Sectionals

Download as txt, pdf, or txt
Download as txt, pdf, or txt
You are on page 1of 1356

AIR-1

MOCK 6
Passage (Q.1-Q.5): On Monday, Assam and Mizoram clashed over a disputed boundary.
Five policemen
and one civilian (all from Assam) were killed and nearly 50 others were injured.
The bloodiest-ever
incident between the two states was the culmination of the discontent that had been
simmering since
October 2020 over incidents of violence and Assam’s residents blocking NH-306, the
lifeline to Mizoram,
for 12 days. While Assam claims Mizoram has encroached on its boundary, Mizoram
maintains the area
belongs to the state.
Mizoram has accused Assam police personnel of entering its territory, indulging in
violence, and claimed
that it only retaliated after tear gas canisters and grenades were hurled by
Assam’s security forces.
Assam has blamed Mizoram of breaching existing agreements and status quo on the
border, and
accused the Mizoram police of firing on its security forces and civilians using
light machine guns. A
violent clash of this nature between two Indian states represents a constitutional
breakdown. And the fact
that the clash played out on Twitter, with chief ministers (CMs) of both states
sparring and urging home
minister Amit Shah to intervene, represents a political and institutional failure.
Incidentally, Mr Shah had
met the CMs of all Northeastern states over the weekend in Shillong where long
pending inter-state
border disputes were discussed.
Mizoram’s border dispute with Assam goes back to 1972 when the former was first
carved out as a Union
Territory (it became a state in 1987). Assam also has border disputes with
Meghalaya, Nagaland and
Arunachal Pradesh. With states in the region either ruled by Bharatiya Janata Party
(BJP) governments
or, as in the case of Mizoram, by constituents of the North East Democratic
Alliance (NEDA), a BJP-led
political platform of anti-Congress parties, there were expectations that border
issues would get resolved.
Instead, the situation appears to have become worse. Monday must serve as a wake-up
call for the
Centre as well as states in the region.

1. According to the passage which of the following options is true?


(a) In Mizoram, the government is of Indian National Congress.
(b) CMs of both the states had asked Amit Shah to intervene in the dispute.
(c) Recently Assam and Manipur clashed over boundary dispute causing death of Assam
police
personnel.
(d) Due to said clash between the two states, 5 police personnel have died who
belonged to Manipur.

2. Why did author expect that the border issues would get resolved in the North-
eastern region?
(a) Because recently, there has been cooperation and peace in the north-eastern
region.
(b) Because the border dispute was resolved in 1972.
(c) Because all the states in the North easter regions are ruled either by BJP or
its alliances.
(d) None of the above.

3. Which of the following can be a suitable title for the passage ?


(a) Assam’s encroachment on Mizoram’s land.
(b) North-eastern States of India – A Disturbed Region
(c) Clash between Mizoram and Assam – An Institutional Failure.
(d) Northeast States of India

4. Which of the following means the same as ‘simmering’ as it is used in the


passage?
(a) carping
(b) tricking
(c) stewing
(d) bickering

5. Which of the following CANNOT be used as a replacement for ‘culmination’ as it


is used in the passage?
(a) nadir
(b) pinnacle
(c) apogee
(d) apotheosis

Passage (Q.6-Q.10): It has been eight working days since the monsoon session of
Parliament began. It
started out with the hope that both the treasury and Opposition benches have a
common interest in a
functional session. The government wanted to push through its legislative agenda
and was prepared to
address the issue of the management of the second wave of Covid-19 this summer. The
Opposition saw
an opportunity to hold the government accountable on public health, price rise and
national security. But
with the Pegasus revelations storming the Indian public sphere, the script changed.
The Opposition has
now demanded that unless there is a full discussion in both Houses on Pegasus and
an enquiry, it won’t
allow Parliament to run. The government has issued two perfunctory statements on
the issue but
doesn’t want a discussion. And so there is a stalemate.
For the sake of India’s parliamentary democracy, it is crucial to break the
stalemate. In principle, the
Opposition is right. The fact that the numbers of key political figures, business
executives, bureaucrats,
activists and journalists were on a potential surveillance list, and some of their
phone devices were found
to be infected with Pegasus — a military-grade spyware — is cause for concern. It
throws up questions
about who was culpable, what it means for the right to privacy, liberty, dignity,
and indeed, the very idea
of democracy. Parliament is the best forum to discuss the issue — and it is indeed
beyond the purview of
just the information technology minister to address it.
The government is in a bind. It cannot admit to procuring and using Pegasus — given
that hacking is
illegal in India and it will raise difficult questions about its democratic
credentials. It cannot deny using
Pegasus — given that this means a foreign government was possibly deploying it
against Indian citizens.
And so it has relied on a defence based on how all interception and surveillance in
India has to be legally
sanctioned. But the fact is that at some point, if not in the House, then in the
courts where a bunch of
petitions have been lined up, the government will have to give a detailed response.
The government may
think that the disruption is costing it little; it is still able to get legislative
business through amid the din
and is able to evade more uncomfortable electoral issues such as price rise. But
the spirit of democracy
dictates that the Opposition must be allowed its say, even if the government has
its way.
6. According to the passage, why is Government “in a bind”?

(a) It cannot admit that it was using Pegasus because hacking is illegal in India.
(b) It cannot deny using Pegasus because it would mean foreign government was using
against Indian.
(c) Both a) and b)
(d) None of the above.

7. Which of the following can be s suitable tone of the passage?


(a) Descriptive
(b) Narrative
(c) Analytical
(d) None of the above.

8. According to the passage, which of the following is not true?


(a) Pegasus is a military grade software.
(b) A number of political figures, business executives, bureaucrats, activists and
journalists were on a
potential surveillance list of Pegasus.
(c) It has been eight working days since the Winter session of Parliament began
(d) The Opposition has now demanded that unless there is a full discussion in both
Houses on Pegasus.

9. Which of the following DOES NOT mean the same as ‘perfunctory’ as it is used in
the passage?
(a) desultory
(b) peremptory
(c) cursory
(d) uninterested

10. Which of the following means the same as ‘din’ as it is used in the passage?
(a) commotion
(b) business
(c) session
(d) None of the above.

Passage (Q.11-Q.15): When BS Yediyurappa resigned as Karnataka’s chief minister


(CM) on Monday
morning, it truly marked the end of an era for both the Bharatiya Janata Party
(BJP) and the state. For the
party, Mr Yediyurappa was the last man standing from the old guard who had retained
a position of
power in the formal governance structure. Now 78, he joined the Rashtriya
Swayamsevak Sangh over
five decades ago and rose up the Jana Sangh and the BJP hierarchy — all the way
from a district to
state unit president; from a member of the legislative assembly first elected way
back in 1983 to a
member of Parliament elected in 2014 (the short national stint was only because he
wanted to stay
focused on the state); from a three-term leader of opposition in the assembly to a
four-term chief minister,
though two of those terms were of less than a week’s duration. But beyond the
formal positions, it was Mr
Yediyurappa’s blood, sweat, and tears that enabled the BJP to cross the rubicon and
become a truly
national party with an imprint in the south.
In the state, Mr Yediyurappa draws his power from the Lingayat community, which
constitutes over 15%
of the population and with a network of sub-religious institutions, wields
disproportionate influence over
politics. But the Karnataka satrap also went beyond his caste base to expand the
BJP’s footprint among
other communities. This multi-caste alliance has been a strength, but has also
generated tensions as is
visible in the battle for succession between leaders of Lingayat and non-Lingayat
communities in the
state. Mr Yediyurappa’s administrative record is hard to judge — for he never
fulfilled a full term in office.
But perhaps in a symbol of the close nexus that begun to mark the nature of India’s
compromised crony
capitalism in the 2000s, he faced corruption allegations on land and mining issues,
most infamously due
to his perceived links with the mining barons of Bellary. This eroded the party’s
image as one with a
difference.
The BJP now has to ensure a smooth leadership transition, keeping all factions and
castes happy. It also
has to ensure that the exit doesn’t have implications for the stability of the
government. Memories of its
electoral setback when Mr Yediyurappa briefly left the party between 2012 and 2014
will be fresh in the
BJP’s mind. And it has to ensure that Mr Yediyurappa does not cash a long shadow
over his successor.
How it navigates these challenges will determine the future of the party in the
state
.
11. According to the passage, which of the following is NOT TRUE about YS
Yediyurappa?
(a) He got first elected to Legislative Assembly in 1982.
(b) Mr Yediyurappa draws his power from the Lingayat community, which constitutes
over 15% of the
population.
(c) He recently resigned as the CM of Karnataka.
(d) None of the above.

12. According to the author, why is YS Yediyurappa’s political career ‘difficult to


judge”?
(a) He had lost a huge number of elections before winning one.
(b) He was hated by some people and loved by others.
(c) He never completed a term in the office.
(d) None of the above.

13. What was YS Yediyurappa biggest contribution towards BJP?


(a) He helped BJP reach South India.
(b) He helped vote bank of BJP by including Muslim communities in it.
(c) Both a) and b)
(d) Neither a) nor b)

14. It is written in the passage - it was Mr Yediyurappa’s blood, sweat, and tears
that enabled the BJP to
cross the rubicon and become a truly national party with an imprint in the south.
What is the meaning of
the phrase “cross the rubicon”?
(a) To become significant in your sphere of work.
(b) To reach a point where you cannot change a decision or course of action.
(c) To be most powerful in a region.
(d) To be able control everything.

15. Which of the following means the same as ‘crony’ as it is used in the passage?
(a) dubious
(b) intimate
(c) sacrilegious
(d) vexatious

Passage (Q.16-Q.20): A month ago, both the Bharatiya Janata Party (BJP) and the
Congress were
struggling with the political situation in two states where they were the governing
party. In Uttar Pradesh,
the management of the second wave triggered criticism of chief minister (CM) Yogi
Adityanath; there was
also criticism of his governance style and alleged patronage to certain castes. The
BJP national
leadership let the controversy play out, uncharacteristically, in public view. But
eventually, it backed Mr
Adityanath’s continuation as CM while making some adjustments, both at the central
and state level, to
accommodate leaders of other castes. With this, it hopes to tap into both the
incumbency vote (in favour
of the CM) as well as the vote of the discontented (who may be disenchanted with
the CM but have faith
in the prime minister).
In Punjab, the acrimonious battle between CM Captain Amarinder Singh and rebel
Navjot Singh Sidhu
played out, with the national leadership allowing Mr Sidhu to undermine the CM.
Eventually, a power#sharing formula was imposed with Mr Sidhu appointed as party
chief. With this, the Congress too hopes
to tap into both the pro-incumbency vote (of supporters of Captain Singh) and the
anti-incumbency vote
(with Mr Sidhu doubling up both as party chief but also chief dissenter against the
state government).
While the context is different, the BJP’s formula is cleaner to execute — back the
CM, while ensuring the
PM is seen as above the fray. The Congress’s formula is harder to execute — since
the CM has to deal
with an official rival in the same party structure.
But both national parties have turned their focus now to two other states. For the
BJP, it appears a
transition in Karnataka is imminent, with BS Yediyurappa being asked to make way
for another leader as
CM. The veteran Karnataka leader has an autonomous base, especially within the
Lingayat community,
and cannot be brushed aside easily. How his possible exit impacts the party’s
fortunes in the south is to
be seen. For the Congress, the next challenge is Rajasthan, where Sachin Pilot has
been waiting for
promises to be implemented. The party leadership is now seeking to ensure the
accommodating of his
loyalists in the state power structure, but whether Mr Pilot’s rival, CM Ashok
Gehlot, will allow this
remains unclear. From UP to Punjab, Karnataka to Rajasthan, both national parties
are attempting to
manage state units with an eye on the next election and generational transition,
while ensuring Delhi’s
writ runs.

16. According to the passage, which of the following options is NOT TRUE regarding
the situation in Punjab?
(a) The governing party in Punjab is BJP.
(b) CM of Punjab is Navjot Singh Sidhu.
(c) Mr Amarinder Singh has been appointed as the party chief.
(d) None of the above.

17. According to the passage, why did BJP include members of certain castes at
Centre and State level?
(a) To accumulate the votes of those castes.
(b) Because those members are highly skilled at specific ministerial work
(c) Because the incumbent ministers were not working well.
(d) None of the above

18. Which of the following can be a suitable title for the passage?
(a) Battle between Sidhu and Amarinder Singh in Punjab
(b) Entry of New Ministers in Central Government
(c) Political Adjustments made by BJP and Congress
(d) None of the above.

19. Which of the following can be used as a replacement for ‘acrimonious’ as it iss
used in the passage?
(a) pectoral
(b) lowly
(c) modicum
(d) rancorous

20. Which of the following does NOT MEAN the same as ‘imminent’ as it is used in
the passage?
(a) miscible
(b) approaching
(c) impending
(d) forthcoming

Passage (Q.21-Q.25): Mirabai Chanu’s silver medal at the Tokyo Olympics was not
unexpected, but it
was a joyous moment when it happened nonetheless. Chanu, who comes from a farming
village near
Imphal, has been one of the world’s best weightlifters in her category, 49kg, for
some time now. She has
won a world championship and holds the world record in one of the Olympics lifts,
the clean & jerk. The
Olympic silver simply adds to the aura and signals, yet again, the pioneering path
forged by athletes
from Manipur.
Kunjarani Devi, the woman who inspired Chanu, was India’s first medallist at the
world weightlifting
championship. She won a silver in 1989, at a time when being a woman weightlifter
in India was almost
unheard of. Devi inspired, and continues to inspire, generations of aspiring
athletes to take up the sport in
India. MC Mary Kom did the same with boxing — opening the way for women to get into
a fighting sport
and dream of big medals. Six world titles and an Olympic medal later, she is still
fighting, in Tokyo. From
the late Dingko Singh to Sarita Devi, and an assembly line of hockey and football
players, Manipur has
an outsized influence on the Indian sporting scene.
Much of that has to do with the love for sport and the sporting culture of the
state. Yet, Manipur’s sporting
infrastructure is below par, even compared to the usual less-than-ideal standards
everywhere in India.
Some states have, in the past decade, started to tap into sporting culture and
encourage its
development. Haryana has done that with boxing and wrestling at the grassroots.
Odisha has done so
with hockey and athletics and built world class infrastructure. Karnataka has some
of the finest sporting
institutes in India now. Manipur would do well to embrace its unique sporting
identity and nurture it in a
truly ambitious manner.

21. Which of the following can be a suitable title for the passage?
(a) Enhancing Manipur’s Sports Infrastructure
(b) Mira Bai Chanu and her Achievements
(c) Weightlifting in Olympics
(d) Increase of Women in Olympic Games

22. Which of the following sports athletes in not mentioned in the passage?
(a) Mirabai Chanu
(b) Sarita Devi
(c) Saina Nehwal
(d) MC Mary Kom

23. Which of the following NOT TRUE according to the passage?


(a) Kunjarani Devi, was India’s first medallist at the world weightlifting
championship
(b) Mirabai Chanu won a silver medal at the Tokyo Olympics
(c) Mirabai Chanu has won six world titles and an Olympic medal.
(d) Karnataka has some of the finest sporting institutes in India now.

24. Which of the following means the same as ‘pioneering’ as it is used in the
passage?
(a) titillating
(b) new
(c) scintillating
(d) victorious

25. Which of the following means the closest to ‘ambitious’ as it is used in the
passage?
(a) blatant
(b) forcible
(c) zealot
(d) determined

Passage (Q.26-Q.30): Mahabaleshwar, a hill station in Maharashtra’s Satara district


in the Western
Ghats, on Friday, recorded the highest ever rainfall in its history: 60cm in the
previous 24 hours. More
than 35 people have died in rain-induced landslides in the state’s coastal Raigad
district. Mumbai
continues to be battered by heavy rainfall and has been placed under orange alert
(“heavy to very heavy
rain at isolated places”). Other parts of the western coast, including cities and
towns in Madhya Pradesh,
Karnataka and Goa, have also recorded exceptional rainfall and flooding in the last
24 hours. The India
Meteorological Department (IMD) classifies rainfall as “extremely heavy” if an area
records 20 cm or
more in 24 hours. An offshore trough running from the Maharashtra coast to north
Kerala coast is causing
such heavy rains and is likely to weaken from July 26, said IMD.
Over the past month, India has seen several rain-related extreme weather events.
Last week, 31 people
died in a series of house collapses after a short burst of intense rain triggered
landslides in Mumbai. Both
Uttarakhand and Himachal Pradesh have seen incessant rainfall, with a flash flood
in the latter. Delhi has
seen some shorter, intense bursts of rain in the last few days, but only after a
slow and delayed start to
the monsoon. These wayward weather events carry not only the indelible sign of a
surging climate crisis
but are also a compelling reminder that the country is under-equipped to tackle
many of its effects, such
as urban flooding and flash floods. But just blaming the climate crisis will be
wrong. Despite some
proactive efforts by some states, India’s development plans are marked by frenzied
and unregulated
construction, even on flood plains, an utter disregard for natural topography and
hydro-geomorphology,
and poor-quality affordable housing. Along with overburdened drainage, these are
only exacerbating the
effects of the climate crisis, leading to large-scale human tragedies and
destruction of property.
In 100 days, the world will witness the Cop26 summit, the vital United Nations
climate talks that open on
November 1 in Glasgow, United Kingdom. The extreme weather events in India and
across the world
(floods across Europe and China, wildfires in the United States, killer heatwaves
stretching into northern
latitudes) are a sharp reminder of what is at stake, and the express need to
accelerate worldwide
measures that are required to control the climate crisis.

26. According to the passage which of the following options is NOT TRUE ?
(a) Mahabaleshwar, a hill station in Maharashtra’s Satara district in the Western
Ghats, on Friday,
recorded the highest ever rainfall in its history
(b) In 100 days, the world will witness the Cop24 summit, the vital United Nations
climate talks that open
in Glasgow, United Kingdom
(c) Recently, more than 30 people died in a series of house collapses after a short
burst of intense rain
triggered landslides in Mumbai.
(d) None of the above.
27. According to the passage which of the following reasons exacerbate the effects
of climate crisis in India?
(a) Overpopulation
(b) Unregulated construction
(c) Deforestation
(d) Wastage of water resources

28. Which of the following is not an ‘extreme weather event which’ is mentioned in
the passage?
(a) Floods in Europe
(b) Wildfires in USA
(c) Heavy Rainfalls in India
(d) None of the above

29. Which of thew following can be used as a replacement for ‘wayward’ as it is


used in the passage?
(a) pivotal
(b) awry
(c) refractory
(d) equitable

30. Which of the following does not mean the same as ‘frenzied’ as it is used in
the passage?
(a) quietude
(b) frenetic
(c) turbulent
(d) manic

LR: The Supreme Court of India in 1996 through the K.R. Lakshmanan judgment has
devised a guiding concept to govern the practice of online gaming/gambling. Games,
where chance
predominates over skill, are forbidden, while games, where skill prevails over
chance, are permitted. In
fact, in comparison to other online games, Courts have been rather demonstrative in
establishing a space
for fantasy sports. Both the Punjab & Haryana and the Bombay High Courts have
concluded that the
games offered by Dream 11 (Fantasy Cricket, Kabaddi, and Football, among others)
demand skill,
knowledge, judgment, and attention and are thus not considered to be coming under
the scope of online
gambling.
Wagering or betting involves the transaction of money or any type of property to
guess the outcome of a
race, game, or any other unpredictable event. Section 30 of the Indian Contract Act
makes agreements
by way of wager, void, and unenforceable. The essentials to wager constitute the
performance of the
bargain depending upon the determination of an uncertain event where both the
parties have mutual
chances of gain and loss and where neither party has control over the happening of
the event one way or
the other.
Unlike games of chance, in E-sports the amount of skill and talent involved in
mastering the game has led
to its professionalization. Also, in E-sports games, the prize money on winning is
pre-declared and does
not depend on the number of people joining the contest as in the case of other
online games.
[Extracted from 'Here’s how online gaming and E-sports are taxed in India' by
Shivani Jha, published 14
June 2021 on The Financial Express

68. Virat created a new gaming platform where football enthusiasts from all over
the world could come
together and make their own football teams by selecting players from different
clubs. Players could bet on
the outcome of the match, which was dependent on whose team was objectively better.
To make an
objectively better team, it was important to have a fundamental understanding of
the strengths and
weaknesses of each member of the team. Does this game fall within the scope of
online gambling?
(a) No, because only games where chance predominates over skill are forbidden and
not games where
skill prevails over chance;
(b) No, because only games where skill prevails over chance are forbidden and not
games where chance
predominates over skill;
(c) Yes, because only games where chance predominates over skill are forbidden and
not games where
skill prevails over chance;
(d) Yes, because only games where skill prevails over chance are forbidden and not
games where
chance predominates over skill.

69. X and Y entered into a contract. The contract stipulated that X would pay 10$
to Y if a head turns up after
every two tosses of a coin and, Y would pay 10$ to X if a tail turns up after every
two tosses of a coin.
The coin was to be tossed by Shamsher Bagawat, who was an expert at tossing coins.
Based on your
reading of the passage, is the contract between X and Y valid?
(a) Yes, because the performance of the contract depends on the determination of an
uncontrollable and
uncertain event where both the parties have mutual chances of gain and loss;
(b) Yes, because the performance of the contract does not depend on the
determination of an
uncontrollable and uncertain event as the coin was to be tossed by an expert;
(c) No, because the performance of the contract does not depend on the
determination of an
uncontrollable and uncertain event as the coin was to be tossed by an expert;
(d) No, because the performance of the contract depends on the determination of an
uncontrollable and
uncertain event where both the parties have mutual chances of gain and loss.

70. Basanti and Gabbar enter into an agreement where Basanti promised to pay Gabbar
a sum of Rs.
55,000 if India wins the world cup. India wins the match and there is an obligation
on Basanti to pay the
amount to Gabbar but she fails to do so. Gabbar initiates a suit against Basanti
for the recovery Rs.
55,000. Will Gabbar succeed in the recovery of the agreed amount?
(a) Yes, because the agreement is void and non-enforceable as it is a wagering
agreement,
(b) No, because the agreement is based on the loss of one party at the gain of the
other,
(c) Yes, because it is dependent on an uncertain event and both the parties have no
mutual interest
except gain or loss,
(d) No, because the agreement is void and non-enforceable as it is a wagering
agreement.

71. FancyBros Ltd launched a mobile application for a game called Chaar Patti. This
game provides a
realistic experience of a casino with real professional dealers, and glamorous
studios. Players can play
Chaar patti from the comfort of their homes and pay amounts for each round of cards
securely through
Paytm. Many novice card players incurred huge debts on the application while
playing Chaar Patti and
were unable to pay their debts. Can FancyBros Ltd recover this amount from these
players?
(a) Yes, because playing Chaar Patti involves mutual chances of gain and loss;
(b) Yes, because Chaar Patti is a game of skill and does not constitute online
gambling or wagering;
(c) No, because Chaar Patti is a game where skill prevails over chance and not
chance predominating
over skill;
(d) None of the above

72. Anuradha launched a new company called Laxmi Ltd. The company organised a
sweepstake competition
where customers were asked to purchase tickets. A lottery was pulled out and the
tickets which
contained the number ‘53’ were awarded cash prizes. Raju had purchased 50 tickets
in the hopes of
winning a cash prize and ended up winning Rs. 1 lakh in total. However, when he
went to claim the
amount, he was turned away. Can he sue Laxmi Ltd for recovery of the amount?
(a) Yes, because a valid contract was constituted between Raju and Laxmi Ltd when
he purchased 50
lottery tickets;
(b) Yes, because lotteries do not have the characteristics of wagers and are
enforceable;
(c) No, because lotteries are fiscally irresponsible and discouraged by the
government;
(d) None of the above.

Passage(Q.73-Q.77): In a recent ruling, the Supreme Court of India observed that


the ‘doctrine of
impossibility’ would be equally applicable to Court orders.
The Doctrine of Impossibility stems from the Latin maxim, lex non cogit ad
impossibilia, signifying that ‘a
man cannot be compelled to do what is impossible’. The genesis of the doctrine is
rooted in the law of
contracts. Statutorily enshrined in Section 56 of the Indian Contract Act, 1872,
this provision allows
contracts to be set aside due to supervening impossibility preventing its
performance.
The application of the doctrine to court orders reflects a shift in the non-
performance of a statutory
obligation to a prospective assumption of non-compliance with the Court’s
directions by the Executive.
Courts often function within their boundaries to enforce directions for public
welfare. Although some of
these directions may be stringent, they go a long way in ensuring that states wake
up to the seriousness
of the matter concerned. Courts generally take into account the response of the
Executive before issuing
directions to them. Thus, it would lead to a lack of clear demarcation between the
mandatory directions
issued by the courts and the advice or opinions given in a general context, which
would make way for the
Executive to excuse themselves from the directions of the court.
As one of the three pillars of democracy, courts are often approached to ensure
that states fulfil their
responsibilities owed to the citizens. This, however, does not excuse the
governments for failing their
duties on the grounds of impossibility and impracticality. Such directions cannot
be circumvented by
claiming ‘impossibility’ or impracticality’ even if it appears to be difficult to
implement.
[Extracted with revisions from 'Extending the Doctrine of Impossibility to Court
Orders: A Conundrum' by
Aisiri Raj and Vishnu Mohan Naidu, published on NLUJ Criminal law blog

73. During the peak of COVID-19 pandemic, the Allahabad High Court issued several
directions such as
providing a minimum number of ambulances in graded towns, requiring new firms to
manufacture
vaccines by borrowing the formula from existing manufacturers, providing oxygenated
beds in all state
nursing homes and upgradation of state medical colleges within a very short period.
The State
government contests that these directions are nearly impossible to implement and
the directions
regarding the manufacture of vaccines could also have international ramifications.
Can the doctrine of
impossibility be invoked against the directions of the Allahabad High Court?
(a) Yes, because the doctrine of impossibility can be invoked against Court orders
when there is
supervening impossibility preventing their performance;
(b) Yes, because though directions are difficult to implement and impractical, they
are technically
possible;
(c) No, because the government cannot be excused for failing their duties on the
grounds of impossibility
and impracticality;
(d) No, because invoking the doctrine of impossibility will lead to a lack of clear
demarcation between the
mandatory directions issued by the courts and the advice or opinions given in a
general context.

74. In the case of River Ganga Organisation v State, the Court ruled in favour of
the NGO, River Ganga
Organisation. The NGO had sought a clean-up of River Ganga, including the ghats and
the
implementation of mandatory awareness drives, setting up of toilets etc. which was
granted by the Court.
Three months later, the State said a lack of funds prevented them from controlling
the rampant
environmental pollution and fulfilling the court order. Can the State invoke the
doctrine of impossibility to
avoid the performance of its duties?
(a) Yes, because a lack of funds can lead to difficulties in the implementation of
environmental policies;
(b) Yes, because a lack of funds constitutes a supervening impossibility which
prevents performance of
the order;
(c) No, because there is a pre-existing duty on the government to address these
problems by prioritising
them and allocating a budget to meet these requirements;
(d) No, because a mere difficulty in implementation due to lack of funds does not
make the order
impossible.

75. Aladin and Jasmine enter into a contract. Aladin agrees to discover treasure by
magic with the help of his
Magic Carpet and a genie. Jasmine had neither seen nor heard of a Magic Carpet or
genie but, she
promises to give a Rolls Royce car to Aladin if he performs his part of the
bargain. Jasmine grows
impatient and demands performance of the Contract. Based on the principles
mentioned in the passage,
can she do so?
(a) No, because discovering treasure by magic with the help of a Magic Carpet and a
genie is not
impossible;
(b) No, because Aladin cannot be compelled to do what is impossible;
(c) Yes, because Aladin can be compelled to do what is impossible;
(d) Yes, because discovering treasure by magic with the help of a Magic Carpet and
a genie is not
impossible.

76. Manjeet booked a banquet hall for his daughter's wedding. The contract with the
owner of the hall
stipulated that the owner would be liable to pay damages to Manjeet if he cancelled
the booking without a
one-month notice. Two days before the wedding, an unforeseen short circuit
culminated into a large fire
which burnt down the hall. Although he felt bad for the owner of the hall, Manjeet
was angry because he
would not be able to book a venue on such short notice. Can he seek damages from
the owner of the
hall?
(a) Yes, because a short circuit does not amount to a supervening impossibility as
the owner should not
have been negligent;
(b) No, because a short circuit amounts to a supervening impossibility and renders
the contract void;
(c) Yes, because a short circuit amounts to a supervening impossibility that can
set aside the contract;
(d) No, because a short circuit amounts to a supervening impossibility that can set
aside the contract.

77. Y leased X’s entire restaurant at a rate higher than usual. Y agreed to pay a
higher price expecting huge
profits as the Olympics were going on and a large number of tourists were expected
in the city.
Subsequently, the government passed an order banning all tourists in the country
due to the rising
number of COVID-19 cases. Consequently, no profit was materialised for Y as
expected. Y pleaded
impossibility because he did not want to pay the higher rent any longer. Will Y’s
plea be accepted?
(a) Yes, because it is impossible for him to attain huge profits in light of the
government order and that
was the entire basis of the contract;
(b) Yes, because it is impossible for Y to perform his end of the bargain as he
would suffer unprofitability;
(c) No, because mere unprofitability cannot render the contract to be frustrated;
(d) No, because the terms of the contract are not impossible to perform for Y and
they are only
unprofitable.

Passage(Q.78-Q.82): The proviso to Section 43D(5) of Unlawful Activities


(Prevention) Act [UAPA] lays
down that a court shall not allow bail if “there are reasonable grounds for
believing that the accusation
against such person is prima facie true,” thus leading to the belief that courts
have no power to grant bail
to a person accused under Sections 15 to 23.
Section 13 mainly concerns itself with disrupting Sovereignty, whereas Section 15
makes punishable, any
terrorist act and opens with the words “any act with intent to threaten or likely
to threaten the unity,
integrity, sovereignty of India, or with intent to strike terror or likely to
strike terror in the people..." by
using lethal weapons, bombs etc.
The conclusion that the court is not to presume the guilt of the accused, and that
the "burden to
demonstrate the prima facie veracity, the allegation must fall upon the
prosecution" is absolutely correct.
When UAPA uses the words “prima facie true”, it meant that the court must accept
the guiltof the accused
persons, even if on broad probabilities. This could happen only if the prosecution
had discharged its initial
duty of establishing a prima facie case.
Why this reasoning is sound can be tested by the following simple explanation. At
the stage when the
court is considering bail, the only material available with the court is that which
has been brought or
collected by the prosecution. So evidently, the court can make up its mind on that
material alone which
needless to say should be sufficient for it to arrive at an opinion that prima
facie the case is true.
[Extracted with revisions from ‘Delhi Riots case: How the Delhi High Court caught
the bull by the horns
while confronting Section 43(D)5 of UAPA' by Anjana Prakash, published 17 June 2021
on bar and
bench

78. Sharjeel participated in a protest against the established, democratically


elected Government of India.
The protest gained widespread media coverage and large masses gathered in the
concerned ground.
Eventually, the crowd became so rowdy that police was ordered to do a lathi charge
to clear the space.
Sharjeel was charged under Section 15 of UAPA. When his case came for hearing, the
state submitted
pictures of Sharjeel addressing the crowd along with videos of his speeches. Has
the state discharged its
burden under Section 43(D)5 of UAPA?
(a) Yes, because the state has submitted enough material on record for the Court to
make up its mind
and decide if the accusations are prima facie true;
(b) No, because the state has submitted enough material on record for the Court to
make up its mind and
decide if the accusations are prima facie true;
(c) Yes, because the state has not submitted enough material on record for the
Court to make up its
mind and decide if the accusations are prima facie true;
(d) No, because the state has not submitted enough material on record for the Court
to make up its mind
and decide if the accusations are prima facie true.

79. Based on the evidence presented by the State against Sharjeel, as enumerated in
the previous question,
the Court denied Sharjeel’s application of bail. The Court found the allegations
against him to be prima
facie true. Sharjeel objected to this decision because he believes that he has a
right to bail. Is the
disposal of Sharjeel’s bail application by the Court, a valid action?
(a) Yes, because jail is the exception and bail is the rule in criminal law;
(b) No, because the Court has the power to grant bail in UAPA cases even if the
allegations are prima
facie true;
(c) Yes, because the Court only needs to make up its mind that the allegations are
prima facie true to
disallow bail application;
(d) No, because there is not enough material for the Court to determine if the
allegations against Sharjeel
are prima facie true.

80. Tabrez was arrested on charges of rioting by the police of Koregaon for
threatening the sovereignty,
integrity and unity of India, under Section 15 of UAPA. Besides the witness
statements of a few people
who were not even on the spot of the alleged rioting, the police did not submit any
other evidence to
substantiate the charges. Can the Court grant bail to Tabrez in this case?
(a) Yes, because there are reasonable grounds for believing that the accusations
against Tabrez are
true;
(b) Yes, because the police did not discharge its burden of proving that the
allegations against Tabrez
are prima facie true;
(c) Yes, because the police discharged its burden of proving that the allegations
against Tabrez are
prima facie true;
(d) No, because there are no reasonable grounds for believing that the accusations
against Tabrez are
true.

81. P and Q prepared petrol bombs with empty glass bottles of Corona beer. They
were loyal citizens and
were happy with the government’s functioning. However, they were very bored with
their miserable,
monotonous lives and wanted to get a ‘kick’ out of doing something different. As a
result, they decided to
throw the petrol bombs in a public park at 2 am in the night when there were hardly
any people in the
park, so as to create a scare. Can they be convicted under the provisions of UAPA?
(a) Yes, because they disrupted sovereignty of the nation under Section 13 of UAPA;
(b) Yes, because they intended to strike terror in the people of the nation under
Section 15 of UAPA;
(c) Yes, because they committed an act with the intention to threaten the unity and
integrity of the nation
under Section 15 of UAPA;
(d) None of the above.

82. P and Q’s antics were published in the newspapers and they were identified as
the culprits behind the
petrol bomb incident after a swift police investigation. The Court was informed
that materials used in
making the petrol bombs, similar to the ones that were thrown in the park, were
found at P and Q’s
house. Can the Court grant bail to P and Q?
(a) Yes, because there are reasonable grounds for believing the accusations against
P and Q are prima
facie true;
(b) No, because there are no reasonable grounds for believing the accusations
against P and Q are
prima facie true;
(c) No, because there are reasonable grounds for believing the accusations against
P and Q are prima
facie true on broad probabilities;
(d) No, because there are no reasonable grounds for believing the accusations
against P and Q are
prima facie true even on broad probabilities.

Passage(Q.83-Q.88): The offense of criminal breach of trust, as defined under


section 405 of IPC, is
similar to the offense of ‘embezzlement’ under the English law. A reading of the
section suggests that the
gist of the offense of criminal breach of trust is ‘dishonest misappropriation’ or
‘conversion to own use’
another’s property, which is nothing but the offense of criminal misappropriation
defined u/s 403.
The essential ingredients of the offense of criminal breach of trust are (1) The
accused must be entrusted
with the property or with dominion over it, (2) The person so entrusted must use
that property, or; (3) The
accused must dishonestly use or dispose of that property or wilfully suffer any
other person to do so in
violation, (a) of any direction of law prescribing the mode in which such trust is
to be discharged, or; (b) of
any legal contract made touching the discharge of such trust.
For criminal breach of trust, the accused is entrusted with property or with
dominion or control over the
property. As the title to the offense itself suggests, entrustment ofproperty is an
essential requirement
before any offense under this section takes place. The language of the section is
very wide. The words
used are ‘in any manner entrusted with property’. So, it extends to entrustments of
all kinds-whether to
clerks, servants, business partners or other persons, provided they are holding a
position of trust. The
term “entrusted” found in Section 405, IPC governs not only the words “with the
property” immediately
following it but also the words “or with any dominion over the property.”
There are two distinct parts involved in the commission of the offense of criminal
breach of trust. The first
consists of the creation of an obligation in relation to the property over which
dominion or control is
acquired by the accused. The second is misappropriation or dealing with the
property dishonestly and
contrary to the terms of the obligation created. The principal ingredients of
Criminal Breach of Trust are
thus ‘entrustment’ and ‘dishonest misappropriation’.

83. The Crime Department is investigating a major bank fraud case. One of the
employees X, who is
entrusted with handling the matter has lost an important file which has information
on the whereabouts of
the accused. Decide.
(a) X has committed criminal breach of trust as he dishonestly disposed of the
property;
(b) X has not committed criminal breach of trust as he was not dishonest in
disposing of the property;
(c) X has committed criminal breach for he was bribed;
(d) X’s liability cannot be determined.

84. Please refer to the facts above. An intel is received that there was a phone
call between the accused and
X before the file went in missing. Decide.
(a) X has committed criminal breach of trust as he dishonestly disposed of the
property;
(b) X has not committed criminal breach of trust as he was not dishonest in
disposing of the property;
(c) X has committed criminal breach for he was bribed;
(d) X’s liability cannot be determined.

85. Anil and Bhadu are working at the police department. A phone call was traced at
Bhadu’s place late at
night. In addition to a phone call, a man wearing plain clothes was also seen
taking money to Bhadu’s
place and documents concerning an important case went missing on the next day. This
is viewed as
credible evidence of bribery. Decide.
(a) Bhadu has committed criminal breach of trust as he dishonestly disposed of the
property;
(b) Bhadu has not committed criminal breach of trust as he was not dishonest in
disposing of the
property;
(c) The very allegation of bribery is enough to show that Bhadu has committed
criminal breach of trust;
(d) Bhadu is a public servant within the meaning of the IPC and thus, he cannot be
held liable for criminal
breach of trust.

86. Albert wanted to give some documents to his junior. The documents were
entrusted to the junior X over a
phone call. X agreed. After hanging up the phone, Albert asked the peon to deliver
the documents to X’s
office. The file never reached X’s office.
(a) The peon is liable for criminal breach of trust as he was entrusted with the
documents.
(b) The peon is not liable for criminal breach of trust as he was not entrusted
with the documents. He
only delivered it.
(c) The peon is liable for criminal breach of trust and X is vicariously liable.
(d) The peon is liable for criminal breach of trust and Albert is vicariously
liable for he was the one who
entrusted the file to the peon.

87. Please refer to the facts above. It turns out that the peon had spilt some
water onto the file. Scared, he
put the file out in the sun and lied to X about the superior not giving the file.
Decide.
(a) The peon is liable for criminal breach of trust as he misappropriated the
property with a dishonest
intention.
(b) The peon is not liable for criminal breach of trust as he did not have a
dishonest intention.
(c) The peon is liable for criminal breach of trust as he lied to X about his
superior and thus has a
dishonest intent.
(d) The mere fact that the peon lied about it, is enough to show the presence of
dishonest intention.

88. Anil took a file along with him with the intent of taking it away from the
police station. Till this time, there is
no evidence of Anil accepting a bribe.
(a) Anil has committed criminal breach of trust as he dishonestly disposed off the
property;
(b) Anil has not committed criminal breach of trust as he was not dishonest in
disposing of the property;
(c) Anil has not committed criminal breach of trust for he has not misused the
property;
(d) Anil’s liability is subject to further evidence of causing the government
wrongful loss which is the
accepted metric of a dishonest intention.

Passage(Q.89-Q.94): The offence of criminal conspiracy is defined under Section


120-A of Chapter V-A
of the Indian Penal Code, 1860. According to Section 120-A, when two or more
persons come together
and agree to do, or cause something to be done, which constitutes an illegal act or
a legal act carried
forward by illegal means, such persons would be guilty of the commission of the
offence of criminal
conspiracy. In simpler terms, conspiracy refers to the meeting of minds for the
commission of an offence.
However, no such agreement would constitute the offence of criminal conspiracy,
unless and until an act
is performed in furtherance of such an agreement. The explanation attached to
Section 120-A makes it
clear that it is immaterial whether the illegal act committed in furtherance of
such an agreement, is the
focal point of the agreement or, is merely incidental to the performance of the
ultimate goal of the
agreement.
Salient features: There must be two or more person involved in the commission of
the offence of criminal
conspiracy; there must be an agreement between the parties; such an agreement must
be for the
commission of an illegal act or the commission of a legal act by illegal means;
When the agreement is for
the commission of a legal act by illegal means, an overt act must have been carried
out by the parties in
furtherance of the same, and the mere agreement is not sufficient to establish the
commission of the
crime.
For the establishment of the offence of criminal conspiracy, the existence of an
agreement between the
parties is a sine qua non. This agreement may be express or implied, the important
factor is consensus
ad idem, i.e., meeting of minds. The agreement must be read as a whole and the
object be ascertained.
It is not necessary that more than one person must always be convicted for the
offence of criminal
conspiracy, it is sufficient if the court is convinced that more two or persons
were actually involved in the
conspiracy. The offender might join the conspiracy from the start itself or at any
time before the
completion of the objective of the agreement, irrespective of the time of joining,
each party to the offence
would be held equally responsible. A criminal conspiracy is said to persist as long
as the parties to the
agreement continue to act in furtherance of the objects of the agreement.

89. A and B decided to rob a house of Mr. D. In the process of doing so, A killed
Mr. D’s son. Is there a
criminal conspiracy under Section 120A?
(a) Yes, both A and B are part of the criminal conspiracy.
(b) No, only A is responsible for the criminal conspiracy as there was no agreement
between A and B for
the murder but only for the robbery.
(c) There is no criminal conspiracy as the essential ingredients for such an act
are prior agreement and
involvement of 2 or more people. .
(d) B has not committed criminal conspiracy.

90. In the above facts, if they blackmailed Mr. D’s guard to help them in the
robbery, is it a criminal
conspiracy?
(a) Yes, all three of them are part of the criminal conspiracy under Section 120A.
(b) Only A and B form part of the criminal conspiracy.
(c) This is a case of robbery and not criminal conspiracy.
(d) There is no criminal conspiracy since there was no prior agreement between the
guard and A & B.

91. On Monday, Ashfaq and Bittu committed theft in a house. In addition, there were
three other persons
Paras, Qasim and Rohit who happened to rob the same house on the same day. All five
of them were
caught by the police. Is there a commission of criminal conspiracy?
(a) All five of them formed the part of criminal conspiracy.
(b) Only Ashfaq and Bittu formed the part of criminal conspiracy.
(c) Only Paras, Qasim and Rohit formed the part of criminal conspiracy.
(d) There is no criminal conspiracy.

92. In the above facts, Qasim wanted to kill the house owner’s son due to his old
rivalry with him but Paras
and Rohit did not agree to this as they only wanted to rob the house. Qasim killed
the owner’s son after
taking their all money and jewellery.
(a) All three of them committed criminal conspiracy.
(b) Only Qasim committed criminal conspiracy.
(c) There is no criminal conspiracy as the prior agreement was only of robbing the
house and not murder
of the owner’s son. Therefore, Paras and Rohit are not responsible for the murder
committed by
Qasim as they had intention to do so.
(d) This is a case of murder and not criminal conspiracy.

93. Paritosh runs a factory that manufactures incense sticks. A few of his workers
let in Wazir and Raman
who robbed the factory.
(a) All of them committed criminal conspiracy as there is meeting of minds in the
commission of the
crime.
(b) Only Wazir and Raman are responsible for criminal conspiracy.
(c) This is a case of robbery and not criminal conspiracy.
(d) No criminal conspiracy is committed.

94. Mr. Bharath is a student of B.E. in Computer Science He loves his computer very
much. He considers his
computer as his close friend and companion. On 1.4.2006, while interacting with his
computer, he hacked
into the Bank account of Mr. Javed and was successful in withdrawing money front
Mr. Javed’s bank
account. He did it to please his girlfriend.
(a) Mr. Bharath has committed an offence.
(b) Mr Bharath has committed cyber crime.
(c) Mr Bharath has committed offence of criminal conspiracy.
(d) Mr.Bharath has not committed the offence of criminal conspiracy.

Passage(Q.95-Q.100): A journalist is not expected to dramatise an incident and


place the subject of the
news report at risk; the Allahabad High Court observed while hearing a case where a
reporter allegedly
egged on a person to die by suicide in front of the Uttar Pradesh assembly,
promising him airtime.
The deceased person's wife is the complainant in this case.
A journalist is not expected to dramatise a sensational and horrifying incident and
make news by putting
his actor in a pitiable condition in danger of death.
The bench also spoke on the role of a journalist "The journalist keeps an eye on
the anticipated or
sudden events happening in the society and brings them to the information of all
the people through
various news media without any tampering; this is his business."
The role of journalists in exacerbating or propagating a situation thrust into
public view was also critiqued
in the aftermath of the passing of actor Sushant Singh Rajput last year, when
several mainstream
television news channels put Bollywood actors on a media trial and conducted
televised investigations on
their own, going to the extent of pronouncing selected people guilty.
If you know someone – a friend or family member – at risk of suicide, please reach
out to them. The
Suicide Prevention India Foundation maintains a list of telephone numbers they can
call to speak in
confidence. Icall, a counselling service run by TISS, has maintained a crowd
sourced list of therapists
across the country. You could also take them to the nearest hospital.

95. Magan is a very prosperous businessman in the community of Hirania Nagar. One
fine day, the news
issues an alert of a possible bombing in the area due to an escaped terrorist named
Mangal. However,
the news fails to display his name correctly and instead of 'Mangal', displays
'Magan', which leads to a lot
of commotion in Magan's vicinity as he was well known throughout the town. This led
to him being
insulted and disgraced in his community, which led to him getting depressed and
committing suicide. The
Media thereafter issued an apology when the state of affairs came to its notice.
Would the media be
liable for his actions herein?
(a) Yes, as his name was severely besmirched, which led to his suicide.
(b) No, as the community knew that he was not the bomber.
(c) Yes, as he was utterly disgraced, and his reputation was destroyed.
(d) No, as any sane person would not equate a well-known businessman with a
terrorist bomber.

96. In the above set of facts, had Magan not committed suicide, would the media
house have still been liable
for his state of affairs?
(a) No, as he did not commit suicide or harm himself in any other way and as
subsequently apologised
to.
(b) Yes, due to the false information being published by them, the media house
should be held liable.
(c) No, as after issuing an apology, everybody would have realised the mistake.
(d) None of the above.

97. Aaj Tak Sarva Satark is a very well-known news station in North India, with
almost all of the households
in that area relying on its bulletins. Recently, they have ventured into the South
Indian region with a view
to capturing the populace therein. Consequently, when they receive a hot tip about
one big gangster
living on the outskirts of Kerala, they immediately decide to report it. To their
dismay, the person living
there was a poor fisherman trying to make ends meet, and someone had pranked them
by sending his
particulars. This leads to his house being flooded by police authorities but to no
avail, and the fisherman
subsequently goes into a shock caused by the entire trauma. Is the news station
liable to be held guilty
for exacerbating and dramatising?
(a) Yes, the news station should be held liable as their reputation led the masses
to believe that the
person living there was a terrorist.
(b) No, as the news station was a relatively new one in this vicinity and had not
yet garnered any support
yet.
(c) Yes, as its support is irrelevant here, they reported false news.
(d) No, as the person living there was not a celebrity, and neither did they pose a
danger to his life.
98. The same news channel as above reported the relationship of Dhanveer Kapoor and
BeepshikaPadukone as being an abusive and toxic one, wherein both the people
involved preach
violence. The people involved, however, did not make any public statement about
their relationship and
were only ever seen as having dinner once in Mumbai. However, after this news
telecast, they received a
lot of backlash from the public, which led to them being pelted with eggs and their
movies flopping. Would
this action of the Media make them liable for the danger to the actors' life?
(a) Yes, as their news roused the public, which led to their movies flopping and
them being assaulted.
(b) No, as their relationship was public knowledge anyway.
(c) Yes, as their statements led to the whirlwind of events in which they were
caught.
(d) No, as their statements did not endanger their lives.

99. The same news channel as above receives a request by Dhananjaya Bhat; a celeb
turned politician who
had allegedly helped a few terrorists enter India through his private cruise. The
celeb now wants to
confess about the same. He contacts the news channel stating his request and is
granted the same.
However, while on primetime, his phone and anxiety medication is taken by the crew
so that the show
isn't interrupted. However, this leads to him having an anxiety attack, and when
that gets out of control,
he suffers from a stroke which leads to his demise. Would this fiasco lead to the
news station being
implicated in the death of the celeb?
(a) Yes, as the actions of the news station led to his ultimate demise.
(b) No, as the news station crew did not force him to surrender his medication.
(c) Yes, as the news channel crew failed to accommodate his health condition.
(d) None of the above.

100. In the above set of facts, after his interview on the news channel, Dhananjaya
gets home but is
immediately surrounded by people all around the house and is pelted with stones and
fire-lit bottles. This
leads to him getting lynched eventually and the public rioting against his family
as being a threat to the
sovereignty of the country. The media learns about this and immediately tries
reaching the parents of the
celeb and continue to do so even after being turned down. Would the media be liable
here?
(a) The media would be liable here both for the death of the celeb and troubling
his parents.
(b) The media would only be liable for troubling his parents.
(c) The media would only be liable for the death of Dhananjaya.
(d) The media will not be liable for either of the events.

Passage(Q.101-Q.104): The Delhi High Court said that a safe house has to be
provided for all types of
couples- inter-faith, inter-caste, those belonging to the LGBTQIA+ community -
facing opposition from
families. While hearing a petition filed by a couple in a similar situation,
Justice Mukta Gupta said.”
Whether it’s inter-caste, inter-religion, inter-community….LGBTQ also covered”.
“SHO Police Station
Mayur Vihar Phase 1 to ensure petitioners taken from office and lodged safely at
the safe house….there
they would be given necessary protection from SHO through beat staff and other
security staff….” The
Court ordered. The Court pointed out that the current arrangement was only a “stop-
gap” measure and
that the couple’s families had to be made parties to the case in the future.
“Someday they will have to
settle on their own. It’s only a stop-gap….They can live at the safe house for some
time but eventually
have to move out….You need to implead the family members….”, Justice Gupta said.
Appearing for the
petitioners, Advocate Utkarsh Singh submitted that his clients were adults but
their families assaulted
them. The petitioners, one of whom was NGO Dhanak For Humanity, argued that the
couple had
approached the DCP concerned, but weren’t provided with a safe house as yet. The
petitioner also
feared being tracked by the families and the people known to them. Judicial
intervention was sought by
the couple on the ground that they also “have the right to pursue their lives with
dignity irrespective of
their sexual orientation”. Not providing them with the safe house was violation of
their fundamental right
guaranteed under Article 21 of the Constitution of India, it was argued.
Source name: Bar and Bench

101. A and B are couple aged 19 and 17, residing in More Vihaar are in the state of
Dailhi. They were
threatened by B’s family that if she doesn’t leave A, they will create trouble for
both of them. The couple
took the help of the state police but the police counselled them and told them to
return to their respective
homes and try to convince their parents. In the light of the passage given above,
select the most
appropriate option.
(a) State police is justified in its action because, the girl is a minor and she
must not live with her lover in
a live-in relationship.
(b) The girl is a minor and at this tender age, children are not able to take the
right decision of their own,
so the state police authority is justified in its action.
(c) Although the girl is a minor, but having A that is her male partner a major, so
police should give a
helping hand to the couple from their families.
(d) Although, the girl is a minor and the male partner is a major and they own a
right to get protection
from the police authority regarding the matter.

102. P and Q were boys who were in love with each other and called themselves a
couple and never hesitated
to express their love in mild way anywhere, even in public. After disclosing their
truth to their families,
they never objected their relationship but had a problem of their open expression
of the same. After
repeated reminder, they didn’t stop doing the same and ultimately families of the
couple ordered them to
get separated, negating which the state police gave them shelter in the safe house.
But frequent
expression of their love agitated the police and the authority ordered them to
leave the place with
immediate effect. In the light of the paragraph, opt out the most appropriate
option.
(a) Providing safe house to a couple resorting to obscenity is nowhere justified
and thus, the police have
done right.
(b) Every couple has the right to express love anywhere to any extent because now
the same sex
relation or marriage has been legalized by the Hon’ble Supreme Court of India.
(c) Couple must be given the allowance to take shelter in the safe house because
mild expression of
love is couple’s right and there is no harm in it.
(d) Couple must be given to take shelter in the safe house because they have been
threatened by their
families if not got separated from each other.

103. B and C, both aged 17 were couple who took shelter in the safe house and were
living there from 3
months escaping from their family members and their acquaintances who were against
their relation.
After repeated counselling and certain advices of the caretaking police authority,
they were adamant not
to confront their families in any form. The police resorted to strict action and
forced them to leave the safe
house and confront their families and convince them about the same. In the light of
the above passage,
opt out the most appropriate option.
(a) Actions of the police of forcing the couple to leave the safe was nowhere
justified as taking the shelter
in the safe house is their right and giving protection to the couples is police’s
duty.
(b) Police’s action was somewhere justified because a couple can’t live like this
whole life, and someday,
they have to face the fact and confront their families to convince them. Thus the
actions of the police
were justified.
(c) Police owns the right and duty to take a decision for the couple for the
betterment of the couple as
they were both minors and it is the duty of the police to make them realize their
mistakes to ensure a
civil society.
(d) Forcing a couple to do anything is nowhere justified and as, the couples were
threatened by their
family members, it was the duty of the police to give them protection.

104. From the above passage, which of the option gives a perfect gist of the
passage?
(a) Couples have the right to be protected by the police in the safe house
irrespective of their explicit
expression of love anywhere to any extent.
(b) Safe house is meant for providing only a temporary shelter to the couples.
(c) Safe house is meant for all the couples irrespective of their sexual
orientation but not for minors living
in a relationship.
(d) Police has the authority over the minor to force them out of the safe house to
confront their families
regarding their relationship because minors are generally not capable of taking
their lives’ decision
decently

CR One hundred years of the CCP is an occasion for the Chinese to celebrate.
Most importantly, disproving Western negativity, the phenomenal success, the
unprecedented growth
rate and eliminating poverty for 800 million people are reasons worthy of
celebrations. It is not correct to
say that the CCP has not accepted past mistakes and taken the necessary corrective
measures. These
have been the bases on which Deng Xiaoping initiated reforms that made possible the
economic miracle
that followed. The Chinese model that Xi applauds is based on an impressive track
record and its
legitimacy is not limited to the renewal of the popular mandate as we are used to
in liberal democracies,
but in projecting a political structure that is free from compulsions of electoral
politics.
The entire effort of the CCP is to remind China and the world of the century of
humiliation that it suffered
because of the Western design to perpetuate China’s semi-colonial and semi-feudal
order. The century
that began with the Opium Wars denied China its rightful evolution and the
supremacy that it enjoyed as
the longest civilizational state in the world. Restoring China to its rightful
place is not an ideological
question but a nationalistic one, which the CCP intends to achieve by the
acquisition of technology and
achieving rough parity by facilitating frontier technology. China is aware of the
wide differences between
the Cold War periods and accepts the fact of an interdependent world. The BRI
project is an example of
that interdependence. The important point for China is the full restoration of its
sovereignty in which all
other considerations become secondary.
Nationalism being the driving force, the CCP accepts mistakes and setbacks, but
emphasises the
economic and cultural aspects that have made it secure and powerful. This reference
by Xi not to accept
any bullying is with regard to the unequal treaties that the Western powers and
Japan inflicted on China
in the past. The whole emphasis is about restoring China to its pre-1840 status.
The point is that China’s
evolution is very different from the West and the CCP’s legitimacy is based on the
centrality of its
capacity to guarantee China’s independence. China reminds the West of its duplicity
as well: When the
latter invested so heavily, singularly contributing to China’s success, it did so
without insisting on Beijing’s
track record on human rights and democratic values. But these have now become of
paramount
importance with the rise of China as a formidable power and rival.
[Excerpt from an article by Subrata Mukherjee, The Indian Express, July 5, 2021:

105. All of the following can be inferred except?


(a) Western countries are also responsible for the economic growth of China.
(b) China’s influence internationally has increased substantially in the last few
years.
(c) China has become the superpower of the world.
(d) China knows the present world order and takes steps keeping that in mind.

106. Which of the following conclusions can be reasonably drawn from the passage
above?
(a) Nationalism is one of the major force behind legitimisation of CCP’s authority
in China.
(b) West has become weaker as compared to China in the present world order.
(c) Xi is the most powerful leader China ever had.
(d) None of the above

107. Which of the following weakens the author’s arguments?


(a) China is trying to increase its influence on other countries of the world.
(b) Even after the phenomenal success and unprecedented growth rate of China,
poverty in China still
exist.
(c) The threat of Chinese acquisition of Taiwan still exist.
(d) None of the above

108. Which of the following is an assumption the author makes?


(a) Human rights and democratic values infringement still exist in China.
(b) CCP has been reasonably successful in strengthening economic prosperity of
China.
(c) Both A) and B)
(d) None of the above

109. If author’s arguments in the passage above are true, which of the following is
most likely to be true?
(a) West has always been and always will be most powerful in the world.
(b) West has started seeing a more powerful China as its rival.
(c) Western countries are more powerful as compared to China.
(d) All of the above

Passage(Q.110-Q.113): There’s a new virus in town and it’s not fooling around. You
can catch it through
face-to-face contact or digitally – that is, via a human or a bot, thanks to
technology. Few of us possess
immunity, some are even willing hosts; and, despite all we’ve learned about it,
this virus is proving more
cunning and harder to eradicate than anyone could have expected.
Misinformation isn’t new, of course. Fake news was around even before the invention
of the printing
press, although the first large-scale journalistic sham occurred in 1835, when the
New York Sun
published six articles announcing the discovery of life on the Moon (specifically,
unicorns and bipedal
beavers). Consider, too, early modern witch hunts, or those colonial myths that
depicted slaves as a
different species; the back-and-forth volleys of anti-Jewish and anti-German
propaganda during the world
wars, McCarthyism’s Red Scare, even communism’s utopian narratives. History teems
with deceit.
What’s different today is the speed, scope and scale of misinformation, enabled by
technology. Online
media has given voice to previously marginalized groups. The transmission of
falsehoods now spans a
viral cycle in which AI, professional trolls and our own content-sharing activities
help to proliferate and
amplify misleading claims. Moreover, these new developments have come on the heels
of rising
inequality, falling civic engagement and fraying social cohesion – trends that
render us more susceptible
to demagoguery. Just as alarming, a growing body of research over the past decade
is casting doubt on
our ability – even our willingness – to resist misinformation in the face of
corrective evidence.
Extracted with edits from Aeon.co:

110. Which among the following best describes the author’s arguments?
(a) The author believes that technology has brought with itself more misinformation
to previously
marginalized groups than ever before.
(b) The author talks about the scale of misinformation in the world of this century
and ways to mitigate its
negative effects.
(c) The author explains that misinformation has been around for a long time but
there is a difference
between misinformation of the past and today.
(d) The author highlights the alarming degree to which misinformation has spread
and how people have
lost their ability to resist misinformation.

111. Which among the following can be inferred from the passage?
(a) Our ability to resist misinformation in spite of corrective evidence may vary
from time to time.
(b) The perception that technology has acted as a catalyst for misinformation is
not free from errors.
(c) The New York Sun incident was one of the first incidents of misinformation to
have surfaced.
(d) An important reason for why misinformation has so much endurance is repetition
of fake news.

112. Which one of the following, if true, presents a strong confirmation to the
author’s arguments?
(a) Online media has given all the resources of deception to the peddlers of
mistruth.
(b) The government has been able to mitigate the effects of misinformation.
(c) A dip in ability to resist misinformation does not render us susceptible to
demagoguery.
(d) None of the above statements strengthen the author’s arguments.

113. If the information set out in the passage is true, which among the following
must be true?
(a) Misinformation can be spread by technology as well as by humans.
(b) The spread of misinformation cannot be altered by anything or any phenomenon.
(c) Both a and b
(d) Neither a nor b

Passage(Q.114-Q.117): Often misunderstood, too much sitting is not synonymous with


too little exercise.
In fact, a person considered highly active might sit far too much throughout their
day: picture an individual
who lifts weights in the morning and runs for two hours in the evening, but sits in
the car during his
commute and at a desk for seven hours throughout the workday. This individual
certainly meets physical
activity guidelines but might still be at risk for detrimental health outcomes due
to excessive sedentary
behavior. It appears that regardless of physical activity level, increased time
sitting corresponds to an
increased risk of mortality.
Though the undeniable association between higher risk for disease and greater time
spent sitting paints a
bleak picture, research investigating the manner in which sitting time is
accumulated offers hope. In one
study, one to two minute breaks from sitting correlated with smaller waists, less
insulin resistance, and
lower levels of inflammation. All of the aforementioned areas are risk factors for
detrimental health
outcomes including cancer and metabolic disorders. Together, these reports hint
that breaks in sedentary
time may become a primary method to protect against its adverse effects.
Despite the pervasive health risks associated with sitting and documented means to
temper them, even
workplaces, where people accumulate most of their sedentary time, have enacted very
few changes. On
the whole, employers do not brainstorm ways to lessen the time their employees
spend sitting and
employees do not clamor for opportunities to escape the confines of their office
chairs. Even hiring
replacement workers increase company costs, disrupt normal workplace flow, and
raise stress levels.
Considering the substantial blow that ailing employees can deal to companies, it
seems more likely that
this lack of workplace changes results from ignorance to company policy
modifications. Then, it is
probable that considerable workplace changes could take place following heightened
emphasis on the
sedentary behavior’s confounding health risks.

114. Which of the following is most supported by the author’s argument?


(a) Anyone who sits down for long will get cancer
(b) No one should sit down for more than an hour any given day
(c) Excessive sedentary behavior is impervious to regular exercise
(d) Some employer’s care about how much time their employees spend sitting down

115. Which of the following can be inferred from the passage?


(a) The impact of sedentary behavior can be controlled by frequent breaks
(b) The present system of working 8 hours a day needs to change
(c) Sitting down is worse when a person is getting some exercise during the day
(d) Both (a) & (b)

116. Which of the following, if true, would seriously undermine the author’s
argument?
(a) Majority of people that sit down in their day without a break are able to
produce more output than
those who don’t
(b) Majority of people that sit down in their day without a break are uncomfortable
with taking a break in
between
(c) Both (a) & (b)
(d) Neither (a) nor (b)

117. The statement “People who have a job that keeps them stationary are likely to
die earlier” is:
(a) Probably True (b) Definitely True (c) Probably False (d) Definitely False
Passage(Q.118-Q.121): As the Chinese Communist Party (CCP) turned 100, its supreme
leader and
China’s President-for-life Xi Jinping claimed his country had never bullied or
subjugated any other nation
nor would it ever do so. But the message that rang across the world was exactly the
opposite. The
reason is simple. Actions speak louder than words.
The CCP has much to be proud of and as much to be ashamed of. Millions of Chinese
perished in the
Cultural Revolution and the Great Leap Forward and an unknown number bore the brunt
of its fury in the
Tiananmen Square protests. Not to forget the millions of Uyghurs currently lodged
in detention camps in
Xinjiang. On the other hand, the CCP uplifted millions of Chinese from poverty with
its Open Doors Policy
or rather, the unnatural marriage of socialism with market economics —Socialism
with Chinese
characteristics. The father of this peculiar formulation, Deng Xiaoping, had
cautioned his comrades to
bide their time until the country became rich before flexing muscles on the
international stage. Three
decades on, Xi surely believes the time has come. China has been clocking a 9.8%
growth rate on an
average since 1979 and was the only country to register growth last year despite
Covid. Evidently the
most powerful Chinese leader since Mao, Xi has arrogated to himself immense power
by ensuring that
the two-term limit on presidency is removed, his own Xi Jinping Thought
incorporated in the Constitution
and the party purged of his opponents.
His reign since 2012 has seen China becoming more assertive, be it in the South
China Sea or the
borders with India, arm-twisting countries economically, upping the ante on Taiwan
and refusing to be
transparent about Covid’s origins. Xi’s Belt and Road Initiative, promotion of
authoritarian capitalism and
export of digital technology appear aimed at reshaping the world with China at the
center stage. Drum#beating nationalism at home and itching for confrontations
abroad, whether Xi’s tenure spells instability in
the CCP remains to be seen. He is definitely in the process of achieving that
abroad.
Extracted with edits from The New Indian Express:

118. Which among the following, if true, presents a strong confirmation to the
author’s arguments?
i. The CCP has committed some war crimes inside their nation in the last three
decades.
ii. The CCP has done some phenomenal work in getting millions of people out of
poverty.
iii. The CCP last year forcefully interfered with the internal affairs of the
nation Hong Kong.
(a) Both I and II (b) Both II and III (c) Both I and III (d) All I, II, and III

119. With which among the following views would the author of the passage be most
likely to agree with?
(a) The claims of Xi Jinping after the CCP turned 100 has substance.
(b) The throne of Chinese Emperor Xi Jinping is at risk of being taken over.
(c) The time for China about which Deng Xiaoping talked about has come now.
(d) The Chinese Communist Party has not done anything phenomenal during its tenure.

120. Which of the following is an assumption(s) based on which the author’s


arguments depend?
(a) Unusual partnerships can sometimes yield positive results.
(b) An average growth rate of 9.8 % per annum for over three decades is uncommon
for nations.
(c) Both (a) and (b) are invalid assumptions
(d) Neither (a) nor (b) are invalid assumptions

121. Which question has not been reasonably answered by the question in the
passage?
(a) For what works should the Chinese Communist Party be proud of?
(b) What changes has the CCP gone after Xi Jinping came to power?
(c) What are the threats to the power wielded by Xi Jinping?
(d) All the above

Passage(Q.122-Q.126): Dumbledore: In modern day Cricket, full-out aggression is


key. The previous
days’ “Gentleman’s Game” is gone. Take Virat Kohli for instance, he is known for
his aggression and he
has captained his side to the number one ranking team in the world. At a time when
the world is in a rat#race for finishing first, the urge to give it back or to get
under the skin of your opponent is the key. It’s a
dog-eat-dog world. It was due to Virat’s infused aggression that India could win
the Australian Test Tour
even with Virat missing the last 3 of the 4 test matches.
Merlin: Oh yeah? If that is the case then why did India not win the first test
match in which Virat was
captaining? They lost the test match then. See, a calm head is what sails you
through dangerous waters.
I cite Dhoni and Ajinkya. They are calm in demeanor but aggressive in tactics. Just
because you set up
an aggressive field for the opposing batsman, doesn’t mean that you have a license
to have a go at the
opposing batsman. Making the correct changes and taking the right steps matter,
aggressive or not. It
was no wonder that under Ajinkya, India won 2 out of those last 3 test matches.

122. What does Dumbledore mean when he says that “It’s a dog-eat-dog world”?
(a) The world of now is not that ethical. Winning is the key and for that
aggression can be used.
(b) How people behave with us, we should behave with them. If someone is
aggressive, we have to be
the same with them.
(c) Only aggression can be able to fight off aggression, otherwise it will
overpower you.
(d) All of the above.

123. Which of the following, if true, is a flaw in Merlin’s argument about the
first test match in which Virat was
captaining?
(a) Being aggressive in tactic means the person is aggressive. Even though the
demeanor is cool, that
person will still be called aggressive.
(b) It was due to bowlers’ dismal performance that India lost the match and nothing
else.
(c) Both (a) and (b)
(d) Neither (a) nor (b)
124. Which of the following is an assumption made by Dumbledore?
(a) Aggression was not a part of old days’ cricket
(b) Nowadays all the Cricket teams are aggressive
(c) Both (a) and (b)
(d) Neither (a) nor (b)

125. Which of the following is used by Dumbledore and Merlin to prove their
arguments?
1. Examples 2. Idioms and phrases 3. Analogies
(a) Only 1 and 2 (b) Only 2 and 3 (c) Only 1 and 3 (d) None of the above

126. Consider the following -


1. A marketing executive, to be successful, goes to the extent of contacting a
competing firm’s
employee and brings out many insider information of its marketing strategy.
2. The marketing executive wants to be successful. He remains calm and does not
lose his head when
he realizes his competition is way ahead of him. He calmly proceeds with the
existing strategy.
3. Thor was beaten by Thanos in the Infinity Wars. However, in the Endgame Wars, he
goes to battle
with Thanos again with the same strategy. But this time, he was less angry than the
first time.
4. Iron-Man understands it will be difficult to beat Thanos, as the latter is much
more powerful than him.
So instead of trying to fight him, he steals the infinity stones from Thanos – the
source of the latter’s
power and then fights him.
Which of the above is a suitable analogy for Merlin’s and Dumbledore’s arguments?
(a) Dumbledore – 1 and 2; Merlin – 3 and 4
(b) Dumbledore – 1 and 3; Merlin – 2 and 4
(c) Dumbledore – 2; Merlin – 3
(d) Dumbledore – 1; Merlin – 4

Passage(Q.127-Q.131):UP’s population policy for 2021-30 unveiled by CM Yogi


Adityanath wants to
incentivize couples to stick to a two-child norm. It’s an idea whose time has long
gone. The past decade
has witnessed a sharp fall in India’s total fertility rates, even in UP, India’s
most populous state. NFHS-4
data from 2015-16 indicates UP’s TFR fell to 2.7 in 2015-16 from 3.8 ten years
prior. Sample Registration
System report for 2018 pegged UP’s TFR slightly higher at 2.9 in 2018 but even this
was a fall of 23.1%
in a decade, bettering the national TFR decline of 18.5%.
Falling TFRs are massive demographic changes catalyzed by education of girls,
economic growth,
migration, falling infant mortality rates and higher institutional births. All
happened without drastic
population control policies. By all indications, UP will also hit replacement TFR
levels of 2.1 in due course
like other states.
Apart from being unnecessary, the approach taken by the UP Law Commission’s draft
Population
(Control, Stabilization and Welfare) Bill is also dangerous. It has prescriptions
portending a bureaucratic
nightmare for ordinary citizens, especially the poor. It requires devoting energies
to identifying state
employees and the general public who qualify for a raft of special incentives.
Disincentives barring
access to welfare schemes and even the PDS for larger families, though not
retrospective, poses
exclusion risks, massive corruption and social discrimination.
Badly conceived laws which trust bureaucracy with inordinate control over people’s
lives are a recipe for
disaster. Ironically, the poor, and especially UP’s rural areas where TFR is 3.0
against 2.1 in cities, will be
penalized for structural deficiencies in schooling, public healthcare, and
employment opportunities. The
potential for widespread social disruption can even undermine any political gains
Adityanath may expect
in the UP assembly polls from the draft bill’s ample references to polygamy. UP
should junk this idea.
Extracted from: The Times of India

127. What is the main conclusion drawn up by the author in the passage?
(a) The Uttar Pradesh government should junk its population policy for 2021-30.
(b) Incentivizing couples for having lesser children is a better population control
measure than penalizing
them for having more children.
(c) The population policy of Uttar Pradesh government will not serve any meaningful
purpose.
(d) Evidence suggests that women in India prefer having lesser children, if given a
choice.

128. Which among the following, if true, may weaken the author’s main argument in
the passage above?
(a) UP’s population is already coming under control so the population policy of
2021-30 will not have any
effect.
(b) Reliable data suggests that India is not being threatened by a ‘population
explosion’.
(c) Both a and b will not weaken the author’s main argument.
(d) Both a and b will weaken the author’s main argument.

129. Which among the following is an assumption made by the Uttar Pradesh
government?
i. Penalizing couples for having more than two children will encourage them to have
more children.
ii. Incentivizing couples for having less than two children will positively impact
the state’s population
numbers.
iii. The population control measure will ensure that the state’s population of a
particular community
declines.
(a) Both I and II (b) Both II and III
(c) All I, II, and III (d) None of the above

130. Which among the following is most likely to be inferred from the passage?
(a) Two-child norm might have been a reasonable measure for population control in
the past.
(b) The poor population of Uttar Pradesh will be made to suffer for their past
mistakes.
(c) Population control measures that may be dangerous are the way to go in the 21st
century.
(d) Results of educating married women about population control have not been
encouraging.
131. Which among the following has been used as a premise by the author to support
his claims?
(a) The TFR of Uttar Pradesh was 3.8 in 2005-06.
(b) The TFR of Uttar Pradesh is projected to be at 1.7 in 2023-24.
(c) Retrospective effects of the population control measure may trigger massive
social discrimination and
corruption.
(d) Both a and c

Passage(Q.132-Q.134): The Indian Medical Association (IMA) recently warned that a


third wave of the
pandemic was inevitable and imminent and appealed to the Central and State
governments not to let
down their guard against COVID-19. It urged them to not allow mass gatherings in
tourist and religious
places without following the COVID-19 appropriate behavior.
In a letter to the Prime Minister, the IMA said that while tourist bonanza,
pilgrimage travel and religious
fervor all were needed, they could wait for a few more months. According to it,
opening up rituals and
enabling people without vaccination to go scot-free in such mass gatherings were
potential super#spreaders of a third wave.
“The past experience of last one and a half years of war with the virus and based
on the emerging
evidences, it is obvious that by making the universal vaccination reach the maximum
possible population
and strictly adopting to Covid appropriate behaviors, we can face the third wave
with confidence and
mitigate its impact,’’ it stated. The IMA national president said, “It is the duty
and responsibility of
everyone at this moment to strictly enforce the Covid appropriate behaviors for a
minimum three more
months and ensure everyone gets vaccinated’’.
Extracted with edits and revisions from The Hindu

132. Which among the following can be concluded to be true according to the
passage?
(a) People who have been vaccinated would not contribute to the possibility of the
third wave.
(b) The author agrees with the concerns of the Prime Minister regarding the onset
of the third wave.
(c) The author believes that everyone should fulfill their duty of following Covid
appropriate behavior at
this moment.
(d) None of the above

133. Based only on the information set out in the passage, with which among the
following may be true?
i. Being complacent in following Covid-19 guidelines may invite the third wave.
ii. Pilgrimage travel and religious fervor should be postponed by a few months.
iii. The IMA is concerned about the possibility of a third wave.
(a) Only I and III (b) Only II and III (c) Only III (d) All I, II, and III

134. It is a mistake to see the electoral success of religious majoritarianism in


recent years as constructed in a
vacuum. Instead, it is precisely the decades-long compartmentalization of different
religious communities,
and the absence of solid state and civil society arrangements in educational
pedagogy, personal
relationships, workplace, to facilitate inter-cultural interaction, and based on
equality and respect, even
under supposedly secular regimes, that has made the soil politically fertile for
the demonization of the
minority, especially the Muslims.
Which among the following best summarizes the main idea of the text?
(a) Perceiving that the electoral success of religious majoritarianism was
constructed in a vacuum is
inaccurate.
(b) The demonization of the minority including Muslims should be stopped by
analyzing the success of
the majority.
(c) Facilitating intercultural interaction presents one of the strongest reasons
for the demonization of the
minority.
(d) Had there been the presence of civil society arrangements in various spheres of
life, the minority
would not be demonized.

mock 7
(Q.1-Q.5): There's a Japanese folk story about a crane who tricks a man into
marrying her by
pretending to be a woman. She knows that her beloved won't want her if he discovers
she's a bird, so at
night she stays awake and tears out her feathers with her beak. CJ Hauser wrote a
beautiful memoir
recently that called on the story: "To keep becoming a woman is so much self-
erasing work." It looks like
a deal. If you do the self-erasure, then in exchange you won't be alone. You
shouldn't take it. Partly just
because it's so horrifically sad. You say you're caring, capable, intelligent, and
you say these things with
an energy that tells me you love them about yourself. Don't pull them out at night.
But the other reason you shouldn't take the deal is that it doesn't do what it says
it will. Even if you paid
up by forfeiting your dignity you wouldn't get what you thought you were buying.
Love borne of pretence
doesn't make us any less alone. Whatever love you'd trick someone into giving you
would be love for an
act, and it's hard not to feel contempt for people you can trick like that. So, you
won't really like them, and
what they like won't really be you. It's a reverse Midas curse, turning what should
be gold into dust. You'd
get love in the same move that would make it worthless.
The thing is, I think you already know that. I love how you talk about yourself.
You know you're great. I
bet you have a drawer with a bunch of string and tape measures in it and a
particular way of banging
flour off your hands. You know you don't want to change. The real problem is that
when we realise the
choice between modifying and being alone was always just a choice between two ways
of being alone,
we're left with just being alone. I could say all the usual things about how love
comes when you give it to
yourself, and how the thoughts that say "no one will ever want me" are liars that
speak most loudly in the
silence after a recent love has closed the door.
But the truth is you might not get the thing you really want. You might not find
love with someone who
values you as you are. It's hard. What I do know is that women like you keep me
going; remind me of the
joy and the strength it takes to spend decades refusing to capitulate to a world
that wants you to be less.
There’s no way around the fact that it will hurt if being yourself means being
unloved, but love has been
wildly overstated as the key to a valuable life. It’s disappointing not to get it,
but it’s worse to disappoint
yourself. Keep your feathers.

1. The passage mentions a 'deal'. Based on the passage, which of the following
fact(s) about this 'deal'
is/are incorrect?
(a) The deal does what it says it will.
(b) This deal is so horrifically sad.
(c) As per this deal, if you do the self-erasure, then in exchange, you won't be
alone.
(d) All are incorrect.

2. For whom the author could have written this particular piece of writing?
(a) Kids
(b) Women
(c) Men
(d) Unmarried people

3. Which of the following words can replace the word "modifying" as used in the
passage's context without
changing the meaning or making the sentence grammatically incorrect?
(a) Alter
(b) Changing
(c) Finishing
(d) Asking

4. Which of the following word best expresses the meaning of the word "pretence" as
used in the context of
the passage?
(a) Facade
(b) Humility
(c) Alliance
(d) Undermine

5. Based on the passage, what is the reverse Midas curse?


(a) Turning everything into gold.
(b) Turning everything into stone.
(c) Turning what should be gold into dust.
(d) Turning everything into diamond.

Passage (Q.6-Q.10): That the public is isolated from the scientific world and there
are stark barriers an
ordinary person must overcome to get an education was recognized over a century
ago. As far as I can
tell, overcoming that isolation and those barriers still constitute our central
dilemma today. A common
solution is to make science less complicated and more entertaining. To this end,
there's been a steady
emphasis on communicating scientific ideas through stories that either dramatize
the working lives of
scientists or speculate about the impact of a scientific idea on society and
culture. Unfortunately, some of
these stories end up with the verisimilitude, complexity, and overall appeal of a
fairy tale. The simplicity
raises scepticism and alienates the reader even further. Obstreperous writers and
scientists have
identified some of the troubling trends in this way of conveying science to the
public. For example: the
fetishization of fMRI imaging and just the general credulity towards phrases like
"studies show" and "the
data says."
Science is a story about ideas - abstract, technical, complex, counterintuitive -
and all the energy and
profundity and drama of science comes from the interactions between these ideas.
These should stay
intact in responsible science communication, not glossed over.
What is to be done? My solution is simple: Encourage the reader to think for
oneself.
If the public can internalize a generalist understanding of science, they would
have a better chance of
finding science interesting on its own terms - not just because they were told ad
nauseam it was
interesting. They would feel a heightened amazement for the depth of knowledge
humanity has achieved,
and a greater reverence for what we don't know, or can't know. Personally, thinking
for myself about
science is the only way I've been able to make sense of it. Self-discovery - that
is, exploring, internalizing,
and modulating ideas - is the iron-nickel core of Enlightenment education; it's
supposed to be the way
human beings learn.

6. According to the author, what is the present day central dilemma?


(a) Barriers to education for the common man.
(b) General public is isolated from the scientific world.
(c) The profundity of science.
(d) Overcoming the public's isolation from the scientific community and barriers to
education.

7. As mentioned in the passage, which of the following qualities of a story


alienates a person from the
scientific concept being conveyed?
(a) Simplicity.
(b) Verisimilitude.
(c) Complexity.
(d) Fairy tale like appeal.

8. According to the passage, which of the following is/are the attributes of a


responsible science
communication?
(a) Phrases like "studies show" and "the data says", must have been used.
(b) It should be a little complex to make it believable.
(c) The ideas, whether complex, abstract, technical or even counterintuitive, must
be kept intact.
(d) The technical and counterintuitive ideas must be glossed over to make it
convincing.

9. As mentioned in the passage, which of the following is true about 'Self


Discovery'?
(a) It's the way humans are supposed to learn.
(b) It is the core of Enlightenment education.
(c) It is exploring, internalizing and modulating ideas.
(d) All the above
10. Based on the passage, which of the following is NOT true?
(a) Simplicity in scientific communication raises scepticism and alienates the
reader even further.
(b) Science is a story about ideas.
(c) Enlightenment is the path to self-discovery.
(d) The author has been making sense of science only by thinking about it himself.
Passage (Q.11-Q.15): When I am manning an information stall,' my beekeeper friend
Paul told me
recently, 'there's one question I'm always asked: "Bees are in trouble, aren't
they?" I tell them:
"Honeybees in the UK, not really; honeybees in the US, yes; wild bee species
globally, yes." Then I cram
in as much detail as I can about the biodiversity crisis before their eyes glaze
over and they wander off in
the direction of the ice-cream van.'
Paul does himself an injustice - I could listen to him rattle on for hours. He is
one of those people with a
knowledge gleaned from years spent watching the bees in his garden, watching the
fields and
hedgerows, chewing the fat with other beekeepers, and sifting through endless
articles and research
papers online. He makes the information interesting, by which I mean he adds colour
to it, adds
experience, adds story, shoehorning little anecdotes and details that make
vertiginous issues such as
biodiversity collapse pressing in a good way - a rallying way, a way that makes one
want to step up and
take note and make a difference.
He is right about honeybee populations, too. Apis mellifera, the western honeybee,
is a member of the
genus Apis, known for its product on of wax and honey, and for the fact that it
lives collectively, as part of
a colony. While losses in the US and Europe over the past two decades have focused
attention on the
plight of this species, at least in the UK and Europe populations now appear
relatively stable.
In the US, where honeybees were brought over with European settlers in the 17th
century, the situation is
more complicated. Beekeepers report winter losses of around 30 per cent (this rose
to nearly 40 per cent
in the winter of 2018-19, the highest figure ever recorded in the 13 years since
the survey commenced).
So, although total hive numbers have improved since the declines brought about by
Colony Collapse
Disorder, this is being achieved mostly through increasingly invasive practices:
beekeepers offset losses
either by replacing colonies or 'splitting' existing ones, taking a portion of
eggs, larvae, bees and food
stores from a healthy colony and placing them in a new hive with a queen reared
specially for the
purpose. Since the reasons driving the high loss rates remain unaddressed,
beekeepers are essentially
having to work harder now than they were a few decades ago just to 'stay in one
place'.
11. The author will agree the most with which of the following statements?
(a) It is important that information should reach to people in an interesting way
to make a piece of
information more acceptable.
(b) Humans are voluntarily concerned about environment, especially biodiversity.
(c) Paul's experience in bee keeping and his reading online articles and research
papers does not make
him an expert on Bees.
(d) Paul is a mediocre orator.
. Page 5 of 40
12. Which of the following finds support from the given passage?
(a) Author is the only friend that Paul has.
(b) Only author believes that Paul knows a lot about bees.
(c) People are not at all concerned about Bees.
(d) People believe that Paul exaggerates or embellishes knowledge of bees.
13. “Then I cram in as much detail as I can about the biodiversity crisis before
their eyes glaze over and they
wander off in the direction of the ice-cream van.” We can infer from the statement
that?
(a) People's concern towards biodiversity is permanent.
(b) The excess information provided by Paul on biodiversity overwhelms people.
(c) The excess information provided by Paul doesn’t interest people.
(d) People are not ready to face the truth about bio-diversity.
14. Which of the following is similar in meaning to the word 'plight' in the
passage?
(a) Boon
(b) Munificent
(c) Replica
(d) Quandary
15. According to the passage, how the beekeepers stabilize the beehive numbers?
(a) They make artificial beehives and set up the colonies there.
(b) They either replace new colonies in the vacant hives or split the colonies into
two and thus form two
hives.
(c) They provide habitat to the wandering bees or those who have lost their way
from their colonies.
(d) None of the above.
Passage (Q.16-Q.20): The huge slabs of stone that make up the most iconic
structures at Stonehenge
came from about 25 kilometres away, according to chemical analysis. Since the
1500s, most Stonehenge
scholars have assumed the 6- to 7-meter-tall, 20-metric-ton sarsen stones came from
nearby
Marlborough Downs, and a recent study by University of Brighton archaeologist David
Nash and his
colleagues has now confirmed that.
Recent studies have traced Stonehenge's bluestones to quarries in the Preseli Hills
of western Wales,
about 300 kilometres (200 miles) away. When another group of archaeologists studied
the chemical
isotope ratios in the cremated remains of people once buried beneath the
bluestones, those researchers
found that many of those people may have come from the same part of Wales between
3100 and 2400
BC. Ancient builders set up the sarsen stones a few centuries after the arrival of
the bluestones. Modern
scholars have only been able to speculate about where the huge boulders came from -
until now.
Sarsen, also called silcrete, is a sedimentary rock mostly made up of quartz sand
cemented by silica
(quartz is just silica in crystal form), formed in layers of sandy sediment. Thanks
to erosion, sarsen
boulders are now scattered in clumps all over southern England. Prehistoric Britons
built monuments like
Stonehenge and Avebury with sarsen boulders, Roman settlers used sarsen bricks to
build their villas,
and medieval people-built sarsen churches and farm buildings. But the largest
sarsen boulders we know
of in Britain today are the ones at Stonehenge.
About 99 percent of the average sarsen boulder is silica, but the other 1 percent
contains trace amounts
of other elements, like aluminium, calcium, iron, potassium, magnesium, and others.
That extra material
is different in different sarsen sources, as it depends on the minerals in the
ground where the rock
formed. Nash and his colleagues used those trace elements as a geochemical
fingerprint to match the
Stonehenge sarsens to their most likely source.
The largest concentration of sarsen in the UK is at Marlborough Downs, an area of
round, grassy hills 25
to 30 kilometres (17 miles) north of Stonehenge. Centuries of archaeologists and
antiquarians have
assumed the Stonehenge sarsens came from the Marlborough Downs, mostly because the
area is
nearby and full of the right material. But that idea had not been scientifically
tested, and the bluestones
. Page 6 of 40
demonstrate that the Neolithic people who built Stonehenge had a far-flung and
complex supply network
- and their own reasons for doing things, often inscrutable to modern researchers.
16. Which of the following is correct as per the given passage?
(a) Most of the times the assumptions made by scholars are found to be correct by
archaeologists.
(b) Sarsen boulder always contains more than five elements.
(c) Chemical analysis plays an important role in tracking the source of origin of
the material used in
building a structure.
(d) The largest concentration of sarsen is usually found in an area of round,
grassy hills.
17. Which of the following is similar in meaning to the word 'speculate' as
mentioned in the given passage?
(a) Abstain
(b) Indulge
(c) Retort
(d) Contemplate
18. Which of the following is the correct definition of Silcrete?
(a) It is a rock formed by sedimentary particles which are a mixture of quartz sand
and silica.
(b) It is a rock made of quartz sand cemented by silica formed in layers of sandy
sediment.
(c) It is a sandy sediment's rock which is highly made up of quartz sand cemented
by silica.
(d) All of these
19. Which of the following is the assumption made by the archaeologists and
antiquarians in the sentence
given below?
Centuries of archaeologists and antiquarians have assumed the Stonehenge sarsens
came from the
Marlborough Downs, mostly because the area is nearby and full of the right
material.
(a) Marlborough Downs is a nearby and accessible location for the material.
(b) Archaeologists and antiquarians have worked for centuries to learn about
Stonehenge sarsens.
(c) Easy accessibility of the required resources is an important factor in
constructing a structure.
(d) None of the above
20. Which of the following sentences talks about the extensive use of Sarsen?
(a) Prehistoric Britons built monuments with sarsen boulders, Roman settlers used
sarsen bricks to build
villas, and medieval people used sarsen to build churches.
(b) About 99 percent of the average sarsen boulder is silica, but the other 1
percent contains trace
amounts of other elements.
(c) The Neolithic people who built Stonehenge had a widely distributed and complex
supply network for
Sarsen.
(d) Sarsen boulders are scattered in clumps all over southern England.
Passage (Q.21-Q.25): The idea of a cafe, or “coffee house” as it was known before,
has fascinated me
since I learnt of its reputation as a vibrant public space where politicians,
doctors, intellectuals, writers
and artists gathered in 18th-Century London. But no amount of cafe-hopping has
resulted in success at
finding the perfect public space for writing, conversations and coffee.
When I find myself at Writer’s Cafe in Gopalapuram, I believe I am closer to
finding that elusive ‘perfect’
cafe. Conversations may be hard to come by, as it is just the initial few weeks
since the place opened,
but a good coffee is just an order away.
Writer’s Cafe has many firsts. The non-profit cafe has been started by restaurateur
M. Mahadevan,
chairman. Oriental Cuisines, in collaboration with Higginbothams, as the legendary
bookstore re-invents
itself.
The lower floor of the two-storied building has wooden tables on tiled floors, and
walls lined with book
shelves, and is just a fraction of the impressive collection on the upper floor,
where Higginbothams has
made at least half of its 18,000 titles available.
. Page 7 of 40
The books on display cover popular fiction, cookbooks, fashion, coffee table books
and magazines. A
dream come true for any voracious reader who can plonk on a comfortable couch and
satiate an appetite
for both good coffee and books here. Karan Manavalan, chef and manager, points out
that apart from
being a space for people to read, write and dine, the place is also conscious about
giving back to society.
All the profits go back to the International Foundation for Crime Prevention and
Victim Care (PCVC) and
to support burn victims with a hope that these women will find a way back into
meaningful employment.
Karan recommends a siphon coffee, and before I know it, a vacuum coffee pot with a
flame underneath
the lower pot is placed on the table. As the water boils, the vapour rises to the
chamber above and a
delectable decoction is formed.
For those visiting Writer’s Cafe for the express purpose of satiating their hunger,
the Swiss-inspired menu
is a novel treat.
As I dig into the flammkuchen pizza, the cheese and pesto fill my mouth as the thin
crust breaks at the
touch of my teeth. The base for the flammkuchen is made in the state-of-the-art
facility at the Cafe, with a
prominent Swiss influence.
I also try out the orange and rocket leaves with ricotta salad, the crunchy leaves
teaming with the sweet
oranges and balsamic dressing to make for a hearty appetizer.
Americanized dishes temper the otherwise interesting Swiss menu and a wide offering
of popular
appetizers, including a variety of fries and toasts, is also available for those
unwilling to take risks with
their orders. Truly a manna from heaven!
21. What was the author's idea of a coffee house?
(a) A cosy corner for conversations with friends, quiet writing with a steaming cup
of coffee by the side.
(b) A perfect replica of the 18th century London public space.
(c) A vibrant public space where politicians, doctors, intellectuals, writers and
artists gathered for a tete#a- tete over a steaming cup of coffee.
(d) A very reputable vibrant public space.
22. What are the 'specials' of the Writers Cafe?
(a) It is a non-profit organization.
(b) Higginbothams is an associate of the cafe.
(c) It offers reading along with coffee.
(d) All of the above
23. How is the Writer's Cafe linked to a social cause?
(a) A non-profit organization runs it.
(b) Its profits are ploughed towards the rehabilitation of burn victims.
(c) Its profits are channelized to a Foundation. k Your answer is incorrect
(d) The profits are not re-invested.
24. In a lighter vein, what would be the ideal and charming title for this passage?
(a) Writer's Cafe
(b) Koffee Express
(c) Coffee with Karan
(d) Intellectual coffee
25. Which of the following is false with respect to the given passage?
(a) Writer's Cafe offers books with coffee for the voracious reader.
(b) Siphon coffee is a special variety of coffee.
(c) Writer's Cafe serves only steaming coffee.
(d) The author is a gourmet with a special love for coffee.
. Page 8 of 40
Passage (Q.26-Q.30): New beginnings are often exciting, yet terrifying- no one
knows what the future
holds. Unlimited possibilities lie behind the door to your new beginning: successes
and failures, ups and
downs, and even some smiles and frowns.
The past is sometimes easier to reflect upon because it is finite, it is done and
cannot offer new surprises
in your life. In comparison, the present and future seem mystical and
uncontrollable. A toolkit for your
attitude may be what it takes to make a smooth transition.
Sometimes you may have to “fake it till you make it” with a positive attitude in a
transitional time. A new
chapter can be terrifying, which brings in negative self-talk. It is common to have
doubts and hesitation;
however you cannot let these thoughts win the battle and defeat you. You may want
to mink of a positive
slogan to get you through the tough transition. For instance, “l have got this!”
may provide you with an
additional boost of confidence and encouragement.
In a transitional time, it is important to be open to new people and new
experiences. To adjust to a new
job, new partner etc. it takes an open mind. You cannot compare your past with your
future. For instance,
if you are in a new relationship, you cannot compare your new partner to your past,
idealized partner,
because if you do, you are setting yourself up for failure. Your present and future
offer you the
opportunity to see things in a new light or try things in a different way.
You may have lofty goals for your future, which is great as long as they are also
realistic. When you enter
a new chapter, it is important to have measurable and realistic stepping stones to
get you to your future
goals. By setting realistic and measurable goals, you are more likely to achieve
them.
Sometimes it just takes time (which can be tough if you’re impatient). New
beginnings are a test of will
power patience and character. You need to give yourself the freedom to fall, stand
up, brush off and try
again. It is only through these mistakes that you can learn how to do things
differently and more
effectively. Persistence pays off because it creates dedication and sweat equity.
At transitional times,
think about the Pareto Principle, or the 80-20 rule, where 80 percent of the
results come from 20 percent
of the effort. Through awareness and persistence, you can tackle the challenges of
a new beginning.
Everyone needs support from loved ones when transitioning into a new chapter
Sometimes you may shy
away from support because you feel like a failure; however, this is often the time
you need support the
most from your friends and family. So, don’t hesitate. Just seek it!
26. Why do you think the author puts forth the idea of ‘a toolkit’ for your
attitude?
(a) A toolkit contains tools with different shapes and sizes, and one cannot be
used in another’s stead.
(b) A toolkit is a combination of differently shaped and sized tools appropriate
for different situations; our
attitude being equated to these tools.
(c) A toolkit is an indispensable thing in our lives, and so is our attitude.
(d) Attitude is like a toolkit helping us tide over difficult situations in our
life.
27. What can the author mean by ‘Fake it till you make it?
(a) To pretend something which you are actually not till the pretence becomes real
for a positive purpose.
(b) Even though you do not have the confidence, you act as if you are full of
confidence.
(c) To fake emotions which you really don’t feel to convince others that they are
for real.
(d) To put on a brave face and to pretend to be happy.
28. According to the passage, why could a new chapter be ‘terrifying’?
(a) A new chapter may have a lot of inflows of negative emotions.
(b) Facing new people and new experiences is always a challenge.
(c) The fear that you may not make it is always haunting.
(d) All the above
29. In what way is the future different from the past?
(a) The past is more familiar, whereas the future has negative possibilities.
(b) The past is predictable, and the future is unpredictable.
(c) Past is limited, and all known, and future is full of unknown.
(d) Past is full of negative experiences, and the future is full of smiles.
. Page 9 of 40
30. Which of the following is untrue?
(a) The Pareto Principle works on the 80-20 rule.
(b) Mistakes are often stumbling blocks for success.
(c) Love from near and dear gives strength to face failures.
(d) Transition and patience are directly proportional.
Directions (Q.66 – Q.105): Read the comprehensions carefully and answer the
questions based on it.
Passage(Q.66-Q.69): The Supreme Court has allowed a Delhi doctor to pursue higher
studies at a
medical college after he was denied admission due to the Delhi Government's
decision to not grant study
leave to doctors during the covid-19 pandemic. The petitioner doctor had completed
five years of service
at Deen Dayal Upadhyay hospital in Delhi. As per the rules, the doctors who
complete five years of
continuous service can avail study leave for higher studies, provided they are not
going to retire after five
years of study and that they would re enter duty. In this case the doctor cleared
the Institute of National
Importance Combined Entrance Test in 2020. A No-Objection certificate was also
issued by the Delhi
Government. However after the counseling at post graduate Institute of Medical
Education and Research
Chandigarh, to pursue MD Pediatrics the doctor was denied study leave. The
government had come up
with a policy in October 2020 wherein the provision of study leaves would be
granted to doctors amid the
covid-19 pandemic. The Appeal was preferred in Supreme Court. The bench noted that
it would be unfair
to deny the appellant the opportunity to enjoy the fruits of his efforts even now
when the covid-19
situation has improved and is in control only because the respondent had not
committed “Apparent
breach of rules and regulations in refusing the appellant study leave”. “Unless
there is a substantial rise
in covid-19 cases the leave application of the appellant shall not be declined”,the
court said. The court
also made it clear that its direction in this case “will not be treated as a
precedent.”
Source name - Bar and Bench
66. A was a meritorious practicing doctor in the hospital ABZ Health in the state
of Dailhy. She has been
practicing as a doctor in that hospital for seven years. However, due to an
inevitable operation of her
spinal cord, she has to take a year rest in the fourth year of her service after
which she continued her
practice. After she has given her service for more than five years, she applied for
the study leave for
which she could pursue her higher studies construing to which the government
allowed her application
and granted her the study leave. In the light of the above passage, choose the most
appropriate option.
(a) Government took the right decision by granting A the study leave as she has
been in service for more
than five years and has been a meritorious, thus should not be barred from grabbing
this opportunity.
(b) Government granted the study leave to A which is a generous gesture, is totally
a helping gesture for
the meritorious students to excel and provide best of their service.
(c) Succumbing to an inevitable operation of spinal cord, A had been through a
break of one year, that is
in the fourth year, failing to serve for continuous five years, should not be
granted the study leave.
(d) As per the guidelines of the Government, A should not be granted the study
leave but taking note of
an inevitable circumstance, the government should consider some other neutral way.
67. D was a doctor practicing in ABM Hospital in the state of Daylhi which was one
of the most contaminated
zones in that time with increasing death rates. Having been in service for more
than five years
continuously in that hospital, he also aspired for higher studies and thus applied
for the study leave. He
was not granted any study leave. However, he merely passed the exam but was denied
to get admission.
He filed a suit against the government of the state. In the light of the passage,
choose the most
appropriate option.
(a) D must be given the study leave as he had been in service for continuous five
years.
(b) D had the caliber to legitimately pass the exam amid the tough situation in
Covid 19 and thus must be
given the leave for higher studies.
(c) The state government’s decision was agreeable of not granting the leave amid
the rising cases in the
pandemic.
(d) State government’s decision was right because mere passing of exam is not at
all enough for the
doctors as it would hamper the quality of services provided by them in this
pandemic
. Page 17 of 40
68. In this paragraph, the bench has clarified that the decision given in this
paragraph cannot be considered
as a precedent. Choose a subtle explanation.
(a) Construing to the guidelines of the state government, the right procedure
should be followed at all
cost.
(b) Instead of any unprecedented circumstances, the doctors should not be given any
break for higher
studies’ leave from their services amid the pandemic situation.
(c) If cases are in control during the pandemic and the student is extraordinarily
talented, giving some
concessions from the guidelines of the state government can be considered.
(d) Doctors must be given a fair chance to pursue their higher studies amid the
pandemic if the situation
is in control and thus study leaves must be granted to them.
69. P was a doctor practicing in a particular hospital in the pandemic period. He
aspired to get higher
education after five years of his continuous service. He made up his mind for
retiring after he pursued his
higher studies and start his own clinic. He applied for the study leave but his
application was denied. In
the light of the above passage, choose the most appropriate option.
(a) Starting own clinic can never be a good option for the patients in the ongoing
pandemic.
(b) As per the guidelines of the state government, P can get a study leave only
when he will return to his
duty after completion of five years of studies.
(c) As per the guidelines, the state government must give him the study leave to
pursue his higher
education because he has completed his five years of service in the hospital and is
eligible to get the
benefit.
(d) Starting own clinic may lead to various profit making malpractices during the
pandemic and may lead
to certain financial and psychological exploitation of the patients.
Passage(Q.70-Q.73): Gujarat’s new law has followed the suit of the other BJP-ruled
states of Uttar
Pradesh and Madhya Pradesh which introduced similar amendments to severely penalise
and regulate
interfaith marriages. The new amendment to the state’s Freedom of Religion Act,
2003, laysdown stricter
punishment for forced conversion through marriage.
While prohibiting conversion through “allurement” or “temptation”, the Act defines
both these ambiguous
terms in such open-ended phrases that it makes the entire provision fertile for
political exploitation. The
Amendment Act introduces the third ground of prohibition – forcible religious
conversion through
marriage.
The amendment act has added more categories to the list of persons who can complain
about instances
of “forced conversion”. Now, under Section 3A, any “aggrieved person” – parents,
brother, sister, or any
other person related by blood marriage, adoption – may file a complaint at the
police station.
Now, “any person who does or omits to do any act for enabling or aiding other
person to commit
fraudulent conversion will be penalised”. The net of punishment is also extended to
those who “counsel”
or “convince” people to commit an “offence” under this Act.
[Extracted from 'Decoding Gujarat’s Draconian New Anti-Conversion Law' by Karan
Tripathi, published
15
70. Julia’s father Junaid had adopted her as a child, when he saw her abandoned at
a fire station. When
Julia grew up, she fell in love with Cyril and decided to marry him. Her father was
against the marriage
because Cyril did not pray to the same God. However, Cyril and Julia went ahead and
got married
anyway. Post her marriage, Julia changed her religion to that of Cyril’s on Cyril’s
insistence. Can her
father file a complaint against this conversion?
(a) No, because Julia has the right to follow the religion of her choice and she
got married voluntarily
(b) No, because Julia has not converted to a different religion by force and
instead chose to do it
voluntarily
(c) Yes, because Julia’s father is an aggrieved person as his daughter converted to
a different religion
post her marriage
(d) None of the above
. Page 18 of 40
71. Romeo and Juliet belonged to different religions but they longed to get
married. Their friend Rajnish
ordained himself online to help them get married. In order to marry each other
traditionally, they had to
belong to the same religion. Romeo did not want to convert to Juliet’s religion and
vice versa. Juliet
asked Rajnish to convert Romeo to her religion with the power vested in him by his
recent ordainment.
Rajnish suggested Juliet to convince Romeo with rosy promises so that he would
agree to convert. Can
Rajnish be held liable for forceful conversion?
(a) Yes, because Rajnish enabled and aided Romeo in fraudulently converting Juliet
for marriage.
(b) Yes, because Rajnish enabled and aided Juliet in fraudulently converting Romeo
for marriage.
(c) No, because Rajnish did not enable or aid Juliet in fraudulently converting
Romeo for marriage.
(d) No, because Rajnish did not enable or aid Romeo in fraudulently converting
Juliet for marriage.
72. In the previous situation, in order to convince Romeo to get married, Juliet
decided to take Rajnish’s
advice. She took Romeo out for dinner and showed him a rosy picture of life after
marriage. She also
promised him that she would never say no to him for anything and never leave him
for someone else. In
the end, she tried to manipulate Romeo by saying that the only way the two of them
could ever stay
together forever, was if Romeo converted to her religion. Is Juliet in violation of
the principles of the
passage?
(a) Yes, because the net of punishment is also extended to those who follow advice
in order to convince
people to get converted.
(b) Yes, because she is trying to lure and tempt Romeo in order to convince him to
convert to her
religion.
(c) No, because she is merely acting on the advice or Rajnish and is herself not in
violation of any law.
(d) None of the above.
73. Shakuni represented the Church. In order to increase the number of church
visitors, he conducted an
online workshop for people from different religions. Therein, he told people that
praying in a Church
makes God hear wishes sooner and as an act of kindness, all church-goers would be
given a supply of
1kg rice for the rest of their life. Impressed with the claims made by Shakuni in
the workshop, several
people converted to Christianity. Can Shakuni be held liable for forceful
conversion?
(a) Yes, because he lured and tempted people to convert by offering them a free
supply of rice for the
rest of their life.
(b) Yes, because he tried to manipulate people by making claims as to the power of
praying in a Church
so as to make God grant wishes sooner.
(c) Both (a) and (b).
(d) No, because Shakuni did not force anyone to convert to Christianity and people
converted on their
own accord.
Passage(Q.74-Q.77): The notion of legal literacy is based on the principle that
every individual must be
aware of their rights and obligations. The maxim 'ignorantia juris nonexcusat,' or
'ignorance of the law is
no excuse,' implies that the Court presumes that every party is aware of the law
and hence cannot claim
ignorance of the law as a defence to escape liability.
The essential public character of a law requires that the law, once properly
promulgated, must apply to
anyone in the jurisdiction where the law applies. Public policy requires that
ignorance of law should be no
excuse. But there is no presumption that everybody knows the law, though it is
often so stated.
The preconceived notion that the law is innately complex and can only be understood
by people
belonging to the legal fraternity only, is a major impediment to legal literacy. It
is only reinforced by
judicial pronouncements that are unnecessarily lengthy and verbose. Another reason
is the legislature's
lack of effort to make the language of laws more accessible to people.
Accordingly, for this maxim to become valid, promulgation of law is of utmost
importance. Further, the
intended audience must be capable enough to understand and know the legislated law.
These
preconditions appear to be quite idealistic but not practical and so is the maxim.
[Extracted with revisions from ‘Ignorance Of Law In India: An Urgent Need To Fight
Legal Illiteracy' by
Akrama Javed & Subodh Singh, published 17 June 2021 on live law
. Page 19 of 40
74. X was an illiterate and simple-minded farmer in a remote village. One day,
while he was tending to his
crops, a buffalo appeared out of nowhere and started trampling on his crops. He
tried to shoo the animal
away from a distance but the buffalo would not budge. Consequently, in order to
save his crop, X shot
the buffalo. Shooting wild animals is illegal under the Wildlife Protection Act,
which was formulated with
due process and published in the gazette in both Hindi and English. Can X plead
defence?
(a) No, because ignorance of the law is no excuse and every person is assumed to be
aware of the law.
(b) No, because public policy requires that ignorance of law should be no excuse.
(c) Yes, because Wildlife Protection Act is not applicable in the present case.
(d) Yes, because X was capable enough to understand and know that killing a buffalo
is a crime.
75. Piramal was a German smuggler. He left Zurich by plane on 27th November 1962
with 34 kilos of gold to
smuggle it into Manila. The plane arrived in Bombay on the 28th but Piramal did not
come out of the
plane. The Customs Authorities searched Piramal when they found that no gold had
been declared by
him and yet he was carrying it in his luggage. It is illegal to smuggle gold in
most countries of the world.
Can Piramal cite ignorance to escape punishment?
(a) Yes, because Piramal was not capable enough to understand and know the law
which prohibits
smuggling of gold as he was German.
(b) Yes, because Piramal was capable enough to understand and know the law which
prohibits
smuggling of gold.
(c) No, because Piramal is presumed to be aware that it is illegal to smuggle gold
and he cannot claim
ignorance of the law.
(d) None of the above.
76. Johnny, a foreigner, killed Janinder in a duel in India. He believed dueling to
be lawful in India. At the time
the law against dueling was passed in India, Johnny was at sea. He argued that he
could not possibly
have been aware of it. Johnny engaged in the duel 2 months after coming back from
his vacation. Can he
cite ignorance of the law?
(a) Yes, because he was incapable of being aware about the existence of such law.
(b) Yes, because he was incapable of being aware regarding the promulgation of such
a law.
(c) No, because he was capable of being aware of the existence of such law.
(d) No, because judicial pronouncements are unnecessarily lengthy and verbose.
77. Ram Swarup was the Head Constable of Kaira Police Station. While investigating
the case of a fake
doctor, he mistakenly arrested a genuine homeopathy doctor. When he found out that
it wasn’t the right
person, Ram Swarup released him from custody. The doctor filed a case against Ram
Swarup for
arresting him without a warrant. Can Ram Swarup cite ignorance?
(a) Yes, because it was an honest mistake and the doctor was subsequently released
from custody.
(b) Yes, because there can be no presumption that everybody knows the law, though
it is often so stated.
(c) No, because there can be no presumption that everybody knows the law, though it
is often so stated.
(d) No, because Ram Swarup was aware that arresting a person without a warrant is
illegal and he
cannot claim ignorance to escape liability.
Passage(Q.78-Q.82): The Calcutta High Court has ruled that voluntary gifts given by
relatives and
friends before or after the wedding to the bride or the bridegroom and which are
not given as a
consideration for marriage but out of love and affection, will not fall within the
definition of 'dowry' under
the Dowry Prohibition Act. Thus, voluntary presents given at or before or after the
marriage to the bride or
the bridegroom, as the case may be, of traditional nature, which are given not as a
consideration for
marriage but out of love, affection on regard, would not fall within the mischief
of the expression 'dowry'
made punishable under the Act, the Court said.
The complaint stated that at the time of marriage, Nirmal Ghosh had given bridal
presents as per his
financial capacity. It was decided before marriage that he would also give a gold
chain to the accused
within six months of the marriage. However, immediately after marriage, the
husband, his mother and
. Page 20 of 40
sister of the husband started abusing her. It was also stated that she was
subjected to physical assault.
On August 4, 2011 the complainant came to know that the daughter had died by
consuming poison.
The court then moved on to explain the definitional tenets of the word dowry. It
said “any demand of
money, property or valuable security made from the bride or her parents or other
relatives by the
bridegroom or his parents or other relatives or vice versa would fall within the
mischief of “dowry” under
the Act where such demand is not properly referable to any legally recognized claim
and is relatable only
to the consideration of marriage.”
Zeb Hasan, Voluntary Gifts given by parents, friends to bride, bridegroom out of
love and affection not
dowry: Calcutta High Court (Bar and Bench, 25 June 2021)
78. Pradeep and Manisha were soon about to be happily married at a grand function
organised by Manisha’s
parents. Before the wedding, the groom’s family made a demand that some of their
close relatives shall
be each given a gold coin, as they have not demanded anything for themselves.
Manisha’s parents
fulfilled the demand and the function concluded successfully. However, when Pradeep
and Manisha
realised about the coins they both filed a complaint under the Dowry Prohibition
Act against Pradeep’s
parents for dowry demand. In the court, the parents claimed that they have not
asked for even a penny
from Manisha’s parents for themselves and therefore are not guilty of the offence.
Choose the correct
option.
(a) Pradeep’s parents shall be found guilty for dowry demand as even when a gift is
given following a
demand, though voluntarily given, qualifies as dowry.
(b) Pradeep’s parents shall not be found guilty for dowry demand as rightly argued
by them that the gifts
were given to the relatives and they demanded nothing for themselves.
(c) Pradeep’s parents shall be found guilty for dowry demand as the coins were
gifted to their relatives
post their demand as a consideration of marriage.
(d) Pradeep’s parents shall not be found guilty for dowry demand as even though the
demand for coins
was made by them, Manisha’s parents voluntarily agreed to gift the coins to the
relatives.
79. Seema was reported deceased after three weeks of her marriage. The autopsy
report suggests that she
was persistently subjected to cruelty which led to her demise. Sumit, her husband
was charged for the
dowry death of Seema, considering past demands for dowry. The prosecution argued
that Sumit had
demanded Seema’s parents to transfer any substantial value to him as per societal
standards at the time.
Contesting the allegations Sumit submitted, that there was no agreement as to
transfer of any definite
value, hence proving that there was no demand for dowry. The money transferred was
a voluntary gift
given by Seema’s parents. Without considering whether Sumit is guilty or not,
decide whether the court
can hear the case as the pertaining to dowry death.
(a) The case shall not be heard as a case of dowry death because there was no
demand for any
particular value from the side of Sumit for which he could have subjected Seema to
cruelty.
(b) The case shall be heard as a case of dowry death because agreement as to a
particular amount is
irrelevant as long as there is demand for money as a consideration for marriage.
(c) The case shall not be heard as a case of dowry death because in absence of any
specific demand
any value transferred by Seema’s parents to Sumit is voluntary in nature.
(d) The case shall be heard as a case of dowry death because the incidents of
cruelty right after the
marriage is an evidence of dowry demand by Sumit.
80. Assertion (A): Gifts, money and property given voluntarily does not qualify as
dowry.
Reasoning (R): All gifts that are not given as consideration for marriage but out
of love and affection are
not actually dowry.
Choose the correct option
(a) Both A and R are true, R is the correct explanation of A
(b) Both A and R are true, R is not the correct explanation of A
(c) Both A is correct and R is incorrect
(d) A is incorrect and R is correct
. Page 21 of 40
81. Mayank was a corporate accountant, who was getting married to Shweta, a lawyer.
After the marriage,
Mayank made demands regarding his share in Shweta’s father’s property, to which he
was legally
entitled as per the law of the land. Enraged by such incessant demands, the father
pressed charges
against Mayank for dowry demand. Upon conviction by the Trial Court, Mayank has
moved to the High
Court for setting aside the order and acquitting him of the offence. Which of the
following is the correct
answer?
(a) The appeal shall be allowed as the husband of Shweta, he is morally entitled to
a share in the
property that would be distributed amongst all his children.
(b) The appeal shall not be allowed as the share in the property is a demand for
dowry which is a
criminal offence under Dowry Prohibition Act.
(c) The appeal shall be allowed as the demand for share in the property is a legal
claim and not a
consideration for marriage.
(d) The appeal shall not be allowed as even though the demand is legal, Shweta’s
father does agree to
voluntarily give the share in the property to Mayank.
82. Against marrying her daughter to Kumar’s son, Sadhna posed a condition of
making her daughter a
partner in Kumar’s business. Kumar agreed happily to the condition and both the
families proceeded with
the wedding. After their marriage, the daughter sold all her share in the business
and separated from the
Kumar’s son. Kumar, enraged by such actions, filed a complaint for dowry demand on
Sadhna and her
daughter. The court refuses to allow the case as one of dowry demand because the
partnership was
voluntarily given as a gift for the wedding. Decide whether the court has given the
right judgment or not.
(a) Court has not given the right judgment as the demand made by bride’s family
does not qualify as
dowry within the legal tenets of law.
(b) Court has given the right judgment as there has been a demand of value as a
condition to marriage
making it a consideration for marriage.
(c) Court has not given the right judgment as Kumar not only voluntarily but also
quite happily made
Sadhna’s daughter a partner in his business, which does not qualify as giving
dowry.
(d) Court has given the right judgment as Sadhna’s daughter selling off the
property and separating soon
after the marriage raise suspicions regarding her intentions behind marrying.
Passage(Q.83-Q.87): Supreme Court granted two weeks time to the Union of India to
file its counter
affidavit in the plea filed challenging the provisions dealing with restitution of
conjugal rights. The PIL has
challenged Section 9 of the Hindu Marriage Act, section 22 of the Special Marriage
Act and Rules 32 and
33 of Order XXI of the Code of Civil Procedure. The petitioner, plea has contended
that the court#mandated restitution of conjugal rights amounted to a "coercive act"
which violates one's sexual and
decisional autonomy, right to privacy and dignity, all of which come within the
purview of right to life and
personal liberty under Article 21.
The said sections provide that when either the husband or the wife has, without
reasonable excuse,
withdrawn from the society of the other, the aggrieved party may apply, by petition
to the district court, for
restitution of conjugal rights and the court, on being satisfied of the truth of
the statements made in such
petition and that there is no legal ground why the application should not be
granted, may decree
restitution of conjugal rights accordingly.
The remedy of restitution of conjugal rights was not recognized by any of the
personal law systems of
India. The same has its origins in feudal English Law, which at that time
considered a wife to be the
chattel of the husband. The Petitioner also seeks reconsideration of the Supreme
Court judgment in
Saroj Rani v. Sudarshan Kumar Chadha by which it had set aside the Andhra Pradesh
High Court's
decision quashing section 9 of the Hindu Marriage Act. In the said judgment, the
court observed that
such a right is inherent in the very institution of marriage itself. There are
sufficient safeguards in Section
9 of the Hindu Marriage Act to prevent it from being a tyranny.
Srishti Ojha &AaratrikaBhaumik, ‘Supreme Court Seeks Centre's Affidavit In
Challenge To Provisions On
Restitution Of Conjugal Rights’ (Live Law, 08 July 2021)
. Page 22 of 40
83. Sheldon and Amy married as a conclusion of love affair. However, after a few
months of their marriage
Amy separated from Sheldon for his being an obsessive husband and persistently
interfering with her
affairs. Amy did not file for a divorce wondering that months of separation might
enlighten Sheldon and
he will be a more liberal and understanding husband. Sheldon, on the other hand,
opted for the legal
process to force Amy to live with him. The court has reserved the order for the
application made under
section 9 of the Hindu Marriage Act. Decide whether the application shall be
allowed or not.
(a) Application shall be allowed as marriage is a pious institution which cannot be
shunned at whim of a
person without any particular reason.
(b) Application shall not be allowed as Amy has a reasonable excuse of
strengthening her position in the
marital tie by abandoning Sheldon for some time.
(c) Application shall be allowed because there is no reasonable excuse at Amy’s end
for justifying her
separation from Sheldon and thus restitution shall be ordered.
(d) Application shall not be allowed as the court shall adopt a more liberal
interpretation of the legal letter
to make orders in the interest of both the spouses without causing prejudice to
either.
84. In which of the following grounds has the section 9 of the Hindu Marriage Act
has been criticized or
challenged:
I. Restitution of conjugal rights is an archaic law enacted as a result of
patriarchy.
II. Inhibiting from exercising one’s sexual autonomy, the section is violative of
right to privacy conferred
by article 21 of the Constitution.
III. The Andhra Pradesh High Court has made correct observation which shall be
mandatorily followed.
IV The remedy of restitution to a spouse cannot be drawn from the personal laws of
any religion in the
country.
Choose the correct option:
(a) I and II (b) IV and II (c) III only (d) I, II and IV
85. Ross and Rachel have been in a marital tie for 8 years. All of a sudden Rachel
feels that she does not
want to be shackled by the marital bonds. So, she leaves their house to live all by
herself in the
Himalayas for finding her inner self. After she left, Ross made an application
under section 9 of the Hindu
Marriage Act for restitution of conjugal rights. Rachel has contested the petition
contending that she has
a valid reason behind leaving and she no more fits in the institution of marriage.
The matter is pending
before the district court for judgment. Choose the correct option.
(a) The court shall allow the application as Rachel separated from Ross without any
reasonable excuse
which warrants strict order of restitution of conjugal rights.
(b) The court shall not allow the application as law cannot force Rachel to stay
with Ross in derogation of
her right to privacy which ensures her sexual autonomy.
(c) The court shall allow the application as the decision of separation in an
institution of marriage is
bound to be a mutual one as allowed in the cases of divorce.
(d) The court shall not allow the application as realising that one no more fits
into the institution and
leaving to explore one’s inner self is a reasonable excuse to separate from spouse.
86. Continuing the above facts, if Rachel has come back from the Himalayas and
instead of moving in with
Ross, she is living with her boyfriend Barry. Ross makes an application as in the
above question, for an
order to force her to move in with him. He refuses to initiate a divorce petition
and desires to live with
Rachel for life. In the view point of the author and the petitioner in the above
passage, decide whether
Rachel can be forced by the court to stay with Ross.
(a) Rachel can be forced to go stay with Ross as he is her husband and the law
prohibits separating from
him unless for a reasonable cause.
(b) Rachel cannot be forced to stay with Ross, and ordering the same would be an
infringement of her
sexual autonomy which is a protected right under right to life and personal
liberty.
(c) Rachel can be forced to stay with Ross as separating for enjoying the company
of boyfriend is a
reasonable cause to abandon one’s husband.
. Page 23 of 40
(d) Rachel cannot be forced to stay with Ross, as she is entitled to choose the
person she wants to live
with, and if this idea is abhorred by Ross he is free to apply for a divorce.
87. Shubham and Jasmine married each other under Special Marriage Act. Soon after
marriage, Shubham
flew to abroad for a business trip and to Jasmine’s surprise Shubham informed her
through a call that he
plans on settling there forever. Jasmine made an application under section 22 of
the Special Marriage
Act. In absence of any response from Shubham, the court agreed with the contentions
of Jasmine that
Shubham left after the marriage and has thus breached the institution of marriage,
and thus passed the
order for restitution of conjugal rights of Jasmine. Shubham’s lawyer argued that
Shubham strongly
refrains from settling with Jasmine and therefore shall not be forced to do so.
Decide whether the appeal
be allowed or not.
(a) The appeal shall not be allowed because there is no legal basis for not
allowing the application and
thus the court is thus entitled to pass an order for restitution.
(b) The appeal shall not be allowed as forcing Shubham to stay with Jasmine amounts
to infringement of
his right to privacy ensured under article 21 of the Constitution.
(c) The appeal shall not be allowed as separation from a spouse, unless for the
reason of divorce, is not
permitted by law.
(d) The appeal shall be allowed as not wanting to stay with one’s spouse is a
reasonable excuse for
separation from them and discourages any order of restitution of conjugal rights by
the court.
Passage(Q.88-Q.93): The safety and the welfare of the State is the supreme law as
comprehended in
the legal maxim – “salus populi suprema lex”. Even the fundamental rights to
profess, practice or
propagate religion, and the right to manage religious affairs, as enshrined under
Articles 25 and 26 of the
Constitution of India, are subject to public order, morality and health. The health
of the citizenry of India
and their right to “life” are paramount. All other sentiments, albeit religious,
are subservient to this most
basic fundamental right.
(Extracted with requisite revisions and edits from‘[Disposal of COVID-19 victim
bodies v. Parsee last
rites] Right to religion subject to public order, morality, and health: Gujarat
High Court’
88. The government of Gujarat issued guidelines namely the “Covid-19 Guidelines on
Dead Body
Management” whereby two modes i.e. either cremation or burial are identified for
the disposal of the
dead body, who died due to Covid-19 as the virus was air-borne and anyone even
slightly exposed or
near the vicinity is likely to get infected and for public health considerations. A
parsi devout, Mr. Daruwala
claimed that for more than 3000 years the Parsee community as per their religious
mandate, laid out the
deceased in a place known as “Dakhmas” (the Tower of Silence) where the body is
kept at a height in a
structure known as “Well/Tower of Silence” to be eaten by vultures and the remains
being exposed to the
Sun to be decomposed. Therefore, the present guidelines should be struck down for
violating the
fundamental right of religion guaranteed under the Constitution. Is his plea likely
to succeed?
(a) No, the permissibility of such a practise would be detrimental to public health
and thus the guidelines
are valid.
(b) Yes, the guidelines would impact the religious freedom of the practitioners of
Parsi religion and being
a minority religion the same should be respected.
(c) No, right to life of the general public would be impacted by such a practice
and thus the guidelines
cannot be invalidated.
(d) Yes, the sentiments of the practitioners of Parsi religion would be offended as
the practice has been
followed as a ritual for 3000 years.
. Page 24 of 40
89. On being questioned about the health impact on the general public due to the
viral nature of the Covid, it
was further contended that this Well/ Tower of Silence is situated at a secluded
place and would not
impact the public health. Is this a valid ground for setting aside the guidelines?
(a) No, the permissibility of such a practise would be detrimental to public health
and thus the guidelines
are valid.
(b) Yes, the guidelines would impact the religious freedom of the practitioners of
Parsi religion.
(c) No, right to life of the general public would be impacted by such a practice
and thus the guidelines
cannot be invalidated.
(d) Yes, the permissibility of such a practise would not be detrimental to public
health as it is held in a
secluded area.
90. It was brought to the notice of the Court that many other states in India have
still not issued guidelines/
law which prohibits the practice of disposal of dead bodies and hence the
guidelines in question should
be struck off. Is the argument likely to succeed?
(a) No, the permissibility of such a practise would be detrimental to public health
and thus the guidelines
are valid.
(b) Yes, the practise has not been adopted in many other States indicates that such
a prohibition is not
necessary.
(c) No, right to life of the general public would be impacted by such a practice
and thus the guidelines
cannot be invalidated.
(d) Yes, the permissibility of such a practise would not be detrimental to public
health as it is held in a
secluded area.
91. Three students of Jehovah’s Witnesses sect studying in a local government
establishment refused to sing
a prayer which praised Hindu Gods. This led to their suspension from the school as
it would disturb the
morality and public order as it goes against the ideal of the institution which was
largely a Hindu
Establishment. Is the act of the school correct?
(a) Yes, religious freedom is subject to public order and morality and hence the
school was correct in
suspending the students.
(b) No, religious freedom guaranteed under Constitution is impacted and there are
no apparent affects to
the morality and public order.
(c) Yes, the act of the students would invoke disobedience amongst the students and
hence the
suspension is necessary.
(d) No, the students have the freedom to speak or even desist from exercising such
a freedom.
92. The school claimed that the suspension is not primarily on a religious ground
but the influence it would
have on other students due to this disobedience. Is the school likely to succeed in
this case?
(a) Yes, religious freedom is subject to public order and morality and hence the
school was correct in
suspending the students.
(b) No, religious freedom guaranteed under Constitution is impacted and there are
no apparent affects to
the morality and public order.
(c) Yes, the act of the students would invoke disobedience amongst the students and
hence the
suspension is necessary.
(d) No, the students have the freedom to speak or even desist from exercising such
a freedom.
. Page 25 of 40
93. A famous temple situated in Kerala decided to prohibit any entry within a
particular area where the Idol of
the Lord was situated and prohibited people from touching the Idols due to the
Covid-19 virus. It was
contended by the petitioners that it is the belief of their religion that touching
the statue and offering
prayers to it is an essential practice. Is the act of the temple violative of right
to religion?
(a) No, religious freedom is subject to public health and hence the temple
authorities were correct in
prohibiting entry of the public.
(b) Yes, religious freedom guaranteed under Constitution is impacted and there are
no apparent affects
to the public health.
(c) No, the discretionary power to determine such questions lies with the temple
authorities.
(d) Yes, an essential practice has been to the devotee which affects their freedom
of religion.
Passage(Q.94-Q.100): Sometimes it happens that people do not convert to another
religion out of fear or
greed, but due to humiliation, as they believe that they will get respect in other
religions. There is no harm
in this, and in the Indian Constitution, all citizens have the right to live life
with dignity. When a person
does not get respect in his/her own house and he/she is neglected, then he/she
leaves the house.
Religion is an object of devotion; it cannot be tied to any particular worship
system.
It has to be remembered that Article 25 (1) guarantees "freedom of conscience" to
every citizen, and not
merely to the followers of one particular religion, and that, in turn, postulates
that there is no fundamental
right to convert another person to one's own religion because if a person purposely
undertakes the
conversion of another person to his religion, as distinguished from his effort to
transmit or spread the
tenets of his religion, that would impinge on the "freedom of conscience"
guaranteed to all the citizens of
the country alike. If someone converts by doing this, then it is not acceptable in
any religion and that is
why the Indian Constitution also does not allow it. Marriage is a sacred
institution under every personal
law and marriage is a sacrament under Hindu law.
(Extracted with requisite revisions and edits from ‘Religion An Object Of Devotion
Which Can't Be Tied
To Any Particular Worship System; 'Akbar-Jodha' A Good Example: Allahabad High
Court’at
94. Ms. Ameena was head over heels in love with Mr. Kadar. The girl was converted
for the sole purpose of
her marriage and that too, against her will. Another reason for such a conversion
was that Mr. Kadar was
already married to Hussainara and it was only under Muslim personal laws he was
permitted to have
more than one wife. Ms. Ameena was made to sign on plain paper and some papers were
in Urdu and
she doesn't know how to read Urdu and prepared a fake Nikah Nama and got married to
her, and
thereafter, he mentally, physically abused her. On getting the opportunity, Ms.
Ameena called the police
and gave a statement against her husband before the magistrate. Is her conversion
valid?
(a) Yes, the conversion was not because of any fear or greed but for the purpose of
consummation of the
sacrament of marriage.
(b) No, the conversion was against the will of Ms. Ameena and is hence not
protected under the Indian
Constitution.
(c) Yes, the act of Mr. Kadar amounts to freedom of conscience and hence his act
would be protected
under the Indian Constitution.
(d) No, Ms. Ameena was abused after marriage and no religion permits such brutal
acts which makes the
conversion null and void.
95. Another issue before the court of law raised was whether the marriage between
Ms. Ameena and Mr.
Kadar would be valid as it was only after the conversion of Ms. Ameena that the
marriage was concluded,
a marriage permitted within the Muslim personal laws. Ms. Ameena wanted to dissolve
their marriage.
Would the conversion if proved to be illegal affect the validity of their marriage?
(a) No, the marriage was conducted after the conversion under Muslim laws and hence
the marriage is
legal.
(b) No, marriage is a sacred sacrament and cannot be dissolved merely because of
conversion against
the will of the person.
(c) Yes, the conversion was conducted against the will of Ms. Ameena and through
fake nikahnama.
(d) Yes, Ms. Ameena can now exercise the tenets of Muslim personal laws and
dissolve the marriage.
. Page 26 of 40
96. A famous spiritual guru, Mr. Toppo organized various spiritual functions to
profess the ideals of
Christianity in India. He was a man of few words but with significant impact on the
attendees. The
attendees were not restricted to one single religion but various religions looking
for peace of mind. The
functions were for the duration of 6 days and on the 7th day all those who wanted
and willing to convert
and adopt Christianity were baptized. This practise was challenged as it amounted
to conversion of
different groups of people. Is the practise legal?
(a) No, the conversion is done willingly by the devotees and will be part of
freedom of conscience as
guaranteed under Indian Constitution.
(b) Yes, the practise involves the conversion of different sects of people and a
third party doesn’t have
the right to convert others.
(c) No, the freedom of conscience is restricted only to a person’s religion of
birth and not the religion s/he
converts into.
(d) Yes, the act of conversion is to get respect in another religion and not
necessarily out of fear or
greed.
97. It was later also found that apart from few attendees who converted at their
will, there was mass
conversion of others through misrepresentation and by fraud. Hence, the function
began to be called by
the media channels as a ‘Function of fraud’. This was opposed by those who were
converted by such
means. Is the conversion of those converted through misrepresentation legal?
(a) No, the conversion is not done willingly by the devotees but by
misrepresentation and fraud adopted
by a third person to convert the devotees.
(b) No, the practise involves the conversion of different sects of people and a
third party doesn’t have the
right to convert others.
(c) Yes, the freedom of conscience is permits the conversion into any religion of
choice.
(d) Yes, the act of conversion is to get respect in another religion and not
necessarily out of fear or
greed.
98. Assume the conversion of those misrepresented is declared as void by the Court
of law, will this impact
the conversion of those who converted by will during the same functions?
(a) Yes, the conversion of all those who converted during the function would be
null and void if the
conversion of even few turns to be illegal.
(b) No, the conversion of all those who converted during the function would not be
affected if done
willingly despite the conversion of even few turning out to be illegal.
(c) Yes, the act of conversion is to get respect in another religion and not
necessarily out of fear or
greed.
(d) No, conversion is a matter of choice and a freedom guaranteed under the
Constitution irrespective of
the mode involved in such a conversion.
99. It was contended by the Advocate of Mr. Toppo that his acts merely involved
spreading the words of God
and is this protected under the Freedom of conscience and hence any conversion done
during the
function would be valid irrespective of the claims of forced, misrepresentation or
fraud involved in such
conversion. Is his argument valid?
(a) Yes, the act of Mr. Toppo is protected under the Freedom of conscience and is
not affected by the
nature of the conversion involved.
(b) No, the act of Mr. Toppo is not protected under the Freedom of conscience as
there is no
fundamental right to convert another person to one's own religion.
(c) Yes, Mr. Toppo is targeted towards spreading the words of the God which is not
an offence under any
law.
(d) No, Mr. Toppo cannot dictate the freedom of religion of others but himself
only.
. Page 27 of 40
100. If the conversions are declared to be illegal, can the organizing of any such
functions by any person
professing a religion be prohibited in their entirety?
(a) Yes, his act of conversion if declared illegal would be indicative of impinging
the fundamental rights of
others and can hence be curtailed in its entirety.
(b) Yes, nobody has the right to dictate the choice of which religion to practise
through acts such as that
of conversion.
(c) No, freedom of conscience of others would be affected if a blanket prohibition
on professing of
religion is done.
(d) No, his act of conversion is independent of his right to profess his religion.
Passage(Q.101-Q.105): The SC has ruled that reservation will be applicable to
persons with a disability
even during promotions in employment.
It also said that the mode of recruitment is no ground to reject reservation
benefits to a person as long as
they are 'disabled' at the time of availing promotion. A division bench dismissed
an appeal filed by the
Kerala government against a judgment of the High Court, which had directed the
respondent to be given
a promotion in the PwD quota though her initial appointment was not under the PwD
quota.
It further added that the source of recruitment should not make any difference as
long as the employee is
PwD at the time for consideration of promotion.
101. Joshila was a havaldar in the Delhi Police, working the day shift and was a
very hard working person,
according to his seniors. He was appointed on compassionate grounds due to his
father having been a
havaldar also in the Delhi Police who had died in an attack on their house when
Joshila was a mere 15-
year-old and was crippled in the same. He was appointed by the PwD quota and had
been dutifully
serving his precinct for over 5 years now, being liable for promotion under the PwD
quota. However, he
took the general test to be appointed as the Head Constable; however, he fails the
same and isn't given
the promotion. Would he be liable for one regardless?
(a) No, as he failed the exam in question and shouldn't be given any promotion.
(b) Yes, even if he failed the exam, he is liable to be promoted under the PwD
quota.
(c) No, as he appeared for the general exam and not the PwD exam.
(d) Yes, as he was supposed to be given a promotion anyway since he was due for it.
102. In the same police chowki, Joshila's constabulary counterpart, Rangeela,
appointed through the General
class, was also due for promotion on account of his meritorious service and was set
to be a Sub#Inspector. The Delhi Police had only one vacancy for a promotion to be
made, which should have ideally
been given to Rangeela since his service was more meritorious than Joshila.
However, on account of
having been appointed as a PwD official, Joshila was given the promotion, which
upset Rangeela. Could
he initiate any action against this oversight of the process?
(a) Yes, he can, as he has wrongly been overlooked while granting the promotion.
(b) No, as according to the vacancy, Joshila was entitled to be promoted first.
(c) Yes, as he was liable to be promoted first as according to his meritorious
service.
(d) No, as Joshila was to be given a promotion, and the sequence at the same level
of designation does
not matter.
103. Shivdasi was a Sub-Inspector in the Kerala police department and was appointed
on account of being
top of her class in the general category. However, in the line of duty, she was
fired upon by some
militants and thus, was rendered lame in one leg. She had been dutifully serving
the police corps for 8
years in her position, entitlingher for two promotions; however, on account of
being disabled in the line of
duty, she was denied the same. Her counterparts had been promoted and had surpassed
her by miles,
but she remained in the same stagnancy. Would she be liable to be promoted?
(a) No, as she has been rightly denied promotion on account of being lame in one
leg.
(b) No, as she would be of no use to the police department in a superior position
since she cannot be of
service as she used to.
(c) Yes, as she was entitled to be promoted and hence, should have been.
(d) Yes, as her disability is of no consequence.
. Page 28 of 40
104. In the above question, had Shivdasi been due for a promotion before her
accident and was promoted
after her disability, would it have been a promotion out of her PwD classification
or a normal promotion?
(a) It would have been a promotion out of her General status and not her PwD
status.
(b) It would have been a promotion out of her PwD status.
(c) It would've been a promotion out of her normal status as a policewoman and not
by classification.
(d) None of the above.
105. CM Buddhan Jhadav, of Maharashtra, was the leader of the SDP political party.
During one of his rallies,
a supporter of the opposition slapped him and shot him in the arm, rendering him as
disabled. He was
about to run for the Governor of the state. However, this stint cost him the
election, as he was surely
about to win if not for his injury. This led to the opposition winning the election
for the Governor. Buddhan
filed an appeal in the High Court for being denied a promotion that he was due.
Would his appeal stand?
(a) His appeal would stand as he was wrongly denied his promotion.
(b) His appeal would be dismissed as he has not been denied any promotion.
(c) His appeal would be quashed since his promotion has not been wrongly denied.
(d) His appeal would not stand as he was not entitled to receive any promotion.
. Page 29 of 40
SECTION - D: LOGICAL REASONING
Passage (Q.106-Q.110): The efficacy of total lockdowns as a public health measure
is yet to be proven.
But we seem to have allowed our imagination and thinking to get trapped into that
logic, regardless of the
social and economic costs to society. Countries that imposed lockdowns during the
pandemic, including
China, have seen a re-emergence of infections while Sweden that did not impose a
lockdown has close
to zero deaths today. However, it’s apparent that localized restrictions on
movements in high positivity
areas for a limited period make sense.
The impact of lockdowns has undoubtedly been painful at both the macro and micro-
levels. Amongst the
worst affected are children, ironically the least vulnerable. Morbidity and
mortality among children have
been comparatively lower. As per a recent UK study, deaths are two per million and
hospitalization under
severe conditions about 1 in 50,000. Most of the children attending schools fall
under the age group of 9
and 15. Studies carried out in the US, Ireland, Norway, Germany and other parts of
the world have shown
very low to negligible transmission of infection in, and due to, schools,
particularly where the discipline of
wearing masks, physical distancing and personal hygiene has been enforced even
moderately. In fact,
most countries have persisted with in-person learning. Only a handful have shut
down schools. India is
one of them.
While we have no information regarding the cohort of children who have been
hospitalized or have died
due to Covid over the past 18 months, as in the case of adults, children with
comorbidities such as
diabetes or obesity are likely to be more vulnerable. Such data, along with
seropositivity studies, need to
be triangulated and analyzed to enable evidence-based policy formulation instead of
panic or speculation
guiding decision making. Available evidence seems to suggest that from a strictly
epidemiological
standpoint, there is weak justification for the stringent and prolonged lockdown of
schools – particularly,
primary schools. We do hear of online classes. But with less than a quarter of the
country having internet
access and the lackadaisical manner of the implementation of online learning by
untrained teachers, the
reach of such instruction to even urban students has been patchy. Students from
rich families attending
“good” schools may have benefitted somewhat from online education, but they are
only a minuscule
section of the learners.
[Extracted with edits and revisions from ‘Indian Express’ Written by K. Sujatha
Rao: August 7, 2021]
106. “Countries that imposed lockdowns, including China, have seen a re-emergence
of infections while
Sweden that did not impose a lockdown has close to zero deaths today.” Which among
the following
conclusions can we draw from this statement?
(a) One of the best strategies to counter the pandemic was to not impose lockdown.
(b) Countries like India would have mitigated the effects of the pandemic by not
imposing lockdowns.
(c) Not imposing lockdowns in the country helped some countries to counter the
pandemic.
(d) Imposing lockdowns does more harm than good to developing nations.
107. Which among the following, if true, most weakens the author’s claims that
schools should be reopened?
(a) The death rate of children aged between 16 and 18 and contracting the disease
is 200 per million.
(b) The reopening of schools puts at risk the child’s adult parents and
grandparents as well of contracting
the disease.
(c) India has shut down the schools since the children in India are much less
vulnerable to Covid than
children in other countries.
(d) Reopening of schools in India will have no significant impact on the quality of
education.
108. Which of the following is the correct expression of the author’s opinion in
the passage?
(a) Online learning is not likely a viable alternative to in-person learning.
(b) It is a human tendency to panic when faced with extraordinary circumstances.
(c) Stringent and prolonged closure of schools cannot be justified on any grounds.
(d) Pressing the panic button when faced with extraordinary circumstances is not
always incorrect.
. Page 30 of 40
109. Consider this part, “We do hear of online classes. But with less than a
quarter of the country having
internet access and the lackadaisical manner of the implementation of online
learning by untrained
teachers, the reach of such instruction to even urban students has been patchy.”
Out of the following
options, if true, which one could weaken the author’s argument in this part of the
passage?
(a) China and India consist of more than 35% of the world’s population and both
these countries have
internet access to 50% of their respective population.
(b) According to the findings of a reliable study, 80% of teachers around the world
have little to no
technical knowledge.
(c) 90% of the population around the world lives in rural areas.
(d) None of the above could weaken the author’s argument in this part of the
passage.
110. As per the passage, which of the following is not an argument for reopening
schools?
(a) We can’t conclude anything about how well imposing lockdowns work.
(b) Children are less vulnerable to Covid as compared to adults.
(c) There is a paucity of ‘good’ schools all around the world.
(d) All of the above
Passage (Q.111-Q.115): India’s coronavirus vaccination drive is hobbling once
again. In the seven days
till Sunday, the number of doses delivered daily on average was 3.5 million – a
long way from the peak of
5.8 million a day in the week after June 21. The country’s vaccination rate surged
to record levels when
the Union government brought the programme back under its control, taking over a
chaotic purchase
mechanism, offering free doses to all adults, and allocating 75% of supplies to
states. Several other
factors too helped boost the drive — the second wave of infections relented,
allowing more people to
come forward, and supply bottlenecks were eased, helping states open up more
vaccination centres. The
pace, it appeared then, was on a steady trajectory that, if built up to 8 to 10
million jabs a day, could have
potentially met the government’s goal of vaccinating all adults by the end of 2021.
Instead, there has been a dip and meeting the target appears almost impossible.
Vaccination trends as
well as remarks from some states suggest two of the persistent problems continue to
exist — unreliable
supply and high levels of hesitancy. Over the last week, reports from Rajasthan,
Maharashtra and Delhi
have suggested that vaccination centres were closed due to lack of doses. At the
same time, wide
variations remain in how different groups of people have taken up vaccines – people
in India’s urban
centres are close to twice as more likely to be vaccinated, but fewer women still
have got jabs than men.
Health care workers on the front lines, especially in smaller towns and villages,
have also found
widespread misinformation around the safety and efficacy of vaccines as being
significant hurdles to
people coming forward.
The slowdown reinforces the need for vaccine-makers to continue expanding
production since delays
can impact population immunity. It also brings back focus on the need for more
vaccines to be approved
and introduced in the market. Later this week, India’s vaccine drive will complete
six months and the
country still continues to rely largely on two manufacturers. But the most
persistent issue may be vaccine
hesitancy, especially in rural India. These regions are normally disadvantaged in
terms of infrastructure
and testing penetration and it is crucial to ensure they are protected, if and when
there is a next wave.
India will need to persuade those left behind, particularly those in the 45-plus
age group, who should
have by now been entirely vaccinated.
[Excerpt from an article titled ‘India will miss its vaccination target’, The
Hindustan Times, July 12, 2021:
111. If author’s arguments in the passage above are true, which of the following is
most likely to be true?
(a) Now it is impossible for India to achieve its goal of vaccinating all adults by
the end of 2021.
(b) Ensuring continuous supply of vaccines and introducing incentives for people
getting vaccinated can
be a potential solution to achieve the target by end of 2021.
(c) Social media is the major cause behind the spread of misinformation related to
vaccination.
(d) None of the above
. Page 31 of 40
112. Which of the following can be inferred from the author’s arguments in the
passage above?
(a) Since the beginning, State governments were responsible for the vaccination
drive.
(b) The vaccination drive policy has been changed at least twice by the Central
government.
(c) People in urban areas have not shown any hesitancy for taking vaccines.
(d) All of the above.
113. Which of the following strengthens the author’s arguments?
(a) A survey showed that 70% of people in rural India are hesitant to take vaccine.
(b) More than 80% health workers have been already vaccinated.
(c) A survey showed that only 75% people in the rural India have been vaccinated so
far.
(d) The government thinks that it can easily achieve its target by end of 2021.
114. Which of the following is an assumption the author makes?
(a) India could achieve its target if more vaccines are approved for vaccination
drive.
(b) The government has not taken any steps to solve the vaccine hesitancy problem
of rural India.
(c) Both A) and B)
(d) None of the above
115. Which of the following, if true, would seriously undermine the author’s
arguments?
(a) The two vaccine suppliers of India are producing vaccines at their full
capacity.
(b) China has vaccinated substantial proportion of its population by approving
various vaccines.
(c) Some state governments have made contract for vaccine supply with some foreign
vaccine
producers.
(d) None of the above
Passage (Q.116-Q.120): COVID-19 has forced South Asia to take a quantum leap in
digitalisation. The
shift to remote work and education has propelled an unprecedented spike in Internet
penetration, with
even smaller nations such as Nepal recording almost an 11% increase in broadband
Internet users. For a
region with threadbare public health infrastructure, the digitisation of health-
care services was a
watershed moment, providing novel solutions to the public health crises.
In India, COVID-19 accelerated the launch of the National Digital Health Mission,
enhancing the
accessibility and the efficiency of health-care services by creating a unique
health ID for every citizen.
The pandemic-induced suspension of bricks-and-mortar businesses spurred South
Asia’s embrace of e#commerce, boosted by digital payment systems. Bangladesh alone
witnessed an increase of 70-80% in
online sales in 2020, generating $708.46 million in revenues.
As one of the world’s poorest regions, a wide digital divide persists in access and
affordability, between
and within the countries of South Asia. Despite having the world’s second largest
online market, 50% of
India’s population are without Internet with 59% for Bangladesh and 65% for
Pakistan.
With monetary and health assistance schemes distributed online, 51% of South Asian
women were
excluded from social protection measures during the pandemic. Children too were at
the receiving end,
with 88% lacking access to Internet powered home schooling. This disruption could
permanently put
children out of school, place girls at risk of early marriage, and push poor
children into child labor costing
economies billions of dollars in future earnings.
Businesses too have paid a heavy price for the gap in digital solutions, whereby
many South Asian firms
failing to embrace e-commerce or other cloud-based technologies to survive the
financial chaos of the
novel coronavirus pandemic. The region recorded a 64% decline in sales, with small
and women-led
firms faring the worst. With COVID-19 transforming work life, the acute skills gap
among youth will
continue, creating unemployment.
[Extracted with edits from TheHindu, written by Syed Munir Khasru: August 07, 2021
. Page 32 of 40
116. Which of the following notions is expressed in the passage to enunciate the
significance of the issue of
digital divide in South Asian countries?
(a) The gap of digital divide increases when the governments take measures to
control it.
(b) Digital divide is an illusionary notion developed during the pandemic to
increase the sales of tech
giants.
(c) The issue of digital divide has come to the fore due to the pandemic and
adequate steps are being
taken.
(d) The gap of digital divide is sure to decrease as the pandemic disappears.
117. As per the passage, which of the following could be most effective for failing
businesses to make a
comeback?
(a) The failing businesses should employ more salesmen to increase their reach at
the ground level.
(b) The failing businesses should focus on the offline market only since this is
what has given them
businesses till now.
(c) The failing businesses should actively engage among themselves to come up with
the best strategy
to recover.
(d) The failing businesses should adopt technology readily and look for digital
solutions to recover.
118. Which among the following has been indicated in the passage? Choose the best
option.
(a) More than half of India’s population is without internet.
(b) Early marriage is not desirable for girls.
(c) People who are without the internet are at least safe from the dangers that the
internet brings in.
(d) None of the above
119. What can be inferred about ‘bricks and mortar’ businesses from the passage?
(a) Companies that employ unethical practices to run their business.
(b) Companies that have a unique style of working.
(c) Companies that possess or lease retail shops, factory production facilities, or
warehouses etc. for its
operations.
(d) Companies that embrace digital solutions to expand their business.
120. What has the author not conveyed regarding India?
(a) Some steps have been taken by the government to reduce the digital divide in
India.
(b) The public health infrastructure in India is not adequate.
(c) A significant chunk of India’s population is still deprived of the internet.
(d) All of the above have been conveyed by the author.
Passage (Q.121-Q.125): The primary difficulty that arises from fetal homicide laws
lies in the
categorization of the fetus as a legal entity that is separate from the mother. In
effect, the fetus is granted
personhood for the purpose of prosecuting violent crimes. This is dangerous,
because the mother no
longer maintains the same set of rights that she did prior to conceiving. For women
to operate as equal
members in society, their personal liberty cannot be determined by their
childbearing status. A potential
person, who is not protected by the Constitution, cannot be granted legal rights
that supersede those of
an already existing person.
The protection and rights that are enjoyed by legal persons (due process, voting,
and equal protection to
name a few) cannot be exercised by a fetus. Therefore, it is unacceptable that a
woman’s rights become
secondary to those of her unborn child. If personhood status is granted to fetuses
(even if only for the
purpose of prosecuting violent crimes), women must relinquish their personal
liberty for the duration of
their pregnancy. This essentially creates a second-class citizenry whose individual
rights are entirely
dependent on whether their womb is vacant or occupied.
. Page 33 of 40
Furthermore, the language in these laws creates an inconsistency with a woman’s
legal right to an
abortion. The right to privacy in the Fourteenth Amendment includes a woman’s
decision whether or not
to terminate her pregnancy. To say that a woman has a legal right to terminate her
pregnancy if she
chooses but yet the death of the same fetus is murder when caused by someone else
(or herself) is
morally problematic. By default, this inequality extends to all women capable of
bearing children because
at any point they too could become pregnant and lose their right to bodily
autonomy. Since men
biologically cannot reproduce, they enjoy the privilege of possessing a full set of
rights that is not
dependent on the existence of another. For women to operate as equal members of
society, they must
also possess this same privilege.
121. Which of the following is most supported by the author’s argument?
(a) Women are discriminated against on the basis of their biology
(b) The rights of men and women are not equal under law
(c) Both (a) & (b)
(d) Neither (a) nor (b)
122. Which of the following can be inferred from the passage?
(a) Abortion is illegal as per the passage
(b) Women are also prosecuted for natural termination of pregnancy
(c) Right to privacy relating to pregnancy termination is not absolute
(d) Both (a) & (b)
123. Which of the following, if true, would seriously undermine the author’s
argument?
(a) Studies suggest that if pregnancy termination is allowed, women may get
influenced to do so more
often than now
(b) Various inanimate objects are given the status of a legal person under law
(c) Both (a) & (b)
(d) Neither (a) nor (b)

124. The statement “As per the existing law, the rights of a fetus supersede the
rights of the mother” is:
(a) Probably True
(b) Definitely True
(c) Probably False
(d) Definitely False
125. Directions:
STRONG ARGUMENTS must be both important and directly related to the question.
WEAK ARGUMENT may not be directly related or may be related to trivial aspects of
the question, and
may be of minor importance.
The question is followed by four arguments. You have to classify them into strong
and weak arguments.
Statement: Advertising is a wasteful expenditure
i. Yes: How many people can read!
ii. No: If it were wasteful, then nobody would advertise.
iii. No: The advertising industry employs a lot of people.
Iv: No: Pleasant pictures are worth a thousand words.
(a) i and iv are weak
(b) only iv is weak
(c) All are weak
(d) ii and iii are weak
. Page 34 of 40
Passage (Q.126-Q.130): The best way to note the second anniversary of nullification
of Jammu &
Kashmir’s special status? Speed up the process to get to the restoration of
statehood – through a
timeline that doesn’t stretch beyond a year. PM Narendra Modi’s meeting with
leaders of J&K parties in
June has shown that Article 370, a long-pending issue in the Supreme Court, may not
become a
stumbling block for political talks. This is a better approach than keeping
everything on hold and waiting
for a judicial outcome. Valley parties should know that BJP promised the
nullification, got a second
consecutive large Lok Sabha majority and kept its promise – jousting with the
governing party on this will
be pointless.
Happily, there’s much cross-party support for restoration of statehood in J&K.
Congress leader Ghulam
Nabi Azad, who hails from Jammu, has suggested that statehood be restored in the
“short-term” before
holding assembly elections. The stated hindrance for holding elections is the
ongoing delimitation
exercise, which has been delayed and has received an extension until March next
year. The Delimitation
Commission’s recent visit to the Valley where it met with political representatives
and its promise of a
“free, transparent” process has allayed some of the fears that the exercise will
whittle down Kashmir’s
share of seats in the legislative assembly. This is one of the steps towards
restoration of democracy in
the region.
While 4G connectivity restoration and the District Development Council elections
were big milestones in
the path to normalcy, continuing detention of many political prisoners remains a
sour note for many
Kashmiris. On the flip side, New Delhi also has to be mindful of the changed
security situation in view of
Taliban’s advances in Afghanistan, apart from the Chinese buildup in eastern
Ladakh. Suspected drone
sightings in recent days after the drone attack in June suggest that cross-border
terror will continue
irrespective of political advances.
Ultimately, it is the economy that matters most to the ordinary person on the
street. The momentous
events of August 5, 2019, and movement and communications restrictions in its wake,
followed by the
pandemic and lockdowns, were back-to-back shocks for J&K’s economic activity.
Greater commercial
engagement with the rest of India is the Valley’s best bet for quick revival. This
would bring back much#needed jobs for the young. Like the 1996 election at the peak
of insurgency that brought Kashmir back
from the brink, all stakeholders must reaffirm their commitment to the political
process. This is the only
road to democracy, statehood, and most importantly, peace and development.
[Extracted with edits and revisions from The Times of India: August 4, 2021]
126. Consider the following statements and denote the statements that are implicit
from the passage.
i. Delimitation exercise is a democratic process.
ii. Releasing political prisoners of the valley would bring peace to the region.
iii. Peace matters the most to an ordinary resident of J&K.
(a) Both i and ii
(b) Only I
(c) Only ii
(d) Both i and iii
127. Which of the following is most appropriate to bring peace in J&K?
(a) Cross border terrorism
(b) Restoration of statehood
(c) Release of political prisoners
(d) 4G connectivity
. Page 35 of 40
128. Consider this statement, ‘Greater commercial engagement with the rest of India
is the Valley’s best bet
for quick revival.’ Which of the following would be the strongest criticism to this
argument?
(a) Commercial engagement with the rest of India would bring in unethical trade
practices.
(b) The rest of India would be initially hesitant to engage with the economy of
J&K.
(c) Increase in commercial engagement with the rest of India would increase cross
border terrorism in
J&K.
(d) All of the above weakens the author’s argument equally.
129. According to the passage, which among the following is correct?
(a) Political advancements in J&K might be a good strategy to mitigate cross border
terrorism.
(b) There is no strategy that would counter the effects of cross border terrorism.
(c) Cross border terrorism is not a serious concern for the people of J&K.
(d) None of the above is correct.
130. Which among the following would be a logical course of action, according to
the author?
(a) The central government should hold more meetings with the stakeholders of J&K.
(b) The government should wait for the judicial outcome on the issue of J&K.
(c) The government should hurriedly restore the statehood of J&K.
(d) The ongoing delimitation exercise in J&K should be quashed.
Passage (Q.131-Q.135): RBI’s biannual report on financial stability, released last
week, flagged the
potential big impact of big technology firms entering the financial sector in a big
way. Technology firms,
whose competitive advantage comes from creating platforms for e-commerce or search
engines, have
been a part of the financial ecosystem mainly through support services such as
payment processors.
However, it’s the next stage when they will have a bigger footprint in lending that
poses challenges for
incumbents like banks, and for regulators too.
A defining feature of big tech firms is the economies of scope built into their
business models. As
technology platforms gain users, their ever-increasing data trail increases the
scope of the products that
platforms offer. Tech firms are uniquely placed in financial services. A bank has
limited information on a
customer. That influences not just the products that can be offered but also the
perception of risks in
offering loans. In contrast, technology platforms have large data on users,
providing an advantage over
traditional lenders. As RBI points out this can further the cause of financial
inclusion by reducing the
information asymmetry between lenders and small borrowers. China and other parts of
Asia, including
South Korea and Singapore, have seen significant penetration of technology firms in
all aspects of the
financial ecosystem.
But these potential benefits come with regulatory challenges. Monetising data
presents unique privacy
challenges. It’s therefore essential that big tech firms function within a data
protection law. India needs
one soon. The overarching characteristic of the financial sector is its
interconnectedness. When financial
activity happens in a regulatory grey zone, systemic risks are concealed. Most
financial crises originate
from concealment of risks. Given this context, RBI needs to be cautious about the
conditions under which
technology firms expand in the financial ecosystem. The primary requirement is that
financial
intermediaries, physical or digital, conform to a uniform regulatory standard.
There are examples from
East Asia where digital banks are allowed to accept deposits but within boundaries
set by the regulator.
To safely harness technology’s benefits, a smart regulatory framework is needed.
[Excerpt from an article titled ‘Tomorrow’s lender’, The Times of India, July 4,
2021:
.
131. Which of the following conclusions can be reasonably drawn from the passage
above?
(a) Technology firms will face certain difficulties if they plan to enter financial
sector.
(b) Regulatory framework need to be transformed to deal with the issues which will
arise due to entry of
Technology firms into financial sector.
(c) India has not seen significant penetration of technology firms into the
financial ecosystem.
(d) None of the above
132. If the following sentence in the passage is true, which of the following must
also be true: “That influences
not just the products that can be offered but also the perception of risks in
offering loans”?
(a) The Technology firms can analyse the data we provide them to determine our
preferences and risk
taking capability.
(b) Data is of great importance as it can be used to develop new products and
services.
(c) Both (a) and (b)
(d) None of the above
133. Which of the following is most supported by the author’s argument?
(a) Technology firms cannot easily work in the financial ecosystem.
(b) Without proper regulatory framework, the increased penetration of Technology
firms in the financial
ecosystem may give rise to certain problems.
(c) The entry of Technology firms in the financial ecosystem should be encouraged
as it will make the
financial ecosystem more efficient.
(d) The role of banks and other financial institutions will get degraded after
entry of Technology firms in
the financial ecosystem.
134. Which of the following can be inferred from the passage?
(a) The entry of Technology firms in the financial sector will make the ecosystem
more exclusive.
(b) RBI is not in favour of allowing the entry of Technology firms in the financial
sector.
(c) The entry of Technology firms in the financial sector may make the ecosystem
more inclusive.
(d) RBI is in favour of allowing the entry of Technology firms to the financial
sector.
135. Which of the following strengthens the author’s arguments?
(a) The user data can be sold, without user consent, to other firms or businesses
in return of monetary
payments.
(b) Economy of scope will limit the entry of Technology firms to the financial
sector.
(c) Without any regulatory framework, exploitation of user and their data could
increase.
(d) Both (a) and (c

mock 8
Directions (Q.1-Q.30): Read the passages carefully and answer the questions.
Passage (Q.1-Q.4): A year and a half after China and Pakistan announced plans for
an economic
corridor the China-Pakistan Economic Corridor (CPEC), to connect “Kashgar to
Gwadar”, the two
countries operationalised the trade route recently, with the first shipment moving
to Gwadar port and on
to the Gulf and Africa. Many of the infrastructure and energy projects that are
part of CPEC, worth an
estimated $46 billion, are already underway. Of this, $35-38 billion is committed
in the energy sector, in
gas, coal and solar energy across Pakistan, with the combined expected capacity
crossing 10,000 MW.
This is roughly double the current shortfall the country experiences. In addition,
the 3,000-km rail and
roadway project is expected to generate 7,00,000 jobs by 2030. While Pakistan sees
CPEC as a game
changer, there are many challenges. There are some misgivings domestically, with
critics questioning the
project’s viability, and some accusing China of launching a second “East India
Company”. There are the
security challenges too, especially in the western areas near the key Gwadar port,
where militants
ranging from Baloch nationalists to the Taliban and the Islamic State have carried
out attacks. Systemic
challenges include project delays in the CPEC’s first year, which the World Bank
warns could prove to be
an impediment to Pakistan’s overall growth. Pakistan-India tensions, unless
contained, too could
endanger sectors of the project where Pakistani troops are engaged in providing
security. Finally, the
economic slowdown in China and the political instability in Pakistan could impact
the project’s future as
well.
However these internal considerations for Pakistan shouldn’t blur the bigger
picture for India: CPEC is
now a reality. In the past India’s reaction to the project, announced a few weeks
before Prime Minister
Narendra Modi’s visit to China in 2015, had turned from dismissal and disdain to
disapproval and then to
outright opposition. India even raised concerns over projects in disputed Gilgit-
Baltistan at the UN
General Assembly. Not only has the project gone ahead despite the objections, but
China now sees
CPEC as a physical link between its One Belt. One Road (OBOR) project and the
Maritime Silk Route
(MSR). India has refused to be a part of either. That Sri Lanka, Bangladesh and
Afghanistan are all on
board the OBOR and the MSR should give India pause. It is important for Delhi to
also take a closer look
at the security implications of the China-Pakistan clinch that is fast drawing in
Russia in the north, all the
way to the Arabian Sea, while China plans a floating naval base off Gwadar.
1. At present, what is the approximate power deficit Pakistan experiences?
(a) Roughly 5,000 megawatts.
(b) Roughly 10,000 megawatts.
(c) Roughly 15,000 megawatts.
(d) Roughly 20,000 megawatts.
2. What could be the probable reason for the accusation that China is launching a
second “East India
Company”?
(a) The likelihood of China trying to bring in imperialism.
(b) The likelihood of China gaining control over other regions in the matter of
trade.
(c) The likelihood of China serving as a catalyst for establishing British
influence.
(d) The likelihood of China commencing spice trade with other countries.
3. Which of the following factor/factors could affect the success of the CPEC
project?
I. Militancy in the north-eastern regions of Pakistan adjoining China.
II. Initial delays in the project.
III. Political instability in Pakistan.
(a) Both I and II
(b) Both II and III
(c) Both I and III
(d) I, II and III

. Page 3 of 40
4. Which of the following can be inferred from the passage?
I. Gwadar is a port city.
II. Gwadar is located on the western part of Pakistan.
III. Gwadar is the fifth largest city in Pakistan.
(a) Only I and III
(b) Only II and III
(c) Only I and II
(d) Only I, II and III
Passage (Q.5-Q.9): We’ve already seen home energy use bottoming out and utilities
freaking out. But
energy efficiency – which often gets short shrift compared to renewables or
electrified transportation -
could be key to limiting climate change to only 1 .5C of warming.
That’s the finding of a study led by Dr Charlie Wilson of the Tyndall Centre for
Climate Change Research
- published over at Nature Energy and reported on by Carbon Brief - which lays out
an achievable if
aggressive scenario for achieving the 1 .5C limit, driven primarily by limiting
energy demand.
Interestingly, the authors of the study are not relying on virtue-driven behaviour
change. Instead, they are
positing the services such as electrified and autonomous transportation - and
digitization of many
consumer goods and services - could deliver better, cheaper and more attractive
results for consumers
while significantly driving down energy demand. (Energy demand related mobility
could faIl 60% in the
North. 59% in the Global South - even as activity levels increase by 29% and 122%,
respectively.) The
study projects that under this “low demand” scenario, emissions could drop rapidly
between 2030 and
2100, eventually entering the ‘net negative’ territory toward the end of the
century.
Now, to be clear, while efficiency and conservation are the primary engines behind
such impressive
figures, they are not being looked at in isolation. The authors also project a
slight global decline in red
meat consumption-largely driven by consumer health concerns and pro-climate policy
signals. (Cutting
back on meat and dairy is, after all, one of the single biggest things we can do
for the climate.) The
scenario outlined also includes an increase in forest cover of 30Dm hectares by
2100, thanks largely to
organized afforestation efforts.
Of course, it’s dangerous to read too much into any one study. Nobody knows how
unpredictable climate
feedback loops may skew our progress toward 1.5 degrees, for example - and it’s a
fair bet that new
technologies and social trends are coming (both positive and negative) that none of
us has predicted yet.
But what this study shows is that limiting demand is one of the surest strategies
we have in minimizing
the negative impacts of climate change, not least because it will make the job of
cleaning up our energy
grids significantly easier.
5. Why does the author say, “Of course, it’s dangerous to read too much into any
one study”?
I. None of the studies have been accurate in predicting energy demands and
conservation.
II. The study in question is conducted by an organization which is funded by
governments of developed
nations.
III. Climate feedback loops and technologies of the future can respectively skew or
aid our progress
towards achieving the goal of limiting temperature rise to 1.5 degrees
(a) Only I
(b) Only III
(c) Only II
(d) Only I and II

. Page 4 of 40
6. According to the study led by Dr Charlie Wilson of the Tyndall Centre for
Climate Change Research,
which of these factor/factors is/are supposed to limit energy demand?
I. Digitization of consumer goods and services.
II Shared transportation.
III. The conscious limiting of energy consumption by people.
(a) Only I
(b) Only II
(c) Only III
(d) Only I and II
7. Which of the following can replace the phrase ‘short shrift’ used in the
passage?
(a) Honourable mention.
(b) Passing reference.
(c) Swift dismissal.
(d) Quickly forgotten.
8. Which of the following is a synonym for the word ‘positing’ as used in the
passage?
(a) Converting
(b) Proving
(c) Proposing
(d) Contradicting
9. Based on the passage, which of the following is/are true?
I. In the Global South, even though activity levels increase by 122%, energy demand
related mobility
could fall by 59%.
II. Limiting energy demand is one of the surest strategies in minimizing the
harmful effects of climate
change.
III. Limiting energy demand will make cleaning up of energy grids easier.
(a) Only I
(b) Only II
(c) Both l and II
(d) All I, II and III
Passage (Q.10-Q.14): Money is not everything, but money is something very
important. Beyond the
basic needs, money helps us achieve our life’s goals and supports the things we
care about most deeply
— family, education, health care, charity, adventure and fun. It helps us get some
of life’s intangibles —
freedom or independence, the opportunity to make the most of our skills and
talents, the ability to choose
our own course in life, financial security. With money, much good can be done and
much unnecessary
suffering avoided or eliminated.
But money has its own limitations too. It can give us the time to appreciate the
simple things in life more
fully, but not the spirit of innocence and wonder necessary to do so. Money can
give us the time to
develop our gifts and talents, but not the courage and discipline to do so.
Money can give us the power to make a difference in the lives of others, but not
the desire to do so. It
can give us the time to develop and nurture our relationships, but not the love and
care necessary to do
so. It can just as easily make us jaded, escapist, selfish, and lonely. How much do
you need? What is it
going to cost you to get it? It is keeping these two questions in mind that gives
us a true sense of
money’s relationship to happiness. If we have less than what we need, or if what we
have is costing us
too much, we can never be happy. We need money to eat, sleep, dress, work, play,
relate, heal, move
about, and enjoy comforts. We should remember in choosing our style that it comes
with a price tag.
Evidence of the psychological and spiritual poverty of the rich and famous fills
our newspapers,
magazines, tabloids, and television programs and hardly needs repeating here. “We
always think if we
just had a little bit more money, we’d be happier” says Catherine Sanderson, a
psychology professor at
Amherst College, “but when we get there, we’re not.” “Once you get basic human
needs met, a lot more

. Page 5 of 40
money doesn’t make a lot more happiness,” notes Dan Gilbert, a psychology professor
at Harvard
University and the author of the new book Stumbling on Happiness.
Yes, we get a thrill at first from expensive things. But we soon get used to them,
a state of running in
place that economists call the “hedonic treadmill”. The problem is not money, it’s
us. For deep-seated
psychological reasons, when it comes to spending money, we tend to value goods over
experiences.
Money should not cost us our soul, relationships, dignity, health, intelligence and
joy in simple things of
life. People who figure out what they truly value and then align their money with
those values have the
strongest sense of financial and personal well-being.
10. What are the intangibles that money can give us?
(a) family, healthcare, entertainment and fun
(b) liberty, opportunity to develop talents, financial security and the ability to
choose our own course.
(c) Houses, cars, the basic needs and the comforts of life.
(d) Specific goals and objectives.
11. What are the negative aspects of money?
(a) It can be a drain on our relationships and rob us of simple joys in life.
(b) It can have a detrimental effect on our health.
(c) It can erode our dignity and intelligence.
(d) All the above
12. What was Dan Gilbert’s observation?
(a) More money brings more happiness.
(b) A little extra money always makes us want more.
(c) The ideal amount of money would be that which meets our basic needs.
(d) People with money always have the strongest sense of financial and personal
well-being.
13. Which of the following is untrue?
(a) Money can help us make a difference in the lives of others.
(b) Money can develop and foster relationships.
(c) Our lifestyle never comes with a price tag.
(d) Money gives rise to selfish and cynical individuals.
14. The author talks about a ‘hedonic treadmill’. What can this refer to?
(a) The machine which facilitates the mapping of ECG of a person.
(b) The inevitable machine test which helps in identifying heart ailments.
(c) A plateau in the level of happiness despite huge fortune and high achievement.
(d) Simple living and high thinking.
Passage (Q.15-Q.20): Historians generally agree that, of the great modern
innovations, the railroad had
the most far-reaching impact on major events in the United States in the nineteenth
and early twentieth
centuries, particularly on the Industrial Revolution. There is, however,
considerable disagreement among
cultural historians regarding public attitudes toward the railroad, both at its
inception in the 1830s and
during the half century between 1880 and 1930, when the national rail system was
completed and
reached the zenith of its popularity in the United States. In a recent book, John
Stilgoe has addressed
this issue by arguing that the “romantic-era distrust” of the railroad that he
claims was present during the
1830s vanished in the decades after 1880. But the argument he provides in support
of this position is
unconvincing.
What Stilgoe calls “romantic-era distrust” was in fact the reaction of a minority
of writers, artists, and
intellectuals who distrusted the railroad not so much for what it was as for what
it signified. Thoreau and
Hawthorne appreciated, even admired, an improved means of moving things and people
from one place
to another. What these writers and others were concerned about was not the new
machinery as such, but

. Page 6 of 40
the new kind of economy, social order, and culture that it prefigured. In addition,
Stilgoe is wrong to imply
that the critical attitude of these writers was typical of the period; their
distrust was largely a reaction
against the prevailing attitude in the 1830s that the railroad was an unqualified
improvement.
Stilgoe’s assertion that the ambivalence toward the railroad exhibited by writers
like Hawthorne and
Thoreau disappeared after the 1880s is also misleading. In support of this thesis,
Stilgoe has unearthed
an impressive volume of material, the work of hitherto unknown illustrators,
journalists, and novelists, all
devotees of the railroad; but it is not clear what this new material proves except
perhaps that the works
of popular culture greatly expanded at the time. The volume of the material proves
nothing if Stilgoe’s
point is that the earlier distrust of a minority of intellectuals did not endure
beyond the 1880s, and, oddly,
much of Stilgoe’s other evidence indicates that it did. When he glances at the
treatment of railroads by
writers like Henry James, Sinclair Lewis, or F. Scott Fitzgerald, what comes
through in spite of Stilgoe’s
analysis is remarkably like Thoreau’s feeling of contrariety and ambivalence. (Had
he looked at the work
of Frank Norris, Eugene O’Neill, or Henry Adams, Stilgoe’s case would have been
much stronger.) The
point is that the sharp contrast between the enthusiastic supporters of the
railroad in the 1830s and the
minority of intellectual dissenters during that period extended into the 1880s and
beyond.
15. The passage provides information as answers to all of the following questions
EXCEPT:
(a) During which period did the railroad reach the zenith of its popularity in the
United States?
(b) How extensive was the impact of the railroad on the Industrial Revolution in
the United States?
(c) Who are some of the writers of the 1830s who expressed ambivalence toward the
railroad?
(d) What arguments did the writers after the 1880s, as cited by Stilgoe, offer to
justify their support for the
railroad?
16. According to the author of the passage, Stilgoe uses the phrase “romantic-era
distrust” in the passage to
imply that the view he referred to was…
(a) the attitude of a minority of intellectuals toward technological innovation
that began after 1830.
(b) a commonly held attitude toward the railroad during the 1830s.
(c) an ambivalent view of the railroad expressed by many poets and novelists
between 1880 and 1930.
(d) a critique of social and economic developments during the 1830s by a minority
of intellectuals.
17. According to the author, the attitude toward the railroad that was reflected in
writings of Henry James,
Sinclair Lewis, and F. Scott Fitzgerald was…
(a) influenced by the writings of Frank Norris, Eugene O’Neill, and Henry Adams.
(b) similar to that of the minority of writers who had expressed ambivalence toward
the railroad prior to
the 1880s.
(c) consistent with the public attitudes toward the railroad that were reflected in
works of popular culture
after the 1880s.
(d) largely a reaction to the works of writers who had been severely critical of
the railroad in the 1830s.
18. It can be inferred from the passage that the author uses the phrase “works of
popular culture” in the
passage primarily to refer to the
(a) work of a large group of writers that was published between 1880 and 1930 and
that in Stilgoe’s view
was highly critical of the railroad.
(b) work of writers who were heavily influenced by Hawthorne and Thoreau.
(c) large volume of writing produced by Henry Adams, Sinclair Lewis, and Eugene
O’Neill.
(d) work of journalists, novelists, and illustrators that was published after 1880
and that has received little
attention from scholars other than Stilgoe.
. Page 7 of 40
19. Which one of the following can be inferred from the passage regarding the work
of Frank Norris, Eugene
O’Neill, and Henry Adams?
(a) Their work never achieved broad popular appeal.
(b) Their ideas were disseminated to a large audience by the popular culture of the
early 1800s.
(c) Their work expressed a more positive attitude toward the railroad than did
those of Henry James,
Sinclair Lewis, and F. Scott Fitzgerald.
(d) Although they were primarily novelists, some of their work could be classified
as journalism.
20. It can be inferred from the passage that Stilgoe would be most likely to agree
with which one of the
following statements regarding the study of cultural history?
(a) It is impossible to know exactly what period historians are referring to when
they use the term
“romantic era.”
(b) The writing of intellectuals often anticipates ideas and movements that are
later embraced by popular
culture.
(c) Writers who were not popular in their own time tell us little about the age in
which they lived.
(d) The works of popular culture can serve as a reliable indicator of public
attitudes toward modern
innovations like the railroad.
Passage (Q.21-Q.25): In 1975, I was the only woman in a male-dominated class who
enrolled for a
brewery programme, but it did not intimidate me, rather encouraged me to be “better
than the rest”!
Subsequently, I failed to find a brew master’s job in India since breweries were
not ready to take the risk
of employing a woman. Driven by a “strong sense of purpose’ undeterred by initial
failures, I took
challenges head-on, and set up India’s first bio tech company, which went on to
become India’s first
publicly listed bio tech company in 2004.
When I started my Bio tech company in 1978. at the age of 25, biotechnology as a
science was not
widely known in India and there were no first-generation women entrepreneurs. I had
no business
background and had limited financial resources. I had to confront an array of
regulatory business
challenges, besides dealing with lack of credibility associated with women. But I
could succeed against
all these odds because I understood that all challenges can be surmounted with
courage and conviction,
passion and perseverance, commitment and determination, and. most important, with a
bold and fearless
attitude.
Times have changed significantly today; women are seen in greater numbers in what
were previously
perceived as “male-dominated” professions. Like most Asian countries, India today
is home to a new
breed of women, who exude self-confidence irrespective of what strata of society
they come from. They
display a sense of self-belief that allows them to excel in any domain and compete
with their male
counterparts on an equal footing. Many of them are outperforming their male
colleagues and assuming
leadership roles in their respective fields. Today, eight out of the top 10 banks
in India are headed by
women, one out of every three startups in India, are led by women. We also find a
very strong
representation of women in middle management levels across diverse sectors like
science and
technology, pharma and bio tech, banking and financial services, information
technology, retail,
engineering, consumer goods, etc.
There are 16,000 women working at ISRO today and a team of eight bright women
scientists from this
group had enabled India’s recent Mangalyaan — Mars mission. Many more ISRO’s
“Rocket Women”
were a part of the team that launched 104 satellites on a single day from
Sriharikota, last month.
Likewise, the recent ABLE report shows that out of 3,000 biotech start-ups in the
last five years, more
than one third were led by women entrepreneurs.
In building my bio tech company, I have created a company where men and women have
mutual respect
and women can work shoulder to shoulder with their male colleagues. It is this
sense of mutual respect
that enables them to work together as a team for the greater common good. More than
50 per cent of our
large scientific pool is represented by women!

. Page 8 of 40
21. The tone of the passage can be said to be…
(a) incendiary.
(b) acclamatory.
(c) provocative.
(d) obsequious.
22. From the passage, which of the phrases/sentences would indicate that the author
could be a woman?
(a) There were no first-generation women entrepreneurs.
(b) Times have changed significantly today; women are seen in greater numbers in
what were previously
perceived as “male dominated” professions.
(c) I failed to find brew masters job in India since breweries were not ready to
take the risk of employing
a woman.
(d) In building my bio tech company I have created a company where men and women
have mutual
respect and women can work shoulder to shoulder with their male colleagues.
23. According to the passage the term “Rocket Women” is indicative
(a) of women who exude self-confidence.
(b) of women who display a sense of self-belief.
(c) of women working on the project Mangalyaan.
(d) of the women who led one third out of the 3,000 biotech start-ups.
24. All of the following statements are validated by the passage EXCEPT
(a) In the late seventies, only women who belonged to business families entered the
field of business.
(b) Unlike in India, where the number of women entering the hitherto male-dominated
professions is
gradually increasing, women are yet to make their prescence felt as burner’s
entrepreneurs in other
Asian Countries.
(c) The percentage of women constituting the scientific pool in India is higher
than that of men.
(d) A few decades ago, woman’s capabilities as a business entrepreneur were viewed
with skepticism.
25. As used in the passage ‘exude” means all of the following, EXCEPT
(a) Emanate.
(b) Absorb.
(c) Ooze.
(d) Radiate.
Passage (Q.26-Q.30): Feeling overwhelmed by the sheer amount of TV shows that are
now on offer?
You're not alone. Ever since The Sopranos aired in 1999, critics have been waxing
lyrical about the
Golden Age of Television: an era of tremendous creativity that's given us smart,
slick series like The
Wire, Mad Men, Breaking Bad and Game of Thrones, TV shows equal to and surpassing
anything that
Hollywood can hope to muster.
Yet it's not just a rise in quality that's been cited as evidence of this so-called
Golden Age, but a
simultaneous rise in quantity. In 2002, there were 182 scripted TV shows on air
across all platforms -
broadcast, cable and digital. By 2016, this number had risen to 455. It's a trend
that, until now, has only
been increasing: in the five years between 2011 and 2016 alone, the number of
scripted shows increased
by 71 per cent.
A big factor in this ballooning is digital platforms like Netflix, Amazon Prime
Video and Hulu investing in
original content. In the past decade, the number of scripted shows streaming online
has skyrocketed from
virtually nil to literally hundreds. This, of course, can be framed as a good thing
- television audiences
have more choice now than ever before. Hence, the 'Golden Age of Television' label.
But not everybody
sees it that way. In July of 2015, John Landgraf, CEO of FX Networks, famously
coined the term "peak
TV", suggesting that there was now a surplus of television being produced, a glut
of scripted shows that
might easily prove overwhelming for audiences. Landgraf said that while the "peak
TV" industry was

. Page 9 of 40
unlikely to "collapse", he foresaw "a contraction" - and now it looks as though we
might finally be reaching
the zenith from which the industry will fall.
As Netflix, Amazon Prime Video and their streaming rivals continue to put out more
and more content –
and despite the appearance of cutting back, Netflix is actually making more new
shows than ever before -
a variety of new players are now also looking to get in on the action. Disney's
streaming platform, primed
for a 2019 launch, is bringing us a roster of new scripted shows based on their
Marvel and Star Wars
properties. Facebook, meanwhile, is said to have spent as much as $1 billion on
developing original
content for its Facebook Watch platform in 2018. But it's not as easy as all that
to become the new
Netflix, and recently we've seen big businesses start to step away from original
scripted content, or at
least begin to exercise more caution.
26. Which of the following is/are true according to the passage?
i. There were 186 scripted Tv shows across all platforms, in 2002.
ii. There were 455 TV shows across all platforms by 2016.
iii. The number of scripted shows increased by 71 per cent, between 2011 and 2016
(a) Only i)
(b) Only iii)
(c) Both i) and iii)
(d) Both ii) and iii)
27. According to the passage, which of the following is/are not one of the "smart,
slick series" of the Golden
Age of Television?
i. The Wire.
ii. Doctor Who.
iii. Breaking Bad.
iv. Chernobyl.
v. Game of thrones.
(a) i), iii) and v)
(b) ii) and iv)
(c) iii) and v)
(d) i), ii) and iii)
28. Which of the following is/are cited as evidence of Golden Age?
(a) A rise in quantity of TV shows.
(b) A rise in the sale of TV sets.
(c) A rise in the quality of TV shows.
(d) Both a) and c)
29. Which of the following correctly describes the term 'peak TV'?
(a) A sudden rise in the sale of TV sets.
(b) A sudden rise in annual subscriptions on online streaming platforms.
(c) A surplus production of scripted TV shows that might be overwhelming for the
audience.
(d) A sudden fall in the sale of TV sets.
30. According to the passage, why can the skyrocketing of number of shows streaming
online, be termed as
a "good thing"?
(a) Audiences have a wide variety of shows to choose from.
(b) Audiences can stream multiple shows, simultaneously.
(c) Digital platforms are coming up with better content.
(d) Digital platforms are promoting local content
Directions (Q.66 – Q.105): Read the comprehensions carefully and answer the
questions based on it.
Passage (Q.66-Q.70): In a bizarre case Bombay High Court recently allowed an
application filed to
quash an order of the Trial Court by way of which Section 498A, IPC proceedings
were initiated for
inflicting cruelty on a woman.
In doing so, Justice AM Badar opined that Section 498A of the Indian Penal Code
(IPC) would not be
attracted for every instance of cruelty against a married woman. After extracting
the provision in the
judgment, the Court said,
"Section 498A of the Indian Penal Code does not come into play in every case of
harassment and/or
cruelty to a married woman. What is required to be shown is willful conduct of such
a nature as is likely to
drive or propel or compel a married woman to commit suicide or to cause grave
injury or danger to her
life, limb or health. It must be shown that acts were of such a nature, as were
sufficient for causing a
married woman to lose her normal frame of mind."
The Court explained that, "the term “cruelty” implies harsh and harmful conduct
with certain intensity and
persistence. It covers acts causing both physical and mental agony and torture. In
order to hold that the
acts amount to cruelty, it must be shown that such acts amount to unbearable,
continuous, repeated acts
of brutality."
On the other hand, Section 498A says - Husband or relative of husband of a woman
subjecting her to
cruelty — Whoever, being the husband or the relative of the husband of a woman,
subjects such woman
to cruelty shall be punished with imprisonment for a term which may extend to three
years and shall also
be liable to fine.
Explanation. — For the purposes of this section, “cruelty” means—
(a) any wilful conduct which is of such a nature as is likely to drive the woman to
commit suicide or to
cause grave injury or danger to life, limb or health (whether mental or physical)
of the woman; or
(b) harassment of the woman where such harassment is with a view to coercing her or
any person
related to her to meet any unlawful demand for any property or valuable security or
is on account of
failure by her or any person related to her to meet such demand.
66. Praful and Hansa are married for 5 years and have a son named Bittu. Praful was
not happy with his wife
Hansa as she was devoting all her time in learning English rather than taking care
of the household. He
once shouted at her and asked her to pay attention to household chores and their
son. Hansa is upset
after this incident. Decide
(a) Praful has committed cruelty for he has induced his wife Hansa to commit
suicide.
(b) Praful has not committed cruelty for mere shouting is not enough.
(c) Praful is liable under domestic violence act for he coerced her to meet an
illegal demand
(d) Praful is trying to dominate her and as a resultant have caused Hansa mental
torture.
67. Bhaiyaji’s wife Bhabhiji is a famous drug peddler in Mumbai. She sources drugs
from African countries
through her links in customs department. Once Bhaiyaji asked Bhabhiji to get him an
expensive stolen
laptop delivered from US. Getting a stolen laptop through customs was difficult and
therefore Bhabhiji
denied Bhaiyaji’s request. As a resultant, Bhaiyaji beat her up black and blue and
Bhabhiji broke her
wrist. Decide
(a) Bhaiyaji is liable to be punished under Section 489A of IPC as he harassed his
wife for an illegal
demand of property.
(b) Bhaiyaji is not liable for the property was legal to be imported.
(c) Bhaiyaji is liable for causing mental harassment to Bhabhiji for refusing his
demands.
(d) Bhaiyaji's liability will depend on the statement given by Bhabhiji in the
court of law.

. Page 19 of 40
68. Heera and Panna are childhood buddies and considers each other as family. Heera
is married to Sona
who is an income tax officer. Panna who runs a grocery business was caught by
authorities for
committing tax fraud. Panna has been harassing Sona to get his case settled and
when Sona couldn’t
get it settled, he defamed her calling her to be a bad character person. Sona
brought as case against
Panna under Section 498A of IPC. Decide.
(a) Panna is liable for he is as good as Heera’s family and harrassed Heera’s wife.
(b) Panna is not liable for he is not a relative and will not be charged under
Section 498A.
(c) Panna is not liable for he is not a relative though he can be charged under
some other law.
(d) Panna is liable for he not only harassed Sona but also defamed her.
69. Mr. Brainless was a staunch believer in Chanakya's philosophy which stated that
from 0-5 years, a child
must always be forgiven, from 5-18 years he must be beaten/rebuked for his mistakes
and after 18, he
becomes a friend of his parents. Mr. Brainless used to beat up his children often
when they committed
mistakes. Their mother, Mrs. Sensible however did not like this beating at all and
often complained.
Decide
(a) Mr. Brainless is liable under Section 498A of IPC as his beating of children
forms part of cruelty.
(b) Mr. Brainless is not liable under Section 498A of IPC for his acts are not
enough to constitute cruelty.
(c) Mr. Brainless is liable under Section 498A of IPC for the mental health of Mrs.
Sensible was affected
by such beating.
(d) Mr. Brainless is not liable. He is only following Chanakya's philosophy.
70. Pooja files a case against her husband Dooja seeking divorce on the grounds of
cruelty, for not taking
her out on vacations and mostly cancelling their movie dates. Dooja in his response
pleaded heavy work
load in his office as a reason for cancellation of plans. Pooja is adamant in
taking her divorce petition
ahead on grounds of cruelty. Decide.
(a) Pooja will succeed as by cancelling plans every now and then Dooja caused
mental trauma to Pooja.
(b) Pooja will not succeed as Dooja’s act doesn’t qualify as harmful conduct done
intentionally.
(c) Pooja will succeed as she is denied her conjugal right of having company of her
husband.
(d) Pooja will not succeed as cancelling plans is not enough and there should be
more act to constitute
harassment.
Passage (Q.71-Q.76): The Punjab and Haryana HC recently held that a judicial order
to collect voice
samples for investigative comparison purposes can't be said to be violative of the
right to privacy.
The Court referred to the SC's ruling in the case of Ritesh Sinha v. State of Uttar
Pradesh, wherein it was
held that a judicial magistrate can direct an accused to provide his voice samples
for investigation even
without his consent.
Further, the Court observed that the voice samples, in a sense, resemble the
fingerprints and handwriting
and that each person has a distinctive voice. In State of Bombay vs Kathi Kalu
Oghad, the SC had held
that there is no infringement of Article 20(3) if an accused person is asked to
give his specimen
handwriting or signature; or impressions of his fingers, palm, or foot to the
investigating officer or under
orders of a court for the purpose of comparison under the Indian Evidence Act.
The Court also directed thus:"…a Judicial Magistrate must be conceded the power to
order a person to
give a sample of his voice for the purpose of investigation of a crime. Such power
has to be conferred on
a Magistrate by a process of judicial interpretation and in the exercise of
jurisdiction vested in this Court
under Article 142 of the Constitution."
[Extracted, with edits and revisions, from “Judicial Order To Collect Voice Sample
For Investigation
Purposes Doesn't Violate Right To Privacy: Punjab & Haryana High Court”, Sparsh
Upadhyay, LiveLaw,
24 July 2021

. Page 20 of 40
71. Allegations against the CM of Budanagarh have been made to the police by
certain sects of people who
believe him to be indulging in unfair practices regarding the defence deals being
made by him. As against
this, a news agency approaches both the parties and decides to hold a televised
debate between them.
To prove his innocence, the CM demands the defendants to provide a voice
note/recording in order to
substantiate their allegations. However, they fail to do so, claiming hearsay.
Would the CM be exempted
of the allegations?
(a) Yes, as the defendants failed to provide enough satisfactory evidence in order
to substantiate their
claims.
(b) Yes, as the recording/voice note was not made available in order to further
their claims.
(c) No, as the voice note in question was only an ancillary piece of evidence and
not a necessary one.
(d) None of the above.
72. In the above-mentioned case, when the charges are framed and the magistrate
takes cognizance of the
matter, the police dig deeper into the matter. However, CBI tried to take over the
investigative functions
claiming to have jurisdiction here in lieu of the matter being of grave importance.
In light of this, before
being appointed as the official investigative agency, the CBI, sure of being
appointed as such,
approaches the CM and obtains his telephone records via his consent in order to
substantiate their
claims. Would the CBI be right in doing so?
(a) The CBI is wrong as they do not have original jurisdiction in all high-profile
matters.
(b) The CBI is incorrect as it cannot just swoop in and take the matter from the
police’s hands like a
vulture.
(c) The CBI is correct in doing so as they have almost been appointed as the
official investigative agency
here.
(d) The CBI is wrong as it does not officially have jurisdiction here.
73. In a matter of delayed supplies and shipments, A and B have been summoned as
being witnesses in
court appointed arbitration proceedings. When asked to produce their call
recordings with the buyer, they
both refuse pleading claiming their right to privacy as being violated due to the
same. Are they correct in
doing so?
(a) Yes, as they have the right to protect their privacy.
(b) No, as the above mentioned judgments clearly provide the courts with the
opportunity to obtain
electronic evidence such as this.
(c) No, as the arbitrator does not have the power to ask for such evidence.
(d) None of the above.
74. In a bank robbery case, all the accused have been summoned in order to provide
their voice samples to
match the same with the CCTV recordings obtained as evidence. They provide the
court with the same
and the proceedings move further. In a turn of events, one of the accused, who
claims to not have been
there while the robbery was taking place, is spotted by virtue of his footprints in
the investigation being
carried out by the police. This leads to his subsequent conviction; however, he
appeals to the HC
pleading his right to privacy being violated. Is he correct in doing so?
(a) He is correct in doing so as he has been wrongly implicated and the evidence
taken without his
content.
(b) He is wrong as he has not right to privacy on account of him being an accused.
(c) He is wrong in doing so as he has not right to privacy on account of being
convicted.
(d) He is wrong to do so as he has no right to privacy in this situation, the
evidence counts even without
his consent.

. Page 21 of 40
75. In a corporate fraud matter, the Company Law Board is set to hear the petition
of Damani v. Junjhunwala.
The claims against Junjhunwala are of defrauding the company of Rs. 20 crore and in
order to verify the
claims, the Company Law Board orders Junjhunwala to submit a voice recording and
handwritten notes
in order to test the documents against other such evidence. Can the CLB order
Junjhunwala to do so?
(a) Junjhunwala has to abide by the orders of the CLB as it has the requisite
jurisdiction to do so.
(b) Junjhunwala has to abide by the order of the court as directed by law.
(c) No, as the CLB is not competent to order the parties for such evidence.
(d) No, as the CLB is not equivalent to HC and does not possess the requisite
authority.
76. The HC of Maharashtra has been appealed to by Mukesh Rehta, a leading
securities trader. The appeal
has been preferred after the verdict of the Securities Appellate Tribunal, who
convicted him for unfair
practices and suspended his license. The HC is pleaded to by the advocate of the
appellants as to put on
record a recording of the SAT’s chairman who has allegedly falsely adjudicated his
case and was paid by
his Mukesh’s competitors. Can the HC allow this petition?
(a) The HC can allow it as it supersedes other courts as being the principal
appellate authority.
(b) The HC cannot allow it as it is not a Securities empowered court.
(c) The HC need not allow the motion as it is up to their discretion.
(d) None of the above.
Passage (Q.77-Q.82): The Supreme Court has issued a slew of directions to be
followed by all courts to
speed up the execution of decrees. The Court issued the following directions:
In appropriate cases, where the possession is not in dispute and not a question of
fact for adjudication
before the Court, the Court may appoint Commissioner to assess the accurate
description and status of
the property.
After examination of parties or production of documents, the Court must add all
necessary or proper
parties to the suit, so as to avoid multiplicity of proceedings and also make such
joinder of cause of
action in the same suit.
In a suit for payment of money, before settlement of issues, the defendant may be
required to disclose
his assets on oath, to the extent that he is being made liable in a suit. The Court
may further, at any
stage, in appropriate cases during the pendency of suit, using powers under Section
151 CPC, demand
security to ensure satisfaction of any decree.
The Executing Court may on satisfaction of the fact that it is not possible to
execute the decree without
police assistance, direct the concerned Police Station to provide police assistance
to such officials who
are working towards execution of the decree.
[Extracted, with edits and revisions, from “Supreme Court issues directions to
ensure speedy execution of
decrees in civil proceedings [Read Judgment]” Shagun Suryam, Bar & Bench, 30 June
2021;
77. Hari Lal was a successful business man who recently passed away, and is
survived by his two
daughters, Sneha and Saumya. Both of them are potential inheritors of the huge
ancestral house and
other properties. However, there was a dispute on the will left by their father,
and thus the dispute went to
Court. On the basis of the facts of the case, the Court decided that Saumya shall
be the rightful owner of
the ancestral property, and both of them shall be joint stakeholders in all other
properties. Saumya
requested the Hon’ble Court to direct the police station for assistance to obtain
the ownership and
execute the Court decree, since Sneha was currently residing in the ancestral
house. Would her request
be executed?
(a) No, as Sneha has not provided any obstruction as of yet with regards to the
execution of the decree.
(b) The decision would depend on the satisfaction of the Executing Court.
(c) Yes, as Saumya is well within her rights to request police execution from the
Executing Court.
(d) Yes, there is a considerable chance that Sneha might take illegal possession of
the house as she is
already residing in the property.

. Page 22 of 40
78. In the situation given in the question above, if it was found out after
assessing the relevant documents
and evidence produced in Court regarding the case, Sneha & Saumya also had a half-
sister who also
had a claim on the various properties, but was not added to the case. However, the
half-sister is not
interested to stake a claim in the properties. Will she be added as a party to the
suit?
(a) Yes, as the same is being done after assessing all relevant documents produced
before the Court.
(b) Yes, as the Court must add all necessary parties to the suit, if any, so as to
avoid multiplicity of suits.
(c) No, as the same is only needed to be done if necessary, and this is not the
case as the half-sister has
already conveyed her interests.
(d) Yes, and a joinder of cause of action must be done in the same suit.
79. Dinesh and Girnar are partners of a business that deals with the manufacturing
and selling of novelty
coffee products. In the course of business, they soon begin to have some money
disputes with one of
their vendors, Tara. After considerable negotiations, Dinesh and Girnar decide to
take the matter to
Court. During the proceedings, their counsel argues that the defendants are liable
to disclose their assets
before the settlement. Would this argument be valid in Court?
(a) Yes, as the Court can direct Tara to disclose her assets, but only till the
pecuniary limit of the suit.
(b) Yes, as the condition of Tara’s financial assets is pertinent to ascertaining
whether she would be able
to make the payments.
(c) No, as the condition of assets being liable to be disclosed is only applicable
on money suits.
(d) No, because if the Court is not certain about Tara’s ability to make the
payment, they can demand
security from her.
80. In relation to the situation presented above, if it is found during the
proceedings that Tara has defaulted in
her payments, not only to Dinesh & Girnar but to other businesses as well, what are
the measures that
the Court can make use of to assure that the petitioners/applicants get their dues?
(a) The Court can direct the nearest Police Station to assist the petitioners in
case of trouble.
(b) The Court can add all of the other vendors who have not gotten their due
payments as proper parties
to the suit.
(c) The Court can direct the defendants to deposit a security to ensure that the
decree is executed.
(d) The Court can impose an additional fee if the default still occurs regarding
the payments.
81. Johnny and Rolland are best friends with Bob, who has recently passed away. He
has bequeathed his
property to both of them in his will. Both Johnny and Rolland are made aware of the
same, and
consequently fall in a court dispute regarding the percentage of ownership. When
the case was recently
admitted in the Hon’ble Court, other necessary parties were also added to the suit
by the Court. The
defendants held that this action of the Court is not valid. Does the claim of the
defendant hold water?
(a) No, as the Court is well within its powers to add the necessary or proper
parties to the suit.
(b) Yes, as proper documents were not examined before the Court issued such
directive.
(c) The validity of the action of the Court depends on the facts and circumstances
of the case.
(d) No, as without this action there is a probability of multiple suits which would
further cause delay in
justice for other cases.
82. Within the context of the above-mentioned question, if the possession of the
property (with the owners
and possessors being Johnny & Rolland) was not disputed, and there were clear
documents to prove the
same, what could have the steps taken by the Court?
(a) Directed for police assistance when the decree had been declared by the Court.
(b) Ordered Johnny & Rolland to declare their assets on oath.
(c) Ordered either Johnny, or Rolland, or both, to deposit a form of security
acceptable by the Court.
(d) Appointed a commissioner to assess the accurate description and status of the
property.

. Page 23 of 40
Passage (Q.83-Q.88): The Gujarat High Court recently reserved its order on the
maintainability of a
petition against the decades-old liquor prohibition in the state. The challenge is
made on the ground of
the emergence of the right to privacy. It is their primary contention that the
right to privacy includes the
right to be left alone and the right to consume liquor within the precincts of
one's home.
In the case of State of Bombay v. FN Balsara, the Top Court primarily considered
the definition of liquor,
which meant— (a) spirits of wine, methylated spirits, wine, beer, toddy, and all
liquids consisting of or
containing alcohol; and (b) any other intoxicating substance which the Provincial
Government may notify.
In the case of Khoday Distilleries Ltd v. State Of Karnataka, the question was
whether the petitioners
have a fundamental right under Article 19(1)(g), to carry on trade in liquor.
The Top Court heavily relied on Article 47, which enjoins upon the State to raise
the level of nutrition and
the standard of living and to improve public health.
The Court was of the opinion that to achieve this object, the State is required to
bring about a prohibition
on the consumption of intoxicating drinks and drugs which are injurious to health.
Such a prohibition may
be complete or partial and it would also include regulation.
[Extracted, with edits and revisions, from “Is There A Fundamental Right To Drink
Liquor? What Courts
Have Held?” Akshita Saxena, LiveLaw, 11 July 2021
83. Rae worked as a secretary in ABC LLP, one of the biggest law firms of the
country. While working late at
the office, she was offered adulterated alcohol by one of the associates, and was
consequently sexually
assaulted. She was threatened that she would lose her job if she raised her voice,
but she went ahead
and filed an FIR anyway. The matter went to the Court, and Rae requested that her
name be removed
from official records in light of her right to privacy, which includes the right to
be left alone. Is such request
valid?
(a) Yes, as the matter is bound to be covered by the media which would be
detrimental to the victim.
(b) No, as the right to be left alone does not include striking off one’s name from
official records.
(c) Yes, as due to societal & cultural reasons, the victim’s identity must be
sheltered from the public.
(d) None of the above.
84. The town of Schitt’s Creek has been recently suffering from a grave problem of
teenage drinking, which
has already caused a couple of motor accidents. Such incidents are tricky to catch
as the teenagers
usually procure alcohol from their houses itself. To curb this problem, the mayor
of the town, Rolland,
decides to completely ban the sale and consumption of alcohol till further notice.
The sellers of alcoholic
beverages are aghast, and approach the Court regarding the same, contending that
this action violates
their Right to Trade. Would their claim be valid?
(a) Yes, as the sellers possess a right to trade under Article 19(1)(g) of the
Constitution.
(b) No, as the State can bring a complete prohibition on the consumption of liquor.
(c) No, as the sellers do not possess the right to trade under Article 19(1)(g) of
the Constitution.
(d) Yes, as it is unfair to shut down a thriving business just to curb underage
drinking, and the action
taken in unproportionate.
85. With reference to the above question, if the ban on the consumption of
alcoholic substances is made on
public consumption of alcohol instead of complete consumption, would it make it
valid?
(a) Yes, as such an action would not be violative of Article 19(1)(g) of the
Constitution.
(b) Yes, as the State is well within its rights to curb such consumption.
(c) Yes, as this would serve the objective of the enactment i.e., to reduce
underage drinking.
(d) None of the above.

. Page 24 of 40
86. The Central Government and other investigative authorities have recently
tracked the emergence of new
intoxicating substances on the street, named ‘Taxi’ and ‘Blizz’. These drugs are
not consumed physically,
but are rather injected directly in the bloodstream. In an attempt to take further
action against their spread
and usage, these drugs were notified as ‘liquor’. Several contentious citizens and
legal practitioners and
filed a petition in the Hon’ble Court claiming this policy action is invalid. Is
their claim correct?
(a) Yes, as the characteristics of these substances do not match the criteria of
‘liquor’ and hence cannot
be characterised as the same.
(b) No, as the Government can notify and include any other intoxicating substance
as ‘liquor’.
(c) Yes, as the Central Government cannot exercise such powers.
(d) No, as they qualify as intoxicating substances which is clear by the fact that
they are injected in the
bloodstream.
87. In the situation described in the question above, if the Central Government
decides to regulate the use of
these substances by limiting their access to only those sections of the population
that require them for
medicinal purposes, will their action be legally valid?
(a) Yes, as the State can regulate the consumption of intoxicating drugs that are
harmful for public
health.
(b) No, as the State can only regulate the consumption of liquor, and these
substances do not constitute
as liquor.
(c) Yes, as such substances are greatly harmful to the health of the collectively
public.
(d) No, as the State can only completely or partially ban such substances.
88. Sinbad has recently shifted to the state of Agrabah in the course of business.
Consumption of any
alcoholic or intoxicating substances is completely prohibited in Agrabah due to the
faith of the majority
people that reside there, along with their notions of morality. Distressed by the
same, he files a public
interest litigation (PIL) in the High Court for the same, claiming that the right
to privacy includes the right
to consume liquor at the privacy of one’s own residence. Will his petition be
maintainable, and are his
claims valid?
(a) His claims are valid; however, his petition would not be maintainable.
(b) His claims are not valid; however, his petition would be maintainable.
(c) It cannot be ascertained whether his claims are valid or not, but his petition
is maintainable.
(d) The maintainability of the petition depends upon the Hon’ble Court; however,
his claims are valid and
legally sound.
Passage (Q.89-Q.94): Anubha Rawat Choudhary, J., acquitted a person convicted under
Section 497
(adultery) of IPC. The grievance of the petitioner was that he had been convicted
under Section 497
which had been declared unconstitutional by the Supreme Court in the case of Joseph
Shine v. Union of
India, and therefore, once the Section itself had been declared ultra-vires to the
Constitution, no
conviction under Section 497 could be sustained. The petitioner further contended
that the evidences on
record would show that he had been convicted on the basis of hearsay evidence and
presumption.
The Court opined that the Supreme Court had struck down Section 497 as
unconstitutional being
violative of Articles 14, 15, and 21 of the Constitution of India. As per Article
141 of the Constitution, the
law declared by the Supreme Court is binding on all the courts within the territory
of India and such law
applies to all pending proceedings.
Upon perusal of the Joseph Shine case, the Bench stated that, “There is no
indication that the same
would apply prospectively and there is nothing like any prospective operation of
law laid down by the
Supreme Court.”
Therefore, the Court held that the conviction and sentence of the petitioner was
legally not sustainable
and both the impugned judgments and sentence need to be interfered under revisional
jurisdiction to
prevent miscarriage of justice to the petitioner.
[Extracted, with edits and revisions, from “Jhar HC | Is the law laid down by the
Supreme Court applicable
prospectively? Will adultery committed prior to Joseph Shine decision attract
conviction when the offence
has been declared unconstitutional pre-conviction?” Editor, SCCOnlinelBlog, 9 July
2021

. Page 25 of 40
89. In a landmark case presented before the Supreme Court, the Court declared the
law of sedition
contained in the IPC as archaic, unconstitutional, and prone to misuse. Arun Roy, a
prolific activist and
environmentalist, was currently in jail on charges of sedition, and his case was
still pending. Upon this
judgment of the Supreme Court, Arun Roy’s counsel files an appeal for his release,
stating that Arun
does not face any charges anymore, and trying him on these charges would be a grave
miscarriage of
justice. Are these claims valid?
(a) No, as the law of the Supreme Court shall apply prospectively.
(b) Yes, Arun’s conviction would not be legally sustainable.
(c) No, as Arun had already been charged before the sedition was declared
unconstitutional.
(d) Yes, as Arun has already served more than enough punishment.
90. Lata has been happily married to Umesh for the past 5 years. However recently,
Umesh has been
extremely busy with the functioning and establishment of his startup, due to which
he is not able to spend
enough time with his wife. Frustrated by the same, Lata starts an extra-marital
affair with her neighbor,
Rajesh. Soon, Umesh comes to know about the same and is furious. Would these
individuals be liable for
adultery?’
(a) Lata & Rajesh would be liable for adultery.
(b) Rajesh and Umesh would be liable for adultery.
(c) Only Rajesh would be adultery.
(d) None of the above.
91. In the situation presented in the question above, had the case already been
filed and had been pending
before Section 497 had been declared unconstitutional, who would have been held
liable for the charges
of adultery?
(a) Lata & Rajesh would be liable for adultery.
(b) Rajesh and Umesh would be liable for adultery.
(c) Only Rajesh would be adultery.
(d) None of the above.
92. Strictly in accordance with the passage, why was Section 497 of the Indian
Penal Code declared as
unconstitutional by the Apex Court? Select the incorrect statement(s)-
i. The Section was violative of the Fundamental Rights enshrined in the
Constitution of India.
ii. Section 497 was ultra-vires the Constitution, hence was held to be
unconstitutional.
iii. The Section did not grant equal treatment towards men and women, and treated
women as property,
and hence was struck off.
(a) Statements (iii) & (ii).
(b) Statements (i) & (ii)
(c) Statements (iii)
(d) All of the above.
93. Owing to the increasing instances of sexual harassment in workplace and
professional settings, the
Supreme Court in its landmark judgement comes up with a set of directives, commonly
known as the
‘Vishakha Guidelines’. Following suit, the Parliament soon passes a law on the same
lines, but however
makes the punishment less stringent. Soon after, a case of sexual harassment in a
law firm appears in
front of the Hon’ble Court, wherein the counsel for the victim argues that the law
laid down by the
Supreme Court in the form of the ‘Vishakha Guidelines’ shall apply prospectively,
as under Article 141. Is
this argument of the counsel valid?
(a) Yes, as Article 141 mandates the law of the Supreme Court to be applicable on
the entire territory of
India.
(b) No, as this is a case of encroachment of the legislative powers of the
parliament by the judiciary.
(c) Yes, as judicial activism is within the confines of the framework of our
Constitution.
(d) No, as the law of the Supreme Court does not apply prospectively.

. Page 26 of 40
94. Strictly in accordance with the understanding of this passage, how can hearsay
evidence be defined?
(a) Indirect evidence, that is usually verbal.
(b) Type of evidence that cannot be admitted in the Court of law, or cannot be
relied upon.
(c) A witness account by an individual who has not directly witnessed the event,
but rather heard it from
somebody else.
(d) The evidence that uses logical reasoning and deduction to arrive at a
conclusion, and not actual
facts.
Passage (Q.95-Q.99): The Supreme Court has held that to prove an offence of
'kidnapping for ransom'
under Section 364A of the Indian Penal Code (IPC) it is necessary to prove that
along with kidnapping,
the kidnapper also threatened to cause death of the victim or hurt the victim. A
Bench of Justices Ashok
Bhushan and R Subhash Reddy stated that for proving an offence under Section 364A,
fulfilment of the
following conditions is essential:
(i) Kidnapping or abduction of any person or keeping a person in detention after
such kidnapping or
abduction; and (ii) threatening to cause death or hurt to such person, or by his
conduct gives rise to a
reasonable apprehension that such person may be put to death or hurt or; (iii)
causes hurt or death to
such person in order to compel the Government or any foreign State or any
Governmental organization
or any other person to do or abstain from doing any act or to pay a ransom.
The top court noted that after establishing first condition, one more condition has
to be fulfilled since after
first condition, word used is “and”. Thus, in addition to the first condition
either condition (ii) or (iii) has to
be proved, failing which conviction under Section 364A cannot be sustained."The
High Court has not
dealt with the grounds taken before it by the accused that no threat to cause death
or hurt was extended
by the accused. From the judgment of the high court, thus, it can be said that
there is no finding
regarding fulfilment of condition (ii)" the Court observed. However, the top court
upheld the conviction
under Section 363 of IPC considering the offence of 'kidnapping' was proven and
thus imposed
punishment of seven years imprisonment and fine of Rs. 5,000.
Debayan Roy, Person guilty of 'kidnapping for ransom' under Section 364A IPC only
if he threatens to
hurt or cause death of victim: Supreme Court (Bar and Bench, 28 June 2021)
<https://www.barandbench.com/news/person-guilty-kidnapping-for-ransom-section-364a-
ipc-threatens#death-hurt-supreme-court-judgment> as accessed on 29 June 2021.
95. Harish, a car mechanic, forcefully took a girl from her home and then called
her parents for a ransom of
ten lacs. When the parents refrained from consenting to any demands made by him, he
sent them a
finger nail of their child without any message. The parents, then, paid the ransom
and after the girl was
dropped back to them, they filed a police complaint against Harish. Police managed
to nab Harish and
charged him under section 364A. In the Court Harish contended the charges on the
ground that he never
made any threat to the life of the girl and the finger nail did not belong to the
girl. The Court noticing that
all the finger nails of the girl are intact dropped the charges under 364A and
charged him with offence
under section 363. Choose the correct option.
(a) The Court has rightly dropped the charges under section 364A as there was no
hurt actually caused
to the girl as evident from all her finger nails being intact.
(b) The Court has wrongly dropped the charges under section 364A as sending a
finger nail to the
parents upon not paying the ransom amounts to extending threat to cause death of
hurt.
(c) The Court has rightly dropped the charges under section 364A as sending a
finger nail does not
amount to grievous hurt which would compel them to pay ransom.
(d) The Court has wrongly dropped the charges under section 364A as an incident of
kidnapping is in
itself induces apprehension of death of hurt.

. Page 27 of 40
96. Irfan has made an appeal before the High Court against him being convicted
under section 364A of the
Indian Penal Code for the kidnapping of a minor. He contended that neither did he
make any threat nor
the statements of the complainants or the arguments of the prosecution suggests
that he threatened to
cause any hurt or death to the minor for demand of ransom. He thus requests that
the charge under
section 364A be dropped and his sentenced be reduced. The State on the other hand
argues that it was
only fear of any hurt to the minor that her parents agreed to pay the ransom. Thus,
Irfan has been rightly
convicted under section 364A. Decide whether appeal shall be allowed or not.
(a) Appeal shall be allowed as it is the liability of the prosecution to prove that
the accused either
threatened to cause either death or hurt to the person kidnapped, in the absence of
any such
evidence Irfan shall not be charged under section 364A.
(b) Appeal shall not be allowed because as pointed out by the State, it was only
the apprehension of
threat to the wellbeing of their daughter that they agreed to pay the ransom to the
accused.
(c) Appeal shall be allowed as there was no threat from Irfan to cause death or
hurt to the victim which is
an essential requisite of section 364A.
(d) Appeal shall not be allowed as the liability is on the Irfan to prove that
there was no threat to hurt the
victim that would attract section 364A.
97. Chaudharys and Maliks were known for their enmity in the town, these were two
rival gangs both into the
business of extortion. One day when Vishal Malik left his home, he was abducted by
Angad Chaudhary.
Due to this enmity, Vishal was physically tortured until he was found by the police
in one of Chaudharys’
warehouses. Angad was charged under section 364A and later arrested by the Police.
The Court
convicted Angad for the offence, who has now made an appeal before the High Court
against the
charges. Which of the following shall be judgment of the High Court?
(a) Angad has been rightly charged with offence under section 364A as he has hurt
Vishal after
abducting him which satisfies all the requisites of the section.
(b) Angad has been wrongly charged with offence under section 364A as the charges
imposed are a
result of the personal enmity between the two groups and there is no proof of any
hurt being caused.
(c) Angad has been rightly charged under section 364A as the animosity towards
Vishal further aid the
claims of physical hurt being caused to Vishal by him thus attracting section 364A.
(d) Angad has been wrongly charged with section 364A as the actions and conduct of
Angad do not
satisfy all the requisites of the section, thus the section will not be attracted.
98. Meera kidnapped her employer’s son with the greed of making some easy money.
She called her
employer to ask for ransom if they wanted their son back. After receiving a
whopping sum of 20 lacs the
boy returned back to home. Later, Meera was arrested by the Police and she was
charged with section
364A and 363 for kidnapping the child. Court acquitted her for absence of any
evidence suggesting that
Meera threatened to cause death or hurt to the victim. The State has challenged the
order before the
High Court as Meera has been released despite kidnapping the child. Choose the
correct option.
(a) Meera has been wrongly acquitted as even though there is no evidence suggesting
threat to cause
hurt or death to the boy, kidnapping is per se an offence punishable by law.
(b) Meera has been rightly acquitted as when there is evident absence of any threat
to cause death or
hurt to the boy she shall not be punished under section 364A.
(c) Meera has been wrongly acquitted as there was a threat of not returning the boy
to her employers if
she does not receive the ransom.
(d) Meera has been rightly acquitted as where an accused has been charged under
incorrect section, the
Court shall make the order in the favour of the accused to not cause prejudice
against their interest.

. Page 28 of 40
99. Assertion (A): Only when kidnapping is coupled with grievous hurt or death is
it an offence under section
364A of the IPC.
Reasoning (R): The offences of kidnapping when not coupled with threat or acts of
causing hurt or death
are offences under section 363 of the IPC.
(a) Both A and R are correct and R is the correct explanation of A
(b) Both A and R are correct but R is not the correct explanation of A
(c) A is correct and R is incorrect
(d) A is incorrect and R is correct
Passage (Q.100-Q.105): In Maharashtra, the Maharashtra Education Institutions
(Regulation of Fee) Act,
2011 and the Amendment Act of 2018 mandate a Parent-Teacher Association (PTA),
Executive
Committee and Divisional Fee Regulation Committee (DFRC) to ensure proper
management and
revision/regulation of school fees. When increasing school fees, the school
management must submit the
details to the Executive Committee at least 6 months before the next academic year.
While giving its
approval, the Executive Committee has the authority to decide the fee afresh and
convey the same within
a period of 30 days from receipt of details of fee proposed by school management.
This must then be displayed on the notice board and school website in English,
Marathi and the
respective medium of the school. In case of indecisiveness on fixing of school
fees, the matter must be
referred to the DFRC. Until a decision is reached, the management must collect the
fee as that of the
previous years, plus 15% or the proposed increased fee.
If parents are aggrieved by the decision of the Executive Committee or the fee
increased by the
management, they can appeal to the DFRC by duly attesting their signatures on a
form within 30 days of
receipt of the decision. In case of further grievance, a further appeal may be
raised to the Revision
Committee within a period of 30 days, but not exceeding 60 days of receipt of the
decision.
(Extracted with requisite revisions and edits from‘Debriefed: The controversy
surrounding payment of
school fees amid the COVID-19 pandemic’
100. St. Anthonies, a renowned educational institution in Thane decided to hike the
educational fees in order
to accommodate the expense to modify the school to the online friendly curriculum
from the current
semester. Since the reason of such online mode of education was the pandemic and to
avoid the loss in
the studies of its students the school didn’t had ample time to intimate the
Executive Committee 6 months
before the start of the next academic year but intimated them only 2 months in
advance. It was
contended by the school that the limit on time period would be detrimental to the
education of its students
as without such a hike in the fee, the modern online modes/ smart classes could not
be adequately
incorporated. This contention however, didn’t budge the decision of the Committee
who emphasized on
the 6 months intimation. Is the act of the Committee correct?
(a) No, the decision of the committee would be detrimental to the education of the
students.
(b) No, the pandemic had an unforeseeable impact which couldn’t have been
ascertained by the School
authorities.
(c) Yes, the delay by the school authorities was of significant duration which is
incurable and unjustifiable
despite the pandemic.
(d) Yes, there is a mandatory requirement of intimation before 6 months to the
Committee.
101. Despite the decision of the Committee, the school went ahead with the revised
fee structure keeping in
mind the welfare of the students as the school management ultimately has the power
to decide the fee on
the basis of the expense incurred and the role of the Committee is merely
discretionary. Is the statement
of the school correct?
(a) Yes, the role of the Committee is merely discretionary and the fee is proposed
by the school
management only.
(b) No, the Executive Committee is empowered to give its approval and even has the
authority to decide
the fee afresh.
(c) Yes, the revised fee by the school management is not groundless but based
premising on the welfare
of the students and the expenses incurred by it.

. Page 29 of 40
(d) No, the Committee is empowered to determine the fee and no objection can be
made against their
decision.
102. The Committee decided against any hike in the fee of the school. Against this
decision, the school made
an appeal to DFRC which supported the decision by the Committee. However, the
school decided to
make a plea before the Court and till then to collect the fee as that of the
previous years, plus 15% or the
proposed increased fee. The parents protested against this hike in fee. Is the
action of the school
justified?
(a) Yes, the school management is empowered to collect the fee as that of the
previous years, plus 15%
or the proposed increased fee till the time a decision is reached.
(b) Yes, the appeal has not yet been determined and decided by the court.
(c) No, the decision has already been reached by the DFRC and hence no hike in fee
can be done by
the school.
(d) No, the Committee has already decided the fee that can be charged by the
school.
103. The next academic year, the school gave the revised fee structure before 6
months in order to avoid any
unfavorable event in the future. However, the committee failed in conveying the
decision within 30 days
as the head of the Committee was affected by Covid and hence a majority decision
couldn’t be arrived at.
The school however protested against this delay and brought an action the
Committee. Is their action
likely to succeed?
(a) Yes, the committee had failed to convey the decision within the prescribed time
fame of 30 days.
(b) No, the circumstances in question were unforeseeable and hence the delay was
not due to any
negligence by the Committee.
(c) Yes, the delay in conveying the decision will affect the bona fide by the
school to ensure the welfare
of its students.
(d) Yes, the school has not been at any fault and had complied with all the
deadlines.
104. Since the school has adopted an online mode of teaching, the school displayed
the notice of increase in
school fee only on the website and even personally mailed the parents of the
students in order to intimate
them. However, the school due to so many precautions already taken about intimation
decided not to go
forth with the notice in the notice board of the school. Mr. Krishnan, a parent of
a student protested
against this failure as it was a mandatory condition and not substitutable. Is the
argument of Mr. Krishnan
correct?
(a) No, the school had already taken sufficient cautions including sending mails to
the parents of the
students.
(b) Yes, the school had failed to provide any sufficient or compelling arguments in
its defence for failure
to comply with such guidelines.
(c) No, the element of displaying in the notice board is merely prescriptive and
not binding.
(d) Yes, school had failed to comply with a mandatory condition of displaying the
fee structure in the
notice board.
105. Mr. Krishnan was against the actions of the institutions and therefore decided
to directly appeal before
the Magistrate Court instead of appealing to the DFRC and therefore the court
rejected the application for
failure to comply with the recourse already provided. It was contended that such an
action of the court will
hinder the justice delivery mechanism. Is the action of the Court correct?
(a) No, the decision of the court will affect the justice delivery mechanism.
(b) Yes, the proper recourse lies with an appeal to the DFRC which had not been
complied with by the
petitioner.
(c) No, the recourse to the court is independent to the remedy with the DFRC.
(d) Yes, court can exercise its discretionary power which will justify its action.

. Page 30 of 40
SECTION - D: LOGICAL REASONING
Direction (Q.106-Q.109): Study the following information to answer the given
questions.
A, B, C, D, E, F, G and H are sitting around a circular table facing the centre. No
two males or two
females are immediate neighbours of each other. A is wife of H. A sits third to the
left of E. F sits second
to the right of D. D is not an immediate neighbour of A or E. H and C are immediate
neighbours of each
other. F is not an immediate neighbour of his wife B.
106. Which of the following is true about G?
(a) G is a male
(b) G sits exactly between F and H
(c) G sits third to the left of E
(d) G sits second to the right of B
107. Who sits third to the left of B?
(a) F
(b) H
(c) D
(d) A
108. How many persons sit between B and F when counted in anti-clockwise direction
from B?
(a) One
(b) Two
(c) Three
(d) Four
109. Four of the following five are alike in a certain way and so form a group.
Which is the one that does not
belong to that group?
(a) H
(b) F
(c) E
(d) G
110. You are given two statements. Identify the nature of the relationship between
them:
Statement A: Two stroke engines cause more pollution than four stroke engine.
Statement B: The new government regulation requires a regular vehicle check for
adulteration of fuel.
(a) A is the cause and B is the effect
(b) B is the cause and A is the effect.
(c) A and B are effects of same common cause.
(d) A and B are effects of independent causes.
111. Identify the correct course of action:
The Intelligence Bureau has received information that terrorists are planning to
attack the water supply of
Bhopal by poisoning the Upper Lake, the main source of water.
A. Security around the lake should be tightened considerably.
B. Nobody should be allowed around the lake.
C. Regular traffic should be diverted away from the lake.
(a) A
(b) B and C
(c) A and C
(d) All the above

. Page 31 of 40
112. The last decade has witnessed an explosion in information and knowledge
leading to rapid and radical
innovation. Unfortunately increasing information has also led to people developing
highly specialized skill
sets. This will result in a drastic slowdown in innovation as people with cross-
functional skills sets as
opposed to specialists are becoming fewer by the day.
Which of the following is an assumption on which the above argument is based?
(a) The talent of people with cross-functional skills is a must-have for
innovation.
(b) Increased specialization makes people get into unnecessary details.
(c) The pace of innovation has been so great that people have not been able to
develop cross-functional
skills.
(d) Innovation is the only way towards progress and development.
113. Hubaira: Anton Chekhov’s stories are sombre and pessimistic, which is a sure
sign of inferior writing. I
have never read a single story of his that ends happily.
Jebel: Chekhov was one of the finest writers of the past 30 years.
Granted, his stories are characterized by sombreness and pessimism, but they are
also wryly humorous,
compassionate, and beautifully structured.
On the basis of their statements, Hubaira and Jebel are committed to disagreeing
over whether
(a) Chekhov’s stories are truly compassionate
(b) Chekhov’s stories are pessimistic in their vision
(c) stories that are characterized by sombreness and pessimism can appropriately be
called humorous
(d) stories that are well written can be sombre and pessimistic
114. Research shows that when people work with a positive mind-set, performance on
nearly every level—
productivity, creativity, engagement— improves. Yet happiness is perhaps the most
misunderstood driver
of performance. For one, most people believe that success precedes happiness. "Once
I get a promotion,
I'll be happy," they think. Or, "Once I hit my sales target, I'll feel great." But
because success is a moving
target—as soon as you hit your target, you raise it again.
Which of the following inferences can be drawn from the given argument?
(a) The happiness that results from success is fleeting.
(b) The habits you cultivate, the way you interact with coworkers, how you think
about stress— all these
can be managed to increase your happiness and your chances of success.
(c) Every business outcome shows improvement when the brain is positive.
(d) All the above
115. Statement: “Use VLCC Liquorice cold cream Cold Cream for fair complexion.” –
An advertisement.
Assumptions:
I. People like to use cream for fair complexion.
II. People are easily fooled.
III. People respond to advertisements.
(a) Only I is implicit
(b) Only I and II are implicit
(c) Only II is implicit
(d) Only I and III are implicit
Direction (Q.116-Q.120): Investment in education yields both private and social
returns. Private returns,
like wages, accrue to individuals. Social returns accrue to society. Textbook
economics suggests that
private returns rise with one’s level of education, but social returns peak at the
elementary levels. For
only when educated people follow rules such as queuing, using washrooms, washing
hands and
protecting public property, the collective returns from such actions generate a
huge social value such as
cleaner, healthier and disciplined societies.
Education is everything that expands our capabilities — as individuals and as
society. The novel
coronavirus pandemic has given us Indians an opportunity to re-evaluate how our
schools should expand
. Page 32 of 40
our capabilities. While academic prowess in math, science and language is
essential, what about the
issue of household chores? Or connecting with the community or nature?
Does learning household chores expand one’s capabilities? For if people do not know
how to keep their
spaces clean, cook their own food, do their laundry, it just needs a shock like a
COVID-19 pandemic to
occur and for a society dependent on the labour of others to feel incredibly
disrupted. Add to it shortages
of essential supplies and overstretched public resources, and social disruption is
inevitable. So, how can
we prepare for the future? [1] The answer: through our elementary schools.
Can we learn from another country? Yes, Japan.
[Extracted with edits and revisions from The Hindu article by Prachi Gupta and
Prajakta Khare, July 29,
2021
116. Which among the following would be a logical corollary to the passage?
(a) Comparison between Indian and Japanese elementary schools
(b) Reasons one should migrate to Japan from India
(c) Reasons the Japanese could counter the pandemic efficiently
(d) Importance of performing house chores
117. In the passage above, the author has not presented support for which of the
following questions?
(a) Why should the Indian elementary school system learn from its Japanese
counterpart?
(b) How can an educated society with social values be beneficial to the nation?
(c) What are some of the learnings from the novel coronavirus pandemic to India?
(d) All of the above questions have been answered by the author.
118. Which among the following options, if true, would strengthen the author’s
argument in statement [1]?
(a) Children who learn household chores in the elementary stage are better able to
manage resources.
(b) The grasping capacity of a child is highest when the child turns into an
adolescent.
(c) Children who are denied elementary education perform relatively similar to
those who acquire
elementary education in handling tough situations.
(d) How well a person can manage resources depends on which university he/she
attends.
119. Which among the following is the correct expression of the author’s opinion?
(a) The objective of improving India’s elementary schools is to become a superpower
in future.
(b) The performance of household chores and connecting with nature do not lie on
the same plane.
(c) Healthier and cleaner societies are the major causes of the novel coronavirus
pandemic.
(d) When educated people do not follow social rules, the benefits of being educated
decrease.
120. Which among the following options strengthens the author’s claim that India
could learn from Japan for a
better elementary school system?
I. Japan ranks among the top in the Program for International Student Assessment
which evaluates
elementary students on their prowess in resource management.
II. The Japanese education policy transitioned from rote memorization-based
approach to 'holistic
development' approach three decades ago.
III. Indian elementary school system is rated as one of the best in the world
recently.
(a) Both I and II
(b) Both II and III
(c) Both I and III
(d) None of the above

. Page 33 of 40
Direction (Q.121-Q.125): What are the best ways to make sure the indifferent
protect themselves from
Covid: vaccine passes for public places, pop-up clinics discounts? How do you
tackle outright
skepticism? When will vulnerable people start receiving booster shots?
These questions are now at the fore in many countries. Across high-income nations,
around half the
population has been vaccinated, allowing life to return to something approaching
normality. In the UK,
hospitalization figures are currently better than anticipated, despite high
infection rates. But the situation
is precarious. The public’s caution may not last; schools will return in September;
we are relaxing travel
restrictions; and there is a marked slowdown in vaccination.
These are important issues. The bigger one, however, is ensuring that the rest of
the world is adequately
protected – especially as travel restrictions are loosened. Vaccines reduce but do
not eliminate
transmission; unless they are very widespread, we not only abandon many countries
to the worst but also
risk our own gains in the process. Internationally, four million cases were
reported to the World Health
Organization last week and around 9,000 people are dying each day. The more widely
that the virus can
circulate, the greater the risk of new, and vaccine-resistant variants emerging.
Covax had planned to make at least 640m doses available worldwide. Its target was
to ensure each
nation could protect at least 20% of its population – high-risk groups – by the end
of this year. But in low#income countries which comprise most of the world’s
population, only around 1.3% of people have been
vaccinated, according to the UNDP. For many poorer places, the main obstacle to
vaccination remains
supply, not demand. While richer nations agonize over how to cajole or induce those
at low risk from
Covid to protect themselves and others by having vaccines, they are denying doses
to high-risk people
elsewhere – even at the risk of wasting doses entirely.
The WHO this week urged wealthier nations to delay using booster shots, saying a
moratorium could
allow it to meet the very modest goal of vaccinating at least 10% of every
country’s population by the end
of September. The Biden administration called this a false choice, declaring that
the US can do both. Yet,
while its purchase of 500m Pfizer doses for Covax is welcome, it is diverting
funding from vaccination
drives in poorer countries to buy them and this should be criticized. Wealthier
nations need to stump up
more for both purchase and delivery of doses. They also need to waive intellectual
property rights to
boost the supply.
Vaccines present us with difficult practical and moral choices. If we look only to
narrow national interests,
we will not just betray the most vulnerable; we may all pay the price.
[Extracted with edits and revisions from The Guardian, August 5, 2021]
121. Which among the following is the most appropriate representation of the main
idea of the passage?
(a) People who are suspicious about vaccines should know that they are insulting
the tedious work of
pharmaceutical companies.
(b) While wealthier nations worry about persuading people to keep themselves safe,
poorer ones
struggle to inoculate their population. In low-income countries, supply of vaccines
is a bigger
challenge.
(c) Wealthier nations are better placed in terms of vaccine supply than their
counterparts due to their
robust health infrastructure.
(d) Vaccine diplomacy is one of the new age diplomatic tactics that have come up
and countries should
exercise caution before moving ahead with it.

. Page 34 of 40
122. Which one is the correct implied expression of the opinion held by the author
of the passage?
(a) Governments should first vaccinate their population before moving ahead with
helping poorer
countries.
(b) In the UK, the authorities should ensure that the number of people getting
vaccinated everyday does
not come down significantly.
(c) The slowdown in the number of people getting vaccinated in wealthier countries
is not a major cause
for concern.
(d) All of the above are a correct representation of the author's opinion.
123. Out of the following events, the implications of which one is likely to be
that wealthier nations lose at least
some of the gains that they have made to counter the pandemic?
(a) Poorer nations are left to fend for themselves for vaccines.
(b) Adequate support for the supply of vaccines is provided to the poorer nations.
(c) The population of wealthier nations becomes complacent.
(d) Both (a) and (c)
124. Which one of the following is correct according to the passage?
(a) The US is one such country which keeps their word after promising something.
(b) In some wealthier countries, life has returned back to normal after most of
their population is
inoculated.
(c) The request of the WHO to delay the use of booster doses by the poorer nations
is to make sure
some percentage of the population of every country is vaccinated.
(d) Covax has a long way to go to achieve its goal of vaccinating 640m people
worldwide.
125. Which one among the following, if true, can weaken the author’s claim that the
US’s action should be
criticized?
(a) The poorer nations in which the US promised to hold vaccination drives are
facing civil war.
(b) The US is one of the strongest nations around the world with the strongest air
force.
(c) The US is one of the largest hoarders of vaccines with some of the most
advanced health
infrastructure.
(d) The diplomats in the US are among the best in the world with the capacity to
negotiate favorably with
the diplomats of every country.
Direction (Q.126-Q.129): If sharks can have a week of their own, why not jellyfish?
Is it because jellyfish
do not have a menacing maw, or a fear-inducing dorsal fin? Is it because no one's
made a sinister movie
where jellyfish terrorize beachgoers? Let's face it, jellyfish are more bothersome
than threatening, which
is why no one would tune into "Jellyfish Week." Sure, their stings hurt and itch,
but sharks can tear a limb
off. If that doesn't get you your own show, nothing will. But what if we told you
there's a type of jellyfish
that is more menacing than the most threatening shark. It goes by the scientific
name Cubozoa, but is
more commonly referred to as the "box jellyfish." Unlike their annoying cousins,
the box jellyfish has a
powerful venom.
The simple facts are these: the number of people killed each year by box jellyfish
is almost the same as
the number of people killed by lightning on golf courses. And the number of people
killed by lightning on
golf courses each year is about the same as the number of people who died falling
from their terrace. All
the horrible myths and gruesome stories aside, therefore, a box jellyfish is in
fact almost as dangerous as
playing on the terrace or a game of golf.
126. Which one of the following is an assumption that the author relies upon in the
IInd part of the passage?
(a) Most incidents involving box jellyfish are fatal.
(b) Box jellyfish are no longer the danger they once were.
(c) The number of fatalities per year is an adequate indication of something's
dangerousness.
(d) A golf course is a particularly dangerous place to be in a thunderstorm

. Page 35 of 40
127. Which one of the following, if true, would most effectively undermine the
author's argument in the IInd
paragraph of the passage?
(a) Although the number of people killed by lightning on golf courses each year is
very small, the total
number of lightning fatalities is many times greater
(b) Terraces are among the safest place of the house; were the author to compare
fatalities from the
other places of the house in general, he would get a much higher figure.
(c) Fish in general-including shark, whale, as well as box jellyfish-kill many more
people than done by
lightning at the golf course.
(d) Statistics show that the number of times people play at terrace each year
exceeds the number of
times people play golf each year, which in turn far exceeds the number of contacts
people have with
box jellyfish each year.
128. The patterns of reasoning in the second paragraph of the above passage closely
resembles the pattern
of reasoning in which one of the following -
(a) In a cricket match Himanshu scored almost equals runs to the runs that are
scored by Pritesh. And
Pritesh scored almost equal runs to the runs which are scored by Arshad. Therefore,
Himanshu is
almost as good batsmen as Pritesh or Arshad.
(b) Himanshu Scored 30 runs in a cricket match and Pritesh scored 90 runs in that
match. Arshad did not
played that match. Therefore, Himanshu is almost as good as Pritesh and Arshad.
(c) Pritesh scored 4 goals in a football match and Himanshu scored 65 runs in a
cricket match.
Therefore, they both are great players.
(d) All of the above.
129. Based on the information in the given passage, which of the following is most
likely to be true?
(a) Sharks are more bothersome and threatening than Jellyfish.
(b) Jellyfishes are more bothersome and threatening than Shark.
(c) Jellyfishes are more bothersome than Sharks and Sharks are more threatening
than Jellyfishes.
(d) Jellyfish is superior to Shark.
Direction (Q.130-Q.135): One of the largest recent innovations has been the
development of genetically
modified organisms or, GMOs. GMOs are plants which have been genetically modified
to tolerate
conditions or chemicals which would normally be detrimental to their survival.
However, all opponents of
GMOs in US disapprove of their widespread use, arguing that because of insufficient
government
regulations GMOs have the potential to do more harm than good. The federal
government should more
strictly regulate GMOs because of confusion between regulatory bodies, how GMOs
effect international
trade and how they carry the risk of disrupting commerce within the United States.
GMOs should be regulated more strictly because the current system contains many
overlaps and gaps
and cannot efficiently monitor the thriving industry of genetic engineering. As
most U.S GMO regulations
are outdated and no longer directly relate to the regulation of GMOs. With the
responsibilities allocated
out between multiple agencies, no comprehensive method of assessment and regulation
exists. As a
result, the regulatory framework surrounding GMOs is imbalanced. In its infancy,
the GMO industry was
small, manageable and its effects could not be seen or felt by the public.
However, thirty years later, its billion-dollar footprint is seen on nearly every
store shelf and in countless
agriculture sectors. The current system is flawed, requiring broader and stricter
regulation to ensure that
the rapidly expanding genetic engineering industry is receiving the attention it
requires. As GMOs begin
to slip through the cracks in the regulatory bureaucracy, the imperfections of the
current system come to
light through the harms seen by many farmers. Regulation of GMOs should be
tightened because due to
the gaps in the regulatory process environmental problems have arisen as a result
of GMO integration
into the agricultural market. These concerns are not limited strictly to American
soil though, the
contamination of GMO crops with conventional crops is beginning to have
international consequences as
well.

.
130. Which of the following statements is in line with the author’s line of
thoughts?
(a) GMO’s are considered to be a sustainable business practice
(b) GMO’s are a must in developing countries
(c) GMO’s cannot be regulated at all to a great extent
(d) The huge GMO market now has become very difficult to be managed
131. Which of the following can be inferred from the passage?
(a) The GMO sector is completely unregulated in the U.S.
(b) GMO’s have started to negatively impact the environment in the U.S.
(c) Both (A) & (B)
(d) Neither (A) nor (B)
132. Which of the following, if true, would seriously undermine the author’s
argument?
(a) GMO exist all around the world
(b) The population today cannot survive without the use of GMO’s
(c) There exist many voluntary GMO regulation organizations that are being abided
by
(d) Pollution caused by GMOs is way more harmful than that contributed by other
sectors
133. The statement “GMO crops are more harmful when left unregulated” is:
(a) Probably True
(b) Definitely True
(c) Probably False
(d) Definitely False
134. What could be an apt heading for the passage?
(a) GMO is the new future
(b) The regulation of GMOs in the U.S.
(c) The Complicity of sustainable practices
(d) What the government is not doing right
135. Which of the following statements would the author definitely not agree to?
(a) Some people who are protesting against the introduction of GMOs are against
technology updation.
(b) The GMOs can distort internal trade of a country if not regulated properly.
(c) There have been rising cases of conventional crops being used in the name of
GMOs
(d) None of the above

mock 9
Passage (Q.1-Q.5): Forecasting is always a highly uncertain activity. The longer
the time frame one is
making predictions about and the more complicated the prediction, the more room
there is for error. This
means that while it might be rational to make a projection about something simple
in the near future, it is
probably pointless to make projections about something complex in the very distant
future. Economists
have known for many years that people tend to discount the future. That means we
put a greater value
on something which we can get immediately than something we have to wait for. More
attention is paid to
pressing short-term needs while longer-term investments go unheeded.
Psychologists have also found that futures that are close at hand seem concrete and
detailed while
those that are further away seem abstract and stylised. Near futures were more
likely to be based on
personal experience, while the distance future was shaped by ideologies and
theories. When a future
seems to be closer and more concrete, people tend to think it is more likely to
occur. And studies have
shown that near and concrete futures are also more likely to spark us into action.
So, the preference for
concrete, close-at-hand futures mean people tend to put off thinking about more
abstract and distant
possibilities.
The human aversion to thinking about the future is partially hard-wired. But there
are also particular social
conditions that make us more likely to give up on the future. Sociologists have
argued that for people
living in fairly stable societies, it is possible to generate stories about what
the future might be like. But in
moments of profound social dislocation and upheaval, these stories stop making
sense and we lose a
sense of the future and how to prepare for it. This is what happened in many native
American
communities during colonialism. This is how Plenty Coups, the leader of the Crow
people, described it:
“When the buffalo went away the hearts of my people fell to the ground, and they
could not lift them up
again. After this nothing happened.”
But instead of being thrown into a sense of despair by the future, Gibson thinks we
should be a little more
optimistic. “This new found state of No Future is, in my opinion, a very good thing
… It indicates a kind of
maturity, an understanding that every future is someone else’s past, every present
is someone else’s
future”.
1. “Psychologists have also found that futures that are close at hand seem concrete
and detailed while
those that are further away seem abstract and stylised.” What can be inferred from
the given statement?
(a) The immediate future seems to be based on randomness with plans in tandem but
distant future seems
more tactical and, therefore, designed as per the need.
(b) The immediate fututure seems to be based on realities with plans in place but
distant future seems more
hypothetical and, therefore, designed as per the need.
(c) The immediate future is lined up with plans in absolutes and the remote future
has nothing to offer.
(d) The nearer future has clarity with regard to all aspects of life; whereas, the
remote future is vague and
random.
2. “The human aversion to thinking about the future is partially hard-wired.” The
interpretation of the word
‘hard-wired’ in the passage is…
(a) Pertaining to or being an intrinsic and relatively unmodified behaviour
pattern.
(b) Pertaining to or being an intrinsic and relatively modified behaviour pattern.
(c) Pertaining to or being an intrinsic and relatively disarranged behaviour
pattern.
(d) Pertaining to or being an intrinsic and relatively vituperative behaviour
pattern.
3. Which of the following is opposite in meaning to the word ‘concrete’ given in
the passage?
(a) Intangible (b) Precise (c) Substantial (d) Pliable

. Page 3 of 40
4. The primary purpose of the passage is…
(a) To bring to fore the various ideologies with regard to social conditions that
play a pivotal role in human
perception of immediate and distant future.
(b) To refute the psychology that projections about distant future are based on
ideologies and theories and
therefore remain unattainable and unrelatable to people whereas, the projection of
the immediate future
is more concrete and likely to take place.
(c) To laud the efforts taken by psychologists to precisely sum up as to why the
predictions of the closer
future find an echo with people as compare to the predictions about the distant
future.
(d) To highlight the fact that forecasting of distant future is prone to errors as
humans are conditioned to see
the immediate future more relatable and concrete; whereas a distant future is one
of abstract, full of
theories and ideologies.
5. The approach taken by the author in the last paragraph is…
(a) Buoyant (b) Dour (c) Neutral (d) Idealistic
Passage (Q.6-Q.10): For Durkheim, religion is endemic to social life, because it is
a necessary feature of
all moral communities. The key term here is sacred. By sacred Durkheim meant
something like,
unquestionable, taken-for-granted, and binding, or emitting a special aura.
Wherever you find the sacred,
thought Durkheim, there you have religion.
There is a sense in which this way of thinking has become entirely commonplace.
When people describe,
say, European soccer fans as religious in their devotion to their home team, they
are drawing on a
Durkheimian conception of religion. They are signaling the fact that fans of this
nature are intensely
devoted to their teams — so devoted, we might say, that the team itself, along with
its associated
symbols, are considered sacred.
We can think of plenty of other contemporary examples: one’s relationship with
one’s child or life partner
may be sacred, some artists view art itself — or at least the creation of it — as
sacred, and
environmentalists often champion the sacrality of the natural world.
The sacred is a necessary feature of social life because it is what enables
individuals to bond with one
another. Through devotion to a particular sacred form, we become tied to one
another in a deep and
meaningful way.
This is not to say that (___) sacred is always (___)good thing. We find the sacred
among hate groups,
terrorist factions and revanchist political movements. Nationalism in its many
guises always entails a
particular conception of the sacred, be it ethnic or civic.
But, at the same time, the sacred lies at the heart of all progressive movements.
Just think of the civil
rights, feminist and gay liberation movements, all of which sacralized the liberal
ideals of human rights
and moral equality. Social progress is impossible without a shared conception of
the sacred.
Durkheim’s profound insight was that despite the negative risks associated with the
sacred, humans
cannot live without it. He asserted that a lack of social solidarity within society
would not only lead
individuals to experience anomie and alienation, but might also encourage them to
engage in extremist
politics. Why? Because extremist politics would satiate their desperate desire to
belong. Thus, we can
sum up the great dilemma of liberal modernity in the following way: how do we
construct a shared
conception of the sacred that will bind us together for the common good, without
falling prey to the
potential for violence and exclusion inherent to the sacred itself?
This question which preoccupied Durkheim throughout his entire life — remains as
urgent today as ever
before.
6. According to paragraph one, we can understand that
(a) Sacred is a sub set of religion.
(b) Religion is a sub set of sacred.
(c) Sacred and religion are two sides of a coin.
(d) Sacred is synonymous to religion.

. Page 4 of 40
7. Which of the following does not find support in the above passage?
(a) For Durkheim, religion is endemic to social life, because it is a necessary
feature of all moral
communities.
(b) The sacred is a necessary feature of social life because it is what enables
individuals to bond with
one another.
(c) Nationalism in its truest form always entails a particular conception of the
sacred, be it economic or
political.
(d) Social progress is impossible without a shared conception of the sacred.
8. Which of the following is the profound insight as mentioned in the passage?
(a) Despite the negative risks associated with the sacred, humans cannot live
without it.
(b) How do we construct a shared conception of the sacred that will bind us
together for the common
good, without falling prey to the potential for violence and exclusion inherent to
the sacred itself?
(c) Through devotion to a particular sacred form, we become tied to one another in
a deep and
meaningful way.
(d) None of the above.
9. The author considers Durkheim’s views as
(a) Condescending (b) Idealistic (c) Revering (d) Denigrative
10. This is not to say that ____sacred is always _____good thing. Fill in the
blanks with the correct articles.
(a) the; a (b) the; the (c) a, a (d) a, the
Passage (Q.11-Q.15): As a nation are we spoilsports? Do we, unintentionally, spoil
all our other sports
and games because of our monomania for cricket?
Sports commentators and others are once again raising this often-asked question in
the context of India’s
showing in the Tokyo Olympics.
Neeraj Chopra’s magnificent javelin throw which speared a Gold, and the Bronze
scored by the men’s
hockey team were both hailed as ‘historic’, which they indeed are: Neeraj is first
Indian to win a medal in
track and field, and the hockey victory came after 41 years.
Nations which in terms of both size and population are tiny in comparison with
India’s vastness, like
Jamaica, Cuba, and microscopic San Marino which has only 33,000 citizens, routinely
outshine India in
the Games.
Why do we, with the advantage of being able to draw on a potential pool of 1.4
billion people, fare so
poorly in the international arena in most forms of sport? And the answer to that
appears to be that all our
energies are focused, with laser-like intensity, solely on cricket.
The superstars in the firmament of cricket glitter with perhaps even greater
glamour than the stellar idols
in the galaxy of Bollywood, that other great Indian passion, and the astronomical
sums that our cricketers
command exceed those of our top-ranking movie icons.
With its alluring aura of mega fame and fortune, cricket attracts a hugely
disproportionate quantum of
talent and sponsorship than all other sports put together. It’s like the spreading
canopy of a giant banyan
tree which doesn’t let anything else grow in its all-encompassing shade.
And the irony of it all is that many, if not most, fans of cricket as a spectator
sport are, by and large,
clueless about the finer points of the game. Maiden over? Who’s she? An IPL
cheerleader? A bride being
sent from here to a Green Card-holding groom in the US? And a silly mid-off? Is
that a foolish stand-up
comic who tells off-colour jokes?
Don’t let cricket hog the limelight to the detriment of all other sports. Apart
from anything else, that’s just
not cricket …
11. Which of the following meanings corresponds with the word “monomania’?
(a) Single-minded in approach towards an objective or a purpose.
(b) Obsessive enthusiasm for all illnesses, especially when one feels to be
afflicted by them all.
(c) A mental condition especially when limited in expression to one idea or area of
thought.
(d) An obsession for collecting rare items or antiques.

. Page 5 of 40
12. Neeraj is first Indian to win a medal in track and field, and the hockey
victory came after 41 years. Select
the option that best illustrates grammatically the underline part of the sentence.
(a) the first Indian to win (b) first Indian in winning
(c) the first Indian in winning (d) the first Indian to winning
13. “And the answer to that appears to be that all our energies are focused, with
laser-like intensity, solely on
cricket.” By using the expression laser-like intensity, the author does which of
the following?
(a) The author uses the expression to draw the attention of the audience to the
plight of other sports.
(b) The author through the expression, wants to showcase his vocabulary knowledge.
(c) The author has superfluously used the expression with no particular though
behind.
(d) The author uses the expression to create an interest in the reader.
14. Don’t let cricket hog the limelight to the detriment of all other sports. Apart
from anything else, that’s just
not cricket … What does the author wants to convey through the expression “that’s
just not cricket…”?
(a) Cricket as a team sport is characterised by a team spirit where members of the
team facilitate each other
towards a shared objective without any one sports person trying to steal the show.
(b) Cricket as a team sport provides a social interaction from which emerges a
situational leader.
(c) Cricket encourages players to think collectively from a team’s perspective that
translates into crucial
lessons on life.
(d) Cricket as a sport has an inbuilt charm that organically draws people to the
sports and has a natural
capacity to hog the limelight.
15. Which figure of speech has been mentioned in the passage?
(a) Satire (b) Pun (c) Irony (d) Alliteration
Passage (Q.16-Q.20): Many people at times stop to consider questions such as: Who
am I, what is my
life’s purpose, and what will bring me fulfilment? Are we simply the body that we
come into this world
with, that ages with time and withers away after death? Are we the mind that grows
astute with
intellectual pursuits, only to evaporate at the time of death? Are we the mind that
grows astute with
intellectual pursuits, only to evaporate at the time of death? Or is there much
more to our ‘selves’?
Spiritual awakening is the moment you begin asking these questions. It marks the
start of your quest to
find answers.
One way people reflect on these questions is by sitting in silence in meditation.
This opens doors to help
us find answers. There is a source of wisdom within you that cannot be accessed
through an internet
search. It is awaiting deep within you. Instead of logging on to the computer, you
can log on to your own
inner essence, soul, through meditation. The answers are already within each and
every one of us,
waiting to be unlocked through the practice of meditation. This will lead you to a
source of wisdom that
will enrich your life.
What is unlimited wisdom? It is not the intellectual knowledge that we learn from
lectures or books; it is
consciousness. Our soul, being the same essence as God, is consciousness. It is a
‘state of knowing all
that is to be known’. When we access the divine wisdom, we reach a state of all-
consciousness, one in
which we know the answers to life’s mysteries and our purpose in life. We see life
as more than a string
of meaningless events and find a lesson and message in all that occurs. Those who
have tapped into
their soul look at life from a refreshing perspective. Rather than be tossed about
on the sea of life,
dashed by every wave, they watch their life like a movie with subtitles, in which
the words at the bottom of
the screen let them know what is going on at the spiritual level. Spirituality is
the process of discovering
our true self. We normally think that perception is possible only through our
bodily sense organs.
However, when we become spiritually aware, we recognise that we can perceive
through the spirit.
16. According to the passage, what helps in getting the answers to the questions
given in the passage?
(a) Sitting in the presence of a guru.
(b) Sitting in meditation.
(c) To go on a journey to find the answers.
(d) Researching on the internet to seek probable answers.

. Page 6 of 40
17. According to the passage, what is unlimited wisdom?
i) It is the intellectual knowledge that we learn from lectures or books.
ii) Our consciousness, which is the soul, the essence of being the same as God.
iii) A state of all-consciousness, one in which we know the answers to life’s
mysteries and our purpose in
life.
(a) Only I (b) Only ii (c) ii & iii but not I (d) All of the above.
18. “Those who have tapped into their soul look at life from a refreshing
perspective. Rather than be tossed
about on the sea of life, dashed by every wave, they watch their life like a movie
with subtitles, in which
the words at the bottom of the screen let them know what is going on at the
spiritual level.” What can be
inferred from the given sentences?
(a) By becoming aware of our consciousness, we become aware and play a more active
and meaningful
role in life rather than merely seeing life go by.
(b) Those who reach soul levels get the enjoy life with a broader perspective and
with multiple choices at
hand.
(c) Those who reach the inner self look at life differently and search for their
role assigned to them by
divinity.
(d) By becoming aware of the surroundings, we find all the answers that lie in the
events all around us.
19. What can be a suitable title to the above passage?
(a) Consciousness: an unlimited wisdom.
(b) Meditation and consciousness.
(c) Answers to the question.
(d) Life from a fresh perspective.
20. Which of the following is not the synonym of the word ‘astute’?
(a) Sagacious. (b) Ingenious. (c) Perspicacious. (d) Imbecile.
Passage (Q.21-Q.25): India’s men’s hockey team – which battled to a bronze medal
against Germany –
and its women’s hockey team – which fought Great Britain valiantly and missed out
on a bronze – have
entered the country’s sporting history.
But we got here after years of hard work and commitment.
To see this, appreciate the grit our teams showed. After the big defeat to
Australia, the first target of the
men’s team was to get into the quarterfinals. The team showed fabulous strength of
character. Heads
could’ve dropped after such a big loss. But they came back and won four matches in
a row – Spain,
Argentina, Japan, and Great Britain in the quarters. This determination came on the
back of steady
improvement after the tough period of 2008-12. You cannot pinpoint one particular
reason for the revival,
it has been the result of cumulatively improving on various fronts. Some of these
reasons are common to
both men’s and women’s teams.
The number one reason is the Hockey India League. A lot of good players came out of
the league.
Birendra Lakra, Manpreet Singh, Mandeep Singh, Amit Rohidas, Surender Kumar – all
these players
really made a name for themselves in the league.Having a strong national league was
of paramount
importance. As was the presence of foreign players. A legend and Olympic gold
medallist like Moritz
Furste was playing. Imagine a 17-18-year-old kid sharing the dressing room with
Furste, you learn a lot in
those two months.
Second, look at the men’s and women’s teams’ backup – the quality of coaches and
the quality of
scientific support staff who are working behind the scenes. Graham Reid, is
definitely one of the top-5
coaches in the world. With the women’s team, there is Sjoerd Marijne, Janneke
Schopman as coaches.
The quality of sports science staff – Robin Arkell for the men’s team, Wayne
Lombard with the women’s
team – is top drawer.
Fitness levels for both teams have been absolutely world class. Also, because of
scientific advisors, open
mindedness of coaches, the rolling substitution was used quite intelligently. We
were doing almost 80
rolling substitutions in a match at this Olympics. Unheard of for India! On an
average, per match, that is

. Page 7 of 40
more than one substitution per minute. That’s why we were able to maintain high
tempo and intensity
throughout the tournament. Hockey is nothing without fitness.
Third, players are now much better looked after in national camps. For that Hockey
India and Sports
Authority of India deserve a huge round of applause. From facilities we used to get
when I was playing
from 2000-2008 to now, the improvement is huge – whether it is in training camps or
while on tours.
21. Which are the three main reasons for the success of men’s and women’s hockey
team in Olympics?
i) A legend and Olympic gold medalist like Moritz Furste was playing with the
national hockey players in
Hockey India League.
ii) The Hockey India League was instrumental in giving India players of
international calibre.
iii) The quality of coaches and the quality of scientific support staff who are
working behind the scenes.
iv) After the big defeat to Australia, the men’s team showed grit and perseverance
to get into the
quarterfinals.
v) The training camps are better equipped with proper facilities, which has ensured
better looking after of
the national players.
(a) i), ii) & v) (b) ii), iii) & iv) (c) i), ii), & iii) (d) ii), iii) & v)
22. The tone of the passage is…
(a) Censuring and thought-provoking
(b) Reflecting and laudatory
(c) Contemplative and somber
(d) Praiseworthy and didactic
23. “That’s why we were able to maintain high tempo and intensity throughout the
tournament.” What was the
reason provided for the given statement?
i) Players are now much better looked after in national camps.
ii) The hockey team had an unprecedented high-spirit for the first time as the
fitness levels for both teams
have been absolutely world class
iii) The hockey team were doing 80 rolling substitution in a match at Olympics,
which ensured that the
players remained indefatigable.
(a) Only I (b) Only iii
(c) i & ii (d) ii & iii
24. All of the following statements are true, except
(a) Indian Hockey team won bronze against Germany in the Olympics.
(b) Birendra Lakra, Manpreet Singh, Mandeep Singh, Amit Rohidas, Surender Kumar –
all these players
really made a name for themselves in the league
(c) Graham Reid, is definitely one of the top-3 coaches in the world.
(d) After losing to Australia, the Indian men’s hockey team came back and won four
matches in a row –
Spain, Argentina, Japan, and Great Britain in the quarters.
25. “You cannot pinpoint one particular reason for the revival, it has been the
result of cumulatively
improving on various fronts.”. The word in bold belongs to which part of speech?
(a) Noun (b) Pronoun
(c) Verb (d) Adverb

. Page 8 of 40
Passage (Q.26-Q.30): Hockey is a physically taxing sport; you have to really look
after the players off the
field. Hockey India and Sports Authority of India (SAI) have invested huge amounts
of money to have
training camps 365 days a year.
Coming to the men’s team, Harendra Singh’s junior team from 2016, which won gold at
the Junior World
Cup in Lucknow, forms the core of the side at the Tokyo Olympics. That’s how it
should be. From 2014 to
2021 – almost seven years – most of these players have been playing together. So,
the team spirit, team
bonding, and camaraderie have been developed over many years. This isn’t a team
that has been built
overnight. Reid, the men’s coach, selected this squad based on current form and
fitness, not on
reputation. He was vindicated. Big names like Ramandeep Singh, SV Sunil and
Akashdeep Singh were
not picked. Between the three of them, they have more than 500 caps and 200 goals
for India. But,
Reid’s policy is that form and fitness come first. He selected players who can
adapt, players who are
extremely fit, players who can play in different positions. Our women players far
exceeded everyone’s
expectations. (____) have been one of the great stories of these Olympics. So many
unsung heroes in
the squad who rose to the occasion. Hockey India must capitalise on the new
popularity of hockey and
promote the game at every level – grassroots to national. I would love to see more
and more sponsors
come forward for hockey. I would love to see thousands of young boys and girls play
hockey all over the
country and not just the traditional hockey nurseries of Punjab, Odisha and Coorg.
Hockey must revive in
nurseries that have dried up – Mumbai, Khadki, Bhopal and Jhansi. I would like to
see hockey spread all
over the country, especially girls’ hockey. I think there is a huge scope to
promote girls’ hockey in the
country. And we need TV. Hockey must be shown regularly on TV. Experts should talk
about the game’s
finer points like it happens in cricket. I hope this doesn’t only happen during the
Olympics,
Commonwealth Games and Asian Games. Hockey must become a TV regular.
Now that the burden of the 41-year wait is gone, hopefully winning will become a
habit again.
26. What does the author mean when he says that one really has to look after the
hockey players off the field?
(a) The hockey players get demotivated fast; therefore, need to kept amused.
(b) Hockey is a physically tiring sports that requires a constant nurturing of the
players even when they are
not on the field.
(c) Hockey is one of the most sought-after sports; hence, the players cannot be
left on their own.
(d) Hockey, being a demanding sport, requires a constant attention towards its
players with regards to
fitness and overall well-being.
27. “He was vindicated.” Why does the author uses this phrase with respect to the
coach Reid?
(a) Coach Reid’s strategy of selecting the players based on form and fitness and
not on reputation paid off in
the Tokyo Olympics.
(b) Coach Reid was adamant in his stand of not selecting players based on
reputation solely got him in
trouble with the authorities.
(c) Coach Reid’s policy of form and fitness over fame as a selection criterion
earned him an international
reputation.
(d) The players of repute with many acclaims did not find a place in the hockey
team as a result of their lack
of form and fitness.
28. “So many unsung heroes in the squad who rose to the occasion.” What is the
inference drawn from the
given sentence?
(a) Hockey players who went unnoticed even after performing well in the Tokyo
Olympics.
(b) The women’s hockey team’s stellar players who weren’t noticed for hard work
made their mark in the
Tokyo Olympics.
(c) The women’s hockey team were criticised for their performance even after giving
a commendable
performance.
(d) None of the above.

. Page 9 of 40
29. (____) have been one of the great stories of these Olympics. Fill in the blank
with the correct pronoun.
(a) Their’s (b) Theirs (c) They’re (d) Them
30. Hockey must revive in nurseries that have dried up – Mumbai, Khadki, Bhopal and
Jhansi. The
underlined context is in reference to which of the following?
(a) Hockey has left these cities.
(b) The places no longer harbour good players.
(c) The places boasted of the best hockey players.
(d) Places that once nurtured young hockey talent

Directions (Q.66–Q.105): Read the comprehensions carefully and answer the questions
based on it.
Passage (Q.66-Q.71): The Supreme Court observed that its order to expedite trial in
criminal cases
against MPs and MLAs should not be understood as a mandate to give out-of-turn
hearing of their
criminal appeals. "High Court shouldn't treat our order as a mandate that appeals
of MPs, MLAs have to
be heard necessarily out of turn", the bench said. The observations were made after
Senior Advocate
Mukul Rohatgi, appearing for a convicted ex-Member of Parliament, told the court
that the hearing of his
client's appeals is being objected to by the complainant on the ground that Supreme
Court's order allows
only expedited trial and not appeals. His appeal has been taken up for hearing on
an expedited basis on
grounds that he is an ex-MP. Mr Rohatgi further said that the complainant filed a
special leave petition
before the top court saying that the appeal cannot be heard on an expedited basis
because the Supreme
Court's order allows only expedited trial and not appeals. The bench observed that
once an MP or MLA
stands convicted, they stand on the same footing as any other convict before High
Court. The Bench
then told Mr Rohatgi "There is no question of priority but also no question of
stopping the hearing of your
appeal. We don't prevent the High Court but we don't want to give priority to your
client by virtue of this
order." We will allow HC to hear the suspension application of Mr Rohatgi's client.
At the same time, we
don't want to give any priority to MPs MLAs in these cases", the bench said.
66. W and H were friends and also partners in a business. Although W was a bigger
stakeholder in that
business, however, there was no conflict regarding the same. Some altercations
arose between the two
and H threatened to divide the share in equal parts otherwise he will get M’s goons
to beat W to death. M
was an MLA in the state of Satter Pradesh. W paid no heed to his threat, as a
result one night, H with his
goons came and thrashed W to death. Consequently, M was arrested along with H and
his goons. In the
light of the above passage, determine the most appropriate option.
(a) H is liable for the death of W as he with M’s goons thrashed W to death.
(b) M is liable for the death of W along with H as H gave the reference of M to
threaten W.
(c) M has nothing to do with H’s goons.
(d) M was falsely implicated by H as an MLA gangster and having goons.
67. G was an MP in a particular area in a particular state. He was an asset for his
area because he had done
massive development works there and people were very satisfied with him. He was
falsely accused of
rape and was arrested and taken into judicial custody in very less time. However,
his appeal was
repeatedly dismissed by the High court. After High Court sustained his sentence of
imprisonment, he
appealed in the Supreme Court to get the remedy. The accused also urged that he was
an MP and his
people were very satisfied with his work and thus his application must be given
first priority to get bail so
that he can do developments in his area. In the light of the passage opt out the
most appropriate option.
(a) G’s plea must be given priority because he was basically an asset for his area
and had done massive
development for the people of that area.
(b) Even G’s plea has to wait for the time to come and can’t be given priority
because he had been
convicted by High Court and the convicted MP should be treated same as other
convicted persons.
(c) G is an accused and not convicted and thus his plea must be heard prior to that
of the others.
(d) G is an MP but cannot be given any special treatment; even he is an MP who is
not convicted yet.
68. An MP was accused of rape of minor and also smuggling of weapons. The matter
was in the High Court
and the same gave sentence of imprisonment and disposed off quickly giving this
case a little more
seriousness from its side. This was challenged in the Supreme Court. The SC wanted
HC to take note of
the case again but prevented HC from giving the application of MP a priority on the
basis that he is an
MP. In the light of the above passage, opt out the most appropriate option.
(a) SC’s actions were justified for doing the same as no MLA MP has to be treated
in a special manner.
(b) SC cannot prevent the MP’s application to be heard in the HC.
(c) SC cannot prevent MP’s application to be heard in HC as it was a heinous
offence done by the MP.
(d) SC has the right to prevent the HC from hearing the application.

. Page 18 of 40
69. B, an ex-MP was convicted of murdering G his brother. He spent 4 months in jail
and during the trial, he
was on bail throughout. He appealed at the Supreme Court against the conviction and
he same was
accepted by SC. Though the trial was delayed and same was contended by B’s
advocate. In the light of
the above passage, choose the correct option.
(a) B’s application has been allowed by SC and the trial should be expedited as B
was ex-MP.
(b) B’’s application has been allowed by the apex court and thus SC is bound to
give priority to B’s trial.
(c) Trial cannot be expedited even though appeal has been accepted.
(d) Trial should be expedited even though appeals have been accepted and expedited.
70. "High Court shouldn't treat our order as a mandate that appeals of MPs, MLAs
have to be heard
necessarily out of turn", the bench said. In the light of the above passage, choose
the interpretation which
is incorrect.
(a) SC can order for expedite trial for MPs ad MLAs on an exceptional basis on its
discretion.
(b) SC has no authority to mandate High Courts to expedite any plea of MP or MLA.
(c) HC do not have to take the orders as a mandate because the convicted should
stand at an equal
pedestal with other convicted, even if the person is an MP or MLA.
(d) HC can expedite the appeals of MP and MLA if necessary but not in the general
sense and the same
cannot be prevented by SC.
71. Construing to the above passage, opt out the main idea of the passage.
(a) Supreme Court has full discretion to order to expedite trials.
(b) Supreme Court’s orders must not be taken as mandate by the High Courts.
(c) High Courts have the discretion to allow any application of MLA and MP to be
granted priority.
(d) High Courts cannot expedite any trial on their own without SC’s orders.
Passage (Q.72-Q.76): Senior journalist Sashi Kumar, founder of Asianet and the
current Chairman of
Asian School of Journalism, has approached the Supreme Court challenging the
constitutional validity of
the offence of sedition under Section 124A of the Indian Penal Code. The applicant
argues that the
provision creates a 'chilling effect on free speech' as it suffers from the defects
of 'vagueness' and
'overbreadth'. The Supreme Court in Kedarnath case imposed a narrower scope of
interpretation, holding
only those matters that had the intention or tendency to incite public disorder or
violence or
disapprobation against the State as legally seditious. There is a need to revisit
the dictum in Kedarnath
case- which had upheld Section 124A after reading it down - in the light of the
latest developments in free
speech jurisprudence after the decisions in Shreya Singhal, KS Puttaswamy on right
to privacy, Navtej
Johar allowing same sex relationship, Joseph Shine decriminalizing adultery.
The applicant says that the recent spurt in cases under Section 124A IPC shows that
the provision was
being applied in a "politicized fashion". "The application of Section 124-A by the
state police machinery
consistently fails to acknowledge the narrow scope for restriction of freedom of
speech laid down in
Article 19(2) and reiterated in Kedarnath cases", the application says.
The law is not a measure simply aimed at the purported objective. It goes beyond
the aim it seeks to
achieve. Also, alternate legal provisions and laws in the Penal Code itself exist
including inherent
vagueness of the terms used in the law, the chilling effect of the law were not
correctly appreciated by the
court. The impact of the impugned law on rights, in particular the rights to
equality and freedom of speech
is perverse, the application says.
‘Section 124A being applied in a Politicised Fashion: Journalist Sashi Kumar moves
Supreme Court
against Sedition Offence’ (Live Law, 10 July 2021)

. Page 19 of 40
72. Vijay, a popular actor, was charged with sedition for comparing Indian stats
related to education,
healthcare and poverty with that of Scandinavian Country, and pointed out the
disappointing growth level
of India. This post received mixed responses but definitely initiated a
conversation across the internet on
the issue. During the trial, Vijay contended that the charges against him are in
violation of judgment of the
Supreme Court in Shreya Singhal, which upheld the freedom of speech and expression
but for the cases
in which exercise of such right causes provocation to violence or disruption in
public order. Decide what
shall be the order of the Court in the light of stated judgment and context of the
passage.
(a) Vijay shall be held liable for posting and circulating seditious material as
the content posted by him
directly assails the integrity of the nation and causing disapprobation against the
government.
(b) Vijay shall not be held liable for posting and circulating seditious material
as the post made by him is
an honest comparison of Indian growth rate with the most developed countries, and
the results per se
were disappointing that could not be blamed on him.
(c) Vijay shall be held liable for posting and circulating seditious material as
the post incited disruption in
the public order which is evident from the online debates and the responses from
the people.
(d) Vijay shall not be held liable for posting and circulating seditious material
as although results were
disappointing, the post was neither intended to nor caused any violence or
disruption in public order.
73. Assertion (A): Sedition is now an archaic law that has lost its significance in
India.
Reasoning (R): The reformative changes owed to the recent rulings of the Court have
widened the
horizon of freedom of speech and expression, and consolidated the fundamental
rights against the
framework adopted from the British reign.
Choose the correct answer in the light of the petition before the Court:
(a) Both A and R are true, R is the correct explanation of A
(b) Both A and R are true, R is not the correct explanation of A
(c) Both A is correct and R is incorrect
(d) A is incorrect and R is correct
74. Donald Grump, an opposition leader and an MP from Patna, was charged with
offence of sedition. The
charge was imposed for calling a protest against the Chocolate Amendment Act under
which government
was willing to import chocolates from neighbouring countries when it simultaneously
failed to create a
market for chocolates manufactured in India. The peaceful protest lasted for a
month before government
made an order for Grump’s arrest to supress the protest. The charges were pressed
on the ground that
the protest has a tendency to give rise to nationwide riots if any incitement is
caused by Grump. Which of
the following is the correct answer?
(a) Grump has been rightly charged as raising voices against the government
policies diffuses hatred
against the government which is per se seditious in nature.
(b) Grump has been rightly charged as a protest lasting this long has the tendency
to turned into a violent
affair, which is a threat to the public order as well.
(c) Grump has been wrongly charged as the protest was to highlight the shortcomings
of the government
policies, and it could not cause either violence or public disorder.
(d) Grump has been wrongly charged as the intention behind the protest was to
support the local
chocolate manufacturers and if intentions are bonafide then the result becomes
irrelevant.
75. After the demise of seven Indian military soldiers during a confrontation, a
nationwide bandh was called
for. The very next day an article, in a widely read newspaper, by Masaba criticized
the bandhs for
causing tremendous loss to the nation’s economy. Later that day, a crowd pelted
stones at Masaba’s
house and to show their anger burnt a few vehicles in her locality including her
own. Later, the
government charged her with sedition for disrespecting the soldiers and causing
violence due to her
speech. Following strictly the ruling of the Court in Kedarnath case, decide
whether Masaba can be
convicted of the offence or not.
(a) Masaba shall be convicted of sedition as making disrespectful remarks against
the military is
synonymous to disrespecting the state machinery which amounts to sedition.

. Page 20 of 40
(b) Masaba shall be acquitted as she just raised genuine concerns regarding the
impact of bandhs, and
the article was neither intended nor had the tendency to incite any violence or
cause public disorder.
(c) Masaba shall be convicted of sedition as her comments in the newspaper led to
mass violence
causing huge destruction of property.
(d) Masaba shall be acquitted as intention of causing disorder or violence is the
sole criteria to determine
whether an act is seditious or not. Since, she did not intend to cause any disorder
or violence she
cannot be guilty of the offence.
76. Ranjan, an activist from the state of Nagaland, has gathered people from the
entire state in Dimapur
district. He alleges the local administration of persecution the local tribal
community in the city. The crowd
has taken over the entire district and has held the administrative officers
hostage. They demand unless
the Prime Minister himself comes to address their questions they will not budge
from their stand. The
government soon took control of the situation with bilateral talks with the crowd.
However, as soon as the
protest cooled down government arrested Ranjan for sedition. The question before
the Trial Court is to
whether charge him for holding some officers hostage which is a trivial offence or
sedition as proposed by
the prosecution, which is one of the gravest offences in India. Decide:
(a) Ranjan shall be charged for sedition because taking over the administration of
a district causes
disruption to public order in the district which is seditious in nature.
(b) Ranjan shall be charged for holding hostages and not sedition as the crowd did
not cause any
violence or any disruption in the public order which would amount to sedition.
(c) Ranjan shall not be charged for any offence as the gathering and non-compliance
was planned to
attract the attention of the governments towards their issue for quick and
permanent redressal of their
issues.
(d) Ranjan shall be charged for sedition as raising voice against the government
machinery caused
disapprobation against the idea of state, which in turn assails the integrity of
the nation.
Passage (Q.77-Q.80): The Kerala HC has held that the display of the name of the
court by state law
officers and central government counsel in the name-boards of their motor vehicles
was contrary to the
Motor Vehicles Act.
The State Emblem of India (Prohibition of Improper Use) Act prohibits the improper
use of the State
Emblem of India for professional and commercial purposes and matters incidental
thereto.
The court emphasised that this Act restricts the use of the emblem on vehicles to
the authorities
mentioned under Schedule II. These authorities include constitutional authorities
and other dignitaries,
who are authorised to display the emblem on their cars.
Since the Act does not provide for the use of the emblem or star plate on any other
motor vehicle, the
court observed that such usage was legally impermissible.
The court observed that the display of the name of the state government in name-
boards was being
permitted on motor vehicles owned by co-operative societies and other societies and
even on vehicles
owned by NGOs.
Several motor vehicles of law officers and Central government counsels were also
permitted to display
the name of the court on the registration plates.
It was observed that motor vehicles likely to endanger the safety of other road
users were being
permitted to be used in a public place by the Registering Authorities in the state.
Therefore, by a
judgment dated 28th October 2019, the court issued directions to be followed by all
motor vehicles in
order to ensure the safety of road users.
77. Aarogya Fetu is a recently defunct co-operative society tied up in the winding-
up process with all the
people involved in its functioning currently trying to receive their share from the
estate. This co-operative
society had a decent estate worth of assets comprising delivery vans, some land,
and a couple of
investments. The co-operative society was registered in Noida and had the stickers
of the local registrar’s
office on their windscreens which had a small national emblem at the top. However,
while using these
vehicles for commuting to the courts involved in the winding-up process, they are
stopped by the local
policemen and are fined for using the emblem. Would they be held liable for using
the stickers?

. Page 21 of 40
(a) They would be liable as they are using the stickers without the authority to do
so.
(b) They won’t be liable since the stickers were still valid.
(c) They will be liable since the stickers were not valid anymore.
(d) They will be liable since the co-operative society is defunct now.
78. Had Aarogya Fetu been an unregistered NGO that was majorly funded by a
government employee,
would they be authorised to use the emblem stickers?
(a) Yes, as they are funded by an agent of the state government.
(b) No, as the government employee is not an agent of the government.
(c) Yes, as the government employee is an instrumentality of the state.
(d) No, as the government employee is not an instrumentality of the state.
79. Javed Fernandes is an IAS officer of the 1982 Uttar Pradesh cadre and had been
involved in the ministry
of defence for 40 years until recently due to retirement. As an officer of the
government, he had certain
perquisites accorded to him, including immunity, preferences and certain other
benefits. One such benefit
was that his vehicles were all earmarked as priority vehicles with the national
emblem and did not have to
stop at traffic lights. However, after retiring, he constantly followed civilian
laws but had not yet gotten his
benefits rescinded. Would he be liable for flouting the restrictions of the Act?
(a) Javed would be liable as he has wrongly been using his benefits post-
retirement.
(b) Javed would not be liable as he has not been using his benefits.
(c) Javed would be liable as regardless of the benefits being used or not; he still
has not revoked his
benefits.
(d) None of the above.
80. In the above case, had Javed been appointed as an amicus curiae in a case by
the Delhi HC in a high
profile case, and had he used his vehicle accorded with benefits to commute to the
HC on a priority
basis, would he be liable for action under the Act?
(a) Javed would be liable since he broke the law by using the emblems and the
benefits.
(b) Javed would not be liable for any action since he used his benefits in
furtherance of justice.
(c) Javed won’t be liable since the Delhi HC has accorded him with enough authority
to bypass civilian
laws in their service.
(d) Javed is liable to be taken action against since he used defunct benefits for
professional purposes.
Passage (Q.81-Q.86): The Family Court cannot deal with a criminal complaint about
offences under the
IPC, the SC remarked. As per S. 7 of the Family Courts Act, a family court can
exercise the jurisdictions
exercisable by any district court of any subordinate civil court under any law for
the following#1. A proceeding between the parties to a marriage for declaring the
marriage to be null and void or,
annulling the marriage or restitution of conjugal rights or judicial separation or
dissolution of marriage.
2. A proceeding for a declaration as to the validity of a marriage or as to the
matrimonial status of any
person.
3. A proceeding between the parties to a marriage with respect to the property to
the parties or of either
of them.
4. A proceeding for an order or injunction in circumstances arising out of a
marital relationship.
5. A proceeding for a declaration as to the legitimacy of any person.
6. A proceeding for maintenance.
7. A suit or proceeding in relation to the guardianship of the person or the
custody of, or access to, any
minor.
It can also exercise the jurisdiction exercisable by a Magistrate of the first
class under Chapter IX
(relating to an order for maintenance of wife, children and parents) of the CrPC.
[Extracted, with edits and revisions, from “Family Court Cannot Try Criminal Cases
Under IPC: Supreme
Court Corrects A 'Wrong Order' Passed In Transfer Petition” LiveLaw News Network,
LiveLaw, 12 August

. Page 22 of 40
81. Dhanya and Ambuj have been married for the past 30 years, and unfortunately for
them, it has not been
a happy marriage. Dhanya has been going through several mental health issues, due
to which she had
went in and out of several rehab centers. This has been severely affecting her
marriage with Ambuj and
her relationship with their ward, Samrath. With a very heavy heart on Ambuj’s
insistence, both finally
agreed to opt for judicial separation. Ambuj filed a case in the Magistrate’s Court
claiming mental agony
and cruelty, and thus seeking a judicial separation. Will their claim stand in
Court?
(a) Yes, as the claims of mental agony and cruelty fall under IPC, and thus should
be dealt with in the
Magistrate’s Court.
(b) No, as mental illness and insanity is a valid exception from criminal
liability.
(c) No, as the case of judicial separation is under the jurisdiction of a Family
Court.
(d) Both B & C are correct.
82. In the same factual scenario as presented above, Dhanya claims that her due to
emotional and mental
cruelty by Ambuj, she had to suffer from a variety of mental health issues and
ultimately go into a variety
of rehab centers. She seeks to claim compensation for the same. Provided that
cruelty is contained in
Section 498A of the IPC, can Dhanya seek the redressal of her issues in Family
Court?
(a) Yes, as ultimately the claims of cruelty are going to be employed to gain a
Judicial Separation.
(b) No, as the Family Court cannot deal with a criminal complaint about offences
under the IPC.
(c) Yes, as the dispute is between a married couple, and hence issues of private
nature should be dealt
with in the Family Court.
(d) No, as cases of cruelty cannot be dealt with in the Family Court.
83. If in the above-mentioned factual situation, a further dispute would have
arisen regarding the custody of
Samrath, their 19-year old child, which Court would they have gone to settle the
same?
(a) They don’t need to go to Court if they can settle the custody through a
contract.
(b) The parties should go to the Family Court.
(c) The Family Court will not have jurisdiction over this matter.
(d) The parties should go to Magistrate’s Court.
84. Abhinav and Trisha have decided to marry each other. Unlike the majority, they
decided to have a small
courthouse wedding and then hold a small event later. A few years later, they
realized that they had
misplaced the certificate of their marriage registration. When they went to the
place wherein it was
registered, the officer claimed that their marriage was not registered correctly,
and hence was not valid.
They want to contest this issue in Court. In which Court will the jurisdiction of
their issue lie?
(a) Their issue will lie in within the jurisdiction of family courts, since the
issue of registration of marriages
can only be dealt with family courts.
(b) Their issue will lie within the jurisdiction of criminal courts, as the officer
wrongfully executed their duty
and caused their marriage to be invalid.
(c) Their issue will lie in within the jurisdiction of civil courts, since the
dispute between the marriage
officer, and Abhinav and Tara is a civil dispute.
(d) Their issue will lie in within the jurisdiction of both criminal and family
courts.
85. Pushpak and Usha have been married for the past 10 years, and despite it being
a love marriage, had
constant bickering and fights. Recently, due to constant stress at work, Pushpak
has been constantly
frustrated and has been taking out his anger on Usha. This has been causing
tremendous stress and
mental trauma for Usha, and thus she takes the decision to file for a divorce in
the Family Court. But
since Pushpak did not want a divorce, the Court released an order stating that the
couple had to stay
together for the next 6 months as a last resort to try to resolve their
differences. Usha contested this order
stating that mental torture and verbal abuse is an offense and thus the Family
Court does not have the
jurisdiction to issue the same. Is her claim valid?
(a) Yes, as mental cruelty is an offense under the IPC, and thus under the
jurisdiction of the criminal
court.
(b) No, as the Family Court possesses the power to issue injunctions arising out of
a marital relationship.
(c) Yes, as Pushpak should be punished for the verbal abuse that he made Usha
endure.
(d) No, as the Family Court possesses the power to issue orders arising out of a
marital relationship.

. Page 23 of 40
86. In the above presented case, after Pushpak and Usha had been separated, a
petition of maintenance
was filed in the Family Court for Usha and her in-laws since Usha was unemployed
and Pushpak’s
parents wanted to live with Usha after the separation. Would their claim stand in
Court?
(a) Both Pushpak’s parents and Usha are entitled to compensation in the Family
Court.
(b) Only Usha is entitled to compensation in the Family Court.
(c) The Family Court has no jurisdiction over the matter.
(d) Both Pushpak’s parents and Usha are entitled to compensation in the Criminal
Court.
Passage (Q.87-Q.91): Union Minister Dr. Jitendra Singh, while in J&K announced that
each Rohingya
Muslim residing in the state will have to “go” as a direct consequence of the CAA-
NRC scheme brought in
by the government.
The matter reached the SC . In paragraph 12 of the Supreme Court’s order, the bench
has agreed with
the submission of the union that the principle of non-refoulement will not operate
in the instant matter as
India is not a signatory to the Refugee Convention of 1951 or the Refugee Protocol
of 1967. However,
the Principle of Non refoulement forms a part of Customary International Law,
thereby making it a
positive obligation of every state, irrespective of their status as a signatory to
the treaty in question.
The International Covenant on Civil and Political Rights which envisages the
principle of non-refoulement
in Articles 6 & 7 wherein it states that no state must effect removal of persons to
places where there is
substantial reason to believe that their life may be under threat (Article 6) or
they may be subjected to
cruel, inhuman and degrading treatment (Article 7).
Article 14 of the constitution of India enshrines the principle of equal treatment
before law. The Supreme
Court has previously held in a plethora of decisions such as the case of State of
Arunachal Pradesh v.
Khudiram Chakma that the protection available under Article 14 is available not
just to Indian citizens but
to all persons. The order of the Supreme Court does not evaluate the identity and
the individual
circumstances of each refugee and instates a blanket arbitrary ban on everyone who
is identified as an
“illegal immigrant” thereby violating Article 14.
Further, the Supreme Court held in the case of Maneka Gandhi v. UOI that due
process is a fundamental
right in itself. The Union Government in 2014 published a circular which contains
detailed procedure as to
how deportation is to be effected on a refugee.
87. The government of Afghanistan is overthrown in a coup. The coup also
destabilized the nation. Fearing
death and persecution, thousands of Ahmadiyya’s cross the border and enter the
territory of India to seek
refuge and protection. The government of India is annoyed by this and eventually
decides to stop taking
in the refugees and gives the local government a deadline of one week to deport all
the refugees. This
decision is challenged in court on grounds of violating the Maneka Gandhi
judgement.
(a) The courts would rule in favour of the government as they have the right to
protect the sovereignty of
the nation and refugees often pose a threat to internal security.
(b) The refugees can’t be deported, it would be a violation of their fundamental
rights. The Indian
constitution gives right to life to everyone, despite their nationality.
(c) The courts would rule in favour of the government as the local government was
given a period of one
week which is enough time ensure the adequate procedure is followed.
(d) The Maneka Gandhi judgement was clearly not followed here as due process is a
fundamental right
in itself and that was not dealt with properly, time frame for evicting refugees
goes contrary to the
judgement.

. Page 24 of 40
88. Was the Supreme Court verdict in the above mentioned passage against customary
international Law
and the principles of non-refoulment?
(a) Yes, customary international law are binding in nature despite a country not
being a signatory to
them.
(b) No, since India is not a signatory to the treat, it is not bound to follow the
above mentioned pact.
(c) India has its own law, principles an treaties. Since it is not a signatory to
the above mentioned treaty,
no international principle has been broken.
(d) Yes, Indian supreme court was at odds with the international law and the court
order violated the
principle of non-refoulment and customary laws.
89. Akash is a Nepali journalist, he has been very critical of the Indian
government and has called out the
misdoings of the ruling regime repeatedly. In the year 2021 he and his family were
on a vacation in the
state of Himachal Pradesh when the local government imposed charges of secession
and terror against
him and he was sent to prison, eventually his right to free trail was violated
multiple times. The Supreme
Court has taken cognizance of the matter, what would it rule?
(a) Akash’s nationality is irrelevant, the Indian constitution grants right to life
to each and every person
and not having a free trail is a violation of the same.
(b) Akash was not an Indian citizen , hence the judgment laid down in the Maneka
Gandhi case would
not be applicable on him as he does not share the same rights with Indian citizens
(c) The supreme court will allow him a free trial as Article 14 of the constitution
of India enshrines the
principle of equal treatment before law.
(d) The Charges of sedition and terror are very serious one. It is the
responsibility of the government to
ensure that strict measures are taken against him and he cannot be treated equally
as an Indian
citizen.
90. Uighurs are a religious minority in China, they have a differs lifestyle ,
culture and way of life. The
Chinese government sees them as a threat and wants to ensure that they give up on
their unique way of
life and indulge with the mainstream. To ensure this they use violent and cruel
methods. Several member
of the community are disenfranchised and escape to neighbouring nation of Vietnam.
China demands
them back from the government of Vietnam, would the government of Vietnam oblige?
(a) They don’t qualify as refugees as the government of china is just trying to
ensure greater integration
with the mainstream population and the practises are not criminal.
(b) As per the Articles 6 & 7 of the International Covenant on Civil and Political
Rights which envisages
the principle of non-refoulement, they would qualify as refugees and the government
of Vietnam
would have to accept them as refugees.
(c) The government of China is not a signatory to the International Covenant on
Civil and Political Rights
hence the Uighurs would not qualify as refugees and the government of Vietnam would
have to oblige
to china.
(d) Article 6 and 7 would ensure protection to the Uighurs and the government of
Vietnam would have to
abide by them.
91. Faiz is a Pakistani businessman, his story of suffering and success has
inspired millions across the
globe. He is seen as the champion of free market and entrepreneurship. Recently,
the Pakistani
government has confiscated his passport citing financial irregularities in his
transactions, fearing
prosecutions he escapes to India. Can be given the status of a refugee?
(a) Yes , he can be given the status of a refugee as his life is in danger and it
is the moral responsibility
of the government to look after those being persecuted.
(b) No, he cannot claim the status of a refugee as he is not facing persecution or
any related threats
based on ethnicity or any other characteristics.
(c) No, based on the international covenants on Civil and Political Rights which
envisages the principle of
non-refoulement in Articles 6 & 7, he cannot be called a refugee
(d) Yes, he can be given the status of a refugee as the government of India is
bound by the principle of
non-refoulment hence cannot turn him away.

. Page 25 of 40
Passage (Q.92-Q.100): It is a settled proposition of law that "forum convenience"
is not a fundamental
right similar to 'access to justice'. 'Access to Justice' is recognized as a
fundamental right. However,
'forum convenience' cannot be claimed as fundamental right and the right to choose
jurisdiction of the
court is again subject to the powers of the Chief Justice. Forum convenience cannot
become 'forum
shopping’. (Only) the Chief Justice, who can allocate the work to a particular
judge, issue a roster,
transfer a case from one judge to another judge, pass an order in respect of
transfer of a case from one
Bench to another Bench and by no stretch of imagination a puisne judge can transfer
a case from one
Bench to another Bench. The aforesaid rule makes it very clear that Constitution of
Benches and
allotment of judicial work/distribution of judicial work has to be done only and
only by Hon'ble The Chief
Justice.
(Extracted with requisite revisions and edits from “'Forum convenience' is not
fundamental right; only
Chief Justice can transfer case from one Bench to another: Karnataka High Court” at
92. Mr. Krishna Gopal (Respondent) was a 78 year old man. The dispute was brought
in the year 2021
before the Court of Karnadama and pertained to the issue of land acquisition of the
property which he
bought in the year 1992. Initially, the Court of Sersi had passed a decision
against Mr. Krishna Gopal and
when the same was at the stage of execution, Mr. Krishna Gopal decided to appeal
against the same.
However, since he now lived far away from the Appellate Court of Sersi, he appeared
before the
Appellate Court of Karnadama and quoted his health reasons to the Registrar General
who in good faith
decided that the case should be transferred to the Appellate Court of Karnadama. Is
his decision of
transfer correct?
(a) Yes, the principle of convenience of the parties has to be adhered to which
will require the case to be
transferred to the court of Karnadama.
(b) Yes, the Registrar General has gave the decision based on good faith and
considering the
surrounding circumstances and would have been the same it the Chief Justice gave
such a decision.
(c) No, the power to transfer a case from one judge to another lies with the Chief
Justice alone.
(d) No, this power of Chief Justice could have been exercised by the Registrar
General if the same had
been directed by the Chief Justice.
93. Mr. Krishna Gopal claimed that if his petition is not accepted then this would
affect his Right of Access to
justice. Is his argument correct?
(a) Yes, the principle of convenience of the parties has to be adhered to which
will require the case to be
transferred to the court of Karnadama.
(b) Yes, the old age of Mr. Krishna Gopal will otherwise hamper his fundamental
right of Right of access
to justice.
(c) No, Right of access to justice is not a fundamental right but merely has
prescriptive and directory
force.
(d) No, his right of access to justice has not been denied as he still has the
right to approach the court of
law although there is merely a bar on the forum of choice.
94. It was found that the property partially lied between the territorial
jurisdiction of both Court of Sersi and
Karnadama and when the suit was initially filed the Chief Justice had decided that
the same should be
governed under the territorial jurisdiction of Court of Sersi. Now, it was the plea
of Mr. KrishnaGopal that
even the Court of Karnadama could try the case as the property undisputedly falls
within the territorial
jurisdiction of this Court although partially. Is his argument correct?
(a) No, the territorial jurisdiction has already been decided by the Chief Justice
and hence, the Court of
Karnadama cannot try the case.
(b) No, the case was initiated in the Court of Sersi and the transfer would affect
and lead to not only the
wastage of administrative and judicial resources and the court of Sersi has become
more equipped
with the case over the years.
(c) Yes, the Court of Karnadama will have territorial jurisdiction as the property
was partially in its
jurisdiction.
(d) Yes, the Chief Justice decision will not prevail as his decision has led to the
ousting of jurisdiction of
one of the courts.

. Page 26 of 40
95. Meanwhile the case continued before the Appellate Court of Sersi where it was
found that one of the
judges Mr. Anna was a distant relative of the petitioner. The Registrar General
observed that bias might
arise because of this relation and in order for justice to not only happen but
should be seen to happen,
decided to transfer the case to another Bench. Is his decision of transfer correct?
(a) Yes, the decision to transfer the case was to avoid any apparent bias in the
course of the
proceedings and to render the decision free of any bias.
(b) Yes, the decision of the Registrar is based on sound reasoning which would have
aided in the
avoidance of any delay in adjudication.
(c) No, the proper course of action would have been that the Registrar should have
transferred the case
to the Chief Justice.
(d) No, the Chief Justice is solely empowered to transfer the case to another bench
and hence the
Registrar was incorrect in transferring the case due to lack of authority.
96. The Chief Justice gave directions to the Registrar to intimate the stakeholders
about the days when the
court sits and what are the matters that are going to be heard by every Bench of
the Court which the
Chief Justice prepared himself. The Puisne Judges stated that this delegation of
the power of the Chief
Justice is incorrect and illegal. Is the argument of the Puisne Judges correct?
(a) Yes, the Chief Justice is solely empowered to take decisions on the matters
relating to where the
Court will sit and to allot cases to the Bench.
(b) Yes, the Registrar is not empowered to take decisions and communicate them
under the name of the
Chief Justice.
(c) No, there is no delegation of power or authority. The Chief Justice has
exercised his permissible
powers and only directed for the intimation of the same to the stakeholders.
(d) No, the Puisne Judges are not empowered to protest against the decisions of the
Chief Justice as he
holds superior position when compared to other Judges.
97. One of the Bench (consisting of 3 judges) dissatisfied and knowing the
delegation to be illegal decided to
adjudicate matters which were present in the initial list of cases as against what
was intimated through
the Registrar. Is there any irregularity in their conduct?
(a) Yes, the Bench cannot go against the legally authorized decision of the Chief
Justice.
(b) No, the Chief Justice had delegated his power to the Registrar and which is
impermissible under the
laws and hence doesn’t hold any binding authority.
(c) Yes, the Bench cannot unilaterally take any decision and if there is lack of
clarity the same can be
clarified but not proceeded against.
(d) No, the Bench had followed the initial list which was a legally and undisputed
list given by the Chief
Justice and will thus have higher effect than the latter directions.
98. Against the direction as mandated in the list by the Chief Justice, a
constitutional bench (5 judges)
decided to transfer a case before themselves due to their expertise in the matter
involved and the same
was also a high-profile case. Is their conduct correct?
(a) Yes, the Bench had an expertise in the adjudication of the subject matter
involved which would aid in
the effective and efficient disposal of the case.
(b) Yes, the Bench consisted of 5 judges who would bear higher power than that of
the Chief Justice and
the list came through a delegated authority.
(c) No, the Bench cannot go against the decision of the Chief Justice who had
already distributed the
work and the cases in the list.
(d) No, the Bench cannot unilaterally take any decision and if there is lack of
clarity the same can be
clarified but not proceeded against.

. Page 27 of 40
99. The Chief Justice being irked by the conduct of the judges decided to initiate
contempt proceedings
against them for non-adherence to the directions given by him. The judges against
whom the action was
taken claimed that the directions should have come from the Chief Justice himself
and not indirectly
through the Registrar. Is the action of Chief Justice correct?
(a) Yes, the judges have failed to adhere and comply with a legally binding
decision of the Chief Justice.
(b) No, the contempt proceeding were initiated out of ire and disobedience to the
authority of the Chief
Justice.
(c) Yes, the Chief Justice is empowered to take any action against those who
disobey his orders.
(d) No, the judges have not adhered to the directions of the Chief Justice due to
the controversial and
disputed status.
100. Ms. Jigna, was a fresh graduate from a Law university and also quite a public
welfare enthusiast, as she
like to call herself. Over the years, she had filed many plaints in the name of
public interest and the Court
has found almost all of them as frivolous and not carrying any substance. The Chief
Justice hence
decided to bar her from filing any fresh plaint for a period of 2 years in any
court. She appealed against
this decision as it affected her right of access to justice. Is her argument
correct?
(a) Yes, right of access to justice is a fundamental right which has been affected
as she has been denied
the right to file any suit or complaint.
(b) No, Ms. Jigna is infamous for her conduct of filing frivolous plaints and hence
such a bar is
necessary.
(c) Yes, just because the initial plaints and complaints turned out to be
frivolous, such a bar is not
proportionate to the conduct.
(d) No, the decision has been taken by the Chief Justice who would be empowered to
impose such bar
on filing of frivolous complaints which has led to the wastage of the court’s
resources, valuable time
and the bar is not indefinite.
Passage (Q.101-Q.105): When a High Court is working under the Code of Civil
Procedure (CPC), it
cannot take aid of Article 226 of the Constitution. No doubt, if there is a case
before the High Court under
writ jurisdiction (Article 226) for violation of fundamental rights, then it has
very wide and unfettered
powers to impose such exemplary/punitive costs. But in a suit governed by the CPC,
the High Court has
very limited scope to impose costs.
There are five provisions in the entire CPC that regulate costs that can be imposed
by courts.
1. Section 35: It states that the court shall have full power to determine by whom
and out of which
property and of what extent the costs be given. But this is not an unfettered power
impose costs;
Section 35 starts with "subject to such conditions and limitations as may be
prescribed," and these
conditions and limitations are provided under Order XX A.
2. Order XX A: It circumscribes the power of courts to impose costs. There are six
heads under which
costs may be imposed. These include expenses in giving notice; expenses of typing
or printing;
charges paid by parties in inspection of records; expenses for producing the
witness; and expenses
for obtaining the copies of decree/judgement in case of appeal or revision. Under
this Order, courts
may take into account all these expenses incurred by parties while accounting the
cost.
3. Section 35A: It empowers the courts to impose compensatory costs on a party
which has filed false
and vexatious claim or defence. The outer limit set for such costs is fixed at Rs.
3,000.
4. Section 35B: This section of CPC also empowers courts to impose cost on parties,
but it is meant
particularly for those cases where parties use delay tactics like unnecessarily
taking adjournment.
[Extracted with revisions from 'Did the Delhi High Court have jurisdiction under
CPC to impose costs of
Rs 20 lakh in Juhi Chawla’s suit?' by HariMudgil, published on bar and bench
101. Mango had a small estate and his Scorpio car was one of his most prized
assets. In his will, he left the
Scorpio to his daughter Frooti. This made Mango's son Maaza very angry and he
disputed the will, even
though it was very clear that the car was left to Frooti. After a long battle of
ten years, the case was
adjudicated in Frooti's favour. But, after all this time the car was left with
almost no value. Its value had
depreciated from 10 lakhs to hardly 50,0000. Can the High Court order Maaza to pay
the difference to
Frooti?

. Page 28 of 40
(a) No, because in a suit governed by the CPC, the High Court has very limited
scope to impose costs.
(b) No, because the power of the High Court to impose such costs is subject to
Section 35, 35A and 35B.
(c) Yes, because the High Court has very wide and unfettered powers to impose such
costs under writ
jurisdiction.
(d) Yes, the High Court may do so under Section 151 to ensure justice to Frooti.
102. Mazdoor was a poor labourer who worked for X. X also ran a drug smuggling
racket on the side. When
he got caught, X got Mazdoor arrested under false charges of smuggling to avoid any
punishment for
himself. 15 years passed and Mazdoor spent all this time in jail as an undertrial
because he didn't have
the money to furnish for his bail bond. When his trial was finally complete, the
judge was shocked to see
the principles of natural justice being violated so blatantly. He ordered the state
to pay him 5 lakhs as
compensation. Is the judge empowered to do so under CPC Jurisdiction?
(a) Yes, the High Court can exercise its inherent power to do justice under Section
151
(b) No, because the High Court can take aid of Article 226 of the Constitution
(c) Yes, because the High Court has very wide and unfettered powers to impose such
exemplary/punitive costs
(d) None of the above
103. Piku was involved in a civil dispute with Mithu. Since the Jabalpur High Court
was very far from her
residence, she had to take an Uber for every hearing. Her taxi expenses for one to
and fro journey from
her house to the Court, and back, amounted to Rs. 750. Till the judgement was
pronounced, she had
made 38 such trips. If the Court is ruling in Piku's favour, can it take this taxi
expense into account while
determining the costs to be paid by Mithu?
(a) Yes, because all such expenses incurred by parties can be taken into
consideration while accounting
the cost
(b) Yes, because Order XXA of CPC empowers the Court to impose costs for such
factors
(c) No, because taxi expenses fall outside the power of the courts to impose costs
under Order XXA of
CPC
(d) No, because additional factors expect those that pertain to the subject matter
of a suit cannot be
considered while imposing costs under CPC
104. Juhi Chawla is a famous Bollywood actress. She filed a suit before the Delhi
High Court seeking an
injunction or prohibition against the setting up of 5G wireless networks in the
country due to the health
hazards associated with the technology. The Court found her claim to be false and
vexatious and was
dismayed to find that the claim was not even well researched and based on half-
baked knowledge. The
Court imposed a compensatory cost of Rs. 20 lakhs on Juhi Chawla. Is it empowered
to do so under the
CPC?
(a) No, because Section 35A of CPC does not empower the court to impose
compensatory costs on a
party which has filed false and vexatious claim or defence
(b) No, because 20 lakhs go beyond the prescribed limit for false and vexatious
claims
(c) Yes, because Section 35A of CPC empowers the courts to impose compensatory
costs on a party
which has filed false and vexatious claim or defence
(d) Yes, because 20 lakhs do not go beyond the prescribed limit for false and
vexatious claims
105. A and B were involved in a property dispute. A was unable to find enough
evidence to prove his claim so
he decided to file a request for joining C, an unrelated party, to the dispute. A
felt that in the time taken by
the Court to dismiss the request for joinder, he would be able to make a stronger
claim. B contested A's
action to be a mere distraction and argued that it was a waste of the Court's time.
If B's contention is
found to be true, can the Court impose any costs upon A?
(a) Yes, because the Court can do so under Section 35B of CPC
(b) Yes, because the Court can do so under Section 35A of CPC
(c) Yes, because the Court can do so under Section 35 of CPC
(d) None of the above

. Page 29 of 40
SECTION - D: LOGICAL REASONING
106. Every religion is riven by caste. Like with religion, 64% Indians say that it
is “very important” to prevent
women from crossing caste boundaries in marriage, and 70% Indians affirm that “most
or all of their close
friends share their caste”. Again, the Survey brings to the fore the central
contradiction of a democratic
nation that is divided by compartmentalized hierarchies. Thus, it is vital to note
that amidst the gathering
clouds of majoritarianism, it is a minority of upper castes that holds the reins of
power, across religions,
and it is the lower castes among the religious minority that face the brunt of
majoritarian attacks.
Which among the following can be concluded from the passage?
(a) Most Indians will prevent women from crossing caste boundaries in marriage.
(b) Less than 30% of Indians have close friends outside their religion.
(c) Caste has adversely impacted the religion of Judaism.
(d) The minority at the top of the pyramid of caste system face frequent attacks.
107. Identify the correct course of action:
There is always traffic congestion at Bharat Talkies square.
A. The traffic police should examine the possibility of diverting traffic in a bid
to ease the congestion.
B. The traffic police should ask a few people from the locality to join them as
volunteers to help improve
traffic management.
(a) Only A should be the course of action
(b) Only B should be the course of action
(c) Neither should be the course of action.
(d) Both should be the course of action
Passage (Q.108-Q.114): That India will have to navigate difficult waters has become
evident in its
External Affairs Minister S. Jaishankar’s recent visit to Iran and Russia. Since
the Joe Biden
administration’s announcement of complete drawdown of troops from Afghanistan, most
of the Ministry of
External Affairs’ time and energy seem to be necessarily consumed in complex
diplomatic manoeuvres,
attempting to explain to its allies and partners in Afghanistan the importance of
the ‘legitimacy aspect’ of
who decides the country’s destiny. Anything ranging from a protracted civil war to
a throwback to the past
with armed conflict between militias is possible in Afghanistan now. Indian
consulates in Jalalabad and
Herat have been closed for some time, and the deteriorating security situation has
now forced India to
pull out around 50 diplomats and security personnel from its consulate in Kandahar.
The fate of its
consulate in Mazar-i-Sharif will also be dictated by realities on the ground. Ever
since the Taliban was
routed in late 2001, India has been dependent on the US security umbrella to secure
its interests in
Afghanistan. New Delhi’s local Afghan allies have also allowed it to expand its
strategic footprints in
Afghanistan. But things have taken a turn for the worse very fast; the Afghan
institutions are hard
pressed to mount a coherent resistance to the Taliban, which seem indifferent to
military fatigue because
they are able to gather the last atom of their physical energy.
108. Which of the following can be inferred by the author in this passage?
(a) Iran and Russia will be key players for India in the USA-less Afghanistan.
(b) The Mazar-i-Sharif consulate will be lost by India in the coming few days.
(c) India will have to be open to talk with the Taliban to maintain peace in
Afghanistan
(d) All of the above
109. Which of the following may be correct as per the passage?
(a) The situation in Afghanistan will only get worse after USA’s exit.
(b) The US withdrawal from Afghanistan will be highly unfavourable for India
(c) Both a and b
(d) Neither a nor b

. Page 30 of 40
110. Which of the following can be a suitable title for the passage?
(a) India at risk in Afghanistan
(b) The worsening conditions post-US-exit in Afghanistan
(c) India’s options in the post-US-exit Afghanistan
(d) The increasing strength of the Taliban
111. Which of the following statements, if true, can weaken the author’s arguments?
(a) The USA decides to not withdraw from Afghanistan and stay back for another 1
rolling year.
(b) The ISIS and Taliban come together to strengthen their foothold in Afghanistan.
(c) The Taliban decides to come into electoral politics with their version of
Sharia laws and gives up arms
(d) Both a and c
112. “The Afghan military is weaker than the Taliban.” This is a/an -
(a) conclusion made by the author
(b) explicit statement made by the author
(c) Inference that can be drawn
(d) Wrong statement
113. Which of the following can be true as per the passage?
(a) India has very less strategic footholds in Afghanistan
(b) It was very difficult before the 21st century for India to gain proper foothold
in Afghanistan
(c) India’s presence doesn’t find much support in Afghanistan
(d) None of the above
114. The line “Anything ranging from a protracted civil war to a throwback to the
past with armed conflict
between militias is possible in Afghanistan now” is a/an -
(a) Inference for the author’s arguments
(b) assumption made by the author
(c) counter-argument to the author’s
(d) None of the above
115. You are given two statements. Identify the nature of the relationship between
them:
A. Drinking water is usually scarce during summers.
B. At least hundred people have been rushed to local hospitals for treatment.
(a) A is the cause and B is the effect
(b) B is the cause and A is the effect.
(c) A and B are effects of same common cause.
(d) A and B are effects of independent causes.
Passage (Q.116-Q.120): The Gujarat High Court decision staying some provisions of
the state’s anti#conversion law, including the one that deems all interfaith
marriages as those solemnized for carrying out
a forceful religious conversion, is extremely welcome. While the decision by Chief
Justice of the High
Court Vikram Nath and Justice Biren Vaishnav is only an interim order, even as the
larger legal challenge
of the Gujarat Freedom of Religion (Amendment) Act, 2021 is still pending, the
ruling sends out a larger
message. It poses a question mark on the constitutionality of not just the Gujarat
law but also similar
legislation in other states, be it Uttar Pradesh, Madhya Pradesh, or Himachal
Pradesh.
The stated aim of recent anti-conversion laws brought in by BJP-ruled states is to
prohibit unlawful
religious conversions. But in doing so, the BJP seems to have hidden intentions.
The law’s vague and
excessively broad provisions give powers to the state to hold a police inquiry to
probe the motives behind
a woman’s change of religion for marriage and to potentially annul such a marriage.
The law also allows
the “aggrieved person, his parents, brother, sister, or any other person related by
blood, marriage or
adoption” to lodge an FIR on just an allegedly unlawful conversion. By shifting the
burden of proof on the

. Page 31 of 40
individual accused of forcibly converting another person, in effect, the law
weaponized communal
prejudice and paranoia and threw the power of the state behind it. Even as these
laws are under
challenge before several high courts, they have become a template for harassment of
citizens. The
interim ruling comes, therefore, as much-needed relief.
The Gujarat HC intervention recognizes the unacceptability of law’s intrusion into
an individual’s private
life even when the state may have a legitimate interest in containing unlawful,
forcible conversions. It
draws a line and says that the state cannot enter the doors of a marriage where
there is no evidence of it
being forced or involving violence. However, even with the progressive intervention
of the judiciary,
decisions are often excruciatingly slow to percolate to the ground. Take the case
of the police across the
country continuing to file cases under Section 66A of the Information Technology
Act despite the apex
court striking it down in 2015. Laws that grant the state police powers to regulate
freedoms must be rolled
back. The Gujarat HC’s reading of the law will likely have a bearing on other
courts where similar laws
have been challenged.
[Extracted with edits and revisions from The Indian Express, dated August 21, 2021]
116. Which among the following is least likely to be inferred from the passage?
(a) The interim order has less value in the eyes of law than a proper judicial
order.
(b) High Courts don’t take cues from orders of other High Courts at all.
(c) The author believes that the Gujarat High Court’s interim order may have a wide
effect.
(d) Other High Courts may take cues from the interim order of the Gujarat HC.
117. Which one of the following, if true, strengthens the author’s arguments?
(a) The anti-conversion law would have legitimized mob interference in an
individual’s private life.
(b) A probe into the anti-conversion law by a high-level committee indicates that
this act has a
progressive nature.
(c) A similar interim order last year from a different High Court did not come as a
relief to the petitioners.
(d) None of the above
118. Which among the following can be inferred from the passage?
(a) All inter-religious marriages are aimed at religious conversion without
exception.
(b) Orders passed by the Supreme Court are not always abided by in India.
(c) Through this law, the government is trying to bridge the differences between
communities.
(d) The intervention of the High Court is not likely to have its desired effect.
119. Which among the following articles of the constitution is most likely to be
the premise of the Gujarat High
Court’s interim order?
(a) Article 14 which guarantees the right to equality on grounds of sex, case,
gender, creed and place of
birth.
(b) Article 26 which guarantees the right to freedom of religion and protection
against interference with
the internal affairs of a religion.
(c) Article 21 which guarantees individual’s right to choice of marriage and
protection against any
interference in private life.
(d) Article 368 which gives the state legislature the power to make laws on anti-
conversion and enforce
such a law.
120. Which among the following is most likely the main idea of the passage?
(a) Dubious legislation such as the ‘anti-conversion’ law in Gujarat may be allowed
to criminalize inter#religious marriages.
(b) Gujarat HC poses a question mark on the constitutionality of not just Gujarat's
‘anti-conversion’ law
but also legislation in other states.
(c) Legislation in other states on ‘anti-conversion’ would be repealed in lieu of
the Gujarat High Court’s
interim order.
(d) There is a difference between the theoretical sense of the anti-conversion law
in Gujarat and its
practical implementation.

. Page 32 of 40
Passage (Q.121-Q.125): Indian agriculture is heavily dependent on the monsoons but
the increasing
variability due to climate change puts millions of farmer families at risk of
losing their livelihoods. Coastal
fishermen are already seeing reduced fish catches and sea-level rise threatens to
inundate their very
homes. For the indigenous people, climate change is a double blow because
greenwashing concepts like
carbon offsets threaten further land alienation in the name of climate solutions.
Another economic group that will bear the brunt of climate impacts is manual
workers (often belonging to
marginalized castes), who work outdoors to earn a livelihood. Farm-labourers,
brick-makers, miners,
street vendors, rickshaw-pullers, construction workers and other informal sector
workers will be regularly
tested with deadlier and more frequent heat waves that will last longer. As
farming, fishing and forestry
become increasingly difficult due to climate-induced shocks, the pace of rural
migration to nearby urban
centers will increase in future. It is a travesty of climate justice that in their
efforts to escape one climate#stressed sector, they will most likely end up in a
sector that will be even worse.
Climate justice is not only about poor people and class struggles. One could argue
that the greatest
injustice in progress is against the young generation who will inherit hostile
earth due to the greed and
inaction of previous generations. If we read between the lines of the IPCC’s
(Intergovernmental Panel on
Climate Change) reports of how warming is already locked in and will continue, it
is safe to conclude that
the fight is more about surviving the future than about having a thriving future.
As more and more children
attain the age of reason, they are starting to see and realize the immensity of
this intergenerational
injustice. India has a huge task ahead to balance justified development needs with
indispensable climate
action. We hope that the stakeholders deliver.
[Extracted with edits and revisions from The Indian Express editorial by Sandip
Chowdhury, dated August
27, 2021.
121. As per the passage, which of the following is not likely to face the brunt of
climate change?
(a) Indian Agriculture that is dependent on monsoons
(b) Manual workers that are not dependent on climate change
(c) Fishermen who are in danger due to rising sea levels
(d) Rural workers who are likely to face frequent heat waves
122. Which of the following weakens the author’s idea of ‘hoping that the
stakeholders would deliver’?
(a) The Indian government feels that it is the developed nations’ job to think
about climate change since
they contribute towards it the most.
(b) The Indian government has and is in the process of taking adequate steps to
counter climate change.
(c) India is among the world’s top countries in the amount of consuming renewable
sources of energy
and this would prevent climate change.
(d) Climate change is not as prevalent and as real as it is often perceived to be,
it is exaggerated by the
developed countries.
123. Which of the following arguments has not been made by the author in the
passage?
(a) The future generations would not be able to survive the wrath of climate
change.
(b) Some sections of society will face the brunt of climate change in future.
(c) Climate justice as a concept is not concerned exclusively about poor people.
(d) All of the above
124. As per the passage, what is least likely to be referred to as a characteristic
of ‘climate-stressed’ sectors?
(a) A sector where the number of jobs has reduced due to climate change
(b) A sector where people are being fired due to reduced efficiency
(c) A sector where the efficiency of the people has gone down due to factors not
under their control
(d) A sector whose market value has gone down due to accelerated climate change

. Page 33 of 40
125. Consider the following statements and mark the statements that can be inferred
from the passage.
I. The intensity of the brunt of climate change would be same for all people.
II. Children of the present who realize the injustice done to them will punish the
present generation in
future.
III. The stakeholders would live up to the expectations of the author.
(a) Only II (b) Only III (c) Only I and II (d) None of the above
Passage (Q.126-Q.130): Children have been victims of the pandemic in a particular
sense. As cases
began to rise and schools were forced to shut down, students were left with no
choice but to accustom
themselves to the ‘new normal’ of online schooling. This magnified the challenges
to learning outcomes
that are often the result of entrenched socio-economic differences. Recent reports
show that these gaps
have manifested themselves not just in access to education and participation rates
as is often discussed,
but also in interest and quality of learning outcomes. Virtual classes have thrown
up significant
challenges for learners and teachers. Online schooling has often been reduced to
drudgery, robbing
young minds of the joy of participatory, collaborative education integral to the
experience of learning in
the physical classroom.
Teachers are not being able to assess whether learners have understood a concept
during online
sessions — a further drop in learning abilities would be a matter of concern in a
country where only 28.1
per cent of students of Class III were able to perform basic subtraction. The loss
is not limited to
knowledge gathering. Online education — the government has been an enthusiastic
endorser of the
medium — seems to extract a significant social price. According to a UNICEF report,
prolonged
confinement at home has deprived children of such skills as social bonding that is
made possible by real
time interactions with peers in school. The impact of these truncated experiences
synonymous with
growing up is likely to be quite severe on the mental health of children.
There is thus an urgent need to assess the impact of online education on learning
abilities. But the
government seems to be in no hurry. Ironically, the Centre has cited the pandemic
as an excuse to
postpone by a year the routine, three-yearly National Achievement Survey conducted
to assess the
learning standards of students of Classes III, V, VIII and X. Reliable data about
the limitations thrown up
by these surveys would have been of critical importance to address the gaps in
online education. But if
the government’s mindless push towards online and vocational education in the
National Education
Policy at the cost of the overall learning experience is any indication,
policymakers are increasingly
viewing education and learning from a narrow, utilitarian perspective. This
instrumental attitude could
prove to be disastrous to India’s pursuit of transforming itself into a knowledge
economy in the future.
[Excerpt from an article titled ‘Lost Interest’, The Telegraph, August 05, 2021
126. If author’s arguments in the passage above are true, which of the following is
most likely to be true?
(a) Online education may not be able to provide the same outcomes as classroom
education.
(b) Very soon online education will completely replace classroom education.
(c) Children have become more innovative in the online setup.
(d) None of the above
127. Which of the following can be inferred from the author’s arguments in the
passage above?
(a) Government has introduced its policy regarding online education after detailed
examination.
(b) Government should reassess its stance and policy push towards online education.
(c) National Education Policy is the most radical policy to reform educational
sector in the Indian history.
(d) All of the above.
128. Which of the following strengthens the author’s arguments?
(a) A report showed that more than 60% of primary class students have become
socially awkward due to
the online education setup.
(b) A report should that more than 40% of high school students have suffered some
mental health related
issues due to online education setup.
(c) A report should that more than 50% of high school students have become more
innovative due to
online education setup.
(d) Both a) and b)

. Page 34 of 40
129. Which of the following may be correct according to author’s views?
(a) Deprivation of social bonding due to prolonged confinement at home could result
in development of
mental health related issues in children.
(b) Learning the skill of social bonding is not very difficult in the online setup.
(c) Both a) and b)
(d) None of the above
130. Which of the following, if true, would seriously undermine the author’s
arguments?
(a) There are more disadvantages of online education as compared to benefits.
(b) A survey showed that only 25% class III students were able to perform basic
subtraction after change
of classroom education to online education.
(c) India can become knowledge economy in the future only by increasing access to
education which can
only be achieved through online education in a country like India.
(d) None of the above
Passage (Q.131-Q.135): Sustainable financing of infrastructure projects in India
has triggered much
debate over the last two decades. Over time and across governments, there are two
strands that have
endured. Getting private money with a long-term horizon is essential as governments
don’t have the
fiscal space to do it on their own. And, the quantum of private money that may come
in is influenced by
the extent to which infrastructure financing is insulated from political risks.
GoI has launched (in August 2021) the National Monetization Pipeline (NMP), which
aims to monetize
assets without transferring ownership. The goal is to raise Rs 6 lakh crore over
four years. To put it in
context, it’s a little less than the Rs 6.45 lakh crore PSUs contributed towards
capital expenditure in GoI’s
budget for 2020-21. Perhaps, to mitigate political risks, NMP classifies land and
buildings as non-core
assets. That leaves mainly roads, railways, airports and pipelines that are already
built as the core assets
that will be monetized. NMP resource mobilization rests mainly on roads (27%) and
railways (25%) to
mop up resources.
The key to success lies in the way the deals are structured. Basic models here are
not new. The main
model envisages a contract between GoI and a private entity to operate an asset for
a specified period.
Among the better-known examples are the PPPs in 2006 of New Delhi and Mumbai
airports under an
operate, maintain and develop model. Another successful model is the one used in
2018 for national
highways called operate, maintain and transfer. The alternative model, which is
also in use, mimics a
mutual fund. Power Grid Corporation, for example, raised funds this year for
earmarked projects through
an Infrastructure Investment Trust IPO.
Going forward, the success of NMP depends on how it navigates two crucial issues.
As ownership is not
being transferred, GoI will retain oversight through the contract period. The
contract, however, needs to
be flexible enough to make it attractive for a private entity. Finding the balance
within a government
framework has been challenging in the past. Another challenge is creating a
regulatory framework to deal
with a monopoly, which is something that will happen in the railway projects under
NMP. Other than
these two main challenges, contracts for entirely new areas such as India’s iconic
JLN Stadium in New
Delhi will enter uncharted territory. NMP represents an alternative to outright
sale of assets. Its success
will depend on execution.
[Excerpt from an article titled ‘Roads, rail & smarts’, The Times of India, August
25, 2021
131. If author’s arguments in the passage above are true, which of the following is
most likely to be true?
(a) There are alternative ways to raise funds and NMP is only one of such ways.
(b) NMP is designed to run by selling the national assets.
(c) PPP will not be adopted for the first time under NMP.
(d) Both a) and c)

. Page 35 of 40
132. Which of the following conveys the main idea expressed in the passage?
(a) NMP resource mobilization rests mainly on non-core assets.
(b) Infrastructure Investment Trust IPO is a much safer way to monetize resources.
(c) There are various challenges involved in the successful implementation of NMP.
(d) None of the above
133. Which of the following can be inferred from the author’s arguments in the
passage above?
(a) NMP has been designed in a way to mitigate political risks.
(b) NMP resource mobilization rests mainly on core assets.
(c) Private entity will have ownership of the assets for the contract period.
(d) Presently the contract is not attractive for a private entity.
134. Which of the following, if true, would seriously undermine the author’s
argument?
(a) NMP already includes a plan to create a regulatory framework to deal with a
monopoly for the assets
going to be monetized.
(b) NMP is the best way to raise funds by monetizing assets.
(c) Earlier some state governments were quite successful in monetizing stadiums in
their state.
(d) All of the above
135. Which of the following conclusions can be reasonably drawn from the passage
above?
(a) Power Grid Corporation is a government owned cooperation.
(b) Government is adopting a completely new model under NMP.
(c) PPPs undertaken earlier were not quite successful.
(d) None of the above

mock 10
Directions (Q.1-Q.30): Read the passages carefully and answer the questions.
Passage (Q.1-Q.5): “…I think my desire to blog about this particular story stems
from the awe-inspiring
way in which Mr Nabokov presents the existential crisis that is humanity through
parallel views. The story
opens with an elderly couple attempting to visit their son in the sanitorium where
he resides due to a
severe derangement. “The story is narrated almost entirely from the perspective of
the elderly parents as
they make their way by train and bus through stormy weather to the facility where
their son is kept only to
be told that he is not well enough to visit. “There they waited again, and instead
of their boy, shuffling into
the room, as he usually did (his poor face sullen, confused, ill-shaven, and
blotched with acne), a nurse
they knew and did not care for appeared at last and brightly explained that he had
again attempted to
take his life. He was all right, she said, but a visit from his parents might
disturb him.
“And so, they begin the miserable return voyage home, the occurrence of lost power,
late bus, storming
weather, injured bird, familiar faces of the past, and misplaced keys all ill omens
of the day building like
an argument in their consciousness. Everywhere around them, the mother sees signs
and symbols as
she struggles to make sense of the world around her and what meanings her
observations (______). In
the middle of the story, the narrator pseudo-shifts, and we get a peek into the
son’s world. He is in an
advanced state of narcissistic paranoia in his world, struggling to make sense of
the world around him
and simultaneously sure that every minutia exists to judge and condemn him.
“It is between these two narrators that we see the existential crisis that all of
us face. Mr Nabokov
highlights the mundane through the extreme. The son represents the need each of us
has to make sense
of our surroundings. We all perceive the universe through our individual filters, a
form of mini-narcissism
that is inescapable for all of humanity. We will never be wholly objective in our
perceptions of the world
because it is invariably filtered through our eyes, ears and brain. We might look
at the dying bird or the
late bus and wonder if it is an ill omen or a sign, like the elderly mother in the
story, or we might pick up
on a totally separate set of observations and attribute entirely different meanings
to them.
1. What understanding can we develop of the word ‘sanitorium’ as mentioned in the
passage?
(a) A place for the treatment of chronic nervous or mental disorders.
(b) A place for the convalescing soldiers who were injured in the war.
(c) A place where specialists treat rare diseases.
(d) A place where troubled boys from dysfunctional families are kept to be
reformed.
2. According to the passage, what is the state of mind of the son?
(a) The son believes the world to be judging and conspiring against him.
(b) The son believes that the parents have condemned him to a sanitarian
(c) The son is in a fantasy land where he is judged for his actions.
(d) The son is disoriented and cannot make out the surroundings.
3. From the first paragraph of the passage, we can understand that
(a) The author is discussing a biography.
(b) The author is giving a brief account of a story.
(c) The author marvel’s Mr. Nabokov’s story-telling.
(d) The author summarises an incident.
4. Everywhere around them, the mother sees signs and symbols as she struggles to
make sense of the
world around her and what meanings her observations ______. Fill in the blank with
the correct form of
the verb.
(a) held (b) hold (c) holds (d) is holding

. Page 3 of 40
5. Through the story, the author is trying to do which of the following?
(a) The author highlights that the same surroundings have different interpretations
for different people.
(b) The author uses the story to emphasize the existential crises of perceiving the
surroundings
according to one’s own understanding.
(c) The author points out that the story is only an extreme reflection of human’s
trying to make sense of
their environment.
(d) All of the above.
Passage (Q.6-Q.10): Louise Mallard has heart trouble, so she must be informed
carefully about her
husband’s death. Her sister, Josephine, tells her the news. Louise’s husband’s
friend, Richards, learned
about a railroad disaster when he was in the newspaper office and saw Louise’s
husband, Brently, on the
list of those killed. Louise begins sobbing when Josephine tells her of Brently’s
death and goes upstairs
to be alone in her room. Louise sits down and looks out an open window. She sees
trees, smells
approaching rain, and hears a peddler yelling out what he’s selling. She hears
someone singing as well
as the sounds of sparrows, and there are fluffy white clouds in the sky. She is
young, with lines around
her eyes. Still crying, she gazes into the distance. She feels apprehensive and
tries to suppress the
building emotions within her but can’t. She begins repeating the word Free! to
herself over and over
again. Her heart beats quickly, and she feels very warm.
Louise knows she’ll cry again when she sees Brently’s corpse. His hands were
tender, and he always
looked at her lovingly. But then she imagines the years ahead, which belong only to
her now, and
spreads her arms out joyfully with anticipation. She will be free, on her own,
without anyone to oppress
her. She thinks that all women and men oppress one another even if they do it out
of kindness. Louise
knows that she often felt love for Brently but tells herself that none of that
matters anymore. She feels
ecstatic with her newfound sense of independence. Josephine comes to her door,
begging Louise to
come out, warning her that she’ll get sick if she doesn’t. Louise tells her to go
away. She fantasizes about
all the days and years ahead and hopes that she lives a long life. Then she opens
the door, and she and
Josephine start walking down the stairs, where Richards is waiting.
The front door unexpectedly opens, and Brently comes in. He hadn’t been in the
train accident or even
aware that one had happened. Josephine screams, and Richards unsuccessfully tries
to block Louise
from seeing him. Doctors arrive and pronounce that Louise died of a heart attack
brought on by
happiness.
6. According to paragraph one,
(a) Brently is the husband of Josephine.
(b) Richards was killed in a railroad disaster.
(c) Louise Mallard must break the news to her sister Josephine.
(d) Richard’s friend is Brently.
7. Which of the following is opposite in meaning to the word ‘oppress’?
(a) Persecute (b) Subjugate (c) Encourage (d) Presage.
8. Josephine screams, and Richards unsuccessfully tries to block Louise from seeing
him. The underlined
part of the sentence may require coherent grammatical structuring. Out of the given
options, choose the
option that is most coherent grammatically. If the underlined portion is correct,
choose ‘no improvement
needed’.
(a) tries unsuccessfully to block
(b) unsuccessfully tries in blocking
(c) tried unsuccessfully to block
(d) No improvement.

. Page 4 of 40
9. What is the primary purpose of the passage?
(a) To give a framework of a novel.
(b) To summarise a story.
(c) To give a highlight of a movie.
(d) To present an outline of an upcoming series.
10. What can be inferred from paragraph three?
(a) The ending in the story is a twist to the plot where Mallard dies of shock on
seeing her husband alive.
(b) The ending in the story is an expected outcome as Mallard dies of happiness on
seeing her husband
alive.
(c) The story ends in an unfathomable way with a weak plot.
(d) The story is an open-ended conclusion leaving the readers to interpret as they
wish.
Passage (Q.11-Q.15): India has emerged as the third-largest start-up ecosystem in
the world after the
US and China, and the pace of growth is not showing any signs of slowing down.
Over the last year, India has added three unicorns every month, taking the total
count to 51, ahead of the
UK (32) and Germany (18). A privately-held start-up company valued at over $1
billion is called a
unicorn.
The US tops the list with 396 unicorns, while China is at the second spot with 277,
showed data from
Hurun Research Institute. India's unicorns are currently worth $168 billion, more
than the GDP of
Telangana at current prices.
Apart from unicorns, however, the number of future unicorns called "gazelles" and
"cheetahs" in India is
growing at an exponential pace, said Anas Rahman Junaid, MD and chief researcher at
Hurun India.
"Gazelle" is a start-up founded after 2000 with the potential to go unicorn in two
years, while "cheetah"
may go unicorn in the next four years. Gazelles have an estimated valuation ranging
from $500 million to
$1 billion, and the valuation of cheetahs range from $200 million to $500 million.
"Preparing the Hurun India Future Unicorn List 2021 has been one of the most
daunting tasks, primarily
due to the positive hyperactivity in Indian start-up ecosystems," said Junaid. "For
instance, five start-ups
that we had as cheetahs at the beginning of our research jumped straight to unicorn
valuations."
Online retail store Zilingo is the most valuable. Gazelle and digital furniture
platform Pepperfry is the most
valuable Cheetah. Mobile Premier League (MPL), Rebel Foods, Cure.fit, Spinny and
Mamaearth are
among a long list of future unicorns that are worth $36 billion, equivalent to one-
third of Delhi's GDP at
current prices.
The top investors in the gazelles and cheetahs are Sequoia, followed by Tiger
Global with 37 and 18
investments, respectively. E-commerce, fintech and SaaS comprise 49% of the future
unicorn list.
Entrepreneurs who feature in Hurun India Rich List have invested in the gazelles
and cheetahs, showed
the data. CRED founder Kunal Shah tops the list with nine start-up investments,
followed by Binny
Bansal with five and Ratan Tata with four. Infosys co-founders Nandan Nilekani (3),
NR Narayana Murthy
(2) and Kris Gopalakrishnan (1) have also invested in start-ups in the list.
With 31 start-ups, Bengaluru is the start-up capital of India, followed by Delhi
NCR (18) and Mumbai (13).
Bengaluru is the home to 11 gazelles and 20 cheetahs, with a cumulative valuation
of $12.4 billion. The
top three cities' headquarters are 68% of these companies.
11. Which of the following is NOT supported in the passage?
(a) India's unicorns are currently more than the GDP of Telangana.
(b) Zilingo is a digital furniture platform
(c) India has a total unicorn of 51.
(d) Unicorns, gazelle and Cheetah are start-ups.

. Page 5 of 40
12. What can be inferred from the start-up ecosystem?
(a) Giant investors form a start-up ecosystem for people with brilliant ideas but a
lack of wherewithal.
(b) A start-up ecosystem is the network of interactions among people, organisations
and their
environment; they can come in many types but are usually better known as start-up
ecosystems of
specific cities or online communities.
(c) A start-up ecosystem collaborates with world organisations to mutually benefit
from the indigenous
resources such as raw material to a skilled workforce.
(d) A start-up ecosystem is a special economic zone created by the governments to
set up small scale
industries.
13. "Preparing the Hurun India Future Unicorn List 2021 has been one of the most
daunting tasks, primarily
due to the positive hyperactivity in Indian start-up ecosystems," said Junaid.
Which of the following is the
part of speech for the highlighted word?
(a) Noun. (b) Verb. (c) Adjective. (d) Adverb.
14. Which of the following best describes the tone of the passage?
(a) Critical (b) Incendiary (c) Objective (d) Contemplative
15. What is the primary purpose of the passage?
(a) To address a malaise taking over the start-ups, which are draining the country
of its vitals.
(b) To bring to fore the initiatives taken by India concerning start-ups and
India's rank in the world as a
start-up ecosystem.
(c) To laud the efforts put in by young entrepreneurs, which has taken India to the
third position as a
start-up ecosystem globally.
(d) To highlight the participation of Binny Bansal, Ratan Tata, Nandan Nilekani, NR
Narayana Murthy
and Kris Gopalakrishnan as investors in various start-ups.
Passage (Q.16-Q.20): Sean (Simu Liu) and his Emma Stone-esque quirky best friend,
Katie
(Awkwafin(a), are two happy slackers. They are uninspired and quite content with
their minimum wage
(valet driver) jobs in San Francisco. The duo's routine banter and unambitious
lives take a dramatic turn
when Sean is compelled to confront his past. A spectacular hand-to-hand combat
sequence aboard a
bus gives away Sean's true identity — Shang Chi, the ultimate warrior.
Shang is summoned by his father, Xu Wenwu, a.k.a. the Mandarin (the legendary Tony
Leung), a
centuries-old Chinese warlord and the bearer of the powerful ten rings. The rings
make him invincible.
Can the righteous son take on his immortal father with questionable morals?
Director Destin Daniel Cretton's latest MCU entry is one of the best solo Marvel
films and most pleasing
to the eye in a long time. Beautiful action choreography dances to spiritually
captivating music by Joel P.
West. The exquisite visuals transport you to the serene landscapes of a mythical
land.
After a rather formulaic Black Widow and watchable MCU tv series, we finally have a
winner that reminds
you what big-screen spectacles are made of. Cretton pays resounding homage to the
exotic culture,
mythology and martial arts of the East while retaining the classic Marvel tropes —
action, situational
humour, bickering best friends and family. He also celebrates platonic love, which
is a rare feat.
The film rides high on emotions, and who better than Wong Kar-Wai's muse — the
iconic Tony Leung to
do the needful. The legendary actor and his soul-piercing eyes command attention,
empathy and anger
without resorting to speech. While Simu Liu's candour and fitness are apt for the
titular role, Tony is the
hero in his way. His charisma is hard to compress for a superhero movie template;
Cretton's brief to Tony
is clear — you do you. A decision that works like magic. He locking eyes with his
screen love interest
Jiang Li (Fala Chen) during their intimate fight sequence is the movie's best
scene. Tony lends a deeper
meaning to his character.
From a brilliant first half to a high on CG climax, Shang Chi feels a tad
predictable in the second half. But
it never fails to keep you on your toes. Before Marvel goes crazier with its
multiverse and erratic
timelines, here's a deeply humane father-son tale that can be watched by the non-
Marvel brigade as well.

. Page 6 of 40
Shang-chi has a certain spiritual calmness to it amidst the gorgeously
choreographed action sequences.
Never has a Marvel movie felt this unique in its execution and treatment. If you
are looking for a film to
end your big screen abstinence, this is the one.
16. The first two paragraphs are doing which of the following?
(a) They are presenting a summary of a film.
(b) They are serving as an introduction to the author's appraisal of the film.
(c) They are portraying various characters acting in the film.
(d) They are building suspense for the coming paragraphs.
17. "Beautiful action choreography dances to spiritually captivating music by Joel
P. West." What is the
author trying to communicate through the statement?
(a) The music takes one into the spiritual realms.
(b) The music makes people dance to its tunes.
(c) The soulful music and the dazzling choreography blend in perfect symmetry.
(d) The incorporeal music and a beautifully choreographed action is the highlight
of the movie.
18. We can conclude from the passage is that the movie...
(a) Is non-fiction; almost a documentary in theology.
(b) Is fictional, based on an imaginary world of superheroes.
(c) Is the first of the series of fictional marvel movies.
(d) Is the first of its kind in the fictional category.
19. "But it never fails to keep you on your toes." What is the meaning of the
idiom, 'to keep you on your toes?
(a) To keep you standing throughout the movie.
(b) To continue directing all your energies and attention to what you are doing.
(c) To cause you to remain alert and ready for anything that may happen.
(d) To keep you in a restive mood without giving you a breathing space.
20. What is the meaning of the word 'banter’ as mentioned in paragraph one?
(a) Serious conversations leading to an amicable understanding.
(b) Arguments bordering on a face-off
(c) Deriding remarks to fend off the company.
(d) Playful and friendly exchange of teasing remarks.
Passage (Q.21-Q.25): The Museum of Failed Products was itself a kind of accident,
albeit a happier one.
Its creator, a now-retired marketing man named Robert McMath, merely intended to
accumulate a
"reference library" of consumer products, not failures per se. And so, starting in
the 1960s, he began
purchasing and preserving a sample of every new item he could find. Soon, the
collection outgrew his
office in upstate New York, and he was forced to move into a converted granary to
accommodate it; later,
GfK bought him out, moving the whole lot to Michigan. What McMath hadn't taken into
account was the
three-word truth that was to prove the making of his career: "Most products fail."
According to some
estimates, the failure rate is as high as 90%. Simply by collecting new products
indiscriminately, McMath
had ensured that his hoard would come to consist overwhelmingly of unsuccessful
ones.
By far, the most striking thing about the museum, though, is that it should exist
as a viable, profit-making
business in the first place. You might have assumed that any consumer product
manufacturer worthy of
the name would have its such collection – a carefully stewarded resource to help it
avoid making errors
its rivals had already made. Yet the executives who arrive every week at Sherry's
door are evidence of
how rarely this happens. Product developers are so focused on their next hoped-for
success – so
unwilling to invest time or energy thinking about their industry's past failures –
that they only belatedly
realise how much they need to access GfK's collection. Most surprising of all is
that many of the
designers who have found their way to the museum have come there to examine – or
been surprised to

. Page 7 of 40
discover – products that their own companies had created, then abandoned. They were
apparently so
averse to dwelling on the unpleasant business of failure that they had neglected
even to keep samples of
their own disasters.
21. All of the following can be inferred from the passage, except
(a) The museum of failed goods was a chance.
(b) Robert McMath started purchasing and collecting a sample of every new item he
could find of
consumer products to make a reference library.
(c) The collection of the new products by Robert McMath was a deliberate sorting of
consumable
products.
(d) Robert McMath's museum became a source study for the designers who wanted to
know the reason
behind the failure of the product.
22. The synonym for the word 'belatedly' as mentioned in the passage, would be:
(a) Tardily (b) Perfunctorily (c) Timely (d) Amicably
23. Which literary device (figure of speech) does the passage reflect?
(a) Humour (b) Allusion (c) Irony (d) Imagery
24. The most suitable title of the passage would be:
(a) A happy accident.
(b) The failed museum.
(c) Reference library of consumer products.
(d) The museum of failed products.
25. Which of the following represents the central idea of the passage?
(a) A brilliant idea of presenting the world of failed goods under a single banner
for the companies to
learn lessons and avoid making the same mistakes by studying their own and
opponents' failed
attempts at a consumable product.
(b) A chanced profitable venture of a museum of failed products started to collect
every new product that
came in the market.
(c) A man beset by an obsession with hoarding consumable items as a random act of
consuming
consumable products to making the collectibles a profitable venture.
(d) An accidental venture starting as a reference library of every new consumable
items to a profitable
venture as a museum of failed products becomes precedence for the companies to
study their and
their rivals' failed goods to prevent history repeating itself.
Passage (Q.26-Q.30): Most of us, the Stoics point out, go through life under the
delusion that it is certain
people, situations or events that make us sad, anxious or angry. When you’re
irritated by a colleague at
the next desk who won’t stop talking, you naturally assume that the colleague is
the source of the
irritation; when you hear that a beloved relative is ill and feel pained for them,
it makes sense to think of
the illness as the source of the pain. Look closely at your experience, though, say
the Stoics, and you’ll
be forced to conclude that neither of these external events is “negative” in
itself. Indeed, nothing outside
your own mind can properly be described as negative or positive at all: what
actually causes suffering are
the beliefs you hold about those things. The colleague is not irritating per se,
but because of your belief
that getting your work finished without interruption is an important goal. Even a
relative’s illness is only
bad in view (____) your belief that it’s a good thing (____) your relatives not to
be ill. Millions of people,
after all, get ill every day; we have no beliefs whatsoever about most of them and
consequently don’t feel
distressed.
For positive thinkers, this would be an argument for trying to replace your
distress-causing beliefs with
upbeat ones. But when thinking about the future, Stoics such as Seneca often
counselled actively
dwelling on worst-case scenarios instead – staring them in the face. Not only does
ceaseless optimism

. Page 8 of 40
make for a greater shock when things go wrong (and they will), imagining the worst
also brings its own
benefits. Psychologists have long agreed that one of the greatest enemies of human
happiness is
“hedonic adaptation” – the predictable and frustrating way in which any new source
of pleasure we
obtain, whether it’s as minor as a new electronic gadget or as major as a marriage,
swiftly gets relegated
to the backdrop of our lives: we grow accustomed to it, and it ceases to deliver so
much joy. It follows,
then, that regularly reminding yourself that you might lose any of the things you
currently enjoy can
reverse the adaptation effect. Thinking about the possibility of losing something
you value shifts it from
the backdrop of your life back to centre stage, where it can deliver pleasure once
more.
26. According to the passage, the delusion has been mentioned with reference to all
of the following, except
(a) Situations as a source of pain.
(b) A loquacious colleague as a source of irritation
(c) An ailing relative as a source of disconcertion.
(d) Beliefs attached to the stimuli as a source of misery.
27. According to the passage, what is a “hedonic adaptation”?
(a) Something or Someone who ceases to be a source of happiness when taken for
granted.
(b) Something or Someone being a constant source of joy even in absentia.
(c) Gadgets that bring back the brain from the state of inertia.
(d) pleasure derived from activities that one has become accustomed to.
28. All of the following are false, except
(a) Stoics are philosophers.
(b) According to the stoics, external events are “negative” in themselves.
(c) Seneca is stoic.
(d) Thinking about the possibility of losing something you value help you in coping
with pain.
29. Even a relative’s illness is only bad in view ____ your belief that it’s a good
thing ____ your relatives not
to be ill. Fill in the correct prepositions in the given blanks.
(a) to; of (b) of; for (c) at; for (d) off; of
30. We can infer from the passage that the author is a…
(a) Reporter (b) Journalist (c) Philosopher (d) Writer

Directions (Q.66 – Q.105): Read the comprehensions carefully and answer the
questions based on it.
Passage (Q.66-Q.68): The Karnataka High Court has held that an intermediary or its
directors/officers as
defined under the Information Technology Act (“IT Act”) would not be liable for any
action or inaction on
part of a vendor/seller making use of the facilities provided by the intermediary
in terms of a website or a
marketplace. On that basis, a single-judge Bench of Justice Suraj Govindaraj
quashed a complaint
against online market place, Snapdeal and two of its its directors, Kunal Bahl and
Rohit Kumar
Bansal."An intermediary as defined under Section 2(w) of the Information Technology
Act or its
directors/officers would not be liable for any action or inaction on part of a
vendor/seller making use of the
facilities provided by the intermediary in terms of a website or a market place,
except otherwise
provided" a single-judge Bench of Justice Suraj Govindaraj said. The only liability
of an intermediary
under Section 79(3)(b) of the IT Act is to take down third-party content upon
receipt of either a court order
or a notice by an appropriate government authority unless otherwise provided , the
Court added. Thus,
an intermediary would not be responsible and/or liable for sale of any item not
complying with the
requirements under the Drugs and Cosmetics Act, 1949 on its platform however the
seller cannot do
away with its liability, the Court made it clear.
66. CMS is an intermediary defined under the Information Technology Act. A store
wants to sell its products
through CMS website. The store registered as a seller in the CMS website. After
registering they started
to sell adulterated products through the website, thereby not complying with the
requirements under the
Drugs and Cosmetics Act, 1949 . Who will be liable for selling adulterated goods?
(a) CMS will be liable for allowing the store to sell adulterated goods;
(b) The store will be liable for selling adulterated goods;
(c) Both CMS and the store will be jointly liable for selling adulterated goods;
(d) Neither CMS nor the store will be liable.
67. By the way of an amendment under Information Technology Act, it is provided
that an intermediary
should not allow the vendor to sell the goods that are prohibited by any law in
India. The President of
India promulgates an ordinance namely ‘Prohibition of Pencils ordinance’. The
ordinance bans the
production, sale and consumption of pencils that are black and red in colour. Utraj
is a store that wants to
sell Red pencils, the store collaborates with an intermediary and starts selling
the pencils. Ryan notices
this and files a litigation in the court against both Utraj and the intermediary.
Will Ryan succed?
(a) Ryan will succeed partially, because only the Utraj will be made liable;
(b) Ryan will succeed and both Utraj and the intermediary will be made liable;
(c) Ryan will not succeed as against Utraj and not intermediary, it is clearly
provided an intermediary
would not be responsible and/or liable for sale of any item;
(d) Ryan will not succeed, because he can’t file a complaint against two parties in
a single case.
68. According to the Judgment of the Karnataka High Court, the IT Act and the
ordinance promulgated by the
President (along with the amendment – reference check Question 2) which of the
following is not true?
(a) Goods prohibited by law can’t be sold through intermediaries;
(b) Goods prohibited by law can be sold through intermediaries,unless otherwise
provided.
(c) Intermediaries will be liable take down third-party content if the court
orders;
(d) Intermediaries will not be liable for the acts of the seller, unless otherwise
provided.

. Page 17 of 40
Passage (Q.69-Q.73): A force majeure clause relieves one or both parties from
liability to perform
contract obligations when performance is prevented by an event or circumstance
beyond the parties’
control. Typical force majeure events may include fire, flood, civil unrest or
terrorist attack etc. Force
majeure is a term used to describe a "superior force" event. The purpose of a force
majeure clause is two#fold: it allocates risk and puts the parties on notice of
events that may suspend or excuse service.
The doctrine of frustration is present in S. 56 of the Indian Contract Act 1852. It
says that any act which
was to be performed after the contract is made becomes unlawful or impossible to
perform, and which the
promisor could not prevent, then such an act which becomes impossible or unlawful
will become void. The
Supreme Court pointed out that the doctrine of frustration could not be availed by
the defendant when the
non-performance of the contract was attributable to his own decision. The doctrine
of frustration comes
into play when a contract becomes impossible of performance, after it is made, on
account of
circumstances beyond the control of the parties or the change in circumstances
makes the performance of
the contract impossible. The Court can give relief on the ground of subsequent
impossibility if it finds that
the whole purpose or the basis of the contract has frustrated by the intrusion or
occurrence of an
unexpected event or change of circumstances which was not contemplated by the
parties at the date of
the contract.
Further, Compensation for loss through non-performance of act known to be
impossible or unlawful.—
Where one person has promised to do something which he knew, or, with reasonable
diligence, might
have known, and which the promisee did not know, to be impossible or unlawful, such
promisor must
make compensation to such promisee for any loss which such promisee sustains
through the non#performance of the promise.
69. According to Hindu Marriage act, insanity is a ground for divorce and to call a
marriage null and void if
they were not aware of the same. Based on this premise decide, when A and B enter
into a prenuptial
agreement to marry each other and before the time fixed for the marriage, A goes
mad, what would be
the status of the contract?
(a) The contract would be frustrated as they had no control over their sanity.
(b) The contract would be enforceable but they would have to file for divorce.
(c) The contract would be frustrated but A would owe B compensation for the same.
(d) The contract would be void as it became impossible to perform after it was
entered into.
70. A entered into a pre-nuptial contract to marry B. B was not aware of the fact
that A was already married
to C. According to the marriage act under which A and B were to be married it was
earlier allowed to
practice polygamy, however later by an amendment it became forbidden by the law to
which A is subject,
to practice polygamy. B sued A for non-performance of contract. Decide?
(a) The contract would be frustrated because B had no knowledge of the polygamy.
(b) The Contract would be frustrated as polygamy is illegal.
(c) The contract would be frustrated for the performance of it became unlawful;
(d) The contract of marriage with both B and C would be frustrated and A owes
compensation to both.
71. A contracts to take in cargo for B at a foreign port. A’s Government afterwards
declares war against the
country in which the port is situated. The contract becomes void when war is
declared. Decide.
(a) The contract is not frustrated as it became void after it was entered into.
(b) The contract was nullified due to force majeure and hence there is not
requirement of a compensation.
(c) The contract was nullified under Sec 56, beyond the knowledge or capacity of
the promise hence
compensation is owed.
(d) The contract will stand to order once the war is over.

. Page 18 of 40
72. There is a Force Majeure Clause in the Contract between Zenab and Siddhant, let
us suppose, that
Siddhant, who was to supply goods to Zenab on certain date and time, faced issues
in relation to
procurement of goods due to mill strike and also because of rise in prices of
goods. In this case, can
Siddhant claim the suspension of performance of Contract on the basis of the Force
Majeure Clause?
(a) Yes, Siddhant can, depending upon the way, the Force Majeure Clause is worded.
(b) Such situation cannot be covered under Force Majeure as it is just a case of
disappointed
expectations and hence Siddhant cannot invoke the clause. It is merely a case of
commercial
hardship.
(c) Siddhant can invoke the Force Majeure clause as the clause is too broad to
cover such situations.
(d) Siddhant can invoke the clause for he could have not anticipated that mills
would go on strike.
73. On 11 February 2020, Covid-19 was declared a pandemic by the WHO, wherein a 6
months complete
lockdown was imposed by the government. On 1st March 2020, Mr. X and Mr. J entered
into a contract for
renovating Mr. X’s house by the end of April 20th, 2020. However due to the earlier
imposed lockdown and
guidelines declared by the Central Government, Mr. J was unable to keep his
promise. Decide upon the
validity of contract and compensation payable.
(a) The contract would be declared frustrated and Mr. J did not have the knowledge
of the pandemic when
he agreed.
(b) The contract would not be frustrated, and he would owe Mr. X compensation
because the pandemic
was declared earlier and could be foreseen.
(c) The contract would be frustrated as there would be no requirements for
compensations also.
(d) The contract would not be frustrated and neither party can reinstate it as both
had the knowledge
about the pandemic way before the made the contract.
Passage (Q.74-Q.79): Seeking bail in the Delhi riots larger conspiracy case
involving charges under
Unlawful Activities Prevention Act (“UAPA”), student activist Umar Khalid told a
Delhi Court that the entire
chargesheet filed by Delhi Police in FIR 59/2020 is a fabrication and that the case
against him is based
on the video clips run by Republic TV and News 18 showing a truncated version of
his speech. Pais
alleged that News18 omitted a crucial statement made by Khalid regarding need for
unity and harmony
from the video telecasted by it. As regards, Republic TV, Pais read out a reply
letter from the channel in
which it stated that its clip was based on a tweet made by BJP member Amit Malviya.
"The footage was
not recorded by our cameraperson. It was tweeted by Mr. Amit Malviya..", the reply
by Republic TV said.
Calling it a "death of journalism" in India, Pais remarked, "Your material is a
YouTube video which is
copied from a tweet. The journalist did not even have the responsibility to go
there. It's not a journalistic
ethic. This is death of journalism."Pais further alleged that News 18 channel
deleted sentences from the
speech, thus changing its meaning and context. "It makes a world of a
difference...a message of unity
based on Gandhiji was given that day and that was termed as a terror," he remarked.
"I have been
framed by the press. Why did they leave other parts of speech? Pais continued that
in his speech, Khalid
was talking about democratic power. He did not call for violence/ violent methods.
74. P was a student union leader in JUN College of Old Dailhy. Due to certain fees
hike in the national
colleges all over the country, he led a protest against the central government’s
policy stating the same in
one of his lines as a ‘tyrannical government’. The news channel named Democratic TV
stated this aspect
as an ‘anti-national attitude’ and regarded P’s statement as a threat to national
security. P was arrested
after which he contended that the action of the police was an infringement to
freedom of speech and
expression. In the light of the passage, determine the most appropriate aspect for
the same.
(a) Democratic TV has done a good job as a fourth pillar of democracy as the anti-
national elements
must be eliminated from the society.
(b) P cannot be arrested because he had not done any anti national sloganeering.
(c) P’s arrest is justified as the term Tyrannical Government can incite hate
against the government in
the minds of people of the country and thus Democratic TV has done the required.
(d) Democratic TV id not justified in its act because the term used by P is not an
anti national
sloganeering and the same has been misinterpreted by the channel. Thus arrest of P
is not justified.

. Page 19 of 40
75. B and S were TV reporters, reporting at the Mantar Jantar in New Dailhy in a
protest against some of the
policies of the Central Government regarding certain bill allegedly favoring
certain community explicitly.
The leader of the protest was and during the protest, construing to some issues, he
gave a reference of a
terrorist organization which was actually a matter of great concern for the
county’s national security. Both
the reporters highlighted the reference of terrorist organization given by the
leader by cutting that specific
part and telecasting repeatedly on TV. Later on that basis the leader was arrested.
Find the liability of the
leader in the light of the above passage.
(a) The leader was falsely accused of anti nationalism by the reporters and thus,
the reporters must get
arrested.
(b) Reporters were primarily doing their jobs and it was the part of their job to
give the reporting of the
protests at any place as it is the media which is the voice of the people which can
be taken to the
government.
(c) Leader should not get arrested because the media persons somehow misinterpreted
the concept due
to miscommunication and the same portion was explicitly cut and telecasted to
tarnish the image of
the leader.
(d) B and S should get the leader arrested because protest like this can bring
unrest and disharmony in
the society and incite anti national feelings in the minds of the people.
76. B was arrested by the police for inciting disharmony and hatred in the minds of
people against the
religion specific community and the Central Government. She said that “The fee that
is being hiked by the
Central Government in our institution along with other national institutions, most
of the students studying
in our institute are ‘minority religion’ and also ‘Dalits’ coming from EWS
(Extremely Backward Section).
And thus, the government is targeting those sections exclusively.” In the light of
the above passage, opt
out the most appropriate option.
(a) B’s arrest is actually justified because she had misinterpreted the policy of
the government and tried
to create the dissatisfaction among the people of the country.
(b) B should not get arrested because as per the fact, the institution actually
consisted of the dalits and
the minorities religious communities and was wrongly framed by the media.
(c) B’s arrest was justified because she had ignored the fact that the government
had raised the fees in
every national institute across the country and she tried to create dissatisfaction
against the
government by her speech.
(d) B’s arrest was justified because she had targeted a specific religion to gain
sympathy for her
demands and nothing else.
77. S was an actress in the country of Indiana, infamous for uploading her mere
antinational tweets,
supporting the terrorist organizations and pseudo feminists and she had many
criminal charges for the
same against her. Few months before, when Indiana had done surgical strike on the
terrorist
organizations in a country called Bakistan, she uploaded a video on Ytubes app and
tweeted the same
saying “Indiana subjugated Bak! Cheers and Bravo!!.” The previous antinational
videos and the present
tweets were taken up by the Secular TV news channel and interpreted “Indiana
subjugated! Cheers and
Bravo!!.” And the same also played the previous antinational videos repeatedly on
the television.
Construing to the same, there arose a demand to arrest S with an immediate effect
and the same was
done. In the light of the passage above, opt out the most appropriate option.
(a) Secular TV has basically handed over a culprit to the hands of law having
numbers of anti national
charges against her. Thus the act was justified by the same.
(b) Secular TV has misinterpreted the video of the actress and thus caused the
miscarriage of journalistic
ethics.
(c) S was liable to be arrested because she had infamously charged for uploading
antinational videos
and tweets.
(d) Secular media had interpreted the same in the wrong way landing S into the
police custody that is not
justified.

. Page 20 of 40
78. "The footage was not recorded by our cameraperson. It was tweeted by Mr. Amit
Malviya..", the reply by
Republic TV said. Construing to the mentioned lines what inference can be drawn?
(a) Republic TV tried to escape liberty from the controversy.
(b) Republic TV wanted to tarnish the image of Umar Khalid and accuse BJP worker
for the same.
(c) Republic TV is basically backed by BJP and any controversy hitting Republic TV
is taken by BJP on
itself.
(d) Republic TV had no journalism ethics from the beginning.
79. When advocate Pais declared the event as ‘Death of journalism’, what did he
mean by the same?
(a) Republic TV being backed by BJP Workers.
(b) Republic TV taking the video from YouTube, instead of going there and checking
veracity of the
statement.
(c) Republic TV’s framing Umar Khalid as anti national.
(d) Republic TV’s act of cutting certain portion of speech to get him arrested.
Passage (Q.80-Q.85): Delhi has granted interim-protection from arrest to Pinki
Chaudhary, who is one of
the main accused in the Jantar Mantar anti-muslim sloganeering incident at New
Delhi. Additional
Sessions judge Ashutosh Kumar stayed the arrest of Chaudhary till Monday and asked
him to join the
investigation whenever required and not to influence witnesses or tamper with
evidence. Chaudhary has
been charged for offences under Sections 188 (disobedience to orders promulgated by
public servant)
269 (negligent act likely to spread infection of disease dangerous to life)
270(malignant act likely to
spread infection of disease dangerous to life 153A (promoting enmity between
religious groups), 120B
(Criminal Conspiracy) Of the Indian Penal Code. During the course of hearing, the
SHO agreed to supply
the video footage of the concerned incident which shows the presence of Chaudhary
and the words
uttered by him. The SHO undertook to place on record the transcript of the same by
the next date of
hearing. The counsel for accused submitted that he was not named in the FIR and he
did not raise and
communal slogans or uttered any words to hurt the religious sentiments of another
community. While
granting protection to Chaudhary, the court also took note of the fact that the
‘organizer’ of the event,
Ashwini Upadhyay has already been granted bail. The accused allegedly took part of
a rally that took on
Sunday in Delhi under the Bharat Jodo Movement against colonial-era laws in the
country here anti#Muslim slogans were said to be raised. Upadhyay along with 5
others were later arrested. Upadhyay was
later granted bail while bail plea of three other accused Preet Singh,Deepak Singh,
Vinod Sharma had
been rejected.
Source Name - Bar and Bench, Author name - Aareeb Uddin Ahmed
80. A was a leader in PJP political party who was there to protest against some
reservation schemes initiated
by the government. P, who was a person of a certain community ‘Islm’, was a police
inspector of that
area tried to control the crowd as it was getting out of hand and ordered to vacate
the area. However,
while P was controlling the crowd, A started arguing that the protest was his
fundamental right and further
started abusing P and started calling him a terrorist group leader, following to
which the group started
calling him the same. A and his fellow beings were arrested immediately. In the
light of the above
passage, opt out the most appropriate option.
(a) A was somewhere justified in arguing because it was his fundamental right and
when police officer
did not adhere to that, he started abusing him as to protect his fundamental right.
(b) A was not justified because abusing any police officer is not a justified act.
(c) However, abusing the police officer was not at all justified, A was justified
somewhere in calling him
terrorist leader because people from P’s community are infamous for being terrorist
group leaders.
(d) A was not justified in his act because he was obstructing a police officer to
discharge his duty,
thereby leading to disobeying orders promulgated by public servant.

. Page 21 of 40
81. W was a religious group member of a certain community called ‘Isml’. He
propagated the message of
love and affection through his teaching among his community. He used to propagate
the teachings
through comparison with other religions in the world and their teachings. He used
to justify his religion’s
teaching by saying that one community called ‘Heend’ use to burn their women after
her husband’s death
even in the present time and say that they will be doing this with women of all the
other communities as
well in order to please their gods and goddesses. He was arrested for the same. Opt
out the most
appropriate option for the same in the light of the above passage.
(a) His arrest is actually justified because the burning women practice was
eradicated years back and
now it is not practiced at all.
(b) His arrest is not justified because the practice of burning wives has been an
age long practice and
thus he can take the defense of truth.
(c) W can’t be arrested because preaching through comparison with other religion
and bringing out their
evil aspects is not bad at all.
(d) He must be arrested because he is empowering his followers to develop hatred
against that religion
Heend.
82. Z was a political leader protesting against government extraordinary price hike
in petrol and diesel.
Government had given the orders that there would be no gathering in a particular
area and the corona
virus protocols must be maintained with social distancing. Z, along with his 10
party workers arrived at
Jantar Santar chowraha and started sloganeering against price. Gradually it was
seen that large amount
of people gathered there and one of them was an infected one from covid who later
infected a large
amount of people there. Z was later arrested for violating the protocol. Opt out
the most appropriate
option.
(a) Z is responsible for he was leading the protest in spite of government’s order
against same, thereby
leading to disobedience of orders.
(b) Z & infected person is responsible for disobedience of orders and doing
negligent act likely to spread
infection of disease.
(c) The infected person was infected himself and he participated in the crowd, and
thus he is responsible
for the same.
(d) The person is liable as he tried to infect the crowd at large by participating
in the protest.
83. P and Q were crossing each other while their shoulders clashed with each other
because of the crowd in
that area. Q belonging to a community called ’Ilam’ shouted on P for not behaving
properly. P, belonging
to community ‘Heend’ compared him with a terrorist leader and started walking away.
Meanwhile other
person named F, of P’s same community started shouting ‘anti Ilam’ stuffs against Q
which provoked the
crowd and in no time, a riot type situation arose in that area with mass vandalism.
In the light of the
above paragraph, opt out the most appropriate option.
(a) P was responsible for the provocation of the crowd and thus, must be arrested.
(b) P, Q and F were conspirators of this event and thus must get arrested.
(c) Q was solely responsible for the promoting enmity between the religions caused.
(d) F was somewhere responsible for promoting enmity between the religions.
84. A, B, C and D were brothers belonging to community called ‘Ilam’. After
offering prayer at their places of
worship, they saw a group of people of community ‘Heend ‘taking a yak to get it
sacrificed in their place of
worship, which an age long tradition and presently followed. C remarked that “this
community never knew
to treat animals well and always exploited them to please their Gods and Goddesses.
We should stay
away from these kinds of communities. We should be compassionate towards animals
and must not kill
them for the sake of sacrifices. However they must be sacrificed same as they are
sacrificing this poor
creature”, and A, B and D agreed upon that. This enraged the opposite group and in
no time, several
people of that community gathered and there was a riot like situation with severe
bloodshed. I the light of
the passage, opt out the correct option.
(a) C was right because sacrificing animals for the sake of gods and goddesses is
not justified.

. Page 22 of 40
(b) C was compassionate towards the poor creature but words like “We should stay
away from these
kinds of communities” and “However they must be sacrificed same as they are
sacrificing this poor
creature” was a provocative statement and thus, he must be held liable.
(c) C must not be held liable because sacrificing was an age old practice in Heend
Community and thus
he has spoken the truth.
(d) C must be held liable as he has spread hatred or enmity between two
communities.
85. Opt out the most appropriate option describing the essence of the paragraph.
(a) If the organizer of any wrong can be released on bail, then it is not possible
for their accomplices to
get the bail.
(b) Video footage in this type of case can be treated as evidence that can be
giving to the court to prove
the guilt of the accused.
(c) It is normal to criticize and give references of the evils of a particular
community to promote the
unification of one’s own religion.
(d) Protestors should be deemed as conspirators of certain crimes.
Passage (Q.86-Q.89): The Kerala HC has called for the Transport Commissioner to
take necessary
steps through the concerned officers in the Motor Vehicles Department to ensure
that no motor vehicle is
permitted to be used in any public place violating the Rules and directions in
place for road safety.
Directions Issued:
Lighting, Light Signalling and Retro-reflectors
Motor vehicles with more than three wheels, trailers and semi-trailers, excluding
agricultural tractor and
special purpose vehicles, shall not be permitted to install any lighting, light-
signalling devices or retro#reflectors.
Detectable and Legible Registration Marks
The registration marks of motor vehicles should be displayed on the licence plate.
These registration
marks should be displayed clearly and legibly.
No Obstructions to Clear Vision of Driver
No object should be placed or hung in front of the windscreen, causing obstruction
to the clear vision of
the driver, both to the front and to his right or left-hand side.
Usage of Official Emblem/Flags
No person, other than those authorised under the State Emblem of India (Regulation
of Use) Rules, shall
use the official state emblem. Only motor vehicles carrying constitutional
authorities/dignitaries are
permitted in public place displaying such official emblems.
Any person who drives or causes or allows to be driven, in any public place a motor
vehicle, which
violates these standards of road safety is liable to be punished for the first
offence with imprisonment up
to three months, or with a fine which may extend to ten thousand rupees or with
both. He shall be
disqualified for holding licence for three months.
[Extracted, with edits and revisions, from “Motor Vehicles: Kerala High Court
Issues Directions On Use Of
Flashy Headlights, Usage of Official Emblem, Flags, Name Boards Etc”, Hannah M
Varghese,LiveLaw, 4
86. Shri Ram Prasaad is a retired IAS officer of the 1963 Bihar Cadre, now living
peacefully with his family in
Uttarakhand. He owns minimal possessions with some land to his name along with a
government
pension in place. He still drives the car that he had bought in his youth with the
national emblem and the
flag on the dashboard. One fine day, while on an errand run to the nearby town, he
is stopped by the
stationed traffic officer on account of misusing the national emblem and flag, and
is fined 2000 rupees for
the same. Is Ram Prasaad liable to pay the fine?
(a) Yes he is liable as he was wrongly using the emblem and flag even after his
retirement.
(b) No, as the use of the symbols was not unlawful in any regard.
(c) Yes, as his retirement disburses him of the privilege to use them
(d) No, as the symbols were affixed when he was an active government signatory

. Page 23 of 40
87. In the above question, had Ram Prasaad been a retired army general granted gun
carrying rights post
his retirement, and had he been carrying the same on the dashboard of the vehicle
that he was driving,
would he have been liable to be stopped and fined/imprisoned or both?
(a) No, as he has been awarded such rights as to carry the gun even after his
retirement.
(b) Yes, as the benefits post retirement do not come under employment of the
government.
(c) Yes, as the act of carrying a gun so precariously amounts to endangerment of
life.
(d) He would not attract the liability since no object is placed caused obstruction
in vision.
88. The Anand Vihar bus terminal is a hustling and bustling bus terminal on the
Delhi-UP border with
thousands of buses traversing through its terminals. It is often stationed with
multiple guards, conductors
and traffic policemen on account of being a rush and accident-prone bus terminal.
On one such occasion,
due to the Diwali rush, the bus terminal was stuffed to the fullest but was
understaffed with traffic
policemen. A bus driven by Sewak Dev was departing from the terminal with tinted
glasses on either side,
which the stationed traffic policeman failed to notice. Unfortunately, the same bus
crashed into a nearby
under-construction flyover, amassing 50 seriously wounded individuals. Is this
oversight bound to be
punishable?
(a) No, as the front screen of the driver was clear of any tinted glass, which is
primarily required to drive.
(b) Yes, as the position of the tinted glass is irrelevant.
(c) No, as the policemen were already occupied to the fullest and no deliberate
oversight was intended.
(d) Yes, as the tinted glass positioned to the sides was unlawful.
89. Girdhar is a van driver employed by a school to ferry children from their
residence to the school and vice
versa. He is very found of music, especially Bollywood songs, and had an expensive
system fitted in his
vehicle through which he blasted the latest tracks as he raced through the streets.
One day, due to the
loud music, he was unable to hear the honk of another car driving alongside, and
crashed into it as he
was making a turn. The other car was unharmed and fled, while Girdhar’s van
suffered from a dent and
his licence plate became disfigured such that nothing on it was visible. He decided
not to go to the police,
and continued driving with his van as it is. A day after, he was caught by the
traffic police. What wrong
had he committed?
(a) Girdhar was blasting music at a very high volume, thus injuring road safety,
and thus should be held
liable for it.
(b) Both the driver of the car and Girdhar should be held liable for causing an
accident and resulting in
injuries to bystanders.
(c) Girdhar would be held liable for not having proper registration marks on his
vehicle.
(d) The driver of the car should be liable instead for causing damage to Girdhar’s
van.
Passage (Q.90-Q.94): “A medical practitioner faced with an emergency ordinarily
tries his best to redeem
the patient out of his suffering. He does not gain anything by acting with
negligence or by committing to
do an irregular act. Obviously, therefore, it will be for the complainant to
clearly make out a case of
negligence before a medical practitioner if charged with or proceeded against
criminally. A surgeon with
shaky hands under fear of legal action cannot perform a successful operation and a
quivering physician
cannot administer the end-dose of medicine to his patient.
The SC has taken a balanced and considerate view of the matter which neither
condones the cases of
callous negligence such as a surgeon forgetting his surgical gauze inside the body
of the patient, or
justifies the carelessness of a medical practitioner within the ordinary skills.”
The notion of negligence in normal parlance can be equalised with carelessness,
breach of care or
caution, dereliction of duty etc. Negligence is the breach of a duty caused by the
omission to do
something which a reasonable man, guided by those considerations which ordinarily
regulate the conduct
of human affairs would do, or doing something which a prudent and reasonable man
would not do. The
negligence existing in the field of medical practices is no different from the
above mentioned approach.
Though there exist no notional distinctness, the treatment that a Medical
Negligence claim demands are
different from its counterparts. The SC observed,

. Page 24 of 40
"The law, like medicine, is an inexact science. One cannot predict with certainty
an outcome of many
cases. It depends on the particular facts and circumstances of the case, and also
the personal notions of
the judge concerned who is hearing the case."
[Extracted from ‘Medical Negligence : When Can A Doctor Be Held Liable For Lack Of
Expertise?’
Published in LiveLaw]
90. Cukoo on her way to college met with an accident and was heavily injured. The
passer-by immediately
took Cukoo to the Life First Hospital and informed the authorities that she needs
immediate attention due
to immense blood loss. At the moment the doctor who was designated at the Emergency
Ward was
heeding to another accident victim and thus the hospital called for another Dr.
Samyak. Dr. Samyak was
on the rounds, checking up with the patient’s progress. When the nurse informed Dr.
Samyak of the
emergency and of patients’critical condition, he said “I’ll quickly finish my daily
rounds and attend to the
patient”. Thereafter, his daily rounds took 30 minutes and then he went to see the
patient. Within minutes
of Dr. Samyak arriving, Cukoo died as she had lost a lot of blood in the said time.
Decide.
(a) Yes, Dr. Samyak will be held responsible as lack of emergency action when so
required, attracts
liability under medical negligence.
(b) Yes, Dr. Samyak will be held responsible as it was because of him that the
patient lost all the blood.
(c) No, Dr. Samyak will not be held responsible as he finished his responsibility
and then attended the
patient.
(d) No, Dr. Samyak will not be held responsible as there were other doctors
available.
91. In the facts as given in Q. No 90., consider the following change in situation.
As soon as the nurse told
Dr. Samyak that there is an emergency, he left all his work and rushed to attend
the patient. On reaching
the ward, he saw that Cukoo was admitted in Ward No. 1 which was not very airy and
the patient could
die of suffocation thus requested the nurses to immediately shift her to Ward No.
2. While the shift was
taking place, Cukoo died. Decide.
(a) Yes, Dr. Samyak will be held responsible as it was because of him that the
patient lost all the blood.
(b) Yes, Dr. Samyak will be held responsible as the situation needed immediate care
and shifting of
wards was not needed at the moment, for he shifted the patient on basis of future
mishappening.
(c) No, Dr. Samyak will not be held responsible as he quickly rushed to attended
the patient.
(d) No, Dr. Samyak will not be held responsible as he acted with due caution taking
in account the
possibility of patient dying due to suffocation, hence not a case of medical
negligence.
92. Dheemi had to undergo a surgery for removing her hernia from her which had
grown in size. The
standard practice is to make an assessment of the allergies of the patient before
administering hernia.
However, Dr. Asmita said that she has a long lasting relationship of 30 years with
Dheemi and is aware
that she has no such allergies. Dr. Asmita went ahead to administer anaesthesia in
normal quantities.
However, Dheemi suffered an allergy due to the anaesthesia and died during the
surgery. Decide.
(a) Dr Asmita will not be held responsible for medical negligence as she
administered normal quantities
of anaesthesia.
(b) Dr Asmita will not be held responsible for medical negligence as it is a
standard practice to check the
charts, but Dheemi was an old client.
(c) Dr Asmita will be held responsible for medical negligence, as she did not adopt
standard practice of
care before the administration of anaesthesia, thus amounting to breach of care.
(d) Dr Asmita will be held liable as her act was not such as a reasonable man would
do.

. Page 25 of 40
93. Dheemi had to undergo a surgery for removing her hernia from her which had
grown in size. The
standard practice is to make an assessment of the allergies of the patient before
administering hernia. Dr.
Asmita checked her charts and accordingly administered her a lower amount of
anaesthesia. However,
Dheemi suffered an allergy due to the anaesthesia and died during the surgery.
Decide.
(a) Dr Asmita will not be held responsible for medical negligence as she
administered decreased
quantities of anaesthesia, keeping account of the allergies.
(b) Dr Asmita will not be held responsible for medical negligence as it was a
reaction which could not
have been predicted with certainity.
(c) Dr Asmita will be held responsible for medical negligence, as she did not adopt
standard practice of
care before the administration of anaesthesia.
(d) Both (a) and (b)
94. Miss. Rajika went to Dr. Dwiwedi for some advice related to her reproductive
health. After hearing the
issue with Rajika’s uterus, Dr. Dwiwedi clarified to Rajika that he is not an
expertise in gynaecology
related matters and that Rajika should seek the medical advice of an expert. He
went ahead to say that
usually he has heard patients take ‘germanium admansismol’, a local medicine for
similar issues. Rajika
without consulting another gynaecology expert, started taking the said medicine as
Dr. Dviwedi had
recommended and developed further infection in her uterus. Decide.
(a) Dr. Dwiwedi will be held responsible for medical negligence as his prescription
of the medicine
resulted in the infection.
(b) Dr. Dwiwedi will be held responsible for medical negligence as when he knew he
doesn’t have the
expertise, he shouldn’t have prescribed any medicine.
(c) Dr. Dwiwedi will not be held responsible for medical negligence as he had
clarified that he is not an
expert in gynaecology related matters.
(d) Dr. Dwiwedi will not be held responsible for medical negligence, as he owed no
duty towards Miss.
Rajika
Passage (Q.95-Q.99): Section 399 of the Companies Act, 2013, specifies the rules
and regulations
governing the inspection, production, and evidence of documents with the Registrar.
In this article, we will
look at the doctrine of constructive notice, the doctrine of indoor management, and
exceptions to the
indoor management rule.
Section 399 allows any person to electronically inspect, make a record, or get a
copy/extracts of any
document of any company which the Registrar maintains. There is a fee applicable
for the same. The
documents include the certificate of incorporation of the company. We know that the
Memorandum and
Articles of Association are public documents. This section confers the right of
inspection to all. Before any
person deals with a company he must inspect its documents and establish conformity
with the provisions.
However, even if a person fails to read them, the law assumes that he is aware of
the contents of the
documents. Such an implied or presumed notice is called Constructive Notice. In
simpler words, if a
person enters into a contract which is beyond the powers of a company, then he has
no right under the
said contract against the company. The Memorandum of Association defines the powers
of the company.
Also, if the contract is beyond the authority of the directors as defined in the
Articles, the person has no
rights.
The doctrine of indoor management is an exception to the earlier doctrine of
constructive notice. Hence, if
an act is authorized by the Memorandum or Articles of Association, then the
outsider can assume that all
detailed formalities are observed in doing the act. This is the Doctrine of Indoor
Management or the
Turquand Rule. In simple words, the doctrine of indoor management means that a
company’s indoor
affairs are the company’s problem. Therefore, this rule of indoor management is
important to people
dealing with a company through its directors or other persons. They can assume that
the members of the
company are performing their acts within the scope of their apparent authority.
Hence, if an act which is
valid under the Articles, is done in a particular manner, then the outsider dealing
with the company can
assume that the director/other officers have worked within their authority.
Exceptions to the Doctrine of
Indoor Management. The Turquand rule or the law of indoor management is not
applicable to the

. Page 26 of 40
following cases: 1) The outsider has actual or constructive knowledge of an
irregularity. 2) The outsider
behaves negligently: The rule of Indoor management does not protect a person
dealing with a company if
he does not initiate an inquiry despite suspecting an irregularity.Further, this
rule does not offer protection
if the circumstances surrounding the contract are suspicious. 3) The doctrine of
indoor management is
applicable to irregularities that affect a transaction except for forgery. In case
of a forgery, the transaction
is deemed null and void.
95. Peter receives a share certificate of ABC Limited issued under the seal of the
company. The company’s
secretary issues the certificate after affixing the seal and forging the signature
of the two directors. Peter
files a lawsuit claiming that the forging of signatures is a part of the internal
management of the company.
Further, he requests the court to stop the company from denying the genuineness of
the document. Is
Peter’s claim valid?
(a) No it’s not as the secretary would be acting on the orders of the directors.
(b) Yes it is not as it is also affixed with the seal of the company which is valid
and hence the suit would be
nullified.
(c) No, as it as forgery is an exception to the rule of indoor management.
(d) Yes it would be but it should happen against the secretary and not the company.
96. In the case of Howard v. Patent Ivory Co. the directors cannot borrow more than
1000 pound without the
consent of the company’s annual general meeting. Directors borrowed 3500 pounds
without the consent
of annual general meeting from another director who took debentures. The director
who lent the money
sued them for violating the rights of the Directors. Will the suit stand?
(a) Yes, it would as the doctrine of indoor management states that it is to be
assumed everything was
done in order.
(b) Yes, it would stand as the withdrawal was for debentures from the director not
the company.
(c) No, it would not as the director had insider knowledge of the irregularity and
was bound to deal in
conformity with provision.
(d) No, it would not stand as the director should not have had 3500 pounds with
himself in the first place.
97. A bill of exchange signed by the manager of a company with his own signature
under the words stating
that he signed on behalf of the company, was held to be forgery when the bill was
drawn in favor of a
payee to whom the manager was personally indebted. The bill, in this case, was held
to be forged
because it purported to be a different document from what it was in fact; it
purported to be issued on
behalf of the company in payment of its debt when in fact it was issued in payment
of the manager’s own
debt. Decide whether the plaintiff will have any remedy.
(a) The company can claim defense of forgery and the irregular conduct of the
manager.
(b) The company can claim the defense of constructive notice as it’s the job of the
receiver to study about
the powers granted by the articles before entering into any such transactions.
(c) The plaintiff can claim that the documents were forged and he was defrauded of
his rights.
(d) The plaintiff can’t claim any remedy for he was having notice of the
irregularity as is mentioned in
passage.
98. In a particular case of T.R. Pratt(Bombay) Ltd. v. E.D. Sassoon & Co. Ltd.,
Company A had lent money to
Company B for mortgaging its assets. The procedure for the same which was laid down
in the Articles for
such nature of transactions were not complied with. The Directors of both the
companies were the same. It
was held by the Court that the lender was aware of such an irregularity and hence
the transaction was not
binding. Decide.
(a) The court was correct in its application, for it is a clear case of
irregularity.
(b) The court was unfair as under both the roles the director acted with different
powers.
(c) The court was correct but it should have applied the doctrine of constructive
notice.
(d) The court was unfair as they did not render to the doctrine of constructive
notice.

. Page 27 of 40
99. In a particular case, X and Y are two directors of a company. A transfer of
shares in the company had
been approved by both X and Y. X was not validly appointed and Y was disqualified
by reason of being
the transferee itself. Decide about the validity of this transfer?
(a) In the case of the transfer would be valid, for doctrine of indoor management
is applicable here;
(b) In the case of the transfer would be not valid, for doctrine of indoor
management is not applicable here;
(c) Both X and Y would not be exempted as they both acted negligently despite
having knowledge of the
facts.
(d) Both would not be exempted as they had insider knowledge of the irregularities.
Passage (Q.100-Q.105): A student who was rusticated from Khwaja Moinuddin Chishti
Urdu, Arabi-Farsi
University on the reason that he was organizing protests arousing the public
against the Citizenship
Amendment Act, 2019 has approached the Supreme Court assailing the Allahabad High
Court's order
affirming University's decision.Interim relief in the form of staying the High
Court's order dated July 23,
2021, along with permission to allow the student to complete his last semester and
to complete
graduation has been sought in the Special Leave Petition.It has been argued before
the Court that the
order of rustication has been passed without giving the BA student a proper
opportunity of hearing and
that even the Chancellor has violated the principle of morality by not personally
hearing the student.It is
further averred that the High Court ought to have considered the grievance of the
petitioner on merits,
rather than summarily upholding the rustication order."The High Court has committed
an error in not
appreciating the reason and dismissing the Writ Petition on the first day itself,"
plea further adds.Relying
on Ranjit Thakur v. Union of India (1987) 4 SCC 611 in which the Top Court had held
that a judgment
which is the result of bias or unjustness or want of impartiality is a nullity and
the trial "coram non- judice",
and that the test of real likelihood of bias is whether a reasonable person, in
possession of relevant
information, would have thought that bias or unjustness was likely, the petitioner
submits that the
university never provided any show cause or charge sheet.
Source Name - Live Law
100. P and Q were friends pursuing LL.B in Jaima Ilsamia college of Law. P was
pursuing his 5th year in the
college whereas Q was in 4th year. Opposing central government’s certain
billsregarding fees hike, P
consolidated hundreds of students for carrying out a protest near Jajantaram
Mamantaram in New
Dailhy. Q appealed to her protestors that the “government is surely going to
eliminate their studies and
drive them out of their colleges and the same would be done in most of the colleges
gradually”. This
statement incited hatred in the minds of people and they wanted the government to
step down, and
consequently P was rusticated from the college. In the light of the above passage,
opt out the most
appropriate option.
(a) P was liable to be rusticated as he had incited hatred against the government
leading to sedition.
(b) Only Q would be liable and not P.
(c) Q& Pare liable to get rusticated for their acts incited the public.
(d) P must be rusticated as he was the mastermind behind all this and must be held
liable.
101. M and N were brothers in a particular college. M used to use mobile phone in
between the lectures but
was caught using the same for the first time and thus his mobile as seized by the
proctor. N tried to
convince the proctor about the same and after certain promises and negotiations, a
conclusion was
drawn upon that M has to pay 5000 rupees as a fine to continue his class otherwise;
he will be rusticated
from the college. N argued referring to the College’s constitution which described
that if a person is
caught doing the same for the first time, he must be given a show cause notice and
nothing else should
be done. However, M and N, both were rusticated from the college on the ground of
unethical conduct.
Opt out the most appropriate option in the light of the passage.
(a) M must get rusticated because of practicing unethical conduct in the classroom
by using mobile
phone.
(b) N should not get rusticated because he was protecting his brother and he had
not done any wrong in
regards to the classroom, but protecting his brother at this point is wrong.

. Page 28 of 40
(c) N and M must get rusticated because advocating a wrong deed; however it must be
show caused
because of the first time is unethical and liable to get the consequences.
(d) N has given the correct reference of the college’s constitution and M and N
should not get rusticated.
102. J was a minority community student who was in the final year in his college.
Due to certain bill initiated by
the central government, he plans to organize a peaceful protest against the same.
Other majority
students, being annoyed by the same, complained about his violent protest
organization plan against the
government going on and got him rusticated from the college. In the light of the
above passage, opt out
the correct option.
(a) However J had organized a peaceful protest, being a college student, it is
unethical to organize
protests against the government as they can lead to incitement of hatred against
the government as
people are more sympathetic to the minorities.
(b) J has the fundamental right to protest against the government irrespective of
the fact that it may incite
hatred against the government.
(c) J should not have been rusticated because he was in his final year and it is
illegitimate to hamper a
student’s career at this moment.
(d) J would not be liable to be rusticated, for his act didn’t arouse or incite the
public.
103. Protests were organized by H the leader of youth congress which was peaceful
in nature and it had the
consent of the college’s administration to carry out a peaceful protest against the
management. One of
the members of the group felt aggrieved as he was not given the chance of public
speaking and thus he
conveyed the message to the administration that H is planning to carry out
vandalism next morning in the
college campus. The administration rusticated him from the college with immediate
effect and he was
arrested for the same. In the light of the above passage, opt out the most accurate
option.
(a) Protest is a fundamental right of every citizen but H must be arrested because
he was planning
vandalism for next morning.
(b) H should have been heard by the college administration, his own version to find
out whether the
same was true or not.
(c) H has been falsely implicated by his fellow member as he was humiliated and
made fun of him during
the protest and thus as an instance of revenge, he had done the same to H.
(d) H should have allowed his fellow member to speak, as he was exercising his
unfettered power of
youth congress leader.
104. "The High Court has committed an error in not appreciating the reason and
dismissing the Writ Petition
on the first day itself". Construing to the line of the above passage, opt out the
appropriate interpretation
of the same.
(a) The high court should have allowed both the parties to be heard.
(b) The high court should have dismissed the appeal after listening to the
petitioner.
(c) The high court should have dismissed the appeal on the day other than the first
day itself.
(d) The high court should not have used it unfettered power in an unethical manner.
105. Construing to the above passage, what is the main idea of the passage that is
being tried to be
conveyed?
(a) The high court should have listened to the appeal of the petitioner and then
dismiss it instead on the
first day itself.
(b) The high court has used its unfettered power to dismiss the appeal of the
petitioner without listening
him on the first day itself.
(c) The college administration has done unethical conduct by not listening to the
petitioner’s version and
rusticating him with immediate effect.
(d) The college was justified in rusticating the petitioner immediately before it
became violent from
peaceful protest.

. Page 29 of 40
SECTION - D: LOGICAL REASONING
Direction: (Q.106): Read the statements and presume that whatever statements given
are true. On the
basis of that, choose the most appropriate conclusion(s) given below.
106. Statements: Some rats are cows. All cows are animals.
Conclusions:
I. All rats are animals
II. Some animals are rats
(a) Only Conclusion I follows (b) Only Conclusion II follows
(c) Both Conclusions I and II follow (d) Neither Conclusion I nor Conclusion II
follows.
107. In the number series 4, 10, 23, 50, 104, 216, 439 the wrong number is
(a) 10
(b) 23
(c) 104
(d) 50

108. Study the following information carefully and answer the questions given
below:
‘Q ÷ R’ means ‘Q is father of R.’
‘Q × R’ means ‘Q is wife of R’.
‘Q + R’ means ‘Q is son of R’.
‘Q – R’ means ‘Q is sister of R’.
How is N related to L, in the given expression L – M + N × O ?
(a) Father
(b) Mother
(c) Sister
(d) Daughter
109. Ravi in his evening walk travelled towards the Sun 3 km, then he turned to his
left walked 2 km. He again
turned to his right and walked 3 km. Finally, he turned to his right walked another
2 km. In which direction
is Ravi walking now?
(a) East (b) West (c) North (d) South
Direction(Q.110): In this question, relationship between different elements is
shown in the statements.
The statements are followed by two conclusions. Study the conclusions based on the
given statements
and select the appropriate answer.
Give answer
(a) If only Conclusion I is true
(b) If either Conclusion I or II is true
(c) If neither Conclusion I nor II is true
(d) If both Conclusion are true.
110. Statements: M < O < U < R > T ; P > R < I < C < L
Conclusions: I. L > M II. O < C

. Page 30 of 40
Passage (Q.111-Q.115): In late 1979, a twenty-four-year-old entrepreneur paid a
visit to a research
center in Silicon Valley called Xerox PARC. He was the co-founder of a small
computer startup down the
road, in Cupertino. His name was Steve Jobs.
Xerox PARC was the innovation arm of the Xerox Corporation. In 1970, Xerox had
assembled the world’s
greatest computer engineers and programmers, and for the next ten years they had an
unparalleled run
of innovation and invention.
Apple was already one of the hottest tech firms in the country. Everyone in the
Valley wanted a piece of
it. So, Jobs proposed a deal: he would allow Xerox to buy a hundred thousand shares
of his company for
a million dollars—its highly anticipated I.P.O. was just a year away—if PARC would
“open its kimono.” A
lot of haggling ensued. Jobs was the fox, after all, and PARC was the henhouse.
Some at PARC thought
that the whole idea was lunacy, but, in the end, Xerox went ahead with it. One PARC
scientist recalls
Jobs as “rambunctious”—a fresh-cheeked, caffeinated version of today’s austere
digital emperor. He was
given a couple of tours, and he ended up standing in front of a Xerox Alto, PARC’s
prized personal
computer.
An engineer named Larry Tesler conducted the demonstration. He moved the cursor
across the screen
with the aid of a “mouse.” Directing a conventional computer, in those days, meant
typing in a command
on the keyboard. Tesler just clicked on one of the icons on the screen. He opened
and closed “windows,”
deftly moving from one task to another. He wrote on an elegant word-processing
program, and
exchanged e-mails with other people at PARC, on the world’s first Ethernet network.
Jobs had come with
one of his software engineers, Bill Atkinson, and Atkinson moved in as close as he
could, his nose almost
touching the screen. “Jobs was pacing around the room, acting up the whole time,”
Tesler recalled. “He
was very excited. Then, when he began seeing the things I could do onscreen, he
watched for about a
minute and started jumping around the room, shouting, ‘Why aren’t you doing
anything with this? This is
the greatest thing. This is revolutionary!”
Xerox began selling a successor to the Alto in 1981. It was slow and underpowered—
and Xerox
ultimately withdrew from personal computers altogether. Jobs, meanwhile, raced back
to Apple, and
demanded that the team working on the company’s next generation of personal
computers change
course. He wanted menus on the screen. He wanted windows. He wanted a mouse. The
result was the
Macintosh, perhaps the most famous product in the history of Silicon Valley.
111. It can be inferred from the passage that
(a) Steve Jobs had at least one founder partner.
(b) Steve Jobs had many founder partners.
(c) Steve Jobs did not have any founder partner.
(d) Steve Jobs was the founder of Xerox PARC.
112. “In 1970, Xerox had assembled the world’s greatest computer engineers and
programmers, and for the
next ten years they had an unparalleled run of innovation and invention.” The
sentence relies on the
assumption that?
(a) The success behind Xerox’s unequaled dominion of innovation and inventions was
a team of the best
engineers and programmers.
(b) To ensure unparalleled success in inventions and innovations, it makes sense to
keep brilliant minds.
(c) A panel of experts leads to unparalleled success in ventures that makes a
company dominate the
field for a long time.
(d) Great minds, with their unique innovations, are behind an organisation’s
hegemony over a period of
time.
113. “Why aren’t you doing anything with this? This is the greatest thing. This is
revolutionary!” from the above
statements, what can one deduce about Steve Jobs?
(a) Steve Jobs was ahead of his contemporaries.
(b) Steve Jobs was a visionary who could see a masterpiece in an innovation.
(c) Steve Jobs was a creative genius who stood the test of time.
(d) Steve Jobs was a revolutionary who had ideas that were way ahead of his time.

. Page 31 of 40
114. “Xerox began selling a successor to the Alto in 1981. It was slow and
underpowered—and Xerox
ultimately withdrew from personal computers altogether.” What is the central flaw
in the given argument?
(a) The above argument takes the cause as the only constant for the effect
produced.
(b) The above argument provides multiple factors to the effect.
(c) The above argument is based on a misinformed conclusion.
(d) The above argument relies on information procured from different sources.
115. The conclusion/conclusions drawn from the passage is/ are…
(i) The tech firm ‘Apple’ was started by Steve Jobs.
(ii) Macintosh was revolutionary as it was among the first to have windows, a menu
on the screen and a
mouse.
(iii) Xerox PARC was the research and development wing of Xerox.
(a) Only (ii)
(b) (i) and (iii)
(c) (i), (ii) & (iii)
(d) None of the above.
Passage (Q.116-Q.120): When we self-diagnose, we look for control factors.
Sometimes we invent them.
The goal of solipsistic anxiety is to find an individual agent that explains our
misery. We eliminate
possibilities one-by-one in hopes that a single cause remains. This is how people
deduce food allergies
and come to workable morning routines (no to coffee, yes to tea; don’t transfer
trains, walk the extra eight
blocks instead). It’s frustrating when changes in lifestyle are not singular but
rather come in waves,
making it harder to identify and explain away the sole source of pain. We prefer
that our personal
problems not be overdetermined.
In the past year, I graduated from college, got a desk job, and bought an iPhone:
the three vertices of the
Bermuda Triangle into which my ability to think in the ways that matter most to me
has disappeared. My
mental landscape is now so altered that its very appearance must be different than
it was at this time last
year. I imagine my brain as a newly wretched terrain, littered with gaping chasms
(What’s my social
security number, again?), expansive lacunae (For the thousandth time, the
difference between
“synecdoche” and “metonymy,” please?), and recently formed fissures (How do you
spell “Gyllenhaal?”).
This is your brain on technology.
I have the sensation, as do my friends, that to function as a proficient human, you
must both “keep up”
with the internet and pursue more serious, analog interests. I blog about real
life; I talk about the internet.
It’s so exhausting to exist on both registers, especially while holding down a job.
It feels like tedious work
to be merely conversationally competent. I make myself schedules, breaking down my
commute to its
most elemental parts and assigning each leg of my journey something different to
absorb: podcast,
Instapaper article, real novel of real worth, real magazine of dubious worth. I’m
pretty tired by the time I
get to work at 9 AM.
In-person communication feels binary to me now: subjects are either private,
confessional, and soulful or
frantically current, determined mostly by critical mass, interesting only in their
ephemeral status. “Maybe
you keep the wrong company,” my mother suggests. Maybe.
116. “When we self-diagnose, we look for control factors. Sometimes we invent
them.” What is the assumption
behind the argument?
(a) The above argument is undeniably the truth.
(b) The above argument has a universal application.
(c) The above argument applies only to people who self-diagnose.
(d) The above argument is an isolated exception.

. Page 32 of 40
117. “It’s frustrating when changes in lifestyle are not singular but rather come
in waves, making it harder to
identify and explain away the sole source of pain. We prefer that our personal
problems not be
overdetermined.” Which of the followings undermines the above argument?
(a) A doctor’s diagnosis of the patient having Lyme’s disease, in the first visit.
(b) A Student’s frustration at not being able to figure out the right career option
out of the glut.
(c) A painting depicting the blooming flower.
(d) A psychologist’s attempt at excoriating layers of misleading psychosomatic
disorders to reach the real
cause.
118. “I imagine my brain as a newly wretched terrain, littered with gaping chasms
(What’s my social security
number, again?), expansive lacunae (For the thousandth time, the difference between
“synecdoche” and
“metonymy,” please?), and recently formed fissures (How do you spell
“Gyllenhaal?”). This is your brain
on technology.” The argument can most reasonably be interpreted as an objection to
which one of the
following claims?
(a) The claim that technology is the be all and end all of our miseries must be
accurately represented to
avoid misleading people.
(b) The claim that technology facilitates the cerebral activities must be made more
precise if we hope to
answer it correctly.
(c) Whether or not there is some truth in the tall claims made by the innovators as
our understanding of
technology is limited.
(d) Whether the present technological advancements are better compared to the
previous ones.
119. “I have the sensation, as do my friends, that to function as a proficient
human, you must both “keep up”
with the internet and pursue more serious, analog interests. I blog about real
life; I talk about the internet.
It’s so exhausting to exist on both registers, especially while holding down a
job.” A reasoning error in the
argument is that the argument
(a) Attempts to infer a value judgment from purely factual premises.
(b) Attributes proficiency and internet activities as co-dependent on each other.
(c) Fails to take into account that the argument may not have a universal
acceptance.
(d) Advocates the use of technology in every area of one’s life.
120. The last paragraph serves to…
(a) present a dichotomy.
(b) address a core issue.
(c) end as an open-ended conclusion.
(d) support the arguments above.
Passage (Q.121-Q.125): Science has a habit of asking stupid questions. Stupid, that
is, by the standards
of common sense. But time and again we have found that common sense is a poor guide
to what really
goes on in the world. So, if your response to the question "Why does time always go
forwards, not
backwards?" is that this is a daft thing to ask, just be patient. Surely, we can
just say that the future does
not affect the past because (duh!) it has not happened yet? Not really, for the
question of where time's
arrow comes from is more subtle and complicated than it seems.
Our everyday experience insists that things only happen one way. Cups of coffee
always get colder,
never warmer, when left to stand. If they are knocked to the floor, the cup becomes
shards and the coffee
goes everywhere, but shards and splashes never spontaneously reassemble into a cup
of coffee. Yet
none of this one-way flow of time is apparent when you look at the fundamental laws
of physics: the laws,
say, that describe how atoms bounce off each other. Those laws of motion make no
distinction about the
direction of time. If you watched a video of two billiard balls colliding and
bouncing away, you would be
unable to tell if it was being run forwards or backwards.
The same time symmetry is found in the equations of quantum mechanics, which govern
the behaviour of
tiny things like atoms. So where does time's arrow come into the picture? There is
a long-standing

. Page 33 of 40
answer to this, which says that the arrow only enters once you start thinking about
lots and lots of
particles. The process of two atoms colliding looks perfectly reversible. But when
there are lots of atoms,
their interactions lead inevitably to an increase in randomness – simply because
that is by far the most
likely thing to happen. Say you have a gas of nitrogen molecules in one half of a
box and oxygen
molecules in the other, separated by a partition. If you take away the partition,
the random movements of
the molecules will quickly mix the two gases completely. There is nothing in the
laws of physics to
prevent the reverse. A mixture of the two gases could spontaneously separate into
oxygen in one half of
the box and nitrogen in the other, just by chance. But this is never likely to
happen in practice, because
the chance of all those billions of molecules just happening to move this way in
concert is tiny. You would
have to wait for longer than the age of the Universe for spontaneous separation to
occur.
121. But time and again we have found that common sense is a poor guide to what
really goes on in the
world. If the view above is correct, it provides a reason for accepting which one
of the following
conclusions?
(a) Because common sense is relied upon its remarkable reasoning, it can be taken
as a marker of
understanding for the events that take place.
(b) If common sense were the guiding factor, we would have a poor study of the
world.
(c) Because of its myopic sensory interpretations and conditioning, common sense
has a limitation in its
inherent nature to encompass events that are beyond its purview.
(d) Common sense renders a miniscule understanding of the world, often leading to a
warped
comprehension of the world around us.
122. “So, if your response to the question "Why does time always go forwards, not
backwards?" is that this is
a daft thing to ask, just be patient.” Through the particular statement, the author
is trying to
(a) Refute the fact that such statements are senseless.
(b) Corroborate the fact that one is devoid of common sense.
(c) Ask the reader to patiently wait till the time the answers are in front of him.
(d) Surprised at the randomness of such questions.
123. Yet none of this one-way flow of time is apparent when you look at the
fundamental laws of physics: the
laws, say, that describe how atoms bounce off each other. Those laws of motion make
no distinction
about the direction of time.
The position taken above presupposes which one of the following?
(i) The interpretation of reality is different from reality itself.
(ii) The one-way flow of time is a myth.
(a) Only (i)
(b) Only (ii)
(c) Both (i) and (ii)
(d) Neither (i) nor (ii)
124. If all statements in the passage are true, then which one of the following
inferences CANNOT be true?
(a) The idea that the past could be a response to the future.
(b) The author’s flouting of the conventional beliefs come from a tenacious grip on
the subject.
(c) Quantum Mechanics deals with studying the behaviour of atoms.
(d) None of the above.
125. We can conclude from the last four lines of the last paragraph that
(a) It is a next to none possibility of witnessing a reverse flow of time.
(b) It can be rarest of the rare case that one can be a witness to the reversel
flow of time.
(c) The witnessing of reverse flow of time can take place once in our lifetime.
(d) The witnessing of reverse flow of time may take many re-births.

. Page 34 of 40
Passage (Q.126-Q.130): We’re not driven only by emotions, of course—we also reason,
deliberate. But
reasoning comes later, works slower—and even then, it doesn’t take place in an
emotional vacuum.
Rather, our quick-fire emotions can set us on a course of thinking that’s highly
biased, especially on
topics we care a great deal about.
Consider a person who has heard about a scientific discovery that deeply challenges
her belief in divine
creation—a new hominid, say, that confirms our evolutionary origins. What happens
next, explains
political scientist Charles Taber of Stony Brook University, is a subconscious
negative response to the
new information—and that response, in turn, guides the type of memories and
associations formed in the
conscious mind. “They retrieve thoughts that are consistent with their previous
beliefs,” says Taber, “and
that will lead them to build an argument and challenge what they’re hearing.”
In other words, when we think we’re reasoning, we may instead be rationalizing. Or
to use an analogy
offered by University of Virginia psychologist Jonathan Haidt: We may think we’re
being scientists, but
we’re actually being lawyers (PDF). Our “reasoning” is a means to a predetermined
end—winning our
“case”—and is shot through with biases. They include “confirmation bias,” in which
we give greater heed
to evidence and arguments that bolster our beliefs, and “disconfirmation bias,” in
which we expend
disproportionate energy trying to debunk or refute views and arguments that we find
uncongenial.
That’s a lot of jargon, but we all understand these mechanisms when it comes to
interpersonal
relationships. If I don’t want to believe that my spouse is being unfaithful, or
that my child is a bully, I can
go to great lengths to explain away behavior that seems obvious to everybody else—
everybody who isn’t
too emotionally invested to accept it, anyway. That’s not to suggest that we aren’t
also motivated to
perceive the world accurately—we are. Or that we never change our minds—we do. It’s
just that we have
other important goals besides accuracy—including identity affirmation and
protecting one’s sense of
self—and often those make us highly resistant to changing our beliefs when the
facts say we should.
126. “Rather, our quick-fire emotions can set us on a course of thinking that’s
highly biased, especially on
topics we care a great deal about.” The underlying thought behind the passage is…
(a) Emotions affect one’s thinking on personal matters.
(b) Emotions lead to a skewed thinking on personal matters.
(c) Emotions neutralises one’s thinking on emotional matters.
(d) One must separate emotions from rational thinking on matters that our personal
to us.
127. We’re not driven only by emotions, of course—we also reason, deliberate. If we
take the preceding
statement as an answer, then which of the following questions is most relevant.
(a) Why emotions are the sole driving force when taking life changing decisions?
(b) Why emotions cloud our judgments to make them non-rational?
(c) Why emotions determine the way we behave?
(d) How can we curtail our overwhelming emotions?
128. “We may think we’re being scientists, but we’re actually being lawyers.” What
can be inferred from the
given sentence?
(a) A scientist’s bent of mind while reasoning is different from a lawyer’s.
(b) While a scientist either accepts a theory or rejects it by providing logical
and scientific evidences in
either case, a lawyer accepts or rejects a theory with a predetermined mindset of
winning.
(c) While a lawyer either accepts a theory or rejects it by providing logical and
scientific evidences in
either case, a scientist accepts or rejects a theory with a predetermined mindset
of winning.
(d) All scientists behave like lawyers and all lawyers behave like scientists when
it comes to defending
their theory.

. Page 35 of 40
129. That people retrieve thoughts that are consistent with their previous beliefs,
and build them to an
argument and challenge what they’re hearing figures in the argument in which one of
the following ways?
(a) It suggests an alternative prospective to the one adopted in the argument.
(b) It sets out a problem the argument is designed to resolve.
(c) It is compatible with accepting the conclusion by presenting as an
illustration.
(d) It summarizes a position the arguments as a whole is directed toward
discrediting.
130. That’s not to suggest that we aren’t also motivated to perceive the world
accurately—we are. Or that we
never change our minds—we do. It’s just that we have other important goals besides
accuracy—including
identity affirmation and protecting one’s sense of self—and often those make us
highly resistant to
changing our beliefs when the facts say we should.
Which one of the following judgments most closely confirms to the principle cited
above?
(a) A scientist presented a theory which challenged the existing theory to the very
core. He faced a
massive backlash to which he responded by presenting more facts. In an interview,
he mentioned
that he cannot change his facts to suit the existing social structure.
(b) Bella took up Medicine as a profession despite lacking the necessary skills.
When reminded of her
folly and being asked to change the course, by studying Architecture, by her
friends, she reasoned
out by explaining in length as how Architecture does not have the as potential as
Medicine does. After
one year of Medicine, she switched to Architecture.
(c) A grandfather wanted to marry off his granddaughter at a very early age,
knowing very well that his
philosophy does not find a place in the modern world. He did not want to change as
change was
against the traditions that he though he represented.
(d) Sam agreed to watch his three-year-old nephew while she played but, becoming
engrossed in
conversion with his friend, did not see him run into the street where he was
attacked by a dog; even
though he intended no harm, Sam’s action was morally bad.
Passage (Q.131-Q.135): The prolonged and seemingly unending nature of the
disruption and losses
make us routinely feel disoriented, anxious and exhausted. Like we need the vaccine
as protection from
the virus, we need to learn ways to immunise ourselves from our inner emotional
turmoil.
Here are three ideas you can consider.
Firstly, we need to cultivate a sense of equanimity – an anchor of stillness within
us so the changing
external scenery doesn’t easily affect us. This doesn’t mean that we are not
affected at all. It’s just that
the threshold at which we experience emotional hijack goes up. Building equanimity
requires deepening
our emotional self-awareness. We need to learn what makes us happy, sad, insecure
and excited; how
frequently we move from one emotional state to the other; and what triggers these
shifts. Ordinarily, we
are on an autopilot mode where certain triggers produce a predictable reaction
within us time after time.
When we are in touch with our changing emotional states, we can quickly catch
ourselves starting to feel
anxious, envious or angry. We can then choose to acknowledge those emotions and
without judging
ourselves, explore alternate ways to respond. That’s how we can break the pattern
of our emotional
reactions.
Secondly, one of the reasons we experience emotional stress is our attachment to
specific outcomes. My
promotion this year; our move to a new home in the next six months; marriage of our
28-year-old in 2021.
Anything that threatens that outcome is an immediate source of anxiety within us.
The way to deal with
this is to open our heart to possibilities of whatever may emerge. Whether that
promotion comes this year
or the next; whether our 28-year-old decides to get married in a year or three.
This requires learning to let
go of our attachment to fixed outcomes and trust the natural process of evolution.
Finally, another reason we struggle to be at peace with the uncertainty of the
future is that we fear and
hate setbacks. We can protect ourselves against this fear by learning to reframe
our relationship with
difficult circumstances. Almost nothing is as good or as bad as it seems. Also, we
can choose to assess
what the situation is trying to teach us.

.
Also, we usually underestimate our innate ability to deal with setbacks. We are
more resilient than we
often believe. In fact, invariably, it’s in such situations that we learn and grow
the most. We simply need
to get more comfortable with being uncomfortable.
131. “Like we need the vaccine as protection from the virus, we need to learn ways
to immunise ourselves
from our inner emotional turmoil.” What is the approach the author has adopted in
the particular
statement?
(a) The author compares effects of external and internal virus.
(b) The author draws an analogy for better comprehension.
(c) The author preaches immunizing oneself against emotional turmoil.
(d) The author predicts the emotionally turbulent times ahead.
132. “This doesn’t mean that we are not affected at all. It’s just that the
threshold at which we experience
emotional hijack goes up.” All of the following are assumptions except
(a) There is a threshold of emotional experience.
(b) The threshold of emotional experience fluctuates.
(c) Increasing the threshold acts as a buffer.
(d) Increasing the threshold of emotional experience guarantees relief from
emotional upheavals.
133. Finally, another reason we struggle to be at peace with the uncertainty of the
future is that we fear and
hate setbacks. We can protect ourselves against this fear by learning to reframe
our relationship with
difficult circumstances. If the above statements are true, then which of the
following reflects ‘reframing’ as
mentioned in the statement?
(a) Going to the astrologer to procure a talisman as a protection against the
uncertainties.
(b) Accepting the uncertainties as part of life, one can detach from the emotions
attached to our future
and learn better coping mechanism.
(c) Embracing the good and bad emotional reactions to the uncertainties and
welcoming all emotional
states as part of life.
(d) Biennially visiting resorts built to help people with manage emotions
effectively through various
modalities.
134. All of the following can be inferred from the statement, except
(a) We must let unsettling events jolt us out of our comfort zone in order to grow.
(b) Constant upheavals can upset the equilibrium and can rob one of peace.
(c) We cannot build an immunity against calamities, but can immunize ourselves
against the negative
outlook towards the emotional turbulence.
(d) Letting go of the immediate need for gratification can be a great stress
buster.
135. The primary purpose of the author is…
(a) To chart out a course of action to immunize one against the unwanted emotional
reactions to the
circumstances.
(b) To highlight some of the laden out paths as sure shot remedies to ease the
troubling mind.
(c) To rule out the fact that the reaction to the negative circumstances require a
reframing of the mind.
(d) To emphasize the need to modify out thinking towards the emotional upheavals
through various
techniques

mock 11

Directions (Q.1-Q.30): Read the passages carefully and answer the questions.
Passage (Q.1-Q.5): My daughter Holly, who is seven years, persuaded me to come in
to give a talk to
her class. Her teacher was really enthusiastic, and in I came. They sat on the
floor, I had a chair, fifty#seven-year-old-eyes gazed up at me. 'When I was your
age, people told me not to make things up,' I told
them.
And eventually one of them asked it. 'Where do you get your ideas?'
And I realized I owed them an answer. They weren't old enough to know any better.
And it's a perfectly
reasonable question.
This is what I told them:
You get ideas from daydreaming. You get ideas from being bored. You get ideas all
the time. The only
difference between writers and other people is we notice when we're doing it.
You get ideas when you ask yourself simple questions. The most important of the
questions is just, What
if...? (What if you woke up with wings? What if your sister turned into a mouse?
What if you all found out
that your teacher was planning to eat one of you at the end of term - but you
didn't know who?)
Another important question is, If only... (If only real life was like it is in
Hollywood musicals. If only I could
shrink myself small as a button. If only a ghost would do my homework.)
And then there are the others: I wonder... ('I wonder what she does when she's
alone...') and If This Goes
On... ('If this goes on telephones are going to start talking to each other, and
cut out the middleman...')
and Wouldn't it be interesting if... ('Wouldn't it be interesting if the world used
to be ruled by cats?')...
An idea doesn't have to be a plot notion, just a place to begin creating. Plots
often generate themselves
when one begins to ask oneself questions about whatever the starting point is.
Sometimes an idea is a person ('There's a boy who wants to know about magic').
Sometimes it's a place
('There's a castle at the end of time, which is the only place there is...').
Sometimes it's an image ('A
woman, sifting in a dark room filled with empty faces.')
Often ideas come from two things coming together that haven't come together before.
('If a person bitten
by a werewolf turns into a wolf what would happen if a goldfish was bitten by a
werewolf? What would
happen if a chair was bitten by a werewolf?')
All fiction is a process of imagining: whatever you write, in whatever genre or
medium, your task is to
make things up convincingly and interestingly and new.
1. "And eventually, one of them asked it. 'Where do you get your ideas?'", Which of
the following/ (s) can be
inferred from the given statement?
(i) The children were not receptive to the author.
(ii) The child was a follower of the author's book.
(iii) The author was waiting for one of the children to come up with that
particular question.
(a) Only (i) (b) Only (ii) (c) (ii) & (iii) (d) All of the following.
2. Which of the following does not find support in the given passage?
(a) The author felt the need to answer befitting the children's understanding of a
seven-year-old.
(b) An idea can come up by asking 'what if….'? 'If only…', 'If this goes on...' and
'Wouldn't it be
interesting if...'.
(c) An idea can be a neighbour, New Zealand or a painting.
(d) An idea can come from reverie.
3. From the given passage, we can say that the author is…
(a) a Sci-fi writer conjuring up images of space, Star Wars and galaxy stories.
(b) a fictional writer who produces a story meant to entertain.
(c) a children's storybook writer blending the old with the new.
(d) a motivational non-fictional writer inspiring the readers with moving quotes.

. Page 3 of 36
4. The author uses which of the following styles of writing?
(a) Prescribing (b) Descriptive (c) Narrative (d) Expository
5. What is the author trying to communicate through the passage?
(a) The author, through the passage, wants to communicate that story writing is for
anyone who is a
creator. A person with prolific imagination is best suited for fiction writing.
(b) The author, through the passage, wants to convey that fiction writing is a
figment of the
imagination and every existing being or non-being contributes as an idea in some
way to shaping
up the story in a convincing and entertaining form.
(c) The author, through the passage, is amusing the children with his anecdote and,
by doing so, is
instilling in them that they too can write with a little bit of imagination.
(d) The author wants to dissuade amateur writers by conveying that writing is a
serious occupation,
especially creative writing, a very specialised skill.
Passage (Q.6-Q.10): Adrienne Boyle, one of Britain's most important flexible-work
activists of the past
century, gazed out the window of her houseboat in Dublin's historic docklands
district. She had a clear
view of Google's 14-storey European headquarters. 'There's not much flexible
working in that one! Lights
on all night, 11pm Saturday night,' she told me, when I spoke to her in December
last year. 'It's flexible all
right, but you're working 60-hour weeks.'
Long before the pandemic made remote work a necessity, employees across Anglo-
America said that
they wanted more flexibility about where and when they work. A 2017 Gallup poll in
the United States
found that 51 per cent of workers would be willing to change jobs for one that
allowed them some control
over their hours, and 35 per cent for one in which the location was flexible. While
flexibility was originally
associated with women seeking to combine paid work with unpaid childcare, it's
since become a key item
on the list of desirable perks for all workers. In the 21st century, flexible work
culture has found its
apogee in large tech companies that have embraced notions such as work-life
balance, family#friendliness and employee wellness as guiding principles.
Yet, even those employees who enjoy the benefits of flexibility - and it's still a
privileged minority - have
found that it doesn't necessarily mean their working lives have become easier or
better. Flexibility can
make it hard to draw boundaries around paid employment, and difficult to
disaggregate work from the rest
of the day. Nor has flexibility at work solved the pressing problems of child or
eldercare, or shifted the
gendered division of housework. While companies such as Google and Facebook like to
trumpet the
benefits which their employees gain from their respective flexible work-hour
policies, they still haven't
fixed the childcare problems of staff, and they still fail to make their benefits
accessible to everyone. The
abrupt restructuring of daily working life for tens of millions due to the pandemic
has also dramatized just
how different 'flexible' work is in different contexts: liberating for some,
imprisoning for others.
Modern-day flexible work policies didn't arise in a sudden moment of crisis, but
from the slow burn of
second-wave feminist activism. In the 1970s, even though growing numbers of women
had entered the
paid workforce, they continued to do a disproportionate share of the childcare and
housework. In the
consciousness-raising and campaign groups that cropped up in the US and Europe,
women increasingly
recognised that what felt 'merely' personal was, in fact, political. A new
generation of activists pushed for
changes in the structure and conditions of paid work. The idea was to render it
more suited to the needs
of workers with caring responsibilities and allow women of all backgrounds to
participate in the economy
on equal terms with men.
6. What would be the underlying premise for the author's statement, "Yet, even
those employees who enjoy
the benefits of flexibility - and it's still a privileged minority - have found
that it doesn't necessarily mean
their working lives have become easier or better"?
(a) Only a few employees reap the flexibility benefits offered, if any, by their
respective employers.
(b) The objective of flexibility is to ease and better the professional lives of
individuals.
(c) The benefits of flexibility in workplaces are disproportionally skewed in
favour of employers as
compared to employees.
(d) All (a), (b) and (c).

. Page 4 of 36
7. Identify the statement(s) that is/are incorrect as per the information given in
the passage.
(a) Working remotely was a privilege or luxury way before the pandemic.
(b) The author is of the view that working 60- hours per week cannot be considered
as flexible.
(c) Flexibility in working hours is more luring than flexibility in location.
(d) More than one option is incorrect.
8. Which of the following sentence(s) can grammatically and contextually follow
immediately after the last
sentence of the passage?
(a) This is most apparent in the first-hand testimony of the women who dedicated
their energies to
reimagining paid work at a time when the 9-to-5, 40-hour working week was the near-
universal
model for profession success.
(b) Feminist activism for what we now call 'flexibility’ was part of a vision for
remaking communities
and supporting the needs of workers as whole human beings.
(c) For this, men were urged to participate more in maintaining home and family.
(d) In the decades since feminists first challenged the structures governing paid
work, the vision at
the heart of their campaigning has been lost.
9. Which of the following can be inferred from the passage?
(a) Flexible work-hour policies are full-proof, from some employer's perspectives.
(b) Saturdays, more often than not, are part of non-working days.
(c) Anglo-American employees were not satisfied with then-current flexibility
policies at their
respective workplaces.
(d) None of the these.
10. What is the contextual meaning of the following as used in the passage?
Apogee
(a) Perigee (b) Culmination (c) Worth or value (d) Voice
Passage (Q.11-Q.15): The erosion of trust in a civil society is one of the greatest
moral challenges facing
the world today. Democratic societies are anxious. Leaders, and the general public,
are worried about
extremism, terrorism and radicalisation. Educators and experts are rightly
concerned about those who
perpetuate approaches that resemble indoctrination. Such threats are making us less
trusting of others,
particularly of those we see as somehow different from ourselves.
A remedy may be found in educating people to be "global citizens", who are not just
caring, but are also
critically engaged with ideas, beliefs and attitudes exhibited across the world.
These global citizens can
help to rebuild the lost trust in civil society in an increasingly diverse and
globalised world.
In order to live among others in a harmonious manner, we must recognise that others
wish to live a good
life, much like ourselves. As Tim Dean points out, the "good life" looks different
for different people at
different times. There is no "one size fits all" when it comes to the good life.
The expression of diversity is
an important component of a world that celebrates liberty. Yet such freedom must be
coupled with
respect and care if we are to have any hope of promoting ethical decision-making on
a local, national or
international level. We must think of ourselves as alike even amidst our diversity.
As individuals who live in communities, we must co-operate in order to achieve our
goals, whether they
be small or large. A life is shaped by our interaction (or lack thereof) with
others: family, friends,
colleagues and strangers. We crave and seek out connection and a sense of
belonging. With the
technological tools available to us, we may now connect at any minute of any day
with people we have
never met in places we have never been.
Trust is one of the most important pro-social attitudes we have. Philosophers such
as Annette Baier and
Martha Nussbaum have written about the vulnerability as well as the necessity of
trust. If we don't trust
others or treat one another with respect and compassion, our interactions can be
unsettling, sources of
anxiety or even fear. Yet, if we approach others as friends - as more or less like
me - a reciprocal and
mutual goodwill is shared and is conducive to pleasant interactions and life-
affirming experiences.
Granted, blind trust is as dangerous as blind faith. Trust, and compassion, must be
accompanied by an

. Page 5 of 36
appropriately engaged critical mind. But we must not go so far as to think that
trust and care are
unreasonable just because the world is a scary place.
11. Which of the following question(s) can be answered from the information given
in the passage?
I. Can a liberal world be imagined with the lone existence of uniformity?
II. Is the idea of 'global citizens' relevant in the contemporary era?
III. What is the role of perception in shaping our experiences in the world?
(a) I and II only (b) I and III only (c) II only (d) II and III only
12. What is the contextual meaning of the following as used in the passage?
Indoctrination
(a) Contamination of an idea or belief with a sub-standard one.
(b) The process of teaching a person or group to accept a set of beliefs
uncritically.
(c) Process of suggesting an alternate idea if the current idea doesn't fructify.
(d) A belief or set of beliefs held and taught by a Church, political party, or
other groups.
13. Which of the following can be inferred from the passage?
(a) A goodwill gesture is always reciprocated by a gesture of a similar kind.
(b) The idea of 'one size fits all' can't be applied to other spheres or realms of
life too.
(c) Being vulnerable necessitates the significance of trust even more.
(d) None of the above
14. Identify the statement(s) that is/are NOT correct as per the information given
in the passage.
(a) People think the world is a less trustworthy place than previous generations.
(b) Compassion and a critical mind can co-exist simultaneously.
(c) The global citizen is someone who recognises others as more alike than
different from themselves,
even while taking seriously individual, social, cultural and political differences.
(d) More than one option is correct.
15. The author is most likely to agree with which of the following?
(a) Reading too many sensationalised news stories may lead us to perceive the world
as an unhappy,
unfriendly place where people are not to be trusted.
(b) Caring about others we have never met and who may seem very different from
ourselves can be a
challenge, particularly in a climate of fear.
(c) It is easy to cultivate the habits of a compassionate yet critical, global
citizen with the
connectedness of our technological world.
(d) More than one option is correct.
Passage (Q.16-Q.20): For the first time in 86 years, physicists are on the verge of
discovering a new
fundamental force of nature which might help unravel the yet unknown parts of the
Universe such as dark
matter and dark energy. The new results might also indicate the discovery of a new
sub-atomic particle,
which is usually discovered every few years. The need for a new force or the new
particle indicates a new
era in physics deviating from what humans have known so far.
The need itself arises from unknown observed behaviour of muons, fundamental sub-
atomic particles
similar to electrons but 200 times heavier. Like the electron, the muon acts like a
tiny magnet and
therefore gets influenced by magnetic fields. When placed in an external magnetic
field, the muon
wobbles. But the latest 'Muon g-2' experiment showed the sub-atomic particles are
not wobbling the way
they should when exposed to known magnetic fields.
A new force of nature or a new fundamental sub-atomic particle is perhaps
influencing the muons, results
announced recently demonstrated. The strength of a muon's internal magnet
determines the measure by
which it wobbles when exposed to a magnetic field. Physicists call this measure the
'g-factor'. In the
Muon g-2 experiment, the muons are exposed to a magnet and a flurry of other sub-
atomic particles

. Page 6 of 36
collectively known as the 'quantum foam'. The particles of the quantum foam pop
into existence and go
out in really short _________ (1) of time.
While the magnet causes the greatest change in the g-factor, the quantum foam can
also influence the g#factor to an extent. The Standard Model allows scientists to
calculate the impacts of both these factors
with high precision but if the quantum foam consists of particles or forces that
are not a part of the
Standard Model, then the g-factor values will change further in unknown ways. This
is what the latest
results of the Muon g-2 experiment show.
“So far we have analysed less than 6 percent of the data that the experiment will
eventually collect.
Although these first results are telling us that there is an intriguing difference
with the Standard Model,
we will learn much more in the next couple of years, “said Chris Polly, co-
spokesperson for the
experiment.
The first time scientists had observed this unknown behaviour of Muons was back in
2001 at the DOE’s
Brookhaven national Laboratory. It has taken science 20 years to make the
measurement of the
discrepancy as precise as possible. Now, the results of the two experiments agree
with each other and
both deviate from the Standard Model.
16. Out of the four alternatives, choose the one which best expresses the meaning
of the word ‘discrepancy’.
(a) Disparity (b) Discordance (c) Reiteration (d) Cosiness
17. Select the most appropriate option to fill in the blank 1.
(a) Duration (b) Interim (c) Gap (d) Prolongation
18. According to the passage, what does the latest results of the Muon g-2
experiment indicate?
(a) That if the quantum foam consists of particles or forces that are a part of the
Standard Model, then
the g-factor values will change further in unknown ways and scientists will be
unable to calculate
the impacts of both these factors with high precision.
(b) That if the quantum foam consists of particles or forces that are not a part of
the Standard Model,
then the g-factor values will remain constant and scientists will be able to
calculate the impacts of
both these factors with high precision.
(c) That if the quantum foam consists of particles or forces that are not a part of
the Standard Model,
then the g-factor values will change further in unknown ways and scientists will be
unable to
calculate the impacts of both these factors with high precision.
(d) None of the above.
19. What does the need for a new force or the new particle suggest?
(a) A new era in physics drifting from the understanding by humans to date.
(b) A new force of nature is influencing the muons.
(c) A new fundamental sub-atomic particle is influencing the muons.
(d) None of the above.
20. We can understand from the passage that the author has…
(a) a complete understanding of the subject matter at hand.
(b) a superficial understanding of the subject matter.
(c) simply reported matters from hand out given at a science conference.
(d) been in the company of the physicists.

. Page 7 of 36
Passage (Q.21-Q.25): From the 16th century to the 19th, scurvy killed around 2
million sailors, more than
warfare, shipwrecks and syphilis combined. It was an ugly, smelly death, too,
beginning with rattling teeth
and ending with a body so rotted out from the inside that its victims could
literally be startled to death by a
loud noise. Just as horrifying as the disease itself, though, is that for most of
those 300 years, medical
experts knew how to prevent it and simply failed to. In 1747, a British doctor
named James Lind
conducted an experiment where he gave one group of sailors, citrus slices and the
others, the traditional
vinegar or seawater or cider. The crewmen who ate fruit improved so quickly that
they were able to help
care for the others as they languished. Lind published his findings, but died
before anyone got around to
implementing them nearly 50 years later.
This brings us to one of the largest gaps between science and practice in our own
time. Years from now,
we will look back in horror at the counterproductive ways we addressed the obesity
epidemic and the
barbaric ways we treated fat people. According to the Centers for Disease Control
and Prevention, nearly
80 percent of adults and about one-third of children now meet the clinical
definition of overweight or
obese. More people live with “extreme obesity” than with breast cancer,
Parkinson’s, Alzheimer’s and HIV
put together.
The medical community’s primary response to this shift has been to blame fat people
for being fat.
Obesity, we are told, is a personal failing that strains our health care system,
shrinks our GDP and saps
our military strength. It is also an excuse to bully fat people in one sentence and
then inform them in the
next that you are doing it for their own good. That’s why the fear of becoming fat,
or staying that way,
drives us to spend more on dieting every year than we spend on anything. The
emotional costs are [x]. I
have never written a story where so many of my sources cried during interviews,
where they double and
triple-checked that I would not reveal their names. My interest in this issue is
slightly more than
journalistic. Growing up, my mother’s weight was the uncredited co-star of every
family drama, why she
never got out of the car when she picked me up from school, why she disappeared
from the family photo
album for years at a time, etc. Obesity has given rise to many health disorders
like diabetes,
osteoarthritis, kidney diseases, heart-diseases, asthma etc. However, the solution
lies not in getting
skinnier but healthier.
21. What is the real point that the author wants to establish in the first
paragraph of the passage?
(a) Scurvy was a disease that had rattled humanity for almost three centuries
before the scientists found
the treatment.
(b) People were oblivious to the fact that scurvy had killed so many people for a
very long time.
(c) There is an inherent myopic characteristic residing in the history of medical
sciences that combats
invention with convention.
(d) When humanity first comes face-to-face with a new disease, there are always
many casualties, and
the losses could also take a mental toll.
22. Which of the following is a suitable title for the passage?
(a) Scurvy and Obesity – the two most misunderstood diseases.
(b) Everything we know about obesity is wrong.
(c) How to reduce obesity.
(d) The fight with the epidemic of obesity.
23. Which of the following summarizes the main point of the passage?
(a) There are very few people who care for the fact that obesity causes mental
problems.
(b) Although obesity is a serious disease, the handling of it should be done with
empathy.
(c) The biggest misconception about obesity has been to reduce weight rather than
to be healthy.
(d) Obesity is a very serious disease; it leads to many other illnesses, and people
should avoid it at all
costs.

. Page 8 of 36
24. A line in the passage has been italicized. One word of that line has been
redacted by [x]. With reference
to the passage, which would be the most suitable word out of the following?
(a) rudimentary (b) unmanageable (c) ground-breaking (d) immeasurable
25. Why has the author included the line “More people live with “extreme obesity”
than with breast cancer,
Parkinson’s, Alzheimer’s and HIV put together” with respect to the context of the
passage?
(a) The author wants to show how big the obesity disease has become nowadays.
(b) The author wants to show the fact that obesity is far more rampant a disease
than breast cancer,
Parkinson’s, Alzheimer’s and HIV put together.
(c) The author wants to show that doctors have been treating obesity in a wrong way
since the
beginning.
(d) The author wants to show the similarity between the two diseases – scurvy and
obesity.
Passage (Q.26-Q.30): The pioneers of the teaching of science imagined that its
introduction into
education would remove the conventionality, artificiality and backward-lookingness
which were
characteristic of classical studies, but they were gravely disappointed. So, too,
in their time had the
humanists thought that the study of the classical authors in the original would
banish at once the dull
pedantry and superstition of mediaeval scholasticism. The professional schoolmaster
was a match for
both of them, and has almost managed to make the understanding of chemical
reactions as dull and as
dogmatic an affair as the reading of Virgil's Aeneid.
The chief claim for the use of science in education is that it teaches a child
something about the actual
universe in which he is living, in making him acquainted with the results of
scientific discovery, and at the
same time teaches him how to think logically and inductively by studying scientific
method. A certain
limited success has been reached in the first of these aims, but practically none
at all in the second.
Those privileged members of the community who have been through a secondary or
public school
education may be expected to know something about the elementary physics and
chemistry of a hundred
years ago, but they probably know hardly more than any bright boy can pick up from
an interest in
wireless or scientific hobbies out of school hours.
As to the learning of scientific method, the whole thing is palpably a farce.
Actually, for the convenience
of teachers and the requirements of the examination system, it is necessary that
the pupils not only do
not learn scientific method but learn precisely the reverse, that is, to believe
exactly what they are told
and to reproduce it when asked, whether it seems nonsense to them or not. The way
in which educated
people respond to such quackeries as spiritualism or astrology, not to say more
dangerous ones such as
racial theories or currency myths, shows that fifty years of education in the
method of science in Britain or
Germany has produced no visible effect whatever. The only way of learning the
method of science is the
long and bitter way of personal experience, and, until the educational or social
systems are altered to
make this possible, the best we can expect is the production of a minority of
people who are able to
acquire some of the techniques of science and a still smaller minority who are able
to use and develop
them.
Adapted from: The Social Function of Science, John D Bernal (1939)
26. In the statement, “The professional schoolmaster was more than a match for both
of them” the word
‘both’ refers to:
(a) The earlier teachers of science.
(b) The forerunners of scientific education and the humanists
(c) The students of the schoolmaster and his fellow teachers
(d) The classical scholars

. Page 9 of 36
27. “A certain limited success has been achieved in the first of these aims but
none in the second.” What
does the author imply through this statement?
(a) The schoolmaster has proved both the earlier teachers of science and the
humanists completely
wrong.
(b) Only privileged members of the community have benefited from a scientific
education.
(c) The students of science have gained some scientific knowledge but have been
unable to develop the
faculty of clear and rational thinking.
(d) Science should be taught through the radio and wireless rather than
conventional teaching methods
to ensure these aims.
28. According to the passage, what is the ‘reverse’ of the scientific method?
(a) To be rational and question every observation and occurrence.
(b) To learn through personal experience.
(c) Anything that is being followed in Britain and Germany.
(d) To accept as true the knowledge in textbooks without questioning it in any
manner.
29. Which of the following statements best sums up the passage?
(a) The pioneers of scientific education have been gravely disappointed.
(b) Persons who have received secondary or public education hardly know more than a
bright student
who may learn through practical experience and knowledge.
(c) The learning of the scientific method is a farce.
(d) Scientific teaching has been unable to renew itself and still follows the
conventional pattern that only
leads to acquiring some knowledge but no development of scientific temperament.
30. Observe the line that has been italicized in the passage. Which of the
following figures of speech has
been used?
(a) Anachronism (b) Simile (c) Cacophony (d) Caesura
Directions (Q.66 – Q.105): Read the comprehensions carefully and answer the
questions based on it.
Passage (Q.66-Q.69): Section 124A, Sedition, of the Indian Penal Code (IPC) reads,
“Whoever by
words, signs, visible representation brings into hatred or contempt or excites
disaffection against the
government shall be punished with imprisonment for life”.
The Supreme Court in Vinod Dua’s case held that sedition would be a prosecutable
offence only in
terms of the law laid down in the 1962 Kedar Nath v State of Bihar judgment, where
it was held that “a
citizen has a right to say or write whatever he likes about the government, or its
measures, by way of
criticism or comment, so long as he does not incite people to violence against the
government
established by law or with the intention of creating public disorder.”
This point was emphasised by the Supreme Court in two cases. In Balwant Singh’s
case, a pro#Khalistan agitator passionately called upon Sikhs to struggle by use of
arms to carve out a separate
state in India. No violence followed and he was acquitted for his mere words.
In Bilal Ahmed Kaloo’s case a similar call for azadi by use of arms without any
violence ensuing
resulted in acquittal. The Supreme Court emphasised that large-scale violence was a
necessary
ingredient of sedition.
The Union was proud to retain this weapon of the war against the people and refused
to repeal the
section, even though the UK government, from whom we inherited this law and many
other countries,
repealed its sedition law decades ago.
Source:- https://thewire.in/law/sedition-and-the-supreme-court-justice-delayed-but-
not-justice-denied
66. While discontent against recent revelations of international usage of Pegasus
spyware software by
various sovereign governments was on rise, one Rohan started a chain of anti-
government tweets,
articles, and also gave speeches to large crowds about how the constitutionally
ensured privacy was
being breached by the government and motivated the masses that to keep us and our
democracy safe,
the citizens must show strength at the right time. Next week, crowds began to march
towards the
Parliament with violent motives. Rohan was arrested and booked under Section 124A
of the Indian Penal
Code. Decide?
(a) Rohan is liable for the offence of sedition because he published tweets,
articles and gave anti#government speeches in greater proportion than the extent of
spyware and privacy threat existed in
India, therefore misleading the public.
(b) Rohan is liable for the offence of sedition as his speech incited violent
motivation among the crowd,
as stated by the Supreme Court in Kedar Nath and Vinod Dua’s case.
(c) Rohan is not liable for the offence of sedition since by showing strength at
right time, he meant not
voting for the sitting government in the upcoming elections, practising democratic
rights.
(d) Rohan is not liable for the offence of sedition since right to privacy is
ensured by the constitution, and
upheld by the Supreme Court in K. Puttaswamy judgment, and a breach by the
government of the
same must be dealt with strictest possible measures, for the safety and security of
the citizens.
67. Khushwant was dissatisfied with the performance of the government in areas of
primary sector
development. He wanted the government to topple no matter how or what. He wanted to
have a large
group of people to support him in his motive. He organised a big rally which saw
participation in large
numbers. In his speech at the rally, he asked the people to join him in his motive
of downfall of the
irresponsive government, even if it meant widespread mayhem. He even made promises
to provide
weapons to everybody who was interested. Lot of people even secured weapons post
the speech, but no
real movement could follow. On the basis of the speech, Khushwant was arrested
under charges of
sedition. Decide?
(a) Khushwant’s arrest is invalid according to Supreme Court’s Vinod Dua judgment.
(b) Khushwant’s arrest is valid as he tried to incite violence through words and
actions, as required under
Section 124A.

. Page 17 of 36
(c) Khushwant’s arrest is invalid as he didn’t want to resort to violence as the
only means, and was open
to consider non-violent options as well.
(d) Khushwant’s arrest is valid since it is sedition to try and topple the
government through undemocratic
means.
68. Shivan was a doctor. His ideology did not always match the government’s behind
various public health
bills, but he was content with the work that the government was doing. A senior
doctor of the same
hospital that Shivan used to work in, Yaman, talked to Shivan trying to sway him
against the
government’s efforts. Shivan’s interest in the topic grew, and taking advantage of
it, Yaman schooled
Shivan to teach the government a lesson by signifying how many human lives which
could be saved are
going to get lost due to government’s recent bill on public health management.
Shivan immediately, on
Yaman’s feeding, organised a crowd and asked them to unite to oppose the government
on the bill. At
first the opposition was peaceful but significant, but when their issues were not
heard, Yaman asked
Shivan to go violent. Shivan asked his followers, and at once violent protests
started. Shivan was
arrested under Section 124A of the Indian Penal Code. Decide?
(a) Shivan is not liable under Section 124A of the IPC since he was only doing what
Yaman was asking
him to.
(b) Shivan is not liable as he legitimately wished to save public lives, as
demanded by his profession of
being a doctor, and protest on his part was needed.
(c) Yaman is liable since he brainwashed Shivan and used him to incite violence.
(d) Shivan is liable as it was his speeches and requests which actually incited
violence.
69. Hakim was the leader of a significant cult in India. Watching the people of his
cult getting oppressed and
ignored, he wanted a separate state carved out from India, where only the people of
his cult lived. The
population of the cult being large enough to support a nation, he tried to justify
and validate his claim by
citing Pakistan’s example. He incited the people of his cult to fight for their
rights and die until they get a
separate state for them. Hakim was arrested under Section 124A of the IPC. His
arrest expanded and
intensified the protests even more. Decide?
(a) Hakim is not liable since Pakistan could also separate from India on the same
basis Hakim’s cult
wants to.
(b) Hakim is not liable as the protests intensified after his arrest. Therefore,
the policemen who arrested
him rather are responsible for expanding and intensifying the protests.
(c) Hakim is liable for sedition as his words incited violence in the followers of
his cult, irrespective of the
justifications he gave.
(d) Whole of Hakim’s cult is liable for sedition because all of them had a
separatist tendencies, and were
incited to commit violence to death.
Passage (Q.70-Q.75): Like it or not, we are all part of various WhatsApp groups. In
fact, nearly all of us
are in more groups than we’d prefer. From project groups to groups for every legal
file that comes into
office, or the random temporary group that was created for a brief discussion.
However, things get serious when we start these groups or are assigned the hallowed
privilege of being
'The Group Admin’. Is being an ‘Admin’ really a privilege, or a potential millstone
around one’s neck? In
other words, what are the implications of this? Can there be, for instance, legal
consequences of
(mal)administering a group or failing to moderate messages/content?
According to the Bombay High Court -
A group administrator cannot be held vicariously liable for an act of a member of
the group, who posts
objectionable content including any defamatory content, unless it is shown that
there was common
intention or pre-arranged plan acting in concert pursuant to such plan by such
member of a WhatsApp
group and the administrator.”
However, in many cases, the current admin may not even be the person who created
the group. For
instance, Junaid Khan, an admin of a WhatsApp group was arrested and charged with
sedition as well as
other offences under the IT Act after a member of the group raised a complaint with
the competent
authorities. However, Junaid had not created the group but became the admin by
default when the

. Page 18 of 36
original creator exited. Hence, Bombay High Court’s interpretation of the law must
come to the rescue of
those like Junaid, who are victims of a negligent police force.
[Extract from Bar and Bench, Bharat Chugh, Siddharth Shivakuma
70. Mr. A created a WhatsApp group of 15 members. After taking prior permission
from A, Mr. B, a member
of the group posted a picture. This picture clearly comes under the ambit of
objectionable content but
others members just comment on the picture. Choose from the most appropriate option

(a) Mr. B is not liable as he did not offend any members of the group
(b) Mr. A is vicariously liable as his consent was taken before posting such
objectionable content
(c) Mr. A can be held vicariously liable after a member of the group reports such
objectionable content
(d) Mr. A cannot be held vicariously liable as the objectionable content did not
offend other members of
the group.
71. Mr. X created a WhatsApp group of 12 members including Mr. Q on July 15, 2021.
On July 19, 2021 after
a heated argument with Mr. H, Mr. X left the group. Due to this, Mr. Q was
automatically made the admin
of the group and he did not have the knowledge of the same. On July 20, 2021 Mr. H
spoke to Mr. Q on
personal chat and took his permission to post a defamatory picture of Mr. X on the
group. Choose from
the most appropriate option –
(a) As Mr. H has posted defamatory content, he shall be held liable under
defamation
(b) Mr. Q being the admin of the group shall be held vicariously liable for the
defamatory content as his
consent was taken before uploading such content to the group.
(c) Mr. Q shall not be held vicariously liable as he was not aware of him being the
admin of the group
(d) Mr. Q shall not be held vicariously liable as Mr. X is not a part of the group
and Mr. H posted such
content due to sudden aggression.
72. Which of the inferences from the passage is the most appropriate?
(a) The admin of a WhatsApp group cannot be held liable for every message as this
is violating his Right
to Privacy.
(b) The admin of a WhatsApp group cannot be held liable for every message as it is
not reasonable to
expect the admin to check every message on the group.
(c) The admin of a WhatsApp group cannot be held liable for every message as some
admins become
the admins without prior notice and cannot be held liable for the messages.
(d) The admin of a WhatsApp group cannot be held liable for every message as even
though they might
consent to the messages on the group, the message is being sent by another member
who holds the
accountability for the message.
73. Ms. G on July 1, 2021 creates a WhatsApp group. After a few days he informs
everyone in the group and
personally texts Mr. U that he will be made the only admin of the group to which he
replies. Mr. T, a
member of the group posts an objectionable audio in the group. Choose from the most
appropriate option
(a) Ms. G being the creator of the group shall be vicariously liable.
(b) Mr. U being the current admin of the group shall be vicariously liable.
(c) No one shall be held vicariously liable as audio does not fall under the ambit
of objectionable content.
(d) Mr. U shall not be held vicariously liable as Mr. T had not taken consent of
the admin before posting
the audio.
74. Ms. X creates a WhatsApp group of 3 college juniors assigned to her as mentees.
Ms. X is the admin of
the group. One day in the college canteen, Mr. F a member of the WhatsApp group
takes a picture of Ms.
H and edits it resulting it in being defamatory. He shows the edited picture to Ms.
X and she laughs and
then suggests Mr. F to share this picture with the other members of the group. Ms.
G, another member of
the group sees this picture and shows it to Ms. H. Ms. H demands the competent
authorities for justice.
Choose from the most appropriate option –
(a) As the picture was shown to Ms. X in the canteen, she shall not be held
vicariously liable.

. Page 19 of 36
(b) Ms. X is the admin of the group and the defamatory content was posted after her
consent and hence
she shall be held vicariously liable.
(c) Ms. X is the admin of the WhatsApp group and she was part of the pre-arranged
plan to share the
defamatory picture on the group. She shall be held vicariously liable.
(d) Both b and c.
75. What is the reason for the Bombay High Court needing to come to the rescue of
admins like Junaid?
(a) He shall not be held vicariously liable as he did not join the group to become
an admin.
(b) He shall not be held vicariously liable as he did not create the group and
cannot be held for any
objectionable content.
(c) He shall not be held vicariously liable as he cannot be expected to put an eye
on all the messages
being sent on the group and be held for any objectionable content after being an
admin by default
and without knowledge.
(d) All of the above.
Passage (Q.76-Q.81): The Calcutta HC recently had the opportunity to extensively
define what
constitutes a "commercial dispute" as contemplated under Section 2(1)(c) of the
Commercial Courts Act,
2015 (2015 Act). The issue in consideration before the Court was whether a plea
filed before the
Commercial Division of the High Court should be tried under the provisions of the
2015 Act or be
adjudicated upon as a regular suit. Herein the court observed that these commercial
disputes are only
capable of being adjudicated by the commercial courts possessing such jurisdiction
as necessary.
The Court referred to 2015 Act wherein a 'commercial dispute' has been defined to
be a dispute arising
out of 'ordinary transactions of merchants, bankers, financiers and traders such as
those relating to
mercantile documents, including enforcement and interpretation of such documents'.
"The commercial purpose would generally mean a transaction by which a person's
commercial/economic
interests may be advanced and would result in an economic benefit to that person.
It would not include
an agreement where profit-making is an incidental outcome of the transaction or may
happen by
accident". Thus, the Court ruled that the 'commercial flavour' of a 'hand loan' is
lost when the money is
loaned under informal terms resulting in uncertainty with regards to recovery of
the amount loaned.
[Extracted, with edits and revisions, from “What Constitutes A 'Commercial Dispute'
Under Commercial
Courts Act, 2015? Calcutta High Court Explains” Aaratrika Bhaumik, LiveLaw, 24
August 2021;
76. With regards to the above-mentioned passage, reason whether a loan given to a
friend would amount to
the subject matter of a commercial dispute in the following scenarios.
(a) Only if the loan carries a return date and interest.
(b) Only if there are mercantile documents provided.
(c) All of the above.
(d) None of the above.
77. Sabharwal enterprises have taken a loan from an individual lender informally so
as to exclude the same
from their financial books. However, upon defaulting on its repayment, the matter
was taken to the
respective courts having jurisdiction of considering commercial disputes. Will the
dispute be adjudicated
therein?
(a) No, as the dispute is not of a commercial nature.
(b) Yes, as the dispute involves two parties of a commercial nature.
(c) Yes, as the parties involved, may be informally involved, but the corpus of the
dispute is of a
commercial nature.
(d) No, as the conditions for the matter to be entertained by commercial courts
have not been satisfied.

. Page 20 of 36
78. Dash is a merchant banker in Gold’s Bank. Lily is his stepdaughter and has
recently been looking for
some business opportunities to invest in and acquire proprietorship of some small
up and coming
businesses. For this, Lily approaches her stepfather to acquire a loan for
investment and start-up/seed
funding purposes and gets the same to the tune of 30 lakhs. Would this agreement be
considered to be
of a commercial nature?
(a) No, as the subject matter of the transaction is not commercial.
(b) No, as the procurement of the loan was not done for a commercial purpose.
(c) No, as the parties are not involved in a commercial transaction.
(d) Yes, as all the conditions for it to be termed a commercial dispute have been
fulfilled.
79. In a loan arrangement between a construction contractor and the Bank of India,
a dispute arose between
the two business parties with regards to the quality of the products being supplied
by the hardware store
owner. As a result of the same the construction contractor sued the store owner for
repayment. The store
owner filed a counter suit alleging mental harassment on account of frivolous
suits. Can the dispute
between the two be termed commercial?
(a) No, as the parties are not in a commercial relationship.
(b) Yes, as the parties are of a commercial nature.
(c) No, as the parties are not of a commercial nature.
(d) Yes, as there existed a mercantile document.
80. Bank of Dena and Vidyut Mallwa entered into a loan agreement as the loanee
being the CEO of an
Investment Banking company and the loaner, being the bank in question. Vidyut
procured the loan and
used the same to invest it in a mutual fund scheme being offered by his own
company. This led to an
insider trading suit being filed against him and action being taken against him and
the bank by SEBI. The
same was contested by the bank as being a third party to the dispute. Is the bank
liable in this scenario?
(a) Yes, as the bank was a valid party in the commercial dispute.
(b) No, as the bank was a third party to the dispute in question.
(c) Yes, as the dispute was of a commercial nature.
(d) None of the above.
81. In the above question, had Mallwa procured a secured loan by proffering a
charge as against his
investment portfolio, and the same had been mentioned in the mercantile documents
prepared by the
bank, would the bank be considered as a party in the matter initiated by SEBI?
(a) Yes, as the bank was a secured creditor and had as much interest in the matter
as Mallwa.
(b) No, as the bank as a mere secured creditor and not a party.
(c) No, as SEBI cannot adjudicate commercial disputes.
(d) Yes, as SEBI adjudicates and initiates commercial hearings.
Passage (Q.82-Q.87): Whether issuance of a legal notice and filing of complaint
case by the petitioner
would amount to ‘abetment’ punishable under Section 306 of IPC?
A person abets the doing of a thing if he firstly, instigates any person to do that
thing; or secondly,
engages with one or more other person or persons in any conspiracy for the doing of
that thing, if an act
or illegal omission takes place in pursuance of that conspiracy, and in order to
the doing of that thing; or
thirdly, intentionally aids, by any act or illegal omission, the doing of that
thing.
As in Gurcharan Singh v. State of Punjab, “It is thus manifests that the offence
punishable is one of
abetment of the commission of suicide by any person, predicating existence of a
nexus between the two,
abetment being the propelling causative factor. To constitute abetment, the
intention and involvement of
the accused to aid or instigate the commission of suicide is imperative. Remoteness
of the culpable acts
or omissions rooted in the intention of the accused to actualize the suicide would
fall short as well of the
offence of abetment essential to attract the punitive mandate of Section 306 IPC.”
The Bench stated that the filing of a criminal complaint by the petitioner was his
legal recourse. It cannot
be said that by filing a criminal complaint against the deceased, petitioner had
the mens rea to instigate
the deceased to commit suicide.

. Page 21 of 36
[Extracted, with edits and revisions, from “Issuing a legal notice and filing a
complaint case against
someone, would that amount to abetment of suicide? Del HC deciphers [Detailed
Analysis]” Devika
Sharma, SCCOnline Blog, 23 August 2021
82. What does Section 306 of the Indian Penal Code Primarily deal with?
(a) Punishment for Suicide.
(b) Attempt to Suicide.
(c) Abetment of Suicide.
(d) Punishment of Abetment of Suicide.
83. Maria is great friends with Rose. However eventually, they soon begin fighting
over a trivial issue and
Maria begins to harbor a deep resentment for Rose. The dislike and resentment
escalate to an extent
that Marie begins to make elaborate plans to kill Rose. When Rose becomes severely
ill, Maria still hopes
that she dies, and suggests ways as to how one could kill her in the hospital. Can
it be said that Maria
possesses mens rea?
(a) No, as Maria merely prepared for the murder, which is not an offense.
(b) Yes, as she possessed the requisite intention and guilty mind.
(c) No, as Rose ultimately died of natural causes and not by the actions of Maria.
(d) Yes, as preparation of a crime is also punishable as an attempt.
84. Ram and Shyam are twin brothers and are extremely close to each other. Both of
them aspire to become
IAS officers, and thus are preparing for the UPSC exam. Ram is brilliant in his
studies, and all the
faculties in their coaching are very sure of his selection. This makes Shyam
jealous, who is not so bright
and fails to achieve such results despite a lot of hard work. When the results of
their preliminary exam
arrived, Shyam gets selected and Ram doesn’t. Absolutely dejected by the same, Ram
commits suicide.
Would Shyam be liable under Section 306?
(a) Shyam would be liable for abetment of suicide as there was the commission of an
act on his part that
resulted in Ram’s death.
(b) Shyam would be liable for abetment as he could also have broken the news
lightly which would not
have resulted in such extreme consequences.
(c) Shyam would not be liable for abetment as there was no act or omission on his
part that led to Ram’s
suicide.
(d) Both (a) & (b) are correct.
85. In the above-mentioned factual scenario, had two days that the results were
announced, Bhanu, a tea#shop owner near Ram’s coaching, had made fun of his shirt;
and later after his unfortunate suicide, if
Ram’s family alleges that Bhanu’s action pushed Ram to the edge to commit suicide
and thus files a case
under Section 306 of the IPC, would Bhanu be liable?
(a) Yes, as Bhanu committed an act that unfortunately resulted in Ram’s death, thus
making him liable
for abetment.
(b) No, as Bhanu did not possess the required intention for abetment of Ram’s
suicide.
(c) No, as the action was too remote and not the driving cause for behind Ram’s
commission of suicide.
(d) Both (b) & (c) are correct.
86. Had in the same factual scenario, it had been later found out that Shyam, along
with his good friend Hari,
had intentionally altered the sheet that contained Ram’s roll number so that a
false result would be
displayed, knowing very well that such an act would lead to Ram taking extreme
measures vis-à-vis his
own life; would the two of them be liable for abetment to suicide?
(a) Yes, as the abetment was the propelling causative factor for Ram’s suicide.
(b) Yes, as there was intention and involvement of both in the suicide, and both
Hari and Shyam had
conspired to commit an offense under S. 306.
(c) No, as Hari did not possess the intention to instigate Ram to suicide, hence
only Shyam will be liable.
(d) Both (a) & (b).

. Page 22 of 36
87. Mr. X is planning to murder Mr. Y owing to a monetary dispute. However, when he
reaches to commit the
offense, he realizes that he forgot the murder weapon. In Mr. Y’s residence, his
servant Z who had
scores of his own to settle, hands him a kitchen knife. Mr. Y then commits the
murder of Mr. Y. Would Z
be also liable of an offense?
(a) Z would be liable of abetment as he intentionally aided, by any act, the
commission of the offense.
(b) Z would be liable of abetment as he conspired with Mr. X to commit an offense,
which later was also
executed.
(c) Z would be liable of abetment as he instigated Mr. X to commit the murder of
Mr. Y.
(d) None of the above.
Passage (Q.88-Q.91): The term “anticipatory breach of contract” refers to the
contract that has been
repudiated even before the contract’s execution has begun. In plain terms, it means
that a contract’s
promisor does not plan to fulfil his or her duties under the contract’s provisions.
It occurs where a party to
a contract is incapable of performing or unwilling to fulfil the contract even
before the contract’s due
date.
An anticipatory breach occurs when:
• The defaulting party denies the other party unconditionally and positively:
Rejection must be
straightforward, clear, and directed at the innocent party. Ambiguous or qualified
refusal is not
enough. Nevertheless, an expression of doubt may indicate a prospective failure to
fulfil the contract,
in which case the non-breaching party may suspend its performance and request
assurance from the
offender.
• The violator is unable to do so due to a certain action. Actions are as important
as words when
repudiating contracts. If the voluntary actions of the defaulting party prevent him
or her from fulfilling
the contractual obligations, it is considered a contract repudiation.
• The contract is transferred to another person. If the contract involves selling
property, it is repudiated
when the property is transferred to a third party.
In such instances, the disgruntled or harmed party may sue for breach of contract
damages. The harmed
party can sue right once or wait until the act is scheduled to be performed.
[Extracted with revisions from 'Post Views: 91 Do Indian courts follow the doctrine
of anticipatory breach
of contract' by horonya Banerjee, published 4 July 2021 on ipleaders blog
88. Singham signed a contract to sell 500 metres of organic silk yarn to Zara by
11th July. On 7th July,
Singham wrote to Zara to say that he has changed his mind and declined his
services. Zara was counting
on Singham’s delivery in order to produce their Spring/Summer Collection in time.
Can Zara sue Singham
on 8th July for breach of contract?
(a) Yes, because Singham repudiated his contract with Zara just after its
execution.
(b) Yes, because Singham expressly rejected to perform his obligation of selling
500 metres of organic
silk yarn to Zara.
(c) No, because Singham was incapable of performing his part of the bargain.
(d) None of the above.
89. Mohanlal pledged to assign all of his interest in the lease of his shop
‘Haldiram sweets’ to Hiralal within
five years after the date of his pledge for the value of Rs. 1,00,000/-. Before the
end of five years,
Mohanlal’s son Mohanlal Jr. attained majority. Mohanlal decided that his son would
continue his sweets
business after him. Consequently, he assigned his interest in the lease to Mohanlal
Jr. Can this be
termed as an anticipatory breach of contract?
(a) Yes, because the pledge was not repudiated by Mohanlal before the pledge’s
execution began.
(b) No, because the pledge was not repudiated by Mohanlal before the pledge’s
execution began.
(c) Yes, because the pledge was repudiated by Mohanlal even before the pledge’s
execution began.
(d) No, because the pledge was repudiated by Mohanlal even before the pledge’s
execution began.

. Page 23 of 36
90. U and V entered into a contract for carriage of certain goods by sea. It was
decided that U would deliver
V’s goods to London via his ship ‘Titanic’. As an essential part of the bargain, V
would pay 20% of the
carriage amount to U as advance and the remaining 80% would be paid on delivery. On
the scheduled
date of loading the goods, V came to the port along with his goods for their
onloading. U refused the
advance payment. Has U committed an anticipatory breach of contract?
(a) No, because U did not repudiate the contract before its execution by refusing
the advance payment.
(b) Yes, because U was incapable of performing the contract which is why he refused
the advance
payment.
(c) No, because refusing the advance payment before execution of the contract does
not lead to a
breach of contract.
(d) Yes, because U repudiated the contract before its execution by refusing the
advance payment.
91. Mehul Choksi was a renowned businessman who was involved in the trade of
diamonds. He took a loan
of Rs. 5 crores from SBI Bank. He was supposed to repay his bank loan by March
2020. However, just
before the due date he became bankrupt, which made it impossible for him to pay
back his loThis is
because he recklessly spent his money in throwing lavish parties instead of
actually using it to further his
business. Can this be treated as a repudiation of the loan agreement with SBI Bank?
(a) Yes, because Mehul did not plan to fulfil his loan obligation at the time of
signing the loan agreement.
(b) Yes, because Mehul is incapable of repaying his loan obligation.
(c) Yes, because Mehul unconditionally and positively refused to repay his loan
obligation.
(d) None of the above
Passage (Q.92-Q.97): A Consumer Court in Bengaluru, while dismissing a complaint
filed by a 35-year#old man against an online matrimonial service provider, said
that a matrimonial portal can only facilitate
marriage and not guarantee marriage.
The bench said:
"It is an admitted fact by the defendant that the complainant obtained its
membership by paying
Rs.59,180. A copy of the cheque is produced. The email document contains the terms
and conditions,
wherein the validity of the service is for 88 days, and the profile shared is from
its website of
matrimonet.com limited. It does not guarantee a positive response at all times and
does not guarantee a
marriage within the service provided."
The complainant claimed that he had been mentally harassed and made to suffer
financially. The service
of the defendant is unscrupulous, dishonest, and deviated from the assurance given
at the time of giving
the membership.
Further, it was argued that the complainant is provided with 14 profiles, and out
of that, the complainant
accepted two profiles. It has provided the services as per the package opted by the
complainant, and
hence complainant cannot claim a refund of the amount, and the complaint is not
maintainable and is
bound by the terms and conditions of the agreement entered into, and its service is
only to facilitate the
marriage and do not guarantee the marriage. The complaint is frivolous, vexatious
with ulterior motives,
and hence, prayed the forum to dismiss the complaint.
92. Sarvesh was repeatedly suggested by his relatives to try the matrimonet
website. He finally created an
account on March 5 and bought the membership for six months for a total amount of
Rs. 30,000.
However, after joining, he wasn't provided any profiles due to his own choices. He
files a complaint
against the website. Is the complaint, by its nature, vexatious?
(a) No, because the matrimonial websites have been made to provide matching
profiles.
(b) Yes, because he did not receive any profiles due to his choices and not due to
any fault of the
website.
(c) No, because the website should have a matching profile encompassing all choices
made by a
member.
(d) Yes, because he was not guaranteed a profile by the website.

. Page 24 of 36
93. Sarvesh, after joining the website, did not get any profile due to the website
being down for 20 days out
of the 60-day package. He files a complaint and claims a refund. Would his claim
stand?
(a) Yes, because he suffered a loss due to the fault of the website and not of his
own.
(b) No, because he still has 40 days left for the membership to end.
(c) Yes, because he could have received matching profiles in the days that the
website was down.
(d) No, because there was no loss since there was no guarantee that he would have
received a matching
profile in the days when the website was down.
94. Since Sarvesh had received so many positive reviews from his relatives, he was
positive that he would
be getting a match before his membership ends. So, upon signing up on the website,
he made a list of
things that are required in a wedding and bought all of them in advance. He did not
get any matches. Can
he claim for refund and reimbursement for the things he bought?
(a) Yes, because he did not receive any matches.
(b) No, because he should have been more prudent before buying all the goods for
the wedding before
even receiving a matching profile.
(c) Yes, because he went through financial loss when he signed up for the website
and did not receive
matching profiles as well as for all the goods he bought in furtherance of the
positive reviews.
(d) No, because the website never guaranteed any matching profiles, and buying all
the goods in
advance was his personal decision.
95. The Matrimonial website guaranteed marriage profiles within 120 days of taking
membership. However,
Rahul received no matching profiles in the whole 80 day plan membership bought by
him. Can he claim a
refund?
(a) Yes, because he did not receive any matches before his membership ended.
(b) No, because he cannot claim for refund in such cases since it is a pure case of
luck.
(c) Yes, because he went through financial loss when he signed up for 80 days and
did not receive a
matching profile.
(d) No, because the website guaranteed matching profiles within 120 days while
Rahul had signed up for
only 80 days.
96. Abhishek signed up on a matrimonial website. He got a matching profile. His
proposal was accepted
online, but later the girl declined the proposal in real life. Can Abhishek claim
for a refund?
(a) Yes, because the Company has breached its terms of providing matching profiles.
(b) No, because there was no breach by the Company.
(c) Yes, because the proposal was declined in real life.
(d) No, because Abhishek should have been more careful before completely believing
that his proposal
has been accepted.
97. Rishi, after signing up for a matrimonial website, started receiving matching
profiles within minutes, Later
he found a profile that he liked and went ahead with it. After the talks for
marriage were in the final
stages, he found out that the profile that he was matched with was a fake profile
matching by a bot
created by developers. Can he claim a refund?
(a) Yes, because Abhishek lost a lot of time and faced mental harassment because of
the fake profile.
(b) No, because Abhishek should have been more diligent before going ahead with the
fake profile.
(c) Yes, because Abhishek signed up to match with real profiles.
(d) No, because there were real profiles on the website that Abhishek could have
matched with, and the
fake profiles were a chance of luck.

. Page 25 of 36
Passage (Q.98-Q.102): In order to claim the benefit under S. 84 IPC (Act of a
person of unsound mind),
it must be proved that at the time of doing the act by reason of unsoundness of
mind the accused is
incapable of knowing the nature of act or he is incapable of understanding that,
what he is doing is either
wrong or contrary to law. It is to be noted that every mental illness is not ipso
facto exempted from
criminal responsibility. It is also well settled that there is a distinction
between legal insanity and medical
insanity.
S.84 lays down the legal test of responsibility in cases of alleged unsoundness of
mind. There is no
definition of "unsoundness of mind" in the IPC. Courts have, however, mainly
treated this expression as
equivalent to insanity. But the term "insanity" itself has no precise definition.
It is a term used to describe
varying degrees of mental disorder. So, every person, who is mentally diseased, is
not ipso facto
exempted from criminal responsibility. A distinction is to be made between legal
insanity and medical
insanity.
(Extracted with requisite revisions and edits from “Varghese v. State of Kerala
Crime No. 173/ 2015
decided on February 1st, 2021 at https://indiankanoon.org/doc/113876583/)
98. Harleen Quinzel was a famous psychiatrist in Gotham City and rendering her
services in the Gotham
Prison. She fell in love with her patient who was popularly known as ‘The Joker’.
Due to her love for him,
she on his request arranged certain weapons including a machine gun in order to
help him escape. She
arranged the weapons and hid them at a place where Joker and his cadre of goons
could easily fetch
them. The plan was successful, but unfortunately for Ms. Quinzel, she was left
alone while ‘the Joker’
broke out of the prison. She was caught by the police later with Joker still at
large. A charge was brought
against her for abetting Joker’s escape. The Advocate for Mr. Quinzel produced
certain reports by a
renowned Psychiatric Institution which has proved that when a person falls in love
with another certain
hormones are released which affect the mental acumen of the person and makes
him/her incapable of
making rational choices which s/he would otherwise have been able to. Thus, the
defense of insanity was
claimed before the Court. Based on this scenario and arguments, can Section 84 be
made applicable on
her?
(a) Yes, according to the report by the renowned Psychiatric Institution it is
proven that the release of the
certain hormone made Harleen incapable of understanding the nature of the act.
Hence, she will be
protected under Section 84 IPC.
(b) No, she was able to understand the nature of her acts and was capable of
understanding them and
thus shall not be protected under Section 84 IPC.
(c) Yes, Harleen Quinzel was medically and legally insane and hence Section 84 will
come to her rescue
and will not be liable for her acts.
(d) No, by the acts of Harleen she is neither medically insane nor legally unsound
and therefore she will
not be protected by the said section.
99. Two years passed but Joker was also unable to forget Ms. Quinzel. However, he
suspected if Ms.
Quinzel would have changed after his betrayal. So, he called her in an abandoned
acid factory and asked
if she would jump in that acid for him. Ms. Quinzel was still in love with him
happily obliged. However, this
led to a depraved and degenerative mental condition where she was unable to process
certain human
emotions like empathy. However, this made her the best killing machine for Joker
who used her to do his
dirty works. During one such instance, while doing her usual job, Ms. Quinzel
killed Robin, the sidekick of
Batman, their archenemy. After killing, she hid the weapon under the ground and
removed all the
evidence which could lead Batman to her and created a scenario which would make the
murder appear
as an accidental death. However, she was caught later by Batman who figured out her
role in the entire
scenario. Joker who also represented Quinzel before the Court claimed her
degenerative mental
condition due to which she would be covered under insanity defense and would not be
liable for her acts.
Is the argument of Joker correct?
(a) Yes, Ms. Quinzel suffered from a depraved and degenerative mental condition and
hence would be
protected under Section 84 IPC.
(b) Yes, Ms. Quinzel was unable to comprehend the nature and consequences of her
action and would
thus not be liable for her act.

. Page 26 of 36
(c) No, Ms. Quinzel was able to comprehend the nature of her act and was able to
understand it and
hence would not be protected under the said Section.
(d) No, Ms. Quinzel was neither medically unfit nor legally unsound but her acts
were motivated by
malice and were illegal.
100. The relationship of Ms. Quinzel and Joker was short-lived and they soon parted
ways. In order to
announce her new found singlehood, she burnt the chemical factory where according
to her everything
began and to finally end things with Joker. At this point of time, she was
interrogated by the police, where
she responded to everything and why she committed the act and is even not afraid of
whatever the
consequences might be. Medical Examination was also conducted where she was
declared a lunatic and
found that she had a relapse of mental illness. Is she eligible for the defence of
insanity under Section 84
of IPC?
(a) No, her mental condition although diagnosed as lunatic is not violent but the
act involved were violent
in nature thus signifying that she was fully aware about the consequences of her
action.
(b) Yes, she has been diagnosed with lunacy as per the medical examination
immediately conducted.
(c) No, her examination and interrogation before the police indicates that she was
able to understand the
nature of her action and their consequences as well and would not be protected
under Section 84
IPC.
(d) Yes, there appears an intersection between medical lunacy and legal insanity in
the present case and
hence Harley can avail the benefit under Section 84 IPC.
101. A prisoner popularly known as Canary got into a fight with her inmate and
after striking her with a sharp
stone ran towards her cell and then put her bed against the cell gate so that she
cannot be caught and
punished. She also threw whatever the belongings were inside the cell to disperse
the prison authorities.
She had earlier been diagnosed with schizophrenia and hence it was the plea of her
advocate that she
should be excused from the offence. Does her act falls under the defence of Section
84?
(a) Yes, schizophrenia is a mental illness which made Canary incapable of
understanding her acts and
thus she can claim protection under Section 84.
(b) No, Canary was able to understanding the nature of her actions as can be
understood from her
actions immediately post the commission of the offence.
(c) Yes, insanity has no precise definition and hence the very fact that Canary
suffered from
schizophrenia will suffice to claim benefits under the said section.
(d) No, the gravity of the offence cannot be ignored while providing the protection
under the claimed
Section.
102. Canary had a family history of schizophrenia and there was absence of motive
also for stabbing her
cellmate. Is she now eligible to claim the defence under Section 84 IPC?
(a) Yes, the absence of motive indicates the degenerative mental health of Canary
and she would thus
be entitled to claim benefits under the said Section.
(b) No, stabbing of the cellmate amply indicates that she possessed the mental
soundness sufficient
enough to commit the crime.
(c) Yes, the family history shall be taken as a determinative factor which would
indicate that her act was
a result of poor mental health.
(d) No, medical unsoundness has to be distinguished from legal unsoundness and thus
despite the lack
of motive or having a family history of schizophrenia will not influence the
protection under Section 84
IPC.

. Page 27 of 36
Passage (Q.103-Q.105): A student who was rusticated from Khwaja Moinuddin Chishti
Urdu, Arabi#Farsi University for the reason that he was attempting to organize
protests against the Citizenship
Amendment Act, 2019, has approached the Supreme Court assailing the Allahabad High
Court's order
affirming University's decision. Interim relief in the form of staying the High
Court's order dated July 23,
2021, along with permission to allow the student to complete his last semester and
to complete
graduation has been sought in the Special Leave Petition. It has been argued that
the order of rustication
has been passed without giving the BA student a proper opportunity of hearing and
that even the
Chancellor has violated the principle of morality by not personally hearing the
student. It is further averred
that the High Court ought to have considered the grievance of the petitioner on
merits, rather than
summarily upholding the rustication order. "The High Court has committed an error
in not appreciating
the reason and dismissing the Writ Petition on the first day itself," plea further
adds. Relying on Ranjit
Thakur v. Union of India (1987) 4 SCC 611 in which the Top Court had held that a
judgment which is the
result of bias or want of impartiality is a nullity and the trial "coram non-
judice", and that the test of real
likelihood of bias is whether a reasonable person, in possession of relevant
information, would have
thought that bias was likely, the petitioner submits that the university never
provided any show cause or
charge sheet.
Source Name - Live Law
103. Protests were organized by H the leader of youth congress which was peaceful
in nature and it had the
consent of the college’s administration to carry out a peaceful protest against the
administration. One of
the members of the group felt aggrieved as he was not given the chance of public
speaking and thus he
conveyed the message to the administration that H is planning to carry out
vandalism next morning in the
college campus. The administration rusticated him from the college with immediate
effect and he was
arrested for the same. In the light of the above passage, opt out the most accurate
option.
(a) Protest is a fundamental right of every citizen but H must be arrested because
he was planning
vandalism for next morning.
(b) H should have been heard by the college administration, his own version to find
out whether the
same was true or not.
(c) H has been falsely implicated by his fellow member as he was humiliated and
made fun of him during
the protest and thus as an instance of revenge, he had done the same to H.
(d) H should have allowed his fellow member to speak, as he was exercising his
unfettered power of
youth congress leader.
104. "The High Court has committed an error in not appreciating the reason and
dismissing the Writ Petition
on the first day itself". Construing to the line of the above passage, opt out the
appropriate interpretation
of the same.
(a) The high court should have followed the doctrine of ‘audi alteram partem’.
(b) The high court should have dismissed the appeal after listening to the
petitioner.
(c) The high court should have dismissed the appeal on the day other than the first
day itself.
(d) The high court should not have used it unfettered power in an unethical manner.
105. Construing to the above passage, what is the main idea of the passage that is
being tried to be
conveyed?
(a) The high court should have listened to the appeal of the petitioner and then
dismiss it instead on the
first day itself.
(b) The high court has used its unfettered power to dismiss the appeal of the
petitioner without listening
him on the first day itself.
(c) The college administration has done unethical conduct by not listening to the
petitioner’s version and
rusticating him with immediate effect.
(d) The college was justified in rusticating the petitioner immediately before it
became violent from
peaceful protest.
. Page 28 of 36
SECTION - D: LOGICAL REASONING
Passage (Q.106-Q.110): The Plastic Waste Management Amendment (PWMA) Rules notified
by the
Centre recently acknowledge the gravity of pollution caused by plastic articles of
everyday use,
particularly those that have no utility beyond a few minutes or hours. Under the
new rules, the
manufacture, sale and use of some single-use goods made with plastic, polystyrene,
and expanded
polystyrene, such as earbuds, plates, cups, glasses, cutlery, wrapping and packing
films, are prohibited
from July 1 next year. The decisions follow recommendations made by an expert group
constituted by the
Department of Chemicals and Petrochemicals. In 2018, India won praise globally for
asserting on World
Environment Day that it would eliminate all single-use plastic by 2022, a theme
that the Prime Minister
has stressed more than once. Yet, policy coherence to achieve the goal has been
lacking. The Central
Pollution Control Board has reported that 22 States have, in the past, announced a
ban on single-use
plastic, but this has had little impact on the crisis of waste choking waterways
and being transported to
the oceans.
At about 34 lakh tons generated in 2019-20, India has a staggering annual volume of
plastic waste, of
which only about 60% is recycled. What is more, a recent study of the top 100
global producers of
polymers that culminate in plastic waste found six of them based in India. It is
unsurprising, therefore, that
in spite of the staggering problem, policymakers have been treading on eggshells.
The international view
is changing, however, and support for a UN Plastic Treaty is growing; the majority
of G7 countries which
is a major organization too are supportive of cleaning up the oceans through a
charter in the interests of
human wellbeing and environmental integrity. India’s policies on environmental
regulation are discordant,
lofty on intent but feeble on outcomes, and plastic waste is no different.
Considerable amounts of plastic
waste cannot be recycled because of lack of segregation, leading to incineration,
while mixing newer
types of compostable plastic will confound the problem. Patchy regulation has led
to prohibited plastic
moving across State borders. Now that the Centre has adopted a broad ban, further
pollution must end.
Microplastic is already found in the food chain, and governments must act
responsibly to stop the
scourge.
[Extracted with edits and revisions from The Hindu]
106. Which of the following could be an explanation to the fact that India has not
been able to eliminate single
use plastic yet?
I. A majority of Indian states have banned the manufacture of single-use plastics.
II. Implementation of the rules against environmental protection has been shoddy
all across the country.
III. The general public does not comply with the government rules regarding the use
of plastics.
(a) I and II
(b) Only I
(c) Only III
(d) Both II and III
107. Which one of the following has been suggested by the author in the passage?
(a) India should ensure that its intentions for environmental protection are backed
by appropriate results.
(b) Presence of microplastics in the food chain is not a cause of concern for the
scientific community.
(c) Close to 20 Lakh tons of waste in India is not recycled annually and this is a
cause of concern.
(d) A UN Plastic treaty is a far-fetched goal, considering that it is not likely to
fulfill its high goals.
108. What does the author mean when he says that policy makers have been ‘treading
on eggshells?
(a) The policy makers are being too cautious so as to not offend someone.
(b) The policy makers are making decisions hurriedly.
(c) The policy makers are not consulting with experts before making a decision.
(d) The policy makers are not competent enough to make decisions.

. Page 29 of 36
109. The author indicates that ‘the findings of a recent study suggest that six of
the top 100 global producers
of polymers that culminate in plastic waste are based found in India’ is a huge
problem. Which among the
following, if true, could weaken the author’s arguments?
(a) The US and China individually have 13 of the top 100 global producers of
polymers that culminate in
plastic waste.
(b) There are 20 nations and 125 producers of polymers that culminate in plastic
waste around the world.
(c) The credibility of the research institute that conducted this study has been
called into question
multiple times in the recent past.
(d) Both (b) and (c)
110. Which among the following can be considered to be likely according to the
passage?
(a) If the government is strict on implementation of the PWMA rules, India will
eliminate the use of single#use goods by July 1 next year.
(b) The use of single-use goods would not be eliminated completely even if the
government implements
the rules strictly and the public complies.
(c) Both a and b
(d) Neither (a) nor (b)
Passage (Q.111-Q.114): The primary idea lies in Mill’s deceptively simple
conception of saying the best
way for one to find fulfillment was to consider, explore and engage with what kind
of life one wishes to
lead and act accordingly; but in order to do even that, one must have the freedom
to think broadly, to
explore and test as many ideas that are encountered as possible. The peculiar evil
of silencing
expression of an opinion is that it robs the human race. It takes from posterity,
as well as the existing
generation, and from those who dissent from the opinion even more than from those
who hold it. If the
opinion is right they are deprived of the opportunity of exchanging for truth: if
it is wrong they lose what is
almost as great a benefit the clearer perception and livelier impression of truth,
produced by its collision
with error.
Thus, if we engage (civilly) with controversial or offensive ideas, we'll either
discover we have been
wrong the whole time - which is, of course, beneficial to us - or we'll show the
dissenter he or she is
wrong - which is also beneficial, since more people will now benefit from having
more robust ideas.
111. Which one of the following best expresses the conclusion presented in the
argument?
(a) Silencing the expression of an opinion is robbing the human race.
(b) Silencing the expression of an opinion harms those who dissent more than those
who agree.
(c) Anyone who agrees with an opinion would not want to silence its expression.
(d) Gaining a clearer perception and livelier impression of truth is a great
benefit
112. What role does the statement “The peculiar evil of silencing expression of an
opinion is that it robs the
human race” plays in the above passage?
(a) It is the premise for establishing conclusion that if the opinion is right they
are deprived of the
opportunity of exchanging for truth.
(b) It is the proof that it will take away the fundamental right of the citizens.
(c) It is the premise of the argument that if the opinion is right they are
deprived of the opportunity of
exchanging for truth: if it is wrong the clearer perception and livelier Impression
of truth, produced by
its collision with error will be lost.
(d) None of the above.
113. Based on the information in the given passage, which of the following is most
likely to be true?
(a) People in general do not know how to listen.
(b) Person, who does not agree with the majority judgment, should remain silent.
(c) Freedom of expression leads to the political uprising in the country.
(d) Silencing a person does not help anyone, because these ideas are now denied to
everyone.

. Page 30 of 36
114. Which of the following statements will actually weaken the main idea of the
author in the passage?
(a) The freedom of speech and expression makes it easier for individuals to spread
false information and
outright lies, but then still pretend that this data is true.
(b) Person, who does not agree with the majority judgment, should remain silent.
(c) People in general do not know how to listen.
(d) All of the above.
Passage (Q.115-Q.119): This has been a topic for extensive research, majority of
which has found out
that if music is introduced at an early stage in a person’s life – say childhood,
that person will be a lot
calmer than others. A study conducted by the Trinity School of Music in London says
so on the basis of a
sample of 8,000 adults. The people who knew to play an instrument tended to react
much more calmly to
an adverse situation than the people who didn’t know how to play any instrument.
People who are avid
listeners of music tended to have lesser health conditions than people who were
averse to music, which
also proves the point. It can be said, music is the best booster for cognitive
skills than anything else.
The difference can be seen prominently. Music is a must in the curriculum of
students in the schools of
Germany. Music is in a way given almost as much importance as education there. The
German children
show a higher problem-solving capability than any other children.
115. Which of the following, if true, most weakens the author’s conclusion?
(a) The German theory of music is the most sophisticated in the world
(b) Most of the prominent classical music composers in the world are Germans
(c) Germany’s education system focuses on practical experiences rather than bookish
knowledge
(d) Germany’s music is most complex in the world
116. If the author’s arguments in the above passage are true, which of the
following must also be true?
(a) A person can get rid of all health conditions if he/she listens to music
(b) A country can produce a more calm-headed population if it makes music must in
school curriculum
(c) A person tends to react in a volatile way if he never listens to music
(d) Germany can become the superpower of the world
117. What role does the sentence that “music is the best booster for cognitive
skills than anything else” play in
the passage?
(a) It supports the author’s conclusion that children having music curriculum will
be better at problem#solving
(b) It is the conclusion that the author draws to his arguments
(c) It supports the author’s claim that Germany has a better music curriculum than
any other country
(d) It negates the author’s arguments that Germany has a better music curriculum
than any other country
118. Which of the following assumptions has been made by the author in this
passage?
(a) Persons who are cheerful are all avid listeners of music
(b) Persons who are fit are all musicians
(c) Persons who have less health conditions tend to be calmer than the rest
(d) Germany is the ideal country in the world
119. Which of the following statements if true refutes the observations made by the
Trinity School of Music?
(a) Adolf Hitler, a German was denied by his father to play the flute and he became
the cruelest dictator
in the world
(b) Albert Einstein, a German had learnt the violin and was a maestro at it and he
became the most
celebrated scientist in the history
(c) Isaac Newton, an Englishman had never touched a musical instrument in his life
and he became a
very influential scientist in the world
(d) Galileo Galilei was fond of listening to music and he gave a new direction to
science with his theories

. Page 31 of 36
Passage (Q.120-Q.123): Life expectancy at birth reflects the overall mortality
level of a population. It
summarizes the mortality pattern that prevails across all age groups - children and
adolescents, adults
and the elderly. As a result of falling age-specific mortality, life expectancy
rose dramatically in the United
States over the past century. Preliminary data for 2003 show that life expectancy
at birth for the total
population has reached an all-time American high level. In combination with
decreasing fertility, the life
expectancy gains have led to a rapid aging of the American population, as reflected
by an increasing
proportion of persons aged 65 and older.
The average life expectancy for the United States population as a whole now is 73.9
years, but children
born in Hawaii will live an average age of 77 years, and those children which are
born in Louisiana will
live for 71.7 years. If a newlywed couple from the state of Louisiana were to begin
their family in Hawaii,
therefore, their children would be expected to live longer than would be the case
if the family remained to
live in Louisiana.
120. Which of the following, if true, would most seriously weaken the conclusion
drawn in the second
paragraph of the passage?
(a) Insurance company statisticians do not believe that moving to Hawaii will
significantly lengthen the
average Louisianan's life.
(b) The governor of Louisiana has falsely alleged that statistics for his state are
inaccurate.
(c) The longevity ascribed to Hawaii's current population is attributable mostly to
genetically determined
factors.
(d) Thirty percent of all Louisianans can expect to live longer than 77 years.
121. Which of the following statements, if true, would most significantly
strengthen the conclusion drawn in the
second paragraph of the passage?
(a) As population density increases in Hawaii, life expectancy figures for that
state are likely to be revised
downward.
(b) Environmental factors tending to favour longevity are abundant in Hawaii and
less numerous in
Louisiana.
(c) Twenty-five percent of all Louisianans who move to Hawaii live longer than 77
years
(d) Over the last decade, average life expectancy has risen at a higher rate for
Louisianans than for
Hawaiians.
122. Which of the following statements can be concluded from the first paragraph of
the passage?
(a) In combination with decreasing fertility, the life expectancy gains have led to
a rapid aging of the
American population.
(b) Life expectancy at birth reflects the exact mortality level of a population.
(c) Life expectancy is the number of years to be lived.
(d) All of the above.
123. What role does the statement “life expectancy rose dramatically in the United
States over the past
century” plays in the above passage?
(a) Forms overall conclusion of the above passage.
(b) Forms premise of the above passage.
(c) Forms premise of the conclusion that in combination with decreasing fertility,
the life expectancy
gains have led to a rapid aging of the American population.
(d) Forms the conclusion of the IInd paragraph of the passage.

. Page 32 of 36
Passage (Q.124-Q.129): The focus on maternal mental health has not become an
integral part of India's
health care, despite India's national mental health program enacted in 1982.
National statistics on
Postpartum depression (PPD) in Indian mothers is lacking as data is limited to
specific geographic
regions. The overall aggregated prevalence of PPD in Indian mothers is estimated to
be 22%, with the
highest prevalence in urban areas (24%) and the southern regions. Southern areas
may have a higher
prevalence due to increased urban slums and domestic violence levels. The higher
prevalence of PPD in
urban areas may be due to over-crowding, higher living costs, and tremendous
pressure on working
mothers.
A girl's birth is not preferred in India as she is typically considered a burden to
her parents. This is due to
the financial liability of her dowry and because she cannot be considered the
successor of her family
lineage. Thus, there are expectations for a woman to deliver a male child.
Discrimination against a female
child may lead to feticide or infanticide, a common practice along the 'infanticide
belt' including Tamil
Nadu, where there is a higher prevalence of PPD.
Additionally, India's cultural constructs have set expectations for "good" mothers,
which lead to stigma
when they are not followed. Childbirth is labeled as the happiest time in a woman's
life; thus, feelings of
anxiety or sorrow are considered taboo. Deviations from these norms, along with the
frustrations of
motherhood, may lead to feelings of guilt. Therefore, help-seeking behaviors are
low in India's southern
and urban regions as breaking this taboo can lead to a loss of reputation within
the community. Breaking
this taboo may further result in social exclusion, discontinuation of financial
resources essential for
livelihood, or even marital separation. These factors demonstrate how culture fuels
stigma against mental
health, resulting in alienation from society.
124. Which of the following statements is in line with the author’s line of
thoughts?
(a) Women’s mental health is completely ignored in India
(b) Mental health as a topic is completely ignored in India
(c) An area having high population-density, high cost of living might have more
number of women
suffering from PPD.
(d) Women who are infertile are the ones who are the most depressed
125. Which of the following can be inferred from the passage?
(a) Generally women in India cannot even complain about being depressed after
childbirth.
(b) Childbirth might not always be the happiest time in a woman’s life
(c) Both (a) & (b)
(d) Neither (a) nor (b)
126. Which of the following, if true, would seriously undermine the author’s
argument?
(a) Most women do not get depressed when they are pregnant
(b) Many women are having a baby before 30 years of age
(c) The rate of female infanticide has increased in the South
(d) None of the above
127. The statement “We are unaware how depression affects all women in India as one
entity” is:
(a) Probably True
(b) Definitely True
(c) Probably False
(d) Definitely False

. Page 33 of 36
128. The line “Breaking this taboo may further result in social exclusion,
discontinuation of financial resources
essential for livelihood, or even marital separation.” serves as the –
(a) conclusion drawn in the argument.
(b) claim that the argument shows to be inconsistent with available evidence.
(c) causal explanation for an observed phenomenon.
(d) premise offered in support of the counter-argument drawn in the argument.
129. Which of the following has been used by the author to establish his arguments?
(a) Hyperbole (b) Simile (c) personification (d) NOTA
130. Statement: With several adoption centres in the city, cases of abandoning a
child have increased these
days.
Assumption I: Such parents think that the babies will be picked up by these
centres.
Assumption II: Most of the parents do not want children.
(a) Only assumption I is implicit
(b) Only assumption II is implicit
(c) Both assumptions I and II are implicit
(d) Neither, assumption I nor II is implicit
131. In the question given below, two statements are given followed by conclusions:
I and II. You have to take
the given statements to be true even if they seem to be at variance from commonly
known facts. Read
the conclusions and decide which of the given conclusions logically follows from
the given statements
disregarding commonly known facts.
Statement 1: All worth are costs.
Statement 2: Some prices are worth.
Conclusion I: No costs are prices.
Conclusion II: All prices are costs.
(a) Only II follows (b) Only I follows
(c) Neither I nor II follows (d) Either I or II follows
132. "All trees are green and Apple is a fruit and hence apple is not green". This
statement is:
(a) Logically valid (b) Logically Invalid
(c) Logically True (d) None of the above
133. There are seven members in a family. N and R are the daughters of G. T is the
cousin brother of R. A is
the wife of S. U is the mother of N. U or S does not have any sibling while G has
only one sibling.
How is S related to T?
(a) Grandmother (b) Father
(c) Aunt (d) Sister
134. If HJ= 9; PAID= 15; then what is the code for DESTINY?
(a) 33 (b) 18 (c) 54 (d) 48
135. Select the correct alternative to indicate the arrangement of the following
words in a logical and
meaningful order.
1. Household Supply
2. Glacier Formation
3. Build Dam
4. Flowing of rivers
5. Generate Electricity
(a) 2, 1, 3, 5, 4 (b) 2, 4, 3, 5, 1 (c) 2, 3, 4, 5, 1 (d) 2, 1, 3, 4, 5

mock 12
Directions (Q.1-Q.30): Read the passages carefully and answer the questions.
Passage (Q.1-Q.5): To critics accustomed to the style of fifteenth-century
narrative paintings by Italian
artists from Tuscany, the Venetian examples of narrative paintings with religious
subjects that Patricia
Fortini Brown analyzes in a recent book will come as a great surprise. While the
Tuscan paintings present
large-scale figures, clear narratives, and simple settings, the Venetians filled
their pictures with dozens of
small figures and elaborate buildings, in addition to a wealth of carefully
observed anecdotal detail often
irrelevant to the paintings’ principal subjects—the religious stories they narrate.
Although it occasionally
obscured these stories, this accumulation of circumstantial detail from Venetian
life—the inclusion of
prominent Venetian citizens, for example—was considered appropriate to the
narration of historical
subjects and underlined the authenticity of the historical events depicted. Indeed,
Brown argues that the
distinctive style of the Venetian paintings—what she calls the “eyewitness style”—
was influenced by
Venetian affinity for a strongly parochial type of historical writing, consisting
almost exclusively of
vernacular chronicles of local events embroidered with all kinds of inconsequential
detail.
And yet, while Venetian attitudes toward history that are reflected in their art
account in part for the
difference in style between Venetian and Tuscan narrative paintings, Brown has
overlooked some
practical influences, such as climate. Tuscan churches are filled with frescoes
that, in contrast to
Venetian narrative paintings, consist mainly of large figures and easily recognized
religious stories, as
one would expect of paintings that are normally viewed from a distance and are
designed primarily to
remind the faithful of their religious tenets. In Venice, where the damp climate is
unsuited to fresco,
narrative frescoes in churches were almost non-existent, with the result that
Venetian artists and their
public had no practical experience of the large-scale representation of familiar
religious stories. Their
model for painted stories was the cycle of secular historical paintings in the
Venetian magistrate’s palace,
which were indeed the counterpart of written history and were made all the more
authoritative by a
proliferation of circumstantial detail.
Moreover, because painting frescoes requires an unusually sure hand, particularly
in the representation
of the human form, the development of drawing skill was central to artistic
training in Tuscany, and by
1500 the public there tended to distinguish artists on the basis of how well they
could draw human
figures. In Venice, a city virtually without frescoes, this kind of skill was
acquired and appreciated much
later. Gentile Bellini, for example, although regarded as one of the supreme
painters of the day, was
feeble at drawing. On the other hand, the emphasis on architecture so evident in
the Venetian narrative
paintings was something that local painters obviously prized, largely because
painting architecture in
perspective was seen as a particular test of the Venetian painter’s skill.
1. In the passage, the author is primarily concerned with
(a) pointing out the superiority of one painting style over another.
(b) citing evidence that requires a re-evaluation of a conventionally held view.
(c) discussing factors that explain a difference in painting styles.
(d) outlining the strengths and weaknesses of two opposing views regarding the
evolution of a painting
style.
2. As it is described in the passage, Brown’s of the use of the eyewitness style in
Venetian narrative
painting suggests that
(a) the painting of architecture in perspective requires greater drawing skill than
does the representation
of a human form in a fresco.
(b) certain characteristics of a style of painting can reflect a style of
historical writing that was common
during the same period.
(c) the eyewitness style in Venetian narrative paintings with religious subjects
was largely the result of
the influence of Tuscan artists who worked primarily in fresco.
(d) the historical detail in Venetian narrative paintings with religious subjects
can be traced primarily to
the influence of the paintings in the Venetian magistrate’s palace.

. Page 3 of 40
3. The author suggests that fifteenth-century Venetian narrative paintings with
religious subjects were
painted by artists who…
(a) were able to draw human figures with more skill after they were apprenticed to
painters in Tuscany.
(b) assumed that their paintings would typically be viewed from a distance.
(c) were a major influence on the artists who produced the cycle of historical
paintings in the Venetian
magistrate’s palace.
(d) were better at painting architecture in perspective than they were at drawing
human figures.
4. The author implies that Venetian narrative paintings with religious subjects
included the representation of
elaborate buildings in part because
(a) the ability to paint architecture in perspective was seen in Venice as proof of
a painter’s skill.
(b) the subjects of such paintings were often religious stories.
(c) large frescoes were especially conducive to representing architecture in
perspective.
(d) the architecture of Venice in the fifteenth century was more elaborate than was
the architecture of
Tuscany.
5. Which of the following would best describe contextually, the meaning of the
phrase ‘circumstantial detail?
(a) Pertinent to the overall narrative style of the painting.
(b) Incidental to the main narrative.
(c) Influenced by the existing circumstance and historical accounts.
(d) Marked by detailed representation of religious tenets.
Passage (Q.6-Q.11): Currently, legal scholars agree that in some cases legal rules
do not specify a
definite outcome. These scholars believe that such indeterminacy results from the
vagueness of
language: the boundaries of the application of a term are often unclear.
Nevertheless, they maintain that the system of legal rules by and large rests on
clear core meanings that
do determine definite outcomes for most cases. Contrary to this view, an earlier
group of legal
philosophers, called “realists,” argued that indeterminacy pervades every part of
the law.
The realists held that there is always a cluster of rules relevant to the decision
in any litigated case. For
example, deciding whether an aunt’s promise to pay her niece a sum of money if she
refrained from
smoking is enforceable would involve a number of rules regarding such issues as
offer, acceptance, and
revocation. Linguistic vagueness in any one of these rules would affect the outcome
of the case, making
possible multiple points of indeterminacy, not just one or two, in any legal case.
For the realists, an even more damaging kind of indeterminacy stems from the fact
that in a common-law
system based on precedent, a judge’s decision is held to be binding on judges in
subsequent similar
cases. Judicial decisions are expressed in written opinions, commonly held to
consist of two parts: the
holding (the decision for or against the plaintiff and the essential grounds or
legal reasons for it, that is,
what subsequent judges are bound by), and the dicta (everything is an opinion not
essential to the
decision, for example, comments about points of law not treated as the basis of the
outcome). The
realists argued that in practice the common-law system treats the “holding/dicta”
distinction loosely. They
pointed out that even when the judge writing an opinion characterizes part of it as
“the holding,” judges
writing subsequent opinions, although unlikely to dispute the decision itself, are
not bound by the original
judge’s perception of what was essential to the decision. Later judges have
tremendous leeway in being
able to redefine the holding and the dicta in a precedential case. This leeway
enables judges to choose
which rules of law formed the basis of the decision in the earlier case. When
judging almost any case,
then, a judge can find a relevant precedential case which, in subsequent opinions,
has been read by one
judge as stating one legal rule, and by another judge as stating another, possibly
contradictory one. A
judge thus faces an indeterminate legal situation in which he or she has to choose
which rules are to
govern the case at hand.

. Page 4 of 40
6. According to the passage, the realists argued that which one of the following is
true of a common-law
system?
(a) It gives rise to numerous situations in which the decisions of earlier judges
are found to be in error by
later judges.
(b) It possesses a clear set of legal rules in theory, but in practice most judges
are unaware of the strict
meaning of those rules.
(c) Its strength lies in the requirement that judges decide cases according to
precedent rather than
according to a set of abstract principles.
(d) It treats the difference between the holding and the dicta in a written opinion
rather loosely in
practice.
7. According to the passage, which one of the following best describes the
relationship between a judicial
holding and a judicial decision?
(a) The holding is not commonly considered binding on subsequent judges, but the
decision is.
(b) The holding formally states the outcome of the case, while the decision
explains it.
(c) The holding explains the decision but does not include it.
(d) The holding sets forth and justifies a decision.
8. The information in the passage suggests that the realists would most likely have
agreed with which one
of the following statements about the reaction of judges to past interpretations of
a precedential case,
each of which states a different legal rule?
(a) The judges would most likely disagree with one or more of the interpretations
and overturn the earlier
judges’ decisions.
(b) The judges might differ from each other concerning which of the interpretations
would apply in a given
case.
(c) The judges probably would consider themselves bound by all the legal rules
stated in the
interpretations.
(d) The judges would regard the lack of unanimity among interpretations as evidence
that no precedents
existed.
9. It can be inferred from the passage that most legal scholars today would agree
with the realists that
(a) linguistic vagueness can cause indeterminacy regarding the outcome of a
litigated case
(b) in any litigated case, several different and possibly contradictory legal rules
are relevant to the
decision of the case
(c) the distinction between holding and dicta in a written opinion is usually
difficult to determine in
practice
(d) the boundaries of applicability of terms may sometimes be difficult to
determine, but the core
meanings of the terms are well established
10. The passage suggests that the realists believed which one of the following to
be true of the dicta in a
judge’s written opinion?
(a) The judge writing the opinion is usually careful to specify those parts of the
opinion he or she
considers part of the dicta.
(b) The appropriateness of the judge’s decision would be disputed by subsequent
judges on the basis of
legal rules expressed in the dicta.
(c) A consensus concerning what constitutes the dicta in a judge’s opinion comes to
be fixed over time
as subsequent similar cases are decided.
(d) Subsequent judges can consider parts of what the original judge saw as the
dicta to be essential to
the original opinion.

. Page 5 of 40
11. Which one of the following best describes the overall organization of the
passage?
(a) A traditional point of view is explained and problems arising from it are
described.
(b) Two conflicting systems of thought are compared point for point and then
evaluated.
(c) A legal concept is defined and arguments justifying that definition are
refuted.
(d) Two viewpoints on an issue are briefly described and one of those viewpoints is
discussed at greater
length.
Passage (Q.12-Q.17): This idea of evolution misses the essential role played by
individual differences
and competition between members of a species in response to environmental
pressures: Individual
cheetahs who can run faster catch more prey, live longer, and reproduce more
successfully; slower
cheetahs lose out, and die out—leaving the species to drift toward becoming faster
overall. Evolution is
the result of random differences and natural selection, not agency or choice.
But belief in the “agency” model of evolution is hard to beat back. While educating
people about evolution
can indeed lead them from being uninformed to being well informed, in some stubborn
instances it also
moves them into the confidently misinformed category. In 2014, Tony Yates and
Edmund Marek
published a study that tracked the effect of high school biology classes on 536
Oklahoma high school
students’ understanding of evolutionary theory. The students were rigorously
quizzed on their knowledge
of evolution before taking introductory biology, and then again just afterward. Not
surprisingly, the
students’ confidence in their knowledge of evolutionary theory shot up after
instruction, and they
endorsed a greater number of accurate statements. So far, so good.
The trouble is that the number of misconceptions the group endorsed also shot up.
For example,
instruction caused the percentage of students strongly agreeing with the true
statement “Evolution cannot
cause an organism’s traits to change during its lifetime” to rise from 17 to 20
percent—but it also caused
those strongly disagreeing to rise from 16 to 19 percent. In response to the
likewise true statement
“Variation among individuals is important for evolution to occur,” exposure to
instruction produced an
increase in strong agreement from 11 to 22 percent, but strong disagreement also
rose from nine to 12
percent. Tellingly, the only response that uniformly went down after instruction
was “I don’t know.”
And it’s not just evolution that bedevils students. Again and again, research has
found that conventional
educational practices largely fail to eradicate a number of our cradle-born
misbeliefs. Education fails to
correct people who believe that vision is made possible only because the eye emits
some energy or
substance into the environment. It fails to correct common intuitions about the
trajectory of falling objects.
And it fails to disabuse students of the idea that light and heat act under the
same laws as material
substances. What education often does appear to do, however, is imbue us with
confidence with the
errors we retain.
12. "This idea of evolution misses the essential role played by individual
differences and competition…." The
beginning of the sentence reflects an idea before. What could be the possible idea
that the author argues
upon?
i) The idea that evolution is driven by natural selection and random differences.
ii) Evolution is essentially a game of species-level strategy.
iii) The idea that evolution is uniform, tactical and well structured.
(a) Only iii (b) ii & iii (c) i & ii (d) I, ii & iii
13. "So far, so good." We can infer from the statement given in the second
paragraph that
(a) The author is convinced of the result of the experiment.
(b) The author is dissatisfied with the result of the experiment.
(c) The author was satisfied with the accurate responses to the experiment up to
the present.
(d) The author knows that the greater accurate responses by the students were
superficial.

. Page 6 of 40
14. The word ‘bedevils' in the context of the passage means
(a) Confound
(b) Harass maliciously
(c) Bewitch
(d) Soothe
15. "Tellingly, the only response that uniformly went down after instruction was "I
don't know." What can be
concluded from the given statement?
(a) The author was expecting the drop in the "I don't know" response.
(b) The only response that showed a significant drop was "I don't know".
(c) The drop in the response "I don't know" was insignificant.
(d) The author was not convinced with the' I don't know' response.
16. Which of the following statement of the passage can be taken as the conclusive
argument from the author's
standpoint?
(a) But belief in the "agency" model of evolution is hard to beat back.
(b) What education often does appear to do, however, is imbue us with confidence in
the errors we retain.
(c) Education fails to correct people who believe that vision is made possible only
because the eye emits
some energy or substance into the environment.
(d) Again and again, research has found that conventional educational practices
largely fail to eradicate a
number of our cradle-born misbeliefs.
17. What education often does appear to do, however, is imbue us with confidence
with the errors we retain.
The following sentence has been divided into sections in the given options. Choose
the option that
contains a grammatical error.
(a) What education often does
(b) appear to do, however, is
(c) imbue us with confidence
(d) with the errors we retain.
Passage (Q.18-Q.24): E. M. Forster’s outlook on the world and his literary manner
though already
thoroughly developed in that epoch, have passed through the subsequent years of
turbulence and
cataclysm with remarkably little modification. Modern revolutions in physics,
psychology, politics, even
literary style, have not escaped his intelligent notice, but they have scarcely
influenced him deeply. His
response to the explosion of the Victorian dream of benevolent progress has been a
modest and orderly
retreat to safer ground— to a tolerant individualism now unmixed with Utopian
dreams, but nevertheless
closer to Victorian ideals than to any of the popular creeds of today. Rather than
conform to bad times,
Forster prefers to remind us cheerfully that his views are atavistic.
The strength of Forster's resistance to the twentieth century is especially
apparent when we place him
beside some of his fellow writers. If Joyce, Lawrence, Pound, and the early Eliot
represent the main
current of the modern literary movement in English, we must admit that Forster's
private stream runs in
an older channel. These others were radical iconoclasts whose rejection of
bourgeois-democratic life was
violent and shattering. Equally shattering was their fragmentation of the polite
cadences of Victorian
literature. In seeing the falseness of the old psychology, they conceived a scorn
for the hypocrite lecteur;
their role as apocalyptic prophets, as naysayers to the boredom and specious
rationality of modern life,
demanded that they be obscure and idiosyncratic. Forster, in contrast, unashamedly
calls himself a
bourgeois and remains faithful to the tradition of calm intelligibility.
The is anti-apocalyptic in both his politics and his literary sense. To some degree
his novels return us to
the congenial Victorian relationship between writer and reader, with its unspoken
agreement over the
usefulness of the sociable virtues and its apotheosis of the happy family. Though
Forster's heroes
struggle against "society" as a body of inhibitions, their revolt is never truly
radical. And Forster's ironical

. Page 7 of 40
style, though it is unsparing in its probing at shams and half-truths, presupposes
a confidence in the
reader's sympathy and good judgment—a confidence that seemed quite archaic to the
other writers
named.
Forster's resistance to modernity may account for the fact that his novels, though
almost universally
esteemed, never won him a cult of fanatical disciples. With a few exceptions,
critics have tended to
explicate and admire his works without becoming heated over the possible merit of
his ideas. Yet Forster
decidedly is a novelist of ideas, and didactic moral content is hardly less
conspicuous in his work than in
Lawrence's. Forster's persistent "moral" is that the life of affectionate personal
relations, disengaged from
political and religious zeal by means of a tolerant eclecticism, is supremely
valuable. This is not a stirring
creed; in fact, it is a warning against allowing oneself to be stirred by any
creed.
18. The author’s primary purpose in this passage is to
(a) discuss E. M. Forster and his writing, particularly in the context of his
reaction to modernity
(b) compare E. M. Forster to other writers of the twentieth century such as Joyce
and Lawrence
(c) affirm that E. M. Forster is as much a novelist of ideas as other modern
writers
(d) suggest that E. M. Forster’s writing is a reflection of not only Victorian
ideals but also Edwardian
19. According to the passage, Forster’s relationship to Victorianism is which of
the following?
(a) He believed Victorian ideals were preferable to those of modernity.
(b) He did not believe in Victorian ideals but nevertheless clung to them.
(c) He considered Victorian ideals to be not only oppressive but also false.
(d) He rejected Victorian ideals, but not so completely as other modern writers.
20. The author most likely refers to Forster saying his “views are atavistic” in
order to
(a) make a case for the importance of individualism to Forster and his work.
(b) isolate Forster as a writer unconnected to the revolutions of the modern world.
(c) emphasize that Forster was an atypical modern writer.
(d) suggest that Forster was an ardent supporter of the popular beliefs of his
time.
21. Select the sentence that proposes an for the lack of passion among some readers
for the ideas often put
forth by Forster.
(a) Though Forster's heroes struggle against "society" as a body of inhibitions,
their revolt is never truly
radical.
(b) Forster’s resistance to modernity may account for the fact that his novels,
though they are almost
universally esteemed, have never won him a cult of fanatical disciples
(c) His response to the explosion of the Victorian dream of benevolent progress has
been a modest and
orderly retreat to safer ground— to a tolerant individualism now unmixed with
Utopian dreams
(d) We must admit that Forster's private stream runs in an older channel.
22. The passage suggests which of the following about other writers who produced
works in the same period
as did Forster?
A. These writers, like Forster, rejected bourgeois values and directed their
writing away from a style of
polite veneer toward peculiar and individualized voices.
B. These writers showed a greater willingness to embrace modernity than did
Forster, who did not give
up his belief in the importance of certain societal virtues.
C. These writers, much like Forster, used their writing to instruct readers about
their moral viewpoint,
even if their styles differed from that of Forster.
(a) A and B (b) B and C (c) A, B and C (d) B only

. Page 8 of 40
23. The author’s reaction to Forster’s novels can best be described as one of
(a) disparagement
(b) scepticism
(c) neutrality
(d) appreciation
24. It can be inferred from the passage that Joyce, Lawrence, Pound, and Eliot in
his early period were all
writers who
A. wrote in a style ahead of the spirit of their time.
B. were often rude and offensive.
C. lacked confidence in their audiences’ sympathy.
(a) A only (b) B only (c) A and B (d) A and C
Passage (Q.25-Q.30): The Centre’s move to allocate 2 crore coronavirus vaccine
doses for universal
coverage of all teaching and non-teaching staff in schools by Teachers’ Day
(September 5) adds a
measure of confidence that resumption of face-to-face classes from September is not
fraught with high
risk. Several States are preparing to reopen schools, mostly for Class 9 and
higher, next month. Some,
including Haryana, Telangana and Gujarat, have announced that they will allow
offline classes even for
younger children. Amidst fears of a third wave of the pandemic, epidemiologists,
academicians and
policymakers have been wrestling with the question of a low-risk trade-off,
balancing protection from the
virus with some bridging of the learning deficit caused by prolonged school
closures that are crippling
future prospects of millions of children. It is encouraging that half of the 97
lakh teachers in the country
have already been immunised, by official estimates, making it feasible to reach the
rest by September 5.
The risk of infection to children in schools remains, however, and must be
addressed with utmost
seriousness, particularly with fast-transmitting virus variants present in all
States. This calls for a coherent
response that incorporates the best learnings from epidemiology and decentralises
decision-making to
the districts based on local circumstances. It is, of course, a step forward that a
vaccine, the three-dose
ZyCoV-D, has been approved for the 12-18 age group, but this is to be administered
only from October.
The reversion to physical classes will, therefore, have to be carefully calibrated.
Vaccination of children over 12 years of age has been allowed in some countries,
but the pandemic’s
course has remained unpredictable. In Israel, one of the most vaccinated countries
with an estimated
78% coverage of the over-12 population, the Delta variant swiftly caused a spike in
infections as
distancing norms, the mask mandate and travel bans were eliminated. In one
instance, an entire class of
students was infected by one unvaccinated child who had been on vacation. This
cautionary tale serves
to emphasize the importance of priority vaccination of children, starting with
those who may have other
health conditions, maintaining safety protocols, and adopting low-cost non-
pharmaceutical interventions
such as good classroom ventilation and open-air instruction wherever feasible.
Credentialed studies in
the U.S. indicate that these are effective measures, along with vaccination. It is
vitally important for the
Centre to share information on the school reopening experience with all States and
issue alerts in real
time to enable decision-making. An empirical approach will also enable
organisations such as the
National Institute of Disaster Management collate useful insights. Parents must be
convinced by
transparent official measures that the health and education prospects of their
children are in safe hands.
25. “Amidst fears of a third wave of the pandemic, epidemiologists, academicians
and policymakers have
been wrestling with the question of a low-risk trade-off…” What can be derived from
the word
‘epidemiologists?
(a) Specialists studying the mental make of school going children during Pandemic.
(b) Specialists studying the emotional impact of diseases and environmental toxins.
(c) Specialists assessing patterns and effects of diseases of children.
(d) Specialists studying and analysing public health concerning a given population.

. Page 9 of 40
26. Which of the following reflects the tone of the passage?
(a) Thought-provoking and Prescribing
(b) Condescending and demeaning
(c) Vituperative and incendiary
(d) Circumventing and Insouciant
27. The author uses the example of Israel in order to
(a) caution against arbitrary decision to discard the necessary precautions.
(b) caution against the Delta variant that is recalcitrant to the vaccination.
(c) give a node to the decision by the government to open opens with all due
precautions.
(d) fend off unnecessary rumours about the Delta variant being a non-pervasive
virus.
28. We can understand from the passage that the author is
(a) censuring the step taken by the government of reopening of schools before the
expected third wave of
pandemic.
(b) circumspect about the government’s decision of opening schools amid the fear of
third wave of
pandemic gripping the children.
(c) embraces the decision by the government to open the school as he feels that the
future of millions of
children is at stake.
(d) gives qualified approval to the decision by the government to open the schools
with prescribing certain
precautionary measures.
29. Which of the following idiomatic expression/phrases would also serve as the
title of the passage?
(a) Throw caution to the wind.
(b) Age of discretion.
(c) Tread on the side of discretion.
(d) On red alert.
30. The reversion to physical classes will, therefore, have to be carefully
calibrated. The highlighted word
belongs to which part of speech?
(a) Noun
(b) Verb
(c) Adverb
(d) Conjunction
Directions (Q.66 – Q.105): Read the comprehensions carefully and answer the
questions based on it.
Passage (Q.66-Q.71): The literal meaning of kidnapping is child stealing.
Kidnapping is of two types i.e.
kidnapping from India and Kidnapping from lawful guardianship. These two forms of
kidnapping may
overlap each other.
Section 360 of Indian Penal Code (IPC) states that whoever takes a person beyond
the limits of India
without the consent of such person or any person legally authorized to give consent
on his behalf then he
shall be liable for the offence of kidnapping from India. Under this offence, it
does not matter whether the
victim is major or minor. The age of consent for the offence of kidnapping is 16
years for boy and 18
years for girls.
Section 361 of IPC states that ‘Whoever takes or entices any minor under [sixteen]
years of age if a male,
or under [eighteen] years of age if a female, or any person of unsound mind, out of
the keeping of the
lawful guardian of such minor or person of unsound mind, without the consent of
such guardian, is said to
kidnap such minor or person from lawful guardianship’. The section is intended for
the protection of
minors and persons of unsound mind. The unsoundness of mind must be permanent and
not temporary
insanity produced due to intoxication.
(Extracted with requisite revisions and edits from ‘All You Need To Know About
Kidnapping and
Abduction’ available at
66. Albeli was a 9-year-old girl and came to visit her grandmother every summer
holidays. She was very fond
of Kulfis
(Ice-creams). A Kulfiwala used to pass from her locality every day and the children
used to gather around
his stall to get their favorite Kulfis. One such day, Albeli got late to get her
Kulfi and saw the Kulfiwala at a
distance. She walked and followed him and after a while she reached him but by that
time she was in
another locality. Albeli’s family registered a complaint against the Kulfiwala for
kidnapping Albeli. Is he
liable for kidnapping Albeli?
(a) Yes, the Kulfiwala took Albeli who is a minor from her legal guardian and hence
this act would
amount to kidnapping.
(b) Yes, there was the element of enticing as the Kulfiwala sold ice-creams of
which Albeli was very fond
of.
(c) No, there is no element of taking away or enticing on the part of the Kulfiwala
and thus there was no
kidnapping.
(d) No, Albeli followed the Kulfiwala at her own free will and thus there was no
kidnapping.
67. Later, after Kulfiwala found that Albeli had followed him came to know that a
reward is soon to be placed
for finding Albeli. He decided to keep her with him for a day until the reward has
been officially
announced. After the reward was announced he took Albeli to her Gradmother’s house
and claimed the
reward. But when the family came to know that it was him whom Albeli had followed
decided to not give
any reward to him and even threatened to initiate police action. Is Kulfiwala
entitled to any rewards?
(a) Yes, the reward was designated to any person who finds Albeli and the Kulfiwala
had brought her
back to the house.
(b) Yes, Albeli had followed the Kulfiwala without him taking any active
advancement to influence or
entice her in order to follow him.
(c) No, Kulfiwala had ulterior motives of returning Albeli and was not motivated by
good faith and
therefore would not be entitled to any reward.
(d) No, Kulfiwala had kidnapped her as he even after becoming aware of the presence
of Albeli he kept
her with him for a day and waited for the reward to be officially announced.

. Page 17 of 40
68. Albeli’s family filed a case against Kulfiwala for kidnapping. Is he liable for
the commission of kidnapping?
(a) Yes, soon after the discovery of Albeli, the Kulfiwala kept her with him
without declaring the same to
her family members till the time the reward was officially announced.
(b) No, there is no element of taking away or enticing and thus there is no
kidnapping of Albeli.
(c) Yes, he enticed her by selling the Kulfis of which she was fond of and took her
away from the legal
guardian.
(d) No, for kidnapping has to be from a legal guardian and the grandmother was not
a permanent
guardian but her parents will be.
69. During the proceedings, it was found that the day Albeli left to follow the
Kulfiwala, she was at their
neighbor’s house and thus even if the act of Kulfiwala was incorrect, the same
would not be kidnapping
under the Penal Code as there is no taking away from Lawful Guardian. Is the
argument favorable to
Kulfiwala to absolve him of all the allegations of kidnapping leveled against him?
(a) Yes, the element of taking away from a lawful guardian has been missing as she
has been at their
neighbour's house.
(b) Yes, he would be partially absolved of his act as despite him not taking from a
lawful guardian, the
other elements to qualify as kidnapping are present.
(c) No, there was still the presence of taking away from the lawful guardian as the
neighbors acted as
guardians.
(d) No, since she was present in the same locality, she would still be treated as
under the guardianship
of her grandmother.
70. Assumingly, while Albeli was kept with the Kulfiwala he even asked her whether
she wants to go home or
she can stay here and have as many kulfis as she want and Albeli, the fan of Kulfis
she was, decided to
choose the latter. It was thus contended that since the consent of the girl herself
has been obtained it
doesn’t amount to kidnapping. Is the argument correct?
(a) Yes, the consent of the person i.e. Albeli has been obtained without any
coercion and out of her free
will.
(b) No, the consent of the person who is legally authorized has not been obtained
as Albeli is a minor
under 18years of age.
(c) Yes, from the conduct of Albeli of following the Kulfiwala it is apparant that
she was mature enough to
understand the nature of her actions.
(d) No, Kulfiwala had entinced Albeli in order to obtain her consent and thus this
will amount to
kidnapping.
71. The health of Albeli’s grandmother has also not been very favorable and she had
started developing long
episodes of Dementia. During one such episode, she left her house and the care of
her sons and in-laws
and walked towards the house of their neighbor Mrs. Kanti and just sat in the
backyard hiding and looking
at her favorite Banyan tree that she associated many memories with. Mrs. Kanti saw
her sitting there but
decided not to intrude upon her thoughts. Meanwhile her family was looking for her.
Is Mrs. Kanti liable
for kidnapping?
(a) Yes, she was taken away from the care and protection of her sons and in-laws
and thus she was
kidnapped.
(b) No, to constitute kidnapping there has to be taking away or enticing which is
absent in the present
case.
(c) Yes, the Banyan tree being present at Mrs. Kanti’s house also amounts to
enticing and thus the act
will be covered under Section 361 IPC.
(d) No, Albeli’s Grandmother is not a minor and kidnapping only covers instances of
females under 18
years of age.

. Page 18 of 40
Passage (Q.72-Q.81): The Supreme Court draws its power from Section 406 of the Code
of Criminal
Procedure, 1908 to transfer criminal cases and appeals pending in one High Court to
another High Court
or from a criminal Court subordinate to one High Court to another criminal Court of
equal or superior
jurisdiction subordinate to another High Court. Justice V.R. Krishna Iyer, in
Maneka Sanjay Gandhi v.
Rani Jethmalani; (1979) 4 SCC 167, laid down as under,
"2. … is not the hypersensitivity or relative convenience of a party or easy
availability of legal services or
like minigrievances. Something more substantial, more compelling, more imperiling,
from the point of
view of public justice and its attendant environment, is necessitous if the Court
is to exercise its power of
transfer…
...5. A more serious ground which disturbs us in more ways than one is the alleged
absence of congenial
atmosphere for a fair and impartial trial. This tendency of toughs and street
roughs to violate the serenity
of the Court is obstructive of the course of justice and must surely be stamped
out. Likewise, the safety of
the person of an accused or complainant is an essential condition for participation
in a trial and where
that is put in peril by commotion, tumult or threat on account of pathological
conditions prevalent in a
particular venue, the request for a transfer may not be dismissed summarily.
(Extracted with requisite revisions and edits from‘To Transfer Or Not To Transfer:
Parameters Followed
By The Supreme Court In Transfer Petitions’ at
72. The Petitioner was considered to be accused for an an offence of attempt to
murder the Former Chief
Justice of the State of Madhya Pradesh who was also his neighbor. The Petitioner
was tried before the
Present Chief Justice of said State, who was the complainant. Therefore, the
Petitioner fearing that he
will not have fair and impartial hearing applied for transfer of the case to some
other High Court. Should
the transfer of the case to another High Court be done?
(a) No, there are no compelling factors for the Court to exercise its power of
transfer.
(b) No, there does not exist any substantial factors or imperiling circumstance
which will require such a
transfer.
(c) Yes, from the perspective of public justice, such a transfer of the case is
necessary.
(d) Yes, the presence of the complainant will lead to the absence of an atmosphere
of fairness and
impartiality.
73. There have been continuous communal tensions between the Meities (Manipuri
people) and the Nagas.
Judicial proceedings were going before the Sessions Judge over the violence that
ensured by the group
led by the accused. According to the accused there is a real possibility of a
physical attack on him and
there was even a risk of intimidation of witnesses who would come in his support by
the opposite
community. Is this the reason sufficient enough to transfer the case?
(a) Yes, the safety of the accused and that of the witnesses is at peril by the
acts of the opposite party
which will require the transfer of the case.
(b) No, there doesn’t exist any actual threat to the safety of the accused and the
witnesses and it is
merely an apprehension on which such a transfer has been sought.
(c) Yes, the communal tensions from the perspective of public justice require such
a transfer of the case.
(d) No, there does not exist any substantial factors or imperiling circumstance
which will require such a
transfer.
74. The police of Nagaland have been renowned for their infamous encounters in the
Nagas and Maities
violence ‘because the Maities protestors were resorting to violent means and were
aiding the accused to
evade their custody’ according to the statement by the police. It was requested
that the investigation be
transferred from the State police to the CBI because of serious allegations made
against the police force
involved in the encounter resulting in the killing of several persons and the
likelihood of safety issues of
other accused and their supporters. Should such a transfer be made?
(a) Yes, the safety of the accused and that of the supporters is at peril by the
acts of the police which will
require the transfer of the case.

. Page 19 of 40
(b) No, there doesn’t exist any actual threat to the safety of the accused and the
supporters and it is
merely an apprehension on which such a transfer has been sought.
(c) Yes, the communal tensions of the community from the perspective of public
justice require such a
transfer of the case.
(d) No, there does not exist any substantial factors or imperiling circumstance
which will require such a
transfer.
75. It is contended that there should be transfer of the chit fund scam in the
states of West Bengal and
Orissa from the State Police to the CBI keeping in view the sensitivity of the
issues involved especially
inter-state ramifications of the scam under investigation and the involvement of
several influential persons
holding high positions of power and having political clout thus putting the safety
of people at peril too.
Should such a transfer be made?
(a) No, there does not exist any substantial factors or imperiling circumstance
which will require such a
transfer.
(b) No, there doesn’t exist any actual threat to the safety of the accused and the
witnesses and it is
merely an apprehension on which such a transfer has been sought.
(c) Yes, the involvement of several influential people holding high positions and
the sensitivity of the case
will affect the fairness and impartiality of the case and the safety of those
involved.
(d) Yes, the sensitivity of the case will prejudice the decision if continued
between the states of West
Bengal and Orissa.
76. A famous sex scandal of Mohali called the Moga sex scandal was being
investigated which disclosed the
name of various senior functionaries of the State Police and political leaders. It
was contended that the
same should be transferred from the State Police to the CBI due to the fear on the
safety of those who
raise voice against these functionaries. This was denied by the Court. Is the
action of the Court justified?
(a) Yes, there does not exist any substantial factors or imperiling circumstance
which will require such a
transfer.
(b) Yes, there doesn’t exist any actual threat to the safety of the accused and the
witnesses and it is
merely an apprehension on which such a transfer has been sought.
(c) No, the involvement of several influential people holding high positions and
the sensitivity of the case
will affect the fairness and impartiality of the case and the safety of those
involved.
(d) No, the sensitivity of the case will prejudice the decision if continued in the
same Court.
77. A young girl, aged 19 years was allegedly killed due to the rampant ragging in
a renowned Government
University. An application was made for the transfer of case to the CBI on account
of coercion by the
college authorities and police, and fabrication of medical evidence in connivance
with the doctors. This
was rejected by the Court for lack of being a compelling cause. Is the Court
justified in doing so?
(a) Yes, there are no compelling factors for the Court to exercise its power of
transfer.
(b) No, there does exist substantial factors and imperiling circumstance which will
affect the fairness of
the trial.
(c) Yes, from the perspective of public justice, such a transfer of the case is not
necessary.
(d) No, the coercion and connivance will be independent of the proceedings before
the Court of law and
hence no transfer is necessitated.
78. A famous scientist Mr. Thakur committed suicide. However, there were traces of
foul play around his
death. In order to inspire public confidence and to do complete justice in the
matter, the Court decided to
transfer all the matter to the CBI. Is the act of the Court correct?
(a) Yes, there are compelling factors for the Court to exercise its power of
transfer.
(b) No, there does not exist substantial factors and imperiling circumstance which
will affect the fairness
of the trial.
(c) Yes, from the perspective of public justice and inspire confidence, such a
transfer of the case is
necessary.
(d) No, the hypersensitivity of the case is not a relevant factor for transferring
a case.

. Page 20 of 40
79. Mr. Anunay was accused of murder of his colleague and was arrested. However,
since he belonged to a
rural and remote area, the place lacked proper and experienced counsels and his
counsel stayed at quite
a distance and Mr. Anunay had to travel a lot just to consult him. He sought the
transfer of the case due
to lack of proper legal services which was affecting his trial being adjudicated
fairly. However, the Court
denied his contention and continued the proceedings. Is the Court correct in doing
so?
(a) No, there exists obstructive factors due to non-availability of easy access to
proper counsel
influencing the course of justice that will necessitate the transfer the case.
(b) Yes, easy availability of legal services is not a factor that will influence
the transfer of the case.
(c) Yes, there are no compelling factors for the Court to exercise its power of
transfer.
(d) No, there does exists substantial factors and imperiling circumstance which
will affect the fairness of
the trial.
80. Mrs. Mugdha, was an 80-year-old lady who suffered from series issues of
arthritis. The Court
proceedings were aking place in a Court situated in another district for which Mrs.
Mugdha has to travel
quite a distance as she was a witness in a prominent case. She quoted her ill
health for transferring the
case to the Court in her city as her ailment was restricting in her movements and
to appear before the
Court regularly. The Court denied the transfer. Is the Court correct in doing so?
(a) Yes, the relative convenience of Ms. Mugdha or her hypersensitivity due to her
arthritis will not be a
factor that will affect the transfer of the case.
(b) No, the fact of Ms. Mugdha being a witness in a prominent case will require the
transfer of the case.
(c) Yes, the grievance raised by Ms. Mugdha is meager and not compelling enough to
transfer the case.
(d) No, the sensitivity and ailment of Ms. Mugdha will influence the transfer of
the case.
81. Mr. Ashutosh was a popular and political face in the locality of Birsingpur. He
was accused under sedition
laws. On the first hearing, the Courtroom was filled with his supporters who
applauded and shouted their
support for Mr. Ashutosh and hurled abuses and threats and sometimes even threw
slippers to his
dissenters. The opposing party sought the transfer of the case which was denied by
the Court. Is the
Court correct in doing so?
(a) Yes, there does not exist any obstructive factors influencing the course of
justice that will necessitate
the transfer the case.
(b) No, the act of the supporters of Mr. Ashutosh is obstructive of the course of
justice and violative of the
serenity of the Court.
(c) No, there are compelling factors for the Court to exercise its power of
transfer.
(d) Yes, there does not exist substantial factors and imperiling circumstance which
will affect the fairness
of the trial.
Passage (Q.82-Q.85): Once a Foreign State opts to wear the hat of a commercial
entity, it would be
bound by the rules of the commercial legal ecosystem and cannot be permitted to
seek any immunity,
which is otherwise available to only when it is acting in its sovereign capacity.
Section 86(3) of the Code
of Civil Procedure [Except with the consent of the Central Government, certified in
writing by a Secretary
to that Government, no decree shall be executed against the property of any foreign
State.] is of limited
applicability and the protection thereunder would not apply to cases of implied
waiver. An arbitration
agreement in a commercial contract between a party and a Foreign State is an
implied waiver by the
Foreign State so as to preclude it from raising a defense against an enforcement
action premised upon
the principle of Sovereign Immunity.
(Extracted with requisite revisions and edits from ‘Foreign State cannot claim
sovereign immunity against
enforcement of arbitral award arising out of commercial transaction: Delhi High
Court’ at
82. India entered into a MoU with Saudi Arabia in order to promote yoga. In
furtherance of this they entered
into an Agreement where India with the aid of its local organization Dharmashala
would be establishing
schools of Yoga at various places in Saudi Arabia and their Government would be
providing with other
amenities along with allocation of land and resources to them within a year. Two
years passed and
despite various reminders, the government of Saudi Arabia did not oblige the
contract. An arbitral award

. Page 21 of 40
was passed against them. When questioned about the same, they contended that
despite the nature of
the contract they would be protected by sovereign immunity being a Government. Are
they correct in
contending this?
(a) Yes, they are the Government of Saudi Arabia and would be protected by
sovereign immunity.
(b) Yes, the involvement of Saudi Government and Indian Government indicates that
the contract has
been entered under sovereign capacities and hence the government would be protected
by virtue of
sovereign immunity.
(c) No, there was a commercial contract between the parties and no immunity can be
claimed by the
Government of Saudi Arabia.
(d) No, nobody can be allowed to enrich themselves unjustly.
83. On February, 2020, the Respondent awarded a contract to the Petitioner for
rehabilitation of Iran
Embassy at New Delhi for a consideration of Rs. 3 crore. Certain disputes arose
during the course of
execution of work whereupon the Petitioner invoked the arbitration clause. The
Respondent participated
in the arbitration proceedings initially. However, there was no appearance later
and an exparte decision
was given against the Respondent. The award was not challenged but was also not
obliged by the
Respondent. Whether consent of the Central Government under Section 86(3) would be
required for the
enforcement of the award?
(a) Yes, they are the Government of Iran and would be protected by sovereign
immunity.
(b) Yes, the involvement of Iran Government indicates that the contract has been
entered under
sovereign capacities and hence consent of Central Government would be required
under Section
86(3).
(c) No, there was a commercial contract and Section 86(3) would be of limited
import.
(d) No, nobody can be allowed to enrich themselves unjustly.
84. On June, 2020, the Petitioner entered into a contract with the Respondent for
supply and distribution of
books to the Respondent at various places in Ethiopia for the contract value fixed
at Rs. 15 crores. The
Petitioner claims to have shipped the goods to the Respondent and completed the
last shipment on
December, 2020. The order was complete with 86 shipments and the books were
distributed in Ethiopia
by the Petitioner’s agent at the pre-decided locations. 86 invoices were also
raised by the Petitioner. The
Respondent after the payment of 10 invoices cancelled the contract by letter dated
24th April, 2021. The
Petitioner initiated the arbitration proceedings as per the arbitration agreement
to recover the balance
amount against the Respondent. The Respondent chose not to participate in the
arbitration proceedings
and was proceeded ex-parte with decision against the Respondent. Respondent didn’t
protest against
the same but also didn’t pay the balance amount. In order to enforce the order,
whether the prior
approval of Central Government as required under Section 86(3) would be required or
not?
(a) Yes, they are the Government of Ethipia and would be protected by sovereign
immunity.
(b) Yes, the involvement of Government indicates that the contract has been entered
under sovereign
capacities and hence consent of Central Government would be required under Section
86(3).
(c) No, there was a commercial contract and Section 86(3) would be of limited
import.
(d) No, because the Government failed to participate in the arbitral proceedings.
85. In the midst of war between Israel and Palestine, Israel (Defendant)
commandeered and ordered certain
goods from a company in India post entering into a arbitration agreement. Defendant
failed to take the
delivery of certain goods and to make payment for them due to which the Company
suffered huge losses
and sued the Secretary of the State to recover damages. An decree against the
defendant was also
given but they didn’t make the payment. Pursuant to which they proceeded to enforce
the decree against
the property of Israel. Decide if the protection under Section 86(3) apply? ?
(a) Yes, the contract involves exercise of sovereign function and would be
protected by sovereign
immunity.
(b) Yes, the involvement of Government indicates that the contract has been entered
under sovereign
capacities and hence consent of Central Government would be required under Section
86(3).

. Page 22 of 40
(c) No, since the consent of Central Government for executing a decree against the
property of any
foreign State is not taken, hence no protection. .
(d) No, since arbitration agreement in a commercial contract is a implied waiver
upon the principle of
Sovereign immunity, hence no protection. .
Passage (Q.86-Q.89): On the 13th of March, 2020, the Ministry of Consumer Affairs,
Food and Public
Distribution, classified Hand Sanitizers as an "Essential Commodity", under the
Essential Commodities
Act, 1955. It has now been inferred from several circulars and orders, issued by
the Drug inspectors
across the nation, that the sale of hand sanitizers would be restricted exclusively
to dealers having drug
licenses, which would curtail its sale, solely to medical shops and chemists.
The legislations regulating the manufacture, import, distribution and sale of drugs
and cosmetics, are
codified in the Drugs and Cosmetics Act, 1940. Chapter IV of the act deals with the
manufacture, sale
and distribution of drugs and cosmetics. According to Section 18(c) of the Act, no
person, either by
himself or on behalf of another person shall manufacture any drug or cosmetic, for
sale or stock or
distribution or exhibition, except under a license that has been issued for the
same. Rule 61 of the Drugs
and Cosmetics Rules, 1945, states, "(1) A license to sell, stock, exhibit or offer
for sale or distribute drugs
other than those specified in Schedules C, C (1) and X and by retail on restricted
license or by wholesale,
shall be issued in Form 20, Form 20A or Form 20B, as the case may be, provided that
a licence in Form
20A shall be valid for only such drugs as are specified in the licence.'’
(Extracted with requisite revisions and edits from ‘Do Hand Sanitizers Require A
Drug License To Be
Sold In Stores?’
86. Dr. Bhaumik, a renowned scientist in the field of development of Medicines,
amongst the growing distress
as to the corona virus and the escalation in the prices of sanitizers decided to
research on affordable
hand sanitizers. After working hard for day and night, he was able to develop an
affordable hand sanitizer
called Jalipattan having an efficacy higher than the present sanitizers against the
corona virus. He does
not have any license for the manufacture of such hand sanitizers. Will this action
amount to violation of
Section 18(c)?
(a) No, there is no manufacturing of drug for sale or stock or distribution or
exhibition and hence there is
no violation of Section 18(c).
(b) No, Dr. Bhaumik is a scientist and is authorized to invent and discover drugs
even without any license
to that effect.
(c) Yes, the manufacturing of drugs without license will lead to sale of spurious
drugs having ill effect on
public health.
(d) Yes, there was manufacture of the drug without a license being issued for the
same.
87. A nearby chemist shop seeing the shortage of certain drug, Wizer which used to
increase immunity
amongst the patients who were infected with the virus wanted to manufacture and
make such drug
available in his shop. He filed for procurement of a license with specification of
this drug. However, there
was certain delay and the vial of this drug had a short shelf life, so knowing that
the drug would otherwise
be wasted, decided to start selling the drug with a discount too. After a few days
he received the license
to manufacture and sell the drug. Was he permitted to make such sale before
obtaining the license?
(a) Yes, the act done by him in good faith in order to tackle the shortage of the
drug.
(b) No, according to Section 18(c) no person can manufacture any drug for sale or
distribution unless he
has the license for the same.
(c) Yes, he obtained the license later and there was no prohibition or issues
preventing the sale by him
under the license.
(d) No, unless the Central or State Government permits, no drug or cosmetic can be
sold.

. Page 23 of 40
88. Vizer, due to its short shelf life has to be disposed off within 10 days after
the vial was opened. Rajesh,
working in the hospital was given the responsibility to dispose off such vials once
expired. However, he
mixed certain other compounds with these vials to increase its concentration and
efficacy and then sold
those vials before the 10th day after such opening of the vial. This drug was sold
in a medical shop of his
brother who had a license to manufacture Vizer under form 20 A and sell the vial of
Vizer and who was
aware that it was an altered form of Vizer sold it in his shop. Is there any
violation of Section 18(c)?
(a) No, the shop where the drug was sold had the license to sell Vizer and hence
there is no violation of
Section 18(c).
(b) Yes, the drug was manufactured by his brother without any scientific expertise
to guarantee a positive
impact.
(c) Yes, a license shall be valid for only such drugs as are specified in the
license and the manufactured
drug by Rajesh is a different drug than Vizer.
(d) Yes, the drug was manufactured by taking into account the shortage of drug and
heavy demands
without any profit motive.
89. Anuradha, amongst the ongoing fear of shortage of hand sanitizers bought 30
bottles of Hand Sanitizer,
Jalipattan from a local chemist shop. But as the covid situations plummeted, she
decided to give half of
these bottles to her friend Arshi. Through the act of such distribution, will this
be a violation of Section
18(c)?
(a) No, there is no element of manufacture as required under Section 18(c).
(b) Yes, there is element of stocking of the Hand Sanitizers which is a requirement
to invoke Section
18(c).
(c) No, she distributed with sanitizer in good faith because covid situation
pulmmeted; .
(d) Yes, there is an element of distributing sanitizer by a non-licensed person
which will invoke the
liability under Section 18(c).
Passage (Q.90-Q.94): The Supreme Court has upheld a judgment of the Madras High
Court which held
that an ex-parte decree (when one of the parties to the suit is not present) passed
against a minor not
represented by a guardian who is duly appointed is a nullity.
A bench comprising Justices Indira Banerjee and V Ramasubramanian was considering
an appeal filed
against the High Court judgment, which had set aside the ex-parte decree passed
against a minor on the
ground that he was not represented by a guardian appointed in terms of procedure
contemplated under
Order XXXII, Rule 3 of the Code of Civil Procedure, wherein upon the appointment of
guardian the Court
had to be duly informed by the Guardian about such appointment. The Court also held
that the failure to
appoint guardian ipso facto will result in prejudice to the minor and it need not
be specially established.
Further, the Supreme Court agreed with the High Court's view that the decree as a
whole required to be
set aside and not merely against the minor defendant.
(Extracted with requisite revisions and edits from ‘Ex-Parte Decree Against Minor
Not Represented By A
Duly Appointed Guardian A Nullity: Supreme Court’ at
90. A suit seeking the specific performance of a sale agreement was decided by the
CourtTrial Court against
the defendants ex parte. A revision application under Section 115 was preferred
before the High Court of
Madhya Pradesh against the refusal of the CourtTrial Court to set-aside the ex-
parte decree. The
CourtTrial Court had dismissed the application to set aside the ex-parte decree on
the ground of
unexplained delay of 862 days in preferring the application. One of the defendants
in the suit was a
minor. While considering the revision, the High Court had summoned the records to
ascertain if the
guardian was duly appointed as per procedure, it was found that there was no proper
appointment of
guardian. Without going into the reasons for delay, the Court sets aside the decree
of the Trial Court as a
nullity. Is the action of the High Court correct?
(a) Yes, the decree was passed against the minor who was not properly represented
by a guardian duly
appointed.
(b) No, the Court had failed to take into account the excessive delay of 862 days.
(c) Yes, one of the defendants involved is a minor and no action can lie against
the minor.
(d) No, since explanation to justify the delay has been made by the defendants.

. Page 24 of 40
91. Based on the facts of previous questions, it was contended by one of the
parties that the defendants
were grossly negligent, first in defending the suit, later in defending the
executing proceedings and then
in seeking to set aside the ex parte decree. Based on the contention, should the
initial decision by the
High Court to set aside the decree change?
(a) Yes, this was the case of gross negligence as the defendants were negligent not
only in defending
the executing proceedings but also in seeking to set aside the ex parte decree.
(b) Yes, as explanation to justify the delay and the negligence has not been made
by the defendants.
(c) No, the decree was passed against the minor who was not properly represented by
a guardian duly
appointed.
(d) No, one of the defendants involved is a minor and no action can lie against the
minor.
92. It was further contended that it was not even a ground raised by the defendant
in their revision petition
before the High Court that the procedure prescribed under Order XXXII, Rule3 of the
Code was not
followed nor that a grave prejudice or injustice has been caused to the
defendant/minor, on account of
the failure of not being represented by a Guardian in Court. Will this change the
fate of the decision by
the High Court as initially decided for setting aside the decree?
(a) Yes, the defendants have failed to argue or protest that proper procedure has
not been followed or
that a grave prejudice or injustice has been caused to the defendant/minor.
(b) Yes, as explanation to justify the delay and the negligence has been made by
the defendants.
(c) No, the decree was passed against the minor who was not properly represented by
a guardian duly
appointed.
(d) No, one of the defendants involved is a minor and no action can lie against the
minor.
93. It was contended by the Petitioners that instead of setting aside the decree in
totality the Court should
only set aside those part of the decree which affects the rights of the minor as
otherwise it will cause
gross injustice to them. The High Court denied accepting their argument. Is the
High Court correct in
doing so?
(a) No, the part which will affect the minor is easily severable with the decree in
totality.
(b) No, gross injustice will be caused to the Petitioners if the decree is set
aside in totality.
(c) Yes, the decree was passed against the minor who was not properly represented
by a guardian duly
appointed.
(d) Yes, one of the defendants involved is a minor and no action can lie against
the minor.
94. During the proceedings, it was found that Mr. Anirvan (uncle of the minor) was
appointed as the guardian
of the minor in the proceedings as per the procedure laid down in Code of Civil
Procedure. However, due
to certain health issues another person, Mr. Nirvan (another kin of the minor)
represented the minor in the
proceedings without intimating the Court about this. Should the Court set aside its
initial decision of
setting aside the decree as the minor was now represented by a guardian?
(a) Yes, the minor is now represented by a guardian and is thus not a nullity.
(b) No, as the appointment of Nirvan was not as the procedure laid down in Code of
Civil Procedure, for
the Court was not intimated of his appointment.
(c) Yes, Mr. Nirvan was also a next of kin to the minor, thus would represent minor
case appropriately
(d) No, Mr. Nirvan had failed to intimate about the change in guardian representing
the minor.

. Page 25 of 40
Passage (Q.95-Q.99): The Lakshwadeep Development Authority Regulation (“LADR”) in
the
Lakshadweep islands allows the administrator to constitute planning and development
authorities for the
purpose of planning the development of areas which he or she identifies as having
‘obsolete
development.’ The regulation defines development as the carrying out of building,
engineering, the
cutting of a hill or any portion thereof, or the making of any material change in
any building or land, etc.
What makes this regulation even more draconian is the power that it vests upon the
authorities to
forcefully relocate the indigenous groups residing in areas that fall under the
proposed development
plans.
While Article 240 of the Indian Constitution allows the president to introduce a
regulation that has the
effect of amending certain parts of central laws applicable to the Union Territory,
the provision allowing
forceful eviction and relocation goes against the very object and purpose of the
Land Acquisition Act,
2013, which was enacted to ensure a humane, participative, informed, and
transparent process for land
acquisition.
Furthermore, the Lakshadweep Prevention of Anti-Social Activities Regulation
bestows upon the
administrator a range of overreaching powers, including the authority to pass an
order detaining an
individual for up to 1 year “with a view to preventing him from acting in any
manner prejudicial to the
maintenance of public order.” Section 6 of the regulation stipulates that such an
order shall not be
invalidated by the fact that some of the grounds are ‘vague,’ ‘non-existent,’ ‘non-
proximately connected to
the person,’ or ‘invalid for any other reason whatsoever.’
The above regulation not only bans cow slaughter, but it also prohibits the natives
from buying,
consuming, selling, transporting or storing beef products in any form. It further
stipulates that any violation
of the same could invite a jail term of up to 10 years, Such actions impinge upon
the right of the
indigenous populations to practice and profess their cultural traditions and
customs, as has been
recognized under Article 11 of the United Nations Declaration on the Rights of
Indigenous
Peoples(UNDIRP).
95. Zubair and his family live in Kavarati, the capital of Lakshadweep islands. he
is a fisherman from the local
tribe and makes a decent living from fishing, recently the local authorities have
forced him and his family
to stop fishing near the coast of Kavarati and vacate their land under the new LADR
laws citing
construction of a resort. His main source of income is threatened. Was the act of
the government legal
according to the LADR?
(a) Yes, the action of the administration is legal according to the newly passed
law as the locals can be
evicted for development purposes and building falls under this category.
(b) No, the LADR law is illegal as it encroaches upon the necessities and means of
revenue of the locals
hence this eviction order is also illegal.
(c) No, construction of a resort would not fall under the category specified under
the LADR , hence the
eviction is illegal
(d) Yes, the actions of the government are definitely legal as tourism as an
important economic activity
and the new laws just aim to promote it.
96. Zubair and his family are deeply aggrieved by the land eviction under the LADR
act and approach the
Kerala High Court seeking justice under the Land Acquisition Act, 2013. What would
be the verdict of the
Court?
(a) Under the LADR act, Zubair and his family must forgo their land for the
development in public
interest, and public interest always supersedes individual interest.;
(b) The Kerala High Court would rule in favor of Zubair and his family as according
to the Land
acquisition act of 2013, he and his family cannot be evicted as the decision that
authorities took didn’t
involve participation from both parties.
(c) The Kerala High Court would rule against Zubair as he was not evicted illegally
or inhumanely by the
government of Lakshadweep and the Land acquisition act of 2013 would not apply;
(d) The high Court would rule in favor of Zubair as his right to land and work
freely was violated. The
Court would stop the eviction order.

. Page 26 of 40
97. Rafi is the member of the Koya tribe in Lakshadweep islands. On the 21st of
March every year all
members of the tribe in a ceremony consume beef and pray to god to easen their
sufferings, as a part of
their custom. According to the new rules under the Lakshadweep Prevention of Anti-
Social Activities
Regulation beef consumption is banned in the islands; the administration hence bans
the locals from
consuming beef. what is the validity of this action under UNDIRP?
(a) No, the action of the administration would be legal as Article 240 grants the
right to introduce relations
that regulate a Union Territory; hence they can ban beef;
(b) No, the action of the administration would be legal in nature as it is in no
way affecting the culture and
lifestyle of the locals and they can switch to other options.
(c) Yes, the action of the administration would be barred as it clearly violates
the rights of the indigenous
populations to practice and profess their cultural traditions and customs
(d) Yes, the action of the administration is definitely violative of UNDRIP as it
is violative of right to
choose their food, as this was recognized in the right to privacy judgement of
Puttuswamy.
98. In the Union Territory of Lakshwadeep, the government decides to create a new
naval base around the
coast. To accomplish this task the government has decided to clear out 250 huts of
the local farmers and
first the government informed them of their plan, entered into a discussion with
people for implementation
of the plan, offered them a period of one year to relocate along with a
compensation of 35 lakh rupees
and alternate employment in the government services. While most of the locals
decide to relocate, five
odd families approach the High Court under the land acquisition act of 2013 citing
inhuman behavior and
forceful eviction. What would the Courts rule?
(a) The Court would rule in favor of the farmers as their right on their own land
was violated and the
treatment meted out to them was clearly unjust and illegal;
(b) The Court would rule in favor of the local families as they cannot be evicted
under the Land
acquisition act of 2013;
(c) The Court would rule against the families as development of the naval base
comes within the ambit of
security of Union Territory;
(d) The Court would rule against the local families as they process was humane,
participative, informed
and transparent as they were given compensation and relief for the said acquisition
and relocation.
99. Rashid is a High school teacher at Lakshadweep High School and had voiced his
dissent on these
recently laid down laws with respect to beef consumption, forceful acquisition of
land to his colleagues.
Pursuant to which he was planning to organize a protest in the capital of the Union
Territory to voice the
concerns of the local tribal population in a peaceful manner. Before the protests
could take place, he is
jailed on the grounds of spreading dissatisfaction in the local masses, was this
arrest legal under the
Lakshadweep Prevention of Anti-Social Activities Regulation?
(a) Yes, as per the Lakshadweep Prevention of Anti-Social Activities Regulation
this arrest of Rashid is
legal as he was a threat to public order, as he was planning to protest.
(b) No, the arrest is completely illegal as public order is a very broad term and
protests does not disturb
the public order of a place, hence the treatment meted out to Rashid is unjust;
(c) Yes, Rashid violated the peace and security of the island by creating
unnecessary fear in the minds
of the people hence he should be charged under the Lakshadweep Prevention of Anti-
Social
Activities Regulation
(d) No, every individual has the right to freedom of speech and expression, he
can’t be arrested he the
right to voice his opinions.

. Page 27 of 40
Passage (Q.100-Q.102): The Haryana State Employment of Local Candidates Act 2021,
is a recent
example of a regional identity enforcing law. It becomes crucial to examine the
legality of such laws from
a constitutional lens. The employer, at his discretion, can restrict the employment
from a particular district
to 10% of the total local candidates to be employed.
Section 5(2)(iii) of the act says: the Designated Officer after inquiry and
evaluation of the application for
exemption, may direct the local candidates to acquire the desired skills and
proficiency. The employer will
have to bear the cost of training the local candidate for him to acquire skills to
increase his employability.
Even when the lack of merit causes the employer to reject the candidate, he can be
forcefully directed by
the Officer to train the concerned candidate and employ him. Lack of a skilled
workforce can adversely
affect the productivity of an industry, which will decrease its competitiveness as
compared to other
industries.
In the most fundamental sense, by following systematic exclusion of non-Haryana job
applicants, this act
is not in line with the concept of equality and thereby violates Article 14.
Discrimination of any citizen on
grounds of religion, race, caste, sex or place of birth is prohibited under Article
15(2). Indeed, the State
can make special provisions for people of Socially and Economically Backward
Classes, Scheduled
Castes and Scheduled Tribes. However, this legislation does not focus on securing
employment for the
youth belonging to these classes exclusively.
Article 16(2) holds discrimination on the grounds of place of birth and residence
unconstitutional. Justice
M. Hidayatullah in the case, A.V.S. Narasimha Rao &Ors. v. State of Andhra Pradesh
and Anr said,
“Parliament exclusively can make any law which prescribes any requirement as to
residence within the
State or Union territory prior to employment or appointment to an office in that
State or Union territory.”
100. Santhals a Scheduled Caste tribe is a tribal community in the state of
Jharkhand, they have been
historically subjected to discrimination and slavery. Till date social indicators
of the community remain
below the state average levels. Moreover, they still face social exclusion and
harassment based on
identity. The chief minister of the State Government has recently announced 5
percent reservation in the
state services for the Santhals of Jharkhand. This reservation policy has been
taken to Court on grounds
of being unconstitutional as it discriminates based on residence as alleged by the
Petitioner. What would
the Courts rule?
(a) The Courts would uphold the reservation policy as states can make special
provisions for people
who’ve been socially and economically backward. Hence the Santhal community which
is a schedule
caste can take the benefits of the reservation.
(b) This step by the Jharkhand government is violative of the constitution as it
violates Article 14 which is
right to equality.
(c) This action of the Jharkhand is inherently discriminatory in nature as it
discriminates on the basis of
residence which is prohibited by the constitution.
(d) The reservation is provided to compensate for social exclusion and
discrimination the community,
hence it would not be unconstitutional.
101. Rajesh is a businessman and owns several factories across the state of
Haryana. His factory makes
cycles and supplies them across the nation, majority of his workers are from the
neighboring state of
Uttar Pradesh and Bihar, after passing of the Haryana State Employment of Local
Candidates Act 2021,
Rajesh was forced to fire majority of his employees and hire local candidates from
Haryana, the locals
don’t possess the requisite skills to work for him , he has asked the government
for exemption, what
recourse does he have?
(a) He can ask for compensation from Haryana government for the loss suffered to
him for forcibly firing
his staff, against his will.
(b) He can seek exemption as lack of a skilled workforce can adversely affect the
productivity of an
industry, which will decrease its competitiveness as compared to other industries.
(c) He can ask the Designated Officer to direct the candidates from Haryana to
acquire the skill of cycle
making, provided he bears the cost for the same.
(d) He can ask the Designated Officer to direct the candidates from Haryana to
acquire the skill of cycle
making, cost for the same will be borne by Designated Authority.

. Page 28 of 40
102. The state of Karnataka has passed a law under which all candidates wanting to
apply for jobs under the
State Government of Karnataka need to possess a domicile certificate of the state
of Karnataka, this
provision has been challenged in Court. The Petitioner has argued that the State
Government does not
have the jurisdiction to make residential qualification. What would the Courts
rule?
(a) The Courts would rule in favor of the Respondents that is the State Government,
as states have the
power and the autonomy to ensure socio - economical upliftment of the local
candidates.
(b) The Courts would rule in favor of the Petitioner as the legislative power to
create residential
qualification for employment is thus exclusively conferred on the Parliament.
(c) The Courts would rule in favor of the Respondents that is the State Government
as they are well
within their rights to provide reservation based on the domicile of the candidate.
(d) The state legislature does not have the jurisdiction as they don’t have the
powers to decide the
conditions for employing candidates.
Passage (Q.103-Q.105): In a counter-affidavit filed before the Delhi High Court
which is hearing
challenges to the new Information Technology Rules, the Union Government has said
that the Rules
seek to “prevent the misuse of the freedom of press by spreading misinformation”
and protect citizens
from fake news in the digital media space, which used to be largely unregulated.
The Information Technology (Intermediary Guidelines and Digital Media Ethics Code)
Rules, 2021, which
were notified in February, impose several obligations on online entities including
an obligation to take
down contentious content quicker, appoint grievance redressal officers and assist
in investigations.
While submitting that there have been past incidents of misinformation on digital
media leading to
disturbance of public order, the Centre appeared to place the onus of
misinterpreting news on the news
providers, saying, “Digital media allows sensational content to be re-circulated in
a different context
leading to misinterpretation by the audience, making it susceptible to being used
as fake news. However
the term fake news is not defined under law. ”
The Union has argued that as opposed to the traditional media, the reach of digital
media is far wider
“which makes it a powerful tool for information campaigns by foreign state and non-
state actors to
influence public opinion in any nation.”
“Online platforms, for commercial reasons, may have a tendency to retain the
consumer on their platform
for a longer period. This results in proliferation and spread of news content that
appears to be
sensational. The risk of false or misleading information is greater over the
internet as the same can be
spread rapidly within the society,”
103. X is a mischievous boy; he loves to play prank on people. He was also very
tech savvy and had his
meme pages in Instagram and Facebook with huge number of followers. One day he
posts a morphed
image of Indian soldiers with a caption that “our soldiers need our support in
these trying times of war”.
The followers of X went wild and the news spread like wildfire. Soon the news
reached Information and
Broadcasting Ministry, and when the ministry asked clarification from X, he says
“from the word war, I
meant the war against covid-19 pandemic”. In the light of above fact and passage
choose the best
option.
(a) The post of X is a fake news, and he is liable for the misuse of right to
freedom of press for it can lead
to sedition.
(b) The post of X is not a fake news, since there is no news in that, he simply
asked to support the
soldiers in the trying times of covid-19 pandemic.
(c) X can be held liable for spreading misinformation, as his post had the
potential to disturb public order
so much so that Ministry of Information and Broadcasting had to intervene.
(d) To prevent people like X from spreading sensational news is the sole reason for
bringing the new IT
rules in the picture to prevent mishaps.

. Page 29 of 40
104. Democratic TV is a news channel infamous for spreading misinformation. One day
they broadcast a
news piece that there has been a communal riot in Lucknow, which is true. This led
to the immediate
shutdown of the shops leading to severe economic loss for the businessmen. In the
light of the above
facts, choose the best option.
(a) The new IT rules are brought for these situations itself; Democratic TV will be
held liable for spreading
misinformation.
(b) Democratic TV will not be held liable for they reported truthful events, and
there was no
misinformation of any kind.
(c) Democratic TV is already infamous for spreading false information, the state
has full right under the
new IT rules to shut this digital media outlet.
(d) Democratic TV not being an online platform is free to publish any kind of news,
and can easily escape
from the clutches of the new IT rules.
105. Zen Zen, a newly established local newspaper company in the State of Bhopal
printed some news in a
newspaper. Considering their business is anew they have captured only the offline
market and not
entered the online market. They published a news about a famous business-man Mr.
Neeraj, which was
later found out to be baseless. Aggrieved by this, Mr. Neeraj approached the editor
to take suitable
recourse, so that the damage cause to him can be compensated. He even threatened
the editor that the
company will be liable under IT Rules, 2011; that strictly goes against spreading
false information. The
editor told Mr. Neeraj that they posted the information after a preliminary check,
without any bad
intention. Select the best option.
(a) The news agency shall be held liable under IT Rules, 2011 as it circulated fake
information.
(b) The news agency will not be liable, since they posted the information after
doing a preliminary check.
(c) The news agency will be liable, as they should have verified the information.
(d) The news agency will not be liable under IT Rules, 2011, as the act of
newspaper won’t come within
ambit of IT Rules.
. Page 30 of 40
SECTION - D: LOGICAL REASONING
Passage (Q.106-Q.108): Answer the questions based on the information given in the
passage.
Just In Time (J1T) is an inventory strategy implemented to improve the return on
investment of a
business by reducing in-process inventory and its associated carrying costs. The
process is driven by a
series of signals, which can be Kanban, that tell production processes when to make
the next part.
Kanban are usually ‘tickets but can be simple visual signals, such as the presence
or absence of a part
on a shelf. When implemented correctly, JIT can lead to dramatic improvements in a
manufacturing
organization’s return on investment, quality, and efficiency. Some have suggested
that “Just on Time’
would be a more appropriate name since it emphasizes that production should create
items that arrive
when needed and neither earlier nor later. Quick communication of the consumption
of old stock which
triggers new stock to be ordered is key to JlT and inventory reduction. This saves
warehouse space and
costs. However, since stock levels are determined by historical demand, if any
sudden demand rises
above the historical average demand, the firm will deplete inventory faster than
usual, thus delaying
delivery and this may cause customer dissatisfaction.
106. Which of the following statements, if true, would most strengthen the author's
argument?
(a) Depletion of inventory can result in substantial unpredictable delays in the
production process.
(b) Customer dissatisfaction can be solved with professional service standards and
prompt
communication.
(c) JIT is not very popular with customers since they do not have advance notice
about production
delays.
(d) Delayed product delivery is one of the common causes of customer
dissatisfaction.
107. Which of the following statements is the author likely to proceed towards?
(a) Japanese manufacturing companies have thus used the JIT strategy to stay ahead
of the
competition.
(b) JIT is the best inventory strategy for manufacturing companies because of the
advantages it provides.
(c) It is necessary that an organization pay attention to flexibility in the JIT
process.
(d) The interpretation of visual signals is extremely important for successfully
implementing the JIT
strategy.
108. Which of the following statements, if true, would most weaken the author's
argument?
(a) It is not necessary that JIT will cause delays in each production cycle.
(b) It is impossible to avoid delays caused by a sudden rise in demand when using
the JIT process.
(c) The customers are usually told of any possible delays well in advance.
(d) Most customers do not mind delays caused by JIT because they understand the
process and the vast
benefits that they get from it in the long term.
Passage (Q.109-Q.112): These questions are based on the following information.
The State Road Transport Corporation will show a seven-minute film on safety in 3
languages for the
benefit of passengers on board on all its luxury buses. Much like those placed in
aircraft, laminated safety
guides will be available for passengers, showing the layout of the bus and some
basic steps they could
follow to get out of the bus in case of an emergency.
(A) The passengers may follow instructions provided in case of an emergency.
(B) There were a number of accidents where passengers could not get out of a bus
while it was on fire.
(C) The Road Transport Corporation intends to ensure the safety of passengers in
case of an
emergency.
(D) Aircrafts give instructions similarly in case of an emergency.
(E) The existing passenger safety measures are probably have not been expressed
adequately.
(F) There was a demand for increased safety measures for passengers of luxury
buses.

. Page 31 of 40
109. Which of the following assumption(s) is/are implicit in the above passage?
(a) Only A
(b) A and F
(c) A and E
(d) E and F
110. Which of the following can be concluded from the above passage?
(a) A and B
(b) Only C
(c) C and B
(d) C and F
111. Which of the following can be inferred from the above passage?
(a) D and F
(b) A and B
(c) Only D
(d) Only B and E
112. Which of the following could be a possible reason for the above situation?
(a) B and F
(b) B and E
(c) D and F
(d) B, E and F
Passage (Q.113-Q.117): These questions are based on the following passage and the
statements that
follow.
To get Russia to change its policy towards Ukraine, the West should reassess its
current strategy of just
imposing sanctions, since it has been ineffective over the past 15 months. Although
such a policy was
successful in the past as in the case of Iran, does not mean it would work in
Russia’s case as well. Unlike
Iran, Russia is a geopolitical giant that still has substantial heft in Central
Asia and Eastern Europe.
(A) The West has imposed sanctions on Russia over the past 15 months.
(B) The West has imposed sanctions on Iran in the past.
(C) The West cannot apply the same strategy while dealing with countries as
different as Iran and
Russia.
(D) A country that is a geopolitical giant cannot be intimidated merely by applying
sanctions.
(E) In geopolitical terms, Iran is not as powerful as Russia.
(F) The West may move International Court of Justice to bring change in Russia’s
policy towards Ukraine.
(G) Evidence suggests that no country has ever remained unaffected by sanctions
imposed on them; the
impact is felt sooner for geopolitically smaller countries; whereas, a larger
geopolitical country may take
as much as two years before the impact is felt.
113. Which of the following is/are assumption(s) implicit in the passage?
(a) Only (G)
(b) Only (C) and (D)
(c) Only (A) and (E)
(d) Only (C), (D) and (E)
114. Which of the following can be concluded from the above passage?
(a) Only (D)
(b) Only (C) and (D)
(c) Only (C) and (E)
(d) None of these

. Page 32 of 40
115. Which of the following can be inferred from the above passage?
(a) Only (A) and (B)
(b) Only (A), (B), and (E)
(c) Only (B) and (E)
(d) Only (E) and (F)
116. Which of the following can be a possible consequence of the scenario given in
the passage?
(a) Only (C)
(b) Only (C) and (D)
(c) Only (D) and (F)
(d) Only (F)
117. Which of the following if TRUE would weaken the above argument?
(a) Only (D)
(b) Only (C) & (D)
(c) Only (G) & (C)
(d) Only (G)
Passage (Q.118-Q.122): Read the passage carefully and answer the questions that
follow.
When a scientific paradigm breaks down, scientists need to make a leap into the
unknown. These are
moments of revolution, as identified by Thomas Kuhn in the 1960s, when the
scientists' worldview
becomes untenable and the agreed-upon and accepted truths of a particular
discipline are radically
called into question. A particular and productive way of looking at the world turns
out to be erroneous in
its essentials. The great scientific revolutions - such as those instigated by
Copernicus, Galileo, Newton,
Lavoisier, Einstein and Wegener - are times of great uncertainty, when cool,
disinterested reason alone
doesn't help scientists move forward because so many of their usual assumptions
about how their
scientific discipline is done turn out to be flawed. So they need to make a leap,
not knowing where they
will land. But how?
To explain how scientists are able to make this leap, the philosopher of science
Bas Van Fraassen in The
Empirical Stance (2002) drew on Jean-Paul Sartre's Sketch for a Theory of the
Emotions (1939). Sartre
was dissatisfied with the major mid-20th-century theories about emotions
(especially those by William
James and Sigmund Freud) that treated emotions as mere passive states. You might
fall in love, or be
gripped with jealousy. It seemed that emotions happened to you without any agency
on your part. Sartre,
by contrast, held that emotions are things that we do. They have a purpose, and
they are intentional. For
example, when we get angry, we do so to seek a solution, to resolve a tense
situation.
In his view, emotions transform the world like magic. A magical act, such as
voodoo, alters the attitude of
the practitioner to the world. Magical spells and incantations don't change the
physical environment, but
they change our world, by shifting our desires and hopes. Similarly, emotions
change our outlook and
how we engage with the world. Applying this idea to scientific practice, Van
Franssen argues that
scientists draw on their emotions when dealing with new, bewildering ideas,
especially those that sprout
up during scientific revolutions.
118. Which of the following can be inferred from the given passage?
(a) Many beliefs held for centuries were dismantled when Newton proposed his
theories.
(b) According to Freud, there is a purpose to the existence of emotions.
(c) Logic alone is the driving force for bringing radical changes in the scientific
world.
(d) Our emotions have magical powers, which means that they cannot be explained by
science.
119. Which of the following, hypothetically, would weaken the hypothesis proposed
by Bas Van Fraassen?
(a) Many scientists have noted how their sense of awe drove their scientific work
in their autobiographical
writings.
(b) Some historians have argued that Darwin's Origin of Species became popular
because it relied on
romantic writing to explain new concepts.

. Page 33 of 40
(c) Einstein changed his field to Physics since Mathematics was much more
competitive and emotionally
taxing.
(d) Newton did his greatest work alone away from his peers as he did not want any
emotional
involvement.
120. Which of the following most accurately expresses the main point of the
passage's last paragraph?
(a) Voodoo magic is similar to science in the sense that both involve emotions to
bring change.
(b) Spells may not change the world around you, but science can.
(c) Emotions can change our perspective, which can be helpful in understanding
path-breaking scientific
concepts.
(d) Scientists don't use emotions to change the world, unlike how magicians use
spells to create a feeling
of change by transforming our expectations.
121. Which of the following, if true, weakens the theory given by Sigmund Freud and
William James according
to the passage?
(a) Scientists suspect that the brain's left frontal cortex is responsible for
feelings such as shame and
jealousy.
(b) Doctors now believe that pain plays an important role in our well-being as it
alerts our body against
possible harm.
(c) Serotonin is the hormone that controls your mood, and lower serotonin levels
can make you gloomy,
which in extreme cases leads to depression.
(d) All of the above.
122. "When a scientific paradigm breaks down, scientists need to leap into the
unknown." Which of the
following is not an example supporting this statement?
(a) Copernicus provided us with the radical idea that the earth revolved around the
Sun when everyone
believed that the Sun revolved around the earth.
(b) Columbus went on a voyage to search for India, leaping into the unknown and
discovering an entirely
new world.
(c) Newton gave us the law of gravity which changed the way we studied heavenly
bodies and had far#reaching consequences in physics.
(d) Darwin published his theory of evolution in the 19th century, which was such a
radical idea that even
the Catholic Church got involved.
Passage (Q.123-Q.126): Read the passage carefully and answer the questions that
follow.
Sleep is the balm that soothes and restores after a long day. It is largely driven
by the body's internal
clock, which takes cues from external elements such as sunlight and temperature.
The body's natural
sleep-and-wake cycle is reasonably attuned to a 24-hour period.
Disruptions of sleep are disruptive to the functioning of many body systems.
Learning, memory, stamina,
general health, and mood are all affected by sleep amount and quality. For many
people, sleep is elusive
or otherwise troubled. In fact, most people, at some point in their lives,
experience difficulty falling asleep
or staying asleep. Potential consequences of consistently poor sleep include
obesity, cardiovascular
disease, and diabetes. Sleep deprivation can also affect judgement and mental
acuity.
Sleep needs differ from person to person and across different age groups. One
person may need a full
eight hours, while another can function with less sleep. Note that falling asleep
as soon as one's head
hits the pillow is not proof that one is a good sleeper. It's more likely an
indication that an individual is
sleep deprived. In general, it should take about 10 to 20 minutes for a person to
drift off. But whether it
takes 20 minutes or 45, if one perceives that it is taking too long, that becomes
one's reality, as it spurs
anxiety about sleep.
Most people have heard the standard sleep hygiene advice: Make sure your bedroom is
cool and dark.
Use your bed only for sleep. Avoid caffeine from mid-afternoon on. And avoid all
screens for at least an
hour before turning in. Falling asleep in front a TV, as 61 percent of adults
confess to having done, is a
problem as well. The most common sleep difficulties are exacerbated by factors like
illness or stress.

. Page 34 of 40
123. Which of the following can be inferred from the passage?
(a) During one's lifetime, one cannot escape experiences related to difficulty
pertaining to sleep.
(b) People suffering from sleep disorders have their sleep-and-wake cycle
reasonably deviated from a
24-hour period.
(c) A person with 8-hours of sleep can be said to be well-rested and better slept
than one with 6-hours of
sleep.
(d) One is sleep deprived if one falls asleep as soon as one's head hits the
pillow.
124. Which of the following form the premise for the statement that "Falling asleep
in front of a TV, as 61 per
cent of adults confess to having done, is a problem as well"?
(a) TV screens' melatonin-inhibiting blue light, when it reaches the eyes, delays
sleep latency by an
average of 10 minutes.
(b) A proper bed is the most appropriate place for sleeping since it helps achieve
refreshing stages of
deep sleep.
(c) TV screen's light penetrates the eyelids, so the brain still processes being
exposed to light, disturbing
the body's internal sleep clock.
(d) TV engages our other sensory organs as well, apart from eyes via vision.
125. Which of the following can be the immediate course(s) of action, apart from
the ones mentioned in the
passage, to address the sleep disorders faced by many people?
(a) Practising meditation regularly to reduce stress.
(b) Wearing a sleep-tracking device that can record when one falls asleep and wakes
up and detect
interrupted sleep.
(c) Consulting a sleep expert.
(d) All (a), (b) and (c).
126. Which of the following would the author most likely agree with?
(a) An individual takes an average of 40 minutes to doze off once he hits the bed;
that individual is likely
to be facing a sleep disorder.
(b) A diabetic individual is most likely to be facing sleep difficulty.
(c) A judge didn't have his/her full quota of sleep last night; he/she is likely to
give a wrong judgement on
an important case today.
(d) None of (a), (b) and (c).
Passage (Q.127-Q.130): Read the passage carefully and answer the questions that
follow.
Smartphone overuse is increasingly understood to be associated with poor mental
health,
developmental, cognitive, and social outcomes, but little is currently known on the
mechanisms that
underpin addiction to screens.
In addition to leading various studies on the effects of smartphones on mental
health, my colleagues and
I in the Culture, Mind, and Brain lab study a range of automatic influences on
cognition and
consciousness like placebo effects, social and cultural influences on healing, and
hypnosis. Jay Olson, a
PhD researcher in our lab, hypothesized that hypnosis and smartphone use may share
common
'automatic' features. Following Olson's hunch, we hypnotized 641 students to
measure their
suggestibility, and also tested their smartphone addiction scores to look for a
possible link. The resulting
study led by Olson was published last month in Frontiers in Psychiatry.
What is hypnosis, and is there a link with smartphone use? Broadly speaking,
hypnosis refers to an
atypical state of attention and consciousness in which voluntary thinking and
action are bypassed by
suggestions - that is, verbal, visual, or other stimuli and cues that can modulate
bodily processes and
sensations that we cannot typically control voluntarily. Hypnotisability refers to
people's propensity to
respond to suggestions. Like extraversion or conscientiousness, hypnotisability is
a relatively stable
personality trait. Some people are highly hypnotizable, others less so. Olson
wondered if highly
hypnotizable people may also be more prone to problematic smartphone use. I
wondered if people who

. Page 35 of 40
are highly receptive to social influences (a trait hypothesized by some researchers
to be linked to
hypnotisability) may also be at higher risk of smartphone addiction.
Our study found a small but stable correlation between hypnotisability and
smartphone addiction. In other
words, hypnotisability appears to moderately predict the severity of smartphone
addiction. The correlation
held across many samples of the same procedure over the course of many months; it
is very unlikely to
be spurious.
127. Which of the following can be inferred from the given passage?
(a) Smartphones are a literal form of hypnosis.
(b) Little is known about why humans are so prone to smartphone addiction.
(c) Hypnosis and smartphone use appears to share opposite features.
(d) The relationship between hypnotisability and smartphone addiction exists, but
there is no causal
relationship between the two.
128. Which of the following can be the course of action to address smartphone
addiction, especially among
children and youth?
(a) They should be hypnotised against the excessive usage of smartphones.
(b) To restrict children from exposure to smartphone, parents need to completely
stop using
smartphones.
(c) We should understand the involuntary nature of smartphone use and plan for
protective and
promotive measures (cognitive behaviour therapy or mindfulness-based interventions)
that target the
automaticity of smartphone use.
(d) The children should be made aware of the number of suicide cases in children as
a result of
increased screen time.
129. The author says, "The correlation held across many samples of the same
procedure over the course of
many months; it is very unlikely to be spurious". Which of the following, if true,
would most strongly
weaken this statement?
(a) Many people are addicted to their phones, in the sense of reporting significant
impairments to their
quality of life as a result of using their phone too much.
(b) The probability of inaccuracy of result(s) increases with the increase in the
number of repeated trials
over the various samples due to involvement and change in external factors over
time.
(c) The rapid spread of misinformation and political polarization on social media
are increasingly
recognized to pose a major threat to our societies.
(d) Large surveys have identified a strong association between decreased face-to-
face interaction,
increased screen time, depressive symptoms, and suicidal tendencies in adolescents
over the past
decade.
130. Which among the following would the author most likely agree with on hypnosis?
(a) In the case of hypnosis, verbal, visual, or other stimuli and cues as
suggestions that can modulate
bodily processes and sensations, are superseded by voluntary thinking and action.
(b) The more people are hypnotizable, the more they are addicted to excessive
smartphone usage.
(c) Smartphone and screen addiction are increasingly understood to present a major
public health issue.
(d) Hypnosis and smartphone use appear to share common features, from automatically
viewing
notifications, automatic and mindless scrolling and browsing, and losing track of
time and the world
around us.
131. Shashi travelled 4km from his house towards North. Then, he took a left turn
and travelled 6 km,
thereafter 4km to his light, then 8km to his left and then 3km to his left.
Finally, he travelled 2 km to his
left to reach the market How far is the market from his house and in which
direction?
(a) 13 km, North-west
(b) 17 km, North-west
(c) 13 km, North-east
(d) 17 km, North-east

.
132. P is the sister of Q, who is the son of A and the brother of E, who is the
daughter of H, whose sister is R.
How is P related to R?
(a) Daughter
(b) Niece
(c) Aunt
(d) Sister
Direction (Q.133-Q.135): There are eight boys — P, Q, R, S, T, U, V and W — who are
to form two
teams of four boys each. T and U cannot be together in the same team. R and S must
be in the same
team. If V and W are in the same team, then U must also be in that team. V
133. In how many ways is it possible to form the teams?
(a) 12
(b) 8
(c) 11
(d) 6
134. If we know that W must always be with either V or Q, then how many possible
ways are there to form the
teams?
(a) 8
(b) 6
(c) 4
(d) None of these
135. Which of the following is a possible team?
(a) T, U, P and Q
(b) T, R, S and V
(c) U, R, S and Q
(d) None of these

mock 13
Directions (Q.1-Q.5): Read the following passage carefully and answer the questions
given below it.
Paragraph 1: If you’re unsure what your blood pressure levels should be, new advice
from the American
College of Physicians (ACP) and the American Academy of Family Physicians (AAFP)
may help. For
many doctors and consumers, uncertainty arose after the 2015 release of results
from a large clinical trial
called SPRINT. Those researchers advised that people with high blood pressure aim
for a systolic
(upper number) of less than 120 mm/Hg. That’s well below what’s recommended by many
expert groups
and the government.
Paragraph 2: But SPRINT included only people at high risk for cardiovascular
disease, so its findings —
and its researchers’ advice on blood pressure levels — didn’t necessarily apply to
those at lower risk.
And getting blood pressure below 120 (as the SPRINT researchers suggested) usually
requires a high
dose of medication or the use of multiple medications. Both significantly increase
the likelihood of serious
side effects, such as fainting from severely low blood pressure or kidney failure.
Paragraph 3: According to ACP and AAFP. If you are 60 or older and have no other
cardiovascular risk
factors (diabetes, high cholesterol, a smoking habit), these guidelines recommend
maintaining a systolic
reading below 150 mm/Hg. Here’s why: The two dozen studies reviewed for the
guidelines suggested
that aggressive treatment to get systolic blood pressure below 140 in people older
than 60 didn’t extend
life or reduce the number of heart attacks. It did, however, possibly lower the
risk of strokes.
Paragraph 4: The new guidelines also recommend that people older than 60 who are at
high risk for
cardiovascular problems or who previously suffered a heart attack or stroke
consider treatment with
medication when their systolic level reaches 140. However, the new guidelines leave
it up to people and
their doctors to determine exactly what blood pressure level works best for them.
That’s because some
people may be able to better tolerate the side effects that can occur with
aggressive medication therapy
to lower blood pressure. If you are younger than 60, the ACP/AAFP guidelines do not
include
recommendations for you. You can follow these 2014 guidelines from the Eighth Joint
National
Committee and the thinking of medical experts from Consumer Reports and Consumer
Reports Best Buy
Drugs: You’re considered to have high blood pressure if your systolic reaches 140
or your diastolic
reaches 90 or above. (Consumer Reports’ medical consultants also suggest a goal of
140/90 for adults of
any age who have diabetes and for those younger than 50 with chronic kidney
disease.) Reaching these
numbers calls for lifestyle changes such as sodium restriction, weight loss and
exercise, and if those
methods are ineffective, medication is required to lower blood pressure.
1. According to the 2nd paragraph of the passage, what's the disastrous result of
high doses of medication
or multiple medications to lower blood pressure?
(a) Either a cardiac arrest or a debilitating stroke
(b) Brain haemorrhage or slurred speech
(c) Paralysis or inability to use one's limbs
(d) Either kidney failure or fainting from low blood pressure
2. What do you understand by cardiovascular problems, as used in the 3rd paragraph
of the passage?
(a) Cardiovascular problems related to high and low blood pressures.
(b) Cardiovascular problems arise from a stroke.
(c) Cardiovascular problems are caused by kidney dysfunction or failure.
(d) Cardiovascular problems are related to the heart and the blood vessels.

. Page 3 of 40
3. According to the 4th paragraph, what advice does the passage give people to have
optimum blood
pressure?
(a) They should restrict intake of common salt, oils and sugar.
(b) They should go to the gym and reduce their weight drastically.
(c) They should adopt new lifestyles--the way they dress, eat.
(d) They should exercise, shed excess weight and restrict salt intake.
4. According to the last paragraph of the passage, why is blood pressure level left
up to people and their
doctors to determine?
(a) There can be a common blood pressure reading for all.
(b) Some people are better capable of tolerating the side effects of aggressive
medication.
(c) Some people have weaker tolerance levels than others have.
(d) Some patients are chronically allergic to medications.
5. The central idea of the passage, as understood from passage, is
(a) How to control high and low blood pressure.
(b) The need to regulate one's blood pressure.
(c) Guidelines about optimum blood pressure.
(d) What to eat and drink to regulate BP.
Directions (Q.6-Q.10): Read the passage carefully and answer the questions that
follow.
In 2014, a month-long bout of dizziness and vomiting brought a 24-year-old woman in
China to the
hospital. She was no stranger to these symptoms: she'd never been able to walk
steadily and suffered
from dizziness nearly her whole life. These were serious, debilitating symptoms.
And yet, they might have
seemed almost mild once CT and MRI scans presented a diagnosis: the woman was
missing the majority
of her brain - in a manner of speaking. Yes, most of the players on the brain's
'stage' were present: the
cerebral cortex, the largest, outermost part of the brain responsible for most of
our thinking and cognition,
was present and accounted for; the sub-cortex and the midbrain, with their myriad
functions involving
movement, memory and body regulation - also present; the brainstem, essential for
controlling breathing,
sleep and communicating with the rest of the body - present and accounted for.
But none of these arenas hold the majority of the brain's currency - neurons, the
cells that fire impulses to
transmit information or relay motor commands. This distinction goes to the
cerebellum, a structure
situated behind the brainstem and below the cerebral cortex. Latin for 'little
brain', the highly compact
cerebellum occupies only 10 per cent of the brain's volume, yet contains somewhere
between 50 and 80
per cent of the brain's neurons. And indeed, it was in this sense that the
hospitalised Chinese woman
was missing the majority of her brain. Incredibly, she had been born without a
cerebellum, yet had made
it through nearly two and a half decades of life without knowing it was missing.
Compare that with strokes
and lesions of the cerebral cortex, whose neuron-count is a fraction of the
cerebellum's. These patients
can lose the ability to recognise colours or faces and to comprehend language - or
they might develop
what's known as a 'disorder of consciousness', a condition resulting in loss of
responsiveness or any
conscious awareness at all.
Understanding consciousness might be the greatest scientific challenge of our time.
How can physical
stuff, for example, electrical impulses, explain mental stuff, for example dreams
or the sense of self? Why
does a network of neurons in our brain feel like an experience, when a network of
computers or a
network of people doesn't feel like anything, as far as we know? Alas, this problem
feels impossible. And
yet, an unmet need for progress in disorders of consciousness, in which the
misdiagnosis rate is between
9 and 40 per cent, demands that we try harder. Without trying harder, we'll never
know if injured patients
are truly unconscious - or unresponsive but covertly conscious with a true inner
life. Without this
knowledge, how can doctors know whether a patient is likely to recover or whether
it's ethical to withdraw
care?

. Page 4 of 40
6. Identify the statement(s) that is/are correct as per the information given in
the passage.
(a) Cerebellum known as the ‘midbrain’ contains all the neurons of the brain.
(b) The woman missing the majority of her brain is talked about only in a
metaphorical sense.
(c) There are situations in which it is right to discontinue medical services for
patients.
(d) The woman was experiencing the symptoms of vomiting and extended period of
dizziness for the first
time in the year 2014.
7. It can be inferred from the passage that
(a) There is a direct mapping between symptoms and diagnosis.
(b) Cerebral cortex is more important than cerebellum.
(c) Misdiagnosis rate between 9 and 40 percent may not be alarming.
(d) CT and MRI reports are inconclusive.
8. The author is most likely to agree with which of the following?
(a) Consciousness is still a mystery for the scientists and doctors alike.
(b) The neuron count of a part of human body is directly proportional to the
severity of the condition in the
situation of loss of it.
(c) The conscious brain is like a democratic society; the unconscious brain like a
totalitarian society.
(d) It is impossible to survive if majority of the neurons are dysfunctional.
9. Which of the following question(s) can be answered from the information given in
the passage?
(a) Can electrical impulses in the brain explain the stuff that dreams are made
upon?
(b) Why can the cerebellum be lost or lesioned without affecting the conscious
mind?
(c) What is the position of brainstem relative to cerebral cortex in the brain?
(d) Did the Chinese woman get the treatment in accordance with the diagnosis of her
condition?
10. Which of the following conveys the state of mind of the author towards the last
paragraph?
(a) Emotional (b) Nonchalance (c) Disquietude (d) Equanimity
Directions(Q.11-Q15): Read the passage carefully and answer the questions that
follow.
Some say language evolved by firelight, with our ancestors sharing stories deep
into the night. Others
suggest it began as baby talk, or as imitations of animal calls, or as gasps of
surprise. Charles Darwin
proposed that language started with snippets of song; Noam Chomsky thought it was
just an accident,
the result of a freak genetic mutation.
Proposals about the origins of language abound. And it's no wonder: language is a
marvel, our most
distinctive capacity. A few slight movements of tongue, teeth and lips, and I can
give you a new idea,
whisk you somewhere else or give you goosebumps. Any thought a human can think, it
would seem, can
be shared on a puff of air. Explaining how this all started has been called the
'hardest problem in science'
and it's one that few can resist. Linguists, neuroscientists, philosophers and
primatologists - not to
mention novelists and historians - have all taken cracks at it.
Over this long and colourful history, one idea has proven particularly resilient:
the notion that language
began as gesture. What we now do with tongue, teeth and lips, the proposal goes,
and we originally did
with arms, hands and fingers. For hundreds of thousands of years, maybe longer, our
prehistoric
forebears commanded a gestural 'protolanguage'. This idea is evident in some of the
earliest writings
about language evolution, and is now as popular as ever. Yet even as the popularity
of the 'gesture-first'
theory has surged, its major weakness - a flaw some consider fatal – has become all
the more glaring.
Early proponents of 'gesture-first' ideas appealed to the intuition that bodily
communication is primitive. In
his Essay on the Origin of Human Knowledge (1746), Etienne Bonnot de Condillac
imagined a boy and a
girl, alone after a deluge, struggling to invent language anew. He described how
the boy, wanting to
obtain some out-of-reach object, 'moved his head, his arms, and all parts of his
body', as if trying to

. Page 5 of 40
acquire it. And the girl got the message. A scene very much like this can be
readily seen today, of course:
a baby in highchair wriggling in the direction of a toy just beyond her grasp. Part
of the primitive aura of
gesture - for Condillac and other early thinkers - stemmed from the observation
that gesture precedes
speech in infancy. Before children can talk, they point, nod, wave and beg.
Perhaps, goes the logic, the
development of language in our species followed this same sequence.
11. Identify the statement(s) that is/are correct as per the information given in
the passage.
(a) It is easier to create signals with one's hands than with one's voice.
(b) Language is what separates humans from others.
(c) There is historical proof that undermines the 'gesture-first' theory of the
origins of the language.
(d) A pliable theory does not have any major flaw.
12. It can be inferred from the passage that
(a) Every human thought can be communicated using language.
(b) Only a few have been interested in the origins of language owing to its
complexity.
(c) Among the theory proposed for origins of language, 'gesture-first' theory has
the least flaw among all.
(d) Trivial changes in gestures emanates a diverse range of emotions.
13. "Linguists, neuroscientists, philosophers and primatologists - not to mention
novelists and historians -
have all taken cracks at it. What can be inferred from the above line?
(a) Linguists, neuroscientists, philosophers and primatologists - not to mention
novelists and historians
have joked about it.
(b) Linguists, neuroscientists, philosophers and primatologists - not to mention
novelists and historians
have derided every attempt at theorizing the evolution of language.
(c) Linguists, neuroscientists, philosophers and primatologists - not to mention
novelists and historians
condemned almost every theory on the evolution of language.
(d) Linguists, neuroscientists, philosophers and primatologists - not to mention
novelists and historians
attempted to put across their theory on the evolution of language.
14. What is the contextual meaning of the following as used in the passage?
Gasp
(a) Say something while catching one's breath.
(b) Be desperate to obtain or consume.
(c) A convulsive catching of breath.
(d) The point of exhaustion or completion.
15. Which of the following question(s) can be answered from the information given
in the passage?
I. What is the major weakness of the gesture-first theory of origins of language?
II. If language began with gestures, why speech dominates communication?
III. With which of the proposals about the origins of language, the author is most
likely to agree?
(a) I and II only
(b) III only
(c) I only
(d) II and III only
. Page 6 of 40
Directions (Q.15-Q.20): Read the passage carefully and answer the questions that
follow.
Desiree's face became suffused with a glow that was happiness itself.
"Oh, Armand is the proudest father in the parish, I believe, chiefly because it is
a boy, to bear his name;
though he says not - that he would have loved a girl as well. But I know it isn't
true. I know he says that to
please me. And mamma," she added, drawing Madame Valmonde's head down to her, and
speaking in a
whisper, "he hasn't punished one of them - not one of them - since baby is born.
Even Negrillon, who
pretended to have burnt his leg that he might rest from work - he only laughed, and
said Negrillon was a
great scamp. Oh, mamma, I'm so happy; it frightens me."
What Desiree said was true. Marriage, and later the birth of his son had softened
Armand Aubigny's
imperious and exacting nature greatly. This was what made the gentle Desiree so
happy, for she loved
him desperately. When he frowned she trembled, but loved him. When he smiled, she
asked no greater
blessing of God. But Armand's dark, handsome face had not often been disfigured by
frowns since the
day he fell in love with her.
When the baby was about three months old, Desiree awoke one day to the conviction
that there was
something in the air menacing her peace. It was at first too subtle to grasp. It
had only been a disquieting
suggestion; an air of mystery among the blacks; unexpected visits from far-off
neighbours who could
hardly account for their coming. Then a strange, an awful change in her husband's
manner, which she
dared not ask him to explain. When he spoke to her, it was with averted eyes, from
which the old love#light seemed to have gone out. He absented himself from home;
and when there, avoided her presence
and that of her child, without excuse. And the very spirit of Satan seemed suddenly
to take hold of him in
his dealings with the slaves. Desiree was miserable enough to die.
She sat in her room, one hot afternoon, in her peignoir, listlessly drawing
through her fingers the strands
of her long hair. The baby lay asleep upon her own bed. One of La Blanche's little
quadroon boys stood
fanning the child slowly with a fan of peacock feathers. Desiree's eyes had been
fixed absently and sadly
upon the baby. She looked from her child to the boy who stood beside him, and back
again. "Ah!" It was
a cry that she could not help; which she was not conscious of having uttered. The
blood turned like ice in
her veins, and a clammy moisture gathered upon her face.
16. We can infer from paragraph one that
(a) The wife does not know her husband too well.
(b) Negrolon had to resort to extreme measures to take respite from the work.
(c) Armand Aubigny was a magnanimous employer.
(d) Desiree wanted a girl instead of a boy.
17. Which of the following describes the character of Armand Aubigny?
(a) Armand Aubigny was a nobleman who had kindness in his heart for everyone.
(b) Armand Aubigny was a god-fearing man who gave prime importance to his family.
(c) Armand Aubigny was a domineering man who was severe in demands or requirements
(d) Armand Aubigny was soft-hearted towards his family but was autocratic towards
his workers.
18. "…unexpected visits from far-off neighbours who could hardly account for their
coming." What is the
author implying through the statement?
(a) The visitors were unknown to the owners who could give no reason for their
visit.
(b) The visitors came to see the mother and the child to bless them.
(c) The visitors came from long distances but gave a flimsy excuse for their
presence.
(d) The visitors had no business with the family, and their presence raised
questions among the local
neighbours.

. Page 7 of 40
19. "The blood turned like ice in her veins, and a clammy moisture gathered upon
her face." Which of the
following words, out of the given options, sums up the meaning of the given
sentence?
(a) Consternation
(b) Bewilderment
(c) Equanimity
(d) Disillusionment
20. Which of the following is not supported by the passage?
(a) Madame Valmonde was Desiree's mother.
(b) The change in the husband's manner was satanic
(c) Desiree was aware of the reason for her husband's change in behaviour.
(d) Desiree was a meek woman who cantered her happiness around her husband.
Directions (Q.21-Q.25): Read the passage carefully and answer the questions.
In the United States, there is a certain stigma behind both public and private
schools. From the ones
portrayed in movies and television, to the rumours of other schools that circulate
through school systems,
most can be written off as not true. The same applies to private and government-run
schools in India,
where private schools are pulling away as the most common form of schooling. This
bodes the question:
Which is better?
The first topic to cover is foreign language. Obviously, students are taught in
local languages, usually
also offering Hindi, Urdu, and/or Sanskrit as secondary languages. These are spoken
by plenty of Indian
citizens, but where private schools go above and beyond by teaching other
languages. English, even
other foreign languages that are taught in America are available in private
schools.
Next up are uniforms. While uniforms are required in all schools in India, private
schools go above and
beyond. They check up on student’s hygiene, uniforms, and general cleanliness to
make sure they are up
to par.
Extracurricular activities are a lot more common and well-developed in private
schools as well. Simple
things like electricity, proper classrooms, and even proper buildings are not
available. While this may not
sound like much, this could totally change the course of all students’ learning
careers.
Additionally, the staff and faculty of private schools are fully accountable for
the children that they teach.
Their grades, behaviour, and well-being of students are in the hands of the school.
However, due to the competitive nature of choosing a school, publicly funded
through the government or
privately owned, there are sometimes very hard choices for parents to decide where
their child should go
to school. Sadly, the quality of education does come down to the area and which
school exactly that the
parents choose, and can range very drastically from good to bad. Especially in more
rural areas of India,
with less options and less quality schools, it can be hard to find the perfect
school. In the long run, as one
may guess, money really pays off in a students’ education. If you are paying for
school in India, it will
most likely be better than the public schooling option in the same area.
To sum up, a student’s best bet is most likely to go to a private school in India,
then government schools,
and then no schooling if unable to afford it. Private schools, because of the extra
money, drastically shift
what is taught, how well it is taught, and how much faculty cares about the
students. India’s government
should financially support public schooling more so now than ever to educate
children and provide a safe
place to learn for all.
21. In paragraph one, the author does which of the following?
(a) The author draws an analogy to introduce a new concept.
(b) The author superimposes one over the other to prove his point
(c) The author draws a parallel to put forward his argument.
(d) None of the above.

. Page 8 of 40
22. Which one of the following is not the reason for private schools faring better
than the government
schools?
(a) The foreign languages such as English and German.
(b) Salubrious conditions of the private schools.
(c) Extracurricular activities of the private schools.
(d) The staff and the faculty of the private schools.
23. What is the approach taken by the author in writing the passage?
(a) The author critically analyses the reasons private schools doing better than
government schools.
(b) The author condemns the governments in lackadaisical approach towards
government schools.
(c) The author provides solutions to combat the sordid condition of public schools.
(d) The author is apologetic to the readers for the matter at hand.
24. “While this may not sound like much, this could change the course of all
students' learning careers."
Which of the following (s) words can replace the underlined word, keeping the
coherency of the sentence
intact?
I. When
II. Though
III. Yet
IV. Rather
V. However
(a) I, II & V
(b) II, V
(c) I, III, IV
(d) II, III,
25. Which of the following is the most appropriate title for the passage?
(a) Private mayhem.
(b) American Vs India.
(c) Government schools Vs private schools.
(d) Private schools Vs government schools
Directions (Q.26-Q.30): Read the passage carefully and answer the question based on
it.
The Compagnie de Indes, similar to the British East India Company, was established
long before
colonization in 1644. They acquired a French trading post in an area in 1674 around
Putucéri. Fittingly,
France called it Pondicherry. Despite a few years of European rule throughout the
years, up until 1954
France had owned the colony, and pictures of it show the inspiration. French
buildings, Government
Square, libraries, language, food, and even some art still shows inspiration from
the time and area.
François Martin, the governor of the French East India Company, set up a trading
centre in Pondicherry,
becoming the chief of French settlement in India. Moving forward, in the early 18th
century, the company
acquired other territories in Mahe, Yanam, and Karaikal.
Great Britain eventually captured all of India in the 1850’s, however allowed the
French to keep all of their
territories. Once India gained independence in 1947, this contradicted that all of
India was under their
own rule. The inhabitants of Pondicherry and the other territories were to choose
what they wanted to do,
and to no surprise, they would choose to be a part of India.
The whole time, however, India and Pondicherry were being used by the French to try
to expand trading
and subtly fight against the power that Britain had in India. The maritime commerce
and trading that
Pondicherry brought were a benefit, and education, industry, and some
infrastructure was a step up from
the previous system. While treaties were being written between the French and
British fighting over who
owned what land, in Europe, the two countries were feuding, especially during the
French and American
Revolutions.
Pondicherry was actually used as a spot for Indian revolutionaries to trade and
stay away from British
forces in pursuit to become a free nation. Formed by three centuries of French
rule, Pondicherry and its

. Page 9 of 40
other territories under French rule were impacted greatly by it. Not only did the
culture and look of it
change, but was a place to plan an eventual rebellion that would free the entire
country of India. Under
multiple European rules, these territories reaped a better education system,
favourable trading with
Europe, and a new style of life (linguistically, culturally, etc.) that cannot be
found anywhere else in India.
Overall, while colonization is not the best, especially for countries that want to
become their own nation,
in this instance, French colonialism in Pondicherry was a cultural change that
partially benefitted natives,
and now that it is under Indian rule, creates a vastly different area in Southern
India to look at for French
culture of the past.
26. The author, through the passage
(a) gives a historical account of how Pondicherry originated.
(b) provides a historical account of the influence of French culture in Pondicherry
because of French
Colonialism.
(c) gives a historical understanding of the trading similarities between the French
East India Company
and the British East India Company.
(d) provides a historical account of the relationship between India and France over
the trade-in
Pondicherry, once India gained independence from British Rule.
27. Which of the following inference(s) can be drawn from the passage?
i. The inhabitants of Pondicherry decided to merge with India without a referendum.
ii. François Martin became the first Governor of the French East India Company and
set up a trading
centre in Pondicherry.
iii. The author was not happy with British Colonialism but had no issues with
French Colonialism.
(a) Only ii
(b) i and iii
(c) Either i or ii, but not iii
(d) None of the following.
28. Which of the following does not find support in the passage?
(a) The Compagnie de Indes was a colonial commercial enterprise.
(b) The French East India Company ruled till 1954.
(c) France and British were clashing, especially during the French and Indian
Revolutions.
(d) Great Britain spread its wings throughout India in the 1850s but allowed the
French to keep their
colonies.
29. “Pondicherry was used as a spot for Indian revolutionaries to trade and stay
away from British forces in
pursuit to become a free nation.” The statement implies that
(a) Pondicherry was the breeding ground of Indian revolutionaries.
(b) Pondicherry was a vantage point for the Indian revolutionary in their fight
against British imperialism.
(c) The French inhabitants of Pondicherry were hostile towards the Indian
revolutionaries.
(d) Pondicherry was used by the British to curb the rising of the Indian
revolutionaries and mitigate any
attempt of strengthening the trading.
30. “…French colonialism in Pondicherry was a cultural change that partially
benefitted natives, and now that
it is under Indian rule, creates a vastly different area in Southern India to look
at for French culture of the
past.” The underlined part can culminate the passage as furtherance of which of the
following subject
matter?
(a) A tourist attraction to savour a colonial mini-France.
(b) A reminiscence for the inhabitants of the French people.
(c) A rejuvenation of Pondicherry as a trading centre between India and France.
(d) A cultural exchange for the Indian and French historians
Directions (Q.66 – Q.105): Read the comprehensions carefully and answer the
questions based on it.
Passage (Q.66-Q.70): ‘Conversion Therapy’, ‘Aversion Therapy’ or ‘Reparative
Therapy’ refers to the
pseudo-scientific practice of attempting to change an individual’s sexual
orientation or gender identity.
Queer people were subjected to extremely traumatising procedures such as corrective
rape and electro#convulsive therapy.
Several bodies of the UN have released statements and studies to prove that same-
sex attraction was
not a mental illness and there is no evidence of an individual’s sexual orientation
being cured by
treatment. They are believed in India as well.
In Laxman Balkrishna Joshi v Trimbak Bapu Godbole, the Supreme Court held that a
doctor who agrees
to treat a patient implies that he has the knowledge and skill for the purpose and
owes a duty of care as
to whether or not to take up the case. A breach of this duty gives rise to medical
negligence. Importantly,
any form of conversion therapy would constitute violation of the fundamental rights
of an individual, In
Navtej Singh Johar v Union of India, the ruling in the Naz Foundation case was
upheld by the Supreme
Court.
Similarly, in Common Cause v Union of India, the Supreme Court held that every
person of sound mental
faculty has the right to make their own decisions pertaining to medical treatments
and their health.
Another course that may be taken against Conversion Therapy is proving that it is
consumer fraud. There
is conclusive evidence that conversion therapy is ineffective and has been
medically disproven. Hence,
claiming to provide a service that is impossible would amount to consumer fraud.
This was the reasoning
used in the Ferguson v JONAH judgment of the New Jersey Superior court and the same
can be applied
to India as well. Conversion therapy would amount to an offence with imprisonment
up to 2 years or fine
up to Rs.10 lakh as per Section 89 of the Consumer Protection Act,2019.
https://rmlnlulawreview.com/2021/07/08/supreme-courts-vinod-dua-verdict-
significant-victory-for-freedom#to-dissent-or-missed-opportunity/
66. Robin is a doctor, he is very renowned in his community and people from across
the state come to
consult him. Recently, Robin has found out that her adult daughter is suffering
from a curable but a
cancerous disease. Robin wants to cure her daughter via surgery as he is sure his
treatment would be
best for his daughter, her daughter refuses and wants to follow the guidelines
provided by other doctors.
Robin is pained by this and during a regular check-up at his clinic performs the
surgery on his daughter
which was successful. Yet she is devasted, can she sue her father?
(a) No, robin is a renowned doctor with a great following across the state, he
knows what is best for his
daughter, and he did the surgery in good faith hence he will not be liable.
(b) He will certainly be liable and can be sued as he did not take the consent of
his daughter depriving
her of her right as per the Common Cause v Union of India case. Robin would be
liable.
(c) Robin would not be liable as he can take decisions on behalf of his daughter
and in this case the
guidelines provided by the Common Cause v Union of India would not apply.
(d) Robin would be liable as he did not possess the adequate skills to carry out
the surgery and he
should have respected his daughters point of view.
67. Rosesh is a homeopathic doctor who believes that he can apparently ‘cure’
people of homosexuality and
has been using this argument to gain traction around the country. A family believes
these claims to be
true and enrols a member in one of his programmes. The individual had to face
trauma in this camp and
eventually sues Rosesh for medical negligence, Rosesh claims he is not guilty. What
will the courts rule?
(a) He will not be liable as he tried is his best and he believed he had the skills
to carry out this treatment.
(b) He will be liable as there is no ‘treatment’ that cures homosexuality, he
claimed to have skills which
he did not possess hence as per the Laxman Balkrishna Joshi v Trimbak Bapu Godbole
case he will
be liable.
(c) He will not be liable as he did not commit these acts with malice in mind but
out of good faith.
(d) None of the above.

. Page 17 of 40
68. As per the above mentioned facts, can Rosesh also be liable under the Ferguson
v JONAH judgment?
(a) No, it would not be correct to sue him under consumer fraud, he will be liable
only for medical
negligence for his act.
(b) Yes, he will be liable for consumer fraud but not medical negligence as he did
what he could to
according to his abilities.
(c) Yes he will be liable for consumer fraud as well as medical negligence as his
acts go against
established Supreme Court judgements.
(d) No, he will not be liable for both offences as medical negligence would also
cover his offence under
consumer fraud hence he shall not be charged under Ferguson vs Jonah
69. Akhil is a 25 year old man, due to a severe accident several parts of his brain
were damaged because of
which he has developed this disorder where he has spells when is unable to
comprehend his thoughts
and decisions accurately. Recently Akhil had to take an important career decision
but because his was
not in a right state of mind at that point, he mother stepped up and chose on his
behalf. After the passing
of the spell his is annoyed on his mother’s decision and decides to sue her. Will
she be liable?
(a) Yes, she will be liable as she did not do what was in the best interest of
Akhil at that time due to
which he had to suffer consequences.
(b) No, she will not be liable as she took the decision without any malice or ill
will and for his benight.
(c) Yes, she will be liable because as per the Common Cause v Union of India, Akhil
was able to take
decisions when he was in the right state of mind so she should have waited.
(d) No, because as per the Common Cause v Union of India, Akhil was unstable during
these spells
caused by his disorder. In these spells her mother can decisions on his behalf and
they would not be
illegal.
70. In the above question, Akhil is currently in the right state of mind but his
mother is worried about him and
decides that she will be taking all of his decisions on his behalf now onwards,
Akhil is annoyed by this.
On the judgement of the Common Cause v Union of India, what would the courts course
of action?
(a) The courts would rule in favour of the Akhil’s mother as she should definitely
have right to take
decisions on her son’s behalf because his mentally unstable.
(b) No, Akhil’s mother cannot take decisions on Akhil’s behalf as he is unstable
during some short spells
only and can decide for him when he is of a stable mind.
(c) The courts would ideally rule in favour of Akhil’s mother as she is a well-
wisher and would not harm
him in any way.
(d) The courts would rule in favour of Akhil as he should have the autonomy to
decide what is best for
him and the decision of Common Cause v Union of India would apply here.
Passage (Q.71-Q.74): The Supreme Court ruled that reservation will be applicable to
persons with
disability (PwD) even during promotions in employment.
It also said that the mode of recruitment – whether or not if the employee
concerned was recruited under
the disability quota or not – is no ground to reject reservation benefits to a
person as long as they are
‘disabled’ at the time of availing promotion. The court further noted that the
absence of rules enunciating
reservation benefits during promotions is no justification to deny rights granted
under the Persons with
Disabilities (Equal Opportunities, Protection of Rights and Full Participation)
Act, 1995.
“Source of recruitment ought not to make any difference but what is material is
that the employee is a
PwD at the time for consideration for promotion”.
Relying on the judgment in the Union of India vs. National Federation of the Blind
(2013), the judges
opined that reservation has to be computed with reference to the total number of
vacancies in the cadre
strength and no distinction can be made between the posts to be filled by direct
recruitment and by
promotion. It further said that if the 1995 Act confines only to recruitment and
not promotions, then it
would negate the provisions of the legislative mandate.

. Page 18 of 40
[Extracted with revisions from ‘Quota for Persons with Disabilities Extend to
Promotions, Not Just
Recruitment: SC’, published 29 June 2021 on The Wire https://thewire.in/law/quota-
for-persons-with#disabilities-extend-to-promotions-not-just-recruitment-sc]
71. Mukesh was born without a left leg. However, he never let his disability become
a handicap and he was
able to do most things with practice over a period of time. He was very inclined
towards sports and could
manage to participate in most sports except activities like running, which required
two legs. He was
selected as a sports teacher in a government school and the children were very
inspired by Mukesh.
However, Mukesh did not receive any promotion despite excelling at his job, while
his peers were
promoted with every Pay Commission Report. Can Mukesh sue the school?
(a) No, because sports teachers must be equipped with complete limb functionality
in order to be eligible
for promotions.
(b) No, because Mukesh was not recruited under the disability quota.
(c) Yes, because Mukesh continues to be a disabled person while consideration for
promotion.
(d) Yes, because no distinction can be made between the posts to be filled by
direct recruitment and by
promotion
72. Rajni worked as an assistant accountant at the Department of Revenue. She was
given employment in
the general category. In 2020, she did not receive any promotion in the March
round. In April, she met
with an accident and lost her left hand, which had to be amputated. She now
contests that she is eligible
for reservation in promotions. Is the Department of Revenue entitled to promote her
at this time?
(a) Yes, because she is disabled at the time for consideration for promotion.
(b) Yes, because quota is no ground to reject reservation benefits to a person as
long as they are
‘disabled’ at the time of availing promotion.
(c) No, because she was not disabled at the time for consideration for promotion.
(d) No, because she was recruited as an employee under the general category.
73. Sunil was a famous wrestler. In 2021 Tokyo Olympics, he received a gold medal
in the sport but severely
injured his knee in the process. Despite several surgeries, he will never be able
to fold his leg ever again.
The government gave him employment in an administrative branch on compassionate
grounds under the
PwD quota. Will he be entitled to reservation at the time of promotion?
(a) Yes, because he was admitted under the PwD quota and reservation in promotions
can only be given
to those who were recruited under such quota.
(b) Yes, because he would be disabled at the time of consideration for promotion.
(c) No, because he would not be disabled at the time of consideration for
promotion.
(d) No, because although he was admitted under the PwD quota, it is irrelevant for
the purposes of
promotion.
74. Leesamma Joseph, whose permanent disability was assessed at 55%, was employed
in the Police
department in 2006 as a typist on compassionate grounds after her brother’s death
in the army. Upon
subsequent promotions and seniority, she was appointed as a cashier on May 5, 2018.
She was not
promoted commensurate to government policy because she was not recruited under the
PwD quota.
However, Joseph argued that she was entitled to promotion as a senior clerk with
effect from July 1, 2010
with all consequential benefits, and as a cashier with effect from May 20, 2016
with all attendant benefits
and thereafter as junior superintendent with effect from the date of her
entitlement based on the
respective Pay Commission Reports. If the alleged grounds are true, can the state
deny her claim?
(a) Yes, because Joseph was disabled at the time of availing promotion i.e., from
2006 to 2018
(b) No, because Joseph was not disabled at the time of availing promotion i.e.,
from 2006 to 2018
(c) Yes, because Joseph was not disabled at the time of availing promotion i.e.,
from 2006 to 2018
(d) No, because Joseph was disabled at the time of availing promotion i.e., from
2006 to 2018

. Page 19 of 40
Passage (Q.75-Q.79): While deciding whether issuance of a legal notice and filing
of a complaint would
amount to abetment of suicide, the Delhi High Court observed that filing of a
criminal complaint by the
petitioner in the case was his legal recourse, as advised to him
The case stemmed from a suicidal death of a person who was in the business of
vintage motorcycles.
When the purchaser did not receive the motorcycle, he was said to have shot off a
legal notice to the
deceased on his visit to India in 2014.
The petitioner subsequently filed a criminal complaint at the local police station.
The Court was informed
that the man left India on the intervening night of December 5 and 6, 2014.
However, on December 9,
2014, the deceased was stated to have died by suicide and left behind a note
blaming the petitioner for
taking such an extreme step.
The Delhi High Court narrowed down the issue to whether the issuance of a legal
notice and filing of a
complaint by the petitioner would amount to abetment of suicide under Section 306
of the Indian Penal
Code.
"It has to be shown that the petitioner did an active/direct act which led the
deceased to commit suicide,
seeing no option. Also, it has to be shown that the petitioner's act must have been
intended to push the
deceased into such a position that they committed suicide. Further, the prosecution
has to show that the
petitioner had the mens-rea to commit the offence,"
The Court opined that abetment involved a mental process of instigating a person or
intentionally aiding a
person in doing a thing.
Source: https://www.barandbench.com/news/litigation/legal-notice-filing-complaint-
abetment-of-suicide#delhi-high-court
75. Abhishek was a shopkeeper in the Somwara locality of New Chhattisgarh. He used
to give out supplies
for a credit period of 10 days owing to the locality all knowing him and him being
the only shopkeeper in
that area. However, after the pandemic hit and everyone was financially crippled, a
huge sum of his credit
was wiped out, he was led to committing suicide. In his note, he had implicated
everyone who had taken
goods on credit for him, which led to them being questioned by the police. Would
this amount to
abetment of suicide?
(a) No, as there was no active act in furtherance of which the suicide had taken
place.
(b) Yes, as the act of taking goods on credit and not paying the amount is an
active act.
(c) No, as the shopkeeper was not obliged to offer credit services.
(d) Yes, as the locals had an obligation to pay him back, which resulted in his
death.
76. In the above case, upon committing suicide, it was found that the note implied
one particular person who
had borrowed 65% of the total borrowings, which had led to him defaulting on his
payments and rents,
and subsequently, to him committing suicide. Would this amount to being abetment to
suicide?
(a) Yes, as the unpaid dues led to him succumbing to his expenses and,
subsequently, dying.
(b) No, as a debtor could not have foreseen the consequences of defaulting on his
payments.
(c) Yes, as the letter clearly denotes the abetment.
(d) No, as the shopkeeper should have foreseen the consequences of giving out huge
credit.
77. In the above case, the shopkeeper was told by the aforementioned creditor to go
die since he won't
receive the money back no matter how much he needs it. Due to this, he committed
suicide and
mentioned him as being the reason for committing suicide. Would this amount to
abetment in this case?
(a) Yes, as the suicide is a clear outcome of the dismissal of payment by the
creditor.
(b) No, as the suicide cannot be accorded to the non-payment.
(c) Yes, as the suicide in question is a direct outcome of the inability of the
shopkeeper to pay his dues.
(d) None of the above.

. Page 20 of 40
78. Aryaman was a 14-year-old living with his parents, who were very strict about
his exams and academics.
He was repeatedly scolded for not scoring enough with finally an ultimatum that if
he didn't pass with
distinction, he would be sent to a boarding school. As a result, when he fails to
score distinction, he
commits suicide, fearing the reaction of his parents. Would this be termed abetment
to suicide?
(a) Yes, as his parents literally drove him to commit suicide.
(b) No, as they did not have the intention for him to commit suicide.
(c) Yes, as he was driven to committing suicide on not scoring distinction.
(d) None of the above.
79. Upon breaking into the flat of a priest who had not been answering any calls
for three weeks, the police
found him dead, hanging by the fan with a suicide note implicating his neighbor who
allegedly used to
consume drugs and other toxins, which were banned for consumption in his religious
beliefs. The
neighbor got arrested for abetment to suicide. Would the charges hold?
(a) Yes, as the neighbor violated his beliefs leading to his death.
(b) No, as the neighbor was not proximate to his death.
(c) No, as the neighbor did not have any intention to kill the priest.
(d) Both b & c.
Passage (Q.80-Q.85): The Rajasthan High Court has refused to extend police
protection to a couple in a
live-in relationship noting that since the woman was already married, granting
protection to the couple
may indirectly amount to giving the Court's assent to such illicit relations.
"It is well settled legal position as expounded by the Division Bench of Hon'ble
Allahabad High Court in
Smt. Aneeta. v. State of UP that live-in relationship cannot be at the cost of the
social fabric of this
country, and directing the police to grant protection may indirectly give our
assent to such illicit relations,"
the Court said in its order.
The petitioner in the matter, a married woman, was allegedly compelled to leave her
matrimonial house.
The petitioners alleged that the respondents were unhappy with the woman living
with another man and
were, therefore, threatening the petitioners. They, therefore, prayed that since
their life was in danger,
police protection may be granted to them. The Court said that giving protection may
amount to giving
assent to illicit relations and rejected the plea for grant of protection.
It, however, said that in case any offence is committed with the petitioners, they
are at liberty to lodge FIR
in a concerned police station or take recourse to available legal remedies.
Recently, the Allahabad High Court had also denied protection to a live-in couple
after finding that the
woman was married to another person, and the Court could not, therefore, permit
"illegality."
Source: https://www.barandbench.com/news/litigation/police-protection-married-
woman-live-in-relation#consent-illicit-relation-rajasthan-high-court
80. Raman and Meghna were married to each other but were plagued with an unhappy
married life which led
Raman to drink excessively and Meghna to looking for support elsewhere. This led
her to Shri, a college
student five years younger than her. They both developed a strong bond with each
other and often used
to spend nights talking to each other. When Raman found out about them, he
spiralled out of his senses
and vowed revenge against Shri. Owing to this, Shri and Meghna implored the police
to provide them
with protection against them. Will their pleas be allowed?
(a) No, as it would amount to a violation of the social fabric of the country.
(b) Yes, as they are not a live-in couple.
(c) No, as no act of violence has been committed yet.
(d) Yes, as their lives are endangered.

. Page 21 of 40
81. In the above case, Meghna and Shri had gotten into a relationship, and upon
getting to know of this,
Raman poured kerosene on Meghna and threatened to set her alight. Would the police
be entitled to
provide them with their protection in this case?
(a) Yes, as there exists a very real threat to Meghna.
(b) No, as it would amount to promoting illicit adulterous behaviour.
(c) Yes, as only Meghna is threatened and not the couple as a whole.
(d) No, as the couple can only seek court-ordered protection.
82. In the present matter, had the appeal been for the protection of Raman from the
extra-marital relations of
Meghna and Shri, would it have been granted?
(a) Yes, as he was being cheated on by the two.
(b) No, as there was no threat to his life by the two individuals.
(c) Yes, as the couple of Meghna and Shri was causing mental agony to Raman.
(d) None of the above.
83. Dama and Wahran are a married couple living together for three years now.
Wahran, the husband, has
recently taken a paramour, Zelda, with whom he has extra-marital relations unknown
to Dama. However,
despite his best efforts, Dama gets to know of his mistress and threatens to kill
her if he doesn't break up
with her and vows to sever all ties. Could the couple of Wahran and Zelda get
protection from judicial
authorities for the preservation of their relationship?
(a) Yes, as there exists a real threat to the life of Zelda and should be granted
protection due to the
same.
(b) No, as Zelda is the mistress of Wahran and is in an adulterous relationship
with a married man.
(c) No, as the adulterous couple being granted protection would violate the social
fabric of the country.
(d) Yes, as the people involved in the relationship are threatened with dire
consequences.
84. Arya and Gendry are a married couple living in Westeros, a backward country
where marrying into a
lower caste is frowned upon vehemently. Stannis is the party leader of the Azaad
Baratheon Party, who
strongly condemns inter-caste marriages. Upon getting to know of Arya and Gendry
being an inter caste
couple, Stannis plans to publicly shame them by painting their faces black and
marching them from the
city temple to the Palace. Seeking protection from this action of the ABP, Arya and
Gendry approach the
local police. Would they be liable to protect Arya and Gendry?
(a) Yes, as Arya and Gendry are innocent individuals who are being persecuted.
(b) No, as it is clear that the legal and social environment of Westeros is clearly
not supportive of inter#caste marriages.
(c) Yes, as the police are anyway supposed to protect the individuals in a society.
(d) None of the above.
85. In the above case, had Arya and Gendry been unmarried and of different castes
living together, would
they still have been accorded the police protection?
(a) Yes, as they are being persecuted for no fault of theirs.
(b) No, as it would violate the social fabric of the country of Westeros.
(c) Yes, as the social fabric of India and Westeros is not the same.
(d) No, as they are violating the accepted norms of their society.

. Page 22 of 40
Passage (Q.86-Q.91): Merely because an arrest can be made as it is lawful does not
mandate that it
must be made, the Supreme Court has said, while observing that personal liberty is
an important aspect
of constitutional mandate. The apex court said if arrest is made routine, it could
cause “incalculable harm”
to the reputation and self-esteem of a person.
A bench said if the investigating officer of a case does not believe that the
accused will abscond or
disobey the summons, he or she is not required to be produced before the court in
custody.
“We may note that personal liberty is an important aspect of our constitutional
mandate. The occasion to
arrest an accused during investigation arises when custodial investigation becomes
necessary or it is a
heinous crime or where there is a possibility of influencing the witnesses or
accused may abscond,” the
bench said in its order passed earlier this week.
The top court said the word ‘custody’ appearing in section 170 of the CrPC does not
contemplate either
police or judicial custody but it merely connotes the presentation of accused by
the investigating officer
before the court while filing charge sheet.
It noted that section 170 of the CrPC does not impose an obligation on the officer-
in-charge to arrest the
accused at the time of filing of charge sheet.
“We have, in fact, come across cases where the accused has cooperated with the
investigation
throughout and yet on the charge sheet being filed, non-bailable warrants have been
issued for his
production premised on the requirement that there is an obligation to arrest the
accused and produce him
before the court.”
Source: https://thewire.in/law/sc-personal-liberty-important-aspect-of-
constitutional-mandate-arrests#shouldnt-be-routine
86. In the investigation stages of a bomb blast in Mumbai, the Nagpara Police
station was made the primary
investigation agency and had been calling up all the prime accused people and
witnesses. One of these
accused was a local school teacher who had been diligently attending to the calls
of the authorities. Upon
being called for his testimony before the magistrate, he was escorted by the police
to the courtroom from
the school. Would this tarnish his reputation in the eyes of prudent public?
(a) Yes, as a school teacher being escorted by the police is bound to tarnish the
reputation of a person.
(b) No, as the people around him had no reason to think low of him.
(c) Yes, as anyone being escorted by the police is bound to affect the public
adversely.
(d) No, as the police are bound to escort a witness.
87. One of the accused in the above case has been residing near the police station
and thus the police have
not had any major difficulties making him comply with their requirements. However,
upon missing the
hearing when he is summoned, the police immediately arrest him from the place of
his residence and
produce him before the magistrate. Are they within their authority to do so?
(a) Yes, as the accused has clearly missed his legal obligation.
(b) Yes, as they possess the requisite authority to arrest and produce accused
individuals.
(c) No, as there is no reasonable inference of any threat from the individual.
(d) No, as the accused’s reputation would be severely tarnished at the sight of his
arrest.
88. In the above case, a prime suspect was produced before the magistrate and his
testimony was recorded,
however it was also pleaded by the suspect that he had been cooperating with the
police and did not
have to be handcuffed and arrested for his judicial obligations. Should the court
take any action in
furtherance of the same?
(a) Yes, as the pleading is reasonable for a person to save their reputation.
(b) No, as the pleading has been made at the wrong stage of the proceedings.
(c) No, as on account of being a prime suspect he was bound to be arrested.
(d) Yes, as the pleading has been made in furtherance of his reputation being
hampered.

. Page 23 of 40
89. Sarvesh, a prime accused in the case has been summoned to the magistrate’s
court on three separate
occasions but owing to his hectic business dealings he has had to travel a lot
domestically leading to him
missing out on the same, about which he has duly informed the authorities. As a
result of this, on his next
summon, the police specially decided to escort him to the courtroom by arresting
him under bailable
charges. Are the police right in doing so?
(a) Yes, as by allowing him to get bail they are not in contravention of reasonable
authority.
(b) No, as the act of arresting him would amount to being an excessive action.
(c) Yes, as his regular absence in obeying the judiciary is a ground for him to be
arrested.
(d) No, as he has not deliberately warranted any police action such as this.
90. Anup Dhirmani is a business magnate in India currently facing shareholder
embezzlement charges in the
Indian courts. Upon the filing of the FIR, he was swiftly arrested by the police,
arrested, and produced
before a magistrate within 4 hours of the same, because police received intel
regarding him planning to
abscond. This led to all the news channels and social media websites exploding with
this information on
account of his reputation as a business tycoon being ridiculed. Would he be liable
for a plea against this
action?
(a) No, as he has validly committed a crime and deserves the treatment that he is
being meted out.
(b) No, as the police are well within their authority to arrest him and produce him
before the magistrate
while handcuffed.
(c) Yes, as any arrest requires a 24 hour period after which the accused is to be
produced before the
judicial magistrate.
(d) None of the above.
91. In the above case, upon hearing of his grandson’s arrest, Jeejabhoi Dhirmani,
the founder of the
Dhirmani group, died of a heart attack. Now Mr. Dhirmani is pleading that the
police was in excess of its
authority insofar as in handcuffing him before his conviction. Can he plead for
relief?
(a) No, as his crimes have been substantiated.
(b) Yes, as he has been wrongly incriminated in this case.
(c) Yes, as he has not been convicted yet.
(d) No, as the death of his grandparent is not an action arising out of the police
action.
Passage (Q.92-Q.95): Once the fact of last seen is established, an adverse
inference can be drawn
against the accused if he fails to explain the circumstances in which he departed
the company of the
deceased, the Supreme Court reiterated.
The Court also noted that the conviction of the accused is based on circumstantial
evidence regarding (i)
Last seen theory; (ii) Motive & (iii) false information provided and subsequent
conduct of the accused.
The court observed that the last seen theory is applied where the time interval
between the point of when
the accused and the deceased were last seen together, and when the victim is found
dead, is so small
that the possibility of any other person being the perpetrator of crime becomes
impossible. The court also
observed that unless the fact of last seen is corroborated by some other evidence,
the fact that the
deceased was last seen in the vicinity of the accused by itself would be a weak
kind of evidence.
The prosecution had relied on case laws, to urge that in situations when the
deceased was last seen in
the company of the accused, a presumption would arise that the said accused
murdered the deceased. If
motive in a case is attributed to an accused(s) and thereafter proved, the
probability of the crime being
committed by the said accused is intensified.
[Extracted, with edits and revisions, from “If Last Seen Theory Is Established,
Accused Should Explain
Circumstances In Which He Departed Company Of Deceased: Supreme Court” LiveLaw News
Network,
LiveLaw, 5 August 2021;

. Page 24 of 40
92. Priyadarshani Mattoo is a law student enrolled in the law dept. of the Delhi
University. She is a bright
student, and catches the eye of Santosh Kumar who relentlessly pursues her despite
being turned down
by her multiple times. She files many complaints with the police and the University
authorities as well, but
to no avail. One night while she was alone in her residence, she was brutally
stabbed multiple times by a
knife, and then eventually died.
The police bring in Santosh Kumar, who had been seen lurking near her residence
half an hour before
the murder. When asked about his whereabouts during the crime, he admitted that he
was near the
deceased’s residence, but refused to reveal the reason for the same, stating that
they were ‘personal’.
Can he be convicted for the murder?
(a) Santosh may not be convicted, as the fact that the deceased was last seen in
the vicinity of the
accused by itself would be a weak kind of evidence.
(b) Santosh may be convicted as he does not successfully explain the circumstances
in which he
departed the company of the deceased.
(c) Santosh may be convicted, as the time interval between the murder and his last
seen whereabouts
was very small.
(d) Santosh may be convicted, as he failed to explain the circumstances around the
fact of last seen,
along with other evidence.
93. In the situation presented previously, had Rama, friend of the deceased, also
visited the residence
around three hours before the murder took place; would it be likely that she would
be held liable of the
murder?
(a) Yes, as she was seen mere two hours before the murder.
(b) No, but her statements would be valuable as evidence for the case.
(c) No, as the fact of the last seen cannot be established.
(d) If motive in a case is attributed to an accused(s) and thereafter proved, then
she could be held liable.
94. In the situation presented previously, there is another person who visited the
deceased a few hours
before her murder. An elderly cook, Ghanshyam, worked at her residence, and left
the place at noon.
However, the call records revealed that merely 20 minutes before the murder he
talked to her on call.
After being questioned, he revealed that it was to inform him of his absence the
next day. Would
Ghanshyam be likely to convicted for the murder?
(a) Ghanshyam would be liable, as he was last seen with the deceased before her
murder.
(b) Ghanshyam would not be liable, as the fact of last seen is not established.
(c) Ghanshyam would not be liable as he could successfully explain the
circumstances in which he was
last seen with the deceased.
(d) Ghanshyam would not be liable as the fact of last seen on its own is a weak
kind of evidence, and
hence not enough to guarantee a conviction.
95. Mr. A is a wealthy baron who lives in his residence with his wife, mother-in-
law, and brother-in-law. One
day, after he comes back from fishing, he has trouble breathing for which he takes
his medicine. After a
short while, he starts to cough violently and foam starts to come out of his mouth.
His mother-in-law
hears the commotion, and immediately proceeds to call a doctor. However, its too
late by the time the
medical help arrives, and Mr. A dies. When the police arrive to conduct their
investigation, the find out
that Mr. A’s brother-in-law, Mr. X, had recently fought with Mr. A to become the
sole nominee of his will,
which was then accordingly executed. Is there enough evidence to convict any of the
family members?
(a) The mother-in-law is likely to be convicted, as she was the one who last saw
the deceased.
(b) The wife is likely to be convicted, as she would be the one would benefit the
most out of Mr. A’s
death.
(c) The brother-in-law is likely to be convicted, as she would be the one would
benefit the most out of Mr.
A’s death.
(d) There isn’t enough evidence to convict any of the family members.
,

. Page 25 of 40
Passage (Q.96-Q.100): Delivering a judgment defining the contours of Section 15 of
the Unlawful
Activities (Prevention) Act, 1967, (UAPA) the Delhi High Court laid down some
important principles upon
the imposition of Section 15, 17 & 18 of the Act. While S. 15 engrafts the offence
of 'terrorist act', S. 17
lays-down the punishment for raising funds for committing a terrorist act and S. 18
engrafts the offence of
'punishment for conspiracy etc. to commit a terrorist act or any act preparatory to
commit a terrorist act'.
The phrase 'terrorist act' has been defined in a very wide and detailed manner
within S. 15 itself, in
court’s opinion, the courts must be careful in employing the definitional words and
phrases used in S. 15
in their absolute literal sense or use them lightly in a manner that would
trivialise the extremely heinous
offence of 'terrorist act', without understanding how terrorism is different even
from conventional, heinous
crime. The court relied on the Supreme Court's decision in the case of Hitendra
Vishnu Thakur, to hold
that, "the extent and reach of terrorist activity must not arise merely by causing
disturbance of law and
order or even public order. Where the court finds that an act or omission is
adequately addressed and
dealt with by the ordinary penal law of the land, the court must not countenance a
State agency 'crying
wolf'.
Defining the contours of "terrorist act" under S. 15 UAPA, the court said, "In our
opinion, the intent and
purport of the Parliament in enacting the UAPA, and more specifically in amending
it in 2004 and 2008 to
bring terrorist activity within its scope, was, and could only have had been, to
deal with matters of
profound impact on the 'Defence of India', nothing more and nothing less.
Shreya Agarwal, 'Terrorist Acts' Under UAPA Only Deal With Matters Impacting
'Defence Of India' And
Not Ordinary Law & Order Problems: Delhi High Court (Live Law, 15 June 2021)
96. Kunti and her associates were arrested by the police and charged with UAPA for
strategically targeting
the Central Government and criticizing its policies on social media. Police seized
documentations which
were found to be a toolkit, which was the formal design for assailing the
government policies. In the court,
the state argued that the act amounts to a terrorist act as it was meant to
incapacitate the government
which would render the nation defenceless. The court agreed with the contentions of
the state and
convicted Kunti and her associates for attempting to perpetrate terrorist
activities under section 18 of the
UAPA. Decide whether the decision has been righty made or not.
(a) Kunti and her associates have been rightly convicted as they intended to render
India defenceless by
launching a planned attack against the government of the nation.
(b) Kunti and her associates have not been rightly convicted as the toolkit was
meant as an attack on the
government policies and to criticize them on social media which is not same as
rendering India
defenceless.
(c) Kunti and her associates have been rightly convicted as strategically breaching
the government
authority and causing distrust against the government is a terrorist act.
(d) Kunti and her associates have not been rightly convicted as they shall rather
be convicted of the
offence of sedition for causing disapprobation against the government than under
UAPA.
97. In a town close to the city of Pune, hundreds of Dalits gathered to commemorate
200th anniversary of the
victory of a Dalit army over an upper caste peshwa. Opposition by some other upper
caste Hindus
agitated violence in the town followed by arsons and mass destruction of property.
Police has charged
the organization and the activists involved under UAPA for committing an act of
terrorism causing loss of
life and property. In the Court the defence argued that neither was the intent of
the celebration to
perpetrate terrorism nor did the aftermath of such celebration amount to an act of
terrorism. Choose the
correct option.
(a) The accused shall be convicted under UAPA as deliberate use of violence against
the state and state
machinery is an active terrorist act.
(b) The accused shall be convicted under UAPA as the offence led to heavy loss of
life and property
which is an act too grave to be tried as an ordinary offence.
(c) The accused shall not be convicted under UAPA as the act was just a
repercussion of the agitation
caused by the upper caste Hindus and not deliberate.
(d) The accused shall not be convicted under UAPA as violence caused in the present
case did not have
any impact on the defence of the nation.

. Page 26 of 40
98. With the intention of liberating the people of Ladakh valley from military
atrocities, Umar decided to join a
militant group that revolts against the army using violent measures. Umar was
determined that as soon
as the army backs off from the valley or eliminate the atrocious practices he would
leave the group.
However, during one such attack over the army he was arrested and charged for
perpetration of terrorism
by the State. In the Court, he stated his intention for using violent measures and
pleaded that he be
charged for general offences under Indian Penal Code and not for terrorism. Which
of the following is the
correct option?
(a) He shall not be charged under UAPA for committing terrorist activities as the
act was not intended to
be an attack against the Indian defence but merely a revolution against the
atrocious measures
adopted by the army in the valley.
(b) He shall be charged under UAPA for committing terrorist activities as the
attacks on the Indian army
would be a matter having profound impact on the Indian defence and thus a threat to
the national
security.
(c) He shall not be charged under UAPA for committing terrorist activities he
intended to leave the group
as soon as the army refrained from adopting any violence or atrocity in the valley
thus giving
evidence that he was merely leading a revolution which if violent shall be dealt
with under IPC and
not UAPA.
(d) He shall be charged under UAPA for committing terrorist activities as
revolution can be initiated and
continued in non-violent means and deliberately using violence ascertains it to be
a terrorist act.
99. Chellam planned inciting a riot in the state of Tamil Nadu by agitating people
over the miserable
conditions of Tamils in Sri Lanka using funds received from terrorist
organizations. On the day, when the
Sri Lankan president was in India, he made a speech which entailed mass violence
including killing of
politicians and heavy destruction of property. It took the state government three
days to supress the
violence. While Chellam was trying to escape he was arrested by the police and
charged for committing a
terrorist act. Contesting the charges against him, Chellam that he merely made a
speech which was
naturally followed by all the violence and thus he has not committed any act of
terror. Decide whether he
shall be found guilty or not.
(a) He shall be found guilty as even if an ordinary offence of causing public order
the riots were meant for
weakening relations of India with Sri Lanka thus decreasing Indian stature on an
international level.
(b) He shall not be found guilty as he disrupted public order and tranquillity by
his speech which is not as
heinous crime as committing deliberate terrorist act.
(c) He shall be found guilty as the riots were planned to cause mass violence and
disruption that led to
deliberate killings and destruction of property rendering India defenceless and
thus amounts to
committing a terrorist act.
(d) He shall not be found guilty as the source of the funds is immaterial if the
act committed was an
ordinary offence enumerated under Indian Penal Code.
100. Assertion (A): General Offences if committed by a person shall not be dealt
with under UAPA.
Reasoning (R): The Court has ruled that such attempts would trivialize the heinous
offences
characterised as acts of terror.
Choose the correct option in context of the above passage:
(a) Both A and R are correct and R is the correct explanation of A
(b) Both A and R are correct but R is not the correct explanation of A
(c) A is correct and R is incorrect
(d) Both are incorrect

. Page 27 of 40
Passage (Q.101-Q.105): Provisions authorising preventive detention cannot be
resorted to deal with
ordinary law and order problem, observed the Bombay High Court while setting aside
an order of
preventive detention passed by the Pune Police Commissioner under the Maharashtra
Prevention of
Dangerous Activities Act. A Bench of Justices SS Shinde and NJ Jamadar observed
that a proper test to
distinguish between the "law and order" and "public order" is two fold: whether the
complained acts lead
to disturbance of the ordinary tempo of life of the community leading to
disturbance of public order; or
whether it merely affects the individual leaving the tranquillity of the society
undisturbed. It is the
propensity and potentiality of the act, disturbing the even tempo of life the
community, that renders it as
prejudicial to the maintenance of public order, the Court said.
The Court opined that the provisions which authorise preventive detention cannot be
resorted to as an
"easy substitute" to deal with an "ordinary law and order problem". It added that
while the detaining
authority is empowered to detain a person, he ought to record a subjective
satisfaction based on the
material placed before it that the "acts and conduct attributed to the proposed
detenu are prejudicial to
the society and fall within the mischief of the provisions which empower the
detention". The Court applied
these principles to the present case to conclude that the objectionable acts
attributed to the petitioner
would fall within the dragnet of "law and order" and can be taken care of by
ordinary laws. "We find it
rather difficult to persuade ourselves to hold that the acts committed by the
petitioner were such that they
disturbed the public tranquility by creating a terror and panic in the society or a
considerable number of
the people in the said locality" the Court said.
Neha Joshi, Preventive detention law cannot be resorted to deal with ordinary law
and order problem:"
Bombay High Court quashes detention order (Bar and Bench, 25 June 2021)
<https://www.barandbench.com/news/litigation/preventive-detention-ordinary-law-and-
order-problem#bombay-high-court> as accessed on29 June 2021.
101. Arjun Gawli was local goon, who is under trial for stabbing two men in broad
daylight. He was released
on bail by the Court. On further inquiry by the police it was discovered that Arjun
and his associates had
been committing offences and were a perpetual danger to the lives and properties of
the people residing
within the local limits of the police station. They were detained, under
Maharashtra Prevention of
Dangerous Activities Act, recording the above mentioned reason post an order by the
Commissioner,
which was sanctioned by the government. Arjun has challenged the detention in a
Special Leave Petition
made before the Supreme Court. Choose whether the appeal shall be allowed or not.
(a) Arjun shall not be detained under the Act as he has already been released on
bail which means there
is no requirement for any custody.
(b) Arjun shall be detained as when the Police Department has the reasons to
believe that preventive
detention is required they enjoy absolute prerogative to do so.
(c) Arjun shall not be detained as the offence of murder is an ordinary issue of
law and order which does
not attract harsh measures such as preventive detention.
(d) Arjun shall be detained as he and his associates were a perpetual danger to
people and property of
the locality thus disrupting the public order at large.
102. Praful was arrested by Police for alleged murder of a businessman. His bail
was denied by the Lower
Court and he was sent to a judicial custody for 90 days. Soon before this period
was about to end, Praful
was detained under the State’s preventive detention statute. Under the law he could
be kept in custody
for a maximum period of a year. Praful has made an application before the Lower
Court for quashing the
order, contending that the detention order was passed without proper application of
mind by the
authorities. Choose the correct option.
(a) Application shall be allowed as the offence of murder does not disturb the
public order and thus does
not warrant a grave measure such as preventive detention.
(b) Application shall not be allowed as the offence of murder is threat to the
society that diffuses a threat
in every member of a locality and thus warrants preventive detention.
(c) Application shall not be allowed as Praful is not fit to be released in the
society which is evident from
his bail being released by the Lower Court.
(d) Application shall be allowed as Praful has already been in judicial custody for
90 days and there is no
further need for further custody.
. Page 28 of 40
103. Baba Samdev is a yoga guru, who following the footsteps of an anti-corruption
leader launched a
nationwide protest. He called for huge gathering at Jantar Mantar in Delhi, from
where the gathering
would move towards the Parliament. The government, apprehending that the protest
would widely disturb
the public order and could lead to violence, denied the request for the gatherings
and the protest.
Regardless of the rejection of the permission, Samdev carried out the gathering,
and as soon as the first
incident of violence was reported Police detained Samdev under Delhi Abolition of
Unlawful Activities Act.
Samdev has challenged his detention in absence of any commission of offence by him.
Choose the
correct option.
(a) Samdev has been rightly detained as his act of calling for a demonstration
despite denial of the
permission by the government causing public disorder.
(b) Samdev has been wrongly detained as right to protest is a fundamental right
which cannot be
restricted unless warranted by an urgent issue of public order.
(c) Samdev has been rightly detained as his acts led to violence during the
demonstration causing
immense public disorder which warrants preventive detention of Samdev.
(d) Samdev has been wrongly detained as it was not him who personally committed any
act of violence
and thus cannot be detained for the offence committed by another.
104. Assertion (A): Any matter of ordinary law and order cannot attract the need
for preventive detention by
the authorities.
Reasoning (R): Only when an offence is capable of causing mass disorder concerning
the security of
entire nation does it require a step as grave as preventive detention.
(a) Both A and R are correct and R is the correct explanation of A
(b) Both A and R are correct but R is not the correct explanation of A
(c) A is correct and R is incorrect
(d) Both A and R are incorrect
105. Salman was a Sri Lankan militant who is alleged of being involved in bomb
blasts in the Sri Lankan
capital. Indian Intelligence received the intel that Salman is planning an attack
in India. Later, he was
nabbed by the Police buying unregistered weapons in the national capital, Delhi. He
was detained under
the Terrorists Act. He made an application in the Court stating that buying
unregistered weapons is an
ordinary offence under the IPC and does not warrant preventive detention. Decide
whether the
application shall be allowed or not.
(a) Application shall be allowed as buying unregistered arms is only an ordinary
offence which cannot be
dealt with by preventive detention of Salman.
(b) Application shall not be allowed as all the facts suggest the potential of
Salman buying the weapons
for an attack on India which is capable of disrupting the entire public disorder.
(c) Application shall be allowed as mere intel is not a conclusive proof that an
act by Salman would
cause any disruption to the even tempo of the life of the community.
(d) Application shall not be allowed as Salman is international terrorist who can
be detained regardless of
his committing or not committing act that warrants such a measure.

. Page 29 of 40
SECTION - D: LOGICAL REASONING
Directions(Q.106-Q.110): Read the passage carefully and answer the questions that
follow.
Athletes get penalized for using performance enhancing drugs. But what about using
nootropics while
working towards exams - or when working in general? Is it ethical, and even
healthy, to use them to
improve cognitive skills?
Nootropics are compounds and supplements that supposedly improve cognitive
abilities. They get their
name from Ancient Greek words: 'νόος' (pronounced 'nóos') and 'τροπή' (pronounced
'tropḗ'), and come
in three forms. These are: stimulants, also known as 'smart drugs' (such as Ritalin
and Adderall),
synthetic compounds (like Noopept and racetams like piracetam), and natural
compounds (such as
caffeine, and herbal ginseng).
While many are thought to be safe for general use, like caffeine and ginseng,
others, such as Ritalin
(methylphenidate) and Adderall (a combination of amphetamine and dextroamphetamine)
are
prescription drugs used to treat sleep disorders and ADHD. Due to limited research
on most nootropics,
little is Known on exactly what they do in the brain. While some are known to
improve skills like attention,
memory, and general processing, their exact results vary from person to person,
depending on genetics,
neurochemistry, general cognitive performance, and more.
Despite this, nootropics are popular for enhancing brain function. Research has
shown that anywhere
between 8% and 35% of college students admit to taking 'study drugs' to enhance
their academic
performance. Meanwhile, a study showed that almost a third of parents think that
stimulants Ritalin and
Adderall boost their children's academic performance, even if they don't have ADHD.
One of the leading
arguments against widespread nootropic use is that they alter biology and give an
unfair advantage to
some over others. An example of this would be students using a drug that enhances
their focus,
attention, and memory from their baseline. These improvements could then lead them
to perform better
on tests than others who did not take the same drugs.
106. Which of the following is the author most likely to agree with?
(a) It is ethical to use nootropics to improve cognitive skills.
(b) A drug can become popular even if its internal functioning is less or not
known.
(c) There should be a ban on the usage of nootropics.
(d) Both, (b) and (c)
107. Which among the following most accurately reflects the author's views on the
usage of Ritalin and
Adderall, as set out in the passage above?
(a) Regulated usage of the given stimulants.
(b) They boost children's academic performance.
(c) A blanket ban on their usage.
(d) These stimulants should only be exported and not to be used locally.
108. Which of the following, if true, would weaken the illustration presented in
the passage against the
widespread use of nootropics?
(a) It ignores underlying factors that already have a strong influence on cognitive
performance.
(b) Studies show that iron deficiency while in the womb and early on in life
negatively affects cognitive
processing.
(c) Both, (a) and (b).
(d) Neither (a) nor (b)

. Page 30 of 40
109. The author says that "While some are known to improve skills like attention,
memory, and general
processing, their exact results vary from person to person, depending on genetics,
neurochemistry,
general cognitive performance, and more". Which of the following, if true, would
most strongly support
this statement?
(a) While babies from lower-income families reportedly tend to hear just 600 words
per hour during their
crucial developmental years, those from higher-income families tend to hear around
2,000 words per
hour.
(b) It is more ethical to provide nootropics to lower-performing individuals to
make up for their deficits.
(c) Students at colleges with more competitive criteria were found to use Ritalin
at over two times the rate
compared to those in less competitive colleges.
(d) None of the above.
110. Which of the following can be the course of action to deal with the situation
of nootropics usage by
students?
(a) Nootropics should be provided only to individuals with below par cognitive
abilities and not for
improving academic performance as these drugs should truly benefit the ones who
actually need
them.
(b) The purpose of attending school should be primarily to learn, and not to pass
or score high in the
exams.
(c) There should be restriction on nootropics based on whether they are harmful or
not in the long run, as
used by the World Anti-Doping Agency (WADA) as a criterion for drug prohibition.
(d) All (a), (b) and (c).
Directions(Q.110-Q.115): Read the passage carefully and answer the questions that
follow.
In 1628, English physician William Harvey put forth a radical theory: blood
circulates. This idea may
sound simple, but it flew in the face of centuries of medical orthodoxy, and over
the next few centuries, it
had an unspeakably large impact on physicians, economists, philosophers, and
political thinkers. In the
words of sociologist Richard Sennett, "A new master image of the body took form."
One particular area
affected by Harvey's ideas was urban planning. Cities expanded at an exponential
rate during the
modern era, and city planners adopted Harvey's idea that healthy living required
free circulation.
Accordingly, they sought to make modern cities that resembled the human body. Wide,
arterial streets
enhanced the movement of people and goods, carrying them swiftly to the commercial
heart of the city. A
bowel-like system of sewers and pipes efficiently emptied the city of waste. And
great green expanses
functioned like lungs, letting people breathe freely. Starting in the 1740s,
European cities began putting
their new visions of the "healthy city" into place, and by the nineteenth century,
the campaign was fully
underway. One of the most obvious innovators was Baron Haussmann, a French official
who carried out
a massive urban renewal program in Paris starting in the 1850s.
Haussmann demolished a number of narrow, dead-end-filled medieval neighbourhoods,
and in their
place, he constructed wide, straight boulevards that shot through the heart of
Paris. Haussmann freely
likened these streets to arteries and veins, as did other European and American
designers. They argued
that if motion was blocked, the city would suffer, much like a person suffers
during a stroke.
By design, these circulatory cities also facilitated the free flow of labour and
capital. In fact, economic
thought was equally influenced by Harvey's ideas of circulation. For example, the
group who coined the
term "economist" promoted the idea that the economy is a circulating flow of income
and output, made
healthy through free trade. Their founding father was Francois Quesnay, a prominent
physician who
frequently relied on medical analogies. Bodily metaphors promoted the vision of
free-flowing traffic,
efficient financial transactions, and smooth supply chains. Today, the value of
circulation has triumphed
in our cities.

. Page 31 of 40
111. Which of the following would the author most likely to agree with?
(a) Our cities are organic and dynamic, not because we have modelled them on
ourselves.
(b) Modern cities are a reflection of our worldview and values since they are
modelled on us.
(c) Clogged street restrict the swift movement, leading to an arrested economy in
the same way as a
clogged artery restricting blood flow, leading to cardiac arrest.
(d) On a metaphorical level, humans are more than capital, motion and efficiency.
112. Which of the following can be inferred from the given passage?
(a) The theory 'blood circulates' is revolutionary.
(b) If free motion in a city is not allowed, that city would suffer.
(c) Free circulation is sufficient for healthy living.
(d) None of (a), (b) and (c).
113. Which of the following can be course of action if we want our cities to truly
reflect us?
(a) Our modern cities, in their present status-quo, truly reflect us; so, there is
no need of a course of
action.
(b) In the tightly packed cities of the modern era, having room to breathe should
be provided, which
would be a true reflection of us.
(c) Cities need to reflect our total health and deeper needs, including the need
for non-commercialized
human connection and mental well-being.
(d) They shouldn't simply fulfil the goals of productivity, goods, and labour
alone.
114. Which of the following best expresses the main point or idea of the given
passage?
(a) Our understanding of the body and human health have changed over the period of
time.
(b) During the rise of the modern city, planners and engineers modelled the fabric
of the city's
infrastructure on the fabric of the human body itself.
(c) We need to reimagine our cities in a more holistic way.
(d) Humans and the human bodies are much more than circulation; on a physical
level, our health
requires more than the unobstructed movement of blood.
115. Based on the analogy and bodily metaphors used in the passage, which of the
following would be
considered as 'body' of the city?
(a) Parks (b) Streets (c) Economy (d) Commercial districts
Directions(Q.116-Q.120): Read the passage carefully and answer the questions that
follow.
The modern philanthropic system bloomed from the vast economic inequality of the
Gilded Age when
wealthy industrial magnates - otherwise known as robber barons - funnelled portions
of their enormous
fortunes into foundations or charity vehicles that directed money toward solving
the social ills of the day.
Several big foundations established during the early twentieth century - such as
the Carnegie
Corporation and the Rockefeller Foundation - today remain major funders in the
arts, education, and
other fields; by design, their endowments were meant to last forever.
Early critics of the philanthropy model insisted that foundations - which commanded
huge sums of tax#sheltered money doled out at the discretion of their trustees with
zero public input - allowed elites to
appear generous with their money, while also disguising how they had amassed their
riches in the first
place. More recently, political scientist Rob Reich has argued that philanthropy
enables a small handful of
elite donors to exert undue influence on public life, thereby perpetuating many of
the same inequalities
they claim to ameliorate, and even eroding democracy. For instance, the Gates
Foundation, which has
an endowment of nearly $49 billion and recently partnered with the state of New
York to "reimagine
education," has notoriously undermined the public school system in the past by
using its enormous
influence and financial resources to push experiments with few or no proven
benefits for students onto
already struggling schools, like breaking up large high schools into smaller ones.
Philanthropy also serves as yet another tax advantage for the already wealthy.
Today philanthropic
institutions' tax breaks cost the U.S. Treasury around $50 billion in lost tax
revenue each year. Despite

. Page 32 of 40
these criticisms, approximately a century after the philanthropic sector was born,
foundations have only
multiplied. As the government continues an austerity regime that withdraws support
for social services,
the arts, education, and other public programs, few non-profits today are in a
position to spurn funding
where they can find it. As a result, even groups pressing for wide-scale social
change and wary of the
strings attached to philanthropic dollars often find themselves trapped in an
uneasy dance with such
funders in order to keep the lights on.
116. Which of the following is the author most likely to agree with?
(a) The Gates foundation has pushed for creating smaller schools which has worked
well for the New
York education system.
(b) Philanthropy has helped the US economy by allowing the wealthy to save more and
by giving them
tax breaks.
(c) The government has cut off funding from non-profits which leaves them with no
alternative except
philanthropists.
(d) Financial crisis has meant that these companies are cutting down on their
funding to non-profits
putting their future in jeopardy.
117. Which of the following, if true, strengthens the argument raised by the author
against the philanthropic
system in the passage?
(a) Most of these foundations are helping the non-profits in times when the
government has withdrawn
funding from them.
(b) Companies take the tax break as they believe that they can help the society
better by directly using
that money to help people.
(c) These companies do not use the money to improve their position in the society
and often donate
anonymously.
(d) None of the above
118. Which of the following is a possible course of action for the non-profits that
the author is likely to agree
with?
(a) The non-profits should stick to government funding as they do not manipulate
them for their own
personal gains.
(b) The non-profits should stick to funding provided by these philanthropic
companies.
(c) The non-profits should rely on individual donations from trusted individuals of
the society and audited
trust funds.
(d) Both a) and b).
119. Which among the following most accurately reflects the author's views on the
Gates foundation?
(a) Great patron of arts and provides lifetime endowments.
(b) Worsened the New York public school system with their experiments.
(c) Improved the New York education system by creating smaller schools with more
personalized
attention to kids.
(d) Got 49 billion dollars’ tax break last year.
120. Which of the following can be inferred from the passage?
(a) Early philanthropists donated to hide money they earned illegally or to save
taxes that they owed to
the government.
(b) The philanthropic donors have used their influence to even undermine democracy.
(c) Non-profits often have to organize dance parties to keep their funding.
(d) Early philanthropists actually wanted to solve the problems of the society
without gaining anything in
return.

. Page 33 of 40
Directions(Q.121-Q.124): Read the passage carefully and answer the questions that
follow.
Perhaps because I spend a lot of time listening to people with crazy opinions, I am
sympathetic to the
view that the only way to live a healthy intellectual life is to expose oneself
constantly to weird or
detestable opinions. But I never sign petitions or open letters. Open letters are
terrible, and you should
never write one or sign one.
Open letters tend to be composed inclusively, so as many people as possible will
sign them. They can
bear no traces of their individual authors, and the easiest way to scrub those
traces is to write in a
numbing, anonymized style, free of idiosyncrasy and wit. Humour is especially
forbidden. What should we
think of a letter signed by Martin Amis that could not possibly have been written
by Martin Amis? I
suppose one interpretation is "Amis must really believe this stuff, if he is
willing to suppress his gag reflex
over a cliché like all too common or we are already paying the price." Another,
more hostile interpretation
is that the signers - the ones considered politically respectable, at least -
couldn't possibly believe this
stuff. So, they must have been tricked into signing the letter, and others
manipulated them into it for
nefarious purposes.
In 1931, after Albert Einstein's theory of relativity became famous, a pamphlet was
published with the title
"100 Authors Against Einstein." Einstein replied, "Why a hundred? If they were
right, one would have
been enough." One of the greatest pleasures of writing for the public is the
pleasure of being right and
watching your readers resist you before they realize you are right. The smaller the
minority in which the
writer finds herself, the greater the pleasure in watching the audience slowly
relent and join her. But the
longer the list, the less the pleasure - and at some point, a writer has to wonder
why she would bother to
sign as one name among dozens, when she could instead reap the much greater rewards
of solitary
righteousness. To be right should be enough.
121. Which of the following, if true, would strengthen the argument present in the
last paragraph of the
passage?
(a) Einstein published the pamphlet knowing that his theory was flawed and only
wanted the publicity that
followed.
(b) Einstein did not care about his haters because he knew he was right all along.
(c) Einstein was pompous and did not care for the opinion of others.
(d) Einstein wanted to prove himself right and ended up getting famous due to the
backlash he received
from other authors.
122. Which of the following is true according to the passage?
I. Open letters are terrible because they do not achieve anything.
II. Open letters are notorious for their bland writing style.
III. Open letters are a good reflection of the individual style of an author.
(a) Only II. (b) Only III. (c) Both I and II. (d) Both II and III.
123. If the passage continues, which of the following can be the central argument
of the next paragraph of the
passage?
(a) Einstein's theory of relativity was incorrect and he had to issue an apology to
the authors who wrote
an open letter against him.
(b) Open letters and petitions have forced the US govt. to change their new policy
about detaining the
immigrants at Mexico border.
(c) Martin Amis later revealed that he had actually been tricked into signing the
open letter and he hated
the style of the letter.
(d) Open letters are a superficial way of showing that you care about a cause
without actually doing any
work that matters.

. Page 34 of 40
124. Which of the following is the author most likely to agree with?
(a) Open letters are usually written and signed by struggling writers who still
haven't developed enough
to have their own style.
(b) Open letters are made for the government and therefore, the language used must
be formal.
(c) Open letters should be avoided by writers as by signing them they lose their
individual identity.
(d) All of the above
Directions(Q.125-Q.129): Read the passage carefully and answer the questions that
follow.
Empathy - the ability to tune into the feelings and perspective of someone else -
is often seen as a
prerequisite for compassion. And research suggests that feeling empathy can
sometimes result in
compassion fatigue. But not all types of empathy are created equal.
Michael Poulin, a professor of psychology at the University of Buffalo, has studied
when empathy can be
beneficial - and when it's simply burdensome. "We use the term empathy in a lot of
different ways," he
says. "Some people use it to mean, 'I feel concern for someone else. I understand
what they feel, and it
moves me to act. But some people use it to mean, 'I'm literally feeling what they
feel. They're suffering,
and therefore I am suffering, too.'
In a study published in 2017, scientists looked at how these two types of empathy
impacted the
physiological stress of someone helping others. Participants watched a video of a
woman in distress, and
while one group was tasked to simply think about how this woman might feel, others
were asked to
imagine how they would feel if they were in her position. The researchers found
that the latter group saw
their cardiovascular stress levels spike from a perceived threat, causing their
blood vessels to constrict.
The former group, who were just thinking about her feelings, experienced no such
distress.
"It is possible to acknowledge another person's distress and want to do something
about it without
necessarily taking that burden on yourself," says Poulin, one of the study's
authors. "When you put
yourself in another person's shoes," he continues, "you are taking a burden on
yourself - you're causing
yourself stress." And in turn, says Poulin, compassion fatigue is much more likely
to happen if people are
routinely trying to practice empathy by literally trying to feel what other people
are feeling. He adds. "I
think we have this cultural belief that suffering is good for you - and that
suffering on behalf of a moral
cause makes it nobler. But suffering doesn't necessarily lead to action."
125. Which of the following, if true, weakens the argument presented by Prof.
Poulin in the last paragraph of
the passage?
(a) Modern religions are built on the system that suffering is atonement of sins
and therefore, good for
you.
(b) Morality dictates that you are only in pain when you have done something wrong.
(c) We must practice empathy to become a better person.
(d) Feeling the pain of others is what makes us human.
126. Which of the following cannot be inferred from the passage?
(a) The concept of empathy is open to interpretation.
(b) Empathy can also lead to physiological changes in some cases.
(c) Practicing empathy results in development of emotional stress.
(d) All of the above
127. Which of the following is the author most likely to disagree with?
(a) Different groups of people can have different reactions to the same situation.
(b) For some people, compassion is necessary to experience empathy.
(c) You can have empathy for your own actions as well.
(d) Empathy can lead to physiological changes in your body in some cases

. Page 35 of 40
128. Which of the following most accurately expresses the main point of the last
paragraph of the passage?
(a) Poulin believes that practicing empathy is an artificial form of inducing
suffering and therefore,
immoral.
(b) Poulin postulates that perhaps the root cause of why we are attracted to
suffering on behalf of others
is our cultural conditioning.
(c) Poulin theorizes that cultures dissuade us from feeling for others, but
morality dictates that we help
others by sharing their suffering.
(d) Poulin says that to truly understand the underlying causes of why humans are
attracted to suffering,
we must turn to religious causes.
129. "I think we have this cultural belief that suffering is good for you - and
that suffering on behalf of a moral
cause makes it nobler. But suffering doesn't necessarily lead to action." Which of
the following if true
contradicts this statement?
(a) Psychiatrists that offer therapy often delve into their patients' pain and help
them improve.
(b) Teachers counsel students through tough times and help them become better.
(c) Consultants advise companies in trouble by finding their weaknesses and
suggesting changes.
(d) Support groups offer safe spaces where people suffering from similar problems
can share their pain
and heal together.
Directions(Q.130-Q.135): The passage given below has three paragraphs. Read it
carefully and answer
the questions that follow.
The analysis by Georg Simmel in 'The Metropolis and Mental Life' (written in 1903)
centres on two
interlocking social forms: money and the city. As they become dominant, they erode
natural rhythms of
production and traditional social bonds. This is liberating: cash does not care
about birth right; it is
'concerned only with what is common to all: it asks for the exchange value'. Yet,
there is a hidden cost:
money reduces what is uniquely valuable to a number, a price. In the right ratio,
fine hand-crafted goods
are equal to mass-produced junk. This devalues commodities - nothing that can be
bought is unique -
while simultaneously accelerating the search for whatever is truly unique and
incomparably valuable.
The city accelerates the calculable logic of money, encroaching even on our
experience of time. Time is
no longer governed by the seasons or celestial bodies but is abstracted and
measured. The city also
compresses space: social and geographical. Diverse classes, strata, cultures,
linguistic groups and
vocations are brought into close proximity. This is why, as Simmel observed, the
German philosopher
Friedrich Nietzsche preached against the city bitterly: it threatened to subsume
his noble individualism
into a mass.
Instead of seeking extremes in the mountains of Sils Maria, Simmel found them in
the metropolitan
crowd, where one can feel the uniquely modern loneliness of passing a thousand
faces without
recognising a friend. Disillusioned by advertising and overstimulation, Simmel
suggested that blasé
individuals search for quality in their last refuge - personality: Man is tempted
to adopt the most
tendentious peculiarities, that is, the specifically metropolitan extravagances of
mannerism, caprice,
preciousness; the meaning of these extravagances lies in form of 'being different',
of standing out in a
striking manner and thereby attracting attention.
130. Which of the following can be inferred correctly from the given passage?
(a) For man, who is always striving, never satisfied, always becoming; love is the
only true human
condition.
(b) If all clocks in a metropolitan city would suddenly go wrong by the least, all
economic life and
communication of the city would be disrupted for a long time.
(c) The problem of modern life is to preserve the individuality of a person's
existence in the face of
overwhelming social forces.
(d) Rapid urbanization and financial speculation propelled the decay of isolation
of an individual from the
society.

.
131. Which of the following assumptions is made based on the given passage?
(a) Before the metropolitan era, there was no disparity among different groups in
the society.
(b) The worth of time was much higher before the invention of money.
(c) Citizens of the metropolitan era face social anxiety and live solitary lives.
(d) In the modern era, anybody is eligible to buy any commodity he wants if he has
abundant money.
132. Which of the following statements weakens the argument that 'money reduces the
price of an entity'?
(a) Most luxury brands are now available online and can be bought or rented for a
certain period of time.
(b) Some people loathe commodities that are produced for the mass and instead look
for designer
commodities that can be customized according to one's desire.
(c) Despite their high prices, cruise tickets often get sold out one month prior to
the voyage dates.
(d) None of the given
133. Which of the following, as evident from the passage, is a challenge faced by
people in this metropolitan
era?
(a) Most people are spoilt for choices in the cities as the availability of
commodities in cities is abundant.
(b) People living in metropolitan areas dislike people from rural areas moving into
urban areas as it
threatens their survival.
(c) As differences between the various social strata and classes have diminished in
cities, urbanized
people have a difficulty moving back to rural areas, even when presented with a
choice.
(d) Urbanisation brings in its wake a constriction of space, both socially and
geography while threatening
to take away individual identity, as the diversities lose uniqueness and
simultaneously causes
loneliness.
134. Which of the following conclusions can be drawn from the last paragraph of the
given passage?
(a) Most philosophers seek seclusion in mountains to let their thoughts flow.
(b) To avoid overstimulation and fading in the crowd, urban man adopts pretentious
ways to seek
attention.
(c) From the beginning of the 20th century, traditions and cultures saw a downfall
in urban areas.
(d) People in urban areas often live in isolation and do not prefer to have family
or friends.
135. “In the right ratio, fine hand-crafted goods are equal to mass-produced junk.
This devalues commodities
- nothing that can be bought is unique - while simultaneously accelerating the
search for whatever is truly
unique and incomparably valuable.” We can infer from the statement that
(a) Excess supply of goods lowers the value of the goods and triggers a demand for
exclusivity.
(b) A fine hand-crafted when produced in mass becomes a junk and loses it value and
at the same tine
people look for a commodity that is unique.
(c) Excess demand leads to lower supply which enhances the value of that commodity
resulting in its
uniqueness.
(d) Any change in the quantity of commodity does not affect the real output but
affects only the price
level
mock 14
Directions(Q.1-Q.30): Read the passages carefully and answer the questions based on
it.
Passage (Q.1-Q.5): The first Neanderthal was found in a limestone cave about forty-
five miles north of
Bonn, in an area known as the Neander Valley, or, in German, das Neandertal.
Although the cave is gone—
the limestone was long ago quarried into building blocks—the area is now a sort of
Neanderthal theme
park, with its own museum, hiking trails, and a garden planted with the kinds of
shrubs that would have
been encountered during an ice age. In the museum, Neanderthals are portrayed as
kindly, if not
particularly telegenic, humans. By the entrance to the building, there’s a model of
an elderly Neanderthal
leaning on a stick. He is smiling benignantly and resembles an unkempt Yogi Berra.
Next to him is one of
the museum’s most popular attractions—a booth called the Morphing-Station. For
three euros, visitors to
the station can get a normal profile shot of themselves and, facing that, a second
shot that has been
doctored. In the second, the chin recedes, the forehead slopes, and the back of the
head bulges out. Kids
love to see themselves—or, better yet, their siblings—morphed into Neanderthals.
They find it screamingly
funny.
When the first Neanderthal bones showed up in the Neander Valley, they were treated
as rubbish (and
almost certainly damaged in the process). The fragments—a skullcap, four arm bones,
two thighbones,
and part of a pelvis—were later salvaged by a local businessman, who, thinking they
belonged to a cave
bear, passed them on to a fossil collector. The fossil collector realized that he
was dealing with something
much stranger than a bear. He declared the remains to be traces of a “primitive
member of our race.”
As it happened, this was right around the time that Darwin published “On the Origin
of Species,” and the
fragments soon got caught up in the debate over the origin of humans. Opponents of
evolution insisted that
they belonged to an ordinary person. One theory held that it was a Cossack who had
wandered into the
region in the tumult following the Napoleonic Wars. The reason the bones looked odd
—Neanderthal femurs
are distinctly bowed—was that the Cossack had spent too long on his horse. Another
attributed the remains
to a man with rickets: the man had been in so much pain from his disease that he’d
kept his forehead
perpetually tensed—hence the protruding brow ridge. (What a man with rickets and in
constant pain was
doing climbing into a cave was never really explained.)
1. "In the museum, Neanderthals are portrayed as kindly, if not particularly
telegenic, humans." Which of the
following best interprets the sentence?
(a) The Neanderthals were pleasant-looking to become an attraction on Telemedia.
(b) The Neanderthals did not behave confidently or looked attractive on television.
(c) The Neanderthals showed a genial face even if the face was not markedly
attractive on the television.
(d) The Neanderthals; disposition came out unattractive to the television viewers.
2. Which of the following draws the inference(s) from the given passage?
I. In the morphing station of the museum in Neander valley, the second profile shot
of a person resembles
the Neanderthal.
II. The Neanderthals lived during the Ice age.
(a) Only I.
(b) only II.
(c) Neither I nor II.
(d) Both I and II.
3. "What a man with rickets and in constant pain was doing climbing into a cave was
never really explained."
Which of the following reveals the literary device used by the author in the above
statement?
(a) Irony
(b) Juxtaposition
(c) Elegy
(d) Satire

. Page 3 of 40
4. Which of the following is closest to the meaning of the word 'salvaged', as used
in the passage?
(a) Reclaim
(b) Rescued
(c) Forfeited
(d) Restored
5. Which of the following best reflects the theme of the passage?
(a) The passage is about the discovery of the first Neanderthal and the contrasting
theories surrounding it.
(b) The passage is on the mystic around the first Neanderthal and eventual
disclosure of the fossil.
(c) The passage is how a fossil site is developed as a fossil park for children
interested in Anthropology.
(d) The passage retells various stories surrounding the first Neanderthal, much to
the amusement of the
author.
Passage (Q.6-Q.10): Reading deSouza’s call for food diversity, I realised what
nagged me was its
deafening silence about one aspect of food ethics — is it right to eat certain
kinds of food at all? It’s time
to make a disclosure here. For last year-and-half, I have gone from being an
occasional non-vegetarian to
the vegan. Almost vegan is much more accurate.
What surprises me though, as a struggling and imperfect vegan, is how people react
to veganism. Some
believe it is a form of food puritanism, which it most definitely is not. Others
dismiss it as a result of some
passing woke trend, an attempt to be a food fashionista. Although there are some
activists who have given
veganism a bad name, very few appreciate that it could also be arrived at through
(________) philosophical
inquiry into the ethics of food, its production and consumption.
The monstrous cruelties that attend industrial factory farming, which author Yuval
Noah Harari described
as probably the worst crime in history, need no repetition here. But if you do not
believe, as some religious
texts have declared, that man was made in God’s image and was placed to have
dominion over every
other living thing on earth, then it is worth at least considering the vegan case,
particularly as we now live
in a world where human survival and nourishment can sustain without animal
slaughter on such a gigantic
scale.
Vegetarians like to think they are more humane about their dietary choices, but
they rarely consider what
goes into the making of dairy products. What it usually means is a long and quick
succession of pregnancies
for cows and buffalos, their calves separated not so long after birth, and their
milk diverted for human
consumption. If the calf is female, then it is raised for another succession of
economically-lucrative
pregnancies. If it’s a male, then it is usually quietly sent to the abattoir.
When it comes to thinking about how our food is produced, we would rather not know,
or deal with our
cognitive dissonances by suppressing what we do know. Allowing oneself to think
critically and candidly
about food may demand making challenging dietary changes. It is this kind of
collective denial that results
in activist campaigns, and even books, devoted to climate change failing to make
even a passing reference
to food. Our food system produces more greenhouse gases than most other sectors,
including
transportation, but we are more comfortable talking about limiting the size of cars
than reducing the harm
to cows or goats. Never mind also that methane, tonne for tonne, is about 30 times
worse in its impact than
carbon dioxide by some estimates.
6. “Almost vegan is much more accurate.” Considering the above statement to be
true, which would be
furtherance to the sentence?
(a) I have allowed myself to, on occasion, eat something with vinegar and chilli
sauce or added extra
oregano to the Pizza rather than risk offending a host and, much worse, cadged a
bite or two of some
tofu sweets.
(b) I have allowed myself to, on occasion, eat something with butter or ghee rather
than risk offending a
host and, much worse, cadged a bite or two of some milk-infused burfis.
(c) I have entirely refrained from indulging in alcohol to the point of mentioning
in advance to the party host
to avoid embarrassment.
(d) None of the above

. Page 4 of 40
7. “For last year-and-half, I have gone from being an occasional non-vegetarian to
the vegan”? The sentence
is grammatically unsuited. From the given options, choose the one which is
grammatically most coherent.
(a) “For the last year-and-half, I have gone from being an occasional non-
vegetarian to a vegan.”
(b) “For the last year-and-a-half, I have gone from being an occasional non-
vegetarian to a vegan”?
(c) “For last year-and-a-half, I have gone from being an occasional non-vegetarian
to a vegan.”
(d) “For the last year-and-a-half, I have gone from being an occasional non-
vegetarian to a vegan.”
8. Although some activists have given veganism a bad name, very few appreciate that
it could also be arrived
at through (________) philosophical inquiry into the ethics of food, its production
and consumption. Which
of the following words from the options is best suited to make the sentence
coherent?
(a) debatable
(b) casual
(c) deliberative
(d) calculated
9. The author, through the passage...
(a) clears misconceptions surrounding misplaced reactions of people towards
veganism.
(b) justifies his reasons for turning a vegan and pointing out the dichotomy
present in people’s outlook.
(c) brings out people’s double standards about their entire focus on climate change
without mentioning
anything about the impact of food on climate.
(d) Both (a) and (c).
10. “When it comes to thinking about how our food is produced, we would rather not
know or deal with our
cognitive dissonances by suppressing what we do know.” What is the author implying
through the
statement?
(a) We are neither in denial nor mishandle the mental discomfort that results from
holding conflicting
beliefs.
(b) We deliberately suppress uncomfortable issues by faking unawareness about them.
(c) We are either in denial or handle the mental discomfort that results from
holding conflicting beliefs by
sweeping them under the carpet.
(d) Despite knowing how food gets into our place, we show complete apathy towards
animal plight.
Passage (Q.11-Q.15): Ford India was in the process of announcing a settlement
package with the
employees in the wake of the US-based auto major deciding to shut its two
manufacturing facilities in the
country under its 'restructuring' exercise, a state minister said on Wednesday.
On Wednesday, after holding a review meeting with the more than 50 auto parts
suppliers to Ford, the
government has received information that Ford was in the process of announcing a
'settlement package'
to the employees.
"Soon after reports emerged about Ford's decision early this month, Chief Minister
M K Stalin discussed
about the possibilities available to safeguard the livelihood of the employees,
including the feasibility of
another automaker taking over the facility.
A parts supplier who had set up shop in 1997 along with Ford, which had commenced
operations at
Maraimalai Nagar, about 45 kms from Chennai, said 80 per cent of the parts
manufactured were supplied
to Ford and his company was earning Rs 80 crore annually.
"We will be hit hard in the wake of Ford closing down the unit. We even had the
land leased from Ford to
produce the auto parts," he said.
Another wiper and blades manufacturer said he had set up a facility in neighbouring
Oragadam at an
investment of Rs seven crore and suggested that the government consider another
auto major taking over
the Ford facility.
A senior official of a company engaged in the production of fuel and braking
systems for Ford India said
they employ about 300 people and were in the process of seeking claims of Rs 25
crore from Ford for the
loss they expect to incur in the wake of Ford shutting down the unit.

. Page 5 of 40
Later, responding to a query, Anbarasan said "Ford has announced its decision to
shut its factory by 2022
and we have received information that it was in the process of announcing a
settlement package to the
employees. There are about 2,638 workers directly employed and 1,421, indirectly."
The Minister said during the review meeting today, the Micro Small and Medium
Enterprises commissioner
and the principal secretary discussed the matter and one of the highlights to
emerge from the meeting was
that 'not many auto parts suppliers were directly involved with Ford'.
"They were not only supplying parts to Ford but also to other auto majors. Those
companies who will be
hugely impacted due to Ford's decision have made a request. We have taken note of
it. We will take it up
with the Chief Minister (M K Stalin) on further course of action", he said.
11. The above passage is…
(a) an excerpt from a newspaper.
(b) a letter from an automobile company.
(c) a fiat from the Chief Minster's Office.
(d) a blog from a writer.
12. What can be inferred from 'restructuring', as mentioned in the passage?
(a) Restructuring of a company occurs when there is a change in overall goals or
transfer of ownership.
(b) Restructuring a company means a sale or a buyout of another company, which
requires an overhaul.
(c) Restructuring a company means significant changes in financial structures and
modes of Operandi,
usually under financial constraints.
(d) Restructuring a company means cost-cutting, such as payroll, or reducing its
size through the sale of
assets.
13. Which of the following finds a mention in the passage?
(a) Ford India had set up its operations at Maraimalai Nagar, about 45 km from
Chennai.
(b) There are about 4059 people employed in Ford, directly or indirectly.
(c) 4059 people are working in the Ford company, directly or indirectly.
(d) About 300 employees are working in the production of fuel and braking systems
department of Ford
India.
14. We can infer from the passage that
I. The two major manufacturing units of Ford India are located in Sholinganallur,
Chennai, Tamil Nadu
and Sanand, Gujarat.
II. Ford India is a subsidiary of the US-based automobile Industry.
(a) Only I
(b) Only II
(c) Both I and II
(d) Neither I nor II
15. What is the primary purpose of the passage?
(a) To expose ford by divulging the reality through insiders.
(b) To give a written account of what has been heard, read or investigated.
(c) To bring out the stories of people associated with Ford India, directly or
indirectly.
(d) To highlight a system whereby companies such as Ford India should be held
accountable for their
employees.

. Page 6 of 40
Passage (Q.16-Q.20): It was a long innings. But the “batsman” is finally out,
making space for the “batter”.
The change in terminology, according to the Marylebone Cricket Club (MCC), the
authority on the game’s
laws, helps “reinforce cricket’s status as an inclusive game for all”. And this is
just as it should be. When
women’s cricket, once dismissed as the also-ran version of the “real game”, has
proved its ability to draw
crowds, bust records and produce cricketing heroes, the shift in language is a sign
that the sporting
establishment is catching up. There are those, like British journalist Piers
Morgan, known to see the
collapse of civilisation at even the slightest stumble towards change, who have
been triggered into outrage.
Others find the new word awkward on the tongue, so strong is the hold of habit.
Fact: In a playground of
“fielders”, “bowlers”, “wicket-keepers” and “umpires”, it was “batsman” that was as
odd as a powerplay in
Test cricket.
Cricket in the 21st century is nothing like it once was — a “gentleman’s” sport
tied up with class and feudal
notions of gentility in Britain and with the “civilising” force of the empire in
the colonies. Well, the former
colonies have gone on to own the game, turning it into a desi spectacle, a money-
spinning one, that cannot
be played with a stiff upper lip. Even the Aussies blush when they read Virat
Kohli’s lips. Women too were
not welcome, with the MCC reluctantly opening its doors to female members as late
as 1998.
But the future of any sport in 2021 rests on how many doors it can open to a
diversity of talent. Cricket
must be glad that it has fresh blood, male and female, running through its veins.
If that means finding a
better word for “third man” and “nightwatchman”, what’s to lose?
16. What does the terminology 'Batter' signify?
(a) A change in perspective.
(b) A change in terminology.
(c) A change of sports.
(d) A gender-neutral status to Cricket.
17. We can infer from the passage that, according to the author, the reaction of
the British Journalist, Piers
Morgan, is reflected through which of the following idiomatic expression?
(a) A tempest in a teapot.
(b) The grass is greener on the other side.
(c) Scrape the barrel.
(d) To not see the wood for the trees.
18. Which of the following gives the best take on Cricket before the 21st century?
(a) Cricket, before the 21st century, had a gentlemanly air and had all the
medieval airs donned by the
upper-class British.
(b) The British saw Cricket as a birth right, belonging to the upper echelon and
carrying the refinement of
a gentleman's game.
(c) Cricket suggested of British air; seen as more of a game played by the upper
echelon of specific
breeding.
(d) Cricket, before the 21st century, was seen as a game belonging to the upper
class to refine the colonies
so dominated.

. Page 7 of 40
19. "Cricket in the 21st century is nothing like it once was — a "gentleman's"
sport tied up with class and feudal
notions of gentility in Britain and with the "civilising" force of the empire in
the colonies. Which of the
following reflects the tone of the author in the given statement?
(a) Condescending
(b) Mulling
(c) Romantic
(d) Deprecation
20. What is the author trying to communicate through the passage?
(a) No longer to the British have the supremacy of the game of Cricket.
(b) Replacing 'batsman' with a gender-neutral term is a sign that Cricket has
changed, a good sign.
(c) Gender-neutral terminology is the new language of the contemporary world.
(d) Sports now has a new language that is all-inclusive.
Passage (Q.21-Q.25): Evergrande is the biggest property developer in China and
among the largest in the
world. Given the size, it is not unreasonable to fear that the direct impact or the
financial contagion can be
large. However, such fears are overstated. First, while still a substantial part of
the economy, the real estate
sector’s contribution to the post-pandemic recovery has been modest. In addition,
there was no expectation
of a significant turnaround in this sector to buoy the recovery in 2022.
Consequently, the impact on growth
is likely to be limited. Second, the financial liabilities of the company, while
large in absolute terms, make
up about 0.65 per cent of China’s corporate and government debt combined (the stock
of total social
financing) and bank loans per se just 0.25 per cent of total bank credit. Linkages
to offshore banks and
financial institutions are even smaller.
So why did the authorities not intervene to prevent the collapse of Evergrande?
Because not doing so is
the strongest signal yet that no corporate is “too big to fail” and that there are
no state guarantees for them.
For years, investors and analysts (including multilateral institutions) have
exhorted China to allow defaults
as a way of eliminating the market’s presumption of implicit state guarantees,
which has obfuscated any
meaningful distinction between good and bad credit leading to a distorted
allocation of capital and
substantial loss of efficiency. While there could be short-term pains and even some
spillover into the wider
financial system, over the medium term this “inaction” is likely to improve the
pricing of credit and efficiency
of capital.
While we expect the Chinese economy to recover in the coming quarters from likely
near-zero sequential
growth in 3Q (on a year-on-year basis, growth should remain above 6 per cent), over
the medium term,
growth is headed to the 4-5 per cent range, with significant rebalancing of the
growth drivers, including
sectoral shifts, redistribution of income towards households and away from
corporates, and a rotation
towards more services consumption. To implement these changes, China’s policy and
regulatory regime
will be altered radically. That process is already underway. And in the cross-hairs
are also countries with
strong trade and financial ties with China. If China changes, so will these
linkages. The world needs to
ready itself for China 3.0.
21. What can be concluded from the first five lines of the first paragraph?
(a) That the unreasonable fear surrounding Evergrande is inflated.
(b) That Evergrande is the largest property developer in the world.
(c) That the fear surrounding Evergrande is hearsay.
(d) That Evergrande, the biggest real estate conglomerate, is filing for
bankruptcy.
22. Which of the following is similar in meaning to the word ‘obfuscated’?
(a) Enlightened
(b) muddled
(c) Revealed
(d) Neutralised.

. Page 8 of 40
23. Which of the following questions acts as a catalyst to bring out the gist of
the given passage?
(a) Is China buckling under financial spillovers?
(b) Is China sending out a strong signal that there are no state guarantees to all
the defaulters?
(c) Why China has decided against saving Evergrande?
(d) Will China’s loss be again for the neighbouring countries?
24. “The world needs to ready itself for China 3.0.” The aforementioned sentence is
a/an
(a) Rhetoric
(b) Open-ended conclusion
(c) Suspense
(d) Climax
25. “First, while still a substantial part of the economy, the real estate sector’s
contribution to the post-pandemic
recovery has been modest.” What is the author implying through the statement?
(a) The real estate sector’s contribution has been exceptional
(b) The real estate sector’s contribution has been unexceptional.
(c) The real estate sector’s contribution has been nil.
(d) The real estate sector’s contribution has been beyond expectation.
Passage (Q.26-Q.30): Globally, about 15 per cent of the population lives with some
form of disability. Of
this, 80 per cent lives in developing countries. Persons with disabilities (PwDs)
are among the most
marginalised groups. They encounter a range of barriers and are more likely to
experience adverse
socioeconomic outcomes. Limited support infrastructure can have a significant
debilitating impact on
everyday life. WHO now considers disability a human rights issue. It emphasises
that people are disabled
by society and not by their bodies.
Over the last 65 years, the overall global literacy rate has increased by 4 per
cent every five years — from
42 per cent in 1960 to 86 per cent in 2019. However, the global literacy rate for
the disabled is as low as 3
per cent with just 1 per cent for females. Ninety per cent of disabled children in
developing countries do not
attend school, says UNESCO. The school drop-out rate is also high due to the lack
of adequate
infrastructure, inaccessible reading material and untrained teachers. An
insignificant number make it to
institutes of higher learning. Lack of education has a trickle-down effect. Most
disabled children are not
equipped with foundational skills for employability.
The pandemic has made us realise how technology is reshaping education. Lockdowns
made schools
rapidly migrate to online education. This metamorphosis of education systems has
far-reaching implications
for disabled children. Online education has the potential to make learning more
accessible for PwDs. It
takes care of physical barriers created by transportation and mobility issues.
Children have the advantage
of accessing learning from the safety and comfort of their homes. It saves them
from unnecessary
inhibitions in attending physical schools. Disabled students in higher education
too can have access to
lectures, libraries and resources without the need to physically navigate remote
campuses.
Internet penetration is increasing fast. As of January 2021, there were 4.66
billion active internet users
worldwide — almost 60 per cent of the global population. In 104 countries, more
than 80 per cent of the
youth population is online. Out of the 830 million young people who are online, 320
million are in China and
India, which are among the countries with the highest incidence of disability.
Advances in the digital economy are creating unprecedented work opportunities for
the disabled, a report
by ILO says. With the rise of the gig economy, most work is being done from homes,
creating new job
opportunities for physically restricted PwDs. Online education could prove to be a
turn of the wheel in the
quest of PwDs to gain a meaningful life.

. Page 9 of 40
26. The phrase ‘turn of the wheel’ means
(a) Digress
(b) Egress
(c) Progress
(d) transgress
27. According to the passage, how will the digital economy help the disabled?
(a) It will provide unparalleled job opportunities.
(b) It will improvise the lifestyle.
(c) It will give the disabled a social standing and independence.
(d) All of the above.
28. The author’s discussion that WHO now considers disability a human rights issue
is meant to...
(a) provide a basis for further benefits that the disable children will get.
(b) help explain that disability is now a social blight rather than remaining a
physical condition.
(c) highlights a particular segment of society instrumental in emotionally
torturing the physically disable.
(d) reinstate the apathy of the government towards the physically disable.
29. “Out of the 830 million young people who are online, 320 million are in China
and India, which are among
the countries with the highest incidence of disability.” Which of the following can
be concluded from the
statement?
(a) India and China put together have more than 40 percent and less than 45 per
cent of youth who are
online.
(b) India and China put together have more than 35 percent and less than 40 percent
of youth who are
online.
(c) India and China put together have more than 25 percent and less than 30 percent
of youth who are
online.
(d) India and China put together have more than 35 percent and less than 37 percent
of youth who are
online.
30. The primary purpose of the passage is to address…
(a) how online education can give disable children greater learning opportunities.
(b) how digital platform is sabotaging the earning opportunities of the weaker
section.
(c) why should there be a disparity in society.
(d) what does the future look for the disable after WHO puts them in the purview of
human rights
Directions (Q.66 – Q.105): Read the comprehensions carefully and answer the
questions based on it.
Passage (Q.66-Q.70): The Delhi high court granted bail to five of the people who
police had claimed were
involved in the murder of Delhi police head constable Ratan Lal and in causing head
injuries to a Deputy
Commissioner of Police (DCP) during the northeast Delhi riots of February, 2020.
The court came down heavily on the ‘vagueness’ of evidence presented by police
against one of the five
and stressed on the duty of the courts to ensure that “the law does not become a
tool for targeted
harassment”.The court noted that, when deciding whether or not to grant bail in a
case of an unlawful
assembly, it must hesitate before concluding that every member in that assembly has
the intention of
accomplishing the unlawful common object.
The court noted that the applicability of Section 149 (every member of unlawful
assembly guilty of offence
committed in prosecution of common object) of the IPC, specifically read with
Section 302 (punishment for
murder), cannot be done on the basis of “vague evidence and general allegations”.
It is the constitutional duty of the court to ensure that there is no arbitrary
deprivation of personal liberty in
the face of excess of state power. Bail is the rule and jail is the exception, and
courts must exercise their
jurisdiction to uphold the tenets of personal liberty, subject to rightful
regulation of the same by validly
enacted legislation.
66. A was driving back home after a verbal spat with one of the senior police
officials of XYZ. While he was on
the way he saw his friend B and three others asking for the lift to a well-known
protest site at XYZ; he
happily gives them a lift. B insists A to stay with them for sometime after which
they can go to a bar to
enjoy; A agrees to this plan of B and waits in the parking lot for B and his
friends. Meanwhile B and his
friends turn violent while protesting and injured one police constable. The police
official with whom A had
a fight a while ago books A, B and the other three friends of B for unlawful
assembly and they are eventually
held liable by the court. Was the decision of the court in consonance with the
above passage?
(a) Yes, since A had indirectly contributed to the violence at the protest site by
giving lift to B and his
friends, even he is liable along with the others for unlawful assembly.
(b) No, although A gave lift to B and his friends, he had not idea that the protest
will turn violent because
of B and his friends, hence apart from A everyone is liable for unlawful assembly.
(c) Yes, before giving lift, A clearly had the knowledge that B and his friends are
going to the protest site
for protesting still he gave them a lift, which makes them collectively liable for
unlawful assembly.
(d) The police official is taking revenge from A by booking him for unlawful
assembly, thus using law for
targeted harassment.
67. Suppose in the previous question B tells A that they have planned to turn the
peaceful protest into a violent
one, A laughs it off. Would this addition of fact, change your answer to the
previous question?
(a) Yes, now that A had the knowledge that the protest is going to turn violent,
all of them can be held liable
for unlawful assembly.
(b) No, although A had the knowledge but there was no common intention amongst all
to turn the protest
into a violent one.
(c) No, A had the knowledge that the protest is going to turn violent, all of them
can be held liable for
unlawful assembly.
(d) Yes, the police official has now solid evidence which cannot be deemed as
‘vague’ by the court, making
all of them liable.

. Page 18 of 40
68. There was a violent protest going on in the Hauz Khas Area of Delhi. The police
were informed abut this,
and a special unit was deployed to control the situation and arrest the offenders.
When this unit reached
the area of violence, they started arresting the protestors under the unlawful
assembly. They also took into
arrest a person named Mahesh, whom they saw engaged on a phone call just beside the
protest area.
Now, Mahesh claims that he was not involved with the protestors in any capacity.
Decide.
(a) Mahesh can be held liable, because it is reasonable to assume that he was one
of the protestors.
(b) Mahesh cannot be held liable, since the fact scenario is questionable.
(c) Mahesh can be held liable, because he was standing so close to the actual
protestors.
(d) Mahesh cannot be held liable, because personal liberty trumps over everything.
69. Assume that during the course of investigation, it is found that Mahesh is a
part of the group; which happens
to be the one involved in this said protest. However, he had no marks of injury
(which were present on the
other members due to the violence) and pleads that he parted ways with the group
when the protest turned
violent. Would this bring any different in your answer?
(a) Yes, as this will make him liable for the unlawful assembly.
(b) Yes, as he went there with a common intention of carrying out the protest.
(c) No, since Mahesh parted ways with the group.
(d) No, since Mahesh was engaged on a call.
70. A, B, C, D, E, F and G were seven friends, who planned a trip to city museum.
When they reached the
museum, the guards didn’t allow A to enter. This happened primarily because A did
not carry his ID card,
which was mandatory to seek entry. Amidst the heated argument, the guards pushed A
out of the gate.
Aggrieved by such kind of behavior, A’s friends started lashing out the guards and
throwing stones on the
artifacts. Meanwhile, museum’s manager informed the police, and these seven friends
were arrested.
Decide.
(a) A is liable, since he started the heated argument.
(b) A is liable, since this entire issue came up because of his negligence of not
carrying the ID.
(c) A is not liable, since he was the victim of guards’ violent behavior.
(d) A is not liable, since there was no common intention of disrupting peace.
Passage (Q.71-Q.75): The Madhya Pradesh High Court has directed the demolition of a
Community Hall
constructed in a park upholding the importance of open spaces of land as a buffer
zone for ecological
balance in the State of Madhya Pradesh alone. The Division Bench observed,
"Once a General Public Park is dedicated to citizens, it is held by the
Municipality in trust on behalf of the
public at large and cannot be put to any other use. Change of its use for any other
purpose by Municipal
body would tantamount to breach of trust; It is imperative that the environmental
factors should weigh
heavily with all the local bodies as also with the Courts while construing a town
planning statute.
Reservation of general open spaces for parks and playgrounds is universally
recognized legitimate
exercise of statutory powers rationally related to the protection of the residents
of the locality from the ill
effects of urbanization."
It also directed that the said space should always be maintained only as a park and
shall not be allowed to
use for any other purpose. The Court read Section 279 of the M.P. Municipalities
Act, 1961, which requires
the Municipal Council to provide places for recreation such as open space, park,
playgrounds, commons,
swimming tanks and amenities for the use and employment of the people and may frame
bye-laws
regulating their use. Section 282 of the 1961 Act allows the State Government to
reserve an area for public
utility use. After making such a reservation, any construction cannot be allowed
except when permitted by
the State Government.

. Page 19 of 40
71. Paradise residence is a newly constructed apartment in the outskirts of
Bhopal,MP and the land on which
it is constructed is taken on lease from Bhopal Municipal Corporation for a period
of 100 years. During the
time of booking the contractor had promised that there will be a community hall and
a park for children in
the premises of the apartment for use by the residents; but the majority of
residents with 50.5% votes
decided to expand the community hall and dismiss the idea of a park. In light of
the above facts, choose
the best option.
(a) The expanded community hall can be created after taking due permission from the
state government
as per section 282 of the M.P. Municipalities Act, 1961.
(b) Since the park has not been dedicated yet to the citizens, the community hall
can be constructed in its
place.
(c) Since the place for a park was already reserved during the booking, retracting
from it would amount to
breach of trust.
(d) None of the above.
72. Suppose in the previous question, the land for the construction of paradise
residence is not given on lease
to the contractor and the entire construction is also being done by the Bhopal
Municipal Corporation. In
that case, choose the best option as to whether a community hall can be constructed
in place of park.
(a) Change of use of park land for any other purpose by Municipal body would
tantamount to breach of
trust.
(b) There will be breach of trust because the majority has voted in the favour of
expansion of community
hall.
(c) Since during the time of construction the municipal corporation had reserved
the place for the creation
of park, it would require the permission of the state government to construct
something else on that
place.
(d) An expanded community hall can be constructed in the place of park
73. There was a lush green park in the Hauz Khas area of Delhi, which was being
extensively used by the local
residents for recreational purpose. However, the local authorities wanted to
construct their office in the
space which was occupied by the park. Decide in the light of Section 282 of the
aforementioned act.
(a) The Park cannot be replaced, as it is amount to breach of trust for it is held
by Municipality in trust on
behalf of public.
(b) The Park cannot be replaced, as it would interfere with the fundamental right.
(c) The Park can not be replaced, unless permitted by State Government;
(d) None of the above.
74. In the previous question, what would be your response if the construction is
permitted by the state
government?
(a) The construction would then be permissible, because the new office would
benefit the citizens.
(b) The construction would not be permissible, since it goes against the public
interest.
(c) The construction would be permissible, because the legislation allows for this
exception.
(d) None of these.
75. Ravindra Nagar is one of the best residential areas in Indore, MP. Glenmark
society is a colony located in
Ravindra Nagar. The residents of Glenmark approached the municipal office with a
complaint that there is
no recreational area in Ravindra Nagar. The municipal officer humbly clarified that
there is no space for to
build a park, as the area is densely populated. Aggrieved by this, the residents
told him to stop the
construction of a new building, which was being carried out by the government. They
insisted that the
building would be illegal, as it is occupying the land where a park could have been
constructed. The
residents mentioned the provision of MP Municipality act as mentioned in the
passage.
(a) The ongoing construction work shall be stopped, as the M.P. Municipalities
act,1961 makes it
mandatory for the government to have a recreational area.
(b) The ongoing construction work shall not be stopped, as setting up a park is not
mandatory for the state
government.

. Page 20 of 40
(c) The ongoing construction work shall be stopped, because it is occupying the
land reserved for a park.
(d) The ongoing construction work shall not be stopped, as it would go against the
order of government.
Passage (Q.76-Q.80): The Kerala High Court on Wednesday dismissed a petition
challenging the integrity
of a Government Order directing further investigation into a criminal case.
The question before the Court was whether an order issued by the State Government
directing to conduct
further investigation of a case was open to challenge by a total stranger to that
case.
The Court initially considered the question of locus standi in the matter.It
recalled that there is no provision
either in the Act or the Code which bars a citizen from filing a complaint for
prosecution of a public servant
who is alleged to have committed an offence.However, the Court noted that the right
to initiate criminal
proceedings does not mean right to challenge or interfere or meddle with the
investigation of a case.
Although the petitioner claimed to be a person relentlessly fighting against
corruption and the nefarious
activities of government servants, the court noted that even if the petitioner is a
crusader against corruption,
it does not confer him any special right to interfere with the investigation of a
case. The Court noted that
the petitioner was a total stranger to the case against the accused pending in the
Special Court.
He was neither the informant nor the complainant in that case. He was not a witness
or a direct victim of
the offence allegedly committed by the accused. He was not a person in any manner
affected by the
impugned order be it directly or indirectly.
On such grounds, the Court found it irresistible to conclude that the petitioner
has no locus standi to
challenge the order issued by the Government directing to conduct further
investigation of the case against
the second respondent.
76. X is a very responsible citizen who has time and again stood up against the
government for their poor
governance and lethargy in solving public grievances. X had a cousin whose wife (W)
once invested in a
particular mutual fund of a reputed and the only government bank which projected a
return of 20% per
annum, but W was not getting this expected return since past few years. X filed a
case against the
government for negligence in handling public money and putting the investors at
risk of losing their money.
Determine if X has the locus standi to file the case.
(a) No, X cannot file the case because he is not directly affected by the
performance of the mutual fund.
(b) Yes, since X’s own family member is affected by the poor performance of the
mutual fund.
(c) Yes, but prima facie, no case seems to arise because, mutual funds are
subjected to market risks and
can be due to falling stock market.
(d) No, X should have filed a case against the bank and not the government to prove
his locus standi.
77. Assume in the previous question, when X goes to file the case, he sees that Y
an assistant of one of the
judges that is the court master is asking everyone in front of his line a certain
money to put the case for
hearing before the court, which is a corrupt practise in itself. Since the court
master was asking everyone
some money, it was safe for X to assume that he will ask money from him as well,
infuriated by this he
comes out of the line, goes home and files an e-complaint against the court master.
Determine if X can file
the e-complaint in the light of the passage.
(a) X cannot file the case because he is not affected directly by the offence
committed by Y.
(b) X can file the case because he was a witness of the offence committed by Y
(c) X does not have a locus standi on the matter because Y is just asking the fees
for the procedure of
case filing to be completed.
(d) Since all the requisites of locus standi is fulfilled in the given facts, X can
file an e-complaint.

. Page 21 of 40
78. Suppose in the previous question, X before leaving the line was actually asked
by Y “Do you want me to
move your file up for 500 bucks” after which X got infuriated, left for home and
filed an e-complaint. Would
this change in facts change your response to the previous question?
(a) Yes, X has locus standi to file the case.
(b) No, X has locus standi to file the case, for he is complainant;
(c) Yes, X does not have locus standi to file the case, for he is not affected
directly by the act.
(d) No, X does not have locus standi to file the case.
79. Ram was a resident of the New Friends Colony in Delhi. On September 18, 2020,
he was arrested as the
prime suspect of a murder. The case had a lot of missing blocks; hence the State
Government ordered a
special investigation by CBI. Anuj, who was a neighbour, and friend of Ram,
objected to the investigation.
He was against this order because he believed that Ram was innocent and went on
stating some of the
facts which were not known before and sought to initiate criminal proceeding. Now,
although the newer
facts were noted by the CBI, they didn’t stop the investigation. Choose the best
option in the light of the
given facts.
(a) The investigation shall be stopped because Anuj has stated some new facts which
were not known
before.
(b) Anuj’s objection will have no effect on the investigation, as the state is
under no such obligation.
(c) Anuj’s objection will have a locus standi on the investigation, as he is not a
stranger to the case.
(d) The investigation shall be stopped for the meanwhile until the state is
convinced to carry out this
investigation even after knowing the new facts.
80. On the evening of 17th December, 2019, there happened to be an execution of a
man named Aman by the
special task force (STF). Although Aman was considered a suspect for several
crimes, Jalaj (a journalist)
was clear with the facts that the special task force executed the wrong person. For
this, he wanted to lodge
an official complaint against the STF for wrongfully killing a person. However, the
authorities declined his
application by stating that he is a complete stranger to this case. Choose the best
option in the light of the
given facts.
(a) Jalaj cannot file the complaint, since he is not even remotely affected by the
execution.
(b) Jalaj can file the complaint, since he is a complainant and an informant in the
said case; .
(c) Jalaj cannot file the complaint, as he is clearly not connected with the case.
(d) Jalaj can file the case as he is a journalist and the duty of a journalist is
to dig out truth and it is his right
to profess his profession
Passage (Q.81-Q.85): The Punjab and Haryana High Court today granted interim
anticipatory bail to
Singer Gurdas Maan in a case registered against him for his comments which hurt the
religious sentiments
of the Sikh community.
An FIR was registered under Section 295A of the Indian Penal Code against Maan by
Paramjit Singh Akali.
After the Additional Sessions Judge, Jalandhar, had denied Maan's plea for interim
relief, he approached
the High Court under Section 438 of the Code of Criminal Procedure, 1973 seeking
anticipatory bail.
Senior Advocate R.S. Cheema, appearing for Maan submitted that no case was made out
under Section
295A of IPC as there was no deliberate or malicious intention.
The Court further remarked,
"It would be pertinent to note at this stage that basic foundation of democracy is
secularism and freedom
of expression. However, with advancement of technology whatever positive social
media might have
provided but one thing is ensured that each and every act or conduct spreads like a
wild fire."
It was also opined that the apology rendered by Maan could have been a step taken
by him to resolve the
issue in the nascent stage. Thus, considering the facts and circumstances in
totality, interim bail has been
granted to Maan. "In the event of arrest, he shall be released on bail subject to
his furnishing adequate bail
bonds to the satisfaction of the Arresting Officer. He is directed to join the
investigation as and when called
for. He shall abide by the conditions as envisaged under Section 438(2) Cr.P.C",
the order adds.

. Page 22 of 40
81. JohnWick is a man of focus, commitment and sheer brawny will. He decided to
read the entire Hindu
mythology and then take a webinar on it. He did read all the books and, in the
webinar, gives out his opinion
that the Bindu mythology does not has strong female characters and in some or the
other way, although
indirectly it does promote patriarchy, however he did not have the intention to
defame any aspect of the
religion. The fans of Bindu mythology filed a complaint against John that he is
defaming the mythology with
his biased statements and has hurt their religious sentiments. Determine his
liability.
(a) John is not liable because his statements are merely in the form of opinions.
(b) John is liable because whatever he said will fall under the ambit of his right
to freedom of expression.
(c) John is not liable because he has just spoken for the Bindu mythology and not
Bindu religion hence
there is no scope of hurting religious sentiments.
(d) John is not liable because he did not have the intent to defame Bindu
mythology.
82. Suppose in the previous question, after reading the Bindu mythology he feels
that no one should read this
because it imparts wrong values and with no intent to defame Bindu mythology, he
says in his webinar that
no one should read it, it is not good, but immediately he realizes that he
shouldn’t have said that and
apologises in the same webinar. The rest of the facts remaining the same, would
this change in facts
change the liability of John?
(a) Yes, because he clearly had the intention to defame Bindu mythology.
(b) Yes, because he has exceeded his right to freedom of expression.
(c) No, because he retracted from his words almost immediately and apologised and
had no intention to
defame.
(d) No, because he resolved the issue at a very nascent stage.
83. Rajesh, Ram, and Shyam were three friends, who used to spend a lot of time
together. On one fine evening,
all the three friends were sitting at a tea stall. During the course of the
discussion, Rajesh started explaining
some historical facts about a religion ‘A’ that he read the previous day. This had
some myth-busters as
well, which were often ignored by the believers of ‘A’. Naresh overheard the
statements of while finishing
his cup of tea. Naresh was an ardent believer of ‘A’, and therefore he became angry
on the statements
made by Rajesh. He approached the nearby station to file a complaint as his
religious sentiments were
deliberately attacked by Rajesh. Choose the best option in the light of the given
facts.
(a) Rajesh has committed a wrongful act by hurting the religious sentiments of
Naresh.
(b) Rajesh has not committed any offence, since he had no intention of targeting
anyone’s religious
sentiments.
(c) Rajesh has not committed any offence, since he was merely telling about the
topic he has read recently.
(d) Rajesh has committed an offence, since he was negligent in explaining such
sensitive topics.
84. In the previous question, assume that Rajesh got engaged in a verbal fight with
Naresh after this incident,
where he uttered (once again) some objectionable comments against religion ‘A’,
knowing well that Naresh
is a believer of A religion. Will this change your answer to the previous question?
(a) No, because Rajesh did not have any intention to hurt Naresh’s sentiments
initially.
(b) Yes, because Rajesh deliberately uttered offensive comments against religion
‘A’ at the end.
(c) No, because Rajesh made objectionable comments in the middle of a fight.
(d) Yes, because Rajesh knew that Naresh was a believer of religion ‘A’.
85. Mahesh went to a club in Indore, where he met a group of people. After talking
to them, Mahesh got to
know that the group is an ardent supporter of DJG (a local political party), which
is led by Mr. Nandish.
Mahesh was a supporter of NDP, which is known to be a rival party of DJG. After a
heated argument about
their beliefs, Mahesh uttered some derogatory remarks about DJG and Mr. Nandish.
Suraj, who was a
member of the other group, felt extremely hurt by the remarks made by Mahesh. He
approached the
concerned authorities to file a complaint under Section 295A, as his beliefs were
targeted by Mahesh
deliberately.Choose the best option in the light of the given facts.
(a) Mahesh is liable under Section 295A, as he made offensive comments against
Suraj’s beliefs.
(b) Mahesh is not liable, as he made the comments against a political party and its
leader.

. Page 23 of 40
(c) Mahesh is not liable, as he was simply stating his own opinions.
(d) Mahesh is liable, as he deliberately uttered such derogatory remarks which
affected Suraj.
Passage (Q.86-Q.90): The Rajasthan Legislative Assembly has inserted a new
provision, namely Section
27-A into the Rajasthan Tourism Trade (Facilitation and Regulation) Act, 2010 via
an amendment,
applicable to the State of Rajasthan alone.
According to this provision, acts such as misbehaviour with tourists which take
place have been made a
cognizable offence (arrest without warrant) in the State, and a non-bailable one if
repeated. The State
initiated this amendment in light of a decision of the High Court in 2017 which
quashed an FIR filed under
the Act on the ground that the offences therein were not specifically provided as
cognizable under the
statute.
Tourism is one of the key industries in Rajasthan, a State visited by several
domestic and foreign tourists
on an annual basis. However, the tourists have often complained of touts, illegal
vendors and undesirable
elements during their visits. Touts, known locally as lapka, are a persisting
trouble at popular tourist sites
in the state. They often mislead and force tourists to shop – usually at exorbitant
prices in connivance with
the establishments – to make money and get a commission.
According to section 27-A,"All the offences punishable under the Act shall be
cognisable and bailable:
provided that the offences punishable under subsection (3) and sub-section (4) of
section 13 of the Act
shall be cognisable and non-bailable."
Section 13 (3) declares that whoever commits a subsequent offence of touting shall
be arrested without
warrant by police and, on conviction, be punished with rigorous imprisonment of up
to three years or a fine
of up to ₹ 30,000 or both.
Similarly, Section 13 (4) lays down that whoever, habitually engages in the
practice of touting even after
having been punished more than once, shall be arrested without warrant by police
and punished on
conviction with rigorous imprisonment of up to seven years or a fine up to ₹ 1 lakh
or both.
86. X was resident of US; he was very fascinated with the rich culture of India and
had always wanted to visit.
He made a plan to visit Rajasthan by 23rd September 2021; accordingly, he left US
on 20th September and
reached Delhi on 21st. With plenty of time in his hand he left early for Rajasthan.
With not more than 20km
of driving he encountered a huge market of all kinds of product; fascinated by the
same he stopped the car
but as soon as he stepped out of the car, a vendor Y came to him, trying to sell
him some illegal artefacts
in Delhi. He rushed out of the place and filed a complaint in a local police
station. Determine the liability of
Y in the light of the passage.
(a) The vendor is not liable because there is no law under which he can be booked.
(b) The vendor can be held liable because of the new law, which has made touting
and illegal vending a
punishable offence.
(c) The vendor is not liable because X was a foreigner and whereas the new law is
only applicable to
Indians.
(d) The vendor is liable because what he is selling is an illegal artefact.
87. Suppose in the previous question X rushed out of the market, reached Rajasthan
and then filed a complaint
in a police station of Jodhpur. Would this change in facts, change your answer to
the previous question?
(a) Yes, the vendor is liable as per the new laws of Rajasthan.
(b) No, the vendor is liable as per the new laws of Rajasthan.
(c) Yes, the vendor is not liable as per the new laws of Rajasthan.
(d) No, the vendor is not liable as per the new laws of Rajasthan.

. Page 24 of 40
88. Suzane was an American citizen, who was a travel blogger. She used to explore
places across the world
that were known for their rich culture and heritage. One fine day, she came across
a post on Instagram,
which highlighted the beauty of Agra and Fatehpur Sikri, Uttar Pradesh. Once she
was there, there were a
couple of local boys who intentionally misguided her by telling her the incorrect
location and also forced
her to shop at exorbitant prices in connivance with the establishments – to make
money and get a
commission. This caused a lot of trouble, as she was completely unaware of the
places. Now, she wanted
to file a complaint. Decide in the light of the aforementioned act in the passage.
(a) Suzane can file a complaint, as she was a tourist who faced inconvenience due
to local boys.
(b) Suzane can file a complaint as the same is mentioned in the new amendment act.
(c) Suzane cannot file a complaint since there is no law which covers her case.
(d) Suzane cannot file a complaint as there was no legal injury in this case.
89. In the previous question, assume that although Suzane went to the incorrect
place where she met her old
friend Asher; Suzane liked the place and started exploring it. However, when Asher
came to know about
the mischief of the local boys on telling intentionally telling her incorrect
location, he went ahead to file a
complaint (the act in passage being applicable though). Decide.
(a) Asher can file a complaint, as Suzane faced trouble due to the local boys
(b) Asher can file the case, as the law Rajasthan Tourism Trade (Facilitation and
Regulation) Act,2010 is
for protection of the tourist.
(c) Asher cannot file a complaint, as telling an incorrect location does not amount
to touting;
(d) None of the above
90. Rajesh is a local resident of Jaipur, Rajasthan. He was a tourist guide and
used to earn quite well. One
day, one of his clients filed a complaint against him for inappropriate behaviour
of touting. Once both the
parties were interrogated, Rajesh was acquitted as he got the benefit of the doubt
(due to lack of witnesses).
After a month, he engaged in a verbal fight with one of his clients, who later on
lodged a complaint against
Rajesh for touting again, pursuant to which he was arrested under Section 13.
Choose the best option in
the light of passage and the given facts whether Rajesh will be entitled to bail.
(a) Rajesh will not be eligible for bail, since he has committed this offense
twice.
(b) Rajesh will get bail, as he was acquitted during the first charge on him.
(c) Rajesh will be liable for touting, and subjected to heavy punishment.
(d) Rajesh will get bail, as this is the first case filed against him for
misbehaviour.
Passage (Q.91-Q.94): The Gauhati High Court observed that section 4 of the Special
Marriage Act does
not save a second marriage contracted by a Muslim male with a Hindu woman, and thus
it would be void.
As per Section 4 of the Special Marriage Act, one of the conditions relating to
solemnization of special
marriages, is that neither party should have a valid spouse living.
The court observed, "It appears that under the principles of Mohammedan law, the
marriage of Muslim
man with an idol worshiper(Hindu) is neither valid nor a void marriage, but is
merely an irregular marriage.
As per section 22 of the principles of Mohammedan law by Mulla (20th edition), the
capacity of marriage
relates to every Mohammedan of sound mind who had entered into the contract of
marriage. The petitioner
not being a Mohammedan, the marriage would not be a marriage under strict meaning
of the
Mohammedan law. In the present case in hand, it is seen that the petitioner was not
married as per
customary Mohammedan law but she was married under the Special Marriage Act, 1954
and that the
provisions of Section 4 of the said Act renders the marriage as valid, for neither
of the parties to the marriage
had any valid spouse living at the time of marriage .Even though the petitioner is
still using her Hindu name
and there is nothing on record to show that the petitioner had accepted the
religion of Islam as her
faith.", Justice Kalyan Rai Surana observed.

. Page 25 of 40
91. Ahmed (Muslim boy) and Simran (Hindu Girl) are 19 and16 years old respectively;
they fall in love and
decide to get married next year. Little did they know that Ahmed was already
married to Fatima when he
was just 2 years old as per Mohammedan law. However as per the law of land which
people of all religions
are bound to follow legal age to marry in India for girls is 18 years old and for
boys it is 21 years old, failing
which if any marriage is contracted then it is void ab initio. Both Ahmed & Simran
already knew that their
parents won’t agree to their marriage so they planned to elope next year but could
not due to Simran’s
board exams. After a couple of years they again plan to elope and marry as per
Special Marriage Act
because Simran and they were successful in executing the plan. Both their parents
want to nullify their
marriage stating it is void ab initio and thus filed a case. Choose the best option
in the light of the given
facts the validity of marriage of Ahmed and Simran’s marriage.
(a) The marriage between Ahmed and Simran is totally valid in the eyes of the law.
(b) The marriage between Ahmed and Simran is an irregular marriage.
(c) The marriage between Ahmed and Simran is void ab initio.
(d) The marriage between Ahmed and Simran is voidable not by their families but by
Fatima.
92. Suppose in the previous question, keeping in consideration the facts of
previous question are pari materia
in this question, Ahmed did not marry anyone before Simran, would this change in
fact, change your answer
to the above question?
(a) Yes, the marriage is valid in the eyes of law.
(b) No, the marriage is valid in the eyes of law.
(c) Yes, the marriage is not valid in the eyes of law.
(d) No, the marriage is not valid in the eyes of law.
93. Rita (a Hindu girl) and Saad (a Muslim boy) met on an online dating site and
fell in love. Although they were
from different religions, they never allowed this to hamper their love life. Both
had whole-heartedly accepted
the differences and used to take part in the religious activities of each other.
Now, they wanted to marry for
this, Rita and Saad married as per Special Marriage Act (“SMA”) and got their
marriage registered under
SMA and then as per both Hindu and Muslim customs for their personal satisfaction.
One day while they
were in market, Heeba (Saad’s 1st wife, a Muslim girl) saw them together. On being
caught, Saad accepted
that Heeba and he were not living together for a couple of years, but they didn’t
get a divorce yet. Now, to
save their marriage Rita said that she has accepted the Mohammedan religion and
hence claimed the
marriage has been solemnized as per the Mohammedan law. Choose the best option in
the light of the
given facts if their marriage would be considered as solemnised under Mohammedan
law.
(a) The marriage can not be solemnized as they have married as per both Hindu and
Muslim customs.
(b) The marriage cant not been solemnized, as the marriage was not performed under
Mohammedan law.
(c) The marriage can been solemnized, as Heeba and Saad were not together anymore.
(d) The marriage can not been solemnized, as Heeba was alive and the same cannot be
permitted.
94. Raju (a Hindu boy) and Himi (a Muslim girl) were dating each other from High
School. Although there were
several roadblocks in their relation, they had strongly passed everything. Now,
both of them were
successful in their respective careers and wanted to take this relationship one
step ahead. They
approached their parents to seek permission to marry. However, their parents denied
this due to religious
differences. Aggrieved by this, Raju and Himi decided that they will marry each
other without anyone’s
permission as per Mohammedan Law. In the light of these statements, choose the best
option.
(a) The marriage is void, as the male should be Muslim for the Mohammedan law to be
applicable.
(b) The marriage will be voidable at the option of the party who is a Muslim, i.e.,
Himi in this case. ‘
(c) The marriage will be valid as both of the parties were major and had the
capacity to enter into a
marriage.
(d) Can’t be determined..

. Page 26 of 40
Passage (Q.95-Q.99): …the sole act of protesting should not be employed as a weapon
to justify the
incarceration of those who are exercising this right," observed the Delhi High
Court as it granted bail to five
accused booked in connection with the murder of Head Constable Ratan Lal and
causing head injuries to
a DCP during the North-East Delhi riots that rocked the national capital last year.
Importantly, while granting them bail, the Court opined thus:
"It is the Constitutional duty of the Court to ensure that there is no arbitrary
deprivation of personal liberty
in the face of excess of State power." "Bail jurisprudence attempts to bridge the
gap between the personal
liberty of an accused and ensuring social security remains intact. It is the
intricate balance between the
securing the personal liberty of an individual and ensuring that this liberty does
not lead to an eventual
disturbance of public order. It is egregious and against the principles enshrined
in our Constitution to allow
an accused to remain languishing behind bars during the pendency of the trial."
the Court finally remarked:
"It is to be noted that the right to protest and express dissent is a right which
occupies a fundamental stature
in a democratic polity, and therefore, the sole act of protesting should not be
employed as a weapon to
justify the incarceration of those who are exercising this right."
95. A and his batchmates were protesting peacefully against the U.P government in
front of Raj Bhawan,
because an internal fraud was exposed by a newspaper that UP civil services board
has manipulated the
marksheets and good ranks were given to those who have bribed the officials. The
board denied all the
allegations and arrested A and his batchmates for tarnishing the image of the
state. Is the act of the
government in accordance to the principle laid down by the HC?
(a) Yes, since there was no fraud by the board and just a conspiracy theory by the
newspaper.
(b) No, since the protest was quite peaceful, the government by arresting them have
breached their right
to protest and express dissent.
(c) There is no need for the government to act in accordance with the principle
laid down by HC because
it was given by Delhi HC, whereas the protest and arrest happened in UP.
(d) No, the government should have only acted after finding were released by the
internal inquiry
committee.
96. Suppose in the previous question, it is soon accepted by the board that fraud
has been committed on their
side, which immediately turns the protest of A and his batchmates violent, which
leads to their arrest. Would
this modification of facts change your answer to the previous question?
(a) Yes, since the protest has now turned violent, the arrest was essential to
maintain public order.
(b) No, the principle laid by the HC does not differentiate between violent and
peaceful protest thus, any
protest leading to arrest is an infringement of the right of the accused.
(c) No, since the protest has now turned violent, the arrest was essential to
maintain public order.
(d) Both b & c
97. There was a group of fifteen people, who were protesting against a recent
municipal order. In the group,
there was a person named ‘A’, who used to get mental attacks which would make him
go a bit violent.
During the protest, A got the mental attack and started hitting random people.
After seeing this, the police
officials thought this to be a violent protest. All the protestors were arrested
for this. Decide.
(a) The police officials are justified in doing so, as it is reasonable to assume
that the protest was violent.
(b) The police officials are justified in doing so, since the protest became
violent.
(c) The police officials are not justified in doing so, since it wasn’t a violent
protest and did not affect
public order.
(d) The police officials are not justified in doing so, since police didn’t try
removing the protestors.

. Page 27 of 40
98. In the previous scenario, imagine that all the members of this group started
hitting people after A did the
same in a fit of rage. They started this after assuming that the other people have
started interrupting the
protest and attacking each other. In this light of this, choose the best option.
(a) The arrest is not justified, since they acted under a wrong assumption.
(b) The arrest is justified, because the protest became violent and disrupting
public order.
(c) The arrest is not justified, since it was only A who started violence.
(d) Arrest is justified here since deprivation of personal liberty is justified
here.
99. A group named ‘D’ (with over twenty members) was protesting in front of the
mayor’s office. After a heated
agreement with the mayor, the group turned violent and started destroying the
public buses and cars.
Seeing this, the mayor informed the police about the violent protest. Meanwhile,
Aniket, who was a close
ally of group ‘D’ came to the spot. He immediately handled the situation and the
group came at peace.
Further, they were trying to engage into a peaceful conversation. However, the
police arrested all the
members of that group along with Aniket. Decide as per the passage.
(a) The arrest is justified, since the group became violent.
(b) The arrest is not justified, since the group was finally at peace.
(c) The arrest is of the group is not justified, because Aniket brought them at
peace.
(d) Arrest is justified of all, except Aniket since they interrupted public order.
Passage (Q.100-Q.106): We usually find out how secure (or insecure) our lives are,
not from the
government. but from organisations and individuals committed to increasing
transparency around these
issues. This time around, an international collaborative investigation titled
“Pegasus Project”, has accessed
a leaked database of phone numbers, supposedly provided by government clients of an
Israeli spyware
firm called NSO.
This is deeply concerning - the government has not categorically denied conducting
surveillance using the
Pegasus spyware. Notably, Section 69 of the Information Technology Act, 2000 (IT
Act) authorises the
government to conduct surveillance on being confirmed about happening of an
offence. The draconian
provision permits the government to intercept, monitor or decrypt information
without any judicial oversight.
However, the provision does not permit the government to install spyware or hack a
mobile device. In fact,
Section 66 read with Section 43 of IT Act, 2000 explicitly criminalizes hacking of
a mobile device.
Moreover, the constitutional standard established in KS Puttaswamy v. Union of
India requires that any
instance of surveillance must be legitimate, proportionate and necessary and in
conformity with law.
Surveillance conducted using the Pegasus spyware, which is extremely intrusive,
does not conform to any
of those requirements, especially since “less intrusive” alternatives are available
to the government such
as availing information from government department/body. While the judgement has
held that Indians have
a fundamental right to privacy, we do not know how private our lives are from the
Government of India. The
fundamental freedoms of citizens cannot be advanced until they are made aware of
the extent to which
they may potentially be infringed.
100. Mr. A resided in the Republic of Killdale. He owned a business of importing
iPhones from the surrounding
countries and sold the same in Killdale. The Government of Killdale observed the
business of Mr. A and
conducted surveillance on his phone on the suspicion that he is importing the
iPhones without paying the
custom duties and did not avail information from the concerned Custom office
department. Choose from
the most appropriate option –
(a) The Government of Killdale has justified reason to conduct surveillance as not
paying custom duties is
a crime against the state and shall be detected and punished immediately.
(b) The Government of Killdale does not have justified reason to conduct
surveillance as the same is not
proved in the instant case ;
(c) The Government of Killdale does not have justified reason to conduct
surveillance as less intrusive
alternatives were available to the government.
(d) Both b) and c)

. Page 28 of 40
101. Mr. C has an import and export business based in Chennai. His business is
registered to import and export
advanced fishing products and can only deal in this domain. An encrypted WhatsApp
chat screenshot leaks
and reaches the police which shows Mr. C making a deal to import drugs via his
business. The police gets
furious and tries to bring Mr. C in for questioning but he is not found at the
registered address after many
attempts and has allegedly fled the state. The police after authorization from IT
Department and court
conducts surveillance on his phone and receives vital evidence sufficient to arrest
him. Choose from the
most appropriate option –
(a) The Police is not justified in conducting the surveillance as they had less
intrusive alternatives available
to them.
(b) The Police is not justified in conducting the surveillance as the same was not
proportionate to the facts
of the case.
(c) The Police is justified in conducting surveillance as they tried less intrusive
means and also have strong
evidence to conduct surveillance on his phone.
(d) The Police is not justified in conducting the surveillance as Mr. C allegedly
fled the state and the same
is not proven.
102. Ms. S is a shop owner in Mumbai. She has been involved in a few criminal cases
of which 2 are still
pending and has also faced jail time once. In July 2021, Mr. F, a famous criminal
gang member being
observed by the police visits the shop of Ms. S to buy 2 packets of cigarettes. The
police suspected criminal
activity due to the past records of both the shop owner and the customer and tapped
and hacked Ms. S’s
mobile phone to ensure there is no criminal activity. Choose from the most
appropriate option –
(a) The Police is justified in conducting surveillance as they have reasonable
doubt of criminal activity and
it is their duty to ensure peace and stability
(b) The Police is not justified in conducting surveillance as Ms. S just sold the
cigarettes to Mr. F and the
same is not enough to breach her right to privacy
(c) Mr. F might have been supplied with some criminal information in the cigarette
packets and the
hence, Police is justified in conducting surveillance
(d) The Police is not justified in conducting surveillance through hacking as it is
not legitimate, necessary
or proportionate and in conformity with law in this case and there are less
intrusive means to ensure
safety.
103. Mr. A is a well known Jihadist in the Kashmir region running a group of
teenage militants. The security
agencies have intelligence reports from reliable sources about this group led by
Mr. A being responsible
for various bombings in Kashmir, killing various civilians including women and
children. They security
agencies on July 19, 2021 got information from a reliable source that Mr. A had
been contacting his teenage
militants and coordinating with them for a massive bombing within 15 days. The
security agencies installed
an over-the-air malware and hacked into Mr. A’s phone in order to tap his phone and
get the call records
as it was a question of the life of hundreds of civilians. Choose from the most
appropriate option –
(a) Information Technology Act, 2000 (IT Act) does not authorise the action taken
by the security agencies.
(b) The security agencies have immunity under Section 69 of the Information
Technology Act, 2000 (IT
Act) and are justified in the phone tapping
(c) As the question for of the lives of hundreds of innocent civilians, the
security agencies are justified in
the phone tapping
(d) There are less intrusive ways than to tap the phone of Mr. A in order to stop
the bombing and the route
chosen by the security agencies is justified.

. Page 29 of 40
104. Which of the following will the author agree with the most
(a) Surveillance is permissible if legitimate, proportionate and necessary;
(b) Surveillance is permissible be it by way of hacking if it is legitimate,
proportionate and necessary and
in accordance with law;
(c) Any act of surveillance if it involves hacking is not permissible as it is
against the law of the country;
(d) Section 69 of the Information Technology Act, 2000 (IT Act) authorizes hacking
as there are less
intrusive means of solving a case.
105. Mr. A is a central government employee working since 18 years. He has been in
constant touch with the
Sri Lankan Naval officers for routine updates concerning national security. CIB, an
Indian Security agency
receives information that Sri Lankan fisherman are going to enter Indian Waters in
order to import illegal
goods into the port of Chennai. In order to confirm this news, CIB tapped and
hacked into Mr. A’s phone
and realised that indeed this information was true. Due to this tapping they could
stop the illegal
import. Choose from the most appropriate option deciding whether such phone tapping
is justified–
(a) Software’s like Pegasus keep track of government employees handling vital
information in order to
combat situations like these and the CIB too has the right to use the same
(b) The right to privacy of Mr. A was not breached as only the calls relating to
his official capacity with the
Sri Lankan fisherman were used to confirm the information of the illegal import
(c) CIB cannot carry out such phone tapping as software’s like Pegasus do not come
under the ambit of
Section 69 of the Information technology Act, 2000
(d) CIB cannot carry out such phone tapping as it is not proportionate, necessary
or legitimate and in
accordance with law.

. Page 30 of 40
SECTION - D: LOGICAL REASONING
Directions (Q.106-Q.110): Read the passage carefully and answer the questions that
follow.
A common scenario in my clinical neuropsychology practice: Jamie's parents tell me
that the school
referred him for an ADHD evaluation and that his teachers report he is unfocused
and can't sit still. He
starts an assignment and then goes onto something else. He is always talking to
others in the class, walks
around looking for something to do, and can't complete his work in a timely
fashion. His teachers report
that he is easily frustrated, gives up, and has begun to say he feels stupid. He
loses focus if something is
the least bit challenging, and his teachers think he needs to try harder.
But at home, we don't think he has ADHD. He can play Minecraft for hours on end and
is so focused he
often doesn't hear us when we tell him it's time for dinner. He sits still without
moving and doesn't want to
stop until he's finished what he is doing. He is also amazing at Minecraft and
other games. He loves to
show us what he is doing and tells us that he solves many problems while playing
his video games. If
anything, we are concerned that he is too focused.
My colleagues and I at Learning Works for Kids and South County Child and Family
Consultants have
conducted research with more than 500 parents of children with ADHD to explore some
of these issues.
We have noticed that many kids with a definitive diagnosis of ADHD can very easily
stay focused while
engaged with screen-based media such as video games, YouTube, and social media.
Leading thinkers
and researchers on ADHD such as Drs. Russell Barkley, Ned Hallowell, and Thomas
Brown minimize
issues of inattention, distractibility, and hyperactivity in their descriptions of
ADHD, instead of focusing on
the impact of poor executive-functioning skills in areas such as organization, time
management, planning,
working memory, and task persistence as the core features of ADHD. Of particular
note is that these
experts characterize individuals with ADHD by their inability to sustain attention
and effort in activities that
do not interest them.
106. Which of the following can we infer from the passage above?
(a) Jamie shows contrasting behaviours when at school and while at home.
(b) ADHD is a disorder of intention rather than attention.
(c) The author is a neurophysiologist by profession.
(d) Only (a) and (c)
107. Which of the following most accurately expresses the main point of the
passage?
(a) Children often like activities that involve screen-based technologies such as
video games and social
media.
(b) We are less likely to see the symptoms of ADHD in children who are engaged in
activities that fully
engage them.
(c) Many children with ADHD will become adults whose attention issues are less
problematic.
(d) The difficulty for many children with ADHD is in shifting attention from
something that engages them to
something that is less engaging.
108. Which of the following can be the course of action to address the issue of
ADHD in children?
(a) Engage a child with ADHD in less engaging areas for a longer duration to make
them stick to the task.
(b) Teens and adults with ADHD are often able to overfocus while using technology.
(c) Identify areas where kids are fully engaged and attentive, and how to nudge,
foster, and promote these
activities in a productive, future-oriented manner.
(d) Children with ADHD should not be allowed to engage in activities which do not
interest them.

. Page 31 of 40
109. Which of the following, if true, would weaken the argument mentioned in the
passage?
(a) Most children with ADHD master computer games featuring challenging skills but
do poorly in skills
requiring consistency.
(b) When kids with ADHD are engaged in their interests, they can be passed on as
normal kids.
(c) The medical insurance of children with ADHD is exorbitantly high, making it
impossible for parents to
afford one.
(d) Children with an intelligent quotient of a genius exhibit the same behavioural
discrepancy as a child
with ADHD.
110. Which of the following would form the premise if the author makes an argument
that parents should make
it easy for their kids with ADHD to expand the scope of interests and foster
learning and expertise in
activities that engage the child?
(a) It is often parents who, intentionally or unintentionally, become an obstacle
in the development of their
kids with ADHD by imposing restrictions.
(b) Kids with ADHD have limited scope in their areas of interest even if given
proper nurturing and
guidance.
(c) A kid with ADHD is better nurtured and handled in schools than at their
respective homes.
(d) While most kids with ADHD are unlikely to become video game designers or
professional gamers,
computer and technology skills are critical in many professions and emerging
fields.
Directions (Q.111-Q.115): Read the passage carefully and answer the questions that
follow.
Did your mom or dad tell you, "Breakfast is the most important meal of the day."
Health professionals,
including me, often advised eating a healthy breakfast, even for people trying to
lose weight. Skipping
breakfast meant that it would be difficult to control eating at the next meal,
typically at lunchtime. I often
told my clients, "If you had dinner at 7:00 last night, and lunch was at noon
today, 17 hours of food
deprivation would result in ravenous hunger, making it likely that you'd eat too
much."
When I offered this advice to my weight-conscious clients I would often get
objections like, "I'm never
hungry in the morning. Why should I eat three or four hundred calories that I don't
want and don't need?" I
would reassure them that having breakfast would protect them from eating too much
at lunch. New findings
suggest that, along with most other health professionals, I was wrong.
A new meta-analysis combined the data from seven controlled studies evaluating the
effects of skipping
breakfast on body composition and cardio metabolic risk factors. Only trials
lasting at least four weeks were
included with a total of 435 adult participants. Five of the trials were conducted
with participants who had
obesity or were overweight, while participants in two of the trials were normal
weight. The results showed
that breakfast skippers had a greater reduction in body weight (1.19 lbs.} compared
to breakfast eaters.
There were no significant differences in Body Mass Index (BMI) or fat mass. LDL
cholesterol increased
significantly in breakfast skippers but no differences were found for blood
pressure, HDL, insulin, fasting
glucose, or other cardio metabolic measures. The researchers concluded that neither
skipping nor
consuming breakfast will have a clinically significant effect on weight loss but
the quality of the breakfast
should be considered.
111. Which of the following can be inferred from the given passage?
(a) Two persons of different weights can have the same or similar BMI.
(b) Having a breakfast ensures that one does not overeat during lunch.
(c) Most of the people are not hungry in the morning.
(d) It is what you eat in the breakfast that will decide the weight loss; skipping
or taking breakfast does not
significantly contribute to weight loss.

. Page 32 of 40
112. Which of the following is the underlying assumption on which the statement
"Only trials lasting at least four
weeks were included with a total of 435 adult participants" is made?
(a) The participants in the trials were available for at least four weeks.
(b) Trials lasting for less than four weeks do not necessarily show any effect on
body composition and
cardio metabolic risk factors.
(c) Every test trial is consistent on certain parameters; especially number of days
and participants.
(d) To reach a proper conclusion on test trials, there is a requirement of a
minimum number of days and
participants.
113. Which of the following can be the course of action to address the dilemma of
having breakfast or not?
(a) Have breakfast that is high in protein and fibre, gives energy throughout the
day and also helps in
building lean muscle mass, unlike carbohydrates.
(b) Different people have different food requirements and there cannot be a single
marker in terms of losing
weight.
(c) Having a hearty breakfast and skipping other meals will help in losing weight.
(d) One should skip breakfast if one is trying to lose weight.
114. Which of the following, if true, would weaken the argument presented in the
passage regarding the trials
conducted?
(a) Both (b) and (c)
(b) Standard operating procedures (SOP) of measuring the effects of breakfast on
body composition and
cardiometabolic risk factors must include trials lasting at least a quarter.
(c) For a fair and accurate analysis, the number of trials with obese or overweight
people should be equal
to that with people of normal weight.
(d) A healthy breakfast should contain whole grains, lean protein, low-fat dairy,
and fruits and vegetables.
115. Which among the following most accurately reflects the main idea, as set out
in the passage above?
(a) Lunch is as important as breakfast, if not more.
(b) Should you skip breakfast if you are trying to lose weight?
(c) Does skipping or having breakfast help in weight loss?
(d) The quality of breakfast determines its consumption.
Directions (Q.116-Q.120): Read the passage carefully and answer the questions that
follow.
As artificial light has spread across cities, roads and through industry,
unadulterated night skies have
become something of a rarity. That's a particular bugbear for astronomers -
artificial light obscures their
view of the faintest objects. It can also have a negative influence on mental
health and changes the ways
animals forage for food and even reproduce. That raises the obvious question of how
to tackle light pollution
effectively. Policy makers need to understand how their regions compare with other
similar ones. The
general view is that areas of high population density pollute more than areas of
low density. But actual light
emissions vary in a more fine-grained and subtle way.
Enter Fabio Falchi at the Light Pollution Science and Technology Institute in
Theine, Italy, and a number
of colleagues, who have developed a way to compare the pollution from similar areas
in an objective way.
Their analysis reveals some surprising effects. For example, densely populated
cities are by no means the
worst offenders. This study has helped to pinpoint important factors such as the
type of infrastructure, the
type of industry and the local differences in energy efficiency, such as the move
in many cities to LED street
lighting which is significantly more energy efficient than conventional sodium
lighting but at least 2 times
brighter. That's why this kind of analysis can help with policy decisions to reduce
the difference.
For Falchi and co, the source of the pollution is secondary to its effect. The
bigger question is whether
communities can come together to solve the problem of light pollution, which is
relatively straightforward in
the greater scheme of things. The researchers paint a stark contrast to more
important but complex
problems such as reducing carbon emissions and tackling global heating. "If we are
unable to solve this
problem, for which the countermeasures are well Known, then our ability of solving
more complex
environmental problems, such as global warming, will remain in doubt."

. Page 33 of 40
116. Which of the following is the author most likely to agree with?
(a) Solving the problem of light pollution is easier when compared to some other
environmental issues.
(b) Promoting the usage of LED lighting is one of the steps taken by the government
to solve the problem
of light pollution.
(c) Light pollution affects only humans as they live in urban areas.
(d) Light pollution was noticed when astronomers found it hard to see objects in
the night sky.
117. Which of the following cannot be a course of action to reduce light pollution
according to the passage?
(a) Investing money into the research done by scientists like Fabio Falchi, so that
they can conduct similar
studies.
(b) Replacing conventional sodium street lamps with LED lighting.
(c) Taking factors like type of infrastructure and type of industry into account
before taking decisions related
to the kind of lights to be used.
(d) Finding an alternative to the LED that is more energy efficient and
environmentally friendly.
118. Which of the following can be inferred from the passage?
(a) Light pollution has also adversely affected animal habits.
(b) Villages are the worst offenders of light pollution.
(c) Light pollution is not a serious issue in the eyes of policymakers.
(d) Communities need to coordinate among themselves to solve the problem of light
pollution.
119. Which of the following most accurately expresses the main point of the last
paragraph of the passage?
(a) Communities need to join hands to combat the menace of light pollution.
(b) Light pollution is a relatively easier problem but solving it would prove if
we're capable of solving larger
problems like global warming.
(c) Global Warming and Carbon Emissions are the more important problems that our
planet faces and we
should not waste our time on light pollution.
(d) Policy makers need to make a decision about how they intend to solve the
problem of light pollution.
120. Which of the following will undermine the arguments presented in the passage?
(a) The research by Falchi is not substantiated by further research on light
pollution.
(b) The research by Falchi has paved the way for finding alternate resources to
combat light pollution.
(c) Falchi's research acts as a precedent in many conferences when discussing light
pollution.
(d) Falchi's research has proven that the problem cannot be solved by policy makers
alone.
Directions (Q.121-Q.125): Answer the questions based on the information given in
the passage.
The economy of India, when measured in USD exchange-rate terms, is the twelfth
largest in the world, with
a GDP of US $1.25 trillion (2008). It is the third largest in terms of purchasing
power parity. India is the
second fastest growing major economy in the world, with a GDP growth rate of 9.4%
for the fiscal year
2006—2007. However, India’s huge population has a per capita income of $4,542 at
PPP and $1,089 in
nominal terms (revised 2007 estimate). The World Bank classifies India as a low-
income economy. India’s
economy is diverse, encompassing agriculture, handicrafts, textile, manufacturing,
and a multitude of
services. Although two-thirds of the Indian workforce still earns their livelihood
directly or indirectly through
agriculture, services are a growing sector and play an increasingly important role
in India’s economy. The
advent of the digital age, and the large number of young and educated populace
fluent in English, is
gradually transforming India as an important ‘back office’ destination for global
outsourcing of customer
services and technical support. India is a major exporter of highly-skilled workers
in software and financial
services, and software engineering. Other sectors like manufacturing,
pharmaceuticals, biotechnology,
nanotechnology, telecommunication, shipbuilding, aviation and tourism are showing
strong potentials with
higher growth rates. India can increase its per- capita income by at least 100% in
the next decade by
concentrating on product manufacturing instead of services. India followed a
socialist-inspired approach
for most of its independent history, with strict government control over private
sector participation, foreign
trade, and foreign direct investment. However, since the early 1990s, India has
gradually opened up its

. Page 34 of 40
markets through economic reforms by reducing government controls on foreign trade
and investment.
Liberal economic policies are responsible for the strong economic growth in the
last decade. The
privatization of publicly owned industries and the opening up of certain sectors to
private and foreign
interests has proceeded slowly amid political debate. Also, India faces a fast-
increasing population and the
growing challenge of reducing economic and social inequality.
121. Which of the following statements is the author likely to go towards?
(a) Other growing economies like Brazil have a much higher per-capita income.
(b) India must concentrate on education to keep up the supply of highly skilled
workers.
(c) Poverty remains a serious problem in India.
(d) The excellent performance of the stock exchanges in the country is an indicator
of the growth potential
in India.
122. Which of the following if true, would strengthen the argument above regarding
liberal economic policies?
(a) The benefits of liberalization have not reached all sections of society in
India.
(b) The average growth rate following liberalization has been constantly above 7%
as compared to an
average of 3% before liberalization.
(c) Socialist policies, if implemented properly without corruption in the
government can lead to the overall
welfare of the people.
(d) Many industrialists and entrepreneurs support liberalization because it suits
their needs.
123. Which of the following assumptions has the author made in the passage above?
(a) Most countries do not have adequately skilled workers to compete with India in
the services sector.
(b) The World Bank is the best authority on world economy.
(c) The Indian government realized a few years ago that India cannot - survive
without liberalization.
(d) The economic growth has not benefited all classes of society in India equally.
124. Which of the following if true would weaken the argument above about
increasing per-capita income by
concentrating on product manufacturing?
(a) Product manufacturing requires heavy investments in infrastructure.
(b) India’s core strength is a huge population of highly skilled and educated
workers which is most suited
to benefit from the highly lucrative global services market.
(c) Other countries like China and Brazil are very strong in product manufacturing.
(d) The profit margins in product manufacturing are much higher than those in the
services sector.
125. The author of the above passage is probably:
(a) an industrialist
(b) an economist.
(c) a journalist.
(d) a politician.

. Page 35 of 40
Directions (Q.126-Q.130): In the form of inference/conclusions are based on the
passages given below.
The passage is followed by five inferences. You are required to examine each
inference separately in the
context of the passage and decide upon its degree of truth or falsity.
The Union Commerce and Industry Minister on Friday unveiled a new corporate logo
for the National Textile
Corporation (NTC), reflecting the new face of the state-owned enterprise as part of
the plans to make it a
world-class textile company. "The Government is committed to making NTC a world-
class eco-friendly
integrated textile company and flag bearer of the sector. The ongoing revival-cum-
modernization
programme and new corporate and brand identity launch by NTC is aimed at achieving
this goal", he said
at a function organized for the purpose.
He said NTC was a key player in the textile sector and its recent moves would
definitely transform it into
an integrated company capable of addressing the needs of the nation. NTC had made a
turn-around within
a short span to emerge as a debt-free company with a highly competitive revival
strategy, he added. He
also launched two new look retail stores of NTC-one in Delhi and another in
Coimbatore, Tami Nadu via
satellite link up. This is a part of revamping of NTC's 93 stores to reflect the
company's new face across
the country.
Speaking on the occasion, the Textile Secretary said the revamp and modernization
steps taken by NTC
in
the recent past in view of the changing times and consumer aspirations of modern
India would help in
instilling confidence in the minds of all stakeholders. The NTC Chairman and
Managing Director said the
new corporate logo reflected the new face of NTC to the world. It had been designed
as free flowing fabric,
indicating the new aspirations and changing aspirations of India.
Apart from re-branding, NTC had developed a new marketing and corporate strategy
that included
revamping of all NTC stores and setting up of new stores, he said. The internal
marketing programme to
employees and key customers includes undertaking a complete programme to bring the
new vision and
culture to its employees. Respective mill managers would be conducting a smaller
unveiling ceremony of
the new logo along with the NTC anthem at all the mills. Besides, all the employees
would be given a small
commemorative badge of the new logo.
126. All these changes look to make NTC one of India's best textile companies.
(a) The inference is definitely true, i.e., it properly follows from the statement
of facts given
(b) The inference is probably true though not definitely true in the light of the
facts given.
(c) The data are inadequate i.e., from the facts given you cannot say whether the
inference is likely to be
true or false.
(d) The inference is probably false though not definitely false in the light of the
facts given
127. The new corporate logo of NTC reflects its new face to the world.
(a) The inference is definitely true, i.e., it properly follows from the statement
of facts given
(b) The inference is probably true though not definitely true in the light of the
facts given.
(c) The inference is definitely false, i.e., it contradicts the statement of facts
given
(d) The inference is probably false though not definitely false in the light of the
facts given
128. The highly competitive revival strategy may have helped make NTC debt-free.
(a) The inference is definitely true, i.e., it properly follows from the statement
of facts given
(b) The inference is probably true though not definitely true in the light of the
facts given.
(c) The data are inadequate i.e., from the facts given you cannot say whether the
inference is likely to be
true of false.
(d) The inference is probably false though not definitely false in the light of the
facts given

.
129. The recent modernization steps taken by NTC may instil confidence in the minds
of all stakeholders.
(a) The inference is definitely true, i.e., it properly follows from the statement
of facts given
(b) The inference is probably true though not definitely true in the light of the
facts given.
(c) The data are inadequate i.e., from the facts given you cannot say whether the
inference is likely to be
true of false.
(d) The inference is probably false though not definitely false in the light of the
facts given.
130. Implementing a programme to bring the new vision and culture to the employees
and NTC.
(a) The inference is definitely true, i.e., it properly follows from the statement
of facts given.
(b) The inference is probably true though not definitely true in the light of the
facts given.
(c) The data are inadequate i.e., from the facts given you cannot say whether the
inference is likely to be
true of false.
(d) The inference is probably false though not definitely false in the light of the
facts given
Directions (Q.131-Q.135): Read the following passage and answer the questions.
In nature, everything is connected. This is equally true of a healthy economy. We
cannot sustain life without
taking care of nature. And we need healthy economies to lift people out of poverty
and achieve the United
Nations Sustainable Development Goals. In our current model, these goals seem to
collide and our
economic pursuits encroach too closely on nature. But nature is what makes industry
possible. We cannot
have human development without a healthy natural world. The bottom line is that
when we damage the
natural world, we damage ourselves. Our growing economic footprint threatens our
own future. With the
projected rise in ocean levels and the average temperature of the planet, large
swaths of land, even whole
countries, will become uninhabitable, triggering mass climate-induced migration.
Since the natural and
economic worlds are linked, similar principles apply to both. In the financial
world, we would not eat into
capital to the point of depletion because that would bring financial ruin. Yet in
the natural world, we have
done this repeatedly. We must treat the natural world as we would the economic
world-protecting natural
capital so it can provide benefits well into the future. We can ensure that the
price of fossil fuel energy
reflects not only production costs but also environmental costs. We must eliminate
energy subsidies that
encourage new fossil fuels or promote overuse and waste. IMF research found the
implicit global subsidy
from undercharging for energy and its environment costs in 2017 was a staggering
$5.2 trillion, or 6.5
percent of world GDP. The private sector can stop supporting industries that damage
the planet and instead
invest in sustainable development. Governments can roll out policies to fight
climate change and the
destruction of nature, for example, through promotion of clean-technology research.
131. According to the author, natural world should be protected as natural capital.
Which of the following
statements is analogous to above reasoning?
(a) In financial world we should not eat into capital to the point of depletion
because that would bring
financial ruin.
(b) Our growing economic footprint threatens our future.
(c) In nature everything is connected.
(d) Our economic pursuits encroach too closely on nature.
132. Consider the following statements from the passage and answer accordingly.
Assertion (A): Climate change is the world’s existential crisis.
Reason (R): Large swaths of land, even whole countries will become
uninhabitable.OPTIONS:
(a) A is true but R is false
(b) A is false but R is true
(c) Both A and R are true and R is the correct explanation of A
(d) Both A and R are true but R is not the correct explanation of A.
. Page 37 of 40
133. Statement: We must eliminate energy subsidies that encourage new fossil fuels
or promote overuse and
waste.
Assumption I: Implicit global subsidy from undercharging for energy and its
environmental cost is reflected
in the world GDP.
Assumption II: Price of fossil fuel energy reflects not only production costs but
also environmental costs.
OPTIONS:
(a) Only assumption I is implicit in the statement
(b) Only assumption II is implicit in the statement
(c) Both assumptions I and II are implicit in the statement
(d) Neither assumption I nor assumption II is implicit in the statement
134. According to the passage, which of the following course of actions will
achieve the United Nations
Sustainable Development Goals?
(a) The private sector should stop supporting industries that damage the planet and
instead invest in
sustainable development.
(b) Healthy economies should lift people out of poverty.
(c) Governments should roll out policies to fight climate change and the
destruction of nature by promoting
clean-technology research.
(d) None of the above.
135. According to the passage what is the problem with our current economic model.
(a) Economic pursuits leverage too much on nature.
(b) Pricing of fossil fuels is not calibrated up to industry mark.
(c) Energy subsidies are not regulated by the governments.
(d) Recapitalization is needed for the current economic world

mock 15

Directions (Q.1-Q.30): Read the passages carefully and answer the questions based
on it.
Passage (Q.1-Q.5): To turn my eyes outwards now, and to say a little about the
relationship between the
Indian writer and the majority white culture in that midst he lives, and with which
his work will sooner or
later have to deal: Common to many Bombay-raised middle-class children of my
generation, I grew up
with an intimate knowledge of, and even sense of friendship with, a certain kind of
England: a dream#England composed of Test Matches at Lord's presided over by the
voice of John Arlott, at which Freddie
Truman bowled unceasingly and without success at Polly Umrigar; of Enid Blyton and
Billy Bunter, in
which we were even prepared to smile indulgently at portraits such as 'Hurree
Jamset Ram Singh', 'the
dusky nabob of Bhanipur'. I wanted to come to England. I couldn't wait and to be
fair, England has done
all right by me, but I find it a little difficult to be properly grateful. I can't
escape the view that my relatively
easy ride is not the result of the dream- England's famous sense of tolerance and
fair play, but of my
social class, my freak fair skin, and my 'English' English accent. Take away any of
these, and the story
would have been very different because of course, the dream-England is no more than
a dream. Sadly,
it's a dream from which too many white Britons refuse to awake. Recently, on a live
radio programme, a
professional humanist asked me, in all seriousness, why I objected to being called
a wog. He said he had
always thought it a rather charming word, a term of endearment. 'I was at the zoo
the other day, 'he
revealed, 'and a zookeeper told me that the wogs were best with the animals; they
stuck their fingers in
their ears and wiggled them about and the animals felt at home. As Richard Wright
found long ago in
America, black and white descriptions of society are no longer compatible. Fantasy,
or the mingling of
fantasy and naturalism, is one way of dealing with these problems. It offers a way
of echoing in the form
of our work the issues faced by all of us: how to build a new, 'modern' world out
of an old, legend-haunted
civilization, an old culture which we have brought into the heart of newer one.
'The ghost of Hurree Jamset
Ram Singh walks among us still. But whatever technical solutions we may find,
Indian writers in these
islands, like others who have migrated into the north from the south, are capable
of writing from a kind of
double perspective: because they, we, are at one and the same time insiders and
outsiders in this society.
This stereoscopic vision is perhaps what we can offer in place of 'whole sight'.
1. The author's experience in England is not the normative experience of an
ordinary Indian because
(a) Like the author, the ordinary Indian has not nurtured a "dream England" or
aspirations of reaching the
dreamland.
(b) Like the author, the ordinary Indian has acquired airs and nuances that are
essentially English, but
are betrayed by his/her complexion.
(c) Unlike the author, the ordinary Indian has a radical sense of nationhood,
citizenship and identity
(d) Unlike the author, the ordinary Indian has no knowledge of the radical cultural
difference that exists
between the two countries.
2. The professional humanist whom the author met regards the term "wog" as a
charming word and a term
for endearment. Which the following statement is true in the light of the comment
of the zookeeper that
the humanist shared?
(a) The humanist thinks that the "wog" is a harmless term, and it becomes more
positive in the light of the
zookeeper's comment.
(b) The zookeeper's comments subvert the harmless connotation of the term wog.
(c) The humanist thinks that the wog is a humanist term, and its meaning remains
the same despite the
zookeeper's comment
(d) The zookeeper comment does not have any implication on the meaning attributed
by the humanist.

. Page 3 of 44
3. “…the dream-England is no more than a dream. Sadly, it's a dream from which too
many white Britons
refuse to awake.” What is the author implying through the statement?
(a) White Britons are happily ensconced in their make-belief world and are in
denial of reality existing in
the UK.
(b) White Britons are happy to carry on with the traditional values and do not want
to adopt new world.
(c) White Britons have readily accepted people from different culture as their own.
(d) The perception of England free of biases is embedded in the white Britons and
are in denial about
their sense of tolerance and fair play.
4. “I can't escape the view that my relatively easy ride is not the result of the
dream…” The highlighted word
(result) belongs to which part of speech?
(a) Noun (b) Verb (c) Adjective (d) Adverb
5. Which of the following reflects the stereotyped vision, as mentioned in the
passage?
(a) A broad vision encompassing and absorbing the society as a whole.
(b) A narrow vision seeing only a limited spectrum in the social context.
(c) A vision full of reverie and fantasy world.
(d) A vision that pierces through the myths existing in the society.
Passage (Q.6-Q.10): Many of us want to be different. We may desire to be more
extroverted, spontaneous,
or productive. On a more concrete level, we may need to exercise three days a week,
consume less
alcohol, or lose 20 pounds. One scientific fact we know for sure is that when we
want to change our
personalities, we have both the capacity and the ability. The quest for personal
development and change
does not imply malcontent but instead suggests the ubiquitous human need to
improve.
Improvement comes in many forms and may include reaching goals such as enhancing
skills, becoming
healthier, or appearing more sociable. However, personal development has
psychological costs that hold
some people back. When we contemplate change, we also acknowledge an existing
deficit or weakness,
a realization that some are ill-prepared to confront. We also must devote
considerable effort to the
challenge. In many cases, change minimally means habit revision and, in some cases,
may result in a
complete reinvention of our personalities, for better or worse.
The capacity to change starts with self-awareness and the creation of a targeted
measurable goal. For
lasting change to occur a few things must happen. First, we must have a compelling
reason to change.
Desire alone, while critical, is insufficient to make change happen. Often the need
for change is suggested
or obvious (such as smoking cessation) but the person does not assess a pressing
need and thus,
compliance is temporary or sporadic. Second, we must understand how to make the
change. Not everyone
recognizes which optimal strategies support the desired outcome. Third, social
support for change
improves the likelihood of change effectiveness and stability. Unless these three
factors are intact, we may
quickly revert back to our bad habits, subsequently feeling disappointed and
frustrated about our failed
attempt.
My dilemma was a result of twenty years of late-night activities. When I started
graduate school, I worked
or attended class during the days and became accustomed to staying up late most
nights to study. It would
not be unusual for me to be pounding the books until after midnight. I needed a few
hours to unwind before
sleep; my typical lights-out time would be 2:00 a.m. Unfortunately, I was unable to
break the unhealthy
habit even though I finished school 16 years ago. The late bedtime did not mean I
would sleep all day, but
it clearly meant that I was not getting enough rest for optimal thinking and
productivity.
6. Identify the statement(s) that is/are correct as per the information given in
the passage.
(a) Desire is a necessary condition for change to occur.
(b) Capacity and ability are used interchangeably at many instances, although they
have different
meanings.
(c) The imminence of need dictates the nature of relevant compliance.
(d) Social support for change diminishes the likelihood of change effectiveness and
stability.

. Page 4 of 44
7. Mr. X joins a well-reputed gymnasium, to help him in his targeted and measurable
weight-loss journey.
However, even after spending considerable amount of time, he isn't able to achieve
the desired result for
himself. Based on the information given in the passage, what could be the most
likely reason for non#achievement of desired goals by Mr. X?
(a) He was often body-shamed by his friends and colleagues at workplace.
(b) The gymnasium didn't have the right tools, equipment and strategy pertaining to
the needs of Mr. X.
(c) He failed to acknowledge the condition of an abnormal BMI (Body-Mass Index)
with him.
(d) He wasn't facing any serious physical or mental issues because of his
disproportionate body-weight.
8. It can be inferred from the passage that
(a) The author used to sleep post-midnight when he/she was in graduate school till
2 am, which he has
been able to change even at present.
(b) Some people give in to the obstacles that come along with personal development.
(c) Social support is usually instrumental in enhancing the probability of change,
but change can also take
place without social support for the change.
(d) It takes considerable amount of time to break out of an unhealthy habit.
9. Which of the following is most likely to be the source of the given passage?
(a) Motivational novel (b) Scientific journal
(c) Newspaper article (d) Sociology textbook
10. What would be the underlying premise for the author's statement "We must devote
considerable effort to
the challenge"?
(a) People don't invest appropriate amount of effort to tackle a challenge.
(b) Effort and its optimum amount is indispensable in addressing a challenge to
achieve the desired result.
(c) The degree of effort required to handle a challenge varies in accordance with
the nature of challenge
at hand.
(d) Challenge can break people, for worse, if appropriate amount of effort is not
given to address the same.
Passage (Q.11-Q.15): The idea to use Navajo for secure communications came from
Philip Johnston, the
son of a missionary to the Navajos and one of the few non-Navajos who spoke their
language fluently.
Reared on the Navajo reservation, Johnston was a World War I veteran who knew of
the military’s search
for a code that would withstand all attempts to decipher it. He also knew that
Native American languages,
notably Choctaw, had been used in World War I to encode messages.
Johnston believed Navajo answered the military requirement for an undecipherable
code because it is
an unwritten language of extreme complexity. Its syntax and tonal qualities, not to
mention dialects, make
it unintelligible to anyone without extensive exposure and training. It has no
alphabet or symbols and is
spoken only on the Navajo lands of the American Southwest. One estimate indicates
that fewer than 30
non-Navajos, none of them Japanese, could understand the language at the outbreak
of World War II.
Early in 1942, Johnston met with Major General Clayton B. Vogel, the commanding
general of Amphibious
Corps, Pacific Fleet, and his staff to convince them of the Navajo language’s value
as code. Johnston
staged tests under simulated combat conditions, demonstrating that Navajos could
encode, transmit and
decode a three-line English message in 20 seconds. Machines of the time required 30
minutes to perform
the same job. Convinced, Vogel recommended to the Commandant of the Marine Corps
that the Marines
recruit 200 Navajos.
11. The author mentions the fact that Navajo “has no alphabet or symbols”, in order
to
(a) emphasize how difficult it is to decipher Navajo language.
(b) suggest a potential drawback of the use of Navajo to secure communications.
(c) explain why so few non-Navajos can speak the language.
(d) highlight the differences between Navajo and other Native American languages.

. Page 5 of 44
12. The passage is primarily concerned with-
(a) examining the complexity of a language for secure military communication.
(b) profiling someone's search for a solution to a problem.
(c) analysing the benefits and drawbacks of an approach.
(d) highlighting the achievement of an individual.
13. Which of the following suggests the tone of the passage?
(a) Factual (b) Diatribe. (c) Lionize. (d) Melancholy.
14. In which year did John meet Major General Clayton?
(a) 1988 (b) 1982 (c) 1942 (d) none of the above
15. Which of the following is the opposite meaning of the word ‘undecipherable’?
(a) Obscure (b) Fathomable (c) Cryptic. (d) Cordial
Passage (Q.16-Q.20): David had a suspicion. He did not know it was that, but that
is what it was. He
suspected that Mother thought he was a good little boy, and he suspected that she
thought Mitchell
Horrigan was a bad little boy. Perhaps Mother had a suspicion, too; she might have
suspected that it was
Mitch who had put a certain notion into David's head—a notion which had to do with
pants. Only you must
not call them pants; they are "trouvers."
But it doesn't really matter in the least what they are called. Mitch had them. He
also had the measles once.
David did not know whether it was the measles part or the pants part that made
Mitch a bad little boy. All
David knew about it was that if he invited Mitch into the yard to climb trees and
give swimming lessons in
the high grass, it usually happened that Mother could think of some important
business for her little boy to
do in the house. It was surprising how many important matters there were for David
to do in the house
every time Mitch came into the yard to play. She might want to show him something,
and perhaps it would
be a turn-over that she wanted to show him, a delicious little half-grown pie
stuffed with strawberries or
with cherries. If Mitch were waiting out under the trees, the toothsome bit of
pastry was always a very
peculiar kind. Mother believed in generosity, but generosity with limitations.
Strawberry turn-over was not
good for Mitch. Mother was positive that it was not good for him. That seemed a
little singular to David, for
he had never noticed anything wrong with Mitch. It does not seem credible that a
boy who owns a real
Indian bow 'n' arrow, which shoots so high he can knock the eye out of an angel
with it, should yet be so
foolish as to have a bad stomach. David had never seen any of one-eyed angels that
Mitch had
knocked down out of heaven with his Indian bow 'n' arrow. Mitch was not the kind to
show all of his
treasures. He didn't even show his bow 'n' arrow. He kept it hid, so that if the
police ever found out about it
they could not get it away from him. If they wanted to arrest him for having it,
that would be all right, but
they should not get hold of his Indian bow 'n' arrow.
16. It does not seem credible that a boy who owns a real Indian bow 'n' arrow,
which shoots so high he can
knock the eye out of an angel with it, should yet be so foolish as to have a bad
stomach.
Which of the following literary device has been used in the sentence?
(a) Anachronism (b) Paradox (c) Personification (d) Metaphor
17. Which of the following is true?
(a) Mitch’s bow n’ arrow was really important to him.
(b) David had measles because of Mitch.
(c) Mitch had no skill at shooting arrows.
(d) All of the above

. Page 6 of 44
18. According to the passage, which of the following options support the claim that
David’s mother did
not like Mitch?
(a) David’s mother always tried to keep him busy with household works when Mitch
came to play.
(b) David’s mother cared for Mitch’s health so she didn’t want him to have the
delicacies she made.
(c) David’s mother believed in generosity without limitations when it came to
Mitch.
(d) David’s mother appreciated Mitch’s influence on him regarding pants.
19. Strawberry turn-over was not good for Mitch. What did the author mean by the
phrase ‘Strawberry
turn-over’ as has been used in the context of the underlined sentence above?
(a) Strawberries must be added.
(b) Sweet strawberries in a buttery, flaky puff pastry.
(c) Strawberries should be replaced by some other berries.
(d) Too many strawberries
20. David had never seen any of one-eyed angels that Mitch had knocked down out of
heaven
with his Indian bow 'n' arrow.
Spot the error in the bold sentence above.
(a) error in article (b) error in adverb (c) error in verb (d) error in noun
Passage (Q.21-Q.25): Globalisation involves a stretching of social and economic
relationships throughout
the world. This stretching is pushed by certain economic policies. Very broadly
this process in India is
termed liberalisation. The term liberalisation refers to a range of policy
decisions that the Indian state took
since 1991 to open up the Indian economy to the world market. This marked a break
with an earlier stated
policy of the government to have a greater control over the economy. The state
after independence had
put in place a large number of laws that ensured that the Indian market and Indian
indigenous business
were protected from competition of the wider world. The underlying assumption of
such a policy was that
an erstwhile colonial country would be at a disadvantage in a free market
situation. The state also believed
that the market alone would not be able to look after all the welfare of the
people, particularly its
disadvantaged sections. It felt that the state had an important role to play for
the welfare of the people.
Liberalisation of the economy meant the steady removal of the rules that regulated
Indian trade and finance
regulations. These measures are also described as economic reforms. What are these
reforms? Since July
1991, the Indian economy has witnessed a series of reforms in all major sectors of
the economy
(agriculture, industry, trade, foreign investment and technology, public sector,
financial institutions etc). The
basic assumption was that greater integration into the global market would be
beneficial to Indian economy.
The process of liberalisation also involved the taking of loans from international
institutions such as tile
International Monetary Fund (IMF). These loans are given on certain conditions. The
government makes
commitments to pursue certain kind of economic measures that involve a policy of
structural adjustments.
These adjustments usually mean cuts in state expenditure on the social sector such
as health, education
and social security. There is also a greater say by international institutions such
as tile World Trade
Organisation (WTO).
Among the many economic factors driving globalisation, the role of transnational
corporations (TNCs) is
particularly important. TNCs are companies that produce goods or market services in
more than one
country. These may be relatively small firms with one or two factories outside the
country in which they are
based. They could also be gigantic international ones whose operations criss-cross
the globe.
21. Which of the following can be inferred from the given passage?
(a) India was not much benefitted by the laws which protected Indian market from
the competition from
outside world.
(b) After Independence India saw the outside world as a place for expansion of
Indian trade.
(c) Even long after independence, India was not able to heal itself from the
colonial wounds.
(d) Indian policy makers did not have much vision to recognize the importance of
liberalization.

. Page 7 of 44
22. Which of the following is the meaning of the word 'indigenous' as mentioned in
the given passage?
(a) of, from, in, or characteristic of a country or language other than one's own.
(b) Unfamiliar and disturbing.
(c) Operating or happening in the local area, not in other countries.
(d) Widely distributed.
23. With which of the following statements the author is least likely to agree?
(a) Importance of WTO is reduced in the world market.
(b) India progressed by leaps and bounds after liberalisation.
(c) After liberalisation taking loan from the IMF is mandatory.
(d) Transnational corporations are the brainchild of WTO.
24. The given passage involves beliefs which set the course of Indian economy after
independence and after
1991, they are
I. Indian market and indigenous business would be at disadvantage in free market
situation.
II. Indian market might fall into the trap of globalisation after the
liberalisation of the economy.
III. Greater incorporation into global market would be beneficial to Indian
economy.
(a) Both I and II (b) Both II and III (c) Both I and III (d) All I, II and III
25. Which of the following can be said about Transnational corporations?
(a) The size of a TNC depends on the size of factories it has across the globe.
(b) TNCs are responsible for producing goods and services for other countries and
exporting them.
(c) TNCs mostly focus on small and medium businesses in other countries.
(d) The TNCs do have a native country
Passage (Q.26-Q.30): I am losing my interest in human beings; in the significance
of their lives and their
actions. Someone has said it is better to study one man than ten books. I want
neither books nor men; they
make me suffer. Can one of them talk to me like the night – the Summer night? Like
the stars or the
caressing wind? The night came slowly, softly, as I lay out there under the maple
tree. It came creeping,
creeping stealthily out of the valley, thinking I did not notice. And the outlines
of trees and foliage nearby
blended in one black mass and the night came stealing out from them, too, and from
the east and west,
until the only light was in the sky, filtering through the maple leaves and a star
looking down through every
cranny.
The night is solemn and it means mystery.
Human shapes flitted by like intangible things. Some stole up like little mice to
peep at me. I did not mind.
My whole being was abandoned to the soothing and penetrating charm of the night.
The katydids began
their slumber song: they are at it yet. How wise they are. They do not chatter like
people. They tell me only:
-sleep, sleep, sleep. The wind rippled the maple leaves like little warm love
thrills. Why do fools cumber
the Earth! It was a man’s voice that broke the necromancer’s spell. A man came
today with his - Bible
Class. He is detestable with his red cheeks and bold eyes and coarse manner and
speech. What does he
know of Christ? Shall I ask a young fool who was born yesterday and will die
tomorrow to tell me things of
Christ? I would rather ask the stars: they have seen him.
26. Why has the author lost interest in human beings?
(a) Because they make the author suffer.
(b) Because the human beings do not bring with them the warmth and the comfort
which comes naturally
with the night, starts and the wind.
(c) Because human beings are not mysterious.
(d) All of the above.

. Page 8 of 44
27. The author has compared the night with:
(a) The Katydids (b) The Necromancer’s spell
(c) Stars (d) All of the above.
28. Why has the author called the katydids’ wise?
(a) Because they sing a slumber song.
(b) Because they tell the author to only sleep.
(c) Because they do not indulge in unnecessary and unimportant talks.
(d) Because they are not fools.
29. Which of the following can be inferred from the passage?
(a) Being close to nature can bring one closer to God.
(b) Nature is just a mystery.
(c) Books and Man are both detestable.
(d) None of these.
30. Which of the following words from the passage mean ‘rough’?
(a) Solemn (b) Caressing (c) Coarse (d) Slumber

Directions (Q.66 – Q.105): Read the comprehensions carefully and answer the
questions based on it.
Passage (Q.66-Q.70): The Uttar Pradesh Government recently criminalized the
conversion of religion for
the sake of marriage by passing the Uttar Pradesh Prohibition of Unlawful
Conversion of Religion
Ordinance, 2020 (“Ordinance”). As per K.S. Puttuswamy v. Union of India an invasion
of life or personal
liberty must meet the threefold requirement of legality, which postulates the
existence of law; need, in terms
of a legitimate State aim; and proportionality which ensures a rational nexus
between the objects and the
means adopted.”
Therefore, based on the above test, we argue that firstly, the Ordinance violates
the fundamental right to
privacy and right of the individuals to practice and profess a faith of their own
choice under Article 25.
Secondly, nothing establishes the State aim in curbing ‘conversion by marriage’ by
terming it unlawful
unless the converting person freely provides that such conversion is not upon their
volition. Thirdly, the
unconstitutional aim is also an encroachment into the personal lives and ensures
that any conversion of
religion undergoes a strict investigation.
Section 8 of the Ordinance requires an individual to submit a declaration of
conversion by free consent
sixty days in advance to the given authority. Section 3 of the Ordinance prohibits
any person to convert or
attempt to convert any other person, directly or otherwise, from one religion to
another by marriage.
However, since the Ordinance requires a declaration and a subsequent investigation,
the marriage had to
be deferred. Furthermore, Section 4 of the Ordinance authorizes any aggrieved
person, or relative to file
an FIR when one contravenes the abovementioned Section 3. In Shakti Vahini, the
Supreme Court held
that when two adults consensually choose each other as life partners, it is
protected under Articles 19 and
21 of the Constitution. It further held that the consent of family or community or
clan is not necessary when
two adults agree to enter into wedlock.
Saikishan B Rathore and Prathiksha Chandrasekhar, ‘Analysing the Unlawful Religious
Conversion
Ordinance, 2020 through the prism of proportionality’ (NLUJ Law Review, 25 March
2021)
66. Dheeraj and Zeenat eloped from their home town Kanpur to marry each other in
Allahabad. Zeenat decided
to convert her religion to Dheeraj’s in order to marry him out of her absolute
will. However, she failed to
make a declaration as required u/s 8 of the Ordinance. Zeenat’s parents objected to
the marriage and thus
registered an FIR against Dheeraj for forceful conversion of their daughter.
Dheeraj was tried by the Court
but later acquitted him and dismissed the case so as to not cause unnecessary
intervention with a couple’s
personal matter. Choose the correct comment on the ruling of the Court in the light
of the author’s views.
(a) The Court has given the incorrect order as if Zeenat failed to make a
declaration it raises a suspicion
against Dheeraj for forcing her to convert her religion upon marrying him.
(b) The Court has given the correct order as if Zeenat forgot or failed to make a
declaration regarding intent
to convert to another religion, and she shall herself be tried for the offence.
(c) The Court has given the incorrect order as under the Ordinance the parents are
allowed to register an
FIR when they have doubts regarding the compliance with section 8 and section 3 of
the Ordinance.
(d) The Court has given the correct order as no third party shall be allowed to
intervene or interfere when
two adults consensually choose to marry or one of them out of free will choose to
convert their religion
upon such marriage.

. Page 17 of 44
67. Which of the following correctly encapsulates the correct meaning of the line
“the unconstitutional aim is
also an encroachment into the personal lives”?
I. The Ordinance strictly scrutinizes every conversion of every individual to
ensure its legitimacy.
II. The Ordinance bars the conversion by any individual before prior approval of
the State Government.
III. The Ordinance gives right to the parents and other family members to raise
objections against
conversion or the marriage in the shroud of legitimate state aim to prevent
forceful conversion.
IV. The Ordinance discriminates between individuals on the basis of religion they
earlier practiced and the
religion they convert into.
Choose the correct option:
(a) I and III are correct
(b) I, II and III are correct
(c) I, III and IV are correct
(d) All are correct
68. Prasoon finds the religion that he currently practices to be discriminatory on
the basis of sex, caste and
class. Thus, he decides to convert into ‘equalism’ which propagates equality among
all the followers.
Prasoon wanted to keep the affair private and thus did not inform anyone including
his parents and friends.
However, when the authorities found out they repudiated the conversion on the
ground that he did not
make a prior declaration regarding the conversion. Prasoon has challenged the order
of repudiation before
the Court. Choose the correct option considering the points raised by the author of
the passage.
(a) The Court shall allow the conversion as the same is a private matter and any
restriction shall be an
infringement of one’s fundamental right to freedom of religion as well.
(b) The Court shall not allow the conversion as the same cannot be permitted if the
requisites under the
Ordinance have not been complied with and free consent is not duly investigated by
the authorities.
(c) The Court shall allow the conversion as Prasoon chose to convert to another
religion out of his free will
because the same propagates the principles of equality which he believes.
(d) The Court shall not allow the conversion as an individual is required to get
both legal and social sanction
without taking such a crucial decision of their lives.
69. Seema, a Hindu, decided to marry her boyfriend Asim, a Muslim. However, Asim
posed a condition that
Seema will have to convert to Islam to marry him. Seema was very reluctant but
eventually agreed as she
could find no way to convince Asim. On the day of their marriage, when the rituals
for conversion were
being performed Police arrived at the venue and arrested Asim u/s 3 of the
Ordinance upon the complaint
by Seema’s parents. Asim is being tried before the Court. Decide whether he shall
be found guilty for
violation of the Ordinance.
(a) Asim shall be found guilty u/s 3 of the Ordinance as posing conversion as a
condition to marriage
amounts to converting her religion upon marriage.
(b) Asim shall not be found guilty u/s 3 of the Ordinance as he did not force Seema
to convert her religion
but just posed it as a condition to marry her.
(c) Asim shall be found guilty u/s 3 of the Ordinance as Seema was reluctant to
convert into Islam and but
did so to marry Asim.
(d) Asim shall not be found guilty u/s 3 of the Ordinance as Seema has herself
agreed to convert to Islam
out of her free will.

. Page 18 of 44
70. When Anuj chose to convert to Christianity after marrying her girlfriend
Cynthia, he made a declaration and
duly informed the authorities regarding his declaration to do so by his own free
consent. However, the
authorities under the Ordinance made an investigation and ordered to withhold the
wedding until the free
consent of Anuj is duly established. Anuj moved to the High Court under article 226
against the order of
withholding the wedding. He contends that the order is in violation of his
fundamental rights. Which of the
following shall be the order of the Court in the context of the passage?
(a) The Court shall rule in favour of the administration as the Ordinance provides
for a strict scrutiny by the
means of an investigation to ensure the free consent of the individual intending to
convert their religion.
(b) The Court shall rule in favour of Anuj as the requirement of declaration and
investigation is already an
interference with his fundamental rights and an order of further delay of marriage
contravenes his
privacy as well.
(c) The Court shall rule in favour of the administration as the state aim of
curbing forced conversion
requires strict measures which are justified on account of such aim.
(d) The Court shall rule in favour of Anuj as he has complied with the requirements
under the section by
declaring his free will to convert to Christianity and thus he shall be allowed to
marry whenever he
wants.
Passage (Q.71-Q.75): Section 295A criminalizes any deliberate or malicious act
intended to offend the
religious sentiment of any class of people. The Section mandates imprisonment for 3
years and/or fine and
is a non-bailable offence. The Harm Principle was proposed by James Mill as the
only justification for
curbing the right to free speech, in other words, any interference with personal
liberty is justified only if it
prevents direct harm to another person. However, Mill qualifies this immunity of
free speech and expression
primarily through two restrictions – first, instigative speech and second, indecent
conduct in public.
Section 295A was designed to protect religious feelings, however, courts have
applied it, as a Harm
Principle, to ensure the maintenance of public order. For instance, in RamjiLalModi
v State of U.P, the
Court while expounding the constitutionality of the Section 295A held that: it only
punishes an aggravated
form of insult to any religion when it is done with a malicious intention to
outrage the religious feelings. The
purpose of ‘protecting religious sentiment’ was made secondary in favour of the
public order defense.
Recently in Shreya Singhal v UOI, it was held that only ‘incitement to violence’
can be restricted under the
reasonable restriction of public order. However, Section 295A being an ‘over-broad
law’ often encapsulates
within itself even the legitimate speech makes it difficult to interpret
‘intentional insult to religious sentiments’
as an equivalent to incitement to lawless action. ‘Outrage’, ‘disgust’ and ‘offend’
are quite vague and
subjective terms which often lead to over-criminalization. For instance, even a
speech containing true
statement could be banned if it outrages religious feelings, by assuming malicious
intention, as held in
Khalil Ahmad v State. However, restriction of speech on grounds of ‘outrage of
religious feelings’ without
public disorder is not constitutionally valid as is not the reasonable restrictions
as provided under Article
19.
KirtiMeena and SandliPawar, ‘Section 295A and Harm-Offence Debate’ (The Criminal
Law Blog NLUJ, 24
July 2021)

. Page 19 of 44
71. Sunderban, a popular OTT platform, launched a show called Rasleela. The show
was based on a political
drama in nation, where family and friends murder each other for gaining the highest
seat of political power
in the nation. This political game was termed as ‘Rasleela’ in the show. A sect of
Hindus were offended by
the name of the show, and contended that the word signifies an event of religious
importance. They further
contended that the show ridicules the Hindu gods and thus shall not only be banned
but its makers shall
be penalized for hurting religious sentiments of the sect. Decide on the guilt of
the makers of the show on
legal grounds given in the passage.
(a) The makers shall be held guilty as the show ridicules a religious event and
outrages the religious
feelings of an entire religion which is punishable u/s 295A.
(b) The makers shall not be held guilty as the show is a political drama and does
not insult a religion or its
beliefs, and has definitely not caused any public disorder or violence.
(c) The makers shall be held guilty as the intention becomes irrelevant as long as
an act or a statement
has outraged the religious feelings of a community.
(d) The makers shall not be held guilty as even if public order was disrupted the
same must be blamed on
the fragile religious beliefs of the communities rather than the artistic
expressions of the makers.
72. M.N. Beg is a renowned artist who showcases his perspectives towards diverse
social issues through his
paintings. His followers believe that the paintings actually propagate a deeper
meaning from what is prima
facie visible. In one such work he painted a naked lady, adorned with jewelries,
caressing her body. His
intention was to glorify the importance of female sexuality, more importantly by
the females themselves.
The work caused furor across vast sections of the society, who objected to the
painting for its alleged
obscenity. Decide whether the painting the work can be penalized under section 295A
or not.
(a) The work shall be penalized u/s 295A as it is not only outrageous to almost
every religion by
disrespecting the women but also obscene which is distinctly punishable under the
IPC.
(b) The work shall not be penalized u/s 295A as painting was a work of art intended
to give a social
message rather than insulting someone’s emotions, and moreover, the painting does
not insult the
religious feelings of anyone belonging to any religion at all.
(c) The work shall be penalized u/s 295A as it caused mass disruption of public
order even though it was
not intended to do have any such effect.
(d) The work shall not be penalized u/s 295A as even if outrageous to a religion,
any work of artistic
expression is protected under freedom of speech and expression.
73. In a show, one of the members dressed up and mimicked Baba Shyaam Ali, who
leads a cult with followers
in millions. The mimicry was intended to be comic but was perceived as denigrating
to Baba and the cult.
Multiple FIRs were registered across the country against the individual and he was
arrested for offence u/s
295A. The accused contended before the court, that the intention behind the mimicry
was not to offend a
person or a sect. The nature of the show itself is to make jokes about people and
cannot be perceived as
an insult to an entire religion. Decide the liability of the accused in the light
of Khalil Ahmad’s ruling.
(a) The accused cannot be found guilty as there was no intention to disrespect
anyone or outrage religious
feelings of any community, and in absence of such malicious intention the offence
was not committed.
(b) The accused shall be found guilty as the mimicry was disparaging to the Baba as
well his followers
outraging their religious feelings, and malicious intention is immaterial in such a
case.
(c) The accused shall not be found guilty as mimicking a Baba does not amount to
disparaging a religion
or religious beliefs of a community, and thus intention is immaterial because other
elements of the
section are not satisfied.
(d) The accused shall be found guilty as the mimicry caused vast public disorder
which is itself punishable
under the provision regardless of it being backed by a malicious intention or not.

. Page 20 of 44
74. Assertion (A): The confines of what is an offence u/s 295A has transformed over
the time.
Reasoning (R): The rulings of the Court now focus on the harm to the religious
feelings of community than
the earlier purpose of preventing disruption of public order.
Choose the correct option:
(a) Both A and R are true, R is the correct explanation of A
(b) Both A and R are true, R is not the correct explanation of A
(c) Both A and R are incorrect
(d) A is incorrect but R is correct
75. A popular standup comedian Munawar Shahrukhi was arrested and booked under
multiple sections
including s.295A for an act which caused riots and violence across the country. An
FIR was filed against
him that in an organized public show under the garb of stand-up comedy the accused
made, prima facie;
scurrilous, disparaging utterances, outraging religious feelings of a class of
citizens of India with deliberate
intendment. In the court this contention was made by the prosecution as well. The
accused contended that
the alleged act was intended to make fun of a song containing the name of a Hindu
God and not the God
or the religion itself. Choose whether Munawar shall be convicted for the act in
the light of reasoning given
by the court in Khalil Ahmad Case.
(a) Munawar shall be convicted for outraging religious feelings of a community as
his act could be
interpreted to be insulting to a religion even though it was not.
(b) Munawar shall not be convicted for outraging religious feelings of a community
as the act was not
intended to insult a religion or its deities, and thus cannot be punished.
(c) Munawar shall be convicted for outraging religious feelings of a community as
his act was found
offensive and also led to violence and riots across the nation.
(d) Munawar shall not be convicted for outraging religious feelings of a community
he just cracked a joke
and a community can take offence on any statement though not really offensive in
nature.
Passage (Q.76-Q.80): Merely because an arrest can be made because it is lawful does
not mandate that
arrest must be made, the Supreme Court recently observed. It also said that Section
170 of the CrPC does
not impose an obligation on the officer in charge to arrest each and every accused
at the time of filing of
the charge sheet. Further, it was also clarified by the Apex Court that,
“Personal liberty is an important aspect of our constitutional mandate. The
occasion to arrest an accused
during investigation arises when custodial investigation becomes necessary or it is
a heinous crime or
where there is a possibility of influencing the witnesses or accused may abscond.
Merely because an arrest
can be made because it is lawful does not mandate that the arrest must be made.”
Furthermore, the court added, “No arrest can be made because it is lawful for the
police officer to do so.
The existence of the power to arrest is one thing and the justification to exercise
it is another. The police
officer must be able to justify the arrest apart from his power to do so. Arrest
and detention can cause
incalculable harm to the reputation and self esteem of a person. No arrest can be
made in a routine manner
on a mere allegation of commission of an offence made against a person. A person is
not liable to arrest
merely on the suspicion of complicity in an offence. There must be some reasonable
justification in the
opinion of the officer effecting the arrest that such arrest is necessary and
justified. Except in heinous
offences, an arrest must be avoided if a police officer issue notice to person to
attend the station house
and not to leave the station without permission would do”
Extracted From, Live Law, August 19, 2021

. Page 21 of 44
76. Manu was discovered murdered at his home on Sunday morning. His brother, who
had returned from a
business trip to Mumbai, called the police. Sonu, Manus's brother, never liked
their next-door neighbor
Shivam. When the police arrived, everyone in the neighborhood, including Shivam,
gathered at Manus
House. Sonu accused Shivam of murdering his brother without any rationale or
emotion, claiming that
Shivam was always arguing about everything with Sonu and that in order to get
revenge, he murdered his
brother. Can the police now arrest Shivam for the same?
(a) The police can do so as Sonu has a reason behind his suspicion.
(b) No, he cannot be arrested as there is no justification for his arrest.
(c) Yes, he can be arrested as the police has the right to arrest anyone who is
under suspicion in a heinous
crime.
(d) Both (a) and (c).
77. In the preceding question, if the police, after investigating the matter,
discover any evidence against Shivam
that strengthens Sonu’s claims against him and arrests Shivam for the same. Can
Shivam now hold the
police accountable for arresting him needlessly and on the basis of mere suspicion,
causing irreparable
injury to his reputation?
(a) Yes, Shivam is right as the arrest was on the basis of mere suspicion.
(b) No, the police arrested Shivam as they had a right to do so.
(c) Yes, according to Shivam they exercised their powers wrongfully.
(d) No, as the police had evidence against Shivam.
78. Shanti is a well-to-do single mother of Arun who divorced her husband owing to
his mistreatment towards
herself and Arun. Shanti’s ex-husband, Karan, is an alcoholic and unemployed. As a
result, even after the
divorce, Shanti continued to get threats from him for money. One fine day, Arun did
not come home, which
worried Shanti that he had been kidnapped, and after 24 hours of his disappearance,
she filed a kidnapping
complaint with the nearby police station. She further claimed that Karan had
abducted her son in order to
satisfy his demands from her. Karan was later taken into custody by police. Is this
correct?
(a) No, as he may or may not be guilty which was not proven.
(b) Yes, as the arrest was not on a mere suspicion against Karan.
(c) No, as this will cause immense harm to Karan’s reputation.
(d) None of the above.
79. In the previous question, after much investigation, Karan found to be innocent,
and the kidnappers were
two of Arun's school punes. Karan now wishes to file a writ petition against Shanti
and the police misconduct
in the case, which has caused him so much harm to his image. Is he capable of doing
so?
(a) Yes, he is right to do so as it caused incalculable harm to his reputation.
(b) No, he cannot as he was not considered accountable on the basis of a mere
suspicion.
(c) Yes, as the police misused their power.
(d) Both (a) and (c).
80. According to the passage, why is there a distinction between the power to
arrest and the right to exercise
it?
(a) So as to maintain the respect of the innocent in society.
(b) So that no unnecessary harm is caused only on the basis of the arrest being
legal without justification.
(c) Both (a) and (b).
(d) None of the above.

. Page 22 of 44
Passage (Q.81-Q.85): The Himanta Biswa Sarma-led Assam Assembly yesterday passed
the Assam
Cattle Preservation Bill, 2021 amid protest from opposition members. The bill is
aimed at regulating
slaughter, consumption and transportation of cattle. The Cattle Preservation Bill
seeks to ensure that
permission for slaughtering cows is not granted in areas that are predominantly
inhabited by Hindu, Jain,
Sikh and other non-beef eating communities or places that fall within a five-
kilometre radius of a temple,
satra, and any other institution as may be prescribed by the authorities. However,
exemptions might be
granted for certain religious occasions.
The bill also seeks to put a check on the transportation of bovines without valid
documents. All offences
under this new legislation are cognizable and non-bailable. According to other
provisions of the Bill, the
slaughter of cattle will be prohibited unless a necessary certificate issued by the
registered veterinary officer
of a particular area has been obtained. The veterinary officer will issue a
certificate only if he believes that
the bovine, not being a cow, is over 14 years of age. A cow, heifer or calf may be
slaughtered only if it is
permanently incapacitated. Those found violating the rules shall be punishable with
imprisonment for a
term not less than three years and up to eight years or a fine that may vary
between Rs 3 lakh and Rs 5
lakh or both. If someone convicted is found guilty of the same or a related offence
the second time, the
punishment will be doubled.
SOURCE: Financial Express August 14, 2021.
81. XYZ slaughterhouse is a famous and well-reputed slaughterhouse in Assam known
for premium quality
Beef. The slaughterhouse only uses healthy, lactating cows so that beef quality is
not compromised and
also the slaughterhouse is constructed on the outskirts of the city to maintain the
religious sentiments of
the non-beef community. Due to new legislation in Assam, the authorities of XYZ
slaughterhouse need
permission to slaughter cattle with a valid certificate. Do you think the
veterinary officer will permit them?
(a) The XYZ slaughterhouse will get valid permission from the veterinary officer as
they will slaughter cattle
outside the city without harming any community religious sentiments.
(b) The XYZ slaughterhouse will not get valid permission from the veterinary
officer because regulating
slaughter consumption in the state of Assam is crucial.
(c) The XYZ slaughterhouse will get valid permission from the veterinary officer
because they will slaughter
only healthy cattle. Hence beef will be free from disease threats.
(d) The XYZ slaughterhouse will not get valid permission from the veterinary
officer because they are not
slaughtering permanently incapacitated cows.
82. Joana in Assam is a town that is populated by Hindus in minority and Muslims in
the majority. The Muslim
butchers are slaughtering cows in the town where Hindu shops and offices are also
established. This lead
to heated debate and turmoil between both the communities and the Hindu community
filed charges against
Muslim butchers under Assam Cattle Preservation Bill, 2021. The Butchers pleaded
that they have valid
permission of legal authorities with certificate. Decide can authorities permit
Muslim butchers considering
the facts of the situation.
(a) Yes, authorities can permit Muslim butchers for slaughtering cows in the town.
(b) No, authorities cannot permit Muslim butchers for slaughtering cows in the town
as the town is
populated by the Hindu community.
(c) No, authorities cannot permit Muslim butchers for slaughtering cows in the town
because Assam Cattle
Preservation Bill, 2021 prohibits the slaughter of cattle in Hindu majority areas.
(d) Yes, authorities can permit Muslim butchers for slaughtering cows in the market
as eating beef comes
under the Right to freedom of food.

. Page 23 of 44
83. On the occasion of Eid ul-Fitr the Magistrate of district Pali in Assam which
is predominately occupied by
Hindu and Jain communities allowed the Slaughter of cows in the District. The order
of the Magistrate
caused riots in the district between Hindu and Muslim communities. Is an order by
the magistrate to
slaughter a cow on occasion is valid?
(a) Yes Magistrate order is valid and he is a magistrate of the district therefore
can take any decision.
(b) No, the Magistrate order is not valid because Assam Cattle Preservation Bill,
2021 prohibits the
slaughter of cattle in Hindu and Jain majority areas.
(c) Due to exceptions of the Assam Cattle Preservation Bill, 2021 the magistrate
order is valid.
(d) Magistrate order is invalid because the slaughterhouse falls within a five-
kilometer radius of a temple.
84. Which of the following are not the Provision of Assam Cattle Preservation Bill,
2021?
(a) The new bill will designate any kind of slaughter of cattle as a cognizable and
non-bailable offense.
(b) The bill proposes slaughtering cows at designated places restricting any kind
of slaughter in areas
resided by non-beef –eating communities.
(c) The bill stated that Cattle can be slaughtered in Assam only with the valid
permission
(d) All of the above are the provision of the Assam Cattle Preservation Bill, 2021.
85. Mr Shyam from the Hindu religion is found slaughtering cattle without a valid
certificate in District Ujjain of
Madhya Pradesh. Will he be liable for offence Under the Assam Cattle Preservation
Bill, 2021, and what
will be the term of punishment for his offence?
(a) He will be liable under the Assam Cattle Preservation Bill, 2021.
(b) He will be liable under the Assam Cattle Preservation Bill, 2021 as
Slaughtering cattle without a valid
certificate is offence under the bill.
(c) He will not be held liable under the Assam Cattle Preservation Bill, 2021.
(d) He will be liable under the Assam Cattle Preservation Bill, 2021 as
Slaughtering cattle without a valid
certificate is offence under the bill and Mr Shyam will be punishable with
imprisonment for a term not
less than three years and up to eight years.
Passage (Q.86-Q.90): In 2017, the Supreme Court, in Independent Thought v. Union of
India, refused to
delve into the question of marital rape of adult women while examining an exception
to Section 375 (rape)
of the Indian Penal Code (IPC) which allows a man to force sex on his wife.
Section 375 of the IPC defines the offence of rape. It lays down which physical
acts are required to make
out the offence, and it is a very broad definition. The second important element of
this definition is consent.
Where these acts are done without the consent of the woman, then the offence of
rape is made out. This
is the general rule, but there is an exception, which says that sexual acts by a
husband with his wife, if she
is 18 years of age and above, would not be rape. While the rest of the provision is
centred on consent, this
exception does not talk about consent at all. Marital rape may be recognised as a
form of cruelty, it may
be a ground for divorce, but it is not punished as rape, which is a very distinct
wrong and has very distinct
terms.
The most compelling argument in support of the marital rape exception as it is
prevalent in our statute
today is that the institution of marriage is sacrosanct and that it should not be
disturbed. But the Constitution
places no importance on any particular institution; the Constitution is unequivocal
in the significance given
to the individual. A person deserves equality, equal protection and autonomy.
Therefore, the exception to
marital rape in itself is unconstitutional and violative of Article 14. Further,
marital rape is an affront to the
dignity of the individual, which is protected under Article 21. Just because the
nature of the relationship
between the victim and the offender is one of marriage, it does not absolve a
person of the crime. The
argument that is built around the institution of marriage and its sanctity needs to
be broken, which is what
has happened in other jurisdictions around us. Once we realise that, our society
will have no choice but to
remove the marital rape exception.
SOURCE: The Hindu Sep 09, 2021 “ Why hasn’t marital rape been criminalised in India
yet?”

. Page 24 of 44
86. Zoya was married to Zakir at the age of 16 years and without Zoya's consent,
her husband is forcefully
making sexual intercourse with her and causing bodily injuries to her. If you are
an advocate and Zoya
seeks your advice on a particular matter. What will be your legal advice
considering the above passage?
(a) A girl child cannot be treated as a commodity and her husband has no right to
intercourse without her
consent.
(b) If a girl is above 15 years of age and married, her husband has the right to
make intercourse with her
without her will.
(c) Zakir will be held liable for making sexual intercourse with her without her
consent.
(d) Muslim Personal law allows legal marriage at the age of 15. Hence Zakir will
not be held liable.
87. A and B are 30 years old married couples. Mr A is used to make forceful sexual
demands with her wife B
without her consent daily. Mrs B is crying inconsolably due to physical and mental
pain and went to court
to charge her husband for the offence of Rape. What remedy is available with Mrs B
and will her husband
be punishable with offence of rape?
(a) Her husband will not be punishable with the offence of rape because Marital
rape is legal in India.
(b) Her Husband will not be liable for any offence because sexual acts by a husband
with his wife if she is
18 years of age and above, would not be rape.
(c) Her Husband will not be liable for any offence because sexual acts by a husband
with his wife are right
of every husband.
(d) Her Husband will not be liable for any offence but marital rape can be a ground
for divorce so Mrs B
can have a divorce.
88. In a party an intoxicated man forcefully made sexual intercourse with a married
woman without her consent.
Even after many pleading and requested he continued to molest and raped her. Decide
will man be liable
for offence of Rape.
(a) Man was intoxicated therefore men's rea is absent in the above situation. He
will not be liable for the
offence of rape.
(b) Man will be liable for the offence of the rape because consent was given by the
woman.
(c) Man will be liable for the offence of rape because he committed sexual acts
without her consent.
(d) Man will not be liable for the offence of the rape because marital rape is
legal in India.
89. What arguments author put forth to bring marital rape under the ambit of
Section 375 of the IPC that defines
the offence of rape?
(a) Marital rape is against the principles of Article 14 and it also violates the
right to dignity of an individual
given in Article 21.
(b) Marriage is sacrosanct and marital rape violates this sacrosanct nature of
marriage.
(c) Women might misuse the law in their favour if marital rape became offence in
India.
(d) To deal with offence of marital rape the personal religion code should play its
role.
90. What could be possible reasons for marital rape legality in India according to
the author?
(a) Marital rape cannot work in India due to social-economic differences in the
country.
(b) Our country is sustaining itself because of the family platform which is upheld
by the marriage.
(c) Because a Good Indian wife must consent to her husband forever.
(d) Once married, women's perpetual consent is implied.

. Page 25 of 44
Passage (Q.91-Q.95): Section 320 of the CrPC provides for compounding or compromise
of two categories
of offences which can be compounded without the permission of the court and where
permission of the
court is required for compromise. Rape does not find mention under any of the
category, for which high
courts use their own power under section 482 of CrPC. Recently a Supreme Court
bench asked a rape
accused to marry her victim. In Saju PR v. State of Kerala, a Supreme Court bench
quashed a “rape
lawsuit” on the basis of a settlement between the accused and the survivor, for
“doing full justice to the
parties concerned.” In Dalbir Singh v. State of Punjab, the charges for rape were
quashed by the High
Court because the case had been compromised and the victim was married to one of
the accused.
In Shimbhu v. State of Haryana it was held that rape is a non compoundable offence.
It is a matter not just
against the victim but against the whole society. The crime of rape should not be
quashed on the basis of
compromise, considering the fact that the consent of victim in this situation is
not absolutely free, but
affected by trauma or helplessness. The apex court observed in the case of Gian
Singh v. State of Punjab
held that compounding would not apply where the nature of offence is very serious
like rape, murder,
robbery, dacoity etc.
Any settlement in rape cases, compromises the woman’s honor. The idea of compromise
should not be
considered, because these are crimes against the body of a woman, which is her own
temple. One cannot
escape prosecution simply because he shows remorse for his crime. This would permit
all those who plead
guilty in court and accept their acts to be left free from implications.
Harshita Dixit, 'Marry your rapist: An unending saga of compromise in Rape Cases'
(The Criminal Law Blog
NLUJ, 23 March 2021)
<https://criminallawstudiesnluj.wordpress.com/2021/03/23/marry-your-rapist-
an#unending-saga-of-compromise-in-rape-cases/> as accessed on 26 September 2021.
91. Shrishti was forced into a sexual activity by her boyfriend, Puneet, when her
mother realized the same, she
filed an FIR against him. However, Shrishti was reluctant to send her boyfriend to
jail and so she married
Puneet. Later, she made an application in the court that both of them have
compromised and there is no
further need for trying Puneet for the offense of rape. District court allowed the
application and dismissed
the case against Puneet. Shrishti’s mother has filed an appeal before the High
Court against the order of
the District Court. Decide as per the passage:
(a) The appeal shall be allowed because as a rape victim Shrishti has suffered
severe trauma and is, thus,
unable to make right decisions for herself.
(b) The appeal shall not be allowed as the precedents suggest that rape can be
compounded upon
marrying the victim of the act.
(c) The appeal shall be allowed as rape cannot be compounded even with the consent
and absolute will
of the victim.
(d) The appeal shall not be allowed as Shrishti has already married Puneet and if
he is convicted and sent
to jail, her life will be further exacerbated.
92. Assertion (A): Marriage does not do full justice to the victim of a rape and
thus should not be compounded.
Reasoning (R): The victim has to, at multiple times, consent due to their
helplessness and compromise
their honour by compromising with the accused.
Choose the correct option:
(a) Both A and R are true, R is the correct explanation of A
(b) Both A and R are true, R is not the correct explanation of A
(c) Both A and R are incorrect
(d) A is incorrect but R is correct

. Page 26 of 44
93. In a village, Hirsa, in Haryana a minor was gang raped by some teenagers. The
matter, as per the prevalent
practise, went before the village panchayat instead of a Court. Panchayat ruled
that if one of the accused
did not come forward to marry the victim, all of them would be hanged. One of the
accused volunteered to
marry the victim and thus all the accused were acquitted. The minor victim, in
paucity of any reasonable
alternative, agreed to the marriage. When a social organization reached out to the
victim and sensitized
her on her rights and penal provisions, the victim chose to file a complaint
against her husband. In the trial
the accused contended that the parties have already compromised and the victim had
given an absolute
consent to the marriage between them. Decide:
(a) The court shall dismiss the case as the parties have already reached a
compromise and in the light of
the precedents where the accused has already married the victim, they shall not be
convicted of the
offence.
(b) The court shall convict the accused and repudiate the compromise between the
parties as the consent
given by the accused was a result of her helplessness and rape cannot be
comppounded upon a
compromise between the parties.
(c) The court shall dismiss the case as no case can be filed for an offense after a
delay of such a long
time, even the courts are bound to not condone the delay by hearing the matter.
(d) The court shall convict the accused and repudiate the claim as the compromise
was suggested and
effected by the Panchayat by assuming the authority of a Court, which is
impermissible under the law.
94. Justin and Hannah have been a couple since they were in school. Hannah always
wanted to marry Justin,
who had also promised to marry her. On their vacation, Justin forced Hannah into a
sexual intercourse
despite her evident dissent to the same. After they returned, Hannah registered an
FIR against Justin for
committing her rape. Justin was arrested and was produced before the court for the
trial. Justin contended
that both of them were soon to marry each other and he is still willing to keep his
promise. The court
accepted the offer by Justin, and ordered him to marry Hannah as soon as possible.
Further, to ensure that
he keeps his promise, the Court stated that an order of acquittal will be made only
after the marriage.
Hannah has appealed the order before the High Court. Choose the correct answer.
(a) The appeal shall be allowed as there is no surety whether Justin would marry
him or abandon her and
abscond if the court gave him a chance.
(b) The appeal shall not be allowed as the couple was already about to marry, and
the forced intercourse,
therefore, will not be a crime if Justin still wants to marry Hannah and save her
from the social humility.
(c) The appeal shall be allowed as an offer to marry Hannah is not a defence to
justify forced sexual
intercourse, further the appeal filed by Hannah suggests that she does not consent
to either the
marriage or the acquittal of Justin.
(d) The appeal shall not be allowed as the Court has already employed measures to
ensure that Justin
does not abandon Hannah after a promise of marriage.
95. Hussein has been serving sentence of 7 years for the offence of rape. As a
result of the incident, the victim
suffered severe shock and has been in a vegitative state since then. Hussein had
already served a
sentence of more than 5 years, when he made an application before the Court for his
release, as he repents
for his actions and wants to serve and take care of the victim. Realizing that the
accused had realized his
crime and he had proposed to take care of the victim, the court ordered his
release. Now decide whether
the order for the release was justified or not in the light of author’s arguments.
(a) The order was unjustified as the severity of the offence does not reduce merely
because the accused
repents for his actions and proposes to take care of the victim.
(b) The order was justified as the accused did not propose to marry the victim
which has been looked down
on by the author of the passage.
(c) The order was unjustified as the sentence has already been given and a part of
it is served as well thus
at such a later stage, no compromise can be made.
(d) The order was justified as the accused is remorseful for his actions and sought
a chance to serve the
disabled victim by taking care of her.

. Page 27 of 44
Passage (Q.96-Q.100): Section 309, IPC which criminalizes attempt to suicide.
Section 115 of Mental
Healthcare Act (MHA) impacts Section 309 IPC. Section 115(1) reads as,
“notwithstanding anything
contained in section 309 of the Indian Penal Code any person who attempts to commit
suicide shall be
presumed, unless proved otherwise, to have severe stress and shall not be tried and
punished under the
said Code.” The matter reached the Supreme Court in P. Rathinam v. UOI. A two-judge
bench declared
Section 309 to be unconstitutional and void. Later, it was overruled by a
Constitution bench of the Supreme
Court in Gian Kaur v. The State of Punjab. The court held that ‘right to die’ is
not included in ‘right to life‘
under Article 21.
Justice DY Chandrachud held in Common Cause v. UOI. It mandates (unless the
contrary is proved by the
prosecution) that a person who attempts to commit suicide is suffering from severe
stress. Such a person
shall not be tried and punished under the Penal Code. Section 115 removes the
element of culpability
which attaches to an attempt to commit suicide under Section 309. It regards a
person who attempts suicide
as a victim of circumstances and not an offender, at least in the absence of proof
to the contrary, the burden
of which must lie on the prosecution.
Justice Nariman in the Navtej Singh Johar judgment held the Government has an
affirmative duty to provide
care, treatment and rehabilitation to such a person with mental stress to reduce
the risk of recurrence of
that person’s attempt to commit suicide. This parliamentary declaration under
Section 115 again is in
keeping with the present constitutional values, making it clear that humane
measures are to be taken by
the Government in respect of a person who attempts to commit suicide instead of
prosecuting him for the
offence of attempt to commit suicide.”
Akanksha, 'The Supreme Court and the Law Commission on Section 309 IPC' (CCJA, 14
September 2020)
96. Karuna, a rape survivor, has been admitted in a hospital for 23 years now.
Right from the incident of rape,
she has been in a vegitative state. Since, she was abandoned by her family, the
hospital management and
staff looking after her filed a Special Leave Petition before the Supreme Court to
permit Karuna to take her
life as she has already been in severe mental stress and physical agony due to
trauma suffered by her.
The hospital staff contended that Karuna is entitled to a death with dignity under
article 21. After listening
to the arguments of the parties, the court has reserved the matter for judgment.
Decide:
(a) The court shall allow the petition as severe mental stress as being suffered by
Karuna is a valid defence
to suicide under section 115 of the Mental Healthcare Act.
(b) The court shall not allow the petition as law in its present state does not
make any effective provisions
for permitting death to a person.
(c) The court shall allow the petition as Karuna’s body is in vegitative state for
decades and her life is
useless, thus prolonging her wait for a dignified death will make her life even
more miserable.
(d) The petition shall not be allowed as no life until it ends by itself shall be
found worthy of death by the
law.
97. Assertion (A): Regardless of the cause of death, the section 115 of the Mental
Healthcare Act provides a
blanket protection to every suicide survivor.
Reasoning (R): Where the prosecution is successful in proving that the mental
stress was not the actual
cause of the suicide, the protection under section 115 can be breached.
Choose the correct option:
(a) Both A and R are true, R is the correct explanation of A
(b) Both A and R are true, R is not the correct explanation of A
(c) A is correct and R are incorrect
(d) A is incorrect but R is correct

. Page 28 of 44
98. Upon reliable information, Police raided a residential complex. When they
reached the location, they found
a person lying on the floor along with all the raw materials for making explosives
in the house. The person
was admitted to the hospital where the doctors discovered that he had consumed
poison. When he gained
consciousness, Police charged him under various sections including section 309 of
the IPC for attempting
rape. During the trial, the prosecution proved that the accused was involved in
terrorist activities and had
consumed poison to commit suicide to escape the liability. The accused contended
that he be granted
protection under section 115 of the MHA, and charged only under other sections and
not 309. Choose the
correct option.
(a) The accused shall be found guilty as the requested defence under section 115
cannot be granted
unless the accused proves the element of mental stress beyond doubt.
(b) The accused shall not be found guilty as he has taken the defence of severe
mental stress granted
under section 115 of the MHA.
(c) The accused shall be found guilty as the prosecution had proven that the actual
cause was to escape
the criminal liability and thus the defence of mental stress cannot be taken.
(d) The accused shall not be found guilty as it is the government’s responsibility
to take humane measures
against the suicide survivours and not charge them for the offence.
99. Saumya had eloped with Asim from their hometown. After being on the run for
months, and being
apprehensive of Saumya’s father they decided to commit suicide. After their
attempt, Saumya had died,
however, Asim survived. Saumya’s father registered an FIR under section 309.
Further, in the court he
contended that Asim committed suicide because he had kidnapped Saumya and then
killed her, thus,
suicide was the only alternative he had. Further, Saumya’s father contended that
due to the above stated
reasons Asim shall not be afforded the protections under section 15 of the MHA.
(a) Asim shall not be convicted of the offence as he committed the suicide under
severe fear and
apprehension of danger from Saumya’s father which is an evidence of mental stress.
(b) Asim shall be convicted of the offence as it has been proven that the primary
cause of the suicide was
to escape criminal liability and not because of any form of mental stress.
(c) Asim shall not be convicted of the offence as Saumya had committed the suicide
and was not murdered
by Asim, therefore he could not be held liable for murder.
(d) Asim shall be convicted of the offence as the defence afforded under section
115 of the MHA can be
used in exceptional circumstances only where the mental disability of the
individual can be medically
proven.
100. Hannah committed suicide, and her two classmates Justin and Bryce have been
charged for the abetment
of the offence. They had constantly bullied Hannah which compelled her to choose
such an option. During
the trial, the court found them both guilty of abetment, however, their lawyer took
the defence of mental
stress under section 115. The lawyer contended that if the defence is granted for
the offence of suicide it
should definitely extend to abetment which is less grave version of the same.
Choose the correct judgment
of the following options.
(a) Both of them shall not be granted the defence of mental stress as the
protection is afforded to the
person who has actually attempted the suicide and not the abettoes who were the
cause of committing
the suicide.
(b) Both of them shall be granted the defence of mental stress as abetment of
suicide is the less grave
version of the offence of suicide and reasonably the defence under section 115
shall extend to abetment
as well.
(c) Both of them shall not be granted the defence of mental stress as they have
already been found guilty
of abetting to the offence and the defence must have been taken at an interim
stage.
(d) Both of them shall be granted the defence of mental stress if they manage to
furnish some medical
evidence suggesting that they are suffering from severe mental stress.

. Page 29 of 44
Passage (Q.101-Q.105): The Punjab & Haryana High Court has held that an
anticipatory bail application
filed under Section 438 of CrPC by a juvenile is not maintainable; that bail is
granted to a juvenile in a
bailable or non-bailable offence, notwithstanding anything contained in CrPC.
The provisions of Section 438 Cr.P.C. are enumerated for granting the bail to the
person who has
apprehension of the arrest. A reading of Section 438 Cr.P.C.’s provisions vis-a-vis
relevant provisions of
the Act would show that a juvenile cannot be arrested, and thus, there is no
question of apprehension of
his arrest. Hence, the petition under Section 438 Cr.P.C. is not maintainable in
case of a juvenile.
The Court further noted that Juvenile Justice Act, 2015 is a complete Code in
itself and section 12 thereof
pertains to bail of a child alleged to be in conflict with the law and mandates
that as soon as the police
apprehend a child, he shall be produced before the JJ Board, without losing any
time.
The provisions of Section 12 would show that when any child in conflict with law is
brought before a Board
then such person notwithstanding anything contained in the Cr.P.C. or in any other
law for the time being
in force, be released on bail with or without surety, the Court noted while
reaffirming that a juvenile cannot
be arrested.
Notably, the proviso to Section 12 mandates that if it appears to the Board that
there are reasonable
grounds for believing that the release of the child in conflict with the law, is
likely to bring that person in
association with any known criminal or expose the said person to moral, physical or
psychological danger
or the person’s release would defeat the ends of justice, then the Board shall
record the reasons for denying
the bail.
Shrutika Pandey, ‘Anticipatory Bail Plea Of Juvenile U/S 438 CrPC Not Maintainable:
Punjab & Haryana
High Court’ (Live Law, 8 July 2021)
101. Raju was a poor ragpicker who was adopted by Mahesh when he was of 8 years.
Hews living with Mahesh’s
family for almost 7 years and had become an integral part of the family. He knew
all the places where the
family jewellery and money was stored. The family also provided him food and
education facilities. One
day when no one was home, there was a burglary happened and all the money and
jewellery was stolen.
Mahesh’s brother filed a complaint in the police station alleging that Raju has
stolen all the jewellery as he
was the only non-family member who knew about the places. Raju in order to save
himself filed for an
anticipatory bail which was later rejected by the district court without recording
any reason in writing for
denying the bail. Decide whether the decision of the district court correct or not?
A. The decision of the district court is correct as Raju was a juvenile and any
application for anticipatory
bail filed by him or on his behalf will not be maintainable.
B. The decision of the district court is correct as a juvenile cannot be granted
anticipatory bail in any
condition as they are required to be kept in judicial custody for their safety.
C. The decision of the district court is not correct as bail is granted to a
juvenile in a bailable or non-bailable
offence, notwithstanding anything contained in CrPC.
D. The decision of the district court is correct as Section 438 of CrPC is not
applicable on a juvenile despite
the offence being bailable.
Choose the correct options.
(a) Only A is correct.
(b) Only C is correct.
(c) Both A & D are correct.
(d) A, B and D are correct.

. Page 30 of 44
102. Seema was in 6th standard when she met Rohan, who was in 11th standard, in the
same school. They soon
starting talking but after a while she noticed some strange behaviour on the part
of Rohan and so she
decided to stop meeting or talking to him. At first, Rohan tried talking to her in
the school but when Seema
complaint about it to the principal, he started stalking her and following her back
to the home. One such
day, he stopped Seema and tried to kiss her and in retaliation she picked up a
stone lying aside and hit
Rohan’s head with it. He suffered serious injuries and later died in the hospital.
Seema was apprehended
by the police and the Juvenile Justice Board denied her bail application on the
ground that the offence is a
non-bailable offence. Decide:
(a) The decision of the Juvenile Justice Board is right as any accused cannot be
granted bail in a non#bailable offence as per the provisions contained in CrPC.
(b) The decision of the Juvenile Justice Board is right as Seema was a juvenile and
all the decisions with
regards to any offence committed by a juvenile lies in the hands of the magistrate
and not the Board.
(c) The decision of the Juvenile Justice Board is wrong as bail is granted to a
juvenile even in a non#bailable offence, notwithstanding anything contained in CrPC
or any other law.
(d) The decision of the Juvenile Justice Board is wrong as the offence committed by
Seema was an
accident and that too while she was acting in self-defence, which is an exception
to the non-bailable
offence of murder.
103. With the above context, when Seema was apprehended by the police for the
offence of murder of Rohan,
what should be next step that the police should make as per the provisions of the
Juvenile Justice Act,
2015?
(a) The police shall produce Seema before the Juvenile Justice Board as soon as she
was apprehended
by them for the murder of Rohan.
(b) The police shall produce Seema before the judicial magistrate and the
magistrate if deems necessary
shall send her to the judicial custody for the reasons recorded in writing.
(c) The police shall keep Seema in their custody for 24 hours in order to record
her statement and then
produce her before the JJ Board within 36 hours of her arrest.
(d) The police shall produce her before the nearest judicial magistrate within 24
hours of her apprehension
and then of the magistrate allows, she shall be released on bail.
104. Mia was an orphan girl and used to live in an orphanage called ‘Bettie’s
Home’. Ms. Bettie was the owner
of the orphanage and also the care taker. On Mia’s 14th birthday, Ms. Bettie called
her into her cabin and
told her that she will now have to work for her business in order to have a bed and
food in the orphanage.
She asked her to deliver a bottle of whiskey to a Mr. Robbins house and then assist
him with whatever he
asks her to do. When Mia went to Mr. Robbins house to deliver the bottle, he asked
her to go to his bedroom
and change her dress and wait. Mia got apprehensive of the work she was asked to do
by Ms. Bettie and
so when Mr. Robbins entered the room, she took a vase and hit it in her head. Mr.
Robbins died after a lot
of bleeding.
Later Mia was arrested by the police and brought before the JJ Board where the
board denied her bail
when Ms. Bettie refused to pay any surety on behalf of Mia. Do you think the
board’s decision was wrong?
(a) The Board’s decision was not wrong as Mia committed murder which is a non-
bailable offence and bail
cannot be granted in such an offence without any surety as per section 437 of CrPC.
(b) The Board’s decision was not wrong because as per section 438 of CrPC, a bail
application filed by a
juvenile is not maintainable in a bailable or non-bailable offence, notwithstanding
anything contained in
Juvenile Justice Act.
(c) The Board’s decision was wrong because Mia shall be released on bail with or
without surety as a
juvenile cannot be arrested and bail is granted to such child in a bailable or non-
bailable offence,
notwithstanding anything contained in CrPC.
(d) The Board’s decision was wrong because Mia did not commit murder but culpable
homicide not
amounting to murder which is a bailable offence and so Mia should be granted bail
even without any
surety as per section 437 of CrPC.

. Page 31 of 44
105. When Mia explained before the Board that in what circumstances she killed Mr.
Robbins, the board had an
apprehension about the kind of work Ms. Bettie was forcing the children of the
orphanage to do. They sent
a complaint to the nearest judicial magistrate and set up an enquiry against Ms.
Bettie for indulging the
orphan kids in forced prostitution. In furtherance of this, the board also denied
Mia’s bail application
recording the reasons that under the apprehension that her release is likely to put
her in physical or
psychological danger or it might also defeat the ends of justice as Ms. Bettie will
try everything to change
Mia’s statement or influence her to speak on her side. Whether the board’s decision
of denying bail to Mia
is correct or not in this situation?
(a) The board’s decision is correct because as per the proviso to section 12, the
Board can deny bail to a
juvenile after recording the reasons, if the release of juvenile is likely to put
them in any kind of danger.
(b) The board’s decision is correct because considering the fact that Mia is an
orphan and she has no
place to live or no family to protect her, the burden ultimately lies on the board
to give her protection
until she is proved guilty or innocent.
(c) The board’s decision is not correct because a juvenile shall be released on
bail in any condition and a
juvenile cannot be arrested, notwithstanding anything contained in any other law
for the time being in
force.
(d) The board’s decision is not correct as the board can only deny bail to a
juvenile when the release of
such a child is likely to bring them in association with any known criminal or
would defeat the ends of
justice, which was not an apprehension in the given situation.

. Page 32 of 44
SECTION - D: LOGICAL REASONING
Directions (Q.106-Q.135): Study the following information and answer the questions
that follow:
Passage (Q.106-Q.110): Nobody can under-estimate the educational value of
travelling. Travelling is a
better teacher than books. It confirms one’s bookish and theoretical knowledge. It
helps us to learn things
in an easier, more lasting, and more lucid manner. Think of a student who has seen
the Taj. Is he not much
better placed to write about the Taj than any of his class-fellows who has never
been to Agra? Travelling
gives first-hand information about things places, persons, and events. It provides
an opportunity for
acquiring a practical knowledge of men and matters and enables a person to fight
the battle of life in a
more successful way. Travelling is essential for the healthy growth of the mind.
Tagore writes: “The health
of the mind cannot be maintained on the ration of books served up in motionless
classes within the prison
walls of a static school.” Traveling converts a student into an awakened scholar in
the open university of
the world. He can have a close look at life, grapple with its varied problems, and
have first-hand knowledge
of the affairs of the world. Knowledge obtained through books is incomplete,
imperfect, and short-lived.
Knowledge obtained through traveling is perfect and permanent. One who has not
travelled remains like a
frog in a well. He is self-centered. He thinks that his view-point is the only
correct viewpoint. He cannot
appreciate the viewpoint of others. He is a prisoner of his set ideas. A widely
travelled person, on the other
hand, comes in contact with numerous persons. Thus, he gets a new vision of life.
His outlook becomes
broad; his sympathies widen. He comes to realize that the view-point of others may
also be correct. During
educational tours, the students come across different people with different customs
and different habits.
This is such a knowledge as cannot be attained otherwise or through books. A
student of science who is
told about atomic reactions outside the laboratory will not learn as much as a
student who is taken to the
laboratory and shown each and everything in practical condition. The latter will
not nurse any doubts about
it because the impression which is formed in his brain’s sensitive plate through
real observation is
permanent and perfect. Tours have a great educational value. Students can learn
much through these
tours. They will not have to bother about books and their knowledge will be more
adequate and more
lasting. Tours play a great role in the general growth of a student’s mind and
heart.
106. What can be inferred from the given passage?
(a) Travelling is considered to be a much better teacher than books because it is
able to strengthen
hypothetical
knowledge and helps us in learning things in an unchallenging and obscure manner.
(b) It can be inferred that knowledge gained from books is deficient, impeccable
and momentary. But
knowledge
acquired through travelling is flawless and insightful.
(c) Travelling is a firsthand knowledge of the world and the people in it. It
refreshes our perspective towards
others; enhances our understanding of the world and its problems; evolves our mind
and mental state;
sensitizes us to the world at large and brings in acceptance of cultural
differences, unlike bookish
knowledge.
(d) A student will be able to understand concepts and subjects better when he is
able to meet people and
comprehend their distinct values and habits. However, knowledge can be inculcated
better when a
student practically goes through the subject.
107. According to the passage, what is the main difference between books and
travelling?
(a) Both books and travelling help people in gaining knowledge. However, knowledge
acquired by travelling
adds wider dimensions to learning and is more fluid than the parochial and static
knowledge gained by
books.
(b) Books procure knowledge that is temporary and flawless, while travelling
reflects permanent and
vitiated knowledge.
(c) Books divest knowledge; on the other hand, travelling help embrace knowledge.
After all, travelling
allows us to meet new people and gain experiences.
(d) Both books and travelling help us in gaining knowledge. After all, both hold
the same importance and
value in a student’s life.
. Page 33 of 44
108. Which of the following addresses the central flaw in the passage?
(a) The author demeans the knowledge gained by books.
(b) The author takes only two sources for gaining knowledge.
(c) The author assumes everyone to be well travelled.
(d) The author discounts the disadvantages of travelling.
109. Why do educational tours have a greater value in a student’s life?
(a) Knowledge gained through tours is far more perpetual and immaculate than the
knowledge acquired
through books.
(b) Tours play a significant part in forsaking a student’s intellectual as well as
emotional development.
(c) Educational tours are the only way to experience true learning.
(d) None of the above
110. Tagore writes: “The health of the mind cannot be maintained on the ration of
books served up in motionless
classes within the prison walls of a static school.” The statement in the passage
plays a role of
(a) Neutralising the contention made in the passage.
(b) Provides extra insight into the passage.
(c) Substantiates author’s point of view.
(d) Negates the preceding sentence.
Passage (Q.111-Q.114): In A New History of Western Philosophy (2004-7), the Oxford
philosopher
Anthony Kenny proposes that philosophy really begins with Aristotle (384-322 BCE)
because Aristotle was
the first philosopher to systematically summarize the teachings of his predecessors
in order to criticize
them. It's an approach that Aristotle's great teacher, Plato, didn't adopt.
We can gain a sense of the radical nature of Aristotle's move if we consider some
of the words he creates
in order to make it. For example, Plato had the word 'analogy' but not the word
'analysis'. The word 'analysis'
was invented by Aristotle. This implies that, whereas Plato assumed that the
purpose of argument was to
point towards truth, Aristotle found that argument could break down the subject
under study, much as
dissection could cut up flowers and fish. Similarly, Plato had the word 'quality'
but not the word 'quantity' -
another word Aristotle coined. It's why Plato is always more interested in oneness,
twoness and threeness
than one, two and three.
His approach to mathematics is contemplative, as is indicated in his story about
Socrates observing two
raindrops colliding to form a single silvery ball of water. "Where did the twoness
- the separation, the duality,
the independence - go?" he has Socrates ask. But Aristotle is different. He can
also contemplate numbers
mathematically. He does argue that '3' is a perfect number because it contains a
beginning, middle and
end; but he's also interested in 'how-muchness' - which is what 'quantity' means.
After this, thinkers became
interested in the calculable aspect of objects in an empirical world. That's
something new. Owen Barfield
writes that, with Aristotle, "The human mind had now begun to weigh and measure, to
examine and
compare; and that weighing and measuring has gone on - with intervals - for twenty-
three centuries". You
could say that, after Aristotle, practical knowledge could be distinguished from
theoretical knowledge.
That's different from the wisdom of myths and traditions, in which those two
aspects are seamlessly
intertwined.

. Page 34 of 44
111. The author writes "That's different from the wisdom of myths and traditions,
in which those two aspects are
seamlessly intertwined." Which of the following, if true, would weaken this
statement?
(a) In Greek mythology, Zeus is the most powerful god and he lived on Mt. Olympus.
(b) Hinduism is a way of life in which the traditions and rituals are timeless
because of their applicability.
(c) The Mahabharata not only explains and delineates scientific concepts which can
be used in our daily
life, but also teaches us values
(d) The Egyptians built pyramids to honour their Gods despite having no modern
machinery.
112. Which of the following most accurately expresses the main point of the second
paragraph of the passage?
(a) Plato was the teacher of Aristotle and therefore, knew much more than
Aristotle.
(b) Aristotle improved his knowledge by building up on the work of his predecessors
and creating new
concepts.
(c) Plato was subjective in his approach to mathematics while Aristotle was
interested in the calculable
aspect of mathematics.
(d) Plato coined the word 'analogy' while Aristotle coined 'analysis' because
Aristotle was a mathematician
while Plato was the philosopher.
113. Which of the following weakens the hypothesis proposed by Anthony Kenny?
(a) Heraclitus was a thinker predating Plato by centuries who first summarized the
teachings of his
predecessors to critique them.
(b) Aristotle had learnt Mathematics from Socrates who invented the concept of
quantity.
(c) Plato had the word 'analogy', but not 'analysis'.
(d) None of the above.
114. Which of the following can be inferred from the passage?
(a) Plato invented the word 'analysis' to dissect and understand ideas better.
(b) Aristotle argued that '3' is the perfect number because it does have a
beginning, a middle or and end.
(c) Aristotle had some role to play in the classification of knowledge as practical
and theoretical.
(d) Plato believed that wisdom provided in legends and myths was too muddled to be
of any use.
Passage (Q.115–Q.119): The electric vehicle market for two-wheelers in India is at
a crossroads. Around
81% of the 18.6 million vehicles sold last year were two-wheelers. Therefore, if
the industry transitions from
one dominated by vehicles run through internal combustion engines to EVs, it will
be in sync with India’s
climate change goal. To quicken the transition, GoI provides a subsidy to lower the
cost of ownership and
some states top up with yet another subsidy. This supportive policy environment has
encouraged both
traditional manufacturers and newer ones such as Ola Electric to enter the EV
market.
The enthusiasm for EVs sometimes leads to calls for unsound policy changes. To
illustrate, in 2019, Niti
Aayog proposed all two-wheelers running on combustion engines be banned by 2025.
Echoing that
approach, Ola’s co-founder has called on incumbents to reject petrol and fully
commit to electricity. These
prescriptions need to be unpacked. Public policy has provided a boost to EVs
through a set of financial
incentives to stoke demand. That’s consistent with the approach to dealing with
climate change. However,
public policy needs to also consider other consequences. Two, in particular, are
important.
Combustion engines’ fuel taxes have emerged as one of the biggest sources of
revenue for the
government, and one that shored up budgets in a pandemic. Revenue stability matters
as governments
perform many indispensable functions. Separately, there are strategic issues to
consider. China dominates
both the processing and manufacturing of Lithium-ion batteries. Moreover, mineral
ores and concentrates
are found in just a few countries, with China again having a key position. A policy
that skews towards a
premature transition to EVs can have unintended consequences. For now, India’s
subsidised EVs should
focus on competing with combustion engine vehicles, which have consistently met
escalating tailpipe
emission standards.

. Page 35 of 44
115. Why EVs are considered the future?
I. A better alternative to combustion engine vehicles.
II. It will help in overcoming the climate issues.
III. It will bring more revenue for the government.
(a) Only I (b) Only II (c) Only III (d) Both I & II
116. What does the author mean by the given statement?
The enthusiasm for EVs sometimes leads to calls for unsound policy changes.
(a) The eagerness to bring in the EVs must be backed by a practical policy change
to avoid unfactored
consequences.
(b) People are enthusiastic about EVs that it increased the sales of Two-wheeler
and rises up to 81%.
(c) The precedents set by earlier policy changes has resulted in huge financial
losses to the government.
(d) India is not ready for the change as it is matter of changing lifestyle, which
cannot happen overnight.
117. What is the major reason for Ola to enter the EV market?
(a) Increased sales of two-wheelers in India by 81%.
(b) To help attain climate change goal.
(c) The supportive environment policy.
(d) Guarantee provided by the government.
118. What is meant by the author by “premature transition to EVs can have
unintended consequences”?
(a) By this statement the author is saying that the decision is very quick and that
it will bring consequences.
(b) Unsound policy changes, Revenue stability and Strategic issues are some of the
factors that need to
be addressed to avoid unseen consequences, before taking the transition head on
(c) EVs are not a safe option to compete with combustion engine vehicles
(d) Premature transition could lead to dire consequences such as territorial and
internal faction.
119. Which one of the following cannot be the assumption behind the passage?
(a) States have the authority to give subsidies.
(b) Policy changes are not immune against ramifications.
(c) The climate change is a reality.
(d) Financial incentives do not encourage demand.
Passage (Q.120-Q.122): Dead wood-eating beetles can cause damage to residential
properties, just like
termites may do. But they repay humans by performing a priceless service: helping
us recycle decomposing
dead trees. Decomposition may have an unpleasant ring to it but it is a fundamental
process in a functioning
ecosystem, ensuring that we are not buried under the huge mass of dead organic
matter that is produced
every year right on our own doorsteps. Dead wood-eating beetles are among the
insect world’s best
decomposers – organisms that digest dead matter and make their own living cells and
tissues out of the
acquired atoms. The vast majority of organic matter produced worldwide every year
is stored in wood,
which is tough, and hard to digest and decompose. Digested wood may be source of
energy, but it is
insufficiently nutritious so that all the organisms developing in dead wood –
beetles but also flies, moths,
and bacteria – struggle with growth, development and maturation.
Still, dead wood eaters are able to survive and thrive on this low-quality food
source. How they do it?
Common knowledge would suggest that the activity of wood-eating beetles’ symbionts
provides them with
nutritionally balanced diets. And we know that beetles are able to synthesize
important organic compounds
out of nutrients furnished by their primary food, the dead wood.
But according to the law of conservation of mass, which dictates that the mass of
the products in a chemical
reaction must equal the mass of the reactants, a complete diet based on pure wood
would be impossible.
The atoms composing nutrients cannot be created out of nothing. It has been
estimated that for wood#eating beetles to consume the atomic composition of their
bodies from wood alone would require

. Page 36 of 44
approximately 40 years for males and 85 years for females, which are bigger. In
fact, the beetle’s growth
period spans at most three to four years in nature.
120. All of the following can be inferred except?
(a) Dead wood-beetles are a boon and a bane depending on their choice of food
(b) Dead wood eaters are a scientific amazement as they survive on an almost
impossible diet
(c) There exist chemical mixtures to decompose wood but they are not as viable
(d) Decomposition is essential for life to exist on Earth
121. Which of the following, if true, would seriously undermine the author’s
argument?
(a) Biologists are yet to completely understand the Dead wood-beetles
(b) A Dead wood-beetle takes about 10 years to completely decompose a block of wood
(c) The dead wood-beetles have a negligible body weight which translates to
negligible energy
consumption
(d) None of the above
122. Which of the following conclusions can be properly drawn from the statements
above?
(a) There exits other organisms that develop in dead-wood
(b) Wood decomposition can only be done by dead wood-beetles
(c) Both (a) & (b)
(d) Neither (a) nor (b)
Passage (Q.123-Q.126): If you have a serious need to memorize, you usually must
take notes. Just what
is it that I think is valuable about note taking? First and foremost is the
requirement for engagement.
Students must pay attention well enough to make decisions about the portion of the
learning material that
will need to be studied later. Paying attention is essential for encoding
information, and nobody can
remember anything that never registered in the first place.
Note taking requires thinking about the material to decide what needs to be
captured for later study. This
hopefully generates questions that can be raised and answered while working your
way through the
learning material. Sadly, in the last few years I notice that in my college
classes, few students take notes.
It is as if they think they can remember everything. The cause may be that teachers
tend to hand out pre#packaged notes. I object to this practice, because it reduces
the level of student engagement and thinking.
Handwritten notes nave special advantages. If done in pencil, items can be erased
or re-arranged. You
can draw diagrams and pictures, which provide visual images to strengthen
memorization and control
spatial layouts. Using different layouts for every page gives each a visual
uniqueness that facilitates
memory. A well-established fact about memory is that spatial location is an
important cue for encoding and
recall. where information is located provides important cues as to what the
information is. The spatial layout
of script and diagrams on a page allows the information to be visualized, creating
an opportunity for a
rudimentary form of photographic memory, where a learner can imagine in the mind's
eye just where on
the page certain information is, and that alone makes it easier to memorize and
recall what the information
is.
123. Which of the following can be inferred from the given passage?
(a) Not everything we read for note taking is important.
(b) Spatial layout helps in creating photographic memory.
(c) Pictures are easier to remember than words.
(d) Pre-requisite(s) to note taking is/are optional.

. Page 37 of 44
124. Which of the following would the author most likely to agree with, as per the
passage?
(a) A child with higher IQ can remember a thing even if it hasn't been registered
in the first place.
(b) There is no engagement when the students learn or memorise via audio medium.
(c) Students in author's class can remember everything.
(d) Using pencil over pen for handwritten notes is recommended.
125. Which of the following would form the premise for the statement made by the
author that "If done in pencil,
items can be erased or re-arranged"?
(a) Note making usually requires modification through the process.
(b) Handwritten notes using other stationeries apart from pencil can't be erased or
rearranged.
(c) Students who studied handwritten notes, made using pencil, scored significantly
higher than students
using laptops.
(d) There are pens and markers available in the market, which facilitate
modification and rearrangement.
126. Which of the following can be the immediate course of action to address the
issue of pre-packaged notes
as raised
in the passage?
(a) Pre-packaged notes should be banned and student should be asked to make their
own notes.
(b) The notes should be structured in a skeleton form that just provides an
organized framework for
students to construct their own notes.
(c) Students should be taught the art of note-making so that they can make their
own notes effectively,
thus negating the need of pre-packaged notes from teachers.
(d) The level of student engagement and thinking should be increased via other
curricular and extra#curricular activities.
Passage (Q.127-Q.131): The experience of having children is something that many of
us have in common.
Despite having this shared experience, no two parents' approach to raising children
is precisely the same.
There are many modern terms for different types of parenting styles; perhaps you
identify with "free-range
parenting" or "attachment parenting," or maybe you like to think of yourself as a
"tiger mom." Among these
new terms, the one that you are likely to hear the most about is called "Helicopter
Parenting."
The term Helicopter Parenting was coined in the 1990s and generally applied to
parents who are overly
involved in their children's lives, especially in academic and achievement-related
activities. Textbook
Helicopter parents tend to remove obstacles that their children face to encourage
them to succeed.
Helicopter parents are generally well-educated, well-resourced parents who are
incredibly well-intentioned,
looking to protect their children from trouble and provide them with as many
opportunities as possible.
Having involved parents is generally a good thing. In fact, helicopter parenting
has been associated with
some positive parenting behaviours, such as frequent advice-giving and providing
children with emotional
support. But constantly hovering can also come at a cost. As children get older,
they seek out more
independence, and it can be a challenge for any parent to relinquish control
slowly. This is especially
difficult for helicopter parents, who tend to claim a large amount of control over
their children's academic
lives.
However, research suggests that inflexibility in maintaining the same level of
control over children
regardless of what's developmentally appropriate can be problematic. In fact,
despite some of the positives
of helicopter parenting, it has also been associated with adverse child outcomes,
such as higher levels of
anxiety and depression, lower ratings of psychological well-being, and a lack of
independence and
ineffective coping skills.

. Page 38 of 44
127. Which of the following would the author most likely to agree with, as per the
passage?
(a) Among the different types of parenting styles in the modern era, Helicopter
Parenting is the most
common and is followed by a majority.
(b) Keeping children from failing promotes failure.
(c) There are more disadvantages of Helicopter Parenting as compared to its
advantages.
(d) All (a), (b) and (c).
128. Which of the following can be inferred from the given passage?
(a) The concept underlying Helicopter Parenting was not present before the year
1990.
(b) No two parents' approach to raising children is precisely the same.
(c) Despite the good intentions of most overinvolved parents, children of
helicopter parents don't
necessarily perform well in school.
(d) A child of 15 years of age will seek out more independence than a child of 10
years of age.
129. Which of the following can be the course of action for helicopter parents
while intervening with their
children's academics and achievement-related activities?
(a) They shouldn't get involved in their children’s academic and achievement-
related activities and let them
choose paths and interests of their own.
(b) They should be flexible in their control over children and gradually provide
autonomy as it helps children
learn how to deal with challenges independently.
(c) They shouldn't allow their children to fail as failure acts as a deterrent in
pursuing the activities of their
interest further.
(d) All of the above.
130. If permissive parents have contrary attributes compared to helicopter parents,
which of the following would
likely be
true for them?
(a) Permissive parents will be less involved with their children's everyday
activities.
(b) Permissive parents will be supportive, yet demanding.
(c) Permissive parents will be the most well-adjusted and the most likely to be
independent and well#socialized.
(d) The children of permissive parents turn out to have the best cognitive
flexibility, which is one of the
predictors of academic achievement.
131. Which of the following would form the premise for the statement, "A textbook
helicopter parent tends to
remove obstacles that their children face to encourage them to succeed"?
(a) Obstacles demotivate children to proceed further and succeed.
(b) Every fall presents an opportunity to correct those mistakes and learn from
them.
(c) Helicopter parents assume that their children aren't mature enough to remove
the obstacles on their
own.
(d) Both (a) and (c).
Passage (Q.132-Q.134): Each of the six persons A, B, C, D, E and F belongs to a
different tribe among P,
Q, R, S, T and U. Each of them belongs to one of the four countries Bhutan, China,
Mangolia and Pakistan.
At least one and at most two persons belong to each of the four countries. Each of
them knows exactly one
language among Spanish. French, Dutch and Swedish. At least one and at most two
know each of these
four languages. No two persons have the same combination of country and language.
The following
information is known about them.
(i) F and C belong to the same country and A and D belong to different countries.
(ii) E belongs to China and tribe T. The person who belongs to tribe P knows
Swedish.
(iii) Two persons know Dutch. Two persons belong to Bhutan and two persons belong
to Pakistan.
(iv) B belongs to Bhutan. Two persons, the one who belongs to the tribe R and B,
know French.
(v) C knows Dutch and belongs to tribe Q.

. Page 39 of 44
(vi) Neither A nor the person who belongs to Pakistan know French.
(vii)The persons belonging to the tribes Q and U know the same language.
132. To which country does A belong?
(a) Bhutan
(b) Pakistan
(c) Mangolia
(d) China
133. To which tribe does F belong?
(a) P
(b) Q
(c) R
(d) S
134. Which of the following is true about the persons who belong to the tribes S
and U ?
(a) They both know the same language.
(b) One of them is F and the other one is B.
(c) They both belong to the same country.
(d) They do not have anything in common.
135. Following question consists of some statements followed by some Conclusions.
Consider the statements
to be true even if they vary from commonly known facts and find out which of the
Conclusions logically
follow(s) the given statements disregarding the commonly known facts.
Statements:
All pipes are leaves.
All pipes are games.
All eagles are games.
Conclusions:
I. No leaves is a game.
II. Some leaves are games.
III. All leaves being eagles is a possibility.
(a) Only II follows. (b) Only III follows.
(c) Either I or II follows. (d) Only II and III follow

mock 16

Directions (Q.1-Q.30): Read the passage carefully and answer the questions that
follow.
Passage: (Q.1-Q.5): That language is a unique human form of communication is hardly
news. Less
obvious is the importance of two non-verbal precursors that are also uniquely
human. Here, we describe
the first such precursor: face-to-face intersubjective relations between an infant
and her mother.
Responding to human faces and sharing eye gaze contribute to a dramatic example of
inter subjectivity.
Andrew Meltzoff, a developmental psychologist, showed that an infant can imitate an
adult's facial
expression, 42 minutes after birth! While the infant is on a pacifier, an adult
protrudes his tongue. After a
minute, the adult removes the pacifier. About two minutes later, the infant
protrudes its tongue.
A new-born infant's ability to imitate is remarkable because the infant has never
seen its own face. To
imitate, it has to match information from two modalities: its perception of the
adult's face and proprioceptive
feedback from the muscular activity of its own face. As Meltzoff commented, "The
self can be felt, but
cannot be seen. The other's face can be seen but not felt. Yet self and others
connect. The other can be
understood as like me, at least in the sense that we can do the same acts".
As impressive as imitation by a new-born infant is, it refers to individual events.
Those events don't do
justice to the interactions that an infant and her mother establish early during
infancy. To capture those
interactions, developmental psychologists have focused on the dyad of infant and
mother, and how their
behaviour is coordinated as they interact face to face.
Even though infants don't produce phonemes (the basic unit of speech) until they
are six months old, they
can make other sounds. And even though they are unable to crawl or stand, they can
smile, move their
hands, turn their heads, and so on. Measurement of infant-mother coordination,
however, can be
problematic. Because it is often too rapid to experience in real-time,
developmental psychologists record it
on audio and/or videotape. That allows the mother's and the infant's gaze,
vocalizations, and facial effect
to be assessed by well- trained observes.
A pioneering study by Mary Bateson recorded the relation between the vocal
responses of two-month-old
infants and their mothers. In response to the mother saying, "What you gonna say?",
"Huh?," "Oh my!,"
"You gonna be a good boy today?" and so on, the infant often responded by cooing,
grunting, whimpering,
and making other infant sounds. Bateson reported a strong correlation between the
mother's utterances
and the infant's vocal responses. Because there was little temporal overlap between
those utterances,
Bateson referred to them as "proto-conversations." That interpretation seems
justified because the infant
and the mother alternated their utterances, in a turn-taking pattern, just as
adults do in real conversations.
1. What is the contextual meaning of the following as used in the passage?
Dyad
(a) Wholesomeness (b) Pair (c) Relationship (d) Intricacy
2. What would be an appropriate title of the passage?
(a) Precursors of language.
(b) Mother and Infant: an intersubjective relation.
(c) Relationship between mother and infant.
(d) Unique communication between infants and adults.
3. What's the purpose behind citing an example in the second paragraph of the
passage?
(a) To delve into technicalities behind the action performed by infants in response
to a stimulus.
(b) To emphasize the point that infants are smarter than adults think they are.
(c) To illustrate the love and affection between infants and mothers as a result of
a strong bond.
(d) To highlight that infant, have exceptional imitating abilities as a response to
non-verbal communication.
4. Identify the statement(s) which is/are correct as per the information mentioned
in the passage.
I. An infant while imitating, bases its response on two modalities to match
information.
II. Face-to-face intersubjective is the only precursor of the language.
III. Interaction between mother and the infant is justified by the impressive
imitation of the infant.
(a) III only (b) I only (c) I and III only (d) II only

. Page 3 of 36
5. What would most likely be the profession of the author?
(a)Neonatologist (b) Language teacher
(c)Child psychologist (d) Paediatrician
Passage (Q.6-Q.10): Too many leaders assume that talking about ethics is something
you do when there
has been a scandal, or as part of an organization's compliance program. Everyone
gets their annual
"ethics flu shot" in the mandatory review of the compliance policy, and all is well
for another year. Nick
Eply, professor at the University of Chicago, in Four Myths about Morality and
Business, says, "It's a myth
to think 'Everyone is different, and everything is relative.' You actually have to
teach people the relative
value of principles relative to choices." Leaders have to infuse everyday
activities with ethical
considerations and design policies and norms that keep ethics top of mind. Jonathan
Haidt, Professor of
Business Ethics at NYU says, "It's important to talk about the positive examples of
ethical behaviour, not
just the bad ones. Focusing on the positive reasons you are in business and
reinforcing the good things
people do strengthen ethical choices as 'the norm' of the organization."
Leaders must accept they are held to higher standards than others. They must be
extra vigilant about not
just their intentions, but how others might interpret their behaviour. While they
cannot control every possible
misinterpretation, leaders who know their people well make careful choices in now
they react to stressful
situations, confront poor performance, how political they are in the face of
controversy, and how receptive
they are to bad news. Above all, even in what might be considered the smallest
"white lie," ethical leaders
are careful not to signal that hypocrisy is okay. As an example, a leader may
casually review an employee's
presentation and provide feedback like, "I think we need to take these two slides
out - that data is
inflammatory and we don't want to derail the ultimate outcome which is to convince
the budget committee
to give us the resources we want." While the leader might presume he has acted in
the best interest of the
group - going to bat for resources they need- the person building the presentation
has just been told, "We
can't tell the entire truth because it could prevent us from getting what we want."
Leaders must put
themselves in the shoes of those they lead to see what unintended messages they may
be sending.
6. According to the passage, why is it important to discuss positive examples of
ethical behaviour?
(a) To make the employees realise the necessity of ethical behaviour.
(b) To encourage people to make ethical choices in the organisation.
(c) To attract the attention of other companies.
(d) To discourage people from making unrequired ethical choices.
7. Which of the following is the meaning of the word 'compliance' as mentioned in
the given passage?
(a) The act of disobeying the authority.
(b) The act of obeying an order, rule or request.
(c) Strengthening something.
(d) Boasting one's abilities.
8. With which of the following statement is the author likely to agree?
(a) Role of a leader is slightly important in an organisation.
(b) Ethical behaviour is important in an organisation.
(c) A leader needs to be apathetic towards its employees.
(d) Ethical behaviour is expected during times of scandal.

. Page 4 of 36
9. Which of the following statement is similar to the last sentence of the given
passage?
(a) Police need to think like criminals as it helps to have the perspective of the
criminal when analysing
possible escape routes, points of entry, and patterns of deception.
(b) The Prime Minster, while addressing a public meeting, compliments the
departments for their
contributions to the well-being of the public and gently reminding them to keep in
my mind nation’s
health as the prime goal.
(c) Hard to prove, easy to join in on, and devastating in impact, ostracization is
a favourite tactic of
workplace aggressor.
(d) Focusing inward and focusing constructively on others helps leaders cultivate
the primary elements of
emotional intelligence.
10. According to the passage, which of the following is the trait of an ethical
leader?
(a) They are careful that hypocrisy does not become a trend in the organisation.
(b) They set their standards same as their employees.
(c) They talk about ethics only when it suits the leaders.
(d) They keep in mind that data should not be inflammatory.
Passage (Q.11-Q.15): Paragraph 1: With an important source of revenue down and the
flow of customers
flattening out, one of the biggest businesses in Georgia — its public university
system — is turning to a
strategy of consolidations and mergers to improve efficiency and cut costs. The
system has shrunk from
35 campuses to 28, helping compensate for a nearly 20 percent cut in state funding
from 2008 to 2016 and
an enrollment that this spring rose only two-tenths of a percent over last year’s
spring semester. More
consolidations are underway, reducing four of the remaining campuses to two. The
universities are putting
some of the resulting $24 million in savings into efforts to reduce the number of
dropouts.
Paragraph 2: Dramatic changes like these are essential, Chancellor Steve Wrigley
told his Board of
Regents in April. “We inherited a system largely conceived in the 1960s,” Mr.
Wrigley said. “But times,
society and students have all changed dramatically.” That is not only true in
Georgia. Other colleges and
universities across the country are also responding (albeit sometimes slowly) to
challenges threatening
their traditional role in society if not their survival.
Paragraph 3: Changes to immigration policies, and resulting resentments, threaten
the crucial supply of
international students, which the consulting firm Dr. Education predicts could cost
universities in the United
States a quarter of a billion dollars in the coming academic year. To fill seats,
colleges are engaged in an
arms race of discounts that they increasingly cannot afford — discounts so deep (as
much as 51 percent)
that, while their sticker prices appear to be rising ahead of the inflation rate,
the schools are actually seeing
their net tuition revenue decline.
Paragraph 4: While public funding for higher education is rising again in some
states, it is still an inflation#adjusted $9 billion behind where it was before
deep cuts were imposed during the last recession, the
Center on Budget and Policy Priorities reported. One reason is that many states
face large pension
obligations for public employees, including those who work at universities; for
some universities, the impact
is becoming more immediate, as states shift this burden directly onto them. More
than half of the $4.1
billion allocated for state universities and colleges in Illinois, for instance,
now goes not to teaching or
research, but to pay retirement costs, the Illinois Policy Institute says.
Paragraph 5: “There’s always been a kind of a wishful thinking that when the
economy gets back to
normal, things will get better. And that is not happening anymore,” said James A.
Hyatt, associate director
of the Center for Studies in Higher Education and former vice chancellor for budget
and finance at the
University of California, Berkeley.

. Page 5 of 36
11. According to Paragraph 1, what is the underlying cause of consolidations,
mergers and shrinking of the
number of universities?
(a) Cut in state funding and decline in student enrollment.
(b) Mass migration of students from Georgia.
(c) Many students underperformed in their tests/exams.
(d) The closure of some universities and colleges.
12. How are the universities proposing to utilize the savings resulting from
consolidations, mergers and
shrinking, as mentioned in Paragraph one?
(a) They intend to use the savings to raise the standards.
(b) They intend to encourage students to score higher grades.
(c) They intend to reduce the number of dropouts and encourage them to stay on.
(d) They intend to award scholarships to Hispanic students.
13. In paragraph three, what does Dr. Education warns in the light of changes to
immigration policies and
resulting resentments, threaten the crucial supply of international students?
(a) The firm predicts a fall in educational standards.
(b) Restrictive changes to immigration would cost the US not only students, but
also dollars.
(c) The firm said many universities would wind up sooner than later.
(d) More discounts should be handed to international students.
14. What major financial constraint do state colleges and universities in Illinois
face according to paragraph 4?
(a) They face shortage of revenue from fees and government grants.
(b) There's a drastic decline in student enrollment.
(c) Every year innumerable dropouts emerge from all colleges and universities.
(d) More than half of the government allocated funds go to paying retirement costs.
15. What do you understand by "wishful thinking", as used in the fifth paragraph of
the passage?
(a) Wishful thinking is an attitude or belief that something you want to happen
will happen even though it
is not likely or possible.
(b) Wishful thinking is a deep-seated attitude which is akin to the saying "if
wishes were horses, beggars
would ride them."
(c) Wishful thinking is a wish list drawn up by a temperamental person in which he
or she lists all that he
or she wants to buy or own, even though they can't at the time.
(d) Wishful thinking is an inveterate belief that everything that you throw into a
wishing well will be realized
very soon, contrary to your experience.
Passage (Q.16-Q.20): Professional services firm Ernst & Young LLP has suggested
coffee growers avail
the services of MGNREGA workers to solve the issue of labour shortage and rein in
the consequent high
production cost. In a draft report prepared by the firm on the strategic road map
for the Indian coffee sector,
the firm suggested the use of MGNREGA labour up to the yielding stage. Currently,
only trenching activities
inside coffee plantations are covered under MGNREGA.
The sector's labour woes reached a boiling point recently after a large population
of immigrant labourers
returned to their hometowns. Ernst & Young was commissioned by the Coffee Board to
undertake a study
on behalf of the United Planters' Association of Southern India and several coffee
growers' associations
including Karnataka Planters' Association, Karnataka Growers' Federation, Kodagu
Planters' Association.
The draft recommended that the Coffee Board formulate cost norms similar to the
convergence guidelines
for rubber. It also suggested that small farmers, who hold up to 10 hectares of
land, should be given access
to MGNREGA labour pool. As per the draft, the coffee sector is currently staring at
several challenges
around cost competitiveness, price volatility and lack of premiumisation. Coffee
farmers are not price
setters, but price takers and Arabica farmers are selling below the cost of
production and have been
incurring net loss in the last five years consecutively.

. Page 6 of 36
Commenting on the recommendations, Coffee Board Chairman M.S, Boje Gowda said, "A
proposal to
make MGNREGA labour available in the coffee sector is a positive thing. However, a
lot more clarity has
to emerge with regard to the specifics of the work. We have to decide what kind of
work can be allotted to
these labourers, because on a coffee plantation certain activity can be done only
by skilled workers.''
Echoing similar sentiments, UPASI president ALRM Nagappan said linking MGMREGA
scheme with the
coffee sector was a constructive idea though a detailed scheme was required. Only
then, the coffee
growers would be able to take advantage of the scheme.
Anil Kumar Bhandari, president, India Coffee Trust, was of the opinion that, for
the record, every coffee
farmer would be in favour of getting MGNREGA labour. However, the draft had not
fleshed out the
modalities of the proposal, it did not explain now it could be set up or what works
could be given to those
labours.
"As we all Know, MGNREGA labour is primarily meant to create rural assets: roads,
culverts, canals public
buildings, public wells etc. The question is: how can individual coffee growers can
use their services when
there is no public asset created? Also, the EY draft said it would save costs
significantly, but did not explain
how," he added. A final report will be out once the Coffee Board decides on cost
norms and guidelines
around the 'nature of coffee work' that could be allotted to MGNREGA workers.
16. Which of the following can be concluded about the plan to provide MGNREGA
workers to Coffee sector?
(a) The plan is not ready to implement and there is a need to decide on a few
things which are crucial for
its implementation.
(b) The plan is substantial but cannot be implemented until all the people in
authority are on the same
page.
(c) The nature of work of the immigrant labourers is no more an issue for the
policy makers.
(d) Creation of public asset is a part of MGNREGA employment scheme but in case of
coffee plantation
this condition is not satisfied
17. Which of the following cannot be definitely true about MGNREGA workers as per
the given passage?
(a) MGNREGA workers' wages are too high.
(b) All the MGNREGA workers are not skilled workers.
(c) MGNREGA covers a few activities inside coffee plantations. D
(d) Both, (a) and (b)
18. Who among the following has been least affected by the challenges in the coffee
sector as per the given
passage?
(a) Coffee Board (b) Coffee growers' associations
(c) MGNREGA labourers (d) Coffee farmers
19. Which of the following can be inferred from the given phrase in the passage?
"Coffee farmers are not price
setters, but price takers."
(a) Coffee farmers do not have the authority to set the price of their product and
some other authority sets
the price.
(b) Coffee farmers are at the mercy of the brutish authority to set the price of
their product.
(c) Coffee farmers are given the prices of the product which they distribute among
themselves.
(d) All of these.
20. According to the passage, what is the assumption behind the following sentence:
We have to decide what
kind of work can be allotted to these labourers, because on a coffee plantation
certain activity can be done
only by skilled workers"?
(a) The labourers which have been talked about in the sentence are skilled workers
but do not know much
about coffee plantation.
(b) The decision about allotting the work to these labourers is one of the most
tiresome tasks.
(c) Coffee plantation has a complicated structure but none of the activities need
expert knowledge to
continue the production.
(d) The labourers which have been talked about in the sentence as the replacement
of earlier workers are
mostly unskilled labourers.

. Page 7 of 36
Passage (Q.21-Q.25): Mother died today. Or, maybe, yesterday; I can’t be sure. The
telegram from the
Home says: YOUR MOTHER PASSED AWAY. FUNERAL TOMORROW. DEEP SYMPATHY. Which
leaves the matter doubtful; it could have been yesterday.
The Home for Aged Persons is at Marengo, some fifty miles from Algiers. With the
two o’clock bus I should
get there well before nightfall. Then I can spend the night there, keeping the
usual vigil beside the body,
and be back here by tomorrow evening. I have fixed up with my employer for two
days’ leave; obviously,
under the circumstances, he couldn’t refuse. Still, I had an idea he looked
annoyed, and I said, without
thinking: “Sorry, sir, but it’s not my fault, you know.”
Afterwards it struck me I needn’t have said that. I had no reason to excuse myself;
it was up to him to
express his sympathy and so forth. Probably he will do so the day after tomorrow,
when he sees me in
black. For the present, it’s almost as if Mother weren’t really dead. The funeral
will bring it home to me, put
an official seal on it, so to speak.
I took the two-o’clock bus. It was a blazing hot afternoon. I’d lunched, as usual,
at Céleste’s restaurant.
Everyone was most kind, and Céleste said to me, “There’s no one like a mother.”
When I left they came
with me to the door. It was something of a rush, getting away, as at the last
moment I had to call in at
Emmanuel’s place to borrow his black tie and mourning band. He lost his uncle a few
months ago.
I had to run to catch the bus. I suppose it was my hurrying like that, what with
the glare off the road and
from the sky, the reek of gasoline, and the jolts, that made me feel so drowsy.
Anyhow, I slept most of the
way. When I woke up, I was leaning against a soldier; he grinned and asked me if
I’d come from a long
way off, and I just nodded, to cut things short. I wasn’t in a mood for talking.
The Home is a little over a mile from the village. I went there on foot. I asked to
be allowed to see Mother
at once, but the doorkeeper told me I must see the warden first. He wasn’t free,
and I had to wait a bit. The
doorkeeper chatted with me while I waited; then he led me to the office. The warden
was a very small man,
with gray hair, and a Legion of Honour rosette in his buttonhole. He gave me a long
look with his watery
blue eyes. Then we shook hands, and he held mine so long that I began to feel
embarrassed.
21. What is the author’s reaction when he is showed sympathy for his mother’s
death?
(a) He is successful in expressing his respects to his mother.
(b) He is awkward and detached with people’s expression of open sympathy.
(c) He is not trying to run away from the people expressing sympathy.
(d) He was overwhelmed by the sympathies he received.
22. Where did the author’s mother pass away?
(a) At a monastery in Algiers.
(b) In the author’s village called Marengo.
(c) Near a village called Marengo.
(d) At an old age home in Marengo.
23. What can be said about the author’s character from the passage?
(a) He was matter-of-fact.
(b) He seems detached from or unwilling to face the harsh truths of reality like
death.
(c) He seems more concerned about the code of propriety than his own feelings.
(d) All the above.
24. What can be said regarding the author’s relationship with his mother?
(a) It bordered on indifference.
(b) It was detached.
(c) It was close and caring.
(d) It was full of anger.

. Page 8 of 36
25. The passage mentions that the author’s boss would express his sympathy ‘when he
sees me in black’.
What does the underlined phrase ‘being in black’ mean?
(a) To see someone in great sorrow or distress.
(b) To avenge someone’s death.
(c) To be in mourning for the death of a close relative.
(d) To be able to unable to engage in social affairs due to someone’s death.
Passage (Q.26-Q.30): The much-anticipated fifth generation (5G) networks are
expected to roll out across
the world by 2020, and are expected to offer faster speeds and more reliable
connections, thus powering
Internet of Things (loT) devices. Some estimates peg 5G data rates to reach a peak
of 20Gbps (gigabits
per second).
However, as telecom operators (telcos) in countries like the US, Japan; South Korea
and China are largely
driving the first 5G rollouts. spectrum sharing will be of the key elements helping
them make the shift.
Spectrum sharing is a technology that allows superimposition of 5G carrier waves
atop existing long-term
evolution (LTE), or 4G. This is important for a country like India that is
currently riding the 4G wave and is
expected to adopt 5G in the next 2-3 years.
Spectrum sharing can dynamically allocate the right bands to users based on their
requirements, David
Hammerwall, from Ericsson, said. For instance, if a 4G and 5G user connects to a
network simultaneously,
each will be allocated the right spectrum. Hammerwall said Ericsson’s technology
works on all LIE bands
because the firm thinks those are the ‘most relevant.
In theory, spectrum sharing allows companies an easier way to shift from their
current LTE-capable (4G)
networks to 5G. In the move to 5G, cash-strapped Indian telcos will have to deal
with the fact that they
already have vast legacy networks deployed, which is a challenge.
“A homogenous network is an idea. But a legacy operator has to add new technology
on top of existing
technologies,” said T.V. Ramachandran, president of Broadband India Forum. However,
Ramachandran
also pointed out that challenges will present themselves. If the 4G network is 5G
ready, then spectrum
sharing can really help, he said. But for legacy networks-2G and 3G-there will
still be a challenge.
According to Hammerwall, in the future LTE will bear the brunt of user traffic.
while 2G remains for legacy
voice traffic. He believes that 3G will die.
26. Which of the following are features of 5G networks?
(a) An estimated peak rate of 20Gbps.
(b) Reliable connections.
(c) Ability to power Internet of Things (loT) devices.
(d) All of the above
27. Which of the following reflects advantages of spectrum sharing?
(a) It will be sharing of the key elements.
(b) It will help in using the legacy networks.
(c) It can allocate the right bands to users based on their needs.
(d) It is important for a country like India that is currently using the 4G.
28. Consider the following idiom/phrase from the passage. Which of the following
options give its correct
meaning?
Bear the brunt
(a) To put up with or make allowances for something or someone.
(b) To suffer the worst part of an unpleasant or problematic situation.
(c) To support or prove a claim or idea by one’s physical presence.
(d) To remember, think about. or consider someone or something.

. Page 9 of 36
29. Which of the following reflects David Hammerwall’s view on future networks?
(a) 4G networks will be mostly used for user traffic.
(b) 2G networks will be not be used for legacy voice traffic.
(c) 3G networks will be used for Internet of things (loT).
(d) 4G and 5G user connecting simultaneously will not find the right spectrum.
30. Which of the following can be a conclusion of the passage?
(a) Spectrum sharing can aid operators in switching from 3G to 5G
(b) 5G network would bring faster speed and better connectivity
(c) Legacy networks would also be used through spectrum sharing
(d) The fifth generation (5G) networks are expected to launch in India in 2020

Directions (Q.66 – Q.105): Read the comprehensions carefully and answer the
questions based on it.
Passage (Q.66-Q.70): We read a lot of news articles in various publications,
newspapers, and so on, but
is there any limit to how much information these pieces may broadcast about people?
or do the authors
have any right to write anything about a prominent figure, even if it seems
offensive to his or her
personality? In quashing a 2016 defamation case against Tamil daily newspaper,
Dinamalar, the Madras
High Court recently cautioned that leaders of the country or the State should not
be disrespected when
articles are printed or published about them. In this case, Justice Bhavani
Subbaroyan voiced a note of
disapproval over the manner in former Chief Minister of Tamil Nadu, J Jayalalithaa
was addressed in
the Dinamalar article in question.
"The petitioner's Newspaper are directed to refrain from printing matters in a
disrespectful manner. It has
been stated 'J', when the said person was the Hon'ble Chief Minister of Tamil Nadu
and should have been
addressed as Hon'ble Chief Minister J Jayalalitha and not as 'J'. While printing
and publishing matters
with regard to the leaders of the Country or State, the petitioners are supposed to
give respect and
address them accordingly," the Court said.
The Court made the observation while allowing a plea to quash the defamation
proceedings initiated
against Dinamalar citing Section 500 (punishment for defamation of public servant
or Leader of Country
and Section 199 (2) of the Code of Criminal Procedure (CrPC), which provides a
special procedure for
initiating prosecution proceedings for the defamation of a public servant or Leader
of Country. The Court
noted that in order to maintain prosecution under defamation intentional false
allegation is a necessity.
Further, Section 199 (2), CrPC, the allegations must directly touch upon the
conduct of the concerned
public servant or Leader of Country in the discharge of his or her public
functions. If the defamatory
statement is personal in nature, the special procedure will not apply and it is
only the concerned person
who has to file the complaint in his or her individual capacity, the Court
explained. The Court, however,
agreed with the petitioners (Dinamlar's Editor and Printer and its publisher) that
the allegations in question
do not touch upon the late Chief Minister's public functions and that the case
could only be one of personal
defamation. The Court, therefore, opined that the complaint filed by the City
Public Prosecutor under
Section 199 (2), CrPC would not stand.
66. Assume Dinamalar wrote an article that injured J Jayalalitha's personal
reputation, and the public
prosecutor subsequently sued Dinamalar under CrPC Section 500 and 199(2). Is it
possible for the public
prosecutor to do this?
(a) Yes, as Jayalaitha is a prominent figure and Dinamalar injured her reputation.
(b) No, as newspapers have the right to publish anything about anyone to make
people aware.
(c) Yes, he can sue Dinamalar under Section 199(2) but not under Section 500.
(d) Yes, he can sue Dinamalar under Section 500 but not under Section 199(2).
67. AZB is a news channel known for criticizing prominent politicians of the Ruling
party and hosting intense
debates on a variety of governmental issues. Sudhir is one of the news reporters
that has acquired a lot of
fame because of the content on this channel. Sudhir discovered on Monday when
reading the daily news
bulletin that the Delhi government is facing public criticism for failing to
organize an adequate number of
beds in hospitals for the poor during this pandemic's suffering. After reading
this, Sudhir said that Delhi's
Chief Minister, Arvind Kejriwal, has no humanity and has nothing to do with the
public. He's a jerk who has
no right to be in charge of the country. Arvind Kejriwal filed a defamation suit
against Sudhir under
Section 199(2) of the CrPC. Is it possible to sue Sudhir?
(a) No, as Sudhir word’s might not have caused harm to Kejriwal’s reputation.
(b) No, as Kejriwal has no right to sue Sudhir under Art. 199(2) CrPC.(provide
reason)
(c) Yes, as Sudhir’s words were imposing attack on Kejriwal’s performance of public
functions;
(d) Yes, as Sudhir tried to defame Kejriwal and therefore, harmed his public
reputation.

. Page 16 of 36
68. Shivam and his family dined at a well-known Mumbai restaurant. Shivam despised
the owner since the
previous time he was there, he had an altercation with him that seemed humiliating
in front of his friends.
This time, Shivam considered exacting his revenge. While eating there, Shivam
pretended to have eaten
something horrible and began to scream that the meal contained poison, causing
everyone to stop eating.
This resulted in a tremendous insult to the restaurant's owner and greatly ruined
the restaurant's
image. However, the owner was eventually able to establish his innocence by testing
the food and doing
a health checkup of Shivam, which resulted in the owner and the restaurant being
found to be innocent.
The owner subsequently filed a defamation lawsuit against Shivam under CrPC
sections 500 and 199(2).
Is it possible to sue Shivam under these provisions?
(a) Yes, as Shivam intentionally tried to defame the restaurant.
(b) Yes, as the restaurant was well-known and the owner had a right to sue him
under the given Section.
(c) Yes, he can sue Shivam under Section 199(2) but not under Section 500.
(d) No, Shivam cannot be sued under this particular Section as it can be only
exercised by leaders of
Country/ State. .
69. Priya is a writer who frequently contributes pieces to the daily magazine
Public Times. She writes on public
officials, their present activities, and related topics. Priya once wrote an
article condemning Narendra Modi,
the Country's Prime Minister, based on all of the facts she gathered from
Government Reports on
Government website. Narendra Modi, on the other hand, read the article and filed a
defamation suit against
Priya under CrPC Section 199(2). Is it possible to sue Priya?
(a) Yes, as she tried to defame the country’s PM, in discharging public function;
(b) No, as the article was fact based and not intentional.
(c) Yes, as Narendra Modi has a right to sue her under Art. 199(2), since she
alleged things on him in
discharge of public functions.
(d) No, as Narendra Modi is not a public servant but Prime Minister hence he has
absolute immunity.
70. According to the given passage, can any public authority sue any person who
makes a speech against
him/her?
(a) Yes, if the speech made is intentional and has false allegations against the
concerned Public Authority
which tries to defame them.
(b) Yes, as anyone who is a public Figure has a deliberate right to sue anyone who
speaks against him/her.
(c) No, only when matters are printed and published with respect to leaders of the
Country.
(d) No, everyone has a right to speech and people can voice against the public
figures too and exercise
their rights.
Passage (Q.71-Q.75): A compromise entered into between the accused and complainant-
victim in a
criminal case is one of the mitigating factors in interfering with the sentence of
the accused, but it cannot
be the solitary basis for a reduction in sentence, the Supreme Court ruled. Other
aggravating and mitigating
factors should also support the accused in order to mold the sentence in favor of
the accused the Bench
ruled.
The decision was rendered on an appeal filed against a judgment of the Bombay High
Court upholding the
conviction of the appellant for the offence punishable under Section 326 (causing
grievous hurt by
dangerous weapons) of the Indian Penal Code and sentencing him to undergo rigorous
imprisonment for
five years with a fine of Rs. 10,000.
"Nonetheless, if one goes through the decisions of this Court (Supreme Court), it
would appear that this
Court takes into account a combination of different factors while exercising
discretion in sentencing, that is
proportionality, deterrence, rehabilitation, etc.," the Court opined.
Compromise, if entered at the later stage of the incident or even after conviction,
can indeed be one of the
factors to absolving the accused of his guilt., the Court conceded.
But at the same time, it cannot be the sole factor, the Court underscored.

. Page 17 of 36
The present was one such case in which the Court cannot grant leniency based on the
compromise, the
Bench said.
71. Lala Sudhanshu was a small-time money-lender in Mumbai who used to provide
informal loans at
comparatively low-interest rates. One such instance was of him lending 10000/- to a
person, Karan, @10%,
which was considered reasonable in the informal lending sector. However, after
receiving the amount from
Sudhanshu, Karan ran away with it, trying to start over in a new city. However, the
police caught up with
him, and he was produced before the local magistrate. Therein, Karan pleaded for
leniency owing to his
lack of circumspection and reached a compromise with Lala. Can the Court pronounce
a reduced sentence
in this matter?
(a) Yes, as the compromise has been validly struck and swift recovery has been
made.
(b) No, as even though the amount was recovered, Lala was potentially duped of his
money.
(c) No, as there are no mitigating factors to support the accused to mold the case
in his favor;
(d) Yes, as the money has been recovered, and punishment does not serve any cause
here.
72. Shiva was a farm laborer working on the farm near the shunting yards of West
Rajasthan Railways. Trying
to earn as much as he could, he often used to pull double shifts working for over
18 hours at a stretch. One
such day, while he was working the night shift after having worked the whole day,
he decided to rest awhile
near the shunting yard where the engineers of the yard used to. To his dismay,
right where he was soundly
asleep, a machine malfunctioned, leaving Shiva with a nasty cut on his leg, which
could possibly cripple
him. The state legal authorities helped him in taking action against the railway
authorities for the same,
who had offered him 20 lakhs as compensation (equivalent to 30 years' worth of his
salary) and additional
healthcare costs along with a job in government sector. He had readily agreed and
was ready to plead that
the authorities be given a reduced punishment, with the engineers responsible also
being let off the hook.
How will the Court rule?
(a) The Court may decide to reduce the punishment as the victim has already been
overly compensated.
(b) The Court shall not absolve the railways & the engineers for endangering his
life.
(c) The Court will absolve both the Board and the engineers.
(d) The Court may only absolve the engineers as they had no fault in this
unfortunate scenario.
73. On his way home from watching a movie, Satya was mugged by one Shahid, who he
later recognized as
being his childhood friend, having fallen upon hard times. However, before Satya
could recognize Shahid,
Shahid had already stabbed him in the leg and had stolen his belongings. Due to
their history as childhood
friends, Satya decided to drop charges against him, and in return, he was given his
belongings with a sum
of 2000/- missing. Thus, the compromise between the two was reached. Can this be
termed as legally
viable?
(a) Yes, as the compromise has been struck in a pre-trial stage.
(b) No, as the compromise is prima facie illegal.
(c) No, as a trial must happen; otherwise, it would be a miscarriage of justice.
(d) Yes, as they are childhood friends and Shahid had no mens rea.
74. Ghess Jhadia was a highly sought-after economic offender, duping the government
of contractual
breaches, most wanted criminal in the territory of India. Before incriminating
evidence against him was
brought to light, he had managed to escape the country, fleeing to a non-
reciprocating country. To his
dismay, the reciprocating treaty between India and the nation of Greece was struck,
where he was hiding,
and the procedure for his extradition was set in motion. His default was of the
tune of 5000 crores, out of
which 4500 crores had been recovered from his estate. While negotiating with Ghess,
his counsel had
offered the Indian Union in the Court of law an amount of 2000 crore more if he saw
a reduced sentence
for his client. Can the Union accede to his demands?
(a) No, ignorance of law is no excuse;
(b) No, compromise is not solitary basis for reduction in sentence;.
(c) Yes, like the offer, being as lucrative as it is, also allows for the criminal
on the run to be brought to
India.
(d) Both (a) & (b).

. Page 18 of 36
75. Justice Bhati, in the most important trial of all his years as an HC judge, was
faced with a very difficult
question. The rape trial of a minor, wherein the victim was left physically
challenged for life and was given
compensation for life, and the convict was facing three life sentences, having
already served 14 years of
the same, had now developed bronchitis, leprosy, and lung cancer and was in acute
pain and thus his case
was under consideration. A plea for a reduced sentence was being preferred by the
convict so as to live
out his twilight years as a free man. The nation was divided on the issue, and the
judiciary was also stumped
to a great extent. Which of the following would be tenably preferred in the present
case?
(a) A reduced sentence for the convict with state-approved compensation for the
minor child.
(b) No reduction in the sentence due to the gravity and severity of the crime being
of such inhumane
descent.
(c) A reduced sentence from rigorous to mere imprisonment with access to medical
support.
(d) A reduced sentence with medical support at all times from the finest hospital
in the country 24x7 being
paid for by the taxpayers' money.
Passage (Q.76-Q.79): When history will look back at the worst communal riots since
partition in Delhi, it is
the failure of investigating agency to conduct proper investigation by using latest
scientific methods, will
surely torment the sentinels of democracy," observed a Delhi Court on Thursday
while discharging three
accused persons in a Delhi Riots case. Pulling up the investigating agency, the
Court observed that the
case appeared to have been solved merely by filing this charge sheet "without any
real effort being made
to trace out eye witnesses, real accused persons and technical evidence" in the
matter. "This Court cannot
permit such cases to meander mindlessly in the corridors of judicial system,
sweeping away precious
judicial time of this Court when the same is open and shut case," the Court said.
While looking at the facts
of the case, the Court took note of the fact that the accused persons were neither
specifically named in the
FIR nor any specific role was assigned to them in the matter. The police seems to
be still busy in filing
supplementary chargesheets therein. The precious judicial time of this Court is
being wasted in giving dates
in those cases. The Court was also of the view that it cannot permit such cases to
"meander mindlessly in
the corridors of judicial system, sweeping away precious judicial time" when the
same is open and shut
case. "The sort of investigation conducted in the instant case and the lack of
supervision thereof by the
superior officers clearly depicts that the investigating agency has merely tried to
pull the wool over the
Court's eyes and nothing else," the Court added.
76. G and H were accused of the riots in state of Dailhy. Wherein H had vandalised
certain police station and
along with certain group of people. G was infamous for organizing public
sloganeering and protests against
the government’s schemes that were not in consonance with their benefits, and had
been arrested earlier
for creating public disorder. After scrutinizing the CCTV footages, both were
charged under sedition and
organizing riots. In the light of the above passage, opt out the most appropriate
option.
(a) G should be charged with sedition as being infamous for organizing the protests
and slogans against
the government which resulted in the riots consequently.
(b) G and H both should be held liable for they are committing communal riots;
(c) H should be held liable for committing communal riots, due to the presence of
technical evidence;
(d) H supported the opinion of G and resorted to vandalism during the riots, hence
he should be held liable.
77. G and B were the Masterminds of State riots that took place in Gujarat in 2002.
They were consequently
arrested for the same by the police and upon investigation a chargesheet was filed.
However, many
evidences against both were tampered and no eyewitness was ready to give evidence
against them
because of their past criminal background. However, police couldn’t find the exact
CCTV footage for the
evidence consequently the matter reached the Court. Decide. In the light of the
passage choose the correct
option.
(a) The Court will discharge the case as no CCTV footage or eye witness was traced;
(b) The Court will not discharge the case as it is a criminal offence which
affected the entire State;
(c) The Court will hold G & B liable for they have committed offence against State
punishable under Section
146 & 147 of Indian Penal Code;
(d) The Court will not hold G & B liable as the investigation was to pull wool over
Court’s eyes.

. Page 19 of 36
78. The Court was also of the view that it cannot permit such cases to "meander
mindlessly in the corridors of
judicial system, sweeping away precious judicial time." Construing to the passage,
opt out the most
appropriate interpretation of this line.
(a) Appalling quality of investigation by the Court due to incompetence by
investigating agency, leads to
wasting Court’s time.
(b) Minimal or frivolous cases must not be reported to the Court and must be
decided by the police itself.
(c) Police incompetent behaviour towards any case.
(d) Courts torment of being incapable to decide the minimal cases in less time.
79. “The police seem to be still busy in filing supplementary chargesheets
therein.” In context of the passage,
find out what the line is trying to convey.
(a) Police generally files chargesheets after the investigation, but in this case
it has filed before the
investigation and merely acted incompetent.
(b) Incompetent behaviour of police while filing chargesheet.
(c) Incompetent behaviour of police during investigation.
(d) Chargesheet merely not necessary in open and close cases.
Passage (Q.80-Q.84): The Calcutta High Court explained the distinction in the
nature of relief between a
‘temporary injunction’ under Order XXXIX Rule 1 and an order for ‘attachment before
judgment’ under
Order XXXVIII Rule 5 of the Civil Procedure Code, 1908. Justice Moushumi
Bhattacharya observed that
while both provisions aim to protect the petitioner by preserving the disputed
property, their applicability
differs when it comes to the nature of property and the stage of proceedings in
question.
Under Order XXXIX Rule 1 a temporary relief is granted to the petitioner in the
event that an imminent risk
to the property in dispute in the suit is caused by the acts of the respondent. As
a result in a bid to preserve
the disputed property, the Court can pass any order as it deems fit in the nature
of a temporary injunction.
Whereas Order XXXVIII Rule 5 applies only at a later stage in a suit when the
petitioner seeks to execute
a decree. This section applies only in respect of orders which lend finality to the
suit and aims at preserving
the state of affairs after the interim stage in the suit is completed.
Further clarifying the difference in the ‘nature of property’ contemplated in the
aforementioned sections,
the Court observed, Under Order XXXIX Rule 1, the property sought to be preserved
is ‘property in dispute
in a suit’, whereas, it is the respondent’s property under Order XXXVIII Rule 5 -
the words used are ‘his
property’. The distinction reinforces the need to preserve the suit property till
final orders are passed in the
former and to secure the petitioner for facilitating execution of a decree in the
latter.
Aaratrika, ‘Distinction Between ‘Temporary Injunction’ And ‘Attachment Before
Judgement’ Under The
CPC: Calcutta High Court Explains’ (Live Law, 28 June 2021)
<https://www.livelaw.in/news#updates/temporary-injunction-and-attachment-before-
judgement-cpc-calcutta-high-court#176384?infinitescroll=1> as accessed on 27
September 2021
80. [A] Order XXXIX Rule 1 of the Civil Procedure Code, 1908 provides relief in the
form of an order for
‘attachment before judgment’ where the property in dispute in a suit is sought to
be preserved.
[B] Order XXXVIII Rule 5 of the Civil Procedure Code, 1908 provides relief in the
form of an order for
‘attachment before judgment’ where the property in dispute in a suit is sought to
be preserved at an
interim stage.
[C] Order XXXVIII Rule 5 of the Civil Procedure Code, 1908 provides relief in the
form of an order for
‘attachment before judgment’ where the petitioner seeks to execute a decree in
respect of orders which
lend finality to the suit.
[D] Order XXXVIII Rule 5 of the Civil Procedure Code, 1908 provides relief in the
form of a ‘temporary
injunction’ at a later stage in a suit and sought to preserve the respondent’s
property.
[E] Order XXXIX Rule 1 of the Civil Procedure Code, 1908 provides relief in the
form of a ‘temporary
injunction’ when there is an imminent risk to the property in dispute in the suit.
Which of the above statements are not true?
(a) Only E (b) A, B & D (c) B, C & D (d) Only D

. Page 20 of 36
81. Mr Jill rented some property for a period of 11 months on 01.07.2018,
comprising of a shop on the ground
floor with two rooms behind in a building, which was initially owned by Mr Joe. Mr
Joe on 07.07.2018
mortgaged with possession the entire building without the knowledge of Mr Jill but
as mortgage money was
not paid, a suit for sale of the mortgaged property was instituted. In the suit a
final decree for sale of the
mortgaged property was passed which was purchased by Ms Kat. Ms Kat then on
02.01.2019
dispossessed Mr Jill of the property and gave orders to strike down the building
and build a shopping
complex in its place. Mr Jill instituted a regular civil suit for his dispossession
from the property and a
temporary injunction against striking down the building. Decide:
(a) A relief under Order XXXVIII Rule 5 shall be requested for the attachment of
the property before the
judgment to stay in possession of the shop and the rooms.
(b) A relief under Order XXXIX Rule 1 shall be requested for a temporary injunction
against the
dispossession and striking down of the building.
(c) A relief under Order XXXVIII Rule 5 shall be requested as the same is the most
accessible option at
the interim stage of a suit.
(d) A relief under Order XXXIX Rule 1 shall be requested as the court will be able
to grant the relief as
soon as the decree is finalized.
82. Mr Kay owned some property situated at village Newari, measuring 8 Acres. After
his demise, his nephew,
Mr Riyas claimed to be the heir and legal representative of late Mr Kay. He started
sowing crops in the field
and his name was also substituted in place of Mr Kay in the revenue records. A
civil suit was filed by him
when he came to know that Mr Raj claimed to be the lawful owner of the land by
virtue of a sale deed
executed by Mr Kay. The court found that the sale deed was never actually
registered at the office of the
Sub- registrar and therefore passed an interim order in favour of Mr Riyas.
Meanwhile, Mr Raj finalised a
sale agreement of the property with a company named Leiweing Co. What is the remedy
available with Mr
Riyas in order to execute the decree passed by the court?
(a) Remedy under Order XXXVIII Rule 5 is available at this stage as the interim
order has been passed
and the same can be executed by the attachment of the property.
(b) Remedy under Order XXXIX Rule 1 is available as an order of injunction can be
used against the
finalization of the sale agreement of the land in dispute.
(c) Remedy under Order XXXVIII Rule 5 is available as it is the only available
method for preserving the
land in dispute.
(d) Remedy under Order XXXIX Rule 1 is available as it allows an injunction even at
a later stage where
the decree has been finalized.
83. Hitesh lent 20 lakh rupees to Riya, for construction of a residential building
at Ramnagar Colony, Chitoor.
She had agreed to repay the amount borrowed with interest at 24% per annum and had
duly executed the
promissory note in favour of Hitesh. However, despite the oral requests, she failed
to repay the due amount.
Hitesh had filed a preliminary suit to recover the money when he came to know that
Riya was trying to sell
the house and move her family to some other place only with an intention to defraud
him. He submitted
before the court that if she succeeds in alienating the only property, he will not
be in a position to recover
his debts even if the decree is passed in his favour. Hence, he filed an
application seeking attachment
before judgment of Riya’s property. Decide.
(a) The requested relief cannot be granted because though the debt has not been
repaid, Riya reserves
absolute right to alienate the property because of her ownership over the property.
(b) The requested relief can be granted as the same can be used as soon as the
court passes an order in
favour of Hitesh.
(c) The requested relief cannot be granted because at a stage where no order has
been passed temporary
injunction is the available remedy.
(d) The requested relief can be granted as the property will be attached by the
court and will be disposed
of as per the outcome of dispute between the two parties.

. Page 21 of 36
84. Jitesh, a rich businessman, made a will in the name of his two sons and a
daughter dividing the property
among them. After his demise, the elder son Pappu took care of all the business and
the properties and
used the same to cover the education expenses of his siblings abroad. After their
return, due to a rift
between them, each decided to take their share in the property. Piya claimed a
family farmhouse
bequeathed to her, however, Pappu rejected her claim as it was mortgaged with the
bank to cover the
expenses of her education. A preliminary suit was filed by Piya in the court
seeking temporary injunction
on enforcement of the security in the form of mortgaged farmhouse property to the
Bank. Whether the court
should grant a temporary injunction or not?
(a) The Court cannot grant a temporary injunction as the farmhouse was mortgaged to
cover the education
expenses of Piya and therefore no cause of action arises in this case.
(b) The Court can grant a temporary injunction as the property could not have been
mortgaged without the
authority of the actual owner of the property.
(c) The Court cannot grant a temporary injunction as at a stage where enforcement
is already taking place,
attachment of the property before judgment is the more suitable remedy.
(d) The Court can grant a temporary injunction because at an interim stage without
any order or decree,
temporary injunction is the requisite relief for preserving the disputed property.
Passage (Q.85-Q.89): Bihar’s new law seeks to set up an armed police force in the
state, named the Bihar
Special Armed Police (BSAP). It will be responsible to maintain public order,
combat extremism, ensure
security of specified establishments, etc.
The most controversial provision of the Act is section 7, which provides that any
Special Armed Police
Officer entrusted with securing an establishment may, without a magistrate’s order
or warrant, arrest any
person who causes, threatens to cause, or attempts to cause harm to an employee of
the establishment
or the officer himself.Further, any person can be arrested either based on
“reasonable suspicion” or if they
commit or attempt to commit a cognizable offence.
Section 8 provides for any Special Armed Police Officer to detain a person even on
the basis of “reason to
believe” that an offence has been or is being committed. He would have the right to
search the offender
and his or her belongings and arrest them.
Article 312 establishes the Indian Police Service. Therefore, the state governments
are empowered to
maintain the police force for the sake of “public order” and that only, hence the
role of the police cannot be
strengthened in administration and management of the state.
The Act places an undue burden on ordinary individuals, who could end up being
subjected to extreme
abuse of power. The burden of proving innocence is also laid up upon the accused
which is
unconstitutional.
This is why Article 21 of the Constitution protects the right to life and personal
liberty of every individual
and reminds the state and people of the requirement to follow procedures
established by law.
No court shall take cognizance of any offence under this Act where the accused
person is a Special Armed
Police Officer except on a report in writing of the facts constituting such offence
and with the previous
sanction of an officer authorized by the Government in this behalf.
https://www.theleaflet.in/is-bihar-a-police-state-in-the-making/
85. Adit is a professor in Bihar university, he has been very critical of the Bihar
government on issues related
to the tribal natives and local indigenous groups and his criticism has often
gained traction and troubled
those in power. He often organises protests and seminars to highlight his cause. In
2021 the BSAP has
arrested him on the grounds of reasonable suspicion of having links with the tribal
Maoists and insurgents.
Was this action of the BSAP correct? Answer as per the views of the author.
(a) No, being supportive of a cause and voicing protest against the government on
public forums cannot
indicate reasonable suspicion.
(b) Yes, his support and sympathy for these groups qualifies as reasonable
suspicion.
(c) No, he has been critical of the state government and his arrest is symbolic to
stifling of dissent.
(d) Yes, the law states that the police can arrest anyone without warrant based on
the charges of
reasonable suspicion.

. Page 22 of 36
86. Ashok is an BSAP officer and he has been found accepting bribes and extorting
businesses asking for
money and other favours, agitated shopkeepers and businessmen approach the courts
against him, the
government refuses the to sanction his report. Under the above mentioned law what
would the courts do?
(a) The courts would continue the case against him as these charges are covered
under the Indian penal
code.
(b) The courts will not be able to proceed as they require government sanction to
prosecute BSAP officials.
(c) The courts are constitutional bodies hence they can continue with the
prosecution of Ashok.
(d) Courts will not be able to prosecute him because he has absolute legal immunity
as he is a BSAP
officer.
87. The government of Bihar has decided that several districts in the state which
are troubled by Naxals and
Maoists would only and completely be administered by the officers of the BSAP. The
officers would be
responsible for everything from public order to sanitation and providing
educational services. This decision
of the government has been challenged in court. Answer in accordance with the
passage.
(a) The court does not have the jurisdiction to rule on the matters relating to
BSAP according to the above#mentioned law.
(b) The courts would rule in favour of the petitioner as the role of the police is
to establish public order and
it cannot be involved in the administration.
(c) Since these areas are infested with Naxals and Maoists, the courts will allow
the BSAP officer to
administer these districts.
(d) Involvement in districts is not under the jurisdiction of BSAP.
88. Does this law passed by Bihar government go against the Article 21 of the
Indian constitution?
(a) Yes, this law puts undue burden on the accused which is a violation of rights.
(b) No, the restrictions and conditions that the law subjects the citizens to are
reasonable in nature.
(c) Yes, this law is against Article 21 of the constitution as this violates the
principles personal liberty and
does not follow the procedure established by law.
(d) No, the law has been passed to curb the problems of extremism, extortion etc.
the law was an essential
requirement for the state.
89. Akhil is a member of the opposition party in Bihar and is agitated by the
governments’ alleged
misgovernance. He often is seen raising his opposition on public platforms, the
government is disturbed
and claims that his opposition stems out of his relations with members of various
gangs. In the month of
March 2021 he was arrested was BSAP on the charges of disturbing the peace of the
state and having
links with members of extortionary gangs. Moreover he is asked by the BSAP officers
to produce evidence
against his will. He says his rights have been violated. Is that true as per the
passage?
(a) His rights have not been violated as the procedure established by law while
arresting him was followed.
(b) His rights have been violated as he has been asked to produce evidence against
himself.
(c) The purpose of the BSAP is to fight these extortionary gangs, Akhil’s arrest
and the responsibility of
burden of proof on him are legal.
(d) He has been forced against his will to produce evidence, this violates his
rights and places undue
burden on him. His rights have indeed been violated as per the constitution.

. Page 23 of 36
Passage (Q.90-Q.93): Fraudulent or dishonest inducement is an essential ingredient
under penal provision
and a person who dishonestly induced any person to deliver any property is liable
for the offence of
cheating, said the Supreme Court Monday. A bench comprising Justices D Y
Chandrachud and M R Shah
said as per section 415 of Indian Penal Code, which deals with cheating, a person
who was induced should
be intentionally induced to deliver any property to any person. Thus, a fraudulent
or dishonest inducement
is an essential ingredient of the offence under Section 415 IPC.
A person who dishonestly induced any person to deliver any property is liable for
the offence of cheating,
the bench said. The judgement came on an appeal filed by a woman Archana Rana
challenging Allahabad
High Court order dismissing her plea for quashing of charge sheet under sections
419, 420, 323
(punishment for voluntary causing hurt) 504 and 506 (criminal intimidation) of the
Indian Penal Code.
90. Rekha and Nikhil are working in a company. Their company comes out with a job
vacancy and they receive
thousands of applicants for that post. One of such applicant Suresh approaches
Rekha in lieu of the job
opportunity. Rekha promised Suresh for the job if he gave her Rs. 5 Lakhs. Suresh
gives Rs. 5 Lakhs to
Rekha in the presence of Nikhil. The results were announced for the job but Suresh
wasn’t selected for the
same. He filed a case against Rekha and Nikhil under various provisions of IPC
including S. 415. Decide
if they are liable for the offence of cheating.
(a) They are not liable for the offence since they did not commit the offence of
cheating.
(b) They are liable for the offence since they promised to give Suresh the job and
took money in lieu of the
same.
(c) She is liable for the offence since she induced Suresh to deliver the money.
(d) She is not liable for the offence since she did not commit the offence of
cheating.
91. Riya owns a diamond jewellery store. Rohan goes into the store to buy a diamond
ring. Riya shows him
sample rings and promises that her diamonds are of the best quality in the whole
city. He buys a diamond
ring from her for Rs. 1 lakh. He later goes to another store to get the purity of
the diamond checked. The
shopkeeper tells him that although the diamond was pure and real, he could find
better quality diamonds
at other jewelers. He files a complaint against Riya u/s S. 415 for cheating.
Decide if Riya has committed
an offence.
(a) She has committed an offence of cheating since she deceived him into buying the
diamonds.
(b) She did not commit the offence since she did not induce him fraudulently.
(c) She would have committed the offence if she had dishonestly induced him into
buying diamond.
(d) She has committed the offence since she cheated him by promising to deliver
best quality diamond.
92. Rajat and Rohan enter into an agreement for delivery of Teak wood chairs. Rajat
took Rs. 5 lakhs from
Rohan for delivery of the same. Rajat promised to deliver the Teak wood chairs
however on the date of
delivery his warehouse caught fire and he could not deliver the chairs despite all
efforts. Rohan files a
complaint against Rajat u/s 415 of IPC. Decide if Rajat if liable for cheating.
(a) He is not liable for the offence since his warehouse caught fire due to which
delivery could not be made.
(b) He is liable for offence since he promised to deliver the chairs and took money
for the same.
(c) He is not liable for the offence of cheating as he had the intention to deliver
the Teak wood chairs to
Rajat as promised.
(d) He is not liable for the offence since he did not induce Rajat to pay him
money.
93. Richa and Swati enter into a contract to renovate Swati’s house. Richa promised
Swati to deliver the house
as per her specifications. Swati instructed Richa to renovate her cupboard with
Teak wood only, Richa
charges extra money for the same. Richa finished the renovation work and delivered
to same on time.
Swati loved Richa’s renovation work and gave her the full payment as per the
contract for her services. A
few months later Swati found that the woodwork done on her cupboard was not of Teak
wood. She files a
case against Richa for cheating. Will Swati be successful in her claim against
Richa?
(a) She will be liable for offence of cheating since she charged money from Swati.
(b) She is not liable for the offence since she completed the work as per Swati
specification and Swati
loved her work.
(c) She will not liable for offence if she had delivered the services as per
specifications.
(d) She is liable for the offence since she committed fraud in delivering her
services and induced delivery
of money as a result.

. Page 24 of 36
Passage (Q.94-Q.99): Rape is a crime, which has a devastating effect on the
survivors; it has been
described as a “beginning of a nightmare”. The aftershocks include depression,
fear, guilt-complex,
suicidal-action, diminished sexual interest. etc., “one becomes afraid of’ writes a
victim, “half the human
race”.
Referring to the pitiable condition of women in society Mr. Justice S. Ahmad
observed that “unfortunately,
a woman in our country, belongs to a class or group of society who are in a
disadvantaged position on
account of several social barriers and impediments and have therefore, been victims
of tyranny at the
hands of men with whom they, unfortunately, under the Constitution “enjoy, equal
status”. “Women also
have the right to life and liberty; they also have the right to be respected and
treated as equal citizens.
Their honour and dignity cannot be touched or violated. They also have the right to
lead an honorable and
peaceful life”.
Rape is a stigma which exists in the society from a long time. The dictionary
meaning of word rape is “the
ravishing or violation of a woman.” She is traumatized after the event; it is very
difficult for a woman to
come out of this trauma. Rape in India is a cognizable offence.
There are many provisions in various Acts. The word rape is legally defined u/s 375
of Indian Penal Code,
1860. It defines the rape and also prescribes its punishment. Whenever a man
penetrates and does sexual
intercourse with a woman without her consent or will it amounts to rape.
Penetration here means that only
a slightest of the touch of penis to vagina amounts to rape, unruptured hymen of
woman does not prove
that rape was not committed. Further the consent of a minor is not valid.
The Supreme Court has expressed strong disapproval of courts casting a stigma on
the victim’s character,
but stigmatization continues, leading to acquittals. Setting aside one acquittal by
a Ludhiana court that
labelled the victim with ‘loose character’ while interpreting her consent to sex,
the Supreme Court in a 1996
judgment said, “The trial court interpreted that the victim was habituated to
sexual intercourse just because
the speculum the doctor used entered her vagina easily and hence she was of loose
character. These
observations lack sobriety expected of a judge. No stigma should be cast against a
victim of sex crime who
is on trial”.
94. Madhur meets Anisha at a party where both of them exchange pleasantries. After
a while, when both of
them are alone and Madhur imposes himself on Anisha and forcefully engages into a
sexual intercourse
even when she protests for the same. Decide.
(a) This shall amount to rape due to absence of consent by Anisha.
(b) This shall not amount to rape due to an exchange of pleasantries.
(c) This shall amount to rape due to a sexual intercourse between them.
(d) This shall not amount to rape but shall fall under assault.
95. Disha and Harsh have known each other for a long time and have engaged in
sexual intercourse at various
instances. All such sexual intercourses involved the consent of the individuals.
However, at one instance,
both the individuals had a sexual intercourse for which Disha did not consent to.
Whether this would amount
to the offence of Rape. Decide.
(a) This shall not amount to rape as they had sexual intercourse at earlier
occasions for which she
consented to.
(b) This shall not amount to rape as consent is vague in the last case.
(c) This shall amount to rape as there was no consent and previous consent is not
relevant.
(d) This shall not amount to rape as the consent was implied.
96. Apoorva went to get together of her college friends. Ramesh was also there and
both of them exchanged
a pleasant conversation. They decided to engage in a sexual activity. Apoorva
consented to one particular
type of sexual activity. However, when they got to the point of intercourse, she
withdrew her consent. Would
this amount to the offence of Rape? Decide.
(a) No, as she consented for the sexual activity before the beginning of the
intercourse.
(b) No, as such withdrawal at such a later stage would not amount to withdrawal.
(c) Yes, as she withdrew her consent for that particular sexual activity.
(d) Yes, as she consented for only one type of sexual activity.

. Page 25 of 36
97. Rahul engaged in a sexual intercourse with Simran who was 15 years old. The
parents of Simran decide
to file a case against Rahul for the offence of rape. However, he contends that the
intercourse had the
consent of Simran therefore cannot be categorized as rape. Decide.
(a) Rahul is not liable as the act was consented by Simran.
(b) Rahul is not liable as the parents cannot file the case but only Simran can.
(c) Rahul is liable as the consent of Simran cannot be considered a valid consent.
(d) Rahul is liable as the parents of Simran can decide for their daughter.
98. Choose an appropriate reason of the following for the lack of gender neutral
laws:
(a) Women are the ones who are the victims of such heinous crime.
(b) Women cannot commit rape due to biological reasons.
(c) Men are the aggressors in the majority of the cases.
(d) Women are the disadvantaged group due to stunted societal structure so they
need more protection.
99. Robin is a police officer who met Malaika at a party. After the party, he told
her that if she did not engage
into a sexual intercourse with him then he would put her in jail and harass the
family members on account
of the powers he possess through his profession. They both enter into a sexual
activity wherein she does
not protest or explicitly resist. Decide.
(a) Robin shall be liable as the consent was not a free consent.
(b) Robin shall not be liable as there was no explicit resistance.
(c) Robin shall not liable due the existence of inherent consent.
(d) Robin shall be liable for coercion but not the offence of rape.
Passage (Q.100-Q.105): The Supreme Court observed that a caretaker/servant can
never acquire an
interest in the property irrespective of his long possession. For no one acquires
ownership in the property
if he/ she is allowed to stay gratuitously, unless the property is transferred onto
his name by the way of a
will.
In this case, the plaintiff sought a declaration that he is a lawful occupier as
caretaker/servant of the sole
owner of the suit property. He also sought a permanent injunction restraining the
defendant to disturb or
evict his peaceful possession of the suit property.
In appeal, the bench observed thus:
After we heard counsel for both the parties and taking into consideration the
material on record, in our
considered view, the Trial Court has committed a manifest error in appreciating the
pleadings on record
from the plaint filed at the instance of respondent no.1-plaintiff who as a
caretaker/servant can never
acquire an interest in the property irrespective of his long possession and the
caretaker/servant has to give
possession forthwith on demand and so far as the plea of adverse possession is
concerned as it lacks
material particulars and the plaint does not disclose the cause of action for the
institution of the suit.
Allowing the appeal, the court rejected the plaint holding that the instant
proceeding is not sustainable on
the first principles of law by the virtue of which property devolves upon immediate
kin of a person.
100. Jamnadas Haveli had been the home of the Garewal household for about 80 years,
including the five
brothers, their wives, a dozen children and their caretaker for 50 years, Hariram.
In a family expedition to
Shirdi, unfortunately the whole of the Garewals had met with an accident and had
died, leaving the haveli
in the care of Hariram. Distraught with this news, Hariram became a recluse and
continued to live in the
house for the next 8 years minding his own business, living off the pension of the
eldest brother of Garewal
family. After 8 years the tax collection authorities came to collect the long
pending property tax dues when
they finally found out about the arrangement. Due to the dues amounting to a
whopping sum of more than
5 figures, Hariram is now being prosecuted by the authorities for not paying the
taxes despite being the
lawful owner of the property. Can an action be brought against him?
(a) No, as a caretaker cannot be made the lawful owner of the property in his
possession.
(b) No, as mere occupation does not amount to possession.
(c) Yes, as he has been availing the facilities of the house for years now, hence
acquired interest.
(d) No, as he was merely residing there, and not claiming possession.

. Page 26 of 36
101. Notwithstanding other facts as above, had Hariram been a far off relative of
the Garewal family, and he
had been instated as the caretaker of the property and had been living with the
family for over a period of
15 years, could he qualify as the subsequent owner of the property?
(a) Yes, as the doctrine of adverse possession a person who is not the original
owner becomes the owner
because of the fact that he has been in possession of the property for a minimum of
12-years.
(b) Yes, as he was the lawful kin of the Garewals and could claim the ownership by
way of right.
(c) No, as the lack of any indication towards the same disqualifies him from
claiming ownership.
(d) No he has not acquired interest in the property in spite of being in long
possession;
102. In the above case, notwithstanding other facts, had the caretaker been made
the legal heir to the property
by the way of executing a will in case of the previous owners’ death, would Hariram
be then entitled to
claim the property as his?
(a) Yes, as he would be considered the owner in law of the property.
(b) No, as under no circumstances can a caretaker become the owner of the property.
(c) No, as the provision for making a caretaker as the owner of the property does
not exist in law.
(d) Yes, as the caretaker has now been validly made the possessor of the property.
103. Shravan was the son of Hira, a business magnate, and was a deadbeat unemployed
bachelor. Fearing for
his son’s reputation and his own, Hira made Shravan a land baron and decided to let
him oversee their
company properties. After 5 years had passed, Hira’s demise led to Shravan stepping
in and taking care
of the company and the household duties. However, owing to his sudden demise, the
absence of a will to
that effect led to question arsing on the ownership of the household properties and
the company vis-à-vis
Shravan. What would be the most plausible reasoning with regards to the same?
(a) Shravan was his only legal heir and both would devolve unto him.
(b) Shravan shall only get the ownership of the company.
(c) Shravan was the property’s caretaker and thus would not be liable for any
ownership.
(d) Shravan would be liable to only get the company ownership as he was merely the
lands caretaker in
this case.
104. Sitaram was the caretaker of the time-share property of three families,
appointed in unison. After 10 years
of enjoying the property thoroughly they decided to sell the same off and the same
was communicated to
Sitaram. Sitaram, who had been living in the same for the last 10 years, requested
them to not do so as he
would lose out on a place to live and would have to look for a new job in his
twilight years. Taking pity on
him the owners decided to not sell the same and gave him the place to live. Can he
be considered the
owner of the property in this case?
(a) Yes, as he was the lawful occupier of the property in question; ;
(b) No, as there was no contract for the sale of the property.
(c) Yes, as he is the immediate kin of the family;
(d) No, as he was gratuitously staying onto the property.
105. In a case of illegal occupation of property by a tenant, the facts were as
follows; Sridevi was a tenant at
Dharam Apartments, where one Mr. Anil was the landlord. She used to pay him 20000/-
rent per month in
exchange for accommodation at his apartment block and had been residing there for
20 years.
Unfortunately, 6 months prior to the action in this case, mr. Anil had died due to
some underlying illness.
Post which she discontinued depositing money into Mr. Anil’s account but continued
staying there. Only
when Mr. Anil’s heirs had come to see the property that they were willed did they
notice that she was still
there and asked her to leave the premises. However, she claimed interest in the
property due to long
possession. Decide whether she had acquired ownership to the same. ?
(a) No, as she was merely a tenant only up to his lifetime and should have vacated
the apartment after his
demise.
(b) Yes, as having lived there for about 20 years, she was entitled to the property
on account of being a tenant.
(c) Yes, as having lived there for about 20 years, she had become the legal
caretaker of the property and cannot
be asked to vacate.
(d) No one does not acquire interest in the property by virtue of long possession.

. Page 27 of 36
SECTION - D: LOGICAL REASONING
Passage (Q.106-Q.110): Almost all the antiquities of ancient India are of a
religious character, or were at
least made for religious objective. Secular art surely exists, for literature shows
that monarchs lived in
luxurious palaces, adorned with lovely murals and sculpture, albeit many of them
have perished. Much has
been said and written about Indian art since. From that time to this, most
authorities on the subject, have
stressed the religious and mystical aspects of Indian art. They admitted the
realism and earthiness of the
earliest sculpture.
It is the full and active life of the times which is chiefly reflected in the art
of ancient India. In all these places,
there is a horror vacui and an intense vitality, which remind us rather of this
world than the next and suggest
to us the warm bustle of the Indian city and the turbulent population of the Indian
forest.
Gothic architecture and sculpture are vertical. Spire and arch point upwards, and
as the style develops, the
spire becomes taller and the arch more pointed. The Christ, saints and angels of
the Middle Ages in Europe
are often disproportionately tall, and their tallness is accentuated by long
garments reaching to the ankles.
Their poses are generally restful, and they rarely smile. Medieval European art was
truly religious; its
conventions seem to have been deliberately designed to lead the worshipper's
thoughts away from the
world of flesh to the things of the spirit. Much of it was the work of pious monks,
or of men with deep
religious vocations.
The tendency of Indian art is diametrically opposite to that of medieval Europe.
The temple towers, though
tall, are solidly based on earth. Gods are young and handsome; their bodies are
rounded and well
nourished. The ideal type is not abnormally tall, but rather short and stocky.
Occasionally they are depicted
as grim or wrathful, but generally they smile, and sorrow is rarely portrayed.
Ancient India's religious art differs strikingly from her religious literature. The
latter is the work of men with
vocations, brahmans, monks and ascetics. The former came chiefly from the hands of
secular craftsmen,
who, though they worked according to priestly instructions and increasingly rigid
iconographical rules, loved
the world they knew, with an intensity which is usually to be seen behind the
religious forms in which they
expressed themselves. In our opinion, the usual inspiration of Indian art is not so
much a ceaseless quest
for the Absolute but as a delight in the world as the artist found it, a sensual
vitality, and a feeling of growth
and movement as regular and organic as the growth of living things upon earth.
106. The Indian art is full of vitality because
(a) Indian art follows a tradition of realism.
(b) The sculptors who carved them were very much men of this world.
(c) People understood and appreciated only what they were familiar with.
(d) This is how the sculptors imagined the ascetics to be.
107. It can be inferred that the phrase ‘horror vacui’ as used in the passage
means:
(a) The art of ancient India manifests a horrible vacuum.
(b) The art of ancient India manifests a dislike for nothingness.
(c) The art of ancient India manifests a passion for horror.
(d) The art of ancient India manifests a dislike for leaving empty spaces.
108. According to the passage, which of the following is the difference between
European and Indian art?
(a) The temple towers, though tall in both, are firmly based on the earth in the
latter.
(b) Gods of the European religion are depicted as tall and those of the Indian
religion are short and stocky.
(c) Religiosity is more evident in the European art than in the Indian art.
(d) Sorrow is seldom portrayed in the European art, whereas the Indian art rarely
depicts smiling faces.

. Page 28 of 36
109. Which of the following are true of ancient Indian religious art and
literature?
(1) The arts were the work of craftsmen while the literature was the work of monks
and Brahmins.
(2) The art was full of earthy sensuality, while not deviating from canonical rules
set by religious literature.
(3) The Indian religious art was solely devoted to afterlife.
(4) Indian temple sculptures are purely religious and mystical.
(a) (1) and (2) (b) (3) and (4) (d) (1) and (4) (d) (2) and (3)
110. The passage…
(a) examines the views of different people regarding art.
(b) compares and contrasts contemporary Indian and European art.
(c) discusses art and sculpture of ancient India.
(d) studies mysticism and religiosity in Indian art and sculpture.
Passage (Q.111-Q.115): Read the passage below and answer the questions that follow:
Some scholars believe the institutional mechanisms affect the ICC’s ability to
prosecute. For example, the
limited institutional powers of the ICC, such as the lack of an enforcement
mechanism, affect its ability to
prosecute crimes. It is difficult to prosecute when it is impossible to enforce
arrest warrants and the states
do not comply with the ICC. The inability of the ICC to pressure these states into
compliance also shows
its limited capacity. However, if the states willingly comply, it is easier for the
ICC to investigate and achieve
the Confirmation of Charges. Clarke contends that the ICC has a big reliance on
states, which affects when
and where the ICC can come in and prosecute. In her analysis, the ICC only steps in
when the national
government is unwilling or unable to prosecute.
This would explain why the ICC has chosen to prosecute certain situations over
others. However, it does
not explain why certain cases achieve the Confirmation of Charges and others do
not. These scholars set
a precedent for testing whether institutional mechanisms play a role in ICC
investigations and prosecutions.
However, I believe that these arguments are lacking. Since the lack of an
enforcement mechanism and the
limited jurisdiction are true for all states, there must be another reason why some
cases are able to achieve
the Confirmation of Charges and others are not.
Scholars contend that the effectiveness of the ICC is based around power politics
and domestic political
calculations. Studies suggest that when domestic punishment is unlikely, ICC
involvement will affect
leaders’ incentives to end their conflicts, significantly decreasing the
probability of termination. At first
glance, this seems counterintuitive. However, when domestic punishment is probable,
leaders will be
preoccupied with this more severe threat, and ICC investigations will have little
impact on their strategic
decision making. While the ICC claims that most of the cases were self-referrals,
many of the “self-referrals”
were actually for the leaders to stay in power and remove political enemies. Very
rarely were they actual
cases in need of justice.
111. What would be needed to achieve Confirmation of Charges by the ICC?
(a) No states have a proper enforcement mechanism when it comes to international
law, which has made
the ICC almost toothless.
(b) ICC requires a proper enforcement mechanism as soon as possible; this would
ensure that the verdict
of the ICC gets carried out in the respective countries to the fullest extent.
(c) The States’ willingness to comply with the decision of ICC as to when it can
prosecute a case or not.
(d) D. The States’ isolation in matters of the ICC’s decisions of investigations.
112. In the passage, a few scholars have come up with the problems faced by ICC,
and the author has given
his take on those problems. Which of the following analogies is in line with the
author's line of thinking?
1. Experts - The teacher can't resort to corporal punishment in a classroom, nor do
they control how the
students' families are. That is the reason for few students scoring well and few
scoring average and
sub-average marks.
Author – All the students have homes to go to; they have two hands, feet, and one
brain. You can't mete
out corporal punishment to anyone – good or bad students, yet why is there still
the division of good and
bad students? It is because few students are serious and few are not.

. Page 29 of 36
2. Experts – People don't listen to the government. Even after saying so many times
that masks are
necessary to safeguard ourselves from COVID-19, people have ventured out without
masks. No
wonder few countries where the rules were followed to the tee have faced the
pandemic well, and the
majority who took it carelessly had to struggle.
Author – The problem is that the government was not severe enough and didn't ensure
compliance with
the safeguard norms by the people. This is the real reason that some countries did
well in tackling the
pandemic and some didn't.
(a) Only 1 (b) Only 2 (c) Both 1 and 2 (d) Neither 1 nor 2
113. Which of the following, if true, would seriously undermine the author’s
argument?
(a) ICC has investigated more international cases than any other organization and
this is a fact that has
been well-researched out.
(b) There are very few cases that warrant the involvement of ICC in domestic
matters. Normally it has been
seen that the internal judicial system of a country takes care of the issues
internally.
(c) Both (A) & (B)
(d) Neither (A) nor (B)
114. The author’s statement that “if the states willingly comply, it is easier for
the ICC to investigate and achieve
the Confirmation of Charges”:
(a) Forms premise of the above passage.
(b) Forms conclusion of the above passage.
(c) Forms assumption of the author to the above passage.
(d) None of the above.
115. The statement “ICC is a useless organization in dispute resolution” is:
(a) Probably True (b) Definitely True (c) Probably False (d) Definitely False
Passage (Q.116-Q.120): J F C Fuller did not invent the tank.
That distinction should probably fall to E L de Mole, an Australian who approached
the British war office in
1912 with a design that was — in the words of historians Kenneth Macksey and John
Batchelor — “so
convincingly similar to those which finally went into service that one wonders why
it was never adopted
from the outset”. But when the British army eventually introduced the tank, it was
J F C Fuller, chief staff
officer of what would later become the tank corps, who understood what to do with
it.
Late in 1917, after almost 400 British tanks had, with modest success, lumbered
across the German lines
at the battle of Cambrai, Fuller applied his radical streak to the problem of using
the tank effectively.
A new and much faster tank, the Medium D, could travel 200 miles at a speed of 20
miles per hour. Fuller
proposed that these tanks would attack the German army’s brain — the string of
German headquarters
miles behind the front line.
A Medium D could roll across the trenches and be on the German command posts in an
hour; Fuller’s
attack would come from nowhere. Air support would disrupt German road and rail
travel.
“Bad news confuses, confusion stimulates panic,” wrote Fuller. His idea was dubbed
Plan 1919. By striking
suddenly at the German command, Plan 1919 would cause the German army to
disintegrate. It would,
Fuller declared, be “the winning of the war in a single battle”.
His astonishing idea became “the most famous unused plan in military history”,
according to his biographer
Brian Holden Reid. But, of course, that is not entirely true. It was used to great
effect, in 1940 — by the
Germans. J F C Fuller had invented blitzkrieg.
The story might be a historical curiosity, had echoes of it not been repeated so
frequently since the British
army stuffed Fuller’s plans for blitzkrieg into a desk drawer. Organisations from
newspapers to oil majors
to computing giants have persistently struggled to embrace new technological
opportunities, or recognise
new technological threats, even when the threats are mortal or the opportunities
are golden.

. Page 30 of 36
In 1970, the photocopying giant Xerox established the Palo Alto Research Center, or
Parc. Xerox Parc
then developed the world’s first personal computer, with a graphical user
interface, windows, icons and a
mouse. Bill Gates of Microsoft and Steve Jobs of Apple observed developments at
Xerox Parc with great
interest. Xerox still makes photocopiers.
116. J F C Fuller did not invent the tank. Through the statement, the author does
which of the following?
(a) The author clears the disseminated misconception
(b) The author categorically puts aside a common perception.
(c) The author draws the attention of the readers towards the truth.
(d) The author expresses regret for the misconception around who invented the tank.
117. Which of the following situation is parallel to the reasoning provided in the
passage?
(a) IBM — the giant of mainframe computing. IBM is a survivor. It predates the
digital computer by more
than three decades. While the performance of computers was being revolutionised by
the
semiconductor, the integrated circuit, the hard drive and the compiler, IBM
maintained a dominant
position without breaking stride.
(b) Unsuccessful organisations stick to their once-triumphant strategies, even as
the world changes around
them.
(c) When Steve Jobs visited Xerox Parc in 1979, and saw a windows-and-mouse
interface for the first
time, he couldn’t contain himself, according to an article by Malcolm Gladwell.
“Why aren’t you doing
anything with this?” he yelled. “This is the greatest thing. This is
revolutionary!”
(d) In 1999, Sony launched the “Memory Stick Walkman”, one of the world’s first
digital music players.
Sony was armed with the iconic Walkman brand, some of the world’s best consumer
electronics
engineers and the talent-soaked Sony-BMG music label. The Memory Stick Walkman went
nowhere
and, two years later, it was eclipsed by a similar product that transformed the
fortunes of a struggling
Apple: the iPod.
118. The arguments presented in the passage can be the answer to which of the
following concern raised by
the author in question form?
(a) Why do ideas slip out of the hands of the executors and fall into the hands of
the innovators?
(b) What were the reasons behind J F C Fuller’s unused radical ideas?
(c) Why do some ideas slip out of the grasp of incumbents, then thrive in the hands
of upstarts?
(d) Why do brilliant ideas never find the daylight and are lost in the darkness?
119. The historical events shared by the author are
(a) anecdotes to show how people miss out on golden opportunities.
(b) to present that a brilliant idea lost by one becomes a golden goose for the
other.
(c) an illustration to prove the mistakes made by Xerox Parc.
(d) to show how Germany used the radical idea by JFC Fuller.
120. What might be the possible explanations to why organisations squander good
ideas?
1. Firms fail because they stick to idea(s) that made them successful,”
2. New technologies are flawed or under-developed at first, so do not appeal to
their conceptualizers.
(a) Only 1 (b) Only 2 (c) Both 1 and 2. (d) Neither 1 nor 2.

. Page 31 of 36
Passage (Q.121-Q.125): In the days when Jean Piaget and Sigmund Freud dominated
thinking about child
development, small children were thought to be irrational, incoherent and
solipsistic in their thinking, and
both easily distractible and unfocused in their awareness of the world. Recent work
in developmental
psychology offers a sharply contrasted picture. Children are unconsciously the most
rational beings on
earth, brilliantly drawing accurate conclusions from data, performing complex
statistical analyses, and
doing clever experiments. And not only does empirical work reveal this about babies
and small children,
but what is, thus, revealed throws light on some of philosophy’s more intriguing
questions about knowledge,
the self, other minds and the basis of morality.
Such are the claims made by philosopher and developmental psychologist Alison
Gopnik in this fascinating
account of the growth of child minds. Gopnik describes how imagination contributes
to the vast amount of
knowledge that children acquire in their first few years. Accumulated knowledge
allows children to think of
alternative ways that the world could be, which in turn helps them to construct
mental maps of the causal
relationships that govern and explain how things work. Imagination also aids them
in forming ideas about
how other people think and why they act as they do. Many children have ‘imaginary
friends’; their ability to
understand others and to change themselves is aided by the possibilities for
exploring alternatives that
such play affords.
Studies of child development also suggest insights into consciousness, one of
philosophy’s most
recalcitrant mysteries. Children appear to have a far more vivid awareness of the
world around them than
adults do, Gopnik, reports, because an adult’s ‘spotlight awareness’ that enables
concentration on specific
features of an environment involves losing the ‘lantern awareness’ that brings the
whole environment to
the forefront of attention. This childhood form of awareness is likened by Gopnik
to how adults feel when
they visit a foreign country; they focus less on particulars and experience
everything more globally because
so much is unfamiliar. Whereas children have a more intense lantern awareness, they
also have less inner
consciousness of the kind that helps manufacture a distinctive sense of self, that
autobiographical centre
of memory and planning which is the ‘me’ in all experience. That explains why they
have less command of
their behaviour, and less sense of the future.
Gopnik’s affectionate and sympathetic enjoyment of the way children think in their
first five years is manifest
throughout her book, but so too is her sensitivity to the deeper philosophical
implications of what their way
of thinking can teach us. The result is absorbing and educative.
121. This passage is most likely an extract from:
(a) A science journal. (b) A book review.
(c) A news magazine. (d) An editorial in a newspaper.
122. From the passage, which of the following would constitute a proper description
of children’s minds?
(a) Children’s minds are as developed and discerning as adult minds, with a full
understanding of being.
(b) Children’s minds are not comparable to adult minds since they function
differently and have less inner
consciousness and a sense of perspective.
(c) Children’s minds tend to obtain a grasp of the whole environment rather than
particular aspects of it,
and thus have a higher philosophical sense of reality.
(d) Children’s minds tend to be exceedingly rational, discerning and innovative,
and possess a kind of
holistic awareness of the surroundings.
123. Which of the following is the process of the growth of children’s minds, as
described in the passage?
(a) Children acquire knowledge through vivid imagination, which helps them
construct mental maps of the
world, and use this imagination to understand how people do or should behave around
them.
(b) Children acquire knowledge with the aid of their imagination, which helps them
construct mental maps
by exploring alternatives to how the world around them works and acts.
(c) Children learn through imagination, and a wide range of such imagination
enables them to learn vital
habits as they grow.
(d) Children possess an intrinsic ability to notice their whole surroundings, which
helps them construct mind
maps that enable a clearer perception of the world.

. Page 32 of 36
124. Which of the following best illustrates the distinction drawn between the
awareness of a child’s mind and
that of an adult?
(a) An adult is able to understand the intricacies of operating a new washing
machine, while a child often
attempts to experiment with the device.
(b) At a wedding, a child excitedly looks around at the decorations, while an adult
is lost in contemplation
of the debts, he incurred to make these decorations possible.
(c) A child enjoys the festive atmosphere of a busy market replete with Christmas
goods, but her mother
is engrossed in looking for items on her shopping list.
(d) A child gets frightened at the sound of thunder and begins to cry, while an
adult is unruffled and merely
draws out her umbrella expecting rain.
125. Which of the following can be inferred from the given options?
(a) According to Alison Gopnik, Children appear to have a far more vivid awareness
of the world around
them than adults do.
(b) Children due to their a distinguishing sense of self, have more command of
behaviour and are sentient
of the future.
(c) Based on their understanding, Jean Piaget and Sigmund Freud offered a grim
picture: smaller children
as incapable of logically constructing their environment; less attentive to their
surroundings and
basically concerned with self as the centre.
(d) Jean Piaget and Sigmund Freud and Alison Gopnik shared same views of children
with respect to their
holistic development.
Passage (Q.126-Q.130): Read the given passage carefully and answer the questions
that follow.
Another Dozen Stories, by Satyajit Ray, translated from the Bengali by Indrani
Majumdar. The book
features never-before translated stories of spooky, magical and astonishing worlds
created by the
polymath. We spoke with Majumdar about these Ray's world of fascinating stories. On
asking Indrani to
tell us about her experience while translating the stories included in the book,
she gave us this answer.
"Ray's style of writing at one level is very simple, colloquial and down to earth.
Yet, this style posed a huge
challenge to me. The opening story in this volume, The Life and Death of
Aryasekhar, written way back in
1968, is one that Ray labelled as 'the only story written with adults in mind'. Not
just the theme, what was
a complete departure was the use of a very strict, formal Bengali style of writing
by Ray. Reading this story,
one was more than impressed, and it revealed Ray's strong hold over his mother
tongue. And here, I put
in real effort and labour over my translation. If readers like reading this book,
the credit goes strictly to the
writer. Though translating the other stories meant for young adults proved less
daunting, it meant reworking
on the structure of certain sentences quite a few times. In Bengali, Ray on
occasions wrote a sentence so
long, it almost amounted to one paragraph. In the original, it works rather well.
But not necessarily so when
transforming it into another language. To ease the flow of reading, I cut it down
to 3-4 sentences to retain
the essence. Translating his quaint use of humour and wit is no less a challenge.
Time and again a twist in the last sentence of a story could never retain the same
charm or offer a tone of
the same surprise while writing in English. I felt translating a language often
meant translating a culture,
too. For instance, a story ends with a mere sentence stating that the protagonist
chose to die in the early
hours on the day of his niece's wedding. It imports a note of a huge tragedy as a
death in a Bengali family
meant no auspicious activities to be observed for the next whole year. I'm sure
such a custom works
elsewhere, too in India."
126. According to the passage, which of the following inferences can be drawn about
Indrani Majumdar?
I. She is a brilliant and wonderfully successful writer.
II. She, as a translator, adds creativity to her work.
III. She has a deep insight about translating, not just words but emotions too.
IV. She likes getting credit for her work.
(a) Only I (b) Only II (c) Both II and III (d) Both I and IV

. Page 33 of 36
127. "Time and again, a twist in the last sentence of a story could never retain
the same charm or offer a tone
of the same surprise while writing in English."
Which of the following, if true, seriously weakens the argument above?
I. English as a language lacks the power of expression.
II. Bengali is better than English.
III. Satyajit's stories had predictable endings.
IV. The English novels have immense power to hold the interest of the audience.
(a) Only I (b) Both II and IV (c) Both III and IV (d) All I, II, III and IV
128. "Ray's style of writing at one level is very simple, colloquial and down to
earth. Yet, this style posed a huge
challenge to me."
The above statement makes which of the following assumptions?
I. Ray's writing style is unmatchable.
II. The translator likes taking up challenges
III. Translation demands retaining the essence of the original work
(a) Only I (b) Only II (c) Both I and II (d) Both II and III
129. Which of the following statement(s) is/are true according to the passage?
(a) It is difficult to translate books written with adults in mind
(b) It is challenging to recreate instances and emotions like the original in the
translated version.
(c) Working on sentence structures is the most daunting task of all.
(d) All of the above
130. Which of the following conveys the main idea expressed in the passage?
(a) Exaggerating the work of Satyajit Ray.
(b) Self-appreciating the efforts of Indrani Majumdar.
(c) Outlining the challenges of a translator.
(d) Comparing English and Bengali.
Passage (Q.131-Q.135): Human beings are self-conscious creatures: we can
conceptualise ourselves as
psychological beings, forming beliefs about who and what we are. We also have
identities: self-beliefs that
are sources of meaning, purpose and value, and that help to constrain our choices
and actions.
In addition to being able to think about ourselves, self-conscious beings can
recognise that we are the
objects of other people's thoughts. This opens up the possibility of a conflict
between our own identities
and how we are perceived by others. This potential for conflict gives us unique
power over each other, and
also makes us uniquely vulnerable: only self-conscious beings can kill with a
glance or die of
embarrassment. Our vulnerability to how others regard us might create obligations
to try to regard others
in some of the ways they desire - ways that are consonant with their own
identities. But what about identities
that we think are false or absurd - or that we simply don't understand?
A plural is a human being who says things like: 'I'm one of many people inside my
head.' Although they are
quite rare (it's impossible to say now rare), plurals are increasingly visible on
social media and in the
occasional popular media article. At present, there is a handbook online about now
to respond to a co#worker's 'coming out' (as the document puts it) as plural. You
might think you've heard of plurals if you've
heard of dissociative identity disorder (DID), because, like plurals, people With
DID experience themselves
as being psychologically multiple. But many plurals don't meet the diagnostic
criteria for DID. Often, this is
because they don't find their plurality per se to be distressing or impairing. In
other cases, it's because they
don't meet the amnesia criterion for DID, since the multiple beings that plurals
experience as being inside
them can share experiences or communicate to each other about their experiences.

. Page 34 of 36
131. Which of the following can we infer about 'plurals' from the given passage?
(a) Plurals might be mistaken for people with dissociative identity disorder.
(b) A plural human being isn't a person at all.
(c) There are identities that we shouldn't respect, because they reinforce unjust
social arrangements.
(d) Plurals live with a discord between what they believe about themselves and what
everyone else
believes.
132. We can infer from the passage that…
(a) Plurality is the by-product of the online social media platforms.
(b) Human beings can counter conflicts by becoming impervious to other’s perception
about self.
(c) What we can learn about respect and identity from plurals.
(d) Human beings have many mental malaises that they are unaware of.
133. Which of the following is most likely to be true had human beings not cared
about what others think of
them?
(a) Human beings would be facing existential crisis.
(b) The element or concept of plurality might not be in existence at all.
(c) There won't be any vulnerability among human beings.
(d) There won't be any conflict among human beings.
134. Which of the following forms the premise for the author's argument that
potential for conflict makes us
powerful and vulnerable at the same time?
(a) By virtue of possessing the element of plurality, we can experience two
contrasting emotions
(powerfulness and vulnerability) at the same time.
(b) Human beings experience identity crisis if they don't feel multiple emotions
all at once.
(c) We can form opinions about others which makes us powerful and at the same time
will be judged by
others, making us vulnerable.
(d) Responding to a particular situation requires a mix set of emotions.
135. Which of the following, if true, would weaken the argument made in the passage
that we are objects of
other people's thoughts?
(a) Out in the social world - that is, the social world outside their heads -
plurals mostly live as if they were
the way that singlets see them.
(b) It generally matters to us that other people respect those identities which
matter to us.
(c) A relationship with plurals would put one in a position to grasp what their
plural identities mean to them
- what this does for them, what it scaffolds or supports in their lives.
(d) None of the above

mock 17

Passage (Q.1-Q.5): Read the following passages carefully and answer the questions
given below it.
Paragraph 1: The government – and Paytm – may not agree, but there are some
downsides to the rising
digitization and connectivity. One is an unleashing of aspirations. Everyone wants
not just what Bengal’s
leftists used to contemptuously dismiss as components of the middle-class Indian
dream — gaadi, baadi,
chaakri (car, home, job) — but a whole lot of other things. From watching the
latest Salman-starrer now —
which is why India is the ‘download’ capital of the world — to the latest phone and
footwear, people desire
for all forms of comfort. This is a downside because the Indian economy is simply
not in a position to create
the kind of ecosystem which will enable all these aspirations to be realized by a
vast majority of the people.
The other downside is the near-instantaneous transmission of unrest. Thanks to
YouTube and WhatsApp,
an expression of farmer unrest can travel from Mannargudi to Mandsaur faster than
any ponderous
government’s attempts to address or contain the disturbance within a particular
area.
Paragraph 2: If we haven’t had more such demonstrations, or bigger unrest, it is
probably because there
hasn’t been an issue big enough to concern all the people across the country at the
same time. The recent
farmer protests, big and significant though they were, left urban India untouched.
But there is one issue
which, sooner or later, may unleash social unrest on a scale which will make the
farmer agitation look like
a toddler’s tantrum — jobs. Or more precisely, the lack of them. That the available
workforce of a country
needs to be occupied in fruitful employment is a no-brainer. The reason global
businesses are investing
billions of dollars in India is its potential to become one of the world’s economic
powerhouses.
Paragraph 3: This potential derives from India’s ‘demographic dividend’ — the
millions of young people
joining the workforce every year. India will add more than 100 million people of
working age between now
and 2025, by which time it will account for one-fifth of the entire world’s
workforce. That creating productive
and remunerative jobs for these aspirants is India’s biggest economic challenge is
also a no-brainer.
Paragraph 4: If that be the case, then, going by the BSE’s own yardstick, India
appears to be in a very
good place. If 96% of India’s workforce is productively employed, as the index
suggests, then Prime
Minister Narendra Modi has more than made good on his pre-poll promise of creating
one crore new jobs
in five years. It also means that India is now well and truly a middle-income
country, with poverty and
deprivation confined to isolated pockets, and many, if not all, in a position to
fulfill most of those newly
unleashed aspirations.
Paragraph 5: Economist Sudipto Mundle argued in a recent paper that “the vast
majority of India’s working
households are still living precariously on the brink of survival”. This is Modi’s
— and India’s — biggest
challenge. It is not just about creating jobs but generating employment that yields
substantially more than
mere sustenance. Otherwise, the ‘aspirational young India’ will turn into an ‘angry
young India’.
1. According to the first paragraph, which of the following is untrue?
(a) Everything is not hunky dory in the digitization projection in India.
(b) A middle class Indian dreams about having a good job, affordable housing and a
sensible car to drive
around.
(c) Salmaan starrers are the only ones which are being downloaded by the Indians as
part of digitization.
(d) Indian economy needs to pass through various upward modes to cater to the
aspirations of its major
chunk of population.
2. In the second paragraph, the author uses the phrase 'a toddler's tantrum'. Which
of the following reflects
the best contextual meaning of the phrase?
(a) The farmers' agitation is still in the infant stage but may become serious in
future.
(b) When compared to the looming job situation, the impact of the farmers'
agitation was a soft fall.
(c) Agitating for rights is child's play for the farmers as they have been
receiving exclusive coverage on the
social media.
(d) Farmers' agitation is like a child's tantrum. It subsides on its own after a
while.

. Page 3 of 36
3. The third paragraph talks of a 'demographic dividend'. What does it imply in the
context of the passage?
(a) When the world over is facing the problem of an aging population, India has a
young human resources
capital.
(b) The population boom in India has been successfully contained.
(c) The B-schools in India are turning out many young people every year adding to
the bleak job market.
(d) There is a vast gap between the aging and young population in India.
4. Which of the following statements connects paragraph three with paragraph four?
(a) According to popular adage, a country can be called as zero unemployed if every
citizen is engaged in
one work or the other.
(b) Dattu is a laborer earning Rs. 5000 a month, barely managing his family needs
and is gainfully
employed.
(c) By 2025, Prime Minister Narendra Modi will be in a position to declare that 96
percent of the workforce
as employed productively, in India.
(d) The age-old adage of 'garibi hatao' will become true in India in 2025.
5. What is the message of caution given by the author in the fifth paragraph?
(a) Indian youth is very restless and may turn angry if not provided with jobs of
their choice.
(b) Poverty can never be eradicated in India.
(c) Prime Minister Narendra Modi needs to work very hard to get rid of poverty in
India.
(d) India accounting for not just sustainable, but a more productive and
remunerative jobs, by 2025 is just
a bubble and needs more introspection.
Passage (Q.6-Q.10): Our planet's natural ecosystems are in trouble. Recent advances
in "big data" and
improved remote sensing tools show us that collective human impacts are leaving
fewer places untouched,
with only 15% of the Earth's land mass formally protected and global biodiversity
declining at an
unprecedented rate.
Global assessments led by scientists, such as the 5th Global Biodiversity Outlook
published this week, and
others endorsed by governments through bodies like the Intergovernmental Panel on
Biodiversity and
Ecosystem Services (IPBES), make it clear that governments are failing to meet
existing global targets for
biodiversity, and that critical ecosystems like coral reefs will be altered to the
point that the biodiversity they
hold, and the services they provide, will be damaged beyond repair.
Corals in particular have shown the most rapid increase in extinction risk of all
assessed species groups,
with studies showing an estimated two thirds of coral reef fish lost compared to
historical reefs as only 2.5
percent of the world's reefs are being actively protected. We must respond to this
remarkable scientific and
political consensus on biodiversity loss with meaningful action. As an
international community, we must do
more to help protect those important ecosystems such as coral reefs, which provide
extraordinary
contributions to both biodiversity (about 25 percent of all marine biodiversity
across about 0.1 percent of
the ocean floor) and human wellbeing (economic and food security for hundreds of
millions of people).
International plans, usually in the form of policy frameworks, are well suited to
globalized threats such as
climate change, but have a hard time addressing the myriad complex localized
threats facing coral reefs
(such as overexploitation, pollution, coastal and industrial development) and other
ecosystems.
They do, however, play an important role in driving consensus around how to measure
and monitor
ecosystems like coral reefs and the benefits they provide to people. They also
galvanise political will, unlock
billions in financing for nature conservation, and provide the impetus to drive
change at a local level.
Currently, all but a few of the world’s governments are in the process of
negotiating updated, consensus#based goals and targets for biodiversity
conservation to replace those that expire in 2020. This “post -2020
global biodiversity framework” will guide the action and investments of the 196
governments that are party
to the Convention on Biological Diversity (CBD). The negotiations are expensive and
complex. Over the
last two years, members of the International Coral Reef initiative (ICRI), a global
partnership of more than
forty governments and more than forty civil society organization with coral reef
expertise, have agreed on
this framework that are critical for coral reefs.

. Page 4 of 36
6. Which of the following reflects the complexity of the international plans for
ecosystem problems?
(a) The plans are usually in the form of policy framework.
(b) Governments don't follow the policy framework for a longer duration.
(c) They play an important role in driving the consensus around how to measure and
monitor Ecosystems.
(d) International plans have a hard time tackling the multitude and complex
localized menaces facing coral
reefs and other ecosystems.
7. In the sentence, "As an international community, we must do more to help protect
those important
ecosystems", Which important ecosystems are being referred to?
(a) Marine ecosystems.
(b) Only coral reefs.
(c) Ecosystems which provide contribution to human wellbeing.
(d) Ecosystems which provide contribution to human wellbeing and biodiversity.
8. Why should we protect coral reefs?
(a) Protecting coral reefs will help nation get funding from World Bank and other
Organisations.
(b) Protecting coral reefs will increase the soft power of a nation.
(c) Coral reefs contribute to the human wellbeing as well as the biodiversity.
(d) Coral reefs help to fight against globalized threats such as climate change.
9. Which of the following can be a suitable title for the passage?
(a) Saving coral reefs: an urgent need.
(b) Role of big data in protecting ecosystem.
(c) How International Coral Reef Initiative was started.
(d) Global Biodiversity post 2020.
10. As per the passage, which of the following sentences is not true?
(a) The natural ecosystem of earth is in trouble.
(b) The existing global targets for biodiversity are not being met.
(c) Members of the International Coral Reef Initiative have agreed on the steps we
must take.
(d) Recent advances in "big data" have helped in the decline of global
biodiversity.
Passage (Q.11-Q.15): One core component of the Belt and road Initiative (BRI) is
the BeiDou Navigation
Satellite System (BDS). BDS, while largely a benign satellite navigation system,
has a key component of
entrapment for economically weaker countries. What exactly are the malign
components of the BDS that
have the potential to entrap countries in technology dependence on Beijing?
The benign parts of BDS can be gauged by the fact that the linked services are
either replaceable, or do
not entail a continuing use of BDS past the implementation-construction phase. To
foster digital economies
in Arab states, Beijing established a BDS in Tunis. This involved the training Arab
state representatives in
transitioning to digital economies, and said training was done on the basis of BDS
to show them the
opportunities and benefits.
For a long time, services we take for granted - like GPS navigation - were
dependent on the US-controlled
Navstar satellites whose services were prohibited from being extended to Iran. A
Memorandum of
Understanding (2015) between China and Iran resolved these issues with BeiDou,
providing high-precision
navigation services, and spawned a host of services based on this technology. In
South Asia, Pakistan's
new Islamabad International Airport's information integration system offers precise
time synchronisation
and coordinated operation of the airport's sub-systems in a "reliable" manner.
Myanmar, Laos, Cambodia, Indonesia, Thailand, and Uganda have used BDS for
agricultural and
infrastructural purposes. These include land, water, and ecology surveys. They have
been able to benefit
by planning their crop cycles and water allocation with great precision, resist
climatic variation, and increase
the accuracy of their national disaster agencies through dependence on these
services. In some of these
countries, like Thailand, power distribution, and goods delivery and tracking are
all linked to BDS.

. Page 5 of 36
On the face of it, all these above-mentioned projects seem benign, with only
benefits to be reaped by the
countries dependent on the system. Yet, each of these associations has masked a
critical element of
dependence. Today, Iran's entire basket of satellite-based services - their
equivalents of Google Maps,
Uber, Zomato, Amazon, etc. - and the massive economic footprint they have spawned
are dependent on
BDS. Any withdrawal of services would be catastrophic for large parts of the
country's urban economy that
depend heavily on this system.
In a similar manner, agricultural systems in several parts of Southeast Asia, as
discussed earlier, have
been enhanced based on BDS technology. Sustaining this expansion is also based on
BDS, as it is used
to plan cropping seasons, adjust water distribution, and compensate for climate.
Indeed, once even basic,
and traditional activities like agriculture become technology-dependent, their
extraordinary increase in
productivity and resistance to climactic variation also depend on the usage of this
technology.
11. Which of the following is referred to the 'critical element of dependence' as
mentioned in the given passage?
(a) The expansion of the BDS Technology and the increase in the number of countries
who accept it.
(b) The utter dependence on the BDS technology, from where going back is not
possible and any departure
from the services of the technology would be disastrous.
(c) The extraordinary increase in productivity and resistance leading to the
betterment of the agricultural
system.
(d) The transfer of the technology from Navstar satellites to BeiDou Navigation
Satellite System in most
Asian countries.
12. What is the author's motive behind writing the fifth paragraph of the given
passage?
(a) To tell the reader how BDS has the ability to entrap countries in technology
dependence.
(b) To tell the reader about overall dependence of Iran on China's various
technologies.
(c) To explain about the pros and cons of BDS for Iran.
(d) To tell the reader about different satellite-based services in Iran.
13. Which of the following is similar in meaning to the word 'benign' as mentioned
in the given passage?
(a) Meticulous (b) Propitious (c) Unerring (d) Solemn
14. With which of the following is the author least likely to agree?
(a) There are many benefits of the BDS technology.
(b) Iran itself is responsible for its own course of action.
(c) BDS technology will eventually cause trouble to the whole world.
(d) The BDS technology largely focus on poor countries.
15. Which of the following is incorrect as per the given passage?
(a) BeiDou Navigation Satellite System is one of the central components of the Belt
and road initiative.
(b) Myanmar, Laos, Indonesia, and Thailand use BDS technology.
(c) BDS technology fails when faced with the situations such as drought.
(d) Agricultural systems in several parts of Southeast Asia have been enhanced
based on BDS technology.
Passage (Q.16-Q.20): Perhaps no other social entity appears more 'natural' than the
family. Often, we are
prone to assume that all families are like the ones we live in. No other social
institution appears more
universal and unchanging. Sociology and social anthropology have over many decades,
conducted field
research across cultures to show how the institutions of family, marriage and
kinship are important in all
societies and yet their character is different in different societies. They have
also shown how the family (the
private sphere) is linked to economic, political, cultural and educational (the
public) spheres.
According to the functionalists the family performs important tasks, which
contribute to society's basic
needs and helps perpetuate social order. The functionalist perspective argues that
modern industrial
societies function best if women look after the family and men earn the family
livelihood. In India studies,

. Page 6 of 36
however, suggest that families need not become nuclear in an industrial pattern of
economy. This is but
one example to show how trends based on experiences of one society cannot
necessarily be generalised.
The nuclear family is seen as the unit best equipped to handle the demands of
industrial society by the
functionalists. In such a family one adult can work outside home while the second
adult cares for the home
and children. In practical terms, this specialisation of roles within the nuclear
family involves the husband
adopting the 'instrumental' role as breadwinner, and the wife assuming the
'affective', emotional role in
domestic settings (Giddens 2001). This vision is questionable not just because it
is gender unjust but
because empirical studies across cultures and history show that it is untrue.
A central debate in India has been about the shift from nuclear family to joint
families. We have already
seen how sociology questions common sense impressions. The fact is that nuclear
families have always
existed in India particularly among deprived castes and classes.
The sociologist A.M. Shah remarks that in post-independent India, the joint family
has steadily increased.
The contributing factor is the increasing life expectancy in India according to
him. It has increased from
32.5 - 55.4 years for men and from 31.7 - 55.7 years for women during the period
1941 - 50 to 1981- 85.
Consequently, the proportion of aged people (60 years and above) in the total
population has increased.
This again is a broad generalisation. But in the spirit of the sociological
perspective, it cautions us against
blindly believing a common-sense impression that the joint family is fast eroding.
And alerts us to the need
for careful comparative and empirical studies.
16. Which of the following strengthens the author's argument that the character of
family, marriage and kinship
is different in different societies?
(a) Importance of family, marriage and kinship is different in different societies.
(b) The private sphere such as family is linked to economic, political, cultural,
and educational spheres.
(c) Family plays an important role in a person's life and give strength to the
individual irrespective of the
societies.
(d) Marriage as an institution is as successful in western culture as much as it is
in the Indian culture.
17. Which of the following is similar in meaning to the word 'perpetuate' as
mentioned in the passage?
(a) Discern (b) Halt (c) Cease (d) Sustain
18. According to the author, the division of roles within the nuclear families is
(a) Justified, but a few sections of the society are against it.
(b) Is not true as per the empirical studies across cultures.
(c) Is not according to the demands of industrial society.
(d) Is a myth with respect to the western societies.
19. According to the passage, why was there a steady increase in joint family in
the post-independent era?
(a) Increase in life expectancy in India.
(b) Increase in the proportion of younger people.
(c) Decrease in the number of aged people.
(d) Increase in the number of nuclear family.
20. What is the author's purpose behind writing the first paragraph of the given
passage?
(a) To realise the importance of family and how it is affected by the economic and
cultural spheres.
(b) To reveal the changing principles of family as joint families are seen very
often.
(c) To highlight how the universal and important social institution i.e., family
can vary as per its public
sphere.
(d) To indicate that all the societies are not same and do not have the same set of
values.

. Page 7 of 36
Passage (Q.21-Q.25): The island of Ireland, whose northernmost part lies a mere 13
miles from Britain,
has been contested territory for at least nine centuries. Britain long gazed with
colonial ambitions on its
smaller Catholic neighbour. The 12th-century Anglo-Norman invasion first brought
the neighbouring
English to Ireland.
In the late 16th century, frustrated by continuing native Irish resistance,
Protestant England implemented
an aggressive plan to fully colonize Ireland and stamp out Irish Catholicism. Known
as “plantations”, this
social engineering exercise “planted” strategic areas of Ireland with tens of
thousands of English and
Scottish Protestants.
Plantations offered settlers cheap woodland and bountiful fisheries. In exchange,
Britain established a base
loyal to the British crown – not to the Pope.
England’s most ambitious plantation strategy was carried out in Ulster, the
northernmost of Ireland’s
provinces. By 1630, according to the Ulster Historical Foundation, there were about
40,000 English#speaking Protestant settlers in Ulster.
Though displaced, the native Irish Catholic population of Ulster was not converted
to Protestantism.
Instead, two divided and antagonistic communities – each with its own culture,
language, political
allegiances, religious beliefs and economic histories – shared one region. Over the
next two centuries,
Ulster’s identity divide transformed into a political fight over the future of
Ireland.
“Unionists” – most often Protestant – wanted Ireland to remain part of the United
Kingdom. “Nationalists” –
most often Catholic – wanted self-government for Ireland. These fights played out
in political debates, the
media, sports, pubs – and, often, in street violence. By the early 1900s, a
movement of Irish independence
was rising in the south of Ireland. The nationwide struggle over Irish identity
only intensified the strife in
Ulster.
The British government, hoping to appease nationalists in the south while
protecting the interests of Ulster
unionists in the north, proposed in 1920 to partition Ireland into two parts: one
majority Catholic, the other
Protestant-dominated – but both remaining within the United Kingdom. Irish
nationalists in the south
rejected that idea and carried on with their armed campaign to separate from
Britain. Eventually, in 1922,
they gained independence and became the Irish Free State, today called the Republic
of Ireland.
In Ulster, unionist power-holders reluctantly accepted partition as the best
alternative to remaining part of
Britain. In 1920, the Government of Ireland Act created Northern Ireland, the
newest member of the United
Kingdom.
In this new country, native Irish Catholics were now a minority, making up
___(A)___ of Northern Ireland’s
1 .2 million people. Stung by partition, nationalists refused to recognise the
British state. Catholic
schoolteachers, supported by church leaders, refused to take state salaries. And
when Northern Ireland
seated its first parliament in May 1921, nationalist politicians did not take their
elected seats in the
assembly. The Parliament of Northern Ireland became, essentially, Protestant – and
its proBritish leaders
pursued a wide variety of anti-Catholic practices, discriminating against Catholics
in public housing, voting
rights and hiring.
21. Which of the given options can be used to complete the blank labelled (A) in
the most appropriate way,
contextually and grammatically?
(a) less by a third
(b) less by one thirds
(c) less than a thirds
(d) less than a third

. Page 8 of 36
22. Which of the following statement (s) is/are NOT TRUE in accordance with the
information provided in the
passage?
I. Britain successfully suppressed Catholicism in Ireland.
II. The British crown was under authority of the Pope.
III. "Plantations" restructured the society of Northern Ireland.
(a) Only I (b) Only III (c) Both II & III (d) Both I & II
23. Which of the following is/are correctly inferred from the given passage?
I. Ulster was chosen as a plantation because of its geographical significance.
II. The people of Ulster actively took part in the conflict.
III. Ulster unionists didn't support the formation of Irish Free State.
(a) Only I (b) Only III (c) Both II & III (d) Both I & II
24. Which of the following words as used in the passage can be replaced by the word
“belligerent”?
(a) Strife (b) Appease (c) Gazed (d) Aggressive
25. Why did Britain want to colonize Ireland?
(a) To finish the Anglo-Norman conquest.
(b) To create a stronger front to defeat the Spanish Armada
(c) To bring more territory under the British crown.
(d) To exploit the resources of Northern Ireland.
Passage (Q.26-Q.30): A murmur from the driver’s seat indicated that I was near my
destination. From the
confines of the car I scanned the dark surroundings with eyes heavy with sleep.
Kochi’s noisy, unruly
evening traffic was around 55 km behind us. I attributed my sluggishness to the
near-silence that rules
Thattekad, a bird sanctuary in the foothills or the Western Ghats in Kerala’s
Ernakulam district.
My eyes were half-closed as I entered my room at the home stay. The lone photograph
in the room - that
of three unattractive birds captioned as Ceylon frog mouth - did little to perk me
up. I remained in this state
until dinner summons. Frog mouth photos stared back from every wall in the house as
I went through the
meal in a haze. By the time I climbed into bed, I’d had an overdose of the
nocturnal grey-brown bird. Or so
I thought.
The morning arrived early. Girish, the guide, took our group of three (two birding
fanatics from Bengaluru
being my companions for the day) into the forest. We walked under a canopy of teak,
mahogany, and
bamboo trees, at a rocky patch. Girish instructed us to look out for the black
baza, a small bird of prey. The
baza didn’t oblige but many others did. In a span of 30 minutes, we spotted the
Asian brown flycatcher, the
orange headed thrush, the Oriental honey buzzard and a variety of woodpeckers.
The little black raptor chose to stay elusive. In a desperate bid to divert
attention from the bird that remained
invisible, I voiced my ‘desire’ to see the unsightly frog mouth. We moved ahead in
search of the new target.
But instead of the grey-brown plumage of the peninsular bird, we were rewarded with
the eye-popping
colors of the Malabar trogon. And before we knew it, the canopy overhead had given
way to the most
spotless blue sky. We reached a water body in the middle of the sanctuary. A
handful of dead tree trunks
jutted out of the water. It was the end of the walk and I was happy to just sit by
and absorb the stillness
around.
The frog mouth had been declared an endangered species in 1983. Improving their
numbers proved difficult
given that the bird lays only one egg on an average every year. More than two
decades on, the number of
frog mouths at Thattekad has climbed close to 50 pairs.
My quest for the bird I had begun to admire continued over the next day. We drove
towards Pandapara,
where the frog mouth had been sighted close to where we stood. We waded through the
knee-deep waters
of a stream before we arrived at another rocky patch. A few minutes later he
pointed to a tree to the left of
the trail we were on. There, in perfect camouflage with the dead leaves, sat a pair
of frog mouths. If one of
them hadn’t blinked I’d have assumed those birds were stuffed!

. Page 9 of 36
26. In the beginning of the passage the author is not very keen on the photograph
of the frog mouth, but the
next day he is desperate to spot it. Why?
(a) The author is an ornithologist out on bird spotting.
(b) He was tired after the journey and the unattractive snaps of the frog mouth did
not help.
(c) The frog mouth was an endangered species and spotting it would be any
ornithologist’s dream.
(d) All the above
27. Why does the author call his friends ‘a couple of birding fanatics?’.
(a) They were fanatic about birds and their habitat.
(b) The author and his friends are ornithologists, and they are so committed to
their profession that they
are passionate about spotting birds.
(c) The author’s friends were interested only in birds and could even skip food for
their sake and the author
finds it difficult to keep pace with them.
(d) The authors friends were ardent bird lovers and kept trekking fanatically till
they spotted the bird they
wished to see.
28. What does the author mean when he says ‘absorbing the stillness around’?
(a) Sitting absolutely still so as not to disturb the birds.
(b) After a long walk in the jungle, the author is just relaxing and stretching his
feet.
(c) The author is an ornithologist and hence is always attracted by nature. Here,
he just sits still feeling
one with the nature.
(d) It is an expression to denote that the scene with the water body was so
ethereal that the author forgot
himself and sat still.
29. Frog is an amphibian and the frog mouth belongs to the avian community. What
can be common between
them?
(a) Both have descended from the frog.
(b) Both eat similar types of insects.
(c) The frog mouth gapes like a frog.
(d) Both have the same ancestors.
30. ‘If one of them hadn’t blinked I’d have assumed those birds were stuffed!’ This
is the last line of the passage.
What do you infer from this?
(a) As the frog mouth was endangered the author could only see the stuffed version
of the birds.
(b) The frog mouths blended so beautifully with the foliage and sat so motionlessly
that only their blinking
could differentiate them.
(c) The author was waiting for the frog mouths to blink so that he could capture
them in his camera.
(d) The frog mouths were so scared with the human treads that they even forgot to
blink

Directions (Q.66 – Q.105): Read the comprehensions carefully and answer the
questions based on it.
Passage (Q.66-Q.70): The Medical Termination of Pregnancy (Amendment) Bill, 2020
was introduced in
Lok Sabha on March 2, 2020 and passed on March 17, 2020. The Act specifies the
grounds for terminating
a pregnancy and specifies the time limit for terminating a pregnancy. Under the Act
a pregnancy may be
terminated up to 20 weeks by a married woman in the case of failure of
contraceptive method or
device. The Bill allows unmarried women to also terminate a pregnancy for this
reason. The requirement
for terminating pregnancy up to 20 weeks requires advice of one doctor. For 20-24
weeks advice of two
doctors is needed and for more than 24 weeks permission from medical board is
needed in case of foetal
abnormalities. Lastly termination of pregnancy at any time can only be done if the
life of pregnant woman
is at stake. The Medical Termination of Pregnancy Act, 1971, nowhere provides for
the express or implied
consent of the husband or anyone other than the woman carrying the child.
Medical Boards: All state and union territory governments will constitute a Medical
Board. The Board will
decide if a pregnancy may be terminated after 24 weeks due to substantial foetal
abnormalities. Each
Board will have a gynecologist, pediatrician, radiologist/sonologist, and other
members notified by the state
government.With regard to privacy a registered medical practitioner may only reveal
the details of a woman
whose pregnancy has been terminated to a person authorized by law. Violation is
punishable with
imprisonment up to a year, a fine, or both.
66. Seema Malhotra was medically examined and she was adamant to get the foetus
aborted but the husband
refused. The husband refused to sign the papers giving his consent to terminate the
pregnancy. Medical
Termination was conducted anyway. Husband filed a suit for the recovery of Rs. 30
lacs towards damages
on account of mental pain, agony and harassment against the wife. Based on the
author's reasoning should
the damages be awarded to the husband?
(a) Damages should be granted since the husband is an integral entity in the future
of child.
(b) Damages should be granted since the husband has undergone mental pain and agony
due to loss of
his child.
(c) Damages should not to be granted since the husband has no locus standi to file
the case.
(d) Damages should not to be granted since the Medical Termination of Pregnancy Act
nowhere provides
for the express or implied consent of the husband.
67. The Medical Termination of Pregnancy (Amendment) Act, carves out an exception
for carrying out
termination of pregnancy, to save the life of pregnant woman irrespective of length
of pregnancy. Doctors
will not be unnecessarily prosecuted if they act in accordance with the rules in
good faith. Dr, Madhu
terminated the pregnancy of Radhika, a minor girl, who is an alleged victim of rape
as the continuation of
pregnancy would result in the loss of her life. The police have decided to
prosecute Dr. Madhu. Decide.
(a) Prosecution shall fail since Dr. Madhu is a doctor of high stature.
(b) Prosecution shall succeed since Dr. Madhu did not take permission from her
family.
(c) Prosecution shall succeed since Dr. Madhu acted in violation of the Medical
Termination of Pregnancy
Act.
(d) Prosecution shall fail since Dr. Madhu acted in good faith so that he can
safeguard her life and put her
out of danger.
. Page 16 of 36
68. Parents should have the right to know of a child's pregnancy or abortion
because they are ultimately
responsible for their child's care until she is eighteen. With the exception of a
few circumstances, parents
are responsible for their children's mental and physical health until they are
adults. A child should not be
allowed to make such a huge and potentially damaging decision about her health
without the parent's
knowledge. Which of the following weakens the author's argument above as per the
passage given.
(a) The parents deserve to know of their child's pregnancy because they will step
up and deal with the
pregnancy more rationally than their child.
(b) Parents may react very negatively to a daughter's pregnancy and reject her at
such a critical phase in
her life, adding to her mental stress and forcing her to terminate the pregnancy.
(c) If the child cannot relate to the parents enough to tell them there is most
probably a reason, aside from
fear of disapproval for it.
(d) The girl regardless of age, is the only person who has the right to decide
whether or not to continue
with the pregnancy.
69. Sarika has 2 sons and was pregnant with another child. Due to fetal
abnormalities she decided to terminate
the 24 weeks pregnancy. The Medical board comprising of gynecologist,
radiologist/sonologist, and other
members notified by the state government gave permission and the pregnancy was
terminated. Was this
the correct procedure to follow as per the passage?
(a) Yes, as it is the prerogative of the Board to decide if a pregnancy may be
terminated after 24 weeks
due to substantial fetal abnormalities.
(b) No, as Sarika was not a rape victim.
(c) No, as a pediatrician was not present in the board.
(d) No, because the consent of two doctors is needed to terminate pregnancy up to
24 weeks.
70. Bindu was a surrogate who agreed to carry the baby of John and Amy. When Bindu
was 20 weeks
pregnant, the doctor discovered that the child will be born with down syndrome.
Hearing this John and Amy
decided that they did not want the child anymore and asked Bindu to abort the
child. When Bindu refused
to terminate the pregnancy, John and Amy went to court to make her abort the child.
Can the court ask
Bindu to terminate the pregnancy as per the passage?
(a) Yes, because the child is of John and Amy.
(b) Yes, because the child has fetal abnormalities.
(c) No, because only the woman carrying the child has a say regarding termination
of pregnancy.
(d) Yes, because the mother is the only one who has a say regarding termination of
pregnancy.
Passage (Q.71-Q.74): Section 378 of IPC states “Any person with the intention to
move or take a movable
property out of the possession of the property holder without his consent then this
is called theft.” The
intention is an important ingredient to perform the crime of theft and it must be
dishonest. The intention
must exist that the taker of the property intends to cause wrongful gain to one
person or wrongful loss to
another person. It is not necessary that the taker of property must have wrongful
gain it would be sufficient
if it causes wrongful loss to the property holder. If a person takes away any
movable property without the
consent of the property holder even though temporarily and with an intention to
return it later on, it would
amount to theft. Where a person takes another’s, property believing on the mistake
of fact that he has the
right to take that property, he is not guilty of theft because there is no
dishonest intention and mistake of
fact is excusable. The property which is permanently attached to the earth is known
as immovable property
and which is not attached with earth and can move from one place to another is
known as movable property
which is defined under section 22 of IPC. The subject matter of theft must be
movable property it must not
be immovable property. Section 379 provides the punishment for theft. It states
“Whoever commits theft
shall be punished with imprisonment of either description for a term which may
extend to three years or
with a fine or with both.”

. Page 17 of 36
71. Aman bought a piece of cloth worth Rs 1000/- from Xoco. He gave it for
stitching to Bunny a tailor for
making a suit for him. After 3 days Aman went to collect his suit. Bunny demanded
the stitching cost from
Aman but he refused and took away his suit without paying him.
(a) Aman is not guilty of theft as he was the owner of the suit.
(b) Aman is guilty of theft because he did not pay the tailor’s money.
(c) Aman is not guilty of theft because he had no dishonest intention.
(d) Aman is guilty because he took the suit from the tailor’s possession without
paying his consent.
72. Sanay is Mr. Manoj’s secretary. One day while working in Mr. Manoj’s office
Sanay removed a file in his
house which contained confidential information, made it available to an outsider
and then returned it to the
office after two days. Decide.
(a) Sanay is not guilty because the file is still in the office.
(b) Sanay is guilty for theft because he took the file without Mr. Manoj’s consent.
(c) Samay is guilty because he had a dishonest intention.
(d) Samay is guilty because he moved the file out of Mr. Manoj’s possession without
his consent with a
dishonest intention.
73. In a village called Zamby, Mohan took Zohan’s three cows against her consent
and distributed them among
three people in his village. Decide.
(a) Mohan is not liable for theft because he was acting in good faith.
(b) Mohan is not guilty for theft because there is no wrongful gain of the thief.
(c) Mohan is guilty of theft because although there was no wrongful gain to the
thief but there was wrongful
loss to the owner.
(d) Mohan is not guilty because the essentials of theft have not been fulfilled.
74. Z was going on a journey and therefore he gave his gold chain to A at that
time. A had the possession of
that gold chain till Z returned. The following day, A sells that gold chain to B.
Decide.
(a) A is liable for theft because the chain was Z’s.
(b) A is liable for theft because he sold the chain without Z’s permission.
(c) A is not guilty as he was entrusted with the chain.
(d) A is not guilty because the chain was not in Z’s possession so A has not
committed theft.
Passage (Q.75-Q.79): Justices M. Duraiswamy and K. Muralishankar of the Madurai
bench of the Madras
high court have held that there is a colossal difference between complaining that a
person is acting against
the government and that person is protesting the policies of the government. The
court was hearing a
petition filed by K Siva, a law graduate. He was not permitted to apply as a member
of the Bar Council of
Tamil Nadu and Pondicherry. A police verification report against Siva had mentioned
that 88 criminal cases
were registered against him between 2017 and 2019.
Most of these cases were lodged after the demonstrations held at Thoothukudi on May
22, 2018 demanding
closure of the Vedanta’s Sterlite copper plant as it had polluted the air and water
and caused health hazards
to the people. Thirteen persons died during the protests, as police opened fire at
them. The Madras high
court had on August 18, 2020 upheld the state government’s closure order. The
Madurai bench has
reasoned that the duties of the state in protecting the environment are basically
the rights of the people.
The bench relied on Article 51-A (g) of Part IV-A (Fundamental Duties), which deals
with the duty to protect
and improve the natural environment and to have compassion for living creatures.
The bench relied on the
Supreme Court’s observation that the mere fact that the petitioner was a part of a
group and shouted
slogans cannot make him guilty of an offence, as the group was impressing upon the
state the need to
follow the Directive Principles of State Policy enshrined in Article 47 of the
constitution.

. Page 18 of 36
75. Student A, troubled by the filthy and unhygienic conditions prevailing in his
neighbourhood causing
widespread cases of dengue and chikungunya, due to the City Municipal Corporation’s
apathy and
negligence, decides to organise a peaceful protest against the local government for
their continued
inaction. Many persons join this protest, shouting slogans against the government
for denying the locality
funds and medical assistance. Soon there is bloodshed and violence when the police
start open-firing at
the protests. Several cases are filed against student A for motivating anti-
government sentiments. Decide
student A’s liability?
(a) Student A should be held liable as he is solely responsible for organizing the
protest which led to people
shouting slogans that were strongly against the government.
(b) Student A should not be held liable as he was too young and therefore incapable
of understanding the
consequences of his actions.
(c) Student A should not be held liable as he was protesting for his rights and
urging the government to
fulfil its own duty.
(d) Student A along with the rest of the protestors should be held liable for
invoking violence against the
government and its machinery.
76. The Members of the Medical Welfare Association (MWA) with relation to the
recent curbs on the import of
a vital drug for diabetes called Glipalamide, following ongoing trade restrictions
by the state government to
increase domestic drug manufacturing capabilities leading to sudden shortfalls in
the availability of this vital
medication, decide to assemble outside the Parliament in organized groups and
demand the government
to resign. They encourage bystanders to join their protest to help them overthrow
the government. Cases
are filed against the MWA for their seditious acts. Decide The MWA’s liability?
(a) The MWA is not liable as they are merely protesting government policy which is
responsible for causing
a huge shortfall in vital medication.
(b) The MWA is not liable as there is huge difference between protesting against
the government and
protesting for collective interest.
(c) The MWA should be held liable for purposely misleading innocent by-standers to
protest for a cause
they don’t personally believe in.
(d) None of the above.
77. Company XYZ located on the outskirts of the village Dholakpur, is allegedly
dumping its chemical waste
into a stream which is the main source of water for most of the villagers. Dholu, a
law graduate from The
Dholakpur Law University decides to file a case against the company for causing
environmental damage
and posing health hazards to the villagers. Dholu, frequently shouts slogans and
organizes protests outside
the Tehsildar’s office to compel the Sarpanch to order the suspension of the
Company’s functioning, but in
vain. He had recently applied for a government job at The Law and Research Centre
in Dholakpur, he was
denied the job on the grounds that he had a criminal background. Adding to his
misery the police too had
filed a charge-sheet against him for criminal conspiracy against the Sarpanch.
Decide if Dholu is eligible to
get the job as per the passage.
(a) Dholu is eligible for the job as protesting for a righteous cause furthered by
article 52-A (g) of Part IV-
(A) (Fundamental Duties) of the Constitution doesn’t account for a criminal act.
(b) Dholu is eligible for the job as he shouldn’t be held guilty for an act which
isn’t an offense and is meant
to encourage public discourse about citizen rights and urge the government to
recognise its own duties
toward its people.
(c) Dholu isn’t eligible for the job as he is notoriously infamous for his actions
against the government.
(d) Dholu can be eligible for the job in the future if he stops protesting against
the government and starts
protesting against the governance.

. Page 19 of 36
78. Kumar Fahid, a prominent environmental activist known for being critical of
many government policies,
started his own show called the Green-Time TV, this show was meant to mirror the
extent of the country’s
environmental loss and measures taken to allay further ecological concerns. It also
featured daily interviews
with youth environmental activists who were keen on raising awareness on government
policies that were
harming indigenous people and tribal groups because of the rapid urbanisation of
several swathes of forest
land. Popular Journalist and Media person, AmitavGhoswami files a case in a
district court against Kumar
Fahid, for his misinformation campaign and calls the show anti-national. The Court
then held that the
remaining episodes of the Green-Time TV should not be allowed to be broadcast as
they are against public
interest. Is the court correct in its interpretation?
(a) Yes, as the facts of the case make it clear that airing inflammatory content
against the government is
useless and that calls for strict action.
(b) Yes, as the content of the show went against the government and its policies,
therefore damaging and
distorting the general public opinion.
(c) No, it is nowhere clear, going by the facts of the case that the show was
against the government.
(d) Yes, as there was a hidden agenda for Kumar Fahid, known for criticising the
government.
79. According to the author of the passage, which of the following would not be in
contravention to the
Fundamental Duty mentioned in the above passage?
(a) Construction of roads and lanes on vulnerable mountainous terrain to connect
cities to
rural/underdeveloped areas.
(b) Allowing chemical effluents to pollute rivers that flow in uninhabited areas.
(c) Creating inhumane conditions for slum-dwellers by dumping the city’s waste in
slum localities.
(d) Producing fossil fuel and mining large amounts of coal that in turn aid climate
change and increase the
global climate temperature.
Passage (Q.80-Q.85): Two adults have a right to choose their matrimonial partner
irrespective of their
religion. The Allahabad High Court has reaffirmed while granting protection to an
inter-faith couple. A
Division Bench granted protection to the inter-faith couple and said that not even
their parents could object
to their relationship.
"It cannot be disputed that two adults have right of choice of their matrimonial
partner irrespective of religion
professed by them...As the present petition is a joint petition by the two
individuals who claim to be in love
with each other and are major, therefore, in our considered opinion, nobody, not
even their parents, could
object to their relationship."
On the said application, the District Magistrate had called for a report from the
concerned police station. As
per the report, the boy's father is not agreeable to the marriage though his mother
is ready for the same.
Both parents of the girl are opposed to the marriage.
In view of the same, the couple approached the High Court, claiming that there was
a threat to their lives.
The Court took note of the fact that both the petitioners are major aged 19 and 24
years.
The Court proceeded to grant them protection while clarifying that the order is not
any final opinion
regarding the age of the petitioners as the findings are only prima facie in nature
for the purposes of
deciding the issue regarding protection of the life of the petitioners.
Source: https://www.barandbench.com/news/litigation/adults-have-right-to-choose-
their-matrimonial#partner-irrespective-of-religion-allahabad-high-court
80. Rama and Shruti are aged 18 and 19, respectively, and have been living in the
same village all their lives.
After having been friends for over 14 years, they finally decided to get married as
they were in love with
each other. As both their families were cordial to each other and were of a
respected caste, they assumed
that their decision would be welcomed. To their dismay, neither of their parents
was amenable to the union
and denied permission outrightly. Can they still get married?
(a) Yes, as they are of consenting age and of the same caste.
(b) No, as they are not adults yet and cannot get married.
(c) No, as their guardians have not yet bestowed upon them the power to consent for
the same.
(d) Yes, as the SC has cleared the position of law on the same by stating that if
the individuals are major,
not even the parents can object to the union.

. Page 20 of 36
81. In another matter of inter-caste marriage Julie, a Christian, and Hamid, an
atheist, the petitioners were
seeking the protection of the SC insofar as protecting their right to marry as an
inter-caste, inter-faith couple.
It was clear that their parents did not consent to the same, and thus, the matter
was under consideration.
Can they be afforded protection?
(a) No, as Hamid does not belong to any faith and cannot be considered to be an
interfaith couple.
(b) Yes, as they are clearly people of two different faith, and their rights are
safeguarded.
(c) No, as the constitution only envisages the right to religion insofar as worship
and practice are
considered.
(d) None of the above.
82. Sudha and Jojo, a homosexual couple, had run away from their home wanting to
marry each other, being
of consenting age and of the same faith. However, before they could appear before
the registrar of
marriages the next day, they were arrested by the police on account of the
complaint filed by their parents.
They were denied marriage on account of the same, and they preferred a petition as
against the High Court
of Delhi. Is their action tenable as per the passage?
(a) No, as homosexual marriages are not legal in India.
(b) Yes, as they are 2 individuals who are adults.
(c) Yes, as their parents do not get to have a say in the matter.
(d) No, as inter-faith marriages are only legal within inter-gender marriages.
83. Kalyan and Saloni were two underage kids, of 15 and 17 years old, and were
madly in love with each other.
This motivated them to marry each other, thus they decided to elope. However, they
were caught by their
relatives before they could get married. When the matter was brought infront of the
court,the court refused
to grant them protection to live together. Is the decision of the Court correct?
(a) It is correct as they were not adults.
(b) It is incorrect since the same was not warranted before pleading for the same.
(c) It is unclear since the nature of a marriage may be dubious.
(d) It is incorrect as the parents cannot object to the relationship.
84. Under which article/provision would an aggrieved inter-faith couple of
consenting age approach the Courts
for granting them protection in marrying each other?
(a) Article 226 of the Constitution of India.
(b) Article 32 of the Constitution of India.
(c) Article 136 of the Constitution of India.
(d) Cannot be determined from the passage.
85. Veeru and Gopa were caught trying to marry each other in the registrar's office
by their parents and were
beaten there by them. Upon bringing an action against their parents and relatives
for brutality and
discrimination, the SC of India had granted them the leave to marry and were
compensated. However, a
parallel claim was filed in the lower courts against the registrar for allowing
them to marry even though the
dissent of the parents was apparent in front of him. Can the action sustain?
(a) No, as the registrar is not personally liable for every marriage.
(b) No, as the registrar bears no liability arising out of the marriages he
registers.
(c) No, as the dissent of the parents is not a valid contention for recourse.
(d) No, as the parents and relatives themselves took the law into their own hands,
breaching public order.

. Page 21 of 36
Passage (Q.86-Q.90): The concept of contempt of Court is several centuries old. In
England, it is a
common law principle that seeks to protect the judicial power of the king,
initially exercised by himself, and
later by a panel of judges who acted in his name. Over time, any kind of
disobedience to judges, or
obstruction of the implementation of their directives, or comments and actions that
showed disrespect
towards them came to be punishable. The law codifying contempt classifies it as
civil and criminal. Civil
contempt is fairly simple. It is committed when someone willfully disobeys a Court
order, or willfully
breaches an undertaking given to Court. Criminal contempt is more complex. It
consists of three forms: (a)
words, written or spoken, signs and actions that “scandalize” or “tend to
scandalize” or “lower” or “tends to
lower” the authority of any Court (b) prejudices or interferes with any judicial
proceeding and (c) interferes
with or obstructs the administration of justice. Making allegations against the
judiciary or individual judges,
attributing motives to judgments and judicial functioning and any scurrilous attack
on the conduct of judges
are normally considered matters that scandalise the judiciary. The rationale for
this provision is that Courts
must be protected from tendentious attacks that lower its authority, defame its
public image and make the
public lose faith in its impartiality.
The punishment for contempt of Court is simple imprisonment for a term up to six
months and/or a fine of
up to ₹. 2,000.
Fair and accurate reporting of judicial proceedings will not amount to contempt of
Court. Nor is any fair
criticism on the merits of a judicial order after a case is heard and disposed of.
86. Mr. Mohit, who is a noted lawyer and activist tweeted that “The CJI is enjoying
his ride on a motorbike
worth INR 40 lakh belonging to a political leader, at a time when he has kept the
Supreme Court in lockdown
mode denying citizens their fundamental rights to access justice”. Decide whether
he is liable for contempt
of Court
(a) Yes, he is liable because he has voluntarily maligned CJI.
(b) No, he is not liable because his statement will be protected as Fundamental
Right of Speech and
expression.
(c) He is not liable because his remark stated the truth.
(d) He is not liable as his tweet was just his opinion without any malice.
87. Mr. A is a public servant who was handling the work of cleaning and sanitizing
Delhi Metro premise. It was
later found that he was using low standard sanitizers for which a suit was filed
against him in Delhi. Judge
found him guilty and imposed a fine of fifty thousand rupees. Mr. A was not
satisfied with the order and he
made a tweet that “Justice is denied to me and it is due to the incompetence of the
judge because he had
personal interest in the matter and knows why he ruled against A I will file an
appeal.” Decide whether Mr.
A is liable for contempt of Court.
(a) Yes, because his remarks made against the Judge are scandalous and can prevent
the course of
justice.
(b) No, he was using his freedom of speech and expression.
(c) No, because he simply stated his opinion.
(d) Yes, because he said that the judge didn’t understand anything at all.
88. Mr. Shashank is a young lawyer who is practicing in Delhi High Court. He filed
a PIL in Delhi HC for removal
of large hoardings of politicians in the vicinity of Court. But, the Court rejected
his PIL without finding any
merits in it. It infuriated Mr. Shashank filed an appeal where he claimed that the
Delhi HC is unable to
understand his grievance and therefore justice was not done to him. Later, one of
the judges of Delhi HC
initiated a contempt proceeding against him. Decide whether he is liable for
contempt of Court.
(a) Yes, because his remarks made against the judge are scandalous and can prevent
the course of
justice.
(b) No, because any citizen can comment or criticize the judgement of this Court
and appeal against it on
merits.
(c) No, because he only said justice was not done to him.
(d) Yes, as he stated that the judiciary failed to address his grievances.

. Page 22 of 36
89. Mr. Xora, a practicing lawyer, writes a blog on a website criticizing a
judgement passed by the Allahabad
High Court for failing to follow the correct position of law settled by the Supreme
Court in an earlier decision.
Whether the Allahabad HC can publish Mr. Xora for committing criminal contempt?
(a) Yes, because his remarks made against the Judge are scandalous and can prevent
the course of
justice.
(b) No, because innocent publication, distribution of matter and reasonable and
fair criticism of judicial act
is permissible.
(c) No, as fair criticism on the merits of a judicial order after a case is heard
and disposed of is not contempt
of Court.
(d) No, as fair criticism on the merits of a judicial order while a case is heard
and disposed of is not contempt
of Court.
90. During the prevalence of Coronavirus pandemic, the Allahabad HC passed an
order, in a Public Interest
Litigation, stating that no eviction or demolition can be done by any
person/authority during this period. Mr.
X was shown a copy of this order by residents of a gated residential society when
he went there to demolish
the society gate. Mr. X tore down the copy of the order handed over to him stating
that “High Court Judges
have got no work to do except passing whimsical orders while sitting in air-
conditioned rooms and
demolished the gate of the society. Whether Mr. X is liable to be punished for
criminal contempt?
(a) Yes, because his statements interfere with, the due course of any judicial
proceeding as he is willfully
breaching order of the Court.
(b) No, he was using his freedom of speech and expression.
(c) No, as this amounts to fair criticism on merits of judicial order;
(d) Yes, because in COVID pandemic he was demolishing the building obstructing the
course of justice.
Passage (Q.91-Q.95): The NDPS Act, 1985 is the principal legislation applicable in
India, through which
the state regulates the operations of narcotic drugs and psychotropic substances.
It provides a stringent
framework for punishing offences related to illicit trafficking in narcotic drugs
and psychotropic substances
through imprisonments and forfeiture of property. Section 27A of the NDPS Act,
1985, prescribes the
punishment for financing illicit traffic and harbouring offenders. The NDPS Act
says that “narcotic drug”
means “coca leaf, cannabis (hemp), opium, popy straw and includes all manufactured
drugs”. Further,
“psychotropic substance” refers to “any substance, natural or synthetic, or any
natural material or any salt
or preparation of such substance or material included in the list of psychotropic
substances specified in the
Schedule.” The aim of the NDPS Act is to prohibit “the manufacture, production,
trade, use, etc. of narcotic
drugs and psychotropic substances”, except for medical or scientific purposes by
medical experts.
The Act provides for lawmakers to expand the list of psychotropic substances or
remove items from it on
the basis, of among other things, “information and evidence which has become
available to it with respect
to the nature and effects of, and the abuse or the scope for abuse of, any
substance (natural or synthetic)
or natural material or any salt or preparation of such substance or material”.
In Sami v. State of Uttar Pradesh, the police caught a person with 25 kilograms of
unlicensed Bhang (a
cannabis preparation) on him. It was held that while the Act defines
cannabis(hemp), there is no explicit
inclusion of bhang under this definition and that it was not a narcotic or
psychotropic substance. Thus, its
possession would not lead to a conviction under the NDPS Act if procurement of
bhang is within permissible
limits of 25 kgs.
91. X and Y are US citizens of 25 and 26 years old respectively. They knew that
consumption of drugs is illegal
in their country so they came India to consume the same. They tried Indian Drugs in
very little amount in
order to understand its quality and nature of substances. But while tasting a very
less amount of drugs
without a doctor’s prescription, both get caught by police officials. Now decide
whether they can be held
liable for consumption of drugs taking the objective of the act in consideration?
(a) No, both of them cannot be held liable as they both are US citizens over whom
Indian laws are not
applicable.
(b) No, both of them cannot be held liable as NDPS act allows for less consumption
of drugs up till 25 kgs
therefore minimal amount of consumption is not illegal.

. Page 23 of 36
(c) Yes, both of them can be held liable as NDPS act prohibits of consumption of
drugs therefore even a
minimal amount of consumption is illegal.
(d) No, because they are majors, they have the right to decide.
92. Cran and Berry are UK citizens of 32 and 28. They knew that consumption of
drugs is illegal in their country
so they came India to consume the same. They tried Indian Drugs - Bhang in very
little amount (5kg) in
order to understand its quality and nature of substances. But while tasting a
minimal number of drugs
without a doctor’s prescription, both get caught by police officials. Now decide
whether they can be held
liable for consumption of bhang?
(a) No, both of them cannot be held liable as they both are UK citizens over whom
Indian laws are not
applicable.
(b) No, both of them cannot be held liable as NDPS act is not applicable on Bhang
in less amount hence
consumption is not illegal.
(c) Yes, as bhang is a narcotic substance and the NDPS Act categorically bans
consumption of the same.
(d) Yes, asmanufacture, production, trade and use of narcotic drug and psychotropic
drug except medical
and scientific purposes is punishable .
93. In Sami v. State of Uttar Pradesh as in the passage, the police caught a person
with 25 kilograms of
unlicensed Bhang (a cannabis preparation) on him. It was held that while the Act
defines cannabis(hemp),
there is no explicit inclusion of bhang under this definition and that it was not a
narcotic or psychotropic
substance. Further if it was held that , its possession would not lead to a
conviction under the NDPS Act.
However, if bhang contained Tetrahydrocannabinol (THC) then it would come under
narcotic and
psychotropic substance. Suresh is a law student and was also a chain smoker. He was
famously known
for his psychotropic substances consuming habits therefore he was renamed as ‘ACID’
among his friends.
But one day when Suresh was about to deliver some of the substances i.e.charas and
bhang (with THC),
he fell inside a police trap and was substantially charged under NDPS act. Decide
can ACID be held liable
for the same under the NDPS act?
(a) No because ACID was not himself consuming these substances, he was only
delivering the same work
as a peddler.
(b) No because Bhang is not covered under NDPS act.
(c) Yes, because consumption, possessionand distribution of psychotropic substances
without a doctor’s
prescription, a person can be booked.
(d) Yes, only and if bhang was more than 25 kgs.
94. Section 54 of NDPS act states “54. Presumption from possession of illicit
articles. In trials under this Act, it
may be presumed, unless and until the contrary is proved, that the accused has
committed an offence
under this Act in respect of: (a) any narcotic drug or psychotropic substance or
controlled substance;
(b) any opium poppy, cannabis plant or coca plant growing on any land which he has
cultivated; (c) any
apparatus specially designed or any group of utensils specially adopted for the
manufacture of any narcotic
drug or psychotropic substance or controlled substance; or (d) any materials which
have undergone any
process towards the manufacture of a narcotic drug or psychotropic substance or
controlled substance.”
X was found to possess opium consequently he was accused of possession and
consumption of drugs
under the NDPS act. In district Court, the accused was asked to produce contrary
evidence that means the
burden of proof was put on the accused. Accused filed a writ petition against
wrongful proceedings taken
in district Court where he raised ‘onus of burden proof shall be on the petitioner,
not on the accused.’
Decide what view can be taken by the High Court?
(a) HC will dismiss the petition in favor of the accused as it is the general
principle of law i.e., the onus of
burden of proof shall lie on petitioner only.
(b) HC will dismiss the petition against the accused as in accordance with section
54 of the NDPS act, the
accused is obliged to provide evidence against the petitioner.
(c) Accused cannot file a writ petition as a person who seeks equity must come with
clean hands.
(d) None of the above.

. Page 24 of 36
95. Maresh is a medical student and was also a chain smoker. He was famously known
for his psychotropic
substances consumption therefore he was renamed as ‘MD’ among his friends. However,
one day he was
found growing coco leaf in his room. The police arrested him instantly. But Maresh
insisted that it was for
his medical science project. Decide can MD be held liable for the same under the
NDPS act?
(a) Maresh is guilty asproduction of narcotic drugs is prohibited except by medical
experts.
(b) Maresh is not guilty as growing coco leaf was a part of his project.
(c) Maresh is guilty as he was found manufacturing coco leaf by growing it in his
room.
(d) Maresh is not guilty as his research might lead to a breakthrough in the field
of medical science.
Passage (Q.96-Q.100): The Supreme Court observed that, in a consumer case, the onus
of proof that
there was deficiency in service employed is on the complainant and without any
proof of deficiency, the
opposite party cannot be held responsible for deficiency in service, Justices
Hemant Gupta and V.
Ramasubramanian observed.
If the complainant is able to discharge its initial onus, the burden would then
shift to the respondent in the
complaint. The rule of evidence before the civil proceedings is that the onus would
lie on the person who
would fail if no evidence is led by the other side. Therefore, the initial burden
of proof of deficiency in service
employed was on the complainant, but having failed to prove that the result of the
sample retained by the
appellant at the time of consignment was materially different than what was
certified by the appellant, the
burden of proof would not shift on the appellant. Thus, the Commission has erred in
law to draw adverse
inference against the appellant.
In this regard, the bench referred to the decision in Ravneet Singh Bagga v. KLM
Royal Dutch
Airlines (2000) 1 SCC 66 in which it was observed thus: The deficiency in service
cannot be alleged without
attributing fault, imperfection, shortcoming or inadequacy in the quality, nature
and manner of performance
which is required to be performed by a person in pursuance of a contract in
relation to any service
employed. The burden of proving the deficiency in service employed is upon the
person who alleges it.
96. Mr. Ghasitaram was witnessing berthing difficulties, cough and fever therefore
his brother Mr. Haldiram
admitted him to KGF hospital, and on primary diagnosis doctors revealed that the
patient was suffering
from covid 19 as well as his preexisting chronic lung dieses ILD and was already in
a critical condition.
Doctors isolated the patient and treated him according to the Covid guidelines of
ICMR despite all that he
died. Haldiram claimed that due to the deficiency of service on the part of KGF
doctors his brother died.
Decide the Hospital’s liability:
(a) KGF hospital is liable as there was a deficiency of service on their part and
due to this Mr. Ghasitaram
died.
(b) KGF hospital is not liable as they did not cause Ghasitaram’s death, he died
due to his medical
condition.
(c) Neither KGF Hospital nor Mr. Haldiram will be liable for there is no deficiency
of services on either of
the parties side.
(d) KGF hospital is not liable as they did no fault or imperfection on their part
that could have led to
Haldiram’s death.
97. Mr. Vex was fond of different types of candles and used to lit them in the
evening every day throughout his
house. One day while he was enjoying his evening with a coffee in candle lights,
his neighbors called him
for small talk and when he returned, his house was on fire, suddenly a passer-by
Mr. Water came who was
afire brigade by a profession came to Mr. Vex’shelp tried to extinguish the fire
however due to his fault fire
got worse and Mr. Vex’s house burned to ashes. Mr. vex filed a complaint against
that fire brigade claiming
deficiency in service.
(a) Mr. Water is liable for deficiency of service on his part as due to his fault
the fire got worse and Mr.
Vex’s house burned to ashes.
(b) Mr. Water is not liable as all the incidence was a mere accident.
(c) Mr. Water is not liable as Mr. Vexdid not employ his services.
(d) Mr. Water is liable for it was his fault, imperfection, shortcoming in
providing services which led to Mr.
Vex’s house on fire

. Page 25 of 36
98. Mr. Baburao Ganpatrao Apte is a cynical man and trusts no one in this world. He
had a very old model of
washing machine of Octopus Company of which the company itself has stopped the
production years ago.
One day the washing machine stopped working therefore he called Mr. Raju the
service man of Octopus
Company to repair it. Mr. Raju repaired it but advised Mr. Babu Rao Apte to buy new
model and left. Babu
Rao Apte was not satisfied with his work and claimed deficiency of service from
Raju. Which of the following
statement is true?
(a) The initial onus of prove that there was no deficiency of service is on Mr.
Raju.
(b) The onus of proof that there was deficiency of service is on neither of the
parties; .
(c) Mr. Baburao Ganpatrao Apte could not claim compensation without providing any
material evidence
and proof of Mr. Raju’s deficiency of service.
(d) The onus of proof that there was deficiency of service is on Octopus Company.
99. Whale Ltd. is engaged in business to Dolphin Ltd. for years, for providing
services for inspection of
groundnut procured by it for the purpose of exporting the same to Greece and Other
lands. However, on a
field inspection of consignment Dolphin Ltd. found that groundnut did not meet the
products specifications
at the time of loading of consignment. Dolphin limited filed a complaint in the
National Consumer Disputes
Redressal Forum. Decide.
(a) Whale Ltd. is liable for deficiency of service as the groundnut inspected by it
did not meet the products
specifications.
(b) Whale Ltd. is not liable as they did no fault or imperfection on his part.
(c) Whale Ltd. is not liable as he is not contractually bound to meet the product
specification and this was
only a mere formality.
(d) Whale Ltd. is liable for there was a fault on their part
100. Which of the following statement is not true in reference to the given
passage?
(a) The burden of proof is always on the person who claims that there was a
deficiency of service on the
part of the service provider.
(b) If sample retained by the appellant at the time of consignment is materially
different than what was
certified by the appellant, the burden of proof would shift on the appellant.
(c) A service provider is needed to act in a manner that a reasonable and prudent
service provider would.
(d) Without any proof of deficiency, the opposite party cannot be held responsible
for deficiency in service.
Passage (Q.101-Q.105): In the wake of the prevailing situation in Afghanistan where
the Taliban have
seized power, India has introduced a new category of e-visa for Afghan nationals to
fast-track their
applications for entry into India. These visas will be valid for six months only
and will be granted only after
security clearance, sources said. “Ministry of Home Affairs (“MHA”) reviews visa
provisions in view of the
current situation in Afghanistan. A new category of electronic visa called “e-
Emergency X-Misc Visa”
introduced to fast-track visa applications for entry into India,” a statement from
MHA.
Sources said the visa is being granted to facilitate the stay of Afghan nationals
fleeing Afghanistan for six
months in India. There is no clarity yet on what will happen after this validity
period expires. India does not
have a refugee policy and grants shelter to foreigners facing persecution in their
countries on a case-to#case basis.
“Such a visa may be granted only with single entry and for the specific duration
taking into account the
purpose of visit. If the visa is granted for a period of stay exceeding 185 days.
This visa will be non#extendable and non-convertible to any other type of visa,”
the MHA policy document on X-Misc visa says.
India has in the past granted long duration visas to Afghan nationals — of all
religions — facing persecution
in that country and a large number of Afghans who fled the country during the first
Taliban takeover. Other
foreigners in this category include those from Pakistan, Iraq, Sudan, foreigners of
Pakistani origin and
stateless persons.

. Page 26 of 36
101. Aaban Mohamad and Shahid were companion of each other and fled away form
Afghanistan on 26th of
March and reached India on 28th of March and are now refugees and have obtained
visa under e#Emergency X-Misc Visa for 6 months. But they stayed in Jammu till
September for complete 6 months, and
have not left Indian frontiers since then. Decide.
(a) They have completed the Time period given, so they have to go through
punishments.
(b) No, they can’t be punished as they are refuges and they are here to shorten
their problem not to
escalate.
(c) Their visa shall now be non-extendable and non-convertible to any other type of
visa.
(d) None of the above.
102. Sundar Pichai an employee at Google in Silicon Village is an Indian Origin
Citizen but is resident of United
Kingdom. He used to come to India often to meet his parents. He wanted to visit
India from UK and
consequentlyapplied for visa under E-Visa under the e-Emergency X-Misc for 6
months. Decide
(a) His visa will be rejected for the visa is for Afghan nationals who are refugees
and fleeing from their
home country;
(b) His visa will be accepted for it is granted for entry into India;
(c) His visa will be rejected for the visa is granted for single entry to India
only;
(d) His visa will be accepted for he has applied for the visa for 6 months as per
the provision.
103. Sharbat Gula and Ashraf Ghani are refugees and migrants from Afghanistan and
Bangladesh as they were
facing persecution in their home country they fled to India, wherein Sharbat Gula
has procured visa e
emergency X Misc Visa, however as India does not have any Refugee policy or have
not signed any
convention regarding it with Bangladesh, Ashraf Gula has not procured any visa. In
the light of the present
facts decide Indian Government declare can Ashraf Ghani an illegal
Migrant/refugees?
(a) Yes, Ashraf Ghani is an Afghan citizen and he can be declared Illegal
migrant/refugee by the competent
Authority.
(b) Yes, Ashraf Ghani can be declared illegal migrant/refugee by the Competent
Authority.
(c) Can’t be determined as this needs to decided case to case basis.
(d) No, Ashraf Ghani can’t to declared illegal Migrant/Refugee as, he lives under,
FRRO and can’t be
declared Illegal as its inhuman and Constitution does not allow us to do that for
it ensures right to live
with dignity.
104. Due to Taliban attack on Afghanistan’s administration in its capital, Many
Afghan citizens have left
Afghanistan, Ram Verma was one of them. He forged a fake passport and have criminal
charges against
him. Considering India is an active member of United Nations High Commissioner for
Refugees, what will
happen?
(a) Ram Verma will not be able to enter India as he has forged the passport.
(b) Ram Verma must have done crimes in Afghanistan including forging passport but
Taliban is bigger
threat than anything, so he should be allowed to enter India.
(c) Ram Verma is not a potential threat to the country, but have criminal charges.
(d) Ram will not be allowed to enter India for he does not have the requisite visa.
105. Government of India has started implementing Citizen Amendment Act (“CAA”) in
India despite the
enforceability of the Foreigners Act, 1946. .The Citizen Amendment Act strictly
refuses Muslims to be the
citizen of country. Shayra, an engineer from profession and Muslim by religion,
when Taliban attacked
Afghan and gained control over government she fled from Afghan to India. Before CAA
came into its
applicability, she had procured emergency visa for 6 months owing to state of
affairs in Afghan. Decide. .
Decide.
(a) Shayra will be allowed to stay in India for the law accords equal protection to
all.
(b) Shayra will not be allowed to stay in India as there is clash between the
provisions of CAA and category
of visa launched by India, hence can not be determined.
(c) Shayra will be allowed to stay in India for she has procured the requisite
visa;
(d) Shayra will not be allowed to stay in India in India as she is a Muslim and CAA
bars the same.

. Page 27 of 36
SECTION - D: LOGICAL REASONING
Directions (Q.106-Q.135): Read the passage carefully and answer the questions that
follow.
Passage (Q.106-Q.108): There is a growing feeling, among those who have the
responsibility of managing
large economies, that the discipline of economics is no longer fit for purpose. It
is beginning to look like a
science designed to solve problems that no longer exist. A good example is the
obsession with inflation.
Economists still teach their students that the primary economic role of government
- many would insist, its
only really proper economic role - is to guarantee price stability. We must be
constantly vigilant over the
dangers of inflation. For governments to simply print money is, therefore,
inherently sinful. If, however,
inflation is kept at bay through the coordinated action of government and central
bankers, the market should
find its "natural rate of unemployment," and investors, taking advantage of clear
price signals, should be
able to ensure healthy growth. These assumptions came with the monetarism of the
1980s, the idea that
government should restrict itself to managing the money supply, and by the 1990s
had come to be accepted
as such elementary common sense that pretty much all political debate had to set
out from a ritual
acknowledgment of the perils of government spending.
This continues to be the case, despite the fact that, since the 2008 recession,
central banks have been
printing money frantically in an attempt to create inflation and compel the rich to
do something useful with
their money, and have been largely unsuccessful in both endeavours. We now live in
a different economic
universe than we did before the crash. Yet the language of public debate, and the
wisdom conveyed in
economic text books, remain almost entirely unchanged.
One expects a certain institutional lag. Mainstream economists nowadays might not
be particularly good at
predicting financial crashes, facilitating general prosperity, or coming up with
models for preventing climate
change, but when it comes to establishing themselves in positions of intellectual
authority, unaffected by
such failings, their success is unparalleled. To this day, economics continues to
be taught not as a story of
arguments but rather as something more like physics, the gradual realization of
universal, unimpeachable
mathematical truths.
106. Which of the following most accurately expresses the main point of the second
paragraph of the passage?
(a) The 2008 financial crisis has affected our financial systems so severely that
we cannot do anything to
make it right.
(b) The 2008 financial crisis has changed many concepts which we assumed and,
therefore, we must
evolve in our understanding of economics.
(c) The 2008 financial crisis made us print a lot of money in an attempt to force
the rich to use their money,
but it did not work.
(d) The 2008 financial crisis should be taught in our textbooks as a reminder to
what we could have done
differently.
107. Which of the following cannot be inferred from the passage?
(a) Economists are not very good at their jobs but boast about their intellectual
prowess.
(b) The 2008 crisis has deemed the assumptions that came with monetarism in the
1980's obsolete.
(c) Economics must evolve as it is not able to solve the problems of the present.
(d) Economics is taught more like Physics
108. Which of the following is the author most likely to agree with?
(a) Governments cannot print excess money as it would lead to inflation.
(b) Today, economists are successfully predicting financial crashes and preventing
climate change.
(c) Banks have printed extra money in the past to induce inflation in the markets
artificially.
(d) Economists are moving towards the mathematical aspect of economics as it
provides more promising
results.

. Page 28 of 36
Passage (Q.109-Q.113): The deliberate or unintentional spread of misinformation,
despite capturing
widespread public attention, remains as rampant as ever, showing up recently in the
form of false claims
about COVID-19 vaccines, the Capitol riot, and many other instances. This
“infodemic” is polarizing politics,
endangering communities, weakening institutions, and leaving people unsure what to
believe or whom to
trust. It threatens the foundations of democratic governance, social cohesion,
national security, and public
health.
Misinformation is a long-standing problem that demands long-term, sustainable
solutions as well as short#term interventions. We've seen a number of quicker,
technological fixes that improve the social media
platforms that supply information. Companies like Facebook and Twitter, for
example, have adjusted their
algorithms or called out problematic content. We've also seen slower, human-centred
approaches that
make people smarter about the media they demand to access online. Evidence-driven
educational
programs, for instance, have made people better at discerning the reliability of
information sources,
distinguishing facts from opinions, resisting emotional manipulation, and being
good digital citizens.
It hasn't been enough. If we're to stop misinformation and its insidious effects,
we need to radically expand
and accelerate our counterattacks. It will take all sectors of society: business,
non-profits, advocacy
organizations, philanthropists, researchers, governments, and more. We also need to
balance our efforts.
For too long, too many resources and debates have focused on changing the
technology, not educating
people. This emphasis on the supply side of the problem without a similar
investment in the demand side
may be a less effective use of time and energy and clearly, it has been the
opposite of the ideal steps that
needed to be taken. While technology-centered, self-policing solutions—filtering
software, artificial
intelligence, modified algorithms, and content labeling—do have the ability to make
changes quickly and
at scale, they face significant ethical, financial, logistical, and legal
constraints.
109. Which of the following statements will the author deem true?
(a) The spread of misinformation is not a recent problem that has cropped up in the
world.
(b) Amidst the huge amount of negative news that’s emanating in the society today
some positive
misinformation is good for us as well, especially during the pandemic times.
(c) Institutions that supply information can themselves solve the problems with
certain steps. Only the
intent from these institutions is needed. It has been lacking amongst them. Their
steps haven’t been
enough.
(d) None of the above.
110. Which of the following, if true, weakens the author's claim?
(a) A startling research observation has been that 70% of the information available
on the internet today is
misinformation.
(b) It has been observed by a large number of studies that major corporations
distribute more information
than individuals.
(c) International laws do not prohibit spreading misinformation by citizens of
another nation.
(d) All people are against spreading and receiving misinformation, yet they are
forced to get it because of
careless policies of organizations supplying them with such false facts.
111. Consider the following instances –
1. Tony Stark receives a message from Steve Rogers (a prankster, by the way) that
the earth is a flat
surface and the people who call it round are mentally unstable. There are also some
supposedly "true
scientific facts" to support this claim. Tony, in his innocence, believes this and
forwards the message
unknowingly.
2. Albert is a pedant and a researcher who devours knowledge. He gets fascinated by
Facebook and
Twitter. After some time, though, he gets frustrated with misinformation and quits
social media
altogether.
3. The new sound space social media Addabaaz has become a field-space for
misinformation with the
celebrity Angana Raha, spreading misinformation and hate. Addahouse changes their
policy that
whenever such instances occur, and they receive complaints from the users, they
shall remove the
problematic content.
Which of the above instances can be said to be examples of what has been mentioned
in the passage?
(a) Both 1 and 2 (b) Both 2 and 3
(c) Both 1 and 3 (d) All 1, 2 and 3
. Page 29 of 36
112. Who should have the most significant responsibility in taking steps to
immediately stop the menace of
misinformation out of the following?
(a) The ordinary people, because they are the victims of the misinformation.
(b) The companies, because they are technologically much more advanced than the
means available to
counter the spread of misinformation, plus the misinformation is happening on their
platforms, so they
are in a better position to stop this.
(c) The political class, because they can educate the common folks in a much better
way than anyone
else.
(d) All of the above have equal responsibility.
113. The statement "Clampdown on information providing organizations is not enough"
is:
(a) Probably True (b) Definitely True
(c) Probably False (d) Definitely False
Passage (Q.114-Q.118): A lot of us think that the World Wide Web and the internet
are the same, but they
aren’t. The Web is the most popular way to access online data through hyperlinks
and websites; while the
internet, is a vast network of computers and servers on which the World Wide Web
operates. The internet
was a tool for scientists, engineers and the military; the web made it accessible
to everyone else.
Berners-Lee worked at CERN, where he developed the very first webpage; it went live
in August 1991, is
still active, and is probably the world’s first website (bit.ly/2SnA7zy). It was in
1994, however, that the World
Wide Web Consortium (W3C) founded by Tim Berners-Lee set up protocols, guidelines
and standards for
the web, and now-familiar terms like TCP (Transmission Control Protocol), IP
(Internet Protocol) and HTTP
(Hypertext Transfer Protocol) were born.
The founding philosophy of the Web was for it to serve as a democratizer and
equalizer, to empower the
long tail and eliminate monopolies and intermediaries. The Web did solve three big
problems for us: an
information problem with search and wikis, a communication problem with email and
messenger tools, and
a distribution problem with file-sharing and e-commerce. But it could not address
the two big problems that
it was supposed to solve: one of trust and security, and another of
disintermediation—its original
philosophy. In fact, the rise of big tech companies has given us intermediaries
that are far more powerful
than ever before. They literally own most of our online data and information. In
that sense, they own us.
Now comes the Block chain technology.
The reason for the excitement around blockchain is that it is supposed to solve our
unsolved problems—
of trust and of inequality—and thereby bring us closer to the original vision of
Time Berners-Lee and his
co-conspirators. So, it is not surprising what he said in the Financial Times, as
he announced that Sotheby’s
would auction off the original source code of the Web: The NFT project was his
“first foray into crypto", but
he saw similarities in his original vision for the web and the philosophy behind
the decentralized network
of Ethereum’s blockchain, which underpins most NFTs. It also resonates with his
latest project, Solid, which
is designed to give us back control of our personal data. “The blockchain and Solid
communities share the
motivations of wanting to empower people," he said, adding that blockchain projects
were motivated by
resistance to central control. Much like the open, democratic and decentralized
origins of the Web.

. Page 30 of 36
114. Which of the following conveys the core message discussed in the passage?
(a) The founding of blockchain technology and its application.
(b) The differences and similarities between internet and the World Wide Web and
their contribution to the
world.
(c) The original vision of web and its later deviation, only to be brought closer
through block chain
technology.
(d) The limitation of web and its failure in providing immunization against trust
and security and
disintermediation.
115. Which of the following is true about the World Wide Web?
(a) The web was invented to act as a democratizer and solve the issue of dearth of
information.
(b) The web is yet to solve the communication and the information problem.
(c) The web solved the issue of intermediation as per its original philosophy.
(d) The author corroborates the perception of the public that World Wide Web and
internet are the same.
116. What does the author mean by stating “They literally own most of our online
data and information. In that
sense, they own us.”
(a) Our data is our identity and those who own our data, own us.
(b) Data is the modern currency of exchange in the tech riddled world.
(c) The big companies do not care about data but about us, the users.
(d) Data is the collection of new information and is the costliest on earth.
117. In the last paragraph, the author ends the passage on a __________ note.
(a) Dismal (b) Hopeful (c) Pensive (d) Confrontational
118. According to the passage, why Berners-Lee is excited about the network of
Ethereum?
(a) Because the work is a complete imitation of his work.
(b) Because he will be well paid by Ethereum.
(c) Because his philosophy behind creating web matches with Ethereum’s technology.
(d) Because of the centralized network of Ethereum’s blockchain.
Passage (Q.119-Q.123): The French mathematician Pierre-Simon Laplace (1749-1827)
believed that the
Universe was a piece of machinery, and that physics determines everything.
Napoleon, who had read up
on Laplace's work, confronted him about the conspicuous absence of a creator in his
theory. 'I had no need
of that hypothesis,' came the reply. Laplace might have said the same thing about
free will, which his
mechanistic universe rendered superfluous.
Since Laplace's day, scientists, philosophers and even neuroscientists have
followed his lead in denying
the possibility of free will. This reflects a widespread belief among theoretical
physicists that if you know
the initial values of the variables that characterise a physical system, together
with the equations that
explain how these variables change over time, then you can calculate the state of
the system at all later
times. For example, if you know the positions and velocities of all the particles
that make up a gas in a
container, you can determine the positions and velocities of all those particles at
all later times. This means
that there should be no freedom for any deviation from this physically determined
trajectory.
Consider, then, that everything we see around us - rocks and planets, frogs and
trees, your body and brain
- is made up of nothing but protons, electrons and neutrons put together in very
complex ways. In the case
of your body, they make many kinds of cells; in turn, these cells make tissues,
such as muscle and skin;
these tissues make systems, such as the heart, lungs and brain; and these systems
make the body as a
whole. It might seem that everything that's happening at the higher, 'emergent'
levels should be uniquely
determined by the physics operating beneath them. This would mean that the thoughts
you're having at
this very moment were predetermined at the start of the Universe, based on the
values of the particle
physics variables at that time.

. Page 31 of 36
119. Which of the following statements may have formed the premise for Laplace's
argument that 'I had no need
of that hypothesis'?
(a) He was an atheist and did not believe in the existence of a sole creator, after
all.
(b) He proved that biological activity at the micro level is literally grounded in
the physical shape of
biological molecules.
(c) He might have proved or formulated the existence of all events in the Universe
on the basis of physics
and mathematical equations, rejecting the plausibility of free will.
(d) He failed to explain some of the events which were beyond the realms and reach
of theories of physics.
120. Which of the following would undermine the argument made in the passage that
if you know the initial
values of the variables that characterise a physical system, together with the
equations that explain how
these variables change over time, then you can calculate the state of the system at
all later times?
(a) Ions are atoms that have become electrically charged because they have lost or
gained an electron.
(b) Heisenberg's uncertainty principle introduces an unavoidable fuzziness and an
irreducible uncertainty
in quantum outcomes. You might know the value of one variable, such as a particle's
momentum; that
means you can't accurately detect another, such as its position.
(c) An apple is suspended from a branch of the tree by a string attached to its
stalk. It would thereby have
been turned into a pendulum, because the string constrained its motion. Instead of
dropping to the
ground, it would have swung back and forth in a circular arc under the branch, with
its state of motion
determined uniquely by its initial position and velocity.
(d) All of the above.
121. Which of the following can we infer from the passage above?
(a) The state of a system is described by what's known as its wave function, which
determines the
probabilities of different outcomes when events take place.
(b) At very small scales, theory of Physics underlies what's happening in the
world.
(c) The structure of the molecules is truly the secret of life.
(d) Outcomes don't depend only on the equations and the initial data.
122. Which of the following is most likely to be true had theories of physics and
mathematical equations failed
to explain some of the earthly events?
(a) Learning and memory offer example of how downward causal effect shapes the
underlying physics.
(b) If you seriously believe that fundamental forces leave no space for free will,
then it's impossible for us
to genuinely make choices as moral beings.
(c) The power of choice enables physiological systems such as the heart and brain
to function in a way
that is enslaved by the lower-level interactions, and choosing the outcomes of the
preferred interactions
from a multitude of options have been predetermined at the elementary level.
(d) None of the above.
123. Which of the following most accurately expresses the main point of the
passage?
(a) The confounding thing for free-will sceptics is that all outcomes don't depend
only on the equations and
the initial data; they also depend on randomization.
(b) In a physically determined world, there is freedom for deviation up to a
certain extent.
(c) There are events or things which Physics is unable to answer or reach to an
acceptable conclusion.
(d) An understanding of physics sees determinism at work in the Universe.

. Page 32 of 36
Passage (Q.124-Q.128): Some of us think. Many of us think that we think. And most
of us never think of
thinking. Very few use the grand prerogative of the mind, many never think, but
they think they do. Very
few of us really think because, to think, we should have the freedom to do so.
We touch the earth with our physical foot; we touch life with our psychological
foot. That is why in temples,
we leave our slippers outside. We leave certain coverings that cover our feet and
enter the temple barefoot.
In the nakedness, you discover the truth. Shoes cover our feet, and thereby we
don’t deeply connect to
God. Our psychological foot, too, is covered by psychological shoes, which are our
conditioning, dogmas,
opinions, and beliefs. With the psychological shoes on, we never touch a life.
Very few of us can look at a flower as it is. When somebody looks at a flower, they
look at it with the
psychology of likes and dislikes, of should or should not, or must or must not.
Therefore, very few of us
really look at the flower; instead, we look at our opinion of the flower- our likes
and dislikes.
So, we never see it as it is. Removing the shoe means removing the psychological
shoes of our
conditioning, our opinions, and dogmas. When you leave these and innocently look at
life, it is then you
see that something very different happens. If you analyze it, you will see that if
we are unhappy in life, it is
not because of what is; it is because of our opinion of how life should be. When
our view of life conflicts
with our life, this makes us unhappy. This unhappiness is a toxin, a disturbance in
our energy field. You
look at situations that you feel should not be, and you get angry. Your anger is
often directed as a
scapegoat, not the real genesis of your unhappiness.
124. What does the author talk about when he says ‘the grand prerogative of the
mind’?
(a) The illusion of thinking while not thinking in reality.
(b) Utilizing the power of mind to think freely.
(c) Not having the freedom to think.
(d) Actually thinking something/anything at all
125. Why has the author mentioned the psychological shoe?
1. To show that it is deceptive.
2. To show how it covers are true understanding of the world.
3. To show how it distorts our thinking.
(a) Only 1 (b) 1 and 2 (c) 2 and 3 (d) All of the above
126. Based on the information above, which is the most accurate?
(a) We are unhappy because we gauge anger from the wrong prism.
(b) To have an objective clarity about happiness, unhappiness and anything in life,
the opinionated shoe
needs to be left out.
(c) The real source of unhappiness and anger lies within us.
(d) The naked soul and the bare feet help connect well with the world.
127. It can be inferred from the passage that the reason of our unhappiness is-
(a) Our opinion is only reason responsible for lack of happiness in our lives.
(b) Consonance in what we think happiness is and what happiness in reality is.
(c) Our psychological shoe, that prevents us from connecting and analyzing things
in their natural colour.
(d) Differentiated opinions on life.
128. Statement: ‘Therefore, we never see it as it is.’
The above statement is:
(a) Conclusion statement. (b) Weakening statement.
(c) Strengthening statement. (d) Parallel reasoning.

. Page 33 of 36
129. In the next few years, molecular biologists will complete mapping the human
genome. This means that
scientists will be able to trace each health problem to a particular gene. It will
then be possible to fix this
problem by fixing the responsible gene. The hope is that several addictions, such
as smoking, alcoholism
and drug abuse, will be curable through gene therapy.
Which of the following assumptions is the above argument based on?
(a) Genetic manipulation is the sole way to cure addictions.
(b) In the future only genetics will matter in the world of medicine.
(c) Addictions are genetically induced, not caused by other factors.
(d) Every human being is addicted to something.
130. One morning after sunrise Deepak while going to school met Raj at road
crossing. Raj’s shadow was
exactly to the right of Deepak. If they were face to face, which direction was Raj
facing?
(a) South (b) North (c)East (d) West
Directions (Q.131): In each of the questions given below three statements are
followed by some
conclusions. You have to take the given statements to be true even if they seem to
be at variance from
commonly known facts. Read all the conclusions and then decide which of the given
conclusions logically
follows from the given statements disregarding commonly known facts.
131. Statements:
Some songs are not tunes.
All tunes are drums.
Some tunes are sticks.
Conclusions:
I. Some drums being songs is a possibility.
II. All those drums which are tunes can be songs.
(a) if only I follows. (b) if only II follows. (c) if either I or II follows. (d)
if both I and II follow.
Direction (Q.132-Q.134): In a family, there are six members- A, B, C, D, E and F. A
and B are a married
couple, A being a male member. D is the only son of C, who is the brother of A. E
is the sister of D. B is
the daughter-in-law of F, whose husband has died.
132. How F has related to A?
(a) Mother (b) Sister-in-law (c) Sister (d) Mother-in-law
133. How is E related to C?
(a) Daughter (b) Aunt (c) Cousin (d) Sister
134. Who is C to B?
(a) Brother (b) Brother-in-law (c) Son-in-law (d) Nephew
Direction(Q.135): In the question below are some conclusions are given followed by
set of statements.
You have to decide from which set of statements the given conclusions logically
follow(s) disregarding
commonly known facts.
135. Conclusions:
I. No puzzle is seating.
II. Some puzzles are coding.
III. Some syllogisms are not seating.
Statements:
I. All puzzles are coding. No seating is syllogism. Some syllogisms are puzzle.
II. No coding is seating. All syllogisms are puzzle. Some Puzzles are coding.
III. Some puzzles are syllogism. All syllogisms are coding. No coding is seating.
IV. No seating is puzzle. Some puzzles are syllogism. All syllogisms are coding.
(a) Only Statement I follows. (b) Only Statement II follows.
(c) Only Statement III follows. (d) Only Statement IV follows.

mock 18

Directions (Q.1-Q.30): Read the passage carefully and answer the questions that
follow.
Passage(Q.1-Q.5): We sometimes say we despair, but what we actually mean is that we
need or want
something strongly and urgently. If the person who wants something strongly and
urgently still has hope
despite, perhaps, a few unsuccessful attempts at securing the desired good, that
person may feel
desperate but is not in despair.
At other times, we may really give up hope, but only about a particular matter or
domain, not about life in
general. For instance, a person may, after changing jobs several times, despair of
his or her own prospects
of finding a suitable occupation. The one who experiences hopelessness about a
particular matter may be
said to despair, but this type of despair has a limited scope. The person afflicted
by it may have interests
or activities that he or she can fall back on. Thus, in Mrs. Dalloway, Virginia
Woolf describes a woman,
Clarissa Dalloway, who becomes hopeless about relationships but finds solace in
solitude. Despairing of
human relationships (people were so difficult), she often went into her garden and
got from her flowers a
peace which men and women never gave her.
There is something appealing about the ability to find ways to cope. That’s perhaps
especially true of an
ability such as Clarissa’s to make use of one’s own inner resources and find peace
in one’s own company.
There are times, however, when despair becomes global: one despairs of one’s life
or future as a whole.
Whatever people or activities are left to fall back on and help prevent a descent
into darkness prove
insufficient. One feels as though one has no lifeline left; that one’s life vest is
just too small to keep one
afloat.
Despair differs from resignation. While a person in both cases lacks hope, the one
who merely resigns him
or herself regards the current level of pain as bearable. If you resign, you accept
the hand you've been
dealt, however grudgingly, and you go on. The person in despair, by contrast, has
not simply lost hope but
judges the pain of the current situation to be intolerable. There is no prospect
for a better future, for sunnier
days, but neither can one go on like this. It is precisely this subjective sense
that the cross of life has
become too heavy to bear, that one does not have enough of a fight left within in
order to continue that we
call despair.
There is something profoundly lonely about despair. A burden that we share with
someone else is typically
not seen as too heavy to bear. Of course, a tragedy may afflict two people
simultaneously and destroy
intimacy instead of bringing the two closer, as may happen when a couple loses a
child. In that case, the
two people may despair despite carrying the same burden for the lack of intimacy
means that each carries
it on his or her own.
1. What is the overall tone of the passage?
(a) Critical (b) Objective (c) Acerbic (d) Apologetic
2. What is the purpose behind writing the third paragraph of the passage?
(a) To illustrate that the feeling of despair becomes so large that it is
impossible to deal with and any ray
of hope is just not enough.
(b) To suggest various ways to deal with the issue of despair as a course of
action.
(c) To find peace in one's own company as a coping mechanism.
(d) To substantiate various types of despair faced by people in general.
3. It can be inferred from the passage that,
(a) There are more differences than similarities between despair and resignation.
(b) There are similarities between desperation and resignation.
(c) A few unsuccessful attempts at achieving the desired result makes one
desperate.
(d) None of the above

. Page 3 of 36
4. A hypothetical or an analogous situation that correctly describes despair is
I. A person after getting rejected three times for a job interview decides to take
a break.
II. A student taking GMAT exam to better his previously secured score in the same
exam.
III. An aspirant after getting declared unfit medically for getting inducted into
the armed forces and
accepting the fact reluctantly chooses to become trainer to train other aspirants.
(a) II and III only (b) I and III only (c) I only (d) I and II only
5. Identify the statement(s) that is/are correct as per the information given in
the passage.
I. Despair can have a limited as well as a globalised scope.
II. One resentfully accepts the given situation and moves forward when one resigns.
III. Some people find peace in solitude, but a majority don't.
(a) I and III only (b) I and II only (c) II and III only (d) All I, II and III
Passage (Q.6-Q.10): Before the grass has thickened on the roadside verges and
leaves have started
growing on the trees is a perfect time to look around and see just how dirty
Britain has become. The
pavements are stained with chewing gum that has been spat out and the gutters are
full of discarded fast
food cartons. Years ago, I remember travelling abroad and being saddened by the
plastic bags, discarded
bottles and soiled nappies at the edge of every road. Nowadays, Britain seems to
look at least as bad.
What has gone wrong?
The problem is that the rubbish created by our increasingly mobile lives lasts a
lot longer than before. If it
is not cleared up and properly thrown away, it stays in the undergrowth for years;
a semipermanent
reminder of what a tatty little country we have now.
Firstly, it is estimated that 10 billion plastic bags have been given to shoppers.
These will take anything
from 100 to 1,000 years to rot. However, it is not as if there is no solution to
this. A few years ago, the Irish
government introduced a tax on non-recyclable carrier bags and in three months
reduced their use by 90%.
When he was a minister, Michael Meacher attempted to introduce a similar
arrangement in Britain. The
plastics industry protested, of course.
However, they need not have bothered; the idea was killed before it could draw
breath, leaving
supermarkets free to give away plastic bags.
What is clearly necessary right now is some sort of combined initiative, both
individual and collective, before
it is too late. The alternative is to continue sliding downhill until we have a
country that looks like a vast
municipal rubbish tip. We may well be at the tipping point. Yet we know that people
respond to their
environment. If things around them are clean and tidy, people behave cleanly and
tidily. If they are
surrounded by squalor, they behave squalidly. Now, much of Britain looks pretty
squalid. What will it look
like in five years?
6. As per the passage, the writer believes the best method of combating the plastic
problem
(a) is by introducing high tax structure against plastics.
(b) by taking on big plastic companies who pressurise the government.
(c) is by cleaning rubbish daily and not letting it pile up.
(d) is by joint venture, both individually and collectively against the menace.
7. As per the passage, the problem with garbage is
(a) that in the daily hustle bustle of modern life we do not pay much attention to
cleanliness and thus, affect
the environment.
(b) that rubbish keeps on accumulating and becomes a visible menace later, when it
is difficult to tackle.
(c) that our isolated and solitary existence has made us forget about the merits of
collaborative ventures,
and the ever-expanding garbage problem is one such example.
(d) that it gets little attention from either the media or the politicians and
thus, remains in the undergrowth
for years.

. Page 4 of 36
8. As per the passage, what did the author observe years ago?
(a) Ireland was in a pathetic and dirty condition until they introduced a tax on
non-recyclable carrier bags.
(b) Honest ministers trying to tackle the garbage problem were dismissed through
joint effort by the plastic
industry.
(c) The garbage situation abroad was much worse compared to that in Britain.
(d) The author was myopic in vision and failed to realise that the small bits of
garbage he had observed
would grow into a serious problem.
9. Which of the following is a common knowledge, as per the passage?
(a) Modern urban life is structured in a way which facilitates garbage
accumulation.
(b) People of all class and credential have lost their will to keep their city
clean.
(c) People do not respond to their immediate surroundings other than vested
interests.
(d) People behave according to what they see around them.
10. Why does the author talk about Michael Meacher?
(a) To show how he tried to follow the Irish example with a tax on plastic bags.
(b) To show how Ireland is the best tackler of contemporary garbage problems.
(c) To show how insidious the plastic industry is.
(d) To show how there are good people in the world, fighting the good fight.
Passage (Q.11-Q.15): The formation of a 'Committee for the Reform of Criminal Laws'
by the Union Home
Ministry with an apparently short time frame and limited scope for public
consultation has caused
considerable disquiet among jurists, lawyers and those concerned with the state of
criminal justice in the
country. Few would disagree with the idea that the current laws governing crime,
investigation and trial
require meaningful reform. There have been several attempts in recent decades to
overhaul the body of
criminal law, comprising the Indian Penal Code of 1860 vintage, the Code of
Criminal Procedure that was
rewritten in 1973, and the Indian Evidence Act that dates back to 1872. However,
comprehensive legal
reform is something that requires careful consideration and a good deal of
deliberation. One criticism
against the latest Committee is that it has begun its work in the midst of a
pandemic. This may not be the
ideal time for wide consultations. Activists and lawyers functioning in the
hinterland may be at a particular
disadvantage in formulating their opinions. The panel's mandate appears quite
broad: "to recommend
reforms in the criminal laws of the country in a principled, effective, and
efficient manner which ensures the
safety and security of the individual, the community and the nation; and which
prioritises the constitutional
values of justice, dignity and the inherent worth of the individual." This is vague
and open to multiple
interpretations. It is also not clear why the Law Commission has not been vested
with this task.
The lack of diversity in what is an all-male, Delhi-based committee has also been
adversely commented
upon. In 2003, the Justice V.S. Malimath Committee on reforms in the criminal
justice system had come
up with some far-reaching suggestions, some of which became part of changes in
criminal law. However,
it also attracted criticism over the suggestion that the standard of evidence be
reduced from "beyond
reasonable doubt" to "clear and convincing". The Justice Verma panel came up with a
comprehensive and
progressive report on reforms needed in laws concerning crimes against women in
2013 in barely one
month, but its speed was probably due to the limited mandate it had. If at all
criminal law is to be reformed,
there should be a genuine attempt to reach a wide consensus on ways to speed up
trials, protect witnesses,
address the travails of victims, improve investigative mechanisms and, most
importantly, eliminate torture.
11. Which of the following can be concluded from the given passage?
(a) The committee is expected to take wide consultations.
(b) Public consultation is a trivial part of introducing a reform in laws.
(c) A diverse committee (including both male and female) for law reforms might not
be welcomed.
(d) The reforms in Criminal Laws require proper deliberation and inclusion of a
wider ambit of consultants
coming to proper consensus.

. Page 5 of 36
12. Which of the following statements is incorrect as per the given passage?
(a) Careful consideration and a good deal of deliberation is required to overhaul
the body of criminal law.
(b) A few will disagree on the fact that India's current criminal laws need
reforms.
(c) For Criminal Law to be reformed, there should be a wider concensus to speed up
trials, to protect
witnesses, to highlight the plight of victims, to improve investigative procedure
and to rule out tortures.
(d) The panel which is established for working on reforms has a vague mandate which
is open for multiple
interpretations.
13. Which of the following is similar in meaning to the word 'overhaul' mentioned
in the passage?
(a) Setback (b) Complication (c) Remodel (d) Aggrandize
14. Which of the following can be the reason that Justice V.S. Malimath Committee
was criticised for its
suggestion to reduce the standard of evidence?
(a) It might help the criminals to easily get acquitted from the punishment.
(b) It might increase the efforts taken by the victim's lawyer to collect the
evidence by itself.
(c) It will make it easier to prove the case against the person who might not be
the culprit and will shatter
hope of reasonable arguments from his/her side.
(d) It has seen positive results in the countries wherever it has been applied.
15. Which of the following can safely be said about the Indian criminal justice
system, as per the given
passage?
(a) Diligent efforts have been taken to reform the laws.
(b) The committee set up for reforms and the people of the country has same opinion
on each of the
reforms presented by the committee.
(c) The Indian criminal justice system is not able to provide timely justice to the
victim.
(d) None of the committees was able to present some far-reaching reforms in their
recommendations.
Passage (Q.16-Q.20): What is a just society? In seeking a defensible conception of
justice, it behooves
us to start with the assumptions of the libertarian perspective, the view that
appears to endorse the least
enforcement of morality. I propose to show that this libertarian view, contrary to
what its defenders usually
maintain, requires a right to welfare; and that further, this right to welfare
(which is also endorsed by a
welfare liberal perspective), leads to substantial equality advocated by
socialists.
Let us begin by interpreting the ideal of liberty as a negative ideal in the manner
favoured by libertarians.
So understood, liberty is the absence of interference by other people from doing
what one wants or is just
able to do. Libertarians characterize their political ideal as requiring that each
person have the greatest
amount of liberty morally commensurate with the greatest amount of liberty for
everyone else. Interpreting
their ideal in this way, libertarians claim to derive a number of more specific
requirements, particularly a
right to life, freedom of speech, press, and assembly; and a right to property.
Here it is important to note that the libertarian's right to life is not a right to
receive from others the goods
and resources necessary for preserving one's life; it is simply a right not to have
one's life interfered with
or ended unjustly. Of course, libertarians allow that it would be nice of the rich
to share their surplus
resources with the poor. Nevertheless, they deny that government has a duty to
provide for such needs.
Some good things, such as providing welfare to the poor, are requirements of
charity rather than justice,
libertarians claim. Accordingly, failure to make such provisions is neither
blameworthy nor punishable. As
a consequence, such acts of charity should not be coercively required. For this
reason, libertarians are
opposed to coercively supported welfare programs.
Now, in order to see why libertarians are mistaken about what their ideal requires,
consider a conflict
situation between the rich and the poor. In this conflict situation, the rich, of
course, have more than enough

. Page 6 of 36
resources to satisfy their basic needs. In contrast, imagine that the poor lack the
resources to meet their
basic needs even though they have tried all the means available to them that
libertarians regard as
legitimate for acquiring such resources. Under circumstances like these,
libertarians maintain that the rich
should have the liberty to use their resources to satisfy their luxury needs if
they so wish. Libertarians
recognize that this liberty might well be enjoyed with the consequence that the
satisfaction of the basic
needs of the poor will not be met; they just think that liberty always has priority
over other political ideals
And since they assume that the liberty of the poor is not at stake in such conflict
situations, it is easy for
them to conclude that the rich should not be required to sacrifice their liberty so
that the basic needs of the
poor may be met.
16. What of the following correctly describes the relationship between the second
and third paragraphs of the
passage?
(a) The second paragraph is negatively co-related with the third paragraph.
(b) The third paragraph elaborates on one of the points the premise of which is
laid down in the second
paragraph.
(c) The third paragraph presents an alternate argument to the argument presented in
the second
paragraph.
(d) The second paragraph discusses an issue whose course of action is being laid
out in the third
paragraph.
17. Identify the statement(s) which is/are incorrect as per the information
mentioned in the passage.
I. The holistic definition of just society can be derived from the passage.
II. The author fully supports the ideas proposed by libertarians.
III. Liberty is the absence of interference by other people from doing what one
wants or is just able to do.
(a) II and III only (b) I and II only (c) I only (d) All I, II and III
18. What is the contextual meaning of the following as used in the passage?
Behoove
(a) fearfully delighted (b) overwhelmingly amused
(c) be incumbent upon (d) be at peace with something
19. What is the overall tone of the passage?
(a) Apologetic (b) Humorous (c) Pedestrian (d) Critical
20. What would be the ideal title for the given passage?
(a) The Idea of Negative Liberty (b) Liberty taking a toll on the poor
(c) Liberty and Equality (d) A Universal Right to Welfare
Passage (Q.21-Q.25): Read the following passage carefully and answer the questions
given below it.
The Indo-Pacific economic space continues to be open to reconfiguration, triggered
as it has been by the
US withdrawal from the Trans Pacific Partnership (TPP) earlier this year. There
have since been many
conjectures on how the region may see a China-led economic order, particularly so
as the Japanese
attempts for a TPP revival have not yielded any concrete positive outcomes so far.
Alternatively, Japan-India relations that have been in the forefront with Prime
Minister Abe's visit to India
earlier this month, are being considered as a possible counter balancing force in
the region. It is, therefore,
relevant and worthwhile to examine the relative placement of China and Japan in
India's economic relations
and discuss India's options in the larger regional economic context.
First, while China is India's top trading partner, Japan is not even among the top
ten trading partners for
India, and has not been so for almost a decade. In 2016-17, India's trade with
China was US$ 71 billion as
against US$ 13 billion with Japan. Interestingly, while India incurs a trade
deficit with both countries, it is
the deficit with China that attracts attention, even though the deficit with both
countries, as a share of total

. Page 7 of 36
bilateral trade, is almost equal at 70 per cent. With Japan, India has a
Comprehensive Economic
Partnership Agreement (CEPA) that was signed in 2011. After six years of its
implementation, growth in
bilateral trade with Japan has been insignificant and in fact, the rate of growth
of exports registered a
significant decline in 2016-17. In contrast, even without a preferential trading
arrangement, India's exports
to China have seen a positive growth over the last year.
Importantly, in the pharmaceutical sector, Japan has been insistent on quality and
regulatory standards,
particularly in respect of pharmaceutical goods which is a major export category
for India not just in bilateral
but in global trade too. Although China also imposes trade barriers for Indian
pharmaceutical exports, a
significant difference is to be noted with regard to their membership of regional
trade formulations - Japan
is a member of both the Regional Comprehensive Economic Partnership (RCEP) and the
now in#suspension TPP; China, like India, is a member only of the RCEP and not of
the TPP. Japan has been
very keen to revive the TPP with or without US membership. A consequence of this
differential membership
is that Japan, in the spirit of TPP trade rules, seeks higher and World Trade
Organisation (WTO)-plus
standards in its bilateral trade transactions that would be very hard for India to
comply with presently and
in the near future.
21. Which of the following is similar in meaning to the word 'conjecture' as
mentioned in the given passage?
(a) Evidence (b) Division (c) Qualm (d) Surmise
22. The author is most likely to agree with which of the following?
(a) Trade between India and China is affected due to the ongoing tension on the
border.
(b) The departure of US from the Trans Pacific Partnership had no effect on the
Indo-Pacific economic
space.
(c) Trade relations between India and Japan are not as flourishing as between India
and China.
(d) The TPP trade rules of Japan in its bilateral transactions with India will be
easy and relaxed.
23. Which of the following can be inferred from the last paragraph of the given
passage?
(a) Japan does not trust India with respect to pharmaceutical projects.
(b) Japan might be motivated by China's step to impose trade barriers on Indian
pharmaceuticals.
(c) Japan is not much concerned about the health and the wellbeing of its citizens.
(d) Japan strictly adheres to the quality and regulatory standards, especially in
the field of pharmaceuticals.
24. Which of the following is referred to by the phrase 'differential membership',
as mentioned in the given
passage?
(a) India- Japan trade relations are bound by different organizations, different
rules, and different standards
in comparison to China.
(b) The differences between the quality of trade between Japan and China with
India.
(c) The repulsive attitude of Japan towards the RCEP and its inclination towards
the standards of WTO in
its bilateral transactions.
(d) The implications of the trade relations are different in case of Japan and
China.
25. Which of the following can be said about the insignificant growth in bilateral
trade between India and Japan?
(a) China has superseded India in trade relations with Japan and has total control
of trade with Japan.
(b) Japan is apprehensive about the quality of imports from India and therefore, is
not conducive to further
the trade relations with India.
(c) India and Japan possibly have a difficult trade agreement with regards to
strict adherence to quality
control and regulations expected by Japan that do not allow for smooth trade
between the two countries.
(d) India is complacent with Japan's affinity towards the implementation of an
agreement equal to or above
the standards of WTO agreement.

. Page 8 of 36
Passage (Q.26-Q.30): I was born in a tribal family in a small village tucked away
in the hills of east Pusad
in eastern Vidarbha. Like many people from my background, I was oblivious to the
outside world, especially
the social arc of the caste society in India. Under the influence of Brahminic
Hinduism, my people practised
untouchability. We kept a cautious distance from the Mahar and Mang people.
After my Class 6, because of our financial condition, I dropped out of school and
worked in a landlord’s
field for six years. While taking the buffaloes out to graze, I chanced upon a book
on Dr B R Ambedkar
which left a lasting impression on my mind and heart. It was so strong that I
decided then and there to
resume my schooling.
I sought admission in the seventh class and did not stop till I had a post-graduate
degree in English. Over
time, I read more of Ambedkar’s literature that took me closer to the Phule, Shahu
and Ambedkarite
movements. I was fully convinced that without following Dr B R Ambedkar, Adivasi
people will have no
salvation whatsoever.
However, for a few years now, a misconception about Dr B R Ambedkar’s contribution
towards Adivasi
communities is doing the rounds among the Adivasi people in some states. Many
tribal social workers,
intellectuals, and even political leaders question Dr Ambedkar’s credibility. They
say that except two or
three articles in the Constitution that seek to uphold Adivasi interests, we find
nothing substantial
concerning the overall development of us forest people. Worse, most Adivasis in
India level a serious
charge against Dr Ambedkar — that he destroyed their identity as Adivasi. He called
the Adivasi people
Scheduled Tribes in the Indian Constitution.
Few know that Dr Ambedkar gave serious thought to developing tribal communities
even before the
promulgation of the Constitution. He held a conference in Mumbai on May 5-6, 1945,
to suggest proper
ways to achieve tribal progress. He had advised that a statutory commission based
on the South African
Constitution be created.
In Annihilation of Caste, Dr Ambedkar has dedicated Chapter 8 to Aboriginal Tribes
in India. He relevantly
delineated the life of Adivasis in India and pointed out the root causes for their
tragic conditions. Why they
remained so backward over the centuries, how their Hindu brethren treated them, and
what ways they
should adopt to escape from this predicament, have found excellent expression in
this small chapter.
I believe Dr Ambedkar did everything possible to improve the plight of the
Adivasis. It is a pity that Adivasis
did not get a visionary leader like him in the post-Independence era. They were
left to the grace of stooges
of political parties. Therefore, the pace of Adivasi progress is too slow. But the
educated younger
generation has now realized the importance of Dr Ambedkar’s teachings.
26. Which of the following is the opposite in meaning to the word ‘annihilation’?
(a) Construction (b) Extermination. (c) Obliteration. (d) Volte-face
27. All of the following are true, except
(a) Dr. Ambedkar’s contribution towards Adivasi communities cannot be marginalised.
(b) There is a lacunae of a visionary leader like Dr. Ambedkar post- independence.
(c) Worse, most Adivasis in India level a serious charge against Dr Ambedkar — that
he destroyed their
identity as Adivasi.
(d) Dr. Ambedkar inappositely portrayed the life of Adivasis in India and pointed
out the root causes for
their plight.
28. Which of the following best reflects the title of the passage?
(a) Dr. Ambedkar: a visionary. (b) Dr. Ambedkar & Adivasis: Some myths, many
truths.
(c) Marginalising Adivasis. (d) Deliberate Maligning of Dr. Ambedkar.

. Page 9 of 36
29. The author, through the passage, does which of the following?
(a) Corroborates the tribal social workers, intellectuals, and even political
leaders questioning of Dr
Ambedkar’s contribution towards Adivasi communities.
(b) Negates the criticism levelled against Dr. Ambedkar’s stance towards other
castes.
(c) Clears the misconception regarding the myth around Dr. Ambedkar’s trivial
contribution towards Adivasi
communities.
(d) Highlights the plight of the Adivasi communities who are at the mercy of the
political stooges.
30. Few know that Dr Ambedkar gave serious thought to developing tribal communities
even before the
promulgation of the Constitution. The underlined part of the sentence contains
grammatical error. From the
given options, choose the one that is grammatically most accurate.
(a) Few know that Dr Ambedkar gave serious thought to developing…
(b) A few know that Dr Ambedkar gave serious thought to developing
(c) Few knew that Dr Ambedkar gave serious thought to developing…
(d) Few know that Dr Ambedkar gave serious thought to develop

Directions (Q.66 – Q.105): Read the comprehensions carefully and answer the
questions based on it.
Passage (Q.66-Q.71): The Bombay High Court has observed that written submissions in
a dispute become
immaterial if the litigant's counsel doesn't rely on them before the Court of the
first instance. The Bench
went on to add that those submissions cannot subsequently be used to challenge any
order.
"Counsel's failure to argue written submissions is not a ground of review or, I
dare say, even appeal. It is
not a ground to assail any order of any Judge of any Court. If the written
submissions were to be relied on,
that ought to have been done during arguments, or, at any rate, while judgment was
being dictated in open
Court or at best shortly after the judgment or order was uploaded. These never-
argued written submissions
cannot be taken in hindsight."
Justice Patel remarked that allowing parties to take grounds in review pleas or in
appeals that
werenotargued initially injects an impermissible level of uncertainty into the
whole decision-making process.
"They must come to a point when a Court must say enough is enough and they cannot
succeed in taking
this ground further.
It was further found that there existed no order for permitting those written
submissions.
With this view, the Court went on to dismiss the review petition after imposing a
heavy cost of Rs.5 lakh
on the petitioners.
It was further observed that "the purpose of the Court is to make time for a
litigant, but no litigant is entitled
to squander or waste the time of the Court. This is as unfair to the Court as it is
to other litigants waiting in
line."
[Extracted, with edits and revisions, from “Counsel's Failure To Argue Written
Submissions Not A Ground
For Review: Bombay High Court” Sharmeen Hakim, LiveLaw, 16August 2021
66. In a case for division of property between three brothers, the counsel of the
first brother had the strongest
case of them all. However, due to oversight on the part of the counsel, he did not
plead a sum of his
arguments, leading to the case being adjudicated in the favour of the other two.
Would the matter be open
to appeal?
(a) No, as non-pleading of the arguments does not stand as a ground for appeal.
(b) No, as the pleadings missed out have no business being revised.
(c) Yes, as this does not prohibit the party from filing an appeal.
(d) Yes, as non-pleaded arguments in written statement are valid grounds for
appeal.
67. In the above case, it also came to light that the counsel for the first brother
failed to argue on one of the
most imperative issues at hand and consequently the Sessions Court failed to take
cognizance of it.
However it was raised in appeal. Would this be termed as wastage of judicial time
by the High Court
respectively?
(a) Yes, amounts to wastage of judicial time, since parties are raising additional
grounds in appeal.
(b) Yes, as the Court was misled by inconsequential arguments.
(c) No, as no diversion per se took place.
(d) No, as the Sessions Court did not recognize it as wastage of judicial time.
68. While hearing the appeal, a plethora of ancillary compensation prayers were
advanced by the counsel for
the aggrieved, which were not advanced before. The HC termed it as being wastage of
judicial time by the
counsel and imposed costs on them. Is the HC correct in doing so?
(a) Yes, as ancillary compensation is immaterial to the issue at hand.
(b) Yes, as ancillary compensation should have been prayed for at the Court of
first instance.
(c) No, as ancillary compensation is not immaterial to the matter at hand.
(d) No, as ancillary compensation is compulsory.

. Page 17 of 36
69. During the stage of arguments, one of the counsels for the two brothers failed
to argue on one of their most
favorable arguments and as a result, was imposed costs upon. This action of the HC
was deemed as being
incorrect. Would this opinion be legally valid?
(a) No, as the HC was correct in imposing the costs on account of wasting the time
of the Court.
(b) No, as the HC was within its powers to impose costs.
(c) Yes, as the HC wrongly believed it to be wastage of judicial time.
(d) Yes, as it is not a counsel’s responsibility to argue all the arguments.
70. In a criminal case, pertaining to trespass and burglary, where the trial Courts
had given their verdict, the
High Court decided to look into the merits of the case on account of some new
evidence coming to light.
Would this stance of the HC be considered as correct?
(a) No, as new evidence does not warrants fresh hearings.
(b) Yes, as new evidence may warrant an appellate Court’s supervision.
(c) No, as new evidence may not warrant fresh hearings.
(d) Yes, as the provisions of Indian Penal Code bestows the High Courts with having
the requisite
superintendence over lower Courts.
71. In the above appeal, had there been no new evidence, but new arguments which
were not pleaded in
earlier filed written statement for the same matter, would the appeal have been
allowed?
(a) No, as new arguments which were not there in earlier written statement cannot
be a ground for appeal.
(b) No, as only new evidence can be grounds for appeal.
(c) Yes, as both new evidence and new arguments are grounds of appeal.
(d) Yes, as new evidence and new arguments along with a fresh appeal are grounds
for the HC to pay
cognizance to the matter.
Passage (Q.72-Q.77): The Allahabad HC held that under the Hindu rituals and
customs, putting vermilion
on the forehead of a woman by a man conveys a man's promise and intention to marry
the woman, which
is sufficient enough for a woman to believe that he would in fact, marry her. The
Court was hearing the
plea of one who sought quashing of the summoning order passed by the CJM, in a
criminal case registered
against him under Section 376 IPC. Further, he also prayed for quashing of Charge
sheet/Final Form and
entire proceedings of the case on the ground that in view of the contents of the
FIR, it was evident that
complainant/victim had consensual sex with him. The accused allegedly refused to
marry her on the ground
that since the daughters from the man's family are married in the family of the
girl, and therefore, a girl
couldn't be brought from that family, where they have already given their daughters
through an alliance of
marriage. The prosecution submitted that the accused/applicant had performed a
ceremony with the
woman, called "Maangbharai”, which is an important step under Hindu Traditions and
Culture leading
towards the marriage-("Saptpadi"). Therefore, it was argued that in the name of
this ceremony, a false
promise to marry was made as it was a sort of consummation of marriage with the
victim. Lastly, it was
also contended that the accused knew it from the beginning about his family
tradition and therefore, despite
knowing this family tradition the assurance given by the accused to obtain the
consent of the victim cannot
be said to be consent free of any blemishes. After hearing the contentions of both
accused and victim High
Court refuses to quash FIR U/s 376 IPC and held that“putting vermillion on victim's
forehead shows man's
intention to marryher and having sex on false promises to marry amount to rape.
72. P was boyfriend of G. Both liked each other and had a plan to marry after one
year. One day, G saw P with
same another girl who was sitting beside him on a bench and both were having a
conversation joyfully. G
immediately broke up with him saying he cannot cheat on her like that. To
concretize his loyalty and love
towards her P put vermillion on her forehead saying “I will always be yours”. One
year later G’s marriage
was fixed with a businessman M and she got married. P filed a complaint against G.
in the light of the
above passage, choose the correct option. Whether there was any intention to marry?
(a) Construing to the passage, it is not given in the facts that P and G ever had
consensual sex and thus
G is not liable to be alleged.

. Page 18 of 36
(b) Putting vermillion on the forehead means a promise to marry someone immediately
as it is an integral
part of Hindu customs.
(c) Putting vermillion on the forehead cannot act as a guarantee of marrying
someone construing to
inevitable circumstances.
(d) Having a joyful conversation with another girl is not cheating and putting
vermillion shows a mere
intention to marry.
73. H and P were batch mates in the college and were in the final year.They were
planning to get married after
a year of the college. H and P were involved in consensual sexual relationship on
the false pretext of them
getting married. However, H liked a girl S in his college and after a year, he
married with her leaving P. P
filed a complaint of rape against H.
(a) H cannot file a complaint of rape as the sexual relationship between the two
was consensual.
(b) H having a mere latent attraction to marry S cannot expose H to a rape
accusation as the sexual
relationship between H and P was consensual.
(c) H has the right to marry anyone he wants because mere sexual relationship which
is consensual cannot
act as a guarantee of marriage between the two individual.
(d) Having a hidden intention to marry another girl can expose H to a rape
accusation.
74. M and W were husband and wife. M was an adulterous man and had an affair with
another woman named
D. one day having boozed and quite intoxicated, having the consensual sexual
relationship between them,
M put vermillion on D’s head and said that now you are my soulmate which D denied
as she didn't want to
get married with him but wanted sexual relationship only. That time they were
caught red handed by M’s
wife and now W accused both of them to be married. In the light of the passage,
point out the correct
option.
(a) According to the passage and construing to the facts given, applying only
vermillion and not resorting
to Saptpadi is not marriage.
(b) Construing to the passageapplying vermillion is an indication of concrete
intention to marry and thus
liable to be accused both of them.
(c) As D didn't want to get married and only wanted sexual favors mere application
of vermillion on the
forehead cannot be a concrete evidence of an intention to marry.
(d) Notwithstanding to the refusal to marriage applying vermillion and having
sexual relation with the
woman can be a ground of rape.
75. E and R were relatives but of the same age group. R’s family daughters were
married to E’s family. E and
R liked each other but E was aware of the tradition that they cannot get married as
the daughters of R’s
family were married to E’s family. However, ignoring the fact, she invited R to
have a sexual relationship
with her and construing to this and unaware of the family traditions, the allowed
the same and in fit of
emotion put vermillion on E’s forehead which she didn't like as she considered it
as a guarantee of marriage
which was possible and accused R for raping her.
(a) There was a consensual sexual relationship between E and R and thus R was not
liable to be accused
of rape.
(b) Applying vermillion lead R to bear the consequences of allegation of rape.
(c) As construing to the passage, if a girl is herself inviting to have a
relationship with her, however not
intending to marry the man, he is not liable to be accused of the same.
(d) E knew the customs of the family and still invited R for the same, thus she was
liable for not construing
to the customs and was not justified in accusing R however application of
vermillion in fit of emotion
was not justified.
76. In the light of the above passage, choose the best interpretation of the
passage.
(a) Applying vermillion to the forehead in any circumstance is a guarantee to marry
someone.
(b) Vermillion and Saptpadi are the two major components of Hindu marriage in any
condition.
(c) Having a clandestine intent and only mere applying of vermillion can lead to
accusation of rape as it is
assumed that marriage also includes consummation i.e. sexual intercourse.
(d) Applying vermillion on forehead and giving false promises is not justified and
leads to allegations of
rape.

. Page 19 of 36
77. In the light of the above passage choose out the incorrect interpretation of
the passage.
(a) Applying vermillion on the forehead is the depiction of a mere intention of
marrying someone.
(b) Saptpadi without applying vermillion cannot be a legit Hindu marriage.
(c) Applying vermillion on the forehead of a rape victim by the accused can nullify
the effect of section 376
of IPC.
(d) Construing to the passage, faking promise and having clandestine intentions can
lead to liability under
certain legal consequences.
Passage (Q.78-Q.82): “Slavery comes in all forms, and in recent times of
technological advancement, data
collection encompasses digital inequality and the illegality of a new era. The
right encompasses certain
rights of correction and erasure namely the right to (i) correct inaccurate or
misleading personal data, (ii)
to complete any incomplete personal data, (iii) update personal data that is out-
of-date, and (iv) erase
personal data which is no longer necessary for the purpose for which it was
processed. Moreover, right to
erasure did not circumferent the bill of 2018, but only emanated in the Personal
Data Protection Bill of 2019
which talks about erasure of personal data belonging a data principal held any data
fiduciary if the data
doesn’t serve any purpose as it violates right to privacy which is a fundamental
right of a person under
Article 21 of the constitution.”
78. Consider this factual situation and answer the questions that follow:
On the State of Madhya Pradesh’s R.T.O. website, there is a tab that enables any
person to find their
vehicle details along with residential details of vehicle owners by entering the
vehicle registration number.
Anirudh Wadhwani wishes to approach the High Court of Madhya Pradesh for this free
availability of vehicle
owners’ residential addresses.Decide whether the writ petition can sustain on the
grounds of right to
privacy.
(a) Yes, the petition is sustainable because a man’s home is his castle and his
address needs to be
protected in all circumstances.
(b) No, the petition is based on false assumption that right to privacy is a
fundamental right, which in reality,
is not a fundamental right as per any Article of Indian Constitution.
(c) No, the petition is a publicity stunt and no such case is sustainable.
(d) Yes, the petition is sustainable as right to privacy is a fundamental right as
per India Constitution and
publicly making available such data is a breach to the same.
79. Jessica, a 23-year-old engineering student, was texting with her boyfriend, who
happens to be a union
minister in central government. All of a sudden, her mother walks in Jessica’s room
only to find out that
she was blushing on her phone while texting someone. Jessica’s mother tried to ask
Jessica to give the
phone to her, so that she could read the chats, to which Jessica refused. Her
mother got aggrieved and
went to file an R.T.I. against her daughter asking for details of the chats in her
phone between Jessica and
her boyfriend. Decide whether the R.T.I. is maintainable.
(a) No, the R.T.I. is not maintainable because Jessica’s boyfriend is a union
minister in central government
and a minister is immune to R.T.I. under Indian law.
(b) Yes, Jessica’s mother has every right to read Jessica’s chats because of the
fact that she raised her
and loves her so much, hence the R.T.I. is maintainable.
(c) No, the R.T.I. is not maintainable because by way of this R.T.I. application,
Jessica’s mother is trying
to breach the privacy of her daughter.
(d) Yes, this R.T.I. is in public interest as it has the chats of a union minister
which is a public record.
80. Rekha and Amitabh who live in Pitampur are about to get married on 02.10.2022.
Onthe night of
30.09.2021, Amitabh fell seriously ill and had to be admitted in S.R.K. Hospital
overnight. He had severe
fever and body ache. The doctor recommended blood test to reveal the cause. Amitabh
agreed for the
same and submitted his blood samples to Being Human Pathology Lab. When the blood
test reports came
out, it was found to everyone’s surprise that Amitabh is HIV Positive. Dr. Reddy,
who was treating Amitabh,
told Rekha about Amitabh being tested HIV Positive. Rekha, being a selfish human,
decided not to marry
Amitabh and left him forever. Amitabh is aggrieved by this and wants to know your
legal opinion on this
matter. Decide.

. Page 20 of 36
(a) Amitabh should go ahead and file a case of criminal conspiracy against Dr.
Reddy, S.R.K. Hospital and
Being Human Pathology Lab as he is being framed in all this.
(b) Amitabh should calm down and let karma take revenge.
(c) Amitabh has no legal remedy available for breach of privacy because disclosure
of such information to
partner who one is about to marry is bonafide in nature and is an exception to the
right to privacy.
(d) Amitabh’s right to privacy is breached and hence, Dr. Reddy is liable for the
same.
81. Rekha, a resident of Pitampur, falls ill one night and had to be admitted in
S.R.K. Hospital overnight. He
had severe fever and body ache. The doctor recommended blood test to reveal the
cause. Rekha agreed
for the same and submitted her blood samples to Being Human Pathology Lab. When the
blood test reports
came out, it was found to everyone’s surprise that Rekha is HIV Positive. Dr.
Reddy, who was treating
Rekha, distributed fliers in Rekha’s workplace, stating that Rekha is HIV Positive
and that everyone should
maintain distance from her. When Rekha went to her office the next week, she was
astonished to notice a
change in everyone’s behavior towards her and was aggrieved by Dr. Reddy’s act of
distributing fliers in
her office announcing that Rekha is HIV Positive. Rekha came to you for legal
opinion on this situation.
Decide.
(a) Dr. Reddy wanted to ensure minimal spread of HIV and hence his actions are
protected under The
Indian Medical Council Act, 1956.
(b) Rekha’s co-workers have right to information under which it is necessary for
them to know Rekha’s HIV
status.
(c) Dr. Reddy has breached Rekha’s right to privacy as he has publicly announced
about her HIV status.
This act is in clear violation of Article 32 of Indian Constitution.
(d) Dr. Reddy has breached Rekha’s right to privacy as he has publicly announced
about her HIV status.
This act is in clear violation of Article 21 of Indian Constitution.
82. The Personal Data Protection Bill, 2019 was introduced in LokSabha by the
Minister of Electronics and
Information Technology, Mr. Ravi Shankar Prasad, on December 11, 2019. Which of the
following
statements is TRUE about this bill?
(a) The Bill seeks to provide for protection of personal data of individuals, and
establishes a Data Protection
Authority for the same.
(b) The Bill allows processing of data by fiduciaries without any consent provided
by the individual under
all circumstances.
(c) Sensitive personal data may be transferred outside India for processing without
any consent by the
individual.
(d) The Bill amends the Information Technology Act, 1964 to delete the provisions
related to compensation
payable by companies for failure to protect personal data.
Passage (Q.83-Q.88): In a judgment delivered on Wednesday (15 September 2021), the
Supreme Court
explained the difference between culpable homicide under Section 304 of the Indian
Penal Code and
murder under Section 300 IPC.
The court observed that, though it is difficult to distinguish between culpable
homicide and murder as both
involve death; there is a subtle distinction between intention and knowledge
involved in both the crimes
"This difference lies in the degree of the act. There is a very wide variance of
the degree of intention and
knowledge among both the crimes.” the bench observed, while stating the following
points to be kept in
mind;
(i) Nature of the weapon used;
(ii) Whether the weapon was carried by the accused or was picked up from the spot;
(iii) Whether the blow is aimed at a vital part of the body;
(iv) amount of force employed in causing injury;
(iv) whether the act was in the course of a sudden quarrel or sudden fight or free
for all fight;
(vi) whether the incident occurs by chance or whether there was any premeditation;
(vii) whether there was any prior enmity or whether the deceased was a stranger;
(viii) whether there was any grave and sudden provocation, and if so, the cause for
such provocation;
. Page 21 of 36
(ix) whether it was in the heat of passion;
(x) whether the person inflicting the injury has taken undue advantage or has acted
in a cruel and unusual
manner;
(xi) whether the accused dealt a single blow or several blows.
The above list of circumstances is, of course, not exhaustive and there may be
several other special
circumstances with reference to individual cases which may throw light on the
question of intention."
83. Greg Chappel was the boss at the place where Virender and Saurav used to work.
They were equally good
employees who were efficient at their jobs and were up for the same promotion.
After the evaluation,
Chappel dropped Saurav out of the workforce and appointed Virender to the promoted
position. This
infuriated Saurav, who had murder on his mind, and he decided to confront Greg
carrying a knife. To his
dismay, as Virender was dressed exactly like Greg, Saurav mistook him for Greg and
stabbed him from
the behind. This resulted in Virender’s death and Saurav was sued for manslaughter.
What offence would
Saurav be liable for?
(a) Culpable homicide as he did not intend to kill Virender
(b) Murder as his intention to kill was irrelevant, a death still took place.
(c) Culpable homicide as he only intended to injure and not kill.
(d) Murder as a stabbing wound would almost certainly kill a person.
84. In the above case, had Saurav not decided to confront Greg but had seen him in
the bar where he was
sitting and was suddenly provoked due to seeing his former employer, would the act
of stabbing Greg be
considered as murder, if Greg succumbs to his wounds?
(a) No, as Saurav was clearly under a grave and sudden provocation.
(b) No, as Greg provoked a disgruntled employee.
(c) Yes, as there is no apparent provocation present here.
(d) Yes, as Gerg succumbs to his wound.
85. Gautam was a senior employee at the firm, close to retirement, who also had a
heart condition which made
him sensitive to loud sounds and sudden surprises, this was known by everyone.
Manish was an energetic
young employee at the same firm who was always ecstatic and jumping around. One
fine day, while
working in his office, Gautam heard a knock, upon telling the person to come in, he
saw Manish enter.
Whilst talking to Gautam, Manish removed his headphones which were blaring loudly.
When he reached
to pause the music, his headphones got disconnected and disturbingly loud music
started playing which
took Gautam by surprise and Gautam’s heart gave out on the spot. What is Manish
liable of here?
(a) Manish is liable of Murder as he was well versed with Gautam’s condition.
(b) Manish is liable of Culpable homicide as he did not intend to kill Gautam.
(c) Manish is not liable because he is a person of goof heart and he also tried to
pause the music.
(d) Manish is not liable at all as has no intention to kill Gautam.
86. Sreesanth and Bhajji were two hot and cold friends. One day while working at
the firm, Sreesanth failed to
do a task that Bhajji asked him to do. This led to Bhajji getting infuriated at
Sreesanth, and in the heat of
the moment, he slapped Sreesanth. What he did not realise was that Sreesanth
suffered from severe
insecurity and inferiority complexand he suffered an anxiety attack due to this,
which resulted to his death
due to a cardiac arrest. Can Bhajji be held liable for murder here?
(a) Yes, as being a friend he should know about his condition.
(b) No, as Bhajji could not have possibly anticipated such a severe reaction to a
mere slap.
(c) Yes, as any prudent man would die of shame anyways if he were to be slapped by
a senior in front of
their peers.
(d) No, as this is a case of culpable homicide not amounting to murder.

. Page 22 of 36
87. Sachin was the foreman in the company’s warehouse, and commanded a team of up
to 200 workers under
his belt. One such worker under his command was Shoaib, who sued to manage the
hooks on the cranes.
One day, while asking for a hook, Shoaib bumped into Sachin and instead of
apologising, antagonised
Sachin, which resulted in a scuffle between the two. In the course of the fight,
Sachin got very angry at
Shoaib’s attempts to get the hooks from where Sachin was managing the whole affair.
This resulted in
Shoaib being slapped by Sachin, who then landed on a hook and died instantaneously.
Can Sachin be
held liable for culpable homicide?
(a) No, as the act of slapping Shoaib could not have resulted in a foreseeable
death.
(b) Yes, as Sachin clearly possessed an enmity towards Shoaib.
(c) No, as Shoaib had clearly provoked Sachin in this case.
(d) Yes, as death of Shoaib is the end result of Sachin’s act.
88. Inzybhaijaan was another worker under Sachin, working in the deliveries
section. Good at handling
deliveries, Inzy made up for his unhealthy physique by working hard and managing
the floor while standing
in one place. One day, while managing the pitch floor, Monty, another worker, saw
him merely standing,
and called him ‘fat’ which resulted in Inzy tripping over the three boxes behind
him. This led to Inzy running
towards Monty and pushing him, making him fall on a stump and dying instantly. Inzy
was sued for murder.
What defense can he avail?
(a) Murder weapon not being involved by him.
(b) Possessing no intention to commit murder.
(c) Being provoked by Monty.
(d) All of the above.
Passage (Q.89-Q.94): The Supreme Court on Tuesday held that educational
qualification is a valid ground
for classification between persons of the same class in matters of promotion.
In the judgment delivered in the case the bench held that such classification on
the basis of educational
qualification is not violative of Articles 14 and 16 of the Constitution.
Holding so, the Court upheld the validity of separate eligibility conditions for
promotion to Supernumerary
Assistant Engineers having diplomas and degrees in Kolkata Municipal Corporation.
(i) Classification between persons must not produce artificial inequalities. The
classification must be
founded on a reasonable basis and must bear nexus to the object and purpose sought
to be achieved
to pass the muster of Articles 14 and 16;
(ii) Judicial review in matters of classification is limited to a determination of
whether the classification is
reasonable and bears a nexus to the object sought to be achieved. Courts cannot
indulge in a
mathematical evaluation of the basis of classification or replace the wisdom of the
legislature or its
delegate with their own;
(iii) Generally speaking, educational qualification is a valid ground for
classification between persons of the
same class in matters of promotion and is not violative of Articles 14 and 16 of
the Constitution;
(iv) Educational qualification may be used for introducing quotas for promotion for
a certain class of
persons; or may even be used to restrict promotion entirely to one class, to the
exclusion of others;
(v) Educational qualification may be used as a criterion for classification for
promotion to increase
administrative efficiency at the higher posts; and
(vi) However, a classification made on grounds of educational qualification should
bear
89. Ramesh and Suresh work in the public works department along with 3 others, are
in the race for a
promotion. Both of them have multiple years of practical experience, and due to the
same, most people
are sure that the promotion would be allotted to either one of them. However, the
promotion is instead
allotted to a man with a comparatively less experience, justifying it by saying
that only he fulfilled the
requirement of a Masters' degree. Ramesh and Suresh are enraged, and take the
matters to Court. Will
their action be successful?
(a) No, their action will not be successful as the other person is more qualified.
(b) Yes, owing to their skill, is a reasonable classification which cannot be
established between them.
(c) Yes, as appointing the person with less experience would not achieve the object
of the position, and
hence is not valid reasonable classification.
(d) None of the above.

. Page 23 of 36
90. If in the situation given above, Ramesh & Suresh do possess Masters' degree,
yet still are not able to
achieve the promotion as the criteria of the position requires that the person
should possess a degree in
Water Conservation from ABC University as a part of their water conservation
initiative. Will this action be
valid?
(a) Yes, as it is a reasonable classification based on educational classification.
(b) No, as a degree of a specific institute will not count as reasonable
classification.
(c) No, as a degree in water conservation does not bear a nexus to the object
sought to be achieved.
(d) Yes, as educational qualification may even be used to restrict promotion
entirely to one class.
91. Suvadhish was a Brahmin pandit of the highest class, considered to be highly
learned in his community.
Jagdish was his arch rival, another pandit, albeit lower in class than Suvadhish
and considered to be a little
less learned. They both were up for the same job in Jal Board of MP, as the
regional supervisor. However,
due to his higher level of education that his caste warrants him with, Suvadhish
got the job, is his
appointment fair in nature?
(a) No, as the Jal Board had equitably given both of them an equal opportunity.
(b) Yes, as they were both discriminated between by the matter of their caste.
(c) No, as Suvadhish was given the appointment on account of his learnedness.
(d) Yes, as Jagdish has been inequitably treated in this matter.
92. In the above case, had the position in Jal board been reserved for only the
Brahmins of the class that
Suvadhish belonged to, would the same entitle Jagdish to relief under articles 14
and 16?
(a) No, as the class of the Brahmins does not fall under any class based
discrimination.
(b) Yes, as the class bifurcation clearly differentiates between the two.
(c) No, as no authority for redressal has been mentioned above.
(d) Yes, as Jagdish was discriminated against on unfair grounds.
93. In the aforementioned case, had the differentiation been made on the basis of
the education that the
different categories offered, and not the categories themselves, would the same
count as being
discriminatory?
(a) No, as the differentiation has been made on the basis of education.
(b) Yes, as the differentiation has been made on the basis of caste.
(c) No, as it does not qualify as being discrimination as it does not disqualify
anyone from the post.
(d) Yes, as the differentiation has been made by disqualifying Jagdish from the
position.
94. What is meant by the phrase, "Classification between persons must not produce
artificial inequalities."?
(a) The classification should be based on natural differences only, such as gender,
intelligence, etc.
(b) The classification can only be between natural persons, and not legal entities.
(c) The reasonable classification should not create inequalities that do not exist.
(d) Both A & B are correct.
Passage (Q.95-Q.100): The Bombay High Court has asked the Maharashtra Government to
file a "better
affidavit" regarding the steps to vaccinate mentally-ill homeless wanderers against
Covid-19, instead of
merely stating the number of inmates vaccinated in psychiatric institutions. A
division bench of Chief Justice
Dipankar Datta and Justice GS Kulkarni said that the State appears to have
vaccinated 1,761 persons of
unsound mind in institutions, where their families could consent for the vaccine.
Previously, the High Court
had asked the Centre and State to reconsider a standard operating procedure (SOP)
to vaccinate the
'floating population' of mentally ill homeless persons, beggars. The bench had also
suggested permanent
tattoos for such persons after inoculation. On Monday, "We can't vaccinate them on
the road. They are
taken to a shelter and then vaccinated there," he said adding that consent would be
required. "Take the
case of the mentally ill. If we allow him to decide, he can be a threat to society.
So why don't you come out
with a policy of how you plan to vaccinate the mentally ill," the Chief Justice
said. In response to Singh's
submissions that Section 100 of the Mental Health Care Act casts a duty on police
officers to identify such

. Page 24 of 36
mentally unsound people and unite them with their families, the court sought to
know if it was being
implemented. "Whether these steps are being taken?" the bench asked the State to
elaborate in their
affidavit. "We notice that the affidavit is silent on the point of those mentally
ill persons who are either
homeless or found wandering in the community," the court observed.
95. PVZ was reputed state governmentwho took the charge of vaccinating all the
beggars across the streets
of the state. The state government was managing its vaccination centers
efficiently. After vaccination, it
took the liberty of tattooing the beggars’ wrist as an indication of a successful
inoculation. However, they
didn't bother to take consent of their families. A legal enthusiast filed a
complaint against the PVZ state
government for not doing its work efficiently. In the light of the passage, opt out
the correct option.
(a) According to the passage, the state government was doing its work efficiently
as it had construed to
the guidelines of the court.
(b) According to the passage, the state government has not fully construed to the
guidelines of the court
regarding the vaccination.
(c) The state government should have taken the consent of the beggars’ families
before vaccination,
though not essentially important.
(d) According to the passage, the tattooing was to be done before the inoculation.
96. The state government construed to fulfilling the conditions of vaccination of
the beggars and the mentally
ill and homeless people. It initiated many vaccination centers across the state for
the same. Beggars were
taken to the centers after rehabilitation with their families and a temporary
tattooing was done in their hands
so that they could get inoculated second time. Same was done for the mentally ill
patients and homeless
people. However, it was seen that there was no decrease in the cases of deaths
among those sections of
society. In the light of the passage, opt out the most appropriate option.
(a) According to the passage given the state government had construed to the basic
guidelines of the
vaccination process.
(b) The state government has partly construed to the guidelines of the ort
regarding the vaccination
according to the passage.
(c) Inoculation for the second time was not done according to the guidelines of the
court construing to the
passage.
(d) Inoculation process was not done in accordance with the guidelines of the
court.
97. P was a homeless man. He took the responsibility of a mentally ill girl of 16
years of age named G who
was thrown out of the house because of her mental condition. Inoculation process
was going on across the
state and he was also asked for the same along with the girl. Homeless man families
had no connection
with him thus the agreed to get vaccinated. He argued about G that her family has
thrown her out of the
house having no connections with her and thus she must get vaccinated upon his
consent. The doctors
denied and searched the home of G before inoculation to take her family's consent.
In the light of the
passage, choose the correct option.
(a) According to the passage, she must get inoculated after the volunteers have
obtained the consent.
(b) Homeless man cannot act as a guardian to give consent as consent of her family
was important.
(c) Homeless man can give consent on behalf of her as she was mentally ill.
(d) Volunteers could vaccinate her without anyone’s consent only if she was
mentally ill and not homeless.
98. E was a mentally ill homeless beggar. He wandered across the streets of Bombay
every day. During the
inoculation drive as he was taken to the vaccination camp where he resisted several
times from taking
vaccine. At last he was forcefully taken to the camp and was vaccinated. In the
struggle to escape the got
very nervous and after the inoculation he got a cardiac arrest and died. In the
light of the passage, describe
the liability of the volunteers who were vaccinating him.
(a) The volunteers should have managed the situation wisely and thus preventing the
same from dying of
cardiac arrest.
(b) The volunteers have construed to the guidelines of the court during the
vaccination.
(c) The volunteers need not to get the consent of mentally ill’s family but the
person’s consent is an
important factor.
(d) The volunteers were not justified in making the beggar nervous by forceful
taking to the camp and
should have dealt with him sensibly.

. Page 25 of 36
99. In the light of the above passage, choose the most appropriate interpretation
of the passage.
(a) The inoculation process has to be construed to the guidelines of the court
regarding the tattooing after
the same.
(b) The tattooing process has to be mandatorily done after the inoculation.
(c) Mentally ill patients are not required to be obtained with the consent of the
family however could be
reunited with family if possible.
(d) Homeless and unsound mind people’s reunion is not an important factor in the
process of vaccination.
100. “If we allow him to decide, he can be a threat to society”. In contrast of the
above passage, opt out the
correct interpretation of this line.
(a) Homeless people are a threat to society as they are vulnerable to indulge in
criminal activities.
(b) Mentally ill people are vulnerable to be infected more than that of normal
people.
(c) Mentally ill people transmit disease faster than the normal people.
(d) Mentally ill people and homeless people must get reunited with their families
before inoculation so they
don’t be a threat to other people during the process.
Passage (Q.101-Q.105): The Supreme Court has issued notice on a special leave
petition which has
raised the argument that criminal proceedings cannot continue after exoneration in
departmental enquiry
on the same allegations. The special leave petition is filed against a judgment of
the Madhya Pradesh High
Court which held that exoneration in departmental proceeding does not entitle an
employee to seek
quashing of criminal proceedings. A bench of Justice Virender Singh of the High
Court had referred to the
Supreme Court judgment in State (NCT of Delhi) vs Ajay Kumar Tyagi (2012) 9 SCC 685
to dismiss the
employees plea to quash the proceedings under Section 482 CrPC. Challenging the
High Court verdict,
Senior Advocate Siddarth Dave, appearing for the petitioner, relied upon the
judgment of the Supreme
Court in Ashoo Surendranath Tewari Vs. Deputy Superintendent of Police, EOW,CB to
argue that criminal
prosecution cannot proceed against the petitioner after being exonerated in a
departmental proceeding
for the same allegations. In the said judgment, a 3-judge bench led by Justice RF
Nariman had held that
in case of exoneration in departmental proceedings on merits and where the
allegation is found to be not
sustainable at all and the person held innocent, criminal prosecution on the same
set of facts and
circumstances cannot be allowed to continue. Taking note of the precedent, a bench
comprising Justices
L Nageswara Rao and BR Gavai issued notice in the special leave petition and stayed
the operation of the
impugned judgment. Source Name - Live Law
101. P was the commissioner.. He was accused of bribery and extortion from a
politician making it a high profile
case. The matter with an immediate effect went to the court when the departmental
proceeding was going
on. The criminal proceeding was in process and the inspector was convicted of the
same on Wednesday
at 11 AM IST. The departmental proceeding concluded on Wednesday 10 AM IST and P
was exonerated
from the charges of the bribery and extortion. In the light of the passage, choose
the correct option.
(a) As P was exonerated in the departmental proceeding, the proceeding against him
in the court can
continue as it was pronouncing the judgment in the matter.
(b) As the court’s judgment was given in the case of the same and thus the
departmental proceeding
exoneration order cannot suffice the same.
(c) The departmental proceeding was more important factor to prove the guilt of the
accused before going
to the court.
(d) P exonerated in the departmental proceeding has no liability to get convicted
in the court’s decision.

. Page 26 of 36
102. T was commissioner of police who was alleged to have taken the bribe from the
tv actor for not arresting
him in hit and run case and the proceedings were going on regarding the same. T was
accused of not
discharging his official duty in a particular area in which the matter was in the
court and the proceeding
was going on in the proceeding department. The commissioner was convicted under the
hit and run bribery
case and was also going to be convicted in the 2nd case also. However, next day
before the judgment was
to be pronounced, T was exonerated from the proceeding and thus the conviction of T
was argued to be
void. In the light of the passage, choose the correct option.
(a) T was convicted by the court but he was exonerated by the department.
(b) However T was convicted by the court, he was exonerated by the proceeding
department before the
judgment thus, and he was not liable to be convicted.
(c) The judgment and the proceeding department orders were on different pedestal.
(d) The judgment in the bribery case will suffice over the proceeding department
order.
103. Departmental proceedings were going on in a case where the police
superintendant was accuses of open
firing on a mob. The criminal proceeding was going on and so as, the departmental
proceeding. In the
same, he was found guilty of the same and was suspended from the duty with an
immediate effect. On the
same day, the court found the SP and acquitted him. The police superintendant filed
a petition against the
orders of the departmental inquiry. In the light of the passage, choose the correct
option.
(a) The petition shall not be entertained.
(b) The petition has no ground to suffice as the departmental inquiry has found him
guilty and after the
same, the court cannot resume its proceedings against the same.
(c) The petition could have entertained if the court had acquitted him before the
departmental inquiry.
(d) It is an arbitrary for departmental inquiry to determine to suffice the
acquittal as the same was done in
similar timing.
104. In the light of the above passage, choose the correct interpretation of the
passage.
(a) The exoneration from the departmental enquiry can suffice the high court’s
order.
(b) The exoneration of the person from the departmental proceedings can impact the
criminal trial going
on against accused.
(c) The high court can use its inherent power to reverse the order or suffice the
order of prior departmental
proceedings.
(d) The high court cannot use its inherent power to dismiss the plea after the
orders of the departmental
proceedings.
105. In the light of the above passage, choose the incorrect option.
(a) The high court’s inherent power can be used to nullify the subsequent order of
proceedings of the
department.
(b) High court’s inherent power has to be used judiciously construing to the
essence of natural justice in
the case of departmental proceedings.
(c) The prior exoneration of the accused from the departmental proceeding is
mandatorily be abide by the
high court in its subsequent orders.
(d) The exoneration of the accused has to be mandatorily abided by the court in any
circumstance.
. Page 27 of 36
SECTION - D: LOGICAL REASONING
Passage (Q.106-Q.110): The ideal woman has always been generic. I bet you can
picture the version of
her that runs the show today. She’s of indeterminate age but resolutely youthful
presentation. She’s got
glossy hair and the clean face, an expression of a person who believes she was made
to be looked at. She
is often luxuriating when you see her – on remote beaches, under stars in the
desert, across a carefully
styled table, surrounded by beautiful possessions or photogenic friends. Showcasing
herself at leisure is
either the bulk of her work or an essential part of it. She has a personal brand,
and probably a boyfriend or
husband: he is the physical realization of her constant, unseen audience,
reaffirming her status as an
interesting subject, a worthy object, a self-generating spectacle with a viewership
attached.
Can you see this woman yet? She looks like an Instagram – which is to say, an
ordinary woman reproducing
the lessons of the marketplace, which is how an ordinary woman evolves into an
ideal. The process
requires maximal obedience on the part of the woman in question, and – ideally –
her genuine enthusiasm,
too. This woman is sincerely interested in whatever the market demands of her (good
looks, the impression
of indefinitely extended youth, advanced skills in self-presentation and self-
surveillance). She is equally
interested in whatever the market offers her – in the tools that will allow her to
look more appealing, to be
even more endlessly presentable, to wring as much value out of her particular
position as she can.
The ideal woman, in other words, is always optimizing. She takes advantage of
technology, both in the way
she broadcasts her image and in the meticulous improvement of that image itself.
Her hair looks expensive.
She spends lots of money taking care of her skin, a process that has taken on the
holy aspect of a spiritual
ritual and the mundane regularity of setting a morning alarm.
Everything about this woman has been pre-emptively controlled to the point that she
can afford the
impression of spontaneity and, more important, the sensation of it – having worked
to rid her life of artificial
obstacles, she often feels legitimately carefree. The ideal woman can be whatever
she wants to be – as
long as she manages to act upon the belief that perfecting herself and streamlining
her relationship to the
world can be a matter of both work and pleasure, or, in other words, of
“lifestyle”. The ideal woman steps
into a stratum of expensive juices, boutique exercise classes, skincare routines
and vacations, and there
she happily remains.
106. “The ideal woman has always been generic.” What can be inferred from the
statement?
(a) An ideal woman does not fit into any role.
(b) An ideal woman is an extraordinary woman, perhaps an all-rounder.
(c) An ideal woman is a woman that most aspire for in the contemporary world.
(d) An ideal woman is an ordinary woman, without any specific identity.
107. Which of the following situation describes the word, meticulous?
(a) The judge gave a disinterested verdict, based on the evidence.
(b) My teacher revels in creativity and out of the box thinking and gives
preference to content than unwitting
errors.
(c) I have a carefree streak. I believe one should not take oneself too seriously.
(d) My sister is thorough with her assignments, checking even the minutest details
before submitting.
108. ‘She spends lots of money taking care of her skin, a process that has taken on
the holy aspect of a spiritual
ritual and the mundane regularity of setting a morning alarm.’ What can be inferred
from the given lines?
(a) For an idealistic woman, self-care is as pious as performing a spiritual ritual
with great fervour and as
necessary as an inherent part of a routine.
(b) An idealist woman will go to great lengths at self-care, considering the
process no less than performing
a grand holy ritual.
(c) For an idealist woman, taking care of herself is as commonplace as setting an
alarm.
(d) Self-care is a propagation of a perfect world that exists in the make-belief
world of an idealist woman.

. Page 28 of 36
109. ‘Can you see this woman yet? She looks like an Instagram – which is to say, an
ordinary woman
reproducing the lessons of the marketplace, which is how an ordinary woman evolves
into an ideal’ Which
of the following indicates a central flaw in the statement?
(a) The author does not take into consideration other online social media
platforms.
(b) The author assumes Instagram as a social media platform tool used only as a
marketing gimmick.
(c) The author assumes every ordinary woman having aspirations for idealism.
(d) All of the following.
110. What is the central idea behind the passage?
(a) To expose the detrimental effects of idealism, devouring the very existence of
women.
(b) To draw attention on an ordinary woman, spending her life walking toward the
idealized mirage of her
own self-image.
(c) To celebrate today’s woman, who is an idealized version of herself and does not
need corroboration.
(d) To honour women by portraying the evolution of a woman in the contemporary
world.
Passage (Q.111-Q.115): The year 2011 was a big one for me. My son was born. We
moved to a new city.
I published a book. But something else happened that was in some ways more
significant: on February 9,
2011, I bought my first smartphone. It didn’t feel like a milestone in my life at
the time. I didn’t note it down
in a diary or commit the date to memory. Only finding a copy of the receipt helped
pin down the day. Yet I
have come to realise that the phone was a very big deal indeed.
Daniel Kahneman, Nobel laureate and author of Thinking, Fast and Slow (UK) (US),
distinguishes between
the “experiencing self” and the “remembering self”. My remembering self dwells upon
the landmark
moments such as the new baby. But my experiencing self is all about the phone. I
spend more time
interacting with it than I do interacting with my children. I am in the presence of
the device more than I am
in the presence of my wife.
As Cal Newport puts it in a new book, Digital Minimalism (UK) (US), we didn’t sign
up for this. My first email
account (1994) received a handful of messages a day, most of them newsletters I
subscribed to in order
to prevent cobwebs forming in my inbox. Facebook (2004) was a curiosity, less
interesting than the latest
computer game.
The first iPhone (2007) had no app store and was originally conceived as an iPod
that made phone calls
— although since “crackberry” had just been named the word of the year by Webster’s
New World
Dictionary, perhaps we should have seen what was coming.
But we didn’t. The hardware and software of the mobile age have gradually and
profoundly entangled
themselves in most parts of most people’s lives. If you are anything like me, you
pick up your phone much
more often than you pick up a knife and fork, and spend far longer reading email
than reading books.
Not that I wish to grumble. These tools are enormously powerful. Without them I’d
need to hire a secretary,
spend hours playing phone tag and give up on working during long journeys by train
and plane. Yes, they
may occasionally distract me during the school nativity play, but the alternative
would have been to miss
the play entirely, because the office and the school are 50 miles apart.
I am not entirely happy with the role these technologies play in my life, but
neither do I want to relinquish
them. I know I’m not alone. For several years now, I’ve been dispensing sporadic
advice about email
overload both to readers and — if I am honest — to myself.
111. In paragraph one, which of the following aligns with the author’s thought
process?
(a) For the author, buying a smart phone carried the same validation as publishing
a book.
(b) The author instantaneously understood the significance of the purchase of the
smart phone.
(c) Smart phone was at the pinnacle of the ascending list of events for the author,
in 2011
(d) For the author, buying a smart phone was one of the milestones, in the year
2011.

. Page 29 of 36
112. Which of the following situations would reflect the ‘experiencing self’?
(a) A friend’s wedding where one had the time of one’s life.
(b) A much-awaited promotion that was two jumps higher than the previous held post
(c) An experience of a horrific fight between two groups that turned into a
bloodshed.
(d) Long walks in the forest as a ritual to de-stress for the day.
113. Which of the followings can be inferred from the given passage?
I. Facebook was launched in the year 2004.
II. The first iPhone was launched in 2007 that had no app store.
III. In the mobile age, people spend more time on phones than on books.
(a) I and III (b) Only II
(c) All of the following. (d) None of the following.
114. The hardware and software of the mobile age have gradually and profoundly
entangled themselves in most
parts of most people’s lives. What is the assumption behind the statement?
(a) Mobiles are today integral part of people’s lives.
(b) Mobiles dictate how people will live their lives
(c) Mobiles have taken over people’s lives.
(d) Mobiles will soon become redundant.
115. The last paragraph does which of the following?
(a) It reveals a vulnerability of human-beings towards harsh decisions.
(b) It brings out one of the addictions of human beings.
(c) It exposes human mettle when taking important calls.
(d) it contradicts the arguments put forth by the author in the previous paragraphs
Passage (Q.116-Q.120): India is often lost in the morass of everyday bureaucracy
and politics which blurs
long term capacity building. Even when attacked, we engage in our favourite sport
of infighting and partisan
politics instead of rallying behind our Prime Minister and military leadership.
Centuries of colonialization
happened not because we were defeated by the Turkic Mughals and the English, but
because we fought
between each other and let invaders capitalise on our insecurities.
The ‘Chalta Hai’ attitude, roughly translated as something in between ‘whatever
works’ and ‘let the show
go on’, is the weakness of our system, keeping it from delivering long-term value.
Jugaad or frugal
innovation, often feted in international management literature, has a synonym as
well, which is cutting
corners. Not the best vibe to fete!
The plurality of our democracy is a celebration of voices, but it does not deliver
quality life for most of our
citizens. Structures for service delivery are inadequate. We spend all our energies
in keeping the system
intact rather than to move the country forward. All movement, such as running on a
treadmill, is, sadly, not
progress. Our politics is geared towards winning elections, not the real meat of
governance and getting our
people a better shot at life.
We need to create a culture where results matter, and ‘moving the needle’ is the
only metric. While writing
this article, I do not forget the diversity of our nation. As an ethnographer, I
live and breathe social justice.
However, social justice is not in being a keyboard warrior ferociously typing away
on Twitter, but in bringing
quality public goods to our people who live on the borders and the heartlands away
from the thriving
economic clusters of our cosmopolitan cities.
Radical ambition moves the paradigm forward. As historian Ramchandra Guha writes,
adopting republic
democracy under universal franchise in 1950 was a radical experiment in India.
Since then, the paradoxical
nature of state capacity in India has been evident. The space programme under
Vikram Sarabhai and the
IIT system began with the state. Yet, the consequent IT boom generating a crop of
top-class tech

. Page 30 of 36
professionals running ‘the valley’ out of Bengaluru emerged despite the government,
not because of it.
Strategic capacity is beyond the shackles of electoral politics and is evident in
the entrepreneurship that
flourishes in the absence of state interventions in India.
Paradox defines us as a culture. Now, millennials must figure out a way to break
this contradiction, do away
with the “Chalta Hai” attitude and create policy and market-oriented solutions for
the five trillion-dollar
economy, which our Prime Minister envisions.
116. What reason does the author give for centuries of colonisation of India?
(a) India did not have sophisticated military powers.
(b) Indians possess slave mentality and hence are subjugated easily.
(c) There was a lot of infighting among Indians themselves which led the invaders
to take advantage of
our insecurities.
(d) India did not put up a robust fight and easily buckled down to foreign
invasions.
117. According to the author, strategic capacities are the realm of
entrepreneurship and they can be achieved
outside government intervention. Which of the following statements, if true negates
the argument?
(a) Entrepreneurs necessarily require government support to flourish.
(b) The entrepreneurial strategies rely on government policies.
(c) Entrepreneurs have built in mechanism of self-sufficiency and are immune
against government
changes and decisions.
(d) Both A and B
118. “India is often lost in the morass of everyday bureaucracy and politics which
blurs long term capacity
building”, What does the author imply by this statement?
(a) Bureaucracy should be dispensed with, as it is a colonial tradition.
(b) The combination of politics and bureaucracy has created a mess and poses a
hindrance to long term
capacity building.
(c) Government intervention cannot lead to capacity building.
(d) Bureaucracy and politics are the pillars on which the long term visions are
built upon.
119. What conclusion can be derived from the argument of the author that “social
justice is not in being a
keyboard warrior”?
(a) Twitter does not have adequate reach in India as internet connections are still
not available in rural
areas.
(b) Internet audience does not read content on policy topics and hence, being a
keyboard warrior would
not achieve anything.
(c) Social justice can be brought about by making public goods available to those
who live in remote and
border areas and not by Twitter activism.
(d) Social justice can only be delivered by active participation and has nothing to
do with social media.
120. According to the passage, which of the following is true?
(a) The English subjugated us because we did not have modern weaponry.
(b) Government needs to be more active and intervene more in order to build
strategic capacities.
(c) Creating a culture where results matter is the need of the hour.
(d) ‘Chalta hai’ is part and parcel of Indian culture and the millennials will have
to embrace the attitude.

. Page 31 of 36
Passage (Q.121-Q.125): A report tabled in Parliament by the Comptroller and Auditor
General (CAG) has
highlighted the grave financial situation of the Railways. The net revenue surplus
dropped alarmingly by
54.8 per cent from Rs. 5,824 crore in 2016-17 to Rs 1,342.81 crore in 2017-18. The
CAG noted that the
decline in generation of internal resources resulted in the Railways' greater
dependence on gross
budgetary support and extra budgetary resources for meeting its capital
expenditure.
The 'Give Up' scheme, started in 2017 to encourage senior citizens to forgo their
train fare concession, has
flattered to deceive. The dismal numbers show that the Railways has failed to
generate awareness among
senior citizens about the initiative aimed at enhancing revenue. The sorry state of
affairs is a far cry from
the days of Lalu Prasad Yadav as Railway Minister in the UPA-I government. His
tenure was not
controversy-free, courtesy the IRCTC hotels' tender allotment scam, but he is
credited with turning around
the Railways' fortunes. Lalu's homespun management model even attracted Harvard and
Wharton
business schools, though the CAG was unimpressed by his 'cash and investible
surplus' claim. He took
the populist route by neither hiking passenger fares nor retrenching workers, while
allowing overloading of
freight wagons and simplifying the freight tariff.
Even as an empowered group set up by the NDA government is looking into the
feasibility of upgrading 50
railway stations to world-class standards and allowing private players to operate
150 trains, the Railways
needs to walk a tightrope — rationalise its workforce on the one hand and scale up
services to meet
passengers' expectations on the other. Timely replacement of old assets and
curtailing wasteful
expenditure would be in order too. The nation's lifeline awaits a new lease of
life.
121. Which of the following further suggests that the railways need to walk a
tightrope?
(a) Indian Railways has to make travel easier for passengers across the country
from connecting remote
areas.
(b) Indian Railways has decided to introduce double-decker trains on many routes.
(c) Indian Railways is suffering from a shortage of coaches as per the recently
introduced plan to improve
the railways.
(d) Indian Railways has determined the optimal level of the staff strength without
having to compromise
with the quality of services offered.
122. Which of the following most accurately describes the role played by the
statement that the sorry state of
affairs is a far cry from the days of Lalu Prasad Yadav as Railway Minister in the
UPA-I government?
(a) It is the conclusion drawn in the passage.
(b) It is a claim that the argument is inconsistent with available evidence.
(c) It is a premise offered in support of the conclusion drawn in the argument.
(d) It describes evidence that the text later refutes.
123. Which of the following can be inferred from the passage?
(a) Railways is one of the most important assets of the country.
(b) The railways department is on the verge of collapsing due to financial losses.
(c) Lalu Prasad Yadav was the best Railway Minister ever in India.
(d) The government is working to popularise the use of railways.
124. Which of the following could have increased awareness about 'The Give Up'
scheme?
(a) Running campaigns on social media platforms which are regularly visited by
youngsters.
(b) Penalise the concession holders for not giving up their concessions.
(c) Preparing a nationwide campaign to encourage more concession holders to give up
their concession.
(d) Sending a personalised letter by Railway Minister to those who give up their
concession

. Page 32 of 36
125. Which of the following, if true, would account for the grave financial
situation that the Indian Railways is in?
(a) The national transporter recorded an operating ratio of 97.33 per cent in 2017-
18.
(b) The Railways spent Rs 98.66 to earn Rs 100, way above the figures in 2010-11
(Rs 93.8) and 2011-
12 (Rs 94.9).
(c) Of the 3.91 crore elderly passengers, only 2.8 per cent opted to give up 50 per
cent rebate and just
3.87 per cent did not avail a fare waiver.
(d) The railways' internal revenue for 2018-19 is estimated at Rs 2.01 trillion, 7%
higher than the revised
estimates for 2017-18
Direction (Q.126-Q.129): The farmers’ agitation in India has attracted worldwide
attention and support.
This is as it should be. Farmers are our annadata. During the Covid-19 pandemic,
while all the sectors
were thrown into a tailspin, the farm sector has sustained us.
The post-Independence history of Indian agriculture has few parallels, due to the
unique success of the
Green Revolution. Within 12-15 years, the country achieved food self-sufficiency.
Ending food imports
helped us save vast fiscal resources which could be used for development and
welfare. Unprecedented
rural prosperity ensued.
However, national-level food self-sufficiency did not result in household-level
food security. Poverty co#existed with prosperity due to inequitable resource
distribution and concentration of land. Its alleviation and
eradication necessitated welfare intervention through the Food Security Act,
MGNREGA, etc. The absence
of effective and equitable land reforms, thus, accounts for the persistence of
poverty.
The story of land reforms in India is a dismal one. Being a state subject, various
states implemented reforms
with varying degrees of effectiveness and equity. But everywhere, the objectives
were the same: Abolition
of feudal landlordism, conferment of ownership on tenants, fixing land ceilings,
distribution of surplus land,
increasing agricultural productivity and production, etc.
Many of the objectives were achieved, many were not. Feudal land relations were
abolished; tenants got
ownership rights. However, owing to manipulations in land records, much surplus
land was not available
for distribution among the landless tillers of the soil, the majority of whom were
the former “untouchables”
and today’s Dalits. Less than one per cent of the total land in the country was
declared as surplus. The
programme was implemented in a country where non-agricultural sectors and
activities were fast
developing, absorbing increasing numbers of the rural population. The relevant
criteria for land entitlement
should have been employment and main source of income.
The ex-tenants, after getting land, became tenant-turned-capitalist-farmers who
effectively made use of
several programmes-Green Revolution technology, bank nationalisation and priority
sector lending,
urbanisation and expanding urban markets. They dominated the small and marginal
farmers, and landless
farm labourers. In the 1970s and 1980s, there was an interlocking of land, labour,
credit and product
markets. Those who controlled land, controlled water, which later promoted water
trade, including drinking
water trade. They cornered a disproportionate share of various subsidies. Many
migrated abroad for quality
higher education and employment. Others reminded in India and occupied important
positions
126. Which of the following best describes the purpose of this passage?
(a) The land reforms program should not be left to the states, as it is likely to
be sabotaged by regional
satraps.
(b) The marginalised are in need.
(c) Programs and schemes for radical land reforms are called for.
(d) Both (b) and (c)

. Page 33 of 36
127. Which of the following is/are the main reason/s behind constant privation?
(a) Extremely disputed increase in rate of wages.
(b) Disempowered and excluded individuals/groups.
(c) Inequitable land amendment measures.
(d) Political browbeating
128. As per the passage, what was the role played by the non-agricultural
industry’s expansion activity towards
rural segment?
(a) The industries displaced the rural residents.
(b) The industries welcomed the rural residents.
(c) The industries took advantage of the rural people.
(d) It cannot be inferred from the passage.
129. As per the passage, should there be any apropos yardstick for obtaining land
prerogatives, if yes, specify
them?
(a) Yes, the relationship between land and caste should have been broken.
(b) No, there should not have been any specific conditions for acquiring land.
(c) Yes, Legal safeguards against market fluctuations should be sought.
(d) Yes, recruitment and a primary source of income.
Passage (Q.130-Q.132): Early in 2020, after a mysterious coronavirus emerged out of
China and then
raced across the globe, a quiet new year took a screeching turn. Stark images of
ventilated patients in
Italian hospital hallways soon filled our newsfeeds. Panic erupted across the West.
One after another,
governments that had been telling their citizens everything was fine suddenly
screamed at everyone to
shelter in place and avoid all human contact. It felt like the modern world had
just met its Black Death.
With no living memory of such scenes, Western audiences reached for the timeless
literature of apocalypse
to make sense of it all. But whereas ancient traditions of end times blamed
spiritual causes for the collapse
of civilisations, we, being the moderns that we are, opted for what we imagined to
be a ‘scientific’ discourse
– the so-called genre of collapsology. Although some modern scholars, such as
Edward Gibbon, Oswald
Spengler and Arnold Toynbee, retained essentially spiritual explanations for
civilizational decline, while
embedding them in empirical ground, those who would shape our interpretation of
COVID-19 came from a
different tradition, one that took inspiration from Thomas Malthus’s 1798 thesis
about the natural
consequences of human development.
Neo-Malthusians credited environmental feedback loops, not moral failings, for
regime collapse. In the
1960s and ’70s, works by Paul Ehrlich and Donella Meadows et al argued that the
world’s population was
growing so fast it would soon outstrip resource supplies, leading to (among other
things) widespread food
shortages. More recently, Jared Diamond wrote of the role that environmental
depletion and diseases
played in the fall of civilisations, and his theory that the collapse of Easter
Island resulted from
overexploitation of the natural environment has enjoyed particular resonance. For
its part, the COVID-19
pandemic revived old theories about the role that diseases played in regime
collapse, and we were
reminded that plagues had laid low the Roman Empire and destroyed European
feudalism.
130. What does the author mean by the following statement?
Western audiences reached for the timeless literature of apocalypse to make sense
of it all.
(a) Western audience started comparing the current situation.
(b) The western audiences started to look for old theories about the pandemics that
happened earlier
(c) Western audiences reached for the timeless literature to know the time of the
revelation of the
civilizations
(d) None of the above

. Page 34 of 36
131. Which of the following is probably true as per the passage?
(a) Pandemics are the reason for destructions and regime collapse
(b) There are spiritual causes for the collapse of civilisations
(c) The role that environmental depletion and diseases played in the fall of
civilisations
(d) All of the above
132. Which of the following examples the author has used in the passage to
strengthen the claim that the
COVID-19 pandemic revived old theories about the role that diseases played in
regime collapse?
(a) Panic erupted across the West, governments that had been telling initially that
everything was fine
suddenly howled at everyone to stay home quarantined and avoid all human contact.
(b) It felt like the modern world had just met its Black Death.
(c) End times blamed spiritual causes for the collapse of civilisations
(d) A and B
133. In the following question, different letters stand for various symbols as
indicated below:
A : Addition
B : Subtraction
C : Multiplication
D : Division
E : Equal To
Fill in the blanks.
5 C 22 B 175 A 95 _____
(a) E 324 D 70 (b) E 876 D 12 (c) E 650 D 13 (d) E 600 D 20
134. Select the letter-cluster that can replace the question mark (?) in the
following series.
GLADIATOR, HMBDIATOR, GLAFKCTOR, GLADIAWRU, ?
(a) KPEDIATOR (b) INCDIATOR (c) HMBDIATOR (d) KLADIATSV
135. In 2020, Dueutsche Lufthansa AG, a state-owned airline of Germany, lost more
than 50% of the foreign
passengers they had previously served on an average each year. Analysts say that
this extreme drop was
a result of a rise in price of tickets for international routes from $55 to $85 per
1,000 kilometers.
Which of the following, if feasible, offers the best prospects for alleviating the
problem of the drop in
passengers as the analysts assessed it?
(a) Cooperating with other airlines to provide more international routes
(b) Allowing foreign passengers to pay the same as in the previous years
(c) Reemphasizing the goals and mission of the airline as serving both domestic
passengers and foreign
passengers
(d) Offering superior VIP service for foreign passengers

mock 19

Passage (Q.1-Q.4): Thirteen Roman Catholic dioceses and some Catholic-related


groups scattered
lawsuits across a dozen federal courts last week claiming that President Obama was
violating their religious
freedom by including contraceptives in basic health care coverage for female
employees. It was a dramatic
stunt, full of indignation but built on air.
Mr. Obama’s contraception-coverage mandate specifically exempts houses of worship.
If he had ordered
all other organizations affiliated with a religion to pay for their employees’
contraception coverage, that
policy could probably be justified under Supreme Court precedent including a 1990
opinion by Justice
Antonin Scalia.But that argument does not have to be made in court, because Mr.
Obama very publicly
backed down from his original position and gave those groups a way around the
contraception-coverage
requirement.
Under the Constitution, churches and other religious organizations have total
freedom to preach that
contraception is sinful and rail against Mr. Obama for making it more readily
available. But the First
Amendment is not a license for religious entities to impose their dogma on society
through the law. The
vast majority of Americans do not agree with the Roman Catholic Church’s anti-
contraception stance,
including most American Catholic women.
The First Amendment also does not exempt religious entities or individuals claiming
a sincere religious
objection from neutral laws of general applicability, a category the new
contraception rule plainly fit. In
1990, Justice Scalia reminded us that making “the professed doctrines of religious
belief superior to the
law of the land” would mean allowing “every citizen to become a law unto him.”
In 1993, Congress required government actions that “substantially burden a person’s
exercise of religion”
to advance a compelling interest by the least restrictive means. The new
contraceptive policy does that by
promoting women’s health and autonomy.
And there was no violation of religious exercise to begin with. After religious
groups protested, the
administration put the burden on insurance companies to provide free contraceptive
coverage to women
who work for religiously affiliated employers like hospitals or universities — with
no employer involvement.
This is a clear partisan play. The real threat to religious liberty comes from the
effort to impose one church’s
doctrine on everyone.
1. According to the given passage, what can be inferred from the last paragraph of
the passage?
(a) The threat to society comes from religious liberty, when one opposes the
church’s doctrine.
(b) It all about power politics and religion is a part of that where one loses its
right to religious liberty as the
church starts imposing its doctrines on its people.
(c) Religious liberty is temporary where ultimately a society indulges itself with
the dogmatic preaching of
the church.
(d) Churches’ forceful imposition of religion harms an individual and, thus, the
societal existence is
hampered.
2. Why has the Roman Catholic Church failed to legally force their anti-
contraceptive stance in America?
(a) Because the Church’s imposition of anticontraception methods is against the
stance of the Constitution.
(b) Because the Church, although, has freedom to preach whatever it wants, it
cannot go against the laws
that protect people.
(c) According to the First Amendment, the Church is allowed to preach anything, but
it cannot impose
anything on anyone via law.
(d) Because Barack Obama’s stance on religious imposition is realistic and it
opposes the Church’s
preaching about not using anti contraceptives.

. Page 3 of 36
3. According to the passage, which of the following statement is true?
(a) Church and Government are both at fault. One tries to impose religion; the
latter tries to oppose the
liberty to practice religion.
(b) Barack Obama’s stance of making anti- contraceptives available for female
workers is not revolutionary.
It’s the way it should be.
(c) One becomes a law unto himself if all the dogmas of a religion are made
superior to the societal laws.
(d) The First Amendment makes religious institutions immune to any law suit or
legal objection.
4. Which of the following is similar in meaning to the underlined expression ‘…but
built on air.’?
(a) Realistic plan or hopes.
(b) Unrealistic; having no foundation.
(c) Exceeding plans or hopes.
(d) Unvalidated expression.
Passage (Q.5-Q.8): In 1966, the American Civil Rights Movement fragmented. Before
this year, civil rights
activists were united, in public at least, in their support of a policy of peaceful
integration of white and black
people. One of the first black activists to publicly declare the death of
integration was Stokely Carmichael.
Carmichael was the leader of a prominent civil rights organisation that had been
‘integrationist’ in the early
1960s: the Student Non-Violent Coordinating Committee (SNCC). But in 1966,
Carmichael called for black
activists in the SNCC to achieve freedom through separatism, and to withdraw from
mixed-race institutions.
In doing so, he quoted from the famous French existentialist, Jean-Paul Sartre.
Carmichael said that by
becoming a black separatist, one was becoming what Sartre had called ‘an anti-
racist racist’. Sartre had
coined this term in 1948, and Carmichael believed that ‘antiracist racism’ – or
exclusion of whites from
black organisations – would allow African-Americans to recover from the sense of
inferiority created by
white cultural and social dominance.
For historians, Carmichael’s use of Sartre’s terminology is interesting. It
indicates the impact that public
intellectuals, such as the French existentialists, can have on cultural and social
change. In this case, Sartre
provided a new term and idea that changed a reference point in public debate.
Sartre reinforced black
separatist aspiration by providing a language through which to express it. The
examples demonstrate how
French existentialism was used by activists to make a public rupture with the
social relations of the past.
Before WWII, social relations were often paternalistic. Paternalism was an ideology
which aimed to reduce
social anxiety by keeping different social groups segregated, allotting them
distinct and separate roles. A
range of groups were perceived to be free under paternalism because they were
protected – women were
protected by men, students by academics, and blacks by whites. Paternalism had
actually eroded in the
inter war period – for instance all adult women got the vote in Britain in 1928,
thereby challenging ideas
they needed political ‘protection’ from men. Nevertheless, paternalist thought
patterns persisted into the
post-war period, and were only pointedly attacked, with the help of French
existentialism and the legacy of
Nazism, in the 1960s.
5. “For historians, Carmichael’s use of Sartre’s terminology is interesting.” Which
of the following is the correct
explanation of this statement?
(a) It is interesting because he proposed a word that would segregate the black
from the white and create
counter racism.
(b) Sartre’s used term ‘anti-racist racism’, which was used by Carmichael was
indicative of the bearing the
public intellectuals made on the cultural and social changes by using distinctive
terms and ideas that
became the testimonials in debates .
(c) Carmichael used Sartre’s terminology to form a separatist agenda for the blacks
by which they can
become independent and need not depend on the whites.
(d) None of the above.

. Page 4 of 36
6. Which of the following sentence is true according to the passage?
(a) Carmichael adapted Sartre’s voice to oppose white supremacy by coining’ freedom
through separation’.
(b) Paternalism is a process which existed during and after the world wars.
(c) Before 1966, civil right activists were supportive of a policy of peaceful
integration of white and black
people, at least in public.
(d) Paternalism aimed to diminish social anxiety by keeping different social groups
integrated, allotting
them uniform roles.
7. What can be inferred from the following passage on paternalism?
(a) Paternalism is a concept by which different groups could remain segregated in a
society avoiding any
form of social anxiety.
(b) By paternalism, dictatorship becomes very easy since it follows divide and rule
policy.
(c) Since many groups remain protected, paternalism promoted open and healthy
relationships among
communities.
(d) Paternalism stayed for a while and then it disappeared from the social context
when integration ideology
reclaimed its place.
8. Which of the following characterizes the writing style of the passage?
(a) Persuasive (b) Expository (c) Narrative (d) Analytical
Passage (Q.9-Q.14): In the Khyber valley of Northern Pakistan, three large boulders
sit atop a hill
commanding a beautiful prospect of the city of Mansehra. A low brick wall surrounds
these boulders; a
simple roof, mounted on four brick pillars, protects the rock faces from wind and
rain. This structure
preserves for posterity the words inscribed there: 'Doing good is hard - Even
beginning to do good is hard.'
The words are those of Ashoka Maurya, an Indian emperor who, from 268 to 234 BCE,
ruled one of the
largest and most cosmopolitan empires in South Asia. These words come from the
opening lines of the
fifth of 14 of Ashoka's so-called 'major rock edicts', a remarkable anthology of
texts, circa 257 BCE, in
which Ashoka announced a visionary ethical project. Though the rock faces have
eroded in Mansehra and
the inscriptions there are now almost illegible, Ashoka's message can be found on
rock across the Indian
subcontinent - all along the frontiers of his empire, from Pakistan to South India.
The message was no more restricted to a particular language than it was to a single
place. Anthologised
and inscribed across his vast empire onto freestanding boulders, dressed stone
slabs and, beginning in
243 BCE, on monumental stone pillars, Ashoka's ethical message was refined and
rendered in a number
of Indian vernaculars, as well as Greek and Aramaic. It was a vision intended to
inspire people of different
religions, from different regions, and across generations.
'This Inscription on Ethics has been written in stone so that it might endure long
and that my descendants
might act in conformity with it,' Ashoka says at the end of the fifth edict. In the
fourth, he speaks of his
ethical project progressing 'until the end of the world', though one year later in
the next edict he offers a
sobering qualification; the project can succeed only as long as it is taken up and
continued - 'if my sons,
grandsons and, after those, my posterity follow my example, until the end of the
world’.
As it turns out, Ashoka 's influence did outlast the short lived Mauryan empire.
Along with Siddhartha
Gautama, the Buddha, whose religion Ashoka did much to establish as a global
phenomenon, Ashoka was
one of the first pan-Asian influences. King Devanampiya Tissa of Sri Lanka (c247-
207 BCE) wanted to
emulate him, as did Emperor Wu of China (502-549 CE); Empress Wu Zetian (623/625-
705 CE) even
wanted to outdo him. And on 22 July 1947, days before India achieved formal
independence from Great
Britain, Jawaharlal Nehru, soon to be the country's first Prime Minister, proposed
to the Constituent
Assembly of India that the independent nation adopt an Ashokan emblem - the wheel
of a chariot - for its
new flag.

. Page 5 of 36
9. Why do you think Ashoka changed his words in the fifth edict which were
mentioned in the fourth edict?
(a) He realised his descendants might not be capable enough to continue his legacy.
(b) He realised that something can only sustain when it is fully adopted by first
his immediate successors
and then the world.
(c) He realised that the stone pillars might not survive till the end of the world.
(d) He might have realised then that future generation will have different opinion
from now.
10. Which of the following argument corresponds with the author's statement:
"Ashoka's influence did outlast
the short lived Mauryan empire"?
(a) The four animals on the Lion capital of Ashoka i.e. elephant, horse, bull and
lion symbolize power,
courage, confidence and pride respectively.
(b) The pillar of Ashoka appeared to have been deliberately destroyed at some
point.
(c) Independent India adopted an Ashokan emblem - the wheel of a chariot - for its
new flag.
(d) He is remembered for the Ashoka pillars and edicts and for sending Buddhist
monks to Sri Lanka.
11. What was the method used to try to preserve the words mentioned on the Stone
pillars for longer time?
(a) Pillars might be carved using a technique unique to that time.
(b) Pillars are made of a special brick which is strong and durable, protecting the
writings on the pillars.
(c) Pillars are surrounded by stone walls on the sides.
(d) There is a roof mounted on four brick pillars which protects the rock faces
from wind and rain.
12. Which of the following is true about Ashoka's message on the stone pillars?
(a) Those messages or teachings are not relevant in today's times.
(b) The message was written in one language and translated to other languages.
(c) The message was inscribed in more than one place across his kingdom.
(d) The message was written in many languages other than Greek and Aramaic.
13. According to the passage, what was Ashoka's intention behind erecting the
inscribed stone pillars?
(a) He will be remembered by the world even after he is gone.
(b) He believed the inscription might endure long as it was written on stone.
(c) Both b) and d)
(d) He believed that his descendants might act according to the values mentioned on
the inscription.
14. What could be possible reason(s) why king Ashoka inscribed following words on
the stone pillars i.e. 'Doing
good is hard - Even beginning to do good is hard.'?
(a) He appears to be a supporter of good deeds but discourage people to follow him.
(b) He appears to believe in doing good but not always as it is hard to be good
always.
(c) He appears to know how difficult it is to do what is right and good and he
might have faced same
situation in his life.
(d) He is a calm man who believes that doing good is something that must be done.
Passage (Q.15-Q.19): The India-Nepal territorial dispute around the Kalpani-
Limpaidhura-Lipulekh
trijunction area stems only in part from the ambiguity around the original boundary
settlement. The present
flare up is a result of a combination of factors: India's strategic concerns;
deterioration in India-Nepal
relations; China's steady inroads into Nepal; and deteriorating India-China
relations.
The India-Nepal border was originally delineated by the 1816 Sugauli Treaty, which
established the river
Kali (Sharda, Mahakali) as the boundary, with territory east of the river going to
Nepal. The Kalapani#Limpaidhura-Lipulekh trijunction territorial dispute centres on
the source of the River Kali. Nepal's stance
is that that the river originates from a stream north-west of Lipulekh, bringing
Kalapani, Limpiyadhura, and
Lipulekh within its territory. India's stand is that the river originates in
springs below the Lipulekh, and
therefore the area falls within Pithoragarh District in India's Uttarakhand state.
Both sides have British-era
maps to assert their positions. India recently inaugurated the Darchula-Lipulekh
pass link road, cutting

. Page 6 of 36
across the disputed Kalapani area, which is used by Indian pilgrims travelling to
Kailash Mansarovar. The
Nepalese government protested this move, pointing out that the construction of the
road amounted to
territorial encroachment. Kathmandu subsequently released a map displaying the tri-
junction area as
Nepalese territory India responded by releasing maps that supported its position
and called for talks for a
resolution of the impasse.
However, Nepal granted constitutional validity to its stance through the
introduction of a constitutional
amendment and began tightening border security measures. Two days after the
inauguration of the link
road, Nepal's foreign minister said that the country plans to increase border
security posts along the India#Nepal border, which according to him were inadequate
in comparison to India's. This is a departure from
the status quo, and points towards a hardening of this international boundary.
The tension over this territorial dispute stems from the fact that it is a
strategic trijunction between India,
China, and Nepal. The Kalapani area is under India's control with lndo-Tibetan
Border Police (ITBP)
observation posts. Increased connectivity in border areas is critical for border
patrolling and quick
mobilisation, and New Delhi views it as being crucial for dealing with "difficult
neighbours."
15. Which of the following can be said about Indo-Nepal relations?
(a) The disputed area between India and Nepal is strategically important for only
India and not Nepal.
(b) Nepal is one of the all-time difficult neighbours of India.
(c) Nepal was dependent on India in trade and defence both.
(d) China's interference in Nepal is one of the reasons for stressful India-Nepal
relations.
16. Which of the following steps indicates towards Nepal's efforts to define the
international boundary more
clearly with India?
(a) Nepal's plan to reduce the border security posts and have a bilateral meeting.
(b) Nepal's plan to increase the border security posts along the India-Nepal
border.
(c) Nepal's protest against the encroachment in the area which it believes to be
its territory.
(d) Nepal's foreign minister indicating that they have inadequate security posts
along the India-Nepal
border.
17. What is primary purpose behind writing the second paragraph of the passage?
(a) It talks about Nepal taking the border infiltration issue very seriously and
action taken by it to strengthen
security in the border area.
(b) It introduces the river which acts as a boundary between the two countries.
(c) It talks about the reason behind the origin of dispute and the actions taken by
both the countries after
the dispute began.
(d) It talks about the multiple reasons which are responsible for the
confrontational attitude of both
countries.
18. What will happen if it is proved that river Kali originates in springs below
the Lipulekh?
(a) The issue will be resolved in the favour of Nepal.
(b) It will alleviate Nepal's concern on India's encroachment in the Nepalese
territory which is actually a
strategic trijunction.
(c) Kalpani, Limpiyadhura and Lipulekh will fall under the territory of India.
(d) It might lead to a new dispute which will be the result of the discontentment
caused due to the unfair
settlement of the presence issue.
19. Which of the following is true about the Sugauli Treaty, as per the given
passage?
(a) Sugauli treaty is the one which demarcated the India-Nepal border in 1816.
(b) The treaty which created a threshold between India- Nepal was registered in
1816.
(c) The India-Nepal Borders had a treaty in 1816 which is known as Sugauli Treaty.
(d) Suguali Treaty is between India, Nepal and China explicitly mentioning the
territorial demarcation.

. Page 7 of 36
Passage (Q.20-Q.24): Poor women in India’s villages are more likely to take up jobs
if their wages can be
deposited into their bank accounts and they can be trained in digital banking, a
September 2019 study by
the US-based National Bureau of Economic Research has concluded. This eases
patriarchal social norms
and increases empowerment among the one section of Indian society with the least
labour market
experience, it added.
If poor, rural women can control their access to wages through bank accounts and
receive adequate
training for handling it, they are more likely to join or continue in the labour
workforce in India, the study
found. It also helped in accommodating changes in gender norms on women going to
work: The study
found that women who received digital deposits and training were more likely to
hold female work in high
regard. Although their husbands did not change their personal beliefs, they became
less likely to report
that husbands suffer social costs when their wives work.
Researchers used randomised control trials to study the effects of channelling
women’s wages under the
Mahatma Gandhi National Rural Employment Guarantee Scheme, into their individually-
controlled bank
accounts, and not the account of the head of their family, typically a man. Women
who received digital
wage deposits as well as the training to use their bank accounts, were found to be
working more, as we
said earlier, in both jobs generated by MGNREGS and the private sector. This
increase occurred even
though the market wage remained static.
The study was conducted in collaboration with the Madhya Pradesh state government
and the rural
development ministry in “socially conservative” areas in the northern pockets of
the state – Gwalior,
Morena, Sheopur and Shivpur.
“Despite robust economic growth, the female labour force participation rate has
declined from 37% in 1990
to 28% in 2015, making Indian women some of the least employed in the world,” the
study noted. India’s
growth trajectory and the well-being of its population, will depend on how well it
uses public policy to lower
barriers to female employment, it said. Policy, when appropriately designed, can
empower women in
homes and even dilute common patriarchal norms, said Charity Troyer Moore, co-
author of the study and
director for South Asia Economics Research at The Whitney and Betty MacMillan
Centre for International
and Area Studies at Yale University.
“By working with women to open accounts, training them on how to use the accounts,
and linking those
accounts to NREGS so they could receive their wages as mandated, we see important
improvements in
women’s financial activity, paid work, especially in the private sector, and views
on women and work,” she
said.
20. Which section does the author refer to when he/she says, “This eases
patriarchal social norms and
increases empowerment among the one section of Indian society with the least labour
market experience,
it added.”?
(a) Urban women (b) Rural women
(c) Daily wage labourers (d) Unemployed women
21. Which of these presents a contrast to the following sentences as mentioned in
paragraph 2: The study
found that women who received digital deposits and training were more likely to
hold female work in high
regard.
(a) Women are unaware of the importance of skilled work.
(b) Financial stability is an incentive that drives more women towards employment.
(c) Women prefer hard cash instead of digital deposits.
(d) Digital deposits and training make women respect their male counterpart more as
they realise the
importance of earning.

. Page 8 of 36
22. What is tone of the fifth paragraph?
(a) Mocking. (b) Cynical. (c) Laudatory. (d) Objective.
23. Given below is a possible inference that can be drawn from the facts stated in
the third paragraph. You
have to examine the inference in the context of the passage and decide upon its
degree of truth or falsity.
"The head of the family, usually a man, can control women of the family by
controlling their money."
(a) Definitely true (b) Probably true
(c) The data are inadequate (d) Probably false
24. Which of the following is true about "social costs" with reference to the
passage?
(a) Patriarchy doesn’t believe in men living off their wives' wages.
(b) Men might suffer humiliation if they have a working wife at home.
(c) Husbands see an earning wife as a personal failure.
(d) None of the above.
Passage (Q.25-Q.30): Galo Rodriguez uses his machete to dig a hole near the small
stream on his farm
in the north-east of Ecuador, on the cusp of the Amazon rainforest. As he digs
there is nothing unusual to
be seen - but when he hits 32cm below the surface, the soil releases a distinct and
pungent smell of
gasoline. More than half of his 35 hectares of land is primary forest, while the
rest is sugar cane or small
trees. But where he digs is devoid of trees or crops. It is covered only by grass.
This area used to be
smothered in oil after a nearby pipeline leaked. The oil filled his stream, killed
of all of his fish and
contaminated the only fresh water source he used for his cattle. The oil sat here
for 10 years before the
company responsible for the pipeline came to clean it up, in 2016. Rodrigo says he
watched as they
collected some 12-15,000 cubic metres of oil off his property, but they didn't
remediate the soil. Today, in
the stream just beside him, blue and green streaks of oil residue can still be seen
in the water.
"For 10 or 11 years, this area didn't produce anything, so we abandoned it," says
the farmer. "Now we plan
to plant guavas and Chaya." Rodriguez is one of dozens of farmers in the north of
Ecuador learning how
to use plants to try to eliminate the oil contamination from his land. This
process, known as bioremediation,
uses living organisms like plants, fungi and microbes to break down pollutants,
including crude oil. There
are several ways this could happen, but most of the hard work to break down crude
oil happens below
ground, where microorganisms are concentrated around the roots of plants and
mineralise, or decompose,
the crude components, making it easier for plants to take up. Some contaminants can
be taken in by the
plant directly and stored in its shoots and roots, or can be evaporated through the
leaves. The course
Rodriguez attends is called "Guardians of the Soil", which is an introduction to
permaculture-based
bioremediation for low-income communities, founded by local resident and
independent researcher, Lexie
Gropper.
Ecuador's northern Amazon rainforest has seen heavy oil contamination since rich
oil fields were
discovered here in the 1960s. One source of contamination was by the oil company
Texaco - later acquired
by Chevron – which dumped billions of gallons of oil waste in the Amazon
rainforest, most of which went
into unlined, open-air pits in the ground. In 1993, thousands of community members
filed a lawsuit against
the company, saying it did not perform any adequate clean up and its drilling
installations continued to
contaminate the area, and demanded they pay for remediation. The oil company
admitted to releasing the
waste, but said it cleaned up its share of the contamination and was legally
cleared of all future liabilities.
Most recently, a court in the Hague found in favour of Chevron. This has turned
into one of the most
complex and longest-running environmental legal battles in history.
25. According to the passage, what is bioremediation?
(a) The process of revitalising the water bodies by using chemicals and machines.
(b) The process of cleaning the ocean in order to get rid of plastics contaminating
The river.
(c) The process of contaminating the soil using living organisms like plants, fungi
and microbes.
(d) The process of using living organisms like plants, fungi and microbes in order
to break down pollutants,
including crude oil, in the soil.

. Page 9 of 36
26. Who founded 'Guardians of the Soil'?
(a) Lexie Gropper, a local resident and an independent researcher.
(b) Galo Rodriguez, a farmer.
(c) Dian Fossey, a research student at Brown University
(d) Milton Friedman, a local resident and an independent researcher
27. Where are micro-organisms present in order to facilitate the breaking down of
crude oil?
(a) Around the roots of the plants.
(b) In the leaves of the plants.
(c) In the stem of the plants.
(d) In the fruit of the plants.
28. What did the community members claim while filing a lawsuit against Chevron in
1993?
(a) The company did not perform any effective clean up activity and continued to
contaminate the area by
its drilling installations.
(b) The company displaced thousands of community members from their ancestral lands
in order to carry
out its operations.
(c) The company was depleting the water table of the area, rapidly.
(d) The company was releasing untreated industrial waste into the local water
bodies.
29. Which of the following is/are TRUE according to the passage?
i. Due to the pipeline leak, oil filled up the near-by stream, thereby killing all
of Rodriguez's fishes.
ii. Ecuador's northern Amazon rainforest has seen heavy oil contamination since
rich oil fields were
discovered here in the 1960s..
iii. Rodriguez' farm did not produce anything for 25-26 years after the pipeline
leakage.
(a) Only i) (b) Only ii) (c) Both i) and ii) (d) Both ii) and iii)
30. Which is similar in meaning to the word ‘pungent’, as mentioned in the passage?
(a) Putrid (b) Strong (c) Mild (d) Stimulating

Directions (Q.66 – Q.105): Read the comprehensions carefully and answer the
questions based on it.
Passage (Q.66-Q.70): A father does not own the daughter to dictate terms and every
child has a right to
use his or her mother’s surname, the Delhi High Court Friday observed.
The court’s observation came while hearing a plea by a minor girl’s father seeking
direction to the
authorities to reflect his name as his daughter’s surname in the documents and not
her mother’s name.
Justice Rekha Palli, however, declined to pass such a direction and said, “A father
does not own the
daughter to dictate that she should use only his surname. If the minor daughter is
happy with her surname,
what is your problem?” The court said every child has a right to use his/her
mother’s surname if he/she
wishes to.
During the hearing, the man’s counsel submitted that his daughter is minor and
cannot decide such issues
on her own and that the child’s surname was changed by his estranged wife.
He claimed that the change in name will make it difficult to avail insurance claims
from the insurance firm
as the policy was taken in the name of the girl with her father’s surname.
The court, which declined to allow the plea, disposed of the petition with a
liberty to the man to approach
his daughter’s school to show his name as the father.
Source: Child has right to use mother's surname: HC,
66. Rini Chakraborthy, a minor of 14 years, has taken the surname of her mother
after the separation of her
parents and changed her name to Rini Das. She, after attaining majority, wants to
claim the benefit of the
Sukanya Samridhi Yojana, which her father had taken for her when she was born. Her
father denies her
claim stating that she has changed her name from the one mentioned on the papers of
the policy. You are
the judge in the case. Decide the matter as per the given principles in the
passage.
(a) The daughter is not eligible to get the benefit because she has willfully
obtained the surname of her
mother, and eventually, her name does not match with the name mentioned in the
policy document.
(b) The daughter, even if, has attained the surname of her mother willfully, will
still be entitled to the benefit
of the scheme because her autonomy over her name does not affect the relationship
with her father.
(c) The daughter is entitled to avail the benefit of the scheme because she cannot
be denied for the same
merely on the grounds of the change of her name.
(d) The daughter, after willfully attaining the surname of mother, has renounced
all such rights over her
father’s assets and other claims.
67. Taking the facts from the previous question, if Rini’s mother remarries to a
person named Satish Ranjan.
Thereafter, she changed her daughter’s name to Rini Ranjan. Rini, after attaining
her majority, claims the
benefit of the Sukanya Samridhi Yojana, which her biological father had taken for
her when she was born.
Her biological father claimed that she has no right to claim the benefit of the
policy since she has changed
her surname. Decide the case as per the given legal principles in the passage.
(a) She can be denied of the benefits over the scheme because her mother remarried
to another person.
(b) She cannot be denied of the benefits because she has attained the surname of
Mr. Satish Ranjan,
willfully, and her right over her biological father’s claims cannot be said to be
renounced.
(c) She cannot be denied of the benefits because the scheme was taken for her
benefits after she attains
majority; hence, it has nothing to do with her personal relations.
(d) She can be denied of the benefits because her name has been changed and does
not match with the
name mentioned in the policy documents.
68. Which one of the following statements best reflects the central idea of the
passage?
(a) A father does not have the dictate/authority over his child and cannot deny her
benefit merely on the
basis of the separation.
(b) Every child has a right to choose his or her name or surname willfully, and no
parent can deny of her
any benefit.
(c) A child has the autonomy over her name; hence, can attain any of the name,
which she likes.
(d) Every child has a right to use any of his/her parents’ surnames.

. Page 16 of 36
69. Subansiri, while she was 15, has decided to keep the surname of her father even
after she has been given
in the custody of her mother after the separation of her parents. Her mother,
Sheela Desai, after attaining
her custody, became her natural guardian and can take decisions on her behalf. She
wants to change her
daughter’s name with her surname. What will be the most appropriate reasoning in
this matter as per the
given information in the passage?
(a) Mother can change the name of her minor daughter because she is her natural
guardian, and can take
decision on behalf of her.
(b) Mother can change the name of her minor daughter because she has been given the
custody of her.
(c) Subansiri has the autonomy over her identity; hence, she can use whatever name
she likes to use as
it is her fundamental right.
(d) Subansiri, even if, minor has the right to keep her father’s surname even after
the separation of her
parents because her mother does not own her to dictate terms.
70. Purva Maurya willfully became Saba Ahmed by taking her mother’s surname and the
religion after the
separation of her parents. After the unfortunate demise of her mother, her father
has been given her
custody. Her father asked her to retain his surname, and religion otherwise she
will not be entitled to
ancestral property. Decide the case as per the legal information given in the
passage.
(a) The Hindu ancestral property can be bequeathed among the Hindus only, hence,
her father is correct
in his actions.
(b) She cannot be denied of her right over her ancestral property merely on the
grounds of the religion she
has adopted.
(c) She cannot be denied of her right over her father’s property because she
willfully obtained her mother’s
identity.
(d) She can be denied of her right over ancestral property because her father is
not dictating any term on
her, but the law says so.
Passage (Q.71-Q.76): The Supreme Court observed that an employee who made a false
declaration
and/or suppressed the material fact of his involvement in a criminal case should
not be entitled to an
appointment or to continue in service as a matter of right.
"Where the employer feels that an employee who at the initial stage itself has made
a false statement
and/or not disclosed the material facts and/or suppressed the material facts and
therefore he cannot be
continued in service because such an employee cannot be relied upon even in future,
the employer cannot
be forced to continue such an employee. The choice/option whether to continue or
not to continue such an
employee always must be given to the employer", the Bench held.
The question is not about whether an employee was involved in a dispute of trivial
nature and whether he
has been subsequently acquitted or not. The question is about the credibility
and/or trustworthiness of such
an employee who at the initial stage of the employment, i.e., while submitting the
declaration/verification
and/or applying for a post, made a false declaration and/or not disclosing and/or
suppressing the material
fact of having involved in a criminal case. If the correct facts had been
disclosed, the employer might not
have appointed him. Then the question is of TRUST.
71. Abhik had gotten through all the rounds of vetting to be employed in Superstore
Stores Co. On the day of
his final interview, he was put through a past vetting process wherein he was given
a form to be filled,
which sought to collect his information regarding any past criminal records that he
may have. He decided
otherwise as his once pickpocketing record of harming his chances of getting a job
at the company. The
company did not find out for the next eight years until he left the offices for a
better job at Megastores Store
Co. Is he liable for an offence in this case?
(a) Yes, as he will betray the trust of the employers in future at Megastores Store
Company.
(b) No, as he did not violate the company's trust in any manner.
(c) No, as his omission was not found out until he resigned.
(d) Yes, as his omission still counts as a breach of trust.

. Page 17 of 36
72. In the above case, other facts notwithstanding, had the company not enquired
about his criminal records,
could he be sued for breach of trust in this case?
(a) No, as Superstores should have enquired about it in the first place.
(b) Yes, as Superstores was entitled to full disclosure on hiring someone.
(c) Yes, as the employer should be furnished all the requisite information a
prudent person deems fit for
disclosure.
(d) No, as there was no outward obligation to disclose his past records.
73. For hiring the foreman in the Superstores stores, there was a rule to collect
all the past information of
foreman from public records. The HR head official failed to collect the past
records from public records,
which the candidate had filled out with a public record. Herein, the information
about the record was found
out by Superstores after four years of him being employed. Would candidate be
liable for breach of trust
here?
(a) No, as he cannot be held responsible for the HR head's irresponsibility.
(b) Yes, as it was his responsibility to submit the form.
(c) Yes, as he was clearly asked for his criminal records and should have furnished
the same.
(d) No, as the disclosure requirement was made apparent here.
74. While vetting Abhik's records, the employer was made aware of a bar-fight 10
years ago, wherein he had
to pay a paltry fine and issue an apology. Would this information be considered
material in his selection as
an employee?
(a) Yes, as he has a valid and existing criminal record.
(b) No, as his criminal record was not a serious one.
(c) No, as he was applying for a mere superstore job.
(d) Yes, as regardless of the record being paltry, it was still a criminal record.
75. Suvarshri was an employee in Tata Motors. While going through her evaluation
therein, it was found that
she was summoned to the Court twice, once to act as a witness and once to pay for
contempt of court fine
for acting hysterically while testifying the first time. Which she fails to
disclose in her form to Tata Motors.
This led to the company terminating her contract of employment on the grounds of
breach of trust. Could
she claim legal recourse for her termination?
(a) No, as she did break the fiduciary relationship in her contract.
(b) No, as the contempt charges were a criminal offence that she should have
disclosed.
(c) Yes, as she was already employed and made the disclosure when asked to.
(d) Yes, as contempt of court is not a criminal offence.
76. While being hired, Subhashish was enquired about any past criminal records of
his. While making these
disclosures, his juvenile record of 15 years back was brought to light, wherein he
was arrested and kept in
custody for a night for pickpocketing his neighbour. Would a record of this affect
his potential hiring?
(a) Since it is a minor record, it would not affect his employment.
(b) No, as the record was 15 years old and very insignificant.
(c) Yes, as he was kept in custody..
(d) Yes, as the disclosure was a mandate and capable of affecting the employment
selection.
Passage (Q.77-Q.82): A Division Bench of Chief Justice S. Manikumar and Justice
Shaji P.
Chaly observed so while adjudicating upon a plea seeking a direction to the
Director of Health Services to
take appropriate disciplinary action. The Bench, however, noted that this was a PIL
registered in a service
matter. Reliance was placed on a number of Apex Court decisions to point out that
it is a settled law that
PILs in service matters cannot be encouraged.
For instance, in N. Veerasamy v. Union of India [(2005) 2 MLJ 564] it was observed
that service matters
are essentially between the employer and the employee and it would be for the State
to take action under
the Service Rules and there is no question of any public interest involved in such
matters.

. Page 18 of 36
The Court further examined another aspect as decided in Dr. Duryodhan Sahu & Ors v.
Jitendra Kumar
Mishra & Ors where it was held that the Administrative Tribunal constituted under
the Act cannot entertain
public interest litigation at the instance of a total stranger.
This position was reinforced in Ashok Kumar Pandey v. State of W.B whereby it was
stated that where the
petitioner has not even a remote link with the issues involved, it becomes
imperative for the court to lift the
veil and uncover the real purpose of the petition and the real person behind it.
Accordingly, the Court found it appropriate to dismiss the petition.
77. X owned multiple showrooms of pre-owned luxury cars in and around Delhi. One of
the staffs, Y in one of
the showrooms of X met with an accident so he hired Z, the son of Y for a month to
fill Y’s position. After
few days of working, Z realised that most of the cars which are being sold are
actually stolen cars which X
does not knows about. Z investigates further and finds that there are several gangs
in Delhi who along with
the help of local police steal cars and then sell it off to people like X. Z files
a PIL for the same. Determine
the maintainability of his PIL.
(a) The PIL will be dismissed, because there is an employee-employer relationship
between X & Z.
(b) The PIL will be entertained, because Z is not a full-time employee of X and
there is no service matter
in the contention.
(c) The PIL will be dismissed because Z has no link with the issues raised in the
PIL.
(d) The PIL will be entertained because the issue it has raised is indeed of a
public interest.
78. Suppose in the previous question. X himself was the one who used to bribe local
policemen and was the
head of the racket which stole the cars which Z did not know. Now with the
established relation between X
& Z and the change of facts, would your answer to the previous question change?
(a) Yes, the PIL will be dismissed.
(b) No, the PIL will be dismissed.
(c) Yes, the PIL will not be dismissed.
(d) No, the PIL will not be dismissed.
79. Aman owned the entire 2nd floor of the building opposite Dalal street, Mumbai
where he had his consultancy
firm. He had 20 employees for the past 5 years. One day one of the employees, Naman
got an intuition
that Aman is involved in insider trading; soon this reached the ears of Aman, and
he immediately fired
Naman and his 9 friends from the firm. This move, made everyone apprehensive that
Aman is indeed up
to something fishy. Ajay, another employee of Aman who was not fired; files a PIL
in Bombay high court
that, the files of the firms need to be made public since it pertains to insider
trading in the National stock
exchange and hence a matter of public interest, he also challenges the firing of
the 10 employees without
any concrete reason, in the same PIL. Determine whether this PIL will be
entertained or not.
(a) The PIL will be entertained because the PIL indeed pertains to public interest.
(b) The PIL will be entertained because there is no element of service matter in
the PIL.
(c) The PIL will be dismissed because the PIL was filed by Ajay who still had an
employer-employee
relationship with Aman.
(d) The PIL will be dismissed because there is no public interest in the issues
raised.
80. Suppose in the previous question, it was Naman who files PIL and not Ajay who
is still the employee of
Aman. Would this change in facts, change your answer to the previous question?
(a) Yes, the PIL will be dismissed.
(b) No, the PIL will be dismissed.
(c) Yes, the PIL will not be dismissed.
(d) No, the PIL will not be dismissed.

. Page 19 of 36
81. X was a tea seller, and had his shop beside ABC Banks Pt. Ltd. With his
frequent visit to the head branch
to serve tea, he realized that the senior managers of the bank are taking bribes to
verify and pass loans
even when the customers are fraud. He files a PIL in High Court that public money
is in danger and the
banks needs to shut down. The bank files the counter claim that X is a total
stranger to the issue since
there is no potential loss to him and only state can take action under service
rules. Is the argument of the
bank valid?
(a) Yes, both the arguments of the bank hold true as per the passage.
(b) The argument is partially valid; only the argument that X is a stranger is
valid.
(c) The argument is partially valid: only the argument that only state can take
action under service rules is
true.
(d) No, both the arguments of the bank are incorrect as per the passage.
82. Suppose in the previous question, X was also the customer of the bank, in that
case would your answer to
the previous question change?
(a) Yes, only one of the arguments stands valid.
(b) No, neither of the arguments are valid.
(c) Yes, neither of the arguments are valid.
(d) No, only one of the arguments stands valid.
Passage (Q.83-Q.88): The Supreme Court set aside a Karnataka High Court judgment
that held that a
divorced daughter would fall in the same class as an unmarried or widowed daughter
for the purpose of a
compassionate appointment.
The Bench reiterated that norms prevailing on the date of consideration of the
application should be the
basis of consideration of the claim for compassionate appointment.
In this case, the writ petitioner's mother was employed with the Government of
Karnataka. After she died,
the writ petitioner filed for an appointment on compassionate appointment.
Karnataka High Court was
directed to consider the application. The High Court interpreted and observed that
a divorced daughter
would fall in the same class as an unmarried or widowed daughter, and therefore, a
divorced daughter has
to be considered on par with 'unmarried' or 'widowed daughter.'
In appeal, the Supreme Court summarized the observations made about the grant of
appointment on
compassionate ground in earlier judgments and concluded:
(i) That the compassionate appointment is an exception to the general rule;
(ii) That no aspirant has a right to compassionate appointment;
(iii) The appointment to any public post in the service of the State has to be made
on the basis of the
principle in accordance with Articles 14 and 16 of the Constitution of India;
(iv) Appointment on compassionate ground can be made only on fulfilling the norms
laid down by the State's
policy and/or satisfaction of the eligibility criteria as per the policy;
(v) The norms prevailing on the date of the consideration of the application should
be the basis for
consideration of the claim for compassionate appointment.
83. Bansi Lal was a reputed employee in PWD, New Delhi. He had a family of four
persons completely
dependent upon him, including his son-in-law, who was a deadbeat stay-at-home
drunk. The only reason
Sheela, his daughter, decided to marry him was because his childhood friend, Mayuri
Ram, had requested
Bansi lal to do so. Due to an ailing heart condition, Bansi died unforeseeably,
which led to a tumult in his
household. In order to get rid of the bedraggling deadbeat, Sheela divorced her
husband immediately, and
the Lal's cut their losses. After her father's rites, she decided to file for a
compassionate appointment in the
PWD where Bansi was posted. Could she be granted the same?
(a) Yes, as the household is now dependent on Sheela for sustenance.
(b) No, as Bansi's appointment does not ensure Sheela’s appointment as well.
(c) No, as she would not fall under the criterion for eligibility mentioned above.
(d) Yes, as she meets the criteria for eligibility to be appointed on the grounds
of compassion.

. Page 20 of 36
84. In the above case, notwithstanding other facts, had Sheela divorced her husband
a month before her
father's demise, would she be liable to be appointed on compassionate grounds?
(a) Yes, as she would qualify as an unmarried daughter.
(b) Not as a matter of right.
(c) Yes, as the whole family now depends on her.
(d) Both A and C.
85. In the same case as above, notwithstanding other facts, if Sheela had known
that her father would
eventually succumb to his illness, would she then have been able to claim an
appointment to her father's
job on compassionate grounds?
(a) Yes, as the knowledge of impending demise does not discount the question of
their sustenance.
(b) No, as she had the knowledge of her father's probable demise and still went
ahead with the divorce.
(c) No, as the divorce in question was premeditated.
(d) No, as she is not eligible for the appointment.
86. In the Jal board of Dehri at Sone, Bihar Ramakant was employed as the regional
head. He used to manage
the entire districts under his belt. He had been working for the Board for almost
30 years before his sudden
and untimely demise. He was very well-liked in the Board, and he and his family
were very well known in
the offices. Due to his sudden demise, the department made haste in appointing one
of his children of the
age of 18 as junior clerks in order to let his family sustain whilst the provision
for compassionate
appointment in the Board was being looked into. It was found out that the Board has
no authority by the
statute to provide any such employment. As a result, Ramakant's son was terminated.
Can he plead for
any recourse in law?
(a) No, as he was wrongly appointed because there was no provision for
compassionate appointment and
thus, cannot plead for recourse.
(b) Yes, as his appointment could not be terminated in an arbitrary manner.
(c) No, as he is not as educated as his father.
(d) Yes, as he was entitled to compassionate appointment in this case.
87. In the same case as above, while terminating the son's appointment, the Board
made it clear that his
mother could be appointed on the same grounds as the provision for the same exists.
Is this provision
constitutional? According to law.
(a) No, as it violates articles 14 and 16.
(b) Yes, as it allows for a family member to be appointed on compassionate grounds.
(c) Yes, as it promotes equality amongst equals.
(d) No, as the same violates article 16.
88. Can article 14 be invoked while deciding the appointment of a Scheduled Caste,
a disabled person and a
general person on compassionate grounds?
(a) Article 14 ensures equality amongst every person; everyone would be liable to
be appointed.
(b) Article 14 ensures quality amongst equals only; everyone shall be subjected to
different provisions.
(c) Article 16 shall be applicable here to ensure no discrimination.
(d) Article 14 shall not be applicable here due to different social classes
persisting here.
Passage (Q.89-Q.94): The SC has recently observed that filing complaints seeking
the removal of one's
spouse from a job amount to mental cruelty.
"The fact that there have been continued allegations and litigation proceedings and
that can amount to
cruelty is an aspect taken note of by this court," the Bench said. The Court
acknowledged that a Constitution
Bench was examining the larger issue regarding the exercise of powers under Article
142 of the
Constitution to dissolve the marriage between consenting parties. However, it noted
that in various
instances, the Supreme Court had exercised this power to grant a divorce.

. Page 21 of 36
The Division Bench observed that the issue before the Constitution Bench had been
pending for the last
five years, and therefore if a marital tie was not working, no purpose would be
served by postponing the
inevitable simply for the pendency of the reference.
The Bench concluded without hesitation that the marriage had not worked and cannot
work, not only on
account of the appellant's second marriage but also because the parties were so
troubled by each other
that they were not willing to even think of living together.
"One of the most difficult situations is where there is irretrievable breakdown of
marriage, but only one of
the parties is willing to acknowledge the same and accept divorce on that account,
while the other side
seeks to oppose it even if it means carrying on with the marriage."
Therefore, the marriage was dissolved not only in the exercise of powers under
Article 142 of the
Constitution of India on account of irretrievable breakdown of marriage but also on
account of cruelty under
Section 13(1)(i-a) of the Act.
89. In a divorce case before family court, the Court observed that the couple was
not amenable at all, and the
husband had been giving her regular loans to run her own parlor for her own
sustenance. The husband
pleaded that the amount of the same be returned to him, in accordance with the
agreement they had signed
@8% rate of interest. When no relief was being expediently delivered to the
husband, he initiated recovery
proceedings for the same. The Court observed that two parallel proceedings would
wrest her out of a lot
of money and thus decided to put an embargo on the recovery proceedings stating
that it amount to cruelty.
Can this decision of the Court be considered tenable?
(a) No, as the recovery proceedings were completely legal.
(b) No, as the recovery proceedings were in accordance with the agreement.
(c) No, as the recovery proceedings cannot be termed as cruelty.
(d) No, as the recovery proceedings are not the domain of the family courts.
90. During the course of the proceedings, it was found that the husband also had
advanced loans to his brother#in-law and father-in-law in their times of need,
which was to be repaid without interest but was due. The
Court let the recovery proceedings run parallel against the brother and the father
but put an embargo on
the proceedings on the recovery proceedings against the wife. Is the Court's
decision viable?
(a) No, as all recovery proceedings should be allowed to run.
(b) No, as all recovery proceedings would not be termed as cruelty.
(c) Yes, as the proceedings against the wife and her family are all tenable.
(d) Yes, as the proceedings against the wife are untenable and cruel.
91. During the arguments stage, the Court found that the marriage was
unsalvageable, even if the couple did
not think so at that stage due to antagonistic exchanges. They opined on couple's
counseling sessions,
which turned out to be disastrous as they did not talk at all, and all the
expensive sessions were a waste
of money. Was the decision of the Courtis justifiable?
(a) No, as the Court was not acting under guidelines of the court.
(b) Yes, as the Court was trying to salvage a marriage relationship.
(c) Yes, as the Court was not supposed to opine on their relation.
(d) No, as the Court was only meant to rule on the divorce.
92. After the counseling sessions, the Court was finally of the opinion that the
marriage was irretrievable and
decided to grant them divorce. However, this made the husband plead for another
chance as he wanted
to give his marriage another try and thus, started begging his wife in the
courtroom is ready to give him one
more chance. Can his pleading sustain?
(a) Yes, the Court should abstain from granting a divorce.
(b) No, as the Court has already opined on granting a divorce.
(c) No, as the arguments stage has been passed over.
(d) Yes, as the Court has complete discretion on ruling over all family matters.

. Page 22 of 36
93. In a separate divorce matter, the Court found that the husband had fired his
wife from his own firm as she
was employed by him since they were looking to divorce each other. The Court found
that firing her would
mean depriving her of her livelihood. Thus, the Court held her termination to be
illegal and cruel. Does
there exist any recourse in law against the action of the Court?
(a) Yes, as he has all the right to terminate her employment in the matter.
(b) Yes, as the termination of employment is a natural reaction to their split.
(c) No, as personal biases have been observed to come into play here.
(d) No, as termination amounts to cruelty.
94. The husband, in the aforementioned matter, also filed for the action of cruelty
since his wife had taken
almost all of the stuffs of their home since her father had gifted the furniture
and everything else to both of
them, due to the husband being destitute back then. Can the husband claim this as
being cruel?
(a) No, as he has a successful means to earn sustainably.
(b) Yes, as a marriage means a share in everything by either of the parties.
(c) No, as all the furniture was given by her father in the first place.
(d) Yes, as taking away the furnishing from home being shared can be termed as
cruelty.
Passage (Q.95-Q.100): The Delhi HC has recently opined that a mere change of
counsel by the parties
would not entitle the respective counsel to recall witnesses examined prior to the
change.
'The change of counsel and the decision of the new counsel to cross-examine any
witness who the previous
counsel did cross-examine is no ground to exercise power u/s 311 of CrPC for
recalling the witness where
due opportunity was afforded earlier. . Lastly, this is a criminal trial and not a
game where, if one party were
afforded an opportunity for recalling any witness for valid reasons, the other
party would ask for chance as
a matter of right without any reason. Accordingly, considering the totality of the
circumstances, this Court
is not inclined to exercise power u/s 311 CrPC to recall PW~38 as no justifiable
grounds exists for the
same. Application is accordingly dismissed."
The fact that the matter here was sub judice since 2006, and the relief for the
petitioners was long overdue,
the change in counsel did not warrant for any recall of witnesses examined or not,
thus, the petition for
allowing even one day for a recall of witnesses could not sustain. Court has
dismissed the application by
stating that Court is not inclined to exercise power u/s 311 CrPC to recall witness
but if foul play by witness
is known after judgment is ground for appeal.
Court also held that senior counsel is more respectable than junior counsel, as
they have more experience
when compared to junior counsel but both stands on the same foot being the counsel
in same matter.
Cross examination of a witness by senior counsel will not stand as ground under
section 311 of CrPC.
95. In the case of Jag Mohan v. Sadmagan, of the year 2018, the Court had finally
progressed to the first stage
of proceedings after the hearings were allowed to continue physically after eight
months of vacation. During
the same, it was found out that the counsel pleading the respondent's case had fake
credentials proving
that he was not an actual lawyer. Upon appointing another lawyer to plead his case,
Sadmagan and his
lawyer pleaded for a fresh trial. Can the Court allow for the same?
(a) No, as the Court cannot completely start the trial afresh.
(b) Yes, this is a case where an exception to the precedent can be made.
(c) No, as there is no provision in law to recall witnesses by a change in counsel.
(d) Yes, as there is evident fraud at play here, and the respondent shall be
allowed to plead for the same.
96. In the above trial, the Court observed that one of the witnesses who had been
examined earlier in the
course of proceedings and was due to be called upon again has died. What possible
course of action could
the Court opt for in this case?
(a) The witness' testimony is invalid. Other witnesses should testify.
(b) The testimony of the deceased witness stands as viable.
(c) The testimony of the deceased cannot be verified.
(d) The Court can order a fresh trial to commence.

. Page 23 of 36
97. In the subsequent proceedings, in the above case, the Court was almost done
hearing the testimonies and
was supposed to tender the judgment. However, via an anonymous letter to the
judge's chamber, the Court
was made aware of foul play in the proceedings as one of the witnesses from the
respondent's contentions
was found out to be staged. Taking note of the same, the Court levied charges of
perjury on the respondent
and his counsel and has ordered a fresh witness procedure under section 311 CrPC.
Can the court order
for the same?
(a) Yes, as the Court is competent enough to call any person as a witness under s.
311 CrPC.
(b) No, as s.311 CrPC does not allow for a fresh witness process to be initiated.
(c) No, as only one witness was found to be planted.
(d) Yes, as other testimonies could also be rigged, and perjury requires strict
action.
98. The Court had finally commenced one last time for tendering the judgment in the
above case, and the
parties had been eagerly waiting for the same. To the respondent's dismay, the same
was tendered in the
petitioner's favor. Upon the judgment having been tendered, the petitioner had
confessed to the respondent
that he had paid his witnesses to lie in front of the Court. Can the respondent
claim any recourse under the
same?
(a) No, as the trial has concluded and the judgment tendered.
(b) Yes, since the witnesses were lying, the same shall be asked to testify again.
(c) Yes, as there exists no actual trial in this case.
(d) Yes, appellate remedies may be availed.
99. In a robbery case, the counsel for the petitioner had recently passed away due
to a long ailment that had
impeded the counsel's practice. Due to that same, his junior had been pleading all
his matters in his stead.
In the present case, due to his deteriorating condition, the senior advocate had
been sending his junior as
his proxy and to plead in his stead. However, after his official demise, the Court
did not object to him being
the principal counsel in the matter. Is this decision of the Court tenable?
(a) No, as the junior counsel has not examined the witnesses or corroborated
evidence.
(b) Yes, as the demise of the lawyer cannot mean a bar on judicial proceedings.
(c) Yes, as the junior counsel was anyway practicing in his stead.
(d) None of the above.
100. In a matter of carjacking, the insurance company had appointed their own
evaluation professional to take
notice of the situation. Whilst the proceedings were underway in the courts, the
petitioner-aggrieved had
testified as the car being of 6 lakhs worth in value which the evaluator found as
being wildly inaccurate.
Due to this reason, the counsel for the petitioner had recused himself to save face
and made his junior as
the principal counsel here. Can the principal counsel recall witnesses in this
case?
(a) Yes, as in order to get the facts correct, he needs to re-examine the
witnesses.
(b) No, as s.311 cannot allow the same to be tendered.
(c) No, as the force of law, in general, prevents the new counsel from recalling
witnesses.
(d) Yes, as the situation here requires clarity in the testimony of the witnesses.
Passage (Q.101-Q.105): 'State under obligation to keep criminal record database
updated - Delhi
HC'
A Delhi Court judge noted that previous involvement of the accused plays a vital
role, when the court
applies its mind to grant of bail/refusal of bail to the accused. Therefore, a
serious responsibility lies on the
shoulders of the SCRB/Delhi Police, to regularly update the database, pertaining to
the accused persons.
In law, a person can be categorized as an under trial, as a convict and in case of
acquittal, there is no
criminal record. If despite acquittal from courts, a system reflects a person as a
criminal without indicating
the fact of acquittal then, in my view, presumption of innocence goes for a toss.
Therefore, heavy onus lies
on the department to maintain its records in such a manner that the police cannot
misuse the information
fed in the system by selectively presenting the data before the courts creating
prejudice against the
accused persons. Therefore, there is a responsibility/duty on the authority
maintaining such a database, to
keep it updated so that correct particulars and information is produced before the
court before arguments

. Page 24 of 36
on bail application are being heard. Failing to do so may be denying a person a
right to his reputation as
well as to his liberty enshrined in Article 21 of Constitution of India.
Therefore, State is under an obligation to keep such criminal databases updated and
it is excepted that the
said exercise be carried out at the earliest, as also, periodically to ensure
justice to one and all. Therefore,
first of all, SHO concerned is directed to file a reply whether information
regarding accused persons is
periodically sent to the SCRB/Delhi Police for updation or not.
Source: Bail matter No. 1876/21, FIR No. 446/21, PS V K South, State Vs. Vikram,
U/s 379/411/34 IPC
101. A person Yuvraj Singh was arrested at 10:15 p.m. on 13.07.2021 for his
casteist remarks in a viral video.
He was presented before the magistrate at 11:00 p.m. After the trial, he was
acquitted by the court on
15.07.2021 at 9:00 p.m., after the hearing went for extended court hours. The shift
of the police personnel
responsible for updating the criminal record database ends at 8:00 p.m. and
therefore Yuvraj’s arrest could
not be updated on 26.07.2021, given earlier backlog. Decide.
(a) Since the shift of police personnel ends at 8:00 p.m., the criminal record
database cannot be updated
when arrests are made after that.
(b) It is the duty of the police to update the criminal record database, and the
record was not updated
regularly in this case.
(c) There is only a twelve days delay, which is not major and can be accommodated.
(d) Yuvraj’s arrest is inappropriate. Fine should have been levied on him.
102. A person Yuvraj Singh was arrested at 10:15 p.m. on 13.07.2021 for his
casteist remarks in a viral video.
He was presented before the magistrate at 11:00 p.m. After the trial, he was
acquitted by the court on
15.07.2021 at 9:00 p.m., after the hearing went for extended court hours. The shift
of the police personnel
responsible for updating the criminal record database ends at 8:00 p.m. and
therefore Yuvraj’s arrest could
not be updated on 26.07.2021, given earlier backlog. His acquittal was not
recorded.. On 29.07.2021, he
was arrested again for theft. During the bail hearing, the court could not gather
his acquittal, and his records
only matched as being arrested on 13.07.2021. The court denied the bail,
considering him to be a repeated
offender. Decide whether the act of court is justiciable?
(a) Yes, since his acquittal was not mentioned in the criminal record database, so
presumption of
innocence is questionable..
(b) No, Yuvraj should not have faced the consequences of the mistake of the police
who did not update
the criminal record database properly.
(c) Yes, as it was police officers’s mistake for not updating the criminal record
database.
(d) None of the above.
103. A learned man, Harishad was arrested on charges of anti-India sloganeering on
18.01.1997. The police
properly recorded all the details of Harishad including the date, time and reason
of his arrest. He was
acquitted after 5 years of trial on 25.12.2002. The criminal record database of
five years before was lost at
the time of his acquittal, due to which his acquittal could not be updated in the
database. Later, when the
database was made online in 2017, the file could be accessed and the records were
made online.
Harishad’s acquittal was not mentioned in the online database. He filed a complaint
against the database
asking his acquittal to be mentioned. Decide.
(a) The complaint is not valid since it has been 15 years of his acquittal.
(b) The complaint is not valid since there is no other case against him which would
incriminate the
possibility of justice, due to the acquittal not being mentioned.
(c) The complaint is valid as the police needs to update the criminal record
database properly and in time.
(d) The complaint is valid as Harishad is a learned man and should be treated
accordingly

. Page 25 of 36
104. A learned man, Harishad was arrested on charges of anti-India sloganeering on
18.01.1997 and was
sentenced to 21 years of imprisonment. The police properly recorded all the details
of Harishad including
the date, time and reason of his arrest. Later, when the database was made online
in 2017, the file could
be accessed and the records were made online. Harishad’s acquittal was mentioned in
the online database.
Decide.
(a) Since the acquittal has been mentioned in the database, the police is abiding
by the orders of the judge
of updating the criminal record database regularly.
(b) Since it has been 20 years of his arrest, such old database is practically no
use now.
(c) There is a mistake on the part of the police in updating the criminal record
database.
(d) Since there is no other case against him so, will not affect justice.
105. Which of the following is true as per the passage?
(a) A Delhi Court judge noted that it is the duty of the SCRB/Delhi Police to
update the criminal record
database frequently, or else Article 21 would be infringed.
(b) A Delhi Court judge noted that it is the duty of the SCRB/Bihar Police to
update the criminal record
database frequently or else Article 20 would be infringed.
(c) A Bihar Court judge noted that it is the duty of the SCRB/Delhi Police to
update the criminal record
database frequently or else Article 21 would be infringed.
(d) A Delhi Court judge noted that it is the duty of the SCRB/Delhi Police to
update the criminal record
database annually or else Article 20 would be infringed.

. Page 26 of 36
SECTION - D: LOGICAL REASONING
Passage (Q.106-Q.110): Ours is an era in which there is little left of nature that
has not been extensively
altered by the activities of human beings. Among proposed remedies is preservation,
setting aside areas
that still remain undistributed and protecting them against human encroachment, and
restoration, bringing
degraded areas back to an unspoiled condition. At first thought, one might suppose
that preservationists
and restorationists would make natural allies, but even a cursory reading of the
relevant literature shows
that all is not harmony and peace between the two groups. The nub of the critique
of preservations is the
claim that it rests on an unhealthy dualism that conceives nature and humankind as
radically distinct and
opposed to each other. Dissatisfaction with dualism has for some time figured
prominently in the
unhappiness of environmentalists with a mainstream industrial society, as in the
writings of Carolyn
Merchant and Theodore Roszak.
However, the writings of the restorationists themselves particularly, William
Jordan and Frederick Turner
offer little evidence to support this indictment. In their view, preservationists
are imbued with the same basic
mindset as the industrial mainstream, the only difference being that the latter
exalts humans over nature
while the former elevates nature over humans. While it is perhaps puzzling that
Jordan and Turner do not
see that there is no logic that requires dualism as a philosophical underpinning
for preservation, more
puzzling is the sharpness and relentlessness of their attack on preservationists,
accentuated by the fact
that they offer little, if any, criticism of those who have plundered the natural
world. The crucial question,
however, about the restorationist outlook has to do with the degree to which the
restorationist program is
itself faithful to the first principle of restoration: that nature and humanity are
fundamentally united rather
than separate.
Rejecting the old domination model, which sees humans as above nature, restoration
theory champions a
model of community participation. Yet some of the descriptions that Jordan and
Turner give of what
restorationists are actually up to, for example, Turner's description of humans as
"the lords of creation," or
Jordan's statement that "the fate and well-being of the biosphere depend ultimately
on us and our
relationship with it" do not cohere well with the community participation model.
Another holistic model
namely, that of nature as an organism might be more serviceable to the
restorationists. As with the
community model, the "organic" model pictures nature as a system of interconnected
parts.
A fundamental difference, however, is that in an organism the parts are wholly
subservient to the life of the
organism. If we could think of the biosphere as a single living organism and could
identify humans with the
brain (or the DNA), or control center, we would have a model that more closely fits
the restorationists' view.
However, to consider humans as the control center of the living earth is to ascribe
to them a dominating
role in nature. Is this significantly different from the old-fashioned domination
model? In both systems,
humans hold the place of highest authority and power in the world. Also, neither
view recognizes any limits
to the scope and range of legitimate human manipulation in the world. This does not
mean that there are
no constraints; only beneficial manipulation should be undertaken. But it does not
mean that nothing is off#limits. A further parallel is that, because the fate of
the world rests on humans, they must have a clear idea
of what needs to be done.
106. Which of the following correctly states the author's primary purpose in the
passage?
(a) Examine the similarities and differences among models for environmental
philosophies.
(b) Formulate a new philosophical model of the relationship between humans and
their environment.
(c) Critique a modern-day environmental philosophy.
(d) Argue that one particular environmental philosophy is more workable than
competing approaches.
107. The author of the passage would probably agree that preservationists
(a) are not critical enough of those who have plundered the natural world.
(b) base their ideas on an unhealthy dualism.
(c) have the same basic mind-set as the industrial mainstream.
(d) have been unfairly criticized by restorationists.

. Page 27 of 36
108. Which of the following best expresses the function of the first paragraph in
relation to the passage as a
whole?
(a) To establish the parameters of an ensuing debate.
(b) To identify problem areas within a school of thought, which are then explored
in greater detail.
(c) To discuss secondary issues as a prelude to a more detailed examination of a
primary issue.
(d) To provide an historical backdrop for a discussion of modern-day issues.
109. In asserting that the organic model might be "more serviceable to the
restorationists" the author implies
that
(a) the descriptions by Turner and Jordan of the restorationists' program conform
more closely to the
organic model than to the community participation model.
(b) the organic model is more consistent than the community participation model
with the principle of
restoration.
(c) the organic model is more consistent with the restorationists' agenda than with
the preservationists'
program.
(d) holistic models are more useful than the dualist model to the restorationists.
110. Which of the following models would the author most likely agree is least like
the other models listed below?
(a) Domination model. (b) holistic model.
(c) community participation model. (d) Dualism.
Passage (Q.111- Q.115): The President of India has sent condolences to Vandana
Mishra’s family, after
her death in Kanpur late Friday night, when the car carrying her to hospital was
stopped for the presidential
convoy to pass. But while the President’s response is in keeping with obligations
of his high office, the
Kanpur tragedy is by no means unique. VVIP convoys have become yet another
manifestation of a political
and administrative culture where public servant’s self-esteem depends on the
distance they can maintain
from the public. Police are overzealous because they respond to a system where
public inconvenience is
extremely low priority.
VVIP convoys in Indian cities are particularly unwelcome. Most cities suffer
terrible traffic congestion. In
Mumbai it is estimated that a trip takes about 53%more time than it should, in
Bangalore 51% and in New
Delhi 47%. On top of this there are few green corridors for ambulances. So, road
blockades set up to
smoothen VVIP movement worsen the situation painfully. There was hope following the
2017 Union
Cabinet decision to end the lal batti raj. But long snaking VVIP convoys aren’t
part of this change. And
MLA’s from Haryana to Himachal Pradesh have tried to retain traffic privileges with
flags atop their vehicles.
By contrast, in America the idea of holding up citizens for hours for the
convenience of politicians would be
laughter. In New Zealand, when the PM’s motorcade was caught over the speed limit,
it was fined for
dangerous driving. In Scandinavian countries, far from reserving a faster lane for
themselves, netas take
public transport shoulder to shoulder with citizens. This is the direction our
democracy must take too. For
VVIP protection, the state must deploy smarter security arrangements, and nix those
convoys. When public
servants are driven, let the public not be driven to despair or worse.
111. Which among the following is the most significant message conveyed by the
author of the passage?
(a) The VVIP system is not suitable for India since it causes inconvenience to
common people.
(b) A public servant’s self-esteem carries a lot more significance than that of a
common man.
(c) It is time that the state does away with convoys and brings up a smarter
security system for VVIPs.
(d) The President should be held responsible for the death of Vandana Mishra.
112. Which of the following courses of action should India take from the various
countries mentioned in the
passage?
I. Hold up citizens for long hours for politicians.
II. Levying a penalty on the vehicles of the politicians for violating traffic
rules.
III. Oblige politicians to use public transport.
(a) Only I (b) Only II (c) Both II and III (d) All I, II and III

. Page 28 of 36
113. Which of the following assumptions must be true?
(a) The tragedy that happened due to the convoy of the President is one of the last
such tragedies.
(b) At least one of the political and administrative VVIP manifestations should be
done away in India.
(c) Deploying smarter security systems for VVIP convoys is a one-stop solution for
the perils of
democracy.
(d) VVIP convoys should not be done away with in a democracy such as India.
114. While giving examples of other countries, which among the following is the
central flaw made by the author?
(a) The traffic situation in Indian and Western countries is broadly similar.
(b) All countries being mentioned are Europeans.
(c) Time is the essence of life.
(d) Politicians should be held responsible for their actions.
115. ‘In Scandinavian countries, far from reserving a faster lane for themselves,
netas (political leaders) take
public transport shoulder to shoulder with citizens.’ Which of the following
statements, from the options,
would weaken the argument?
(a) The political leaders have been put on a pedestal by the people who wish to be
in awe of their leaders.
(b) The political leaders are known for their indifference towards development and
people may vent out
their anger on the leaders, if the leaders were to mingle with people.
(c) The leaders being the representative of people face constant death threats and
are under the radar of
those who want to sabotage the development, cannot be treated as commoners as they
hold position
of relevance.
(d) The leaders are representative of people and conducting themselves with
humility will only strengthen
their support amongst their people.
Passage (Q.116-Q.120): Research shows that intergenerational shared sites increase
the health and well#being of both young and older participants, reduce social
isolation, and create cost efficiencies. They are
joyful places. And unsurprisingly, the concept isn’t unpopular. Americans are
overwhelmingly in favor of
shared sites. Shared sites benefit and build the health-care workforce as well. One
such plan operates a
high-school program that has graduated nearly 900 at-risk students. The program has
not only sparked
student interest in health-related careers but also helped them develop a range of
useful life skills. Finally,
attracting and retaining staff—a chronic problem in the care industry—is less of an
issue at shared sites,
because employees can more easily meet their own family caregiving needs, and many
find joy working
with both children and seniors.
And yet, while successful shared sites exist in countries around the world, there
are fewer than 150 of them
in the United States, compared to tens of thousands of age-segregated care
facilities around the country.
The answer is that this relatively simple care model, it turns out, is difficult to
pull off, complicated by funding
silos and cumbersome regulations. Most funders that support care facilities focus
on either childcare or
eldercare, forcing operators of shared sites to seek and manage separate funding
sources. Furthermore,
the staffing plans, emergency evacuation procedures, and other regulations that
govern care sites for
younger people differ from those designed for older populations.
To solve this problem, the first step is increasing awareness. Simply making
intergenerational connection
more visible through the design of buildings, community engagement, and promotional
materials is one
way to do this. Further, a shared site that draws on the talents and resources of
the local community, and
that invites a range of groups to take part in its successes and tribulations will
more likely prosper over the
longer term.
. Page 29 of 36
116. Which of the following is most supported by the author’s argument?
(a) As the building plans required for eldercare is extremely different from the
same being required for
the childcare, shared sites can’t be a good idea always in the society.
(b) The staffing required for shared sites are easy to get as lots of people have
showed their interest in
joining these organisations and owing to the popularity and unique cross-
generational experience it
attracts fair interest among the youth.
(c) Though the concept of shared sites is a popular one, its advantages and
conveniences need to reach
the people to a much larger extent than what it is presently
(d) Both B and C

117. Which of the following can be inferred from the passage?


(a) Intergenerational shared sites have offered investors a scope for investing in
a tax-free and
economic-liability free area. The returns from these sites are guaranteed.
(b) According to research, many adults have shown that they prefer shared sites
over normal day care
centers. The sole reason for this has been the intergenerational experience that
they get from these
centres is unique and truly health- and mind-fulfilling.
(c) It shall be very important that the people of the local area are brought up to
speed with the
advantages of these shared sites. This can be one of the techniques to pique
interest among people
about shared sites.
(d) Intergenerational shared sites, though come with many benefits, do come with
limitations in terms of
increased costs and isolating the health care workers form critical care.

118. As per the author, why are shared sites a good initiative?
(a) Shared sites have allowed the government to solve two purposes with a single
operation – taking
care of the elders as well as the children in one single space. This is huge
problem that has been
solved.
(b) Instead of investing in two areas, the investor invests just for one centre,
which means there is a huge
scope of savings that generates from these centres.
(c) The health care benefits and workforce gets built form the shared sites,
creating an awareness of
careers for the younger generation.
(d) The staffing plans, emergency evacuation procedures, and other regulations that
govern care sites for
younger people differ from those designed for older populations.
119. Which of the following, if true, would most seriously undermine the author’s
argument?
(a) Studies suggest that young children are incapable of being very friendly and
are prone to violence
towards people elder to them.
(b) Some senior citizens do not have the energy and mental faculties to entertain
and engage younger
people.
(c) Majority of working people prefer to send their aged parents to old age homes
rather than keeping
them at home.
(d) The interaction between the older and the younger generation can prevent mental
and emotional
disturbances prevalent amongst the youth.
120. The author’s statement that “Research shows that intergenerational shared
sites increase the health and
well-being of both young and older participants” forms the:
(a) premise of the above passage. (b) conclusion of the above passage.
(c) assumption of the author. (d) It is just a general statement.

. Page 30 of 36
Direction (Q.121–Q.125): India is one of the largest producers of food grains
globally. Though its
agriculture is very small-scale, the yield from farming is low and the pressure on
land is enormous. This
needs to change with the help of digital precision technology. India’s agricultural
policies have seen a major
boost since the Modi government came to power in 2014 and decided to focus on
doubling farmers income
by 2022. The government has also set an ambitious food grains production target of
291.1 million tons for
2019-20, an increase of 2.6% compared to the previous year, citing a favourable
monsoon in the current
season. The government has chalked out plans to achieve the goal; from agriculture
productivity, soil health
cards, crop insurance, irrigation, total mechanization, technology, animal
husbandry and allied activities.
But is it on the right track, many ask. The Consortium of Indian Farmers
Associations (CIFA), India’s apex
professional farmer’s organization allege ineffective policy level interventions on
behalf of farmers from all
over India. It said recently that to double farmers income, the government needs to
implement a consistent
export-import policy and proactive market intervention to attain fair prices for
agricultural produce.
121. What can be inferred from the given passage?
(a) Modi government’s focus on doubling farmers income by 2022 is not an easy ride.
(b) Indian agricultural production is minuscule because of the massive pressure on
land.
(c) India requires an effective export or import policy to reap devious prices for
agricultural produce.
(d) The Consortium of Indian Farmers Associations is India’s nadir professional
farmer’s organization.
122. Which of the following may best explain the term ‘animal husbandry’?
(a) A controlled cultivation, management, and production of wild animals.
(b) Animals bred and raised for consumption.
(c) A controlled cultivation, management, and production of domestic animals.
(d) Animals bred and raised for sport, and research.
123. If the information contained in the passage is true, then it is also true that
(a) India needs a different set of solutions for agriculture and for those working
the land.
(b) India needs to shift from basic farming to more efficient, sustainable, and
productive farming.
(c) Today’s agriculture policies fail to recognise how crop choices, input costs,
and the supply chain are
intertwined, perpetuating marginal farming.
(d) However, even if we doubled or tripled our output, would that double or triple
farmer’s earnings? No,
since a glut would reduce prices.
124. Which of the statements is true about CIFA?
(a) They suggested implementation of consistent export and import policies.
(b) They believe pre-emptive market mediation can lead to an increase in
agricultural production.
(c) It is the topmost organization that protects farmers’ interest.
(d) All of the above
125. Which of the following statements discloses the intention of the writer?
(a) To emphasize the need to implement CIFA’s agricultural policies and rules.
(b) To laud the actions of the BJP government in achieving greater heights in
Indian production and income
of the farmers.
(c) To highlight the need to focus on ambitious and proactive advances in the
fields of Indian agriculture.
(d) To show concern over the overambitious goals of the Modi government.

. Page 31 of 36
Passage (Q.126-Q.130): Death is in the air. According to a recent study by ‘The
Network’, C40, among 61
global megacities, Calcutta recorded the highest number of premature deaths on
account of air pollution
caused by coal-fired thermal power plants in 2019. It is also projected to witness
10,000 premature deaths
by 2030. Among Indian cities, it is followed by Mumbai. This is not surprising. In
spite of much-lauded
efforts in clean energy, the carbon footprint of India’s coal-processing industry
remains stark. The country
has the world’s second highest number of operating coal power plants, none of which
is scheduled to be
phased out by 2030. In fact, a draft of the National Electricity Policy 2021
revealed plans to continue building
coal-fired plants as they provide a cheap source of power. The rampant use of dirty
coal in existing plants
only compounds the problem — 84 per cent of the power stations supplying
electricity to Bengal do not
comply with official regulations for sulphur dioxide emissions. ‘Clean coal’
alternatives are not viable
solutions either, since their contributions to greenhouse gas emissions and air
pollution are high. Clearly,
a move away from coal as a whole is necessary.
Policies, however, continue to be guided by false assumptions. While rural
households are believed to
contribute majorly to coal consumption, coal accounts for about 70 per cent of
India’s electricity generation,
consumed at a much higher rate in urban centres. What available data suggest is a
worrying gap between
India’s commitment to adopt clean energy sources and the reality on the ground. To
meet the 2030
emissions goal committed at the Paris Pact, India would need to reduce coal
capacity in and around its
C40 cities by about 22 per cent. Yet, going by the current coal plans, it is
expected to increase by 20 per
cent instead. To bridge this gap, which is unlikely, policymakers need to look
beyond populist steps. While
the Ujjwala scheme promises free liquefied petroleum gas — an environment-friendly
alternative to coal —
to poor households, the government seems oblivious to the plight of the Indian
middle class, struggling to
afford LPG because of soaring prices. Investments in renewable energy need to be
increased manifold —
it constitutes just 33 percent of the country’s recent spending on energy
infrastructure. Populism must be
junked and changes in public lifestyle encouraged in policy for India to cleanse
the air.
126. Which among the following can most likely be inferred from the passage?
(a) The policy makers tend to appeal to the ordinary people while making decisions
on coal.
(b) The Ujjwala scheme caters to the demands of all the economic groups of the
country.
(c) India has a proven track record on keeping the promise it makes, especially
about environment
protection.
(d) None of the above can be inferred from the passage.
127. Which of the following can be considered as an opinion of the author? Choose
the best answer.
(a) The author is bewildered by the findings of ‘The Network’.
(b) The author is not concerned with the data released by ‘The Network’.
(c) The author believes that the data released by ‘The Network’ is not credible.
(d) The author agrees with the findings of ‘The Network’.
128. Out of the following, which one would be considered a logical course of action
by the author?
i. The government takes measures to keep the prices of the Liquefied Natural Gas
under control,
especially for the middle class.
ii. The government focuses on obsolescence of coal in the market to reduce the
pollution levels.
iii. The policy makers, henceforth, make genuine attempts at providing the needed
solution and do not
take up a policy to please the majority.
(a) Only I (b) Only I and II (c) Only I and III (d) All of the above
129. Which of the following is not likely to be inferred of the passage? Choose the
most appropriate option.
(a) India would reduce its coal capacity in and around its C40 by 20 percent.
(b) The policy makers should make sure that it makes policy decisions based on
credible pieces of
evidence.
(c) India is unlikely to meet its 2030 emissions goal committed at the Paris Pact.
(d) India has made commendable progress in the industry of clean energy.

. Page 32 of 36
130. Consider this statement, ‘A transition to 100 per cent cleaner and greener
energy could generate 18-27
per cent more productivity across India.’ Which among the following roles would
this piece of evidence, if
true, plays towards the arguments of the author?
(a) This piece of evidence would support the author’s arguments.
(b) This piece of evidence would weaken the arguments made by the author.
(c) This piece of evidence is unlikely to play a role towards the author’s
arguments.
(d) This piece of evidence is likely to be an assumption made by the author.
Passage (Q.131–Q.135): The decision by ASEAN to exclude Myanmar’s military junta
from its annual
summit held on October 26-28 is a major setback for the Generals’ attempts to gain
regional legitimacy for
their brutal regime. Ever since it seized power by toppling the democratic
government of Aung San Suu
Kyi in February, the junta has unleashed a reign of terror claiming an estimated
1,000 lives. Ms. Suu Kyi
has been in detention since the coup and is facing absurd charges such as
“illegally owning walkie-talkies”.
Thousands of others were arrested by the military, notorious for its reprisal of
democratic protests in the
past. But this time, the crisis seems much worse. Months after the seizure of
power, the junta, led by Gen.
Min Aung Hlaing, is still struggling to restore order. If in the past the National
League for Democracy (NLD),
Ms. Suu Kyi’s party, had upheld non-violence even in the face of repression, this
time, NLD leaders have
called for a “revolution”. The remnants of the old regime have formed a National
Unity Government, which
claims to be the true representative of Myanmar. In cities, protests slid into
armed fighting between pro#democracy protesters and security personnel, while in
the jungles, anti-junta groups joined hands with
rebels for military training. The situation was so grave that the UN Special Envoy
warned this month that
Myanmar had descended into a civil war.
One of the regional groupings with some leverage over the junta is ASEAN. In April,
Gen. Min Aung Hlaing
was invited to Jakarta for emergency talks with ASEAN members. The bloc asked him
to immediately end
violence, start the reconciliation process and allow a regional special envoy to
meet with all stakeholders,
including Ms. Suu Kyi. None of these requests was met. A special envoy was
appointed as part of the
ASEAN plan, but he was not allowed to meet Ms. Suu Kyi. Regime violence, political
crises and strikes and
counter-attacks by protesters have all pushed Myanmar to the brink of collapse.
Violence might allow them
to hold on to power for now, but that is not sustainable. The ASEAN snub is a
reminder that continuing
violence could cause regional isolation of the regime, which could worsen the
crisis. The international
community should continue to put pressure on the junta and urgently start a
reconciliation process.
131. Which among the following sentences from the passage is the best
representation of the central idea of
the passage?
(a) The ASEAN snub is a reminder that continuing violence could cause regional
isolation of the regime,
which could worsen the crisis.
(b) Most recent reports suggest that the junta has been systematically torturing
political prisoners.
(c) Months after the seizure of power, the junta, led by Gen. Min Aung Hlaing is
still struggling to restore
order.
(d) Ms. Suu Kyi has been in detention since the coup and is facing absurd charges
such as “illegally owning
walkie-talkies”.
132. Which of the following can be validly inferred from the passage?
(a) The stance of the National League for Democracy on the military junta has
remained firm.
(b) The ASEAN does not have sufficient control over Myanmar’s military junta.
(c) The situation in the forests of Myanmar is better than that of the cities.
(d) Aung San Suu Kyi is the sole person who can curb the military junta.

. Page 33 of 36
133. What could be attributed to be the author’s opinion, according to the passage?
I. The snub of the ASEAN is a reminder for the military in Myanmar to leave the
country, otherwise face
consequences.
II. The ASEAN snub is not likely to play any significant role in Myanmar’s military
coup.
III. The military this time has cracked down on its protesters more brutally than
what it did in the past.
IV. The leaders of the National League for Democracy in the second time round of
the coup by military
junta are not up for a non-violence protest.
(a) Only I (b) Only II (c) Only III (d) Only IV
134. Which among the following is a future possibility of the military rule in
Myanmar?
(a) The military rule in Myanmar is likely to accommodate peaceful protests.
(b) The military rule in Myanmar is likely to try to suppress any resistance.
(c) The military rule in Myanmar is likely to give up its rule and let democracy
prevail.
(d) The military rule in Myanmar is likely to treat its political prisoners
humanely.
135. The international community should continue to put pressure on the junta and
urgently start a reconciliation
process.’ The author through the passage does which of the following?
(a) This statement is not likely to play any significant role in the formation of
the passage.
(b) This statement is likely to weaken the arguments made by the author.
(c) This statement is an attempt by the author to make a suggestion.
(d) This statement is an assumption made by the author

mock 20

Passage(Q.1-Q.6): The passage given below is followed by a set of questions. Choose


the most
appropriate answer to each question.
The story of Robinson Crusoe, few people know, is based on a real life incident.
The son of a cobbler,
Alexander Selkirk was a wayward young man, with little respect for authority.
Abject conditions at sea and
the cruelty of the captains made the sailors miserable in those days. It was not
surprising that Alexander
became rebellious and malevolent when he became a sailor.
In 1704, he was Sailing Master on a ship; when it anchored for repairs near the
desolate island of Juan
Fernandez about 650 kilometres west of Chile. They were looking for gold, which
they often got by
plundering other ships. In the days that followed, Alexander hatched a conspiracy.
He instigated the other
sailors to leave the ship and remain on the island. They would declare a mutiny.
Perhaps, Alexander
reasoned, that the Captain would accept their demands if he believed that his men
would refuse to sail
otherwise. Unfortunately for Alexander, the crew played the Judas. The Captain,
getting to know of
Alexander’s part in the planned mutiny, left him behind on the island as he was a
bad influence on the men.
He was provided with a few necessities, among them, a copy of the Bible.
Alexander, marooned on an island populated only by wild cats and goats became adept
at hunting and his
food soon comprised of fish, turtles and meat. He also made clothes with goat skin.
Although a cobbler’s
son, he could not make shoes. Running barefoot after goats had hardened the soles
of his feet. He read
the Bible again and again and slowly took to reading it aloud. He spoke and sang to
the cats and learnt to
milk goats.
Once, during his stay on the island, a Spanish ship anchored near the island and
the crew rowed in.
Alexander was petrified and hid in the thick foliage. In those days Spain and
England were at daggers
drawn. Fortunately, the Spaniards left after an unkempt brief rest.
In February 1709. two English ships sailed in to collect fresh water and shoot
goats. Alexander rushed to
them for succour. He looked strange with his hair. beard and goat skin clothes. His
rescuers understood
him with great difficulty. His speech had changed a great deal. His vocabulary had
shrunk and he had to
grope for words. They did, however, manage to understand his story finally.
Alexander took a job as a sailor on one of the ships and reached London in 1711. He
returned home with
a large fortune. However, he ran out of his fortune in two years and had to return
to sea. Alexander’s
adventure became well-known.
Eight years later. Daniel Defoe gave the story a new shape with many twists,
calling it The Adventures of
Robinson Crusoe. It was now the story of a man who was shipwrecked on an island and
lived alone for an
unbelievable twenty eight years.
1. Match the appropriate word in the list below with a word used in the
comprehension.
(i) Incite (A) instigated
(ii) A person or thing that aids or helps (B) succour
(iii) Dishevelled (C) unkempt
(iv) Mutinous (D) adept
(E) rebellious
(a) (i)-B (ii)-C (iii)-D (iv)-E
(b) (i)-C (ii)—E (iii)-A (iv)—D
(c) (i)-A (ii)-B (iii)-E (iv)-D
(d) (i)—A (ii)-B (iii)—C (iv)—E

. Page 3 of 40
2. What does the author mean by the phrase ‘with little respect for authority?’
(a) Lack of concern for anyone.
(b) Lack of concern for superiors and rules.
(c) Lack of concern for friends and foes.
(d) Lack of discipline.
3. Why did Alexander become reckless and malevolent when he became a sailor?
(a) Because he was inherently undisciplined and wayward as stated in the
comprehension.
(b) Alexander knew that indiscipline wasn’t tolerated, and was seasick due to the
abject conditions and
wanted to go back home.
(c) Because of abject conditions on sea and maltreatment.
(d) A combination of the factors in reasons (1) and (3).
4. Where would you most probably find such a piece of work?
(a) Archaeological paper.
(b) Research paper of a literature scholar.
(c) A book by Daniel Defoe.
(d) A book of fables.
5. What does the author mean that the crew played Judas?
(a) Judas was a keen follower of Christ and the crew, too, were keen followers of
Alexander.
(b) The crew betrayed Alexander, even after being an integral part of the plot.
(c) The analogy with Judas is with reference to betrayal; whereby Judas is an
eponym of betrayal.
(d) The crew, too, were part of the plot along with Alexander and successfully
implemented it.
6. Why was Alexander petrified when the crew of the Spanish ship rowed in?
(a) Because he may be considered as an enemy.
(b) He was totally out of touch with people, and the first sight after a long
interval of 28 years scared him.
(c) He wasn’t sure of the intentions of the Spanish sailors.
(d) He considered the Spaniards to be traitors.
Passage(Q.7-Q.11): Read the following passage carefully and answer the questions
that follow.
Charles Dickens was a British novelist, journalist, editor, illustrator and social
commentator who wrote such
beloved classic novels as Oliver Twist, A Christmas Carol, Nicholas Nickleby, David
Copperfield, A Tale of
Two Cities and Great Expectations. Dickens is remembered as one of the most
important and influential
writers of the 19th century.
Dickens was born Charles John Huffam Dickens on February 7, 1812, in Portsmouth, on
the southern
coast of England. The famed British author was the second of eight children. His
father, John Dickens, was
a naval clerk who dreamed of striking it rich. Charles' mother, Elizabeth Barrow,
aspired to be a teacher
and school director. In 1822, the Dickens family moved to Camden Town, a poor
neighbourhood in London.
By then the family's financial situation had grown dire, as John Dickens had a
dangerous habit of living
beyond the family's means. Eventually, John was sent to prison for debt in 1824,
when Charles was just
12 years old.
Following his father's imprisonment, Dickens was forced to leave school to work at
a boot-blacking factory
alongside the River Thames. At the run-down, rodent-ridden factory, Dickens earned
six shillings a week.
It was the best he could do to help support his family. Looking back on the
experience, Dickens saw it as
the moment he said goodbye to his youthful innocence, stating that he wondered "how
[he] could be so
easily cast away at such a young age." He felt abandoned and betrayed by the adults
who were supposed
to take care of him. These sentiments would later become a recurring theme in his
writing.
Much to his relief, Dickens was permitted to go back to school when his father
received a family inheritance
and used it to pay off his debts. But when Dickens was 15, his education was pulled
out from under him
once again. In 1827, he had to drop out of school and work as an office boy to
contribute to his family's

. Page 4 of 40
income. As it turned out, the job became a launching point for his writing career.
Within a year of being
hired, Dickens began freelance reporting at the law courts of London. Just a few
years later, he was
reporting for two major London newspapers. In 1833, he began submitting sketches to
various magazines
and newspapers under the pseudonym "Boz." In 1836, his clippings were published in
his first book,
Sketches by Boz. In the same year, Dickens started publishing The Posthumous Papers
of the Pickwick
Club. His series, originally written as captions for artist Robert Seymour's
humorous sports-themed
illustrations, took the form of monthly serial instalments. The Posthumous Papers
of the Pickwick Club was
wildly popular with readers. In fact, Dickens' captions were even more popular than
the illustrations they
were meant to accompany.
7. Which of the following is/are true according to the passage?
i) Dickens was one of the most prominent writers of the 19th century.
ii) Dickens could go back to school when his father received family inheritance and
had paid off all his
debts.
iii) Dickens' captions were less popular than the illustrations they were meant to
accompany in 'The
Posthumous Papers of the Pickwick Club'
(a) Both i) and ii) (b) Only ii) (c) Both i) and iii) (d) Only iii)
8. According to the passage, which of the following works is/are not authored by
Charles Dickens?
i. Oliver Twist
ii. A Christmas Carol
iii. David Copperfield
iv. Gulliver's travels
v. A Tale of Two Cities
(a) Only ii) (b) Only iv) (c) Both ii) and iv) (d) Both i) and ii)
9. Which of the following is accredited to Charles Dickens?
(a) He provided a clear portrait of the Victorian-era underclass.
(b) He provided a hazy picture of the Victorian-era underclass.
(c) He provided a hazy picture of the Victorian-era upper-class
(d) He provided a clear portrait of the Victorian-era upper-class.
10. Which of the following became a recurring theme in Dickens' novels?
(a) The feeling of abandonment and faithfulness by adults who are supposed to take
care of children.
(b) The feeling of abandonment and betrayal by adults who are supposed to take care
of children.
(c) The feeling of protection and betrayal by adults who are supposed to take care
of children.
(d) The feeling of providing security and trustworthiness by adults who are
supposed to take care of
children.
11. Which of the following correctly defines the meaning of 'pseudonym' as used in
the passage?
(a) A fictitious name, especially one used by an author.
(b) The real name of an author.
(c) The author's family name.
(d) The name used by predecessors.
Passage(Q.12-Q.16): Read the following passage and answer the questions that
follow.
Oceans cover over 70% of our "blue" planet and are vital to its health. For
instance, carbon moves in and
out of the ocean and can be stored there for thousands of years. Oceans are also a
source of food and
livelihood to millions of people, and to the economies of coastal countries. They
are also the largest
habitable space on the planet and house many different organisms.
But there's a great deal that scientists still don't know about the world's oceans.
The "deep sea" is traditionally defined as below 200m. Usually light from the sun
can't reach these depths
and they are home to organisms that have special adaptations to live here. These
waters are often in

. Page 5 of 40
remote areas and are beyond the reach of all but specialist technologies;
therefore, much of the deep sea
remains underexplored. Exploration is always revealing species that are new to
science. Many of these
could be directly important to humans, for example, some contain specific compounds
that may aid
medicinal advances.
Seychelles and the Maldives are now jointly launching a new deep-sea scientific
mission in the Indian
Ocean that is focused on seamounts - large landforms that rise from the ocean floor
but don't reach the
surface. Because of a limit in equipment and experts, there have not been any
systematic biological
surveys of this region at these depths before. Historically, this type of research
has been near countries
with better access to resources, such as those on the shores of the Atlantic and
Pacific Oceans.
The mission of the "First Descent: Midnight Zone" is to understand what lives in
the water, from the surface
to the seabed. We also want to know how this changes, from waters in Seychelles to
the Maldives. This
information will eventually be available on open access databases, building on the
global knowledge of the
deeper ocean for other scientists and policymakers. We hope that this information
enables countries to
understand how to manage their oceans better.
Our expedition is made up of scientists from many different disciplines who are
coming together to
document biological, physical and chemical parameters. This will provide us with
valuable baseline data
which can also be used to predict life in other sites that we couldn't explore. The
gear we will use ranges
from traditional oceanographic technologies to newly developed equipment. The most
advanced piece of
technology we will use is the full depth submersible, it looks like an underwater
pod that can go to extreme
depths. This enables us to explore the steep slopes of the seamounts. This will
allow us to film and record
transects of the seabed and also take samples of specific organisms of interest
with the manipulator's arm.
We expect to find cold-water coral reefs and gardens of soft corals and sponges -
all home to diverse life.
This expedition will take five weeks, operating 24 hours a day.
12. Which of the following is/are true according to the passage?
i. Oceans are insignificant to the earth's health.
ii. The light from the sun cannot reach below 200m.
iii. Historically, deep-sea research has been confined to countries with better
access to resources such as
those on the shores of the Atlantic and Pacific Oceans.
(a) Only i) (b) Only ii) (c) Only iii) (d) Both ii) and iii)
13. Which of the following is/are false about the oceans according to the passage?
(a) There's a lot that scientists still don't know about the world's oceans.
(b) The largest ocean on Earth is the Pacific Ocean, it covers around 30% of the
Earth's surface.
(c) The oceans accommodate one-third of the total living species on earth.
(d) Both b) and c)
14. According to the passage, what are 'seamounts'?
(a) These are small landforms that rise from the ocean floor and reach the surface.
(b) These are large landforms that rise from the ocean floor but don't reach the
surface.
(c) These are large landforms that rise from the ocean floor and reach the surface.
(d) These are small landforms that rise from the ocean floor and reach the surface,
causing a tsunami.
15. According to the passage, which of the following countries are jointly
launching a deep-sea scientific
mission in the Indian Ocean?
(a) India and Mozambique. (b) Comoros and Madagascar.
(c) Seychelles and the Maldives. (d) Somalia and Maldives.
16. According to the passage, which of the following does not illustrate that
oceans are vital to the earth's
health?
(a) Oceans can store carbon for thousands of years.
(b) Oceans provide food and livelihood to a large number of people.
(c) Oceans are the largest habitable spaces on earth and are home to many different
organisms
(d) The oceans regulate drastic climate changes on the earth.

. Page 6 of 40
Passage(Q.17-Q.21): Read the given passages and answer the questions that follow
each passage.
Flying a helicopter is tricky, especially when hovering. You use your left hand to
raise and lower the
collective-pitch lever (to climb or descend), your right hand to move the cyclic-
pitch joystick (to go forwards,
backwards and sideways) and both feet to work the anti-torque pedals (to point the
nose). At first it all
seems like an impossible dance, but with plenty of practice and careful co-
ordination, it can be mastered.
Flying a drone, by comparison, is a lot easier. Some can be operated with little or
no experience using only
a smartphone app.
One passenger drone undergoing flight tests is the Volocopter VC200. With 18
separate rotors it might
seem to be an ungainly contraption, but its makers. e-volo, a company based in
Karisruhe, Germany, claim
it is more stable than a conventional helicopter It is certainly more
straightforward to fly and can be operated
with just one hand. Twisting the joystick makes the Volocopter turn left or right
and pushing an “up” or
“down” button makes it climb or descend.
The idea behind the Volocopter and similar craft under development is that, like a
drone, they are packed
with sensors, including gyroscopes, accelerometers and magnetometers which,
combined with an on#board computer system, means the aircraft flies largely
autonomously.
The technology is sufficiently advanced that there is nothing to stop passenger
drones taking to the air,
provided they can meet the same safety standards as other light aircraft and are
flown by trained pilots.
Aviation authorities have in the past worked with companies and flying enthusiasts
to develop special
training programmes for other new types of aircraft, such as powered hang-gliders
and microlights.
Some envisage going further still, allowing passenger drones to provide autonomous
air-taxi services. A
bit like using an Uber app to call a cab, a pilotless drone would be summoned to
whisk you away to your
destination.
Unmanned drones can already be flown under existing guidelines. They must be kept
in line of sight, below
400 feet (122 metres) and away from people. To use a drone for commercial purposes,
the operator must
undertake an approved training course. Exemptions to the line-of-sight rule will be
allowed for some flights,
such as those making deliveries.
The attraction of drones is their ease of operation. By turning two of the rotors
clockwise and two
anticlockwise it counters the twisting effects of torque produced by a single-rotor
helicopter. Moreover,
whereas a helicopter needs to vary the pitch of its blades (the angle at which they
attack the air) in order
to manoeuvre, the multiple rotors on a drone have a fixed pitch. The drone instead
manoeuvres by
independently changing the speed of one or more of its rotors under computer
control. As this set-up
requires fewer and less complex moving parts than a helicopter, it makes drones
simpler, cheaper to build
and maintain, and potentially more reliable.
17. As understood from the passage,
I. Passenger drones that provide autonomous air-taxi services may be a thing of the
near future, is the
vision of some.
II. The German company e-volo is also developing a drone that you can sit on like a
motorbike.
III. Flying a helicopter can be mastered with sufficient practice and careful co-
ordination.
(a) Only I (b) I and III (c) I and II (d) II and III
18. Which is the flying machine that appears awkward, clumsy and unnecessarily
complicated?
(a) Helicopters. (b) Powered hang-gliders.
(c) Volocopter VC200. (d) Microlights.
19. Which among the following is NOT TRUE about the VolocopterVC200?
(a) It is manufactured by the German company e-volo.
(b) The joystick helps in sideways movement as well as ascent or descent.
(c) It has sensors, including gyroscopes, accelerometers and magnetometers.
(d) It uses a hybrid power system in place of conventional batteries.

. Page 7 of 40
20. Rules for operating unmanned drones specify that they should be kept in line of
sight at all times. The
drones excluded from this rule are
(a) air-ambulances
(b) those conveying VIPs
(c) those making deliveries
(d) those conveying evacuees during a disaster
21. Compared to helicopters, drones are apparently better by virtue of
I. their ease of operation.
II. less moving complex parts, making it reliable.
Ill. the variable pitch of the rotors.
(a) Only I (b) I and II (c) I and III (d) I, II and III
Passage(Q.22-Q.26): The World Bank regards the $37 million grant as money well
spent on a landmark
scheme that will help bring grid electricity to the 90 per cent of Congolese who
lack it. Most
environmentalists and many in the aid community disagree. They say the dam is a
white elephant and that
its power will mainly benefit urban elites, mining companies and the export market.
What the DRC’s poor
need, they say, is de-centralised, low carbon energy sources such as solar panels.
The disagreement over
Inga 3 is a microcosm of a wider debate about how best to bring electricity to
people who lack it. And the
argument is not just pitting the likes of the World Bank against environmentalists.
The Breakthrough Institute, a California environmental think tank known for its
iconoclastic stance, recently
published a report called Our High-Energy Planet. In it, co-author Alex Trembath
argues that promoting
solar panels and other low-carbon energy technologies is “neo-colonialist, morally
unacceptable and
increasingly irrelevant”. The charge is that solar enthusiasts are sacrificing
economic development for the
poor on the altar of their environmental concerns.
The same debate surfaced at a recent meeting on low-carbon energy, organised by the
University of
Sussex’s Sussex Energy Group at the Royal Society in London, where researchers
presented an analysis
of the spread of domestic solar power in Kenya. Over 300,000 homes are now fitted
with panels, an
achievement that the university’s David Ockwell praised as an example of “pro-poor,
low-carbon
development”.
Or is it? As Ockwell himself remarked later in conversation, a couple of panels on
the roof can charge
phones and run a few lights and a radio but would be no good for anything more
demanding, like boiling a
kettle. Most Kenyans would probably prefer to be hooked up to centralised power,
but the grid only reaches
one-fifth of the country.
In other words, it is not obvious that low-carbon is necessarily pro-poor. And its
widespread adoption might
lock poor communities into a low-carbon future that is also low-energy and low-
income.
Which brings us back to the Breakthrough Institute’s report. It slams environment
groups and aid agencies
who make a fetish of off-grid, low-energy power while giving “big” low-carbon
energy like nuclear and
hydroelectric the thumbs down. The institute says this is both unethical and
counterproductive. It argues
that the world’s poor need a “massive expansion of energy systems” or they will be
condemned to a future
of continued poverty.
The trouble is that neither side is wholly convincing. The Breakthrough report has
little to say about the
implications of its strategy for the climate. The small is- beautiful crowd,
meanwhile, have yet to explain
where their endless expanses of solar panels will take the poor.

. Page 8 of 40
22. Which of the following is the focal point of this passage?
(a) How to sustain economic growth while protecting the climate.
(b) How to provide for the world’s poor without affecting the climate.
(c) How to counter the forces that are trying to propel the world’s poor into an
economically uncertain
future.
(d) How to find the best way to provide electricity to the world’s poor.
23. The Breakthrough institute condemns the attitude of environmental groups
towards the electricity project
unethical because
(a) these groups sacrificing economic development for the poor on the altar of
their environmental
concerns.
(b) these groups favour the rich and discriminate against the poor.
(c) these groups don’t care for the economic future of the globe.
(d) these groups do not try to find a solution that is suitable for the
environment.
24. According to the passage, the environmentalists oppose Inga3 dam because
(a) it is a project that will incur high costs and will benefit certain class and
businesses.
(b) it is a low carbon energy generation project that will not reach the masses.
(c) it is a massive project that will reach ninety percent of the Congo population.
(d) it supports their overall agenda.
25. According to the passage, why does the author say that “neither side is wholly
convincing”?
(a) Because none of them care for the financial elevation of the poor.
(b) Because both sides have good intentions but present utopian ideas of
implementation.
(c) Because both sides are yet to address the lacunae of their respective aims.
(d) Because both sides have contradictory ideas which can’t survive together.
26. In the passage, why does the author report Ockwell’s conversation on low power
generation capacity of
solar panels?
(a) To show that the project, despite the hype, doesn’t prove useful for the Kenyan
people in fulfilling their
minimal energy needs.
(b) To show that the project, though well meaning, doesn’t help the Kenyan people
to fill more demanding
and long-term power solutions.
(c) To show that the project, though ambitious and low-carbon in nature, doesn’t
prove useful for the poor
of the world in a long-term capacity.
(d) To show that the project, though with a noble aim, fails to meet the demanding
requirements of the
Kenyan people.
Passage(Q.27-Q.30): Stoicism was founded in Athens by Zeno of Citium in the early
3rd century BC, but
was famously practiced by the likes of Epictetus, Cato, Seneca and Marcus Aurelius.
The philosophy
asserts that virtue (such as wisdom) is happiness and judgment based on behavior,
rather than words.
That we don’t control and cannot rely on external events, only ourselves and our
responses.
But at the very root of the thinking, there is a very simple, though not easy, way
of living. Take obstacles in
your life and turn them into your advantage, control what you can and accept what
you can’t.
In the words of Epictetus:
“In life our first job is this, to divide and distinguish things into two
categories: externals I cannot control,
but the choices I make with regard to them I do control. Where will I find good and
bad? In me, in my
choices.”

. Page 9 of 40
Amazingly we still have access to these ideas, despite the fact that many of the
greatest Stoics never wrote
anything down for publication. Cato definitely didn’t. Marcus Aurelius never
intended for Meditations to be
anything but personal. Seneca’s letters were, well, letters and Epictetus’ thoughts
come to us by way of a
note-taking student.
And so it was from their example, their actions, we find real philosophy.
Because other than their common study of the philosophy, the Stoics were all men of
action—and I don’t
think this is a coincidence. Marcus Aurelius was emperor of the most powerful
empire in the history of the
world. Cato, the moral example for many philosophers, defended the Roman republic
with Stoic bravery
until his defiant death. Even Epictetus, the lecturer, had no cushy tenure—he was a
former slave.
And this shouldn’t really be that surprising…
The modern day philosopher and writer Nassim Nicholas Taleb defines a Stoic as
someone who
“transforms fear into prudence, pain into transformation, mistakes into initiation
and desire into
undertaking.”
Using this definition as a model we can see that throughout the centuries Stoicism
has been a common
thread though some of history’s great leaders. It has been practiced by Kings,
presidents, artists, writers
and entrepreneurs. Both historical and modern men illustrate Stoicism as a way of
life.
Prussian King, Frederick the Great, was said to ride with the works of the Stoics
in his saddlebags because
they could, in his words, “sustain you in misfortune”.
27. Which of the following can be inferred from ‘Stoicism’?
(a) ‘Stoicism is a philosophy that emphasizes on controlling responses and
behaviour to uncontrollable
external events.
(b) Stoicism is a philosophy that highlights transformation of external events
through one’s actions.
(c) Stoicism refers to seeking wisdom by controlling how we react to external
events.
(d) Stoicism is defined by optimistic celebrations of area without control by
becoming oblivious to external
events.
28. According to the passage, which one of the following is true?
(a) Stoicism is a philosophy which has little to do with destiny and free will.
(b) Stoicism is essentially meant for those nearing the end of life.
(c) Epictetus, Cato, Seneca and Marcus Aurelius though from different walks of
life, practiced Stoicism
(d) According to Nassim Nicholas Taleb, a stoic transforms fear into initiation,
pain into transformation,
mistakes into prudence and desire into undertaking.
29. Stoicism is a philosophy which can be defined as
(a) Relational (b) Analytical (c) Obstructive (d) Metamorphic
30. Which of the following best describes the tone of the passage?
(a) Romantic. (b) Objective (c) Incensed. (d) Effusive
Directions (Q.66 – Q.105): Read the comprehensions carefully and answer the
questions based on it.
Passage (Q.66-Q.70): In a judgment delivered on Wednesday, the Supreme Court
explained the difference
between culpable homicide under Section 304 of the Indian Penal Code and murder
under Section 300
IPC.
The court referred to the following observations made in Pulicherla Nagaraju &
Nagaraja Reddy v State of
Andhra Pradesh which outlined considerations that should weigh with courts, in
discerning whether an act
is punishable as murder, or culpable homicide not amounting to murder:
"Many petty or insignificant matters - plucking of a fruit, straying of cattle,
quarrel of children, utterance of
a rude word or even an objectionable glance, may lead to altercations and group
clashes culminating in
deaths. Usual motives like revenge, greed, jealousy or suspicion may be totally
absent in such cases.
There may be no intention. There may be no premeditation. In fact, there may not
even be criminality. At
the other end of the spectrum, there may be cases of murder where the accused
attempts to avoid the
penalty for murder by attempting to put forth a case that there was no intention to
cause death. It is for the
courts to ensure that the cases of murder punishable under Section 302, are not
converted into offences
punishable under Section 304 Part I/II, or cases of culpable homicide not amounting
to murder are treated
as murder punishable under Section 302. The intention to cause death can be
gathered generally from a
combination of a few or several of the following, among other, circumstances;
(i) nature of the weapon used;
(ii) whether the weapon was carried by the accused or was picked up from the spot;
(iii) whether the blow is aimed at a vital part of the body;
(iv) the amount of force employed in causing injury;
(v) whether the act was in the course of sudden quarrel or sudden fight or free for
all fight;
Source: https://www.livelaw.in/know-the-law/supreme-court-distinction-between-
culpable-homicide#murder-181800
66. Two lawyer friends were once talking on a controversial topic. One of them,
Amulya made a religious point,
which hurt the sentiments of the other one, Mulyawaan. Mulyawaan got angry. He
found a nail cutter
nearby. He picked the nailcutter and threw at Amulya. The nail cutter hit the knees
of Amulya. Amulya had
undergone knee replacement surgery recently. His stitches opened and due to
overflow of blood, he died
instantly. Amulya did not know about the surgery. Decide.
(a) Mulyawaan is liable for murder under section 302.
(b) Mulyawaan is liable for culpable homicide under section 304 for culpable
homicide not amounting to
murder.
(c) Mulyawaan is liable for culpable homicide under section 300.
(d) Mulyawaan is not liable for culpable homicide since knee is not a vital organ
and he was not aware of
the surgery.
67. Two lawyer friends were once talking on a controversial topic. One of them,
Amulya hated Mulyawaan for
his views ever since they had known each other. Amulya once made a religious point,
which hurt the
sentiments of the other one, Mulyawaan. Mulyawaan got angry. Amulya after knowing
about the knee injury
of Mulyawaan pulled out a nail cutter from his pocket and threw it at his stitches,
which opened and Amulya
died from overflow of blood. Decide.
(a) Mulyawaan is liable for culpable homicide amounting to murder under Section
302.
(b) Mulyawaan is liable for culpable homicide amounting to murder under Section
304.
(c) Mulyawaan is liable for culpable homicide amounting to murder under Section
300.
(d) Mulyawaan is not liable for murder as it is grave and sudden provocation..

. Page 16 of 40
68. Class 11th students of a school were in the sports ground during the sports
period. Sohit and 12 other
students were playing basketball and were blaming Vinesh for cheating. As a result
a fight broke up
between Sohit and Vishesh. One of the student Abhay, who was not allowed to play
with them since he
couldn’t be divided in the team, hopped into the fight and hit a stone on Vishesh’s
head, who started to
bleed and died. Everybody blamed Sohit. Decide.
(a) Sohit is liable for culpable homicide under Section 304 as he shouldn’t have
pointed out the cheating
and started the fight in the first place.
(b) Abhay is liable for culpable homicide under Section 304 as he shouldn’t have
aimlessly entered
between people who were fighting.
(c) Nobody is liable since they were 11th class students and each one of them is
minor.
(d) Abhay is liable under Section 302 since he intentionally hit on Vishesh’s head.
69. A and B were good friends, who had an altercation one day. They used to throw
things at one another
playfully for a long time. During this altercation, A threw a knife towards B
aiming at his head, which rather
hit his chest, with mild force. The knife pierced through his heart and B died on
the spot. Decide.
(a) A is liable for culpable homicide amounting to murder under Section 302.
(b) A is liable for culpable homicide not amounting to murder under Section 304.
(c) A is liable for murder under Section 300.
(d) A is not liable, since he was friends with B.
70. Which of the following is correct according to the given passage?
(a) It is for the courts to ensure that the cases of murder punishable under
Section 304, are not
converted into offences punishable under Section 302, or cases of culpable homicide
not amounting
to murder are treated as murder punishable under Section 302.
(b) It is for the courts to ensure that the cases of culpable homicide punishable
under Section 302, are
not converted into offences punishable under Section 304, or cases of murder not
amounting to
culpable homicide are treated as murder punishable under Section 302.
(c) It is for the courts to ensure that the cases of murder punishable under
Section 302, are not
converted into offences punishable under Section 304, or cases of culpable homicide
not amounting
to murder are treated as murder punishable under Section 302.
(d) It is for the courts to ensure that the cases of murder punishable under
Section 300, are not
converted into offences punishable under Section 304, or cases of culpable homicide
not amounting
to murder are treated as murder punishable under Section 304.
Passage (Q.71-Q.75): In Thwaha Fasal vs Union of India, the Supreme Court has acted
in its introspective
jurisdiction and deconstructed the provisions of the Unlawful Activities
(Prevention) Act (UAPA) with a great
sense of legal realism. This paves the way for a formidable judicial authority
against blatant misuse of this
draconian law.
In this case from Kerala, there are three accused. The police registered the case
and later the investigation
was handed over to the National Investigation Agency (NIA). During the
investigation, some materials
containing radical literature were found, which included a book on caste issues in
India and a translation
of the dissent notes written by Rosa Luxemburg to Lenin. There were also leaflets
that were allegedly
related to Maoist organizations.
Thus, the provisions of the UAPA were invoked. Against the first accused, Allen
Shuaib, offences under
Sections 38 and 39 of the UAPA and 120B of the Indian Penal Code (IPC) were
alleged. Section 38 deals
with “offence relating to membership of a terrorist organization” and Section 39
deals with “offence relating
to support given to a terrorist organization.” Section 120B of the IPC is the penal
provision on punishment
for criminal conspiracy. Against the second accused, Thwaha Fasal, over and above
these charges,
Section 13 of the UAPA was alleged — which is the provision about punishment for
unlawful activities.
The Supreme Court, after a comprehensive examination, upheld the trial judge’s
finding that the materials,
prima facie, do not show any “intention on the part of both the accused to further
the activities of the terrorist
organization”. It found fault with the High Court for not venturing to record,
prima facie, findings regarding
charges against Thwaha, whose bail was set aside by the High Court.

. Page 17 of 40
The Supreme Court was emphatic and liberal when it said that mere association with
a terrorist organization
is not sufficient to attract the offences alleged. Unless and until the association
and the support were “with
intention of furthering the activities of a terrorist organization”, offence under
Section 38 or Section 39 is
not made out, said the Court. Mere possession of documents or books by the accused
at a formative young
age, or even their fascination for an ideology, does not ipso facto or ipso jure
make out an offence, the
Court ruled.
The judgment also exposes the hypocrisy of the law of, the UAPA. Unlike the
Criminal Procedure Code,
the UAPA, by virtue of the proviso to Section 43D(2), permits keeping a person in
prison for up to 180 days,
without even filing a charge sheet. Instead of presumption of innocence, the UAPA
holds presumption of
guilt of the accused. Section 43E of the Act expressly says about “presumption as
to the offences”.
According to Section 43D(5), jail is the rule and bail is often not even an
exception though when health of
a person is in question it should be given utmost importance to consider the bail
as matter of right. The
Court, in Thwaha Fasal, refused to construct this Section in a narrow and
restrictive sense. This analysis
has to some extent, liberalised an otherwise illiberal bail clause.
Source: A new jurisprudence for political prisoners, November 8th, 2021, The Hindu,
71. Keshvan was arrested by the Kullam Town Police Station in the state of Mastaka
in connection with the
investigation of a crime, wherein he was alleged to have the possession of
pamphlets narrating "armed
revolution and violence against Sindhian constituency", supporting the banned
organization namely,
Communist Party of Sindhia (Maoist) (hereinafter the CPS (M)). It was also found
that he had frequent
contacts with one Sheshvan, who is the member of the urban action team of the CPS
(Maoist) through
WhatsApp. It was also found in the investigation that he had close acquaintance
with the 'Janakaya
Manushyata Prasthanam', which is recognised, identified and scheduled as a
terrorist organization in the
law.
You are the legal representative on behalf of Keshvan; considering the fact that
laws in the Union of Sindhia
are pari materia to the laws of the Union of India, provide the appropriate defence
in favour of your client
as per the legal information given in the passage.
(a) The arrest is not valid because there was no intention on the part of the
accused to further the activities
of the terrorist organization.
(b) The arrest is not valid because merely having the possession of the pamphlets
does not hold him liable
under the provisions of the alleged law.
(c) The arrest is not valid because merely having contact with the member of the
organization does not
show culpability.
(d) The arrest is not valid because the accused has not be informed about the
charges imposed on him,
which is a Fundamental Right of an arrestee.
72. Considering the facts from the previous question, supposedly, it was found that
Keshvan has been
appointed as the regional leader of the CPS (M) in Kullam Town, he has been
organizing rallies and actively
recruiting the students in the organization. Whether he can seek the defence
against the charges imposed
against him under the UAPA? Answer the question as per the legal information given
in the passage.
(a) Yes, he can seek the defence because the alleged act does not show his
intention to indulge in any of
the terrorist activities.
(b) Yes, he can seek the defence because mere appointment as a regional head of the
organization does
not denote his active participation in the any of the terrorist activity of the
organization.
(c) No, he cannot take the defence because his appointment, and his act of
recruiting the students for the
organization denotes his intention to commit terrorist activities
(d) No, he cannot take the defence because the alleged act of recruiting the
students for the organization
denotes his intention to further the activities of the organization

. Page 18 of 40
73. Manzar Sheikh has been arrested by the Akaba Police Station in the state of
Udhyan under the provisions
of the Unlawful Activities Prevention Act, and he was arrested on March 25th, 2021.
An order dated August
21st, 2021 passed by the Roster Judge, Trial Court, whereby the Court on an
application submitted by the
learned Additional Public Prosecutor for extending the period of investigation for
further 90 days was
accepted. An application under Section 167 (2) has been filed seeking statutory
bail on the completion of
180 days. Decide the matter as per the legal information given in the passage
extending the period of
investigation with arrest in valid or not?
(a) The arrest is valid under the provisions of the UAPA because the arrest can be
done for 180 days
without filing the charge-sheet before the Court.
(b) The arrest is valid under the provisions of the UAPA because the Trial Court is
empowered to allow the
detention of the person for an indefinite time under the Act.
(c) The arrest is not valid under the provisions of the UAPA because the arrest can
be done for 180 days
without filing the charge-sheet before the Court.
(d) The arrest is not valid because the person has not been given the opportunity
to defend the extension,
which is against the principles of Natural Justice.
74. Dr. Sittavan, a professor of Political Science in the Jhalawar University in
the state of Dholvira. He is in
affiliation with the organization named Yalgar Parishad, which is a recognized as
unlawful organization
under the Unlawful Activities Prevention Act (hereinafter the UAPA) of the Union of
Rakhigarhi. He was
found promoting the objectives of the organization among his students in the
University. He was also found
to facilitate the arms deal for the organization. He was charged under Section 13,
16, 17, 18, 18-B, 20, 38,
39 and 40 of the Act. While he was in custody, certain health issues were raised in
him due to his age, and
emergency medical care was needed, a writ petition has been filed before the High
Court of Dholavira by
his family to seek the release of him on the medical grounds.
Decide the case as per the legal information given in the passage.
(a) The petition is valid because the arrest under the provisions of the UAPA was
not valid considering the
health condition of the person.
(b) The petition is valid because the arrest under the provisions of the UAPA was
not valid because the
alleged act of the arrestee does not denote his intention to further the terrorist
activities.
(c) The petition is not valid because the arrest under the provisions of the UAPA
was valid because the
arrestee was affiliated with the organization, and has done certain acts actively
for the furtherance of
the organization.
(d) The petitioner is valid because the person so arrested has though indulged in
the terrorist activities, is
in need of urgent medical assistance.
75. Napoleon and the members of his gangs were in the target of the police of the
state of Ostrich in the Union
of Narnia from past some years for their alleged activities of transporting young
people from Narnian
territory to the foreign territory, and the police administration has got the
secret information that the same
was happening as a recruitment process for the terrorist organization. Once they
were caught red-handed
by the police authority, and were charged under the provisions of the Unlawful
Activities Prevention Act.
During the investigation, they have admitted the act of transporting young people,
but they have stated that
they did not know about the purpose of the transportation, as they were merely
contacted to transport the
human for a decided monetary commission. Considering the fact that the laws of the
Union of Narnia is
pari materia to the laws of the Union of India, decide the case as per the legal
information given in the
passage.
(a) Their arrest under the UAPA is valid as their act was alleged to be in
furtherance for the terrorist
activities.
(b) Their arrest under the UAPA is valid because their act of transporting the
young people denotes their
active participation in the illegal activities.
(c) Their arrest under the UAPA is not valid because their act of transporting the
young people does not
show their intention to work for the furtherance of the terrorist organization.
(d) Their arrest under the UAPA is not valid because as per the law the active
association with the terrorist
organization is essential, which the accused did not have.

. Page 19 of 40
Passage (Q.76-Q.80): The Supreme Court on Friday said any attempt of booth
capturing and bogus voting
should be dealt with an iron fist because it ultimately affected the rule of law
and democracy.
"The essence of the electoral system should be to ensure freedom of vote to
exercise their free choice.
Nobody can be permitted to dilute the right to a free and fair election," a bench
of Justices D Y Chandrachud
and M R Shah said.
It said the trial court has rightly convicted all the accused were the members of
the unlawful assembly in
prosecution of the common object, namely, “to snatch the voters list and to cast
bogus voting”.
Taking a grim view of election-related offences, the court referred to its previous
judgment in the case of
'People’s Union for Civil Liberties Vs Union of India' (2013) which stated that
freedom of voting is a part of
the freedom of expression.
"Secrecy of casting vote is necessary for strengthening democracy and to ensure
that a voter casts his
vote without any fear of being victimized. Democracy and free elections are a part
of the basic structure of
the Constitution. The election is a mechanism, which ultimately represents the will
of the people," the bench
said, citing the judgment.
Source: Can't permit anybody to dilute free and fair polls: Supreme Court,
76. Malagaon Desham Party, the ruling party in the state of Sauhadra has enhanced
the routes of the buses
for the voters from the far-fledged area to bring them to the polling booth.
Telaniram, a member of the
opposition party filed a complaint stating this exercise is to be a violation of
free and fair election. Decide
the case as per the given legal information in the passage.
(a) The alleged act is not violative of free and fair election because it is not
infringing the secrecy of votes.
(b) The alleged act is violative of the free and fair election because it will
influence the voters towards the
ruling party.
(c) The alleged act is not violative of the free and fair election because it is
duty on the part of the
government of conduct election with maximum participation of voters.
(d) The alleged act is violative of the free and fair election because it is
violative of model code of conduct
during the election process.
77. In the constituency of Bhanupur, only 34% of voters have arrived to cast their
votes till 2:00 pm on the
voting day. Sambha, a worker of the Samanjasya Samarta Party, took the voters’ list
and started the
absentees forcefully to the polling station to make them cast their votes. Jaydev,
a reporter from Jagrit
Duniya filed a complaint against the Samanjasya Samarta Party for booth capturing
and unlawful assembly
by the supporters of the party. Whether anyone will be liable for violating the
free and fair election?
(a) The alleged act is violative of the free and fair election because the member
of the party was indulged
in bogus voting, and hence, the party will also be liable.
(b) The alleged act is not violative of free and fair election because it is
ensuing that the people cast their
votes in larger number.
(c) The alleged act is violative of the free and fair election because , and hence,
the party will also be liable.
(d) A political party cannot be held liable for the act of its worker because
Sambha was neither authorized
nor instructed to do so.
78. In the state of Tintina, where most of the populations are farmers, the
elections were held on July 10th,
2021. During the election, a group of farmers came to cast their vote but instead
of doing so, they started
raising slogans against the recent farms’ bills passed by the Union government of
Vimana. They sat on the
peaceful protest against the bills outside the polling booth. Considering the fact
that the law of the Union
of Vimana is pari materia to that of the Union of India, decide the case as per the
given information in the
passage that whether farmers are liable for violation of free and fair election.
(a) The farmers will be liable because they are infringing the right to free and
fair election by way of putting
the disruptions in the election.
(b) The farmers will not be liable because it is their fundamental rights to
freedom of speech and expression
to raise their voice against the law, and while doing so, they are not infringing
the others’ right to vote.
(c) The farmers will be liable because they have captured the booth, and were
influencing the voters.

. Page 20 of 40
(d) The farmers were not infringing the secrecy of casting votes, and were neither
victimizing the votes in
the same. Hence will not be liable.
79. Mriganka Sonewal, a candidate from the Rajnitik Alinkrit Party in the by-
election for the constituency of
Vibhanipur, went for the door-to-door campaign. During her campaign, she herself
handed over the voters’
slips at their door. Jorawar, a public-spirited person filed a petition against her
for violating the norms of a
free and fair election and secrecy of election. Decide the case as per the legal
information given in the
passage.
(a) She will be liable as her alleged act would defeat the purpose of the free and
fair election, because the
secrecy of the votes, and privacy of the voters are at stake.
(b) She will be liable as her alleged act infringes the free and fair election
because it will influence the
election process in an unreasonable manner.
(c) She will not be liable as her alleged act will not infringe the free and fair
election because it is common
practice during the election campaign.
(d) She will not be liable as her alleged act will not infringe the free and fair
election because her act does
not attract the instances of booth capturing, or bogus voting.
80. Assertion (A): Right to vote is a fundamental right of persons enshrined under
Article 19 of the Constitution.
Reasoning (R): While a statute can decide the modalities of voting, the act of
voting is a part under Article
19(1) (a) of the Constitution.
Choose the correct option:
(a) Both A and R are true, R is the correct explanation of A
(b) Both A and R are true, R is not the correct explanation of A
(c) A is correct and R are incorrect
(d) A is incorrect but R is correct
Passage (Q.81-Q.85): Perjury petition can't be deferred or delayed - Karnataka HC
The Karnataka High Court recently observed that consideration of complaints
regarding perjury should not
be deferred or delayed by courts.
The definition of perjury as considered in the Indian laws is, “Whoever
intentionally gives false evidence or
tampers with evidence in any stage of a judicial proceeding, or fabricates false
evidence for the purpose of
being used in any stage of a judicial proceeding, shall be punished with
imprisonment of either description
for a term which may extend to seven years, and shall also be liable to fine; and
whoever intentionally gives
or fabricates false evidence or tampers witness in any other case, shall be
punished with imprisonment of
either description for a term which may extend to three years, and shall also be
liable to fine.”
Justice Krishna S Dixit said "Act of perjury is treated as a heinous offence in all
civilized societies;
consideration of complaints with regard to the same cannot be deferred or delayed;
otherwise there is all
possibility of the fountain of justice being polluted."
"... The evil of perjury has assumed alarming proportions in cases depending on
oral evidence and in order
to deal with the menace effectively, it is desirable for the Courts to use the
provision more effectively and
frequently, than it is presently done..."
The court said, "The inner voice of this decision appears to have fallen on the
deaf ears of the learned
Judge of the court below." Further, it said "The learned trial Judge ought to have
considered petitioner's
subject application with due seriousness and at the earliest point of time, there
being no justification for
deferring its consideration since it touched purity of judicial proceedings."
81. A wife, Samai, filed a perjury case against her husband, Xavier in Delhi High
Court. Xavier had gifted his
child Sam an iPhone and gave him treats every now and then in order to ask him to
testify in the court that
his mother, as a matter of husband, beat him. He testified for the same when the
case was filed by Xavier
against Samai. Samai then filed a case of perjury against Zavier. The petition was
delayed for three months
before the matter could come up for hearing. Decide.
(a) The husband can be subject to ten years of imprisonment.
(b) No fine would be collected from Xavier on account of perjury.

. Page 21 of 40
(c) In light of the facts mentioned, nothing can be done since the case is in
Delhi.
(d) It is not advisable to provide an iPhone to the child.
82. A murdered B in Vijapura, Karnatak and a murder trial was going on against him
on this account. The same
murder was witnessed by C. D, a police officer threatened C to not testify against
A otherwise he will be
punished. The same was found out and a perjury complaint was filed against D. The
trial court deferred
the perjury case until the murder case is decided. Decide.
(a) This is a wrong committed on the part of the trial court.
(b) It is logical for the trial court to defer the perjury petition until the
decision of the murder trial.
(c) D can be subjected to seven years of imprisonment.
(d) C shall be punished for perjury.
83. A person A, filed a case against C of corporate tax theft of Rs. 1.35 crores. C
had made fabricated
documents through P which proved that the said tax was paid to the government, and
the documents
passed the verification test as well. P’s name occurred during the trial and he was
called for testifying,
and on the basis of his testification, a perjury case was filed against P. At this
time, an urgent petition of a
superstar’s kid came up. The Court deferred the case for 14 days, then for 28 days
and then for 2
months. Decide.
(a) Since the superstar’s kid’s case would be followed by the whole nation, it is
necessary for the court to
devote more resources on that so as to keep the faith of the general public in
judiciary upright.
(b) Life imprisonment should be awarded to people like P, since they ruin the
country by theft of corporate
tax, fabricating evidence and corruption.
(c) The perjury petition should not have been deferred, otherwise there was all
possibility of the fountain
of justice being polluted.
(d) Offence of perjury is established against P.
84. A person A, filed a case against C for corporate tax theft of Rs. 1.35 crores.
C had made fabricated
documents through P which proved that the said tax was paid to the government, and
the documents
passed the verification test as well. P’s name occurred during the trial and a few
pages of his personal
diary could be accessed. On the basis of the same, his computer was accessed and
checked and it was
found that he had created a software just to fabricate documents pertaining to tax
theft. A’s document of
fabrication of non-payment of tax could also be accessed. The case was delayed by
the court for Passover.
Decide.
(a) Since there has been no requirement of oral evidence, the case does not pertain
to perjury.
(b) Seven years of rigorous imprisonment would be granted to P.
(c) Deferring the case is a wrong committed on the part of the court.
(d) P will be liable for the three years of rigorous imprisonment.
85. Which of the following is correct?
(a) Justice Krishna Dixit said, “Act of perjury is treated as a heinous offence in
all civilised societies” and
can be punished with seven years of rigorous imprisonment and fine.
(b) Justice Krishna Dixit said, “Act of perjury is treated as a heinous offence in
all civilised societies” and
can be punished with seven years of imprisonment or fine.
(c) Justice Krishna Aiyyer said, “Act of perjury is treated as a heinous offence in
all civilised societies” and
can be punished with seven years of imprisonment and fine.
(d) Justice Krishna Dixit said, “Act of perjury is treated as a heinous offence in
all civilised societies” and
can be punished with seven years of imprisonment and fine.

. Page 22 of 40
Passage (Q.86-Q.90): Setting aside the conviction order passed against a Husband
for abetting the suicide
of his wife, the Allahabad High Court last week held that separating wife from his
own life could not be a
reason which could come under the category of the abetment. At the outset, the
Court noted that the trial
court had found that the appellant used to torture the deceased mentally and that
he had created such a
situation before the deceased by separating her from his life that she was not left
with any other option but
to commit suicide and therefore, the Trial Court held him guilty of abetting the
suicide of wife.
However, the High Court opined that this finding of the trial court was not in
consonance with the settled
position of law regarding abetment, as the Court noted that before a person may be
said to have abetted
the commission of suicide, he must have played an active role by an act of
instigation or by doing a certain
act to facilitate the commission of suicide.
Significantly, the court noted that in order to bring the case under Section 306
IPC abetment of suicide, it
must have been proved that the husband instigated the deceased to commit suicide or
that he engaged
with one or more persons in any conspiracy to abet the deceased to commit suicide
or that he intentionally
aided by any act for abetting her to commit suicide.
86. A and B were batchmates in XYZ public school. A was new admission there and was
a little naive, when
other students realised this about A, they started bullying him. B was also one of
the bullies but did not
bully A as much as other students, but whenever he did, it always made A break into
tears. On the last day
of the school before summer vacation, most of the students except B bullied A.
Infuriated by the same, A
rushed to his home wrote a suicide note wherein he stated that B and other students
bullied him, and then
he cut his wrist to bleed to death. The police arrested B on the charges of
abetment to suicide, determine
his liability.
(a) B is not liable because he was not the one who bullied him on the last day.
(b) B is liable because he had been constantly bullying A in the past.
(c) B is not liable because he was quite sensitive towards A and did not bully him
like other students.
(d) B is liable because he always made A cry out of frustration.
87. Suppose in the previous question, after the summer vacation everyone stopped
bullying A. soon the
midterm exams approached and A fails in the exams since he was not in the mental
space to study due to
constant bullying in the past. A became morose with his result and jumped off the
school building. The
police arrested the school principal for abetting the suicide of A. Determine the
principal’s liability.
(a) Not the principal but B and A’s other batchmates are liable for abetting A’s
suicide.
(b) B and the other students are not liable because the bulling had stopped after
the summer vacations.
(c) The principal is liable because he failed A, without being considerate towards
his mental health which
directly led to A committing suicide.
(d) The principle is not liable because he did not have any active role in A’s
suicide.
88. X, Y & Z 10th class student played in the terrace of their apartment, which was
of mere one floor. One day
they decided to invent a new game ‘Jump master’ where each participant would have
to jump off the terrace
and whoever has the best landing will win the round. Accordingly, they all prepared
to jump, X jumped and
the other two gave him 7 points, Y jumped and the other two gave him 8 points. Z
was quite afraid that he
might die but X & Y motivated him and he jumped; but made a crash landing, stuck
his head in a rock and
died immediately. The police arrests X & Y for abetting Z’s suicide, determine
their liability
(a) X & Y are not liable because they did not have any intent to abet Z’s suicide.
(b) X & Y are liable because they did motivate Z to jump; with the active role in
Z’s suicide, they satisfied
the conditions for abetment.
(c) X & Y are not liable because the conditions of abetment to suicide is not met.
(d) X & Y are not liable because volenti non fit injuria will be applicable in this
case.
. Page 23 of 40
89. Suppose in the previous question, Z had suicidal tendencies since the past few
months known to X & Y,
and he saw in this game an opportunity to jump and end his life, but he was
hesitant to jump, fearing that
he might survive. X and Y motivate him by saying, “jumping is easier than the fact
that our lives are full of
despair and with each passing second we come closer to our ultimate destiny, i.e.,
death”. Z gets motivated,
jumps and dies and X & Y are held liable for abetting his suicide. Would this
change in fact, change your
answer to the previous question?
(a) Yes, X & Y are not liable.
(b) No, X & Y are not liable.
(c) Yes, X & Y are liable.
(d) No, X & Y are liable.
90. Mina and Tina are supermodels and best friends. One day they had a fight
regarding a new contract of a
reputed company, in heat of the moment Mina punches Tina on her face, the latter
falls on the floor injuring
and eventually deforming her face. Tina loses all her business contracts, morose by
the same she jumps
of her apartment and dies. Determine the liability of Mina in the light of the
given fact and passage.
(a) Mina is liable for grievous hurt because she deformed the face of Tina.
(b) Mina is liable for abetment to suicide because it is due to her that Tina
committed suicide.
(c) Mina is not liable..
(d) Mina is only liable for puncing Tina, thereby causing injury and nothing else.
Passage (Q.91-Q.95): The Supreme Court in an 11 judge bench reiterated that the
scope of judicial review
on the quantum of punishment imposed in disciplinary proceedings is limited.
"Even in cases where the punishment imposed by the disciplinary authority is found
to be shocking to the
conscience of the Court, normally the disciplinary authority or the appellate
authority should be directed to
reconsider the question of imposition of penalty",
The court added that it is only in rare and exceptional cases where the court might
to shorten the litigation
may think of substituting its own view as to the quantum of punishment in place of
punishment awarded by
the competent authority that too after assigning cogent reasons.
The court also noticed the principles set out in Lucknow Kshetriya Gramin Bank and
Another vs. Rajendra
Singh in this regard:
1. When charge(s) of misconduct is proved in an enquiry the quantum of punishment
to be imposed in a
particular case is essentially the domain of the departmental authorities.
2. The courts cannot assume the function of disciplinary/departmental authorities
and to decide the
quantum of punishment and nature of penalty to be awarded, as this function is
exclusively within the
jurisdiction of the competent authority.
3. Limited judicial review is available to interfere with the punishment imposed by
the disciplinary authority,
only in cases where such penalty is found to be shocking to the conscience of the
court.
4. Even in such a case when the punishment is set aside as shockingly
disproportionate to the nature of
charges framed against the delinquent employee, the appropriate course of action is
to remit the matter
back to the disciplinary authority or the appellate authority with direction to
pass appropriate order of
penalty. The court by itself cannot mandate as to what should be the penalty in
such a case.
Source: https://www.livelaw.in/top-stories/supreme-court-judicial-review-
disciplinary-proceedings#punishment-union-of-india-vs-ex-constable-ram-karan-185409
91. Delhi Police constituted an investigation-cum-punishment bench with three
senior police officers to conduct
an enquiry of Mr. Himmat Singh. The bench found Mr. Himmat guilty of misusing the
resources of the police
for 2 days and suspended him from his post, while suggesting a fine of Rs. 4 lakhs
besides recommending
a termination letter against him. On further appeal in the Supreme Court in full
bench, said that the
punishment is utterly disproportionate. The Court reduced Mr. Himmat’s punishment
to suspension for a
month and the fine of Rs. 10,000. Decide.
(a) The Supreme Court’s action is totally correct as the investigation bench’s
punishment was utterly
disproportionate.

. Page 24 of 40
(b) The Supreme Court’s action is totally correct as it is Supreme Court so can
reverse its judgment and
was beyond the power of investigation bench to give punishment.
(c) The Supreme Court’s action to reduce the punishment is not correct as it is
beyond its powers to reduce
the punishment by itself.
(d) The Supreme Court’s action to reduce the punishment is not correct as it should
rather have directed
the investigation bench to reconsider the sentence rather than reducing the
punishment by itself.
92. In a school in Shimla, there was a 13 year old student who was always involved
in mischievous activities.
He was involved in lot of violent fights, never completed his homework on time, and
even vandalised
school’s property a few times. The school fined him Rs. 20,000 and gave him a
suspension for a month
after enquiry of his actions, and neither allowed him to study there, nor provided
him with the School
Leaving Certificate (SLC), so he could not study in any other school as well. He
appealed in the Supreme
Court, which ordered the school to provide him with the SLC and reduce the fine to
Rs. 10,000. Decide.
(a) The Supreme Court’s action was correct as the school cannot withhold a
student’s SLC and bar him
from studying anywhere as it is against the Fundamental Right to Education.
(b) The Supreme Court’s action was incorrect as it should rather have directed the
school to reconsider
the punishment rather than altering the punishment itself.
(c) The Supreme Court can alter the punishment given by a school as there has been
no enquiry for the
student. Only an analysis of his doings has been conducted.
(d) None of the above.
93. Mumbai Police conducted routine annual enquiry against all police officers. In
the course of the enquiry,
three police officers were suspended for a month after the enquiry. These police
officers had not appeared
for duty on time multiple times during the past year. One of these police officers
requested a judicial review
of the same. Decide.
(a) The judicial review would stand since the enquiry is routine and suspending
only three police officers
when everyone in the country is late at work shatters the conscience of the court.
(b) The judicial review would be conducted since one month suspension for being
late is not a proportionate
punishment.
(c) The judicial review would not be conducted as only one of the police officers
requested the judicial
review and not the other two.
(d) Since nothing in this shatters the conscience of the court, the limited
judicial review cannot be invoked
for the purpose of this case.
94. A police officer arrested the son of a superstar in a drugs case. Later, the
superstar’s son is released on
bail and an enquiry is set up against the police officer to gather whether or not
he had ulterior motives
during/for the arrest. No ulterior motives as such were found but the police
officer was suspended from the
case. The court ordered to put him back on the case, unless and until any ulterior
motive is concretely
proved.
(a) The court’s action is correct since no ulterior motive whatsoever has been
found and the police officer
has still been suspended from the case.
(b) The court’s action is incorrect since the court should rather have directed the
enquiring authority to
reconsider their stance on the suspension of the police officer.
(c) The court’s action is correct as it is very necessary for the courts to
maintain a check over such
preposterous suspensions of the police officers by taking up suo moto cases.
(d) None of the above.

. Page 25 of 40
95. Which of the following is incorrect according to the given passage?
(a) As per the principles set in Lucknow Kshetriya Gramin Singh and Another vs.
Rajendra Bank, when
charges of misconduct are proved in an enquiry the quantum of punishment to be
imposed in a
particular case is essentially the domain of the judicial review.
(b) As per the principles set in London Kshetriya Gramin Bank and Another vs.
Rajendra Singh, when
charges of misconduct is proved in an enquiry the quantum of punishment to be
imposed in a particular
case is essentially the domain of the by-laws.
(c) As per the principles set in Lucknow Kshetriya Gramin Bank and Another vs.
Rajendra Singh, when
charges of misconduct is proved in an enquiry the quantum of punishment to be
imposed in a particular
case is essentially the domain of the departmental authorities.
(d) As per the principles set in Lucknow Gramin Bank and Another vs. Rajendra
Singh, when charges of
misconduct is proved in an enquiry the quantum of punishment to be imposed in a
particular case is
essentially the domain of the judicial authorities.
Passage (Q.96-Q.99): The Supreme Court has held that a person who is entitled to
the benefit of
reservation in either of the State of Bihar or State of Jharkhand will not be
entitled to claim benefit of
reservation simultaneously in both the successor States.
Allowing such simultaneous claim will defeat the mandate of Articles 341(1) and
342(1) of the Constitution,
the Court observed. Those who are members of the reserved category and are resident
of the successor
State of Bihar, while participating in open selection in State of Jharkhand shall
be treated to be migrants
and it will be open to them to participate in general category without claiming the
benefit of reservation and
vice-versa.
The bench said that the collective readings of the provisions of the Bihar
Reorganisation Act, 2000, makes
it apparent that such persons whose place of domicile on or before the appointed
day (November 15, 2000)
was of Bihar now falling within the districts/regions which form a successor State,
that is Jharkhand under
Section 3 of the Act, 2000 became ordinary resident of the State of Jharkhand.
At the same time, so far as the employees who were in public employment in Bihar on
or before November
15, 2000, apart from those who have domicile of either of the district which became
part of Jharkhand,
such employees who have exercised their option to serve in Jharkhand their existing
service conditions
shall stands protected by virtue of Section 73 of the Act, 2000 and vice-versa.
https://enalsar.informaticsglobal.com:2278/top-stories/reservation-category-person-
cant-claim-quota#benefits-simultaneously-in-two-successor-states-supreme-court-
180031LiveLaw dated 22/08/2021
96. Aman & Aarti are happily married couples living in Patna. They went for winter
vacation to Dhanbad, where
Aarti resided with her parents before marrying Aman. On the midnight of 14th
November2000, Aarti gave
birth to a baby boy who was named Manoj. Soon after the birth of Manoj, Aman &
Aarti go back to Patna..
Years later, Manoj is appearing for the banking exams at Bihar and wants to claim
reservation benefits, is
he entitled for the same?
(a) No, since Manoj was born in the successor state of Jharkhand, he will be
entitled to claim reservation
benefits from the state of Jharkhand only.
(b) Yes, although Manoj was born in the successor state of Jharkhand, but by the
virtue of him living in
Bihar all his life will entitle him to claim reservation benefits in Bihar.
(c) No, Manoj is entitled to get reservation benefits only from the state of
Jharkhand, if he is given the
reservation benefit in Bihar as well it will defeat the mandate of Articles 341(1)
and 342(1) of the
Constitution.
(d) Yes, since there is an ambiguity around the domicile of Manoj, he will get the
benefit of both the
states to claim reservation.

. Page 26 of 40
97. Mr. Dinesh was an income tax officer in revenue department of Bihar, where he
acquired the domicile.
After the state reorganization, he opted to serve in the Lohardaga district of
Jharkhand. He shifted to
Jharkhand on 19th February, 2000 and gave birth to a child named Raj on 22nd
September, 2001. After
growing up, Raj wanted to appear for the state PCS exam of Jharkhand. However, his
application for
claiming reservation was denied by the authorities, stating the absence of a
domicile of the state of
Jharkhand. Decide.
(a) The authorities can reject his application, since his parents lived in Bihar
for the most of their lives.
(b) The authorities cannot reject his application, since Raj was born in Jharkhand.
(c) The authorities can reject his application, since his father had a domicile of
Bihar.
(d) The authorities cannot reject his application, since his father was posted in
Jharkhand.
98. In the previous question, what will be your answer if Mr. Dinesh would have
been serving in the State of
Bihar, while Raj took birth in Rajasthan and completed his schooling from a
residential school of
Jharkhand?
(a) Raj will be entitled to claim reservation, since he was born in Jharkhand.
(b) Raj will be entitled to claim reservation, since he spent his entire life in
Jharkhand.
(c) Raj will not be entitled to claim reservation, since his father had a domicile
of Bihar and served in the
same state.
(d) Raj will not be entitled to claim reservation, as this will defeat the idea of
avoiding benefits from
multiple sources.
99. Shyam was a public servant in Ranchi (capital of the successor state of
Jharkhand), which was technically
his place of origin and domicile as well. However, he went on serving in the state
of Bihar after the
reorganization in 2000. Now, his daughter Meera wanted to claim reservation in
Bihar. Decide on this.
(a) Meera can avail reservation because her father was serving in Bihar.
(b) Meera can avail reservation because public servants can avail such benefits.
(c) Meena cannot avail reservation because there is no provision is for the
services in Jharkhand.
(d) Cannot decide as Meera’s place of origin is not defined.
Passage (Q.100-Q.105): Not only does office of profit only find a fleeting mention
in the Constitution as a
ground for disqualification of Members of Parliament under A.102 and Members of
Legislative
Assemblies under A.191, but the ambiguity surrounding the law also has not
prevented its unhindered
usage.
Now, while the intent behind adding it as a ground for disqualification in the
Constitution is clear - legislators
should not feel obligated to the executive in any way, which could influence their
decision-making process
while they are discharging legislative functions, and to avoid any conflict b/w the
duties and interests of a
legislative member, why the need to define it was not felt is still not clear.
In Pradyut Bordoloi v. Swapan Roy, Supreme Court again clarified the law and laid
down four broad
principles for determining whether an office attracts constitutional
disqualification. First: Whether the
government exercises control over appointment, removal, and performance of the
functions of the office.
Second: whether the office has any remuneration attached to it, Third: whether the
body in which the office
is held has certain specific powers like releasing money, allotment of land,
granting licenses, etc., and
fourth: whether the office enables the holder to influence by way of patronage.
Another significant judgment
on the matter, came in the case of Jaya Bachchan v. Union of India where the court
said that, "payment of
honorarium by government of India, in addition to daily allowances like
compensatory allowances, rent-free
accommodation, and chauffeur-driven car at the state expense, are clearly in the
nature of remuneration
and a source of pecuniary gain and hence constitute office of profit."

. Page 27 of 40
100. The government gave out a contract to X with a profit margin of 22% to
construct a canteen in the premises
of the parliament. After 3 months when almost half of the construction was
complete, X decided to contest
in the local panchayat elections and he eventually won the same with huge margin.
But as soon as he
started working there, someone filed a complaint in the election commission that X
holds an office of profit.
Determine whether X can be disqualified for holding an office of profit or not.
(a) X can be disqualified since he is the elected representative of the panchayat
as well as the contractor
of a government project at the same time.
(b) X cannot be disqualified for holding an office of profit since the contract of
canteen will not come
under the ambit of office of profit.
(c) X can be disqualified since he cannot extract profit from two departments
simultaneously.
(d) X cannot be disqualified since his contract of the canteen cannot alter his
decision-making process
during his discharge of his legislative duties.
101. Suppose in the previous question, instead of getting a contract of canteen
construction, X got employed in
a PSU bank in the post of zonal manager. With the rest of the facts remaining the
same, would your answer
to the previous question change?
(a) Yes, he can be disqualified for holding an office of profit.
(b) No, he can be disqualified for holding an office of profit.
(c) Yes, he cannot be disqualified for holding an office of profit.
(d) No, he cannot be disqualified for holding an office of profit.
102. Y was very enthusiastic of entering into politics but due to financial
restrains he had to take up a job to
support his family. Accordingly, he worked very hard to crack UPSC examinations and
he became an IAS
officer. After few years of service, he made some contacts in political arena and
decided to contest for
municipality elections. He contested, and he was declared winner and thus he
eventually started working
in both the portfolios. Determine as per the passage whether he is ought to be
disqualified or not?
(a) He cannot be disqualified since he is not holding any office of profit.
(b) He can be disqualified since he is holding any office of profit.
(c) He cannot be disqualified since the contention of office of profit per se will
not be applicable in Y’s
case.
(d) He can be disqualified since he contested for the election without resigning
from his post of an IAS
officer.
103. Suppose in the previous question, instead of becoming an IAS officer Y chose
to become a government
school teacher. With the rest of the facts remaining the same, would your answer to
the previous question
change?
(a) Yes, he can be disqualified for holding an office of profit.
(b) No, he can be disqualified for holding an office of profit.
(c) Yes, he cannot be disqualified for holding an office of profit.
(d) No, he cannot be disqualified for holding an office of profit.
104. Z was a reputed overseas businessman who resided in India. Due to his
extensive network all over the
Europe and his expertise in international trade and commerce, he was given an
honorary seat in the
European chambers of commerce and trade, which was a direct subsidiary of European
Union’s
international trade vertical. After few months Z decides to contest for Lok Sabha
elections. Determine if he
can be disqualified for holding office of profit.
(a) He can be disqualified since he satisfies the principles laid by the supreme
court.
(b) He cannot be disqualified since his office of profit is not in India.
(c) He can be disqualified since he cannot two offices of profits simultaneously.
(d) He cannot be disqualified since the very contention of office of profit will
not be applicable in the given
case.

. Page 28 of 40
105. Suppose in the previous question, Z was given a similar position in the Indian
counterpart of the European
chambers of commerce and trade by government of India. With the rest of the facts
remaining the same,
would your answer to the previous question change?
(a) Yes, he can be disqualified for holding an office of profit.
(b) No, he can be disqualified for holding an office of profit.
(c) Yes, he cannot be disqualified for holding an office of profit.
(d) No, he cannot be disqualified for holding an office of profit.

. Page 29 of 40
SECTION - D: LOGICAL REASONING
Passage(Q.106-Q.110): With the advent of the Covid-19 pandemic, normal life — in
India and across the
world — has been fundamentally disrupted. More worrying is the fact that the virus
has also adversely
affected life expectancy. A recent study has revealed that life expectancy at birth
of both men and women
in India decreased by around two years — from 69.5 years and 72 years in 2019 to
67.5 years and 69.8
years, respectively — in 2020 owing to Covid-19. The term refers to the average
number of years that a
new born is expected to live if the mortality pattern at the time of birth remains
constant in the future. Given
that the number of Covid deaths in the country has by now crossed 4.5 lakh — this
is the official tally —
the drop in life expectancy is not surprising. The pandemic has even undone a
decade’s efforts to increase
life expectancy in the country — it has now regressed to the 2010 figures. But this
is no reason to lose
heart. Experts argue that such a decline is not uncommon after a health crisis of a
massive scale.
The parity in the decline in life expectancy across the gender divide can be
misleading: the impact of the
pandemic on men and women has been far from equal. Globally, as in India, more
women lost their jobs
during Covid-19 than men. During the lockdown, this difference amounted to 40
percentage points. The
burden of unpaid care work at home, exacerbated by the pandemic, also fell
disproportionately on female
shoulders. However, government outreach and palliative measures have not been
weighted appropriately
to correct this imbalance — even as India administered one billion jabs, official
data indicate that 6 per cent
fewer women are getting vaccinated. Other marginalized constituencies are being
affected unevenly too.
The pandemic’s wrath has fallen even more harshly on the transgender community —
many have
reportedly been deprived of even the subsistence allowance of Rs 1,500 pledged by
the Union government.
The inoculation campaign has, thus, been smeared by existing social cleavages. That
the pandemic would
curtail life was expected; but it is the duty of the government to ensure
vulnerable constituencies are not
disenfranchised — socially, economically and in terms of health. Vigilance and
closer monitoring of the
health and rightful benefits of women and those on the margins are key to equitable
healthcare and quality
of life.
[Extracted with edits and revisions from The Telegraph]
106. What is the central message of the passage?
(a) Covid 19 pandemic has decreased life expectancy, which is a matter of grave
concern.
(b) The dip in life expectancy brought about by Covid 19 is not as disturbing as
the impact of the pandemic
on different constituencies.
(c) Different groups of people suffered differently during the pandemic and this
was not a surprise.
(d) The government and the authorities failed to curb the effects of the pandemic
and failed collectively.
107. Which of the following is true, as per the passage?
i. A new born would live for at least 67.5 years, if it was born in 2020.
ii. A new born would live for at least 69.5 years, if it was born in 2019.
iii. The decrease in life expectancy in men during the pandemic is sharper than
that of women.
(a) Both i and ii are correct (b) Only iii is correct
(c) All are correct (d) None of the above
108. Which of the following would strengthen the author’s claims in the passage?
(a) Life expectancy in Africa had dropped drastically owing to the HIV/AID
epidemic, but it recouped in a
few years.
(b) There is a condition called Lipoma which makes one ineligible for inoculation
against Covid 19, this is
more common in women than men.
(c) The Europe till date has not recovered in life-expectancy after the Spanish
flu.
(d) Decline in women’s job was at its ebb pre-Covid.
. Page 30 of 40
109. What can be attributed to be the author’s opinion from the passage?
(a) The impact of Covid 19 on different groups of people was surprisingly not very
different.
(b) The lockdown imposed due to Covid 19 was unprecedented and unexpected.
(c) The impact of Covid 19 on different gender groups - men, women and transgender
was different.
(d) The Union Government does a better job than state governments in terms of
making the society more
welcoming towards vulnerable groups.
110. What does the author mean by this statement, “Government outreach and
palliative measures have not
been weighted appropriately to correct this imbalance.”?
(a) The government had the opportunity to correct the imbalance created by Covid
19, but its span in terms
of measures were unevenly distributed, leaving many unimpacted.
(b) The government could not make sure that all the groups are treated equally when
the natural floods
took so many lives.
(c) The gender gap was created due to the pandemic and it was the duty of the
government to further
increase this divide.
(d) The State governments and the Union government should be held accountable for
the gender divide
that they have created.
Passage(Q.111-Q.115): In its most recent meeting, the monetary policy committee of
the RBI chose to
maintain the policy status quo, keeping the benchmark repo rate at 4 percent and
continuing with its
accommodative stance, presumably as long as is necessary to revive growth. But the
decision was not
unanimous. One of MPC’s members voted not only against continuing with the
accommodative stance, but
also argued in favour of raising the reverse repo rate. Concerns over how long the
MPC can continue with
its policy stance appear to be gaining traction. Even though the other members
stuck to their earlier
positions, there appears to be a shift in the tone of the commentary, with another
member arguing that
“over-stimulus as after the global financial crisis, with delay resulting in sharp
adjustment, has to be
avoided.” So far, the MPC has continued to attach primacy to growth considerations.
In their most recent
comments, most members highlighted the uneven nature of the recovery — the scarring
of small business
and the informal sector. According to another MPC member, of the 404 industries for
which data is
available, 63.4 percent are operating below 2018-19 levels. The RBI Governor also
argued that the
“informal sector is likely to take even longer to recoup as the impact of the
second wave on this sector was
relatively more pronounced.” But the space for continuing with its current stance
appears to be receding.
On the inflation front, the MPC members were cognisant of the risks, as, even
though headline inflation
has dipped — CPI fell to 4.35 per cent in September, down from 5.3 per cent in
August — core inflation
remains elevated. As the member who voted against continuing with the accommodative
stance noted,
“inflationary pressures are beginning to show signs of greater persistence than
anticipated earlier”.
Economists expect inflation to inch back upwards. According to the RBI’s
projection, it is expected to dip
to 4.5 per cent in the third quarter, rising thereafter to 5.8 per cent in the
fourth quarter. The withdrawal of
policy support is likely to be gradual with the process playing out at multiple
levels — beginning with
normalisation of liquidity, which has begun, followed by a hike in the reverse
repo, a shift in the stance from
accommodative to neutral, culminating in a repo rate hike. However, the duration
over which this plays out
is uncertain. Perhaps a few more months of data will provide greater clarity over
the durability of the
recovery, allowing the MPC to firm up its exit plans.
[Extracted with edits and revisions from The Indian Express]
111. Which of the following is true according to the passage?
(a) There is a clear plan over the duration of the withdrawal of policy support.
(b) The RBI chose to continue with the present repo rate with the consent of all
its members.
(c) The decision of the RBI to maintain the status quo met with some resistance
from the members of the
MPC.
(d) The fall of CPI refers to an increase in the inflation rate, according to the
Reserve Bank of India.

. Page 31 of 40
112. Out of the following, which one is the best representation of the main message
of the author in the passage?
(a) It will need greater clarity over durability of recovery for RBI to firm up its
moves towards policy
normalisation.
(b) Policy normalisation is a distant goal for the RBI, firstly it needs to bring
the members of the MPC to an
agreement.
(c) The informal sector was one of the hardest hit sectors due to the second wave
and all efforts must be
made to revive it.
(d) The difference in opinion of the members of the MPC over a subject of
particular concern is disturbing,
which might hamper its efficiency.
113. Which of the following can be inferred from the given passage?
(a) Disagreement between members of an organization is a sign of its bleak future.
(b) The RBI is a body which aggravates the already fragile situation of the weaker
sectors.
(c) The RBI’s stance on growth considerations is likely to change in future.
(d) The monetary policy committee of the RBI, in the recent meeting, chose to
maintain the policy status
quo, keeping the benchmark repo rate at 4 percent.
114. For which of the following claims has the author presented some pieces of
evidence?
(a) Till now, the stance of the Monetary Policy Committee on growth considerations
has not changed.
(b) The decision of the Monetary Policy Committee to maintain the status quo was
not unanimous.
(c) The withdrawal of policy support by the Monetary Policy Committee is likely to
be abrupt.
(d) Both a and b
115. ‘In its most recent meeting, the monetary policy committee of the RBI chose to
maintain the policy status
quo, keeping the benchmark repo rate at 4 percent and continuing with its
accommodative stance,
presumably as long as is necessary to revive growth. But the decision was not
unanimous. One of MPC’s
members voted not only against continuing with the accommodative stance, but also
argued in favour of
raising the reverse repo rate.’ Which of the following reflects the assumption(s)
behind the passage?
I. There are more than one committee that governs the monetary policies of the RBI.
II. The decision to maintain the policy status quo with regard to repo rate was
unanimous.
(a) Only assumption I is implied.
(b) Only assumption II is implied.
(c) Both assumptions I and II are implied.
(d) Neither assumptions I nor II is implied.
Passage(Q.116-Q.120): In the current public unrest in France over the rising price
of the baguette are
echoes of what happened 232 years ago. Back then, the fact that a staple had been
priced out of the reach
of the masses led to the French Revolution and the dramatic, bloody collapse of the
ancient régime. The
situation isn’t quite so dire right now — the president of the French Confederation
of Bakeries and Pastry
Shops (FCBPS) has said that unlike then, bread is still available, even if it’s
more expensive.
Around the world, the unaffordability or non-availability of food has, from time to
time, led to mass unrest,
and even the collapse of governments. Bread riots had preceded the fall of the
Bastille in 1789, which was
stormed in part because the starving sans culottes were looking for grain.
Similarly, in 1918 a precipitous
rise in the price of rice caused riots in Japan, which led to the resignation of
Prime Minister Terauchi
Masatake and his cabinet. And consider how frequently that Indian pantry staple —
the onion — has caused
electoral convulsions: From fuelling public anger against Indira Gandhi’s
government in the pre-Emergency
months to powering her resurgence in the 1980 general elections. Onion price rise
was also one of the
factors in the ousting of the BJP in the 1998 Delhi Assembly elections.
France’s current surge in the price of the baguette is due to bad harvests in
Russia, which have led to a
global rise in the price of wheat. Greater energy prices have also made ovens more
expensive to operate
and the heat is being felt by the nation’s famous boulangeries, as well as the
average consumer of the

. Page 32 of 40
long, baton-shaped bread which is seen as an icon of French culinary heritage. As
history teaches us,
when people complain about basic foodstuff being unaffordable, governments can’t
just ask them to eat
cake.
[Extracted with edits and revisions from The Indian Express]
116. Which among the following can be deduced from the passage?
(a) The President of FCBPS feels that there is no difference between the food
crisis in the past and the
present.
(b) There have been a lot of protests and mass movements against the unavailability
or high prices of
exquisite food.
(c) There is no option available to the French Government other than asking its
people to eat cake.
(d) None of the above.
117. If the information set out in the passage is true, then which of the following
must be true?
(a) There is a link between the high operational costs of Ovens and food prices.
(b) Sometimes a food staple can go out of stock without the fault of the country
where the crisis has taken
place.
(c) The non-affordability or non-availability of the food has led to mass riots
time and again.
(d) An average consumer can withstand the heat of the food prices or the non-
availability of the food by
studying the pattern of food production and consumption.
118. Which of the following can be inferred about the various food crises around
the world?
I. Unaffordability of staple diet can lead to mass movements.
II. Unavailability of food can lead to fall of governments.
III. Some leaders of Japan were forced to resign for the unavailability of food.
(a) Only I and II (b) Only II (c) Only I and III (d) All I, II and III
119. What can be inferred about the present food crisis in France?
(a) The situation in France is so grave that there is no food to feed people.
(b) The unavailability of the food concerned is not entirely due to the failure of
the French government.
(c) The present food crisis in France is likely to repeat history, with the second
French revolution coming
in.
(d) None of the above.
120. ‘As history teaches us, when people complain about basic foodstuff being
unaffordable, governments can’t
just ask them to eat cake.’ What can be inferred from the given argument?
(a) The government cannot offer something that is contrary to the expectations of
the people in order to
save itself.
(b) History is witness to unrest in various countries as and when the governments
have failed to meet even
the basic foodstuff due to inflation.
(c) The history indicates that the wrong alternatives or policies by the government
with regard to food prices
and availability led to public unrest and subsequent government fallings.
(d) If people demand the basic foodstuff, the government cannot present exclusive
food as an alternative
to appease people in order to avoid ousting.

. Page 33 of 40
Passage(Q.121-Q.125): Read the passage below and answer the questions that follow:
Humans have always used stories to make sense out of our chaotic world. When our
ancestors had to kill
animals they felt were kindred spirits to survive, they created myths to help them
come to terms with it.
When they invented agriculture, they created myths that glorified graft and
highlighted the seasonal nature
of existence. When they began to settle, humans created myths imbuing cities with
transcendence. Fast#forward to the volatile times we live in today, where people
increasingly recognize that tackling problems
like climate change, inequality, and health care requires a systems approach. The
work of systems change
involves seeing systemically—looking at the elements, interconnections, and wider
purposes of systems—
and acting systemically. Story plays a vital role in helping us do both of these
things.
Story has many different qualities that make it useful for the work of systems
change. It’s a direct route to
our emotions, and therefore important to decision-making. It creates meanings out
of patterns. It coheres
communities. It engenders empathy across difference. It enables the possible to
feel probable in ways our
rational minds can’t comprehend. When it comes to changing the values, mindsets,
rules, and goals of a
system, story is foundational.
Supporting niches of innovation is another important part of systems change. This
not only affects the
elements in a system, but also can transform the nature of the relationships in a
system and ultimately its
purpose. A core part of this work involves illuminating outliers: the mavericks,
pioneers, and intrapreneurs
who are already trying to make change happen. Story plays two important roles here.
First, systems
changers can increase the profile and impact of outliers by amplifying their
stories. Second, they can use
story to create unifying narratives of change, adding momentum to innovative
movements.
121. Which of the following statements will the author deem the truest?
(a) There are some lessons that cannot be taught without storytelling.
(b) Storytelling cannot be used to provide all kinds of messages and information.
(c) Systemic change can only be brought by storytelling.
(d) Historical evidence points towards the use of storytelling to generalize
certain phenomenon to make
human actions make sense.
122. Which of the following conclusions can’t be properly drawn from the statements
above?
(a) The work of systems change involves looking at things more wholistically from a
wider perspective.
(b) The work done by the change-makers of the society must be exaggerated and
presented as stories.
(c) Storytelling is a powerful tool that can be used to harmonize and mobilize
people for various causes.
(d) Story is extraneous when trying to change the values, mindsets, rules, and
goals of a system
123. Which of the following options can be said to be proper instances mentioned in
the passage?
I. Chandler Bing wants to reach the grapes hanging on the high branch of a tree. He
tries a lot to climb,
but fails. He ultimately thinks that its better he didn’t eat those grapes, because
those grapes are sour.
II. Joey watches a movie where the authority asks his people to make sacrifices of
the animals for the
higher good of humanity.
(a) Only I (b) Only II (c) Both I and II (d) Neither I nor II.
124. Which of the following, if true, might weaken the author’s claim?
(a) Various people prefer to have debates over topics rather than monologue
discussion
(b) People’s intelligence and analytical ability makes them see through the
specious justifications behind
the stories.
(c) Some problems cannot be solved by creating stories revolving around them.
(d) Form the yore, myths have been created to gradually adapt people to system
change.
125. The author’s statement that ‘A core part of this work involves illuminating
outliers: the mavericks, pioneers,
and intrapreneurs who are already trying to make change happen.’
(a) Forms premise of the above passage.
(b) Forms conclusion of the above passage.
(c) Forms assumption of the above passage.
(d) Forms inference of the above passage.

. Page 34 of 40
Passage(Q.126-Q.130): Read the passage given below and answer the question that
follows.
Indian women excelled in the most decorated Olympic Games for India so far. There
is no reason for it to
be otherwise in any other field, especially education, given the right support. As
a nation, we can ill-afford
to ignore half the potential workforce if we aspire to be an economic powerhouse.
As a society, women can
be the pivot to bring about critical and lasting social transformation. As
individuals, they deserve a shot at
being the very best they can.
First, as an immediate step, in every locality, a mohalla school or a community
learning programme should
be started with appropriate Covid norms, if the local disaster management
authorities and the state
governments permit. Evidence from the Ebola pandemic shows that continued
engagement with
educational activities reduces drop-outs in a statistically significant way. NITI
Aayog, with the help of civil
society organisations, had started a community programme led by volunteers called
“Saksham Bitiya” in
28 aspirational districts where more than 1.87 lakh girl students were trained in
socio-emotional and ethical
learning. Such initiatives should be replicated to ensure more girls do not drop
out of schools during the
pandemic.
Second, to predict likely drop-outs, a gender atlas comprising indicators that are
mapped to key reasons
for school drop-outs should be developed. Teachers should also be trained in all
the scholarships and
schemes available that provide economic support to girls and their families for
continuing their education.
Third, there is a need to revise the National Scheme of Incentive to Girls for
Secondary Education in areas
or states with high prevalence of drop-outs and early child marriages. The
scholarship amount may be
increased and tied to the completion of graduation, with yearly scholarships paid
to students upon
successful completion of each year of their undergraduate degree.
State governments need to leverage existing schemes to design interventions to
promote women in higher
education. The recently modified viability gap funding scheme includes provisions
for social infrastructure
projects, including education. For greenfield projects in higher education, 60 per
cent of the funding can be
accessed as viability gap funding from the Central and State governments.
The pandemic has brought unprecedented challenges for educators and students,
especially for those on
the margins, including girls. However, with recent experiments and learning
experience, informed targeting
of ample resources and an agile policy environment, this challenge could well prove
to be an opportunity.
126. The passage can be best termed as:
(a) Prescriptive (b) Descriptive (c) Abstract (d) Narrative
127. The author’s main purpose is to:
(a) Suggest that learning experience is more important.
(b) Highlight that the government has to make sure that the girls do not drop out
of school.
(c) Propagate that economic support to girls and their families is required.
(d) Warn that without girl education, a healthy society cannot thrive.
128. The newly amended viability gap funding plan contains
(a) Facilities such as cultural participation as a part of social economic
projects.
(b) Services such as providing midday meal to girl child and take care of all the
nutrition in their daily diet.
(c) PPP (Public Private Partnership) Arrangements for social infrastructure
projects.
(d) None of the above.

. Page 35 of 40
129. ‘As a society, women can be the pivot to bring about critical and lasting
social transformation. As individuals,
they deserve a shot at being the very best they can.’ Which of the following
assumption is behind the
argument
(a) Women, individually and socially can bring transformative changes.
(b) Women empower a nation.
(c) A women can help other women in bringing critical and lasting social
transformation.
(d) A society cannot survive without women.
130. Being assertive, the author claims that the women have the potential to be the
catalyst for
(a) removing the long-lasting stigma of non-education in the society.
(b) a drastic change in the level of incomes and employment generation.
(c) a lasting change in the performance of the assets and shares.
(d) a deep-rooted social change in the communities that we all live in.
Passage(Q.131-Q.135): Passage A: Interdependence between the Chinese and American
economies
would create conditions to prevent conflict and channel China to shoulder more of
the burden in
international affairs (lessening the burden on the United States), lift people out
of poverty, and set
conditions for gradual, evolutionary democratization that would cement better Sino-
American relations and
benefit the cause both of global order and of human rights. A trade war risks all
of that by incentivizing
conflict and removing the linkages which might cushion tensions, and, by negatively
impacting the standard
of living of both Chinese and Americans who depend on the trading relationship, can
be seen as unethical
in that regard.
Passage B: The trading relationship has allowed a Chinese regime that is
antithetical to liberal values at
home and to the existing international system to acquire more power and resources,
which it has used to
both pursue greater capabilities to act in the world (often at odds with U.S.
preferences), but also to more
effectively repress its citizens at home. Disconnecting the U.S. and Chinese
economies, despite the short#term pain, is ethical in the long run for removing any
tacit U.S. support for China’s unliberal practices at
home which are at odds with American values but also to lessen the economic and
technological bases
from which China is emerging as a near-peer competitor to the U.S.
131. The guiding principle of Passage A is that
(a) Avoiding cooperation with entities in conflict with one’s value is ethical.
(b) Avoiding something that negatively impacts standard of living is unethical.
(c) Promoting free trade and economic interdependence to avoid conflict is ethical.
(d) Promoting cooperation with entities antithetical to liberal values is ethical.
132. The guiding principle of Passage B is that:
(a) Encouraging cooperation with those who are in conflict with one’s value is
ethical
(b) Avoiding something that negatively impacts standard of living is unethical
(c) Promoting free trade and economic interdependence to avoid conflict is ethical
(d) Avoiding cooperation with those that are antithetical to liberal values is
ethical
133. Both the author of Passage A and the author of Passage B would agree with
which one of the following?
(a) A trade war between US and China will benefit US in the long run.
(b) A trade war between US and China will create negative impact on the trades on
both sides.
(c) A trade war between US and China will reduce the standard of living of US
citizens.
(d) A trade war between US and China can be effectively addressed through
discussions.

.
134. Which one of the following most accurately characterizes the author of passage
B’s response to the author
of passage A?
(a) The author of passage B falsely accuses the author of passage A of
contradicting their own views.
(b) The author of passage B unfairly directs his argument against the author of
passage a personally.
(c) The author of passage B uncovers a hidden assumption underlying the author of
passage A’s position.
(d) The author of passage B contradicts the reasons the author of passage A cites
in favor of her
conclusion.
135. U.S. imposed six waves of tariffs on a number of imports of China. The US’
losses mounted steadily over
the year, as each wave of tariffs affected additional countries and products from
washing machines to steel
and aluminum. The US’ losses hit the hardest after the sixth wave, when the U.S.
levied $200 billion in
Chinese imports with a 10 percent tariff.
If the above statements are true, based on the reasoning of the author of Passage
B:
(a) The loss is justified as China suffered greater loss.
(b) The loss is not justified as it hurt American citizens.
(c) The loss is not justified as it is against liberal practices.
(d) The loss is justified for long-term consideration

mock-21
Directions (Q.1-Q.30): Read the following passage carefully and answer the
questions that follow.
Passage (Q.1-Q.6): Step into the study in Della Burnside's five-storey home and a
clementine corner sofa and
matching partition wall, a crimson rug and poof and an exuberant pink feather
flapper girl lamp vie for your
attention. "The colours all over the house are shocking and bright," says Burnside.
"They suit my personality
better than my old house, which was more restrained." That was a late Victorian
property, also in Clerkenwell,
central London, and it was "dark, with lots of wood panelling and traditional
furniture". Fog House, which she
bought in 2016, gets its name from the sandblasted wall of glass on the top floor
of the converted warehouse.
Once occupied by Marc Quinn, the YBA who made a sculpture of his head out of his
own frozen blood, the
building has itself been given fresh blood, courtesy of leading architect Sir David
Adjaye, who added a glass#clad parapet and a cantilevered glazed extension on the
back.
"I tell visitors to look for the house that looks like a prison," says Burnside, an
investor and solicitor. It prepares
them for the stark black exterior with frosted windows. Cross the threshold,
though, and you might think you're
in a funky library-cum-lifestyle pod. Almost an entire wall of the double-height
space is lined with bookshelves
that sit on a steel gantry, the titles ranging from travel to politics to old
favourites from Burnside's childhood. "I
like the feel of having books in a house," says Burnside. "You can't beat it." Peer
over the railings and you'll see
a blob of blue, curves of steely grey, rounds of black and flashes of red and white
- it's Burnside's home gym,
the equipment bouncing off a mirrored wall.
"This space was previously used as a lounge," she says, "but I wanted a lounge in
the light. I use the gym every
day - I'm very disciplined. I like constantly going up and down stairs, too," she
says. "It keeps me fit." She's less
keen on the twisting plywood staircase that leads to the top floors. "Apparently
it's a very fancy design," she
laughs. "Personally, I don't like it that much. I might quite like to paint it, I
mentioned when I was looking round.
There was shock and horror at that." It's what you get if you buy through the
design-led estate agency The
Modern House, as Burnside did.
Each gleaming floor is open plan, with colour functioning as a focal point or room
divider - sometimes both. In
the showstopper top floor, glazed on three sides and home to the kitchen, dining,
living area and roof terrace,
purple is the eye candy. It is a hand-me-down from Elton John, an old friend of the
previous owner, Janet Street#Porter, who commissioned David Adjaye to create Fog
House.
1. Who was an old friend of Janet Street-Porter?
(a) Marc Quinn (b) Broke Stuart (c) Elton John (d) Della Burnside
2. Which of the following facts about Della Burnside's old house is correct in the
context of the passage?
(a) It had a big kitchen. (b) It had a swimming pool.
(c) It was a late-Victorian property. (d) It had bright colours.
3. Which one of the following is a design-led estate agency as per the passage?
(a) The Old House. (b) The Modern House.
(c) The Peculiarity. (d) The Design House.
4. How was the gym place previously used?
(a) As a dining room (b) As a studio (c) As a storeroom (d) As a lounge
5. “…a crimson rug and poof and an exuberant pink feather flapper girl lamp vie for
your attention…” Which of
the following best expresses the author’s emotional state?
(a) The author is caught off guard by the splendour of the décor that is state-of-
the- art.
(b) The author is bemused at the child-like simplicity of the house owner.
(c) The author is shocked at the myriad colours that are at display in the house
and is unable to utter a sentence.
(d) The author is surprised at the bright colours of the décor; as it each item was
competing for and seeking
attention of the onlooker.

. Page 3 of 36
6. Which of the following word can replace the word "keen" as used in the context
of the passage without changing
the meaning or making the sentence grammatically incorrect?
(a) Embrace (b) Abate (c) Eagerly (d) Avid
Passage (Q.7-Q.11): Salcomp, one of Apple's major component suppliers, has bought
Nokia's defunct handset
factory near Chennai for close to $30 million (Rs215 crore), setting the stage for
the revival of what was once
India's poster child for local manufacturing but which was shut down in late 2014
after a tax dispute between the
Finnish company and local authorities. Salcomp is expected to begin operations at
the plant by March next year,
and ramp up exports from there to about $2 billion by investing $300 million over
the next five years. Once fully
functional, the new unit is expected to generate employment for close to 10,000
people. Salcomp's acquisition
of the handset manufacturing plant - it was once the world's largest, producing 100
million handsets a year and
employing some 12,000 people - coincides with iPhone maker Apple's strategy to
expand production in India
for the local and global markets. Apple is gradually shifting away from China amid
the Sino-US trade war.
The American company sources chargers from Salcomp, which is the largest supplier
for it globally and which
has made India a huge base over the past few years. Apple's major contract
manufacturers, Wistron and Foxconn,
have also been ramping up investments in the country. The iconic smartphone major's
other component suppliers
such as Flex, Sunwoda Electronic Co, CCL Design (Suzhou) and Shenzhen Yuto
Packaging Technology have
also set up units in India and are manufacturing parts and accessories here. The
Nokia handset plant, set up in
2006, at its peak was the world's largest and showcased India's local manufacturing
prowess. But since 2013,
when India issued a Rs 2,500 crore withholding tax demand on the company, Nokia,
along with Vodafone Group
and Shell, was also identified with the aggressive stance of India's tax
authorities, causing an ________ among
foreign investors. The plant was shut in November 2014, after it was left out of
Nokia's deal to sell its devices
business to Microsoft earlier that year due to the tax dispute.
7. Which is the world's largest handset manufacturing plant currently?
(a) Wistron (b) Apple (c) Salcomp (d) None of the above
8. How much is Salcomp expected to invest over the next five years in the plant?
(a) $300 million (b) $100 million (c) $200 million (d) $350 million
9. When was the Nokia handset plant set up in India?
(a) 2013 (b) 2006 (c) 2014 (d) 2007
10. Which of the following word can fit grammatically and contextually in the blank
given in the passage?
(a) Calmness (b) Uproar (c) Chaos (d) Outrageous
11. Which of the following word best expresses the opposite meaning of the word
"peak" as used in the context of
the passage?
(a) Height (b) Pinnacle (c) Nadir (d) Apex
Passage (Q.12-Q.16): The internet was built on open standards and interoperability,
but networks tend to
balkanize. Every now and then, we see attempts to re-open the internet. Last month,
Twitter announced Bluesky,
a project to create an open protocol for social media. If adopted, Bluesky would
allow anyone to spin up a Twitter
clone and host a social network. Content could be shared across platforms - for
example, a Twitter user could
retweet a post from a Bluesky version of Facebook, allowing her followers to
interact with the post without
switching accounts.
There are plenty of other examples. One of the older ones is XMPP, an open protocol
developed for social
networking and chatting. The project began in 1999 and was adopted and later
abandoned by AOL Instant
Messenger, Facebook Chat, and Google Talk. A more recent introduction comes from
Mastodon, a decentralized
social network as well as a software, in the form of ActivityPub.

. Page 4 of 36
Take Wikipedia, regarded as a definitive source of truth by factcheckers on
Facebook and YouTube. After
criticizing Wikipedia's oligarchic editing process, a former co-founder created the
Knowledge Standards
Foundation to promote an open protocol where anyone can host wiki-type
encyclopaedia pages. Custom reading
interfaces can then fetch pages from sources preferred by the individual user.
Then there's Google Search. Google's prowess doesn't come from hundreds of
petabytes stored on index servers,
although that does create a massive barrier to entry for competitors. The company's
dominance comes from its
proprietary ranking algorithm, designed to deliver the worthiest links to the top
of a user's search results. Google
has been accused of de-ranking certain sites, promoting political biases, or even
spreading disinformation. An
open protocol for Google Search could allow anyone to set up a web server to
receive queries and forward them
to the index servers, then rank results using competing algorithms. The approach
could be extended to any
content platform.
The challenge, of course, is making a profit. It's hard for platforms to monetize
an open protocol. Jack Dorsey
explains how an open protocol might be good for Twitter: "It will allow us to
access and contribute to a much
larger corpus of public conversation, focus our efforts on building open
recommendation algorithms which
promote healthy conversation, and will force us to be far more innovative than in
the past." No company truly
wants to be forced to innovate, but the first two points are valid. As a non-
dominant social network, Twitter
could benefit from accessing external content, just like Google gains access to new
data each time a Gmail user
corresponds with an external email address.
Distributed platforms also give companies a way to avoid the messy problem of
content moderation. Google
contracts with over 10,000 search quality raters who spend their days evaluating
controversial search results
while Facebook has hired 15,000 moderators to manage problematic content. With a
federated internet, these
companies would not have to employ so many moderators or worry about creating a
catch-all policy that attempts
to please everybody.
12. According to the passage, which of the following is/are TRUE?
I. Google gains access to new data, each time a Gmail user corresponds with an
external email address.
II. Google's dominance comes from its proprietary ranking algorithm, designed to
deliver the worthiest links to
the top of a user's search results.
III. It is easy for platforms to monetize and make profits through an open
protocol.
(a) Only I & III (b) Only II (c) Only I & II (d) Only III
13. According to the passage, what is 'Mastodon'?
(a) A microblogging website like Twitter and Facebook.
(b) An online publishing platform to write blogs.
(c) A social news aggregation, web content rating, and discussion website.
(d) A decentralized social network as well as a software.
14. According to the passage, which of the following is NOT TRUE?
(a) If adopted, Bluesky would allow only a privileged few to spin up a Twitter
clone and host a social network.
(b) XMPP was an open protocol developed for social networking and chatting that
began in 1999 and was later
abandoned.
(c) Distributed platforms give companies a way to avoid the messy problem of
content moderation.
(d) The internet was built on open standards and interoperability.
15. Which of the following reflects the tone of the passage?
(a) Reflective (b) Analytical (c) Quixotic (d) Aggressive

. Page 5 of 36
16. According to the fourth paragraph of the passage, which of the following
activities has Google been accused of?
I. Promoting political biases.
II. De-ranking certain sites.
III. Avoiding payment of taxes.
IV. Spreading disinformation.
(a) Only I & IV (b) Only I, II & III (c) Only I, II & IV (d) Only II & IV
Passage (Q.17-Q.21): William James, the father of Western psychology, in 1902
defined spiritual experiences
as states of higher consciousness, which are induced by efforts to understand the
general principles or structure
of the world through one's inner experience. At the core of his view of
spirituality is what we might call
'connectedness', which refers to the fact that individual goals can be truly
realised only in the context of the
whole - one's relationship to the world and to others.
Traditionally, this spiritual state has been described as divine, achievable
through contemplative and embodied
practices, such as prayer, meditation and rhythmic rituals. Indeed, this higher
state of consciousness and
connection has been reported in many spiritual traditions, ranging from Buddhism to
Sufism and Judaism to
Christianity. However, recent neuroscientific research shows that the same state
can be achieved by secular
practices too. Scientific and creative epiphanies with their accompanying ecstatic
states characterised by a sense
of unity and bliss are similar to religious experiences, with both involving a
higher state of presence and
observation. Many geniuses, such as Albert Einstein and the mathematician Srinivasa
Ramanujan, reported
spiritual-like states during their revelations or breakthroughs. But these don’t
have to be the rare experiences of
a chosen few. They can be reached in daily life. As the Novel laureate and poet
Czeslaw Milosz put it:
‘Description demands intense observation, so intense that the veil of everyday
habit falls away and what we paid
no attention to, because it struck us as so ordinary, is revealed as miraculous.’
I’m a neuroscientist and, among other things, I study the way that spiritual states
are reflected in the brain and
other parts of the body. Spiritual practices have been shown to be closely linked
to self-awareness, empathy and
a sense of connectedness, all of which can be correlated with the frequency of
brainwaves as measured by
electroencephalogram (EEG). Studies using EEG have demonstrated conflicts between
our behaviour, thinking,
feeling and communication. On the other hand, expert meditators demonstrate more
'harmonious' brain waves,
which could be indicative of greater synchrony or connectivity within and across
different neural areas. In short,
spirituality, similar to love, has physiological effects in the brain and body, and
EEG provides a window on these
changes.
Research suggests that we can do more than just measure this kind of activity. We
can also train our brains to
behave in a more ‘aware’ way by engaging in activities that facilitate greater
connection or neural
synchronisation. Higher synchronisation has been found following the practice of
different contemplative
paradigms, such as meditation and prayer. Neuronal synchronisation also correlates
with feeling more self#connected, which can, in turn, further increase empathy,
creativity and social effectiveness.
17. Which of the following is not one of the suggestions made by the study that
used EEG?
(a) We can teach others to stay more active and aware.
(b) We can engage in activities that make us more aware.
(c) We can involve in activities that make our neurons more connected.
(d) We can train our brain towards higher synchronisation makes us feel self-
connected.
18. According to William James, spiritual experience is defined as
(a) Spiritual traditions.
(b) The higher state of consciousness.
(c) Scientific and creative manifestation of a divine being.
(d) Principles of the world.

. Page 6 of 36
19. In the context of the passage, what is meant by the statement "which could be
indicative of greater synchronicity
or connectivity within and across different neural areas"?
(a) Spirituality is a brain state we can all reach whether or not we want.
(b) That neuronal synchronisation creates conflicts between our thoughts and
actions.
(c) We can create a life that changes our focus from basic needs to values.
(d) That neuronal synchronisation enhances our brain harmony, creating
physiological effects akin to the feeling
of love.
20. Which of the following words can act as the synonym of 'embodied', as used in
the passage?
(a) disintegrate (b) hide (c) excluded (d) incorporate
21. As per the passage, what is/are the prerequisites to truly achieve one's
purpose?
(a) One’s state of self-awareness.
(b) One’s awareness of one's relationship with the world.
(c) One’s awareness of one's relationship with others.
(d) All (a), (b) and (c)
Passage (Q.22-Q.27): The discovery of natural selection, by Darwin and Alfred
Russel Wallace, must itself be
counted as an extraordinary philosophical advance. The principle remained unknown
throughout the more than
2,000-year history of philosophy ranging from the Greeks to Hume, Kant and the
Victorian era. The concept of
natural selection had remarkable power for explaining directional and adaptive
changes. Its nature is simplicity
itself. It is not a force like the forces described in the laws of physics; its
mechanism is simply the elimination
of inferior individuals. This process of non-random elimination impelled Darwin’s
contemporary, philosopher
Herbert Spencer, to describe evolution with the now familiar term “survival of the
fittest.”
The truly outstanding achievement of the principle of natural selection is that it
makes unnecessary the invocation
of “final causes”—that is, any teleological forces leading to a particular end. In
fact, nothing is predetermined.
Furthermore, the objective of selection even may change from one generation to the
next, as environmental
circumstances vary.
A diverse population is a necessity for the proper working of natural selection.
(Darwin’s success meant that
typologists, for whom all members of a class are essentially identical, were left
with an untenable viewpoint.)
Because of the importance of variation, natural selection should be considered a
two-step process: the production
of abundant variation is followed by the elimination of inferior individuals. This
latter step is directional. By
adopting natural selection, Darwin settled the several-thousand year- old argument
among philosophers over
chance or necessity. Change on the earth is the result of both, the first step
being dominated by randomness, the
second by necessity.
Darwin was a holist: for him the object, or target, of selection was primarily the
individual as a whole. The
geneticists, almost from 1900 on, in a rather reductionist spirit preferred to
consider the gene the target of
evolution.
For 80 years after 1859, bitter controversy raged as to which of four competing
evolutionary theories was valid.
Darwin’s theory clearly emerged as the victor during the evolutionary synthesis of
the 1940s, when the new
discoveries in genetics were married with taxonomic observations concerning
systematics, the classification of
organisms by their relationships. Darwinism is now almost unanimously accepted by
knowledgeable
evolutionists. In addition, it has become the basic component of the new philosophy
of biology.
A most important principle of the new biological philosophy, undiscovered for
almost a century after the
publication of On the Origin of Species, is the dual nature of biological
processes. These activities are governed
both by the universal laws of physics and chemistry and by a genetic program,
itself the result of natural selection,

. Page 7 of 36
which has moulded the genotype for millions of generations. The causal factor of
the possession of a genetic
program is unique to living organisms, and it is totally absent in the inanimate
world. Because of the backward
state of molecular and genetic knowledge in his time, Darwin was unaware of this
vital factor.
22. Which of the following, as per the passage, is an example of a 'diverse
population'?
(a) A group of scientists trying to achieve a breakthrough in a technology.
(b) A group of theorists trying to debate the validity of their findings.
(c) A group of people from eclectic fields trying to propagate a law that protects
women.
(d) A group of teenagers enjoying a school camp.
23. Which of the following best describes the architecture of natural selection?
(a) It is an entity which can neither be controlled nor can be channelized.
(b) It is a philosophy which integrates within itself the laws and theories of
natural sciences.
(c) It is a deterministic philosophy whereby moral end goals can fit into
biological ones.
(d) It is a philosophy based on directional and adaptive changes; the process of
non-random elimination.
24. The theory of natural selection provided an important shift in existential
perception. It can be characterised as
(a) Individual as a whole. (b) Aesthetic to practical.
(c) Reductionist to genetic. (d) Chaotic to determined.
25. Darwin’s theory shaped the philosophy of chance or necessity as a
(a) homogenous concept. (b) abstract conception.
(c) duality of natural selection. (d) temporal feature.
26. A significant shift in our interaction with lifeless entities through theory of
evolution is:
(a) the perception that as long as species evolves, its natural surroundings also
evolve.
(b) the notion that existence on earth is a process which governs both living and
non-living beings.
(c) the understanding that rocks and stones also evolve following the physical
laws.
(d) the knowledge that additional factors control the living beings.
27. What do you think is the essence – either positive or negative - of the shift
between governing biological factors
being laws to governing factors being concepts?
(a) Laws are more stable and thus helpful in the long run.
(b) Concepts are not fixed and thus variety arises out of them.
(c) Laws can never be taken out of context and random species functions become
redundant.
(d) Concepts can be manipulated for ulterior motives.
Passage (Q.28-Q.30): Amid all the hubbub a few years ago about the Existentialist
movement in France, it
seems that nobody, not even the Existentialists themselves, took the trouble to
make one cardinal point that
would have cleared up a great deal of misunderstanding. Yet this point is a very
simple one, so simple that it is
surprising that it got lost in the scuffle. It is nothing less than the fact that
Existentialism is not a philosophy at
all—at least not the kind of philosophy that should have stirred the professional
contentiousness of the various
philosophic schools now current in America. This does not mean that Existentialism
is merely a brand of
impassioned rhetoric, which it may have been in some of its adherents, or merely a
new literary genre—perhaps,
worst of all, only a clumsy effort at poetry. On the contrary, it has a very good
right to the name “philosophy,”
almost, we might say, a right of primogeniture. Its aim, in fact, is nothing less
than to restore to this name its
ancient and primitive meaning, a meaning which covers much of the territory we
moderns assign to religion, and
one which centuries of specialized learning have obscured. Unless we understand
this point, we shall not
understand what is at the center of Existentialism, and, therefore, shall not see
how it is related to our time and
what it can hope to do for us.

. Page 8 of 36
Nietzsche once observed, in one of those passing pregnant asides that constitute
the real richness of his work,
that the achievement of a philosophy was an altogether different business for the
modern European (of
Nietzsche’s time) than it was for ancient man or even for the modern Oriental. When
we speak of a man as
having a “philosophy,” we tend immediately to think of this as something he has to
tell us, to convey verbally,
rather than as a truth that has been lived. A philosophy, thus, is taken as a
system of propositions held to be true
on purely intellectual or rational grounds. But for the ancients, and for the
Orientals, the business of achieving a
philosophy was one that engaged the whole man, his total being, and was not pursued
simply as one specialized
department of knowledge among others. In the spirit of Nietzsche’s remark,
Kierkegaard had attacked the
Hegelian professors of his time as being philosophers without any real philosophic
existence: they had a system
of propositions to teach but the system itself was a means of forgetting the
concrete realities of human life.
28. Which of the following can be inferred from the given passage following the
footsteps of ancient philosophers?
(a) Existentialism has suffered a setback with the rise of American school of
philosophy.
(b) The core essence of a philosophy should engage the whole man, his association
with the world.
(c) A philosophy fails to express the real problem which modern man is feeling.
(d) Existentialism has always been the primitive source for understanding
philosophy.
29. Which of the following justifies Nietzsche’s observation in case of philosophy?
(a) Idea of philosophy has altered over time.
(b) Nietzsche’s philosophy clashed with the ideas of the ancient philosophers.
(c) A philosophy should not be mere rational conjectures, but a concrete reality of
a man’s existence.
(d) Concept of man hasn’t changed over time.
30. Which of the following should be the aim of modern existentialism?
(a) The philosophers should not look back in time; instead, alter the
Existentialist theories with respect to modern
man.
(b) To restore its ancient and primitive meaning, which covers much of the
territory the moderns assign to
religion.
(c) It’s important to go through the existentialist philosophies of the past before
imposing it on modern
philosophers.
(d) Modern philosophers should preach the ancient theory of existentialism in the
modern times

Directions (Q.66 – Q.105): Read the comprehensions carefully and answer the
questions based on it.
Passage (Q.66-Q.71):The Supreme Court on Thursday rapped Gujarat Government for
issuing a notification
dated October 29, 2021 as per which a Scrutiny Committee was constituted for
granting ex-gratia compensation
to COVID victims. The Court observed that the constitution of the scrutiny
committee was an attempt to
overreach the directions in the judgment delivered, where a slew of directions were
issued regarding the
procedure for issue of death certificates and disbursal of compensation to COVID
victims. The bench in their
order said, "It appears that an attempt has been made to overreach the directions
issued by this Court." The bench
also directed petitioner's counsel to serve a copy of the application to the
Solicitor General to enable him to take
remedial steps and come out with a clear and simplified formula/notification so
that the amount of compensation
as ordered by the Court is paid to the victim at the earliest and without any
further harassment. Adding that the
directions issued by the Court for payment of compensation to the family members of
the persons who died due
to COVID-19 were very clear, the bench observed that there was no requirement at
all of constituting the Scrutiny
Committee for the purpose of awarding the compensation. In the October 4 judgment,
the Supreme Court had
approved the ex-gratia compensation of Rs.50,000 for the kin of COVID victims as
recommended by the
National Disaster Management Authority.
Source Name - Live Law
66. Scrutiny committee was constituted for granting ex-gratia compensation to covid
victims. However there was a
set of direction of the Supreme Court construing to the fact that there will be no
scrutiny committee for the
dispersal of compensation to the same. The Supreme Court rapped the State
Government for the same. In the
light of the passage choose the correct Option.
(a) The Supreme Court was not justified because there was a need of scrutiny
committee for the dispersal of
compensation to covid victims.
(b) There was no need of scrutiny committee at all.
(c) The court directed that there will be a scrutiny committee that will keep an
eye on the dispersal of
compensation to covid victims.
(d) The court created a set of direction including that there will be no scrutiny
committee for the disbursement
of the compensation to the covid-19 victims.
67. The state government constituted a scrutiny committee to keep an eye on the
disbursement of the compensation
to the covid victims. However the court taking note of the same declared that it
had never set the directions to
constitute the scrutiny committee for the same and it was also clarified that the
constitution of scrutiny committee
was an attempt to overreach the directions of the judgement by the court. In the
light of the passage choose the
correct Option.
(a) The court was justified because it had set the orders for other committees and
not the scrutiny committee.
(b) The court was not justified for deciding the same because there was a need of
the scrutiny committee for the
disbursement of the compensation to the covid-19.
(c) The court was justified for desire in the same.
(d) As there was an attempt to overreach the judgment of the Supreme Court the
court was justified for deciding
the same.

. Page 15 of 36
68. There was a scrutiny committee that was constituted by the State Government for
granting ex gratia
compensation to covid victims. The court observed that constitution of scrutiny
committee was an attempt to
overreach the directions in the judgement delivered by the Supreme Court. However
Supreme Court did not
explicitly said that there was no requirement at all of constituting the scrutiny
committee for the purpose of
awarding the compensation. In the light of the passage choose the correct Option.
(a) As the court did not explicitly clarified the same the court was not justified
for deciding the same.
(b) According to the passage is the Supreme Court was justified for deciding the
same.
(c) As there was no attempt to overreach the judgement of the Supreme Court the
Supreme Court was justified
for deciding the same.
(d) The Supreme Court was not justified for deciding in the same.
69. It was alleged by the state government that the court gave the direction for
constitution of the scrutiny committee
for granting the compensation to the covid-19 outbreak. However, it was clarified
by the court that there was no
need for the constitution of scrutiny committee for granting the ex-gratia
compensation to the victims of the
covid. In the light of the passage is the correct Option.
(a) As it was declared by the court that there was an important factor for
constitution of scrutiny committee the
court was not justified for deciding the same.
(b) As the court itself declared that it was important for constituting a scrutiny
committee the court was not
justified for deciding the same further.
(c) The Supreme Court clearly declared that there was no need of constituting the
scrutiny committee so the
court was justified in the same.
(d) There was a need of scrutiny committee by the state government so the court was
not justified.
70. In the light of the passage choose the correct Option.
(a) The Supreme Court clearly observed that there was a need of constitution of
scrutiny committee by the state
government.
(b) There was no need of scrutiny committee as declared by the court.
(c) As it was the matter of disbursement of the compensation to the covid-19
victims it was important to
constitute a scrutiny committee.
(d) State government has no right to constitute a scrutiny committee for the
process of disbursement of
compensation to covid-19 victims.
71. In the light of the passage choose the misinterpreted Option.
(a) There was no need of constituting a scrutiny committee for disbursement of
compensation to covid-19
victims.
(b) There was a need for constituting scrutiny committee for keeping an eye on the
disbursement of
compensation to covid-19 victims.
(c) The court clearly observed that there was no need of constituting scrutiny
committee for covid-19 victims’
compensation process.
(d) The court was not happy with the decision of the state government regarding
constitution of scrutiny
committee in the process of compensation granting to covid-19 victims.

. Page 16 of 36
Passage (Q.72-Q.76): Negligence is a failure to behave with the level of care that
someone of ordinary
prudence would have exercised under the same circumstances. The behavior usually
consists of actions, but can
also consist of omissions when there is some duty to act (e.g., a duty to help
victims of one's previous
conduct). Primary factors to consider in ascertaining whether the person's conduct
lacks reasonable care are the
foreseeable likelihood that the person's conduct will result in harm, the
foreseeable severity of any harm that
may ensue, and the burden of precautions to eliminate or reduce the risk of harm.
Four elements are required to
establish a prima facie case of negligence:
1. The existence of a legal duty that the defendant owed to the plaintiff
2. defendant's breach of that duty
3. plaintiff's sufferance of an injury
4. proof that defendant's breach caused the injury (typically defined through
proximate cause)
Typically, if the defendant had a duty to act, did not act (resulting in a breach),
and that breach caused an injury,
then the defendant's actions will be classified as misfeasance. There are several
ways to determine whether the
defendant had a duty to act (note: this is NOT an exhaustive list):
1. The defendant engaged in the creation of the risk which resulted in the
plaintiff's harm
2. Voluntary undertaking: The defendant volunteered to protect the plaintiff from
harm
3. Knowledge: The defendant knows/should know that his conduct will harm the
plaintiff
4. Business/voluntary relationships: ex: business owner and customer; innkeeper and
guest; land possessor who
opens her land to the public; person who voluntarily takes custody of another
person
Contributory negligence is the plaintiff's failure to exercise reasonable care for
their safety. In such a case there
will be no liability on the defendant.
72. Zoe, parked her car on the side of a busy street, took her iPod and whilst
being completely engrossed in the
iPod, trying to choose the rigt song she started crossing the road on foot. There
were no traffic lights in the
vicinity which were to be followed. A car, being driven by Aman, hit Zoe and Zoe
got injured. Zoe sued Aman
for compensation. Decide Aman's liability.
(a) Aman is liable to pay the compensation as he drove negligently and hit Zoe.
(b) Aman is liable to pay the compensation as he did not fulfil his responsibility
to drive carefully.
(c) Aman is not liable because Zoe's negligence contributed to the injuries.
(d) Aman is not liable because he was not responsible to look out for Zoe in a
place where there was no path for
crossing for pedestrians.
73. A pedestrian, Slimey, wrongly parked his car on the side of a busy street, and
started crossing the road on foot
when the pedestrian light was green. A car, being driven by Andre, hit Slimey and
Slimey got injured. Slimey
sued Andre for compensation. Decide his liability.
(a) Andre is liable to pay the compensation as he did not fulfil his responsibility
of driving carefully and hit Z.
(b) Andre is liable to pay the compensation as he was aware that his actions could
cause such injuries but still
went ahead and hit Slimey intentionally.
(c) Andre is not liable because Slimey's negligence contributed to the injuries.
(d) Andre is not liable as Slimey should have waited for him to go.
74. B, while riding his cycle on the road, met with an accident, when A, accidently
opened his car door to exit the
vehicle without ensuring no one is around, hitting the cycle in the process. B sued
A for compensation. Decide.
(a) A is liable to pay the compensation as his negligence caused the accident. He
was not careful enough to
check before opening the door.
(b) A is not liable to pay the compensation as he was in a hurry.
(c) A is not liable because Z's negligence contributed to the accident.
(d) A is liable as he should have known that opening the car door without checking
first, can injure someone.

. Page 17 of 36
75. An inebriated man, Zo, was walking on the train tracks, when a train arrived on
the tracks, causing Zo's death.
A guard, appointed by the railway authorities, was on duty at the time of the
incident, but failed to notice A
walking on the tracks and was therefore unable to save him. Zo's family sued the
railway authorities. Decide.
(a) The railway authorities are liable to pay the compensation because it was the
guard's negligence which
caused the accident.
(b) The railway authorities are liable to pay the compensation because the guard
did not fulfil his duties.
(c) The railway authorities are not liable to pay the compensation because Zo also
acted negligently.
(d) Zo cannot claim compensation in this situation as this is a case of Volenti Non
Fit Injuria..
76. The term rule of the last opportunity means the last opportunity to avoid an
accident. If in a situation both the
plaintiff and the defendant are negligent on their part and whosoever has the last
opportunity of avoiding such
consequences fails to do so will be held responsible for such accident solely. Z,
while visiting a friend, callously
parked his bike outside the friend's house, which was on a busy main street. A
while driving his car on the said
road at a high speed, hit Z's bike. Z sued A for compensation equivalent to the
damages caused to his vehicle.
Decide.
(a) A is liable to pay the compensation as he was riding at a high speed on the
main street carelessly.
(b) A is liable to pay the compensation as there was contributory negligence.
(c) A is not liable as Z should not have parked the vehicle on the road.
(d) A is not liable as he couldn’t see Z’s bike.
Passage (Q.77-Q.80): A plethora of cases have cropped up across High Courts on
whether certain online games
amount to online gambling. The courts, while distinguishing online gaming from
gambling, have dissected
whether the game in question is a game of chance or one of skill.
The Kerala high Court recently heard a batch of petitions challenging the State
government's ban on online
rummy. It ruled that the notification banning a game when played for stakes is
arbitrary, violative of Articles 14
and 19(1)(g) and therefore unconstitutional, since the element of skill is
predominant for winning such games.
It was also held by the Court that a notification in relation to a game which is
explicitly exempted from the
provisions of the Kerala Gaming Act under Section 14, cannot be curtailed. In
addition to this, it was noted that
Online Rummy does not come within the purview of 'gambling' or 'gaming' and
providing a platform for playing
the game cannot be restricted.
The Madras High Court most recently rendered Part II of the Tamil Nadu Gaming &
Police Laws (Amendment)
Act, 2021, which imposed a blanket ban on online games like Online Rummy and Online
Poker, unconstitutional.
The said part of the Amendment specifically banned betting or wagering in
cyberspaces and also games of skill
if played for a wager, bet, money or other stakes. The Bench ruled that imposing a
complete ban would be
violative of the ‘least intrusive test’ and thereby in conflict with Article 19 (1)
(g) of the Constitution (right to
practice any profession or to carry on any occupation, trade or business).
77. The state of Humararashatra brought a law called Humararashatra prohibition of
obscene dance in Hotels,
Restaurant and Bar Rooms under the Protection of Dignity of Women (Working therein)
Act, 2016 which
completely prohibits any kind of dance performance in an eating house, room or beer
bars due to which many
local bar dancers lost their livelihood and became unemployed. The bar dancers
challenged the law before the
court stating it be violative of article 19 (1) (g). Decide?
(a) The act violates right to practice any profession of choice of bar dancer.
(b) The act does not violate right to practice any profession of the bar dancers as
it ensures standards of decency.
(c) The state actions are arbitrary and would be violative of the ‘least intrusive
test’.
(d) Both A and C.

. Page 18 of 36
78. During the unprecedented times of covid, to keep the sport spirits of students
alive and keep them busy, the
educational institutional association came up with an app to organize a virtual
sport meets of the different schools
that includes different online games based on the skills of the students, and the
participants can also win money
however the Tamil Nadu Gaming & Police Laws (Amendment) Act, 2021, bans such online
games decide
whether the ban is reasonable or not?
(a) The ban is reasonable as the game involves monetary compensation.
(b) The ban is not reasonable as the involvement of money does not amount to
wagering.
(c) The ban is reasonable as it amounts to wagering in the cyberspace.
(d) The ban is not reasonable as the game is primarily based on skill.
79. In the season of IPL, Mr. Makhija and Mr. Chindi developed an online Fantasy
Sports Game in which one can
create a fantasy sports team based on a real-life players of upcoming IPL matches
and can participate in contests
and the participants win cash prizes based on the players performance in the real
life match. However the Tamil
Nadu Gaming & Police Laws (Amendment) Act, 2021, also bans such online games decide
whether the ban is
reasonable or not?
(a) The ban is reasonable as the games involve money .
(b) The ban is not reasonable as involvement of money does not amount to wagering.
(c) The ban is reasonable as it amounts to wagering in the cyberspace.
(d) The ban is not reasonable as game is primarily based on the skill.
80. “Ludo” is a very popular gaming app of the decade and with the introduction of
actual monetary rewards it has
reached to another level of popularity among the people. Basically it is based on
the board game of “ludo”
however the Tamil Nadu Gaming & Police Laws (Amendment) Act, 2021, also bans such
online games decide
whether the ban is reasonable or not?
(a) The ban is reasonable as the games involve monetary rewards.
(b) The ban is not reasonable as involvement of money does not amount to wagering.
(c) The ban is reasonable as it amounts to wagering in the cyberspace.
(d) The ban is not reasonable as game is primarily based on the skill.
Passage (Q.81-Q.86): The Supreme Court on Thursday enquired the whereabouts of
former Mumbai Police
Commissioner Param Bir Singh, who was recently declared as an absconder by a Mumbai
Magistrate after his
non-appearance in an extortion case. A bench comprising Justices Sanjay Kishan Kaul
and MM Sundresh was
considering the petition filed by Param Bir Singh seeking protection in the cases
of corruption and extortion filed
against him. "Where are you? Are you in the country? Outside the country", Justice
Kaul asked Senior Advocate
Puneet Bail who appeared for Singh, as soon as the matter was taken. The senior
counsel replied that the queries
can be answered only by the filing counsel. The Advocate-on-Record told the bench
that the petitions have been
filed by the power-of-attorney holder of Singh. "You have not joined any
investigation. If you are sitting abroad
and approaching the Court...Our suspicion might be wrong...if the Court gives a
favourable order only then he'll
come back. It might be so", Justice Kaul observed. The bench said that no
protection can be granted until his
whereabouts are known. "No protection, no hearing till we have the answer to the
question - where are you?"
Justice Kaul said. The counsel then sought for time till Monday to get
instructions. The petition was filed against
the September 16 judgment of the Bombay High Court which dismissed as not
maintainable the petitions filed
by him challenging the two enquiries orders issued by the State Home Ministry for
allegedly violative service
rules and the second over allegations of corruption.
Source Name - Live Law

. Page 19 of 36
81. P was a police commissioner of the state. He was accused of case regarding
extortion. However he was
absconding and no one knew his whereabouts. However the court denied granting
protection to P unless his
whereabouts were known to the court. In the light of the above passage was a
correct option.
(a) The court was right in its position as according to the passage it is evident
that a person having no clear
whereabouts known to Court cannot be granted protection.
(b) The court was justified in giving protection to the police commissioner.
(c) The court was not justified for deciding the same.
(d) The court was justified for deciding the same but the police commissioner was
not absconding.
82. D was a police commissioner who was stuck in a case regarding the extortion.
However he was running away
from the proceeding. However the court granted the protection to the police
commissioner without knowing his
whereabouts. In the light of the above passage choosethe correct option.
(a) The court was justified for granting him the protection.
(b) The Court was not justified for granting him the protection.
(c) As his whereabouts were not known it was important for the protection to be
granted to him.
(d) According to the passage the court should not have granted protection to him.
83. S was a police commissioner of the state who was alleged for extortion in a
case. His council pleaded for
protection for the police commissioner. However the court observed that the court
did not know his whereabouts
and the court denied him giving protection at any cost unless his whereabouts are
known. In the light of the
above passage choose the correct option.
(a) The court was right in its approach.
(b) As it was necessary for the court to know his whereabouts the court was
justified for denying the same.
(c) Ashe was not absconding so the court was not justified.
(d) As he was a police commissioner he should be granted protection and the court
was not justified.
84. P was a police commissioner who was alleged to be involved in extortion and the
proceeding was to be held
against him. However any whereabouts of him were not known. As he was absconding,
the counsel pleaded for
the police commissioner infront of the court that he should be granted protection
so that he can take part in the
proceeding. However the court did not grant the protection until his whereabouts
are known to the court. In the
light of the above passage choose the correct option.
(a) As he was a police commissioner he should have been granted the protection.
(b) As no whereabouts of him were known the Court was justified in denying him
protection.
(c) The court was wrong in its decision.
(d) The court was absolutely right in its decision.
85. In the light of the passage choose the correct option.
(a) Whereabouts of a person is one of the important factors taken into account
before being granted protection.
(b) Whereabouts of the person is not an important factor for giving protection.
(c) The court has no power to deny protection to a police commissioner.
(d) Every person has a right to protection and the court was not justified in the
passage.
86. In the light of the passage choose the misinterpreted option.
(a) Whereabouts of a person is not an important factor for being granted
protection.
(b) Whereabouts of a person is required by the court to grant him protection.
(c) Court has the right to deny granting protection to the accused person.
(d) Whereabouts of a police commissioner is also important for granting him the
protection by the court.

. Page 20 of 36
Passage (Q.87-Q.91): Intoxication is a state of mind in which a person loses self-
control and his ability to judge.
Intoxication is a defense available to criminal defendant on the basis that,
because of the intoxication, the
defendant did not understand the nature of his/her actions or know what he/she was
doing. The defense of
intoxication typically depends on whether the intoxication was voluntary or
involuntary and what level of intent
is required by the criminal charge, it is involuntary when the person suffering
from it has had the intoxication
administered to him either (a) by force (against his will), or (b) by fraud
(without his knowledge). The expression
'without his knowledge 'means ignorance of the fact that what is being administered
to him is or contains or is
mixed with an intoxicant. An act which is against the 'will' must be an act which
the performer performed not
out of his own conscious volition but on the compulsion by some outside agency by
overpowering or paralyzing
his will by overt physical acts.
Under the Indian Penal Code the criminal liability under intoxication is mentioned
under section 85 and 86.
Section 85 deals with offences committed under the influence of drugs or alcohol
which is caused by fraud or
coercion. Section 86 deals with intoxication which is self-induced. Bablu alias
MubarikHussain V. State of
Rajasthan, in this case SC examined Section 85 of IPC and held that evidence of
drunkenness, the evidence
which proves that the accused is incapable of forming the wrongful intent has also
been considered along with
the other facts, and then it should be proved of the accused person has the
intention to commit crime. These
sections do not protect someone who voluntarily consumed intoxicants as the person
loses his mental ability
because of his consensual act i.e. by self-induced intoxication.
87. A went to party with his friends. A was a teetotaler. B, C and D who attended
party with A asked him to drink
alcohol. A rejected the offer. C made fun of him and started shouting, “A was a
looser and was not bold enough
to try alcohol.” B, C and D persuaded him to drink alcohol to prove that he is not
a looser.. A accepted under
the condition that he would not drink more than a glass. When A was about to drink
alcohol his girlfriend E
came to the party. A went to talk to her. Disappointed with the missed opportunity
to get A drunk C and D mixed
strong whiskey to A's bottle of soft drink. A drunk it and later attacked E under
the influence of alcohol. Decide:
(a) Defense of intoxication is not available to A as he had accepted to try one
glass of alcohol.
(b) Defense of intoxication is available to A as whiskey was administered without
his knowledge and consent.
(c) No one shall be held guilty as it was a case of misunderstanding between A and
his friends as their challenge
was abruptly cut short due to the arrival of E.
(d) Defense of intoxication is available to A as he had not accepted to try strong
whiskey but only the simple
alcohol.
88. A was a police officer. A along with B was consuming alcohol while on night
duty around the city. A and B
came across D who was escaping with valuables stolen from E's house. A, instead of
chasing to stop D, fired 2
rounds of bullet towards D. Under the intoxication A had confused D with E who was
chasing to stop D. E
succumbed to the injuries. Decide
(a) A shall not be liable as he was a police officer and is protected under state
sovereignty.
(b) The defense of intoxication is available to A, as he was unaware about his
possible encounter with a robber.
(c) The defense of intoxication is not available to A, as he voluntarily consumed
alcohol with his own free will
and consent while going for night rounds.
(d) The defense of intoxication is not available to A, as he is a police officer
and should have been more
responsible while going on rounds.

. Page 21 of 36
89. Z prepared a new alcohol-based drug which could intoxicate a person for up to 5
hours. Z expected it to be a
huge success amongst drug users. To test its effect, he forcibly made his neighbor
Y, consume it saying it was a
health therapy for back spasm. Y consumed it and thereafter assaulted his wife.
Decide:
(a) Y shall be held liable, as he is a sane adult and has enough maturity not to
get tricked.
(b) Y could plead the defense of intoxication as it was administered forcibly
without his consent and true
knowledge about its ingredients.
(c) Y cannot plead the defense of intoxication as the act was committed 3 hours
after the administration of the
drug.
(d) The defense of intoxication under section 85 cannot be pleaded as it is limited
only to alcohol and not for
other drug consumptions.
90. R was an alcoholic. One night when R came home drunk, his wife served him
dinner. After finishing the dinner
when he was about to leave, his wife handed him over a list of vegetables to be
bought for the next day. In a fit
of anger, he took kerosene and poured it over his wife and lighted her. After
hearing her scream he tried to
extinguish the fire with a blanket. R was charged under section 304 of IPC. He
pleaded the defense of
intoxication. Decide.
(a) R can claim the defence as he was in an intoxicated state.
(b) R cannot claim the defence as he knowingly poured kerosene on his wife.
(c) R cannot claim the defence as he was willfully intoxicated and was in a state
to understand the consequences
of his act.
(d) R is guilty of murder and should be hanged.
91. M was a retired army official who came back home in an intoxicated state after
attending a late-night party
withhis friends and drinking a lot of alcohol. While entering his home, he saw a
robber trying to steal his fridge.
He immediately took his baseball bat and bashed him. The next morning, he saw his
daughter lying dead near
the fridge. He pleaded the defence of intoxication. Decide.
(a) M can plead the defense as he was under the illusion that the robber was trying
to steal his fridge. He bashed
him to protect his property.
(b) M is liable as he was under voluntary intoxication, and cannot plead the
defense of intoxication.
(c) M is not liable as he was not in a state of mind to understand the nature of
his acts.
(d) M is liable under section 85 of IPC as he willfully consumed alcohol.
Passage (Q.92-Q.95): The Kerala High Court has held that if a party enters into a
second marriage when the
appeal of the decree of divorce of the first marriage is still pending, he/she will
not be guilty of the offence of
bigamy under Section 494 of the Indian Penal Code if the appeal is subsequently
dismissed. While allowing a
petition filed under Section 482 of the Code of Criminal Procedure to quash the
complaint alleging bigamy, the
Court ruled that the Hindu Marriage Act does not override the right of appeal.
Upon exploring the offence of bigamy, the Court laid down the statutory pre-
requisites essential to constitute the
offence: (i) the accused must have contracted first marriage (ii) he must have
married again (iii) the first marriage
must be subsisting (iv) the spouse must be living. Further, the second marriage
should be void by reason of it
taking place during the lifetime of the first husband or wife.
The Court also noted that amended Section 15 of the Hindu Marriage Act deals with
the stage in which a divorced
person can validly enter into a second marriage. It says that after the decree of
dissolving the marriage, either
there is no right of appeal or if there is such a right, the time for appealing has
expired without an appeal having
been presented or an appeal has been presented, but has been dismissed
subsequently, it shall be lawful either of
the party to the marriage to marry again. In the instant case, although the husband
had entered into a second
marriage during the stay order, the appeal ended in dismissal subsequently
confirming the decree of divorce.
[Excerpt from: Live Law, Hannah M Varghese, September 28, 2021
https://www.livelaw.in/news#updates/section-494-ipc-second-marriage-while-divorce-
decree-was-stayed-appeal-dismissed-later-no#offence-bigamy-kerala-high-court-
182587]

. Page 22 of 36
92. Mr. Radhe, a male Hindu, 21-years of age, resident of Surat (Gujarat) married
Ms. Kishori, a female Hindu.
After 8 years of marriage, the couple had difficulty in having an offspring due to
medical reasons and the wife
filed for a divorce. The same was granted on July 8, 2021 on grounds of medical
reason. On August 7, 2021 Mr.
Radhe decided to marry Ms. Meher, a close friend of his for 6 years. They also used
to meet in hotels and go on
vacations during the marriage of Mr. Radhe and Ms. Kishori. But their marriage was
conducted with all the
customary rites and practices and was solemnized on August 25, 2021.
(a) The second marriage is valid.
(b) The second marriage is void.
(c) Mr. Radhe has committed the offence of Bigamy as they had an affair while the
marriage was subsisting.
(d) The first marriage is not valid.
93. Mr. Ravi and Ms. Shama solemnized their marriage on February 28, 2019 under the
Hindu Marriage Act 1955
following all customary rites and rituals. Unfortunately, the Husband developed a
venereal disease which is a
valid ground for divorce under the said act. The wife filed the petition on August
18, 2021 to get a divorce and
dissolve the marriage. Ms. Yami who was immune from the disease Mr. Ravi had due to
certain natural abilities
by birth met Mr. Ravi and they fell in love. They married on October 4, 2021 under
the Hindu Marriage Act
1955 following all customary rites and rituals. Choose the most appropriate option
(a) Disease shall not be any grounds for divorce and no dissolution of marriage
shall take place
(b) The second marriage is valid as a petition for divorce has been filed
(c) Mr. Ravi has committed the offence of bigamy.
(d) The fate of the petition shall decide the offence of bigamy under Section 494
of the IPC.
94. Ms. Seema and Mr. Pranjal solemnized their marriage on January 4, 2020 under
the Hindu Marriage Act 1955
following all customary rites and rituals. Subsequently the family of the
bridegroom started to abuse and cause
mental agony to the bride due to insufficient dowry. This being a valid ground for
divorce, Ms. Seema filed for
a petition and the same was granted on July 5, 2021 making the marriage stand
dissolved. Mr. Pankaj filed an
appeal in accordance with the procedural law on September 4, 2021 by contending
that it was his family which
caused the mental agony and not himself and the decree of divorce shall be
reversed. Subsequently, he had been
attracted towards his colleague who also liked Mr. Pankaj and they solemnized their
marriage under the Hindu
Marriage Act 1955 following all customary rites and rituals on October 15, 2021.
Choose the most appropriate
option –
(a) The second marriage is void by the fact of first marriage being valid as the
petition is still pending.
(b) The first marriage is dissolved.
(c) The fate of the petition shall decide the offence of bigamy under Section 494
of the IPC.
(d) Mr. Pankaj has committed the offence of bigamy under Section 494 of the IPC.
95. What will logically be the consequence if the appeal against the decree of
divorce succeeds in a case involving
a second marriage?
(a) The second marriage remains valid. (b) The offence of bigamy is attracted.
(c) The second marriage is dissolved. (d) The first marriage remains dissolved.
. Page 23 of 36
Passage (Q.96-Q.100): Section 499 of the IPC defines what amounts to criminal
defamation and few subsequent
provisions specify what the punishment for having committed defamation would be.
Section 499 states
defamation could be through words – spoken or intended to be read, through signs,
and also through visible
representations. These can either be published or spoken about a person with the
intention of damaging reputation
of that person, or with the knowledge or reason to believe that the imputation will
harm his reputation. Section
500 stipulates an imprisonment of up to two years, with or without fine, for
someone held guilty of criminal
defamation. However, criminal defamation is a compoundable offence and parties can
seek a closure of the case
by reaching a compromise.
Defamation can take two forms – libel (by writings) and slander (by spoken words).
In order to establish that a
particular statement – written or spoken – is defamatory, it must be proved that it
is false, defamatory and
published and lowered the reputation of a specific person or an identifiable set of
people in the eyes of the general
public.
Truth is generally considered to be a defense to defamation as a civil offence but
under criminal law, truth is not
a defense except in a limited number of circumstances. Besides the statement or
writing being demonstrably true,
it also requires to be proved that the imputation was made for public good.
Accusations, censure or imputation
made in good faith by persons having lawful authority are also a few exceptions to
a charge of criminal
defamation.
96. Two employees in an office wanted to poke fun at their colleague and began to
reveal private messages he had
sent to other female colleagues in the office. He wished to file a case against
them for criminal defamation. The
two defendents are claiming the defense of the publication being true. Will he
succeed in filing a case for criminal
defamation?
(a) Yes, as their publication was true but did not involve public good..
(b) Yes, such a communication brought him great shame in his workplace.
(c) No, since the publication was not false but of a matter that happened.
(d) No, since the employees only wished to poke fun and not injure his reputation.
97. The Chief Minister was involved in some shady businesses and wasregularly
criticized by the people for his
strange policies. One such journalist published a report of the CM being involved
in a corruption racket with
evidences he collected from a sting operation.. This report was cited by many,
including the opposition, and led
to a huge fall in support. Will a case of criminal defamation succeed against the
journalist?
(a) No, he cited the truth in public interest only.
(b) Yes, the report injured his reputation.
(c) No, the statements he made were true.
(d) Yes, the journalist published the same with the intent to injure his
reputation.
98. A college student while browsing online came across disparaging information
about a fellow student from his
class. He circulated the same among his friends and soon it became viral. He later
came to know that the
information was fake. The victim filed for defamation under Section 499 IPC and
won. Was the court, right?
(a) Yes, he found the information with the intent to harm the reputation of the
victim.
(b) Yes, he published information which was neither true nor made for public good
with the intention to cause
harm to his reputation.
(c) No, as he only found and circulated it but did not make the statement.
(d) No, he only circulated among his friends and did not intend for it to go viral.

. Page 24 of 36
99. A letter written in Urdu was sent to B by C. Therefore, he needed another
person to read it to him. The letter
contained derogatory remarks about B. A knew that B would need a translator to
comprehend the letter. Decide.
(a) C is not liable as he only knew how to write in Urdu.
(b) C is liable as he was aware that B could not understand Urdu and would require
help of someone else.
(c) B is liable as he asked someone else to read the letter.
(d) C is liable as he made defamatory statements with full intention and knowledge
that were published as a third
person read the same.
100. The defendants published an article stating that ‘Harold Limestone, a
Camberwell man had been convicted of
bigamy’. The story was true of Harold Limestone, a Camberwell barman. The action
for defamation was brought
by another Harold Limestone, the barber. Decide.
(a) The defendants are liable for libel.
(b) The defendants are not liable for defamation as they did not intend to harm the
reputation of the barber.
(c) The defendants are not liable as they have freedom of speech and expression.
(d) The defendants are liable as the words were considered to be understood as
referring to the plaintiff.
Passage (Q.101-Q.105): The couple has questioned a 2015 notification by which the
Director-General of
Foreign Trade moved the import of human embryos from the 'restricted' category to
the 'prohibited' category
except for scientific research.
In their petition under Article 226, the couple has questioned notification by
which the import of human embryos
was prohibited. The classification in the 2015 notification creates a blanket ban
which is arbitrary, irrational, and
violative of Article 21 of the Constitution. The citizens' right to have their
biological child, as per their desire
and cannot be curtailed or violated by executive fiat. Such reasonable expectation
is not illegal or immoral.
The petitioners have claimed that perusal of both the Bills, i.e., the Surrogacy
(Regulation) Bill 2019 and the
Assisted Reproductive Technology (Regulation) Bill 2020, do not prohibit surrogacy.
However, none of them
clarifies the aspect where the embryo is cultivated out of the couple's sperm and
ovary for use in the near future.
Lastly, the couple contends that the right to have their own children is a part of
the Fundamental Right to live a
meaningful life. However, their fundamental rights are being "trampled" upon by the
various Government
Authorities by not issuing No Objection Certificates.
"Every woman has the right to have her own child either naturally or through IVF or
through surrogacy. By
imposing a ban on having a child, is violating the Petitioners Fundamental Rights
and Constitutional
Guarantees." However, in India only altruistic surrogacy is permitted in India, not
commercial surrogacy.
101. Svetlana and Boris were a Russian couple looking to have a child but were
unable to have one due to infertility.
They were looking into IVF clinics but were unable to find any authentic ones in
Russia due to their stringent
laws. They started looking into Indian clinics and found good options. They had
their frozen embryos saved in
the fertility banks and finally decided to take them to India for fertilization.
However, due to the high tariff rate
imposed by the Indian government, they were having a hard time paying the same.
They contested for relief in
the Indian SC under art. 226. Would it sustain?
(a) No, as there is no violation of their fundamental rights here.
(b) Yes, due to the high tariffs, they were being deprived of having a child.
(c) No, as there exists no right in law for them to plead to the Indian courts for
relief.
(d) Yes, as they are entitled to become parents under the Indian Constitution.
102. In the above matter, other facts notwithstanding, the issue arose to be that
the Indian government had imposed a
blanket ban on the import of cryo-embryos due to the unavailability of safe IVF
facilities in India. This was
contested by the couple as being in violation of their FRs. Are they liable for
relief?
(a) No, as the restrictions placed are via the law and competent authorities.
(b) Yes, as the restrictions placed are arbitrary and unjust in every manner.
(c) Yes, as the SC is competent to provide them with relief as under the
Constitution of India.
(d) No, as the restrictions placed are not arbitrary or irrational.

. Page 25 of 36
103. Due to the unavailability of the IVF facilities in India, the couple decided
to choose Austria as the alternative
destination, where the IVF facilities did not carry a lot of tariffs due to not
having a stable government. This
factor did not turn out favorable for them due to the arbitrary pricing and
facilities extended by the IVF clinics
there. They decided to take action against the same but were aware that Austria did
not have a constitution.
Would they be liable to receive any relief from the Judiciary there?
(a) No, as the lack of a valid constitution deprived them of their rights.
(b) No, as FRs need to be guaranteed by a written constitution and a valid
government.
(c) Yes, as the FRs of a person cannot be deprived due to an unwritten
constitution.
(d) Cannot be determined. .
104. Mike and Amanda are an infertile couple looking for a surrogate mother. They
decided to ask Mike's step-sister,
Phoebe, who is a masseuse. Phoebe agrees to do so.. Can this arrangement be held
legal?
(a) Yes, as this would be termed charitable surrogacy, which is not prohibited in
India.
(b) No, as this would be termed commercial surrogacy, which is prohibited in India.
(c) No, because the agreement would be considered incestuous as sexual relations
between a brother and sister
is illegal.
(d) None of the above.
105. Which of the following articles involve invoking the jurisdiction and pleading
for the protection of Fundamental
Rights to the High Courts?
(a) Article 227 (b) Article 226 (c) Article 32 (d) Article 21.

. Page 26 of 36
SECTION - D: LOGICAL REASONING
Directions (Q.106-Q.110): Read the passage carefully and answer the given
questions:
There was a time when India was merely a back-office for the rest of the world. But
in less than a decade, the
Indian tech and start-up ecosystem have shed that yoke to become the third-largest
start-up hub in the world,
behind only the US and China. In the next decade, India can emerge as an innovation
hub setting global standards.
Technology matters, now more than ever before. Futurist Ray Kurzweil has made it a
habit to make remarkable
predictions about how technology will shape the future. We will merge with machines
by 2045, he says, pointing
to a future where our biology will be made up of equal parts technology and
physiology. If technology were to
be the backbone of our existence as a species, then it becomes all the more
relevant that it is developed with
equal participation from women too. There is an urgent need for better gender
diversity in technology.
Globally, men dominate the STEM workforce and educational map. When I was doing my
engineering in the
1980s, there were only seven women doing the course. Times have changed since then.
At the undergraduate
level, 51 percent of student enrolments are by males and 49 percent by females
across India; for PhDs, it is 56.18
percent by males and 43.82 percent by females; and for integrated levels, it is
57.50 percent for males and 42.50
percent for females. While more needs to be done to encourage more women to
continue pursuing their
education, there has been good progress over the last few decades.
Technology has always brought out far-reaching innovation, and more women in tech
will offer that vital and
much-needed balance to a male-dominated industry. Venture capital will encourage
such innovation. It typically
creates disruption and enables wealth creation. The venture capital ecosystem in
India plays a key role by
investing in innovative ideas, helping these ideas achieve product-market fit, and
in scaling up. The combined
valuation of some of the world’s biggest venture capital-funded companies may well
exceed the GDP of some
countries. Young start-ups that struggle to obtain capital from banks have
benefited from venture capital firms
that offer them the funding to realize their ideas, transforming society in the
process. Yet, women-led start-ups
often receive markedly reduced funding.
We should inspire more women globally to pursue careers in STEM and create the
platform and avenues that
make it easier for them to achieve that. We need to create clearer pathways for
women to enter into STEM
careers and stay there. It is, therefore, important to expand the talent pipeline
and make gender diversity the core
to hiring. We must work diligently to erase any unconscious bias in hiring or
promotions. Corporate rewards that
highlight progress in objective hiring and policies that are women-friendly need
the spotlight and encouragement.
106. “There is an urgent need for better gender diversity in technology,” what
rationale does the author give for this
assertion?
(a) Women empowerment is the need of the hour, and there is no reason why women
should be kept excluded
from the field of technology.
(b) Women will complement the thought process of men and would bring a different
perspective to the IT
industry.
(c) Technology may form the backbone of our existence, and hence it is imperative
that women be accorded a
greater role in the technology field.
(d) Both A and C
107. The author presents statistics for enrolments and argues that there has been
progress in including women in the
STEM field. Which of the following statements, if true, weakens the author’s
argument?
(a) Many women in India are not allowed to work after marriage, and hence, their
educational degrees rarely
matter.
(b) The number of seats in engineering and medical colleges has been significantly
reduced over the past years.
(c) The universities now have a mandated quota of allocating at least 20% of the
seats to women students.
(d) The dropouts rate of women in the STEM field after the initial enrolment is
between 70 and 80 percent.

. Page 27 of 36
108. Which of the following is true according to the passage?
(a) India of the 1980s did not have sufficient technological prowess and hence,
served as the world’s back#office.
(b) India has developed some disruptive technologies in the recent years and would
be the global hub for start#ups in the next couple of years.
(c) For any technological sector to grow, the diversification of its workforce on
the basis of gender is required.
(d) In less than a decade, the Indian tech and start-up ecosystem become the
second-largest start-up hub in the
world.
109. “Technology has always brought out far-reaching innovation, and more women in
tech will offer that vital and
much-needed balance to a male-dominated industry”, Identify the assumption
underlying the statement.
(a) Women would bring their own skillsets to the field and hence would complement
the men.
(b) Women and men have different skillsets.
(c) Inclusion of women in a male-dominated technology industry will bring fresh
perspective to innovation.
(d) Innovation can be brought about only if different perspectives are fused.
110. The author argues that gender diversity should be core to hiring; which of the
statements given below, if true,
would weaken the author’s argument?
(a) Skills should be the sole metric on which a prospective candidate should be
assessed.
(b) Women may bring a different perspective, but their managerial skills are seen
to be not strong.
(c) Gender diversity can be a part of a mix of metrics used to assess candidates,
with the most important metrics
being the skillsets of the candidates and the value they add to the company.
(d) None of the above.
Directions(Q.111-Q.115): Read the passage carefully and answer the given questions.
Over the last few decades, the dynamics of armed conflict have changed. The shift
from interstate to intrastate
conflicts has meant that the vast majority of victims are civilians, and the
emergence of non-traditional tactics
of war such as sexual violence in conflict has further increased their insecurity.
Systematic rape has been used
as a weapon of war in ethnic cleansing campaigns by militias in Rwanda and the
former Yugoslavia and more
recently by terrorist groups such as ISIS.
Terrorists and militias are not the only perpetrators of sexual violence. United
Nations (UN) peacekeepers have
also taken advantage of conflict zones and routinely exploit vulnerable populations
by engaging in sexual
exploitation and abuse (SEA). It is abhorrent that the blue helmets, which are seen
as symbols of peace, become
another source of insecurity for those they are mandated to protect. Statistical
analysis demonstrates that SEA is
a constant and endemic problem within UN peacekeeping, with an average of 50 cases
a year since 2010, and
31 cases filed between July and September 2017 itself. This is particularly
alarming since only a fraction of cases
is reported, given most victims do not feel comfortable seeking redress.
Indian UN peacekeepers have not been immune from allegations of SEA, with three
cases of SEA registered
against them between 2010 and 2013. However, it is important to note that while
being one of the largest
contributors of troops, with almost 200,000 troops spanning 50 missions, India has
one of the lowest cases of
SEA and has not faced any allegations after 2013. This has been achieved by
ensuring strict adherence to the
UN’s ‘zero-tolerance policy’ against SEA and making considerable changes to the way
Indian peacekeepers are
recruited, trained and prosecuted. This suggests that the Indian experience may
indeed have much to offer by
way of lessons to the rest of the world.
111. What is the reason behind the author’s assertion that the majority of victims
being civilians has increased?
(a) Armed conflicts have become more brutal.
(b) Weaponry has advanced.
(c) There has been a shift from inter-state to intrastate conflicts.
(d) Victims belong to low social strata.

. Page 28 of 36
112. What conclusion can you derive from the passage?
(a) Rape is a war weapon.
(b) Indian Peacekeeping force has not been involved in SEA since 2013.
(c) Indian Peacekeeping force can teach lessons to the world about integrity.
(d) Indian Peacekeeping force are the best in the world.
113. “Terrorists and militias are not the only perpetrators of sexual violence.
United Nations (UN) peacekeepers have
also taken advantage of conflict zones and routinely exploit vulnerable populations
by engaging in sexual
exploitation and abuse (SEA)”, what is the reason behind this assertion of the
author?
(a) Changing nature of conflicts.
(b) Statistical analysis demonstrating that SEA is a constant and endemic problem
within UN peacekeeping.
(c) Peacekeeping forces rarely contribute to the maintenance of peace.
(d) There has been a shift in the war strategies from inter-state to intra-state.
114. Which of the following assumptions, if true, weakens the author’s statement
that the shift from inter-state to
intra-state conflict is responsible for the majority of victims being civilians?
(a) Terrorists use fear-inducing tactics, hence, the rise in the majority of
victims being civilians.
(b) Modern warfare is not fought by armies alone.
(c) Bombing of cities by armies were responsible for wiping off of entire
populations.
(d) All of the above
115. According to the author what is the cause behind Indian UN peacekeeping forces
having lower cases of SEA
against them since 2013?
(a) Indians started sending more females for peacekeeping missions.
(b) India stopped participating in peacekeeping missions.
(c) Through strict adherence to the UN’s ‘zero-tolerance policy’ against SEA.
(d) Indians have very strict moral values.
Directions(Q.116-Q.120): Study the following information and answer the questions
that follow:
Leadership is the action of leading a group of people or an organization. Again, it
can also be said that leadership
is an art of motivating a group of people to achieve a common goal. In order to be
a good leader, one needs to
have some unique leadership qualities or leadership skills. First of all, honesty
is the foremost quality that a
leader needs to be a successful leader. A good or successful leader is always
honest. A dishonest person can’t
lead his group smoothly. On the other hand, a good leader always inspires his
followers and motivates them in
achieving the goal. He also possesses good communication skills so that he can
communicate with his group. He
keeps a constant eye on his followers too. Simultaneously an efficient leader has
decision-making capabilities
too. He can take a quick decision as per the situation. Leadership in education or
educational leadership is a
combined process that unites the wisdom of the triangle i.e., the teachers, parents
and students. The main goal
of educational leadership or leadership in education is to strengthen the quality
of education. In educational
leadership, the teachers, students, parents and those who are involved in the
process put their efforts together to
upgrade the quality of education. A dream of success is prepared through
educational leadership. On the other
hand, educational leadership also prepares a good learning environment for the
students. The teachers are
considered the founder of educational leadership; an organization can’t be imagined
without a leader. Leadership
in an organization creates a crystal-clear vision for the organization. A leader in
an organization motivates the
employees to reach the goal. He also shows them the vision of success. The growth
of the organization solely
depends on the influence of leadership in the organization. As a whole, leadership
plays a vital role in the success
and development of an organization. Leadership in management and leadership in an
organization sounds almost
the same. But both are a little different from each other. Management is a part of
an organization. To manage an
organization in a smooth way a good leader is required. Management leadership is
required to maintain a smooth
relation between the authority and the employees. In an organization, it is almost
impossible for the higher

. Page 29 of 36
authority to maintain a relationship or to motivate the employees all the time. The
leader does it and leads the
employee towards the goal.
116. According to the passage, which of the following statements provide incorrect
information.?
(a) Leadership is the activity of driving a gathering of individuals or an
association. Once more, it can likewise
be said that initiative is a specialty of propelling a gathering of individuals to
accomplish a shared objective.
(b) An exploitative individual can't lead his gathering easily. Then again, a
decent pioneer consistently rouses
his adherents and inspires them in accomplishing the objective. He likewise has
great relational abilities, so
he can speak with his gathering.
(c) Initiative in an association makes a clear vision for the association. An
innovator in an association rouses the
workers to arrive at the objective. He likewise conceals the vision of
accomplishment.
(d) The principle objective of instructive initiative or administration in
schooling is to fortify the nature of
training. In instructive initiative, the educators, understudies, guardians and the
individuals who are
associated with the cycle set up their efforts to update the nature of training.
117. What can be inferred from the given passage?
(a) A good leader who leads through examples in order to motivate people to a
shared vision has many qualities
that are necessary for sustenance. The qualities of good leadership such as
probity, communication skills,
motivational skills and problem-solving skills are required for a leader of any
field.
(b) The executive’s administration is needed to keep a smooth connection between
the higher position and the
workers. In an association, it is practically conceivable for the more significant
position power to keep a
relationship or to rouse the workers constantly.
(c) A productive pioneer has dynamic capacities as well. He can accept a fast
choice according to circumstance
requests. Administration in schooling or instructive authority is a joined cycle
that joins the ignorance of the
triangle i.e., the educators, guardians and understudies.
(d) None of the above
118. Which of the following statements strengthen the argument discussed in the
passage about an honest leader?
(a) Trustworthiness is the chief quality that a leader should have in order to lead
people, as honesty is the core
value between relationships and a leader should exhibit the mental courage to be
honest with his people for
their sustaining growth.
(b) An honest leader, being true to his words, should have the mental strength to
discourage his people wherever
the leader feels that the managerial task is beyond the capacity of workers.
(c) A dishonest leader may initially win over people but his specious arguments
reveal the true intent, losing the
trust of people in the long run.
(d) Honesty does not work all the time and a leader should at times use white lies
to prevent people from knowing
the dynamics of certain situations.
119. In an association, it is practically unimaginable for the more significant
person in power to keep a relationship
or______. The leader does it and leads the employees towards the objective.
(a) To propel the employees constantly
(b) To push the workers erratically
(c) To encourage the members sporadically
(d) To discourage the workers consistently.
120. According to the passage, why is a good leader required in management?
(a) It is convenient for a higher authority in a company to maintain a partnership
or inspire workers all the time.
(b) A leader placates the workers to accomplish the purpose. He also shows them a
vision of success.
(c) The leader’s initiative is needed to keep a smooth connection between the power
and the representatives.
(d) The growth of the organization solely depends on the influence of leadership in
the organization.

. Page 30 of 36
Passage(Q.121-Q.125): Indian universities have retained an archaic process of
admissions based solely on
qualifying examinations – ostensibly to introduce objectivity and fairness. This,
however, is a false equivalence:
Objectivity by itself doesn’t guarantee fairness. Exams create a single point of
entry and fail to account for
impediments like differences in socio-economic status, personal conditions at the
time of the exam, the vast
difference in educational outcomes among schools based on location and funding, and
other factors. This is not
to say that the ECA quota is without its issues; but it offers the prospect of a
more inclusive admission process.
Premier universities across the world have adopted a more holistic admissions
criterion, where a student’s
academic merit is often weighed alongside their contribution to extracurricular
activities, and other social or
professional efforts. DU, on the other hand, has become notorious for its towering
cutoffs – going as high as 99
per cent. In this environment of cut-throat academic competition, the ECA
(Extracurricular activity) quota offers
a much-needed respite to those who went beyond the confines of the classroom and
developed other aspects of
their personality. Students who devoted innumerable hours in honing their non-
academic talents cannot suddenly
be told to compete with others who did not make such investments of time and
effort. This is where the ECA
quota balances things out; it offers students a chance to be rewarded in fields
they truly excel in.
For a few students, the ECA quota has been life-changing. Some hadn’t scored enough
marks in the Class 12 to
secure a seat in any DU college. Recently a few of them finished their masters from
a foreign university. Had it
not been for the ECA quota, this journey would have never been possible.
Extra-curricular activities aren’t just hobbies that students at DU pursue; they
are often the most defining part of
their college lives. They enable students to form new perspectives about society,
register dissent against
oppression, share their ideas with the world, and pursue myriad undiscovered paths.
It is no surprise then that
some of the country’s greatest artistic talent has emerged from this university.
By scrapping the ECA quota, not only have talented students been denied the
opportunity of studying at DU, but
DU has been deprived of the brilliant and powerful ideas and expressions these
students would have added to its
environment.
121. According to the passage, which of these is correct about ECA?
1. ECA quota offers students a chance to be rewarded in fields they truly excel in
other than academic
excellence.
2. ECA quota has provided a more level-playing field for students.
3. Students of diverse non-curricular strata emerge from the corridors of the
university.
4. A student makes up an institute as much as an institute makes a student
(a) 1, 2 and 4 (b) 2 and 3 (c) 3 and 4 (d) 2,3 and 4
122. Consider the following statements regarding ECA and find the correct one#1.
ECA is of no relevance in the present days.
2. The only talents emerging from the university belong to ECA.
3. ECA counts on inclusion of students who can’t make it through academic admission
process.
4. It finds relevance beyond the confines of study room.
(a) 1 and 2 (b) 2 only (c) 3 only (d) 2, 3 and 4
123. The comparison of Indian admission process to premier universities serves
which purpose#1. Outlines the flaw in our system.
2. Lines the holistic approach International Universities have in assessing a
student application.
3. Highlights the hard work put in by young talents in honing their skill on non-
academic front.
4. Highlights that exam-based admission is faulty.
(a) 1 only (b) 2 only (c) 3 only (d) All of the above

. Page 31 of 36
124. The passage implies that exposure to students from ECA quota leads to#1.
Emergence of a pool of artists.
2. Platform for a pool of brilliant students.
3. Pursual of varied unexplored fields.
4. Students gaining new perspective and sharing idea with others.
(a) 1 and 2 (b) 1 and 4 (c) 1 and 3 (d) All of the above
125. In the context of the passage, the statement- “Exams create a single point of
entry and fail to account for
impediments”, can be said to be
(a) Aligned with the overall passage.
(b) Misaligned with the reasoning given by the author.
(c) Takes the passage on the same lines.
(d) A parallel argument.
Passage (Q.126-Q.130): Recently, two noted advocates declared that they will not
appear for a Chinese App
against the GOI. The same became the talk of the town praising them for showing
their patriotism. This meant
that any other lawyer accepting the brief for them would be considered ‘ANTI
NATIONAL'. This refusal has
surely opened up a discussion as the rules of the profession demands that an
advocate is bound to accept any
brief and the refusal "must be exceedingly rare".
Now what is an exceptional circumstance for an advocate to refuse a brief, has not
been defined anywhere. But
if one wants to know the answer, it could be traced to the customs and traditions
followed by the bar since time
immemorial.
The variety of cases handled by the Sr. Advocates and their likes could tell us,
the kind of matters, that an
advocate can and should accept.
Looking at the variety, it becomes impossible to ascertain an exceptional
circumstance, under which an advocate
could refuse to accept a brief and therefore it becomes important to understand if
at all under any circumstance,
acceptance of a brief by a lawyer could be considered as less Patriotic or ANTI
NATIONAL.
The Supreme Court in the case of A.S.Mohammed Rafi vs State Of Tamilnadu has
observed: "In our opinion,
such resolutions are wholly illegal, against all traditions of the bar, and against
professional ethics. Every
person, however, wicked, depraved, vile, degenerate, perverted, loathsome,
execrable, vicious or repulsive he
may be regarded by society has a right to be defended in a court of law and
correspondingly it is the duty of the
lawyer to defend him"
We may give some historical examples in this connection. When the great
revolutionary writer Thomas Paine
was jailed and tried for treason in England in 1792 for writing his famous pamphlet
`The Rights of Man' in
defence of the French Revolution, the great advocate Thomas Erskine (1750-1823) was
briefed to defend him.
Erskine was at that time the Attorney General for the Prince of Wales and he was
warned that if he accepts the
brief, he would be dismissed from office. Undeterred, Erskine accepted the brief
and was dismissed from office.
However, his immortal words in this connection stand out as a shining light even
today: "From the moment that
any advocate can be permitted to say that he will or will not stand between the
Crown and the subject arraigned
in court where he daily sits to practice, from that moment the liberties of England
are at an end.
Indian lawyers have followed this great tradition. The revolutionaries in Bengal
during British rule were
defended by our lawyers; the Indian communists were defended in the Meerut
conspiracy case; Razakars of
Hyderabad were defended by our lawyers; Sheikh Abdulah and his co-accused were
defended by them, and so
were some of the alleged assassins of Mahatma Gandhi and Indira Gandhi"

. Page 32 of 36
126. A few renowned Sr. Advocates from India have been appointed as Queens Counsel
in the United Kingdom.
Based on the passage, what can we infer from this?
(a) Only senior advocates can appear and represent another Country such as the
United Kingdom.
(b) Only with the Government’s permission can an advocate appear and represent
another Country such as the
United Kingdom.
(c) Any advocate can appear and represent another Country such as the United
Kingdom, of whom India has a
dreadful & devastating history.
(d) None of the above.
127. The Bar Council of Maharashtra and Goa passed a resolution prohibiting its
members from representing a
terrorist who was involved in Bombay Blasts. Based on the passage, evaluate the
following statements and mark
the right answer choice.
(a) The resolution passed by the bar council is against the principles of the
constitution.
(b) The resolution passed by the bar council is against the principles of natural
justice.
(c) The resolution passed by the bar council is illegal.
(d) The resolution passed by the bar council is legal.
128. Which of the following is a valid conclusion based on the passage?
(a) The Government cannot make a law prohibiting anyone from representing a person
in the court of law.
(b) Every advocate has an absolute duty to defend anyone who approaches him.
(c) Every person has a right to be defended against any charges levelled against
him.
(d) An advocate is within his jurisdiction to refuse any person as he deems fit.
129. What is the flaw with author’s argument about lawyers accepting to take up the
case of Chinese App being
perceived as anti-national?
(a) Author’s argument is based on a wrong premise.
(b) The author draws a wrong conclusion.
(c) The author is unfamiliar the subject matter.
(d) The author draws a wrong analogy.
130. Which of the following can be inferred from the passage?
(a) Lawyers cannot refuse to accept a brief.
(b) Exceptions to the general rule demanding the lawyers to accept a brief exist.
(c) A lawyer is free to refuse to defend a case at all times.
(d) Exceptional circumstances as to when a lawyer can refuse to take a case are
well defined.
Directions(Q.131-Q.135): These questions are based on the following information.
Five people Anand, Bhanu, Chandu, David and Eshwar buy TV’s of different companies
Onida, LG, Samsung,
Panasonic and Sony. Each of them have AC’s among Onida, Samsung, Videocon, LG and
Panasonic.
(i) The person who buys Onida TV also buys Samsung AC and Chandu owns a Panasonic
one.
(ii) David buys Samsung TV.
(iii)Neither Anand nor David have an LG product and Bhanu has Onida AC.
(iv) None of them buys TV and AC’s of the same company.
(v) The person who has Videocon AC has Sony TV but he is not Anand.
131. Who has Samsung AC?
(a) Anand (b) David (c) Bhanu (d) Chandu
132. Who has Onida AC?
(a) Anand (b) David (c) Bhanu (d) Chandu

. Page 33 of 36
133. Who has Sony TV?
(a) Anand (b) Eshwar (c) Chandu (d) Cannot be determined
134. Six aero-planes P, Q, R, S, T and U took off from an airport at different
timings. Q took off after S and before
T. R took off after S and before Q. If U took off after P but it is not the last
one to take off, then which is the last
aero-plane to take off?
(a) Q (b) T (c) S (d) R
135. In the given question three statements are given followed by two conclusions I
and II. You have to consider the
statements to be true even if they seem to be at variance from commonly known
facts. You have to decide which
of the given conclusion, if any, follow from the given statements.
Statements:
All snakes are reptiles.
Some reptiles are not fishes.
Some lizards are snakes.
Conclusions:
I. Some reptiles are lizards.
II. Some fishes are lizards.
(a) Only conclusion I follows. (b) Only conclusion II follows.
(c) Both I and II follow. (d) Neither I nor II follows

mock 22
Directions (Q.1-Q.30): Read the following passage carefully and answer the
questions that follow.
Passage (Q.1-Q.5): Controversy has surrounded many major development and
infrastructure projects in India,
such as the Sardar Sarovar dam on the river Narmada, the Chilka Lake in Orissa, the
Konkan Railways, the East
Coast Road, etc. Objections to these projects pertain to the extent of
environmental destruction and uprooting of
human settlements such projects may cause. But these environmental and social costs
have been justified by the
government and other protagonists as essential for any kind of development. This
dichotomy reflects the essence
of the debate around sustainable development. The process of resolving the
perceived conflict between
environment and development in all these issues, and the actual solutions that are
worked out, will indicate
whether the concept of sustainable development is implementable in a country like
India.
Not only in India but almost everywhere in the world, ‘sustainable development’ has
become the new buzz#word. Every international agency-from the World Bank to the
UNICEF –and almost every country is talking of
it. But what does this sustainable development mean?
“Sustainable development is the development that meets the needs of the present
without compromising the
ability of future generations to meet their own needs.” This definition has been
offered by the World Commission
on Environment and Development (WCED) in its report ‘Our Common Future’ (1987) and
widely accepted. It
can also be defined as economic progress in which the quantity and quality of our
stocks of natural resources
(like forests) and the integrity of biogeochemical cycles (like climate) are
sustained and passed on, unimpaired
to future generations.’
The eminent Indian economist, Sukhamoy Chakraborty has pointed out that the success
of ‘sustainable
development’ lies in the fact that it says nothing precise and, therefore, means
anything to anybody! For a logging
company, it can mean sustained projects; for an environmental economist it can mean
sustained stocks of natural
forests; for a social ecologist it can mean sustained use of the forest, and for an
environmentalist, it can mean a
clean heritage for our children. As a western joke goes now, sustainable
development for multinational
companies, many of which have also embraced the concept, means simply ‘sustained
growth’ or ‘sustained
profits’.
Environmental degradation has already been massive. Yet not many seem to be aware
of it. The natural resources
are being exploited without much consideration for future generations. Because of
the holes in the ozone shield
and the accumulation of greenhouse gases, the world as a whole may already be on a
critical threshold.
Indications of ecological degradation only reveal that economic growth of the
present kind which depends on
consuming the earth’s natural and environmental resources is not sustainable.
1. The phrase ‘Sustainable development’ (para 3 and 4) primarily refers to:
(a) Lucrative expansion that allows an equilibrium between the amount and the
condition of resources for
distribution to the future generations.
(b) Passing on of sullied natural resources and stock to the future generations,
without compromising on its
quantity and quality.
(c) A type of decadence that fulfils the demands of the present without adjusting
the needs of the future
generations.
(d) Economic progress that allows sporadic growth and development, without
hindering its supply to the future
generations.

. Page 3 of 36
2. What is the contradiction that (paragraph one) reflects the essence of the
debate around sustainable development?
(a) Sustainable development around the world incurs social costs that is inherent
in economic progress.
(b) Developmental projects affect the intensity of environmental damages and
stagnation of human
establishments.
(c) Despite of the environmental damage and social issues, the government has been
dissuading development at
the cost of the pertaining issues.
(d) Authorities have warranted the ecological and social ramifications of economic
development as
indispensable for progress.
3. Which of the following statements conclude the opinion of the Indian economist,
Sukhamoy Chakraborty?
(a) According to Sukhamoy Chakraborty, sustainable development lacks an imprecise
meaning. Hence, it can
be perceived by the people according to their understanding.
(b) According to Sukhamoy Chakraborty, sustainable development doesn’t have a lucid
definition and can be
projected as per different understandings.
(c) According to Sukhamoy Chakraborty, the notion of sustainable development has
been unsuccessful because
it builds a resolute idea within the society.
(d) According to Sukhamoy Chakraborty, sustainable development is defined in a
manner as to leave no scope
for misconstruction.
4. What can be inferred from the given passage?
(a) Most of the multinational companies and billion-dollar projects look at
sustainable development as a
precarious growth or an astatic progress.
(b) The existing generations and human communities are misusing natural resources
with awareness and concern
for the future generations.
(c) Sustainable development has become a fashionable and pervasive jargon all
around the world, with a
convenience of interpretation.
(d) According to reports, imminent economic progress is only viable by exploiting
natural resources. Hence, it
isn’t considered to be sustainable.
5. Which of the following statements accurately expresses the main point of the
passage?
(a) Sustainable development leads to perpetual growth and progress.
(b) Economic progress and advancement at the cost of environmental depravity can be
resolved with concerted
efforts.
(c) Dissipation of greenhouse gases in the environment is leading to fast paced
economic degradation.
(d) Today’s economic progress rely on utilizing natural resources is not
sustainable for the future generation.
Passage (Q.6-Q.10): “Literacy”, the ability to read and write, is often equated
with “education”, but is not the
same. Literacy is a step to education. For me, education is the complete
development of a person in terms of
knowledge, sensibility and most important, behaviour in different situations. Let
us take an example of a person
who knows very well to read and write. He is very fluent in English. Thus, we can
say that he is literate. But can
we say that he is well educated? I doubt. I can say he is educated only if his
knowledge and information gained
is reflected in his actions.
A person who is literate in a language is considered illiterate when he goes to a
foreign land having another
language but his behaviour will reflect his educated mind everywhere. Literacy
cannot make a person wise, but
education can. An educated man can only bring around changes in society and
contribute to the development of
society, which leads to the development of the nation.
To make people and children educated, we must take care of the teaching
methodology. We must have efficient
and eminent teachers who can educate the young generations as the mind once
enlightened cannot become dark.

. Page 4 of 36
The students of different capabilities must be mixed together in a proper order
without discrimination so that
they can learn from each other.
Everybody has his or her own capability and interest. We must encourage them to
excel in the fields in which
they desire to grow. They must be provided with the option to develop themselves.
If we judge a fish by its
ability to climb a tree then we are making a fool of us only.
6. As per the above passage, the author considers literate to:
(a) A person who is fluent in English.
(b) A person who is completely developed in terms of his knowledge and behaviour.
(c) A person who knows how to read and write.
(d) A person who took his education from efficient and eminent teachers.
7. On the basis of the above paragraph, what can be deduced of a person who is
excellent in terms of his knowledge,
sensibility and behaviour in different situations?
(a) He is educated, but nothing can be judged about his literacy.
(b) He is literate but not educated.
(c) He is neither literate nor educated.
(d) He is both literate and educated.
8. A person who is literate in a language can be considered illiterate
(a) When he goes to a foreign land speaking foreign language.
(b) If he cannot contribute in the development of his society.
(c) If he cannot read and write even though his overall behaviour is good.
(d) When he goes to a foreign land having another language.
9. According to the above passage, which of the following statements is not
correct?
(a) A person’s behaviour reflects his educated mind.
(b) Education cannot make a person wise, but literacy can.
(c) If a mind once enlightened cannot become dark.
(d) Different people have different capabilities and interests.
10. What, according to the above passage, cannot contribute in making people and
children educated?
(a) Encourage them to excel in the fields in which they desire to grow.
(b) The students of different capabilities must be mixed together in a proper order
without discrimination while
teaching.
(c) Teach them to behave in public.
(d) Efficient and eminent teachers.
Passage (Q.11-Q.15): Glance at any page of news about the movie business, and
you’ll spot a headline
announcing that Hollywood is dumbing down, that product placement is on the rise,
and that dialogue is being
whittled away to grunts and one-liners so that films can be exported with the
minimum of redubbing. From
ancient times, to the present days, there are increased reports of cinemas closing,
of piracy depriving the industry
of zillions of pounds, of television offering more sophisticated narratives than
its slow-witted multiplex cousin.
But the 100 inventive films in our selection indicate that, despite all this, we
may actually be living in a golden
age of cinema. It’s notable that the tally doesn’t rely on the giants of the late
20th Century: Spielberg and Scorsese
have just one film apiece; Mike Leigh, Ken Loach, Werner Herzog and the Dardenne
brothers are absent; and
the only Coppola represented isn’t Francis but his daughter Sofia. And yet the old
guard is hardly missed. There
is a wealth of astonishing films out there if you know where to find them - and
knowing where to find them has
never been easier.

. Page 5 of 36
Critics have an unfair advantage when it comes to happening upon the best stuff,
admittedly. If you’re lucky
enough to be paid to write about film, or, better still, to report from festivals
which serve up the cream of the
crop, you may well have a rosier view of the state of cinema than someone who has
to choose between Suicide
Squad and The Legend of Tarzan on a Friday night.
But none of us is as far from a life-enhancing film as we used to be. In the 20th
Century, anyone who was in the
mood for a monochrome musing on the roots of Nazism (The White Ribbon: 18) would
have had a tricky job.
Even in the heyday of art-house cinemas and video stores, you had to wait a long
time for a particular auteurist
gem to wash up on your shores - and that’s assuming you had heard of it at all. But
in the 21st Century, anyone
with an internet connection should have no problem venturing beyond films that
depict cities being flattened in
3D.
Today, we’re all just a click or two away from an intriguing trailer on YouTube, an
appetite-whetting essay on
a scholarly website, and an enthusiastic discussion in a message board. Another few
clicks and we can order the
film itself, either on DVD or download. It’s true that watching a taut Iranian
domestic mystery (A Separation:
9) at home may not be the same as seeing it in a crowded cinema. But - for the
film-maker as well as the viewer
- it’s better than not watching it at all.
11. Which of the following can best be concluded from the above passage?
(a) In the present times, people are less interested in going to the movie theatre
than ever before.
(b) In the present times, loss of revenue is of prime concern to the Hollywood film
makers.
(c) There is a shift that has taken place in Hollywood from its inception to the
21st century.
(d) The new globalized world, where everything is just a click away, has several
complications.
12. According to the passage, who has a better understanding of cinema?
(a) Those who watch movies on big screen of the cinema.
(b) Those who watch movies at their home comfortably and sensibly.
(c) Those who read other people’s views about films.
(d) Those who are directly or indirectly a part of cinema.
13. Which of the following inferences can be drawn from paragraph four of the
passage?
(a) Internet has been a source of enlightenment for the people from 20th to 21st
century.
(b) Watching films has become easy in the 21st century.
(c) There has been a transition in film viewing in the 21st century because of the
digital world.
(d) All the major developments have taken place in the 21st century.
14. ‘Hollywood is dumbing down, that product placement is on the rise, and that
dialogue is being whittled away to
grunts and one-liners so that films can be exported with the minimum of redubbing.’
What can be inferred from the expression, ‘dumbing down’?
(a) Hollywood is simplifying so as to be intellectually undemanding and cater to
wide audience.
(b) Hollywood movies are made complicated so as to be intellectually demanding and
cater to selective
audience.
(c) Hollywood is unable to cater to the demands of the foreign audience and is
fading out.
(d) Hollywood has applied new strategy to playing down the dialogues and sharpening
the actions.
15. Which of the following is nearest in meaning to the word ‘rosier’ in the
context of the passage?
(a) Better (b) Healthier (c) Glowing (d) Brighter

. Page 6 of 36
Passage (Q.16-Q.20): The Red Fort, from where all our honourable prime ministers
have, ever since
Independence, addressed the nation, in the presence of happy people, was barricaded
from all sides this year,
while our present hon’ble Prime Minister was addressing the nation, on Independence
Day. This time the Red
Fort had been made secure, safe, hidden from public vision and approach with huge
bunkers.
The ratio of security measures adopted actually indicate the ratio of fear! The
question is: Who are we afraid of?
Who are we protecting ourselves from? Our own brothers? What are we threatened
with? Haven’t the people of
India co-existed peacefully for more than 70 years? Where has our trust for each
other gone? What has happened
to our “peaceful coexistence”? Has it suddenly evaporated into thin air? Why have
we suddenly become a threat
to each other?
A natural question in sequence would be, whether this fear is real or is it simply
a purposefully designed illusion
of fear, that is eating away into our very existence. In Parliament, did we need
such a huge army of marshals to
protect the chair? Did the treasury side actually feel physically threatened by the
Opposition? Are we enemies
to each other or simply two sides with different ideologies? Protest and dissent,
of course, have always been a
way of democratic expression. Then what is this fear and insecurity indicative of?
The truth is that in a democracy, swaying public opinion is crucial. And in order
to gain it, fear is always used
as a tool. Ever since the inception of human civilisation, mankind has admired a
saviour or a liberator. The prince
of the fairy tales, who could kill the monster or the giant, became the unsung hero
of his times. In order to become
an unsung hero today, a monster is needed. And if there is no monster, one is
created! So, in a country of diverse
cultures like India, communities become the monster, from which the saviour has to
keep the people scared all
the time. In order that people keep looking up to the saviour to keep them
protected.
One political party demonises one community and makes a section of the population
perceive that it is only they
who can save them from the demon or the dragon or the monster or the giant. While
the other political party does
the same vice versa.
16. From the passage, it can be inferred that:
(a) The barricading of various prominent areas is being done rigorously.
(b) The façade of panic is spread among masses, to keep the ball rolling with
politics.
(c) Many fabricated ideologies are coming into play off late.
(d) Fright is the key to success for achieving any desired position in politics.
17. As per the passage, we are part of the political ideology that is
(a) Inherently a cultural mindset.
(b) Part of the ancestral legacy.
(c) Progressive than its counterparts.
(d) A creation of pseudo trepidation.
18. What is the role of the so-called ‘saviour’, in countries, where the
communities became devilish?
(a) to aid the people round the clock.
(b) to nurture the good set of value.
(c) to time and again frighten the populace.
(d) to bar the unwanted responses from the majority.
19. The tone of the passage is:
(a) Condemning (b) Idolatry (c) Fussy (d) Pedantic

. Page 7 of 36
20. What can be best substitute of the phrase ‘eat away’?
(a) To act maturely in front of the gentry.
(b) To continually bother or worry someone.
(c) To demolish the setup of the fake pretense.
(d) To incite the immature people.
Passage (Q.21-Q.25): The father of our nation, Mahatma Gandhi, once remarked,
“Youth are agents for
transformation”. The history of modern India would be incomplete without
acknowledging the role played by
students and youth of this country.
Many social revolutions and changes were brought about through politically
conscious and socially responsible
students, who raised their voices against existing inequities. Students have been
the face of the Indian
independence movement. In fact, the youth have often taken up certain causes and
inspired many political parties
to take up the same subsequently.
Education has a social agenda. The agenda is to develop our human resources, which
meet the requirements of
society. An educated citizenry is the greatest asset for any democratic society.
Students are known for their
readiness to fight for all the right causes because their thoughts are pure and
honest. They are always at the
forefront, questioning injustice. Any keen observer of Indian society would notice
that in the past few decades,
no big leader has emerged from the student community. This appears to be correlated
with diminished
participation of students in social causes after liberalisation. The importance of
students’ participation in a
modern democracy cannot be played ______.
It is necessary for you (students) to take part in current debates. You must have a
clear vision. It is essential that
more and more well-meaning, forward-looking, and upright students like you enter
public life. You must emerge
as leaders. After all, political consciousness and well-informed debates can steer
the nation into a glorious future
as envisioned by our Constitution. A responsive youth is vital for strengthening
democracy.
It is, therefore, necessary for students to realise the importance of their
relationship with society. Students are an
integral part of society. They cannot live in isolation. Students are guardians of
freedom, justice, equality, ethics,
and social equilibrium. All this can be achieved only when their energies are
properly streamlined. When the
youth become socially and politically conscious, the basic issues of education,
food, clothing, healthcare, shelter,
etc. would come into focus in the national discourse. The educated youth cannot
remain aloof from social reality.
You have a special responsibility.
The youth of today is driven by idealism and ambition. Idealism without ambition
may not achieve any positive
results. Ambition without idealism can be dangerous.
21. According to the passage, which of the following is not supported by the
passage?
(a) The educated youth cannot remain aloof from social reality.
(b) Education has a social agenda.
(c) The history of modern India would be complete without acknowledging the role
played by students and
youth of this country.
(d) Students are known for their readiness to fight for all the right causes
because their thoughts are pure and
honest.
22. “Youth are agents for transformation”. What can be inferred from the statement
of Mahatma Gandhi?
(a) Youth are the catalysts of reforms and changes.
(b) Youth represent changes.
(c) Youth are the agents for rebellion.
(d) Only youth can change the face of a country.
23. The importance of students’ participation in a modern democracy cannot be
played ______. Fill in the correct
word to make the sentence coherent.
(a) up (b) down (c) over (d) along

. Page 8 of 36
24. What is the author trying to communicate through the passage?
(a) Youth must have a social, economic and political voice and participation in
modern democratic India.
(b) Ambition without idealism is dangerous.
(c) Youth must actively participate in debates and speak against the injustices
being meted out to the weaker
section of the society.
(d) Youth are the flag bearer of law and order in the society and must uphold the
basic values of the society.
25. Which of the following sentences will be a coherent carry forward of the
author’s point of view?
(a) The two are separate and isolate and combining the two can lead to perilous
outcomes.
(b) Do not allow narrow and partisan issues to dominate the nation’s thought
process.
(c) The harsh reality is that even after the students enter professional
universities, the focus is on classroom
learning, and not on the world beyond the classroom.
(d) Combine the two in the right proportion and enable our country to emerge as one
of the most powerful and
harmonious.
Passage (Q.26-Q.30): And while you are at it, give me one hundred Abraham Georges
as well -- individuals
who step out of their context and set a different example can have such a huge
impact on the imagination of so
many others. One day in February 2004, I was resting in my hotel room in Bangalore
when the phone rang. It
was a young Indian woman who said she was attending a private journalism school on
the outskirts of the city
and wanted to know if I would come by and meet with her class. I've learned over
the years that these sorts of
accidental invitations often lead to interesting encounters, so I said, 'What the
heck, sure. I'll come.' Two days
later I drove ninety minutes from downtown Bangalore to an open field in which
stood a lonely journalism school
and dormitory. I was met at the door by a handsome middle-aged Indian man named
Abraham George. Born in
Kerala, George served in the Indian army, while his mother immigrated to the United
States and went to work
for NASA. George followed her, went on to study at New York University, (NYU),
started a software firm that
specialized in international finance, sold it in 1998 and decided to come back to
India and use his American#made fortune to try to change India from the bottom --
the absolute bottom -- up.
One thing George learned from his time in the United States was that without more
responsible Indian
newspapers and journalists, the country could never improve its governance. So, he
started a journalism school.
So how is journalism important? Uncovering the truth is contingent upon ethical,
skilled journalists who tell a
story accurately. Whether reporting in a war or areas of media censorship, it is
their job to gain access and bring
the world closer to people. New technology has made that easier – bringing the
world closer to disaster.
Journalists are people actively engaging with the world, through pen and
technology. Supplying journalism
education will fill the void in the profession and encourage those who would like
to punch story ideas to local
reporters on how to do it, and in the end, create a more participatory society.
As we sat in his office sipping juice, it quickly became apparent to me, though,
that as proud as he was of his
little journalism school, he was even more proud of the elementary school he had
started in a village outside
Bangalore populated by India's lowest caste, the untouchables, who are not supposed
to even get near Indians of
higher caste for fear that they will pollute the very air they breathe. George
wanted to prove that if you gave
these untouchable children access to the same technologies and solid education that
have enabled other pockets
of India to plug in and play with the flat world, they could do the same. The more
he talked about the school, the
more I wanted to see it and not talk journalism. So as soon as I finished speaking
to his journalism students, we
hopped into his jeep, along with his principal Lalita Law, and set out on a two-
hour drive to the Shanti Bhavan
school which was located about ten miles and ten centuries from the outskirts of
Bangalore. The word 'wretched'
does not even begin to describe the living conditions in the villages around the
school.
26. What is the author of the passage commending Abraham George for?
(a) Abraham George went to the author's country.
(b) Abraham George followed in his mother's footsteps.
(c) He set a different example leaving a huge impact on many others.
(d) Abraham George studied at New York University.

. Page 9 of 36
27. Why did a young lady from a private journalism school invite the author?
(a) She wanted the author to contribute to the journalism school on the outskirts
of Bangalore.
(b) The young lady had known the author’s achievements for long
(c) She wanted the author, a journalist, to speak to and inspire the journalism
students.
(d) The young lady wanted to make friends with the author.
28. What do you understand by the phrase “... to change India from the bottom --
the absolute bottom -- up'?
(a) To pull out India from the bottomless pit.
(b) To scrape the bottom of the barrel.
(c) To progress upward from the lowest levels.
(d) To let the decline in certain fields, iron out
29. According to the passage, which of the following statements is TRUE?
(a) George's mother taught at New York University.
(b) George's software firm went bankrupt in 1999.
(c) George wanted to use his American-made fortune to enrich the lives of all the
untouchables in India.
(d) George realized that better governance in India depends on newspapers and
journalists.
30. Why does the author of the passage say that the word 'wretched' cannot describe
the living conditions in the
villages around the school?
(a) The author was being diplomatic and tactful in describing the conditions.
(b) The living conditions were so deplorable that even ‘wretched’ did not do
justice to describe the conditions.
(c) The living conditions were barely human and the author could not come up with
any other word.
(d) The author did not like to hurt anyone, least of all his Indian hosts.

Directions (Q.66 – Q.105): Read the comprehensions carefully and answer the
questions based on it.
Passage (Q.66-Q.70): In a judgment the Supreme Court explained the difference
between culpable homicide
under Section 304 of the Indian Penal Code and murder under Section 302 IPC.
The court then referred to the decision in State of Andhra Pradesh v Rayavarapu
Punnayya 1976. "In the scheme
of the Penal Code, "culpable homicide" is genus and "murder" its specie. All
"murder" is "culpable homicide"
but not vice- versa. Speaking generally, culpable homicide sans special
characteristics of murder, is culpable
homicide not amounting to murder. For the purpose of fixing punishment,
proportionate to the gravity of offence,
the Code practically recognises three degrees of culpable homicide. The first is
murder punishable in Section
302 of IPC, what may be called, "culpable homicide of the first degree". The second
may be termed as "culpable
homicide of the second degree". This is punishable under the first part of Section
304. Then, there is "culpable
homicide of the third degree".i.e. culpable homicide not amounting to murder is
punishable under the second
part of Section 304.”
Culpable homicide is murder, if the act by which the death is caused is done with
the intention of causing death
or if it is done with the intention of causing such bodily injury as the offender
knows to be likely to cause the
death of the person to whom the harm is caused.
Culpable homicide is not murder if the offender causes death of a person, whilst
deprived of the power of self#control by grave and sudden provocation or anything
done in the exercise of the right of private defence exceeds
the power given to him by law and causes the death of the person, or being a public
servant or aiding a public
servant acting for the advancement of public justice exceeds the power given to him
by law, or when the person
whose death is caused, being above the age of eighteen years, suffers death or
takes the risk of death with his
own consent.
66. A man, A had an affair with B’s wife C. B got to know this. B was tracking A
for a few days. B hit A’s car one
morning and went away. D, A’s driver was driving the car that day. C got hurt badly
in the accident. Decide.
(a) B is liable for culpable homicide of first degree.
(b) B is liable for culpable homicide amounting to murder.
(c) B is liable for culpable homicide not amounting to murder.
(d) None of the above.
67. A police man was on duty at a check post. A truck driver was asked to stop,
which he didn’t abide by and rather
increased the speed to skip the check post. The same truck driver had stolen
jewellery from the police man’s
house an hour ago, secretly. The police man climbed the moving truck and entered
it. He pushed the driver hardly
than required so he would stop the truck and from the push, the driver fell from
the truck, came beneath the tyre
and died. Decide the culpability of the police officer.
(a) The police officer is liable for culpable homicide of first degree, since he
had an intention to kill the man
who stole jewellery from his house.
(b) The police officer is not liable since he was performing statutory duty.
(c) The truck driver is liable for theft and obstructing statutory duty of a police
officer by not stopping the truck
at the check post.
(d) The police officer is liable for culpable homicide not amounting to murder.

. Page 17 of 36
68. A police man was on duty at a check post. A truck driver was asked to stop,
which he didn’t abide by and rather
increased the speed to skip the check post. The same truck driver had killed the
police officer’s brother 2 years
ago and fled in front of the police officer, who was helpless then. He wanted to
take revenge ever since. The
police officer identified the truck driver, who also recognised the police officer,
climbed the moving truck and
entered it. He intentionally pushed the driver hardly than required outside the
truck, who from the push, fell from
the moving truck, came beneath the rear tyres and died. Decide the culpability of
the police officer.
(a) The police officer is liable for culpable homicide not amounting to murder,
since he just wanted the driver
to stop the truck.
(b) The police officer is liable for culpable homicide of first degree.
(c) The police officer is liable for culpable homicide of first degree, but the
truck driver is also liable for not
stopping the truck at the check post as required.
(d) The police officer is liable for culpable homicide second degree.
69. Two friends were once talking on a controversial topic. One of them, Sumit made
a point, which hurt the
sentiments of the other one, Nikhil. Nikhil got infuriated by Sumit’s statement. He
found an axe nearby. He
picked the axe and hit at Sumit on being provoked by Sumit’s point. The axe hit at
the chest of Sumit, who died
due to excessive blood loss, but he could have been saved if taken to hospital at
right time. Decide.
(a) Nikhil is liable for culpable homicide of first degree.
(b) Nikhil is liable for culpable homicide of second degree.
(c) Nikhil is liable for culpable homicide of third degree.
(d) Nikhil is liable for murder as a prudent man will know hitting by axe will
cause death of a person.
70. Which of the following is correct according to the passage?
(a) In the scheme of the Penal Code, culpable homicide is genus and murder its
specie. Speaking generally,
culpable homicide sans special characteristics of murder, is murder not amounting
to culpable homicide.
(b) In the scheme of the Penal Code, culpable homicide is specie and murder its
genus. Speaking generally,
culpable murder sans special characteristics of murder, is culpable homicide not
amounting to murder.
(c) In the scheme of the Penal Code, murder is genus and culpable homicide its
specie. Speaking generally,
murder sans special characteristics of culpable homicide, is culpable homicide not
amounting to murder.
(d) In the scheme of the Penal Code, culpable homicide is genus and murder its
specie. Speaking generally,
culpable homicide sans special characteristics of murder, is culpable homicide not
amounting to murder.
Passage (Q.71-Q.76): The Delhi High Court has directed the police to ensure that
temporary stay in a children's
home is provided to the minor girl, who left her parental home after allegedly
being forced to marry an old man
and her parents not letting her study, as an interim measure till September 6, the
next date of hearing. Girl below
18 years cannot marry, as lawful age for marriage is 18 years and above. After
attaining majority person has
right to choose partner of his own choice for marriage. The Court heard a petition
filed by a minor girl through
her next friend claiming that she left her parental home on 17th June, 2021 since
her parents were allegedly
beating her, not letting her go out of the house, refused to let her study and
intended to marry her to an old man.
During the course of hearing on Friday, the Court interacted with the girl through
video conferencing wherein
she apprised the Court that she is not inclined to live with her parents. "It would
be, thus, appropriate to talk to
the parents of the petitioner to find out the problem involved as also with Mr.
Gaurav, who has filed the present
petition as the next friend of the petitioner," the Court ordered "SHO, PS,
Badarpur is directed to ensure that the
petitioner is taken along with the lady Constable to Children's Home, Nirmal Chhaya
and lodged there today
itself, till the next date of hearing, as she is a minor and be brought to this
Court on the next date," it added. The
Court also directed the SHO to inform about listing of the matter to the parents in
writing and also asked the
girl's next friend to be present before Court. Solving such type of matter is
crucial as all cannot be allowed to
stay in rescue home or in a children's home forever, thus police can take necessary
steps to achieve the same.
The court held that parents can take decisions that are justified and necessary but
can neither restrict child from
studying further nor can force to marry any person.
Source Name - Live Law

. Page 18 of 36
71. J was 17 years old girl living with her parents. She was a bright student in
her school. She liked a boy named G
in her school 2 years senior than him. There was a mutual relationship between both
of them and J wanted to
marry him after completing her college. But her parents wanted their daughter to
get married immediately and
thus they compelled to her to marry him. In the light of the above passage, opt out
the correct option.
(a) There is a mutual consent between the two and thus she should marry G.
(b) She should complete her college and then choose to marry him.
(c) Parents have the right to choose best for their children and thus their action
is justified.
(d) Parents cannot compel their girl but they are justified by choosing the same
boy for her marriage.
72. Y was a girl of sixteen years. She was residing with her boyfriend in a live in
relationship after having some
altercations with her parents. She decided to marry her boyfriend but was
restrained by her parents, and upon
insistence for the same, she filed a criminal complaint against her parents about
harassing the couple. In the light
of the above passage, opt out the correct option.
(a) Her parents were harassing the couple and thus liable to be arrested.
(b) Her parents were considerate about her life as she is minor thus the steps
taken were justified.
(c) Boy should have restrained from marrying a minor because she was harassed by
her family about the same.
(d) The girl is justified for living and marrying to him as it is her right.
73. D was a girl of 19 years who was raped by an old man of 62 years in a village.
When the matter went into court,
the court ordered the old man to marry the girl which was opposed by her parents.
In the light of the above
passage, chose the correct option.
(a) The court was justified in deciding the same.
(b) The court was somewhere not justified in doing the same.
(c) The old man should let the girl tie a rakhi on his wrist construing to the
previous judgment of the court.
(d) The girl has the right to choose whom she wants to marry, thus decision of
court is not justified.
74. H was a girl of 18 years who was living with her boyfriend of 22 years of age
in a rescue home basically meant
for sheltering the harassed couples. Her parents wanted her to marry immediately
with a 20 year boy who was
their friends’ son. She didn't want to marry him. She and her boyfriend were living
there for 11 months. The
police authority reminded them to vacate the premise multiple times and resolve the
matter with parents. On
refusal to vacate the rescue home, the police forced them out of the same. In the
light of the above passage, opt
out the correct option.
(a) Police were not justified in forcing them out of the premise.
(b) Police should take care of the harassed couple and let them live in the
premise.
(c) Solving such type of matter is crucial and police authority reminded them
multiple times thus, the police
was justified in forcing them out.
(d) The girl has no right to marry 22 year old boy.
75. Construing to the passage above, which of the interpretation is the most
correct.
(a) Raping a girl and then offer of marriage by the same person can nullify the
offence to some extent.
(b) Parents have no right to compel their child for marriage at any cost.
(c) Parents can regard some compulsions on their children for their betterment in
life but cannot compel for
things not justified.
(d) Parents cannot compel their minor to get married to any person unless she
herself wants to get married at
that age.
76. Construing to the above passage, which of the following is an incorrect
interpretation of the passage.
(a) Compelling minor girl to get married to any person even by the parent is not
justifiable.
(b) Compelling any minor even by the parents to marry a person more than her age is
not justified.
(c) If a minor wants to get married by her own choice, the parents have no right to
impose any compulsion or
restrictions.
(d) Compelling a minor for the betterment of her life is totally justified.

. Page 19 of 36
Passage (Q.77-Q.82): Observing that Nehru Place market area in the national capital
should not become a slum,
the Delhi High Court sought suggestions from the South Delhi Municipal Corporation
on the management of
squatters in the area. Justice Manmohan and Justice Navin Chawla was hearing a suo
moto case registered by
the Court following a fire incident at a building in the market. The Court said
that prima facie it was of the view
that till the time another bench headed by Justice Vipin Sanghi decides the
constitutionality of the Town Vending
Committee, it will allow only 95 vendors for hawking in the market area, subject to
verification of their names.
The Court also added that the said vendors will strictly comply with the terms of
hawking and vending, will not
have a permanent structure and will be confined to the hawking permitted area.
Additionally, the Court said that
the fire brigade must be able to reach the area in case of emergency and that the
vendors must ensure that the
goods are removed and are confined to the embarked area only. "The enormity of the
problem caused by the
hawkers and vendors at the Nehru Place area can be seen from a video circulated on
social media which shows
that it was difficult for the fire tenders to access the building where the fire
broke out. It also shows the complete
lack of any cleanliness or maintenance on the part of the Municipal Authorities,"
the Court had said. It was found
by the Court that the hawkers and vendors had claimed the pavement space which was
meant for circulation of
pedestrians.
Source Name - Live Law
77. There was fire Breakout in the building. However the area was a densely covered
with the hawkers and vendors.
The fire brigades were not able to get clearance off the path and so they couldn't
reach on time and couldn't work
out efficiently. The court on observing the same sort suggestions from the South
State municipal corporation on
the management of the vendors and hawkers. In the light of the above passage choose
the correct option.
(a) According to the passage the court did the right thing.
(b) The court was not justified for doing the same according to the passage.
(c) As the hawkers had right to profess occupation as it is a right given in the
constitution of India the court was
not justified.
(d) Owing to the problems faced by the fire brigades to reach on time due to
hawkers and vendors the court was
justified for seeking suggestions from the South state municipal corporation.
78. Squatters are the people who to claim their right over some area which is not
authorised for them. There were
some squatters who worked as vendors and hawkers in a particular area and they had
occupied certain pavement
spaces which were meant for circulation of the pedestrians. However this caused
some problems in in the
particular aspect when there was a fire break out and there was a need of fire
brigade which could not get the
clearances because it was difficult to pass through the presiding hawkers in that
particular area. Observing the
same the court shot suggestions from the South state municipal corporation on the
management of such squatters
in the area. In the light of the above passage choose the correct option.
(a) The court was not justified for seeking suggestions from the South State
municipal corporation.
(b) The court was not justified for seeking the same because the hawkers had the
right to profess their occupation
in the area.
(c) The court was justified for seeking suggestions on the management of the
squatters because it was very
difficult for the pedestrians to move around because of these squatters as it was
their fundamental right.
(d) The court was justified for seeking suggestions on the management of squatters
because the fire brigade
could not get the clearance of the roads due to those hawkers and it was difficult
for the pedestrians to move
around as hawkers and vendors had claimed the pavement spaces.

. Page 20 of 36
79. It was seen in a video that the fire brigade was not able to get through the
road because of the vendors and the
hawkers that squatted the land and claimed the pavement that was meant for
circulation of the pedestrians.
However construing to the problems court after seeing the video tape observed that
there was a need to seek
suggestions from the south state municipal corporation on the management of the
quarters and the vendors and
hawkers in the area so that the fire brigades could get the clear road and reach
the area in case of emergency. In
the light of the above passage choose the correct option.
(a) The court was not justified for seeking the suggestion from the south state
municipal corporation for the
management because hawkers and vendors did not claim the pavement space but
squatted the roads upon
which they could carry out their profession.
(b) The court was justified for seeking suggestions from the south state municipal
corporation.
(c) As construing to the video it was seen that the fire brigade must reach the
place in the time of emergency so
the court was justified for seeking suggestions from the state municipal
corporation on the management of
squatters and vendors.
(d) The court was not justified for seeking suggestions from the south state
municipal corporation.
80. There were certain vendors and hawkers on the road that covered up the road
completely for carrying out their
business. But they did not clean the pavement part that was meant for the
circulation of the pedestrians so there
is no problem for them. However a fire broke out in an area and the fire brigade
was not able to reach on time in
the time of emergency because of those vendors. Construing to the same the court
sought suggestions from South
State municipal corporation on the management of those hawkers and vendors on
account for the fire brigade
that must be able to reach the area in case of emergency. In the light of the above
passage choose the correct
option.
(a) The court was right in seeking suggestions from South State municipal
corporation.
(b) The court was not justified for seeking the suggestions from the South State
municipal corporation because
the vendors and carrying out their profession.
(c) As the hawker and vendors did not claim the pavement part the court was not
justified for seeking suggestions
for the management of those vendors and hawkers.
(d) As there was difficulty for the fire brigade to reach on time in the time of
emergency the court was completely
justified for seeking suggestions from the South state municipal cooperation for
the management of those
vendors and hawkers.
81. In the light of the above passage choose the correct option.
(a) Carrying out the profession is a fundamental right in the constitution of India
and vendors and hawkers were
justified to clean the pavement spaces in the passage.
(b) The vendors and hawkers in the passage can claim the pavement part that was
meant for circulation of
pedestrians.
(c) The hawkers and vendors cannot claim the pavement part off the road that was
meant for circulation of the
pedestrians but they can install their shops and goods on the road.
(d) As there is a need for quick action of the fire brigade in the time of
emergency so the vendors and the hawkers
have no right to install there shops on the roads are in the pavement that was
meant for circulation of
pedestrians.
82. In the light of the above passage choose the incorrect option.
(a) The pavement part is meant for circulation of public and so the vendors cannot
claim for install of shop over
that.
(b) The pavement part is for the pedestrians so the hawkers and vendors cannot
claim their right over that for
circulation of pedestrians.
(c) There is a need for clearance of the roads on the account of the fire brigades
in the time of emergency.
(d) The court can seek suggestions from the state municipal corporations for the
management of hawkers and
vendors.

. Page 21 of 36
Passage (Q.83-Q.88): The Supreme Court has observed that while issuing an
interlocutory direction at an
interim stage while exercising jurisdiction under Section 482 of the Code of
Criminal Procedure, the High Court
must furnish reasons. "Even at the interim stage, the High Court must demonstrate
an application of mind and
furnish reasons for issuing any interlocutory direction, which is capable of being
tested before this Court in an
appropriate case", a bench observed. The Supreme Court was considering the case
which was an appeal against
a judgment of the Gujarat High Court which quashed criminal proceedings exercising
powers under Section 482
CrPC. The Supreme Court took a critical view of the fact that the High Court's
interim direction was not
supported by any reason. "The interim direction amounted to an unnecessary
interference in the investigative
process envisaged under the CrPC. In a petition filed under Section 482 CrPC filed
by some of the accused, the
High Court directed that the investigation may continue but the charge-sheet be
filed only with its permission.
A draft charge-sheet was placed before the High Court for offences punishable under
Sections of the IPC. Taking
note of the contents of draft charge sheet, the High Court quashed the FIR against
some of the accused. The
Supreme Court further held that the High Court could not have quashed the
proceedings on the basis of a "draft
charge sheet". "...the High Court cannot place reliance on a "draft charge-sheet"
which is yet to be placed before
the Magistrate to quash the criminal proceedings under Section 482", the Court
said.
Source Name - Live Law
83. There was an investigation going on against an accused. However there was a
draft chargesheet that was placed
before the high court for the offences punishable under IPC against the accused.
But the high court taking the
note of contents of the draft charges sheet quashed the FIR against the accused. A
petition was filed against the
decision in the supreme court and the supreme court held that the High court could
not have quashed the
proceedings on the basis of draft charges sheet. In the light of the above passage
choose the correct option.
(a) The High court was not justified for quashing the proceedings on the basis of
draft charge sheet.
(b) The high court was justified for doing the same as according to the passage it
is clarified itself.
(c) The supreme court was not justified for determining the same for the High
Court.
(d) The supreme court was justified for doing the same.
84. Investigation process was envisaged under the sections of CrPC against an
accused. However petition was filed
by the accused before the high court and the high court directed that the
investigation may continue but the
charge sheet must be filed only with its permission. After the draft chargesheet
was placed before the High court
the high court quashed the FIR against the accused on the basis of the same. In the
light of the above passage
choose the correct option.
(a) The High court was justified for quashing the FIR against the accused according
to the passage.
(b) According to the passage the High court was not justified.
(c) As the supreme court had held that the High court could not have quashed to the
proceedings on the basis of
draft chargesheet so the high court in the present case was not justified.
(d) The high court have the power to quash the FIR against the accused on the basis
of charge sheet.
85. There was an appeal filed by the accused in a case where the investigation was
going on against him and he filed
a petition before the High court. The High court directed that the investigation
may continue but the charge sheet
must be filed with its permission. The high court relied upon the draft charges
sheet filed but the investigating
process and on that basis the high court quashed the FIR against the accused. The
supreme court upheld the
decision. In the light of the above passage choose the correct option.
(a) The Supreme Court was justified for upholding the decision of high court.
(b) The High court was justified for quashing the FIR proceedings on the basis of
drafted chargesheet.
(c) The supreme court was not justified for the same.
(d) According to the passage the Supreme Court in the passage determined that the
high court cannot place
Reliance on a draft charge sheet which is yet to be placed before the magistrate
and so cannot quash the FIR
proceedings thus Supreme Court was not justified.

. Page 22 of 36
86. An appeal was filed by the petitioner regarding the investigation going on
against him. The High court directed
and the investigation that the investigation may continue but the charge sheet
should be filed with its orders.
However the high court and directed the investigating agency to file a draft charge
sheet before it and so the high
court relied upon the same and quashed the FIR against the accused who was the
petitioner. The Supreme Court
opined that the high court cannot be its Reliance on a draft chargesheet and so
cannot quash the FIR. In the light
of the above passage choose the correct option.
(a) The high court cannot place Reliance on the draft charge sheet so the Supreme
Court was justified.
(b) Reliance can be placed upon the draft chargesheet and the quashing of FIR can
be done so the supreme court
was not justified.
(c) Supreme court was not justified for deciding the same.
(d) Supreme court was justified for deciding the same as the court into the
passage.
87. In the light of the passage choose the correct option.
(a) Reliance can be placed upon by the High court on the draft charge sheet in some
circumstances.
(b) Reliance cannot be placed on the draft charge sheet in any case by the High
court.
(c) The Supreme Court cannot direct the high court upon non Reliance on the draft
charge sheet to quash the
FIR.
(d) FIR can be quashed on the mere Reliance on the draft chargesheet.
88. In the light of the above passage choose the misinterpreted option.
(a) Reliance cannot be placed upon the draft chargesheet to quash the FIR.
(b) The supreme court has decided that High court cannot quash the FIR on mere
Reliance on the draft charge
sheet that is yet to be filed before the magistrate.
(c) In certain circumstances Reliance on draft charge sheet can be placed upon.
(d) The high court in any case cannot quash the FIR on the basis of draft charge
sheet.
Passage (Q.89-Q.94): A legal notice has been sent to Commissioner of Hyderabad
Police Anjani Kumar seeking
immediate cessation of illegal search of mobile phones that are reportedly being
carried out by police officers
across the State by the court. The notice also sought the initiation of
disciplinary proceedings against the
concerned officers as per the provisions of the Hyderabad Police Act. The notice
has further sought disclosure
about whether any prior warrants or departmental instructions had been obtained by
the concerned police officers
before searching and accessing mobile phones of citizens. Reliance has been placed
on various new reports
which published that police officers on October 27, 2021 were seen stopping random
passers-by and checking
their mobile phones and private conversations for any mention of drugs. In cases
where messages with keywords
such as "ganja", "weed" and "stuff" were found, the persons were reportedly being
sent to police stations. "
Subsequent actions include demands for access to mobile phone devices carried on
their person and thereafter
proceeding to search the contents of such mobile devices including searching
private messaging applications",
the notice highlighted. The legal notice further averred that police officials have
not been conferred with any
powers under the Code of Criminal Procedure (CrPC) to stop ordinary citizens and
request them to unlock their
mobile phone devices and search the contents of their messages. "It is apparent
that the police officers are acting
on fear and abusing their wide powers and coercive presence to treat all and any
persons as suspects without any
reasonable basis.
Source Name - Live Law

. Page 23 of 36
89. P was a student who was going to his college from his bike when he was stopped
by a policeman. The police
man requested him to unlock his mobile phone and thoroughly scrolled through his
WhatsApp chats, whether he
was interested in the trading for consumption of drugs. The student went to the
court and complain about the
same. A legal notice was sent to the commissioner of police. in the light of the
above passage choose the correct
option.
(a) The court was justified in issuing the legal notice.
(b) The court was not justified in issue in the legal notice as it was the duty of
the police to discharge its official
duty and take a track the consumption and trading of the drugs in the state.
(c) The police have not influenced the boy’s right to liberty and right to privacy.
(d) There was no infringement of right to Liberty and right to privacy.
90. L was going somewhere with his friend when they were stopped by the policeman
who requested both of them
to unlock their mobile phones and allow the policeman to scroll through their
WhatsApp chats as to confirm that
they were not in dust in the trading or consumption of drugs. This action of the
police was challenged by the boy
in the court and the court issued a legal notice to the commissioner of police. In
the light of the above passage
choose the correct option.
(a) The right to privacy and right to Liberty of the boy and his friend were
infringed.
(b) The boy was not liable to allow the policeman to scroll through his WhatsApp
chats to confirm whether he
was interested in the trading of drug and consumption.
(c) The police have the power to investigate for the same and does was not liable
for the legal notice by the
court.
(d) The court was not justified in sending the legal notice to the police.
91. J and k were going to their colleges on their bikes when they were stopped by
two policemen who were searching
the WhatsApp chats of the students to confirm whether they were industry in
consumption of drugs. When the
police came to K, they found the word weed in one of his chats and immediately he
was sent to the police station.
In the light of the above passage choose the correct option.
(a) The police was not justified to send the student to the police station on a
mere basis that the word weed was
written in his chats.
(b) The police was discharging his official duty and the boy was liable to be sent
to police station as the word
weed was written in his chats.
(c) According to the passage the police had the authority to randomly check the
WhatsApp chats of the people.
(d) The fundamental right to privacy of the boy was infringed.
92. B was sent to the police station by the police man upon searching his random
WhatsApp chats in which the
words like drugs and weed were found in many places. However practically he was not
interest in these things
but the police suspected that the repetition of these words imply that the boy was
interested in the same drug
practices and thereby stopped him. This was challenged by the boy and a legal
notice was sent to the
commissioner of police. In the light of the above passage choose the correct
option.
(a) The police had the power to investigate whether a person was interested in the
malpractices like drugs and
weed.
(b) The police had no authority to investigate the random WhatsApp chats.
(c) The right to liberty and the right to privacy was infringed in this case.
(d) The legal notice was not justified to be sent to the police.
93. In the light of the above passage choose the correct option.
(a) The police have the full authority to randomly check the WhatsApp chats to
ensure whether the person is
interested in the malpractices of drugs and weed.
(b) The police have no legal basis to randomly check the chats of the people.
(c) The police is justified in randomly checking the chats as it is a part of the
discharge of its official duty.
(d) Police have the absolutely authority to check a WhatsApp chats backed by the
legal bases of CRPC.

. Page 24 of 36
94. In the light of the above passage choose the incorrect option.
(a) The police has no legal basis to randomly check WhatsApp chats.
(b) The police cannot search the random WhatsApp chats of the people in absolute
sense as according to the
passage.
(c) In the present case in the passage the legal notice was primarily sent to curb
the illegal action of the police
in the veil of investigation.
(d) There is a discharge of the official duty of the police and thus the police can
check the WhatsApp chats of
the people.
Passage (Q.95-Q.100): The Delhi High Court has disposed of the plea moved by Pinjra
Tod activist, Devangana
Kalita, seeking copies of the video footage pertaining to violence during an anti-
CAA protest at Jafrabad on
February 23 last year. She is an accused in the case. Justice Subramonium Prasad
however said that it will be
open for the Magistrate to consider Kalita's application afresh in case she files
the same. The Court also observed
that a reading of the Trial Court's order rejecting her application indicated that
the same was passed because the
investigation qua other accused persons in the matter was not completed. " Now the
investigation is complete.
The stage of taking cognizance has reached” Court added. Devangana was arrested by
the Delhi Police on May
23 last year in relation to the FIR registered over Jafrabad sit-in protest. She
was granted bail by the Delhi High
Court on a finding that the Delhi police had failed to produce any material to show
that she instigated women of
a particular community or gave hatred speech leading to violence. Devangana had
demanded copies of the video
footage relating to violent incidents in Jafrabad. Earlier, she had moved the Trial
Court stating that even though
a final report had been filed by the Delhi Police, she was not provided with the
electronic material that formed
part of such report. Her request was however denied by the Court of a Metropolitan
Magistrate, stating that the
plea was 'premature' and the Court is yet to take cognizance of the final report.
Source Name - Live Law
95. S was an activist who was accused for hate speech in a state riot. She had
demanded the video footage of the
state riot which was however denied by the court. However at that stage when the
investigation was going on
the case was considered premature. In the light of the above passage choose the
correct option.
(a) The court was not justified in denial of the same as the case was premature.
(b) As there was investigation going on and the case was premature the court could
not provide the activist the
video footage of the riot in the state.
(c) The court was not justified in denying the activist the video footage as it was
the right of an accused to gain
the same.
(d) The court was justified in denial of production of the video footage.
96. An activist was accused of splitting hatred among the community's during the
riot by her hate speech. Where in
custody she demanded the video footage of cctv of the state riot in which she was
seen spreading hatred by her
head speech. The case was being investigated by the police and the court denied the
production of the video
footage to the accused. In the light of the above passage choose the correct
option.
(a) The court was justified in not giving the video footage.
(b) The court was not justified in denying the production of the same to the
accused.
(c) The court was justified in denial of the production of the CCTV footage to the
accused as the investigation
was going on.
(d) When the investigation was going on the accused had no right to claim any video
footage from the court.

. Page 25 of 36
97. When the court is yet to take cognizance the case is deemed to be premature. L
was accused of spreading hatred
in a state riot by his hate speech. In the course of time he was arrested where the
police failed to produce the
video footage of him spreading the hatred by his hate speech. However he demanded
the video footage which
was denied by the court because the court hadn't taken cognizance yet. In the light
of the above passage choose
the correct option.
(a) The court was not justified in denying the production of the video footage.
(b) The court was justified in denying the video footage as the case was premature.
(c) As the case was not premature and police failed to produce the video footage of
the same the court was not
justified in denying the production of video footage.
(d) The court was justified in denying the production of video footage.
98. M was arrested by the police for spreading hatred by his hate speech in state
riot. However there was CCTV
footage of the same but the police for some consequential reasons failed to produce
the same to the accused
however this was challenged by the accused in the court. The investigation was
completed and only it was upon
the court to take cognizance. The court however denied the production of the
footage.. In the light of the above
passage choose the correct option.
(a) The court was not justified in denying for the same as the investigation was
complete.
(b) The court was justified in denying for the same as it was upon the court to
take cognizance and then supplied
the video footage to the accused.
(c) The court was not justified in denying the video footage to the accused.
(d) The case was not premature the judge was obliged to provide the video footage
to the accused.
99. In the light of the above passage choose the correct option.
(a) The court in every sense after it had taken the cognizance and the
investigation is completed obliged to
provide the accused with the video footage as required by the accused.
(b) The court is obliged to produce any video footage if the investigation has been
completed.
(c) The accused has the right to be provided with the video footage in any case.
(d) Prematurity of the case is not very important factor for production of video
footage to the accused.
100. In the light of the above passage choose the misinterpreted option.
(a) The prematurity of a case has relevance in the aspect of producing the video
footage to the accused.
(b) It is upon the court to take cognizance after the investigation has been done
in aspect of producing the video
footage to the accused.
(c) The accused owning a right to be provided with the video footage after the
court has taken cognizance of the
same is not an important factor according to the passage.
(d) If the court has not taken the cognizance and the investigation is not complete
the court is not bound to order
the production of video footage to the accused.
Passage (Q.101-Q.105): Chief Metropolitan Magistrate Gajender Singh Nagar was of
the opinion that the
testimony of the woman on her allegation of being pulled by hair and arm by Khosla
and the threat that she will
not be allowed to practice from Tis Hazari Court was "absolutely truthful and
creditworthy."
The Court was of the view that the act of pulling someone from hair and arm would
result in bodily pain and
thus offence under Section 323 (voluntarily causing hurt) IPC was made out as
bodily pain was inflicted
voluntarily on the complainant.
On the argument that the complainant could not bring any independent witness to
corroborate her version, the
Court was of the view:
"It is common knowledge that nowadays people are becoming self-centered and they
find it safe to keep numb
even if they see an injustice being done to any person. This is becoming a harsh
reality these days. These days
nobody comes forward to save someone or to stand witness for someone unless and
until one has personal
interest in the matter."
"The complainant was an advocate, she was aware about all the legal provisions, if
she had to cook up a story
she could have easily made up a story that in the late hours after dark she was
attacked or molested by the

. Page 26 of 36
accused which is not the case here. In the present case the complainant has alleged
the attack in the broad
daylight in presence of a number of lawyers, soon before the visiting of Civil
Judge. It is impossible for a person
to so minutely cook up a story as allegedly done by the complainant."
101. Shriram was an employee of RVS Technologies, which is a product-based IT
company. He was a dedicated
employee, and earned respect for working with tight deadlines. One fine day, the
management decided to merge
RVS Technologies with another IT company to fulfil the goal of expansion. After the
merger, Shriram was
transferred to another office, where Mr. Rajeshwar was the team lead. Mr. Rajeshwar
was not very welcoming
to Shriram, and they started having disagreements very often. Further, they had a
heated argument where Shriram
hit Rajeshwar with a heavy stick. However, the attack had just caused bruises to
Rajeshwar. Now, he wants to
file a complaint against Shriram under Section 323 for causing hurt. Decide.
(a) Shriram is liable under Section 323, since he threw a heavy stick on Rajeshwar.
(b) Shriram is not liable under Section 323, since his act merely caused bruises.
(c) Shriram will be liable under Section 323, since he voluntarily inflicted
injuries on Rajeshwar.
(d) Shriram will not be liable, since he did this during an argument where
Rajeshwar was aggressive as well.
102. Suppose in the previous question, Shriram threw the heavy stick which missed
Rajeshwar by an inch. Will this
change your answer?
(a) Yes, since this was an intentional attack.
(b) Yes, since there was no injury inflicted upon Rajeshwar.
(c) No, since Shriram started the disagreement by himself.
(d) No, since all the conditions listed under Section 323 are satisfied.
103. Ramesh and Suresh were neighbours and attended the same tuition classes while
preparing for engineering
entrance examination. Both of them were exceptionally good in studies, and used to
score almost the same in
every mock test. Due to this, they were always competing with each other and soon
became rivals. They started
having frequent fights, to prove their mettle. On one fine evening, Ramesh was
flying his remote-control
helicopter near his house. In order to tease Suresh, Ramesh started flying his
helicopter over him. Suddenly, the
remote stopped working and the helicopter fell on Suresh’s head. This caused some
serious injury to Suresh,
who decided to sue Ramesh under Section 323 of IPC. Decide.
(a) Ramesh is liable, since he was intentionally flying the helicopter over Suresh.
(b) Ramesh is not liable, since he was in complete control of the helicopter before
it broke down.
(c) Ramesh is liable, since his act caused serious injuries to Suresh.
(d) Ramesh is not liable, since he had no intention of hurting Suresh.
104. Suppose in the previous question, Ramesh knew beforehand that the helicopter’s
remote is in a dilapidated
condition and might stop working at any point. Will this change your response to
the previous question?
(a) Yes, since Ramesh knew that the helicopter might break down and he still went
on flying it over Suresh.
(b) No, since Ramesh still had no intention of hurting Suresh.
(c) Yes, since Ramesh wanted to tease Suresh due to their enmity.
(d) No, since Ramesh was trying his best to have complete control over the
helicopter.
105. Shivam and Shyam were the captains of their respective school’s cricket teams.
Both of them were very
passionate about the game, and hence played with utmost dedication. During one of
the matches between these
two teams, Shivam sledged Shyam to distract him. Although Shyam ignored this at
that point of time, he was
convinced that Shivam needed to be taught a lesson. Once the match was concluded,
Shyam and his friends
surrounded Shivam and threatened him that they would beat him to death if he
sledged him in future. After this
heated conversation, they allowed Shivam to leave. During the way back to his home,
Shivam was under extreme
emotional turbulence, which resulted in an accident. Now, Shivam wants to sue Shyam
for his injuries under
Section 323. Determine the liability of Shyam.
(a) Shyam is liable, since he threatened Shivam which later on led to the accident.
(b) Shyam is not liable, since he did not inflict any sort of physical injury on
Shivam.
(c) Shyam is liable, since he voluntarily caused hurt to Shivam.
(d) Shyam is not liable, as his acts were completely unconnected to Shivam’s
injury.

. Page 27 of 36
SECTION - D: LOGICAL REASONING
Directions (Q.106-Q.135): Read the following passage carefully and answer the
questions that follow.
Passage(Q.106-Q.110): Among literally dozens of new books on Apollo, all timed to
coincide with the 50th
anniversary of the Moon landing in July 2019, two books exemplify both old and new
tropes. James Donovan’s
Shoot for the Moon has been touted by none other than Apollo 11 astronaut Michael
Collins, who recently
remarked, “This is the best book on Apollo that I have read.” Donovan’s strength is
his breezy and journalistic
writing style that weaves together a vast and complicated set of stories from all
levels of Apollo.
If Donovan’s book is a by-the-numbers retelling that is also a familiar one, Eight
Years to the Moon by Nancy
Atkinson takes us into less-familiar aspects of the history, benefiting in many
ways from a burst of recent
scholarship. This book functions as a kind of corrective to the ubiquitous heroic
narrative of politicians and
astronauts.
The overall beats of Donovan’s story will be familiar to many. For example, he
captures the intimate world of
Wernher von Braun, the visionary German rocket designer who was captured by
Americans after World War II.
Von Braun was brought to Texas and eventually to Alabama, where, in the 1960s, he
directed the project to
develop the giant Saturn V rocket that sent Apollo to the Moon.
Very succinctly, Donovan describes von Braun as “a handsome, charming ex-SS officer
who had been the chief
architect of an ambitious rocket program that had killed thousands during the war—
and who now spread the
gospel of space exploration to Americans in Walt Disney TV specials.” What he
leaves unsaid is as important:
that more people died building the V2—as slave prisoners in Nazi concentration
camps—than the rocket actually
killed. Although Donovan glosses over this, Von Braun’s team’s culpability for
crimes against humanity is
fundamental to any reckoning with the legacy of Apollo.
Donovan’s narrative is a well-crafted one. It is one of the rarest of Apollo books
that manages to weave together
the political, the technical, and the heroic. We get vivid descriptions of managers
struggling with difficult
decisions, such as the directive that sent the first Apollo spacecraft around the
Moon in late 1968, a
recommendation that was one of the riskiest in the entire program.
In Donovan’s hands, the actual landing mission, Apollo 11, reads like a thriller.
We know the ending, of course,
but the contingencies of the mission—points at which the flight could have failed—
are so many, that their
cumulative power communicates how minuscule the margins were. In a million possible
scenarios, the Eagle
lunar module lander crashes on the Moon with no way back for the astronauts. They
did, of course, come back,
and Donovan’s narrative wraps up rather abruptly with a few cursory paragraphs
about the postflight visits of
the Apollo 11 astronauts to foreign countries. He concludes on a pithy note: “By
the time the crew returned
home, they understood that their lives would never be the same.”
106. Which one of the statements given in options presents a contrast to the
following sentence as mentioned in
paragraph 2?
“If Donovan’s book is a by-the-numbers retelling that is also a familiar one, Eight
Years to the Moon by Nancy
Atkinson takes us into less-familiar aspects of the history, benefiting in many
ways from a burst of recent
scholarship.”
(a) Atkinson's book is more likely to surprise the reader.
(b) Atkinson's readership consists mostly of scholarship students.
(c) Most books have a predicated storyline.
(d) Donovan's book uses a linear narrative.

. Page 28 of 36
107. According to the passage, which of the following are the ‘old tropes’
exemplified in James Donovan’s Shoot for
the Moon?
I. Involvement of Nazi scientist.
II. Characters with heroic narratives.
III. Familiar characters.
IV. Known timeline.
(a) I, II and III (b) II and IV (c) II, III and IV (d) I, II, III and IV
108. Given below is a possible inference that can be drawn from the facts stated in
the third paragraph. You have to
examine the inference in the context of the passage and decide upon its degree of
truth or falsity.
“Wernher von Braun was among the chief architects of the Saturn V rocket that sent
Apollo to the Moon”
(a) Definitely true (b) Probably true
(c) The data are inadequate (d) Probably false
109. Which of the following is not an assumption that supports the arguments
presented in the fourth paragraph?
(a) Von Braun was interested in the science behind the rockets, rather than their
implications.
(b) The high labour costs for building the V2 rockets alone made it an ambitious
project.
(c) Donovan skimming over von Braun’s criticism was a half-truth.
(d) The success of a venture may be hiding ugly truths.
110. The Author has criticized which one of the following factors of James
Donovan’s Shoot for the Moon?
(a) Over dramatization of politics.
(b) Both C & D
(c) The ending.
(d) Von Braun's actions during World War II.
Passage(Q.111-Q.115): Health is a very relatable topic for children. It plays a
crucial role in the physical and
mental development of a child. They can shape their life by maintaining their
health at this stage of life. Providing
a healthy life to children is the liability of their parents. The health of the
children depends much on the awareness
of their parents. Many factors affect the health of an individual or the entire
family. Some of these factors are
income & social status, education, environment, culture, health facilities, etc.
All of these factors hugely affect
the health of the people. In many cases, gender becomes an important factor in
deciding the health of people.
The women in society struggle a lot in the absence of all the required facilities.
There are some basic rules that
we can follow to avoid health issues. The first thing that we should work on is
'stress'. Stress is the mother of all
diseases. Though mental stress is natural and inevitable, holding it for a long
time can make us sick. We should
avoid having mental stress for a long time. Waking up early in the morning and
having a walk can help you best
in your health. This is the only time we get the fresh air to breathe in. The
morning walk is like a whole-body
exercise. It provides energy to work for the complete day tirelessly. Being
positive and social with your
environment is also helpful in keeping you healthy. Good health is the need and
desire of every individual. A
healthy life has become very rare in the current polluted world. A healthy life is
the result of routine exercise, a
decent diet, and enough meditation. A healthy body refers to a balanced body;
neither an underweighted nor an
over weighted one. An unbalanced body is an invitation to many dangerous diseases
like stroke, diabetes, high
blood pressure, gout, Osteoporosis, Anaemia, etc. Being physically fit will bring
the advantage of staying away
from these diseases. Our mood will also be cheerful and active all the time, and
thus, we can be an inspiration
for others. We experience quick fatigue during illness. We feel like we don't have
energy. It is because of
nutrition that our body lacks. A healthy body will always remain energetic and
active. It is also necessary to live
a longer, happier, and blessed life. Mental and social activeness is equally
necessary for us. Our various emotions
and our ability to deal with them imply our mental and social health. Every one of
us goes through many stresses
daily.

. Page 29 of 36
111. What can be inferred from the given passage?
(a) A sound body will consistently stay fiery and apathetic to stress. It is
likewise important to live a more
extended, more joyful and favoured life.
(b) A fit and healthy life is the aftereffect of routine exercise, a good eating
regimen, and enough reflection. A
salubrious body alludes to a fair body; neither an underweighted nor an over
weighted one.
(c) Mental and social vivacity are equally trivial for us as physical health. Our
different feelings and our capacity
to manage them infer our psychological and social wellbeing.
(d) The principal thing that we should deal with is 'stress'. Stress is the mother,
everything being equal. Even
though the psychological pressure is regular and avertible, holding it for quite a
while can wipe us out.
112. Which of the following statements support the main idea discussed in the
passage?
(a) A wholesome wellbeing of an individual, a child or the whole family depend on
various factors such as
economic, cultural, social, education and management of stress. These variables
gigantically influence the
soundness of the individuals.
(b) Wellbeing is a truly relatable point for youngsters. It assumes an essential
part of the physical and mental
advancement of a kid. They can shape their life by keeping up their wellbeing at
every phase of life.
(c) A healthy life is the result of routine exercise, a decent diet, and long hours
of meditation that can work
wonders on the overall well-being of people.
(d) Women neglect their health by being health conscious of their families, and in
the process taking too much
stress upon themselves by neglecting their well-being.
113. ‘Stress is the mother of all diseases.’ Which of the following is the
assumption behind the statement?
(a) No diseases mean no stress
(b) The more the stress, the more the diseases.
(c) Stress begets diseases.
(d) Stress helps combat diseases
114. Which of the following statements provides honest information about the
passage?
(a) Getting up late in the early mornings and having a walk can help you best in
your wellbeing.
(b) Being positive and socially active at all costs of your current circumstances
are useful in keeping you going.
(c) An unsound body will consistently stay enthusiastic and dynamic.
(d) Being genuinely fit will bring the upside of avoiding infections and diseases.
115. What are the factors that affect the wellbeing of a person?
(a) Competent diet (b) Economic wellbeing
(c) Regular activities (d) Emotional well-being
Passage(Q.116-Q.120): The issue isn’t only that sustainable business practices
don’t scale, though this is true
and pertinent. The problem is that they displace meaningful action. A business that
pursues “sustainability” as
conventionally understood becomes, in the media’s eye and in customer perception, a
“green” company,
absolved of doing anything else. Such firms don’t have to undertake the hard work
of political activism that
might actually drive down global emissions like political advocacy, use of public
voice, testimony in
Washington, noisy, uncomfortable coalition building and peer pressure, divestment,
or public calling-out of bad
behaviour. Hell—they don’t even need to cut their emissions to be labelled a
leader. They just need to aspire to
it. But this approach may actually be worse than a distraction.
In so many cases, the dodge becomes open duplicity. Corporations learned to weave
the funeral shroud of their
carbon credits and waste-reduction by day and undo it at night with their corporate
contributions and lobbyists.
“Leaders” in the corporate sustainability movement are more or less silent on
policy. Meanwhile, with the vast
bulk of their action and messaging focused on how they are greening their own
operations, they give money to
politicians that are climate deniers. The duplicity often extends beyond climate
into social responsibility and
justice, causes that are part and parcel of the climate movement.

. Page 30 of 36
Over the years, these sorts of dismal discoveries led me to an even unhappier
place: The actions businesses take
under the banner of win-win, profitable, and good-for-the-planet corporate
sustainability were exactly and
precisely what the fossil fuel industry would want them to do. These moves ensure
that businesses take
responsibility for the climate problem only as their own individual emissions
challenge, instead of seeing it as a
systemic issue. It creates a focus on sustainability actions so lame and small-ball
that they could never and would
never disrupt the fossil fuel industry’s hammerlock on governance.
116. Which of the following statements is in line with the author’s line of
thoughts?
(a) Sustainable business practices are a hoax in the present times.
(b) No corporate entities practice sustainability in actuality.
(c) Sustainable business practices are used as a garb to cover for the shift from
significant action.
(d) It is impossible to force corporates to practice real sustainable business
practices.
117. Which of the following can be inferred from the passage?
(a) Intent to cut emission is considered not good enough to be termed as a
visionary in the industry
(b) Corporation that are big on sustainable business practices rarely comment upon
policy
(c) Fossil fuel industry would want reduction in global emissions.
(d) Sustainable business practices participate in political activism.
118. Which of the following, if true, would seriously undermine the author’s
argument?
(a) Without the aid of corporates, it may not be possible to reach sustainable
action in reducing global
emissions.
(b) Majority of corporates are actually invested in turning green but cannot find
the right infrastructure that
will bring a considerable change.
(c) The governmental policies are needed to be upgraded as they are archaic.
(d) Pollution caused by individuals is way more harmful than that contributed by
firms.
119. The statement “The corporate’s approach towards sustainability is
individualistic rather than systemic” is:
(a) Probably True (b) Definitely True
(c) Probably False (d) Definitely False
120. What could be an apt heading for the passage?
(a) Corporates are lying to their consumers
(b) The Complicity of Corporate Sustainability
(c) Government needs to bring a change
(d) Climate change is real: Here’s how to fight it
Passage(Q.121-Q.125): It is extremely important to clarify what exactly early
intervention is. A program is
considered “early” if it occurs from before birth until early adolescence, and
before the onset of delinquent
behavior. Research has shown that the later the intervention occurs in the child’s
life, the more therapeutic effort
is required to return the child to a pattern of normal development. The results of
high-quality early prevention
programs can be tremendous. Looking specifically at preschool programs and parent
educational services that
improve school readiness, they help to set a pattern that prevents delinquency in
later years. Children who
participate are less likely to drop out and perform delinquent behavior because
they have had better early school
experiences and a stronger commitment to education. Early interventions also show
increases in IQ scores and
executive functioning, better elementary school achievement, and lower rates of
aggression and other antisocial
behavior.
Early intervention programs offer a support system of parental involvement and
education that works to improve
family functioning and with that, child functioning. This aspect of dealing with
the family also makes these
programs more comprehensive, which is another factor of good programs. An important
point to make is that no

. Page 31 of 36
child is inaccessible. In fact, the greater risk factors a child has, the more they
will benefit from additional support
such as a strong and encompassing program.
Even in terms of cost, these programs succeed. Especially since today the majority
of money in crime prevention
goes towards incarceration. If that same money could be used for prevention
programs instead, the results would
be outstanding.
121. Which of the following can be inferred from the passage?
(a) Prevention programs can function even without family support
(b) Early interventions start as early as prenatal to almost adolescence.
(c) Juvenile delinquency can be anticipated from birth itself
(d) Early intervention stalls the mental development of a child.
122. Which of the following statements will the author deem the truest?
(a) Children through prevention programmes return to a pattern of normal
development before the onset of
delinquent behaviour.
(b) Children in prevention programmes are programmed to turn law abiding citizens
later in time
(c) Children in prevention programmes achieve higher IQs than others.
(d) Children in prevention programmes are trained to give therapeutic training
later in life.
123. The author’s statement that “Children who participate are less likely to drop
out and perform delinquent behavior
because they have had better early school experiences and a stronger commitment to
education” forms
(a) A premise of the above passage
(b) A conclusion of the above passage
(c) An assumption of the above passage
(d) An inference of the above passage.
124. Which of the following assumptions has been made by the author?
(a) Instead of spending on incarceration, if money could be used for prevention
programs, the results would be
outstanding.
(b) Many people don’t know what exactly “early intervention” is.
(c) Children who require prevention programs have low IQ before the start of the
programs.
(d) Early intervention is required for every child from inception.
125. What does the author indicates in the last paragraph?
(a) The cost of early prevention programs is high.
(b) The cost incurred for prevention programs is better than that on incarceration.
(c) Children who require prevention programs are unable to afford it for
themselves.
(d) The cost incurred on incarceration cannot be compared with the cost generated
for intervention
programme.
Passage(Q.126-Q.130): Below is given a passage followed by several possible
inferences that can be drawn
from the facts stated in the passage. You have to examine each inference separately
in the context of the passage
and decide upon its degree of truth or falsity.
In commodities’ business, size does matter. This is common wisdom. The Indian Sugar
Industry, the second
largest in the world after Brazil, has traditionally been fragmental, which led to
widespread sickness and a large
number of mills going bankrupt, a situation exacerbated by a slew of government
controls which are
meaningfully, getting diluted since August 1998. It’s now been more than seven and
a half years since the
industry was de-licensed. No official permission is required either to build a new
factory or for brownfield
expansion plan, except that there must not be any violation of command area norms,
even then, there aren’t many
who have the capacity to play the volumes game at the cyclic sugar business.

. Page 32 of 36
126. India has not yet been able to consolidate its firm stand in the international
sugar market.
(a) Inference is definitely true, i.e., it properly follows from the statement or
fact given.
(b) Inference is probably true though not ‘definitely true’ in the light of the
facts given.
(c) ‘data are inadequate’ i.e. from the facts given you cannot say whether the
inference is likely to be true or
false.
(d) Inference is ‘probably false’ thought not ‘definitely false’ in the light of
the facts given.
127. At present the Indian sugar industry has been made considerably free from
government controls.
(a) Inference is definitely true, i.e., it properly follows from the statement or
fact given.
(b) Inference is probably true though not ‘definitely true’ in the light of the
facts given.
(c) ‘Data are inadequate’ i.e., from the facts given you cannot say whether the
inference is likely to be true or
false.
(d) Inference is ‘probably false’ thought not ‘definitely false’ in the light of
the facts given.
128. Prior to 1998, Indian sugar industry was considerably lower in the world
ranking or large nations.
(a) Inference is definitely true, i.e. it properly follows from the statement or
fact given.
(b) Inference is probably true though not ‘definitely true’ in the light of the
facts given.
(c) ‘Data are inadequate’ i.e. from the facts given you cannot say whether the
inference is likely to be true or
false.
(d) Inference is ‘probably false’ thought not ‘definitely false’ in the light of
the facts given.
129. Most of the bankrupt sugar mills in India are funded by the government to
revive their units.
(a) Inference is definitely true, i.e., it properly follows from the statement or
fact given.
(b) Inference is probably true though not ‘definitely true’ in the light of the
facts given.
(c) ‘Data are inadequate’ i.e., from the facts given you cannot say whether the
inference is likely to be true or
false.
(d) Inference is ‘probably false’ thought not ‘definitely false’ in the light of
the facts given.
130. The Indian sugar Industry has comparatively smaller units in comparison to
other major sugar producing nations.
(a) Inference is definitely true, i.e., it properly follows from the statement or
fact given.
(b) Inference is probably true though not ‘definitely true’ in the light of the
facts given.
(c) ‘Data are inadequate’ i.e., from the facts given you cannot say whether the
inference is likely to be true or
false.
(d) Inference is ‘probably false’ thought not ‘definitely false’ in the light of
the facts given.
Passage(Q.131-Q.135): Ms. Ryan and Ms. Markova have found what they call three
zones of existence: comfort,
stretch, and stress. Comfort is the realm of existing habit. Stress occurs when a
challenge is so far beyond current
experience to be overwhelming. It’s that stretch zone in the middle activities that
feel a bit awkward and
unfamiliar where true change occurs.
“Getting into the stretch zone is good for you,” Ms. Ryan says in “This Year I
Will...” “It helps keep healthy. It
turns out that unless we continue to learn new things, which challenges our brains
to create new pathways, they
literally begin to atrophy, which may result in dementia, Alzheimer’s and other
brain diseases. Continuously
stretching ourselves will even help us lose weight, according to one study.
Researchers who asked folks to do
something different every day listen to a new radio station, for instance, found
that they lost and kept off weight.
No one is sure why, but scientists speculate that getting out of routines makes us
more aware in general.” She
recommends practicing a Japanese technique called kaizen, which calls for tiny,
continuous improvements.
“Whenever we initiate change, even a positive one, we activate fear.” “If the fear
is big enough, the small steps
in kaizen don’t set off fight or flight, but rather keep us in the think creativity
and playfulness.” Simultaneously,
take a look at how colleagues approach challenges, Ms. Markova suggests. We tend to
believe that those who
think the way we do are smarter than those who don’t. That can be fatal in
business, particularly for executives

. Page 33 of 36
who surround themselves with like-thinkers. If seniority and promotion are based on
similarity to those at the
top, chances are strong that the company lacks intellectual diversity.
“Try lacing your hands together,” Ms. Markova says. “You habitually do it one way.
Now try doing it with the
other thumb on top. Feels awkward, doesn’t it? That’s the valuable moment we call
confusion when we fuse the
old with the new.”
AFTER the churn of confusion, she says, the brain begins organizing the new input,
ultimately creating new
synaptic connections if the process is repeated enough. But if, during the creation
of that new habit, the “Great
Decider” steps in to protest against taking the unfamiliar path, “you get
convergence and we keep doing the same
thing over and over again,” she says. “You cannot have innovation,” she adds,
“unless you are willing and able
to move through the unknown and go from curiosity to wonder.
131. Three situations in the form of statements are presented. Categorise each in
comfort, stretch, or stress. Choose
from the options that accurately represents the categories.
Situation I. A boy unable to cope with the engineering course recedes to
depression.
Situation II. A resolution by a man to pick up a new skill every year to have the
cutting edge in beauty industry.
Situation III. A doctor taking rounds in the ICU ward as a routine.
(a) I-Comfort, II-Stretch, III-Stress (b) I-Stress, II-Stretch, III-Comfort
(c) I-Stretch, II-Comfort, III-Stress (d) I-Stress, II-Comfort, III-Stretch
132. ‘It turns out that unless we continue to learn new things, which challenges
our brains to create new pathways,
they literally begin to atrophy, which may result in dementia, Alzheimer’s and
other brain diseases.’ Which of
the following is the assumption behind the statement?
(a) Learning new activities keep cerebral diseases at bay.
(b) A challenging activity recedes brain diseases.
(c) Stagnancy leads to degeneration of the organs.
(d) Learning new things everyday keeps the brain functional.
133. All of the following are not inferences, except
(a) Kaizen is a Japanese technique.
(b) Getting into the stretch zone is overwhelming.
(c) “Whenever we initiate change, even a positive one, we activate fear.”
(d) The reasons are unknown for the loss in weight due to learning new activity.
134. Identify the statements as based on the given options
Statement I: It helps keep healthy.
Statement II: Getting into the stretch zone is good for you.
(a) Statement I is an assertion and statement II is the reason.
(b) Statement I is a reason and statement II is the assertion.
(c) Both statements are assertions.
(d) Both statements are reasons.
135. Which of the following will weaken the contention given in the passage?
(a) Japanese technique kaizen evades the extreme fight or flight mode.
(b) Intellectual diversity is a dissonance in thinking leading to new and
innovative researches.
(c) The brain is unable to process a new learning skill that challenges mental
acuity, but responds to a strenuous
physical activity.
(d) A state of confusion in the brain leads to processing the new input, ultimately
creating new synaptic
connections if the process is repeated enough or as an isolated case of recognising
the process in the first
attempt.

mock 23
Directions (Q.1-Q.30): Read the following passage carefully and answer the
questions that follow.
Passage(Q.1-Q.5): The capabilities that scientists have developed to engineer the
genetics of plants and animals
could have a profound impact on the control of insects and insect-related diseases.
One approach would be to
introduce, into the pest population, individuals that have been genetically altered
to carry genes that interfere
with reproduction. Researchers are also evaluating the possibility of replacing
natural mosquito populations with
populations that are unable to support normal parasite development. In addition,
work is underway to produce
mites and insects that are highly effective enemies of crop pests. For instance, a
transgenic version of a mite is
helping to control spider mites in almond orchards. There are, however, associated
concerns. One is that
arthropods (in contrast to transgenic crops) have the ability to make sudden
changes in their diets and even in
their genes, coupled with the fact that they can readily move from one area to
another. Furthermore, it may be
that the foreign gene from the genetically engineered organism can be transferred
to other arthropod species.
Other researchers are directing their attention to functional analysis of the genes
that assure the virulence of
various parasites. One of the most vicious malaria parasites, for example, protects
itself by manufacturing a
protein that expels drugs before they can be fully effective. Having identified and
isolated these genes, scientists
are attempting to develop a compound that will interfere with the action of the
protein. This in turn, should permit
engineering the genes within the parasite to either prevent or reverse the
resistance of these organisms to various
drugs
In related studies, genetic engineers have achieved impressive successes in
designing agricultural plants with
improved resistance to insects and viruses. Initially such efforts were directed to
improving plant resistance.
Today the aim has been expanded to include protecting the harvested crop. In
certain crops (cereal grains, beans,
peas) weevils and other insects can cause losses during storage equal to those
incurred during the growing season.
1. According to the passage, why is it insufficient to control plant damage or
losses due to pests?
(a) Plants can resist pests whereas harvested crops cannot.
(b) Researches have so far been working on improving plant resistance only.
(c) Damage to stored crops, by pests, is equally prevalent.
(d) Plants are resistant to pesticides.
2. Which of the following describes the organisation of the passage?
(a) A hypothesis is made, and the various implications are discussed.
(b) An observation is made and the different aspects are detailed to support it.
(c) A specific case is presented, the pros and cons are discussed and a conclusion
is arrived at.
(d) A generalization is made, specific cases are provided to substantiate it.
3. ‘Efforts towards producing mites and insects that are highly effective enemies
of crop pests’ is a case of
(a) a poison used as antidote to another poison.
(b) developing a population of genetically altered species to counter the harmful
ones.
(c) killing damage causing insects by insects of the same breed.
(d) using chemicals as pesticides to eradicate the entire population of mites and
insects.
4. The use of transgenic versions is likely to be ineffective in arthropods because
of all of the following abilities of
arthropods, EXCEPT:
(a) their migration.
(b) transformation of genes.
(c) ability to transfer genetically engineered foreign genes to others.
(d) resistance to insects and viruses.

. Page 3 of 36
5. Select the word which is most nearly the SAME in MEANING to the word VIRULENCE,
as used in the
passage.
(a) malignity (b) forbearance (c) courtesy (d) solitude
Passage(Q.6-Q.10): Archimedes of Syracuse is hailed by some as the greatest
mathematician of antiquity.
Ignoring a clunky numerical system and devising his own, he demonstrated the nature
of infinity and calculated
a value for pi. He realized that the volume of an object can be measured by how
much water it displaces, a
discovery he is said to have made in the bath and which caused him to run naked
through the streets of the city
shouting “eureka” (“I have found it”). Now, a new use for an old particle
accelerator is poised to reveal more of
his thoughts.
Research at the Stanford Synchrotron Radiation Laboratory in California, led by Uwe
Bergmann, will attempt
to read the final pages of the oldest known manuscript of Archimedes’ palimpsest.
Much of the text has already been read, but between 10% and 20% remains hidden. In
places, the ink from the
original text is no more than a faint stain, so the scientists studying it need a
technique that is sensitive to very
low concentrations of ink. The answer, they hope, will be X-ray fluorescence
imaging. The ink contains iron,
and traces of iron can be made to reveal themselves when bombarded with X-rays.
Iron atoms have 26 electrons
in different orbitals around their nuclei. An X-ray turned to the correct energy
can knock an electron out of the
innermost orbital. This makes the system of orbitals unstable and an electron from
an orbital further out rapidly
fills the hole. As this replacement electron falls into place, it emits an X-ray at
a second specific energy. A
detector captures each X-ray having this energy, building up an image of the ink
dot by dot.
In principle, this could be done using any suitable X-ray source, but the Stanford
Synchrotron is so powerful that
it can reveal in a day what might take others months. That power, though, comes at
a price. To ensure the beam
does not loiter on the parchment and damage it, the researchers have had to design
the procedure so that the X#ray beam cannot be on when the manuscript is
stationary.
To help read the pages damaged by forgers, the researchers also plan to fire X-rays
into the backs of the
parchment leaves and measure fluorescence from elements other than iron–including
zinc and barium – that are
found in the paint but not in the ink. They hope this will help to determine if a
mark is just paint or whether there
may be text underneath.
A two-week test run has proved successful. The first page of the manuscript, which
had previously proved
illegible, revealed some of its secrets. The X-ray imaging showed its contents.
Indeed the test was so successful
that Dr. Bergmann hopes to obtain images that will reveal missing text from “The
Method of Mechanical
Theorems”, the only copy of which is contained in the palimpsest. Such an
achievement would allow scholars
to cry out their own “eureka!”.
6. Archimedes is hailed as the greatest mathematician because
(a) he devised a numerical system to replace an unyielding one.
(b) he demonstrated the nature of infinity.
(c) he calculated a value of pi.
(d) All of these
7. The main objective of the scientists of the Stanford Synchrotron Laboratory was
to
(a) to read the final pages of Archimedes’ palimpsest.
(b) study the techniques of X-ray fluorescence imaging.
(c) check the veracity of Archimedes’ findings.
(d) to formulate mechanical theorems.
8. The difference between the Stanford Synchrotron and an ordinary X-ray source is
that
(a) the former is more accurate.
(b) the Stanford Synchrotron is faster and powerful.
(c) the latter is more expensive.
(d) an ordinary X-ray source is comparatively more sophisticated.

. Page 4 of 36
9. According to the passage, which of the following statements is NOT true?
(a) Ink of very low concentration is found in Archimedes writing on the parchment.
(b) X-ray fluorescence imaging is a technique that can highlight sensitive to very
low concentrations of ink.
(c) Apart from iron, elements like zinc and barium are also found in the ink used
in the parchment.
(d) Archimedes proved that the volume of an object can be measured by the amount of
water it displaces.
10. Which among the given options is closest in meaning to the capitalized word as
used in the passage.
ILLEGIBLE
(a) esoteric (b) indecipherable (c) ambiguous (d) explicit
Passage(Q.11-Q.15): Despite decades of governmental efforts to thwart the use of
illicit drugs, it is certain that
drug abuse will continue to be a major social problem. There are varying reasons
for this conclusion, but one
thing is increasingly clear: there will be no “quick fixes” or easy solutions to
the existing problem.
Dilemmas in searching out solutions to the drug problem are illustrated, in part,
by arguments over which drugs
are good or bad for the user. Additionally, controversy centres on the issue of
legalization of certain drugs that,
according to some, are relatively harmless to the casual user, and, when controlled
and legally prescribed, create
a series of different problems.
The use of the term “drug abuse” illustrates, in part, the confusion surrounding
the drug issue. To some people,
any use of an illicit drug is drug abuse, while to others, using a drug to the
extent that it compromises his or her
physical or psychological well-being is a more accurate interpretation of the term.
The latter definition would
infer that one can take drugs and use them responsibly (a premise that itself is
controversial).
The current state of affairs involving drug use clearly indicates that pervasive
use of dangerous substances is
widespread.
Adding to the physical dangers of substances abuse is the reality that drugs alter
a person’s behaviour.
Psychoactive drugs alter a person’s mood, perception, attitudes, and emotions. As a
result. In addition, there is
growing concern about the relationship between mind and mood altering substances
and violent crime in society
at large.
There can be no doubt that drugs abusers account for a disproportionate amount of
both violent and property
crime. Although the precise relationship between drugs and crime is still dubious,
there are several
manifestations of this relationship that deserve close attention. The first is the
relationship between crime and
addiction. Drug addicts frequently find that the cost of their habit exceeds their
ability to pay for the drugs. If the
addict cannot finance his or her habit, he or she will commonly turn to other
sources of income, such as
prostitution, burglary, and robbery, rather than seek treatment. The alternative is
to become physically ill from
the symptoms of withdrawal.
Drug users face the additional problem of acquiring their drugs in an underground
marketplace, which brings
them into contact with a wide variety of criminal actors.
Finally, drugs are both an intuitive escape and a natural occupational recourse for
many. Drugs provide a quick,
although illusory, escape from the problems of poverty, unemployment and
underemployment, poor education,
and a myriad of other social problems faced by inner-city youth. drugs provide a
quick route to material success
and accumulation of wealth for others. It has become an attractive occupational
alternative, that is an “easy way
out”.
11. According to the passage the term ‘drug abuse’ itself illustrates, in part, the
confusion surrounding the issue
because
(a) there can only be either use or abuse of a drug.
(b) all the substances used as drugs cannot be called ‘harmful’.
(c) an illicit drug is one which is unlawfully sold and so the use of the drug by
itself cannot be called ‘abuse’.
(d) for some, any illicit drug use is abuse, while for some it is an abuse only
when its use crosses an acceptable
limit.

. Page 5 of 36
12. Which of the following explains the author’s views about the nexus between drug
addiction and crime?
(a) Convinced about the relationship.
(b) Doubtful of the nexus.
(c) Doubtful of the exact nature but clear that there is a relationship.
(d) Convinced that there is no relationship.
13. The tone of the passage is
(a) cynical (b) consternating (c) satirical (d) pleading
14. Based on the passage, which of the following reflects the meaning of the word,
‘Thwart’?
(a) negate. (b) obstruct. (c) postpone. (d) neglect.
15. The word opposite in meaning to dubious is,
(a) certain. (b) problematic. (c) determined. (d) distorted.
Passage(Q.16-Q.20): Returning for a moment to the movie ideal of a mathematician,
we note that sloppy clothes
have not been the invariable attire of great mathematicians. All through the long
history of mathematics about
which we have fairly detailed knowledge, mathematicians have paid the same amount
of attention to their
personal appearance as any other equally numerous group of men. Some have been
fops. others slovens; the
majority, decently inconspicuous.
The psychological peculiarities of great mathematicians are another topic in which
there is considerable interest.
But on the general question, not much can be said till psychologists call a truce
and agree among themselves as
to what is what. On the whole the great mathematicians have lived richer, more
virile lives than those that fall to
the lot of the ordinary hard — working mortal. Nor has this richness, been wholly
on the side of intellectual
adventuresomeness. Several of the greater mathematicians have had more than their
share of physical danger
and excitement, and some of them have been implacable haters — or, what is
ultimately the same expert
controversialists.
This brings us to what at first sight may seem like a significant trait of
mathematicians — their hair-trigger
quarrelsomeness. Following the lives of several of these men, we get the impression
that a great mathematician
is more likely than not to think others stealing his work, or disparaging it. or
not doing him sufficient honour,
and to start a row to recover imaginary rights. Men who should have been ______
such brawls seem to have
gone out of their way to court battles over priority in discovery, and to accuse
their competitors of plagiarism.
Another ‘psychological” detail of a similar sort is more disturbing. Envy is
carried up to a higher level. Narrow
nationalism and international jealousies, even in impersonal pure mathematics, have
marred the history of
discovery and invention to such an extent that it is almost impossible in some
important instances to get at the
facts or to form a just estimate of the significance of a particular man’s work for
modern thought. Racial
fanaticism — especially in recent times — has also complicated the task of anyone
who may attempt to give an
unbiased account of the lives and work of scientific men outside his own race or
nation.
An impartial account of western mathematics, including the award to each man and to
each nation of its just
share in the intricate development, could be written only by a Chinese historian.
He alone would have the
patience and the detached cynicism necessary for disentangling the curiously
perverted pattern to discover
whatever truth may be concealed in our variegated occidental boasting.
16. It can be inferred that the passage is an extract from
(a) an introduction to the biographical sketches of some mathematicians of the
western world.
(b) a serious psychological study of what makes men of genius narrow minded and
parochial.
(c) a research paper trying to ascertain the precise individual contributions of
various mathematicians.
(d) an introduction to a collection entitled, ‘The eccentricities of Great
Mathematicians.

. Page 6 of 36
17. With which of the following statements would the author be least likely to
agree?
(a) A significant number of great mathematicians have been quarrelsome and mean to
their contemporaries in
the field.
(b) Mathematicians as a class revel in controversy.
(c) A serious involvement with mathematics appears to predispose a person to
peevishness and irascibility.
(d) Most mathematicians have lived richer, virile lives than other men.
18. The author’s proposal to hand over the job of unravelling the complications to
a Chinese historian is meant to.
(a) suggest the impossibility of the task that a Chinese historian is capable of
fulfilling.
(b) highlight the proverbial Chinese ingeniousness in these matters.
(c) specifically highlight the Chinese penchant for detached cynicism.
(d) suggest the impartiality of the Chinese in writing biographies.
19. According to the author, the narrow nationalism in which some of the great
mathematicians have indulged, has
led to all the following complications, except
(a) It has distorted the facts pertaining to individual contribution to
mathematics.
(b) It has harmed the progress of mathematics.
(c) It has given biased accounts of life and work of individual mathematicians.
(d) It has unnecessarily complicated the history of discovery.
20. ‘Men who should have been _______such brawls seem to have gone out of their way
to court battles over priority
in discovery…’ Fill in the correct preposition, from the given options, to make the
sentence coherent.
(a) below. (b) in. (c) above. (d) for.
Passage(Q.21-Q.25): Because economic and social phenomena are so forbidding, or at
least so seem, and
because they yield few hard tests of what exists and what does not, they afford to
the individual a luxury not
given by physical phenomena. Within a considerable range, he is permitted to
believe what he pleases. He may
hold whatever view of this world he finds most agreeable or otherwise to his taste.
As a consequence, in the interpretation of all social life, there is persistent and
never-ending competition between
what is right and what is merely acceptable. In this competition, while a strategic
advantage lies with what exists,
all tactical advantage is with the acceptable. Audiences of all kinds most applaud
what they like best. And in
social comment, the test of audience approval, far more than the test of truth,
comes to influence comment. The
speaker or writer who addresses his audience with the proclaimed intent of telling
the hard, shocking facts
invariably goes on to expound what the audience most wants to hear.
Just as truth ultimately serves to create a consensus, so in the short run does
acceptability. Ideas come to be
organised around what the community as a whole or particular audiences find
acceptable. And as the laboratory
worker devotes himself to discovering scientific verities, so the ghost writer and
the public relations man concern
themselves with identifying the acceptable. If their clients are rewarded with
applause, these artisans are deemed
qualified in their craft. If not, they have failed. By sampling audience reaction
in advance, or by pre-testing
speeches, articles and other communications, the risk of failure can now be greatly
minimised.
Numerous factors contribute to the acceptability of ideas. To a very large extent,
of course, we associate truth
with convenience-with what most closely accords with self-interest and personal
wellbeing or promises best to
avoid awkward effort or unwelcome dislocation of life. We also find highly
acceptable what contributes most to
self-esteem.

. Page 7 of 36
21. Physical phenomena do not offer the kind of luxury the author refers to because
they
(a) are difficult to understand
(b) are easy to understand.
(c) have little to do with self-esteem.
(d) are concrete and specific and can readily be verified to ascertain their
correctness.
22. In which of the following respects does the public relations man’s attitudes
significantly differ from that of the
laboratory worker?
(a) His devotion to duty.
(b) The latter’s methodical approach to his work.
(c) The latter’s search for truth.
(d) His desire to excel in his profession.
23. A suitable title for the passage would be
(a) The role of truth in social realities.
(b) Truth and human nature.
(c) Human nature in economics and sociology.
(d) The slow road from acceptability and the truth.
24. With which of the following statements would the author be least likely to
agree?
(a) Truth in the matter of economic and social phenomena is least likely to be
interpreted differently by people.
(b) In social phenomena, what is merely acceptable may be far different from the
truth.
(c) People do not easily accept truths about social phenomena that are likely to
cause uncertainty and change in
their lives.
(d) In general people accept as truth, things that help to add to their self-
esteem.
25. It can be inferred from the passage that
(a) veritable intentions of plain speak is abandoned for a painless approval.
(b) strategic advantage is defined by verity, while tactical advantage by public
approval.
(c) conceptualization merits public approval to forestall failure.
(d) convenient truth marks the essence of popular mandate in society.
Passage(Q.26-Q.30): NASA's Perseverance rover keeps making history. The six-wheeled
robot has converted
some carbon dioxide from the Martian atmosphere into oxygen, the first time this
has happened on another
planet, the space agency said Wednesday. "This is a critical first step at
converting carbon dioxide to oxygen on
Mars," said Jim Reuter, associate administrator for NASA's space technology mission
directorate.
The technology demonstration took place on April 20, and it's hoped future versions
of the experimental
instrument that was used could pave the way for future human exploration. Not only
can the process produce
oxygen for future astronauts to breathe, but it could make hauling vast amounts of
oxygen over from Earth to
use as rocket propellant for the return journey unnecessary.
The Mars Oxygen In-Situ Resource Utilization Experiment -- or MOXIE -- is a golden
box the size of a car
battery, and is located inside the front right side of the rover.
Dubbed a "mechanical tree," it uses electricity and chemistry to split carbon
dioxide molecules, which are made
up of one carbon atom and two oxygen atoms. It also produces carbon monoxide as a
byproduct. In its first run,
MOXIE produced 5 grams of oxygen, equivalent to about 10 minutes of breathable
oxygen for an astronaut
carrying out normal activity.
MOXIE's engineers will now run more tests and try to step up its output. It is
designed to be able to generate up
to 10 grams of oxygen per hour.

. Page 8 of 36
Designed at the Massachusetts Institute of Technology, MOXIE was built with heat-
resistant materials like
nickel alloy and designed to tolerate the searing temperatures of 1,470 degrees
Fahrenheit (800 Celsius) required
for it to run.
A thin gold coating ensures it doesn't radiate its heat and harm the rover.
MIT engineer Michael Hecht said a one ton version of MOXIE could produce
approximately 55,000 pounds (25
tons) of oxygen needed for a rocket to blast off from Mars.
Producing oxygen from Mars' 96 percent carbon dioxide atmosphere might be a more
feasible option than
extracting ice from under its surface then electrolyzing it to make oxygen.
Perseverance landed on the Red Planet on February 18 on a mission to search for
signs for microbial life. Its
mini helicopter ‘Ingenuity’ made history this week by achieving the first powered
flight on another planet. The
rover itself has also directly recorded the sounds of Mars for the first time.
26. As per the passage, which of the following statements is true?
I. Perseverance is a four-wheeled robot.
II. 96 percent of Mar's atmosphere contains carbon dioxide.
III. MOXIE produces carbon monoxide as a by-product.
(a) Both I and II (b) Both II and III
(c) All I, II, and III (d) Only I
27. Which of the following statement is/are false with reference to the passage?
I. MOXIE is located inside the front left side of the rover.
II. The process can make hauling vast amounts of oxygen over from Earth to use as
rocket propellant for the
return journey unnecessary.
III. MOXIE can tolerate the searing temperatures of 1,450 degrees Fahrenheit.
(a) All I, II, and III (b) Both I and II (c) Both II and III (d) Both I and III
28. Which of the following is MOST SIMILAR in meaning to the given word as used in
the passage?
Searing
(a) Blistering (b) Diminishing (c) Knackering (d) Holistic
29. Which of the following statements would the author agree with the most?
(a) Materials like nickel alloy was used in MOXIE to protect it from the bone-
chilling cold.
(b) The conversion of Mars' 96 percent carbon dioxide into oxygen can pave the way
for future human
exploration.
(c) MOXIE can produce 5 grams of oxygen that can provide 20 minutes of breathable
oxygen to an astronaut
carrying out normal activity.
(d) MOXIE is dubbed as a biological tree that can be used to split carbon dioxide
molecules.
30. What do the author mean by 'pave the way for future human exploration'?
(a) The humans can go and settle on Mars if carbon present in the Mar's atmosphere
can be converted into
breathable oxygen.
(b) Tourism can be flourished on Mars by converting its carbon dioxide into oxygen.
(c) Mars can be converted into a scientific centre for performing experiments like
converting carbon dioxide
into oxygen.
(d) The conversion of carbon dioxide into oxygen will make it possible for the
humans to explore the unknown,
and push the boundaries of scientific and technical limits

Directions (Q.66 – Q.105): Read the comprehensions carefully and answer the
questions based on it.
Passage (Q.66-Q.71): The Supreme Court observed that the principle of equal pay for
equal work could not be
applied merely on the basis of designation.
In this case, the court had to examine the claims made by Private Secretaries
(employed in the Eastern Central
Railways for parity in pay with their counterparts working in the Railway Board
Secretariat Stenographers
Service/Central Administrative Tribunal ("CAT").
Interpreting the Sixth Central Pay Commission report, the court noticed that the
aspect of disparity between the
Secretariat and the field offices was a matter taken note of by the Commission
itself. Court also observed that
commission makes the recommendations about the slab of salary not for the salary of
each any every employees
of statutory or government authority. There would have been no requirement to make
these separate
recommendations if everyone was to be treated on parity on every aspect. "We are
fortified in the view we are
seeking to adopt; the principle of equal pay for equal work cannot be applied
merely on the basis of designation
if work of person is different in many aspects like shift, time, amount of work,
field work etc. but if equal amount
of worked is performed then equal pay for equal work shall be allotted to the
employees. While dealing with the
5th Pay Commission recommendations with respect to functional requirements, it was
held that there was no
question of any equivalence on that basis.; it has been held that courts ought not
to interfere if the Commission
itself had considered all aspects regarding salary and after due consideration
opined that absolute equality
ought not to be given for statutory and government entities". Source:
https://www.livelaw.in/top#stories/supreme-court-principle-equal-pay-for-equal-
work-designation-180664
66. Surbhi and Vineet are two employees in Yes Bank employed as the tellers at the
deposit podium working in
same shifts. During the annual appraisals Surbhi getting a 10% raise and Vineet
getting a 5% pay cut. Getting to
know of this, Vineet complained to the remuneration committee, who resorted to the
company policy, which
stated that remunerations of floor employees are based on committee discretion. Can
he have any recourse in the
matter?
(a) No, as the remuneration committee has clear discretion in deciding packages.
(b) Yes, as there is clear discrimination between the two employees.
(c) No, as the committee has been authorized by the law to do it.
(d) No, as the question only mentions statutory and not private entities.
67. In the above case, had if two have been employed in the Supreme Court of India
as stenographers in the
courtroom of a Justice, would they still have had any recourse in law?
(a) No, as the Pay Commission has specified the packages to be so.
(b) Yes, as the Commission has power to decide salary of each employees.
(c) Yes, as both are working on the same post of stenographers.
(d) No, as the employees are two different entities.
68. In a dispute between a railway TC and the Zonal head of operations, the quantum
of the dispute, arising out of
the pending salary payments of the TC left to be sanctioned by the Zonal Head, was
not fixed and formed the
crux of the same. This led to the dispute being pleaded in the Supreme Court. The
arguments were heard and
examined, and it was brought to light that the salary to be given to the TC was not
decided in the pay commission
report. May the SC decide the quantum of the same?
(a) No, as the same can only be decided by the Pay Commission.
(b) No, as the salary of the TC was to be decided before the employment starts.
(c) Yes, as the salary is unclear in the present matter.
(d) Yes, as the salary has not been decided by the Commission here.

. Page 15 of 36
69. In a dispute between two cleaning workers in the Parliament of India, Raman and
Ramya, a male and female,
respectively, wherein Raman was the petitioner who had initiated the suit for pay
disparity as he was earning 30
rupees less than Ramya per hour. However, Ramya pleaded that the Commission had
specified the same in the
reports, and the parity should hold good. Can the SC consider Raman's case?
(a) No, as the position of law is clear in the case.
(b) No, as the Commission has clearly specified the pay disparity.
(c) Yes, as there are no special designations involved.
(d) Yes, as there appears to be a clear case of discrimination in the matter.
70. Arising out of the matter, the petition alleged that the Pay Commission reports
were unconstitutional as they
were in contravention of the anti-discriminatory provisions in force. Can the court
consider the same?
(a) No, as the Pay Commission has authority regarding the same.
(b) Yes, as the Commission has clearly erred in the matter.
(c) No, as the Commission is not a statutory authority.
(d) Yes, as the matter appears to have erred in the matter.
71. The latest Pay Commission specified revised Judges' salaries as having a pay
cut of 5% for all SC Judges. One
of the SC judge challenged the same, and upon admission of the petition, presided
over the matter, along with
four other judges. Is this action tenable?
(a) No, as the Judge is presiding over his own matter.
(b) Yes, as the coram has four other judges, and his vote would not account for a
deciding vote.
(c) No, as the matter can only be considered by the Commission.
(d) Both a & c.
Passage (Q.72-Q.77): In a bid to eradicate the evil of manual scavenging, which has
claimed several lives in the
past, the Madras High Court held that Municipality Heads would be personally liable
in case any person is found
to be indulging in manually cleaning the sewers. So, MCD heads may take cognizance
of the complaints or may
report it to department to eradicate the evil of manual scavenging.
It also directed the heads of corporations and municipalities to file a written
undertaking to the effect that no
manual scavenging work would be permitted to be undertaken within their
jurisdiction.
"All Commissioners of Municipal Corporations across the State and heads of the
Municipalities should file
written undertakings to the effect that no manual scavenging work would be
permitted to be undertaken within
the relevant corporation or municipality areas."
The court added that it may ultimately pass an order directing that such an
undertaking be furnished by any
commissioner connected with a corporation or head of the municipality "when
assuming office in future."
The court further observed that heads of municipal bodies and commissioners of
corporations would be held
personally liable if it was discovered that manual scavenging activities were
taking place within their jurisdiction.
The State was also directed to obtain appropriate machines or improve the sewer
lines in order to ensure that no
manual scavenging is necessary anywhere in the State. It will also be the
responsibility of the heads of
Corporations and of Municipalities to ensure that no private person also indulges
in any manual scavenging
activity or engages any other in such regard, the court ordered further.
Source: https://www.livelaw.in/news-updates/madras-high-court-no-manual-scavenging-
municipalities-file#undertaking-181281
72. It was reported in the local newspaper of Nalasopara that hefty manual
scavenging was being carried out, and
the information about the same was sent to the Municipal Corporation of the area.
Upon receiving information,
the office responded by saying that the current head has not been appointed and
thus, no recourse can be accorded
until the same has been done. Can the Municipality be held responsible here?
(a) Yes, as the department is suomoto liable for the same.
(b) No, as there is no one to pin the responsibility on.
(c) Yes, as the complaint can still be registered.
(d) No, as the presence of the head is mandatory to file a complaint.
. Page 16 of 36
73. When the above-mentioned claims were investigated by the corporation, the
personnel did not find any viable
leads or evidence pointing towards the same was found. The corporation was brought
into question regarding
the alleged activity and now wanted to take action against the informant for being
wrongly appealed to. Can they
do so?
(a) No, as the informant liability is not discussed..
(b) No, as the informant did not possess any mal-intent.
(c) Yes, as the informant wrongly brought the corporation into the question,
(d) Yes, as his actions would make him liable for slander.
74. Upon the investigation being conducted, it was observed that no manual
scavenging per se was being done, but
they did observe that a ball was being retrieved by a child from a nearby sewer. Is
this activity due to being
reported?
(a) No, as this was not liable to fall under the definition of scavenging.
(b) No, as no actual cleaning was being done.
(c) No, as a child cannot be convicted of the same.
(d) No, as the activity is only to be investigated by the head of the Municipality.
75. On his way to work one day, Sujal, the MCD head, saw blatant scavenging being
done in front of him. He was
appalled at the State of affairs in his own Municipality and decided to take action
against the same. Who should
he complain to?
(a) The MCD head can take action on his own right there.
(b) The MCD head should report it to his department.
(c) Either of the above.
(d) The Corporation head does not have suomotu powers.
76. In the Municipality of Nalasopara, a new head was finally appointed, i.e.,
Shama. As her duty to her domain, she
decided to wrap up all the previously reported complaints. Can she open complaints
that she wasn't privy to?
(a) Yes, as it is her job as the head of the Municipality.
(b) Yes, as the Municipality can take care of all the previous complaints made to
it.
(c) No, as the rule of double jeopardy prevents her from doing so.
(d) None of the above.
77. The residents of Shraddha Apartments had been complaining about a blocked sewer
in front of their residential
blocks. However, since the corporation was not taking cognizance of the same, the
residents decided to clean the
sewer on their own. This activity was reported to the Municipality, which decided
to take action against the
residents. Are they liable to be punished?
(a) No, as the residents were picking up the slack of the Municipal Corporation.
(b) Yes, as for whatever reason, manual scavenging was being done.
(c) No, as mere sewer cleaning cannot be equated with manual scavenging.
(d) Yes, as manual scavenging is a crime, even if done with bona fide intent.
Passage (Q.78-Q.83): The Kerala High Court recently pulled up the police force for
using derogatory terms like
using abusive words while addressing citizens and directed the State Police Chief
to instruct all officers to treat
citizens with respect and file a report. It was said that the use of disrespectful
and derogatory words to address
citizens mentally harass them and runs contrary to the Constitutional morality and
conscience of our country.
The High Court felt it necessary to affirm that such actions on the part of the
police are unacceptable in modern
democratic society and this court will act forward to give consequence if such
directions are not followed. In this
regard, the court directed the State Police Chief to remind all officers of their
obligation to treat and address
citizens with respect and submit a report about action taken towards that end.
"I direct the State Police Chief to issue necessary instructions, by way of a
Circular or otherwise, to all members
of the Force under his command that they shall address the citizens using
acceptable vocatives and shall not use
the aforementioned or such other words or phrases. Consequently, the State Police
Chief will act as per the

. Page 17 of 36
aforementioned directions and inform this court about the steps taken in this
regard through a report to be filed
within two weeks from the date of receipt of a copy of this judgment," the court-
ordered.
Source: https://www.barandbench.com/news/abusive-derogatory-language-by-police-
against-citizens-contrary#to-constitutional-morality-kerala-high-court
78. Suman, a paraplegic of 25 years, was addressed by the policemen near the
parking lot as being 'disabled' and
was asked to 'drive safely.' Being a modern paraplegic was aware that that
terminology was outdated, and people
with such afflictions were now supposed to be addressed as being differently-abled.
Being offended by the
misnomer, she decided to file a suit for mental harassment. Would it sustain?
(a) Yes, as she was inappropriately addressed.
(b) No, as the police officer did not intend to do so.
(c) No, as the term mentioned above was not an offensive one.
(d) Yes, as the term was enough to offend the differently-abled.
79. While in hot pursuit, the cops chasing a robber try to provoke him and
subsequently get his attention. In order to
do so, he called him out and shouted loudly, "yo mama so ugly," this gathered the
robber's attention as he
immediately took offense and headed up to the cop to beat him up. He was caught
while running towards the
aggressor policeman and tried to reason with them for the same. While in court, he
also pleaded for mental
harassment and entrapment being employed to catch him. Can his pleas be heard?
(a) No, as he himself was a wrongdoer and cannot claim amnesty.
(b) No, as one should always come to the court with clean hands.
(c) Yes, as the methods employed by the policeman were deplorable.
(d) None of the above.
80. Upon arresting the daughter of an MLA, the arresting officer was being
misbehaved with by the daughter by not
cooperating with the staff and even went on to berate them for misbehaving with a
'lady.' He was addressed as a
dog several times by the same woman. Can he bring an action for mental harassment
by a civilian?
(a) No, as there exists no such offence.
(b) No, as the police officer had enough authority not to have the civilian cause
any trouble.
(c) No, as politicians and their kin have diplomatic immunity in such matters.
(d) No, as no law allow the police to sue the people, hence no offence exists.
81. Avanish was a security guard at Galaxy apartments and usually used to be posted
at the night shift. One night,
while on night duty, he was on duty when a car pulled over in front of the
apartment gate. In there, a shoddily
dressed person was seated. Seeing this, Avanish told him to get lost and shouted,
'Get lost, you beggar.' Turns
out the person sitting inside was Balwan Bhai, one of the homeowners in Galaxy
Apartments. He decided to
bring an action against the guard. In the light of the passage would the action
will sustain?
(a) Only for defamation, as there is no policeman here.
(b) Only for mental harassment as it affected Balwan.
(c) For both of the above.
(d) No such action will sustain.
82. A local policewala in the district of Bhilwara, the birthplace of India's Prime
minister, referred to him as 'chai
wala' while watching the PM address on TV. Enraged by this, the local supporters of
the PM decided to file a
complaint against him. Would the complaint sustain?
(a) No, as the PM himself has not filed the complaint.
(b) No, as the Policewala did not do anything wrong.
(c) Yes, as the statement was published and defamatory in nature.
(d) None of the above.

. Page 18 of 36
83. A senior police officer was seen misbehaving with a group of college students
right outside their college while
making them strip and dance for 'being too forward.' This was recorded on tape by a
journalist and was sent to
the students to take action for the same. What action can be taken?
(a) The state police chief can take the action since he is the prime authority over
policemen under his area.
(b) The local court can take the action as they have the power to summon and
penalise the policemen.
(c) The HC can take the action due to this being a case of breaching constitutional
morality.
(d) Both b and c.
Passage (Q.84-Q.88): August 24, 2021, was the fourth anniversary of P u t t as w
amy v . U ni o n of In di a ,
the landmark digital rights case in which the Indian Supreme Court ruled that the
right to privacy is a fundamental
right under the Indian Constitution. Since then, the government, instead of
overhauling the surveillance law
framework and enacting robust data protection mechanisms, has used public safety
and national security
arguments in court and in parliament to deflect concerns about violations of
privacy rights.
The recent the Information Technology (Intermediary Guidelines and Digital Media
Ethics Code) Rules, 2021.
These rules target internet intermediaries, including social media services,
digital news services, and curated
video streaming sites. While the government says they are aimed at curbing misuse
of social media, including
for the spread of “fake news,” they allow greater governmental control over online
content, and would
seriously undermine rights to privacy and freedom of expression online.
Hacking is illegal under Indian law, and thus as far the Indian government has not
said whether it used Pegasus
to hack into devices. The Indian government’s claims that it has sufficient
safeguards to prevent unauthorized
surveillance have no basis, the groups said. In India, the legal regime for
surveillance is governed by the 1885
Telegraph Act, along with the 2000 Information Technology Act, which if government
want may take back if
wishes to do so.
Under these laws, which have been challenged in Indian courts, the executive branch
has extremely broad powers
of surveillance that are devoid of any meaningful safeguards, with no judicial
authorization or independent
oversight.
Even though the Supreme Court has twice stated, in 1997 and in 2017, that an order
of surveillance can be passed
only when strictly necessary and if there is no other alternative, the lack of
independent scrutiny and effective
reporting mechanisms result in lack of accountability.
84. The Union government of India, has made a new law which mandates Aadhaar
mandatory for all government
related schemes and services, Aadhaar is Unique identity card that contains all
biometric information of the card
holder, X a beneficiary of one of the scheme protested against this new law saying
it violates his Right to privacy
and must be taken back, Is X contention valid?
(a) X contention is not valid because in the current scenario the usage of Aadhaar
for gov. schemes is for welfare
purposes, and welfare of people is more important than privacy.
(b) Right to privacy doesn’t apply to government schemes because they are meant for
people, and with biometric
system it fastracks the process.
(c) The government may not take back this law unless it is challenged and stuck by
some constitutional courts.
(d) X contention is valid as it violates Right to privacy because, biometric
information is very critical personal
information, and hence law must be taken back.

. Page 19 of 36
85. Jammu & Kashmir, state in India where there is high prevalence of militancy and
insurgency etc, the security
forces in Jan 2020 hacked into a mobile device of a leader of the Student Union
group in J &K to check whether
he is linked with any militant group , Can security forces do such hacking without
any permission under Indian
Law?
(a) The hacking of mobile device of the leader is valid because it is prerogative
for them to do such things.
(b) The question of permission doesn’t apply to security forces because, such
rights are vested with them to
protect people.
(c) The question of permission is valid, and the security forces need to have an
independent scrutiny body and
other mechanism before proceeding with hacking.
(d) Though independent scrutiny , Independent oversight or judicial authorizations
desirable, there is right to
privacy but under the current law security forces have power to proceed without any
permission to hack any
devices.
86. In the same state of J&K , the Security forces got an intelligence that one of
the militant group leader planning
on a bomb attack, the security forces hacked the militant head phone and with
intelligence from hacked mobile
they avoided a major attack, Is hacking valid and why?
(a) The hacking of mobile in this case is valid because it’s a matter of national
security.
(b) The hacking of mobile in the present case is valid because, its case of
necessity and with inputs of intelligence
of an attack, there is urgency and case of strict necessity, hence valid.
(c) The hacking in the present case is not valid since, there must is no
independent scrutiny in place, there is
need check for unfettered power.
(d) Both A and B gives a valid reasoning for the hacking.
87. spranks, a spyware by NSO a Israel firm gives spyware only to government bodies
,many groups in India like
newspaper heads, journalists, policy makers, former supreme court judges phones got
attacked with this spranks
spyware by government, all of the victims have one thing in common i.e. they are
critical about current
government. There is huge uproar against this action, if this hacking is done by
government, is it authorized to
do such hacking?
(a) The government is not authorized do such hacking because, there is no strict
necessary for surveillance in
the present case, it is just to crack down dissent.
(b) The government is not liable in hacking in the present case, since the actual
hacking was done by a Israel
firm not the government.
(c) The government is liable for hacking and the present government should be
dragged to the court and be
punished accordingly.
(d) The government is authorized to do such hacking because, people who are
critical to government may
threaten the integration and security of the country.
88. The new Information Technology (Intermediary Guidelines and Digital Media
Ethics Code) Rules, 2021, is
aimed is curbing fake news and expand more control over online content, the uproar
against this is that these
rules can seriously undermine rights to privacy and freedom of expression , In
which of the following scenario
the uproar can be ascertained and justified ?
(a) A twitter account that is involved into making and spreading fake news, and
using deep fakes as means to
spread information. After the new rules, government making this content remove.
(b) A YouTube channel that is critical of government, and make people aware about
wrong doings of
government etc. after the new rules, government making this content remove.
(c) A foreign social media regime spread across various social media, making hatred
content and spreading lies
and disrupting the harmony in India between religious groups,, after the new rules
government making this
content remove.
(d) Taking down a fake news on news website that is published on vaccines and
promoting anti– vax during
covid pandemic. After the new rules, government making this content remove.

. Page 20 of 36
Passage (Q.89-Q.93): Section 11 (Nullity of marriage and divorce- Void marriages)
of the Act had considered
following marriage to be void: -
1. Where at the time of marriage any party has a living husband or wife i.e.,
bigamous marriage is void
2. Where parties to the marriage fall within sapinda relationship i.e., same blood.
A person cannot marry in the
same family i.e., to a person from five generation from the paternal side, three
generations from the maternal
side, The parties are within the prohibited degree of relationship.
According to Sec 3(g) “degrees of prohibited relationship” - two persons are said
to be within the “degrees of
prohibited relationship”- (I) if one is a lineal ascendant of the other; or (ii) if
one was the wife or husband of a
lineal ascendant or descendant of the other; or (iii) if one was the wife of the
brother or of the father’s or mother’s
brother or of the grandfather’s or grandmother’s brother or the other; or (iv) if
the two are brother and sister,
uncle and niece, aunt and nephew, or children of brother and sister or of two
brothers or of two sisters.
A marriage is voidable on either side of the party is known as voidable marriage.
It will be valid unless the
petition for invalidating the marriage is made. This marriage is to be declared
void by a competent court under
the Hindu Marriage Act, 1955. The parties of such marriage have to decide whether
they want to go with such
marriage or make it invalid. The following are the grounds of voidable marriage
under Hindu law: Under-age,
Mental disorder of spouse, Repeated attacks of Insanity, Marriage by force or
fraud, Pregnant with other than
the child of the petitioner. If a person is not heard for the time of 7 years will
be presumed dead.
[Extracted from, https://nyaaya.org/marriage-and-divorce/hindu-marriage/sapindas/]
89. Bigamy is the offence of marrying a second time by one who has a former husband
or wife still living and the
marriage between themselves is still continuing. Section 494 of the Indian Penal
Code lays down provisions on
punishment for the offence of bigamy that is, marrying again during life-time of
husband or wife. A, a Hindu
aged 32 years married Z (Hindu) who resided in England before returning to India.
After 2 years into marriage
A realized that Z was already married to X in England and had not obtained a
divorce. A left Z and married M.
Z filed a case of bigamy against A. Decide.
(a) A is liable for bigamy as he did not divorce Z before marrying M. Thus, he has
entered into bigamous
marriage.
(b) A is liable for bigamy as his marriage with Z was voidable at his option.
Therefore, he had to obtain a decree
of nullity.
(c) A will not be liable for bigamy as his previous marriage with Z was void ab
initio. Z had a living spouse at
the time of her marriage with A, thus making it a void marriage.
(d) Instead of A, Z should be charged with bigamy as she was the one who entered
into a marriage while her
previous marriage subsisted.
90. The terms “custom” and “usage” are defined in Hindu law which mean any rule
that has gained the force of law
among Hindus in any local area, tribe, community, group, or family after being
consistently and universally
maintained for a long period. A was married to B who was his paternal cousin
sister. Both A and B offered pinda
to the same ancestor. Their marriage was held void under prohibited degree of
relationship. The parties to the
marriage contended that there have been a couple of instances of cousins getting
married in their family, hence
it should be treated as a custom. Decide
(a) The marriage will be valid as it is the custom in the family of A and B to
marry cousins.
(b) The marriage will be valid as both A and B are adults and can marry anyone they
want to.
(c) The marriage will be held void as a couple of instances is not sufficient to
prove an existence of custom
allowing marriage of Sapindas.
(d) The marriage will be held voidable at the option of the groom and her family.

. Page 21 of 36
91. Marriage by force or fraud- If one party is forced into a marriage by force or
by fraud then that party can annul
the marriage within 1 year from the discovery of force or fraud. A took a loan of 1
lakh from B. A was unable
to pay back the loan. B coerced A to marry his daughter C to him, failing which he
would increase the interest
by 25%. Marriage took place 4th April 2020. On 15th May 2020, C daughter of A got
to know that her father
was coerced to marry her to B. On 5th April 2021 she filed a petition to annul the
marriage. Decide.
(a) It is a void marriage. The marriage stands annulled as it is not valid in the
eyes of law.
(b) It is a voidable marriage, But C cannot annul the marriage as it is more than 1
year from the date of marriage
(c) It is a voidable marriage at the option of B as he was compelled to marry her
because A failed to pay the
loan.
(d) It is a voidable marriage at the option of C. The marriage can be annulled as
it has not been 1 year since she
realized about the involvement of force and fraud in her marriage.
92. Pick the option with the correct order of validity of marriages.
(a) A married B. At the time of marriage B was pregnant with the baby of her former
paramour C.
(b) A married B in the year 2000. In the year 2002 he married C without divorcing
B.
(c) Z married Y. Y had a disorder which because of which he could not copulate.
This came to light only after
the marriage.
(d) A married B. One day B left overnight and went missing. A waited for 7 years. A
married C after 8 years
after B went missing.
(a) Voidable, Void, Voidable, Voidable (b) Void, Void, Voidable, Voidable
(c) Voidable, Valid, Void, Voidable (d) Voidable, Void, Voidable, Valid
93. Which of the following statements regarding voidable marriage is false?
(a) Voidable marriage is valid unless the petition for invalidating the marriage is
made.
(b) Voidable marriage can be invalidated only at the option of one party.
(c) The party which does not have the power to invalidate the marriage can simply
walk out of it.
(d) Marriage of a 20-year-old male with an 18 -year-old is an example for voidable
marriage.
Passage (Q.94-Q.98): Wrongful restraint means preventing a person from proceeding
in any direction where
such a person has the right to proceed. The obstruction may be caused by causing it
impossible or difficult or
dangerous to proceed. To constitute an offence under this section the person who
was obstructed must have the
right to proceed in a particular direction. The ingredients are
(1) Voluntarily obstruct any person.
(2) To proceed in a particular direction.
(3) The person obstructed must have the right to proceed.
There is no offence under this section where a person obstructs a private pathway
claimed by the way of a right
of assessment over his land and which right was not admitted he does not commit the
offence of wrongful
restraint. Punishment of this section has been given under section 341 of IPC.
Section 340 of IPC deals with the offence of wrongful confinement. Wrongful
confinement can be said to be is
species of ‘wrongful restraint’ as defined in the section. It is a total restrain
of a person, from proceeding beyond
the certain circumscribing limit. There can be no wrongful confinement when there
is no desire to proceed the
certain circumscribe limit, or if it was consented by the person affected. Where
there are no voluntary instructions
that the person as to prevent that person from proceeding in the direction in which
such person has right to
proceed does not constitute an Offence under the section. The punishment of
wrongful confinement has been
given under section 342 of IPC.
Source: https://timesofindia.indiatimes.com/readersblog/lawpedia/wrongful-restrain-
and-wrongful#confinement-35462/

. Page 22 of 36
94. P was stopped by Q to go on a particular path which was one of the two ways for
the river rafting venue. Q stated
that the water level is high and it is dangerous to move in that direction, knowing
the fact that there is nothing
like that. Decide?
(a) Q will be liable as he obstructed the path of P where P had the right to
proceed.
(b) Q will be liable as he obstructed the path of which P had right and that too in
a bad faith.
(c) Q will not be liable as he genuinely showed concern towards P.
(d) Q will not be liable as P was free to take an alternative path.
95. A made a circle around his son B who was standing in the middle of a ground and
told him that if he tries to
move out of the circle then A will shoot him. Decide whether it is a case of
Wrongful Confinement?
(a) A will be liable for Wrongful Confinement as physical boundaries are not
needed.
(b) A will not be liable for Wrongful Confinement as physical confinement is a
necessary component.
(c) A will be liable for Wrongful Confinement as A has a legal authority over B.
(d) A will not be liable for Wrongful Confinement as he did not restrict B
completely.
96. Which of the following is not Wrongful Restraint?
(a) P called Q and told him that he should not be going to shop as the rioting
situation was there in the city.
(b) P has a right to proceed in a path but is threatened by Q that if he proceeds
then P will place a ferocious dog
after him.
(c) P wilfully obstructed Q to use the well with a bad faith that the water is
impure in a particular season which
was not true.
(d) P build a wall on the path which was also used by Q daily in bad faith.
97. A was taken to a police station for some enquiry related to a case and he was
there for 2 days without any
reasonable ground. The police officer stated that A was free to move without
permission inside the station. Will
the Police officer be liable for wrongful confinement?
(a) Police officer will be liable as he was not performing his official duty
correctly.
(b) Police officer will not be liable as A was free to move out of the police
station. C
(c) Police officer will be liable as he violated the law and confined A illegally
within a limited boundary.
(d) Police officer will not be liable as A was not put behind the bars with a
limited space.
98. Xolo is a police officer. One day he arrested Joker a petty thief. Joker asked
his friend Tom to get a bail order
for him. The magistrate granted him bail, however Xolo insisted that Joker is a
habitual offender and should be
put behind the bars. Following this he kept him locked up. Joker has filed a
petition for wrongful confinement.
Decide.
(a) Xolo is not liable as he knew that joker is a habitual offender.
(b) Xolo is protected under sovereign function.
(c) Xolo is liable as he did not follow bail orders and kept joker in lock up.
(d) Xolo is liable as he restrained joker’s movement.
Passage (Q.99-Q.103): The doctrine of proportionality emanates from the two key
elements of a constitutional
structure i.e. rule of law and democracy. Democracy entails rights and liberties,
and on the other hand the rule
of law empowers the state to sanction limitation on them to maintain the rule of
law and order. But, however,
according to a proportionality tool that is derived from this doctrine.
In the simplest of terms, this doctrine means that the ambit of limitation on any
right must be proportional to the
purpose/objective sought to be achieved through the limiting law. It ensures that
the encroachment on any right
is not disproportionate to the objective of the law. This doctrine is meticulously
defined by Arhon Barak, Former
President of the Supreme Court of Israel, in his book titled "Proportionality:
Constitutional Rights and their
Limitation" where he proposes that the central philosophy that governs the
proportionality doctrine is to shield
the individual and his/her rights from the arbitrary brute power of the state, and
conceives of the proportionality
doctrine as having four components namely- proper purpose, rationale connection,
necessity and balancing.
Where the chief focus of proper purpose is to look at the purpose for which the
limitation on rights is brought,

. Page 23 of 36
meaning that in a given constitutional setup the legislative intended purpose is
rational or not. In Modern Dental
College and Research Centre, the Supreme Court designed four- part proportionality
test that are- (i) that the
measure is designated for a proper purpose (ii) that the measures are rationally
connected to the fulfilment of the
purpose (iii) that there are no alternative less invasive measures, and (iv) that
there is a proper relation between
the importance of achieving the aim and the importance of limiting the right.
[Excerpt from, https://www.livelaw.in/columns/universal-declaration-of-human-
rights-constitution-national#security-185312 ]
99. Mr. Edward Cullen was the prime minster of the state of twilight with a very
thin majority who implemented
emergency in the state citing internal disturbance on the suspicion that the
opposition going to demand floor test
and suspended all the fundamental rights, put opposition party leaders in jail and
imposed censorship on the
media. Decide?
(a) The actions of Mr. Edward Cullen are valid as the constitution of twilights
provides powers to its prime
mister to proclaim emergency when he is satisfied.
(b) The actions of Mr. Edward Cullen are not valid as the measure taken by him is
arbitrary in nature.
(c) The actions of Mr. Edward Cullen are valid as limiting the rights in the
security interest of nation are valid.
(d) The actions of Mr. Edward Cullen are not valid as nothing could take away the
fundamental rights of an
individual.
100. The government of India was given the input by the security services that
Khalistan supporters are conspiring to
use farmers’ protest to unleash mayhem in India. It was also brought to the notice
of the government that Internet
facilities are being used by these Khalistan supporters to spread rumours, defame
the government and to incite
violence. Based on these inputs, the government of India decided to prohibit use of
internet in the State
temporarily. Decide?
(a) The actions of government are not valid as right to internet is a fundamental
right inherent under right to life
and personnel liberty thus could not be taken away.
(b) The actions of government are valid as the measures taken are proportional to
the purpose.
(c) The actions of government are not valid because the measures taken are not
proportional to the purpose.
(d) The actions of government are valid as government is empowered to limit the
fundamental rights of an
individual.
101. Mr. Gulam was an employee of the corporation for 20 years and in his career
there was not even an allegation
of misconduct against him however a disciplinary enquiry was initiated against him
for misplacing a file
entrusted to him, which amounted to misconduct under by-law. The committee after
completing its enquiry
suspended him. Decide?
(a) The actions of the committee are valid as Mr.Gulam was proven guilty of for
misplacing a file entrusted to
him.
(b) The actions of the committee are not valid at all.
(c) The action of the committee are valid as suspension from service is adequate
disciplinary action for
misplacing a file entrusted to him.
(d) The action of the committee are not valid as suspension from service is an
extreme measure for misplacing
a file entrusted to him.
102. Due to the uncertainties and high risks in crypto currency exchange, the
central bank of the state of Rabri issued
a circular prohibiting the regulated Banking and Non Banking Financial Institutes
from providing any banking
services to the business involved in crypto currency exchange. Decide?
(a) The circular of central bank of state of Rubri is valid as crypto currency
exchange involves uncertainty and
high risk
(b) The circular of central bank of state of Rubri is not valid at all.
(c) The circular of central bank of state of Rubri is valid as prohibiting Banking
Institutes from providing any
banking services to the business involved in crypto currency exchange is an
adequate measure.
(d) The circular of central bank of state of Rubri is not valid as prohibiting
Banking Institutes from providing
any banking services to the business involved in crypto currency exchange is
extreme measure.

. Page 24 of 36
103. Assertion (A): protection of rights of each and every individual, however
subjected to the limitations imposed
by the laws, forms basic tenets of a democratic nation.
Reason (R): a healthy balance between the individual rights and the rights of
citizens as a whole is must in a
democratic country.
(a) Both (A) and (R) are correct (b) (A) is correct but (R) is incorrect
(c) Both (A) and (R) are incorrect (d) (A) is incorrect but (R) is correct
Passage (Q.104 and Q.105): Article 14 of the Constitution provides that every
person is equal before the law
and enjoys an equal protection of law. Any discrimination on the grounds of
sex/gender is prohibited under
Article 15. This, however, does not prevent the State from making special
provisions for the benefit of women
under Article 15(3). The purpose is to remedy the historic disadvantage faced by
women and to empower them.
Any legislation drafted hereunder cannot be subject to a constitutional challenge
on the ground of unequal
treatment alone. The Maternity Benefit Act and other childcare leave are enacted
under this provision, coupled
with the State’s duty under the Directive Principles to ensure maternal health.
Drafted with this good intention
of empowering mothers, gender-specific parenting rights may, however, do more harm
than good.
Protective legislation, however well-intentioned, must be understood in the social
context under which benefit
is sought to be provided. Notable academician Professor Sandra Fredman in her book
on Discrimination Law
(2011) has cautioned against protective legislation which might appear to remedy
disadvantage, but in fact
reinforces sex stereotypes for women. In a similar tone, the Supreme Court has also
warned against the dangers
of protective legislation and held that it must not create classifications that
perpetuate legal, social or economic
inferiority of women.
Reason being that such classifications may perpetuate sex stereotypes and therefore
contribute to discrimination
rather than overcome it. To achieve this, the Court has also consistently rejected
the State’s attitude towards
pregnancy, motherhood and domestic obligations towards children and family as roles
only being attributed to
women. These stereotypes, premised on assumptions about socially ascribed roles of
gender which discriminate
against women, are against the spirit of Article 14 of the Constitution.
[Extracted from, https://www.barandbench.com/columns/gendered-childcare-in-india-
time-for-he-for-she]
104. In the democratic country of Nadia, due to the age long patriarchal setup,
women’s participation in political,
legislative and judicial bodies are minimal even after the fact that women
constitute nearly 50% of its population,
therefore the government of Nadia has decided to give reservation to the women in
legislative and judiciary
bodies. Decide?
(a) Reservation for women in both legislative and judiciary bodies are against the
spirits of article 14 and thus
constitutionally invalid.
(b) Reservation for women in both legislative and judiciary bodies are protected
under article 15 (3) and
therefore constitutionally valid.
(c) Reservation for women in legislation is valid as women constitutes 50% of
Nadia’s population thus their
representation is must however reservation of women in judiciary is invalid as
reservation could harm the
integrity of such institution.
(d) None of the above.
105. In a democratic country of Sugar Rush, there was a law related to adultery
which prohibits and criminalizes, any
consensual sex between a married women and man outside her marriage without the
consent of that women’s
husband. These laws portraits women as innocent and the man with whom she had sex
as a perpetrator and
sentences him to imprisonment. Decide the author’s insight toward the adultery laws
of sugar rush.
(a) Authors appreciate such laws as these laws safeguards women and are enacted for
the welfare of the women.
(b) Author vehemently condemns such laws for being violative of sprit of article 14
of the constitution as the
law only sentences the man not the married women where both are at fault.
(c) Author appreciates such laws as they are protected under article 15(3).
(d) Author vehemently condemns such laws as it encourages stereotypical gender-
based assumptions.

. Page 25 of 36
SECTION - D: LOGICAL REASONING
Direction (Q.106-Q.110): Read the passage given below and answer the question that
follows:
The latest World Inequality Report makes for a sobering read. It details the rising
levels of income and wealth
inequality across countries. As far as income is concerned, the richest 10 percent
of the global population
currently takes 52 per cent of global income, whereas the poorest half of the
population earns 8.5 percent of it.
The picture is worse when it comes to wealth inequalities.
India is one of the worst performers. “India stands out as a poor and very unequal
country, with an affluent elite,”
states the report. While the top 10 per cent and top 1 per cent hold respectively
57 per cent and 22 per cent of
total national income, the bottom 50 per cent share has gone down to 13 per cent.
It is not just the inequality in
income and wealth that plagues India. The report also points to extreme gender and
carbon inequality. For
instance, at 18 per cent the female labour income share in India is one of the
lowest in the world. Inequality is
so high in the country that when India is removed from calculations, the global
bottom 50 percent income share
rises.
Alarming as these findings are, they are not entirely surprising. That’s because in
India’s case the primary
instrument — that is, fast economic growth — to reduce poverty and counter
inequality has been faltering for a
while. GDP growth has been rather iffy since the Global Financial Crisis of 2008
and has completely lost its
momentum since the start of 2017. For a relatively poor country such as India, the
most durable and dependable
way to reduce inequality is to increase the size of the GDP. That is the first
policy lesson for the government.
However, as evidence from across the world has shown, fast GDP growth alone doesn’t
help, especially when it
comes to tackling inequalities in accessing education and health. That is the
second key policy lesson. A good
starting point in this regard would be for the government to improve the quality of
data on inequality within the
country.
106. Which of the following reflects the main idea of the passage?
(a) It is rather perplexing that India is one of the worst performers according to
the World Inequality Report.
(b) There are a few key lessons that India needs to learn from the latest World
Inequality Report.
(c) India could have easily shown well in the latest World Inequality Report, had
it learnt the lessons from the
previous report.
(d) The primary objective of the World Inequality Report is to draw lines between
income and wealth of an
individual.
107. What is the role of the first paragraph in the given passage?
(a) The first paragraph contains the main idea of the given passage.
(b) The first paragraph gives reasons on why India is the worst performing country.
(c) The author briefs the readers about the passage in the first paragraph.
(d) The first paragraph plays an insignificant part in the passage.
108. What is the reaction of the author on India’s dismal performance in the World
Inequality Report?
(a) The author was perplexed at India’s performance and blames it on the
administration.
(b) The author expected India to perform even worse in the given report.
(c) The author was not completely surprised and presented a few lessons regarding
it.
(d) The author is not surprised at all on India’s dismal performance.
109. Which of the following is not one of the findings from the World Inequality
Report?
(a) There is no difference between the income levels and the wealth levels.
(b) A minority of the population holds the majority of wealth.
(c) In India, the top ten percent holds close to sixty percent of the national
income.
(d) India presents a dismal picture when it comes to wealth inequalities.

. Page 26 of 36
110. According to the author, which of the following steps may help India improve
its performance in the next
inequality report?
I. By only improving GDP growth.
II. Increasing the size of GDP.
III. Improving the quantity of data on the country’s inequality.
(a) Only II (b) Only I and III (c) Only II and III (d) All I, II and III
Direction (Q.111-Q.115): Read the passage given below and answer the question that
follows:
Happy families are all alike; but every unhappy family is unhappy in its own way.
This is what Leo Tolstoy
thought in his time. But the twenty-first century seems to have turned Tolstoy’s
argument on its head. Unhappy
families now seem to have one — singular — thread of discontent: difference of
opinion. Hard data may be
difficult to come by just yet, but psychologists seem to be echoing news reports
that suggest that more and more
young people are choosing to cut ties with parents over clashes in cultural values.
Formally known as
‘estrangement’, most of these ‘break-ups’ between parents and children tend to be
initiated by the latter. Families
have always harboured differences of opinion: loud arguments are the hallmark of,
say, the argumentative Indian
clan. The disapproval of individualism and juvenile autonomy — purportedly another
leading cause of
estrangement — within familial settings is not new either.
What has changed, according to experts, is a growing awareness of mental health,
leading, in turn, to deeper
appreciation of how toxic or abusive family relationships can affect psychological
well-being. There has also
been a corresponding rise in self-dependence; fewer children are now reliant on
their parents for livelihood or
property. However, the principal cause of estrangement — rifts based on political
opinion and values — is a
growing area of concern. In the course of its evolution, the family has found
ingenious ways of fusing the political
with the personal. What, then, has changed so fundamentally? The answer seems to be
the prevailing —
deepening — crisis in consensus. Individuality, it would seem, has birthed an
inflexibility of opinion, so much
so that a difference in views is now tantamount to a moral transgression. The
fractured political constituency is
encouraging these chasms within other institutions. Social media — that other fount
of shrill opinion and false
narratives — is equally dismissive of divergent points of view.
What an embittered, opinionated but divided family — and the world? — needs are
greater investments in
conversation, empathy and consensus. Isolation — a social pathology that is
spreading like a pathogen — breeds
ignorance. The challenge across social settings is to debate differences without
demonizing them.
111. Which among the following best represents the purpose of the author?
(a) The author apparently does not have any particular purpose for writing this
passage as many conflicting
opinions have been expressed.
(b) The author tries to find out why the entire world is moving on one single path
despite the existence of multiple
paths.
(c) The author is exploring the reasons for the new relationship that has surfaced
between young adults and their
families.
(d) The author focuses on exploring the reasons for increasing fractured families
due to difference in political
ideologies, among other factors.
112. Out of the following, which one is most likely to be an opinion held by the
author?
(a) There may be some recourse to a divided family - they can still be united.
(b) An Indian family is not likely to be divided at all - they have always
harboured differing opinions.
(c) Juvenile autonomy is one such cause of more families getting divided which was
not present earlier.
(d) Unhappy families now seem to have one — singular — thread of discontent:
difference of cultural set up.

. Page 27 of 36
113. Out of the following, which can weaken the author’s arguments?
(a) Individuality has begotten rigidity of opinions; and the differences in opinion
are seen as moral offence.
(b) It is extremely difficult to find out a precise reason for families getting
divided.
(c) One of the reasons for the estranged family is the awareness of mental health
and children want to dissociate
from parents who are toxic to their mental health.
(d) One can be embittered over a difference of opinion, but division of family is
not the solution.
114. Which of the following can be inferred from the given passage?
I. The author of the passage is in agreement with the arguments presented by Leo
Tolstoy.
II. If members in the families do not perceive differences of opinion as moral
transgression, it is likely that there
would be fewer families getting divided.
III. Reducing the political differences within the families would stop families
from estrangement.
(a) Only I (b) Only II (c) Both II and III (d) All I, II and III
115. According to the second paragraph of the passage, which of the following has
not led to estranged families?
(a) Differences in opinion among the families leading to rifts between them.
(b) Growing awareness of mental health leading to realization of how good families
can be to one’s well-being.
(c) More young people getting self-dependent, relying less now on their parents for
livelihood or property.
(d) Social media — that other fount of shrill opinion and false narratives — is
equally dismissive of divergent
points of view.
Direction (Q.116-Q.120): Read the passage given below and answer the question that
follows:
Alongside overseeing land administration, revenue collection, health, education,
disaster management and other
goings-on in the district, Madhya Pradesh collectors will, henceforth, judge
‘entertainment’ as well. Recently,
the MP home minister announced that the state government has finalised guidelines
whereby anybody wanting
to shoot a film, OTT drama or even an advertisement will have to submit a complete
script to the collector’s
office, and the officer will decide if there is anything ‘objectionable’ in it
before saying yes or no. Presumably,
MP, a relatively poor state that can do with all the business it gets, doesn’t mind
losing the film shoot business.
It’s quite likely that filmmakers, blessed with an abundance of choices in India
and abroad, will sidestep a state
where local administrators will be culture arbiters.
The larger point is of course that the MP's decision is terrible news in a
democracy, and one that prides itself in
the soft power of its film and entertainment industry. India is experiencing some
kind of a filmmaking paradigm
shift as OTT productions, as well as some movie hall releases, made here get good
response from critics and
audiences abroad, and movies made in dozens of countries around the world are seen
and liked by Indians. India
is a major market and content producer for OTT because it has the audience and the
talent – but this rich
ecosystem is predicated on creative freedom.
MP has pioneered a rule that, if replicated, can easily deal a body blow to that
freedom. Bad ideas have a way of
replicating across states – witness how many states have adopted the self-defeating
rule on private industry
reserving jobs for locals. If other state governments want to prove they are as
“culturally conservative” as MP,
a flourishing industry with a global footprint will suffer. So will India’s
reputation. And that has consequences
outside the film industry.
116. Which of the following best sums up the main idea of the passage?
(a) A blow to India’s reputation would have consequences outside the film industry.
(b) The recent announcement in Madhya Pradesh of local administration as culture
arbiters and scrutinizers is
against the creative freedom and business of the film industry.
(c) The recent announcement in Madhya Pradesh of local administration as culture
arbiters and scrutinizers is a
harbinger of doom to India’s democracy.
(d) The film industry needs to regulate itself, else the government would step in.

. Page 28 of 36
117. Out of the following, which can be inferred from the passage?
(a) The broader consequences of the development in MP would be elsewhere, other
than the blow to the film
business.
(b) The Indian film industry is already on a downward curve, with the new
development accelerating the decline.
(c) There is an increasing behavioural change of Indian filmmakers making films
outside the country.
(d) The officer would likely give the nod to the script, which he finds
‘objectionable’.
118. For which of the following claims has the author not presented any evidence or
information to strengthen?
I. Madhya Pradesh’s decision is bad news for democracy.
II. Indian films are doing average in other countries.
III. Madhya Pradesh collectors have other departments to take care of than the
entertainment industry.
(a) Only I (b) Only II (c) Only III (d) I, II & III.
119. The author argues that creativity may suffer due to MP’s new rule. What among
the following, if true, can counter
this argument?
(a) The filmmakers would leave the film industry and look for better opportunities.
(b) The new rule in MP is limited to filmmakers taking permission for shooting in
sensitive areas.
(c) The filmmakers would make the content such that their scripts get a nod from
the relevant officer.
(d) The filmmakers will file a petition in the Supreme Court against the decision.
120. Which of the following can be deduced from the passage?
(a) Audience and Talent alone cannot keep India’s film industry sailing.
(b) The filmmakers would maneuver their way around the state of Madhya Pradesh.
(c) The other states especially will not follow the unreasonable rule announced in
Madhya Pradesh.
(d) The officer would not give a nod to any script that he peruses.
Direction (Q.121-Q.125): Read the passage given below and answer the question that
follows:
After years of stability, the Office for National Statistics reported that alcohol-
specific deaths in 2020 had
increased by 19% from 2019, counting 8,974 deaths coded as caused by alcohol
misuse, with three in four being
from alcoholic liver disease. The national lockdown saw an increase in abstention,
but also in heavy drinking.
Harm can increase without greater total consumption: the distribution matters, not
just the average.
Drinking too much alcohol raises the risks of many diseases, so analysts try to
estimate total numbers of deaths
attributable to alcohol consumption – they modelled about 19,200 alcohol-related
deaths in England in 2019,
around four times the direct count of number of alcohol-specific registrations. The
method requires many
assumptions, such as 11% of breast cancer deaths in women over 75 being due to
alcohol, and a major recent
change in such attributable fractions cut estimated alcohol-related death figures
by around 23%.
People might like a single number to settle an issue but we cannot count everything
directly and analysts may
need to construct a range of statistics to improve our understanding. Many tens of
thousands of deaths every year
are attributed to air pollution, but until an inquest in 2020, it was never given
as an official cause of an individual
death.
Influenza also demonstrates the limitations of counting. Between 2013 and 2020,
only around 600 people in
England and Wales died with influenza as the direct underlying cause each year. Yet
England’s public health
agency estimated in 2020 that there were around 15,000 “influenza-attributable
deaths” in the 2016-17 season
alone. That figure comes from the FluMomo model, which picks out periods with high
mortality over a curved
seasonal baseline, attributing those deaths to influenza or extreme temperatures.
This approach has some
weaknesses, including potential overestimation and insufficient corrections for
registration delays.

. Page 29 of 36
121. Which of the following is the main message of the author in the passage?
(a) Deaths attributable to alcohol, air pollution and flu must be modelled rather
than counted.
(b) Explaining a health issue is close to impossible, with so many factors that
need to be taken into account.
(c) Analysis of various factors is really crucial, before making any deduction
about a health issue.
(d) A health issue that impacts the entire world is much more difficult to explain
than the one whose impact is
endemic.
122. Which of the following can be definitely deduced from the passage?
(a) 11% of women over the age 75 who died due to breast cancer died due to alcohol.
(b) In the year 2016-2017, England saw over 15000 people die due to influenza.
(c) There were a total of 8794 alcohol related deaths globally.
(d) None of the above
123. What can be validly inferred from the given passage?
(a) It is possible to quantify the number of alcohol induced deaths without making
some assumptions.
(b) There is a possibility that the number of reported alcohol-related deaths is
not accurate.
(c) The FluMomo model is the best available model to estimate the number of deaths
related to something.
(d) Deaths attributable to alcohol consumption are isolated from the deaths
attributable to a specific disease.
124. Which of the following is an assumption made by the author?
A. 11% of breast cancer deaths in women over 75 was due to alcohol.
B. A major recent change in some attributable fractions cut estimated alcohol-
related death figures by around
23%.
(a) Only I. (b) Only II. (c) Both I and II. (d) Neither I nor II.
125. What role does the following statement (based on the last information given in
the second paragraph) play
towards the arguments of the author, “An inquest aids in attributing some health
issue to an official cause of an
individual death.”?
(a) This statement is an assumption made by the author.
(b) This statement can be labelled as one of the supporting ideas of the passage.
(c) This statement is a claim made in the passage without any supporting evidence.
(d) This statement plays no part in the passage.
Direction (Q.126-Q.130): Read the passage given below and answer the question that
follows:
How should schools reopen after the pandemic closure? In the midst of discussions
about learning deficits,
remediation, accelerated learning, and so forth, what is clear is that it should
not be business as usual. Education
is not a race. It is a child’s journey to fulfilling his potential. Reopening
schools should be an opportunity to
rethink the teaching and learning process itself.
Tagore became one of India’s foremost philosophers of education. As a child, he had
dropped out of formal
school and was taught at home. For him, education meant much more than rote
learning: “The highest education
is that which does not merely give us information but makes our life in harmony
with all existence.” He dismissed
any pedagogy that sought to cut children off from the world around them: “We are
made to lose our world to
find a bagful of information instead. One of the students at this school in rural
Bengal was Amartya Sen. In his
memoir Home in the World (named after Tagore’s classic novel of the freedom
movement, Ghare Baire or
‘Home and the World’), Sen writes about his years at Santiniketan. In the Dhaka
school where he began his
education, he had not at all been motivated to study. Contrary to the principal’s
expectations that all his students
should “shine”, he had ranked 33rd in a class of 37.

. Page 30 of 36
In 1941, due to fears of the possible Japanese bombing of the cities, Amartya was
sent to Santiniketan. Here, he
discovered the freedom of learning at his own pace. There was no pressure to excel
in terms of grades or exam
performance. This was immediately liberating: “I became what would count as a good
student only when no one
cared whether I was a good student or not.” Indeed, there seemed to be an inverse
relationship between grades
and originality.
Santiniketan’s approach was to help the whole child to learn through exploration -
art, music, curiosity, and the
careful observation of nature. The environment was stimulating. There was no
corporal punishment. “School
was fun in a way I had never imagined school could be,” writes Sen. The atmosphere
of freedom and thoughtful
reasoning shaped his educational attitudes, including his response to inequities
around him: For example, by
running night classes for tribal children from neighbouring villages.
There are many lessons here for the Indian education system today. Joyful and
creative learning should not be
an indulgence meant only for privileged children. Every child should be able to
learn in an atmosphere that is
free, reflective and affirming. They should be able to relate new concepts to what
they are already familiar with
within their own lives.
Karnataka’s Vidyagama programme, which began with a group of committed teachers
creating informal, outdoor
learning circles or “vataara shaale” during the pandemic-where children gathered
with a teacher for in-person
teaching in small groups, in outdoor community spaces; not bound by blackboards and
textbooks, but learning
interactively, through stories and activities - has been acknowledged as an example
of an alternate model of
teaching and learning.
The pandemic itself can be a starting point for inquiry-based learning: Children
can be encouraged, within the
safe space of the learning circles, to discuss what they saw, experienced and
learned during the pandemic.
Rather than being cooped up inside small, cramped schoolrooms through the day,
extending lessons into outdoor
spaces, where feasible, will also improve ventilation, which is perhaps next in
importance only to vaccination
during the continuing pandemic.
Symbolically, too, an open classroom presents itself to fresh winds and ideas. We
should really open up our
education system.
126. What is the main idea of the passage the author is stressing over?
(a) To start giving education in open premises rather than being cooped up in small
classrooms to improve
ventilation and present itself to fresh winds and ideas.
(b) To take pandemic as an opportunity to re-think the education system because
education meant much more
than rote learning and is not bound by blackboards and textbooks.
(c) To fill the gaps present in the Indian Education system and to improvise it as
per the industry set-up.
(d) To understand the need for improving the education system.
127. What does the author mean by the following statement?
“We are made to lose our world to find a bagful of information instead”
(a) The children need to learn wholistically from the surroundings rather than
restricted disseminating of
information.
(b) Our education system separates children from inside the schools to provide very
little information.
(c) The world made us lose everything to provide us very less information
(d) Only information is the way to garner knowledge about the world.

. Page 31 of 36
128. Why is the author stressing over the topic “post-pandemic opportunity to re-
think education”?
(a) Now everything is going online and so does the education system.
(b) Post- pandemic, the education and examination system are completely disturbed
now.
(c) Reopening schools should be an opportunity to rethink the teaching and learning
process itself.
(d) Post pandemic, the schools should completely overhaul the education system as
it is redundant.
129. What is/are the lesson/ (s) the author talked about in the passage for the
betterment of the Indian education
system?
(a) Learning interactively, through stories and activities
(b) Every child should be able to learn in an atmosphere that is free, reflective
and affirming.
(c) Both (a) and (b)
(d) Children figure out the smart approach to learning despite the restrictive
information.
130. What does the author mean by inquiry-based learning?
(a) One must inquire and understand what the children want to learn and let them
learn the same.
(b) Children should be encouraged and provided with learning circles, to discuss
what they saw, experienced
and learned.
(c) Time to time inquiring about the learning pattern and understanding of the
children through the same and
make improvements on a daily basis.
(d) To send the teacher into the world to inquire about the new reforms in the
education systems and bring in
the best learning process to adopt to our education system.
Direction (Q.131-Q.135): Read the passage given below and answer the question that
follows:
The government’s new definition of a surrogate mother is a woman genetically
related to the intending couple,
married with a child of her own, aged between 25 years and 35 years allowed to be a
surrogate only once in her
lifetime. The intending couple meanwhile requires an official certificate to opt
for surrogacy. On August 5, the
Lok Sabha passed, with almost no debate, The Surrogacy (Regulation) Bill, 2019. The
surrogacy bill prevents
single parents, same-sex couples, divorced or widowed persons, transgender persons,
live-in partners, and
foreign nationals from using a surrogate mother. The bill now awaits assent by the
Rajya Sabha, the Parliament’s
upper house.
I. Proven infertility through diagnosed medical conditions should indeed be the
only criteria to permit
surrogacy, said Manasi Mishra from the Centre for Social Research.
II. The bill says that the Centre and the State governments will appoint an
appropriate authority to issue a
“certificate of essentiality” and “eligibility certificate” to the intending
couple.
III. If the bill becomes law, same-sex couples, single-parents, and live-in couples
will not be allowed to opt for
surrogacy, as they currently can.
IV. "By denying surrogacy to these groups – single parents, homosexuals,
transgenders – you are denying
them their rights, making it a regressive or limited view of what a family is”,
said Gargi Mishra, a gender
rights lawyer at Sama.
V. Experts and activists said banning commercial surrogacy will also take away the
livelihoods of women
who rented out their wombs and would deny women rights over their own bodies.
VI. Some said the ban should be followed by modification of adoption processes, so
single people and
homosexual couples can have children.
131. Which of the given statement states a course(s) of action to be taken by the
government to enforce the bill?
(a) Only V (b) Only II (c) Both I & IV (d) Only I, III & IV
132. Which of the given statement(s) criticises the actions taken by the
government?
(a) Only VI (b) Only II (c) Both I & VI (d) Only IV & V

133. Which of the following statement(s) support (s) modifications to the Surrogacy
Bill?
(a) Only II (b) Only V (c) Both I & VI (d) Only II, III & V
134. Which of the following supports the Surrogacy Bill mentioned in the stanza?
(a) Only VI (b) Only I (c) Both II & IV (d) Only II, III & V
135. Which of the given statement(s) describes direct impact(s) of the Surrogacy
Bill?
(a) Only VI (b) Only II (c) Both I & VI (d) Both III & V

mock 24
Directions (Q.1-Q.30): Read the following passage carefully and answer the
questions that follow.
Passage(Q.1-Q.5): As many of us would have already realized, life is indeed a drama
with each of us doing
varied roles at various stages of our lives, that resonate with our personality and
temperament. Every role has its
own identity and purpose. But unlike in a staged drama, in life people are free to
choose the roles they want to
play. For example, you might have to get married in a drama, whereas in life you
have a choice: you can either
get hitched or remain single.
Hence, when people have the power to decide what role they want to play it is
imperative that they make wise
choices that would result in positive consequences for themselves as well as to
their fellow humans.
While role play on a stage can, in most cases, be a joyful experience, the real
roles we play bring with them
several challenges and problems that need to be overcome if we want to play our
parts to perfection.
It takes a lot of effort on the part or every person to strive and achieve
excellence in their various roles. Today
everyone has several opportunities to try out diverse roles, and it is up to the
individual to choose roles of their
liking, or roles that they believe would help them fulfill their needs,
aspirations.
It is up to the discretion of the individual to take up a single role or multiple
roles on the basis of their ability to
execute it/them. A homemaker is no less worthy than a working woman; it is simply a
choice made by the
individual.
It is necessary to ensure that we are fulfilling each role to the best of our
abilities. And if we find a lag then we
must make sure that we make up for the loss and rework our strategies. And unlike
in a stage play, our real roles
come with a lot of emotional bonding, which requires us to be more sensitive in
handling the roles that we
execute.
It also calls for more commitment and acumen as finally it is these roles that
determine our inner happiness and
well-being. In this drama of life it is important that we respect our coactors and
work together in peace and
harmony. We must never gauge another individual’s role worth based on social,
economic, political, racial or
cultural factors as each role, however trivial it might appear to be, has its
importance in its own space. We must
always strive to prove our role’s worth and give back to society.
It is more likely that people would remember an affluent individual for his
contribution to society rather than
any empire he would have built for himself. So if we want to leave a mark even
after we have left the stage it
can only be through the good work that we would leave behind by comprehending our
own roles better.
So if we contribute to society even in a small way, it would make a world of
difference to future generations.
1. What is the stark difference between drama and life?
(a) In drama, characters have predetermined roles but life roles are extempore.
(b) In drama you do not have a choice but in life you do.
(c) both (a) and (b)
(d) none of the above.
2. Which of the following is untrue?
(a) People should choose their roles wisely for positive results.
(b) Life is synonymous with conflicts and issues and needs immense courage to sail
through.
(c) Life today has become easy-going and it is not difficult to achieve eminence in
various fields.
(d) Life abounds with options which call for right decision making.
3. If we consider that multitasking has become the order of the day. How do you
justify it?
(a) Today, everyone is taking up diverse roles to fulfill their aspirations.
(b) People who have the ability to manage several things take up multiple roles.
(c) Real roles have emotional overtones and need handling in a sensitive manner.
(d) Women multitask as a home maker and office managers.

. Page 3 of 36
4. Happiness is derived from
(a) multitasking. (b) properly judged and dedicated roles.
(c) emotional bonding. (d) reworking our strategies.
5. What would be the best way to leave our footprints on the sands of time?
(a) By building a memorial.
(b) By doing small good things for the society.
(c) By understanding ourselves and contributing to society.
(d) By respecting our co-stars and working in harmony.
Passage(Q.6-Q.10): Some people, including some young, strong and bright women, feel
feminism is no more
necessary. It is a thing of the past because girls and women have equal rights now.
I wish it was so.
On the one hand, women have broken many barriers; they have proven themselves in
every area of work and
life, we have pretty good laws and policies, but at the same time violence against
women is not less, if not more,
and this is so all over the world: in India the female male ratio continues to dip.
For the first time in human history, there are fewer women than men on this planet
thanks mainly to gynocide
(killing of women) in India and China. The employment rate of women in India in the
formal sector has been
going down while the GDP has been rising
Our religions, customs and traditions, our languages not just in India but
everywhere, continue to be patriarchal.
The media and advertisements are filled with misogynistic (woman hating) images,
stories and item numbers.
Therefore, for me feminism is still relevant and required. My first point is that
feminism is perhaps the most
badnaam (infamous), purposely discredited, misinterpreted and misunderstood -ism.
There are all kinds of totally
ridiculous rumors about feminism. For decades corporate media has spread falsehoods
about feminism. I find it
totally amazing that most middle class men have heard about bra burning feminists,
while they seem to have
missed hearing about the relentless struggles of feminists in South Asia against
dowry deaths, female feticide or
rape.
Despite this, I am happy to call myself a feminist. I was delighted when HH the
Dalai Lama said recently that
“if speaking for the rights of women is being a feminist, then I think I am a
feminist”.
The word feminism is derived from the French word Femme, which means woman.
Feminism looks at the world
through women’s eyes. Why do we this? We look at the world through women’s eyes
because for the last couple
of thousand years the world has been looked at and spoken about through men’s eyes.
Almost all religious texts have been written by men; mainly men have formulated
laws in most Parliaments of
the world; mainly men have made laws and passed judgments; men editors have
interpreted and controlled news
and so on. Therefore, a balanced view is desperately required and for that we need
to look at the world through
women’s eyes.
Another reason for looking at the world through women’s eyes is the fact that women
are at the bottom of all
social, political and economic hierarchies. Hence when we look at the world through
women’s eyes, we look at
it through the eyes of the most oppressed and exploited members of our societies.
South Asian feminists have
defined feminists thus. Anyone who recognizes that women are discriminated against
within families, at the
place of work and in society in general, and who takes action against this
discrimination is a feminist”. According
to this definition, men can also be feminists.
6. Which part of speech is feminism?
(a) noun (b) particle (c) verb (d) infinitive
7. Which of the following is untrue?
(a) The empowerment of women is inversely proportional to the violence rate against
them.
(b) Gender bias is still very much existent
(c) Female infanticide and female feticide have taken their toll in India.
(d) Women employment rate in the formal sector in India is inversely proportional
to the GDP growth.

. Page 4 of 36
8. The idiom ‘it is a man’s world’ is reflected in
(a) there are fewer women than men on this planet thanks to gynocide.
(b) our beliefs in religion, customs, traditions and even languages are
patriarchal.
(c) media and advertisements are filled with misogynistic information.
(d) the GDP has been growing faster than the women employment rate.
9. Which of the following is UNTRUE?
(a) Ridiculous rumors abound regarding feminism.
(b) Corporate media always spreads fake messages about feminism.
(c) Middle class men are also aware of dowry deaths, female infanticide and female
feticides.
(d) A feminist need not belong to the female gender
10. It is important for feminism to view the world through women’s eyes because,
(a) the word feminism is derived from the French word ‘femme’.
(b) being the affected strata of society, women will be able to give an unbiased
and unprejudiced view.
(c) we have had enough of texts and books from men’s viewpoint.
(d) feminism begins at home.
Passage(Q.11-Q.15): When it comes to family, where does love stop and duty begin?
Sometimes that’s easily
answered: evolutionary instinct moulds a parent’s love for their children into
something fierce and
uncomplicated. Broaden out the focus to siblings, adult children, ageing parents,
aunts and uncles, and the answer
is less straightforward.
Britain’s more individualistic approach to family is often contrasted with family
cultures in southern Europe.
There, young people tend to leave the parental home later, and it is much more
common to find three or even
four generations of the same family living under the one roof. But as the UK’s
housing crisis has given way to a
“boomerang generation”, and as the shrinking amount of state funding for older care
leaves more families to
fend for themselves, there are signs that we might be starting to embrace a more
Mediterranean approach to
family life. The question we’re not asking is: at what cost?
In Victorian Britain (1837-1901), working-class young people left home in their
early teens to enter domestic
service, at one point the country’s biggest source of jobs. Half a century ago,
baby boomers came of age in a
world of cheap housing and plentiful jobs, which eased their route to independence.
The number of young people
going to university, many of them moving away from home, has ballooned from just 2%
immediately after the
Second World War to over 40% today. Around 1990s.
At the other end of the lifespan, as people live longer - though often with
protracted spells of poor health - we’ve
developed a professional care sector so that adult children don’t have to give up
work to look after ageing parents.
By contrast, in countries such as Japan and Italy, where there is a greater
expectation that families will step up,
there has been less reliance on professional care: across much of Italy, fewer than
one in 50 over-85s live in a
care home, compared with one in six in the UK
That generous baby-boomer deal is a thing of the past: one in five 25- to 29-year-
olds now lives with his/her
parents. The eye-watering costs of childcare mean more parents are relying on
grandparents helping out to enable
them to go back to work. And the rising cost of elderly care means those who need
it are often reluctant to fork
out. Rather than dip into the value of their house to pay, they would prefer to
pass the property on to their
children. These financial shifts are being compounded by government policy. The
tripling of tuition fees and the
axing of maintenance grants have loaded further debt on to young graduates. Last
year the government revived
its plans to scrap support for the housing costs of low-income 18- to 21-year-olds.
Cuts to social care have
implicitly shifted the balance of responsibility for looking after ageing parents,
and ministers have explicitly said
families must pitch in more.

. Page 5 of 36
11. What, according to the passage, is meant by “boomerang generation”?
(a) Young people who leave the parental home after completing their education.
(b) Young people who rebel against the existing system.
(c) Young educated people who share a home with their parents.
(d) Young people who adopt a Mediterranean approach to life.
12. Which of the following can be inferred from the passage about Britain’s
approach to family?
(a) It is influenced by its economy and its education.
(b) Job opportunities have always had a positive influence on Britain’s approach to
family.
(c) Cheap housing is the factor responsible for the independence of the young.
(d) Families are founded on economics of the world.
13. ‘Baby boomers’ are
(a) people having more than one baby.
(b) young children with good parental support.
(c) children of working class.
(d) persons born in the years following the World War II.
14. The passage suggests that austerity measures adopted by the British government
are likely to
(a) create tension in family life. (b) have a bearing on family life.
(c) affect the elderly negatively. (d) be detrimental to the society.
15. With which of the following statements is the author likely to agree with?
(a) People in England are generally very frugal.
(b) Generational separation in England is determined by economic realities.
(c) Spending habits in England have changed.
(d) Grandparents in England are no more considered a burden on the family.
Passage(Q.16-Q.20): The Centre is working on an industrial policy for backward
areas of the northeast as well
as the Himalayan States to subsidise the cost of capital, a senior official said.
“It is a challenge to access funds
in India,” said Department of Industrial Policy and Promotion (DIPP) Secretary
Ramesh Abhishek.“Cost of
funds is high, and the government is working to bring this down and make funds
easily accessible. We have
interest subsidy schemes specifically for more difficult areas like the northeast.
We are trying to come out with
an industrial policy for backward areas of northeast and the Himalayan States and
we are going to see how we
can subsidise the cost of capital in these areas,” he said at an interactive
session on Make in India initiative
online. Mr. Abhishek further said that norms were yet to be announced by the
concerned regulators, including
the RBI and the SEBI, regarding regulations for online lending platforms. “Those
platforms are doing a good
job. So we are hoping that regulators will come out soon with good and positive
regulations so online platforms
are able to do their business,” the DIPP Secretary said. On state-wise start-up
policy, he said as of now there are
16 states that have their respective start-up policies and the Centre has asked the
remaining states to soon join
the initiative. Noting that start-up policy is important for economic growth, he
said startups are the biggest job
creators. “We have to turn our youth into job givers not job seekers. Creating a
startup ecosystem is also
important,” he said. It is this perception that will help drive the economy in all
aspects. The development of
agriculture sector is a key priority area for the Centre, he said. The government
is encouraging organic farming
across the country and he has sought suggestions in this regard from the public.
Mr. Abhishek also said that
Invest India — the national investment promotion agency — in the last one and a
half years had promoted foreign
investments to the tune of $70 billion, of which already $3.1 billion worth of
investments had come into the
country. “This kind of hand holding was never done by the Centre earlier,” Mr.
Abhishek added. On the matter
of “troubles” relating to conversion of agricultural land to nonagricultural land
for its commercial use, the official
said it was a “tricky issue” and sought suggestions from stakeholders so that the
Centre can form a view on it
and then forward them to the state governments as land issues fall under the
jurisdiction of states.

. Page 6 of 36
16. What is the main objective of the Government behind all these steps?
(a) To encourage the people of north-east
(b) To promote business and commerce in coastal areas
(c) To make the north east region developed in every way where finance plays the
major role.
(d) To alleviate the poor health conditions of the people
17. Which of the following best explains the impact of the perception that the
youth should be job-givers and not
seekers?
(a) It helps to increase the economy of the nation and create better job
opportunities in the nation.
(b) It elevates the inclination of students towards higher education.
(c) The youth of the country gives a tough competition to the outside countries.
(d) The imbalance created in the economy after world war of the country gets
balanced.
18. Why are start-up policies encouraged in the mentioned states?
(a) They help the people to be dependent.
(b) They create more job opportunities for people.
(c) They create a better balance in the nation.
(d) The Government earns support in these regions.
19. Which of the following is a significant step in the development of these (north
eastern) regions?
(a) Regulations in the RBI (b) Establishment of online lending platforms
(c) More agricultural activities (d) None of these
20. Two statements (I) and (II) are given below. These statements may be either
independent causes or may be
effects of independent causes or a common cause. One of these statements may be the
effect of the other
statement. Read both the statements and decide which of the following answer choice
correctly depicts the
relationship between these two statements.
I. It is a challenge to access funds in India as the cost of funds is high.
II. The government is working to bring this down and make funds easily accessible.
(a) Statement (I) is the cause and statement (II) is the effect.
(b) Statement (II) is the cause and statement (I) is the effect.
(c) Both the statements (I) and (II) are independent causes.
(d) Both the statements (I) and (II) are effects of independent causes.
Passage(Q.21-Q.25): Ever since the 1980s the fashion had been to make companies as
lean as possible,
outsourcing all but your core competencies, expanding your just-in-time supplier
system around the globe,
loading up with debt to “leverage” your balance-sheet. Companies that hoarded cash
— even ones as good as
Toyota and Microsoft — were viewed with suspicion.
No longer. For many big American companies, the day of reckoning came when the
deepening financial crisis
brought about the abrupt closure or the overnight commercial-paper market. This
briefly sent even the most solid
companies into a desperate scramble to find money to meet such basic obligations as
paying their staff. Since
then, the guiding principle for managers everywhere has been to gather up whatever
cash they can find, and then
keep as much of it as possible for as long as possible.
But most of the panic is still hidden. In Britain, solid corporate giants are
finding it harder to rollover routine
loans. Across Europe, nervous accountants say they will need to see more proof that
firms are “going concerns”
before they sign off year-end accounts
The cash squeeze is a huge problem for the world economy, because as firms cut
discretionary spending wherever
they can, the result is likely to be a corporate version of what John Maynard
Keynes called the “paradox of
thrift”. Every firm does what is prudent for itself, but by cutting its spending,
it slows down the economy still
further and thus hurts everybody, including itself. This will only reinforce the
need for expansionary monetary
and fiscal policy to boost demand; and also for more direct support in credit
markets.

. Page 7 of 36
For the few lucky hoarders, this is a time to feel both smug and predatory. Cash-
rich drugs firms, have all said
that the financial turmoil presents an opportunity for them to buy biotechnology
companies at knock-down
prices.
For the non-hoarders, there is a balance to be struck. In the short term, some of
the old ways to perk up your
share price now seem suicidal. Huge dividends or share buybacks have to be regarded
as reckless. What was
once seen as evidence of corporate fitness for the moment looks like anorexia. More
padding — in the form of
cash in the bank — will be necessary to secure a clean bill of health. Likewise,
ultra-lean supply chains no longer
look like such a brilliant idea when you have to find cash to keep afloat a
supplier that cannot get even basic
trade credit. “Just in time is giving way to “just in case”.
And there will also come a time when the necessity to safeguard cash is not so all-
consuming. But even if cash
does become more plentiful, it is doubtful whether today’s generation of managers
will be quite so cavalier about
taking it for granted. That change in attitude, more than anything else, will be
the legacy of this credit crunch for
the corporate world.
21. The passage is chiefly concerned with
(a) warning that the supplier system has to be trimmed down.
(b) demonstrating that cash transactions are preferable to transactions on credit.
(c) arguing for a critical review of corporate cash flow requirements.
(d) recommending a way out of recession by increasing spending.
22. It can be inferred from the passage that the ‘paradox of thrift in Keynesian
economics is due to which of the
following?
(a) Increased savings represents a diminishing circular flow of money.
(b) Decreased savings represents an increasing circular flow of money.
(c) An expansionary fiscal policy stimulates the economy in anticipation of a
business-cycle contraction.
(d) An expansionary monetary policy increases the supply of money to stimulate the
economy in anticipation of
a business-cycle contraction.
23. The passage warns of which of the following as dangers to a corporate “going
concern” status?
(a) A high degree to which a business utilizes borrowed money
(b) Reacquisition by a company of its own stock by distributing cash
(c) An ultra-lean supply chain management
(d) All of the above
24. The “not so cavalier” attitude mentioned in the concluding lines of the passage
is best defined as one that
(a) treats cash as dispensable where credit is easily available.
(b) recognizes that cash is hard to come by.
(c) Just in time supplier system
(d) All of the above
25. The author cites the fact that the drug firms are buying companies at very
cheap prices in order to do which of
the following?
(a) Point out that acquisitions are the way for growth.
(b) Illustrate how downturns tend to be good times for certain sectors.
(c) Warn that companies that are strapped for cash are quickly cannibalized.
(d) Imply that cash-hoarding firms now outperform indebted firms

. Page 8 of 36
Passage(Q.26-Q.30): All of us who have worked in any organisation would have had
our fair share of bosses
good and bad. The good bosses are those who take you under their wings, teach you
the ropes and guide you
through the organisational maze. They are those who inspire ordinary people to do
extraordinary things. There
are also bosses who bark and snarl their instructions or orders, point a critical
finger at every opportunity and
even tick you off at meetings and seminars. The human mind is geared to sift the
good memories from the bad
and retain them to dwell on in tranquil moments. The bad memories are best given a
decent burial.
However the published results of some recent research at the University of
Manchester on workplace behaviour
brought to my mind a few bosses I would rather not have encountered. The research
states that bosses who
display psychopathic, arrogant, brutal tendencies on their subordinates are likely
to engage in counterproductive
behavior. Also, quite often you find narcissists in top management cadre who
believe in the principle “vex and
boss”. All the aggressive, toxic rhetoric and constant bullying can whittle down
your ego and reduce you to a
robot.
I remember, whenever a particular boss called us we stepped into his chamber as
though towards a guillotine.
Full of himself, totally in love with his own voice, he would crush us with his
demeaning comments. He was the
oracle, he was the final word. No wonder, when he retired and visited the office,
he was given a wide berth and
a cold shoulder.
Next was another who was equally famed for his rudeness and did not hesitate to
fling files from his table.
Though I did not myself have any files flung at me. I got my share of tongue-
lashing. One consolation was
they were uniform in their behaviour with everyone.
Of course there were those who played favourities and humiliated those who were
not. The third boss was a clear
misogynist who faulted me over everything while all my male colleagues were
cheerfully forgiven. He took
immense delight in correcting and re-correcting my drafts and returned my papers
with unsavoury comments.
So, it was a mailer of particular pleasure when in the years to come I got past him
on the career ladder
What gives power to one individual over another? Is it rank, class or privilege
that he thinks bestows on him the
divine right to humiliate and terrorise “lesser” mortals around?
Meryl Streep, the screen legend, while receiving the Golden Globe Award, made a
reference to a person in a
powerful position who mocked a reporter with disability. She said she considered it
inexcusable to attack
“someone he outranked in privilege, power and the capacity to fight back”. She
added; “That instinct to humiliate
when it is modelled by someone on a public platform, it filters down into
everyone’s life because it gives
permission for others to do the same.’
However in the end, one realises the futility of all power and ultimate impotence
of all those who exercise such
power.
26. The author is of the view that
(a) Both (b) and (d)
(b) bosses who are disdainful of their subordinates are looked upon with awe.
(c) bosses who are roughshod over their underlings are likely engaging in
counterproductive behaviour.
(d) bosses who are domineering are generally popular in the organisation.
27. The last sentence of para 3 states, “When he retired and visited the office, he
was given a wide berth and a cold
shoulder” In the given context, what do you understand by the phrase, shown in
italics?
(a) The former boss was accorded a cold, dispassionate welcome.
(b) The employees stayed well away from the former boss with deliberate
unfriendliness.
(c) The employees had a lukewarm interaction with the former boss, and grumbled
among themselves.
(d) The former boss’s visit was a damp squib.

. Page 9 of 36
28. Which among the following is / are TRUE according to the passage?
I. Good bosses educate their subordinates on the established way of doing things.
II. Bosses who have extreme love and admiration for themselves trouble people
aggressively.
III. Studies reveal that celibate bosses are less vehement, less arrogant, and
rarely domineering.
(a) Only I (b) Only II (c) I and II (d) I and III
29. According to the passage, who are not good bosses?
(a) who take you under their wings
(b) teach you the ropes
(c) guide you through the organisational maze
(d) who bark and snarl their instructions
30. What does the phrase "I got my share of tongue- lashing" mentioned in the
passage implies?
(a) The author was spared the wrath of the boss.
(b) The author was rude to the boss.
(c) The author was severely scolded by the boss on occasions.
(d) The author was complimented by the boss

Directions (Q.66 – Q.105): Read the comprehensions carefully and answer the
questions based on it.
Passage(Q.66-Q.70): Those who did not comply with the lockdown were extensively
prosecuted under Section
188 of the Indian Penal Code, 1860 (the “IPC”). In a statement to the states, the
Home Ministry stated that
violators of the containment measures will face penalties under the Disaster
Management Act, 2005 (the “DMA”)
as well as Section 188 of the IPC. Consider the following provisions and those that
are related:
Section 188 of the IPC punishes people who disregard an order issued by a public
servant with imprisonment
ranging from one to six months. Section 188 of the IPC is the provision under which
penalty is administered for
individuals who violate instructions issued under the Epidemic Diseases Act, 1897
(the “EDA”). Section 51 of
the DMA punishes two types of offences: impeding any official or employee of the
government or anybody
authorised by any authority under the DMA in the exercise of their
responsibilities, and refusing to comply with
any instruction made by the authorities under the DMA. On conviction, the sentence
can be increased to one
year, or two years if the refusal results in the loss of life or an impending
danger.
Section 505 of the IPC punishes anyone who publish or circulate anything that is
intended to provoke fear or
panic with three years’ imprisonment or a fine, or both. Section 54 of the DMA
allows for up to a year in prison
for anyone who issue or circulate a false alert or warning about a disaster or its
severity or extent. According to
Section 52 of the DMA, anybody who makes a fraudulent claim for “any relief,
assistance, repair, reconstruction,
or other benefits” from any governmental authority can face up to two years in
prison and a fine.
Any authority under the DMA has the ability to demand resources such as people and
material resources,
premises such as land or buildings, or sheds and automobiles for rescue operations.
Despite the fact that the
DMA provides for compensation, anybody who disobeys such an order faces
imprisonment for up to a year.
[Edited and extracted from,
https://viamediationcentre.org/readnews/NjE=/ENFORCEMENT-OF-SECTIONS#188-269-270-
OF-IPC-FOR-LOCKDOWN-VIOLATION]
66. Rohit has been trapped at home for weeks, under lockdown, and is becoming bored
since he has had nothing to
do for a long time as his school-leaving tests are over, and the entrance exams he
has been studying for have
been postponed. He decides to have some fun and manipulates a picture on his
computer, which depicts Rohit’s
school in ruins. Rohit then sends the photograph to his WhatsApp contacts with the
remark, “This is what our
school will look like in 100 years!” Jyoti, one of Rohit’s classmates in the
WhatsApp group where he shared the
photos, misses the caption and believes their school has been destroyed. She
becomes terrified, and her parents
submit a complaint with the police, who want to charge Rohit under Section 505 of
the IPC. Will the police case
be successful?
(a) Yes, since Rohit spread something that was likely to frighten his peers.
(b) Yes, since Rohit should have acted more maturely during tough moments such as
the lockdown.
(c) No, because Rohit just sent the photograph to his friends and did not
‘circulate’ it.
(d) No, because the image and caption were unlikely to induce fear or alarm.
67. After the situation with the picture was settled, Rohit began spending an hour
every evening standing on his
balcony, gazing out over his neighbourhood. One day, he spotted a few medical
personnel dressed in protective
clothing knocking on his neighbour, Mr. Kapoor’s door. Because Mr. Kapoor had
claimed specific Covid-19-
like symptoms, the personnel had come to take a sample from him for a Covid-19
test. The personnel were
employees of the local government hospital and were executing on directions from
their superior, a DMA
authority. Mr. Kapoor, on the other hand, refused to unlock his door, forcing the
workers to return. Later that
day, the police informed Mr. Kapoor that if he did not comply and produce a sample
for testing, he would be
charged under Section 51 of the DMA. Is Mr. Kapoor in violation of Section 51 of
the DMA?
(a) Mr. Kapoor is not a government official, thus no.
(b) Yes, as Mr. Kapoor had, however, prevented government personnel from carrying
out their duties.
(c) Yes, because Mr. Kapoor was required to produce a sample after reporting
symptoms.
(d) No, because the DMA authority’s directions only extended to the medical
professionals, not Mr. Kapoor.

. Page 16 of 36
68. Along with the above facts, Mr. Kapoor refused to produce a sample despite
police warnings that he may face
charges under Section 51 of the DMA. In order to get a sample and establish that
there was no spread in Mr.
Kapoor’s neighbourhood, the police chose to talk with him again, and this time they
informed him that they
would also file a case against him under Section 188 of the IPC. Is Mr. Kapoor in
violation of both Section 188
of the IPC and Section 51 of the DMA?
(a) No, a person cannot be tried and punished for the same offence under two
separate sections of the law.
(b) No, because Section 188 of the IPC only applies to wrongdoings committed by a
public servant.
(c) Yes, Mr. Kapoor had violated both Section 51 of the DMA and Section 188 of the
IPC by refusing to provide
a sample.
(d) No, neither Section 51 of the DMA nor Section 188 of the IPC have been violated
by Mr. Kapoor.
69. Mr. Kapoor eventually agrees to provide a sample and is assured that his
findings would be available in 24 hours.
While waiting for the findings, Mr. Kapoor becomes irritated and sends a letter to
the local District Magistrate,
stating his family has been terribly impacted by the Covid-19 outbreak, and that
his family should be
compensated because he was suffering from the illness himself. Mr. Kapoor’s test
results, on the other hand, are
negative for the illness which he already knew. The police are now furious with Mr.
Kapoor and intend to charge
him with breaking Section 52 of the DMA. Is Mr. Kapoor in violation of Section 52
of the DMA?
(a) No, since he was concerned about his test findings and was entitled to
compensation.
(b) Yes, since he submitted a false compensation claim with the District
Magistrate.
(c) Yes, because he originally refused to provide a sample to medical personnel.
(d) No, because the results of his tests were negative.
70. Mr. Kapoor receives a phone call from one of his employees a few days after the
events recounted in the
preceding questions. The employee informs Mr. Kapoor that the warehouse where he
stored the products, he
dealt in had just flooded due to a storm. When Mr. Kapoor hears this, he rushes
across the street to Rohit’s house.
Mr. Rishabh, Rohit’s father, also works for Mr. Kapoor, and Mr. Kapoor informs Mr.
Rishabh that he must
immediately hand over the keys to his car to Mr. Kapoor, as he intends to use it to
retrieve the goods from the
warehouse. Mr. Kapoor further informs Mr. Rishabh that because Mr. Rishabh works
for him, Mr. Kapoor is
an‘authority,’ and so has the power to requisition the automobile from Mr. Rishabh
under the DMA. Is Mr.
Kapoor, correct?
(a) No, under the DMA, the ability to requisition resources is only accessible to
an authority under the DMA,
not to an employer.
(b) No, under the DMA, the authority to requisition resources is only available to
rescue people, not goods.
(c) Yes, because an employer is an ‘authority’ in the eyes of an employee, Mr.
Rishabh must hand over the keys
to his automobile to Mr. Kapoor.
(d) Yes, because the loss of the products would have a negative impact on Mr.
Rishabh’s livelihood.
Passage(Q.71-Q.75): Suicide abetment is a criminal offence under the Indian Penal
Code of 1860. Section 306
of the IPC provides “If any person commits suicide, whoever aids and abets the
commission of such suicide will
be punished with imprisonment for a term of up to 10 years, and shall also be
liable to fine.”
Suicide abetment is a serious crime that is prosecuted in a Sessions court and is
cognizable, non-bailable, and
non-compoundable. A cognizable offence is one for which a police officer can make
an arrest without a court
order. A non-bailable offence is one in which bail is granted at the discretion of
the court rather than as a matter
of right.
A non-compoundable offence is one in which the complainant’s complaint cannot be
withdrawn even if the
complainant and the accused have reached an agreement of compromise. A case
containing a non-compoundable
offence cannot be withdrawn by the court, and every such complaint must be followed
by a trial in which
evidence is held against the accused.
The crime of abetment of suicide has two main components. The first is a suicidal
death. The second component
is the accused’s intent to aid in such suicide. Legally, whether a death is a
suicide or not is a factual finding,
which means evidence must be analysed to determine whether or not the death was a
suicide.

. Page 17 of 36
In common parlance, the term “suicide” is freely used to all cases of self-
destruction, although suicide is never
assumed. A suicide determination is established when the deceased individual is
considered to have known the
likely outcome of what the self-harm is about to inflict to the person and
nevertheless commits it willfully.
Once this finding is reached, the purpose of the individual charged of abetment of
suicide is investigated.In
demonstrating any crime, the accused’s actions reveal his or her purpose. Supreme
Court in multiple judgments,
including one in 2002 in the case of ‘Sanjay Singh v State of Madhya Pradesh,’ have
found that a speech or
statement made in haste or fury does not constitute abetment of suicide.
Furthermore, if the deceased individual is judged to be extremely sensitive in
comparison to a reasonable person,
the court has said that the accusation of abetment to suicide will be reduced.
[Extracted from, https://timesofindia.indiatimes.com/india/disciplining-a-student-
not-abetment-to-suicide#sc/articleshow/86797335.cms]
71. Ms. Lucy, a Chennai resident, has been charged and arrested for Sedition, a
non-bailable offence under Indian
law. She has asked for bail in this regard. Can the Court grant her bail?
(a) No. Because sedition is a non-bailable offence, the accused cannot be granted
bail.
(b) No, because sedition is such a serious offence against the nation’s interests,
the Court will not give bail to
such an accused.
(c) Yes. Bail is granted at the discretion of the court and is not guaranteed as a
right to the accused.
(d) Yes, Ms. Lucy has bail as a matter of right.
72. Money and Aby have been great friends since they were children. They attend the
same school, return home
together, play together, and have been inseparable since they were very young.
However, they had a tremendous
quarrel one day, and Money exclaimed on the rush of the moment that she hates Aby
and wishes he would kill
himself. Aby was devastated by this and committed suicide in his sadness. Will
Money be prosecuted for aiding
and abetting suicide?
(a) Yes. Money will be held accountable since her statements had a direct impact on
Aby’s choice to commit
suicide.
(b) No. Money’s remarks were spoken in haste;therefore, it cannot be assumed that
Money intended to trigger
Aby’s suicide.
(c) Yes. Money had been Aby’s best friend since childhood and should have been more
aware of the impact her
remarks may have on Aby.
(d) No, Aby’s suicide was his personal choice and hence even if anybody influenced
him, he should be sensible
enough to commit suicide.
73. Assume, in the preceding situation, that a lawsuit has been initiated against
Money based on the Complaint filed
by Aby’s parents. They subsequently decide to forgive Money because of their son’s
connection with Money.
As a result, they wish to withdraw the case. Will their parents be able to do the
same?
(a) Yes. Since they filed the complaint, they have the ability to withdraw it at
any time.
(b) Yes. The Court will enable the parents to withdraw the lawsuit due to the
unique circumstances surrounding
Aby and Money’s friendship.
(c) No. Such charges of abetment to suicide cannot be withdrawn under the law.
(d) No, since once a complaint is filed, it cannot be withdrawn, and if the
individual who files the case does so,
he or she is held liable for wasting government officials’ time.
74. In the preceding situation, it was discovered that Aby had a sensitive
disposition and was easily injured even in
simple issues. Will Money be acquitted as a result of this new information?
(a) Yes, if the deceased individual is deemed to be very sensitive in comparison to
a reasonable person, the
Court will never hold a person guilty of aiding and abetting suicide.
(b) No. The deceased’s sensitivity is an element that can aid Money’s case, but it
is not a guarantee of her
acquittal.
(c) No. Money should have been more cautious given Aby’s sensitive disposition.
(d) Yes, even if it was an unknown fact, but then this makes clear that Money from
nowhere is liable for abetment
to suicide as it was Aby’s mental condition which made him do so.

. Page 18 of 36
75. Which of these Courts, according to the Paragraph, has the authority to try a
case of Abetment to Suicide?
(a) First Class Judicial Magistrate (b) Second Class Judicial Magistrate
(c) Chief Judicial Magistrate (d) Sessions Judge
Passage(Q.76-Q.80): Intellectual property (IP) is a term referring to a brand,
invention, design or other kind of
creation, which a person or business has legal rights over. Almost all businesses
own some form of IP, which
could be a business asset.
Copyright- The rights of authors of literary and artistic works (such as books and
other writings, musical
compositions, paintings, sculpture, computer programs and films) are protected by
copyright, for a minimum
period of 50 years after the death of the author. The owner of copyright work can
generate wealth not only by
exploiting it himself but also by sharing it with others for mutual benefits. This
can be done by way of assignment
or licensing of copyright.
Trademark - A trademark is a word, group of words, symbol or combination of these
that distinguishes the
products of one competitor from the products of other competitors in the
marketplace. It has to be unique,
distinct, and capable of being represented graphically. Also, should be capable of
distinguishing the goods or
services of one person from those of others a brand can be registered under the
trademark law.
Patent- A patent is an exclusive right of monopoly granted for a non-obvious
invention, which is a product or a
process that provides, in general, a new way of doing something, or offers a new
technical solution to a problem.
Term of every patent in India is 20 years from the date of filing of patent
application, irrespective of whether it
is filed with provisional or complete specification. However, in case of
applications filed under PCT the term of
20 years begins from the international filing date accorded under PCT. In case of
contract of service right to
patent belongs to the employer. Monopoly is granted through which the inventor
monitors and controls the
availability of the invention to the public.
[Extracted from, https://www.wto.org/english/tratop_e/trips_e/intel1_e.htm]
76. A, wrote a novel after getting inspired from the Harry Potter series. ZXY a
publishing house came forward to
publish the book of A. They took over the copyright of the book with a payment of
INR 100. Later A realised
he was cheated as other publishing houses came forward with much lucrative offers.
A sold his publishing rights
to ZZZ publishers. A was sued by ZXY for copyright infringement. Decide.
(a) A has not infringed copyright deal as he was cheated by ZXY thus making the
copyright contract null and
void.
(b) A has infringed the copyright deal as from the moment he signed the deal for
INR 100, he had repudiated
his rights over the book.
(c) A has not infringed the copyright deal as it is his original work inspired by
the Harry Potter series and he too
deserves to be successful like J.K Rowling.
(d) A has infringed the copyright deal as he had to take permission from ZXY
publishers before selling it to ZZZ
publishers.
77. A started a clothing company named ‘Regal’ which turned out to be successful. A
obtained a trademark over the
name Regal. B started a clothing company named ‘Regel’. A filed a suit for
trademark infringement against B.
Decide.
(a) B has not committed trademark infringement as his company name is different
than that of A.
(b) B has committed trademark infringement as the name of B’s company is similar to
that of A’s and likely to
be confused as both deal with similar and identical goods.
(c) B has not committed trademark infringement as his name is different to that of
A’s name and is not likely to
be confused.
(d) B has committed trademark infringement as it is a ploy from him to confuse
people with the well-known
brand of A.

. Page 19 of 36
78. Z was hired by ZYZ Company to work in their research wing which was developing
Covid- 19 vaccine. While
working, Z came up with a new combination which was extremely effective, while
tested on animals. Z went
ahead to claim patent over the formulae but was challenged by ZYZ board. They felt
they should have the patent
as he was their employee acting under the company’s supervision. Decide.
(a) Z will not be granted with the patent as the efficiency of the vaccine is still
very much uncertain.
(b) Z will not be granted the patent as he was an employee of ZYZ Company who was
specifically hired for the
purpose of research. Hence, the company should become the patent holder.
(c) Z will be granted a patent, as it was his individual effort and skill that made
the creation of formulae possible.
(d) The company cannot obtain the patent only because they hired Z. Z was hired in
the research wing but was
not explicitly asked to discover the vaccine.
79. Which of the following statements regarding patents is incorrect?
(a) It grants the inventor a monopoly grant through which the inventor monitors and
controls the availability of
the invention to the public.
(b) The patent owner holds the exclusive right for 20 years from the date of
registration.
(c) A patent can be obtained for an innovation or modification on a previously
available product/instrument.
(d) Patents should be useful and non-obvious.
80. Choose the right set of intellectual properties for the below given examples.
A. Y wrote a poem which was purchased by U-series for the sake of distribution and
monetization.
B. X came up with a new tagline- ‘Miraculously tasty’ for his home-made product
distribution company.
C. A, invented a new machine which could zero the gravity around 10 metres of its
radius.
D. B, a dancer, made a video of his boneless dance. The video went on to become a
huge hit.
(a) Copyright, trademark, patent, copyright
(b) Trademark, patent, copyright, copyright
(c) Copyright, patent, trademark, trademark
(d) Patent, trademark, copyright, copyright
Passage(Q.81-Q.85): Disapproving the blanket orders passed by a single judge of the
Rajasthan High Court to
not list applications for bail and suspension of sentence as urgent matters during
the lockdown, the Supreme
Court has observed that the right to apply for bail is an individual right implicit
in Articles 14, 19 and 21 of the
Constitution.
The Court has observed that such blanket bans would suspend Fundamental Rights of
individuals and block
access for seekers of liberty to apply for bail. These orders were challenged
before the Supreme Court by the
Rajasthan High Court itself.
"Such an order also has the effect of temporarily eclipsing statutory provisions"
the Supreme Court said. While
noting that the blanket order was passed by the High Court without compliance of
procedure established by law,
the Bench has observed that blanket order prohibiting listing of bail application
or applications for suspension
of sentence in appeals also infringes upon the right of personal liberty of
incarcerated persons.
Such right has been taken away by judicial order, without compliance of procedure
established by law, which in
our constitutional jurisprudence, is akin to "the due process" dictum." the Bench
said "Right to apply for bail is
an individual right implicit in Articles 14, 19 and 21 of the Constitution", the
judgment authored by Justice Bose
stated.
The Court also observed that the right of an accused, an under trial prisoner or a
convicted person awaiting appeal
court's verdict to seek bail on suspension of sentence is recognised in Sections
439, 438 and 389 of the 1973
Criminal Procedure Code. Court also held that granting a bail is discretion of
court on basis of merit of the case.
https://enalsar.informaticsglobal.com:2278/top-stories/right-to-apply-for-bail-
individual-right-articles-14-9-21-
supreme-court-high-court-of-rajasthan-v-state-of-rajasthan-ll-2021-sc-523-182834
Live law dated29/09/21

. Page 20 of 36
81. X was arrested on the charges of possessing drugs. He filed a writ petition in
the Delhi High court stating that
the arrest is violative of his right under article 19. The high court dismissed his
petition. Are the facts given in
consonance with the aforementioned Supreme Court ruling?
(a) Yes, but X should have filed a bail application instead of a writ petition.
(b) Yes, but there is no obligation on the court to entertain X’s petition
(c) No, X’s right under article 19 has been breached and the court should have
entertained his petition.
(d) No, sections 439 and 389 of Criminal Procedure code will not be applicable in
the present case.
82. Suppose in the previous question, X filed the same petition in Rajasthan High
court but it got dismissed, would
this change in facts change your answer to the previous question?
(a) Yes, the dismissal of the petition is against the Supreme Court ruling.
(b) No, the dismissal of the petition is against the Supreme Court ruling.
(c) Yes, the dismissal of the petition is not against the Supreme Court ruling.
(d) No, the dismissal of the petition is not against the Supreme Court ruling.
83. ‘Ram’ was arrested on the charges of murdering a person named ‘Shyam’. During
the trials, it was found that
Shyam was alive and the entire case was a false accusation. Now, Ram filed a
petition to quash the trials.
However, the court was reluctant to quash the case since there were other charges
against Ram in the similar
case. Decide the applicability of aforementioned decision by the Supreme Court.
(a) The court is under an obligation to quash the trial, as not doing so would
infringe the fundamental rights of
Ram.
(b) The court is not under any such obligation, as the decision is applicable for
the bail applications.
(c) The court is under an obligation because the judgement came from the apex
court.
(d) Ram shall file a bail application instead to avail the rights.
84. In the previous question, will you change your response if Ram has filed an
application of interim bail, but the
court is not granting the bail. To defend its stance, court has stated that there
are certain new angles in the case
which needs to be explored.
(a) Yes, because the Supreme Court has made it clear that bail is one of the
fundamental rights of an accused.
(b) No, the court can still reject a bail application on its merits.
(c) Yes, bail shall be granted as the application is merely for an interim one.
(d) No, as there are certain new angles in the case which shall be explored first.
85. Which of the following can be conclusively derived from the passage?
(a) The bail applications shall be listed mandatorily, and there shall be no ban on
this.
(b) The order by Supreme Court is only applicable to the state of Rajasthan.
(c) The order deals with other ‘rights of an accused’ as well.
(d) Right to bail is a recognised right under the relevant provisions of Indian
constitution as well.
Passage(Q.86-Q.90): The Supreme Court observed that a High Court can quash criminal
proceedings in exercise
of its inherent powers under Section 482 Cr.P.C., even if the offences are non-
compoundable and the compromise
is reached after conviction.
The bench headed by CJI NV Ramana added that criminal proceedings involving non-
heinous offences or where
the offences are predominantly of a private nature, can be quashed irrespective of
the fact that trial has already
been concluded or appeal stands dismissed against conviction.
While allowing the appeal, court therefore concluded:
We thus sum up and hold that as opposed to Section 320 Cr.P.C. where the Court is
squarely guided by the
compromise between the parties in respect of offences 'compoundable' within the
statutory framework, the extra#ordinary power enjoined upon a High Court under
Section 482 Cr.P.C. or vested in this Court under Article 142
of the Constitution, can be invoked beyond the metes and bounds of Section 320
Cr.P.C. Nonetheless, we
reiterate that such powers of wide amplitude ought to be exercised carefully in the
context of quashing criminal

. Page 21 of 36
proceedings, bearing in mind: (i) Nature and effect of the offence on the conscious
of the society; (ii) Seriousness
of the injury, if any; (iii) Voluntary nature of compromise between the accused and
the victim; & (iv) Conduct
of the accused persons, prior to and after the occurrence of the purported offence
and/or other relevant
considerations.
https://enalsar.informaticsglobal.com:2278/top-stories/supreme-court-criminal-
cases-predominantly-private#nature-section-482-crpc-compromise-182751 live law
dated 29/09/21
86. X & Y were friends but X was always jealous of Y due to his booming business. X
mixed poison in Y’s drink
leading to Y’s death immediately. With a thorough investigation it became quite
apparent that X was the one
who killed Y. X is convicted by the district court of Allahabad, but Y’s family
wants to compromise since X has
small kids and Y’s family doesn’t want to ruin their life. Accordingly, the court
quashes the criminal proceedings
against X. Was the court right in doing so?
(a) Yes, the court has inherent powers to quash criminal proceedings.
(b) Yes, the court has satisfied all the prerequisites under section 320 of the
Cr.P.C
(c) No, the court does not have the power to quash criminal proceedings.
(d) No, the court exceeded its power since the offence was murder which is a
heinous crime and it is non#compoundable.
87. Suppose in the previous question, X & Y lived in Delhi and the case was being
heard by Delhi High court. With
the rest of the facts remaining same, will this slight modification in facts change
your answer to the previous
question?
(a) Yes, the both the courts have power to quash the criminal proceedings.
(b) No, both the courts do not have the power to quash criminal proceedings.
(c) Yes, the court has the power to quash the criminal proceedings.
(d) No, the court does not have the power to quash the criminal proceedings.
88. Jai and Veeru were two friends, who grew up together in the city of Rampur.
They fell in love with the same girl
named ‘Basanti’. Once they came to know about this fact, both of the friends were
deeply hurt. In a fit of rage,
Jai published some posters, which had some defamatory content about Veeru. After
seeing this, Veeru lodged a
complaint against Jai, and he was convicted for the offence of defamation. However,
they both had a heartfelt
talk and thought of taking back the case. Pick the most suitable option.
(a) The case shall be quashed, since the parties have arrived at a compromise.
(b) The case can be quashed, since the parties have arrived at a compromise and the
offence was not that grave.
(c) The case cannot be quashed, as the case was not dealt by the High Court.
(d) The case shall be quashed, since this is an inherent power of the courts.
89. In the previous question, will you change your answer if Jai would have killed
Veeru? Provided that Jai has tried
reaching at a compromise with Veeru’s family?
(a) Yes, since the offence is grave in nature and injury is serious.
(b) No, since the both the parties have tried reaching a compromise.
(c) Yes, since both Jai and Veeru were extremely close friends.
(d) No, since the proceedings can be quashed anytime as mentioned under the powers
of courts.
90. Which of the following cases cannot be quashed even if the parties have reached
a compromise?
(a) A stole the purse of B, which led to him missing the flight due to lack of
documents (that were stolen).
(b) A broke into B’s house, but left as he saw the thief catching mechanism.
(c) A pushed B against the wall in a fight, due to which B’s leg got fractured.
(d) A stealthily evaded the security check-in to smuggle certain illicit drugs in
the flight.

. Page 22 of 36
Passage(Q.91-Q.94): The Bench held that it was incorrect to say that the sale deed
was without consideration.
Relying on S.25 of the Indian Contract Act, 1872 it held that:
"Section 25 of the Contract Act is to the effect that an agreement without
consideration is void but if a document
is registered on account of natural love and affection between the parties standing
in a near relation to each other,
then such an agreement is not void." (Para 20)
Observing that the parties are in near relations and in the present case the elder
brother had come to the help of
younger brother by discharging his debts, the Bench held that the sale deed cannot
be said to be without
consideration.
Further, the Bench affirmed the Appellant's reliance on Bellachi (Dead) by LRs v
Pakeeran, which held that:
"The burden of proof regarding the genuineness of document lies upon the vendee and
that in case of a registered
document, there is a presumption that it was executed in accordance with law."(Para
17)
In response to the Respondents argument that the sale deed was obtained by
misrepresentation and fraud, the
Bench held that:" in terms of Order VI Rule 4 of the Code, in all cases in which
the party pleading relies on any
misrepresentation, fraud, or undue influence shall state in the pleadings the
particulars with dates and items in
the pleadings. The extract from the written statement or the plaint does not show
that there is any pleading of
misrepresentation or fraud. If party relaying on certain pleadings if not raised
initially cannot be raised further
in a suit”.
https://enalsar.informaticsglobal.com:2278/top-stories/section-92-evidence-act-
oral-evidence-sham-document#supreme-court-182906 Live Law 01/10/2021
91. X and Y were neighbours and knew each other since their childhood. X purchased
a brand-new Harley Davidson
but due to crunch of time he hardly got the opportunity to ride it. He enters into
an agreement with Y wherein he
agreed to give his bike to Y and Y agreed return his bike after 6 months. After 2
months, X felt the need of
having his bike back, but Y refused. X filed a suit in the court of law that Y must
return his bike because the
agreement was void ab initio. Is the argument of X in consonance with the
aforementioned passage?
(a) No, the given fact situation comes under the exception of section 25 of the
Indian Contract Act.
(b) Yes, the agreement was void.
(c) No, consideration was fulfilled for both the parties and hence a valid
agreement.
(d) Yes, order VI rule of the code of civil procedure will come in the rescue of X.
92. Suppose X and Y were best friends and had lots of love and affection between
them when they entered into the
agreement. Would this change your answer to the previous question?
(a) No, the agreement is void as per section 25 of the Indian contract act.
(b) Yes, the agreement is not void as per section 25 of the Indian contract act.
(c) Yes, the agreement is void as per section 25 of the Indian contract act.
(d) No, the agreement is not void as per section 25 of the Indian contract act.
93. Minie entered into an agreement with one of her colleagues named ‘Rishi’
regarding her jewelleries. All the
documents were in accordance with law and the agreement was completed. However,
Rishi defaulted on the
payment, which compelled Minie to file a law suit. During the proceedings and
calculation, she realised that
Rishi has deceived her in getting a greater amount of gold than mentioned in the
agreement. She now wanted to
argue for fraud as well. Decide.
(a) She can argue for fraud under the same suit, as the issue is related to the
same case.
(b) She cannot argue for fraud under the same suit, as it was not mentioned in the
plaint initially.
(c) She can argue for fraud, as Rishi took more gold than what was initially agreed
upon.
(d) She cannot argue for fraud, as she should have calculated the same before
entering into the agreement.

. Page 23 of 36
94. Which of these would represent a close relationship to effectuate an agreement
without consideration under
Section 25 as mentioned above?
(a) Two colleagues, who have worked in the same office.
(b) Two passengers, who met in a train and became good friends over the time.
(c) Two neighbours, who have recently shifted to a colony named ‘Shaantivan’.
(d) None of these
Passage(Q.95-Q.100): The law does not ask a law-abiding citizen to helplessly
suffer a bodily injury or assault
and do nothing to save himself/herself or any other person, when confronted with
imminent and unlawful
aggression.
In Chapter IV (Section 96 to 106) of ‘General Exceptions’ of the Indian Penal Code
(IPC), the law recognizes
and protects the right of a person of self-defence of the body.
The cardinal principle underlying the doctrine of the right of private defence is
that when an individual or his
property is faced with danger and immediate aid from the State machinery is not
readily available, then that
individual is entitled to protect himself/herself or his/her property even by using
force, if need be.
Section 96 provides that 'nothing is an offence which is done in the exercise of
right of private defence'. Section
97 provides that every person has a right to defend his/her own body or the body of
any other person against any
offence affecting the human body, though subject to the restrictions contained in
Section 99, which limits the
right of private defence to the extent of harm caused. In other words, it permits
the right of private defence to be
exercised without the use of excessive force and reasonable force shall be used.
Section 100 read with Section 106 extends the right of private defence to cause
death in certain occasions
involving apprehension of death, grievous hurt, assault with the intention of
committing rape, and others. It is
thus clear that a person saving herself or any other person from an offence of rape
or attempt to rape, is entitled
to use maximum force to stop the offender from committing such a crime, subject of
course to certain limitations.
The right of private defence of the body commences as soon as a reasonable
apprehension of danger to the body
arises from an attempt or threat to commit the offence, though the offence may not
have been committed. It
continues as long as such apprehension of danger to the body continues.
(Extracted with requisite revisions and edits from‘Understanding the nuances of the
right to private defence:
Criticism of the Bulandshahr trial court conviction’at
https://www.barandbench.com/columns/understanding#the-nuances-of-the-right-to-
private-defence-criticism-of-the-bulandshahr-trial-court-conviction)
95. Shweta was an avid shopaholic and could spend hours browsing through the
stores. During one such expedition
of hers in SarojiniChowk, she saw a man at a close distance who was holding a knife
and was swinging it at her.
This gave her goosebumps and being a lady of strong will and to teach that man a
lesson, she picked a pebble
from one of the showcase vases and threw it at him injuring him in the process as
the pebble had hit him on his
head. The stranger started bleeding profusely. A claim was later brought against
Shweta for her rash conduct to
which she took the defence of private defence. Is her claim likely to hold well?
(a) Yes, in the absence of any state machinery at the immediate disposal of the
person, s/he is entitled to protect
it by any means.
(b) No, there was no immediate threat or danger to Shweta to entitle her to
exercise the right to private defence
and hence she cannot claim the defence.
(c) Yes, there was threat to the body of Shweta as the swinging of knife at her is
a reason enough to invoke a
fear and for the protection of her body it was reasonable enough to have exercised
the right to private defence.
(d) No, the force i.e. hitting with a pebble on head was disproportionate to the
apparent threat in the situation.

. Page 24 of 36
96. During the investigation, Shweta revealed that she had earlier seen the same
man assaulting a lady but at that
time she couldn’t gather the courage and this incident haunted her day and night
and when she saw the same
man with a knife, she knew something has to be done to end this menace and hence
acted in the manner as stated.
Is her rash conduct protected under the right to private defence?
(a) Yes, her alleged rash conduct is owing to the fact of ill precedents and she
had acted in good faith in order
to prevent any future incidents. Hence, she has acted in private defence.
(b) No, the force used by her was disproportionate to the threat in hand of
swinging knife. She could have instead
used a less offensive object for protecting her person.
(c) Yes, private defence extends to the protection of not only the body of the
person exercising it but also for
the body of others and in this case, both the incidents were committed/likely to be
committed.
(d) No, despite the fact of the alleged incident of assault against the lady there
was no apparent/ immediate threat
on Shweta in the present scenario to entitle her to exercise the right to private
defence.
97. It was not the first time when SarojiniChowk had reported such hooliganism. It
seemed like an everyday news
of the shopping hub to be infested with such nefarious hooligans. One such day, Ms.
Sunaina saw a Stranger
trying to rob an old lady trying to snatch her purse and trying to rob her at
gunpoint. She gathered all the courage
and picked a cane lying nearby and hit the stranger with all her power making the
stranger fall instantly and
losing consciousness with blood oozing out of his head. Can Ms. Sunaina claim the
defence of private defence?
(a) Yes, the right of private defence can be used to protect the person of not only
oneself but also others in order
to tackle any impending threat or harm and hence Ms. Sunaina can claim the said
defence.
(b) No, the incident of blood oozing out of the perpetrator's head indicate that
that the force used against him
was disproportionate and hence she would not be entitled to the defence.
(c) Yes, since Sunaina was located near the incident, there was a high possibility
of being a victim of similar
attack on her after the robber had snatched the purse of the old lady and in order
to prevent such a harm to
herself, it was necessary to nip in the bud.
(d) No, the right to private defence as the name suggests ‘private' is for the
protection of one's own body and not
meant for the protection of others.
98. It was found by the investigating authority that the police headquarters were
situated at four blocks across the
place of incidence and would have taken only 5-7 minutes to report to them instead
of Sunaina herself taking the
law in her hands. Would she be entitled to the right of private defence?
(a) No, since the aid from the State machinery was available at her perusal, she
could have utilized that instead
of using any force on the perpetrator and therefore she is not entitled to any
right of private defence.
(b) Yes, the threat was immediate and the police headquarters cannot said to be
situated at a place to provide
instant resolve to the present crisis and hence the right to private defence would
still be available to Sunaina.
(c) No, since the police headquarters were situated nearby it is apparent that she
could have easily shouted in
order to invoke attention of the people and also likely that of the police instead
of taking law in her own
hands.
(d) Yes, right to private defence can be used to protect not only the body of
oneself but also of others.
99. Assuming the gun which the stranger was carrying was a counterfeit and the old
lady being unaware about the
fact was carrying a paper cutter and stabbed the stranger with that and ran away
with as fast as possible. Can the
old lady claim the right to private defence?
(a) No, in order to claim the right to private defence the threat must not only be
immediate but also real and in
this case the gun being used was itself a counterfeit and therefore no right of
private defence can be claimed
by the old lady.
(b) Yes, the old lady was unaware of the fact whether the gun was a counterfeit or
not and hence the threat or
harm that may have been caused to her or her property were immediate for her and
this entitling her to the
right to private defence.
(c) Yes, since the state machinery was not readily available at her disposal, she
was correct and entitled to have
exercised the right to private defence.
(d) No, despite being unaware of the genuineness of the gun she used force that was
disproportionate to the harm
being posed to her which was mostly constrained towards the taking away of the
purse.

. Page 25 of 36
100. After two days, it was revealed that the stranger had died due to excessive
bleeding as medical assistance was
delayed to him. Can the old lady’s claim of the right to private defence hold good
in the Court of law?
(a) No, since the act caused the death of the perpetrator it is apparent that the
force being used by the old lady
was disproportionate to the harm actually caused and hence her claim of private
defence shall not hold good.
(b) Yes, the death of the perpetrator is due to the negligence of the medical staff
in causing the delay and hence
no liability can be attributed to the old lady.
(c) No, the present factual matrix does not involve any instances of apprehension
of death, grievous hurt, assault
with the intention of committing rape, and others where the causing of the death in
the exercise of the right
of private defence can be justified.
(d) Yes, since the harm was done keeping the fact that the gun was real and hence
the likely outcome of his
death was not intended or caused due to the direct conduct of the old lady.
Passage(Q.101-Q.105): When a tort is committed, meaning that a defendant’s actions
interfered with the
plaintiff’s person or property, a plaintiff’s consent will excuse the defendant of
the wrongdoing. Although a
defendant’s conduct may be considered immoral, or harmful, if the plaintiff allows
these interferences to occur,
then the defendant is not considered to have committed a tort.
Consent occurs when a plaintiff displays a willingness to participate in the
defendant’s conduct. This consent
can be express or implied. Consent may not always excuse a defendant of liability.
Sometimes consent is
ineffective under certain conditions. If the plaintiff lacks the capacity to
consent, is coerced into consenting, or
consents under false pretenses, the consent is not valid as a defence to the tort.
This incapacity must interfere with the plaintiff’s ability to weigh the benefits
and consequences of the
defendant’s suggested conduct. A person suffering from bouts of insanity cannot be
expected to be able to give
proper consent and anyone who takes advantage of that fact and puts him under any
risk of injury shall not have
the defence of consent.
(Extracted with requisite revisions and edits from ‘General Defenses In Torts’
athttps://www.lawctopus.com/academike/general-defenses-in-torts/)
101. Manoj, was an avid fan of cricket and with India-Pakistan match being
organized in his city, it enough for fans
like him to pay for premium ticket just to get hold of the match. On the day of the
match, while Manoj was
watching the match, a ball by one of the players Babur went straight to his
shoulder and injuring him in the
process. He brought an action against Babur for having made the shot at the place
where the audience were
sitting which he could have avoided, him being a professional player. Will Babur be
liable for professional
misconduct?
(a) Yes, Babur would be liable for professional negligence as he is expected to
maintain a least level of due
diligence before exercising a shot like this.
(b) Yes, there is no implied or expressed consent given by Manoj for sustaining any
such harm which he
withstood due to the negligence of a professional player and there was no
willingness to partake in such a
conduct of Babur.
(c) No, Manoj has implicitly consented to any such harm which occurred during the
match including the injury
sustained by him during the match by buying the tickets of the stadium.
(d) No, Manoj is expected to take due care and diligence in order to divert any
harm and hence Babur would not
be liable for the harm.
102. It was argued by Manoj that the stadium authorities have promised him while
giving the tickets (kindly note that
he brought premium tickets to watch the match) that it is the highest point of the
stadium and there has not been
a single instance when the ball would have reached that high ever in the history of
the cricketing tournament.
However, the history changed as Manoj was injured with the shot. He claims that
there were false pretenses
made by the stadium authorities and therefore, they would be liable. Is his
argument correct?
(a) Yes, the consent was given by Manoj on the false pretense of safety by the
stadium authorities and therefore
the consent was not a valid consent and Manoj would be entitled to compensation.
(b) No, there was no false pretenses by the stadium authorities as their promise
was based on factual accuracy
and Manoj has consented to such harm caused during the match.

. Page 26 of 36
(c) Yes, Manoj has not consented to take part in the defendant’s conduct and
therefore Babur cannot be excused
from any liability.
(d) No, there is an implied consent by Manoj for any harm that might or likely to
occur in the stadium and no
liability can be attributed to any person whatsoever.
103. Pakistan lost the match despite the tremendous performance by Babur.
Infuriated by the performance of his team
and in the dressing room, he got into a fight with a fellow teammate and struck him
with the stump, thereby
injuring him in the process. Babur claimed that he cannot be made liable as all the
teammates have consented to
any injury caused and resulted during the match. Is his argument correct?
(a) No, there was no consent given by the teammates to withstand any harm incurred
due to any physical
altercation and the consent is given only for harm caused due to acts which are
part of the game.
(b) Yes, any altercation the subject matter of which is pertaining the sports in
question will excuse Babur of any
liability and therefore his argument is correct.
(c) No, the altercation has occurred not within the stadium but inside the dressing
room and therefore no
protection from liability can be ascribed to Babur and he would be liable for his
acts.
(d) Yes, such altercations are a part of sportsman spirit and should be taken in
the same sense as law does not
taken into account of trivial matters.
104. During the match, the roof of a part of the stadium collapsed which led to
injuring a substantial number of
audience who came to watch the match. They decided to bring an action against the
stadium authorities for not
maintaining the stadium in proper condition which led to such a casualty. To this,
the stadium authorities pleaded
that the audience has consented to any such harm by entering the premises and
therefore there would be no
liability on the stadium authorities. Is the argument by the authorities correct?
(a) Yes, there is no negligence that can be directly attributed to the stadium
authorities and hence they would
not be liable for the injury sustained by the audience.
(b) No, there is no consent given by the audience present in the stadium for the
harm caused due to the negligence
of the stadium authorities and hence stadium authorities can be made liable.
(c) Yes, implied consent has been given by the audience by entering the premises
and using the facilities in the
stadium and hence they are required to take their own personal level of care and
due diligence.
(d) No, once a person enters inside the premises of the stadium, the stadium
authorities would be responsible for
any harm that the spectators might incur inside the stadium despite having
consented implicitly to the same.
105. Due to the intense match and the subsequent loss of Pakistan, an enraged fan
threw his gloves and the
autographed ball over to opponent team supporter and a fight ensued between them
injuring both of them in the
process. Has the opponent team supporter consented impliedly to sustaining of harm
by entering into the
premises?
(a) Yes, any reasonable person can reasonably foresee such altercation due to the
high intensity drama involved
during the match of India-Pakistan and therefore the opponent team supporter
impliedly consented to
sustaining of such harm.
(b) Yes, since both the supporters are lawful possessor of the stadium tickets both
of them cannot be made liable
for the physical altercation as they were liable for contributory negligence.
(c) No, the consent has to be in a written format although such consent can be in
the form of terms and conditions
printed at the backside of the ticket.
(d) No, there is no consent given by the opponent team supporter for such a harm
caused due to any physical
altercation as it is not a part of the usual course of the match.

. Page 27 of 36
SECTION - D: LOGICAL REASONING
Directions(Q.106-Q.135): Read the passages and answer the questions.
Passage(Q.106-Q.110): It is unfortunate that yet another important bill with far-
reaching implications for our
democracy has been passed by both Houses of Parliament without a discussion. The
Election Laws (Amendment)
Bill 2021, which provides for the linking of electoral cards with the Aadhaar
number, was passed by a voice vote
amidst protests from the Opposition. Some Opposition members demanded that it be
referred to a parliamentary
panel. Since the provisions of the bill related to serious matters like
citizenship, voting rights and personal
identity, there should have been a detailed debate on its provisions. The
government seemed to be in a hurry and
the demand for referring the bill to a parliamentary committee was rejected. Law
Minister Kiren Rijiju, who
introduced the bill, told the Lok Sabha that some proposals which are part of the
bill had already been
recommended by the Standing Committee of Law and Personnel. This was an inadequate
reply and there was a
need for more clarity on the matter.
The bill raises some genuine and serious concerns. A voter ID card is given only to
citizens; an Aadhaar number
is only a tool to verify identity, and is available to all residents, including
non-citizens. A linkage between the
two is unnatural and problematic. It has been pointed out that it can potentially
lead to a situation in which non#citizens may get the voting right or bona fide
citizens may be denied the right. Some critics have mentioned the
possibility of disenfranchisement of voters with the help of the new law. Electoral
registration officers can
demand the Aadhaar number of those who seek a voter ID and even of those who
already have the card. Refusal
can mean denial of the voting right, though the linking is now said to be optional.
But if such linking was optional,
then why make a law?
The bill also faces criticism on the ground that it violates the Supreme Court
judgement in the Puttaswamy case
which has made privacy a fundamental right. Linking the voter ID with Aadhaar
violates the right to privacy as
defined by the judgement, and the competence of the government to legislate on the
matter has been questioned.
The Supreme Court judgement on Aadhaar had limited its use to welfare schemes,
whereas voting is a
constitutional right. The use of Aadhaar in matters related to citizenship might go
beyond its scope as prescribed
by the court.
106. Which of the following is the main idea of the passage?
(a) A bill with consequences for our democracy should not have been passed without
debate and discussion.
(b) Despite progressive aspects, linking electoral rolls with Aadhaar raises
apprehensions.
(c) Attempts to long pending electoral reforms are welcome, however the bill should
not have been passed
without debate.
(d) Since linking Aadhaar with a Voter ID card is optional, there was no reason to
make a law.
107. Out of the following, which one provides strength to the author’s arguments?
(a) The members of the Standing Committee of Law and Personnel are merely the
puppets of the ruling party.
(b) A detailed investigation and analysis of each of the provisions of the
Electoral Bill was done before passing
it.
(c) The government called for the public discussion of the Aadhar Bill but the
general public did not pay heed.
(d) All of the above provides strength to the author’s arguments.
108. It is evident from the passage that the author feels:
(a) The electoral reforms bill fails to not impress as far as passing the muster of
the SC judgement is concerned.
(b) There has already been a judgement in the Indian jurisdiction that deals with
the right to property.
(c) There does not exist any ambiguity as far as the provisions of the electoral
reform bill is concerned.
(d) There is a legitimate concern that the line between voluntary and mandatory
linking may be breached.

. Page 28 of 36
109. Which option weakens the author’s arguments the most?
(a) Reforms in the electoral matters of India were long overdue.
(b) Linking Aadhaar with Voter ID will help weed out fraudulent voters.
(c) Linking Aadhaar with Voter ID will increase the risk of multiple enrollments.
(d) The possibility of disenfranchisement of voters with the help of the new law is
a possibility
110. Which of the following must be true for the author’s arguments in the passage
to hold ground?
(a) There is no need to make a law on something that is optional.
(b) If right to property was not a fundamental right, then this bill would have
little room for criticism.
(c) If a bill passes the scrutiny of the parliamentary panel, then all the concerns
of the bill can be ignored.
(d) Linking the voter ID with Aadhaar gives the government transparency into
nefarious activities by criminal
elements.
Passage(Q.111-Q.115): Results of the exercise carried out by the J&K Delimitation
Commission are in the
public domain even though the proposals haven’t been published in the gazette.
Jammu is to get six more seats
in the proposed assembly and Kashmir will be given one more. It will lead to a
split of 43 seats for Jammu and
47 to Kashmir in a 90-member assembly. Kashmir’s politicians, rarely on the same
page, are unanimous that
their region got a raw deal. They still have time to convince the Commission. The
J&K Delimitation
Commission is a unique body, owing its existence to the provisions of the Jammu and
Kashmir Reorganisation
Act, 2019. The Act set the terms for the Commission. In addition to population,
other aspects such as physical
features, communication and conveniences were to be considered in the delimitation
exercise. To put the
extraordinary situation of J&K in perspective, it’s important to consider Andhra
Pradesh’s bifurcation. The 2014
AP reorganisation law states that the assembly seats in the successor states will
be increased. That exercise has
been put off till the first census after 2026, which is when there can be a
national delimitation exercise.
Chief election commissioner Sushil Chandra, a member of the J&K Delimitation
Commission, observed that it’s
not a mathematical exercise, but must reflect political aspirations. That’s the nub
of the matter. It’s the reason
the Commission now needs to engage politicians from Kashmir. Advancing the
delimitation exercise for just
J&K serves no purpose if it cripples a nascent political process. Reactions to this
exercise should also make us
rethink the forthcoming national delimitation exercise. The Constitution initially
linked Lok Sabha seats allotted
to a state to its population. Subsequent amendments froze the population reference
point to the 1971 Census till
the first census after 2026. It’s meant to protect states that took a lead in
lowering fertility rates.
A delimitation exercise represents a shift in political power. As population is the
main basis of seat allocation,
an inherent problem is it dilutes the political clout of states and regions that
have done well in realising the goal
of population control. India has over time overcome fissiparous tendencies that
weakened national integrity.
Given this backdrop, a national delimitation exercise is best avoided because of
the danger of unintended
consequences. And in J&K, the Commission should find a way to address the fears of
Kashmir before it publishes
its outcome.
111. Out of the following, which one is not one of the supporting ideas of the
passage?
(a) J&K Delimitation Commission should address fears of Kashmiri people.
(b) India should rethink about the 2026 national delimitation plan.
(c) Indian national integrity has weakened to a new low.
(d) Advancing the delimitation exercise for J&K should be mindful of budding and
new political process.
112. With which of the following is the author most likely to agree with?
(a) The intention of the recent Delimitation exercise in J&K may be to fuel
communal disharmony.
(b) The ruling party is setting double standards for the north and south of India.
(c) The Delimitation Exercise in J&K is a unique exercise for several reasons.
(d) Giving population importance in a delimitation exercise is beneficial to
national integrity.

. Page 29 of 36
113. It can be inferred from the passage that:
(a) Delimitation exercise is one of the features of a democratic society.
(b) Imposing heavy handed measures in Kashmir is not sustainable.
(c) A national delimitation exercise may weaken national integrity.
(d) The govt. should focus on guaranteeing jobs instead of making illicit moves.
114. Consider the following statement: “The J&K Delimitation Commission has invited
the views of the public and
political parties until the end of next month.” What role does this statement play
towards the passage?
(a) It weakens the author’s arguments.
(b) The author is likely to welcome this move.
(c) It does not affect the author’s arguments.
(d) It is an inference that can be drawn from the author’s arguments.
115. The author cites Andhra Pradesh’s bifurcation to
(a) Drawn an analogy between Jammu and Kashmir and Andhra Pradesh’s situation.
(b) To highlight the discrepancy in exercise of the J&K Delimitation Commission.
(c) To mention the contradiction between Jammu and Kashmir and Andhra Pradesh’s
situation.
(d) To challenge the decision taken by the J&K Delimitation Commission.
Passage(Q.116-Q.120): The sweeping victory of Gabriel Boric, in the presidential
run-off election, is a
testimony to how Chile has changed. One of the bastions of free market orthodoxy in
Latin America, Chile has
been rocked by anti-inequality protests for more than two years. Mr. Boric, one of
the protest leaders who
promised to “bury neoliberalism” during his campaign, built an alliance of social
democrats and communists
that took on the Republican Party’s José Kast. While Mr. Boric promised to build a
more equitable society in
one of the most unequal countries, Mr. Kast, a defender of the military regime,
positioned himself as a candidate
of the economic status quo and blamed migrants, terrorists and narco-traffickers
for Chile’s agonies. The
pollsters had predicted a narrow lead for Mr. Boric, but his 12-point triumph over
Mr. Kast by securing about
56% of the votes marked the strongest political comeback of the Chilean left, which
had undergone systemic
persecution during the U.S.-backed military dictatorship of General Augusto
Pinochet. Gen. Pinochet, who
toppled the socialist President, Salvador Allende, in 1973, laid the foundations of
Chile’s neoliberal state. His
regime fell in 1990, but the state apparatus he built survived, including the
Constitution. Now, when an elected
Constituent Assembly is writing a new Constitution for Chile, bringing an end to
Pinochet’s influence, the
country will have the most left-wing President since Allende.
Mr. Boric has promised to fight the “privilege of the few” and tackle poverty and
inequality. He has opposed
big-ticket mining projects as part of his climate protection plan. He wants to
raise taxes by 8% of GDP, abolish
the unpopular private pension funds, shorten the working week to 40 hours, raise
the minimum wage and create
a universal health-care system. These promises were the crux of the progressive
electoral platform he built. Mr.
Boric, who will be sworn in next month, faces the daunting challenge of walking the
talk. His legislation agenda
would be met with strong opposition in Parliament: the Senate is evenly split
between the right and the left, and
in the 155-member Chamber of Deputies, his coalition has only 37 MPs. Sagging
growth and high inflation
would limit the new government’s spending agenda. If he goes ahead with the plan to
raise taxes on the
corporations, abolish private pensions and waive off student debt, the private
capital and the old political
establishment would revolt, like what happened in the other left-ruled states in
Latin America. Mr. Boric’s
victory has put wind in the sails of Chile’s left-wing politics, but he should be
ready for a storm as he seeks to
take on the Pinochet consensus.
116. The main idea of the passage is best represented by:
(a) Mr. Boric’s victory is significant and so will be the challenges in his tenure.
(b) The victory of the new President elect in Chile is a victory for Chileans.
(c) Left wing politics is the best form of politics for a country like Chile.
(d) Mr. Boric with an easy victory would face massive challenges going ahead.

. Page 30 of 36
117. Which of the following is true according to the passage?
(a) Mr. Boric would fulfill most of the promises that he made during the polls.
(b) Mr. Boric would not be able to walk the talk during his tenure.
(c) The believers of the Pinochet consensus advocate left-wing politics.
(d) None of the above
118. Which among the following can be inferred from the passage above?
(a) Mr. Boric is contradicting himself by banning big mining projects if he wants a
more equal society.
(b) Through the election, it is clear that Chile wants to elect conservative
politicians.
(c) Controlling inflation would help the Boric government fulfill its promises.
(d) José Kast represented the Democratic Party.
119. The author has made a few claims in the passage. For which of the following
has the author presented a few
pieces of evidence to support his claims?
I. The Boric government would face stiff opposition in the Parliament.
II. It would be difficult for Boric to fulfill his ambitious promises.
III. The Chilean left had undergone systemic persecution under Pinochet.
(a) Both I and II (b) Both I and III (c) Both II and III (d) All I, II, and III
120. What challenges is the Boric government not likely to face in its tenure?
(a) Active opposition. (b) Soaring growth rate.
(c) Protests from the people. (d) An easy tenure.
Passage(Q.121-Q.125): According to Jewish and Catholic traditions, human beings are
the most advanced living
organisms on earth, in part because of their capacity to speak and to think
creatively. While animals such as
nonhuman primates or human-animal chimeras may exhibit human-like cognitive
capacities, they will still be
considered nonhuman animals according to both traditions. In contrast, personhood
status and rights reflect
society’s perception and understanding of whether an organism possesses human-like
cognitive capacities. Thus,
society has the right to use its moral compass to decide what animals should
possess personhood status.
Another consideration for granting personhood status is the definition of “life.”
Jewish thought maintains that
only a living organism can attain personhood. This criterion conflicts with the
decision of Saudi Arabian
government officials, who recently became the first to grant full citizenship to a
robot, the artificially intelligent
Sophia, developed by Hanson Robotics.
Biotechnologies can elicit complex ethical challenges that need to be discussed and
addressed. I believe that an
animal that has been engineered with human brain cells or human neural organoids
does not necessarily attain a
human state. Being human requires the creation of an embryo from human gametes or
being born from a human.
However, a human-animal chimera that expresses basic human-based cognitive
capacities or human-like
behaviors (that is, advanced communication skills, the ability to override basic
instincts, etc.) can be granted
personhood status reflected in legal or moral rights not normally given to animals.
For example, society can
confer these organisms with limited autonomy and prohibit their use in clinical
drug trials based on our
perception of their ability to provide consent. As with other medical ethical
dilemmas, personhood status of any
living organism must be assessed on a case-by-case basis.
121. The passage is mainly concerned with which of the following?
(a) Considerations used as moral compass for granting personhood to human-animal
chimera.
(b) Features and characteristics of human-animal chimeras.
(c) Ways in which human-animal chimeras have been granted personhood.
(d) Problems in granting personhood to the closest relatives of human beings.

. Page 31 of 36
122. Which among the following techniques has the author used to make arguments in
this passage?
(a) The author tries to bring in different perspectives about a question before
giving more credibility to a
particular perspective.
(b) The author employs arguments based on cause and effects.
(c) The author asserts arguments of different authors to strengthen his point of
view.
(d) The author brings forth various perspectives and concludes by asserting his
point of view concerning the
subject matter.
123. According to the author, which of the following is the most important
consideration before granting nonhumans
personhood?
(a) Human-like cognitive abilities. (b) Moral and legal ethics.
(c) Advanced communication skills. (d) None of the above.
124. Based on the passage, which of the following is most likely to be the
profession of the author?
(a) A professor of Biology. (b) A political scientist.
(c) Author of a science-fiction. (d) A politician
125. What is the main purpose of the author behind introducing Jewish and Catholic
traditions?
(a) The author is trying to find some points of difference between the two
traditions.
(b) The author advocates Jewish traditions to grant human-animal chimeras’
personhood.
(c) The author tries to bring in different perspectives about granting human-animal
chimeras’ personhood.
(d) The author is trying to explain how Jewish and Catholic traditions are similar
in granting something
personhood.
Passage(Q.126-Q.130): The world is familiar with the cycles of boom and bust in the
energy market. It would
be understandable, therefore, if the current surge in the price of natural gas in
Europe and its rippled impact on
the price of coal and oil is seen as just one more twist in the sector’s business
trajectory. That would be a mistake.
For, this time, there is the added dimension of its consequence on the pace, nature
and course of the “green
transition”. Were this dimension ignored, the price hike would incentivize an
increase in the production of fossil
fuels and that would run counter to not just public sentiment but also the efforts
to shift to a clean non-fossil fuel
energy system. The nub of the dilemma for governments created by this latest price
shock is to find a way to
navigate the long-term imperatives of decarbonisation and also manage the political
and social backlash from
consumers impacted by high electricity and fuel costs. COP 26 should add the
resolution of this dilemma to its
agenda. It will be a recurring issue. The price of natural gas in Europe has shot
up by approximately 600 per cent
over the past 12 months. And, in a reversal of the conventional feedback loop
wherein the price of oil would lead
to a change in the price of gas, this time it is the price of gas that has pushed
up the prices of oil and coal. This
is because as the former became increasingly unaffordable, consumers turned to the
latter. There are many
reasons for this price surge, but they distill down to the perennial influencers of
the energy market — the interplay
of demand, supply and geopolitics. On the demand side, the strongest driver has
been the global economic
recovery. Added to that are the micro factors of the drop in hydropower in Brazil
and China because of drought,
the reduction in wind power because of unfavourable wind conditions in the North
Sea and the underperformance
of nuclear reactors in Europe. On the supply side, there have been three economic
blockers and one geopolitical
bottleneck. The economic blockers were the cold wave in Texas in February this
year, which froze gas wells and
throttled the export of US LNG, the start-up of the maintenance work suspended
since 2020 on account of Covid#19. Matters have been compounded by the diversion of
US LNG cargoes destined for Europe to Asia and low
inventories.
126. According to the given passage, which of the following statements can be
attributed to the author?
(a) The price of oil and the price of gas are remotely connected.
(b) The reasons for the price surge of gas are alien to what we have seen before.
(c) This price surge of gas is unlikely to be a one-off event.
(d) The most important factor of the deflation of the price of gas is yet to be
ascertained.

. Page 32 of 36
127. Out of the following, which among the following is not one of the supporting
factors of the rise in prices of oil?
I. The global economic recovery is one of the strongest reasons for the given
event.
II. The cold wave in Texas was an enabling factor.
III. The drought in China was an important cause for the price surge.
(a) Only I (b) Only I and II (c) All I, II, and III (d) None of the above.
128. Which of the following can be validly inferred from the given passage?
(a) The current surge in the price of gas is unlikely to have been perceived as an
abnormal event.
(b) The recovery of the global economies is the most important factor in the
increase in the price of gas.
(c) The price of natural gas in Europe has shot up by approximately 600 per cent
over the past 12 months..
(d) The price of gas is inversely proportional to the price of oil.
129. Out of the following, which one is not true from the passage?
I. If the price of gas in Europe one year ago was Rs. 100 per litre, it would have
become Rs. 600 per litre.
II. Consumers have turned to natural gas since it is perceived that fossil fuels
are a major source of pollution.
III. The governments have a task cut out for them as far as managing the backlash
of the people due to the rise
in the price of gas is concerned.
(a) Only II (b) Only III (c) Only I and II (d) I, II & III
130. Which among the following is not one of the arguments of the author?
(a) The current surge in the price of gas is different from previous such events.
(b) The major contributors to this price surge are not entirely different from
previous causes.
(c) The price hike of gas would lead people to consume more fossil fuels.
(d) On the demand side, the strongest driver has been the global economic recovery.
131. Eight persons are sitting around a circular table facing inward. B is sitting
second to the right of O. U is sitting
third to the left of A. N is sitting to the immediate left of J. N is sitting
opposite to R. M is sitting to the immediate
right of B. Who is sitting second to the left of R?
(a) J (b) U (c) A (d) M
132. In the following question, two statements are given and three conclusions are
given below these statements.
Choose the option which shows the conclusions which logically follow from the given
statements, disregarding
commonly known facts.
Statements:
Only a few leashes are ropes.
Only a few ropes are cords.
Conclusions:
I. All ropes are leashes.
II. Some leashes can be cords.
III. Some ropes are not cords.
(a) Only I and II follow (b) Only II follow
(c) Only I follow (d) Only III and II follow

. Page 33 of 36
133. In the question below, two statements are given followed by conclusions: I, II
and III. You have to take the given
statements to be true even if they seem to be at variance from commonly known
facts. Read the conclusions and
decide which of the given conclusions logically follows from the given statements
disregarding commonly
known facts.
Statements:
All hair are black.
All black are eyes.
Conclusions:
I. No hair is an eye.
II. All hair are eyes.
III. Some eyes are black.
(a) Either I or II follows (b) Only II and III follows
(c) Only II follows (d) Only III follows
134. Select the letter cluster that can replace the question mark (?) in the
following series.
PROCASTINATION, PSOCASTINATIPN, PSPCASTINATJPN, PSPDASTINAUJPN, ?
(a) PSPDBSTINBUJPN (b) PSPDBSTINAUJPN
(c) PSPDBTUINATJPN (d) PSPDBTUINAUJPN
135. A boy started walking towards north and travelled 2 km before he turned right.
He then travelled 3kms further.
Then he repeated the process of taking left and travelling 3km and then taking
right and travelling 4km, two
times. What is the shortest distance between his current position and his starting
point?
(a) √117 km (b) √231 km (c) √123 km (d) √185 km
mock 25
Directions (Q.1-Q.30): Read the following passage carefully and answer the
questions that follow.
Passage(Q.1-Q.4): A rich heritage of dance comes down to us from our ancestors.
Ever since man was created,
he created the art of movement i.e., dance. Each of the world’s greatest
civilizations has produced its own dances.
In India, too, dance is rooted in the pre-historic past, for long before dance grew
to be a complex art, human
beings swayed, turned, stamped rhythmically and took the first steps of motion. It
was an expression of joy. We
notice that the movements that started as only a joyous expression blossomed into
specific dance patterns, which
again got codified into rules of performance, thus emerging as classical dances.
Classical dances when brought on proscenium faced a major change in repertoire as
well as in presentation. But
modern proscenium also inspired dance productions that do not adhere to any rule.
They incorporate any kind
of movement. The popular term, ‘Creative Dance’ is often coined for such dance
productions. Its birth is in the
conscience of the choreographer’s mind.
The inspiration for the choreographer is the socio-economic and political
conditions that affect a sensible person,
thereby provoking him to react to social issues and express through the medium of
dance. Therefore, we can say
Creative Dance is a product of consciousness and it bears the identity of the
choreographer or the conceiver.
In the world of dance, we come across another classification i.e., modern dance.
The thoughts about modern
dance emerged in America just after the First World War. Three Americans, Isadora
Duncan, Louis Fuller and
Ruth St Denis revolted against the prevalent tradition and spurred to seek new
forms of dancing. Each considered
himself/herself an artist rather than entertainer. In Europe we get modern dancers
like Lavan, Wigman, Jack
Delacroize, Herald Krutzberg and in Asian continent we have Taksumi Hijikata, Min
Tanaka, Yan Mei Ki and
our Udayshankar.
Turning to our country India, we observe that this kind of outburst took place
after the Second World War. The
socio-economic and political crisis that occurred during the freedom movement
became the inspiration to new
dance thoughts here.
The IPTA came up with its theatre productions and dance dramas bearing distinct
stamp of contemporary
relevance. Two torchbearers of the new dance movement in India were Rabindranath
Tagore and Udayshankar.
Rabindranath in Shantineketan tried to make a synthesis of the classical dances and
folk dances of India. He did
not hesitate to use even dances from Java, Bali and German modern dances in his
compositions. On the other
hand, Udayshankar tried to blend the western dance presentation with eastern dance
concept. Both Rabindranath
and Udayshankar were conscious of the contemporary relevance of their choreographed
themes. Thus, we find
Tagore writing Chandalika and UdayShankar choreographing ‘Man and Machine’ and
‘Rhythm of Life’.
Many are of the opinion that any creation in dance is a ‘Creative Dance’. But to
fall into the Creative Dance
category, the choreographer must be able to offer new or original thoughts with
contemporary relevance. The
artistic director is free to borrow from any dance form, classical, folk, Indian or
foreign. The movements should
dissolve in the body and bring out a new dimension and the blending should
transform the whole into a new
vision.
1. According to the author, Creative Dance
(A) seems to be flexible enough to include any type of dance movements in it.
(B) do not adhere to any rules
(C) is the product of choreographer’s creativity.
(D) sometimes may reflect the prevailing socio-economic and political conditions.
(a) A, B and C are true. (b) A and C are true.
(c) A, C and D are true (d) All of the above

. Page 3 of 36
2. Identify the correct statements regarding Uday Shankar.
(A) His themes are contemporary issues.
(B) He choreographed, among many dances, “Man and Machine”.
(C) He amalgamated the western dance presentation with eastern dance concept.
(a) only A and C. (b) only A and B.
(c) A, B and C. (d) only B and C
3. Who can be credited with the synthesis of classical and folk dances?
(a) Duncan, Fuller and St. Denis. (b) Rabindranath Tagore.
(c) Uday Shankar. (d) Lavan, Min Tanaka.
4. As understood from the passage, classical dances
(A) need to follow certain rules of performance.
(B) cannot be termed as expressions of joy.
(C) form a different type of ritual dance.
(D) are unique to each economy.
(a) only A is true. (b) only C is true.
(c) B and D are true. (d) A and D are true.
Passage(Q.5-Q.9): Born out of the forces of globalization, India’s IT sector is
undertaking some globalization
of its own. In search of a new source of rapid growth, the country’s outsourcing
giants are aggressively expanding
beyond their usual stomping grounds into the developing world, setting up
programming centers, chasing new
clients, and hiring local talents.
Through geographic diversification, Indian companies hope to regain some momentum
after the recession. This
shift is being driven by a global economy in which the US is No longer the
undisputed engine of growth. India’s
IT powers rose to prominence largely on the decision made by the American
executives, who were quick to
capitalize on the cost-saving to be gained by outsourcing non-core operations, such
as system programming and
call centers, to specialists overseas.
Revenues in India’s sector surged from $4 billion in 1998 to $59 billion in the
last fiscal year. But with the
recession, NASSCOM forecasts that the growth of India’s export of IT and other
business services to the US and
Europe will drop at most 7% in the current fiscal year, down from 16% last year and
29% in 2007-08.
Factors other than the crisis are driving India’s IT firms into the emerging world.
Although the US still accounts
for 60% of the export revenue of India’s IT sector, the emerging market is growing
faster.
Tapping these more dynamic economies won’t be easy, however. The goal of Indian IT
firms for the last 30
years has been to woo clients outside India and transfer as much of the actual work
as possible back home, where
lower wages for highly skilled programmers allowed them to offer significant cost
savings. With costs in other
emerging economies equally low, Indian firms can’t compete on price alone.
To adapt, Indian companies which are relatively unknown in these emerging nations
are establishing major local
operations around the world, in the process of hiring thousands of locals. Cultural
conflicts arise at times while
training new recruits. In addition, IT firms also have to work extra hard to woo
business from emerging-market
companies still unaccustomed to the concept of outsourcing. If successful, the
future of India’s outsourcing sector
could prove to be as bright as its past.

. Page 4 of 36
5. What is the author trying to convey through the phrase “India’s IT sector is
undertaking some globalization of
its own”?
(a) India has usurped America’s position as the leader in IT.
(b) The Indian IT sector is competing with other emerging nations for American
business.
(c) The Indian IT sector is considering outsourcing to developing economies.
(d) Indian IT firms are engaging in expanding their presence internationally.
6. Which of the following factors(s) made the services offered by the Indian IT
attractive to the US?
(A) Indian IT companies had expertise in the not so common core operation.
(B) The US lacked the necessary infrastructure and personnel to handle core
operations.
(C) The inability of other equally cost-efficient developing countries to comply
with their strict policies.
(a) None (b) Only (A) (c) Only (A) and (B) (d) Only (C)
7. Which of the following has caused Indian IT firms to change the way they conduct
business in developing
countries?
(a) The volume of work being awarded cannot be handled by Indian firms.
(b) The demands of these markets are different from those of traditional
operations.
(c) Wages demanded by local workers are far higher than what they pay their Indian
employees.
(d) Stringent laws which are not conducive to outsourcing.
8. What do the NASCCOM statistics about Indian IT export indicate?
(a) Drops in demand for IT services by Europe and the US is at most 7 percent.
(b) Indian IT firms charge exorbitantly for their services.
(c) Indian has lost out to other emerging IT hubs.
(d) The Indian IT sector should undergo restructuring.
9. What is the opposite meaning of “woo” which is given in the passage?
(a) Court (b) seek attention (c) elude (d) presence
Passage (Q10.-Q.14): Bovine tissue used in the manufacturing of artificial heart
valves can now be kept alive
for as long as 45 years, a city surgeon has demonstrated, thanks to a special
chemical solution he has formulated.
This breakthrough effectively extends the life of each of these critical devices by
an equivalent number of years.
In contrast, the mechanical heart valves currently in use to meet an annual demand
of around 20,000 have a life
of only 15 years. They also cost twice as much.
Dr Dhani Ram Baruah, who runs an artificial heart valve manufacturing facility at
SEEPZ here, presented his
findings at the 39th Annual Conference of the Indian Association of Cardiovascular
and Thoracic Surgeons at
Hyderabad in February. Studies over three years have shown that the preservative
solution in which Baruah cures
the fresh tissue obtained from animal hearts, keeps the tissue alive for periods as
long as 45 years. The
breakthrough is in the fact that, thanks to swift tissue harvesting and curing
techniques, the mitochondria in the
cells, the tiny powerhouses that generate the energy to fuel cellular activity, can
be kept alive and functioning.
"The advantage of these valves is obvious," Dr Baruah says. "Apart from lasting
forever, the risk of the tissue
tearing or cracking because of the strain of continuous opening and closing during
every heart beat is minimised."
The solution he uses retards the calcification, the process of gradual hardening of
the tissue, if not suppressing it
completely. Thus, the durability of the device is also increased.
In India, where there is a very high incidence of rheumatic heart disease, these
valves hold out hope especially
since mechanical valves are usually not suitable for people younger than 50. Valves
manufactured in Dr Baruah's
laboratories here have been exported to Hong Kong and Sweden over the last three
years. "We are very satisfied
with the performance of the device and its added advantage of requiring no
anticoagulation." Dr Ho says. "There

. Page 5 of 36
is a tremendous significance for many developing countries in which the use of
anti-coagulants after valve
replacement has been less than ideal if not problematic." Anti-coagulants are
essential for patients with
mechanical valves.
An improved version of the valves used in Hong Kong was sent to the Karolinska
Institute in Sweden last year,
Dr Baruah says. The durability of this batch of 90 "third-generation" valves, as
Baruah calls them, has been
enormously improved because they are without stents, the circular frames with which
earlier valves were fitted.
The mechanical valves are not very efficient, either. The majority of them close
too early in the heart's pumping
cycle, leaving behind as much as half the blood that would have been pumped out by
a more efficient device.
And studies show that 65 percent of patients on mechanical valves don't survive
beyond 13 years. The remainder
usually have problems like paralysis and loss of memory.
Government officials whom Dr Baruah has met so far have all expressed interest in
his work - his tale is almost
parable for these times of export and economic liberalisation - but so far nothing
concrete has come of it. No
other heart valve researchers in the country have reached commercial production. In
fact, the material being used
for the biological valves in institutes in the south is porcine tissue, which does
not have sufficient strength to
stand up to the strain. Still, the battle is to the strong. And Baruah is best
placed to ride the winds of liberalisation.
10. What is/are the advantage(s) of an artificial heart valve in comparison to the
mechanical heart valve?
I. The artificial heart valve extends human life by a number of 50 years.
II. The artificial heart valve has proved to be less expensive than the mechanical
heart valve.
III. The artificial heart valve has high rate of rejection in comparison to the
mechanical heart valve.
(a) Only II (b) Only III (c) I and III (d) I and II
11. In the light of Dr Baruah's studies, which of the following is/are true?
I. The tearing or cracking of the tissue in artificial heart valves due to the
strain of continuous opening and
closing is minimised.
II. The preservative solution in which the fresh tissue is cured is obtained from
the hearts of pigs.
III. This particular artificial heart valve can be successfully used only on
patients under the age of 50.
(a) Only I (b) Only II (c) I and II (d) I, II and III
12. According to the passage, what are the drawbacks of the mechanical heart valve?
I. It has a short life span.
II. It involves high risk of paralysis and memory loss.
III. Succeeds only in 13 years old patients
(a) Only III (b) I and II (c) II and III (d) I, II and III
13. What is meaning of the given word “parable”?
(a) short stories with a moral (b) prose
(c) sonnets (d) poetry
14. Which of the following is a suitable title for the above passage?
(a) The Winds of Liberalisation - not yet Strong enough.
(b) Breakthrough in Bovine Tissue.
(c) Take Heart - Bovine help is on the way.
(d) The Superiority of the Artificial Valve.

. Page 6 of 36
Passage(Q.15-Q.19): “Not religion, but religious dogma” leads to conflict,
“Religion should graduate into
spirituality”. These two epigrammatic statements of President Abdul Kalam are worth
pondering over. It has
been said that more blood has been shed over the cause of religion than any other
cause. This is confirmed by
what is happening today. It is a paradox that religion is at once a cohesive and a
divisive force. If religion with
its potential for good has turned into a destructive force, it is but a perversion
of religion.
Religion in its deepest sense has laid the foundation of moral order. It has
extended the bounds of human
sympathy and underscored the values of humanity, charity in the sense of love and
tolerance, faith in a
transcendental divine order, respect for the sanctity of all forms of life and
observance of the decencies of life.
All values advocated by religion are threatened by greed, violence, exploitation,
competitive religion,
consumerism, commercialisation and such forces. It looks as if religion has become
its own enemy. Every sphere
of human activity from the individual and domestic to the international level is
torn with chaos and conflict.
The spirit of scepticism generated by science has resulted in an erosion of values.
The world is broken into
narrow domestic walls. “We have grasped the mystery of the atom and rejected the
Sermon on the Mount. The
world has achieved brilliance without wisdom and power without conscience. We know
more about war than
about peace, more about killing than about living” (General Omar S. Bradley).
We boast about our civilisation. There is nothing particularly civilised in
travelling by plane or living in
airconditioned comfort. Speed, quantity, sophisticated lifestyle and bank balance
do not make for civilisation.
Human beings have been described as the elder brothers of the animals and the
younger brothers of the gods.
Civilization fulfils itself when they are elevated from animality to spirituality.
Religion and spirituality have their
role in this transforming and regenerating process. Their role should be redefined
and reformulated in the modern
context.
Religions are many, but religion is one. There is no relative superiority of one
religion over another. One should
go beyond denominational religion to understand religion. What the world needs is a
fellowship of faiths on a
common march towards a common goal. “Man must evolve for all human conflicts a
method which rejects
revenge, aggression and retaliation. The foundation of such a method is love.”
These words of Martin Luther
King sums up the essence of religion.
15. Which of the following opinions regarding ‘religion’ and ‘spirituality’ has
been quoted by the author?
(a) Religion and spirituality are synonymous words.
(b) Religion should not be equated with spirituality.
(c) Religion is practised by observing spirituality.
(d) Religion should lead a person to spirituality.
16. What can be understood from the author’s statement that one should go beyond
denominational religion to
understand religion?
(a) Being attached to a particular religion will not help in understanding
religion.
(b) If one does not understand his own religion, one cannot understand any
religion.
(c) One should try to understand religion by analysing all the religions.
(d) spirituality prevents one from understanding religion.
17. According to the author, what is the paradox about religion?
(a) It is sometimes uniting and at other times dividing.
(b) It starts as a unifying force but ends as a divisive force.
(c) It unites and divides people at different points of time in different ways.
(d) It happens to be both uniting and dividing at the same time.

. Page 7 of 36
18. The author calls religion its own enemy…
(a) because it has led people to destroy themselves.
(b) because has divided people by greed, violence, exploitation, competitive
religion, consumerism,
commercialisation and such forces
(c) because religion is infested with negative elements.
(d) because people who practice religion have only become sinners.
19. What is the meaning of epigrammatic used in the 1st paragraph?
(a) long stories
(b) novellas
(c) a short saying that expresses an idea in a clever way.
(d) enigma
Passage(Q.20-Q.24): Infant mortality, the number of babies that die before their
first birthday for every 1,000
live births, is a powerful indicator of the quality of life in a community. High
infant mortality has been linked to
poor maternal health and inter-generational poverty in families.
Across the world, there is a link between government per-capita spending on health
and infant mortality. This
partly explains why India has historically had one of the highest infant mortality
rates, despite its rapid GDP
growth, at 1.4% of GDP in 2014. According to the WHO, India’s spending lags that of
Sub-Saharan Africa. To
some extent, public spending also explains variations within the States. A study
using data from 1983-84 to
2011- 12 showed that per-capita spending on health in Indian States was the biggest
predictor of infant mortality,
followed by female literacy and urbanisation. Other studies, however, have found
smaller impacts of public
spending. This is because public health expenditure is only a part of the story,
say experts. While it does help in
setting up healthcare infrastructure, the willingness of people to access this
infrastructure is crucial. On this
count, southern States like Kerala and Tamil Nadu outperform, because of widespread
literacy. “People here are
aware of the danger signs of infant illnesses and seek help promptly,” says Ravi
Prakash Upadhyay, a public
health researcher at Delhi’s Centre for Health Research and Development, Society
for Applied Studies. Literacy
and greater female autonomy in these States, also seen in Manipur, mean women give
birth at a later age and
wait longer between births
This cuts infant mortality, but isn’t affected by public health spending. Even the
transport infrastructure of a
State can have a role in reducing infant mortality, because the longer people take
to reach hospitals when their
child is sick, the higher the risk of death.
Around two-thirds of infant deaths happen in babies less than four weeks old. A
Lancet paper published as part
of the Million Death Study in 2011 found the biggest drivers of neo-natal deaths in
India to be premature birth
and low birth weight, neo-natal infections, and asphyxia and trauma. Asphyxia is a
medical condition in which
insufficient or no oxygen and carbon dioxide are exchanged on ventilator basis, and
trauma can be caused by an
injury or shock. For these two hospitalizisation is crucial. After the first month,
diarrhoea and pneumonia become
the leading causes of death. While low birth weight can be prevented if the mother
is well-nourished, diarrhoea
can be avoided by exclusively breast-feeding the child in the first six months. “If
the mother resorts to infant
formula, the water used to prepare it is often contaminated. This can lead to
diarrhoeal deaths,” says Dr.
Upadhyay.
Despite Kerala’s low infant mortality, the hilly regions of districts like Kasargod
have historically lagged behind.
“The settlements here are not very big for the government to provide healthcare
infrastructure,” explains Mr.
Arokiasamy. Similarly, the Vidarbha region of Maharashtra has suffered, while there
are differences in eastern
and western Uttar Pradesh. Such intra-State variations can be as big as inter-State
variations, but do not get as
much attention from policymakers.

. Page 8 of 36
20. According to the passage, which of the following is unlikely to be a cause of
infant mortality?
(a) The general public’s reluctance to avail of available health facilities.
(b) Insufficient funds at the disposal of the State for health care.
(c) Delaying conception to a relatively later age.
(d) Little gap between births.
21. What function does the second paragraph perform?
(a) It deters the government to increase its per capita spending on health.
(b) It delineates the reasons why infant death rate is high in some States.
(c) It asserts that spread of literacy can reduce infant mortality.
(d) It badmouth the efforts put in by Kerala, Tamil Nadu and Manipur in reducing
infant mortality.
22. After reading the passage, which of the following would you recommend to reduce
asphyxia and trauma?
A. Ensure that women go to hospital to deliver their babies.
B. Provide nutritious food to pregnant women.
C. Immunize the new born.
D. Give cash incentive for delivery in hospitals.
(a) (A) and (D) (b) (B) and (D) (c) (A) and (C) (d) (A) and (B)
23. What is the title of the passage?
(a) GDP and health care (b) Healthcare crises World Wide problem
(c) Infant mortality in India (d) literacy and neo natal infections.
24. According to the passage, states having very low infant mortality should pay
attention to which of the
following?
(a) Encouraging hospital deliveries.
(b) Building more health care infrastructure.
(c) Launching a campaign to increase gap between births.
(d) Providing health care facilities at thinly populated and inaccessible areas.
Passage (Q.25-Q.30): The Chinese were among the first foreigners to do trade with
the island of Sumatra. Six
hundred years ago, villages would have been but infinitesimal specks in an
inconceivably vast and sublime rain
forest. In 1416, a Chinese report on Sumatra noted that “There are in the forests
immense quantities of wild
rhinoceroses, which the king lets catch by men.” The Rhinos, the author goes on to
explain, would be sent to
China as “tribute” to the emperor. Later, in the midst of compiling a list of
agricultural products and minerals to
be found in Sumatra, the author’s mind drifts back to something even more valuable,
and he abruptly ends his
list by reminding his Chinese reader: “Besides, there are Rhinoceroses.”
There are still rhinoceroses in Sumatra today, perhaps as few as 30, and they are
still hunted. According to that
15th century Chinese account, as well as the testimony of early European visitors
and explorers, rhinoceroses
once swarmed on the island. Yet their population has all been but wiped out. What
happened? The answer is
pretty straightforward: They were hunted and slaughtered for their horns. Many of
those horns were sent to
China, where they were used in so-called traditional medicine. The trend continues
to this day, and it will
continue until the last Rhino has been hunted out of Sumatra’s protected areas.
A similar story can be told about another species of Asian megafauna: Tigers. In
the 19th century vast swaths of
Sumatra and Java were “infested” with tigers, which would pick off mailmen and
laborers wandering around the
backroads and the edges of plantations and carry them off into the jungle for
dinner.. In the 1930s, French Captain
Henry Baudesson, overseeing the construction of a railway in southern Vietnam,
wrote that the tiger’s
“supremacy has hardly yet been seriously challenged” in the hinterlands, adding,
“In Indo-China the tiger is the
hunter and man the hunted. It is estimated that there were 100,000 tigers in the
wild in 1900; today their numbers

. Page 9 of 36
are down to about 3,200. The Javan tiger was declared extinct in the 1980s; the
last tiger of China’s Yunnan
province was snared and eaten by a poacher in 2009.
Asian elephants have been ruthlessly slaughtered for their ivory across Asia for
years, and their numbers are
declining. Habitat destruction is probably the key driver for this megafauna’s
demise, and their African cousins
have it worse due to the fact that their tusks are much bigger and therefore more
valuable, but Asian elephants
have it bad too. In the eastern Thai provinces of Rayong and Changseao, local media
report almost daily on
human-elephant conflicts, and in Myanmar wild elephants are being killed for their
skin, which is used to make
clothing, jewelry, and, of course, traditional medicine. These products are sold in
the border town of Mong La
and virtually all of the customers are from China. Elephants were once found as far
north as the outskirts of
Beijing, but Chinese farmers and elephants don’t get along, and the pachyderms were
pushed south or simply
killed. In Mark Elvin’s brilliant The Retreat of the Elephants, mention is made of
Chinese recipes for barbecued
elephant trunk.
25. Why the author drifted reader’s mind by mentioning the phrase “Besides, there
are Rhinoceroses.”?
(a) The author wanted to end his report abruptly.
(b) The author wanted to give the description of island of Sumatra.
(c) The author wanted to divert the reader’s mind towards the value that Rhinoceros
held above everything else.
(d) The author wanted to humour the readers.
26. What causes the increase in level of hunting of Rhinoceros on the island of
Sumatra?
(a) The poaching of the Rhinoceros is the latest trend in the North Eastern
countries.
(b) The decorative material used by the rhino’s horns is the cause of their hunting
and poaching.
(c) The horns of Rhinoceroses are used as medicine in China.
(d) The parts of the rhino are imported and exported for reasonably high price.
27. In the context, what does ‘infested’ suggest with reference to the tigers?
(a) Hunting and poaching of tigers for their skin, whiskers and fur.
(b) The danger for tigers of being extinct.
(c) Area of Sumatra and Java covered with Tigers.
(d) The damage caused by the Tigers of the Sumatra and Java.
28. Which of the following is the appropriate title of the passage?
(a) Imbalance of ecosystem. (b) Clearing of forests.
(c) China’s penchant for Wildlife. (d) All of these
29. How can it be inferred that African Elephants’ condition was worse than the
Asian elephants?
(I) African elephants are more poached for they are more dangerous.
(II) The clothing jewelry and medicine made of African elephants’ skin are more
valuable.
(III) The tusks of African elephants are more valuable and bigger than Asian
elephants.
(a) Only (I) (b) Only (III) (c) Both (I) and (II) (d) All are correct
30. The term pachyderm used in the passage refers to:
(a) It refers to Mark Elvin
(b) It refers to Chinese Farmers
(c) It refers to Elephants
(d) If refers to Barbecued elephant trunk

Directions (Q.66 – Q.105): Read the comprehensions carefully and answer the
questions based on it.
Passage (Q.66-Q.71): Setting aside the conviction of a person in a murder case, the
Supreme Court recently
held that merely because a person revealed the hideout of the victim to the
murderous mob, he cannot be
presumed to share the common object of the unlawful assembly.
A bench comprising Justices Sanjay Kishan Kaul and MM Sundresh struck a word of
caution by observing that
the Courts must guard against the tendency of convicting mere passive onlookers of
the crime using the medium
of Section 149 of the Indian Penal Code for sharing the common object of the
unlawful assembly.
The Court observed: "The Court must guard against the possibility of convicting
mere passive onlookers who
did not share the common object of the unlawful assembly. There must be reasonable
direct or indirect
circumstances which lend assurance to the prosecution case that they shared common
object of the unlawful
assembly. Not only should the members be part of the unlawful assembly but should
share the common object
at all stages. This has to be based on the conduct of the members and the behaviour
at or near the scene of the
offence, the motive for the crime, the arms carried by them and such other relevant
considerations". An assembly
of five or more persons is designated an “unlawful assembly”, if the common object
is to do an unlawful act
under section 141 of IPC. For deciding the liability of a person one should be the
member of such assembly
followed by whether that person share common object or not.
https://enalsar.informaticsglobal.com:2278/top-stories/section-149-ipc-person-
disclosed-victims-hideout-not#part-unlawful-assembly-common-object-murder-case-
supreme-court-186819 Live Law dated 2/12/21
66. Some people were protesting outside the gate of parliament against few bills
which were passed by the parliament
in the ongoing winter session. X was also furious by the new bills and decided to
throw a grenade in the
parliament campus to showcase the angst of the people protesting outside. X asked
his friend Y to assist him in
purchasing a grenade from dark web, and Y obliged. The following day, Y also
dropped X outside the gate of
the parliament and thereafter went to his college. Soon, X executed his plan, but
was caught by the police.
Determine Y’s liability under section 149 of the IPC.
(a) He is not liable because he was not a member of the unlawful assembly.
(b) He is not liable because he did not share a common objective with anyone.
(c) He is liable because he satisfies all the ingredients under section 149 of IPC.
(d) He is liable because he contributed directly in the commission of the crime.
67. Suppose in the previous question, as soon as Y dropped off X, his bike got
punctured and thus he stayed with X
until a mechanic arrived. With the rest of the facts remaining the same, would your
answer to the previous
question change?
(a) Yes, he is liable under section 149 of IPC.
(b) No, he is liable under section 149 of IPC.
(c) Yes, he is not liable under section 149 of IPC.
(d) No, he is not liable under section 149 of IPC.
68. Z and his friends failed in the final semester of their college. They were
furious and demanded a re-check of
their answer sheets, but the administration denied. Agitated by the same they
gathered outside the residence of
the vice-chancellor and started abusing him, but it all went into vain. Suddenly Z
got the idea to burn the house
of the VC; Z got the petrol and lighter but as soon as he prepared to throw it over
the gate, police arrived and
arrested him. Police wants to book Z and his friends under section 149 of IPC.
Determine their liability.
(a) They are not liable since no offence has been committed by the group.
(b) They are liable because they gathered outside the VC’s residence illegally and
hurled abused at him.
(c) They are not liable since the ingredients under section 149 are not satisfied.
(d) They are liable because there was a common object and intention amongst the
group to burn down the VC’s
house.

. Page 17 of 36
69. Suppose in the previous question, the police failed to arrive on time and the
VC’s house was burnt to ground,
with severely injuring the VC. With the rest of the facts remaining the same, would
your answer to the previous
question change?
(a) Yes, the group will be liable under section 149 of the IPC.
(b) No, the group will be liable under section 149 of the IPC.
(c) Yes, the group will not be liable under section 149 of the IPC.
(d) No, the group will not be liable under section 149 of the IPC.
70. P took some money from Q but failed to return them even after several warnings;
infuriated by the same Q along
with few other friends, went to P’s house to threaten him. When Q reached P’s
house, everyone threatened and
abused P but Q also slapped him multiple times. P filed a police complaint against
Q and his friends, and the
police wants to book them under section 149 of IPC. Determine their liability.
(a) They are liable because they have satisfied all the ingredients of section 149.
(b) They are not liable because they have not satisfied all the ingredients of
section 149.
(c) They are liable because they all shared a common objective and acted in the
furtherance of the same.
(d) They are not liable because there was no common objective to assault P.
71. Suppose in the previous question, when Q was slapping P, another friend R also
got incited and kicked P
repeatedly and so did his other friends. With the rest of the facts remaining the
same, would your answer to the
previous question change?
(a) Yes, the group will be liable under section 149 of the IPC.
(b) No, the group will be liable under section 149 of the IPC.
(c) Yes, the group will not be liable under section 149 of the IPC.
(d) No, the group will not be liable under section 149 of the IPC.
Passage (Q.72-Q.77): The Supreme Court has observed that once the prosecution
establishes the existence of
three ingredients forming part of "thirdly" in Section 300 of the Indian Penal
Code, it is irrelevant whether there
was an intention on the accused part to cause death.
"It does not matter that there was no intention even to cause the injury of a kind
that is sufficient to cause death
in the ordinary course of nature. Even the knowledge that an act of that kind is
likely to cause death is not
necessary to attract "thirdly".
Quoting from Virsa Singh case, the Supreme Court mentioned the ingredients of
Section 300 IPC "thirdly" as
follows: "To put it shortly, the prosecution must prove the following facts before
it can bring a case under S.
300, "Thirdly";
First, it must establish, quite objectively, that a bodily injury is present;
secondly, the nature of the injury must
be proved; these are purely objective investigations. Thirdly, it must be proved
that there was an intention to
inflict that particular bodily injury, that is to say, that it was not accidental
or unintentional, or that some other
kind of injury was intended. Once these three elements are proved to be present,
the enquiry proceeds further
and, fourthly, it must be proved that the injury of the type just described made up
of the three elements set out
above is sufficient to cause death in the ordinary course of nature. This part of
the enquiry is purely objective
and inferential and has nothing to do with the intention of the offender"
https://enalsar.informaticsglobal.com:2278/top-stories/supreme-court-section-300-
ipc-intention-to-cause#death-immaterial-prosecution-proves-ingredients-thirdly-
186671 Live Law dated 01/12/21
72. X wanted to kill his neighbour Y; accordingly, he prepared a cake with some
poison in it and served it to Y. Y,
an aficionado of bakery products, happily eats it, only to find himself later with
an upset stomach. When he
discovers from a mutual friend that X had mixed poison in the cake he was served,
he files a complaint under
section 300 of IPC. Determine X’s liability in the light of the above facts.
(a) X is not liable because Y did not die due to X’s actions.
(b) X is not liable because there is no bodily injury to begin with.
(c) X is liable because he had the clear intention to kill Y.
(d) X is liable because he had the knowledge about his actions.

. Page 18 of 36
73. Suppose in the previous question, instead of having upset stomach, Y developed
carcinogenic ulcers in the
stomach. With the rest of the facts remaining the same, would your answer to the
previous question change?
(a) Yes, section 300 will be attracted.
(b) No, section 300 will be attracted.
(c) Yes, section 300 will not be attracted.
(d) No, section 300 will not be attracted.
74. P and Q were twins but due to some complications during their birth, Q was
rendered with intellectual disability.
When they were still young, Q stole two pistols from his father’s drawer and forced
P for a duel. P unwillingly
took part in the duel but unfortunately Q shot him at point blank range causing his
death and seeing his brother
dead Q started crying. Determine Q’s liability under section 300 of IPC.
(a) Q is liable because he satisfies all the ingredients of section 300.
(b) Q is not liable because he can take the defence of unsound mind.
(c) Q is liable because he shot Q at point blank range, hence he had the knowledge
of what he was doing.
(d) Q is not liable because he does not satisfy the required ingredients of section
300.
75. Suppose in the previous question, instead of being intellectually disabled, Q
was physically disabled. With the
rest of the facts remaining the same, would your answer to the previous question
change?
(a) Yes, Q is liable under section 300 of IPC.
(b) No, Q is liable under section 300 of IPC.
(c) Yes, Q is not liable under section 300 of IPC.
(d) No, Q is not liable under section 300 of IPC.
76. Y and Z were friends but one day they started fighting over some issue, so much
so that they were ready to kill
each other; in the heat of the moment, Y grabbed a pencil and stabbed in Z’s
stomach. The pencil was sharp
hence it pierced the skin of Z, but there was not much bleeding. Few days later Z
realized that he developed
sepsis due to graphite of the pencil penetrating his inner layer of skin and within
a span of 3 days, he died.
Determine Y’s liability under section 300 of IPC.
(a) Y is liable because it was due to his actions that Z died.
(b) Y is not liable because he does not satisfy all the ingredients of section 300.
(c) Y is liable because he had the intention to kill Z and acted in the furtherance
of the same.
(d) Y is not liable because, Z’s death was unforeseen and an accident.
77. Suppose in the previous question, instead of a sharp pencil, Y used a rusted
yet sharp knife. Few days later Z
realized that he developed sepsis due to the rust of the knife penetrating his
inner layer of skin and within a span
of 4 days, he died. With the rest of the facts remaining the same, would your
answer to the previous question
change?
(a) Yes, Y is liable under section 300 of IPC.
(b) No, Y is liable under section 300 of IPC.
(c) Yes, Y is not liable under section 300 of IPC.
(d) No, Y is not liable under section 300 of IPC.

. Page 19 of 36
Passage (Q.78-Q.81): The Gauhati High Court recently observed that the police
cannot use their uniform as a
shield when any allegation of unlawful conduct at the police station is raised
against them. The mandate to obtain
sanction from government under Section 197 of the Code of Criminal Procedure (CrPC)
for taking cognizance
of offences against police personnel by court, applies only if the alleged act is
reasonably connected to the
discharge of official duty, the Court reiterated.
"Only because they are in the police station with their uniform, they cannot take
the shield the uniform to protect
their unlawful conduct," the Court made it clear. In case of an act of policeman or
any other public servant not
connected with the official duty, there can be no question of sanction, single-
judge Justice Rumi Kumari
Phukan ruled.
However, if the act alleged against a policeman is reasonably connected to
discharge of his official duty, it does
not matter, if the policeman has exceeded the scope of his power and/or beyond the
four corners of the law, the
Court added. "Supreme Court has unanimously clarified that in the criminal
misconduct and misdemeanour on
the part of a public servant, not to be treated as an act of discharge of his
official duties. The learned trial court
in its order has rightly placed reliance upon the decision of Rajib Ranjan and Ors.
v. R. Vijaykumar, wherein it
has been held that while discharging his official duty if a public servant enters
into criminal conspiracy or
indulges in criminal misconduct, same cannot be treated as an act of discharge of
official duties," the High Court
observed. The requirement of sanction under Section 197 arises only if the act if
connected to the discharge of
official duties not moral duty, the Court emphasised.
https://enalsar.informaticsglobal.com:2276/news/section-197-crpc-police-cannot-use-
their-uniform-shield#unlawful-conduct-gauhati-high-court Bar and Bench dated
03/12/21
78. X was a police officer and was on holiday with his friends. One day when he was
sitting in the park with his
family, he saw that few men were trying to molest a girl; following his impulses
and instinct he picked up a rod
nearby and hit one of the men in the head which caused his death, only to realize
later that, the entire scene was
fake and was being enacted for a movie scene. Determine if X can claim protection
under section 197 of CrPC.
(a) X cannot claim protection because the committed act was unrelated to his
discharge of official duties.
(b) X cannot claim protection because he exceeded his scope of power.
(c) X can claim protection because he had a genuine reason to believe that the girl
is actually being molested.
(d) X cannot claim protection because his actions were not proportionate to the
crime he witnessed.
79. Suppose in the previous question, it was not a shoot for some movie, but the
molestation was real in its truest
sense. With the rest of the facts remaining the same, would your answer to the
previous question change?
(a) Yes, X can claim protection under section 197 of CrPC.
(b) No, X can claim protection under section 197 of CrPC.
(c) Yes, X cannot claim protection under section 197 of CrPC.
(d) No, X cannot claim protection under section 197 of CrPC.
80. Y was a police officer and was assigned to catch a wanted criminal alive in
Mumbai with no casualties; but when
Y was chasing that criminal, for the safety of himself and his fellow mates, he had
to shoot his driver in the leg
to stop him them escaping from car, but unfortunately the bullet hit the driver’s
spine and he died on the spot.
Determine if Y can claim protection under section 197 of CrPC.
(a) Y cannot claim protection because he exceeded the powers given to him.
(b) Y can claim protection because his act was in accordance of the discharge of
official duties.
(c) Y cannot claim protection because he disobeyed the orders that no casualties
should be reported.
(d) Y can claim protection because he had no control over the events, and he did
not have the intention to kill
the driver.

. Page 20 of 36
81. Suppose in the previous question, Z was the police officer who was assigned the
task and not Y, to catch the
criminal and for his assistance he called his friend Y, who was also a police
officer. With the rest of the facts
remaining the same, would your answer to the previous question change?
(a) Yes, Y can claim protection under section 197 of CrPC.
(b) No, Y can claim protection under section 197 of CrPC.
(c) Yes, Y cannot claim protection under section 197 of CrPC.
(d) No, Y cannot claim protection under section 197 of CrPC.
Passage (Q.82-Q.86): The Supreme Court observed that mere failure to avoid the
collision by taking some
extraordinary precaution does not in itself constitute negligence.
To establish contributory negligence, some act or omission, which materially
contributed to the accident or the
damage, should be attributed to the person against whom it is alleged, the bench
comprising Justices Hemant
Gupta and V. Rama subramanian observed.
"13. Therefore, the entire reasoning of the High Court on Issue No.1 is riddled
with inherent contradictions. To
establish contributory negligence, some act or omission, which materially
contributed to the accident or the
damage, should be attributed to the person against whom it is alleged. In Pramod
kumar Rasik bhai Jhaveri vs.
Karmasey Kunvargi Tak and Others this Court quoted a decision of the High Court of
Australia in Astley v.
Austrust Ltd. , toy up hold that "…where, by his negligence, one party places
another in a situation of danger,
which compels that other to act quickly in order to extricate himself, it does not
amount to contributory
negligence, if that other acts in a way which, with the benefit of hindsight is
shown not to have been the best
way out of the difficulty". Contributory negligence is the ignorance of due care on
the part of the plaintiff to
avoid the consequences of the defendant’s negligence. In fact, the statement of law
in Swadling v. Cooper , that
"though mere failure to avoid the collision by taking some ordinary precaution,
does in itself constitute
negligence…", was also quoted with approval by this Court."
[Extracted from, https://www.livelaw.in/top-stories/supreme-court-contributory-
negligence-k-anusha-shriram#ll-2021-sc-571-183676]
82. Mr. Popi is truck driver who used to deliver dairy product of Rohan dairy to
all nearby cities by 12’o clock. One
day due to some mechanical issue in the lorry Mr. Popi got late for his deliveries
so he was rushing and his speed
of lorry was more than permitted, while speeding he was continuous giving horn to
other to clear his path
however in his hurry he collided with Mr. sunny’s car who failed to took his car
aside from his path and drives
in wrong lane. Mr. popi claimed the defense contributory negligence Decide?
(a) Mr. popi can claim contributory negligence because even after his continuous
horns sunny did not take his
car aside of the path.
(b) Mr. popi can’t claim contributory negligence.
(c) Mr. popi cannot claim a contributory negligence because mere failure to avoid
colliation by taking
extraordinary precaution does not amounts to negligence.
(d) There was no negligence on part of sunny.
83. The car in which the husband of Rohi, Mr. Ronit was travelling, dashed against
a lorry that was going in
front, when the driver of the lorry Mr. jolly allegedly stopped it all of a sudden
without any signal or
indicator. Mr. Ronit suffered serious injuries and died on the spot. Mr. Jolly
claimed defense of contributory
negligence claiming that Mr. Ronit could have avoided the incident by turning to
other side. Decide
(a) Mr. ronit has contributed to the accident as he could have easily avoided the
accident by turning to other
side.
(b) Mr. ronit has not liable to the accident at all.
(c) Mr. ronit has contributed to the accident as he has the opportunity to avoid
accident but he did not acted
reasonably.
(d) Mr. Ronit is not liable for contributory negligence.

. Page 21 of 36
84. Mr. A who was driving a car without headlights at a galloping speed on a
highway in a night and saw Mr. B
driving his car in the wrong side from a long distance but ignored and when the car
came close there is no
opportunity left to avoid the accident and both the car collided. Decide?
(a) Mr. A is liable as he could have avoided the accident, had he taken the
precaution and thus has contributed
to the accident.
(b) Mr. A is not liable at all as Mr. B was negligent on his part.
(c) Mr. A is liable as he did not act reasonably.
(d) Mr, A has not contributed to the negligence.
85. Ronald Patterson, parked his truck illegally in the middle of the road as he
needed to have his vehicle towed.
When confronted by the unexpected obstruction on the road, Mr. Donald attempted
unsuccessfully to avoid
colliding with Mr. Patterson’s vehicle. Patterson sued for the injuries he
sustained as a result of the accident.
(a) Mr. Donald is liable for negligence as failed to take the reasonable care while
driving his car
(b) Mr. Donald is not liable for negligence as tried to avoid the collation by
taking extraordinary precaution
(c) Mr. Donald is neither liable for negligence nor contributed to the accident.
(d) Mr. Donald is liable for negligence.
86. Assertion (A): mere failure to avoid the accident by taking some ordinary
precaution does not in itself constitute
negligence.
Reason (R): to constitute contributory negligence there must be an act or omission
contributing to the incident.
(a) Both (A) and (R) are correct
(b) (A) is correct but (R) is incorrect
(c) Both (A) and (R) are incorrect
(d) (A) is incorrect but (R) is correct
Passage (Q.87-Q.91): The Kerala High Court recently held that as per the provisions
of the Prevention of
Corruption Act, 1988, actions of public servants which may be contrary to
departmental norms do not amount
to criminal misconduct unless it was done with dishonest intention.
"Dishonest intention is the gist of the offence under Section 13(1)(d) of the Act.
Mere conduct and action of a
public servant, without dishonest intention but contrary to departmental norms, do
not amount to criminal
misconduct," the judgment stated.
As per said section, a public servant is said to commit the offence of criminal
misconduct if he -
(i) by corrupt or illegal means, obtains for himself or for any other person any
valuable thing or pecuniary
advantage; or
(ii) by abusing his position as a public servant, obtains for himself or for any
other person any advantage; or
(iii) while holding office as a public servant, obtains for any person any valuable
thing or pecuniary advantage
without any public interest.
The Court held that dishonest intention is the gist of the offence under Section
13(1)(d) of the Act.
"Mere violation of procedure in the appointment does not lead to an inference that
the Members of the Managing
Committee had a dishonest intention in making such appointment especially when the
appointment was ratified
by the competent authority," the Court opined.
On an inspection of the facts and evidence of the case, the Court found that
dishonest intention cannot be
established, and it, therefore, allowed the appeals and quashed the charge sheets
filed against the petitioners.
Source https://www.barandbench.com/news/prevention-of-corruption-act-actions-
contrary-to-norms-but#without-dishonest-intention-not-criminal-misconduct-kerala-
high-court

. Page 22 of 36
87. Rudradev was the junior secretary to the Minister of Railways. He had planned
to purchase a car recently. In
furtherance of the same, he decided to visit the nearest car dealership and enquire
about the cars there. As soon
as the salesman there saw his ID being one of a bureaucrat, his behaviour changed,
and he became friendlier by
a great margin, enabling Rudradev to get a great deal on the car he wanted to buy.
As soon as he was given a
favourable option, he took it. Would this amount to criminal misconduct?
(a) Yes, as he should've realized, the behaviour of the salesman changed due to his
position.
(b) No, as Rudradev did not intend for any favouritism to occur.
(c) No, as Rudradev did not possess any dishonest intent.
(d) Yes, as being a junior secretary, he was sure to be given special treatment.
88. Rahul was the son of the Minister of Textiles and was involved in a drug case
wherein he was being investigated
by the police. Upon getting to know of the same, his father, Raj Balwant, called up
the inspector in charge and
told him to 'handle the matter with discretion, hearing this, and fearing for his
job, he dropped the case and let
Rahul go. Would this matter be considered as being criminal misconduct under the
conditions above?
(a) No, as Raj did not specify any criminal intent above.
(b) No, as no malicious intent is apparent above.
(c) Yes, as a minister calling up a policeman is an apparent sign of criminal
intent.
(d) Yes, as being the child of a minister, Rahul was bound to get off the hook
eventually.
89. Ex-PM, Mr Sarvodaya, had fallen on hard times, which made him get by in tough
times by resorting to a lot of
different tactics. However, there was one specific place that used to cater to him
in a very special manner, under
the belief that he was still an important government official. Making use of this,
he used to exploit his luck there,
until one day, when the owner got to know of this scam that he was pulling and
threw him out, suing him for
criminal misconduct by a public servant. Is this action sustainable?
(a) Yes, as the ex-PM was wrongly taking benefits of his positions.
(b) Yes, as being a public servant, he was not entitled to any special treatment by
any individual.
(c) No, as he was not a public servant anymore.
(d) No, as he did not ask for any special treatment on his own.
90. After winning the PM elections, the new Candidate-elect, Shri Namo Shah, was
tasked with appointing his
cabinet for his term. To the dismay of the opposition, the cabinet that he
appointed was full of his own party's
people, and no other person was appointed. This was brought up in the Parliament as
being inviolate of the
provisions above since , the opposition was stripped of their representation. Would
Namo be held responsible?
(a) No, as the Parliament is not a valid authority for him to be punished by.
(b) No, as the cabinet has been duly appointed.
(c) No, as none of the above conditions are met.
(d) No, as Namo is the duly elected PM, and his word is final.
91. One of the cabinet ministers, Dr Raivardhan Harsh, the minister for health in
the central government, was having
his son being treated by the finest hospital in the country, all at his
expenditure. However, he was noticing some
delays and dismissive behaviour by the doctors toward his son's treatment, which
led him to believe that he might
not be getting proper treatment. He decided to call the doctor, from the PM office,
in order to make him realize
how serious he was for the treatment to take place. As soon as the call got placed
and received by the doctor, he
started the surgery for his child, and the same was completed in no time, with the
child on his way to recovery.
Would this amount to disorderly conduct according to the above conditions?
(a) Yes, as he obtained an unfair pecuniary advantage here.
(b) Yes, as he exploited his position for personal gain here.
(c) No, as he did not use his own position to exploit the doctor.
(d) No, as he merely placed the call and did nothing to further gain any unlawful
advantage.

. Page 23 of 36
Passage (Q.92-Q.97): The Kerala High Court recently held that a son-in-law cannot
claim any legal right over
his father-in-law's property unless explicit promises are made.
The High Court observed that the father-in-law is paying tax on the property and
building and has been residing
in the same. The Court also found it difficult to hold that the defendant is a
member of the family.
"The defendant is the son-in-law of the plaintiff. It is rather shameful for him to
plead that he had been
adopted as a member of the family, subsequent to the marriage with the plaintiff's
daughter."
"The rightful owner(father-in-law) filed a suit for injunction restraining him from
entering into the property.
The residence of the defendant (the appellant son-in-law), if any, in the plaint
schedule building is only
permissive in nature. The defendant cannot contend that he is in legal possession
of the suit property or the
building”, the order stated.
The son-in-law had approached the High Court against the orders of lower courts
which had granted a permanent
injunction to the father-in-law, interdicting the son-in-law from trespassing into
the plaint schedule property or
interfering with his peaceful possession and enjoyment of the said property.
The question considered by the Court was whether a son-in-law has any legal right
over his father-in-law’s
property and building.
Moreover, since the father-in-law had been paying taxes and residing in the
building and had only filed the suit
of permanent injunction against his son-in-law and not his own daughter, the Court
found it difficult to hold that
the son-in-law has been adopted as a member of the family.
Source: https://www.barandbench.com/news/son-in-law-does-not-have-any-legal-right-
over-father-in-laws#property-kerala-high-court
92. Rana was the son in law of the Duggabattis, a famed caste wherein he had
married their eldest daughter.
Whenever he used to visit them, he was always reminded to feel at home and was
often told that it was his home
as well. After ten happy years of marriage, his wife, Sylvia, had passed away,
leaving him alone. Fearing this
feeling of being alone, he went to this in-laws' place. To his dismay, he was not
let in and was told to go away
since he reminded them of their daughter, and they were still grieving. Could Rana
lay a claim to the place in
the matter above?
(a) Yes, as he was often told that the place was his as well.
(b) No, as mere affectionate words cannot be held as promises.
(c) No, as he was the son-in-law and was welcome there only at their behest.
(d) Yes, as he was a part-owner of the property via his wife.
93. In the above case, had the father in law enticed Rana into marrying his
daughter by showing off his property,
Rana only married into the family because of the same, would he have a valid claim
to the property?
(a) Yes, as the agreement for marriage was based upon the same.
(b) No, as marriage cannot be equated with a contract.
(c) No, as the object of this contract is unlawful, and thus the contract is void.
(d) None of the above.
94. In the above case, if the father-in-law explicitly made promises for always
providing Rana with shelter then,
would he then be liable to lay claim to the property?
(a) Yes, as there is an explicit and binding contract upholding his claim.
(b) No, as this is a mere social agreement that is not binding in nature.
(c) No, as the nature of the promise is affectionate and not legally binding.
(d) Yes, as the son-in-law has been permitted for the same.

. Page 24 of 36
95. Sheena had split up with her husband after 20 years of marriage, nearing 45
years of life for her. After her
divorce, she was very distraught and wanted to be with her parents in her childhood
home. After having gone
there, she started feeling much better and even decided to work around the house
and make small changes here
and there. This troubled her parents, who wanted her to live her life on her own,
independently. Upon coming to
know of this, she was very upset and decided to file for an actual share in the
property. Could she do so?
(a) No, as she has legally ceased to be a member of the family.
(b) Yes, as she is still a member of the family.
(c) No, by marrying, she implicitly disavowed her claim to any of her parents'
properties.
(d) Yes, as her claim to the property is merely permissive.
96. After falling out with his wife, Nora, Habib, the husband, decided to visit his
father-in-law, Azeem, with whom
he had excellent relations. Azeem had told Habib that he would be welcome to stay
with them anytime, for as
long as he wanted. Taking this as an indication of help from a friend, Habib
decided that he didn't want to go
back home to Nora ever again and wanted to stay with his friend. Under this belief,
he started to live with Azeem
and had resided there for more than four months, when Nora decided to apologize and
wanted Habib to come
back. Habib denied doing so, under the belief that this was his house now. Did he
have a valid claim to that
house?
(a) No, as he was only overstaying his welcome.
(b) Yes, like the way he was being treated was akin to a family member.
(c) Yes, as he was under bona fide belief that he was entitled to stay there.
(d) No, as the property in question was still his father-in-law's.
97. Shrey was the son-in-law of Dhyankar, Dhyankar was laid off from his job and
had been unemployed for about
two years now. Shrey had been helping Dhyankar with his rent and taxes for that
period, but eventually, due to
the recession even he was laid off. Luckily, Dhyankar's house was liability-free.
Thinking that his father-in-law
would return the favour, he went to his house. However, Dhyankar was in no mood to
repay his debts by allowing
Shrey to stay, so he decided to not permit him inside his house. Furious at this
behaviour, Shrey filed a suit for
partial ownership of the property. Could lay a claim for the same?
(a) Yes, since he had helped Dhyankar pay his liabilities arising out of the house.
(b) No, since mere help cannot be equated with part ownership in the property.
(c) No, as all the help provided by Shrey was in furtherance of his amicable
relationship he had no intention to
permit him to stay in the house.
(d) Yes, as he had become a family member by helping pay for the family property.
Passage (Q.98-Q.102): The right against exploitation enshrined in article 23 and 24
of the Indian constitution
guarantees human dignity and protects people from any such exploitation. Thus,
upholding the principles of
human dignity and liberty upon which the Indian constitution is based.
Clause 1 of article 23 prohibits the trafficking of human beings, beggar any
similar form of forced labour. It
also states that any contravention of this provision is punishable under concerned
law. It explicitly prohibits:
Human trafficking: this refers to the sale and purchase of human beings mostly for
the purpose of sexual slavery,
forced prostitution or forced labour.
Beggar: this is a form of forced labour which refers to forcing a person to work
for no remuneration.
Other forms of forced labour: this includes other forms of forced labour in which
the person works for a wage
less than the minimum wage. This includes bonded labour wherein a person is forced
to work to pay off his debt
for inadequate remuneration, prison labour wherein prisoners sent in for rigorous
imprisonment are forced to
work without even minimum remuneration etc. However, if a person voluntarily agrees
to do work or to
do additional work for the remuneration of a certain benefit for a return then
there is no forced labour.
Clause 2 of article 23 of the constitution states that this article does not
prevent the state to impose compulsory
services for public purposes. It also states that while doing this, the state must
not make any discrimination on
grounds of religion, race, caste, class or any of them.
Hence, article 23 has a very wide scope by ensuring that a person is not forced to
do anything involuntarily. For
instance, it forbids a land-owner to force a landless, poor laborer to render free
services. It also forbids forcing a
woman or child into prostitution.

. Page 25 of 36
98. Dharmapur, a village in India, has a long-standing tradition. It is that each
home provides the village leader with
one day of free labour on a monthly basis. This ritual was followed by all
residents of the village without
controversy. One day, a village youngster named ramu complained against this
ritual, claiming that it was
infringing on his fundamental right as guaranteed by article 23 of the Indian
constitution. Decide:
(a) It is violation of article 23 as the leader is not providing any wages or
remuneration to the
free labour provided by them and hence a form of forced labour.
(b) It is not a violation of article 23 as villagers were following this tradition
of providing free labour from time
immemorial and hence state cannot interfere in such issue.
(c) It's not a violation of article 23 as none of the villagers are protesting
against the tradition except ramu.
(d) It’s a violation of article 23 as the leader is not providing any wages or
remuneration to the
free labour provided by them and hence will come under a category of beggar as
prohibited by clause 1 of
article 23.
99. Ram works as group project manager at a famous MNC in Mumbai. He was very
hardworking employee of the
company and was always appreciated by the project leader Mr. Kapoor for the same.
To speed the process of
launch of their new product Mr. Kapoor asked ram to do overtime (forced him) for a
month
without any remuneration. Ram though resisted but do not wanted to leave the
project in between kept on doing
overtime. Ram was paid his regular salary which did not include his overtime
remuneration. When he asked for
his overtime salary, company asked him to resign. Ram filed a complaint stating
that the company should be
held accountable for indulging his employees to do forced labour.
(a) Company is liable as ram worked against his will and was also not paid any
remuneration for the overtime
he did.
(b) Company is not liable; ram should understand that he is a group project manager
and it’s the requirement of
the project so as to speed up the launch.
(c) Company is liable as article 23 prohibit any kind of human exploitation.
(d) Company is not liable as ram is not willing to work according to their schedule
and hence, they stand correct
in terms of firing ram.
100. Sudhanshu was working as a licensed porter with Howrah railway station. His
sister’s marriage was around the
corner so he was in need of some extra funds. Railway recruiter announced vacancy
for a part time laborer, who
needs to work only 2 hours a day after the official working hours. I.e., from 7pm
to 9pm. Sudhanshu got to know
of this opportunity and voluntarily agreed to a two-hour additional labour
agreement with the railway authority
for which he will be paid less remuneration. Choose the correct statement.
(a) Article 23(1) not violated as Sudhanshu voluntarily agreed to do so hence will
not come under the category
of forced labour.
(b) Article 23(1) not violated as sudhanshu is in need of some extra money and this
opportunity will help him
earn some extra pennies for his sister's marriage.
(c) Article 23(1) violated as he was paid less remuneration and will come under the
category of forced labour.
(d) Article 23(1) not violated as Sudhanshu voluntarily agreed to take this job and
not forced to do it, hence will
not come under the category of forced labour.
101. The ministries of women and children's development and education both require
data for their surveys. The
research topic was to see if a nuclear or joint family is more successful in terms
of future planning for their
children. The state government instructed government school teachers to go door to
door and gather data for the
study. All of the government teachers who took part in the survey claimed that they
were forced to do so, and
that this was a breach of article 23. Choose the correct statement.
(a) It’s a violation, as state forced them to do such survey and provided them no
remuneration for same. Hence
violation under article 23 clause 1.
(b) It’s a violation of article 23 as state cannot impose compulsory services for
public purposes.
(c) It's not a violation of article 23 as state can impose compulsory services for
public purposes under article 23
clause 2.
(d) It’s violation of article 23 state did not provide them any remuneration and
hence the govt. Teachers will fall
under the category of forced labour.

. Page 26 of 36
102. Lockdown in India has affected a huge number of transient specialists. The
absence of food and fundamental
conveniences, loss of business, dread of obscure, and absence of social help was
significant () s behind battle in
this enormous piece of populace. Because of the lock-down, more than 300 deaths
were accounted for, with
reasons going from starvation, suicides, fatigue, street and rail mishaps, police
mercilessness, and refusal of
opportune clinical consideration. In times like these an NGO names ‘maa jesa” took
in custody of all those
children who lost their parents to covid-19. But behind the curtains they started
selling those children in the name
of adoption to some other parties. NGO started transferring custody of children to
any person wanting to help
them without conducting police verification or following a proper procedure of
transferring custody of a child
in consideration of money. Ngo was booked for the offence of child trafficking. Now
choose the correct
statement.
(a) Ngo is liable for the offence of trafficking which was made explicitly
prohibited and also punishable by
concerned law for same under article 23 clause 1 of Indian constitution.
(b) Ngo is not liable as in the absence of any guardians of the orphans, it took
the custody and can pass on
the same to whomsoever they want.
(c) Ngo is liable for the offence of trafficking and should get punished for the
same under article 23 clause 1 of
Indian constitution.
(d) Ngo is liable as it indulges in an act which was made explicitly prohibited and
punishable under article 23
clause 1 of Indian constitution.
Passage (Q.103-Q.105): A "bailment" is the delivery of goods by one person to
another for some purpose, upon
a contract that they shall, when the purpose is accomplished, be returned or
otherwise disposed of according to
the directions of the person delivering them. The person delivering the goods is
called the "bailor". The person
to whom they are delivered is called the "bailee".
According to section 150 of ICA 1872, The bailor is bound to disclose to the bailee
faults in the goods bailed,
of which the bailor is aware, and which materially interfere with the use of them,
or expose the bailee to
extraordinary risks; and if he does not make such disclosure, he is responsible for
damage arising to the bailee
directly from such faults.
If the goods are bailed for hire, the bailor is responsible for damage, whether he
was or was not aware of the
existence of such faults in the goods bailed. If the goods have been hired, then
the knowledge of the fault would
not matter as it is the duty of the bailor in this case to check the goods before
giving it under bailment.
The two types of bailors according to Section 156 of Indian Contracts Act, 1872
are:
 Gratuitous:
The bailor is expected to disclose the faults in his knowledge. There is benefit of
only one party as bailee does
act without any benefit. It can be terminated by bailor at any time even though the
bailment was for a specified
time or purpose. Section 163 states that the bailor, in the case of gratuitous
bailment, has the right to demand the
goods back from the bailee before the expiration period. However, if the bailee
incurs any necessary cost in the
process, bailor shall be liable to pay.
 Non-Gratuitous:
A non-gratuitous bailor has a greater responsibility. He/she would be liable
regardless of his knowledge about
the fault. In Non-Gratuitous bailment there is occurrence of mutual benefit.
As per Section 151 of Indian Contracts Act, 1872, the bailee is entitled to take
care of the goods like a reasonable
man in the same way he would have taken care if the ownership of the goods was with
him. In Giblin v.
McMullen, the court pointed out that “a gratuitous bailee is bound to take the same
care of property entrusted to
him as a reasonable, prudent and careful man may fairly be expected to take his
property of the similar
description.” Therefore, the bailee is bound to take reasonable care whether the
bailment is gratuitous or non#gratuitous. Additionally, the obligation of a bailee
includes not only the duty to take all reasonable precautions
to obviate the risks but also the duty of taking all proper measures for the
protection of the goods when such
risks had already occurred.

. Page 27 of 36
103. Raman hired a carriage (including a pair of horses and a driver) from the
rentals Pvt. Ltd. for a specific journey.
However, during the journey, a bolt in the underpart of the carriage broke which
resulted in Raman being injured.
Rentals Pvt. Ltd. Contended that they were not aware of the default so could not
make such disclosure and hence
not liable. The court held that
(a) Rentals Pvt. Ltd is liable as it was their duty to supply a carriage as fit for
the purpose for which it is hired
as care and skill can render it.
(b) Rental Pvt. Ltd. is liable as knowledge of the fault would not matter as it is
the duty of the bailor in this case
to check the goods before giving it under bailment.
(c) Rental Pvt. Ltd. is not liable as it’s the duty of Raman to check the carriage
before hiring it.
(d) Rental Pvt. Ltd. is not liable as they had no knowledge of default in the
carriage.
104. Chandler out of gratitude lent his sports bike to joey as he was very fond of
it and always wanted the same for a
long road trip to Ladhak for 4 months. Even though the sports bike was in an
inoperable state, there were
concerns regarding its performance during a long drive. Joey incurred Rs. 10,000 on
the bike’s repair. On
returning from the tour, Joey had to handover the bike to Chandler. Joey asked for
the amount he spent on repair
to which chandler denied. Decide.
(a) Denial to pay is justified as chandler lent his bike out of gratitude and never
wanted repairs.
(b) Denial to pay is justified as bike was in an inoperable state, and it was Joey
who wanted repairs.
(c) Denial to pay not justified as Joey incurred necessary cost.
(d) Denial to pay not justified as chandler is duty bound to pay same.
105. Kanu and Manu were close friends since childhood. Manu knew that Kanu is a
trained horse rider, and she tried
to take out of habit every horse for a stride. Manu bought a new horse named Tyson,
who was famous racing
horse. Manu discovered after a week that Tyson has developed an infection in his
lower abdomen owing to which
hitting him there will make it go wild which could be risky. Kanu on seeing Tyson
asked Manu to exchange it
for the purpose of countryside ride with kanu’s horse. Manu agreed to the same.
Kanu out of habit took Tyson for
stride in the course of which he was hitting it on her lower abdomen. Horse got
wild and smashed Kanu on a big
rock resulting in hospitalization of Kanu. Kanu filed a claim for damages against
Manu. Manu contended since
Kanu exchanged the horses, cannot claim damages. Decide.
(a) Kanu will succeed as Manu failed to mention Tyson's infection.
(b) Kanu will succeed in his claim as he took the horse in exchange and not in
bailment.
(c) Kanu will not succeed as she took Tyson in exchange and not in bailment.
(d) None of the above.

. Page 28 of 36
SECTION - D: LOGICAL REASONING
Directions(Q.106-Q.135): Read the passages and answer the questions.
Passage (Q.106-Q.111): The suspension of 12 Opposition MPs from the Rajya Sabha for
the duration of the
Winter Session was unfair and unreasonable, and the reasons cited for the drastic
action are inadequate and
unconvincing. The MPs were suspended for their alleged unruly conduct on the last
day of the monsoon session
in August when marshals were called after Opposition members stormed the well of
the House during the passage
of a bill. The government proposed suspension of the members under Rule 256 of
Rules of Procedure for unruly
conduct and for lowering the dignity of the House. Chairman Venkaiah Naidu has
stuck to the decision as,
according to him, the members have shown no remorse for their conduct. Opposition
parties have condemned
the decision as undemocratic and unprecedented, and violative of rules and
procedures.
It is pointed out that Rule 256 could only be invoked in an ongoing session and
cannot be used for taking action
against members for their past conduct. There are other violations of procedure,
too. Rule 256 is not frequently
used, and it cannot be claimed that the actions of the MPs, though not right, were
so uncommon as to invite such
a drastic response. Parliament has seen much worse and more shameful scenes and
members of the ruling party
too have figured in them in the past. The Opposition has also said, and rightly so,
that the government’s action
is selective in nature, punishing some but not others.
The Opposition thinks that the government took a deliberate decision to suspend the
MPs because that would
make it easy for it to get its legislative business done in the Rajya Sabha where
it does not have a majority. If
that is true, that is bad intent and is against the basic principles and practice
of parliamentary democracy. An
important norm in parliamentary practice is that it is the government’s duty to
ensure that the business of the
House is conducted in an orderly manner. The Opposition had a real and genuine
grouse that the government
was not responsive to it and did not accept its legitimate demands for discussions
on many important issues. It
is unfortunate if the Opposition is pushed into provocation and then punished. The
attack on the Opposition is
of a piece with the attacks on criticism and dissent that is seen in politics and
in wider society. It shrinks and
diminishes democracy.
[Extracted with edits and revisions from Deccan Herald]
106. Which of the following is the correct expression of the author’s opinion as
stated in the passage?
(a) Indian democracy is in deep danger considering how the opposition is silenced
in this country.
(b) The opposition members have to be punished if they do not show any remorse for
their actions.
(c) There is no nexus between the government and the opposition in this country.
(d) The acts of the Opposition against the government were genuine and such harsh
punishment was uninvited.
107. Which of the following is the central theme of the passage?
(a) Unfairness and unreasonableness of the suspension of MPs.
(b) Effect of the suspension of MPs on Indian democracy.
(c) Importance of dissent in a parliamentary democracy.
(d) Rule 256 of Rules of Procedure and its nuanced provisions.
108. Which of the following can be inferred from the passage?
i. The suspension of MPs violates more norms than just some of the provisions of
Rules of Procedure.
ii. A calculated decision to suspend the MPs is against the principles of
democracy.
iii. If the present suspension of MPs is justified, then there have been more
shameful acts in the past that are
justified as well.
(a) Both i and ii
(b) Both i and iii
(c) Both ii and iii
(d) All i, ii and iii

. Page 29 of 36
109. Which of the following, if true, strengthens the arguments of the author in
the passage?
(a) It was a minister of the ruling government who theorized that the Opposition
holds equal say in the
Parliament.
(b) An MP was spared for the same offense keeping in mind the political situation
in Punjab.
(c) The actions of the Parliamentarians in the present case was more disgraceful
than anything in the past.
(d) Rule 256 exists to be invoked if any disgraceful act takes place in and has
been invoked in the past on many
occasions.
110. As per the passage, what can be deduced?
(a) The government action to suspend the MPs was deliberate and uncalled for.
(b) The suspension of MPs was done according to the rules and procedures.
(c) Since rule 256 is not frequently used, it should not be used to punish the
Opposition.
(d) None of the above
111. According to the author’s contention, how does he see the democracy shrinking
and diminishing?
(a) By the ruling government curbing the voice of dissent and criticism of the
opposition party by invoking rule
256.
(b) By the opposition sabotaging the workings of the parliament by creating ruckus.
(c) By the ruling government taking extreme measures of invoking rule 256,
especially when the opposition
failing for the first time.
(d) By the opposition threatening to walk out of the parliament in the midst of the
session.
Passage (Q.112-Q.116): For decades now, India’s global influence has been greater
than China’s in only one
sphere: soft power. Yet, as the former Indian foreign secretary, Nirupama Rao, has
warned, that relative strength
too is now at risk of waning, largely because of New Delhi’s own actions — or
inactions. The arrows in India’s
quiver are many: from its stature as the world’s largest democracy and its immense
diversity to the moral
authority of M.K. Gandhi’s model of successful non-violent resistance that was
adopted by Nelson Mandela,
Martin Luther King and other icons. The addition of yoga, spiritualism, delicious
cuisines and Bollywood to this
mix has led to the creation of an enviable and powerful cocktail that should be
hard to match for an authoritarian
State, which, until recently, was inward looking and uneasy with its own ancient
traditions. But as Ms Rao
pointed out, these advantages would count only if India upholds constitutional
values, including human rights
and sensitivity towards minorities.
At a time when militant far-right groups feel emboldened to issue public calls for
genocide against a religious
minority, it is vital for India to pay heed to this message. This is especially so
for the government of Prime
Minister which routinely highlights India’s rich history and significant
achievements — although the line
between myth and fact is occasionally blurred — to pitch the country as a
vishwaguru or global teacher. To be
sure, China’s own aggressive ‘wolf diplomacy’ of late is hardly conducive to
Beijing’s projection of soft power.
In Hungary, large protests against a proposed Chinese university campus have forced
a referendum. But with an
economy six times the size of India’s, and with superior technological and military
prowess, China has resources
New Delhi does not possess. The Indian Council for Cultural Relations (ICCR), the
agency primarily responsible
for the country’s soft power expansion, has 38 centres abroad. China, by contrast,
has more than 550 Confucius
Institutes spread across over 150 countries, with the largest number in America,
its bitter rival. Earlier this year,
while India had to suspend exports of Covid-19 vaccines because of a domestic
shortage caused by botched
planning, China was flooding Latin America and Africa with its vials. To compete
again in soft power, India
must reconnect with its core strengths.
[Extracted with edits and revisions from The Telegraph]

. Page 30 of 36
112. Which among the following is the central idea of the passage?
(a) The soft power of India has waned to a level that is beyond redemption.
(b) India should rework at its soft power to compete with China in this area.
(c) India and China’s global influence are equal because of India’s soft power.
(d) India and China are the global players at the moment with the USA keeping up.
113. It can be inferred from the passage that:
(a) China’s own aggressive ‘wolf diplomacy’ of late is hardly conducive to
Beijing’s projection of soft power.
(b) India does not have enough areas to work upon its soft power in the short run.
(c) Helping needy countries in crises plays a role in increasing soft power.
(d) The author’s suggestions to increase the soft power are in contrast to that of
Ms Rao.
114. By mentioning that China has more technological, economic and military
prowess, the author
(a) breaks away from the arguments made in the previous few sentences.
(b) weakens the main argument of the passage.
(c) extends his argument from the previous few sentences.
(d) gives a brief explanation of why the author thinks that India competes with
China.
115. Which of the following can be concluded from the passage?
(a) Most of Confucius Institutes of China are located in America.
(b) India should develop more ICCR centres in countries economically below it.
(c) India should walk the talk with regards to being the global teacher.
(d) India does not need to wield soft power in such harsh times.
116. As per the passage, which of the following is a challenge for India?
(a) To manifest India as a land of opportunities
(b) To restrict access to economic opportunities to its neighbours
(c) To increase global influence through soft power
(d) Both a and c are correct
Passage (Q.117-Q.121): One of the strangest things about the human mind is that it
can reason about
unreasonable things. It is possible, for example, to calculate the speed at which
the sleigh would have to travel
for Santa Claus to deliver all those gifts on Christmas Eve. It is possible to
assess the ratio of a dragon’s wings
to its body to determine if it could fly. And it is possible to decide that a yeti
is more likely to exist than a
leprechaun, even if you think that the likelihood of either of them existing is
precisely zero.
Everyone knows that “impossible” is an absolute condition. “Possible versus
impossible” is not like “tall versus
short.” Tall and short exist on a gradient, and when we adjudge the Empire State
Building taller than LeBron
James and LeBron James taller than Meryl Streep, we are reflecting facts about the
world we live in. But
possibility and impossibility are binary, and when we adjudge the yeti more
probable than the leprechaun, we
aren’t reflecting facts about the world we live in; we aren’t reflecting the world
we live in at all. So how, exactly,
are we drawing these distinctions?
In the fourth century B.C., Aristotle sat down to do some thinking about
supernatural occurrences in literature.
On the whole, he was not a fan; in his Poetics, but he did allow that, if forced to
choose, writers “should prefer
a probable impossibility to an unconvincing possibility.” There’s a reason
Aristotle addressed this advice to
writers and artists. A little more than two millennia later, a very different kind
of artist got somewhat more
specific. Although Walt Disney is best remembered today for his Magic Kingdom, his
chief contribution to the
art of animation was not his extraordinary imagination but his extraordinary
realism. “We cannot do the fantastic
things, based on the real, unless we first know the real,” In short order, the
cartoons emerging from his workshop
started exhibiting a quality that we have since come to take for granted but was
revolutionary at the time: all
those talking mice, singing lions, dancing puppets, and marching brooms began
obeying the laws of physics.

. Page 31 of 36
It was Disney, for instance, who introduced to the cartoon universe one of the
fundamental elements of the real
one: gravity. Even those of his characters who could fly could fall, and, when they
did, their knees, jowls, hair,
and clothes responded as our human ones do when we thump to the ground. Other laws
of nature applied, too.
Witches on broomsticks got buffeted by the wind. Goofy, attached by his feet to the
top of a roller-coaster track
and by his neck to the cars, didn’t just get longer as the ride started plunging
downhill; he also got skinnier,
which is to say that his volume remained constant. To Disney, these concessions to
reality were crucial to
achieving what he called, in an echo of Aristotle, the “plausible impossible.” Any
story based on “the fantastic,
the unreal, the imaginative,” he understood, needed “a foundation of fact.”
117. ‘One of the strangest things about the human mind is that it can reason about
unreasonable things.’ What can be
inferred from the statement?
(a) The human mind can apply logic only to logical things.
(b) The human mind can apply logic even to the most illogical things.
(c) The human mind can apply logic to even the most unnecessary things.
(d) The human mind can apply logic to even the most indiscriminate things.
118. According to the passage, how is ‘tall vs short’ and possibility vs
impossibility different from each other?
(a) The former is based on concrete facts, whereas the latter on a prospect.
(b) The former and latter can be adjudged based on their reality.
(c) The former is a likelihood, whereas the latter is absolute.
(d) The former accounts for reality, whereas the latter for fiction.
119. “To Disney, these concessions to reality were crucial to achieving what he
called, in an echo of Aristotle, the
“plausible impossible.” The argument of the author depends on which one of the
following assumption(s)?
I. The analogy drawn between Aristotle and Walt Disney is based on the similarity
in thoughts.
II. The ideology of Aristotle resonated in Disney’s work.
(a) Only I.
(b) Only II.
(c) Both I & II.
(d) Neither I nor II.
120. Which one of the following would strengthen the author’s conclusion?
(a) A successful readership or viewership of a fictional writer or an author
depended on its relatability with the
readers.
(b) The closer the fiction is to the existing facts, the more successful would be
the readership and the viewership.
(c) The hiatus between the fiction and fact depends upon the readers and viewers’
ability to digest the fantastic.
(d) The more absurd the laws of nature, the more acceptable is the fictional
readership or viewership.
121. If the information given in the passage is true, which of the following
closely represents Aristotle’s contention?
(a) An animation character depicted as an alien that when leaps to the sky reaches
the space.
(b) An out of body experience of going to heaven.
(c) An elf falling from the witch’s broom and crashing onto a little boy’s roof.
(d) A deer chased by the lion runs at its optimal speed.

. Page 32 of 36
Passage (Q.122-Q.126): Putting down that extra bit of cash for a logo and other
luxuries, generally speaking,
begins later in life, once one is well established. As I see it, self-indulgence,
on one’s own buck. It is one of the
few privileges of adulthood, blowing up money you have earned, as you see fit.
Youth, in any case, doesn’t need
embellishment. When you’re bursting with vitality and optimism that experience
hasn’t chipped away yet, you
look good in anything. If values have fallen in place correctly, the young have an
innocence uncorrupted by a
need for projection. It’s not important, yet, to wear clothes that make a statement
about a financial background.
That is the sad plight of middle agers, who need acquisitions to cheer themselves
up as consolation for their
unfulfilled hopes and dreams. We fool ourselves by justifying expensive purchases
as growing discernment for
the finer things of life. But it’s always about something else, ennui, or
existential dissatisfaction.
Generation Z, unfortunately, has grown up on social media that fuels a thirst for
something other than what we
have. Historically, there’s been an unbridgeable chasm between the lifestyles of
youth and middle age, but
nowadays the lines between teenage and adult lives are blurred. They are faced with
the same daily barrage of
advertisements we are, that insidiously suggest this or that product will make them
happy. Considering how
difficult it is for mature people to handle peer pressure, the younger lot are not
equipped to resist being told what
they ought to have. Brandishing the right labels has become the yardstick of self-
worth too early in life.
122. What is the assumption behind the first line of the passage?
(a) One is usually well established later in life.
(b) One does not need logos early in life.
(c) One is not well established early in life.
(d) One should spend on logos only when one is well established.
123. Which one of the following inferences can be drawn from paragraph one?
(a) The vivacity and the positivity are the embellishments of the Youth.
(b) Experience corrodes the liveliness in the youth.
(c) Experience is the embellishment of the middle-aged.
(d) Youth does not need embellishment.
124. ‘If values have fallen in place correctly, the young have an innocence
uncorrupted by a need for projection.’
Which of the following enfeebles the given statement?
(a) The projection of values in the young is a constant demand in a world which is
inherently skeptical.
(b) Innocence is a projection onto itself and needs no validation.
(c) A young can be innocent and yet may desire projection every now and then.
(d) Innocence when projected becomes corrupted and the values fall out of place.
125. According to the passage, paragraph two
(a) deepens the chasm between the generation Z and the generation Y.
(b) relies on the author’s accuracy of providing the correct information.
(c) defies earlier premise given in paragraph one
(d) sketches the challenges faced by generation Z.
126. Which of the following would be a suitable carry forward to the last lines of
the passage?
(a) Climate change activists like Greta Thunberg, briefly, took the shine off
conspicuous consumption.
(b) It’s a sermonising cliche to say that materialism and consumerism are terrible
wastes of time.
(c) At an age when exploring the world and notching up experiences should be a
priority, the focus has moved
to building a wardrobe.
(d) There’s a frustrating gap between what we think we need and what we actually
need.

. Page 33 of 36
Passage (Q.127-Q.130): Despite the economy being in a slump, 2021 has been one of
the best years for the
stock markets and for IPOs, from which close to Rs 1.20 lakh crore has been mopped
up. While most stocks
have done well post listing, some start-ups like Paytm have been big flops. This
has prompted SEBI chief Ajay
Tyagi to nudge merchant bankers to review their due diligence standards. He wants
them to better explain the
basis of the pricing of the issue, especially for the new-age companies, many of
which are loss-making. That is
a tall task given that, in many cases, there is simply no rationale for the
valuations. Financial experts have valued
these businesses way below the valuations that they have earned via the IPOs. If
retail investors are getting
carried away by the hype, they have only themselves to blame.
Even so, it is necessary to encourage start-ups to list on the Indian bourses. To
this end, SEBI is taking cognizance
of the changing corporate ownership structures, by which the traditional family-
owned constructs are giving way
to frameworks where there are several owners or partners, but no distinct owner.
For instance, the regulator
suggests a limit be imposed on the number of shares that can be put on sale by
existing shareholders—those who
own more than 20% of the pre-issue capital—by way of an Offer for Sale (OFS). The
idea seems to be to try and
prevent large chunks of equity from being offloaded in one go so that price
discovery is not hurt. While promoters
could skirt the rule by capping their holdings at 19.5%, it will limit the supply
of promoter shares coming into
the market.
Ultimately, though, such factors matter less than the intrinsic strength of the
business and the quality of the
management. The business can’t be considered to be weak simply because several
promoters want to pare their
stakes. Conversely, even if promoters hold on to their stakes, it is no guarantee
the business is in great shape. If
the promoters and the management are committed, they will not let down the minority
shareholders, no matter
what their ownership levels. In India, however, both have a chequered history.
[Extracted with edits and revisions from The Financial Express]
127. Which of the following can be correctly attributed to the SEBI? Choose your
answer based on the passage only.
(a) The Chairmanship of SEBI is not in the right hands.
(b) SEBI is a regulatory body in the Indian Stock Market.
(c) The functions of SEBI include controlling the management of a business.
(d) SEBI can intervene in government functioning.
128. Identify the idiom that best fits for the first sentence of the given passage.
(a) Every cloud has a silver lining.
(b) Out of the frying pan; into the fire.
(c) When it rains, it pours.
(d) Misery loves company.
129. It can be inferred from the passage that:
(a) The strength of the business and its management are the two factors that
exclusively control share prices.
(b) Limiting the supply of promoter shares in the market is desirable.
(c) The task given to the merchant bankers can be handled with ease.
(d) SEBI does not have the authority to taking cognizance of the changing corporate
ownership structures.
130. Why does the author believe that the promoters could cap their holdings at
19.5%?
(a) According to SEBI, no person can hold more than 20% of a company’s ownership.
(b) In this way, they would be able to limit the supply of promoter shares in the
market.
(c) The promoters would try to do so to protect the price discovery.
(d) None of the above is correct

. Page 34 of 36
Passage (Q.131-Q.133): Society’s apathy toward sleep has, in part, been caused by
the historic failure of science
to explain why we need it. Sleep remains one of the least great biological
mysteries. All of the might problem#solving methods in science - genetics,
molecular biology, and high-powered digital technology - have been
unable to unlock the stubborn vault of sleep. Minds of the most stringent kind,
including Nobel Prize-winner
Francis Crick deduced the twisted-ladder structure of DNA, famed Roman educator and
rhetorician Quintilian,
and even Sigmund Freud had all tried their hand at deciphering sleep’s enigmatic
code, all in vain.
131. The author’s central complaint in the above argument is that:
(a) society has failed to give sleep its due credit because of science’s failure in
emphasizing its importance.
(b) great scientists have not given sleep its due credit in their research and
failed to create a mark in society.
(c) people and society in general have overestimated the value of sleep.
(d) people don’t realize how much sleep is necessary.
132. Which of the following, if true, would most strongly challenge the author’s
contention in the paragraph?
(a) Lack of sleep or sleep related disorders rarely kill a patient.
(b) Research has found that accidents caused by sleep deprived drivers exceed those
caused by alcohol and drugs
combined.
(c) Many doctors across the globe have started to acknowledge that dreams play a
very important role in self
therapy.
(d) The importance of sleep is part of the cultural folklore of a majority of
communities in the world.
133. Which of the following is an assumption implicit in the argument?
(a) Sleep is an unscientific concept.
(b) Science is rarely able to handle stubborn concepts.
(c) Science sometimes fails to resolve mysterious concepts.
(d) Sleep deprivation accounts for its mysterious nature.
134. A person put some sweets in five boxes – B1, B2, B3, B4 and B5. The number of
sweets in B2 is less than that
of B3 as well as B1. The number of sweets in B1 is less than that of B4 but more
than that of B5. The number
of sweets in B3 is less than that of B5. Which of the following statement is false?
(a) B2 has the least number of sweets
(b) The number of sweets in B1 is more than the number of sweets in three boxes.
(c) The number of sweets in B3 is more than that of B1
(d) The total number of sweets in B1 and B5 is more than the total number of sweets
in B3 and B2
135. Point U is 20 m to the south of point R which is 9 m to the east of point P.
Point T is 15 m to the north of point
P which is 20 m to the east of point S. Point V is 35 m to the north of point Q
which is 21 m to the west of point
U.How far and in which direction is point Q with respect to point T?
(a) 41 m, North West
(b) 37 m, South West
(c) 29 m, South West
(d) 37 m, North East

mock 26
Directions (Q.1-Q.30): Read the following passage carefully and answer the
questions that follow.
Passage (Q.1-Q.4): Success has many fathers. No wonder, then, the paternity suits
are flying in microfinance’s
lending small amounts to help the poor pull themselves out of poverty. Thanks first
to charities and later,
international financial institutions like the World Bank; micro founder has been
shown to work. Now,
philanthropists such as Bill Gates and Pierre Omidyar (the founder of eBay), are
using their own charities to
pour money into the field. So, increasingly, is the for-profit sector, including
“socially responsible” investors
and capitalists, more interested in the bottom line than the poverty line?
Micro finance is a promising way to get credit to parts of the country starved of
capital. So it is a pity that all
these lenders are competing to support the same, small groups of micro finance
institutions that cater to the most
creditworthy borrowers. It would be better for the poor if the IFI’s and donors
left the best credit risks to profit#seeking lenders and concentrated instead on
those still stuck outside the system.
No doubt that sounds ungrateful. The private sector shunned the risk-out of
ignorance and a lack of expertise.
The pioneering work of donors means that there are now some 10000 micro finance
institutions lending an
average of less than $300-400 million to poor borrowers worldwide.
As a result, micro finance has become profitable. Top tier micro lenders no longer
need subsidies or even
commercial loans from IFI’s or philanthropists.
The inter–American development bank has acknowledged that the best micro lenders
can finance loans by
attracting commercial investors. It is busily selling equity stakes in its
portfolio of micro finance investments.
But other development groups are less willing to cut the apron strings. They
continue to devote scarce aid dollars
to the micro lenders that need the least. Having nurtured these outfits when for
profit would not; they now want
to bask in their successes. Some philanthropists, too, prefer to take the safe
route and invest in stable, profitable
top tier micro finance groups.
This trophy lending is harmful. By subsidizing micro finance groups that do not
need it, aid bodies and
philanthropists discourage private money, which cannot compete with their soft
terms. In the long run, this harms
micro financers, because it slows down their integration into the financial-
services industry. Aid money is better
spent where commercial cash fears to tread-such as on the next generation of micro
finance institutions. Subsidies
are often needed to lend to the rural poor, where small, scattered population make
it hard for commercial lenders
to cover their costs. Donor funds could be used to invest in technology such as
mobile payment, which promise
to cut the costs of providing micro credit. Top micro finance institutions
themselves may need help in expanding
into insurance and other financial products for the poor.
Only a fraction of the world’s 500 million impoverished “micro entrepreneurs” have
access to the financial
system. There is not enough donor or “socially responsible” money in the world to
meet the demand. That’s why
micro finance needs private-sector capital. Aid agencies, philanthropists and well
meaning “social” investors can
help attract it by investing only where commercial outfits will not. When the
children come of age, the best
parents step aside.
1. The author rues the fact that
(A) Too much aid money is flowing into micro finance.
(B) Competition has emerged in the micro finance sector leading to wastage of time
and energy.
(C) International financial institutions and donors do not move on to the poor who
have been left out of financial
net.
(D) Micro financing has become a commercial venture.
(E) Even micro lenders who can find funds are still dependent on aid agencies.
(a) Only (A) and (C) (b) Only (A), (B) and (C)
(c) Only (B) and (D) (d) Only (C) and (E)

. Page 3 of 40
2. Areas where “commercial cash fear to tread” are
(A) Villages with a small and scattered population where costs of lending are high.
(B) Insurances and other financial products for the poor.
(C) The rural poor who cannot offer any security with whom they work.
(D) The places where there is more credit-worthiness.
(E) The micro-finance groups that are heavily subsidized.
(a) Only (A) (b) Only (A) and (B)
(c) Only (B) and (D) (d) Only (C) and (D)
3. Micro finance has become profitable because
(a) There are about 10000 micro institutions operating worldwide.
(b) International financial institutions are prepared to take a chance since they
are helping the poor.
(c) Small amounts are lent to a large number of people thereby reducing the risks.
(d) Many development groups continue to devote scarce aid dollars to micro lenders
that need the least.
4. When the author concludes that the best parents step aside when “children come
of age “, he implies that personal
and institutions funding the field of micro finances…
(a) Must move away from outfits that are profitably mature and can find funds by
themselves.
(b) Do not need to find entrepreneurs any more to start a venture.
(c) Must move forward into next generation ventures that had hitherto been touched.
(d) Must move into areas like improving regulations and infrastructure.
Passage (Q.05-Q.09): The people must use their power to make laws to prohibit the
business model of
“surveillance capitalism”. The provision (______) services on the internet in
return for comprehensive behaviour
collection through the individual’s personal devices should be equally prohibited
to all parties. Neither Twitter
nor Amazon nor Koo nor Reliance should be allowed to play with our data in an
unbridled way. This is an
environmental regulation, like prohibiting the discharge of particular poisonous
chemicals into the water or the
air, or like prohibiting a fraudulent business practice. It should apply to all.
Beginning from the people’s right to be protected against (________) data practices
allows us to attack many
problems at the root, rather than proceeding branch by branch, one company at a
time. Requiring all service
providers on the internet to minimise behaviour collection will cause a redesign of
services.
We need the rich services of the 21st century internet that make our lives better.
But we don’t need those services
to be paid for by monitoring our every move; in fact, as we have learned, that’s
too high a price for society and
democracy to pay.
5. Which of the following reflects the tone of the passage?
(a) Reproaching (b) Lionizing (c) Quixotic (d) Analytical
6. ‘Beginning from the people’s right to be protected against (________) data
practices allows us to attack many
problems at the root, rather than proceeding branch by branch, one company at a
time.’ Which of the following
word , as a fill in. will make the sentence coherent?
(a) humane (b) prey (c) predatory (d) quarry
7. Which of the following reflects the title of the passage?
(a) Surveillance Capitalism: a bane
(b) Surveillance Capitalism: a boon
(c) Protecting people’s rights
(d) Twenty first century internet

. Page 4 of 40
8. What can be inferred from the passage?
(a) The laws pertaining to protecting the privacy of people with regard to the
provision of services on the internet
are adequate.
(b) The laws pertaining to protecting the privacy of people with regard to the
provision of services on the internet
are inadequate.
(c) The laws pertaining to protecting the privacy of people with regard to the
provision of services on the internet
are outdated.
(d) The laws pertaining to protecting the privacy of people with regard to the
provision of services on the internet
are stringent.
9. The provision (____) services on the internet in return for comprehensive
behaviour collection through the
individual’s personal devices should be equally prohibited to all parties. Fill in
the correct preposition to make
the sentence coherent.
(a) of (b) by (c) over (d) in
Passage (Q.10-Q.14): The opening of the official files by the British Government in
the first week of February
concerning ‘Black Wednesday’ (September 16, 1992) – the day that saw the exit of
the British pound from the
European Exchange Rate Mechanism (ERM) – forms a compelling reason for us in India
to take up a similar
advocacy concerning public access to information on economic affairs in India.
There has been substantive
movement at both towards greater openness, transparency and accountability in
governance, the practice of
secrecy in the conduct of economic policies has remained unaffected by these
developments. Secrecy in the
realm of economic policy is recognised as legitimate exception to the law
pertaining to disclosure of information.
The documents released under the recently amended Freedom of Information Act,
reveal significant
disagreements between Margaret Thatcher and the Chancellor of the Exchequer, Nigel
Lawson, over Britain’s
decision to join the ERM; the failure of the succeeding government to make a
realistic assessment of the potential
impact of German unification on the British economy; lack of contingency plans to
deal with the crisis over
interest rates (recession at home required lower interest rates while the need to
keep the pound within the ERM
band required higher interest rates); the Bank of England’s ill-advised move to
spend around $39 billion
(approximately £ 20 billion) in purchasing pounds and the record loss of £ 3.3
billion in a single day of Treasury
money.
While the democracies represented a superior form of government than aristocracies,
as decision making
apparatuses they proved to be more susceptible to public control and opinion than
the latter. The political
compulsions of increasing popular representation challenged the existing modus
operandi of a closed system of
governance. Institutionalization of secrecy, in other words, became a political
necessity for Western
democracies.
Thus while formally it was the Agadir crisis – the deployment of Geman warship in
the Moroccan port in July
1911 that justified the unusually hasty passage of the Official Secrets Act by the
British Parliament – the real
reasons for secrecy lay in the assault that democratisation was inflicting on the
traditional hold exercised by
ruling classes over the State. It is indeed ironical that a practice that in part
supported the project of colonialism
did not see the end of day with decolonisation. Secrecy in governance remained very
much a legitimate exercise
of state power, justified in the name of national security.
The overwhelming secrecy and mystification with respect to economic policy in
India, especially negotiations
relating to structural adjustment, serves as a relevant illustration. Structural
adjustment in 1991, it may be
recalled, was largely justified by the present Prime Minister, then Finance
Minister, as a measure to bail India
out of the financial crisis.

. Page 5 of 40
10. What, according to the author, is the true cause behind democracies resorting
to secrecy?
(a) Proliferation of crime in the western societies.
(b) Democracies led to loosening of hold by the rulers more susceptible to public
control and opinion and
Secrecy in governance remained very much a justification in the name of national
security.
(c) With technological innovations throwing open official secrets, it becomes all
the more necessary to maintain
secrecy.
(d) Democracies being less vulnerable to public opinion and control.
11. Existence of official secrecy under which form of government is found as
ironical by the author?
(a) Oligarchy. (b) Monarchy. (c) Democracy. (d) Aristocracy.
12. Making public the files concerning ‘Black Wednesday’ revealed
(A) A miscalculation in plans to meet the interest rates crises.
(B) Bank of England’s miscalculated move to buy pounds.
(C) deployment of German Warships in the Moroccan Port in July 1911.
(D) failure to make an evaluation of the impact of unification of Germany on the
British economy.
(a) B, C and D only (b) A, B, C and D (c) A, B and D only (d) A, B and C only
13. The passage deals with official secrecy kept in
(a) economic matters. (b) political matters.
(c) defence matters. (d) legal matters.
14. What was the reason given by the Government for resorting to structural
adjustment?
(a) To help India come out of a financial crisis.
(b) To help India become a global power.
(c) To obtain loan assistance from international agencies.
(d) To introduce reforms in sectors of primary education, health and nutrition.
Passage (Q.15-Q.19): Former President Pranab Mukherjee was once described by a
magazine as “the man who
knew too much.” His autobiography is not a bare-all memoir. The Turbulent Years,
Part Two of his
autobiography was a bit of a damp squib, probably because the author was the
occupant of Rashtrapati Bhavan
when the book was released. Part Three of Mukherjee’s memoirs, which covers the
period is far more candid
about the behind-the-scenes maneuvering in the government and the Congress party.
Mukherjee occasionally resorts to the ploy of citing media reports of that time to
convey what he wishes to say.
Thus, when Sonia Gandhi appointed Manmohan Singh instead of him as Prime Minister,
he quotes the surprise
in the media that while he had vast experience in government, Singh’s bio-data was
that of a civil servant with
just five years as a reformist finance minister. Sonia Gandhi prevailed on
Mukherjee and he reluctantly
acquiesced.
As Prime Minister, Manmohan Singh relied heavily on Mukherjee, who assumed many
prime ministerial
responsibilities. It was Mukherjee who presided over some 95 GoMs (Group of
Ministers). The preponderance
of GoMs was a way of establishing an effective decision- making process by
virtually bypassing cabinet
meetings. The author admits candidly that he has a short fuse. At one stage, with
the PM’s blessings, he asked
all officials to leave the cabinet room. He then yelled at his ministerial
colleagues that, in his long experience,
he had never seen such cabinet meetings where endless discussions led to no
decision. The Singh government
quietly substituted the GoMs as the decision-making bodies to get around the
unending talk shops that were the
cabinet meetings.

. Page 6 of 40
In explaining the circumstances of his own appointment as President, Mukherjee
simply states the fact and lets
the readers reach their own conclusions. Sonia Gandhi had reservations about
Mukherjee as a Congress candidate
even in the 2007 presidential election when the Left parties proposed his name.
affairs. Once again, in 2012,
Mukherjee’s name made the rounds and there seemed a general consensus on his name,
except for her.
Curiously, Sonia Gandhi finally agreed to Mukherjee as the party’s choice only
after Mamata Banerjee spoke
out vehemently against Mukherjee’s candidature. Banerjee’s actions at this juncture
were puzzling. She kept
oscillating. She first proposed Mukherjee’s name to Gandhi, then she took a U-turn
and announced her support
for a second term for Abdul Kalam in public the next day. But Mukherjee does not
spell out in this case just what
was Banerjee’s strategy.
15. It can be inferred from the second paragraph that
(a) Pranab Mukherjee himself thought that Manmohan Singh was not the ideal choice
as the Prime Minister.
(b) Pranab Mukherjee would have been more effective as the PM than Manmohan Singh
if Singh had been
chosen.
(c) Manmohan Singh was the apt choice as the PM.
(d) Pranab Mukherjee was disillusioned when he was not appointed the PM.
16. Which of the following is not true about The Turbulent Years, Part Two,
according to the passage?
(a) It disappointed readers.
(b) Mukherjee is blunt in his narration.
(c) It is silent on many developments during the period that the book covers.
(d) It was published during Mukherjee’s presidency.
17. Which of the following cannot replace the ‘short fuse’ as used in the passage?
(a) Quick Temper (b) Irritation (c) Anger (d) Vitriol
18. According to the passage, GoMs replaced cabinet meetings because
(a) cabinet meetings were found to be ineffectual.
(b) the President once lost his temper.
(c) cabinet meetings usually stonewalled decision making.
(d) the cabinet meetings took a long time to reach a consensus.
19. If you were to interview Mr. Mukherjee what question would you ask him?
(a) Do you think Sonia Gandhi was unfair with you?
(b) What do you think are your shortcomings as an administrator?
(c) Why did Mamata Banerjee oppose your candidature in public?
(d) Who, according to you, was a more suited Prime Minister, if not you or Dr.
Manmohan Singh?
Passage (Q.20-Q.25): The Arms Trade Treaty (ATT) is the first multilateral treaty
to regulate the international
trade in conventional arms. In this respect, it is a landmark treaty for the
international community, trying to find
and establish mechanisms to control unregulated flows of arms. Since its entry into
force on December 24, 2014,
130 states have signed the treaty and so far 92 became State Parties. However, when
looking at the regional
divide of ratifications and accessions on a world-wide scale, the Asia-Pacific
shows by far the lowest numbers
in global comparison. The past years didn’t result in much progress; efforts to
tighten arms transfer controls in
the region seem to be at a stalemate. What is the reason for this inactivity, given
so many countries in the region
face significant small arms proliferation problems which pose threats to both
national and regional security?
The ATT is to be seen in the greater scope of what is commonly known as export
control, or, more positively
connoted, Strategic Trade Control (STC). The main security-enhancing effect of the
treaty is to be found in the
requirement to introduce comprehensive control systems to make sure that exports,
imports, transits, and

. Page 7 of 40
transshipments of conventional weapons will not be diverted and end up in the hands
of illicit actors. Thus, the
ATT can serve as another instrument inside the global anti-terrorism toolbox. The
treaty scope requires states to
introduce legislation and establish comprehensive control systems to perform case-
by-case risk assessments. To
do so, states must have competent national licensing authorities that check
relevant control lists of military items.
Questions of the end-use and the end-user of weapons shipments are thus essential
for such authorities to consider
when reviewing applications for arms transfer licenses. Possible cases when a
license must be denied include
those where arms might end up in situations where crimes against humanity occur.
Comparing the Asia-Pacific to other developing regions, it becomes obvious that
acceptance and popularity of
the ATT has progressed rather slowly. Up to date the number of ATT ratifications
within the Asia-Pacific
remains at an absolute low of only six out of 53 countries. It is within the
Pacific Islands where one can see
probably the most forward movement in putting the ATT in practice on a regional
basis. This is mainly due to
the work of the Pacific Island Forum Secretariat, with the help of the government
of New Zealand. The
Secretariat published an ATT blueprint legislation in an effort to support the
Pacific states in their ambitions to
ratify and implement the treaty framework. Amongst its 18 member countries,
ratifications by Tuvalu, Samoa,
and signatures to the treaty by Kiribati, Nauru, and Palau prove these regional
efforts to be at least partly
successful. Australia and New Zealand as strong ATT supporters also played a major
role in these efforts to
make ATT gain momentum in the region.
In eastern Asia, Japan and South Korea are the sole active supporters of the
treaty. For many years, South Korea
has viewed STC as an important security instrument, mainly preventing North Korea
from acquiring products
that may add to its conventional and unconventional weapons arsenal After the Tokyo
subway sarin gas attack
in 1995, Japanese authorities started to give the overall topic of STC ever more
emphasis and also began to
promote the goal of establishing a tight regional network of sensitive items
controls amongst their neighbors.
20. According to the passage, How Arms Trade Treaty can be referred as Strategic
Trade Control?
(a) The Arms Trade Treaty ensures the tightening of the exports, imports, transits
and transshipments of
conventional weapons controls.
(b) The Arms Trade treaty ensures the national and regional security.
(c) The Arms Trade Treaty is introduced to ensure that the haulage of the weapons
will not fulfill some illegal
purpose.
(d) All are correct.
21. Which of the following cannot be inferred from the passage?
(a) There are many countries which face small arms proliferation problems posing
threat to national and regional
security.
(b) The purpose and success of the treaty made many other countries join the
agreement.
(c) Arms Trade Treaty is a multilateral treaty in which there are 92 State Parties.
(d) The Arms Trade Treaty is signed among the countries to control unregulated
flows of arms and, hence,
ensuring national security.
22. According to the passage, how can the purpose of the arms trade treaty be
fulfilled?
(I) By issuing limited number of arms for avoiding illegal use.
(II) By keeping check on the control lists of military items that were transported.
(III) The purpose can be fulfilled by evaluating the risks.
(a) Only (I) is correct (b) Only (II) is correct
(c) Both (I) and (III) are correct (d) Both (II) and (III) are correct

. Page 8 of 40
23. According to the passage, what had made the Arms Trade Treaty ratification
progress?
(a) The legislation published by the developed country in order to make the treaty
implemented and ratified by
the pacific states.
(b) The advantages of the treaty towards national security attracted many other
countries for the treaty
ratification.
(c) The supporters of the treaty like Australia and New Zealand had played a major
role in making the treaty
ratified by other member nations.
(d) Both (a) and (c)
24. Which of the following is the appropriate title of the passage?
(a) Arms proliferation problems. (b) Peace among nations.
(c) National and regional security. (d) The Arms Trade Treaty in the Asia- Pacific.
25. According to the passage, what is, are the perspective(s) of the East Asian
countries regarding the treaty?
(I) Despite increased counter terrorism attacks, east Asian countries has not taken
any further step towards
signing the treaty.
(II) East Asian countries like Japan being the active supporter check relevant
control lists of military items among
its neighbours.
(III) South Korea supports the treaty as it prevents other countries from acquiring
products.
(a) Only (I) is correct (b) Only (II) is correct
(c) Both (I) and (III) are correct (d) Both (II) and (III) are correct
Passage (Q.26-Q.30): Read the given passages carefully and answer the questions
that follow them.
When the United States went to war in Afghanistan in 2001, the Taliban’s despicable
treatment of women was
cited by First Lady Laura Bush as one of the main reasons for going to war. Yet,
since that regime fell 15 years
ago, the
Afghan government has neither included women in the peace building process, nor has
it stemmed the endemic
rate of violence against them.
2016 was the bloodiest year since the year of the US invasion. While the Taliban
has lost power, it continues to
operate, Afghan women continue to endure “parallel justice” for supposedly “immoral
activities”.
Afghanistan, a dangerous country in the world to be a woman, educates only 15% of
its girls. 60% are married
off by age 16. Fatwas have been issued for girls not to attend school and even the
small handful of women who
managed to enter politics has been targeted. Assassination attempts have been made
on women in public service.
Political leaders, directors of women’s affairs and police chiefs have been killed
in recent years.
The fallacy of liberating women as part of the war cry has turned out to be yet
another illegitimate reason for
this seemingly never-ending conflict. Afghan women are now dealing with not only an
epidemic of violence
inside their homes — but also in society in general. The prolonged war has
exacerbated this. There was a road
less traveled, which may have ensured a different outcome, but it seems to have
fallen on deaf ears.
Sadly, women were left out of almost all political participation and little has
changed. Their calls for disarmament
were ignored, and the efforts of brave women such as Malalai Joya to prevent
warlords from taking power were
unsuccessful. She was instead removed from her governmental position.
In 2001, we had hoped that the international community would listen to the voices
of Afghan women, but the
failure to do so and the dire situation of Afghanistan today shows that few lessons
have been learned. Discussions
on including women in decision-making related to ending conflict and ensuring peace
have not been acted upon.
Transitional governments supported by the UN were almost entirely male in
Afghanistan. And a decade later,
exactly the same mistake was made in Libya.
Both countries are now in a virtually impossible position of political stalemate.
In Libya, on the day of elections,
a brilliant constitutional lawyer and political activist Salwa Bugaighis was
murdered — her political platform

. Page 9 of 40
was simply to build peace. The Libyan Women’s Platform for Peace (LWPP), which she
co-founded, carries on
her work, with major obstacles to overcome.
Meaningfully including women in rebuilding peace in war-torn countries seems like
an obvious solution to all
of this.
Enabling women to be part of processes which secure their future and those of their
families and the societies
they live in is not only the right thing to do, it’s also the most effective thing
to do politically and economically.
26. After going through the passage, what is the single important thing you infer?
(a) Any society reluctant to give women their rightful life can never progress
economically or politically.
(b) Afghanistan and Libya are still backward due to their barbaric attitude towards
women.
(c) Suppressing women and denying them education is injustice done to the entire
society.
(d) Educate a woman and you educate the entire family.
27. The fall of the Taliban regime has not improved the situation in Afghanistan
because
(a) the Taliban has not been totally rooted out.
(b) the mindset of the people has not changed and women are punished citing
supposedly immoral crimes.
(c) the UN has been supporting entirely all-male governments.
(d) All the above.
28. The author calls the liberation of women a fallacy because
(a) women in Afghanistan are not educated and women who dared to be different are
targeted and killed.
(b) dissipating violence against women was one among the war agenda but this has
only aggravated the situation.
(c) Afghan women have to now face domestic as well as social violence.
(d) the prolonged Afghan war has aggravated the agony faced by women.
29. What do Malalai Joya and Salwa Bugaighis have in common?
(a) They dared to be different.
(b) They were both terminated.
(c) They were the first ladies of their respective countries.
(d) They accepted the administration.
30. What can be the remedy for the stalemate in war torn Afghanistan and Libya?
(a) The UN can with the help of world countries take the initiative by sending
delegations for sorting out the
issue.
(b) Women should be given a fair chance in contributing to nation building.
(c) A government headed by a woman with an all women cabinet can do the magic.
(d) A and b

Directions (Q.66 – Q.105): Read the comprehensions carefully and answer the
questions based on it.
Passage (Q.66-Q.70): The provision related to divorce by mutual consent under Hindu
law has been inserted in
the Hindu Marriage Act, 1955, as Sec 13B, by an amendment in the year 1976. In
cases where both the parties
to a divorcedo not want to live together due to non-compatibility and they want to
end their marriage, they can
go for a petition of divorce with mutual consent under section 13B. The following
requirements have to be
fulfilled: The parties must have to live separately since at least one year, The
parties are unable to live together,
and both the parties must have agreed to dissolve the marriage.
In Sureshta Devi v. Om Prakash, the Supreme Court while pronouncing judgment stated
that the expression
living separately indicates the parties not living like husband and wife even if
they are living under the same
roof. What is important is that the parties have no desire to live together or to
perform the marital obligations
and both the parties are living separately for a period of at least one year before
the presentation of the case.
Non-compatibility between the spouses plays an important role and should try to
protect their marriage.
However, after fulfilling all of the above requirements and filing a petition for
divorce by mutual consent, the
parties must have to wait till the period of six months which is termed as cooling
period unless there are any
reasonable exceptional circumstances. Jurisdiction of the court should not be a
major issue in filing for divorce
as the petition can be filed within the local limits of the ordinary civil
jurisdiction of where the marriage was
solemnized or where either of the parties currently resides. The parties to a
marriage must be living separately
for at least one year before filing the petition.
The doctrine of irretrievable break-down of marriage as a ground of divorce as per
the recommendations of the
71st report of the Law Commission of India is not available to the High Court’s as
it is solely exercised by the
Supreme Court under Article 142 of the Constitution, thus in case of requirement
such cases shall be referred to
Supreme Court.
[Edited and extracted from:https://www.lawyered.in/legal-disrupt/articles/divorce-
mutual#consent-dushyant-tiwari/]
66. Reema and Rohan got married in May 2019 and due to their profession, they are
living in different cities
separately since last two years. One fine day on mutual agreement, they decided to
take divorce, as they found
no meaning to their marriage as living in different cities. Can they get divorce
under Section 13B?
(a) No, because merely living separately does not grant the permission under
Section 13B.
(b) Yes, because the divorce can be granted on mutual agreement under Section 13B.
(c) No, because marriage is not a contract under the Hindu Marriage Act.
(d) Yes, because the circumstances of the case compel them to do so.
67. Aashi and Ajay have, during lockdown, realized that they have huge
compatibility issues, which have been
creating chaos in their lives since they got married in 2019, which solemnized in
Jaipur. They were living
separately for past six months; Aashi is in Jaipur and Ajay is in Delhi. They
mutually decided to file a divorce
application in Jodhpur. Decide the matter?
(a) They cannot be granted divorce as living separately for at least one year prior
to the filing of application is
statutory requirement under the Act.
(b) They cannot be granted divorce as the application can be filed only in Jaipur,
where the marriage was
solemnized.
(c) They can be granted divorce as they mutually decided to get separate through
divorce under Section 13B.
(d) None of the above

. Page 17 of 40
68. Treya and Shreya, are a Hindu couple, married under Special Marriage Act. They
are living separately for more
than one year due to non-compatibility in their marital relations. They want to
seek divorce by mutual consent
under Section 13B. Suggest them with appropriate legal solution under section 13B.
(a) They can get the divorce under Section 13B because they have mutually decided
to do so and circumstances
strengthen their application.
(b) They cannot get the divorce as they married under the Special Marriage Act.
(c) They cannot get the divorce as Section 13B does not apply in their case.
(d) They can get the divorce under Section 13B because even if they married under
Special marriage Act, Hindu
marriage is governed under Hindu Marriage Act.
69. Mansi and Rohan got married in 2019. However, both of them were not happy with
their marriage anymore and
were allegedly having extramarital relationships. They were living apart without
ever visiting each other for
more than a year. They blamed each other for their suffering and unhappiness. Both
alleged each other to be
involved in a series of illicit relationships but denied ever being involved in
such relationships themselves.
Decide.
(a) Mansi and Rohan can get divorce under mutual consent as both do not want to
stay together.
(b) They cannot get the divorce under Section 13B as their illicit relationships
haven’t been proved.
(c) They can get divorce under section 13B as both of them have been living
separately for more than
one year and are unable to reconcile.
(d) They cannot get divorce under 13B but can get under 13(1).
70. Ajay and Arti got married on 23 December 2019 as per Hindu rites and
ceremonies. However soon thereafter
differences started arising and both of them did not want to stay together. Ajay
started torturing Arti and also
tried to poison her once. Arti couldn’t bear this any longer and decided to file a
divorce petition, however the
court directed Arti to stay in the matrimonial house as the 6 months cooling period
is necessary. Ajay also
apologized to Arti for his behavior. Arti has filed a petition in High court.
Advice Arti.
(a) High court have jurisdiction, since it is a family law matter.
(b) Waiting period/Cooling period is necessary to maintain the sanctity of the
institution of marriage.
(c) The matter should be referred to the Supreme court under article 142 as the
waiting period as mentioned in
the act can be waived depending upon the nature and circumstances of case.
(d) Arti should file a case of attempt to murder.
Passage (Q.71-Q.75): Expressions such as “joint liability”, “common liability”,
“constructive liability”,
“vicarious liability” etc. convey, more or less, the same connotation in the sphere
of law.” Vicarious liability” is
strictly the liability which a person incurs for the acts of his “servant” or
“agent” committed in the course of
employment as such. The most appropriate expression in this context of “joint
liability” would be “constructive
liability” which is used to denote the liability of a person for an offence which
he has not actually committed but
cannot escape from its responsibility. The person concerned becomes liable in law
for the consequence of the
act of another, despite the fact that he has not done it himself. Thus, an act
committed by another person will be
attributed to the accused if such an act is done “in furtherance of common
intention falling under Section 34
IPC” or done “in prosecution of the common object falling under Section 149 IPC”.
Section 149 prescribes the principle of “vicarious or constructive liability” on
all the members of an unlawful
assembly (An assembly of five or more persons) where an offence is committed by any
member of such assembly
in prosecution of the common object of that assembly or that such of the members of
that assembly knew it that
such an offence was likely to be committed. While the first limb of Section 149 IPC
envisages
“the commission of an offence” by a member of an unlawful assembly in prosecution
of the common object of
that assembly, the second limb of the Section contemplates “knowledge” on the part
of a member of the unlawful
assembly regarding the likelihood of such offence being committed in prosecution of
the said common object.
[Edited and extracted from:https://www.livelaw.in/columns/section-149-ipc-supreme-
court-viramvirma-v#state-of-madhya-pradesh-ocular-testimony-medical-evidence-
common-intention-187446]

. Page 18 of 40
71. Leader of a naxalite group, Rupawati made an assembly of 25-30 peoples with an
object to dismantle the polling
booths during elections in her state. However, due to some medical emergency she
could not be present or join
the assembly that day but still the rest of member did what they decided. Decide?
(a) Rupawati is liable along with other members under sec 149 as she incited all
the members to dismantle the
polling booths.
(b) Rupawati is not liable under sec 149 as she was neither present nor did
anything wrong.
(c) Rupawati is liable along with other members under sec 149 as she shares a
common object to all the members
of the assembly.
(d) Rupawati is not liable sec 149 as she does not share a common object to all the
members of the assembly
and if it was so, she would have joined it whatsoever the situation would be.
72. Unions of employee of different banks were planning to go on strike and
organize a peaceful rally in opposition
of the privatizations of government banks. However, when they were protesting some
member of the union
became violent and destructed the properties of RBI. Decide the liability of all
the member.
(a) All the members of the unions are liable for the violence and destruction of
the properties of RBI as they
formed an assembly to destruct the properties of RBI.
(b) All the members of the unions are not liable for the violence and destruction
of the properties of RBI as the
assembly was not formed for such activities.
(c) All the members of the unions are liable for the violence and destruction of
the properties of RBI as they
have common object to destruct the properties.
(d) Only the violent members of the unions are liable for the violence and
destruction of the properties of RBI
as they only destroyed the property.
73. Mr. Santa and Mr. Banta were twin brothers who had a dispute over who was born
first. One day Mr. Santa with
his five friends went armed with gun and lathis to forcibly make his brother admit
that he was born first and on
denial, Santa shot him dead. Decide
(a) Only Santa is liable for murder as he is the one who shot Mr. Banta dead.
(b) Santa and all his five are liable for murder under sec 302 read with 149 of IPC
as they assembled with the
common object.
(c) Santa’s friends are not liable for murder as they never had the intentions to
kill Mr. Banta.
(d) Santa and all his five are not liable for murder under sec 302 read with 149 of
IPC as they did not have the
common object to kill Mr. Banta.
74. Ranchor, Raju, Farhan and Chatur all were great friends and fans of BTS which
is a music band from Russia.
When they heard that a BTS concert is going to happen in India they could not
resist and bought the ticket as
soon as possible however made a plan to meet them personally and planned to kidnap
guard and in doing so
Chatur kidnapped the security guard Viru. Decide
(a) They all are liable under sec 149 of IPC for kidnapping Viru as they had the
common object to kidnap.
(b) They all are liable under sec 149 of IPC for kidnapping Viru as the assembly
was illegal.
(c) Only Chatur is liable for kidnapping Viru.
(d) They all are not liable under sec 149 of IPC for kidnapping Viru
75. A group of more than 30 people of bajarang dal with Lathis, guns and Kalikh was
roaming in large number on
the Valentine’s Day to punish all the lovers for unchaste Indian culture and in
doing so one of the party members,
Kalin Bhaiya beat one of such person who was caught by the Bajarang dal involved in
such activities, Guddu
who was opposing their ideologies. Guddu died on the spot. Decide.
(a) Only kalin bhaiya is liable for the death of guddu as he killed him.
(b) The whole Bajrang dal is liable for the death of guddu as assemble with the
common object.
(c) The whole Bajrang dal is not liable for the death of guddu as they never
assemble with the intentions to kill
anyone.
(d) The whole Bajrang dal is liable for the death of guddu as the assembly was
unlawful.

. Page 19 of 40
Passage (Q.76-Q.79): An order of custody of minor children either under the
provisions of the Guardians and
Wards Act, 1890 or the Hindu Minority and Guardianship Act, 1956 is required to be
made by the court treating
the interest and welfare of the minor to be of paramount importance. It is not the
better right of either parent that
would require adjudication while deciding their entitlement to custody. The desire
of the child, wherever
possible, coupled with the availability of a conducive and appropriate environment
for proper upbringing
together with the ability and means of the parent concerned to take care of the
child are some of the relevant
factors that have to be taken into account by the court while deciding the issue of
custody of a minor.
Object and purpose of the Guardians and Wards Act, 1890 is not merely physical
custody of the minor but due
protection of the rights of ward’s health, maintenance and education. In
considering the question of welfare of
minor, due regard has, of course, to be given to the right of the father as natural
guardian but if the custody of
the father cannot promote the welfare of the children, he may be refused such
guardianship.
Children are not mere chattels nor are they toys for their parents. Absolute right
of parents over the destinies and
the lives of their children, in the modern changed social conditions must yield to
the considerations of their
welfare as human beings so that they may grow up in a normal balanced manner to be
useful members of the
society and the guardian court in case of a dispute between the mother and the
father, is expected to strike a just
and proper balance between the requirements of welfare of the minor children and
the rights of their respective
parents over them.
Better financial resources of either of the parents or their love for the child may
be one of the relevant
considerations but cannot be the sole determining factor for the custody of the
child. It is here that a heavy-duty
is cast on the court to exercise its judicial discretion judiciously in the
background of all the relevant facts and
circumstances, bearing in mind the welfare of the child as the paramount
consideration.
The word “welfare” used in Section 13 of the Hindu Minority and Guardianship Act,
1956 has to be construed
literally and must be taken in its widest sense. The moral and ethical welfare of
the child must also weigh with
the court as well as its physical well-being. Though the provisions of the special
statutes which govern the rights
of the parents or guardians may be taken into consideration, there is nothing which
can stand in the way of the
court exercising its parenspatriae jurisdiction (parent of the nature in law)
arising in such cases.
[Extracted from ‘Custody of Children’, Published by SCC Blog]
76. Gaurav Nagpal and SumedhaNagpal got married on 14.10.1996 and the child from
their wedlock was born on
15.11.1997. Gaurav Nagpal alleges that his wife, SumedhaNagpal abandoned the child
on 8.8.1999. However,
few years later, SumedhaNagpal filed a petition for guardianship of the child. Her
lawyer argued that the
distancing between the mother and the child, is causing the mother to suffer from
mental disorders like acute
anxiety and severe depression. Decide.
(a) Guardianship should be granted to Sumedha the welfare of the child is best
known to his mother.
(b) Guardianship should be granted to Sumedha as if not, then her medical condition
will worsen further.
(c) Guardianship should not be granted to Sumedha as in cases of guardianship, the
welfare of the child is
paramount, not the better right of the parent.
(d) Guardianship should not be granted to Sumedha as she abandoned the child on an
earlier occasion.
77. In the above case, the advocate appearing on behalf of Sumedha also argued that
given the poor financial
condition of Gaurav and his alcohol addiction, he will not be able to provide a
safe and healthy environment for
the upbringing of the child. However, on the other hand the advocate appearing on
behalf of Gaurav argued that
Gaurav is the natural guardian and thus has a right to custody of the child under
law. Decide.
(a) Gaurav should be given the custody as due regard has to be given to the right
of the father as natural guardian
(b) Gaurav should be given the custody as he took care of the child when his mother
abandoned him earlier.
(c) Gaurav should not be given the custody on the ground that he alone is the
natural guardian, as both parents
are considered equal.
(d) Gaurav should not be given the custody as he cannot promote the welfare of the
child due to his addiction
and financial status.

. Page 20 of 40
78. In continuation to the facts given, the advocate appearing on the behalf of
Gaurav argued that the Sumedha’s
large area of accommodation and financial affluence cannot be a determinative
factor. Determine.
(a) The argument is valid as large area of accommodation and financial affluence
cannot ensure welfare of child.
(b) The argument is valid as these factors cannot be the sole determining factor to
determine the custody of a
child.
(c) The argument is not valid as large area of accommodation and financial
affluence can ensure welfare of
child.
(d) The argument is not valid as Gaurav’s own financial condition is not conducive
for the upbringing and
welfare of the child.
79. The court finally decided in the favour of Gaurav for having the custody of the
child. However, Sumedha was
dissatisfied and went to appeal. The lawyer appearing on behalf of Sumedha argued
that the court has no locus
to decide the welfare of the child. Decide.
(a) The argument is not valid as court has parens patraie jurisdiction and that
gives it the locus to decide the best
interest of the child.
(b) The argument is valid as from the facts it was cleared that Sumedha was more
able to keep the child’s best
interest in mind.
(c) The argument is valid as the custody of the child has to be a mutual decision
of the parents and the court has
no business to interfere.
(d) The argument is not valid as there was no mutual agreement between the parents
and thus this turned into a
civil dispute.
Passage (Q.80-Q.84): The Supreme Court has observed that protection under Section
53A of the Transfer of
Property Act, 1882 is available to a person who is put in possession pursuant to an
agreement of lease in his
Favor though no lease has been executed and registered. Under the Transfer of
Property Act, 1882 Section 53A
that deals with doctrine of part performance states that:
This section states that when a transferor or someone on his behalf transfer’s his
immovable property for some
consideration in a written manner that is signed by him to the transferee or
someone on his behalf on some terms
with reasonable certitude. And in the advancement of such contract, he had used his
right of part performance to
exercise possession over the property or be in partly use of it and acted on such
contract. This also includes his
wish to willingly perform his part of contract and also if he has already performed
on his part.
Therefore, if there is no registered or proper completion of this procedure of
transferring of property as prescribed
by the law but such agreement was achieved then the transferor cannot enforce
against transferee his rights where
the transferee has taken the possession or is continuing to be in possession. Only
if the term of such contract
describes any such exception, then only transferor can do so. Also, if the
transferee is not aware about the contract
or this doctrine still his rights shall not be affected. So, this is what the
section reads as.
The bench, while dismissing the appeal, observed:
"It is clearly held by this Court that defence under Section 53A of the Transfer of
Property Act, 1882 is available
to a person who has agreement of lease in his favor though no lease has been
executed and registered.
(Source: Transfer of Property Act] Protection U/s 53A Available To Person Put In
Possession and Has
Agreement of Lease in His Favor: SC, LIVELAW)

. Page 21 of 40
80. Harry and Jacky were two brothers who shared a 2-acre land. On the shared site,
Harry intended to build a cement
plant. Harry likewise requested Jacky to lease his portion of the property.
Following that, Jacky and Harry signed
a written contract with reasonable terms. Jacky expressly said no to register or
execute the lease deed by saying,
“you’re my brother”. After seeing Harry's plant's tremendous success, Jacky
presented Harry with an eviction
notice out of envy. Can Harry use Section 53A of the 1882 Transfer of Property Act
as a defense? Decide.
(a) No, because the lease deed is neither executed nor registered, Jacky cannot
claim defense under Section 53A
of the 1882 Transfer of Property Act.
(b) Yes, because the said section protects the possession of persons who have acted
on a contract of lease with
reasonable terms but in whose Favor no valid lease deed is executed or registered.
(c) Yes, because Jacky signed a written contract with reasonable terms with harry.
(d) No, because cement plant is also constructed on Jacky's part, thus cannot claim
any defense.
81. In a similar situation, if the written agreement signed by both parties had a
stipulation saying that the lease would
only be for two years, after which Harry would be required to vacate the portion
owned by Jacky. After reviewing
the agreement, both sides signed the agreement. After two years, Jacky served Harry
with an eviction notice.
Decide
(a) Harry cannot claim defense under section 53A of Transfer of property act,
because he signed the agreement
after a careful read.
(b) Harry cannot claim defense because the agreement stipulated a two-year eviction
timeframe, Jacky's eviction
notice is valid. Thus, defense cannot be claimed.
(c) Jacky can claim defense under section 53A of Transfer of property act, because
there exists an agreement of
lease in his Favor.
(d) Harry cannot claim defense under section 53A of Transfer of property act,
because the deed has not be
been registered and executed.
82. Bablu's home was situated in the very famous khan market of Delhi. Since his
ground floor was of no use, he
vacated the same and put it on lease to a bank called “Lena bank”. Due to certain
problem faced by the bank
they shifted without giving any prior notice. Now Bablu's friend Rony approached
him and asked to lease out
the space to him for opening a saloon. Since Rony was a good friend, Bablu don’t
want to say no straight to his
face. Also, Rony raised no affirmation as to consideration, Bablu nodded and said
“we will see”. Upon Bablu's
oral statement, Rony took possession of the said space and also started the saloon.
Can Rony take doctrine of
part performance as defense in case of encroachment by the transferor?
(a) Rony cannot take defence of part performance as one conditions was not
satisfied.
(b) Rony can take defense of part performance as there exists contract between the
transferor and transferee.
(c) Rony can take defense of the doctrine of part performance as possession has
been given to Rony and he
partly made use of such by starting the saloon.
(d) Rony cannot claim because there was no transfer made for some consideration in
a written manner.
83. Gram Panchayat of Dholpur village gave certain land to farmers on lease for
cultivation purpose. An agreement
was signed between the parties and the same was executed in form of a deed. Farmers
in exercise of part
performance started using the land for cultivating crops, and pulses. Gram
Panchayat knew that farmers did not
know about the doctrine of part performance so cannot be taken as defense in case
of encroachment by the
transferor. A notice of eviction was served to the farmers to vacate the land on a
false contention that it is acquired
by the Govt for construction purpose. What will you suggest to the farmers?
(a) Farmers can still use the doctrine of part performance as a defense since their
rights will not be affected by
their lack of knowledge of the doctrine.
(b) Farmers cannot take the defense as it was gram panchayat’s decision to vacate
the land.
(c) Farmers can take the defense as they were being encroached by the transferor on
false contention.
(d) Farmers cannot take defense because the land was given to them for cultivation
and not on lease.

. Page 22 of 40
84. Choose the correct statement.
(a) A person who has a lease agreement in his favor has a defense under Section 53A
of the Transfer of Property
Act, 1882, even though the lease has not been executed and registered.
(b) Section 53A of the Transfer of Property Act, 1882 protects the possession of
persons who have acted on a
contract of sale and in whose Favor no valid lease deed is executed or registered.
(c) If the transferee is not aware of his right under the doctrine of part
performance, he cannot claim it afterward.
(d) There is no requirement for a written agreement and consideration between the
parties, and hence no
execution and registration of a deed under Section 53A of the Transfer of Property
Act of 1882.
Passage (Q.85-Q.89): The Bombay High Court, issuing guidelines for cases regarding
Sexual Harassment of
Women at the Workplaces only, has directed that such matters will be heard either
in-camera or in the judge's
chambers, orders are not to be passed in open court and should not be uploaded on
the official HC website
either.
A bench of Justice Gautam Patel has further barred from publishing proceedings
under the Sexual
Harassment of Women at the Workplace (Prevention, Prohibition and Redressal) Act,
2013 or reporting on a
judgment without the court's permission.
Breach of the guidelines or publishing any party's name or their information by
anyone, even if it is in public
domain would amount to contempt of court, as per these guidelines.
"Both sides and all parties and advocates, as also witnesses, are forbidden from
disclosing the contents of any
order, judgment or filing to the media or publishing any such material in any mode
or fashion by any means,
including social media, without specific leave of the court," the court said about
media disclosure.
GUIDELINES
PUBLIC ACCESS – Orders Can't be Published Without Court's Direction
1. In all orders, the endeavor will be to anonymize the identities of both the
parties.
(a) If any order is to be released into the public domain, this will require a
specific order of the Court.
(b) This will be on the condition that only the fully anonymized version of the
order of judgement is let into the
public domain for publication.
HEARINGS AND ACCESS
(a) The Registry will not permit anyone other than the Advocate-on-Record with a
current and valid Vakala
Nama to take inspection or copies of any filing or order.
(c) Only the advocates and the litigants are permitted to attend hearings. Support
staff (clerks, peons, etc), must
leave the Court.
BREACH – Contempt of Court
(a) The prohibition on publishing the names, address or other personal identities
of the parties is absolute. And
in no matter, prohibition will amount to violation of fundamental rights guaranteed
under constitution of India
as rights are subject to restrictions.
(b) It will continue to apply where that information about the parties has been
obtained by using the contents of
a judgment or order to discover information already in the public domain.
(c) All persons, including the media, are required to ensure strict compliance with
these conditions of anonymity.
Failure to do so will be a contempt of court.
(SOURCE: No Media Reporting, Public Disclosure Of POSH Case Judgments Without Prior
Approval :
Bombay High Court Issues Guidelines To Shield Anonymity, LIVE LAW)
85. Rajni, a TCS employee, was subjected to sexual harassment by his boss, who made
racist and sexist statements
about her during an online meeting. A case was filed under the Sexual Harassment of
Women at the Workplace
(Prevention, Prohibition and Redressal) Act, 2013 by Rajni against his boss. Rajni
was also a frequent social
media user. To expose his boss's misconduct, she published his boss's comments on
social media on her social
account. Does this amount to contempt of court?
(a) No, it will not amount to contempt of court as she did not reveal her identity
in the post.
(b) No, it will not amount to contempt of court because she as victim of assault
had all rights to expose the
accused.

. Page 23 of 40
(c) Yes, because both parties' identities must be protected and cannot be disclosed
in any situation thus, it will
amount to contempt of court.
(d) Yes, because publishing the names, address or other Personal information of the
parties is absolutely
prohibited. Thus, amount to contempt of court.
86. Neelam is a second-year law student interning with renowned lawyer of the
Supreme Court, Jarvis salve. Mr.
Jarvis salve took Neelam to assist with him on a case of rape happened in a jungle.
Opposite counsel alleges
contempt of court. Deci
(a) It will be contempt of court since Neelam will come under the category of
supporting staff.
(b) It will be contempt of court because only the advocates and the litigants are
permitted to attend hearings.
(c) It will not be a case of contempt of court because proceedings concerned were
of rape in a jungle.
(d) It will not be a case of contempt because Neelam is a law intern under advocate
Jarvis salve.
87. Nirmala a victim of sexual harassment at his workplace, file a case under POSH
act against the accused. The
accused was a known name in the business industry. Since it’s a high-profile case,
various media houses
approached Nirmala to come live on their channel. Nirmala gave her consent as she
wanted to expose the accused
in front of public. Nirmala goes live for the video to go public. And the video was
banned as a case was filed
against the media house for contempt of court. Decide
(a) It was a case of contempt of court because media houses did not strictly
complied with the conditions of
anonymity.
(b) It was not a case of contempt of court because media house made the video live
with due approval of court.
(c) It was not a case of contempt of court because Nirmala gave her free consent to
go live.
(d) It was not a matter of contempt of court since the accused would be exposed in
public for their awful
conduct.
88. Following the guidelines given by the court or cases regarding Sexual
Harassment of Women at the Workplaces,
all the major media houses across the went on a strike stating that such
prohibition under the guidelines may
make media reporting on POSH cases completely out of bounds. They also filed a
petition for violation of their
fundamental right to speech and expression guaranteed under constitution of India.
(a) Contention is valid since the guidelines are putting an absolute prohibition on
publication of such
proceeding.
(b) Contention is invalid because such absolute prohibition on publication will not
amount to violation of right
to freedom and speech.
(c) Contention is invalid because such publication will harm the identity of
parties involved.
(d) Contention is valid because such guidelines can be used by accused to suppress
the victim.

89. While reporting a case of sexual harassment at work, XYZ news media house cited
a similar incident,
information of which is already in the public domain. The information of similar
event had been extracted from
a judgement available public domain. While reporting media house adhered to all of
the guidelines and also
secured prior court consent for reporting the case of sexual harassment at
workplace but not for citing a similar
incident. A case against contempt of court was filed against the media for
publication of the other similar
event. Decide
(a) Not a case of contempt of court since the information of similar cited incident
is already in public domain.
(b) A case of contempt of court because the guidelines also extend to the
information already available in public
domain.
(c) Not a case of contempt of court because media house adhered to all the
guidelines while reporting the matter.
(d) A case of contempt because as such publication becomes a matter of breach of
privacy for concerned parties.

. Page 24 of 40
Passage (Q.90-Q.94): The Allahabad High Court last week quashed an FIR registered
against a Muslim under
Section 366 (Kidnapping, abducting or inducing woman to compel her marriage, etc.)
of IPC for allegedly
kidnapping and marrying a Hindu girl.
The Court was hearing the joint plea filed by the Woman (Wife/petitioner no.1) and
Man
(Accused/husband/petitioner no.2) seeking quashing of the FIR claiming that the
Woman had left her paternal
home out of her own sweet will and being a major girl, she was free to take her
choice to embrace Islam and
perform marriage with the accused.
It was asserted by the couple that no offence under Section 366 IPC would be made
out as petitioner no.1 is a
major girl and that the entire criminal case lodged by respondent no.4 was nothing
but an abuse of the process
of the law.
it was observed by the Court that right to live with a person of his/her choice
irrespective of religion professed
by them, is intrinsic to right to life and personal liberty."
Importantly, the Bench of Justice Pankaj Naqvi and Justice Vivek Agarwal had also
remarked thus,
"We fail to understand that if the law permits two persons even of the same sex to
live together peacefully then
neither any individual nor a family nor even the state can have an objection to the
relationship of two major
individuals who out of their own free will are living together."
https://enalsar.informaticsglobal.com:2278/news-updates/inter-religious-marriage-
woman-major-right-choose#partner-allahabad-high-court-quashes-fir-366-ipc-husband-
181287 Live Law dated 09/09/21
90. X and Y are deeply in love and want to stay together, but the catch in their
case is both are already married to A
& B respectively and X is a Muslim whereas Y is a Hindu. X & Y without divorcing
their partners marry each
other. B, the husband of Y files a complaint against X for manipulating Y. X&Y
argue that they have their
respective rights to life and personal liberty and hence no one should interfere.
Determine if their situation is in
consonance with the HC ruling given in the above passage.
(a) X & Y are well within their rights to live together and no one should
intervene.
(b) X & Y have exceeded their right to personal liberty because what they are doing
is not lawful.
(c) X and Y are living together with their own free will and the FIR shall be
quashed since there is no
manipulation by X.
(d) X and Y should have divorced their former partners before marrying each other.
91. Suppose in the previous question, suppose X and Y don’t marry but enter into a
live-in relationship, would that
change your answer to the previous question?
(a) Yes, X & Y are well within their rights to enter into such arrangement.
(b) No, X & Y are well within their rights to enter into such arrangement.
(c) Yes, X & Y have exceeded their rights and their relationship is illegal.
(d) No, X & Y have exceeded their rights and their relationship is illegal.
92. Jai and Arjun were two adult boys who were in love. However, the rules of their
local custom prohibit same sex
relationship. Now, since they were deeply in love, they ran away from their houses
to live together in a private
flat. When their families came to know about this, a case was filed against the
boys for running without consent
of their families. Also, they were accused of violating the local customs. Decide.
(a) They have committed an offence, because same sex relationship is prohibited in
their custom.
(b) They have not committed an offence, since they have complete autonomy of their
love life.
(c) They have committed an offence, since they ran away without the consent.
(d) They have not committed an offence, since they are adults and same sex
relationship is not illegal.

. Page 25 of 40
93. Suppose in the previous question, Jai and Arjun started living in a flat
without providing genuine identity to the
owner. They did this in order to stay away from violence and harm which their
community may inflict on them.
It is also quite apparent that their families were so angry that they could have
shot these boys. Will this change
your response, to the previous question?
(a) Yes, because they have breached the law by escaping without the consent of
their families.
(b) No, because they are adults and they know what they are doing.
(c) Yes, because they have breached the law by being in a same sex relationship
without giving genuine contact
details/identity.
(d) No, although they did provide a false identity to stay together, it does not
dissolve the fact that their
relationship per se is legal.
94. A (girl) and B (boy) were in love, who have decided to marry each other. They
knew that their parents won’t
allow this marriage, since they both belonged to different communities. Hence, they
ran away to marry. After
marrying, B started forcing A to embrace his religion. However, A denied the same.
Aggrieved by this, B started
subjecting A to violence. When A’s family came to know about this, they filed a
complaint against B. B, in his
defence, stated that A came here in her will and this is their private matter.
Decide.
(a) B has not committed a wrong, as A came with B in her right state of mind and
sweet will.
(b) B has committed a wrong, since A never consented to the violence.
(c) B has not committed a wrong, as they are adults and they will sort their
issues.
(d) B has committed a wrong, as religious conversion is against the law.
Passage (Q.95-Q.98): Needless to mention that bail is a rule and its rejection is
an exception. Bail in non-bailable
offence cannot be refused without assigning strong reason although bail is a
discretion of the court and discretion
of bail cannot be exercised arbitrarily while in bailable offence it’s a matter of
right. The allegedly committed
non- bailable offence attributed to the applicant does not carry life imprisonment
or death penalty debarring this
court to exercise discretion of bail in favour of the applicant. As such, this
court has a sufficient reason to exercise
discretion of bail in favour of the applicant in case of offence punishable with
life imprisonment or death
penalty... without going into the depth of the merits of the case, the instant
application is allowed and the accused
person is admitted to interim bail provided he will furnish surety and personal
bonds before the SHO concerned.
(Extracted with requisite revisions and edits from ‘Jammu and Kashmir Court grants
interim bail to man accused
of 'promoting enmity' for comment on non-local officials’ at
https://www.barandbench.com/news/litigation/jammu-and-kashmir-court-bail-man-
accused-promoting-enmity#non-local-officials)
95. During an interaction with the local Kashmiri populace, Mr. Sufiyan said 'I can
have expectations from you
because you are Kashmir-based and you can understand us. I can grab you by the
collar and seek answers. But
what expectations can I have from officers who are outsiders?’ He was booked under
Section 153A, a bailable
offence for promoting enmity between the groups. His bail application was rejected
as the allegations were
considered serious enough to dismantle the peace in the valley. Is the denial of
bail justified?
(a) Yes, Mr. Sufiyan promoted enmity between the groups by his address to the
Kashmiri polulace.
(b) Yes, Bail is the discretion of the court and can be denied by the court.
(c) No, bail has to be the rule and should only be rejected as an exception and
it’s his right to get bail.
(d) No, bail has been denied arbitrarily by the Court.
96. Mr. Sufiyan was also charged under some non-bailable offence after a few days.
After some days interim bail
was granted to Mr. Sufiyan but he was unable to furnish the requisite security and
was hence kept within the
confines of the judicial custody. Is keeping him within the confines of judicial
custody justified?
(a) Yes, Mr. Sufiyan failed to provide with requisite security as required by the
Court and can thus be kept into
custody.
(b) No, bail is a rule and its rejection should be an exception.
(c) Yes, offence for which Mr. Sufiyan is charged off is serious in nature and thus
security should mandatorily
be furnished.
(d) No, keeping Mr. Sufiyan within the confines of the custody is a harsh measure
adopted by the court.

. Page 26 of 40
97. A court clerk, Taarak was arrested for bribery which he had accepted from Mr.
Sufiyan to deliver a note to his
family. Bribery is a bailable offence however when application for bail was filed,
the same was rejected as
according to court if the people of court are involved in such activities, it would
portray the justice system under
poor light and hence the court has to exercise it discretion in denying the bail as
making the exception of rejection
a rule. Is the reasoning of the Court correct?
(a) Yes, the act of Taarak portrays the justice system under poor light and hence
his bail should be denied.
(b) No, as bail should be a rule and its rejection should be an exception as the
offence is a bailable offence so is
a matter of right.
(c) Yes, Court has authority to exercise discretion under such circumstances of
offences involving bribery.
(d) No, bail should always be granted by the court as it would otherwise be against
the constitutional mandate.
98. In the aforementioned question, after much arguments and counters, the bail
application was accepted with the
requirement of personal bond and 3 sureties. He was ready to furnish a personal
bond, however he could manage
only 2 sureties as he was new to the State with his relatives and acquaintances
from outside the State. The court
hence denied his application. Can his bail application be denied on such a ground?
(a) No, bail should be accepted as a rule and its rejection an exception.
(b) No, the special circumstances revolving around Taarak should be taken into
consideration by the Court.
(c) Yes, the requirement of furnishing sureties has not been complied by Taarak and
thus the bail application
cannot be given effect to.
(d) Yes, the court has the discretionary power to decide the fate of the bail
application.
Passage (Q.99-Q.102): Vadipetty Police had booked the petitioner accused who went
for a sight-seeing trip
along with his family for offences under Sections 120B [Criminal Conspiracy], 122
[waging war against the
state], and 507 [Criminal Intimidation by Conspiracy] of IPC.
He was also booked for Section 505(1) (b) of public mischief for posting the
pictures with the contentious caption
on his Facebook page. However, Section 505(1) (b) prescribes the punishment for:
"Making, publishing, or
circulating any statement, or rumour, or report, with intent to cause, or which is
likely to cause, fear or alarm to
the public or to any section of the public whereby any person may be induced to
commit an offence against the
State or against the public tranquillity."
Provided that it does not amount to an offence, within the meaning of this section
when the person making,
publishing or circulating any such statement, rumour or report, has reasonable
grounds for believing that such
statement, rumour or report is true and makes, publishes or circulates it in good
faith and without any such intent
as aforesaid.
Court's Observations: For penal provisions like Section 399 and Section 122 of IPC,
even the preparatory stage
for the commission of a crime can be made an offence but the provision has to be
strictly construed and
preparation must meet a higher threshold. About Section 122 of IPC which the
petitioner was charged with, the
court further states that waging war against the state requires crossing certain
stages. Mobilisation of men and
accumulation of arms and ammunition through a concerted effort are instances of
preparing to wage war against
the state. However, all of such must meet the higher threshold proscribed by the
strict construing of the penal
statute to the peculiar facts of the case, notes the court. About the conduct of
the petitioner, the court noted that
the petitioner merely took photographs of a scenic place during a family trip with
an intended-to-be funny
caption. "No weapon or prescribed material was recovered from the petitioner. The
petitioner neither intended
to wage war nor did he commit any act towards preparation therefore he is
acquitted."
https://enalsar.informaticsglobal.com:2278/news-updates/right-to-be-funny-madras-
high-court-quashes-fir#over-humorous-facebook-post-188121 Live Law dated 21/12/21

. Page 27 of 40
99. X was a news reporter. During the winter session, the parliament passed a bill
which if got consent of the
president will turn the entire country into a communist society. X posted this news
on his Facebook page, which
spread like wildfire. People, fearing that their private properties might be
seized, they walked down the streets,
protesting violently. Determine liability of X as per section 505 (1) of IPC.
(a) X is liable because his Facebook post led to the riots.
(b) X is liable because his post led people to commit offence against the state.
(c) X is not liable because he wrote the post simply because he was a journalist.
(d) X is not liable because he did not have the intent to cause violent protests.
100. Suppose in the previous question, X was not a journalist and just some
landlord of Chandigarh who posted the
news from his twitter account. With the rest of the facts remaining the same, would
your answer to the previous
question change?
(a) Yes, X is liable under section 505(1).
(b) No, X is liable under section 505(1).
(c) Yes, X is not liable under section 505(1).
(d) No, X is not liable under section 505(1).
101. Y was a Japanese historian; he was working on a particular contention about
the independence of India. When
he completed his research, he published that research paper in the magazine of the
Central University. Someone
read that research paper and made it viral on Instagram, which led to violent
protest outside the Indian Parliament.
Determine the liability of Y.
(a) Y is liable because the publication has incited violent protests.
(b) Y is liable because it was obvious to acknowledge that the publication can turn
people against the
government.
(c) Y is not liable because he did not have the intention to incite violence.
(d) Y is not liable because he not an Indian national and hence provisions of IPC
won’t be applicable on him.
102. Suppose in the previous question, Y was an Indian Historian but was pursuing
his PhD from the central
University of Japan. With the rest of the facts remaining the same, would your
answer to the previous question
change?
(a) Yes, Y is liable under section 505(1).
(b) No, Y is liable under section 505(1).
(c) Yes, Y is not liable under section 505(1).
(d) No, Y is not liable under section 505(1).
Passage (Q.103-Q.105): A Delhi Court has framed charges against five men in a case
concerning the North East
Delhi riots observing that they were part of an armed riotous mob which had fired
at three public witnesses with
an intention to kill. Additional Sessions Judge Amitabh Rawat framed charges
against Mohd.Juber, Mohd.
Aamir, Samshuddin, Mohd. Barik and salman under Sections 147 (rioting), 148
(rioting, armed with deadly
weapon), 307 (attempt to murder) read with Section 149 (unlawful assembly) of IPC.
However, the Court
discharged them for offences punishable under Sections 336 (act endangering life or
personal safety of others)
and 120B (criminal conspiracy) of IPC.
An assembly of five or more persons is designated an “unlawful assembly”, if the
common object of the persons
composing that assembly is to commit any offence.
.The said accused persons themselves did not fire nor was pistol or bullet
recovered in the present case. As far
as charge u/s. 120 B IPC is concerned, there is nothing on record to show prior
meeting of the minds for the
purpose of criminal conspiracy but there was an armed unlawful assembly of riotous
mob consisting of accused
persons and which had fired at the public witnesses with an intention to kill them,
for the offence under section
120 B of IPC pre meeting of mind is most important rather than other facts." the
Judge said.
The Court was of the view that there was a firing by the said riotous mob
consisting of the accused persons in
prosecution of their common object to do rioting and to kill three witnesses.

. Page 28 of 40
"Thus, accused persons are liable u/s. 149 IPC but discharged of the offence
punishable u/s. 120B IPC," the
Court added. The development came in FIR 116/2020 registered at Jafrabad police
station under Sections 147,
148, 149, 336, 120B and 307 of IPC.
https://enalsar.informaticsglobal.com:2278/news-updates/delhi-riots-four-men-part-
of-armed-mob-that-fired#with-intention-to-kill-charged-unlawful-assembly-187694
Live Law 15/12/21
103. P, Q, R, S and T were thieves and used to collectively loot shops and
apartments. One day P discovered a
jewellery shop, with negligible security; since stealing from that shop would
essentially mean a jackpot, P
thought that he will call his friends and surprise them. When everyone assembled
with their respective guns, P
said that they will only loot the cash drawer, but he had in his mind to loot the
entire shop, which he did not
disclose to anyone. When they barged inside the shop, 4 security guards came out of
the corners, completely
bamboozling them. Q panicked and shot them all. Determine their liability.
(a) They are liable under section 149, because it was an unlawful assembly.
(b) They are liable under section 120B, because there was a prior meeting of mind
between the accused.
(c) They are liable under both section 149 and 120B because they satisfy all the
ingredients therein.
(d) They are not liable under any of the sections because the ingredients are not
satisfied.
104. X and Y filed an application for renewal of their shop license but that was
rejected by the Circle Office of that
district. In order to protest the arbitrary rejection, they went to CO’s residence
and stated shouting at him to come
out of his house, but instead the security guards came out and threatened them to
go away. X in the heat of the
moment, picked up a flower pot and threw it inside the gate which unfortunately hit
one of the guards, thereby
killing him. In the light of the facts, choose the best option
(a) They are liable under section 120B, because they both decided to go to CO’s
house together.
(b) They are liable under section 149 because they have caused death of a guard.
(c) They are liable under both section 120B and 149, because the ingredients are
satisfied.
(d) They are not liable under any of the sections.
105. Suppose in the previous question, X and Y decided to keep pistol with them so
that if required can use it. And
later when they were being asked to go away, Y fired that pistol in the air, which
unfortunately hit the guard and
killed him. With the rest of the facts remaining the same, would your answer to the
previous question change?
(a) Yes, they are liable under section 149.
(b) Yes, they are liable under section 120B.
(c) No, they are not liable under either section 149 or 120B.
(d) No, they are liable under section 149.

. Page 29 of 40
SECTION - D: LOGICAL REASONING
Passage (Q.106-Q.110): Crypto markets have suddenly mushroomed after bitcoin
emerged on the scene. Since
then, many such assets have emerged with varying degrees of financial engineering.
Nearly all cryptos are based
on blockchain, but all blockchain is not crypto. The fact that the RBI sandbox
permits innovation in blockchain
only proves this point.
Issues involving cryptos can be seen at three levels, each of which is equally
important. The first is its impact on
sovereignty. Second is its interaction with financial markets and third is the
value proposition (VC) that the entire
concept of crypto brings to the economic debate.
Commentators and experts who have observed trends in artificial intelligence have
predicted that the algorithmic
world that will emerge in coming years will stress the very concept of the nation
state that emerged from the
Treaty of Westphalia. Blockchain technology and by extension crypto are important
components of this virtual
world. Some of the variants of cryptos such as the stable coin clearly indicate
that these are attempts to create
systems of money that incorporate features of price stability that imply a parallel
monetary system. Thus,
unrestricted co-opting of VC clearly dilutes the sovereign function of money
creation, clearly impacting the
revenues of RBI. Concerns pertaining to money laundering, terrorist threats and
narco-trading also come under
this category given the high value and anonymity offered by crypto currencies.
Once any decision is taken to co-opt the supposed innovation, the question will
arise on how it interacts with the
formal system. As of now cryptos have been recognised as assets or commodities and
as a medium of exchange.
Their role as units of account or legal tender is rather limited. They may offer a
store of value given their short
supply. From a banking point of view, certain issues do arise. Since VCs are not
legal tenders, they cannot be
used in the discharge of debt. Thus, banks cannot accept VCs to close a loan
account. Second, can banks lend in
fiat by accepting VCs as collateral assuming the VC is an asset? If this is so,
what should be the capital
requirement if VCs are accepted as collateral? What will be the impact on the
bank’s non-maturing liabilities if
there is a flight of deposits towards VC?
At a deeper level, the very idea of VCs and the way they are designed are
incompatible with the fractional system
of banking. The fluctuations in interbank liquidity require that money supply
adjusts to system requirements. If
money supply undergoes compositional change in favour of VCs, this ability will be
curtailed thus accentuating
the crisis.
106. ‘Nearly all cryptos are based on blockchain, but all blockchain is not
crypto.’ Which of the following leads to a
valid conclusion?
(a) Some blockchains are not cryptos. (b) Some blockchains are cryptos.
(c) Either statement I or statement II. (d) IV All blockchains are definitely not
cryptos.
107. All of the following are inferences, except
(a) Bitcoin is one of the assets of crypto currency.
(b) Cryptocurrency works on artificial intelligence.
(c) Cryptocurrency has collateral impact on an economy.
(d) Cryptocurrency does not come without issues.
108. ‘Some of the variants of cryptos such as the stable coin clearly indicate that
these are attempts to create systems
of money that incorporate features of price stability that imply a parallel
monetary system.’ The aforementioned
statement reflects which of the following?
(a) Inference (b) Fact (c) Judgement (d) Assumption

. Page 30 of 40
109. According to the passage, what are the limitations of Cryptocurrency?
(a) Cryptocurrency is not a legally accepted mode of payment; therefore, it cannot
be used as a clearance of a
debt or closing of a loan account by the banks.
(b) Cryptocurrency is in the process of acquiring a legal recognition after which
it will create a parallel monetary
system.
(c) Cryptocurrency is not a legally accepted mode of payment, which can accelerate
money laundering, terrorist
threats and narco-trading.
(d) Cryptocurrency is an evanescent virtual fad that will lose its sheen in the
coming years.
110. Which of the following questions are pertinent to the passage with regard to
issues concerning cryptocurrency if
some information in the passage is to be considered as an answer?
(a) What should be the capital requirement if cryptocurrencies are accepted as
collateral?
(b) What will be the impact on the bank’s liabilities and assets if cryptocurrency
exists as a parallel monetary
system?
(c) What will be the impact on existing investments in mobile payment and UPI
technology?
(d) Do we have data to support the view that investments in cryptocurrency are
permeating all households,
specifically the vulnerable ones?
Passage (Q.111-Q.115): The IMF’s latest World Economic Outlook sums up the
challenges ahead in the report’s
title: ‘A long and difficult ascent’. With COVID-19 having already extracted a toll
of over a million lives,
accompanied by an evisceration of livelihoods and output in economies,
prognosticating the economic future
even as the pandemic rages on is an unenviable task. The Fund’s economists have
gamely sought to make
forecasts for world output through 2020, 2021 and into the medium term. While the
global economy is projected
to shrink 4.4% this year, reflecting a less severe contraction than the 5.2% drop
estimated in June, output is seen
rebounding at a marginally slower 5.2% pace in 2021. The IMF has based its revision
on “better-than-anticipated
second-quarter GDP out-turns, mostly in advanced economies” where activity improved
after lockdowns were
eased, as well as signs of a stronger recovery in the July-September quarter. But
the IMF has been prudent in
pointing out that even as the world economy ascends out of the depths it plunged to
in April, following the
worldwide lockdown, there remains the danger of a resurgence in infections that is
prompting countries in Europe
to reimpose at least partial closures. And the risks associated with predicting the
pandemic’s progression, the
unevenness of public health responses, and the extent to which domestic activity
can be disrupted, magnify the
uncertainty.
Pointing out that the pandemic is set to leave scars well into the medium term ‘as
labour markets take time to
heal, investment is held back by uncertainty and balance sheet problems, and lost
schooling impairs human
capital’, IMF Chief Economist Gita Gopinath contends that global growth will
gradually slow to about 3.5% in
the medium term. With the cumulative loss in output relative to the pre-pandemic
projected path estimated to
more than double to $28 trillion over 2020–25, efforts to improve average living
standards are certain to be
severely set back. Observing that the pandemic is set to widen inequality between
economies and within nations,
the Fund has urged greater international cooperation. It is imperative for all
countries to work closely to ensure
that new treatments and vaccines are made available to all since wider and faster
availability of medical solutions
could boost global income by almost $9 trillion by end-2025, reducing income
divergence, she says. With no
visibility yet on vaccine availability, the IMF has also stressed the need for
policymakers to persist with direct
income support for the most vulnerable and regulatory forbearance for stressed but
viable firms. The message is
clear. In a world as interconnected as it is today, the cost of economic insularity
would only be more protracted
pain for all.

. Page 31 of 40
111. “With COVID-19 having already extracted a toll of over a million lives…………is
an unenviable task.” What
can be concluded from the given statement with regards to IMF’s forecasting?
(a) Though IMF in its report has summed up the future economic growth with the
ongoing COVD-19 as a
prolonged and difficult rise, forecasting about future economic growth with no
immediate respite is not an
endeavour that most would like to take on.
(b) Forecasting future economic growth during unrelenting COVID-19 is a task that
only IMF can do with an
admirable precision.
(c) Though IMF in its report has summed up the future economic growth with the
ongoing COVD-19 as a
protracted and difficult rise, forecasting about future economic growth with no
immediate respite is an
endeavour that most would like to take on.
(d) The uncertainty of the future economic growth based on the present markers
makes future economic
predictions a herculean task even for an organization as mammoth as IMF.
112. The IMF’s prediction of the marginally slower pace of 5.2% pace in 2021 is
based on the assumption that
(a) The rebound pace of the recovery in July to September quarter spills over to
the next quarter.
(b) The rebound pace of the recovery in July to September quarter will be less in
the next quarter.
(c) The rebound pace of the recovery in July to September quarter will be way more
in the next quarter.
(d) None of these.
113. What is the primary purpose of the passage?
(a) To highlight the measures taken by IMF in bringing about more economic
sustainability around the world.
(b) To elucidate IMF’s prognosis on economic outlook from 2020 till 2025.
(c) To bring to fore the plunging numbers of the global growth by the IMF and its
future implications on the
world economies.
(d) To rebuff the IMF’s forecast of 2021 global growth by clearing the
misconception.
114. The message is clear. In a world as interconnected as it is today, the cost of
economic insularity would only be
more protracted pain for all. We can infer from the statement that
(a) For world economies to work together towards finding a possible solution to
Covid-19.
(b) Economic and social peripherality by the economies of the world will lead a
sustainable global economic
growth.
(c) An inclusive approach by the world economies to avoid a delay in getting back
on their feet.
(d) Having a narrow provincial viewpoint of isolated economic welfare in the
globally interconnected world will
impede sustainable economic growth world-wide.
115. The message is clear. In a world as interconnected as it is today, the cost of
economic insularity would only be
more protracted pain for all. What possible remedy could be deduced out of the
given statement?
(a) To adopt the policies of the developed world by the developing ones to insure
minimum economic damage.
(b) To be more exclusive with a vision to be wary of drastic and impulsive
policies.
(c) A call for more international cooperation as the world economy recovers slowly.
(d) To persist with indirect income support for the most vulnerable and regulatory
forbearance for stressed but
viable firms.
Passage (Q.116-Q.120): Preliminary results of Sunday’s Presidential election in
Bolivia point to an emphatic
victory for the former President Evo Morales’s Movement Toward Socialism (MAS)
party almost a year after
he was ousted by protesters and the military. While the official results will not
be announced for days, exit polls
and independent counts give his hand-picked candidate, Luis Arce 53% of the popular
vote against his main rival
Carlos Mesa’s 29.5%. Mr. Mesa, who was President between 2003 and 2005, has
conceded the election. This is
as much a victory for Mr. Morales, Bolivia’s first indigenous leader, as it is for
Mr. Arce. Mr. Morales left the
country, first for Mexico and then for Argentina, after the Generals asked him to
stand down in November 2019.
Since then, Jeanine Añez, a right-wing conservative Senator, has been the acting
President, rolling back many

. Page 32 of 40
of Mr. Morales’s policies and going after his supporters. But MAS mobilised its
supporters and fought back. Mr.
Arce promised voters that if elected, he would carry forward the pro-poor socialist
legacy of Mr. Morales, while
MAS portrayed Mr. Mesa as a representative of the pre-Morales elites, whose rapid
privatisation and pro-market
policies had triggered frequent mass protests. The results demonstrate that the
voters chose the equity-oriented
socialist stability, which MAS offered, over the free-market conservatism of its
rivals.
The new leader could learn from the achievements and mistakes of Mr. Morales,
credited with turning around
South America’s poorest country economically. Under his government, Bolivia saw a
drop in extreme poverty,
from 33% of the population in 2006 to 15% in 2018. He also stepped-up public
investments, opened more schools
and health clinics, built roads and nationalised the oil and gas industry, all the
while ensuring that the economy
continued to expand. These policies helped MAS build a strong connect with the
poor, a base which continued
to back the party despite last year’s political turbulence. At the same time, his
push to stay in power beyond the
term limits set by the Constitution helped the Opposition organise itself. He got
the ban lifted by a constitutional
court after his bid failed in a referendum. This raised questions about the
legitimacy of his candidacy in the
October 2019 election, which he won but was accused of fraud, leading to protests
and his ouster. Mr. Arce’s
biggest challenge would be to continue Mr. Morales’s welfare policies, while
keeping the battered economy on
track. Moreover, the anti-Socialist and mostly white opposition is now more
powerful, after ousting Mr. Morales
and having run the interim government for a year with the U.S.’s support. Mr. Arce
can keep the galvanised
opposition at bay only by continuing MAS’s socio-economic “revolution” to expand
its support base in a divided
country.
116. The write up about the result of the Presidential election in Bolivia is based
on the assumption that
(a) Mr. Morales conceded defeat even before the election polls.
(b) Exit polls and independent counts are heralds of the actual results.
(c) Exit polls and independent counts have a hundred percent accuracy record.
(d) Exit polls and independent counts can precipitate certain impressions in the
minds of the voters towards a
presidential candidate.
117. We can conclude from the given passage that
(a) Mr. Morales will come for another term as the President of Bolivia, assuming
the exit polls to be the true
indicator of the actual results.
(b) Luis Arse will become the President of Bolivia, assuming the exit polls to be
the true indicator of the actual
results.
(c) Carlos Mesa will become the President of Bolivia, assuming the exit polls to be
the true indicator of the
actual results.
(d) The exit polls cannot be considered as true marker of the actual polls and the
victory can go either side.
118. We can infer from the passage that
(a) Luis Arce is the face of the MAS party where the actual victory is of Mr.
Morales.
(b) Luis Arce represents the true leader of the MAS party backed by the former
President Mr. Morales.
(c) Luis Arse won hands down against Carlos Mesa who conceded defeat even before
the actual results.
(d) Luis Arse will face stiff competition from the opposition with regards to his
staunch policies against the
elitist.
119. What further logical conclusion can be drawn going by the writer’s stance?
(a) The road ahead for the winning party is a smooth sail if it represents anti-
socialist movement.
(b) The winning party will have to appease the poor voters in order to win the next
election.
(c) The winning party has a tough journey ahead as it not only has to replicate the
socio-economic model but
also to yield no opportunity to the opposition to find any chance of a usurp.
(d) None of these.

. Page 33 of 40
120. Which of the following will strengthen the author’s stance regarding MAS
party?
(a) MAS Party was able to complete full term after winning the elections without
continuing Mr. Morales legacy.
(b) MAS Party continued to expand its support base with the soliciting elite votes.
(c) MAS Party strengthened its ties with the neighbouring countries with respect to
bilateral trades ensuring
foreign investments to strengthen the economy.
(d) MAS party invited many privatized sectors to ensure a better infrastructure,
thereby, making Bolivia an
attractive destination for foreign trade.
Passage (Q.121-Q.125): India’s poor progress on nutritional indices must dispel the
hubris surrounding strong
economic growth for years, and turn national focus on persisting hunger, wasting
and stunting among children.
This year’s Global Hunger Index (GHI) places India in the company of Sudan — rank
94 among 107 countries
— with the unedifying assessment of the national situation as “serious”. The
country’s score of 27.2 is the worst
among BRICS countries, and inferior to Pakistan, Sri Lanka, Bangladesh and Nepal.
The GHI is developed
around wasting and stunting (under five), the share of the population with
insufficient caloric intake, and child
mortality. Index scores by international agencies have been critiqued as flawed for
choosing the wrong weights
in scoring, and also for not including genetics and social determinants, but that
would be a narrow view. The
evidence from the NFHS-4 of 2015-16 is not very different. The reality is that
national policy has no appetite
for a radical transformation in the delivery of adequate nutrition especially to
women and children, and has paid
inadequate attention to achieving diet diversity through the PDS. On the other
hand, the country is widely seen
as falsely equating energy calories with a diverse diet. The existing deprivation
has been aggravated by the
pandemic, with food inflation putting pressure on depleted incomes or meagre
pensions and savings.
The NFHS-4 found that under-five stunting from chronic undernourishment stood at
38%, and wasting, a result
of acute lack of nutrition, at 21%. These data represent some progress, at a drop
of about 10 percentage points
in both categories compared to a decade earlier, although steady economic
prosperity should have yielded a far
bigger social dividend. The latest GHI measure is a reminder that much work is
necessary to bring the true
benefits of the National Food Security Act to the unreached, not merely as hunger
mitigation through cereals,
but as nourishment through a diverse diet that includes fat, protein and
micronutrients. But there are worrying
indications that the Centre has chosen the wrong course. In its pursuit of fraud
within the PDS, it is inclined to
take hard measures that would deprive the disabled and the elderly of even cereals,
by insisting on biometric
verification to get supplies. Strengthening the PDS, with a focus on women’s
health, would lead to healthier
pregnancies, and stronger supplemental nutrition under the ICDS scheme would give
children a better chance at
all-round development. The importance of sustained, immediate intervention is
further underscored by recent
findings of International Food Policy Research Institute scholars that three out of
four rural Indians cannot afford
a balanced, nutritious diet. The right to food would be meaningless if it leaves a
large section of Indians hungry,
stunted and wasted.
121. The first four lines of the passage suggest which of the following?
I. Nutritional indices are an important aspect of economic progress.
II. Nutritional indices are counter indicative of the perceived economic growth of
India.
III. Sudan has a poor representation of nutritional indices according to Global
Health Index (GHI).
(a) Only I follows (b) Only II follows (c) II and III follow (d) All follow.
122. This year’s Global Hunger Index (GHI) places India in the company of Sudan —
rank 94 among 107 countries
— with the unedifying assessment of the national situation as “serious”. What is
the logical explanation of the
inclusion of the following statement in the passage?
(a) The statement is the assumption on which the entire passage stands.
(b) Global Hunger Index (GHI) is a serious enough marker to draw the writer’s
attention to the plight.
(c) The statement is a rhetoric to bring to focus the ineptitude of the current
governance.
(d) Global Hunger Index marker is not a correct representation of India’s position
with respect to Nutritional
Indices.

. Page 34 of 40
123. Which of the following would most undermine India’s score as worst amongst the
BRICS countries?
(a) If the respective scores of the countries Pakistan, Srilanka, Bangladesh and
Nepal were less than India.
(b) If the respective scores of the countries Pakistan, Srilanka, Bangladesh and
Nepal were more than India.
(c) If the score 30 were represented as one of the best scores with regards to
Nutritional indices.
(d) If the score 0 were represented as one of the worst scores with regards to
Nutritional indices.
124. If the above contents in the passage are considered to be true, then what is
the likely consequence of stunting
and wasting in children under five?
I. Undernutrition and malnutrition would lead to high child mortality.
II. Wasting and Stunting in children under five will result to chronic nutritional
deficiencies leading to largely
unproductive adult.
III. The stunted and wasted children under five are the outcome of under nourished
pregnant mothers.
(a) Only II (b) I and II (c) II and III (d) Only III
125. We can conclude from the passage that Stunting and Wasting can be remedied
through
(a) A concerted effort by the government with a focus on providing supplemental
diet consisting of Fat, Proteins
and Micronutrients as compared to a narrow focus on hunger relief.
(b) A consistent supply of cereals to the under nourished mothers and children
through a vigilance on biometric
verification.
(c) Strengthening the National Food Security Act by making necessary amendments.
(d) A consolidated effort by the government to focus only on women’s health and
pregnancy thereby preventing
stunting and wasting at the source itself.
Passage (Q.126-Q.130): Since the Supreme Court’s highly controversial judgment in
the ADM Jabalpur case
in 1976, whereby it was held that during the Emergency the right to life and
personal liberty of a citizen under
Article 21 of the Constitution would remain suspended, the judiciary has not only
come under critical scrutiny
but also under attack. The judgment has since been overruled, but its reversal goes
to prove the fact that judges
are not infallible, and that “to err is human”. Recently, the Chief Justice of
India, in his own mild way, protested
against the attack on judges. One can understand his pain and agony, but he too
knows that judges do not, and
should not live in ivory towers. They sit in open court, speak out under the public
gaze and deliver judgments
that come into the public domain. As the judiciary is one of the pillars of
democracy, and the Constitution entrusts
judges with the task of protecting the constitutional rights of the people,
especially the right to life and liberty,
the consumer of justice has every right, and would be fully justified in critically
examining, and commenting
upon each and every word of the judges spoken or written, howsoever unpalatable it
may be. Of course, none
has the right to make personal attacks on judges.
126. All of the following can be assumptions, except
(a) The reader may not be aware that right to life and personal liberty is about
Article 21 of the Constitution.
(b) Supreme Court’s decisions can come under scanner.
(c) Supreme Court’s judgments can be overruled.
(d) The Chief justice of India is entrusted to shield the judges from attacks.
127. One can understand his pain and agony, but he too knows that judges do not,
and should not live in ivory towers.
The author puts forth the argument
(a) as a counterargument.
(b) as an assertion.
(c) as a repudiation to the protest by the Chief Justice of India.
(d) as an affirmation.

. Page 35 of 40
128. Which of the following can be inferred from the passage?
(a) The judges can falter in their judgments.
(b) The judgments given by the judges are irrevocable.
(c) The Supreme Court gave a highly controversial judgment in the ADM Jabalpur case
in 1976.
(d) The Chief Justice of India has no say in the working of the Supreme Court.
129. The author would most agree with which of the following verdicts of the bench
in favour of the reversal of the
decision of the Supreme Court with regard to the ADM Jabalpur case in 1976?
I. Civilised state can contemplate, in many events, an encroachment upon life and
personal liberty without the
authority of law.
II. Life and personal liberty are alienable to human existence.
III. The right to life has existed even before the advent of the Constitution. In
recognising the right, the
Constitution does not become the sole repository of the right.
(a) Only I. (b) Only II (c) Only III (d) II & III
130. What is the main idea behind the passage?
(a) The author through the passage highlights the weakness of the judicial system,
and appeals for an infallible
judgement.
(b) The author through the passage highlights that the judgements of Supreme Court
are not water tight and
consumer of justice is fully justified in critically examining the judiciary.
(c) The author through the passage berates the protest of the Chief Justice of
India, and point out that his protest
as unreasonable.
(d) The author through the passage addresses the judges to come out of their ivory
towers and be mindful of the
judgements.
Passage (Q.131-Q.135): While the pandemic has forced many in the confines of their
homes, the worst hit are
the elderly who are devoid of any human contact and without any contact in the real
world. Among others, health
care, medical services and care for the elderly is something that is becoming hard
with times like these. So, what
is it that can bring a drastic change in the life of elderly; how can ageing be
better with smarter technology? Let
us discover.
Big data and artificial intelligence (AI). With the amalgamation of these two
technological advanced tools, it is
possible now to usher in personalized medicine, treatments that are tailor made for
individual, and all this brings
us to revolutionize the health care system. It brings together all the aspects
including information on the lifestyle
of the individual, biological aspect and clinical aspect all these help in creating
a unique blueprint of the
individual or the patient. It becomes easier to begin with target therapies for
achieving the best outcome whether
it is about prevention or to manage the disease. Early detection of disease becomes
significantly easier that
provides the medical practitioner with the power to easily diagnose and provide
accurate treatment for the same.
Life threatening diseases then can be handled in a better manner like the onset of
cancer and Alzheimer’s.
Monitoring the home of the elderly, providing timely medical supplies and help,
smart heating devices or
temperature control and other such things makes it easier for the ageing people to
lead a comfortable life.
Helping the aged people deal with the isolation is yet another factor that is
essential to be considered as they are
more at risk than the rest of the population.
The time elderly people were active socially that was pre-pandemic time their
longevity was positively impacted.
They felt healthier and happier that kept them from falling ill frequently, but
with the onset of pandemic things
changed dramatically.
Digital technology is providing a breakthrough in relation to wellbeing for the
elderly. Connecting people with
elderly population and running their errands, maintaining contact with them and
taking on tasks for them,
checking upon them is all made possible. Facilitating real-time interactions helps
in reducing social isolation.

. Page 36 of 40
131. According to the author, big data and artificial intelligence will help the
elderly
(a) By bringing in treatments that are tailor made for individuals.
(b) By including information on the lifestyle of the individual, biological aspect
and clinical aspect all these help
in creating a unique blueprint of the individual.
(c) By early detection of disease becomes significantly easier that provides the
medical practitioner with the
power to easily diagnose and provide accurate treatment for the same.
(d) All of the above.
132. ‘Monitoring the home of the elderly, providing timely medical supplies and
help, smart heating devices or
temperature control and other such things makes it easier for the ageing people to
lead a comfortable life.’ What
is the assumption behind the argument?
(a) The aging people require monitoring on the basic functioning of life.
(b) The aging people are unable to manage the simplest of tasks.
(c) The aging people live alone without anyone monitoring.
(d) The aging people are unable to perform the basic functioning.
133. Which of the following will undermine the contention presented in the passage?
(a) With the Pandemic out of the system, life will resume back to normal, even for
the elderly people.
(b) The aging people need support of the NGOs and only NGOs can make the life
better for the aging people.
(c) The aging people are technology savvy and are able to figure out artificial
intelligence systems.
(d) The ageing people with their weak faculty and fugacious memory are unable to
perform a task as simple as
dialing a number on their mobile.
134. If the arguments in the passage are true, how will artificial intelligence
help in making the ageing people’s life
comfortable?
(a) Using AI, the elderly can be made to learn multiple tasking, thereby easing
their lives and keeping them busy
so that they do not feel isolated.
(b) Using AI with human values, data related to each elderly fed into the system
used for analysis, medical health
care and other services available through smart devices is where technology will
make a viable impact.
(c) Data related to each elderly will be fed into the system that can help in
assisting medical emergencies and
hospital care.
(d) With the help of artificial intelligence, facilitating real-time interactions
will help in reducing social isolation.
135. Which of the following reflects the main idea of the passage in brief?
(a) Ageing reverse with smarter technology. (b) Ageing better with smarter
technology.
(c) Aging made more comfortable. (d) Aging post Pandemic

mock 27
Directions (Q.1-Q.30): Read the following passage carefully and answer the
questions that follow.
Passage (Q.1-Q.5) Speaking at the Royal Television Society’s autumn conference, the
former media minister
suggested that the dominance of global streaming giants such as Netflix, Amazon and
Disney was endangering
the future production of distinctively British content. In future, public service
broadcasters would need to be
legally directed, he suggested, to create programmes embodying a kind of
“Britishness” exemplified by old
favourites such as Dad’s Army or Only Fools and Horses.
As critics such as the historian David Olusoga have pointed out, attempting to
define a supposedly core
Britishness in a multicultural, evolving society would be a highly contentious
exercise. It would also be a futile
one in an industry that is entering a fast-moving and unpredictable golden age.
Startling figures released recently
underline that investment from the streamers is helping to transform the scope,
ambition and range of TV and
film production. Traditional broadcasters such as Sky and the BBC are following
suit, spending more on big#budget content. As the year comes to an end, overall
investment in shows costing at least £1m an episode is two#thirds greater than the
previous record set in 2019 – before the pandemic.
Public service broadcasters have a special responsibility to attend to the state of
the nation and must always be
given adequate resources with which to do that. Small-scale creativity and
innovation must also be protected in
an age of big-budget production. But portraying “Britishness”, in this transformed
landscape, is becoming a
perpetual work in progress. A transatlantic focus on greater diversity has led to
milestone portraits of
multicultural Britain in which, for example, black lives and experience have been
belatedly foregrounded.
Michaela Coel’s I May Destroy You – on the BBC and HBO – and Steve McQueen’s Small
Axe films for the
BBC broke new ground in this regard. Luther, the detective series starring Idris
Elba, also broadcast by the BBC,
is now to be made into a film by Netflix. The hit Netflix drama Bridgerton is more
in the “made-for-global”
category, but – filmed in Bath and produced by a black American – it showcases both
Regency England and a
full range of contemporary British acting talent.
Interviewed in the Radio Times earlier this month, the former Doctor Who actor
David Tennant joined Mr
Olusoga in questioning the government’s desire to protect nebulously defined
“British values” in television
programming. They are right. At this time of new possibilities, proliferating
platforms and cultural cross#fertilisation, the last thing our broadcasters need is
a nostalgic attempt to force the future of high-end British
drama to conform to a narrow, selective version of the past.
1. What among the following is the central argument of the author in the passage?
(a) In the present context, attempting to broadcast Britishness in global streamers
and protecting “British values”
in the TV and film industry would be a contentious exercise.
(b) The “British values” are outdated in the present context and they need to be
phased out systematically.
(c) The filmmakers should look forward to making more films with “British values”
since the number of such
films has decreased drastically.
(d) Public service broadcasters should be given freedom to choose what to broadcast
and promote.
2. Which among the following can be attributed to be the author’s opinion?
(a) The future production of British content has been put into danger by the
dominance of global streaming
giants.
(b) At least companies like Disney, which are British, should strive to promote
more British content on their
platforms.
(c) Trying to coerce the makers in the TV and film industry to make more “British
content” is likely to reap
little benefits.
(d) Actor David Tennant and Mr Olusoga are irrational people since they make claims
without any strong
evidence.
. Page 3 of 40
3. What does the author mean when he makes this statement, “But portraying
“Britishness”, in this transformed
landscape, is becoming a perpetual work in progress.”?
(a) The author suggests that portraying business would reap benefits in the long
run.
(b) The author suggests that there is not much sense in portraying narrow,
selective version of the past British
values in today’s multicultural and dynamic age.
(c) The author believes that gradually, the portrayal of “Britishness” would bring
in positive results.
(d) The author vehemently negates the presence of Britishness in the programmes
shown in digital media, TV
and film industry.
4. Which of the following is most dissimilar to the word ‘proliferate’?
(a) Multiply (b) Mushroom
(c) Dwindle (d) Snowball
5. What role does the penultimate paragraph play?
(a) This paragraph provides evidence to the claim made in the previous paragraph.
(b) This paragraph provides evidence to support the claim made in the last
paragraph.
(c) This paragraph does not contain any pieces of evidence to support or weaken any
claim.
(d) This paragraph provides evidence to weaken the claim made in the previous
paragraph.
Passage (Q.6-Q.10): From conquering polio, or HIV/AIDS, to the elimination of ozone
depleting substances,
the history of humanity is full of success stories for collective action. According
to research by the Oxford-based
‘Our World in Data’ organization, humanity is on average better off today than many
decades ago. Despite the
unprecedented population growth over the past century, we live during one of the
most peaceful, most
progressive, and stimulating eras in history. We are more apt to die from unhealthy
lifestyle choices, suicide, or
old age than from hunger, war, terrorist attacks, or transferrable illnesses. Since
1820, global poverty has been
reduced from 94% to 9.6% in 2015, and global income has increased on average
tenfold with falling global child
mortality rates. Also, literacy has increased—from 12% in 1820 to 87% in 2014—and
most countries are now
ruled by democratic governments. This progress would not have been possible without
massive amounts of
energy, economic globalization, and exponentially growing technologies, all of
which must now become
sustainable. This was all created by the collective application of human curiosity,
innovation, creativity, a sense
of wonder, and purpose. This should give us hope because creating better societies
to ensure the future of life on
Earth despite the grand global challenges can only occur if we believe it can be
done.
To better understand how social transformation could be achieved within the context
of planetary boundaries for
many generations to come, it is important to take a closer look at the underlying
factors influencing it through
the lens of integral theory by Ken Wilber that has been successfully applied in
more than 50 disciplines from
medicine to economics, investing, and business. Explicating integral theory here
would not be that relevant;
however, its roots are embedded in evolutionary theory and in the Platonic values,
the True, the Good, and the
Beautiful. Integral theory can help identify the missing pieces in the current
paradigms that are failing us, and
integrate all of reality, its exterior as well as interior dimensions, such as
culture, emotions, and spirituality.
6. Which among the following has not been discussed in the passage?
(a) What is the importance of Integral theory?
(b) How has social transformation been achieved in the present context?
(c) How has collective work impacted mankind?
(d) What changes can we see at present from a few decades ago?
7. Which among the following words is closest in meaning to ‘paradigm’?
(a) Pattern (b) Antithesis (c) Opposite (d) Contrast

. Page 4 of 40
8. Which among the following can be inferred from the passage?
(a) The factors that have brought massive changes in our lifestyle do not require
any improvement.
(b) Social transformation in the present context has been achieved through the use
of Integral theory.
(c) Population growth has not stalled the massive efforts that have been put to
bring peace in the world.
(d) Integral theory can help identify the missing pieces in the current paradigms
that are failing us.
9. Which of the following reflects the tone of the passage?
(a) Critical and cynical. (b) Nostalgic and reminiscent.
(c) Commiserating and consoling (d) Reassuring and encouraging.
10. Compared to 1820, which of the following has not been on the rise in 2015?
I. Global poverty
II. Income levels
III. Sex ratio
IV. Child mortality
(a) Only I and II (b) Only II and IV (c) Only I, and IV (d) Only I, II and IV
Passage (Q.11-Q.15): Foreign Secretary Harsh Vardhan Shringla was in Myanmar this
week to assess the
ground situation and to nudge various sides to seek mutually acceptable solutions
to the multiple crises marring
one of India’s most important neighbours. New Delhi, of course, has vital interests
in Myanmar that it would
like to protect and enhance. While the West has made democracy the sole prism of
its Myanmar policy, India
doesn’t have that luxury. Much like most other immediate neighbours of Myanmar,
India has been keen to push
back against the Myanmar military’s authoritarian tendencies even as its multiple
interests have ensured that it
keeps its channels of communication open with all stakeholders.
Myanmar has been in turmoil since February when the military seized control of the
country in a coup and
detained Aung San Suu Kyi and other leaders of her National League for Democracy
(NLD). India had been
categorical from the very beginning that the gains made by Myanmar over the last
decades on the path towards
democracy should not be undermined. Earlier this month, after Suu Kyi was sentenced
to four years (later
reduced to two years) in jail for provoking dissent against the military and
breaching Covid-19 rules, New Delhi
responded by expressing its “deep concern” over “any development that undermines
these processes and
accentuates differences” and expressed its hope that “keeping their nation’s future
in mind, efforts would be
made by all sides to advance the path of dialogue.”
But for India, challenges emanating from Myanmar and adjoining areas do not cease
depending upon the
complexion of the government. And direct engagement with the State Administration
Council (SAC) and other
stakeholders can no longer be put on hold. Last month’s deadly attack on an Assam
Rifles convoy near the
Myanmar border in Manipur was a reminder about the proclivity of China for creating
trouble in the Northeast,
especially at a time when border tensions along the LAC remain high. The Covid-19
pandemic has also had an
impact because of the porous border between India and Myanmar. For India, a
humanitarian crisis as a result of
the pandemic-induced economic crisis in Myanmar would be a lose-lose situation and
it should be prevented
with the utmost urgency.
11. Why does India cannot engage with Myanmar only on the basis of whether it is a
democracy or not?
(a) Myanmar had played an important role in making India a democracy.
(b) India has crucial interests in Myanmar it wants to protect and enhance.
(c) Myanmar has always supported India when India has been in trouble.
(d) India has supported Myanmar a lot during bad times, so it will continue to do
so.

. Page 5 of 40
12. Which of the following can reasonably be inferred from the passage?
(a) India is in favour of a democratic government over military rule.
(b) India, the only neighbouring country bordering Myanmar should lend its hands to
it.
(c) The borders between India and all its neighbours are guarded heavily by the
soldiers.
(d) India faces different challenges from Myanmar, depending on the type of
government.
13. What would be a step taken by India which the author would approve of? Choose
the best option.
(a) India engages with the stakeholders of Myanmar in a direct way.
(b) India brings Myanmar to the UN platform to engage democratic discussions.
(c) India allows its citizens to engage directly with the citizens of Myanmar.
(d) India cuts its ties with its neighboring countries, not including Myanmar.
14. Which of the following is true according to the passage?
(a) India is the only democratic country neighbouring Myanmar.
(b) The economic condition of Myanmar is not in a fix.
(c) Myanmar’s decades of efforts have been rendered useless.
(d) None of the above.
15. What is the correct meaning of ‘proclivity’?
(a) Something that is done frequently. (b) Tendency to do something.
(c) Something this is done rarely. (d) Aversion to do something.
Passage (Q.16-Q.20): It is widely acknowledged that the pressing global crises
today are societal rather than
purely environmental issues. Challenges such as climate change and global warming,
the loss of biodiversity, or
the global water crisis call for deep societal transformations. Even the most
adamant natural scientists or
advocates of technological solutions concede that addressing the current challenges
requires societal efforts since
environmental, social, cultural, and economic issues are inextricably interlinked
in today’s crisis.
Despite a high level of consensus on the diagnosis, there is great dispute about
how to initiate the necessary
change towards a more sustainable society. Political top-down strategies have
undeniably had some degree of
success in the past. International climate agreements, for example, set boundaries
for greenhouse gas emissions
and stimulated change in energy supply in many countries of the world. Global
education programs, on the other
hand, brought questions of sustainable development to the classroom and broadened
curricula worldwide.
Yet it has become obvious in recent years that top-down approaches often face
significant obstacles to
implementation and are not sufficient to increase the speed and depth of the needed
societal transformations.
First, because they tend to impose “one size fits all” solutions that discount the
need for culturally and regionally
differentiated pathways towards global sustainability. Second, top-down approaches
often disregard the
knowledge and expertise of everyday actors and ignore their desire for making their
own choices instead of
executing imposed strategies. Transformations towards living sustainably are much
more likely to be accepted
if they are developed jointly by everyday people, specific stakeholders, and
policy-makers at all levels working
together with academic experts and scientists.
A first pillar for pushing forward social transformations is to create (more)
laboratories of change in the public
sphere. Local and regional governments, for e.g., can serve as a model for how to
spark, develop and implement
technological and social innovations at the very scale at which global change
becomes tangible. Education is
another key factor to facilitate change and shape societal transformations.
Educational institutions and
organizations like schools and universities, and also centers for adult education,
or museums promote
understanding of the world and help build capacities for transformative action.
Given the complex nature of
today’s “wicked” problems, however, traditional ways of organizing knowledge must
be called into question
and new forms of teaching and learning need to be developed. A third pillar of
societal transformation is the

. Page 6 of 40
development of a new aesthetic for dealing with the natural and the social world.
Un-sustainable development
is deeply linked to culturally embedded mindsets and resulting daily routines and
habits. How we do things
depends very much on what they signify to us, and how we see the world and our
place in it. The arts in all their
forms can provide novel perspectives on the relationships of humans to the natural
world and to each other, and
help envision and catalyze societal change.
16. According to the passage, the characteristic/ characteristics of a top-down
approach is/ are
(a) It gives people the freedom to use their own strategies.
(b) This approach is not suitable for increasing the speed and depth of social
change.
(c) This approach fails to recognize that there can be different paths to global
sustainability.
(d) Both b and c are correct
17. It can be inferred from the passage that
(a) Environmental issues at the moment need more attention than societal issues.
(b) Attention should be paid to discussions on how to move towards a more
sustainable society.
(c) Solving cultural issues would not help much in solving environmental issues.
(d) The top-down approach to solving societal issues has been grossly unsuccessful.
18. In the given passage, which of the following does the author suggests to push
social transformations?
(a) Giving enough attention to the field of education can help.
(b) The traditional form of education should be done away with.
(c) Little to no attention needs to be given to arts for this objective.
(d) Too much attention has already been given to social transformation.
19. Consider the following statement, “Global education programs, on the other
hand, brought questions of
sustainable development to the classroom and broadened curricula worldwide.” What
role does it play in the
passage?
(a) This statement supports the main idea of the passage
(b) This statement supports the claim that top-down approach is unsustainable.
(c) This statement weakens the author’s claim that top-down approach has achieved
success.
(d) This statement is a complement to the previous two suggestions made in the
passage.
20. Which of the following mean the same as ‘aesthetics’?
(a) Artistic. (b) Comeliness (c) Creative (d) All of the above
Passage (Q.21-Q.26): Most people think of Sigmund Freud as a psychologist or a
psychiatrist. But he was
neither. He was trained as a neuroscientist and went on to create a new discipline
that he called ‘psychoanalysis’.
But Freud should also be thought of as a philosopher – and a deeply insightful and
prescient one at that. As the
philosopher of science Clark Glymour observed in 1991:
Freud’s writings contain a philosophy of mind, and indeed a philosophy of mind that
addresses many of the
issues about the mental that nowadays concern philosophers and ought to concern
psychologists. Freud’s
thinking about the issues in the philosophy of mind is better than much of what
goes on in contemporary
philosophy, and it is sometimes as good as the best …
For neuroscientific researchers, the daunting scientific challenge of figuring out
how the brain works (without
the benefit of the sophisticated technologies available today) was compounded by
the equally formidable
philosophical challenge of explaining the relationship between the electrochemical
impulses coursing through a
massively complex network of neurons and the experiential fabric of our subjective
mental lives – our thoughts,
values, perceptions, and choices.

. Page 7 of 40
Unlike most scientists today, the neuroscientists and psychologists of that era
understood that science is
inevitably rife with philosophical assumptions. For the most part, they worked
within a paradigm that they had
inherited from the 17th-century polymath René Descartes. Two components of the
Cartesian intellectual tradition
were especially relevant to their work. One concerned the ‘mind-body problem’ The
other concerned what might
be called ‘the mind-mind problem’ The first of these was primarily of interest to
neuroscientists, while the second
was mainly of interest to psychologists.
Each of us has direct access only to our own mental states, and we cannot be
mistaken about those states. This
implied that psychological research should proceed by means of introspection, which
is why the first
psychologists came to be known as ‘introspectionists’.
During the course of the 19th century, the Cartesian concept of mind-body dualism
came under increasing
pressure. Scientists of the mind responded to this sort of challenge with two
explanatory strategies, both based
on the assumption that nothing that is mental can be unconscious, and nothing that
is unconscious can be mental.
Some granted that ostensibly unconscious mental states were indeed mental but
insisted that they were not really
unconscious. According to this view, a person’s consciousness can be split apart,
resulting in a ‘main’
consciousness and one or more ‘sub’- consciousnesses, an approach that’s sometimes
called dissociationism.
A second strategy was to accept that ostensibly unconscious mental states are
genuinely unconscious, but to deny
that they are mental. Advocates of this dispositionalist approach believed that the
(non-physical) mind is distinct
from the (physical) brain, and that only the brain processes behaviour. They
believed that mental states
accompany these physical processes, but denied that they make any contribution to
human behaviour.
Both of these theories were also harnessed to explain the puzzling phenomenology of
mental illness. Consider
the splitting of the mind associated with the mental disorder that was then known
as hysteria. It was natural to
explain this phenomenon as the splitting or fragmentation of a single self into
several others. As the philosopher
and psychologist William James put it in The Principles of Psychology (1890):
A hysterical woman abandons part of her consciousness because she is too weak
nervously to hold it together.
The abandoned part may meanwhile solidify into a secondary or subconscious self.
This ‘dispositionalist’ story
was also deployed to throw light on psychiatric disorders.
This ‘dispositionalist’ story was also deployed to throw light on psychiatric
disorders. The fact that mentally ill
people often engaged in compulsive behaviours that they could neither control nor
understand could perhaps be
explained by unconscious brain activity that is somehow cut off from the person’s
mind. In cases of compulsive
hand-washing, for instance, the sufferer seemed to be hostage to alien forces
within himself that were not part
of his consciousness, and therefore – according to the prevalent assumptions of the
time – not mental at all.
21. Which of the following is true about Sigmund Freud according to the passage?
(a) Freud made use of ideas without acknowledging his predecessors for the sources
of those ideas.
(b) Current neurologists are re-evaluating the works of Sigmund Freud in the light
of Cartesian theory of mind#body dualism knowledge.
(c) Freud’s contributions to the development of psychoanalysis have been greatly
appreciated by Clark
Glymour.
(d) Philosophers should re-examine Freud’s place in neuro-analysis theory.
22. Which of the following, if true, most strongly supports the psychologists’
interpretation of the study regarding
dispositionalist?
(a) A person’s consciousness can be split apart, resulting in a ‘main’
consciousness and one or more ‘sub’-
consciousnesses.
(b) The human behaviour indicating acceptance or rejection are exhibited by trauma
and emotional imbalances.
(c) Each of us has direct access only to our own mental states, and we cannot be
mistaken about those states.
(d) The mind is distinct from the brain and only the brain processes behaviour.

. Page 8 of 40
23. According to the passage, which of the following can be inferred about mental
illness?
(a) For neuroscientific researchers, the unnerving scientific challenge was to
figure the working of the brain and
explain the correlation of the physical functioning of the brain and the
behavioural aspects of metal life such
as– our thoughts, values, perceptions, and choices.
(b) Freud believed that when we explain our behaviour to ourselves or others
(conscious mental activity), we
rarely give a true account of our motivation.
(c) The neuroscientists finally figured out two strategies in explaining the mental
illnesses which were
dispositionalists and dissociationism.
(d) The basic dilemma of all human existence is that each element of the psychic
apparatus makes demands
upon us.
24. The word ‘ostensibly’ means
(a) Lightly (b) Apparently (c) Beautifully (d) Lively
25. Which of the following can be inferred from the last paragraph of the given
passage?
(a) A mentally ill person’s activities may be related to his/her mind and not the
unconscious pineal lobe.
(b) A mentally ill person’s activities can be explained by his\her unconscious
brain.
(c) A mentally ill person’s activities are a result of only dispositionalist nature
of the unconscious brain.
(d) A mentally ill people often fail to differentiate real from apparent.
26. One can infer from the passage that the author is
(a) A journalist. (b) A philosopher. (c) A psychologist. (d) A layman
Passage (Q.27-Q.30): James Lovelock defined Gaia as a complex entity involving the
Earth’s biosphere,
atmosphere, oceans, and soil, the totality constituting a feedback or cybernetic
system which seeks an optimal
physical and chemical environment for life on this planet.
The theory was initially, according to Lovelock, a way to explain the fact that
combinations of chemicals
including oxygen and methane persist in stable concentrations in the atmosphere of
the Earth. Lovelock
suggested using such combinations detected in other planets’ atmospheres would be a
relatively reliable and
cheap way to detect life, which many biologists opposed at the time and since.
According to critics of Lovelock’s
theory, “organisms could not act in concert as this would require foresight and
planning from them Dawkins
claimed ‘there was no way for evolution by natural selection to lead to altruism on
a Global scale’.
Later other relationships such as the fact that sea creatures produce sulfur and
iodine in approximately the
quantities required by land creatures emerged and helped bolster the theory. Rather
than invent many different
theories to describe each such equilibrium, Lovelock dealt with them holistically,
naming this self- regulating
living system after the Greek goddess Gaia. The Gaia Hypothesis has since been
supported by a number of
scientific experiments and provided a number of useful predictions, and hence is
properly referred to as the Gaia
Theory.
His initial hypothesis was that the biomass modifies the conditions on the planet
to make conditions on the planet
more hospitable - the Gaia Hypothesis properly defined this ‘hospitality” as a full
homeostasis.
Lovelock’s initial hypothesis, accused of being teleological by his critics, was
that the atmosphere is kept in
homeostasis by and for the biosphere.
Aside from clarifying his language and understanding of what is meant by a life
form, Lovelock himself ascribes
most of the criticism to a lack of understanding of non-linear mathematics by his
critics, and a linearizing form
of greedy reductionism in which all events have to be immediately ascribed to
specific causes before the fact.

. Page 9 of 40
One of the criterion of the empirical definition of life is its ability to
replicate and pass on their genetic
information to succeeding generations. Consequently, an argument against the idea
that Gaia is a “living”
organism is the fact that the planet is unable to reproduce.
Lovelock, however, defines life as a self-preserving, self-similar system of
feedback loops like Humberto
Maturana’s autopoiesis; as a self-similar system, life could be a cell as well as
an organ embedded into a larger
organism as well as an individual in a larger inter-dependent social context.
Maturana and Lovelock changed this with the autopoiesis deductive definition which
to them explains the
phenomenon of life better. Reproduction becomes optional: bee swarms reproduce,
while the biosphere has no
need to. Lovelock himself states in the original Gaia book that even that is not
true; given the possibilities, the
biosphere may multiply in the future by colonizing other planets, as humankind may
be the primer by which
Gaia will reproduce. Humanity’s exploration of space, its interest in colonizing
and even terraforming other
planets, lends some plausibility to the idea that Gaia might in effect be able to
reproduce.
27. Which of the following statements is not a feature of the Gaia hypothesis?
(a) The Gaia hypothesis can be reduced to the level of oxygen, methane and other
elements in the atmosphere.
(b) Individual-level effects can translate to planetary homeostasis.
(c) Gaia is capable of reproduction depending on the efforts of the humans.
(d) The Earth behaves as a single, self-regulating system comprised of physical,
chemical, biological and human
components.
28. In the context of this passage, the word “teleological’ most nearly means:
(a) The philosophical study of design and purpose.
(b) All things are designed for an inherent purpose they serve rather than the
cause by which they arise.
(c) Contrasted with metaphysical naturalism, which views nature as lacking design
or purpose.
(d) The idea that form is defined by function.
29. Which of the following reflects the title of the passage?
(a) The Gaia Hypothesis: Critics’ views.
(b) The Gaia Hypothesis: James Lovelock’s analysis.
(c) The Gaia Hypothesis: The Holistic Nature of Planet Earth.
(d) The Gaia hypothesis: The fractured Nature of Planet Earth.
30. The given sentence from the passage is divided into four parts. Choose the part
that carries a usage error. It there
is no error, select option D as their answer.
One of the criterion of the empirical definition of life is its ability to
replicate and pass on their genetic
information to succeeding generations.
(a) One of the criterion of the empirical definition
(b) of life is its ability to replicate and pass
(c) on its genetic information to succeeding generations
(d) No error
Directions (Q.66 – Q.105): Read the comprehensions carefully and answer the
questions based on it.
Passage (Q.66-Q.70): The Telangana High Court in a recent judgment laid down that
the life of the fetus or to
be born child cannot be placed at higher pedestal than that of the life of the
woman. The court observed that
Constitutional Courts have the power under writ jurisdiction to direct termination
of pregnancy, even when the
length of pregnancy is beyond the statutory limit of twenty-four weeks as per the
Medical Termination of
Pregnancy (Amendment) Act, 2021 (The Act).
The court also observed that, the dignity, self-respect, healthy living are facets
of right to life and personal liberty
enshrined under Article 21 of the Constitution of India, which also include right
of a woman to make a choice
of pregnancy and terminate pregnancy, in case, where pregnancy is caused by rape or
sexual abuse or for that
matter unplanned pregnancy, subject to reasonable restrictions under law."
Section 3 of Medical Termination of Pregnancy Act (Post Amendment in 2021) Section
3 of Medical
Termination of Pregnancy Act, 1971 deals with scenarios when pregnancies may be
terminated by registered
medical practitioners. As per the Medical Termination of Pregnancy (Amendment) Act,
2021, the upper limit
for medical termination of pregnancy is 24 weeks as opposed to 20 weeks which was
the position before the
amendment. The termination can take place when a pregnancy occurs as a result of
failure of any contraceptive,
or where pregnancy has been caused by rape or sexual assault, or there are
substantial fetal abnormalities but not
beyond the prescribed upper limit (position before the amendment).
The court relied upon the latest decision in xxx v. Union of India, in which the
High Court of Kerala allowed
the termination of pregnancy even when the period of gestation had reached 26 weeks
taking into account the
traumatic experience of the victim and the possible genetic disorders of the unborn
child. It held that when the
period of gestation exceeds the period prescribed in Sections 3 and 4 of the
Medical Termination of Pregnancy
Act, 1971, medical termination can be carried out only by an order of court.
(SOURCE: 'Woman Has Right Not to Carry Pregnancy, Subject to Restrictions':
Telangana High Court Permits
Rape Victim to Terminate 26 Weeks Old Fetus, LIVELAW)
66. Lavina, an 18-year-old girl, went to his uncle's house with her family for
dinner. When no one was in the home,
she was harassed and raped by two of his relatives in the backyard. Lavina had been
instructed to keep her mouth
shut. After a while, as Lavina's health continued to decrease, she was referred to
a doctor. It was then discovered
that she was 23 weeks pregnant. Can she terminate her pregnancy at this point
according to the law before
amendment?
(a) No, she cannot terminate her pregnancy as she was a major.
(b) Yes, she can terminate her pregnancy under Section 3 of Medical Termination of
Pregnancy Act (Post
Amendment in 2021).
(c) No, she cannot terminate her pregnancy since it would amount to violation of
right to life of an unborn child.
(d) No, she cannot terminate her pregnancy since it exceeded the upper limit set by
the pre-amended statute.
67. Considering the above situation, what will be the correct statement if she
wishes to end her pregnancy according
to the recent high court judgment?
(a) She cannot terminate her pregnancy as the length of pregnancy is way beyond
statutory limit.
(b) She has the right to terminate her pregnancy because it was the consequence of
harassment and rape by her
relatives.
(c) She cannot terminate her pregnancy as the life of the fetus or to be born child
cannot be placed at lower
pedestal than that of the life of the woman.
(d) She can terminate her pregnancy even when the length of pregnancy is beyond the
statutory limit.

. Page 17 of 40
68. On account of a fake promise to marry and settle, Sudhir disguised to mala to
come under physical relation with
him. When mala got pregnant Sudhir refused to accept that it was his child and
refused to marry. Mala was 28
weeks pregnant, which is beyond statutory limit of twenty-four weeks as per the
Medical Termination of
Pregnancy (Amendment) Act, 2021. Mala filed a petition before Court to grant an
order for termination. Does
High Court have power to direct termination of pregnancy, even when the length of
pregnancy is beyond the
statutory limit prescribed?
(a) Yes, the high court has power under writ jurisdiction to direct termination of
pregnancy when the length of
pregnancy is beyond the statutory limit.
(b) No, the high court has power to direct termination of pregnancy only when the
length of pregnancy is under
the statutory limit.
(c) No, the high court cannot direct termination of pregnancy as the physical
relationship between mala and
Sudhir was consensual.
(d) Yes, the high court has power as Sudhir refused to marry mala.
69. Continuing on similar facts, Mala proceeded with medical termination without
seeking court approval, despite
the fact that the gestation period exceeded the length prescribed in Sections 3 and
4 of the Medical Termination
of Pregnancy Act, 1971. Is her termination lawful?
(a) It is lawful because, according to a recent decision, when the gestation period
exceeds the length prescribed
in Sections 3 and 4 of the Medical Termination of Pregnancy Act, 1971, medical
termination can be carried
out.
(b) It is unlawful as, when the gestation period exceeds the length prescribed in
Sections 3 and 4 of the Medical
Termination of Pregnancy Act, 1971, medical termination can only be carried out by
court order.
(c) It is lawful as it is Mala’s right under Article 21 of the Constitution of
India, which also includes the right of
a woman to make a choice of pregnancy and terminate the pregnancy, in case, where
pregnancy is caused
by rape or sexual abuse or for that matter unplanned pregnancy.
(d) It is lawful as Constitutional Courts have the power under writ jurisdiction to
direct termination of pregnancy,
even when the length of pregnancy is beyond the statutory limit of twenty-four
weeks as per the Medical
Termination of Pregnancy (Amendment) Act, 2021.
70. ASSERTION: Article 21 of the Indian Constitution, also includes the right of a
woman to choose her pregnancy
and terminate it if the pregnancy is caused by rape or sexual abuse, or if the
pregnancy is unexpected.
REASON: The High Court of Kerala allowed the termination of pregnancy and held that
even when the period
of gestation had reached 26 weeks taking into account the right of a woman to
choose her pregnancy and
terminate it if the pregnancy is caused by rape or sexual abuse, or if the
pregnancy is unexpected.
(a) Both A and R are true and R is correct Explanation of A.
(b) Both A and R are true but R is not correct Explanation of A
(c) A is true but R is false.
(d) A is false but R is true.
Passage (Q.71-Q.75): The Punjab and Haryana High Court has recently observed that
in a Multiple-Choice
Question (MCQ) based examinations, a question which has no single, unique or 'most
appropriate answer' (i.e.,
suspected question) becomes incapable of being asked. The Bench further held that a
suspected question in an
MCQ based examination needs to be deleted so that no student gets advantage, or is
denied advantage, because
of evaluation of such questions. It was also noted that such objection of suspected
question should be filed before
declaration of result or evaluation. Therefore, apart from the suspected question,
no objections may be accepted.
In an objective multiple-choice question, where the candidate has to merely mark a
correct response, a question
which has no single, unique or 'most appropriate answer' (i.e., suspect question)
becomes incapable of being
asked. This may be because the answer requires an Explanation and argumentation or
reasons for its justification,
which is an exercise permissible for the exam where the format is subjective and
not objective," the Court opined.
Thus, for subjective pattern, the same approach cannot be applied. It was also
noted that such objection should
be filed before declaration of result.
. Page 18 of 40
In light of this, the Court limited its decision to the following question: If
pursuant to the recommendations of
the Expert Committee who found that question Nos.25 & 62 (General Ability Paper)
were ambiguous/confusing,
could those at all be deleted? And, whether the Commission was competent to cause
such deletion?
To answer the query, the Court referred to Apex Court's ruling and held as follows:
"…the examining authority, guided by the experts in the subject, is well equipped
and, thus, rightly authorized
to decide the answer-key and in that process delete the suspected questions.
Further, whether a question is framed
aptly or is vague, ambiguous or has multiple correct answers and, therefore,
required to be deleted is the exclusive
domain of the subject experts. Thus, ordinarily this Court in exercise of power of
judicial review would not
interfere with the opinion of the experts unless shown to be conclusively erroneous
or flawed." It was further
noted by the court that that such objection should be filed before declaration of
result and if failed to file
objection before result is not entitle to raise further objection.
(Source: MCQ Exams- "Question Having No 'Most Appropriate Answer' Is Incapable of
Being Asked, Needs to
Be Deleted": P&H High Court, Live Law)
71. Kapil was an average student with excellent general knowledge. He was also
studying for the CLAT and other
legal entrance exams. He used to study solely from the consortium's materials. He
participated in a mock test
conducted by a consortium of NLUs. It was a multiple-choice question (MCQ) exam.
There is no suitable or
right response to one of the multiple-choice questions in the GK section. Following
the exam, an objection form
was distributed to the students. Can Kapil submit a petition on the basis of such a
disparity, claiming that a
suspect question in an MCQ-based test must be deleted?
(a) Yes, he can file a petition on the basis that the concerned question has no
single, unique or 'most appropriate
answer’.
(b) Yes, he can file an objection to the consortium only.
(c) No, he cannot file an objection as it was a mock paper and not the original
entrance paper.
(d) No, he cannot file an objection as he has no proof to show such disparity.
72. Continuing with similar facts, suppose Kapil raised such an objection after the
results were announced, claiming
he missed out on the merit list because one question was incorrect. Can he file an
objection after the result has
been declared? Decide.
(a) He cannot file an objection petition after the result has been declared.
(b) He cannot file an objection on grounds that he missed out on the merit list
because one question was
incorrect.
(c) He can file an objection as a question which has no single, unique or 'most
appropriate answer' (i.e., suspect
question) becomes incapable of being asked.
(d) He can file an objection because suspect question in an MCQ based examination
needs to be deleted as there
was an error in question and evaluation.
73. In a similar pattern CLAT conducts it PG entrance exam. Which has both
objective I.e., Multiple-Choice
Question (MCQ) based question as well as an additional section of 20 marks which is
subjective in nature. Kapil
when he wrote PG entrance exam. He found out that the question asked in subjective
section was completely
erroneous. Can he file a petition seeking deletion of such question alleging
similar approach should be applied
in for suspect question in subjective section as applied in an MCQ based
examination so that no student gets
advantage, or is denied advantage, because of evaluation of such questions?
(a) No, he cannot file a petition as the said error was found in subjective
section.
(b) Yes, he can file a petition as paper was objective I.e., Multiple-Choice
Question (MCQ) based.
(c) No, he cannot bring a petition because a similar approach cannot be applied to
a subjective pattern.
(d) Yes, he can file a petition as it comes under the category of suspect question.

. Page 19 of 40
74. Ram wrote MPPSC examination in MCQ pattern. There he found out that the static
GK section was altogether
copied from an online mock series, as all questions are repeated. Though there was
no such question reported
which has no single, unique or 'most appropriate answer' (I.e., Suspect question).
Can ram file an objection in
such a scenario to the examining authority?
(a) Ram can rightly raise an objection as examining authority is rightly authorized
to decide in such scenario.
(b) Ram cannot raise an objection as it was not an MCQ based question paper.
(c) Ram cannot raise an objection as there was no suspect question in the paper.
(d) Ram can raise an objection the paper was completely copied.
75. Continuing on above facts, the examining authority and expert is of the opinion
that there exists no fault or
ambiguity in the paper. Ram filed a case against a high court for a judicial review
challenging the decision of
examining authority and experts in the subject declaring no fault or ambiguity in
the paper.
(a) Court can rightly interfere in matter if it is proved to be conclusively
erroneous or flawed.
(b) Court cannot interfere with the opinion of the experts unless shown to be
conclusively erroneous or flawed.
(c) Court can interfere as the opinion of the experts seems to be conclusively
erroneous or flawed.
(d) Both (a) and (c).
Passage (Q.76-Q.80): The Patna High Court on Monday observed that in absence of any
connection between
the act alleged and the property recovered under the Unlawful Activities
(Prevention) Act, it cannot be assumed
that those properties were acquired by the terrorist act.
The observation came from a single judge bench comprising of Justice Birendra Kumar
while dealing with a
petition filed by the family members of an accused person challenging the order of
Designated Authority
allowing approval under sec. 25 of UAPA with regards to seizure of certain
properties which were not connected
with the acts of terrorism so alleged against the accused.
Observations of the Court
"Section 25 of the UAP Act requires that the Investigating Officer must have
"reason to believe" that any property
in relation to which an investigation is being conducted represents "proceeds of
terrorism".”The reason to
believe" must be on the basis of specific, reliable and relevant information." The
Court observed at the outset.
In view of this, the Court observed that the police report submitted did not show
any specific reliable or relevant
information to form a believe that the property so seized were "proceeds of
terrorism".
"In absence of any connection between the act alleged and the property recovered,
it cannot be assumed that
those properties were acquired by the terrorist act. In such case the seizure
exercise is held to be illegal and
arbitrary".
The Court held.
"To attract the mischief of penalty for being member of an unlawful association
under Section 10 of the UAP
Act, it must be established that the association was declared unlawful by a
notification issued under Section 3
of the UAP Act. In the case on hand, there is no evidence that to which of the
unlawful association the accused
were supplying the arms. Hence, it cannot be ascertained whether that association
was declared unlawful
association or not. Likewise, Section 13 of the UAP Act which provides punishment
for unlawful activities is,
prima facie, not attracted in absence of identity of the unlawful association."
(SOURCE: Pat HC | To attract penalty for being a member of an unlawful association,
the association must be
one declared unlawful by a notification under UAP Act, 1967; HC declares seizure
exercise illegal and arbitrary,
LiveLaw)

. Page 20 of 40
76. Madan and Mohan live in Jammu and Kashmir, which seems to be a controversial
region between India and
Pakistan. Residents of J&K are occasionally subjected to violence as a result of
military forces activity, as well
as terrorist attacks. Madan and Mohan, angered, vowed to put an end to this once
and for all. To discuss their
proposal further, they approached a secret organization that had been proclaimed
unlawful. They began meeting
and discussing the various sorts of techniques and materials needed to build a
bomb. They also traded various
material samples for the same. Through a confidential source, police learned about
the conspiracy and detained
both Madan and Mohan under the Unlawful Activities Prevention Act of 1967. The
investigation officer also
recovered the material in investigation. Is the detention justified? Decide.
(a) The act of detention is Not justifiable is as the bomb has not been planted
yet.
(b) The act of detention is Not justifiable is as they were just discussing and not
done anything in furtherance of
their common intention.
(c) The arrest is justified since the police officer has reason to believe that the
material under investigation is
terrorist proceeds.
(d) The arrest is justified as the investigation officer recovered the explosive
material from the accused.
77. In a similar circumstance, a young teenager named Shaheen was inspired by Mohan
and Madan's actions and
attempted to finish Madan and Mohan's quest. He began gathering material on the
internet and was simply
waiting for the right opportunity. Police on investigation and seizure found
nothing but his online browsing
history. Report submitted did not did not show any specific or relevant information
to form a believe that the
property so seized were proceeds of terrorism. Is seizure legal?
(a) The exercise entered into by the IO in making seizure of property was legal.
(b) The exercise entered into by the IO in making seizure of property was not legal
as the report failed to show
that property so seized did not represent any proceeds of terrorism.
(c) The exercise entered into by the IO in making seizure of property was legal as
the police found relevant
browsing history which shows proceeds of terrorism.
(d) The exercise entered into by the IO in making seizure of property was not
illegal as police made seizure on
relevant information.
78. On suspicion of engaging in illicit activities under UAPA, police performed a
seizure and detained Raju and his
friends. The police took this action because they were watching Raju and his
friend's computer's online activity.
The browsing history reveals a pattern of unauthorized access to prohibited
websites. Despite the fact that no
substantial evidence was recovered by the police, the suspects were placed on the
terrorist watch list. In addition,
no declaration of unlawful association was made under the UAPA. Police filed a
charge sheet against all the
accused under section 10 UAPA for being member of an unlawful association. Decide.
(a) Charge sheet filed is incorrect as it was a matter violation of right to
privacy.
(b) Charge sheet filed is correct as the browsing history shows unauthorized access
to prohibited websites.
(c) Charge sheet filed is incorrect as the association have not been declared
unlawful under UAPA.
(d) Charge sheet filed is correct as police suspicion of engaging in illicit
activities under UAPA was found
correct.
79. Rohan and Rahul were delivery boys, taking all sort of orders. Police on
information alarmed all check-post
station to check every vehicle passing. Looking at the police party, three persons
from the vehicle started fleeing
and managed their escape. Police caught one of the three accused and they
identified that Rohan and Rahul were
the person who fled away. Nothing was recovered from the physical possession of the
arrested person. However,
from the vehicle, a pistol along with other accessories were recovered for which
the arrested accused produced
relevant paper. Besides that, some Naxal literature were also seized from the
vehicle and the arrested persons
disclosed that Rahul and Rohan used to supply arms to the Naxals which has been
declared unlawful. Police on
assumption that the properties were acquired by the terrorist act, seized the
property investigated and also
detained the accused under UAPA. Decide
(a) Police action was illegal and arbitrary as there exists no connection between
the alleged act and the property
recovered.
(b) Police action was not illegal and arbitrary as police recovered a pistol and
Naxal literature.
(c) Police action was illegal as they should have detained Rahul and Rohan.
(d) Police action was illegal as it was not approved by designated authority.

. Page 21 of 40
80. In a similar situation, assume that police caught all the three accused fleeing
with Naxal arms for which they
have no relevant papers, will then the arrest will be legal under UAP Act?
(a) No, as there exists no connection between the alleged act and the property
recovered.
(b) No, as there was no approval under sec. 25 of UAPA of designated authority with
regards to seizure of
properties.
(c) Yes, as there exists connection between the alleged act and the property
recovered.
(d) Yes, as the accused were found in possession of Naxal arms.
Passage (Q.81-Q.85): The Patna High Court has recently ruled that the demand
of Chanakya National Law
University (CNLU) with respect to payment of facilities fees and examination fees
from the students is 'arbitrary
and illegal' in view of the fact that students had not attended classes and had not
availed these facilities due to
the Covid-19 pandemic.
Court observed that mere purchase of books and other infrastructure does not
entitle the university to demand
fees towards facilities fees and library fees especially since students are not
benefited by the aforesaid facilities.
The Court further placed reliance on the Supreme Court judgment in Indian School,
Jodhpur v. State of
Rajasthan, insisting on payment for facilities not provided to students would
amount to profiteering which must
be avoided by the educational institutions.
"In law, the school management cannot be heard to collect fees in respect of
activities and facilities which are,
in fact, not provided to or availed of by its students due to circumstances beyond
their control. Demanding fees
even in respect of overheads on such activities would be nothing short of indulging
in profiteering and
commercialization. It is also observed that students are not liable to pay fees for
unutilized facilities during the
relevant period of pandemic year 2020-21, the Court stated further.
The Court also placed reliance on DPSP which is non-justiciable under Article 41 of
the Constitution which
provides for the right to secure the right of work, to education and to public
assistance in certain cases of
unemployment, old age sickness and disability. Similarly, Article 46 of the
Constitution deals with education of
weaker sections of the people, it was noted further.
(Source: Educational Institutions Should Be Sensitive About Aftermath of The
Pandemic': Patna HC Sets Aside
CNLU's Demand for Payment of 'Library Fees' & 'Examination Fees', livelaw)
81. Raman had recently taken admission in Texas law school. The counselling process
was scheduled to be healed
in physical mode but due to increase in number of covid cases, the university
allowed online counselling. Further,
Raman did not get a chance to visit his college as, the university vide an official
memorandum dated August 7,
2020, declared to function in an online mode. Only lecture facility was started on
online portal and Raman was
a regular student. Raman was asked to pay hostel fees, library fees and other
facilities fees. To which Raman
filed a petition against the authorities to waive off all the fees including the
academic tuition alleging that, these
facilities were not availed by him in full access. Is the demand for fees under
different heads arbitrary and illegal?
(a) It is not wholly arbitrary and illegal as the university is running classes
online.
(b) It is not arbitrary and illegal as the university is working online and thus
providing all the services.
(c) It is arbitrary and illegal as students had not attended classes physically and
had not availed any of the
facilities due to the Covid-19 pandemic except online lectures delivered by
university on its portal they can
only claim tuition fee.
(d) It is arbitrary and illegal as only online lectures were delivered and no other
facilities.
82. Rocky, an NRI student in 3rd year of ANCT College of Engineering. Since it was
his final year, he did not
attended any class nor took any facility availed by the college. He also without
giving any notice, started living
outside the campus hostel. College served him a notice in demand to pay his hostel
fees, library fees, tuition fees
and other facilities fee. Is the demand for fees under different heads arbitrary
and illegal?
(a) It is not arbitrary and illegal as it was rocky who didn’t availed the
facilities and attended classes and did not
inform college regarding the same.
(b) It is not arbitrary and illegal as it was rocky’s personal choice to not attend
the class and avail any other
facility provided by college.
(c) It is arbitrary and illegal as rocky neither attended classes and nor availed
these facilities.
(d) It is arbitrary and illegal as college cannot demand fees for facilities didn’t
availed by the students.

. Page 22 of 40
83. Aakash is from a lower-middle-class background. His parents own an antique
store on Shimla's Mall Road. The
family's earnings are insufficient to meet all of Aakash's needs. He was sent to
one of Shimla's top schools under
the BPL quota. Due to the pandemic, the family's earnings were flat, and they were
unable to pay Aakash's
education. As a consequence, school officials decided to expel Aakash from their
school for failing to pay his
fees on time. Aaksh's parents filed a petition against the school for expelling him
in violation of Articles 41 and
46 of the Indian Constitution. Decide school’s action be challenged?
(a) School’s action of expelling Aakash is violative of Articles 41 and 46 of the
Indian Constitution as he has
right to education, which has been denied by school authorities. Thus, it can be
challenged.
(b) School’s action of expelling Aakash cannot be challenged as right to education
provided under Articles 41
and 46 of the Indian Constitution.
(c) School’s action of expelling Aakash cannot be challenges as they failed to pay
fees on time.
(d) School’s action of expelling Aakash can be challenged as the action is
arbitrary and illegal.
84. During the Covid period, Dhanush's College began to construct a huge library.
So that when students return to
college after the pandemic, they may make use of it. In order to do this, in
addition to the online tuition fee,
college officials requested that students contribute to the building of the library
and the purchase of books under
“other facilities”. Decide
(a) Such a request will be arbitrary and illegal as student did not availed the
facility.
(b) Such a request will be not be arbitrary and illegal as the college is building
the library for students only.
(c) Such a request will not be arbitrary or illegal because the students will avail
the facility once they return to
college.
(d) Such a request is arbitrary because students had no say in the construction of
a library for their own use.
85. Janki public school and its administration decided to begin therapy sessions
for young children suffering from
depression and anxiety during the Covid crisis. It was an optional session, no one
is compelled to mandatorily
attend the session. Each session will cost 1000 rupees per hour. Ramesh filed a
petition alleging that the school
administration's action is purely for profit and commercialization. And thus,
unlawful. Decide
(a) The court will rule that the school administration's actions were illegal since
they were done to make a profit
from the children.
(b) The court will rule that the school administration's actions were not illegal
as they were done to help students
suffering from depression and anxiety during the Covid crisis.
(c) The court will rule that the school administration's activities were unlawful
since the students were charged
excessive fees. Thus, indulging in profiteering and commercialization.
(d) The court will rule that the school administration's activities were not
unlawful as the sessions were optional
and not compulsory.
Passage (Q.86-Q.90): To combat the threat of fake news, a country’s IT Actinclude
the following provisions:
Any person who sends or shares, by means of a computer resource or a communication
device, —
 any information that is grossly offensive or has menacing character; or
 any information that he knows to be false, but for the purpose of causing
annoyance, inconvenience, danger,
obstruction, insult, injury, criminal intimidation, enmity, hatred, or ill will,
 any electronic mail or electronic mail message sent with the intent of causing
annoyance or inconvenience,
or to deceive or mislead the addressee or receiver regarding the origin of such
messages, is punished by
imprisonment for up to three years and a fine.
The terms “electronic mail” and “electronic mail message” refer to a message or
information created, transmitted,
or received on a computer, computer system, computer resource, or communication
device, including
attachments in text, images, audio, video, and any other electronic record that may
be transmitted with the
message.

. Page 23 of 40
86. Salman gets a video forwarded to his WhatsApp on his mobile-phone from his
friend Vinay, which shows a
particular religious community is talking against other community. Thinking it to
be true he posts this on Twitter
from his mobile-phone. His Twitter-follower Amir understands this is a morphed and
doctored video and makes
a complaint in the police-station against Salman for spreading false information.
Is Salman’s action punishable
under this law?
(a) Yes, he has posted a false information which is punishable under the law
(b) No, he didn’t know that the video was fake and did it out of his unknowing and
hence cannot be punished
(c) Yes, the content he has posted could have started violence and therefore, is
punishable under the law
(d) No, since he has posted this from his mobile-phone, his action can’t come under
this law
87. Pranjal had gone for an admission-interview in LawSpin University. His best
friend Alok pranks him by creating
a fake email ID of LawSpin University and sending an e-mail to him saying the
latter has got the admission and
should start in the college from the next day. Pranjal believes the e-mail and goes
to the college the next day
only to find that he has been pranked and that the results of the interview are
still yet to be declared. Can Alok’s
action be punishable under this law?
(a) No, this was a mere prank; this doesn’t come under the above law
(b) No, Pranjal should have checked with the HR of the company before believing
(c) Yes, Alok could have ended damaging Pranjal’s career; this indicates criminal
intent
(d) Yes, Alok did this to annoy Pranjal which is against the law
88. Santosh makes up a false-story of menacing character that he communicates over
the phone to his friend Kamal
in his village. Kamal believes the story and tells it to other people. The story
that Santosh had cooked up had
communal overtones and starts off violence in his locality. Can Santosh’s action be
judged punishable under this
law?
(a) No, Santosh hasn’t sent any electronic message and cannot be held under the law
(b) Yes, Santosh has used a communication device to spread the false information of
menacing character and
can be held under the given law
(c) No, the other people should have verified the story and should not have merely
believed on the oral
conversations
(d) Yes, as Santosh’s intentions of spreading misinformation are very clear and
hence, he should be held for the
same
89. Nitish, a politician was allegedly involved in a corruption charge but was
later acquitted by the Supreme Court.
This had happened three years earlier. Now his political rival Naidu, posted on his
Twitter that Nitish was
convicted in a corruption charge three years earlier. This was done just near the
election. Is Naidu’s action
punishable under this law?
(a) No, Naidu is saying something that has happened and is the truth and contains
no misinformation
(b) Yes, Naidu is knowingly spreading misinformation so as to create an impact on
Nitish’s political career
(c) No, this is actually a case of defamation rather than a case under this law
(d) Yes, Naidu is trying to electorally damage Nitish’s image
90. Keshav Swami, a well-known religious leader, passes away. He has a large
following that wants the state
government to give them a day off in commemoration of his death. The state
administration agrees and issues a
notice of an off-day on its official Twitter account. Ravi, a daily wage earner and
Swami devotee, expresses his
displeasure with this move by replying on the tweet “Why should the whole
production of the state suffer?” Our
mental recollection is sufficient.” Ravi receives extensive criticism for his
remark, and the Swami’s organisation
files a complaint against him, and the police prosecute him under the
aforementioned IT Act, claiming that his
remark has created displeasure among the public. Are the police correct?
(a) Yes, his comment has caused annoyance to many people who are followers of the
Swami and thus, he shall
be charged
(b) No, he has not posted any false information and stated what the government must
have actually done.
(c) No, his comment was not aimed at causing annoyance but was merely putting an
opinion..
(d) Yes, as Ravi being a follower of Swami and speaking against him created
disturbance and hence, he shall be
charged for the same.

. Page 24 of 40
Passage (Q.91-Q.94): The Central Government of Avalon, a secular country,
introduces a rule that the public
perceives as very communalizing and contrary to the ethos of Avalon’s foundation.
As a result, there are several
mass demonstrations. Anjali is swayed by the demonstrations and gives a statement
that attracts the attention of
law authorities. Her main remarks were as follows:
 I suggest a large chakka-jaam to block off connections between the government and
the border state of
Avalon (the state which will be most affected by the law). Only then can the
government grasp our demands;
else, this government will be deafeningly year to our issues.
 We will sever railway tracks and highways, as well as burn down government
buildings, so that it takes at
least a month for them (the government) to clean things on the ground. It is our
job to keep the state apart
from Avalon.
 It is critical that we fight this bill in a constructive manner. It is
detrimental to both Hindus and Muslims.
All Avalon citizens have the right to free speech and expression under Article
17(2) of the nation’s Constitution.
This article is subject to various limitations, including sovereignty and integrity
of state, the security of the state,
cordial relations with other states, public order, decency, or morality, or in
regard to contempt of court,
defamation, or incitement to an offence. Through this Article state can impose
restriction on freedom to speech
but it does not create a liability over person. According to Section 178 of the
nation’s Penal Code, “Whoever,
by words, either spoken or written, or by signs, or by visible representation, or
otherwise, brings or attempts to
bring into hatred or contempt, or excites or attempts to excite disaffection
towards the Government established
by law in Avalon shall be punished with imprisonment for life, to which fine may be
added, or with fine and said
to commit an offence of sedition.” Citing these rules, Anjali was arrested on
accusations of sedition, which
prevented any violence from erupting as a result of her speech’s motivation.
91. Are the authorities incorrect in charging her of sedition over her speech?
(a) No, the action taken by the authorities is as per the Article 17(2) of the
Constitution
(b) No, the action taken by the authorities is as per the Section 178 of the
nation’s Penal Code
(c) Yes, according to Article 17(2), Anjali was just exercising her right to free
speech
(d) No, as Anjali’s speech was inciting people and the authorities have made an
action against her taking into
consideration the Article 17(2) as well as Article 178 of the Constitution and the
nation’s penal code
respectively.
92. The laws of Avalon define violence as “a behaviour by an individual or
individuals against another person or
group of people or property that wilfully threatens, provoke, seeks, or inflicts
damage.” Do the actions suggest
in Anjali’s speech fall under the category of violence?
(a) Yes, Anjali is asking to isolate the country from the state
(b) No, Anjali has not asked to inflict any physical harm to anyone
(c) Yes, Anjali has asked to cut-off railway tracks and roads
(d) No, Anjali has just given a speech, which cannot cause anyone or anything
physical harm.
93. Anjali’s defence brought forth a previous Supreme Court judgement where it
said, “Sedition is not the same
thing as a strong criticism of the government established by law upon the measures
or acts of Government by
lawful means if it is for the betterment of the society”. Should the lawyers of
Anjali ask the Supreme Court to
follow this judgement as rule of precedent and acquit Anjali?
(a) Yes, Anjali in her speech mentions that what she proposes it for the benefit of
the Hindus and Muslims, thus
she wants the betterment of the condition of the people
(b) Yes, Anjali doesn’t advocate riots or violence against civilians in her
speeches and these methods were used
by the freedom-fighters of many countries when they were under colonial rule
(c) No, Anjali’s speech doesn’t suggest non-violent methods to be followed by the
protestors
(d) Yes, the motive of the speech is only to protest against the law brought by the
Government and not the
removal of the Government itself

. Page 25 of 40
94. If the Supreme Court in one of its earlier judgements had mentioned that “A
speech, whatever its substance, may
only be called seditious if it has resulted in violence.”, if the law of precedence
is applied then will Anjali’s
defence get strengthened by this ruling?
(a) No, her speech had mentions of committing violent acts
(b) Yes, her speech hasn’t led to any violent acts
(c) No, her speech could have led to violent acts, but she was apprehended before
that
(d) Even if Anjali’s speech didn’t lead to any violence as of now, but she shall
not be left free and shall be held
accountable as the speech she made can even lead to a bloodbath in the country.
Passage (Q.95-Q.99): The Karnataka High Court has reiterated that a Court hearing a
petition for quashing of
FIR/ charge sheet under Section 482 CrPC cannot appreciate evidence as the same
lies within the domain of the
trial Court.
The court on considering the same observed that,
"Though under Section 482 of Cr.P.C the charge sheet can be quashed, the said
jurisdiction cannot be invoked
for quashing the charge sheet by appreciating the evidence."
"It must be highlighted that appellate Courts ought not to routinely re-appreciate
the evidence in a criminal case.
This is not only for reasons of procedure, expediency, or finality; but because the
trial Court is best placed to
holistically appreciate the demeanor of a witness and other evidence on record,"
the bench said.
The provision under section 482 CrPC confers the HC with an inherent power to quash
an FIR or a complaint,
upon satisfaction of well-established parameters that have been laid down in
decisions such as State of Haryana
v. Bhajan Lal (“Bhajan Lal”) and R.P. Kapur v. State of Punja (“R.P. Kapur”). The
parameters laid down
in Bhajan Lal and R.P. Kapur includes the following:
 If the allegations made in the FIR/complaint, even if taken at face value, do not
prima facie constitute any
offence or make out any case against the accused.
 If the allegations made in the FIR do not disclose any cognizable offence, which
justifies a police
investigation under Section 156(1) of the CrPC.
 If the allegations made in the FIR/complaint and the evidence collected in
support of the same do not disclose
the commission of any offence, and do not build any case against the accused.
 If a criminal proceeding is based on mala fides, or the proceeding is maliciously
instituted with an ulterior
motive.
(Source : while deciding the petition under Section 482 of Cr.P.C, evidence cannot
be appreciated as it lies within
the domain of the Trial Court, Live law)
95. Taban Khan was robbed of Rs 30,000 by three boys in May 17, 2001 in Bhopal. The
police caught the trio three
days later from a bus stand and recovered a knife from the accused Anwar and other
two co-accused. The trial
court held all three accused guilty of robbery with attempt to cause grievous hurt
and sentenced them to seven
years rigorous imprisonment. The convict Anwar approached the High Court which
dismissed the charge of
robbery under IPC by re-appreciating the evidence in a criminal case. Is the action
of High court under section
482 Cr.P.C justifiable?
(a) It is not justifiable as high court has no power to quash an FIR or a complaint
under section 482 of CrPC.
(b) It is justifiable since the stated jurisdiction under section 482 of the CrPC
can be exercised for quashing the
charge sheet based on the evidence.
(c) It is not justifiable as jurisdiction under section 482 of CrPC cannot be
invoked for quashing the charge by
appreciating the evidence.
(d) It is justifiable as High court has inherent power to quash an FIR or a
complaint.

. Page 26 of 40
96. Continuing with the same facts, assuming the HC dropped the charges against the
accused after concluding that
the allegations in the FIR/complaint and the evidence collected in support of the
same do not prima facie establish
an offence or make out a case against the accused Anwar. Is the action of High
court under section 482 Cr.P.C
justifiable?
(a) It is not justifiable as the trial Court is best placed to holistically
appreciate the demeanor of a witness and
other evidence on record.
(b) It is justifiable as evidence collected in support of the same do not disclose
the commission of any offence,
and do not build any case against the accused.
(c) It is not justifiable as evidence collected in support prima facie establish an
offence of robbery.
(d) It is justifiable since Taban Khan has an ulterior motive and the criminal
proceeding was based on malice.
97. Sudha was married against her will, and thus has no liking towards her husband
and her in laws. To get a divorce
she filed a false complaint of cruelty against her husband and in-laws. The trial
court after taking all necessary
evidence on record found husband and his parents guilty of the offence charged. The
husband made an appeal to
high court under section 482 of Cr.P.C. The high court after seeing the evidence on
record concluded that the
allegations made in the complaint and the evidence collected in support of the same
do not disclose the
commission of any offence as the proceeding is based on mala fide intent, and hence
dismissed the
charges. Decide
(a) The HC can quash the criminal proceeding as there exists no offence of cruelty.
(b) The HC cannot quash the criminal proceeding as HC under Section 482 CrPC cannot
appreciate evidence as
the same lies within the domain of the trial Court.
(c) The HC can quash the criminal proceeding as the proceeding is maliciously
instituted with an ulterior
motive.
(d) The HC cannot quash the criminal proceeding as appellate Courts ought not to
routinely re-appreciate the
evidence in a criminal case.
98. Ravish and Juned were charged for the offence of robbery and Grevious hurt
under provision of IPC. Trial court
after appreciating the evidence held two accused guilty of robbery with attempt to
cause grievous hurt and
sentenced them to seven years rigorous imprisonment. The accused made an appeal
under section 482 of CrPC
to dismiss the charges against the accused on grounds the proceeding is maliciously
instituted with an ulterior
motive. Decide
(a) If the proceeding is proven to be founded on deception, the Court of Appeal has
the authority to dismiss it.
(b) HC cannot quash the proceeding as appreciating the evidence comes under the
domain of trial court.
(c) HC can quash the proceeding as two accused were falsely implicated in a
criminal proceeding.
(d) HC cannot quash the proceeding as Court hearing a petition for quashing of FIR/
charge sheet under Section
482 CrPC cannot appreciate evidence.
99. ASSERTION- The trial Court is best placed to evaluate a witness's demeanor and
other evidence on the record
holistically.
REASON - The provision under section 482 CrPC confers the HC with an inherent power
to quash an FIR or a
complaint.
(a) Both A and R are true and R is correct Explanation of A.
(b) Both A and R are true but R is not correct Explanation of A
(c) A is true but R is false.
(d) A is false but R is true.

. Page 27 of 40
Passage (Q.100-Q.105): The tort of Negligence makes sure that in case a person
fails to exercise the due
reasonable care that a reasonable person should have exercised, then he will be
punished for it. Such failure must
cause injury to a person to make the negligence punishable.
Before the tort of Negligence is established, the court needs to determine whether
the accused owed a duty of
care towards the affected party or not.
Duty of care as per a dictionary definition means, “a moral or legal obligation to
ensure the safety or well-being
of others.”
This duty of care need not be necessarily a legal duty. Rather it could be
reasonable care that a person was
expected to take in such circumstances.
The duty breached must be by the absence of the exercise of reasonable care by the
accused. Also, the injury
caused due to such breach should also be foreseeable.
An injury must be caused to the plaintiff because of the breach of duty. A ‘but
for’ test is conducted for this i.e.
‘but for’ the breach of duty, such injury would have not been caused to the
plaintiff.
(Extracted with requisite revisions and edits from at
https://lawctopus.com/clatalogue/know-all-about-the-tort#of-negligence/)
100. All Cure Hospital (ACH) has been a renowned hospital in Delhi. Due to the
pandemic, it designated its Wing B
for the treatment of Covid patients and those coming to get inoculated. Avanti, a
college student, went to get the
second dosage of the vaccine, Covishield. Dr. Samadhaan was to administer her the
vaccine. However, due to
the busy schedule of the doctor and lack of manpower, he gave the second dosage of
Covaxin to Avanti. On
realizing this Avanti brought a claim against the hospital for negligence, it is
shown in studies and governmental
notifications permitting the switching of the vaccines (both the dosages were not
required to be of same vaccine
and would not have any side-effects). Will Avanti’s claim for negligence succeed?
(a) Yes, the hospital authorities will be vicariously liable for Dr. Samadhan’s
negligence and Avanti will be
entitled to damages.
(b) Yes, since the vaccine not intended has been administered to Avanti it
indicates a breach of legal duty on the
part of Dr. Samadhan and therefore he would be liable for negligence.
(c) No, there appears to be no apparent injury by the act of Dr. Samadhan and hence
Avanti’s claim for
negligence will fail.
(d) No, since Dr. Samadhan exercised all reasonable care and caution while
administering the vaccine and it not
due to any personal breach of duty but lack of manpower which resulted in switching
of the vaccines.
101. When Avanti got to know that a ‘wrong’ vaccine has been administered to her,
she fainted out of panic and
tension and claims that this under the damages for negligence. Is her claim likely
to succeed?
(a) No, the injury caused to Avanti is not foreseeable and therefore no damages can
be claimed for administering
the ‘wrong’ vaccine.
(b) No, since the vaccine being administered was not a wrong vaccine perse as was
confirmed by the government
and studies undertaken therefore no liability on any resultant outcome of the
vaccine too.
(c) Yes, hospital and Dr. Samadhan will be vicariously liable for the injury and
harm caused to Avanti.
(d) Yes, because of the breach of duty of Dr. Samadhan, an injury has been caused
to Avanti and therefore her
claim will succeed.

. Page 28 of 40
102. Avanti was undertaking some renovation activities in her house and for that
purpose she kept all the scraps etc.
at back of the house which was not accessed by anyone. On the occasion of
Sankranti, a kite landed at the
posterior part of the house. A kid, Raju went to catch the kite and a nail pierced
through his feet. A claim for
negligence was brought against Avanti for not taking due care and caution as
required. Will such a claim
succeed?
(a) No, Avanti is not liable for any injury caused to trespassers in her land and
therefore the claim is likely to
fail.
(b) Yes, injury is caused by the lack of care and caution on the part of Avanti as
scraps which could cause injury
are required to be properly disposed off.
(c) Yes, Avanti is required to exercise due care and caution to even the
trespassers and any harm or injury caused
due to lack of such exercise will make her liable for negligence.
(d) No, Avanti had exercised all reasonable care and caution as is expected from a
reasonable person and
therefore would not be liable for negligence.
103. Raju had always been a mischievous kid who frequently landed into troubles and
injured himself. Once, he was
running at a sidewalk trying to balance in his newly bought roller skates. The
municipality was undertaking some
construction activities but had forgotten to place the warnings/ roadblocks. Raju
saw the pothole open in front at
the very last moment and unable to balance himself fell into it thereby fracturing
his bones. Can the municipality
be made liable for negligence?
(a) No, there is contributory negligence on the part of Raju also as he was unable
to balance himself in the roller
skates and used them in the public road.
(b) Yes, there was a breach of duty on the part of the municipality which resulted
in the injury to Raju and
therefore they would be liable for negligence.
(c) No, the injury to Raju was not foreseeable by any reasonable person and
therefore no liability can be
attributed to the Municipality for negligence.
(d) Yes, municipality would, however, be partly responsible and not entirely as
this is a case of contributory
negligence.
104. When the grandmother of Raju heard of this unfortunate news, she got a cardiac
arrest and was hospitalized.
They decided to club this claim along with the claim for Raju’s injury. Are both
the claims likely to succeed?
(a) No, only the claim with respect to injury to Raju will succeed and the claim as
to cardiac arrest to
grandmother will fail as it is not foreseeable.
(b) Yes, applying the but-for test, Municipality could be made liable for the
injury to the grandmother of Raju
(cardiac arrest) as the injury would not have occurred but for the negligence of
Municipality.
(c) No, only the claim as to grandmother’s injury will succeed and the claim as to
Raju’s injury will fail because
of contributory negligence.
(d) Yes, both the claims will succeed as they are direct impact of the negligence
of municipality.
105. Before Raju fell into the pothole, a passer-by knowing that the road is under
construction and that there is a huge
pothole in front didn’t care to warn Raju or attempted to stop him. A suit is
brought against him for not exercising
due care and taking cautions. Will the claim succeed?
(a) Yes, duty of care is not restricted to only legal duty but also moral duty
which has not been exercised by the
passer-by and therefore he would be liable.
(b) No, there was no duty breached or absence of exercising any reasonable care by
the passer-by and hence he
cannot be made liable.
(c) Yes, it was but-for the failure of the passer-by to warn Raju that the injury
would not have occurred.
(d) No, there is no constitution duty on the passer-by to warn or caution Raju and
therefore there is no breach of
duty and no liability.

. Page 29 of 40
SECTION - D: LOGICAL REASONING
Passage (Q.106-Q.110): Prime Minister Modi last week elaborately pitched India as
an investment destination
that could serve as a manufacturing hub at the heart of global supply chains. The
pitch made at the U.S.-India
Strategic Partnership Forum comes in the backdrop of the government’s keenness to
use the disruptions the
COVID-19 pandemic has caused to the cross-border movement of goods as an
opportunity to lure potential
investors, especially those looking to relocate from China, to India. This tact is
consistent with recent initiatives
to explore supply-chain synergies with other economies, including Japan, as an
escalating border feud casts a
shadow over India’s economic and trade ties with its northern neighbour. The
reasoning appears to be that if
even a few multinational enterprises can be drawn to set up manufacturing bases,
either by shifting facilities or
as new additional plants, then not only does the Indian economy stand to gain FDI,
new jobs and tax revenue but
it also makes a statement. Clearly, officials must have advised Mr. Modi that U.S.
businesses were the ideal
target given the worsening relationship between Washington and Beijing and the
ongoing trade stand-off
between the world’s two largest economies. On the face of it, the approach seems
inarguably sound. The rub,
however, lies in the government’s recent ‘Aatmanirbhar Bharat’ initiative, of
making India more self-reliant.
Over the decades, it has been established that global FDI investors prioritise and
are even willing to pay a
premium for policy stability and largely barrier-free access to local and
international markets. The drive for self#reliance has spurred several Ministries
to urge companies and industry sectors to replace imports with ‘Made in
India’ substitutes. From the Shipping Ministry’s call for the design and
manufacture of indigenous tugboats to
auto component makers being told to abjure foreign parts, the thrust of the
initiative is evidently ‘import
substitution’. It is hard to imagine any potential foreign investor in
manufacturing being ready to source capital
goods locally — assuming they are available — even at the cost of possibly
compromising on quality or price
or both. Betraying the government’s anxiety, Mr. Modi took pains to stress that the
push for self-reliance should
not be interpreted as India turning its back on the world. Separately, from the
market access perspective, India’s
decision to not join the RCEP multilateral trade pact would put investor companies
seeking to tap consumers in
RCEP member countries at a tariff disadvantage. Interestingly, most of the recent
FDI announcements have been
by way of stake acquisitions in existing businesses, and predominantly in the
services sector. Attracting FDI into
manufacturing will require the government to convince investors that it is
committed not merely in words but in
deeds as well to an open, barrier-free global trade and investment order.
106. Which of the following will strengthen Indian government’s call to lure
potential foreign investors post COVID
disruptions in cross border movements of economic good?
I. To boost indigenous supply chains of goods pan India from products sourced
locally under Aatma- nirbhar
Bharat initiative.
II. To ease the foreign investment policies and to expedite procedures of
initiating a start up to lure foreign
companies to invest in India.
III. To open barrier-free global trade and investment in order to attract foreign
companies.
(a) Only I (b) Only II (c) II and III (d) All of the above.
107. If the initiative of Aatmanirbhar Bharat finds resonance with the indigenous
companies, what further logical
conclusion can be drawn in terms of India making a statement globally?
I. The indigenous manufacturers have to be quality conscious and their products
competitively priced to use
the supply-chain synergies with the other economies.
II. Self-reliance with regard to goods will drive the economy ensuring a lesser
need for FDIs and a sounder
global trade opportunity.
(a) Only I (b) Only II (c) Neither I nor II (d) Both I and II

. Page 30 of 40
108. We can infer from the statement that Aatmanirbhar Bharat initiative is
(a) A complementary to the decision of wooing American industries to India as the
new industrial investment
destination hub.
(b) A complementary to the decision of inviting China to invest in FDIs building
stronger ties with the neighbour.
(c) Conflicting to the decision of pitching India as the new manufacturing hub to
the US after the China-US
trade stand-off.
(d) A neutral initiative with no bearing on inviting US companies to India as a new
investment destination to
boost economic growth.
109. “Interestingly, most of the recent FDI announcements have been by way of stake
acquisitions in existing
businesses, and predominantly in the services sector.” The core thinking behind
making FDI announcements is
that
(a) The prospects of luring foreign investors towards service sector have a better
standing.
(b) Foreign investors are looking to investing in the service sector in India.
(c) Manufacturing sector does not hold attractive prospects for foreign investors
to take a risk.
(d) Manufacturing sector is robust and self-reliant.
110. The assumption behind the passage is that
(a) India’s efforts to attract capital will result in failure of Aatmanirbhar
Bharat initiative.
(b) India’s efforts to attract capital will result in a flood of FDI till investors
see policy stability.
(c) India’s efforts to attract capital will not result in a flood of FDI till
investors see policy stability.
(d) A foreign manufacturer will be reluctant to source a locally procured raw
material.
Passage (Q.111-Q.115): Following Friday’s talks in Moscow between the Defence
Ministers of India and China,
the prospects of an imminent diplomatic solution to the continuing stand-off along
the LAC do not appear bright.
The statements issued by the two sides have underlined the sharp differences in how
New Delhi and Beijing
have continued to view the unprecedented developments along the border since May,
when China deployed
troops in large numbers and sought to unilaterally redraw the LAC in several areas.
Defence Minister Rajnath
Singh “categorically conveyed” India’s stand, emphasising that China’s actions
“were in violation of the bilateral
agreements”. He also expressed hope that both sides would be able to resolve the
ongoing situation “peacefully
through dialogue”. His Chinese counterpart, General Wei Fenghe, appeared to only
reiterate the stand conveyed
by China in recent statements that it had no blame to bear for this summer’s
developments. He said “the
responsibility lies entirely with the Indian side”, while China “kept maximum
restraint to prevent potential
escalation”. He called on India to “immediately withdraw its troops”. He did, also,
add that both sides should
“stay committed to resolving the issue through dialogue and consultation” and “make
joint efforts to meet each
other halfway”.
As External Affairs Minister S. Jaishankar said last week, diplomacy is the only
way out of the crisis, and that
can only happen “if both sides understand that it is in each of their best
interests if the events of this summer are
not repeated”. The problem, so far, has been a stark mismatch between China’s
statements and the actions of its
troops. Its consistent labelling of India as the aggressor this summer contradicts
the reality that India has, since
May, ceded about 1,000 square kilometres in Ladakh to Chinese control. If China’s
diplomats have spoken
repeatedly of the need to keep in mind “the big picture” of bilateral ties, the
actions of its military on the ground
have suggested an intent that is precisely the opposite, emphasising achieving
tactical gains at the border over
the broader strategic relationship. Until that calculus changes, India will have to
be prepared to be tested along
the border and to stand its ground over the long haul. India has signalled its
intent to do so with the latest
developments on August 29 in Chushul. If the statements following the Moscow meet
did not exactly inspire
confidence, both sides will have the chance to reassess the situation when Mr.
Jaishankar will likely meet his
counterpart, Wang Yi, at a meeting of SCO Foreign Ministers on September 10.
Military talks can occasionally
help to avert a flare-up, but the two neighbours need to work toward a diplomatic
solution to ensure undisturbed
peace and quiet along the border.

. Page 31 of 40
111. “Following Friday’s talks in Moscow between the Defence Ministers of India and
China, the prospects of an
imminent diplomatic solution to the continuing stand-off along the LAC do not
appear bright.” What is/are the
assumption/assumptions that is/are inbuilt in the given statement?
I. A dialogue between countries can resolve a dispute.
II. Talks between countries to arrive at a diplomatic solution does not yield
result.
III. LAC is the bone of contention between India and China.
IV. A dialogue between the countries is not restricted to their respective
countries.
(a) Only I (b) Only III (c) I, III and IV (d) I, II and III
112. Which of the following reflects the situation taken by China from India’s
standpoint?
(a) A paradox with China not walking the talk.
(b) A mismatch of actions with every stand.
(c) A consistency in the statements and actions by China.
(d) A soft approach towards India with no intentions of backing out.
113. Which of the following is in line with the author’s thought process?
(a) Diplomatic solution is the only way to ease the India and China border
disputes.
(b) Unable to resolve their border disputes themselves, India and China should seek
international court of Justice.
(c) India and China must lose no time in finding a diplomatic solution to their
border disputes.
(d) It will be a long haul before India and China will come to a mutual
understanding with respect to border
disputes.
114. What is the primary purpose of the passage?
(a) To mention the strained border dispute between India and China which is
damaging bilateral relationship.
(b) To emphasize on the need to take a firm stand against China, possibly a
military intervention.
(c) To bring to light the Moscow talks between India and China and to emphasize the
need for a diplomatic
solution.
(d) To discuss the strained relations between India and China and its repercussion
on neighbouring countries.
115. What can be a possible conclusion to the given passage if the passage were to
be drawn further?
(a) Clashing interest can sabotage bilateral relationship with no going back.
(b) Countries cannot find internal harmony till the borders are safe.
(c) The last resort falls upon China’s shoulders if it wants to have a better
harmony with its neighbours.
(d) A diplomatic solution is the way to stabilize the bilateral relations to ensure
a better overall understanding
between the two countries.
Passage (Q.116-Q.120): Venus, the hottest planet in the solar system, has not
enjoyed as much recent attention
as Mars, as far as space missions are concerned. With surface temperatures of above
460° Celsius that can melt
even a metal like lead, and a heavy atmosphere of carbon dioxide, the planet was
considered hostile to life. This
despite its being similar in size to the Earth and rocky, so much so that it is
often called the Earth’s “sister planet”.
There was some excitement when the European Space Agency’s mission, Venus Express,
found signs of ozone,
made of three oxygen atoms and considered a biomarker, in the upper atmosphere of
Venus, in 2011. But the
recent discovery of traces of phosphine, another biomarker, in its atmosphere has
just given the search for extra#terrestrial life a shot in the arm. Phosphine, a
compound of one phosphorous atom and three hydrogen atoms, is
given out by some microbes during biochemical processes. In an atmosphere rich in
carbon dioxide, it is likely
to get destroyed soon. However, the researchers estimate that phosphine forms about
20 parts per billion of
Venus’s atmosphere. This fact, when added to the hostile conditions on its surface,
yields tantalising possibilities
— of phosphine’s survival through extraordinary chemistry and thermodynamics or the
stubborn triumph of
biology and life.

. Page 32 of 40
116. What can be inferred by the statement that Venus has not enjoyed as much
recent attention as Mars?
(a) Venus, being Earth’s sister planet has been overlooked as a possible supplicant
to life on it.
(b) Venus as a planet, being the hottest planet, cannot support any form of extra-
terrestrial life as Mars does.
(c) Space agencies paid attention to Mars as they believed that Mars has all the
biomarkers to support extra#terrestrial life.
(d) Space agencies have not paid attention onto Venus as they believed the planet
did not hold any signs of extra#terrestrial life.
117. Which of the following is not an assumption on which the arguments in the
passage stand?
(a) Phosphine is one of the biomarkers indicating life on a planet.
(b) Signs of Ozone on Venus was not a sufficient indicator of a possible signs of
extra-terrestrial life.
(c) Planet Mars has been found more conducive to supporting biomarkers for extra-
terrestrial life than planet
Venus till recently.
(d) With the recent findings of another biomarker Phosphine, Venus will become a
targeted focus of attention
for space missions.
118. What is the primary purpose of the passage?
(a) To draw attention to the possibility of planet Venus supporting extra-
terrestrial life.
(b) To bring to fore the negligence on part of the space mission agencies in
overlooking planet Venus as an
indicator of having extra-terrestrial life.
(c) To highlight the new possibility that planet Venus may be more conducive for
space missions than plant
Mars.
(d) To underline the fact that other planets may also hold the possibility of
extra-terrestrial life.
119. Which of the following will weaken the fact that planet Venus may be
supporting extra-terrestrial life?
1. Mars has ozone, phosphine and other biomarkers along with surface temperature
similar to earth which
makes Mars an ideal condition to sustain life.
II. Phosphine forming 20 parts per billion of Venus’s atmosphere is not sufficient
to sustain life under the hostile
conditions of the planet.
(a) Only I (b) Only II (c) Both I and II (d) Neither I nor II.
120. What logical implication can be drawn from the fact the scientists discover
possible sign of life on planet Venus?
(a) It will be an impossibility to corroborate any of the discovery as part of the
space mission because of hostile
topography of planet Venus.
(b) NASA and ISRO will now launch a space mission to planet Venus to explore the
possibility of biology on
the planet.
(c) Because of such high temperatures, the possibility of flying at a certain
height and sending down high heat
resistant drones would be more feasible than a landing to confirm the findings.
(d) The focal of attention will now shift from Mars to Venus because of Venus’
marked similarity with Earth,
after the new findings.
Passage (Q.121-Q.125): As the United Nations commences the 75th session of the
General Assembly, the need
for internal reforms to suit the 21st century could not be starker. Volkan Bozkir,
the Turkish diplomat and
politician who is the incoming president of the UNGA, has voiced concern that the
structure of the 15-member
Security Council ought to be more democratic and representative. But action has
been long overdue on the
demand, especially from the so-called Group of 4 (G4) countries — Brazil, Germany,
India and Japan — which
advocate a permanent seat for all of them. Meanwhile, the veto powers that the
UNSC’s five permanent members
enjoy is an anachronism in this age. This instrument is often wielded as a blunt
weapon to shore up their
geopolitical interests, regardless of the disastrous consequences for the victims
of armed conflict. The push for
reform gathered momentum following the unilateral declaration of war by the United
States and the United
Kingdom, against Iraq, in 2003. The General Assembly’s 122nd plenary meeting in
2008 decided to facilitate

. Page 33 of 40
the reform process through the Inter-Governmental Negotiations framework (IGN) on
equitable representation
as well as expansion of the UNSC Though the General Assembly’s adoption of a 2015
resolution to allow the
IGN on the basis of a framework document generated some enthusiasm, it was dampened
by the U.S., Russia
and China being opposed to serious reform of the Council. The G4 bemoaned earlier
this year that the IGN
process might have outlived its purpose given the absence of a negotiating document
which alone could provide
a structure to the deliberations. In any case, the exercise has been deferred in
view of the COVID-19 pandemic.
India’s election in June as a non-permanent member of the UNSC, obtaining 184
votes, was a diplomatic
triumph, notwithstanding that it was the lone contestant for the Asia-Pacific seat.
But in a sign of the difficulties
ahead to achieve New Delhi’s ultimate objective, reforms to the UN figured no more
than as part of a broader
vision in the declaration to commemorate the organisation’s 75th anniversary. The
political and economic
architecture of the emerging global order that the allied powers shaped at the end
of World War II has been
altered since then. The UN remains unreflective of the current trajectory,
especially in the strategic and economic
arenas. The multilateral framework now faces an unprecedented challenge — to
fashion a collective response to
humanity’s biggest problems, which include global warming and the pandemic.
Paradoxically though, the post#war order faces an existential threat to its
stability from the revival of nationalism across the globe, with some
of the powers that enshrined common principles and rules willing to discard them.
All countries must have the
voice to influence policy.
121. We can infer from the first sentence of the passage that
(a) Even after, seventy-five years, the United Nations is following a beaten path
despite in need for internal
reforms.
(b) The need for internal reform within the United Nations to cater to the 21st
century is more conspicuous than
ever.
(c) The United Nations is ready for the 21st century as it has made all the
necessary internal reforms.
(d) There is a remarkable need for internal reforms in the United Nations for the
next century.
122. What can be drawn further if India were to realize its ultimate objective?
(a) India will find a permanent seat with veto power along with the five nations at
the security council.
(b) India will be the first to represent Asia-Pacific as a permanent member of the
security council.
(c) India will have more of a say in the current trajectory especially in building
global strategies.
(d) All of the above.
123. “This instrument is often wielded as a blunt weapon to shore up their
geopolitical interests, regardless of the
disastrous consequences for the victims of armed conflict. If the above statement
is true, what are the likely
consequences, in case of a war between nations?
(a) The nations with the power to veto will have hegemony over other nations and
will use it to their advantage
to further their own interests.
(b) The nations with the veto power will remain neutral in case of war between
nations to thwart any attempts
at disrupting global peace.
(c) The warring nations will find support from one of the nations with veto powers
which will put others at a
risk.
(d) There will be a situation very similar to World War II, if the nations at war
are protected by any of the
countries with the veto power.
124. What is the assumption behind the passage?
(a) The United Nations has outlived its existence with nothing to contribute in the
21st century.
(b) A need for a more democratic and a broader representation at the Security
Council with regards to
participation in policy making by other nations.
(c) The G4 countries need to find a voice to say their piece at the General
Assembly.
(d) India should now be more vociferous in demanding a permanent seat amongst the
five nations with veto
power

. Page 34 of 40
125. Paradoxically though, the post-war order faces an existential threat to its
stability from the revival of nationalism
across the globe, with some of the powers that enshrined common principles and
rules willing to discard them.
What is the basis behind the statement?
(a) The presence of United Nations is even more a need of the hour today.
(b) We are closer to an impending world war.
(c) Nationalism across globe is threatening the existence of the United Nations.
(d) None of the above.
Passage (Q.126-Q.130): A blanket gag order against the media is often fraught with
serious consequences for
both free speech and the citizen’s right to receive information. Orders by
different courts, restraining the media
from reporting on particular cases or programmes from being telecast, have drawn
attention this week to
questions of prior restraint, media freedom and the right of people facing
investigation to a fair trial. A quite
unusual and legally questionable decision has been the interim order of the Andhra
Pradesh High Court imposing
a ban on the media, and even social media, from mentioning anything in relation to
an FIR filed by the police
against a former Advocate General of the State and others. It is unusual in the
sense that there appears to be no
material to justify such censorship other than an allegation by the petitioner that
it is a “foisted” case. It is also
accompanied by an order staying the investigation itself. It is indeed open to a
High Court to grant a stay on
investigation in extraordinary cases. When political vendetta is alleged against
the government of the day, that
too by someone who had served a previous regime as a law officer, the need for
media coverage and public
scrutiny is all the greater. How the petitioner would benefit from the complete
absence of any reportage is
unclear. It prevents legitimate comment even to the effect that there is no
substance in the allegations.
Injunctions against publication can either be an order to prevent possible
defamation or invasion of privacy, or
one aimed at protecting the fairness of a trial or investigation. The Supreme Court
did hold in Sahara vs. SEBI
(2012) that the Court can grant preventive relief on a balancing of the right to
free trial and a free press. However,
it favoured such temporary restraint on publication “only in cases of real and
substantial risk of prejudice” to the
administration of justice or a fair trial. Meanwhile, the Supreme Court, on the
same day, passed a more important
interim order stopping the telecast of the remaining episodes of a series on
Sudarshan News on entirely different
grounds. Holding that the programme — four episodes were aired — was nothing but
vilification of Muslims,
the Court found it necessary to interdict the telecast of more episodes. The Court
seems to have made a distinction
between freedom of expression and propagation of hate. In recent years, there have
been quite a few instances,
especially in Karnataka, of omnibus interim injunctions against all media houses
obtained by some people solely
to prevent any news reporting about themselves. While claiming to be defamed by one
publication, they sue all
media outlets and obtain open-ended stay on publications, including those that are
hardly interested in writing
about them. As a matter of principle, courts ought to avoid omnibus orders against
publication. Such orders are
often to the detriment of the right to know.
126. “A blanket gag order against the media is often fraught with serious
consequences for both free speech and the
citizen’s right to receive information.” The inference drawn from the statement is
(a) A partial restraining court order of media’s freedom of expression can result
in misconstrued information
floating in public.
(b) A Government’s influence on courts’ decisions on freedom of expression can lead
to serious outcomes in
future, hampering democracy.
(c) A complete restraining of media expression by the court with regards to a case
frequently result in detrimental
ramifications.
(d) A complete ban by the courts on the media’s free speech is the order of the
day, as media usually interferes
with an ongoing trial.

. Page 35 of 40
127. By using a particular case as an example in paragraph one, the author is
trying to
I. Highlight the vendetta politics to malign the incumbent government without any
substantial proof.
II. Bring to fore a rare legally unjustifiable decisions by the court with respect
to media freedom.
III. Support the state court’s decision with regard to the free speech and
citizen’s right to receiving information.
IV. To adumbrate the arbitrary decisions taken by the courts with adverse future
implications with respect to free
speech and right to information.
(a) Only I (b) Only II (c) I and III (d) II and IV
128. Which of the following, if true, will weaken author’s stand on the legitimacy
of the court’s decisions?
I. A decision to curtail free speech of media that may pose a threat to national
interest is constitutionally right.
II. The courts have been vested with special powers to totally take away
expressions of free speech under
extraordinary circumstances.
III. Precedents have existed in similar situations where courts have given
injunctions on restraining media’s
freedom of expression even if the legitimacy of such orders have been in question.
(a) Only I (b) Only II (c) II and III (d) None of the above
129. A rational summation of author’s line of thought would be
(a) Courts must avoid omnibus orders against media and publication without actual
risk of prejudice.
(b) Courts need to pay heed to the information provided by the media that can
actually facilitate a sound verdict.
(c) Courts should refrain from arbitrary orders that can sabotage their influence
on people.
(d) Courts should be mindful of the impact their decisions make on the activists
filing the case.
130. The technique adopted by the author in the two paragraphs, is
(a) Paragraph one exposes the legitimacy of the court orders regarding the ban on
free speech in a specific case,
citing this as an unprecedented decision and paragraph two is an elaboration of the
arbitrary decision by the
court.
(b) The author uses paragraph one as a stage setting for his outlook on the
legitimacy of a decision by the state
court in a specific case and uses paragraph two to highlight as to what constitutes
right orders and giving his
own perspective.
(c) The author uses both the paragraphs to sound off the courts with regards to
their decision-making capacity
by citing specific cases.
(d) The author uses paragraph one to highlight the glaring discrepancies of the
court’s order pertaining to
publications in general and uses paragraph two to illustrate impeccable decision-
making capabilities of
Supreme-court.
Directions (Q.131-Q.133): Study the following data carefully and answer the
questions accordingly.
A certain number of people are sitting in a straight line facing the north
direction. At most six people are sitting
between H and J. Four people are sitting between Z and H. G sits second to the
right of L. H sits immediate right
of P. Six people are sitting between J and Z. More than three people are sitting
between P and G. Five people
are sitting between P and L.
131. How many people are sitting in this arrangement?
(a) 15 (b) 13 (c) 14 (d) 11
132. Find the correct statement.
(a) Three people are sitting to the right of L (b) P and H are immediate neighbors
(c) Two people are sitting between J and H (d) Z is not sitting at the end
133. How many of them sit to the left of J?
(a) Three (b) Seven (c) Five (d) Nine

. Page 36 of 40
134. In a certain code language, 'children of sky' is coded as '712', 'sky is blue'
is coded as '205' and 'of blue syrup' is
coded as '154'. Which phrase would the code '407' represent in the same code
language?
(a) syrup is children (b) children blue syrup
(c) sky blue syrup (d) syrup of children
135. Arrange the following things in a logical and meaningful order.
1) Pound
2) Milligram
3) Tonne
4) Gram
5) Quintal
(a) 2, 4, 1, 5, 3 (b) 4, 1, 5, 2, 3 (c) 1, 4, 3, 2, 5 (d) 3, 4, 1, 5, 2
mock 28
Directions(Q.1-Q.30): Read the following passage carefully and answer the questions
that follow.
Passage(Q.1-Q.6): There's only one company crazy enough to take on a centuries-old
product that is beloved
by people of all ages and backgrounds across the globe: Amazon. There's also only
one company that could
actually pull it off: Amazon. The Amazon company consistently pulls off hit
products that overtake the market,
even when the odds are stacked against them.
Then, in a world dominated by rectangular touchscreens, Amazon introduced a
cylindrical, voice-activated smart
device with no immediately clear use-case. Now Alexa is a household name and loved
by people around the
world. Amazon’s projects are always on the cutting-edge of the current consumer
technology landscape. They
push boundaries and the market usually goes along with it in a big way. The company
reported net revenue of
$177.87 billion in 2017. And though the majority of their revenue still comes from
online retail product sales,
products like the Kindle and Echo have turned Amazon into a tech superpower and
have paved the way for
Amazon to shape the future of consumer tech interactions.
Part of their ability to build grand slam products is their famous customer –
centric mindset. But it’s more than
that. The key to Amazon’s success lies in their ability to deliver products that
are somewhere in between what
customers say they want and what they don’t know to ask for yet. Finding this sweet
spot means a lot of research,
iterations, and the occasional total failure. But as long as Amazon is trying and
testing ideas, they’re learning
more about what users want and how to anticipate what comes next. This is the
mission-critical information.
There’s so much to say about how Amazon became an e-commerce behemoth and tech
leader.
But the process that Amazon uses to build and market consumer tech products is more
important for other
startups to understand. With the Kindle and the Echo-both of which are pretty
different products – Amazon was
able to take over entirely new product categories and win the hearts of the market.
How Amazon digitized the
“last bastion of analogue” so successfully that the Kindle became more popular than
they could have imagined.
The book has never needed an update. Amazon didn’t need to build an e-reader to
become a successful company.
But the Kindle has been one of the big keys to the company’s growth over the past
decade because it proved that
Amazon could bring out great consumer products.
Selling books was a great way for Amazon to get a start in e-commerce. In 1994,
Bezos created Amazon on the
belief that the internet was a great place to sell books. He saw a huge, viable
market: books are standardized all
over the world, and there’s no need to see a book in person before you buy it.
Amazon staked their claim on the
internet, and they quickly became the leading online retailer of books.
1. From which of the following categories, does Amazon get the majority of its
revenues?
(a) Tech products like the Kindle. (b) Online product retail sales.
(c) Cloud computing services. (d) It's rocket building business.
2. What did the success of Kindle mean for Amazon?
I. The company could work on breakthrough technology.
II. The company could build great consumer products.
III. The company could provide great products at affordable prices.
(a) Both I & II (b) Only I (c) Both I & III (d) Only II

. Page 3 of 36
3. Which of the following has/have helped Amazon in being a successful company?
I. Their famous customer-centric mindset.
II. Their ability to deliver products that are somewhere in between what customers
say they want and what they
don't know to ask for yet.
III. Finding a match between innovative and price friendly.
(a) Both I & II (b) Both II & III (c) Both I & III (d) All of the above
4. Which of the following attributes of Amazon has been called as ‘the mission-
critical information’ by the author?
(a) Amazon's projects are always on the cutting-edge of the current consumer
technology landscape.
(b) Amazon pushes boundaries and the market usually goes along with it in a big
way.
(c) Amazon tries and tests ideas, which enable them to learn more about what users
want and how to anticipate
what comes next.
(d) The Amazon company consistently pulls off hit products that overtake the
market, even when the odds are
stacked against them.
5. Which of the following attributes describe(s) Alexa, as talked about in the
passage?
I. The only colour variants are black and blue.
II. It's cylindrical.
III. It's a voice-activated smart device.
(a) Both I & II (b) Both II & III (c) Only II (d) Both I & III
6. Which of the following would be the antonym of 'viable', as used in the passage?
I. Visionary
II. Impractical
III. Illogical
IV. Pragmatic
(a) Only I (b) Both II & IV (c) All of I, II & IV (d) Both II & III
Passage(Q.7-Q.10): By regarding the expanding universe as a motion picture, you can
easily imagine ¯running
the film backward. If you do so, you find the universe getting smaller and smaller,
and eventually you come to
the moment when its whole mass is crammed into an infinitely dense point. Before
that time it didn’t exist, or at
least it didn’t exist in its present form.
Though there is some controversy about its exact age, most cosmologists would be
inclined to agree that the
universe has existed for about ten to twenty billion years. For scale, this can be
compared to the four-and-a-half#billion-year age of the solar system, the time
since the disappearance of the dinosaurs (sixty-five million years),
and the age of the human race (about three million years).
The event that marked the beginning of the universe was christened the Big Bang;
the term has now entered the
vernacular of our culture. Originally the name referred only to the single
initiating event; now, however,
astronomers have come to use it to mean the entire developmental process of the
birth and expansion of the
cosmos.
The simple statement that the universe had a beginning in time is by now so obvious
to astrophysicists that few
give it a second thought. Yet it is a statement that has profound implications.
Most civilizations embrace one of
two opposite concepts of time. Linear time has a beginning, duration, and end;
cyclical time, as its name suggests,
continues around and around forever. In a universe that functions through cyclical
time, the question of creation
never arises; the universe always was and always will be. The minute you switch to
linear time you immediately
confront the vexing question not only of creation, but also of the Creator.
Although there is no logical reason for
the assumption, many people believe that if something comes into existence, it must
do so in response to the
actions of some rational being. Because of that belief, astronomers, even though
they resist becoming involved

. Page 4 of 36
in theological discussion, find themselves in one when they posit the Big Bang
universe. It puts them squarely
in the middle of an age-old debate.
One common misconception about the Big Bang that should be disposed of immediately
is the notion that the
universal expansion is analogous to the explosion of an artillery shell. The
galaxies are not like bits of shrapnel
speeding away from a central explosion. The raisin-in-dough analogy is a more
satisfactory way to think about
the whole process. The raisin-in-dough analogy is a more satisfactory way to think
about the whole process.’
The raisin-in-dough analogy with regards to the universe is that the expanding
universe is like a rising dough
with raisins in it. (The raisins in the universe are the galaxies) As the bread
rises, the raisins move away from
each other but are still held together by the dough.
7. In the context of the passage, the phrase ‘the Big Bang’ refers to
(a) Cosmology.
(b) the beginning of God.
(c) the birth and expansion of the cosmos.
(d) the birth of Earth.
8. Which of the following reflects the title of the passage?
(a) The Big Bang: the beginning of the Universe.
(b) Both (c) and (d)
(c) The concept of time has two different interpretations.
(d) Universe is a big challenge.
9. A common misconception regarding the Big Bang is that
(a) it cannot be defined in the temporal sense.
(b) the expansion of the universe is similar to the explosion of arteries.
(c) the Big Bang does not follow a linear time sequence and cyclical time sequence.
(d) the Big Bang is still refuted by a large number of astrophysicists.
10. What can be drawn from the raisin-in-dough analogy regarding the universe?
(a) The universe has been compared to the dough that is expanding and there may be
galaxies that remain close,
held together by the universe.
(b) The universe has been compared to the dough that is expanding and there may be
galaxies that have come
closer and are held together by the universe.
(c) The Universe is like a raisin that moves away upon its expansion.
(d) The universe has been compared to the dough that is expanding and there may be
galaxies that have moved
away but are still held together by the universe.
Passage(Q.11-Q.15): Inflation is in the news. Double-digit inflation persists,
concentrated in prices of food and
necessities. Yet, some economists and financial experts believe there is little
reason for concern. There is a boom
in purchases of consumer durables. The middle class is prospering. The poor are
better-off with the NREGA.
And people are no longer afraid of inflation. Such a worldview is, to say the
least, misleading. It needs a reality
check.
The woman in the household or the man in the street is not persuaded by statistics
on rates of inflation. In the
mind of the citizen, there is a ‘price perception index’ which is based on prices
paid. The social and political
threshold of tolerance for inflation in India has always been low, because a large
proportion of the population is
poor and an even larger proportion does not have index-linked incomes. Governments
are sensitive to inflation
because elections have been lost on the price of onions. In the past, persistent
double-digit inflation was
unacceptable to people and to governments. This time around, there are no visible
signs of anger among the
people, just as there are no obvious signs of anxiety in the government. Why?

. Page 5 of 36
Silence on the part of people is a puzzle. But there are some plausible
explanations. For the rich, small in number
and large in influence, food is such a negligible part of their expenditure that
food prices do not matter. For the
middle class in the private sector, beneficiaries of rapid economic growth, incomes
have increased significantly
more than prices and expenditure on food as a proportion of their household budgets
has come down.
For the middle class in the government sector, following the Sixth Pay Commission,
higher salaries combined
with substantial arrears paid, meant that their purchasing power increased
considerably more than prices. In the
past, whenever there was double-digit inflation, it was these two segments of the
middle class with a voice that
organised protest and shaped opinion.For the poor, making ends meet to simply
provide food for their families
is such a struggle that they have no time to protest. Their silence does not mean
acceptance. It is just that they
do not have a voice.
Most important, perhaps, the government does not quite know what to do. Some hope
that inflation will come
down in six months, but relying on statistics or words is not enough. Some assert
that inflation is the price of
growth, even if those who lose from inflation are not those who gain from growth.
Some believe that raising
interest rates and tightening credit would help combat inflation, without
recognising that this inflation is
attributable to supply-demand imbalances rather than excess liquidity; if the
diagnosis is wrong the prescription
cannot work.
It would be a serious mistake for the government to conclude that people are now
willing to live with higher
inflation or that their tolerance is greater than before. Persistent inflation,
particularly in food, hurts the poor.
Slowly but surely, resentment mounts. The number of people affected could be as
much as half our population.
Even if they do not have a purchasing power in a market economy, come election
time, they do exercise their
right to vote in a political democracy.
11. The author of the passage attributes inflation to
(a) the price of growth and development.
(b) imbalances in demand and supply.
(c) government complacency.
(d) excess liquidity.
12. The passage can be described as
(a) analytical. (b) informative. (c) descriptive. (d) discursive.
13. In the author’s opinion, the view of economists and financial experts on
inflation
(a) is no cause for concern. (b) is unrealistic.
(c) is dangerous. (d) is baseless.
14. Which of the following ideas is NOT suggested in the passage?
(a) The purchasing power of the middle class employed in the government sector has
gone up.
(b) Double digit inflation can turn political fortunes.
(c) The rich spend rarely on food.
(d) Hike in food prices does not worry the rich.
15. ‘Governments are sensitive to inflation because elections have been lost on the
price of onions.’ Which of the
following would be a suitable summation of the statement?
(a) The price of the onion decides the government.
(b) Sensitive inflation before elections.
(c) The government’s destiny.
(d) The political price of onion.

. Page 6 of 36
Passage(Q.16-Q.20): Paragraph 1: The fireworks have been ordered. Street parties
are planned. The Iraqi
government has prepared a week of festivities to mark the fall of Abu Bakr al-
Baghdadi's self-proclaimed
caliphate. Three years after seizing control of the great alluvial plains of the
Tigris and the Euphrates, Islamic
State, which has claimed so many victims in north-western Iraq, Syria and beyond,
is finally dying. American#led forces in Syria breached the old city walls of IS's
capital, Raqqa, on July 4th. In Mosul, in Iraq, all but the
last alleyways of the Old City were back in government hands.
Paragraph 2: Finding a backdrop from which to celebrate the liberation of Mosul
will be difficult, though.
Between them, IS and the coalition have destroyed too many shrines and mosques to
leave much of historic
value, including the al-Nuri mosque dating back to Crusader times from where Mr
Baghadi proclaimed himself
caliph. Gone is the Jewish quarter, the markets with their monasteries, and the
lattice balconies and sculptured
masonry of another Sunni silk road city.
Paragraph 3: The odour of putrefaction hangs in the hot air. Cameramen entering the
honeycomb of the Old
City return with footage of bodies, old and young, lying under blankets in front
rooms for days. Many more are
buried under the rubble. Thousand trapped in the alleys IS still holds are without
water or food. Iraqi soldiers
maintaining a siege prevent aid workers from getting food to those inside.
Paragraph 4: Previous battles for the city have been short-lived. In 1918 and 2003,
opposition melted away
when British and American forces respectively marched in. Even in the earlier
battle for east Mosul, SI fighters
coded one neighbourhood after another, and retreated west over the Tigris and into
Syria. But besieged in their
western redoubt, they have nowhere else to go. Unlike during the fall of Aleppo to
the Assad regime earlier this
year, no one has negotiated safe passage in the final stages of battle. The
fighting goes on alley by alley.
Paragraph 5: The clear-up operation will do much to determine whether Iraq’s
government wins the war as
well as the battle. Mosul’s exodus has realised the UN’s worst-case predictions.
About 9,00,000 of the city’s 2m
people have been displaced; 7,00,000 are still homeless. The prime minister, Haider
al-Abadi, wants to get people
home quickly, but the UN says 2,00,000 have none to return to. Most of the latter
come from poor
neighbourhood, like the Old City, where IS found many recruits. Strewn across Iraq,
they may now spread their
anger countrywide.
16. According to the passage, which of the following is the nearest interpretation
of the expression, 'hanging in the
hot air'?
(a) To be uncertain of something.
(b) Something which doesn't go away.
(c) Something which has become deadly.
(d) To be lost forever.
17. Based on the passage, which of the following is true?
I. In 1918 and 2003, opposition melted away when Irish and American forces
respectively marched in.
II. Cameramen entering the honeycomb of the Old City return with footage of bodies,
old and young, lying
under blankets in front rooms for months.
III. Thousands trapped in the alleys IS still holds are without water or food.
(a) Only I (b) Only II (c) Only III (d) Both I & III
18. According to the passage, why does the author say that the celebration of the
liberation of Mosul will be difficult?
(a) Because although the IS have fallen, their leader Abu Bakr al-Baghdadi is still
at large.
(b) Because too much history of the city has been uprooted.
(c) Because it will take time to organize and regulate the people and, hence, the
city can't celebrate before doing
that.
(d) Because Mosul is under the siege of the British and American forces.

. Page 7 of 36
19. According to the passage, what does the author want to communicate through the
second paragraph?
(a) The author is describing the present state of the city of Mosul.
(b) The author is sceptical regarding the way forward for the city.
(c) The author is criticising the government for planning celebrations after such a
disaster.
(d) Both (a) and (c)
20. Which of the reflects the writing style adopted by the author for the passage?
(a) Narrative (b) Expository (c) Persuasive (d) Descriptive
Passage(Q.21-Q.25): Recently, in a court case, lawyers argued that the deity in a
temple is a legal entity and
therefore, the deity’s legal rights have to be protected. The deity’s legal rights
and desires and wishes are
expressed through the hereditary priests who manage the temple and who have been
doing so for several
generations. This draws our attention to a very important but overlooked theme in
Indian society where Gods
play a significant role in establishing society. Deities have always played an
important role in social engineering
and we have to think about it carefully.
In ancient tribes how did you bind people together? Everybody had a different
opinion, this could have led to
tensions between members over resource allocation, job allocation. One of the ways
in which the tribe was kept
together was by establishing an impersonal deity, to whom all are beholden and this
deity’s wishes were
expressed through the dreams of shamans and everybody submitted to this dream.
Modern institutions are essentially impersonal entities that are treated by law, as
a person and, therefore with
rights and responsibilities. In ancient times, this role of an institution was
played by a tribal god or village god
(grama devata) or the clan god (kula devata).
Authority came from a supernatural force. The assumption here was that supernatural
and impersonal entities
are fairer. However, life is never so simple as the gods cannot speak, just as an
institution cannot speak. An
institution expresses its will through its board of directors and its management
committee, in the same way a
deity expresses its will through its management committees, the priests and
trustees. The earliest idea of
institution comes from the Roman Empire, where the senators owed allegiance not to
the king or Caesar but to
the city of Rome. Later, Rome was replaced by the Church and the priestly class
functioned in the name of the
church.
In India, many people abhor the idea of giving taxes to the state but they have no
problem in giving large
amounts of money, even shares of their companies, to the deity because they feel
they will receive karmic
dividends. Thus, temples are flush with funds which are then managed by the
committees of the temple, which
in a way embodies the will of the deity in a temple. With the help of the temple,
and its impersonal deity, the
priests helped the king bind various clans and communities and tribes. Indian
people believe in Karmic Dividends
and they think that by making donations to religious institutions, they will earn
the blessings of God and by
paying income tax they do not get anything in return.
One can argue that deities in India, and their temple culture, through the priestly
class functioned as an
impersonal proto-institution that bound communities together and helped legitimize
the kingship. It made the
kingdom not the king’s property but the king’s responsibility. The king was merely
a trustee who took care of
the deity’s people. But the quality of the deity’s will like the quality of an
institution’s value is a function of the
biases of the management committee. It is time their ‘divine’ dreams moved out of
medieval times and became
more in line with the modern times.
21. What would be the appropriate antonym for the word ‘abhor’ as used in the above
passage?
(a) Fancy (b) Regard (c) Prefer (d) Adjourn

. Page 8 of 36
22. What view does the author have on the role of deity in the country’s social
fabric?
(a) The author does not believe in this notion as he thinks human beings are solely
responsible for everything.
(b) The author likes the idea of a supreme power and is ecstatic with the
possibility of creating a world founded
around a God so that people can serve the power.
(c) The author is cautious about the deity's relation to the nation's social system
(d) The author is of the view that, Indians are unreasonable as they donate
everything to the deity without
actually seeing where it goes.
23. Why do Indians detest paying taxes but like to donate money to religious
institutions?
(a) When one makes donations, income tax is withheld and, thus, people make
donations in order to obtain
income tax deductions.
(b) Income Tax is paid to government without getting anything in return, however
making donations to temples
gain karmic blessings from God.
(c) Income Tax payment is obligatory and, thus, people detest it.
(d) Making donations to a temple makes people happy and fulfils their emotional
needs.
24. Which among the following is synonym of the word Allegiance as used in the
passage?
(a) Belief (b) Credibility (c) Loyalty (d) Commitment
25. Which of the following statements are true in the sense of temples and
organization being similar?
(a) Temples and Organizations have overflowing money but they don’t know where to
use it.
(b) They both are entities which are regulated by rules and laws.
(c) The responsibility of both relies on the management force's mindsets.
(d) They both believe in supremacy and control of the minds of people.
Passage(Q.26-Q.30): Scholars have always been people of the book, so it seems wrong
that this faithful
companion has been put on the defensive. Part of the problem is knowing what we
mean exactly when we say
"book." It's a slippery term for a format, a technology, a historical construct,
and something else as well.
The book has a long story to tell, one that might be organized around four epochal
events, at least in the West.
In the beginning was the invention of writing and its appearance on various
materials. The second was the
development during the first years of the Christian era of the codex—the thing with
pages and a cover—first as
a supplement and eventually as a replacement for the older technology of the
scroll. The third was the European
deployment of movable type, in the 15th century. And the fourth is, of course, the
digital revolution in the middle
of which we find ourselves today.
When we say "book," we hear the name of a physical object, even if we're thinking
outside the codex. The codex
bound text in a particular way, organizing words into pages, and as a result
literally reframed ideas. The static
text image on my desktop is the electronic cousin of late antiquity's reading
invention. When my screen is still,
or when I arrange text into two or four pages, like so much visual real estate, I
am replicating a medieval codex,
unbinding its beautifully illuminated pages. Yet reading digitally is also a
scroll-like engagement - the fact that
we "scroll down" connects us to a reading practice that dates back several
millennia. One of the things that book
historians study is the change in, and persistence of, reading technologies over
time, and what those historians
have demonstrated is that good technologies don't eradicate earlier good
technologies. They overlap with them
- or morph, so that the old and the new may persist alongside yet another
development. Think Post-its, printed
books, PCs, and ipods, all in the same office cubicle.
The concept of the "history of the book" is a scholarly new discipline, that
evolved as recently as the 1950s.
Book history's objective was analysis of the function of the book in culture, and
since the 1970s, it has continually
expanded its scope, emerging as a trading zone among various disciplines, where the
work of librarians,

. Page 9 of 36
archivists, and scholarly publishers can intersect with the work of traditional
scholars and theorists, all members
of what is now called the "knowledge industry."
In the long night of culture, we knowledge workers are restless sleepers. We need
dreamers - in technology and
science as well as the arts. Right now we are walking through two great dreams that
are shaping the future of
scholarship, even the very idea of scholarship and the role "the book" should play
within it.
26. Why does the author say that the codex 'literally reframed ideas'?
(a) Because it coincided with the emergence of new ideas in the literary field
(b) Because it changed the way written expression was presented, from a scroll to
pages bound between covers.
(c) Because it was the precursor to the e-book that has pages framed on the screen.
(d) Because it was a new, despised and significant, medium of written expression.
27. Which of the following best describes the concluding paragraph?
(a) The paragraph uses analogy to represent the significant contribution, by
science and art, to the development
of the book.
(b) The paragraph depicts the authors visualisation of how the book will shape
science and technology.
(c) The paragraph aptly concludes the discussion on the evolution of the book by
giving a glimpse of its future
shape.
(d) The paragraph is a metaphorical representation of the influence of art and
technology on the future of the
book and scholarship.
28. The author refers to the Post-its, printed books, PCs and iPods found in the
office cubicle
(a) to demonstrate that electronic gadgets have not been able to replace the
printed word.
(b) to suggest that the good old technologies are the ones that evolve into the new
ones.
(c) to show that good old technologies may evolve into changed forms but are not
wiped out by newer
technologies.
(d) to exemplify how new technologies borrow ideas from older forms that have
proven themselves.
29. The author calls the book 'a slippery term' because
(a) it encompasses not just a form and technology but beyond the limiting
definition.
(b) it has been changing its form in response to changes in technology, and is
therefore easy to define.
(c) it has altered so drastically that it is difficult to think of a scroll and an
e-book as essentially the same thing.
(d) the changes over time have affected how it is perceived and received by the
general public.
30. What is the author trying to communicate through the passage?
(a) The book is a slippery term as it entails collections of historical data and
collates them into texts.
(b) The book cannot be narrowed down to a physical text as it is a reading
technology that morphs the older
technology and the newer technology to merge into the future.
(c) The book is a broad concept of the accumulated knowledge through text of the
past that gets eradicated as
the new text of present knowledge evolves.
(d) None of these

Directions (Q.66 – Q.105): Read the comprehensions carefully and answer the
questions based on it.
Principle (Q.66-Q.71): Criminal Conspiracy is defined under section 120 A of the
Indian Penal Code. It refers
to an agreement between two or more persons to commit an offence or carry out an
illegal act by unlawful means.
The proviso to the section sheds more light on the scope of liability as follows:
No agreement except an agreement to commit an offence shall amount to a criminal
conspiracy unless some act
besides the agreement is done by one or more parties to such agreement in pursuance
thereof subject to the details
discussed below.
The Explanation to the section further entails that it is immaterial whether the
illegal act is the ultimate object of
such agreement, or is merely incidental to that object.
Thus, in situation where the agreement is not regarding commission of offence an
act in pursuance is required
but in case of an agreement of commission of offence, mere agreement is sufficient
to make the offenders liable.
An act includes omission as well. Section 43 of the IPC states that “The word
illegal is applicable to everything
which is an offence or which is prohibited by law, or which furnishes ground for a
civil action ; …”.
The word civil includes breach of contract.
Thus, the following are the necessary ingredients in order to constitute
conspiracy:
1. There must be two or more persons;
2. There must be an illegal act or an act in an illegal way;
3. There must be a meeting of minds;
4. There must be an agreement regarding the same thing.
Source: https://www.barandbench.com/columns/criminal-conspiracy-law-applicability
66. Rita and Geeta wanted to kill their employer, Ramesh. They both met one day and
agreed to kill Ramesh with a
knife. On the day of killing, Geeta killed Ramesh with the knife whereas Rita was
not there on the crime scene.
Decide whether they both can be liable for conspiracy?
(a) Only Geeta will be liable as Rita did not kill Ramesh.
(b) Both Rita and Geeta will be liable for conspiracy as it was planned by both of
them.
(c) Neither Geeta nor Rita will be liable for conspiracy and Geeta will be liable
for murder.
(d) Only Geeta will be liable as Rita abandoned the plan before killing Ramesh.
67. A, B and C were arrested on the charge of committing criminal conspiracy for
forging documents. However, the
documents could not be forged and they were arrested. In the trial A and B were
acquitted on merits. Can an
offence of criminal conspiracy be made out against C?
(a) Yes, C can be made liable as he wasn’t acquitted.
(b) No, C cannot be made liable as the documents were not forged.
(c) No, C cannot be made liable as A and B were acquitted.
(d) Yes, C can be made liable as forging of document is immaterial.
68. Ram and Shyam were jogging in a park when Ram shared with Shyam that he is
planning to kill his wife Shagun.
Shyam didn’t dissuade him from killing Shagun as he dismissed it as a joke. Next
day Shagun is killed and Ram
names Shyam as a co-conspirator. Will Shyam be liable?
(a) Yes, Shyam will be liable as he didn’t stop Ram from killing Shagun.
(b) Yes, Shyam will be liable because he knew about Ram’s plan.
(c) Yes, Shyam will be liable because Ram discussed about his plan with him and he
didn’t refuse him which
amounts to agreement.
(d) No, Shyam will not be liable because he didn’t agree to Ram’s plan.

. Page 17 of 36
69. Vikram and Tanisha are both post graduates and plan to elope and decide to get
married against the wishes of
their parents. When the news of their marriage reaches Tanisha’s parents, her
father suffers a massive heart attack
due to shock and dies. Tanisha’s mother files a case of criminal conspiracy against
the couple. Will they be
liable?
(a) No, the couple will not be liable because they didn’t agree to commit any
offence or an illegal act or an act
in an illegal way.
(b) Yes, the couple will be liable as they knew that the news of their marriage
will shock the parents and adverse
things could happen.
(c) Yes, the couple will be liable as they had planned to elope and marry.
(d) None of the above.
70. Sameer and Tisha had planned to breach their contract with Kareem. Three days
later they breached the contract
and Karim filed a complaint against Sameer and Tisha. Can Sameer and Tisha be held
liable under criminal
conspiracy?
(a) Yes, Sameer and Tisha can be held liable as what they had planned and done an
illegal activity.
(b) Yes, Sameer and Tisha can be held liable as breach of contract is an offence.
(c) No, Sameer and Tisha cannot be held liable as remedy for breach of contract is
compensation.
(d) No, Sameer and Tisha cannot be held liable because breach of contract is not a
criminal offence.
71. Sameer and Tisha had planned to breach their contract with Kareem. Three days
later Kareem came to know
about their plan and filed a complaint against Sameer and Tisha. Can Sameer and
Tisha be held liable under
criminal conspiracy?
(a) Yes, Sameer and Tisha can be held liable as it is immaterial whether the act
was actually done.
(b) Yes, Sameer and Tisha can be held liable as breach of contract is a wrong and
they had planned to commit
the illegal act.
(c) No, Sameer and Tisha cannot be held liable as no overt act was done.
(d) No, Sameer and Tisha cannot be held liable because breach of contract is not a
criminal offence.
Principle (Q.72-Q.77): Section 84 I.P.C. lays down that "nothing is an offence
which is done by a person who
at the time of doing it, by reason of unsoundness of mind, is incapable of knowing
the nature of the act, or that
he is doing what is either wrong or contrary to law".
It deals with the deficiency of will due to weak intellect due to medically
diagnosed unsoundness. The legal
conception of insanity differs considerably from the medical conception. It is not
every form of insanity or
madness that is recognised by law as a sufficient excuse. To establish a defence on
the ground of insanity, it
must be clearly proved that, at the time of committing the act, the party accused
was labouring under such a
defect of reason, from disease of the mind, as not to know the nature and quality
of the act he was doing; or, if
he did know it, that he did not know he was doing what was wrong"
The principle is based upon the maxim furiosi nulla voluntas est. The burden of
proving the existence of
circumstances bringing the case under Section 84 I.P.C. is thrown on the accused
and Section 105 of the Indian
Evidence Act directs that "the court shall presume the absence of such
circumstances".
Source: https://www.livelaw.in/top-stories/depression-not-unsoundness-of-mind-for-
s-84-ipc-sc-junks-appeal#against-dismissal-by-ex-armyman-165257 and Section 84, IPC
72. A father dreams that God is commanding him to sacrifice his son. Next morning
the father bathes in a river and
dresses him in new clothes. Thereafter, he takes the son in front of the God and
slits his throat. The son dies.
After returning he tells everything to his brother without any remorse. Later, when
police arrives he attempts to
hide himself. Decide whether the father can be given defence of Section 84, IPC.
(a) No, the father cannot be given the defence of Section 84, IPC as the son is
killed by him.
(b) No, the father cannot be given the defence of Section 84 because he knew the
nature of his act is contrary to
law.
(c) Yes, the father can be given the defence because he believed that the God
commanded the sacrifice.
(d) Yes, the father can be given the defence because his conduct displays that he
is unaware about the nature of
his act.

. Page 18 of 36
73. X was suffering from schizophrenia and he believed that Y wanted to kill him.
One day he thought that Y is
attaching him with a knife and in an attempt to save himself he pushes Y down the
stairs and Y dies. The lawyers
contesting on behalf of X wish to use the defence under Section 84. Decide.
(a) X should not be given a defence under Section 84 because he has committed an
offence.
(b) X should not be given a defence as he could have called police.
(c) X should be given a defence because he was suffering from schizophrenia which
led him to genuinely believe
that Y wanted to kill him.
(d) X should be given a defence because he acted in private defence.
74. X was suffering from schizophrenia which was medically diagnosed and he
believed that Y wanted to kill him.
One day he saw Y and planned to kill Y although at that time he didn’t apprehend
that Y wish to kill him. He
took a knife and killed Y. He took the defence that he was suffering from
schizophrenia. Decide.
(a) X should not be given a defence under Section 84 because he knew the nature of
the act he was doing while
he was doing it.
(b) X should be given a defence because he has been medically diagnosed with
schizophrenia.
(c) X should be given a defence because he was suffering from schizophrenia which
led him to genuinely believe
that Y wanted to kill him.
(d) X should be given a defence because he acted in private defence.
75. A Husband got extremely mad at his wife and in the rage of his anger he
couldn’t distinguish right from wrong
and grievously injured his wife. The family members kept on shouting that he has
gone mad. He only stopped
when neighbours came. When the wife complained against him, he took a defence of
Section 84 and stated that
something came over him and he couldn’t understand what he was doing. Decide.
(a) Husband shouldn’t be held liable because he was so angry that he couldn’t
understand what he was doing.
(b) Husband shouldn’t be held liable because the family members also stated that he
has gone mad.
(c) Husband should be held liable as getting angry is not unsoundness of mind.
(d) Husband shouldn’t be held liable because he wasn’t in his senses.
76. A person was undergoing treatment for unsoundness of mind. He recovered from
his illness and started to lead
a normal life. One day he purchased a gun and started following his colleague. When
the colleague got suspicious
and turned, the person hid himself. Later that night, the person went inside the
colleague’s house when he knew
nobody would be inside except his colleague and shot him dead. After a few weeks of
this incident, he again
developed symptoms of unsoundness. Should he be given defence of Section 84?
(a) Yes, he can be given the defence of Section 84 as his illness resumed.
(b) Yes, he can be given the defence of Section 84 as he did not know about the
nature of his act.
(c) No, he cannot be given the defence of Section 84 as his illness was not
continuous.
(d) No, he cannot be given the defence of Section 84 as he was not suffering from
unsoundness while he was
doing the act.
77. An accused was suffering from unsoundness of mind. He developed hatred for
human life and one day he killed
his wife. In the trial, he didn’t raise the defence of Section 84. Will he be
liable?
(a) No, the court shall presume that he is unsound.
(b) Yes, he will be liable as the burden of proving unsoundness lies on the
accused.
(c) Yes, the accused should be liable because he knew what he was doing.
(d) No, the accused shouldn’t be liable as he was suffering from unsoundeness.

. Page 19 of 36
Principle (Q.78-Q.83): In law of tort, the damage caused may be intended or
unintended. However, in case of
unintended damage the test of deciding whether a person is liable for the damage
caused is through the test of
directness and test of reasonable foresight. Test of directness makes a person
liable for all the wrongs done by
him irrespective of the fact whether he could foresee it or not. Test of reasonable
foresight or remoteness of
damage means a person will be held liable for the consequences which an ordinary
person under that
circumstance can foresee. A person cannot be held liable for ad infinitum
consequences. In deciding whether the
claimed damages are too remote, the test is whether the damage is such that it must
have been considered by the
wrongdoer as a possible result of the breach. If it is, then it cannot be
considered too remote. Actual knowledge
must be shown knowledge is not merely imprudence and carelessness. A tortfeasor, if
he was negligent cannot
escape liability by saying that the plaintiff suffered an injury because he was
working under extreme
circumstances and therefore the harm caused is much worse than what could be
foreseen due to the negligence.
Source: https://www.jstor.org/stable/24752817
78. A ragpicker threw a lighted cigarette on the haystack he had just collected on
the road. The haystack caught the
fire and burnt the complete compound to the ashes. Decide whether the ragpicker can
be held liable under
remoteness of damages.
(a) The ragpicker can only be held liable for burning the haystack.
(b) The ragpicker cannot be held liable as he could not have foreseen the building
catching fire.
(c) The ragpicker can be held liable because he could have foreseen that the
haystack is in a building premise
and putting a lighted cigarette on haystack can burn down the building.
(d) The ragpicker cannot be held liable because he didn’t intend to destroy the
building.
79. A batsman hit the ball in a stadium. The ball crossed the walls of the stadium
and it was the first time something
like this ever happened. The ball hit a person who was walking outside the stadium
in the head an injured him.
The person is claiming damages from the batsman under the test of directness.
Decide.
(a) The batsman will be liable as it doesn’t matter if he could foresee the injury
or not.
(b) The batsman will not be held liable as he did not intend to cause this injury
to the person.
(c) The batsman will not be held liable because he could not have foreseen a man
walking outside the stadium.
(d) The batsman will be held liable because the man got injured.
80. A batsman hit the ball in a stadium. The ball crossed the walls of the stadium
and it was the first time something
like this ever happened. The ball hit a person who was walking outside the stadium
in the head an injured him.
The person is claiming damages from the batsman under the test of reasonable
foresight. Decide.
(a) The batsman will be liable as it doesn’t matter if he could foresee the injury
or not.
(b) The batsman will not be held liable as he did not intend to cause this injury
to the person.
(c) The batsman will not be held liable because he could not have foreseen that the
ball will cross the stadium.
(d) The batsman will be held liable because the man got injured.
81. A man was working in a chamber where he was exposed to carcogenic elements. He
was given a suit for his
protection by the employer. However, the quality of the suit was not good. One day
during the course of his
employment, the element fell onto the man which caused an injury to the man. The
man was diagnosed with
cancer which was accelerated by the injury. The man died ultimately. Action was
brought against the employer.
Decide
(a) The employer should not be liable as per test of directness as the man died of
cancer.
(b) The employer should not be liable as he could not have foreseen that the man
will die of cancer.
(c) The employer should be liable because the man got cancer and ultimately died
due to bad quality suit.
(d) The employer should not be liable as the person was voluntarily working in the
factory.

. Page 20 of 36
82. An oil spill occurred in an ocean due to negligence of a ship owner. The ship
owner tried his best to clean the
surrounding water to avoid damage. Few months later, it was reported that as an
outcome a lot of aquatic life
died on a shore far away. The fishermen sued the shipowner. Decide by applicability
of remoteness of damage.
(a) The ship owner will be liable because he didn’t cleanse the water properly.
(b) The ship owner will be liable because he could foresee that the spilled oil
will be harmful for the aquatic life.
(c) The ship owner cannot be liable because he could not foresee that the outcome
of the spill will be seen far
away after a few months.
(d) The ship owner cannot be held liable because he took corrective measures.
83. A scientist was experimenting on a genome sequencing in his lab. The experiment
went wrong and the scientist
was astonished to see the spread of the resultant virus Novid around the world
within a span of few days. Can
he be made liable under reasonable foresight?
(a) The scientist could not have fathomed such an outcome of the experiment and
thus, he can’t be liable.
(b) The scientist could not have known that the experiment will go wrong and thus,
he cannot be held liable.
(c) The scientist must be aware about the risks attached with such an experiment
and should have taken
precautions; thus, he will be liable.
(d) The scientist’s experiment has led to such disastrous outcomes, thus he should
be made liable.
Principle (Q.84-Q.89): An acceptance to a proposal must be absolute and
unqualified. According to Mulla, it is
called the Mirror rule. This means that the proposal and the acceptance must
correspond. If the acceptance is
conditional it does not amount to a valid acceptance, rather it amounts to counter
proposal. Proposal can only be
accepted by that person to whom it is made and while accepting the proposal must be
within the knowledge of
the acceptor. Another important characteristic of a proposal is that it should be
capable of creating legal relations
and this can be deciphered from the intention of the parties. Sometimes one
signifies to another not the
willingness but just information to attract customers. This is known as invitation
to proposal. A proposal can
only be revoked until acceptance is made.
84. An employer offered an incentive scheme for workers who would work 10 hours a
day on Wednesday. Kashish
usually worked for 10 hours a day. However, she was absent on Wednesday. When on
the salary day she saw
Meeta being paid incentive for working 10 hours a day, she came to know about the
scheme and demanded her
incentives as well. The employer denied the incentives to her. Decide
(a) Kashish will not be paid the incentives as she never accepted the incentive
offer.
(b) Kashish will be paid incentives because she fulfilled the conditions of the
offer.
(c) Kashish will not be paid incentives because only the employer can decide who
should get incentives.
(d) None of the above.
85. Shefali went to a coaching institute to enquire about CLAT classes. However,
the coaching institute informed
Shefali that they have limited seats which will soon get filled. The coaching
institute further informed her about
the requisite fee for 12 months. Shefali told them that she would join the classes
after discussing this with her
parents and left.
Shefali came back to find that all the seats have been booked and got very upset.
She claimed that she had made
a valid acceptance. Decide.
(a) No, Shefali had made a conditional acceptance.
(b) No, Shefali had made an offer earlier that the institute had accepted.
(c) Yes, Shefali had said that she would join the classes.
(d) Both (a) and (b)

. Page 21 of 36
86. Shefali went to a coaching institute to enquire about CLAT classes. However,
the coaching institute informed
Shefali that they have limited seats which will soon get filled. The coaching
institute further informed her about
the requisite fee for 12 months. Shefali told them that she would join the classes
after discussing this with her
parents and left.
Shefali came back to the institute to submit the fee but the institute denied the
admission. Shefali raised the
question that the institute is not right in revoking the proposal once an
acceptance has been made. Decide.
(a) The institute is wrong in revoking their proposal after Shefali has accepted
their proposal.
(b) The institute has not made an offer. The act of the institute is just an
invitation to the proposal.
(c) Shefali is right in claiming that she has made valid acceptance.
(d) The institute cannot deny admission as it is against trade practices.
87. Shefali went to a coaching institute to enquire about CLAT classes. However,
the coaching institute informed
Shefali that they have limited seats which will soon get filled. The coaching
institute further informed her about
the requisite fee for 12 months. Shefali told them that she would join the classes
after discussing this with her
parents and left.
Shefali came back to the institute and stated that she wants to pay the requisite
fee to study for 15 months. The
institute accepted the offer stating that they accept the offer to let her join the
classes for 12 months. Is this a
valid acceptance?
(a) The institute has not made a valid acceptance to the proposal and acceptance
don’t correspond.
(b) Shefali has made a valid acceptance as she has agreed to pay the requisite fee.
(c) The institute cannot make a valid acceptance to an invalid proposal.
(d) Shefali could not have made a valid proposal after she had left the institute
without confirming.
88. Shefali went to a coaching institute to enquire about CLAT classes. However,
the coaching institute informed
Shefali that they have limited seats which will soon get filled. The coaching
institute further informed her about
the requisite fee for 12 months. Shefali told them that she would join the classes
after discussing this with her
parents and left.
Shefali returned to the institute and offered them to let her study for the
requisite fee but she can only pay the
fee in installments as opposed to one-time payment as they had earlier discussed.
What is the nature of Shefali’s
statement?
(a) This is a valid acceptance as Shefali has agreed to pay the requisite amount.
(b) This is an absolute and qualified acceptance.
(c) This is a counter proposal.
(d) This is neither an acceptance nor a proposal.
89. A Husband offered his wife to come back to their matrimonial home in return of
a written and registered promise
of yearly trips together. The wife agreed to it. After a year, the wife sued the
husband as he failed to take her on
yearly trips as promised. Decide
(a) Wife cannot sue the Husband as the intention of husband was not to create legal
obligations.
(b) Wife cannot sue the Husband because they share domestic relationship.
(c) Wife can sue the Husband because the Husband wanted to create contractual
obligations.
(d) Wife cannot sue the Husband as the husband casually promised to get her back.

. Page 22 of 36
Principle (Q.90-Q.94): Article 25 of the Constitution guarantees to all persons the
freedom of conscience and
the right to freely profess, practise and propagate religion. These rights may be
exercised within the confines of
one’s private space or with the participation of the public, subject to public
order, morality and health.
The Supreme Court, in 2018, had banned the bursting of hazardous firecrackers
during Diwali throughout Delhi
NCR. The Court held that assuming this is a part of religious practice, if it
threatens the health and lives of people
(living in Delhi and NCR), it is not entitled to protection under Article 25 (being
subject to Article 21).
The Fundamental Right to Freedom of Religion is not an absolute right and is
subject to public order, health,
morality and other fundamental rights and any religion constitutes three kinds of
activities:
1) Pure religious rituals- The Government cannot interfere with this right as
guaranteed under Article 25 but it
is subject to public order, health, morality and other fundamental rights.
Example: How to perform a religious ritual, how to worship etc.
2) Activities closely associated with religion- Such activities can be reasonably
controlled by the State.
Example: Organizing Melas(fairs), use of loudspeakers
3) Non-religious activities: Such activities can be controlled by the State.
Example: Acquiring land for religious purposes, management of religious institutes
etc.
Source: https://www.barandbench.com/columns/courts-on-covid-19-health-and-
religious-freedom
90. Government of India created a trust “XYZ Shrine Trust” and took control of
management affairs of a temple.
The family who traditionally took care of the management of the temple objected to
the creation of this Trust
stating that the creation of this Trust violates their fundamental Right and they
also objected to the ban on use of
loudspeaker in the morning. The family specifically objected to the right of
appointment of Chief Priest being
given to the Trust.
Which of the following statement is true?
(a) Creation of XYZ Shrine Trust is unconstitutional.
(b) Creation of XYZ Shrine Trust is constitutional but government cannot interfere
in management of religious
affairs of the temple.
(c) Creation of XYZ Shrine Trust is constitutional and the government can interfere
in management of religious
affairs like banning use of loudspeaker but cannot appoint the Chief Religious
Priest.
(d) Creation of XYZ Shrine Trust is constitutional and government can interfere in
management of religious
affairs and can also appoint the Chief Religious Priest.

91. The government has decided to stop organization of a mahapooja owing to the
spread of Coronavirus. The
religious groups have started protesting against this step of the government
stating that it violates their
fundamental right of religion as doing this mahapooja is a vital activity in their
religion. Decide whether stopping
religious gatherings amidst the rise in virus violative of right to religion.
(a) The government is wrong in stopping public gatherings as the government cannot
interfere in core religious
rights.
(b) The government can put restrictions on right to religion on the basis of public
order, health, morality and
other fundamental rights.
(c) The government can put restrictions on organizing a pooja because it is not a
core religious activity.
(d) The government can restrict fundamental right of religion without any
justification.
92. A temple did not allow people of certain caste to enter its premises. A
petition was filed to stop this practice.
The temple authorities stated that this is an age old tradition which should not be
interfered with. Decide.
(a) The Court cannot force the temple to allow people from the restricted caste as
this is an age old tradition.
(b) The Court can force the temple to allow people from the restricted caste as the
practice is violation of right
to equality which is a fundamental right.
(c) The Court cannot interfere with the temple management as every person has the
right to freely express,
practice, profess their religion.
(d) The Court can only advice the temple authorities to let the members of
restricted caste enter the premises.

. Page 23 of 36
93. A religious group had a vital ceremony wherein all the followers had to dance
naked within the closed premise
of a hall dedicated to the ceremony and only the followers were allowed to take
part in the ceremony. A group
of lawyers objected to this custom as being obscene. Decide whether the practice
can be stopped.
(a) The practice cannot be stopped as only followers are allowed to participate.
(b) The practice cannot be stopped as it is a vital ceremony and it does not
violate public order, health, morality
and other fundamental rights.
(c) The practice should be banned as it is immoral in nature.
(d) The Court can ask the religious group to change the nature of the practice in
the light of obscenity.
94. A government order took away the land that was allotted in the name of a temple
and offered adequate
compensation. However, the religious group challenged the order as being violative
of their fundamental right
to religion. Decide.
(a) The order takes away the fundamental right to religion as the government cannot
take away the land meant
for religious use.
(b) The order does not take away the fundamental right to religion as the
government is offering adequate
compensation for it.
(c) The order does not take away the fundamental right to religion as acquiring
land is a non-religious activity.
(d) The government should have consulted with the religious group and should have
taken away the land with
their consent.
Passage (Q.95-Q.99): The Supreme Court recently observed that comments made during
the course of the
mediation or settlement proceedings should not be taken on record, as the same
would impede conciliation and
will be contrary to the principle of confidentiality [Arjab Jena @ Arjab Kumar Jena
v. Utsa Jena @ Pattnaik].
A Division Bench of Justices Sanjiv Khanna and Bela M Trivedi therefore, directed
the observations in an order
by the Orissa High Court be erased from the record.
"We disapprove the observations made in the impugned order which refer to the
comments made during
the course of the mediation or settlement proceedings. The High Court should not
have taken the aforesaid
comments on record, as the same would impede conciliation and is contrary to and
impinges on the
principle of confidentiality," the Court said.
The Court was hearing an appeal moved against the order of the Orissa High Court
which had permitted the
appellant-father to meet his child subject to certain conditions.
After expunging the offending observations, the top court proceeded to consider the
matter on merits.
It noted that the matter had been remitted back to the family court and therefore,
requested the family court to
decide the application for joint custody, visitation right etc., in accordance with
the law within a period of 4
months.
The Court also permitted the applicant-father to interact with his child in the
presence of the counselor attached
to the Family Court.
"The first such meeting would take place on 21.01.2022 at 03.00 PM. The counselor
would fix the date and time
of subsequent meetings as per her/his discretion, keeping in view the request made
by the parties. The counselor
would be at liberty to submit his/her report before the Family Court in sealed
cover. It would be open to the
Family Court to decide whether the report, if submitted, should be made available
to parties. It will be open to
the Family Court to pass interim order/direction as are considered necessary and
appropriate," the apex court
directed.
However, the Court clarified that the observations and the directions given in its
order and the High Court order
would not be construed as findings recorded or expression of opinion on the merits
of the facts and contentions
of the parties. The court also observed that if the interim appeal is for the
progress of the mediation, so it may be
taken into the record. Court also held that any appeal after the mediation award
can be take into record and it
doesn’t impede conciliation efforts.
Source: Article published in Bar&Bench on 10th Jan, 2022
https://www.barandbench.com/news/courts-should#not-take-on-record-comments-made-
during-mediation-settlement-proceedings-supreme-court

. Page 24 of 36
95. Aryan filed a suit against Sunny in Secunderabad city civil court on a matter
pertaining to interior design of
Aryan’s house. Upon the agreement of both the parties the matter was sent to
mediation. Both the parties made
their respective claims before the mediator. Sunny puts up an argument that the
proceedings of the said mediation
must be submitted before the Secunderabad court by the mediator. Decide the
legality of the argument put up by
Sunny.
(a) Yes, the proceedings of the mediation must be submitted to the Secunderabad
court as the suit was filed
initially in the Secunderabad court
(b) No, if the proceedings are submitted before the court, it violates the right to
privacy of the parties to the suit
(c) Yes, the proceedings must be submitted before the court because it brings the
dispute to an end
(d) No, the proceedings need not be submitted before the court because it is up to
the mediator to decide whether
the report should be submitted before the court
96. The Andhra High Court in an order against interim appeal of mediation
proceeding between family on disputes
related to matters of property and have passed an order which puts stay on the
rights of specific X party of the
family in the proceedings, Is the observations made by the high court can be taken
into record in the mediation
proceedings, Decide?
(a) Yes , as the observations made by the High court being a constitutional court
should be taken into record by
the mediation proceedings.
(b) No, the mediation proceedings are different from court proceedings’ where the
goal of the former is
settlement whereas the latter is to pass an order.
(c) Yes, the observation made in the interim appeal is for the progress of the
mediation ,so it may be taken into
the record.
(d) No, the observations made by the high court cannot be taken into record for the
sake of mediation because it
affects mediation proceedings.
97. Why does taking observations of comments made during the course of the
mediation or settlement proceedings
should not be taken on record affects the mediation proceedings?
(a) The comments taken during the mediation proceeding shouldn’t be taken as they
affect the understanding
between parties in first place to take up mediation.
(b) The observations made during the mediation proceeding contain confidential
information which affects the
process of mediation after the judgment.
(c) Taking such observation into record would impede conciliation and
confidentiality.
(d) Both A&C.
98. A mediation process has been started between two corporate companies, after the
first meeting one of the parties
challenged the appointment of one of the mediators, the Madras HC while hearing the
appealed matter, took
proceedings into record which contain certain trade strategies of a party of
mediation, Decide?
(a) The Court, while hearing the matter, appealed the right in taking the record of
the proceedings as it is
necessary to resolve the issue at the disposal.
(b) The HC in the present case need not take proceedings into record which could
impede the conciliation
proceeding of mediation.
(c) The HC is wrong in taking the proceedings into record because it contains
confidential information of one
of the parties.
(d) The HC is wrong in taking the proceeding into record because the recorded
information is not necessary to
arrive at the appealed matter.

. Page 25 of 36
99. The Hyderabad Mediation Center , recently mediated a matter between X and Y and
passed mediation award in
favor of Y and aggrieved by the mediation finality ,X appealed the matter as per
Arbitration & Mediation
Act,1996 in the Telangana HC, as part of the hearings looked into facts of the case
and taken into the record of
proceedings of mediation ,Decide?
(a) The court is wrong in doing so as it would affect the confidentiality and
impede the conciliation process
between the parties.
(b) The mediation matters cannot be appealed to the high court and the award of
mediation is final and binding.
(c) The HC ,in taking record of proceedings of the mediation, violated the recent
SC case of Arjab Jena Arjab
Kumar Jena v. Utsa Jena Pattnaik .
(d) The court is right in taking record of the proceedings of mediation because the
matter has already been
decided at the mediators. It is currently at appeal on merits of the mediation
award.
Passage (Q.101-Q.105): The Gujarat High Court has recently observed that as per the
Muslim law, as enforced
in India, a Husband has not been given any fundamental right to compel his wife to
share his consortium with
another woman (husband’s other wives or otherwise) in all circumstances. The bench
observed thus while
holding that in a suit filed by the husband for restitution of conjugal rights, a
woman cannot be forced to cohabit
with her husband even by way of a court’s decree. In this case, the Court was
dealing with the wife’s challenge
to the decision of the Family court that had directed her to go back to her
matrimonial home and perform conjugal
obligations. However, overturning the family court’s order, the Court, referring to
the object behind Order XXI
Rule 32(1) and (3) CPC, held that no person can force a female or his wife to
cohabit and establish conjugal
rights and if the wife refuses to cohabit, in such case, she cannot be forced by a
decree in a suit to establish
conjugal rights. Gujarat High Court Importantly, while dealing with this case, the
bench also emphasized that
Muslim law doesn’t encourage polygamy as an institution, but only tolerates the
same and therefore, the first
wife of a Muslim husband can decline to live with her husband (who has married
another woman) on the ground
that the Muslim law permits polygamy but doesn’t encourage it.
Source Name – Live Law
Source Link – https://www.livelaw.in/news-updates/muslim-husband-no-fundamental-
right-compel-wife-share#consortium-another-woman-gujarat-hc-188546
100. Areena and Parvez married in 2017. After the marriage things began to
deteriorate and the relationship between
the two became very troublesome to carry out. The husband filed suit for
restitution of the conjugal rights when
Areena left her husband and began to live with her parents. The family court had
directed Areena to go to Pervez
and cohabit with him. What is the obligation of 25egal in this scenario?
(a) Areena has the obligation to go to Parvez house and cohabit with him because
Parvez is her husband.
(b) Parvez cannot file the suit of restitution of conjugal rights because arena can
be forced to cohabit with her
husband.
(c) Arena has the right to live with her parents because the court cannot force
Areena to live with her husband
Parvez.
(d) Areena has no obligation to return to Parvez because she cannot be forced to
cohabit with him.
101. Afreen and Iqbal were married to each other in 2015 under the Muslim law.
After 2016 their relationship began
to become sore and it was very difficult for them to handle each other. A suit of
restitution of conjugal rights
was filed by Iqbal and the matter was in the family court. The family court
directed Afreen to cohabit with Iqbal
at any cost. However the wife blatantly refused to cohabit with Iqbal after the
decree came. What is the liability
of Afreen for defying the orders of the court?
(a) Afreen can be held punishable for defying the order of the court because the
same amounts to contempt of
the court.
(b) Afreen has the liability to cohabit with Iqbal at any cost because he is her
husband.
(c) Afreen can choose to not to cohabit with Iqbal because she cannot be forced to
cohabit with him if she refuses
to do the same.
(d) The decree has come in the favour of Iqbal and Afreen has the liability to
follow the order of the court and
not to refuse to cohabit with Iqbal.

. Page 26 of 36
102. Fatima who lived in Udaipur with her husband Alam had some altercation with
him on some issue. The
altercation took shape of a heated argument and consequently Fatima decided to
leave her husband and go to her
parents and live with them. Alam decided to marry another woman or shared his
Consortium with another
woman. The family court ordered Fatima to go back to her matrimonial home and
perform the conjugal rights
on the basis that as per the Muslim law husband has the fundamental right to compel
his wife to share his
Consortium with another woman. What is the best option regarding the fact that can
be opted out?
(a) The decision of the family court was right because Alam had the fundamental
right to compel his wife.
(b) Fatima was not in a right position to leave her husband because she had the
obligation to perform conjugal
rights with Alam.
(c) The family court was right in ordering Fatima to return to matrimonial home
because Alam had married
another woman after Fatima left her matrimonial home.
(d) Fatima cannot be forced to live with her husband because Alam had no
fundamental right to compel his wife
to perform the conjugal rights.
103. Asim decided to marry Parveen who was another woman after Malaika had decided
to leave Asim due to some
arguments between them. Malaika left her matrimonial home and began to live with
her parents in Lucknow.
After the matter of restitution of conjugal rights was in the family court the
family court had directed Malaika to
go back to her matrimonial home. However she refused to obey the orders and refused
to cohabit with her
husband because he had married another woman and also claimed that polygamy is not
encouraged in Muslim
law. Decide the position of Malaika in this matter.
(a) Polygamy is totally tolerated in the Muslim law and there is no harm in it.
(b) Mailaika is right because it was her fundamental right to refuse to cohabit
with her husband.
(c) Malaika was wrong in this matter as she has to perform he conjugal obligations
as directed by the court
however Asim married Parveen.
(d) Malaika cannot be compelled to do conjugal obligations as polygamy is not
encouraged in Muslim Law.
104. Construing to the passage and relying upon the decision of the family court
choose out the correct option.
(a) If a suit of restitution of conjugal rights has been filed by the husband, the
woman cannot be forced to cohabit
with her husband.
(b) There is no obligation on a women under the Muslim law to cohabit with her
husband if she had left her
matrimonial home and the decree of the court had been passed in favour of the
husband for restitution of
conjugal rights.
(c) The wife has to return to her matrimonial home and perform the conjugal
obligations when there is decree
of the court in favour of the husband regarding the restitution of conjugal rights.
(d) In any circumstance the husband under the Muslim law cannot compel his wife to
perform conjugal
obligations regarding a suit filed by the husband for restitution of conjugal
rights.
105. Construing to the passage and relying upon the facts and aspects discussed
regarding the Muslim law choose out
and option which cannot be the correct interpretation of the passage.
(a) Under the Muslim law husband has no fundamental right to compel his wife to
share his Consortium with
another woman in some circumstances.
(b) When there is a decree of the family court in favour of the husband regarding
the restitution of conjugal
rights of women has no obligation to return to her matrimonial home.
(c) In any circumstance a woman cannot be compelled by her husband to perform the
conjugal obligations.
(d) Under Muslim law polygamy is tolerated.

. Page 27 of 36
SECTION - D: LOGICAL REASONING
Passage (Q.106-Q.110): The main task of education and research is to train people
to perform in future economic
and technological environments with many unknowns. Workers prepare to tackle
unknown problems using
instruments yet to be developed. In truth, we know embarrassingly little about
tomorrow’s jobs. Interdisciplinary
and holistic education and research are indispensable as the workplace combines
cognitive skills with teamwork
and debate with focus on adaptability, replacing silo thinking with a flexible
approach that applies knowledge
from multiple sectors that at first appearances may not seem relevant.
Education and research are increasingly out of touch with demand for skills. The
tendency to focus on cognitive
skills, including the STEM topics of science, technology, engineering, and
mathematics, cannot preclude the
productivity benefit of soft skills – applying knowledge and finding opportunities
offered by technology. In
recent decades most countries have fallen into the trap of overextending cost-
benefit analyses while training
students to solve yesterday’s problems. Governments enforce short-term fiscal
planning on education programs,
yet measuring social skills is not as easy as calculating STEM competences.
Much attention is devoted to high-calibre education and research and for good
reason, but demand trends suggest
that human factors may be more essential than normally assumed. For example,
health, nonstop improving of
skills and entertainment may prove to be the growth sectors of the future – and
main job providers. Human
maintenance will grow almost exponentially steered by demographics and a growing
proportion of elderly
citizens. Human improvement, the ability to try new technologies should not be
overlooked as higher
productivity embedded in new technology only blossoms if humans have the skill to
manage technology.
Entertainment follows from a shorter work life and longer retirements. Elderly
people are more active than
previous generations and demand health care, entertainment, social networks and
communication. They need
coaches, presenting huge job openings.
106. What has the author conveyed regarding the current pattern of Education and
Research?
(a) Current education and research patterns have no productivity benefits.
(b) It is less inclined towards providing STEM competences.
(c) It has somehow lost touch with the need for soft skills.
(d) It nowhere relates to applying knowledge and finding opportunities offered by
technology.
107. Which of the following can be deduced from the above passage?
(a) There is a need to identify the evolving job trends and focus on many in-demand
cognitive skills.
(b) Acquiring academic knowledge and mapping the set standards should be the
ultimate goal.
(c) Overcoming silo thinking and gaining flexibility at work must be a priority.
(d) The current education and research patterns need to focus more on enhancing
soft skills to help tap the
growing sectors of the future.
108. As per the passage, which of the following does not correctly represent the
author’s view regarding education
and research?
(a) Education and Research must focus on skills that involve conscious intellectual
effort.
(b) Education and Research trains people to enable them to perform in future
economic and technological
environments.
(c) One cannot do away with human improvements that relates to the ability to try
new technologies.
(d) The new cognitive working environment demands extracting the students out of
the archaic solving of
yesterday’s problems.

. Page 28 of 36
109. Which of the following steps must be taken to help education and research
become Interdisciplinary and holistic?
(a) Preparing workers to tackle unknown problems of the future.
(b) Condemning the system that lays much importance to cognitive skills and STEM
competences.
(c) Realising the value of social skills and inculcating human improvement along
with the existing focus on
cognitive skills and STEM knowledge.
(d) Enforcing more short-term fiscal planning on education programs.
110. In the above passage, what does the author mean when he says “Much attention
is devoted to high-calibre
education and research and for good reason, but demand trends suggest that human
factors may be more essential
than normally assumed.” Which of the following reflects the central flaw in the
reasoning above?
(a) The government’s focus is more on high-calibre education and research than
focusing on human factors.
(b) The author assumes the high-calibre education and research does not take into
account human factors.
(c) High-calibre education and research and human factors are intra-connected.
(d) The demand for the human factors in the future will diminish the high-calibre
education and research.
Passage (Q.111-Q.115): In February 1922, a violent mob set a police station on fire
at Chauri Chaura with 22
policemen trapped inside. The home secretary at that time called it a “rebellion
against the Raj” but for Mahatma
Gandhi, it was an “index finger” that pointed the way to possible anarchy and he
called off the civil disobedience
movement. He even went on a fast for five days as self-punishment for the violence.
He was experimenting with
the means of non-violence, and, for him, means were as necessary as the end. He
wrote in Young India: “They
say ‘means are after all means’. I would say means are, after all, everything.
There is no wall of separation
between means and ends.”
As the year winds down with the protracted farmers’ protest and the deadlock looks
like continuing into the new
year, this is a moment for reflection and concern.
Democracy needs a free and safe space for the expression of ideas but because there
is no Gandhian leadership
in and around us, the shadow of fear lurks. Although the government so far has
shown restraint, visuals of
barricades being broken, reports of cell phone towers being damaged, protests in
Patna on Tuesday — all of
these could erupt any time. We have already seen the tragic suicides of two farmers
and any incident can take
their peaceful protest into the tunnel of violence. That is why it is extremely
important to break the deadlock
through dialogue and minimise any chance of violence. That is why the Mahatma is an
inspiration — for both
sides.
For, his belief that means are as important as the end ensured that he never
compromised on the means. Just the
opposite seems to be the norm today — we compromise on the means, irrespective of
our best intentions. Today,
as the market evolves, we are becoming more consumers than citizens. The towering
aspirations of consumerism
have overshadowed the core values of citizenship.
111. Chaura Chauri has been illustrated to
(a) To highlight the fact that the incident then and the incident now bears
remarkable similarity.
(b) To bring into cognizance that Chaura Chauri incident had an inspirational
leader who had the following of
the masses but there is a leadership lacuna at present.
(c) To incorporate the same principles that means should be in accordance with the
ends.
(d) To show a stark contrast between the approaches adopted by two leaders.
112. All of the following is implied through the statement except
(a) There is a need for an inspirational leader such as Mahatma Gandhi in the
present circumstances.
(b) There lurks a fear in expression of ideas that it can take a swift turn for
violence.
(c) No protest remains peaceful when it takes the shape of masses.
(d) Gandhiji found Chauri Chaura incident as defiance against the ideologies of
non-violence.

. Page 29 of 36
113. “They say ‘means are after all means’. I would say means are, after all,
everything. There is no wall of separation
between means and ends.” If the above statement is true, then which of the
following will be in harmony as a
conclusion with Mahatma Gandhi’s ideology?
(a) All is well that ends well.
(b) A violent pursuit to bring in non-violence.
(c) Peace at all cost.
(d) None of the above.
114. The opponents could effectively defend their position against the author's
stance by pointing out that
I. Democracy has been a misguided concept where predators prey on freedom of
expression as a means to a
twisted end.
II. Right to dissent and protests turning violent are the norms that are the new
normal.
III. Even an inspirational leader’s influence has limitations and the ideologies
tend to lose their sheen as the
protestors grow in number.
(a) Only I
(b) Only II
(c) Only III
(d) I and III
115. ‘Today, as the market evolves, we are becoming more consumers than citizens.
The towering aspirations of
consumerism have overshadowed the core values of citizenship.’ The assumption
behind the statement is that
(a) Consumerism aspirations and core citizenship values do not align together.
(b) Consumerism overshadows citizenship values.
(c) There is decadence of core values of a citizen with the rise in his
consumerism.
(d) Consumerism is the means to fulfilling economical aspirations.
Passage (Q.116-Q.120): The Green Revolution-essentially the promotion of capital-
intensive industrial
agriculture-was more of a Cold War stratagem than a humanitarian initiative, as
recent histories have forcefully
argued. After independence in 1947, peasant movements led by communists had mounted
fierce pressure on the
Indian National Congress, the ruling political party, to redistribute land from
landlords to peasants. But the
Congress, beholden to landlords for electoral support in rural areas, was unwilling
to implement comprehensive
land reforms. In this context, the U.S. government promoted the Green Revolution to
preempt a Soviet-style
“Red Revolution,” as U.S. Agency for International Development administrator
William Gaud stated in a speech
in 1968. It comprised subsidized fertilizers and irrigation, rice and wheat
varieties bred to absorb high fertilizer
doses, and state-led training programs to assist farmers in transitioning to new
practices. Given the expense, it
was rolled out only in a few, well-endowed districts of Punjab and a few other
states. Because bumper
productions inevitably depress prices, farmers were guaranteed procurement through
state-run mandis or market
yards at MSPs declared in advance. State procurement was therefore crucial to
transforming Punjab into India’s
breadbasket.
But as many argued, the Green Revolution package created more problems than it
solved. By the 1980s, even
the geographically limited package proved fiscally onerous. As state support
declined, the problem of
unremunerative prices and debt escalated.
The PM hails the farm laws as watershed reforms that will usher in a new era of
prosperity for farmers backed
by corporate investments. On the face of it, they allow private buyers to purchase
farm produce outside of the
supervision of and without the payment of taxes and fees to mandis; limit state
intervention in retail prices; and
provide a framework for farming on contract to corporations. In their details,
however, the farm laws intrude
upon the regulatory powers of state governments and intensify the already severe
power asymmetry between
corporate houses and the mass of Indian farmers, nearly 86 percent of whom
cultivate less than two hectares.
Farmers fear that the laws portend a total hollowing out of the state-regulated
procurement at mandis. To this

. Page 30 of 36
day, mandis signal prices with regular announcements of MSPs, and if they are
weakened any further than they
already have been, farmers will be fully exposed to debilitating price pressures.
Underlying this broad base of discontent is the failure of the Green Revolution.
Even a celebratory review in
2003 was forced to concede that the principal benefit of the package was lower food
grain prices, whereas the
vast majority of farmers and agricultural laborers had suffered declines in
incomes. In short, the Green
Revolution secured cheap cereals in exchange for justice and ecological
sustainability.
No amount of tinkering on the marketing end will fix a fundamentally warped and
unsustainable production
model, and therefore the government must concede the immediate demand to withdraw
the three laws. But, to
actually secure a viable future for farmers, we must abandon the Green Revolution
paradigm and adopt
agroecological, diverse, decentralized and just agrarian and food systems.
116. What was the reason behind the creation of MSPs?
(a) Farmers needed to satisfy basic needs so the state in order to help them,
became active in procuring food
grains at a minimum price.
(b) The Landlords had re-distributed the land and the state in order to support
farmers came up with MSP.
(c) An increase in production leads to reduction in prices and, hence, MSP was
required to ensure that the
farmers produce more than enough and also receive a minimum price for their yield.
(d) The protest by the farmers with respect to the green Revolution led to the
creation of MSPs.
117. ‘Given the expense, it was rolled out only in a few, well-endowed districts of
Punjab and a few other states’ what
is the assumption for the above statement?
(a) An expensive process resulted in Green Revolution.
(b) Things that are expensive should be tested before being released on a large
scale to avoid errors and loss of
money.
(c) Support should only be provided to well-endowed districts for higher yield.
(d) Well-endowed districts had a higher capability of giving positive results.
118. What can be derived from the given statement, ‘even the geographically limited
package proved fiscally
onerous’?
(a) The green revolution was brought in only few areas.
(b) The green revolution was very expensive and India’s economy took a hit due to
the same.
(c) Even if the green revolution was introduced in only certain areas, it still
proved to be expensive.
(d) None of the fiscal schemes proved profitable.
119. Which of the statements are false and not in congruence with the above
passage?
(a) Green Revolution was only a tactic of the US for Cold War rather than being an
agricultural reform.
(b) Green Revolution resulted in lower incomes and lower prices for the farmers.
(c) The Green revolution ended the food scarcity in India and hence, now we do not
need laws regulating prices
of food produce.
(d) To secure farmers there is a need to adopt agroecological, diverse,
decentralized and just agrarian and food
systems.
120. What would be the conclusion of the given passage?
(a) Green Revolution was a cold war tactic and did not bring any positive effect.
(b) The regulatory regime over agriculture would be ended by the farm laws.
(c) Green Revolution would just make the products cheaper, farm laws would just
weaken the farmers more.
(d) If a model is inaccurate, then its mistakes cannot be corrected by marketing it
as a humanitarian measure.

. Page 31 of 36
Passage (Q.121-Q.125): Within a span of a few months, India has transitioned from
facing a shortage of hospital
beds to showing off crowded holiday resorts. Fresh out of a deadly second wave,
viral videos of clogged roads
to hill stations and packed markets are symptomatic of the rapid recovery in
economic activity as the worst of
the lockdown restrictions have ended.
Growth indicators so far suggest a “Teflon economy” in the short term — a shallow
dent in May’s economic
activity followed by a recovery in June, back to April’s levels. The external,
investment and industrial sectors
have been relatively resilient, with consumption and services bearing the brunt.
Notwithstanding signs of some
fatigue in ultra-high frequency indicators in July, damage from the second wave
seems largely limited to April#June 2021.
Nevertheless, a K-shaped recovery means light cracks on the top conceal much larger
structural fault lines below.
The Pew Research Centre estimates that the pandemic has led to India’s poor rising
by 75 million, while the
middle and upper-middle class has shrunk by 39 million. A recent survey by the ILO
finds that the worst-hit —
MSMEs and their informal workforce — have struggled to access the government’s
pandemic support
programmes. These more structural scars may become blurred in the GDP data in
coming quarters as the
economy rapidly normalises alongside strong global growth, fiscal activism, and
easy financial conditions, but
will almost certainly affect the medium-term growth story.
However, in the near term, there are two impending macro pivots to navigate. First,
is the vaccine pivot. The
“ultimate unlocking” of the economy remains contingent on a critical mass getting
vaccinated, which on
materialising should trigger a revival in consumer and business sentiment. The
vaccine pivot is also an effective
insurance policy against a possible third wave — a risk to near-term growth. The
lacklustre pace of vaccination
in July and fresh information on the vaccine pipeline suggest that there are risks
of a delay in the pivot taking
place in August, and to our baseline projection of around 50 per cent of the
population being vaccinated by end#2021. However, the uptick in the pace of
vaccination over the last few days and
higher seroprevalence reported in some states are welcome news.
The second pivot is that of policy. When inflation is under control, then flush
liquidity and ultra-accommodative
monetary policy will help kill two birds with one stone- ensuring easy financial
conditions, and helping control
borrowing costs of the government’s expansive borrowing programme.
121. Which of the following best describes the purpose of this passage?
(a) To encourage rising trust among people for the recently shown GDP data.
(b) To discourage higher seroprevalence reported in some states.
(c) To explain the effects of pandemic on Indian economy.
(d) To highlight the need for vaccination and rising risks and delays.
122. What does author want to imply through “these more structural scars”?
(a) The accretion of the poor by more than 7 crores due to pandemic.
(b) A fall-off in more than 3 crores middle and upper-middle class.
(c) The striving of MSMEs for accessing government’s pandemic support programmes.
(d) All of the above.
123. The author uses the expression Teflon economy to
(a) To highlight the unsoundness of the Indian economy because of pandemic.
(b) To stress upon the Indian economy’s resilience despite lockdown onslaughts.
(c) To suffer the worst part of an unpleasant situation.
(d) To be looking for solutions in the wrong places, especially during the
lockdown.

. Page 32 of 36
124. Which of the following cannot be inferred from the passage?
(a) Flush liquidity and ultra-accommodative monetary policy are the answers for
fiscal issues.
(b) Loss from the second wave was not bound to April quarter.
(c) All the external and investment sectors have been affected, but the consumption
and services took the most
hit.
(d) India had been facing the shortage of hospital beds during pandemic.
125. “The lackluster pace of vaccination in July suggests that there is a risk of a
delay in the pivot.” Which of the
following sentences from the passage reflects the reversal of this?
(a) The arguments of a slight increase in the speed of vaccination in past few
days.
(b) The arguments of having greater levels of pathogens found in many people, as
measured in blood serum.
(c) The unlocking of the economy relies on population getting vaccinated.
(d) Both A and B
Passage (Q.126-Q.130): Over a hundred people have succumbed to the mystery virus
that originated in China's
Wuhan province and it has been confirmed in at least 10 other countries. The
Chinese authorities have
acknowledged that the virus has affected 4,500 people. But modeling by researchers
at Imperial College London
suggests that 70,000 to 100,000 people could be affected. The WHO, which last week
desisted from describing
the situation as a public health emergency, has asked countries in Southeast Asia
to remain vigilant. The outbreak
has evoked memories of the SARS epidemic of 2002-2003, which killed nearly 800 and
affected more than 8,000
people worldwide. It was also ascribed to a coronavirus and manifested similar
symptoms - fever, cough and
shortness of breath. Antibiotics do not work against such viral pneumonia and there
are no vaccines against them.
However, there are reassuring differences between the situation in 2002-3 and the
one today. For one, China's
response to the current outbreak is markedly different from the way it dealt with
SARS. In contrast to its secretive
ways 17 years ago, Beijing informed the WHO, shared the virus's gene sequence with
the world, imposed travel
restrictions and quarantined 50 million people. Moreover, experts suggest that the
current coronavirus outbreak
is not as virulent as the one in 2002-3 - it has a mortality rate of less than 3
percent as compared to SARS's death
rate of nearly 10 percent. But we are still looking at preliminary data, and
scientists are likely to know the exact
magnitude of the problem in the coming weeks.
126. Which of the following strengthens the author's argument?
I. Chinese authorities are not revealing the real pictures of the virus outbreak
due to international backlash.
II. Chinese authorities do not have the technology to predict the number of people
affected by the virus outbreak.
(a) Only I
(b) Only II
(c) Both I & II
(d) Neither I nor II
127. Which of the following is the most logical and rational inference that can be
made from the above passage?
(a) China’s response to the outbreak is extremely deviant from the standard
protocol.
(b) The current virus outbreak is a mystery.
(c) In a globalized world, the chances of the flu spreading fast are high.
(d) Flu vaccines should be developed as soon as possible.
128. Which of the following statements weakens the author’s argument in the
passage?
(a) WHO advises against the application of any travel or trade restrictions on
China based on the information
currently available. It does not recommend that travelers take any specific
measures either.
(b) Sharing the virus' gene data will help other countries to quickly identify the
virus, provide care, and also
develop specific diagnostic kits, drugs and even vaccines.
(c) The ratio of the people affected by the virus to the death occurred by the
virus is lower than the death
occurring from normal flu.
(d) The current virus is much more virulent than its predecessors.

. Page 33 of 36
129. Which of the following is/are the most logical assumption made by the author?
I. It is more difficult for vulnerable members of the population, that is, the
elderly or those with existing
respiratory or immune problems, to protect themselves against the spread of
Coronavirus.
II. India has issued a travel advisory asking citizens to follow certain
precautionary measures while visiting
China.
III. The mortality rate of less than three percent does not indicate a virulent
nature of the virus.
(a) Only I
(b) Only II
(c) Only III
(d) None of the above.
130. Given below are pairs of events ‘A’ and ‘B’. You have to read both the events
‘A’ and ‘B’ and decide their
nature of relationship. You have to assume that the information given in ‘A’ and
‘B’ is true and you will not
assume anything beyond the given information in deciding the answer.
Event A: The WHO, which last week desisted from describing the situation as a
public health emergency, has
asked countries in Southeast Asia to remain vigilant.
Event B: Over a hundred people have succumbed to the mystery virus that originated
in China's Wuhan province
and it has been confirmed in at least 10 other countries.
(a) If ‘A’ is the effect and ‘B’ is its immediate and principal cause.
(b) If ‘A’ is the immediate and principal cause and ‘B’ is its effect.
(c) If ‘A’ is an effect but ‘B’ is not its immediate and principal cause.
(d) If ‘B’ is an effect but ‘A’ is not its immediate and principal cause.
131. Five Athletes - A, B, C, D and E participated in a race and finished the race
at five different timings, not
necessarily in the same order. C is neither the first nor the last one to finish
the race. Either B or D finished the
race third and B finished the race immediately before C and immediately after D. If
A finished the race after D,
then who is the last one to finish the race?
(a) A
(b) E
(c) C
(d) Cannot be determined
132. Choose the set of statements where the last statement can be logically derived
from the preceding three.
Statements:
1) Some birds are animals.
2) All animals live in jungle.
3) Some animals cannot fly.
4) All birds can fly.
5) Some birds do not live in jungle.
6) Some animals fly.
(a) 3, 2, 4, 1
(b) 2, 1, 4, 6
(c) 1, 4, 3, 5
(d) 2, 3, 1, 5
133. Identify the missing number: 4 and 8, 11 and 20, 23 and 39, 42 and _____
(a) 62
(b) 55
(c) 58
(d) 67

. Page 34 of 36
134. A is taller than F, who is shorter than three persons. C is shorter than D,
who is the tallest. B is taller than E. C
is neither shorter than F nor taller than A. Who is the second shortest person
among the given?
(a) A
(b) B
(c) C
(d) E
135. In a certain code language. “BREAK” is written as “23615455” How is “XEROX”
written in that code
language?
(a) 24154610
(b) 24105460120
(c) 2410546012
(d) 2410546024

mock 29
Directions (Q.1-Q.30): Read the following passage carefully and answer the
questions that follow.
Passage (Q.1-Q.5): The Myanmar junta has made it plain it will stop at nothing to
suppress the protests against
its February 1 coup. The near daily death toll of pro-democracy protestors is a
distressing reminder of the promise
of Myanmar just until a couple of months ago, and how the country’s military has
turned the clock back by at
least a decade, if not more. The gunning down of protestors is a devastating attack
on the people of Myanmar,
who have mostly peacefully demanded a return to democracy. The violence that the
military has unleashed on
the people after usurping power following the sweeping electoral victory of Aung
San Suu Kyi’s National League
for Democracy, shows it has no intention of being hemmed in by democracy, even of
the hybrid kind written
into Myanmar’s Constitution. On Sunday, the military shot down 39 people, the
highest single-day death toll,
taking the total number of people killed just for voicing protests against the
junta since the coup to more than
100. Suu Kyi remains in detention, and has been accused, along with other leaders
of the NLD, of corruption by
the military regime. There is a crackdown on media, and at least 10 journalists
have been detained without
charges.
The coup seems to have taken the world by surprise. Last week, the United Nations
Security Council asked the
military rulers “to exercise utmost restraint”. The US, Britain and Canada have
individually announced sanctions
such as travel bans and asset freezes on Myanmar’s top army generals. There has
been talk of freezing arms
sales, with South Korea already announcing suspension of defence exchanges with
Myanmar. But Russia and
China, which have close relations with the Myanmar military, may take the edge off
such action. ASEAN has
more or less accepted the new leadership, weakly calling for Suu Kyi’s release.
Japan, the second-biggest
investor in Myanmar after China, has said it is “considering” how to respond to the
developments.
India, the biggest democracy in the region (and indeed the world), which has close
links with both the Myanmar
military and with Suu Kyi and the NLD, has been silent after its initial expression
of “concern”. In the Nineties,
India teetered between its storied friendship with Suu Kyi, and the need to engage
with the military amid
concerns that Indian militant groups in the Northeast were being given safe haven
in Myanmar. In the first decade
of the 21st century, the rivalry with China for influence in the region made that
engagement even more necessary.
But the moral dilemma for fence-sitters may be sharper today. Colombo’s invitation
to Myanmar for the April 1
BIMSTEC foreign ministers’ meeting has triggered protests in Sri Lanka and Myanmar,
and will bring home
that dilemma for Delhi, too. Speaking in another context, External Affairs Minister
S Jaishankar has said India
is not looking for Western approval on India’s democratic credentials. What
democratic values does India, then,
stand for? Its words and actions on Myanmar will provide an indication.
1. What is the inference drawn with respect to the hybrid kind of democracy as
mentioned in Myanmar’s
constitution?
(a) A cross between autocracy and democracy with more elements of democracy than
autocracy.
(b) A mixed kind of regime combining autocratic features with democratic ones and
can simultaneously hold
political repressions and regular elections.
(c) An unprecedented mention in the constitution that actually defies the very
weave of democracy.
(d) A suppressed rule where democracy only serves theoretically and has no voice in
the political corridors.
2. Which of the followings does/do not hold true with regards to the passage?
I. China and the US are the fence sitters with regards to the military coup in
Myanmar.
II. India’s response to the military coup in Myanmar has resonated with US, Britain
and Canada.
(a) Only I
(b) Only II
(c) Both I and II hold true
(d) Both I and II do not hold true.

. Page 3 of 40
3. The word ‘Junta’ in the context of the passage refers to
(a) The military of Myanmar ruling the country, especially after a coup d’état and
before a legally constituted
government has been instituted.
(b) An insurgency by a small group in Myanmar to overrule democracy and place
autocratic rule.
(c) A council with the power to usurp a regime on finding discrepancy with the
incumbent ruling.
(d) A military coup to bring in communism rather than democracy in Myanmar.
4. The central idea behind the passage is
(a) To censure the stand taken by the Myanmar military with respect to dishonouring
the constitution of
Myanmar.
(b) To censure the fence-sitters who are merely watching in silence while the
people of Myanmar are being
subjugated by their military.
(c) To highlight the condition of Myanmar after the military coup and respective
stand being taken by the
countries with respect to the military coup in Myanmar, especially India.
(d) To draw the attention of India towards the plight of Myanmar and to persuade it
to take a stronger stand on
moral grounds.
5. The word similar in meaning to the word ‘teetered’ in the context of the
passage, is:
(a) Balanced (b) Pilot (c) Vacillated (d) Decided
Passage (Q.6-Q.10): Spain is a country in love with food, renowned for everything
from tapas to trailblazing
chefs to simple, elegant recipes that have endured for generations. So, it may seem
counterintuitive, perhaps
even heretical, to say that the most important thing about a Spanish lunch is not
the food. But it's true. Before
you spill your gazpacho, let me say that Spanish people don't take the food part of
lunch lightly; far from it. As
a Spaniard in love with food in general, and lunch in particular, I for one
approach the subject of where to eat
with the same level of thought and research that some people put into buying a new
car.
Steady yourselves foodies; but in Spain the purpose of going out for lunch isn't
just eating, it's catching up with
friends or family, telling stories and laughing away the stress caused by things
that, with a little perspective, you
come to realise don't matter anyway. Food matters a lot in Spain, but the social
aspect of it matters even more.
Lunch, for example, doesn't end when people can't eat another bite. That's when the
sobremesa starts. There is
no equivalent word in English, though the concept is simple: sobremesa is the time
you spend at the table after
you’ve finished eating. Usually, there’s laughter involved, and almost always the
kind of easy, convivial
conversation that only the pleasures of a big meal can inspire.
On a personal level, the sobremesa is fundamental. As a chef, when I see people
spending time at the table after
lunch, I feel that it’s sign that everything has gone well, but often times people
enjoy themselves even more than
during the meal itself. The sobremesa can be magical. When I moved to Madrid from
Zaragoza, I got in touch
with Ben Curtis, a British blogger who has lived in Spain for 20 years and has
probably taught more people
about Spanish customs than anyone else. We’d been emailing about things related to
Spanish culture for some
time, but we’d never met, so I suggested we go out for a beer. He wisely suggested
we go out for lunch instead.
It went so well that we’ve been having lunch more or less once a week for the past
six years.
In my experience, avantgarde food doesn’t lend itself to a good sobremesa because
too much attention gets
devoted to the food itself. That’s why I prefer classic, unpretentious casas de
comida, or family restaurants,
where the food is home-style, made from well-cooked, simple ingredients. I know Ben
feels the same way
because we have often explored this important subject in leisurely chats after
robust meals, the white tablecloth
sprinkled with breadcrumbs and splotched with red-sine stains. My informal research
suggests that the better
the food, the better the sobremesa, but tellingly, you can eat mediocre food and
still have a great lunch if you’re
with the right company.

. Page 4 of 40
6. What happened in the aftermath of the lunch between the British blogger and the
author?
(a) The two became the best of buddies.
(b) They started having lunch more frequently.
(c) They collaborated on a book.
(d) They set out to educate people of sobremesa.
7. ‘So, it may seem counterintuitive, perhaps even heretical, to say that the most
important thing about a Spanish
lunch is not the food. But it's true.’ Which of the following is implied by the
author?
(a) The author, at the risk of sacrilege, admits that the Spanish lunch while
considered as an ethereal experience
is not the most rewarding experience of having the lunch.
(b) The author woes that Spanish lunch is given more weightage than it deserves
when there is more to Spain
than its lunch experience.
(c) The author believes that to disregard Spanish lunch is akin to blasphemy;
therefore, it requires a mention by
tourists.
(d) The author regards the food of Spanish lunch as one of the most heavenly
experiences that a tourist gets on
visiting Spain.
8. ‘Before you spill your gazpacho, let me say that Spanish people don't take the
food part of lunch lightly; far from
it.’ Which of the following reflects the figure of speech in the given statement?
(a) Synecdoche. (b) Metaphor. (c) Epigram. (d) Hyperbole.
9. Which of the following is the closest antonym of "unpretentious", as used in the
passage?
I. Modest
II. Purified
III. Flashy
IV. Straightforward
(a) Only II (b) Only III (c) Both I & IV (d) All I, II & IV
10. The author’s attitude towards Sobremesa is
(a) Inhospitable (b) Animated (c) Detachment (d) Liberal
Passage (Q.11-Q.15): Russian President Vladimir Putin sought a confrontation with
the West and he got one.
So now what? Diplomatic efforts to defuse the crisis appear to have stalled. In the
past month, Putin has escalated
his long proxy war in two eastern provinces and accelerated a massive Russian troop
build-up that now surrounds
Ukraine on three sides. The United States, in response, has put thousands of troops
on high alert and the President
of America is weighing a range of further measures, from severe economic sanctions
to sending troops.
At the centre of it all is Putin, an enigmatic leader with a quest for power and a
deep resentment of the West.
With the world watching for a possible war, we reached out to the smartest Russia
and Putin watchers we know
to ask what might be next — and what the U.S. should do. These observers are the
first to tell you that Putin is
impossible to predict — but we asked them to do it anyway. Some said they expect
Putin to invade, while others
believe he is likely to give diplomacy more time. Some pointed to key experiences
in Putin’s personal history
that could impact how the crisis plays out. And when asked how Biden (American
President) should respond,
our experts offered a wide spectrum of options, from sending more troops into the
region to taking Russia’s
concerns more seriously to toning down the rhetoric from Washington.
According to Shevtsova, President Putin has become a “Master of Suspense”. Suspense
provokes confusion in
the West, whereas war could unite the West. Putin’s challenge now is to prevent
suspense from turning into a
bluff. In the end, he will have to choose between war and farce — which means
humiliation, and he is not ready
for that. However, about the view of Menon, despite the prevailing pessimism, he
believes the failed Geneva

. Page 5 of 40
talks opened several diplomatic channels, improving prospects for diplomacy. So,
Menon expects Putin to wait,
assess the results, keep his powder dry, but not rush to invade Ukraine.
McKew says that Putin will continue blackmailing and an attack is inevitable, there
is a new security reality in
the region; we are debating things that were not debatable before the escalation.
Russia keeps inching their
geopolitical vision forward upon the West and Ukraine may lose if they do not stand
and fight now. Now, on the
other hand, according to Pifer, the Biden administration has set the correct
framework: Try to extend the
diplomatic path while making clear the costs of a Russian military assault — more
punitive sanctions, more
Western military assistance to Kyiv, and a bolstering of NATO presence on its
eastern flank. It’s good to see the
administration increasing military assistance to Ukraine. The Americans appear to
be preparing for an order to
move some troops to Europe.
11. How did America retaliate when Russia expanded its long-running proxy war in
two eastern provinces in the
last month and expedited a vast Russian force build-up that now encircles Ukraine
on three sides?
(a) The US withdrew the troops from that area, and the president considered a
variety of additional steps, that
included not interfering with the affairs of a foreign country.
(b) The US asked Putin to step back politely as it could be the start of a snowball
effect for the third world war.
(c) The US put thousands of soldiers on vigilance, and the president is considering
a variety of additional steps,
including harsh economic measures for the deployment of troops in that area.
(d) The US has put tens of thousands of soldiers on high alert, and the president
is considering a variety of
additional steps, but he is not supporting the idea of investing a huge chunk of
money for the brawl with the
Russians.
12. Based on the passage, what comes out as the biggest difference between Putin
and Biden?
(a) Putin is unforeseeable in this approach, while the strategy of Biden can be put
down to words.
(b) Putin is a war-hungry dictator, while Biden is a pacifist.
(c) Putin is an impeccable strategist, while Biden is wet behind his ears.
(d) Putin is anxious about the current balance of power in the world, while Biden
is fine with the ongoing
situation.
13. Why is it that maintaining suspense in the West is an essential part of Putin’s
strategy and what are the two
possible choices for him in this matter?
(a) Conflict will cause disarray in the west, but the suspense on part of Putin has
the potential to unify the west
and his choice is between war and absurdity in the end.
(b) Putin’s suspense with regard to the strategy will lead to perplexity in the
west; however, a war would only
serve to unify the west; therefore, the two choices with Putin are either to war or
face humiliation.
(c) Putin’s silence will cause confusion in the West, but the war has the potential
to unify the east and the only
choice he has is war.
(d) The conflict between Putin and Biden will cause disarray in the west, but the
suspense has the potential to
unify it and the only choice that Putin has is to remain silent.
14. Which one of the following expert’s opinions hints towards the least aggressive
attitude of either Putin or Biden
in power?
(a) Shevtsova. (b) Menon. (c) McKew. (d) Pifer.

. Page 6 of 40
15. Which one of the following statements can be identified as the most plausible
explanation behind the reason for
Russia’s attempt to capture Ukraine?
(a) The US has sent tens of thousands of troops on high alert, and America is
considering a variety of further
steps, including harsh economic penalties and the deployment of troops.
(b) Ukraine has abundant reserves of coal, iron ore, natural gas, manganese,
graphite, sulphur and mercury.
(c) Russia's geopolitical agenda continues to advance toward the West, and Ukraine
may lose if they do not rise
up and fight now.
(d) Putin is a power-hungry statesman and he has a strong disdain of the West.
Passage (Q.16-Q.20): Known for their lotus flowers and rows of orchards, three
counties, southwest of Beijing,
are set for a hi-tech industrial makeover. On April 1, Chinese President Xi Jinping
announced that the Xiongan
New Area (XNA) would be formed from the merger of Rongcheng, Xionxian and Anxin
administrative districts.
The XNA will become the latest edition of China's famous New Areas, which have been
at the heart of the
country's meteoric rise and transformation into a modern economy. President Xi,
with his XNA, is only following
the pioneering trail carved by Deng Xiaoping, Mao Zedong's successor. Deng was the
first to establish the
Shenzhen economic zone. A stone's throw from Hong Kong, which opened a pipeline of
global finance, the
Shenzhen experiment established real confidence in China's ability to leapfrog into
the modern industrial age.
Following Deng's footsteps, his successor, Jiang Zemin, set up the Pudong New Area
(PNA) - the pride of
Shanghai. The Huangpu river separates the historic Shanghai from the PNA, which
extends all the way to the
East China Sea. Stunning high rises, such as the Oriental Pearl Tower, the Jin Mao
Tower and the Shanghai
World Financial Centre in the PNA, impart a distinct flavour of power and
commercial success to Shanghai.
President Xi is also planning big with the XNA. Not many details have surfaced yet,
but according to a Morgan
Stanley estimate, 2.4 trillion Yuan, or $290 billion, would be invested in this
mega city, 160 km from Beijing.
The venture, which would include new clusters of hi-tech industry, brand new
universities, housing projects,
schools and hospitals, is expected to add as much as 0.4 percentage points to
China's annual economic growth.
It is estimated that the XNA, where 2.5 million people will eventually reside, will
emerge as the largest
infrastructure project in China's history. President Xi's announcement has
triggered a chain of events, typical of
a boomtown going stratospheric. From the faraway corners of China, early-bird small
businessmen and
speculators, sensing mega-profits, are rushing to the upcoming city. Rentals have
already shot up. However,
environmental groups are voicing concerns regarding the mega plan.
The Baiyang Lake, around which the XNA will rise, is already in deep decay. The
lake and its surrounding water
bodies cover an area of 366 sq. km, forming north China's biggest wetland. But
toxic pollutants from nearby
factories are channelled to the bottom of the lake through hidden pipelines. Unless
urgent remedial measures are
taken, copious amounts of additional waste will land up in the lake, when the
area's population rises from the
current 2,00,000 to 3,00,000 to 2.5 million or more. Han Dongmei, deputy director
of the Chinese Academy of
Sciences, fears the problem will get worse as the plan materializes. "It is easy to
predict what will happen as
more people swamp in," she observed. The Baiyang lake could turn into an ecological
"disaster. However, many
others do not share her views. Professor Cui Baoshan, dean of the school of
environment in Beijing Normal
University, says plenty could still be done to save the lake. He points out that
the government could shut down
all polluting factories, nearby farms could use less pesticide, and water-treatment
plants could be established on
an industrial scale.
16. Choose the option which best describes the meaning of the phrase "a stone's
throw" as used in the passage.
(a) A profitable place (b) A wasteful place
(c) A long distance (d) A short distance

. Page 7 of 40
17. Which of these is the primary concern of the environmental groups about the
mega plan of the establishment of
Xiongan New Area?
(a) Large number of trees will be cut for this mega project which will result in
environmental hazards.
(b) Toxic pollutants from factories already channelled to the Baiyang Lake through
hidden pipelines have
decayed the river and with the increase in population and infrastructure, it will
be an ecological disaster.
(c) It is estimated that 2.5 million people will reside in XNA, which gives rise to
additional waste and scarcity
of potable water.
(d) This ambitious project is leading to high land rates and rentals will shoot up
giving rise to social differences.
18. Which of these statements is not true about the establishment of XNA proposed
by Chinese government recently?
(a) This project (XNA) is expected to add as much as 0.4 percentage points to
China's annual economic growth
with 2.5 million people residing at the site.
(b) The XNA would be formed from the merger of Rongcheng, Xionxian and Anxin
administrative districts
which are famous for lotus flowers and orchards and situated southwest of Beijing.
(c) According to an estimate, 2.4 trillion Yuan, or $290 billion, would be invested
in XNA mega city, to make
it equivalent to or more successful than the already established PNA and SEZ.
(d) XNA would impart an amalgamation of power and commercial success with its high
rise economical hubs
like Oriental Pearl Tower, the Jin Mao Tower and the Shanghai World Financial
Centre.
19. Choose from the following that best describes the meaning of the phrase
"boomtown going stratospheric", as
used in the passage?
(a) A community that is undergoing growth of economic activities giving rise to
pollution level.
(b) A community sitting on the verge of disaster due to extremely high rate of
urbanization
(c) A community that experiences a sudden growth in business and population from
scrap to an exceptionally
high position
(d) A community that showcases its country's culture, art and economy at the same
time due to highly sensitive
inhabitants
20. According to the author, with the plan of XNA, Chinese president Xi is
following the pioneering trail carved by
former president Deng Xiaoping who was the first to establish Shenzhen economic
zone. Which of these may
be the possible outcomes of these mega projects?
1. These projects open a way of global finance and investments in the country and
help in leapfrogging into the
modern industrial age.
2. These projects give an opportunity to small businessmen and speculators to earn
profit and grow eventually.
However, it also gives rise to high rates of living.
3. These projects will end up polluting the environment to an extent that it will
impossible to re-establish the
green belt and thereby turning the entire site into an ecological disaster.
(a) Only 1 and 2 (b) Only 2 and 3 (c) Only 1 and 3 (d) All 1, 2 and 3

. Page 8 of 40
Passage (Q.21-Q.25): A few years ago, ascending the Upper Mississippi in the
Autumn, when its waters were
low, I was compelled to travel by land past the region of the Rapids. My road lay
through the Half-Breed Tract,
a fine section of Iowa, which the unsettled state of its land-titles had
appropriated as a sanctuary for coiners,
horse thieves, and other outlaws. I had left my steamer at Keokuk, at the foot of
the Lower Fall, to hire a carriage,
and to contend for some fragments of a dirty meal with the swarming flies, the only
scavengers of the locality.
From this place to where the deep water of the river returns, my eye wearied to see
everywhere sordid, vagabond
and idle settlers; and a country marred, without being improved, by their careless
hands.
I was descending the last hillside upon my journey, when a landscape in delightful
contrast broke upon my view.
Half encircled by a bend of the river, a beautiful city lay glittering in the fresh
morning sun; its bright new
dwellings, set in cool green gardens, ranging up around a stately dome-shaped hill,
which was crowned by a
noble marble edifice, whose high tapering spire was radiant with white and gold.
The city appeared to cover
several miles; and beyond it, in the back ground, there rolled off a fair country,
chequered by the careful lines of
fruitful husbandry. The unmistakable marks of industry, enterprise and educated
wealth, everywhere, made the
scene one of singular and most striking beauty.
It was a natural impulse to visit this inviting region. I procured a skiff, and
rowing across the river, landed at the
chief wharf of the city. No one met me there. I looked, and saw no one. I could
hear no one move; though the
quiet everywhere was such that I heard the flies buzz, and the water-ripples break
against the shallow of the
beach. I walked through the solitary streets. The town lay as in a dream, under
some deadening spell of loneliness,
from which I almost feared to wake it. For plainly it had not slept long. There was
no grass growing up in the
paved ways. Rains had not entirely washed away the prints of dusty footsteps.
[ Excerpts from The Mormons, by Thomas L. Kane]
21. Which of the following literary devices has been used in the sentence, ‘For
plainly it had not slept long.’?
(a) Personification (b) Symbolism (c) Anaphora (d) Simile
22. Which of the following can be considered the main purpose of the second
paragraph of the passage?
(a) Describing the tiredness of the author due to the journey.
(b) Pursuing the reader about the pros that the industrial development can bring to
a city.
(c) Describing the visual imagery of the city that the author discovered during his
journey.
(d) Highlighting the scenic beauty of the landscape.
23. According to the passage, which of the following is NOT true?
(a) The city was bustling when the author visited it.
(b) The view of the city suggested that it was a vast township.
(c) The city was situated beside a river.
(d) The author was travelling through the roads of Iowa.
24. The first paragraph of the passage is constructed on which of the following
tones?
(a) Cynical (b) Dogmatic (c) Emotional (d) Gloomy
25. Why did the author say that his eyes were wearied?
(a) Because he had a long journey that made his eyes physically tired.
(b) Because the impaired quality of appearance of the vast land he travelled
through made him feel weary from
within.
(c) Because the road he travelled through was too bright and colourful which
affected his sight.
(d) Because he suffered from a sickness that affected his vision.

. Page 9 of 40
Passage (Q.26-Q.30): Monlam literally means supplication, but in practice it is the
name of the great Tibetan
festival performed for the benefit of the reigning Emperor of China, the offering
of prayers to the deities for his
prosperity and long life. The festival commences either on the 3rd or 4th of
January, according to the lunar
calendar, and closes on the 25th of the month. The three days beginning on New
Year’s Day and ending with
the 3rd are given up to the New Year’s Festival, and from the following day, the
great Monlam season sets in.
In order to make arrangements for the coming festival, the priests are given
holiday from the 20th of December.
Holiday, however, is a gross misnomer, for the days are spent in profane pleasures
and in all sorts of sinful
amusements. The temples are no longer sacred places; they are more like gambling-
houses—places where the
priests make themselves merry by holding revels far into the night. Now is the time
when the Tibetan priesthood
bids good-bye for a while to all moral and social restraints, when young and old
indulge themselves freely to
their heart’s content, and when those who remain aloof from this universal practice
are laughed at as old fogeys.
I had been regularly employing one little boy to run errands and to do all sorts of
work. In order to allow him to
enjoy the season, I engaged another boy on this occasion. I might have dispensed
with this additional boy
altogether, for as the two boys never remained at home, and even stayed away at
night, it was just as if I had had
no boy at all. And so for days and days religion and piety were suspended and in
their places profanity and vice
were allowed to reign supreme.
The wild season being over—it lasts about twelve days—the Monlam festival
commences. This is preceded by
the arrival of priests at Lhasa from all parts of Tibet. From the monasteries of
Sera, Rebon, Ganden and other
large and small temples, situated at a greater or less distance, arrive the
contingents of the priestly hosts. These
must number about twenty-five thousand, sometimes more and sometimes less,
according to the year. They take
up their quarters in ordinary houses, for the citizens are under obligation to
offer one or two rooms for the use
of the priests during this season, just as people of other countries are obliged to
do for soldiers when they carry
out manoeuvres in their neighbourhood.
[ Excerpts from Three Years in Tibet, by Ekai Kawaguchi ]
26. Which of the following can be inferred from the passage?
(a) The festival of Monlam is celebrated in the winter season.
(b) Monlam permanently gets rid of priests’ sinful pleasures of life.
(c) Soldiers play an important role in the festival of Monlam.
(d) Tibetan priests are all old men.
27. According to the passage which of the following is false?
(a) Two of the boys that the author appointed were very hard working.
(b) Lhasa monastery makes several arrangements for their guest priests to stay
there during the festival.
(c) The main intention behind the festival of Monlam is the welfare of people.
(d) All of the above.
28. Why did the author say that holiday is a gross misnomer?
(a) Because the priests are never allowed to rest before the festival commences.
(b) Because the priests are never allowed to leave the holy premises of their
respective temples or monasteries.
(c) Because during that time period priests indulge themselves in morally corrupt
activities.
(d) Because during that time no one was allowed to have fun except the royal
family.
29. Which of the following words can be used in place of the word ‘manoeuvres’
without changing the context?
(a) Operations (b) Negotiations (c) Controls (d) Plays
30. Which of the following reflects the writing style of the author?
(a) Analytical (b) Persuasive (c) Narrative (d) Expository

Directions (Q.66 – Q.105): Read the comprehensions carefully and answer the
questions based on it.
Passage (Q.66-Q.69): Section 2(a) of the Indian Contract Act 1872 defines proposal
as ‘When one person
signifies to another his willingness to do or to abstain from doing anything, with
a view to obtaining the assent
of that other to such act or abstinence, he is said to make a proposal’.
Following are the essential conditions for a valid offer:
 Intention to create legal obligation
 Express or implied
 Certainty of terms
 Silence as acceptance
 Expression of willingness to do or to abstain from anything
 Offer must be communicated: communication of a proposal is complete when it comes
to the knowledge of
the person to whom it is made. This may be express or implied.
 Cross offers: when two parties make identical offers to each other, in ignorance
of each other’s offer, the
offers are termed as ‘cross offers’. Such offers do not constitute acceptance of
one’s offer by the other thus
they do not lead to a completed agreement.
 When the offeree offers a qualified acceptance of the offer subject to
modifications and variations in terms
of the original offer, he is said to have made a counter offer.
 Invitation to offer: in a valid offer there is an expression of willingness to do
or abstain from doing an act
with a view of obtaining assent from the other while in case of invitation of offer
a party without expressing
his final willingness proposes certain terms on which he is willing to negotiate.
There in an invitation to offer
the party does not make an offer but invites the other party to make an offer on
those terms
 The communication of a proposal is complete when it comes to the knowledge of the
person to whom it is
made. The communication of acceptance is complete against the proposer when it is
put in course of
transmission to him, so as to be out of the power of the acceptor. The
communication of acceptance against
the acceptor gets complete when the proposer comes to know about the acceptance.
(Extracted with requisite revisions and edits from at
https://www.google.com/amp/s/lawctopus.com/clatalogue/indian-contract-act-
introduction/amp/)
66. Ram and Ghanshyam were best friends who discussed everything and all the
updates in their life with each other.
One such day, Ram was telling about selling one of his bike to someone who is
willing to pay a decent price of
50k for the same. Next day Ghanshyam goes to his house with the quoted prize and
says he will buy his red
Royal Enfield bike. Ram however, refuses to sell it and says he never intended to
sell that bike but it was his
metallic red Glamour bike which he wanted to sell. Was an offer been made?
(a) Yes, the details as to the bike and the price has been told to Ghanshyam in
clear terms thus an offer has been
made.
(b) Yes, Ram had signified his intention to Ghanshyam of selling his bike for the
purpose of obtaining the
consent from Ghanshyam or any other purchaser willing to pay the requested sum and
hence an offer has
been made.
(c) No, there was no certainty as to the bike which was to be sold and no offer was
made by Ram to Ghanshyam.
(d) No, there is no offer in the present scenario but only invitation to offer by
Ram to Ghanshyam.

. Page 16 of 40
67. Ram waited for a long time to purchase his red Royal Enfield (RE). When the RE
was brought in the market, he
was first to do the booking and purchase the bike. On the day of Sankranti (Indian
festival), he went to make the
payment and purchase the RE. However, due to huge demands for RE, the price shot up
(as the festival discounts
were scrapped by the shop) which was also communicated for all the pre-bookers. Ram
however proposes the
same amount (discounted) and tells Raman, the shop-owner that he can keep 5k as his
commission if he sells the
bike at that price. Can the proposal by Raman be understood as accepted by Ram?
(a) No, Ram had made a counter-offer to Raman by quoting the new price to Raman and
hence the proposal has
not been accepted.
(b) Yes, there is no material alteration in the terms of the offer and the amount
quoted by Ram is the one at
which the RE was initially sold (festival discounts), therefore the offer by Raman
is accepted by Ram.
(c) No, Ram had made a cross offer to Raman and hence no valid contract has been
culminated between the
parties.
(d) Yes, there is certainty in the terms of the offer and the acceptance is also
made in the same respect and on
the same lines, therefore a valid acceptance has been made by Raman.
68. With schools remaining close due to covid pandemic, one Ms. Rinku decided to
utilize this time by cultivating
her long lost love for arts. She went to a stationary shop and gave her purchase
list to the shopkeeper. One such
item mentioned was an Easel, which she could clearly see was present in the shop
(which was also an antique
piece by the looks of it) but the shopkeeper declined to sell it. According to Ms.
Rinku he couldn’t do so as he
has already made the offer which he couldn’t withdraw now. Is she correct?
(a) Yes, by keeping the objects in the shops is a clear indication of the general
offer being made which could be
accepted by anyone including Ms. Rinku and therefore, the shopkeeper cannot refuse
to later rescind the
offer once it is accepted.
(b) No, since Ms. Rinku had not negotiated as to the details of the offer made by
the shopkeeper including the
color or price of the easel, the shopkeeper is free to rescind the offer.
(c) Yes, by taking the purchase list of Ms. Rinku, the shopkeeper bound himself
with selling all materials which
are enlisted and are present in his shop.
(d) No, there was no offer made by the shop but it was merely an invitation to
offer and hence the shop-owner
is correct in doing so.
69. While Ms. Rinku was returning from the shop, she stopped an auto and asked him
to take her to Shivajinagar
(where her house was located). The autowala quoted the fare to be 150 bucks. Ms.
Rinku however told him that
she will not ride his auto for more than 120 bucks. Autowala remains mum and
signifies her to sit in the auto.
She rode the auto and at the destination paid 120 bucks to which the autowala
disputes and says she is required
to pay 150 bucks only. Is Ms. Rinku correct in paying 120 bucks?
(a) No, there was no acceptance on the part of the autowala to offer her the ride
at 120 bucks and hence he is
eligible to the 150 bucks.
(b) Yes, there was an implied acceptance by the autowala to take Ms. Rinku at the
price quoted by her i.e. 120
bucks.
(c) No, by riding the auto there is an implied consent on the part of Ms. Rinku to
pay the amount quoted by the
autowala i.e. 150 bucks.
(d) Yes, this is a situation of making cross offers which has been accepted by the
autowala and is only entitled
to 120 bucks for the ride.

. Page 17 of 40
Passage (Q.70-Q.73): In the law of torts, there is a duty on every person do acts
with reasonable care in order
to avoid any harm which may occur due to their failure of taking such care. For
e.g., if a person is driving his
car, he has a duty to drive the car safely and within speed limits so that no
accident occurs which can also harm
any other person.
In case a person gives his consent to doing of an act which leads to him getting
injured, then even if an injury is
caused by the other person, he cannot claim any damages from that person because
the act was one for which he
voluntarily consented. The consent of the plaintiff acts as a defence and this
defence is called volenti non fit
injuria which means to a willing person no injury happens.
There are 2 essential elements in this defence:
 The plaintiff has the knowledge of the risk
 The plaintiff with the knowledge of risk has voluntarily agreed to suffer the
harm.
When a plaintiff gives his consent for an act such consent should be free from any
coercion, fraud or any other
such means by which the free consent can be affected. If the consent is given for
an act which is not allowed by
law then, even on the fulfilment of all the essential conditions of this defence,
the liability cannot be escaped and
thus in such cases, this defence becomes inoperative. The defence of volenti non
fit injuria is not applicable in a
case where the defendant has been negligent. Thus only where there is no negligence
by the defendant, he can
claim this defence to escape liability.
(Extracted with requisite revisions and edits from
https://blog.ipleaders.in/volenti-non-fit-injuria/)
70. Ikaris went with his friends on an adventure trip to Himachal where they
subscribed for various adventure sports
and risk involved with it. One such sport was river rafting. Due to the currents of
the waters, the raft tumbled
despite all the cautions by the organizers and due to cold feet couldn’t move an
extent and thereby his leg hits a
rock. He brought a claim against the organizers for damages. Is his claim likely to
succeed?
(a) Yes, Ikaris has only agreed to the harm which are a direct consequence of
rafting however hitting with the
rock indicates that there was lack of due care and caution on the part of the
organizers to ensure a safe
passage.
(b) Yes, there was absence of reasonable care and preparedness on the part of the
organizers who failed to timely
rescue Ikaris after the raft tumbled.
(c) No, Ikaris had agreed and had full knowledge of the risks involved in the sport
and thus cannot recover any
damages from the organizers.
(d) No, there was negligence on the part of Ikaris as he failed to take into
account all the factors which might
cause him harm before agreeing for the sport.
71. Thena has been working in the mine industry for many years and had noticed that
one of the pulley used to draw
the minerals has been rusting and might break at any time. This fact was
highlighted by her to the administration
time and again but the administration paid no heed to her requests and assured her
of the safety. So, she continued
to work. One day, the pulley broke and few rocks fell over her and fractured her
leg. She brought a claim for
damages against the administration. Is her claim likely to succeed?
(a) Yes, there was negligence on the part of the administration who failed to take
necessary actions required and
therefore would be liable to pay damages to Thena.
(b) No, Thena was already aware of the damaged pulley and despite this her working
in the mines indicates that
she has voluntarily agreed to suffer harm and is therefore not entitled to any
damages.
(c) Yes, the administration made false assurances and would thus be bound by the
law of promissory estoppel
and Thena would be entitled to damages for the same.
(d) No, Thena was herself negligent in her conduct as she continued to work despite
the present state of the
pulley.

. Page 18 of 40
72. Gilgamish went to an eye surgeon for his temporal eyesight issues. Dr. Ajak
prescribed an eye surgery to him
for recovery which according to her was the only mode for cure. Dr. Ajak had failed
to inform her that there is
a possibility (although very slight) of complete loss of vision. However, the
surgery led to the complete loss of
vision for her. He brings in a claim for damages against Dr. Ajak to which Dr. Ajak
claims that she had consented
to all such scenarios as the failure of the surgery is a natural outcome and need
not be expressly informed. Is her
argument correct?
(a) Yes, Dr. Ajak performed the surgery with all due care and caution required in
the field and hence cannot be
made liable for any claims.
(b) No, Gilglamish neither had the knowledge of the risk involved nor agreed to
suffer any such harm voluntarily
and therefore the argument by Dr. Ajak is incorrect.
(c) No, Dr. Ajak failed to take all the necessary precautions and due care required
to perform the surgery and
hence would be liable to pay damages to Gilgamish.
(d) Yes, Gilgamish had impliedly consented to all the risks involved in performing
the surger including the loss
of sight and hence would not be entitled to any damages from Dr. Ajak.
73. Overtaker was a famous wrestler in the WWD (World Wrestling Domain). He has
been a part of various fights
and emerged victorious. During one such fight, things got heated a little and the
fight got real and intense which
led to Overtaker hurting the other player, John badly. John brings a claim for
damages against him. Will such a
claim likely to succeed?
(a) No, there was no negligence attributable to Overtaker or any false play
involved and hence John is not
entitled to any damages.
(b) Yes, John has not consented to serious injuries but to only minor injuries
which might result in the normal
course of the game.
(c) Yes, the terms and conditions of the WWD Agreement has to be seen in order to
invoke and estimate the
liability of Overtaker.
(d) No, John has consented to such a harm by being a part of such fights and
therefore is not entitled to any
damages.
Passage (Q.74-Q.77): Section 182 of the Contract Act, 1872, defines “Agent" as "a
person employed to do any
act for another in dealing with third parties”. The person for whom such act is
done, or who is represented, is
called the principal. It is only when a person acts as a representative of another
in business dealings and
negotiations, between that other person and a third party, he is legally an agent
of the person on whose behalf he
is acting.
Usually, an agent occupies no personal liability while contracting for his
principal and therefore, it is not
necessary that he should be competent to contract. An agent has a fiduciary duty to
act loyally for the principal’s
benefit in all matters connected with the agency relationship. The first and the
foremost duty of every agent is to
carry out the mandate of this principal. He should perform the work which he has
been appointed to do. Any
failure in this respect would make the agent absolutely liable for the principal’s
loss.
According to Section 211, an agent is bound to conduct the business of his
principal according to the directions
given by the principal and to keep himself within the confines of his authority. In
the absence of directions, the
agent has to follow the custom which prevails in businesses of the same kind and at
the place where the agent
conducts such businesses. When the agent acts otherwise, if any loss be sustained,
he must make it good to his
principal, and if any profit accrues, he must account for it.
[Extracted with revisions from ‘Duties of an Agent’ by Dipti Khatri, published on
lawctopus
https://www.lawctopus.com/academike/duties-agent/ ]

. Page 19 of 40
74. S was a merchant involved in the trade of raw cotton. R was the agent of S.
consignment of raw cotton was to
be collected by a trader. S instructed R to store the consignment in a water-proof
warehouse till the same was
collected by the concerned trader. R selected a famous warehouse and stored the
consignment there, but forgot
to confirm if the warehouse was waterproof. At night, there was a heavy rain and
the warehouse got flooded. As
a result, the entire consignment was destroyed. Is R liable for this destruction?
(a) Yes, because R did not follow S’s instructions properly and he is absolutely
liable to compensate for the loss
caused due to his negligence
(b) Yes, because as an agent, R occupies no personal liability while contracting
for his principal
(c) No, because R followed S’s instructions and his mandate ends after doing
performing the assigned task
loyally
(d) No, because as an agent, R occupies no personal liability while contracting for
his principal
75. Manishq was a popular jeweller. He was exporting his jewellery to Sudan and
appointed a commission agent X
to take care of the jewellery in the intermediate. X was instructed to store the
jewellery in a safe place and insure
the same. X also charged a premium for insuring the jewellery. Subsequently, X
stored the jewellery in a safe
godown but failed to insure the goods. The goods were stolen. Can Manishq claim the
loss from X?
(a) No, because when an agent acts otherwise than the directions given by the
principle he must not account for
any loss accrued as a consequence of his disobedience
(b) No, because there was no prevalent custom in the concerned trade which made it
necessary for X to insure
the jewellery
(c) Yes, because X failed to insure the jewellery despite clear instructions so he
is liable to make good for the
loss accrued to Manishq
(d) Yes, because there was a prevalent custom in the concerned trade which made it
necessary for X to insure
the jewellery
76. In the previous fact situation, assume that Manishq did not ask X to insure the
jewellery and X did not charge a
premium for the same. Manishq simply asked X to safely and securely store the
jewellery till it was exported. It
is an established practice in the jewellery business to always insure the jewellery
as the concerned goods tend to
be precious and extremely expensive. Since no directions were given in this regard,
X did not insure the
jewellery. The jewellery was stolen. Is X now liable to Manishq for the loss?
(a) Yes, because X departed from the instructions of Manishq and is liable to
compensate for the loss caused as
a result of this disobedience
(b) Yes, because X was liable to follow the established practice of insuring the
goods which is prevalent in the
jewellery business even if no express directions were given by Manishq
(c) Yes, because was liable to follow the established practice of insuring the
goods which is prevalent in the
jewellery business because express directions were given by Manishq
(d) No, because X departed from the instructions of Manishq and is liable to
compensate for the loss caused as
a result of this disobedience
77. Seeta was appointed as an agent to sell a house. She received an offer of Rs. 1
lakh which she promptly
communicated to her principal. The latter accepted it provisionally “subject to
contract”. Subsequently, Seeta
received a higher offer of Rs 2 lakhs, which she failed to pass on to the
principal. As a result, the house was sold
off for Rs. 1 lakh. Can the principal claim a balance of Rs. 1 lakh from Seeta?
(a) No, because Seeta followed the mandate of her principal and communicated all
the offer to them
(b) No, because Seeta is not liable to compensate for the loss of Rs. 1 lakh as the
sale of the house was “subject
to contract”
(c) Yes, because Seeta is liable to make good the principal’s loss in terms of the
difference in the two prices and
she should have communicated and accepted the better offer
(d) Yes, because Seeta was given express directions to sell the house at the best
possible price and she should
have sold the house to the buyer offering Rs. 2 lakhs

. Page 20 of 40
Passage (Q.78-Q.82): “Habeas Corpus” is a Latin term which literally means “you may
have the body”. The
writ is issued in the form of an order calling upon a person by whom another person
is detained to bring that
person before the court and to let the court know by what authority he has detained
that person. Thus, the main
object of the writ is to give quick and immediate remedy to a person who is
unlawfully detained by the person
whether in prison or private custody. Supreme Court has held that production of the
body of the person alleged
to be unlawfully detained is not essential. Writs can be issued only by High Courts
and Supreme Court and not
a district court or a court below the level of High Court.
The writ of habeas corpus is generally applied by a person who is illegally
detained but in certain cases, an
application of habeas corpus can be made by any person on behalf of the prisoner
i.e., a friend or a relative.
Where the detenue is in judicial custody pursuant to remand order passed by
jurisdictional magistrate or any
competent court, the habeas corpus petition is not maintainable. The writ of habeas
corpus cannot only be used
for releasing a person illegally detained but it will also be used for protecting
him from treatment inside jails.
The writ or order in the nature of mandamus would be issued when there is failure
to perform a mandatory duty.
The writ of mandamus does not lie against a private individual or any private
organization because they are not
entrusted with a public duty. The duty sought to be enforced must be a public duty,
that is, duty cast by law. A
private right cannot be enforced by the writ of mandamus.
Source: Constitution of India by V.N. Shukla, latestlaws.com.
78. A lived in Hubly. A was arrested on 6th January, 2020 in relation to a riot
held between two groups. His family
members were not informed about his arrest till 19th January, 2020. B, A’s brother
went to Hubly High Court
and pleaded before the court to know about his brother and to issue a writ of
habeas corpus desiring to know the
whereabouts of A. Choose the correct option.
(a) B’s pleading will be accepted by the court and the writ will be issued.
(b) B’s pleading shall be rejected because not B but only A can file for the writ.
(c) B’s pleading will be rejected because A is the mastermind of the riot.
(d) B’s pleading will be accepted however the writ will not be issued.
79. After analysing the legality of the above mentioned passage, choose the
incorrect legal statement from the
following.
(a) Writ of habeas corpus can be filed by the detenu or his relative or friend.
(b) Production of body is an essential ingredient of habeas corpus.
(c) The writ or order in the nature of mandamus would be issued when there is
failure to perform a mandatory
duty.
(d) A public duty and not a private duty can be enforced by the writ of mandamus.
80. In Question No. 78, let’s assume that A has been convicted by the district
court and sent to judicial custody for
30 days. Choose the correct option now.
(a) B’s pleading will be accepted by the court and the writ will be issued.
(b) B’s pleading will be rejected and A will be released as soon as possible.
(c) B’s pleading will be rejected owing to the judicial custody ordered by the
district court.
(d) B’s pleading will be accepted and writ of mandamus will be issued.
81. A was a farmer in a small village in district Midnapur. A went to a farer rally
on 03.02.2020 and did not return
home till 09.03.2020. Desiring to know about A’s whereabouts, A’s brother G went to
the local police station to
file a FIR which was denied by the station incharge. G then went to Midnapur
District Court and requested to
issue a writ of habeas corpus and mandamus. Choose the correct answer.
(a) G’s request will be entertained because G is a relative of A and can file for
habeas corpus writ.
(b) Writ of habeas corpus only will be issued by Midnapur Court and not of
mandamus.
(c) No writ will be issued as Midnapur District Court is not competent to issue any
writ.
(d) G’s request to issue mandamus will only be entertained and not of habeas
corpus.

. Page 21 of 40
82. In Question 16, let’s assume A has been convicted of waging war against nation
and has been sent to jail for 10
years and was brutally tortured there. A wrote a letter regarding the same to G. G
even after making many
applications was not allowed to meet A since last 1 years. Decide the legal
recourse available to G.
(a) G can request the Midnapur High Court to issue a writ of mandamus against
Midnapur District Court.
(b) G can request Midnapur High Court for the issuance of writ of habeas corpus
owing to brutal torture of A in
jail.
(c) G’s request to issue habeas corpus will be denied by Midnapur High Court
because G already knows where
is A.
(d) Only Supreme Court and not Midnapur High Court can assist G in issuance of writ
of habeas corpus.
Passage (Q.83–Q.87): Conversion is an act of wilful interference, without lawful
justification, with any chattel
in a manner inconsistent with the right of another, whereby that other is deprived
of the use and possession of it.
Thus, conversion would be caused if the chattel belonging to another person is
interfered with in a manner, which
is inconsistent with the rights of that person entitled to that chattel.
Conversion can be committed in many ways. Conversion by Taking is when a person
takes the chattel out of the
possession of the owner. The demand for chattel is not needed for its return to its
rightful owner. In these
circumstances it becomes the duty of the person in whose custody the chattel is, to
return it to its owner.
Conversion might be by detention as well, however such is the case only when it is
averse to the owner or other
such persons who are entitled to possession and is without any lawful
justification. It is essential in this case that
the person interfering must have shown an intention to keep the thing in defiance
of the owner. A person is guilty
of conversion by wrongful delivery when he, without lawful justification, deprives
a person of his goods by
delivering them to someone else, so as to change the possession. If a person gives
some other person the right to
title of a good, which belongs to somebody else without lawful justification, that
person would be deemed to be
guilty of conversion by wrongful disposition or by parting with goods. A mere
damage which falls short of actual
destruction cannot be said to have amounted to conversion. The destruction caused
to the chattel of the party
must be such that either the identity of that article is deformed or it is no
longer of the same use as that it can be
normally used.
Source: Extracted, edited and recreated from: legalserviceindia.com,
indiankanoon.org.
83. X bought the crop of maize from Y's field by a contract of sale and took the
delivery of the crop. However, in
spite of Y's repeated demands, X did not pay him the money for the same. Has X
committed the tort of conversion
in any of its forms?
(a) Yes, by detaining the amount to which Y is entitled to.
(b) Yes, by wrongfully taking Y's crops.
(c) No, as he bought to crop from Y.
(d) No, as X had a right to take the crop from Y.
84. X was a shop owner in whose premises Y, a customer of his, forgot his bicycle.
Fearing that it might get stolen
in the night, X kept the bicycle in his garage before locking up the garage in the
evening. When Y came to look
for the same at night, he did not find it in the premises of X's shop. Can Y sue X
for conversion ?
(a) Yes, as it was X's duty to return the bicycle to its owner.
(b) Yes, as X did not have any lawful justification for moving the bicycle.
(c) No, as X did it with lawful justification.
(d) No, as there was no change in the possession of the bicycle.
85. X was the owner of a valuable painting which was stolen by a thief. The thief
sold the painting to an art collector,
Y, who auctioned it off to Z. Upon finding out about the ownership of the painting,
X sues Y for conversion.
Decide the legal consequences accordingly.
(a) Y is liable for conversion by taking as he bought the painting from a thief.
(b) Y is liable for conversion by wrongful disposition as he bought the painting
from a thief.
(c) Y is liable for conversion by detention as he set up a title in defiance of the
owner of the painting.
(d) Y is liable for conversion by wrongful delivery with goods as he wrongful
conferred a title on Z.

. Page 22 of 40
86. Following the above mentioned hypothetical situation, X went to Z to ask for
his painting back. Z refused to
give it back to X as he had bought it at a fair price and had no connection to the
person who had stolen the
painting originally. X, getting enraged with the injustice being done to him, threw
a bottle of ink at the painting
which completely defaced the painting. Z sues X for conversion. Will he succeed in
his suit ?
(a) No, as X had a lawful justification to deface the painting.
(b) No, as X was the rightful owner of the painting.
(c) Yes, as X destroyed the painting without a lawful justification.
(d) Yes, as Z was the rightful owner of the painting.
87. P gave his car for repair at Q's garage. After the repairs were done, Q told P
the amount due, but P asked him to
give the car back on credit and promised to pay him back. Q refused to give P his
car back until he paid the dues
for the work done by Q. in order to prevent P from taking the car forcefully, Q put
the car in the garage and
locked its door. Decide accordingly.
(a) Q is liable for conversion by detention.
(b) Q is not liable for conversion, as Q act is lawfully justified.
(c) Q is liable for conversion by wrongful disposition, as Q locked the car inside
the garage with mala fide
intention
(d) Q is liable for conversion by taking, as Q was not the owner of the car
Passage (Q.88-Q.92): Generally, a person is liable for his own wrongful acts and
one does not incur any liability
for the acts done by others. In certain cases, however, vicarious liability, that
is the liability of one person for the
act of another person, may arise. In the words of Lord Chelmsford: “It has long
been established by law that a
master is liable to third persons for any injury or damage done through the
negligence or unskillfulness of a
servant acting in his master’s employ. The reason for this is, that every act which
is done by a servant in the
course of his duty is regarded as done by his master’s order, and, consequently it
is the same as if it were the
master's own act." Vicarious liability is attracted only upon fulfilment of some
constituents that there must be a
relationship of master-servant in a certain kind, the wrongful act must be related
to the relationship in a certain
way and the wrong has been done within the course of employment.
The liability of the employer arises for the wrongs committed by his servant and
not in respect of wrongs
committed by an independent contractor. A servant and independent contractor are
both employed to do some
work for the employer but there is a difference in the legal relationship which the
employer has with them. A
servant is engaged under a contract of services whereas an independent contractor
is engaged under a contract
for services. A servant is an agent who is subject to the control and supervision
of his employer regarding the
manner in which the work is to be done. An independent contractor is not subject to
any such control. He
undertakes to do certain work and regarding the manner in which the work is to be
done, he is his own master
and exercises his own discretion.
Preeti Singh, ‘Vicarious Liability in India’ (Legal Services India)
<http://www.legalservicesindia.com/article/1634/Vicarious-Liability-in-India.html>
as accessed on 12 January
2022.

. Page 23 of 40
88. National Law University Odisha (University) hired a contractor for renovation
and painting of the entire campus,
and dictated the colour scheme as well as the renovation to be performed. As per
the general practice, the workers
are expected to tie the bucket of the paint so that it does not fall and injure
anyone. However, one such worker
forgot to follow it, and a bucket fell injuring one of the students, who has
claimed compensation for the physical
injury and mental trauma suffered by them. In the suit he has made both the
contractor and the University a party.
University claims that they have hired workers under contract for service, and the
contractor shall be solely
liable. Student contends that the safety of a student in the premises of the
university administration and thus
university is also liable for the act of the worker. Decide:
(a) The University alone shall be liable to pay compensation as it is their
responsibility to take care of the
wellbeing of the students in the premises, which they have failed to perform.
(b) Both the University and the contractor have the joint liability to pay
compensation as the worker was
employed by the contractor and working under control of the University as per its
directions.
(c) Neither shall be liable to pay compensation as the worker was expected to tie
the bucket as per the
professional practice, and no one would be liable for this negligence and
unprofessionalism of the worker.
(d) Contractor alone shall be liable for the negligence of the worker, as he was
the employer and the university
had only hired them as independent contractors.
89. Ashraf was a freelance journalist, who was engaged on a casual basis, and was
employed under a series of
contracts by different news channels as and when required. However, whenever he did
reporting for a news
channel, he was told the requirements and was portrayed as a journalist of such
channel. Once while interviewing
a politician Ashraf asked some defamatory questions and made some malignant
remarks, which he was strictly
prohibited to do under the contract with the news channel. The politician has filed
a suit for defamation against
the reporter and the news channel to compensate him for his loss. Choose the
correct answer.
(a) Ashraf himself is liable as he was an independent contractor of the news
channel and defamation attracts
tortious liability to pay the compensation which cannot be imposed on the channel.
(b) Ashraf himself is liable as the contract with the news channel strictly
prohibits him from using defamatory
language, and thus the channel cannot be held liable for defamation.
(c) The news channel shall be liable as being the employer, it is liable for all
kinds of acts done by Ashraf in the
course of employment, whether negligent or not, professional or not.
(d) The news channel shall be liable as Ashraf was being represented as journalist
of the news channel while
asking the defamatory questions, which makes the channel directly liable to pay the
compensation.
90. The defendants in the present case engaged a band called ‘The Lovefist’ to play
at their dance hall. The
defendants have given a list of all the songs to the band to be played, and
exercised very severe and direct control
over the band in terms of the durations for which each song shall be played, the
loudness of the songs and even
the instruments to be used for each song. During the event, the band played two
songs, the owners of the
copyright of such songs filed a suit for breach of copyright against the defendants
and claimed compensation.
Choose the correct option.
(a) The defendants shall not be liable for compensation as they had hired the band
as independent contractors
and thus are not liable for playing the songs without permission of their owners.
(b) The defendants shall be liable for compensation as even though the band was
hired for the time being, the
nature of control exercised by the defendants on them make them vicariously liable
for breach of copyright.
(c) The defendants shall not be liable for compensation as they had already given
the list of songs to be played
at the event, and defiance of their directions will end their liability with
respect to acts of the band as their
employee.
(d) The defendants shall be liable for compensation as the event was organized by
them and whether or not the
band was a servant of the defendants, they will be liable for the breach of
copyright.

. Page 24 of 40
91. Prajjawal called a cab for the airport from his home. He informed the driver of
the location and also the route he
prefers to take to the airport. Since he was getting late, he also requested the
driver to drive a little faster. The
driver, keeping in mind the paucity of time, drove rashly even on crowded streets.
The cab hit a pedestrian,
resulting in a fracture in their arm. The pedestrian has sued the cab driver, who
in the court argues that he was
merely following directions by Prajjawal, who exercised control on him as his
employer for the time being.
Decide:
(a) Prajjawal is vicariously liable as he had control over the destination, the
route and even the speed at which
the car was being driven, and thus was the master of the driver for the time being.
(b) Prajjawal is not vicariously liable as he was merely stating his requirements
and exercised no control over
the manner in which they were to be performed by the driver.
(c) Prajjawal is vicariously liable as he had not objected to the driver rashly and
thus approved of it, making him
jointly liable for the accident along with the driver.
(d) Prajjawal is not vicariously liable as the pedestrian has only sued the cab
driver and he is not even a party to
the suit for compensation to be paid.
92. Assertion (A): Master is liable only for the acts done by the servant done in
the course of their employment and
not otherwise.
Reasoning (R): Acting negligently while performing duties of employment per se
exclude such actions from the
course of employment.
Choose the correct option:
(a) Both A and R are correct, R is the correct explanation of A.
(b) Both A and R are correct, R is not the correct explanation of A.
(c) A is correct and R is incorrect.
(d) Both are incorrect.
Passage (Q.93-Q.97): Divorce in general means the breakage or dissolution of
marriage with the help of the
law, so that one can leave his or her spouse and become free from marital duties
with some exceptions. Law
grants divorce on multiple grounds given under section 13 of the Hindu Marriage
Act, 1955 including cruelty.
Every matrimonial conduct, which may cause annoyance to the other, may not amount
to cruelty. Mere trivial
irritations, quarrels between spouses, which happen in day-to-day married life, may
also not amount to cruelty.
Cruelty in matrimonial life may be of unfounded variety, which can be subtle or
brutal. It may be words, gestures,
or by mere silence, violent, or non-violent.
Judicial pronouncements adjudicate cruelty on case to case basis but have
identified some incidents as cruelty
such as leveling of false allegation by one spouse about the other having alleged
illicit relations with different
persons outside wedlock, or when the other spouse levels an allegation that their
spouse is a mental patient, or a
husband cannot ask his wife that he does not like her company, but she can or
should stay with other members
of the family in the matrimonial home. In Naveen Kohli vs. Neelu Kohli [AIR 2004
All 1] held Physical violence
is not absolutely essential to constitute cruelty. A consistent course of conduct
inflicting immeasurable mental
agony and torture may constitute cruelty. In conclusion, it is sufficient that if
the cruelty is of such type that it
becomes impossible for spouses to live together. If the intention to harm, harass
or hurt could be inferred by the
nature of the conduct or brutal act complained of, cruelty could be easily
established. But the absence of intention
should not make any difference in the case. The cruel treatment may also result
from the cultural conflict between
the parties.
Avinash Gadhre, ‘Cruelty as ground for divorce’ (Legal Services India)
<http://www.legalservicesindia.com/article/1900/Cruelty---as-a-ground-for-
Divorce.html> as accessed on 12
January 2022.

. Page 25 of 40
93. Ashi had resided in a metropolitan area for all her life before her father
married her off to a landlord in a village.
Only if adjusting to the new life was not enough, she was also compelled to
contribute to the chores. While she
managed to excel at everything else, she always got disgusted when making uple (cow
dung cakes). She had
repeatedly requested not to be asked to perform the task any further, but she was
always forced to perform it.
Eventually, she returned to her father’s home and filed for divorce on grounds of
cruelty. Choose the correct
option:
(a) Divorce shall not be allowed as there was no cruelty inflicted on Ashi, every
housewife performs such chores
and it does not make living together impossible for the spouses.
(b) Divorce shall be allowed as being forced to do a chore despite repetitive
requests to do otherwise caused
mental agony to Ashi which eventually made her to leave her husband’s house.
(c) Divorce shall not be allowed as mere trivial irritations which happen in day to
day life is not a sufficient
reason to take divorce on grounds of mental cruelty.
(d) Divorce shall be allowed because Ashi was forced to settle in a rural setting
without her consent and thus her
marriage only was enough to constitute mental cruelty.
94. Siddhart filed an application for divorce with her wife Shehnaz on the grounds
of cruelty. He argued that Shehnaz
made him leave his parents’ house, disrespected and maligned him publicly, and got
intimate with someone else.
Moreover, she withdrew 10 lacs from their joint account and deposited the same in
her own account for personal
use. In defence Shehnaz proved that Siddhart immorally cohabited with another lady
right from their marriage,
and thus her actions were justified. Siddhart on the other hand was incapable of
disproving the allegations.
Decide whether the court shall allow the application or not.
(a) The court shall reject the application as Shehnaz’s actions were justified on
the account of his husband being
cohabiting with another woman, during the course of their marriage.
(b) The application shall be allowed because it is evident from the facts that the
couple is incapable of living
together and thus shall be separated despite the wife not wishing to do so.
(c) The court shall reject the application because it is a renowned principle of
law that no one shall be allowed
to benefit from their own wrong, and Siddhart shall not be allowed to request
divorce when he is himself
guilty of adultery.
(d) The court shall allow the application as facts unambiguously suggest that
Sehnaz was inflicting mental agony
on Siddhart which amounts to cruelty.
95. Rehana and Akash had eloped because their parents were against an inter-caste
marriage. Soon after marriage
Rehana was being asked to change her lifestyle to conform with his culture. Also
Akash wanted to raise their
children as per his traditions, whereas Rehana wanted them to decide their own
faith as they grow up. Over the
time Rehana felt that her religion and faith is being subdued by Akash’s and that
she, at times, is unintentionally
being forced to perform rituals which her own faith does not allow. She has filed
for divorce from Akash, who
argues that he never intended to subject Rehana to any mental cruelty but that he
will not allow practise of any
other religion in his house. Decide:
(a) Rehana has been subjected to mental cruelty as even though there was no
specific intention, there was a
cultural conflict between the parties that Akash does not want to resolve.
(b) Rehana has not been subjected to mental cruelty as she should have foreseen the
restraints on practise of her
faith when she eloped with a person from another caste.
(c) Rehana has been subjected to mental cruelty as she was not satisfied with her
life after the marriage, and was
not entitled to give opinion regarding the life of her children.
(d) Rehana has not been subjected to mental cruelty as there was no intention to do
so, and because she should
have confronted Akash before directly applying for divorce.

. Page 26 of 40
96. Munna, a businessman, married Madhuri because of compulsion from his father.
Unhappy in the wedlock, he
entered into a void marriage with Chandni without getting a divorce from Madhuri.
Chandni always insisted on
settling with Munna’s family but he always managed to delay it, giving some
pretext. Once when Chandni
reached Munna’s home without asking him, she found out that he was already married
to Madhuri, and had
concealed it all this while. Victim of the betrayal, Chandni filed for a divorce in
the Family Court on the grounds
of mental cruelty suffered due to the incident. Choose the correct answer to decide
if she shall be granted the
divorce or not.
(a) Chandni shall not be granted the divorce as the court shall also consider Munna
was compelled for his first
marriage, and he truly wishes to live with Chandni only.
(b) Chandni shall be granted the divorce as hiding a previous marriage from her is
undoubtedly subjecting her
to mental cruelty which entitled her to a divorce.
(c) Chandni shall not be granted the divorce as theirs was a void marriage and
therefore Chandini cannot use
divorce as a remedy.
(d) Chandni shall be granted divorce because despite her repeated requests Munna
never took her to his parents’
house and never accepted her publicly which is undeniably cruelty.
97. With all the facts remaining the same, Madhuri also files an application for
divorce from Munna after getting to
know that he has remarried with Chandni despite being married to her. Decide
whether there is ground to grant
her divorce from Munna.
(a) Madhuri cannot be granted divorce as the subsequent marriage of Munna is void
in the eyes of law, and thus
there arises no basis for claiming cruelty.
(b) Madhuri can be granted divorce as Munna had secretly married Chandni which
suffices as a basis for
claiming mental cruelty, and thus asking for divorce.
(c) Madhuri cannot be granted divorce as Munna was forced into marriage with
Madhuri and thus he was the
one suffering mental cruelty and not Madhuri.
(d) Madhuri can be granted divorce as Munna had never accepted her as his wife, and
it is impossible for the
two to live together anymore.
Passage (Q.98-Q.101): Section 17 of the Indian Contract Act, 1872 defines fraud as
an act that has been
committed by a party or his agent, to a contract with the intent to deceive or
induce the other party. Mere silence
or nondisclosure of facts would not constitute a wrongful act unless the
circumstances of the case are such that
the individual has a duty to speak and inform the other party of the facts, but
they remain silent or the party’s
silence is equivalent to expression. The other party to the contract is misled and
suffers damages as a
consequence.
For example, an employer guarantees that the servant is honest and hardworking.
However, he concealed the
fact that the servant is also employed elsewhere and is guilty of acts such as
dishonesty. Here, the past conduct
of a servant is a material fact and should be disclosed to the other employer. An
omission to mention such facts
would imply that such facts do not exist. Therefore, it can be noted that this
contract which has been induced by
non-concealment of material facts is invalid.
Each party to a contract of marriage is bound by a duty to disclose every material
fact. If the accurate facts are
not revealed, the other party can break off the engagement and revoke the contract.
In the case of Anurag Anand
v. Sunita Anand, it was held that caste, income, age, nationality, religion,
educational qualifications, marital
status, family status, financial status, would be considered as material facts and
circumstances. Similarly, under
Section 55(i)(a) of the Transfer of Property Act, 1882, the seller is under an
obligation to reveal to the buyer any
material defect or shortcoming in the property or in the seller’s title of which
the seller is aware.
[Rachit Garg, ‘Does silence amount to fraud’ (December 22, 2021, ipleaders)
<https://blog.ipleaders.in/does#silence-amount-to-fraud/> as accessed on 13
January, 2022]

. Page 27 of 40
98. In which of the following cases, silence amounts to fraud?
(a) A pharmaceutical company deliberately does not disclose to its dealers at the
time of sale that their medicine
is yet to be tested on humans.
(b) When the owner of a car remains silent when asked if the car has ever had an
accident by a prospective
buyer.
(c) When the real estate developer does not disclose to the buyer that the building
has failed to pass the required
safety standards.
(d) All a, b & c
99. Riya and Rakesh’s wedding was celebrated on 12th July 2018 at Jullundur. The
negotiations for this marriage
took place between their parents in the month of April 2018, where Riya’s father
assured Rakesh’s family that
Riya’s character was unblemished. She was educated and was 20 years of age.
Rakesh’s mother also mentioned
that she hoped that Riya had never been in any relationships with other men before
her marriage to which Riya’s
father smiled but did not say anything in reply. Soon after their marriage, Rakesh
came to know from a distant
relation that Riya had an abortion of a child born to her as a result of an illicit
connection with another person
before her marriage with him. He filed a petition to annul the marriage on the
grounds of fraud? Decide.
(a) Riya’s father did not say anything in affirmation as to Riya having no other
relationship with other men
before her marriage with Rakesh and therefore, it does not amount to fraud.
(b) Riya’s father assured Rakesh’s family that Riya’s character was unblemished and
so having an abortion of a
child born out of an illicit connection with another men is opposite of his
statement and therefore amounts
to fraud.
(c) Riya’s father did not conceal any material fact which would have misled
Rakesh’s family and they would
have suffered damages as a consequence. Therefore, it does not amount to fraud.
(d) Rakesh gave his wilful consent to the marriage and did not object to it while
solemnizing the marriage and
therefore there was no fraud done by Riya or her family.
100. With the above context, after 4 years of his marriage with Riya, Rakesh
received an anonymous letter from a
man who contended that Riya is actually 27 years old now and was married to a man
named Jeevan. After a year
of her marriage with Jeevan, she took all the jewellery from the house and ran away
with some other man with
whom she also had a child which she later aborted. The letter also contended that
she had filed a case of domestic
violence against Jeevan which was later found to be a false allegation. Based on
the contents of the letter, Rakesh
filed for divorce from Riya and submitted that the marriage was induced by the
concealment of material facts
and therefore it is invalid and shall be annulled. Decide whether Rakesh is correct
or not?
(a) Yes, Rakesh is correct as any concealment of age or marital status of a party
in a contract of marriage amounts
to concealment of a material fact which would cause damage to the other party in
the contract.
(b) No, Rakesh is incorrect as Riya’s father had himself given assurances to
Rakesh’s family without any inquiry
from their side.
(c) Yes, Rakesh is correct as had he known all the information prior to the
marriage, he would have never done
it in the first place.
(d) No, Rakesh is incorrect as abortion or previous cases filed against Jeevan were
irrelevant and immaterial to
the marriage between Riya and Rakesh, thus, there is no case of fraud.

. Page 28 of 40
101. Anuj was selling their ancestral home in Old Bhopal, to buy a lavish bungalow
in the Arera colony in New
Bhopal. After the house was sold and the title was transferred, a person filed an
application in the court claiming
that he is entitled to a share in the house, and thus title transferred to the
buyer is not absolute. To Anuj’s surprise
this person was his step-brother, from his father’s previous marriage, he had not
known about. The buyer now
seeks to revoke the sale agreement on the grounds of fraud. Choose the correct
answer.
(a) The sale contract shall be annulled because Anuj gave the impression that his
title over the property is
absolute while entering into the contract, which amounts to fraud.
(b) The sale contract shall not be annulled because Anuj had no knowledge about his
title in the house being
defective, and therefore he has not committed any fraud.
(c) The sale contract shall be annulled because it cannot be concluded unless the
stepbrother, who also has title
over the property, agrees to the sale of the house to the buyer.
(d) The sale contract shall not be annulled because the suit as to title over the
property is sub judice, and therefore
shall not be the reason for annulment.
Passage (Q.102-Q.105): In its affidavit filed last year, the Union government urged
the court to dismiss the
petition with costs and claimed that it has adopted a balanced approach on cannabis
by empowering the state
governments to permit, control and regulate the cultivation of any cannabis plant,
production, manufacture,
possession, transport, import inter-state, export inter-state, sale, purchase,
consumption or use of cannabis
(excluding charas) for medical, scientific and industrial purposes subject to the
permission from state
government.
The Union government stated that the present legal framework regulating the usage
of cannabis did not violate
Articles 14 (right to equality), 19(1)(g) [freedom of trade], 21 (right to life) or
other fundamental rights
guaranteed under the Constitution.
“There is no complete ban on cannabis under NDPS Act but can be used for medical,
scientific, industrial,
horticultural purposes by taking requisite permissions from respective state
governments,” the affidavit filed by
the director, narcotics control, department of revenue, Ministry of Finance said.
“The state governments are empowered to license the cultivation of cannabis for
industrial and scientific purpose.
On similar lines, it may be inferred that the cultivation for industrial/
horticultural purposes, as provided in
Section 14 of NDPS Act, can be considered by the state government,” it added.
In its response, the Union government further clarified that under the NDPS Act,
there was a clear distinction
between the various parts of cannabis plant, namely the fibre, flower and the seed
and does not treat all of them
and their derivative equally.
“The seeds and leave when not accompanied by the tops have not been included in the
definition of cannabis.
Bhang which is an edible preparation of cannabis is also not controlled under the
NDPS Act,” it said.
(Source : Cannabis Not Completely Banned, Medical Use Allowed': Centre Tells Delhi
HC, The Wire)
102. Ram and Shyam ran a chemist shop. According to reports, the government has
permitted only registered medical
plants/factory to use cannabis in the production, manufacturing and sale for
medical purposes. Ram and Shyam
created a cannabis solution with a dug Dolo and stated that it was beneficial in
treating a Covid-19 patient. They
were prohibited from selling the substance by police. Can ram and Shyam take the
defense of sale of cannabis
plant for medical purpose.
(a) Yes, they can take the defense as the substance as the government has permitted
to use cannabis in the
manufacturing and manufacture of medical products.
(b) No, they cannot take the defense as ram and Shyam ran a chemist shop nor a
medical plant.
(c) Yes, they can take the defense as the solution containing cannabis is proved to
be effective in treating patients
of covid-19.
(d) No, they cannot take the defense as they Ram and Shyam did not took permission
from the Govt for sale.

. Page 29 of 40
103. Continuing on similar facts, can Ram and Shyam claim for violation of
fundamental rights guaranteed under
Article 19(1)(g) .i.e., freedom of trade for prohibition of selling drug which has
cannabis as an essential
component ?
(a) Yes, they can claim as it violates Article 14.
(b) Yes, they can claim the violation as they were prohibited from selling the
drug.
(c) No, they cannot claim violation of article 19(1)(g) as legal framework
prescribed no such violation.
(d) No, they cannot claim violation of fundamental right as the prohibition on sale
was legal.
104. Rahul and his pal Shyam were in illegal possession of cannabis in an amount
that exceeded the permissible limit
for possession. In order to avoid being caught, Rahul sowed the seeds in one of his
pots outside his residence.
Their Neighbour filed a complaint with the authorities. Rahul and Shyam were both
arrested. Determine the basis
on which the court will levy charges against them.
(a) For keeping in possession illegal quantity of cannabis.
(b) Using it for interests other than medical, scientific, or industrial purposes.
(c) Both A & B
(d) For illegal cultivation of cannabis without license.
105. Rahul, a young scientist and researcher at a private research institute,
discovered the formula for treating Covid
patients. Because his research involves the use of cannabis for scientific
purposes, he presented an application
to the state government for the necessary approval. The state government denied his
application due to a lack of
research. In defiance of the state government's decision, Rahul contracted with
cannabis merchants for the
purposes of his research. When caught, Rahul alleges that he used cannabis for his
scientific research. Decide
(a) Rahul will be held liable as he was not granted requisite permissions from
respective state governments.
(b) Rahul will not be held liable as it was used for scientific and medical
purposes.
(c) Rahul will be held liable as there is complete ban under NDPS act without prior
permission.
(d) Rahul will not be held liable as there is no complete ban on cannabis under
NDPS Act.

. Page 30 of 40
SECTION - D: LOGICAL REASONING
Passage (Q.106-Q.110): After more than a decade’s wait, a museum was set to be
inaugurated in 1921 as a
memorial to the Prince of Wales. Built in the Indo-Saracenic style, the memorial
would have art, archaeology
and science sections, and represent the Bombay Presidency and Sind. It would also
encompass the “Oriental
region”, including Tibet, Yunnan, Syria and Iran.
However, when Edward VIII, the Prince of Wales, landed in Bombay in November 1921,
he was greeted by
non-cooperationists and riots that lasted three days. To the great disappointment
of the museum trustees, the
royal inauguration did not materialise. Then on January 10, 1922, the memorial was
inaugurated by the collector
of Bombay, JP Brander, and the wife of the governor, Lady Lloyd. With no prince in
attendance, the memorial
fulfilled its destiny as the Prince of Wales Museum of Western India.
The museum, renamed Chhatrapati Shivaji Maharaj Vastu Sangrahalaya (CSMVS), Kala
Ghoda, is currently
home to 70,000 objects, 26 galleries, and exhibition spaces that match
international standards — all of which
are managed by a staff of 220. The much sought-after destination attracted a
footfall of 10 lakh annually pre#pandemic — tourists, picnickers, school students,
art historians, numismatics, and zoologists.
It celebrates its centenary this year. In the pandemic, CSMVS was shut for 16
months, and has survived solely
on patronage. It’s a strong reminder of its tumultuous origins. As delayed as its
opening was in 1922, the building
had been operational since it was completed in 1914. Forgoing its intended purpose,
the building first operated
as Lady Hardinge Hospital for Indian soldiers in World War I. Then again, during
the Spanish Flu pandemic
of 1918-20. In World War II, the main building of the otherwise functional museum
was closed for five years to
allow the Red Cross to function.
106. If the statements in the given passage are true, then which of the following
must be true?
(a) Non-cooperation movement started in or before 1921.
(b) People have largely contributed to making the museum.
(c) CSMVS is the first museum to be introduced audio guides.
(d) Prince Edward, the prince of Wales arrived in Bombay after the riots.
107. Which of the following is not true about the museum?
(a) It served as hospital for many injured solider during the times of WWI.
(b) It was inaugurated without the presence of Prince of Wales.
(c) The museum is now known with a different name.
(d) The museum was renamed in 1955.
108. Which one of the following is an assumption on which the argument relies?
(a) The museum has served beyond mere collections of artefacts.
(b) Where there is building, there is a service centre inside.
(c) The museum is now called as Chhatrapati Shivaji Maharaj Vastu Sangrahalaya.
(d) Pandemic has taken away the glory of the museum’s historical relevance.
109. Which of the following does not find support in the passage?
(a) The museum represents the art, technology and science of the times.
(b) The museum served as hospital for many injured solider during the times of WWI.
(c) The museum is now known with a different name.
(d) The museum still supports a hospital facility for various treatments.

. Page 31 of 40
110. Which of the following premise supports delaying the inauguration of the
museum?
(a) The inauguration was kept on hold for more than a decade.
(b) The inauguration was delayed because of the wars and calamities.
(c) When prince of wales came, the hostilities happened.
(d) The building was not completed at the stipulated time.
Passage (Q.111-Q.115): Seven out of 10 adults in the country have received two
doses of the Covid-19 vaccine,
as India completed one year of the world’s largest vaccination drive on Sunday.
While achieving saturation
coverage of the adult population the 15-18 age group continues to be the priority,
the government is looking at
two new engagements in the Covid vaccination drive over the coming weeks, The
Indian Express has learned.
One, exploring further the scientific data on the “mixing” of vaccines and, two,
considering vaccination for
children of age younger than 15. Top government sources said the National Technical
Advisory Group on
Immunization (NTAGI), which provides scientific inputs for the government’s
vaccination policy, will be
examining the advanced clinical trial data on using a vaccine based on a platform
different from the first two
doses, for the third, “precaution” dose in the 60-plus age group. “The primary work
that NTAGI is doing at
present is looking at a heterologous regimen within the existing groups which have
been permitted precaution
doses. Because that is where the clinical trials within the country are at an
advanced stage,” a senior government
official said. “The working group on Covid-19 within NTAGI is in close contact with
the institutions conducting
the heterologous clinical trials. Maybe data would be submitted to the drugs
controller general by the end of
January,” the official said. The sources also said that the government would look
at lowering the age floor for
the vaccination of children once a “substantial part of the 15-18 group” is
covered. “NTAGI has given a general
recommendation on vaccinating children starting from the older ones. They have said
that you can cover in a
staggered manner, top downwards. Within that recommendation, the government took
the decision that first, the
15-18 group would be covered. Once we have vaccinated a substantial part of the 15-
18, we will lower the age
limit,” a top source said.
111. Which of these, is correct according to the passage?
(a) 70 percent of adults are now jabbed with two doses successfully.
(b) There are future plans to vaccinate children younger than 15.
(c) Third dose might be different from the earlier one to create more efficiency.
(d) All of the above
112. What is the central idea of the given excerpt?
(a) Government is aiming to target all groups of people for vaccination and
combination of vaccines as booster
doses.
(b) Government is targeting to jab in different vaccine for booster dose.
(c) Government is looking to be the first country to have a hundred percent
vaccination drive for all age groups.
(d) National Technical Advisory Group on Immunization (NTAGI) provides scientific
inputs for the
government’s vaccination policy.
113. Find which of the following is an assumption for the statement?
STATEMENT: Seven out of 10 adults in the country have received two doses of the
Covid-19 vaccine.
(a) 3 out of 10 are probably not fully vaccinated.
(b) 3 out of 10 have received at least 1 dose.
(c) 3 out of 10 have not received a single dose.
(d) Minimum two vaccines are required against Covid-19.

. Page 32 of 40
114. Which of the following conclusion can be drawn from the passage?
(a) The government of India is expediting the vaccination drive for children in the
age group of 15 to 18 so that
it can start with the under 15 age group.
(b) Once all the people of India are vaccinated, the virus will die a natural death
with each variant becoming
feeble.
(c) 70% adults twin-jabbed, younger kids could be part of the coverage.
(d) The government will go ahead with the vaccination drive for the age group of
under fifteen, when the
National Technical Advisory Group on Immunization (NTAGI) will give a green signal.
115. Which of the following, if true, weakens the author’s argument?
(a) Vaccinating everyone will increase the immunity.
(b) Vaccinating all groups of people will prevent deadly wave of Covid-19 in the
country.
(c) Vaccinated people are a threat and they invite death and increase the mortality
rate in the country.
(d) Vaccinating younger group of people will encourage adults.
Passage (Q.116-Q.120): A crowded three-story ferry carrying around 80 passengers
caught fire on the sugandha
River in southern Bangladesh early Friday, killing at least 40 people and injuring
the remaining, officials said.
The fire broke out around 3 am on Friday in the engine room of the Barguna –bound
MV Abhijan- 10 launch
that started the journey from Dhaka, police and fire service officials said, adding
that scores of other passengers
were missing. Officials familiar with the rescue said 40 people were killed in the
blaze, included nine being
drowned but some private television channels, quoting officials, reported 41
deaths. Officials said they were
carrying out the autopsy of the 30 people at Jhalakathi’s main government hospital
ahead of handing them over
to relatives.
I have assigned an official team to find out the casualty figure, the deputy
commissioner of Barishal district told
the private Samoy TV. "The record suggests 310 listed passengers were travelling in
the ferry, but we assume
the actual number was much higher, junior minister for shipping Khaled Mahmud
Chowdhury told reporters
after he visited the accident site. He said three separate investigations have been
launched to find out the
background and other details of the pre-dawn accident. Doctors at the Sher-e-Bangla
Medical College Hospital
in the neighboring Barishal district said they were currently treating 70 people
while fire service officials said
some 50 more were being treated at other health facilities.
116. Which of the following is most similar to the above-mentioned incident?
(a) A driver driving a car at high speed, resulting in him losing control because
the brake fails and the car collides
with the tree when the driver tries to stop the car.
(b) A cyclist trying to do stunts, leading to the cycle spiraling out of control
and crashing into a tree, a few miles
before the destination.
(c) A train of passengers got derailed leading to the death of many passengers
(d) A speeding bus carrying passengers beyond the capacity overturns and catches
fire leading to several death
and many injured
117. Which of the following can be inferred from the given statements?
I. Accident caused death of all those travelling in the ferry.
II. Ferry had a mechanical malfunction.
III. Weather was horrendous and the sea was not silent that day.
(a) Only I (b) Only II (c) II & III (d) None of the above.
118. Which of the following can be the logical corollary of the given passage?
(a) The cause of the fire was being investigated at the time of reporting the
incident.
(b) Ferry caught fire from a gas leak.
(c) Ferry failed under pressure.
(d) Ferry had a mechanical fault.

. Page 33 of 40
119. Which of the following does not find a support in the passage?
(a) There was an accident due to which 40 or more passengers died.
(b) Ferry was big as it had three stories.
(c) The ferry was carrying passengers more than its capacity.
(d) Ferry caught fire after dawn.
120. ‘I have assigned an official team to find out the casualty figure, the deputy
commissioner of Barishal district told
the private Samoy TV. "The record suggests 310 listed passengers were travelling in
the ferry, but we assume
the actual number was much higher, junior minister for shipping Khaled Mahmud
Chowdhury told reporters
after he visited the accident site.’ Which of the following assumption/ assumptions
is/are valid from the given
options?
I. Deputy commissioner has the authority to assign an official team to investigate
into the cause of the accident.
II. Khaled Mahmud Chowdhury was the junior minister for shipping.
III. The disclosure of the listed passengers may have been tampered with.
(a) Only I (b) Only II. (c) I & III (d) I & II
Passage (Q.121-Q.125): Snakes are elongate in body form and therefore potentially
subject to disturbance to
cardiovascular function when they assume upright or vertical postures. Extant
species of snakes occupy a
diversity of habitats, and several lineages have evolved fully or semi-aquatic
forms while others have become
highly arboreal. From a comparative and phylogenetic perspective, snakes include
forms that routinely
experience gravity stress, such as in climbing (scansorial) species, especially
climbing in trees (arboreal), as well
as others that might be regarded as evolutionarily “deconditioned” to gravity, such
as fully aquatic species. Thus,
within a single taxon that has a body form sensitive to gravity, it is possible to
sample a range of gravitational
adaptations attributable to diversification of habitat demands and behaviors.
A relation between gravity and anatomic position of the heart has been demonstrated
in comparative studies of
cardiovascular function in snakes: the hearts of aquatic species are significantly
closer to the body center than
are those of arboreal and scansorial species, while semi–aquatic and relatively
non–climbing terrestrial species
appear intermediate. At extremes, the heart occupies positions as little as 15% of
the total body length from the
head in some arboreal species, whereas the position is nearly 45% of the total body
length from the head in
aquatic species. The relative heart position is independent of body mass and
length, and there is no apparent
tendency for a reduction of heart–head distance in longer snakes within a habitat
category. In terrestrial snakes,
the increased hydrostatic pressure of the blood column is compensated by increased
systemic arterial blood
pressure that leads to greater heart work; conversely in aquatic species, a heart
position near the body center
potentially reduces the work necessary to perfuse the body, and working against
gravity is not an issue.
The lungs of snakes are elongated structures with a single, large axial chamber in
most species. They are divisible
into anatomically and functionally discrete segments. The respiratory region is
highly vascular and is termed the
vascular lung for purposes of the present discussion. The remainder of the lung,
having lumen continuous with
the vascular segment, virtually lacks respiratory parenchyma and continues for
variable distances as a saccular
lung. While the entire pulmonary structure extends for much of the length of the
body cavity, the relative lengths
of the vascular and saccular segments vary considerably among species. In arboreal
and scansorial species, the
vascular segment has been shown always to be very short, beginning usually near the
heart and extending less
than 20% of the total body length. In some species, the vascular segment is less
than 9% of the total body length.
On the other hand, vascularized parenchyma in lungs of totally aquatic species
extends a much greater length of
the body cavity, and in some species occupies virtually the entire body cavity.
This includes a vascular portion
anterior to the heart, called the tracheal lung. The differences in vascular
segments appear to represent an absolute
constraint on the length of pulmonary vasculature in the scansorial species, for
gravitational pressures develop
in the pulmonary vasculature in proportion to the absolute height of the blood
column and can easily exceed the
threshold for creating serious pulmonary edema. Filtration of fluid into the
reptilian lung has been estimated to
exceed that of mammals by two orders of magnitude, and the leakiness of reptilian
lungs has been characterized

. Page 34 of 40
as a “wet lung” syndrome. When sea snakes are tilted head-up outside of water, the
lung develops severe edema
and even capillary rupture in the dependent segment.
121. The passage primarily aims at studying which of the following relations?
(a) Taxonomy and cardiopulmonary anatomy.
(b) Cardiopulmonary physiology and habitat.
(c) Structure of lungs and topology.
(d) Position of heart and topography.
122. According to the passage, which of the following is likely to be true for the
arboreal species of snakes.
(a) Anterior hearts decrease cardiac work with respect to cranial flow and minimize
drops in cranial pressure
and flow during head-up climbing.
(b) Position of the anterior hearts in arboreal snakes is relative to the total
length of the snake body.
(c) The volume and the hydrostatic pressure of the blood column increases in the
arboreal species which is
compensated by increased systemic arterial blood pressure.
(d) Anterior hearts of the arboreal snake species pulsate at a higher rate in
comparison to the aquatic snake
species.
123. Which of the following is not corroborated by the passage with respect to the
respiratory region of the arboreal
and scansorial species?
(a) Shorter length of the vascular segment in relation to the total body length
helps prevent the risk of pulmonary
edema.
(b) Gravitational pressure develops in proportion to the absolute height of the
blood column which is controlled
by a constraint on the length of pulmonary vasculature.
(c) Saccular lung is not longer in the arboreal and scansorial species in
comparison to the aquatic species.
(d) Arboreal and scansorial species have conditioned themselves to the profound
effects of gravity on body fluid
dynamics.
124. From the passage it can be gathered that the capillary rupture could have been
prevented in a sea snake during
tilted head-up position outside of water if?
(a) Systemic arterial blood pressure would have been greater than atmospheric
pressure.
(b) Heart would have been enclosed in a thoracic cavity as in case of mammals.
(c) Heart would have been placed at the posterior most end of the body.
(d) The vascular segment of respiratory region would have been smaller in
comparison to the saccular lung.
125. The excerpt is likely to have been taken from
(a) A flora and fauna section of a newspaper.
(b) Textbook on reptiles
(c) Journal on cardiopulmonary physiology
(d) Research paper on Evolution of snakes.
Passage (Q.126-Q.130): Dozens of Afghan women from all walks of life held a protest
in Kabul demanding the
Taliban to provide government jobs and equal representation in society, the media
reported. Some women who
participated in a protest were former employees of the Independent Administrative
Reform and Civil Service
Commission, who lost their jobs following the Taliban takeover in August 2021, TOLO
News reported. One
such participant, Ferozan Amiri, said: "We the women working in the Independent
Administrative Reform and
Civil Service Commission, and the 28 percent active force of the government
department, have suffered major
damage with the fall of the Islamic Republic of Afghanistan and the Taliban's
ruling on government departments
on August 15, 2021."
The protesters also released a resolution which comprised five points -- allowing
females to work; women's
meaningful inclusion in the government's decision-making body about women's
affairs; formation of policies to

. Page 35 of 40
support women's rights; preservation of posts which were occupied by women; and the
creation of a safe
environment for women. "Women are concerned over the fate of jobs in the
(Independent Administrative Reform
and Civil Service Commission). You know, this institution is neutral. Around 28 per
cent of the civil service is
filled by women," TOLO News quoted another protester, Khujusta Ilham, as saying.
Meanwhile, Taliban officials have said that there has been no decision to exclude
women from the government
structure. "The issue of women's working activity in (government) departments is
under discussion. After an
assessment, if the women's presence is needed in any department, they will be
working in the same posts and the
same departments," said Bilal Karimi, a deputy spokesman for Taliban. To consider
the recognition of the current
Afghan government, the international community requires the observation of human
rights, women rights, and
the formation of an inclusive government.
126. What can be ideal theme for the passage?
(a) Taliban diminished the rights to women.
(b) Women want their jobs back.
(c) To protect themselves from authoritative government, women are protesting.
(d) Both B and C.
127. What does the Author indicate when he quotes “To consider the recognition of
the current Afghan government,
the international community requires the observation of human rights, women rights,
and the formation of an
inclusive government”?
(a) International community will attack Taliban soon.
(b) Afghan women should reach out to international community.
(c) US is the chief head of the community.
(d) International community is doing whatever it can, to help women in Afghanistan.
128. Choose the best conclusion for the given passage.
(a) Women protest is the result of the Taliban acquisition over Afghanistan.
(b) Afghan Women’s rights should be restored.
(c) Afghan Women must be treated equally and they should be allowed to work.
(d) All of the above.
129. Which one of the following can be inferred from the passage?
I. Women’s rights were demolished when Taliban conquered Afghanistan.
II. The situation of women in Afghanistan will get better following the protest.
(a) (I) is an implicit assumption.
(b) (II) is an implicit assumption.
(c) Both (I) & (II) are implicit assumptions.
(d) Neither (I) nor (II) is an implicit assumption.
130. Which of the following is an implicit assumption, according to the third
paragraph?
(a) Taliban is on talks to restoring women’s posts.
(b) Taliban may add female member in talks and discussion when it will be required.
(c) International community is watching the situation in Afghanistan.
(d) All of the above.
131. K's father G is A's brother. M, who is A's sister-in-law, is R's daughter. T
has two sons only. K is granddaughter
of R. A, who is a male, has only one sibling. How is M related to T?
(a) Sister (b) Niece (c) Granddaughter (d) Daughter-in-law

. Page 36 of 40
Passage (Q.132-Q.134): These questions are based on the following information.
Four models of cars – Honda, Ford, Toyota and Hyundai – were tested on two
parameters – acceleration and
mileage. The cars were ranked from 1 to 4 on both parameters such that the one with
the highest value in each
parameter is ranked the first, the second highest is ranked the second, and so on.
Further, no car got the same
rank in both the parameters. Neither Honda nor Ford were the first or the last in
either of the parameters. Ford
has lesser mileage than only Hyundai.
132. Which car has the highest acceleration?
(a) Toyota (b) Honda (c) Ford (d) Hyundai
133. How many of the given cars have a greater acceleration than Ford?
(a) 0 (b) 1 (c) 2 (d) 3
134. Which car has both better mileage and better acceleration than Honda?
(a) Toyota (b) Ford (c) Hyundai (d) None of these
135. All pink is red. Apple is red. In logical language, therefore:
(a) Apple is pink (b) Apple is not pink
(c) Pink can be apple (d) Apply may or may not be pink

mock 30

Directions(Q.1-Q.30): Read the following passage carefully and answer the questions
that follow.
Passage(Q.1-Q.5): Commuters driving on Mumbai roads experience potholes via the
rattle and shake of their
bones. The Indian economy’s unevenness is a similarly bumpy ride endured, despite
multiple assertions that the
worst is over and a glorious future lies ahead. There are a lot of hopes riding on
the Budget but the journey
promises to get bumpier. So, better strap up.
The Indian economy is entering a crucial stretch and how it behaves in the coming
months will depend on the
government’s annual Budget exercise that is just around the bend. The political
economy, as usual, will have an
outsized influence on the Budget. India is mid-way between two general elections –
the last one was in 2019.
In the next few weeks, crucial polls are due in Uttar Pradesh, Punjab, Goa,
Uttarakhand and Manipur, with about
690 seats in the balance. Next year, it will be the turn of Gujarat, Karnataka,
Meghalaya, Mizoram, Nagaland,
Himachal Pradesh and Tripura to vote, with an almost equal number of seats in
contention. And then 2024 is the
big gig.
The Bharatiya Janata Party needs to win as many state elections as possible – its
reputation hinges on victories
in Uttar Pradesh and Punjab. The BJP’s poor run over the past two years, especially
the humiliation in West
Bengal and Tamil Nadu, has left a visible dent in its image as an infallible
election-winner.
Unarguably, the Budget (to be announced on February 1) will be Finance Minister
Nirmala Sitharaman’s
opportunity to play Santa, handing out giveaways and free passes to crucial voter
blocs. It might then make sense
to assess the state of the economy and figure out where the Budget could be headed.
Spoiler alert: lots of bumps
ahead.
The first stop is the statistical ministry’s advance estimate of the full year’s
(India’s fiscal year runs April to
March) gross domestic product or GDP. Multiple skeins are intertwined here.
India’s real GDP (adjusted for inflation) for 2021-22 is likely to grow by 9.2%
over the previous year. Various
components from the demand side of the equation – private consumption, government
consumption, or capital
expenditure – have shown the first signs of a hesitant recovery. But keep that
party on hold.
Remember this is only an “advance estimate”, or a guesstimate about the March 2022
picture based on economic
activity till November-December. It will be updated once again by end of January.
The real picture will emerge
only on May 31. There are other reasons to hold off premature celebrations.
1. Which of the following literary device has been used in the first paragraph of
the passage?
(a) Allegory (b) Allusion (c) Alliteration (d) Metaphors
2. Select the meaning of the word ‘skeins’.
(a) A length of thread or yarn, loosely coiled and knotted.
(b) The bottom part of a tree left projecting from the ground after most of the
trunk has fallen or been cut down.
(c) Cause (someone) to feel confused because they cannot understand something.
(d) A flock of wild geese or swans in a flight.
3. Select the option which can be inferred from the passage
(a) The seats in Gujarat, Karnataka, Meghalaya, Mizoram, Nagaland, Himachal Pradesh
and Tripura are a little
less than Uttar Pradesh, Punjab, Goa, Uttarakhand and Manipur.
(b) The seats in Gujarat, Karnataka, Meghalaya, Mizoram, Nagaland, Himachal Pradesh
and Tripura exactly
equal to Uttar Pradesh, Punjab, Goa, Uttarakhand and Manipur.
(c) The seats in Gujarat, Karnataka, Meghalaya, Mizoram, Nagaland, Himachal Pradesh
and Tripura are more
than Uttar Pradesh, Punjab together.
(d) The seats in Gujarat, Karnataka, Meghalaya, Mizoram, Nagaland, Himachal Pradesh
and Tripura are
extremely less when compared to Uttar Pradesh, Punjab, Goa, Uttarakhand and
Manipur.

. Page 3 of 40
4. Which of the following can be inferred from the passage?
(a) BJP had the image of an infallible political party which was tarred after
losing West Bengal and Tamil Nadu
in elections.
(b) BJP still has the image of an infallible political party despite losing West
Bengal and Tamil Nadu in elections.
(c) BJP never had the image of an infallible political party from the time it first
one the election with a huge
margin.
(d) BJP has completely lost its image of being a weak party after losing West
Bengal and Tamil Nadu.
5. Select the option which is the characteristic of the word, ‘guesstimate’
(a) Guestimate is a portmanteau. (b) Guestimate is a formal word.
(c) Guestimate is a slang. (d) Guestimate is a fragmented word.
Passage(Q.6-Q.10): As we write this, the light is failing and the zombies, those
strange monsters recently
released into our midst, are roaming the fair streets of our city. They appear each
morning and again at each
day’s end, carrying briefcases and book bags, riding bicycles, walking, or, most
ominously, taking the local
trains. It is up to us to discover their origins and the best means for dispatching
them. This Zombie Apocalypse,
as those who read the papers already know, has now reached dangerous proportions.
Some have referred to its
creeping nature as “insidious” or “spooky.”
Yes, Dear Reader. You already know which members of our society we speak of, and
the knowledge may send
a ripple of fear shivering down your spine. The Zombie Lawyer Apocalypse has
targeted those who most want
to engage our business, political, and public sectors. We are talking, of course,
about law students and lawyers.
Zombies, driven by achievement, depressed by environment, lost, alone, and angry,
are rampant in the legal
profession and in law schools. They roam the halls of their employers and their law
schools, wreaking havoc on
all of those who meet them, threatening to spread their virus and to eat the brains
of the living. The legal
profession, with its rich history of both civic and business engagement, its public
service mission, and its focus
on human interactions and human institutions, is uniquely situated to embrace the
lessons of the Zombie Lawyer
Apocalypse and to intervene in the spread of the disease and other factors that
cause legal zombies. As an
institution, we are in a position to produce, not zombies, but fully realized
beings who care about accomplishment
and relationships, about meaning and not only money, and about themselves, their
peers, and the broader world
around them.
Martin Seligman writes, “If you want well-being, you will not get it if you care
only about accomplishment. If
we want our students to flourish, we must teach that the positive corporation and
the individuals therein must
cultivate meaning, engagement, positive emotion, and positive relations as well as
tending to profit.”
If we are to stem the tide of law student and lawyer depression and emerge as
victors in the battle of the Zombie
Lawyer Apocalypse, we must take these words to heart and begin to change the way we
engage with one another
and with our institutions. We must move away from a culture of dehumanizing
competition and away from the
notion that the legal discipline is a purely instrumental one, devoid of human
emotion, engagement, and ideas.
Instead, we must embrace the tenets of mindfulness, ethical decision-making, and
positive psychology in order
to build a profession and professional education system that encourages individuals
and organizations to flourish.
For, as Seligman tells us, “when individuals flourish, health, productivity, and
peace follow.” If we can shift the
culture of the legal profession and the legal academy and begin to work toward
flourishing, then we can end the
Zombie Lawyer Apocalypse and breathe life back into those who have been harmed by
the profession?
6. What is the synonym of word INSIDIOUS used in the passage
(a) Honest (b) Forthright (c) subtle (d) None of these

. Page 4 of 40
7. Choose the appropriate title for the passage-
(a) The problem without solution (b) The Zombie Lawyer Apocalypse
(c) How a zombie turns into a lawyer. (d) Reasons why not to go in the legal field.

8. Which of the following will be the result of us altering our interactions with
one another and our institutions?
(a) We will stop producing zombies that failure in all spheres of life.
(b) We will reduce the number of law students and lawyers who are depressed.
(c) We will remove all the lawyers and law students from the institutions that are
zombies.
(d) It will firmly establish the root of legal discipline as an insensitive field.
9. Why has Martin Seligman been quoted by the author –
(a) Martin Seligman is a renowned jurist. (b) Martin Seligman is his teacher.
(c) Martin Seligman is his best friend. (d) Cannot be determined.

10. Determine the tone of the passage.


(a) Sceptic (b) Jubilant (c) Awed (d) Prescriptive
Passage(Q.11-Q.15): What do you think of when you hear the phrase “when pigs fly"?
Do funny images of pigs
with wings fill your mind? This common saying is used to express the idea that
something happening is highly
unlikely or even impossible.
For example, if you ask your parents when they're going to buy you a brand-new
Ferrari, the answer might very
well be: “When pigs fly!" You can spend a lot of your time scanning the skies for
flying pork, but your chances
of seeing a flying pig are about the same as seeing a shiny new red Ferrari in the
driveway.
So why pigs? And why flying? Why couldn't people just say, “Never. Not a chance!"?
People have used colourful phrases for centuries to get a particular message
across.
These phrases convey a sense of humour and allow people to put a positive spin on
things without seeming so
negative. Instead of simply saying “no," someone can communicate the same message
in a way that makes it
sound — technically — possible.
After all, couldn't pigs fly one day? Granted, their weight and lack of wings weigh
heavily against their chances
of taking flight. Plus, there's the whole question of desire. Do pigs have the urge
to fly? Without that internal
motivation, the seemingly-impossible will probably never become possible.
No one is certain exactly who developed the phrase “when pigs fly." An old
reference to pigs flying appears in
Lewis Carroll's Alice's Adventures in Wonderland. However, scholars believe Carroll
may have picked up the
phrase from the Scottish, who allegedly had been using the phrase for a couple
hundred years.
They may be right. John Withals' 1616 English-Latin dictionary lists the proverb
“pigs fly with their tails
forward." That similar phrase was apparently used back then as a clever reply to
people who made overly#optimistic statements.
There are also those who believe the phrase may have gotten its start in America.
Long ago, before the Industrial
Revolution, farming was the primary occupation for most Americans. It's not
surprising that many common
sayings that developed back then had an animal or farm background.
For example, long ago, farmers used to transport pigs for slaughter along rivers on
small barges. If it was foggy,
people on shore could only see the pigs' heads above the fog. This made them look
like they were flying through
the clouds. This might have given rise to the popular saying.

. Page 5 of 40
Other countries also use animals (1) ______ figures of speech (2) ________
represent the impossible. For
example, many countries use the phrase “when cows fly." Other countries, such as
France and Spain, use phrases
like “when frogs grow hair" or “when hens grow teeth."
11. Why do the phrases that have a pre-industrial origin, have animals as one of
their common themes?
(a) Because animals were a major source of entertainment back then.
(b) Because animals were considered as sacred back then.
(c) Because farming was the principal source of income back then.
(d) None of these
12. ‘Other countries also use animals (1) ______ figures of speech (2) ________
represent the impossible. For
example, many countries use the phrase “when cows fly." Other countries, such as
France and Spain, use phrases
like “when frogs grow hair" or “when hens grow teeth." Fill in blanks (1) and (2)
with appropriate prepositions.
(a) for, to (b) In, to (c) To, of (d) of, on

13. The origin of the phrase, when the pigs fly, can be traced back to (as used by
Lewis) –
(a) Scotland (b) Australia (c) America (d) Cannot be determined.

14. Give one word for – the people who make overly-optimistic statements –
(a) Panglossian (b) Gullible (c) Thespian (d) Naïve

15. Choose the appropriate figure of speech that applies to the underlined line in
the passage.
(a) Litotes (b) Oxymoron (c) Euphemism (d) Pun
Passage(Q.16-Q.20): Klara’s boss introduced the hybrid policy in September 2021,
when UK government
guidance recommending home working came to an end: Tuesdays and Thursdays would be
home-working days,
with the remainder of the week spent in the office during normal contracted hours.
“Having a permanent hybrid
set-up initially came as a relief," says Klara. "After years of full-time office
work, it felt like I finally had control
over my work schedule and busy home life.” As the months rolled by, however, the
novelty of hybrid work
soon gave way to hassle and a jarring one-day-in, one-day-out routine. “I feel
settled and focused on the days
that I work from home,” says Klara. “But by the evening I dread having to go back
in: sitting at my desk for
eight hours a day in a noisy office, staring at a screen, readjusting to exactly
how it was before Covid.”
Another employee, Justina feels she now has two workplaces to maintain – one in the
office and one at home.
“It involves planning and a stop-start routine: taking my laptop to and from the
office every day, and
remembering what important things I’ve left where,” she adds. “It’s the
psychological shift – the change of
setting every day – that’s so tiring; this constant feeling of never being settled,
stressed and my productive home
working always being disrupted.” In a recent global study by employee engagement
platform Tinypulse, more
than 80% of people leaders reported that such a set-up was exhausting for
employees. Workers, too, reported
hybrid was more emotionally taxing than fully remote arrangements – and,
concerningly, even full-time office#based work.
In theory, hybrid offers the best deal for both employer and employee. It combines
pre-Covid-19 patterns of
office-based working with remote days, in a working schedule that would allow both
in-person collaboration
and team building, as well as greater flexibility and the opportunity for focused
work at home. It seemed a win#win for workers; in one May 2021 study, 83% said they
wanted to go hybrid after the pandemic. “There was a
feeling that hybrid would be the best of both worlds,” says Elora Voyles, an
industrial organisational psychologist
and people scientist at Tinypulse, based in California. “For bosses, it means they
retain a sense of control and
that they can see their workers in person. For employees, it offers more
flexibility than full-time in the office and
means they can work safely during the pandemic.”
A familiar routine can act as a well-worn groove that allows flow, but carving out
new daily habits – involving
a less consistent schedule between workplaces – can chip away at cognitive
resources. “Moving to hybrid has

. Page 6 of 40
the potential to disrupt someone’s home-working routine,” explains Gail Kinman, a
chartered psychologist and
fellow of the British Psychological Society. “Hybrid practises haven’t become
second nature yet, so it takes
greater energy, organisation and planning. You have to form new strategies – hot
desking, planning commutes
– that you wouldn't need if you were fully remote or in-person.”
16. Why is the hybrid working model is referred to as a win-win set-up for both the
employer and the employees?
(a) Hybrid model lets the company cut down variable costs like electricity and
employees are also given freedom
of work from home.
(b) Studies have shown that productivity increases when companies follow the hybrid
model and in the long
run, it is beneficial for both the company and the employees.
(c) On one hand, in-person cooperation and team development can be achieved and on
the other hand, increased
flexibility and the ability to focus upon work at home are available.
(d) The win-win situation is just a facade as no good can come out of the hybrid
model for both the company
and the employees.
17. Choose the correct antonym for the word 'dread'
(a) Fear
(b) Trepidation
(c) Agitation
(d) Confidence
18. Even though Klara found hybrid work to be exciting at first, why is it that the
excitement of hybrid employment
gradually gave way to annoyance?
(a) She enjoyed working from home as she could attend to her chores, but the hybrid
model coerced her to drop
the idea as it became compulsory for her to attend office on some days.
(b) She felt that she had two workplaces to attend to; the first being her home and
the second being her office.
(c) Both (a) and (b)
(d) The constant shifting between working from home and working from the office was
disastrous for her as
working from home was relaxing and she could concentrate, but the next day, she had
to sit at the desk and
readjust.
19. Which of the following reflects the tone of the passage?
(a) Satirical (b) Populist (c) Speculative (d) Vituperative
20. How can we make hybrid work a success in today’s world?
(a) By following the UK government’s instruction which states that Tuesdays and
Thursdays would be home#working days, with the balance of the week spent in the
office during standard contractual hours.
(b) By maintaining two workplaces, one in the office and one at home.
(c) By creating a comfortable atmosphere at the office, which can be achieved by
arranging home-like structural
integration in cabins, providing sleeping pods, sofas, time-friendly policies, etc.
(d) By making more effort, organizing, preparing and trying to develop new methods,
such as hot desking and
commute planning.

. Page 7 of 40
Passage(Q.21-Q.25): Read the passage carefully and answer the questions.
In the next week, Great Britain’s energy regulator will announce the steepest rise
ever in its energy price cap,
effectively saddling millions of households with an annual energy bill of close to
£2,000. The blow to household
finances follows almost six months of record high energy market prices because of
the global gas crisis. Despite
the deepening gloom facing bill payers, ministers are yet to agree on a package of
measures to prevent a national
energy crisis. After a fourfold surge in energy market prices across Europe,
households will pay an average of
54% more for energy this year than in 2020. The bank warned that the biggest
increases would be felt by Italy
and the UK.
While European governments have moved to protect households from the full brunt of
the global energy crisis,
the UK government has remained silent. The UK’s failure to act comes despite
desperate calls from groups
representing vulnerable households, small businesses and economists, which fear
that record-high energy bills
threaten to unlock economy-wide inflation and a cost-of-living crisis. In Great
Britain, the next energy price
increase is scheduled to be announced on 7 February, and households will be hoping
that new support measures
can be agreed upon before it takes effect from 1 April. Potential measures include
cutting the 5% rate of VAT
on energy bills, or moving the policy costs levied on energy bills to general
taxation. More radical ideas include
setting up a “stabilisation mechanism” to give top-up payments to energy suppliers
depending on the market
price. The Treasury is expected to remain tight-lipped on whether households can
expect a reprieve until its
spring statement in late March.
The Netherlands, like the UK, is highly reliant on gas for electricity generation
and home heating. Unlike the
UK, its government took action within weeks of energy markets reaching record highs
to provide a multibillion#euro package of measures to protect households and small
businesses. The cabinet agreed in October to cut
energy taxes to save households an average of €400 a year. In addition, some €150m
is being set aside to boost
home insulation. A further €500m will be used to compensate small firms in the form
of lower energy taxes.
The French government also wasted no time in agreeing with a package of measures to
soften the blow of rising
energy bills, after an increase in fuel duty in 2018 that triggered a widespread
outcry over the cost of living and
led to protests by the gilets Jaunes (yellow vests). The French government has
already cut some electricity taxes
to help slow the rise in home energy bills at an estimated cost to the state of
€8bn. It will also use its powers to
squeeze the state-owned electricity company EDF to lower the cost of electricity by
charging well below the
market rate for the electricity it generates.
21. Give an appropriate title to the passage.
(a) How to tackle the energy crisis in today’s world?
(b) The detrimental energy crisis that looms over the European Countries.
(c) UK’s systematic approach to solve the domestic energy crisis.
(d) Tackling of energy crisis by UK and other European countries.
22. Which one of the following outcomes was feared by the people of the UK that
would be caused by the energy
crisis?
(a) Higher cost of living and inflation.
(b) 5% rate of VAT on energy bills.
(c) Unemployment and shutting down of various businesses.
(d) Transferring policy expenses from energy bills to general taxes.
23. Which one of the following statements is INCORRECT in context to the passage?
(a) If compared to the Netherlands and France, the approach taken by the UK to
solve the energy crisis is almost
similar to them.
(b) The stabilization system adopted by the UK included grant top-up payments to
energy suppliers based on
market prices.
(c) Energy taxes were cut down in the Netherlands, which was beneficial to the
homeowners of the country.
(d) After a rise in fuel duty in 2018, the French government immediately decided on
a package of measures to
cushion the impact of rising energy bills.

. Page 8 of 40
24. What is the tone/attitude of the author in the passage?
(a) Sarcastic (b) Vitriolic (c) Analytical (d) Obsequious
25. How will the household finances be affected due to the crisis of gas in Great
Britain?
(a) Companies, big and small, will run out of resources and instigate huge-scale
unemployment.
(b) Households would spend fifty-four per cent more on energy this year than in
2020 which will adversely
affect the household finances.
(c) Rise in gas prices will hamper the vehicle industry, leading to the economy
getting affected.
(d) Nothing would be affected as there is no relation between the energy crisis and
the household finances.
Passage(Q.26-Q.30): Read the passage carefully and answer the questions.
Why is my awareness here, while yours is over there? Why is the universe split in
two for each of us, into a
subject and an infinity of objects? How is each of us our own centre of experience,
receiving information about
the rest of the world out there? Why are some things conscious and others
apparently not?
The mind-body problem enjoyed a major rebranding over the last two decades. Now
it’s generally known as the
“hard problem” of consciousness, after philosopher David Chalmers coined this term
in a now classic paper and
further explored it in his 1996 book, “The Conscious Mind: In Search of a
Fundamental Theory.” Chalmers
thought the mind-body problem should be called “hard” in comparison to what he
called the “easy” problems of
neuroscience: How do neurons and the brain work at the physical level? Of course,
they’re not actually easy at
all. But his point was that they’re relatively easy compared to the truly difficult
problem of explaining how
consciousness relates to matter.
Over the last decade, my colleague, University of California, Santa Barbara
psychology professor Jonathan
Schooler and I have developed what we call a “resonance theory of consciousness.”
We suggest that resonance
– another word for synchronized vibrations – is at the heart of not only human
consciousness but also animal
consciousness and of physical reality more generally. It sounds like something the
hippies might have dreamed
up – it’s all vibrations – but stick with me. All things in our universe are
constantly in motion, vibrating. Even
objects that appear to be stationary are in fact vibrating, oscillating,
resonating, at various frequencies. Resonance
is a type of motion, characterized by oscillation between two states. And
ultimately all matter is just vibrations
of various underlying fields. As such, at every scale, all of nature vibrates.
Something interesting happens when different vibrating things come together: They
will often start, after a little
while, to vibrate together at the same frequency. They “sync up,” sometimes in ways
that can seem mysterious.
This is described as the phenomenon of spontaneous self-organization. Mathematician
Steven Strogatz provides
various examples to illustrate this “sync” – his term for resonance – in his 2003
book “Sync: How Order Emerges
from Chaos in the Universe, Nature, and Daily Life,” including when fireflies of
certain species come together
in large gatherings, they start flashing in sync, in ways that can still seem a
little mystifying. Based on the
observed behaviour of the entities that surround us, from electrons to atoms to
molecules, to bacteria to mice,
bats, rats, and on, we suggest that all things may be viewed as at least a little
conscious. This sounds strange at
first, but “panpsychism” – the view that all matter has some associated
consciousness – is an increasingly
accepted position with respect to the nature of consciousness. The panpsychist
argues that consciousness did not
emerge at some point during evolution. Rather, it’s always associated with matter
and vice versa – they’re two
sides of the same coin.

. Page 9 of 40
26. Why did the adjective ‘hard’ come into the mind of Chalmers when he was
referring to the mind-body problem?
(a) The concept of mind-body problems or rather the relation between consciousness
and matter is so complex
that scientists, as well as researchers hardly want to explore its depth as the
theories are dependent upon
one’s subjective interpretation.
(b) If compared to the complications of neuroscience that basically deal with the
working of the brain and
neurons at the physical level, it is really arduous to explain the relationship
between consciousness and
matter.
(c) It doesn’t go with the resonance theory of consciousness and thus, it is
absolutely hard to explain the
underlying principles of the mind-body problem.
(d) If put in comparison to the complications of neuroscience that basically deal
with the working of the brain
and neurons at the physical level, it is really easy to explain the relationship
between consciousness and
matter.
27. Which one of the following statements does NOT associate itself with the
resonance theory of consciousness?
(a) This theory states that if we let positive vibrations into our body, the body
shall react positively and the
opposite will happen in case we let the negative vibrations affect our body.
(b) Resonance, in general, lies at the centre of human as well as animal awareness,
and tangible reality.
(c) Even objects that we view as fixed actually vibrate, oscillate, and resonate at
different frequencies.
(d) This concept of resonance theory of consciousness may have been dreamt (in
their conscience) by the hippies
as they believe that it’s all about the vibrations.
28. What is the tone of the author in the passage?
(a) Belligerent (b) Condescending
(c) Thought-provoking (d) Dogmatic
29. Which one can be identified as the main reason behind the mystery of the
certain kinds of fireflies that congregate
in big groups and begin flashing in unison?
(a) This phenomenon is still a mystery and no valid theory that explains its
occurrence has ever been presented
till today.
(b) The fireflies consciously congregate in big groups and flash in unison as they
are controlled by factors such
as the weather and the rotation of the earth.
(c) The occurrence of planned self-organization which states that when vibrating
objects come together through
planning, they typically begin to vibrate at the same frequency after a short time.
(d) The phenomenon of sync on how order emerges from chaos in the Universe, nature,
and daily life, which
states that when vibrating objects come together, they typically begin to vibrate
at the same frequency after
a short time.
30. Which of the following is INCORRECT regarding the theory of ‘panpsychism’?
(a) It concurs that awareness did not develop at any time throughout evolution; it
is actually constantly linked
to matter and the matter is linked to awareness.
(b) Panpsychism supports the idea that everything, from atoms to humans, is at
least a little aware.
(c) The panpsychist argues that consciousness did emerge at some point during
evolution. Rather, it’s always
associated with matter and vice versa – they’re two sides of the same coin.
(d) Panpsychism is becoming a more widely recognised viewpoint in accordance with
the nature of awareness
Directions (Q.66 – Q.105): Read the comprehensions carefully and answer the
questions based on it.
Passage (Q.66-Q.70): Court's observations
Responding to the contention of DGP Haryana that no such law exists to bring
interrogation rooms under the
CCTV Coverage, the Court, referring to the SC's order, observed thus: "As per
Article 142 of the Constitution
of India, the Supreme Court, in the exercise of its jurisdiction, may pass such
decree or order as is necessary for
doing complete justice in any cause or matter pending before it, and any order or
decree so passed would be
enforceable throughout the territory of India. Further, the law declared by the
Supreme Court would be binding
on all courts as per Article 141.
In this regard, the Court said that as per the SC's directions, interrogation rooms
are to be covered under CCTV
surveillance, and therefore the DGP, Haryana, the DGP, Punjab, as also the DGP,
U.T., Chandigarh, have been
directed to file affidavits as to whether the aforesaid directions of the Supreme
Court have been complied with
or not. If not complied DGP will be responsible for not complying with the
guidelines.
"Consequently, and obviously, non-compliance of the directions issued by the
Supreme Court in Paramvir Singh
Saini's' case, would amount to contempt of Court and this court would, naturally,
also be bound to ensure that
the directions issued by the Supreme Court are actually carried out at ground level
by the States and Union
Territory falling within the jurisdiction of this court. Hence, the aforesaid
direction to the DGPs," the Court
further added. Lastly and importantly, the Court also directed that not just in the
case of the present petitioner,
but in the case of every person who is in police custody or is being taken into
police custody, all provisions of
the Cr.P.C., including Section 41-B, 41-C, 41-D and 54, 55 and 55-A would be
meticulously followed, with
compliance reports in that regard to be made a part of the report under Section 173
of the Cr.P.C., as regards
even medical examination necessary to be conducted in terms of Section 55-A
thereof.
(Source: Entire Police Station Including Interrogation Rooms Need to Be Under CCTV
Coverage As Per SC's
Order: High Court To Punja HC, LIVELAW)
66. While analyzing Section 497 of the IPC, which gives a husband the exclusive
right to prosecute the person with
whom the wife committed adultery by engaging in sexual intercourse with him by an
order, the Supreme Court
issued a decree decriminalizing adultery as an offence under Section 497 of the IPC
for doing complete justice
in a matter pending before it. Sahil questioned the Supreme Court's ruling,
claiming that no such law exists that
may decriminalize an offence under the IPC. Decide
(a) In the exercise of its jurisdiction, the Supreme Court, under Article 142 of
the Indian Constitution, may pass
such decree or order as is necessary for doing complete justice in any cause or
matter pending before it, and
any order or decree so passed would be enforceable throughout the territory of
India.
(b) In exercise of its original jurisdiction, the Supreme Court, under Article 141
of the Indian Constitution, may
pass such decree or order as is necessary for doing complete justice in any cause
or matter pending before it,
and any order or decree so passed would be enforceable throughout the territory of
India.
(c) In the exercise of its jurisdiction, the Supreme Court, under Article 142 of
the Indian Constitution, may or
may not pass such decree or order as is necessary for doing complete justice in any
cause or matter pending
before it, and any order or decree so passed would be enforceable only in part of
the state mentioned in the
said decree.
(d) In the exercise of its jurisdiction, the Supreme Court, under Article 141 of
the Indian Constitution, may pass
such decree or order as is necessary for doing complete justice in any cause or
matter pending before it, and
any order or decree so passed would be binding on all courts.

. Page 17 of 40
67. As per the direction of court, interrogation rooms are to be covered under CCTV
surveillance, and therefore the
DGP, Madhya Pradesh, the DGP, Chhattisgarh, as also the DGP, of Gujrat, have been
directed by Supreme Court
to file affidavits as to whether the aforesaid directions of the Supreme Court have
been complied with or not.
following the guidelines passed by the court, all the police station in the said
states are now under CCTV
surveillance but the person concerned failed to file affidavits as to whether the
aforesaid directions of the
Supreme Court have been complied with or not. Decide
(a) It will amount to violation of Article 141 of the Constitution of India.
(b) It will amount to violation of Article 142 of the Constitution of India.
(c) It will amount to contempt of Court.
(d) It will not amount to contempt of Court as directions of the Supreme Court have
been complied with.
68. Suresh was arrested by police on ground of sedition. As the charges were
serious, he was detained for a period
of 70 days, which is more than a period prescribed for detention under police
custody. Suresh was beaten black
blue in a room with CCTV surveillance. Also, when presented before the magistrate
police officials failed to
provide medical examination reports of the accused as they failed to conduct a
medical examination of the
accused. Suresh's wife can file a complaint against the police officials?
(a) Contempt of court for non-compliance of the directions issued by the Supreme
Court.
(b) Failing to conduct medical examination necessary to be conducted in terms of
Section 55-A.
(c) Failing to produce compliance reports in that regard to be made a part of the
report under Section 173.
(d) For detaining Suresh beyond the prescribed limit.
69. Dinesh and his father were arrested by police for failing to follow state
guidelines given in the aftermath of
Covid-19. The accused were arrested and detained by police officers, and before
they could be produced before
a magistrate, police reported them dead. When officials were approached for CCTV
footage of the interrogation,
they refused, claiming that there was no electricity at the police station that
night. An investigation by a special
team revealed that there were no CCTV cameras placed in any area of the police
station. Decide who will be
held liable for not complying with the guidelines issued by SC.
(a) Police officials who arrested the accused will be liable for contempt of court.
(b) DGP of the said jurisdiction will be liable for contempt of court.
(c) Both police and DGP will be liable for contempt of court.
(d) No one is liable for contempt of court as the reason for death of accused is
not known yet.
70. ASSERTION: The Court said that as per the SC's directions, interrogation rooms
are to be covered under CCTV
surveillance, and therefore the DGP, Haryana, the DGP, Punjab, as also the DGP,
U.T., Chandigarh, have been
directed to file affidavits as to whether the aforesaid directions of the Supreme
Court have been complied with
or not.
REASON- As per Article 141 the law declared by the Supreme Court would be binding
on all courts.
(a) Both A and R are true and R is correct explanation of A.
(b) Both A and R are true but R is not correct explanation of A
(c) A is true but R is false.
(d) A is false but R is true.
Passage (Q.71-Q.75): The Indian Penal Code (IPC) has various basic defences that
exonerate criminal liability,
based on the concept that even if a person commits an offence, he cannot be held
accountable. This is because
the person's conduct was justified at the time of the offence, or there was a lack
of mens rea. Exceptions such as
Mistake of Fact and Accident, among others, are possible when a person is
misinformed about the presence of
certain facts and the conduct is performed without criminal intent.
Mistake of Fact: This exemption applies where an accused has misinterpreted a fact
that eliminates a criminal
element. This legal weapon can be utilised if the accused is able to demonstrate
that he or she was mistaken
about the presence of certain facts or was unaware of the existence of such facts.
However, the said mistake must
be of fact rather than of law. In same spirit, Section 76 of the IPC states “act
done by a person bound, or

. Page 18 of 40
mistakenly believing himself bound, by law”- Nothing constitutes an offence if it
is committed by a person who
is, or thinks himself to be, obligated by law to do it due to a mistake of fact
rather than a mistake of law.
Accident: Using this defence, a person might avoid criminal liability when their
actions are the result of an
accident. Such an act must be unintentional. The purpose of the law is not to
penalise a person for matters over
which they have no control. In such spirit, Section 80 of the IPC states, “accident
in doing a lawful act, - nothing
is an offence which is done by accident or misfortune, and without any criminal
intention or knowledge in the
doing of a lawful act in a lawful manner by lawful means and with proper care and
caution”.
[Edited and extracted from: https://www.barandbench.com/columns/criminal-
conspiracy-law-applicability]
71. Ram is a wanted criminal, and his picture is displayed across Chandigarh's
police stations and public locations.
There is also a monetary reward for anyone who is successful in apprehending him
and published the picture of
Ram. Ketaki is walking through the municipal park one day when she notices a man
who looks like Ram from
the picture. She follows him around the park, and when he gets near enough, she
knocks him hard in order to
catch him and denounce him to the police. However, following closer inspection, it
is discovered that the man
Ketaki injured is not Ram. Can Ketaki use the defence of mistake of fact?
(a) Yes, as Ketaki genuinely thought she had spotted a criminal.
(b) Yes, as Ketaki hit the person with the intention of helping the police.
(c) No, as the act committed by Ketaki was illegal in itself.
(d) No, as Ketaki should have determined the identity first and then hit him.
72. Karan and Varun are brothers who live on the third floor of a building. Anish,
their next-door neighbour, feels
the two brothers are smuggling wheat because he observes bags of wheat grains being
moved in and out of their
flat on a regular basis. Under this impression, he contacts the local police
station to report wheat smuggling.
However, when the police come and conduct an investigation, Anish's concerns are
proven to be unfounded. Can
Anish claim the defence of mistake of fact?
(a) No, as he was ignorant about the law and not facts.
(b) Yes, as he reported the activity to the police in good faith.
(c) No, as he should have first checked for himself then should have called the
police.
(d) Yes, as if it were not for Anish, the act would have gone unreported.
73. Kamlesh, a farmer, has a plot of land on which he grows wheat for the wholesale
market in the neighbouring
city. One day, he found that his crops had been uprooted, and he assumed that it
was the result of a Boar's
activities, as Boars had frequently been sighted in and around that region. On the
second day, he discovered his
crops uprooted once more and felt activity in the field. As there is no law that
makes shooting animal an offence
so he drew his bow and arrow and shot at the location where he sensed movement. It
was discovered that he had
injured a person. Can Kamlesh use the accident defence?
(a) No, as before shooting and arrow, Kamlesh should have determined the identity
of the miscreant.
(b) Yes, as Kamlesh shot an arrow believing it to be a Boar, and did not intend to
hurt a person instead.
(c) No, as Kamlesh shot an animal which is an illegal act in itself.
(d) Yes, as Kamlesh has all the rights to protect his field and crop, irrespective
of who attempts to destroy it.
74. Anam and Awez engage into a brawl in the middle of the road when the former's
car collides with the latter's
bumper plate. Anam was joined by her husband and a two-year-old child. In the midst
of the fight, Awez tried
to push Anam but accidentally pushed Anam's husband, who was holding the two-year-
old child, who
subsequently fell on the road. The child was seriously injured in the fall and had
to be transported to the hospital.
Can Awez claim the defence of accident to justify the child's injury?
(a) Yes, as he did not have the criminal intent or the knowledge to the hurt the
child.
(b) Yes, as he only intended to scare Anam and not cause any injury to anyone.
(c) No, as Awez should have forseen the result of his actions.
(d) No, as Awez’s actions did not constitute a lawful act done in a lawful manner.
. Page 19 of 40
75. Consider the factual circumstance described in the preceding question, in which
a mad dog invaded the conflict
scene and attempted to bite Anam's husband. In an attempt to chase away the dog,
Awez hits it, inadvertently
striking Anam's husband, who falls, and the child, who is severely hurt and must be
brought to the hospital. The
child eventually dies. Can Awez now use the defence of accident?
(a) No, as he hit at the dog with the criminal intent of harming it, irrespective
of who was injured as a result.
(b) Yes, as he did not have the criminal intent to cause the death of the child.
(c) No, as Awez’s actions were not done in a lawful manner.
(d) Yes, as Awez could not have foreseen the death of the child.
Passage (Q.76-Q.80): Some acts may be so harmful that the government must control
them with severe
penalties. For example, the law may impose a punishment even if the harm was caused
by someone else's
negligence. This is exactly what happens when the rule of strict liability is
applied. This regulation is crucial for
commercial and other operations that have the potential to cause serious harm.
Recently, six factory workers
died and 22 others were hospitalised after they inhaled toxic fumes leaking from a
chemical tanker parked near
a factory in Gujarat's Surat district. The strict liability principle is a crucial
concept in tort law. This idea is based
on the inherent harm that some activities can cause.
The law of strict liability is based on the well-known English case of Ryland's v.
Fletcher. It is claimed that if
someone keeps anything on his property for his own benefit, it should not escape
and cause harm to others. If it
escapes, the owner of such thing is obligated to compensate the victim even if he
was not negligent. There are,
however, a few exceptions to this rule:
i. Act of God: An act of God is a sudden, direct, and irresistible natural act for
which no one can reasonably
prepare. It can cause harm no matter how many safeguards are taken. Tsunamis,
tornadoes, earthquakes,
unusual rains, and other natural disasters are examples of acts of God. Any damage
caused by these activities
is not subject to strict liability.
ii. Wrongful act of a third party: Damages can occasionally be caused by the
involvement of other parties.
Renovation work in one apartment, for example, may create some inconvenience in
another. In this case, the
tenant who is subjected to the annoyance cannot sue his landlord. He may only sue
the individual who is
refurbishing the other apartment.
iii. Plaintiff's own fault: In some cases, the plaintiff may be at fault for the
harm he sustains. In such instances,
he cannot shift the responsibility to someone else, no matter how much he suffers.
In the case of MC Mehta v. Union of India, the Supreme Court of India used a
stricter version of the concept
of strict liability i.e. absolute liability. In this case, hazardous oleum gas had
escaped from a Shriram foods
and fertiliser industries factory. The gas has caused extensive harm to individuals
and enterprises in the
surrounding area. Similar happened in the Surat case where the chemical was being
illegally discharged from
the tanker when a toxic gas leaked from it and spread to nearby areas, due to which
26 workers present in
the dyeing and printing mill and surrounding areas fell unconscious following which
they were rushed to the
hospital.
No strict liability exceptions shall apply in some instances, according to the rule
of absolute liability. This is
often used when there are a large number of victims of a transgression. As a
result, those who do harm will
face unlimited liability for appropriately compensating victims.
[Edited and extracted from: https://www.firstpost.com/india/gujarat-six-factory-
workers-dead-22-
hospitalised-after-inhaling-toxic-fumes-10261861.html]
76. Aryan owns a rottweiler, one of the most vicious dog breeds. He lives in a
heavily populated area, and one day,
when Aryan was on his way to work, the dog escaped from the apartment building and
bit one of the children in
the colony in his hand. The child's right hand had to be amputated. Is Aryan
capable of defending any of the
exceptions given in the passage?
(a) Yes, as the plaintiff (the child) was at fault for having come in way of the
dog.
(b) No, as Aryan was the owner of the ferocious dog and was responsible to take
care, he should have been more
careful.
(c) Yes, as Aryan cannot be made liable for every mistake of his dog.
(d) No, as Aryan should not have brought such a ferocious dog to a crowded locality
in the first place as it is
illegal.

. Page 20 of 40
77. Consider the factual circumstance mentioned in the preceding question, in which
an earthquake caused the door
of Aryan's house to become unfastened after he had gone for work. When he returned
home, he learned that the
dog had left the house and harmed many people in his neighbourhood. Can Aryan use
any of the available
defences?
(a) No, because Aryan was aware of the dog's ferocity, he should have secured him
properly.
(b) Yes, as the earthquake was an act of God, and therefore, beyond Aryan’s
control.
(c) No, as Aryan should not have brought such a ferocious dog to a crowded locality
in the first place.
(d) Yes, as Aryan cannot be made liable for every mistake of his dog.
78. Kamal is a young trainee at the Police Academy, which is immediately close to
the local panchayat office. Kamal
goes to firing range every morning, which is in a compound surrounded by a fence,
beyond which lies the village.
During one of these drills, Kamal fires a shot that misses the target and strikes a
villager who was passing by
beyond the fence. Can Kamal be held accountable?
(a) No, as it was the part of his training to shoot at the target.
(b) Yes, as Kamal carried on a dangerous activity resulting in an escape which
caused harm.
(c) No, as it is the villager who should not have been roaming around the police
academy which such firing
activities are being conducted.
(d) Yes, as Kamal should have been aware that the bullet from his pistol could
injure anyone.
79. Lionheart Constructors is constructing a road through a remote region when it
comes upon a rocky ledge. Despite
the fact that the region is quite close to a residential neighbourhood, they decide
to blast away the rock. A child
playing in a nearby yard gets struck by a flying rock, resulting in a serious
laceration. Can the company be held
responsible?
(a) No, as the company could not have foreseen the flying rock hitting the child.
(b) Yes, as the activity of blasting rock is an inherently dangerous activity near
a housing district.
(c) No, as the child should have been more careful while near a blasting site.
(d) Yes, as the company should not have carried out blasting activity near a
housing district.
80. In the preceding question, as the rocky promontory is being blasted, a little
rocky hill located above caves in,
burying the entire housing area located in the low ground. Hundreds are said to
have been injured or killed. What
is the nature of Lionheart Construction Company's liability?
(a) Lionheart constructors are strictly liable for the event as it cans till claim
some exceptions to the situation.
(b) Lionheart constructors are not liable for the event as it could not have
foreseen the hill caving in.
(c) Lionheart constructors can claim the defence of act of God as the hill caving
in was not a direct result of the
blasting of rock.
(d) Lionheart Construction company is absolutely liable for the event, subject to a
court of law determining the
same.
Passage (Q.81-Q.82): A country’s pharmaceutical industry strategy states that –
Medicine availability and
affordability must be assured.
 The main price elements are who will provide them, who will pay for them and
their earnings, the production
and procurement dynamics, and finally the pricing problem. The above criteria will
have to converge to guarantee
medications are available to those who need them without putting a financial
burden. That is, in the end, the
essence of universal health coverage in general. Because out-of-pocket spending is
a significant contributor to
poverty, and medication spending is a significant component to that out-of-pocket
spending, cost, availability,
and quality is essential.
 Drug-price ceilings should not be imposed arbitrarily. The production cost must
be clearly defined. The
market price may be defined as the cost plus the margins that should be deducted
from the basic manufacturing
price. Otherwise, ceilings become increasingly arbitrary over time.
 It is critical to protect the domestic pharmaceutical sector and steps towards
the same should be taken while
making the policies. Acquisition of a well-developed indigenous business by
multinational profiteers should be

. Page 21 of 40
prohibited. To some extent, the public sector’s capability to prepare for
unforeseen events must be preserved,
particularly in the manufacture of pharmaceuticals for specific diseases;
nevertheless, this should not jeopardise
the company’s ability to produce.
 Trusting the private sector in terms of production and contribution of essential
pharmaceuticals of assured
quality is crucial, but encouraging them to keep their costs reasonable through
sufficient restrictions is also
important.
The country should have a stable, reasonable, and predictable policy that guides
both the manufacturing sector
and the health sector, which procures and uses medications. However, pricing must
be flexible in response to
changing circumstances and unique new realities.
[Edited and extracted from,https://www.livemint.com/opinion/online-views/new-laws-
for-our-pharma-sector#must-focus-on-reforms-11634057160516.html]
81. The government promises to rid the country of dengue infection by 2022. The
government supports
pharmaceutical firms to develop effective vaccines and anti-disease medications.
However, the drug’s
production costs are quite high. The government creates health ID cards with all
people’s Unique Identification
numbers and registers their bank accounts with it. The government then implements
subsidies for patients to
obtain medications in exchange for UIDs of health-cards that are immediately
remitted to their bank accounts.
Many states are holding legislative assembly elections this year. As per policy,
offering this subsidy is a bad
idea, according to the opposition. Is the opposing viewpoint, correct?
(a) No, the government has made efforts to lower out-of-pocket expenses.
(b) Yes, the government has put the information of the entire public at risk
through the health cards.
(c) No, getting the country dengue-free is part of the Government’s vow, thus it
must be done.
(d) Yes, this is an election gimmick and should not be done in an election year for
petty electoral advantages
82. MonkeyPox spreads out in the country immediately after this strategy is
implemented. Savlon, a private firm,
develops its immunisation and offers the pharmaceuticals at a reasonable price in
accordance with the policy.
However, the increased demand raises the price of the drug’s main components. This
forces Savlon to raise the
drug’s market price, making it unavailable to those on the poorest end of society.
A family member of a deceased
affected individual brings the corporation to court on the claim of breaking the
above-mentioned policy. Savlon
displays its financial statements, stating that it will be unable to manufacture
the pharmaceuticals at the prior
pricing. Should the Court decide in Savlon’s favour?
(a) No, the rapid increase in price should not be permitted because it broke the
country’s balanced pricing
structure.
(b) No, the firm was required to maintain the price in accordance with the law’s
public-sector capacity.
(c) Yes, because the corporation is a private company, it may choose its own
pricing.
(d) Yes, because the price of the crucial ingredient has risen, the market price
has risen as well.
83. Dengue cases have increased in the country. It has had a far greater impact on
the lower income group than on
the medium and upper income groups. The government imposes a rule requiring
pharmaceutical businesses to
charge a lot lower price to the lower-income group, a little higher price to the
middle-income group, and the
highest price to the higher-income group. Is the government correct in implementing
the policy of differentiated
pricing?
(a) No, this is discriminatory pricing and is not a stable pricing regime
(b) No, this type of pricing would cause a slew of issues for the administration
and society
(c) Yes, price can vary depending on who is paying and their income.
(d) Yes, the government may request that the pharma sector maintain their pricing
low.

. Page 22 of 40
84. The government proclaims that pharmaceutical businesses shall set their price
caps in such a way that the basic
price remains consistent with the basic production price and the surplus too
remains consistent with the basic
production price. This is being challenged by private corporations. They claim that
this is against to policy. Is
this correct?
(a) Yes, this will maintain the price very low, and according to the policy, this
will not protect their interests.
(b) Yes, this regulation will have an impact on medicine availability.
(c) No, this is in accordance with the policy’s proposed price ceiling.
(d) No, it is up to the government to set the price ceiling while keeping
medication availability in mind.
85. The government imposes a 100 percent Foreign direct Investment rule in the
country’s pharmaceutical industry
via an automatic route. This action has been met with strong opposition from the
country’s pharma enterprises.
Is the government correct in enacting this law in accordance with policy?
(a) No, 100 percent FDI in the industry will eliminate the country’s small-pharma
enterprises.
(b) Yes, foreign investment will result in a significant improvement in the quality
of medications.
(c) No, this move will make purchase of the country’s pharma-companies extremely
simple thus will impact the
domestic pharmaceutical sector.
(d) Yes, 100 percent FDI would boost the efficiency of the country’s pharma-
companies, resulting in job
creation.
Passage (Q.86-Q.90): In a nationwide programme to take the third eye to the sky,
the Ministry of Civil Aviation
(MoCA) has called for more effective utilisation of drone applications and urged
the Ministry of Home Affairs
to deploy unmanned aerial vehicles for surveillance, situational analysis, crime
control, VVIP security, disaster
management, etc. Further, the Ministry of Civil Aviation has instructed all drone
owners to voluntarily register
their drones before January 31, failing which they will be fined with a jail term
of up to two years or a fine of
₹10 lakh. All drones in India must be registered on the Digital Sky portal and have
a unique identification
number, according to the rules established by the Directorate General of Civil
Aviation (DGCA). All drone
operators must also have an Unmanned Aircraft Operator Permit (UAOP). Both went
into effect on December
1. Drones in India must have a unique identification number, with the exception of
nano drones flown up to a
height of 50 feet in an enclosed premises and remotely piloted aircraft operated by
security and intelligence
services. The Indira Gandhi Rashtriya Uran Academy is also being considered as a
drone school by the
government. Users will be required to place trackers on their drones. Given the
security concerns of utilising this
technology, the government is eager to compile a database of drone users. Once that
is determined, tracking such
equipment’s will be critical to ensuring safe operations even in green zones or
regions where they would be
permitted. According to a draft issued in November 2017, consumer drones are
classified into five categories:
Nano – less than or equal to 250 grams, micro – more than 250 grams but less than
or equal to 2 kilogrames,
mini – more than 2 kg but less than or equal to 25 kg, small – more than 25 kg but
less than or equal to 150 kg,
and big – more than 150 kg.
The majority of consumer drones produced today fall into the Nano and Micro
categories. Because Indian cities
are expanding vertically, having nano drones floating around without any oversight
may result in privacy
violation. As permission does not empower operator to interface in one’s privacy.
They might also be used for
immoral purposes such as business espionage, trespassing, spying, unauthorised
photography, and burglary.
India has witnessed several instances of unidentified drone activity in recent
years. Operations at the Indira
Gandhi International Airport in Delhi were suspended for about 40 minutes on August
20, 2017, after a pilot
spotted a drone nearby. There was a similar scare on July 25, 2018, and again on
August 9.
[Edited and Extracted from,https://www.thehindu.com/news/national/use-drones-more-
effectively-civil#aviation-ministry/article38157077.ece]

. Page 23 of 40
86. Kumar had purchased a drone, obtained an operator permission, and registered
the drone in the Digital Sky site.
He began utilising the drone for educational purposes. Is he capable of doing so?
(a) Yes, as he has obeyed all DGCA guidelines, hence he is permitted to fly the
drone
(b) No. The DGCA has not been alerted of the educational motive.
(c) Yes, he is permitted to fly it because he is doing it for educational purposes
(d) No. He should have registered with the DGCA rather than the Digital Sky site.
87. Rahul had purchased a toy drone for his ten-year-old daughter’s birthday. After
the notification was issued, he
registered the drone on the Digital Sky site and obtained an operator permission
for himself. His daughter had
learned how to fly the drone from him and had begun to fly it. Nonetheless, his
family had legal issues as a result
of this situation. What may be the explanation behind this?
(a) Rahul allowed his underage daughter to operate the drone
(b) Rahul was not required to register for a toy-drone.
(c) Rahul should have obtained an operator authorization for his daughter.
(d) Toy drones can cause issues in the aviation industry. They are not permitted.

88. Kashish registered his drone for commercial use with the Digital Sky platform.
He obtained his own operator’s
licence. He installed a tracker on his drone. Kashish suspects his next-door
neighbour is up to no good. He now
wishes to fly the drone over his neighbour’s house in order to keep an eye on him.
Is he capable of doing so?
(a) Yes, he has all of the necessary permissions and a tracker, so he is free to
use his drone
(b) Drone operators for espionage must attend Uran school in order to use them.
(c) Yes, because this act will be performed in the best interests of society
(d) No. This is a clear invasion of privacy of the neighbours
89. Anuj specialises in wedding photography and videography. He employs a drone; a
nano-drone, to be precise,
since it weighs less than 250 gm. He employs this to give his videography a
competitive advantage. After
receiving the notification, he assumed that he would be able to fly the drone for
one wedding after 31st before
registering it, as registering it would take time. Is he capable of doing so?
(a) Yes, he can do it since it is a nano-drone
(b) No. Because he will be using a drone with a camera, he must first register it
(c) Yes. He can fly the drone since it will only operate in a specific area.
(d) No. All drones need to be registered with Digital Sky portal
90. What is the major objective of enacting this new regulation, according to the
legislation and the new notification?
(a) The government wants to limit civilian drone usage;
(b) The government wants to create a database of drones in the country;
(c) The government wants to regulate drone usage so that it is not used
unethically.
(d) The government wants to control drone usage so that it does not impede
aviation.
Passage (Q.91-Q.95): The Epidemic Diseases Act is routinely enforced across the
country for dealing with
outbreaks of diseases such as swine flu, dengue, and cholera. The Act, which
consists of four sections, aims to
provide “for the better prevention of the spread of Dangerous Epidemic Diseases.”
Section 2 empowers state governments/UTs to take special measures and formulate
regulations for containing
the outbreak. The section includes special measures to be taken by the Centre to
“prescribe regulations as to
dangerous epidemic disease.” It also includes the detention of people or any vessel
that come from international
shores and are seen potent to spread the epidemic in the country.
The Act was formulated pre-independence mainly to control plague in the late 1800s.
The provisions of the Act
state that the government can fine people or imprison them for violating rules and
regulations that will be set to
contain the outbreak. According to Section 4 of the act, no suit or other legal
proceedings shall lie against any
person for anything done in good faith under the act. The Act can be used to
restrict the movement of suspected
coronavirus patients to prevent further spread of the disease.

. Page 24 of 40
[Edited and extracted from: https://indianexpress.com/article/explained/explained-
what-is-the-epidemic-act-of#1897-govt-has-invoked-to-fight-coronavirus-6309925/]
91. A dangerous contagious sexual disease had spread in Jaipur. The disease was
very similar to HIV but more
lethal. As it could spread to healthy persons even by coughing and sneezing besides
blood or bodily fluids. Mrs.
Mona, daughter of the former King of Malwa region travelled to Jaipur from USA. She
was to be tested for the
disease as was the rule made by State of Rajasthan for anyone coming from outside
India or other states to Jaipur.
Mrs. Mona refused citing herself to be dignitary. Decide.
(a) Special exception will be created for Mrs. Mona as she hails from a royal
family.
(b) Mrs. Mona can be tested for the disease without any exceptions or excuses.
(c) Mrs. Mona can be tested for the disease only after giving her compensation
commensurate to her position in
the society.
(d) Ladies are generally less likely be infected by a contagious disease as per a
recent report.
92. A bubonic plague had spread in Maharashtra. It was caused by flies which pick
up the plague causing bacteria
from faeces of rodents and infect food sources by sitting over it. The death rate
was almost 60%. A few medicines
showed some efficiency in reducing symptoms. However, there was no particular
medicine for the treatment of
the disease. The State of Maharashtra under The Epidemic Diseases Act, acquired a
few private hospitals as the
government hospitals were too less in number, compared to the size of the people
infected by the disease. A
compensation depending upon the market rate, as is the norm, was provided to the
acquired hospitals. Choose
the most appropriate option.
(a) The State of Maharashtra cannot acquire private land or hospitals.
(b) Prior consent of the Central government has to be taken before any acquisition
of land is made by the State
government.
(c) The action by the State of Maharashtra is legally sound.
(d) Only the Central government is empowered to act under The Epidemic Diseases
Act.
93. After the floods in Kerela, some exotic species entered the backwaters. They
were fished and sold in local
markets in Kottayam. The fish fetched a high price due to its delicious taste. A
month later many people in the
district developed diarrhea. A few even died. The numbers multiplied each day. A
central scientific team was
immediately dispatched. The team concluded that a novel fungi strain to which
Indians are not immune has
entered the human body through the exotic. The strain could spread even through
touch with the infected fish.
Also human to human transmission is possible. The government immediately sealed all
the fish markets in
Kottyam and restricted movement of the local people. The borders of the district
were totally sealed. Decide.
(a) The action by the government is not legally sound as the such arbitrary
restriction of freedom of movements
is violative of the Constitution.
(b) Essential supply must be allowed as otherwise the supply chain would be
impacted leading to loss of
livelihood for hundreds of people.
(c) The government should impose lockdown in the entire state of Kerala and not
just the district of Kottayam.
(d) The action by the government is legally sound as the disease was novel, deadly
and infectious.
94. Harsh loved Sona and proposed her for marriage. Sona however, was not
interested in him and disparagingly
rejected him. Harsh was dumbstruck and decided to do something great in life. He
worked very hard and finally
became a top-class doctor. He was later promoted to the District Health Officer
Gorakhpur District. In the
meantime, Sona got married to Satish and shifted to Salempur, a town in Gorakpur.
Taking advantage of his
position Harsh declared the area where Sona lived a ‘Containment Zone’ and imposed
strict curfew citing
increasing cases of mosquitoes borne disease, which though true were not at all
dangerous. All shops were
ordered to be closed and the entire area was cordoned off. Advise Sona and Satish.
(a) They should lodge a complaint against the actions of Harsh as they seem mala
fide.
(b) They should meet Harsh in person and apologize and seek his forgiveness.
(c) Satish must divorce Sona and let her marry Harsh else Harsh would make their
life a hell.
(d) Harsh should impose curfew on the house of Sona and Satish only. By doing so,
he would not cause any
problem to others who did not do any harm to Harsh

. Page 25 of 40
95. Sasha went to Maldives on a vacation with her family. After returning, she
developed some flu like symptoms.
On getting inspected from a local physician, a test was prescribed. The test
revealed that she suffered from very
mild viral infection which gets cured on its own in a week’s time. The physician
gave her some symptomatic
medicines and asked her to report back after 6 days. On the fifth day the symptoms
totally eased. If you are the
District Administrator of the district where Sasha lived. You get to know about
Sasha’s case. What is the most
reasonable action you would take, based on the passage?
(a) I will invoke The Epidemic Diseases Act and order sealing of the borders of the
entire district.
(b) I will invoke The Epidemic Diseases Act and order quarantining of Sasha and all
her family members who
went on a trip to Maldives.
(c) I will invoke The Epidemic Diseases Act and do testing for the flu virus of the
entire population of the
district.
(d) I will not invoke The Epidemic Diseases Act and let the status quo prevail.
Passage (Q.96-Q.100): The Delhi High Court has modified the conviction and sentence
of a man from sec. 397
(robbery or dacoity with an attempt to cause death or grievous hurt) of Indian
Penal Code to sec. 392 (punishment
for robbery) as the prosecution had failed to prove the use of a deadly weapon.
Justice Mukta Gupta was of the view that the prosecution is required to prove the
nature of the weapon of
offence used specially in the case of knife or blade.
"In the absence of the use of a deadly weapon being proved by the prosecution, the
conviction of the appellant
for offence punishable under Section 397 IPC cannot be sustained and is required to
be modified to an offence
punishable under Section 392 IPC," the Court said.
Perusing the testimonies of the prosecution witnesses, the Court said that it is
trite law that even if the weapon
of offence is shown after snatching had taken place for running away along with
snatched article, offence under
sec. 397 IPC is attracted.
"Thus, if the offender uses the deadly weapon at the time of committing robbery or
dacoity which would include
even the fear of instant death or instant hurt or wrongful restrain or an attempt
to cause death or hurt or wrongful
restraint even while carrying away or attempting to carry away the property
obtained by theft, the act of the
offender will fall within the four corners of Section 397 IPC," the Court said.
Though it is not essential that the weapon of offence should be recovered to prove
the nature of the weapon used
and that a deadly weapon was used at the time of commission of the offence,
however, the prosecution is required
to prove the nature of the weapon of offence used specially in the case of knife or
blade.
(Source : Prosecution Must Prove 'Nature Of Weapon' Used During Robbery Was Deadly
For Upholding
Conviction U/S 397: Delhi High Court , live law )
96. Radha went to the nearby local vegetable market to get some vegetables. She is
an extremely miserly lady since
she is always bargaining with the vendors. When she was returning from her office
one day, she saw some goons
on the street who she identified as vegetable vendors. They came to exact vengeance
on Radha. Radha saw the
vendors approaching her and noticed that they were holding a knife-like instrument,
but because it was dark, she
didn't spot it correctly and in fear she collapsed. They took her handbag and phone
and ran. What section will
they be punished under?
(a) section 397 of IPC as the robbers attempted to cause grievous hurt to Radhain
addition radha saw a knife
like weapon in hands of vendor.
(b) Section 397 of IPC as it was an act of revenge in addition to robbery.
(c) Section 392 of IPC as it was a case of robbery.
(d) As it was entirely an act of retribution, the thieves attempted to do serious
harm to Radha, and hence it was
punishable under sections 397 and 392 of the IPC.

. Page 26 of 40
97. Chinu owns a jewelry business in Indore's Sarafa Market. He usually closes his
shop since he does not trust any
of his employees with the locker or the shop. Three masked individuals ganged up on
him one day as he was
locking his shop and demanded that he open the shop's safe. When the robbers left,
Chinu attempted to phone
police from his landline, but before he could dial, the robbers pulled a sharp
knife around his neck and abandoned
him on a highway. Despite the prosecution's best efforts to show that the criminal
used a deadly weapon, Chinu
was in shock and could not recall anything from that night. Decide
(a) They will be arrested and tried under section 392 of IPC.
(b) They will be arrested and tried under 397 of IPC.
(c) They will be arrested and tried under for (robbery or dacoity with an attempt
to cause death or grievous hurt)
under section 392 of IPC.
(d) The court will modify the sentence from section 392 of IPC to section 397 of
IPC.
98. Rupali lived in a 46-story building with his only child Ruhan while her husband
was in Singapore. The building
was completely secure, and Rupesh, her husband, could track Rupali and his son
Ruhan's whereabouts at any
moment using an app. One day, a gang of thieves broke into Rupesh's flat and
attempted to steal all of the
luxurious items in it. Because Rupali teaches karate, she decided to fight with
them, but the other thief
backstabbed her with a sharp blade. Everything was recorded on CCTV. Decide
(a) The prosecution will be required to prove the nature of the weapon used.
(b) The prosecution will be required to prove that the weapon of offence is
recovered.
(c) The prosecution will be required prove the nature of the weapon used and that a
deadly weapon was used at
the time of commission of the offence.
(d) The respondent can submit recordings of CCTV to prove its case.
99. Choose the correct reference taken from passage :
(a) It is essential that weapon used in the offence must be recovered.
(b) It is essential that the offender use the deadly weapon to attract the offence
under section 392 of IPC.
(c) It is essential for an offender to use a deadly weapon for committing an
offence of theft.
(d) If the prosecution proved that the deadly weapon is used in committing the
offence the court can modify the
conviction and sentence of a man from sec. 392 (punishment for robbery) to sec. 397
(robbery or dacoity
with an attempt to cause death or grievous hurt) of Indian Penal Code.
100. Dhara was alone in the house when a man named Aakash entered and showed him a
video of her spouse Naman
being trapped in a room with a gun on his head. He also threatened Dhara's husband
with death if she did not
hand over all of the gold and cash to him. Ramesh's act punishable under?
(a) Section 392 of IPC as no deadly weapon was used at the time of commission of
offence by the robber.
(b) Section 397 of IPC will not be attracted as the video was fake.
(c) Section 397 will be attracted as the act of offender induced instant death or
instant hurt to Naman.
(d) Section 397 will not be attracted as there was no harm caused to Dhara.
Passage (Q.101-Q.105): In a significant pronouncement, the Bombay High Court on
Monday held that media
trial during criminal investigation interferes with administration of justice and
hence amounts to 'contempt of
court' as defined under the Contempt of Courts Act, 1971.
The Court also held that media reports interfering with criminal investigation,
before the initiation of trial, can
amount to interference with administration of justice.
"...we hold that any act done or publication made which is presumed by the
appropriate court (having the
power to punish for contempt) to cause prejudice to mankind and affect a fair
investigation of crime as well as
a fair trial of the accused, being essential steps for "administration of justice",
could attract sub-clause (iii) of
section 2(c) of the CoC Act depending upon the circumstances and be dealt with in
accordance with law"
Trial by media/pre-judgment while a police investigation is in progress could lead
to interference
with/obstruction to "administration of justice", the Court said.

. Page 27 of 40
"The expression "administration of justice” in section 2(c) (iii) of the Contempt
of Courts Act is sufficiently
broad to include civil as well as criminal justice. The stage from which
"administration of justice” commences
may be prior to institution/initiation of judicial proceedings.
The Bench has also held that the Press Council of India guidelines on reporting
shall be applicable to the
electronic media as well.
Some of the guidelines pronounced by Bench in the open Court are as follows:
 Publishing a confession alleged to have been made by an accused as if it is an
admissible evidence letting
the public know about its inadmissibility should be avoided;
 While reporting suicide, to suggest that the person was of weak character, should
be avoided;
 Reconstructing of crime scenes, interviews with potential witnesses, leaking
sensitive and confidential
information should be avoided during the investigation;
 Investigative agencies are entitled to keep secrecy about the ongoing
investigation and they are under no
obligation to divulge information.
Source: Source: media trial during criminal investigation interferes with
administration of justice and hence
amounts to 'contempt of court', Livelaw)
101. CC national, a news reporting channel was reporting a suicide case of a famous
business tycoon, who reportedly
was died in her own washroom in a bathtub due to a cardiac arrest. The reason of
death was shared by DGP in a
public announcement. The reporting of news channel was straight to the point and in
accordance with Press
Council of India guidelines. Does it amount to interference with administration of
justice?
(a) No, it does not constitute to interfering with the administration of justice
because it causes no harm to
humanity.
(b) Yes, it does amount to interfering with the administration of justice because
it can affect police investigation.
(c) Yes, it does amount to interfering with the administration of justice because
it can affect a fair investigation
of crime as well as a fair trial of the accused.
(d) No, because the reporting was direct and to the point, it does not amount to
interfering with the administration
of justice.
102. Assume, in a similar situation to the one described above, CC national news,
prior to the police investigation ,
and aired a separate show titled "who killed the tycoon"? They also called a
potential witness, who said he saw
someone strange go to his house on the day she died. People's brains were thrown
into disarray as a result of the
comment. According to witness evidence in the mainstream media, police began to
consider murder as an option.
Determine the liability of the CC national.
(a) CC national is not liable for interference with administration of justice as
they have exercised their right of
free speech and expression.
(b) CC national is not liable as interviews with potential witnesses does not
amount to interference with
administration of justice.
(c) CC national is liable as media trial interferes with administration of justice
by interviewing potential witness
and hence amounts to 'contempt of court'.
(d) CC national is liable for contempt of court as interviews with potential
witnesses was to be avoided during
media reporting.
103. Continuing with similar circumstances, CC national alleges that it does not
amount to interference with
administration of justice on grounds that the case was only in the investigation
stage and not proceeded for a
trial. Can CC national avail such defense?
(a) Yes, they can avail a defense as media trial while a police investigation is in
progress does not lead to
interference with "administration of justice.
(b) No, they cannot avail themself of the defense because the stage at which
"interferes with the administration
of justice" begins may be prior to the institution/initiation of judicial
proceedings.
(c) Yes, they can use the defense because the case is still being investigated by
police.
(d) No, they cannot use the defense because a media trial during a criminal inquiry
interferes with the
administration of justice and thus constitutes "contempt of court."

. Page 28 of 40
104. A well-known young Bollywood actor is said to have died at her residence. On
the face, it appeared to be a
suicide. This news was exploited by several news channels to increase their TRP.
AGP news, one such news
station, released a confession by an anonymous person claiming to be a close friend
of the actor. He disclosed in
the confession that the actor was going through a financial difficulty and that the
movie mafia "we all know
who" is solely to blame for the actor's murder. The news channel also argued for
the legitimacy and admissibility
of the aforementioned confession and pleaded with the judiciary for speedy justice.
Decide
(a) ABP news channel is liable for interference in the administration of justice.
(b) ABP news channel is liable for an offense of contempt of court.
(c) Both A & B
(d) ABP news channel is liable for interference in the administration of justice
only as they published a
confession as a piece of admissible evidence.
105. Choose the correct statement:
(a) The phrase "administration of justice" in section 2(c)(iii) of the Contempt of
Courts Act is not broad enough
to embrace both civil and criminal justice.
(b) Investigative agencies have the right to maintain secret about ongoing
investigations, but they are also
obligated to divulge information.
(c) Reconstructing of crime scenes, and interviews with potential witnesses are
common practices under a media
trial.
(d) Trial by media while a police investigation is ongoing could not disrupt the
"administration of justice."

. Page 29 of 40
SECTION - D: LOGICAL REASONING
Passage(Q.106-Q.110): Tolerance is a virtue badly needed in this diverse,
impatient, angry world.
Etymologically owing its origin to the Latin word tolerate, it implies enduring.
The word has however evolved
to mean much more. It has come to mean one who is free from hate... and prejudice —
somebody who is big
enough to accept differences in thought and practice in others. UNESCO’s
Declaration of Principles on
Tolerance describes it best. ‘Tolerance,’ the Declaration states, ‘is respect,
acceptance and appreciation of the
rich diversity of our world’s cultures, our forms of expression and ways of being
human. Tolerance is harmony
in difference. It is not only a moral duty; it is also a political and legal
requirement’.
The essence of tolerance thus is self-control. Tolerance ensures acceptance and
consequently less friction.
Intolerance is a slippery slope that so very easily leads to acts of violence. This
is the worrying aspect of
intolerance. An intolerant world is necessarily an imperfect world. Ideas are
stifled. Debate suffers, fear thrives,
creativity dies. Tolerance implies freedom of thought. The test of tolerance comes
when we are in the majority.
It means going the extra mile to instil a sense of confidence in the minority. It
is not a concession. And the
principle of Tolerance extends equally to those in the minority. They cannot thumb
their noses at the practices
of the majority and expect a tolerant response. Tolerance is ultimately a two-way
street, wherein we not only
accept differences but celebrate them. This is the hallmark of civil society
It does appear easy, but tolerance is difficult. As Yuval Noah Harari writes,
tolerance is not a Sapiens trademark.
Therefore, it becomes all the more important and urgent. Our world is increasingly
becoming pluralistic and
culturally and religiously diverse, and requires acceptance of these differences.
Tolerance has to be taught. The
key to such inculcation is education ‘in the most positive sense of the word, i.e.,
as nurturing and guiding’.
Education, whereby tolerance is taught, is the best investment societies can make
for a better future.
Prescribing limits as to what can be tolerated and what cannot is challenging.
Should ideas that are intrinsically
intolerant be tolerated? Does tolerance mean permissiveness? Does it mean that a
person, who despite having
the power to stop an objectionable practice and does not do so because he is
tolerant, is guilty of hypocrisy? Karl
Popper has termed this the ‘The Paradox of Tolerance’. These are troubling
questions for which there are no
easy answers. Governments all over the world have taken upon themselves the role of
prescribing limits. They
have increasingly demonstrated a strong streak of intolerance. While valid security
concerns have dictated this,
a constant review is essential – restrictions cannot become the norm.
Not all tolerance, however, is a virtue. We should never forget the lament of
Martin Niemoller, who in Nazi
Germany, tolerated all the evil that was happening around him since it did not
concern him, till “they came for
me, and there was no one left to speak for me.” So, it is alright to be intolerant
— if it means standing up for
eternal values of right and wrong
106. The author’s views presented in the passage can be best described as:
(a) Argument from Analogy - perceived similarities are used as a basis to infer
some further similarity that is
yet to be observed.
(b) Inductive Generalization - proceeds from a premise about a sample to a
conclusion about the population.
(c) Argument from Authority - a claimed authority’s support is used as evidence for
an arguments conclusion
(d) Causal Reasoning - one that focuses specifically on how something has caused,
or has led to another thing.
107. All of the following has been implied in the passage EXCEPT?
(a) Tolerance is not essentially an inherent or inbuilt behavioural trait.
(b) Tolerance can sometimes be perceived as a vice depending on the situation.
(c) Intolerance is the major threat to democracy where debate suffers, fear
thrives, creativity dies.
(d) Governments are forced to prescribe limitations that are intolerant when the
situations demand, but it cannot
become a routine practice.

. Page 30 of 40
108. Which one of the following best describes the organization of the passage?
(a) Several concerns that are currently being debated are investigated.
(b) A problem is found, and its causes and principles are summarized.
(c) A number of theories are examined, and one is recommended.
(d) Two important scholars' works are summarised.
109. The author has quoted ‘UNESCO’s Declaration of Principles on Tolerance’ in
order to
(a) Weaken the earlier assumption. (b) Strengthen the earlier premise.
(c) Reach a conclusion. (d) Neutralize an ideology
110. The author’s style can best be described as
(a) Perspicuous. (b) archaic (c) esoteric (d) objective
Passage(Q.111-Q.115): In 1960, during an American trip, when a journalist asked the
popular and admired
Indian politician couple, who belonged to two different political parties, “What
happens in India when husband
and wife are leaders in opposing political parties?”, both replied: “Peaceful co-
existence”. They further
laughingly added, “if we had been in America, we probably would have been divorced
twice. Political rivalries
haven’t extended into our married life.”
The example of husband and wife occupying the topmost positions in two foremost
opposing political parties of
the country and leading an ideal harmonious life with each other is unparalleled in
Indian democracy and history.
That couple was Sucheta Kriplani and J B Kriplani (Acharya Kriplani); both were
ardent Gandhians, socialists,
independence activists and leading politicians of pre and post-independent India.
J B was much senior to Sucheta in public life and had already established himself
as a strong national leader.
Sucheta, who was the professor of history at BHU, was still finding her ground.
Gradually Sucheta not only
started rising, but during the Quit India phase (1942), she emerged as the star of
the underground movement that
shook the roots of the foreign government. She also became the right hand of Gandhi
in his constructive work
in villages and relief and rehabilitation work in the riot-torn region of Noakhali
(1946).
`Due to her contributions, Sucheta was elected a member of the Indian Constituent
Assembly from the United
Provinces (now Uttar Pradesh). On August 14, 1947, when India as a nation was about
to be born, the Constituent
Assembly held its Independence Session which started at 11 o’clock in the night. It
was the most momentous
and reflective juncture when in that historical Session just before Jawaharlal
Nehru’s Tryst with Destiny speech,
Sucheta Kriplani exhilarated the Assembly by the soul-stirring recitation of Vande
Mataram. It was a pinnacle
moment for her because she not only opened the Session with her singing but also
concluded it by reciting Saare
Jahan Se Achha and the National Anthem.
Sucheta had a sweet, rich and soulful voice and her Bhajans had been the regular
feature of Gandhi’s prayer
meetings during 1946-47. However, behind a soft voice was an iron-willed and gutsy
person, who in spite of
being highly influenced by the ideas and philosophies of her political guru,
Mahatma Gandhi, and her husband,
remained fiercely independent, made her own choices and chartered her own course.
In spite of belonging to
opposing political parties in the Lok Sabha, Sucheta and J B epitomised mutual
tolerance and integrity in politics
by staying together cordially.
In 1963, the unstoppable Sucheta became the first woman chief minister of an Indian
state— Uttar Pradesh, the
largest one. It was a remarkable feat given fewer women in politics at that time.
From teaching history to creating history, during her eventful journey of life,
Sucheta Kriplani stood out as one
of the most dynamic political personalities with a splendid record of service to
her credit. She played a prominent
role in shaping the contours of independent India.
111. In establishing their views in the passage, the author employs which of the
following techniques?
(a) Assert shared beliefs by quoting the authorities.
(b) Counterargue the belief systems of various institutions.
(c) Cite a luminary’s achievements as the passage's main theme.
(d) Theorise about cause and effects.

. Page 31 of 40
112. All of the following are explicitly mentioned in the passage, EXCEPT…
(a) The powerful politician couple, Sucheta Kriplani and J.B Kriplani during their
American trip voiced their
opinion about political rivalries not playing any part in their marriage
(b) Sucheta Kriplani was a staunch Gandhian.
(c) Acharya Kriplani encouraged Sucheta to join the freedom struggle, besides
introducing her to Gandhi
himself.
(d) Sucheta is especially remembered for her role in the Quit India Movement of
1942.
113. Which of the following can be inferred as a view the author will most likely
agree with?
(a) By dint of her honesty, level-headedness, intelligence and hard work, Sucheta
Kriplani became one of the
prominent faces of the Indian freedom struggle.
(b) It was due to the guidance and influence of her husband, Sucheta Kriplani
deviated from her political journey.
(c) Sucheta’s marriage to freedom fighter J.B Kriplani was to some extent shaped by
Gandhi.
(d) Sucheta’s marriage to JB Kriplani leaned her towards philosophies of Mahatma
Gandhi.
114. The author asserts that Sucheta, a history professor at BHU, was still finding
her ground. What prompts the
author to make this statement?
(a) J.B Kriplani was 20 years senior to Sucheta and he had much more experience in
life.
(b) J.B Kriplani was already a prominent figure in the Indian political set up
while Sucheta without any prior
political experience had just embarked on her new journey.
(c) Sucheta Kriplani was yet to find out which political party she wanted to join,
while J.B Kriplani was already
a part of a major party.
(d) Sucheta was merely a history professor at BHU and becoming a political leader
was much more demanding.
115. Which of the following, if true, will strengthen the author’s stance on the
power couple’s harmonious
relationship?
(a) Both husband and wife despite having different political loyalties, never
questioned each other, thereby,
respecting each other’s choices.
(b) Sucheta in later years discharged her duties as a very able and careful
housewife.
(c) Sucheta speaks of her husband J.B Kriplani fondly in her autobiography.
(d) Their marriage was opposed by both their families, including Mahatma Gandhi,
but they eventually got
married.
Passage(Q.116-Q.120): Democracy is deceptively simple in both form and appearance,
but it is intricate and
complex in its real-life manifestation. The fact of the matter is that democracy
when wrongly practised becomes
worse than dictatorship or totalitarianism. Its essence will be lost if not handled
with sensitivity. As a nation, we
take pride in proclaiming that ours is the largest democracy in the world. Though
this is a plausible affirmation,
the real danger lies in our perpetuating certain wrong practices and corrupting the
very soul of democracy.
Sycophancy is one such practice, defiling the sanctity of democracy.
Sycophancy is defined as ‘obsequious behaviour towards someone important in order
to gain advantage’. The
tendency to flatter someone in power, and do things normally considered demeaning
to human dignity with the
ulterior motive of pleasing someone in authority is one of the most obnoxious
sights to see. That this is done by
the ones believed to be educated and cultured makes it utterly repulsive. In doing
so, the dignity of the sycophant
is compromised, which is the very negation of a democratic way of life.
Bureaucrats touching the feet of ministers, helping the master with his shoes,
opening the door of the car when
the boss wants to get in or out, etc., are regular sights.
Some might argue that it is part of our culture. Sorry, it’s not. Showing reverence
to elders and gurus by touching
their feet may be justified, if it is voluntary and done with no ulterior motive.
It is a hallowed Indian tradition to
show respect to elders and receive their blessings. It is not a hypocritical act to
curry favour with someone
powerful.

. Page 32 of 40
Is sycophancy harmless? It’s not entirely so. For instance, there are some people
who perfect the art of
sycophancy to enviable heights. Egoistic bosses will be bowled over easily. The
shallower the master, the easier
it is to flatter him.
Ask anyone practising these things. They do it because that’s how it is usually
done. They do it to please the
master, with the ulterior aim of getting some favour. They take comfort in the
belief that it would do them some
good. At least, it will not endanger their prospects. That would obviously mean
that people do these things not
because they are happy or willing to do it. Perhaps they will be cursing the
recipient of their sycophantic
genuflections even as they seem to be paying obeisance to him/her. Look at the
hollowness of it all!
The flip-side of a culture of sycophancy is that honest people who do not want to
enact this charade will soon be
in the bad books of the master. Such innocent mortals suffer in this murky
situation. Their merit or efficiency
would not count, thereby demoralising the really capable hands. That certainly ill-
behoves a modern nation.
The centrality of citizens and their dignity should not be negotiable in a true
democracy. Sycophancy is nothing
but hypocrisy glorified to feed the grandiose delusions of those in power.
Obviously, it flies in the face of an
authentic democracy.
116. Which of the following statements best conveys the core thinking behind the
passage?
(a) The passage seeks to outline the principles of democracy.
(b) The passage sheds light on India as the largest democracy in the world.
(c) The passage seeks to highlight the insignificance of the practice of
Sycophancy.
(d) The passage talks about the practice of Sycophancy and the implications behind
it.
117. The primary purpose of the passage is to
(a) describe a commonplace practice and its potential drawbacks and inaccuracies
(b) propose a temporary solution to a problem that mars a system
(c) analyze a frequent source of disagreement between the governments on core
issues.
(d) explore the implications of a finding that leads to optimal functioning.
118. The author proclaims “Look at the hollowness of it all!” What propels the
author to make this statement?
(a) To denounce the hypocrisy that people who practise sycophancy will most likely
be cursing the recipient of
their sycophantic genuflections even as they are trying to please them.
(b) To describe that sycophancy is a problematic practise that is emptying the
country of its moralistic fabric.
(c) To propose that the dignity of the sycophant is compromised, which is the very
negation of a democratic
way of life.
(d) To state that Bureaucrats touching the feet of ministers, helping the master
with his shoes is unacceptable.
119. Through the passage, the author primarily seeks to
(a) define a set of terms. (b) outline a new approach.
(c) describe a current problem. (d) present historical information.
120. ‘Sycophancy is defined as ‘obsequious behaviour towards someone important in
order to gain advantage’. The
assumption behind author’s defining the term is that
(a) The reader may not be aware of the term sycophancy.
(b) The reader may not be aware of sycophancy.
(c) The reader is aware of the term sycophancy.
(d) The author places very little value of readers’ calibre.

. Page 33 of 40
Passage(Q.121-Q.125): Almost 11 months after the United States’ leading
telecommunications companies won
bids for $81 billion worth of C-band radio spectrum to roll out 5G services, the
much-awaited introduction hit a
major snag this week after the country’s leading airlines warned of massive flight
disruptions if the wireless
technology was put into operation, especially around the nation’s airports. In a
compromise on the eve of the
planned roll-out for last week, AT&T and Verizon agreed to delay introduction of
the new wireless service near
key airports. The two major telcos’ commitments notwithstanding, several domestic
and international airlines
flying to the U.S. have announced major rescheduling as well as the possibility of
cancellation of flights to
several destinations citing warnings from the Federal Aviation Administration (FAA)
and aircraft makers that
accurate functioning of radar altimeters in some aircraft may be affected by the 5G
radio frequencies. At the
heart of the impasse lies the fact that both the 5G services and some flight
equipment operate on the same C#band radio spectrum, with only the frequencies
varying.
The fact that the world’s largest economy is now faced with the risk of large-scale
domestic and international air
travel disruptions as a result of the relatively belated roll-out of 5G telecom
services points to the peculiar
problems of the U.S. market, including the particular frequencies allotted there
for 5G. The U.S. telcos have
suggested the FAA and airlines need to do more to find solutions to the deployment
of the wireless technology
near airports. The FAA on its website has pointed to the specifics of the proposed
5G roll-out in the U.S. with a
comparison to the situation in France and asserted that some key differences pose
challenges. The administration
will have their task cut out in pushing to hammer out a long-term solution to this
impasse so as to minimise any
further disruption to global travel and trade sectors.
121. Which option best conveys the main idea of the given passage?
(a) The benefits of an established 5G service would balance the inconvenience it
caused during its inception.
(b) Travel disruptions due to the roll out of 5G services calls for solutions.
(c) Temporary disruptions in other forms of travel would not have made this small
issue a big fuss.
(d) 5G services should not be rolled out until a fool proof plan of action is
tailored.
122. If the information given in the passage is true, which among the following
must be true as well?
(a) Turning on some wireless towers near the airports would solve the issue.
(b) The likelihood of the administration finding a long-term solution is low.
(c) The US administration would find the solution to the issue quickly.
(d) Radar altimeters play a significant role in maintaining the safety of an
aircraft.
123. Among the following options, which option provides most strength to the claim
made in the first sentence of the
second paragraph?
(a) The economic situation of the US is facing major challenges and the situation
is far from stable.
(b) The US has a first mover advantage in rolling out 5G services and expanding the
network.
(c) Roll out of 5G services in countries like India and Pakistan did not disrupt
flight services.
(d) Large-scale domestic and international air travel disruptions are frequent in
the US.
124. With which among the following options would the author most agree?
(a) If the situation in France and America regarding the proposed 5G roll-out was
similar, then the flight
disruptions would have been avoided.
(b) If the radio spectrum of the 5G services and some flight equipment were
different, the problem would not
have been as big as it is.
(c) The FAA is a team with under qualified individuals and more needs to be done to
prevent such major
disruptions in future.
(d) In the long term, the FAA needs to clear and allow the vast majority of the US
commercial aeroplane fleet.

. Page 34 of 40
125. Whose claim is being strengthened by the following piece of evidence? “5G
power levels are lower in France,
with even the planned temporary nationwide lower power levels in the U.S. still
expected to be 2.5 times more
powerful.”
(a) That of the FAA. (b) That of the author.
(c) That of the Airline Companies. (d) That of the US President.
Passage(Q.126-Q.130): The NHRC has done the right thing in directing the Ministry
of Home Affairs and the
Arunachal Pradesh government to submit an action taken report against the racial
profiling and relocation of the
Chakma and Hajong communities in the northeastern State. They had fled their homes
in the Chittagong Hill
Tracts in erstwhile East Pakistan (present-day Bangladesh) after losing land to the
construction of the Kaptai
dam on the Karnaphuli river in the early 1960s. They had sought asylum in India and
were settled in relief camps
in Arunachal Pradesh. Since then, they have been well integrated in villages in the
southern and south-eastern
parts of the State. In 2015, the Supreme Court directed the State to grant them
citizenship, but this had not yet
been implemented. In a judgment in 1996, the Court had stated that the “life and
personal liberty of every Chakma
residing within the State shall be protected”. In light of these orders and given
that most of the Chakma/Hajong
community members were born in the State and have been living peacefully, the
Arunachal Pradesh Chief
Minister’s announcement, in August 2021, that they would be relocated outside the
State and that steps would
be taken for a “census” of the communities was clearly unwarranted. The so-called
State-driven census would
have amounted to a racial profiling of the two communities that have also been the
subject of an antagonist and
nativist campaign by organisations such as the All-Arunachal Pradesh Students’
Union.
It is difficult, but not impossible, for any State government in the northeast to
balance the interests of native
tribal communities and those of legitimately settled refugees and their progeny.
Special rights guaranteed in the
Indian Constitution in these States in order to protect the tribal people, their
habitat and their livelihoods, have
more than occasionally been misinterpreted as favouring tribal nativism with
overblown demographic fears
fanning hatred for communities such as the Chakma/Hajong in Arunachal Pradesh and
Mizoram. Unfortunately,
political forces have also limited themselves to using ethnic fissures for power
and sustenance. Uprooting
communities that fled their homelands under duress and have since been well settled
in their adopted areas,
contributing to the diversity of culture and the economy, would be a violation of
their rights and repeating a
historic wrong. A dialogue between the State government, civil society and those of
the Chakma/Hajong
communities would go a long way in addressing concerns in implementing the Court
judgment of 2015, rather
than the course currently adopted by Itanagar. Implementing the NHRC directive
should be a step in the process
to reverse that course.
126. Which option best represents the main point in the given passage?
(a) The Chakma and Hajong community deserve citizenship and not racial profiling.
(b) Giving refugees citizenship would help India’s diplomatic interests in future.
(c) Political factions demanding the ouster of refugees do not have civic sense.
(d) The rest of India’s integration with the north east is important for our
society.
127. What can be said to be an assumption made by the author in the passage?
(a) A society can be well integrated with another society in the course of a few
decades.
(b) Integrating a society with another well placed society takes more than twenty
years.
(c) A party can be called a liberal party only when it keeps the interests of all
members of a society.
(d) Communities seeking asylum in another country have no right to citizenship.
128. If the information given in the passage is true, then which of the following
is most strongly supported?
(a) Implementation of the Supreme Court’s pronouncement of 2015 is relatively easy.
(b) Chakma and Hajong communities would have to look after themselves on their own.
(c) It may take over 5 years to implement a Supreme Court’s judgement.
(d) The political scenario of Arunachal Pradesh is quite different from the rest of
India.

. Page 35 of 40
129. What opinion does the author form about the directive of the NHRC?
(a) The position taken by some political leaders in Arunachal Pradesh should be
rejected.
(b) The NHRC should go ahead and single-handedly implement its directive.
(c) The position is by far the worst that has been taken by some political leaders
of Arunachal Pradesh.
(d) The directive of the NHRC could well be a turning point for some communities.
130. Statement I: It is difficult, but not impossible, for any State government in
the northeast to balance the interests
of native tribal communities and those of legitimately settled refugees and their
progeny.
Statement II: Unfortunately, political forces have also limited themselves to using
ethnic fissures for power and
sustenance.
Which option best depicts the relationship between Statements [1] and [2]?
(a) Statement [1] and [2] are mutually unexclusive.
(b) Statement [1] strengthens the claim made in [2].
(c) Statement [2] lends support to the claim made in [1].
(d) Statement [1] weakens the position of the author in [2].
Passage(Q.131-Q.135): As the joint statement at the end of the India-Central Asia
virtual summit last week
noted, ties between India and the region have been historically close, with
“civilisational, cultural, trade and
people-to-people linkages”, but the lack of access to land routes, and the
situation in Afghanistan are among the
biggest challenges. Hosted by Prime Minister Narendra Modi with the Presidents of
the five Central Asian
Republics (CARs), it was a first, building on years of dialogue. The summit also
came after the meeting of NSAs
in Delhi, where they built on several common themes of concern and priority. To
begin with, there is the problem
of routing trade — a paltry $2 billion, spent mostly on Kazakhstan’s energy exports
to India. In comparison,
China’s CAR trade figures have exceeded $41 billion — they could double by 2030.
With Pakistan denying
India transit trade, New Delhi’s other option is to smoothen the route through
Iran’s Chabahar port, but that will
involve greater investment in rail and road routes to Iran’s northern boundaries
with the CARs, something India
is hesitant to do in the face of U.S. sanctions. A third option is to use the
Russia-Iran International North-South
Transport Corridor via Bandar Abbas port, but this is not fully operational and at
least two CARs (Uzbekistan
and Turkmenistan) are not members. India too, has dragged its feet over TAPI gas
pipeline plans (Turkmenistan#Afghanistan-Pakistan-India), due to supply guarantees,
given the tensions with Pakistan. Finally, there is
Afghanistan: the tenuous link between Central Asia and South Asia, where after the
Taliban takeover, there is
no official government, a humanitarian crisis is building, and there are worries of
terrorism and radicalism
spilling over its boundaries. Each theme has been outlined in the summit joint
statement as areas to work upon.
They have also agreed to more structured engagement, including the setting up of
joint working groups, on
Afghanistan and Chabahar, and more educational and cultural opportunities.
While the attempt by India to institutionalise exchanges and press the pedal on
trade, investment and
development partnerships with the CARs is timely, it is by no means the only
country strengthening its ties here.
China is now the biggest development and infrastructure partner to the countries.
The CAR Presidents held a
similar virtual summit with Chinese President Xi Jinping earlier. Pakistan has also
increased its outreach to the
CARs, signing transit trade agreements, offering trade access to the Indian Ocean
at Gwadar and Karachi. India
will need to move nimbly to ensure it stays in step with the changes, and to make
certain the future of ties more
closely resembles the deep ties of the distant past.
131. Which of the following pieces of evidence strengthens the author’s arguments?
(a) Kazakhstan’s energy exports to India are highest in the world in terms of pure
value.
(b) Russia is a clear winner when it comes to strategically engaging with the CAR.
(c) The ties between the United States and Iran are great, to say the least.
(d) India has smooth access to land routes through Central Asia regarding trade.

. Page 36 of 40
132. Based on the information in the passage, which of the following is strongly
supported?
(a) India’s options to engage meaningfully in trade with the CAR are limited.
(b) India’s attempts to increase its cultural ties with the CAR would be futile.
(c) India’s attempts to engage with the CAR have come in a bit too late.
(d) India’s attempt to engage meaningfully in trade with CAR treads on perils.
133. The central argument of the passage is best represented by:
(a) India must ensure that other countries do not make meaningful ties with the
countries in the CAR.
(b) India’s ties with the countries in the Central Asian Region should revolve
around modern projects.
(c) India must not allow its contemporaries to keep it out of the mix when it comes
to the CAR.
(d) The challenges are many, India must stay in step with the changes in the
Central Asian Region.
134. Out of the following, with which one of the following would the author agree
with?
(a) India’s virtual summit with the Central Asian region is a novel concept.
(b) India’s option to engage with the CAR through Afghanistan is far from ideal.
(c) China’s investment in the CAR region is more than that in its BRI project.
(d) India has the first mover advantage when it comes to engaging with the CAR.
135. “As in other parts of the extended neighbourhood, the gap between India’s
reach and grasp in the CAR is quite
large.” What role does this statement play towards the passage?
(a) This statement strengthens as well as weakens the arguments made in the
passage.
(b) This statement weakens the arguments made in the passage.
(c) This statement can be inferred from the information in the passage.
(d) This statement does not have a direct link with the passage

mock 31
Directions(Q.1-Q.30): Read the following passage carefully and answer the questions
that follow.
Passage (Q.1-Q.5): During the second half of World War II, a soldier named
Yossarian is stationed with his Air
Force squadron on the island of Pianosa, near the Italian coast in the
Mediterranean Sea. Yossarian and his
friends endure a nightmarish, absurd existence defined by bureaucracy and violence:
they are inhuman resources
in the eyes of their blindly ambitious superior officers. The squadron is thrown
thoughtlessly into brutal combat
situations and bombing runs in which it is more important for the squadron members
to capture good aerial
photographs of explosions than to destroy their targets. Their colonels continually
raise the number of missions
that they are required to fly before being sent home, so that no one is ever sent
home. Still, no one but Yossarian
seems to realize that there is a war going on; everyone thinks he is crazy when he
insists that millions of people
are trying to kill him.
Yossarian’s story forms the core of the novel, so most events are refracted through
his point of view. Yossarian
takes the whole war personally: not swayed by national ideals or abstract
principles, Yossarian is furious that his
life is in constant danger through no fault of his own. He has a strong desire to
live and is determined to be
immortal or die trying. As a result, he spends a great deal of his time in the
hospital, faking various illnesses in
order to avoid the war. As the novel progresses through its loosely connected
series of recurring stories and
anecdotes, Yossarian is continually troubled by his memory of Snowden, a soldier
who died in his arms on a
mission when Yossarian lost all desire to participate in the war. Yossarian is
placed in ridiculous, absurd,
desperate, and tragic circumstances—he sees friends die and disappear, his squadron
get bombed by its own
mess officer, and colonels and generals volunteer their men for (------) most
perilous battle in order to enhance
their own reputations.
Catch-22 is a law defined in various ways throughout the novel. First, Yossarian
discovers that it is possible to
be discharged from military service because of insanity. Always looking for a way
out, Yossarian claims that he
is insane, only to find out that by claiming that he is insane he has proved that
he is obviously sane—since any
sane person would claim that he or she is insane in order to avoid flying bombing
missions. Elsewhere, Catch#22 is defined as a law that is illegal to read.
Ironically, the place where it is written that it is illegal is in Catch#22 itself.
It is yet again defined as the law that the enemy is allowed to do anything that
one can’t keep him from
doing.
https://www.sparknotes.com/lit/catch22/summary/
1. According to the passage, what is the meaning of Catch-22?
(a) Catch-22 is any paradoxical, circular reasoning that catches its victim in its
illogic.
(b) Catch-22 is an endless cycle of logically consistent arguments put forth one
after the other.
(c) Catch-22 is a measure or policy whose effect is the opposite of what was
intended
(d) Catch-22 is a situation presenting two equally undesirable alternatives.
2. The mission, wherein the death of Snowden occurred, brought a significant change
in Yossarian. What was that
change?
(a) He was continually troubled by his memories.
(b) He acquired a strong desire to die beside his friend.
(c) He acquired a strong desire to live and be immortal.
(d) He lost his will to fight the in the war.
3. What is the tone of the author?
(a) Satirical (b) Sceptical (c) Cynical (d) Ironical
4. Who among the following is/are representative of brutish selfishness?
(a) Yossarian. (b) Snowden. (c) Generals. (d) Pianosa.

. Page 3 of 36
5. Insert the appropriate article in the above italicised line to make it
grammatically correct#Yossarian is placed in ridiculous, absurd, desperate, and
tragic circumstances—he sees friends die and
disappear, his squadron get bombed by its own mess officer, and colonels and
generals volunteer their men for
(-------) most perilous battle in order to enhance their own reputations.
(a) A (b) An (c) The (d) No insertion required
Passage (Q.6-Q.10): Child psychology is the study of subconscious and conscious
childhood development.
Child psychologists observe how a child interacts with their parents, themselves,
and the world, to understand
their mental development.
Everyone wants their child to have healthy development, but it’s not always clear
if a child’s behaviour is a
symptom of a normal stage in development or a sign of an abnormality. Child
psychologists can help you
understand the difference. Understanding the normal and abnormal psychological
patterns of a child can help
parents understand how to best communicate and connect with their child, teach
their child coping mechanisms
for managing emotions, and help their child progress and thrive in each new
developmental stage.
Child psychologists can also identify abnormal behaviours early, help detect the
root of common behavioural
issues such as learning issues, hyperactivity, or anxiety, and help children work
through early childhood trauma.
They can also help to prevent, evaluate, and diagnose developmental delays or
abnormalities such as autism.
Physical development in children is typically a predictable sequence of events.
Your child holds their head up,
rolls over, crawls, walks, and runs, in that order. Your Child Psychologist can aid
your paediatrician in observing
your child’s physical development, and if there are any abnormalities that could
indicate developmental
irregularities. Child Psychologists will observe your child’s progression toward
the milestones of development
to ensure that your child is physically developing normally. Major delays in
physical development may reveal
other underlying developmental issues that can then be addressed early on.
The medical understanding of childhood cognitive development has greatly changed
over the recent years.
Emotional and social development are deeply intertwined. Emotional development
refers to how the child feels,
understands, and expresses their emotions. Emotional development is expressed in
very young children through
the expression of basic emotions like fear, joy, anger, and sadness. As the child
develops, more complex emotions
such as confidence, hope, guilt, and pride emerge. Emotional development also
includes a child’s ability to feel
and understand the emotions of other people through empathy. Learning to regulate
and express emotions
appropriately is difficult for many children. Helping children understand their
emotions early can have a
powerful impact on current and future emotional development. A Child Psychologist
can help your child
understand and express their emotions in a healthy, positive way.
Emotional development highly informs social development. Social development is
about how the child develops
the values, awareness, and social skills necessary to relate to the people around
them; their parents, peers,
authority figures, and animals. A child’s early relationships can have a huge
impact on their development of age#appropriate social skills. Trust, friendship,
conflict management, and respect for authority are examples of social
development.
6. Based on the passage, in what way is Child psychology helpful?
(a) Child psychology helps an adult identify abnormal behaviours early, help detect
the root of common
behavioural issues such as learning issues, hyperactivity, or anxiety, and help
children work through early
childhood trauma.
(b) Child psychology develops a child’s social values, awareness and social skills
necessary to relate to the
people around them
(c) Child psychology helps in observing a child’s interaction with their parents,
themselves, and the world by
understanding their cognitive, emotional and social development.
(d) It makes the parent more conscious about the abnormality of their child.

. Page 4 of 36
7. According to the passage, which among the following is not true?
(a) Identifying the abnormality in the early stage can solve problems like autism.
(b) Psychologists study the activity of the child and can identify if there is any
abnormality in the behaviour.
(c) Learning issues, hyperactivity, or anxiety are issues that are dealt by the
Child psychologist.
(d) The medical understanding of childhood cognitive development has advanced in
transforming a child’s
mental make-up.
8. ‘Emotional development highly informs social development.’ Which of the
following is the most suitable
interpretation of the statement in isolation?
(a) Emotional and social developments are parallel to each other.
(b) Emotional and social developments are interwoven.
(c) Social development is not really important at an early age.
(d) Emotional development indicates social development.
9. Which of the following is a suitable title for the passage.?
(a) Definition of Child Psychology.
(b) Scope of Child Psychology.
(c) Introduction to Child Psychology.
(d) Child’s Psychology.
10. What is the main function of psychologists while working with children?
(a) To point out the disabilities of the child and provide proper medicine.
(b) To identify the root cause of the psychological, cognitive and social problems
of a child.
(c) To guide the parents to communicate in the right way with the children.
(d) To collate the data of behavioural aspects of children for progressive
analysis.
Passage (Q.11-Q.15): Many people seem older than they are. Research into the causes
of premature aging has
shown that stress has a lot to do with it, because the body wears down much faster
during periods of crisis. The
American Institute of Stress investigated this degenerative process and concluded
that most health problems are
caused by stress. Researchers at the Heidelberg University Hospital conducted a
study in which they subjected
a young doctor to a job interview, which they made even more stressful by forcing
him to solve complex math
problems for thirty minutes. Afterward, they took a blood sample. What they
discovered was that his antibodies
had reacted to stress the same way they react to pathogens, activating the proteins
that trigger an immune
response. The problem is that this response not only neutralizes harmful agents, it
also damages healthy cells,
leading them to age prematurely. The University of California conducted a similar
study, taking data and samples
from thirty-nine women who had high levels of stress due to the illness of one of
their children and comparing
them to samples from women with healthy children and low levels of stress. They
found that stress promotes
cellular aging by weakening cell structures known as telomeres, which affect
cellular regeneration and how our
cells age. As the study revealed, the greater the stress, the greater the
degenerative effect on cells.
These days, people live at a frantic pace and in a nearly constant state of
competition. At this fever pitch, stress
is a natural response to the information being received by the body as potentially
dangerous or problematic.
Theoretically, this is a useful reaction, as it helps us survive in hostile
surroundings. Over the course of our
evolution, we have used this response to surroundings. Over the course of our
evolution, we have used this
response to deal with difficult situations and to flee from predators. The alarm
that goes off in our head makes
our neurons activate the pituitary gland, which produces hormones that release
corticotropin, which in turn
circulates through the body via the sympathetic nervous system. Nonetheless, the
stress to which human beings
are subjected today is clearly harmful. Stress has a degenerative effect over time.
A sustained state of emergency
affects the neurons associated with memory, as well as inhibiting the release of
certain hormones, the absence
of which can cause depression. Its secondary effects include irritability,
insomnia, anxiety, and high blood

. Page 5 of 36
pressure. As such, though challenges are good for keeping mind and body active, we
should adjust our high#stress lifestyles in order to avoid the premature aging of
our bodies.
11. Which of the following reflects the main objective of the passage?
(a) To make aware the readers the path to a healthy life by adjusting high-stress
situations to avoid the premature
aging of our bodies.
(b) To help readers combat high stress lifestyles in order to avoid premature aging
of our bodies.
(c) To Highlight the factors leading to premature aging of our body.
(d) To clear misconception surrounding stress, especially when stress is important
for survival.
12. With reference to the passage, what does the expression “frantic pace” mean?
(a) Peaceful activities.
(b) Anxiety-driven activities.
(c) Overzealous activities.
(d) Leisurely activities.
13. “Challenges are good to keep mind and body active”. Which among the following
supports the statement?
(a) Challenges increase the ability to fight and sometimes lead to a difficult path
in life.
(b) Challenges make one sick and hyper leading to more stress.
(c) The change that comes along challenges motivates people and increases their
capacity to work optimally.
(d) To some extent, challenges help to keep the mind and body nimble, and this
helps us to survive or evolve
with the situation.
14. From the passage, it can be inferred that the author is
I. A reporter.
II. A researcher.
III. A biologist.
IV. A scientist.
(a) I & II (b) II & IV (c) Only III (d) Either II or IV
15. What is the tone of the author in the passage?
(a) Quixotic (b) Pedestrian (c) Analytical (d) Commiserating
Passage (Q.16-Q.20): The COVID-19 pandemic has triggered new ways of learning. All
around the world,
educational institutions are looking toward online learning platforms to continue
with the process of educating
students. The new normal now is a transformed concept of education with online
learning at the core of this
transformation. Today, digital learning has emerged as a necessary resource for
students and schools all over the
world. For many educational institutes, this is an entirely new way of education
that they have had to adopt.
Online learning is now applicable not just to learn academics but it also extends
to learning extracurricular
activities for students as well. In recent months, the demand for online learning
has risen significantly, and it
will continue doing so in the future.
For many students, one of the biggest challenges of online learning is the struggle
with focusing on the screen
for long periods of time. With online learning, there is also a greater chance for
students to be easily distracted
by social media or other sites. Therefore, it is imperative for the teachers to
keep their online classes crisp,
engaging, and interactive to help students stay focused on the lesson.
Another key challenge of online classes is internet connectivity. While internet
penetration has grown in leaps
and bounds over the past few years, in smaller cities and towns, a consistent
connection with decent speed is a
problem. Without a consistent internet connection for students or teachers, there
can be a lack of continuity in
learning for the child. This is detrimental to the education process.
Students can learn a lot from being in the company of their peers. However, in an
online class, there are minimal
physical interactions between students and teachers. This often results in a sense
of isolation for the students. In

. Page 6 of 36
this situation, it is imperative that the school allow for other forms of
communication between the students, peers,
and teachers. This can include online messages, emails and video conferencing that
will allow for face-to-face
interaction and reduce the sense of isolation.
Online learning requires teachers to have a basic understanding of using digital
forms of learning. However, this
is not the case always. Very often, teachers have a very basic understanding of
technology. Sometimes, they
don’t even have the necessary resources and tools to conducts online classes. To
combat this, it is important for
schools to invest in training teachers with the latest technology updates so that
they can conduct their online
classes seamlessly.
Many parents are concerned about the health hazards of having their children spend
so many hours staring at a
screen. This increase in screen time is one of the biggest concerns and
disadvantages of online learning.
Sometimes students also develop bad posture and other physical problems due to
staying hunched in front of a
screen.
16. Which of the following can be concluded from the passage?
(a) Online education is the newest way of learning after COVID-19
(b) Online education is the only way left for uplifting the education system
(c) Online education has various disadvantages that has to be gradually sorted.
(d) Both A and C
17. Which of the following gives the most appropriate meaning of the word, hunched?
(a) A supine position that causes bending of the spinal curvature.
(b) To squat in a manner that one’s body is bent backwards.
(c) A position where the back is humped in a convex because of abnormal spinal
curvature
(d) A position where the spinal curvature is straight, and the neck is slightly
bent.
18. Which among the following is not the disadvantage of online education?
(a) Problem with internet connectivity.
(b) Less efficiency in teaching skills.
(c) Easily accessible from every part of the world.
(d) Health hazards for children.
19. “Online education is causing a major feeling of isolation” what does this refer
to?
(a) people are getting more open towards education.
(b) Students feel estranged from the company of their peers.
(c) Online education has lessened the social interaction of students.
(d) One of the advantages of online education.
20. What can be the best title for the passage?
(a) Tread cautiously with online education.
(b) Do away with the old, get on with the new.
(c) Online education: a new fad.
(d) Online education: Teachers’ nightmare.

. Page 7 of 36
Passage (Q.21-Q.25): Soon after World War II, Winston Churchill was visiting the
White House when he is
said to have had an uncanny experience. Having had a long bath with a Scotch and
cigar, he reportedly walked
into the adjoining bedroom – only to be met by the ghost of Abraham Lincoln.
Unflappable, even while
completely naked, Churchill apparently announced: “Good evening, Mr President. You
seem to have me at a
disadvantage.” The spirit smiled and vanished.
His supposed contact with the supernatural puts Churchill in illustrious company.
Arthur Conan Doyle spoke to
ghosts through mediums, while Alan Turing believed in telepathy. Three men who were
all known for their
razor-sharp thinking, yet couldn’t stop themselves from believing in the
impossible. You may well join them.
According to recent surveys, as many as three quarters of Americans believe in the
paranormal, in some form,
while nearly one in five claim to have actually seen a ghost.
Intrigued by these persistent beliefs, psychologists have started to look at why
some of us can’t shake off old
superstitions and folk-lore. Their findings may suggest some hidden virtues to
believing in the paranormal. At
the very least, it should cause you to question whether you hold more insidious
beliefs about the world.
Some paranormal experiences are easily explainable, based on faulty activity in the
brain. Reports of poltergeists
invisibly moving objects seem to be consistent with damage to certain regions of
the right hemisphere that are
responsible for visual processing; certain forms of epilepsy, meanwhile, can cause
the spooky feeling that a
presence is stalking you close by – perhaps underlying accounts of faceless “shadow
people” lurking in the
surroundings.

Out-of-body experiences, meanwhile, are now accepted neurological phenomena, while


certain visual illusions
could confound the healthy brain and create mythical beings. For example, one young
Italian psychologist looked
in the mirror one morning to find a grizzled old man staring back at him. His later
experiments confirmed that
the illusion is surprisingly common when you look at your reflection in the half
light, perhaps because the brain
struggles to construct the contours of your face, so it begins to try to fill in
the missing information – even if that
leads to the appearance of skulls, old hags or hideous animals.
https://www.bbc.com/future/article/20141030-the-truth-about-the-paranormal
21. What is the main reason behind the occurrence of a paranormal experience?
(a) Aberrant activity in brain.
(b) Neurological occurrence.
(c) Illusions created by the eyes.
(d) People’s firm beliefs in old folklores.
22. According to the passage, which of the following men has had a visual illusion?
(a) Alan Turing (b) Abraham Lincoln
(c) Winston Churchill (d) Arthur Doyle
23. After reading the passage, determine the belief of author in the existence
supernatural elements?
(a) Firm believer. He believes in them and regards them as part of real world.
(b) Sceptic. He is unclear towards his stand.
(c) Rejects this idea.
(d) Cannot be determined.
24. ‘Some paranormal experiences are easily explainable, based on faulty activity
in the brain.’ Identify the part of
speech of the above underlined word-
(a) Noun (b) Verb (c) Adverb (d) Adjective

. Page 8 of 36
25. Unflappable, even while completely naked, Churchill apparently announced: “Good
evening, Mr President. You
seem to have me at a disadvantage.” The spirit smiled and vanished. Give the
synonym of the word in bold –
(a) Befuddled. (b) Imperturbable. (c) Regaled. (d) Disconcerted.
Passage (Q.26-Q.30): Inside our body can be found the bloodiest of battlefields,
where millions of organisms
are massacred daily, without cease. It is a battle waged by our body's robust
immune system against a wide
variety of pathogenic bacteria, virus, fungi, and parasites. What makes the defence
mechanism powerful is the
two-level protection conferred by the immune system. The innate immune system that
serves as the first line of
defence is not antigen-specific; it readily targets all pathogenic organisms the
moment they enter the body. The
antigen-specific adaptive immune mechanism acts as the second line of protection to
keep us healthy. This year's
Nobel Prize in Physiology or Medicine has been awarded to Bruce A. Beutler, Jules
A. Hoffmann, and Ralph
M. Steinman for revolutionising our understanding of the immune system by
discovering the key principles that
activate the defence mechanism. Beutler and Hoffmann will share half the prize
money for discovering the
receptor proteins that recognise micro-organisms and activate the innate immunity.
In 1996, Hoffmann found
that the Toll gene was responsible for sensing pathogenic micro-organisms and that
its activation was required
for mounting innate immune response. Two years later, Beutler discovered that
components of micro-organisms
bind to Toll-like receptors located on many cells. The binding activates the innate
immunity, which results in
inflammation and destruction of the pathogens.
The other half of the prize money was awarded to Steinmann for discovering, way
back in the 1970s, that
dendritic cells were responsible for adaptive immunity. As they are antigen-
specific, dendritic cells take time
to react to an invading organism on first exposure; but immunological memory allows
them to react more
rapidly to the same antigen on subsequent exposures. This is the attribute
researchers exploit while designing
preventive vaccines. Adaptive immunity holds great medical promise. The immune
system can be directed to
attack the tumour. Blocking the excessive production of cytokines when diseases
show up can ameliorate
autoimmunity. Even preventing autoimmune diseases may become possible when certain
cells of the immune
system are successfully silenced. Steinmann will go down in history as not just a
highly worthy Nobel Prize
winner. He was (as a Rockefeller University statement explains) "diagnosed with
pancreatic cancer four years
ago ... his life was extended using a dendritic-cell based immunotherapy of his own
design," and he died three
days before his Nobel was announced.
26. The primary objective of the passage is
(a) To explain the reason behind the success of Defence mechanism in the body.
(b) To expatiate that the antigen specific immune system acting as a second line of
protection works better as
a defence mechanism than innate immune system acting as a first line of defence
mechanism
(c) To bring to fore the injustice meted out to the Nobel Prize winners, Bruce A.
Beutler and Jules A
Hoffman, by attenuating their works and giving more emphasis to Ralph M Steinman.
(d) To highlight the works of the Nobel Prize winners (in their field) who gave a
new direction to our
understanding, by discovering the key principle that activate the defence mechanism
and where one
scientist corroborated his theory by applying it on himself.
27. The statement which is not in concord with the text is,
(a) Ralph M. Hoffman found that the Toll gene was responsible for sensing
pathogenic micro-organism and
that it was required for mounting innate immune response.
(b) Steinman was awarded the Nobel Prize posthumously
(c) The innate immune system that serves as the first line of defence is not
antigen- specific
(d) Steinman, in 1970, discovered that dendritic cell was responsible for adaptive
immunity.
28. "Even preventing autoimmune……..are successfully silenced". The word silenced as
used in the passage, means
(a) Dead (b) Dormant (c) Resurged (d) Suppressed.

. Page 9 of 36
29. According to the passage, Dendritic cells are
(a) Pathogen-specific (b) Antigen-specific
(c) Autoimmune (d) Parasites
30. The tone in the passage, is;
(a) Laudatory (b) Belligerent (c) Neutral (d) Satirical

Directions (Q.66 – Q.105): Read the comprehensions carefully and answer the
questions based on it.
Passage (Q.66-Q.71): High Court stated that it is trite law that even if the weapon
of offence is shown after
snatching had taken place for running away along with snatched article, an offence
under Section 397 IPC is
attracted and if the offender fails to acquire the property, it will be an attempt
to robbery.
Section 390 CrPC provides that in a robbery, there is either theft or extortion. It
is further provided that theft is
‘robbery’ if, in order to committing of the theft or in committing the theft, or in
carrying away or attempting to
carry away property obtained by theft, the offender, for that end, voluntarily
causes or attempts to cause to any
person death or hurt or wrongful restraint, or fear of instant death or of instant
hurt or of instant wrongful restraint.
When the offender uses deadly weapon while committing robbery or dacoity, he is
punished under Section 397.
Extortion is “robbery” if the offender is in the presence of the person put in
fear, and commits the extortion by
putting that person in fear of instant death, of instant hurt, or of instant
wrongful restraint to that person or to
some other person, and, by so putting in fear, induces the person, so put in fear
then and there to deliver up the
thing extorted.
The Bench stated that it is not essential that weapon of offence should be
recovered to prove the nature of the
weapon used and that a deadly weapon was used at the time of the commission of the
offence, however, the
prosecution was required to prove the nature of the weapon of offence used
especially in the case of knife or
blade.
Court held that since from the evidence of the prosecution witnesses the size and
sharpness of the blade was not
proved, the prosecution failed to prove that the appellant used a deadly weapon.
(Extracted with requisite revisions and edits from ‘Whether a ‘blade’ would be
covered under S. 397 IPC as a
deadly weapon? Del HC explains in view of settled position of law’by Devika
Sharmaat
https://www.scconline.com/blog/post/2022/02/04/whether-a-blade-would-be-covered-
under-s-397-ipc-as-a#deadly-weapon/)
66. In reference to above stated passage, which is true about deadly weapon:
(a) Knife is a deadly weapon.
(b) Deadly weapon must be used for commission of Robbery.
(c) Weapon which is deadly must be produced before court.
(d) Some weapons become deadly depending on its nature.
67. Mahesh went to Rakesh and asked him for money so that he can pay the expenses
for his mother’s medical
treatment. Rakesh denied giving him the amount and asked to leave the place. Mahesh
later on calls Rakesh and
says that he will injure him badly if he does not give him the money. He says that
his friend is present in his
vicinity and has a knife with which he will hurt him. Decide whether Mahesh is
guilty of his act:
(a) Mahesh is guilty for Robbery under Section 392.
(b) Mahesh is not guilty for Robbery.
(c) Mahesh is guilty under Section 397.
(d) Mahesh did not injure Rakesh and did not commit any offence.
68. Mohan entered into the house of Raka. He saw that nobody is the home and stole
jewelry from his house. While
he was running away with the jewelry Raka saw him. He tried to stop Mohan but
failed to do so. To stop Raka,
Mohan kept the jewelry aside on the road and picked up a rock and threw it at Raka.
The stone did not hit Raka.
Decide whether Mohan is guilty:
(a) It is not robbery as Mohan kept the stolen jewelry aside.
(b) Mohan just committed theft as he stole the Jewelry from Raka.
(c) Mohan committed robbery as he was running away with stolen jewelry.
(d) It is punishable under Section 397 as Mohan used stone which is a deadly
weapon.
. Page 17 of 36
69. A sees that B is going to market is carrying money with him. A wanted to get
the money from B and pushed B
down asking him for money. He says that B should deliver the money to him and then
only he will allow him to
go. While A was holding him down him down, B sees the knife that A was carrying in
his pocket. B somehow
loosens the grip of B and runs away. B later on files a complaint against A. Decide
for which act can A be
punished:
(a) A is guilty of attempt to robbery.
(b) A is guilty of robbery.
(c) A is guilty of robbery with deadly weapon.
(d) A is guilty of extortion.
70. X approaches Y with a gun in his pocket and tells Y that he does not give all
his valuables to X, he will shoot
him. He tells that Y he has a gun with him. Y was scared and therefore, unwillingly
handed over his property. X
went away after taking all the property from Y. Y saw Z around and asked for his
help to catch X who was
running away. When Z runs towards X, he took out his gun and pointed it towards Z
and threatened to shoot
him. Decide the offence for which X is guilty:
(a) He is guilty of Robbery with deadly weapon as he used the gun.
(b) He is guilty of Robbery done in furtherance of extortion.
(c) He is guilty of robbery done in furtherance of theft.
(d) He is not guilty of robbery as he did not use the gun to threaten Y to give up
his valuables.
71. In furtherance of above passage, decide which is false in relation of robbery:
(a) Robbery can be committed even without fear of instant death, instant hurt or
instant wrongful restraint.
(b) Theft or extortion is a pre-requisite for commission of robbery.
(c) A person must be put in fear while committing robbery by extortion.
(d) While committing Robbery in Section 397 weapon should be used before or at the
time of stealing the
property.
Passage (Q.72-Q.77): “Every agreement by which any one is restrained from
exercising a lawful profession,
trade or business of any kind is to that extend void. There is one exception to
this rule-that if the goodwill of a
business has been sold, an agreement to refrain from carrying on similar business,
if it appears to the Court to be
reasonable, would be protected and would be enforced.”
In Supreme Court’s decision of Superintendence Company of India v. Krishna Murgai,
the Supreme Court while
discussing the objective behind Section 27 of the Act analyzed the difference of
negative covenant between an
employer-employee and a seller-purchaser and stated that a negative covenant
between the employer – employee,
pertains to performance of personal service which is altogether different in
substance from purchase and will
have vastly different social and economic implications. The essential line of
distinction is that the purchaser is
entitled to protect himself against competition on the part of his vendor, while
the employer is not entitled to
protection against mere competition on the part of his servant. A restrictive
covenant ancillary to a contract of
employment is likely to affect the employee’s means or procuring a livelihood for
himself and his family to a
greater degree than that of a seller, who usually receive ample consideration for
the sale of the goodwill of his
business.
An employment contract generally includes restraint of trade clause to protect the
interest of the employer after
an employee leaves their organization or business for the following reasons:
 Non-compete clause
 Non-solicitation clause
 Non-recruitment clause
 Confidentiality clause
In Independent News Service (P) Ltd. v. SucheritaKukreti, in the context of Section
27 of the Contract Act, it
was held that the right saved thereby to be a facet of Article 21 of the
Constitution of India.

. Page 18 of 36
Extracted with requisite revisions and edits from ‘For a contract to be
enforceable, restraint of trade clause
must be reasonable: Post-termination non-compete clauses are permissible in
employment contracts under S.
27 of Contract Act? District Court explains’ by Devika Sharma
athttps://www.scconline.com/blog/post/2022/02/03/for-a-contract-to-be-enforceable-
restraint-of-trade-clause#must-be-reasonable/)
72. M sold unlicensed guns for his livelihood. He used to check the details,
documents and took other necessary
precaution before selling the gun to a person. His mother was worried that such a
business might threaten the
safety of M. M explained to his mother that he sells the gun after doing a
background check. His mother did not
listen to him and asked him to sign a contract with her promising that he will stop
selling gun. M unwilling
signed the contract due to the pressure from his mother. Decide whether the
contract is valid:
(a) No, as the contract restrict M from selling gun and therefore, is void under
Section 27.
(b) No, as no contract was formed as M did not give free consent and signed
contract due to the pressure from
her mother and therefore, is void.
(c) Yes, as M’s mother just stopped his from selling gun and did not stop him from
doing any other trade.
(d) Yes, as the trade of M is not lawful.
73. Mehul is an advocate and he was hired by Tarun as his advocate. Tarun was very
concerned about his privacy
and did not want his information to be leaked. He asks Mehul to sign a contract to
act as his advocate and to not
work as the advocate of any person in future. Decide the validity of the contract:
(a) The contract is void.
(b) The contract is valid as Tarun is just trying to protect his interest.
(c) The contract is void to the extent on restricting Mehul.
(d) The contract can be declared void by Mehul.
74. In accordance with above stated passage, decide which statement is true:
(a) Trade, profession and business cannot be restricted at all.
(b) The restriction imposed on trade must be reasonable.
(c) Imposition of restriction does not violate any right.
(d) The restriction imposed must be reasonable to the parties to contract.
75. John was hired by a multi-national company after his graduation. He signed a
contract with the company without
going through the contract completely. One of the terms of the contract was that
John cannot ever work in the
same line of business as his company after he leaves the job. When John learned
about the term of the contract,
he was furious. He contended that the contract is void as the condition restricted
his profession. Decide the
validity of the contract:
(a) John has already signed the contract and will be valid.
(b) The contract is void as it restricted the profession of John.
(c) The contract is valid as the company as the employer can impose such
restriction.
(d) The contract is void as the restriction imposed by company is unreasonable.
76. Which of the following is true about restriction imposed by employer on
employee:
(a) Employer cannot restrict his present employees.
(b) Restriction must be reasonale.
(c) Employer cannot restrict regarding confidential information
(d) Employer can impose restriction by a contract after the person has stopped
being the employee.

. Page 19 of 36
77. Decide which is true about imposition of restriction by employer on employee
and seller on purchaser:
(a) The degree of affect on the interest of parties can be ground to distinguish
employer-employee contract from
seller-purchaser contract.
(b) Competition can be involved as a sole ground of restriction by the employer.
(c) The restriction by seller creates a positive right in favor of purchaser.
(d) There exists no social and economical implication in restriction imposed by
purchaser on seller as he is
getting consideration for the goodwill of his business.
Passage (Q.78-Q.83): The Supreme Court's judgment in Gargi v. State of Haryana
which had elaborated on
circumstantial evidence, which is not the first choice of evidence, stating that
"circumstantial evidence is the one
whereby other facts are proved from which the existence of the fact in issue may
either be logically inferred or
at least rendered more probable."
Furthering referring to the Apex Court's judgment in Sharad Birdhichand Sarda v.
State of Maharashtra where
the panchsheel principles pertaining to acceptance of circumstantial evidence were
laid down, the bench
reiterated those
 The circumstances from which the conclusion of guilt is to be drawn should be
fully established
 The facts so established should be consistent only with the hypothesis of the
accused's guilt (and no other)
 Circumstances should be of conclusive nature and tendency
 Such circumstances should exclude every other hypothesis
 The chain of evidence must be so complete that it leaves no reasonable ground to
conclude or even point to
the innocence of the accused and must show in all human probability that the
offence has been committed by the
accused.
Further holding that "extra-judicial confession cannot be the sole basis of
conviction and cannot be relied on
when surrounding circumstances are improbable and create suspicion", the bench
noted that since the chain of
events does not lead to the result of accused committing the crime of murder, the
trial court had erred in
convicting the accused and the conviction of the accused was set aside and
miscellaneous applications pending
stood closed.
(Extracted with requisite revisions and edits from ‘Judges To Tread Cautiously In
Circumstantial Evidence,
Can't Allow Conjectures & Suspicion To Take Place Of Proof: Telangana High Court'
by Rashmi Bagriat
https://www.livelaw.in/news-updates/telangana-high-court-circumstantial-evidence-
proof-judges-to-tread#cautiously-191742)
78. In furtherance of above passage, decide the correct statement relating to
circumstantial evidence:
(a) Circumstantial evidence is relied upon when there is no direct evidence.
(b) Circumstantial evidence is needed in every case irrespective of the
availability of direct evidence.
(c) A person cannot be convicted solely on the basis of circumstantial evidence.
(d) The court cannot deny accepting circumstantial evidence.
79. The case before the court is that whether Mahesh killed Rakesh or not. Rakesh’s
body was found after he was
shot. There exists no eye witness to prove the same. The police find that Mahesh
was present at the same place
where Rakesh’s dead body was found. It was also found that Mahesh had made an
attempt in the past to injure
Rakesh and wanted to take revenge from Rakesh. The police also found that Mahesh
called Rakesh before his
death and also went to market to purchase a knife. Decide whether there exists
circumstantial evidence to convict
Mahesh:
(a) Yes, as the circumstances establish a chain of evidence which prove that Mahesh
is guilty.
(b) Yes, as the Mahesh being present at the same place fully establish guilt
conclusively.
(c) No, as there exist discrepancy in the chain of evidence and therefore, it
creates a doubt.
(d) Yes, as Mahesh made attempt in past to injure Rakesh which proves his intention
for murder.

. Page 20 of 36
80. In accordance with the above passage decide which statement is true:
(a) Circumstantial evidence creates no connection in between the fact in issue and
the facts proved by it.
(b) For court to rely on circumstantial evidence just proving any of the facts is
enough.
(c) Circumstantial evidence establishes a direct relation between proving a fact
and existence of fact in issue.
(d) Circumstantial evidence establishes an inverse relation between proving a fact
and existence of fact in issue.
81. The police is investigating the theft at X’s house. The police find that Z and
Y were aware of the fact that X is
not at his place. Z has a criminal record of theft in past. The police are sure
that the theft was committed by a
single person. Z was aware of the fact that where X has stored his valuables at his
house. Y was nearby X’s place
at the time of theft. However, there is no evidence to show that Y entered X’s
house. Decide who can be guilty
for theft:
(a) Z as he has past criminal record and Z was aware of X’s house.
(b) Z and Y both can be the accused due to which none of them can be deemed guilty.
(c) Y as guilty as he was aware of X’s house and was present nearby at the time of
theft.
(d) Z and Y both are accused and therefore, both are guilty.
82. Arun is suspected to have killed Hemant. The facts show that Arun is a person
of good character and have always
regarded Hemant as his brother. Arun was found to be having bought knife from a
store nearby. When Arun was
returning back to his house his neighbor found that his clothes were covered in
blood and Arun was very anxious.
Arun later on burnt his clothes late at night which was seen by his neighbor.
Decide whether there exist evidence
to prove Arun’s guilt:
(a) No, as the circumstances show that Arun treated Hemant like his brother and
therefore, his guilt cannot be
conclusively established.
(b) No, as the circumstances give rise to two hypothesis one of the guilt of Arun
and another of innocence of
Arun.
(c) No, as there is absence of chain of evidence to prove guilt of Arun beyond
reasonable doubt.
(d) Yes, as the circumstances prove beyond reasonable doubt the guilt of Arun.
83. In furtherance of above stated passage decide the false statement regarding
circumstances:
(a) The circumstances can relate to anything to prove any possibility.
(b) Circumstances should show the absence of all other possibilities.
(c) Circumstances relevant to fact in issue are to be considered for establishing
guilt.
(d) Circumstances must be consistent with the confession so that it can be relied
upon.
Passage (Q.84-Q.89): Commission noted that as per the Supreme Court decision in
DrMukhtiar Chand v. State
of Punjab, Ayurveda, Siddha, Unani and Homeopathy practitioners can prescribe
allopathic medicines only in
those states where they are authorized to do so by a general or specific order made
by the State Government
concerned.
Further, the Coram expressed that, Few State Governments have authorized AYUSH
doctor(s) it by some special
order(s) to prescribe medicines of allopath, but in our view, that does not
authorize the doctor to deviate from
the standard of care which results into wrong diagnosis and prescribe wrong
medicines.
Coram relied on the Supreme Court’s decision in PoonamVerma v. Ashwin Patel,
wherein it was held that “the
doctor must not only be qualified, but he must also be registered with the
appropriate Medical Council in order
to practice as a doctor. A homoeopath would not have knowledge about allopathic
medicines and its drug actions,
so administration of allopathic treatment by a homoeopath would be proof enough to
establish negligence”.
Coram noted the 4 points on which the OP 2 failed:
 Failure to correctly diagnose the condition as SJ syndrome but treated the
patient for Measles.
 The dose of steroid 4mg Dexa was inadequate for the patient; the dosage should be
calculated as per kg of
body weight.
 For treatment of SJ syndrome the appropriate drugs of choice would be
Cyclosporine with
Methylprednisolone and the dosage to be per kg body weight of the patient.

. Page 21 of 36
 There was a delay and wrong referral of the patient to the physician instead of a
skin specialist.
Therefore, the patient deserved enhanced just and reasonable compensation.
(Extracted with requisite revisions and edits from ‘Minor treated for “Measles”
instead of “Stevens-Johnson
Syndrome” due to wrong diagnosis and leading to medical negligence: Read detailed
report on NCDRC’s
decision’ by Devika Sharmaathttps://www.scconline.com/blog/post/2022/01/28/medical-
negligence-6/)
84. Alok is a 27 years old man. He has certain medical condition due to which he is
unable to perform his daily
activities. He consults Mahesh who is an expert in ayurvedic medicine. Mahesh
prescribed him certain
ayuervedic medicines. He further said that if Alok does not get relief from the
medicine, he should take some
antibiotic to relieve his pain. Mahesh suggested the names of the antibiotics to
Alok orally and told him that he
should once talk to his doctor once before talking the medicine. Decide whether act
of Mahesh is correct:
(a) No, as he acted negligently.
(b) Yes, as Mahesh just wanted to relive the pain of Alok and acted in good faith.
(c) No, as he should not prescribe antibiotic to Alok.
(d) Yes, as he just made a suggestion did not prescribe the antibiotic to Alok.
85. In the last question, if Mahesh wrote on a slip prescribing certain ayuervedic
medicine and an antibiotic. Mahesh
prescribed it in good faith without any intent to cause him harm and to the best of
his knowledge. Decide whether
the act of Mahesh is correct in accordance to the general tone of the passage
considering his State has not allowed
the same:
(a) Yes, as Mahesh acted in good faith.
(b) No, as Mahesh is not authorized to prescribe antibiotic.
(c) Yes, Mahesh acted as per his knowledge and to benefit Alok.
(d) No, as Mahesh should have gotten consent of Alok.
86. In accordance with the above passage, decide which statement is correct in
regard to medical negligence:
(a) It can be only punished by imprisonment.
(b) Medical negligence will not occur when state government authorizes anayuerveda
practitioner.
(c) Medical negligence may occur even after authorization from state government to
the practitioner.
(d) It won’t occur when practitioner has knowledge.
87. A is a very bright student and wanted to become a doctor. He got admitted into
the medical school. One day
while he was on his way to college he saw that a person fainted and to make the
person recover he administered
a medicine to him. Later on the patient suffered from side effects due to the
medicine and he sued A for medical
negligence. Decide whether the patient would succeed:
(a) Yes, as A is not a registered doctor.
(b) No, as A administered him medicine to save him.
(c) No, as A did not prescribe medicine to the patient.
(d) No, as A acted best to his ability.
88. Mukund is a homeopathy practitioner in state of V. The state published a
general order allowing homeopathy
practitioner in the state to prescribe allopathic medicine to their patients.
Mukund never studied about allopathic
medicine and did not have knowledge about it. He prescribed allopathic medicine to
one of his patients. The
patient suffered after consuming the medicine and sued Mukund for medical
negligence. Decide whether the
patient would succeed:
(a) Yes, as Mukund if a homeopathy practitioner.
(b) Yes, as Mukund had no knowledge about allopathic medicine.
(c) No, as state government authorized Mukund to prescribe allopathic medicine.
(d) No, as the patient consumed the medicine with his choice.

. Page 22 of 36
89. In the last question, if Mukund is of 29 years old and government order
authorized the practitioner above 30
years. Mukund was aggrieved by the order of government and challenged it. Whether
government make such an
order:
(a) No, as government cannot just allow limited people.
(b) Yes, as government can do anything they wish.
(c) No, as government cannot authorize a homeopathy practitioner to do something
they have not studied.
(d) Yes, as the government can make specific order.
Passage (Q.90-Q.95): Section 52 incorporates the doctrine of lispendens and
stipulates that during pendency of
any suit or proceeding in which any right to immoveable property is, directly or
specifically, in question, the
property, which is the subject matter of such suit or proceeding cannot be
transferred or otherwise dealt with, so
as to affect the rights of any other party to such a suit or proceeding. It is to
maintain the status quo unaffected
by the act of any party to the litigation pending its determination.
Order 39 Rules 1 and 2 deals with cases in which temporary injunction may be
granted. It is a discretionary relief
exercised by the courts.
The distinction between the two is that an order of temporary injunction is pre-
emptive in nature restraining the
act of alienation by party to the suit where there is such a danger, whereas
Section 52 T.P. Act comes into play
after the alienation takes place during pendency of the suit. If an order of
temporary injunction is passed and
transfer is restrained, the question of applicability of Section 52 of T.P. Act
will not arise as then there will be
no transfer pending litigation.
In view of the primary object of grant of temporary injunction to maintain the
status quo till the adjudication of
the rights of the litigating parties, the Court held that Section 52 of TP Act does
not operate as a bar to grant of
temporary injunction under Order 39 Rules 1 & 2 CPC, in the discretion of trial
court, on fulfillment of pre#conditions (prima facie, balance of convenience and
causing irreparable loss or injury in his favour) for grant of
temporary injunction, restraining alienations as well.
(Extracted with requisite revisions and edits from ‘Section 52 Transfer Of Property
Act Does Not Bar Temporary
Injunction Against Alienation Of Property: Andhra Pradesh High Court’ by
JagritiSanghi at
https://www.livelaw.in/news-updates/andhra-pradesh-high-court-section-52-transfer-
of-property-act-bar-to#grant-of-temporary-injunction-190679)
90. Mehul and Manisha were indulged in a suit for partition of their father’s
property. The general practice in their
family is that the daughter does not acquire the property of the father. Mehul
wanted some money so that he
could pay for certain expenses of his deceased father. Mehul thought of selling the
house which has been
succeeded from their father. Mehul sold the house and Manisha challenged the act of
Mehul. Decide whether
Manisha would succeed:
(a) Yes, as during pendency of suit Mehul cannot sell the house.
(b) No, as since Mehul sold the house to pay expenses of his deceased father.
(c) No, as Mehul had right over the house.
(d) No, as in accordance with general practice Manisha cannot acquire property of
her father.
91. Nitin and Yash have a suit going on between them regarding a land and garden.
The court is hearing the suit on
daily basis and is trying to resolve the dispute as soon as possible. Yash had
right over the garden concerned.
Mahi learnt that Nitin who has the possession of the property was disposing the
garden. Mahi was not having
any right on the garden but filed a case challenging the transfer. Decide whether
Mahi would succeed:
(a) Yes, as Mahi’s right has been affected.
(b) No, as Mahi can file a suit later on.
(c) Yes, as transfer was made during pendency of suit.
(d) No, as Mahi is not a party to the suit.

. Page 23 of 36
92. In reference to the above passage, decide which of the following the correct
relationship between Section 52 and
Order 39:
(a) They both are contradictory to each other.
(b) They are complementary to each other.
(c) They both cannot be claimed as a relief is a same suit.
(d) They both can be claimed as a preventive measure.
93. Anand and Renu are a married couple. They planned to get a divorce and filed a
suit for divorce. At the same
time Renu filed a suit claiming the ownership of car which was in the possession of
Anand. Anand did not want
Renu to get the car and therefore, intentionally sold the car to another person.
Renu filed a case challenging the
sale under Section 52. Decide whether she would succeed:
(a) No, as Renu should wait for the case of divorce to be decided before filing the
case for ownership of car.
(b) Yes, as transfer was made during pendency of suit.
(c) Yes, as transfer was made to intentionally cause injury to Renu’s right.
(d) No, as the property involved in the pending suit is car.
94. A and B has a suit going in between them regarding a land title. A contended
that he rented the land to B. When
he demanded the land back from B, he was denied due to which he filed a case. A
also applied to court to grant
temporary injunction against B restraining him from entering on the land. A
presented the document in relation
to the land to depict his ownership. A contended that if he is not granted
injunction, he will lose his source of
income. He also notified court that B has acquired the income arising from the
land. Decide whether Court can
grant temporary injunction:
(a) No, as there is no prima facie case.
(b) Yes, as all the requisites has been fulfilled.
(c) No, as there in no balance of convenience in favour of A.
(d) No, as no injury is caused to A.
95. In accordance with above passage which statement is correct regarding temporary
injunction:
(a) It is a matter of right which Court must grant.
(b) Temporary injunction makes changes in the rights of parties.
(c) It is granted after transfer has taken place.
(d) Court must be satisfied that all the pre-requisites of temporary injunction
have been fulfilled before granting
it.
Passage (Q.96-Q.100): Ms. Tanu Bedi, Counsel for appellant had vehemently argued
that the defense counsel
was not given any opportunity to cross-examine the victim in the trial court and
thus, the testimony of the victim
could not be relied on for convicting the accused-appellant. In reply the learned
Counsel stated that they "had
slept over their right to do so" as they did not submit any questionnaire at the
time of examination of the victim,
nor did they make an application for cross-examination. They did not even raise any
objection and argument
regarding the same during the trial either.
After scrutinizing the submissions made, the Court had to look into question:
Whether the testimony of a child
be the sole reason for the conviction of the accused-appellant? The Court then
proceeded to peruse S.33 of
POCSO Act, noting the legislature's wisdom of providing a special, specific
procedure for recording the evidence
of child witnesses and clarified that "in cases of sexual assault against Children,
the first and most important
piece of evidence is always the statement of the child victim Themselves."
Referring to the apex court's judgments in Sanjay Kumar Valmiki v. State which held
that if the trial court is
satisfied that the child witness before it is unlikely to be tutored and has
deposed of their own will and volition,
then the evidence given by such child witness should be treated with the same
regard as that of any other witness
as given in Sec 118, the High Court averred that there is no embargo on child
testimony in S.118 of Evidence
Act, that talks about ‘who may testify’ and the duty is cast on the courts
regarding allowing or disallowing such
evidence on the grounds that child does not understand the questions put to him or
to provide rational answers

. Page 24 of 36
to such questions and if the answers to these questions are in negative then there
is no justification to disregard
such testimony. Thus, the court stated that the Child witness's testimony was
substantial and enough under S.118
of the Evidence Act.
(Source : No Embargo In Treating Testimony Of A Child Witness As The Sole Basis For
Conviction: Punjab
And Haryana High Court, livelaw)
96. Ramu is a ten-year-old boy who is unable to speak. Every morning, he goes with
his mother to get water from
a nearby well. When they both went to the well one day, they encountered an
impediment in the form of a large
tree that had fallen owing to severe rain. She told Ramu to stay put until she
returns. Ramu eventually heard his
mother's screams from the other end, but due to the tree blocking his view, he
couldn't see what was going on.
Though he heard his uncle Chetan's voice, he couldn't be sure because he couldn't
see his face. Raju was
summoned by the court since he was the only witness present at the scene. Ramu
heard everyone saying that
chetan might have killed his mother. Chetan killed Ramu's mother, according to
Ramu's testimony. Is the court
likely to accept Ramu's statement under section 118 of the Evidence Act?
(a) The court will not allow Ramu’s testimony as he can’t speak.
(b) The court will not allow Ramu’s testimony as the statement was tutored due to
his tender age.
(c) The court will allow the testimony of Ramu as he testifies in her testimony
Chetan’s criminal liability.
(d) The court will allow Ramu’s testimony as he heard Chetan’s voice at the crime
scene.
97. In the same circumstances as above, what procedure will the court employ to
extract Ramu’s testimony?
(a) Direct and to-the-point questions are preferred.
(b) The question should be phrased in a way that the child witness can understand.
(c) The questions must be put forth in form of yes or no.
(d) When obtaining the evidence of a child witness, the court must ensure that the
statement made by the child
is made of their own free will and volition.
98. Which of the following statements correctly depicts the essential of section
118 of the IEA?
(a) The testimony of child witness should always be supported by corroborative
evidence.
(b) The testimony of a witness should be untutored and of free will.
(c) The testimony of child witness should be tutored and of free will.
(d) The testimony given by a major, sound mind is admissible in addition to Child
witness's testimony is
substantial and enough under S.118 of the Evidence Act.
99. Which of the following is not true, based on your understanding on the subject
of child witness as discussed in
the above passage:
I. In the case of rape or assault, a juvenile witness can also be subjected to
cross examination by the defence
lawyer, and is not limited to judicial testimony.
II. In situations involving criminal offences against children, the first and most
essential piece of evidence is
always the child victim's own statement.
III. The courts are tasked with allowing or disallowing such evidence on the basis
that the child does not
understand or provide rational answers to the questions posed to him, and that if
the responses to these
questions are negative, there is reason to reject such testimony.
IV. Upon court’s satisfaction, the testimony of a child witness can be considered
substantial and sufficient under
S.118 of the Evidence Act.
(a) II & III
(b) 1&III
(c) II& IV
(d) ONLY III

. Page 25 of 36
100. Chandini a 6
th class student file a complaint against his sports teacher for sexual assault as
he used to touch her
inappropriately on several occasion. She in her testimony stated “I know what is
good touch and bad touch”.
Court was very impressed by the statement of Chandini as she had been raised well
by her parents and has
knowledge of good and bad touch. Based on this testimony of Chandini the court
convicted the teacher under
POCSO Act without taking any other substantial evidence on record. Can the
statement provided by Chandini
stands acceptable under section 118 of IEA?
(a) Yes, it should be allowed because chandini's testimony is not tutored and given
of her own free will.
(b) It is unacceptable because a court cannot convict an accused based solely on
the testimony of a minor
witness.
(c) It is acceptable because the statement of the child victim has toalways be the
first and most essential piece
of evidence in situations of sexual abuse against children and used to determine
guilt.
(d) It is not acceptable because, because child testimony is not always reliable,
the court should have reviewed
substantial evidence before convicting the accused.
Passage (Q.101-Q.105): The Delhi High Court has observed that an FIR filed under
the Protection of Children
from Sexual Offences (POCSO) Act, 2012 cannot be quashed on the ground that the
victim after attaining
majority decided to compromise the matter with the accused.
A single judge bench comprising of Justice Subramanian Prasad observed thus:
"Exercising jurisdiction under Section 482 Cr.P.C to quash an offence under POCSO
Act would go against the
intention of the legislature which has brought out the special enactment to protect
the interests of children. The
FIR cannot be quashed on the ground that the victim after attaining majority has
decided to compromise the
matter with the accused."
The Court was dealing with a petition filed under sec. 482 CrPC seeking quashing of
FIR registered under sec.
354 (Assault of criminal force to woman with intent to outrage her modesty), 354D
(Stalking),
506 (Punishment for criminal intimidation), 509 (Word, gesture or act intended to
insult modesty of a
woman), 34 (Common intention) of IPC and sec. 10 (Punishment for aggravated sexual
assault) of the
POCSO Act. The petition was filed on the ground that both the prosecutrix and the
petitioner have compromised
the matter.
The Court, relied on the Statements of Objects and Reasons of the POCSO Act which
reads "heinous crime
like rape cannot be quashed by the High Court by exercising power under Section 482
Cr.P.C. even if the
prosecutrix and the accused have entered into a compromise."
In view of this, the Court dismissed the petition after observing that the Court
was not inclined to quash FIR in
a case wherein petitioners were accused of sec. 10 of the POCSO Act.
(Source : POCSO- FIR Cannot Be Quashed On The Ground That Victim Decided To
Compromise Matter After
Attaining Majority: Delhi High Court live law )
101. Rupesh gave his neighbour Mailini a ride because she was running late for work
and had missed her bus. During
the ride, Rupesh made Malini very uncomfortable by repeatedly hitting the brakes in
order to keep Malini from
holding him. Even after arriving at the office, she was disturbed by Rupesh's
actions. In these circumstances,
can Malini file a complaint against Rupesh?
(a) Rupesh act will not be punished under section 10 of POCSO act for committing
sexual assault on malini but
under IPC.
(b) Rupesh act will be punished under 354 of IPC for Assault of criminal force to
woman with intent to outrage
her modesty.
(c) Rupesh act will not be punished under any penal provision as he had no
intention to outrage her modesty.
(d) Rupesh act will be punishable under section 509 of IPC for act intended to
insult modesty of a woman.

. Page 26 of 36
102. A group of four people, A, B, C, and D, raped a girl named Jeenu aged 17 years
5 months as a form of retaliation
for her declining D's proposal. D did nothing but abetted A, B, and C for
committing sexual assault on
Jeenu. Determine the culpability of all the alleged perpetrators:
(a) All the accused will be liable under penal provision of IPC for raping jeenu.
(b) D will be punished under POCSO as he is minor while others will be punished
under provisions of IPC.
(c) All the accused will be punished under the POCSO act and provision of IPC will
be applicable on them.
(d) Only A, B and C will be punished under POCSO act as D is not an accomplice in
the crime only the abetter.
103. A petition was filed in the high court under section 482 for the quashing of a
FIR filed under section 10 of the
POCSO Act (Punishment for Aggravated Sexual Assault). The prosecutrix said that the
accused, a distant
relative, had begun to reside in her home and that, in the absence of her parents,
the accused "started looking at
her with odd eyes." Furthermore, the accused, along with his other two nephews,
allegedly threatened her with
death over her photographs and even threatened to kill her parents. The petition
was filed after a settlement was
reached between the prosecutrix's mother and the defendants. Decide
(a) The court will quash the FIR as there was compromised made between prosecutrix
mother and the
defendants.
(b) The court will not quash the FIR since a solution was reached between the
prosecutor's mother and the
defendants rather than the prosecutor and the defendants.
(c) The court will quash the FIR as the victim is a minor and is represented by her
mother.
(d) Even if the parties come to some agreement, the court will not dismiss the FIR.
104. Choose the correct statement:
(a) The POCSO act recognises the fact that any person including a child can be
prosecuted as a sexual offender.
(b) Sections of the POCSO Act may not be added by the police in the First
Information Report (FIR) whenever
a sexual offence is committed against a child.
(c) Court can quash an FIR under section 482 of CrPC.
(d) An FIR can charge someone with rape under section 376 of the IPC then it cannot
charge under Pocso Act..
105. Assertion: Court can only quash an FIR if compromise is reached between the
parties in case of offences
punishable under IPC.
Reason: If the parties have reached an agreement through free consent, an
application under section 482 of the
CrPC can be submitted to dismiss the charges.
(a) Both A and R are false but R is not correct explanation of A .
(b) Both A and R are true and R is correct explanation of A.
(c) A is true but R is false.
(d) A is false but R is true.

. Page 27 of 36
SECTION - D: LOGICAL REASONING
Passage (Q.106-Q.110): Fortunes of many political families are on the line in the
upcoming five state assembly
elections and for a change these elections are seeing some family members on
opposing sides of the political
divide. After a prolonged schism, Akhilesh Yadav has marginalised everyone else in
his big family, which had
earned SP much public scorn for being a family enterprise. With no future in SP,
his sister-in-law Aparna Yadav
has joined BJP. The Badal family domination of the Akali Dal, meanwhile, is
yielding diminishing returns to a
movement that has dominated Punjab politics for decades.
Dynastic succession is a reality in Indian politics except for parties like BJP and
fading CPM, CPI – though these
too periodically make allowances for scions proving their mettle. That none of
BJP’s top rung leaders are dynasts
allows Prime Minister Narendra Modi to go on the offensive against big political
families like Gandhi’s. With
umpteen regional parties originally ranged against Congress now confronting BJP,
the latter is sharpening the
“parivarvaad” attack against dynasties of satraps like M Karunanidhi, Bal
Thackeray, Mulayam Singh, Lalu
Prasad and lately Mamata Banerjee.
With one plank of the political attacks against him centred on “family rule”,
Akhilesh has paid a backhanded
compliment to BJP for ridding SP of his family members. But political elites down
to the district level are actively
invested in prolonging their own fiefdoms. Democracy in India has retained many of
the dynastic and feudal
characteristics of the medieval political economy that preceded it. With
personality cults flourishing, creating a
line of succession for the immediate family was a primary instinct for those who
seized outsized influence in
political parties at the national, state and local levels.
But there seems to be recognition in the political class that popular acceptance
for nepotism cannot be pushed
beyond a point. Yadav family politicians are conspicuously missing at Akhilesh’s
side in these elections. Badaun
MP Sanghamitra Maurya hasn’t betrayed her 2019 mandate by crossing over to SP, even
though her father and
senior OBC leader SP Maurya has. In Punjab and Uttarakhand, Congress is attempting
a one family, one ticket
rule – discomfiting the likes of Charanjit Singh Channi and Harish Rawat. BJP has
likewise denied the
prestigious Panaji seat to Manohar Parrikar’s son. Indian politics and its economy
are in dire need of reforms
that improve opportunities for the youth. Dynastic politics is one of the ways
those doors just don’t open wide
enough.
106. Which option best represents the main idea of the passage?
(a) Succession in dynastic politics in India is an ominous sign for many political
parties.
(b) Dynastic politics are keeping India away from becoming a world leader.
(c) India’s dynastic politics is losing its charm as some cracks are showing in
India’s dynastic politics .
(d) The BJP should be awarded for not being a dynastic political party.
107. It can be inferred from the passage that:
(a) The BJP is the only party with little to no practice of dynastic succession.
(b) Making allowances for scions is not a move away from nepotism.
(c) Mamata Banerjee is one of the front runners in encouraging dynastic politics.
(d) Making allowances for sons of political leaders is a move away from nepotism.
108. Out of the four marked statements, which one of the following is the odd one
out?
(a) Fortunes of many political families are on the line in the upcoming five state
assembly elections and for a
change these elections are seeing some family members on opposing sides of the
political divide.
(b) But there seems to be recognition in the political class that popular
acceptance for nepotism cannot be pushed
beyond a point.
(c) Yadav family politicians are missing at Akhilesh’s side in these elections.
(d) Democracy in India has retained many of the dynastic and feudal characteristics
of the medieval political
economy that preceded it.

. Page 28 of 36
109. Which of the following would present a clear challenge to the claims of the
author? Consider the following
statements to be true.
I. The present Home Minister is from the CPI, whose father was also a political
leader.
II. The present party President of the BJP was given the position on merit, even
though his father was a Member
of Parliament.
(a) Only I (b) Only II (c) Both I and II (d) Neither I nor II.
110. Which option most accurately and correctly represents the author’s opinion?
(a) It becomes virtually impossible to rule out the competence of dynastic parties,
once it is in power.
(b) The present practice of dynastic politics has its roots in the history of
India.
(c) The CPM is on a declining curve and there are no hopes for it to be back in
contention.
(d) The Samajwadi Party has paid the heftiest price for being a party based on
dynasties.
Passage (Q.111-Q.115): NHS waiting lists are out of control. Around 7.5 million
people are queueing for
hospital treatment in the UK, around 6 million of them in England. There are
differences in the policies adopted
by the devolved administrations to reduce backlogs. In England, it is clear that an
increased role for the private
sector is the government’s plan.
The details of the recovery plan promised by ministers are still being finalised,
with NHS bosses resisting what
they regard as unrealistic targets. Finding a way to increase surgical capacity, so
that more operations can happen
more quickly, is the priority. The impact of delays is both social as well as
economic. But there are worrying
signs that any new contracts will tip the system’s overall balance further in
favour of for-profit providers, and
away from the NHS.
The unprecedented pressure that the NHS is under ought to lead the government to
invest in it, not compel it to
rely on businesses to do its work. Yet as with education, where the government
chose to buy a package of
pandemic catch-up tuition from outside partners rather than funding schools
themselves, ministers are opting to
meet the additional health needs of the population through the private sector.
The role of the private sector in carrying out operations on behalf of NHS England
is well established. Labour’s
health spokesman, Wes Streeting, has said he would use private firms to reduce
waiting lists. Outsourcing also
happens in Northern Ireland, Scotland and Wales, although to a lesser extent. But
by forcing NHS England to
continue to rely on private sector backup, while refusing to commit to long-term
workforce planning as
advocated by a former health secretary, the government is deliberately expanding
the role of the market.
Additional NHS funding from higher national insurance should help ease pressures.
But the problems in some
areas are chronic, long predating the pandemic, and a worsening staffing crisis in
the social care sector is already
having knock-on effects.
Of course, ministers continue to declare their support for the NHS, which they know
is valued highly by a public
that sees clearly how much worse people fare under for-profit healthcare systems
such as in the US. But their
actions in beefing up the role of private providers, while refusing to take the
steps that would help to secure the
NHS’s long-term future, speak louder than their words.
111. The claim made in which of the following statements is being strengthened by
this piece of evidence? “People
waiting for cataracts can’t see properly; people who need joint replacements may
struggle to walk.”
(a) The role of the private sector in carrying out operations on behalf of NHS
England is well established.
(b) The impact of delays is both social and taxing on people’s health.
(c) There are differences in the policies adopted by the devolved administrations
to reduce backlogs.
(d) Finding a way to increase surgical capacity, so that more operations can happen
more quickly, is the priority.

. Page 29 of 36
112. If the information given in the passage is true, then which of the following
can be inferred from the passage?
(a) There is nothing wrong with the NHS getting more than just backup support from
private players.
(b) There is a difference between what ministers say about the NHS and what they do
for it.
(c) The Ministers would make sure that the recovery plan of the NHS is stalled.
(d) The parliamentarians genuinely care for the NHS and want it to regain its lost
glory.
113. What has the author not discussed in the passage?
(a) Stance of ministers on the NHS.
(b) Steps taken by the government to ease pressure from NHS
(c) Benefits of increased role for the private sector in NHS
(d) Suggestions that could ease pressure from NHS
114. Which of the following can be attributed to be the opinion of the author?
(a) Private players would better manage the workload which has been assigned to the
NHS.
(b) For the NHS to get back to its previous position, it is important that the
support of private players is
completely removed.
(c) Private healthcare system such as that in the US is well suited for the public
of the UK.
(d) None of the above
115. ‘The details of the recovery plan promised by ministers are still being
finalised, with NHS bosses resisting what
they regard as unrealistic targets. Which of the following is the assumption behind
the passage?
(a) The NHS authorities do not agree with the recovery plans.
(b) The discrepancy in the recovery plan will delay the reducing of the backlog of
cases.
(c) There is a discrepancy in how the recovery plan is being perceived by ministers
and NHS bosses.
(d) The recovery plan of NHS is untenable.
Passage (Q.116-Q.120): Days after meeting Prime Minister Narendra Modi in Delhi,
Russian President
Vladimir Putin held a summit via video conference with Chinese President Xi
Jinping. While the two meetings
may have focused on bilateral issues, the conversations appear to have overlapped
in unusual ways. According
to a senior Kremlin official, after discussing with Mr. Modi India’s problems with
Chinese aggression, which
were raised publicly during the visit by Defence Minister Rajnath Singh, Mr. Putin
“briefed” Mr. Xi on his talks
in Delhi. The official then indicated that a trilateral summit of the leaders of
Russia, India and China (RIC) could
be held in the near future, which would pick up on the Modi-Putin-Xi conversation
during an RIC summit on
the sidelines of the Osaka G-20 summit in 2019. However, much has occurred between
that summit and today.
Since April 2020, the two leaders have not spoken directly once, and while they
have attended the same
multilateral summits (BRICS, SCO, G-20, etc.), it would be hard to see them
engaging in a face-to-face format.
While India-Russia defence and bilateral ties have considerably strengthened,
especially after the Modi
government’s decision to go ahead with its purchase of the S-400 missile defence
systems despite the U.S. threat
of sanctions, New Delhi must tread cautiously in its trilateral and multilateral
cooperation with Moscow and any
hint that Russia could play a facilitator of talks with China must be scrutinised
closely. Russia and China have
consolidated their support for each other in the face of U.S. concerns over Russian
action against Ukraine and
Chinese action on Taiwan. While India must continue to walk the tightrope between
Moscow and Washington,
and its partners in Eurasia versus those in the Indo-Pacific, it needs to
disentangle these threads from the very
potent threat it faces directly and bilaterally from its northern neighbour, where
it has little choice but to follow
an independent path.

. Page 30 of 36
116. Which of the following best represents the central point of this passage?
(a) Russia and India should continue to work together and solve their bilateral
issues.
(b) India’s relation with Russia is similar to that friend who always stands by
your side when in need.
(c) Any offer from Russia to become a facilitator between India and China should be
scrutinised closely by New
Delhi.
(d) Ties between India and China have reached a new low in the last two years,
which is beyond redemption.
117. With which of the following would the author be most likely to agree with?
(a) All is not well with the ties between Russia and the US.
(b) There are a lot of cultural similarities between India and Russia.
(c) Bilateral relations between different entities must be kept separate.
(d) Russia should not have brought up India while engaging with China.
118. Which of the following pieces of evidence lend support to the author’s
arguments about the ties between India
and China?
(a) There was a major border conflict between the soldiers of India and China in
2020.
(b) In 2018, a Chinese platoon held 20 Indian army soldiers and released them only
after much trouble.
(c) China had reserved its opinion on the recent $4 Billion deal between India and
Russia.
(d) China has lent support to Russia in matters of Ukraine.
119. Which of the following may be deduced, based only on the passage?
(a) The geographical location of India is south of China.
(b) The main subject matter of Russia and China in the recent meeting was India.
(c) Mr. Xi and Mr. Modi would not engage in a face-to-face meeting again.
(d) New Delhi does not care one bit about the sanctions imposed by the US.
120. Which of the following would be helpful to know to ascertain whether Russia
should become a facilitator for
Delhi to engage with China?
(a) China has invested $400 Billion in Russia; Russia’s GDP is $1 Trillion.
(b) Russia has equal and good bilateral relations with India and China.
(c) The ties between India and America are at an all-time low.
(d) Both a and b are correct.
Passage (Q.121-Q.125): The passage given below is followed by a set of questions.
Choose the most appropriate
answer to each question.
From the philosophical point of view, the most brilliant results of the new method
are the exact theories which
we have been able to form about infinity and continuity. We know that when we have
to do with infinite
collections, for example, the collection of finite integer numbers, it is possible
to establish a one-to-one
correspondence between the whole collection and a part of itself. For example,
there is such a correspondence
between the finite integers and the even numbers, since the relation of a finite
number to its double is one-to#one. Thus, it is evident that the number of an
infinite collection is equal to the number of a part of this collection.
It was formerly believed that this was a contradiction: even Leibnitz, although he
was a partisan of the actual
infinite, denied infinite number because of this supposed contradiction. But to
demonstrate that there is a
contradiction, we must suppose that all numbers obey mathematical induction. To
explain mathematical
induction, let us call by the name “hereditary property’ of a number, a property
which belongs to n + 1 whenever
it belongs to n. Such is, for example, the property of being greater than 100. If a
number is greater than 100, the
next number after it is greater than 100. Let us call by the name “inductive
property” of a number a hereditary
property which is possessed by the number zero. Such a property must belong to 1,
since it is hereditary and
belongs to 0; in the same way, it must belong to 2, since it belongs to 1; and so
on. Consequently, the numbers
of daily life possess every inductive property. Now, amongst the inductive
properties of numbers is found the

. Page 31 of 36
following. If any collection has the number n, no part of this collection can have
the same number n.
Consequently, if all numbers possess all inductive properties, there is a
contradiction with the result that there
are collections which have the same number as a part of themselves. This
contradiction, however, ceases to
subsist as soon as we admit that there are numbers which do not possess all
inductive properties. And then it
appears that there is no contradiction in infinite number. Cantor has even created
a whole arithmetic of infinite
numbers, and by means of this arithmetic, he has completely resolved the former
problems on the nature of the
infinite which have disturbed philosophy since ancient times.
The problems of the continuum are closely connected with the problems of the
infinite and their solution is
effected by the same-means. The paradoxes of Zeno the Eleatic and the difficulties
in the analysis of space, of
time, and of motion, are all completely explained by means of the modern theory of
continuity. This is because
a non-contradictory theory has been found, according to which the continuum is
composed of an infinity of
distinct elements; and this formerly appeared impossible. The elements cannot all
be reached by continual
dichotomy; but it does not follow that these elements do not exist.
From this follows a complete revolution in the philosophy of space and time. The
realist theories which were
believed to be contradictory are so no longer, and the idealist theories have lost
any excuse there might have
been for their existence. The flux, which was believed to be incapable of analysis
into indivisible elements,
shows itself to be capable of mathematical analysis, and our reason shows itself to
be capable of giving an
explanation of the physical world and of the sensible world without supposing jumps
where there is continuity,
and also without giving up the analysis into separate and indivisible elements.
121. What is the way in which the author of the above passage presents his views?
(a) Underlines the advantages of a new method, explains them, and glorifies the
method.
(b) Introduces a new method, explains it, relates its usefulness and then
exemplifies it.
(c) Disagrees with an existing norm, states ways of changing, and typifies it.
(d) Counters a hypothesis, exposes its loopholes and revokes it.
122. According to the passage, in what way has the problem of the infinite number
been resolved?
(a) By stating that the property that the number of an infinite collection is equal
to the number of a part of this
collection, is a contradiction.
(b) By understanding that the property that the number of an infinite collection is
equal to the number of a part
of this collection, is no more supposed to be a contradiction.
(c) With possible solutions, the property that the number of an infinite collection
is equal to the number of a part
of this collection, can erase a contradiction.
(d) The contradiction has finally become an established property.
123. Extending the author’s logic to different situations, which of these is he
most likely to agree with?
(a) The remedy for all diseases is possible to develop as there is an acceptance of
its presence in the subconscious
mind.
(b) The subconscious mind cannot be formed solely of electric signals. The emotions
must play a role in it.
(c) “Emotions are immeasurable” is a statement that need be revoked in order that
the human mind pursue its
goal of learning.
(d) Learning about learning is an odious task and one in which there should be no
investing of time.
124. According to the passage, what is the relation between the inductive property
and the hereditary property?
(a) An inductive property is an example of the hereditary property.
(b) An inductive property is an extension of the hereditary property in a totally
unknown field.
(c) A hereditary property logically follows an inductive property.
(d) An inductive property is an extension of the hereditary property in a known
field.

. Page 32 of 36
125. The supposed contradiction that is stated in the passage related to the
infinite number is that
(a) the infinite number cannot be a part of the whole and, hence, cannot be
realized.
(b) the infinite number can be a part of itself and, hence, cannot be realized.
(c) all numbers obey mathematical induction.
(d) all the numbers do not obey mathematical induction.
Passage (Q.126-Q.130): Religious leaders are supposed to be detached and
‘liberated’ from the affairs of the
world, but the manner in which dozens of Lingayat seers have openly dabbled in
politics by lobbying for the
continuation of B S Yediyurappa as Karnataka Chief Minister is an indication of how
far they have strayed from
the path of renunciation. The doctrine of separation of Church and State calls for
a distance between the political
apparatus and religious organisations, but this line has been breached all too
often. The political clout that these
men in ochre robes wield has never been in doubt, but to dictate to a political
party that a Chief Minister should
not be removed only because he belongs to their community does not auger well for
democracy and secularism.
It is a different matter though that, irrespective of the party, it is usually an
undemocratic ‘high command’ that
has the final say in the choice of the Chief Minister, and not the MLAs as mandated
by the Constitution.
Nonetheless, religious and spiritual leaders have no business to play the role of
power brokers.
While all political parties are guilty of pandering to various mutts and their
heads in view of their supposed
influence over their followers, the BJP has taken it to an altogether different
level in its bid to shred secularism,
a basic requirement of democracy, and wield its Hindutva ideology to gain political
dominance. It has come back
to bite the party, as the seers threaten to derail the BJP’s electoral prospects if
it dares replace Yediyurappa. The
BJP has already had a taste of it during the 2013 Assembly elections, when the
seers threw their weight behind
Yediyurappa who had floated his own outfit, the Karnataka Janata Party, leading to
the BJP’s defeat in many
seats. It is no secret that many religious leaders interfere in the government’s
functioning and exert pressure on
the Chief Minister to induct their favourites as ministers and officers in plum
posts. But what is most unfortunate
is that the effect of what the seers have said openly has the effect of dividing
people on the basis of caste and
community.
If the seers are keen on a life of politics, they should drop the pretence of
renunciation and contest elections, as
Yogi Adityanath and others have done. People look up to seers and saints with great
reverence. They bring no
honour to themselves by indulging in political manoeuvres. Politics and religion
will both retain legitimacy only
so long as they remain separate.
126. What is the central theme of the passage?
(a) Religious leaders are supposed to be detached from the affairs of the world.
(b) Chief Minister should not be removed only because he belongs to their
community.
(c) Seers had always been and will remain in politics.
(d) The intervention of the religious leaders in politics does not bode well for
democracy.
127. The doctrine of separation of Church and State calls for a distance between
(a) Non- political and religious organisations.
(b) Political apparatus and religious organisations.
(c) Various political apparatus.
(d) Religious organisations.
128. All of the following can be inferred except
(a) The case of removing B S Yediyurappa as Karnataka Chief Minister is not
unprecedented.
(b) No political party in India is untouched to playing up to the religious
sentiments.
(c) There are incidents of religious seers who joined politics after renunciation
of religion sentiments.
(d) Religious leaders, at times, are the backbones behind a prominent leader.

. Page 33 of 36
129. Directions: Each question has a set of sequentially ordered statements. Each
statement can be classified as one
of the following.
Facts, which deal with pieces of information that one has heard, seen or read, and
which are open to discovery
or verification (the answer option indicates such a statement with an ‘F’)
Inferences, which are conclusions drawn about the unknown, on the basis of the
known (the answer option
indicates such a statement with an ‘I’).
Judgments, which are opinions that imply approval or disapproval of persons,
objects, situations and occurrences
in the past, the present or the future (the answer option indicates such a
statement with a ‘J’).
I. The doctrine of separation of Church and State calls for a distance between the
political apparatus and
religious organisations, but this line has been breached all too often.
II. Nonetheless, religious and spiritual leaders have no business to play the role
of power brokers.
III. During the 2013 Assembly elections, the seers threw their weight behind
Yeddyurappa who had floated his
own outfit, the Karnataka Janata Party, leading to the BJP’s defeat in many seats.
IV. It is no secret that many religious leaders interfere in the government’s
functioning and exert pressure on the
Chief Minister to induct their favourites as ministers and officers in plum posts.
V. People look up to seers and saints with great reverence.
(a). FJFFJ (b). JJIFI (c). FIIFJ (d). JIFFJ
130. ‘Politics and religion will both retain legitimacy only so long as they remain
separate.’ Which of the following
is the assumption behind the passage?
(a) Politics and religion are two sides of the same coin.
(b) Religion is drops of oil in political water
(c) Politics and Religion can retain legitimacy if only they remain separate from
each other.
(d) Religion and politics are parallel lines.
Passage (Q.130-Q.135): 2022 offers a hint of hope after a year of gloom. Vaccines
are rolling out, and we might
permit ourselves visions of normality as the battle against the COVID-19 pandemic
at last swings in our favour.
While we may soon prevail against COVID-19, we can’t ignore an increasing problem
that the fight against the
virus has worsened. Plastics have been deployed in great quantities as a shield
against COVID. But little attention
has been paid to where the increased plastic waste will end up. The sad irony is we
were on the cusp of real
victories against plastic pollution just as the coronavirus pandemic began.
In 2019, Prime Minister Narendra Modi committed to completely phase out single-use
plastics by 2022. The
commitment called for better arrangements to collect, store, and recycle single-use
plastic. The movement was
also international. The UN Environment Programme, with the support of Norway and
Japan, undertook a
multiyear assessment of how plastic finds its way into riverways, and ultimately
the ocean, through projects like
CounterMEASURE. And National Geographic’s “Sea to Source: Ganges” Expedition
brought together four
countries, including India and Bangladesh, to holistically study plastic pollution
within the Ganges River basin.
The pandemic halted and, in some cases, reversed much of this progress. Plastics,
especially single-use plastics,
became more ubiquitous. Masks, sanitiser bottles, personal protective equipment,
food packaging, water bottles:
Life came to be ensconced in a plastic shell.
In time, this plastic will disintegrate into tiny particles of less than five
millimetres — known as microplastics
— and move through water bodies and farm soil to enter the food we eat and the air
we breathe. We know that
only 9 per cent of all plastic ever produced has been recycled, while 79 per cent
of all plastic produced can be
found in the world’s landfills and in our air, water, soil, and other natural
systems. Plastic doesn’t belong in our
bodies and it doesn’t belong in nature. But plastic is still important. Its central
role in durable goods, medicine
and food safety means that it is not practical to get rid of entirely. Instead, we
must be more thoughtful about
where, when and how we use it. We need an approach that includes reducing the
manufacture of new fossil fuel#based plastics, improving waste collection and
disposal, and developing and using alternatives.

. Page 34 of 36
131. Which one of the following is an assumption that the author relies upon in the
passage?
(a) Plastic waste disposal is the next bigger challenge the government takes on
after the subjugation of COVID#19.
(b) Tackling Plastic waste is as challenging as dealing with Covid-19.
(c) With the increase of plastic use during the Covid times, the plastic waste
disposal will emerge as a major
environmental challenge for the government.
(d) There is a cure for Covid-19 but there is no cure for perils of plastic usage.
132. ‘But plastic is still important. Its central role in durable goods, medicine
and food safety means that it is not
practical to get rid of entirely.’ Which one of the following conclusions can be
correctly drawn from the
statements above?
I. One can never get rid of use of plastics.
II. Plastic is here to stay till an alternative is found.
III. The use of plastic will prevail for certain commodities for a very long time.
(a) Only I (b) Only II (c) II and III (d) I and III.
133. ‘The commitment called for better arrangements to collect, store, and recycle
single-use plastic.’ Which of the
following statement/(s) from the passage itself/themselves belies the above
statement?
I. We know that only 9 per cent of all plastic ever produced has been recycled,
while 79 per cent of all plastic
produced can be found in the world’s landfills and in our air, water, soil, and
other natural systems.
II. Plastic doesn’t belong in our bodies and it doesn’t belong in nature.
III. Life came to be ensconced in a plastic shell.
(a) Only I (b) Only II (c) I and II (d) I, II & III.
134. Life came to be ensconced in a plastic shell. What can be inferred from the
given statement?
(a) Life revolved around plastics with the coming of Pandemic.
(b) Plastic use became a way of life during pandemic.
(c) People realized more and more the importance of plastic during pandemic.
(d) The use of plastics declined during pandemic.
135. Which of the following will the author least likely to agree with
(a) There is an urgent need to consolidate efforts nationally and internationally
to address the mayhem of use of
plastics.
(b) To work towards finding an alternative organic source to stop the use of
plastics.
(c) To start various programmes for public awareness and appeal to the minimum use
of plastics.
(d) To bring a draconian law to deal with the inordinate use of plastics

mock 32
Directions(Q.1-Q.30): Read the following passage carefully and answer the questions
that follow.
Passage(Q.1-Q.5): Snyder Appleby was generally considered by the boys as one of the
meanest fellows in
Euston, and that is the reason why they called him “Cider Apples”; for those, as
everybody knows, are most
always the very poorest of the picking. So, the name seemed to be appropriate, as
well as a happy parody on that
to which he was really entitled. He was the son, or rather the adopted son, of
Major Arms Appleby, who, next to
President Vanderveer, was the richest man in Euston, and lived in the great,
rambling stone mansion that had
been in his family for generations.
The Major, who was a bachelor, was also one of the kindest-hearted, most generous,
and most obstinate of men.
He loved to do good deeds; but he loved to do them in his own way, and his way was
certain to be the one that
was contrary to the advice of everybody else. Thus, it happened that he determined
to adopt the year-old baby
boy who was left on his doorstep one stormy night, a little more than sixteen years
before this story opens. He
was not fond of babies, nor did he care to have children about him. Simply because
everybody advised him to
send this one to the county house, where it might be cared for by the proper
authorities, he declared he would do
nothing of the kind; but would adopt the little waif and bring him up as his own
son.
As the boy grew, and developed many undesirable traits of character, Major Appleby
was too kind-hearted to
see them, and too obstinate to be warned against them.
“Don’t tell me, “he would say, “I know more about the boy than anybody else, and am
fully capable of forming
my opinion concerning him.”
Thus Snyder Appleby, as he was called, because the name “Snyder” was found marked
on the basket in which
he had been left at the Major’s door, grew up with the fixed idea that if he only
pleased his adopted father he
might act about as he chose with everybody else. Now he was nearly eighteen years
of age, big and strong, with
a face that, but for its coarseness, would have been called handsome. He was fond
of display, did everything for
effect, was intolerably lazy, had no idea of the word punctuality, and never kept
an engagement unless he felt
inclined to do so. He always had plenty of pocket money which he spent lavishly,
and was not without a certain
degree of popularity among the other boys of Euston. He had subscribed more largely
than anybody else to the
Steel Wheel Club upon its formation, and had thus succeeded in having himself
elected its captain.
[ Excerpts from Cab and Caboose: The Story of a Railroad Boy by Kirk Munroe]
1. According to the passage, which of the following is true?
(a) Snyder Appleby was a year old when he was adopted.
(b) Snyder Appleby was never given sufficient pocket money
(c) Major Appleby didn't treat his adopted son well.
(d) Snyder Appleby was rude to his foster father.
2. Which of the following adjectives doesn't accurately describe the protagonist?
(a) Indolent. (b) Industrious. (c) Tardy. (d) Spendthrift.
3. Choose the word with closest with OBSTINATE used in the passage
(a) stubborn (b) skeptical (c) cynical (d) flexible
4. He was not fond of babies, nor did he care to have children about him.
Identify the part of speech of the italicised word in the above sentence.
(a) Adverb (b) Adjective (c) Verb (d) Noun

. Page 3 of 36
5. Which of the following reflects the most suitable title for the passage?
(a) Major Appleby
(b) The warm hearted Snyder Appleby.
(c) Snyder Appleby: the underprivileged waif.
(d) The accidental journey of Snyder Appleby.
Passage(Q.6-Q.10): Read the following passage and answer the questions that follow:
There is a certain malady of the mind induced by too much of one thing. Just as the
body fed too long upon meat
becomes a prey to that horrid disease called scurvy, so the mind fed too long upon
monotony succumbs to the
insidious mental ailment which the West calls “cabin fever.” True, it parades under
different names, according
to circumstances and caste. You may be afflicted in a palace and call it ennui, and
it may drive you to commit
peccadillos and indiscretions of various sorts. You may be attacked in a middle-
class apartment house, and call
it various names, and it may drive you to cafe life and affinities and alimony. You
may have it wherever you are
shunted into a backwater of life, and lose the sense of being borne along in the
full current of progress. Be sure
that it will make you abnormally sensitive to little things; irritable where once
you were amiable; glum where
once you went whistling about your work and your play. It is the crystallizer of
character, the acid test of
friendship, the final seal set upon enmity. It will betray your little, hidden
weaknesses, cut and polish your
undiscovered virtues, reveal you in all your glory or your vileness to your
companions in exile—if so be you
have any.
If you would test the soul of a friend, take him into the wilderness and rub elbows
with him for five months! One
of three things will surely happen: You will hate each other afterward with that
enlightened hatred which is
seasoned with contempt; you will emerge with the contempt tinged with a pitying
toleration, or you will be close,
unquestioning friends to the last six feet of earth—and beyond. All these things
will cabin fever do, and more. It
has committed murder, many a time. It has driven men crazy. It has warped up and
distorted character out of all
semblance to its former self. It has sweetened love and killed love. There is an
antidote—but I am going to let
you find the antidote somewhere in the story.
Bud Moore, ex-cow-puncher and now owner of an auto stage that did not run in the
winter, was touched with
cabin fever and did not know what ailed him. His stage line ran from San Jose up
through Los Gatos and over
the Bea Creek road across the summit of the Santa Cruz Mountains and down to the
State Park, which is locally
called Big Basin. For something over fifty miles of wonderful scenic travel, he
charged six dollars, and usually
his big car was loaded to the running boards. Bud was a good driver, and he had a
friendly pair of eyes—dark
blue and with a humorous little twinkle deep down in them somewhere—and a human
little smiley quirk at the
corners of his lips. He did not know it, but these things helped to fill his car.
[ Excerpts from Cabin Fever by B. M. Bower]
6. ‘It will betray your little, hidden weaknesses, cut and polish your undiscovered
virtues, reveal you in all your
glory or your vileness to your companions in exile—if so be you have any.’ The
author through the lines wants
to
(a) Emphasise the term. (b) Present the dichotomy.
(c) Caution against the disease. (d) Prophesise doomsday.
7. What did the author mean by "you will be close, unquestioning friends for the
last six feet of the earth - and
beyond."?
(a) One can be close friends enough to go to the undergrounds and start living
there together.
(b) The bond so formed between friends will remain till the last breadth and will
spill over to the afterlife.
(c) One can be close friends enough only after their death.
(d) The bond so formed between friends will remain till the last breadth and will
spill over to the next life.
8. Choose the word closest to ennui used in the passage
(a) boredom (b) courageous (c) contentment (d) None of these

. Page 4 of 36
9. Observe the following sentences and identify the idiom that is synonymous with
touch.
(a) glum where once you went whistling about your work and your play.
(b) It has sweetened love and killed love.
(c) True, it parades under different names, according to circumstances and caste.
(d) If you would test the soul of a friend, take him into the wilderness and rub
elbows with him for five months
10. Which of the following words can be used in the place of the word ‘peccadillo’
without changing the context of
the passage?
(a) Trifling offence (b) Discombobulated
(c) Ingress (d) Ingratiate
Passage(Q.11-Q.15): Read the following passage and answer the questions that
follow:
Tom Jones was a stout lad of sixteen, with frowzy brown hair, crowned by a brimless
straw hat, and his pants
looked as if they had been turned inside out and outside in, upside down and
downside up, and darned and
patched and re-darned and patched again, until time, and labour, and cloth enough,
such as it was, had been used
to fabricate a number of pairs of pants. As for boots–for his lower extremities
were not wholly destitute of
protection–they might have come down to him as an heir-loom from a pauper of a
preceding generation. But
what mattered it to him that his clothes were threadbare, many-hued, and grotesque?
or that his boots let the
deep, rich soil in at sides and toes? Was he not a “squatter sovereign,” or the son
of one, free in his habits as the
Indian that roamed the prairies of his frontier home? He had not heard of “the
latest fashion,” and paid no
attention to the cut of his garments, although, it must be confessed, he sometimes
wished them a trifle sprucer
and more comfortable. His home, as I have hinted, was on the prairie. Nevertheless,
the family domain was an
unpretending one.
The squatter’s son looked about him with a dissatisfied air. “I do wish,” he
soliloquized, “that I could see
something of the world, and do something for myself. Here we’ve been changing
around from one place to
another, doing nothing but raise a few potatoes and a little corn, living in a
miserable cabin, where there are no
schools, and scarcely any neighbours. It’s too bad to spend all our days so. I
believe we were made (---) something
better; and, as the minister told us Sunday, we ought (---) try and be somebody,
and not float (---) as the stick
on the stream. I’m sure it isn’t, and never was, to mother’s mind; and, as to
father–” And here he stopped and
pondered, as if trying to solve a mystery.
The rows of hills were covered with the bountiful returns brought up to the light
of day by Tom’s well-used hoe.
It was not, however, the size, quality, or number of the potatoes that most
interested Tom just then. The fact that
they were all out of the ground; that the corn was cut and stacked, and the
pumpkins ready to be housed; that the
fall work could be finished by that afternoon’s sun-setting,–stirred him strangely;
for he had of late begun to
question the future, to learn what it had in store for him. He had come to realize,
in a degree, that that future
would be very much what he chose to make it. And serious dissatisfaction with the
past and the present filled his
heart with disquiet.
[ Edited and recreated from The Cabin on The Prairie by Dr. C. H. Pearson ]
11. The main theme of the passage revolves around:
(a) Narrating the protagonist’s situation briefly.
(b) Describing how poverty had hit all teenagers of the prairie.
(c) Describing the hardships and challenges of farming.
(d) Narrating the friendship between Tom and the squatter's son.
12. Which of the following can be an appropriate title of the passage?
(a) Life of a farmer all over the world (b) Life and Dreams of Tom Jones.
(c) A Friendship in the Prairie. (d) A Daydreamer’s Paradise.
13. Which of the following words from the passage means ‘an act of speaking one's
thoughts aloud when by oneself
or regardless of any hearers, especially by a character in a play.’?
(a) Soliloquy. (b) Destitute. (c) Grotesque. (d) Trifle.

. Page 5 of 36
14. ‘Tom Jones was a stout lad of sixteen, with frowzy brown hair, crowned by a
brimless straw hat, and his pants
looked as if they had been turned inside out and outside in, upside down and
downside up, and darned and
patched and re-darned and patched again, until time, and labour, and cloth enough,
such as it was, had been used
to fabricate a number of pairs of pants.’ The above lines reflect which of the
following literary device?
(a) Litotes. (b) Hyperbole (c) Allegory (d) None of these
15. I believe we were made (---) something better; and, as the minister told us
Sunday, we ought (---) try and be
somebody, and not float (---) as the stick on the stream.
Choose the correct option carrying the right order and combination of the missing
prepositions from the italicised
sentence above.
(a) of, to, by (b) for, at, along (c) for, to, along (d) of, at, by
Passage(Q.16-Q.20): Read the following passage and answer the questions that
follow:
For three days we had been congratulating ourselves that we were on the eve of the
greatest battle in history.
Around us in silent might, two armies slept on their arms. From the border of far
Mongolia for a hundred and
eighty miles eastward lay the line of the Japanese trenches, and for forty miles
deep every Manchu hut and
village sheltered the soldier or coolie patriot of the Island Emperor. Above the
roads for endless miles hung the
heavy powdered dust of Mongol soil; like a mist unstirred by any wind, it rose from
the plodding of the feet of
limitless thousands of men and animals, pushing forward for the last great struggle
of a mighty conflict. Regiment
after regiment fresh from home, poured along the Japanese made arteries, for the
blood of an army corps. Now
and again the khaki-coloured battalions at the command of an officer halted at the
side of the road while a battery
of artillery, with clanking chains and creaking limbers, trotted through the
thickening clouds of dust that settled
on one like flour. Cavalry, red cross, transport, coolies, bridge trains and
telegraph corps gave place the one to
the other in rapid succession. In eighteen months’ association with the Japanese,
we had not seen such activity.
“The Peace Conference at Portsmouth has failed” we told ourselves, and leaving the
extreme front of the army,
where we had been visiting the cavalry outposts, we turned our horses’ heads for
the thirty-mile ride to the
headquarters of General Nogi, to which we had been attached since May. All our talk
was of the coming of the
great battle and of the preparations which we must make for a three weeks’ campaign
in the saddle, and more
important still, how we should arrange an open line of communications from the
ever-changing front of the
prospective struggle to the cable office in the rear.
Covered with dust an eighth of an inch deep, we rode into Fakumen, our
headquarters, late on the afternoon of
September 4th. At the door of a Chinese bean mill, where for four weary months we
had been awaiting the call
to action, stood a Japanese orderly. As we dismounted, he saluted and respectfully
handed me one of the Japanese
charactered envelopes of the Military field telegraph. Turning my horse over to my
Japanese boy I opened it,
and read the word “Return.”
The Russo-Japanese War was over, and even before the armies themselves knew that
the end had come, my chief
in his office in faraway Chicago had sent the word over the cable which meant as
much as reams of explanation.
The same night the London Times reached half around the world and ordered home its
special correspondent
with the Japanese armies in the field.
[ Excerpts from The Cable Game, by Stanley Washburn]
16. “The tree is like the god of the forest”.
Identify the correct option that used the same literary device as the sentence
above.
(a) Cavalry, red cross, transport, coolies, bridge trains and telegraph corps gave
place the one to the other in
rapid succession.
(b) Above the roads for endless miles hung the heavy powdered dust of Mongol soil;
like a mist unstirred by
any wind, it rose from the plodding of the feet of limitless thousands of men and
animals, pushing forward
for the last great struggle of a mighty conflict.
(c) Around us in silent might, two armies slept on their arms.
(d) As we dismounted, he saluted and respectfully handed me one of the Japanese
charactered envelopes of the
Military field telegraph.

. Page 6 of 36
17. Which of the following can be inferred as true according to the passage?
(a) The Japanese trenches were in the east of the Mongolian Border.
(b) The author was associated with the Japanese for less than a year.
(c) Peace conference was held at a place called Nogi
(d) None of the above
18. Which of the following words mean ‘slow-moving and unexciting.’?
(a) Unstirring (b) Plodding (c) Trotted (d) Halted
19. It can be inferred from the passage that:
(a) The Russo-Japanese war was considered to be a battle of minor significance.
(b) The author and his team were staying permanently at the cavalry outposts.
(c) General Nogi’s office was in Czechoslovakia.
(d) The author was a special correspondent of the London Times.
20. Which of the following writing tones has been used in the line?
“For three days we had been congratulating ourselves that we were on the eve of the
greatest battle in the history”
(a) Emotional (b) Acerbic (c) Ethical (d) Euphemistic
Passage(Q.21-Q.25): Read the following passage and answer the questions that
follow:
The desire to preserve to future ages the memory of past achievements is a
universal human instinct, as witness
the clay tablets of old Chaldea, the hieroglyphs of the obelisks, our countless
thousands of manuscripts and
printed volumes, and the gossiping old story-teller of the village or the backwoods
cabin. The reliability of the
record depends chiefly on the truthfulness of the recorder and the adequacy of the
method employed. In Asia,
the cradle of civilization, authentic history goes back thousands of years; in
Europe the record begins much later,
while in America the aboriginal narrative, which may be considered as fairly
authentic, is all comprised within
a thousand years.
An examination of the aboriginal American calendars affords a good idea of the
comparative importance attached
by the Indian and by the white man to the same event. From the white man's point of
view many of the things
recorded in these aboriginal histories would seem to be of the most trivial
consequence, while many events which
we regard as marking eras in the history of the plain’s tribes are entirely
omitted. Thus, there is nothing recorded
of the Custer campaign of 1868, which resulted in the battle of the Washita and
compelled the southern tribes
for the first time to go on a reservation, while the outbreak of 1874, which
terminated in their final subjugation,
is barely noticed.
On the other hand, we find noted such incidents as the stealing of a horse or the
elopement of a woman. The
records resemble rather the personal reminiscences of a garrulous old man than the
history of a nation. They are
the history of a people limited in their range of ideas and interests, such
materials as make up the chronicles of
the highland clans of Scotland or the annals of a medieval barony.
It must be remembered, however, that an Indian tribe is simply a large family, all
the members being interrelated;
this is particularly true of the Kiowa, who number only about 1,100. An event which
concerns one becomes a
matter of gossip and general knowledge in all the camps and is thus exalted into a
subject of tribal importance.
Moreover, an event, if it be of common note in the tribe, may be recorded rather
for its value as a tally date than
for its intrinsic importance.
[ Edited and recreated from Calendar history of the Kiowa Indians. (1898 N 17 /
1895-1896 (pages 129-444),
by James Mooney]
21. According to the passage:
(a) Members of any Indian tribe are correlated with each other.
(b) Kiowa is not a part of an Indian tribe.
(c) An event was always recorded in an Indian tribal history solely for its
intrinsic importance.
(d) Europe has one of the earliest records of civilization.

. Page 7 of 36
22. Which of the gives a brief account of the passage?
(a) The preservation of history is a human instinct for future records. The
trustworthiness of the record depends
chiefly on the veracity of the recorder and the competence of the method that is
employed. Asia records the
latest genuine records followed by Europe, and the American history is more recent
with the study
examination of the aboriginal American calendars affords a good idea of the
comparative importance
attached by the Indian and by the white man to the same event. The white man’s
records omit details that
they considered trivial, and the records are more the personal reminiscences than
the history of a nation
limited in their scope of ideas and interests. But Indian tribes are one big clan;
a family, especially Kiowa.
(b) The preservation of history is a human instinct for future records. The
trustworthiness of the record depends
chiefly on the veracity of the recorder and the competence of the method that is
employed. Asia records the
earliest genuine records followed by Europe, and the American history is more
recent with the study
examination of the aboriginal American calendars affords a good idea of the
comparative importance
attached by the Indian and by the white man to the same event.
(c) The preservation of history is a human instinct for future records. The
trustworthiness of the record depends
chiefly on the veracity of the recorder and the competence of the method that is
employed. Asia records the
earliest genuine records followed by Europe, and the American history is more
recent with the study
examination of the aboriginal American calendars affords a good idea of the
comparative importance
attached by the Indian and by the white man to the same event. The white man’s
records omit details that
they considered trivial, and the records are more the personal reminiscences than
the history of a nation
limited in their scope of ideas and interests. But Indian tribes are one big clan;
a family, especially Kiowa.
(d) The preservation of history is a human instinct for future records. The
trustworthiness of the record depends
chiefly on the shrewdness of the recorder and Asia records the earliest genuine
records followed by Europe,
and the American history is more recent with the study examination of the
aboriginal American calendars
affords a good idea of the comparative importance attached by the Indian and by the
white man to the same
event. The white man’s records omit details that they considered trivial, and the
records are more the personal
reminiscences than the history of a nation limited in their scope of ideas and
interests. But Indian tribes are
one big clan; a family, especially Kiowa.
23. Which of the following word has the opposite meaning of ‘forget’?
(a) Subjugation. (b) Reminiscence. (c) Fugacious (d) Barony
24. Thus, there is nothing recorded of Custer campaign of 1868, which resulted in
battle of the Washita and
compelled the southern tribes for the first time to go on reservation, while the
outbreak of 1874, which terminated
in their final subjugation, is barely noticed. The author through the statements
wants to
(a) Highlight the discrepancy of the various historical events.
(b) Point out the fact that the white men omitted important events.
(c) Emphasis for the need for an authentic recording of the event.
(d) Negates the role of the Indian tribes in keeping the authentic historical
event.
25. The author believes that:
(a) Studying an aboriginal American calendar provides a good perspective on the
comparative importance of
events between the Indians and the white people.
(b) No parts of the history of the tribes are ever omitted.
(c) The Kiowa is a large tribe.
(d) Asia’s history doesn’t date back to much longer time periods than European or
American.
Passage(Q.26-Q.30): Read the following passage and answer the questions that
follow:
Once on the sidewalk, Mr. Peaslee turned to the right toward the house of his
neighbour, Mr. Edwards. Edwards
was a younger man than Peaslee, perhaps forty-seven. His business was speculating
in lumber and cattle, and in
the interest of this he was constantly passing and re passing the Canadian border,
which was not far from
Ellmington. In the intervals between his trips, he was much at home. He was a
stern, silent, secretive man, and
simply because he was so close-mouthed there was much guessing and gossip, not
wholly kind, about his affairs.

. Page 8 of 36
Mr. Peaslee found the front door of the Edwards house standing open in the trustful
village fashion, and, with
his well bonded neighbour he walked in without ringing with neighbourly freedom,
walked in without ringing.
He turned first into the sitting-room, where he found no one, and then into a rear
room opening from it. This
obviously was a boy's "den." On the table in the centre were a checkerboard, some
loose string, a handful of
spruce gum, some scattered marbles, a broken jack-knife, a cap, a shot-pouch, an
old bird's nest, a powder-flask,
a dog-eared copy of "Cæsar's Commentaries," open, and a Latin dictionary, also
open. In a corner stood a fishing#rod in its cotton case; along the wall were
ranged bait-boxes, a fishing-basket, a pair of rubber boots, and a huge
wasp's nest. Leaning against the sill of the open window was a double-barrelled
shotgun, and on the sill itself
were some black, greasy rags and a small bottle of oil.
Various truths might be inferred from the disarray. One was that Mr. Edwards was
generous to his son Jim, and
another was that there was no Mrs. Edwards. Further, it might be easily enough
guessed that Jim had been lured
from the study of Latin, in which pretty Miss Ware, who was his teacher at the
"Union" school, was trying to
interest him, by the attractive idea of oiling his gun-barrels, and that something
still more attractive—perhaps a
boy with crossed fingers, for it was not too late for swimming—had lured him from
that. At any rate, Jim was
not there.
Mr. Peaslee, still bent on finding Mr. Edwards, moved toward the open window. But
he could see no signs of
life anywhere. None of the household was, however, far away. Jim was in the loft of
the barn, where he was
carefully examining a barrel of early apples with a view to filling his pockets
with the best; the housekeeper had
merely stepped across the street to borrow some yeast, and Mr. Edwards, who had a
headache, was lying down
in the chamber immediately above Jim's den.
[ Excerpts from The Calico Cat, by Charles Miner Thompson]
26. Choose a suitable title for the passage above
(a) The curious Mr. Peaslee. (b) Mr. Edward and his his housekeeper
(c) Jim’s life. (d) The Neighbourhood diaries of Ellmington.
27. The author would agree with which one of the following?
(a) Mr. Edward was a man who liked travelling.
(b) Mr. Peaslee and Mr. Edwards had good terms as neighbours
(c) Jim was a studious lad.
(d) Mr. Edward had strict disciplinary rules for Jim.
28. Which of the following can be inferred from the passage?
(a) Canadian border is far from Ellmington.
(b) Mr. Edward in interval between his trips, he never stayed at home
(c) Mr. Edward had no wife, but only a housekeeper.
(d) Jim was very interested in taking Latin classes.
29. ‘He was a stern, silent, secretive man, and simply because he was so close-
mouthed there was much guessing
and Directions (Q.66 – Q.105): Read the comprehensions carefully and answer the
questions based on it.
Passage(Q.66-Q.71): We all have the legal right of peaceful enjoyment. Enjoyment
here means to be able to act
as one pleases, but obviously, without unreasonably interfering with another
person’s enjoyment. This tort of
nuisance arises where there is an unreasonable or undue interference in one’s
peaceful enjoyment of something
by another.
The undue interference is thus one which is unjustifiable. From the above, it is
sufficiently clear that malice is
not an important element in nuisance. That is because, once again, the foundation
of the Tort of Nuisance is the
interference with the enjoyment of another’s property and not the interference per
se.
Public Nuisance occurs when a person does an act which causes annoyance to the
public in general. In cases of
public nuisance, the persons affected have a common claim against the defendant.
Despite that, a claimant may sue the culprit individually. That particular claimant
will have to prove these things:
1. That he has suffered an injury which is greater than that suffered by the rest
of the public.
2. Such an injury must be direct and not merely consequential.
3. The injury must be substantial, not fleeting.
Private nuisance is when the act causing discomfort or inconvenience affects a
specific people.
This will depend on a) the sensitivity of the plaintiff, and b) the locality in
which the nuisance has occurred. An
act does not become actionable solely due to the higher sensitivity of the
plaintiff towards a normal act.
The defence of statutory authority applies to acts which are incidental to those
done in pursuance of a law.
However, this defence will be vitiated if the act was done with negligence, and a
claim of nuisance can be
maintained.
(Extracted with requisite revisions and edits from ‘Understanding The Tort of
Nuisance’ by Aparna Shukla at
https://lawctopus.com/clatalogue/understanding-the-tort-of-nuisance/ )
66. Uma has rented out her house to Mahesh so that he can live there. Mahesh
without the consent of Uma started
construction in her house. Uma was aggrieved by the construction undertaken by
Mahesh and asked him to stop
the construction. Uma said that Mahesh is interrupting her enjoyment of the house.
Mahesh did not stop the
construction. Uma filed a complaint against Mahesh for causing nuisance. Decide
whether Uma would succeed:
(a) Yes, as her enjoyment is interrupted.
(b) Yes, as Mahesh was causing undue interference.
(c) No, as Mahesh had no intent to cause interference.
(d) No, as Uma had no legal right of peaceful enjoyment.
67. Mehul has a pet dog. Rahul who is his neighbor does not like dogs and was
always irritated by Mehul’s dog.
Mehul’s dog used to bark a lot at night. Rahul was unable to sleep due to the
barking. He filed a case against
Rahul that he is causing nuisance. Decide whether Mehul would succeed:
(a) No, as Rahul always hated Mehul’s dog and Mehul has no ill-intent.
(b) No, as the dog did not enter on Rahul’s property to cause his disturbance.
(c) Yes, as the barking of dog caused Rahul undue interference.
(d) No, as Mehul did not cause any interference on his own.
68. In furtherance of the above stated passage, decide which statement is true:
(a) Nuisance can never be caused by statutory authority does its statutory duties.
(b) Nuisance cannot be caused against one’s own property.
(c) In public nuisance compensation cannot be granted to an individual.
(d) Interference without intention would not amount to nuisance.

. Page 18 of 36
69. The government department of electricity is vested with the duty to manufacture
electricity. The electricity is
produced by constructing a dam. Due to the construction, cement mixed in the air
and landed on the agricultural
land and house of the people residing nearby. They all filed a suit to claim
compensation stating that
compensation from electricity department. Decide whether people would succeed:
(a) No, as every person apply separately for nuisance caused to them.
(b) Yes, as it is a case of public nuisance.
(c) No, as no injury was caused to people.
(d) No, as the electricity department is performing its statutory duty.
70. In the last question, if the dam has been constructed by the electricity
department but suffers from normal wear
and tear. The department fails to do the maintenance of dam. One fine day, due to a
crack in the dam, the water
is flooded in Ramesh’s agricultural land and destroys his crops. Decide whether
Ramesh would succeed for
nuisance:
(a) No, as the department was performing its statutory duty.
(b) Yes, as the department acted negligently.
(c) Yes, as the department will not be treated differently just because it belongs
to government.
(d) No, as the damage was caused due to the normal process of wear and tear and
department did nothing wrong.
71. Manish and Mona are neighbors. Manish had heart issues and could not deal with
noises even a little. One day
Mona was watching news on television and Manish could not bear the volume of
television. Manish was so
much troubled by it that he had to be hospitalized. Manish filed a suit against
Mona for nuisance. Decide whether
Manish would succeed:
(a) No, as Mona was doing a normal act and did not cause any undue interference.
(b) Yes, as due to Mona’s act Manish was hospitalized.
(c) Yes, as Mona caused undue interference.
(d) Yes, as Mona should have been extra cautious considering the special condition
of Manish.
Passage(Q.72-Q.77): The Delhi High Court has clarified that the phrase "first
learns" used in the Limitation Act
1963 vide Articles 68 and 91(a) in its Schedule means "actual knowledge" of
misappropriation and not merely
"speculative knowledge."
Limitation under Article 68 pertains to specific movable property lost, or acquired
by theft, or dishonest
misappropriation or conversion. The period of limitation is 3 years, time from
which begins to run when the
person having the right to the possession of the property first learns in whose
possession it is.
Similarly, Article 91(a) pertains to wrongfully taking or detaining any specific
movable property lost, or acquired
by theft, or dishonest misappropriation, or conversion. The period of limitation is
three years, time for which
begins to run when the person having the right to the possession of the property
first learns in whose possession
it is.
In this backdrop, Justice Prathiba M. Singh held:
"the language `first learns' clearly means actual knowledge and not speculative
knowledge. The knowledge
cannot be by means of inference but ought to be clear knowledge. Such knowledge
should include the factum as
to the exact location of the movable property, as to who is in possession of the
same or the proceeds thereof."
On the aspect of Burden of Proof of proving actual knowledge, the Court citing
Standard Chartered quoted as
follows:
"Obviously where a person has a right to sue within three years from the date of
his coming to know of a certain
fact, it is for him to prove that he had the knowledge of the said fact on a
particular date, for the said fact would
be within his peculiar knowledge."
(Extracted with requisite revisions and edits from ‘Limitation Under Art. 68 &
91(a) Runs From Date Of Actual
Knowledge' Of Misappropriation: Delhi High Court’ by PrateekChakraverty at
https://www.livelaw.in/news#updates/delhi-high-court-limitation-act-article-68-91-
date-of-knowledge-of-misappropriation-190982)

. Page 19 of 36
72. Monesh owned a diamond necklace. He pledged the necklace to Jayant and
therefore, the necklace was in
custody of Jayant. Rakesh stole the necklace but Jayant was not aware about the
theft initially and found about
it later on. However, Monesh came to know about the theft on the day of the
offence. Monesh filed a case against
Rakesh for stealing his necklace. Decide till when can Monesh file the case:
(a) Monesh cannot file a case and Jayant can file a case.
(b) Three years since Rakesh stole the necklace.
(c) Three years since Monesh comes to know about the theft.
(d) Three years since Jayant comes to know about the theft.
73. In the last question, if Jayant plans to file complaint for the theft against
Rakesh. Then when can Jayant file the
complaint:
(a) After 3 years when Jayant got to know about the theft.
(b) Within 3 years from the date when Jayant got to know that property has been
stolen.
(c) Within 3 years from the date when Jayant got to know that stolen property is in
Rakesh’s possession.
(d) Within 3 years from when date when Jayant has reasonable doubt about property
being stolen.
74. In accordance with the above stated passage, decide which statement is true:
(a) The time limit fixed by Art 68 is not compulsory.
(b) Court has the discretion to decide the period of limitation under Article
91(a).
(c) The knowledge must be about exact place and person in whose possession property
is to start period of
limitation.
(d) Speculation of misappropriation formed on reasonable ground is enough to start
period of limitation.
75. X is the agent of Y who is authorized with his land to keep it in safe custody.
X leased the land to Z which is
against the directions of Y. Y come to know about the misappropriation of land by
X. Decide when Y can file a
complaint under Article 91(a):
(a) Within 3 years of knowledge of misappropriation to Y.
(b) The case cannot be brought under Article 91(a).
(c) Within 3 years of commission of misappropriation.
(d) No case can be filed as X is the agent of Y.
76. Rahul misappropriated a cheque that he found on a road on 04/03/17. He gets the
cheque cashed on 06/03/17
from bank. The cheque belonged to Manu who came to know on 15/03/17 that the cheque
is in the possession
of Rahul who has misappropriated it. Manu wants to file a case against Rahul.
Decide when can Manu file a
complaint:
(a) On or Before 04/03/2020. (b) On or Before 06/03/2020.
(c) On 15/03/2020. (d) On or Before 15/03/2020.
77. In the last question, when Manu files the case on 15/03/2020. Rahul contented
that Manu came to know that the
cheque is in the possession of Rahul before 15/03/2017. Decide who will have to
prove that when was knowledge
of possession received:
(a) Manu. (b) Rahul. (c) Police. (d) Court.
Passage(Q.78-Q.83): The Delhi High Court recently observed that a wife making
serious allegations of criminal
conduct against her husband and his parents and not being able to establish the
same amounts to cruelty as a
ground for divorce under the Hindu Marriage Act, 1955 (Neelam v. Jai Singh).
The Bench held the same while dismissing the wife’s appeal against the judgment of
a family court which had
granted a divorce decree to the husband and dissolved the marriage. The Court held,
"The mere fact that she made serious allegations of criminal conduct against the
respondent and his parents –
which she could not establish before the Court, was sufficient to constitute acts
of cruelty against the respondent."

. Page 20 of 36
The husband as well as his parents was subsequently taken into custody. Both the
husband and his parents were
acquitted in August 2015 and an appeal against the acquittal was also dismissed in
January the next year.
After the family court granted the husband a divorce decree on the ground of
cruelty, the wife approached the
High Court in appeal to seek restitution of conjugal rights under Section 9 of the
HMA which was also denied
on the grounds of cruelty.
“How can the respondent be expected to allow the appellant into his life in these
circumstances? The faith and
trust – which is the foundation of a matrimonial bond stood completely demolished
by the aforesaid conduct of
the appellant. For a man to see his parents to be taken into custody and being
incarcerated even for a single day
would have caused immense and untold pain and agony to him...Did she not know that
their conviction would
have led to their being sentenced to imprisonment? Therefore, her conduct of not
opposing the bail application
is neither here, nor there."
The Court, however, issued notice to the husband on the aspect of grant of
permanent alimony of the wife and
maintenance of their minor child.
Source: https://www.barandbench.com/news/litigation/wife-making-unproven-
allegations-of-criminal-conduct#is-cruelty-to-husband-delhi-high-court
78. In a case of divorce, filed by the wife, Ahaana, the contentions placed against
her husband by her were that he
was a drunkard and a molester. However, she failed to substantiate the allegations
against him, thus resulting in
him being exonerated of the charges but being granted the divorce all the same.
Should the divorce stand in this
case?
(a) Yes, as she could not substantiate her allegations, making divorce valid.
(b) No, as the grounds for divorce were flimsy and bound to be dismissed.
(c) No, as the grounds for divorce are not in accordance with the HMA.
(d) Yes, as a divorce requires no grounds to be granted.
79. In the above case, had the wife failed to substantiate the allegations of
cruelty as against her husband, and had
the same led to a petition for divorce being filed by the husband, would the pleas
of the wife for restitution of
conjugal rights be allowed ?
(a) No, as she filed serious allegations of criminal conduct and failed to
substantiate the same.
(b) Yes, as no allegation of cruelty could be made.
(c) No, as the husband has been deeply affected by the same.
(d) Yes, as the wife should be allowed means of sustenance.
80. In another case of domestic troubles, a divorce was filed for by the husband,
claiming that his wife had been
overly abusive of his mother and this amounted to cruelty. After the divorce had
been given on the same grounds,
the wife had suddenly fallen on hard times due to no source of sustenance for her
own endeavours. Thus, she
sought to file for restitution of conjugal rights in the court. Is the court liable
to grant her restitution?
(a) No, as the divorce was granted on firm grounds.
(b) No, as she can be given maintenance for her affairs.
(c) No, as the ground of cruelty has been satisfied against her.
(d) No, but she is entitled to maintenance.
81. Shubhra had brought allegations of rape against her husband and had obtained
divorce from him via the same.
Thereafter, due to not being given any maintenance, it had become difficult for her
to sustain herself on scraps.
Thus she demanded the court that restitution of her conjugal rights be made
possible for her to obtain a decent
standard of living. What relief can the court grant her?
(a) None, as she was the one who obtained the divorce.
(b) None, as she had received the relief by way of divorce.
(c) Adequate maintenance may be granted to her.
(d) Restitution of her conjugal rights will not be granted to her.

. Page 21 of 36
82. In a case of theft being alleged by the wife as against her husband and his
family members, the court had found
them not guilty of the same and had also held that these were arm twisting tactics
that were being employed by
the wife to extort money from the husband. After the dismissal of the case, the
husband had denied to live with
his wife forever and had thrown her out of the house. The wife, to counteract the
same, approached the court for
restitution of her conjugal rights as being his lawfully wedded wife. Would the
court oblige?
(a) Yes, as the couple is still married.
(b) No, as the wife has clearly exceeded the bounds of her conjugal relations.
(c) No, as the wife has perverse intentions that are apparent.
(d) No, as the wife made allegations of serious criminal conduct and could not
substantiate the same.
83. In the peaceful colony of Nanda Vihar, Kamal and Rosa used to reside, who were
a childless married couple
frustrated with each other. During a family outing, while dressing up for the
occasion, Rosa could not find her
necklace, leading to her bursting out on Kamal under the apprehension of him
stealing it. As such, she made a
huge ruckus and the neighbours in the vicinity heard it all. When the necklace was
found in Rosa’s cupboard
later on, Kamal, infuriated at her, decided to leave, never to come back. In this
case, can Rosa plead for restitution
of conjugal rights?
(a) Yes, as they have not been separated here
(b) No, as the couple are clearly not amicable and should remain separated.
(c) Yes, as no suit has been filed yet on false allegations.
(d) No, as the husband deserves to be separate from his wife on account of being
accused of theft.
Passage(Q.84-Q.89): The Delhi HC has directed all Principal District and Sessions
judges in the State to issue
circulars mandating that all concerned District and Sessions Judges in the State
have to intimate the Judicial
Officer and no one else about their intention to take leave at least one day in
advance, and the Judicial Officer
has to update the same on the Delhi District Court website immediately.
The communication by the Registrar General of the High Court also stated that the
circular should mandate that
in case of any hardship or unforeseen exigency or when leave is applied at the last
moment, intimation should
be sent through electronic means to the judicial officer so that the same is
received at the office of Principal of
District and Sessions Judge before 10 am, enabling the updation of the same on the
website for intimation of all
concerned.
"I am further directed to request you to issue direction for maintaining a Register
in your respective offices
containing following fields:
a) Name of the Judicial Officer
b) Date(s) for which leave is sought
c) Date and Time of receiving the leave application in the office
d) Date and time of uploading the intimation on the website," the communication
said.
Source: https://www.barandbench.com/news/litigation/delhi-judges-should-mark-their-
leave-one-day-in#advance-delhi-high-court
84. Once the above mentioned guidelines were passed, they were posted all over the
court complex for the judges to
get familiar with the same. One judge, Bhola J. had left early that day and was
unaware of the changed stance of
affairs. He had taken a leave the next day and the same was not notified until 11
am on the day, leaving a number
of advocates and their clients furious for not being heard. Would Justice Bhola
face a criminal liability?
(a) No, as he was unaware of the changed position.
(b) Yes, as ignorance of law is no excuse.
(c) No, he will not be held liable.
(d) Yes, as anything that the court utters becomes law.

. Page 22 of 36
85. After passing the above mentioned directions, the SC took suo motu cognizance
of the same and passed an order
mandating the same to be done by the judicial officers of every court complex. In
the Tis Hazari Court, the
judicial officer had done the same, and Justice Basudev had failed to report to him
about his absence, being
unaware of the position of law. Would the judge be criminally liable in this case?
(a) Yes, as the judge did not know the position of law in the matter.
(b) No.
(c) No, as judges have bureaucratic immunity in such cases.
(d) Yes, as the judge has been held in neglect of the position of law.
86. Judge RP Bhadoria of the Delhi HC was a very punctual and stringent judge known
for his stoic mannerisms
and expeditious hearings. After having accrued a decent number of vacation days, he
had finally decided to go
on a vacation to Maldives with his family. As a result of it, he failed to inform
the Judicial officer of the same,
which resulted in about a week’s worth of hearings not being addressed. Would he be
liable for negligence in
this matter?
(a) No, as he only took up his accrued vacation days.
(b) No, as he is a HC judge.
(c) Yes, as all judges are bound by the directives mentioned above.
(d) No, as the directives are not meant for judges who have a good record.
87. In the above case, had Justice Bhadoria been a District court judge, and had he
sent in a message for his vacancy
one day prior to the judicial officer, only to have him take notice of the same
post 11 am the next day, would he
be held liable for non-compliance of the directive?
(a) Yes, as he did not report his absence in time.
(b) No, as he had sent in the intimation timely.
(c) Yes, as the judicial officer would be liable here for not taking note of the
same.
(d) No, as the present matter does not clearly show either as being negligent.
88. In the middle of a crucial hearing, the Judge had to leave owing to a family
emergency, and had adjourned the
further matters. He had no time in the next two days due to his wife having been
admitted to a hospital due to
cardiac arrest. This led to his absence of two days from the court without any time
for him to notify the officer
of the same. Would he be held liable for this absence?
(a) No, as the matter was urgent and crucial.
(b) Yes, as the Judge should have left a message making his absence known.
(c) No, as the Judge was preoccupied completely and had no chance of reporting the
same.
(d) Yes, as his absence would have caused disruption, wasting court’s time..
89. Judge Sharma had to take a leave of absence on one particular day, and after
informing the Asst. Judicial officer
of the same, he had left. The assistant officer failed to inform the Judicial
officer of the same, which led to a
plethora of complaints being tendered to the officer for not reporting the same on
time. Who shall be held liable
in this case?
(a) The assistant shall be held liable due to his negligence.
(b) The Judicial officer shall be held liable for not reporting the same.
(c) The Judge shall be held liable for not duly reporting his absence.
(d) No one shall be held liable in this case.

. Page 23 of 36
Passage(Q.90-Q.95): There is a need to improve legal literacy among general public
including educated people,
Supreme Court judge Justice AS Oka has stated, stressing that courts cannot go by
public sentiments when
delivering judgments and have to be strictly in accordance with established laws.
General public has to be made aware about the processes involved in law, Justice
Oka emphasized.
In this regard, he also pointed out how judges are criticized on public platforms
and social media. “We find that
after court decisions, there are remarks online which can't be read in court. You
suddenly realize it's your order
being talked about on social media," the judge quipped.
The judge said that lack of legal literacy has especially meant that educated
people do not understand the reasons
for pendency of cases, the difference between bail and acquittal, and that
sentencing in criminal courts cannot
be bound by sentiments of public.
He, therefore, urged the Bar Associations to incorporate the need of improving
legal literacy among the general
public in their code of ethics. The judge also said that his personal view is that
live-streaming of court
proceedings should be allowed but in a limited manner.
Source: https://www.barandbench.com/news/courts-cannot-give-judgments-based-public-
sentiments-supreme#court-judge-as-oka
90. After the above case, once the judgment was delivered, another case about
inter-community riots had come up
before the court. In order to deliver a judgment that sates both the communities,
the judge delivered a pacifist
natured judgment in accordance with laws. This was heavily criticized as the media
said that the judge was
influenced by the socio-legal environment of the country. Has the judge erred in
rendering the judgment?
(a) No, as the judge was right in not infuriating the riots.
(b) Yes, as the judge was influenced by the situation.
(c) Yes, as the judge was influenced by his overwhelming emotions.
(d) No, as the decision was in accordance with laws.
91. In the above case, the judge had been informed that there were radicals outside
the courthouse ready to pelt him
with stones. After which, he had delivered the same judgment, which resulted in the
radicals vacating the
courthouse premises. This was interpreted as an act of emotions overwhelming the
judge, leading to him being
criticized for the same. Is this a case of apparent emotional bias?
(a) No, as the judge was not influenced by the radicals.
(b) Yes, as the judgment is a clear product of the judge being harassed.
(c) Yes, as the judge bent in a manner that would placate the radicals.
(d) No, as the judgment was in accordance with laws.
92. The High Court of Madhya Pradesh, had sentenced AZB, a murder accused, to
death, for having bombed the
Bombay Central Railway station, killing 109 people in the name of Jawanism, his
religion. Whilst a small sect
by universal standards, Jawanism had garnered a decent following in India with
nearly 200,000 people following
the same. His matter was now under appeal before the SC which had reduced the death
sentence to a life sentence
fearing civilian retribution from other jawanists, clearly in violation of laws. Is
the SC right in doing so?
(a) No, as the SC clearly diverted justice under influence.
(b) No, as the SC bowed down to an unruly evil in this case.
(c) No, as the court has not given the judgment in accordance with laws.
(d) No, as the SC has clearly undermined the HC’s decision illegally.

. Page 24 of 36
93. Sudha was a woman, who was raped and then murdered in a backwater village of
Jhabua, resulting in the case
being heard by the HC who had declared the same as being liable for death sentence
being given to the accused.
Her parents were being pressured into redacting the case from the HC or facing the
brunt of the goons in cahoots
with the accused. The death sentence was executed, and as a result of the same, her
parents were also murdered.
This was made to be a red-letter day, as the court was being criticized for not
considering the position of the
parents in this case. Has the court erred in delivering its judgment?
(a) No, as the judgment was strictly in accordance with the laws.
(b) Yes, as the major driving cause in the case should have been the further lives.
(c) Yes, the court should have firstly considered the position of the parents.
(d) No, as the court did not deviate from delivering the judgment.
94. SujivWalia was a business tycoon in India, employing 10000 people within India,
leading the country in
employing the people. He was convicted recently in a banking scam, held liable for
defrauding a consortium of
banks out of 700 crore rupees in the last quarter. After being convicted, his
company ran into insolvency and
about 8500 people were rendered unemployed, leading to a massive offload resulting
in an economic turmoil.
The SC was appealed to in order to overturn its judgment in convicting him. Should
the Court do so?
(a) No, as court cannot deliver judgments on public sentiments.
(b) No, as the court is not an arbitral tribunal.
(c) Yes, as the court should consider the situation of turmoil in the country.
(d) Yes, as the court is supposed to take steps to further peace and tranquility.
95. Judge Victor, a SC judge and Joris Bohnson, the English minister of foreign
affairs, were childhood friends as
they had gone to school together in Sweden. In an unfortunate turn of events, Joris
was implicated in a matter of
defrauding the Indian government in a bilateral treaty, thereby making him an
accused answerable to the SC of
India. Hearing of this, Victor was infuriated and wanted his friend to answer for
his crimes. He interferedt in on
the bench, and held him guilty of the same. Is this action of Justice Victor
correct?
(a) Yes, as he ensured that justice was swiftly done.
(b) No, as he has violated the laws of justice.
(c) No, as he has not given the judgment in accordance with the laws.
(d) Yes, as Joris deserved to be brought to justice.
Passage(Q.96-Q.101): The Supreme Court has recently observed that a Family Court
cannot deal with a criminal
complaint about offences under the IPC. As per the Family Courts Act, a family
court can exercise the
jurisdictions exercisable by any district court of any subordinate civil court
under any law for the following#1. A proceeding between the parties to a marriage
for declaring the marriage to be null and void or, annulling
the marriage or restitution of conjugal rights or judicial separation or divorce or
dissolution of marriage.
2. A proceeding for a declaration as to the validity of a marriage or as to the
matrimonial status of any person.
3. A proceeding between the parties to a marriage with respect to the property to
the parties or of either of them.
4. A proceeding for an order or injunction in circumstances arising out of a
marital relationship.
5. A proceeding for a declaration as to the legitimacy of any person.
6. A proceeding for maintenance.
7. A suit or proceeding in relation to the guardianship of the person or the
custody of, or access to, any minor.
It can also exercise the jurisdiction exercisable by a Magistrate of the first
class under Chapter IX (relating to an
order for maintenance of wife, children and parents) of the CrPC.
https://www.livelaw.in/news-updates/supreme-court-family-court-cannot-try-criminal-
cases-ipc-wrong-order#179433

. Page 25 of 36
96. After the divorce of a couple, Shardul, their child, was embroiled in a dispute
for custody of his person. The
mother made certain allegations regarding his father’s abhorrent behaviour and
excessive drinking. The father
made allegations about his mother’s philandering and foul-mouthedness. Is this
dispute triable by the Family
courts?
(a) The dispute is triable by the Family Courts.
(b) The dispute is triable by Criminal courts.
(c) The dispute is triable by both courts.
(d) The dispute is not triable by any of these courts.
97. KKD and BBC are to brothers involved in a property dispute wherein they were
equal shareholders of the land.
However, KKD wanted to sell the same because during a burglary in their home, one
of the robbers had killed
the other, marking the property as former scene of crime. BBC however, is not of
the opinion for selling it and
wants to retain it. Can the family courts entertain the same?
(a) No, as the property in the question is a scene of crime.
(b) No, as there is no consensus between the principal owners of the property.
(c) Yes, as it falls within the jurisdiction of the family courts.
(d) No, this does not fall within the jurisdiction of family courts..
98. Anjali had assumed her husband dead until recently, when in a prisoner
exchange, the warring state had released
the list of POWs. She had been waiting for him to return for 8 years, after which
she had him declared legally
dead and remarried. The husband filed a case against Anjali on the grounds that her
second marriage is
illegitimate. Can the family courts rule on this subject matter?
(a) Yes, as this is a suit related to validity of a marriage.
(b) No, as this is a suit concerning warring nations and should be tried by army
courts.
(c) Yes as this is a suit for restitution for conjugal rights.
(d) Yes, as this suit is related to declaring legitimacy of a person.
99. In the above case, had the husband not been a POW but a criminal in the warring
state absconding from the
police there, and had come back after being missing for 8 years. Would the matter
then be considerable by the
family courts?
(a) Yes, as the matter is of validity of marriage.
(b) No, as he has not been presumed dead.
(c) No, as being a criminal, he is firstly to be tried by a criminal court.
(d) Yes, as the matter remains to be of restitution of conjugal rights.
100. Surjana left her husband, Jagga, for abusing her physically for years and
years on end. Now the matter has come
up in the family courts of Delhi for the same. Can the Family Courts entertain the
matter?
(a) No, as the matter is primarily criminal in nature.
(b) Yes, as the matter is primarily of family courts.
(c) Yes, as the matter has already been initiated in a family court.
(d) None of the above.
101. In the above case, had the wife actually filed for divorce, and had filed for
maintenance post-divorce, which
courts would have the requisite jurisdiction to entertain the same?
(a) The family court would have the requisite jurisdiction.
(b) The trial court would have the requisite jurisdiction.
(c) Any court of first instance can entertain a matter of domestic abuse.
(d) For the matter of divorce and maintenance, family court will have jurisdiction,
but for cruelty only a criminal
court will have jurisdiction.

. Page 26 of 36
Passage(Q.102-Q.105): The Madras HC urged the TN Bar Council to exercise its
suomotu powers against
lawyers found acting to demean or prejudice the legal fraternity even if no formal
complaint is lodged.
"The Tamil Nadu Bar Council in consultation with the Bar Council of India, in the
larger interest of the legal
fraternity, shall look into the issue of evolving a mechanism for initiation of
suomotu proceedings against such
of those members, who indulge in activities, which are prejudicial and demeaning
the interest of the legal
fraternity as a whole", stated the order.
This led the Court to express concern that the Bar Council seldom uses its suomotu
powers to act against lawyers
for unprofessional conduct.
"Any unprofessional conduct of a member of the legal profession, coming to the
knowledge of the Bar Council
through the visual media for which no complaint emanates from any quarter, can the
Bar Council allow that
instance to go unnoticed for the mere reason that the Advocates Act does not
envisage suomotu action. It
observed that instances of advocates going beyond their brief are on the increase.
"Though this Court cannot give any positive direction to the Bar Council to do one
thing or the other, as it is not
vested with jurisdiction ... it is high time the Bar Council enforces the power and
authority to initiate action
suomotu on the incidents, which comes to its knowledge through the digital/print
media, for which there is no
complaint given by any individual," the order said.
"Advocates should not take law into their own hands on the premise that they are
the custodian of the law; on
the contrary, the advocates being the custodian of law, are bound to act within the
legal framework, even if there
is a violation of law and establish the rule of law through the well-defined
mechanism."
102. Ramesh, an advocate in Delhi, during the period of lockdown, went outside his
house to purchase medicine.
Police officials on duty stopped Ramesh while he was proceeding in his car. It was
found out that Ramesh was
not in possession of a valid pass for going out during the lockdown period. Ramesh
quarrelled with the police
officials on duty. He, while claiming that he is an advocate, castigated the police
officials on duty and threatened
them that they would be stripped off of their uniforms if they tried to intervene
and cause hindrance to the
movement of the petitioners.
(a) The High Court of Delhi cannot issue directions to the Bar Council of Delhi to
act in a certain way.
(b) The Bar Council of Delhi must not initiate suo-motu proceeding against Ramesh
because Ramesh is a part
of the legal fraternity and does not envisage suo-motu actions
(c) The High Court of Delhi vests with the jurisdiction of issuing directions to
the Bar Council of Delhi to act
in a certain way.
(d) The bar council has to initiate an action if the high court orders for same.
103. The quarrel hereinabove was recorded by some passer-bys and was uploaded on
Twitter tagging the Police Dept.
The Tweet was quoted as saying, "Delhi Police hampering the movement of a free
individual, disgraceful
conduct by law maintain authorities". Delhi police now wants to initiate an action
against the people tweeting,
advise them so.
(a) Delhi Police can initiate action for Defamation against Ramesh and the
tweeters.
(b) Delhi Police can initiate action for Defamation against the tweeters only.
(c) Delhi Police can urge the Bar Council to take Suomoto cognisance of the matter
herein as it happened on
social media, and no complaint arose from it.
(d) None of the above.

. Page 27 of 36
104. During a hearing in the Delhi HC, while the matter was being heard, one of the
counsels for the respondents
started heatedly arguing, often bordering on slander, as against the petitioners.
This led to the counsel for
respondent ultimately implying that the petitioners are two-bit thugs who are just
here to earn unlawfully from
damages. This was noted by the judge during the hearings, and the counsel was
reprimanded for the same by the
judge in a passing manner. However, the petitioners were highly offended by this
and implored the judge to
direct the Bar Council to take action against them separately for this slur. Would
they be right in doing so?
(a) Yes, as the counsel herein used a highly derogatory racial slur that insinuated
that the petitioners were of no
character.
(b) Yes, as the comments made herein were so inherently derogatory as to render the
petitioners slandered and
the respondents liable to be punished by the Council.
(c) No, as the courts do not hold the jurisdiction for the same..
(d) No, as the suomotu power of the Bar Council is not an appellate mechanism.
105. During a divorce case the counsels from both the sides start arguing fiercely,
each trying to get their clients the
majority share in their joint assets. This leads to several bouts of heated
exchanges wherein the counsels heavily
bantered, often insinuating depravity and hurled heavily veiled insults at each
other. During one of these
exchanges, one of the counsels casually mentioned that 'women cheat and are
casanovas', and the judge, a female
of high renown, was offended at this and held him in contempt of the Court,
pleading the bar to take strict action
against lawyers who casually demean the opposite gender in a slanderous manner. Is
this action of the Court
liable to be struck down?
(a) No, as the counsel is displaying unprofessional conduct. The Court has rightly
pleaded the Council to take
cognisance of it.
(b) No, as the judge herein has rightly exercised her judicial expertise and
discretion while pleading the Council
to take action.
(c) Yes, as the Courts do not have the power to implore the Council to take action
where the cause of action was
very casual such as this.
(d) None of the above.

. Page 28 of 36
SECTION - D: LOGICAL REASONING
Passage(Q.106-Q.110): Opposing the PILs related to “marital rape”, the Centre has
told the Delhi high court
that, among other things, misuse of Section 498 of the IPC related to dowry
harassment and lack of a mechanism
to verify when consent stands withdrawn by the wife are concerns because of which
India should move
cautiously and not follow other countries blindly on the issue. This has been
conveyed to the court in a written
submission.
“Various other countries, mostly Western, have criminalised marital rape but it
does not necessarily mean India
should also follow them blindly. This country has its own unique problems due to
various factors like literacy,
lack of financial empowerment of a majority of females, mindset of society, vast
diversity, poverty, etc and these
should be considered carefully before criminalising marital rape,” the government
submitted earlier this month.
It has pointed out that marital rape is not defined in any statute or law. While
rape is defined under Section 375
of IPC, “defining marital rape would call for a broad-based consensus in society.
As to what constitutes marital
rape and what would not constitute marital rape needs to be defined precisely
before a view on its criminalisation
is taken,” it said.
Solicitor General Tushar Mehta had told the HC that the Centre was considering a
“constructive approach” to
the issue and has sought suggestions from several stakeholders and authorities on
comprehensive amendments
to the criminal law. He had said criminalisation of marital rape involves “family
issues” as well as the dignity of
a woman and cannot be looked at from a “microscopic angle” and sought some more
time. But the bench made
it clear it would continue hearing other stakeholders till the Centre opens its
arguments.
106. ‘Various other countries, mostly Western, have criminalised marital rape but
it does not necessarily mean India
should also follow them blindly.’ Choose the most logical conclusion from the given
statement.
(a) India looks to Western countries for setting legal precedents.
(b) Western countries have criminalised rape after following some other countries.
(c) India always looks at Western countries as precedent.
(d) Indian law is highly influenced by the West.
107. Which of the following, if true, would most likely weaken the author's
statement that Centre has sought
suggestions from several stakeholders and authorities on comprehensive amendments
to the criminal law.
(a) The Centre always does whatever it wants, but this time it is trying to make
collective decisions.
(b) The Centre is trying to influence the decision of several authorities against
the court.
(c) The Centre has sought the suggestions of stakeholders and authorities that toe
the Centre’s line.
(d) The Centre wants to involve various stakeholders and authorities before
committing to amending the criminal
law to avoid far reaching repercussions.
108. Which of the following reflects the central theme of the passage?
(a) Court's ambition to make martial rape a criminal offence is an inspiration from
the West.
(b) The central is not against criminalising marital rapes, but does not want to
shoulder the responsibility.
(c) The centre wants the court to move cautiously and consider various factors
before criminalising marital rape
as India is different from the western countries.
(d) The Court’s decision of criminalising marital rape will affect a lot many
marriages adversely.
109. Consider the following statement from the passage and answer accordingly.
Assertion (A): Solicitor General Tushar Mehta had told the HC that the Centre was
considering a “constructive
approach” to the issue and has not sought suggestions from several stakeholders and
authorities on
comprehensive amendments to the criminal law.
Reason (R): He had said criminalisation of marital rape involves “family issues” as
well as the dignity of a
woman and cannot be looked at from a “microscopic angle” and sought some more time.
OPTIONS:
(a) A is true and R is false.
(b) A is false and R is true.
(c) Both A and R is true and R is the correct explanation of A.
(d) Both A and R is true and R is not the correct explanation of A.

. Page 29 of 36
110. Which of the following, if true, most strongly supports the argument?
(a) Marital rape law must be considered after much deliberation and broad-based
consensus.
(b) Martial rape law should not exist in India looking at the diversity in culture.
(c) Marital rape is a serious offence and should have a well-defined law against
the criminal offence.
(d) The court is the keeper of law, and it will base its decision using utmost
discretion.
Passage(Q.111-Q.115): It is slow work — subverting independence. That the work is
in progress was indicated
by the remarks of the Union law minister, Kiren Rijiju, about ‘coordination’ of
constitutional bodies with the
executive. As the law minister, he cannot have any doubt regarding the importance
of independence of
constitutional bodies such as the Election Commission. The whole point of free and
fair elections is defeated —
and democracy made into a travesty — if the EC, for example, acts in coordination
with the law ministry headed
by an elected politician. But it is precisely this that formed the context of Mr
Rijiju’s remarks; he praised the EC
for its cooperation after a meeting with the election commissioners that he had
called. The impropriety of the
proceeding had been criticized by members of the Opposition and the former chief
election commissioner, S.Y.
Quraishi. The EC may call officials for administrative tasks, but its commissioners
cannot be summoned by the
executive. Moreover, the ‘coordination’ was in the matter of a controversial
change. The election laws
(amendment) bill, 2021 links Aadhaar with voter rolls which, according to the
Opposition, is likely to
disenfranchise a number of voters apart from breaching privacy.
So the context of the Union law minister’s paean to coordination defines what he,
or his government, means by
the term. He has even spoken to the judiciary about the importance of coordination;
should justice ‘coordinate’
with the executive then? The independence of officials of autonomous institutions
was being undermined for
quite some time, whether through the delay in appointing the necessary number of
vigilance commissioners for
the Central Vigilance Commission or through amending the law so that information
commissioners in the Central
Information Commission became dependent on the government for their tenure and
salaries. Such moves add
more weight to the criticism from the former Supreme Court judge, Madan B. Lokur,
Mr Quraishi and others
that followed Mr Rijiju’s praise of coordination. They pointed out the lack of
checks against ‘executive
overreach’, focusing on the fact that, unlike the CEC, the other two election
commissioners do not have
constitutional protection from removal and therefore feel insecure. The process of
their selection and
appointment is faulty, being dependent on the executive. But that is what this
government desires; executive
overreach was implicit in the law minister’s high-minded benevolence.
111. According to the author, what might be the intention of the law minister in
making such a remark?
(a) The law minister wants more interference by the executive in the EC.
(b) The law minister wants the EC to surrender itself to the hands of the
Executive.
(c) The law minister should not care about the opinions made by the author.
(d) The law minister wants specific boundaries to be drawn between the EC and the
executive.
112. Which of the following is strongly supported by the information given in the
passage?
(a) The criticisms made by personalities like Madan Lokur have no real value.
(b) An elected politician is not meant to be a person of honesty and integrity.
(c) Giving constitutional protection to an officer makes him/her more independent.
(d) Only very few elected politicians are there who really want to serve the
country.
113. Out of the following, which is the best representation of the author’s opinion
about the coordination that the law
minister has remarked?
(a) The coordination between the executive and judiciary would adversely affect
democracy in India.
(b)The coordination would make the democracy of India more vibrant and healthy.
(c)The coordination remarked by the law minister would not be good for India’s
democracy.
(d)The coordination between the bodies referred by the law minister would make
India an unruly state.

. Page 30 of 36
114. With which of the following would the author not agree?
(a) The government of the day desires the Election Commission to be independent.
(b) The post of an EC Commissioner is too powerful to be summoned by the executive.
(c) The coordination of the constitutional bodies is the underlying strength of
democracy.
(d) The Election Commission as the constitutional body must be given an independent
status.
115. What is the role played by the following statement in the passage? “Persons
appointed as the law minister are
expected to be well read about the constitutional bodies.”
(a) This statement is a counter premise.
(b) This statement is likely to be an assumption made by the author.
(c) This statement would weaken the arguments made by the author.
(d) This statement is the premise of the passage.
Passage(Q.116-Q.120): Russian Foreign Minister Sergey Lavrov’s comment, after
receiving a written response
from the US to Moscow’s security demands, that ‘Russia doesn’t want wars’ raises
hopes of a diplomatic solution
to the Ukraine crisis. Last week saw a flurry of diplomatic activities aimed at de-
escalating the situation. Besides
the U.S.’s written response to Russia, which could set the stage for further
diplomatic talks in the coming weeks,
French President Emmanuel Macron has held talks with his Russian counterpart
Vladimir Putin. Mr. Macron
and German Chancellor Olaf Scholz are also reportedly trying to revive the stalled
Minsk process that sought to
find a peaceful solution to Ukraine’s internal conflict between Kiev and the
Russia-backed separatists in the
eastern Donbas region. Mr. Putin has said the U.S. response does not address
Russia’s core security concerns.
Fears of a military conflict are still there. Russia, which has mobilised thousands
of troops on its border with
Ukraine, in Belarus in Ukraine’s north and in Transnistria (a breakaway region from
Moldova) in its south-west,
has a clear military advantage. But the West’s willingness to press ahead with
diplomatic options and Russia’s
reciprocity suggest that neither side is in a hurry for armed conflict.
There is a strong case for de-escalation. Mr. Putin has already achieved many
things, without a shot being fired.
He has got the Western leadership to talk to him over the contested issue of NATO’s
expansion, which Russia
has long been complaining about. By putting a gun to Ukraine’s head, he has
effectively drawn a red line in
Russia’s relations with the West. Also, as the U.S. and its allies are scrambling
for economic measures to punish
Russia “if it invades Ukraine”, the 2014 Russian annexation of Crimea and Moscow’s
continued support for
separatists inside Ukraine have practically become non-issues. Nobody is
threatening to punish Russia for not
returning Crimea to Ukraine. The status quo has been redrawn. Besides, the U.S. has
said it is open to discuss
some of Russia’s security concerns, including missile deployments in Eastern Europe
and military exercises in
sensitive regions. Russia should accept the U.S. proposals, de-escalate the crisis
around Ukraine and opt for more
dialogue on critical issues, including NATO’s eastward expansion. If it still goes
ahead with an attack, which
U.S. President Joe Biden said could happen, it could well be a mistake. Russia
might be in a position to overrun
Ukraine militarily, but what comes next in Europe’s largest country is as
unpredictable as it can get. If the post#9/11 military adventures of the U.S. are
any lesson, it is that great powers could clinch swift victories against
weaker countries but could fail miserably in sustaining those victories. Mr. Putin
should not walk into the same
mistake and thereby push Europe back into the darker days of the Cold War.
116. Which of the following best sums up the main idea of the passage?
(a) Russia should do more to reduce tensions along its border with Ukraine.
(b) Russia should be mindful of the West before attacking Ukraine.
(c) Russia would do well to de-escalate and solve its issues with the US
diplomatically.
(d) Ukraine should bandwagon with its neighbours to take on Russia.
117. The information given in the passage most strongly supports which of the
following inferences?
(a) The Minsk process will be completed in the foreseeable future.
(b) Avoiding a conflict between Ukraine and Russia is good news for India.
(c) Russia can learn from the mistakes committed by the US in some regard.
(d) The European front has a tendency to accommodate Russia in most matters.

. Page 31 of 36
118. What is the opinion of the author regarding the U.S. proposals to Russia?
(a) Russia is left with much to desire for; the proposals won’t be much.
(b) Acceptance of the U.S. proposals by Russia is advisable.
(c) There is nothing more that Russia could have asked for.
(d) Russia, being a powerful military, knows how to tackle such situations.
119. Out of the following, which statement would provide strength to the author’s
arguments?
(a) Russia has downsized its military personnel by 30% to control its economic
downfall.
(b) The U.S. dealt with the perpetrators of the 9/11 attacks with a heavy blow.
(c) Last week, a large team of diplomatic personnel were sent by Russia to Europe
to talk about Ukraine.
(d) Much can be attained with coercion and threat as has been in the case of Russia
and Ukraine.
120. With which of the following would the author of this passage disagree?
(a) Russia will go ahead and attack Ukraine with full force.
(b) Ukraine’s stature in Europe is among the least preferred nations.
(c) Russia is not comfortable with NATO’s eastward expansion.
(d) Both a and b are correct
Passage(Q.121-Q.125): Western Uttar Pradesh, which goes to the polls in the first
phase of the Assembly
elections in the state, has been plunged into a poll campaign marked by the most
undesirable forms of communal
and casteist rhetoric and mobilisation. This region of the state has much
significance in this round of elections
because of the churn that seems to have happened after the last state polls. The
Jats, who are the dominant
community in many constituencies, had supported the BJP in the last elections. But
the community has been
estranged from the party after the year-long farmers’ protest. The alliance between
the Samajwadi Party (SP)
and the Jat-dominated Rashtriya Lok Dal (RLD) has emerged as a strong challenger to
the BJP. The BJP had
won 52 out of the 71 seats in the region 5 years ago. The SP had won 16. If the Jat
votes shift to the SP-RLD
alliance, the BJP is bound to suffer losses.
That possibility has led the BJP to unleash a blatantly communal and casteist
campaign in the region. The Union
Home Minister has led the charge, and he has made some most unseemly and
inappropriate statements, unworthy
of his position. He made the astounding statement that the “ideology of both the
BJP and Jats is the same as both
have fought the Mughal invaders’’. Was the Home Minister of the country using
“Mughals” to mean today’s
Muslim citizens? He also made references to the communal riots in a UP district
that happened around a decade
ago in a way that seemed to hold out a threat that if the BJP was not voted into
power, there would be a repeat
of those riots. The Prime Minister used his virtual rally on Monday to remind the
people of the riots. The BJP
leaders are trying to woo the Jats back with promises, warnings and references to
the riots and the most ridiculous
claims.
The Home Minister, who has the responsibility to maintain peace and amity between
communities and to take
action against those who disrupt the peace, is himself trying to create dissensions
between sections of people.
They raise moral and political issues that make his continuance in the government
untenable. The Chief Minister
and several Union and state ministers have also made the most divisive statements
and tried to create hostilities
between communities. It is unfortunate that the Election Commission of India has
not found any of the statements
worthy of action. Appeals to religious and caste sentiments violate the model code
of conduct and should attract
firm action from the commission. Successive elections have seen a progressive
deterioration in the standards of
campaigning. Unfortunately, they have seen a deterioration in the functioning of
the Election Commission, too.

. Page 32 of 36
121. Which piece of information would be helpful to know while evaluating the
author’s argument(s) in the second
paragraph?
(a) Was the BJP in existence in the 16th and 17th centuries to fight the Mughals?
(b) Whether the Election Commission has the power to take action against misdeeds?
(c) Whether unleashing a blatantly communal and casteist campaign has led to
victory for the concerned party?
(d) Whether Jats will always remain on the side of BJP irrespective of the
differences.
122. The main idea of the passage is best represented by:
(a) The Election Commission’s inaction on the Home Minister’s untenable position is
unfortunate.
(b) The Home Minister would do well to take his unfortunate remark back if he wants
peace in the country.
(c) The politicians' way of campaigning in poll states - based on religion and
caste, needs to stop.
(d) Communal harmony is disrupted when the Election Commission does not take action
against the evil.
123. Which among the following courses of action would the author of this passage
logically agree with?
(a) The Election Commission disqualifies the BJP from conducting polls in the
upcoming elections.
(b) The populace of Uttar Pradesh does not elect the BJP as their elected
representatives in the polls.
(c) The BJP fulfils all the promises made by its representatives to the Jat
community before elections.
(d) None of the above is correct
124. Out of the marked statements, choose the statement that is the odd one out.
Use the concepts of Logical
Reasoning to answer this question.
(a) The Home Minister, who has the responsibility to maintain peace and amity
between communities and to
take action against those who disrupt the peace, is himself trying to create
dissensions between sections of
people.
(b) They raise moral and political issues that make his continuance in the
government untenable.
(c) The Chief Minister and several Union and state ministers have also made the
most divisive statements and
tried to create hostilities between communities. It is unfortunate that the
Election Commission of India has
not found any of the statements worthy of action.
(d) Successive elections have seen a progressive deterioration in the standards of
campaigning.
125. If the information given in the passage is true, it logically follows that
I. Deterioration in the functioning of the Election Commission spells doom for
India’s democracy.
II. Winning back the trust of the Jat Community would win the UP election for the
BJP.
III. The Jat Community was one community that took part in the year-long farmer’s
protest.
(a) Both I and III (b) Only III (c) Both II and III (d) All I, II and III
Passage(Q.126-Q.130): The CPM-led Kerala government’s decision to bring an
ordinance to limit the powers
of the anti-corruption watchdog is questionable. By turning the quasi-judicial
institution into a toothless advisory
body, whose orders will no longer be binding on the government and could be
appealed in a court of law, the
ordinance, which is awaiting the governor’s assent, will effectively neutralise the
Lokayukta Act, a model
legislation.
The Opposition has spoken out against the proposed amendment and alleged that it is
aimed at containing the
political fallout in the event of the Lokayukta passing adverse orders on
complaints against Chief Minister
Pinarayi Vijayan and Higher Education Minister R Bindu. Ahead of the assembly
elections in March last year,
the then higher education minister, KT Jaleel, was forced to resign following a
ruling by the Lokayukta: In a
case pertaining to an allegedly illegal appointment of a relative of the minister
in the state minorities corporation,
it had held that Jaleel’s conduct was unbecoming of a minister. Bindu has been in a
spot ever since Governor
Arif Mohammad Khan publicly spoke about her writing to him in connection with the
reappointment of a vice#chancellor. A complaint has been filed against Bindu before
the Lokayukta in the VC appointment case. The
government has refuted the Opposition’s charge that the ordinance has been drafted
to protect the ministers and

. Page 33 of 36
has claimed that the proposed changes are in line with two high court rulings. But
its defence appears
unpersuasive. Its attempts to whittle down the powers of the Lokayukta also fly
against the CPM’s own stated
position that a strong Lokpal at the Centre and empowered Lokayuktas in the states
are necessary to check public
corruption. In a 2011 statement, the party had said that the “battle against
corruption, in order to be effective
today, can be achieved only through a comprehensive reform of our political, legal,
administrative and judicial
systems and not through one-off or piece-meal measures. The establishment of an
effective Lokpal institution is
one such measure.” It also argued that Lokayuktas should be set up in the states
“on the lines of the Lokpal” with
“all state government employees, local bodies and the state corporations under
their purview”. CPM general
secretary Sitaram Yechury reiterated this position several times in Parliament.
The Kerala government needs to recall the Lokayukta ordinance. Failing to do so
could be viewed as a
compromise on the part of the Left towards strengthening institutional checks
against public corruption.
126. What role does the following piece of evidence play towards the arguments made
by the author in the passage?
“No provision of appeal in the Lokayukta law gives it unreasonable powers and
authority to make arbitrary
decisions.”
(a) This statement would likely weaken the author’s arguments.
(b) This statement would likely strengthen the author’s arguments.
(c) This statement can be inferred from the author’s arguments.
(d) This statement is an assumption made by the author.
127. Which of the following can be attributed to be the author’s opinion?
(a) The substance of the Lokayukta body is that its orders are binding.
(b) Governments that are left-aligned function better in states such as Kerala.
(c) The Lokpal and Lokayuktas are unnecessary and unconstitutional bodies.
(d) Illegal appointment of relatives of a minister as office holders is unlawful.
128. If the information given in the passage is true, which among the following
must be true as well?
(a) The Kerala government would banish the proposed amendment if its ministers are
reinstated.
(b) The proposed amendment is not likely to pass the muster in a court of law.
(c) The CPM’s stance on Lokayuktas today is in contrast to what it was in the past.
(d) An ordinance is most likely a type of law making process which cannot be taken
down.
129. Which of the following roles does the CPM party statement of 2011 play in the
passage?
(a) The statement weakens the author’s arguments.
(b) The statement strengthens the opposition’s arguments.
(c) The statement strengthens the author’s position.
(d) The statement strengthens the CPM party’s argument.
130. Which of the following sentences express an opinion?
(a) Governor Arif Mohammad Khan publicly spoke about Bindu’s writing to him in
connection with the
reappointment of a vice-chancellor.
(b) KT Jaleel was forced to resign from his post last year.
(c) CPM’s defence on bringing in the ordinance appears unpersuasive.
(d) Lokayukta is a quasi-judicial institution.
Passage(Q.131-Q.135): Japan's Cabinet approved a record 5.4 trillion-yen ($47
billion) defense budget for fiscal
2022 on Friday that includes funding for research and development of a new fighter
jet and other “game#changing” weapons as Japan bolsters its defence capabilities in
response to China’s growing military might and
its tensions with Taiwan.

. Page 34 of 36
The 1.1% budget increase for the year beginning in April is the 10th consecutive
defense spending increase and
is in line with Japan’s pledge to the United States to strengthen its own defense
capabilities to tackle increasingly
challenging security issues in the region. The budget, which still needs to be
approved by parliament, includes a
record 291 billion yen ($2.55 billion) for defense research and development, up 38%
from the current year.
Of that, 100 billion yen ($870 million) is for development of the F-X fighter jet
to replace Japan’s aging fleet of
F-2 aircraft around 2035. It would be Japan's first domestically developed fighter
jet in 40 years.
Japan and Britain recently announced joint development of a future demonstration
fighter jet engine and agreed
to explore further combat air technologies and subsystems. The project includes
Mitsubishi and IHI in Japan and
Rolls-Royce and BAE Systems in the U.K.
As China’s military build-up extends to cyberspace and outer space, Japan's Defense
Ministry is also pushing
for research into artificial intelligence-operated autonomous vehicles for aerial
and undersea use, supersonic
flight, and other “game-changing” technologies.
131. Based on the passage, select the statement that can most plausibly be inferred
from the author's reasoning.
(a) Japan's cabinet approved a defense budget with a 1.1% increase amid Taiwan
concerns.
(b) Japan is bolstering its defense capabilities in response to China's growing
military might.
(c) There is a deep environment of concern in Japan's cabinet.
(d) Japan military budget for the fiscal year is an unprecedented amount.
132. Which of the following is in is in line with the passage?
(a) Japan is planning to fight with Taiwan in future.
(b) Japan is planning to establish its good relationship with China
(c) Japan is planning to become the strongest defense in the world.
(d) Japan is planning to develop its defense mechanism.
133. In the following question given below, a statement is given followed by two
assumptions numbered I and II. An
assumption is something supposed or taken for granted. You have to consider the
statement and the following
assumptions and decide which of the assumption(s) is/are implicit in the statement.
STATEMENT: China holds the biggest defense military budget in the world. Japan has
increased its military
budget.
ASSUMPTION(S):
I. Japan is trying to compete with China.
II. Japan fears China.
(a) Only assumption I is correct (b) Only assumption II is correct
(c) Both I and II are correct (d) Neither I nor II are correct
134. Which of the following is most likely to weaken the author's argument?
(a) Britain and Japan have been working together to create innovative military
technology.
(b) Japan is strengthening its defense capabilities to tackle increasing security
issues.
(c) Japan bolsters its defence capabilities in response to the US’s request to aid
it in military combat against its
enemies.
(d) AI operated autonomous vehicles will be introduced soon with the help of
increased budget.
135. Which of the following questions is most likely to find an answer in the
passage?
(a) Are China and Taiwan going to combat Japan?
(b) Why is Japan increasing its military budget?
(c) Have Japan and Britain become close friends?
(d) None of the above, not wholly kind, about his affairs.’ What is the author
implying through the lines?

I. People associate negative connotations with those who disclose information about
themselves.
II. People with a candid disposition tend to be perceived favourably by others.
III. People perceive negatively those who keep to themselves.
IV. A person with a shy disposition never gets a favourable opinion.
(a) Only II (b) I and IV (c) Both II and III (d) Either II or III
30. But he could see no signs of life anywhere.
Which of the following literary device has been used in the italicised sentence
above?
(a) Metonymy (b) Satirical (c) Hyperbole (d) Juxtaposition
Directions (Q.66 – Q.105): Read the comprehensions carefully and answer the
questions based on it.
Passage(Q.66-Q.71): We all have the legal right of peaceful enjoyment. Enjoyment
here means to be able to act
as one pleases, but obviously, without unreasonably interfering with another
person’s enjoyment. This tort of
nuisance arises where there is an unreasonable or undue interference in one’s
peaceful enjoyment of something
by another.
The undue interference is thus one which is unjustifiable. From the above, it is
sufficiently clear that malice is
not an important element in nuisance. That is because, once again, the foundation
of the Tort of Nuisance is the
interference with the enjoyment of another’s property and not the interference per
se.
Public Nuisance occurs when a person does an act which causes annoyance to the
public in general. In cases of
public nuisance, the persons affected have a common claim against the defendant.
Despite that, a claimant may sue the culprit individually. That particular claimant
will have to prove these things:
1. That he has suffered an injury which is greater than that suffered by the rest
of the public.
2. Such an injury must be direct and not merely consequential.
3. The injury must be substantial, not fleeting.
Private nuisance is when the act causing discomfort or inconvenience affects a
specific people.
This will depend on a) the sensitivity of the plaintiff, and b) the locality in
which the nuisance has occurred. An
act does not become actionable solely due to the higher sensitivity of the
plaintiff towards a normal act.
The defence of statutory authority applies to acts which are incidental to those
done in pursuance of a law.
However, this defence will be vitiated if the act was done with negligence, and a
claim of nuisance can be
maintained.
(Extracted with requisite revisions and edits from ‘Understanding The Tort of
Nuisance’ by Aparna Shukla at
https://lawctopus.com/clatalogue/understanding-the-tort-of-nuisance/ )
66. Uma has rented out her house to Mahesh so that he can live there. Mahesh
without the consent of Uma started
construction in her house. Uma was aggrieved by the construction undertaken by
Mahesh and asked him to stop
the construction. Uma said that Mahesh is interrupting her enjoyment of the house.
Mahesh did not stop the
construction. Uma filed a complaint against Mahesh for causing nuisance. Decide
whether Uma would succeed:
(a) Yes, as her enjoyment is interrupted.
(b) Yes, as Mahesh was causing undue interference.
(c) No, as Mahesh had no intent to cause interference.
(d) No, as Uma had no legal right of peaceful enjoyment.
67. Mehul has a pet dog. Rahul who is his neighbor does not like dogs and was
always irritated by Mehul’s dog.
Mehul’s dog used to bark a lot at night. Rahul was unable to sleep due to the
barking. He filed a case against
Rahul that he is causing nuisance. Decide whether Mehul would succeed:
(a) No, as Rahul always hated Mehul’s dog and Mehul has no ill-intent.
(b) No, as the dog did not enter on Rahul’s property to cause his disturbance.
(c) Yes, as the barking of dog caused Rahul undue interference.
(d) No, as Mehul did not cause any interference on his own.
68. In furtherance of the above stated passage, decide which statement is true:
(a) Nuisance can never be caused by statutory authority does its statutory duties.
(b) Nuisance cannot be caused against one’s own property.
(c) In public nuisance compensation cannot be granted to an individual.
(d) Interference without intention would not amount to nuisance.

. Page 18 of 36
69. The government department of electricity is vested with the duty to manufacture
electricity. The electricity is
produced by constructing a dam. Due to the construction, cement mixed in the air
and landed on the agricultural
land and house of the people residing nearby. They all filed a suit to claim
compensation stating that
compensation from electricity department. Decide whether people would succeed:
(a) No, as every person apply separately for nuisance caused to them.
(b) Yes, as it is a case of public nuisance.
(c) No, as no injury was caused to people.
(d) No, as the electricity department is performing its statutory duty.
70. In the last question, if the dam has been constructed by the electricity
department but suffers from normal wear
and tear. The department fails to do the maintenance of dam. One fine day, due to a
crack in the dam, the water
is flooded in Ramesh’s agricultural land and destroys his crops. Decide whether
Ramesh would succeed for
nuisance:
(a) No, as the department was performing its statutory duty.
(b) Yes, as the department acted negligently.
(c) Yes, as the department will not be treated differently just because it belongs
to government.
(d) No, as the damage was caused due to the normal process of wear and tear and
department did nothing wrong.
71. Manish and Mona are neighbors. Manish had heart issues and could not deal with
noises even a little. One day
Mona was watching news on television and Manish could not bear the volume of
television. Manish was so
much troubled by it that he had to be hospitalized. Manish filed a suit against
Mona for nuisance. Decide whether
Manish would succeed:
(a) No, as Mona was doing a normal act and did not cause any undue interference.
(b) Yes, as due to Mona’s act Manish was hospitalized.
(c) Yes, as Mona caused undue interference.
(d) Yes, as Mona should have been extra cautious considering the special condition
of Manish.
Passage(Q.72-Q.77): The Delhi High Court has clarified that the phrase "first
learns" used in the Limitation Act
1963 vide Articles 68 and 91(a) in its Schedule means "actual knowledge" of
misappropriation and not merely
"speculative knowledge."
Limitation under Article 68 pertains to specific movable property lost, or acquired
by theft, or dishonest
misappropriation or conversion. The period of limitation is 3 years, time from
which begins to run when the
person having the right to the possession of the property first learns in whose
possession it is.
Similarly, Article 91(a) pertains to wrongfully taking or detaining any specific
movable property lost, or acquired
by theft, or dishonest misappropriation, or conversion. The period of limitation is
three years, time for which
begins to run when the person having the right to the possession of the property
first learns in whose possession
it is.
In this backdrop, Justice Prathiba M. Singh held:
"the language `first learns' clearly means actual knowledge and not speculative
knowledge. The knowledge
cannot be by means of inference but ought to be clear knowledge. Such knowledge
should include the factum as
to the exact location of the movable property, as to who is in possession of the
same or the proceeds thereof."
On the aspect of Burden of Proof of proving actual knowledge, the Court citing
Standard Chartered quoted as
follows:
"Obviously where a person has a right to sue within three years from the date of
his coming to know of a certain
fact, it is for him to prove that he had the knowledge of the said fact on a
particular date, for the said fact would
be within his peculiar knowledge."
(Extracted with requisite revisions and edits from ‘Limitation Under Art. 68 &
91(a) Runs From Date Of Actual
Knowledge' Of Misappropriation: Delhi High Court’ by PrateekChakraverty at
https://www.livelaw.in/news#updates/delhi-high-court-limitation-act-article-68-91-
date-of-knowledge-of-misappropriation-190982)

. Page 19 of 36
72. Monesh owned a diamond necklace. He pledged the necklace to Jayant and
therefore, the necklace was in
custody of Jayant. Rakesh stole the necklace but Jayant was not aware about the
theft initially and found about
it later on. However, Monesh came to know about the theft on the day of the
offence. Monesh filed a case against
Rakesh for stealing his necklace. Decide till when can Monesh file the case:
(a) Monesh cannot file a case and Jayant can file a case.
(b) Three years since Rakesh stole the necklace.
(c) Three years since Monesh comes to know about the theft.
(d) Three years since Jayant comes to know about the theft.
73. In the last question, if Jayant plans to file complaint for the theft against
Rakesh. Then when can Jayant file the
complaint:
(a) After 3 years when Jayant got to know about the theft.
(b) Within 3 years from the date when Jayant got to know that property has been
stolen.
(c) Within 3 years from the date when Jayant got to know that stolen property is in
Rakesh’s possession.
(d) Within 3 years from when date when Jayant has reasonable doubt about property
being stolen.
74. In accordance with the above stated passage, decide which statement is true:
(a) The time limit fixed by Art 68 is not compulsory.
(b) Court has the discretion to decide the period of limitation under Article
91(a).
(c) The knowledge must be about exact place and person in whose possession property
is to start period of
limitation.
(d) Speculation of misappropriation formed on reasonable ground is enough to start
period of limitation.
75. X is the agent of Y who is authorized with his land to keep it in safe custody.
X leased the land to Z which is
against the directions of Y. Y come to know about the misappropriation of land by
X. Decide when Y can file a
complaint under Article 91(a):
(a) Within 3 years of knowledge of misappropriation to Y.
(b) The case cannot be brought under Article 91(a).
(c) Within 3 years of commission of misappropriation.
(d) No case can be filed as X is the agent of Y.
76. Rahul misappropriated a cheque that he found on a road on 04/03/17. He gets the
cheque cashed on 06/03/17
from bank. The cheque belonged to Manu who came to know on 15/03/17 that the cheque
is in the possession
of Rahul who has misappropriated it. Manu wants to file a case against Rahul.
Decide when can Manu file a
complaint:
(a) On or Before 04/03/2020. (b) On or Before 06/03/2020.
(c) On 15/03/2020. (d) On or Before 15/03/2020.
77. In the last question, when Manu files the case on 15/03/2020. Rahul contented
that Manu came to know that the
cheque is in the possession of Rahul before 15/03/2017. Decide who will have to
prove that when was knowledge
of possession received:
(a) Manu. (b) Rahul. (c) Police. (d) Court.
Passage(Q.78-Q.83): The Delhi High Court recently observed that a wife making
serious allegations of criminal
conduct against her husband and his parents and not being able to establish the
same amounts to cruelty as a
ground for divorce under the Hindu Marriage Act, 1955 (Neelam v. Jai Singh).
The Bench held the same while dismissing the wife’s appeal against the judgment of
a family court which had
granted a divorce decree to the husband and dissolved the marriage. The Court held,
"The mere fact that she made serious allegations of criminal conduct against the
respondent and his parents –
which she could not establish before the Court, was sufficient to constitute acts
of cruelty against the respondent."

. Page 20 of 36
The husband as well as his parents was subsequently taken into custody. Both the
husband and his parents were
acquitted in August 2015 and an appeal against the acquittal was also dismissed in
January the next year.
After the family court granted the husband a divorce decree on the ground of
cruelty, the wife approached the
High Court in appeal to seek restitution of conjugal rights under Section 9 of the
HMA which was also denied
on the grounds of cruelty.
“How can the respondent be expected to allow the appellant into his life in these
circumstances? The faith and
trust – which is the foundation of a matrimonial bond stood completely demolished
by the aforesaid conduct of
the appellant. For a man to see his parents to be taken into custody and being
incarcerated even for a single day
would have caused immense and untold pain and agony to him...Did she not know that
their conviction would
have led to their being sentenced to imprisonment? Therefore, her conduct of not
opposing the bail application
is neither here, nor there."
The Court, however, issued notice to the husband on the aspect of grant of
permanent alimony of the wife and
maintenance of their minor child.
Source: https://www.barandbench.com/news/litigation/wife-making-unproven-
allegations-of-criminal-conduct#is-cruelty-to-husband-delhi-high-court
78. In a case of divorce, filed by the wife, Ahaana, the contentions placed against
her husband by her were that he
was a drunkard and a molester. However, she failed to substantiate the allegations
against him, thus resulting in
him being exonerated of the charges but being granted the divorce all the same.
Should the divorce stand in this
case?
(a) Yes, as she could not substantiate her allegations, making divorce valid.
(b) No, as the grounds for divorce were flimsy and bound to be dismissed.
(c) No, as the grounds for divorce are not in accordance with the HMA.
(d) Yes, as a divorce requires no grounds to be granted.
79. In the above case, had the wife failed to substantiate the allegations of
cruelty as against her husband, and had
the same led to a petition for divorce being filed by the husband, would the pleas
of the wife for restitution of
conjugal rights be allowed ?
(a) No, as she filed serious allegations of criminal conduct and failed to
substantiate the same.
(b) Yes, as no allegation of cruelty could be made.
(c) No, as the husband has been deeply affected by the same.
(d) Yes, as the wife should be allowed means of sustenance.
80. In another case of domestic troubles, a divorce was filed for by the husband,
claiming that his wife had been
overly abusive of his mother and this amounted to cruelty. After the divorce had
been given on the same grounds,
the wife had suddenly fallen on hard times due to no source of sustenance for her
own endeavours. Thus, she
sought to file for restitution of conjugal rights in the court. Is the court liable
to grant her restitution?
(a) No, as the divorce was granted on firm grounds.
(b) No, as she can be given maintenance for her affairs.
(c) No, as the ground of cruelty has been satisfied against her.
(d) No, but she is entitled to maintenance.
81. Shubhra had brought allegations of rape against her husband and had obtained
divorce from him via the same.
Thereafter, due to not being given any maintenance, it had become difficult for her
to sustain herself on scraps.
Thus she demanded the court that restitution of her conjugal rights be made
possible for her to obtain a decent
standard of living. What relief can the court grant her?
(a) None, as she was the one who obtained the divorce.
(b) None, as she had received the relief by way of divorce.
(c) Adequate maintenance may be granted to her.
(d) Restitution of her conjugal rights will not be granted to her.

. Page 21 of 36
82. In a case of theft being alleged by the wife as against her husband and his
family members, the court had found
them not guilty of the same and had also held that these were arm twisting tactics
that were being employed by
the wife to extort money from the husband. After the dismissal of the case, the
husband had denied to live with
his wife forever and had thrown her out of the house. The wife, to counteract the
same, approached the court for
restitution of her conjugal rights as being his lawfully wedded wife. Would the
court oblige?
(a) Yes, as the couple is still married.
(b) No, as the wife has clearly exceeded the bounds of her conjugal relations.
(c) No, as the wife has perverse intentions that are apparent.
(d) No, as the wife made allegations of serious criminal conduct and could not
substantiate the same.
83. In the peaceful colony of Nanda Vihar, Kamal and Rosa used to reside, who were
a childless married couple
frustrated with each other. During a family outing, while dressing up for the
occasion, Rosa could not find her
necklace, leading to her bursting out on Kamal under the apprehension of him
stealing it. As such, she made a
huge ruckus and the neighbours in the vicinity heard it all. When the necklace was
found in Rosa’s cupboard
later on, Kamal, infuriated at her, decided to leave, never to come back. In this
case, can Rosa plead for restitution
of conjugal rights?
(a) Yes, as they have not been separated here
(b) No, as the couple are clearly not amicable and should remain separated.
(c) Yes, as no suit has been filed yet on false allegations.
(d) No, as the husband deserves to be separate from his wife on account of being
accused of theft.
Passage(Q.84-Q.89): The Delhi HC has directed all Principal District and Sessions
judges in the State to issue
circulars mandating that all concerned District and Sessions Judges in the State
have to intimate the Judicial
Officer and no one else about their intention to take leave at least one day in
advance, and the Judicial Officer
has to update the same on the Delhi District Court website immediately.
The communication by the Registrar General of the High Court also stated that the
circular should mandate that
in case of any hardship or unforeseen exigency or when leave is applied at the last
moment, intimation should
be sent through electronic means to the judicial officer so that the same is
received at the office of Principal of
District and Sessions Judge before 10 am, enabling the updation of the same on the
website for intimation of all
concerned.
"I am further directed to request you to issue direction for maintaining a Register
in your respective offices
containing following fields:
a) Name of the Judicial Officer
b) Date(s) for which leave is sought
c) Date and Time of receiving the leave application in the office
d) Date and time of uploading the intimation on the website," the communication
said.
Source: https://www.barandbench.com/news/litigation/delhi-judges-should-mark-their-
leave-one-day-in#advance-delhi-high-court
84. Once the above mentioned guidelines were passed, they were posted all over the
court complex for the judges to
get familiar with the same. One judge, Bhola J. had left early that day and was
unaware of the changed stance of
affairs. He had taken a leave the next day and the same was not notified until 11
am on the day, leaving a number
of advocates and their clients furious for not being heard. Would Justice Bhola
face a criminal liability?
(a) No, as he was unaware of the changed position.
(b) Yes, as ignorance of law is no excuse.
(c) No, he will not be held liable.
(d) Yes, as anything that the court utters becomes law.

. Page 22 of 36
85. After passing the above mentioned directions, the SC took suo motu cognizance
of the same and passed an order
mandating the same to be done by the judicial officers of every court complex. In
the Tis Hazari Court, the
judicial officer had done the same, and Justice Basudev had failed to report to him
about his absence, being
unaware of the position of law. Would the judge be criminally liable in this case?
(a) Yes, as the judge did not know the position of law in the matter.
(b) No.
(c) No, as judges have bureaucratic immunity in such cases.
(d) Yes, as the judge has been held in neglect of the position of law.
86. Judge RP Bhadoria of the Delhi HC was a very punctual and stringent judge known
for his stoic mannerisms
and expeditious hearings. After having accrued a decent number of vacation days, he
had finally decided to go
on a vacation to Maldives with his family. As a result of it, he failed to inform
the Judicial officer of the same,
which resulted in about a week’s worth of hearings not being addressed. Would he be
liable for negligence in
this matter?
(a) No, as he only took up his accrued vacation days.
(b) No, as he is a HC judge.
(c) Yes, as all judges are bound by the directives mentioned above.
(d) No, as the directives are not meant for judges who have a good record.
87. In the above case, had Justice Bhadoria been a District court judge, and had he
sent in a message for his vacancy
one day prior to the judicial officer, only to have him take notice of the same
post 11 am the next day, would he
be held liable for non-compliance of the directive?
(a) Yes, as he did not report his absence in time.
(b) No, as he had sent in the intimation timely.
(c) Yes, as the judicial officer would be liable here for not taking note of the
same.
(d) No, as the present matter does not clearly show either as being negligent.
88. In the middle of a crucial hearing, the Judge had to leave owing to a family
emergency, and had adjourned the
further matters. He had no time in the next two days due to his wife having been
admitted to a hospital due to
cardiac arrest. This led to his absence of two days from the court without any time
for him to notify the officer
of the same. Would he be held liable for this absence?
(a) No, as the matter was urgent and crucial.
(b) Yes, as the Judge should have left a message making his absence known.
(c) No, as the Judge was preoccupied completely and had no chance of reporting the
same.
(d) Yes, as his absence would have caused disruption, wasting court’s time..
89. Judge Sharma had to take a leave of absence on one particular day, and after
informing the Asst. Judicial officer
of the same, he had left. The assistant officer failed to inform the Judicial
officer of the same, which led to a
plethora of complaints being tendered to the officer for not reporting the same on
time. Who shall be held liable
in this case?
(a) The assistant shall be held liable due to his negligence.
(b) The Judicial officer shall be held liable for not reporting the same.
(c) The Judge shall be held liable for not duly reporting his absence.
(d) No one shall be held liable in this case.

. Page 23 of 36
Passage(Q.90-Q.95): There is a need to improve legal literacy among general public
including educated people,
Supreme Court judge Justice AS Oka has stated, stressing that courts cannot go by
public sentiments when
delivering judgments and have to be strictly in accordance with established laws.
General public has to be made aware about the processes involved in law, Justice
Oka emphasized.
In this regard, he also pointed out how judges are criticized on public platforms
and social media. “We find that
after court decisions, there are remarks online which can't be read in court. You
suddenly realize it's your order
being talked about on social media," the judge quipped.
The judge said that lack of legal literacy has especially meant that educated
people do not understand the reasons
for pendency of cases, the difference between bail and acquittal, and that
sentencing in criminal courts cannot
be bound by sentiments of public.
He, therefore, urged the Bar Associations to incorporate the need of improving
legal literacy among the general
public in their code of ethics. The judge also said that his personal view is that
live-streaming of court
proceedings should be allowed but in a limited manner.
Source: https://www.barandbench.com/news/courts-cannot-give-judgments-based-public-
sentiments-supreme#court-judge-as-oka
90. After the above case, once the judgment was delivered, another case about
inter-community riots had come up
before the court. In order to deliver a judgment that sates both the communities,
the judge delivered a pacifist
natured judgment in accordance with laws. This was heavily criticized as the media
said that the judge was
influenced by the socio-legal environment of the country. Has the judge erred in
rendering the judgment?
(a) No, as the judge was right in not infuriating the riots.
(b) Yes, as the judge was influenced by the situation.
(c) Yes, as the judge was influenced by his overwhelming emotions.
(d) No, as the decision was in accordance with laws.
91. In the above case, the judge had been informed that there were radicals outside
the courthouse ready to pelt him
with stones. After which, he had delivered the same judgment, which resulted in the
radicals vacating the
courthouse premises. This was interpreted as an act of emotions overwhelming the
judge, leading to him being
criticized for the same. Is this a case of apparent emotional bias?
(a) No, as the judge was not influenced by the radicals.
(b) Yes, as the judgment is a clear product of the judge being harassed.
(c) Yes, as the judge bent in a manner that would placate the radicals.
(d) No, as the judgment was in accordance with laws.
92. The High Court of Madhya Pradesh, had sentenced AZB, a murder accused, to
death, for having bombed the
Bombay Central Railway station, killing 109 people in the name of Jawanism, his
religion. Whilst a small sect
by universal standards, Jawanism had garnered a decent following in India with
nearly 200,000 people following
the same. His matter was now under appeal before the SC which had reduced the death
sentence to a life sentence
fearing civilian retribution from other jawanists, clearly in violation of laws. Is
the SC right in doing so?
(a) No, as the SC clearly diverted justice under influence.
(b) No, as the SC bowed down to an unruly evil in this case.
(c) No, as the court has not given the judgment in accordance with laws.
(d) No, as the SC has clearly undermined the HC’s decision illegally.

. Page 24 of 36
93. Sudha was a woman, who was raped and then murdered in a backwater village of
Jhabua, resulting in the case
being heard by the HC who had declared the same as being liable for death sentence
being given to the accused.
Her parents were being pressured into redacting the case from the HC or facing the
brunt of the goons in cahoots
with the accused. The death sentence was executed, and as a result of the same, her
parents were also murdered.
This was made to be a red-letter day, as the court was being criticized for not
considering the position of the
parents in this case. Has the court erred in delivering its judgment?
(a) No, as the judgment was strictly in accordance with the laws.
(b) Yes, as the major driving cause in the case should have been the further lives.
(c) Yes, the court should have firstly considered the position of the parents.
(d) No, as the court did not deviate from delivering the judgment.
94. SujivWalia was a business tycoon in India, employing 10000 people within India,
leading the country in
employing the people. He was convicted recently in a banking scam, held liable for
defrauding a consortium of
banks out of 700 crore rupees in the last quarter. After being convicted, his
company ran into insolvency and
about 8500 people were rendered unemployed, leading to a massive offload resulting
in an economic turmoil.
The SC was appealed to in order to overturn its judgment in convicting him. Should
the Court do so?
(a) No, as court cannot deliver judgments on public sentiments.
(b) No, as the court is not an arbitral tribunal.
(c) Yes, as the court should consider the situation of turmoil in the country.
(d) Yes, as the court is supposed to take steps to further peace and tranquility.
95. Judge Victor, a SC judge and Joris Bohnson, the English minister of foreign
affairs, were childhood friends as
they had gone to school together in Sweden. In an unfortunate turn of events, Joris
was implicated in a matter of
defrauding the Indian government in a bilateral treaty, thereby making him an
accused answerable to the SC of
India. Hearing of this, Victor was infuriated and wanted his friend to answer for
his crimes. He interferedt in on
the bench, and held him guilty of the same. Is this action of Justice Victor
correct?
(a) Yes, as he ensured that justice was swiftly done.
(b) No, as he has violated the laws of justice.
(c) No, as he has not given the judgment in accordance with the laws.
(d) Yes, as Joris deserved to be brought to justice.
Passage(Q.96-Q.101): The Supreme Court has recently observed that a Family Court
cannot deal with a criminal
complaint about offences under the IPC. As per the Family Courts Act, a family
court can exercise the
jurisdictions exercisable by any district court of any subordinate civil court
under any law for the following#1. A proceeding between the parties to a marriage
for declaring the marriage to be null and void or, annulling
the marriage or restitution of conjugal rights or judicial separation or divorce or
dissolution of marriage.
2. A proceeding for a declaration as to the validity of a marriage or as to the
matrimonial status of any person.
3. A proceeding between the parties to a marriage with respect to the property to
the parties or of either of them.
4. A proceeding for an order or injunction in circumstances arising out of a
marital relationship.
5. A proceeding for a declaration as to the legitimacy of any person.
6. A proceeding for maintenance.
7. A suit or proceeding in relation to the guardianship of the person or the
custody of, or access to, any minor.
It can also exercise the jurisdiction exercisable by a Magistrate of the first
class under Chapter IX (relating to an
order for maintenance of wife, children and parents) of the CrPC.
https://www.livelaw.in/news-updates/supreme-court-family-court-cannot-try-criminal-
cases-ipc-wrong-order#179433

. Page 25 of 36
96. After the divorce of a couple, Shardul, their child, was embroiled in a dispute
for custody of his person. The
mother made certain allegations regarding his father’s abhorrent behaviour and
excessive drinking. The father
made allegations about his mother’s philandering and foul-mouthedness. Is this
dispute triable by the Family
courts?
(a) The dispute is triable by the Family Courts.
(b) The dispute is triable by Criminal courts.
(c) The dispute is triable by both courts.
(d) The dispute is not triable by any of these courts.
97. KKD and BBC are to brothers involved in a property dispute wherein they were
equal shareholders of the land.
However, KKD wanted to sell the same because during a burglary in their home, one
of the robbers had killed
the other, marking the property as former scene of crime. BBC however, is not of
the opinion for selling it and
wants to retain it. Can the family courts entertain the same?
(a) No, as the property in the question is a scene of crime.
(b) No, as there is no consensus between the principal owners of the property.
(c) Yes, as it falls within the jurisdiction of the family courts.
(d) No, this does not fall within the jurisdiction of family courts..
98. Anjali had assumed her husband dead until recently, when in a prisoner
exchange, the warring state had released
the list of POWs. She had been waiting for him to return for 8 years, after which
she had him declared legally
dead and remarried. The husband filed a case against Anjali on the grounds that her
second marriage is
illegitimate. Can the family courts rule on this subject matter?
(a) Yes, as this is a suit related to validity of a marriage.
(b) No, as this is a suit concerning warring nations and should be tried by army
courts.
(c) Yes as this is a suit for restitution for conjugal rights.
(d) Yes, as this suit is related to declaring legitimacy of a person.
99. In the above case, had the husband not been a POW but a criminal in the warring
state absconding from the
police there, and had come back after being missing for 8 years. Would the matter
then be considerable by the
family courts?
(a) Yes, as the matter is of validity of marriage.
(b) No, as he has not been presumed dead.
(c) No, as being a criminal, he is firstly to be tried by a criminal court.
(d) Yes, as the matter remains to be of restitution of conjugal rights.
100. Surjana left her husband, Jagga, for abusing her physically for years and
years on end. Now the matter has come
up in the family courts of Delhi for the same. Can the Family Courts entertain the
matter?
(a) No, as the matter is primarily criminal in nature.
(b) Yes, as the matter is primarily of family courts.
(c) Yes, as the matter has already been initiated in a family court.
(d) None of the above.
101. In the above case, had the wife actually filed for divorce, and had filed for
maintenance post-divorce, which
courts would have the requisite jurisdiction to entertain the same?
(a) The family court would have the requisite jurisdiction.
(b) The trial court would have the requisite jurisdiction.
(c) Any court of first instance can entertain a matter of domestic abuse.
(d) For the matter of divorce and maintenance, family court will have jurisdiction,
but for cruelty only a criminal
court will have jurisdiction.

. Page 26 of 36
Passage(Q.102-Q.105): The Madras HC urged the TN Bar Council to exercise its
suomotu powers against
lawyers found acting to demean or prejudice the legal fraternity even if no formal
complaint is lodged.
"The Tamil Nadu Bar Council in consultation with the Bar Council of India, in the
larger interest of the legal
fraternity, shall look into the issue of evolving a mechanism for initiation of
suomotu proceedings against such
of those members, who indulge in activities, which are prejudicial and demeaning
the interest of the legal
fraternity as a whole", stated the order.
This led the Court to express concern that the Bar Council seldom uses its suomotu
powers to act against lawyers
for unprofessional conduct.
"Any unprofessional conduct of a member of the legal profession, coming to the
knowledge of the Bar Council
through the visual media for which no complaint emanates from any quarter, can the
Bar Council allow that
instance to go unnoticed for the mere reason that the Advocates Act does not
envisage suomotu action. It
observed that instances of advocates going beyond their brief are on the increase.
"Though this Court cannot give any positive direction to the Bar Council to do one
thing or the other, as it is not
vested with jurisdiction ... it is high time the Bar Council enforces the power and
authority to initiate action
suomotu on the incidents, which comes to its knowledge through the digital/print
media, for which there is no
complaint given by any individual," the order said.
"Advocates should not take law into their own hands on the premise that they are
the custodian of the law; on
the contrary, the advocates being the custodian of law, are bound to act within the
legal framework, even if there
is a violation of law and establish the rule of law through the well-defined
mechanism."
102. Ramesh, an advocate in Delhi, during the period of lockdown, went outside his
house to purchase medicine.
Police officials on duty stopped Ramesh while he was proceeding in his car. It was
found out that Ramesh was
not in possession of a valid pass for going out during the lockdown period. Ramesh
quarrelled with the police
officials on duty. He, while claiming that he is an advocate, castigated the police
officials on duty and threatened
them that they would be stripped off of their uniforms if they tried to intervene
and cause hindrance to the
movement of the petitioners.
(a) The High Court of Delhi cannot issue directions to the Bar Council of Delhi to
act in a certain way.
(b) The Bar Council of Delhi must not initiate suo-motu proceeding against Ramesh
because Ramesh is a part
of the legal fraternity and does not envisage suo-motu actions
(c) The High Court of Delhi vests with the jurisdiction of issuing directions to
the Bar Council of Delhi to act
in a certain way.
(d) The bar council has to initiate an action if the high court orders for same.
103. The quarrel hereinabove was recorded by some passer-bys and was uploaded on
Twitter tagging the Police Dept.
The Tweet was quoted as saying, "Delhi Police hampering the movement of a free
individual, disgraceful
conduct by law maintain authorities". Delhi police now wants to initiate an action
against the people tweeting,
advise them so.
(a) Delhi Police can initiate action for Defamation against Ramesh and the
tweeters.
(b) Delhi Police can initiate action for Defamation against the tweeters only.
(c) Delhi Police can urge the Bar Council to take Suomoto cognisance of the matter
herein as it happened on
social media, and no complaint arose from it.
(d) None of the above.

. Page 27 of 36
104. During a hearing in the Delhi HC, while the matter was being heard, one of the
counsels for the respondents
started heatedly arguing, often bordering on slander, as against the petitioners.
This led to the counsel for
respondent ultimately implying that the petitioners are two-bit thugs who are just
here to earn unlawfully from
damages. This was noted by the judge during the hearings, and the counsel was
reprimanded for the same by the
judge in a passing manner. However, the petitioners were highly offended by this
and implored the judge to
direct the Bar Council to take action against them separately for this slur. Would
they be right in doing so?
(a) Yes, as the counsel herein used a highly derogatory racial slur that insinuated
that the petitioners were of no
character.
(b) Yes, as the comments made herein were so inherently derogatory as to render the
petitioners slandered and
the respondents liable to be punished by the Council.
(c) No, as the courts do not hold the jurisdiction for the same..
(d) No, as the suomotu power of the Bar Council is not an appellate mechanism.
105. During a divorce case the counsels from both the sides start arguing fiercely,
each trying to get their clients the
majority share in their joint assets. This leads to several bouts of heated
exchanges wherein the counsels heavily
bantered, often insinuating depravity and hurled heavily veiled insults at each
other. During one of these
exchanges, one of the counsels casually mentioned that 'women cheat and are
casanovas', and the judge, a female
of high renown, was offended at this and held him in contempt of the Court,
pleading the bar to take strict action
against lawyers who casually demean the opposite gender in a slanderous manner. Is
this action of the Court
liable to be struck down?
(a) No, as the counsel is displaying unprofessional conduct. The Court has rightly
pleaded the Council to take
cognisance of it.
(b) No, as the judge herein has rightly exercised her judicial expertise and
discretion while pleading the Council
to take action.
(c) Yes, as the Courts do not have the power to implore the Council to take action
where the cause of action was
very casual such as this.
(d) None of the above.

. Page 28 of 36
SECTION - D: LOGICAL REASONING
Passage(Q.106-Q.110): Opposing the PILs related to “marital rape”, the Centre has
told the Delhi high court
that, among other things, misuse of Section 498 of the IPC related to dowry
harassment and lack of a mechanism
to verify when consent stands withdrawn by the wife are concerns because of which
India should move
cautiously and not follow other countries blindly on the issue. This has been
conveyed to the court in a written
submission.
“Various other countries, mostly Western, have criminalised marital rape but it
does not necessarily mean India
should also follow them blindly. This country has its own unique problems due to
various factors like literacy,
lack of financial empowerment of a majority of females, mindset of society, vast
diversity, poverty, etc and these
should be considered carefully before criminalising marital rape,” the government
submitted earlier this month.
It has pointed out that marital rape is not defined in any statute or law. While
rape is defined under Section 375
of IPC, “defining marital rape would call for a broad-based consensus in society.
As to what constitutes marital
rape and what would not constitute marital rape needs to be defined precisely
before a view on its criminalisation
is taken,” it said.
Solicitor General Tushar Mehta had told the HC that the Centre was considering a
“constructive approach” to
the issue and has sought suggestions from several stakeholders and authorities on
comprehensive amendments
to the criminal law. He had said criminalisation of marital rape involves “family
issues” as well as the dignity of
a woman and cannot be looked at from a “microscopic angle” and sought some more
time. But the bench made
it clear it would continue hearing other stakeholders till the Centre opens its
arguments.
106. ‘Various other countries, mostly Western, have criminalised marital rape but
it does not necessarily mean India
should also follow them blindly.’ Choose the most logical conclusion from the given
statement.
(a) India looks to Western countries for setting legal precedents.
(b) Western countries have criminalised rape after following some other countries.
(c) India always looks at Western countries as precedent.
(d) Indian law is highly influenced by the West.
107. Which of the following, if true, would most likely weaken the author's
statement that Centre has sought
suggestions from several stakeholders and authorities on comprehensive amendments
to the criminal law.
(a) The Centre always does whatever it wants, but this time it is trying to make
collective decisions.
(b) The Centre is trying to influence the decision of several authorities against
the court.
(c) The Centre has sought the suggestions of stakeholders and authorities that toe
the Centre’s line.
(d) The Centre wants to involve various stakeholders and authorities before
committing to amending the criminal
law to avoid far reaching repercussions.
108. Which of the following reflects the central theme of the passage?
(a) Court's ambition to make martial rape a criminal offence is an inspiration from
the West.
(b) The central is not against criminalising marital rapes, but does not want to
shoulder the responsibility.
(c) The centre wants the court to move cautiously and consider various factors
before criminalising marital rape
as India is different from the western countries.
(d) The Court’s decision of criminalising marital rape will affect a lot many
marriages adversely.
109. Consider the following statement from the passage and answer accordingly.
Assertion (A): Solicitor General Tushar Mehta had told the HC that the Centre was
considering a “constructive
approach” to the issue and has not sought suggestions from several stakeholders and
authorities on
comprehensive amendments to the criminal law.
Reason (R): He had said criminalisation of marital rape involves “family issues” as
well as the dignity of a
woman and cannot be looked at from a “microscopic angle” and sought some more time.
OPTIONS:
(a) A is true and R is false.
(b) A is false and R is true.
(c) Both A and R is true and R is the correct explanation of A.
(d) Both A and R is true and R is not the correct explanation of A.

. Page 29 of 36
110. Which of the following, if true, most strongly supports the argument?
(a) Marital rape law must be considered after much deliberation and broad-based
consensus.
(b) Martial rape law should not exist in India looking at the diversity in culture.
(c) Marital rape is a serious offence and should have a well-defined law against
the criminal offence.
(d) The court is the keeper of law, and it will base its decision using utmost
discretion.
Passage(Q.111-Q.115): It is slow work — subverting independence. That the work is
in progress was indicated
by the remarks of the Union law minister, Kiren Rijiju, about ‘coordination’ of
constitutional bodies with the
executive. As the law minister, he cannot have any doubt regarding the importance
of independence of
constitutional bodies such as the Election Commission. The whole point of free and
fair elections is defeated —
and democracy made into a travesty — if the EC, for example, acts in coordination
with the law ministry headed
by an elected politician. But it is precisely this that formed the context of Mr
Rijiju’s remarks; he praised the EC
for its cooperation after a meeting with the election commissioners that he had
called. The impropriety of the
proceeding had been criticized by members of the Opposition and the former chief
election commissioner, S.Y.
Quraishi. The EC may call officials for administrative tasks, but its commissioners
cannot be summoned by the
executive. Moreover, the ‘coordination’ was in the matter of a controversial
change. The election laws
(amendment) bill, 2021 links Aadhaar with voter rolls which, according to the
Opposition, is likely to
disenfranchise a number of voters apart from breaching privacy.
So the context of the Union law minister’s paean to coordination defines what he,
or his government, means by
the term. He has even spoken to the judiciary about the importance of coordination;
should justice ‘coordinate’
with the executive then? The independence of officials of autonomous institutions
was being undermined for
quite some time, whether through the delay in appointing the necessary number of
vigilance commissioners for
the Central Vigilance Commission or through amending the law so that information
commissioners in the Central
Information Commission became dependent on the government for their tenure and
salaries. Such moves add
more weight to the criticism from the former Supreme Court judge, Madan B. Lokur,
Mr Quraishi and others
that followed Mr Rijiju’s praise of coordination. They pointed out the lack of
checks against ‘executive
overreach’, focusing on the fact that, unlike the CEC, the other two election
commissioners do not have
constitutional protection from removal and therefore feel insecure. The process of
their selection and
appointment is faulty, being dependent on the executive. But that is what this
government desires; executive
overreach was implicit in the law minister’s high-minded benevolence.
111. According to the author, what might be the intention of the law minister in
making such a remark?
(a) The law minister wants more interference by the executive in the EC.
(b) The law minister wants the EC to surrender itself to the hands of the
Executive.
(c) The law minister should not care about the opinions made by the author.
(d) The law minister wants specific boundaries to be drawn between the EC and the
executive.
112. Which of the following is strongly supported by the information given in the
passage?
(a) The criticisms made by personalities like Madan Lokur have no real value.
(b) An elected politician is not meant to be a person of honesty and integrity.
(c) Giving constitutional protection to an officer makes him/her more independent.
(d) Only very few elected politicians are there who really want to serve the
country.
113. Out of the following, which is the best representation of the author’s opinion
about the coordination that the law
minister has remarked?
(a) The coordination between the executive and judiciary would adversely affect
democracy in India.
(b)The coordination would make the democracy of India more vibrant and healthy.
(c)The coordination remarked by the law minister would not be good for India’s
democracy.
(d)The coordination between the bodies referred by the law minister would make
India an unruly state.

. Page 30 of 36
114. With which of the following would the author not agree?
(a) The government of the day desires the Election Commission to be independent.
(b) The post of an EC Commissioner is too powerful to be summoned by the executive.
(c) The coordination of the constitutional bodies is the underlying strength of
democracy.
(d) The Election Commission as the constitutional body must be given an independent
status.
115. What is the role played by the following statement in the passage? “Persons
appointed as the law minister are
expected to be well read about the constitutional bodies.”
(a) This statement is a counter premise.
(b) This statement is likely to be an assumption made by the author.
(c) This statement would weaken the arguments made by the author.
(d) This statement is the premise of the passage.
Passage(Q.116-Q.120): Russian Foreign Minister Sergey Lavrov’s comment, after
receiving a written response
from the US to Moscow’s security demands, that ‘Russia doesn’t want wars’ raises
hopes of a diplomatic solution
to the Ukraine crisis. Last week saw a flurry of diplomatic activities aimed at de-
escalating the situation. Besides
the U.S.’s written response to Russia, which could set the stage for further
diplomatic talks in the coming weeks,
French President Emmanuel Macron has held talks with his Russian counterpart
Vladimir Putin. Mr. Macron
and German Chancellor Olaf Scholz are also reportedly trying to revive the stalled
Minsk process that sought to
find a peaceful solution to Ukraine’s internal conflict between Kiev and the
Russia-backed separatists in the
eastern Donbas region. Mr. Putin has said the U.S. response does not address
Russia’s core security concerns.
Fears of a military conflict are still there. Russia, which has mobilised thousands
of troops on its border with
Ukraine, in Belarus in Ukraine’s north and in Transnistria (a breakaway region from
Moldova) in its south-west,
has a clear military advantage. But the West’s willingness to press ahead with
diplomatic options and Russia’s
reciprocity suggest that neither side is in a hurry for armed conflict.
There is a strong case for de-escalation. Mr. Putin has already achieved many
things, without a shot being fired.
He has got the Western leadership to talk to him over the contested issue of NATO’s
expansion, which Russia
has long been complaining about. By putting a gun to Ukraine’s head, he has
effectively drawn a red line in
Russia’s relations with the West. Also, as the U.S. and its allies are scrambling
for economic measures to punish
Russia “if it invades Ukraine”, the 2014 Russian annexation of Crimea and Moscow’s
continued support for
separatists inside Ukraine have practically become non-issues. Nobody is
threatening to punish Russia for not
returning Crimea to Ukraine. The status quo has been redrawn. Besides, the U.S. has
said it is open to discuss
some of Russia’s security concerns, including missile deployments in Eastern Europe
and military exercises in
sensitive regions. Russia should accept the U.S. proposals, de-escalate the crisis
around Ukraine and opt for more
dialogue on critical issues, including NATO’s eastward expansion. If it still goes
ahead with an attack, which
U.S. President Joe Biden said could happen, it could well be a mistake. Russia
might be in a position to overrun
Ukraine militarily, but what comes next in Europe’s largest country is as
unpredictable as it can get. If the post#9/11 military adventures of the U.S. are
any lesson, it is that great powers could clinch swift victories against
weaker countries but could fail miserably in sustaining those victories. Mr. Putin
should not walk into the same
mistake and thereby push Europe back into the darker days of the Cold War.
116. Which of the following best sums up the main idea of the passage?
(a) Russia should do more to reduce tensions along its border with Ukraine.
(b) Russia should be mindful of the West before attacking Ukraine.
(c) Russia would do well to de-escalate and solve its issues with the US
diplomatically.
(d) Ukraine should bandwagon with its neighbours to take on Russia.
117. The information given in the passage most strongly supports which of the
following inferences?
(a) The Minsk process will be completed in the foreseeable future.
(b) Avoiding a conflict between Ukraine and Russia is good news for India.
(c) Russia can learn from the mistakes committed by the US in some regard.
(d) The European front has a tendency to accommodate Russia in most matters.

. Page 31 of 36
118. What is the opinion of the author regarding the U.S. proposals to Russia?
(a) Russia is left with much to desire for; the proposals won’t be much.
(b) Acceptance of the U.S. proposals by Russia is advisable.
(c) There is nothing more that Russia could have asked for.
(d) Russia, being a powerful military, knows how to tackle such situations.
119. Out of the following, which statement would provide strength to the author’s
arguments?
(a) Russia has downsized its military personnel by 30% to control its economic
downfall.
(b) The U.S. dealt with the perpetrators of the 9/11 attacks with a heavy blow.
(c) Last week, a large team of diplomatic personnel were sent by Russia to Europe
to talk about Ukraine.
(d) Much can be attained with coercion and threat as has been in the case of Russia
and Ukraine.
120. With which of the following would the author of this passage disagree?
(a) Russia will go ahead and attack Ukraine with full force.
(b) Ukraine’s stature in Europe is among the least preferred nations.
(c) Russia is not comfortable with NATO’s eastward expansion.
(d) Both a and b are correct
Passage(Q.121-Q.125): Western Uttar Pradesh, which goes to the polls in the first
phase of the Assembly
elections in the state, has been plunged into a poll campaign marked by the most
undesirable forms of communal
and casteist rhetoric and mobilisation. This region of the state has much
significance in this round of elections
because of the churn that seems to have happened after the last state polls. The
Jats, who are the dominant
community in many constituencies, had supported the BJP in the last elections. But
the community has been
estranged from the party after the year-long farmers’ protest. The alliance between
the Samajwadi Party (SP)
and the Jat-dominated Rashtriya Lok Dal (RLD) has emerged as a strong challenger to
the BJP. The BJP had
won 52 out of the 71 seats in the region 5 years ago. The SP had won 16. If the Jat
votes shift to the SP-RLD
alliance, the BJP is bound to suffer losses.
That possibility has led the BJP to unleash a blatantly communal and casteist
campaign in the region. The Union
Home Minister has led the charge, and he has made some most unseemly and
inappropriate statements, unworthy
of his position. He made the astounding statement that the “ideology of both the
BJP and Jats is the same as both
have fought the Mughal invaders’’. Was the Home Minister of the country using
“Mughals” to mean today’s
Muslim citizens? He also made references to the communal riots in a UP district
that happened around a decade
ago in a way that seemed to hold out a threat that if the BJP was not voted into
power, there would be a repeat
of those riots. The Prime Minister used his virtual rally on Monday to remind the
people of the riots. The BJP
leaders are trying to woo the Jats back with promises, warnings and references to
the riots and the most ridiculous
claims.
The Home Minister, who has the responsibility to maintain peace and amity between
communities and to take
action against those who disrupt the peace, is himself trying to create dissensions
between sections of people.
They raise moral and political issues that make his continuance in the government
untenable. The Chief Minister
and several Union and state ministers have also made the most divisive statements
and tried to create hostilities
between communities. It is unfortunate that the Election Commission of India has
not found any of the statements
worthy of action. Appeals to religious and caste sentiments violate the model code
of conduct and should attract
firm action from the commission. Successive elections have seen a progressive
deterioration in the standards of
campaigning. Unfortunately, they have seen a deterioration in the functioning of
the Election Commission, too.

. Page 32 of 36
121. Which piece of information would be helpful to know while evaluating the
author’s argument(s) in the second
paragraph?
(a) Was the BJP in existence in the 16th and 17th centuries to fight the Mughals?
(b) Whether the Election Commission has the power to take action against misdeeds?
(c) Whether unleashing a blatantly communal and casteist campaign has led to
victory for the concerned party?
(d) Whether Jats will always remain on the side of BJP irrespective of the
differences.
122. The main idea of the passage is best represented by:
(a) The Election Commission’s inaction on the Home Minister’s untenable position is
unfortunate.
(b) The Home Minister would do well to take his unfortunate remark back if he wants
peace in the country.
(c) The politicians' way of campaigning in poll states - based on religion and
caste, needs to stop.
(d) Communal harmony is disrupted when the Election Commission does not take action
against the evil.
123. Which among the following courses of action would the author of this passage
logically agree with?
(a) The Election Commission disqualifies the BJP from conducting polls in the
upcoming elections.
(b) The populace of Uttar Pradesh does not elect the BJP as their elected
representatives in the polls.
(c) The BJP fulfils all the promises made by its representatives to the Jat
community before elections.
(d) None of the above is correct
124. Out of the marked statements, choose the statement that is the odd one out.
Use the concepts of Logical
Reasoning to answer this question.
(a) The Home Minister, who has the responsibility to maintain peace and amity
between communities and to
take action against those who disrupt the peace, is himself trying to create
dissensions between sections of
people.
(b) They raise moral and political issues that make his continuance in the
government untenable.
(c) The Chief Minister and several Union and state ministers have also made the
most divisive statements and
tried to create hostilities between communities. It is unfortunate that the
Election Commission of India has
not found any of the statements worthy of action.
(d) Successive elections have seen a progressive deterioration in the standards of
campaigning.
125. If the information given in the passage is true, it logically follows that
I. Deterioration in the functioning of the Election Commission spells doom for
India’s democracy.
II. Winning back the trust of the Jat Community would win the UP election for the
BJP.
III. The Jat Community was one community that took part in the year-long farmer’s
protest.
(a) Both I and III (b) Only III (c) Both II and III (d) All I, II and III
Passage(Q.126-Q.130): The CPM-led Kerala government’s decision to bring an
ordinance to limit the powers
of the anti-corruption watchdog is questionable. By turning the quasi-judicial
institution into a toothless advisory
body, whose orders will no longer be binding on the government and could be
appealed in a court of law, the
ordinance, which is awaiting the governor’s assent, will effectively neutralise the
Lokayukta Act, a model
legislation.
The Opposition has spoken out against the proposed amendment and alleged that it is
aimed at containing the
political fallout in the event of the Lokayukta passing adverse orders on
complaints against Chief Minister
Pinarayi Vijayan and Higher Education Minister R Bindu. Ahead of the assembly
elections in March last year,
the then higher education minister, KT Jaleel, was forced to resign following a
ruling by the Lokayukta: In a
case pertaining to an allegedly illegal appointment of a relative of the minister
in the state minorities corporation,
it had held that Jaleel’s conduct was unbecoming of a minister. Bindu has been in a
spot ever since Governor
Arif Mohammad Khan publicly spoke about her writing to him in connection with the
reappointment of a vice#chancellor. A complaint has been filed against Bindu before
the Lokayukta in the VC appointment case. The
government has refuted the Opposition’s charge that the ordinance has been drafted
to protect the ministers and

. Page 33 of 36
has claimed that the proposed changes are in line with two high court rulings. But
its defence appears
unpersuasive. Its attempts to whittle down the powers of the Lokayukta also fly
against the CPM’s own stated
position that a strong Lokpal at the Centre and empowered Lokayuktas in the states
are necessary to check public
corruption. In a 2011 statement, the party had said that the “battle against
corruption, in order to be effective
today, can be achieved only through a comprehensive reform of our political, legal,
administrative and judicial
systems and not through one-off or piece-meal measures. The establishment of an
effective Lokpal institution is
one such measure.” It also argued that Lokayuktas should be set up in the states
“on the lines of the Lokpal” with
“all state government employees, local bodies and the state corporations under
their purview”. CPM general
secretary Sitaram Yechury reiterated this position several times in Parliament.
The Kerala government needs to recall the Lokayukta ordinance. Failing to do so
could be viewed as a
compromise on the part of the Left towards strengthening institutional checks
against public corruption.
126. What role does the following piece of evidence play towards the arguments made
by the author in the passage?
“No provision of appeal in the Lokayukta law gives it unreasonable powers and
authority to make arbitrary
decisions.”
(a) This statement would likely weaken the author’s arguments.
(b) This statement would likely strengthen the author’s arguments.
(c) This statement can be inferred from the author’s arguments.
(d) This statement is an assumption made by the author.
127. Which of the following can be attributed to be the author’s opinion?
(a) The substance of the Lokayukta body is that its orders are binding.
(b) Governments that are left-aligned function better in states such as Kerala.
(c) The Lokpal and Lokayuktas are unnecessary and unconstitutional bodies.
(d) Illegal appointment of relatives of a minister as office holders is unlawful.
128. If the information given in the passage is true, which among the following
must be true as well?
(a) The Kerala government would banish the proposed amendment if its ministers are
reinstated.
(b) The proposed amendment is not likely to pass the muster in a court of law.
(c) The CPM’s stance on Lokayuktas today is in contrast to what it was in the past.
(d) An ordinance is most likely a type of law making process which cannot be taken
down.
129. Which of the following roles does the CPM party statement of 2011 play in the
passage?
(a) The statement weakens the author’s arguments.
(b) The statement strengthens the opposition’s arguments.
(c) The statement strengthens the author’s position.
(d) The statement strengthens the CPM party’s argument.
130. Which of the following sentences express an opinion?
(a) Governor Arif Mohammad Khan publicly spoke about Bindu’s writing to him in
connection with the
reappointment of a vice-chancellor.
(b) KT Jaleel was forced to resign from his post last year.
(c) CPM’s defence on bringing in the ordinance appears unpersuasive.
(d) Lokayukta is a quasi-judicial institution.
Passage(Q.131-Q.135): Japan's Cabinet approved a record 5.4 trillion-yen ($47
billion) defense budget for fiscal
2022 on Friday that includes funding for research and development of a new fighter
jet and other “game#changing” weapons as Japan bolsters its defence capabilities in
response to China’s growing military might and
its tensions with Taiwan.

. Page 34 of 36
The 1.1% budget increase for the year beginning in April is the 10th consecutive
defense spending increase and
is in line with Japan’s pledge to the United States to strengthen its own defense
capabilities to tackle increasingly
challenging security issues in the region. The budget, which still needs to be
approved by parliament, includes a
record 291 billion yen ($2.55 billion) for defense research and development, up 38%
from the current year.
Of that, 100 billion yen ($870 million) is for development of the F-X fighter jet
to replace Japan’s aging fleet of
F-2 aircraft around 2035. It would be Japan's first domestically developed fighter
jet in 40 years.
Japan and Britain recently announced joint development of a future demonstration
fighter jet engine and agreed
to explore further combat air technologies and subsystems. The project includes
Mitsubishi and IHI in Japan and
Rolls-Royce and BAE Systems in the U.K.
As China’s military build-up extends to cyberspace and outer space, Japan's Defense
Ministry is also pushing
for research into artificial intelligence-operated autonomous vehicles for aerial
and undersea use, supersonic
flight, and other “game-changing” technologies.
131. Based on the passage, select the statement that can most plausibly be inferred
from the author's reasoning.
(a) Japan's cabinet approved a defense budget with a 1.1% increase amid Taiwan
concerns.
(b) Japan is bolstering its defense capabilities in response to China's growing
military might.
(c) There is a deep environment of concern in Japan's cabinet.
(d) Japan military budget for the fiscal year is an unprecedented amount.
132. Which of the following is in is in line with the passage?
(a) Japan is planning to fight with Taiwan in future.
(b) Japan is planning to establish its good relationship with China
(c) Japan is planning to become the strongest defense in the world.
(d) Japan is planning to develop its defense mechanism.
133. In the following question given below, a statement is given followed by two
assumptions numbered I and II. An
assumption is something supposed or taken for granted. You have to consider the
statement and the following
assumptions and decide which of the assumption(s) is/are implicit in the statement.
STATEMENT: China holds the biggest defense military budget in the world. Japan has
increased its military
budget.
ASSUMPTION(S):
I. Japan is trying to compete with China.
II. Japan fears China.
(a) Only assumption I is correct (b) Only assumption II is correct
(c) Both I and II are correct (d) Neither I nor II are correct
134. Which of the following is most likely to weaken the author's argument?
(a) Britain and Japan have been working together to create innovative military
technology.
(b) Japan is strengthening its defense capabilities to tackle increasing security
issues.
(c) Japan bolsters its defence capabilities in response to the US’s request to aid
it in military combat against its
enemies.
(d) AI operated autonomous vehicles will be introduced soon with the help of
increased budget.
135. Which of the following questions is most likely to find an answer in the
passage?
(a) Are China and Taiwan going to combat Japan?
(b) Why is Japan increasing its military budget?
(c) Have Japan and Britain become close friends?
(d) None of the above

mock 33
Directions(Q.1-Q.30): Read the following passage carefully and answer the questions
that follow.
Passage(Q.1-Q.5): The efficacy of total lockdowns as a public health measure is yet
to be proven. But we seem
to have allowed our imagination and thinking to get trapped into that logic,
regardless of the social and economic
costs to society. Countries that imposed lockdowns, including China, have seen a
re-emergence of infections
while Sweden that did not impose a lockdown has close to zero deaths today.
However, it’s apparent that
localised restrictions on movements in high positivity areas for a limited period
make sense.
The impact of lockdowns has undoubtedly been painful at both the macro and micro-
levels. Amongst the worst
affected are children, ironically the least vulnerable.
Morbidity and mortality among children have been comparatively lower. As per a
recent UK study, deaths are
two per million and hospitalisation under severe conditions about 1 in 50,000.
Studies carried out in the US,
Ireland, Norway, Germany and other parts of the world have shown very low to
negligible transmission of
infection in, and due to, schools, particularly where the discipline of wearing
masks, physical distancing and
personal hygiene has been enforced even moderately. In fact, most countries have
persisted with in-person
learning. Only a handful have shut down schools. India is one of them.
While we have no information regarding the cohort of children who have been
hospitalised or have died due to
Covid over the past 18 months, as in the case of adults, children with _____ such
as diabetes or obesity are likely
to be more vulnerable. Such data, along with seropositivity studies, need to be
triangulated and analysed to enable
evidence-based policy formulation instead of panic or speculation guiding decision
making.
Available evidence seems to suggest that from a strictly epidemiological
standpoint, there is weak justification
for the stringent and prolonged lockdown of schools – particularly, primary
schools. We do hear of online classes.
But with less than a quarter of the country having internet access and the
lackadaisical manner of the
implementation of online learning by untrained teachers, the reach of such
instruction to even urban students has
been patchy. Students from rich families attending “good” schools may have
benefitted somewhat from online
education, but they are only a minuscule section of the learners.
The impact of this policy is generational and has undone a lot of the gains in
education. A large number of
children are now joining the workforce– they are missing out on the joys of
learning.
1. From the passage, it can be inferred that
(a) Affluent students only can attend competent schools.
(b) The children currently looking for employment may not be not equipped to enter
the workforce.
(c) Rather puerile students are able to attend online education.
(d) The children joining the workforce will have to do an on the job learning to
cover up for the inadequate
learning.
2. It is evident that from a pandemic stance, lengthy lockdown of schools
(a) remains absolutely unjustified.
(b) is strictly perilous for the students.
(c) is accounted for inadequately.
(d) explains the weak regime imposing diktat.

. Page 3 of 36
3. For the three major reasons mentioned in the passage, which one of the
followings would be not related to
‘negligible transmission of infection’?
(a) Discipline of wearing full-sleeves clothes.
(b) Maintaining physical distancing along with personal hygiene.
(c) Enforcing fixed number of students per class.
(d) Both A and C
4. Children with such as diabetes or obesity are likely to be more vulnerable. Fill
in the blank with the word from
the options that makes the sentence coherent.
(a) comorbidities (b) seropositivity (c) geniality (d) Machiavellian
5. The word ‘lackadaisical’ is the antonym of:
(a) uninterested (b) enervated (c) enthusiastic (d) offhand
Passage(Q.6-Q.10): A proposed change in what can be classified as "green energy" by
the European Union will
have disastrous consequences for global climate efforts, if its greed-driven
leadership is not stopped soon. On
New Year's Eve, the European Commission (EC) quietly distributed a draft proposal
to EU member states to
classify gas- and nuclear-powered electricity generation as "green energy options"
eligible for investment
incentives. The underhanded timing of the proposal's release over a holiday weekend
was likely intentional, as
the new "taxonomy" would almost completely undo what tenuous progress was made at
last year's COP26
climate summit. The EC seemed to realize that proposing environmental destruction
as a matter of continental
policy would not be popular with the public, and so distributed the draft at a time
when public attention would
be elsewhere.
The public has since caught up, however. The proposed changes have been met with
bitter protests, and for good
reason. The new rules would effectively derail what has been fairly rapid progress
in transitioning to renewable
energy across Europe and completely end any hope of the EU collectively achieving
its emissions reduction
commitments by 2030. As the EU has until now aggressively marketed itself as the
benchmark for climate
mitigation action as far as energy is concerned, its latest move will likely
encourage other countries to similarly
roll back their own efforts. Under the proposal, which would go into effect in 2023
if approved by a majority of
EU member states, permits for new nuclear plants could continue to be issued until
2045, while permits for work
needed to extend the life of existing nuclear plants could be issued until 2040.
For gas plants, the proposal
establishes a deadline of 2031 for new construction permits, and makes the non-
binding stipulation that carbon
emission limits should be set "well below" those of comparable coal plants.
The reasoning behind the proposed changes is that they would help lower-income
countries transition from coal
power to eventually sustainable sources. The proposal, however, represents a
significant step backward from
what the EU has already achieved. Under its current commitment, the EU would have
to reduce emissions from
gas power plants by 32 to 37 percent by 2030, along with replacing coal plants, in
order to meet its target.
Increasing gas power obviously will not accomplish that. Nuclear power is likewise
not emissions-free, although
it is certainly a far cleaner option. CO2 emissions from operations in most modern
nuclear plants (those with
either advanced reactors or the even newer third-generation EPR pressurized water
reactors) range from 18.2 to
24.4 grams of CO2 per kilowatt-hour (gCO2/kWh). When related emissions from fuel
processing and
construction are factored in, the emissions increase to 40 to 116 gCO2/kWh.
The bigger problem with nuclear power, as critics (most notably the Austrian
government) have pointed out, is
that the new rules would contravene existing EU legal definitions. Under the
current taxonomy, investments and
activities can only be described as "sustainable" if they both contribute to
climate objectives and do no significant
harm to other environmental objectives, including circular economy, biodiversity
and pollution reduction
objectives.

. Page 4 of 36
6. The author is-
(a) Presenting a hypothetical situation.
(b) Portraying a bleak situation.
(c) Portraying his happiness on the situation.
(d) Objectively describes the current environment situation.
7. Which of the following is the closest meaning to the word “Taxonomy”?
(a) Disorder (b) Segregation (c) Unification (d) Nomenclature
8. All of the following can be inferred from the sentence except
(a) The author is unhappy with the proposed changes in classifying gas- and
nuclear-powered electricity
generation as "green energy options".
(b) The draft proposal to EU member states to classify gas- and nuclear-powered
electricity generation as "green
energy options" was a covert attempt to avoid public uproar.
(c) The public found the proposed changes unpalatable.
(d) The new rules would effectively derail what has been fairly rapid progress in
transitioning to renewable
energy across Europe.
9. According to the passage, which of the following is/ are true?
I. EU has until now aggressively marketed itself as the benchmark for climate
mitigation action.
II. The bigger problem with nuclear power is that the new rules would contravene
existing EU legal definitions.
(a) Only I (b) Only II (c) Both I and II (d) None of the above
10. Which of the following reflects the overall tone of the author?
(a) Rabble-rousing (b) Neutral (c) Acclamatory (d) Disquieting
Directions (Q.11-Q.16): As humans continue to rapidly expand the scope of their
domination of nature -
bulldozing and burning down forests and other natural areas, wiping out species,
and breaking down ecosystem
functions - a growing number of influential scientists and conservationists think
that protecting half of the planet
in some form is going to be key to keeping it habitable. The idea first received
public attention in 2016 when
E.O. Wilson, the legendary 90-year-old conservation biologist, published the idea
in his book Half Earth: Our
Planet's Fight for Life. "We now have enough measurements of extinction rates and
the likely rate in the future
to know that it is approaching a thousand times the baseline of what existed before
humanity came along," he
told The New York Times in a 2016 interview.
Once thought of as aspirational, many are now taking these ideas seriously, not
only as a firewall to protect
biodiversity, but also to mitigate continued climate warming. One of the major
reasons for adoption of these
extreme preservation goals is a 2019 report by the Intergovernmental Science-Policy
Platform on Biodiversity
and Ecosystem Services (IPBES), which found that more than 1 million species are at
risk of extinction. That
report concluded that it's not only species that are at risk, however. The myriad
life-support functions that these
species and ecosystems provide also are threatened – everything from clean water
and air to flood control and
climate regulation, food and a host of other services.
The ambitious goal of protecting and restoring natural systems on a large scale is
shared by a number of groups
and people. The Wyss Campaign for Nature is working in partnership with the
National Geographic Society to
support the goals of the so-called "30x30" movement, a highly ambitious initiative
that aims to protect 30% of
the planet, on land and at sea, by 2030.
All eyes are now on the Convention on Biological Diversity (CBD), a multilateral
treaty created by the United
Nations to write a 10-year biodiversity plan. The 2010 CBD meeting called for 17%
of the terrestrial planet to
be protected in some form and 10% of the oceans by 2020. That goal was not reached
- currently about 16% of

. Page 5 of 36
the terrestrial planet has been protected, and less than 8% of marine ecosystems.
So, reaching the 2030 goal
would require a near doubling of land protections and a quadrupling of ocean
protections - all in the next decade.
It's a daunting challenge, even if the will is there, with some countries - notably
Brazil and the US - moving in
the opposite direction. Brazilian President Jair Bolsonaro has opened up the Amazon
rainforest to an onslaught
of land-clearing, logging and agricultural development. And last year the Trump
Administration eliminated the
Landscape Conservation Cooperative Network, an Obama-era program that created 22
research centres to tackle
landscape-level conservation problems across the US.
11. Which of the following is/are true according to the passage?
i. The book Half Earth: Our Planet's Fight for Life is authored by E.O. Wilson.
ii. In a 2016 interview to the New York Times, E.O. Wilson said that we do not have
enough measurements of
extinction rates.
iii. The goal of protecting and restoring natural systems on a large scale is
shared by a number of groups and
people.
(a) Only i) (b) Both i) and iii) (c) Only ii (d) Both ii) and iii)
12. What was the finding of the 2019 Intergovernmental Science-Policy Platform on
Biodiversity and Ecosystem
Services (IPBES) report?
(a) It found that less than 1 million species were at risk of extinction.
(b) It found that more than 1 million species were at risk of extinction.
(c) It found that more than 3 million species were at risk of extinction.
(d) It found that less than 3 million species were at risk of extinction.
13. Which of the followings is working in partnership with the Wyss Campaign for
Nature?
(a) National Geographic Society.
(b) United Nations Environment Programme.
(c) World Wide Fund for Nature.
(d) Earth Day.
14. Which of the following correctly defines the aim of '30x30' movement?
(a) To protect 30% of the existing wildlife in their natural habitat.
(b) To build 30 new dams across the world.
(c) To destroy 30% of the natural resources.
(d) To protect 30% of the planet, on land and at sea, by 2030.
15. According to the passage, what is Convention on Biological Diversity (CBD)?
(a) A unilateral treaty, formed by the United Nations, in order to chart out a 10
years biodiversity plan.
(b) A multilateral treaty, formed by the United Nations, in order to chart out a 10
years biodiversity plan.
(c) A unilateral treaty formed by the United Nations, in order to chart out a 15
years biodiversity plan.
(d) A multilateral treaty, formed by the United Nations, in order to chart out a 20
years biodiversity plan.
16. What did the Landscape Conservation Cooperative Network do?
(a) It created 33 research centres to tackle landscape-level conservation problems
across the US.
(b) It created 22 research centres to tackle ocean-level conservation problems
across the US.
(c) It created 22 research centres to tackle atmospheric-level conservation
problems across the US.
(d) It created 22 research centres to tackle landscape-level conservation problems
across the US.

. Page 6 of 36
Passage: (Q.17-Q.21): Two years into a raging pandemic, Corona continues to have
most of civilization in a
chokehold. As expected, people haven’t taken kindly to the fact that their prayers,
wishes, science-approved
vaccines and safety measures have done next to nothing to make it go away. Needless
to say, fear, fraying
tempers and fraught emotions have come to the fore, creating a toxic climate that
claims as many victims as the
variants of a virus.
People need to present this calamity with a united front and yet, we have seldom
been more divided. We can’t
agree on whether the vaccines are lifesavers guaranteed to save humanity or a
placebo concocted by the
pharmaceutical companies to make trillions and profit from collective suffering in
collusion with heartless
capitalists. The double-jabbed and the vaccine sceptics are butting heads leading
to explosive results with World
No 1 tennis champion, Novak Djokovic, being the most high-profile casualty.
The outspoken Novak Djokovic has been one of the most visible vaccine sceptics and
his stubborn stance has
endeared him to his fans who were infuriated at the public humiliation of a great
champion by slimy politicians
while earning him the dire wrath of most others. His detractors sought to drown him
in a wave of social media#engineered derision, mocking his spiritual beliefs and
the pseudoscience he supposedly peddles. This inability
to find a middle ground in light of the ever-widening chasm between science and
spirituality is our biggest failure
in modern times and it needs to be bridged. We must make the attempt to develop a
system of knowledge that is
free from the fallacies of science and the failings of religion. In the tussle
between faith and intellect, neither can
hope to subsume the other which is how we have arrived at this hopeless impasse.
It wasn’t always this way. Ibn Sina—polymath, philosopher and physician whose
innovative theorizing in
metaphysics elevated the soul to the realm of the intellect—gave the world the
Canon, the foundation of modern
medicine which was taught as a textbook in Europe and the Islamic lands.
Michelangelo was a devout Catholic
who risked eternal damnation to perform dissections on corpses to satisfy his
scientific curiosity about anatomy
which he felt was crucial to enhance his prowess as a sculptor and painter.
India’s great mathematicians like Aryabhata and Brahmagupta managed to find a way
to reconcile the demands
of their faith with science, making unequalled contributions to the study of
numbers even as they gazed upon the
stars. These are the giants we must emulate. After all, differences in race,
culture and beliefs notwithstanding,
diversity is a beautiful thing especially when the divisive elements find a way to
beat the odds and coexist in
truth and harmony.
17. Choose the option which interprets the following line, ‘Corona continues to
have most of civilization in a
chokehold’.
(a) Covid have gripped the whole civilization.
(b) Covid had closely tied civilization to us.
(c) Covid has restrained civilization and continues to do so.
(d) Covid has retrained humans to be civilised and continues to do so.
18. Which of the following is appropriate title for this passage?
(a) Science of the spirituality.
(b) Spirituality behind the science.
(c) Schism between Science and Spirituality.
(d) Divination between science and spirituality.
19. According to the passage, select the correct statement?
(a) Docile stance of Novak Djokovic has endeared him to the politicians.
(b) There is a toxic climate due to the air quality during COVID.
(c) People can present this calamity with segregated view.
(d) Fear, fraying tempers and fraught emotions have come to the fore with the new
virus variant.

. Page 7 of 36
20. What, according to the passage, is the main/significant work of Ibn Sina?
(a) Innovative theorising in metaphysics.
(b) The foundation of modern medicine
(c) Elevated the soul to the realm of the intellect
(d) To perform dissections on corpses.
21. What, according to the author, is the achievement attained by India’s great
mathematician?
(a) Quarrel between science and faith.
(b) To restore demands of faith with science.
(c) To reconcile divine with science.
(d) The relation between science and faith.
Passage(Q.22-Q.26): Northern Ireland has been plunged into political crisis amid
reports that the first minister,
Paul Givan, is to resign over Brexit. Such a move, reported by the BBC’s Stephen
Nolan show, would threaten
the future of the government and trigger the resignation of the deputy first
minister, Michelle O’Neill, Sinn
Féin’s (opposition party) leader in Northern Ireland. Neither leader can stay in
power if the other quits. The
reported move comes just days after the DUP gave Brussels a 21 February deadline to
resolve the dispute over
Brexit checks and just hours after Ireland’s agriculture minister, Edwin Poots,
also a DUP representative, ordered
a halt to Brexit checks on food and farm products coming into Northern Ireland from
Great Britain.
The resignation of the first and deputy first minister would not trigger the
immediate collapse of the Irish
executive allowing it to stumble on till the elections in May. Under the rules, the
parties would have six weeks
to nominate replacements. But critics say the resignation of the two ministers will
further destabilise Northern
Ireland and prevent key decisions on an upcoming budget from being made. Doug
Beattie, the UUP leader, said
that the resignation would not create leverage in the negotiations over the Irish
Sea border but “create more
destabilisation here in Northern Ireland”.
Simon Coveney told the BBC: “It will create more hardship for the people of
Northern Ireland; We will not be
able to get a budget agreed and through, and at the end of it all, the protocol
will still be there.” The latest DUP
manoeuvres are being seen by rivals as positioning ahead of the May elections and
come amid repeated threats
by the party leader, Sir Jeffrey Donaldson, to quit Stormont over the Brexit
checks. The ruling party’s popularity
has fallen over its handling of Brexit, leaving Sinn Féin in pole position to be
the largest party for the first time
according to recent opinion polls.
Alliance party MP Stephen Farry said the resignation of the first minister would be
“an act of huge harm” to
Northern Ireland. The Northern Ireland executive would be unable to approve a
three-year budget that is
currently out to consultation. Also under threat would be the appointment of a
victims’ commissioner to deal
with troubles legacy killings and injuries and a grant scheme to give householders
a £200 grant against rising
energy bills. The Traditional Unionist Voice leader, Jim Allister, told the BBC’s
Stephen Nolan it was “about
time” the first minister resigned, saying it would be “better later than never” if
Givan carried out his threat.
“There can never be a settlement on this issue,” he said of the Northern Ireland
protocol. “The protocol is worse
than a pandemic for our constitution position, the protocol kills the union [of the
United Kingdom]”, said Allister.
22. What is Doug Beattie’s take on the resignation of Paul Givan?
(a) It will not give weight to the discussions over the Irish Sea boundary but
bring stability in Northern Ireland.
(b) It will expedite the process of stability in Northern Ireland and also give
leverage to the discussions over the
Irish Sea boundary.
(c) It will give leverage to the discussions over the Irish Sea boundary, leading
to further instability in Northern
Ireland.
(d) It will cause further instability in Northern Ireland than provide weight to
the discussions over the Irish Sea
boundary.

. Page 8 of 36
23. What is the advantage that the Sinn Féin party has achieved because of the
decline of the reputation of the ruling
party?
(a) It can influence the recent Brexit turmoil.
(b) It can lure in the ministers from the ruling party to join them due to the
decline in their reputation.
(c) It has earned the probability of becoming the largest party for the first time.
(d) It can provide the budget for the coming fiscal year.
24. Which one among the following options can be the reason behind the resignation
of the first minister, Paul
Givan?
(a) Brexit, where the DUP’s setting a deadline of February 21 for Brussels to
resolve the Brexit check dispute
and the Ireland agriculture minister’s order to stop the Brexit checks on food and
farm products coming from
the UK to Northern Ireland.
(b) Rivals perceive the new DUP maneuvers as positioning ahead of the May polls,
and they come after the
party's leader, Sir Jeffrey Donaldson, who has threatened to quit Stormont over the
Brexit checks.
(c) A three-year budget that is presently out for comment would be impossible for
the Northern Ireland
administration to adopt.
(d) Givan realized that his party is bound to lose the majority in the upcoming
elections and hence, he showed
the white feathers, along with some ministers in his cabinet.
25. According to Stephen Farry, which one among the following is NOT a massive act
of harm to Northern Ireland,
following the departure of Givan?
(a) A grant plan to provide homeowners with a £200 payment to help with rising
energy prices, would be
jeopardised.
(b) Providing any negotiating advantage in the Irish Sea border talks would be out
of the question.
(c) The appointment of a victims' commissioner to deal with troubled legacy deaths
and injuries would be
affected.
(d) A three-year budget that is presently out for comment would be impossible for
the Northern Ireland
administration to adopt.
26. Why will the situation in Northern Ireland still be distressful even though the
resignations of the first and deputy
first ministers would not result in the Irish government collapsing immediately?
(a) The parties would have six weeks to designate successors under the guidelines.
(b) It would further lead to instability in Northern Ireland and prevent essential
decisions to be made on a budget
that is forthcoming.
(c) The opposition leader will take advantage of the turmoil, owing to the absence
of leaders in the ruling party
and this will lead to riots and protests.
(d) The vacant post will lead to competition among the ministers, eventually
leading to factions within a single
party.
Passage(Q.27-Q.30): Elizabeth was born nearly 500 years ago, in 1533. Her father,
King Henry VIII, had six
wives. When Elizabeth was two, her father ordered her mother’s execution. ‘Off with
her head!’ Elizabeth was
very clever, but she had a difficult childhood with her bad-tempered father and so
many stepmothers. She was
put in prison in the Tower of London for two months when she was twenty. Elizabeth
became queen in 1558.
‘Your sister is dead. You are the new queen.’ The English people liked Elizabeth
and called her ‘Good Queen
Bess’. But Elizabeth was often bad tempered and bossy. ‘I said give me my wig!’
Elizabeth sent explorers sailing
around the world. In 1577, Sir Francis Drake set sail in his ship, the Pelican.
‘Goodbye, Francis. Discover some
new countries and bring me back some Spanish treasure!’
In 1588, the Spanish king sent a huge fleet of ships to attack England. It was
called ‘the Spanish Armada’.
Elizabeth gave a famous speech to inspire her army. ‘My loving people! I know I
have the body of a weak
woman, but I have the heart and stomach of a king. I will live or die amongst you
all, but I know we shall have
a famous victory!’ Spain lost the battle.

. Page 9 of 36
Elizabeth’s cousin, Mary, was Queen of Scotland. Mary had to run away from Scotland
and she asked Elizabeth
to protect her. ‘Please protect me, Elizabeth!’ Elizabeth was afraid that Mary
wanted to become Queen of
England. She locked Mary up. Nineteen years later she ordered Mary’s execution.
‘Off with her head!’ Elizabeth
was queen for 45 years. It was a golden age for England. She died in 1603, aged 70,
and James I, Mary’s son,
became king. ‘Queen Elizabeth is dead. Long live King James!’
27. In which year, Elizabeth’s mother was got executed by his father?
(a) 1533 (b) 1558 (c) 1577 (d) 1535
28. According to the passage, which of the following statements is incorrect?
(a) In 1588, the Spanish king sent a huge fleet of ships to attack England.
(b) Mary, the Queen of Scotland, was locked up by Queen Elizabeth for nineteen
years, later to be executed.
(c) Elizabeth gave a famous speech to inspire her army and her army won the battle.
(d) The queen of Scotland said, “I have the heart and stomach of a king”.
29. The main objective of the author is to
(a) give a brief biographical anecdote on Queen Elizabeth.
(b) provide background information on Queen Elizabeth.
(c) showcase the accomplishments of Queen Elizabeth.
(d) Narrate a story of Queen Elizabeth from birth till her becoming the queen.
30. ‘My loving people! I know I have the body of a weak woman, but I have the heart
and stomach of a king.” Which
of the following literary devices is reflected in the given statement?
(a) Allegory (b) Imagery (c) Consonance (d) Paradox
Directions (Q.66 – Q.105): Read the comprehensions carefully and answer the
questions based on it.
Passage (Q.66-Q.70): In a noteworthy decision, the Kerala High Court on Wednesday
has ruled that the admin
of a WhatsApp group cannot be held vicariously liable if a member of the group
posts objectionable content in
the group.
Justice Kauser Edappagath observed that this was so because vicarious liability in
criminal law can only be
fastened when a statute prescribes so. The Court also recalled that it is the basic
principle of criminal
jurisprudence that mens rea must be an ingredient of an offence and both the act
and intent must concur to
constitute a crime.
In the facts and circumstances of the case, the specific question to be answered
was whether the petitioner could
be vicariously held liable for the act of the first accused. The Court noted that
vicarious liability in civil and
service matters arises usually because of some or the other legal relationship
between two people which may be
anyone for example there lies a legal relationship between doctor-nurse, master-
servant, minor-guardian etc.
However, relying upon a few precedents, it was found that vicarious criminal
liability can be fastened only by
reason of a provision of a statute and not otherwise. Therefore, since no special
penal law creates vicarious
liability, it was held that an Admin of a WhatsApp group cannot be held liable for
the objectionable post by a
group member.
The Judge also emphasized that there was nothing on record to suggest that the
petitioner had published or
transmitted or caused to be published or transmitted in any electronic form the
alleged obscene material or he
browsed or downloaded the said material or, in any way, facilitated abusing
children online. Since the basic
ingredients of the offences alleged are altogether absent as against the
petitioner, the Court found it a fit case
where it can exercise its extraordinary jurisdiction under Section 482 of Cr.P.C.
(Source: Breaking: Admin Of WhatsApp Group Not Vicariously Liable For Objectionable
Post By Group
Member: Kerala High Court, livelaw)
66. JUGNU and PREETA were two admin who created a group called “FRIENDS”. In March
2020, JUGNU, the
accused posted a porn video depicting children engaged in sexually explicit acts in
the group. Accordingly, a
crime was registered against both the accused under Sections 67B (a),(b) and (d) of
the Information Technology
Act, 2000 and Sections 13, 14 and 15 of the Protection of Children from Sexual
Offence Act. Decide
(a) For publishing sexually explicit content in the Whatsapp group, both jugnu and
preeta will be held
vicariously accountable.
(b) Only Jugnu will be held accountable, as Preeta lacked mens rea and did not
participate in a malicious way.
(c) Jugnu will be held accountable because both the conduct and the intent were
found to be malicious.
(d) Jugnu will be held accountable for publishing or transmitting child abuse
online and facilitating it not Preeta.
67. Continuing with the same circumstances as the last one, what would be the best
argument in Preeta's favour to
allow her to escape vicarious criminal liability?
(a) In the absence of a special penal law creating vicarious liability, an Admin of
a WhatsApp group cannot be
held liable for the objectionable post by a group member.
(b) In the absence of a special penal law creating vicarious liability, an Admin of
a WhatsApp group cannot be
held liable for the objectionable post by another group admin.
(c) A vicarious criminal liability can be fastened only by reason of a provision of
a statute and not otherwise.
(d) Absence of both the act and intent to constitute a crime.

. Page 16 of 36
68. Para 3 of the passage states, “The Court noted that vicarious liability in
civil and service matters arises usually
because of some or the other legal relationship between two people. Thus, the
wrongful act of the servant is
deemed to be the act of the master as well. “The doctrine of liability of the
master for act of his servant is based
on the maxim:
(a) Respondeat superior (b) qui facit per alium facit per se
(c) only A (d) Both A and B
69. Chose a statement which best describes the context of the passage:
(a) In criminal law intention is an essential constituent of the crime, but in law
of torts intention is irrelevant.
(b) Vicarious liability refers to a situation where someone is held responsible for
the actions or omissions of
another person.
(c) The liability of master and servant is joint and several in exceptional cases.
(d) The liability of the admin of a WhatsApp group and its members is joint and
several.
70. A new station waggon was purchased by the Phillips family. They chose this car
since the entire family would
be able to use it. It was big enough to suit everyone's needs. Harry, the youngest
son, was 17 when he decided
to take the car out for a spin. He decided to spin the tires at a stop sign to
impress a bunch of girls. However, he
forgot to put his foot on the brake, forcing the car to crash into the junction,
resulting in a three-car pile-up.
Determine who will be held liable if civil vicarious liability is imposed.
(a) Harry will be held liable for his negligence.
(b) Harry will not be held liable as he is a minor.
(c) The entire family can be held liable.
(d) Harry can be only held liable when a statute prescribes so.
Passage(Q.71-Q.75): The Meghalaya High Court has held that Magistrate cannot employ
his power under
Sections 451/457, Cr.P.C. when trial or inquiry has not been set in motion.
Sections 451 and 457 deal with an
order for custody and disposal of property pending trial and procedure by police
upon seizure of property
respectively.
While setting aside the order of the Magistrate under such Sections, the Single
Judge Bench of Justice W.
Diengdoh observed,
"…at the time of passing of the original impugned order, the matter was still under
investigation by the Customs
officials and the stage of prosecution has not yet commenced as evident from the
fact that the relevant sanction
for prosecution by the Principal Commissioner of Customs or Commissioner of Customs
have not yet been
issued to enable the Court to take cognizance of the offence. Therefore, the
learned Magistrate in the absence of
a trial or inquiry could not have passed the said impugned order under Section
451/457 Cr.P.C." court also
observed that if the case is in pursuance with the special law no court will
interfere in it.
"It may be mentioned that the Customs Act, 1962 is a special Act while the Code of
Criminal Procedure deals
with the general law. It is also well settled that the provisions of the Special
Act will override the provisions of
the general law. The principles that general law yields to special law should they
operate in the same field on
same subject will be applicable here and if does not happen so then general law
will be applicable."
(Source: Magistrate Can't Use Power U/S 451/457 Cr.P.C. When Trial Or Inquiry Has
Not Been Set In Motion:
Meghalaya High Court, livelaw)
71. On the basis of specific information, the revenue officers, shilong unit had
intercepted three trucks on
09.04.2021. On being searched, a total number of 1500 bags of foreign origin black
pepper weighing about
75,000 KGs were seized. The three drivers and two helpers were arrested the said
trucks were also seized under
the relevant provisions of the Customs Act, 1962 ("the Act"). The accused will be
tried and prosecuted under the
provision of the Customs Act. Owners of the truck moved to high court seeking
release of the items seized as
they were seized beyond the period prescribed in Code of criminal procedure. Choose
a suitable alternative in
the light of information given in the passage and facts regarding the jurisdiction
of High Court:
(a) The high court will have jurisdiction to try the case and hence can order for
release of items seized on bonds.

. Page 17 of 36
(b) The high court cannot encroach upon the jurisdiction of special law.
(c) The principles that sates general law yields to special law should they operate
in the same field on same
subject will be applicable here.
(d) Because the prescribed timeframe for seizing goods has been surpassed, the high
court will have jurisdiction.
72. Both Criminal Procedure Code and Passports Act provide impounding seizure of
passports respectively. What
will be the position of special law if both the acts do not operate in the same
field on same subject?
(a) Special will yield to general law.
(b) General law will prevail.
(c) Court will draw a harmonious construction between the two statutes.
(d) General Law will prevail over special law.
73. Jeetu was detained by a court order on murder accusations, and his passport was
subsequently seized
(confiscated) by the investigating authorities after the order of court under the
provision of CrPC(General law).
Criminal Procedure Code and Passport act provides for seize(confiscation) of
passport by court and impounding
of passport by police officer respectively.. Jeetu makes an application under
section 482 of the HC to have the
investigation authorities' order overturned. Decide keeping mind that passport act
holds the position of special
law.
(a) Jeetu will succeed as an investigating authority, which is governed by general
law, has trampled on the
jurisdiction of special law.
(b) Jeetu will not succeed as investigating authorities which is governed by
general law encroached upon the
jurisdiction of special law.
(c) Jeetu will succeed as the court will order in jeetu’s favour.
(d) Jeetu will not succeed as the principles that states general law yields to
special law should they operate in
the same field on same subject will not be applicable here.
74. In Latin Maxim special Provision prevails over the general provisions do means
(a) Generalia specialibus non derogant.
(b) Contemporanea Exposition Est Fortissima in lege
(c) Ut res Magis valeat quampareat.
(d) Noscitur a sociis
75. ASSERTION: Special law can be made and will prevail over general law.
REASON: The principles that states general law yields to special law should they
operate in the same field on
same subject will be not be applicable in case if the provision of special law is
in conflict with that of fundamental
rights.
(a) Both A and R are true and R is correct explanation of A.
(b) Both A and R are false but R is not correct explanation of A.
(c) A is true but R is false.
(d) A is false but R is true.
Passage (Q.76-Q.80): Justice Jasjit Singh Bedi has held that in addition to there
being a requirement of mens
rea pointing to instigation by the accused to goad the deceased into committing
suicide, there must be a
proximate and live link present between such abetment and the consequent suicide,
to establish the offence of
abetment to suicide under S.306 of IPC.
The ingredients of abetment must be fulfilled to establish the abetment of suicide
under Section 306 of IPC and
merely a suicide note with the name of the accused shall not suffice for the same,
Punjab and Haryana High
Court has held. The apex court had further added that the words uttered in a
quarrel or words uttered in the spur
of a moment could not be taken to be laced with mens rea.
The High Court further referred to the Apex Court's judgments in Netai Dutta v.
State of West Bengal,
S.S.Chheena v. Vijay Kumar Mahajan & Anr., and Gurcharan Singh v. the State of
Punjab, where it had been

. Page 18 of 36
held that "to constitute abetment, the intention and involvement of the accused to
aid or instigate commission of
suicide are imperative and any severance or absence of these constituents would
militate against this indictment."
Further stating that "Contiguity, continuity, culpability and complicity of the
indictable acts or omission are the
concomitant indices of abetment.", the Supreme Court had succinctly put that "S.306
of IPC criminalizes the
sustained incitement for suicide."
(Source: S. 306 IPC | Merely Being Named In Suicide Note Doesn't Establish Guilt Of
Accused, Proximate Link
Between Alleged Abetment & Suicide A Must: P&H HC)
76. Jaswinder Singh had found that his son had committed suicide by hanging from
the girder by tying a bed sheet
around his neck and a note was found in front of his chest stating that he had
committed suicide because of
harassment by the accused. Accused had been charged with S.306 of IPC. Choose an
appropriate alternative that
will work in the accused's favour.
(a) Prima facie requirement of mens rea pointing towards instigation by the
accused.
(b) There must be a link between abetment and the consequent suicide.
(c) Ingredients of abetment are not fulfilled.
(d) Suicide note with the name of the accused.
77. Determine the culpability of accused in the circumstances given above?
(a) The accused cannot be held accountable under section 306 of the IPC because
there was no purpose or
involvement on the part of the accused to aid or provoke the commission of suicide.
(b) The accused cannot be held accountable under section 306 of the Indian Penal
Code since there is no
proximate and live link between the abetment and the later suicide.
(c) The letter be treated as dying declaration and hence the accused will be liable
under the offence of abetment
to suicide under S.306 of IPC.
(d) Both (A) and (B)
78. Assume, in the same circumstances as given in Q76, that the accused had a
quarrel with the deceased the day
before the incident and left the town. Because the deceased was unable to cope with
the situation, he decided to
commit suicide after being told he was useless. S.306 of the IPC had been filed
against the accused.
(a) The accused will be charged with abetment to suicide under Section 306 of the
Indian Penal Code.
(b) Since he immediately left town and was not present at the time of the incident,
the accused would not be
held guilty for the offence of abetment to suicide under S.306 of the IPC.
(c) As a proximate and live link exists between such abetment and the subsequent
suicide, the accused will be
held accountable for the offence of abetment to suicide under S.306 of the IPC.
(d) The accused will not be held accountable since he lacked mens rea.
79. ASSERTION: S.306 of the IPC do not decriminalizes the prolonged incitement to
suicide.
Reason: The indicators of abetment are contiguity, continuity, culpability, and
complicity of the indictable acts
or omissions.
(a) Both A and R are true and R is correct explanation of A.
(b) Both A and R are true but R is not correct explanation of A .
(c) A is true but R is false.
(d) A is false but R is true.
80. Chavi had a very bad influence on Martha, his girl friend. She even told him
that if at any point he leaves her,
she will commit suicide. Threatened by her continuous warnings, chavi broke up
Martha. Martha on the next
day died consuming a rat poison. Decide the culpability of chavi in the present
circumstances under the offence
of abetment to suicide under S.306 of IPC.
(a) Chavi should be held accountable since he broke up with Martha despite knowing
that she would commit
suicide if he leaves her.
(b) Chavi should not be held liable as he was threatened by Martha’s psychotic
nature.

. Page 19 of 36
(c) Chavi should be held liable asthere is a proximate cause between the acts of
chavi’s breaking up with Martha
and subsequent suicide.
(d) Chavi should not be held liable as he did not abetted neither had the intention
or involvement in aiding or
instigating commission of suicide.
Passage(Q.81-Q.84): The Delhi High Court has reiterated the various necessary
factors to be kept in mind before
making a person vicariously liable for offences committed by a company under sec.
138 of the Negotiable
Instruments Act, 1881.
Justice Subramonium Prasad listed out the following factors as laid down by the
Supreme Court in various
judgments:
- The primary responsibility is on the complainant to make specific averments as
are required under the law in
the complaint so as to make the accused vicariously liable otherwise such complaint
will be dismissed. For
fastening the criminal liability, there is no presumption that every Director knows
about the transaction.
-Section 141 does not make all the Directors liable for the offence. The criminal
liability can be fastened only
on those who, at the time of the commission of the offence, were in charge of and
were responsible for the
conduct of the business of the company.
-Vicarious liability can be inferred against a company registered or incorporated
under the Companies Act, 1956
only if the requisite statements, which are required to be averred in the
complaint/petition, are made so as to
make the accused therein vicariously liable for offence committed by the company
along with averments in the
petition containing that accused were in charge of and responsible for the business
of the company and by virtue
of their position they are liable to be proceeded with.
-The person sought to be made liable should be in charge of and responsible for the
conduct of the business of
the company at the relevant time. This has to be averred as a fact as there is no
deemed liability of a Director in
such cases.
The Court added that it was for the petitioner to establish in trial that he was
not responsible for the conduct of
the business of the company owing to his age and that the mere ipse dixit of the
petitioner that he is 80 years of
age and was unable to manage the affairs of the company could not be accepted and
the complaint could not be
quashed on that basis under section 482 of CrPC.
(Source: Delhi High Court Reiterates Factors To Be Considered Before Making A
Person Vicariously Liable
For Offences By Company Under S.138 NI Act, livelaw)
81. The passage talks about vicarious liability of the Directors of a company under
Sections 138 and 141 of the
Negotiable Instruments Act, 1881. In light of the passage's context, choose an
incorrect statement:
(a) The Court explained the duty of a Magistrate while issuing process and his
power to dismiss a complaint.
(b) It is settled law that at the time of issuing of the process the Magistrate is
required to see only the allegations
in the complaint and where allegations in the complaint or the charge-sheet do not
constitute an offence
against a person, the complaint is liable to be dismissed.
(c) If, at the time the offence was committed, the person accused was in charge of,
and responsible for the
conduct of business of the company and if statutory compliance of Section 141 of
the NI Act has been made,
the High Court cannot quash the proceedings against the person accused under
Section 482 CrPC.
(d) Merely being a director of a company is not sufficient to make the person
liable under Section 141 of the
Act.
82. Last para of the passage provides that it was for the petitioner to establish
in trial that he was not responsible for
the conduct of the business of the company owing to his age and that the mere ipse
dixit of the petitioner that he
is 80 years of age and was unable to manage the affairs of the company could not be
accepted and the complaint
could not be quashed on that basis. What does the expression “ipse dixit” mean?
(a) A person's own assertion without relying on any authority or proof.
(b) A dogmatic and unproven statement.
(c) A person's own assertion relying on any authority or proof.
(d) A statement proved on the basis of evidence.

. Page 20 of 36
83. Radhe filed a complaint against his company's directors under section 141 of
the NI Act. It was claimed that
starting in 2019, employees' salaries, including the complainant's, began to be
delayed, and the corporation failed
to meet statutory requirements such as PF, ESI, and other benefits. It was also
said that, because the dues and
arrears of salary were rising, the complainant had offered to take a salary cut at
the request of the Director. When
the cheques were presented for encashment, however, they were returned with the
note "insufficient fund."
Choose a statement that will be taken into account when the directors are held
vicariously accountable for the
company's operations.
(a) It has to be averred as a fact as there is no deemed liability of a director in
such cases.
(b) The person sought to be made liable is in charge of and responsible for the
conduct of the business of the
company at the relevant time.
(c) Complainant to make specific averments as is required under the law in the
complaint so as to make the
accused vicariously liable.
(d) Complainant to make specific averments as is required under the law that all
directors are aware of the
transaction.
84. ASSERTION: Accsued has the right to approach court under Section 482 CrPC, to
quash the process under
Sections 138 and 141.
Reason: Section 141 extends criminal liability on account of dishonour of cheque in
case of a company to every
person who at the time of the offence, was in charge of, and was responsible for
the conduct of the business of
the company.
(a) Both A and R are true and R is correct explanation of A.
(b) Both A and R are true but R is not correct explanation of A
(c) A is true but R is false.
(d) A is false but R is true.
Passage(Q.85-Q.89): A Delhi Court has observed that for a contract to be
enforceable, the restraint of trade
clause included in it must be reasonable. It added that the reasonability can be
ascertained on the grounds of
time period, geographic location and scope of work.
Doing a legislative analysis of sec. 27 of the Indian Contract Act which states
that every agreement by which
any one is restrained from exercising a lawful profession, trade or business of any
kind, is to that extent void,
the Court said:
"There is one exception to this rule-that if the goodwill of a business has been
sold, an agreement to refrain from
carrying on similar business, if it appears to the Court to be reasonable, would be
protected and would be
enforced." Another exception is found in the Partnership Act.
It is a sufficient justification, if the restriction is reasonable, that is, in
reference to the interests of the parties
concerned and reasonable in reference to the interests of the public, so framed and
so guarded as to afford
adequate protection to the party in whose favour it is imposed, while at the same
time it is no way injurious to
the public," the Judge added.
The Court cannot restrain the defendant from using his own acumen or skills to
compete with the plaintiff in the
same kind of business. The restriction which the plaintiff is seeking to impose
upon the defendant is of permanent
and unlimited nature and is not reasonable," the Court said.
Accordingly, the plaintiff's application was dismissed.
(Source: Restraint Of Trade Clause Must Be Reasonable For A Contract To Be
Enforceable: Delhi Court,
Livelaw)

. Page 21 of 36
85. In which of the following agreements, restraint of trade is valid?
(a) An agreement to purchase an intangible asset owned by, and associated with, the
operation of a business
entity.
(b) Contracts involving the buying illegal weapons, murder for hire contracts, and
contracts to buy drugs.
(c) Two similar business owners, in a partnership, came to an agreement that only
one of their factories would
work at a time and the profit will be shared between them.
(d) All of the above.
86. Which one of the following statements is incorrect?
(a) A restraint of trade clause in a contract of employment may restrict an
employee's freedom to contract both
during and after the termination of his employment.
(b) In some circumstances where severance is possible a court will strike out an
unreasonable part of a restraint
of trade and allow the reasonable part of the clause to be enforced.
(c) A restraint of trade clause may be valid if the person seeking to enforce it
can show it is reasonable in the
circumstances.
(d) If a restraint of trade clause is too wide the courts can change its wording to
make it reasonable.
87. Sunita has been engaged as a senior manager by the XCL agency. She had to sign
a bond with the company
stating that she would stay for at least three years and that if she decides to
leave early, she would be barred from
working in a similar business for a year. For bounty hunting criminal refugees, the
company collaborates with
Los Angeles police departments. Sunita was not allowed to work in a similar
industry as she had access to
sensitive information that could be exploited against the public good. Determine
whether the reasonable
restriction clause is valid.
(a) It is valid as it in reference to the interests of the parties concerned.
(b) It is not valid as restriction of trade and business is a violation of
fundamental right guaranteed under article
19 (1) (g) of COI.
(c) It is not valid as one cannot restrain the defendant from using his own acumen
or skills to compete with the
plaintiff in the same kind of business.
(d) It is valid as the restriction is justified on grounds of public interest.
88. COLA COCO wants to sell their goodwill to Co Pepsi. What will NOT be a fair
restraint in a trade deal, according
to your interpretation of the passage?
(a) Restraining seller from using firms name in the market.
(b) Restraining seller from presenting himself as carrying on the business of the
firm.
(c) Restraining the buyer solicit the custom of persons who were dealing with the
firm before its dissolution
(d) The seller can carry on any business similar to that of the firm within
specified local limits with reasonable
justification
89. ASSERTION: A negative covenant that the employee would not engage himself in a
trade or business or would
not get himself employed by any other master for whom he would perform similar or
substantially similar duties
is not therefore a restraint of trade.
REASON: Negative covenants operative during the period of the contract of
employment when the employee
is bound to serve his employer exclusively are generally not regarded as restraint
of trade and therefore do not
fall under Section 27 of the Contract Act.
(a) Both A and R are true and R is correct explanation of A.
(b) Both A and R are true but R is not correct explanation of A
(c) A is true but R is false.
(d) A is false but R is true.

. Page 22 of 36
Passage(Q.90-Q.94): Right to appeal in criminal law jurisprudence acquires a
special position, since it draws
upon the ideological base of Article 21 of the Constitution of India. As per
Article 21 of the constitution, life and
personal liberty of a person cannot be taken away by state except in accordance
with the procedure established
by law. The procedure established by law has been creatively interpreted by the
Supreme Court to include within
its scope the implicit guarantee of a fair, just and reasonable procedure. Right to
appeal in guarding against any
suffering due to errors of the court ensures fairness of this procedure and
therefore forms an essential part of the
criminal law system.
A complainant is a person who prefers a complaint as defined under Section 2(d) of
CrPC to the magistrate, upon
which the court can subsequently take cognizance of the offence under Section
190(1)(a), CrPC.
Section 378(4) provides the complainant with the right to an appeal against an
order of acquittal passed by a
criminal court to the High Court. However, this right is subject to an important
condition i.e. grant of special
leave to appeal by the HC to appeal from such order, upon application made by the
complainant. It is important
to note that such leave is important for the "very presentation of appeal", i.e.
memorandum of appeal can only
be presented when a grant of appeal has been made by the HC. Now, as clear from the
legislative framing of
Section 372 proviso, this right to appeal is not restricted by the condition of
obtaining any leave from the High
court in case of FIR.
The current position discriminates between a person who is only a victim and not a
complainant, and a person
who is a victim and also a complainant. While the person in the first category has
the right to appeal under
Section 372, without any limitation of obtaining a special leave; the second
category has to mandatorily obtain
the same.
[Extracted with revisions from ‘Right To Appeal Against Acquittal Under Criminal
Procedure Code, 1973 : The
Rule Of Victim v. Complainant’ published on livelaw
https://www.livelaw.in/lawschool/law-school-cover#story/right-to-appeal-crpc-
acquittal-order-victim-complainant-section-327-section-378-187753]
90. X committed theft against Y's property by stealing his bike from his
possession. Y makes a complaint of the
same under Section 2(d) of CrPC, upon which cognizance is taken under Section
190(1)(a), and subsequently an
acquittal order is passed by the court. The lower court acquits X. Does Y have to
obtain a special leave to file
appeal against X's acquittal?
(a) Yes, because Y is only a complainant and not a victim since he has made a
complaint under Section 2(d) of
CrPC
(b) Yes, because Y is both a complainant since he made a complaint under Section
2(d) of CrPC as well as a
victim since he suffered a loss of his property
(c) No, because Y is only a complainant and not a victim since he has made a
complaint under Section 2(d) of
CrPC
(d) No, because Y is a victim since he suffered a loss of his property and so he
does not face any limitation of
obtaining a special leave
91. The Government passed a new law called SADA. Under this law, any person could
be arrested in the interest of
national security. However, unlike other laws which allow an individual to be
granted bail as well as to be
produced before a magistrate within 24 hours of arrest, SADA expressly prohibited
these rights. Mohan is
arrested in the interest of national security under SADA. His bail application was
also not heard. Can he contest
that this is a violation of his personal liberty?
(a) Yes, because Mohan’s personal liberty has been curtailed because his bail
application was not even heard
(b) No, because Mohan's personal liberty has not been curtailed at all and hearing
the application of bail is up to
the discretion of the Court
(c) Yes, because SADA lays down a fair, just and reasonable procedure for granting
of bail
(d) No, because Mohan' personal liberty has been curtailed in accordance with the
procedure established by law

. Page 23 of 36
92. Shakti made a complaint to the magistrate against Sonam for forgery. Sonam was
Shakti’s servant who had
forged Shakti’s signature on a cheque of Rs. 1 lakh in Sonam’s name. When Shakti
found out about this, she
immediately filed a complaint with the magistrate. When the matter was heard by the
district court, Sonam was
acquitted because Shakti was unable to give cogent proof that it was Sonam only who
had forged the signatures.
Shakti wanted to appeal against this decision and filed a special leave to appeal
in the High Court. However, the
leave to appeal was dismissed. Shakti is very sure that it was Sonam who forged the
cheque. Can she appeal
against Sonam’s acquittal?
(a) Yes, because Shakti is a complainant and in order to file an appeal against the
decision of the district court,
she needs to obtain a special leave to appeal.
(b) Yes, because Shakti has already filed for a special leave to appeal against the
decision of the district court
before the High Court.
(c) No, because Shakti has to obtain a special leave to appeal in order to appeal
against Sonam’s acquittal but
the same has been denied.
(d) No, because Shakti has already filed for a special leave to appeal against the
decision of the district court
before the High Court.
93. In the previous situation, the factual scenario changed. The High Court granted
Sonam’s special leave to appeal
after Shakti was acquitted by the lower court. While the lower court did not grant
her justice, Shakti was resolved
to take up the matter with higher authorities. With this change in facts, is Shakti
now in a position to file an
appeal against Shakti before the High Court?
(a) Yes, because Shakti is a complainant and in order to file an appeal against the
decision of the district court,
she needs to obtain a special leave to appeal.
(b) Yes, because Shakti has obtained a special leave to appeal from the High Court
(c) No, because Shakti is a complainant and in order to file an appeal against the
decision of the district court,
she needs to obtain a special leave to appeal
(d) No, because Shakti is a victim and she did not need to obtain a special leave
to appeal in the first place
94. R subjected Q to unwanted physical advances. Despite Q being visibly
uncomfortable, R continued his predatory
behaviour. This left Q in a mentally and physically disturbed state and she decided
to file an FIR. A trial
commenced and R was let go scot-free. This development was very upsetting for Q as
no justice was meted out
to her as a victim. She wants to file an appeal against the decision of the trial
court. If Q has not obtained a
special leave to appeal, can she still file an appeal before the High Court?
(a) No, because Q is a complainant so in order to appeal against R’s acquittal, she
needs to obtain a special leave
to appeal
(b) No, because Q has the right to appeal under Section 372 without any limitation
of obtaining a special leave
(c) Yes, because Q is a complainant so in order to appeal against R’s acquittal,
she needs to obtain a special
leave to appeal
(d) Yes, because Q has the right to appeal under Section 372 without any limitation
of obtaining a special leave
Passage(Q.95-Q.99): A Supreme Court bench comprising Justices KM Joseph and S.
Ravindra Bhat observed
that “Practices or rules or norms rooted in historical prejudice, gender
stereotypes and paternalism have no place
in our society”. The Court said this in its recent judgment while disapproving a
gender-based stereotype. The
Supreme Court held that the condition imposing a gender cap as to the number of
women or men, who can
perform in orchestras and bands in licenced bars is unconstitutional.
The court added that, such measures which claim protection in reality are
destructive of Article 15(3) as they
masquerade as special provisions and operate to limit or exclude altogether women’s
choice of their avocation.
In case there were any real concern for the safety of women, the state is under a
duty to create situations
conducive to their working, to run that extra mile to facilitate their employment,
rather than to thwart it, and
stifle their choice.
. Page 24 of 36
Article 15(3) states that the state is open to make special provisions for women
and children. However, the court
have clarified that such special provisions can be made in favour of and not
against women. It implies that state
can discriminate in favour of women against men but not vice versa.
The SC have repeatedly emphasized that whenever challenges arise, particularly
based on gender, it is the task
of the judges to scrutinize closely, whether, if and the extent to which the
impugned practices are rooted in
historical prejudice, gender stereotypes and paternalism. Such attitudes have no
place in our society
Article 19(1)(g)- All citizens shall have the right to practise any profession, or
to carry on any occupation, trade
or business. The fundamental right is not absolute but subject to any existing law
or subsequent law imposing
reasonable restrictions in the interests of general public.
(SOURCE: https://www.livelaw.in/top-stories/supreme-court-orchestra-bars-gender-
cap-unconstitutional#hotel-priya-a-proprietorship-vs-state-of-maharashtra-192337,
https://indiankanoon.org/doc/1218090/)
95. The State of Maharashtra enacted a law wherein it restricted women from working
in liquor shops. A widow,
Samaira who has a son is the only bread-earner in her family. Because of the newly
enacted law, she was unable
to maintain the education of her son and the daily food for her family. Suppose you
are a lawyer known to
Samaira and she asks for your help. What would you suggest her?
(a) I would suggest her to file a writ petition before the SC because her
fundamental right under Article19(1)(g)
and Article 15(3) is violated.
(b) I would suggest her to find any other source of living as it is a law brought
in public interest.
(c) I would suggest her to keep working in the liquor shop as she has a son to feed
and teach.
(d) I would suggest her to ask the government for an alternative source of income.
96. Taking the same facts as above into consideration. Suppose Samaira approaches
the Supreme Court against the
law. In light of the above passage, what shall the Supreme Court (SC) do?
(a) The SC shall strike down the law only on the ground that the fundamental right
of women under Article
19(1)(g) is violated.
(b) The SC shall not strike down the law because it is imposing reasonable
restriction in the interest of general
public.
(c) The SC shall not strike down the law because the state is empowered to make
special provisions in the
interest of women.
(d) The SC shall strike down the law on the ground that it is violative of Article
19(1)(g) and also Article 15(3).
97. The Union of India enacted a policy under which, a condition was made in
respect to women army officers. The
condition read that, ‘a serving unmarried women army officer must obtain prior
permission of the authorities
before marriage’. A group of women officers offended by the said policy, challenges
it for being against them
before the SC. Decide its validity in the light of above passage?
(a) The policy is invalid down because it is violating right to privacy of women.
(b) The policy is invalid because it has kept in its ambit only unmarried women and
not men and hence
discriminatory.
(c) The policy is invalid because the state can discriminate in favour of women and
not against them.
(d) The policy is valid because it is in interest of women, where life becomes
quite different after marriage.
98. Suppose, India from few months back is witnessing an increasing rate of
prostitution (the practice or occupation
of engaging in sexual activity with someone for payment). Since, there is no law to
prevent it, the media is
debating this issue on a wide scale and criticising the government. In the light of
above passage, decide what
should the government do?
(a) The Sate shall immediately enact a law banning prostitution.
(b) The State shall enact a law criminalising prostitution.
(c) The State shall enact a law preventing forceful prostitution.
(d) The State shall not do anything because it falls under freedom of profession.

. Page 25 of 36
99. Suppose a woman from Rajasthan challenges before the SC, the traditional
practice of compulsorily doing
ghunghat infront of her elder in-laws. In the light of the above passage, how is
the SC supposed to deal?
(a) The SC shall not interfere with the practice because it a tradition.
(b) The SC shall interfere and find out whether it as a gender stereotype.
(c) The SC shall not interfere because it may be choice of some women.
(d) The SC shall interfere and strike down the practice without investigating the
matter.
Passage(Q.100-Q.105): The Supreme Court on Tuesday held that if the prosecution has
failed to prove the basic
facts alleged against the accused, the burden of proof cannot then be shifted to
the accused by resorting to Section
106 of the Indian Evidence Act A Bench held that Section 106 of the Evidence Act is
not intended to relieve the
prosecution from discharging its duty to prove the guilt of the accused.
Section 106 reads as, when any fact is especially within the knowledge of any
person, the burden of proving that
fact is upon him.
"The prosecution having failed to prove the basic facts as alleged against the
accused, the burden could not be
shifted on the accused by pressing into service the provisions contained in Section
106 of the Evidence Act," the
Court said.
The Court also placed reliance on its 1956 judgment in Shambu Nath Mehra vs. State
of Ajmer which had held
that Section 106 of the Evidence Act is designed to meet certain exceptional cases
in which it would be
impossible, or at any rate disproportionately difficult, for the prosecution to
establish facts within the accused's
knowledge.
The accused had been charged and held guilty by the Sessions Court for offences
under Sections 302 (murder)
read with Section 34 (criminal act with common intention) and Section 201 of the
Indian Penal Code (causing
disappearance of evidence of office, or giving false information to screen
offender).
The Bench relied on the judgment in Sharad Birdhichand Sarda v. State of
Maharashtra, and opined that “it
is a settled position of law that circumstances how so ever strong cannot take
place of proof and that the guilt
of the accused have to be proved by the prosecution beyond reasonable doubt.”
Specifically on the contention of the prosecution regarding Section 106, the Court
said that the same was
misplaced since the provision is not intended to relieve the prosecution of burden
of proof. Thus accused is
acquitted.
Source: https://www.barandbench.com/news/prosecution-cannot-shift-burden-of-proof-
to-accused-by#resorting-to-section-106-indian-evidence-act-supreme-court
100. During a murder trial, the accused in the case, A, was being tried for same
and his guilt was proven adequately
by the prosecution. Now the court gave A the opportunity to prove his innocence in
front of the court by
defending him. Has the court done so correctly?
(a) Yes, as the court was obliged to give A the opportunity to defend himself.
(b) No, as the burden of proof is being shifted here in the case.
(c) No, as the court should satisfy the guilt based on the prosecution’s arguments
only.
(d) Yes, as the prosecution herein has proven guilty beyond doubt, thus opportunity
to defend should be given
to the accused.
101. In the above case, had the prosecution not been able to prove the guilt beyond
reasonable doubt, but it was
established that the accused was an escaped offender, would the court be liable to
hold the accused guilty for
murder unless he is able to prove his innocence?
(a) No, as that would amount to the court showing bias in this case.
(b) Yes, as the accused herein is a repeat offender and deserves the strictest
punishment for his erstwhile crimes.
(c) Yes, as the accused also deserves an opportunity for defending himself and
thus, should be allowed to
contend.
(d) No, as the prosecution failed to prove the innocence so burden of proof will
not shift.

. Page 26 of 36
102. While considering a murder case, the prosecution had presented all
circumstantial evidence, and was about to
move on to summoning and presenting eyewitnesses, however, satisfied by the
evidence; the court allowed for
the prosecution to skip the same and held the accused as guilty of the offence. Was
the court correct in doing so?
(a) No, as the matter here was not proven beyond reasonable doubt here.
(b) No, as the court jumped the gun in not hearing the witnesses.
(c) Yes, as the court has complete discretion in deciding the matter and being
satisfied with the extent of the
proof showed.
(d) Yes, as the court was satisfied of the guilt here and decided to expeditiously
dispose of the trial.
103. In the above case, if the guilt been proven beyond reasonable doubt by the
prosecution, and in furtherance of the
same, the court allowed section 106 to be invoked in order to obtain the facts
pertaining to the accused within
his knowledge, would the same amount to a contravention of law here?
(a) No, as the guilt was proven beyond reasonable doubt.
(b) No, as the section was invoked only to obtain particular facts and not for
proving guilt.
(c) Yes, as the court cannot invoke the section without the prosecution praying for
the same.
(d) Yes, as the court has untenably shifted the burden of proof on the accused
instead of the prosecution.
104. Abhay and Kanishka were two contracting parties who were now arguing before
the court since the allegations
were being presented wherein it was prayed by Abhay that the contract was sullied
and breached by Kanishka
and should be punished for the same. The court, in order to obtain facts pertaining
to why the breach occurred
allowed for section 106 IEA to be invoked and ordered Kanishka to make submissions
appropriately and testify.
Is the court correct in doing so?
(a) Yes, as the court is empowered to get to the bottom of all the matters.
(b) No, as the matter here does not fall within the ambit of the passage above.
(c) No, as the IEA does not apply to civil proceedings.
(d) Yes, as the court herein is a criminal court and should be allowed to determine
the standard of guilt here.
105. In the above mentioned case, assuming that section 106 is applicable to civil
proceedings, and the plaintiff wants
to prove the intent of breaching the contract beyond reasonable doubt here, can the
court invoke the section 106
and burden of proof be shifted to kanishka?
(a) Yes, as the breach of a contract is a criminal offence.
(b) No, as the breach of contract is not a criminal offence here.
(c) Yes, as the offender has committed a criminal offence and can be made to
testify in furtherance of section
106.
(d) No, as the extent of proof is not mentioned.

. Page 27 of 36
SECTION - D: LOGICAL REASONING
Passage(Q.106-Q.110): The launch of a new digital rupee in the upcoming year will
boost financial inclusion
in Asia’s No.3 economy and eventually make more Indians cut their dependence on
physical cash, according to
accountants, currency exchange operators and crypto experts. The move will also
provide a cost-efficient
alternative to cash and make international transactions easier, they said. The
launch “would give great comfort
and trust in the use of digital currency,” Lily Vadera, a former executive director
at the Reserve Bank of India
told DH.
It “would provide a cost-efficient alternative to cash as the cost of printing,
storing, transportation and
distribution of currency can be reduced considerably and hopefully bring down the
dependence of cash in the
economy,” said Vadera, who is currently a senior advisor at law firm Cyril
Amarchand Mangaldas. The
comments came just a day after Finance Minister Nirmala Sitharaman used her budget
speech to reveal that the
Reserve Bank of India will launch its digital currency in the year starting April
2022 and tax the income from
the transfer of virtual assets at 30 per cent.
The digital rupee, or the central bank digital currency (CBDC), could be exchanged
for cash, Prime Minister
Narendra Modi clarified, adding that it will make online payments more secure and
give a push to the digital
economy down the lane. Denizens of the digital currency world agreed. The experts
see RBI determining the
supply of the digital rupee and also tracking how it gets distributed to various
individuals If the CBDC runs on
blockchain technology, it will be extremely advantageous to the average Indian,
they pointed out.
“The RBI’s digital currency will help even those who do not have a bank account as
it can be digitally
transferred”, Bhagaban Behera, CEO & Co-Founder, Defy said. “This type of
transaction wouldn't need to pass
through multiple banks and take several days. It could all happen nearly
instantaneously on one digital ledger.
Consumers also wouldn't need a commercial bank account to use CBDC”.
“Being on blockchain will also make international transactions simpler - a person
in the US or Europe can easily
transact with the Indian Rupee, if anyone prefers to,” Subburaj told DH. One
disadvantage at the start would be
the fact that only a small share of the Indian Rupee in circulation may be on the
blockchain, and hence, large
transactions may have to wait, he said. Others such as Amit Jaju, Senior Managing
Director at Ankura
Consulting’s India arm urged the government to closely evaluate the broader
concerns related to money
laundering, fraud, data privacy and cyber security tied to cryptocurrencies.
Bengaluru-based chartered
accountant B E Kumar Prasad urged people to pick the RBI-backed digital rupee over
unregulated crypto assets
citing the lower risks generally associated with an asset regulated by a central
bank.
106. Which of the following can be inferred from the passage?
(a) Experts see digital rupee as a big boon to India.
(b) Experts criticize the digital rupee.
(c) Experts focused on evolution of the digital rupee.
(d) Experts present a dismal future for digital rupee.
107. “It would provide a cost-efficient alternative to cash as the cost of
printing, storing, transportation and
distribution of currency can be reduced considerably and hopefully bring down the
dependence of cash in the
economy,” said Vadera, who is currently a senior advisor at law firm Cyril
Amarchand Mangaldas. Which of
the following (s) is/are assumptions behind the lines?
Assumptions:
I. The alternative to cash is good for the economy.
II. The law firm Cyril Amarchand Mangaldas is a reputable firm.
III. Vadera is one of the senior advisors at Cyril Amarchand Mangaldas.
IV. Reliance on cash in the economy will significantly condense with the
alternative to cash.
V. Cash currency involves the cost of printing, storing, transportation and
distribution of currency.
(a) I, III & IV (b) III, IV & V (c) I, & II (d) All of the following.

. Page 28 of 36
108. Managing Director at Ankura Consulting’s India arm urged the government to
closely evaluate the broader
concerns related to which of the following?
I. Money laundering
II. Fraud
III. Data privacy
(a) Only I (b) Only II (c) Only III (d) All of the above
109. Based on the understanding of the passage, two statements have been provided.
Choose from the options, the
relationship between the two statements.
Statement 1: The launch of a new digital rupee is in the upcoming year.
Statement II: The launch will boost financial inclusion in Asia’s No.3 economy.
(a) If ‘I’ is the effect and ‘II’ is its immediate and principal cause.
(b) If ‘I’ is the immediate and principal cause and ‘II’ is its effect.
(c) If ‘I’ is an effect but ‘II’ is not its immediate and principal cause.
(d) If ‘II’ is an effect but ‘I’ is not its immediate and principal cause.
110. In paragraph four, what can be inferred from Bhagaban Behera’s saying?
(a) The saying is advisory. (b) The saying is motivating.
(c) The saying is pessimistic. (d) The saying is critical.
Passage(Q.111-Q.115): The Body Farm, known officially as the University of
Tennessee Anthropological
Research Facility, is a gruesome place. It is a hectare of land near Knoxville, cut
off from the rest of the world
by razor wire, that has, for more than three decades, been at the forefront of
forensic science. It is both a
laboratory which examines how corpses decay in different circumstances, so that
matters such as time of death
can be established more accurately, and a training facility for those whose jobs
require an understanding of such
processes.
To study a body forensically, though, you first have to find it. For a corpse
dumped in a city this is hard enough.
If the burial site is a forest it can be near impossible. Searchers must cover huge
amounts of ground, and may
therefore not do so as thoroughly as might be desirable. Vegetation broken by
people burying bodies is easy to
overlook. And soil perturbed by digging tends not to remain perturbed for long once
it has been exposed to wind
and rain.
For homicide detectives, then, woodlands are a problem. At least, they have been
until now. For Neal Stewart,
co-director of the Tennessee Plant Research Centre, another part of the university,
reckons that a bit of botanical
thinking brought to bear on the matter may turn trees from being cover for the
disposal of bodies to signposts
showing just where they are hidden.
The most obvious is fertilization-for bodies are good fertilisers. Calculations
suggest that a decaying adult human
body releases about 2.6kg of nitrogenous compounds (mostly ammonia) into the
surrounding soil. Such an
overdose would surely have consequences for nearby plant life. In particular, it
would increase chlorophyll
production, and thus cause a perceptible greening of plants near a buried body. In
principle, this would be true
of the decay of the body of any large animal. But the remains of wild creatures,
left on the surface, are usually
scavenged quickly. People with a human body to dispose of, generally prefer to
inter it so that it cannot be seen.
A more subtle change in the foliage near a buried body would be brought about by
any cadmium present within
its flesh and bones. Cadmium is rare in nature, but not in some human bodies.
Smokers, and also those who work
in industries involving welding or electroplating, have high concentrations of this
metal. Cadmium is easily taken
in by plants through their roots and, once present in their leaves, affects the
structure of a molecular complex
called photosystem two, which houses chlorophyll. That changes the way this complex
absorbs and reflects light.
This, in turn, affects the colour of the leaves.
. Page 29 of 36
111. Which of the these presents a contrast to the following sentence as mentioned
in paragraph three:
“a bit of botanical thinking brought to bear on the matter may turn trees from
being cover for the disposal of
bodies to signposts showing just where they are hidden.”
(a) Botany provides insight on how to notice unusual patches of plants.
(b) The difference in trees near the disposed bodies and those that are not can
only be spotted by botanists.
(c) The soil around the area where the bodies have been buried as a disposal have a
rare sighting of rodents
scavenging the soil.
(d) Trees are preferred sites of disposed bodies as they provide natural
camouflage.
112. Which of the following is not an assumption that supports the arguments
presented in the second paragraph?
(a) Forensic science doesn't include study of the location of the disposed bodies.
(b) Forests cannot be easily mapped.
(c) Cities do not have many areas of perturbed soil.
(d) Presence of human intervention is hard to detect in a natural environment.
113. What processes does the author refer to when he/she says, “an understanding of
such processes” in paragraph
one?
(a) The process of corpse decaying under different circumstances.
(b) The process of studying forensic science.
(c) The process of isolating a facility.
(d) The process of fortifying laboratories.
114. According to the passage, why do people with a human body to dispose of,
prefer to inter it so that it cannot be
seen?
(a) They do not want it to be scavenged by wild animals.
(b) They care about the environment.
(c) They do not want it to be found by anyone.
(d) It is illegal to dispose of human bodies.
115. According to the passage, which of the following statements would the author
most agree with?
(a) Human bodies should be offered as an alternative to chemical fertilizers.
(b) Every discoloured plant should be dug up to find buried bodies from underneath.
(c) Bodies found devoid of cadmium cannot be declared as murders.
(d) None of the above
Passage(Q.116-Q.120): As environmental regulation is far from a Formula 1 race,
speed should not be the key
criteria for assessing it. This is why the Union environment ministry’s plan to
rate State Environment Impact
Assessment Authorities (SEIAAs) by faster green clearances raises questions. The
rating is being undertaken
within the ease of doing business (EoDB) framework. While balancing economic and
environmental interests is
complex and necessary work for governments to do, the specific mandate of SEIAAs is
to safeguard the
environment. The ranking that really matters here is how India’s air, water and
land quality compare to other
countries today. Of course it is not in public interest to have the clearance
process mired in red tape either. In
this regard NDA has done well to rectify the UPA-era logjam. Prescribed timeline is
down to 75 days, from the
previous 105 days. But we cannot keep speeding down this path illimitably. Plus, it
is not as if states’ EoDB
rankings are sole determinants of actual investment flows. Tamil Nadu ranks lower
than both UP and Bengal.
But the state attracts some very big projects. Ola’s e-scooter plant in Krishnagiri
being a recent example.
Consider that SEIAA members will get the top score if they undertake site visits in
less than 10% of cases, but
the bottom score if visits are undertaken in over 20% cases. This is a strange
incentive given that these state
authorities are supposed to be staffed by experts in local ecology precisely so
they can rigorously analyze the

. Page 30 of 36
impact of a particular project. [1] Even though the pandemic has forced grant of
clearances through virtual
meetings, this is not a desirable policy when it comes to the future of our
ecologically sensitive areas. From
Uttarakhand to Kerala, SEIAAs need to be independent of both business and
governments. They should put the
environment first, and last.
116. The argument of the author depends on which of the following assumptions?
I. Excessive interference of bureaucracy increases the time taken to clear
environmental projects.
II. Not all discussions that can happen via in person mode can happen through
virtual meets.
(a) Only I. (b) Only II. (c) Both I & II. (d) Neither I nor II.
117. The author’s arguments logically support which of the following courses of
action?
(a) The prescribed timeline to clear environment projects is reduced to 50 days.
(b) More and more virtual meets are organized to clear environmental projects.
(c) The Union government does away with rating SEIAAs of every state.
(d) None of the above courses of actions are logically supported by the passage.
118. Which of the following would lend strength to the author’s arguments?
(a) States are likely to hurry the passing of environmental projects to get better
ratings.
(b) Speed ratings would increase the sense of competition and long due projects
would be cleared on priority.
(c) India ranks low globally in the various environmental ratings such as that of
air, land and water.
(d) The public has the right to information about various environmental projects
that are being cleared in their
states.
119. What can be said to be true about statement [1]?
(a) This statement strengthens the claim made by the author in the previous
sentence.
(b) This statement is by itself a complete argument, containing a claim and a
premise.
(c) This statement presents counter arguments to a claim made by the author in the
previous statement.
(d) This statement contains a claim made by the author, though no support has been
provided.
120. What can be concluded from the information given in the passage?
i. The prescribed timeline of clearing environmental projects has been reduced to
approximately 30% of the
previous timeline.
ii. The previous timeline for clearing environmental projects is close to 40% more
than what it is today at
present.
iii. UP and Bengal are better in clearing environmental projects than Tamil Nadu.
(a) Only i and ii (b) Only i and iii (c) Only ii (d) Only iii
Passage(Q.121-Q.125): The Commonwealth Games is just five months away, the Asian
Games is seven months
and the Indian Olympic Association is in shambles. The two along with the Olympics
are the three major events
to which the IOA sends the names of athletes—not just to the organising nations but
also to the sports ministry
for approval. Besides players and coaches, the IOA also selects officials who would
visit the Games under
different capacities. It facilitates accreditation for visiting media too.
With the fight between IOA president Narinder Batra and secretary Rajeev Mehta
raging, there seems to be
uncertainty at an inopportune moment. The athletes and coaches of course would be
selected by the respective
federations but for other officials and the media, there is no clarity. Especially
after the latest fight when Batra
invalidated the official email ID and IOA website, last week on January 8, which
Mehta claimed was illegal. At
another level, this sparring over not sharing the email password seems at best
juvenile.
What is more perturbing is Mehta’s statement that there is no executive council
because the tenure got over on
November 14 when it finished four years in office and as per the IOA constitution,
it has ceased to exist. This

. Page 31 of 36
too would complicate matters. The IOA has not called for an annual general meeting
or a special general meeting
to extend the tenure of the executive council as of now.
The matter pertaining to the election and revised constitution to align it with the
Sports Code is still pending in
the Delhi High Court. With the tussle between the president and secretary becoming
bitter, things are expected
to get worse. There is no sign of truce after a meeting before the AGM on November
19. With big-ticket events
just a few months away, the IOA should get its house in order. At the same time,
one hopes the court decides on
the matter soon. Otherwise, the chaos has the potential to rip apart the current
dispensation in the IOA,
irrespective of whose side one is on. As of now it seems the situation is getting
out of hand.
121. Out of the following, what has been suggested by the author in the passage?
Choose the best option.
(a) The crisis at the IOA should be solved diplomatically.
(b) The timing of the tussle in the IOA does not help its course.
(c) The IOA would soon organise a special meeting to sort things out.
(d) Athletes would not be able to take part in the upcoming competition.
122. If the information in the given passage is true, then which of the following
can be concluded?
(a) The Olympic Games would take place in the month of July.
(b) The Commonwealth Games would take place in the month of May.
(c) The Asian Games are scheduled to take place in the month of February.
(d) None of the above can be concluded from the information given in the passage.
123. The structure of paragraph four in the context of the given passage is best
depicted by:
(a) The author has presented solutions to the ongoing problems.
(b) The author gives reasons for why the crisis is not going to improve.
(c) The author explains that the problem may not be easily solved.
(d) The author portrays the body concerned in a bad light.
124. Out of the given statements, which of the following statement is a factual
statement?
Statement I: The Commonwealth Games is just five months away; the Asian Games is
seven months and the
Indian Olympic Association is in shambles.
Statement II: The IOA has not called for an annual general meeting or a special
general meeting to extend the
tenure of the executive council as of now.
Statement III: With the fight between IOA president Narinder Batra and secretary
Rajeev Mehta raging, there
seems to be uncertainty at an inopportune moment.
Statement IV: The matter pertaining to the election and revised constitution to
align it with the Sports Code is
still pending in the Delhi High Court.
Statement V: At another level, this sparring over not sharing the email password
seems at best juvenile.
(a) II & IV (b) I, II & IV (c) III & V (d) I & IV
125. The arguments of the author that the tussle within the IOA has come at an
inopportune moment is weakened by
which of the following pieces of evidence?
(a) Narinder Batra and Rajiv Mehta are childhood friends, they will sort out their
problems soon, without any
external intervention.
(b) A novel disease has emerged and is spreading exponentially, a complete lockdown
would be required to stop
the spread for over a year.
(c) Narinder Batra and Rajiv Mehta were at loggerheads in preceding events, leading
to the two teams not
qualifying in the international sports events.
(d) The tussle in the IOA does not affect the teams qualifying for the major
sporting events as the selections of
the players to overall management was finalized a year ago.

. Page 32 of 36
Passage(Q.126-Q.130): India may be on course to introduce e-passports, with
embedded microchips and
futuristic technology, to improve security and make immigration clearance a breeze
at border posts worldwide,
but some basic flaws at the ground level need a quick resolution. The detection of
as many as 200 passports
made on the basis of fake documents in the region in the recent past has resulted
in hectic activity to plug the
gaps, and hold the officials accountable for the lapses. The police chiefs of
Punjab, Haryana and Chandigarh
need to accord due importance and urgency to the Regional Passport Officer’s letter
that calls for strict adherence
to the standard operating procedures to be followed for the police verification of
each applicant.
An obvious recommendation is to visit the residence of the applicant as well as
enquire from neighbours, putting
an end to the casual approach of calling the applicant along with two witnesses to
the police station for
completing the verification process. The use of the Crime and Criminal Tracking
Network and Systems (CCTNS)
has also been suggested to check the crime record of the applicant instead of
checking it manually only at one
police station. Haryana has already taken the lead by suspending police officials
on charges of negligence in
clearing applications, and ordering re-verification of passports issued in some
districts last year.
A clear message of a crackdown has to go out to tighten the noose around
fraudsters, who prey on the gullible
and offer a variety of illegal options. The hi-tech paraphernalia found by the
Special Cell of Delhi Police as it
busted a fake visa racket earlier this week shows that even in this age of
biometrics and advanced security
features, there is no dearth of those willing to trap and those eager to take the
bait, no matter the risk involved.
Fast-track action could be a deterrent.
126. Which among the following is one of the opinions of the author regarding
passports in India?
(a) E-passports perfectly fit in as far as India is concerned.
(b) The authorities should up the ante as far as verification is concerned.
(c) E-passports need to be scrapped due to so many fraudulent activities.
(d) The burden of proof of the address should be of the applicant.
127. The argument of the author depends on which of the following assumptions?
(a) The applicants cannot convince any of their neighbours to give modified
statements to the authorities.
(b) All the passports that have been fraudulently issued are due to the mistakes of
the officials.
(c) There is a huge possibility that the fraudsters responsible for making forged
passports will not be punished.
(d) E-passport is a bullet-proof approach to tackling the ground level need.
128. Which among the following has been used as a premise to support the author’s
argument?
(a) E-passports would make immigration clearance a smoother process.
(b) Enabling e-passports is a significant step towards becoming a developed nation.
(c) The present police verification process is not very safe and secure.
(d) Fraudsters will find a way to navigate around the e-passports.
129. Out of the following, which option, if true, lends support to the author’s
arguments?
I. Fast track action gives the impression that all fraudsters would be nabbed and
punished.
II. In a credible survey conducted last year, 75% of the population said that they
would not apply for passports
without due diligence.
III. Fraudsters have a clearcut workaround for every security feature that the
authorities have or can add in future.
(a) Only I (b) Only I and II (c) Only I and III (d) Only II and III
130. With which of the following options would the author of the passage not agree
with?
(a) Many e-passports have been issued through the use of fake documents.
(b) Use of CCTNS would make the verification process more efficient.
(c) Re-verification of passports is possible in today’s day and age.
(d) A strict adherence to the standard operating procedures to police verification
of each applicant.

. Page 33 of 36
Directions(Q.131-Q.133): These questions are based on the following information.
Five boys – P, Q, R, S and T – are of different heights and are sitting in a row,
in the decreasing order of their
heights from left to right. Two persons sit in between R and T. T is shorter than
at least two other boys. Q is
neither the tallest nor the shortest. S is sitting adjacent to both Q and T.
131. Who is the tallest person?
(a) P (b) R (c) S (d) Cannot be determined
132. Who is sitting exactly in the middle of the row?
(a) R (b) Q (c) S (d) Cannot be determined.
133. Which of the following pairs of persons always sit adjacent to each other?
(a) Q and S (b) S and T (c) R and Q (d) More than one of the above
134. If each letter of the English alphabet is assigned an even numerical value in
increasing order such that they are
multiples of 3, such as A = 6, B = 12, and so on, then what will be the code of
'NOUGAT'?
(a) 849612644690 (b) 7296124486126
(c) 8490126426120 (d) 486012602480
135. S, R and M are siblings of each other and only R among them is a female
member. Q is S's son and W is Q's
grandfather. N is the sister-in-law of S and has no children. How is N related to W
?
(a) Daughter-in-law (b) Sister
(c) Sister-in-law (d) Mother

mock 34

Directions (Q.1-Q.30): Read the following passage carefully and answer the
questions that follow.
Passage (Q.1-Q.5): The moment everyone remembers from ‘The Maltese Falcon” comes
near the end. when
Brigid O’Shaughnessy (Mary Astor) has been collared for murdering Sam Spade
(Bogart)’s partner. She says
she loves Spade. She asks if Spade loves her. She pleads for him to spare her from
the law. And he replies, in a
speech some people can quote by heart, “I hope they don’t hang you. precious, by
that sweet neck. The chances
are you’ll get off with life. That means if you’re a good girl, you’ll be out in 20
years. I’ll be waiting for you. If
they hang you. I’ll always remember you.”
Cold - Spade is cold and hard. like his name. When he gets the news that his
partner has been murdered. he
doesn’t blink an eye. He didn’t like the guy. Beats up Joel Cairo (Lorre) not just
because he has to, but because
he carries a perfumed handkerchief. and you know what that meant in a 1941 movie.
He turns the rough stuff on
and off Loses patience with Green Street; throws his cigar into the fire; smashes
his glass; barks out a threat.
slams the door and then grins to himself in the hallway, amused by his own act. How
do Bogart and Huston get
away with making such a dark guy the hero of a film? Because he does his job
according to the rules he lives by,
and because we sense (as we always would with Bogart after this role) that the
toughness conceals old wounds
and broken dreams.
John Huston had worked as a writer at Warner Bros. before convincing the studio to
let him direct. “The Maltese
Falcon” was his first choice. even though it had been filmed twice before by
Warners (in 1931 under the same
title and in 1936 as “Satan Met a Lady”). “They were such wretched pictures.”
Huston told his biographer.
Lawrence Grobel. He saw Hammett’s vision more clearly; saw that the story was not
about plot but about
character; saw that to soften Sam Spade would be deadly; fought the tendency for
the studio to pine for a happy
ending.
The plot is the last thing you think of about “The Maltese Falcon.” To describe the
plot in a linear and logical
fashion is almost impossible. It’s all style. It isn’t violence or chases, but the
way the actors look, move, speak
and embody their characters. Under the style is attitude: hard men. in a hard
season. in a society emerging from
Depression and heading for war. For an hourly fee, Sam Spade will negotiate this
terrain. Few Hollywood heroes
before 1941 kept such a distance from the conventional pieties of the plot.
1. Which of the following CANNOT be inferred from the passage?
(a) The plot of the movie, The Maltese Falcon, is not its main attraction.
(b) Bogart who plays Sam Spade was a cold and ruthless man.
(c) The Maltese Falcon broke some conventional mode of filmmaking.
(d) People find some vulnerability in Sam Spade’s hardened demeanour.
2. The author of the passage will most likely agree with?
(a) It is impossible to portray a movie in linear and logical fashion.
(b) The Warner Bros. studio knew that The Maltese Falcon was about the character,
not the story.
(c) Hammett wanted his readers to view Spade as a maverick character.
(d) The Maltese Falcon earned huge profits for its makers.
3. Which of the following best describes the tone of the passage?
(a) Argumentative (b) Analytical (c) Censuring (d) Lionizing
4. The Maltese Falcon was released in:
(a) Before 1936 (b) 1936 (c) 1941 (d) After 1941

. Page 3 of 36
5. ‘Hollywood heroes before 1941 kept such a distance from the conventional pieties
of the plot.’ Which of the
following reflects a similar meaning to the word ‘pieties’ in the context of the
passage?
(a) Hypocrisies (b) Wickedness (c) Dissidence (d) Virtues
Passage (Q.6-Q.10): My granddaughter Shivani came to India after almost five years.
Born and living in the
US, she was inquisitive about the culture, history and the changing scenario of
India, the country of origin of her
parents. I wanted her to like the country. My son and daughter-in-law shifted to
the US in the late 1980s. They
frequently visited India but the pandemic had restricted their trips. However,
Shivani could not come earlier as
she was in college, then in a new job, followed by the pandemic. Now with the Covid
situation improving, they
came to Chandigarh in December.
We went to Amritsar the next day. Shivani and her father, both photography
enthusiasts, went to the Golden
Temple, taking photographs of the rising sun and the temple. We joined them later.
It was a memorable
experience for her to be in the sanctum sanctorum. While walking around the
sarovar, we visited the ‘Guru ka
langar’ hall. She was amazed to see the volunteers preparing meals in huge
utensils, serving hundreds of people
at a time, collecting and cleaning the used thalis and glasses, keeping them ready
for the next round. People were
sitting together on the floor and eating wholesome vegetarian food, regardless of
religion, caste, gender or
ethnicity. She had never seen such voluntarism – kar seva. It made an indelible
impression on her young mind.
From the Golden Temple, we walked over to the Jallianwala Bagh, where she took her
time to see how history
was created when a peaceful crowd was made the target of shooting by the British-
led soldiers, killing and
injuring more than 1,500 hapless people. Here, she got a mixed feeling of pain,
anger and pride. From the epitome
of religion to the place of martyrdom, Shivani saw a slice of India’s glorious
history. In the afternoon, we drove
to the Attari-Wagah border between India and Pakistan to watch the Beating Retreat
ceremony. The atmosphere
was charged with nationalistic fervour and constant shouting of slogans for the
country. Just as the sun was about
to set, the bugles sounded on both sides and the flags of the two countries were
lowered for the day
simultaneously. This was a climax of heightened nationalistic sensitivity.
The two evenings, we went to traditional dhabas for dinner. We had to manoeuvre our
way in small lanes,
between rickshaws, motorcycles and pedestrians. But it was fun and we had simple,
delicious, vegetarian food
— hot crisp naans, paranthas, dal and a vegetable. Shivani could not believe that
the whole meal cost just $3 per
person. The following day, we returned to Chandigarh. My elder son and daughter-in-
law got together for a
family reunion. Before leaving for the US, she mentioned that she had loved every
moment and experience of
the trip and said firmly, ‘I have to come regularly every year to see more of
India, our great nation.’
The magic of India had worked!
6. In context of the passage, which of the following is NOT TRUE?
(a) Shivani came to India after almost five years.
(b) While walking around the altar, the Shivani and her family visited the ‘Guru ka
langar’ hall.
(c) Shivani could not believe that the whole meal cost just $3 per person.
(d) The two evenings, Shivani and her family went to traditional dhabas for dinner.
7. All of the following can be inferred from the passage, except
(a) Shivani was enamoured with the Indian culture.
(b) Jallianwala Bagh evoked inconsistent feelings in Shivani.
(c) Beating Retreat ceremony is the highpoint of Attari-Wagah border.
(d) Shivani carried with evanescent memories of the selfless services of the
voluntaries.
8. Which of the following best reflects the writing style adopted by the author?
(a) Expository. (b) Narrative. (c) Polemical. (d) Persuasive.

. Page 4 of 36
9. ‘The atmosphere was charged with nationalistic fervour and constant shouting of
slogans for the country.’
Which of the following is the correct part of speech of the highlighted word?
(a) Adjective. (b) Adverb. (c) Verb. (d) Noun.
10. What is the author trying to communicate through the sentence, ‘The magic of
India had worked! With respect
to the daughter’?
(a) Nobody can remain immune to the charms of India.
(b) India can charm tourists with its history.
(c) The purpose was accomplished.
(d) The mission of the author was successful.
Passage (Q.11-Q.15): The climate crisis is set to profoundly alter the world around
us. Humans will not be the
only species to suffer from a calamity. Huge waves of die-offs will be triggered
across the animal kingdom as
coral reefs turn ghostly white and tropical rainforests collapse. For a period,
some researchers suspected that
insects may be less affected, or at least more adaptable, than mammals, birds and
other groups of creatures. With
their large, elastic populations and their defiance of previous mass extinction
events, surely insects will do better
than most in the teeth of the climate emergency?
Sadly not. At 3.2C of warming, which many scientists still fear the world will get
close to by the end of this
century, half of all insect species will lose more than half of their current
habitable range. This is about double
the proportion of vertebrates and higher even than for plants, which lack wings or
legs to quickly relocate
themselves. This huge contraction in liveable space is being heaped onto the
existing woes faced by insects from
habitat loss and pesticide use. “The insects that are still hanging in there are
going to get hit by climate change
as well,” says Rachel Warren, a biologist at the University of East Anglia, who in
2018 published research into
what combinations of temperature, rainfall and other climatic conditions each
species can tolerate.
Some insects, such as dragonflies, are nimble enough to cope with the creeping
change. Unfortunately, most are
not. Butterflies and moths are also often quite mobile, but in different stages of
their life cycle they rely on certain
terrestrial conditions and particular plant foods, and so many are still
vulnerable. Pollinators such as bees and
flies can generally move only short distances, exacerbating an emerging food
security crisis where farmers will
struggle to grow certain foods not just due to a lack of pollination but because,
beyond an increase of 3C or so,
vast swaths of land simply become unsuitable for many crops. The area available to
grow abundant coffee and
chocolate, for example, is expected to shrivel as tropical regions surge to
temperatures unseen in human history.
The climate crisis interlocks with so many other maladies – poverty, racism, social
unrest, inequality, the
crushing of wildlife – that it can be easy to overlook how it has viciously
ensnared insects. The problem also
feels more intractable. “Climate change is tricky because it’s hard to combat,”
says Matt Forister, a professor of
biology at the University of Nevada. Use of chemical is responsible for decrease in
number of insects overtime
“Pesticides are relatively straightforward by comparison but climate change can
alter the water table, affect the
predators, affect the plants. It’s multifaceted.” Insects are under fire from the
poles to the tropics. The Arctic
bumblebee, Bombus Polaris, is found in the northern extremities of Alaska, Canada,
Scandinavia and Russia. It
can survive near-freezing temperatures due to dense hair that traps heat and its
ability to use conical flowers, like
the Arctic poppy, to magnify the sun’s rays to warm itself up.
11. Which of the accompanying is NOT true, keeping in view the subject of the
passage?
(a) Butterflies and Moths are more prone to extinction than dragonflies.
(b) While tropical countries experience temperatures unprecedented in human
history, the space accessible to
cultivate bountiful coffee and chocolate is anticipated to increase.
(c) Climate change is difficult to tackle with the woes increasing further in
future.
(d) In terms of being more dangerous, climate change gets an upper hand over the
use of pesticides.

. Page 5 of 36
12. What is the tone of the author in the passage?
(a) Acerbic. (b) Apathetic. (c) Condescending. (d) Disconcerting.
13. Give a suitable title for the passage.
(a) The catastrophic effect of the changing climate.
(b) How the speed of climate change is unbalancing the insect world?
(c) How climate change is proving to be lucrative to the insect world?
(d) The intensity of inimical insects.
14. According to the passage, which one of the following is NOT one of the effects
that shall lead to the insect’s
decrease in number, over time?
(a) Destruction of their habitat. (b) Use of chemicals to kill them.
(c) Pollutants in soil and water. (d) Drastic reduction in available living space.
15. Which one of the following assumptions by some researchers made them draw the
conclusion which highlighted
the fact that climate change would prove to be the least malicious for the insects.
(a) Their enormous, adaptable numbers and resistance to prior mass extinction
occurrence.
(b) Insect species diversity per area tends to decrease with higher latitude and
altitude.
(c) They move in hoards to different areas where the climate is more suitable for
their existence and proliferation.
(d) Some of them go into hibernation when they encounter inclement weather and
after it passes, they revert to
their original form and number.
Passage (Q.16-Q.21): In contrast to India's continued ambiguity over the legality
of cryptocurrencies, its stance
on introducing an official digital currency has been reassuringly clear and
consistent over time. And, four years
after an inter-ministerial committee recommended that India launch fiat money in
digital form, the Reserve Bank
of India has indicated that pilot projects to figure out its viability are likely
to be launched soon. In a speech a
few days ago, T. Rabi Sankar, Deputy Governor, RBI, said, 'RBI is currently working
towards a phased
implementation strategy and examining use cases which could be implemented with
little or no disruption.' The
clarity is welcome, given that the much-awaited Cryptocurrency and Regulation of
Official Digital Currency
Bill, 2021, is yet to be introduced. In recent years, the significant rise of
private cryptocurrencies such as Bitcoin
and Ether has spooked central banks throughout the world, and pushed the case for
official digital currencies.
Mr. Rabi Sankar himself cited a 2021 BIS survey of central banks, which found that
86% were actively
researching the potential for such currencies, 60% were experimenting with the
technology, and 14% were
deploying pilot projects. China, having already engaged in pilot projects for its
digital RMB, is in fact
planning a major roll-out soon. There has been little doubt, therefore, that India
needs a digital rupee. The
important questions are about the details and the timeline.
There are crucial decisions to be made about the design of the currency with
regards to how it will be issued, the
degree of anonymity it will have, the kind of technology that is to be used, and so
on. It is possible that the
question of the degree of anonymity, especially, will be quite a challenging one.
While official digital currencies
can borrow the underlying technology feature of private cryptocurrencies, they
significantly differ from the latter
in their philosophy and goals. Also, to be considered are possible impacts of the
introduction of an official digital
currency on people, the monetary policy, and the banking system. There are risks to
be considered as well, not
the least of which will be those emerging from cyberattacks. What is more, many
laws need to be amended to
make the digital rupee a reality. So, while India might have done exceedingly well
in digital payments in recent
years - the Deputy Governor said they have grown at a compounded annual growth rate
of 55% over the last five
years - the digital rupee will be something else altogether. Notwithstanding all
these challenges, it would seem
that the answer to Mr. Rabi Sankar's speech title, 'Central Bank Digital Currency -
Is This the Future of Money',
is a yes.

. Page 6 of 36
16. What was RBI's intention behind launching pilot projects?
(a) To check the viability of currency in digital form.
(b) To gain the trust of customers.
(c) To check the security risks associated with it.
(d) To check the response of potential customers.
17. Which of the following bills has to be introduced to give clarity on the
perspective of digital currency?
(a) Cryptocurrency and Regulation of Official Digital Capitalisation Bill, 2021.
(b) Cryptocurrency and Regulation of Official Digital Currency Bill, 2022.
(c) Cryptocurrency and Regulation of Official Digital Currency Bill, 2021.
(d) Currency and Regulation of Official Digital Currency Bill, 2021.
18. Which of the following was found in the BIS survey of Central banks?
I. 86% of Central Banks were actively researching for the potential of digital
currency.
II. 14% of Central Banks were deploying the pilot projects for digital currency.
III. 60% of Central Banks have already issued the digital currency.
(a) I and II (b) I and III (c) II and III (d) Only I
19. Which of the following words can replace the given word- spooked without
changing the meaning of the
sentence.
(a) Soothed (b) Startled (c) Frightened (d) Apprehensive
20. Which of the following statements is TRUE about the passage?
I. Digital currency are being used in all the developed countries.
II. The digital payments have grown at a compounded annual growth rate of 55% in
the last 5 years.
III. Crucial decisions are required around the degree of anonymity of digital
currency.
(a) Only I (b) Only II (c) Only III (d) II and III
21. What is the strategy of RBI to implement the digital currency?
(a) The strategy is to bring all banks together and work towards the security
issues of digital currency
(b) It wants the implementation to be held in a phased manner after examining the
use cases
(c) It wants participation of individuals in suggesting different measures to
assist its implementation
(d) All the possible challenges should be marked clear before implementation
Passage (Q.22-Q.25): A rapid, technology-driven transformation is taking place
within the global economy.
India and China are playing decisive roles in this transformation as emerging
advanced technology superstates.
China is five to ten years ahead of India in most respects, but India is ahead in a
few areas and is closing the gap
in others. Chinese exports are far larger for manufactures, while India has the
lead for business services. There
is growing competition for foreign direct investment featuring large R&D
components. Indian multinational
companies, especially when including outward foreign direct investment, are ahead
of Chinese companies not
only for business services, but in the pharmaceutical, automotive and steel sectors
as well. The likely courses
over the coming two to five years is that it is highly likely that India will
continue its 8% to 10% annual growth,
while China is equally likely to experience a "hard landing" adjustment from
excessively export-oriented to more
domestically-oriented growth, including slower growth for at least a couple of
years.
Both nations face obstacles to continued high growth, but India is on a more
balanced growth path, with growing
momentum from private sector investment, including for urgently needed
infrastructure. China has had higher,
10% to 11% growth, but half or more of the growth has been accounted for by
increased exports of manufactures
and related investment. External and internal pressures to shift toward more
domestically-oriented growth,
including through a major revaluation of the yuan, are building, but the
restructuring faces formidable obstacles.

. Page 7 of 36
The US needs a forceful and comprehensive policy response to this rapid rise of
China and India to advanced
technology superstate status. In the short to medium term, economic policy issues
will dominate, while over the
longer-term, foreign policy and national security issues will become more
important.
22. Which of the followings can be inferred from the lines in the passage, ‘while
China is equally likely to experience
a "hard landing" adjustment from excessively export-oriented to more domestically-
oriented growth, including
slower growth for at least a couple of years.’?
(a) China will become a more advanced country than any other country in the world,
in near future.
(b) China is embracing an undesirably rapid decline in economic growth in the
coming years.
(c) China is turning to become a more domestically driven economy rather than
export-oriented economy
(d) China will import more than it exports in the coming years.
23. Which of the followings is/are not among the sectors wherein India is ahead of
China?
(i) Steel sector.
(ii) Automotive.
(iii)Pharmaceutical.
(iv)Services sectors.
(v) Manufacturing export products.
(a) All, except (v) (b) Only (iv) and (v)
(c) All, except (i), (ii) and (iv) (d) Only (v)
24. Why India is on a balanced growth path compared to China?
(i) India’s impetus is from the growing private sector investment, and a focus on
its investments in
infrastructure; whereas, China's growth largely depends on exports of manufactures.
(ii) India's annual growth rate will continue at 8% to 10%; whereas, China's growth
rate 10% to 11% likely to
decline.
(iii)A major revaluation of the yuan will lead to downfall of China's economic
growth, but Indian currency value
will remain the same.
(a) Only (i) (b) Only (iii) (c) Both (i) and (iii) (d) All (i), (ii), and (iii)
25. Which of the following sentence(s) has/have "respects" used in the same context
as used in the passage?
(i) Throughout life, we rely on small groups of people for love, admiration,
respect, moral support, and help.
(ii) Religious respects and admiration for the priest led many of the parishioners
to ignore the bad things he did.
(iii)In many respects, the conditions proposed by the Bush Administration are
similar to those proposed by the
Carter Administration.
(a) Only (i) (b) Only (iii)
(c) Both (i) and (iii) (d) All (i), (ii), and (iii)
Passage (Q.26-Q.30): Read the following passage carefully and answer the questions
given after the passage.
Certain words/phrases have been printed in bold to help you locate them while
answering some of the questions.
In India, the University Grants Commission, UGC, has been the regulatory body
responsible for maintaining
standards in higher education, while addressing challenges of globalisation.
Processes of UGC mandated
“standardisation” have in particular impacted social sciences and humanities
research in Indian universities. Over
the years, UGC has linked institutional funding to ranking and accreditation
systems like NAAC and NIRF. In
order to evaluate institutions, these bodies have evolved “objective” criteria,
which rank universities based on
faculty research measured by citations in global journal databases like SCOPUS.
Even for faculty promotions
and eligibility for research supervision, it is articles published in such
databases that are considered valid. In
comparison, importance granted to research outputs like books or other forms is
declining.
The insistence of publication in journals fails to distinguish between the varied
trajectory of disciplines. It is
more of an imposition of STEM (science, technology, engineering and management)
criteria on social sciences

. Page 8 of 36
and humanities. While in STEM disciplines, research is often highly objective and
quantified, and conclusions
can be published more easily as reports, short studies or articles, in social
sciences and humanities research is
subjective, analytical and argumentative. Within social sciences, too, there are
some disciplines like economics
where research methods can possibly be more empirical and quantitative. On the
other hand, in disciplines like
history, sociology, politics, philosophy, psychology and literature, researchers
spend years writing books that
engage with ideas in complex ways. The same is always not possible in a series of
articles published over years.
In devaluing books as authentic forms of research, UGC does major disservice to
doyens of social sciences and
humanities whose globally acclaimed books have contributed significantly to
knowledge building.
Tyranny of “peer review” is another phenomenon that haunts scholars. Since
continuous production and
publication has become necessary for professional growth, teachers spend most of
their productive time writing
articles and getting them published, thereby missing out on quality engagement with
pedagogy and research.
This has led to a surfeit of articles on any possible subject competing with each
other for citations. Moreover,
with long review processes associated with journal articles, it is not really as
efficient a measure of research
output as is claimed.
While professional growth in universities bases itself on quantified indices, the
process of peer review itself is
subjective, and depends upon the knowledge, inclination and availability of time of
the particular reviewer. It is
not unheard of getting two opposite reviews for the same article, sometimes even
expecting an author to revise
the entire argument that she may be making. Researchers in interdisciplinary areas
face even greater challenges
as their reviewers might come from conventional fields.
26. Which of the following reflects the central idea of the passage?
(a) An imposition of STEM criteria by UGC on social sciences and humanities is
unjustified, as in STEM
disciplines, research is often highly objective and quantified, and conclusions can
be published more easily
as reports, short studies or articles, in social sciences and humanities research
is subjective, analytical and
argumentative.
(b) Precise point-based measurements currently applied to measure knowledge
production appear to be
misplaced in knowledge ecosystems of the global south.
(c) Parameters measuring industrial productivity, help in assessing the relevance
of knowledge created and
disseminated in any societal context, including those with an industrial culture
like in Europe and America,
remains unanswered.
(d) Control and governance of knowledge in academia are often not compatible with
the educational systems,
especially the UGC mandated “standardisation” in social sciences and humanities
research in Indian
universities.
27. What can be inferred from the phrase ‘peer review’?
(a) Peer review is an evaluation of scientific, academic or professional work by
others working in the same field.
(b) Peer review is a biased evaluation scientific, academic or professional work by
others working in the same
field.
(c) Peer review is what the researchers spend most of their productive time; in
writing articles and getting them
published, thereby missing out on quality engagement with pedagogy and research.
(d) Peer review is an evaluation of the articles, journals, academic researches by
the students under a professor
from the same field.
28. Why does the author use the word ‘tyranny’ with respect to peer review?
(a) Because of it being a criterion for appraisal of an academician’s progression.
(b) Because it compels the professors to deviate from their purpose.
(c) Because of the inherent feature of the word, peer review.
(d) Because of the despotic approach by the UGC.

. Page 9 of 36
29. Which of the following is the best title of the passage?
(a) Indian higher education requires reforms.
(b) Indian higher education is down in the dumps.
(c) UGC: a failed regulatory body.
(d) Peer review.
30. Based on the contents of the passage, the author is
(a) A journalist (b) A reporter
(c) An academician (d) An education minister

Directions (Q.66 – Q.105): Read the comprehensions carefully and answer the
questions based on it.
Passage(Q.66-Q.70): The Supreme Court on Thursday dismissed the appeal filed by
Loop Telecom and Trading
Limited assailing the decision of TDSAT of dismissing their pleas seeking refund of
Entry Fee of Rs 1454.94
crores paid for grant of Unified Access Service Licenses.
The bench of Justices DY Chandrachud, Surya Kant and Vikram Nath observed that Loop
Telecom was in pari
delicto with the Department of Telecom and the then officials of the Union
government and that they were
beneficiary of the ―First Come First Serve policy which was intended to favour a
group of private bidding
entities at the cost of the public exchequer.
On 2 February 2012, the Supreme Court Court by its judgment in Centre for Public
Interest Litigation v. Union
of India declared that the policy of the Union government for allocation of 2G
spectrum on a ―First Come First
Serve basis was illegal. The judgement resulted in quashing of the UASL's granted
by the Union.
Referring to Section 65 of Indian Contract Act, 1872 court observed that in
adjudicating a claim of restitution,
the court must determine the illegality which caused the contract to become void
and the role the party claiming
restitution has played in it. If the party claiming restitution was equally or more
responsible for the illegality (in
comparison to the defendant), there shall be no cause for restitution. (Para 52)
Indian Contract Act, 1872 - Section 56 - Doctrine of Frustration discussed - The
applicability of Section 56 of
the Indian Contract Act is not limited to cases of physical impossibility. The
doctrine of frustration states that
frustration occurs when an unforeseen event renders performance of a contract
impossible or radically different
from that originally contemplated by the parties. No party is considered at fault.
If a contract is found to be frustrated, it is automatically terminated. All future
obligations of the parties to the
contract are discharged.
(Source: Case: Loop Telecom and Trading Limited vs Union of India | CA 1447-1467 of
2016 | 3 March 2022
Citation: 2022 LiveLaw (SC) 238)
66. The passage in Para 2 gives reference to an observation made by the court in
the present case. It was observed
that Loop Telecom was in pari delicto with the Department of Telecom. What is meant
by the expression “in
pari delicto”?
(a) A contract by which one party promises the other party to save from loss which
may be caused either by the
conduct of the promissor or by the conduct of any other person.
(b) Both the parties being equally or more responsible for making the contract
frustrated.
(c) Loop telecom was equally responsible for the illegality.
(d) Both parties were equally responsible for the illegalities.
67. Shyamlal used to deliver freshly baked bread throughout the province of
Hogwarts. He used to pay octroi, a tax
levied by the municipality to enter their territory, at the entry. Shyamlal used to
pay the entry tax every time he
entered the province for which receipt is given, until he discovered that
municipalities were illegally charging
shyamlal. Octroi is collected only when a person enters the province with goods.
Shyamlal files a claim for
monetary reparation. Decide
(a) Shyamlal will succeed as he was illegally charged extra tax.
(b) Shyamlal will succeed as in the present case the party claiming restitution was
not responsible for the
illegality.
(c) Shyamlal will not succeed as he was in pari dalicto with the municipality.
(d) Shyamlal will not succeed as he has no contract with the municipality.

. Page 16 of 36
68. Continuing with the same scenario as above, what if Shyamlal used to bribe the
municipality for not examining
his items on specific occasions under the guise of tax? Can Shyamlal later allege
that he was wrongfully charged
an additional tax every time he enters the province?
(a) No, he cannot claim so as he was in pari dalicto with the municipality.
(b) Yes, he can because he was charged an additional tax for entering the province.
(c) No, he cannot claim that he was charged extra because he bribed the authorities
under the pretence of tax.
(d) No, he cannot be charged extra because it is against the law.
69. CodeALFA had agreed to construct tunnels for the rail authority on the premise
that works be done 24 hours a
day, seven days a week in order to complete the work on schedule. Residents' noise
complaints resulted in an
injunction from court that limited labour hours. The rail authority refused to pay
the increased fees, claiming that
CodeALFA had not followed the terms of the contract. Decide
(a) The contract stands frustrated as the contract became impossible to perform.
(b) The contract do not stands frustrated as it is not limited to the cases of
physical impossibility.
(c) The contract stands frustrated because of a foreseen event.
(d) The contract does not stand frustrated merely because one party is not able to
perform the contract.
70. Simran used trivago app to purchase an "All in one travel and seating package"
in order to attend a BTS concert
organized by trivago. This allows her to a one-day return train ticket from Bhopal
to Delhi, as well as a front#row seat to the concert. Unfortunately, the event was
cancelled due to the rise of a warlike situation between
India and China. Simran claims that her contract with trivago was frustrated. Which
of the following assertions
is more likely to be adopted by the court in determining the legal position between
Simran and Trivago?
(a) The contract has been frustrated because the common purpose of both parties has
been destroyed.
(b) The contract has been frustrated on grounds of common mistake.
(c) Warlike situation terminates the contract and no party is considered at fault.
(d) The contract is not frustrated as Simran can still travel to Delhi.
Passage(Q.71-Q.76): The Supreme Court observed that interest on compensation under
the Workmen's
Compensation Act 1923, shall be paid from the date of the accident and not the date
of adjudication of the claim.
Ajaya Kumar Das was working as a labourer. Due to an accident, he suffered multiple
injuries in his abdomen
and kidney.
A claim for compensation was lodged before the Workmen compensation-cum Assistant
Labour Commissioner,
Odisha. Allowing his claim, the Commissioner directed that a compensation of Rs 2,
78,926 must be paid
together with interest at the rate of 12 per cent per annum on the principal sum
awarded from the date of accident
till the deposit. Though the High Court refused to interfere with the compensation
amount, it held that Das is not
entitled to any other interest on the compensation awarded except the accrued
interest.
In appeal, the bench comprising Justices DY Chandrachud and Dinesh Maheshwari noted
that Section 4A of
the Workmen's Compensation Act stipulates that the Commissioner shall direct the
employer to pay interest of
12% or at a higher rate, not exceeding the lending rates of any scheduled banks
specified, if the employer does
not pay the compensation within one month from the date it fell due shall be
recovered from the employer by
way of penalty.
“This Court emphasises that the applicant is entitled to interest from the date of
accident while rejecting the
submission that the award of interest should be after the expiry of 30 days from
the date of accident. Thus, there
was no legal basis for the High Court to delete the order of payment of interest.”
the bench observed.
Source: https://www.livelaw.in/top-stories/supreme-court-workmen-compensation-act-
interest-date-of#accident-190772

. Page 17 of 36
71. Suraj Industries was being sued by its waged workers for denying one of their
member’s fair compensation on
getting injured while performing his workerly duties. The court ventured into the
question of whether there was
a claim payable in the present case, as the employment status of the waged worker
was disputed by the company
as being not contractual, thereby, excluded from the ambit of the law above. How
shall the court rule?
(a) The company is correct as the worker is not a contractual employee.
(b) The company is not correct as the worker, contractual or not, deserves
compensation.
(c) The workers would be held as correct, since the term worker includes all sorts
of workers.
(d) Not discernible as not provided in the principle above.
72. In the above question, if the court discernibly decided that the worker was
supposed to receive compensation for
the injury caused to him, the date of compensation to be paid from was to be
adjudicated. How shall the court
rule?
(a) Compensation is payable from the date of the medical expenses incurred by the
injured party.
(b) Compensation is payable from the date of detection of injury by the company.
(c) Compensation is payable from the date of the incident and the injury taking
place.
(d) Not discernible as no law to the same effect has been provided above.
73. While granting interest to an aggrieved employee in a dairy produce factory,
the court had instructed the
employer, who had allowed other medical benefits of his employees, to also pay
11.75% interest to his employee
upon the adjudicated claim. Was the court right in doing so?
(a) Yes, as the passage specifies the compound interest limit as 12% to be payable
here.
(b) No, as the rate of interest provided above has been incorrectly mentioned.
(c) Yes, as the worker has been treated fairly, otherwise through benefits and
amenities.
(d) No, as the employer has been unfairly demanded an exorbitant rate to be
payable.
74. In the above case, had the claim been adjudicated the same day as on which the
injury occurred, and the interest
thereupon was offered by the employer as 11.5%, which was disbursed by him the same
day as well, would the
situation of interest herein be correctly conducted?
(a) Yes, as there was no delay and hence the interest was aptly paid.
(b) No, as the interest should have been paid at 12%.
(c) No, as the interest should have been paid upwards of 12%.
(d) Yes, as the claim was adjudicated and paid quickly, leading to the employee
receiving quick relief.
75. Barjatya Industries was being sued by its workers for having risked their lives
while making them work during
the pandemic, which also led to three workers asthmic for a period of 4 months, and
was now before the Supreme
Court. Therein the Supreme Court had ordered a compensation of 12 lakhs to be given
to each person, and 15
lakhs to the severely affected. Moreover, as the claim was due on March 1st, and
was not paid by April 1st, the
interest accrued thereupon had been held to be 12% as well. Can the court direct
the company to make such
payment?
(a) No, as the interest accrued thereupon has not become payable yet.
(b) Yes, as the court has ordered fair and just compensation in this case.
(c) No, as the compensation paid herein is unfair as it is too exorbitant for the
current scenario.
(d) Yes, as the SC has complete discretion in ordering the compensation that it has
deemed fair.
76. In the above case, the court had decided the amount to be compensated on March
1, 2022 and the payment was
scheduled to be made on 30 March 2022, however, the injury herein took place on
January 1, 2022 and the
amount was disputed by the company on March 3 2022. Compute the date from when the
interest should be
payable.
(a) The interest is payable from March 1, as the amount payable was not known
before that.
(b) The interest is payable from January 1, since the injury occurred then.
(c) The interest is payable from March 30, since the payment is to be made then.
(d) The interest is payable from March 3, as the amount was still under contention
then.

. Page 18 of 36
Passage(Q.77-Q.80): Merely granting protection for long time would not be a ground
to extend the benefit of
anticipatory bail to the accused, when the applicant is otherwise disentitled for
anticipatory bail", the Gujarat
High Court has held.
Justice Ilesh J. Vora was hearing an application under Section 438 of CrPC seeking
pre-arrest bail in connection
with an FIR for offences under Sections 307, 397, 452, 324, 323, 143, 147, 148, 504
and 506(2) of IPC.
The Applicant moved an anticipatory bail which was rejected by the Sessions Court
stating that the alleged
offence was prima facie serious and that custodial investigation was necessary.
Noting the Supreme Court's opinion in P Chidambaram vs Directorate of Enforcement
[2019 (9) SCC 24]
wherein it was observed, "Ordinarily, arrest is a part of procedure of
investigation to secure not only the presence
of the accused, but several other purposes. Power under Section 438 is an
extraordinary power and same has to
be exercised sparingly. The privilege of pre-arrest bail should be granted only in
exceptional cases. The judicial
discretion conferred upon the Court has to be properly exercised after application
of mind as to the nature and
gravity of accusation; possibility of fleeing from justice and other factors to
decide whether it is a fit case for
grant of anticipatory bail or not", the Bench refused to grant anticipatory bail to
the Applicant.
(SOURCE: Case Title: HIRENBHAI HITESHBHAI PATEL Versus STATE OF GUJARAT : Citation:
2022
LiveLaw (Guj) 49)
77. The Applicant has requested anticipatory bail. Applicant and seven others,
armed with a washbasin pipe and a
metal bolt, allegedly assaulted the victim Anand, causing severe head injuries and
stealing approximately INR
30,000 in cash. When the first informant attempted to intervene, the Applicant
threatened them with dire
consequences if they filed a FIR. The entire incident was recorded on CCTV. Decide
(a) The applicant will be granted anticipatory bail by the trial court because all
accused have the right to seek
pre-arrest bail under section 438 of the CrPC.
(b) The applicant will not be granted anticipatory bail because pre-arrest bail
should be granted only in
exceptional circumstances.
(c) At the discretion of the court, the applicant will be granted anticipatory
bail.
(d) Applicant will not be granted bail as the alleged offence was prima facie
serious.
78. Assume, in the same facts as above, that the applicant is a serial offender of
petty crimes and thus remains under
the constant surveillance of law and order. Can the applicant use this as a basis
for grant of anticipatory bail?
(a) Yes, because he is being surveilled, the applicant has no chance of evading
justice.
(b) No, as remaining under a constant surveillance would not be a ground to extend
the benefit of anticipatory
bail to the accused.
(c) Yes, it is plausible if the applicant is otherwise eligible for anticipatory
bail.
(d) Cannot be determined because the facts do not provide complete information.
79. Choose a statement do not illustrate an anomaly of the context of the passage:
I. The very purpose for the provisions relating to anticipatory bail is to ensure
that no person is confined in any
way until and unless held guilty.
II. Protection for long time would be a ground to extend the benefit of
anticipatory bail to the accused, when
the applicant is entitled for anticipatory bail.
III. Section 438 confers discretionary and sparing powers on the court to grant an
order of anticipatory bail.
IV. If the offence alleged is of serious nature, the personal liberty granted to
the accused in for of an anticipatory
bail will be cancelled.
(a) I & II (b) II & IV (c) I, II &IV (d) III

. Page 19 of 36
80. Madhu was forced to marry Anuj against her will. However, after the marriage,
she confronted Anuj and her in
laws, stating that if she was forced to do anything against her wishes, she would
file a FIR for domestic violence
and assault. Madhu arrived home late one day, which drew the attention of all the
residents of the society. They
began to talk negatively about Madhu. In retaliatory action, Madhu filed a false
domestic violence case against
her husband and in-laws, while feigning burns and bruises. Decide
(a) The husband and in-laws can apply for anticipatory bail.
(b) When in case of domestic violence a FIR is filed, a woman is presumed to be a
victim; therefore, in such
cases, arrest is part of the investigation procedure to secure not only the
presence of the accused, but also
several other purposes.
(c) Because Madhu filed a completely bogus FIR, the Court's judicial discretion
must be properly exercised after
considering the nature and gravity of the accusation.
(d) The grant of anticipatory bail will be based on whether or not the case
qualifies for the grant of anticipatory
bail.
Passage(Q.81-Q.85): The Supreme Court observed that the jurisdiction of a Court
under the Contempt of Courts
Act would not cease, merely because the order or decree of which contempt is
alleged, is executable.
The court added that, irrespective of whether or not a decree is executable, the
question to be considered in
determining whether a case for contempt has been made out is, whether, the conduct
of the contemnor is such as
would make a fit case for awarding punishment for contempt of Court.
In this regard, the court observed thus:
"Further, it is trite law that the jurisdiction of a Court under the Act, would not
cease, merely because the order
or decree of which contempt is alleged, is executable under law, even without
having recourse to contempt
proceedings. Contempt jurisdiction could be invoked in every case where the conduct
of a contemnor is such as
would interfere with the due course of justice; vide Rama Narang vs. Ramesh Narang
– [(2006) 11 SCC 114.
Contempt is a matter which is between the Court passing the order of which contempt
is alleged and the
contemnor thus provision of appeal will not be applicable in this regard ;
questions as to executability of such
order is a question which concerns the parties inter-se. The power of the Court to
invoke contempt jurisdiction,
is not, in any way, altered by the rights of the parties inter-se vide Bank of
Baroda vs. Sadruddin Hasan Daya –
[(2004) 1 SCC 360]."
The court observed that when a party which is required to comply with the terms or
directions in an order has
not done so within such time as stipulated in the order, two options are available
to the party which was required
to comply with such order:
(a) give an explanation to the Court as to the circumstances due to which the party
could not comply with the
order of the Court;
(b) seek for further time to comply with the order of the Court.
[Source: Live Law, “Contempt Jurisdiction Would Not Cease Merely Because Order is
Executable: Supreme
Court”]
81. Mr. Chaman is a renowned Businessman and owns a huge factory. A case before the
Labour Court was filed by
his workmen for deducting salary due to COVID Lockdown. The Labour Court passed
interim order directing
Mr. Chaman to return the deducted salary while the matter is pending for final
order. However, Mr. Chaman did
not pay the salary and rather filed an Appeal before the High Court. High Court
decided in favor of Mr. Chaman,
However, before High Court decided in his favour, the Labour Court held that Mr.
Chaman has committed
contempt by not complying with its order. Decide.
(a) Mr. Chaman was not liable for contempt as the order of Labor court was
ultimately reversed by High Court
(b) Mr. Chaman will be liable for contempt irrespective of whether or not the High
Court set aside the order of
labor court
(c) Mr. Chaman will not be liable for contempt as the non-compliance was not
deliberate.
(d) Can’t say as the facts are not complete to decide if the reason for non-
compliance was satisfactory or not.

. Page 20 of 36
82. Neha and Reema are colleagues. However, one day there was a huge fight between
the two which led to physical
altercations between them. In this altercation, Reema was suffered minor injuries
and a case of hurt was filed
against Neha. The Court advised the parties to settle the matter outside the court.
However when Neha and
Reema failed to settle the matter, the Court invoked its contempt jurisdiction to
punish both Neha and Reema.
Decide
(a) Neha and Reema will be liable for contempt since they failed to comply with
Court’s order
(b) Neha and Reema will not be liable for contempt as the directions of the Court
were mere advisory in nature
and hence not binding.
(c) Neha and Reema will be liable for contempt even when the directions of the
court were advisory in nature.
(d) Neha and Reema will not be liable for contempt as whether or not to settle the
matter is the discretion of
parties.
83. In the matter Anil Goel versus Income Tax Dept, The Income Tax Tribunal had
directed the Income Tax
Department to refund a sum of INR 4 Lakhs to Mr. Goel within a period of 7 days.
However, when the Income
Tax Dept. failed to refund the money, a case of contempt was filed by Mr. Goel
against the Income Tax Dept.
In the contempt case, the court was satisfied with the reasoning given by the Dept.
regarding delay caused in
refunding the money and hence the contempt case was dismissed. Aggrieved by this
order, Mr. Goel filed an
Appeal. Decide if the Appeal is maintainable in light of the passage?
(a) The appeal is not maintainable as the contempt proceedings does not involve any
third party
(b) The appeal is maintainable because any person aggrieved by any court’s order
can file an appeal
(c) The appeal is not maintainable as the contempt proceedings are a matter between
the Court and Contemnor
(d) This question can’t be answered since the reason for non-compliance is not
provided.
84. In light of your understanding of the passage above, pick the incorrect
statement
(a) Power to punish for contempt is an inherent power of every court in the
country.
(b) A person faces contempt proceedings only on deliberate failure to comply with
Court’s order.
(c) If the Court is satisfied that there is a satisfactory reason for not complying
with the order, the court may
drop contempt proceedings.
(d) Power to punish for contempt is a responsibility as well as discretion of the
Court.
85. Harish and Radhe are business partners and have been working together for about
20 years now. However, owing
to current financial crisis, there arose some disputes between the two and
ultimately the matter reached the Court.
After listening to both parties, the Court directed Harish to pay a sum of INR 2
lakhs to Radhe. However, instead
of paying the sum, both parties settled the matter at INR 50,000. Will such act
amount to contempt of the court?
(a) Both Harish and Radhe shall be liable for contempt as there was clear violation
of the Court’s direction
(b) Only Harish will be liable for contempt as he violated the court’s direction
(c) None shall be liable for contempt as parties settled the matter and no Court
has the jurisdiction to interfere
in a matter which is settled between the parties inter se.
(d) Only Radhe shall be liable for contempt as he should not have interfered with
due course of justice by settling
the matter with Harish.
Passage(Q.86-Q.90): Justice Robert H Jackson of the United States Supreme Court, in
Brown v. Allen (1953),
famously quipped,
"We are not final because we are infallible, but we are infallible only because we
are final."
A slight variation of Justice Jackson's words has emerged in India for Supreme
Court of India…. Article 137 of
the Constitution of India confers it the power to review any judgment pronounced or
order made by it…
It is rare for the Supreme Court to entertain and accept a review petition…. As far
back as in 1940, Chief Justice
Gwyer, speaking for the Federal Court in Raja Prithwi Chand Lall Choudhry etc. v.
Rai Bahadur Sukhraj Rai &
Ors had observed:
"This Court will not sit as a court of appeal from its own decisions, nor will it
entertain applications to review
on the ground only that one of the parties in the case conceives himself to be
aggrieved by the decision. It would

. Page 21 of 36
in our opinion be intolerable and most prejudicial to the public Interest if cases
once decided by the Court could
be re-opened and re-heard: "There is a salutary maxim which ought to be observed by
all Courts of last resort
-- Interest reipublicae ut sit finis litium…."
The Supreme Court in Kamlesh Verma v. Mayawati summarised the principles for
entertaining review petitions.
(i) Discovery of new and important matter or evidence which, after the exercise of
due diligence, was not within
the knowledge of the petitioner or could not be produced by him;
(ii) Mistake or error apparent on the face of the record;
(iii)Any other sufficient reason - which has been interpreted to mean a reason
sufficient on grounds at least
analogous to the principles mentioned above.
[ Source- Bar and Bench, “[The Viewpoint] How final are decisions of the Supreme
Court,” published on March
07, 2022 <https://www.barandbench.com/view-point/how-final-are-decisions-of-the-
supreme-court> ]
Based on your understanding of the passage, answer the following questions:
86. Radha and Shyam are married since 2005. However, in the year 2015, certain
issues arose between the two and
Radha filed a divorce case on the ground of cruelty. However, the case was opposed
by Shyam. The lower court
granted divorce by citing that since Shyam came home drunk every night, it amounts
to cruelty in marriage. The
same was approved by High Court. Ultimately, Shyam filed an Appeal before the
Supreme Court. Supreme
Court refused to entertain the Appeal. Thereafter, 2 years later, Shyam found love
letters belonging to Radha
and Ankit from her Almirah at his house. On the basis of these love letters, he
filed a review Petition. Decide if
the Supreme Court will entertain the review petition.
(a) The Supreme Court will entertain the review petition because there is a
discovery of new and important
matter or evidence.
(b) The Supreme Court will not entertain the review petition because there is no
mistake or error apparent on
the face of record.
(c) The Supreme Court will not entertain the review petition because the new
evidence found could have
reasonably be known with due diligence.
(d) The Supreme Court will entertain the review petition in the interest of
justice.
87. According to you what should be the most appropriate meaning of “We are not
final because we are infallible,
but we are infallible only because we are final”
(a) The sentence means “We are not final because we are right, we are right because
we are final.”
(b) The sentence means, “we are not right because we are final, we are final
because we are right.”
(c) The sentence means, “if we are right, we are final, if we are not right, we are
not final.”
(d) The sentence means, “if we are final, we are right, if we are not final, we are
not right.”
88. According to the passage, select the most appropriate reason as to why Courts
are reluctant to entertain a review
petition.
(a) Because the Supreme Court is already burdened with a lot of cases and to avoid
pendency of cases, it is
reluctant to entertain review petition.
(b) Because the Supreme Court is right and final in all cases.
(c) Because finality of decision and adjudication of rights between the parties to
a matter is very important to
ensure that there is no miscarriage of justice and there is end of litigation.
Hence, it is allowed only in most
exceptional circumstances.
(d) Because the State and litigants will have to incur a lot of cost to conduct
review of all cases.

. Page 22 of 36
89. ABC ltd and XYZ Ltd. are two companies engaged in manufacturing of crackers.
However, due to a govt’s
notification all production of crackers is banned with immediate effect. It was
challenged by both ABC Ltd and
XYZ Ltd before the High Court. High Court refused to entertain the matter citing
the reason of public health and
environment. It was challenged before the Supreme Court. However, Supreme Court
after an elaborate hearing,
decided in favor of ABC Ltd and XYZ Ltd citing that the two companies were not
given a chance to be heard
before the notification was announced by the Govt. Aggrieved by the decision of
Supreme Court, the Parliament
decided to review the decision of Supreme Court and passed a law banning production
of crackers. Based on the
passage decide if such review was correct.
(a) No, the review is not correct because there was no discovery of new and
important material/evidence.
(b) No, the review is not correct because there was no mistake or error apparent on
the face of record.
(c) Both (a) and (b)
(d) Neither (a) nor (b)
90. Out of the following options, which of the options qualify to fall under the
category of “Mistake or error apparent
on the face of the record”?
(a) Failing to take into consideration the evidence already in the files before the
Supreme Court.
(b) Failing to consider that there might be a different opinion on the question of
law, which could have been
more acceptable by the majority of public.
(c) Failing to consider the impact a decision might have on the policies of the
government in power.
(d) All three above
Passage(Q.91-Q.95): Rajbir Sehrawat. J., contemplated and answered the interesting
question asked in the
recruitment test on which the dispute of the petitioner revolves around.
“73. Which of the following schedule of the Constitution is immune from judicial
review on the grounds
of violation of fundamental rights?
A) Seventh Schedule B) Ninth Schedule C) Tenth Schedule D) None of the above”
Petitioner sought a correct answer to the above said question asked in Recruitment
Test. The petitioner answered
and claimed that ‘D’ (none of the above) is the correct answer. Whereas,
respondents communicated that
according to them ‘B’ (9th Schedule) is the correct answer. It was submitted by the
petitioner that on account of
wrong answer taken by the respondents; the right of the petitioner to seek public
employment has been
jeopardised.
Here were two approaches taken by Court:
 Simple answer - Under Article 31-B of Constitution of India, immunity is provided
to the laws included in
the ‘Ninth Schedule’ from being declared as null and void on the ground of such law
being violative of
fundamental rights. On the contrary, the Supreme Court has upheld the Article 31-B
and accordingly, the
laws included in Ninth Schedule; despite the same being directly violating the
fundamental rights of the
citizen affected by such law. “Hence, the answer to the question asked in the above
said exam has to be,
necessarily, ‘B’ (Ninth Schedule).”
 Complex answer - While answering the question the Court relied upon the judgment
of Supreme Court in R.
Coelho v. State of Tamil Nadu, (1999) 7 SCC 580, where it was held that, after
24.4.1973, no amendment
for adding laws to the Ninth Schedule is immune from judicial review and the test
would be whether it
violates the basic feature of the Constitution. Essence of certain rights conferred
by Part III was also upheld.
However, the Court observed that the complex answer did not satisfy the requirement
of the question asked
in the test.
The Court while dismissing the petition did not find any substance in the argument
of the petitioner to justify the
answer presented by him. It was held that, “the language of Article 31-B grants
immunity to the law included in
the Ninth Schedule from being declared void on the ground of the violation of the
fundamental rights; as was,

. Page 23 of 36
otherwise, mandated by Article 13 (2). As observed in foregoing paragraphs, Article
31-B which grants this
immunity, as such, has not been set aside by the Hon’ble Supreme Court, till
today.”
Court held that article 13(2) states that no state may make laws that take away or
abridge an individual's
fundamental rights. If done so then can be challenged and protected by court
subject to ninth schedule while
considering other parameters. The Court opined that the question asked needed lots
of explanations for being
answered correctly and does not, straightway admitted the precise answer,
therefore, this question or a question
analogous thereto, if asked as a multiple choice type of question, has the
potential of damaging the right of citizen
to get public employment; like the petitioner of the present case.
(Source: P&H HC | Answer to the question on ‘fundamental rights vis-a-vis judicial
review’ considered as
‘National Confusion’ as different interpretation possible, scconline.com)
91. Choose a sentence(s) that present an anomaly with the language of the Ninth
Schedule of the Indian Constitution.
I. Amendments added to the ninth schedule are not immune from judicial review.
II. The Ninth Schedule contains a list of laws that are not void on violation of
fundamental right in court.
III. The Supreme Court has judicially reviewed laws included in the Ninth Schedule
in a number of decisions,
held that they directly violate the fundamental rights of the citizens affected by
such laws thus such laws are
void.
IV. The Ninth Schedule is an approved schedule, despite the fact that it violates
Article 13 (2) of the Indian
Constitution.
(a) I, II and IV (b) III (c) II & III (d) III & IV
92. The government made an amendment in the disaster management act that states
that in the event of a pandemic,
no one is allowed to refuse vaccinations that are being administered in the
interest of public safety and preventing
the spread of deadly viruses. Some citizens claimed that it violated the
fundamental right to health, which is
enshrined in Article 21 of the Indian Constitution. Will the petition be able to be
sustained?
(a) Yes, the Indian constitution allows for the challenge of any law that violates
a fundamental right enshrined
in Part 3 of the constitution.
(b) No, because the law was enacted to benefit the general public and hence the
petition won't be sustained.
(c) Yes, because such law will be declared null and void on the grounds that it
violates fundamental rights.
(d) No, because the facts are insufficient if it would have been the case of
amendment made under ninth schedule,
it would have come under the purview of judicial review.
93. Answer the following question in light of the given information presented in
the context of the passage;
Whether an Act or Regulation which, or a part of which, is or has been found by the
Supreme Court to be
violative of any of the Articles 14, 19 and 31 can be included in the Ninth
Schedule?
(a) Yes, despite the fact that it directly violates the fundamental rights of the
citizen affected by such act or
regulation.
(b) It is not at the court's discretion because such acts and laws are protected
under Article 31 B of the Indian
Constitution.
(c) yes, because such legislation is not immune from judicial review because it
violates fundamental rights.
(d) Cannot be determined due to insufficient information provided in the passage.
94. Sudhir took the Haryana law recruitment officer examination, and in the revised
answer key, a question answered
in the objective type examination shows two alternatives as the correct answer. As
a result, additional candidates
were adjusted in the final recruitment list, and Sudhir's rank dropped from AIR 150
to AIR 200. Sudhir, although
being qualified, brought a complaint against the respondent, jeopardising the
petitioner's ability to advance in
public service. Decide whether sudhir’s contention is maintainable or not?
(a) Sudhir's complaint is valid since the respondent's later adjustment of two
possibilities without offering an
explanation amounts to a breach of the petitioner's right to seek public
employment.
(b) Sudhir's claim is invalid because he's a qualified applicant.

. Page 24 of 36
(c) Sudhir's claim is true, as his ranking has slipped from 150 to 200.
(d) Sudhir's contention is viable and will be heard by the court on the grounds
that the petitioner's right to seek
public employment should not be jeopardised.
95. Assertion: The concept of a universal civil code can only be adopted through
the ninth Schedule because the
matter has been in discussion for long as enforcing it would violate the right to
religion guaranteed under Part
III of the Indian Constitution.
Reason: laws enacted under Ninth Schedule of the Constitution are immune from
judicial review on the grounds
of violation of fundamental rights.
(a) Both A and R are true but R is not correct explanation of A.
(b) Both A and R are true and R is correct explanation of A.
(c) A is true but R is false.
(d) A is false but R is true.
Passage(Q.96-Q.100): In an important verdict on Tuesday, the Karnataka High Court
dismissed various
petitions challenging a ban on Hijab in education institutions. The full bench of
the HC concluded its hearing in
the Hijab case saying that prescription of school uniform by the State is a
reasonable restriction on the students'
rights to wear a dress of their choice under Article 19(1)(a) and thus, the
Government Order issued by the
Karnataka government dated February 5 is not violative of their rights.
"The school regulations prescribing dress code for all the students as one
homogenous class, serve constitutional
secularism", the Court observed.
It is too farfetched to argue that the school dress code militates against the
fundamental freedoms guaranteed
under Articles, 14, 15, 19, 21 & 25 of the Constitution and therefore, the same
should be outlawed by the stroke
of a pen", the Court held.
No "reasonable accommodation" can be given for hijab; allowing it can lead to
"social-separatedness" and
offends the feel of uniformity, the Petitioners had relied on the 'principle of
reasonable accommodation' to
contend that they should be permitted to wear hijab of structure and colour that
suit to the prescribed dress code.
A Full Bench of the High Court comprising Chief Justice Ritu Raj Awasthi, Justice
Krishna S Dixit and
Justice JM Khazi today held that wearing of hijab is not a part of Essential
Religious Practice in Islamic faith
and thus, is not protected under Article 25 of the Constitution. The Court further
rejected the reliance placed by
the Petitioners on a judgment of the Constitutional Court of South Africa, in
KwaZulu-Natal and Others v Pillay,
which upheld the right of a Hindu girl from South India to wear a nose ring to
school.
It held, "Constitutional schemes and socio-political ideologies vary from one
country to another, regardless of
textual similarities. A Constitution of a country being the Fundamental Law, is
shaped by several streams of
forces such as history, religion, culture, way of life, values and a host of such
other factors. In a given fact matrix,
how a foreign jurisdiction treats the case cannot be the sole model readily
availing for adoption in our system
which ordinarily treats foreign law & foreign judgments as matters of facts."
(Source: Hijab row: Karnataka HC dismisses petitions, rules wearing Hijab not
essential practice, excerpt taken
from The Business Today)
96. Following multiple petitions and challenges, the Army altered policy in 2017
and now permits Sikh soldiers to
wear religious accoutrements and turbans with few restrictions as it is an
essential religious practice followed by
Sikhs community. The union government has also agreed to the rule. A petition has
been filed in the Supreme
Court, arguing that allowing Sikhs to wear religious accessories in army uniforms
would be discriminatory
against other religions. Decide
(a) The petition is maintainable on the grounds that allowing one religious group
to wear religious accessories
while denying others constitutes discrimination.
(b) The petition is unsustainable since the union permitted the practise because it
was an integral practise in the
Sikh community.
(c) The petition is unconstitutional because authorising such activity infringes
fundamental freedoms granted
by Articles 14, 15, 19, 21, and 25 of the Constitution.

. Page 25 of 36
(d) According to the paragraph, if it is a necessary activity, it cannot be
prohibited because it is protected by
Article 25 of the Constitution.
97. In a new law adopted by the Indian government, the government from the rightist
party secured a majority vote.
According to the law, all schools in India, whether governmental and non -
governmental, would henceforth be
required to wear exclusively Indian uniforms, with no skirts or shirts permitted.
Girls will wear a suit with a
chunni, while boys will wear a kurta and pyjama. The justification offered for
implementing such a law is that it
will promote and protect Indian culture from degrading. Isthe newly passed
legislation in accordance with article
19(1)(a)?
(a) No, as stated in article 19 (1) (a), students have the freedom to wear any
dress they like.
(b) Yes, because article 19 (1) (a) also includes some reasonable restrictions.
(c) Yes, because the uniform school dress rule violates the fundamental freedoms
protected by Articles 14, 15,
19, 21, and 25 of the Constitution.
(d) No, because the union's goal of instituting a uniform clothing code for all
students is consistent with Article
14 of the Indian Constitution.
98. Through an enactment, the government of India forbade women over the age of 16
from wearing western attire.
Decide
(a) The law violates fundamental right guaranteed under article 19 (1) (a).
(b) The law is valid as it allows the state to put some reasonable restriction.
(c) The being violative of fundamental right provided under article 25.
(d) Cannot be ascertained on grounds of incomplete information provided in the
facts.
99. Continuing with the similar facts presented above, suppose the law has been
enacted without giving any reason
for his enactment. Now select a statement that, in your opinion, best sums up the
context of the passage and is
also in line with the pertinent facts given.
(a) Such an act will be declared null and void since it violates the fundamental
rights provided by Article 19 (1)
(a) of the Indian Constitution.
(b) Such an act shall be declared null and void to the extent that its provisions
violate fundamental rights
demands.
(c) The act will not be fully justified unless the government provides a reasonable
explanation for passing such
legislation.
(d) The act is unconstitutional because, according to Article 19 (1) (a), every
citizen has the right to dress as he
wishes in public and also violative of article 14 on grounds of discrimination.
100. Assertion: The precedent of a foreign court is not admissible in Indian
courts.
Reason: Regardless of linguistic similarities, constitutional frameworks and socio-
political ideologies differ
from country to country.
(a) Both A and R are false and R is not correct explanation of A.
(b) Both A and R are true and R is correct explanation of A.
(c) A is true but R is false.
(d) A is false but R is true.

. Page 26 of 36
Passage(Q.101-Q.105): An Ahmedabad trial court’s decision to sentence as many as 38
people to death for
the 2008 bomb blasts that killed 56 people in Gujarat tears to shreds the judicial
circumspection urged by
the Supreme Court of India in Bachan Singh v. State of Punjab (1980) on the
question of capital punishment.
Judge A.R. Patel accepted the prosecution’s claim that the role played by over
three dozen people satisfied
the “rarest of rare” doctrine, while sentencing 11 others to life.
The convicts have already spent 13 years in jail and their defense lawyers intend
to appeal. Over the past
several decades, death sentences pronounced by trial courts have mostly been
overturned by the higher
judiciary. It is to be hoped that the 38 men will also have their lives spared when
their appeals are heard. At
the very least, their cases deserve closer scrutiny and well-considered
adjudications.
The fact that all of the 38 are Muslim, while Hindus found guilty of mass murder on
the same scale have
been spared the death penalty is bound to raise questions about the true meaning of
‘rarest of rare’. While
sentencing Babu Bajrangi, Maya Kodnani and others for their roles in the Naroda
Patiya massacre of 97
Muslims in the 2002 Gujarat riots, judge Jyotsna Yagnik ruled out imposing capital
punishment because
she said it went against “human dignity”.
But to stay with Bachan Singh for a minute, what did the Supreme Court majority in
1980 say after weighing
in great detail the arguments for and against the death penalty?
“…[F]or persons convicted of murder, life imprisonment is the rule and death
sentence an exception. A real
and abiding concern for the dignity of human life postulates resistance to taking a
life through law’s
instrumentality. That ought not to be done save in the rarest of rare cases when
the alternative option is
unquestionably foreclosed.” (emphasis added)
Justice Bhagwati wrote: “Death penalty in its actual operation is discriminatory,
for it strikes mostly against
the poor and deprived sections of the community and the rich and the affluent
usually escape from its clutches.
This circumstance also adds to the arbitrary and capricious nature of the death
penalty and renders it
unconstitutional as being violative of (the Indian Constitution’s) Articles 14
(equality before the law) and 21
(protection of life and personal liberty).”
(Source: Death Penalty is State-Sponsored Murder, the Indian Judiciary Must Put a
Stop to Executions,
Excerpt taken from The Wire)
101. Para 1 of the passage states that Ahmedabad trial court’s decision to sentence
38 people to death for the
2008 bomb blasts that killed 56 people in Gujarat tears to shreds the judicial
circumspection urged by the
Supreme Court of India in Bachan Singh v. State of Punjab (1980) on the question of
capital punishment. Chose
a statement most likely to describe correct context of the excerpt?
(a) It was held in Bacchan singh v State of Punjab, trial court’s can only
pronounce capital punishment in rarest
of the rare case.
(b) The apex court urged judicial caution in Bachan Singh v. State of Punjab on the
issue of capital
punishment.
(c) The apex court cautioned lower judiciary to consider aspect of human dignity
while deciding on the issue
of capital punishment.
(d) The apex court ruled that for persons convicted of murder, life imprisonment is
the rule and death sentence
an exception.
102. Which of the following, when discussing a case of capital punishment, appears
anomaly with the author's point
of view presented in the passage?
(a) The role played by approximately to 38 people in the bomb blast do not passes
the "rarest of rare" doctrine
test.
(b) Capital punishment orders issued by trial courts are not plausible to be
reversed on appeal.
(c) When deciding on a case of capital punishment, natural justice rules should be
considered.
(d) The death penalty should only be used as a last resort when all other options
have been exhausted.

. Page 27 of 36
103. In a 2015 case of mob lynching, a mob broke into a jail in Dholakpur, Uttar
Pradesh, and lynched an accused
rapist who awaiting capital punishment the next day. In the present case, what will
the court's decision be?
(a) The court will have to punish the mob as the real and abiding concern for the
dignity of human life postulates
resistance to taking a life against law’s instrumentality.
(b) The court cannot punish the mob because the accused is already awarded the
death penalty.
(c) Though the conduct of mob will be considered against the instrumentality of
law, the act of mob is justified
in light of the immoral act committed by the accused.
(d) Uncertain because the passage does not provide enough information to reach a
conclusion.
104. What would be considered an incorrect observation on capital punishment based
on the context of the passage?
(a) The order of capital punishment always tends to be reversed by Supreme Court in
case of an appeal.
(b) Capital punishment awarded in its actual operation will not be rendered
unconstitutional.
(c) Capital punishment in general is considered unconstitutional being violative of
article 14 and 21 of
constitution of India.
(d) A genuine and abiding concern for the dignity of human life necessitates
resistance to taking a life through
the use of the legal system.
105. Anand was arrested for murder and marital rape of his wife after his in-laws
filed a complaint. The bench,
believing that this is a case of the rarest of the rare, sentences the accused
Anand to death. Decide
(a) The sentence tears to shreds the judicial circumspection urged by the Supreme
Court of India on the question
of capital punishment.
(b) The sentence is valid as the bench opined it to be a perfect case of rarest of
the rare case.
(c) The sentence is valid as it is not discriminatory and arbitrary.
(d) The sentence is not valid as it is unconstitutional.

. Page 28 of 36
SECTION - D: LOGICAL REASONING
Passage (Q.106-Q.110): The WHO chief in a recent briefing, noted that 90 million
cases of coronavirus have
been reported since the Omicron variant was first identified 10 weeks ago. His
statement comes in the context
of many countries easing their restrictive movement measures amid public fatigue.
From WHO’s perspective,
the blanket lifting of restrictions poses a problem as most people appear to
believe that Omicron is less
threatening compared to previous variants and that two shots of vaccines are an
adequate defence against the
virus. He underlined that a narrative that “preventing transmission is no longer
possible and no longer necessary”
had taken hold and this was problematic. This was false, he underscored at the
briefing, as the virus continues to
evolve and four of the six WHO regions globally are reporting an increasing trend
in deaths.
Britain, France, Ireland, the Netherlands and Finland are on the path of easing
COVID-19 restrictions. In India
too, with current evidence pointing to a fall in the daily caseload, several States
have moved to ease movement
restrictions. WHO has also said that the newly emerged variant, BA.2, is as
transmissible as Omicron and that
all measures needed to contain the original Omicron variant are applicable to it
too. After facing criticism that it
did not move soon enough in 2020 to alert the world of the magnitude of the
calamity that it awaited, WHO,
which takes a global view of the crisis, cannot be faulted for airing concerns from
the evidence available so far.
It has also consistently warned that the pandemic cannot be over until all regions
of the world are sufficiently
vaccinated and that economic inequity continues to be a driver of the pandemic. The
coronavirus, while secular
in its infectiousness, affects nations differently. The richer ones can afford to
bear the consequences of disrupted
social activity a little longer than the rest. Just about half the world has been
fully vaccinated; unfortunately, so
far, the available vaccines are only equipped to protect against disease rather
than infection. WHO must use its
influence to continue to encourage vaccination and step in with advice and
expertise to help countries access
necessary doses. Framing the pandemic as a war that humanity must ‘win’ was useful
to accelerate the
development of vaccines. However, science is not equipped yet to predict the future
trajectory of the coronavirus;
COVID-appropriate behaviour, vaccines and accessible health care remain the only
credible defenses.
106. What is the central theme of this passage?
(a) Disease prevention with COVID-appropriate behaviour and vaccination is still
necessary.
(b) WHO, this time cannot be faulted for airing concerns from the evidence
available so far.
(c) Covid Is the new normal and we need to learn living with it.
(d) Omicron is less threatening compared to previous variants.
107. As per the passage, which of the following approaches can be the most
effective course of action to curb the
spread of covid?
(a) Science is not equipped yet to predict the future trajectory of the virus, and
we should accept and start living
in this new normal.
(b) As preventing transmission is no longer possible, WHO must use its influence to
continue to encourage
vaccination.
(c) Government should focus on improving health facilities and encouraging people
to follow covid protocols
like wearing a mask.
(d) Government should provide boost to economy first as pandemic cannot be over
until economic inequity
exists.
108. Which of the following can be inferred from the above passage?
(a) Earlier WHO faced criticism that it did not move soon enough in 2020 to alert
the world of the magnitude
of the virus when it started spreading in China.
(b) Covid affects nations differently and the richer ones can much easily tackle
the virus. But if we need to stop
the spread, economic inequity needs to come to an end.
(c) Two shots of vaccines are not an adequate defense against the virus.
(d) Four of the six WHO regions globally are reporting an increasing trend in
cases.

. Page 29 of 36
109. “WHO, which takes a global view of the crisis, cannot be faulted for airing
concerns from the evidence available
so far.” In the context of the statement, which of the following strengthens the
author’s opinion?
(a) IIIT Kanpur recently published a report stating that preventing transmission is
no longer possible.
(b) WHO has done a lot by providing covid vaccines to poor countries this year.
(c) WHO didn’t warn about a new variant which later caused a large number of
deaths.
(d) WHO kept on warning India about incoming third wave, which actually came and
peaked in Jan 2022.
110. In the above passage, which of the following is not true about COVID?
(a) Covid has multiple variants.
(b) India, like other countries, is also facing cases of covid.
(c) There exists a vaccine to counter covid infections.
(d) Easing covid restrictions is one of the courses of action which countries are
adopting.
Passage (Q.111-Q.115): West Bengal Chief Minister Mamata Banerjee’s outburst
against Governor Jagdeep
Dhankhar on Monday was not a first but it brought to the fore, yet again, the role
of the Governor in relation
with the elected government and legislature. Mr. Dhankhar and his counterparts in
Tamil Nadu and Maharashtra
appear to be testing the limits of their power and confronting the elected
governments and legislatures in recent
weeks. Tired of Mr. Dhankhar’s constant tirade against her on Twitter, Ms. Banerjee
blocked him on the
platform. The Governor then sent her a message for “dialogue and harmony amongst
constitutional
functionaries” but promptly posted that too on Twitter. The Chief Minister said the
Governor was trying to treat
the elected government as “bonded labour. Mr. Dhankhar also has withheld assent to
the Howrah (Amendment)
Bill 2021, delaying polls to the civic body. He has made allegations of impropriety
in welfare schemes,
questioned Government claims about investments in the State, and taken up the
cudgels for the Opposition BJP.
In Maharashtra, Governor has stalled the election of Speaker since the post fell
vacant. He has taken
umbrage over the amendments in the legislative rules for holding the Speaker’s
election through voice
vote instead of secret ballot. The Governor’s view that the State Assembly cannot
decide its own rules is
unacceptable to the ruling coalition, but is being cheered by the Opposition BJP.
Mr. Koshyari had in the past
batted for the BJP, supporting its demand for a special session of the Assembly on
women’s safety and security.
He had refused to accept the recommendation of the Council of Ministers on the
nomination of 12 members to
the Legislative Council, until the matter reached the High Court. In Tamil Nadu,
governor has not acted upon
the T.N. Admission to Undergraduate Medical Degree Courses Bill, adopted by the
Assembly in September
2021.The Bill relates to a question of State-Centre relations, as it proposes to
dispense with the NEET Exam in
the State. NEET has been criticised for curtailing State powers, and the Governor’s
delay in processing the Bill
is only aggravating the situation. Some of these issues may require debate and
discussion before resolution. But
any imperial overtones of Governors can only do harm to the constitutional scheme
of things.
111. What is the central idea of this passage?
(a) The tussle between chief ministers and the governors of respective states
points out to a broader constitutional
issue.
(b) The governor should act in cooperation with Chief Minister of West Bengal in
all matters.
(c) The tussle between Chief Minister and the Governor highlights the disruption of
all constitutional ethos in
the recent past.
(d) Both West Bengal and Maharashtra are fighting Against BJP.
112. The author is most likely to agree with which of the following statements?
(a) The Governors of majority of the States are currently facing tussle with State
Governments
(b) The Centre state tussle is limited to three states.
(c) NEET Exam curtails the powers of the States.
(d) West Bengal is not the only state facing the Centre- State tussle.

. Page 30 of 36
113. Which of the following, if true, weakens the author’s argument?
(a) Indian constitution provides that the governor should decide in case of
conflict between states and Centre.
(b) State Government is restricting the discussions on the matter in Tamil Nadu.
(c) The Governor of Madhya Pradesh works in cooperation with the State Government.
(d) Other states also face such tussles, and they are very common now.
114. Which of the following strengthens the stance of the Governor of Tamil Nadu?
(a) NEET has lost its relevance in the contemporary world, as its format is
archaic.
(b) NEET is very prestigious, and the Central Government has always conducted it
with probity.
(c) The governor has taken a very long time to process the bill.
(d) Constitution prescribes that governor can take time as per his discretion to
process bills.
115. What purpose does the boldfaced statement in the passage serves?
(a) It is an argument to support how governors are misusing their powers.
(b) It is a premise to support how governors are misusing their powers.
(c) It is a premise supporting that Centre- State tussles in West Bengal are not
distinctive.
(d) It is an argument supporting that Centre- State tussles in West Bengal are not
distinctive.
Passage (Q.116-Q.120): The findings of the latest study by the PGI, Chandigarh, on
drug abuse in Punjab come
as no surprise: substance use in the beleaguered state continues rampantly, with
more than three million addicts
— mostly men — in the vicious grip of some harmful drug or the other. More
importantly, the survey, which is
aimed at giving muscle to the preventive steps and strategies, indirectly exposes
another well-known but bitter
truth: the failure of the successive governments to break the backbone of the drug
mafia over the past couple of
decades. Taming the drug lords with quick and deterrent penalties holds the key to
preventing the vulnerable
youth from going astray and wasting away.
The youngsters must be steered towards the right path — socially, mentally and
physically. Only an able Gennext
can pull the stressed state out of its depths of socio-economic ruin and propel it
to the days of resplendent glory
and prosperity. Nothing signifies more the looming dark clouds over Punjab than the
exodus of hordes of
promising young men and women to foreign lands. Parents, too, are willing to put
everything at stake to send
their wards abroad as they fear for them falling into the trap of substance abuse
here. The huge consignments of
drugs smuggled in regularly and little corresponding headway in catching and
punishing the influential operators
tells the bleak tale of a Punjab bled dry. The dreams of a bright future in the
state are fast turning into nightmares.
The huge burden of substance dependence and addiction inflicting the state is
apparent from the fact that 18 of
the 272 most affected districts by drug abuse in the country are in Punjab. But the
road map to ending this
menace, including educating students about its ill effects and banning liquor and
tobacco sale around schools
and colleges, have hit road bumps. For example, the government’s ‘Tu Mera Buddy’
scheme launched in 2018
to make Punjab ‘nasha mukt’ was later found to have been diluted to being the
‘Buddy Group’, a virtual platform
for learners during the Covid era of online teaching.
116. What is the best representation of the main point of the passage?
(a) Substance abuse has led Punjab to become one of India’s backward states.
(b) Curbing substance abuse is key for the prosperity of the state of Punjab.
(c) Punjab has two faces: one with its able farmers and the other with its addict
men.
(d) Punjab would have been one of India’s premier states, if not for substance
abuse.
117. Out of the following, which statement would weaken the arguments of the
author?
(a) The tax collected due to the transactions of drugs in Punjab form the most of
the government's earnings.
(b) Most parents who want to send their children abroad in Punjab work day and
night to gather money for it.
(c) The CPM government which was formed in 2012 was able to stop the supply of
drugs for a day.
(d) None of the above is correct.

. Page 31 of 36
118. For which of the following statements does the passage have some support?
(a) If Punjab becomes a drug free state, less of its youth population would leave
the state.
(b) Substance abuse is so prevalent in the state of Punjab that it is not
practically possible to root it out.
(c) Women of the state of Punjab don’t do drugs due to the less addictive nature of
the drugs on women.
(d) If the situation in Punjab goes more out of control, then it risks losing all
of its youth population.
119. Out of the following, which statement has been used as a premise to support
the author’s arguments?
i. ‘Tu mera buddy’ scheme was diluted to ‘Buddy group’.
ii. The youth of Punjab are more talented than those of other states.
iii. Punjab’s dream of becoming a prosperous state seems shattered.
(a) Only I (b) Only i and ii
(c) Only i and iii (d) Only ii and iii
120. The structure of the second paragraph of the passage is best represented by:
(a) Some disturbing developments are presented which might spell doom for the
concerned state.
(b) Some data are cited to highlight the gravity of the problem of Punjab.
(c) Some additional data are cited to help the readers know that the situation is
not as bad as it seems.
(d) Some suggestions are presented and some steps taken by the government are
presented later.
Passage (Q.121-Q.124): SADLY, addressing the important issue of proper labelling on
packaged food products
— it has a direct bearing on the consumers’ health — in our country tends to be an
indeterminately prolonged
affair, plunging, meanwhile, lakhs of people into the financially and mentally
straining rut of being disease#ridden. The authorities drag their feet on striking
an acceptable balance between the protection of consumers’
right to precise information of ingredients and the powerfully rich manufacturers’
lobby. It is widely known that
the intake of the addictive ultra-processed food items, being overly packed with
harmful sugar, salt, sodium and
fats, is linked to the proliferation of diabetes and heart ailments. That India
worrisomely ranks at number two in
such diseases and their prevalence is growing should have goaded the Food Safety
Standards Authority of India
to fast-track its decisions. But even eight years down the line, it is still
debating whether the labelling should
include a health warning along with the quantity of sugar, salt, sodium and fats
(that consumers want) or just
mention their amount (that producers want) on the packet. There is still a long way
to a resolution, draft proposal
and final issuance of norms, covering manufacturers, retailers and wholesalers.
Particularly impacted by the inordinate delays in the enforcement of such
guidelines are the estimated 1-3 per
cent of the population dependent on information warnings of food allergens such as
gluten (a protein present in
wheat, rye, barley), milk, soybean, egg, nuts, seafood etc. The intake of even
minuscule quantities of the allergen
may be enough to trigger a flare-up and cause huge misery to the patients and their
families. Food allergies cause
nearly 30,000 emergency treatments and 100 to 200 deaths per year in India.
There is much to learn from some western countries that have imposed food labelling
rules for over 15 years
now. Equally impressive are the high awareness levels there, ensuring allergen-
sensitive hospitality by hotels
and restaurants. Another related field begging for our government’s consideration
is the compensation doled out
by Italy to celiac disease patients for their life-long medically prescribed
dependence on expensive gluten-free
diet.
121. In context of the passage, which of the following question arises?
(a) Is proper labelling of packaged food necessary?
(b) Are our food labelling rules new?
(c) Should a label of health warning be there on packaged foods?
(d) Should India be answerable for the inordinate delays in enforcement of
labelling on packaged food products
guidelines?

. Page 32 of 36
122. According to the passage, which of the following is/are TRUE?
I. Food allergies cause nearly 30,000 emergency treatments.
II. The important issue of proper labelling on packaged food products is theme of
the passage.
III. The intake of minuscule quantities of the allergen may not be enough to
trigger a flare-up and cause huge
misery to the patients.
(a) Only III (b) I and II (c) I, II and III (d) Only I
123. Chose the correct option based on the appropriate reason for the assertion?
Assertion (A): There is still a long way to a resolution, draft proposal and final
issuance of norms, covering
manufacturers, retailers and wholesalers.
Reason (R): India ranks at number two in such diseases and their growing prevalence
should have goaded the
Food Safety Standards Authority of India to fast-track its decisions.
(a) Both A and R are true but R is not the correct explanation of A.
(b) Both A and R are true and R is the correct explanation of A.
(c) A is true but R is false.
(d) Both A and R are false.
124. What conclusion can be drawn from the passage?
(a) India is in top list for food safety.
(b) India provides precise information of ingredients on packaged foods.
(c) India is lagging behind in giving health warning on packaged foods.
(d) Consumption of addictive ultra-processed food items with the precise labelling
of ingredients leads to less
risk of disease.
Passage (Q.125-Q.129): Religion should be kept a private affair of individual
citizens, but that is an ideal case
scenario. Whether we like it or not, elements of religious practices have become
part of the culture of believers,
and the harmonious co-existence of such customs with the ones of those who practice
no religion is a pre#condition for a society’s progress. Religion being a general
term, the courts have elaborated on this constitutional
proviso and brought essential religious practices under its protection. It is in
this background that one should
view the controversy raging in Karnataka about Muslim girl students wearing the
hijab (headscarf).
As the Karnataka high court rightly observed while hearing the petition of the girl
students, every citizen of this
country should go by reason, the law, and the Constitution; not passion and
emotion. This is an advice to all
those who have taken to violent protests and also to those who come up with
whatabouteries to argue their points.
They must realize that the Constitution does not mandate banishing of religion;
instead, it affords people the
right to practice it. The question is how to make it least obtrusive in a secular
system. The situation as of today
is that while sane persons would be busy seeking to address the question as
suggested by the high court, vested
interests and extremists have almost hijacked the issue. Protests against the hijab
are taking place all across
Karnataka now, and it is highly likely that it shall spread to other states, too.
The Constitution and the law are made with a view to address issues as citizens of
a nation, and not as enemies.
It must not be done in a mechanical way in all situations; how a decision would
impact the lives of law-abiding
people must be a consideration the court should keep in mind. A total ban on the
hijab, which Muslims claim to
be an essential religious practice, will result in Muslim women either avoiding
multi-religious and secular
institutions or, worse, becoming dropouts. At the same time, the court cannot give
in to the pressure of identity
politics and communalists and allow each one to wear the dress of their choice. It
will, thus, be called upon to
come out with a solution that protects the right to practise religion and the right
to education while refusing to
yield to the demands of religious obscurantists of all hues. It should ensure that
better sense prevails.

. Page 33 of 36
125. What is the central idea of this passage?
(a) Religion should be kept a private affair of individual citizens and not a
public affair in educational
institutions.
(b) The right of practicing religion and that of education needs to be balanced
within the constitutional scheme.
(c) Everyone has right to practice their cultural practices and customs.
(d) The whole controversy is result of extremists and people with vested interests.
126. The author is most likely to agree with which of the following course of
action?
(a) A uniform school dress and a prohibition on wearing of religious cloths and
symbols in educational
institutions.
(b) People should strictly follow the law and the Constitution to avoid any such
conflicts.
(c) People should be allowed to wear dresses of their choices
(d) Various factors need to be composed, and their social impact should also be
considered.
127. “The situation as of today is that while sane persons would be busy seeking to
address the question as suggested
by the high court, vested interests and extremists have almost hijacked the issue”
points out in which of the
following options, the possible logical flaw in the argument of the author?
(a) The argument contains no stated or implied relationship between People having
vested interests and
extremists.
(b) The premises of the argument are stated in such a way that they exclude the
possibility of drawing any logical
conclusion.
(c) The conclusion is derived without any premise to support the argument.
(d) The author draws a specific conclusion out of one case.
128. Which of the following cannot be inferred from the passage?
(a) The author supports the stance taken by the court in this matter.
(b) Author supports that hijab must be allowed in educational institutions.
(c) The author is not in favor of allowing each one to wear the dress of their
choice.
(d) The issue can spread to different parts of the country.
129. “Every citizen of this country should go by reason, the law, and the
Constitution; not passion and emotion”.
Which of the statement, if true, will weaken the statement?
(a) The author himself has cases of violation of law and constitutional norms.
(b) Religious passion and emotions are the driving forces behind people’s seeking a
religion, and the constitution
regards religious sentiments as sacrosanct.
(c) Following law and constitutional norms makes a society free from conflicts.
(d) Religious sentiments defy all possible logic, and people place religion above
Constitution, laws or reason.
130. Following question contains 6 statements followed by 4 sets of combinations of
3.
Choose the set in which the statements are logically related.
A. Some weight producing things are tasty.
B. All weight producing things are pastries.
C. Some pastries are weight producing.
D. Some pastries are not tasty.
E. All pastries are tasty.
F. All pastries are weight producing.
(a) ABD (b) ADE (c) EFA (d) ADF

. Page 34 of 36
131. Rena travels 10 km North turns left and travels 4km and then again turns right
and covers another 5km. He then
turns to right hand side and travels another 4 km. How far is he from the starting
point of his journey?
(a) 15 km (b) 4 km (c) 5 km (d) 10 km
Directions (Q.132-Q.135): Study the following information and answer the questions
given below.
Eight persons A, B, C, D, E, F, G and H are sitting around a rectangular table and
all of them are facing towards
the center of table. All of them like different colour viz. Red, yellow, white,
black, pink, violet, orange and green
but not necessarily in the same order. Two persons sit on each side of table. No
two successive persons are sitting
together according to alphabetical order. For Example: A does not sit next
(adjacent) to B; similarly B does not
sit next (adjacent) to C and so on.
A sits third to left of G who likes green colour. Only one person sits between A
and C. The one who likes white
colour sits second to right of C. Only one person sit between A and H. Only one
person sits between B and F
who likes pink colour.
The persons who like red and orange colour sit on the same side. E neither like red
nor orange colour. Three
persons sit between the one who likes black colour and the one who like red colour.
C does not like black colour.
The one who likes yellow colour does not sit with adjacent to F.
132. Who is sitting third to the left of C?
(a) E (b) A (c) F (d) H
133. Who is sitting opposite to D?
(a) A (b) C (c) F (d) E
134. Who among the following likes yellow?
(a) E (b) D (c) C (d) B
135. Who among the following likes black?
(a) E (b) D (c) C (d) b

mock 35
Directions (Q.1-Q.30): Read the following passage carefully and answer the
questions that follow.
Passage (Q.1-Q.5): In her dream she is fourteen, running home from work on
precarious little heels, down the
paved street, as molten metal plip-plops around her, dotting the cement with
perfect silver discs. Her mother lags
six feet behind, following her footsteps, urging her to tread carefully. A V2
rocket launches overhead, triggering
the timer in her mind. She waits in the signature silence for the rocket to find
its mark, as it careens noiselessly
through the night sky. “One,” she shouts, “Two. Three. Four.” “We have plenty of
time, Lizzy, plenty of time.”
Her mother is shouting, over and over, no panic, just a calm, reassuring scream. It
is the same dream as always.
Lizzy reaches behind and grabs her mother’s hand, pulls her into the shelter of
their porch, watching as the house
across the road explodes into a heap of searing, flaming rubble.
My mother, Lizzy, is ninety-seven, and I think of her as a survivor. Of life and
time. Not long ago she had a
mother, husband, sister and two daughters. Now she has me. Twice a week I open the
door to her house and look
left, peering through the curtained darkness, along the hall and into her bedroom.
I can see she is still sleeping.
Under the formless camouflage of blankets, her shape has diminished and become
frail, although her carers joke
that she inhales each meal. They also say she is still stubborn but in a cute way.
She has passed the irritating
cantankerousness of old age and drifted without notice into the gentle humility of
a disappearing mind. There is
not enough time in a day to see her more often, constructing a life so much more
important, exciting,
exasperatingly busy than it is.
“I’m so grateful you look after me, so glad,” she repeats. I accept her gratitude,
even though guilt battles with
honesty, and pulses through my brain at sleepless midnights. In her dream, she is
eighteen, with the blonde
permed hair and arched brows of a movie star. Her husband George, the love of her
life, is away at war, and she
fears he will never come home. He writes sometimes, although there are no mailboxes
in the Pacific, and home
is his last thought as suicide planes fall burning out of the sky. The blackened
casualties of war sleep beside him
in rows on the smoking deck. She wakes sobbing, drenched in fear, and spots the
picture of a bride and groom
on her bedside. Reassured, she remembers the sixty years of devoted life with him,
recalls they had plenty of
time and longs for more.
I try to wake her but she is in a deep sleep. I know the conversation if she wakes.
I will ask her if she has eaten
and she will say she’s not hungry. I will try to cajole her into the warming
sunlight, pressure her to have tea and
a chat. She will refuse. She will tell me she is happy to be curled up in her warm
bed, that she is tired.
1. Why does Lizzy suddenly wake up weeping and embraced with apprehension?
(a) Her daughter neither takes care of her nor takes out time to talk to her.
(b) In her dream, she considers herself an eighteen-year-old lady when her husband
was serving in the war zone
and his life was in great danger.
(c) The memories of war and how her mother and she survived haunts her to date.
(d) Lizzy at ninety-seven is stuck in the state of timelessness and the memories of
the war are the only vivid
memories that come to haunt her in her dreams.
2. ‘My mother, Lizzy, is ninety-seven, and I think of her as a survivor. Of life
and time’ The author implies through
the statement that
I. Lizzy has stood the test of time.
II. Lizzy braved war, and the associated emotional and mental trauma associated
with the war.
III. Lizzy survived the loss of her husband and a daughter to the war.
(a) Only I (b) II and III
(c) Only I and II (d) All I, II and III

. Page 3 of 40
3. Which of the accompanying is NOT seen by Lizzy in her dream when she is fourteen
years of age?
(a) Her mother and she evade the catastrophic effect of a rocket that destroys the
houses across the road.
(b) Her mother panickily instructs Lizzy to get away from the rocket they have a
paucity of time.
(c) She is being followed by her mother who asks her to proceed carefully.
(d) She is hurrying down from work to home on the paved street on little heels.
4. Which of the accompanying impeccably explains the present condition of Lizzy?
(a) She has been the same in terms of health, however, stubborn in a lousy way;
also, she's slipped past the
unpleasant ill-temperateness of old age and into the soft humility of a vanishing
intellect.
(b) She has become puny and weak, however, stubborn in a good way; also, she's gone
into the unpleasant ill#temperateness of old age and vanishing intellect.
(c) She has become puny and weak, however, stubborn in a good way; also, she's
slipped past the unpleasant ill#temperateness of old age and into the soft humility
of a vanishing intellect.
(d) She has become tiny and vulnerable of health, living in the past with a mind
that has seen better days, yet
carrying the images of her life vividly.
5. What is the tone of the author in the passage?
(a) Commiserating (b) Acerbic (c) Contemptuous (d) Doleful
Passage (Q.6-Q.10): The prospects for a political career in Germany for a thirty-
year-old Austrian without
friends or funds, without a job, with no trade or profession or any previous record
of regular employment, with
no experience whatsoever in politics, were less than promising, and at first, for a
brief moment, Hitler realized
it.
He had returned to Munich at the end of November 1918, to find his adopted city
scarcely recognizable.
Revolution had broken out here too. Though a moderate Social Democratic government
under the real power in
Bavarian politics passed to the Right.
What was the Right in Bavaria at this chaotic time? It was the Regular Army, the
Reichswehr; it was the
monarchists, who wished the Wittelsbach back. It was a mass of conservatives who
despised the democratic
Republic established in Berlin; and as time went on, it was above all the great mob
of demobilized soldiers for
whom the bottom had fallen out of the world in 1918, uprooted men who could not
find jobs or their way back
to the peaceful society they had left in 1914, men grown tough and violent through
war who could not shake
themselves from ingrained habit.
Armed free-corps bands (______) all over Germany and were secretly equipped by the
Reichswehr. At first they
were mainly used to fight the Poles and the Balts on the disputed eastern
frontiers, but soon they were backing
plots for the overthrow of the republican regime. In March 1920, one of them, the
notorious Ehrhardt Brigade,
led by a freebooter, Captain Ehrhardt, occupied Berlin and enabled Dr. Wolfgang
Kapp, a mediocre politician
of the extreme right, to proclaim himself Chancellor.
In Munich, at the same time a different kind of military coup d’etat was more
successful. On March 14, 1920,
the Reichswehr overthrew the Hoffmann Socialist Government and installed a right-
wing regime under Gustav
von Kahr. And now the Bavarian capital became a magnet for all those forces in
Germany which were determined
to overthrow the Republic, set up an authoritarian regime. It was in this fertile
field in Munich that Adolf Hitler
got his start.
When he had come back to Munich at the end of November 1918, he had found that his
battalion was in the
hands of the “Soldiers’ Councils”. Shortly thereafter the Communist regime was
overthrown, Hitler began what
he terms his “first more or less political activity”.

. Page 4 of 40
Apparently Hitler’s service on this occasion was considered valuable enough to lead
the Army to give him further
employment. He was assigned to a job in the Press and News Bureau of the Political
Department of the Army’s
district command. The German army, contrary to its traditions, was now deep in
politics, especially in Bavaria,
where at last it had established a government to its liking. To further its
conservative views, it gave the soldiers
courses of “political instruction”, in one of which Adolf Hitler was an attentive
pupil. One day, according to his
own story, he intervened during a lecture in which someone had said a good word for
the Jews. His anti-Semitic
harangue apparently so pleased his superior officers that he was soon posted to a
Munich regiment as an
educational officer, a ‘Bildungsoffizier’, whose main task was to combat dangerous
ideas – pacifism, socialism,
democracy; such was the Army’s conception of its role in the democratic Republic it
had sworn to serve.
This was an important break for Hitler, the first recognition he had won in the
field of politics he was now trying
to enter.
6. Going by the passage, which of the following is one of Hitler’s ideologies that
led him to rise in power?
(a) Anti-Semitism (b) Democracy (c) Socialism (d) Pacifism
7. Which of the following statements is NOT true about the armed free corps?
(a) They sprang up all over Germany and were secretly equipped by the Reichswehr.
(b) They overthrew the Hoffman socialist government and installed a right wing
regime.
(c) Initially, they were used to fight the Poles and the Balts on the disputed
eastern frontier.
(d) One of the Armed free corps, led by Captain Ehrhardt, occupied Berlin.
8. Which among the following ultimately represented the Right wing in Bavaria as
political chaos prevailed?
(a) The monarchists.
(b) The Reichswehr.
(c) The great mob of demobilized soldiers.
(d) Conservatives.
9. As inferred from the passage, the German Army promoted its conservative views
(a) through press and news bureau.
(b) by giving courses in political instruction for soldiers.
(c) by encouraging revolutions to fulfil its desire to establish revolutions as a
permanent condition.
(d) by setting up an authoritarian regime.
10. ‘Armed free-corps bands (______) all over Germany and were secretly equipped by
the Reichswehr.’ Which of
the following expression will fill in the blank accurately to make the sentence
coherent?
(a) Beefed up. (b) Sprang up. (c) Pulled up. (d) Eked out.
Passage (Q.11-Q.15): What characterized Kautilya’s Bharat was true of British India
as we shall see
subsequently. But as a comment on the Indian state, society and kingship, we can
venture a generalized
conclusion. India’s strategic culture got internalized, remained fixated upon
curbing within rather than combating
the external, and created a yawning chasm of mutual suspicion between the state and
the citizen. This signal
failure - the establishment of a confident, viable and efficient Indian state,
nourished by effective institutional
instruments, and sustained by a willing and cooperative citizenry - has become a
political and cultural trait; it
both proscribes the form and constricts the functioning of the Indian state, even
today. In the process, it has
prevented India from developing its true power.
The ethos of the Indian state was crippled by another great failing. Not just
occasional, often an excessive, and
at times ersatz pacifism, both internal and external, has twisted India’s strategic
culture into all kinds of
absurdities. Many influences have contributed to this: an accommodative and
forgiving Hindu milieu; successive

. Page 5 of 40
Jain, Buddhist, and later Vaishnav-Bhakti influences resulting in excessive piety
and, much later, in the twentieth
century, ahimsa or non-violence.
The remarkable aspect is not that this pacifist thought developed, but that,
despite it, so much else about warfare
as an instrument of policy about the craft of war, about valour and heroism
remained. That despite the combined
cultural influences of such pacifist faiths, Islamic conquest of parts of India
needed many centuries of strife; that
even at the height of its glory and spread, the Mughal Empire did not encompass the
whole of India, and that it
was in reality not so much a ‘Mughal’ empire as a political-military alliance, a
coalition of the principal Rajput
feudatories of the period and the Mughals.
Even as India and its many constituents were each of them separately self-governed,
also well governed and
prosperous, the country at large assured in the steady functioning of a highly
developed civilization and culture,
where art and science and music and literature flourished, yet they failed to
‘serve for the national and political
unification of India and failed in the end to secure it against foreign invasion,
the disruption of its institutions
and an age of long servitude’.
India has never been nationally and politically one. As a constituent of strategic
culture, a sense of history, of a
recording of it, evaluating and assessing it, and then utilizing it as an input in
decision-making has not been there.
This absence of a written historical account of India can be variously explained:
on account of a lack of unity,
there being no one India; that the Indian tradition is more oral; that religious
texts, in any event have always had
greater merit. No matter what the causes, the consequence of this absence (of a
sense of history) has significantly
affected the development of India’s strategic thought. Whereas ancient Indian texts
on every conceivable subject
abound – on art and dance and drama, most abundantly on philosophy – there are
none, other than Kautilya, that
have detailed the military science of India. There is another factor: of geography,
a sense of territory. Both were
absent: a territorial consciousness, and a strategic sense about the protection of
the territory of residence.
11. ‘The remarkable aspect is not that this pacifist thought developed, but that,
despite it, so much else about warfare
as an instrument of policy about the craft of war, about valour and heroism
remained.’ Which of the following
reflects the figure of speech for the given situation?
(a) Caesura (b) Satirical. (c) Paradoxical. (d) Ethos.
12. According to the passage, all the following are true, EXCEPT:
(a) In spite of the pacifist faith, Islamic conquest over parts of India needed
many centuries of strife.
(b) The Mughal empire encompassed the whole of India.
(c) The political-military alliance was between the Rajput feudatories and the
Mughals.
(d) An accommodative and forgiving Hindu milieu; successive Jain, Buddhist, and
later Vaishnav-Bhakti
influences resulted in inordinate piety.
13. The author holds that
(a) the absence of a written historical account of India has affected the
development of its strategic thought.
(b) Indians feared the loss of their culture and civilization.
(c) the absence of history can be attributed to Kautilya and his military science.
(d) there was an excess of territorial consciousness, and a strategic sense about
the protection of the territory of
residence.
14. Which of the following best reflects the title of the passage?
(a) Defending India.
(b) The ethos of the Indian state.
(c) Absence of territorial consciousness: a historical account.
(d) Kautilya’s Bharat.

. Page 6 of 40
15. Which of the following reflects best the meaning of the word ‘yawning’ in the
context of the passage?
(a) Sighing. (b) stretching. (c) Drowsy. (d) Deepening.
Passage (Q.16-Q.20): We’re at a major tipping point in education. According to a
recent survey, 48% of teachers
admitted that they had considered quitting within the last 30 days. Of that number,
34% said they were thinking
about leaving the profession entirely. Understaffing has plagued schools for years,
but it’s now reaching epic
proportions. Teachers and administrators alike are stressed, overworked and at the
end of their rope. After the
tremendous pressures of the past two years, they have nothing more to give. They
are already giving
everything—time, energy, mental wellbeing, and heart. They’re beyond tired. They’re
exhausted. Conditions in
the education field have always trended toward demanding, but today they’re a
recipe for burnout—which
teachers experience almost twice as much as other government employees.
At the same time, teachers are very hard to replace. The specialization and
requirements inherent to the field of
education make it extremely difficult to expand the talent pool, as other fields
are often able to do. In order to
reach and teach students effectively, teachers must forge a human connection with
them. Today’s younger
generations simply will not move forward in their education and career journey
without that connection. This is
a non-negotiable; it’s just who they are. The vast majority of teachers truly want
to forge that meaningful
connection with students. In fact, for many, it was the driving force behind their
decision to enter the profession.
But, understaffed and overworked as they are, many simply have no time to show
students that they see, hear,
and care about them. Survival mode—where many teachers have lived for the past two
years—doesn’t allow
much room for relationship building. This creates a vicious cycle. Students aren’t
performing, so more burdens
are placed on teachers to help students hit the mark, thus decreasing teachers’
time and bandwidth to forge a
human connection with students that is the basis for all learning. Teachers’ legs
are cut out from under them, yet
they’re still expected to carry their students across the finish line. It’s a
gridlock.
Teachers and students spend hours every day together—and unfortunately, stress
isn’t easy to hide. Even teachers
that don’t actually mention their stress to the class manifest it in a thousand
small ways that young people can
observe. The elevated and prolonged levels of teacher stress are warning the next
generation not to become
teachers. Who wants that kind of life, for that kind of money? Obviously, fewer
people entering education will
only exacerbate the teacher and substitute shortage for the long term. The skills
gap is going to hit the education
field hard. There just won’t be enough teachers to go around. As current educators
flee the profession and the
next generation avoids entering it, we may see class sizes skyrocket—further
straining the teachers that remain.
Without a healthy student-teacher ratio, the quality of instruction, the individual
time spent with each student,
and any vestige of a human connection will inevitably drop.
16. Elaborate the ‘vicious circle’ in teaching that the author talks about?
(a) The stress of teachers is at an all-time high, and it's scaring the next
generation away from becoming
educators and this circle will continue as long as the repugnancy exists in the
profession.
(b) Teachers fail to form a relationship with students and therefore, students
can’t perform well and again, more
pressure is placed on the students to meet their goals, reducing the student’s time
to establish the personal
connection with teachers.
(c) Due to overwork and less staff, teachers fail to form a relationship with
students and hence, students can’t
perform well and again, more pressure is placed on teachers to help them meet their
goals, reducing teachers'
time to establish the personal connection with students.
(d) A sequence of reciprocal cause and effect in which two or more elements
intensify and aggravate each other,
leading inexorably to a worsening of the situation.

. Page 7 of 40
17. Which one of the following statements among the following is NOT meant by the
author?
(a) The stress of teachers is at an all-time high, and it's scaring the next
generation away from becoming
educators.
(b) The education industry will be badly hurt by the skills gap and there will
simply be insufficient teachers to
go around.
(c) Some teachers are extremely capable of masquerading their stress from their
students.
(d) The quality of education, personal time spent with each pupil, and any
semblance of a human connection
would all suffer if there isn't a good student-teacher ratio.
18. Which of the accompanying enlightens the fact that the profession of teaching
is arduous to substitute?
(a) Forming a connection with the students and then imbibing the values of a
particular subject is a tough nut to
crack.
(b) It is extremely unlikely to extend the talent pool in teaching unlike the other
fields because the expertise and
criteria are intrinsic in the general education curriculum.
(c) Without connection, today's younger generations will be unable to progress in
their studies and careers.
(d) Becoming a teacher calls for subject matter expertise and also, a huge amount
of monetary support and
patience is indispensable that most people can’t afford in the modern world.
19. Which of the accompanying is NOT in alignment with the idea of the passage?
(a) Staff shortages have been a problem in schools for a long time, but it has now
reached monumental levels.
(b) Teachers are extremely exhausted as they have given their all—their time,
energy, mental health, and hearts.
(c) Most of the teachers and administrators are anxious, overworked, and at their
wit's end.
(d) Teaching is supposed to be a sinecure profession, but teachers are almost as
busy as other government
employees.
20. What is the tone of the author in the passage?
(a) Cautioning and Commiserating. (b) Cautioning and gloomy.
(c) Caustic and deriding. (d) Condescending and berating.
Passage (Q.21-Q.25): Like many negative emotions, regret is often seen as a purely
undesirable feeling – one
that we should quash whenever possible. Psychologists, however, have shown that it
can be an eminently useful
emotion. “It would be a very, very bad idea, I think, to eliminate regrets in your
life,” says Aidan Feeney, a
professor of psychology at Queen’s University Belfast. “It’s one mechanism for
learning how to improve your
decision-making – a signal that maybe you need to rethink your strategy.” Regret is
a complex emotion since it
involves counter-factual thinking, he points out. It requires the capacity to
imagine alternative courses for events
that have already happened and the capacity to compare and contrast those different
outcomes to determine
which you would have preferred. Due to this complexity, young children are often
unable to feel regret, and the
emotion tends to emerge around age six or seven.
Feeney’s research has tested how emotion is essential for developing an
understanding of delayed gratification
– our ability to put off a small reward now for a greater reward later. Working
with Teresa McCormack, he
presented a group of six-to-seven-year-olds with two boxes. The boxes were equipped
with a timed lock, with
one set to open after 30 seconds and the other after 10 minutes. (Sand timers
placed beside each box showed the
children how long they would have to wait for it to unlock.) The children were told
they could choose to pick
one box to gain their prize. This task was a bit unfair since the children didn’t
know what each box contained,
meaning that most opted for the one that opened first, which contained two candies.
Only after they had made
their decision, they were told that if they had waited for the other box to open,
they could have had four candies
instead – doubling their prize. After the children had learnt this fact, the team
tested whether they felt any regret
for having made the wrong decision. The following day, the psychologists then
presented the children with the
same task again. They found that the children who had developed a sense of regret
were much more likely to
wait for the bigger reward, compared to the children who did not yet entertain the
emotion.

. Page 8 of 40
Regret, it seems, helped them to become more patient so they could subdue the
temptation to go with the
immediate pleasure. The delayed gratification of this kind is essential to form
self-control and is thought to be
very important for people’s success in life. If you can put off the pleasure of
playing a computer game to study
for exams, for example, you are more likely to get a place at a good university,
which will, in turn, lead to more
stable finances for the future. Regret’s fundamental role in our cognition may
explain why so many people
experience it so frequently. Pink points to one study, from 1984, that examined the
conversations of
undergraduate and married couples. Within these recordings, regret was the second
most discussed emotion after
love.
21. Which of the accompanying is NOT a good effect of regret as concluded by Feeney
after he had experimented
with a group of six-to-seven-year-olds with two boxes?
(a) Building of self-control, owing to delayed gratification.
(b) Resisting the impulse to pursue immediate gratification.
(c) The ability to be more attentive can be imbibed.
(d) Regret helped them to build patience.
22. Which of the accompanying is the conclusion that the author wants us to draw
from the fact that after love, regret
is the second most discussed emotion?
(a) Regret is widely perceived as an unpleasant emotion that should be avoided at
all costs.
(b) Feeney's study has looked at how emotion plays a role in learning about
deferred rewards.
(c) The significance of regret in human cognition.
(d) Regret assisted the children in being more patient, allowing them to resist the
impulse to pursue immediate
gratification.
23. Which one of the following can serve as a perfect title to the passage?
(a) The complex case of regretting.
(b) Children’s brain programming to elude regretting.
(c) Regret-the unnecessary evil.
(d) Not regretting means not growing up.
24. Why did some of the children feel the complex emotion of regret after opening
the first box?
(a) After opening the first box, the children were informed that if they awaited
the other box’s opening, they
could’ve received four chocolates instead of two.
(b) None of them regretted it because regret is a difficult emotion for young
children to feel, and it usually
appears around the age of six or seven.
(c) After opening the first box, the children found out that it was empty and the
second box contained four
chocolates.
(d) After opening the first box, the children found out that the other children who
had waited for the second box
to open were given chocolates and they weren’t given any.
25. Which of the accompanying can NOT be induced as a view that Aidan Feeney will
most certainly concur with?
(a) As regret includes counter-factual thinking, it can be termed as a complicated
emotion.
(b) Regret is frequently viewed as an unpleasant emotion that should be avoided at
all costs.
(c) Regretting is one of the techniques of enhancing your judgmental abilities.
(d) It is a lousy idea to erase regrets from one's life.

. Page 9 of 40
Passage (Q.26-Q.30): There is something eerily prescient about Cut Piece, the 1964
performance art piece by
Yoko Ono, in which the artist sat silent and motionless, with a pair of scissors
before her, allowing members of
the audience to do what they wanted. Some were hesitant, snipping off small pieces
of her clothes, while others
were bolder, slashing through large chunks of fabric. It reflected the strong
strain of violence and sadism that
lies, barely contained, beneath the surface of even the most genteel, art show-
going audience, just like, three
years later, the vitriol that Ono drew for allegedly breaking up The Beatles would
bare the toxic, misogynistic
side of rock and roll fandom.
The myth that Ono is to blame for the demise of one of the greatest musical acts of
all time was born in 1970
and it’s taken half a century for it to finally be laid to rest. In a three-part,
eight-hour-long documentary, Get
Back, filmmaker Peter Jackson uses footage from the three-week period when The
Beatles wrote and rehearsed
the songs that would eventually make it onto Let It Be to draw a portrait of a
group of musicians who are clearly
drifting apart, with each individual keen to pursue his own artistic vision.
Perhaps it had to do as much with her race, as with her gender, but almost from the
start, Ono was singled out
for the fandom’s ire for the Beatles’ breakup. The documentary shows that she was
only one of the many guests
— which included Linda Eastman (Paul McCartney’s wife-to-be), Maureen Starkey
(Ringo Starr’s wife), Pattie
Boyd (George Harrison’s wife) and Shyamsunder Das (Harrison’s friend) — at the
sessions. Yet, Ono was the
one described as “dragon lady”, her name passing into the cultural lexicon as the
term used to describe any
woman who is seen as controlling or interfering. The world owes Ono an apology.
26. Which of the following best reflects the interpretation of the expression
‘eerily prescient’?
(a) Spookily clairvoyant. (b) Uncannily perceptive.
(c) Unnervingly divinatory. (d) Cacophonously revelatory.
27. The author, thorough paragraph one indicates that
(a) The 1964 performance art piece by Yoko Ono revealed the underlying aggressive
and unkind streak wrapped
up in a polished mannerism.
(b) The 1964 performance art piece by Yoko Ono was a deliberate attempt by the
artist to bring out the hypocrisy
of the audience.
(c) The reaction of the audience reflects a misplaced and misguided anger vented
out at an innocent victim.
(d) The reaction of the audience reflects a deep-seated hostility for Yoko Ono, in
specific and for women in
general.
28. All of the following can be inferred from the passage, except?
(a) The author believes that accusation on Yoko Ono that she is responsible for the
disbanding of the Beatles is
baseless.
(b) Yoko Ono suffered the viciousness of the fans of the Beatles.
(c) ‘Dragon Lady’ is eponymous to officious women.
(d) Targeting Yoko Ono was a random act because of her race and gender.
29. Which of the following reflects the tone and attitude of the author towards
Yoko Ono?
(a) Distrustful and Armoured. (b) Justifying and approval- seeking.
(c) Assertive and defensive. (d) Friendly and humane.
30. What is the central idea behind the passage?
(a) To highlight the three-part, eight-hour-long documentary, Get Back from
filmmaker Peter Jackson that lays
to rest the controversy surrounding the break-up of the Beatles.
(b) To highlight the underlying racial discrimination and misogynistic conduct
among people that is portrayed
on innocent people.
(c) The author demanding a public apology from the fans of Beatles for a malicious
ranting vented at Yoko Ono
for her alleged breakup of the Beatles.
(d) To bring to fore the mistreatment meted out to Yoko Ono for a baseless
allegation of her breaking up the
Beatles.
Passage (Q.66-Q.70): In a landmark order, the High Court of Tripura ordered the
police to refrain from
prosecuting a man who was earlier arrested over a social media post. Court also
barred the police from making
any further arrest in connection with the case.
The order was passed by the Chief Justice on January 10, 2020 after lawyers moved
the court against the arrest
and harassment of a Congress youth activist Arindam Bhattacharjee over his post on
a social media platform
criticising the Bharatiya Janata Party’s online campaign in support of the
Citizenship Amendment Act (CAA).
The Chief Justice in his order broadly remarked that posting on social media was
tantamount to a “fundamental
right” applicable to all citizens, including government employees.
The order of the High Court came within days of the Supreme Court adding legal heft
to freedom of speech and
expression in its ruling on the Kashmir internet shutdown. In its order (Anuradha
Bhasin v. Union of India), the
Supreme Court recognized that “freedom of speech and expression of the citizens...
over the medium of internet”
is protected under Article 19 of the Indian Constitution. It ruled that an
indefinite suspension of internet services
would be illegal under Indian law as orders for internet shutdown must satisfy the
tests of necessity and
proportionality. The Court held that though the Government was empowered to impose
a complete internet
shutdown, any order(s) imposing such restrictions had to be made in public interest
and was subject to judicial
review.
The Tripura High Court heard another petition filed by one Lipika Paul, who had
been suspended from the state
fisheries department. The Court ruled:
“As a Government servant the petitioner is not devoid of her right of free speech,
a fundamental right which can
be curtailed only by a valid law. She was entitled to hold her own beliefs and
express them in the manner she
desired, of course subject to not crossing the borders laid down in sub-rule (4) of
Rule 5 of the Conduct Rules.”
The court then set aside the suspension order and directed the government to
release Paul her retiral benefits
within two months.
[Source: “After SC, how Tripura High Court added muscle to freedom of speech &
expression”, The Print,
https://theprint.in/judiciary/after-sc-how-tripura-high-court-added-muscle-to-
freedom-of- speech#expression/348902/ ; “Social media posting is a fundamental
right, rules Tripura High Court”, The Hindu,
https://www.thehindu.com/news/national/social-media-posting-is-a-fundamental-right-
rules-tripura-high#court/article30551332.ece and “Bhasin v. Union of India”, Global
Freedom of Expression, Columbia
University, https://globalfreedomofexpression.columbia.edu/cases/bhasin-v-union-of-
india/ ]
66. The State of Emergency faced a spate of communal riots. Upon investigation, it
was found that there were several
groups on a popular internet messaging application, GitHub, that played a role in
instigating people. It urged
people to resort to violent means to teach a lesson to the minority community in
their respective neighbourhoods.
As a result, the State of Emergency shut down the internet services for an
indefinite period to prevent any such
further incidents like this. John, a resident of a locality in Emergency filed a
writ petition before the Supreme
Court stating that the indefinite shutdown of internet services violates his
freedom of speech and expression
under Article 19 of the Constitution. Which of the following best characterises the
most likely Supreme Court
judgement, according to your interpretation of the passage?
(a) It will direct the State of Emergency to do away with the indefinite suspension
of internet services.
(b) It will direct the operators of Github to monitor the content being circulated
on their messaging service.
(c) It will dismiss John’s writ petition as he did not show any actual damage done
to him because of the internet
shutdown.
(d) It will dismiss John’s writ petition as the Supreme Court does not have the
power to review such a decision
of the State

. Page 18 of 40
67. Taking into account the factual situation described in the above question, the
order for the shutdown of internet
services in the State of Emergency was to operate for a period of one month, after
which the services would be
resumed in their full capacity. In such a scenario also, John decided to file a
writ petition challenging the order
of the government as being violative of his fundamental right of speech and
expression under Article 19 of the
Constitution. Which of the following best characterises the most likely Supreme
Court judgement, according to
your interpretation of the passage? (Consider that all the laws applicable to India
are applicable to the State of
Emergency, and it has adopted the same Constitution)
(a) It will hold the shutdown of internet services as illegal and uphold John’s
right to freedom of speech and
expression.
(b) It will uphold the government’s power to impose such a restriction in the State
of Emergency.
(c) It will dismiss John’s petition as being frivolous and impose costs.
(d) It will direct the State to ensure that no such shutdown is imposed in the
future.
68. Anmol was a state government employee, working with the anti-corruption bureau.
While he was in service, he
noticed that a few of his colleagues were in the habit of accepting bribes in order
to not prosecute people on
charges of corruption. He did not object to such a practise but also never
practised it. However, once he retired
from service, he became very active on social media. He joined an online campaign
called “India Fights
Corruption”, where he was made in charge of the content developing team. He started
regularly posting the
corrupt practices at his former place of employment, although never naming any
official or colleague. One day,
he was arrested by the police for defaming the office of the anti-corruption
bureau. Does this amount to a
violation of his right to freedom of speech and expression?
(a) No, as regular code of conduct for retried officials requires them to not speak
ill about their former places of
employment without necessary sanction.
(b) No, as his fundamental right to speech and expression can be subject to
reasonable restrictions.
(c) Yes, as he had already retired from the place of his employment.
(d) Yes, as posting on social media is a fundamental right of all citizens.
69. Taking into account the factual situation described in the above question,
Anmol joined the online campaign of
“India Fights Corruption” while he was still in service. He posted on social media
about his office, naming it and
the practices prevalent there without naming any of his colleagues or other
officials. He was dismissed from his
position with the reason that his posts were political in nature and brought infamy
to the office. Decide whether
Anmol’s right to freedom of speech and expression have been violated in such a
situation.
(a) No, as he was not entitled to post about his current place of employment.
(b) No, as the official rules did not allow him to bring infamy to his department.
(c) Yes, provided he did not breach any rules of conduct that governed his stay in
the office.
(d) Yes, as the right to express one’s opinion on social media is available to
government employees.
70. Consider that Mark, a delegate from Australia, opted for an Indian work visa
and came on deputation to the same
department where Anmol worked, for a period nine months. Both of them became very
good friends and started
noticing the corrupt practices of Anmol’s colleagues. They both started posting
about such practices on their
social media accounts, criticizing the impunity with which their colleagues worked.
Which of the following
statement(s) is true, as per the passage?
I. Anmol and Mark, both government employees, have the right to post on social
media, which is protected by
Article 19 of the Constitution.
II. Only Anmol is bound by the rules of conduct at his workplace, whereas Mark is
not.
III. Anmol has the absolute freedom to post on social media, without being held
liable for his posts.
(a) Only III
(b) Both II and III
(c) Only I
(d) None of the above

. Page 19 of 40
Passage (Q.71-Q.75): The Supreme Court has held that merely proving the kidnap of a
person is not sufficient
for conviction for the offence of 'kidnapping for ransom' under Section 364A of the
Indian Penal Code. It must
also be proved that there was threat to cause death or harm to the kidnapped person
or the kidnapper, by his
conduct, gave rise to a reasonable apprehension that such person may be put to
death.
The Court held that the essential ingredients to be proved by the prosecution for
proving the offence under
Section 364A IPC are as follows :
(i) Kidnapping or abduction of any person or keeping a person in detention after
such kidnapping or abduction;
and
(ii) threatens to cause death or hurt to such person, or by his conduct gives rise
to a reasonable apprehension that
such person may be put to death or hurt or;
(iii)causes hurt or death to such person in order to compel the Government or any
foreign State or any
Governmental organization or any other person to do or abstain from doing any act
or to pay a ransom.
"Second condition is also a condition precedent, which is requisite to be satisfied
to attract Section 364Aof the
IPC", the Court said.
However, from the evidence on record regarding kidnapping, it is proved that
accused had kidnapped the victim
for ransom, demand of ransom was also proved.“The offence of kidnapping having been
proved, the appellant
deserves to be convicted under Section 363.”
(Source : Kidnapping For Ransom - Necessary To Prove Threat To Cause Death Or Harm
For Conviction Under
Section 364A IPC : Supreme Court, live law)
71. Tarun went to a picnic organised by the school and returned to school at around
3:00 pm. Usually, he would wait
for a regular (fixed) auto to drop him home from school but unfortunately on the
said date, the same did not turn
up. He then took another auto, after being advised the same by his father. The
accused, who was driving the said
auto, took the child to the house of his sister and demanded a ransom of Rs.2 lakhs
from the child’s father. The
accused further threatened to harm the child if his father filed a complaint.
Determine the statutory provisions
under which the accused will be prosecuted.
(a) If the second criterion is met, the accused will be held accountable under
section 346A of the IPC, which is
a prerequisite for triggering section 364a of the IPC.
(b) If it is proven that the accused kidnapped the victim for ransom and that a
demand for ransom was made, the
accused would be held accountable under section 363.
(c) The accused cannot be held accountable under section 364 A since the
prerequisites for attracting the offense
under the said section have not been met a priori.
(d) The accused will be charged with kidnapping for ransom, and not kidnapping or
abducting for the purpose
of murder.
72. Continuing with the same facts as before, what does the prosecution need to
prove in order to convict the accused
of 'kidnapping for ransom' under Section 364A of the Indian Penal Code?
I. Kidnapping or abduction with apprehension that there was threat to cause death
or harm to the kidnapped
person.
II. Kidnapping for ransom where the kidnapper, by his conduct, gave rise to a
reasonable apprehension that
such person may be put to death.
III. Kidnapping or abduction which caused hurt or death to such person in order to
compel the Government or
any foreign State or any Governmental organization or any other person to do or
abstain from doing any act
or to pay a ransom.
IV. All of the above.
(a) I (b) II
(c) EITHER I OR II OR III (d) IV

. Page 20 of 40
73. Kunwar Sen, Dwarki, Ram Charan and Janter Singh came at the house Hem Raj and
took his son Vijay Pal
(deceased) by saying that they were going to Kashipur in search of some job.
Accordingly Vijay Pal accompanied
them. On the next day at around 5:00 pm, when Hem Raj was returning from river
after fishing, he saw that one
slipper of his son was lying near the land of one Bhoopal Jatav. On suspicion, Hem
Raj made search around that
place and found that dead body of his son Vijay Pal was lying in the sugarcane
field of one Sukhey Jatav. There
was sign of pressing his neck on his body and one of his slippers was also lying
nearby. It was alleged by Hemraj
that, his son was murdered by Kunwar Sen, Dwarki, Ram Charan and Janter Singh by
pressing his neck.
Thereafter he filed a complaint against all the 4 accused under section 364 A IPC.
Decide
(a) They will be charged under sections 364, 302/34 of IPC for committing murder of
Vijay pal (deceased) by
abduction.
(b) They will be charged under Section 364A IPC for the offence of 'kidnapping for
ransom'.
(c) They will not be charged under section 364a IPC for the offence of 'kidnapping
for ransom'.
(d) They will be charged under section 363 of IPC.
74. Choose a statement that is/are not correct in light of the passage:
(a) Kidnapping or abduction, which Section 364A IPC envisages, must be with the
intention to commit his
murder or with the knowledge that the person, kidnapped or abducted, would be put
to danger of being
murdered.
(b) if the kidnapping or abduction is with an intention that the person, kidnapped
or abducted, be murdered or if
the person is kidnapped or abducted with the knowledge that the victim is likely to
be put to danger of being
murdered, offence under Section 364A IPC would be complete irrespective of the fact
as to whether the
victim is, eventually or actually, murdered or not.
(c) It is explicit that kidnapping or abduction is a basic and necessary ingredient
to attract penal provisions of
Section 364A IPC
(d) If there was no such objective when the abduction was perpetrated, but later
the abductors murdered the
victim, Section 364A IPC Would be attracted.
75. Assertion: ‘A’ and ‘B’ are brothers. ‘A’ wanted to marry ‘C’, but she did not
want to. ‘A’ asked ‘B’ to abduct
‘C’ so that he can marry her. ‘B’ did as was asked from him and took ‘C’ from her
house to ‘A’. Here ‘B’ is
guilty of the offence under section 364A of IPC as he abducted a woman, ‘C’ with
the knowledge that would be
compelled into marriage.
Reason: It is explicit that kidnapping or abduction is a basic and necessary
ingredient to attract penal provisions
of Section 364A IPC.
(a) Both A and R are true and R is correct explanation of A.
(b) Both A and R are true but R is not correct explanation of A
(c) A is true but R is false.
(d) A is false but R is true.
Passage (Q.76-Q.80): The Supreme Court observed that the proof of demand of bribe
by a public servant and
its acceptance by him is sine quo non for establishing the offence under Section 7
of the Prevention of Corruption
Act.
The failure of the prosecution to prove the demand for illegal gratification would
be fatal and mere recovery of
the amount from the person accused of the offence under Section 7 or 13 of the
Prevention of Corruption Act
would not entail his conviction there under, the bench comprising noted.
The bench noted the following observations made into the judgment in P.
Satyanarayana Murthy v. District
Inspector of Police, State of Andhra Pradesh and another, the court said:
The proof of demand of illegal gratification, thus, is the gravamen of the offence
under Sections 7 and 13(1) (d)
(i) and (ii) of the Prevention of Corruption Act and in absence thereof,
unmistakably the charge therefore, would
fail. Though obtainment is important ingredient under section 13. Mere acceptance
of any amount allegedly by
way of illegal gratification or recovery thereof, dehors the proof of demand, ipso
facto, would thus not be
sufficient to bring home the charge under these two sections of the Act. As a
corollary, failure of the prosecution

. Page 21 of 40
to prove the demand for illegal gratification would be fatal and mere recovery of
the amount from the person
accused of the offence under Section 7 or 13 of the Prevention of Corruption Act
would not entail his conviction
there under."
(SOURCE: PC Act - Mere Acceptance of Amount, Without Proof Of Bribe Demand, Will
Not Establish Offence
Under Section 7: Supreme Court, LIVELAW)
76. Nishil recently relocated to Bhopal from London after completing his master's
degree. To enrol as a driving
licence holder of Indian nationality, he had to pass the driving test. When Nishil
reached at the RTO office, he
was astounded to discover that there is no system. People are being awarded with
their driving licences by
the RTO as if they were selling bananas. Nishil decided to conduct a sting
operation on one of the officers, and
he attempted to give bribe. The officer took the money, believing it to be for
something else. Can the officer be
held responsible under Section 7 of the Prevention of Corruption Act?
(a) Yes the officer can be held responsible as he took the money.
(b) No, the officer is not to blame because it was simply a sting operation.
(c) Yes the officer is responsible as acceptance of bribe is sine quo non for
establishing the offence under Section
7 of the Prevention of Corruption Act.
(d) No, the officer cannot be held liable as mere recovery of amount from the
person accused of the offence
would not entail his conviction there under.
77. Continuing with the same facts as the last one, suppose Nishil hired an agent
who will deliver his license to his
residence. The agent has a tie with the RTO office, and on demand of same, he
bribes the officer to obtain the
license as soon as possible because it takes up to 2 to 3 months to prepare the
same. Nishil’s license is delivered
on the same day. Decide
(a) RTO agent will be held liable under Section 7 of the Prevention of Corruption
Act for bribing the officer.
(b) Officer who took the bribe can be held liable under section under Section 7 of
the Prevention of Corruption
Act.
(c) Nishil will be held accountable for triggering the trend of bribery.
(d) Officer will not be held liable because there is no proof of demand of illegal
gratification.
78. Choose an anomaly of the context of the passage:
I. Recovery of tented money is not sufficient to sustain the charges unless and
until the recovery is preceded
with demand of illegal gratification and acceptance of the same.
II. Filing of cases without gratifying the essential mandated ingredients of
statutory provision is completely
illegal and unjustified.
III. The evidence regarding the demand and acceptance of a bribe leaves room for
doubt and does not displace
wholly, the presumption of innocence, the charge cannot be said to have been
established.
IV. Element of obtainment is the prime ingredient for Section 13.
V. None of the above
(a) V (b) II & III (c) I & IV (d) II & IV
79. Consider the following scenarios and select the one in which the offence under
Section 7 of the PC Act does not
apply:
(a) Ravi gifts Mukesh, a public servant, a house in Andheri in return for Mukesh
giving fast approval to Ravi's
building construction project.
(b) Mukesh is a tax assessment officer. Ravi gives Mukesh's sons admission for free
in the school run by Ravi's
family. This is so that Mukesh does not fine Ravi for failing to pay his income
tax. But he anyhow charged
Ravi with the penalties.
(c) Mukesh, a municipality officer is paid Rs. 10,000 by Ravi so that Mukesh will
award the project for building
a road in the locality to Ravi's company.
(d) All of the above

. Page 22 of 40
80. Assertion : Mere acceptance of any amount allegedly by way of illegal
gratification or recovery thereof, dehors
the proof of demand.
Reason : Prosecution should have to prove beyond reasonable doubt the acceptance of
illegal gratification.
(a) Both A and R are false and R is not correct explanation of A.
(b) Both A and R are true and R is correct explanation of A.
(c) A is true but R is false.
(d) A is false but R is true.
Passage (Q.81-Q.85): While explaining that to prove an offence under Section 306,
the Prosecution must satisfy
the ingredients of Section 107 first, Justice Sandeep N Bhatt of the Gujarat High
Court refused to interfere with
the impugned judgement and quash the order of acquittal. While doing so, the Bench
delved into the terms
'abetment' and 'instigation' under Sections 306 and 107 of the IPC, at length.
Section 306. of the Indian Penal Code defines 'Abetment of suicide' as an offence
punishable under this code.
If any person commits suicide, whoever abets the commission of such suicide, shall
be punished with
imprisonment of either description for a term which may extend to ten years, and
shall also be liable to fine.
S. 107 Abetment of a thing.—A person abets the doing of a thing, who—
(First) — Instigates any person to do that thing; or
(Secondly) —Engages with one or more other person or persons in any conspiracy for
the doing of that thing, if
an act or illegal omission takes place in pursuance of that conspiracy, and in
order to the doing of that thing; or
(Thirdly) — Intentionally aids, by any act or illegal omission, the doing of that
thing. Explanation 1.—A person
who, by wilful misrepresentation, or by wilful concealment of a material fact which
he is bound to disclose,
voluntarily causes or procures, or attempts to cause or procure, a thing to be
done, is said to instigate the doing
of that thing.
It cannot be said that the abusive language, which had been used by the appellant
on 25.07.1998 drove the
deceased to commit suicide. Additionally, the Court referred to Ramesh Babulal
Doshi vs. State of Gujarat
(1996) 9 SCC 225) to affirm that the Appellate Court cannot substitute its own view
by reversing the acquittal
into conviction and this is a cardinal principal of criminal jurisprudence. (1996)
9 SCC 225) Ramesh Babulal
Doshi V. State of Gujarat.
"When the trial Court renders its decision by acquitting the accused, presumption
of innocence gathers strength
before the Appellate Court. As a consequence, the onus on the prosecution becomes
more burdensome as
there is a double presumption of innocence."
Accordingly, the Bench dismissed the Appeal.
(SOURCE: Prosecution Must Satisfy Section 107 Requirements To Prove An Offence
Under Section 306 Of
IPC: Gujarat High Court)
81. Chavi was well aware that Radha was madly in love with him and will do
everything he ask. They took a short
trip to Lonavala one time. On the way, Chavi asked Radha what crazy things she
could do in love, and Radha
replied, "If you ask me to jump down a hill, I'll do that for you." When Chavi
reached the top, he inadvertently
asked Radha to jump in a funny way. Radha took a leap. Is Chavi liable under
Section 306 of the IPC?
(a) There was no intention on part of Chavi, hence, not liable.
(b) First condition of S. 107 attracts Chavi’s liability under section 306.
(c) Chavi abetted Radha to jump down the hill, hence liable.
(d) Chavi will be held liable if prosecution satisfies the ingredients of Section
107 first.

. Page 23 of 40
82. Sanju is a well-known Bollywood actor from the 1990s. He met Jim at a private
party, and there he introduced
Sanju to the world of drugs. Sanju was also going through a rough patch in his
life, so he turned to drugs for
solace. Sanju was sent to rehab after his father became concerned that his son had
become an addict. Doctors
warned Sanju that if he didn't quit, he would die. Jim secretly went to the rehab
Centre and gave Sanju the drug
in the form of a brownie. Sanju died as a result of a drug overdose. What crime
will Jim be charged with?
(a) Jim will be charged under Section 306 read in conjunction with Section 107 of
the IPC.
(b) Jim will be charged with instigating, aiding and abetting under Section 107 of
the IPC.
(c) Jim will not face any charges because all he did was start giving Sanju some
drugs.
(d) Jim will not be held liable because he was unaware of Sanju's critical
condition.
83. Government of India launched a helpline number for those who suffered from
severe anxiety and depression the
term of COVID. Rahul feeling anxious and depressed one day as he was living alone
for a period of 8 month
and had no one to take care called the helpline. He asked for help as he was having
suicide thoughts and needed
to talk to someone. Talking to helpline assistant did not helped and even escalated
the situation to worse by
stating that you are of no use, you should die. Rahul tried committing suicide by
drinking a bottle of whitener
placed on his study table and died. Decide who will be held liable?
(a) Helpline assistant will be held liable for instigating rahul to take such
decision.
(b) Rahul will be held liable if the attempt was not successful.
(c) Can’t be determined.
(d) Government will held vicariously liable for the acts of his servants in
instigating rahul to attempt suicide.
84. Author mentions in its last Para that “When the trial Court renders its
decision by acquitting the accused,
presumption of innocence gathers strength before the Appellate Court. As a
consequence, the onus on the
prosecution becomes more burdensome as there is a double presumption of innocence."
Which other
presumption of innocence is referred to by the court?
(a) There shall be no such presumption of innocence once a person is convicted by a
trial court.
(b) An accused is innocent until proven guilty.
(c) An accused is guilty until proven innocent.
(d) Presumption of innocence.
85. Assertion: To prove an offence under Section 306, the ingredients of Section
306 and Section 107 must be
considered because they are not mutually exclusive events.
Reason: Prosecution Must Satisfy Section 107 Requirements To Prove An Offence Under
Section 306 Of IPC.
(a) Both A and R are true but R is not correct explanation of A.
(b) Both A and R are true and R is correct explanation of A.
(c) A is true but R is false.
(d) A is false but R is true.
Passage (Q.86-Q.90): The Bench comprising Chief Justice Arvind Kumar and Justice
Ashutosh Shastri recalled
the observations made by a previous Bench in this regard:
"The debate as regards the rights of encroachers over public land vis a vis the
right to shelter should come to an
end. The right to shelter and encroachment are two different facet. An encroacher
may save himself from being
forcibly evicted only if during his period of stay over the encroached public land
any enforceable legal right has
crystallized in his favour. Otherwise, merely by asserting the Right to Shelter ,
an encroacher, over public land,
cannot say that he cannot be evicted."
The Bench referred to Ahmedabad Municipal Corporation vs. Nawabkhan Gulabkhan and
others [AIR 1977 SC
152] to conclude:
"If the encroachment is of a recent origin the need to follow the procedure of
principle of natural justice could
be obviated in that no one has a right to encroach upon the public property and
claim the procedure of opportunity

. Page 24 of 40
of hearing which would be a tedious and time-consuming process leading to putting a
premium for high handed
and unauthorised acts of encroachment and unlawful squatting. On the other hand, if
the Corporation allows
settlement of encroachers of a long time for reasons best known to them, and
reasons are not far to see, then
necessarily a modicum of reasonable notice for removal, say two weeks or 10 days,
and personal service on the
encroachers or substituted service by fixing notice on the property is necessary."
Thus, if situation falls other than these condition then it was for the Court to
decide in exercise of its constitutional
powers whether the deprivation of life or personal liberty is by procedure, which
is just, fair and reasonable.
Keeping in view, these facts and precedents, Court dismissed the Petition with the
permission to the Petitioner
to avail any other remedy with State authorities.
(Source: Bandhkaam Mazdoor Sangathan vs State Of Gujarat
Case citation: 2022 LiveLaw (Guj) 26)
86. The retired government employees were allowed to continue residing in their
government allotted residential
apartments under a state-sponsored rehabilitation scheme. Under this scheme, the
State was rehabilitating all
those people who were victims of terrorist violence and it had allowed the retired
government officials to stay in
the government residences till they were rehabilitated under the scheme. What will
be the case in an event of
eviction?
(a) The retired government has provided the accommodation under the scheme of right
to shelter.
(b) The retired government officers are granted accommodation under a state-
sponsored rehabilitation scheme
and the same can be enforced as a legal right.
(c) The retired government officer cannot claim right to shelter over the
encroached public land until an
enforceable legal right has crystallized in his favour.
(d) The retired government officer cannot claim right to shelter over the
government allotted residential
apartments as it was allotted under a state-sponsored rehabilitation scheme.
87. In which of the following situations the principles of natural justice be
waived in the event of an encroachment
eviction:
(a) It will be the discretion of the court to decide in exercise of its
constitutional powers whether the deprivation
of life or personal liberty is by procedure, which is just, fair and reasonable or
otherwise.
(b) It can be waived off in the event where the encroachment is of a recent origin.
(c) It can be waived off in an event where he Corporation allows settlement of
encroachers.
(d) It can be obviated if during the period of stay over the encroached public land
no enforceable legal right has
crystallized in his favour.
88. Slum dwellers who were evicted as encroachers from the roads outside the
railway station were promised a new
shelter. Even after repeated reminders, the state did not provide the Dwellers with
a place to settle. Can dwellers
claim from the state the right of encroachers in relation to the right to shelter
as promised by the state?
(a) No, since this deprivation of personal liberty is carried out in accordance
with procedure established by law.
(b) Yes, because encroachers are promised shelter by the state, hence dwellers
claim from the state the right of
encroachers in relation to the right to shelter is valid.
(c) No, because no legal right in lieu of dwellers has been coalesced.
(d) No, because the right to shelter and encroachment are two distinct issues.

. Page 25 of 40
89. A trade union working for human rights, had filed a PIL seeking the
rehabilitation and resettlement of slum
dwellers and restraint on Respondent rail authorities from evicting the dwellers
from the slum colony, a
permanent establishment, in pursuit of the construction of Ahmadabad-Mumbai Bullet
Train. What will be ruled
by the court in this case?
(a) Court will allow the petitioners to avail any other remedy with the concerned
authorities.
(b) It will be up to the Court, in exercising its constitutional powers, to
determine whether the deprivation of life
or personal liberty is accomplished through a procedure that is just, fair, and
reasonable or otherwise.
(c) Since the encroachment is not of recent nature, the need to follow the
principle of natural justice and cannot
be obviated.
(d) Dwellers cannot avoid eviction by claiming the right to shelter in their
defense.
90. Assertion: If a person's life or personal liberty is taken away through a
procedure that is not just, fair, or
reasonable, it will be considered arbitrary action.
Reason: It was for the Court to decide in exercise of its constitutional powers
whether the deprivation of life or
personal liberty is by procedure, which is just, fair and reasonable or otherwise.
(a) Both A and R are true but R is not correct explanation of A.
(b) Both A and R are true and R is correct explanation of A.
(c) A is true but R is false.
(d) A is false but R is true.
Passage (Q.91-Q.95): The Section 148-A of Code of Civil Procedure, 1908 talks in
brief about the Caveat
Petition. A Caveat Petition is a precautionary measure which is undertaken by
people usually when they are
having a very strong apprehension that some case is expected to be filed in the
Court of Law, regarding their
interest in any manner.
The word 'Caveat' is not defined in the Code of Civil Procedure, 1908, however, in
the case titled Nirmal Chand
Vs Girindra Narayan, AIR 1978 Calcutta 492, the Hon'ble High Court defined the word
'Caveat' as under;
The term 'caveat' is very common in testamentary proceedings. A caveat is a caution
or warning giving notice to
the Court not to issue any grant or take any step without notice being given to the
party lodging the caveat. It is
a precautionary measure taken against the grant of probate or letters of
administration, as the case may be, by
the person lodging the caveat. Such a person lodging a caveat in probate may not be
a necessary party to such
an application, but he may be affected by an order or have any interest of person
related to him that may be
passed on such an application.
The whole concept and the object behind giving right to a person to lodge caveat in
such a suit or proceedings
before any Court, is to ensure that before any order is passed at the instance of
the person instituting the suit, the
person whose interest is likely to be affected, be given an opportunity of hearing.
In essence, it is intended to
ensure that the person likely to be affected, has a right to oppose the prayer of
the applicant in such suit or
proceedings and that applicant or the plaintiff may not get an ex-parte order in
his favour in the absence of
'Caveator' / 'Applicant' who has an interest in the matter.
The expression in a suit or proceeding instituted or about to be instituted in a
Court occurring under Section
148A of the Code of Civil Procedure, 1908 necessarily relates to civil proceedings
in a Civil Court, as this
expression has to be read in the light of Section 9 of Code of Civil Procedure,
1908, which stipulates that Courts
shall have jurisdiction to try all suits of a civil nature.
(Source: Legalserviceindia.com)
91. The second paragraph discusses testamentary proceedings. What exactly is a
testamentary proceeding, in your
opinion?
(a) A Testamentary Court is only concerned with finding out whether or not the
testator executed the
testamentary instrument of his free will.
(b) In cases of testamentary succession, the Will appoints an Executor to handle
the affairs of the Testator on
death.

. Page 26 of 40
(c) All testamentary cases should be of civil nature.
(d) A civil suit where a Letter of Administration is issued by a competent
authority or a court and appoints the
Administrator officer to dispose of the property of a person.
92. Jhanvi's father is one of the world's wealthiest people, according to Forbes.
She was Mr. Oberoi's only daughter.
Mr Oberoi was aware that he had been receiving regular death threats as a result of
his influential position and
contacts with the underworld. For the same reason, he does not wish to involve his
only daughter, Jhanvi, in his
business. He also advised Jhanvi to start a new business, in which he will invest.
Jhanvi, on the other hand,
refused and said that she would build her own empire. After several years, when Mr
Oberoi's will was read out
to the court for probate, it was revealed that all of his property and assets had
already been transferred to two
charitable organisations. Mr Oberoi's wife filed a lawsuit in court, alleging
suspicion in the will. Jhanvi, who
was married to Suresh, relocated to London following their marriage. In this
situation, what options does Jhanvi
have?
(a) Jhanvi has no recourse because she will no longer be a coparcener because she
declined to be a part of her
father's estate.
(b) Jhanvi has no remedy because she will no longer be a coparcener after her
marriage also not an Indian
resident, hence cannot file a caveat.
(c) Jhanvi can also be a co-plaintiff in her mother's complaint, which is expected
to be filed in the Court of Law,
relating her interest in any way.
(d) Jhanvi may file an application with the court asking the court not to make any
grant or take any action without
giving notice to the party filing the application.
93. Continuing with the preceding facts, for in what feasible circumstances can
Jhanvi file a caveat application?
(a) Was concerned that she would not be made a successor to her father's property.
(b) Concerned that the court could issue probate in her absence.
(c) Issuing a notice to the court not to make any grant or take any action without
first notifying the party who
lodged the caveat.
(d) She has not joined as a party to her mother's application.
94. According to your understanding of the passage’s context, is the application of
caveat mandatory in testamentary
proceeding?
(a) No, it is just a precautionary measure.
(b) It is followed on the pretext of principle of natural justice i.e., the person
whose interest is likely to be
affected, be given an opportunity of hearing.
(c) Yes, as no exparte order can be passed in the absence of 'Caveator' /
'Applicant' who has an interest in the
matter.
(d) No, it is just a precautionary measure and can be filed only in case of a civil
suit.
95. Assertion: A civil suit in which a competent authority or a court issues a
Letter of Administration and appoints
an Administrator officer to dispose of a person's property is referred to as a
grant of probate and thus a civil suit.
In such a circumstance, a caveat can be filed by any person, even if they have no
interest in the case.
Reason: A person filing a caveat is not required to be a party to such an
application, but he may be impacted by
any order issued in response to such an application. The phrase in a suit or
proceeding instituted or about to be
instituted in a Court, as used in Section 148A of the Code of Civil Procedure,
1908, must refer to civil
proceedings in a Civil Court.
(a) Both A and R is false and R is not correct explanation of A.
(b) Both A and R is true and R is correct explanation of A.
(c) A is true but R is false.
(d) A is false but R is true

. Page 27 of 40
Passage (Q.96-Q.100): The effect that religion has had on the progress of Indian
society is tremendous.
Therefore, the emergence of India as a secular state in the mid-twentieth century
was a remarkable social,
political, and religious phenomenon. Though religion remains important in India,
and still exists in the public
sphere, the country has successfully retained its secular character. Secularism
became ideal in religious India
due to communalism because the former was understood as an answer to the later." To
make secularism
acceptable, freedom of religion was guaranteed. As such, the framers of
Constitution provided that no religion
would be given preference over another and permitted the practice of any religion.
Citizens would be free to
follow and practice their religion in their private affairs, and thus, the State
would not impose a uniform civil
code despite a constitutional mandate to do so. Further, the State would not
interfere in religious affairs so long
as they did not affect certain other basic rights protected by the Constitution.'
Secularism in India, unlike the West, was not designed to create a wall of
separation between church and state.
It was shaped to assure minorities that their culture, religion, and identity would
be protected and that a
majoritarian view would not be imposed on them. Thus, Indian secularism goes much
beyond mere state
neutrality in matters of religion.
It was in pursuit of these objectives that Pandit Nehru declared: “The government
of a country like India, with
many religions that have secured great and devoted followings for generations, can
never function satisfactorily
in the modern age except on a secular basis.” This principle of equality goes
beyond the assurance that everyone
has the freedom to practice their religion, which was codified in Article 25 of the
fundamental rights chapter of
the Constitution which is available to the citizen of India.
Alongside balancing the diverse milieu of the nation by providing for its own blend
of secularism, the Indian
Constitution also sought to create a progressive society based on scientific
temper. It thus mandated that the State
should intervene in religious affairs only if social welfare demanded it.
[Source- Prof. Faizan Mustafa, Jagteshwar Singh Sohi, “Freedom of Religion in
India: Current Issues and
Supreme Court Acting as Clergy”, BYU Law Review 2017
https://digitalcommons.law.byu.edu/cgi/viewcontent.cgi?
article=3113&context=lawreview ]
96. The union Home minister has tabled a bill on Uniform Civil Code (UCC) before
the Lok Sabha (the lower house
of the Indian Parliament). The code aims to make uniform laws for marriage and
divorce for all religions. The
Bill is passed with 100% votes in favour and has now become a binding law. It is
challenged by the religious
minority groups on ground that it is beyond the power of Indian Parliament. Decide
if it is within the power of
Indian parliament?
(a) It is within the power of the Indian Parliament and hence the challenge is not
valid
(b) It is not within the power of the Indian Parliament as UCC violates Freedom of
Religion in Article 25
(c) It is not within the power of the Indian Parliament as it violates the
principle of secularism
(d) Both (b) and (c)
97. The Author opines that there is a difference between “secularism” in India and
“Secularism” in the West like
US, UK. Which of the following options is incorrect with respect to that
difference?
(a) In the West, Secularism believes in total non-interference with all religious
institution and groups.
Secularism in India means that the state is neutral to all religious groups but not
necessarily separate and
hence supports all religions.
(b) The Western concept of Secularism does separate state and church. In India, all
expression of Religion is
manifested equally with support from the state and the state can protect any
religion which is being
discriminated against.
(c) Both (a) and (b)
(d) Neither (a) nor (b)

. Page 28 of 40
98. In the small town of Kerala, there are 70% Muslims, 25% Hindus and 5%
Christians. There was a piece of land
on which only one place of worship can be constructed, i.e. either a church, or a
temple or a mosque can be
constructed. The Government has to decide on to whom to give the said land. In
light of the passage, decide
which of the following options aptly reflect the ethos of the Indian Constitution?
(a) The state must not interfere at all and maintain neutrality and let the parties
decide amongst themselves.
(b) The state should actively intervene and ensure that all the religions are able
to have a place of worship to
profess their religion.
(c) The state should leave it to the judiciary to decide such dispute
(d) The state should support only the religious minority and hence give the land to
Christians.
99. The Government has criminalised the act of Triple Talaq by a Muslim man for a
Muslim wife. Similarly, the
government has banned giving-taking dowry in Hindu Marriages. It is argued that
Triple Talaq and Dowry were
a part of Islam and Hindu Religion and hence banning such activities is a violation
of the Freedom of Religion.
However, it is not a sound argument. Decide why, in light of the passage?
(a) The state can decide which activity is religious and which is not and hence the
state was correct to hold that
Triple Talaq and dowry are not religious activities.
(b) The state has to ensure that there is freedom of religion to all citizens.
However, the freedom might be
curtailed under circumstances of social welfare.
(c) The state has equally banned practices of both the religions. Since the idea of
neutrality by state is maintained,
the banning of such practices is allowed.
(d) The idea of secularism in India provides that the state should actively
interfere in all religious activities of
the people and hence, it can ban certain activities.
100. The state of Karnataka has recently held that the banning of hijab (a scarf
worn by Muslim women) by schools
in Karnataka is valid on the grounds of secularism. Only by referring to the
opinion of the author in the passage
above, decide if such ban is valid.
(a) The ban is valid because the idea of secularism implies state neutrality and
therefore, the uniform worn by
students has to be neutral.
(b) The ban is valid because the state can interfere with religious activities on
the ground of social welfare.
Wearing Hijab is against social welfare activities of school and hence can be
banned.
(c) The ban is not valid because the state can interfere with religious activities
only on very minimal grounds
like social welfare. Since there is no violation of social welfare by wearing hijab
in school, the ban is valid.
(d) The ban is not valid because the state cannot interfere with religious
activities and banning a religious scarf
shows religious intolerance of the state towards a particular religion.
Passage (Q.101-Q.105): 'Lotteries' is a species of gambling activity and hence
within the ambit of 'betting and
gambling' as appearing in Entry 34 List II. It is only lotteries organized by the
Government of India or the
Government of State in terms of Entry 40 of List I which are excluded from Entry 34
of List II. List I gives
power to union to make laws w.r.t. subjects of list I while list II gives power to
states w.r.t. make laws If lotteries
are conducted by private parties or by instrumentalities or agencies authorized, by
Government of India or the
Government of State, it would come within the scope and ambit of Entry 34 of List
II. The State Legislatures
have the power to tax lotteries under Entry 62 of List II. (Para 124) State of
Karnataka v. State of Meghalaya.
In a significant judgment, the Supreme Court has held that State legislatures have
the competence to levy tax on
the lotteries organized by other states.
Holding so, a bench comprising Justices MR Shah and BV Nagarathna allowed the
appeals filed by the States
of Karnataka and Kerala challenging the judgments of Karnataka and Kerala High
Courts which held that they
lacked the legislative competence to levy tax on the lotteries organized by other
states like Nagaland, Meghalaya
and Sikkim.
In 2010, the Karnataka High Court had held that the State lacked the legislative
competence to enact the
Karnataka Tax on Lotteries Act, 2004 and struck it down. Likewise, in 2020, the
Kerala High Court struck down

. Page 29 of 40
the Kerala Tax on Paper Lotteries, Act, 2005. The States were directed to refund
the tax collected under these
legislations. Now, the Supreme Court has set aside these judgments of the Kerala
and Karnataka High Courts.
Thus, the scope and ambit of lotteries organised by Government of India or
Government of State under Entry 40
of List I is only in the realm of regulation of such lotteries. The said Entry does
not take within its contours the
power to impose taxation on lotteries conducted by the Government of India or the
Government of State.
Lottery schemes by the Government of other States are organised/conducted in the
State of Karnataka or Kerala
and there are express provisions under the impugned Acts for registration of the
agents or promoters of the
Governments of respective States for conducting the lottery schemes in the State of
Karnataka and the State of
Kerala. This itself indicates sufficient territorial nexus between the respondents–
States who are organising the
lottery and the States of Karnataka and Kerala.
(Source: States Have Legislative Competence To Levy Tax On Lotteries Run By Other
States: Supreme Court
Allows Appeals Of Kerala & Karnataka, live law)
101. Last Para of the passages mentions the expression “Territorial nexus between
the respondents States who are
organising the lottery and the States of Karnataka and Kerala”. Chose a
statement(s) that correctly depicts the
meaning of the expression in passage’s context.
(a) Laws made by a state legislature are not applicable outside that state, except
when there is a sufficient nexus
between the state and the object.
(b) Laws enacted by the Kerala and Karnataka legislatures are applicable to one
another as long as the state and
the object have a sufficient nexus.
(c) There is Territorial nexus between States who are organising the lottery and
the States of Karnataka and
Kerala.
(d) Laws made by a state legislature of Karnataka and Kerala are applicable to
other states who are organising
the lottery in the state Karnataka and Kerala as they have express provision for
registration under the
impugned acts.
102. Chandini Enterprises holds annual lotteries all over the world, which are
featured in World Times, an
international newspaper. The edition's readers include top government officials,
MNCs, international firms,
Indian firms, and so on. Chandini Enterprises is based in Karnataka, although it is
registered with the Government
of India under Entry 34 List II. The enterprises conducted their annual lottery
competition but restricted its
audience to the state of Nagaland, Meghalaya and Sikkim. What are the ramifications
of such a restriction?
(a) Since the firm is an authorised agency for conducting lotteries by the
Government of India, it falls under the
scope and ambit of Entry 34 of List II.
(b) Under List II Entry 62, the state legislatures of Nagaland, Meghalaya, and
Sikkim has the authority to tax
lotteries.
(c) The state legislatures of Nagaland, Meghalaya, and Sikkim have the authority to
charge taxes on lotteries
conducted by the Chandini enterprise, which is based in the state of Karnataka.
(d) Unable to determine since the passage does not provide so.
103. The state of Arunachal Pradesh authorised instrumentalities approached the
state of Kerala for the conduct and
organisation of lotteries in the state of Kerala. The Kerala government levied a
tax on lotteries held in the state
of Kerala under the Kerala Tax on Paper Lotteries Act of 2005. Is it possible for
the state of Nagaland to be
exempted from the levy tax?
(a) Yes, on grounds that state Legislatures do not have the power to tax lotteries.
(b) No, as the State Legislatures have the power to tax lotteries only under Entry
62 of List II organised by the
Government of State in terms of Entry 40 of List I.
(c) Yes, on grounds that state legislature of Kerala under the Kerala Tax on Paper
Lotteries Act of 2005, lacked
the legislative competence to levy tax on the lotteries organized by other states.
(d) No, as there are express provisions under the impugned Acts for levy tax on the
lotteries organized by other
states.

. Page 30 of 40
104. Assertion: Article 19(1) (g) of the Constitution declares that all citizens
shall have the right to practice any
profession, or to carry on any occupation, trade or business, subject to reasonable
restrictions under clause (6),
which the State may impose in the public interest.
Reason: lotteries encourage a spirit of reckless propensity for making easy gain by
lot or chance, which leads to
the loss of the hard-earned money of the undiscerning and improvident common man.
Therefore, such activities
could not possibly have been intended by our Constitution-makers to be raised to
the status of trade, commerce
and intercourse and to be made the subject-matter of a fundamental right guaranteed
by Article 19(1) (g).
(a) Both A and R are true but R is not correct explanation of A.
(b) Both A and R are true and R is correct explanation of A.
(c) Both A and R are false.
(d) A is false but R is true.
105. Choose a statement(s) which do not represent an anomaly from the context of
the passage given.
(a) Gambling is a state subject, and only states in India are entitled to formulate
laws for gambling activities
within their respective states.
(b) lotteries organized by the Government of India or the Government of a State
have been taken out of the ambit
of Entry 34, List II by Entry 40, List I.
(c) It was once the rule that state legislatures lacked the legislative competence
to levy tax on the lotteries
organized by other states.
(d) Lotteries organized by the Government of India or the Government of State are
authorized to levy taxes on
lotteries organized by the Government of India or the Government of State.

. Page 31 of 40
SECTION - D: LOGICAL REASONING
Directions (Q.106-Q.135): Read the passage given below and answer the questions
that follow#Passage (Q.106-Q.110): Time seems to change its nature in prison. The
present hardly exists, for there is an
absence of feeling and sensation which might separate it from the dead past. Even
news of the active, living and
dying world outside has a certain dream-like unreality, immobility and an un-
changeableness as of the past. The
outer objective time ceases to be, the inner and subjective sense remains, but at a
lower level, except when
thought pulls it out of the present and experiences a kind of reality in the past
or the future.
As Auguste Comte said, we live dead men's lives, encased in our pasts, but this is
especially so in prison where
we try to find some sustenance for our starved and locked up emotions in memory of
the past or fancies of the
future. There is a stillness and everlastingness about the past; it changes not and
has a touch of eternity, like a
painted picture or a statue in bronze or marble. Unaffected by the storms and
upheavals of the present, it
maintains its dignity and repose and tempts the troubled spirit and the tortured
mind to seek shelter in its vaulted
catacombs. There is peace there and security, and one may even sense a spiritual
quality.
But it is not life unless we can find the vital links between it and the present
with all its conflicts and problems.
It is a kind of art for art's sake, without the passion and the urge to action
which are the very stuff of life. Without
that passion and urge, there is a gradual oozing out of hope and vitality, a
settling down on lower levels of
existence, a slow merging into non-existence. We become prisoners of the past and
some part of its immobility
sticks to us.
This passage of the mind is all the easier in prison where the action is denied and
we become slaves to the routine
of jail life. Yet the past is ever with us and all that we are and that we have
comes from the past. We are its
products and we live immersed in it. Not to understand it and feel it as something
living within us is not to
understand the present. To combine it with the present and extend it to the future,
to break from it where it cannot
be so united, to make of all this the pulsating and vibrating material for thought
and action—that is life.
106. Based on the reading of the passage, what can be a logical inference about
prison life?
(a) Only past memories exist, present does not exist.
(b) One becomes a prisoner of the past in a prison.
(c) Prison life is a routine life without any changes.
(d) The objective life dies, subjective life remains.
107. What can be concluded from the sentence given below?
‘It maintains its dignity and repose and tempts the troubled spirit and the
tortured mind to seek shelter in its
vaulted catacombs.’
(a) Individuals resort to the past for rescuing from the difficulties of the
present.
(b) Present is open and known while the past is hidden and obscure.
(c) Disappointed by the present, one finds relief in undisclosed memories of the
past.
(d) We never forget the past achievements and dignity even if tortured by the
present.
108. Which of the following statements the author would most likely disagree with?
(a) Passion is required to maintain the flow of meaningful actions in life.
(b) To break free from the past, take action in the present to achieve the goals of
the future, is life.
(c) As the inertia increases, the life detaches from the non-existence.
(d) It is recommendable to take actions challenging the difficulties of the present
and avoiding sticking to the
past.

. Page 32 of 40
109. Which of the following best describes the organisation of the passage?
(a) A hypothesis is presented and explained by the author.
(b) A widely accepted belief of an individual is described and implications are
suggested.
(c) Controversial views of an individual are propagated with valid arguments.
(d) An observation was made and explained in detail with the use of personal
thoughts and illustrations.
110. What is the most critical message of the author in the passage?
(a) Romanticisation of the eternal past doesn't solve today's problems.
(b) Learn from past mistakes to envision the future.
(c) Desire to not revel in the past is achieved by developing tools of rationality.
(d) Past is eternal and soothing while the present is stressful, future is
fanciful.
Passage (Q.111-Q.115): Bhagwat Gita’s popularity and influence have not waned ever
since it was composed
and written in the pre-Buddhistic age, and today its appeal is as strong as ever in
India. Every school of thought
and philosophy looks up to it and interprets it in its way. In times of crisis,
when the mind of man is tortured by
doubt and is torn by the conflict of duties, it has turned all the more to the Gita
for light and guidance. For it is a
poem of crisis, of political and social crisis and, even more so, of crisis in the
spirit of man. Innumerable
commentaries on the Gita have appeared in the past and they continue to come out
with unfailing regularity.
Even the leaders of thought and action of the present-day—Tilak, Aurobindo Ghose,
Gandhi—have written on
it, each giving his interpretation. Gandhiji bases his firm belief in non-violence
on it, others justify violence and
warfare for a righteous cause.
The poem begins with a conversation between Arjuna and Krishna on the very field of
battle before the great
war begins. Arjuna becomes the symbol of the tortured spirit of man, which, from
age to age, has been torn by
conflicting obligations and moralities. From this personal conversation, we are
taken step by step to higher and
more impersonal regions of individual duty and social behaviour, of the application
of ethics to human life, of
the spiritual outlook that should govern all. There is much that is metaphysical in
it, and an attempt to reconcile
and harmonize the three ways for human advancement: the path of the intellect or
knowledge, the path of action,
and the path of faith. Probably more emphasis is laid on faith than on the others,
and even a personal god emerges,
though he is considered as a manifestation of the absolute.
It is a call to action to meet the obligations and duties of life, but always
keeping in view that spiritual background
and the larger purpose of the universe. Inaction is condemned, and action and life
have to be by the highest ideals
of the age, for these ideals themselves may vary from age to age. The yuga dharma,
the ideal of the particular
age, has always to be kept in view. Because modern India is full of frustration and
has suffered from too much
quietism, this call to action makes a special appeal.
111. What is the paradox reflected in the passage regarding Bhagavat Gita?
(a) The Bhagavat Gita is written effectively but everyone interprets it
differently.
(b) It advocates violence and non-violence both at a time.
(c) A book written in the pre- Buddhist era finds relevance even in the internet
era.
(d) It suggests the path of action and the path of faith simultaneously.
112. What can be the most logical reason for so many interpretations of the
Bhagavat Gita continuously appearing?
(a) The text is obscure and esoteric, it is liable to be misunderstood.
(b) It has duality in its recommendations that is interpreted differently by all
authors.
(c) Everyone finds others’ interpretations missing a point or more from the book.
(d) Bhagavat Gita exudes different perspective open to different interpretation.

. Page 33 of 40
113. What is a rationale assumption of the utility of Bhagavat Gita for a CEO of a
big business house?
(a) To win the ethical dilemma of competition and the inevitably of physical
failures.
(b) The CEO can find it useful to understand leadership and management.
(c) The CEO can learn to make balance in his tough professional and personal life
to achieve more.
(d) It motivates him to take decisions (maybe tough) and fulfil his obligations
without fear.
114. What is the inherent message of the author in the sentence given below?
Because modern India is full of frustration and has suffered from too much
quietism, this call to action makes a
special appeal.
(a) An inertia in policymaking is to be overcome to make decisions for social
betterment.
(b) Here, the author is demanding a revolutionary action from the people of India
leaving quietism.
(c) It is a call to action that resonates with the Yug Dharma and is based on
spiritual principles.
(d) India has to rise from its long slumber and get ready to compete with the
world.
115. All of the following are the correct inferences of paragraph one, except?
(a) The various commentaries on Bhagavat Gita will continue ad infinitum.
(b) Spiritual crisis is one of the crises of the human sufferings.
(c) The leaders of thought and action are influenced by the teachings of Bhagavat
Gita.
(d) Bhagavat Gita transits space and boundaries.
Passage (Q.116-Q.120): We have seen that the two essential ingredients in the
sentiment of justice are, the
desire to punish a person who has harmed, and the knowledge or belief that there is
some definite individual or
individuals to whom harm has been done. Now it appears to me, that the desire to
punish a person who has
harmed some individual is a spontaneous outgrowth from two sentiments, both in the
highest degree natural, and
which either are or resemble instincts; the impulse of self-defence, and the
feeling of sympathy. It is natural to
resent, and to repel or retaliate, any harm done or attempted against ourselves, or
against those with whom we
sympathise. The origin of this sentiment is not necessary here to discuss. Whether
it be an instinct or a result of
intelligence, it is, we know, common to all animal nature; for every animal tries
to hurt those who have hurt, or
who it thinks are about to hurt, itself or it’s young. Human beings, on this point,
only differ from other animals
in two particulars.
First, in being capable of sympathising, not solely with their offspring, or, like
some of the more noble animals,
with some superior animal who is kind to them, but with all human, and even with
all sentient beings. Secondly,
having a more developed intelligence, which gives a wider range to the whole of
their sentiments, whether self#regarding or sympathetic. By virtue of his superior
intelligence, even apart from his superior range of sympathy,
a human being is capable of apprehending a community of interest between himself
and the human society of
which he forms a part, such that any conduct which threatens the security of the
society generally, is threatening
to his own, and calls forth his instinct (if instinct it be) of self-defence. The
same superiority of intelligence
joined to the power of sympathising with human beings generally, enables him to
attach himself to the collective
idea of his tribe, his country, or mankind, in such a manner that any act hurtful
to them, raises his instinct of
sympathy, and urges him to resistance. The sentiment of justice, in that one of its
elements which consists of the
desire to punish, is thus, I conceive, the natural feeling of retaliation or
vengeance, rendered by intellect and
sympathy applicable to those injuries, that is, to those hurts, which wound us
through, or in common with, society
at large.
116. Based on the reading of the passage, the idea of justice is based on an –
(a) Inference (b) Judgement (c) Assumption (d) Fact
117. Which of the following finds relevance in the passage?
(a) The concept of justice in human beings is rooted in their attempt to show
superiority over animals.
(b) Social sympathies coupled with the higher intellect levels shape the concept of
justice in humans.
(c) Self-defence, retaliation and sympathies are the prime driving forces behind
the demand for justice.
(d) Natural instinct of self-defence with retaliation is the conception of justice
in every harm.

. Page 34 of 40
118. What is the central theme of the passage?
(a) Social justice is essentially relatable to the natural instincts of anger,
reaction and sympathy.
(b) The idea of justice originated in human intelligence on principles of mutual
co-existence and community of
interests.
(c) Idea of social justice is not based on natural instincts but the larger social
interests driven by intelligence and
sympathy.
(d) The idea of justice exists due to the sentiments to retaliate even in humans
with higher intelligence and
sympathy.
119. What is the relationship between the first paragraph and the second paragraph?
(a) Cause and effect. (b) Premise and explanation.
(c) Mutually Independent. (d) Complementary annexation.
120. What is an analogical corollary based on the facts presented in the passage?
(a) A mob attacking its rival for territorial occupancy.
(b) A judge pronouncing the death penalty for a serial killer.
(c) Some union members protesting against a possible pay cut.
(d) A group of elephants attacking visitors disturbing their habitat.
Passage (Q.121-Q.125): A ‘health-war’ is underway in the global centre of pandemic
– Europe. Movement
restrictions across all the cities have deserted the streets, malls, cafes and
cultural heritage sites. Human life does
not exist. The scene is no less than of a dystopian movie scripted with empty
streets, strange face masks, surgical
gloves and self-isolation of the new normal. The restrictions of the Nazi-era were
insipid and surreptitiously free.
W.H.O, passing new ambiguous and dreading statements related to the Coronavirus on
an unending basis looks
like the Greek god announcing doomsday.
Europe, being the centre of the epidemic, holds a specific responsibility in the
way we react to this situation, and
in how we behave also towards others beyond our shores. Covid-19 is a danger that
needs to be addressed
collectively, not only by Europe or by Asia in isolation. The collective approach
is mysteriously missing to
tackle the Covid-19. Why can the success of the Paris climate agreement be not
repeated when all the world
leaders stood together and vowed to mobilise their resources against a common
enemy. However different, both
these perils have this in common: they transcend all national boundaries and they
threaten lives. The “us” versus
“them” logic of nationalists and populists becomes absurd in the face of these
phenomena.
Videos of Italians in lockdown singing from their balconies to keep their spirits
up have been admirable. Could
some of that gusto spread and morph into a Europe-wide flurry of videos chanting
our empathy and willingness
to show solidarity with one another? Wherever we may live and whatever language we
may speak, sending that
kind of message across our “Corona-centre” continent would hold special meaning,
surely not just for ourselves
now, but for others also, and perhaps for the future as well.
Trust in institutions and resistance to fake news are being put to the test.
Pessimists will say our European
capacity to come together and show generosity or even elementary openness to others
has already been entirely
blunted by the crises of the past decade (our numbness to Syria’s killing fields
is, to me, the greatest case in
point). We are no doubt now in severe, introspective, fear-and-fragmentation
European mode.
But for those who still believe we can be a community of a kind, and that our
continental space (or the world
beyond) should not be turned into an “arena”, now is the time to ask ourselves how
we will want to look back at
this phase of our collective history. How will we want future generations to look
back at us, and what kind of
message do we want to send to the rest of the world?
121. What can be inferred about the restrictions of the Nazi era?
(a) Nazism is a form of fascism against democracy and the parliamentary system.
(b) Nazi era was full of anti-communism and the use of eugenics into its creed.
(c) Nazism was spotted with curbs on liberties and free movements, equally
claustrophobic in this era.
(d) The idea of lockdowns might be rooted in this era of restrictions of the Nazi
era.

. Page 35 of 40
122. What cannot be assumed about the ‘dystopian scenes’ as stated in the passage?
(a) Dystopias are societies in cataclysmic decline, with characters who battle
environmental ruin, technological
control, and government oppression.
(b) Dystopian literature is a form of speculative fiction that began as a response
to utopian literature.
(c) Dystopian fiction can be a way to educate and warn humanity about the dangers
of current social and political
evils.
(d) None of these.
123. ‘Europe, being the centre of the epidemic, holds a specific responsibility in
the way we react to this situation,
and in how we behave also towards others beyond our shores.’ Which one of the
following most clearly identifies
an error in the author’s reasoning?
(a) The author cites irrelevant data.
(b) The author presents on incorrect conclusions.
(c) The author makes incompatible assumptions.
(d) The author mistakes an effect for a cause.
124. Which of the following message does the author wants to pass to future
generations?
(a) That citizens can show the way in times of disasters while the political class
fails.
(b) Ordinary virtues are never absent in the period in need of solidarity.
(c) To use our shared knowledge to steer toward meeting global goals on climate
change and sustainable
development.
(d) We wish that you do not take the present good life for granted and that you
work for a collective future
125. Which of the following strengthens the arguments of the author?
(a) Advanced European countries have the necessary expertise to help Asian
countries.
(b) A synergy of efforts from the diverse competent groups can stop the spread of
the pandemic.
(c) Coronavirus is fast mutating in different countries and information needs to be
shared.
(d) The period to come out of a pandemic depends on the concentrated action of the
world community.
Passage (Q.126-Q.130): Who is the person responsible for a failed drug therapy
going wrong on a patient? Legal
questions are bound to raise to hold someone accountable and liable to be punished.
If the responsible person or
persons providing the accounting fail to furnish an acceptable solution for the
conduct, then liability will be
imposed. Liability is legal recognition that an acceptable accounting was not
provided by a party who was in a
position of responsibility when a problem occurred.
The pharmacy law defines a set of responsibilities for pharmacists and for others
who are formally involved with
medication use. Pharmacy law protects patients from harm that might occur if
medications were used in ways
that unreasonably increase the risk of their causing harm. Some pharmacy laws
relate to all drugs and the hazards
of using them for therapeutic reasons. Other laws relate to a subset of drugs that
have the potential for abuse for
performance enhancement, and these laws seek to restrict inappropriate use while
not interfering with legitimate
use. In studying pharmacy law, the primary goal is to address the question: “How
far should the government go
to protect people who use medications from the consequences of their own choices in
drug therapy?” This is
both an empirical question and a normative question because it asks both what the
rules are and what the rules
ought to be. To fully appreciate pharmacy laws, it is important to understand what
is required and why, as well
as understand that pharmacy law is dynamic and can be changed to protect the public
in ways that extend beyond
the status quo.
Pharmacy is regulated at the federal and state levels. Federal rules primarily
relate to the drug product, while
state rules primarily relate to the people who practice pharmacy and the practice
sites within which they perform
their professional duties. As a general rule, pharmacists are required to comply
with the most restrictive law, if
both state and federal law address a specific issue and if they conflict on that
issue. For example, if under federal

. Page 36 of 40
law a particular drug is not restricted to prescription-only sale, but under state
law, there is such a restriction,
pharmacists may not sell the drug without a prescription, because state law would
be stricter. Administrative
agencies such as the board of pharmacy, at the state level, and the Food and Drug
Administration or the Drug
Enforcement Administration, at the federal level, make rules for pharmacists and
they enforce their own rules as
well as rules made by the state and federal legislatures.
126. What can be a logical reason for pharmacists to comply with more restrictive
one out of federal and state laws?
(a) Stricter law would be better than less strict law.
(b) It leaves no room for mistakes and harm to the patient.
(c) It ensures that room for mistakes is minimal.
(d) Federal authorities are more responsible than state authorities.
127. All of these are mandatory chapters in a book titled - ‘An introduction to
pharmacy laws’ EXCEPT -
(a) Understanding the composition of drugs.
(b) How to market the drugs to the right patients?
(c) Responsibilities of druggists in dealing with special cases.
(d) Appendix of State and Federal Laws.
128. What is the main objective of the author in the passage?
(a) To highlight the importance of pharmacy laws.
(b) To explain the necessity of pharmacy laws.
(c) To explain the concept of pharmacy laws.
(d) To describe the enforcement procedure of pharmacy laws.
129. What is implied in the statement given below?
“How far should the government go to protect people who use medications from the
consequences of their own
choices in drug therapy?”
(a) Government fails to understand the dynamics of the pharmacy laws.
(b) Some patients do not understand the legality of taking prohibitive drugs.
(c) It is not easy to frame the rules while the same drug use can be legal and
illegal as well.
(d) Government cannot frame rules to stop the wilful use of some drugs by patients.
130. Which of the following is a case of drug abuse as per the understanding of the
passage?
(a) A sportsperson taking a prohibited drug to increase his chances.
(b) A child taking a prescribed drug to increase his intelligence quotient.
(c) Legal use of the euthanasia drug to take the life of a person.
(d) A young divorcee taking sleep-inducing pills after consulting her family
doctor.
Passage (Q.131-Q.135): There is a general tendency in an achiever’s mind not to
think too much of those
fundamental questions which appear to be beyond reach, but rather to concentrate on
the problems of life—to
understand in the narrower and more immediate sense what should be done and how.
Whatever ultimate reality
may be, and whether we can ever grasp it in whole or in part, there certainly
appear to be vast possibilities of
increasing human knowledge, even though this may be partly or largely subjective,
and of applying this to the
advancement and betterment of human living and social organization.
There has been in the past, and there is to a lesser extent even today among some
people, an absorption in finding
an answer to the riddle of the universe and life. This leads them away from the
individual and social problems
of the day, and when they are unable to solve that riddle, they despair and turn to
inaction and triviality, or find
comfort in some dogmatic creed. Social evils, most of which are certainly capable
of removal, are attributed to
original sin, to the unalterableness of human nature, or the social structure, or
(in India) to the inevitable legacy
of previous births.

. Page 37 of 40
Thus, one drifts away from even the attempt to think rationally and scientifically
and takes refuge in
irrationalism, superstition, and unreasonable and inequitable social prejudices and
practices. Indeed, even
rational and scientific thought does not always take us as far as we would like to
go. There is an infinite number
of factors and relations all of which influence and determine events in varying
degrees. It is impossible to grasp
all of them, but we can try to pick out the dominating forces at work and by
observing external material reality,
and by experiment and practice, trial and error, grope our way to ever-widening
knowledge and truth. For this
purpose, and within these limitations, the general Marxist approach, fitting in as
it more or less does with the
present state of scientific knowledge, seemed to me to offer considerable help. But
even accepting that best
available approach, the consequences that flow from it and the interpretation of
past and present happenings
were by no means always clear.
131. What conclusion can be drawn from the passage?
(a) It is futile to try to solve the mystery of the universe that is beyond the
human faculties.
(b) People tends to cover their failures by resorting to convoluted questions of
life and the universe.
(c) Social problems need a scientific attitude for their solutions, not irrational
subjective thoughts.
(d) Inaction leads to superstitions and irrational approaches that cannot solve
social problems.
132. Which of the following action would most likely get approved by the author?
(a) Development of an R&D centre in a state.
(b) Opening of a meditation branch in school.
(c) Leaving worldly life to a secluded place in the Himalayas.
(d) Finding the links of social evils in religion.
133. What can be inferred about the mind of an achiever?
(a) A focused mind concentrated on fundamental questions.
(b) A progressive mind facing social questions.
(c) A mind equipped with technical knowledge and science.
(d) Mind with high intelligence and emotional quotient.
134. Which of the following, if true, weakens the claim of the author in the
passage?
(a) Solving the social problems lead to the progress of civilisations, not wasting
time on the unanswerable
questions.
(b) People who cannot achieve success in practical life starts to take the path of
spirituality.
(c) Social evils are erased by the efforts of pragmatic people, not by the
spiritual people.
(d) We are involved in the materialistic race to achieve more while questions of
our existence are still
unanswered.
135. What is the approach of the author in the passage?
(a) Critical (b) Analytical (c) Convincing (d) Coercive

mock 36
Directions (Q.1-Q.30): Read the following passage carefully and answer the
questions that follow.
Passage (Q.1-Q.5): The Turkish government launched a coordinated attack by many
media outlets, against me,
accusing me of being an enemy of Turkey. The reason for this was my recent
commentary asking President Joe
Biden not to sell Turkey the F-16 fighter jets that Erdogan is asking to buy. I
want to make it clear from the
beginning. I am not anyone’s enemy. I am a Greek-American, I love Greece and I do –
and I will continue to do.
And its interests demand that these state-of-the-art planes not be sold to Turkey
because a time may come when
they will be used to kill Greeks, to occupy Greek territory, and to grab even more
territory in Cyprus.
So, it is both a duty and an obligation and my pleasure to do everything in my
power to prevent such a
development. Turkey’s attack on me, an attack that is one of the greatest honours
in my life, however, reveals
some things that I find useful to note:
Firstly, it is obvious that the Turkish government attaches great importance to
being able to buy these aircraft. It
is for this reason that they try to silence those who they think might stand in the
way of their goal. The fact that
they include me among them is my honour.
Secondly, in contrast to the majority of the political – and journalistic – class
in Athens, the Turkish government,
it turns out, attaches great importance to the role of the Greek- American
community in serving Greece’s national
interests. Perhaps Turkey, as the recipient of the fruits of the efforts of the
Community against it, has a better
understanding of the effectiveness of our actions than Athens does.
Thirdly, the Turkish attack on me proves the attention they are paying to our
community and consequently the
care with which they are watching The National Herald. This is not only because
Ankara is aware that our
newspapers are read in the Community from one end of the United States to the
other, but also because Turkish
officials are aware that it is closely monitored by the U.S. government and is,
therefore, likely to have an impact
in this area.
In my commentary on the F-16s, which angered Turkey, I also expressed my concern
that in the context of the
U.S.-Russia crisis over Ukraine, Turkey’s role would be upgraded, from a
geopolitical point of view, and that
Erdogan will return to the role of ‘good boy’ and good ally of NATO, persuading
America to erase the recent
sins he committed against it and sell him the planes. Unfortunately, “before the
rooster crowed three times,” this
view has been confirmed. During his visit to Ukraine a few days ago, Erdogan
announced that he had agreed to
increase the supply of long-range drones he has sold to that country, which had
already angered the Russian
government.
This decision is a classic move by Erdogan with the obvious aim of appeasing
America. I repeat that under no
circumstances should Biden fall into this trap. Based on recent events – and to say
nothing of earlier ones – there
can be no doubt about Erdogan’s unreliability. If Biden sells the planes to him,
then, after Erdogan has achieved
his goals, he will turn again against America and possibly, more aggressively
against Greece. If what I write
makes me an ‘enemy’ of Turkey, then I gladly accept my guilt.
1. What could be the appropriate title of the passage?
(a) My answer to Turkey- I am guilty, as charged.
(b) Turkey and Me- A Rivalry.
(c) Effect of US- Russia crisis on Turkey.
(d) Turkish interest in Greeks.
2. Why does the author not want President Joe Biden to sell Turkey the F-16 fighter
jets?
I. Because the time may come when the F-16 fighter jets will be used to kill the
Turks.
II. Because the time may come when the F-16 fighter jets will occupy Greek
territory.
(a) Only I. (b) Only II. (c) Both I and II (d) Neither I nor II.

. Page 3 of 36
3. Which of the following cannot be inferred from the passage?
(a) It is both a duty and an obligation of the author to do everything in his power
to curtail Erdogan’s intentions.
(b) Author’s commentary on the F-16s angered Turkey.
(c) Turkey’s attack on the author is one of the greatest honours of his life.
(d) The author expressed his concern that in the context of the U.S.-Russia crisis
over Ukraine, Turkey’s role
would be degraded.
4. One can infer from the passage that the author is by profession?
(a) A commentator. (b) A journalist.
(c) A Greek-American. (d) A Greek citizen.
5. Which of the following can be concluded from paragraph five?
(a) The National Herald is read only in Turkey.
(b) The National Herald is read only in New York.
(c) The National Herald is read from one corner of the U.S to other.
(d) The National Herald is read Globally.
Passage (Q.6-Q.10): The Mahatma’s legacy in modern India has, more often than not,
been honoured by way
of political pageantry. Successive governments and their leaders have made it a
point to observe the occasions
of M.K. Gandhi’s birth and death with ritualistic shows of reverence. This year,
January 30 — Gandhi’s death
anniversary — was not an exception. The Prime Minister led the nation in paying
homage to the martyr. What
is new — and worrying — is the not-so-ritualistic ‘burial’ of the Mahatma’s ideals
at the hands of the regime.
The evidence is substantial and stark. A gathering of sadhus called upon the
government to declare India, a
secular republic, as a Hindu rashtra — a majoritarian State that Gandhi fought
against all his life — that too on
the occasion of his 74th death anniversary. Bigots, these days, also have the
courage to call for genocide against
India’s minorities. Meanwhile, academic institutions are being goaded to adopt
dubious curriculum in the name
of glorifying faith. These transgressions are possible because the State is
complicit in them, either through silence
or tacit endorsement.
Gandhi’s burial can be attributed to several factors. Among them, two deserve
special attention. The first — the
more obvious — is a sustained ideological and, subsequently, institutional assault
on his vision by India’s present
minders. The other — less discussed — is liberal India’s failure to ensure that the
great man’s philosophy
percolated into the very soil of the nation so as to secure fraternity. These
challenges, however, underline the
formidable moral, philosophical and political strength of the Gandhian way of life.
But the task of resurrecting
the Mahatma is not a political duty. It cannot be left to the hands of politicians
who seek to appropriate or reject
him. It is a public responsibility that needs to be nurtured — shared — by every
Indian citizen.
6. Which of the following can be inferred from the passage?
(a) Gandhi’s metaphorical burial can be attributed to two factors.
(b) The burial of the Mahatma’s ideals at the hands of the regime is worrying.
(c) The task of resurrecting the Mahatma is a political duty bestowed upon the
politicians.
(d) Under the government’s watch India’s regression into frenzied polarization on
non- religious grounds has
been the most discernible.
7. What is the central theme of the passage in brief?
(a) Gandhi’s Burial.
(b) Democratic development
(c) The regime – The Peril of the Majority.
(d) The regime and Gandhi- Eye to eye.

. Page 4 of 36
8. The expression, Gandhi’s burial, is…
(a) Metaphorical. (b) Satirical. (c) Ironical. (d) Rhetorical.
9. The task of resurrecting the Mahatma is-
(a) A political duty. (b) A social duty.
(c) A public responsibility. (d) A social responsibility.
10. What does the author mean by “ritualistic shows of reverence”?
(a) A display of respect. (b) A procedural respect.
(c) A normal respect. (d) unpredictable respect.

Passage (Q.11-Q.15): Since the days of the Nixon administration, nearly 50 years
ago, American policy has
aimed to steer China out of its isolation to prosper in the global market economy.
There needn’t be a conflict
between the established superpower and the emerging giant, the best strategic minds
argued. As it grew rich,
they reasoned, China could be integrated peacefully into the institutional
framework built by the Western powers
from the rubble of World War II. The proposition fit with the “liberal peace” view
of foreign relations: that
nations engaged in intense economic intercourse would find it too costly to go to
war. American businesses that
flocked to China to tap its cheap labour and huge consumer market after its entry
into the World Trade
Organization in 2001 enthusiastically agreed with the approach.
That strategy, it appears, is now over. President Trump’s announcement that the
United States would impose a
battery of tariffs against as much as $60 billion worth of Chinese goods while
restricting Chinese investments in
American technology companies has set policy onto a different, more belligerent
path. China is now, in the
president’s words, an “economic enemy.” Interestingly, not all scholars have
opposed the change of tone. Many
foreign policy experts agree that China is not playing by the rules. American
businesses, which typically
endorsed forbearance to protect their market access to China, have grown frustrated
at its appropriation of their
intellectual property.
John Mearsheimer, a foreign policy expert at the University of Chicago, doesn’t buy
the idea of liberal peace. In
“The Tragedy of Great Power Politics,” published in 2001, he wrote, “A wealthy
China would not be a status
quo power but an aggressive state determined to achieve regional hegemony.” And yet
Mr. Trump’s hard-line
approach to China seems destined to fail. Taking a stand against China’s abusive
behaviour is not necessarily
wrong. The problem with the president’s game plan is that it is inconsistent with
all the other diplomatic
initiatives he has taken so far. The tangle of stabs and swipes at allies and
rivals alike, in the service of ill#conceived goals like closing a trade deficit,
serves China more than it does the United States. “Trump has been
a godsend for China,” noted Eswar Prasad, an expert on trade at Cornell University
who once headed the China
division at the International Monetary Fund. “China has manipulated the rules, but
Trump’s response is
counterproductive.”
China is clearly chafing at the rules put together by the West. It openly disagrees
with the post-war apportionment
of the South China Sea. It is more than willing to flout the World Trade
Organization’s intellectual property rules
to build its domestic technological expertise. Several Chinese initiatives — One
Belt, One Road effort to build
infrastructure to connect to Central Asia, the Regional Comprehensive Economic
Partnership, it hopes to
negotiate with its Asian neighbours, and its Asian Infrastructure Investment Bank —
are aimed at building an
institutional framework to rival the trade agreements and multinational financial
institutions supported by the
West.

. Page 5 of 36
11. What is the main theme of the given passage?
(a) The American policy of uplifting China globally appeared counter-productive for
America in the long run.
(b) The American policy to help China grow in the Global market backfired with
matters worsening because of
Trump’s flawed China Policy that makes China the come out as the winner.
(c) It describes the failure of the American policies in promoting China as the
Asian superpower.
(d) It describes America’s attempt at reviving China globally backfiring, with
President Donald Trump trying
to salvage the situation by adopting hardline approach against China.
12. According to the best strategic minds, why wouldn’t there be any conflict
between China and America?
(a) China was expected to tow the lines set by the Western powers.
(b) Some of the issues China has put on the table were legitimate.
(c) China was expected to disintegrate peacefully out of the institutional
framework built by the western
countries.
(d) China was expected to merge seamlessly into established charter framed by
America and its allies.
13. Which of the following was being cited as the reason behind the failure of
President Trump’s hardline approach?
(a) Mr. Trump’s tangle of stabs and swipes at allies and rivals alike made him
unpopular with China.
(b) The diplomatic initiatives that President Trump had taken so far were not
compatible with his game plan.
(c) President Trump maintained that Americans had been outwitted by savvier trading
partners, which derailed
attempts made by him.
(d) Many economists and international-relations experts assumed that President
Trump’s tariffs could play out
much the same way, isolating the United States.
14. Which of the following words has a meaning similar to the word “belligerent”?
(a) Halcyon. (b) Veracious. (c) Hostile. (d) Tranquil.
15. What can be understood from the quote, “A wealthy China would not be a status
quo power but an aggressive
state determined to achieve regional hegemony.”?
(a) China will not be happy playing a passive role of preserving things as they are
when it has regional dominion
in mind.
(b) China has a mind of its own and will not play along with the established norms
dictated by others.
(c) China is happy to tag along with the status quo of the West, but simultaneously
aims to achieve regional
hegemony.
(d) The Western powers are trying to beat China at its own games of achieving
regional hegemony.
Passage (Q.16-Q.20): Nowadays, we find the lack of discipline rampant in every
sphere of human life, be it
political, social, economic and even in education. It is extremely difficult to
proceed with constructive work
when people are prone to indiscipline. To fight against disruptive tendencies and
indiscipline, radical treatment
of the mind is required. Here we feel the necessity of moral and spiritual values
in human life.
There are two ways of treating diseases - pathological and symptomatic. In
pathological treatment, the root cause
of the disease is ascertained first and then the remedy is prescribed. The process
of symptomatic treatment may
be easier, but it does not have a lasting effect. It may give temporary relief,
while treatment through the
pathological process brings about permanent relief.
To determine the root cause of unrest, we ought to first determine the meaning of
the self. Ignorance of our real
Self is the cause of unrest, discord and anxiety. The real Self is not the physical
home. It is something other than
the gross and subtle bodies. We consider the body to be the person as long as we
observe the consciousness in
it. The moment the body is (__) of consciousness, it loses its personality. 'I' am
'i' when the conscious entity, that
is, the entity that thinks, feels and wills is present in me, and 'i' am 'not-I,'
when it is absent in me. Hence, the
entity whose presence and absence makes me, 'me' and 'not-me' respectively, must be
the person, the real 'me'.

. Page 6 of 36
We cannot live independently. We all are interconnected and coexisting, though
retaining our own individual
characteristics. Now, the differences among us all are unavoidable as we are
conscious units. There is a problem
to find a common ground and interest for the solution of these differences. A sense
of common interest can be
fostered among individuals if they learn that they are interconnected.
16. Choose the statement(s) that is/are incorrect as per the given passage.
(a) The ‘I’ in the personality is the real self, which is the conscious self.
(b) Moral and spiritual values treat the unruly tendencies of human life through
drastic measures.
(c) The pathological treatment is finding the consciousness residing in oneself.
(d) A co-existence is interconnectedness on a collective level without losing
individual attributes.
17. What can be a solution to differences among individuals in society?
(a) Individuals need mental peace to understand each other
(b) Spiritual guidance in our lives.
(c) The realisation of the fact that all humans are interlinked.
(d) Disciplining oneself to realise the importance of human interaction.
18. ‘The moment the body is (_____) of consciousness, it loses its personality. 'I'
am 'i' when the conscious entity,
that is, the entity that thinks, feels and wills is present in me, and 'i' am 'not-
I,' when it is absent in me.?’ Which
of the following is unsuited as the word for the blank?
(a) Bereft (b) Devoid (c) Bereaved (d) dispossessed
19. We can infer that the nature of the passage is…
(a) Theological (b) Sociological (c) Satirical (d) Philosophical
20. Which of the following represents the 'i' am 'not-I,'?
(a) The mind that is not the hub of spiritual and mindful learning.
(b) The mind is not the root cause of all inconsistencies in humans.
(c) The mind that is aware of its existence with the sentients.
(d) The mind that is simultaneously destructive and disciplined.
Passage (Q.21-Q.25): I wonder why I am up here on this stage when I’d rather be at
home, when being at home
would be so much more comforting. And I wonder why all of you are sitting there in
the audience, when so
many of you would also be happier at home.
At home, you can wear your pyjamas. No one is going to snub you or disappoint you.
It is less depressing to
think the same thoughts you thought yesterday, than to have the same conversation
you had last week. Why did
we go out? My father never goes out. His emotional life is absolutely even keel. He
is a deeply rational person.
He doesn’t see the advantages.
For many years I have asked myself, why do you spend time with other people? but I
never really attempted to
come up with an answer. I always believed I was asking myself a rhetorical
question, but this week I thought I
would try and find an answer, because a question you ask yourself a thousand times
eventually deserves to be
answered.
And I figure if I know why I go out, I might feel less suspicious of myself for
going out. I might criticize myself
less. I might be able to look around a party without thinking, what a fool – why
did you come – you should have
stayed at home.
The other day, I was sitting alone in a Mexican restaurant and wondering whether it
is possible to quit people,
and good old Alan Carr came to mind. It’s maybe because I recently ended a
relationship, and also have not been
spending much time in my city, and my body has been experiencing very similar
sensations as it did when I gave
up cigarettes two years ago; it’s a physical ache that comes and goes, that’s
almost painful, a sort of gaping
emptiness, a void that needs to be filled. It often seems like the only way to cure
myself of this craving is to give
. Page 7 of 36
in –Not until you tear yourself from everyone you love does it appear that you are
actually physically addicted
to people. The longing for a person is almost identical to the longing for a smoke.
It’s weird.
Anyway, I am not a stoic. My response to withdrawal – which has been to flee into
semi-soothing rebound
relationships – has prevented me from being able to stand before you today and
declare with confidence that it
is possible to renounce people, to bear the weeks of physical withdrawal symptoms,
and thereafter attain the
qualities that Alan Carr claims the non-smoker is in possession of: “health,
energy, wealth, peace of mind,
confidence, courage, self-respect, happiness and freedom.”
But though it wasn’t recent, I have spent time alone in the past, and in my
memories of these times – the happiest
times of my life – I really did seem possessed of substantially more courage,
confidence, self-respect, freedom,
energy, and peace of mind, than those times when I’ve surrounded myself with
people.
And if that’s the truth, and my memory’s not lying – why go out?
21. The author seems to be
(a) An author. (b) A student.
(c) A motivational speaker. (d) A loner.
22. Which of the following describes the tone of the author in the passage?
(a) Reflective (b) Nostalgic (c) Contrite (d) Boorish
23. ‘I always believed I was asking myself a rhetorical question, but this week I
thought I would try and find an
answer, because a question you ask yourself a thousand times eventually deserves to
be answered.’ Which of the
following figures of speech the author has deployed?
(a) Hyperbole (b) Rhetoric (c) Epigram (d) Understatement
24. ‘The longing for a person is almost identical to the longing for a smoke.’ The
analogy has been drawn by the
author based on
1) One’s emotional response to withdrawal from a situation or a person.
II) One’s relationship quotient with the others.
III) The manifestations of symptoms i.e almost physical like symptom
IV) A response of fleeing into semi-soothing rebound relationships.
(a) Only I (b) I & III (c) II & III (d) I & III & IV
25. Which of the following describes the author’s father’s personality?
(a) A perspicacious bent of mind with quicksilver responses.
(b) A deeply rational man weighing the pros and cons, and a stoic.
(c) A quiescent man with deep analytical skills, and devoid of emotional responses.
(d) A reticent man with a pragmatic approach to life, and a calm disposition.
Passage (Q.26-Q.30): In our mouths or in print, in villages or in cities, in
buildings or in caves, a language
doesn’t sit still. It can’t. Language change has preceded apace even in places
known for preserving a language
in amber. You may have heard that Icelanders can still read the ancient sagas
written almost a thousand years
ago in Old Norse. It is true that written Icelandic is quite similar to Old Norse,
but the spoken language is quite
different—Old Norse speakers would sound a tad extra-terrestrial to modern
Icelanders. There have been
assorted changes in the grammar, but language has moved on, on that distant isle as
everywhere else.
It’s under this view of language—as something becoming rather than being, a film
rather than a photo, in motion
rather than at rest—that we should consider the way young people use (drum roll,
please) ‘like’. So deeply
reviled, so hard on the ears of so many, so new, and with such an air of the
unfinished, of insecurity and even
dimness, the new like is hard to, well, love. But it takes on a different aspect
when you consider it within this
context of language being ever-evolving.
Because we think of like as meaning “akin to” or “similar to,” kids decorating
every sentence or two with it
seems like overuse. After all, how often should a coherently minded person need to
note that something is similar

. Page 8 of 36
to something rather than just being that something? The new like, then, is
associated with hesitation. It is common
to label the newer generations as harbouring a fear of venturing a definite
statement.
Let’s start with, ‘so we’re standing there and there were like grandparents and
like grandkids and aunts and
uncles all over the place.’ That sentence, upon examination, is more than just what
the words mean in isolation
plus a bizarre squirt of slouchy little likes. Like grandparents and like grandkids
means an entire family popped
up in this space we expected to be empty for our use, but in fact, it really was a
whole family. In that, we have,
for one, factuality—“no, really, I mean a family.”
And in that, note that there is also at the same time an acknowledgment of counter-
expectation. The new like
acknowledges unspoken objection while underlining one’s own point (the factuality).
Like grandparents
translates here as “There were, despite what you might think, actually
grandparents.” Another example: I opened
the door and it was, like, her! That sentence is uttered to mean “As we all know, I
would have expected her
father, the next-door neighbour, or some other person, or maybe a phone call or e-
mail from her, but instead it
was, actually, her.” Factuality and counter-expectation in one package, again.
26. ‘Language change has preceded apace even in places known for preserving a
language in amber.’ Which of the
following is the closest interpretation of the sentence?
(a) Language is dynamic and eludes any attempts to fossilise it.
(b) Language escapes evolutions and attempts to conserve it in in- situ.
(c) Conservative attempts to preserve language fail as language escapes and
evolves.
(d) Language breaks barriers and evolves to don many interpretations.
27. That sentence, upon examination, is more than just what the words mean in
isolation plus a bizarre squirt of
slouchy little likes. What does the expression ‘bizarre squirt of slouchy’ mean in
the context of the passage?
(a) Random and unstructured appearances of the word ‘likes’ in the sentence.
(b) Odd and loosely fitted spurts of ‘likes’ in sentences that make it non
sensible.
(c) Unpleasantly spurting word ‘like’ in the sentence makes it interesting.
(d) Strangely fitted ‘like’ in the sentence that leaves the readers clueless.
28. Which of the following states the main point of the author?
(a) How ‘like’ the ubiquitous and overused word associated with young people
represents the ever-changing
English language.
(b) ‘Like’ transformed from something occasional into something more regular,
representing the evolution of
language.
(c) ‘Like’ is not just a tic of heedless, underconfident youth.
(d) The new ‘Like’ is associated with hesitation.
29. Which of the following best represents the title of the passage?
(a) Many interpretations of ‘Like’.
(b) ‘Like’ is not just like.
(c) A reviled ‘Like’.
(d) The Evolution of 'Like'.
30. In the context of the passage, which of the following is the opposite of the
word ‘apace’?
(a) Lightning (b) rapidly (c) gradually (d) static

Passage (Q.66-Q.70): The Supreme Court observed that a sale pursuant to public
auction cannot be set aside on
the basis of some offer made by third parties subsequently and that too when they
did not participate in the
auction proceedings.
Under normal circumstances, unless there are allegations of fraud and/or collusion
and/or cartel and/or any other
material irregularity or illegality, the highest offer received in the public
auction may be accepted as a fair value.
Otherwise, there shall not be any sanctity of a public auction the bench comprising
Justices MR Shah and BV
Nagarathna said.
The sale pursuant to the public auction can be set aside in an eventuality where it
is found on the basis of material
on record that the property had been sold away at a throw away price and/or on a
wholly inadequate consideration
because of the fraud and/or collusion and/or after any material irregularity and/or
illegality is found in
conducing/holding the public auction. After the public auction is held and the
highest bid is received and the
property is sold in a public auction in favour of a highest bidder, such a sale
cannot be set aside on the basis of
some offer made by third parties subsequently and that too when they did not
participate in the auction
proceedings and made any offer and/or the offer is made only for the sake of making
it and without any serious
intent."
(Source: Sale Pursuant To Public Auction Cannot Be Set Aside On The Basis Of Some
Offer Made By Third
Parties Subsequently: Supreme Court, live law)
66. Nandini learned that the state government is holding a public auction for a
plot of land near his village. Because
she did not want to be engaged in the actual sale, she hired an agent and
registered the land to his name. The sale
deed has been executed in the agent's favour. Nandini filed a suit against the
agent for a declaration of title to the
property seeking setting aside of sale deed executed pursuant to public auction.
Decide
(a) The suit is maintainable as Nandini hired the agent to buy the property for
her.
(b) The suit is not maintainable as the sale deed has been executed in favour of
the agent.
(c) The suit is not maintainable as there shall not be any sanctity of a public
auction.
(d) The suit is not maintainable as a sale cannot be set aside on the basis of some
offer made by third parties
subsequently and that too when they did not participate in the auction proceedings.
67. Continuing with the same circumstances as above: Nandini realized that her
prospects of winning the auction
were slim, so she contacted other prospective buyers and formed a pact with them to
bid the lowest amount
possible so that when her agent place the highest price, he would get the property.
And then we'll all sell the
property at a higher price. Despite the fact that this was insider information, the
state authorities are also involved
in the swindle. In such cases, determine the validity of the public auction sale.
(a) The sale is invalid as the bidder engaged in anti-competitive practices and
hence illegal.
(b) The sale is valid as the highest offer received in the public auction may be
accepted as a fair value.
(c) The sale is invalid as the bidder involved in the creation of cartel.
(d) The transaction is null and void because the state authorities were also
involved in the scam, making the
public auction illegal.
68. Nazia obtained a loan from Syndicate Bank; however, due to failure in repaying
the loan, the bank foreclosed on
it. After gaining formal possession of the mortgaged property, which had been given
as a surety for the loan's
timely repayment, the property was put up for sale. The respondent herein was the
highest bidder whose bid was
accepted resulting into issuance of the sale certificate. Contention of Nazia is
that the bank via notification dated
29.09.18 published in the newspaper fixed the date for sale as 8.10.18, inviting
prospective buyers for the
bidding. The explanation given is that the public notice issued for the said sale
was defective as 30 days time
which is mandatorily required was not given. Decide
(a) The sale deed is valid as the respondent here is the highest bidder whose bid
was accepted resulting into
issuance of the sale certificate.
(b) The sale deed is invalid as there was irregularity in conduction of the
auction.

. Page 16 of 36
(c) Can’t be determined as it cannot be said to be a public auction.
(d) The sale deed is valid as a sale pursuant to public auction cannot be set aside
on the basis of some offer made
by third parties subsequently and that too when they did not participate in the
auction proceedings.
69. Chose a statement where the sale pursuant to public auction cannot be set
aside:
I. The worth of the property was substantially higher, and the highest bid was for
less than the actual value of
the land.
II. The auction advertisement is published in the public interest.
III. It is permissible to cancel the auction or sale in favour of the highest
bidder based on some improper
representations made by third parties.
IV In an eventuality where the property had been sold away at a throw away price
and/or on a wholly inadequate
consideration.
(a) II & III
(b) I & II
(c) III & IV
(d) II
70. Assertion: State authorities refused Maahi's participation in a public auction
based on her previous records of
property transfer fraud.
Reason: The sale pursuant to the public auction can be set aside in an eventuality
where illegality is found in
conducing/holding the public auction.
(a) Both A and R are true and R is correct explanation of A.
(b) Both A and R are true but R is not correct explanation of A
(c) A is true but R is false.
(d) A is false but R is true.
Passage (Q.71-Q.75): The hydra headed Amazon—Future—Reliance dispute also throws up
important
questions concerning the role of judicial review in arbitration. Future had applied
to the Delhi High Court under
Article 227 of the Constitution of India to declare the continuation of the
arbitration proceedings as contrary to
law.
To set a context, the first thing to note is that the High Courts review action of
a wide range of bodies is under
Article 226 of the Constitution of India (commonly known as the writ jurisdiction).
Article 227 vests with the
High Courts the power of superintendence over all Courts and tribunals within its
territory. Arbitral tribunals fall
under the High Court's jurisdiction under both these provisions. Both provisions
are part of basic structure of the
Indian Constitution. This doctrine was enunciated in the iconic Kesavananda Bharati
v. State of Kerala, (1973)
4 SCC 225, in which it was argued, that there are certain limitations to amending
the Constitution. For instance,
the majority in power could not amend the Constitution to say that India would no
longer be a democracy.
So, a provision like Section 5 of the Arbitration and Conciliation Act, 1996 (A&C
Act) that restricts intervention
of any judicial authority except as provided under the A&C Act does affect the High
Court's jurisdiction to
intervene. But the availability of this plenary power does not mean that the Indian
High Courts judicially review
all aspects of arbitration. The Supreme Court has tried to devise some standards.
In Deep Industries Limited v.
ONGC Limited and another, 2019 SCC OnLine SC, the Supreme Court ruled that the High
Courts should
interfere against those orders of arbitral tribunal which are patently lacking
inherent jurisdiction. This was a case
where an arbitral tribunal's interim order was unsuccessfully appealed.
The one and only question to be asked in the Amazon case was with reference to the
orders under challenge: did
the tribunal patently lack inherent jurisdiction to pass the orders under
challenge? The answer is a self-evident
no. The A&C Act confer on the tribunal the authority to regulate its procedure. The
tribunal was later going to
consider the issue of termination application filed by the Future group. A party
cannot scuttle the power by
arguing at what point in time the tribunal should exercise that authority.

. Page 17 of 36
[Source- Live Law “Judicial Review of Arbitration: How Far Can The Constitutional
Courts Go?”
https://www.livelaw.in/law-firms/law-firm-Articles-/judicial-review-arbitration-
singapore-international#arbitration-centre-194045 published on March 12, 2022]
71. Based on the passage above, decide which of the following options correctly
describe the power of High Court
under Article 27 vis-à-vis Arbitration Act?
(a) The Power of the High Courts under Article 227 is a part of basic structure.
(b) The power of the High Courts under Article 227 is undoubtedly a Constitutional
power but to be used
sparingly only
(c) The power of the High Courts under Article 227 is undoubtedly a Constitutional
power but is to be used
only in exceptional cases where the lower Courts have passed an order without any
jurisdictional authority
(d) Both (a) and (b)
72. ABC Ltd versus XYZ Ltd. had entered into a contract. However, owing to certain
contractual disputes, the matter
reached Arbitration under the Arbitration Act. As soon as the Arbitration
Proceedings were initiated, XYZ Ltd.
filed a writ petition under Art 226 praying that the Arbitrator has framed wrong
questions of law. Is the said writ
petition maintainable in light of Section 5 of the Arbitration Act?
(a) No, the writ petition is not maintainable as Section 5 restricts any judicial
authority to intervene except as
provided under the Act.
(b) Yes, the petition is maintainable as Section 5 restricts any judicial authority
to intervene except as provided
under the Act. However, the Arbitration Act provides the High Court with the power
to interfere in case of
jurisdictional issues.
(c) Yes, the petition is maintainable as the High Courts have power to entertain
writ petition under Article 226
in exceptional cases and the present facts represent that exceptional situation.
(d) No, the petition is not maintainable as the High Courts have power to entertain
writ petition under Article
226 in exceptional cases and the present facts do not represent that exceptional
situation.
73. Sumit and Keshav entered into a contract which provides for an arbitration
clause to be invoked in case of non#payment of dues by either party. Certain
disputes regarding quality of goods supplied by Sumit to Keshav arose.
Sumit initiated arbitration proceedings against Keshav. Keshav challenged the
maintainability of arbitration
before the Hon’ble High Court under Artivle 226/227 of the Constitution. Decide if
High Court should interfere?
(a) High Courts can interfere as power under Article 226/227 is a part of basic
structure of the Constitution and
cannot be taken away.
(b) High Courts can interfere because the arbitration clause who’s being invoked
beyond the jurisdiction of the
arbitrator.
(c) The High Court cannot interfere because the jurisdiction of the arbitrator is
not in issue in the said writ
petition
(d) The High Court cannot interfere because Section 5 of the Arbitration Act
specifically restricts interference
by any judicial authority, except Supreme Court
74. An arbitration clause in the contract between the parties read as follows:
“In event of any dispute, difference, breach, arising out of or in connection with
the instant Agreement, the
parties shall resort to amicably settle the matter. However, in case the matter is
not amicably settled, then the
parties shall mutually appoint an Arbitrator. In no case, can the parties approach
any judicial forum, especially
High Court under Article 226/227, to challenge any clause or settlement or
jurisdiction under this clause.”
When the matter was referred to the Arbitration, one of the Parties challenged it
before the Hon’ble High Court
under Article 226. The High Court was of the view that the Arbitrator has exceeded
its jurisdiction. The said
order of the High Court was challenged before the Supreme Court on the grounds that
the Arbitration Clause
ousts the jurisdiction of all judicial forums and hence in no case, can an
Application under Article 226 be filed.
Decide.

. Page 18 of 36
(a) The Supreme Court will set aside the High Court’s decision because High Court
has erred in interfering in
the Arbitration Process because the arbitration clause specifically excluded the
interference by High Court
even in the case of jurisdictional issues.
(b) The Supreme Court will not set aside the High Court decision because the power
under Article 226 and
227 is part of basic structure of the Constitution and hence cannot be taken away.
(c) The Supreme Court will not set aside the High Court decision because the
arbitration clause is violative of
the well settled position of law that the High Court can interfere in case of
jurisdictional issue of the
arbitrator.
(d) The Supreme Court will set aside the High Court decision because the issue of
jurisdiction was wrongly
decided by the High Court.
75. In a matter pertaining to incomplete construction of a building, under a
contract, the building owner invoked the
arbitration clause as per the contract. While the arbitration proceedings were
going on, the Arbitrator directed
the construction contractor to pay interim compensation till the pendency of
arbitration proceedings. Such
directions were challenged before the High Court under Article 226 and 227 on the
grounds that an Arbitrator
ought not to have awarded such heavy interim compensation as it is against its
powers. Should High Court invoke
its power under Article 226 and 227 to grant relief?
(a) No, because the question is not related to jurisdictional issues.
(b) Yes, because the question is related to jurisdictional issues.
(c) No, because the question is related to exceeding the powers which is not an
issue of jurisdiction
(d) Yes, because the question is related to exceeding the powers which is an issue
of jurisdiction.
Passage (Q.76-Q.80): Bhatia, a former employee, had registered a sexual harassment
complaint against
Scoopwhoop CEO Sattvik Mishra and his wife.
A Delhi Court has observed that expression of a victim's trauma or experience is
his or her fundamental right
which can only be curtailed if it is falls under four broad categories.
Additional Civil Judge Preeti Parewa listed the four categories as under:
- Libel, slander or defamation;
- Contempt of Court;
- Offends against decency or morality, and
- Undermines the security or tends to overthrow the State.
It said "A legal right / injury has to be established before considering any case
for grant of injunction which
prima- facie appears to be absent in the present case.
(Source : Sexual Harassment | Expression Of Victim's Trauma A Fundamental Right,
Can Only Be Curtailed
Under 4 Heads Like If Defamation, Contempt, Etc.: Delhi Court, livelaw)
76. Dhanush worked as an office boy at Metamobile. He was sacked by the corporation
after overhearing a talk
between the CEO and one of his coworkers about how the company is illegally
importing internal parts for
making mobile phones from an alien enemy. As soon as they fired Dhanush, he penned
an Article about how
companies are now engaging in anti-competitive methods to enhance profit sales. He
also mentioned an incident
that happened with him anonymously. Choose a correct statement.
(a) The Article is a prima facie proof of irreplaceable injury caused to the
company’s goodwill.
(b) The Article purports to be the expression of victim’s trauma hence cannot be
curtailed.
(c) The Article comes under the category of defamation and hence can be categorized
as libel.
(d) The Article is an example of freedom of speech and expression enshrined under
Article 19 of the Constitution
of India.

. Page 19 of 36
77. Choose the statement that best illustrates the passage's anomaly:
(a) An interim relief in form of injunction can be sought in matters associated
with defamation on establishing
prima facie proof of a legal right / injury.
(b) In the event of contempt of Court, the plaintiff may seek an injunction against
the defendants.
(c) If it is proven that the one party has suffered irreparable harm, the party has
the right to seek relief against the
fundamental right of expression of a victim’s trauma or experience.
(d) None of the above.
78. A mother uploaded a video on social media wherein she was getting painted by
her two minor children and the
purpose to do so was to teach sex education to them. The said act and uploaded on
social media with the heading
— “Body Art and Politics”. Choose an option based on your understanding of the
passage's context.
(a) The act of mother can be protected under expression of a victim’s trauma or
experience.
(b) The mother's act will not be protected because it is against decency and
morals.
(c) Because it was a consensual act, the act of the mother can be protected under
expression of a victim's trauma
or experience and cannot be curtailed under any circumstances.
(d) Posting such a video on a social media platform may harm the mother's and
children's reputations, and thus
may be restricted.
79. Assertion: "Ask people not to buy the book or read it," Justice Yashwant Varma
said while dismissing the petition
filed by Advocate Vineet Jindal, alleging that Khurshid had compared Hindutva to
groups like ISIS and BOKO
HARAM in his book.
Reason: Court found that matter does not fit under the broad category of
undermining national security or
attempting to topple the government, hence should not be prohibited.
(a) Both A and R are true and R is correct explanation of A.
(b) Both A and R are true but R is not correct explanation of A
(c) A is true but R is false.
(d) A is false but R is true.
80. The passage's closing paragraph expresses Before examining any cause for an
injunction, a legal right or injury
must be proven. In tort, the expression legal injury, where the person in whom the
right is vested is entitled to
bring an action is best described by the maxim:
(a) Damun sine injuria
(b) Injuria sine damnum
(c) Ubi jus, ibi remedium
(d) Actio personalis moritur cum persona.
Passage (Q.81-Q.85): Reservations in promotions to members of SC/ST community can
be provided but certain
conditions, in toto, must be met. There must be quantifiable data to show that
there is backwardness of the SC/ST
community, there is inadequate representation of the community and lastly such
reservations in promotions must
not lose sight of efficiency. Freedom to Speech and Expression under Article 19(1)
of the Indian Constitution
can be restricted on limited grounds of, inter alia, Sovereignty and Integrity of
India, Public Order, Decency and
Morality, Security of State, Friendly relations with foreign countries, Contempt of
Court, Incitement to an
offence. Hurting religious sentiments of a person is an offence. Reasonable
restrictions can be put on right to life
guaranteed by the Indian Constitution under Article 21. Such restrictions must be
by procedure established by
law and such procedure must be just, fair and not arbitrary.
[Edited and Extracted from; https://www.india.com/news/india/sc-refuses-to-dilute-
conditions-for-reservation#in-promotion-for-sc-and-st-says-collection-of-
quantifiable-data-mandatory-5209828/]

. Page 20 of 36
81. According to a government study, about 0.02 percent of senior level teaching
staff posts in Indian Institutes of
Technology (IITs) across India are held by members of the SC/ST community. It also
demonstrates that the
SC/ST community is socially backward. The government passed legislation reserving
3% of senior professor
promotions for members of the SC/ST community. Decide.
(a) Reservations are eating away merit in India. Therefore, the law passed is not
valid in the eyes of law.
(b) Reservations are leading to brain drain in India.
(c) The law is most likely valid in the eyes of law.
(d) The law is not valid in the eyes of law as the conditions are not satisfied.
82. Kunal was born and raised in Jammu and was pursuing his further education there
as well. In college, he got
acquainted with Virat, another Jammu local. Virat was a covert Pakistani agent
spreading discontent among
Jammu and Kashmir residents, which Kunal and others were unaware of. Kunal was
misled into believing that
Kashmir should be independent. Kunal shared many posts on his Facebook timeline,
along with manipulated
images and videos provided by Virat. The messages went viral, and there were
reports of violence against the
Indian Army stationed in Jammu. Kunal’s Facebook account was banned by the
authorities. Decide.
(a) Kunal’s right to freedom of speech and expression is violated.
(b) Virat is the real culprit and his social media account should be blocked and
not that of Kunal.
(c) Both Kunal and Virat must be arrested as their posts amounted to distress.
(d) The government’s action is justified in the instant case.
83. Mr. Shukla was involved in tribal rights activities. He led numerous peaceful
protests against government
policies that displace tribals in the name of national development. The government
passes legislation prohibiting
any form of protest, whether peaceful or violent, against any official action. Mr.
Shukla has been arrested in
accordance with the law. Decide.
(a) Mr. Shukla’s arrest is unjustified since the legislation is unjust and
unreasonable.
(b) Mr. Shukla’s arrest is unjustified because his recommendation was not taken
into account when the statute
was enacted.
(c) Mr. Shukla’s arrest is justified since he violated the new legislation.
(d) Mr. Shukla’s detention is justified since the legislation is enacted through
legal procedures.
84. The government passes a new law in response to an increase in rapes. Among the
key modifications made is that
the victim’s evidence is final and can be utilised to condemn the accused solely on
the basis of it. The minimum
sentence is life in jail with castration, and the maximum is death. Decide.
(a) The law is not valid as many reported rape cases are fake.
(b) The law is harsh but valid as it is enacted by procedure established by law.
(c) The law is not just to the accused and is arbitrary.
(d) The law is a welcome step and will help stop the increasing number of rape
crimes in the country.
85. Shifa is a state level kabaddi player and has won many laurels in the same.
While playing kabddi, she wears full#length tights instead of shorts which other
girl’s wear. Adding to this, she wears a hijab so that to cover her neck
and hair so as to maintain her modesty and to subscribe to her religious faith.
While playing a match in Mumbai,
Saloni, her team mate itself, captured a picture of Shifa and posted on social
media with caption “#stupidity at
peak #ashamed of showing body #man gropes if skin exposed”. The post gets viral and
when Shifa discovers it,
she files a complaint against Saloni. Decide.
(a) It is common for women to wear shorts in kabaddi. The dress by Shifa is weird
and hence, Saloni’s post is
justified.
(b) Shifa’s dress is not subscribing to norms of modernity and must be shunned at
all costs.
(c) Saloni has hurt the religious sentiments of Shifa and therefore, her post can
be blocked.
(d) Saloni has the right to free speech and expression and therefore, can post
anything on her social media
account.

. Page 21 of 36
Passage (Q.86-Q.90): The Supreme Court opined that while determining testamentary
or non-testamentary
dispositions, the Courts should not pass value judgment on relationships between
the transferor and the transferee
as long as the document is validly executed.
The Supreme Court observed that voluntariness and animus are necessary for the
execution of a valid gift deed.
"When a person obtains any benefit from another, the Court would call upon the
person who wishes to maintain
the right to gift to discharge the burden of proving that he exerted no influence
for the purpose of obtaining the
document.", the bench comprising Justices MR Shah and Sanjiv Khanna observed in a
judgment delivered on
Monday.
The gift of immovable property will be effective when the gift deed is registered
with the appropriate Registrar
or Sub-Registrar. The gift of movable property is effective when the gift deed is
registered or by delivery of the
property. When the gift deed is registered, the transfer of the property from the
donor to the donee takes place
immediately, and the parties need not go to Court for its execution.
Registration of the gift deed is mandatory when the donor wishes to gift immovable
property to the donee. A gift
deed must be executed out of love and affection towards the donee without any
consideration in return.
The donor should have a sound mind and must be competent to enter into agreements
at the time of making the
gift.
A minor is incapable of gifting property as he/she is incapable of entering into
agreements. However, the
guardian of a minor can accept the gifts given to a minor on his/her behalf. The
donor should make a gift without
any consideration, i.e. the donor should not receive anything from the donee for
making the gift.
(SOURCE: Gift Deed In Favour Of Live-In Partner: Supreme Court Says Courts Should
Not Pass 'Value
Judgment' On Relationship between Donor & Donee If There Is Valid Execution, live
law)
86. Raju's father has promised to buy him an iPhone if he passes his higher
secondary exam with flying colours. He
also brought the phone and hid it in the cupboard for Raju to gift him on the day
of his results, whatever it may
be. Raju discovered the phone a day before the results and took it into his
possession. Decide
(a) Raju has no ownership rights to the phone, as it is still owned by Raju's
father.
(b) Raju has no legal claim to the phone.
(c) Raju's father has legal ownership of the movable property until raju takes
possession of it.
(d) Even after Raju took possession of the movable property, Raju's father retains
legal ownership over the
movable property.
87. Continuing with the same facts as above, suppose Raju's father gives him the
phone on his result day because he
topped in the entire state, and he also makes an announcement stating that he is
thinking to gift his bungalow
named "Mannat" to his son Raju. Decide
(a) Both gifts are valid because they were given out of love and affection for the
donee with no expectation of
recompense.
(b) Only phone delivery will be accepted as a valid gift in this case.
(c) The transfer of immovable property will necessitate mandatory registration.
(d) Both gifts are valid because the donor was of sound mind when they were made.
88. Radha is madly in love with the Mayur, a wealthy rich businessman. They decided
to marry, but Radha stipulates
that she will only marry Mayur if he transfers all of his real estate assets in the
name of her minor child, Rohan.
In a counter-offer, Mayur accepted Radha's offer but added, "only if you agree to
marry me a week later." Radha
agreed. Is the gift valid?
(a) Gift is valid as a minor can receive the gift through a guardian.
(b) Gift is not valid as it is not registered yet.
(c) Gift is valid as voluntariness and animus are necessary for the execution of a
valid gift deed are present.
(d) Gift is not valid as donor accepting a consideration in return.

. Page 22 of 36
89. Hardei was an old illiterate lady who used to live in a village with her
sister’s son Keshav. Gian Chand is the
son of Hardei’s brother, whereas Keshav is her sister’s son. Gian Chand contended
that late Hardei had gifted
the land to him during her lifetime vide gift deed dated 23rd December 1985.
Keshav, on the other hand, claimed
that he was a tenant in occupancy of the land for over 15 years, a fact admitted by
Hardei before the revenue
authorities. Keshav had therefore acquired rights over the land. Who has a better
chance to acquire rights over
the said property in dispute?
(a) Keshav, as his possession over the said disputed property has been admitted by
Hardei.
(b) Gian Chand as Hardei executed a gift deed in favour of him during her lifetime.
(c) No one as it states incomplete information.
(d) Gian Chand will not acquire rights over the said property as there is no
registration of the said gift deed.
90. Choose a statement which do not provides an anomaly with the context of the
passage given:
I. In the event of a transfer of movable or immovable property, the donor bears the
burden of proof that the
transfer is valid.
II. A testamentary disposition is one that is made without the use of an
instrument.
III. In order for a gift deed to be valid, the Court must examine the relationship
between the donor and the donee.
IV. A gift deed is only valid if it is registered.
V. None of the above.
(a) II & IV
(b) V
(c) I, II, III & IV
(d) IV
Passage (Q.91-Q.94):The Supreme Court observed that once it is established that all
the accused came at the
place of incident with a common intention to kill the deceased, it is immaterial
whether any of the accused who
shared the common intention had used any weapon or not and/or any of them caused
any injury on the deceased.
"Once it has been established and proved by the prosecution that all the accused
came at the place of incident
with a common intention to kill the deceased and as such, they shared the common
intention, in that case it is
immaterial whether any of the accused who shared the common intention had used any
weapon or not and/or
any of them caused any injury on the deceased or not.", the bench observed.
Section 34 in the Indian Penal Code. [34. Acts done by several persons in
furtherance of common intention. —
When a criminal act is done by several persons in furtherance of the common
intention of all, each of such
persons is jointly liable for that act in the same manner as if it were done by him
alone.
Therefore, the Court allowed the appeal and restored the conviction recorded by the
Trial Court.
(Source: State of MP vs Ramji Lal Sharma | CrA 293 OF 2022 | 9 March 2022
Citation: 2022 LiveLaw (SC) 258
Coram: Justices MR Shah and BV Nagarathna)
91. Z worked as a security guard at the AZB bank. He is aware of all the bank's
security checks and concocts a plan
with B, his brother, to rob the bank with a shotgun. On the day of the robbery, B
came with a riffle, which had
not been discussed in the plan of robbery. B also happens to cause harm to a number
of the bank's customers.
Will Z be held accountable for B's actions?
(a) Yes, Z will be held liable as both shared the common intention to rob the bank.
(b) No, Z will not be held liable for the actions of B as Z has not shared the
intention to cause harm to the
customers of the bank.
(c) Yes, Z will held liable for the actions of B as it is irrelevant whether the
accused shared common intention
or not once they arrive at the place of incident.
(d) Yes, Z will be held liable as long as they shared common intention while coming
at the place of incident to
rob the bank.

. Page 23 of 36
92. Continuing with similar circumstances as given above, Z and B will be held
liable for?
(a) Z and B both will be held liable for robbery with harm caused to the customers.
(b) Z will be liable for robbery and B will be liable for both robbery and harm
caused to bank customers.
(c) Z will not be liable for harm cause to bank customers but only for robbery.
(d) Cannot be determined.
93. Section 34 of the I.P.C. establishes liability based on common intention.
Consider the following examples:
I. The weapon used in the crime was discovered in A's residence.
II. A voluntarily obtained the weapon of crime in order to aid the criminal gang.
III. A was forced to obtain the weapon of offence due to a threat to his life.
IV. The weapon was provided upon receipt of the weapon's value (sale).
Which of the following situations reflect (s) the correct ingredients in relation
to Section 34?
(a) I and II
(b) I, II and IV
(c) Only II
(d) All of the above
94. A and B decided to kidnap Taimur, a celebrity kid who is always accompanied by
security. The following are
the requirements of the principle of joint liability under Section 34 of the Penal
Code. Select an incorrect
response.:
(a) A and B involved in a criminal act of kidnapping a child.
(b) A and B have done the criminal act in furtherance of common intention of both A
and B.
(c) A and B shared the common intention, they may or may not participate in the
commission of the act
constituting the crime.
(d) Common intention between A and B of kidnapping implies a pre-arranged plan
between the two.
95. After returning home inebriated, Naman used to beat up on his wife. It's partly
her fault because she used to give
her money to him, knowing perfectly well that he'd use it to buy alcohol. One day,
Naman struck Rishita with
the broken glass bottle, knowing well that it may inflict serious injury or perhaps
death. Rishita was unable to
withstand the blow and died instantly. In order to avoid getting caught, Naman
dumped Rishita's body into the
Ganga. Determine Naman's culpability.
(a) Naman is liable for murder of Rishita as he was completely aware of the
circumstances though in state of
intoxication.
(b) Naman is liable as he came at the place of incident with the intention of
killing Rishita.
(c) Naman is not liable as he was intoxicated and do not had intention to kill
Rishita.
(d) Cannot be determined as facts lack complete information.
Passage (Q.96-Q.100): The Kerala High Court has recently ruled that the occurrence
of a commercial difficulty
or hardship to perform a contract is not an excuse to back out from contractual
obligations which the parties had
agreed to in the first place.
A Division Bench of Justice Anil K. Narendran and Justice P.G. Ajith kumar while
dismissing a petition,
observed that merely because the pandemic has made the performance of a contract
inconvenient, it was not a
good reason for a party to retract from their obligations. The doctrine of
frustration as per Section 56 of the
Contract Act will not apply merely because of commercial hardships.
"Frustration of contract happens when the execution of contract is wholly
impossible.” It states that an agreement
to do an act that is unlikely in itself is void.
There are 3 Ingredients of Section 56:
It renders a contract to perform an act invalid under the following circumstances:
When the act becomes
impossible to perform after the contract is made, or when the act becomes
unconstitutional due to incidents that
the promisor could not avoid.

. Page 24 of 36
Where a person has agreed to do something that he knew, or should have known with
due diligence, was
impossible or unlawful, and which the promisee did not know was impossible or
unlawful, the promisor would
be held liable to compensate the other party for any damages incurred as a result
of the promise’s non#performance.
A resulting alteration in the law or in the legal situation that affects a contract
and prevents the contract’s
performance is a well-known cause of frustration under Section 56.
(Source : No Frustration Of Contract Due To Mere Commercial Hardships Caused By
Pandemic: Kerala High
Court, live law )
96. Jamna Ben is the zamindar of Palghar village. Because his friend Rusheel needed
land for agriculture, Jamna
Ben leased a portion of his land to Rusheel for three years beginning in January
1947. The deed was not registered
in accordance with the law, it was based on an oral agreement. However, due to
partition, the land became a part
of another nation. As a result, Rusheel is unable to use the property due to
communal reason. Rusheel files a
lawsuit against Jamna Ben for breach of contractual obligation. In light of the
information provided in the passage
what would be the fate of contract in the event like that of partition?
(a) Because contract performance is completely impossible, the doctrine of
frustration of contract will apply.
(b) Rusheel cannot compel Jamna Ben to perform contractual obligations as they were
based on an oral
agreement.
(c) Jamna Ben may refuse to perform the contract because the deed was never
registered.
(d) The doctrine of frustration will not be applicable as the deed was never
registered and the contract was based
on an oral argument.
97. In the same fact situation as described above, with the modification that Jamna
Ben was well aware that the
portion of land she was selling to Rusheel became a part of another nation's
territory after partition . Jamna Ben
registered the sale deed. On the same day, Rusheel discovered that Jamna Ben had
duped him into purchasing
land that became another nation's territory after partition. In this case, can
Rusheel claim for compensation from
Jamna Ben for any damages incurred as a result of the promise’s non-performance?
(a) No, Rusheel cannot seek compensation from Jamna Ben since the contract is null
and void due to the
impossibility of contract fulfilment.
(b) No, Rusheel may claim compensation from the government because the contract has
been ineffective due to
the impossibility of its implementation due to a change in the legislation.
(c) Yes, Jamna Ben was aware that she would be unable to fulfil the agreement's
contractual obligations.
(d) The contract is void since it is based on the performance of an impossible act,
hence no compensation will
be provided.
98. Chand follows a faith that permits polygamy. Zenith agrees to marry Bilal
despite the fact that she is already
married to Chand and being forbidden by the law to which she is subject to practice
polygamy. Later, after
learning of the deal, Chand forced Zeenat to retract from her obligations. I Bilal
liable for compensation?
(a) Yes, Zeenat is liable to compensate to Bilal for the loss caused to him by the
non-performance of her
promise.
(b) She is not obligated to compensate Bilal since he knew that Zeenat was not
permitted by law to perform
such contract.
(c) She is not required to compensate because Zeenat was not authorised by law to
perform the contract.
(d) Yes, since Zeenat vowed to marry him but Chand forced her not to.
99. SMITA FOOD INDUSTRIES is a well-known brand in the food supply industry for
producing loose bottles for
food packaging. They had manufacturing plants across the country. They enter into
an agreement with an Irish
company named Nimbola to produce soft drink bottles at a lower cost for
distribution in India. The deadline for
completion of task is in a month. The factory that makes the bottles was destroyed
by a natural disaster. SMITA
FOOD INDUSTRIES did send the consignment to be manufactured at their other plant
but they were unable to

. Page 25 of 36
complete the assignment on time. Can SMITA FOOD INDUSTRIES claim business
difficulty or hardship as a
justification to avoid performing a contract?
(a) Yes, they can make an excuse since the factory was burned down due to a natural
disaster, making the
fulfilment of a contract inconvenient.
(b) No, they cannot take commercial difficulty or hardship to perform a contract as
an excuse.
(c) Yes, because they attempted to finish it but were unable to do so due to the
hassle caused by moving the
material from one manufacturing unit to another.
(d) No, as the act became impossible to perform after the contract is made.
100. Which of the following statements correctly depicts the essential of
frustration of contract?
1. Any act which was to be performed after the contract is made becomes unlawful or
impossible to perform,
and which the promisor could not prevent, become voidable at the part of promisee.
2. Impossibility of performance and frustration are often interchangeable
expression.
3. Frustration automatically brings the contract to an end at the time of the
frustrating event.
4. A frustrated contract is valid until the time of the supervening event but is
automatically ended thereafter.
(a) 1 and 4
(b) 2 and 4
(c) 2 and 3
(d) 2, 3 and 4
Passage (Q.101-Q.105): The High Court was of the view that the power of a
legislative body to repeal a law is
co-extensive with the legislative body's competence to enact such law.
The Supreme Court observed that a law passed by the legislature is good law till it
is declared as unconstitutional
by a competent Court or till it is repealed. The very declaration by a Court that a
statute is unconstitutional
obliterates the statute entirely as though it had never been passed, the bench
comprising Justices L. Nageswara
Rao, BR Gavai and BV Nagarathna observed.
Referring to various earlier judgments, the bench noted these principles:
I. A statute which is made by a competent legislature is valid till it is declared
unconstitutional by a Court of
law.
II. After declaration of a statute as unconstitutional by a Court of law, it is non
est for all purposes.
III. In declaration of the law, the doctrine of prospective overruling can be
applied by this Court to save past
transactions under earlier decisions superseded or statutes held unconstitutional.
IV. Relief can be moulded by this Court in exercise of its power under Article 142
of the Constitution,
notwithstanding the declaration of a statute as unconstitutional.
Article 245 of the Constitution which gives Parliament the power to make laws also
gives the legislative body
the power to repeal them through the Repealing and Amending Act. Article 142 gives
the Supreme Court the
power to exercise judicial activism whenever required and do complete justice and
in many cases, the Supreme
Court has done that. But the Supreme Court also exercised judicial restraint in
many cases where the issue should
be resolved by the legislature or the executive.
(SOURCE : Law Made By A Legislature Is Valid Till It Is Declared UnConstitutional
By A Court Of Law:
Supreme Court , LIVELAW)
101. Which of the following is not true, based on your understanding of the element
of judicial activism undertaken
by the Supreme Court in the case discussed in the above passage:
(a) The decisions of the Supreme Court are binding on all Courts within the
territory of India.
(b) The Indian Courts seem to have pioneered this unique use of suo moto powers to
declare a statute
unconstitutional.
(c) The Supreme Court has the authority to declare unconstitutional law passed by
the legislature.
(d) The Supreme Court can rule a statute unconstitutional if the Legislature lacked
the legislative authority to
pass it in the first place.

. Page 26 of 36
102. The parliament enacted a bill stating that if someone is driving while under
the influence of alcohol, a zero#tolerance policy must be applied. The Supreme
Court of India issued guidelines for granting liquor sales licences
on national and state highways across the country. The Supreme Court has made it
clear that all states and union
territories are forbidden from granting liquor licences along state and national
roadways. Can Supreme Court
issue directions under the ambit of Article 142 to do complete justice ?
(a) No as the apex Court has no authority to encroach the territory of legislature
by enacting new laws.
(b) Yes, the Court can issue directions under Article 142 of Constitution of India
to do complete justice.
(c) No, because the judiciary cannot employ its judicial activism power in every
case, and it must also show
judicial restraint.
(d) Yes, the Supreme Court can issue remedial orders in the exercise of its power
under Article 142 of the
Constitution, even if a statute has been declared unconstitutional.
103. Under the subject of union list, the state assembly of Goa passed a new state
alcohol control act in 2012. Due to
a lack of legislative competence to implement the 2012 Act, the bill was declared
unconstitutional by the
Supreme Court. The Goa state legislature passed another act in 2018 to repeal the
2012 Act by include a saving
clause in the Repealing Act, 2018. Is the legislature authorized to repeal a
legislation that it was not authorized
to enact in the first place?
(a) Yes, as they are competent to repeal the law as they have no authority to enact
it in first place.
(b) No, the legislature cannot repeal a law that they did not have the authority to
enact in the first place.
(c) Both the acts will be declared unconstitutional by the apex Court on ground of
lack of competence to enact
such law.
(d) The act of 2018 will be held unconstitutional by applying the doctrine of
prospective overruling.
104. The High Court held in the above passage that a legislative body's capacity to
repeal a law is co-extensive with
the legislative body's competence to enact the law. What could be the explanation
for the Court's observation?
(a) It is in accord with the power conferred on Parliament by Article 245 of the
Indian Constitution.
(b) It can be described as a Court's obiter dictum.
(c) A statute enacted by an incompetent legislature is invalid.
(d) For all intents and purposes, a statute enacted by an inept legislature is null
and void
105. Assertion: An act passed by government which infringed the right of privacy of
an individual was held
unconstitutional by the Court of law.
Reason: The Court granted remedies in accordance with Article 142 of the
Constitution to save past transactions
under earlier decisions superseded or statutes held unconstitutional.
(a) Both A and R are true but R is not correct explanation of A.
(b) Both A and R are true and R is correct explanation of A.
(c) A is true but R is false.
(d) A is false but R is true.

. Page 27 of 36
SECTION - D: LOGICAL REASONING
Directions (Q.106-Q.135): Read the following passages and answer the questions that
follow.
Passage (Q.106-Q.110): On July 20, GOI made two eye-catching observations in
Parliament about the second
Covid wave. It said states had reported that there were no deaths due to medical
oxygen shortage, or due to lack
of testing and treatment. This is plainly inconsistent with collective experience.
April 2021 to June was the worst
phase in the Covid trajectory, and large parts of the country were locked down as
healthcare infrastructure was
overwhelmed. GoI’s data showed that 2.35 lakh deaths, or 56% of all Covid
fatalities till date, took place in those
three months. Its beggars’ belief that not one death was due to the oxygen supply
crisis. The problem is states
and hospitals are probably using record-keeping protocols to avoid grim facts. A
larger, related question is
whether India is undercounting Covid fatalities. There’s been plenty of research on
this. The most commonly
used source now is the Civil Registration System (CRS), a record of deaths
maintained by states. While this
system does not offer data on just Covid deaths, it provides a sense of the “excess
mortality” in 2020-21 that can
be assumed to have been influenced largely by the pandemic. An alternative
indicator is the GoI’s Sample
Registration System (SRS), a demographic survey, which is available till only 2019.
Arvind Subramanian, a
former GoI chief economic advisor, and his associates, recently estimated that,
under some assumptions, using
CRS shows that excess mortality between April 2020 and June 2021 was 3.4 million.
There’s an argument
against using CRS, which is that it’s not as accurate as GoI’s SRS. That’s partly
true. Data of 2019 shows that
in the southern states there’s no mismatch between CRS and SRS. But many other
states’ CRS underestimate
deaths.
Deaths are often used as a proxy for governance in political rhetoric. Politics
over deaths provides perverse
incentives to undercount and underplay the severity, helped by the fact that Covid
death registration guidelines
are too tight. That’s deeply unfair to families of victims of the second wave. They
deserve an honest answer. The
way out is for GoI to do a proper SRS, a survey that’s been in place for 50 years.
GoI’s data shows that so far
there are 4.18 lakh Covid fatalities, which is 1.34% of people who tested positive.
But if sero surveys indicate
that the number of people infected is far larger than test data, what’s the real
scale of fatalities? The answer can
only come through a nationwide demographic survey.
[Source: Politics of death: There should be a nationwide ....
https://timesofindia.indiatimes.com/blogs/toi#editorials/politics-of-death-there-
should-be-a-nationwide-demographic-survey-to-truly-measure-covid#fatalities/ ]
106. Which of the following, if true, strengthens the idea that there is a pressing
need for a nationwide
demographic survey to gauge the accurate number of deaths occurring due to the
COVID-19 pandemic?
(a) Various independent and unbiased institutions have pointed out that
undercounting of deaths in India
occurring due to public health disasters may further aggravate the crisis.
(b) The Government of India has tried hard to gauge the exact number of the
fatalities associated with the
pandemic and so far, they have succeeded partially.
(c) Various petitioners have pointed out that we do not need a nationwide
demographic survey as long as the
undercounting does not occur.
(d) The government of India is seriously considering the requirements for the
nationwide demographic survey.
107. Which of the following, if true, weakens the idea that there is a pressing
need for a nationwide
demographic survey to gauge the accurate number of deaths occurring due to the
COVID-19 pandemic?
(a) A nationwide demographic survey is only going to increase the government’s
expenditure, which can be put
to better use to boost health infrastructure.
(b) A nationwide demographic survey will surely help us in getting a clearer
picture regarding the number of
deaths occurring due to the pandemic.
(c) Recently, a similar nationwide survey was successfully carried out by the U.S,
which sees a dump in the
COVID fatalities.

. Page 28 of 36
(d) The government’s attempt at gauging accurate data may offset panic among people
which may be detrimental
to the economy.
108. “Deaths are often used as a proxy for governance in political rhetoric.
Politics over deaths provides
perverse incentives to undercount and underplay the severity, helped by the fact
that Covid death
registration guidelines are too tight.” What is the author trying to convey through
this statement?
(a) The governments have always tried to shy away from all the negativity
surrounding the deaths of people
from public health crisis.
(b) It is not like the government is running away from taking all the
responsibility for the deaths caused by the
pandemic. It’s just that this matter should not be politicized.
(c) Deaths have always been considered a substitute for politics, marked by the
morally wrong incentives to
undercut the casualties caused by the pandemic and helped by the strictness of
COVID-19 death registrations.
(d) The government has a hidden motive to undercount the deaths caused by the
pandemic as part of the political
strategy and expression, which is also helped by the stringent COVID-19
registrations.
109. As per the given passage, which of the following statements is not true?
(a) The Government of India is deliberately trying to undercount the number of
casualties associated with the
COVID-19 pandemic.
(b) Both Civil Registration System (CRS) and Sample Registration System (SRS) can
be trusted to some extent.
(c) Most of the people died during the deadly second wave of the pandemic probably
went unrecorded.
(d) A nationwide demographic survey is needed to weed out the problem of
misreporting of cases and gauge the
actual scale of the fatalities.
110. Which of the following statements is true with respect to the passage?
(a) The CRS is just as reliable as SRS.
(b) The CRS is much more reliable than the SRS.
(c) The CRS is much less reliable than the SRS.
(d) The CRS and the SRS are reliable in some instances and not reliable in the
others.
Passage (Q.111-Q.115): Fyodor Dostoevsky was arrested and exiled by an imperial
Russian government,
“Russian Salad” was invented by a Belgian and, at the height of the Cold War,
Andrei Tarkovsky became one
of the most celebrated filmmakers at both the Cannes and Venice film festivals.
During the current conflict in
Ukraine — and the ensuing outrage in many quarters against Vladimir Putin’s
expansionism — an Italian
university has suspended a course on Dostoevsky, a cafe in Kerala has taken Russian
Salad off its menu and
there have been boycotts and bans on Russian dancers, singers, artists and
filmmakers across Europe. While the
cultural assault on Russian artists may come from a place of solidarity with the
people of Ukraine who are
suffering, it is, in practice, unwise and counterproductive.
If there is one lesson from the protracted conflicts of the 20th century, it is
this: Artists can bridge divides and
keep the doors open when governments cannot. In the subcontinent, after all, Indian
films and Pakistani artists
have continued to thrive across the border. Not even the most ardent anti-Putin
activist would claim that all
Russians support the invasion of Ukraine, or that the protests against the war in
the country are not a welcome
sign of dissent. But perhaps the reason for the bellicose calls for eschewing all
things Russian has less to do with
Russia and more to do with the nature of moral posturing in the internet age.
The move to topple Dostoevsky from Europe’s literary canon, or to “cancel” Russian
Salad is reminiscent of the
sort of comical virtue signalling often seen on social media. On the internet,
anonymous posts and loud groups
are the tools used to intimidate and excoriate people that users find disagreeable,
or even others associated with
them. In the real world, however, matters are more complex. Dostoevsky is not
Putin’s creation; a ballet dancer
is not a bomber.
. Page 29 of 36
Source: https://indianexpress.com/article/opinion/editorials/russian-scapegoats-
fyodor-dostoevsky-cold-war#vladimir-putin-7807481/
111. Which of these statements, if true, weakens the argument of author the most?
(a) The relationship of India and Pakistan has deteriorated due to complete ban of
films from one country to
another.
(b) Two countries at war will have people favouring their respective countries, and
people from different world
will show their pleasure or displeasure through social media.
(c) The politics of a country should be kept separated from its culture.
(d) The people of a country represent its mood, culture and political sentiments,
and one cannot isolate people
from their country.
112. Which of the following can be inferred from the passage?
(a) The boycott is a result of actions by Russian state.
(b) The boycott reflects the virtue signalling on internet.
(c) Russian Salad was invented by a Belgian at the height of the Cold War.
(d) Russia has not done a correct thing by attacking Ukraine.
113. The author is most likely to agree with which of the following statements?
(a) Russia is not correct in taking such actions
(b) We should support Ukraine, but such boycott is not the solution
(c) Other countries should not interfere within the affairs of Ukraine and Russia.
(d) Boycott of Russian artists and culture is not the right way to support Ukraine.
114. What purpose the boldfaced statement in the passage serves?
(a) It is a premise supporting the stance of Ukraine over the issue.
(b) It's a premise supporting author's argument regarding the boycott.
(c) It's the argument of the author.
(d) It's a counter premise against the boycott of artists.
115. Which of the following lines reflects the central theme of the passage?
(a) Arrest of Fyodor Dostoevsky is bad and counterproductive.
(b) Boycotts, calls to cancel Russian artists and literary figures are
counterproductive and unwise.
(c) Dostoevsky is not Putin’s creation, a ballet dancer is not a bomber.
(d) Countries should come together in support of Ukraine, but boycott of artists is
not the right way.
Passage (Q.116-Q.120): MORE than two years after the first Covid-19 case was
detected in India, the actual
death toll continues to be obscured by inaccuracies. Going by the official count,
at least 5.15 lakh people have
succumbed to the virus so far. However, this figure is widely regarded as just a
conservative estimate in view of
large-scale underreporting and undercounting of casualties. Last month, the Supreme
Court had said that ‘official
statistics are not true on death figures’, while taking a serious note of the
rejection of a large number of
compensation claims by several state governments. Now, the court has hinted at
ordering a probe into the
allegations that some doctors are issuing fake Covid certificates to help
ineligible people get the ex gratia
payment of Rs 50,000 each.
A thorough and time-bound investigation is needed to separate the genuine claims
from the fraudulent ones. It’s
probable that many persons who lost their family members to Covid might still be
running from pillar to post for
compensation, while unscrupulous elements have pocketed the money in connivance
with corrupt doctors.
Thousands of claims were rejected last year on the grounds that the death
certificate did not mention Covid as
the cause of death, prompting the court to stipulate that if a patient had died
within 30 days of being declared
Covid-positive, his or her family was entitled to compensation ‘without any further
conditions’. The order’s twin

. Page 30 of 36
objectives were to cut red tape and expedite the payment of relief, but it seems
that the relaxation is being grossly
misused. No wonder ever since the court expanded the eligibility criterion, the
deaths officially recorded by state
governments have been hugely outnumbered by the claims.
This mismatch will persist unless a nationwide audit is conducted to establish
clarity on the real number of Covid
deaths. All states and union territories must review their medical records and take
corrective action wherever
irregularities are detected. The officials responsible for lapses in documentation
and the doctors indulging in
fraud should be brought to book. A calamity must not be allowed to become an
opportunity for shady operators
to make a quick buck.
(Source :- https://m.tribuneindia.com/news/editorials/covid-deaths-376171)
116. What is the central idea of this passage?
(a) Covid has highlighted the gross discrepancy in the claims for compensation
where the claims have
outnumbered the actual deaths due to Covid.
(b) Thorough probe is needed to establish clarity on the real number of Covid
deaths to weed out fake claims
for compensation.
(c) A nation-wide audit is the only way to establish clarity on the real number of
Covid deaths to weed out false
claims for compensation.
(d) The Supreme Court decision regarding compensation has resulted in various
problems.
117. The author is least likely to agree with which of the following course of
action?
(a) The amount of ex gratia compensation for Covid deaths to be given to the real
victims.
(b) To punish all the culprits who made false claims for compensation.
(c) The Supreme Court should overrule its ruling regarding compensation.
(d) A nationwide audit regarding the covid deaths should be conducted.
118. Which of the following, if true, most weakens the author’s argument?
(a) The nation-wide audit is ineffective, if the claims of the death as per the
doctor is attributed to something
other than Covid.
(b) The correctness of government data regarding Covid deaths has been questioned
by various eminent
scientists.
(c) The compensation of 50 lakh is sufficient and government cannot grant more
compensation due to budget
constraints.
(d) The doctors involved in fraud will continue doing frauds, even if some action
is taken against some.
119. Which of the following can be said as an inference from the passage?
(a) Officials and doctors indulging in these unfair practices should be suspended
with immediate effect.
(b) The author is not satisfied with how covid was managed in India.
(c) 5.15 lakh people have succumbed to the virus so far.
(d) The Supreme Court ruling although was of good intent, but resulted in some
adverse consequences.
120. What purpose the boldfaced statement in the passage serves?
(a) It is a statement supporting the author’s argument that there is lack of
clarity regarding data.
(b) It is a statement supporting the author’s argument that there is lapses in
Covid cases reported.
(c) It is the argument of the author which he has later substantiated by data and
arguments.
(d) It is the unstated part of the author’s argument.

. Page 31 of 36
Passage (Q.121-Q.126): Two separate benches of the Supreme Court did the right
thing by red-flagging the
disturbing practice of courts accepting information from government agencies in
sealed envelopes. First, in a
case involving the Bihar government, a three-judge bench headed by Chief Justice N
V Ramana made it clear
that it wanted all arguments to be presented in open court. The CJI’s displeasure
was echoed later in the day by
Justice D Y Chandrachud during the hearing on an appeal against the Centre’s ban on
the TV Channel, Media
One. The channel had gone off air on after the Kerala High Court upheld the ban by
relying on documents
submitted by the Centre in a sealed envelope. But when the government repeated this
practice before the apex
court, it was pulled up by a three-judge bench led by Justice Chandrachud that
stayed the ban.
Unfortunately, in recent years, the Supreme Court has itself played a role in
perpetuating the tendency to seek
public-interest related information in sealed envelopes. In the Rafale aircraft
case, the Court accepted the
government’s argument that the matter pertained to the Official Secrets Act. In the
NRC exercise in Assam (that
led to about 19 lakh citizens being excluded from the list), the apex court sought
details from the NRC
coordinator in a sealed cover with neither the government nor the affected parties
being allowed to look at them.
And in the case involving corruption allegations against the CBI director, the
court insisted that the Central
Vigilance Commission submit its report in a sealed cover, ostensibly to maintain
public confidence in the agency.
In a democracy, only a small set of acts by public agencies must remain in the
realm of secrecy – delicate
international negotiations or those that relate to sensitive aspects of security,
details about survivors of sexual
assaults or child abuse. The principles of natural justice demand that all parties
in litigation get a fair chance to
scrutinise evidence. Citizens are entitled to know the reasons for court verdicts
and subject them to scrutiny.
Such transparency is also one of the sources of legitimacy of the institution of
the judiciary. The three-judge
bench in the Media One case has said that it will expand the ambit of the case to
deal with sealed cover
jurisprudence. It will be watched closely.
{Source: https://indianexpress.com/article/opinion/editorials/supreme-courts-red-
flagging-of-sealed-cover#jurisprudence-is-welcome-7823350/}
121. Which of the following assumptions are implied in the passage?
I. The Apex Court benches can issue a red-flag against the lower courts.
II. The practice of court accepting the sealed envelope should not be allowed in
any matter before the court.
III. Supreme Court’s recent decisions red flagging “sealed cover jurisprudence” is
welcome.
IV. Sealed envelope practice shall continue as it is important to safeguard
sensitive information regarding
sensitive matters relating to security.
(a) Only III (b) II & IV (c) I & III. (d) I, III & IV.
122. The author is most likely to agree with which of the following actions by the
government -
(a) Adopt the practice of never using sealed envelope to present information in
court.
(b) Adopt the practice of using sealed envelope to present information in court
(c) Adopt the practice of using sealed envelope to present information in court
only in exceptional
circumstances.
(d) The government should bring all the sensitive information in public domain.
123. In a democracy, only a small set of acts by public agencies must remain in the
realm of secrecy”
Which of these statement, if true, strengthens the statement?
(a) Right to information is not a licensee to get access to all information by the
government.
(b) Any information that poses a national threat is an exception to transparency in
democracy.
(c) Right to information is an absolute right and cannot be subjected to
restrictions.
(d) In a democracy, the affairs of the State should be in public domain except for
sensitive matters.

. Page 32 of 36
124. What is the primary purpose of the second paragraph of the passage?
(a) Analyses the previous cases of Supreme Court relating to the practice of giving
information in sealed
envelope.
(b) Author makes an argument regarding Supreme Court’s role of giving information
in sealed envelope, and
then gives premises to support that argument.
(c) To explain the rulings of some of the case laws regarding the practice of
giving information in sealed
envelope.
(d) To illustrate how government misuses its power to hide information from the
citizens infringing their
fundamental right.
125. Which of the following can be inferred from the passage?
(a) In a democracy, nothing can be hidden in the name of secrecy.
(b) Supreme Court has always spoken against the practice of giving information in
sealed envelope.
(c) The case regarding Mediaone is the first case where the court has gone against
the practice of giving
information in sealed envelope.
(d) The objection raised by the Supreme Court with regard to presenting information
in a sealed envelope by the
government was inadvertently started by the Court itself.
126. What purpose does the boldfaced statement in the passage serve?
(a) It is an argument supporting the main idea of the passage.
(b) It is a premise supporting the argument of the author that Supreme Court has
played a role in continuing the
sealed envelope practice.
(c) It gives examples of cases where Supreme Court has itself played a role in
perpetuating the tendency to seek
public-interest related information in sealed envelopes.
(d) It is one of the arguments of the author regarding the practice of accepting
information in sealed envelope.
Passage (Q.127-Q.131): THE Haryana khap panchayats have always been accused of
pursuing a patriarchal
regressive agenda. Now, some khaps of Jind, Bhiwani and Hisar seem to have shed
their traditional approach to
social customs as they realise that their old diktats have been a hurdle in the
advancement of the people. Their
resolve to not only campaign for the enrolment of all children in government
schools but also seek better
educational facilities is bound to lay the foundation of a progressive state. Since
the campaign aims to tackle
problems at the village level, the fight to overcome long-standing issues like
those pertaining to inadequate or
absent teaching staff is likely to bear better results. If this problem is tackled,
the rising trend of preference for
costly and dubious private schools may be reversed. Incidentally, the need to
spruce up government schools has
been acutely felt in the past couple of years. Many parents are today unable to
afford the private schools after
the economic blow of the Covid-induced lockdowns and job losses.
Equally importantly, this rising of the village elders is designed to address the
hugely unfair gap in the quality
of education received between urban and rural students and their subsequent career
placements. On their agenda
is a push for enabling the children in computer literacy as well as introducing
them to the emerging ecosystem
of start-ups, research, security systems, data and digital curriculum. Earlier, the
khaps have commendably
worked towards changing the backward mindset regarding health and ugly gender
biases like female foeticide
and child marriage.
This melting of the deeply entrenched norms is significant, for it paves the way
for a more widespread
acceptance. Other khaps must also sit up and take note. The transformation of the
state towards an educated and
developed society rests on each one of these influential grassroots-level
organisations taking up this crusade.
Their combined force is needed to stir the moribund government system out of its
inertia and fast-track the efforts
towards improving the state of its schools and the quality of education imparted.
{Source : https://www.tribuneindia.com/news/editorials/khaps-progressive-move-
378401}

. Page 33 of 36
127. “The transformation of the state towards an educated and developed society
rests on each one of these
influential grassroots-level organisations taking up this crusade.” Which of the
following assumptions
are implied in the argument?
I. The makeover of a state starts with the foundational organisations.
II. An educated and developed society relies on struggles taken on by grassroot
level organisations.
III. Grassroot level organisations are fully equipped to take up the movement of
transformation at the micro
levels.
IV. To bring some change in education, it is necessary to follow a bottom-up
approach.
(a) Only I (b) I, & IV (c) I, III & IV (d) I, II, III & IV
128. The author is most likely to agree with which of these statements: -
(a) The need to spruce up government schools has been acutely felt in the past
couple of years.
(b) No other Khap panchayats have been accused of a regressive patriarchal mindset.
(c) Khap panchayat earlier had regressive mindset, but now they are adopting
progressive practices like
promoting education.
(d) Khap panchayat’s campaign of promoting education is based on promoting
contemporary curriculum
meeting the needs of today.
129. Given below are pairs of events ‘A’ and ‘B’. You have to read both the events
‘A’ and ‘B’ and decide
their nature of relationship. You have to assume that the information given in ‘A’
and ‘B’ is true and you
will not assume anything beyond the given information in deciding the answer. Marks
answer;
Event A: Many parents are today unable to afford the private schools after the
economic blow of the Covid#induced lockdowns and job losses.
Event B: Equally importantly, this rising of the village elders is designed to
address the hugely unfair gap
in the quality of education received between urban and rural students and their
subsequent career
placements.
(a) If ‘A’ is the effect and ‘B’ is its cause.
(b) If ‘A’ cause and ‘B’ is its effect.
(c) If both A and B are independent causes.
(d) If ‘B’ is an effect but ‘A’ is not its immediate and principal cause.
130. What purpose the boldfaced statement in the passage serves?
(a) It is premise used to support author’s argument.
(b) It is one of the arguments of the author which have been later substantiated.
(c) It is the counter-premise to support the main idea of the passage.
(d) It is an assertion based on misconception that the author uses to counter
through premises.
131. Which of the following reflects the central idea of this passage?
(a) To bring lasting impact on education, it is necessary to follow a top-down
approach.
(b) The recent steps of khap panchayat are progressive, and other khap panchayats
should also follow it.
(c) Khap panchayat is regressive and patriarchal, but has also made some
contribution for the society.
(d) The khaps have commendably worked towards changing the backward mindset
regarding health.

. Page 34 of 36
Direction (Q.132-Q.135): These questions are based on the following information
Each of the seven students P, Q, R, S, T, U and V belong to one of the three teams
A, B and C and can play only
one game – either Chess, Carroms or Badminton. There are three boys and four girls
among them. Each team
has at least one boy and at least one girl. Three of these seven play Carroms and
two each play Chess and
Badminton. Q belongs to B and he plays Chess. Neither T nor P is in team B. S and
her sister U play Badminton
but are in different teams. V does not play Carroms and he is in team C and S is
the only other player in team C.
R is placed in the same team along with U.
132. Which of the following groups plays Carroms?
(a) PRU (b) PRT (c) RST (d) RTU
133. Who belong to team A?
(a) PT (b) UT (c) RT (d) RU
134. Who among the following plays Carroms?
(a) P (b) V (c) Q (d) U
135. Which of the following teams consists of two girls?
(a) A (b) B (c) C (d) A or c

mock 37 { go babes, you can do it}


Directions (Q.1-Q.30): Read the following passage carefully and answer the
questions that follow.
Passage (Q.1-Q.5): At the heart of America is a packed bag. A proponent of westward
expansion is rightfully
struck at the heart of a particularly American brand of freedom: the ability to get
to a new place. And while
freedom of movement has never been equally distributed, potentially the most
defining migration the nation has
ever seen was the Great Migration, when millions of Black Americans fled the South.
Isabel Wilkerson, the
historian and author of ‘The Warmth of Other Suns’, captured the essence of this
mass movement: “They did
what human beings looking for freedom, throughout history, have often done. They
left.”
But what happens when leaving is no longer an option? In the US, that’s what we’re
witnessing right now:
“Americans, it seems, are finding themselves increasingly locked into places that
they wish to escape,” two
psychologists grimly proclaim in a new paper studying the cultural effects of
residential stagnation. Study authors
Nicholas Buttrick and Shigehiro Oishi cite research showing that when you compare
today’s Americans to
people in the 1970s, people who said they intended to move from a place are 45
percent less likely to have done
so. The paper finds that as residential mobility has gone down, so have “levels of
happiness, fairness, and trust
among Americans.”
How could declining mobility lead to these changes? Buttrick and Oishi explain that
moving to a new place
severs social bonds, and in a new town, far from home, newcomers are forced to
define themselves with context#free personality traits i.e., ‘I am hardworking’ or
‘I am intelligent’ rather than by their relationships to locals like
they might in their hometown. Importantly, all those researchers have found are
correlations: No one has yet
established that declining mobility causes any psychological changes. And another
caveat — while some data
exists related to how much Americans were moving in the 1700s and 1800s, it is only
since 1948 that the
researchers have a “reliable annual rate of residential mobility, making it
difficult to draw strong conclusions
regarding the cultural effects of residential mobility in the longer term.”
Another note of caution is that residential mobility is not independent of economic
growth, settlement patterns,
religiosity, and more. In other words, it could be something else that is driving
some or all of this correlation.
The researchers claim that while things like unemployment and GDP growth have
cyclical patterns, mobility
rates have been declining steadily since 1948 through booms and busts alike. And
the psychologists’ work builds
on a body of economic and political science literature that has raised the alarm
for decades about declining
interstate mobility and its negative effects on financial and personal freedom.
Buttrick and Oishi delineate the
cultural markers of a mobile society- individualism, optimism, and tolerance and on
the other hand, a stable
society- security, and a strong sense of the difference between ingroups and
outgroups. This growing shift toward
the latter could explain much of what has happened to America’s political system in
recent decades.
1. Why can’t researchers draw firm conclusions about the cultural influences of
displacement in the long run?
(a) The levels of happiness, fairness, and trust among Americans are dependent upon
residential mobility.
(b) Researchers only have firm sources to confirm the annual rate of residential
mobility since 1948 and before
that, they don’t have the exact data.
(c) Residential mobility is independent of advancement in economy, settlement
outlines, religiosity, etc.
(d) The economic growth, settlement patterns, religiosity, and more factors make
the assessment and collation
of data almost an impossibility.
2. Which of the accompanying is correct with reference to the passage?
(a) Isabel Wilkerson upheld the Great Migration by stating that the people who left
did exactly what people
had done throughout history to embrace freedom.
(b) According to a study, levels of happiness, justice, and loyalty among Americans
have decreased as
residential migration has mitigated.
(c) Buttrick and Oishi claim that if held in comparison, Americans, in recent
times, are forty-five percent less
likely to move from a place if compared to Americans in the 1970s.
(d) All of above

. Page 3 of 40
3. Which of the following points that are expressed in the passage are TRUE?
A. Much of what has transpired to America's political system in recent decades
could be explained by this
rising tilt towards a stable society that ensures security, and a strong sense of
the difference between
ingroups and outgroups.
B. The psychologists' work is independent of economic and political science
literature that has been warning
us about the decline in interstate mobility.
C. Residential migration is influenced by a variety of factors including progress
in the economy, settlement
patterns, faith, and more.
(a) A and B (b) B and C (c) A and C (d) All A, B and C
4. What is the tone of the author in the passage?
(a) Satirical (b) Pedestrian (c) Introspective (d) Populist
5. According to the study in the passage, how do the factors of happiness and
reliability shift with the mobility
shift?
(a) Moving to a new place dissolves social ties, and newcomers are forced to
associate with context-free
personality rather than by their connections with local residents, as they might in
their birthplace.
(b) Moving to a new place strengthens social ties, and newcomers are forced to
specify themselves in a new
town, far from home, by context-free personality rather than by their connections
with local residents, as
they might in their birthplace.
(c) Moving to a new place breaks social ties, and it is unconditional for newcomers
to specify themselves in a
new town, far from home, by context-free personality rather than by their
connections with local residents,
as they might in their birthplace.
(d) Moving to a new place breaks social ties, and newcomers are forced to specify
themselves in a new town,
far from home, by contextual personality and also by their connections with local
residents, as they might in
their birthplace.
Passage (Q.6-Q.10): I noticed that with each year that passed, it felt like my
attention was getting worse. It felt
like things that require a deep focus, like reading a book, or watching long films,
were getting more and more
like running up and down an escalator. I could do them, but they were getting
harder and harder. And I felt like
I could see this happening to most of the people I knew. I felt like it was
particularly bad for some of the young
people in my life. For a long time, I told myself, “Well, every generation
struggles with attention.” You can read
letters from monks 1,000 years ago where one of them says to the other, “My
attention isn’t what it used to be.”
It’s just that you’re getting older, and as you get older, your mind deteriorates
and you mistake your deterioration
for the deterioration of the world. We need to understand that our attention did
not collapse, our attention has
been stolen from us by these very big forces. And that requires us to think very
differently about our attention
problems.
So, the problem isn’t the tech, it’s the business model? The heart of this is the
business model. So, you open
Facebook or any of the mainstream social media apps and those companies begin to
make money immediately
in two ways. The first way is obvious. You see ads and they make money from the
ads. The second way is much
more important. Everything you do on Facebook is scanned and sorted by Facebook to
build a profile of you.
As people in Silicon Valley always say, you are not the customer of Facebook,
you’re the product they sell to
the advertisers by revealing your preferences. The whole machinery, this whole
business model has an effect:
Every time you pick up Facebook and scroll, they make money. And every time you put
it down, their revenue
streams disappear. So, all of their algorithmic power, all of their engineering
genius, some of the cleverest people
in the world are dedicated toward one goal: “How do I get you to pick up his phone
more often and scroll as long
as he possibly can?”
We also have another pretty good reason for short attention span, although there
isn’t a consensus, that we sleep
significantly less than we used to. Only 15 percent of Americans wake up feeling
refreshed. The figures for the

. Page 4 of 40
US are staggering. I go through them in the book for the lack of sleep in the
United States. I think the figure, if
I remember rightly, is something like 37 percent of Americans sleep less than seven
hours a night. When I spoke
to Dr. Charles Czeisler, who’s one of the leading experts in the world on sleep at
Harvard Medical School, he
said that we sleep so much less, that alone would cause a significant attention
crisis.
6. Which of the accompanying is LEAST likely to be the view of the author keeping
the third paragraph in mind
as a reference?
(a) Dr. Charles Czeisler indicated that we sleep so much that it produces a
substantial impact on the weakening
of our attention span.
(b) The fact that sleep deprivation is one of the main reasons for our short
attention span is not undisputed.
(c) Sleeping less than seven hours a night is detrimental to our brain and is a
reason for the shortening of our
attention span.
(d) A very smaller number of individuals in America get such a good amount of sleep
that they feel refreshed.
7. Which one of the following statements is the main reason for Facebook’s
intention of building a profile on us?
(a) The fact that they consider us a product and show us advertisements to earn
money in return.
(b) The fact that they spy on us, our routine and sell them to the government.
(c) The fact that we’re a product that they sell to advertising companies by
divulging our preferences.
(d) When people sign up for Facebook, many of them choose to upload their contacts
to the service so that
they can find other people to connect with.
8. Why is it said that the business model is the main culprit behind our
deteriorating attention span?
(a) The algorithm of Facebook is built in such a manner the advertisements follow
one after the other based on
viewers’ preference.
(b) It invades our privacy and steals our data and we become slaves to these big
companies like Facebook.
(c) New applications are being launched that are more colourful and contain more
updated features and it is
really tough to keep our hands off them.
(d) The algorithm of Facebook is built in such a manner that we are bound to use it
and waste our time and this
generates profit for the business.
9. In the context of the passage, which of the following best reflects the meaning
of the word ‘profile’?
(a) Outline. (b) Silhouette. (c) Report. (d) Summary.
10. What consolation had the author given to himself for his mitigating attention
span before drawing the conclusion
that his attention span was snatched away from him?
(a) The tech and the business model are designed to decrease our attention span.
(b) It's only that he was getting on in years and as one gets older, one’s mind
weakens.
(c) Sleep deprivation was the fundamental reason for his mind getting weaker day by
day.
(d) The lousy eating habit that the author was going through was the reason for his
mind’s deterioration.
Passage (Q.11-Q.15): The origin of the moon is a mystery as old as the tides.
Although the moon has enthralled
observers for eons, planetary scientists have struggled to trace its ancestry. The
manned lunar landings of the
Apollo programme were supposed to help by collecting hundreds of pounds of moon
rocks. Three perfectly
respectable theories for lunar origin had been proposed. All that was needed to
choose among them was a chunk
or two of the object in question. But the lunar samples may as well have been made
of blue cheese. The evidence
was ambiguous. The result was utter confusion.” In the midst of this
disappointment, a few stalwart scientists
came up with yet another concept. Called the giant-impact theory, it proposed that
the moon was a kind of divot
nicked off when the ancient Earth collided with another celestial object fully half
its size. The theory may have
been far-fetched, but it was consistent with the Apollo findings. Its advocates
said it might also explain certain
dynamics of Earth-moon interactions. Proving it seemed impossible.

. Page 5 of 40
In the past five years, a new generation of technology and talents has emerged to
resuscitate the giant-impact
theory. Faster computers have allowed theorists to construct more credible models
of impacts dynamics, and 21
century geophysics has dispensed with some of the formerly troublesome “facts”.
“It’s fair to say the giant#impact theory has more truth in it than any other
theory”, says Melosh.
Superficially Tim Swindle, planetary scientist, says there’s not much difference
between lunar rocks and “rocks
out on the parking lot”. Under magnification, there are hints of an unearthly
provenance. In spectrometric tests,
some of the dirt also shows signs of exposure to the solar wind–charged particles
that stream from the sun and
bombard the moon, which, unlike Earth, has neither an atmosphere nor a magnetic
field to shield it. But the
mineral composition of moon rocks – including elements such as iron, silicon,
magnesium, and manganese –
resemble that of Earth rocks, says Swindle. And the distribution of oxygen isotopes
– a feature that, like a local
accent, tells scientists what part of the solar system a rock hails from – is also
identical to that of terrestrial
geology. “The rocks are fundamentally made of the same stuff as the rocks on
Earth,” Swindle says. The
similarities between moon and Earth rocks were compatible with each of the three
now-defunct models of lunar
origin. In the fission scenario, for example, a furiously spinning early Earth
tosses some of its mass into orbit.
The co-accretion model has the moon forming at the same time and place as Earth
did, by condensing from an
enormous disk of gas and dust surrounding the early sun. In the capture theory,
Earth’s gravitational field grabs
a wandering moon from nearby space. If the moon calved off Earth or formed anywhere
in the same
neighbourhood, it should look a lot like Earth.
But the Apollo samples differ in important ways from Earth rocks. For one thing,
moon rocks are drier than
anyone expected. It may seem obvious that rocks are dry. But on Earth, water
molecules seep into spaces within
the crystal lattices of minerals. Scientists recently estimated that hydrated
minerals in Earth’s lower mantle could
store about five times as much water as the oceans hold. Scientists have long
known, for example, that the density
of the moon is little more than half that of the Earth, that the moon lacks an iron
core like the one in Earth’s
innards. Seismic readings by Apollo astronauts confirmed that the lunar core is
small to nonexistent.
11. As per the passage, the statements which support Tim Swindle’s ideas are
(I) The magnetic field of the moon shields it from the energy particles which
stream out from the sun.
(II) The distribution of oxygen isotopes on the moon is similar to that on the
earth.
(III)The mineral composition of earth and moon rocks is found to be similar.
(a) I and II (b) II and III (c) I and III (d) I, II and III
12. ‘But the lunar samples may as well have been made of blue cheese.’ What can be
inferred from the given
statement?
(a) The lunar samples did not carry any distinct features to be rendered different
from the earth’s mantle.
(b) The lunar samples were as rare as the blue cheese; therefore, carrying a
distinct geological significance.
(c) The lunar sample were as rare as the origin of the blue cheese.
(d) The metaphorical reference of blue cheese is to suggest that the lunar samples
were as ordinary as the blue
cheese.
13. Select the options where the statements have been correctly identified as true
or false.
(I) The crystal lattices of the minerals found in rocks in the earth’s core hold
large volumes of water.
(II) The lunar core is of negligible size.
(III)The earth’s inner crust lacks an iron core.
(IV)The giant impact theory was found to be absolutely credible.
(a) I – T, II – F, III – F, IV – F
(b) I – T, II – T, III – F, IV – F
(c) I – F, II – F, III – T, IV – F
(d) I – T, II – T, III – F, IV – T

. Page 6 of 40
14. The theory which could successfully explain the origin of the moon was the
(a) The Fission model. (b) The co-accretion model.
(c) The Capture theory. (d) None of the above.
15. ‘In the midst of this disappointment, a few stalwart scientists came up with
yet another concept.’ All of the
following are the synonyms for the word stalwart. Select the synonym that is
nearest to the meaning, in the
context of the passage.
(a) Daring. (b) Unwavering. (c) Brawny. (d) Loyalist.
Directions (Q.16-Q.20): Read the following passage carefully and answer the
questions given below it. Certain
words or phrases have been printed in bold to help you locate them while answering
some of the questions.
Passage: INDIA is the fastest-growing aviation market on the planet, according to
the International Air
Transport Association (IATA), an industry trade group. No thanks to government.
Reforms in India’s aviation
industry have moved at a pace it takes to travel from Delhi to Hyderabad by foot,
rather than by an aeroplane.
India's aviation boom has long been a story of quantity over quality. Its airlines
carry nine times as many
passengers today as they did before deregulation in 1994. No-frills carriers,
unheard of two decades ago, provide
two-thirds of domestic capacity. Taken in isolation, this boom in budget travel is
no bad thing for an emerging
economy like India. Creaking airport infrastructure and stringent regulations that
strangle airline profitability
can make flying a bad experience.
Hopes of reform were running high after the election in May 2014. The aviation
ministry had two troublesome
regulations in its sights. The first, the so-called 5/20 rule, prohibits start-ups
from flying internationally until
they have acquired 20 aircraft and completed five years of operations. Given the
low rewards on offer in the
crowded and price-sensitive domestic market, this rule ring-fences many of India's
most lucrative routes for
incumbent airlines. The second regulation, the Route Dispersal Guidelines, forces
all carriers to devote a
percentage of their networks to public-service-obligation (PSO) flying. That
entails serving unprofitable, remote
regions such as north-east India, the Andaman Islands, and Jammu and Kashmir.
Lowering State-imposed fuel
taxes was identified as another priority, though its urgency has since waned with
the downturn in oil prices
(which, in turn, lifted airline profitability forecasts for 2015/16).
For local start-ups like Vistara, a joint venture between Mumbai-based TATA Group
and Singapore Airlines,
delayed reforms are a double-edged sword. Though eager to see the back of the 5/20
rule, Phee Teik Yeoh,
Vistara’s chief executive, grumbled that politicians “have come up with a
replacement model which appears to
be even more complicated”.
Alas, letting market forces prevail in the aviation sector has never been a strong
point of Indian policymakers.
The country currently has 33 "non-operational" airports, according to the Airports
Authority of India. Popularly
dubbed “ghost airports”, facilities such as Jaisalmer Airport in Rajastan, which
was built in 2013 to accommodate
300,000 passengers a year but recently closed without flying a single one, are a
relic of successive, failed
attempts at promoting economic growth in small cities and townships. With the
government vowing to build 200
low-cost airports over the next 20 years, many more are expected. India's well-
intentioned airports policy—much
like its egalitarian Route Dispersal Guidelines—ultimately has too much faith in
the public sector's ability to
stage manage private-sector growth. Shiny new airports bereft of passengers do
little to spur the growth of local
economies.
For the majority of its citizens, flying is still an expensive impossibility. That
partly explains why the
government tinges its aviation policies with populist shades, even as it talks up
the need for reform.

. Page 7 of 40
16. Which of the following, according to the passage, is not true?
(a) Two decades ago, no-frills carriers were non-existent.
(b) Strict rules are not the only aspect that destroys the profitability of
airlines and make flying an awful
experience.
(c) Shiny new airports devoid of passengers are not likely to do much to urge local
economies to grow.
(d) The ghost airports are getting a lifting up as part of the India's well-
intentioned airports policy.
17. Which of the following does the sentence underlined in last paragraph of the
passage suggest?
(a) For majority of the citizens in India, flying is cheap.
(b) For majority of the citizens in India, flying is neither cheap nor expensive.
(c) For majority of the citizens in India, flying is not at all cheap.
(d) For majority of the citizens in India, flying is not an option.

18. Which of the following as understood from the passage accounts for the
prosperity in India’s aviation industry?
(a) Strict adherence to government regulations.
(b) Improved air-traffic management.
(c) The number of passengers carried by its airlines.
(d) The kind of services provided by its airlines.
19. As understood from the passage the 5/20 rule demands or necessitates which of
the following in order to start
international operations?
(a) A fleet of 5 aircraft and operational experience of 20 years.
(b) A fleet of 20 aircraft and operational experience of 5 years.
(c) A fleet of 20 aircraft and operational experience of 5 months.
(d) A fleet of 20 aircraft and operational experience of 5 weeks.
20. Which of the following reflects the author’s tone in the passage?
(a) Qualified approval for the aviation industry.
(b) Qualified approval for the government’s aviation policies.
(c) Censuring the government for inept handling of the aviation industry.
(d) Ruminating the future of the aviation industry in the present government’s
hands.
21. Which of the following sums up the central idea of the passage?
(a) India's aviation boom: a sordid story.
(b) The government’s unfailing attempts at aviation revival.
(c) The stupendous aviation boom.
(d) The government’s bumbling aviation boom.
Passage (Q.22-Q.26): Today, African economies export low-value-added goods relative
to their imports. Instead
of growing their own food to feed their people, countries import foodstuffs. While
some nations export crude
hydrocarbons, many more import refined petrochemicals such as gasoline. The right
to bring in these essentials
is handed over to a politically connected business "rentier" class that has a
vested interest in the status quo. There
is a demand for jobs, a hunger for education and a desperate need for health in
Africa. Yet leaders are caught in
a (______): if they create money to spend on social cohesion, they risk increasing
food, energy and capital goods
imports, and increasing their trade deficit. That puts downward pressure on the
national currency. A weak
exchange rate means that imports of basic necessities will be more expensive.
History is littered with examples
of violent revolutions preceded by price spikes.
Economic orthodoxy has no answer. Its textbooks would have African governments
instructing central banks to
borrow US dollars to prop up the local currency and prioritising foreign creditors
with austerity. Africa's stunted

. Page 8 of 40
development demonstrates that poor states continue to be impoverished by being
integrated into the world system
through a relationship of unequal economic exchange with wealthy states. An
alternative African strategy would
see governments spending on public services and on increasing food and renewable
energy sovereignty, while
cracking down on corruption. This provides a way out of the current development
trap.
State-owned enterprises and a competitive domestic private sector would help Africa
evade activities demanded
by the global north. As African countries become increasingly digital, data will be
power in economic
governance - and local entities must be its custodian, not transnational
corporations. Trade agreements between
countries of similar income levels are more beneficial for them compared with the
World Trade Organization's
framework. The African Continental Free Trade Area, created by 54 of the 55 AU
nations, is a good start. African
economies would benefit by producing green industrial goods that rich countries
take for granted, but whose
mass production has not reached the continent. It would be in Europe's interest to
help as more Africans would
be able to find jobs at home, pressure to migrate would ease.
22. According to the passage, which of the following can be inferred?
I. The economies of Africa continue to be impoverished.
II. Africa needs to create currency to increase their imports of basic amenities.
III. The political affluent cartel has a vested interest in Africa’s stunted
growth.
(a) Only I (b) Only II (c) Both I and III (d) Only III
23. Which of the following can be inferred from the downward pressure on national
currency?
(a) A downward pressure on national currency brought about by an increase in the
demand for the foreign
currency due to an increase in demand for imports leads to devaluing of currency.
(b) A downward pressure on national currency brought about by a decrease in the
demand for the foreign
currency due to a decrease in demand for imports leads to devaluing of currency.
(c) A downward pressure on national currency brought about by an increase in the
demand for the foreign
currency due to an increase in demand for exports leads to devaluing of currency.
(d) An upward pressure on national currency brought about by an increase in the
demand for the foreign
currency due to an increase in demand for imports leads to rise in value of
currency.
24. Which of the following words would most suitably fill in the blank given in the
passage to make the sentence
coherent?
(a) dire straits. (b) catch 21. (c) confusion. (d) dilemma
25. According to the passage, which of the following is/are the proposition(s) made
by the author that could improve
the economic situation in Africa?
I. Beginning trade agreements between countries of similar income levels.
II. Prop up the local currency by borrowing US dollars.
III. Increase the money spent on public services and renewable energy.
(a) Only I (b) Only II (c) Both II and III (d) Both I and III
26. Which of the following could be the most appropriate title for the passage?
(a) African economy and the world.
(b) The African Continental Free Trade.
(c) Rejuvenating African economy.
(d) Unstable African Currency.

. Page 9 of 40
Passage (Q.27-Q.30): Do animals have free will? Probably, the answer to that
question would be agreed by
most people to be a fairly obvious "no." The concept of free will is traditionally
bound up with such things as
our capacity to choose our own values, the sorts of lives we want to lead, the
sorts of people we want to be, etc.
and it seems obvious that no non-human animal lives the kind of life which could
make sense of the attribution
to it of such powers as these. But in thinking about free will, it is essential,
nevertheless, to consider the capacities
of animals. Even if animals cannot be said to have full-blown free will, animal
powers of various sorts provide
a kind of essential underpinning for free will which philosophers who focus too
exclusively on the human
phenomenon are forever in danger of ignoring. And these simpler capacities are
interesting enough to raise many
philosophical issues all by themselves; indeed, I would argue that they raise the
most discussed problem in this
area of philosophy all by themselves. For they are, in my view, hard to accommodate
within certain conceptions
of the universe in which we live - what might be called mechanistic or
deterministic conceptions of that universe.
This makes it very useful and important to think about the simpler capacities from
a philosophical perspective.
Instead of asking, as philosophers constantly do, whether free will is compatible
with determinism, we should
first ask ourselves whether even the simpler powers which constitute what I call
animal agency are consistent
with it. And it might conceivably be that the answer to this question is "no,"
which would boon a new and
interesting kind of light on the free will debate.
27. Which of the following questions can be answered conclusively from the
information contained in the passage?
I. Do animals have free will?
II. What essential thing should be taken into account when dealing with the subject
of 'free will' of animals?
III. What are the most discussed problems in the area of philosophy which deals
with the 'free will' of animals?
(a) I only (b) II only (c) III only (d) I and III only
28. "Do animals have free will? Probably, the answer to that question would be
agreed by most people to be a fairly
obvious "no". Which of the following gives the reason for the obvious negative
response from people?
I. Most people believe that the free will that humans enjoy is similar to that
exercised
by animals.
II. Nature commands every animal, and the beast obeys but man experiences the same
impression, but he
recognizes himself free to acquiesce or to resist.
III. Most people have limited understanding of the perceived concept of free will,
and on finding them missing
in animals, it is perceived as a negation of free will in animals.
(a) Both II and III (b) Only I
(c) None of the above (d) Only III
29. The author presents the passage in a _________manner. Which of the following
will fill in the blank most
accurately to reflect the approach of the author?
(a) Polemical (b) Facilitating (c) Exploratory (d) Abrupt
30. Which of the following words replaces the highlighted word in the passage in a
grammatically and contextually
correct way?
(a) Shade (b) Shed (c) Banal (d) Trite
Passage (Q.66-Q.70): Restrictions on the Freedom of Association: In Haji Mohd.v.
District Board , Malda ,
it was held that a restriction requiring a teacher to take prior permission to
engage in political activities is a
reasonable restriction. It aimed at preventing teachers from getting mixed up with
political institutions. For, a
teacher is not merely a citizen but he has to be under certain terms and discipline
of employment. Sometimes,
restrictions are put to protect the sovereignty and integrity of India.
A Government order requiring municipal teachers not to join unions other than those
officially approved was
held to impose earlier restraint on the right to form association and union, which
was in the nature of
administrative censorship, and thus invalid in Ramkrishna v. President, District
Board, Nellore.
The constitutionality of the Hindi Sahitya Sammelan Act, 1962 was questioned in
Damayanti v. Union of India
as a violation of Article 19(1) of the Constitution. The petitioner belonged to an
organisation. The Act altered
the organization's composition and added new members. As a result of this change,
members who willingly
formed the association were now obligated to act in the association with other
members over whom they had no
choice in admission. The Supreme Court held: The Act violated the rights of the
original members of the society
to form an association guaranteed under Art 19 (1) (c). “The right to form an
association”, the Court said,
“necessarily implies that the person forming the association also has the right to
continue to be associated with
only those whom they voluntarily admit in the association.
Any law that introduces members into a voluntary association without giving the
members the opportunity to
keep them out, or that takes away the membership of those who have willingly joined
it, is a law that violates
the freedom to create an association." The Supreme Court further stated that
fundamental rights (in this case, the
right to association) apply solely to legislation enacted by the government or
administrative acts taken by the
government, and do not apply to private individuals' actions.
SOURCE: Extracted little from
http://www.legalservicesindia.com/article/2302/freedom-of-association.html
and some excerpts taken from the judgement of Dharma Dutt v. Union of
India#https://www.advocatekhoj.com/library/judgments/index.php?go=2003/november/
44.php The passage has been
further edited and revised.
66. Kisan Mazdoor Union Limited, a cotton textile cooperative private company,
offered an employment contract
of two years to Sarvagya Jain. One of the clauses in the employment contract
provided that Sarvagya must join
Kisan Mazdoor Sangh, one of the trade unions active in Kisan Mazdoor Union Limited.
Decide which of the
following propositions can be most reasonably inferred through the application of
thegiven legal passage ?
(a) The employment contract offered to Sarvagya to joinKisan Mazdoor Sangh is legal
as it does not restrict his
freedom not to join any association.
(b) The condition requiring Sarvagya to join Kisan Mazdoor Sangh cannot bind him as
it impinges on his
freedom not to join any association.
(c) Sarvagya cannot claim a fundamental right to freedom of association against
Kisan Union Limited and
therefore, the contract would bind him even though his freedom of association is
restricted.
(d) The employment contract infringes Sarvagya’s freedom to decide with whom to
associate and therefore is
legally not enforceable.
67. Following the facts of the previous question, what would be the legal
consequences if Parliament enacts a law
which requires every employee to join the largest trade union in their workplace
mandating Sarvagya Jain to join
Kisan Mazdoor Sangh ?
(a) Such a law would merely govern private action to which fundamental rights do
not apply.
(b) Such a law would not curtail any individual’s right to freedom of association.
(c) Neither the employment contract, nor the law of the parliament would be
enforceable as they would curtail
the freedom of association.

. Page 17 of 40
(d) The law of Parliament would violate an individual’s freedom not to join any
association and therefore be
unconstitutional.
68. Following the facts of the question preceded by the previous one, what would be
the legality if Kisan Mazdoor
Union Limited enter into an agreement with Kisan Mazdoor Sangh wherein the former
agrees to hire only the
existing members of Kisan Mazdoor Sangh as employees ?
(a) The agreement would be illegal as it would curtail the union members’ right to
decide with whom they would
like to associate.
(b) Such an agreement would infringe the union’s right to decide with whom to
associate and therefore is legally
not enforceable.
(c) The agreement would not be enforceable as it would infringe upon the employer’s
right not to joinan
association.
(d) The constitutionality of this agreement cannot be contested on grounds of
contravention of fundamental
rights as such rights are not applicable to private persons
69. Which among the following cannot be referred from the given passage?
(a) Right to association also includes the right to alter the objective of one’s
association.
(b) Right to association can be invoked against policies made by the state as it is
not absolute innature.
(c) Both a and b can be inferred from the above passage.
(d) Neither a nor b can be inferred from the above passage.
70. Rajasthan has recently witnessed a 20 day long violent protest by the workers
from different factories in the state
demanding a safe and secured work environment. Fearing that such protest might fire
up again, the government
passed a legislation restricting formation of any worker’s union and holding
gatherings of any existing union.
Albert, the head of the department at a textile factory, wishes to form a worker’s
union to support the cause of
the workers. He was restricted under the newly passed legislation.
Albert challenges the same in court of law. Decide on the legality of this case.
(a) Albert will succeed as it is a violation of the fundamental right to freedom of
association.
(b) Albert will not succeed as he can join some already existing worker's union.
(c) Albert will not succeed as the government can put reasonable restrictions on
fundamental rights.
(d) None of the above are appropriate.
Passage (Q.71-Q.75): Supreme Court in Joginder Kumar v. State of U.P., had dealt
with the contours of Article
21 of the Constitution of India with regard to the arrest of an accused to the
effect that the power to arrest cannot
be exercised in isolation, and that it must have justification for the exercise of
such power, as no arrest can be
made in a routine manner on a mere allegation of commission of an offence made
against a person, without
reasonable satisfaction reached after some investigation as to the genuineness and
bonafides of a complaint and
a reasonable belief qua the person’s complicity and the need to necessitate such
arrest.
Further, in Arnab Manoranjan Goswami v. State of Maharashtra, Supreme Court
reiterated the value of the
personal liberty enshrined under Article 21 of the Constitution of India. The
Supreme Court further emphasized
that the basic rule behind bail jurisprudence is “to bail not jail”. The Court went
on to observe that it is our
earnest hope that our courts will exhibit acute awareness of the need to expand
that footprint of liberty and use
our approach as a decision-making yardstick for further cases for the grant of
bail.
The fraudulent and dishonest intention should be present since inception for an
offence of Cheating.
Further, it was stated that since the investigation was complete, there was no
apprehension of tampering with
any documents, influencing witnesses or absconding from the trial. Hence, Court
satisfied the triple test laid
down by the Supreme Court in P. Chidambaram v. Directorate of Enforcement. The
triple test involves:
1. That the accused is not at “flight risk”; or
2. That there are no chances of tampering with evidences; or
3. That there is no likelihood that the accused shall influence the witnesses.

. Page 18 of 40
(Extracted with requisite revisions and edits from ‘Law on Bail | Investigation
complete, charge sheet filed,
accused in jail since 6 months: Read whether Del HC grants bail’ by Devika Sharma
at
https://www.scconline.com/blog/post/2022/02/08/law-on-bail/)
71. Sunil saw a job opportunity which was posted by a University. The University
was looking for a professor and
posted the qualification for the job. The qualification required was the applicant
must have procured Ph.D. Sunil
was enrolled in a Ph.D. program but did not complete his degree. He honestly
thinking that he will receive his
degree before the date of interview and therefore, applies for the job. Sunil was
selected by the University
believing that he holds Ph.D. Sunil did not deny accepting the job offer. The
university later on found out that
Sunil did not possess the required qualification and filed a complaint against
Sunil. Decide whether University
would succeed:
(a) No, as there was no dishonest intention from the inception when Sunil applied
for the job.
(b) No, as when Sunil applied for the job there was fraudulent act done by him but
there was no dishonest
intention.
(c) Yes, as when Sunil accepted the job he committed the offence of cheating as
there was fraudulent and
dishonest intention.
(d) No, as when Sunil applied for the job there was dishonest intention but there
was no fraudulent act done as
Sunil acted in good faith.
72. In accordance with the above stated passage decide which statement is true
regarding bail:
(a) Article 21 prohibits curtailment of personal liberty which includes jail.
(b) The court must prefer to jail a person.
(c) Court must be very cautious while deciding the application of bail.
(d) Court must decide the application of bail on its own without taking into
consideration any fact.
73. A is accused of killing B. He could not be found by the police as he kept on
moving from one place to another.
Later on, after trial began, A was found by police and was produced before court. A
filed a bail application before
the court. A contended that he will be present before the court when ordered.
Decide whether bail can be granted
to A or not:
(a) Yes, as A has already been found by police.
(b) Yes, as he promised to present before the court.
(c) No, as he was found by police with difficulty and had history of running away.
(d) Yes, as although he made attempt to run away in past no such act was done after
trial started and therefore,
thus he won’t run away.
74. M is accused of stealing goods from N’s house. The stolen goods have not been
found by the police. The police
has intel that if M is left loose, he will make the stolen goods disappear. M was
caught by police and police
contended that he should be jailed. The police found out that the stolen goods are
kept outside the jurisdiction of
the court. The court issued an order to restrict the movement of M just to the
jurisdiction of the court. M applied
for bail from this restriction. Decide whether court should grant bail:
(a) No, as there is a risk of him running away due to which court also made an
order restricting his movement.
(b) No, as he could make the stolen goods disappear and therefore, will make it
difficult to recover it.
(c) No, as there was risk of him running away and influencing the witnesses.
(d) Yes, as bail cannot be denied just because of certain assumptions of police.
75. In accordance with the above stated passage, decide what is true regarding
bail:
(a) The flight risk, tampering of evidence and influencing of witness must all
exist in a case to deny bail.
(b) The triple test lays down the sequence in which all the factors to consider
bail application must exist.
(c) Only one of the three factors in triple test must exist in a case.
(d) One of the factors out of triple test laid down must exist for denying bail.

. Page 19 of 40
Passage (Q.76-Q.80): Civil Defamation - Section 306 of the Indian Succession Act
which speaks of the rights
of administrators and executors of the estate of the deceased, does not bar family
members and near relatives
covered by Section 499 of the Indian Penal Code from seeking injunction - A right
in tort may arise when any
imputation concerning a deceased person harms the reputation of that person, if
living or is intended to be hurtful
to the feelings of his family members or other near relatives.
Mere hurting of sensibility is not defamation, if the person said to be defamed is
not lowered in character or
credit in the eyes of others.
The IPC safeguards a person’s reputation under Chapter XXI sections 499-502.
Section 124A of the Code
prohibits defamation of the state, and Section 153 of the Code prohibits defamation
of a class, such as a
community, and Section 295A prohibits hate speech that offends religious
sentiments.
Civil defamation is based on harm to a private entity, but criminal defamation is
based on harm to community.
The criminal law of defamation is codified, but the civil law of defamation is not.
According to Section 500 of
the IPC, defamation is punishable by up to two years in imprisonment or a fine.
Interim Relief - The court has to consider the prima facie case made out by the
applicant for interim relief, both
on the question of locus standi to sue, if questioned and on the merits of the
prayer for interim relief. The Court
also has to consider the balance of convenience. (Para 21)
(Source: Mere Hurting of Sensibilities Not Defamation; CBFC Certificate Prima Facie
Shows Film Not
Defamatory)
76. Select an alternative that do not illustrates an anomaly of the passage's
context:
(a) Both civil defamation and criminal defamation are codified laws.
(b) Civil defamation is based on harm to the community, but criminal defamation is
based on a private entity.
(c) Section 500 of the IPC makes criminal defamation a crime.
(d) None of the above.
77. Arya is a writer based in Noida. Arya’s daily ranting and obnoxious antics see
him added to the no fly list, being
labeled an anti national, beaten up in a restaurant, and more. Despite all those
issues, the biggest thorn in his life
as a famous pulp fiction writer is his rival and India’s best-selling author, Hetan
Bagat. As Arya launched his
new book, he requested his readers to provide constructive feedback on it. A reader
highlighted one of the book's
excerpts and compared it to Arya's life. Decide
(a) It is not defamation as the author has exposed his work to the judgment of the
public or spectators.
(b) It is not defamation as it is an honest feedback.
(c) It is defamation as the comment harmed the reputation of Arya in the eyes of
public.
(d) It is defamation since the imputation affected Arya's sensibilities.
78. Arya launched his new book, a pulp fiction. A reader posted one of the book's
paragraphs on witter a social
media platform, related it to Arya's life, claiming that it is the reason his wife
is leaving him. Decide
(a) It amounts to defamation as it affects the reputation of the person.
(b) It does not amount to defamation as it is honest review.
(c) It does not amount to defamation as Truth is an absolute defence.
(d) It amounts to criminal defamation, and the reader will be prosecuted under
provision of Chapter XXIsections
499-502 of IPC.
79. Chetan is a Chartered Accountant, and he used to make fun of all the Company
Secretaries, saying that they are
inferior to them in terms of income and societal acceptance. Chetna, a Company
secretary who is also his sister,
was outraged by his words and filed a defamation complaint against him. Will she be
successful?
(a) She will succeed as Section 153 of IPC prohibits defamation of a class.
(b) She will succeed as Section 153 of IPC prohibits defamation of a community.
(c) She will not succeed as Chetan addressed a remark on the Class of CS and not
specifically on Chetna.
(d) She will not succeed since a class action suit must be brought by a set of
people.

. Page 20 of 40
80. Assertion: In a religious conclave two known communities defamed each other.
Hence prosecuted under
provisions of IPC.
Reason: Section 295A prohibits hate speech that offends religious sentiments.
(a) Both A and R are true but R is not correct explanation of A.
(b) Both A and R are true and R is correct explanation of A.
(c) A is true but R is false.
(d) A is false but R is true.
Passage (Q.81-Q.85): A contract represents a binding agreement between two or more
parties. If you abandon
your business or one party fails to perform its obligations, the other parties may
sue for breach of contract. There
are, however, a number of defenses to a breach of contract claim. For example, if
there is evidence the parties
have ignored or “abandoned” the contract, the agreement is no longer enforceable in
court.
Abandonment is normally understood, in the context of a right and not in the
context of a liability or obligation
- a party to a contract may abandon his rights under the contract leading to a plea
of waiver by the other party,
but there is no question of abandoning an obligation - the refusal to perform the
obligations, can perhaps be
termed as breach of contract and not abandonment or breach.
The Supreme Court, on Wednesday, held that the refusal of a contractor to continue
to execute the work, unless
the reciprocal promises are performed by the other party, cannot be termed as
abandonment of contract.
Moreover, when material alteration takes place in the terms of the contract due to
the act of one party, the other
party can choose not to perform the original contract and it would not amount to
abandonment.
Referring to Section 67 of the Indian Contract Act, 1872, the Court stated - "...if
any promisee neglects or refuses
to afford the promisor reasonable facilities for the performance of his promise,
the promisor is excused by such
neglect or refusal."
"The refusal of a contractor to continue to execute the work, unless the reciprocal
promises are performed by the
other party, cannot be termed as abandonment of contract. A refusal by one party to
a contract, may entitle the
other party either to sue for breach or to rescind the contract and sue on a
quantum meruit for the work already
done."
(Source: with revision and edits from, Shripati Lakhu Mane v. The Member Secretary,
Maharashtra Water
Supply and Sewerage Board and Ors., livelaw)
81. Mehar was a world-class shooter. She was a current exchange student at Harvard
Law School. In the winter
Olympics, she also won gold medals. Her journey drew the attention of ACC news, a
global news channel. They
sought Mehar for interviews in order for her to convey her personal story and her
journey from India to Harvard
to the Olympics. JJ, a contract killer, held Mehar captive in her own apartment and
forced her to sign a contract
with him, which included a condition that if she won gold in the following Olympics
season, she would be paid
Rs 100 crores. Mehar signed the contract since she thought it was a fair bargain.
What would constitute contract
abandonment in these circumstances?
(a) Mehars’s waiving off her share of 100 Crores after winning the gold medal.
(b) Mehar not being able to gain any place in the upcoming Olympic season due to
her poor performance.
(c) Mehar’s being absent on the day of game.
(d) All of the above.
82. Prem moved to Canada lately. Prem married Poonam in court on April 10th, 2021,
before heading to Canada
and promised to take her there within a month. Since then, an year has passed
during which Poonam has fought
tooth and nail to persuade Prem to let her accompany him to Canada in order to
fulfil their marriage obligation.
Prem reciprocates by inviting Poonam to travel to Canada on the condition that she
passes the IELTS
(International English Language Testing System). Poonam is adamant about not doing
so because she wants
Prem to take her with him. Can Poonam claim breach of contract?
(a) Yes, as it is Prem’s duty as a husband to fulfill matrimonial obligation.

. Page 21 of 40
(b) No, as the refusal to perform the obligations can perhaps be termed as
abandonment and not breach of
contract.
(c) Yes, the refusal to perform the obligations can perhaps be termed as breach of
contract and not abandonment.
(d) If considered marriage a contract, then perhaps it can be termed as breach of
contract.
83. Smiti and Kanha planned a lavish Indian wedding at the Six Senses Fort Barwara
in Rajasthan. They've hired a
variety of musicians to perform at the wedding's multiple ceremonies. Badshah, B
prakk, and Salim Merchant
were among the singers asked to perform at the wedding. The wedding planners
emailed the erroneous invitation
to the artists regarding their performance dates, which caused a lot of confusion
throughout the wedding events.
Due to change in the dates, all of the artists had to cancel at the last minute.
Decide accountability of artists?
(a) The artist shall be held liable for contract violations.
(b) The sole responsibility for the cancellation of live performances is on the
wedding planner's error.
(c) The artist will not be held accountable for breach of contract since the
wedding planners' negligence led to
the cancellation.
(d) Wedding planners bear complete responsibility for failing to fulfill their
obligations and hence no
accountability of artists arises.
84. Continuing on the similar facts as above, can artist claim compensation from
the organizers?
(a) The artist can claim the compensation for the entire event as they had events
planned ahead which they had
to cancel.
(b) The artist cannot claim any compensation.
(c) The artist can claim compensation under section 67 of Indian contract act.
(d) The organizers can claim compensation from the artist as they have cancelled
the show last minute.
85. What defence can artists take in case they are sued for breach of contract?
Answer in accordance to the passage.
(a) Yes, as there is breach on part of organizers.
(b) No, as it was just a mistake on part of wedding organizers so as to the dates
of event.
(c) Yes, only if the other party claims breach of contract.
(d) Yes, on the basis that the organizers ignored the contract.
Passage (Q.86-Q.90): There have been numerous instances where the media has been
blamed and accused of
conducting the trial of the accused by passing the "Verdict" according to their
investigation before the judgement
is passed by the Court. It is essential that the trial must be carried out by the
Court and not the media. The trial
by the media is certainly an undue interference in the procedure of delivery of
justice.
The legislature has a great responsibility to perform while drafting laws on media,
ensuring that their freedom is
not curtailed. Media has the right to discuss and comment on the case judgments but
they have no right or
freedom to start a trial on sub-judice matters. The right of the accused to have a
fair trial is always more important
than the freedom of media before starting the trial of the pending case. Media
trial hinders the purpose of justice.
It becomes clear that the influence of the media had a more negative effect rather
than a positive effect (except
for a few exceptions). The Courts should properly regulate the media. The Courts
should not grant free hand to
the media in the Court proceedings as they are not some event of the sport.
The most favourable way for legislating the media is by exercising the contempt of
court to penalize the ones
who interfere with the basic code of conduct. The Supreme Court has approved in a
number of cases the use of
contempt powers by the Courts against the newspapers and media channels. Freedom of
speech and expression
cannot be allowed to the media to an extent to prejudice the trial itself.
(Source: Famous Cases of Media Trail, thelegalserviceindia.com)

. Page 22 of 40
86. Rachel (model turned barmaid) was working as a part-time waitress in a
restaurant owned by socialite Monica
in Saket, South Delhi, was shot dead by Phoebe Gandhi, the daughter of former MP,
Mr Gandhi, when she
refused to sell her and her companions liquor. This case rose to the top of the
list of examples where public
pressure and the media persuaded the justice system to take a second look at this
case. Though phoebe got
acquitted initially as the Delhi police failed to sustain the grounds on which they
had built up their case after
public outcry due to the media coverage of the case, the Delhi High Court sentenced
her to life imprisonment.
Decide
(a) The media is not involved in the case since there was also a public outcry for
justice to be done.
(b) The media cannot be blamed because it has the right to free speech and
expression.
(c) It may be considered a media trial.
(d) The media trial can be claimed to have impacted the decision.
87. A well-known future Bollywood actor was found hanging in his own flat in
Bandra, Mumbai. At first glance, it
appeared to be a suicide. The case quickly gained attention after the actor was
discovered dead under unusual
circumstances, but it was also revealed by the media reports that his purported
girlfriend, Riya, could be the
cause for him taking this drastic measure. It was also stated that he was
despondent and was taking anxiety pills,
as well as seeing a psychiatrist. The media inflated the case to the point where it
published multiple reports
accusing various people of being suspects in the actor's suicide. Riya, summoned by
ED, and then his brother.
What will the court's ruling be in this particular case?
(a) It will be held as a media trial, and the court may issue an injunction against
media organisations that
distribute false information.
(b) It will be case of contempt of court.
(c) The court can issue contempt proceeding against the media house spreading such
baseless news.
(d) Freedom of speech and expression cannot be allowed to the media to an extent to
prejudice the trial itself.
88. Choose a statement which illustrates an anomaly of the context of the passage
I. Media handling an issue and a matter under consideration should be mutually
exclusive events.
II. Control over media trials can only be achieved through stringent legislation
enacted by the legislature.
III. The use of the media in a trial is an unwarranted intrusion into the process
of delivering justice.
IV. A media trial can expose an innocent person to judicial scrutiny, affecting his
reputation in society.
(a) I,II (b) II (c) III, IV (d) II, IV
89. Assertion: A trial by electronic media, press or by way of public agitation is
anti-thesis to the rule of law and can
lead to a miscarriage of justice.
Reason: The media's freedom of speech and expression cannot be allowed to prejudice
the trial itself.
(a) Both A and R are true but R is not correct explanation of A.
(b) Both A and R are true and R is correct explanation of A.
(c) A is true but R is false.
(d) A is false but R is true.
90. The passage states that it becomes clear that the influence of the media had a
more negative effect rather than a
positive effect (except for a few exceptions). What, in your opinion, could be the
few exceptions? Choose among
the following statement.
(a) Due to so much outrage in the media which brought multiple amendments in the
laws including the Juvenile
Justice Act.
(b) The personal life of the accused had been pierced by the tormenting eyes of the
media which paved the way
for fresh debate in the murder trial issue of the accused.
(c) A case through which the journalism ethics had been again under the
controversial debate due to their
meddling with the personal matter of the accused.
(d) All of the above

. Page 23 of 40
Passage (Q.91-Q.95): A recent Judgment of the Supreme Court read:
On a consideration of the rival submissions, the court observed to have no option
now, but to pass this order with
a view to safeguard the fundamental rights of citizens during this grim period in
our country's history. This Court
had readily agreed to the request of the learned Advocate General of Karnataka, for
time to revert to this Court,
with a practical solution to the problem faced by the residents in Kasargod
District, in Kerala, who were
prevented from travelling to Mangalore, in Karnataka, for urgent medical treatment.
The said restrictions
imposed by the State of Karnataka, through the blockades erected for the purpose,
has resulted in the loss of
many lives in the last two days.
The right of a citizen to move freely throughout the territory of India, subject to
reasonable restrictions that may
be imposed in the interests of the sovereignty and integrity of India, the security
of the State, public order etc. is
recognised under Art.19 (1)(d) of our Constitution. A citizen also has a
fundamental right to life and personal
liberty guaranteed to him by the State under Art.21 of our Constitution. Both these
rights are simultaneously
infringed in the case of a resident of the State of Kerala when he/she is denied
entry into the State of Karnataka
for availing medical treatment, or is deprived of essential articles of food that
are being transported into the State
through blockades erected by the State of Karnataka. Restrictions imposed on the
transportation of essential
articles of food would amount to a breach of the rights protected under Arts.301-
304 of our Constitution.
The National Highways come under the administrative jurisdiction of the Central
Government and the provisions
of the National Highways Act clearly provide for the maintenance of such highways
by the Central Government,
and even provide for penal measures to be taken against anyone blocking such a
highway. The arterial roads that
connect Mangalore in Karnataka, to Kasaragod in Kerala, are part of the National
Highway network and it is
therefore the duty of the Central Government to ensure that the said roads are kept
free of blockades. No doubt,
restrictions may be imposed in times of a national emergency such as the present,
but when the guidelines issued
by the Central Government under the Disaster Management Act itself permits travel
for urgent medical treatment,
then the said guidelines nave necessarily to be enforced by the Central Government
through the removal of the
blockades that prevent such travel.
Directed, the Central Government to forth with intervene in the matter and ensure
that the blockades erected by
the State of Karnataka, on the National Highways connecting the said State to the
State of Kerala, are removed
forthwith, and without any further delay, so as to facilitate the free movement of
vehicles carrying persons for
urgent medical treatment, across the border between the two States. We may re-
iterate that we expect the Central
Government to act expeditiously in this matter, taking note of the human lives that
are at stake.
(Excerpt taken from the judgement of JUSTICE K S PUTTASWAMY (RETD.), AND
ANR. ..Petitioners
VERSUS UNION OF INDIA AND ORS)
91. Even after the ordered pronounced by the apex court on the issue of blockade of
national highway by Karnataka
government, the said order has been overlooked. The Karnataka government decided to
go against the order and
block the road that connects its district to the aforementioned district of Kerala,
what can be the most likely
outcome?
(a) Penal measures can be taken against the Karnataka government as it disobeyed
the court’s order.
(b) The blockade will have to removed as it is a time of emergency.
(c) The Karnataka government will have to serious consequences of disobeying apex
court’s order leading to
contempt.
(d) The central government will take over the government of Karnataka as it has
been creating chaos in the said
times of emergency.

. Page 24 of 40
92. Archana a resident of Sonauli, a town, near city of Maharajganj in Maharajganj
district in Uttar Pradesh, India.
It located on the Indo-Nepal Border and is a well-known and most famous transit
point between India and Nepal.
Residents of Sounali and the town across the border, "Chichau," have a friendly
relationship, allowing for cross#border student education. The state government of
Uttar Pradesh issues an order prohibiting such movement.
Archana, a scholar, petitions the court for the ban to be lifted. Will she be
successful?
(a) Yes, as the constitution recognizes the right of a citizen to freely.
(b) No, but if she establish such ban can only be ordered by central government.
(c) Yes, as Archana was unable to travel for education and it affected her family
too so the government will
have to remove the ban.
(d) No, as the ban doesn't violate any fundamental right of the citizen
93. If the Central Government passes new guidelines banning even the travel for
urgent medical treatment from one
state to another, then what would change in the outcome of question 1?
(a) The blockade will continue.
(b) The blockade will be immediately lifted.
(c) The Karnataka government will have to compensate the family of affected people.
(d) The Karnataka government can put blockades in the entire state.
94. The state government of Y passes an order restricting the transportation of
items. Sumit, a young law student,
believes that if he approaches the high court he can get the restriction lifted.
Will Sumit succeed? Mark the option
which provides a valid outcome supported by a valid argument in the context of the
passage.
(a) Yes, because it violates a citizen's right to move freely throughout the
territory of India
(b) Yes , because there is legal provision for transportation of essential items.
(c) Yes, because when a citizen is deprived of essential articles it violates his
right to life and personal liberty.
(d) No, restrictions imposed on the transportation of articles would not amount to
a breach of the rights
95. What would be the most likely reaction of the Supreme Court if the Central
government delays taking action on
the matter of dispute between the two states?
(a) The Court will rebuke the government for being slow in the matter of grave
concern such as this and ask
them to act expeditiously
(b) The court will wait for the Centre's guidelines
(c) The court will go ahead and take decisions on its own
(d) The court will not get involved in a matter involving two different governments
Passage (Q.96-Q.101): Joint liability is a term used for people who have committed
an act in pursuance of a
common intention, where each of the persons is liable in the same manner, as this
act was done by them alone.
Section 34 IPC, states the joint liability of partners in crime. The section only
provides for the constitution of
joint liability, not the punishment. This section is only a rule of evidence and
does not constitute a substantive
offence. It provides for the principle of constructive liability.
There are a few principles that guide the application of the concept of common
intention. It is necessary that
there has been a prior conspiracy relating to that act. When the offence is proved
only on the basis of
circumstantial evidence, the allegations of common intention cannot be established
in the absence of meeting of
minds. One of the basic most requirements is that there must be some criminal act.
Such a criminal act must be
done by “several persons.” Such a criminal act however would not include acts which
are merely invalid in the
law. An example would be a minor man marrying a minor woman for the marriage is
merely void and not a
crime. The common intention doctrine states that there should be an antecedent to
the occurrence. A clear
distinction is made between common intention and common object is that common
intention denotes action in
concert and necessarily postulates the existence of a pre-arranged plan implying a
prior meeting of the minds,
while common object does not necessarily require proof of prior meeting of minds or
pre-concert. Common
intention has to be ascertained in the investigations, however, it is not necessary
that it happens before the
occurrence only. It might happen during the occurrence of the act as well. The
intention is curated at any given

. Page 25 of 40
point in the action. Usually, an overt act would be sufficient to show that there
is common intention; however
the absence of the same also does not vitiate the applicability of common
intention. The law requires that the
accused must be present on the spot during the occurrence of the crime and take
part in its commission; it is
enough if he is present somewhere nearby.
96. A, B and C decided to commit burglary. They broke into a locked house. However,
before they had finished
their work, a domestic servant appeared from the out-house and started shouting. A,
B and C left the house and
started running away. They were pursued by a small crowd. A, on being caught by X,
one of the persons pursuing
them, stabbed him with a knife kept at road side and ran away. By the time B and C
had disappeared. X died on
account of the stab wounds. Later, the police arrested all the three. They were
charged for attempted burglary
and murder of X.
(a) Along with A, B and C are also guilty of murder because A stabbed X in
furtherance of common intention.
(b) Along with A, B and C are also guilty of murder because A, at the time of
stabbing X, was acting on behalf
of B and C and he wanted to save not only himself but B and C as well.
(c) A alone is guilty of murder because though there was common intention to commit
the offence of burglary,
there was no common intention to commit the offence of murder.
(d) None of these.
97. Please refer to the facts above. While A was stabbing X, B stood next to A in a
jeep honking for A to come in
jeep after stabbing, do a clean job and run as soon as possible. Decide the
liability of A and B.
(a) Along with A, B and C are also guilty of murder because A stabbed X in
furtherance of common intention
(b) A alone is guilty of murder because though there was common intention to commit
the offence of burglary,
there was no common intention to commit the offence of murder
(c) Along with A, B is also liable for the offence as he stood next to A in a jeep
waiting to rescue A from the
crowd.
(d) None of these
98. Abhishek, Boru and Chetan conspire to rob Peter’s house for he had an affair
with Abhishek’s sister. When they
were committing such robbery, they were caught and thus they had to run. During
such running Abhishek was
caught by Rathore. He stabbed Rathore. Boru was waiting right next to the place of
murder on a jeep. After
Abhishek stabbed Rathore, he went away along with Boru in the jeep. On their way
they found Chetan running
and they took Chetan along with them to run away quicker. Decide
(a) All of Abhishek, Boru and Chetan are liable for murder. The very fact that they
escaped together shows that
common intention was present in all the 3.
(b) Abhishek alone is guilty of murder because though there was common intention to
commit the offence of
burglary, there was no common intention to commit the offence of murder
(c) The three had only run to save their lives from the mob. They acted in self-
defence and hence are not liable
(d) Abhishek and Boru are liable. Chetan joining them in their jeep after the
murder had been committed does
not show common intention
99. A has romantic attachment with B for the past 25 years. Both of them want to
marry each other but their parents
are not agreeing to the same. A, along with his friends C and D decides to run away
with B and marry in the
nearby temple. Decide
(a) Elements of Section 34 are satisfied for there is presence of common intention
(b) Elements of Section 34 are not satisfied for there is nothing showing pre-
meditation
(c) Elements of Section 34 are satisfied for the four should have spoken to their
parents
(d) Elements of Section 34 are not satisfied for no criminal act has been committed

. Page 26 of 40
100. Please refer to the facts above. A is 20 years of age and B is 17 years of
age. Both of them go and marry in the
nearby temple. Decide
(a) Elements of Section 34 are satisfied for there is presence of common intention
(b) Elements of Section 34 are not satisfied for there is nothing showing pre-
meditation
(c) Elements of Section 34 are satisfied for the four should have spoken to their
parents
(d) Elements of Section 34 are not satisfied for no criminal act has been committed
101. A is a mastermind who controls all criminal activities in the city. He wishes
to get the Mayor of the city murdered
for the Mayor refused to give him a highway tender. He hires 3 goons to murder the
Mayor and went to a trip to
USA so that no suspicion is brought on him.Goons successfully murder the Mayor.
Decide
(a) A and the goons are liable for murder for there is presence of common intention
(b) Only A is liable for the murder for he hired these goons to commit the offence
(c) Only the goons are liable for the murder. A was not in the vicinity during
commission of the crime
(d) None of them are liable. A was justified in hiring the goons to murder the
Mayor.
Passage (Q.102-Q.105): A floor test is a motion through which the government of the
day seeks to know whether
it still enjoys the confidence of legislature in the seats therefrom the time of
announcement of election. In this
procedure, a CM appointed by the Governor can be asked to prove majority on the
floor of the Legislative
Assembly of the state.
 A confidence motion or a vote of confidence or a trust vote, is sought by the
government in power on the
floor of the House.
 It enables the elected representatives to determine if the Council of Ministers
command the confidence of
the House.
 The idea underlying the trust vote is to uphold the political accountability of
the elected government to the
State legislature.
 A no-confidence motion, or vote of no-confidence, or a no-trust vote, can be
sought by any House member
to express that they no longer have confidence in the Government.
This happens both in the parliament and the state legislative assemblies. In
situations when there are differences
within a coalition government, the Governor can ask the Chief Minister to prove
majority in the house. The chief
minister has to move a vote of confidence and win a majority among those present
and voting. If the confidence
motion fails to pass, the chief minister has to resign. The idea behind a floor
test is to ensure transparency in the
constitutional process
If there is more than one person staking claim to form the government and the
majority is not clear, the governor
may call for a special session to see who has the majority. This test is called as
Composite Floor Test.
These are the modes by which voting can be conducted:
● Voice vote: In a voice vote, the legislators respond orally.
● Division vote: In case of a division vote, voting is done using electronic
gadgets, slips or in a ballot box.
● Ballot vote: Ballot box is usually a secret vote - just like how people vote
during state or parliamentary
elections.
Following the vote, the person who has the majority will be allowed to form the
government. In case there is a
tie, the speaker can cast his vote. When no party gets a clear majority, the
governor can use his discretion in the
selection of chief ministerial candidate to prove the majority as soon as possible.
In a situation where the governor has reasons to believe that the council of
ministers headed by the chief minister
has lost the confidence of the House, constitutional propriety requires that the
issue be resolved by calling for a
floor test.
The Governor’s requirement to have a trust vote does not “short-circuit” any
disqualification proceedings
pending before the Speaker. A Governor need not wait for the Speaker’s decision on
the resignation of rebel
MLAs before calling for a trust vote.

. Page 27 of 40
The pro-tem speaker's role is crucial in conducting a floor test. Conventionally,
the longest serving House
member is nominated as pro tem speaker, whose role is limited to administering oath
to new MLAs and
conducting the election of the full-time speaker.
102. In the state of Bihar KDU was ruling the government in coalition with HJP. In
the assembly of 120 seats KDU
has 42 seats, HJP has 40 seats, MNC has 17 seats and 21 seats went to independent
candidates. In the mid of the
5-year term, the coalition between KDU and HJP was broken and HJP claimed that KDU
does not have number
in the assembly to form the government. They went to the governor and asked him for
a chance to form the
government as they claimed to have the support of the 21 independent candidates.
Governor using his
discretionary power called for a special session and asked HJP to prove their
majority in the house.
(a) The governor will ask HJP to prove their majority.
(b) The governor can use his discretion to choose the CM candidate as there is no
clear majority.
(c) The governor can call for a floor test to give a chance to KDU to prove their
majority in the house.
(d) The governor does not need to give chance to KDU first as it is clearly visible
that they have lost the majority
in the house.
103. In the state of UP, elections were conducted. Four main rival parties, A, B,C&
D contested against each other
apart from few independent candidates. In the assembly of 220 seats Party A won 91
seats, Party B won 15 seats,
Party C won 89 seats and Party D won 20 seats whereas 5 seats were won by
independent candidates. The
majority mark was of 111 seats. There was a deadlock situation as the top two
parties wanted to form the
government.
(a) The governor should invite the Party with maximum seats first and give them a
chance to form the
government.
(b) The governor should call a special session and ask for composite floor test.
(c) The governor, on his discretion, can select any person as CM candidate and then
will ask him to prove his
majority as soon as possible.
(d) The governor with the consultation of the speaker will select a CM candidate
and ask him to prove his
majority.
104. In the assembly of 110 seats, Rashtriya Ekta Sangh won 62 seats and enjoyed a
clear majority in the state
assembly. Few months after forming the government, 14 MLAs of the ruling party
including few ministers
started to have dispute with the CM and also send their resignation to the speaker.
The speaker has not accepted
the resignation of those MLAs and neither they are expelled from the party. The
opposition had a meeting with
the governor informing him about the current political turmoil in the state. The
governor asks the CM to prove
his majority in the floor test.
(a) The Governor is not discharging his duty within the constitutional framework
and is acting on the advice of
the opposition, therefore it is not valid.
(b) Governor is encroaching upon the power of the Speaker who has still not decided
on the resignation of the
MLAs, therefore it is not valid
(c) It is well within the power of the Governor to ask for a floor test when he
believes that the government might
have lost majority, therefore it is valid.
(d) Governor doesn’t have to wait for Speaker’s decision but there was no
reasonable reason for him to believe
that the government has lost the majority, therefore it is not valid.

. Page 28 of 40
105. Surendra Pratap Rathore is a MLA from the ruling party in the state of Madhya
Pradesh. He was not given any
portfolio in the government. For past few months, he had been side-lined by the
party high command as well as
the government of the state because of his contrasting views on some policies.
Agitated by this, he brought a no
confidence motion in the house against the government. His motion was supported by
the opposition parties. He
was confident that the government will fall.
(a) He cannot bring a No Confidence Motion in the house because he belongs to the
party that is in power.
(b) He cannot bring a No Confidence Motion in the house because he is part of the
government and cannot go
against the government.
(c) He can bring a No Confidence Motion in the house as he is against the policies
of the government.
(d) He can bring a No Confidence Motion in the house as he is believes that the
government will not have
sufficient number in the house.

. Page 29 of 40
SECTION - D: LOGICAL REASONING
Passage (Q.106-Q.110): Our country cannot close eyes or console itself in the
matter of economic problems
by merely saying that it is not only our economy but the world economy is also in
turbulent waters. The
effects of global economic recession have been more disastrous in developing and
underdeveloped countries
than in the developed countries. Moreover, the wild fire of terrorism has
aggravated the agonies of people in
the Third World. Some world stalwarts attribute the global recession to the joint
effect of war and
international competition in the business sector.
Indian corporate leaders had been sounding alarm bells over the alleged harmful
effects of international
competition. “India for Indians first”, “we should have opened the Indian economy
for Indian businessmen and
then invited foreigners”, “strengthen Indian industry first, and then invite
multinational companies” – are some
of the typical comments appearing in the dailies and weeklies. Many contend that
they do not have a “level
playing field”. International players have superior technology and financial
leverage, and they will destroy
Indian manufacturing industry. India may eventually become a nation of farm
products and services.
Trade has over the years shrunk India’s manufacturing base, both in terms of value
addition and employment.
There is a distinct irony here. Traditional classical theory argues in favour of
free trade; yet in the Indian case,
its role has been more of deprivation for providing a stimulus to growth. The
adversity’ impact of import
liberalization is more pronounced on intermediate and capital goods industries than
on the consumer goods
industries. Again, between the two affected categories the capital goods sector is
worse hit. Since the
intermediate and capital goods industries have relatively large income and
employment generating linkages, their
erosion would have a direct negative effect on value-added and employment. Further,
their erosion also implies
the erosion of India’s industrial base.
106. Which of the following statement weakens the demand for ‘India for Indians
first’?
(a) It is a well-connected global world; nationalism has no place in it.
(b) Indian manufacturers get raw materials from the international markets.
(c) International companies employ millions of Indian youths at a better salary.
(d) The impact of free trade is limited to some sectors only. \
107. What is the apprehension of Indian corporate leaders that has been highlighted
in the passage?
(a) Indian economy will remain dependent on agriculture in case of the weak
manufacturing industry.
(b) Multinational companies will offer better technical products and ruin their
markets.
(c) International companies have an undue advantage over them in their mammoth
resources.
(d) Indian companies are not developed to compete with foreign countries.
108. What can be a logical deduction about the impact of terrorism on the economy?
(a) Terrorists aggravate the agonies of the people and destroy the industries.
(b) With reduced tourism and the destruction of infrastructure, the economy is
indirectly affected.
(c) Terrorism creates an environment of fear that affects the economy.
(d) To tackle the problem of terrorism, valuable resources are unnecessarily
wasted.
109. If one more paragraph is added to the passage to strengthen the points raised
by the author, what can be its
location and topic of discussion?
(a) In the beginning, strong foundations of the Indian economy in recent times.
(b) After the last paragraph, the government’s support to domestic industries.
(c) In the middle of the passage, disruptive strategies of MNCs.
(d) At the start of the passage, increasing grasp of globalization.

. Page 30 of 40
110. What is the impact of free trade in the Indian context?
(a) It has proved to be insipid. (b) It has activated growth.
(c) It has been detrimental. (d) It has eroded competition.
Passage (Q.111-Q.1115): Black money and tax evasion eating away the social and
moral framework of Indian
economy are undermining the socioeconomic objectives and are responsible for
manifest and lavish
consumption. Black money and tax evasion foster concentration of economic power in
the hands of undesirable
groups in the country. Apart from the huge share in Swiss Bank Accounts, the
Income-tax evaders in India have
deposited their monies in the developed countries of the world such as U.S.A, U.K,
France, Germany, Canada,
Isle of Man, Dubai, etc.
In view of the facts set out so far, it becomes necessary to look at the extent of
compliance of tax laws in India.
Though many estimates of black money have been coming forth, an attempt was made to
determine the extent
of tax evasion in the Mumbai Income Tax charge, which collected about 35% of the
Income Tax collections of
the country and 43% of the corporate tax collections. The study was made on the
basis of results of the survey
and search cases for all the years covered by such cases. It came to light that
none of the taxpayers concerned
declared for taxation purposes anything more than 25% of their true incomes after
2003.
There was, therefore, every reason to believe this estimate. However, there appears
to be higher tax evasion in
the case of companies. Some of the companies have shown their entire capital as
having come from the
countries regarded as Tax Havens. Considering the extent of Indian money stacked in
Swiss Bank Accounts,
and bank accounts of the developed countries, and comparing the same with the
annual income tax
collections of the Central Government, it appears that the real income admitted for
taxation purposes
is less than 25% . The extent of evasion appears to be very much higher in the case
of companies as the
companies have resorted to evolution of tax evasion devices in the accounts and
such methods have
not yet been properly investigated by the Income Tax Department. There are
companies which have
camouflaged their capital investments and shown it in the books as if it is
explained capital for income tax
purposes.
The deprivation of the tax dues to the Central Government by the taxpayers of the
country has resulted in huge
budgetary deficits, and consequent lack of financial capacity of the government in
promoting economic growth
to its fuller potential. The Government has been running continuous revenue and
fiscal deficits and the budgetary
position of the Central Government displays a pathetic picture for many years. The
great Kautilya states: “Public
Finance is the mainstay of the State and from the treasury comes all the power of
the Government.”
111. What does the author imply in the sentence given below-?
‘Black money and tax evasion are eating away the social and moral framework of
Indian economy.’
(a) Black money leads to social and economic disharmony in the country.
(b) Black money and tax evasions come from immoral practices and leads to social
and economic inequality.
(c) Black money and tax evasions come from immoral practices affecting the social
and economic welfare
policies.
(d) Tax evasion leads to immoral black money that is against Indian social and
moral ethics.
112. What is the purpose of the author in the passage?
(a) Revealing the fact that Indian money stashed in foreign banks is black money.
(b) Money evaded through the tax in a country is known to be black money.
(c) To highlight how black money is impending the development objectives of the
country.
(d) To emphasize the need for the government to nab the tax evaders.

. Page 31 of 40
113. What is the critical message in the statement of Kautilya, as given in the
passage?
(a) Financial Stability is most important for governance.
(b) Strength of the government can be estimated from its financial health.
(c) Without monetary support government cannot work powerfully.
(d) Tax collected by the government facilitates government to work powerfully.
114. What is a logical and rational inference based on the passage?
(a) Less than 25% of actual income is submitted for taxation purposes.
(b) Less than 25% of people in India submit actual tax.
(c) More than 75% of income is black money in India.
(d) About 25% of income is available for taxation by the government.
115. What is a valid assumption based on the passage?
(a) India fails to control corruption that leads to the accumulation of black
money.
(b) Companies have tax evasion devices including lobbying for their interests.
(c) Foreign Banks charge high fees from the customers to hide their black money.
(d) Income tax department fails to detect the stratagem of tax evaders.
Passage (Q.116-Q.120): The decade-old proposal to link all of India's major rivers
with one another was revived
with much fanfare last year. Most political parties welcomed it then as a solution
to the country's drinking water
and irrigation problems. But it has not taken long for the proposal to come face to
face with the hard reality of
planning what will be the largest project ever taken up in India. A number of
States, from Punjab in the north to
Kerala in the south, have expressed their opposition to a transfer of river waters
from their territory to other
States. This is only one of many reasons why the ambitious, many would say
unrealistic schedules for the
execution of the project have already been thrown out of gear.
The high–level task force on the project was expected to prepare the schedule for
completion of feasibility studies
and estimate the cost of the project by the end of April this year. It was to then
come up in June with the options
for funding the project. It was also expected to convene a meeting in May/June of
State Chief Ministers and
obtain their agreement and cooperation. None of these deadlines has been met and
there is no indication that
these events will take place in the near future. This is not surprising, for while
the interlinking proposal has been
spoken about for decades, all the complex engineering, economic, environmental and
social issues involved in
the project have never been carefully studied.
In fact, the one Government committee that did examine aspects of the proposal to
some extent, the National
Commission for an Integrated Water Resources Development Plan, was in 2015
ambivalent about the benefits
of interlinking the country's rivers. The drought of recent years was the context
in which the proposal to build a
grid connecting India's rivers was revived. Before another drought leads to another
round of active interest in the
project, it is necessary to come up with answers to two broad sets of questions.
The first question is, what will be the total costs and benefits of a river grid
project in economic, environmental
and social terms. The second will be, what are the different options to meet the
future requirements of water and
is the interlinking proposal the best among them. Answers to these questions will
have to address issues in
agricultural technology, patterns of water use, extraction of ground and surface
water resources, efficiency in the
consumption of water in crop cultivation, resource mobilization, human displacement
and changes in the
environment. A plan on such a scale and of such complexity as the proposal to link
the country's rivers can be
taken up only after a range of such substantive issues are analysed threadbare.

. Page 32 of 40
116. What is the main objective of the author in the passage?
(a) To highlight the objections raised by some states against the proposed project.
(b) To point out reasons as to why the proposed interlinking of rivers could not be
achieved in time.
(c) To critically analyze the pros and cons of linking all of India's major rivers
with one another.
(d) To bring to fore the political wrangling over a public utility scheme.
117. Based on the passage, which of the following is true?
(a) Though the project had been well planned, the delay occurred because of
unforeseen circumstances.
(b) The government committee, set up to examine the project, was very positive
about the outcome of the project.
(c) The water shortage of recent years gave an impetus to the ongoing proposal of
interlinking of India’s major
rivers.
(d) The river-linking project could not progress beyond the draft stage because the
techno-commercial validation
was unconvincing.
118. What could be a likely source of the passage?
(a) Project Report of NITI AYOG.
(b) A college textbook.
(c) Government of India gazette.
(d) An Integrated Water Resources Development report.
119. The author would most likely support the idea that
(a) River integration is best for meeting all water problems of states.
(b) River integration needs to be done after careful analysis of its benefits.
(c) River integration is an ambitious project and cannot be totally realized.
(d) River Integration is not possible due to geographical constraints.
120. What can be understood about the river’s integration project?
‘All the complex engineering, economic, environmental and social issues involved in
the project have never been
carefully studied’.
(a) Fancifully envisaged. (b) Injudiciously conceptualised.
(c) Non-clamantly implemented. (d) Improvidently designed.
Passage (Q.121-Q.125): The average price of diesel at the pump has just gone to a
new high, 100 plus Rs/ litre.
This came just a few days after we were told that inflation was falling. A lot of
people were upset – and
understandably so. High fuel prices drive up the cost of doing business and the
cost of living. Experts and callers
all chimed in with suggestions as to why the price is so high – excessive duty,
supply constraints in Europe,
Russia – Ukraine fight, refinery problems, the weakness of the rupee against the
dollar, price-fixing, tensions in
the straits of the Middle East. Diesel is a key energy price.
Diesel price adjustments have lagged international prices in recent years, and
budgetary subsidies have
ballooned. At the same time, such low prices and subsidies are providing incentives
for misuse, shifts to diesel
use such as luxury sports utility vehicles, escalating imports in an energy-
insecure country, and increased
pollution loads. Diesel is a heavy contributor to particulates and black soot and
to asthma, cancer, and heart
disease. On the other hand, political economy arguments are that diesel is a widely
used fuel for public transport,
budgetary subsidies are offset by central and state value-added tax (VAT), excise
and sales taxes, and finally,
diesel prices in India are high relative to incomes. Is there merit in these
arguments?
One way of testing for this formally is to compare diesel prices in India with
those prevailing in other countries,
adjusted for PPP incomes, as well as relative energy abundance. Other things being
equal, countries that export
oil (such as the Middle Eastern ones) or are relatively diversified, energy-
abundant countries (such as Canada
and the United States) can afford to keep domestic prices lower than energy-
insecure countries (such as India).

. Page 33 of 40
The evidence shows that just such a predicted relationship indeed holds. But even
accounting for this, diesel
prices were already 20 per cent below predicted levels for India in 2019; the
divergence has since doubled as
global oil prices have surged 45 per cent (from US$ 60/barrel Brent prices in 2016
to US$ 120 currently), while
domestic price adjustments have not followed. Diesel prices need a large adjustment
now (as China, for example,
has recently undertaken), given subsidies, pollution and public health costs.
Charging high road and vehicle taxes
is another option (that Singapore uses).
121. What is a valid assumption based on the passage regarding the course of action
the Indian government
can choose?
(a) Indian government can reduce excise duty on diesel to relieve the public from
high diesel prices.
(b) To set off the increased prices of brent, a new tax might be introduced.
(c) To maintain subsidies, road tax and vehicle taxes is a viable option.
(d) Government might increase excise duty on diesel to adjust with the global
prices.
122. Which of the following statement is incongruent with the views of the author
in the passage?
(a) Diesel is a luxury fuel and should not be subsidized as would be misused by the
rich who drive SUVs.
(b) Diesel is a public utility fuel and should be subsidized; otherwise, inflation
would go up.
(c) Many factors decide the price of diesel which include the global events and
relationships of local currency
with respect to the dollar.
(d) Diesel is one of the main producers of pollutants that can produce different
diseases.
123. Based on the passage, what is the most rationale long term policy a country
like India shall have?
(a) Decrease its dependence on Diesel.
(b) To change its position from an energy insecure country to an energy safe
country.
(c) Make the interrelated factors (which affect Fuel prices) positively on its
side.
(d) Increase the price of fuel at a market rate so as to balance off things.
124. What would be the stand of the author regarding a proposal to increase the
taxes on diesel?
(a) Definitely support it.
(b) Definitely not support the proposal.
(c) May support it conditionally.
(d) Likely to support for a short period.
125. What is the correct understanding of the expression ‘political economy
arguments’ from the passage?
(a) Political interests are behind the low diesel prices in India.
(b) Indians cannot afford the high diesel prices due to low incomes.
(c) How public policy on diesels will affect society.
(d) Increasing fuel prices can impact the political parties’ prospects.
Passage (Q.126-Q.130): Recent research has highlighted the notion that people can
make judgments and choices
by means of two systems that are labelled here, tacit (or intuitive) and deliberate
(or analytic). Whereas most
decisions typically involve both systems. This chapter examines the conditions
under which each system is liable
to be more effective. This aims to illuminate the age-old issue of whether and when
people should trust
“intuition” or “analysis.” To do this, a framework is presented to understand how
the tacit and deliberate systems
work in tandem. Distinctions are also made between the types of information
typically used by both systems as
well as the characteristics of environments that facilitate or hinder accurate
learning by the tacit system. Next,
several experiments that have contrasted “intuitive” and “analytic” modes on the
same tasks are reviewed.
Together, the theoretical framework and experimental evidence lead to specifying
the trade-off that characterizes
their relative effectiveness. Tacit system responses can be subject to biases. In
making deliberate system
responses, however, people might not be aware of the “correct rule” to deal with
the task they are facing and/or
make errors in executing it.

. Page 34 of 40
The idea that judgments and choices involve distinctive analytic and intuitive
components resonates with most
people’s everyday experiences. It is also an idea that has been discussed by
philosophers and scientists across at
least two millennia. More recently, the distinctive nature of intuitive and
analytic thought has been the subject
of much psychological research with many theorists postulating so-called “dual
models” of thought. Accepting
this dichotomy, a natural question is whether and when one form of thinking is more
“valid” (however defined)
than the other. At one extreme, it is tempting to think that analytic ways of
making decisions must be better.
After all, a large part of the educational process involves teaching people to
think more analytically under the
assumption that people’s untrained intuitive processes will lead them astray. On
the other hand, there is a mass
of anecdotal evidence that supports the use of intuition (as well as much that does
not!) and the term intuition
itself is often accorded a mystical status akin to truth. Moreover, people
sometimes find themselves in situations
where their analysis contradicts their intuitions. What should they do?
126. What is the most relevant observation made in the passage given above?
(a) Decision making involves analytical processes.
(b) In decision making, intuitive thoughts hinder everyday experiences.
(c) Analytical and intuitive thoughts work in tandem in cases of everyday decision
making.
(d) It is difficult to ascertain which though process to apply in decision making –
analytical or intuitive.
127. What can be concluded about the decision-making process in human beings?
(a) It is eclectic in nature. (b) It is a complicated process.
(c) It is a dualistic process. (d) It is capricious and unpredictable.
128. Given below are pairs of sentences ‘A’ and ‘B’ from the passage. You have to
read both the sentences ‘A’ and
‘B’ and decide their nature of relationship. You have to assume that the
information given in ‘A’ and ‘B’ is true
and you will not assume anything beyond the given information in deciding the
answer. Marks answer;
(a) If ‘A’ is an assertion and ‘B’ is its reason.
(b) If ‘A’ is the cause and ‘B’ is its effect.
(c) If both ‘A’ and ‘B’ are inferences.
(d) If both ‘A’ and ‘B’ are analysis.
Sentence A: This chapter examines the conditions under which each system is liable
to be more effective.
Sentence B: This aims to illuminate the age-old issue of whether and when people
should trust “intuition” or
“analysis.”
129. Which of the following inference is not based on the passage?
(a) There is a possibility of partisan approach in implementations of decision
based on intuition.
(b) Our education system does not consider the role of intuition in the thought
process.
(c) Tacit and deliberate systems work together to guide decision making process.
(d) None of these.
130. Which of one the following is tactic thinking as described in the passage?
(a) Answering an out of syllabus question in the exam.
(b) Learning how to drive a car.
(c) Planning a surprise party for the wife.
(d) Selecting who to sit with in a class on the first day of a new school.

. Page 35 of 40
Passage (Q.131-Q.135): Himalayan glaciers are undergoing rapid mass loss but rates
of contemporary change
lack long-term (centennial-scale) context. Here, we reconstruct the extent and
surfaces of 14,798 Himalayan
glaciers during the Little Ice Age (LIA), 400 to 700 years ago. We show that they
have lost at least 40 % of their
LIA area and between 390 and 586 kms of ice; 0.92 to 1.38 mm Sea Level Equivalent.
The long-term rate of ice
mass loss since the LIA has been between − 0.011 and − 0.020 m w.e./year, which is
an order of magnitude
lower than contemporary rates reported in the magazine ‘Nature’.
The MIT research paper shows the vulnerability of the Himalayan ecosystem,
something several scientific
studies, including the one by the intergovernmental panel on climate change, have
highlighted in the short term;
this leads to the lake formations due to meltwater, and these could overflow or
burst, leading to flash floods. The
papers’ authors have warned that the number and size of these lakes are increasing,
so continued acceleration in
mass loss can be expected; however, there is little consensus on how such glacial
retreat will impact hydrological
resources in the long run.
Some studies suggest a severe water shortage due to glacier loss. The Himalayan
glaciers: climate change, water
resources and water security, a 2012 report by the national research council,
suggests that at lower elevations,
glacier retreat is unlikely to cause significant changes in water availability over
the next several decades, but that
at higher elevations, there could be altered water flow into the river basin. Snow
and glacier melt contribute more
than 50% of runoff in the Indus River system and around 20% in the upper Ganga
basin, according to Divecha
Centre for climate change.
The Hindukush Himalayas (HKH) experienced a temperature rise of about 1.3 degrees
Celsius between 1951
and 2014. As a result, several areas of HKH have experienced a declining trend in
snowfall and witnessed
significant glacier retreat, a report by the ministry of earth sciences has
flagged.
Future warming in the HKS region, which is projected to be in the range of 2.6-4.6
degrees Celsius by the end
of the century, will lead to profound hydrological and agricultural impacts in the
region. It has concluded that
the government shall start long term glacier- monitoring programme to understand
the likely impact on
agriculture, water availability, and the possibility of disasters downstream.
131. What are the main points of concern raised in the passage?
1. The problem of retreating glaciers in Hindukush.
2. The deteriorating climate in the Himalayan region.
3. The global impact due to climate change.
(a) Only 1 (b) Only 1 and 2 (c) Only 1 and 3 (d) All 1, 2 and 3
132. What can be the possible implications of lakes formed by glaciers?
1. The lakes can burst and cause floods in lower regions.
2. The lakes will impact hydrological resources in the long run.
3. The flash floods in the regions can fill these lakes.
(a) Only 1 (b) Only 2 (c) 1 and 3 (d) 1,2 & 3
133. What can be concluded about the different reports given by Nature, MIT ,
National Research Council and
Intergovernmental panel in the passage?
(a) All the reports contradict one another.
(b) These reports are complementary.
(c) These reports make mutually exclusive observations.
(d) One report is dependent on the other.

. Page 36 of 40
134. What can be a valid source of the passage?
(a) The magazine ‘Nature’.
(b) The report of the ministry of earth sciences.
(c) A newspaper editorial.
(d) An environment scientist’s observation.
135. What might not be an objective of the Glacier Monitoring Programme?
(a) It will note down the changes in hydrological resources in the region.
(b) The programme will find out the possibilities of new agricultural techniques.
(c) It shall monitor any change to soil fertility and capability of agricultural
produce.
(d) It shall monitor the level of water in lakes formed in the upper region.

MOCK 38

SECTION-A: ENGLISH LANGUAGE


Directions (Q.1-Q.30): Read the following passage carefully and answer the
questions that follow.
Passage (Q.1-Q.5): "To teach is to touch lives forever" is a popular phrase that
teachers like to be reminded
of. I know this because they almost all have it framed on their desk, upon their
office walls as posters, or have a
t-shirt with the theme on it. Another is "To teach a child is to change the world."
I haven't been a teacher, but I
work with children and with children's health at The Little Gym, and I volunteer at
the homeless centre's
children's activity room. I can't say that I've touched or changed their lives, but
I know that they've changed my
view on love more than anything else has.
All other humans and I first learned about love from our parents. From birth, we
are nurtured and given
everything needed for survival. Through this dependence, we develop affection for
our parents. When we start
to learn more about the world, we come in contact with others. I am one of the many
people that try to develop
a child's life for the best. I am probably one of the least important people in the
process of the child's mental and
physical growth. But, some children make me feel like the most important person in
the world, that only a few
worthy people are lucky enough to see a child's special light and smile.
A child's smile is transparent. Their love can be seen through their smile. Their
smile and happiness is so innocent
and unselfish, that sometimes, we have to feel like a child to feel that kind of
love. Working with kids reminds
me to bring myself back to childhood. I have to think, what made me happy when I
was a young boy, what
frustrated me, and how I can make that situation better for the child. The groups
of children that I work with
may compare and contrast to each other in different ways. The Gym's kids are
generally well behaved and easy
to please. The homeless centre's kids are disrespectful, and not easy to please. A
favourite activity at the homeless
centre is for the children to throw toys around the room when they can't get their
way. These kids need extra
attention to calm down and to explain sharing to. But if I was a raging kid, I
wouldn't want to be talked to. I
would need other entertainment to keep my mind on and fast.
This is why kids are considered the gifts of god. Kids are and always will be
exceptional to me because I have
fallen in love with kids and my profession after working with them for so long.
Thus, I do not think I would want
to be in any other profession ever.
1. There is an emboldened quote in the opening statement of the passage. How is the
author aware of this well#known quote about teachers?
(a) The author is aware about this because he has seen many teachers wearing a
garment with the quote
emblazoned across it.
(b) The author is aware of this because he has himself used the quote many times to
inspire future
generations of teachers.
(c) The author is aware of this because he has seen many teachers with this quote
framed outside their
respective houses.
(d) The author is aware of this because he has seen many teachers who have the
quote printed and pasted
in their bedrooms as a poster.
2. What do you think is the author's profession?
(a) The author is a teacher by profession and inspires other people to choose his
line of career.
(b) The author is a linguist by profession and helps little kids to understand
tough languages.
(c) The author is a regular employee at a local gymnasium for little kids.
(d) The author is employed by a school where he is a part of the support staff to
care for kids.

. Page 3 of 40
3. According to the author, which of the following is NOT a characteristic of a
child's smile?
(a) According to the author, a child's smile is unclouded.
(b) According to the author, a child's smile is reflective of their affection.
(c) According to the author, a child's smile is sinless.
(d) According to the author, a child's smile is self-absorbed.
4. According to the passage, why would children forever be special to the author?
(a) It is because the author has no kids of his own, and considers all kids he
works with, as his own.
(b) It is because the author has developed a deep fondness for kids which he can't
let go of.
(c) It is because the author has realised that kids are very easy to please in
comparison to anyone else.
(d) It is because the author has gotten so used to kids that he fears leaving them.
5. Which of the following is TRUE about the children at the 'homeless centre'?
(a) It is not at all tough to please the kids at the homeless centre.
(b) The kids at the homeless centre are polite and extremely obedient and
reverential towards the author.
(c) The kids at the homeless centre always keep their toys in an organised manner.
(d) The kids at the homeless centre are different from the kids at the gym
Passage (Q.6-Q.10): On September 3, 2016, a magnitude 5.8 earthquake struck just
northwest of Pawnee,
Oklahoma, causing moderate to severe damages in buildings near the epicentre. It
was the largest ever recorded
in the state. The Pawnee earthquake followed the dramatic increase of seismic
events in the central United States
beginning in 2009, associated with the increase of underground wastewater disposal
by oil and gas operators.
This and other events in the area raised public concerns and led governmental
agencies to shut down injection
wells and establish new regulations regarding wastewater injections.
While human-caused earthquakes have been documented for more than a century, their
increasing number
reported worldwide has drawn much scientific, social and political attention. Such
earthquakes are related to
industrial activities such as mining, construction of water dams, injection of
liquids such as wastewater and
carbon dioxide, and extractions associated with oil and gas exploitation. With the
ever-increasing demand for
energy and mineral supplies worldwide, the number of human-caused earthquakes is
expected to rise in the
upcoming years. Some of the largest and more destructive earthquakes of the past
few years have been related
to man-made activities, such as the 2008 magnitude 7.9 Wenchuan (China) earthquake
and the 2015 magnitude
7.8 Nepal earthquake.
In most of the cases industrial activities do not induce earthquakes. But this
becomes problematic when such
activities are close to active faults. In this case, even small stresses
underground caused by man-made activities
can destabilise faults, inducing earthquakes.
In Europe, where the population density is higher than the United States, public
concern over man-made
earthquakes is greater. In Southern Europe, which has a higher risk of natural
occurring earthquakes, public
tolerance on induced earthquakes due to industrial activities is even more limited.
The deadly 2012 Emilia (Italy)
earthquake sequence became a topic of sustained public debate and political
discussion, based on the proximity
of the earthquake epicentres to an oil field. The previous cases illustrate some of
the coming challenges to be
faced with man-made earthquakes. The ability to distinguish between natural and
human-induced earthquakes
can be difficult or even impossible, especially in seismically active regions,
while in other cases the risk
associated with industrial activities is significantly underestimated. Such
problems pose novel challenges for risk
mitigation and economic growth, especially in seismically active regions such as
Southern Europe.
To significantly reduce such hazards, regulations are required that include hazard
modelling as well as
assessment before and during industrial activity that might perturb regional stress
fields. Such regulations were
recently issued in North America, including California, Oklahoma, Ohio and Texas,
as well as in and Canada.
In Europe, the EU has not yet issued any such regulations, but guidelines have been
put forth in some countries
that have experienced induced earthquakes, including the Netherlands, Switzerland,
the UK, Germany, France
and Italy.

. Page 4 of 40
6. The increasing number of human-caused earthquakes have not drawn __________
attention.
(a) Social (b) Political (c) Religious (d) Scientific
7. Why does the author feel that human caused earthquakes are expected to rise in
the upcoming years?
(a) Because the demand for energy and mineral resources is increasing worldwide.
(b) Because the demand for petroleum is increasing worldwide.
(c) Because the demand for gold and other precious metals is increasing worldwide.
(d) Because the demand for fossil fuels is increasing worldwide.
8. Which of the following earthquakes are mentioned in the passage as examples of
"largest and more destructive
earthquakes" related to man-made activities?
(a) The 2004 magnitude 9.1 Sumatra (Indonesia) earthquake and the 1920 magnitude
7.8 Ningxia (China)
earthquake.
(b) The 1923 magnitude 7.9 Kanto (Japan) earthquake and the 1948 magnitude 7.3
Turkmenistan earthquake.
(c) The 2010 magnitude 7.0 Haiti earthquake and the 1976 magnitude 7.5 China
earthquake.
(d) The 2008 magnitude 7.9 Wenchuan (China) earthquake and the 2015 magnitude 7.8
Nepal earthquake.
9. Which of the following industrial activities is not mentioned in the passage as
one of the ways that induces man#made earthquakes?
i. Construction of water dams.
ii. Mining.
iii. Injection of liquids such as wastewater and carbon dioxide.
iv. Creating tunnels in mountains and underground constructions.
v. Extractions associated with oil and gas exploitation.
(a) Both i) and iv)
(b) Only iv)
(c) i), iii) and iv)
(d) ii) and v)
10. Why is public concern over man-made earthquakes higher in Europe as compared to
the United States?
(a) Because population density is higher in Europe as compared to that in US.
(b) Because European industries are more developed and advanced as compared to
those in the US.
(c) Because European industries are less developed and advanced as compared to
those in the US.
(d) Because population density is lower in Europe as compared to that in US.
Passage (Q.11-Q.15): Over the course of her career, Tiffany Jung experienced
different challenges maintaining
a balance between her professional and personal life. When she began her career at
a law firm, Jung says there
was no such thing as work-life. Employees were expected to log significant hours,
be available at all times, and
even cancel vacations at a moment’s notice. Seeking a change of pace, she moved to
an in-house position at a
high growth tech company, but despite its strong focus on company culture, work-
life balance became tricky in
new and unexpected ways. “They had an open floor plan, meaning there was pressure
to always seem available,
but also busy,” she says. “There was also a sense of pressure to participate in
company social events that would
take place after work hours, which meant employees had to fit work culture into
their personal time.” Before
long, Jung says she was having trouble sleeping, felt unmotivated at work, and
began identifying signs of
burnout. Eventually, she decided to leave the company to take a work sabbatical,
before re-entering the
workforce part-time as a freelancer.
TRUCE Software’s CEO, Joe Boyle, says that those who began their careers in an era
when bringing work home
was a conscious decision have grown accustomed to establishing and enforcing that
divide themselves. On the
other hand, those who started working in an era where they could be reached 24-7
via email, Slack, Zoom, or

. Page 5 of 40
any number of other workplace tools, typically expect their employer to support and
model a strong work-life
divide.
The transition to widespread remote work made issues related to work-life balance
impossible to ignore, but
Boyle says those lines were already starting to blur before COVID-19, in large part
due to technology. It’s a
topic to which Boyle has given a lot of thought. TRUCE provides software that can
be programmed to limit
access to non-work-related applications when a mobile device is on company
property, and remove work-related
applications as soon as they clock out. Boyle adds that once those boundaries are
crossed and new expectations
are set, they often don’t regress.
Establishing work-life boundaries is neither strictly a matter of corporate policy
or personal responsibility, but
should be a shared commitment by both employees and employers, argues Deniece
Maston. Unless organizations
establish and commit to policies that ensure workers feel confident being truly
“off the clock” during non-work
hours, employees often need to decide between establishing their own boundaries—
which could come at a cost
to their reputation—or allow work to disrupt their downtime. Rather than choosing
between two undesirable
options, Maston says employees should engage in an honest dialogue with their
managers about their personal
needs and responsibilities. Maston says that many organizations boast about their
work-life balance policies in
job postings, but setting policies that encourage that divide—such as banning
after-hours work communications
or encouraging staff to take time off—is only effective if they’re followed by
leaders.
11. What exactly does the software do that is provided by TRUCE?
(a) When a mobile device is on corporate property, TRUCE software can be set to
give access to non-work#related applications and upload work-related applications
as soon as they clock out.
(b) When a mobile device is on corporate property, TRUCE software keeps track of
the sites and access points
of an employer and notifies the company if any suspicious activity is tracked
during log-in hours.
(c) When a mobile device is on corporate property, TRUCE software can be set to
block access to non-work#related applications and delete work-related applications
as soon as they clock out.
(d) When a mobile device is on corporate property, TRUCE software can be set to
relegate the work-related
applications as a carry over to the following day.
12. According to the passage, which of the following statements are true?
A. Maston recommends that employees must be candid with their bosses about their
personal requirements
and tasks to ensure work-life balance.
B. Many companies brag about their work-life balance rules in job advertising and
most of the time, it is a
sham to attract good employees from other organizations.
C. Maston states that work-life balance must be a common playground for both the
corporate as well as the
employees.
(a) Only A and B
(b) Only B and C
(c) Only A and C
(d) A, B and C
13. What is the tone of the author in the passage?
(a) Provocative (b) Speculative (c) Pedestrian (d) Populist
. Page 6 of 40
14. What is the major difference that is drawn by Joe Boyle, between employees of
the past and employees of the
present era?
(a) The employees of the past couldn’t decide for themselves if they wanted to take
their work home and weren’t
accustomed to maintaining the boundary, but the employees of the present have a
choice in that matter owing
to technology; hence, they don’t expect their employers to create the work-life
balance for them.
(b) The employees of the past could decide for themselves if they wanted to take
their work home and were
accustomed to maintaining the boundary, but the employees of the present have no
choice in that matter
owing to technology; hence, they expect their employers to create the work-life
balance for them.
(c) The employees of the past could decide for themselves if they wanted to take
their work home and weren’t
accustomed to maintaining the boundary, but the employees of the present have no
choice in that matter
owing to technology; hence, they don’t expect their employers to create the work-
life balance for them.
(d) The employees of the present can decide for themselves if they wanted to take
their work home and they are
accustomed to maintaining the boundary, but the employees of the past had no choice
in that matter owing
to technology; hence, they expected their employers to create the work-life balance
for them.
15. Which of the accompanying was NOT faced by Tiffany when she employed herself in
an in-house role at a high#growth software firm?
(a) Cancelling vacation at a moment’s notice.
(b) Sleep irregularity.
(c) Lack of motivation at work.
(d) Signs of fatigue and exhaustion.
Passage (Q.16-Q.20): Coral reefs are often compared to cities, an analogy that
captures both the variety and the
density of life they support. The number of species that can be found on a small
patch of healthy reef is probably
greater than can be encountered in a similar amount of space anywhere else on
Earth, including the Amazon rain
forest. Researchers who once picked apart a single coral colony counted more than
eight thousand burrowing
creatures belonging to more than two hundred species. Using more sophisticated
genetic-sequencing techniques,
scientists recently looked to see how many species of crustaceans alone they could
find. In one square metre at
the northern end of the Great Barrier Reef, they came up with more than two hundred
species—mostly crabs and
shrimp—and in a similar-size stretch, at the southern end, they identified almost
two hundred and thirty species.
Extrapolating beyond crustaceans to fish and snails and sponges and octopuses and
squid and sea squirts and on
through the phyla, scientists estimate that reefs are home to at least a million
and possibly as many as nine million
species.
This diversity is even more remarkable in light of what might, to extend the urban
metaphor, be called reefs’
environs. Tropical seas tend to be low in nutrients like nitrogen and phosphorus.
Since most forms of life require
nitrogen and phosphorus, tropical seas also tend to be barren; this explains why
they’re often so marvellously
clear. Ever since Darwin, scientists have been puzzled by how reefs support such
richness under nutrient-poor
conditions. The best explanation anyone has come up with is that on reefs—and here
the metropolitan analogy
starts to break down—all the residents enthusiastically recycle.
“In the coral city there is no waste,” Richard C. Murphy, a marine biologist who
worked with Jacques Cousteau,
has written. “The byproduct of every organism is a resource for another.” Corals
are not only the architects of
the system, they’re also the repairmen; without their ceaseless maintenance,
there’s just “rapidly eroding rubble.”
According to Roger Bradbury, an ecologist at Australia National University, if
reefs were to disappear, the seas
would look much as they did in Precambrian times, before fish had evolved. “It will
be slimy,” he has observed.
Worldwide, some five hundred million people rely on reefs for food, protection,
income, or a combination of all
three. Attaching a monetary value to these goods and services is difficult—some
entire nations are composed of
reefs—but estimates run as high as three hundred and seventy-five billion dollars a
year.

. Page 7 of 40
16. Which of the following best reflects the title?
(a) Unnatural selection.
(b) Coral reef: a habitat
(c) The Great Barrier Reef
(d) Corals: architects of the system.
17. Which of the following can be inferred from the passage?
(a) Without corals, seas will not be able to sustain life.
(b) Corals are responsible for the economies of many countries.
(c) Corals are rapidly eroding rubble, if they cease the role of being the
architects of the system.
(d) Five hundred million people rely on reefs for their sustenance.
18. Ever since Darwin, scientists have been puzzled by how reefs support such
richness under nutrient-poor
conditions. The phrase ‘ever since’ belongs to which part of speech?
(a) Preposition (b) Conjunction (c) Adjective (d) Adverb
19. Based on the passage, how do the reefs differ from the urban cities?
(a) In diversity and density of sustaining of life.
(b) In sustaining different species in their natural habitat.
(c) In contributing to the economy of a country.
(d) In not wasting by-products through recycling.
20. Which of the following best summarises the passage?
(a) Coral reefs are a natural habitat of millions of species in water and on land,
ceaselessly maintaining and
repairing the environment so that no waste is produced. They contribute to the
economies of various countries
by contributing food, protection, income, or a combination of all three.
(b) The Coral reefs are the architect and the repairmen of the aqua- ecosystem
sustaining many species of
crustaceans, while giving optimal environment system for species to thrive even
under adverse conditions
along with supporting economies by adding a large amount of a monetary value.
(c) Coral reefs under sea are analogical to the urban cities in terms of diversity
and density of population,
sustaining millions of species, and if reefs were to disappear, the life on earth
will cease to exist.
(d) Coral reefs as an architect, ceaselessly sustaining millions of species in seas
under nutrient-poor conditions
and as repairmen, recycling the waste such that the by-product of every organism is
a resource for another
along with contributing to the economies of many countries abound in reefs.
Passage (Q.21-Q.25): Plato-who may have understood better what forms the mind of
man than to some of our
contemporaries who want their children exposed only to 'real' people and evade
events-knew what intellectual
experiences make for true humanity. He suggested that the future citizens of his
ideal republic begin their literary
education with the telling of myths, rather than with mere facts of so-called
rational teachings. Even Aristotle,
master of pure reason, said, "The friends of wisdom is also a friend of myth."
Modern thinkers who have studied myths and fairy tales from a philosophical or
psychological viewpoint arrive
at the same conclusion, regardless of their original persuasion. Mirecea Eliade,
for one, describes these stories
as "models for human behaviour [that], by that very fact, give meaning and value to
life." Drawing on
anthropological parallels, he and other suggest that myths and fairy tales were
derived from, or give symbolic
expression to, initiation rites or rites of passage-such as the death of an old,
inadequate self in order to be reborn
on a higher plane of existence. He feels that this is why these tales meet a
strongly felt need and are carriers of
such deep meaning.
Other investigators with a deep-psychological orientation emphasize the
similarities between the fantastic events
in myths and fairy tales and those in adult dreams and daydreams-the fulfilment of
wishes, the winning out over

. Page 8 of 40
all competitors, the destruction of enemies - and conclude that one attraction of
this literature is its expression of
that which is normally prevented from coming to awareness.
There are, of course, very significant differences between fairy tales and dreams.
For example, in dreams more
of-ten than not the wish fulfilment is disguised, while in fairy tales much of it
is openly expressed. To a
considerable degree, dreams are the result of inner pressures which have found no
relief, of problems which
beset a person to which he knows no solution and to which the dream finds none. The
fairy tale does the opposite:
it projects the relief of all pressures and not only offers ways to solve problems
but promises that a 'happy'
solution will be found.
We cannot control what goes on in our dreams. Although our inner censorship
influences what we may dream,
such control occurs on an unconscious level. The fairy tale, on the other hand, is
very much the result of common
conscious and unconscious content having been shaped by the conscious mind, not of
one particular person, but
the consensus of many in regard to what they view as universal human problems, and
what they accept as
desirable solutions. If all these elements were not present in a fairy tale, it
would not be retold by generation after
generation. Only if a fairy tale met the conscious and unconscious requirements of
many people was it repeatedly
retold, and listened to with great interest. No dream of a person could arouse such
persistent interest unless it
was worked into a myth, as was the story of the Pharaoh's dream as interpreted by
Joseph in the Bible.
21. What can be understood from the words of Aristotle that the friend of wisdom is
also a friend of myth?
(a) Wisdom and myth are interrelated; people drawn towards wisdom will be drawn
towards myth.
(b) One who is inclined towards philosophy will be inclined towards myths.
(c) Intellectual experiences and myth go hand in hand in forming the rational
thinking mind.
(d) People seeking wisdom cannot remain untouched by the myth from where they will
draw the wisdom.
22. What can be inferred from the passage regarding the inherent difference between
a dream and a fairy tale?
(a) Fairy tale is a projection of universally desirable solutions to universal
problems; whereas, dreams are the
outcomes of mind’s struggles without remedies.
(c) Fairy tales are covert venting out of the steam; whereas, a dream project inner
struggles without exit.
(d) Dreams are reflections of a person’s unfulfilled desires; whereas fairy tales
are reflections in a controlled
settings with a deliberate emphasis on wish fulfilment.
(d) Fairy tales and dreams are two sides of the same coin in manifestation of wish
fulfilment.
23. Which of the following best reflects the tone of the passage?
(a) Matter-of-fact. (b) Opinionated. (c) Prosaic. (d) None of the above.
24. The death of an old, inadequate self in order to be reborn on a higher plane of
existence.’ Which of the following
figure of speech is inherent in the sentence?
(a) Hyperbole (b) Metaphor (c) Personification (d) Allegory
25. What does the author communicate through the last paragraph?
(a) The brain is instrumental in controlling the external and internal thoughts and
projects it in the form of a
fairy tale that then becomes a folklore.
(b) The conscious mind blends the internal and the external information and plays a
pivotal role in the shaping
of the fairy tale which results in it becoming a legend.
(c) A fairy-tale becomes a folklore because the conscious mind shapes the oblivious
thoughts of the mind as
well as the universal cognisant thoughts in the form of a myth that meets the
requirements of many people.
(d) dream is an uncontrolled projection at an unconscious level that is
individualistic; hence, cannot have a
universal requirement.

. Page 9 of 40
Passage (Q.26-Q.30): The essence of being human is illustrated beautifully through
an Ubuntu proverb in Zulu
culture, and this was often recalled by Nobel peace prize awardee and archbishop,
Desmond Tutu: “I am a person
through other people. My humanity is tied to yours.”
He would explain it as follows: “One of the sayings in our country is Ubuntu – the
essence of being human.
Ubuntu speaks particularly about the fact that you can’t exist as a human being in
isolation. It speaks about our
interconnectedness … We think of ourselves far too frequently as just individuals,
separated from one another,
whereas you are connected and what you do affects the whole world. When you do
well, it spreads out; it is for
the whole of humanity.” That is, the butterfly effect is as true among people of
the world as it is true in the
intricate web of nature, wherein all beings, animate and inanimate, are closely
interconnected.
Scholar-researchers Jacob Mugumbate, University of Wollongong, and Andrew Nyanguru,
University of
Zimbabwe, sum up Ubuntu philosophy as ‘I am because of who we all are.’ Such an
attitude would likely lead
us to the essence of the summum bonum philosophy that denotes the “fundamental
principle on which some
system of ethics is based, that is, the aim of actions, which, if consistently
pursued, will lead to the best possible
life.”
Mugumbate and Nyanguru published a paper on the place of Ubuntu philosophy in the
study and practice of
social work in terms of its impact on ethical and social issues that require deeper
understanding coming from a
wider, humanistic perspective. Something that is desperately needed, in a world
being riven by meaningless
divisions and conflicts.
When we dehumanise others, we end up dehumanising ourselves, said Tutu, who felt
that to heal both spirit and
body, one had to learn to look at the positive aspects of any person, group or
situation rather than focus only on
the negative aspects. Both Tutu and Nelson Mandela stressed on the significance of
magnanimity, reconciliation
and forgiveness rather than on revenge and retribution. This was the spirit in
which the famous Truth and
Reconciliation proceedings took place in South Africa to help both victims and
perpetrators of the Apartheid to
overcome all the unpleasantness and tragedies, inspire forgiveness and herald a new
era of hope, faith and love
that would lead to peaceful and dignified co-existence.
What we are as individuals is the sum of all that went before us; of all that we
experience now and of all that
could possibly happen in the future.
A beautiful Ubuntu practice Tutu often related, involves community members sitting
around a person who has
committed wrong, and speak of all the good that person has done, focussing on all
his positive qualities, for a
few days in a row, in a sort of purification, a revelatory ritual. The belief is
that all of us are good, but we do
sometimes make mistakes, which is really a cry for help.
26. What can be inferred from the quote, “I am a person through other people. My
humanity is tied to yours.”?
(a) A person’s humanity is tied to another person.
(b) Humanity depends on mutual human existence.
(c) Communal human existence is mutually exclusive.
(d) A person’s identity is linked with another person paving way for mutual
humanity.
27. Which of the following best reflects the title for the passage?
(a) The essence of Humanity.
(b) We’re connected.
(c) My humanity is tied to yours.
(d) Ubuntu philosophy.

. Page 10 of 40
28. Which of the following conveys the same meaning as the word ‘Apartheid’?
(a) Racism. (b) Jingoism (c) Ethnicity (d) Altercation
29. Based on the passage, what can be inferred from the expression ‘dehumanise’?
(a) Devalue a person based on the nature of crime.
(b) To regard, represent or treat a person as less than human.
(c)To treat a person less than its community.
(d) To divest a person of individuality by depriving that person of human
qualities.
30. Which of the following sentences will come as the conclusion to the last
paragraph?
I. We must override divisions and heal the scarred collective psyche.
II. What we are as individuals is the sum of all that went before us; of all that
we experience now and of allthat
could possibly happen in the future.
III. Unity and affirmation have more power to change behaviour than shame and
punishment.
(a) Only I (b) Only II (c) Only III (d) I & III.

SECTION - C: LEGAL REASONING


Passage (Q.66-Q.70): Read the following passage and in the light of the same answer
the questions that follow.
It is often said that section 295A of the Indian Penal Code (IPC), which is an
Indian variant of the blasphemy
law, violates the secular character of the Constitution. It interferes with ideals
of social reform of religion as
envisaged under the Constitution by disallowing fair criticism of religion. In
modern states practicing secularism,
offenses related to religion aim to uphold law and order and do not necessarily aim
to protect God or faith.
Section 295A was introduced whereby public peace would be disturbed by large-scale
religious hostilities. The
only useful purpose the provision could serve is the protection of law and order.
Section 295A Code provides
that any insult or attempt to insult religion, or the religious beliefs of a class,
with the deliberate and malicious
intention of outraging religious sentiments shall be punishable. This section was
enacted to make malicious acts
insulting the religion, or outraging the religious feelings, of any class of
citizens, punishable as offenses relating
to religion. For the application of this section, there must be a proximate link
between speech/act and public
disorder, and not a remote, or fanciful connection. Titles do not depict religion.
Fair academic criticism of
religion cannot be construed as blasphemy. Engagement with religion invariably
necessitates fair criticism of
religion and its practices. Freedom of Speech and expression is a fundamental right
in the constitution of various
countries including India and is also a Human Right. For a law to be a reasonable
restriction upon the freedom
of speech in the interests of public order, it would need to be limited to
situations where there was a degree
of proximity between the proscribed speech, and the possibility of public disorder.
Such laws are enacte d to
promote harmony in the society.
(Source- Surbhi Karwa And Shubham Kumar. (2019, September 5). Blasphemy Laws and
Secularism in India.
Https://Www.Epw.in/Engage/Article/Blasphemy-Law-Antithetical-Indias-Secular-Ethos.
Retrieved April 1,
2022, from https://www.epw.in/engage/article/blasphemy-law-antithetical-indias-
secular-ethos)
66. A film, by the name ‘Ganga’, suffered massive criticism post-release. The name
of the protagonist was Ganga
and she played the role of a prostitute. An FIR was issued against the director and
the producer of the film by
the members of Bhole Sena under section 295A of IPC. According to members of Bhole
Sena, Ganga is a pious
word for them, naming the protagonist who is playing the role of a prostitute has
hurt their religious sentiment.
The film was prohibited from being released. Bhole Sena does not represent the
thoughts of the entire class. The
accused in his statement said that- “Film is a work of art and it’s a fictional
tale. Shakespeare once said, what is
in a name? That which we call rose by any other name would be as sweet. There is a
reason why a disclaimer is
shown at the beginning of the film.” Decide.
(a) The Producer should be held liable under section 295A because his film outraged
the religious sentiments of
the citizens.
(b) The Producer should be held liable under section 295A because his film is
disrespectful to a pious figure of
a particular religion.
(c) The Producer should not be held liable under section 295A because the
allegations are far-fetched.
(d) The Producer should not be held liable under section 295A because the
allegations are unnecessary.
67. A group of atheists formed an association to promote atheism. All of them were
Christians by religion before
choosing atheism. They gathered in public one day and started promoting their
purpose, one of the members
stood up and tore the copy of the Holy Bible which belonged to him, and said “ I
give up.” The other members
also supported him by cheering for him. This led to a brawl between him and the
people standing by to listen.
All of them were arrested by the police under section 295A of IPC. Decide-
(a) All of them are not liable under section 295A, only the person who tore the
Holy Bible shall be booked.
(b) All of them are not liable under section 295A, only the members of the
association should be held liable.
(c) All of them are liable under section 295A.
(d) All of them are not liable under section 295A, only the head of the association
to promotes atheism is liable.

. Page 19 of 40
68. In the light of the passage, which of the following statements best describe
purpose Blasphemy laws in India?
(a) Blasphemy laws are created to protect and upkeep the principles of the
Constitution.
(b) Blasphemy laws are created to maintain peace and tranquility in society.
(c) Blasphemy laws are created to penalize every act that is disrespectful to the
beliefs of people.
(d) Blasphemy laws are created to protect the spiritual interests of people.
69. Keeping in mind the facts from Q.66. the director of the film ‘Ganga files a
petition in the Supreme Court, stating
that his fundamental right i.e. right to freedom of speech and expression under Art
19(1)(a) got violated. Decide
whether there was a violation of Article 19(1)(a).
(a) No, there is no violation of Article 19(1)(a) the allegations were correct and
the film had blasphemous content
in it.
(b) No, there is no violation of Article 19(1)(a) because reasonable restrictions
can be imposed wherever
necessary.
(c) Yes, there is a violation of Article 19(1)(a) because the content of the film
cannot be said to be offensive to
the religious sentiments of the people and restriction on release amounts to a
violation of freedom of speech
and expression.
(d) Yes, there is a violation of Article 19(1)(a) because the film contains a
social message and the director has a
right to express his views freely.
70. Which statement summarises the passage?
(a) Though blasphemy laws in India protect the freedom of some individuals to
practice religion as they see fit
without insult or unjust attack, blasphemy laws inherently limit other individuals'
freedom of speech.
(b) Blasphemy laws in India have blatantly failed because of secularism.
(c) Blasphemy laws in India disallow the fair criticism of religion.
(d) Blasphemy laws in India are not much wider because India is a secular country.
Passage (Q.71-Q.75): The Supreme Court has observed that the Environmental
Protection Act does not prohibit
grant of ex post facto Environmental Clearance absolutely.
"Where the adverse consequences of denial of ex post facto approval outweigh the
consequences of
regularization of operations by grant of ex post facto approval, and the
establishment concerned otherwise
conforms to the requisite pollution norms, ex post facto approval should be given
in accordance with law, in
strict conformity with the applicable Rules, Regulations and/or Notifications.",
the court observed.
The court noted that, in Alembic Pharmaceuticals Ltd. v. Rohit Prajapati, it had
deprecated ex post facto
clearances, but did not direct closure of the units concerned but explored measures
to control the damage caused
by the industrial units. In this regard, the bench made the following observations:
"The 1986 Act does not prohibit ex post facto Environmental Clearance. The Court
cannot be oblivious to the
economy or the need to protect the livelihood of hundreds of employees and others
employed in the project and
others dependent on the project, if such projects comply with environmental norms.
There can be no doubt that the need to comply with the requirement to obtain EC is
non-negotiable. A unit can
be set up or allowed to expand subject to compliance of the requisite environmental
norms. EC is granted on
condition of the suitability of the site to set up the unit, from the environmental
angle, and also existence of
necessary infrastructural facilities and equipment for compliance of environmental
norms. To protect future
generations and to ensure sustainable development, it is imperative that pollution
laws be strictly enforced. Under
no circumstances can industries, which pollute, be allowed to operate unchecked and
degrade the environment.
Ex post facto environmental clearance should not be granted routinely, but in
exceptional circumstances taking
into account all relevant environmental factors and amount of fault. The deviant
industry may be penalised by
an imposition of heavy penalty than damage on the principle of 'polluter pays' and
the cost of restoration of
environment may be recovered from it.”
[Source- LiveLaw, “Ex post facto Environmental Clearance…”
https://www.livelaw.in/top-stories/ex-post#facto-environmental-clearance-can-be-
granted-in-exceptional-circumstances-supreme-courtpahwa-plastics-pvt#ltd-vs-dastak-
ngo-2022-livelaw-sc-318-195027 published on March 25, 2022]

. Page 20 of 40
71. Raadhe chemicals Pvt limited, is a newly incorporated company which
manufacturers pesticides and fertilizers.
The company has just started a new plant in the city of Mathura. However, the
company has not taken
environmental clearance from the pollution Control Board. After 5 months of its
operations, it applies for
environmental clearance. Will the court allow ex post facto clearance in this case?
(a) The court will allow ex post facto EC as the Supreme Court usually allows such
clearance in commercial
interest of the parties.
(b) The court will not allow ex post facto EC as there are no exceptional
circumstances.
(c) The court will allow ex post facto EC as the plant has been functioning from
five months and it is an
exceptional circumstance.
(d) The question can’t be decided as the facts are not complete.
72. Sailesh Construction Ltd. purchased a land for construction of towers for
residential flats. Since Construction
activities causes pollution, the Company took environmental clearance from the
Pollution Control Board.
However, despite taking clearance, the construction activities caused massive
pollution due to usage of old
machinery. The residents living nearby brought a case for compensation, due to
health hazards caused by the
Construction Activities. Decide the liability of the company.
(a) The company shall not be liable as they have already obtained Environmental
Clearance.
(b) The company shall be liable as every polluter who causes pollution has to bear
the damages.
(c) The company shall be liable only to the extent the environmental damage is
above the normal pollution
norms.
(d) The company shall not be liable as they are undertaking construction
activities. Hence it is an exceptional
circumstance if the pollution is being caused for a limited period of time.
73. The Hon’ble Supreme Court has held that in exceptional circumstances, ex-post
facto environmental clearance
might be granted. Which of the following options seem to not fall within the
category of “Exceptional
circumstance”?
(a) This court allows ex-post facto clearance as not giving clearance shall render
more than 5000 employees as
unemployed.
(b) This court allows ex-post facto clearance as not giving clearance shall disrupt
the demand and supply of
essential medicines in the light of ongoing pandemic.
(c) This court allows ex-post facto clearance as not giving clearance shall render
the owners of the factory into
huge debts and losses
(d) All of the above.
74. Sunil and Suresh wanted to set up a chemical plant near Delhi NCR for
manufacturing chemical dyes for clothes
and other textiles. They had set up a plant by December 2021 and immediately
applied for environmental
clearance. However, the Pollution Control Board refused to provide the clearance
citing faulty pollution control
mechanisms in place. Despite the refusal, they hired about 100 people to work in
the factory. The Pollution
Control Board sued both of them and also shut down the factory. Sunil and Suresh
contend before the Court that
ex-post facto clearance be granted to them. Decide.
(a) The court should grant them ex post facto clearance as not giving clearance
will render 100 people
unemployed. This is an exceptional circumstance.
(b) The court should not grant ex post facto clearance because the clearance was
already denied to them and
hence the question of providing ex post facto clearance does not arise.
(c) The court should grant them ex post facto clearance because the reason why the
clearance was denied was
not correct.
(d) The court should not grant ex post facto clearance because the company is not
involved in any exceptional
business.

. Page 21 of 40
75. Aqua pharmaceuticals limited is a company engaged in the manufacture of
medicines. Such medicines produced
are extremely beneficial for the children suffering from pneumonia. However, a
report of the pollution Control
Board shows that the company has regularly defaulted on complying with various
environmental norms under
the Environmental Protection act. The report also says that the environmental
clearance is not obtained.
Consequentially, the pollution Control Board directed the company to stop its
operations. The matter reaches
before the Supreme Court for grant of ex post facto clearance. Will the Supreme
Court allow ex-post facto
clearance?
(a) No, the Supreme Court will not allow ex post facto clearance as there is no
exceptional circumstance.
(b) Yes, the Supreme Court will allow ex post facto clearance as there is an
exceptional circumstance that is to
prevent the children from pneumonia.
(c) No, the Supreme Court will not allow expost factory clearance because the
company has regularly defaulted
on complying with various environmental norms and also not obtained the
environmental clearance
(d) Yes, the Supreme Court will allow expost factory clearance because the
Environmental Protection act does
not absolutely bar providing such clearance.
Passage (Q.76-Q.80): While allowing the plea filed by the petitioner(accused) under
Articles 226 and 227 of
the Constitution of India read with Section 482 of CrPC, Justice Asha Menon
observed,
"There is no prima facie evidence that he had any mala fide intent in keeping the
ammunition. The safety of
passengers was not threatened. The possession was not conscious."
As per the prosecution, the petitioner was unable to prove that he was not in the
conscious possession of the
cartridges. The counsel for the petitioner on the other hand argued that the
petitioner was not in conscious
possession of these two cartridges and that the same could not be used for any
threat purpose, in the absence of
any fire arm. Reliance was placed on several Supreme Court decisions to argue that
in cases like the present one,
where there was no "conscious possession", the courts have been quashing the FIRs.
The High Court noted that Coordinate Benches of the Court in various decisions,
including Adhiraj Singh Yadav
v. State, where 1 or 2 live cartridges have been found in the possession of the
accused, have taken a view that
mens rea or mala fide intention must be present supporting a "conscious possession
plea" of the prosecution and
in the absence of any such evidence, even prima facie, there would be no reason to
deny the relief to the
petitioner.
Thus, in totality of the circumstances, the Court observed that since no mala fides
or mal-intention is evident
from the facts and the record, powers under Section 482 Cr.P.C. are required to be
exercised in the present matter.
"Additional Standing Counsel, conceded that the petitioner had a valid Arms License
issued to him in
Punjab...The bullets recovered were relatable to the licensed weapon, " it noted.
The court on exercising its power
under section 482 of CrPC accepted the petition and decided in favour of
petitioner.
Source : Case Title: KARAMJIT SINGH v. STATE (N.C.T. OF DELHI)
76. According to your interpretation of the passage, when will a person accused of
being in “conscious possession”
will be convicted?
(a) As per the prosecution, the petitioner was unable to demonstrate that he was
not in conscious possession of
the cartridges.
(b) The petitioner's attorney, on the other hand, stated that the petitioner was
not aware of the existence of these
two cartridges and that they could not be used for any threat purpose in the
absence of a fire arm.
(c) Where there was no "conscious possession," the courts will dismiss the FIRs.
(d) Where 1 or 2 live cartridges were discovered in the accuser’s possession, the
prosecution has taken the
position that mens rea or mala fide intention must be present to establish a
"conscious possession plea.

. Page 22 of 40
77. The passage's context refers to "conscious possession" on several points.
Choose a statement(s) that defines the
term accurately in the light of the passage given.
(a) The mere possession of ammunition without knowledge does not constitute an
offence under the Arms Act.
(b) Only the conscious possession of a firearm or ammunition, where the individual
is fully aware of the
possession, can constitute an offence.
(c) When there is no prima facie proof of malfeasance on the part of the accused in
storing the ammunition and
the passengers' safety is not jeopardised, the possession is said to be not
conscious.
(d) The possession is deemed to be conscious when an individual keeps the
ammunition with the malicious
intent and safety of the passenger is jeopardised.
78. Neeraj is a Jammu & Kashmir native. His village has been declared under the
Armed Forces (Special Powers)
Act (AFSPA), 1958, an act of the Indian Parliament that allows the Indian Armed
Forces special powers to
preserve public order in "disturbed areas." Army officials caught Neeraj for
possessing two missiles, one
grenade, and two land mines. As his lawyer, you chose a statement that will be your
line of action in this case.
(a) As accused lawyer, you will file a petition for quashing of FIR under Articles
226 and 227 of the Constitution
of India read with Section 482 of CrPC.
(b) In the absence of a fire arm, the accuser’s counsel may present a plea that the
accused was not in conscious
possession of the ammunitions and that they could not be used for any threat
purpose.
(c) As accused counsel, you may submit a plea that the accused was not aware of the
ammunition thus was not
in conscious possession.
(d) The burden of proof will be on accused to proof is innocence beyond doubt.
79. Continuing with the same facts as above, who has the burden of proof in cases
involving "conscious possession
of ammunition"?
(a) Both accused and prosecution been given chances to present their respective
case to the court.
(b) Prosecution bears the burden of proof to that mens rea or mala fide intention
must be present supporting a
"conscious possession plea".
(c) Counsel for petitioner bears the burden of proof to show that that the
petitioner was not in conscious
possession of these two cartridges and that the same could not be used for any
threat purpose, in the absence
of any fire arm.
(d) Prosecution need to proof that the petitioner was unable to prove that he was
not in the conscious possession
of two missiles, one grenade, and two land mines.
80. Neeraj is a Jammu & Kashmir native. His village has been declared under the
Armed Forces (Special Powers)
Act (AFSPA), 1958, an act of the Indian Parliament that allows the Indian Armed
Forces special powers to
preserve public order in "disturbed areas." Neeraj has in possession two missiles,
one grenade, and two land
mines. He boarded the bus to Srinagar with the intention of planting a human bomb
on the bus. Neeraj is going
to be the human bomb. But, before anything wrong happens, he was apprehended by the
soldiers at one of the
checkpoints between the two cities. Decide
(a) Neeraj will be brought up on charges of being in conscious possession of
ammunition.
(b) Neeraj will not be held accountable because he was the human bomb and was thus
rescued by the Indian
army at their checkpoints.
(c) Neeraj will be held accountable since he boarded the bus with the objective of
deploying a human bomb.
(d) Neeraj will not be held liable as the safety of passengers was not threatened.

. Page 23 of 40
Passage (Q.81-Q.85):The bench of UU Lait and Indira Banerjee, JJ has explained that
Section 12 of the
Specific Relief Act, 1963 has to be construed in a liberal, purposive manner that
is fair and promotes justice.
“A contractee who frustrates a contract deliberately by his own wrongful acts
cannot be permitted to escape scot
free.” While hearing a case relating to sale of land in the year 1984, the Court
held that Section 12 of the Specific
Relief Act is to be construed and interpreted in a purposive and meaningful manner
to empower the Court to
direct specific performance by the defaulting party, of so much of the contract, as
can be performed, in a case
like this.
“To hold otherwise would permit a party to a contract for sale of land, to
deliberately frustrate the entire contract
by transferring a part of the suit property and creating third party interests over
the same.”
The Court explained that the relief of specific performance of an agreement was at
all material times, equitable,
discretionary relief, governed by the provisions of the Specific Relief Act 1963.
Even though the power of the
Court to direct specific performance of an agreement may have been discretionary,
such power could not be
arbitrary. The discretion had necessarily to be exercised in accordance with sound
and reasonable judicial
principles.
After the amendment of Section 10 of the Specific Relief Act, the words “specific
performance of any contract
may, in the discretion of the Court, be enforced” have been substituted with the
words “specific performance of
a contract shall be enforced subject to …”. Hence, “the Court is, now obliged to
enforce the specific performance
of a contract, subject to the provisions of sub-section (2) of Section 11, Section
14 and Section 16 of the S.R.A.
Relief of specific performance of a contract is no longer discretionary, after the
amendment.”
Referring to suits relating to sale of land, the Court explained that, “an
agreement to sell immovable property,
generally creates a right in personam in favour of the Vendee. The Vendee acquires
a legitimate right to enforce
specific performance of the agreement.”
(SOURCE: With revision and edits from https://www.scconline.com/blog)
81. Jubin, Salim, and Javed are three coparceners whose father has left them 300
acres of land. They later leased the
land to Farhan via a registered lease deed, granting Farhan the right to possession
and use of land for profit, i.e.,
to run his business on the property owned by three brothers. Salim met his uncle in
Dubai, who is highly
interested in purchasing a land in India for future prospects. Salim proposed to
his uncle a proposal in which he
would sell his land to him in exchange for 3 crores in cash. His uncle paid him in
cash and accompanied Salim
back to India to complete the required formalities for the property to be
transferred into his name. What impact
will the sale have on Farhan’s right to make use the said land?
(a) As the transaction will alter the status of the lessee in the current case,
Farhan will be given a fair chance to
explain his claim and seek justice.
(b) Farhan merely hold rights over the land as leasee, hence the sale has no
bearing on Farhan's right to make
use of the land.
(c) Farhan has the right to seek remedies under Section 12 of the Specific Relief
Act.
(d) Farhan may seek remedy by invoking Section 12 of the Specific Relief Act, which
gives the court the
discretion to adjudicate the matter rationally in accordance with sound and
reasonable judicial principles.
82. Continuing the similar facts as above, suppose the three brothers promised
farhan to sale the said land once he
completes his lease of 1 year. But before that Salim sold the land to his uncle.
Can farhan claim that the said
transaction between Salim and his uncle is invalid?
(a) No, because proving the terms of an oral contract in the case of a breach can
be difficult.
(b) Yes, because he can seek justice and also request the Court to order the
defaulting party to perform as much
of the contract as is possible in a case like this.
(c) Yes, although proving the terms of an oral contract in the case of a breach can
be difficult, this sort of contract
is legally binding.
(d) Yes, but only if Farhan can establish the contents of the oral agreement in
writing.

. Page 24 of 40
83. Choose a statement(s) from the following illustration where the right in
personam exists in favour of the vendee.
I. In a suit for specific performance of a contract by a member of an undivided
Hindu family to sell his share,
it is not permissible to join the other members of the family as defendants merely
with a view to obtaining
partition and possession of the alleged vendor's share as against them.
II. A sold his car to B. A has the right to receive the sale proceeds.
III. Mr. Y has a suitcase full of cash. This money belongs to Mr. Y exclusively.
IV. B loaned money to C. The right to recover the money belongs only to B.
(a) I (b) I, II and III (c) I &II (d) None
84. The plaintiff, a firm that operates hotels in several locations, furnished the
defendants with a space on leave and
a license for a period of 12 years. Under these terms, the defendant agreed to make
a loan of a specified sum to
the plaintiff. Meanwhile, the plaintiff had addressed a letter to the defendant
requesting that the property be
vacated before the expiration of the time period specified. In exchange, the
defendant requested that the deed be
renewed. There was a fire outbreak one day in the plaintiff’s hotel which affected
the suit premises as well. The
defendant vacated the place soon after. Can defendant sue the plaintiff for
specific performance of contract under
section 10 of the specific relief act?
(a) No, because Section 10 of the Specific Relief Act can only be invoked at the
court's discretion.
(b) Yes, the Court has the authority to enforce specific performance of a contract.
(c) Yes, the order of specific performance will be given at the court's discretion.
(d) No, because the defendant vacated the premises and cannot now be granted any
relief.
85. Assertion: Vendee a person, who buys the land or the property, acquires a
legitimate right to enforce specific
performance of the agreement in case of an agreement to sell.
Reason: Right in personam gives the person rights against one person or party to
the contract. It generally will
correspond with a duty imposed on the said person or party.
(a) Both A and R are true but R is not correct explanation of A.
(b) Both A and R are true and R is correct explanation of A.
(c) A is true but R is false.
(d) A is false but R is true.
Passage (Q.86-Q.90): Most government or Public Sector Unit (PSU's) contracts are
wedged with an 'interest
barring' clause.
Under section 31(7) of the Arbitration and Conciliation Act, 1996, the arbitral
tribunal has the competence to
levy interest on the arbitral award. However, section 31(7)(a) of the Act begins
with the phrase 'Unless otherwise
agreed by the parties'; this brings a material change to the scheme of the power of
the arbitrator to award interest.
The Supreme Court in the case of Garg Builders vs. Bharat Heavy Electricals
Limited(BHEL), (CIVIL APPEAL
NO.6216 OF 2021) has ruled that, "Section 31(7)(a) of the 1996 Act which allows
parties to waive any claim to
interest including pendente lite and the power of the Arbitrator to grant interest
is subject to the agreement of the
parties…. Thus, when there is an express statutory permission for the parties to
contract out of receiving interest
and they have done so without any vitiation of free consent, it is not open for the
Arbitrator to grant pendent lite
interest. When there is no clause then it will be discretion of arbitrator to grant
the interest."
Thus, the law on the issue is straight as an arrow with no-scope of further
interpretation by appellate court unless
pendent lite interest is waived off in contract and has not been applied in case.
Appellate shall not intervene when
there is an interest barring clause in the contract. Nonetheless, a clause that
subdues the very genesis of
compensation (in this case interest) is prima facie colourable and thereby demands
a meticulous scrutiny.
In cases of government contracts wherein an interest barring clause is engrafted in
the agreement, it is necessary
to look at the outlook in toto. In these types of contracts, the parties are on an
unequal footing. This is because
of the fact that one entity is an awfully big enterprise (for e.g. a PSU) while the
other contracting party has a
considerably lessor bargaining power in front of it. This creates a situation where
the party having an upper hand
can conveniently demand a favorable contractual term on its sole will and without
much opposition from the

. Page 25 of 40
other party. This essentially erodes the component of free choice from the
contract. This position of law is
arbitrary as the parties are not placed at the same pedestal while entering the
contract. Unless otherwise agreed
by the parties' from section 31(7)(a) will be an 'up to scratch' approach in the
current state of affairs.
(Source- Livelaw, “Freedom of Contract and Interest Barring Clauses..”
https://www.livelaw.in/columns/section-317a-of-the-arbitration-act-interest-act-
1978-section-23-of-the-indian#contract-act-194547 published on March 20, 2022)
86. In the above-mentioned passage, the author has criticised certain element of
current legal position. Which of the
following options aptly summarise such criticism by the Author?
(a) The author has criticized the interest-barring clause in the government
contracts.
(b) The author has criticized section 31(7) of the Arbitration Act.
(c) The author has criticized unfair competition between the two parties entering
into the contract.
(d) The author has criticized the Arbitration Act for giving too much freedom to
the parties.
87. Bob the Builder Ltd. is a construction company, engaged in construction of
government guest houses since 2005.
In the year 2015, they entered into a contract with the State of Maharashtra for
the construction of a guest house
in the city of Bombay. The contract provided that the construction should be
completed in 5 years and if the
construction is completed within the time, then the government shall make the
payment within 6 months. The
government failed to make the payment within 6 months. Bob the Builder invoked the
arbitration clause and
claimed an interest of 5% on delayed payment. The government argues that such
interest payment cannot be
made in light of the interest-barring clause in the contract. The arbitral tribunal
awards no interest to Bob the
Builder on the delayed payments. Bob the Builder challenges the arbitral award
before the High Court. Should
High Court intervene in the set decision of the arbitral tribunal?
(a) The High Court should not interfere as there is an interest barring clause in
the contract.
(b) The High Court should interfere because the interest barring clause is
discriminatory and puts the parties on
an equal footing.
(c) The High Court should interfere because section 31(7) subdues the very genesis
of compensation.
(d) The High Court can interfere if it is satisfied that interest barring clause is
arbitrary and leads to unfair
measures for the other party.
88. “This essentially erodes the component of free choice from the contract. This
position of law is arbitrary as the
parties are not placed at the same pedestal while entering the contract”. By this
statement what was the opinion
of the author?
(a) Competence of Parties is hampered as are not on equal pedestal.
(b) Free consent of Parties is effected due to big enterprises hampering their
choices.
(c) Undue Influence is done by big enterprises.
(d) Coercion is done by big enterprises to enter other party into the contract.
89. Ridhi and Nidhi enter into a contract for the supply of paintings, being 20 in
number, by the end of the year.
However, Ridhi failed to supply this said paintings to Nidhi, thereby invoking the
arbitration clause in their
agreement. The arbitrator decided in favour of Nidhi, and also awarded interest add
the rate of 5% per annum
from the date of default to supply the paintings till the date of arbitral award.
Ridhi challenged the payment of
interest clause before the High Court alleging that the contract didn’t provide for
payment of any interest. Will
the High Court interfere in the decision of the arbitrator?
(a) The High Court will interfere because the Contract did not provide for any
pendente lite interest, and
therefore, the interest cannot be given by the Arbitrator.
(b) The High Court may interfere if it finds that the interest is exorbitant to
take away free choice of the parties.
(c) The High Court will not interfere because the Contract is silent on the aspect
of interest and therefore, the
Arbitrator has power to grant pendente lite interest.
(d) None of the above.
. Page 26 of 40
90. Raheja distributors and Karthik suppliers entered into a contract for supply of
goods from Raheja to Karthik.
However, Raheja failed to supply the goods on time to Karthik, thereby invoking the
arbitration clause in the
agreement. The Arbitration clause provided that there shall be no pendente lite
interest. The Arbitrator decides
in favour of Karthik and also provides for payment of interest at the rate of 5%
from the date of award till the
date of payment. Raheja challenges the arbitrator’s award on the ground that since
the agreement waived off the
interest, the arbitrator cannot avoid such interest. Will the High Court interfere
in the set judgement?
(a) The High Court will not interfere as there is no pendente lite interest is
imposed.
(b) The High Court will not interfere as court does not have power in pendente lite
interest.
(c) The High Court will interfere because the arbitration clause provided that
there shall be no pendente lite
interest.
(d) The High Court will interfere because as per section 31(7), the arbitrator
cannot grant pendente lite interest
if the parties to the contract have waived off such interest.
Passage (Q.91-Q.95): The concept and doctrine of Principles of Natural Justice and
its application in Justice
delivery system is not new. It has by now assumed the importance of being, so to
say, "an essential inbuilt
component" of the mechanism, through which decision-making process passes, in the
matters touching the rights
and liberty of the people. Its application is extended to all those authorities who
have upon them, the
responsibility of determining civil rights or obligations of the people The Hon'ble
Supreme Court quoted:
“Natural Justice has many colours and shades, many forms and shapes and, save where
valid law excludes, it
applies when people are affected by acts of authority. It is the bone of healthy
government, recognised from
earliest times and not a mystic testament of judge-made law…. Today its application
must be sustained by current
legislation, case law or other extant principle, not the hoary chords of legend and
history. Our jurisprudence has
sanctioned its prevalence even like the Anglo-American system."
The first principle is that 'No man shall be a judge in his own cause' i.e., to
say, the deciding authority must be
impartial and without bias. It Implies that no man can act as a judge for a cause
in which be himself has some
Interest, may be pecuniary or otherwise.
The next principle is audi alteram partem, i.e., no man should be condemned unheard
or that both the sides must
be heard before passing any order. A man cannot incur the loss of property or
liberty for an offence by a judicial
proceeding until he has a fair opportunity of answering the case against him.
The third principle which has developed in course of time is that the order which
is passed affecting the rights
of an individual must be a speaking order with reasons in writing. A bald order
requiring no reason to support it
will be arbitrary.
[Source- J.T.R.I Journal, “Principles of Natural Justice” by Justice Brijesh Kumar,
published in 1995,
http://ijtr.nic.in/articles/art36.pdf]
91. Which of the following options aptly summarise the importance of principles of
natural justice in legal system?
(a) The principles of natural justice must be embedded in every law enacted by the
parliament
(b) The principles of natural justice must be embedded in every decision taken by
the High Court, Supreme
Court or any other court of law in India
(c) The principles of natural justice must be embedded in every decision affecting
the rights and liabilities of a
person
(d) The principles of natural justice must be embedded in every administrative
decision.
92. Sadhna gave a loan of INR 50,000 to her business friend Ridhi for a period of 2
years. Ridhi was supposed to
repay the money at the end of 2 years. However, after 2 years, Ridhi refused to
make the repayment. Sadhna
filed a recovery suit against Ridhi. On the first day of the hearing, the Court
issued a notice to Ridhi directing
her to be present on next date of hearing. However, she failed to be present before
the Court. The Court again
issued notice on 2nd, 3rd and 4th hearing too. However, Ridhi did not appear.
Ultimately, the Court directed that
the decision shall be taken ex-parte. Finally, the court directed Ridhi to pay a
compensation of INR 1,00,000 to
Sadhna. Ridhi challenges the said order before the Appellate Authority citing
violation of natural justice. Decide
(a) There is a violation of natural justice as no party should be condemned
unheard.

. Page 27 of 40
(b) There is no violation of natural justice as the court had already provided
enough chances to her to present
her case.
(c) There is a violation of natural justice as no party should be condemned
unheard, even though she didn’t
appear before the court on various occasions.
(d) There is no violation of natural justice as the court had already provided her
reasonable opportunity to present
her case which might affect her rights and liabilities.
93. ITC ltd. is a huge public limited company and has thousands of shareholders.
One such shareholder is Mr. Daivat,
who is a judicial officer in District Court, Saket. On one occasion, a case titled
Tata Ltd. versus ITC Ltd. came
to be listed before Mr. Daivat. Mr. Daivat proceeded to hear the matter and passed
an order against ITC Ltd.
This order was challenged by ITC Ltd. citing violation of natural justice as he is
a shareholder of ITC Ltd. Mr.
Daivat responded that since the order was passed against ITC Ltd., hence there
cannot be any violation of Natural
Justice. Decide.
(a) There is a violation of natural justice as a person cannot be a judge in its
own case.
(b) There is a violation of natural justice as a person cannot be a judge in its
own case even if the order has been
passed against a party in which the judge had interest.
(c) There is no violation of natural justice as the judge was not a party in its
own case since it was merely a
shareholder.
(d) There is no violation of natural justice as the judge was merely a shareholder.
Despite being a shareholder
of ITC limited, he passed an order against ITC itself, therefore, one cannot say
that there was violation of
natural justice.
94. Subodh was arrested on charge of sexual harassment of her neighbour. After
being in judicial custody for 14
days, he moves a bail Application before the Hon’ble High Court. After hearing both
the parties, the Court was
of the opinion that Subodh has been charged for similar offences in past too.
Hence, the bail was rejected. The
Bail Order read as, “Heard the parties. Bail denied.” Subodh challenges this order
before the Supreme Court on
ground of violation of natural justice. Decide
(a) There is no violation of natural justice as the Judge heard both the parties
before denying bail.
(b) There is no violation of natural justice as the Judge heard the parties and
also had valid reasons to deny bail.
(c) There is no violation of natural justice as the judge heard both the parties,
was impartial and also had valid
reasons to deny bail.
(d) None of the above.
95. Shiven and Viren are brothers and have a dispute regarding an ancestral
property. Ultimately, the dispute reached
the Court. At the stage of evidence, both the parties filed various documents as
evidence in order to establish its
ownership over the property. However, after the stage of evidence was over, Viren
found another document
which was of extremely relevant in deciding the case in his favour. However, the
Court refused since the stage
of evidence was already over. Viren ultimately lost the case. He challenged the
decision on grounds of violation
of natural justice by not allowing Viren to produce an additional document. Decide.
(a) There is a violation of natural justice as the Judge ought to have allowed him
to place on record all relevant
documents, even at a later stage
(b) There is no violation of natural justice as the Judge only has a duty to
provide a reasonable opportunity to
present its case and place all documents on record.
(c) This question can’t be decided as the facts are not clear over the nature of
document.
(d) None of the above.

. Page 28 of 40
Passage (Q.96-Q.100): Statements made by an accused in regards to a confession
during a criminal proceeding
admitting his guilt before a judge or magistrate is known as judicial confession or
formal confession. A
confession made by an accused before a Judge or Magistrate is admissible under
Section 164 of the Criminal
Procedure Code. It is known as a plea of guilt and a conviction could be made based
on a judicial confession but
the condition that needs to be satisfied is that the confession should be voluntary
and based on truth. It is the
duty of the judicial officer or magistrate to ensure that the accused is protected
as per the provisions of Article
20(3), which states No person accused of any offence shall be compelled to be a
witness against himself. The
evidentiary value is dealt with under Section 80 of the Indian Evidence Act that a
judicial confession is
substantive evidence and though a conviction can be made on the sole ground of it
as a rule of caution it should
be corroborated with other evidence.
A confession made involuntarily was inadmissible and unreliable and retracted so it
cannot be treated as a
corroborative piece of evidence either. Confession made to a person, during self-
conversation or before anybody
including a Judicial Magistrate in personal capacity is known as extra-judicial
confession or informal confession.
It is admissible even if it is overheard by others and that piece of evidence can
be proved if the Court is satisfied
that it is substantive evidence that goes against the accused but it should be
corroborated with some other
evidence.
The extra-judicial confession, if made voluntarily and with a sound mind, is not
merely weak evidence owing to
the presumption that extra-judicial confession is a weaker kind of evidence. Its
reliability depends upon the facts
and circumstances of the case and the credibility of the witnesses.
(Explanation : https://blog.ipleaders.in/all-you-need-to-know-about-false-
confession/)
96. Which of the following statements may be interpreted as true based on the text
of the passage concerning
substantive evidence?
(a) The evidence on the basis of which a fact is proved and which requires no
corroboration.
(b) Both substantial and corroborative evidence are a must in case of conviction
based upon judicial confession.
(c) Substantive evidence is either direct or circumstantial or both.
(d) Corroborative evidence is the evidence used to make substantive evidence more
concrete.
97. Four teenagers who were presumed to have committed the heinous offence of rape
were arrested. When they
were brought before the magistrate within twenty four hours of their detention, the
police persuaded them to
acknowledge the offence themselves because it would cost them less imprisonment.
Police also told them that
all of the evidence discovered was against the four accused. Accused, seeing no
chance for justice, confessed to
the crime in front of the judge. What may be the probable ruling in this case?
(a) Confession of the all the accused is not admissible as the accused is protected
as per the provisions of Article
20(3).
(b) Confession made by an accused before a Judge or Magistrate is admissible under
Section 164 of the Criminal
Procedure Code.
(c) Confession of all the accused will not be admissible until corroborated with
other evidence.
(d) Confession of all the accused will be admissible only if corroborated with
other evidence.
98. Madhu was accused of killing her own cousin in a lonely flat situated at
Bandra. During the investigation, she
admitted to the crime but also indicated that she was afraid of Manish's suspicious
behaviour. Initially, she was
coping with his split personality, but when he attempted to kill her, she had no
choice but to kill him in self#defence. The report of the accused confessing to the
crime was provided to the court when the accused was being
presented in court. As a result, the court made its verdict and found Madhu guilty.
Isthe court'sruling acceptable?
(a) Court’s ruling is not correct as it’s solely based on confession of accused.
(b) Court’s ruling is not correct as it is based on extra judicial confession which
is not corroborated.
(c) Court’s ruling is correct as it is based on accuser’s confession.
(d) Cannot be determined as facts lack information.

. Page 29 of 40
99. Continuing with the same facts as stated above, suppose Madhu was harassed and
made a victim of custodial
violence. In her confessional statement before the court, she admitted to her crime
as well asthe terrible treatment
she received during the inquiry. Can she be shielded from such atrocities?
(a) Yes, it is the duty of the judicial officer that the accused is not exploited.
(b) No, magistrate’s duty is ensure that the accused is confessing the crime
voluntarily and stating the truth.
(c) Yes, as she confessed the crime voluntarily and was stating the truth.
(d) Can’t be determined as facts lack information.
100. Assertion: It is the judicial officer's or magistrate's responsibility to
guarantee that the accused is safeguarded
in accordance with the rules of Article 20(3).
Reason: No person accused of an offence shall be compelled to be a witness against
himself.
(a) Both A and R are true but R is not correct explanation of A.
(b) Both A and R are true and R is correct explanation of A.
(c) A is true but R is false.
(d) A is false but R is true.
Passage (Q.101-Q.105): Once the express terms have been identified, there is the
question of interpretation. The
document setting out the parties' agreement must be interpreted objectively: it is
not a question of what one party
actually intended or what the other party actually understood to have been intended
but of what a reasonable
person in the position of the parties would have understood the words to mean. The
starting point for ascertaining
the objective meaning is the words used by the parties. These are interpreted
according to their meaning in
conventional usage, unless there is something in the background showing that some
other meaning would have
been conveyed to the reasonable person. Thus, the terms of the contract must be
read against the "factual matrix";
that is, the body of facts reasonably available to both parties when they entered
the contract.
The "parol evidence" rule provides that evidence cannot be admitted to add to, vary
or contradict a written
document. Therefore, where a contract has been put in writing, there is a
presumption that the writing was
intended to include all the terms of the contract, and neither party can rely on
extrinsic evidence of terms alleged
to have been agreed which are not contained in the document. This presumption is
rebuttable, and extrinsic
evidence is admissible, if the written document was not intended to set out all the
terms on which the parties had
agreed. The parol evidence rule prevents a party from relying on extrinsic evidence
only about the contents of a
contract (and only express terms), and not about its validity (such as the presence
or absence of consideration or
contractual intention, or where a contract is invalid for a reason such as
incapacity).
(Source: http://www.a4id.org/wp-content/uploads/2016/10/A4ID-english-contract-law-
at-a-glance.pdf)
101. What may be said to be best describe express terms in a contract, according to
your understanding of the passage?
(a) Express terms are ones that the parties have set out in their agreement and
subject to no interpretation.
(b) Incorporation without express reference depends on the intention of the
parties, determined in accordance
with the objective test of agreement.
(c) Consideration clause expressly stated in an agreement cannot be relied upon
extrinsic evidence.
(d) If the written document was meant to give forth all of the terms on which the
parties had agreed, extrinsic
evidence is admissible.

. Page 30 of 40
102. Astha and Bhargavi launched a firm in a partnership agreement, and they also
obtained a collateral loan from
Dena Bank. Sanjeev, the manager, assisted the two in obtaining a loan with a
cheaper interest rate. The loan was
put out in Astha's name, and Bhargavi became the guarantor. In the event of a
disagreement, both parties agreed
to dissolve the partnership and divest the agreement's assets and obligations, if
any. Bhargavi refrained from
repaying the money because it was in her name. To escape from the liability in the
court of law, what options
does Astha have?
(a) Astha can also hold Bhargavi liable for loan repayment because the loan was
obtained in lieu of a partnership
agreement between the two.
(b) Astha cannot hold bhargavi liable for debt repayment because the loan was
obtained in astha's name rather
than bhargavi's.
(c) As the guarantor, Astha can hold Bhargavi accountable for debt repayment.
(d) Astha cannot hold Bhargavi responsible since the rule of parol evidence applies
in this case.
103. Choose a statement which can be construed true, in the light of given passage,
about parol evidence rule:
(a) The rule of parol evidence is a rule of contract law.
(b) The rule of parol evidnce is applied to all agreements enforceable by law in
india.
(c) Rule of parol evidence do not bars the admission of any extrinsic evidence if
the written document was not
intended to set out all the terms on which the parties had agreed.
(d) Consideration is one of the exceptions to the rule of parol evidence.
104. Chinu came into an agreement with kanishk that he will buy him a “DOVE”, if he
loses the bet. According to
kanishk, dove denotes to an American personal care brand owned by the British
multinational consumer goods
company Unilever. Whereas, in chinu’s interpretation, Dove is an American brand of
chocolate. Chinu loses the
bet, but he did provide the chocolate for Kanishak as promised. Can Kanishak sue
Akash for failing to satisfy
the terms of the agreement in the event of a losing the bet?
(a) Yes as the document setting out the parties' agreement must be interpreted
objectively.
(b) No, as it was kanishk’s fault that he misunderstood what chinu intended to say.
(c) Yes, since there is evidence in the backdrop that another interpretation would
have been given to a reasonable
person.
(d) No, because the body of information was fairly known to both parties when the
contract was signed.
105. Assertion: All express terms are implied in an agreement but not vice versa.
Reason: When a contract is put in writing, there is a presumption that the document
was meant to incorporate all
of the terms of the contract.
(a) Both A and R are true but R is not correct explanation of A.
(b) Both A and R are true and R is correct explanation of A.
(c) A is true but R is false.
(d) A is false but R is true.

. Page 31 of 40
SECTION - D: LOGICAL REASONING
106. Two officers Rathore and Suresh investigated a case of theft in which three
gems – an Emerald, a Ruby and a
Sapphire were stolen. The thieves who stole the stones are Moby, Roxy and Biyani.
Rathore reported that: “Moby
did not steal Emerald. Roxy stole Sapphire” Suresh reported that: “Sapphire was
stolen by Roxy. Biyani stole
Ruby”
If it is known that one statement of each of them is correct and the other wrong,
then who among the following
stole Sapphire?
(a) Roxy (b) Moby (c) Biyani (d) Cannot be determined
107. In a certain code, WHITE is coded as TPRNJ, then which of the following words
can be coded as DXBFV?
(a) BLACW (b) PRANK (c) BYAQK (d) BLACK
108. Pointing to a person in a photograph, Naveen said to Murali, “He is the father
of your sister’s only sibling’s wife
and also he is my wife’s brother”. How is Naveen’s wife related to Murali’s wife?
(a) Mother (b) Mother-in-law (c) Aunt (d) Niece
Passage (Q.109-Q.113): Women as a constituency provide a decisive edge in electoral
contests. It's led to
welfare policies and poll promises tailored to address their specific needs and
subsequent analyses of the impact
of these. Welfare, however, has limitations. It is a safety net, not a tool to
empower women. Empowerment will
come with jobs, which are often the gateway to financial security and a sense of
self-worth, and political power.
There are signs that political parties are now beginning to move beyond targeted
welfare schemes to measures
which can truly empower women.
Two recent examples are worth noting. Delhi's government is trying to get more
women to own and operate
public transport through measures such as reserving licences and joint ownership.
In Tamil Nadu, the DMK's
success in urban local body polls led it to appoint women as mayors in 11 of the 20
corporations, including
Chennai. Important as these measures are, they are not foundational. The
precondition in empowering women
is to impart confidence that they can access public spaces without fear. Data shows
a strong correlation between
perceptions of safety and women's participation in the workforce.
GoI's employment data for the last full pre-pandemic year, 2018-19, showed that
53.6% of the population in the
15-59 age group was in the job market. The discrepancy between women and men was
stark. A mere 26.5% of
the women were in the labour force as compared to 80.3% of the men. It's the
regional variation that foregrounds
the safety factor. Southern states, Goa, Maharashtra, HP, Chhattisgarh and Sikkim
were among states with a
relatively high percentage of women in the workforce. Among states trailing the
national average were UP,
Bihar, West Bengal and Delhi. There are other reasons why so few women look for
work. But unsafe public
spaces are the most important.
(Source –The Times of India, 19th march)
109. Which of these statement, if true, weakens the author’s argument?
(a) Haryana which has one of the lowest numbers of women in workforce has unsafe
working spaces.
(b) Maharashtra is known for being one of the most unsafe places for women.
(c) Lesser payments, family pressure are also one of the prominent reasons for less
number of women in
workforce.
(d) Women’s lacks equal status and needs empowerment.
110. The author is most likely to agree with which of the following actions by the
government: -
(a) Coming with new welfare measures for women and implementing the already
existing ones.
(b) Providing reservations for women in work force to increase their
representation.
(c) By making the public and work spaces more sensitive and safer for women.
(d) By promoting education, training and professional development for women.

. Page 32 of 40
111. Which of the following can be said as assumption of the above passage?
(a) The discrepancy between women and men working in labor force is stark.
(b) Welfare policies will not themselves lead to empowerment of women.
(c) Unsafe public spaces are one of the most important causes of lack of women’s
representation in workforce.
(d) Unsafe public and work spaces are one of the contributing factors to low
representation of women in
workforce.
112. What purpose the boldfaced statement in the passage serves?
(a) It is premise used to support author’s argument.
(b) It is one of the arguments of the author which has been supported above.
(c) It is the premise to support the central idea of the passage.
(d) It plays the role of counter-premise in the passage.
113. What is the central idea of this passage?
(a) There's a strong correlation between safe public and work spaces and women in
jobs.
(b) Political parties are now beginning to move beyond targeted welfare schemes.
(c) There are less number of women in the job market compared to men.
(d) Safety concerns supersede any other factor in women environment.
Passage (Q.114-Q.119): The UN Security Council’s decision to authorise a new
mandate for the organisation
in Afghanistan is the most emphatic step taken by the world body in tackling the
myriad problems the country
has been facing ever since the Taliban takeover. According to the UNSC resolution
which was adopted by a vote
of 14-0 with Russia abstaining, the UN mission in Afghanistan is authorised to
promote gender equality, the
empowerment of women and girls, human rights and an inclusive and representative
government. There is no
direct reference to the Taliban, but it urges the mission to work in “close
consultation with all relevant political
actors and stakeholders....” The Taliban have called for international help but
shown no signs of sharing power
or respecting the basic rights of Afghans. In the 1990s, the Taliban excluded girls
and women from education
and work and were hostile to ethnic and religious minorities. The Sunni
fundamentalist group formed a Pashtun#dominated male government that was reluctant
to allow girls to attend primary schools; older girls are still denied
education.
But the Taliban’s fundamentalism and their lack of legitimacy should not prevent
the international community
from working to ameliorate the suffering of the Afghan people. The country is going
through one of its worst
humanitarian crises. Before the Taliban takeover, two-thirds of the Afghan
government’s expenditure came
through donations. As no country has recognised the Taliban as the legitimate
rulers of Afghanistan, these
donations have dried up since August. The U.S.’s decision to freeze $9 billion in
assets belonging to the Afghan
central bank has deepened the crisis. Even government employees have not been paid
their salaries for months.
Its economy is expected to contract by 30% this year and nearly every Afghan
citizen could be living in poverty
by mid-2022, according to the UN. The international community is reluctant to step
in over fears that the Taliban
would use the aid to consolidate their power and resist further demands for
reforms. But the international
community cannot just look away when Afghans face mass starvation. With the fresh
mandate, which got the
support of almost all the major powers, the UN mission should start engaging the
Taliban. This does not mean
that the member countries should offer quick recognition to the Taliban regime.
They should offer humanitarian
assistance to the Afghans in consultations with the Mullahs, while at the same time
putting pressure on them to
accept at least short-term reforms and take measures to respect basic human rights.
(SOURCE - https://www.thehindu.com/opinion/editorial/helping-afghanistan-the-hindu-
editorial-on-the-un#mission-in-afghanistan/article65237823.ece/amp/)
114. ‘The international community is reluctant to step in over fears that the
Taliban would use the aid to consolidate
their power and resist further demands for reforms.’ The following sentence is
(a) A fact. (b) A judgement (c) An inference (d) A premise

. Page 33 of 40
115. The author is most likely to agree with which of the following actions by the
countries?
(a) Giving humanitarian assistance to the Afghans as well as legal recognition to
Taliban.
(b) Preparing an international alliance of militaries to fight against the
oppression of Taliban.
(c) Conducting peace campaigns and UN meeting discussing the situation at
Afghanistan.
(d) International communities to come forward in supplying food and other support
to Afghans.
116. Which of the following, if true, strengthens the author’s argument?
(a) Only 2% of Afghanistan’s population have enough food, according to the World
Food Programme.
(b) Taliban doesn’t want to involve international community in the matters of
Afghanistan.
(c) Taliban has brought some positive changes, such as strengthening of army for
national security.
(d) Afghanistan has always been a poor nation and, therefore, present conditions
are no different.
117. Which of the following conclusions cannot be derived from the passage?
(a) Afghanistan’s economy majorly relies on fundings by other countries.
(b) The Taliban’s’ regime is a mandate of the people of Afghanistan.
(c) Taliban is an extremist organization believing in absolute power.
(d) Afghanistan may see an unprecedented economic slump.
118. Which of the following statements, if true, weakens this statement, “The
Taliban’s fundamentalism and their
lack of legitimacy should not prevent the international community from working to
ameliorate the suffering of
the Afghan people”?
I. International law requires legal recognition of a government as the precondition
for providing humanitarian
assistance to a country.
II. Afghanistan's large number of working women are not allowed to go to their
places of work.
III. International community can help Afghans by providing humanitarian assistance
to them.
IV. Fundamentalism, inherently runs on the stringent diktats of conserving extreme
and parochial views that do
not serve interests of people at large.
(a) Only I. (b) Only IV (c) I & IV (d) I, III & IV
119. Assuming the information and arguments above to be true, which one of the
following will be true?
(a) International community can overlook the legitimacy of a regime.
(b) The financial assistance will reach the suffering people of Afghanistan.
(c) Taliban will run the country effectively, once wherewithal is in place.
(d) Taliban will eventually have to accede to short-term reforms and respect basic
human rights.
Passage (Q.120-Q.124): During its first stint in office in Delhi in 2013, the party
AAP that had emerged from
the Anna movement appeared to rely more on a politics of confrontation and protest
vis a vis the Centre — a
vestige of its roots in the anti-corruption campaign – rather than the hard labour
of governance. Then, in its last
attempt at capturing power in Punjab in 2017, the AAP’s prospects seemed dented by
the fact that it did not have
a chief ministerial candidate, was accused of flirting with Khalistani elements and
its “high command” was seen
to be micro-managing the state unit from Delhi. Clearly, the party leadership took
cognisance of these
shortcomings, running a campaign that highlighted its achievements in health and
education in Delhi, naming a
CM candidate and steering clear of controversial or polarising issues. The politics
of solutionism that it has
showcased in Delhi may not be enough, however, to address the complex issues
plaguing the state.
Both industry and the young are fleeing Punjab and drug addiction continues to
increase. Tensions around alleged
acts of sacrilege complicate the dynamics in the border state where memories of the
decade lost to terrorism
haven’t faded. To build on its victory, the AAP must now construct a “Punjab model”
that responds to the strong
desire for change that has propelled it to power with such a large mandate.
[Source: https://indianexpress.com/article/opinion/editorials/punjab-assembly-
election-aap-win-delhi-model#7814069/ ]
. Page 34 of 40
120. The author is most likely to agree with which of these statements:
(a) AAP failed in 2017 elections in Punjab due to its unfit chief ministerial
candidate.
(b) The party could emerge a winner as a model of governance if it brings about a
metamorphically driven
changes in Punjab.
(c) The solutionism approach that AAP has achieved in Delhi will help the party to
emerge as loved government
in Punjab.
(d) AAP construction of “Punjab Model”, will further increase chances of party to
win in other states as well.
121. Which of the following can be inferred from the above passage?
(a) AAP government have always been clear with their decisions of closing best CM
candidate, highlighting its
achievements and campaigning, which led to its victory in Punjab in a short period.
(b) Most of the people from Punjab are migrating and the addiction in the state
have increased to extreme levels.
(c) The Punjab model by AAP requires a reconstruction, addressing the issues
plaguing the state.
(d) Demonstrating the AAP's solutionism in Delhi will help the party become a
successful party in Punjab.
122. Which of the following, if true, weakens the last statement of second
paragraph?
(a) To capitalise on its triumph, the AAP must now develop a paradigm that
addresses the tremendous demand
for change that pushed it to power with such a wide mandate.
(b) The victory of AAP in Punjab was a result of it being seen as a last resort
amid disillusionments with other
party candidates.
(c) The showcase of Delhi Model was one of the major reasons that AAP could succeed
in Punjab elections.
(d) Some of Punjab population is migrating, and the state's addiction has reached a
high, which will create
hindrance for a Punjab model construction.
123. What purpose “The party leadership took cognizance of these shortcomings,
running a campaign that highlighted
its achievements in health and education in Delhi” in the passage serves?
(a) To bring out the flaw of AAP's replication of the model in Punjab elections.
(b) To highlight a tactical political maneuvering by AAP upon realizing its past
mistakes.
(c) To indicate the AAP’s follies in previous election, and learn a lesson for the
next campaign.
(d) To bring to fore loopholes in AAP’s electoral campaign of 2017 as lesson to be
learnt for other parties.
124. “AAP’s victory of Punjab can be sustained if it diligently works on the issues
of drug addiction and border
tensions.”
The above statement is:
(a) Probably true (b) Probably false (c) Definitely true (d) Definitely false
Passage (Q.125-Q.129): At a snap virtual meeting of the Quadrilateral Security
Dialogue or Quad, comprising
India, the U.S., Australia and Japan, leaders discussed the crisis of Russia’s
invasion of Ukraine along with more
traditional topics of interest for the Dialogue, including territorial and maritime
security across the Indo-Pacific.
In the joint statement, issued after the summit, the four nations reaffirmed their
commitment to a free and open
Indo-Pacific, “in which the sovereignty and territorial integrity of all states is
respected and countries are free
from military, economic, and political coercion”. The latest Quad meeting was in
part likely motivated by the
concern of the U.S., Australia, and Japan that India, in not explicitly condemning
Russian President Vladimir
Putin’s decision to launch a ground offensive across the Russia-Ukraine border and
to bomb Ukrainian
infrastructure, might not be on the same page as the other Quad members vis-à-vis
this conflict. They have not
only condemned Russia’s aggression but have also slapped Kremlin elites and
organisations linked to them with
crippling sanctions. India, contrarily, has abstained from three UN resolutions
condemning Russia. There is also
a considerable difference on the Russia-Ukraine issue in terms of the individual
readouts of the Quad members.
While the U.S., Australia and Japan directly called out Russia’s attempt to
unilaterally force changes to the status
quo in Ukraine and vowed not to let such action occur anywhere in the Indo-Pacific,
India’s readout only
referenced Ukraine in passing, in the context of establishing a new humanitarian
assistance and disaster relief

. Page 35 of 40
mechanism for this cause. The Quad, cannot afford to alienate India, a critical
partner in the global-strategic plan
to balance the rise of China as a potential Asian hegemon.
[Source: https://www.thehindu.com/opinion/editorial/quadrilateral-queasiness-the-
hindu-editorial-on-indias#stance-in-russia-ukraine-crisis/article65198277.ece]
125. Which of the following, if true, strengthens the above arguments?
(a) The situation in Ukraine is complicated and it can be immediately resolved with
the help of powerful
organisations, such as Quad.
(b) The situation in Ukraine in critical which can be only resolved by
intervention.
(c) The situation in Ukraine is tense, but intervention of third countries may
aggravate the conflict.
(d) In 2014 when Russia attacked Crimea, sanctions were put in place against Russia
which crippled its
economy, thereby ending the war.
126. What is the central idea of the passage?
(a) India’s stance of abstaining may prevent the conflict to bring it to an end.
(b) Though difference in opinions, Quad has taken pro-active role in Russia-Ukraine
crisis, and is disillusioned
with India siding with Russia.
(c) Quad has been dedicating its focus to Indo-pacific region to control China’s
hegemon with or without India
as its strategic partner.
(d) Despite India’s non-committal stance on Russia, the Quad cannot dispense with
India as part of a strategy to
keep in check China’s Asian dominance.
127. ‘India, contrarily, has abstained from three UN resolutions condemning
Russia.’ What is the assumption behind
the statement?
(a) India does not agree with the UN resolutions.
(b) India reflects Russia.
(c) India supports Russia against Ukraine.
(d) India believes in taking neutral stand to protect itself
128. Which of the following can be inferred from the passage above?
(a) Quad is much focused on Indo-Pacific region, especially after Russia-Ukraine
conflict.
(b) UN resolutions play a major role to pacify the world order during wartime
situations.
(c) The quad members have slapped Kremlin elites and organisations linked to them
with crippling sanctions.
(d) India has played the least role amongst the countries in easing tensions
between the conflicting countries.
129. Which of the following will follow the last sentence as a conclusion to the
passage?
(a) The idea that Quad countries or even Russia can force sovereign nations with a
proud history of non#alignment to pick a side in a complex geopolitical conflict is
quite passé.
(b) New Delhi continues to be guided by the 21st century variant of its non-
alignment paradigm.
(c) India cannot be forced to pick a side in the conflict, but Russia could test
its resolve.
(d) Yet, India may find its resolve and patience with Russia tested should Russian
occupying forces begin
committing war crimes and human rights violations.

. Page 36 of 40
Passage (Q.130- Q.132): These questions are based on the information given below.
As part of the annual meeting, the CEO of a company called five of the employees to
his office. Each of them
has a different designation among Regional Manager, General Manager, Assistant
Manager, Sr. Systems
Engineer and Sr. Accountant and earns Rs.90,000, Rs.80,000, Rs.60,000, Rs.55,000
and Rs.55,000 per month
not necessarily in the same order.
Stephine, the Assistant Manager earns less than Gagan. Ramesh earns the least
amount. The Regional Manager
earns the highest and the Sr. Systems Engineer earns the least. Gagan is not the
Regional Manager. Prakash earns
more than Krishna, Ramesh is not the Sr. Systems Engineer.
130. How much does Stephine earn (in Rs.)?
(a) 80,000 (b) 60,000 (c) 55,000 (d) 90,000
131. What is the designation of Prakash?
(a) Regional Manager (b) General Manager
(c) Sr. Accountant (d) Assistant Manager
132. Which of the following is a correct combination of Employee, Designation and
Earnings per month (in Rs.)?
(a) Gagan – Sr. Accountant – 80,000
(b) Krishna – Sr. Systems Engineer – 55,000
(c) Ramesh – General Manager – 55,000
(d) More than one of the above
Passage (Q.133-Q.135): “Sometimes fate is like a small sandstorm that keeps
changing directions. You change
direction but the sandstorm chases you. You turn again, but the storm adjusts. Over
and over you play this out,
like some ominous dance with death just before dawn. Why? Because this storm isn’t
something that blew in
from far away, something that has nothing to do with you. This storm is you.
Something inside of you. So all
you can do is give in to it, step right inside the storm, closing your eyes and
plugging up your ears so the sand
doesn’t get in, and walk through it, step by step. There’s no sun there, no moon,
no direction, no sense of time.
Just fine white sand swirling up into the sky like pulverized bones. That’s the
kind of sandstorm you need to
imagine.
And you really will have to make it through that violent, metaphysical, symbolic
storm. No matter how
metaphysical or symbolic it might be, make no mistake about it: it will cut through
flesh like a thousand razor
blades. People will bleed there, and you will bleed too. Hot, red blood. You’ll
catch that blood in your hands,
your own blood and the blood of others.
And once the storm is over you won’t remember how you made it through, how you
managed to survive. You
won’t even be sure, in fact, whether the storm is really over. But one thing is
certain. When you come out of the
storm you won’t be the same person who walked in. That’s what this storm’s all
about.” – Haruki Murakami, Kafka On The Shore.
133. What does the author imply through “symbolic storm” in the passage?
(a) The experiences of life.
(b) The Problems of life disguised as fate.
(c) Good things that life has to offer to you.
(d) The challenging destiny that one cannot escape.
134. All of the following can be inferred from the passage, except?
(a) Challenges alter the way one looks at life.
(b) Challenges can render you helpless.
(c) One can escape challenges by facing them.
(d) None of the above.

. Page 37 of 4

MOCK 39
SECTION-A: ENGLISH LANGUAGE
Directions (Q.1-Q.30): Read the following passage carefully and answer the
questions that follow.
Passage (Q.1-Q5): Given how few voters enjoy paying them, politicians rarely
trumpet the advent of new taxes.
But the passage of a new goods-and-services tax (GST) in India’s upper house on
August 3rd is a deserved
exception. Well over a decade in the making, the new value-added tax promises to
subsume India’s miasma of
local and national levies into a single payment, thus unifying the country’s 29
states and 1.3 billion people into
a common market for the first time. The government of Narendra Modi, never averse
to over – hyping what turn
out to be modest policy tweaks, has enacted its most important reform to date.
Both the central government and powerful state legislatures impose a dizzying array
of charges. Because the
rates differ between states, making stuff in one and selling it in another is often
harder within India than it is in
trade blocs such as NAFTA or the European Union. Queues of lorries idle at India’s
state boundaries much in
the same way they do at international borders.
That should change with the GST, essentially an agreement among all states to
charge the same (still to be
decided) indirect tax rates. Businesses are thrilled at the idea of being able to
distribute their products from a
single warehouse, say, rather than replicating supply chains in each state. Thick,
exception-riddled tax codes—
car sales are liable to six different levies at various rates, depending on the
length of the vehicle, engine size and
ground clearance, for example—are to be replaced with a single GST rate to be
applied to all goods and services.
Better yet, the GST will be due on the basis of value added. That avoids businesses
being thwacked by taxes on
the entire value of the products they buy and sell rather than just the value they
create — a situation that often
made it cheaper to import stuff rather than make it locally. Just as importantly,
by requiring businesses to
document the prices at which they buy inputs and sell products (unless they wish to
pay higher taxes), it will
force vast swathes of the economy into the reach of the taxman.
Economists and technocrats have long backed the GST, which they think could boost
economic output by 1-2
percentage points a year.
Because the tax overhaul requires a new amendment to the constitution, and
therefore the backing of at least 15
state legislatures, it will take several months to enact. Few expect it to be
derailed, but a deadline of April 2017
seems unlikely to be met.
Such nitty-gritty will be fought over in the “GST council”, a novel body which will
represent both state and
federal executive branches but looks likely to be dominated by ministers sitting in
New Delhi.
Indeed, the new council and the tax it will administer go against a recent trend
for decentralising power from New
Delhi to the various state capitals. Powerful chief ministers sitting in the
provinces will be more dependent on
revenue collected federally and less on purse-strings they control themselves.
Money will shift from (richer)
states that make things towards (poorer) ones that consume them, too. The advent of
a single tax to rule them all
may come to shape Indian politics as much as it does the economy.
1. Which of the following could be an implication of the uniform taxation system to
be implemented shortly?
(a) There will be unpredictable consequences.
(b) Financial autonomy of manufacturing states would be affected.
(c) It will increase the revenue collection of states
(d) Revenue generation from origin-based taxes will increase while that from
destination-based taxes will suffer
losses.
2. The Goods and Services tax has been touted to do which of the following?
(a) Boost economic activity by a few percentage points each year.
(b) Ease the burden on the common man by doing away with multiple taxes.
(c) Ensure transparency in taxation.
(d) All of the above.

. Page 3 of 40
3. Which of the following is the primary objective of the primary Goods and
Services tax?
(a) To manage a slew of central and state levies.
(b) To divide all cobweb taxes into various categories.
(c) To dismantle inter-state barriers to trade and create a united market of 1.3
billion people.
(d) To take care of the cascading effect of direct taxation.
4. How is the proposed GST framework different from the existing tax scenario in
India?
(a) The GST is a comprehensive tax that will include all other taxes levied.
(b) The GST shall subsume all border taxes.
(c) The GST shall be levied in addition to all local and national level taxes.
(d) Credits of input taxes paid at each stage will be available at every stage of
value addition, which makes GST
essentially a tax only on value addition at each stage.
5. From the passage, it is clear that the Goods and Services tax has been
(a) welcomed as a breakthrough to unify India's economy to attract foreign direct
investment.
(b) hailed as an important and much-needed tax reform in the country.
(c) lauded as the biggest reform in India so far.
(d) denounced as a tax reform that will not benefit the economy.
Passage (Q.6-Q.10): In 2003, Anthony M. Perks came up with an anatomical
explanation for Stonehenge, the
prehistoric monument in England whose precise purpose is a mystery. “Stonehenge
could represent,
symbolically, the opening by which Earth Mother gave birth to the plants and
animals on which the ancient
people so depended,” he wrote in an essay published in a medical journal. It could
depict, he suggested, “the
human vulva, with the birth canal at its centre.” The vulva hypothesis is one of
the myriad theories that have
proliferated around Stonehenge, which was constructed some 4,500 years ago. He also
highlighted that while it
was built at roughly the same time as the Sphinx and the Great Pyramid of Giza, we
know far more about those
Egyptian sites. Incomplete knowledge of Stonehenge has turned it into a riddle that
is now part of its identity.
Some believe it to be an astronomical calculator or an observatory that helped
demarcate the seasons. Others
view Stonehenge as a place of healing, a kind of prehistoric Lourdes, which hosted
hordes of pilgrims. In the
1960s and ’70s, the site was thought to be imbued with magical and mystical powers,
and it became a hot spot
for hippies and open-air festivals. Today, it’s a focal point of New Age
counterculture and environmental
activism. Stonehenge also attracts plenty of alien-origin theories, prompted by the
belief that human beings could
not possibly have raised those structures by themselves. According to these
theories, Stonehenge was built by
extra-terrestrials, and it’s actually a landing pad for spacecraft.
“Stonehenge was an important place that people went to, to be together as a
community,” said Neil Wilkin. He
described the site as a mix between a town hall and a cathedral, where people
mingled for both religious and
social reasons. Another recent discovery revealed that some of the pilgrims who
helped build Stonehenge stayed
at Durrington Walls, a nearby settlement which, at its peak, contained around 1,000
temporary houses.
Stonehenge was built at a time of drastic population decline and dispersal, said
Mike Parker Pearson. There were
few, if any, villages, and society was “trying to create a sense of unity and
collaboration among its members,”
he explained. Built on the site of an ancient cemetery, Stonehenge was a “monument
of remembrance,” he said,
and an “expression of unity” that pulled people together in the pursuit of a common
endeavour. Analysis of
stable isotopes — meaning atoms that have additional or missing neutrons — is being
used to study bones, tooth
enamel and food residues on pots and elsewhere to determine what a person of the
time ate and how far they
moved around. Tooth enamel contains a kind of chemical record of the climatic and
geological conditions in
which a person grew up, allowing archaeologists to work out how far people
travelled from their birthplaces and
to map out migration and mobility, Pearson explained.

. Page 4 of 40
6. How did the interpretation of Stonehenge among the people modify with the
passage of time?
(a) With the passage of time, the belief of Stonehenge being a spacecraft landing
platform by the aliens started
spreading.
(b) During 1960-the 70s, the place was told to be imbibed with magic, crowded by
hippies and outdoor events
and now, it is a hub for New Age counterculture and ecological movement.
(c) Stonehenge sparked a slew of alien-origin ideas, powered by the belief that
humans could not have built
those monuments on their own, whereas before, it was said to be imbibed with magic
and outdoor events.
(d) None of the above.
7. Which of the accompanying indirectly disapproves the theory that Stonehenge was
created by the extra#terrestrials?
(a) The discovery of thousands of temporary homes of pilgrims at Durrington Walls,
a nearby village and they
assisted in the construction of Stonehenge.
(b) Stonehenge could symbolise the entrance through which Earth Mother gave life to
the plants and animals
that the ancient people relied on so heavily.
(c) The location was described by Neil Wilkin as a cross between a town hall and a
cathedral, with people mixing
for religious and social purposes.
(d) A map was discovered that contained the engineered layout of the structure and
the language used in it was
identical to the dialect used by the ancient people.
8. Which of the arguments were put forward by Mike Parker Pearson in respect to
Stonehenge?
A. If there were any communities, they endeavoured to develop a sense of oneness
and co-operation among its
members by building the structure.
B. It was a monument of recollection and an expression of solidarity that brought
people together in the pursuit
of a single goal.
C. It was built at a time of rapid population explosion and the influx of a large
number of people in that area.
(a) Only A and B (b) Only B and C (c) Only A and C (d) A, B and C
9. Which of the following theories were held by people about Stonehenge?
A. The extra-terrestrials had built it and more precisely, it was a spacecraft
launching platform.
B. A healing site that attracted throngs of pilgrims.
C. It was used for calculating astronomical factors.
(a) Only A and B (b) Only B and C (c) Only A and C (d) A, B and C
10. Which of the following views or ideas are NOT from Anthony M. Perk’s account of
Stonehenge?
(a) He speculated that Stonehenge must be a representation of the human genitalia
with the centre as the birth
canal.
(b) The mystery of Stonehenge is the part and parcel of its attraction.
(c) Like the Sphinx and the Great Pyramid of Giza, the information about Stonehenge
is still a mystery.
(d) Stonehenge is a symbolic representation of the aperture through which earth
provided sustainability to early
humans.
Passage (Q.11-Q.15): Edison had a very specific and peculiar way of interviewing
research assistants for his
labs. He'd invite candidates out for a meal and then order soup for the table. "The
reason for this soup test was
that the famous inventor wanted to see if the applicants added salt and pepper
before tasting what was in their
bowl, or if they waited until they tasted it before proceeding with the seasoning,"
Martin explains. "Edison
immediately rejected the premature seasoners, as he reasoned he didn't want
employees who relied on
assumptions. In his opinion, those who were content to abide by preconceived
notions had no place in his
business, because the absence of curiosity and willingness to ask questions were
antithetical to innovation."
This trick would not be at all practical in today's business environment. Using
restaurant meals to screen
candidates doesn't exactly scale as plenty of today's candidates would have an
allergy or dietary restriction that

. Page 5 of 40
precluded whatever soup you ordered. But even more fundamentally the idea just
sounds crazy. Is there anything
besides nutty professor idiosyncrasy behind Edison's soup-based interviewing
technique? While using soup
alone to assess job applicants is clearly not a great idea, reading about Edison's
unusual approach to hiring did
remind us of a body of research on job interviews that actually suggests Edison's
idea isn't as crazy as it first
sounds.
First off, it's important to know that study after study shows that interviews, as
they're usually conducted, are
pretty close to useless. Asking people questions, even expert-recommended
behavioural or hypothetical
questions tend to advantage slick talkers over the actually competent. Interviewers
are also notoriously swayed
by biases and irrelevant details of self-presentation. Also, in some cases,
interviewers become subconsciously
biased towards good-looking candidates. What does modern science suggest instead?
Perhaps not so
surprisingly, just testing candidates on the actual skills and competencies
required to do the job. A trial
assignment, sample work project, or domain-specific test far outperforms just
talking with candidates about their
previous work experience, character, and goals.
In short, actions speak much louder than words. This means maybe Edison's weird
soup test wasn't so crazy after
all. When someone seasons their food is a real-world behavioural indicator of how
they think through problems
and, as such, is more likely to reveal something true about their mindset than
abstract discussions. The takeaway
here isn't that you need to take potential new hires out to lunch (unless you're
hungry or obsessed with table
manners). The point is that Edison was right about the fundamental truth of
interviewing. If you want to really
understand who candidates are and what they can do, design ways to observe them
solving relevant problems.
You'll always get a better sense of a person from what they do than from what they
say.
11. Which of the following points are expressed by the author to support the idea
that the interviews that are
conducted in today’s world are actually useless?
A. Attractive people seem to get an upper hand in the selection process of
interviews in some cases.
B. Biases and unimportant elements of self-presentation are famously swaying
interviewers.
C. Smooth talkers tend to win out over the truly skilled candidates.
(a) Only A and B (b) Only B and C (c) Only A and C (d) All A, B and C
12. Which of the accompanying is NOT correct in reference to the passage?
(a) Techniques must be designed to see how candidates handle important tasks if
they genuinely want to know
who they are and what they can achieve.
(b) We can get a broader picture of a person by analysing what they say rather than
what they do.
(c) Seasoning one's food is a genuine behavioural predictor of how one thinks about
problems; hence, is more
likely to indicate something true about one's attitude than theoretical
conversations.
(d) In conclusion, the author mentions that the soup test by Edison is actually
effective, though he expressed his
doubts before.
13. Which of the accompanying was the principle held by Edison on which he rejected
the candidates at the soup
test?
(a) The candidates, who added salt and pepper after tasting the soup, were the ones
who didn’t use assumptions
in their life as they weren’t content to follow the traditional conceptions and
hence, they were rejected as
they lacked curiosity and a readiness to ask questions that were contradictory to
creativity.
(b) The candidates, who added salt and pepper before tasting the soup, were the
ones who used assumptions in
their life as they were content to follow the traditional conceptions and hence,
they were rejected as they
lacked curiosity and a readiness to ask questions that were contradictory to
creativity.
(c) The candidates, who added salt and pepper before tasting the soup, were the
ones who used assumptions in
their life as they were content to follow the traditional conceptions and hence,
they were selected as they
brimmed with curiosity and a readiness to ask questions that were synonymous to
creativity.

. Page 6 of 40
(d) The candidates, who added salt and pepper after tasting the soup, were the ones
who used assumptions in
their life as they were content to follow the traditional conceptions and hence,
they were rejected as they
lacked curiosity and a readiness to ask questions that were contradictory to
creativity.
14. What is the tone of the author in the passage?
(a) Introspective (b) Obsequious (c) Humanistic (d) Vitriolic
15. Why does the author say that the soup test by Edison might be impractical in
today’s world?
(a) Many of today's prospects have allergies or dietary restrictions that would
prevent them from eating whatever
soup you ordered.
(b) Due to the fast-paced world, there isn’t enough time to call in each and every
candidate to a restaurant and
conduct the soup test on them.
(c) There isn’t anything to Edison's soup-based interviewing approach other than
crazed professor idiosyncrasy.
(d) With the setting in of the digital world, many of the technicalities of the
past interview styles are archaic, to
say the least.
Passage (Q.16-Q.20): Democrats are getting push-back from Republicans over their
calls to require Americans
to use coronavirus passports. President Biden is reportedly working with tech
companies to create an app
showing proof of vaccination. If the “passports” were used on a national scale,
businesses would require
Americans to show their passports to be able to use their services. New York became
the first state to enforce
such a rule using IBM’s Excelsior Pass app, which shows businesses a user’s
personalized QR code.
However, several Republicans are pushing back against the idea and are saying it
infringes on Americans’
individual freedoms. Critics are lambasting Democrats who are promoting vaccine
passports while arguing it
shows their hypocrisy. House Republican Whip Steve Scalise said he finds it strange
that Democrats are quick
to oppose and vilify voter ID laws yet they support Americans having to show IDs
when going about their daily
lives.
“Considering that Democrats want to require vaccine IDs for people to conduct their
basic daily activities, they
now have zero grounds to object to voter ID laws,” Scalise said in an interview.
“If under Democrat logic, you should need an ID to enter even a grocery store,
surely there wouldn’t be an
objection to showing an ID to legally vote.”
Additionally, Ohio Rep. Jim Jordan took to Twitter to condemn the Biden
administration for supporting the idea
of showing vaccination passports. He tweeted:
The Biden Administration:
–Considering a “vaccination passport” for Americans.
–But doesn’t seem to care about passports when it comes to illegal migrants
crossing the southern border.
Freshman Rep. Madison Cawthorn, R-N.C., strongly condemned the idea of vaccine
passports.
“…We must make every effort to keep America from becoming a ‘show your papers
society,'” Cawthorn told
Fox News. “The Constitution and our founding principles decry this type of
totalitarianism.”
“America faces a dangerous future when its leader’s ideology shares more
commonalities with Leninism than
liberalism,” the North Carolina Republican added.
Other Republicans are expressing concerns over privacy issues that could arise from
mandating the apps. They
believe the ethical problems outweigh the potential benefits of the passports.
In the meantime, Florida Gov. Ron DeSantis is the first GOP lawmaker to take
concrete steps against the
measures. The Republican said he will take executive action if the federal
government and private sector move
to impose the passports. DeSantis added, the nation should not have to take such
drastic measures as long as
health officials can effectively vaccinate Americans.
New York has already developed its own version called the “Excelsior Pass.”
Democratic Gov. Andrew
Cuomo’s office says the digital app doesn’t display the user’s health information
(but does not say if it is there
in the background), and that it will be required to gain entry to large venues like
NBA arenas Madison Square
Garden and Barclay’s Center. Theaters, businesses and stadiums have also been
listed as requiring the vaccine
passport for entry. No word on whether restaurants, schools, supermarkets, malls,
etc. will make the “vaccine

. Page 7 of 40
passport” mandatory for entry. Israel is already using a “green passport” to allow
only people who have their
vaccines or have recovered from COVID-19 to attend public events, such as concerts.
16. According to the passage, which of the following is/are true?
I. New York has already developed its own version called the “Excelsior Pass.”
II. New York became the last state to enforce such a rule.
III. Several Republicans are pushing back against the idea and are saying it
infringes on Americans’ individual
freedoms.
(a) Only II (b) Only I and III (c) Only I (d) None of the above
17. On what is President Biden reportedly working with tech companies?
(a) To create an app showing proof of passport.
(b) To create an app for showing current health status.
(c) To create an app for entry and exit from America.
(d) To create an app showing proof of Coronavirus vaccination.
18. What is the meaning of “no word on” in context of the last paragraph?
(a) There has been no mention about whether restaurants, schools, etc. will make
the vaccine passport mandatory
for entry.
(b) There has been an argument about whether restaurants, schools, etc. will make
the vaccine passport
mandatory for entry.
(c) There has been detailed mention about whether restaurants, schools, etc. will
make the vaccine passport
mandatory for entry.
(d) There has been no mention about whether large venues, etc. will make the
vaccine passport mandatory for
entry.
19. What is the meaning of the underlined phrase, critics are lambasting democrats?
(a) Excruciating democrats. (b) Complementing democrats.
(c) Condemning democrats. (d) Commending democrats.
20. “If under Democrat logic, you should need an ID to enter even a grocery store,
surely there wouldn’t be an
objection to showing an ID to legally vote.” According to the Republicans, which of
the following word describes
the attitude of the Democrats?
(a) Aristocracy. (b) Hypocrisy. (c) Arrogance. (d) Complacency.
Passage (Q.21-Q.25): At the eternal level, nothing has changed. The sunrise on
January 1 is guaranteed to be
no different from what we experienced on the day before when we told ourselves that
the previous year had
come to an end. Likewise, the water in the pipes on New Year’s Day will fail
exactly like it did on December
31, or 30. The electricity too might drop for an hour or two. Consider the power of
our abstract thought. We have
divided and parceled up Time itself: A.D./ B.C, The Middle Ages, Modern Era, the
21st century, and more. Until
fresh evidence is produced, we are telling ourselves that our species is 70,000
years old — give or take. We have
surrendered to the mythology of the calendar so completely that we are now obsessed
with the passage of years.
We want to either stop it or reverse it. Some people believe that its signs can be
reversed with treatments; we
even make arrangements to preserve our own blood in case we need our past to repair
the possible disaster in
some distant future.
Concepts of timelessness have always bothered humankind ever since we grew
conscious of ourselves as
memory-making creatures. We train ourselves to be mindful of how rapidly things
grow, ripen and work towards
their end. It is something we constantly live with. The Buddhists spend days
drawing elaborate mandalas and
then wipe them out. Likewise, we have the Kalamezhuthu of South India, which too,
is fashioned arduously with
colours and materials found in nature. This too is rubbed out after the ritual
performances end. One of the

. Page 8 of 40
purposes of these acts is to remind ourselves of the impermanence of our own lives
and their products in order
to warn people not to get too attached to the idea of themselves — self-seriousness
being just next door to
narcissism.
“Send me your response by the end of the day” is a frequent demand (bordering on
the discourteous) on email
or phone — another time-related pressure designed to destroy efficiency and
accuracy. And yet another part of
the artificial framework into which we have gracefully inserted ourselves. The
outsourcing of human memory
and ingenuity to electronic data banks is the first step towards our species
becoming redundant someday. If a
smart truck doesn’t need a driver and a robot can do your housework, where does
that leave human cleaners and
drivers? And yet — can any machine replace a teacher like Jothi Thiagarajan? She
places a globe on her student’s
desk, asks him to close his eyes and spin it. She then says: “Place your finger
anywhere on the globe to stop it.
Open your eyes, and tell me what materials you would use to build a house if you
were living where your finger
is?”
It is only in the classroom that one can arrest the spinning of the globe. The only
way we can manage the march
of time is to manage ourselves. For that, we need to look within, examine ourselves
and live with awareness.
Then perhaps we master time by making our days meaningful.
21. A suitable title for the passage would be?
(a) Losing time (b) Time- A Losing Game
(c) Transcending Time (d) Timelessness
22. The word “parceled up” in the context of the passage means?
(a) Allocated the time. (b) Time is cohesive in nature.
(c) Incorporated the time. (d) Wrap up the time.
23. Which of the following has been bothering humankind ever since we grew
conscious of ourselves as memory#making creatures?
(a) Concept of Humankind. (b) Concept of Timelessness.
(c) Concept of Mindfulness. (d) Concept of Humanism.
24. The Kalamezhuthu of South India is fashioned arduously with which of the
following?
I. Colours of nature.
II. Materials found in nature.
III. Inner beauty of a person.
(a) I and III (b) Only II (c) Only III (d) I and II
25. “Send me your response by the end of the day” The tone reflected in the above
is?
(a) Biased. (b) Authoritative. (c) Skeptical. (d) Reflective.
Passage (Q.26-Q.30): I will be going back to my village tomorrow. My brother has
died,” our cook announced
matter-of-factly. “Oh no!” I exclaimed, my hands instinctively coming up to my
heart as I connected with his
inevitably deep grief. “I’m so sorry. That’s horrible. Is there anything I can do
for you?”
“Anyway, he was old, and it is all in God’s plan,” he replied stoically, definitely
not reeling in the ocean of grief
I’d expected.
Moored to my western understanding about life, death seems like a horrible
calamity. Unless, of course, one is
very sick and suffering, any death before a very ripe old age, meaning late 80s or
even 90s, is a grave tragedy.
In India, however, life is seen as that which we are compelled to endure in order
to fulfil our karmic debts, to eat
the metaphoric fruits of our past karma, with the ultimate goal being freedom from
the cycle of life, and eternal
merging into God.
I have found myself struck, again and again, by Indians’ ability to walk toward
death so freely, even eagerly.
In the west, we tend to see illness and death as a failure. We even see ageing as a
failure.

. Page 9 of 40
This collective aversion to the inevitable is largely due to the separate boxes
we’ve stuffed life and death into.
Funerals are either closed casket or the deceased is dressed up and beautified so
completely that they look like
they’re going to the prom rather than into a grave. We have clear-cut boundaries
between places of grieving and
places of celebration, times of grieving and times of rejoicing, times and places
where we whisper or wail about
death, while the rest of the time we pretend it’s only one of several options.
In India, cremations take place at cremation ghats, or riverbanks, and the fires
and smoke are seen by all around.
Death is woven seamlessly into the fabric of life.
Here, in India, sadness at the loss of a loved one feels more like a wave on the
surface of the ocean, turbulent for
a while but mitigated by the mourners’ anchoring in the peace and stillness of the
depths of the ocean. They seem
to have an inherent grounding in ‘This is all God’s plan. We must accept it.’
A lot of this, of course, is due to theological differences in cultures. In the
Hindu tradition, while the loss of a
loved one is sad for us, ultimately it is seen as good for the deceased, for they
have walked one step closer to
merging with God.
But the biggest difference is the belief that ultimately we are not our bodies. And
as Krishn says in the Bhagwad
Gita, we are the eternal soul, that which cannot be burnt by fire, wet by water,
dried by the wind, or cut with
knives. So, while in the midst of grief it is hard for anyone to be philosophical,
nonetheless that awareness, that
cultural and spiritual river of Knowing flows beneath and through the grief,
enabling an awareness that we won’t
be going anywhere, and it will be merely like moving from one room to another room.
26. We can infer from the passage that
(a) The author is a Christian with a deep understanding of Indian religion.
(b) The author is a westerner residing in India.
(c) The author believes that the theological ideologies of India and the west have
common threads.
(d) The author is highly influenced by the Indian culture.
27. ‘Moored to my western understanding about life, death seems like a horrible
calamity.’ What does the underlined
word mean in the context of the passage?
(a) Tied. (b) Secured. (c) Linked. (d) Chained.
28. What is the author trying to communicate?
(a) Unlike the West, death in India is seen as woven seamlessly into the fabric of
life with the deeply entrenched
belief that the soul is immortal.
(b) The West perceives death shallowly and in a compartmentalized manner with a
collective aversion towards
it.
(c) The theological differences in cultures of West and India are stark, especially
concerning the outlook towards
death.
(d) For the author, death is some kind of spiritual and cultural osmosis, the
absolute readiness to go at any
moment, the conscious surrender to whatever is God’s plan.
29. In India, which of the following helps to understand death based on theological
perspective?
(a) Aversion. (b) Acceptance. (c) Communication. (d) Ideology.
30. Which of the following reflects the most appropriate title for the passage?
(a) The Perception of Life and Death.
(b) Ideological and Cultural Differences.
(c) God’s Plan.
(d) Life, Death and the River of Knowing.

SECTION – C: LEGAL REASONING


Passage (Q.66-Q.71): Offer and acceptance.
One party must make the offer and the other party must give its assent to such
offer, thereby accepting it.
Acceptance of offer results in promises that form agreement. The acceptance must be
communicated in an
express or implied mode. Such offer must express willingness to do or abstain from
doing to obtain other’s
assent. The offer may be express or implied but the intention to form a contract
must be there and such offer
must be communicated.
Intention to create legal obligation
There is no express provision in the Indian Contract Act 1872 that makes an
intention to create legal obligation
mandatory but over the years, various judgements have settled the position making
Intention to create legal
obligation as an essential condition.
Consideration is must.
As per section 25 of the Act, an agreement not supported by consideration is void.
A contract without the
consideration will become Nudum Pactum (Bare Contract). Such consideration must be
real and illusionary. The
adequacy of the consideration must is not necessary. But Section 25 lays down a few
exceptions where an
exception without consideration is not void and the examples of such exceptions
include an agreement made on
account of love and affection between parties, an agreement where it is a promise
to compensate, a person who
is already voluntarily done something for the promisor etc.
The parties are competent to contract.
Any person who has attained the age of 18 years, is of sane mind and is not
disqualified by any law is said to be
Competent to form a contract. A contract entered by a minor is void ab initio and
no obligation arises from it.
A minor can plead his minority as a defence in a suit, thus the rule of promissory
estoppel is not applicable.
Contracts must not be uncertain or vague
As per section 29 of the Act, a contract is said to be certain if terms of the
contract can be understood in the way
it was intended to be understood and also are not ambiguous and vague. Such
certainty ensures the consensus ad
idem among the parties and hence trying to prevent any kind of dispute in future.
(Extracted with requisite revisions and edits
fromhttps://lawctopus.com/clatalogue/brief-introduction-to#essentials-of-
contracts/)
66. Madhu went to a party with her friends where a stranger came to her and offered
to sell his watch (seeing which
Madhu was already in awe with it). Madhu despite not knowing who the stranger was
agreed to purchase the
watch at a specified sum of money to be decided later. Has a promise come into
existence?
(a) No, the consideration has still not been decided upon and therefore the
agreement still remains vague as to
consideration and therefore no promise has come into existence.
(b) Yes, the offer by the stranger has been accepted by Madhu and thus a promise
has come into existence.
(c) No, the promise has still not been formed as the details as to the other party
to the contract is not clear and
is thus ambiguous.
(d) Yes, the intention of the parties, in particular the intention to enter into a
contract and to sell the watch is not
clear and therefore no legal promise has come into existence.
67. In the same party, the bracelet worn by Madhu was appreciated by her friend
Rani who expressed her wish to
purchase the same. Madhu in a jovial manner said “Why only bracelet, if you want
why don’t you purchase my
house as well and I will even sell it to you for Rs. 1000.” Next day Rani came to
her house and offered Rs. 1000
to purchase the house. Madhu, however, refused. Has Madhu breached the contract?
(a) Yes, all the essential of the agreement are present in the present factual
context i.e. offer, acceptance, and
valid consideration.
(b) No, the consideration is inadequate for a house and therefore the contract has
not come into existence,
therefore no question of breach can be raised.
(c) Yes, since the offer by Madhu to sell the house has been accepted by Rani any
derogation will amount to
breach of the contract.

. Page 17 of 40
(d) No, there was no intention to create any binding legal intention intended by
Madhu to sell her house and
hence no agreement has been reached between the parties and therefore no question
of breach exists.
68. Ankita when she was a 15 years of age wanted to purchase a violin however, her
parents refused to purchase the
same knowing that she had a concentration of a butterfly and would soon get bored
of it. So, she decided to
purchase it herself and went to a nearby store. She bought the violin on credit and
paid 25 percent of the amount
through her pocket money and promised to pay the rest within next 20 days but
failed. After a month, the
storekeeper went to her in order to extract money and tried to enforce the contract
as she not only purchased the
product but also used it. Can the contract be enforced?
(a) Yes, not enforcing the contract would result in unjust enrichment of Ankita and
hence equity demands the
enforcement of the contract.
(b) No, contract with Ankita would be void from the beginning as she is a minor and
cannot be enforced.
Therefore, no money can be extracted from Ankita by the storekeeper.
(c) Yes, since Ankita has reaped the benefits from the transactions, the
storekeeper is entitled to the remaining
amount from her.
(d) No, since the shopkeeper failed to demand the money within 20 days as agreed
between the parties, it
indicates that he was sleeping over his rights and therefore is not entitled to the
enforcement of contract.
69. The storekeeper asserted that he was not aware that Ankita was not yet a major
but also any reasonable man
would mistake her to be a major. Will this aid to his prosecution of Ankita and for
the enforcement of contract?
(a) No, the fact that Ankita is a minor would entail that the contract is void ab
intio and therefore the contract
cannot be enforced.
(b) No, the failure of the shopkeeper to demand money within 20 days as agreed
between the parties indicates
that he was sleeping over his rights and therefore is not entitled to the
enforcement of contract.
(c) Yes, since Ankita has reaped the benefits from the transactions, the
storekeeper is entitled to the remaining
amount from her.
(d) Yes, according to the reasonable man test, Ankita appeared to be a major and
since it is a civil wrong she
can be accordingly treated to enforce the contract.
70. Brijmohan has always wanted to buy the black horse from Rajesh from his
collection/ stud of high breed horses.
One day, Rajesh told him that he wanted to sell his horse and if Brijmohan wants
they can agree on a specified
amount and then he will sell the horse. Elated by the news, they agreed on an
amount which was paid instantly
online and next day Brijmohan went to take the horse. However, to his utter
disappointment the horse being sold
was a white one and not the one which was thought by Brijmohan. Can he enforce the
contract and obtain the
black horse itself?
(a) Yes, the terms of the agreement are clear as Brijmohan has always indicated his
intention of purchasing the
black horse on various occasions and therefore giving rise to a reasonable notion
that Rajesh intended to sell
the black horse.
(b) Yes, the contract has all the essential elements of the contract namely the
offer, acceptance and consideration
and after the payment of consideration it is Brijmohan who has the say in the horse
he intends to buy.
(c) No, the principle of caveat emptor will be applicable and Brijmohan should have
been vigilant on the details
of the contract before making the purchase.
(d) No, the contract was vague and hence lacked the essential of certainty;
therefore, it cannot be enforced by
Brijmohan.

. Page 18 of 40
71. Brijmohan claims that if Rajesh is unwilling to sell the black horse then he
should atleast refund the amount due
to lack of clarity in the contract terms. However, Rajesh refused saying that the
contract has been formed between
them as there is valid offer, acceptance and consideration. Can there be refund of
amount to Brijmohan?
(a) Yes, there was no consensus ad idem between the parties as to the horse
intended to be sold and no contract
itself has come under existence therefore in the absence of the formation of the
contract the amount can be
refunded to Brijmohan.
(b) No, the contract has all the essential elements of the contract namely the
offer, acceptance and consideration
and refund of the amount will constitute alteration of the agreement which cannot
permitted.
(c) No, altering the terms of the contract would amount to breach of the original
offer and therefore the refund
would not, even if made, be done in entirety and be subjected to valid deductions.
(d) Yes, however as to the amount Brijmohan is entitled to is dependent on the
terms and conditions of the
contract between the parties.
Passage (Q.72-Q.77): Defamation is an injury to the reputation of a person. It is
basically the publication of
statements which harms the reputation of the person in the eyes of reasonable
minded persons of the society.
There are certain defences to defamation namely:
1. Truth or justification
Truth acts as an absolute defence, even if the statement is not entirely, but
substantially true. Since, a claim of
defamation does not depend on intent, the defendant cannot rely on the fact that
s/he was just passing on rumours
or hearsay, or that s/he honestly believed the statement to be true.
2. Fair and bona fide comment
A fair and bona fide comment made in public interest will not be considered
defamatory. This is because
some amount of legitimate criticism in such matters is justifiable. The loss
occurred is damnum sine injuria, and
therefore, non-actionable. The comment must satisfy the objective test; could any
man honestly express that
opinion on the proved facts; even though the comment satisfies the objective test
the defence can be defeated if
the plaintiff proves that the defendant was actuated by express malice.
2. Privileges
The defence of Privilege is given to a person who holds some special status vis-à-
vis the circumstances. This
privilege can be further classified into Absolute Privilege and Qualified
Privilege. As the name suggests, the
former is a complete bar against an action of defamation even if the statement is
defamatory (e.g. comments
made during Parliamentary or Judicial Proceedings, or any other case where the
entity in question holds this
privilege).
In case of Qualified Privilege, an action will be successful if the comments were
made maliciously. Such
Privilege arises in case of:
1. Legal, social or moral duty (owed towards one’s kin or employers)
2. For self-protection
3. For protection of common interest
4. For protection of public good
But, this privilege is lost when the defendant goes beyond the limits of interest
which the party/parties hold(s) in
the matter or abuses the said privilege (say, by making a malicious comment and
then using the privilege as
defence).
(Extracted with requisite revisions and edits
fromhttps://lawctopus.com/clatalogue/the-tort- of-defamation/)
72. Mrs. Akhter was a big gossipmonger. Her life has always revolved around the
rumours and gossip of others. The
people in her locality have also known her this habit quite well. During a kitty
party, she told few of her friends
that their neighbour Mrs. Rama was having an affair with some colleague of hers and
that’s why her relationship
with her husband was also at a turmoil. However, nobody took her words seriously
and took her words for a
grain’s worth. When Mrs. Rama got to know about this rumour she brought a claim
against Mrs. Akhter. Can
Mrs. Akhter be made liable for defamation?
(a) No, the statements by Mrs. Akhter are true and justified and hence she cannot
be made liable for defamation.

. Page 19 of 40
(b) No, Mrs. Akhter would not be liable for defamation as everybody knew her habit
of spreading rumours and
did not believe in her words.
(c) Yes, Mrs. Akhter made public some baseless rumours which hampered the repute of
Mrs. Rama.
(d) No, since these are rumours, by their very nature they are supposed to be far
from reality and is not to be
believed.
73. A year later, it was heard that Mrs. Rama was divorcing her husband for her
neighbour whom she has been dating
for more than a year. Which of the following can aid Mrs. Akhter‘s defence?
(a) Mrs. Akhter would be protected by the defence of truth and would not be liable
for defamation.
(b) Mrs. Akhter would be protected by the defence of fair and bonafide comment.
(c) Mrs. Akhter would not be protected by any of the defences as her statements
were defamatory so as to hamper
the reputation of Mrs. Rama.
(d) Mrs. Akhter would not be protected by any of the defences as her statements
were untrue and baseless at the
time they were made.
74. RKR was an infamous film critic who was known for his highly nitpicking of
famous actors. With the release of
much awaited film of Lakhan Khan, he released a video critiquing his work in the
film. He commented on how
the film was far from reality and quoted the logical fallacies in the film. He also
pointed out how the role of a
specially abled seems to have been mocked by Lakhan through his acting. Lakhan
instituted a suit for defamation
against RKR. Would his claim succeed?
(a) No, RKR’s comment would be covered under the defence of truth and
justification.
(b) No, RKR’s comment would be covered under the defence of fair and bonafide
comment.
(c) Yes, since the comments be RKR are based out of malice, he cannot avail the
defences.
(d) Yes, RKR has to discharge the burden that the comments made by him are based on
true facts.
75. In the Monsoon Session of the parliament, there was huge uproar on certain new
reforms made by the incumbent
government for the farmers which according to many were regressive in nature.
Leader of the opposition during
the session remarked ‘Our PM although boasts of his nationality as Indian is
actually a Canadian as his father
was a citizen there and he graduated from a college in Canada and that is why he
cannot relate with the feelings
of the common man and is only driven by money.’ Can a suit of defamation be brought
against him?
(a) No, he would be protected by an absolute privilege as the comment was made
during parliamentary
proceedings.
(b) Yes, since the remarks of the Leader of opposition bear no relevance to the
session, he cannot be protected
with the defence of absolute privilege.
(c) No, the suit of defamation is likely to fail on the grounds of truth or
justification as the fact of nationality is
a factual detail.
(d) Yes, since the words of the Leader of Opposition hamper the repute of the PM,
he will be liable for
defamation.
76. The leader of Opposition didn’t curtail his comments in the session only but
rather remarked during their political
party nation wide get together, how the incumbent PM has not contributed anything
towards the economy and
has rather been investing in mafias in order to compel others to adhere to his
ideology. He also made certain
unsavoury remarks against him. When the claim of defamation was brought against
him, he claimed the defence
of absolute privilege as he is the Member of Opposition and is entitled to similar
benefits in and out of the
parliament. Is his defence likely to succeed?
(a) Yes, since the remarks made by the leader of opposition are merely the
extension of his speech in the
parliament, he will be protected absolutely.
(b) Yes, the leader of opposition will be protected by virtue of his position as a
leader of opposition and cannot
be made liable for defamation.
(c) No, it is the entity i.e. the parliament that is covered under the defence of
absolute privilege and outside the
entity, remarks made in personal capacity would not be covered by the defence.

. Page 20 of 40
(d) No, since his remarks were baseless merely to malign the image of the hon’ble
figure like PM, he would be
liable for defamation.
77. Ms. Anupama was a reporter in a nation-wide magazine “The Update”. In one such
edition, in the editorial she
wrote an unsavoury article about the MLA of their constituency. With the elections
around the corner, the MLA
filed a suit for defamation against Ms. Anupama. When the matter reached the court,
Anupama pleaded that her
words were true, however, she was not able to back her claims with proofs so as to
provide a proper justification
and stated that she believed in good faith that the remarks in the article were
true. Is the claim of MLA likely to
succeed?
(a) No, Anupama would be protected by the defence of truth or justification and
therefore would not be liable
for defamation.
(b) No, since Anupama believed in good faith that the statements in the magazine
are true, she could not be
made liable for defamation.
(c) Yes, the mere fact that a defamatory article was published with elections round
the corner indicates the
malicious intention of Anupama and therefore she would be liable.
(d) Yes, since Anupama is unable to back her claims with proper evidences and
intention is immaterial in the
claim of defamation, MLA’s claim is likely to succeed.
Passage (Q.78-Q.82): Exactly a week after International Women's Day, the Excise
Department of
Kerala registered a case against the Manager of a bar in Kochi for deputing three
Russian women as bartenders
during its launch party. The matter reached the excise officials after a video of
the women bartending in style
took social media by storm. The Manager was primarily booked for employing women at
the bar, violating the
Kerala Foreign Liquor Rules.
Back in 2007, while dealing with an identical issue, the Supreme Court acknowledged
the growing presence of
women in all spheres. A Division Bench of Justices S.B. Sinha and Harjit Singh Bedi
proclaimed: "In the last 60
years, women in India have gained entry in all spheres of public life. They have
also been representing people
at grass root democracy. They are now employed as drivers of heavy transport
vehicles, conductors of service
carriage, pilots etc. al. Women can be seen to be occupying Class IV posts to the
post of a Chief Executive
Officer of a Multinational Company. They are now widely accepted both in police as
also army services."
Two years later, the Madras High Court in Vasantha R. v. Union of India & Ors ruled
that a provision that denies
an opportunity for women to work during night hours when they are desirous of doing
so for the betterment of
their employment prospects is violative of Articles 14, 15 and 16 of the
Constitution. It remarked, "there is no
reason or rhyme to deny them employment or livelihood, which throws more
opportunity, the potential
employment cannot be denied on the sole ground of sex when no other factor arises."
However, even in a world that has witnessed active female participation in almost
all sectors, the recent case in
Kerala bears testimony to the fact that women are still combating workplace
discrimination. Despite a surplus
of resolutions and policies accentuating the need for a fairer treatment of women,
there remain crevices the law
has not yet sealed. As conceded by the Kerala High Court in 1994, women continue to
be at a fundamental
disadvantage in a male-dominated society and this disadvantage is still visible in
all spheres of life.
Laws that prohibit women from being employees in bars undoubtedly qualify as gender
discrimination which is
expressly interdicted under Article 16(2) of the Constitution. Apart from this,
such laws go against other
Fundamental Rights guaranteed under the Constitution such as Articles 14, 15, 19
and 21. Yet the rule remained
in force until very recently.
(Source: can't Women Serve Liquor in Bars? Constitutional Analysis of Legal
Restrictions, know the law)

. Page 21 of 40
78. Under the Punjab Excise Act, 1990, which expressly prohibited employment of any
man under the age of 25
years or any woman in any part of such premises in which liquor or intoxicating
drug is consumed by the public.
Reema, a widow with a kid, was in dire need of a job. The bartender, however, is
unable to hire her since it
is prohibited. What will be the correct decision if the matter reaches SC?
(a) The legislation Punjab Excise Act, 1990 of can be termed unconstitutional.
(b) The legislation undoubtedly qualifies as unconstitutional and is expressly
interdicted under Article 16(2) of
the Constitution.
(c) Reema’s fundamental under Article 16(2) of the Constitution right has been
violated.
(d) Reema can claim legislation to be held gender discriminatory under Art 16(2)
and it is violative of Articles
14, 15, 19 and 21 of the Constitution.
79. Which of the following best describes author’s perception on bar on women’s
employment in restaurants and
bars?
I. Needless to say, all policy decisions and statutory regimes that enforce such
protective discrimination
potentially serve as double-edged swords as they can be violative of rights too.
II. This tension between the right to employment and security can only be resolved
when the Government
realizes that enacting legal provisions limiting women's employment opportunities
in its attempt to protect
them is a misguided presumption of its role as Parents Patriae.
III. The safety of women has been by far the most commonly used justification for
restricting them from making
independent choices, both in their personal and professional lives.
IV. The hospitality industry continues to shut its doors at women simply by virtue
of gender differentiation. The
laws have aggravated the scepticism and resistance towards female bartenders who
were gradually becoming
the changing face of the industry.
(a) All of the above (b) III and IV
(c) I (d) I & III
80. Assertion: What is troubling is the fact that despite decisive judicial
precedents from the Supreme Court as well
as High Courts giving the green light for women to work as bartenders, there are
still legal hindrances attempting
to prevent their ascendancy in the sphere of bartending.
Reason: These prohibiting Rules are violative of the basic fundamental rights
guaranteed under the Constitution.
(a) Both A and R are true but R is not correct explanation of A.
(b) Both A and R are true and R is correct explanation of A.
(c) A is true but R is false.
(d) A is false but R is true.
81. Chaitanya a man, has been denied an opportunity of employment to perform and be
a part of orchestras and
bands in licensed bars “hard rock café”. The bar has its own rule that while it
allows male members as their
customers, but it only allows women to play and perform in orchestras and bands
performances. Decide
(a) Chaitanya cannot claim violation of article 14, 15 and 16 of the Constitution.
(b) Chaitanya claims of discrimination will not be entertained by the courts.
(c) Chaitanya was only barred from performing and not from entering the bar due to
the bar's own rules.
(d) Chaitanya’s claim of discrimination will succeed as it is based on denial of
employment on grounds of sex.
82. Female dancers are no longer permitted to perform in Maharashtra bars that
serve alcoholic beverages. The
Bombay High Court issued an order stating that only a certain number of male and
female dancers are permitted
each club or hotel selling alcohol owing to the pandemic. Is HC's ruling
discriminatory since it imposes a cap on
the number of women to perform in licensed bars, according to your interpretation
of the passage?
(a) No, as cap on the number of women to perform in licensed bars does not amount
to discrimination.
(b) No, it is not discriminatory as it does not put a blanket ban.
(c) Women continue to be at a fundamental disadvantage in a male-dominated society
and this will prove to be
a positive step towards gender neutrality.
(d) It is discriminatory as many women are denied work opportunities.

. Page 22 of 40
Passage (Q.83-Q.88): Section 14 of the ICA defines the term “free consent” and
states that any consent which
has been given by a party pursuant to coercion, undue influence, fraud,
misrepresentation or mistake shall not
be treated as free consent.
These elements whose existence make a consent to be treated as not ‘free’ can be
understood as follows:
• Coercion – In simple words, coercion means compelling a person to do something by
using force or giving
threats. Section 15 of ICA states that coercion means:
1. Committing or threatening to commit any act forbidden by the Indian Penal Code
or
2. Illegally detaining or threatening to detain any property to the prejudice of
any person with the intention of
causing him/her to enter into an agreement.
The Indian Penal Code defines offences punishable in India so if any person obtains
the consent of a party to an
agreement by doing/threatening to do commit any offence punishable under the Indian
Penal Code, the consent
shall be said to have been obtained by coercion.
The contract can be termed as voidable if there is any minute chance of the
existence of coercion and that it is
completely dependent on the disgruntled party. When coercion is performed by the
state, the state has the
complete right of maintaining peace even it has to reasonably coerce its subjects
in doing what is required for
maintaining essential decorum. It also stops private individuals from committing an
offence which they might
commit and damage the authenticity of the contract.
(Extracted with requisite revisions and edits from
https://lawctopus.com/clatalogue/legal-reasoning-free#consent-for-clat-2022/amp/)
83. Sugarcane (Production and Distribution) Regulations, 2021 were enacted by the
State Government of Uttar
Pradesh. According to the regulations, the State Government is free to either make
or to not make an offer to the
producer of the canes and if such an offer is made by the State Government then the
cane producer will be bound
to accept such an offer at standard prices in public interest. This regulation was
challenged before the Court by
one such producer Mr. Kanhaiyya who was made the offer of purchase by the State
Government on the ground
that the regulation amounts to coercion on the other party under the Indian
Contracts Act and cannot be enforced
on the producer. Is he correct?
(a) Yes, the regulations would be void for arbitrariness and would not be
enforceable before any court of law.
(b) No, there is no coercion involved in the sale and purchase of the canes and
hence any such regulation can be
enforced as it would tantamount to free consent.
(c) Yes, since coercion is compelling a person to do something by using force or
giving threats, the regulation
would be equivalent to compelling the producer to sell the canes without any choice
as to the same, therefore
this would amount to coercion.
(d) No, since the regulation intends for the purchase the canes in furtherance of
public interest, the same is valid
and can be enforced against Mr. Kanhaiyya.
84. Since, Mr. Kanhaiyya was unwilling to sell his sugarcane produce to the State
Government due to their standard
pricing which according to him was lesser when compared to pricing which he might
be able to fetch if he sells
the produce otherwise on the market. Agitated by this, a representative of the
State Government, Mr. Santosh
locked the premises of the storage area for the canes and threatened that the same
would not be released unless
Mr. Kanhaiyya enters into an agreement with the State Government and at a reduced
pricing due to the delay he
had caused them. The agreement is entered between the parties. Can such an
agreement be enforceable?
(a) No, the contract is entered into by illegal detention of the property which
would amount to coercion and
hence is not enforceable.
(b) Maybe, since the passage as well as the factual matrix are silent on whether
the detaining of property would
be punishable under IPC or not, no reasonable conclusion can be made whether the
act by Mr. Santosh
amount to coercion and to its enforceability.
(c) Yes, since the law of the sovereign requires Mr. Kanhaiyya to deliver the
produce at the standard prices and
he is not entitled to obstruct the same and therefore, the agreement can be
enforced.

. Page 23 of 40
(d) No, since the terms of the agreement do not provide the other party (the
producer) an opportunity to negotiate,
the agreement cannot be enforced for lack of consent.
85. Mr. Kanhaiyya agreed to abide by the agreement and takes the advance payment as
well from Mr. Santosh. Can
he do so?
(a) No, since the agreement is entered into by coercion the same cannot be made
enforceable by Mr. Kanhaiyya.
(b) No, the agreement is voidable at the option of the government and hence only
they can decide on the
enforceability of the agreement.
(c) Yes, since Mr. Kanhaiyya had entered into the agreement with free consent with
Mr. Santosh, the same
would otherwise be also enforceable.
(d) Yes, since the agreement is voidable at the option of the affected party, Mr.
Kanhaiyya can abide by the
agreement.
86. Ramesh entered into an agreement with Chhotelal for disbursement of loan of Rs.
50,000 which has to be repaid
within 10 days from the date of such disbursement and on the failure of which 25%
rate of interest will be
charged on the unpaid amount. It was only after a month that Chhotelal returned for
the repayment of Rs. 50,000.
However, Ramesh demanded for the interest amount as well. The case had now reached
before the court where
it was the argument of Chhotelal that Ramesh had coerced him by his knowledge of
the fact that he is having an
extra-marital affair outside the marriage and Ramesh would tell Chhotlal’s wife
about the same if he tries to
negotiate the terms of their agreement. Fearing the disclosure of this truth,
Chhotelal entered into the agreement.
The disclosure was based on a true fact and the same is not punishable under IPC,
the same argument was raised
by Ramesh. Is the argument of Ramesh correct?
(a) No, the terms of the agreement are lopsided in favour of Ramesh and are
prejudicial to Chhotelal in terms of
interest, for instance.
(b) No, Ramesh had threatened to disclose the fact of Chhotelal’s extra-marital
affair to the latter’s wife and
therefore this would amount to coercion.
(c) Yes, since Chhotelal entered into the agreement fully knowing the terms of the
same, he cannot later say
after the execution that they are unenforceable.
(d) Yes, since the threat did not involve an offence punishable under IPC, the
agreement between Ramesh and
Chhotelal is valid and enforceable.
87. Shailja bought a new phone Appsung launched recently. While she was showing her
phone to her friend Lalita,
who was in awe with the phone threatened her that if she doesn’t give the phone to
her, she would infect her
phone with deadly virus (Lalita was a computer prodigy). Seeing no options and for
the moment, Shailja
delivered the phone to Lalita and later filed a claim for coercion. It was the
contention of Lalita, that since her
act is not covered under IPC, the claim of Shailja for coercion is incorrect. On
the other hand, Shailja contended
that her act is punishable under IT Act and even that would make the entire
transaction equivalent to coercion.
Is Shailja’s argument correct?
(a) Yes, since Lalita’s act is punishable under IT Act, the threat to commit any
act which is punishable under
law would amount to coercion and thus, the transaction will be voidable.
(b) Yes, as coercion means compelling a person to do something by using force or
giving threats, this situation
would also amount to coercion.
(c) No, committing or threatening to commit an act which is punishable under IPC is
only covered under
coercion, and since Lalita’s act is punishable only under IT Act, the same would
not amount to coercion.
(d) No, since Shailja had willingly given her phone to Lalita, she cannot later
approbate or reprobate at the same
time.

. Page 24 of 40
88. Lalita had always wanted to purchase the Appsung phone, however, her parents
never gave in to her demands
and request. One day, she climbed up the parapet of their terrace and told her
family that she will jump off the
parapet unless they promise to buy her the Appsung mobile. Finally, her father gave
in to her request and after a
few days when Lalita reminded her father of the promise, he refused to buy the
phone informing that her threat
of committing suicide is a criminal offence (which is a known fact) under the penal
code and if she continues to
pester him for the phone, he will file the charges against her. Does the act of
Lalita amount to coercion?
(a) No, the consent from her parents for purchasing the phone was free and devoid
of any threats from Lalita
and hence the agreement is binding on them.
(b) No, since Lalita’s act didn’t have any effect on her parents who didn’t do
anything to further the purchase of
the phone was undertaken by Lalita’s parents, the same cannot be called coercion.
(c) Yes, threat to commit suicide is punishable under IPC and hence her threat
would also amount to coercion.
(d) Yes, although Lalita’s act would amount to coercion, however owing to the fact
that her parents also
threatened her of filing criminal charges against her, who should have otherwise
come with clean hands to
the court, the contract would be a nullity due to the doctrine of clean hands.
Passage (Q.89-Q.93): The Single Judge Bench of the Patna High Court, comprising
Justice P.B. Bajanthri in
the case of Jai Prakash Mishra v. The State of Bihar and Others has initiated
contempt proceedings against The
Secretary and Joint Secretary, Law Department, Government of Bihar for disobeying
the order of the Court to
reappoint the Public Prosecutor.
The Court referred to Halsbury view on Contempt that “The Breach of an undertaking
given to the Court on the
faith of which the Court sanctions a particular course of action or inaction is
misconduct amounting to contempt.
So also is disobedience of an injunction order of Court”.
Therefore, the Court held that the Secretary and Joint Secretary, Law Department,
Government of Bihar shall be
present in the Court to face contempt petition. So long as there is an order of the
Court which requires compliance
not only by parties but even third parties were not parties to the proceeding but
have knowledge of the same they
should be liable for contempt for disobedience of such order or obstructing
execution of the same whether the
order is valid or irregular unless order is stayed by a competent Court till it is
stayed it has to be obeyed.
Any act to lower the dignity of the court in the eyes of the people should be
condemned. Thereby, to insulate the
institution from unfair criticism and prevent a fall in the judiciary’s reputation
in the public eye, the concept of
contempt of court has been instituted. Contempt of court could be largely defined
as willful disobedience to court
orders as well as interference with the administration of justice and overt threats
to judges.
(Source: https://www.latestlaws.com/case-analysis/order-of-courts-valid-or-
irregular-should-be-obeyed-if#contempt-action-is-to-be-averted-patna-high-court-
184032/)
89. A female employee filed a sexual harassment complaint against the CJI of India
under the Sexual Harassment
of Women at Workplace (Prevention, Prohibition, and Redressal) Act, 2013, which
became a major topic of
discussion as the media questioned the judiciary's very existence. They also dubbed
it the "instance that knocks
the judiciary's credibility." Is it possible for the court to submit a complaint
against the aforementioned media
company for making insulting statements against the judiciary, the torch bearer of
the justice?
(a) Yes, since the media attempted to lower the judiciary’s reputation in the eyes
of public.
(b) No, since journalism, as the fourth pillar of democracy, has exercised its
right to free speech and expression.
(c) Yes, as the news organization ridiculed and questioned the judiciary's very
existence.
(d) No, because the critique was reasonable and fair.

. Page 25 of 40
90. Many high court justices have been accused of being corrupt, unbiased, and
dependent by Justice Khanna, who
was infamous for his actions committed in a courtroom. Prime Minister Narendra Modi
has been given notice to
take severe action against his fellow judges. Even after the Supreme Court barred
him from doing any
administrative or judicial duties, he filed many suo moto cases against his fellow
judges who voted for his
removal and has been constantly reaching out to media and making extremely bad
statement about the judiciary’s
independence. Can a contempt proceeding be filed against Justice Khanna?
(a) Yes, because the contemnor's conduct was both lowering the reputation and
interfering with the court's
proceedings.
(b) No, as the contempt proceeding cannot be filed against a judge.
(c) Yes, since contempt can be brought against anybody.
(d) Yes, as any publication that attacks an individual judge or the court as a
whole, casting unwarranted and
defamatory perceptions over the character of the judges should be included within
the meaning of
scandalizing the court.
91. Mehar was ordered capital punishment in a case where she was accused of
murdering her entire family of seven
members. It was a once-in-a-lifetime case in which a court, for the first time in
the history books of capital
punishment, sentenced a woman to death. Niharika, a newbie, was assigned the Mehar
Salam case as a probono
because the authorities had yet to comply with the court's decision. Since that
time, ten years had elapsed since
the court's order was issued. Who can be held accountable for the situation?
(a) All those in charge of the authorities who were aware of the death penalty
order.
(b) All parties to the proceeding, including third parties, who were not parties to
the proceeding but were aware
of it.
(c) Only Public officials who disobeyed the court's order.
(d) If the order is stayed by a competent Court, then no one will be held liable.
92. In a similar situation to the one described above, the order of capital
punishment was postponed by the
appropriate court when fresh evidence became available. In order to serve justice
and in line with the norm of
criminal law that no accused shall be considered guilty unless proven beyond the
court's satisfaction, the court
stayed a death sentence order. Can the authorities then be held accountable for
disobeying a court order?
(a) Yes, since a competent court has imposed a stay in the case.
(b) No, since only until an order is not stayed by the court, it must be followed.
(c) Yes, it is not required to obey if it is stayed.
(d) No, it must be followed regardless of subsequent orders unless a stay order has
been issued by the same
bench.
93. On the basis of defamation and whether a genuine certificate has been issued by
the Certification authority,
Dharma Productions was issued an injunction against the exhibition of their film
"Gangu Nath” in theatres. A
pirated version of the film has been floating about on "Anygram" since it was
already released in several states
before the order. Decide
(a) This act of piracy will be termed as contempt of court as it negates court’s
injunction order.
(b) This will not amount to contempt of court’s order.
(c) This will consider as contempt as disobedience of an injunction order of Court
is also contempt.
(d) Cannot ascertain as facts lack complete information.

. Page 26 of 40
Passage (Q.94-Q.99): Information contained in a document, if replicated, can be the
subject of theft and can
result in wrongful loss, even though the original document was only temporarily
removed from its lawful custody
for the purpose of extracting the information contained therein.
The following requirements need to be established in order to make out a case of
theft under Section 378 IPC:
• Dishonest intention
• To take away movable property
• For wrongful gain
• To cause wrongful loss
Whoever, intending to take dishonestly any moveable property out of the possession
of any person without that
person’s consent, moves that property in order to such taking, is said to commit
theft. The offence of theft is
complete a soon as the property is moved in order to such taking. It is not
necessary that the person from whose
possession the property is taken is the true owner or has the real title of the
property.
(Extracted with requisite revisions and edits from ‘Temporary removal of document
for replication of content
can be the subject of Theft, Supreme Court’ at
https://www.barandbench.com/news/temporary-removal-of#document-for-replication-of-
content-amounts-to-theft-supreme-court)
94. The Hogwords School was planning on organizing a fund raiser for the
underprivileged students in the school
and in order to attract the audience they decided to organize a magic show
performed by the great magician P.K.
Sharma. Tickets were getting sold out at a rampant rate. One of the members of the
organizing committee, Ritik,
who was also a student at Hogwords school and in-charge of all tickets, decided to
hide certain show tickets so
as to reserve them for his family members who were yet to buy the tickets since
they were out of town and were
coming the next day. However, Ritik was sceptical that the tickets would last till
then decided to hide them so
that his parents could buy them the next day. On being asked by the faculty adviser
about the status of sale, he
said that all the tickets have been sold out and therefore they had to close the
sale and send other prospective
audience home. Has Ritik committed theft?
(a) Yes, the hiding of the tickets by Ritik caused wrongful loss to other
prospective audience who were deprived
of the tickets.
(b) Yes, the hiding of the tickets by Ritik indicates his dishonest intention which
caused wrongful loss to others
in order to cater to Ritik’s gains and hence he would be liable for theft.
(c) No, the tickets were already in his own possession and can therefore not be
appropriated so as to move it
from the lawful possession.
(d) No, the hiding of the tickets would not result in any wrongful gain to Ritik as
at the end he is going to sell
the tickets to his family.
95. Next day, parents of Ritik came back and bought the tickets saved by Ritik
despite having been told by Ritik
how he managed to save the tickets for them. Are the parents of Ritik liable for
theft/ cooperating with a thief?
(a) Yes, they would be liable for conspiracy to commit theft as they were aware of
how the tickets were obtained
and reserved through an improper mode and supported it.
(b) No, no offence has been committed by them as they lawfully bought the tickets
and therefore any allegations
against them are likely to fail.
(c) Yes, Ritik did not have the title over the tickets and therefore had no right
to sell the tickets after claiming
that the tickets have got over. Thus, any tickets transferred by him cannot hold
validity as it would be an
object of theft/ misappropriation.
(d) No, since the parents of Ritik came to know about the commission of the offence
after it was committed,
they cannot be made liable for any offence.

. Page 27 of 40
96. Certain critical documents of Hogwords School were kept in the restricted area
of their library including audit
reports etc. Mr. Anujay, who was the principal of a competing school secretly
tiptoed to the library and took the
documents and made copies of them and later put the documents back in their place,
taking away only the copies
of the document. Is he guilty of theft?
(a) No, since there is no loss to the Hogwords School as the original documents
were kept back in their place,
there is no theft being committed.
(b) Yes, wrongful loss has been caused to the Hogwords school by the loss of
confidential information which
was taken with a dishonest intention and without consent by Mr. Anujay, thus he is
guilty of theft.
(c) No, the dishonest intention of Mr. Anujay cannot be traced from the given
factual matrix and thus one of the
ingredients of theft remains absent.
(d) Yes, the documents containing confidential information were replicated without
anyone’s consent and
therefore Mr. Anujay would be guilty of theft.
97. On the day of the magic show, P.K. Sharma was performing his renowned magic
trick called ‘Here and there’
in which he used to make certain personal belonging of an audience member disappear
which used to magically
transport to some other place on the amphitheatre. This time he called Mr. Ratan
(Former Chief Minister of the
State) who was an audience member. Once he arrived on the stage, P.K. Sharma made
his watch disappear
without him knowing or consenting to the same. The watch was an imported watch
gifted to him during one of
the official visits to United Kingdom. Has P.K. Sharma committed theft?
(a) No, there was no dishonest intention to steal the watch which also no intention
to cause any wrongful gain
to P.K. Sharma and hence no theft has been committed by P.K. Sharma.
(b) Yes, the watch was moved without the consent of Mr. Ratan and caused wrongful
loss to him, therefore P.K.
Sharma would be guilty of theft.
(c) Yes, there was wrongful gain to P.K. Sharma as the disappearance of watch would
cause monetary gain to
him in the form of tickets sold through his popularity.
(d) No, there is an implied consent by every audience member as they bought the
tickets knowing of such
performances especially if it is a renowned trick.
98. It was finally time to bring the watch back. The much anticipated part of the
grand trick was to see as to where
the watch will appear. However, something seemed wrong. P. K. Sharma was unable to
bring the watch back.
Even after multiple attempts he failed, just to carry an apologetic face for having
lost the watch. It was not a
trickery but reality. He had lost the watch. Mr. Ratan got very angry but had no
recourse except for the legal
means. Is a case is instituted against P. K. Sharma for theft, would he be liable
for the same?
(a) Yes, the watch was moved from its rightful owner, Mr. Ratan causing wrongful
loss to him and therefore P.
K. Sharma would be guilty of theft.
(b) No, P. K. Sharma would not be liable for theft because Mr. Ratan had consented
to the giving of the watch
for a magic trick and thus consented to the risks involved as well.
(c) Yes, the fact that P. K. Sharma had performed this trick earlier warrants that
he was in a position to bring
back the watch and he avoided the bringing back of the same due to dishonest
intentions which forms an
ingredient of theft.
(d) No, there was no dishonest intention coupled with wrongful gain, two of the
ingredients for theft and hence
P. K. Sharma would not be guilty of theft.
99. After a month passed, one of the apprentice of P. K. Sharma, after seeing the
pathetic state and the mental hassle
P.K. Sharma was undergoing in the trial revealed that since he was already aware
where the watch would appear,
he showed his sleight of hand and stole the watch and returned the watch back to P.
K. Sharma so that he can in
turn return it to the rightful owner. P. K. Sharma later thought that after having
undergone so much of trouble,
he should show Mr. Ratan of what he is capable of and decided to keep the watch
with himself. Is P. K. Sharma
guilty for theft?
(a) No, since the watch was not taken out of possession with a dishonest intention,
it does not amount to theft.

. Page 28 of 40
(b) No, there is no legal obligation on P.K. Sharma to return the watch as it was
not stolen by him and the watch
has to be returned by the actual culprit.
(c) Yes, after the watch was delivered to him, he kept the watch with dishonest
intention causing wrongful loss
to Mr. Ratan and having a wrongful benefit in turn.
(d) No, the watch was not delivered to him by the true owner but by his apprentice
who himself did not title over
the watch.
Passage (Q.100-Q.105): The Kerala HC recently held that a rape survivor's sexual
history is immaterial in a
rape case and will have no bearing on the credibility of the testimony of such a
survivor. A Single Judge
bench held that a rape survivor being habituated to sexual intercourse or being "a
girl of easy virtue" would not
be grounds to absolve an accused of rape charges.
The Court categorically held that even when a survivor admits in cross-examination
that she has had sexual
relationships with another person previously, the same does not in any way affect
the credibility of her testimony.
"Even in a case where it is shown that the victim is a girl of easy virtue or a
girl habituated to sexual
intercourse, it may not be a ground to absolve the accused from the charge of rape.
Even assuming that the
victim is previously accustomed to sexual intercourse, that is not a decisive
question. On the contrary, the
question which is required to be adjudicated is, did the accused commit rape on the
victim on occasion
complained of. It is the accused who is on trial and not the victim,"
"Her evidence need not be tested with the same amount of suspicion as that of an
accomplice. The nature of
evidence required to lend assurance to the testimony of the prosecutrix must
necessarily depend on the facts
and circumstances of each case. Only if the Court finds it difficult to accept the
version of the prosecutrix on
its face value, it may search for evidence, direct or circumstantial, which would
lend assurance to her
testimony.
Section 375 of the Indian Penal Code defines rape as "sexual intercourse with a
woman against her will, without
her consent, by coercion, misrepresentation, or fraud or at a time when she has
been intoxicated or duped, or is
of unsound mental health and in any case, if she is under 18 years of age."
Source: https://www.barandbench.com/news/girl-habituated-to-sexual-intercourse-or-
being-of-easy-virtue-not#grounds-to-absolve-accused-of-rape-charges-kerala-high-
court
100. Shreya had given her consent to have intercourse with Bhatav on Tuesday when
they were on a date after having
two glasses of wine. However, after having slept immediately after the wine at the
table, Bhatav took her to her
house and left her there safely. They met again on Thursday for lunch when Bhatav
finally approached her for
intercourse assuming that he had her consent. To his dismay, she pulled away and
did not entertain his advances.
Would he be liable for committing rape in this case?
(a) No, as she had given him her consent.
(b) Yes, as she was intoxicated while having given her consent.
(c) No, as he did not have intercourse with Shreya.
(d) Yes, as he inappropriately manhandled her.
101. Shubhi and Ruben were a couple always at crossroads wanting to have kids.
Ruben wanted to have kids, while
Shubhi did not. As a result them they would get in fights all the time. One day,
frustrated due to all the waiting,
Ruben forced himself upon her in order to have kids. This resulted in an
altercation between the two while Ruben
was forcing an intercourse with her. Once he was done, she rushed to the police and
reported him. Would he be
liable for having raped her?
(a) No, as he was her husband.
(b) No, as he only wanted to have kids.
(c) Yes, as he forced her to have intercourse with him.
(d) Both a & b.

. Page 29 of 40
102. In the above case, had Shubhi been the one forcing Ruben to have kids and had
forced her upon him, would it
still amount to having committed rape?
(a) No, as she had a valid concern about having kids.
(b) No, as men cannot be raped.
(c) Both a and b.
(d) Yes, as he was violated inappropriately.
103. In the above case, had Shubhi and Ruben not been a married couple but just a
dating couple, would the Act of
Shubhi forcing herself on Ruben be considered as committing rape?
(a) Yes, as they are not a married couple, and Ruben did not consent to the same.
(b) No, as Ruben is not protected under law.
(c) No, as Ruben consented to the same while getting into a mutual relationship.
(d) Yes, as Shubhi violated his person.
104. Rinki was notoriously infamous in her friend group for having terrible
relationships over the years. One such
relationship was her recent one with a guy named Toby. To no one's disappointment,
the relationship ended
badly, with Toby having violating Rinki. On her continous struggle during the
process, he slapped her and left.
She reported the same to the police, and they charged him for rape. Is this action
of the police correct?
(a) Yes, as he succeeded in violating Rinki.
(b) Yes, as Rinki did not consent to have intercourse with Toby.
(c) No, as Rinki exhibits a pattern of bad behaviour and cannot be trusted.
(d) No, as Rinki has a clearly flawed judgment and cannot be trusted.
105. In the above-mentioned case, had Rinki had a pastrecord of having multiple
relations with multiple men and
catfishing men, would her charges and testimony be considered reliable?
(a) No, as she exhibits a clear behaviour of repeated offending.
(b) No, as she deliberately catfishes men into having intercourse with her.
(c) Yes, as she was still raped in this case.
(d) Both a & b

ECTION - D: LOGICAL REASONING


Directions (Q.106-Q.110): Read the following passage in in light of the same answer
the questions that follow.
In some instances, more than one option may be the answer to the question; in such
a case, please choose the
option that most accurately and comprehensively answers the question.
Almost all known snake species eat their prey whole, in a single and sometimes
monumental gulp. But scientists
have found an exception to this general rule in the cat-eyed water snake (Gerarda
prevostiana), a small serpent
native to mangrove swamps throughout Southeast Asia. A new study has found that
these snakes will attack and
eat crabs up to five times larger than their jaw can accommodate.
Picky eaters, they only go after freshly molted crabs in the 10-to-15-minute period
after the animals shed their
old shells, according to the study, published recently in the Biological Journal of
the Linnean Society. "They are
quite the little gourmands," says study leader Bruce Jayne, a professor of biology
at the University of Cincinnati.
Their strange feeding style begins with a bite, and a tight grasp of the crab's
main body, or carapace. They then
make a loop with their body pulling the prey through it repeatedly until the crab
is deformed or breaks apart,
Jayne explains. Next, they eat the individual pieces, or if the carapace is still
too big, sometimes they just pull
off the legs and eat them one by one, Jayne says.
After capturing cat-eyed water snakes in Malaysia, and later Singapore, Jayne and
colleagues took some into the
lab and placed them in mud-bottom tanks where the creatures formed individual
tunnels and stayed put. "They
are very shy snakes." Jayne says. When they initially put crabs on top of the mud,
the animals showed no interest.
Why weren't the snakes feeding? The colleagues remained stumped until Jayne came
upon a clue from an
unrelated animal, the queen snake (Regina septemvittata). This North American
creature prefers to prey upon
freshly molted crayfish, snacking on the animals when they are softer and easier to
swallow. If cat-eyed water
snakes were similar, it would explain why some crabs recovered from inside the
stomach of snakes they captured
in the wild appeared squishy, probably from being newly molted rather than being
partially digested.
With the help of Peter Ng, a crab specialist at the University of Singapore, the
researchers acquired a large
collection of the prey animals, so that they would have some freshly molted
specimens. It worked. When they
placed freshly molted crabs in the tanks, the snakes would "zoom right out of the
burrow, and instead of moving
slowly, their heads would thrash back and forth." and then they'd attack. Jayne
says.
The researchers also examined the eating habits of two related species, the white-
bellied mangrove snake
(Fordonia leucobalia) and Cantor's water snake (Cantoria violacea), which
respectively eat hard-shelled crabs
and snapping shrimp.
They found that the white-bellied mangrove snakes also have a very strange
attacking method, striking at crabs
with a closed mouth and pinning them down into the mud. They then wrap their body
around the crab and bite
it, sometimes pulling off individual legs, before downing the whole thing. Such
unusual behaviors show how
snakes have evolved creative ways to hunt crustaceans, a most unusual prey item
with a hard outer body and the
ability to fight back.
106. The central idea of this passage is
(a) Why some crabs in snakes' stomachs appeared to have a strange texture.
(b) Why captive cat eyed snakes were not interested in eating hard shell crabs.
(c) How molting patterns of certain hard shell crustaceans affect snakes.
(d) How some snakes have evolved unusual attacking behaviors to consume difficult
prey.

. Page 31 of 40
107. Scientists' observation of the queen snake quashed which of their earlier
assumptions?
(a) That the queen snake and cat eyed water snake were completely unrelated
animals.
(b) That the crabs from the cat eyed water snake's stomach were partially digested.
(c) That the queen snake ate crabs like the cat eyed water snake, not crayfish.
(d) That the queen snake's diet was not restricted to freshly molted prey.
108. All of the following statements are true, EXCEPT
(a) Cat eyed water snakes are picky eaters as they sometimes eat a crab's legs one
by one.
(b) Cat eyed water snakes are not very social animals and prefer to be alone.
(c) Cat eyed snakes' normal mode of movement when not feeding, tends to be slow.
(d) Crustaceans prey have the ability to fight back when attacked.
109. Each of these statements describes the strange feeding style of cat eyed water
snakes, EXCEPT
(a) Making a loop with their body in order to pull the prey through it and deform
or break it.
(b) Picking on crabs that have freshly molted in the past 10-15 minutes.
(c) Partially digesting the crab beforehand so that it is softer and easier to
consume.
(d) Pulling off the crab's legs and eating them one by one.
110. Why has the author mentioned the examples of the white-bellied mangrove snake
and Cantor's water snake?
(a) To show that there are other snakes who have developed unusual attacking
methods.
(b) To demonstrate that these species related to the cat eyed water snake had
almost the same feeding habits.
(c) To show the study carried out on other snake species who consume only freshly
molted crabs.
(d) To show that cat eyed water snakes are an exception to the general rule of
snakes eating prey whole.
Directions (Q.111-Q.115): read the following passage in in light of the same answer
the questions that follow.
Last year this time, cases were below 5,000 a day, encouraging several States and
the Centre to claim that the
pandemic was over, though within a matter of weeks there was a resurgence fuelled
by the Delta variant which
birthed a summer of catastrophe. There is, however, a crucial distinction between
then and now in that over 75%
of those over 15 years are now fully vaccinated in India. A small and growing
number of those over 60 have had
the third dose. India is fortunate in that it does not have to battle vaccine
hesitancy in a large measure. The initial
scepticism regarding the vaccines not having passed the typical stages of vaccine
approval saw a certain degree
of hesitation, but very soon it emerged, in April and May last year, that India’s
main problem was an
insufficient number of vaccines. Though India today has administered nearly 178
crore vaccine doses and has
several indigenously developed vaccines that have been approved in emergency mode
by authorities, there are
still serious questions on supply. Currently, vaccine demand is low and the
vaccination drive is in ‘mop up
mode’ and administering second doses. But were the pandemic situation to suddenly
turn for a fourth wave
to take shape, there would be a spike in demand for vaccinations for children,
particularly those below 15, as
well as booster doses for adults. The experience of Covaxin’s manufacturer being
unable to ramp up vaccinations
in time during the crisis months ought to be a persistent reminder to other
biotechnology companies that having
vaccines is very different from being ready with a seamless supply chain. The
Indian government has still
not made public a timeline for when vaccines from Biological E, Gennova and Zydus
Cadila will be practically
available for mass use. Though the world is occupied with a different crisis, India
must not let its guard down
and should insist on companies being ready with a measurable timeline.
[Source: The Hindu]
111. What is the central idea of the passage?
(a) Vaccination plays a major role in combating pandemic; hence a robust supply
chain is must.
(b) Vaccine hesitancy is one of the reasons for decrease in demand for the vaccine.
(c) Supply of Indigenous vaccines will prevent emergence of Fourth wave.
(d) Children are more susceptible to pandemic; hence, to cater their needs, India
must bolster supply
chain.

. Page 32 of 40
112. Which of the following, if true, most weakens the author’s argument with
reference to the statement “The initial
scepticism regarding the vaccines not having passed the typical stages of vaccine
approval saw a certain degree
of hesitation, but very soon it emerged, in April and May last year, that India’s
main problem was an insufficient
number of vaccines”?
(a) Due to lack of adequate transportation and logistical facilities (b) vaccines
were not able to reach to
the public healthcare centres.
(b) It was found by DGCI that around half the vaccines administered were (d) either
wasted or went
expiry.
(c) It was shown that about 92% of people who died due to pandemic, their cause of
death was not getting
vaccinated.
(d) The demand for vaccine in the category of children below the age of 15 years is
at all-time high.
Owing to this fact, they are not administered the doses.
113. Which of the following can be inferred from the above passage?
(a) Demand for the vaccines for the age group above 15 was well catered to during
initial waves of the pandemic.
(b) India exported immense number of vaccines which led to default in its domestic
supply.
(c) Vaccines continues to act as an elixir during third wave of the pandemic.
(d) Vaccines have prevented people from infection.
114. Which of the following, if true, strengthens the last two lines of the
passage?
(a) The approval of vaccination in a limited time frame may result speculations on
efficacy of the vaccines.
(b) The pandemic waves will continue as each wave leads to a new variant that
reaches its peak in a certain
frame of time.
(c) Since, majority of the population has been vaccinated, it is unlikely that a
fourth wave will emerge.
(d) Indigenous vaccines require a shorter time frame for production and supply than
imported vaccines.
115. What conclusion can be drawn from the passage?
(a) India is not ready for the fourth wave.
(b) There was an adequate supply of vaccines during the Delta variant.
(c) The need for vaccines will continue in the near future.
(d) India has always been weak in the execution during emergencies.
Directions (Q.116-Q.120): Read the following passage in in light of the same answer
the questions that follow.
Last week, the Union government announced that merchandise exports from India have
crossed $400 billion in
the current financial year. In fact, for the full financial year, which ends on
March 31, this number is expected
to touch $410 billion. Two things stand out about this performance. One, in value
terms, this level of goods
exports is far higher than the previous record of $330 billion, which was achieved
in 2018-19. Two, this
achievement is even more remarkable when one takes into account that the current
financial year witnessed two
waves of the Covid pandemic, especially the vicious second wave at the start of the
financial year from April to
July. However, for this growth to sustain, it is important to understand the true
nature of this growth and what
caused it.
The government has claimed that “there was a detailed strategy in place, including
specific targets set — country#wise, product-wise & EPC-wise (Export Promotion
Council), monitoring and course correction behind the
achievement of the export target”. While these efforts need to be commended, it is
also important to recognise
that India’s exports are also a function of several global factors. Two factors
stand out in particular. One is the
overall rate of economic growth in the world. This matters because a strong
economic recovery implies there
will be demand for Indian goods in other countries. Two, the amount of easy money
available in the global
economy. The ongoing financial year — 2021-22 — provided a conducive environment on
both counts. Many
western economies recovered from the Covid dip quite fast, thanks to massive
government spending in those

. Page 33 of 40
countries. Moreover, in response to the pandemic, most central banks in the
developed world also expanded their
balance sheets and provided cheap credit. Both factors had a salutary effect on
India’s exports.
But close to the start of a new financial year, most central banks in the West are
winding down their balance
sheets and raising interest rates in a bid to contain inflation levels that are at
multi-decade highs. Russia’s
invasion of Ukraine has further exacerbated inflationary concerns. Overall, the
outlook on economic growth,
too, has taken a beating. Globalisation, which was under threat even before the
pandemic, is likely to come under
renewed pressure with more and more countries wanting to become self-sufficient or
at least reduce dependence
on foreign goods. (_____) Lastly, the government must not forget that higher prices
had a huge role to play in
India reaching the $400 billion target. As a percentage of the GDP, merchandise
exports are far from where they
were a decade ago.
116. It can be said that the difference in increase in the merchandise exports in
percentage from 2018-2019 till the
latest time (March 31st) mentioned in the article would be
(a) More than twenty four percent.
(b) Around twenty four percent.
(c) Twenty four percent.
(d) More than twenty five percent.
117. Which of the following can be inferred from the passage?
(a) Global policies that helped India hit $400 billion in exports are changing.
(b) The merchandise exports from India that have crossed $400 billion in the
current financial year present a
dismal growth rate in the export sector.
(c) Russia’s invasion of Ukraine has further exacerbated inflationary concerns.
(d) The current levels in India’s exports are due to the concerted and sole efforts
by the government’s detailed
strategy in place, including specific targets set — country-wise, product-wise &
EPC-wise monitoring and
course correction.
118. One is the overall rate of economic growth in the world. This matters because
a strong economic recovery implies
there will be demand for Indian goods in other countries.’ What is the underlying
assumption behind the passage?
(a) Global economic growth is inversely proportional to Indian economic growth.
(b) Global economic growth is directly proportional to Indian economic growth.
(c) Global economic growth is exclusive of Indian economic growth.
(d) Global and Indian economic growth are mutually dependent.
119. Which of the following will logically follow as an argument filling the blank
in the passage?
(a) The global policy landscape has become less challenging for India and will
least demand renewed efforts at
the domestic level to raise the competitiveness of the exporters.
(b) The global policy landscape has become more challenging for India and will
demand renewed efforts at the
domestic level to raise the competitiveness of the exporters.
(c) The global policy landscape has become extremely challenging for India and will
demand renewed efforts
at the domestic level to raise the competitiveness of the exporters.
(d) The global policy landscape remains stagnant for India as there are no renewed
efforts at the domestic level
to raise the competitiveness of the exporters.
120. ‘As a percentage of the GDP, merchandise exports are far from where they were
a decade ago.’ Which of the
following conclusion can be drawn from the given argument?
(a) The merchandise exports have contributed more to the GDP than a decade ago.
(b) The merchandise exports have contributed less to the GDP than a decade ago.
(c) The contribution of merchandise exports to the current GDP is equal to the
contribution of merchandise
exports to the GDP of the previous decade.
(d) The merchandise exports have contributed significantly more to the GDP than a
decade ago.

. Page 34 of 40
Directions (Q.121-Q.125): read the following passage and in the light of same,
answer the questions that follow
Maharashtra’s Minister of Minority Affairs and Skill Development and Nationalist
Congress Party (NCP) leader
Nawab Malik is the second Minister in the Maha Vikas Aghadi (MVA) alliance to be
arrested by the Enforcement
Directorate (ED). The case against Mr. Malik pertains to a transaction in 1999 in
which a property was sold to
one of the companies belonging to him for a price ostensibly much lower than its
actual worth. The Additional
Solicitor General alleged that there was enough evidence to make a case under the
Prevention of Money
Laundering Act. The ED has also alleged that the property sale was done by an
individual, Salim Patel, an
associate for Dawood Ibrahim’s sister Haseena Parkar, since deceased. Mr. Malik’s
advocate has argued that the
power of attorney for the property sale was made in 1999, much before the
Prevention of Money Laundering
Act ,2002 (PMLA) came into force and that there is no retrospective action in
criminal law. It should be noted
that Mr. Malik has been vocal in his criticism of agencies such as the Narcotics
Control Bureau and the way they
have conducted investigations against individuals in Mumbai. Unlike the case with
former Home Minister Anil
Deshmukh, who resigned after his arrest last year, the NCP and its chief Sharad
Pawar have ruled out Mr. Malik’s
resignation alleging that this is an attempt by the BJP to destabilise the MVA
government. It is for the courts to
find if there is merit in the ED’s case against Mr. Malik. But there have been
several instances where the PMLA
has been used by the government at the Centre as a weapon to target politicians
from the opposition, their
relatives, and activists recently. The PMLA was enacted in 2002 in line with
India’s global commitment to
combat money laundering, particularly related to crimes involving drugs and
narcotics. But as petitions filed in
the Supreme Court against the draconian use of the Act have pointed out, the ED
conducted 1,700 raids and
investigations in 1,569 cases in the last decade (2011-20) but could obtain
convictions in only nine cases.
Meanwhile, as the petitioners in the case against the PMLA’s use have argued in the
Supreme Court, taking
recourse to the Act should not be “arbitrary, vague and fanciful”.
[The Hindu]
121. Which of the following can be construed as the central idea of the passage?
(a) PMLA has lost its relevance and core purpose.
(b) Statutes which are used as a deterrent are being covered under the whims of
politics.
(c) Ruling party has been consistently trying to silence the vocals by fanciful use
of legislation.
(d) PMLA has effectively curbed the corruption, thereby realising its true purpose.
122. Which of the following can be inferred from the above passage?
(a) The agency is using the offences scheduled in the PMLA in an overbroad sense
without limiting itself to the
chief purpose of the Act.
(b) There has been increase in number or arrest and raids carried out by agency and
least convictions since the
enactment of Act.
(c) Former Home Minister Anil Deshmukh, who was arrested last year, was critical of
ruling government and
central government agencies.
(d) Courts from time to time, particularly the Supreme court, have stepped in and
voiced concerns regarding
blatant misuse of such Acts.
123. Which of the following can be inferred from the following passage?
(a) India consents to various international conventions to combat menace of money
laundering.
(b) The courts have time and again failed to protect the autonomy of individuals by
not preventing them from
clutches of this horrendous act.
(c) There have been various instances of Supreme Court making vague interpretation
with respect to several
other statutes.
(d) The ruling government has consistently attempted to destabilise opposition
parties in other states through
this draconian Act.

. Page 35 of 40
124. Which of the following course of action, would the author most unlikely to
agree?
(a) There exists a political rumble between Ruling party at centre and various
coalition party in other states,
which can be tackled through deliberations.
(b) It is need of the hour for the courts to step in to balance intent of
legislature with Rights of individuals.
(c) It is essential to regulate the power of agencies like ED to prevent political
victimisation of innocents.
(d) State shall enact their respective laws with respect to money laundering,
corruption so as to avoid further
tussle between the centre and the states.
125. Which of the following is the assumption behind the passage?
(a) The Enforcement Directorate in the Department of Revenue, the Government of
India is responsible for
investigating the offences of money laundering under the PMLA.
(b) The Enforcement Directorate under the Government of India is responsible for
investigating the offences of
money laundering under the PMLA.
(c) The Enforcement Directorate coordinates and strengthens the efforts of the
national and international
intelligence.
(d) The scheduled offences are separately investigated by the Enforcement
Directorate mentioned under several
acts, for example, the local police, CBI, Customs departments or any other
investigative agency.
Directions (Q.126-Q.130): Read the following passage in in light of the same answer
the questions that follow.
If lack of data was an impediment to roll out action plans against antimicrobial
resistance (AMR), now that
excuse has been yanked off. The recent publication of The Lancet’s global burden of
bacterial antimicrobial
resistance — an elaborate and studied estimate validated by using counterfactual
analysis for the first time —
comes at a time when the world seems to have lost steam to mount a robust AMR
policy. But the report makes
it clear that no slacking can be allowed on this front any longer; it estimated
that 4.95 million deaths were
associated with bacterial AMR in 2019 alone. It also identified the pathogens and
pathogen-drug combinations
that cause such resistance. Bacterial AMR occurs when the drugs used to treat
infections become less effective,
as a result of the pathogens becoming resistant to the drugs. This happens due to
indiscriminate use of antibiotics,
availability of antibiotics over the counter, poor hygiene and sanitation, and poor
infection control practices in
hospitals. While data on the exact number of deaths might not have been available,
there was no doubt about the
alarming nature of associated mortality and morbidity. [1] And yet, few nations
have a policy to counter this
pernicious problem.
In 2008, when the NDM1 enzyme that renders bacteria resistant to a range of
antibiotics was traced back to
India, it served as an urgent call for action. India released its own AMR action
plan in 2017, and announced a
task force for implementation. By 2019, Kerala and Madhya Pradesh had rolled out
State action plans. Since
then, little progress has ensued. The Chennai Declaration, a consortium of doctors
and health-care institutions
against AMR, was also formed in 2012 to draw up a road map. The ban on Colistin in
the poultry, aqua farming
and animal feeds supplements sectors, which India enforced from July 2019, was
considered a strong strike in
countering the AMR challenge. [2] No doubt AMR offers humanity one of the most
complex challenges that
it has faced; but the recognition that solutions are not only in the realm of
science is necessary. [3] Scientific
publications have established the correlation between AMR and poor hygiene, lax
administrative
governance and poor ratio of public-private expenditure. While the scientific
community looks for solutions
in its ken, governments must raise the standard of living for citizens, provide
them accessible and affordable
quality health care, besides regulating the sale and use of antibiotics. Not doing
so in studied haste will only land
up eroding the significant health-care gains India has proactively, and painfully
at times, secured over the years.

. Page 36 of 40
126. Which among the following is the best representation of the main idea of the
passage?
(a) Lack of data has been a significant impediment to expediting medical
facilities.
(b) India’s health-care gains remain at risk of being eroded, thanks to
antibiotics.
(c) India should look forward to making the local medical practices mainstream by
promoting the quality health
care systems.
(d) India must combat antimicrobial resistance by regulating the use of
antibiotics, and raise the quality
healthcare.
127. Out of the following, which piece of evidence lends support to the author’s
arguments?
(a) 18 of India’s 28 States are still framing their Antimicrobial Resistance (AMR)
action plans.
(b) According to a research study, the difference between the estimated deaths and
actual deaths due to bacterial
AMR is immense.
(c) Both a and b are correct.
(d) Neither a nor b is correct.
128. If the information in the passage is true, then which of the following is
strongly supported?
(a) Taking a casual approach against antimicrobial resistance (AMR) is likely to
cost the world dearly.
(b) India’s health-care over the years has come without any significant cost.
(c) A crisis on a different front disabled the states from formulating action plans
against the antimicrobial
resistance (AMR).
(d) A robust approach against bacterial AMR would ensure that it is eliminated in
the long run.
129. What is the best representation of the relationship between the two boldface
statements ([2] and [3])?
(a) Statement [2] forms the premise of the claim made in statement [3].
(b) Statement [3] forms the premise of the claim made in statement [2].
(c) Statement [2] and [3] are claims made to support the main argument.
(d) Statement [2] and [3] are counter arguments of each other.
130. Which of the following can be inferred from the first few lines of the
passage?
(a) Lack of data can no longer be a justification for inaction against
antimicrobial resistance (AMR).
(b) Lack of data had been the cause for delayed action against antimicrobial
resistance (AMR).
(c) Sufficient data was available with the world to act against the against
antimicrobial resistance (AMR), and
the government’s inaction cannot be attributed to it.
(d) The defence of lack of data to roll out action plans against antimicrobial
resistance (AMR) has been suddenly
withdrawn.
Directions (Q.131-Q.135): Read the following passage in in light of the same answer
the questions that follow.
Sonia Gandhi’s charge against social media giants of bending their own content
norms to favour politicians of
the ruling party, and aiding in spreading disinformation, cannot be brushed aside
as the lament of a party chief
unable to come to terms with the rapid downward slide. The Congress’ worries go
much beyond anything that
an independent social media, free of bias, can mend. But any attempt by a foreign
business to meddle in electoral
politics in India, as being alleged, is an issue of grave concern. The serious
repercussions of such an eventuality,
and the template it sets for abuse and misuse, call for scrutiny and debate in
Parliament and the high offices of
the Election Commission alike.
Media reports claiming that Facebook offered the BJP cheaper deals by bending hate
speech rules for poll
advertisements raise several uncomfortable questions. As also the inference to a
toxic ecosystem of proxy
advertisers posing as news media, bypassing election laws and suppressing the voice
of all those ‘speaking
up against the government’. Notwithstanding the customary denial, Meta platforms
need to come clean on
pointed allegations of systematic interference and not providing a level playing
field to political parties. Since

. Page 37 of 40
much of the bluster on social media platforms tethers on religious fault lines,
allowing it all to go can only be
seen as a breach of trust and fomenting social disharmony.
The influence of social media has set new rules of the game for the political
class, and the party structures have
responded accordingly. The Congress cannot absolve itself of its complacency,
functional flaws and the
shortcomings within that get reflected in the social media space, especially when
the opponents only seem to be
perfecting its mastery. Even on such matters of national importance that should
raise the hackles across party
lines, what is glaringly missing is the collective, coherent voice of the
Opposition. Its absence is no less
damaging.
(SOURCE- https://www.tribuneindia.com/news/editorials/social-media-scrutiny-37867 )
131. Which of the following reflects the central idea of the passage?
(a) It is time to take strong actions against those accused for misuse of social
media.
(b) As the matters pertains to national security, the members of the ruling party
who were involved in such deals
should be punished.
(c) The biased, misuse of social media for spreading misinformation by political
parties does not bode well for
the political ecosystem.
(d) The opposition party making allegations against the ruling parties of misuse of
social media cannot be
absolved of its own flaws and shortcomings.
132. The author is most likely to agree with which of the following courses of
action?
(a) Persons involved in spreading misinformation on social media should be coerced
to submit their resignations.
(b) There should be a proper discussion and debate on these allegations.
(c) As the allegations are politically motivated, they should not be taken
seriously.
(d) Social media sites should process the information as a filtering procedure
before sanctioning such data.
133. Which of the following, if true, weakens the author’s argument?
(a) The ruling party has denied all of these allegations as baseless.
(b) By and large, the social media content is perceived by the public as lacking
any merit or value.
(c) The allegations made are serious and can be cleared only after a proper
investigation.
(d) The opposition has itself in the past been involved in allegations of media
tampering.
134. What purpose the boldfaced statement in the passage serves?
(a) It is a premise supporting the stance of social media platforms over the issue.
(b) It provides background information or context regarding the issue.
(c) It is an argument of the author regarding why social media platform should come
up with reply to pointed
allegations.
(d) It's a premise supporting the argument regarding responsibility of META.
135. Author is most likely to agree with which of the following statements?
(a) Attempt by a foreign business to meddle in electoral politics in India is not
entirely a new situation.
(b) The allegations lack any basis and should not be taken seriously.
(c) Other opposition parties are in favour of the ruling party over the issue as
they have not come in support of
the opposition party.
(d) The social media allegation concerns national security and requires debate
regarding it.

MOCK 40
SECTION-A : ENGLISH LANGUAGE
Directions (Q.1-Q.30): Read the following passage carefully and answer the
questions that follow.
Passage (Q.1-Q.5): Human beings have an inherent and limitless desire for all-round
victory in all spheres of
life. In any sphere of life, important or unimportant, it is unthinkable that
people will be content to lead
insignificant lives.
The idea of moving along a set path, eternally subservient to nature, has always
been repugnant to human
psychology. Sometimes, however, people are compelled to submit to the laws of
nature due to extreme
circumstantial pressure, and this is due to their lack of sufficient intellect and
stamina. All human sadhana is
merely an effort to overcome internal weaknesses.
Not all human energy, however, is exhausted in the effort to remove psychic
imperfections. As people have to
maintain their existence in this seemingly adverse physical world, naturally they
must make constant endeavours
in the physical sphere. In order to conquer static Prakriti, the Supreme Operative
Principle, bhautik vijnana,
physical science, evolved. In Sanskrit, vijnana stands for Brahmn vijnana,
intuitional science, or adhyatma
vijnana, the science of spirituality.
Physical science plays its part by discovering new formulas to assist in such
invention. Had physical clash not
existed, the creation and evolution of the human mind would never have been
possible.
1. What does Vijnana mean in English?
(a) Intuitional (b) Science (c) Spirituality (d) Efforts
2. ‘The idea of moving along a set path, eternally subservient to nature, has
always been repugnant to human
psychology.’ What can be inferred from the given lines?
(a) Humans likes to challenge nature.
(b) Humans are averse to the idea of towing the lines.
(c) Humans like to assert rather than accept natural course.
(d) Human alter the natural course to assert their superiority.
3. ‘Had physical clash not existed, the creation and evolution of the human mind
would never have been possible.’
What can be understood by ‘physical clash’?
(a) A physical clash is the coming together of the human mind against the physical
world.
(b) A physical clash is a confrontation of the physical science with the physical
world.
(c) A physical clash is an encounter of the human psyche as against human sadhana.
(d) A physical clash is the creation of an invention due to the clash between human
mind and the physical world.
4. ‘All human sadhana is merely an effort to overcome internal weaknesses.’ Which
of the following is the correct
representation of ‘sadhana’ as mentioned in the context of the passage?
(a) Devotion (b) Energy (c) Sweat (d) Nature
5. Which of the following reflects the primary purpose of the author?
(a) The author propounds that the physical science is in direct clash with the
physical world as a result of human
assertiveness.
(b) The author advocates that human energy is directed towards changing the static
laws of the nature for which
they create many inventions.
(c) The author disregards the failings of the mind in overcoming internal
weaknesses.
(d) The author proposes that the evolution of human mind is a result of their
constant endeavours in the physical
sphere.

. Page 3 of 40
Passage (Q.6-Q.10): Although no one thinks that Plato simply recorded the actual
words or speeches of Socrates
verbatim, the argument has been made that there is nothing in the speeches Socrates
makes in the Apology that
he could have not uttered at the historical trial. At any rate, it is fairly common
for scholars to treat Plato’s
Apology as the most reliable of the ancient sources on the historical Socrates. The
other early dialogues are
certainly Plato’s own creations. But as we have said, most scholars treat these as
representing more or less
accurately the philosophy and behaviour of the historical Socrates—even if they do
not provide literal historical
records of actual Socratic conversations. Some of the early dialogues include
anachronisms that prove their
historical inaccuracy.
It is possible, of course, that the dialogues are all wholly Plato’s inventions and
have nothing at all to do with
the historical Socrates. Contemporary scholars generally endorse one of the
following four views about the
dialogues and their representation of Socrates:
The Unitarian View:
This view, more popular early in the 20th Century than it is now, holds that there
is but a single philosophy to
be found in all of Plato’s works (of any period, if such periods can even be
identified reliably). There is no
reason, according to the Unitarian scholar, ever to talk about “Socratic
philosophy” (at least from anything to be
found in Plato—everything in Plato’s dialogues is Platonic philosophy, according to
the Unitarian). One recent
version of this view has been argued by Charles H. Kahn (1996). Most later, but
still ancient, interpretations of
Plato were essentially Unitarian in their approach. Aristotle, however, was a
notable exception.
The Literary Atomist View:
We call this approach the “literary atomist view,” because those who propose this
view treat each dialogue as a
complete literary whole, whose proper interpretation must be achieved without
reference to any of Plato’s other
works. Those who endorse this view reject completely any relevance or validity of
sorting or grouping the
dialogues into groups, on the ground that any such sorting is of no value to the
proper interpretation of any given
dialogue. In this view, too, there is no reason to make any distinction between
“Socratic philosophy” and
“Platonic philosophy.” According to the literary atomist, all philosophy to be
found in the works of Plato should
be attributed only to Plato.
The Developmentalist View:
According to this view, the most widely held of all of the interpretative
approaches, the differences between the
early and later dialogues represent developments in Plato’s own philosophical and
literary career. These may or
may not be related to his attempting in any of the dialogues to preserve the memory
of the historical Socrates
(see approach 4); such differences may only represent changes in Plato’s own
philosophical views.
Developmentalists may generally identify the earlier positions or works as
“Socratic” and the later ones
“Platonic,” but may be agnostic about the relationship of the “Socratic” views and
works to the actual historical
Socrates.
The Historicist View:
Perhaps the most common of the Developmentalist positions is the view that the
“development” noticeable
between the early and later dialogues may be attributed to Plato’s attempt, in the
early dialogues, to represent
the historical Socrates more or less accurately. Later on, however (perhaps because
of the development of the
genre of “Socratic writings,” within which other authors were making no attempt at
historical fidelity), Plato
began more freely to put his own views into the mouth of the character, “Socrates,”
in his works. Plato’s own
student, Aristotle, seems to have understood the dialogues in this way.
6. What is the purpose of the author in writing the article?
(a) Highlighting scholars by presenting their views that these scholars have sought
to augment fairly scant
evidence by employing different methods of ordering the Plato’s dialogues.
(b) Bringing to fore the varying degrees of controversy over Plato’s works, and
highlighting four views about
Plato’s dialogues and their representation of Socrates.
(c) Highlighting the authenticity of Plato’s dialogues by presenting four views
about his dialogues and their
representation of Socrates.
(d) To bring forth the prevalent view that Plato’s earlier works bespoke Socrates’
ideology which Plato presented
as his.

. Page 4 of 40
7. How is Unitarian View different from Literary Atomist View?
(a) Unitarian view holds that there is but a single philosophy to be found in all
of Plato’s works, while the
Literary Atomist view believes that all philosophy to be found in the works of
Plato should be attributed only
to Plato.
(b) Unitarian view holds that there is but a single philosophy to be found in all
of Plato’s works, while the
Literary Atomist view the “difference noticeable between the early and later
dialogues may be attributed to
Plato’s attempt to bespeak Socrates’ views.
(c) Unitarian view holds that there is but a single philosophy to be found in all
of Plato’s works, and that
‘Socratic philosophy’ is to be discounted in Plato’s dialogues, while the Literary
Atomist view reject
completely any relevance or validity of sorting or grouping the dialogues into
groups, on the ground that any
such sorting is of no value to the proper interpretation of any given dialogue.
(d) Unitarian view exempts Socratic view completely, while the Literary Atomist
view Plato’s dialogues as
relevant to Plato only.

8. Which of the following represents the writing style of the author?


(a) Persuasive (b) Creative (c) Narrative (d) Expository
9. How is Developmentalist View different from Historicist’s View?
(a) Developmentalist View hold that the earlier works of Plato’s as “Socratic” and
the later ones “Platonic, even
though they cannot corelate the ‘Socratic View’ to be actually the dialogues of
Socrates; whereas, Historicist
View held that the early and later dialogues may be attributed to Plato’s attempt,
in the early dialogues, to
represent the historical Socrates more or less accurately, but the later views were
the authentic view of Plato
bespoken as Socrates’ views.
(b) Developmentalist View held that the earlier works of Plato representing
Socratic views, and his later views
were his own with a distinct relationship between Socratic view by Plato and the
authentic work of Socrates;
whereas, the Historicists View held that the early and later dialogues may be
attributed to Plato’s attempt, in
the early dialogues, to represent the historical Socrates more or less accurately.
(c) Historicist View held that the early works of Plato represented his authentic
dialogues, and the later dialogues
were heavily influenced by Socratic view; whereas the Developmentalist View holds
that there is a difference
between the early Plato’s work bespoke his true genuine self, but the later part of
his work was influenced
by the ideologies of Aristotle.
(d) Historicist View held that that there is but a single philosophy to be found in
all of Plato’s works, and that
‘Socratic philosophy’ is to be discounted in Plato’s dialogues; whereas, the
Developmentalist View
represented the earlier works of Plato’s as “Socratic” and the later ones
“Platonic, even though they cannot
corelate the ‘Socratic View’ to be actually the dialogues of Socrates.
10. ‘It is possible, of course, that the dialogues are all wholly Plato’s
inventions and have nothing at all to do with
the historical Socrates.’ The underlined part belongs to which of the following
parts of speech?
(a) Noun (b) Adjective (c) Adverb (d) Pronoun
Passage (Q.11-Q.15): Russia on Saturday sent a pair of long-range nuclear-capable
bombers on patrol over its
ally Belarus amid spiraling tensions over Ukraine. The Russian Defense Ministry
said that the two Tu-22M3
bombers practiced interaction with the Belarusian air force and air defense assets
during a four-hour mission.
The flight followed several similar patrols over Belarus, which borders Ukraine to
the north. Russia has denied
plans of attacking its neighbor Ukraine, but urged the US and its allies to provide
a binding pledge that they
don’t accept Ukraine into NATO or deploy offensive weapons and roll back the
alliance deployments to Eastern
Europe. Washington and NATO have rejected the demands.
The West has called on Russia to pull back an estimated 100,000 troops from areas
near Ukraine, but the Kremlin
has responded by saying it will station troops wherever it needs to on the Russian
territory. A fireball rises from
an explosion during the joint exercises of the armed forces of Russia and Belarus
at a firing range in the Brest

. Page 5 of 40
Region, Belarus February 3, 2022. As the tensions over Ukraine soared, the Russian
military has launched a
series of war games spreading from the Arctic to the Black Sea. The Russian troop
deployment to Belarus raised
concerns in the West that Moscow could stage an attack on Ukraine from the north.
The Ukrainian capital is just
75 kilometers (50 miles) from the border with Belarus.
Belarus’ authoritarian leader Alexander Lukashenko, who has increasingly relied on
the Kremlin’s political and
financial support amid bruising Western sanctions triggered by his crackdown on
domestic protests, has called
for closer defense ties with Moscow and recently offered to host Russian nuclear
weapons.
As war fears mounted, Ukrainian authorities launched a series of drills for
civilians to prepare for a possible
Russian invasion. “I am here to learn how to defend myself, defend my relatives and
also understand how to act
in the situation,” Kyiv resident Ilya Goncharovian said after taking part in drills
on the outskirts of the Ukrainian
capital.
Amid the standoff over Ukraine, US President Joe Biden has ordered 2,000 US-based
troops to Poland and
Germany and shifted 1,000 more from Germany to Romania in a show of the US
commitment to NATO’s eastern
flank. Earlier this week, Russian President Vladimir Putin has signalled Moscow’s
readiness for more talks with
Washington and its NATO allies.
As part of high-level diplomacy to ease the tensions, French President Emmanuel
Macron is set to head to
Moscow and Kyiv on Monday and Tuesday, while German Chancellor Olaf Scholz will
travel to Kyiv and
Moscow on Feb. 14-
Amid the tensions with the West, Putin on Friday attended the opening of the Winter
Olympics in Beijing and
met with China’s leader Xi Jinping to strengthen the two countries’ alliance. In a
joint statement, Putin and Xi
declared their opposition to any expansion of NATO while affirming that the island
of Taiwan is a part of China.
11. What is the writing style for this passage?
(a) Narrative. (b) Persuasive. (c) Expository. (d) Ironic.
12. What is the appropriate title for this passage?
(a) The non-negotiation movement.
(b) Russia vs. Ukraine.
(c) Russian bombers fly over Belarus amid Ukraine tensions
(d) Russia bombs Ukraine.
13. Which of the following statement is correct?
(a) Putin supported the mission secretly.
(b) Russia has conducted a series of joint drills with Belarus.
(c) Alexander Lukashenko called for closure of military ties with Belarus.
(d) US President Joe Biden has ordered 1,000 US-based troops to Poland.
14. Read the following statement and determine who said it and when:
“I am here to learn how to defend myself, defend my relatives and also understand
how to act in the situation,
(a) Alexander Lukashenko said it after requesting for closure military ties.
(b) Joe Biden said it after sending the 2000 troops to Poland.
(c) Ilya Goncharovian said it after taking part in drills on the outskirts of the
Ukrainian capital.
(d) Vladimir Putin said it, in case, Ukraine attacks Russia.
15. What is the central idea of the joint statement of Putin and Xi?
(a) Agreeing upon the NATO.
(b) Agreeing that the island of Taiwan is a part of China.
(c) Opposition to expansion of NATO.
(d) Supporting the expansion of NATO.

. Page 6 of 40
Passage (Q.16-Q.20): That large animals require a luxuriant vegetation, has been a
general assumption which
has passed from one work to another; but I do not hesitate to say that it is
completely false, and that it has vitiated
the reasoning of geologists on some points of great interest in the ancient history
of the world. The prejudice has
probably been derived from India, and the Indian islands, where troops of
elephants, noble forests, and
impenetrable jungles, are associated together in every one's mind. If, however, we
refer to any work of travels
through the southern parts of Africa, we shall find (_______) in almost every page
either to the desert character
of the country, or to the numbers of large animals inhabiting it. The same thing is
rendered evident by the many
engravings which have been published of various parts of the interior. Dr. Andrew
Smith, who has lately
succeeded in passing the Tropic of Capricorn, informs me that, taking into
consideration the whole of the
southern part of Africa, there can be no doubt of its being a sterile country. On
the southern coasts there are some
fine forests, but with these exceptions, the traveller may pass for days together
through open plains, covered by
a poor and scanty vegetation. Now, if we look to the animals inhabiting these wide
plains, we shall find their
numbers extraordinarily great, and their bulk immense. We must enumerate the
elephant, three species of
rhinoceros, the hippopotamus, the giraffe, the boscaffer, two zebras, two gnus, and
several antelopes even larger
than these latter animals. It may be supposed that although the species are
numerous, the individuals of each
kind are few. By the kindness of Dr. Smith, I am enabled to show that the case is
very different. He informs me,
that in lat. 24', in one day's march with the bullock-wagons, he saw, without
wandering to any great distance on
either side, between one hundred and one hundred and fifty rhinoceroses - the same
day he saw several herds of
giraffes, amounting together to nearly a hundred. Dr. Smith describes the country
passed through that day, as
'being thinly covered with grass, and bushes about four feet high, and still more
thinly with mimosa-trees.' The
numbers indeed of the lion, panther, and hyena, and the multitude of birds of prey,
plainly speak of the abundance
of the smaller quadrupeds: one evening seven lions were counted at the same time
prowling round Dr. Smith's
encampment. Dr. Smith also informs me that the vegetation has a rapid growth; no
sooner is a part consumed,
than its place is supplied by a fresh stock. There can be no doubt, however, that
our ideas respecting the apparent
amount of food necessary for the support of large quadrupeds are much exaggerated.
The belief that where large
quadrupeds exist, the vegetation must necessarily be luxuriant, is the more
remarkable, because the converse is
far from true. Mr. Burchell observed to me that when entering Brazil, nothing
struck him more forcibly than the
splendour of the South American vegetation contrasted with that of South Africa,
together with the absence of
all large quadrupeds.
16. According to the passage, the author counters which of the following
assumptions?
(a) The large animals depend on meagre flora.
(b) The smaller animals can thrive in scanty vegetation.
(c) The large animals require verdant vegetation.
(d) Animals can thrive even in scanty flora.
17. ‘If, however, we refer to any work of travels through the southern parts of
Africa, we shall find (______) in
almost every page either to the desert character of the country, or to the numbers
of large animals inhabiting it.’
Which of the following words will fill in the blank to make the sentence coherent?
(a) illusions (b) allusions (c) elusions (d) delusions
18. Which of the following inferences concerning the southern part of Africa hold
true with regard to the observation
made by Dr. Andrew Smith?
(a) Barring some pockets of dense forests, the southern part of Africa holds an
enormous fauna, especially large
animals despite the barren ness of the land.
(b) The southern part of Africa boasts of the greatest bulk of fauna, especially
the larger animals despite the
sterile condition of the land
(c) The southern parts of Africa are unfertile, and the scattered forests contain
the bulk of large and small
animals.
(d) The southern part of Africa mainly bears a bulk of large and small animals
defying the assumptions that large
animals require a luxuriant vegetation.

. Page 7 of 40
19. What possible reason has been cited for the thriving of fauna in the southern
part of Africa despite scanty
vegetation?
(a) The rapid replenishing of the consumed vegetation.
(b) The conducive climate of the southern part of Africa.
(c) The minimal tampering of nature by humans.
(d) The presence of large number of quadrupeds.
20. Why does Dr. Andrew Smith need to enumerate the large animals he encounters on
his expedition?
(a) To reiterate the theory of geologists. (b) To indicate an intriguing
phenomenon.
(c) To illustrate earlier account. (d) To negate author’s counter-assumption.
Passage (Q.21-Q.25): Last year, I put together this list of the most iconic poems
in the English language; it’s
high time to do the same for short stories. But before we go any further, you may
be asking: What does “iconic”
mean in this context? Can a short story really be iconic in the way of a poem, or a
painting, or Elvis?
Well, who knows, but for our purposes, “iconic” means that the story has somehow
wormed its way into the
general cultural consciousness—a list of the best short stories in the English
language would look quite different
than the one below. When something is iconic, it is a highly recognizable cultural
artifact that can be used as a
shorthand—which often means it has been referenced in other forms of media. You
know, just like Elvis. (So,
for those of you heading to the comments to complain that these stories are “the
usual suspects”—well, exactly.)
An iconic short story may be frequently anthologized, which usually means
frequently read in classrooms,
something that can lead to cultural ubiquity—but interestingly, the correlation
isn’t perfect. For instance,
Joyce’s “Araby” is anthologized more often, but for my money “The Dead” is more
iconic. Film adaptations and
catchy, reworkable titles help. But in the end, for better or for worse, you know
it when you see it. Which means
that, like anything else, it all depends on your point of view—icon status is (like
most of the ways we evaluate
art) highly subjective.
So, having acknowledged that there’s no real way to make this list, but because
this is what we’re all here to do,
here are some of the most iconic short stories for American readers in the English
language—and a few more
that deserve to be more iconic than they are.
21. What does ‘iconic’ means to the author?
(a) A story that has gradually become part of the collective cognizance of a
culture as an identifiable expression.
(b) A story that gradually reaches a particular culture only to become indelible in
the minds of the people of that
culture
(c) A story that gradually finds its way into different cultures as an artefact for
collective purposes.
(d) A story that entrenches the psyche of a culture in a manner that it becomes a
popular form of expression.
22. ‘Well, who knows, but for our purposes, “iconic” means that the story has
somehow wormed its way into the
general cultural consciousness.’ The sentence reflects which of the following
figure of speech?
(a) Personification. (b) Metaphor (c) Onomatopoeia (d) Synecdoche
23. What does the expression ‘cultural ubiquity’ mean in the context of the
passage?
(a) An all-round presence of culture. (b) A pervasive culture of expression.
(c) A collective cultural pursuits. (d) A uniformity in shared knowledge in
culture.
24. Why does the author say that icon status is (like most of the ways we evaluate
art) highly subjective?
(a) An evaluation of a form of expression as an art is individual.
(b) An evaluation of a form of expression as an art is skewed.
(c) An evaluation of a form of expression as an art is general.
(d) An evaluation of a form of expression as an art is deceptive.

. Page 8 of 40
25. The author in the passage seems to be
(a) A linguist (b) An editor (c) An author (d) Any of the above.
Passage (Q.26-Q.30): It was quite by accident I discovered this incredible invasion
of Earth by lifeforms from
another planet. As yet, I haven't done anything about it; I can't think of anything
to do. I wrote to the Government,
and they sent back a pamphlet on the repair and maintenance of frame houses.
Anyhow, the whole thing is
known; I'm not the first to discover it. Maybe it's even under control.
I was sitting in my easy-chair, idly turning the pages of a paper backed book
someone had left on the bus, when
I came across the reference that first put me on the trail. For a moment I didn't
respond. It took some time for the
full import to sink in. After I'd comprehended, it seemed odd I hadn't noticed it
right away.
The reference was clearly to a nonhuman species of incredible properties, not
indigenous to Earth. A species, I
hasten to point out, customarily (________) as ordinary human beings. Their
disguise, however, became
transparent in the face of the following observations by the author. It was at once
obvious the author knew
everything. Knew everything--and was taking it in his stride. The line (and I
tremble remembering it even now)
read:
...his eyes slowly roved about the room.
Vague chills assailed me. I tried to picture the eyes. Did they roll like dimes?
The passage indicated not; they
seemed to move through the air, not over the surface. Rather rapidly, apparently.
No one in the story was
surprised. That's what tipped me off. No sign of amazement at such an outrageous
thing. Later the matter was
amplified.
...his eyes moved from person to person.
There it was in a nutshell. The eyes had clearly come apart from the rest of him
and were on their own. My heart
pounded and my breath choked in my windpipe. I had stumbled on an accidental
mention of a totally unfamiliar
race. Obviously non-Terrestrial. Yet, to the characters in the book, it was
perfectly natural--which suggested they
belonged to the same species.
And the author? A slow suspicion burned in my mind. The author was taking it rather
too easily in his stride.
Evidently, he felt this was quite a usual thing. He made absolutely no attempt to
conceal this knowledge. The
story continued:
...presently his eyes fastened on Julia.
Julia, being a lady, had at least the breeding to feel indignant. She is described
as blushing and knitting her brows
angrily. At this, I sighed with relief. They weren't all non-Terrestrials. The
narrative continues:
...slowly, calmly, his eyes examined every inch of her.
Great Scott! But here the girl turned and stomped off and the matter ended. I lay
back in my chair gasping with
horror. My wife and family regarded me in wonder.
"What's wrong, dear?" my wife asked.
I couldn't tell her. Knowledge like this was too much for the run-of-the-mill
person. I had to keep it to myself.
"Nothing," I gasped. I leaped up, snatched the book, and hurried out of the room.
26. ‘A species, I hasten to point out, customarily (_________) as ordinary human
beings.’ Which of the following
words would fill in to make the sentence coherent?
(a) parading (b) veiling (c) cloaking (d) masquerading
27. What does the story suggest?
(a) The story suggests a man losing control of his grip on reality, but this is
manifested in the world as an inability
to differentiate between metaphorical and literal language.
(b) The story suggests a man hallucinating, and unable to demarcate between the
real life and the world of
fantasy.
(c) The story suggests irony by way of showing how a man can lose grip of reality
and manifested it as his
inability to differentiate the real content from the false.
(d) The story suggests the binary fission, and how a man can lose the sensibilities
and travel to a fantastical
world that he considers as his world.
28. What does the author mean when he uses the idiomatic expression ‘run-of- the -
mill’ for his wife?
(a) Ordinary (b) Extraordinary (c) Unintelligent. (d) Immature.
29. The protagonist in the story is
(a) A man of conviction. (b) Of a flighty character.
(c) Is impressionable (d) Is perspicacious
30. ‘...his eyes slowly roved about the room. Vague chills assailed me. I tried to
picture the eyes. Did they roll like
dimes? The passage indicated not; they seemed to move through the air, not over the
surface.’ The author of the
novel uses short sentences
(a) For the ease of the reader. (b) To create interest in the reader.
(c) To tell the story in brief. (d) To take off the burden of using pompous words

SECTION – C: LEGAL REASONING


Directions (Q.66 – Q.105): Read the comprehensions carefully and answer the
questions based on it.
Passage (Q.66-Q.71): Charas is the separated resin, which can either be in crude or
purified form which is
obtained from the cannabis plant. This also includes the concentrated preparation
and resin known as hashish
oil or liquid hash. Ganja is the flowering or fruiting tops of the cannabis plant
(excluding the seeds and leaves
when not accompanied by the tops). Any mixture with or without any neutral
material, and of the above forms
of cannabis or any drink prepared therefrom is also included in the definition of
cannabis under the NDPS Act.
The NDPS Act prohibits sale and production of cannabis resin and flowers. However,
it does not prohibit the
use of seeds and leaves. This allowed India to continue its large scale consumption
of bhang and after 25 years
of exemption period, India passed the NDPS Act adopting the same definition of
cannabis. Section 10 of the
NDPS Act allows states to permit and regulate cultivation of any cannabis plant,
production, manufacture,
possession, transport, import and export inter-state, sale, consumption or use of
cannabis (excluding charas) or
the manufacture of medicinal opium or any preparation containing any manufactured
drug from materials which
the maker is lawfully entitled to possess or the manufacture and possession of
prepared opium from opium
lawfully possessed by an addict registered with the State Government on medical
advice for his personal
consumption. Under Section 14 there is a special provision for cannabis, where the
government may allow
cultivation of a cannabis plant “for industrial purposes only of obtaining fibre or
seed or for horticultural
purposes.”
The punishment for producing, manufacturing, possessing, selling, purchasing,
transporting, importing inter#State, exports inter-State or using cannabis is as
follows:
For small quantity: Rigorous imprisonment for a term that may extend to six months
or a fine that may extend
to Rs 10,000, or both. For greater than small quantity but lesser than commercial
quantity: Rigorous
imprisonment may extend to 10 years with a fine that may extend to Rs 1 lakh.
For commercial quantity: Rigorous imprisonment will not be less than 10 years, but
may extend to 20 years and
a fine of not less than Rs 1 lakh (which can be extended to two lakhs).There is a
separate law for juveniles
(Section 18 of Juvenile Justice Act) and those below 18 cannot be prosecuted under
the NDPS Act.
(Extracted with requisite revisions and edits from ‘Cannabis: Your guide to what’s
legal and what’s not in India’
available at https://www.scconline.com/blog/post/2020/09/25/cannabis-your-guide-to-
whats-legal-and-whats#not-in-india/)
66. Aman had always been fascinated to try new ‘things’ in life despite being a
school boy waiting to attain majority
which was far by two long years. From some of the seniors, he got to know that
Ganja gives the feeling of
attaining moksha and being the kind of person Aman was, it sounded like a must try.
After certain contacts and
references he was able to procure the same in small quantity. However, he became
addicted to same and soon
thereafter started procuring the Ganja in greater quantity (lesser than commercial
quantity). He was apprehended
and was sentenced to imprisonment for six months which was opposed by the
prosecution as being less and that
Aman should be sentenced for using quantity above small quantity and should be
punished for maximum
imprisonment i.e. 10 years. Is the plea correct?
(a) Yes, the offence involves the possession and use of Ganja to a quantity higher
than small and hence the
maximum punishment to which Aman can be subjected can be 10 years.
(b) No, the punishment of six months is sufficient for offenders who are young and
not repeated offenders
involved in offences of less serious nature.
(c) Yes, it should be seen what is the exact quantity determined by the State
Government for small or higher
quantities.
(d) No, Aman cannot be punished under NDPS and hence not only the claim of the
defence will fail but also the
punishment initially decided is incorrect.

. Page 17 of 40
67. The State Government of U.P. decided to legalize sale, use or consumption of
resins in the State through the
NDPS (U.P.) Rules, 2021. This legislation was challenged by the “Society towards
rehabilitation and against
Narcotics” on the ground that the action of State Government is illegal to which
the State Government pleaded
Section 10 of the NDPS Act. Is the State Government’s legislation likely to stand
as legal in the Court of law?
(a) No, Section 10 of the NDPS Act does not empower the State Governments to
legislate to the contrary terms
than the Central legislation on matters dealing with resins and hence the Rules are
likely to be declared void.
(b) Yes, Section 10 of the NDPS Act allows states to permit and regulate
cultivation, manufacture or
consumption etc. of any cannabis plant and hence the Rules are going to hold as
legal in the Court of law.
(c) No, any such legislation before coming into effect has to be permitted and
taken approval from the Central
Government as NDPS Act is a Central legislation and hence the Rules are likely to
be declared null and void.
(d) Yes, State Governments are empowered to legislate and incorporate as per their
discretion and suitability
certain amendments to the Central Acts without any requirement of approval from
Centre.
68. The State Government of U.P. later permitted the sale of cannabis leaves and
seeds due to the
upcoming KumbhaMela at designated spots and to legalize the consumption by the
Sadhus on the occasion.
Certain Sadhus were however caught with leaves with the flower portion not been
completely detached from the
plant. They pleaded that since the flowering portion only forms a very
insubstantial portion in the plant, it can
be disregarded as the intention of the Government was unequivocally to protect
Sadhus’ interest. Is their
argument correct?
(a) Yes, the intention of the government behind the legislation has to be seen
which was to protect the interests
of the Sadhus and hence their act of possessing the flowering portion of the
cannabis is valid and cannot be
proceeded against.
(b) No, State Government has only legalized the sale and consumption of cannabis
leaves and seeds. The
consumption and sale of flowering portion constitutes cannabis within NDPS Act
which is an offence under
the Act.
(c) Yes, since the flowering portion forms an insignificant portion of the
legalized product i.e. leaves and seeds,
the Sadhus were correct in their plea and the possession of flowering part can be
disregarded.
(d) No, the Sadhus are liable to be punished for the maximum duration of six months
if they are in possession
of cannabis in small quantities.
69. During the KumbhaMela, at selected places certain outlets were established
which served bhaang to the devotees
as prasada along with providing leaves and seeds of cannabis in a limited and
specified quantity authorized by
the State Government. However, due to the huge number of devotees thronging over
the place, they started
serving the leaves and seeds along with flowers but cautioning them to remove the
flowers before consuming.
Are the shop owners guilty of any offence?
(a) No, applying the reasonable person test it can reasonably be understood that
catering to the requirements of
the crowd would create difficulties and provided the sale was made with caution no
liability can be invoked
against the shop owner.
(b) No, the caveat with the sale of the flowering portion in addition to the sale
of the authorized part leads to the
transfer of the liability on the buyer from that of the seller.
(c) Yes, since the sale of only leaves and seeds (and Bhaang) has been permitted by
the Government hence any
deviance from such guidelines will invoke liability.
(d) Yes, considering the nefarious effects cannabis might have on the health of the
individual, the sale by the
outlet will invoke liability to cater to public interest.

. Page 18 of 40
70. One of the devotees carrying the leaves of the cannabis with him was travelling
to his home in Maharashtra after
the spiritual journey in U.P. However, he was caught by the police for the illegal
possession of cannabis as it
was not authorized in Maharashtra unlike U.P. Can he be arrested for the possession
of the cannabis in
Maharashtra?
(a) No, since the procurement of the cannabis was done in a state where the
procurement was legal, he cannot
be arrested on those grounds.
(b) Yes, since the devotee was caught with cannabis in a state where such a
possession is not legal he can be
arrested on that ground.
(c) No, the arrest would be dictated by the sole discretion of the government of
U.P. on whether inter-state sale
or possession in another state is permitted by it or not.
(d) Yes, however such an arrest cannot be made on the sole ground of possession for
personal use as it is
authorized by the State Government of U.P. but such arrest can be made for sale of
cannabis.
71. Vimal was authorized by the State Government to possess and consume small
quantities of Ganja for personal
use for some medicinal purposes on a daily basis. Vimal however used to buy the
Ganja on a weekly basis and
in a higher quantity to avoid buying every day. Since he was recovering from his
addiction, he used to avoid the
consumption on certain days and used to sell it for some gain. Has he committed any
offence?
(a) Yes, the authorization for possession and consumption of Ganja is limited to
personal use and daily limits
and cannot be sold to others therefore Vimal would be liable for sale of the
cannabis.
(b) No, since he was only procuring the quantity which he is authorized to take, he
has not committed any
offence.
(c) Yes, since he procured the quantity higher than what was authorized by the
State Government, he will be
liable for the offence.
(d) No, since the procurement of the cannabis was for medicinal purposes, no
offence has been committed by
Vimal.
Passage (Q.72-Q.75): Patent grants exclusive monopoly rights to the patent holder
or inventor for twenty years.
After the expiry of such a period, the innovation falls into the public domain.
Article 21 of the Constitution guarantees protection of life and personal liberty
to every citizen. Since the right
to health is integral to the right to life, the government must ensure that the
patent holders do not exercise their
exclusive right over their patented products for a long time, giving them unfair
exploitation of the patent.
Section 3(d) of the Patents Act, 1970 was amended to ensure that patented products
do not stay patented for a
long time by making minor or insignificant modifications. This Amendment had aimed
to prevent ‘patent
evergreening’.
Evergreening refers to the practice whereby pharmaceutical firms extend the patent
life of a drug by
obtaining additional 20-year patents for minor reformulations or other iterations
of the drug without
necessarily increasing the therapeutic efficacy.
Compulsory licensing is “authorizations, permitting a third party to make, use or
sell a patented invention without
the patent owner’s consent.”
Section 84 of the Indian Patents Act, 1970 provides for the conditions under which
a compulsory license can be
granted regarding the invention. The conditions are, when:
1. The reasonable requirements of the public concerning the patented invention have
not been satisfied, or
2. The patented invention is not available to the public at a reasonable price or
3. The patented invention is not used in India.
Whena patent holder hasn’t made the necessary steps to provide the drugs at a
reasonable price, the controller or
any other person interested in the product can step in and offer or apply for a
compulsory license.
This ensures that the patent-holding company doesn’t have unmitigated power to
manufacture and distribute the
drug.
[Extracted with revisions from 'What is the Impact of TRIPS on Pharmaceutical
Industry in India? A
Comprehensive Analysis' by Mani Abinaya K, published on
lawctopushttps://www.lawctopus.com/academike/impact-of-trips-on-pharmaceutical-
industry/]

. Page 19 of 40
72. Novartis was an international pharmaceutical company, filed an application for
granting a patent for an
anticancer drug named ‘G’. Already a patent for G had been granted to another
pharmaceutical company.
Novartis sought a new patent over G by claiming that they had made several
innovations on its formulation that
increased its efficacy by using a substance called ‘Imatinib Mesylate’. However,
upon enquiry the Patent Office
found that Imatinib Mesylate is merely a new form of a known substance that was
already used in drug G.
Further, the efficacy remained the same and there were improvements only in the
therapeutic nature of the drug.
Should Novartis be granted the patent for G?
(a) No, because it is not an instance of patent evergreening as Novartis has not
made significant innovations
over G.
(b) Yes, because Imatinib Mesylate is a new form of a known substance even if it
was already used in G.
(c) No, because there was no increase in the efficacy of G or the substances used
in its formulation.
(d) Yes, because it is not an instance of patent evergreening as Novartis has made
significant innovations over
G.
73. Bayer was a pharmaceutical company that held the patent rights over an
ingredient ‘N’, which was used to treat
liver and kidney cancer. Bayer was granted the patent in 2010, after years of
research and development to bring
N to the market. However, despite 10 years in the market, Bayer was not able to
meet the adequate medicine
supply and sold N at exorbitant rates because it was manufactured in Switzerland.
Natco is a new pharmaceutical
company. Can it apply for a compulsory license to manufacture N?
(a) Yes, because all the requirements as per Section 84 of the Indian Patents Act,
1970 under which a compulsory
license can be granted regarding the invention are fulfilled.
(b) No, because Bayer has made the necessary steps to provide N at a reasonable
price by setting up a
manufacturing plant in Switzerland.
(c) Yes, because interested persons like Natco can step in to apply for a
compulsory license because it would
violate the exclusive monopoly rights conferred by a patent to Bayer.
(d) No, because Bayer has unmitigated power to manufacture and distribute N as it
has been conferred by
exclusive monopoly rights
74. Natco applied for a compulsory license for N on 1st January 2021. Meanwhile,
Bayer had started importing
equipment from Switzerland in December 2020 to set up a drug manufacturing plant in
Maharashtra. Bayer had
started taking these measures so that they could decrease the price of N from Rs.
3,00,000 to Rs. 1,50,000. With
these developments in place, can Natco be granted a compulsory license for N?
(a) Yes, because all the requirements as per Section 84 of the Indian Patents Act,
1970 under which a compulsory
license can be granted regarding the invention are still fulfilled.
(b) Yes, because a price of Rs. 1,50,000 is still not affordable or reasonable for
consumers in the market and this
hinders their right to health.
(c) No, because Bayer has not taken the necessary steps to provide N to consumers
at a reasonable price.
(d) None of the above
75. Jollibee was a reputed pharmaceutical company which manufactured and
distributed various medicines for
different ailments. They sold a drug ‘X’ which helped to control high blood
pressure and a drug ‘Y’ which helped
to lower cholesterol. Many patients used to purchase both X and Y as high blood
pressure and cholesterol levels
tend to affect together. After years of research and development, Jollibee created
a drug ‘Z, which was a 2-in-1
medicine for high blood pressure and cholesterol’. Z used various substances which
were already present in X
and Y. Can Jollibee apply for a new patent for Z?
(a) No, because Z is merely a minor reformulation to create another iteration of
drugs X and Y.
(b) No, because Z is an attempt at patent evergreening by Jollibee as there is no
innovation in the formulation
or therapeutic efficacy of this new drug.
(c) Yes, because Z is a new drug altogether and provides benefits which are
different from X and Y taken
separately.
(d) Yes, because Z is not an attempt at patent evergreening since the modification
is novel, significant and can
tackle both high blood pressure and cholesterol.

. Page 20 of 40
Passage (Q.76-Q.80): Vicarious liability as a concept of law has been with us since
the development of the
traditional doctrine of tort law relating to the liability of employers for the
acts committed by his employee
within the course of his employment.
Increasingly, criminal law became an important site for the imposition of vicarious
liability. Whether a company
can be held criminally liable for the actions of its employees where the law
creating the relevant offence is silent
on this question?
Recently, in Iridium India Telecom ltd. versus Motorola Inc., Motorola sold a
technology product to Iridium that
was accompanied by assertions and promises by Motorola that allegedly turned out to
be false. Iridium bought
a cheating case against Motorola. The case was brought not against Motorola’s
employees but against Motorola
itself. Under IPC, cheating requires an intention to deceive. Motorola argued that
a corporate body, being an
artificial person, is not capable of a mental state and therefore cannot be held
criminally liable. The Supreme
Court rejected this argument after it considered the modern approach to the problem
of corporate criminal
liability in the English Courts.
Of particular relevance to this discussion is the Supreme Court’s reference to the
House of Lords decision in
Tesco Supermarkets Ltd. versus Nattrass, where it was held that, in absence of a
specific statutory or common
law exception, the principle of corporate criminal liability was not based on the
vicarious liability of employer
for the acts of its agents and employees. Instead, it was based on the concept of
attribution. A company cannot
think and act on its own as it is a juristic personality. It thinks and acts
through its employees. In other words,
the mental state and action of its employees are attributed to the company.
This is a legal Fiction but a necessary legal fiction in order for separate legal
personality of the company to
sustain itself over a period of time. Otherwise, the company would not be able to
sign contracts, acquire property,
negotiate with business partners, sue and be sued and make public disclosures and
statements.
[Source- Prof Nigam Nuggehali, “Vicarious Criminal Liability for the Corporate
Officers in India: Problems and
Prospects” Jamia Law Review (2015)”]
76. Shyam works in ABC Corp Ltd., a multinational company engaged in the production
of preserved food. He
works at the post of Assistant Manager, production and is responsible for assuring
the quality of preserved food
manufactured. Due to gross negligence, there was a faulty batch of 100 cupcakes,
which were supplied to a
school. After eating those cupcakes, 80 students fell seriously ill. A case under
the Food Security Act was brought
by the school authorities against ABC Corp ltd. As per Sec 18 of this Act, “for
negligent food manufacturing
and packaging, the company and its officers shall be jointly and severally liable
with imprisonment and fine”.
Decide the liability in light of principle developed in Tesco Supermarkets Ltd.
(a) As per the principle of Attribution, ABC Corp Ltd. shall be liable for the
criminal acts of its employee.
(b) As per the principle of Attribution, ABC Corp Ltd. shall not be liable as
negligence is not a criminal act.
(c) As per the principle of Attribution, ABC Corp Ltd. shall not be liable as
Assistant Manager is not an
employee.
(d) None of the above.
77. Shantanu and Wasim are directors of Folly Chemicals Ltd, a company. In order to
expand their business, there
was a dire necessity of funds. To make quick money, Shantanu and Wasim entered into
a deal with Sooraj
Chemicals Ltd., by forging its documents. However, this fraud was unearthed by
Sooraj Chemicals Ltd. later on.
Sooraj Chemicals Ltd., brought a fraud case against Shantanu and Wasim personally.
However, Shantanu and
Wasim question the maintainability of case filed by Sooraj Chemicals Ltd. (a
company). Decide if the case is
maintainable
(a) The suit is not maintainable since the case against Folly Chemicals Ltd. can
only be brought by the directors
of Sooraj Chemicals and not by Sooraj Chemicals itself.
(b) The suit is maintainable since Sooraj Chemicals Ltd. is a legal personality
vested with all legal entitlements
of a natural person and thus can file a case against another.
(c) The suit is maintainable since anyone can file a case against criminal acts of
the other
(d) The facts are incomplete to answer this question.

. Page 21 of 40
78. As per the author of the above-mentioned passage, there is one necessary legal
fiction. Which of the following
options aptly highlight that legal fiction which is being discussed by the author?
(a) The distinction between principle of vicarious liability and Principle of
Attribution is not a natural distinction
but a legal distinction.
(b) The Company is a creation of statute and hence vested with all legal
entitlements of a legal person.
(c) The distinction between tortious liability and criminal liability of an
employer for acts done by its employees.
(d) All of the above
79. As per the passage, which of the following factors is responsible for the
invocation of principle of Attribution in
case of crimes committed by the employees of a company?
(a) The intention of the wrong-doer is the primary factor in imputing criminal
liability.
(b) The Company is nothing but an artificial person clothed with the rights and
duties of a natural person.
(c) Neither (a) nor (b)
(d) Both (a) and (b)
80. Radhe is the owner of Satnam Departmental Store (A sole proprietor firm). He
earns huge profits in a year and
accordingly, has to pay a huge amount of tax liability. However, agitated by the
fact that the area where his
Departmental Store is situated does not get proper water and electricity supply, he
decides not to pay tax from
next year. Accordingly, he starts evading his tax liability. Income Tax Department
sues Satnam Departmental
Store for punishing with respect to tax evasion. Will this case be maintainable?
(a) The case will be maintainable as per the Principle of Attribution which shall
hold Satnam Departmental store
(b) The case will not be maintainable since the Principle of Attribution will not
be applicable
(c) The case will be maintainable under Vicarious Liability as evasion of tax is
not a criminal liability
(d) The case will not be maintainable since Radhe did not have any criminal
intention while evading tax.
Answe: B
Passage (Q.81-Q.85): Absolute privilege exempts defamatory statements made during
judicial proceedings; by
government officials; by legislators during debates in the parliament; during
political speeches in parliamentary
proceedings; and communication between spouses.
When a person making the statement has a legal, social or moral duty to make it and
the listener has an interest
in it, then the defence of qualified privilege is allowed. Generally, such a
defence can be availed in case of
reference for a job applicant; response to police enquiries; fair criticism of a
published book or movie in review;
communication between parents and teachers; communication between employers and
employees; and
communication between traders and credit agencies.
Court while dwelling upon absolute and qualified privileges, in the context of
defamation, observed as:
“67. Even the issue of absolute privilege has remained a subject-matter of
considerable debate. Is absolute
privilege absolute in the sense of being infinite? As late as 1998, in the decision
reported as Waple v. Surrey
County Council[5], it was held:
The absolute privilege which applies to statements made in the course of judicial
or quasi-judicial proceedings
and in the documents made in such proceedings, would only be entitled where it was
strictly necessary to do so
in order to protect those who were to participate in the proceedings from being
sued themselves.
(Source: https://www.scconline.com/blog/post/2021/02/12/defamation-2/)
81. Which of the following statements will be granted absolute immunity from being
defamatory?
I. A statement once made in a pleading filed before a court of law, is a
publication of such statement.
II. The divorce petition containing defamatory statements.
III. Sam was a witness in a criminal case, and despite his damaging testimony,
Ralph was found not guilty.
Ralph sued sam for defamation.
IV. In a trial, the prosecutor said in court, "Quentin is a murderer." Quentin, was
given a not guilty verdict.
(a) All of the above (b) II, III and IV
(c) II and IV (d) III and IV

. Page 22 of 40
82. Which of the following statements accurately describes the essentials of
absolute privilege?
I. In a civil action for defamation, the plea of absolute privilege has been held
to be a good defense.
II. Absolute privilege protects a statement as no action would lie for it, however
false and defamatory it may
be, even though, it was made maliciously and with an improper motive.
III. Laws of defamation recognise that statements made in public can be protected
from prosecution in a
Court of law under certain circumstances.
IV. Absolute privilege attaches to public statements made in certain circumstances.
(a) II and III
(b) I, II and IV
(c) II and IV
(d) More than two options might be correct.
83. On Monday, a fist-fight broke out in South African parliament as members of the
left-wing Economic Freedom
Fighters (EFF) party tried to prevent president Jacob Zuma of the ruling African
National Congress (ANC) from
making an address. During the protest, security guards forcibly removed 20 members
of the EFF party. It was
the second confrontation to take place in two weeks. Prime Minister Justin Trudeau
offered another apology for
actions that led to his physical confrontation with opposition MPs in the House of
Commons. The event was
featured in prime time news, and footage of the incident was made public.
Parliament accused the media source
of defamation. The media source alleges absolute privilege. Decide
(a) The media source will be held liable for publishing footages of incident as
they directly interfere with the
reputation of the representatives in the minds of the public.
(b) The media source will not be held liable for publishing the footage as
statements made during the debates in
parliament are protected under the defense of absolute privileges.
(c) The media source will be held liable as they cannot claim the defense of
absolute privilege in the present
scenario.
(d) The media source will not be held liable for defamation as the statements made
by the defendant against the
character of the plaintiffs could not be said to be absolutely privileged.
84. Choose a statement which can be construed as mark of distinction between
absolute and qualified privilege:
I. Qualified privilege is used when the statement made is true, but absolute
privilege can be invoked even if
the defamatory remark made is false.
II. A speech given by a Member of Parliament in a parliamentary proceedingis
covered under absolute privilege.
However, if the Member of Parliament makes statement outside parliament regarding a
author of a book ;
he can avail the defence of qualified privilege.
III. Absolute privilege can be utilized as an in rem defence, although qualified
privileges can only be asserted
in personam.
(a) I
(b) II and III
(c) II
(d) All of the above
85. Assertion: Government and political matters are proper subjects for public
discussion and such discussion is
covered by the defense of qualified privilege.
Reason: It is not necessary to prove that the matter published concerned an issue
of public interest to establish
the defense of qualified privilege.
(a) Both A and R are false.
(b) Both A and R are true and R is correct explanation of A.
(c) A is true but R is false.
(d) A is false but R is true.

. Page 23 of 40
Passage (Q.86-Q.90): Remedy is one of these fundamental rights historically
recognized in our legal system
as central to the concept of ordered liberty. "The principle that rights must have
remedies is ancient and
venerable. In 1703, the right to a remedy was expressly recognized in Anglo-
American law. In Ashby v. White,
the Chief Justice of the King's Bench stated:
If the plaintiff has a right, he must of necessity have a means to vindicate and
maintain it and a remedy if he is
injured in the exercise or enjoyment of it; and indeed it is a vain thing to
imagine a right without a remedy; for...
want of right and want of remedy are reciprocal.... Where a man has but one remedy
to come at his right, if he
loses that he loses his right.
Similarly, the United States Supreme Court from its earliest time has recognized
the bedrock principle that
deprivations of law require remedies. In Marbury v. Madison, Chief Justice Marshall
endorsed the common law
requirement mandating a remedy for every wrong: It is a general and indisputable
rule, that where there is a legal
right, there is also a legal remedy by suit or action at law, whenever that right
is invaded.... For it is a settled and
invariable principle in the laws of England, that every right, when withheld, must
have a remedy, and every
injury its proper redress.
(Source: Excerpt from an article by TRACY A. THOMAS, SAN DIEGO LAW REVIEW,
86. The passage talks about the famous case of ashby V White, which states, where
there is right, there is remedy.
Which of the following maxim can be referred to from the given context of the
passage?
(a) Ubi jus remedium (b) Ubi jus, ibi remedium
(c) Ubi jus remedy (d) Ibi jus ubi remendium
87. The state government of Ohio provides funding to all Ohio's public schools. The
common schools filed a lawsuit
against the government, claiming that the state's conduct of solely funding public
schools is arbitrary and violates
Ohio's constitutional promise of a "thorough and efficient system of common
schools." Decide
(a) The suit filed by common school will prevail and be entitled of damages.
(b) The suit filed by common school will fail as the state has not made any
distinction among common school.
(c) The suit filed by common schools will prevail as state’s action is in direct
violation of the constitutional right
of Common Schools.
(d) The suit filed by common schools will not prevail as there is no violation of
legal right.
88. Continuing with the same circumstances as the last one, if it is proven that
the state's conduct is not arbitrary,
would the court's ruling be overturned even though there is breach of Ohio's
constitutional provision of a
"thorough equal and efficient system of common schools? Will common school have any
remedy?" Decide
(a) Yes, as where there is right there is remedy.
(b) No, as where there is no arbitrary action on state’s end, the maxim cannot be
invoked.
(c) Yes, as there is violation of legal right.
(d) No, the court’s ruling will not be changed but as there is violation of legal
right and hence common schools
are entitled for remedy.
89. Assertion: A meaningful remedy is one that is minimally adequate and effective
at ensuring the protection of
the attendant right.
Reason: Where state court remedies are arbitrary and unreasonable, the Court has
found a violation of Due
Process.
(a) Both A and R are false.
(b) Both A and R are true and R is correct explanation of A.
(c) A is true but R is false.
(d) A is false but R is true.

. Page 24 of 40
90. Chandu a differently abled person was stopped at the airport for checking.
Passengers with disability had to
remove their prosthetic limbs put it in the X-ray machine. Many passengers felt the
procedure was humiliating
and violating their right to life. Rules also required passengers to vacate their
wheelchair for screening by CISF.
A complaint was filed with the court and it was ruled that differently abled
persons with prosthetic limbs should
not be asked to remove the prosthetics at airport security checks. In view of the
information provided in the
passage, on what grounds may the court's verdict be accepted?
(a) Lifting a person with disability during air travel or security check-up is
inhumane and violates their human
dignity, and that this should be done only with the person’s consent.
(b) The petitioners had moved the Supreme Court under Article 32 of the
Constitution for putting a system in
place so that other such differently-abled persons do not suffer similar agony.
(c) The petitioners had moved the Supreme Court as the practice violates their
fundamental rights under Articles
21 of the Constitution.
(d) Cannot be ascertained as the passage lacks sufficient information.
Passage (Q.91-Q.95): For long, mandamus was primarily a tool for restoration of
office or a right from which
a person had been wrongly deprived. But in the present context, the very essence of
a prayer for a writ of
mandamus, to put it in the words of Stanley Alexander de Smith, is “an allegation
of a Contempt of the Crown
consisting in the neglect of a public duty”.
Mandamus being the mode of enforcement of a public duty, is also the definition
that best fits the Indian
conception of it. The Supreme Court in Hari Krishna Mandir Trust v. State of
Maharashtra recently recapped the
line of decisions to this effect and reaffirmed that a High Court, under Article
226 of the Constitution, not only
has the power to issue the writ of mandamus, but also the duty to do so, where the
government or a public
authority has failed to exercise or has wrongly exercised discretion conferred upon
it.
There was some movement away from the writ courts as being the primary forum of
remedy for those seeking
the enforcement of various rights not covered under the ordinary civil laws.
A series of developments, such as the Shah Committee, the Administrative Reforms
Committee, various Law
Commission reports suggesting the setting up of various tribunals etc, all reached
a crescendo with the judgment
of a seven-judge bench of the Supreme Court in L Chandra Kumar judgment. Though the
Court ultimately held
that the power of the High Courts to issue writs could not be curtailed, the
tribunals were placed as the forum of
first resort for service disputes that might have otherwise come to the High Courts
as writ petitions.
The decline of the writ of Mandamus in nineteenth century UK was because of the
reduction in
maladministration. The reason for revival and expansion of the scope of mandamus in
India today lies in the
dysfunctionality in the tribunals and the faith of the people in the High Courts
and the Supreme Court as the right
fora to solve their problems.
[Source- Bar and Bench, “Why the Writ of Mandamus…” by Vikram Hegde, published on
27 March 2922
https://www.barandbench.com/columns/writ-of-mandamus-declined-19th-century-uk-
flourishes-21st-century#india]
Based on the passage above, answer the following questions
91. Based on the passage above, which of the following options aptly summarise the
author’s opinion?
(a) The writ of mandamus is rising in India due to increase in maladministration by
public servants.
(b) The writ of mandamus is a tool by which the Constitutional Courts may interfere
where the public authority
has failed to carry out its legal duties.
(c) The Tribunals were formed to look into the cases of failure on part of public
authorities to fulfil its legal
duties, however, the Tribunals have failed to live up to that expectation, thereby
giving rise to writ of
mandamus
(d) all of the above

. Page 25 of 40
92. In which of the following situations can a person file a writ of Mandamus
before the Constitutional Courts of
the Country?
(a) Failure on part of an individual/private and public body to carry out is legal
duties.
(b) Failure on part of any public body/authority to carry out its legal duties.
(c) Failure on part of any public body/authority to carry out the duties as
expected by the public
(d) All of the above.
93. Harish and Sarla are a married couple and expecting a baby soon. They ask a
doctor of a government hospital to
conduct a sex determination ultrasound. However, the doctor refuses to perform any
such procedure. They file a
writ of mandamus before the High Court seeking directions against the doctor of the
Government Hospital to
fulfil his duties. Decide, if the High Court shall interfere.
(a) The writ of mandamus shall not be maintainable as the doctor has not failed to
perform its legal duties.
(b) The writ of mandamus shall be maintainable as the doctor has failed to perform
its expected duties.
(c) The writ of mandamus shall not be maintainable as the doctor is an individual
and no writ of mandamus can
lie against an individual.
(d) The rate of mandamus shall be maintainable as the couple has been wrongly
deprived of its right to know
the sex of the child.
94. Shreyansh purchased a pressure cooker from a well-known utensils brand, Dawkins
under the company Dawkins
pvt. Ltd. However, there was a certain defect in the cooker and he asked Dawkins to
replace the cooker. Dawkins
refused to entertain his grievance. Shreyansh wants to file a complaint against
Dawkins. His lawyer suggests
him to file a writ of Mandamus against Dawkins before High Court as public at large
may affect if any accident
took place because of defect. Decide how accurate is the lawyer’s suggestion
(a) The lawyer’s suggestion was wrong as the aggrieved person should first approach
the consumer
forum/tribunal and in case of inaction by the tribunal, he should file a writ of
mandamus.
(b) The lawyer’s suggestion was wrong as no writ of mandamus can lie against
Dawkins.
(c) The lawyer’s suggestion was right as he can approach the High Court for an
inaction on part of Dawkins to
address public grievance.
(d) Both (a) and (b)
95. Daya has incorporated her private company along with her friend Babita. Daya
had 51% Shareholding in the
company, whereas Babita had 49% shareholding. When certain issues cropped up
between Daya and Babita,
Daya passed a board resolution to remove Babita and appoint her friend Stuti as a
director. Babita challenges
this decision before the National Company Law Tribunal (NCLT). However, the
Tribunal fails to listen to the
case for 3 years. She files a writ petition before the High Court against the
inaction of NCLT. Decide if such writ
petition is maintainable.
(a) Daya can approach High court under Art 226 for a writ of mandamus.
(b) Daya cannot approach high court under Art 226 for a writ of Mandamus because
the NCLT has been unable
to hear the application due to huge pendency.
(c) Daya can approach High court under Art 226 for a writ of mandamus because of
inability of the tribunal to
take speedy actions
(d) Daya cannot approach High court under Art 226 for a writ of mandamus since no
such writ can lie against a
private company.

. Page 26 of 40
Passage (Q.96-Q.100): There are three dimensions to the varying ways in which this
question is addressed in
different legal settings. First, the measures taken in response to crimes committed
by children may be tailored to
the age of the child, so that restrictions are placed on the punishments attached
to convictions, and allowance
may be made for a range of welfare or educational interventions. Second, most legal
regimes now identify an
age below which no child can be held legally responsible for criminal offences
(doli incapax, Strafunmündig,
oder schuldunfähig), a line between the absence or presence of any criminal legal
subjectivity.There are three
dimensions to the varying ways in which this question is addressed in different
legal settings. First, the measures
taken in response to crimes committed by children may be tailored to the age of the
child, so that restrictions are
placed on the punishments attached to convictions, and allowance may be made for a
range of welfare or
educational interventions. Second, most legal regimes now identify an age below
which no child can be held
legally responsible for criminal offences (doli incapax, Strafunmündig, oder
schuldunfähig), a line between the
absence or presence of any criminal legal subjectivity.There are three dimensions
to the varying ways in which
this question is addressed in different legal settings.
First, the measures taken in response to crimes committed by children may be
tailored to the age of the child, so
that restrictions are placed on the punishments attached to convictions, and
allowance may be made for a range
of welfare educational interventions. Second, most legal regimes now identify an
age below which a child cannot
be held legally responsible for criminal offences (doli incapax), a line between
the absence or presence of any
criminal legal subjectivity.
No matter what a child under this age does, even if there was evidence that the
child had the required criminal
intent, they cannot commit a crime under defense of doli incapax. Third, a
‘transition zone’ of conditional
criminal responsibility is generally identified, where a child’s capacity to
understand that their action is seriously
and legally wrong is a matter for the court’s consideration. The ages to which this
concept applies varies along
with the age of criminal incapacity. The upper limit of such a ‘transition zone’,
the age of full criminal
responsibility, indicates when such a person is presumed to have the require
understanding of legal right and
wrong ‘as entirely as if he were forty’. My focus here will be on the second and
third dimensions, because these
are the issues which vary the most allowance for conditional criminal
responsibility is frequently treated as if
were equivalent to a higher age of criminal incapacity.
....the rule operates, essentially and invariably, (i) to make ‘guilty knowledge’
an ingredient in every offence
charged against a child, and (ii) to require the prosecution to adduce evidence,
other than the actus reus itself,
from which such knowledge can be inferred....Its effect is to import into every
criminal offence with which a
child is charged a mens rea consisting in, at least, knowledge of the wrongfulness
of the actus reus.
What is required to rebut the presumption of doli incapax is a demonstration that
the child understood their act
as ‘seriously wrong’, rather than merely mischievous or naughty. The presumption
and the possibility of its
rebuttal operates as the marker of a ‘zone of transition’ from an existence outside
the criminal law, as an ‘innocent
child’, albeit still governed by welfare and education, to an existence governed by
criminal law as well. It is this
transition which transforms ‘horseplay’ and ‘bullying’ into ‘assault’,
‘manslaughter’ and ‘murder’, ‘shoplifting’
into ‘theft’, and ‘mischievousness’ into ‘criminality’
(Source:
https://www.researchgate.net/publication/
272304421_Doli_Incapax_and_Its_Vicissitudes_Childhood_and_Cri
minal_Responsibility_)
96. The author in the passages states that, to rebut the presumption of doli
incapax is a demonstration that the child
understood their act as ‘seriously wrong’, rather than merely mischievous or
naughty. This rebuttal can be
attributed to which of the following maxim?
(a) Doli Capax
(b) Doli incapax
(c) De minimis non curat lex
(d) Furiosis nulla voluntas est

. Page 27 of 40
97. When a child is charged for a criminal offence, the author expresses his or her
point of view in the first dimension.
What measures here is suggested to address the problem?
(a) Put a limit to child age’s beyond which he/she cannot be prosecuted.
(b) Punishments associated with the crime should not be limited to the age of the
children.
(c) Punishment should be given to the child in accordance with child’s age.
(d) The author suggests minimalist punishment for children so that room is made for
a variety of welfare
educational interventions.
98. Toon In with Me is an American live-action/animated anthology comedy television
series that airs weekday
mornings on MeTV. The show is hosted in live-action segments by Bill, MeTV's
"cartoon curator", along with
his puppet friend, Toony the Tuna. Richie, a die-hard admirer of the show, never
misses a single episode. Richie's
parents are working parents; therefore they don't give a hoot about what Richie
watches on television. Every
episode, the Host Bill would conclude it by shooting on Toony the Tuna, leading him
to disappear. Because
Richie's father is a cop, she is familiar with what a gun is and how it is used, as
depicted in the show. One Sunday,
while playing with her father, she pulled out the gun and fired a shot at him. In
view of the maxim doli Incapax,
what accountability does Richie have here?
(a) Richie will not be held accountable due to his age.
(b) Richie will not be held accountable since a child below 7 years cannot be held
legally responsible for criminal
offences.
(c) Riche is accountable because she was familiar with what a gun is and how it is
used thus forms mens rea.
(d) Richie cannot be held liable since she was unaware of the wrongfulness of the
actus reus.
99. Continuing with the same facts as above, what will the decision be if the court
finds conditional criminal liability
on the part of the minor who is invoking doli incapax?
(a) It will be up to the court to decide whether or not to convict Richie.
(b) Richie will be deemed to have the necessary understanding of legal right and
wrong and so held accountable.
(c) The prosecution must present evidence other than the actus reus from which
conditional criminal liability on
the part of the minor can be inferred.
(d) Richie cannot be held liable under the rule of Doli Incapax.
100. Assertion: The court always presumes child’s innocence in a criminal matter.
Reason: The rule of doli incapax, a line between the absence or presence of any
criminal legal subjectivity.
(a) Both A and R are true but R is not correct explanation of A.
(b) Both A and R are true and R is correct explanation of A.
(c) A is true but R is false.
(d) A is false but R is true.
Passage (Q.101-Q.105): Truth and justice have prevailed. Congratulations to ICJ for
a verdict based on
extensive study of facts. I am sure Kulbhushan Jadhav will get justice. - PM
Narendra Modi.
It goes without saying that in a major legal and diplomatic victory for India and
also simultaneously in a major
legal and diplomatic setback along with global embarrassment for Pakistan, the
International Court of Justice
(ICJ) has held upfront that Pakistan violated the Vienna Convention in the
Kulbhushan Jadhav case and it should
review and reconsider his conviction and sentencing while allowing India consular
access to the Indian national.
Importantly, the ICJ ruling said unequivocally that the stay on the death sentence
pronounced on Jadhav must
remain. It minced no words in saying clearly, categorically and convincingly that,
the court considers that a
continued stay of execution constitutes an indispensable condition for the
effective review and consideration of
the conviction and sentence of Jadhav.
To be sure, while indicating its unhappiness with the judicial process regarding
Kulbhushan Jadhav, the ICJ said
that, Court considers it imperative to re-emphasize that the review and
reconsideration of the conviction and

. Page 28 of 40
sentence of Jadhav must be effective. This clearly comes as a rap on the knuckles
of Pakistan's opaque way of
trial of Jadhav by military courts which is the biggest proof that it was just a
sham trial and everything was pre#decided! It is a no-brainer that this sharp
observation of ICJ clearly tantamount to an open indictment of Jadhav
being tried before secret military black courts where the evidence against him and
his legal defence remains
unknown! How can this by any standard be termed as fair trial?
What's more, while rejecting all the major contentions put forward by Pakistan, the
ICJ said the Vienna
Convention was applicable in the Jadhav case regardless of allegations that he was
engaged in espionage. It also
conveyed unambiguously that, Pakistan must inform Jadhav without further delay of
his right under Article 36
and allow and arrange for his legal representation. Pakistan should be ashamed that
ICJ has to remind it that
what all legal rights should be provided to Jadhav! Government should know any
person arrested has a right to
consult a counsel.
(Source: https://www.legalservicesindia.com/law/article/1260/26/Review-And-
Reconsider-Conviction-And#Sentencing-of-Jadhav-ICJ-To-Pakistan)
101. What, in your opinion, is the best reason for the ICJ judgment on the stay of
the death penalty awarded to Jadhav
by the Pakistan court?
(a) The court believes that a further stay of execution is required for a proper
review and analysis of Jadhav's
conviction and sentence.
(b) The court believes that a continuous stay is unnecessary since a proper review
and analysis of Jadhav's
conviction and sentence is more vital.
(c) The court believes that the Pakistani courts acted on their whims and fancies,
jeopardising the criminal justice
system and violating natural justice principles.
(d) The court orders a stay of execution for Jadhav since his execution is in
confirmation with the Vienna Treaty.
102. Shahida, a mute six-year-old girl, lives with her parents in the hills of
Sultanpur, Pakistan. Razia, with the help
of her husband, plans to take her to the shrine of Sufi saint Nizamuddin Auliya in
Delhi, India, through the
Samjhauta Express, in the hopes that it may restore her speech. The train pauses
for repairs on its route back to
Pakistan, and Shahida jumps off to save a lamb while Razia is sleeping. Shahida is
unable to re-board the train
because it has already left. Panicked, she boards a freight train, expecting it
will take her in the same direction,
but it takes the other route and lands her at Kurukshetra, India. When Pawan, an
Indian, finds that she is from
Pakistan, he promises to take shahida home on his own, despite the fact that he
lacks a passport and a visa. Pawan
and shahida are able to enter Pakistan over the border barriers, but they are
apprehended shortly thereafter on
suspicion of Pawan being an Indian spy. Decide
(a) The Pakistan court can penalise Pawan for an act of espionage based on the
circumstances surrounding his
arrest.
(b) The Pakistani court cannot punish Pawan for an act of espionage since he did
not enter Pakistan's territorial
jurisdiction with malice.
(c) A Pakistani court cannot convict Pawan on suspicion of spying for India.
(d) A Pakistani court has the authority to hold Pawan for illegally entering
Pakistani territory.
103. According to the passage in Paragraph 2, the ICJ granted India consular access
to the Indian national. What
meaning may be derived from the phrase "Consular access"?
(a) Consular access is available, even if the nationality of the arrested person is
disputed.
(b) Consular access is the ability of foreign nationals to have access to consulate
or embassy officials of their
own country in the host nation.
(c) India had demanded the consular access to former Indian Navy officer Kulbhushan
Jadhav in Pakistan under
the rules of the Vienna Convention.
(d) The Vienna convention allows foreign nationals who are arrested or detained to
have the access.

. Page 29 of 40
104. Assertion: Article 36 of the Convention does not exclude from its scope
certain categories of persons, such as
spies or terrorists and this Article applies to all nationals of State parties in
the territory of other State parties.
Reason: Accepting Pakistan’s argument would be very dangerous, because, otherwise,
any state can deny the
right to consular access by describing the actions of a foreign national as
espionage and terrorist activities.
(a) Both A and R are true but R is not correct explanation of A.
(b) Both A and R are true and R is correct explanation of A.
(c) A is true but R is false.
(d) A is false but R is true.
105. Continuing with the factual circumstances described in 37, imagine Pawan being
prosecuted by a Pakistani court
on charges of espionage. Which sentence best depicts the author's point of view
toward the innocuous individual
who is being falsely accused on purpose?
(a) Not allowing Pawan to meet with embassy officials from his home country in the
host nation would be a
risky move on the side of the host country.
(b) Regardless of the allegations, Pawan should be provided with a legal counsel.
(c) Pawan can only be prosecuted in military courts if he has been appointed legal
representation under Vienna
Convention article 36.
(d) All of the aforementioned.
. Page 30 of 40
SECTION - D: LOGICAL REASONING
Passage (Q.106-Q.110): Read the following passage. In light of the same, answer the
questions that follow.
“Because things are the way they are, things will not stay the way they are”. These
words of German poet Bertolt
Brecht have always struck me as a socio-political analyst. In the context of the
recent Uttar Pradesh assembly
election results, they become more relevant. Why did a larger section of Dalits
vote for the Bharatiya Janata
Party (BJP)? This question seems to trouble many political observers and a section
of the public. It surprised a
section of the metropolitan intelligentsia, who want to see reflections of their
ideology and aspirations in the
marginalised.
The UP election campaign can be considered as a text to explore this question.
While caste, religion, women,
youth appeared as overt categories of mobilisation by various political parties, an
agenda for the poor did not
come up in the political discourse. It is interesting to observe that the BJP
emphasised the welfare of the poor
(“garib kalyan”) as one of the important components of its electoral campaigning
strategy. The BJP’s UP election
in-charge, Dharmendra Pradhan, emphasised this agenda from his earliest meetings.
When Narendra Modi
started his rallies, his core argument was that BJP had delivered on development
with special emphasis on social
welfare schemes such as Ujjwala Yojana, PM Awas Yojana, free ration, etc. So,
Dalits and marginals appeared
in their discourse in two ways, as labharthi (beneficiaries) and in the campaign
around the poor. The BJP tried
to see Dalits and marginal communities not through caste but with the tinge of
economic class. They successfully
drew a bigger circle through the politics of governance and development, unlike
political parties that approached
these communities through ideas of identity politics.
Interestingly, the welfare of the poor, the core of the liberal and socialist
discourse in Indian electoral politics, is
being incorporated by the BJP in its language of developmental politics through
ideas of garib kalyan and
labharthi. Through its connection with these beneficiaries, the BJP tried to create
a new identity, that of a
development-aspirant community, named recently as vikas yoddha by PM Narendra Modi.
The process of this
shift became quite clear when Modi said in one of his interviews that if showing
concern for the poor and
marginals may be called as socialist concern, “I am a socialist in that sense”.
106. “Because things are the way they are, things will not stay the way they are”.
Which of the following situations
aptly describe the implication?
I) The current Russia-Ukraine crisis will re-define the policy decisions of other
countries.
II) With the rampant social-media covering aspects of the political scenario, the
governments cannot win the
next Lok-Sabha elections based on the current manifesto.
III) The polarised strategies and sanctions by various countries demands a self-
reliance on part of the government
to come up with economic policies boosting growth in the future.
(a) Only II (b) Only III (c) I & II (d) I & III
107. All of the following can be inferred from the passage, except
(a) The author is a socio-political analyst.
(b) The reflections of the ideology and aspirations of the metropolitan
intelligentsia did not come to pass with
respect to the UP elections.
(c) The BJP’s core emphasis was to incorporate the Dalits in the larger scheme of
welfare of the poor in the UP
election.
(d) Political parties, other than BJP, circumvented the ideas of identity politics.

. Page 31 of 40
108. Which of the following reflects the central point of the author in the
passage?
(a) Shifts in political choice are not the outcome of election campaigns alone, but
a result of the long-term
developmental strategies adopted and delivered by the BJP.
(b) The win for BJP in UP elections was the result of stressing the caste,
religion, women, youth appeared as
overt categories, and delivering on development with special emphasis on social
welfare schemes for the
poor as the covert strategies.
(c) The calculated social engineering-based representation worked in BJP’s favour,
but the new social chemistry
formed by the party through its governance strategies dented various identity-based
mobilisations too.
(d) It is true that humans can’t be free from identities but sometimes new, broader
identities become more
effective than conventional ones.
109. Which of the following will not be in consonance with the stance of the author
in the passage?
(a) The UP elections showcased the growing agency of the marginal.
(b) Identity politics loses its sheen in the current UP elections.
(c) Welfare-Hindutva combination won the vote in UP
(d) The contesting political parties other than BJP focussed on caste, religion,
women, youth, but the agenda for
the welfare of the poor was left out.
110. ‘The process of this shift became quite clear when PM Modi said in one of his
interviews that if showing concern
for the poor and marginals may be called as socialist concern, “I am a socialist in
that sense”. Which of the
following is/are the conclusions drawn from the passage?
(a) PM Modi is a socialist.
(b) PM Modi appears to be socialist.
(c) PM Modi is a socialist in only one sense.
(d) None of the above.
Passage (Q.111-Q.115): Are popular songs today happier or sadder than they were 50
years ago? In recent years,
the availability of large digital datasets online and the relative ease of
processing them means that we can now
give precise and informed answers to questions such as this. A straightforward way
to measure the emotional
content of a text is just to count how many emotion words are present. How many
times are negative-emotion
words - “pain', 'hate' or 'sorrow'- used? How many times are words associated with
positive emotions - 'love, joy'
or 'happy-used? As simple as it sounds, this method works pretty well, given
certain conditions. This is a possible
technique for what is called 'sentiment analysis'. Sentiment analysis is often
applied to social media posts, or
contemporary political messages, but it can also be applied to longer timescales,
such as decades of newspaper
articles or centuries of literary works.
The same technique can be applied to song lyrics. For our analysis, we used two
different datasets. One contained
the songs included in the year-end Billboard Hot 100 charts. These are songs that
reached wide success, at least
in the United States, from The Rolling Stones "(I Can't Get No) Satisfaction' to
Mark Ronson's 'Uptown Funk".
The second dataset was based on the lyrics voluntarily provided to the website
Musixmatch. With this dataset,
we were able to analyse the lyrics of more than 150,000 English-language songs.
These include worldwide
examples, and therefore provide a wider, more diverse, sample. Here we found the
same trends that we found in
the Billboard dataset, so we can be confident that they can be generalised beyond
top hits.
English-language popular songs have become more negative. The use of words related
to negative emotions has
increased by more than one third. Let's take the example of the Billboard dataset.
If we assume an average of
300 words per song, every year there are 30,000 words in the lyrics of the top-100
hits. In 1965, around 450 of
these words were associated with negative emotions, whereas in 2015 their number
was above 700. Meanwhile,
words associated with positive emotions decreased in the same time period. There
were more than 1,750 positive#emotion words in the songs of 1965, and only around
1,150 in 2015. Notice that, in absolute number, there are
always more words associated with positive emotions than there are words associated
with negative ones. This

. Page 32 of 40
is a universal feature of human language, also known as the Pollyanna principle,
and we would hardly expect
this to reverse: what does matter, though, is the direction of the trends.
The effect can be seen even when we look at single words: the usage of 'love' for
example, practically halved in
50 years, going from around 400 to 200 instances. The word 'hate, on the contrary,
which until the 1990s was
not even mentioned in any of the top-100 songs, is now used between 20 and 30 times
each year. Our results are
consistent with other, independent analyses of song moods, some of which used
completely different
methodologies, and focused on other characteristics of the songs.
111. According to the passage, what can be inferred about sentiment analysis?
(a) It's an analysis used to measure the reason behind why sad songs are more
popular than happy songs
(b) It's an analysis used to measure the number of negative words as against
positive words in a song
(c) It's an analysis used to measure the effect of negative emotions over positive
emotions
(d) It's an analysis used to measure negative words and their emotional impact
112. Which one of the following can LEAST likely be inferred about the Billboard
hot experiment?
(a) The experiment works on similar lines as that of understanding the sentiment
analysis.
(b) It comprised of two datasets, one measuring the popularity of the songs and the
other to understand the lyrics.
(c) The experiment provided diverse trends across both the datasets used for
analysis.
(d) It concluded that during a specific time period there was a noticeable increase
in negative emotions.
113. What is the Pollyanna principle?
(a) The fact that there are always more negative emotions attached to a song than
positive emotions.
(b) The fact that positive emotional words have a better impact than negative
emotional words.
(c) The fact that in absolute numbers, more positive words are associated with
songs than any other area.
(d) The fact that in general, more words are associated with positive emotions than
negative emotions.
114. Which of the following most closely reflects the structure of the passage?
(a) The passage introduces the concept of sentiment analysis and then provides
examples of the same.
(b) The passage provides various examples to prove that negative emotions are more
widely accepted as
compared to positive emotions.
(c) The passage introduces the concept of sentiment analysis and further proves the
same through conducting
various studies.
(d) The passage highlights how emotions play a vital role in deciding whether the
songs are happy or sad.
115. Which of the following is the central flaw in the given argument related to
sentiment analysis?
(a) A higher number of words whether positive or negative is society’s reflection.
(b) The words used in the song do not depict the negative emotional state.
(c) Words describe the state of the lyricist or the singer.
(d) A simple technique of counting words measures emotional content.
Passage (Q.116-Q.120): India’s agriculture sector is all set to create an all-time
high production record this year.
According to the Second Advance Estimates of Production of Foodgrains for 2021-22,
released by the Ministry
of Agriculture and Farmers Welfare last week, India’s total foodgrains production
will cross 316 million tonnes.
This is 1.7 percent higher than the total production the year before. But what
truly captures the remarkable growth
in India’s farm production is the stark trend in the past six years: Foodgrains
production has gone up from 252
million tonnes in 2015-16 to 316 million tonnes now; rising every single year.
Contrast this with the performance
in the six years preceding 2016-17 — production fluctuated between 244 and 265
million tonnes.
While overall farm production has increased, there are diverging trends. Among
cereals, coarse grains such as
jowar, bajra and ragi are expected to see a decline in output while maize is
expected to buck the trend. Thanks
to good monsoons, rice production, both in kharif and rabi seasons, is expected to
increase by almost 3 per cent.

. Page 33 of 40
Wheat production, too, is expected to go up by 2 per cent. Pulses are expected to
see their output grow by almost
6 per cent with the exception of tur, which is likely to dip by 7 per cent.
Oilseeds are expected to see a production
growth of 3.3 per cent, thanks to significant increases in mustard and soybean,
making up for the fall in groundnut
production. Among the key cash crops, sugarcane is expected to see a jump of over 4
per cent while cotton
production may decline by over 3 per cent. The overall production growth suggests
that no Indian will go to bed
hungry.
There are two policy concerns emerging from this data. One, what happens to food
inflation. For instance,
wholesale inflation has been 10.5 per cent in wheat, 14.5 per cent in maize, over
23 per cent in oilseeds and 45
per cent in cotton. While domestic production is one big factor in influencing
prices, the minimum support prices
announced by the government (pulses) as well as the international prices (oilseeds)
of these commodities also
impact them. The combination of these factors will play out in the year ahead. For
example, lower production in
cotton when prices are already high will raise the raw material costs of the
domestic textile industry, weakening
its competitiveness. The other concern would be to ensure improved remuneration for
farmers. The ironic —
and troublesome — aspect of India’s sustained increase in farm production in the
past 5-6 years has been the
concurrent rise in farmers’ distress as the terms of trade have worsened.
116. Which of the following can be inferred from the given passage?
i. The rise in foodgrains production does not necessarily lead to a decline in
farmer's concerns.
ii. The amount of growth in foodgrains production capacity in the last few years is
unusual.
iii. The growth in foodgrains production of almost all food items is similar to
each other.
(a) Only iii (b) Only i (c) Only i and ii (d) Only ii and iii
117. The author’s arguments are based on which among the following assumptions?
(a) India’s supply chain is adequately good, making sure every Indian gets enough
food to survive.
(b) 316 million tonnes of food production would be delivered to every Indian’s
doorstep.
(c) Mustard and Soyabean are insignificant parts of the oil production industry in
India.
(d) India’s agriculture production is1.7 percent higher than the total production
the year before.
118. With which of the following would the author agree with?
(a) An increase in farm production for one year should remove the distress of the
farmers.
(b) Farmers have the most important role to play in growing India’s food grains
production; hence, they should
be paid the highest.
(c) Unless a significant attention is given to the policy on terms of the trade, an
increase in food grains
production may not necessarily bring down the farmer’s distress.
(d) India will become a global player in the cotton industry if the price of cotton
is decreased and production
increased.
119. The author has used which of the following as a premise to support his
argument?
(a) The production of cotton should be reduced when its prices are so high.
(b) India’s total foodgrains production last year was fifty percent more than what
it was five years ago.
(c) Minimum support prices decided by the government play a significant role in
deciding food prices.
(d) The foodgrains production levels in India have grown every single year in the
five years.
120. Which of the following would weaken the author’s arguments in the second
paragraph?
(a) The overall production of maize is lower than that of jowar, bajra and ragi.
(b) The overall production of cotton forms 5 percent of total food production,
which is double than sugarcane.
(c) The composition of soybean and mustard oil composition in the total food
production is less than 3 percent.
(d) The weak warehouse structure and distribution channels impact the reach of the
agrarian products to the
consumers.

. Page 34 of 40
Passage (Q.121-Q.125): Unlike 2017, when the BJP formed the government, the
Congress had emerged as the
single largest party with 28 seats, the ruling party had a clear-cut majority on
its own this time in the state of
Manipur. Besides, the BJP government has received post-election support from the
Janata Dal (United) and the
Nagaland Peoples’ Front, with six and five MLAs each. The clear verdict from the
electorate this time should
help Mr. Singh run a stable government free of the shenanigans that dominated his
previous tenure. Mr. Singh
also ran a spirited campaign to secure an electoral majority for the BJP, with some
of his initiatives such as the
“Go to Hills” and “Go to Villages” bearing fruit.
The stable majority should give Mr. Singh the ballast to focus more on governance
and address the immediate
needs of the State. Manipur has a higher literacy rate — 79.85% compared to the
country’s average of 74.04%
— besides achieving a medium human development index of 0.697, as of 2019. Only
Sikkim and Mizoram have
better indices in the North-east. But chronic unemployment, especially of the
youth, remains a key concern that
needs to be tackled by the Government. The recurring border conflicts in villages
in Manipur and Nagaland, with
strikes and blockades, need to end quickly, and a majority government will be
better placed to address this
ticklish problem. While the BJP remained silent on the unpopular Armed Forces
(Special Powers) Act in the
course of the election and still managed to win handily in the State, it cannot
assume that the electorate agreed
with the views of the Union government and the party on the Act. This could come up
as a problem, yet again.
As for the weakened Opposition in the State, the BJP’s facile victory opens up
fresh challenges for the Congress’s
beleaguered leadership even as smaller parties such as the National Peoples’ Party
have tried to warm up to the
BJP by offering outside support. Manipur is a vital border State, with a history of
extremism and ethnic violence
whose embers still remain. While a stable government is best placed to work out a
lasting peace and focus on
livelihood issues, it will be in the Opposition’s best interests to keep the
Government on its toes by acting
maturely.
121. Which of the following best captures the essence of the passage?
(a) North-East region has witnessed historical conflicts and tremors.
(b) BJP has effectively worked on its political manifestos in Manipur region.
(c) Mr. Singh’s electoral majority for BJP in Manipur can sustain only through
effective functioning by
addressing local issues.
(d) N. Biren Singh’s role in winning the BJP a stable majority helped him return as
the Chief Minister of
Manipur.
122. Which of the following can be concluded from the above passage?
(a) The previous regime in the state of Manipur have boosted economic growth of the
state.
(b) The Armed Forces (Special Powers) Act remains an eye sore for the people of
Manipur that cannot be
overlooked by BJP.
(c) BJP has won with clear majority this time after a cut-throat competition.
(d) Coalition government helps to ripen benefits of policies to everyone; hence,
decentralising the power.
123. The author is most likely to agree with which of the following statements?
(a) Though unelected, opposition parties have a critical role to play in
development of a region.
(b) BJP does not have support or liking from the other parties in the region.
(c) The border and protest issues cover a vast portion of challenge to be addressed
by the incumbent govt.
(d) Manipur has been performing consistently well in various demographic
indicators, leading to a continuous
rise in its human development index.
124. ‘While a stable government is best placed to work out a lasting peace and
focus on livelihood issues, it will be
in the Opposition’s best interests to keep the Government on its toes by acting
maturely.’ Which of the following
reflects the assumption behind the argument?
(a) Opposition’s role in keeping the incumbent government streamline cannot be
discounted.
(b) Only a government that finishes its tenure can bring the lasting peace to a
state.
(c) The work of the opposition is to restrict the incumbent government from its
ambitious plans.
(d) The opposition party must ensure that Peace and livelihood issues are key areas
are looked into.

. Page 35 of 40
125. ‘The clear verdict from the electorate this time should help Mr. Singh run a
stable government free of the
shenanigans that dominated his previous tenure.’ Which of the following, if true,
will weaken the authors claim
made in the argument?
(a) During the previous tenure, the state witnessed highest inflow of refugees
amongst the NE region from the
bordering countries.
(b) Half of the term of current incumbent of BJP has passed, and the state suffers
from horrendous problem of
stagflation.
(c) Coalition govt enabled to work for overall betterment of people due to which
literacy rate increased.
(d) The major portfolio was with the BJP in the previous tenure; therefore, the
governance was dictated by the
BJP party.
Passage (Q.126-Q.130): When K. Eric Drexler popularized the word 'nanotechnology in
the 1980's, he was
talking about building machines on the scale of molecules, a few nanometers wide -
motors, robot arms, and
even whole computers, far smaller than a cell.
Much of the work being done today that carries the name 'nanotechnology' is not
nanotechnology in the original
meaning of the word. Nanotechnology, in its traditional sense, means building
things from the bottom up, with
atomic precision. This theoretical capability was envisioned as early as 1959 by
the renowned physicist Richard
Feynman. Based on Feynman's vision of miniature factories using nanomachines to
build complex products,
advanced nanotechnology, sometimes referred to as molecular manufacturing, will
make use of positionally#controlled mechanochemistry guided by molecular machine
systems.
Mihail (Mike) Roco of the U.S. National Nanotechnology Initiative has described
four generations of
nanotechnology development. The current era, as Roco depicts it, is that of passive
nanostructures, materials
designed to perform one task. The second phase introduces active nanostructures for
multitasking for example,
actuators, drug delivery devices, and sensors. The third generation is expected to
begin emerging around 2010
and will feature nanosystems with thousands of interacting components. A few years
after that, the first integrated
nanosystems, functioning much like a mammalian cell with hierarchical systems
within systems, are expected
to be developed.
Unfortunately, conflicting definitions of nanotechnology and blurry distinctions
between significantly different
fields have complicated the effort to understand the differences and develop
sensible, effective policy. The risks
of today's nanoscale technologies (nanoparticle toxicity, etc.) cannot be treated
the same as the risks of longer#term molecular manufacturing (economic disruption,
unstable arms race, etc.). It is a mistake to put them together
in one basket for policy consideration - each is important to address, but they
offer different problems and will
require different solutions. But as a general-purpose technology, it will be dual-
use, meaning it will have many
commercial uses and it also will have many military uses-making far more powerful
weapons and tools of
surveillance. Thus, it represents not only wonderful benefits for humanity, but
also grave risks. A key
understanding of nanotechnology is that it offers not just better products, but a
vastly improved manufacturing
process. A computer can make copies of data files - essentially as many copies as
you want at little or no cost. It
may be only a matter of time until the building of products becomes as cheap as the
copying of files. That's the
real meaning of nanotechnology, and why it is sometimes seen as "the next
industrial revolution."
Nanotechnology not only will allow making many high-quality products at very low
cost, but it will allow
making new nanofactories at the same low cost and at the same rapid speed. This
unique (outside of biology,
that is) ability to reproduce its own means of production is why nanotech is said
to be an exponential technology.
How soon will all this come about? Conservative estimates usually say 20 to 30
years from now. However, it's
not too early to begin asking some tough questions and facing potential issues. Who
will own the technology?
Will it be heavily restricted or widely available? What will it do to the gap
between rich and poor? How can
dangerous weapons be controlled and perilous arms races be prevented? Many of these
questions were first
raised over a decade ago, and have not yet been answered. If the questions are not
answered with deliberation,
answers will evolve independently and will take us by surprise, the surprise is
likely to be unpleasant.

. Page 36 of 40
126. It can be inferred from the passage that, as an exponential technology,
nanotechnology can make it possible
(a) for large manufacturing systems to evolve rapidly from small ones.
(b) for computers to become more efficient in their outputs.
(c) for high-quality products to be made at low cost.
(d) to spread widespread destruction.
127. Which of these has not been claimed about nanotechnology in the passage?
(a) It is the next revolution in industry.
(b) Effective policies regarding nanotechnology has not been framed because of
conflicting definitions and
blurry distinctions between different fields.
(c) The nanotechnology today is far from the concept traditionally envisioned.
(d) It has the ability to reproduce in a way far more efficient than biology.
128. The term 'positionally controlled mechanochemistry' as mentioned in the
passage most likely refers to:
(a) A process whereby mechanical energy is transformed into chemical energy.
(b) A process whereby reactive molecules attached to molecular mechanical systems
are guided and brought
together in planned sequences. positions and orientations.
(c) A process whereby reactive molecules encounter one in different positions
through random thermal motion
in a liquid or vapor.
(d) A futuristic process that will impede rather than enhance the progress of
nanotechnology.
129. The predicted development of nanotechnology over four generations is similar
to:
(a) The development of human civilization over the Stone Age, the Feudal Age, the
Industrial Age and the
Technological Age.
(b) The Darwinian theory of the evolution of populations over several generations
through a process of natural
selection.
(c) The stages in the growth of more and more complex capabilities in the human
body through birth. infancy,
adulthood and death.
(d) None of them
130. With which of the following statements is the author most likely to disagree?
(a) Any policy enacted to deal with the immediate risks of nanotechnology cannot be
significantly different from
that required to deal with the long-term risks associated with it.
(b) Any integrated policy on nanotechnology cannot ignore the effect nanotechnology
will have in widening the
gap between the rich and the poor.
(c) Because it is a matter of time before building products becomes as dishonest as
copying them, the
development of nanotechnology may well signal the advent of the next
industrialization.
(d) Nanotechnology being a dual-use technology may see it being applied only in the
sphere of military
technology in the future

MOCK 41
ECTION-A: ENGLISH LANGUAGE
Directions (Q.1-Q.30): Read the following passage carefully and answer the
questions that follow.
Passage (Q.1-Q.5): When my daughter was little, I became fixated on a schoolhouse a
few blocks from our
apartment—a Tudor-style storybook cottage, with red trim and a brick chimney and a
playground all of the
wood. Its first-floor windows were concealed by tall bushes of a deep impossible
green, and everything that
childhood should be was waiting for my daughter behind them, or so I believed. When
I went inside, my
expectations were met. The children, aged two to six, were serious and serene,
occasionally speaking to each
other in low, considerate tones. They stacked blocks, strung beads, and arranged
letter boards. When it was time
for “walking on the line”—a morning custom in which the children followed a line of
tape on the floor, around
and around, silent and judiciously spaced—I felt overcome by a sense of dazed
compliance.
This was our local Montessori school, and I had convinced myself that, with a bit
of scrimping and bootstrapping,
I could somehow find the money to send my daughter there. I scheduled her required
interview; afterwards, the
director told me, “Oh, she’s a dream,” and at that moment I would have signed a Sea
Org contract in exchange
for a year of my kid’s enrolment. But when I reviewed the numbers, the following
weekend, I concluded that I
could pay the tuition only if I went into credit-card debt—and, really, if that
qualifies as being able to “afford”
something, what can’t you afford? I withdrew her application, and, to self-soothe,
I bought a Montessori-ish
hundred-piece counting board for her off Amazon. She barely touched it, and I gave
it away after her toddler
brother expressed an interest in eating the numbers.
For the Montessori-curious parent on a budget, there is consolation in the wide and
lasting influence of the
movement’s founder, Maria Montessori, the Italian physician and educator whose
ideas and innovations are
ubiquitous even in the preschools that do not bear her name. The eschewing of
individual desks in favour of mats
and child-size tables, the primacy of hands-on learning, daily observances such as
“circle time” and “choice
time” - all of these elements of early-childhood education are indebted to
Montessori’s philosophy.
At the turn of the twentieth century, it was revolutionary to think that a child’s
education could be child-centred—
shaped according to his or her actual brain and body. Montessori and her many
disciples made this common
sense. What’s more, they believed something that still seems counterintuitive
today: that children are, in their
essence, methodical, self-directed beings with a strong work ethic, perfectly
capable of deep concentration, and
that their tendency toward inattention and disruption can be a reasonable response
to disharmonious
surroundings. This most orderly and tranquil of educational philosophies had its
beginnings in the grimmest and
most chaotic of circumstances. In 1897, Montessori, one of the first women in Italy
to earn a medical degree,
had recently graduated from the University of Rome and was volunteering at the
school’s psychiatric clinic,
where her responsibilities entailed visits to the city’s ghastly insane asylums.
1. Which of the accompanying are the innovative ideas by Maria Montessori that have
been applied in the
Montessori schools?
I. The essentiality of hands-on learning.
II. The daily observances such as ‘circle time’ and ‘choice time’.
III. The usage of mats and child-size tables instead of individual desks for
children.
(a) I and II (b) II and III (c) I and III (d) I, II and III
2. What is the tone of the author in the passage?
(a) Dogmatic (b) Commiserating (c) Laudatory (d) Introspective

. Page 3 of 36
3. Which one of the following activities did NOT enchant the author when she
entered the local Montessori school?
(a) The teachers or rather psychologists who trained the children to control their
emotions and harness them in
a positive manner.
(b) The children organized letter boards, piled blocks, and threaded beads.
(c) The children, who ranged in age from two to six, were solemn and calm,
occasionally conversing in quiet,
attentive tones to one another.
(d) The walking on the line ritual where the youngsters walked a line of ribbon on
the floor, back and forth,
quiet and neatly spaced.
4. Which of the accompanying are CORRECT in reference to the passage?
A. Montessori and her followers conform that children are systematic, autonomous
beings with a strong work
code, perfectly capable of deep attentiveness, and failing these can be indicative
of disharmonious
surroundings.
B. Montessori, after graduation, volunteered at the school's psychiatric clinic,
where her responsibilities
included visits to the city's dreadful mad asylums.
C. At the beginning of the twentieth century, it was quite a common belief that a
child's education could be
child-centred that is shaped around his or her cognition and health.
(a) Only A and B (b) Only B and C (c) Only A and C (d) All A, B and C
5. How did the author compensate when she couldn’t enrol her child into the local
Montessori school?
(a) She decided to get into a credit card debt and anyhow enrol her kid into the
school.
(b) She purchased a Montessori-style hundred-piece counting board for her child.
(c) She got a tuition teacher who was experienced as a Montessori school teacher to
teach her kid at home.
(d) She joined a Montessori school as a teacher for her child to get the benefit of
admission.
Passage (Q.6-Q.10): Ankara has close ties to both Kyiv and Moscow, and Erdogan
believes his country can
play a key role in defusing the tensions. He has suggested in the past that Turkey
could be a venue for possible
peace efforts. Ukrainian President Volodymyr Zelenskyy (L) with Turkey's President
Recep Tayyip Erdogan
review the honor guard in Kyiv, Ukraine.
Turkey’s President Recep Tayyip Erdogan arrived in Kyiv on Thursday for talks with
Ukraine’s President
Volodymyr Zelenskyy as his country walks a tightrope trying to balance its
relations with both Russia and
Ukraine. A key NATO member in the strategically important Black Sea region, Turkey
has been urging a
diplomatic solution to the crisis and has repeatedly offered to mediate between the
two. Ankara has close ties to
both Kyiv and Moscow, and Erdogan believes his country can play a key role in
defusing the tensions. He has
suggested in the past that Turkey could be a venue for possible peace efforts.
Last week, Erdogan said it would not be “rational” for Russia to invade Ukraine,
and that Turkey would do
whatever is necessary as a NATO member. However, he has also spoken of a need for a
“meaningful dialogue
with Russia” to resolve any “reasonable” security concerns it may have and to
explain to Moscow why some of
its requests “are not acceptable.”
Turkey has historic relations with Ukraine and strong ethnic ties with Ukraine’s
Crimean Tatar community.
Ankara has spoken out against Russia’s 2014 annexation of the Crimea, vowing never
to recognize it. Before
departing for Kyiv, Erdogan underlined Turkey’s commitment to Ukraine’s territorial
integrity and sovereignty.
Ankara has increased defense cooperation with Ukraine in recent years. It has sold
Kyiv armed Barakat TB2
drones which have been used against pro-Russian separatists in eastern Ukraine’s
Donbass region, angering
Moscow. The two countries also plan joint defense industry production projects.
Several agreements, including
a free trade deal, are expected to be signed during Thursday’s visit — which
coincides with the 30th anniversary
of the establishment of diplomatic ties between Turkey and Ukraine. The situation
over Ukraine has put Turkey
in a bind. The NATO member has been trying to repair its frayed ties with the U.S.
and other alliance members,

. Page 4 of 36
following its controversial decision to buy advanced Russian air defense
technology. But at the same time, it
cannot afford to damage its ties with Moscow.
Turkey also needs to tread carefully with Russia in Syria. Ankara needs Moscow’s
approval to continue its
presence in northern Syria, despite the two supporting opposite sides in Syria’s
civil war. In 2020, 37 Turkish
soldiers were killed in Russian-backed airstrikes against rebels in Syria’s last
rebel-held Idlib province. To
further complicate matters, Russia is a major source for Turkey’s natural gas and
is currently building the
country’s first nuclear power station.
6. Select the appropriate title for the above-mentioned passage?
(a) Mediation between the two countries
(b) A mediating role for Turkey’s President Erdogan in Ukraine crisis?
(c) Information on Ukraine crisis.
(d) Turkey vs. Ukraine.
7. Which of the following represents the writing style of the author?
(a) Persuasive (b) Creative (c) Narrative (d) Analytical
8. Select the incorrect statement according to the passage.
(a) Turkey is a key NATO member in the strategically important Black Sea region.
(b) Turkey has strong ethnic ties with Ukraine’s Crimean Tatar community.
(c) Erdogan said it would be “rational” for Russia to invade Ukraine.
(d) Russia is currently building the country’s first nuclear power station.
9. Which of the following is an inference drawn from the passage?
(a) Turkey’s historic relations with Ukraine and strong ethnic ties with Ukraine’s
and bilateral ties with Moscow
requires it to tread carefully as a mediator by not offending any of the countries.
(b) Turkey cannot side with Moscow at the cost of offending Ukraine with which it
has historical relations.
(c) Moscow may retaliate by severing all bilateral ties with Turkey if it so much
catches a whiff of betrayal from
Turkey.
(d) Russia has no interest in Turkey playing the adjudicator, and trying to play a
key role in defusing the tensions
between Russia and Ukraine.
10. Which of the following is the correct interpretation of the idiomatic
expression ‘walks a tightrope’?
(a) A catch 22 situation.
(b) A situation where both options cannot be lost, if things do not workout in
one’s favour.
(c) A situation where navigating allows very little or no error.
(d) A situation where one is bound to make a mistake irrespective of how carefully
one treads.
Passage (Q.11-Q.15): Most of us have heard the saying that some folks "work to
live" and others "live to work."
When employers are looking for one type but hire another, big problems arise. An
extensive new report from
Bain that digs into workers' changing expectations for their jobs suggests not. The
authors spent a year surveying
20,000 workers in 10 countries as well as conducting in-depth interviews with more
than 100 employees.
They concluded that there are four work orientations. These archetypes help us
better understand what it takes
for different individuals to find a sense of purpose at work, the report states. If
you know who you're dealing
with, you're better placed not only to hire the right person for the right role but
also to help your existing team
stick around. The first one is the Operators. Of the traditional work-to-live type,
the reports say "operators find
meaning and self-worth primarily outside of their jobs. When it comes down to it,
they see work as a means to
an end. They're not particularly motivated by status or autonomy, and generally
don't seek to stand out in their
workplace. They tend to prefer stability and predictability. Thus, they have less
interest in investing to change

. Page 5 of 36
their future compared with other archetypes. At the same time, operators are one of
the more team-minded
archetypes, and often see many of their colleagues as friends."
The second is Givers. "Givers find meaning in work that directly improves the lives
of others. They are the
archetype least motivated by money. They often gravitate toward caring professions
such as medicine or
teaching, but can also thrive in other lines of work where they can directly
interact with and help others. Their
empathetic nature typically translates into a strong team spirit and deep personal
relationships at work. At the
same time, their more cautious nature means they tend to be forward planners, who
are relatively hesitant to
jump on new opportunities as they arise," according to the report's authors. The
third one is Artisans. "Artisans
seek out work that fascinates or inspires them. They are motivated by the pursuit
of mastery. They enjoy being
valued for their expertise, although they are less concerned with status in the
broader sense. Artisans typically
desire a high degree of autonomy to practice their craft and place the least
importance on the camaraderie of all
the archetypes. While many find a higher purpose in work, this is more about
passion than altruism," the report
claims.
The fourth one is explorers. A free-spirited type, the report says "explorers value
freedom and experiences. They
tend to live in the present and seek out careers that provide a high degree of
variety and excitement. Explorers
place higher-than-average importance on autonomy. They are also more willing than
others to trade security for
flexibility. They typically don't rely on their job for a sense of identity, often
exploring multiple occupations
during their lifetime. Explorers tend to adopt a pragmatic approach to professional
development, obtaining only
the level of expertise needed."
11. Which of the accompanying is INCORRECT in reference to the passage?
(a) Explorers find their sense of purpose and identity in the job they do.
(b) Artisans are passionate about their work and they endeavour to improve and
master their job, as well as they
aren’t concerned with their status.
(c) Explorers prefer to live in the present and look for jobs that offer a lot of
diversity and excitement to them.
(d) Artisans desire greater independence in their line of work where they can bloom
to their full potential.
12. Which of the following statements are correct in the case of the Givers?
A. They are reluctant to grab new opportunities.
B. They find amity by helping others and so, they gravitate towards careers like
teaching and medicine.
C. The value of money is the least motivating factor in their perspective.
(a) Only A and B (b) Only A and C (c) Only B and C (d) All of them.
13. Which of the accompanying perfectly describes the advantage that employers
might get if they know what type
of a worker a person is?
(a) The archetypes allow the employers to grasp what it requires for various people
to have a sense of purpose
at work.
(b) Employers can enjoy the advantage of not only employing the ideal individual
for the job but also keeping
their current team.
(c) The Human Resources policy of the companies is bound to apply this rule and
most of the time, they don’t
have a say in this matter.
(d) The archetype allows employees to focus on their prototypes and apply in
companies with the same
examples, which works for the employers too.
14. Which of the accompanying is INCORRECT in the case of the Operators?
(a) They are one of the essential elements to build a team for a project.
(b) They have an aversion towards permanence and uniformity.
(c) They don’t consider work to be motivational and for them, it is just a way to
support their life.
(d) They don’t look forward to leaving a mark in their work culture.

. Page 6 of 36
15. Why can we rely on the analysis by Bain regarding the types of workers?
(a) Bain was himself a core member of a reputed company and he was a fantastic
observer of people and also,
he was a renowned psychologist.
(b) They came up with the theory that some staffs work to live while others live to
work.
(c) A year was devoted by the author to analyse twenty-thousand workers in ten
countries and carry out personal
interviews with more than a hundred employees.
(d) He devoted half his life in analysing thousands of workers in ten countries,
and carry out personal interviews
with more than a hundred employees.
Passage (Q.16-Q.20): Administrators and teachers took their time to learn how to
insert these nouns into the
grammar of everyday parlance. Politicians and company heads investing in education
were quicker. Journalists
noticed the change and performed their expected role of spreading the word(s). The
Programmed of Action
(1992) awakened and exhorted ageing vice-chancellors to recognize how important a
role “innovation” was
going to play in the impending dawn of the new century. Syllabi had to be revamped
to make them capable of
inspiring the innovative spirit of youth whose imagination had been stifled by
nursery teachers. Universities took
up the challenge of repairing what had been damaged during thoughtlessly playful
kindergarten years. A fresh
narrative of educational reform was born.
A few years ago, I met a young man who had designed a five-day training module to
instill the spirit of innovation
among school teachers. A whole range of VIPs had endorsed this effective module.
When I met its maker, the
remarkable module had already been administered to nearly half a million teachers,
turning them around from
their fixed pedagogic ways. Among the teachers he had trained, a few dozen had been
selected for recognition
as leaders. State governments were vying with each other to arrange the five-day
jab of innovation among their
listless teachers.
According to a recent report in this newspaper, an American innovator had attracted
a record number of big
investors in a new device capable of detecting a wide gamut of potential illnesses
from a few drops of blood.
The device radically cut down the price and hassle of a standard blood test. This
disruptive health-tech device
ruled the American market for several years before being revealed to be a fraud.
The success of that project helps us figure out the intimate relationship between
“innovation” and “narrative”.
Both achieved the status of keywords. Together, they marked the arrival of a new
culture. However, one was
more important than the other. While narrative carried sustained weight, in the
final analysis it was subservient
to its verbal mate, innovation. That is where the new goal of teaching and research
lay — in sculpting a mind
that could habitually innovate in any sphere of choice, including the art of
creating a narrative.
Innovation thus became the supreme purpose of the humble teacher’s labours.
Universities set up cluster resource
centers where exam-weary youngsters might seek refuge to assemble a new device or
invent a solution for an
old nagging problem. The mission to carve out such a space in schools had to wait
for a little while, but now that
wait is over. Entrepreneurship has finally become a “subject” and teachers have
been trained to handle it
effectively. Their focus is on nurturing young adults who do not hanker after a
job; instead, they create
employment for others. The innovation resides in the narrative.
16. Which are the two central nouns used in this passage?
(a) Innovation and technology. (b) Technology and education.
(c) Education and narrative. (d) Innovation and narrative.
17. Which of the following is the correct title for this passage?
(a) How ‘innovation’ and ‘narrative’ came to dominate education.
(b) Innovative ideas leading the education system.
(c) Innovation and educative schemes.
(d) The parallels of innovation and narrative.

. Page 7 of 36
18. Which of the following is/are correct statement(s)?
(a) During the1950 the two nouns silently rolled into India’s academic world.
(b) Administrators and teachers eventually learned to insert innovation and
narrative into the grammar of
everyday parlance.
(c) An African innovator had attracted a record number of big investors in a new
device capable of detecting a
wide gamut of potential illnesses from a few drops of blood
(d) Innovation and narrative both are equally important in today’s day and age.
19. Which of the following best expresses the meaning of ‘narrative’ in brief in
the context of the passage?
(a) Story line. (b) Historical account.
(c) Plot. (d) Statement.
20. Which of the following cannot be inferred from the passage?
(a) Innovation carries more weightage than narrative.
(b) Innovation and narrative are potent agents of change.
(c) The art of creating a narrative is exclusive of the art of innovation.
(d) The art of creating a narrative is inclusive in the art of innovation.
Passage (Q.21-Q.25): Before radio stations started calling it "country" after World
War II, a man at an Ozark
fiddle contest or a church gathering was listening to "hillbilly" or "old
familiar." His music spoke to a life lived
not in opposition to the city but on its own plain terms. In the late 1800s, folk
songs lamented the "vacant chair"
at the supper table left by the Civil War. In the 1930s, the proto-country of Bob
Wills and his Texas Playboys
dunked the Victorian music-hall love song into the swing-time sweat of the Friday
evening barn house stomp -
the soundtrack to unwinding after work. Hank Williams - the genre's great, self-
immolating hero, a star at twenty#three, dead in a Cadillac at twenty-nine -
articulated the pain of failed love in a yodel influenced by blackface
singers. What united the music was a catholicity of origin and an unerring
obsession with life inside the home.
Not a protest song but the reality of that vacant chair. Not social commentary but
the brutality of love, the rhythm
of work and leisure, the steady roll of the seasons, the passage from birth to
marriage to death.
While radio had a history of tacit integration, record sales demanded photographs
of artists and categories to put
them in, and by the late '20s most major labels had a "hillbilly" and a "race"
subsidiary. After World War II,
those terms changed to "country" and "rhythm and blues." On one of the first major
country stations, WSM - the
call sign standing for the slogan of its insurance-company sponsor, "We Shield
Millions" - whites listened to
black artists without knowing it, every Saturday on a program called the Grand Ole
Opry. One of the great songs
of 1930, for example, Jimmie Rodgers's "Blue Yodel No. 9," featured Louis Armstrong
on trumpet and his then#wife, Lil Hardin, on piano. But Armstrong wasn't credited
on the record. As late as the 1960s, labels released
black country artists like Charley Pride without photos on the record sleeves, a
reversal of the white enthusiasm
for blackface and minstrel skits.
The wide road to country's commercialization opened in 1941, when WSM started
broadcasting the Opry live
from the Ryman Auditorium in Nashville. The Ryman was known as the "mother church
of country music," and
the nickname was literal. The Ryman had been built as a church in the late
nineteenth century by a riverboat
captain as a gift to the evangelist who reformed him. When it was converted into a
music hall, listeners still sat
on pews. It was during this decade that a young boy growing up in Florida, the
child of a single mother, would
tune in to the Opry broadcast every Saturday and be inspired by the song writing
and the harmonies, which he
loved as much as he loved the church singers, he heard on Sunday mornings. In 1962,
Ray Charles recorded his
album of blues-inflected country, Modern Sounds in Country and Western Music,
paying tribute to the hybrid
music of his youth. He added horns and swung the tempo. Charles's album went to
number one on the pop charts
that year.

. Page 8 of 36
21. The passage is primarily concerned with:
(a) The rise of Ray Charles as a singer in the 1960's.
(b) The origins of 'Hillbilly' and other racist terms.
(c) The origins of 'Country' music.
(d) The rise and fall of ‘country’ music
22. According to the passage, which of the following was not an inspiration for
themes in early country music?
(a) The influence of Catholic sect in the region.
(b) The cycle of changing seasons.
(c) The various stages of life.
(d) Social issues and causes.
23. Which of the following is the author most likely to agree with?
(a) Early country music was based on protest songs and commentary about social
issues.
(b) Ray Charles was discriminated at the Ryman because he was black.
(c) Radio shows like Grand Ole Opry did not play the music of black artists like
Charley Pride.
(d) Hank Williams was one of the famous country singers and his songs were mostly
about heartbreaks.
24. Which of the following is an example of discriminatory racist practices
employed in the early years of country
music?
(a) Using the term 'rhythm and blues' for the music.
(b) WSM playing music by black artists for white audiences.
(c) Louis Armstrong and his wife not being credited for Jimmie Rodgers's "Blue
Yodel No. 9".
(d) Ray Charles became popular but never got a chance to perform at the Ryman
because of his race.
25. What united the music was a catholicity of origin and an unerring obsession
with life inside the home. Not a
protest song but the reality of that vacant chair." What is being alluded by the
use of the phrase 'vacant chair'?
(a) The families who deserted the region due to anti-Catholic persecution.
(b) The family members who lost their lives in the civil war.
(c) The black people who were killed by the white supremacists and racists.
(d) The black artists who remained unknown because their pictures were not
published on the record sleeves.
Passage (Q.26-Q.30): They look like mirrors: 32 rectangles neatly arranged in eight
rows on the rooftop of a
supermarket called Grocery Outlet in Stockton, California. Shimmering beneath a
bright sky, at first glance they
could be solar panels, but the job of this rig is quite different. It keeps the
store from overheating. Tilted toward
the sun, the panels absorb almost none of the warmth beating down on them; they
even launch some into space,
improving the performance of the systems that keep things inside cold. The feat
relies on a phenomenon called
radiative cooling: Everything on Earth emits heat in the form of invisible infrared
rays that rise skyward. At
night, in the absence of mercury-raising daylight, this can chill something enough
to produce ice. When your
car's windshield frosts over, even if the thermometer hasn't dipped below freezing?
That's radiative cooling in
action. To Aaswath Raman, who was a key mind behind Grocery Outlet's shiny tiles,
that effect seemed like an
opportunity. "Your skin, your roof, the ground, all of them are cooling by sending
their heat up to the sky," he
says.
Raman, is the co-founder of SkyCool Systems, a start-up trying to flip the script
on the technology we depend
on to create chill. As the world warms, demand for air conditioning and
refrigeration is going up. But these
systems themselves expel a tremendous amount of heat, and the chemical compounds
they use can escape
skyward, where they act as a planet-warming greenhouse gas. According to the
Birmingham Energy Institute in
the UK, these substances and the power involved accounted for at least 11 percent
of global greenhouse gas
emissions in 2018. By 2050, more than 4.5 billion air conditioners (A) / 40 percent
of all electricity (B) / are
projected to consume nearly (C) / and 1.6 billion refrigerators (D). If it goes
mainstream, SkyCool's tech -

. Page 9 of 36
and similar approaches in the works from competitors and other researchers - could
slow the cycle by naturally
lowering building temperatures and easing the energy burden on conventional
methods.
26. The rig installed on the rooftop of a Grocery Outlet in Stockton works on a
specific principle. Which of the
following reflects the one?
(a) The feat that is there on the rooftop of a Grocery Outlet works on a phenomenon
called radiative cooling:
the process by which a body absorbs heat to stay warm.
(b) The rig installed on the rooftop of a Grocery Outlet in Stockton works as per
the radiative cooling: the process
by which various bodies absorb heat from the surrounding to keep the environment
cool.
(c) The design works as per the phenomenon of radiative cooling: the process by
which a body loses heat and
cools down.
(d) The device installed on the rooftop of a Grocery Outlet in Stockton relies on
radiative cooling: the process
by which a body absorbs moisture from the environment to keep itself cool.
27. Which 'opportunity' does the statement, ‘To Aaswath Raman, who was a key mind
behind Grocery Outlet's shiny
tiles, that effect seemed like an opportunity’, talk about?
(a) Raman thought he had a chance to build his own company.
(b) By applying the principles of radiative cooling by naturally slowing down the
cycle of rising temperature of
the earth by taking down the burden from mother nature.
(c) By seeing an opportunity to install solar panels over his own grocery store by
applying the principles of
radiative cooling.
(d) Raman saw establishing a manufacturing plant of air conditioners and
refrigerators, as an opportunity.
28. In the given passage, what does the phrase 'mercury-raising' mean?
(a) Moving. (b) Chilly. (c) Lifeless. (d) Hot.
29. In the passage, a sentence is highlighted, divided into four parts and jumbled,
which when arranged will form a
logical and coherent sentence. Rearrange the parts to form a meaningful sentence
and choose the option that
represents the correct arrangement.
(a) BCAD (b) CBDA (c) CDAB (d) ADCB
30. SkyCool Systems, Raman's start-up has the goal to 'flip' which script?
(a) Increase the production of air conditioners and refrigerators.
(b) SkyCool Systems have enabled radiative cooling in special types of air
conditioners and refrigerators.
(c) Overturn our dependency on technologies like air conditioning and refrigeration
to create chill.
(d) Raman has a plan to create new air conditioners and refrigerators using the old
ones.

ECTION - C: LEGAL REASONING


Passage (Q.66-Q.70): In a lawsuit, the de minimis doctrine is applied by a court to
avoid resolving trivial matters
that are not worthy of judicial scrutiny. Its application sometimes leads to an
action being dismissed, especially
when the only redress being sought is for a nominal sum, such as a dollar. When
appropriate, the appellate courts
also use the de minimis doctrine.
A legal term which means too small to be meaningful or taken into account;
immaterial. As a matter of policy,
the law does not encourage parties to bring legal action where the impact of the
breach is negligible for technical
breaches of rules or agreements. De minimis exceptions are commonly included in
contracts to limit the use of
covenants or other restrictions so that they do not apply in circumstances where
the failure to comply with the
restriction has negligible impact.
Not all courts agree when and how the doctrine should be applied. Before making a
decision to apply “de
minimis,” the courts consider the wrong and the amount of harm involved. The maxim
is said to be a pure
“exercise of judicial power and nothing else.”
This maxim is also recognized by Section 95 of IPC. This section is intended to
prevent the punishment of
negligible mistakes or trivial offenses. Whether the act, which amounts to an
offense, is trivial would
undoubtedly depend on the nature of the injury, the party’s position, the knowledge
or intention with which an
offending act is performed, and other related circumstances. Under this provision,
these cases are considered
innocent by the public, even though they fall within the letter of the penal law,
not yet within its spirit, or
throughout the world. In other words, the harm resulting from an offense is so
small and trivial that no person of
ordinary sense and temper would complain about it.
(SOURCE: https://blog.ipleaders.in/)
66. The content of the passage refers to a legal maxim that reads, "The law does
not encourage parties to seek legal
action if the impact of the breach is trivial for technical breaches of rules or
agreements." Which of the following
legal maxim is specifically discussed in the passage?
(a) de minimis non curat lex
(b) de minimis curat non lex
(c) curat non lex de minimis
(d) de minimis lex non curat
67. Rohan was speeding along a sandy road, and his motor wheels kicked up some dust
on Yaman, a pedestrian's
clothing. Rohan's vehicle created so much dust that other drivers couldn't see the
road well, resulting in Yaman
getting hit by another car due to a hazy road. Decide upon the maxim’s application
in this case?
(a) The court will consider the instance of Yaman being hit by another car, and so
it will not be deemed a trivial
matter.
(b) The court will dismiss this as a trivial matter because there is no one to
blame but the sandy road.
(c) If Rohan is dragged into the matter; he can take up the defence of the maxim de
minimis.
(d) The maxim of de minimis can be used to defend the motorist that injured Yaman.
68. In which of the following illustration, the maxim de minimis will not be
applicable:
(a) Ujjawal was involved in food adulteration and sold a food item unfit for human
consumption.
(b) B, a passerby, touched a fireman on the arm to attract his attention to another
part of a burning building; the
fireman filed a battery claim.
(c) A promised B that they would go see a movie together on Sunday. A did not
appear in the theatre on the
given day. As a result, B suffered emotional stress and agony. He filed a lawsuit
against A for monetary
damages.
(d) All of the above.

. Page 17 of 36
69. Nandan has the exclusive right to fish in a particular body of water. Nandan's
brother Chandan casts a fishingnet
therein and pulls it out to get revenge on Nandan. Nandan filed a petition for tort
of trespass. The plaintiff claimed
that he had the exclusive right to fish and that the defendant violated it by
trespassing and fishing. Decide
(a) As it is a trivial matter of retribution between the two brothers, the
plaintiff will not succeed.
(b) Given the defendant's tortious act, the plaintiff will succeed.
(c) The plaintiff will not succeed since the maxim de minimis will apply in this
case.
(d) Uncertain since the decision will be made at the discretion of the court.
70. Assertion: Applying some force to another does not always suggest a criminal
assault. Quite the contrary, there
are many examples of incidental touch that cannot be considered criminal conduct.
Reason: In the application of statutes, the Court is not bound to a strictness at
once harsh and pedantic. Where
there are very slight irregularities, the law does not intend that the penalties
should be inflexibly severe.
(a) Both A and R are true but R is not correct explanation of A.
(b) Both A and R are true and R is correct explanation of A.
(c) A is true but R is false.
(d) A is false but R is true.
Passage (Q.71-Q.76): Information contained in a document, if replicated, can be the
subject of theft and can
result in wrongful loss, even though the original document was only temporarily
removed from its lawful custody
for the purpose of extracting the information contained therein.
The following requirements need to be established in order to make out a case of
theft under Section 378 IPC:
• Dishonest intention
• To take away movable property
• For wrongful gain
• To cause wrongful loss
Whoever, intending to take dishonestly any moveable property out of the possession
of any person without that
person’s consent, moves that property in order to such taking, is said to commit
theft. The offence of theft is
complete a soon as the property is moved in order to such taking. It is not
necessary that the person from whose
possession the property is taken is the true owner or has the real title of the
property.
(Extracted with requisite revisions and edits from ‘Temporary removal of document
for replication of content
can be the subject of Theft, Supreme Court’ at
https://www.barandbench.com/news/temporary-removal-of#document-for-replication-of-
content-amounts-to-theft-supreme-court)
71. Arun, a 14-year old schoolboy, was quite famous for his academic acumen.
However, what people didn’t know
that he was a kleptomaniac too. One day while he visited the house of his friend
Kabir, he saw anewly released
game CD and seeing that he picked it. However, since there was no space or
opportunity to carry the CD, he hid
it inside arug in the room.Thereafter,he left thehouse planning to returnduring
Kabir’s absence.Later, he got
occupied and soon enough forgot about the CD. Is he guilty of theft?
(a) Yes, he is a kleptomaniac and his act of taking the CD will amount to theft and
he will be liable for the same.
(b) No, he is not liable as kleptomania is a condition which requires medical
attention and since it’s an urge to
steal, there is absence of any dishonest intention.
(c) Yes, there is dishonest intention so as to deprive Kabir of his CD and thus
Arun would be liable for theft.
(d) No, Arun did not ultimately take the property i.e. CD withhim and would thus
not be liable for theft.
72. That day Arun had come to study at Kabir’s house and forgot to bring his pencil
box and thus borrowed apen
from him. However, while leaving he forgot to return the pen and instead kept it in
his pocket. Is he guilty of
theft?
(a) No, Arun did not possess any dishonest intention to take away the pen and thus
would not be guilty of theft.
(b) Yes, Arun was a kleptomaniac and thus his acts would fulfil all the ingredients
required to make a person
liable for theft.

. Page 18 of 36
(c) No, since law does not take into account trivial matters the allegationof theft
is likely to get dismissed by the
Court of law.
(d) Yes, there is wrongful loss to Kabir which is sufficient to make Arun liable
for theft.
73. When Arun was going back to his home, he found an expensive ring lying on the
road. He picked the ring
immediately and without inquiring about the owner of the ring kept it in his
pocket. Later, he went home. Is he
guilty of theft?
(a) Yes, he had the dishonest intention to dispossess someone of the valuable ring
causing wrongful loss to such
person and would thus be liable for theft.
(b) No, the property was not in the possession of anyone. Therefore, there is no
theft committed by Arun.
(c) Yes, Arun had the moral and legal obligation to inquire about the rightful
owner of the ring instead of keeping
the ring with himself and is thus liable for theft.
(d) No, Arun was a kleptomaniac and by virtue of the fact that it’s a condition/
urge not within his control, he
did not possess dishonest intention and would not be liable for theft.
74. The next day, the news broke of the missing ring which belonged to a minister’s
wife who came there for an
official visit but their ring was stolen from the hotel which somehow might have
landed on the road. After seeing
the news, Arun thought it best to keep shut and did not return the ring. Is he
liable for theft?
(a) No, the property was not in the possession of anyone at the time it was picked.
Therefore, there is no theft
committed by Arun.
(b) Yes, he had the dishonest intention to dispossess someone of the valuable ring
causing wrongful loss to such
person and would thus be liable for theft.
(c) No, the ring was originally stolen by someone else and therefore Arun cannot be
made liable for the theft
committed by someone else.
(d) Yes, Arun was a kleptomaniac and making him liable for his acts is a way to
improve him and get rid of the
problem.
75. There was a graveyard situated near the house of Kabir. One day while he was
returning from school he found a
shining bracelet over a body about to be buried. He went ahead and took that
bracelet with him. Is he guilty of
theft?
(a) No, there was no dishonest intention as it was taken from a dead body and would
thus not qualify as theft.
(b) Yes, even dead bodies have rights under law and cannot be disposed of the
valuables with which they are
buried.
(c) No, the property was not in the possession of anyone. Therefore, there is no
theft committed by Kabir.
(d) Yes, there is dishonest intention coupled with wrongful gain to Kabir and thus
he would be liable for theft.
76. Arun saw an Ivory tree in the garden of his neighbour. Being fascinated by the
beauty of the tree, he chopped
off few of the branches of the tree and carried it home. Is he guilty of the
offence of theft?
(a) No, tree is an immovable property whereas theft is restricted to movable
property only and therefore there is
no theft committed by Arun.
(b) Yes, as soon as the branches were chopped off those turned into movable
property and thus theft would
occur.
(c) No, there was no dishonest intention as the reason for taking the branches of
the tree was pure fascination
and not ill intention and would therefore not qualify as theft.
(d) Yes, since ivory tree is a valuable tree taking away any part of it is an
offence under State laws and Arun
would be punished under the same.

. Page 19 of 36
Passage (Q.77-Q.80): In Hegarty v. Shine, it was held that mere concealment of
facts is not considered to be a
fraud so as to vitiate consent. Here, the plaintiff’s paramour had infected her
with some venereal disease and
she brought an action for assault against him. The action failed on the grounds
that mere non-disclosure of facts
does not amount to fraud based on the principle ex turpi causa non orituractio i.e.
no action arises from an
immoral cause. In some of the criminal cases, mere submission does not imply
consent if the same has been
taken by fraud which induced mistake in the victim’s mind so as to the real nature
of the act. If the mistake
induced by fraud does not make any false impression regarding the real nature of
the act then it cannot be
considered as an element vitiating consent.
(Source: With revision and edits from Ipleaders.com)
77. Assertion: No action arises from an immoral cause.
Reason: No court will lend its aid to a man who founds his cause of action upon an
immoral or an illegal act.
(a) Both A and R are false.
(b) Both A and R are true and R is the correct explanation of A.
(c) A is true but R is false.
(d) A is false but R is true.
78. Changu and Mangu agreed to sell a plot of land to Bheem, which has already been
sold to Nandu. Both
successfully sold the land to Bheem and accepted token money from him. When Changu
later demanded for his
share, Mangu refused to pay him. Can Changu sue Mangu for not paying him his share
of the profit from the
sale?
(a) Yes, because both parties were aware that the nature of the act was illegal but
yet engaged into an agreement
to pursue a common course of action.
(b) No, because both parties were aware that the act was illegal and immoral, and
hence no action can be brought
in this case.
(c) Yes, because Changu is entitled to his share of profit, and simple hiding of
facts is not deemed a deception
sufficient to void consent.
(d) No, Changu cannot sue Mangu since the court will not hear a claim based on
immoral grounds.
79. Continuing the similar factual situation as stated above in Q12, on what
grounds, can Bheem file a case against
Mangu and Changu for fraud and resulting in void consent?
(a) Yes, as his consent for buying the land has been taken by fraud.
(b) No, Bheem cannot sue Changu and Mangu owning to immoral cause.
(c) Changu and Mangu cannot claim the defence of immoral cause.
(d) Bheem cannot file a case against Changu and Mangu.
80. Mike Ross, a talented young college dropout, is hired as an associate by Harvey
Specter, one of New York's best
lawyers despite being a college dropout who never attended law school. In US
courts, the client's attorney takes
an oath to represent only ethically right clients. Mike took the oath, but it was
later discovered that Mike’s client
was guilty of the crime. The court fined him $100 and sentenced him to one year in
jail. Mike asserted that
because he does not have a license to practice, he cannot be convicted and so
cannot be penalized for swearing
a false oath. Decide
(a) Mike’s claim will be successful as he was never an attorney hence cannot be
prosecuted.
(b) Mike’s claim will not be successful as no one can claim illegality as a
defence.
(c) Mike’s claim will be successful as he did not know that his client was indulged
in unethical activities.
(d) Mike’s claim will be successful as it is not based on immoral cause.

. Page 20 of 36
Passage (Q.81-Q.85): In torts, the object behind remedying a party is to take the
aggrieved party back to the
status or position that they were enjoying before the occurrence of tort. When two
or more persons unite to cause
damage to another person, then they will be liable as joint tortfeasors. All those
who actively participate in the
civil wrong commission are joint tortfeasors. Based on the percentage of damage
caused by his negligent act,
each joint tortfeasor is responsible for paying a portion of the compensation
granted to the complainant.
According to the principle of contribution, the defendant who pays more than his
share of the damages, or who
pay more than he is at fault, may bring an action to recover from the other
defendant. When two or more persons
join together for common action, then all the persons are jointly and severally
liable for any tort committed in
the course of such action. When the same injury is caused to another person by two
or more persons as a result
of their separate tortious acts, this results in several concurrent tortfeasors.
(Source: https://blog.ipleaders.in/law-of-torts)
81. The petitioner (victim) was constantly stalked, and the offender eventually
established a fake account of her and
sent filthy messages to the victim’s acquaintances. A modified necked photo was
also plastered on the wall of
the victim's hostel. A complaint was filed against the tortfeasor for defamation.
Determine whether the violation
will be tried as a civil or criminal offence.
(a) It will be the discretion of the court.
(b) It can be tried in both ways as defamation in tort is a civil crime and in
criminal law a criminal offence.
(c) The offender can be tried for actively participating in a civil wrong.
(d) Cannot be determined.
82. Rahul and Manoj lived in the Star Homes Society, in Houses 4 and 6,
respectively. The plot adjacent to the
society was under construction, resulting in noise and air pollution for the
residents of the star houses. For this
purpose, most house owners in the society installedreflective insulating shields in
front of their houses. Ritesh,
the resident of H.No. 5, was not convinced for fitting the same in his house. In
his absence, Rahul and Manoj
brought the shield for his house as well and had it installed. When Ritesh returned
after a week, he discovered
that the shield’s netwas blocking the home from receiving proper sunlight. He
finally filed a complaint for
trespass against Rahul and Manoj. Decide accountability of Rahul and Manoj as joint
tortfeasors.
(a) As joint tortfeasors, Rahul and Manoj would be held accountable for causing
Ritesh harm.
(b) Rahul and Manoj will not be held liable as joint tortfeasors because they had
no intent to cause Ritesh harm.
(c) Rahul and Manoj will not be held accountable since they did not commit a tort
against Ritesh and had no
intent to cause him harm.
(d) Rahul and Manoj will be held accountable as concurrent tortfeasors.
83. Kapsch Metro JV has commissioned the Delhi Gurgaon Expressway with 3 Toll
Plazas with a total of 59 toll
lanes. The largest toll plaza has a total of 32 + 4 reversible toll lanes. Since
the toll is located at a slanting height,
it daily reports at the least one incident of accident in the lane due to sudden
stoppage of vehicle. One such day,
Ramesh stopped his vehicle behind a truck that had come to a sudden stop. He was
then struck from behind by
a vehicle driven by the defendant Jayesh which was struck by a vehicle driven by
the defendant Mayank. The
exact sequence of the collisions could not be determined with certainty because
they all occurred within a very
short time frame. Can bothbe held liable as joint tortfeasors?
(a) Yes, Jayesh and Mayank will be liable as joint tortfeasors.
(b) Jayesh and Mayank will be liable for negligence for causing damage to
plaintiff’s vehicle.
(c) Yes, bothwill be held liable as joint tortfeasors and based on the percentage
of damage caused by
theirnegligent acts, each joint tortfeasor is responsible for paying a portion of
the compensation granted to
the plaintiff.
(d) No, they will be held liable as concurrent tortfeasors.

. Page 21 of 36
84. Siddharth was beaten black and blue by his seniors for not following their
commands. He was badly injured as
Ramesh broke his leg by hitting it with an iron rod and then Suresh blindfolded him
and pushed him downthe
stairs, resulting in various fractures. Siddharth’s parents filed a complaint
against both Ramesh and Suresh.
Decide.
(a) Both Suresh and Ramesh will be held liable for committing a tort against
Siddharth.
(b) Neither Ramesh nor Suresh will be liable for committing a tort against
Siddharth.
(c) Neither Ramesh nor Suresh will be held liable as they did not participate in
the civil wrong.
(d) Cannot be determined as facts lack information.
85. Assertion: The liability of wrongdoers is joint and several i.e. each is liable
for the whole damage.
Reason: A tortfeasor who has been held liable to pay more than the share of the
damages, can claim contribution
from the other joint tortfeasors.
(a) Both A and R are false.
(b) Both A and R are true and R is not the correct explanation of A.
(c) A is true but R is false.
(d) A is false but R is true
Passage (Q.86-Q.90): The Delhi high court was dealing with a petition, which sought
the filing of ordinary
criminal cases (as opposed to anti-terrorism ones) against certain leaders of the
ruling party, who had created
an alternate ‘Hindu political mass’ in juxtaposition to the ‘Muslim protestors’.
The former have not been
identified thus in popular discourse, although the prosecution in Khalid’s case
repeatedly refers to the latter,
rebuked protestors as ‘Muslim critical masses’. I am choosing to call them that
since they were mobilised in
response to the impliedly Muslim protests.
The protests were painted as being false, fabricated, funded by external agencies
and internal dissenters to
disturb the government, and therefore prima facie ‘anti-national’.
The counter protest on the other hand, was presented as a natural ‘nationalist
response’, which sought to clear
the streets (and political discourse) of anti-nationals. The organisers, amongst
whom was at least one minister
of state, and another sitting Member of Parliament, had consistently, over days,
raised incendiary slogans
against the ‘traitors to the nation’.
The Delhi high court did not see any threat in the mobilisation. It said, almost in
a ‘stream of consciousness’
kind of manner:
“Suppose you have said something just for creating mahaul (atmosphere) and all
this, I think there is no mens
rea because some other political parties say something else. Everybody is
addressing their constituencies and
mobilising their constituents. That speech has been done for the purpose of
mobilising the constituency. If
you’re saying something with a smile then there is no criminality, if you’re saying
something offensively,
then criminality. Because we are also in a democracy…you also have the right to
speech and all these things.”
According to the higher court, there is place in our vibrant democracy for
political and social mobilisations,
but the lower court inexplicably thinks that the same acts constitute a terrorist
offence. We are left no wiser
as to why ‘Muslim critical mass’ is a dangerous thing, while ‘Hindu political
society’ is only a corrective.
The Supreme Court has clarified in the case of PravasiBhalaiSangathan v. Union of
India (2014) that the
harm done by hate speech is not measured by its offensive value alone, but rather
by how successfully and
systematically it marginalises a people: by dubbing them ‘anti-nationals’, by
delegitimising their grievances
as motivated (and externally funded) and by colouring their mobilisation as
dangerous activity. Such speech
that excludes people from political and social spaces is not democratic. In fact,
it causes a ‘democratic
deficit’, because it labels dissenters as ‘anti-nationals’ and therefore outside of
acceptable political discourse.
(SOURCE: The Prima Facie Anti-National, https://thewire.in/law)

. Page 22 of 36
86. Choose a remark that, based on your understanding of the paragraph, would be
considered not an incidence
of hate speech:
I. A Muslim leader giving a speech to mobilize its constituents hat says “La
ilahaillallah; Muhammad –ar#Rasululullah”, and then proceeds to explain that the
phrase means that the Law of Allah is above all and its
usage is evidence of the Muslims’ plotting against secularism, democracy and
nationalism.
II. Governments strive to motivate citizens to vote in elections and other forms of
voting.
III. The court is convinced that the 'Muslim critical mass' has no genuine issues
and is merely looking for trouble.
IV. There is a mention of Muslim students at JNU who express a willingness to
participate in chakkajam in as
many as 50 cities throughout India.
(a) I and III (b) III and IV (c) I and IV (d) II
87. On the 9th of February 2010, several Motilalal Nehru University (MNU) students
staged a demonstration on
their campus in protest of the capital penalty meted out to Fazal Guru, the 2001
Indian Parliament attack convict,
and Saqbool Khan, a Kashmiri separatist. Despite the University administration
revoking permission for the
event just before it was scheduled to begin owing to protests by members of the
student union, the event went
ahead. A video was disseminated by the Indian television station Zew news in which
a small number of people
shouted "anti-India" slogans while wearing masks, according to a later inquiry. The
slogans were criticized by
many individuals, including political leaders and students of MNU. Decide if the
protest can be considered as
antinational?
(a) The protest will be considered prima facie anti national.
(b) The protest will not be taken into account since in a democratic state;
everyone has the right to free
expression.
(c) Because the protest is anti-national in nature, it will be labeled as such.
(d) The protest will not be considered as anti national as anything said
offensively attracts criminality.
88. The CM of Delhi, along with all of his party's followers, began a protest
against the Indian government for
refusing to implement the anti-lokpal law; the protest was led by Anna Hazare, a
well-known political leader.
The demonstration created such a stir across the country that every news station
conducted a public poll to gauge
popular opinion. Is this a protest that is anti-national?
(a) No, since the demonstration was not obnoxious, it was lawful.
(b) Yes, since the protest was insulting with the intent of disrupting public
order, making it anti-national and
illegal.
(c) Yes, because the demonstration was provocative and hence may be classified as
anti-national.
(d) Yes, because the protest's goal was to get the law passed without causing any
disruption to the government.
89. Assertion: Hate speech contributes to a "democratic deficit" by labelling
critics as "anti-nationals" who are
therefore excluded from legitimate political debate.
Reason: According to the higher judiciary, political leaders' mobilization against
anti-nationals does not
constitute a terrorist act.
(a) Both A and R are true but R is not correct explanation of A.
(b) Both A and R are true and R is correct explanation of A.
(c) A is true but R is false.
(d) A is false but R is true.

. Page 23 of 36
90. Aisha Ghoosh is an MNU student pursuing a master's degree in philosophy at
MNU's School of International
Relations. The election of the student union is always the center of attraction for
the nation, as the MNU is
defamed of being an anti-national institution, and various anti-nationalist
protests are held during election
campaigns, which are funded by external agencies in order to disrupt public order
and government. It was also
alleged that the winning candidate is beholden to the influence and desires of
external agencies. As she was the
leader of such a demonstration, Aisha, the chosen candidate for student union, is
now said to be an anti national.
Decide
(a) Aisha won the election fair and square and hence cannot be blamed for being an
anti national.
(b) It can be said that Aisha is mobilizing her constituents and the protests can
be said to be addressing their
constituencies and mobilising their constituents.
(c) Aisha cannot be claimed be anti national as it was just an assertion that the
winning candidate is beholden
to the influence and desires of external agencies.
(d) Aisha will be held as anti national as she was the leader of such protests.
Passage (Q.91-Q.95): After hearing both the sides Court stated that the expression
‘reason to believe’ appearing
in sub-section (1) of Section 82 Cr.P.C. means sufficient cause to believe. Section
26 IPC also explicates that a
person is said to have ‘reason to believe’ a thing if he has sufficient cause to
believe that thing but not otherwise.
Thus, ‘reason to believe’ that a person against whom warrant is issued is either
absconding or concealing himself
should be reflected by the material placed on record before the Court. The use of
expression ‘so that such warrant
cannot be executed’ further implies that the person must be attributed with the
knowledge that such warrant has
been issued against him and his abscondence or concealment is intentional.
HC noted that the issuance of process under Section 82 Cr.P.C. and pronouncing a
person as ‘proclaimed
person’ or PO entail serious consequences, including not only deprivation of
personal liberty of a person, but
also attachment of properties and initiation of proceedings under Section 174A IPC
against such person. HC
stated that no judicial order is complete without reasons and it is expected that
every Court, who passes an order,
should give reasons for the same. From the impugned order, it is further apparent
that the learned ACMM did
not record any reasons for his belief that the petitioner was intentionally
avoiding service/process
After evaluating submissions made by both the parties the Court held that “the
impugned order reflects non#application of mind by the learned Judge, as the
petitioner, who is not accused of any of the offences punishable
under Sections enumerated in Section 82(4) Cr.P.C., could only have been pronounced
a ‘proclaimed person’ if
the process did not otherwise stand vitiated, and not declared PO. Accordingly, the
present petition is allowed
and the impugned order is set aside.”
(Source: latestlaws.com)
91. On the eve of Aisha's birthday, Jhanvi, a teetotaler, got inebriated by her
companions. In good faith, she was
inebriated since all of her pals were drinking and only Jhanvi was left out.
Because everyone had to stay at
Aisha's farmhouse for the weekend, everyone arrived prepared to stay. Jhanvi
received a call from her mother
informing her that her father had had a heart attack, and without waiting to inform
anybody, she grabbed her car
keys and drove to Pune. Jhanvi crashed into another car in the middle of the
highway, killing an entire family of
four. Choose a statement that best fits the principle presented in the passage.
(a) Jhanvi had grounds to believe she shouldn't drive since she was inebriated.
(b) Her pals handed her drinks having sufficient reason to believe that Jhanvi
wouldn't be able to ingest it since
she was a teetotaler.
(c) Jhanvi must have had good grounds to believe she shouldn't drive since she was
inebriated.
(d) Cannot be determined due to a lack of information in the facts.

. Page 24 of 36
92. Continuing with the same circumstances as before, what would the court's ruling
be if the matter is brought
before it?
(a) The court will have grounds to conclude that Jhanvi was inebriated
involuntarily.
(b) The court will not reach a decision unless it has grounds to believe that
something is true.
(c) Jhanvi may have been able to avoid the drive, according to Court.
(d) The court cannot rule Jhanvi a proclaimed person without a valid basis.
93. Sudheer discovered a one-of-a-kind diamond ring beside the train rails. He took
the ring in his hand and kept it
for a while. He had been losing money in his business since he had retained the
ring, so he decided to sell it to
one of his associates, Raja, who he considered unlucky. As soon as he decided to
sell the ring, he discovered that
the police were looking for the same thief who had stolen the royal family's ring.
Fearing being caught, he hurried
to his buddy Rohan and urged him to buy the ring for Rs. 20, which might have cost
hundreds of rupees. Decide
(a) Rohan can be said to have reason to believe that the ring might be a stolen
property.
(b) Rohan should buy the ring as it is being sold at such a low price.
(c) Rohan should take the ring and report it to the police.
(d) It can be said that Rohan might not know that the ring has been stolen but he
has reasonable grounds to infer
that it has been stolen.
94. Choose a statement which illustrates an anomaly from the context of the
passage:
I. Section 26, IPC signifies the existence of such facts and circumstances which
are sufficient in the ordinary
course of nature to believe a thing, but not otherwise.
II. What is sufficient cause in a given case depends upon the facts and
circumstances of each case such as
education, knowledge, intelligence, position and status of the receiver, the time,
place and the host of other
factors.
III. A person can be supposed to know where there is a direct appeal to his senses
and a person is presumed to
have a reason to believe if he has sufficient cause to believe the same.
IV. Section 26 I.P.C. explains the meaning of the words "reason to believe" and
“sufficient cause to believe”.
(a) I & IV (b) II & III (c) II & IV (d) None of the above
95. In a case appellant admittedly was a licensed stamp vendor and he was found in
possession of counterfeit stamps.
Choose a statement in respect section 126 IPC?
(a) Appellant had "knowledge" and "reason to believe" that the stamps which he had
in his possession and which
he was selling or offering to sell, were counterfeit ones.
(b) Appellant had "reason to believe" that the stamps which he had in his
possession and which he was selling
or offering to sell, were counterfeit ones.
(c) Its upon the court to decide whether or not Appellant had "reason to believe"
that the stamps which he had
in his possession and which he was selling or offering to sell, were counterfeit
ones.
(d) Can’t say as facts lack sufficient information.
Passage (Q.96-Q.100): A plea of non est factum is one that the signer of the
document disclaims as his act by
showing that there was not existing at the time of the plea any valid execution of
the document on his part. One
thing which is very apparent is that the defendant’s signature in such a document
is invalid and of no effect. It
must be noted that the protection offered by the law is to ensure that a person is
not held bound by the terms of
a document which the same should not have signed but had signed as a result of
fraud or mistake. The plea of
non est factum applies to those persons who are unable to read, owing to blindness
or illiteracy and who,
therefore, had to trust someone to tell them what they were signing. Also, it is
applicable to those persons who
are permanently or temporarily unable, through no fault of their own, to have
without explanation and real
understanding the purport of a particular document whether that may be due to
defective education, illness or
innate incapacity. A significant point to bear in mind is that the plea belongs to
the realm of civil law of contract

. Page 25 of 36
and not to the realm of criminal law. An important point to, also, bear in mind is
that the plea of non est factum
can be employed both as a sword and shield, as it can be the basis of an action as
well as a defense to an action.
(Source: https://www.bhu.ac.in/law/blj/Banaras%20Law%20Journal%202020%20Vol
%2049%20No.%202.pd)
96. Which of the following statements depicts the essentials of plea of non est
factum?
I. A heavy onus is placed on the person who pleads non est factum to prove that not
only was there a lack of
consent but also there was no lack of negligence.
II. There must be fraudulent misrepresentation as to the character of the document
executed.
III. The plaintiff-appellant never intended to sign what she did sign.
IV. The plea of non est factum can be used only as a shield for protecting innocent
signatories to a contract and
cannot act as a sword to harm the interests of innocent third party.
(a) All of the above (b) II & IV (c) II & III (d) I, II & III
97. Which of the following statements is incorrect?
(a) The Latin phrase non est factum literally means "it is not his/her deed”.
(b) It is not a defense in contract law to allow a person to avoid the stipulations
in a contract that she may have
signed because of certain reasons such as mistake as to the kind of contract.
(c) The doctrine provides a remedy when a document has been forged; when it is
quite "literally" true to say that
"it was not my deed".
(d) The doctrine is also applied to situations where someone had signed due to
fraudulent representations or had
not an inkling of an idea as to what she was signing.
98. Ram (defendant) had signed a document stating that he had received money from
the plaintiff as full settlement
for his land. He later refused to execute the transfer document and the plaintiff
sued for specific performance.
The defendant said that he had not read the document, as he was illiterate. He
alleged that it had been explained
to him as being a document concerning a loan by the plaintiff to the defendant to
purchase tools and equipment
to build a house on the land as a joint enterprise. Decide
(a) Ram can claim defense of non est factum.
(b) Plaintiff can claim defense of non est factum.
(c) Ram cannot claim defense of non est factum.
(d) Plaintiff cannot claim defense of non est factum.
99. A, the property owner, issued a power of attorney authorizing his mother B to
represent him in all matters during
his stay in the United States. C, A's sister, duped B into signing a document
transferring the property to C. The
property was then mortgaged by C. In this case, the mother pleaded non est factum,
contending that the transfer
and subsequent charge on the property were null and void from the start. What will
be the ruling of the court?
(a) B cannot use the defense since she was tricked by C and is not an illiterate.
(b) B can effectively assert the non est factum defense since she did not know the
nature of the document that
she signed.
(c) B can validly assert non est factum since she was unaware of the contents of
the documents.
(d) B's claim will be successful, and signed documents are void and invalid.
100. Assertion: ignorantia facti excusat ignorantia juris non excusat.
Reason: Defence of Non est factum is based on mistake of facts.
(a) Both A and R are true but R is not the correct explanation of A.
(b) Both A and R are true and R is correct explanation of A.
(c) A is true but R is false.
(d) A is false but R is true.

. Page 26 of 36
Passage (Q.101-Q.105): "A mere apprehension of a breach of law and order is not
sufficient to meet the standard
of adversely affecting the "maintenance of public order"", the bench comprising
Justices DY Chandrachud and
Surya Kant observed while quashing a detention order.
In this case, an order of detention was passed against the detenu on 19 May 2021
under the provisions of Section
3(2) of the Telangana Prevention of Dangerous Activities and White Collar or
Financial Offenders Act 1986.
The High Court's writ jurisdiction under Article 226 extends to protecting the
personal liberty persons who have
demonstrated that the instrumentality of the State is being weaponized for using
the force of criminal law.
The order of detention was challenged before the High Court in a petition under
Article 226 of the Constitution.
The Division Bench of the High Court dismissed the petition.
In appeal, the bench noted that the order of detention was passed nearly seven
months after the registration of
the first FIR and about five months after the registration of the second FIR. The
court, therefore, observed:
In this case, the apprehension of a disturbance to public order owing to a crime
that was reported over seven
months prior to the detention order has no basis in fact. The apprehension of an
adverse impact to public order
is a mere surmise of the detaining authority, especially when there have been no
reports of unrest since the detenu
was released on bail on 8 January 2021 and detained with effect from 26 June 2021.
The nature of the allegations
against the detenu are grave."
The bench observed that the two FIRs which were registered against the detenu are
capable of being dealt by the
ordinary course of criminal law. The court further observed:
"The personal liberty of an accused cannot be sacrificed on the altar of preventive
detention merely because a
person is implicated in a criminal proceeding. The powers of preventive detention
are exceptional and even
draconian. Tracing their origin to the colonial era, they have been continued with
strict constitutional safeguards
against abuse. Article 22 of the Constitution was specifically inserted and
extensively debated in the Constituent
Assembly to ensure that the exceptional powers of preventive detention do not
devolve into a draconian and
arbitrary exercise of state authority. "
Source: Mallada K Sri Ram vs State of Telangana | 2022 LiveLaw (SC) 358 | CrA 561
of 2022 | 4 April 2022:
Coram: Justices DY Chandrachud and Surya Kant)
101. Chandan and his father Nandan have been apprehended by the police for keeping
their store open even after the
curfew time starts. They could have been given a warning but were arrested and
detained by the police authorities
for several days. Choose statement which best describes the context of the passage
in relation with the facts
given.
(a) The arrest will be deemed illegal and unlawful.
(b) The arrest will be legal because both Chandan and Nandan were in violation of
the curfew by keeping their
stores open over the curfew hours.
(c) Despite the fact that the father-son duo had been wrongfully held, they are
certain to have broken the law.
(d) The authorities should have a compelling reason to apprehend the duo based on a
fear of a disruption of
public order.
102. Select a statement that best illustrates the passage's context:
(a) Constitution of India safeguards under Article 22 Protection against arrest and
detention in certain cases.
(b) For invoking the Writ under Article 226 of the constitution, it is necessary to
illustrated how the state's
instrumentality is being deployed in order to use the power of criminal law.
(c) There may or may not be an imminent fear of a disruption to public order as a
result of a crime for the
purpose of detention.
(d) Article 22 of the Constitution guarantees that the extraordinary powers of
preventative detention do not
descend into a harsh and arbitrary use of state power.

. Page 27 of 36
103. Chandini, a member of the SC/ST community who was detained by police as a
preventive measure, was held in
police custody for several days because she was too poor to pay a counsel. She
didn’t have any family; therefore
no one came to her aid. Reena, a newcomer to the legal field, took up Chandini’s
case. What will Reena's next
line of action as a Chandini lawyer be?
(a) The detaining authority was blissfully ignorant and completely unaware of the
detenus' rights and the
accompanying responsibility imposed on them.
(b) It can be claimed that the detaining authority, by failing to inform the
detainees of their constitutional rights,
had hindered them from making effective representation at the earliest opportunity,
resulting in a breach of
their constitutional right under Article 22 of the Constitution.
(c) Since no legal representative requested the release of Chandini, the police
have no right to release the
detainee until a legal representative arrives.
(d) The Supreme Court has established, as a general norm that the detaining
authority is required to inform the
detainees of their specified rights, and failure or neglect to do so would void the
orders of custody.
104. Assertion: Once the detenus' said constitutional right i.e., Personal liberty
is held to have been infringed, the
question, as to whether it caused them prejudice or not, does not arise at all.
There is inbuilt prejudice in the
infringement of the said right itself.
Reason: Mere failure or omission to apprise the detenus of their aforesaid rights
would result in invalidating the
detention order.
(a) Both A and R are true but R is not correct explanation of A.
(b) Both A and R are true and R is correct explanation of A.
(c) A is true but R is false.
(d) A is false but R is true.
105. Dheerubhai, an infamous don who has been accused of being engaged in several
illicit drug dealings, has already
been granted anticipatory bail, which implies that the authorities cannot arrest
him if any drug-related case arises
in future. Dheerubhai was travelling to the Maldives when he was stopped from
flying overseas and his passport
was taken by officials on the grounds that he was a proclaimed felon. As a result,
he missed his trip and lost 2
million pounds by not being able to attend the meeting and seal the contract. Which
of the following arguments
will work most in Dheeru Bhai's favour?
(a) There has been no report of a crime committed by Dheeru Bhai, and he has also
been granted bail.
(b) The airport authority acted outside of its authority, and has no right to
prevent someone from travelling
abroad.
(c) The detaining authority's fear of a negative influence on public order is
purely speculative.
(d) An accuser’s personal liberty cannot be sacrificed on the altar of preventative
detention just because he or
she is involved in a criminal action.

. Page 28 of 36
SECTION - D: LOGICAL REASONING
Directions (Q.106-Q.135): Read the passage carefully and answer the following
questions.
Passage (Q.106-Q.110): [a] The Karnataka High Court has once again come to the
rescue of Bengalureans by
putting a stop to protests and processions on city roads. [b] It has rightly
restricted such activity to Freedom Park,
the venue designated for demonstrations. For two days in the past week, the
Congress padayatra demanding
implementation of the Mekedatu reservoir project held the city to ransom,
gridlocking the roads and leaving
citizens in distress. [c] That the court had to take up the issue and pass
strictures is a comment on the insensitivity
of the party to people’s hardship. This propensity to make political statements at
the cost of the common man is
not limited to any one party, and neither is it the first time that the HC has had
to intervene to bring order to
Bengaluru’s roads in the aftermath of a rally. As far back as 2008, a JD(S) rally
had paralysed the city, prompting
the judiciary to propose guidelines for traffic management. Unfazed, the party
repeated its feat, leading to
possibly the worst gridlocks the city had seen, with school children stranded on
roads till late evening.
At a time when the collective mindspace is occupied by students stranded in war-hit
Ukraine, and more recently,
the hijab row, the Mekedatu rally will not find much resonance with the public. [1]
The Congress appears to
have miscalculated and certainly does not have its finger on the people’s pulse.
[2] If anything, the padayatra
appears jinxed: it had run afoul of the HC for blatant violation of Covid-19 norms
in January, and this time for
causing inconvenience to the public. The court is only completing the task it took
up a year ago when it suo motu
initiated a PIL petition on huge traffic jams caused due to frequent protests on
public roads. [d] The Calcutta and
Rajasthan HCs have made similar observations and imposed restrictions on political
rallies, which only shows
that our elected representatives need the courts to prevent them from acting in an
irresponsible manner.
106. In the given passage, which of the following has been assumed by the author?
(a) Blocking the streets of the city would pressurize the sitting government to
meet the demands.
(b) Protesting in the venue designated for demonstrations would not affect the life
of the general public.
(c) High Courts are the highest authority as far as the state of Karnataka is
concerned.
(d) Public expects the elected representatives to act in a responsible manner, and
situation of courts intervening
should not be recurrence.
107. Out of the following options, select the one that strengthens the arguments in
the passage?
(a) A political roadshow by a political party caused the death of a school boy who
reached hospital late due to
traffic jams.
(b) The decree given by the Calcutta and Rajasthan High Courts don’t hold much
value in Karnataka.
(c) The opposition party holds the lion’s share (close to 95%) of the number of
illegitimate protests that take
place each year.
(d) The Mekedatu reservoir project is projected to provide direct benefits to 60%
of Karnataka’s population.
108. The author of the passage would most agree with which of the following
options?
i. The Mekedatu reservoir project does not hold much significance with the public
in the present context.
ii. High Courts are the last resort for the general public when the politicians
hold the cities at ransom.
iii. The Congress party has a fair idea about the mood of the public of the city of
Bengaluru.
(a) Only I (b) Only i and ii
(c) Only i and iii (d) All i, ii and iii
109. The relationship between the given two statements [1] and [2] is best
represented by which of the following
options?
(a) Statement [1] is the hidden premise based on which statement [2] is claimed.
(b) Statement [2] is the premise based on which statement [1] is claimed.
(c) Statement [2] is based on statement [1], but it is not the hidden premise.
(d) Statement [2] is the hidden premise based on which statement [1] is claimed.

. Page 29 of 36
110. Select the statement out of the many statements lettered as (a), (b), (c) and
(d) given in the passage that is the
odd one out. Use the concepts of logical reasoning to arrive at your decision.
(a) [a] (b) [b] (c) [c] (d) [d]
Passage (Q.111-Q.115): Americans were primarily rural people in the early 19th
century. Cities had few
restaurants until the 1830s and 1840s. Most that did exist were for very rich
people. It took the emergence of a
new urban life to spark the creation of the kind of eating and drinking
establishment that would enshrine the
straw in American culture: the soda fountain.
P2. Carbon dioxide had been isolated decades before, and soda water created with
predictably palate-pleasing
results, but the equipment to make it was expensive and unwieldy. It wasn’t until
the gas was readily available
and cheap that the soda fountain became prevalent. In the 1870s, their technical
refinement met a growing market
of people who wanted a cold, sweet treat in the city.
P3. At the same time, the Civil War had intensified American industrialization.
More and more people lived in
cities and worked outside the home. Cities had saloons, but they were gendered
spaces. As urban women fought
for greater independence, they, too, wanted places to go. Soda fountains provided a
key alternative. Given the
female leadership of the late-19th-century temperance movement, soda fountains were
drafted onto the side.
Sodas were safe and clean. They were soft drinks.
P4. By 1911, an industry book proclaimed the soda fountain the very height of
democratic propriety. “Today
everybody, men, women and children, natives and foreigners, patronize the fountain”
said The Practical Soda
Fountain Guide.
111. Which of the following can be inferred from paragraph one?
(a) 1830s and 1840s were considered the early nineteenth century.
(b) The restaurants in the early nineteenth century catered to the affluent class.
(c) Fountain soda was the result of the emergence of urban and rural life in the
1830s and 1840s.
(d) The fountain soda was the result of the isolation process of carbon dioxide
decades ago.
112. Which of the following will be in dissonance with author’s thought-process?
(a) Fountain soda reflected the liberal activism.
(b) Fountain soda symbolised the American industrialism, the creation of urban
life, and the changing gender
relations.
(c) Part of the popularity of the fountain soda was its cold and sweet disposition.
(d) Soda was patronised by the environmentalists because it was safe and clean
option.
113. ‘It took the emergence of a new urban life to spark the creation of the kind
of eating and drinking establishment
that would enshrine the straw in American culture: the soda fountain.’ The above
statement is a/an
(a) Fact. (b) Inference. (c) Assumption. (d) Judgment
114. What role does paragraph four play in the overall organisation of the passage?
(a) It serves as a conclusion to paragraph three.
(b) It serves as a conclusion to the overall orgainsation of the passage.
(c) It serves as an illustration to reinforce the stance of the author.
(d) It is an isolated illustration that is a precedent to the following arguments.
115. Which of the following enquiries would not yield an answer from the given
passage?
(a) From the perspective of the fountain soda, what is the relevance of modern
capitalism?
(b) What led to the fountain soda become the very height of democratic propriety by
the political parties?
(c) What made fountain soda the essence of the American urban life?
(d) How did soda fountain become instrumental in abridging the gender gap?

. Page 30 of 36
Passage (Q.116-Q.120): Turf wars and bloodshed are synonymous with West Bengal
politics, particularly
during the election season, with successive governments often fanning the flames
rather than dousing them. The
killing of eight persons, including two children, in Birbhum district —apparently
in retaliation for the murder of
a Trinamool Congress (TMC) panchayat leader — adds another tragic chapter to this
never-ending saga of
violence. The Calcutta High Court has ordered a CBI investigation into the gruesome
incident, even as Chief
Minister Mamata Banerjee — in the damage-control mode — got a TMC block president
arrested and pulled up
the local police for not acting in time. The Birbhum horror has provided the BJP,
the state’s main Opposition
party, ample ammunition to target the ruling TMC. Even though the TMC has promised
to cooperate with the
CBI, the fierce inter-party rivalry and Centre-state tussle seen in recent years do
not bode well for a free and fair
probe.
The Left, during its decades-long rule in Bengal, banked on violence and coercion
to force its rivals into
submission. It was Birbhum that had witnessed the lynching of 11 labourers in July
2000 during Jyoti Basu’s
tenure, while more than 20 lives were lost in clashes during the 2008 panchayat
polls. The Mamata-led TMC has
inherited this unenviable legacy of repression. With the BJP gaining ground in
Bengal on the back of its strong
performance in the 2019 Lok Sabha elections, the two parties have frequently locked
horns, right from the top
brass to the grassroots-level workers. Last year’s Assembly elections were also
marred by bloodshed, with four
voters falling to bullets fired by Central security personnel.
The onus is on all stakeholders to discourage the culture of violence that is not
only a blot on Bengal but is also
impeding the state’s progress. It needs political will to rein in the cadres and
uphold the rule of law. With warring
parties relying on muscle power to silence their detractors, it would be a
Herculean task to weed out the criminal
elements and help Bengal turn over a new leaf.
(Source - https://www.tribuneindia.com/news/editorials/birbhum-killings-380770)
116. What is the central idea of this passage?
(a) The high court ruling brings again the issue of political violence in Bengal.
(b) Culture of violence is the bane of Bengal politics and needs to be stopped.
(c) The burden rests on all stakeholders to stop violence in Bengal.
(d) Birbhum horror shows the sad state of affairs in Bengal.
117. Which of the following, if true, weakens the author’s argument?
(a) Other states face more political violence than West Bengal, and are known for
more gruesome acts than West
Bengal.
(b) There are some parties that do not use violence in west Bengal.
(c) The cases of bloodsheds during electoral process have been on a steady decline,
with violence in Birbhum
district being an unprecedented case.
(d) Bengal has the greatest number of deaths due to political violence amongst all
states.
118. The author is most likely to agree with which of the following statements?
(a) All the parties involved in such incidents should be debarred from contesting
elections.
(b) The people involved in such killings should be given the taste of their own
medicines.
(c) People should kindly request them to abandon such practices of killings.
(d) Political parties should themselves discourage such practice.
119. What purpose does the boldfaced statement in the passage serve?
(a) It is a premise used to support author’s argument.
(b) It is one of the arguments of the author which have been later substantiated.
(c) It is the premise to support the main idea of the passage.
(d) It is a claim that is refuted by the author.

. Page 31 of 36
120. According to the passage, which of the following can be a course of action to
reduce Political violence?
(a) All parties and people involved in the political process should discourage acts
of violence.
(b) State of West Bengal should bring in CBI to probe the offences.
(c) The court should take cognizance of the matter and should give orders.
(d) All political parties’ antisocial elements should be incriminated during the
times of elections.
Passage (Q.121-Q.124): The man who seeks to create a better order of society has
two resistances to contend
with; one that of Nature, the other that of his fellow-men. Broadly speaking, it is
science that deals with the
resistance of Nature, while politics and social organization are the methods of
overcoming the resistance of men.
The ultimate fact in economics is that Nature only yields commodities as the result
of labour. The necessity of
some labour for the satisfaction of our wants is not imposed by political systems
or by the exploitation of the
working classes; it is due to physical laws, which the reformer, like everyone
else, must admit and study. Before
any optimistic economic project can be accepted as feasible, we must examine
whether the physical conditions
of production impose an unalterable veto, or whether they are capable of being
sufficiently modified by science
and organization. Two connected doctrines must be considered in examining this
question: First, Malthus'
doctrine of population, and second, the vaguer, but very prevalent, view that any
surplus above the bare
necessaries of life can only be produced if most men work long hours at monotonous
or painful tasks, leaving
little leisure for a civilized existence or rational enjoyment. I do not believe
that either of these obstacles to
optimism will survive a close scrutiny. The possibility of technical improvement in
the methods of production
is, I believe, so great that, at any rate for centuries to come, there will be no
inevitable barrier to progress in the
general well-being by the simultaneous increase of commodities and diminution of
hours of labour.
This subject has been specially studied by Kropotkin, who, whatever may be thought
of his general theories of
politics, is remarkably instructive, concrete and convincing in all that he says
about the possibilities of
agriculture. Socialists and Anarchists in the main are products of industrial life,
and few among them have any
practical knowledge on the subject of food production. But Kropotkin is an
exception. His two books, **The
Conquest of Bread" and "Fields, Factories and Workshops." are very full of detailed
information, and, even
making great allowances for an optimistic bias. I do not think it can be denied
that they demonstrate possibilities
in which few of us would otherwise have believed.
Malthus contended, in effect that population always tends to increase up to the
limit of subsistence, that the
production of food becomes more expensive as its amount is increased, and that
therefore, apart from short
exceptional periods when new discoveries produce temporary alleviations, the bulk
of mankind must always be
at the lowest level consistent with survival and reproduction. As applied to the
civilized races of the world, this
doctrine is becoming untrue through the rapid decline in the birth-rate; but, apart
from this decline, there are
many other reasons why the doctrine cannot be accepted, at any rate as regards the
near future. The century
which elapsed after Malthus wrote, saw a very great increase in the standard of
comfort throughout the wage#earning classes, and, owing to the enormous increase in
the productivity of labour, a far greater rise in the
standard of comfort could have been effected, if a more just system of distribution
had been introduced. In former
times, when one man's labour produced not very much more than was needed for one
man's subsistence, it was
impossible either greatly to reduce the normal hours of labour, or greatly to
increase the proportion of the
population who enjoyed more than the bare necessaries of life.
121. Which of the following is this passage chiefly concerned with?
(a) To assert that it is not possible either to reduce the hours of labour or to
improve the general wellbeing of
the masses.
(b) To reinforce the general resistances that the reformer has to contend with if
he seeks to create a better order
of society.
(c) To argue that the perceived general resistances that a reformer has to contend
with are sustainable on scrutiny.

. Page 32 of 36
(d) To argue that the obstacles to optimistic economic projects can be sufficiently
altered to reduce labour and
to promote general wellbeing.
122. "An unalterable veto" as used in the passage would most likely be illustrated
by which of the following?
(a) A part of the desert where irrigation is not possible.
(b) A mountainous region with several rivers.
(c) A part of land covered by snow for several months.
(d) A city with a high population density.
123. The writer is likely to agree with which of the following?
I. Kropotkin was not without knowledge on the subject of food production.
II. Men will have little leisure if surplus above bare necessaries are to be
produced.
III. A better standard of comfort could be effected through an equitable
distribution system.
(a) I only (b) I and III only
(c) II only (d) II and III only
124. Based on the details available in the passage, which of the following will
make Malthus doctrine of population
less supportable?
(a) A just and efficient distribution system failed to improve the standard of
living of the working classes.
(b) The current level of population on earth does not pose any threat either to the
planet or to mankind.
(c) Increase in productivity was followed by a remarkable improvement in the
overall standard of living.
(d) The bulk of the mankind remains at the lowest level of existence concerned only
about their survival.
Passage (Q.125-Q.129): Empowering citizens has been a persistent challenge for
democratic polities. Over the
years, most democracies, including India, have usually adopted a rather
instrumental approach to resolving the
problem — through doles. The Indian experience is singular in this respect.
Governments, irrespective of
political affiliations, promise freebies before every election. These include
pledges to provide education,
employment, healthcare and such other basic amenities for the people. Electoral
pledges are not the problem per
se. What must, however, be kept in mind are two issues. First, these promises, more
often than not, are broken
by those in power, to be resurrected before every electoral contest. Second, the
packaging of these promises
reveals the asymmetrical relationship between politicians and people. This
imbalance has an embedded element
of condescension: it is suggestive of the political class using — weaponizing —
amenities that are, in essence,
people’s rights.
This problematic relationship between doles and democracy was highlighted by
Priyanka Gandhi Vadra recently.
During a virtual rally in Uttar Pradesh, she stated, quite correctly, that the
future — be it of children or that of
the nation — cannot be built on free rations. Meaningful empowerment needs serious
investment. The
consequence of this paternalism, as Ms Vadra pointed out, is there for all to see:
the transition of public
entitlements into a form of political capital that can end up weakening collective
opposition and, in turn,
perpetuate the grip of people’s representatives on power. What this necessitates is
reflection on the nature of
democracy’s compact on the part of the people — politicians, who have profited from
this lopsided arrangement,
are unlikely to be in favour of changing it. There should be a greater, collective
endorsement for a culture of
accountability. This means that leaders, no matter how lofty their rhetoric should
be held accountable for their
failure to provide rudimentary facilities. And also for curating what they are
mandated to do as a form of service.
The need for contemplation on doles and democracy has a contemporary resonance.
India’s ruling regime
accuses parties interested in the upliftment of minorities of ‘appeasement’. But
how is it any different from
making — and then breaking — commitments towards welfarism?

. Page 33 of 36
125. Which among the following is the best representation of the central idea of
the passage?
(a) There issue of doling out freebies in a democracy during electoral pledges that
need to be addressed.
(b) Weaponizing amenities that are people’s rights is practical in a democratic
setup.
(c) The relationship between doles and democracy is a complex one to understand.
(d) The people of India deserve their politicians since they want free amenities
only.
126. The arguments of the author require which among the following to be assumed?
(a) Priyanka Gandhi Vadra does election rallies only in the state of UP.
(b) The basic rights of people cannot be handed out as privileges.
(c) Public entitlements have always been turned into weapons by the politicians.
(d) Freebies democracy are intertwined in the very nature of the ideology.
127. With which of the following would the author of the passage logically agree
with?
(a) Leaders with more public support should be given harsher punishments on
default.
(b) The general public should be made aware of the fact that the promises of the
politicians are half-hearted.
(c) In elections, the politicians have been weaponizing amenities to which the
general public is entitled.
(d) Empowering citizens has been a persistent challenge for democratic polities
which consider freebies as bane.
128. Out of the following, which one is an expression of an opinion rather than a
fact?
(a) The rise of paternalism has led to the breakdown of democracy.
(b) The view of Priyanka Vadra on the relation between doles and democracy is
informed.
(c) Governments, irrespective of political affiliations, promise freebies before
every election.
(d) Education, employment, healthcare and such other basic amenities for the
people.
129. What among the following roles would the following statement play towards the
passage? “The practice of
making the same promises in every election and not fulfilling them saves a lot of
work for the politicians.”
(a) This statement would strengthen the arguments of the author.
(b) This statement would weaken the arguments made in the passage.
(c) This statement would strengthen the argument of Mrs. Vadra.
(d) This statement is an isolated argument that is unsupported in the passage.
Passage (Q.130-Q.134): Achieving a commendable milestone, India’s exports have
crossed the $400-billion
mark in the current financial year that ends next week. The previous record was
$330.07 billion in 2018-19, but
the exports soon dipped to $292 billion in the Covid-hit fiscal 2020-21. The
upswing is being attributed to a
healthy performance by sectors such as petroleum products, engineering, gems and
jewellery, chemicals,
pharmaceuticals and fabrics. The welcome trend shows that the ‘Make in India’
initiative is on the right track
and our MSMEs (micro, small and medium enterprises) are making a strong recovery
after the upheaval caused
by the pandemic. It is laudable that India has set itself an ambitious but
achievable export target of $450-500
billion in 2022-23 and is making a push for fresh free trade agreements and trade
concessions with major
economies and regional blocs.
India’s imports in 2021-22 have risen to $589 billion so far, making the trade
deficit surge to around $189 billion.
Last month, the country’s imports increased by 36 per cent, with the value of
inbound shipments of petroleum
and crude oil shooting up by 69 per cent in the backdrop of the Russia-Ukraine
conflict. The ever-widening gap
between imports and exports is a cause for concern. The overall trade deficit,
which was $102 billion in the
2020-21 financial year, is estimated to cross $200 billion in 2022-23. The
appreciable trade deficit with China,
one of India’s top business destinations, underlines the skewed state of affairs.
The production-linked incentive schemes for 13 key sectors of manufacturing seem to
be the best bet to reduce
the import bill and boost exports. With the war in eastern Europe causing trade
disruptions, the government is

. Page 34 of 36
finding it tough to cater to the burgeoning domestic as well as global demand. The
resurgence of Covid-19 in
several Asian and European countries is another stumbling block. The challenge lies
in ensuring effective
coordination among various stakeholders — right from district-level production
units to the overseas market —
and dealing with the bottlenecks on priority.
130. what is the central idea of this passage?
(a) Increasing exports is a welcome and commendable trend but reducing trade
deficit remains a challenge
(b) India has achieved a commendable milestone by crossing 400 billion mark.
(c) India's achievement looks great but it isn't really as the trade deficit has
also surged.
(d) India needs to manage its trade deficit and ensure coordination among various
stakeholders.
131. Which of the following can be inferred from the above passage?
(a) There has been an increase in India's export which will result in higher
revenue.
(b) Covid 19 pandemic has extremely affected the economy of various countries,
including India.
(c) India is on the way of economic recovery which will continue further.
(d) India's exports have risen showing that the initiatives of government are on
good track.
132. Which of the following, if true, weakens the author’s argument?
(a) Exports are increasing, but there are various issues that India still needs to
tackle.
(b) Maximum exports by India are done by large scale enterprises.
(c) India is facing trade deficit which is going to be trouble for India.
(d) India will face the adverse consequences of Russia Ukraine conflict.
133. The author is most likely to agree with which of the following action by the
government: -
(a) Stop trading with China to prevent the trade deficit.
(b) India should pay attention to increasing trade deficit.
(c) India should ensure coordination among different market players at micro and
macro levels.
(d) India should invest more in MSME and should help them more in economic recovery
134. What purpose the boldfaced statement in the passage serves?
(a) It is the argument of the author which later has been substantiated in the
passage.
(b) It is the premise of the author to support the arguments in the passage.
(c) To give a context and background information about India’s exports.
(d) It's a counter premise which, later, has been rebutted by the author.

MOCK 42
Part of the Most Comprehensive & Consistently Successful Study Material & Test
Series Module, spanning across
both Physical and Online Programs in the entire Country. As a result, LegalEdge was
able to engineer Clean-Sweep#Landslide figures of a handsome 64 Selections under
100 ranks, and a whopping 273 selections under 500 ranks in
CLAT 2021. With AILET being no different, a total of 34 of our students found their
way into NLU, Delhi in 2021.
In a nutshell, every second admit in a Top National Law School in 2021 came from
the LegalEdge Preparation
Ecosystem.
MOCK COMMON LAW ADMISSION TEST 2022
MOCK CLAT #42
Duration : 120 Minutes
Max. Marks : 150
Centre Name : __________
Candidate Name : _____________
Batch : _____________
Contact No. : _____________
INSTRUCTIONS TO CANDIDATES
1. No clarification on the question paper can be sought. Answer the questions as
they are.
2. There are 150 multiple choice objective type questions.
3. There is negative marking of 0.25 for every incorrect answer. Each question
carries ONE mark. Total marks are
150
4. You have to indicate the correct answer by darkening one of the four responses
provided, with a BALL PEN
(BLUE OR BLACK) in the OMR Answer Sheet.
Example: For the question, "Where is the Taj Mahal located?", the correct answer is
(b).
The student has to darken the corresponding circle as indicated below:
(a) Kolkata (b) Agra (c) Bhopal (d) Delhi
Right Method Wrong Methods
5. Answering the questions by any method other than the method indicated above
shall be considered incorrect and
no marks will be awarded for the same.
6. More than one response to a question shall be counted as wrong.
7. Do not write anything on the OMR Answer Sheet other than the details required
and, in the spaces, provided for.
8. You are not required to submit the OMR Answer Sheet and Test Paper after the
test.
9. The use of any unfair means by any candidate shall result in the cancellation
of his/her candidature.
10. Impersonation is an offence and the student, apart from disqualification, may
have to face criminal prosecution.
11. You have to scan the QR code only after completion of offline test.
12. You cannot leave the examination hall without punching your answers on the
portal.
Scan this code after the test
to punch in your answers
(Test ID: 2509571)

. Page 2 of 40
SECTION-A: ENGLISH LANGUAGE
Directions (Q.1-Q.30): Read the following passage carefully and answer the
questions that follow.
Directions (Q.1-Q.5): Read the passage carefully and answer the questions.
In ancient Greece, Stoic philosophers believed the Earth would be periodically
destroyed by fire in a ritual
cleansing before starting again. Stoics usually believed this event occurred when
civilisation was at its very
height of sophistication and complexity like ours is now. In an echo of the modern
environmental movement,
Stoics believed that when the intact and perfect balance of nature called Gaia was
interfered with, then collapse
was inevitable. Roman Stoic, playwright and political adviser Seneca believed the
apocalypse would take the
form of a flood. James Romm, in his excellent biography of Seneca, wrote of
Seneca’s unease as Rome expanded
beyond its territorial boundaries, “As in the biblical tale of the Tower of Babel,
the very complexity of civilisation
seemed to carry the seeds of its own destruction. Where once a single ship had
disturbed the natural order, Rome
had now filled the seas with traffic, scrambling the races and dissolving global
boundaries. In Seneca’s view, the
ceaseless advance of the empire would turn the cosmos itself into an enemy. When
everyone could go
everywhere, when no boundary remained intact, total collapse might not be far off.”
The nostalgia for summer's past is a trap. If we desperately wish to get back to
the past, if we think it’s even
possible to return, then we are deluded. It’s been coming at us for some time,
hundreds of thousands of reports
predicting this disorder. The latest this week came from the Intergovernmental
Panel on Climate Change which
said along with more extreme weather events, there was “very high confidence” that
some natural systems had
already experienced irreversible change. But for those of us in this age of
collapse, trapped in a mental construct
of what summer should look and feel like, we are surprised and appalled when the
predictions become reality.
A denial of reality is baked into the rhetoric. This week the New South Wales
premier, Dominic Perrottet,
described the Lismore floods as a “one-in-1,000-year event”, as if it was a freak
occurrence, a matter of timing,
unlikely to happen again for another 1,000 years. There is a macabre comfort in
this, that we will not live through
this again – that is, until we realise these one-in-1000-year events keep
happening, they keep piling up before
our very eyes, daring us to wake up from our dreams and longings of normalcy. What
more will it take?
To look the climate crisis in the face, to see it for what it is, to stop believing
this is a temporary aberration and
that we will go back to those long, succulent, perfect summers, is to take the
first steps in addressing the climate
problem. Because there is a problem. The problem is both climate change itself and
our mindset: that the last
few years are abnormal, that one day, hopefully soon, we will return to “normal” –
and that golden seasonal
rhythm we knew in the past will return unbroken.
1. What is the first step to address the climate problem?
(a) By shaking off the fact that we wouldn’t live long enough to encounter the
apocalypse of the earth.
(b) By tackling climate change head-on, accepting that climate change is not a
deviation and recognizing that
we can’t remake the earth the way it was.
(c) By formulating policies that will be for the betterment of the earth, by each
country and the world as well.
(d) By tackling climate change from a distance, not accepting that climate change
is not a deviation and
recognizing that we can remake the earth the way it was.
2. What is the tone of the author in the passage?
(a) Humanistic
(b) Euphemistic
(c) Obsequious
(d) Populist

. Page 3 of 40
3. Why did the expansion of Rome cause Seneca’s restlessness?
(a) A single ship had formerly disrupted the natural balance, Rome had now clogged
the seas with traffic.
(b) Seneca held the view that flood would be the part and parcel of the apocalypse.
(c) Seneca thought that the empire's unrelenting progress was against nature and
the total collapse was inevitable
if everyone could roam everywhere, with no restrictions.
(d) Seneca thought that the empire's progress was against the nature, and the
incessant advance of the empire
would turn the cosmos towards an enemy.
4. Which of the accompanying is INCORRECT in reference to the passage?
(a) The stoics from Greece believed that the earth shall be destroyed by flood when
society was at its pinnacle
of refinement and variety, as it is in the present times.
(b) Stoics thought that when nature's broken and flawed balance, known as Gaia, was
disrupted, the collapse
was inescapable.
(c) Contemporary philosophers studying ancient Greece thought that the Earth would
be gradually engulfed in
flames in a ceremonial cleaning before beginning again.
(d) All are incorrect.
5. Which of the following statements are CORRECT in reference to the passage?
A. There is a slight chance that we can return to the summer’s past.
B. Some people are shocked when some obnoxious predictions are proved to be true.
C. Some of the ecological systems had already undergone permanent alteration, in
addition to severe weather
events.
(a) Only A and B
(b) Only A and C
(c) Only B and C
(d) All A, B and C
Directions (Q.6-Q.10): Read the passage carefully and answer the questions.
Ukraine defence minister sees no threat of invasion from Russia. Ukrainian Defence
Minister Alexey Reznikov
said he had received no information so far indicating the possibility of Russia’s
invasion of his country shortly.
He further added that Russia had not created any strike groups which could indicate
the launch of any offensive
and categorically refuted the media reports indicating the likely attack of Russia
on Ukraine. Some reports had
indicated that Russia might attack Ukraine on Feb 20, 2022, the final day of the
Olympic Games in Beijing. The
US-led Western nations have of late been spreading accusations about Russia’s
likely invasion of Ukraine.
However, Russia has refuted these claims as empty and unfounded, promoting
tensions. Russian sources say that
Russia did not cause any threat to anyone; however, it did not rule out the
likelihood of incitements aimed at
justifying such accusations. Russia also warned that any attempt to use military
force to resolve the south eastern
Ukraine crisis would have serious repercussions.
On Dec17, simultaneously, the Russian Foreign Ministry issued the draft agreements
between Russia and the
US on security assurances and actions for safeguarding the security concerns of
Russia and NATO member
states. The document was intended to address, among other things, NATO’s ongoing
‘military development’ of
the Ukrainian territory. The two sides had numerous rounds of discussions,
including a personal meeting between
the top diplomats of Russia and the US, Foreign Minister Sergey Lavrov and US
Secretary of State Antony
Blinken. Meantime, NATO is sending additional forces in Eastern Europe due to the
situation around
Ukraine. Such reports have surfaced earlier to state that Russia allegedly plans to
attack its neighbour. These
reports mainly appeared from Ukraine to blow such unfounded fears to aid the US-led
NATO alliance.
In April 2021, Kyiv quoted a sizable amassing of Russian forces near Crimea,
Russia’s peninsula, in 2014.
Western powers aided demonstrators in overthrowing Ukraine’s elected, pro‐Russia
president, Viktor

. Page 4 of 40
Yanukovych. The troop movements led to severe strains between Russia and NATO,
which were lessened by
the end of April when the Russian units pulled back from their forward positions.
Intermittent reports have appeared in the past to indicate that Russia allegedly
intent huge expansionist plans on
the pretext of Ukraine. Those reports were unfounded, and the latest allegations
should be viewed with healthy
scepticism. Russia has sufficient reasons to hold out against any desire to occupy
Ukraine. Russia wouldn’t like
to acquire a corrupt, dysfunctional economy that would be a significant source of
drainage of Moscow’s financial
resources. Although Ukraine’s eastern part, which is mainly Russian‐speaking, might
not oppose tooth and nail
to Moscow’s rule, the strongly nationalistic western portion of the country would
not like it.
Ukrainian President Volodymyr Zelensky called on the West, led by the US, not to
create panic amid the build#up of Russian troops on his country’s borders, reported
adding that the warnings of an imminent invasion were
putting Ukraine’s economy at risk. The only reason that might compel Russia to take
such a misguided step
would be the efforts of the US to integrate Ukraine into the folds of NATO to
establish a staging area for Western
military power. Directly against Russia.
6. The above passage is probably?
(a) A work of fiction.
(b) An article.
(c) A reporting.
(d) A narrative.
7. Which of the following best describes the above passage, in brief?
(a) Ukraine in danger.
(b) Invasion in Ukraine- A reality or a build-up.
(c) Russia- The biggest alliance of Ukraine.
(d) Russia allegedly planning to occupy Ukraine.
8. According to the passage, which of the following is least likely to be true?
(a) The latest allegations that Russia allegedly intent huge expansionist plans on
the pretext of Ukraine should
be viewed with morbid scepticism.
(b) The US-led nations have of late been spreading accusations about Russia’s
likely invasion of Ukraine.
(c) Russia wouldn’t like to acquire a corrupt economy that would be a significant
source of drainage of
Moscow’s financial resources.
(d) Some reports indicated that Russia might attack Ukraine on the final day of the
Beijing Olympic Games.
9. The document issued by the Russian Foreign Ministry intended to address-
(a) Multiple NATO military exercises that have bothered Russia.
(b) Ukraine to blow such unfounded fears to aid the US-led NATO alliance.
(c) NATO’s ongoing ‘military development’ of the Ukrainian territory.
(d) NATO’s interference in the military development of the Russian Troupe around
Ukraine.
10. What does the author mean by using the phrase “tooth and nail” in paragraph
three?
(a) Ukraine’s eastern part will not oppose calmly to Moscow’s rule.
(b) Ukraine’s eastern part will not oppose furiously to Moscow’s rule.
(c) Ukraine’s eastern part will be neutral to Moscow’s rule.
(d) Ukraine’s eastern part will fight with all weapon’s possible, including tooth
and nail.

. Page 5 of 40
Directions (Q.11-Q.15): Read the passage carefully and answer the questions.
A few decades ago, Ronald Reagan said, “Nothing like an attack from outer space to
unite all of mankind.” The
deadly Covid-19 isn’t from outer space but it might just as well be!
After all the imaginary ‘enemies’ writers have conjured up to assemble chills and
thrills — transformers, placid
lakes, open crypts, anacondas and bird monsters — we have a real and present danger
in our midst, an entity
capable of impersonal mass destruction no one can spot till it is actually inside a
human. Where did it come
from? A bat, a fish? The truth is that it came from the activities of a small band
of hairless apes who infested
every part of the planet and upset ecological balances with no regard for nature or
scientists who have for years
been warning us about exposure to wildlife. They have also campaigned against trade
in animals which do not
usually have any contact with humans.
How will it change our perspective, immediate and long term? While day and night
sweep up the seas and
mountains and the Earth turns on its axes, we suddenly see ourselves not as
individuals but as a part of the huge
chain of humanity? Though every generation quite unnecessarily tells the next that
the future depends on them,
our future does indeed depend on the next gen who, each and every one of them, has
been exposed and before
emotional maturity, to the primary fear of survival or death. Every child expects
its parents or grandparents to
protect it from disasters and a child’s fears are probably deeper than an adult’s
because they are both unspoken
and fantastical.
But children see that their parents are also afraid and perhaps controlling their
hysteria with some difficulty.
Even otherwise, school teachers will tell you that they are handling children whose
early childhood and primary
school years are filled with experiences their caretaker generation never had. And
now this! While all of us have
worried about the effects of nuclear warfare, guided missile conflicts and land-
mines planted all over the world,
the prospect of an untimely end has suddenly reached every single member of our
race.
When millions who sold tea or green coconuts or flowers and fruit on pavements have
no idea how to feed their
families, people who are better off have at least temporarily, begun meditating on
what we shouldn’t be doing
— living only for ourselves. This is an appeal to schools to take Value Education
into classrooms and help
children to deal with emotions. Or else we will have to prepare for a different
kind of chaos as the present trauma
begins to affect students more deeply than the anxiety their academic progress
might pose. Perhaps we can
remind ourselves of what Archibald MacLeish wrote 52 years ago when he saw the
first photographs of the Earth
taken from the Moon. “To see the Earth as it truly is, small and blue in that
eternal silence where it floats, is to
see ourselves as riders on the Earth together, brothers on that bright loveliness
in the unending night.”
11. The author is primarily concerned with which of the following?
(a) Child’s fear are deeper than the adult’s.
(b) It is time to stop putting the fate of the planet in the hands of the children
and ensure they are better equipped
to handle the future.
(c) Activities of a small band of hairless apes who infested every part of the
planet and upset ecological balance.
(d) A change in immediate and long-term perspective to value with regards to the
ecology and education to the
children.
12. Which of the following best describes the last sentence of the passage?
(a) The Earth is small and blue in colour where we are the riders and there are
endless nights.
(b) The Earth is small and full of blue because of the water, and is a home to
human beings that must do
everything to sustain its existence.
(c) The Earth should be seen as a small living planet hanging in the vastness of
the universe, silently sustaining
the lives, should be ridden together.
(d) The Earth, which is a beautiful speck in the universe, sustaining beings, must
be nurtured by humans, all and
sundry in preserving its beauty and bounty till eternity.

. Page 6 of 40
13. ‘While day and night sweep up the seas and mountains and the Earth turns on its
axis, we suddenly see ourselves
not as individuals but as a part of the huge chain of humanity?’ Which part/parts
of the sentence contain/(s) a
grammatical error?
I. While day and night sweep up the seas and mountains
II. we suddenly see ourselves not as individuals
III. Earth turns on its axes,
IV. huge chain of humanity?
(a) Only IV
(b) Only II
(c) I, III & IV
(d) III & IV
14. What according to the author is the meaning of “Hairless apes”?
(a) Humans Beings.
(b) Apes without hair.
(c) All beings without hair.
(d) Human beings without hair.
15. An appropriate title for the passage could be?
(a) Saving Earth.
(b) Saving Present.
(c) Saving Ourselves.
(d) Saving Apes.
Directions (Q.16-Q.20): Read the passage carefully and answer the questions.
A team of scientists has identified the olfactory receptors for two common odor
molecules: a musk found in
soaps and perfumes and a compound prominent in smelly underarm sweat. The research
team also discovered
that more recent evolutionary changes to these olfactory receptors alter people’s
sensitivities to those odors. The
work was published in PLOS Genetics on Thursday.
“There’s a molecule called androsterone,” said Joel Mainland, a neuroscientist at
Monel Chemical Senses Center
in Philadelphia and an author of the new study. “And we know that some people smell
that molecule as urine,
some people smell that molecule at sandalwood and some people don’t smell it at
all.” With that said, genetic
changes aren’t the only thing (________) smell interpretation. “One is genetic and
the other is experience, which
includes things like the culture you grew up in,” said Hiroaki MA tsunami.
The researchers then looked for associations between olfactory receptor genes and
odors, as well as variations
within those genes and their potential effect on perception of the odor. By
sampling a large, diverse population
of people, the researchers were able to home in on odors whose perception was based
in genetic differences
between people, rather than cultural or experiential factors. That led them to
molecules including trans-3-methyl#2-hexenoic acid and calanolide.
Trans-3-methyl-2-hexenoic acid is considered one of the most pungent compounds in
underarm sweat.
Calanolide is a synthetic musk often described as having a floral, woody odor
that’s used in perfumes and
cosmetics, but also things like kitty litter. The research team was able to
identify olfactory receptor variants for
those odors. In the case of the underarm odor, most people with the more
evolutionarily recent gene variant
found it more intense. The opposite was true for calanolide.
The calanolide findings were particularly striking, with some participants unable
to smell the musk at all. “It’s
really rare to find an effect that’s as large as what we saw for this one receptor
on the perception of the musk
odor,” said Marissa Lamarck, a neuroscientist at the University of Pennsylvania who
was an author of the study.
MA tsunami views this work as another example of human olfaction being more complex
than people initially
thought. He said that, although the major findings in the study involved just two
scents, they’re adding to
evidence that “odorant receptors as a group have extraordinary variety.” The
authors think their findings support
a hypothesis that has been criticized that the primate olfactory system has
degenerated over evolutionary time.

. Page 7 of 40
Kara Hoover, an anthropologist at the University of Alaska Fairbanks who was not
involved in this research but
who studies the evolution of human smell, is not convinced by that hypothesis in
the first place. “Why is reduced
intensity assumed to be degradation?” she asked. “Maybe other things are becoming
more intense or odor
discrimination is improving. We know too little to make these conclusions.”
For Hoover, these findings stirred up other evolutionary questions. “Our species is
really young,” she said. “Why
this much variation in such a short period of time? Is there an adaptive
significance?”
16. What is not the evolutionary question stirred up by Hoover?
(a) Are species really young?
(b) Why there is so much variation in such a small period of time?
(c) Is there an adaptive significance?
(d) Why is there so much variance in such a long period of time?
17. Which of the following is the correct title for the given passage?
(a) Olfactory evolution generation per generation.
(b) Changes in olfaction and bad odor.
(c) Body odor may smell worse to you than your ancient ancestors.
(d) Why is bad odor smelling worse than our ancient ancestors?
18. Which of the following is not a correct statement?
(a) A team of scientists has identified the olfactory receptors for two common odor
molecules: a musk found in
soaps and perfumes and a compound prominent in smelly underarm sweat.
(b) Androsterone is the molecule which variant people smell valiantly.
(c) Cultural differences are solely responsible for the interpretation of smell.
(d) Trans-3-methyl-2-hexenoic acid is considered one of the most pungent compounds
in underarm sweat
19. Read the following statement and choose the option which is defined by this
statement:
“And we know that some people smell that molecule as urine, some people smell that
molecule at sandalwood
and some people don’t smell it at all.”
(a) Trans-3-methyl-2-hexenoic acid
(b) Trans amino acid
(c) Androsterone.
(d) Calanolide.
20. ‘With that said, genetic changes aren’t the only thing _________ smell
interpretation.’ Which of the following
word will fill in the blank to make the sentence coherent?
(a) Underlying.
(b) Understating.
(c) Overpowering.
(d) Understanding
Directions (Q.21-Q.25): Read the passage carefully and answer the questions.
In ‘The Structure of Scientific Revolutions’, the philosopher of science Thomas
Kuhn observed that scientists
spend long periods taking small steps. They pose and solve puzzles while
collectively interpreting all data within
a fixed worldview or theoretical framework, which Kuhn called a paradigm. Sooner or
later, though, facts crop
up that clash with the reigning paradigm. Crisis ensues. The scientists wring their
hands, re-examine their
assumptions and eventually make a revolutionary shift to a new paradigm, a
radically different and truer
understanding of nature. Then incremental progress resumes. For several years, the
particle physicists who study
nature’s fundamental building blocks have been in a textbook Kuhnian crisis.
The crisis became undeniable in 2016, when, despite a major upgrade, the Large
Hadron Collider in Geneva still
hadn’t conjured up any of the new elementary particles that theorists had been
expecting for decades. The swarm

. Page 8 of 40
of additional particles would have solved a major puzzle about an already known
one, the famed Higgs boson.
The hierarchy problem, as the puzzle is called, asks why the Higgs boson is so
lightweight — a hundred million
billion times less massive than the highest energy scales that exist in nature. The
Higgs mass seems unnaturally
dialled down relative to these higher energies as if huge numbers in the underlying
equation that determines its
value all miraculously cancel out.
The extra particles would have explained the tiny Higgs mass, restoring what
physicists call “naturalness” to
their equations. But after the LHC became the third and biggest collider to search
in vain for them, it seemed
that the very logic about what’s natural in nature might be wrong. “We are
confronted with the need to reconsider
the guiding principles that have been used for decades to address the most
fundamental questions about the
physical world,” Gian Giudice, head of the theory division at CERN, the lab that
houses the LHC, wrote in 2017.
At first, the community despaired. “You could feel the pessimism,” said Isabel
Garcia, a particle theorist at the
Kavli Institute for Theoretical Physics at the University of California, Santa
Barbara, who was a graduate student
at the time. Not only had the $10 billion proton smasher failed to answer a 40-
year-old question, but the very
beliefs and strategies that had long guided particle physics could no longer be
trusted. People wondered more
loudly than before whether the universe is simply unnatural, the product of fine-
tuned mathematical
cancellations. Perhaps there’s a multiverse of universes, all with randomly dialled
Higgs masses and other
parameters, and we find ourselves here only because our universe’s peculiar
properties foster the formation of
atoms, stars and planets and therefore life. This “anthropic argument,” though
possibly right, is frustratingly
untestable. Many particle physicists migrated to other research areas, “where the
puzzle hasn’t gotten as hard as
the hierarchy problem,” said Nathaniel Craig, a theoretical physicist at UCSB. Some
of those who remained set
to work scrutinizing decades-old assumptions.
21. Which of the accompanying is an INCORRECT statement?
(a) The question of why the Higgs boson is so light is the part and parcel of the
hierarchy problem.
(b) The generation of new particles from the Large Hadron Collider would’ve solved
the mystery behind the
Higgs boson particle.
(c) The Large Hadron Collider in Geneva, despite a substantial upgrade, failed to
produce any of the new
elementary particles that physicists had been anticipating for ages.
(d) All are correct.
22. Which of the following statements are correct in respect to the ‘anthropic
argument’?
A. There might be a multiverse of universes, all with arbitrarily dialled Higgs
masses and other parameters.
B. Our universe's atypical properties incite the production of atoms, stars,
planets, and so life in general.
C. After the failure of the Large Hadron Collider, people started to strongly
believe if the universe is normal,
the result of fine-tuned mathematical validations.
(a) A and B
(b) B and C
(c) A and C
(d) All A, B and C
23. What is the actual definition of ‘paradigm’ in reference to this passage?
(a) When a crisis shows itself, scientists re-examine their assumptions, and
eventually move to a new model, a
radically different and more accurate view of nature.
(b) Scientists opt for gradual progress by presenting and solving conundrums while
interpreting all facts
collectively under a predefined perspective or conceptual foundation.
(c) Scientists opt for fast progress by offering and resolving puzzles while
interpreting limited facts collectively
under a new perspective or conceptual foundation.
(d) When a crisis in a particular field shows itself, the scientists dismantle the
earlier theories and begin afresh
with new perspectives.

. Page 9 of 40
24. Which of the accompanying is an INCORRECT statement?
(a) When facts don’t validate themselves with a paradigm, scientists re-examine
their assumptions and
eventually make an innovative shift to a new paradigm.
(b) In comparison to these greater energies, the Higgs mass appears to be
unnaturally dialled down, as if massive
numbers in the underlying equation that determines its value mysteriously cancel
out.
(c) After the failure of the Large Hadron Collider, it seemed that the very
judgement about what’s normal in
nature might be incorrect.
(d) The "anthropic argument" can be tested despite being flawless.
25. What was Gian Giudice’s view after the failure of the Large Hadron Collider?
(a) He was of the opinion that the fault of the Large Hadron Collider was the
inefficiency to get a hold of the
larger picture that new elements can’t be created nor can be destroyed.
(b) There is an essentiality for maintaining the same guiding principles that form
the base of the physical world.
(c) There is a requirement for reassessing the guiding principles that form the
base of the physical world.
(d) There is a need to re-examine the scientific studies and innovations based on
the guiding principles that form
the base of the physical world.
Directions (Q.26-Q.30): Read the passage carefully and answer the questions.
An empty canvas sits on an easel in front of the window with an expanse of tropical
landscape behind it. The
paints are neatly arranged next to it; the paintbrushes dry and waiting. I want to
paint on it. I can feel the pressure
of art yet to be expressed inside me. Over the last few months, it has felt like
these creations inside me waiting
to be born are on backorder. And yet, I cannot bring myself past the exhaustion to
sit down and paint.
A close friend of mine has a saying... Starting a business is like making a plane
take off of the runway. Everyone
involved has to put everything they have into it and in the beginning, it is tense
and sometimes rocky and then
after a bit, it stabilizes.
This retreat centre (though it ties in with my mission and current business
perfectly) is in reality a brand new
business. Since the second we decided to go ahead with it, it has taken every last
drop of sweat. every breath and
every minute for every person involved. None of us have been able to focus on our
respective role within the
organization because of everything we have had to get done. Instead of doing my
role. I have had to sort out
details with design and direct contractors and workers and shop for necessities and
haul things in pick-up trucks.
At this point I feel guilty for having ever been upset at the price I had to pay
for a hotel or any other hospitality
service having seen what goes into the back end of a retreat. It is literally a
crazy amount of work and it feels
like the minute something goes right, something else goes wrong. And it feels
impossible to make it so that every
person feels good. The kind of pressure we are all under has made the team’s
collective shadows and wounds
surface. On top of that the shamanic nature of this property itself is a catalyst.
The intensity level of it makes it
feel like you’re on a spiritual plant medicine without having to actually ingest
anything.
To add to it all, the collective hate group which is so vehemently opposed to my
career as well as me as a person
has been dedicating themselves wholeheartedly to trying to destroy this retreat
centre. Essentially, the minute
that we announced this retreat centre, they began planning how to shut it down.
I wish that people could walk for a week in my shoes. I’m overwhelmed. Part of me
wants to just go into cryo#sleep and wake up years from now to years to see if
anything has already been sorted out and is finally running
smooth.
The pressure I am under now is the very reason that people choose to stay small. I
am acutely aware that it is
much safer socially to do so. That is sad to me. I can see now first-hand what
people are trying to avoid by
holding themselves back. The reality is that launching anything in the hopes of
success, just like you do when

. Page 10 of 40
you push the throttle on a plane to launch it into the airs is a risk. It is a risk
that you’ll never be ready for. And
so, rather than provide any solution today, I would like to end this message by
inviting you to ask yourself the
question... What am I afraid of that is currently making me hold myself back? By
taking a risk to ‘really go for
it’, whatever that may be, what is it that I risk?
26. According to the passage, the author can be best described as:
(a) An architect
(b) An entrepreneur
(c) A priest
(d) A broker
27. How does the author feel towards hospitality service?
(a) He envies them for the profits they make.
(b) He feels vindicated that his business does better than most hospitality
services.
(c) He empathizes with them for the hard work they have put in to provide their
services.
(d) He feels pity towards hospitality service for the hard work they have put in to
provide their services.
28. The pressure I am under now is the very reason that people choose to stay
small. We can imply from the above
that:
(a) People succumb under pressure.
(b) People choose to be under pressure.
(c) People eliminate pressure.
(d) People cannot endure pressure.
29. What is the purpose of the author to want the reader to find answers to the
questions he puts up in the end of the
passage?
(a) The author does not want the readers to experience the kind of pressure that he
is enduring.
(b) He wants the readers to reflect on their lives by seeking answers to what stops
them from taking risks.
(c) The author wants the readers to take a chance on some big ventures.
(d) The author wants the readers to take some risks to understand that there are no
risks at the end of the hard
work.
30. The “collective hate group” that the author mentions, mostly likely to refers
to
(a) Society.
(b) Members of parliament.
(c) Competitors.
(d) Friends and family
ECTION - C: LEGAL REASONING
Passage (Q66.-Q.70): Many a times, especially in defamation cases the identity of
an anonymous person posting
a comment as a blog, become a crucial question. In Independent Newspaper Inc. v.
Brodie, the court of Maryland
held that the court can direct the disclosure of details about the anonymous person
subject to the following
conditions: i. Reasonable notice has been given to the anonymous person that his
identity was being requested
by the plaintiff. ii. Such person has been given sufficient time to oppose
disclosure of details. iii. Alleged
defamatory statement has been specified in the notice. iv. There is a prima facie
case; and v. It has been proved
that balance of convenience lies in favour of disclosure of his identity. In India,
the Code of Criminal Procedure,
1973 empowers the police to call for records from telephone companies and Internet
service Providers (ISPs) to
identify the person who has made a transaction; and obtain his log information
content contained in an e-mail/
chat account and IP address details. In civil cases, courts can issue notices to
such ISPs directing them to submit
the required information. Normally, Internet Service Providers will not entertain
direct request for disclosure of
identity of a person, and even if the complaint is made against a fake blog or
impersonating profile on website,
an Internet Service Provider in whose favour balance of convenience lies will at
most block the subject page
from access but do not reveal the person who created it.
(Source:https://www.bhu.ac.in/law/blj/Banaras%20Law%20Journal%20%202015%20Vol
%2044%20No.%20
2.pdf)
66. Assertion: Courts have wrestled with producing an appropriate standard for
revealing the identity of an
anonymous blogger who posts allegedly defamatory material on a message board or
website.
Reason: The standard created is far too sympathetic to anonymous bloggers and fails
to address important issues
facing victims of defamation.
(a) Both A and R are true but R is not correct explanation of A.
(b) Both A and R are true and R is correct explanation of A.
(c) A is true but R is false.
(d) A is false but R is true.
67. An anonymous individual had tweeted on alleged violence in the state following
Mamata Banerjee-led All India
Trinamool Congress’ assembly election victory. In the tweet, the person urged Prime
Minister Narendra Modi
to show his “Virat Roop” from the “early 2000s” to “tame” Banerjee in Bengal,
sparking anger among some
Twitter users, who asked the US-based platform to take action. The referenced
account has been suspended for
repeated violations of Twitter Rules specifically our Hateful Conduct policy and
Abusive Behavior policy. Will
the anonymous individual have any redress against the internet service provider's
arbitrary action?
(a) No, since the internet service provider blocked the user's account after
receiving a complaint.
(b) Yes, because no notice was provided to the anonymous individual and sufficient
time for them to object to
the disclosure of details.
(c) No, since there is a prima facie case of slander.
(d) Yes, because the balance of convenience favors disclosing his identify.
68. A criminal complaint has been filed against an anonymous commenter who has
commented on every social
media post by BJP leader Shri Narendra Modi. The complainant considers the
accused's statements such as
"chowkidaar chor hai," "choron ka sardar," and "Commander-in-Thief" to be
defamatory to the whole political
party and has thus attempted to prosecute the anonymous commenter under Section 500
of the Indian Penal
Code. Decide
(a) The court will order for disclosure of information about the anonymous
individual.
(b) The Internet service provider must provide the necessary information on the
anonymous user who submitted
defamatory comments.
(c) The Internet service provider must give enough notice and opportunity to
anonymous person defends his
case before blocking his account.
(d) the police may request the disclosure of information about the anonymous
individual.

. Page 18 of 40
69. Meera writes a regular blog, most of which are political in nature. The topic
revolves around the lack of
government responses to queer people, who are constantly denied basic rights. Who
always have to battle for
their basic rights as a form of equality. The gay community accused Meera of
defamation, claiming that the
material is targeted at laymen and that amongst them they are a disadvantaged
community that would remain
such as neither the government nor the law-making agencies are interested in
elevating the position of queer
people in society. Meera's blog was blocked and deleted without even notifying her.
Decide
(a) The action taken against Meera was unjust since she was not given a fair chance
to defend herself.
(b) The action taken against Meera is not inappropriate because the internet
service provider has the right to
directly block the user in such cases.
(c) The action taken against Meera is unjust since she was not served with the
notice.
(d) The action taken against Meera is inappropriate as an internet service provider
cannot unilaterally block a
user's account.
70. Chetna, is the winner of an MTV entertainment show, India's next top model. Her
reputation skyrocketed after
she was seen in the city whilst handing food packs to the needy. Fans admired the
actress's compassion and
gratitude towards other humans. Dheeraj, the camera guy for Etv, a programme that
provides frequent updates
on the entertainment industry, used to write her current location in the comment
section of her Instagram account
from his fake account, so that her followers could reach her. This became a major
issue for the actress after a
while, since she is now being followed by creeps and stalkers on a regular basis.
The actress filed a criminal
complaint against the fake account. In the current situation what will be the next
resource of law enforcement
agencies?
(a) Dheeraj's bogus account might be directly blocked by the Internet service
provider.
(b) The court will issue an order requiring the Internet service provider to reveal
the identification of the accused.
(c) The police will request that the Internet service provider supply the essential
information on the offender so
that the investigation may proceed.
(d) The police can immediately request information on the accused from the Internet
service provider.
Passage (Q71.-Q.75): The trademark registration and enforcement landscape in India,
as far as legal provisions
are concerned, is no different from that in other Commonwealth jurisdictions. A
trademark is a type of
intellectual property, and typically a name, word, phrase, logo, symbol, design,
image, or a combination of these
elements. There is also a range of non-conventional trademarks comprising marks
that do not fall into these
standard categories. The owner of a registered trademark may commence legal
proceedings for trademark
infringement to prevent unauthorized use of that trademark. However, registration
is not required. The owner of
a common-law trademark may also file suit, but an unregistered mark may be
protectable only within the
geographical area within which it has been used or in geographical areas into which
it may be reasonably
expected to expand.
In situation of contradiction between registered mark and passing off both can
exist simultaneously with the
registered mark without infringing the right of one another. The following
trademarks are not registrable in India
on grounds of inherent registrability: i) those devoid of any distinctive
character; that is to say, not capable of
distinguishing the goods or services of one person from those of another person;
ii) those of such nature as to
deceive the public or cause confusion; iii) those containing or comprising any
matter likely to hurt the religious
susceptibilities of any class or section of citizens of India; iv) those comprising
or containing scandalous or
obscene matter. Trademark infringement is a violation of the exclusive rights
attached to a trademark without
the authorization of the trademark owner or any licensees (provided that such
authorization was within the scope
of the license). Infringement may occur when one party, the “infringer”, uses a
trademark that is identical or
confusingly similar to a trademark owned by another party, about products or
services which are identical or
similar to the products or services which the registration covers. An owner of a
trademark may commence legal
proceedings against a party that infringes its registration.

. Page 19 of 40
(Source-Trademark Laws in India. (n.d.).
Http://Www.Legalservicesindia.Com/Article/307/Trade-Mark-Law#in-India-&-Its-
Violation.Html. Retrieved April 18, 2022, from
http://www.legalservicesindia.com/article/307/Trade-Mark-Law-in-India-&-Its-
Violation.html)
71. Which of the following statement(s) is/are the suitable illustration(s) of
trademark infringement?
I) A food joint under the name Tom’s kitchen, sues Mr. Figg because their pet cat’s
name is also Tom.
II) Sony Corporation filed a suit claiming dilution of its well-known SONY
trademark against a sole proprietor
running tours and travels business under the name, Sony Tours and Travels.
III) A person is free to record a TV program to view it later, but he transfers or
distributes it to others.
IV) X, a pharmaceutical company made a medicine registered under the mark AMOXYL in
2005 in class 5 in
respect of Pharmaceutical goods. Y, another pharmaceutical company made a medicine
registered under the
mark LYMOXYL in 2007 in the same class.
(a) I and II
(b) II and III
(c) I and III
(d) II and IV
72. KT, the famous pop star from Dhanbad has a large fan base and is extremely
popular amongst pop music lovers.
KT Entertainment was incorporated to manage the artist’s escalating career. The
Cute Toys company was making
a large business by selling miniature dolls of the artist who can sing few lines on
compositions and cashing on
his popularity. KT Entertainment company was extremely aggrieved and filed for a
permanent injunction from
infringing the artist’s right of publicity and false endorsement leading to passing
off. KT Entertainment company
argued that the unauthorized or unlicensed use of the artist’s reputation for goods
or services will deceive the
public. Decide-
(a) The allegations by KT Entertainment company are appropriate because this would
lead the public into
believing that the goods and services are associated with the singer.
(b) The allegations by KT Entertainment company are appropriate because Cute Toys
company did not take
consent from the artist.
(c) The allegations by KT Entertainment company are inappropriate because both the
businesses are different
and that will not cause any confusion to people in general.
(d) The allegations by KT Entertainment company are inappropriate because it is not
an infringement of
trademark. .
73. France Industries Limited, the Plaintiff, found hardware fittings and bathroom
accessories retailer using their
well-known mark ‘PINE’ and hence filed a trademark & copyright infringement suit
before the Court of law.
The defendant had filed two applications for registration of the mark PINE and
pirated artwork that was
advertised. Decide-
(a) There is a trademark infringement because the parallel names are likely to
cause a sense of muddle in the
minds of the customer and no license is being obtained from plaintiff.
(b) There is a trademark infringement because the defendant used the same name
without the consent of the
plaintiff.
(c) There is no trademark infringement because both the businesses are different
and they will never overlap.
(d) There is no trademark infringement because the defendant has the fundamental
right to choose guaranteed
by the Constitution.
. Page 20 of 40
74. Ibis is a company that has launched a mango drink by the name CHILLZ. It has a
tagline that goes- “ Hardum
fresh !” and registered the same. One other Rakesh and Co. also launched a grape
drink by the name FIZZ later
in the same year. It has a tagline that goes- “Every time aur bhi fresh !” Ibis
company sued Rakesh and Co. for
trademark infringement. Decide
(a) There is a trademark infringement because the names are analogous.
(b) There is a trademark infringement because the taglines are analogous.
(c) There is no trademark infringement because both the drinks differ in flavor.
(d) There is no trademark infringement because the taglines are poles apart.
75. from those of another person; ii) those of such nature as to McDonald’s, a
multinational fast food corporation,
which has developed it’s business in past 300 years. McDonald’s is international
chain famous for its
hamburgers, cheeseburgers, and French fries sued a person named Sunny for Trademark
infringement. All rights
regarding the McDonald’s business are reserved with the company under The Trademark
Act. Sunny is a famous
street food vendor selling 100 types of Dosas. He is running a hereditary shop all
the dishes in menu are same
since 500 years but has not registered name of any dish. On his menu, he has an
item called “Sunny special Mc
Dosa.” Decide suit for violation of trademark can be brought or not?
(a) Sunny is guilty of trademark infringement because the right to use the prefix
‘Mc’ was reserved with
Mcdonald's.
(b) Sunny is guilty of trademark infringement because the itinerary sold by both
Mcdonald’s and Sunny were
the same.
(c) Sunny is not guilty of trademark infringement.
(d) Sunny is not guilty of trademark infringement because Sunny has the fundamental
right to choose guaranteed
by the Constitution.
Passage (Q76.-Q.80): The State of Gujarat and its Director of Prosecution came
under the criticism of the
Supreme Court for not filing appeal against a High Court order granting bail to the
accused in a case where a
Dalit man was brutally murdered while he was collecting scrap outside a factory. A
bench comprising Justice
MR Shah and Justice BV Nagarathna noted that the High Court's bail order was "most
perfunctory and casual",
but the State did not appeal against it. Aggrieved with the High Court order, the
complainant (the widow of the
victim) filed appeal before the Supreme Court. While allowing the appeal and
setting aside the High Court's
judgement in Jayaben V. Tejas Kanubhai Zala & Anr, the bench said, "By not filing
the appeals by the State
against the impugned judgments and orders releasing the accused on bail in such a
serious matter, the State has
failed to protect the rights of the victim. We are of the opinion that this was the
fit case where the State ought to
have preferred the appeals challenging the orders passed by the High Court
releasing the accused on bail. In
criminal matters the party who is treated as the aggrieved party is the State which
is the custodian of the social
interest of the community at large and so it is for the State to take all the steps
necessary for bringing the person
who has acted against the social interest of the community to book."
Rejecting the submissions made by accused(s) counsel regarding non cancellation of
bail since more than 2.5
years were passed and there were no allegation of misuse of liberty, the bench in
the judgement authored by
Justice MR Shah said,
"As per the settled preposition of law, cancellation of bail and quashing and
setting aside the wrong order passed
by the High Court releasing the accused on bail stand on different footings. There
are different considerations
while considering the application for cancellation of bail for breach of conditions
etc., and while considering an
order passed by the Court releasing the accused on bail. Once, it is found that the
order passed by the High Court
releasing the accused on bail is unsustainable, necessary consequences shall have
to follow and the bail has to
be cancelled."
(SOURCE : 'State & Director Of Prosecution Failed In Duties' : Supreme Court Pulls
Up Gujarat Govt For Not
Appealing Against Bail Order In Murder Case, LIVE LAW )

. Page 21 of 40
76. Jana was a well-known criminal, and his lawyers were always able to keep him
out on bail. However, this time
he was convicted of a crime he did not commit. Decide
(a) State should act as the custodian of Jana as he was framed wrongly this time.
(b) State should act as a custodian for aggrieved person for the social interest of
the community at large.
(c) State will not act in favor of Jana as he is not the aggrieved party here.
(d) In any circumstance, the state cannot act as a guardian for the aggrieved
individual.
77. In criminal matters, Expression “State as a custodian” refers to :
(a) Custodian to the aggrieved party.
(b) Custodian of social interest of the community at large.
(c) Custodian to both aggrieved party as well as social interest of the community
at large
(d) Custodian against the person who has acted against the social interest of the
community to book.
78. Ram works as a security guard for the RBI. He was in the most senior position
and had been working with the
bank for a decade to keep a tight handle on security. Ram was severely injured by a
gunshot that directly hit his
right leg during a robbery, rendering him incapacitated and ineligible to return to
his duty with the bank. The
accused in the trial was recognized, but because the prosecution offered no
substantial proof, he was acquitted
on bail. Can Ram seek cancellation of the accuser’s bail because the Court's order
releasing the accused on bail
is unsustainable?
(a) The order is sustainable as the court rightly appreciated all the evidence
presented before the court by
prosecution.
(b) The order is not sustainable and ram can seek cancellation of accuser’s bail as
the act done by the accused
acted against the social interest of the community.
(c) The order releasing the accused on bail cannot be said unsustainable because
the prosecution failed to provide
substantial proof to prove accused guilt.
(d) The order releasing the accused on bail is sustainable even if the accused
acted against the social interest of
the community.
79. Continuing with facts of above case, what remedy can ram avail in such a
situation?
(a) Ram can seek the assistance of the state to defend the victim's rights and take
all necessary efforts to bring
the person who has acted against the social interests of the community to justice.
(b) Ram has the right to file an appeal in a court of law since the state failed as
a custodian of the social interests
of the community as a whole.
(c) Ram has no further options in this circumstance.
(d) The court's decision is final, and no appeal can be filed by the plaintiff if
the prosecution fails to show the
accuser’s guilt.
80. Choose an alternative that provides an anomaly of the passage's context:
(a) In criminal matters the party who is treated as the aggrieved party is the
State which is the custodian of the
social interest of the community at large.
(b) If an order of the lower court is not unsustainable, the order passed by the
Court releasing the accused on
bail can be cancelled.
(c) In an event of state failing to file an appeal on victim’s behalf, it amounts
to violation of right of the victim.
(d) All of the above.

. Page 22 of 40
Passage (Q81.-Q.85): If you suffer accidental personal injury or damages through
the fault of another, that
person or business is legally responsible (liable) and can be required to pay
compensation. To determine
responsibility, the court looks for negligence--carelessness by one of the parties
involved. Whoever is determined
to have been less careful (i.e. more negligent), is legally responsible for at
least part of the damages incurred.
Compensation is generally awarded based on the strength of your documentation and
degree of injury. A legal
professional is recommended in Personal Injury cases over all others in order to
maximize your compensation
in the face of the Insurance companies, which normally defend such cases. Personal
injury cases are serious
matters. They often involve grave injury, permanent disability, and even death.
Victims depend on the personal
injury, lawyer to recover financial damages that are required to cover their
medical treatments, replace
permanently lost income, and compensate for their pain and suffering.
(Source: http://www.legalservicesindia.com/articles/piov.htm)
81. A retired cop seeks vengeance for his murdered family by hiring two misfit
criminals Jai and veeru to apprehend
the wicked dacoit Gabbar Singh. They found Gabbar's secret hideout after
successfully carrying out their plan.
Both Jai and Veeru were hiding away on opposing hills, covering Gabbar's domain.
Both took shots at Gabbar,
but only Veeru's struck the right side, whilst Jai's shot hit Veeru in the chest.
Determine Jai's liability in this
case?
(a) veeru cannot be held accountable since both jai and Veeru have a single purpose
in to kill Gabbar.
(b) Jai will be held accountable for his recklessness.
(c) Jai will not be held accountable as both jai and Veeru took shots at Gabbar.
(d) Jai will be held accountable only if it is proven that he was less cautious.
82. Continuing with the facts given in above question under what circumstance is
jai entitled for the award of
compensation?
(a) Mere injury is sufficient to claim the compensation.
(b) Compensation is granted in case injury is serious.
(c) Negligence of other party must be proved.
(d) All of the above.
83. The cleaning staff at the national hospital has been instructed to sweep the
floor first thing in the morning since
there is a risk of a patient or his or her attendant slipping while in a hurry. The
workers mopped the floor with
soap water on Sunday, when there were less visitors. A nurse stumbled while racing
to attend to a patient in the
emergency room who needed urgent treatment. Will the situation be considered
negligent on cleaning staff’s
part?
(a) Yes, because the cleaning staff is obliged to exercise due care at all times.
(b) No, since it is their duty to clean the floor.
(c) Yes, because the nurse and the housekeeping staff both contributed to the
accident.
(d) No, because the workers were not careless while doing their tasks, hence it was
not a case of negligence.
84. Continuing with the above-mentioned factual circumstance, how much compensation
can be granted to the
nurse?
(a) The nurse will be entitled to compensation only after proving the negligence of
other party.
(b) Whoever is determined to have been less careful will be awarded the
compensation.
(c) The nurse will be awarded the based on the degree of injury.
(d) Cannot be ascertained as facts lack complete information.

. Page 23 of 40
85. Assertion: If a person experiences an injury as a result of a car accident and
was not at fault, he or she may be
able to seek compensation for injuries.
Reason: The plaintiffs need to have medical documentation detailing the injuries
sustained, insurance
information.
(a) Both A and R are true but R is not correct explanation of A.
(b) Both A and R are true and R is correct explanation of A.
(c) A is true but R is false.
(d) A is false but R is true.
Passage (Q86.-Q.89): The traditional approach to criminal law has been that a crime
is an act that is morally
wrong. The purpose of criminal sanctions was to make the offender give retribution
for harm done and expiate
his moral guilt; punishment was to be meted out in proportion to the guilt of the
accused. In modern times more
rationalistic and pragmatic views have predominated. One widely accepted principle
of criminal law is the rule
against retroactivity, which prohibits the imposition of ex post facto laws (i.e.,
laws that would allow an
individual to be punished for conduct that was not criminal at the time it was
carried out). The rule restricts
the authority of judges to declare new offenses (though not necessarily to expand
the scope of old ones by
interpretation). One of the most-important general principles of criminal law is
that an individual normally cannot
be convicted of a crime without having intended to commit the act in question. With
few exceptions, the
individual does not need to know that the act itself is a crime, as ignorance of
the law is no excuse for criminal
behaviour. Thus, if a person believes that an act is perfectly legal and
intentionally performs that act, the legal
requirement of criminal intention is met.
(Source: https://www.britannica.com/topic/crime-law/General-principles-of-criminal-
law)
86. Assertion: Every wrong that is not morally right will be considered as crime.
Reason: Morality and criminal law frequently overlap.
(a) Both A and R are false and R is not correct explanation of A.
(b) Both A and R are true and R is correct explanation of A.
(c) A is true but R is false.
(d) A is false but R is true.
87. The author states that “In modern times more rationalistic and pragmatic views
have predominated”. What
exactly does the author mean by this?
(a) The meaning of a proposition is to be found in the practical consequences of
accepting it, and that unpractical
ideas are to be rejected.
(b) A pragmatist can consider something to be true without needing to confirm that
it is universally true.
(c) Criminal sanction is modern era is generally dealing crime in a practical and
rationalistic way rather
following ideas or principles.
(d) More rationalistic and pragmatic viewpoints have prevailed in current times.
88. On 8 November 2016, the Government of India announced the demonetisation of all
₹500 and ₹1,000 banknotes
of the Mahatma Gandhi Series. The government also gave the public enough time to
exchange and deposit old
Rs 500 and Rs.1,000 banknotes with government banks, with a warning that if anyone
is caught with the scrapped
illegal tender beginning January 1st, 2017, he or she will face a fine,
imprisonment, or both. Ranu a 100 year old
lady wishes to deposit his savings of life in a bank in 2018. When her
representative Shyam reached the bank to
deposit money, he was arrested by the authorities for illegally keeping the
scrapped currency notes being holding
a status of illegal tender for so long and then coming to the bank and claiming its
acceptance. Decide
(a) Shyam's arrest is legitimate since he satisfied the threshold of criminal
intent.
(b) The arrest of Shyam is illegal since he lacks the required mens rea for
committing the crime.
(c) Ranu should be jailed instead of Shyam since he was only functioning as Ranu's
agent and hence had no
criminal intent to do an unlawful act.
(d) Shyam's arrest is legitimate since ignorance of the law is not an excuse.

. Page 24 of 40
89. Consider the following illustrations and select a statement that qualify as a
crime:
(a) A mother knowingly slaps his son for doing a wrong act.
(b) A mother knowingly slaps his sons for failing in exams.
(c) Knowing that the government announced janta curfew on every Sunday with no
exception, then also jatin
went to the medical for buying essential medicine of cancer which is necessary for
treatment.
(d) Salman bhai killed a black buck, a rare species, in self defence.
Passage (Q90.-Q.94): Usually when a contract is breached, the court orders specific
performance of the contract
or directs the defaulting party to pay damages to the aggrieved party. Frustration
renders such defaulting party
absolved of that breach, because of circumstances that made the contract unable of
execution. The concept of
frustration in India has two elements: Section 32 and Section 56. Further, it
differentiated between the two by
observing that while an impossibility under section 32 is an internal force of the
contract, the impossibility is
external in nature under section 56. The Indian courts however, seemed to have
forgotten this distinction in
subsequent cases and have ignored section 32 while interpreting the law of
frustration. The test of impossibility
to result in frustration has to be of such a nature, that the contract is incapable
of being performed in the
reasonable foreseeable future. In this case, the impossibility was a temporary
impossibility since the war would
end in the near future, and thus the contract was capable of being performed at a
later stage. However, instead
of concisely interpreting that delay did not make performance of a contract
impossible, the court delved into
expanding the meaning of 'Impossibility' under section 56.
Source: https://www.legalserviceindia.com/legal/article-8368-is-doctrine-of-
frustration-itself-is-frustrated-by#judicial-interpretations.html
90. Choose a statement which is correct in the light of the passage given?
I. The fundamental rule of contract law holds that both parties to a contract must
fulfil their contractual duties.
II. Doctrine of frustration relieves a party from performing its duties under a
contract.
III. An impossibility that was within the contemplation of the parties comes under
section 32.
IV. a supervening impossibility, which could have never been imagined by the
parties, comes under the purview
of section 56
(a) II
(b) I, III & IV
(c) II, III & IV
(d) All of the above
91. Across town at The Kings Roost in Silver Lake, Roe Sie is a local artisan flour
miller and supplier. The Kings
Roost is also McLaughlin's supplier for San Pedro Sourdough. The effects of two
years of ongoing drought are
finally hitting crops across the entire country. The USDA released a report on July
6 that rated 98% of the
country's wheat fields as being on land that is experiencing some form of drought.
Roe has raised the price of
breads and other items that require wheat flour as an essential ingredient as a
result of reduced wheat harvest.
Many contractors sued Roe for raising the price over what was originally agreed
upon in the supply contract
thereby for breach of contract. Decide
(a) Roe will be held liable for breach of contract as a party should not be
absolved solely on the basis of its
obligation becoming arduous.
(b) Roe will not be held liable for breach of contract as it became impossible for
her to comply with contractual
terms by virtue of a supervening event under section 32.
(c) Roe will be held liable as frustration has to be of such a nature, that the
contract is incapable of being
performed in the reasonable foreseeable future.
(d) none of the above.

. Page 25 of 40
92. Assertion: It will not be entirely untrue if one says that the principles of
frustration are being frustrated by the
judicial interpretations.
Reason: No party is considered at fault. If a contract is found to be frustrated,
(a) Both A and R are true but R is not correct explanation of A.
(b) Both A and R are true and R is correct explanation of A.
(c) A is true but R is false.
(d) A is false but R is true.
93. Choose a remark from the context of the passage that characterizes an anomaly:
I.Frustration of a contract makes the contract void, and discharges the parties of
the contractual obligations.
II.Frustration of a contract occurs without the fault or control of either party,
and therefore, a party should not
be made to compensate in such event.
III.It is hoped that the Indian judiciary sheds some light into such issues and
provide a suitable remedy for cases
of frustration of contracts.
IV.If the contract is unable to be performed, the contract is frustrated.
(a) I & III
(b) II & III
(c) III
(d) None of the above
94. The landowner engaged into an agreement for agricultural use of a property
located in PoK, and it was said that
the legal battle between Pakistan and Kashmir would be mostly won by India, and the
area would become part
of Indian Territory. The landowner leased the property to one of the farmers for
agricultural purposes. The
territory was thereafter taken over by the Pakistani army permanently. What will be
the outcome of the current
case?
(a) The landowner will have to pay for damages aggrieved by the farmer.
(b) The landowner will be taking defense of doctrine of frustration.
(c) The situation would be eligible for frustration of contract.
(d) Since it was not a case of temporary impossibility and thus would lead to
frustration of the contract.
Passage (Q95.-Q.99): A tort is a civil wrong that causes harm to another person by
violating a protected right.
The specific right give rise to the unique “elements” of each tort. Elements are
the essential facts that are required
to be proven. Courts impose liability for torts to compensate an injured party for
an act or an omission that causes
harm. One is never “guilty” of a tort, as that is a term from the criminal law that
implies a violation of some
moral standard. One who commits a tort is a tortfeasor; the tortfeasor is “liable,”
rather than guilty. Tort liability
is meant to monetarily reimburse the tort victim for the harm caused them by the
tortfeasor. Other remedies are
also possible, in restitution or injunctions. A tort may arise from intentional
acts, or from negligent acts
(frequently an omission when there was a duty to act), or from the violation of a
statute. The idea of tort law is
that people are liable for the consequences of their actions. Under most tort laws,
the injury suffered by the
plaintiff does not have to be physical. Torts may include causing emotional
distress or a violation of personal
rights (e.g., the “right to privacy”). There are different types of torts based on
the rights violated.
(Source: https://www.researchgate.net/publication/329718260_Tort_and_Liability)
95. Assertion: Damages must be proved to a certainty; estimates are not sufficient.
Reason: The concept of proximate cause is a legal fiction that acts as a check on
imposing damages that are too
speculative or remote.
(a) Both A and R are false and R is not the correct explanation of A.
(b) Both A and R are true and R is correct explanation of A.
(c) A is true but R is false.
(d) A is false but R is true.

. Page 26 of 40
96. Delhi has a bunch of frightful entertainment venues for adventurous souls. And
the Haunted House is one of the
Scariest House in Delhi mall. The entryway to the haunted mansion warns all guests
that it is "not for faint
hearted people" Ravi, an adventure seeker, purchased a ticket for the house's
thrilling haunted tour, as promised
by the organizers. Ravi began hallucinating following the visit. He experiences and
observes things that his peers
cannot fully comprehend. Ravi filed a lawsuit against the organizers because the
visit disrupted his mental
serenity and caused him emotional suffering. Decide
(a) The organisers will be held legally responsible because the victim experienced
severe mental distress after
visiting the haunted house.
(b) The organisers will not be held liable for tort because Ravi consciously and
deliberately put himself in danger
and negligence exists on ravi’s part.
(c) The organizers will be held liable since the tortfessor cannot escape liability
by simply posting a disclaimer
on the board and thus negligent
(d) Ravi will not be held entitled for any claims since the defendant is not at
fault.
97. A short coach journey from the centre of Argentina's thriving capital city
Buenos Aires is the most dangerous
zoo in the world, Zoo Lujan. What attracts the many visitors is that you don't just
get to look at the animals
through cage bars, but you are allowed inside to touch the animals. A terrified
elephant that was startled by the
visitors, attacked a group of tourists while on safari. They filed a case alleging
that they obeyed clear instructions
to remain silent and then also got injured as a result. It was claimed that the zoo
has not been feeding animals
properly lately. Decide
(a) Zoo will be held liable as it was negligent.
(b) Zoo will not be held liable for the inevitable and unforeseeable accident.
(c) Zoo will be held liable as they owed a duty of care towards its visitors.
(d) Zoo will not be held accountable as a tort only arises from intentional acts.
98. Following an earthquake measuring of 2.5 on richer magnitude, a lion escaped
from the zoo by breaking low
quality bars. One of the visitors died after falling prey to the lion. Who will
bear liability for the injuries?
(a) Zoo officials will be held guilty for their negligence.
(b) Zoo authorities will be liable for their negligence.
(c) The zoo authorities will be held liable for keeping a dangerous animal, the
escape of which resulted in the
plaintiff's injury.
(d) The authority of the zoo will not be held liable since the earthquake will be
deemed an inevitable event.
99. The statistics for dying on a tandem skydive are even less. Over the course of
the past decade, there has only
been one tandem student fatality per every 500,000 jumps, which is a. 0002% chance
of dying. Chandu dies of
a heart stroke while skydiving. He also signed a waiver from which eliminates
liability of any injuries caused
while performing the adventure. Decide.
(a) Signing the waiver form totally absolves the defendant's liability in the
current case.
(b) Chnadu signed the waiver hence will be unable to seek compensation from the
organizers.
(c) Chandu will not be reimbursed for a damage caused by the defendant's omission.
(d) Chnadu will not be entitled to any compensation since the defendant was not
negligent.
Passage (Q100.-Q.105): The Bombay High Court last week acquitted a person of murder
charges on the ground
that extra-judicial confession made by the accused to a stranger cannot be the
basis of conviction.
A Bench of Justices Sadhana Jadhav and Prithviraj Chavan observed that an extra-
judicial confession would
in normal course be made only to a person in whom the confessor reposes faith and
not a passer-by with whom
one was recently acquainted.
“Accused had given graphic details of the act committed by him including the role
of each of the accused persons
and the manner in which they had killed both the deceased. It is rather very
difficult to accept that the accused
would make an extra judicial confession to a stranger,” the High Court noted.

. Page 27 of 40
The Bench thus acquitted the petitioners of charges of murder, causing
disappearance of evidence and common
criminal intention. With the case centered on circumstantial evidence, the
prosecution highlighted an extra#judicial confession by the accused to one of the
witnesses as a key piece of evidence.
The defence stated that accused had "only a stray acquaintance" with the witness
and therefore, the confession
would not inspire confidence. The Court accepted this contention, noting that it
was a well-known fact that the
accused worked as a waiter in a hotel which the witness used to visit once a week.
The Bench noted that the accused had no reason to repose faith in a customer of the
hotel.
“It is also clear that there is no independent corroboration to the alleged extra
judicial confession. The manner
in which it is said to have been made appears to be improbable and imprudent,” the
Bench added.
As a result, the High Court stated that while an accused might be convicted based
on an extrajudicial confession,
the confession must inspire trust, which it did not in this case. Even in cases of
deciding culpability on basis of
extra-judicial confession testimony shall be corroborated then person can be held
liable.
Source: https://www.barandbench.com/news/cannot-accept-confession-made-to-stranger-
bombay-high-court#acquits-murder-accused
100. Shubh had committed the murder of the man who had married his ex-wife, but now
was depressed by the same
and was desperately feeling like confessing. He had already tried confessing to a
tree but to no avail and was
feeling gloomier by the day. One such day, while drinking in a bar, when he had had
one too many drinks, he
finally blurted his secret to the barkeep who promised that he would stay put.
However, the second Shubh left
the barkeep ran to the nearest police stated and ratted on Shubh. Would this amount
to extra-judicial confession?
(a) No, as Shubh did not intend to confess, and was inebriated at the time.
(b) Yes, as Shubh, although inebriated, still intentionally confessed.
(c) No, as there is no relationship of trust here.
(d) Yes, as a reasonable trust has been formed here.
101. In the above case, assuming other facts to be true, had Shubh confessed here
to his long-estranged brother with
whom he had not spoken to for 15 years, and had recently gotten in touch with,
instead of the barkeep, would
the relation of trust being founded?
(a) No, as here a mere familial relation was not present.
(b) No, as the two, even though brothers, do not seemingly have trust and
confidence.
(c) Yes, as they are brothers and are bound to trust each other.
(d) Yes, as a mere decade or two cannot chip away at the strong familial ties
between two brothers.
102. While considering a theft case, the evidence before the court was very foggy
and unclear, and the same led to
the court then moving on to eyewitness’s evidence wherein two people were standing
at place of incident, who
had attested as being present at the scene of crime, had been asked to testify. One
of these claimed that the
accused himself confessed to him about committing the theft and had then run away
which is confirmed by other
person standing. Would this person’s testimony amount to be admitted as extra-
judicial confession?
(a) Yes, as he bore witness to the confession that was rendered by the accused.
(b) Yes, as he heard the confession same was reaffirmed by other person too.
(c) No, as there is no relation of trust present here.
(d) No, as one cannot identify as both an eyewitness and one who has heard
confession.
103. In the above case, had the extra-judicial confession been made to a police
officer by the accused, who had wanted
to repent for his crime and wanted his sentence to be reduced, would the same be
admissible?
(a) Yes, as the intent behind the confession is one to repent.
(b) Yes, as he has approached a fiduciary authority to confess to.
(c) No, as there is no relation of trust present here.
(d) No, as the police officer did not take the confession in front of a magistrate.

. Page 28 of 40
104. After having committed a murder, Jai had desperately wanted to get it off his
chest, and in doing the same, he
confessed about it to Veeru, who had just cleared his magistrate level exam but the
same was not known to Jai.
Veeru, being a judicial officer, decided to take an action against his friend and
thus proceedings were intiated
against Jai wherein Veeru had decided to testify against his friend. Is there a
relation of trust in the present case?
(a) Yes, as both Jai and Veeru were friends and Jai trusted Veeru.
(b) No, as Veeru had suppressed material information from Jai regarding his status
as a magistrate.
(c) Yes, as Jai had confessed to his friend under confidence and there was a
relation of trust here.
(d) No, as Veeru had betrayed Jai in not informing him about his judgeship.
105. In the above case, had Jai known that Veeru was a police officer, and yet, had
continued to confess to his friend
about the crime committed by him, would the same amount to there being admissible
extra-judicial confession
being made?
(a) No, as Veeru is in a dominant position here and cannot act as a neutral third
party here.
(b) Yes, as Veeru, although a police officer, is still in a relation of trust with
Jai.
(c) No, as Jai should have known better than to confess to a police officer.
(d) Yes, as even police officers can enter into relationships of trust.

. Page 29 of 40
SECTION - D: LOGICAL REASONING
Directions (Q.106-Q.110): Read the following passage and in light of the same,
answer the questions that
follow.
Kerala Governor Arif Mohammed Khan has posed a pertinent question for the state's
political class to answer.
"Is it fair to finance cadres of political parties using public money?" he asked,
questioning the practice followed
by successive governments to deploy party cadres in large numbers in the personal
staff of ministers.
Unlike the Centre and most states, in Kerala, such persons are appointed as
permanent employees on a co#terminus basis so that they would be eligible for
statutory pension.
While a Union Cabinet minister is entitled to a 15-member personal staff, the
majority of who are on deputation
from government service, in Kerala a minister can have double the number. The LDF
regime has cut the personal
staff strength of a minister to 25 while keeping it at 30 for the CM. In addition,
the speaker, deputy speaker and
chief whip can have 25 members as personal staff. Currently, there are about 400
such direct recruits in service
- almost everyone is an active member of a party part of the ruling front. All of
them draw handsome salaries
too, ranging from Rs 23,000 to Rs 1,60,000.
The state exchequer shelled out Rs 170 crore in the last five years for paying
salaries of personal staff. In 1994,
the Congress-led government, with the wholehearted support of the opposition,
introduced a statutory pension
for all such staff who complete three years of service, that too with retrospective
effect from 1982. Later, the
rules were amended to include the provision for family pension so that the spouse
will get the money if the
pensioner dies. The pension payout for personal staff has grown 100 per cent
between 2013–14 and 2019–20. In
2013, a contributory pension scheme was introduced for government employees. Yet,
personal staff of ministers
whose only eligibility to be in service is their loyalty to their parties are still
covered by statutory pension.
The Governor’s outburst has helped invite attention to an issue buried under the
carpet by the collective
conspiracy of political parties. With its coffers being empty and no schemes for
additional revenue generation in
sight, the government needs to be prudent in spending the money it has. Doing away
with pensions for personal
staff will be a right step in that direction.
106. The information given in the passage supports which of the following
statements?
(a) The government's step of resisting pensions to its members is imprudent.
(b) Cutting the personal staff strength from 30 to 25 is harsh on the staff
members.
(c) The personal staff is drawing way more salaries than what they are entitled to.
(d) The Kerala governor is concerned about where the public money goes.
107. Which of the following is an assumption behind the author’s arguments?
(a) The funds of the government can be used to take care of other needs.
(b) 25 personal staff members is not sufficient for a state-level minister.
(c) A family is left with no sources of income if the pensioner of the family dies.
(d) Public money also funds the public servants.
108. The arguments of the author are strengthened by which of the following pieces
of evidence?
(a) The Unnati scheme for the empowerment of girls in Kerala is way underfunded to
produce any meaningful
results.
(b) The amount of money spent on the well-being of ministers in Kerala is the least
in the country.
(c) Statutory pension for all government staff who complete three years of service,
that too with retrospective
effect from 1982, is deserving.
(d) The government officials work for the betterment of the public, and certain
perks acts as a motivation to
continue to serve the public.

. Page 30 of 40
109. What is the relationship between the two statements in the passage? Choose the
best answer.
Statement 1: With its coffers being empty and no schemes for additional revenue
generation in sight, the
government needs to be prudent in spending the money it has.
Statement 2: Doing away with pensions for personal staff will be a right step in
that direction.
(a) Statement [1] forms the premise of the claim made in statement [2].
(b) Statement [2] forms the premise of the claim made in statement [1].
(c) Statement [1] and [2] are claims made to support the main argument.
(d) Statement [1] and [2] are counter arguments of each other.
110. What is the main reason for the author to suggest that the staff members of
ministers should not be given
pensions?
(a) They were unqualified to secure their respective jobs.
(b) Their pension is putting a dent on the government’s coffer.
(c) The public money can’t be spent on such menial jobs.
(d) The numbers of the personal staff are enormous.
Directions (Q.111-Q.115): Read the following passage and in the light of same,
answer the questions that
follow.
Sri Lanka is facing an economic crisis with long queues in front of petrol
stations, steep rise in prices of essential
commodities and frequent blackouts. Although the COVID-19 pandemic precipitated a
crisis of trade imbalance,
the fundamentals of the Sri Lankan economy have always had serious issues. Debt,
both domestic and foreign,
has been a major problem. Even in February 2020, hardly a few months after Gotabaya
Rajapaksa assumed
office as President, his elder brother and Prime Minister Mahinda Rajapaksa, during
his visit to New Delhi,
wanted India to reschedule the loan. Over the last three months, India has provided
assistance of $2.4 billion
including a $500 million loan deferment and $1 billion credit line to enable the
supply of essential commodities.
Apart from approaching Beijing, Colombo has also sought help from the International
Monetary Fund, shedding
its earlier reservation of taking help from the agency. As soon as the shortage of
certain essential commodities
ends, which the government expects before the start of the Sinhala-Tamil New Year
(which falls in the middle
of April), steps should be taken for economic recovery.
Compulsions of electoral politics should not come in the way of tough measures such
as restructuring the
administration of concessions and subsidies. Mr. Gotabaya Rajapaksa should also use
the scheduled meeting
with the Tamil political leadership to create a road map on the issue of political
devolution and economic
development of the war-affected northern and eastern provinces, among the areas
badly hit by the current crisis.
Perhaps, Tamil Nadu has already started feeling the impact of the crisis with the
reported arrival of 16 persons
from Sri Lanka, including six women and seven children, through illegal means.
Tamil Nadu was home to nearly
three lakh refugees after the anti-Tamil pogrom of 1983. Regardless of the motive
of those who have reached
Tamil Nadu clandestinely, the authorities, both in India and Sri Lanka, should
ensure that the present crisis is
not used to step up smuggling activities and trafficking or whip up emotions in
both countries. On the contrary,
the crisis should be used as an opportunity for New Delhi and Colombo to thrash out
a solution to the Palk Bay
fisheries dispute, a longstanding irritant in bilateral ties.
111. Which of the following represents the central idea of the passage?
(a) Sri Lanka's economy is dipping owing to various causes, with ripple effects in
India.
(b) The crippling effect of pandemic can be witnessed in developing countries.
(c) India has a benevolent attitude towards the neighbouring Sri Lanka.
(d) Political imbalance in a country indirectly leads to economic downturn.

. Page 31 of 40
112. Which of the following can be inferred from the passage?
(a) Deliberations and negotiations have proven to be the major sources in
mitigating the crisis.
(b) New Delhi aims to prevent the crisis as fear of China’s influence looms in the
country.
(c) Political vows and frequent changes in the regimes hinder economic stability of
a nation.
(d) Tamil Nadu and Sri Lanka share historical socio-economic and cultural ties.
113. Which of the following is most likely to be agreed with, in consonance with
the above passage?
(a) IMF, an international organisation, lends economic aid to developing countries.
(b) Palk Bay fisheries dispute has led to disruption of mutual ties between both
India and Sri Lanka.
(c) There have been several instances of cross-border smuggling of goods between
India and Sri Lanka.
(d) Sri Lankas’ economic crisis has the potential to convert into a refugee crisis
in India.
114. Which of the following can be assumed from the first paragraph of the above
passage?
(a) Rajapaksa family, apart from creating political turmoil, has been the cause for
economic slowdown as well.
(b) Concessions and subsidies as a tool of electoral political compulsion have
played a part in the Sri Lankan
economic crisis.
(c) Demand for essential commodities, once fulfilled, will lead to economic
rebound.
(d) Sri Lanka has been facing situation of spiking current account deficit since
Rajapaksa govt took control of
the country.
115. What can be most plausible conclusion from the above passage?
(a) A drastic restructuring of economic affairs is the way out for the Sri Lankan
government.
(b) Instances of Sri Lanka imply that developing country’s economy is further
aggravated by the pandemic.
(c) India has manageable forex reserves which makes it viable for aiding its
neighbour.
(d) India should help Sri Lanka during the crisis and also resolve some of the
bilateral issues.
Directions (Q.116-Q.120): Read the following passage and in light of the same,
answer the questions that
follow.
Despite several Supreme Court rulings in recent years on restricting the
manufacture, sale and use of firecrackers,
flagrant violation of norms continues to plague the Indian fireworks industry,
whose worth is estimated to be
around Rs 3,000 crore. Earlier this week, six migrant workers were killed and 14
others suffered burn injuries in
an explosion at an illegal factory in Himachal Pradesh’s Una district. The factory
had no licence for making
firecrackers and was also not registered with the industries department. Last
month, seven lives were lost in
mishaps in Tamil Nadu’s Sivakasi, the hub of firecracker manufacturing. Safety
concerns are not properly
addressed even by the authorised industrial units, with employees handling
explosives and other chemicals
without requisite training.
It was in October three years ago that the apex court had banned the production and
sale of all crackers — except
the ‘green’ ones (made without toxic ingredients) and those with reduced emissions.
The court had also ordered
that cracker should be sold only by licensed traders and their decibel levels
should be within permissible limits.
The bursting of firecrackers aggravates air pollution during the festival season,
adversely impacting people with
respiratory ailments. On Diwali last year, Delhi and some other states had imposed
a blanket ban on all kinds of
crackers, even though its enforcement left a lot to be desired.
An outright ban no doubt faces practical constraints, but it is feasible to reduce
the availability and supply of
firecrackers. Making the licensing process tougher would go a long way in ensuring
that manufacturing
operations are scaled down considerably. At the same time, strict monitoring is
needed to weed out the illegal
factories. The government-approved manufacturers who show blatant disregard for
safety rules, endangering the
lives of workers, should be delicensed. The attempts to circumvent the ban —
selling products containing
harmful chemicals under fake ‘green cracker’ labels and QR codes — must be scuttled
by the regulatory
authorities. The public can do its bit by shunning firecrackers altogether or
limiting their use to the bare
minimum.

. Page 32 of 40
116. Which among the following is the main point of the passage?
(a) It is high time to scale down production of firecrackers as a response to
safety concerns and an increasing
air pollution.
(b) The governments and the courts have failed in their duty to keep India’s air
clean.
(c) Green crackers should become the new norm during the festival seasons.
(d) Illegal factories and owners should be handled strictly according to the law to
curb ascending production of
firecrackers.
117. Which among the following, if true, weakens the author’s arguments?
I. The factory in Himachal Pradesh that claimed several lives was not used for
storing firecrackers.
II. The tax collected in the firecrackers industry contributes to 0.1% of India’s
GDP.
(a) Only I
(b) Only II
(c) Both I &II
(d) Neither I nor II.
118. Out of the following, what can be inferred from the given passage?
I. A complete ban on firecrackers would be the government’s best foot forward.
II. Green crackers are better firecrackers than some other forms of them.
III. Shutting off illegal factories may result in less casualties due to
firecrackers.
(a) I and II
(b) Only II
(c) II and III
(d) I, II and III
119. Which among the following has been assumed by the author in the first two
sentences of the last paragraph?
(a) Reduced availability and supply of firecrackers would instigate manufacturers
to establish illegal factories.
(b) Factory owners would not take the illegal route to produce firecrackers if they
don’t get the manufacturing
licence.
(c) The authority of the Supreme Court falls on the entire territory of India.
(d) An outright ban on burning firecrackers will ensure the final products move out
of India.
120. Which of the following is not true according to the information given in the
passage?
(a) The Supreme Court’s decision of banning the production of all crackers, with
some exceptions came in 2018.
(b) The passage contains some suggestions for the general public to take to
contribute to the cause.
(c) The passage contains some suggestions for the authorities to undertake to
reduce the manufacturing
operations.
(d) An outright ban on the sale and purchase of firecrackers will bring in its wake
certain challenges.
Directions (Q.121-Q.125): Read the following passage and in light of the same,
answer the questions that
follow.
In refusing to entertain ‘sealed covers’ submitted by the government or its
agencies, the Supreme Court has made
a noteworthy and welcome shift away from this unedifying practice. At least two
Benches have spoken out
against it. Recently, in the Muzaffarpur shelter home sexual abuse case, Chief
Justice N.V. Ramana wondered
why even an ‘action taken’ report should be in a sealed envelope. The use of
material produced in a ‘sealed
cover’ as an aid to adjudication is something to be strongly discouraged and
deprecated. However, it gained
much respectability in recent years, with contents withheld from lawyers appearing
against the government, but
being seen by the judges alone. Unfortunately, in some cases, courts have allowed
such secret material to
determine the outcome. In a recent instance, the Kerala High Court perused
confidential intelligence inputs
produced in a sealed envelope by the Union government to uphold the validity of
orders revoking the
broadcasting permission given to Malayalam news channel Media One on the ground of
national security.

. Page 33 of 40
It is quite disconcerting to find that courts can rule in favour of the government
without providing an opportunity
to the affected parties to know what is being held against them. In this backdrop,
it is significant that the Supreme
Court has decided that it will examine the issue of ‘sealed cover jurisprudence’
while hearing the channel’s
appeal. For now, the apex court has stayed the revocation order and allowed the
channel to resume broadcasting.
It is true that the law permits the submission of confidential material to the
court in some cases. In addition,
courts can order some contents to be kept confidential. The Evidence Act also
allows the privilege of non#disclosure of some documents and communications. The
government usually justifies the submission of secret
material directly to the court, citing national security or the purity of an
ongoing investigation. Courts have often
justified entertaining material not disclosed to the parties by underscoring that
it is to satisfy their conscience.
[Source: The Hindu]
121. Which of the following represents the central idea of the passage?
(a) Sealed cover defies the conscience of justice delivery system in the country.
(b) Courts proceeding with the practice of sealed cover devalue the rights of
parties to the case.
(c) Supreme court has been criticising the practice of sealed cover.
(d) The practice highlights decaying of basic freedoms to the accused.
122. Which of the following is the author most likely to agree with in accordance
with passage?
(a) Sealed cover as a practice should be discouraged as it takes away all the
rights of accused.
(b) Judges are often been criticised and are in spotlight when they promote such
practices.
(c) Sealed cover makes the state avoid deep scrutiny of the proportionality of its
restrictions on freedom.
(d) Courts in all the cases have vociferously argued against this arbitrary
practice.
123. Which of the following can be inferred from the above passage?
(a) The time has come for courts to intervene and red-flag the ad-hoc practice.
(b) Majority of High courts have justified this practice under the garb of
“national security” concerns.
(c) Supreme court’s views on the practice have an influential impact on several
high courts.
(d) Judiciary is to be seen more inclined towards executive by favouring such
practices.
124. Which of the following will be most appropriate course of action to curb or
limit this practice?
(a) The action can be regarded as just only when it passes through proportionality
test.
(b) The act which mandates such practice should be repealed.
(c) The practice has entered into realm of justice delivery system and courts have
to be equipped to it.
(d) High Courts should start exerting their influence to restrict the practice in
equivalent courts.
125. What purpose does the boldfaced statement serve in the passage?
(a) It is an argument to support how the practice is a worrying cause in judiciary.
(b) It is a premise to support how this practice erodes the tenets of justice.
(c) It is a premise to support that courts have broadened the concept of justice.
(d) It is an argument to support that courts have expanded their inquisitive role
to uphold the democratic value.
Directions (Q.126-Q.130): Read the following passage and in light of the same,
answer the questions that
follow.
In recent weeks, Beijing has repeatedly blamed NATO for the crisis in Ukraine.
While claiming to stay neutral,
it has moved to reaffirm ties with Russia, which Beijing on Monday described as
“rock solid”. When the two
countries’ leaders met for a summit on February 4, China backed Russia on its
concerns on NATO’s eastward
expansion in Europe, and Russia returned the favour with both criticising the U.S.
Indo-Pacific strategy. Beyond
their already deep political and economic linkages, these mirrored concerns on U.S.
alliances are emerging as a
powerful binding glue in the China-Russia axis. New Delhi will need to consider how
this will impact its close
relations with Russia. By explicitly equating the Quad, which is not a military
pact, with other security
agreements, China now also appears to be clearly situating India as a part of the
U.S. “exclusive club”. New

. Page 34 of 40
Delhi has rejected that notion. Only last month, External Affairs Minister S.
Jaishankar said “interested parties”
were making a “lazy analogy of an Asian NATO” and India was not a U.S. treaty ally.
Indeed, some in New
Delhi have come to view Beijing’s aggressive moves along the LAC in 2020 as a
warning sign to deter India#U.S. relations. India’s firm, and correct, response has
been to hold the line, and continue deepening ties not only
with the U.S. and the Quad but also other Indo-Pacific partners to underline it
will not be swayed. The other
message from India has been that sensitivity to concerns has to be mutual, and
cannot be demanded from one
side when ignored by the other; China’s relations with Pakistan being a case in
point. Mr. Wang did acknowledge
that recent “setbacks” in ties suited neither India nor China — a view New Delhi
shares. The two sides will meet
on March 11 for the next round of military talks to take forward LAC disengagement.
As India and China
continue to seek a much-needed modus vivendi to restore ties from the lowest point
in decades and ensure peace
on the border, they will also need to have a broader conversation about global
currents that are reshaping their
bilateral relations.
[Source: The Hindu]
126. Which of the following forms the central idea of the passage?
(a) The relations in geopolitics are tensed which could further add on to land
border disputes between India#China.
(b) China has been sceptical of growing Indo-Russia relations since India maintains
close relations with Russia.
(c) India wants to reaffirm its ties with China, thereby bringing normalcy in
relations.
(d) Russia-China relations have been on a strong edifice owing to their shared
interests in geopolitics, while
sending India subtle messages.
127. Which of the following can be inferred from the above passage?
(a) Geopolitical tensions have an influence in shaping bilateral ties of countries.
(b) India has a flexible stance to maintain when it comes to deepen its ties with
Russia.
(c) NATO’s eastward expansion is the sole reason for robust ties between China-
Russia.
(d) China has insensitive relations with Pakistan, causing further escalations at
LAC.
128. Which of the following statements is most likely to be agreed with?
(a) China since the formation of Quad, has been repeatedly blaming it as “Asian
NATO”
(b) China and Russia have been sharing cordial position on economic and political
front.
(c) India-US relations have been hampered owing to threats emerging from LAC.
(d) Sino-Indian trade has not been impacted owing to tensions escalating at LAC.
129. Which of the following can be concluded from the above passage?
(a) India is keen on building QUAD a military alliance to counter crisis at LAC.
(b) NATO’s expansionist policy should be discouraged to de-escalate further
tensions.
(c) A balance of power is a win-win situation to incorporate sound negotiations.
(d) Russia has tried to balance its stance on China and India.
130. ‘Beyond their already deep political and economic linkages, these mirrored
concerns on U.S. alliances are
emerging as a powerful binding glue in the China-Russia axis.’ The aforementioned
statement is
(a) A fact. (b) An inference. (c) A judgement. (d) A suggestion.

. Page 35 of 40
Directions (Q.26-Q.30): Read the following passage and in light of the same, answer
the questions that follow.
Signs of a confrontation between Raj Bhavan and the elected government in a State
are not infrequent in the
country. The onus often appears to be on the Chief Ministers to avert a
constitutional crisis rather than pursue a
confrontational course. One way of seeing these developments is to attribute them
to the appointment of those
who have been politically active in the recent past as Governors and the partisan
role they play as agents of the
Centre. However, the problems may have to do with the way they understand their own
powers. Constrained by
the ‘aid and advice’ clause in their routine functioning, some Governors seem to be
using the discretionary space
available to them to keep regimes on tenterhooks. A Constitution Bench of the
Supreme Court laid down in 1974
that the President and Governor shall “exercise their formal constitutional powers
only upon and in accordance
with the advice of their Ministers save in a few well-known exceptional situations”
— “situations” also
illustratively listed. Yet, there is the extraordinary situation of some Governors
not acting upon requests to grant
clemency or assent to Bills.
Much of the conflict arises due to the Constitution itself. It fixes no time-frame
for the Governors to act, and
contains, in Article 163, an unusual power to choose what is in their discretion
and what is not, with the courts
being barred from inquiring into whether any advice and, if so, what advice was
given. The Sarkaria Commission
on Centre-State relations recommended no change in this scheme, but it is time it
is revisited. While as the
‘lynchpin’ of the constitutional apparatus, Governors indeed have a duty to defend
the Constitution and
encourage or caution the elected regime, the impression that Governors are not
obliged to heed Cabinet advice
persists in some areas. At a time when regional political forces are actively
seeking to be heard by the Centre, it
may be time that the provisions relating to the Governor’s role are amended.
Identifying areas of discretion,
fixing a time-frame for them to act, and making it explicit that they are obliged
to go by Cabinet advice on
dealing with Bills can be considered. In addition, as suggested by the M.M. Punchhi
Commission, ending the
practice of burdening Governors with the office of Chancellor in universities
should also be considered.
131. If the information given in the passage is true, then which of the following
can be logically inferred from the
passage?
(a) The Supreme Court’s decision of 1974 is being bypassed by some governors.
(b) The provisions of the Constitution are crystal clear, with little
discrepancies.
(c) Governors play the roles of the Centre’s agents, which is unfortunate.
(d) The Constitution has failed miserably in defining the roles of the governor.
132. Which of the following pieces of evidence would strengthen the arguments of
the author?
(a) Disputes between the governor and the chief minister in states very often lead
to constitutional crises due to
which the citizens suffer.
(b) Fixing time frames for the governor to approve the bill has led to wrong
decisions in the neighbouring
countries.
(c) There are no regional parties in India that present a real challenge to the
party at the centre.
(d) Much of the conflict arises due to the Constitution itself that has given
unlimited powers to a governor.
133. The best representation of the main idea of the passage is:
(a) The disputes between the chief minister and the governor should be resolved in
a timed-manner.
(b) The time may have come for clarifying the role and functions of the governor.
(c) The Constitution should be rewritten to accommodate the governor and the chief
minister.
(d) Governor’s exceeding their constitutional limits symbolises the colonial
mindset.
134. Which among the following is an expression of opinion rather than fact?
(a) Governors have a duty to abide by the Constitution of India.
(b) Three previous governors of Tamil Nadu created a constitutional crisis.
(c) Governors need to use their powers in a discretionary manner.
(d) Chief Ministers are ranked below the governors in the hierarchy.

. Page 36 of 40
135. The author has used which of the following as a premise to support his claims?
I. Time frame should be fixed for the governors to assent or dissent to the bill.
II. The Constitution has not fixed a time frame for the governors to take
decisions.
III. The Supreme court in 1974 gave unlimited powers to the President and the
Governors.
(a) Only I
(b) Only II
(c) II and III
(d) I and II

MOCK 43

ECTION-A: ENGLISH LANGUAGE


Directions (Q.1-Q.30): Read the following passage carefully and answer the
questions that follow.
Passage (Q.1-Q.5): Psychologists have been studying burnout for five decades, and
certain researchers have
long warned of burnout within their ranks. In the last two years, the cultural
status of burnout has radically
changed. No longer is “burnout” a specialized term describing a state of depletion
among workers in certain
strenuous human-services professions. Burnout is now a conflagration, blazing
through the ranks of elite
professionals with greater firepower. Everyone, from veterinarians to Amazon
account managers, complains of
burnout; the New York Times seems on the verge of creating a burnout beat if its
churn of coverage is any
indication. How did “burnout” become a keyword of our age?
The pandemic, of course, has much to do with the term’s newfound popularity. The
stress and social dislocation
resulting from a poorly managed, seemingly interminable public health emergency put
limits on what workers
could tolerate. Yet burnout’s ubiquity cannot be attributed to Covid alone.
Culturally while exhaustion among
nurses, teachers, and other frontline workers accounts for some of the upticks in
burnout talk, the term has been
seized most avidly by highly educated remote workers in such fields as technology,
finance, and media. Is
burnout, then, really a syndrome resulting from chronic workplace stress, as the
World Health Organization has
classified it? Is it a form of depression? Or is it a mark of illusion with which
certain workers can evade additional
responsibilities?
The psychologist Christina Maslach sees burnout as having three components:
exhaustion; cynicism or
depersonalization and a sense of ineffectiveness or futility. Exhaustion is easy to
brag about, inefficacy less so.
Accounts of the desperate worker as labour-hero ignore the important fact that
burnout impairs your ability to
do your job. A “precise diagnostic checklist” for burnout, also providing medical
aid to people who are the real
sufferer of burnout Malesic writes, would curtail loose claims of fashionable
exhaustion, while helping people
who suffer from burnout seek medical treatment.
Malesic, however, is interested in more than tracing burnout’s clinical history. A
scholar of religion, he diagnoses
burnout as an ailment of the soul. It arises, he contends, from a gap between our
ideas about work and our reality
of work. Americans have powerful fantasies about what work can provide: happiness,
esteem, identity,
community. The reality is much shoddier. Across many sectors of the economy, labour
conditions have only
worsened since the 1970s. As our economy grows steadily more unequal and
unforgiving, many of us have
doubled down on our fantasies, hoping that in ceaseless toil, we will find whatever
it is we are looking for,
become whoever we yearn to become. This, Malesic says, is a false promise. While
the book rarely veers into
polemic, it has a strong moral-religious bent. It is an attack on the cruel idea
that work confers dignity and
therefore that people who don’t work—the old, the disabled—lack value. On the
contrary, dignity is intrinsic to
all human beings, and in designing a work regime rigged for the profit of the few
and the exhaustion of the many,
we have failed to honour one another’s humanity.
1. Which one among the following is NOT one of the questions that have been asked
in reference to burnout?
(a) Is burnout an outcome of prolonged professional tension?
(b) Is it a form of depressive disorder?
(c) Is it true that burnout is limited to people working in the IT industry or
similar professions that require mental
cognition?
(d) Is it a form of a facade that laid back workers have come up with to take
minimum responsibilities?
2. Which of the accompanying is CORRECT in reference to the words of Maslach?
A. It's simple to boast about tiredness but it's harder to boast about
inefficiency.
B. A specific diagnosis for burnout will reduce false claims of tiredness while
also providing medical aid to
people who actually suffer from burnout.
C. Tiredness, loss of efficiency and futility are the three components of burnout.
(a) Only A and B (b) Only B and C (c) Only A and C (d) All are correct.

. Page 3 of 44
3. How has the standing of burnout changed over the last two years culturally?
(a) The effect of Covid has changed the definition of burnout over the years as the
pressure and social
disarticulation put limitations on what workers could tolerate.
(b) Burnout is no longer a term that touches upon the exhaustion among workers in
tiring professions but rather
it is now an inferno that shows itself in professionals with mightier firepower.
(c) Burnout was just an impression among the professionals who worked for fifty-
five to sixty years of age and
now due to remote work, it has reached among people in their twenties.
(d) Burnout no longer afflicts the adults; it is now prevalent amongst children of
different age groups.
4. What is the tone of the author in the passage?
(a) Humanistic (b) Laudatory (c) Provocative (d) Speculative
5. Which of the accompanying is INCORRECT in reference to the passage?
(a) Malesic supports the idea that the origin of burnout is due to the difference
in our vision and pragmatism of
our work.
(b) Since the 1970s, labour conditions have deteriorated in many sectors of the
economy.
(c) Malesic says that all human beings must feel dignified only when they work.
(d) Americans have high expectations from their work but the reality is altogether
different.
Passage (Q.6-Q.10): With so much conflicting financial advice available, it can be
hard to keep it all straight,
much less figure out which one is best for you. Some approaches are old-fashioned,
tried and true; others are
trendy—popularized by financial experts with large media platforms. You may have
heard of some of these
buzzed-about strategies, but what do they mean? The first is Bogleheads. This
strategy advocates for saving at
least 20% of your income, investing early and often, never trying to “time the
market,” finding a risk profile that
is not too high or too low, investing in broadly diversified low-cost (low expense
ratio) index funds, and staying
the course through the stock market’s ups and downs.
In his speech “Investing with Simplicity” Bogle said, “Simplicity is the master key
to financial success.” He
explained that rather than doing extensive research on and tracking of individual
stocks, Bogleheads advocate
achieving portfolio diversification by following a simple investing philosophy: the
creation of a three-fund
“lazy” portfolio that includes a total stock market index fund, a total
international stock index fund, and a total
bond market fund (a percentage that Bogle recommends should be equivalent to your
age—if you’re 40, allocate
40% of your portfolio to bonds). Then, outside of periodic rebalancing, set it,
forget it, and watch your money
produce returns with the market over time.
The next technique is FIRE (Financial Independence Retire Early). You may be a
candidate for this aggressive
strategy if you place a high priority on retiring early, earn a high income and
have the discipline to invest 50-
75% of your income yearly. While the idea behind FIRE is not new, the strategy
gained popularity more recently
courtesy of bloggers like Peter Adeney, the former software engineer behind Mr.
Money Moustache, who retired
at age 30. Essentially, FIRE comes down to extreme budgeting, serious view of
stocks and shares controlled
spending, and low-cost investing—living frugally and cutting out luxuries in order
to put a minimum of 50% of
your income into inexpensive index funds.
The next method is called the 50/30/20 Rule. This budgeting strategy is: 50% on
needs, 30% on wants, and 20%
on savings or paying off debt. In this paradigm, half your income is allocated to
essentials such as rent, mortgage
payments, groceries, utilities, and gas. Thirty percent goes towards things like
entertainment, self-care,
restaurants, shopping, the gym, or vacations. And 20% is funnelled into savings,
retirement, investment, or debt
payments. Another strategy is the “HELOC strategy”; this somewhat risky approach
involves using a line of
credit—typically a home equity line of credit (HELOC)—like a savings or checking
account to pay for your
monthly expenses and make principal payments on a loan, usually a mortgage.
According to Money, “The basic
idea is to strategically spread your money across different debt products in order
to minimize interest payments
and maximize the amount going to pay down your mortgage principal.”

. Page 4 of 44
6. Which of the following is NOT one of the premeditated conditions that is
indispensable to master the FIRE
financial strategy?
(a) Having the quality to invest half or three-fourths of your income.
(b) Keeping a casual view of stocks and shares and embracing a simplistic approach
to invest.
(c) Be at the receiving end of high-income sources.
(d) Having the vision to retire early in life.
7. Which one of the following statements perfectly explains the HECLOC strategy?
A. It includes carefully dividing your money across several debt products in order
to reduce interest payments
and increase the amount that goes toward paying down your mortgage principal.
B. It is a risky approach in terms of financial strategy.
C. It involves the use of a credit facility like a home equity line of credit—a
savings or checking account to pay
for monthly bills and make principal payments on a loan, usually a mortgage.
(a) Only A and B (b) Only B and C (c) Only A and C (d) All A, B and C
8. Which of the accompanying is NOT a piece of advice that is given by Bogle?
(a) Variation of investment in the stocks in an uncomplicated manner is the key.
(b) He advocated the creation of the three-fund lazy portfolio.
(c) With the omission of monthly rebalancing, set your stocks and fail to recall
it, and observe your money
receive market returns with the passage of time.
(d) Selling all your assets that include even inheritance and keeping them in a
bank or investing in stocks and
using the interest to meet your daily expenses.
9. Explain the ‘50/30/20’ Rule in reference to this passage.
(a) Fifty percent of the income must be spent on items that are requisite for
survival, thirty percent for
entertainment and other amenities and the rest for savings, investments or repaying
loans.
(b) Fifty percent of the income must be spent on savings, investments or repaying
loans, thirty percent for items
that are requisite for survival and the rest for entertainment and other amenities.
(c) Fifty percent of the income must be spent on savings, investments or repaying
loans, thirty percent for
entertainment and the rest for items that are requisite for survival.
(d) Fifty percent income must be spent on entertainment; thirty percent on
investments or repaying loans, and
twenty percent on items that are important for survival.
10. Which of the following statements perfectly highlight the Bogleheads’ financial
strategy?
A. Investing in highly diversified low-cost index funds.
B. Sticking your neck out for maximum profit.
C. Investing frequently and from the initial stages and never endeavouring to time
the market.
(a) Only A and B (b) Only B and C (c) Only A and C (d) All A, B and C
Passage (Q.11-Q.15): When the nine sprinters lined up for the final at the Olympic
Stadium in London, Bolt
was obviously the cynosure of attention. He was the reigning champion and this was
to be his last competitive
race in an individual event. Just about everybody wanted him to sign off with a
gold medal. True, Bolt's sketchy
form this season had been of some concern to aficionados. He had been way below his
career-best timing. There
were niggling injuries to worry about. In the heats, starts had been sluggish and
the finish far from explosive.
But hey, this was Bolt. Never short on hunger for winning, always capable of coming
up with a special effort to
stamp his authority on a sport he had dominated for over a decade, like no sprinter
before him. What could stop
him now?
Ironically, it was to be American Justin Gatlin, his long-standing nemesis, who had
twice been suspended for
drug-taking. Bolt was slow off the blocks. Where in the past he would make up for
this with strong loping strides
mid-race and an explosive kick at the finish, this time it was not to be. That Bolt
finished third- with young

. Page 5 of 44
Cristian Coleman winning silver- is of academic interest. It was gold he was
seeking, and he was beaten to it.
That he clocked his season’s best timing (9.95 seconds) is of no recompense or
consequence. The rest of the
field too was flabbergasted at the turn of events. An overwhelming feeling of
despondency hung over the
stadium, felt even by those watching the race on TV. Everybody wanted Bolt to win
because there had been none
like him before.
Past champions like Jesse Owens and Carl Lewis had been more versatile, but lacked
the same panache,
flamboyance and, certainly, the speed. True, timings improve as a human being
evolves, but even so, Bolt looked
like he was cut from a different cloth. When he started, it was said Bolt was too
tall to sustain the high knee
action so vital in sprints. But he only got better and better. Scepticism later
centred on his showmanship. This
will be his downfall, argued critics. In fact, this became his method of self-
motivation. The comparison with
Muhammad Ali is valid to an extent, for the latter had several other dimensions to
his persona beyond sport. But
like the great boxer, Bolt was colourful, not averse to braggadocio, and loved to
psyche his opponents.
Like Ali did for boxing, Bolt made sprinting sexy and rescued it from its comatose
condition. He filled stadium
wherever he ran. The unending string of successes inspired hundreds of young men
and women – and not just in
his native Jamaica- to take to sprinting. Perhaps most importantly, he pulled
athletics out of the mire of mistrust
caused by drug- taking athletes. For a long spell, rivals and officials, frazzled
by his achievements, put Bolt
under intense scrutiny to see if he was unclean. As yet, there hasn’t been any
taint on him. The immediate- and
not unfounded- parallel to Bolt’s final race comes from cricket. In Don Bradman’s
last Test innings, Bradman
needed just four runs to finish with a Test career average of 100. As it happened,
he got out for a second- ball
duck, finishing with 99.94. Yet the failure of both Bradman and Bolt in their final
performance, deeply tragic in
one sense, find redemption when juxtaposed with Aristotle’s Barbara. Within the
framework of human
limitations, their achievements have not just been extraordinary, but almost
immortal. Hence nothing hampers
the legacy created in decades
11. Which of the following is true as per the passage?
I. Everybody wanted Bolt to win because there had been none like him before.
II. Past champions like Jesse Owens and Carl Lewis had not been more versatile, and
lacked the same panache,
flamboyance and, certainly, the speed.
III. For a long spell, rivals and officials, frazzled by his achievements, put Bolt
under intense scrutiny to see if
he was unclean.
(a) Only I (b) Only II (c) Only III (d) Both I & III
12. According to the passage, which of the following was supposedly targeted by the
critics to become the reason(s)
for Bolt's downfall?
I. When he started, it was said Bolt was too tall to sustain the high knee action
so vital in sprints.
II. It was later said that his showmanship would take him down.
III. His unending string of successes.
(a) Only II (b) Both I & II (c) Both II & III (d) Both I & III
13. ‘But like the great boxer, Bolt was colourful, not averse to braggadocio, and
loved to psyche his opponents. The
boldfaced word belongs to which parts of speech?
(a) Noun. (b) Verb. (c) Adjective. (d) Adverb.
14. Which of the following could be concluded from the passage?
I. Even though Bolt failed to win a gold in his final race, it would not hamper his
decade worth of legacy.
II. Bolt defined sprinting in a way, which was never done before, in turn, bringing
it to the fore.
III. Bolt did not manage to win a gold in his final, failing in posing his season's
best timing.
(a) Only I (b) Only II (c) Only III (d) Both I & II

. Page 6 of 44
15. Which of the following reflects the most suitable title for the passage?
(a) Bolt: indefatigable.
(b) A bolt from the blue.
(c) Bolt: a colourful personality.
(d) Bolt: an untainted legacy.
Passage (Q.16-Q.20): To be self-confident is the imperative of our time. While
women and girls are subjected
to intense appearance pressures and unrealistic body ideals, the beauty industry is
announcing that “confidence
is the new sexy.” Similarly, as women suffer profound inequality at work, some
employers are offering
“confidence training” courses. As feminist cultural analysts, we began to notice
the rise of these messages in the
early 2010s. They stretched across many unrelated spheres: the welfare system,
consumer culture, and even
international-development initiatives. We expected that confidence might just be
having a moment. But several
years later, the obsession with women’s self-confidence seems to only be ramping
up.
Even the military has gotten in on the act: In a 2020 recruitment campaign, the
British army addressed potential
female soldiers with the promise that joining the forces would give them true and
lasting self-esteem—unlike
the superficial pseudo-confidence that “can be reapplied every morning,” like
makeup or false eyelashes. By
now, these exhortations are so ubiquitous that they have come to constitute a kind
of unquestioned common
sense; the self-evident value of women’s confidence has been placed beyond debate.
Of course, we are not against confidence. Would anyone genuinely want to position
themselves against making
women feel more comfortable in their skin? But we are sceptical of the consequences
of the cultural prominence
of this imperative. And after a decade of research, we’ve concluded: Confidence is
both a culture and a cult. It
is an arena in which meanings about women’s bodies, psyches, and behaviour are
produced, circulated,
negotiated, and resisted. This cult isn’t all bad. But just as it opens up many
possibilities for change, it also
renders much unintelligible. Whatever the problems faced by women or girls, the
implied diagnosis offered is
typically the same: She just needs to believe in herself.
In recent years we’ve seen a seemingly contradictory move—the vulnerability turns.
Indeed, many of the
champions of confidence culture—such as the female celebrities Serena Williams,
Melinda French Gates, and
Michelle Obama—have confessed their self-doubts, experiences of impostor syndrome,
and other struggles.
Although this focus on vulnerability might seem to challenge the confidence
imperative, ultimately it can
reinforce this culture. These confessions address some insecurities and structural
sources of vulnerability, such
as poverty, hunger, ill-health, racism, and sexism. Though these messages primarily
target women, appeals to
men’s confidence are also evident across various entities. For example, one popular
male life coach claimed that
he’d teach men to “operate on the highest, most optimized level of performance,”
“become a remarkable leader,”
and “reach social mastery.” By contrast, those who promote self-confidence in women
tend to focus on
overcoming internal problems—even in realms where this wouldn’t seem to make sense,
such as financial
advice. An accountant and writer who aims to financially empower women, for
example, promised: “five ways
to make managing your money an act of self-love.”
16. How does the concept of confidence vary in the case of men and women?
(a) Women are instigated to join the army to gain self-confidence instead of
pseudo-self-assurance and it is not
in the case of men.
(b) Men are programmed to be confident and self-assured owing to parental
upbringing and societal structure
whereas women are a victim of both of these.
(c) Men are taught to perform tasks at the highest level, display leadership
qualities and master the art of
socializing and women on the other hand are taught to deal with internal issues
such as handling their money
and accounts.
(d) Women are taught to overcome their fears and insecurities, along with handling
financial juggles, while men
are taught to embody a more positive self-image through acts of self-love.

. Page 7 of 44
17. How does the acceptance of vulnerability by some famous female celebrities
beneficial in some aspects?
(a) It strengthens the culture of confidence and touches upon the negative aspects
of hunger, illness, sexism and
racism.
(b) It weakens the culture of confidence and barely touches upon the negative
aspects of hunger, illness, sexism
and racism.
(c) It exposes the fine line between actual self-assurance and confidence on one
hand and counterfeit boasting
on the other hand.
(d) It highlights the glaring vulnerabilities of women with respect to low self-
esteem, and at the same time
protects the culture of confidence.
18. Which of the accompanying is NOT a conclusion that has been reached by the
author and his team?
(a) Confidence, while it creates a lot of potential for change, also makes a lot of
things inexplicable.
(b) Confidence is a space where interpretations about women's bodies, minds, and
behaviours are created,
distributed, debated, and opposed.
(c) Confidence is not only a culture but also a cult.
(d) The problems that are faced by women, their implicit diagnosis is always the
same and it is that they must
seek validation from society.
19. What is the tone of the author in the passage?
(a) Pedestrian (b) Thought-provoking
(c) Provocative (d) Speculative
20. Which of the following statements are CORRECT in reference to this passage?
A. Because women face significant workplace inequity, several firms are providing
"confidence training"
classes.
B. The author and his team anticipated that such self-assurance would be fleeting
and their prediction was bull’s
eye.
C. The British army told budding female soldiers that joining the army would bestow
upon them the gift of
actual and lasting self-assuredness instead of fake confidence.
(a) Only A and B (b) Only B and C (c) Only A and C (d) All A, B and C
Passage (Q.21-Q.25): While I stood drinking in the beauty of this placid scene, I
became conscious of an
alteration. In a moment the sole porter emerged from his midday nap, operated a
signal that clanked noisily into
position, and then ambled slowly towards me for my return-half-ticket, whilst I
remarked that his red amiable
face and easy-going gait were in perfect harmony with the tranquil surroundings.
A wisp (___) smoke on the horizon with a dark snake crawling (_____) it announced
the approach of the train.
As it drew nearer, the deep silence of the place was gradually displaced by a
creaking of brakes and a hissing of
steam.
Save for me, no one entered the train and no one alighted. The porter with
leisurely expertness trundled a couple
of milk churns on board, the door was slammed, the guard signalled to the driver,
and we moved off, leaving the
small station once more to its drowsy silence.
21. All of the following can be inferred from the passage, except
(a) The author alighted the train.
(b) The author was the sole traveller from the station.
(c) The station had a calm ambience.
(d) The porter was multitasking.

. Page 8 of 44
22. As it drew nearer, the deep silence of the place was gradually displaced by a
creaking of brakes and a hissing of
steam. The literary device used in the passage is
(a) Onomatopoeia (b) Alliteration (c) Oxymoron (d) Irony
23. In the context of the passage, which of the following would be an apt synonym
for the word ‘gait’?
(a) Run (b) Walk (c) Smile (d) Expression
24. ‘A wisp of smoke ____ the horizon with a dark snake crawling ______ it
announced the approach of the train.
Fill in the blanks with correct prepositions.
(a) In, behind (b) On, beneath (c) Under, above (d) At, below
25. Which of the following best reflects the title of the passage?
(a) The traveller. (b) The station. (c) The porter. (d) The train.
Passage (Q.26-Q.30): The breadth of Hindu religion accommodates a large variety of
religious concepts.
Sarvepalli Radhakrishnan called it 'Hindu hospitality'. As a result, we see varied
forms of practice of Hinduism.
Shankaracharya was responsible for the 'synthesis' of Hindu religion. Shankara
brilliantly unified the different
practices into one religion that we call Hinduism today.
The concept of God in Upanishads is the 'Brahman' or the Universal Soul. Every
human being has an atman that
resembles the Universal Soul that is temporarily separated from it. It is compared
to a fire that is the Supreme
Being, from flames of which come the sparks, which are the human souls. However,
the human soul has an
undiminished desire to join its Creator. The newer religions like Christianity and
Islam also profess that God
created man in His own image.
Brahman of the Upanishads is the impersonal, transcendent power that is responsible
for all creation and the
cosmos. This World Soul can only be described as what it is not, only in negative
terms. The world soul is
without qualities. It is nirguna, nirakara, nirvishesha and nirupadhika. There is,
however, a danger of describing
the Supreme Being in negative terms. This renders it totally impersonal and dispels
a positive image of God.
Buddha circumvented this problem by calling the eternal spirit nothing but
righteousness or dharma, the codes
of ethical living that all humans should follow. It is the essence of Buddha's
dharma/teachings. Though atheistic
at its inception, Buddhism ultimately used Buddha himself as its icon and God.
Eventually, even Hindus accepted
Buddha as one of Vishnu avatars.
Ordinary Hindus needed a God they could conceptualize, for worship. To go about
their daily lives with devotion,
the common folks needed a symbol, a personal God. Upanishads that described Nirguna
Brahman as the
Universal Soul also offered a Saguna Brahman, who is the antithesis of Nirguna
Brahman. But, the central belief
of Hinduism remained unshaken that all these forms of godheads sprung from one
Universal Soul. This makes
Hinduism unique in the sense that it is a monotheistic religion with a pantheon of
manifested forms of God. The
common misconception that Hindu religion is polytheistic is simply untrue.
Shankara is credited with bringing unity to Hindu religion when it was dangerously
close to schism, especially
between Shiva and Vaishnava cults. Which begin to rise before Shankara. Hindus also
believe in the laws of
karma. All actions, intended or unintended, result in karma. It is the account of a
person's action. Karma is
independent of God and religion. Apostasy from religion does neither absolve nor
vitiate one's karma. Whether
one is theist or not, he is going to face consequences of his actions. God does not
enjoin man to behave in a
certain way. The basic guidelines of ethics and truth are described in the Hindu
Dharma document but the actions
of a person are left to his or her discretion. However, there are consequences for
all karma, good or bad, and
these are permanently etched in the 'memory' of the soul. The manner of this
linkage between jiva-atman and
parama-atman, differs in the three major philosophies of Hinduism - Advaita,
Vishishtadvaita and Dvaita.

. Page 9 of 44
26. Which, among the following is a misconstrued statement of the passage?
(a) Hinduism believes in Universal soul.
(b) Buddhism was atheistic at its inception.
(c) The World Soul is nirguna (without qualities).
(d) Shankara is credited with bringing disarray to Hinduism.
27. The passage mentions comparison or similarity between the following sets of
religions, except
(a) Hinduism and Buddhism.
(b) Buddhism and Jainism.
(c) Hinduism and Christianity.
(d) Hinduism and Islam.
28. Which of the following CANNOT be inferred from the passage?
(a) Shiva and Vaishnava cults existed before the period of Shankara.
(b) A person will be affected by his karma even if they are irreligious.
(c) A person has no freedom in his actions, as God wills everything.
(d) Buddhism is considered as a subsection of Hindu religion by the Hindus.
29. What, according to the author, drives a person to follow the path of Dharma in
Hinduism?
(a) The realization that all doings will result in bad consequences.
(b) The belief that that every action of the soul whether witting or unwitting,
good or bad generates a
consequence.
(c) Following the path of Dharma is the best way to realize God.
(d) The path of Dharma enables a soul to renounce desire, and worldly passions.
30. The main purpose of the author in writing this passage may be to
(a) highlight "Unity in Diversity' in Hinduism.
(b) describe the contribution of Shankaracharya to Hinduism.
(c) compare and contrast the ideals of Hinduism with that of other religions
(d) explain the various facets of Hindu religion
ECTION - C: LEGAL REASONING
Passage (Q.66-Q.71): The doctrine of privity of a contract is a common law
principle which implies that only
parties to a contract are allowed to sue each other to enforce their rights and
liabilities and no stranger is allowed
to confer obligations upon any person who is not a party to contract even though
contract the contract have been
entered into for his benefit. The rule of privity is basically based on the
‘interest theory’ which implies that the
only person having an interest in the contract is entitled as per law to protect
his rights.
A party to a contract is excused from performance if it depends upon the existence
of a given person, if that
person or becomes too ill to perform. Thus, where the nature or terms of a contract
require personal performance
by the promisor, his death or incapacity puts an end to the contract.
The Calcutta High Court reiterated that an agreement is not discharged by the death
of a party and shall remain
enforceable by or against the legal representatives of the deceased party provided
the right to sue in respect of
the cause of action survives.
Single-Judge Chief Justice Prakash Srivastava was of the view that even though the
legal representatives are not
signatories to the arbitration agreement, but being the legal representatives of
the original signatory of the
agreement, they would be bound by the agreement.
It is clear from Section 40 of the Arbitration Act that an arbitration agreement is
not discharged by the death of
any party thereto and on such death it is enforceable by or against the legal
representatives of the deceased, nor
is the authority of the arbitrator revoked by the death of the party appointing
him, subject to the operation of any
law by virtue of which the death of a person extinguishes the right of action of
that person.
(Extracted with requisite revisions and edits from ‘Arbitration agreement not
discharged by death of parties;
enforceable against legal representatives: Calcutta High Court’ at
https://www.barandbench.com/news/litigation/arbitration-agreement-not-discharged-
by-death-of-parties#enforceable-against-legal-representatives-calcutta-high-court)
66. Aman Grover entered into a partnership deed with one Radhey for starting a
pathological laboratory business in
Calcutta. The co-partner to the deed (Radhey) had executed a power of attorney in
favour of his wife, Radha.
The provisions of the partnership deed provided that in case of any dispute between
the parties arising out of this
deed, it shall be adjudicated according to the Arbitration and Conciliation Act,
1996 and the place would be New
Delhi. It also provided that the arbitration panel shall consist of 3 arbitrators
(one appointed each by every party
and the third would be jointly appointed with mutual consent by the parties. A
dispute arose as to the partnership
deed and a notice was sent by Aman to Radhey for appointing of the arbitrator.
Meanwhile, Radhey died of a
heart attack before any arbitrator could be appointed. A reminder notice was sent
by Aman to his wife to appoint
the arbitrator and to resolve the issue. However, it was the contention of Radha
that she was not a party to the
Deed and hence she cannot be subjected to the provisions of the Deed. Is the
contention of Radha correct?
(a) Yes, according to the doctrine of privity of contract, the legal
representatives cannot be made liable as they
were not signatories to the Deed.
(b) No, death of the signatory i.e. Radhey would not excuse his legal
representatives from the obligations
enshrined under the Deed.
(c) Yes, since only a power of attorney was issued under the name of Radha who
didn’t had any interest/ was
not a beneficiary to the agreement, she cannot be subjected to the provisions of
the deed the contents of
which were not authorized by her.
(d) No, since the Deed doesn’t deal with any personal and specific performance by
Radhey, the privity of
contract can be extended to include the legal representatives.
67. One of the sons of Radhey, Mohan contended that on the liquidation of the
partnership between the deceased
and Aman, equitable adjustment has not been made between the parties and that issue
shall also be resolved by
the arbitration tribunal/ panel. It was however, stated by Aman that not only Mohan
is not a signatory to the
Deed but no claims can be brought by the legal representatives which was not
initially notified to the signing
party during their life and law only provides for bringing claims against the legal
representatives. Is the
contention of Aman correct?

. Page 17 of 44
(a) No, the death of the signatory would not extinguish the rights of the legal
representatives and thus Mohan is
entitled to bring a claim against Aman.
(b) Yes, the death of the signatory will extinguish all rights existing with the
signatory during the time he was
alive.
(c) No, legal representatives cannot be termed as strangers to the agreement and
therefore will have right to sue
or be sued after the death of the signatory.
(d) Yes, according to the doctrine of privity, no stranger to an agreement is
permitted to sue the party to the
agreement.
68. As per one of the provisions of the Partnership Deed, it was provided that
Radhey would be designing their office
as he had completed masters in interior designing. The intention of such a
provision was for cost-cutting purposes
and therefore, after his death, it was the contention of Aman that now the interior
designing should be undertaken
by Radha or Mohan (being the legal representatives of Radhey) or they should
undertake to pay for such amount
as would be incurred during the course of the activity by third persons. The legal
representatives however refused
to exercise either of the options and it was their contention, that after the death
of Radhey his performance
obligation also ceases and now the legal representatives cannot be made to perform
such obligations. Is the
contention of the legal representatives correct?
(a) Yes, the performance obligations personal to Radhey will frustrate after the
death and the legal
representatives cannot be made liable to incur such personal obligations.
(b) No, since the performance obligations as per the deed was vested not only with
Radhey but Aman also gave
the alternative to make payment for hiring of any third person to substitute for
Radhey’s obligations.
(c) Yes, since the legal representatives are not signatories to the deed, there
exists no right to sue or be sued
under the deed against the legal representatives of Radhey.
(d) No, since there was no personal performance to be exclusively performed by
Radhey, the legal
representatives will continue to be liable for specific performance.
69. Radhey was also required to purchase the furniture from one of the shops in
their locality. The order was placed,
however, after his death, the legal representatives decided to not continue with
the obligations despite continuing
being party to the business on behalf of the deceased till liquidation has been
finally fructified. Their claim was
since they were not party to the agreement, they cannot be made to follow the
obligations and the contract is
terminated on the death of Radhe due to personal performance requirements. Is their
argument correct?
(a) Yes, the contract is frustrated by the death of the signatory i.e. Radhey and
therefore no specific performance
as to the obligations can now be claimed by Aman.
(b) No, since the legal representatives continued to be a part of the business,
they have to oblige all the
obligations and will have all the rights once vested with Radhey.
(c) Yes, the legal representatives were not signatory to the deed and hence only
those provisions which are for
the benefit of the legal representatives can be made applicable.
(d) No, there was no personal performance by Radhey contingent upon the existence
of Radhey and thus the
contract cannot be excused or terminated merely by virtue of the death of Radhey.
70. There was already a pending dispute between the two partners where the
arbitrator was already appointed by the
parties as per the terms of the deed. The decision was, in an oral hearing, given
against Radhey but the written
decision was yet to be formulated after the next hearing. However, after the death
of Radhey, the appointment
of the panel itself was challenged by the legal representatives who challenged that
post the death of the arbitrator
the authority of the arbitrator gets revoked and now the parties shall be required
to re-appoint the arbitrators and
to conduct the arbitration afresh. Is the contention of the legal representatives
correct?
(a) Yes, in the interests of justice and to serve the interests of Radhey’s legal
representatives, the arbitrators
should be appointed afresh.
(b) Yes, since one of the signatory has died and right to sue and be sued would
vest with the legal representatives,
therefore the right to appoint the arbitrator of their choice should also be vested
with them.

. Page 18 of 44
(c) No, since the act of the legal representatives for the removal of the
arbitrator was motivated by malice since
they apprehended that the decision might be given against them, the contention of
the legal representatives
is bound to fail.
(d) No, the death of the signatory would not discharge the authority of the legally
appointed arbitrator and s/he
would continue to have the authority has was enshrined.
71. As per another agreement between Radhe, Aman with a delivery person, Sandeep,
it was required that the
delivery of the laboratory essentials shall be made within 3 days of the placing of
the order. However, Sandeep
failed to deliver the goods on time. The legal representatives of Radhey decided to
bring a claim against Sandeep
in the court of law. It was the contention of Sandeep that only the signatories to
the agreement can bring a claim
against him and no one else. Is the contention of Sandeep correct in light of
doctrine of privity?
(a) Yes, only the signatories can bring a claim for the breach in services by
Sandeep and not the legal
representatives.
(b) No, the agreement shall remain operational even after the death of one of the
signatories to the agreement
and the right of the deceased signatory shall now vest with the legal
representatives.
(c) Yes, the agreement involves giving of services by Sandeep for the partners, the
legal representatives shall
bear no responsibility under the terms of the agreement.
(d) No, the doctrine of privity shall not be applicable as one of the signatories
has died and would be required
to be represented by his legal representatives.
Passage (Q.72-Q.77): In Mukund Dewangan, a 3-judge Bench had held that a person
holding a driving licence
in respect of "light motor vehicle", could on the strength of that licence, be
entitled to drive a "transport vehicle
of light motor vehicle class" having unladen weight not exceeding 7500 kgs. A bench
comprising Justice
Amitava Roy, Justice Arun Mishra and Justice Sanjay Kishan Kaul held in Mukund
Dewangan that a separate
endorsement in the LMV driving licence is not required to drive a transport vehicle
having unladen weight below
7500 kilograms.
A transport vehicle and omnibus, the gross vehicle weight of either of which does
not exceed 7,500 kg would be
a light motor vehicle and also motor car or tractor or a road roller, 'unladen
weight' of which does not exceed
7,500 kg and holder of a driving licence to drive class of "light motor vehicle" as
provided in section 10(2)(d) is
competent to drive a transport vehicle or omnibus, the gross vehicle weight of
which does not exceed 7,500 kg
or a motor car or tractor or road-roller, the "unladen weight" of which does not
exceed 7,500 kg.
Section 7 provides that no person can be granted a learner’s licence to drive a
transport vehicle unless he has
held a driving licence to drive a light motor vehicle for at least one year.
Section 14 deals with the licence to
drive motor vehicles. The proviso to Section 14(2)(a) provides that in case of a
licence to drive a transport vehicle
carrying goods of dangerous or hazardous nature, it shall be effective for a period
of one year. However, in case
of any other licence, it would be effective for a period of 20 years. Rule 5 of The
Central Rules Motor Vehicles
Rules, 1989 (hereinafter referred to as “the Rules”) makes a medical certificate
issued by a registered medical
practitioner mandatory for in case of a transport vehicle, whereas for a non-
transport vehicle, only a self#declaration is sufficient. For the purpose of all
meanings: "transport vehicle" means a public service vehicle, a
goods carriage, an educational institution bus or a private service provider
vehicle and "unladen weight" means
the weight of a vehicle or trailer including all equipment ordinarily used with the
vehicle or trailer when
working, but excluding the weight of a driver or attendant, products; and where
alternative parts or bodies are
used the unlade weight of the vehicle means the weight of the vehicle with the
heaviest such alternative part
or body.
Extracted with requisite revisions and edits from ‘Does LMV Licence Enable One To
Drive Transport Vehicle
Weighing Less Than 7500KG? SC 3-Judge Bench Doubts 'Mukund Dewangan' Decision;
Refers To Larger’ at
https://www.livelaw.in/top-stories/lmv-license-enable-one-to-drive-transport-
vehicle-weighing-less-than#7500kg-sc-3-judge-bench-doubts-mukund-dewangan-decision-
refers-to-larger-bench-193949)

. Page 19 of 44
72. In one of the early winter mornings, Mukesh was driving a truck and heading
towards the construction site where
the construction of a new mall was taking place after the removal of slums which
stretched far and wide earlier.
The truck carried around 1400 kgs of sand and 600 kgs of concrete. The driver
weighed 70 kgs and the weight
of the truck itself was approximately 5450 kgs and engine weight of 500 kgs. The
truck met with an accident
while rushing towards the construction site and killed two people instantly. When
the compensation was claimed
by the family of the deceased’s, the insurer company denied the claim on the ground
that Mukesh had not
followed the legal requirements as the gross weight of the vehicle was clearly
above the lawful limits without
any separate endorsements and insurance doesn’t cover negligent acts. Is their
(insurers) contention bound to
succeed in a court of law?
(a) Yes, the gross weight of the truck was clearly above the prescribed limits
under law i.e. 7500 kgs and
therefore the insurer is not liable to pay compensation for the negligence of
Mukesh.
(b) Yes, despite the weight of the truck, the insurers cannot be made liable for
the negligence of the insure.
(c) No, the gross weight of the truck means the weight of the truck which is 5450
kgs that is below the prescribed
7500 kgs limit.
(d) No, the gross weight of the truck was below the prescribed limits under law
i.e. 7500 kgs and hence insurer
is bound to pay the compensation to the deceased’s family.
73. Mukesh was by nature a helpful person. One day, he saw a farmer (50 kgs) with a
buffalo (550 kgs) waiting to
reach a local hart (market) to sell his buffalo. On knowing that the farmer was
getting delayed, Mukesh offered
a ride till the hart. The truck weighing 5450 already carried concrete weighing
1500 kgs. The truck met with an
accident. Will the insurance company be liable to pay compensation to the
deceased’s?
(a) No, the gross weight of the truck is 7550 kgs which is above the prescribed
limits under law i.e. 7500 kgs
and therefore Insurance Company would not be liable to pay compensation.
(b) Yes, the gross weight of the truck is below the prescribed limits under law and
therefore the insurance
company would be liable to provide compensation to Mukesh.
(c) Yes, since the act of Mukesh was out of good faith and without malice, the
legal requirements can be relaxed
in such a scenario.
(d) No, the gross weight of the truck exceeded the prescribed limits under law and
therefore the insurance
company is not liable to pay compensation to deceased’s family.
74. Rama bought a new car ‘Duster’. She had a learner’s license for about a 3
months and then directly obtained the
permanent license to drive the car. She was driving the car one day, and saw few
girl scouts on the way who had
missed their bus and had to perform at Janpath on Republic Day. Before the actual
performance day, they had to
complete the training of one more month and they informed Rama how it is
challenging for them to catch the
bus as they come from distant areas. Rama, out of kindness, told them that she
comes from around that area and
can give them a lift every day for a month. After around 20 days, the car met with
an accident injuring the
passengers and Rama. Insurance Company denied their claim for compensation of
medicals on the ground that
she never had a learner license before using the vehicle for providing
transportation to the girl scouts. Is the
contention of the insurance company correct?
(a) No, the requirements of a transport vehicle are not applicable on Rama.
(b) No, Rama provided lift to the girl scouts on good faith and without charging
them for the transportation and
therefore it would be incorrect if the medical expenses are not compensated by the
insurers.
(c) Yes, the statutory requirements supersede the good faith principle and
therefore Rama is not eligible for the
medical expenses from the insurers.
(d) Yes, the vehicle is used for providing transportation to the girl scouts and
therefore the mandatory
requirement of possessing one learner’s license for one year is breached in this
case.

. Page 20 of 44
75. It was also found by the insurance company that Rama had not submitted the
medical certificate showing her
fitness at the time of insurance and had merely submitted a self-declaration form.
The insurance company was
also blind-sided as they didn’t know that the car would be used for providing
transportation to people. Thus, they
brought a claim for fraud against Rama. Is their claim likely to succeed?
(a) Yes, Rama would be liable for fraud as she intentionally didn’t submit the
medical certificate to the insurers.
(b) No, it is the duty of the insurers to make sure that the necessary
documentations are present before approving
any insurance and therefore the effect of breach of duty by the insurers cannot be
cast upon Rama.
(c) Yes, the insurance covered only the usage of the vehicle for personal use and
nothing beyond that, therefore,
the claim of the insurers would survive.
(d) No, there was no fraud by Rama as met the required condition.
76. Manoj has been in the business of transporting sulphuric acid and hydrochloric
acids to the chemical treatment
plants for the past 5 years and was known to the facilities staff of the plant. The
chemicals transported were to
be kept in closed and non-reactive containers. However, during the evenings, he
used to deliver construction
materials to some other areas and the delivery of chemicals was carried only 4
times a month otherwise his
vehicle was devoted to delivering of construction materials. One day while he was
delivering chemicals, he was
stopped by the staff from entering the premises as his license was dated 1.5 years
ago and he didn’t carry any
renewed license. He always ensured that the weight of the vehicle was below 7500
kgs. However, he contended
that since the vehicle was mostly used for transporting non-hazardous materials,
his license will be valid for 20
years. Is his contention correct?
(a) Yes, since the vehicle was majorly devoted to the carrying and delivering of
construction materials i.e. non#hazardous materials, the license would be valid for
20 years.
(b) No, construction material can also qualify as hazardous materials due to its
very nature and thus, in any case,
Manoj’s contention would be incorrect.
(c) No, since he was delivering hazardous substances, his license for transport
would be valid for a period of 1
years only and therefore Manoj’s argument is incorrect.
(d) No, the fact that the vehicle was to carry materials to a chemical factory is
sufficient to qualify it under a
hazardous activity and therefore, the contention of Manoj would be incorrect.
77. Rath wanted to start operating bus services in his city under his name. He
hired drivers who had experience in
the sector. However, he himself had never held a driving license to drive a light
motor vehicle. He decided to
abstain from the driving and delegate the exercise to only the licensed employees.
One day, one such drivers met
with an accident and seeing no other vehicles and drivers around him, he decided to
drive the bus himself to take
him to the hospital. While driving back, he was caught by the traffic police, who
imposed Challan (penalty) on
him due to non-possession of license. Is the act of the traffic police justified?
(a) No, since Rath was driving the bus not to provide services to the passengers,
therefore the requirement of a
license to drive a public vehicle would not be applicable.
(b) Yes, Rath didn’t possess the requisite license to drive a transport vehicle and
therefore penalty can be
imposed on him by the traffic police.
(c) No, since Rath only drove the vehicle when no other aid was available and in
good faith to protect the life
of a person, he cannot be subjected to any penalties.
(d) Yes, despite the act of Rath being in good faith, this exception would no
longer be applicable since the
emergency had averted and Rath was returning from the hospital.

. Page 21 of 44
Passage (Q.78-Q.82): The gross abuse of the tool of ‘interview’ to deliberately
deny entry to aspirants from
marginalised sections in educational institutions and public sector employment on
the pretext that they lack
even minimum merit is not a new phenomenon.
However, lack of adequate data on the marks awarded to reserved category aspirants
as well as the
conspicuous silence of the mainstream media, which itself is considered highly
casteist, has meant that such
issues of gross injustice can never garner the attention they deserve.
The implementation of the RTI Act has ensured to an extent that such deliberate
attempts of awarding
abysmally low marks, which in a majority of cases is due to the caste prejudice of
‘upper’ caste interviewers,
will not go unreported, nut candidates are not aware of their right regarding RTI.
This coupled with access
to several social media platforms where such incidents spread like wildfire have
immensely contributed in
building a public opinion against such acts of injustice.
While the Supreme Court ruled in early 2020 that there is no fundamental right to
reservations in
appointment and promotions under articles 16(4) and 16(4A) of the constitution,
Article 16(1) and 16(2)
assured citizens of the equality of opportunity in employment or appointment to any
government office.
Also, Article 15(1) generally prohibits any discrimination against any citizen on
the grounds of religion,
caste, sex or place of birth.
Additionally, Article 29(2) bars discrimination against any citizen with regard to
admission to educational
institutions maintained by the government or receiving aid out of government funds
on the grounds of
religion, race, caste, etc. Notwithstanding the provisions in these articles,
caste-based discrimination is quite
rampant in selection processes in public institutions.
(Source: https://thewire.in/caste/no-indian-institution-is-free-of-caste-based-
discrimination-in-viva-exams-and#interviews)
78. The selection panel at GB Pant Social Science Institute of Allahabad, found
none of the 16 OBC candidates
who had been called for interview suitable for the position of assistant professor.
The position of associate
professor for the OBC category was also left vacant for the same reason – ‘none
found suitable’. Can this
instance be termed as violation of any fundamental rights discussed in the passage
given?
(a) Yes, since the institute purposefully did not choose any OBC candidate who came
exclusively for an
interview for an associate professor position in the OBC category.
(b) No, since the institute did not find any candidate "fit" for the position,
which is a self-explanatory reason
for not selecting any OBC candidate who came specifically for an interview for an
associate professor
position in the OBC category.
(c) Yes, because the institute discriminated against OBC candidates when recruiting
for associate professor
positions.
(d) No, the institute did not discriminate against OBC candidates when recruiting
for associate professor
positions in OBC category.
79. What might be considered the cause of the castiest incidents mentioned in the
passage?
(a) There is no openness in the selection process and candidates are unaware of the
right to information as
a basic fundamental right.
(b) No Transparency in the selection process where caste has been revealed to the
members of the interview
panels.
(c) In selection processes, the criterion "none found appropriate" cannot be used
more than once.
(d) In interview panels, observers from the reserved group come from outside the
institute, and they should
have greater input.

. Page 22 of 44
80. The result of PhD entrance exams of Jawaharlal Nehru University, one of the
India’s finest, were announced.
In it, a number of candidates from reserved categories, Scheduled Castes, Scheduled
Tribes and OBCs were
given abysmally low marks – between one to four out of 30 in the viva voce or oral
examinations. Where
people from other caste are given 22-28 marks out of 30. Decide.
(a) This is an apparent infringement of the fundamental rights guaranteed under the
Indian constitution.
(b) This is an evident example of discrimination based on caste.
(c) The incidence where only a few number of candidates belonging to reserved
category getting low marks
cannot be regarded as discriminatory.
(d) The incidence where all of the candidates belonging to reserved category
getting low marks can be
regarded as discriminatory.
81. Nandini alleges that during her M.Phil and PhD days at JNU, there were frequent
reports of the cases where
students from reserved categories were given even given a zero mark in their viva
voce examinations despite
doing relatively well in their written exams. She states this could have been a
subject of every bonafide
enquiry. Chose a statement that supports her contention.
(a) Nandini’s contention of bonafide enquiry is baseless.
(b) Nandini’s contention of bonafide enquiry can find its base in Right to
information.
(c) Nandini in addition of invoking his right to information for bonafide enquiry
can also take help of
various social media platforms to raise public opinion against such acts of
injustice.
(d) The only recourse nadini has is of invoking right to information. Help of
social media platform to raise
awareness can be proved helpful.
82. Assertion: Article 15 is the protector of the oppressed and a shield against
discrimination.
Reason: Although the ideals are enshrined in the constitution. Inequalities still
exists even today in our society.
(a) Both A and R are true but R is not correct explanation of A.
(b) Both A and R are true and R is correct explanation of A.
(c) A is true but R is false.
(d) A is false but R is true.
Passage (Q.83-Q.87): The expression “special leave to appeal” in Article 136(1) of
the Constitution was adopted
by the framers from the Government of India Act, 1935. Article 136 has been couched
in the widest possible
terms with almost no restrictions, of any nature.
136. Special leave to appeal by the Supreme Court.-
(1) Notwithstanding anything in this Chapter, the Supreme Court may, in its
discretion, grant special leave to
appeal from any judgment, decree, determination, sentence or order in any cause or
matter passed or made by
any court or tribunal in the territory of India.
(2) Nothing in clause (1) shall apply to any judgment, determination, sentence or
order passed or made by any
court or tribunal constituted by or under any law relating to the Armed Forces.
However, it must be remembered that Article 136 does not confer a “right to appeal”
but only a “right to apply”
for special leave which, if granted, confers a right to appeal so long as the leave
is not revoked. Despite being
couched in widest words, the Supreme Court, on its own accord, does not entertain
special leave petitions in
cases particularly when there are concurrent findings of fact, save in exceptional
cases, such as cases of perversity
or impropriety, violation of principles of natural justice, error of law or errors
of record or misreading of
evidence. The Supreme Court invokes the power under Article 136 in “exceptional
circumstances” where the
Supreme Court is duty bound by its constitutional duties. The Constitutional Bench
in Dhakeshwari Cotton
Mills Ltd. v. Commissioner of Income Tax, West Bengal has observed that
“ …. It is, however, plain that when the court reaches the conclusion that a person
has been dealt with arbitrarily
or that a court or tribunal within the territory of India has not given a fair deal
to a litigant, then no technical
hurdles of any kind like the finality of finding of facts or otherwise can stand in
the way of exercise of this power
because the whole intent and purpose of this article is that it is the duty of this
court to see that injustice is not

. Page 23 of 44
perpetuated or perpetrated by decisions of courts and tribunals because certain
laws have made the decisions of
these courts or tribunals final and conclusive…
Based on the passage above, answer the following
83. Based on the passage above, it can be seen that there are certain restrictions
imposed on the use of Supreme
Court’s power, apart from the restriction specified in clause (2) of the Article.
What is the source of these
restrictions?
(a) Government of India Act 1935
(b) Constitution of India
(c) The judgment in Dhakeshwari Cotton Mills Ltd. v. Commissioner of Income Tax,
West Bengal
(d) None of the above
84. Radha was raped by 3 men on the Wednesday night, while she was coming back from
her office. Being
traumatized by the same she went to the police 2 days later. However, the police
authorities refused to entertain
her complaint on the ground that she has come to them after 2 days of the incident.
Despite her repeated requests,
the police does not register her FIR. Aggrieved by the same, she files a special
leave to appeal before the
honourable Supreme Court requesting them to direct the police authorities to
register her complaint. Will the
court entertain this Special Leave to Appeal?
(a) The court will entertain her appeal since it involve grave injustice by police
authorities.
(b) The court will not entertain her appeal since it doesn’t have any pertinent
question of law.
(c) The court will not entertain her appeal since it is not an exceptional
circumstance
(d) None of the above
85. Varsha is an extremely poor lady who has been thrown out of her house by her 2
sons in the year 2001. She hired
a pro bono lawyer to fight her case of wrongful eviction against her sons. The
matter was first listed in the District
Court in the year 2002. However, on the first day itself the court simply adjourned
the matter to the next date
vide its order dated 30.01.2002. Since then, it has been 20 years the matter has
not come up for hearing.
Aggrieved by the same she applies through a special leave to appeal against the
order dated 30.01.2002 of the
District Court. Will the court entertain this special leave to appeal?
(a) The court will entertain the appeal as it is an exceptional circumstance, since
the poor woman has been
denied justice since 20 years.
(b) The court will not entertain the appeal because pendency of cases is a common
feature of the Indian legal
system and hence not an exceptional circumstance.
(c) The court will not entertain the appeal because the court does not have any
jurisdiction to hear a special leave
to appeal from such an order of the District Court
(d) None of the above
86. Based on the observations of the Supreme Court in the judgment of Dhakeshwari
Cotton Mills Ltd. v.
Commissioner of Income Tax, West Benga, which amongst the following options aptly
summarise the purpose
of Article 136 of the Constitution?
(a) The Supreme Court, being a court of final order, has been given exceptional
powers to supervise the decision
of lower courts in order to do justice.
(b) The Supreme Court, being a court of equity, has a duty and responsibility to
ensure that there is no hindrance
in ensuring complete justice to a litigant.
(c) The Supreme Court, being the highest court, has been given exceptional powers
to avoid the long hassle of
litigating before various courts and tribunal.
(d) The Supreme Court, being a court of national importance has been given
exceptional powers to decide
pertinent question of laws.

. Page 24 of 44
87. ONGC limited has filed an Arbitration case against reliance India limited with
respect to certain dispute over oil
refineries. When this came into public knowledge, the share price of both the
companies started falling rapidly,
in anticipation of filing of the case. Both the companies applied to Supreme Court
through a special leave to
appeal, seeking relief to expeditiously resolve the matter so that there is no
adverse impact on the share price.
Will the court entertain the special leave to appeal?
(a) The Supreme Court will not entertain this special leave to appeal since there
is no pertinent question of law,
neither there are exceptional circumstances.
(b) The Supreme Court will not entertain this special leave to appeal since it does
not fall within the scope of
Article 136
(c) The Supreme Court will entertain this special leave to appeal because there are
exceptional circumstances
which justify interference by the Supreme Court.
(d) The Supreme Court will entertain this special needs to appeal because Article
136 is written in widest words
and the court can interfere to do complete justice.
Passage (Q.88-Q.91): A bench headed by Chief Justice of India NV Ramana and also
comprising Justices Surya
Kant and Hima Kohli had on April 27 last week, ordered the government to file its
response by April 30. The
bench had also directed that the case be listed for final disposal on May 5 while
making it clear that no
adjournment will be granted.
The Court is seized of a batch petitions challenging the constitutionality of
Section 124A.
The Court had highlighted that provision was used by the British to quell the voice
of Indian freedom fighters
like Mahatma Gandhi and Bal Gangadhar Tilak before the country gained independence.
It is now being misused when someone does not like the views of another person and
there is no accountability
from the executive, CJI Ramana had observed. It stated that a statute criminalising
expression based on
unconstitutionally vague definitions of ‘disaffection towards Government’ etc. "is
an unreasonable restriction on
the fundamental right to free expression guaranteed under Article 19(1)(a) and
causes constitutionally
impermissible ‘Chilling Effect’ on speech." Article 19(1)(a) provides that all
citizens shall have the right to
freedom of speech and expression. However, this right will be subject to reasonable
restrictions in the interest of
sovereignty, public order, decency, morality or in relation to contempt of court.
"Dispute is it is a colonial law and was used by British and suppress freedoms and
used against Mahatma Gandhi
Bal Gangadhar Tilak. Is this law still needed after 75 years of independence? Our
concern is misuse of the law
and no accountability of the executive," CJI Ramana had said.
Section 124A provides: “Whoever, by words, either spoken or written, or by signs,
or by visible representation,
or otherwise, brings or attempts to bring into hatred or contempt, or excites or
attempts to excite disaffection
towards, the Government established by law in [India], shall be punished with
[imprisonment for life], to which
fine may be added, or with imprisonment which may extend to three years, to which
fine may be added, or with
fine.
Explanation 1- The expression "disaffection" includes disloyalty and all feelings
of enmity.
Explanation 2- Comments expressing disapprobation of the measures of the Government
with a view to obtain
their alteration by lawful means, without exciting or attempting to excite hatred,
contempt or disaffection, do not
constitute an offence under this section.
Explanation 3- Comments expressing disapprobation of the administrative or other
action of the Government
without exciting or attempting to excite hatred, contempt or disaffection, do not
constitute an offence under this
section.”
(Extracted with requisite revisions and edits from ‘Sedition: Central government
seeks more time to file response
to Section 124A challenge in Supreme Court’ at
https://www.barandbench.com/news/sedition-central#government-seeks-more-time-file-
response-section-124a-challenge-supreme-court)

. Page 25 of 44
88. Mr. Dua was a known youtuber and during the covid pandemic he was so frustrated
with the take of the
government that he decided to release a video on the same. This video containing
the following remarks:
“The government neither has nor is taking any measures to establish adequate
testing facilities and the
government is making false disclosures to the people on the availability of the PPE
kits, sanitizers and ventilators
and that there is insufficient information on them.”
An FIR was lodged against him on the ground that a video like this not backed by
evidences will provoke the
population, cause panic and generate terror among the population by making false
allegations. These rumours
were merely spread to stir public dissatisfaction, resulting in panic and
individuals disobeying the lockdown
conditions. Will Mr. Dua be liable under Section 124A of the IPC?
(a) Yes, the words of Mr. Dua are sufficient to incite hatred, contempt or
disaffection against the incumbent
government and therefore Mr. Dua would be liable under Sedition.
(b) No, the facts asserted by Mr. Dua are premised on true facts and ground reality
and therefore Mr. Dua will
not be liable for sedition.
(c) Yes, youtube as a media platform has the capability of influencing youth and
even the slightest incitement
can have the outcome of affecting exciting the feelings of enmity.
(d) No, the comments made by him would amount to disapprobation of the government
measures and therefore
an action against Mr. Dua for Sedition would not be maintainable.
89. It was the claim of Mr. Dua, that he made the video in Bengali and there were
very few Bengali followers of his
channel and therefore his words although may appear to bring enmity would not
impact the ‘public’, and no
public dissatisfaction can thus be comprehended. Is his argument correct?
(a) No, Section 124A doesn’t prescribe any limits as to number of people who should
be impacted to perpetrate
the offence of sedition.
(b) Yes, the surrounding circumstances including number of followers, language in
which the speech/ remark
was made would be determinant in deciding the culpability of the offence.
(c) No, culpability under Section 124A is dependent upon the actual harm rather
than the attempted harm and
therefore, no liability can be invoked upon on Mr. Dua.
(d) Yes, the term ‘public’ is subjective and therefore whether the remarks were
capable enough to stir disloyalty
amongst the public cannot be deduced.
90. Few college students were so motivated by the words of the Mr. Dua that on the
occasion of a political rally,
they attacked the rally with hockey sticks, knives etc. thereby injuring the many
people who were the part of the
procession. Can Mr. Dua be made liable for sedition?
(a) No, Mr. Dua would be protected by the freedom of speech and expression and
therefore cannot be held liable
under sedition.
(b) Yes, since the remarks of Mr. Dua in his channel incited the college students
which led to them attacking the
rally, therefore Mr. Dua would be liable.
(c) No, regardless of the fact that there was an actual attack on the rally after
being motivated from the words of
Mr. Dua, the words of Mr. Dua were merely disapprobation and disapprovals and
therefore he cannot be
made liable for sedition.
(d) Yes, the fact that there was an actual mob attack based on the remarks by Mr.
Dua is sufficient to indicate
that his words were seditious.

. Page 26 of 44
91. Two journalists were covering on the recent riots that took place in
Shahjahanabad regarding the forceful eviction
of the illegal occupants without any notice. Agitated with the plight of the
people, they commented in a rally that
“Till when would you want to be spectators. Today your homes are being taken away
from you, the days when
your kids would be on the street begging is not far away. The current government
has always been propounding
the majority religion and the interests of the minorities has been mocked upon by
them time and again. We
should come out of our homes and bring the change by taking government down. A
protest was organized with
affect to their words and the journalists along with the mob were ready to revolt
against the incumbent
government. They carried smoke guns etc. in order to protect them in case the
government resists. However,
things did not work out as before they could reach the rendezvous, the were caught
by the police and were put
behind bars. Can the two journalists be charged under sedition despite the protest
not having taken place?
(a) Yes, the remarks by the two journalists are sufficient to cause disaffection
amongst the people and it is clear
from the factual matrix that they intended to protest against the government.
(b) Yes, it is apparent from the factual matrix that the two journalists intended
to organize protest against the
lawfully elected government and therefore would be liable.
(c) No, the words of the two journalists are merely disapproval of the government
policies and therefore the
journalists would fall under the exception of sedition.
(d) No, the remarks made by the journalists is based on good faith and conscience
and for a greater good and no
actual act took place as they were caught by the police and therefore they cannot
be made liable under
sedition.
Passage (Q.92-Q.96): From time immemorial, one of the primary functions of the
state has been to maintain
law and order and ensure that justice prevails. This has been a function that
remained unchanged even when the
state was evolving from a police state to a welfare state. The citizens pay taxes
every year to the state and the
officials for the smooth functioning of all the three organs of the state.
Prolonged pre trials and back log in cases resulting in undue delay in justice will
affect the credibility and
reliability of the judiciary which is the corner stone of a legal system. With the
introduction of sections 265A#256 L to the Code of Criminal Procedure,1973 by the
Criminal Law (Amendment) Act of 20051 the legislature
has officially induced plea bargaining into the Indian Legal system to curb the
problem of back logging of cases
in the Indian Courts and to alleviate the suffering of under trial prisoners. The
induction of plea bargaining will
be beneficial in contributing to reforming our criminal justice system.
A plea bargain is a contractual agreement between the prosecution and the accused
concerning the disposition
of a criminal charge. However, unlike most contractual agreements, it is not
enforceable until a judge approves
it. If accused is of opinion that it will benefit him may enter is such agreement.
The Indian concept of plea bargaining is inspired from the Doctrine of Nolo
Contendere. It has been incorporated
by the legislature after several law commission recommendations. This doctrine has
been considered and
implemented in a manner that takes into account the social and economic conditions
prevailing in our country.
There are three types of plea bargaining; i) charge bargaining; ii) sentence
bargaining; and iii) Fact bargaining.
Negotiating for dropping some charges in a case of multiple charge or settling for
a less grave charge is called
charge bargaining. Where the accused has an option of admitting guilt and settling
for a lesser punishment it is
sentence bargaining. Lastly, negotiation which involves an admission to certain
facts in return for an agreement
not to introduce certain other facts is fact bargaining.
(Source: http://www.manupatra.com/roundup/326/Articles/Plea%20bargaining.pdf)
92. What according to your understanding of the maxim Nolo Contendere mean?
(a) The accused is given the opportunity to respond to the charge.
(b) The accused may enter a guilty plea to the alleged crime or to a lesser
offence.
(c) The Judge may accept the plea if he is satisfied that it is voluntary and in
accordance with the actual wishes
of the accused.
(d) A plea by which a defendant in a criminal prosecution accepts conviction but
does not plead or admit guilt.

. Page 27 of 44
93. Assertion: Justice delayed is justice denied.
Reason: A person accused of an offense may file the application of plea bargaining
in trials which are pending.
(a) Both A and R are true but R is not correct explanation of A.
(b) Both A and R are true and R is correct explanation of A.
(c) A is true but R is false.
(d) A is false but R is true.
94. Which of the following statements presents an oddity with passage’s text?
I. The plea bargaining is beneficial for both the prosecution and the defense
because there is no risk of complete
loss at trial.
II. In India, the reason behind the delay in trials is many e.g. the operation of
the investigative agencies as well
as the judiciary, personal interest of lawyers etc.
III. The provisions of Plea Bargaining do not provide for an independent judicial
authority to evaluate plea#bargaining applications.
IV. As per the legal provision dealing with Plea bargaining, it is a voluntary
mechanism which is only entertained
when accused opts it willingly.
(a) IV
(b) All of the above
(c) I, II & III
(d) I and III
95. The government files an indictment against drug trafficking. Federal agents
stuck john with over five kilograms
of cocaine. Five kilograms triggers a sentence involving many years in prison, so,
he agrees to plead guilty to
the offense in exchange for the prosecution's stipulation that he possessed less
than five kilograms. Can his plea
be considered for disposition of the criminal charge?
(a) No, as he was found in possession with over five kilograms of cocaine
(b) Yes, as it is enforceable upon approval of the judge.
(c) Yes, as the accused always has an option of admitting guilt and settling for a
lesser punishment.
(d) No, as the accused only has an option of admitting guilt and settling for a
lesser punishment upon approval
of the judge.
96. Which of the following illustrations depicts plea bargaining correctly?
I. The prosecution expenses Ross with both robbery and simple attack. The parties
agree that Ross will plead
to the attack charge and that the prosecution will dismiss the theft charge.
II. Sammy agrees to plead to the charge of resisting arrest, and the prosecution
agrees to recommend that the
judge not sentence him to jail time will settle with fine.
III. The defendant pleads in exchange for the prosecutor's stipulation that certain
facts led to the conviction. The
omitted facts would have increased the sentence because of sentencing guidelines.
IV. The prosecution charges chandler with burglary, but he pleads guilty to
trespassing and the prosecution
dismisses the burglary charge.
(a) I and IV
(b) I and III
(c) II and III
(d) All of the above.

. Page 28 of 44
Passage (Q.97-Q.100): Article 226 gives High Courts the ability to issue
instructions, orders, and writs to any
person or authority, including the government. Whereas, Article 227 gives High
Courts the power of
superintendence over all courts and tribunals in the territory over which they have
jurisdiction.
High Court’s power under Article 227 to be plenary and unfettered but at the same
time, the High Court should
be cautious in its exercise.
“…in cases, where the High Court exercise its jurisdiction under Article 227, such
exercise is entirely
discretionary and no person can claiming it as a matter of right.”
Jurisdiction of superintendence under Article 227 is for both administrative and
judicial superintendence.
Therefore, the powers conferred under Articles 226 and 227 are separate and
distinct and operate in different
fields. Jurisdiction under Article 227 is exercised by the High Court for the
vindication of its position as the
highest judicial authority in the State.
Scope of the power under Article 227 of the Constitution cannot be exercised
overriding the provisions of the
Special Enactments, wherein the specific reliefs are provided for redressal.
(SOURCE: https://www.scconline.com/blog/post/tag/civil-proceedings/)
97. What according to your understanding of the passage is the nature of the power
entrusted onto High Court under
Article 227?
I. The nature of superintendence is administrative as well as judicial.
II. High Court should not interfere for correcting mere error of facts or, with a
finding of the subordinate court
which is within the jurisdiction of such court.
III. The power under Article 227 of the Constitution of India is exercised to keep
the subordinate courts within
the bounds of their authority, thus, this power is to be used sparingly.
IV. When the subordinate court has assumed a jurisdiction which it does not have,
the High Court may step in
to exercise its supervisory jurisdiction.
(a) I
(b) All of the above
(c) I & III
(d) Only IV
98. Assertion: when there is a conflict, general and special provision, the later
will prevail.
Reason: Article 227 of the Constitution cannot be invoked overriding the provisions
of the Special Enactments
that do not provide for special redressal.
(a) Both A and R are false and R is not correct explanation of A.
(b) Both A and R are true and R is correct explanation of A.
(c) A is true but R is false.
(d) A is false but R is true.
99. Rita filed a writ suit for divorce from his spouse under the grounds of the
Hindu Marriage Act because he was
tormenting her indirectly on occasions. Not covering her everyday expenses and also
demotivates her for her
future career prospects. Rita, who was overwhelmed, filed for divorce. She also
claimed that she was subjected
to mental harassment. The harassment complaint was filed particularly under the
domestic violence legislation.
Can Rita ask for redress immediately under 227 of the Indian Constitution?
(a) No, because the high court's power and 227 is exercised suo moto by the court.
(b) Yes, as an individual, you have the right to seek prompt remedy before the High
Court.
(c) Yes, but it will be based on the High Court's discretion rather than an
individual's right.
(d) No, the High Court's exercise of jurisdiction under Article 227 is purely for
administrative authorities.

. Page 29 of 44
100. Continuing with the same facts as stated above, what do you believe the court
will rule on the mental harassment
case filed under the Domestic Violence Act, the remedy for which has been
addressed?
(a) The court will not interfere in mental harrassment actions brought under the
Domestic Violence Act.
(b) They are not permitted to intervene in proceedings for mental harrassment
brought under the Domestic
Violence Act.
(c) C. The high court has jurisdiction for the justification of its status as the
highest judicial authority in the
State.
(d) The high court will not interfere in procedures brought under the provisions of
the Special Enactments, if
particular remedy is provided for redressal.
Passage (Q.101-Q.105): In filing a quick appeal against the grant of statutory bail
to lawyer-activist
SudhaBharadwaj, the NIA has displayed nothing but pique and petulance over a well-
reasoned order of the
Bombay High Court. The bail order itself is a much-delayed relief, considering that
the right to ‘default bail’ had
accrued to her as early as January 2019, on completing 90 days in prison and when
there was neither a charge
sheet nor a lawful order extending the time limit for filing it from 90 to 180
days. The High Court is right in
concluding that the Sessions Court had no jurisdiction to grant such an extension,
and subsequently take
cognisance of the charge sheet filed in February 2019, when a duly constituted
Special Court under the NIA Act
was already functioning in Pune. Further, the court has given the benefit of
default bail — an indefeasible right
under Section 167(2) Cr.P.C. that arises when the investigating agency fails to
submit its final report within the
stipulated period — only to Ms. Bharadwaj, as only her application was pending at
that time; while eight others
had not specifically sought bail on that ground, even though they had questioned
the legality of the manner in
which the court had taken cognisance of the case against them. On this, case law
favours the view that if one
fails to seek statutory bail at the appropriate time, and a charge sheet is laid
subsequently, the right to default bail
is extinguished. Another instance where right to bail will be extinguished is when
required condition by court is
not fulfilled.
The NIA’s appeal exemplifies the hard-line approach of the Union government in
prosecuting the
BhimaKoregaon case under the Unlawful Activities (Prevention) Act based on a
dubious premise that some
violent incidents that occurred in the aftermath of the Elgaar Parishad event, on
December 31, 2017, were part
of a sinister Maoist conspiracy. A local investigation against the attack on a
commemoration event organised by
Dalits transmogrified into an anti-terrorism probe. After convincing the Supreme
Court that it was not a case of
suppression of political dissent, the Centre pursued the probe vigorously, and got
bail denied to everyone — save
for a temporary respite on medical grounds to Telugu poet-activist Varavara Rao.
There is also no sign of an
early trial. There are reports that some purported evidence in this case may have
been planted remotely on their
devices. It is unfortunate that courts seem to be considering bail only on medical
grounds, and in this one case,
on the ground of default. It is time they examined the merit behind the sweeping
claims in the charge sheet and
also took heed of Supreme Court judgments that have granted bail even under UAPA if
the trial is unlikely to be
completed in the foreseeable future. Accused be given protection and benefit of
default bail if investigating
agency fails to meet the required condition, because it’s not fault of arrested
person.
Source: https://www.thehindu.com/opinion/editorial/pique-and-petulance-on-sudha-
bharadwajs-bail-and-nias#appeal/article37831751.ece
101. In the Union of Virata, the national investigation authority, hereinafter the
NIA has arrested a Human Right
lawyer Ms. Rama Swamy on the charges of her indulging in the unlawful activities
under the preventive detention
law on February 20th, 2020. On the application of the NIA, the Court has ordered
her arrest for 60 days. After
the completion of the arrest, she was consecutively awarded the judicial custody
till October 15th, 2020 for 180
days. She filed for seeking the bail before the Special Court on 25th August 2020,
which was subsequently
rejected. Against the same, she filed an appeal before the High Court. Considering
the fact that the laws of the
Union of Virata is parimateriato the laws of the Union of India, decide the case.
(a) She cannot be granted the benefit of Section 167 because the term of 180 days
has not been completed in the
month of August.

. Page 30 of 44
(b) She cannot be granted the benefit of Section 167 because the Court herself has
provided the investigation
authority the time till October 15th 2020 to file the complete charge sheet.
(c) She cannot be denied the benefit of Section 167 because the term of 180 days
gets completed in August.
(d) She cannot be denied the benefit of Section 167 after the completion of the
term so provided in the law, it
becomes the Fundamental Right of the person to seek the default bail
102. Taking the facts from the previous question, supposedly, she was kept in house
arrest by the NIA without the
order of the Court for more than six months. Whether she would have been able to
seek the default bail? Decide
the case seeking the legal information given in the passage.
(a) She would be able to seek the default bail because the term of 180 days gets
expire without filing the complete
charge sheet against her, and thereafter, it becomes her right to seek default bail
(b) She would not be able to seek the default bail because she was kept in house
arrest without the order of the
Court for the said term; hence, she cannot take the benefit of the provision
(c) She would be able to seek the default bail because she was kept in house arrest
without the authorization of
the Court, which curtails her personal liberty and hence, also violative of her
Fundamental Right
(d) Both A and C
103. In the Union of Madeena, a serving Naval Commander Amrul Disht, arrested by
the central investigation
authority, the CBI in the alleged leak of confidential information on maintenance
and purchase of naval
equipment, under the provisions of the Official Secrets Act on September 2nd, 2020.
On November 2nd, 2020,
the CBI filed a charge sheet, wherein there was no mention regarding the
investigation being carried under the
Official Secrets Act (hereinafter the OSA), though the same was being carried in
this case itself. Whether he
would be eligible to get the default bail considering the fact that the limitation
for filing a charge sheet under the
OSA is 60 days, not 90 days and the laws of the Union of Madeena is pari material
to the laws of the Union of
India. Decide the case.
(a) He would be eligible to get the default bail because charge sheet filed was
incomplete in nature and hence
after the completion of the term of 60 days, he will be eligible to seek the
default bail
(b) He would be eligible to get the default bail because in the charge sheet, there
was no mentioning of the
investigation being carried under the Official Secrets Act
(c) He would not be eligible to get the bail because the charge sheet was filed
within the prescribed term of 60
days; hence, after the charge sheet being duly filed, the benefit of Section 167
cannot be availed
(d) He would not be eligible to get the bail because the charge sheet so filed
within the prescribed time cannot
be held incomplete merely on the reason that there was no mentioning regarding the
investigation being
carried under the Official Secrets Act
104. Assertion(A): The rights of an undertrial under the Constitution cannot be
allowed to be defeated on
technicalities of procedure.
Reasoning (R): Right to seek 'default' bail is a fundamental right and an
indefeasible part of right to personal
liberty under the Constitution which cannot be suspended even during a pandemic
situation.
Choose the correct option:
(a) Both A and R are false
(b) Both A and R are true, R is not the correct explanation of A
(c) A is correct and R is incorrect
(d) A is incorrect but R is correct

. Page 31 of 44
105. Mr. Mogambo, who was a gangster and mastermind of several violent activities
in the past was awarded default
bail by the High Court on January 15th, 2019 after the failure of the state to file
a charge sheet within the
prescribed period. The High Court released the man on default bail on furnishing of
a personal bond of ₹ 25,000
and a surety of the like amount. Supposedly, Mr. Mogambo is not able to furnish the
bond, and the surety, whether
his bail could be rejected on the same ground? Decide the case as per the legal
information given in the passage.
(a) The right to seek default bail is an absolute right and hence, cannot be denied
merely on the ground of the
accused not being able to furnish the bail
(b) The right to seek default bail cannot be denied on the ground of the accused
not being able to furnish the bail
because the Court has the option to seize his property to gather the bail amount
(c) The right to seek default bail is not an absolute right because it depends upon
the fulfilment of conditions
(d) The right to seek default bail is a statutory right and hence, it is the
discretion of the Court to provide or
reject the same on the condition of the fulfilment of the terms so attached with
the bail

. Page 32 of 44
SECTION - D: LOGICAL REASONING
Passage (Q.106-Q.110): Read the passage below and answer the questions that follow:
P1. The internet opened up a world that was truly revolutionary. But now, now that
we're all online, and any
novelty to this fact has worn off, the internet has closed that world. We now only
have to interact with people
who agree with us; if I use Twitter as my primary news source, as so many people
do, I can carefully curate my
feed to exclude anyone who disagrees with me about anything. (And if someone who
slips in there who does, I
can call them a horrible person.) this is now accepted public policy. You don't
have to find anyone to contradict
you, if you don't want to.
P2. This isn't just common practice now: This is how you win. The entire strategy
for succeeding at anything,
whether it's winning elections, selling a product or attracting visitors for your
Website, revolves around pitching
yourself as loudly as you can to those people on your side and turning those who
disagree with you into the worst
version of themselves, demonizing them into something subhuman and venal. Nuance is
tossed out, even if you
know a situation is desperately nuanced, in favour of quick points and splash;
we've all become the New York
Post.
P3. This is simply how communication is done now. The idea of unifying anyone on
anything is passé, old
thinking, a waste of time. A horrible tragedy happens, and your first reaction,
rather than taking a moment to
mourn or quietly search for some grace and peace, is instead to start screaming and
claiming that those with
whom you disagree have blood on their hands. You are rewarded with this by the top
slot on the news, a video
that goes viral, and everyone on your side applauding you. After all, every time
you say something loudly and
strongly enough, the people who agree with you tell you how great you are.
P4. It can be so demoralizing, so exhausting, to watch this day after day after
day. We have begun to shout at
those with whom we disagree as if they are terrible drivers and we're within the
safety of our own cars; they're
the anonymous, faceless monsters we shower with the worst possible motives, just
because they happen to be in
our way when we're in a hurry. Except they can hear us. And so can everyone else.
So, one tries to find hope.
P5. I tend to find it outside, where people, you know, are. Because we drop this
act during those strange,
disorienting times when we find ourselves in mixed company, lo, real life. The
things we do online, or when we
think someone is watching, we don't do these things in the real world. In regular,
everyday life, we accept all the
time that those who disagree with us exist; sometimes we even like them. They're
our families, they're our friends,
they're our neighbours, they're the people we open the door for at the supermarket.
They're human beings, idiot,
scared, just-trying-to-hang-on human beings like every single one of the rest of
us.
106. We now only have to interact with people who agree with us; if I use Twitter
as my primary news source, as so
many people do, I can carefully curate my feed to exclude anyone who disagrees with
me about anything. What
is the underlying assumption behind the passage?
(a) If one wants selective news feed and interaction with the like-minded people,
Twitter is the place to go to.
(b) Twitter allows sifting through of news feed according to likes and dislikes.
(c) People are unable to swallow an opposite view.
(d) One tends to associate with those that agree with one’s thought-process.
107. Which one of the following best indicates a flaw in the argument about tossing
out of nuance?
(a) People are incapable of segregating the pitches and the nuances.
(b) People are capable of segregating the pitches and the nuances.
(c) People are only interested in loud pitches.
(d) People rarely follow such advertisements or sites that pitch loudly.

. Page 33 of 44
108. Which one of the following, if true, most seriously weakens the contentions
given in the third paragraph of the
passage?
I. If you are an anti-vaccine activist, you can read so much "information"
supporting your position that, as far
as you can tell, you are right.
II. You don't have to be right; you just have to be louder than the other guys.
III. We accept human frailty, that we do not share a cerebral cortex with every
other person on the planet and
therefore will not always see eye-to-eye on all matters.
(a) Only I
(b) Only II
(c) Only III
(d) I & III
109. Two arguments have been provided in the form of statements. Select the correct
relationship between them from
the given options.
Statement I: Pitching loudly isn't just common practice now: This is how you win.
Statement II: Pitching loudly is simply how communication is done now.
(a) Statement I is an assertion; statement II is a reason.
(b) Statement I is an assertion; statement II is also an assertion.
(c) Statement I is a cause; statement II is an effect.
(d) Statement I is a fact; statement II is a judgement.
110. Which of the following is a conclusion drawn from the passage?
(a) How internet has closed off the world and made us a world of closed chambers.
(b) How internet has turned people into zombies that follow a culture mindlessly.
(c) How online news drowns the dissenting voices permanently.
(d) How internet has become a loudspeaker of a particular thought and
decompartmentalise views in closed
chambers.
Passage (Q.111-Q.115): Fishermen from Tamil Nadu keep getting caught with alarming
regularity in the
territorial waters of Sri Lanka for “poaching”. [1] Yet, the stakeholders concerned
have yet to demonstrate
the alacrity required for well-known solutions. In the latest development, the Sri
Lankan Navy arrested 22
fishermen who are from Nagapattinam and neighbouring Karaikal, on Wednesday. There
are already 29
fishermen in custody in Sri Lanka. As per an estimate, Sri Lankan authorities have
also impounded 84 boats.
The frequency with which Tamil Nadu’s fishermen allegedly cross the International
Maritime Boundary Line,
despite being aware of the consequences, highlights their level of desperation
driven by livelihood concerns.
This is, however, not to absolve them of their culpability in endangering Sri
Lanka’s marine biodiversity. The
vexatious problem has also been aggravated by events over the past month — the
reported death of two Jaffna
fishermen following “mid-sea clashes with their Tamil Nadu counterparts” on January
27 and 29, subsequent
protests by northern Sri Lankan fishermen, and the reported auctioning by Sri Lanka
of 140 impounded boats.
There has been no word from Colombo on permitting fishermen-devotees to attend, in
March, the annual festival
of St. Anthony’s Church at Katchatheevu.
Apart from getting the arrested fishermen released, the governments of the two
countries should fix a date for an
early meeting of the Joint Working Group, last held two years ago. They should also
facilitate the resumption of
talks at the level of fisherfolk, especially from Tamil Nadu and the Northern
Province. Sri Lanka should be
proactive as its citizens in the North bear the brunt of the alleged acts of
transgression. Besides, its positive
actions would be in tune with what the Prime Ministers of India and Sri Lanka
agreed at the virtual summit in
September two years ago— to “continue engagement to address the issues related to
fishermen through regular
consultation and bilateral channels”. New Delhi should also consider providing
additional incentives and
concessions to fishermen of the Palk Bay districts of Tamil Nadu to elicit a better
response from them for its
. Page 34 of 44
deep sea fishing project. It could also propose assistance for the fishermen of the
Northern Province as a gesture
of goodwill. [2] There is no paucity of ideas in the area of the Palk Bay fisheries
conflict, but adequate
action on the part of the stakeholders is found wanting.
111. If the information given in the passage is true, then which of the following
statements is supported?
(a) Providing incentives to the fishermen of some districts of Tamil Nadu would
stop them from violating Sri
Lanka’s boundary.
(b) There are not enough solutions about the fishermen issue between Sri Lanka and
India.
(c) Events other than the arrest of Indian fishermen have escalated the fishermen
issue.
(d) If the Sri Lankan authorities do not stop arresting fishermen, India would
bring this issue to the United
Nations.
112. Which of the following pieces of evidence would lend support to the author’s
arguments?
(a) Several talks have been held between the authorities of India and Sri Lanka
over the past two years, but to
no avail.
(b) According to a research study, Tamil Nadu’s fishermen are among the least paid
fishermen of India.
(c) According to the UN Convention on the Laws of the Sea, fishermen who have
transgressed international
boundaries lose their citizenship.
(d) The ideas to resolve the Palk Bay fisheries conflict suffers from lack of a
dialogue.
113. The relationship between the two boldface statements 1and 2 is best depicted
by?
(a) Both these statements are counterarguments of each other.
(b) Statement1 provides the premise for the claim made in statement 2.
(c) Statement 2 provides the premise for the claim made in statement 1.
(d) Both the statements 1 and 2 have claims made on similar lines.
114. The author has used which of the following as a premise in the passage?
i. Sri Lanka should be proactive in finding solutions.
ii. The Sri Lankan navy has arrested around two dozen Indian fishermen.
iii. The authorities are not motivated enough to find solutions to the problem.
(a) Only II
(b) Only I and II
(c) Only II and III
(d) All of the above.
115. In each question below is given a Statement followed by two Assumptions
numbered I and II. An assumption is
something supposed or taken for granted. You have to consider the Statement and the
following assumption and
decide which of the assumption is implicit in the Statement.
Statement: The frequency with which Tamil Nadu’s fishermen allegedly cross the
International Maritime
Boundary Line, despite being aware of the consequences, highlights their level of
desperation driven by
livelihood concerns.
Assumption 1: Concern for livelihood takes precedence over everything else.
Assumption II: The livelihood of fishermen is fraught with danger.
(a) If only assumption I is implicit.
(b) If only assumption II is implicit.
(c) If neither I nor II is implicit.
(d) If both I and II are implicit.
of Tamil Nadu, and not all fishermen.

. Page 35 of 44
Passage (Q.116-Q.120): Office workers could - fingers crossed for the salaried
class - be in for their best
increments this year since the pandemic struck. [1] One in three companies is
giving pay hikes in the double
digits as the average rate converges to 9%. Pay hikes in industries that have
pulled out of the Covid-19 pandemic
trough are ratcheting up as companies grapple with the biggest mass resignation in
over a decade. The skill
supply disruption has pushed salary increases in India way past those in other
emerging markets, and employers
are offering greater flexibility in working conditions - hybrid or work from home
included - in an effort to retain
staff. Companies have upped their talent-retention initiatives and the pressure on
payrolls has spread to sectors
that saw demand collapsing in the first wave of lockdowns.
[2] Salaries appear poised for a breakout in hi-tech clusters. [3] The information
technology industry added
almost half-a-million workers in the year to March 2022, as revenues grew at their
quickest pace since the global
financial meltdown. The Indian technology industry is earning a quarter of its
revenue at home, and niche skills
are driving the digitisation of the broader economy. [4] A survey of employers by
HR consultancy Aon finds
ecommerce, infotech and professional services at the top of the salary sweepstakes
with hikes above 10%.
Manufacturing, metals and mining are expected to see relatively lower increments.
But these, too, should be
upwards of 7%. Talent is most scarce in data analytics, engineering, and sales and
marketing, reveals a study by
Mercer.
These findings predate the breakout of hostilities between Russia and Ukraine, and
should be read in the context
of surging energy prices. Yet, the longer trend that has equipped a third of the
country's 5 million infotech
workers with digital skills is fairly entrenched. The salary hike projections
signify India Inc's efforts to build a
more resilient workforce. The pandemic has brought to the fore the reskilling
premium.
116. The central argument of the passage is best represented by:
(a) India Inc’s efforts to build a resilient workforce seems to have paid off.
(b) The pandemic brought with itself the great economic slowdown.
(c) The hike in the employees’ salaries would weaken the ‘great resignation’.
(d) Efforts need to be made to bring in fresh talent in some industries.
117. Out of the following, which option most weakens the author’s arguments?
(a) The salaries of the employees who chose to work from home are being slashed.
(b) The employers are organising workshops at regular intervals for the betterment
of their employees.
(c) The increments for the office workers and pay hikes in some industries is a
knee jerk response to Covid-19.
(d) The increments for the office workers and pay hikes in some industries were due
for a long time.
118. The author’s arguments require which of the following to be assumed?
(a) The relationship between Russia and Ukraine could impact the salary hikes.
(b) The energy prices have no nexus with the salaries of office employees.
(c) Companies that would increase salaries in double digits have more workforce
than those with single digit
hikes.
(d) The Indian technology industry grew more than twenty-five percent since the
Pandemic.
119. Out of the given options, select the statement out of the given statements in
the passage marked as 1,2,3 & 4,
that is the odd one out. Rely on the concepts of opinion statements and factual
statements to arrive at your answer.
(a) [1] (b) [2] (c) [3] (d) [4]
120. With which of the following options would the author of the passage agree
with?
(a) Russia is going to capture the territory of Ukraine.
(b) Less supply has surged the energy fuel prices to an all-time high.
(c) An 8% increment is not necessarily too bad for the employees.
(d) HR Consultancy Aon has a record of presenting discreditable information.

. Page 36 of 44
Passage (Q.121-Q.125): The accepted definition of creativity is production of
something original and useful,
and that's what's reflected in the tests. There is never one right answer. To be
creative requires divergent thinking
(generating many unique ideas) and then convergent thinking (combining those ideas
into the best result).
When you try to solve a problem, you begin by concentrating on obvious facts and
familiar solutions, to see if
the answer lies there. This is a mostly left-brain stage of attack. If the answer
doesn't come, the right and left
hemispheres of the brain activate together. Neural networks on the right side scan
remote memories that could
be vaguely relevant. A wide range of distant information that is normally tuned out
becomes available to the left
hemisphere, which searches for unseen patterns, alternative meanings, and high-
level abstractions.
Having glimpsed such a connection, the left brain must quickly lock in on it before
it escapes. The attention
system must radically reverse gears, going from defocused attention to extremely
focused attention. In a flash,
the brain pulls together these disparate shreds of thought and binds them into a
new single idea that enters
consciousness. This is the "aha!" moment of insight, often followed by a spark of
pleasure as the brain recognizes
the novelty of what it's come up with.
Now the brain must evaluate the idea it just generated. Is it worth pursuing?
Creativity requires constant shifting,
blender pulses of both divergent thinking and convergent thinking, to combine new
information with old and
forgotten ideas. Highly creative people are very good at marshaling their brains
into bilateral mode, and the more
creative they are, the more they dual-activate.
Like intelligence tests, Torrance's test—a 90-minute series of discrete tasks,
administered by a psychologist—
has been taken by millions worldwide in 50 languages. Yet there is one crucial
difference between IQ and CQ
scores. With intelligence, there is a phenomenon called the Flynn effect—each
generation, scores go up about
10 points. Enriched environments are making kids smarter. With creativity, a
reverse trend has just been
identified and is being reported for the first time here: American creativity
scores are falling.
The potential consequences are sweeping. The necessity of human ingenuity is
undisputed. A recent IBM poll
of 1,500 CEOs identified creativity as the No. 1 "leadership competency" of the
future. Yet it's not just about
sustaining our nation's economic growth. All around us are matters of national and
international importance that
are crying out for creative solutions, from saving the Gulf of Mexico to bringing
peace to Afghanistan to
delivering health care. Such solutions emerge from a healthy marketplace of ideas,
sustained by a populace
constantly contributing original ideas and receptive to the ideas of others.
It's too early to determine conclusively why U.S. creativity scores are declining.
One likely culprit is the number
of hours kids now spend in front of the TV and playing videogames rather than
engaging in creative activities.
Another is the lack of creativity development in our schools. In effect, it's left
to the luck of the draw who
becomes creative: there's no concerted effort to nurture the creativity of all
children.
121. As per the passage, what is the assumption behind creativity?
I. Divergent and convergent thinking are prerequisites to creativity.
II. Creativity is an abstract.
III. Creativity is the combining of unique ideas into an unprecedented idea.
(a) Only I (b) Only I (c) I & II (d) I, II & III
122. All of the following can be inferred from the passage, except
(a) Left brain activates the obvious facts and familiar solutions.
(b) The ‘aha!’ moment is the peak of creative moment.
(c) Flynn effect is concerned with Creativity Quotient.
(d) The right hemisphere stores the remote memories.
123. Which of the following explains divergent thinking?
(a) Generation of many different ideas in a free-flowing, unstructured manner.
(b) Generation of multitude of connected ideas shaping into a unique idea.
(c) A cognitive process that is methodical and powerful.
(d) A cognitive process of narrowing down of possible solutions until an ideal
solution is reached.

. Page 37 of 44
124. Study the two activities, and decide the thinking that needs to be applied
between divergent and convergent or
both.
Statement I: Narrowing down colours choices to decorate your living room with.
Statement II. Thinking of different possibilities of colours to decorate your
living room with.
(a) Statement I is an example of divergent thinking, and statement II is an example
of convergent thinking.
(b) Statement I is an example of convergent thinking, and statement II is an
example of divergent thinking.
(c) Both statements I and II are examples of divergent thinking.
(d) Both statements I and II are examples of convergent thinking.
125. Given below are pairs of events ‘A’ and ‘B’. You have to read both the events
‘A’ and ‘B’ and decide their
nature of relationship. You have to assume that the information given in ‘A’ and
‘B’ is true and not assume
anything beyond the given information in deciding the answer. Marks answer;
Event A: All around us are matters of national and international importance that
are crying out for creative
solutions.
Event B: There's no concerted effort to nurture the creativity of all children.
(a) If ‘A’ is the effect and ‘B’ is its immediate and principal cause.
(b) If ‘A’ is the immediate and principal cause and ‘B’ is its effect.
(c) If ‘A’ is an effect but ‘B’ is not its immediate and principal cause.
(d) If ‘B’ is an effect but ‘A’ is not its immediate and principal cause.
Passage (Q.126-Q.130): As most of India grapples with emotions to come to terms
with the tragic news of the
death of two of our students in Ukraine, besides experiencing anxiety over the fate
of those who still remain
stranded in eastern parts of that country, there’s more disturbing news trickling
in. As per latest reports citing
some Indian students who have run aground since the Russian invasion began, local
authorities and the military
in Kharkiv are now assaulting these students, pointing guns at them and pushing
them out of bunkers and trains
arranged to evacuate people from the region. The scenes are reminiscent of
apartheid and unadulterated
discrimination: After trains open their locked doors at a station, the authorities
allow only the Ukrainians —
children first, their mothers followed by other women and, lastly, Ukrainian men —
to board. All attempts to
contact the Indian embassy were wasted as the beleaguered students could not get
through. The embassy in Kyiv
was already shut down on March 1. The Ukrainian officials on patrol around town
harbour a grudge against the
Indian Government as they reportedly told these students that when India was not
supporting the war-torn nation,
why should they help Indians?
Of the 18,000-20,000 Indian students in Ukraine, only 6,000 have been brought back
so far. The students,
especially girls, are braving snowfall without shelter and awaiting evacuation at
Ukraine’s eastern borders. The
normal bond between average Ukrainians and Indians is tellingly captured in the
true story about a student who
turned down an offer to be evacuated because she wanted to stay with her landlady’s
two young daughters to
take care of them after the family man, a civilian, volunteered to enlist in the
Ukrainian Army. But this initial
mood of euphoria and pride being an Indian seems to be steadily giving way to
hopelessness and despair against
the Government among the Indian expatriates. If India had not extended tacit
approval to Russia’s invasion of
Ukraine earlier this week at the UN Security Council and the UN General Assembly
meetings, Ukraine’s Army
and general populace might not have turned against hapless Indian students, who are
innocent young adults, like
those of any other nationality. India’s neutral stance is certainly not helping
Indians survive easily in Ukraine
though it might have helped the cause of Indian diplomats.
126. According to the author, what can be a logical course of action?
I. India sends its army personnel to rescue the Indian students caught in Ukraine.
II. India escalates the process of rescuing its remaining students in Ukraine.
III. India goes out and openly criticises the Russian army for invading Ukraine.
(a) Only (b) Only II (c) Only III (d) Only II and III

. Page 38 of 44
127. The arguments of the author depend on which of the following assumptions?
(a) By assaulting the Indian students, India would take action against Russia.
(b) If India takes action against Russia, Indian students would not be assaulted.
(c) The stance taken by the government towards Ukraine in the Russia-Ukraine
conflict has affected the stranded
Indian students in Ukraine.
(d) The Indian government is solely responsible for the plight of the Indian
students stranded in Ukraine, as they
suffer from apartheid and discrimination.
128. If the information given in the passage is true, then which of the following
can be said to be true as well?
(a) India has provided direct support for Russia in its invasion of Ukraine.
(b) Indian students have not done anything to incite the Ukrainian officials.
(c) India should go ahead and take some diplomatic action against Russia.
(d) India’s neutral stand has hurt the sentiments of countries supporting Ukraine.
129. The arguments of the author are weakened by which of the following options?
(a) The Indian government had given a fair warning to the Indian students two
months preceding the Russia#Ukraine conflict to evacuate Ukraine.
(b) The military capabilities of Ukraine are far superior to Russia.
(c) Many Indian students who pursue their education from Ukraine build lifelong
relationships with the locals.
(d) The Indian government was in constant dialogue with the Ukrainian government to
aid in the evacuation of
the Indian students.
130. Which of the following can be inferred with reference to the Indian students
in Ukraine, from the author’s stance?
(a) As hardships pile on, stranded Indian students in Ukraine are looking at
another facet of life.
(b) As hardships pile on, stranded Indian students in Ukraine are losing hope with
Russia.
(c) As hardships pile on, stranded Indian students in Ukraine are looking at India
to help them.
(d) As hardships pile on, stranded Indian students in Ukraine have lost hope with
the Indian government.

mock44
ECTION-A: ENGLISH LANGUAGE
Directions (Q.1-Q.30): Read the following passage carefully and answer the
questions that follow.
Passage (Q.1-Q.6): Karl Heinrich Marx (\May 5, 1818-March 14, 1883) was a German
philosopher, political
economist, sociologist, humanist, political theorist and revolutionary credited as
the founder of communism.
Marx's approach to history and politics is indicated by the opening line of the
first chapter of The Communist
Manifesto (1848): "The history of all hitherto existing society is the history of
class struggles". Marx argued that
capitalism, like previous socioeconomic systems, will produce internal tensions
which will lead to its destruction.
Just as capitalism replaced feudalism, capitalism itself will be displaced by
communism, a classless society
which emerges after a transitional period-socialism - in which the state would be
nothing else but the
revolutionary dictatorship of the proletariat.
On the one hand, Marx argued for a systemic understanding of socioeconomic change.
He argued that it is the
structural contradictions within capitalism which necessitate its end, giving way
to communism:
"The development of Modern Industry, therefore, cuts the ground from under its feet
the very foundation on
which the bourgeoisie produces and appropriates products. What the bourgeoisie,
therefore, produces, above all,
are its own grave-diggers. Its fall and the victory of the proletariat are equally
inevitable.”
- (The Communist Manifesto)
While Marx was a relatively obscure figure in his own lifetime, his ideas began to
exert a major influence on
workers' movements shortly after his death. This influence was given added impetus
by the victory of the Marxist
Bolsheviks in the Russian October Revolution, and there are few parts of the world
which were not significantly
touched by Marxian ideas in the course of the twentieth century.
Karl Heinrich Marx was born in Trier, in the Kingdom of Prussia's Province of the
Lower Rhine, the third of
seven children. His father, Heinrich Marx (1777-1838), born Herschel Mordechai, the
son of Levy Mordechai
(1743-1804) and Eva Lwow (1753-1823), Marx's mother was Henriette née Pressburg
(1788-1863).
Soon after losing his job as editor of Rheinische Zeitung, a Cologne newspaper,
Karl Marx was married to Jenny
von Westphalen, the educated daughter of a Prussian baron, on June 19, 1843 in
Kreuznacher Pauluskirche, Bad
Kreuznach. Their engagement was kept secret at first, and for several years was
opposed by both the Marxes and
Westphalens. From 1844 to 1848. Marx enjoyed a very comfortable lifestyle, with
income derived from the sale
of his works, his salary, gifts from friends and allies; a large inheritance from
his father's death, long delayed,
also became available in March 1848. During the first half of the 1850s the Marx
family lived in poverty and
constant fear of creditors in a three room flat on Dean Street in Soho. London.
Marx and Jenny already had four
children and three more were to follow. Of these only three survived to adulthood.
Marx's major source of income
at this time was Engels, who was drawing a steadily increasing income from the
family business in Manchester.
This was supplemented by weekly articles written as a foreign correspondent for the
New York Daily Tribune.
Inheritances from one of Jenny's uncles and her mother who died in 1856 allowed the
family to move to
somewhat more salubrious lodgings at 9 Grafton Terrace. Marx generally lived a
hand-to-mouth existence,
forever at the limits of his resources, although this did extend to some spending
on relatively bourgeois luxuries,
which he felt were necessities for his wife and children given their social status
and the mores of the time.
1. Karl Marx and his wife's engagement was kept secret because
(a) The engagement would have created an uproar amongst the communists.
(b) Their engagement was opposed by both Marx's parents as well as his wife's
parents.
(c) Of the Marx's Rabbi descendance.
(d) Of Marx's admiration for figures such as Voltaire and Rousseau.
2. Which of the following best reflects the meaning of the word ‘proletariat’?
(a) Aristocracy. (b) Dictators. (c) Working class. (d) Political leaders.

. Page 3 of 40
3. According to the passage, which of the following statements is false?
(a) Marx was an originator of communism.
(b) Marx was the third of the seven siblings.
(c) Marx’s ideas exerted influence late in his life.
(d) Marx wrote for the New York Daily Tribune.
4. It was Marx's belief that
(a) the faulty structure of capitalism would lead to its destruction.
(b) Communism would be replaced by socialism.
(c) Dictatorship is the solution to all socio-economic problems.
(d) Capitalism begets feudalism.
5. Which of the following statements would the author not agree with?
(a) Marx witnessed both a life of poverty and a life of luxury.
(b) Marx's ideology and principles attained attention posthumously.
(c) Marxian ideology was restrained to a small part of Europe mainly Germany.
(d) Marx believed that Capitalism would eventually give rise to communism.
6. "The development of Modern Industry, therefore, cuts the ground from under its
feet the very foundation on
which the bourgeoisie produces and appropriates products. Which of the following
figures of speech does the
line represent?
(a) Oxymoron (b) Antithesis (c) Simile (d) Paradox
Passage (Q.6-Q.11): Evidence for the prolonged presence of potentially-life-
supporting, salty, acidic water on
the surface of Mars claims top honors as the Breakthrough of the Year, named by
‘Science’ and its publisher,
AAAS, the non-profit science society. With the help of remote-sensing spacecraft,
NASA’s two hardy little
robotic explorers performed the first true geologic field explorations on another
planet. Their findings suggest a
wet, warm Mars that could have been capable of cradling life billions of years ago,
when life on Earth was getting
its start.
These findings plus nine other research advances make up ‘Science’s’ list of the
top ten scientific developments
in 2004, chosen for their profound implications for society and the advancement of
science. South Korean
researchers made headlines worldwide this year when they announced that they had
cloned a human embryo,
the first evidence that this technique could work with human cells. The
researchers’ intention was to derive
embryonic stem cell lines that could help researchers understand complex diseases
or eventually produce
genetically matched replacement cells for patients. With an understanding of how to
chill the two basic types of
atoms into a single quantum state or condensate under their belts, researchers got
down to probing these strange
forms of matter in 2004. They learned how condensates’ behaviour changes as atoms
grow further apart, and
they created a solid condensate, complementing earlier successes with gas and
liquid versions.
The stretches of ‘junk DNA’ that lie within genomes proved this year to be far more
important than previously
thought. This DNA, found between genes and between a gene’s protein-coding regions,
turns out to be essential
for helping genes turn on at the right time and in the right place. Astrophysicists
discovered the first known pair
of pulsars, spinning neutron stars that shoot out jets of radiation. Further
studies of these whirling objects may
provide the most stringent examination yet of Einstein’s general theory of
relativity.
While evidence of an ancient, wet Mars grabbed the spotlight in 2004, advances in
our understanding of water
itself flowed freely as well. If they hold up, new results on the structure and
chemical behaviour of water could
reshape fields from chemistry to atmospheric sciences. ‘Public-private
partnerships’ emerged as a formidable
force in 2004, affecting the way medicines are developed and delivered to
developing countries. Joint ventures

. Page 4 of 40
by foundations, rich countries, academics, pharmaceutical companies and other
groups were behind several
prominent initiatives this year, including a malaria vaccine trial and efforts to
provide anti-HIV drugs.
Researchers hit on a new way to identify life forms too small and too remote to see
with the naked eye. They
collected water from environments as different as the Sargasso Sea and a deep
abandoned mine and then
sequenced the genes floating in it. This work has turned up new genes and genomes
alike.
7. ‘Junk DNA’ was so called because
(a) it was found in a large mess. (b) it was found in disorder.
(c) it was found in large numbers. (d) it was thought to be of little value.
8. According to the passage, when could Mars have been capable of supporting life?
(a) Along the origin of Earth. (b) When the life on earth getting its start.
(c) After life started on Earth. (d) Life on Mars is a mere speculation.
9. According to the passage, what was the purpose behind cloning a human embryo?
(a) To produce humans with desired characteristics.
(b) To produce humans with healthy physique.
(c) To enable doctors to supplant diseased cells with new ones.
(d) To cure infertility in humans.
10. Which among the scientific developments listed is not an invention or a
discovery?
(a) Condensing two types of atoms into a single quantum state.
(b) Public-private partnerships affecting the way medicines are produced.
(c) Way to identify life forms too small and too remote to see with the naked eye.
(d) Spinning neutron stars that shoot out jets of radiation.
11. What does ‘species diversity’ mean?
(a) Presence of large number of living beings.
(b) Living beings with divergent characteristics.
(c) A large variety of living beings.
(d) Living beings which are inimical to one another.
Passage (Q.12-Q.15): Just five survivors remain today from the three Soviet
divisions which liberated
Auschwitz concentration camp in January 1945. I am the youngest – I was only 19
when the war ended. But the
events of 60 years ago are as fresh in my memory as if they happened yesterday.
Although I was just 15 years
old, I was immediately conscripted. We were kept in reserve, but when I turned 17,
I was sent to the front in
January, 1953; we liberated Kursk. It was a bloodbath: a whole regiment was killed
in three hours. Later, I was
badly wounded in the chest in the battle of Kursk.
On recovery, I caught up with my regiment, under the command of General Vasily
Petrenko, who died not long
ago. He was a great commander. Under him we liberated Lvov in the summer of 1944,
and on January 19, 1945,
we freed Krakow, a beautiful ancient city. At about 4 a.m. on January 27 we
approached Oswiecim (Auschwitz).
It is a small town on the Sola River. We did not even know there was a
concentration camp there.
The Germans had far better weapons than us, and their rations were excellent, not
like the gruel we had. It was
mild for January. There was no snow, which we needed to melt in our pots to get
water.
We won that war with our bodies. It was tough in Auschwitz, too. The Germans
deployed artillery and
submachine guns outside the camp. They shot at us from the watchtowers and
barracks. The prisoners could
barely walk: they looked like shadows or ghosts, they were so skinny. Some could
not even move, others were

. Page 5 of 40
supported by friends. They tried to talk to us, but we could not understand them:
there were people from different
countries, including many Jews from France, Poland and even Palestine.
At the time of our assault there were 7,000-10,000 people in the camp – I learned
after the war that the Germans
had earlier shipped hundreds of thousands of prisoners to Germany and continued to
use them for forced labour.
But those left behind were barely alive.
They had not had time to blow up anything or plant mines. There was a huge
construction site next to the camp:
prisoners were building a chemicals plant. There were not just camp inmates working
there, but also tens of
thousands of civilians shipped from the Soviet Union.
The grim barracks stood in rows and, from a distance, looked like a factory – and
it was a real factory of death.
I saw a great deal in the war, but nothing so horrible or awful as that camp. The
experience gave us a new energy
and determination to put an end to the abomination of Nazism.
I do not believe humanity will forget the suffering of the victims of Auschwitz,
nor the bloodshed by their
liberators. Anyone who witnessed such a nightmare would do anything possible to
prevent it from happening
again.
12. The narration focusses on
(a) the inhuman nature of the Nazis.
(b) the difficulties faced by the Russians in dealing with the Auschwitz inmates.
(c) the pitiable plight of the people in the Auschwitz concentration camp.
(d) the course that World War II traversed.
13. The Auschwitz camp is described as a ‘factory of death’ because
(a) this was where people died or were put to death in large numbers.
(b) the inmates of the camp either barely survived or died due to the atrocities.
(c) the inmates were forced to work in chemical factories, hazardous to health.
(d) a large number of soldiers from both sides died.
14. From the passage it can be understood that the inmates of the concentration
camp were
(a) Germans. (b) Russian civilians.
(c) Jews belonging to some countries. (d) People and Prisoners of war from various
countries.
15. ‘Although I was just 15 years old, I was immediately conscripted.’ Which of the
following is the closest synonym
for the underlined word?
(a) Escaped. (b) Submitted. (c) Enrolled. (d) Imprisoned.
Passage (Q.16-Q.20): For those who haven’t been following the ongoing drama in the
politics of the United
Kingdom, here’s the basic storyline: after an initially botched response, where it
ignored the seriousness of the
Covid-19 virus, Boris Johnson’s government declared strict lockdown protocols for
the whole country; as 2020
and 2021 unfolded, Britain witnessed, and suffered under, further incompetencies
and the lack of concern from
Johnson and his team; scandals to do with the government also began to emerge.
These substantial accusations
of flagrant disregard for rules he had himself made for the rest of the country
have now got Johnson teetering on
the edge of the precipice.
Like many current counterparts in other countries, Johnson is famous for never
knowingly speaking the truth.
Like other prime ministers and presidents, we know, Johnson has his own — typical —
methods of avoiding
answering hard questions; a favourite trick is what one might call the ‘chaff
release’ method, named after the
bunches of tiny aluminium or zinc strips warplanes release as decoys to
electronically confuse missiles coming
at them.

. Page 6 of 40
Before he became leader of the Labour Party, Starmer had been Director of Public
Prosecutions and head of the
Crown Prosecution Service, the government department that prosecutes criminals in
court.
Krishnan Guru-Murthy, one of Britain’s seniormost television anchors, confronts
Nadine Dorries, secretary for
culture and one of the few remaining Johnson loyalists in the cabinet, and asks her
bluntly: “How can you have
a Prime Minister just repeating fake news like that?” Dorries’ response is to
shuffle sideways, saying first that
she knows nothing about Starmer’s background, second, reaching for whataboutery,
that Starmer himself also
said things that were inappropriate and, third, that she only has Guru-Murthy’s
word about Starmer’s record.
None of this washes with the TV journalist. “He’s just repeating a meme!” says
Guru-Murthy before adding,
“He said things that were untrue, he misled the House today!”
It’s clear that Johnson’s government is in complete shambles, generously self-
garnished with disgrace and
vicious mendacity. It’s clear that the British prime minister is incompetent,
completely indifferent to the fate of
the poorest people under his care as he bends over backwards to serve his rich
friends, that he is completely
narcissistic, and willing to blow apart any principle or truth that obstructs his
continuation in power. This much
is closely comparable to what we are facing in India today.
Whether they stem from geographical or demographic scale, from history or culture,
what should concern us are
the differences in what can and can’t happen from this point on. For instance, it’s
unlikely that the Conservative
Party or the Tory press will allow any kind of a personality cult to encrust around
Johnson. As the suffering
audience of our own, ongoing grotesque fancy-dress party, we will have to arrive at
our own route to ending the
mehfil of the few.
16. In many current counterparts in other countries, Johnson is famous for what?
(a) Never knowingly speaking the truth. (b) Never knowingly speaking the lie.
(c) Knowingly speaking the truth. (d) Never unknowingly speaking the truth.
17. Which of the following is true regarding the second paragraph?
(a) Johnson’s favourite trick is the chaff release method.
(b) Johnson avoids answering hard questions.
(c) Chaff release method is named after bunches of tiny aluminium strips.
(d) All of the above.
18. Which of the following title captures the central theme of the passage?
(a) Political Gambling. (b) Boris Johnson- In full retreat.
(c) Fair play by Boris Johnson (d) Government in complete shambles.
19. Which of the following correctly interprets the idiomatic expression ‘bending
over backwards’?
(a) To try very hard to do everything for someone at the cost of one’s risk.
(b) To try extremely hard to help or to please someone, even if causes you trouble.
(c) To try hard enough to stick to the principles, and not relent to pressures.
(d) To try extremely hard to please everyone, even at the cost of one’s harm.
20. ‘…and willing to blow apart any principle or truth that obstructs his
continuation in power. The underlined word
belongs to which parts of speech?
(a) Adjective (b) Adverb (c) Pronoun (d) Conjunction

. Page 7 of 40
Passage (Q.21-Q.25): Polydectes announced a large banquet where each guest was
expected to bring a gift.
Polydectes requested that the guests bring horses, under the pretense that he was
collecting contributions for the
hand of Hippodamia, "tamer of horses". The fisherman's protegé had no horse to
give, so asked Polydectes to
name the gift, for he would not refuse it. Polydectes held Perseus to his rash
promise, demanding the head of the
only mortal Gorgon, Medusa, whose very expression turned people to stone. Ovid's
anecdotal embroidery of
Medusa's mortality tells that she had once been a woman, vain of her beautiful
hair, who lay with Poseidon in
the Temple of Athena. In punishment for the desecration of her temple, Athena
changed Medusa's hair into
hideous snakes "that she may alarm her surprised foes with terror".
Athena instructed Perseus to find the Hesperides, who were entrusted with weapons
needed to defeat the Gorgon.
Following Athena's guidance, Perseus sought out the Graeae, sisters of the Gorgons,
to demand the whereabouts
of the Hesperides, the nymphs tending Hera's orchard. The Graeae were three
perpetually old women, who had
to share one eye and one tooth among them. As the women passed the eye from one to
the other, Perseus snatched
it from them, holding it ransom in return for the location of the nymphs. When the
sisters led him to the
Hesperides, he returned what he had taken. From the Hesperides, he received various
treasures: a knapsack
kibisis to safely contain Medusa's head, winged sandals to fly, Hades' helm of
invisibility to hide. Hermes loaned
Perseus an adamantine sword, while Athena gave him a polished shield. Perseus then
proceeded to the Gorgons'
cave. In the cave he came upon the sleeping Stheno, Euryale and Medusa. By viewing
Medusa's reflection in his
polished shield, he safely approached and cut off her head. From her neck sprang
Pegasus ("he who sprang") and
Chrysaor ("bow of gold"), the result of Poseidon and Medusa's meeting. The other
two Gorgons pursued Perseus,
but under his helmet of invisibility he escaped.
21. Which of the following cannot be implied from the passage?
(a) Stheno and Euryale were Gorgons.
(b) Perseus received several magical items from Hermes.
(c) Perseus was able to escape after chopping off Medusa's head.
(d) Perseus kept his word.
22. Explain the use of the phrase “anecdotal embroidery” in the passage.
(a) Anecdotal embroidery refers to the art of overlaying a form of text with
another.
(b) Medusa’s story was told by Ovid in order to distract the reader from Perseus.
(c) It is often used to explain an element of the story previously unknown to the
reader.
(d) Ovid elaborated on a point in Perseus’ tale to tell Medusa’s story.
23. Why did Perseus view Medusa’s head in his shield?
(a) Because Medusa’s looks could actually kill.
(b) Because her looks were enough to alarm a surprised foe with terror.
(c) Because he wanted Medusa to die by looking at her reflection.
(d) Because he didn't want to wake up the other sleeping Gorgons.
24. According to the passage, which of the following is incorrect?
(a) Polydectes announced a large banquet.
(b) In the banquet, Polydectes expected every guest to bring a gift.
(c) Medusa was the only mortal Gorgon at the time of the story.
(d) Gorgon requested the guests to bring horses.
25. Which of the following is the most likely source of the passage?
(a) A book on mythological tales. (b) A book on the history of Persian empire.
(c) A book on moral education. (d) A book on the basics of philosophy.

. Page 8 of 40
Passage (Q.26-Q.30): What’s happening in the Great Lakes during those long, frigid
months when they’re often
covered partially or completely with ice? A casual observer — and even experts —
might be inclined to say,
“Not much.” Lake scientists have long considered winter a season when aquatic
activity slows. Most do their
field studies at other times of the year. But researchers now think more is going
on in the bitter depths than
previously believed — including activity influenced by climate change.
“We’ve been ignoring winter on the Great Lakes for so long,” said Ted Ozersky, a
lake biologist with the
University of Minnesota Duluth, who announced the “Winter Grab” expedition
Thursday. “There are lots of
ways in which ice and winter conditions can affect the ecosystem. We don’t fully
understand all of them. We
have a general idea of how it should work but, in many cases, we haven’t done the
footwork to see,” Ozersky
said. Crews from more than a dozen U.S. and Canadian universities and government
agencies will make their
way onto frozen sections of lakes. That’s typically around the time of maximum ice
cover. They’ll take what
Ozersky described as a midwinter “snapshot,” measuring characteristics such as
light levels at different depths,
water movement and the presence of carbon, bacteria and nutrients that feed fish
but also can damage the
environment.
One reason for the growing interest in winter’s effects on the lakes is how winter
itself is changing. Great Lakes
ice cover has declined steadily since the 1970s and some projections indicate it
could become scarce later this
century. While that might boost the cargo shipping industry, the results for lake
ecology are unknown. Ice is “a
dramatic physical force,” Ozersky said, influencing everything from the exchange of
carbon dioxide between air
and water to light penetration and the water column’s thermal structure. Such
characteristics can determine how
much plankton is available for fish. Nearshore ice can protect fish eggs and
prevent crashing waves from eroding
shorelines.
“We know it’s important but because we haven’t studied it, there are many areas
where we don’t fully understand
the effect of having or not having ice,” Ozersky said. The University of Michigan’s
Cooperative Institute for
Great Lakes Research, a partnership with the National Atmospheric and Oceanic
Administration, is funding this
month’s excursion.
“Winter is the season that is altered the most with climate change,” said
Marguerite Xenopoulos. She will take
samples from Lake Ozersky, who proposed the quest and will be visiting Lake
Superior’s he helped establish a
research collaboration called the Great Lakes Winter Network. Its studies could
influence practical matters such
as upgrading shoreline infrastructure and planting winter cover crops to prevent
erosion, he said. “As we get this
information, we’ll hopefully have a better idea what the loss of winter will mean
and how to adjust management
practices to mitigate the harmful effects,” Ozersky said.
26. Which of the following reflects the central idea of the passage?
(a) Scientists have taken granted that very less activity takes place Great Lakes
in winters.
(b) Scientists race Great Lakes to gather winter data on ways in which ice and
winter conditions can affect the
ecosystem.
(c) Scientists question what happened in the Great Lakes last year.
(d) Scientist are concerned about they do not fully understand the effect of having
or not having ice.
27. What is the reason for the growing interest in winter’s effects on the lakes?
(a) Winter is altered most with climate change.
(b) Ice is a dramatic physical force.
(c) Winter affects the ecosystem in many ways.
(d) How winter itself is changing.
28. Which of the following is not true in the context of Ted Ozerky?
(a) Ozersky helped establish a research collaboration called the Great Lakes Winter
Community.
(b) Ozersky said that near-shore ice can protect fish eggs and prevent crashing
waves from eroding shorelines.
(c) Ozersky will be visiting Lake Superior’s Apostle Islands and Lake Michigan’s
Green Bay.
(d) Ice is a dramatic physical force.

. Page 9 of 40
29. What does the author mean by “midwinter snapshot”?
(a) Long lasting effect of winter. (b) Extensive effect of winter.
(c) A glimpse at midwinter. (d) An overview of midwinter.
30. What does the author mean by “we haven’t done the footwork to see”?
(a) We have not gone around to understand ways in which ice and winter conditions
can affect the ecosystem.
(b) We have worked much to understand ways in which ice and winter conditions can
affect the ecosystem.
(c) We have shown sincerity to understand ways in which ice and winter conditions
can affect the ecosystem.
(d) We have not shown much interest to understand ways in which ice and winter
conditions can affect the
ecosystem
ECTION - C: LEGAL REASONING
Passage (Q.66-Q.70): Abortion is a highly debateable around the globe due to its
socio-medical, religious as
well as constitutional aspects. There exists a continuous question on woman's
rights with regards to her will to
abort, as is the abortion rights really a free right to a woman or is it a
conditional right and also whether women's
right to abort or the foetus (unborn child's) rights to live should be focused
upon.
Issues related to abortion in India raises questions under Article 21 of the
Constitution as there is Right to privacy
granted as a fundamental right that takes under its purview immunity from outside
interference in matters that
are personal and intimate such as marriage, child-rearing and the use of
contraception and along with this the
Right to life with dignity is given which permits abortion for reasonable reasons
for the dignified life of the
woman and it is a woman's absolute right to live with human dignity.
The Indian penal code of 1860 under section 312-316 has made the process of induced
abortion punishable that
is causing to miscarriage which is not done in good faith for saving the life of
the woman is considered as a
crime. Under these sections miscarriages which are not done for the purpose of
saving the life of a woman is
punishable and if the "woman is quick with the child" i.e. when movements can be
felt by the mother in her
womb it is considered as culpable homicide.
Intent of legislature in formation of the medical termination of pregnancy act 1971
was aimed to legalize the
process the abortion in exceptional cases and the main purpose of the act or the
intent of legislation was to
provide the medical filed, doctors, medical staff, nurses 'a safeguard' from the
investigations under IPC as the
laws were very strict and many illegal and unsafe abortions were done.
(Source: https://articles.manupatra.com/article-details/The-Right-To-Abortion-
Understanding-the-Rights-by#using-Rules-and-Aids-of-Interpretation)
66. A petition was filed in the court for getting permission for a 20 week post
pregnancy as under the MTP act. The
unborn child was suffering from heart defect which was diagnosed by the medical
practitioner after 20 weeks of
the pregnancy and the condition was very serious at that when the child would be
born he would need to have
an operation of heart and a pacemaker which may could also harm him in other ways,
this petition was dismissed
on the grounds that the medical board appointed for an opinion said that the child
would not suffer any serious
disability in life and with the help of a heart surgery everything could be okay.
Determine if this is a violation
of Nikita constitutional rights provided under article 21.
(a) Yes, because the doctor's refusal to grant the abortion request directly
violates Nikita's right to live with
dignity under Article 21.
(b) No, because article 21 protects the right to life of the foetus.
(c) Yes, because the woman has the right to life with dignity, which allows for
abortion in some circumstances.
(d) No, as miscarriages done when movements can be felt by the mother in her womb
is considered as culpable
homicide.
67. "foetus's right to life" under Article 21 is alleged to be debatable issue in
the opinion of the author. Choose an
appropriate remark on this subject in accordance with the information provided in
the passage.
(a) There are numerous debates about the stand of law in this regard and various
judgements have given different
views.
(b) the article 21 of Constitution guarantees right to life with liberty and right
to privacy to a woman within the
right as a freedom to choose on her own whether she wants abort the child or not
under reasonable condition.
(c) The Indian Constitution allows abortion under the fundamental right under
Article 21 as right to life with
personal liberty but still there is various restriction, checks and balances placed
and there is a requirement to
meet different needs of the society.
(d) The constitution on one hand states that a woman should have complete freedom
towards her body and her
decisions but on the other hand the MTP Act restricts this right as a woman herself
is not allowed to decide
regarding her own will to abort or not.

. Page 17 of 40
68. Assertion: More than anything a person's right to abortion is a person's right
to their own body.
Reason: There is a recurring dilemma about a woman's rights in relation to her
desire to abort, if women's right
to abort or the foetus' right to life should be prioritised.
(a) Both A and R are true but R is not correct explanation of A.
(b) Both A and R are true and R is correct explanation of A.
(c) A is true but R is false.
(d) A is false but R is true.
69. Choose a sentence that perfectly balances the position of MTP, IPC, and article
21 on women and foetus’s right
to life:
I. Laws which criminalise all abortions, except those required to save a mother’s
life, were unconstitutional
and violated the right to privacy of a pregnant woman.
II. Right should be balanced against the right of the state’s legitimate interest
in protecting both the pregnant
woman’s health and the potentiality of human life at various stages of pregnancy
under the provision of
various laws.
III. Abortion under any circumstances, except danger to a pregnant woman’s life,
was illegal in India.
IV. The MTP Act overrides the IPC by allowing a woman to get an abortion within the
first 12 weeks of
pregnancy, provided a registered medical practitioner diagnoses grave danger to the
pregnant woman’s
physical and mental health.
V. It was observed that as a result of these stringent provisions of the law, the
health and lives of many women
seeking abortions were being compromised.
(a) I, II , III and V
(b) II
(c) III and V
(d) V
70. Choose a statement which correctly describes the position the women with
respect to right to abortion in the
present scenario:
(a) A woman can terminate her pregnancy, under certain circumstances in which a
pregnancy can be terminated
at any stage.
(b) It is clear that the objective of enacting the MTP Act was far from any
consideration of a woman’s right to
make reproductive choices.
(c) The right to have an abortion is a facet of the right to privacy guaranteed
under Article 21.
(d) Although women’s reproductive decisional autonomy has been recognised by the
Supreme Court, its full
realisation would only be possible if the provisions of the IPC making abortions a
criminal offence were to
be struck down, and a woman’s decision to terminate her pregnancy be left up to her
and her doctor.
Passage (Q.71-Q.75): Conspiracy means a combination of two or more person for
unlawful purpose. The rules
relating to offence of criminal conspiracy under Indian Penal Code, 1860 punishes
several persons who are
directly or indirectly engaged in commission of any illegal act or omission.
The definition of criminal conspiracy under Indian Penal Code, 1860 incorporates
three main ingredients of the
offence which are:
One of the main requirements for constituting the offence of criminal conspiracy is
involvement of two or more
person who will subsequently agree for the commission of any offence.
The second requirement for constitution the offence of criminal conspiracy is an
agreement between the parties.
There should be consensus ad idem (meeting of minds) between the parties as an
agreement cannot exist unless
and until there is meeting of minds between the parties. The word agreement itself
includes the word 'agree'
which stipulates that the parties to the agreement should agree for the same thing,
if it not so then no agreement
would exist between the parties.

. Page 18 of 40
The last but not the least requirement for constituting the offence of criminal
conspiracy is commission of an
'illegal act'. The most important ingredient for constituting an offence under
criminal law is the presence of mens
rea, any act would not be considered illegal unless and until it incorporates men
rea.
According to Black law dictionary Criminal conspiracy is defined as: The
combination or confederacy between
two or more persons formed for the purpose of committing, by their joint efforts,
some unlawful or criminal act,
or some act which is innocent in itself, but becomes unlawful when done by the
concerted action of the
conspirators, or for the purpose of using criminal or unlawful means to the
commission of an act not in itself
unlawful.
Generally the charge of some other substantive offences is also included along with
the offence of criminal
conspiracy against the accused.
(Source: https://articles.manupatra.com/article-details/The-offence-of-criminal-
conspiracy-under-Indian#criminal-Law)
71. Which of the following statements is correct in respect to information provided
in the passage?
(a) Mere agreement to commit an offence is punishable.
(b) Agreement to commit civil wrong or any act prohibited by law were not
punishable.
(c) This section will apply in offence where either the ultimate object or means of
committing the act or omission
is illegal.
(d) No agreement except agreement for commission of an offence shall amount to a
criminal conspiracy unless
parties have done some act besides the agreement.
72. Assertion: I.P.C. defines criminal conspiracy as an agreement of two or more
persons to do or cause to be done
an illegal act.
Reason: A person may not be able to commit the offence himself alone but if he
enters into an agreement with
other, to commit that Offence.
(a) Both A and R are true but R is not correct explanation of A.
(b) Both A and R are true and R is correct explanation of A.
(c) A is true but R is false.
(d) A is false but R is true.
73. Chetna visited the Jageshwari Temple in Chanderi Dist. Ashoknagar. On the holy
day of Navami, she witnessed
several beggars receiving massive quantities of food from visitors. When she
witnessed the food being
completely squandered and the beggars having sufficient food for the day, she
advised others not to provide any
food to them today, but instead to distribute it to other beggars who sat a mile
away from the temple. All of the
beggars present in the temple perished of hunger since no one fed them following
Chetna's proposal. Will this
be construed as an incident of criminal conspiracy?
(a) Yes, Chetna, with the support of other visitors, persuaded the throng not to
feed the beggars.
(b) No, as Chetna did not plot with anyone, thus it cannot be concluded as an
incidence of criminal conspiracy.
(c) No, but if Chetna and the mob had intended to kill the beggar by not giving
them with food, then it would
be a case of criminal conspiracy because all of the ingredients had been satisfied.
(d) Yes, because the beggars went hungry when no one fed them after Chetna's
proposal.
74. There were two ration in the town of Patna -- one standing on the name of
respondent Kailash Prasad Singh and
the other in the name of respondent Shyam Sunder Singh. Respondent Adya Prasad was
the Munshi of the first
shop while respondent Ragho Prasad was the Munshi of the later shop. Respondent
Ambika Prasad was a helper
of Adya Prasad. Occasionally he worked for the other shop also. The case against
the respondents was that
between October 1992 and February 1994 they conspired to cheat the Ration Office at
Patna by presenting forged
challans and obtaining ration from authorities on the basis of those forged
challans and thereafter taking delivery
of grains from the Government godowns. The accused Gourav was the license holder of
a shop and there were
incidence that he used to visit the offices. Will Gourav be liable for the offence
of criminal conspiracy?
(a) Yes, as he was aware of the passing challans and must have benefited from the
scam.

. Page 19 of 40
(b) No, because he had no idea his servents were receiving rations from government
godowns by submitting
false challans.
(c) All parties were in consensus ad idem while engaging in an illegal act.
(d) Everyone, with the exception of the accused, consented while engaging in an
illegal act of getting rations
from government godowns by presenting false challans.
75. Choose a statement which presents an oddity with passage’s text:
I. In a criminal conspiracy, meeting of minds of two or more persons for doing an
illegal act is the sine qua
non but proving this by direct proof is not possible.
II. Conspiracy being a continuing offence continues to subsist till it is executed
or rescinded or frustrated by
the choice of necessity.
III. It is not necessary that all conspirators should agree to the common purpose
at the same time.
IV. A man may join a conspiracy by word or by deed.
(a) I and II
(b) I and IV
(c) All of the above
(d) II
Passage (Q.76-Q.80): The argument favoring death penalty is often founded on the
deterrent theory of
punishment. This theory assumes that the motive behind awarding punishment is to
deter other members of the
community from committing the same crime. Since people fear death the most, death
penalty serves as the most
effective deterrence among people. However this theory has been severely criticized
by several criminologists
jurist.
Justification for the continuance of death penalty is the retribution theory of
punishment. This theory is founded
on the principle of lex talionis and argues that a just punishment is one which
inflicts the same amount of pain
to the offender as he caused to the victim. The retentionists argue that since
death penalty is prescribed in those
cases where the accused has caused the death of another person it is very much
reasonable and justified that law
takes the life of the offender to give retribution to the family of the deceased.
However, the abolitionists expose
the absurdity in this hypothesis by arguing that modern punishment is not founded
on the principle of retribution
because if this was the case then no punishment can be given to rapists, thieves
and whole lot of other criminals.
Instead they argue that reformation and rehabilitation is the primary purpose of
punishment, which is defeated if
death penalty is awarded to an offender.
The Supreme Court while holding that Bachan Singh case was the law on the subject
listed the following
guidelines emerging from that case : Firstly, death penalty should be granted only
in rarest of the rare case.
Secondly, while sentencing the accused the circumstances of the crime and that of
the accused both are relevant
and neither of them could be left out from consideration. Thirdly, the fundamental
guiding principle is that death
sentence is exception to the general principle of awarding life imprisonment and
hence it should be used only in
sporadic cases where the facts and circumstances warrants nothing but the death of
the accused and any other
punishment would be wholly inadequate. Fourthly, before awarding death sentence the
sentencing Court must
draw a ‘balance sheet of all the aggravating and mitigating circumstances.’ After
giving weightage to the
mitigating circumstances and if the balance is still tilted in favor of the
aggravating circumstances only then
death penalty should be awarded.
The Ravji decision was delivered by the Supreme Court in 1996 and was detected 13
years later in 2009 by
another bench of same Court in Santosh Kumar Satishbhushan Bariyar v. State of
Maharashtra, the bench besides
lamenting over the fact that a per incuriam judgment was being incorrectly followed
with authority by other
Courts did not attempt to undo the wrong committed and perhaps save those
individuals who due to reliance on
an incorrect precedent were sentenced to be hanged by neck till death.
(SOURCE: https://articles.manupatra.com/article-details/The-Meandering-Course-Of-
Death-Penalty#Sentencing-In-India-A-Critical-Analysis)

. Page 20 of 40
76. What can be asserted true about the deterrent and retribution theory discussed
in the passage?
I. Both deterrence and retribution have become individual theories whose thoughts
are constantly being
discussed of either being good or bad.
II. By imposing a method or means of deterrence, the offender will be cautioned not
to do another offence of
similar or related nature (again) or else he or she will have to experience the
same punishment given to him
or her before.
III. Retribution is getting even with the felon while deterrence is doing something
to the felon so as to deter him
and other would-be felons from doing the same wrong doing
IV. Retribution is just like saying the famous quote ‘an eye for an eye, a tooth
for a tooth’. Whereas, at a glance,
deterrence seems to be a very harsh principle.
(a) All of the above
(b) II
(c) I, II and III
(d) I and III
77. Anupama- a married lady, who had a child of around 10 years at the time of
commission of crime, killed another
married lady and her 2 year old child because she suspected that the deceased (her
husband) was having an extra
marital affair with another widower for whom she had deep seated affection and also
an extra marital affair. The
Trial Court and the High Court had both awarded death sentence to the accused.
Choose a statement which
provides for a court order that is based on both punishment theories: retributive
of the accused and deterrent of
the conduct committed.
(a) While deciding the appeal of the accused, the Supreme Court had observed that
while deciding the sentence
of an accused the focus should not only be on the crime committed but also on the
criminal.
(b) While deciding the case crime and criminal are treated as equally material when
the right sentence has to be
picked out.
(c) After duly considering not just the severity of the crime committed but also
the overall social and other
circumstances of the criminal, the Court gave due regard to the fact that the
accused was a female, a young
age, hence, can be imposed a deterrent punishment.
(d) Cannot be ascertained as the acts lack complete information.
78. The accused and one Bhole were involved in a long standing family feud. The
accused in heat of passion
murdered one of the family members of Bhole for which he was sentenced to life
imprisonment. However, after
being released from the jail on the occasion of Gandhi Jayanti he gruesomely
murdered Bhole and his friend
Jethalal. The Trial Court awarded death sentence which was duly confirmed by the
High Court. The Supreme
Court, while, reversing the sentence was of the opinion that death sentence shall
not be awarded. What, in your
opinion, is the basis for the Supreme Court's disagreement with trial and High
court orders?
(a) Capital punishment is justified in only those cases where public interest,
social defence and public order
warrants.
(b) The circumstance does not pass the test laid down to decide whether to impose
death sentence or life
imprisonment.
(c) The case of the accused does not fall under the category of rarest of the rare
case.
(d) Death sentence is warranted when due to the crime committed by the accused, the
normal life in the
community is paralyzed and there is absolute destruction of the social order.

. Page 21 of 40
79. Shyam murdered his wife and three minor children and attempted to murder his
mother and neighbors’ wife. He
was convicted and sentenced to death by the two Lower Courts. While considering the
case of the accused, the
Court erroneously believed that in a criminal trial while sentencing, ‘It is the
nature and gravity of the crime and
not the criminal.’ which must be considered. Hence, without even justifying that
the case falls within rarest of
the rare category the Supreme Court confirmed the death sentence. Decide the
validity of the judgment.
(a) Since the court did not follow the instructions issued by the Supreme Court in
Bachhan Singh on the
imposition of the death penalty, this is a per incurium decision.
(b) The court acted negligently by only considering aggravating circumstances and
failing to draw a balance
sheet of all aggravating and mitigating circumstances.
(c) The court's conclusion is correct since the circumstance meets requirements of
the rarest of the rare doctrine.
(d) The court's conclusion is wrong because the facts do not meet the guidelines
presented in bacchan singh’s
case.
80. The passage in last para refers to “per incurium judgment”. What according to
your understanding of the passage,
does the phrase mean?
(a) Judgemnet through which the court ordered death penalty on the accused.
(b) A judgemnet which set the wrong precedent in line.
(c) Refers to a judgment of a court which acted carelessness and has been decided
without reference to a statutory
provision.
(d) A decision of the Court which is not mistaken. A decision of the Court is
Binding Precedent if given Per
Incuriam.
Passage (Q.81-Q.85): Territorial Jurisdiction is very important for a trial. An
inappropriate choice of territorial
jurisdiction can cause administrative inconvenience such as problems in the
collection of evidence. Jurisdiction
becomes also important from the perspective of the various stakeholders for
criminal justice administration.
Witnesses, victim and accused can face problems in travelling across jurisdictions.
Generally, in common law
countries, an offender is tried in local limits in which the offence has been
committed. The same has been used
in section 177 of Code of Criminal Procedure, 1973 (CrPC). Under section 406 of
Code of Criminal procedure,
Supreme Court is exception to the general principles. As per the section, the SC
can transfer case from one court
to another court if it feels it is expedient for the ends of the justice. The main
requirement of the section is that
court should have reasonable apprehension as non-transfer of trial would lead to
injustice. The broad categories
for which Supreme Court have granted transfer of the case under section 406 were
given by Supreme Court in
Nahar Singh case are:
1. When court feels that the state machinery or the prosecution is acting with
accused and the same would result
into miscarriage of justice due to lack of care by the prosecution.
2. The material of the case shows that the accused may influence the witnesses and
may cause physical harm
to complainant.
3. Inconvenience and hardships could be caused to the complainant/the prosecution,
accused, and the witnesses.
The economic burden which would be borne by State Exchequer in reimbursing
travelling and other expenses
of the official and non-official witnesses.
4. The atmosphere of the place is communally surcharged which indicates that the
state and prosecution would
not be able to holding fair and impartial trial for the reason of the accusations
made and also such nature of
crime to the accused.
5. There is an existence of some kind of material which leads to inference that the
witnesses and other persons
are so hostile that they could interfere with the course of justice.
(Source: https://www.barandbench.com/columns/investigation-territorial-
jurisdiction-transfer-under-section#406-crpc-some-stray-thoughts)

. Page 22 of 40
81. Amit is convicted of an offence for assassinating two sitting judges of High
Court of Patna. Amit thinks that he
would not be granted adequate justice in High Court of Patna as both the deceased
judges were known to the
current sitting judges in the High Court. Decide whether the Supreme Court will
order the transfer to other court?
(a) The transfer will not be ordered as facts do not warrant to order the transfer
of the case.
(b) The transfer of case will be ordered as the judges might have bias for the
accused.
(c) The transfer of cases will not be ordered as the judges are allowed to judge
irrespective of any conflict.
(d) The transfer of the case will not be ordered as the witnesses might turn
hostile.
82. Ram Sikka, Chief Minister of the State of Uttar Pradesh, was accused for the
charges of corruption under the
Prevention of Corruption Act. During the trial proceedings, 70 witnesses turned
hostile and some of the witnesses
were grievously hurt by a faction of people. As a counsel for CBI, which of the
following statement would
strengthen your case to transfer the case to another jurisdiction?
(a) The witnesses that were grievously hurt were offered police protection but
denied.
(b) The Chief Minister, who controls the police, has not resigned from the post of
Chief Minister.
(c) The administrative costs for transferring the case to nearest High Court will
cost 10 crores to exchequer
(d) The government in Union Legislature is not from same party as the Chief
Minister.
83. The accused in a double murder case was very influential person. He threatened
witness and family of victims
and the petitioner several time complained to police. In addition to that, many
witnesses turned hostile, and
witnesses did not respond to summons due to fear of accused. One of the victims
approached Supreme Court to
transfer the case. As counsel of victim, which statements will weaken your case?
(a) The accused is son-in-law of Deputy Chief Minister of the State.
(b) The state has set up a special court for deciding similar cases expeditiously.
(c) The witnesses that turned hostile were denied police protection when asked.
(d) The prosecutor has been given strong powers to order the police with regard to
the investigation. The same
is being done by him vehemently.
84. The accused was former Home Minister of State and was out on bail in relation
to Sohrabuddin abduction case
from two years. CBI appealed for annulment of bail and to transfer the case on the
grounds that firstly, non#cooperative state police; secondly, a witness of
Sohrabuddin abduction, Kanhaiya, was also killed in another
fake encounter by Gujarat Police and, on the other hand, state authorities tried to
conceal the relation of the full
fact of two encounters with public prosecutor. Decide whether the Supreme Court
will order the transfer to other
court.
(a) The transfer will not be ordered as facts do not warrant to order the transfer
of the case.
(b) The transfer of case will be ordered as the state machinery that the
prosecution is acting with accused.
(c) The transfer of cases will not be ordered.
(d) The transfer of the case will not be ordered as the witnesses might turn
hostile.
85. The accused was facing charges regarding Coimbatore serial bomb blasts case.
The accused pleaded to transfer
the case from Coimbatore to Chennai. It was alleged that he is a member of Pakistan
intelligence, and was
responsible for bomb blast, and that there was communal disturbance in the area,
and also that no lawyer is ready
to defend the accused. The respondents on the other hand stated that lawyers have
already appeared for accused
at various courts. Which one of the following factors will favor the case of
accused?
(a) The communal disturbance is surcharged in state.
(b) Most of the witnesses are from Tamil Nadu.
(c) The State of Tamil Nadu has ensured free, fair and impartial trial.
(d) Prison in Coimbatore has less prisoners than authorized accommodation while
Chennai are overcrowded.
. Page 23 of 40
Passage (Q.86-Q.90): It is the contract and tort law including case laws emanating
from them which bear the
credit of evolving the Product Liability jurisprudence in India. The origins of the
Product Liability principles
can be traced back to the common law concept of "caveat venditor" which implies
"let the seller beware". It in
turn places the burden on the seller to be responsible and liable for any problem
that the consumer might
encounter with the service or a product. Where under contract law the principle of
warranty governs the consumer
regime, on the other hand under tort law the principles of negligence and strict
liability are responsible for the
development of jurisprudence.
The law of torts is uncodified in India. Thus, the courts are guided by the
principles of justice, equity, and
conscience in addition to the established tort law principles of negligence, strict
liability, and duty of care.
Negligence is referred as lack of observance of reasonable or due care as a result
of which an injury or loss is
suffered. Thus, the three essential elements required to be proved in any claim for
negligence are -
(a) The existence of a duty of care
(b) A breach of such duty of care, and
(c) Damage or injury resulting from such breach
The principle of Strict Liability operates when the products are unreasonably
defective and dangerous that the
seller would be made liable for any proprietary loss or personal injury.
Consequently, the damages awarded under tort product liability claims can be
exemplary or punitive in addition
to compensatory as the objective of awarding such damages is to punish the
breaching party for its lack of care
and non-observance of relevant conduct.
(Source: https://articles.manupatra.com/article-details/Product-Liability-Laws-in-
India)
86. vaishali was sipping his favorite soft drink in canteen when simran interruptd
her by saying, what you are
drinking, is as good as the cleaner at home. Shocked by the facts, vaishali dug a
little deeper and took a look at
the ingredients in the most popular soft drinks available in the market. The key
ingredients were sugar, caffeine
and phosphoric acid, which give them their tangy flavour and acidity. She also
found out that Phosphoric acid,
a colourless, odourless crystalline liquid, is actually commonly found in toilet
cleaners. Vaishali filed a suit
against Cola Coco. Determine her odds of winning in the negligence complaint filed
with the Consumer
Commission?
(a) The suit filed against the soft drink company will be successful as it included
a dangerous ingredient which
might directly affect the health of the consumer.
(b) The suit filed by vaishali will be unsuccessful against Cola Coco.
(c) The suit filed against cola Coco will be succeed as the seller not only is
negligent but also strictly liable
under the tort law of product liability jurisprudence.
(d) The suit filed will succeed as Strict Liability operates when the products are
unreasonably dangerous.
87. Chaitanya, 18 years old brought a lawsuit against malboro, manufacturer of
premium cigarettes. He alleged that
cigarette companies manufactured and marketed products that the companies knew to
be defective. Choose an
argument in favour of the defendant that will allow them to avoid culpability.
(a) The defendants contended that since the plaintiffs were aware of the inherent
health dangers of cigarette
smoking, there could be no guilt because no obligation arose with respect to the
plaintiff here.
(b) Just because a product is risky does not make it defective.
(c) Cigarettes are inherently dangerous, but not defective.
(d) All of their ingredients have been disclosed to the government the federal
government has never requested
removal of any ingredient in Philip Morris cigarettes.

. Page 24 of 40
88. Choose a statement which is correct in light of the passage given:
(a) If an aggrieved consumer suffers an injury or loss owing to a breach of
contractual warranty, damages can
be awarded.
(b) A breach of a warranty under contract Law can give rise to a claim for damages
in a product liability action.
(c) Warranty is defined as stipulation collateral to the main purpose of the
contract, the breach of which gives
rise to a claim for damages but not to a right to reject the goods or services and
consider the contract as
repudiated.
(d) The term 'Product Liability' implies the responsibility of seller of goods to
compensate for injury caused by
defective goods that it has provided for sale.
89. The passage mentions the concept of caveat venditor. What does it mean?
(a) It is for the buyer to satisfy himself that the goods which he is purchasing
are of the quality which he requires
or if he is buying them for a specific purpose, that they are fit for that purpose.
(b) The maxim fosters consumer welfare by making the seller, manufacturer, and
service providers accountable
for the quality of goods produced or services offered.
(c) It is for the seller to satisfy the consumer that the goods which he is
purchasing are of the quality which he
requires or if he is buying them for a specific purpose, that they are fit for that
purpose.
(d) The onus is on the sellers to ensure the quality of product and if not done so
will be liable for the same.
90. A children’s toy called “Aqua Dots” was found to have a coating that could
react by producing a dangerous drug
if swallowed by a child. Moose Enterprise developed the toy and for the work of the
Chinese lab it hired to
manufacture the toy. The product was the subject of a recall in 2020 amid reports
that nine children in the
states and three in India became sick after swallowing the beads. Tests showed
products sold by aqua dots
Pvt. Ltd. were coated with a chemical that, when ingested, metabolizes into gamma
hydroxybutyrate (GHB),
more commonly known as the "date-rape" drug. The compound can induce breathing
problems, nausea,
vomiting, unconsciousness, coma and death. Who will be held accountable in this
case under the caveat
venditor doctrine?
(a) The seller will be held liable along with the manufacturer as he sold the final
product to the seller.
(b) Only the seller will be held liable as the doctrine restrains the seller from
taking advantage of the weaker
position of the consumer.
(c) Cannot ascertain as the facts lack appropriate information.
(d) Seller along with Moose Enterprise and Chinese lab who was hired to manufacture
the toy will be held liable.
Passage (Q.91-Q.95): The Supreme Court observed that a suit for permanent
injunction is not maintainable
against the true owner of the property when the dispute with respect to title is
settled against the plaintiff.
Once the suit is held to be barred by limitation qua the declaratory relief, the
prayer for permanent injunction,
which is a consequential relief can also be said to be barred by limitation, the
bench comprising Justices MR
Shah and BV Nagarathna observed:
However, the trial court granted the relief of permanent injunction. The First
Appellate Court dismissed
defendant's appeal. Dismissing the Second appeal, the High Court held that the
relief of permanent injunction
sought by the original plaintiff can be said to be substantive relief and not a
consequential relief and therefore,
the trial court was justified in granting the permanent injunction in favour of the
plaintiff.
The court noted that the main reliefs sought by the plaintiff in the suit were
cancellation of the sale deed and
declaration and thus the prayer of permanent injunction can be said to be a
consequential relief.
"Therefore, the title to the property was the basis of the relief of possession. If
that be so, in the present case, the
relief for permanent injunction can be said to be a consequential relief and not a
substantive relief as observed
and held by the High Court. "
Therefore, once the suit is held to be barred by limitation qua the declaratory
relief and when the relief for
permanent injunction was a consequential relief, the prayer for permanent
injunction, which was a consequential
relief can also be said to be barred by limitation. It is true that under normal
circumstances, the relief of permanent

. Page 25 of 40
injunction sought is a substantive relief and the period of limitation would
commence from the date on which
the possession is sought to be disturbed so long as the interference in possession
continuous. However, in the
case of a consequential relief, when the substantive relief of declaration is held
to be barred by limitation, the
said principle shall not be applicable.
(Source: Relief Of Permanent Injunction Against True Owner Cannot Be Granted When
Title Dispute Is Settled
Against Plaintiff : Supreme Court,Livelaw)
91. Dhanur filed a suit against the title of the property which is in possession of
Dinesh, his tenant. Dispute is settled
against the plaintiff. Can dhanur file a separate suit for injunction?
(a) No dhanur cannot file a separate suit as it is barred by limitation.
(b) Yes, dhanur can file a separate suit as it is not barred by limitation.
(c) No , dhanur cannot file a separate suit as the principle of res judicata will
be applicable.
(d) None of the above.
92. Assume, in the aforementioned set of facts, the title dispute is resolved in
favour of the plaintiff. What will the
status of consequential relief be as provided in the passage?
(a) Relief for permanent injunction will be barred by limitation.
(b) Since plaintiff is the owner, suit for permanent injunction is not
maintainable.
(c) It is held to be barred qua declaration relief.
(d) As it was a substantive relief, it will not be disallowed.
93. Select an alternative that do not illustrates an anomaly of the passage's
context:
(a) If the title to the property was not the basis of the relief of possession, the
relief for permanent injunction can
be said to be a consequential relief.
(b) Once the suit is held to be barred by limitation, and when the relief for
permanent injunction was a
consequential relief, the prayer for permanent injunction, which was a
consequential relief cannot be said to
be barred by limitation.
(c) Once the dispute with respect to title is settled and, the suit by the
plaintiff for permanent injunction shall
maintainable against the true owner.
(d) In the instant case, the appellant succeeded in the legal dispute, and it was
determined that the defendant
lacked standing to seek permanent injunction relief since it was not a substantial
relief, and thus was barred
by limitation.
94. According to the paragraph, how should the period of limitation be determined
in a declaration claim where the
relief of injunction is consequential in nature?
(a) It does not begin on the date the possession is sought to be disturbed.
(b) It persists indefinitely as long as the interference in possession continues.
(c) It commence from the date on which the possession is sought to be disturbed so
long as the interference in
possession continuous.
(d) Cannot be determined.
95. Assertion: In a suit of permanent injunction along with declaration, the relief
for permanent injunction can be
said to be a substantive relief and not a consequential relief.
Reason: Consequential relief is something which is followed by substantial relief.
(a) Both A and R are true but R is not correct explanation of A.
(b) Both A and R are true and R is correct explanation of A.
(c) A is true but R is false.
(d) A is false but R is true.

. Page 26 of 40
Passage (Q.96-Q.100): The Kerala High Court has recently ruled that if there exists
suspicious circumstances
surrounding the execution of the will, it is the onus of the propounder to remove
all those reasonable doubts in
the matter and the test to be applied in this connection is satisfaction of
judicial conscience before the grant of
probate. Once probate is granted for will it shall be executed without any further
delay.
A Division Bench of Justice P.B. Suresh Kumar and Justice C.S Sudha observed that
suspicion cannot be
removed by the mere assertion of the propounder that the will bears the testator's
signature or that the testator
was in a sound state of mind at the time when the will was made or that the wife
and children of the testator who
would normally receive their due share in his estate were disinherited because the
testator might have had his
own reasons for excluding them.
"The presence of suspicious circumstances would therefore make the initial onus of
the propounder of the will
heavier and in cases where the circumstances attended upon the execution of the
will is shrouded by suspicions,
the propounder must remove all those suspicions which are legitimate before the
document can be accepted as
the last will of the testator. In the event of executor’s absence beneficiary will
be propounder"
In the event of the death of the executor of a will, the maxim actio personalis
moritur cum persona does not apply
to probate proceedings initiated by the executor before his death. An executor in
applying for probate is not
fighting a personal action but fighting for the interests of all the beneficiaries
under the will and that therefore
the action of an executor in applying for a probate is not in substance a personal
action. If the executor fails in
his duty, any of those whom he represents are entitled to intervene and carry on
the proceedings with a formal
modification’ that the prayer must then be for letters of administration with the
will annexed.
(Source: Proof Of Will : Onus Is On The Propounder To Remove Suspicious
Circumstances : Kerala High Court,
livelaw)
96. Mr Lokhandawala has made a will in favour of his three sons, excluding his only
daughter Chandini. Mr
Lokhandwala was a successful businessman. Chandini on death of his father learned
about this, when all of his
children were told about his will, that she had no share in her father's property.
Chandini claims suspicion in the
will, claiming that how can such a wealthy man leave nothing for his daughter?
Determine who bears the burden
of removing all reasonable doubts in the matter.
(a) Chandini as she contends the suspicion in the will.
(b) The three son’s who seeks to execute the will being correct
(c) Mr Lokhandawala , who is the testator of the will.
(d) The onus will be on the propounder represents in probate proceeding.
97. In the same situation as above, if executor is seeking for validity of will but
during the probate proceeding he
died. In such situation who will be responsible for clearing the reasonable
suspicion in the will in the event of
the executor's death?
(a) Propounder’s representative i.e., his heir who will succeed him.
(b) The person or entity that the propounder represents in probate proceedings.
(c) Legal heirs of the testator i.e. his son
(d) Cannot be determined
98. Passage provides that, In the event of the death of the testator, the maxim
actio personalis moritur cum persona
does not apply to probate proceedings initiated by the executor before his death.
What does the expression “actio
personalis moritur cum persona” mean ?
(a) A personal right of action do not die with the person.
(b) A personal right of action dies with the person.
(c) In case of will, a personal right of action does not die with the person.
(d) a future gift given in expectation of the donor’s imminent death and only
delivered upon the donor’s death.

. Page 27 of 40
99. Jethalal, a businessman, wrote a will in which he stipulated that all of his
property, businesses, and basically all
movable and immovable assets bequeathed to an ashram called Ganga Ashram. Ganga
Ashram initiated the
probate proceedings because a probate is required if the Will includes immovable
assets. Legal heirs of Jethalal
claim suscpision in his will as to why he will not include his family members in
the inheritance of Jethalal's
property. Choose an argument that would be most compelling in favour of Ganga
Ashram.
(a) Jethalal made the will in sound mind.
(b) Jethalal executed the will in presence of the witnesses.
(c) Verifying the signature of jethalal on the instrument.
(d) Upon satisfaction of judicial conscience.
100. Assume, in the same circumstances as stated above, the court granted probate
for Jethalal's will because no one
objected to the will. Can family members raise suspicions about the will after it
has been granted probate?
(a) Yes, because whenever there are suspicious circumstances surrounding the
execution of the will, it is the
proponent's responsibility to dispel all reasonable doubts.
(b) No, because no objections can be raised after the grant of probate.
(c) Yes, because the proponent's assertion cannot dispel suspicion.
(d) No, because the propounder must remove all legitimate doubts before the
document can be accepted as the
testator's last will and testament.
Passage (Q.101-Q.105):The Dharwad Bench of the Karnataka High Court recently
directed Superintendent of
Police of Uttara Kannad district to enquire into and take suitable action against a
police inspector for failing to
register a first information report (FIR) in a cognisable offence [Ganesh S Hegde
v. State of Karnataka and
Ors].
The Court framed the following issue for consideration:
“Whether on information being received, either on the helpline or directly to a
police station, the concerned
Officer can carry out an enquiry, requiring the complainant to attend an enquiry
before registering of a
complaint?”
Justice Suraj Govindaraj noted that the conduct of the said officer was in
violation of the directions issued by
the Supreme Court in Lalita Kumari.
"Whenever any information is received disclosing a commission of a cognizable
offence either on phone or in
written form, there is no preliminary enquiry which is permissible and FIR is
required to be registered by the
person receiving information. It is only when the information received does not
disclose a cognizable offence, a
preliminary enquiry could be conducted to ascertain if there is a cognizable
offence committed or not. If the
enquiry discloses the commission of a cognizable offence an FIR must be
registered," the Court said.
The Court added that in such cases, the top court had made clear that action must
be taken against the erring
officials who do not register an FIR on request of person whose request is denied
by officer.
A preliminary enquiry may be made in matrimonial disputes/family disputes,
commercial offences, medical
negligence cases, corruption cases, or cases in which there is significant delay in
initiating criminal prosecution,
such as over 3 months' delay without a satisfactory explanation, it was stated. The
Court noted that once
information which discloses a cognizable offence is provided by any citizen to the
police helpline or to the police
station, an FIR ought to be registered by the police.
Trespass into the property of the petitioner and removal of areka nut crop is a
cognizable offence under Sections
441 (criminal trespass) and 427 (mischief causing damage to the amount of fifty
rupees) of the Indian Penal
Code.
(Source : Take action against Police Inspector for not filing FIR in cognisable
offence: Karnataka High Court,
Bar and Bench)

. Page 28 of 40
101. Nandu hired a security guard to guard his property, which is still under
construction, in order to keep the
construction materials safe while the boundary wall is being constructed. Twenty
thugs arrived and robbed the
entire construction material. When the security guard attempted to call the
helpline, the officer instructed him to
file a FIR offline. Can Nandu file a complaint against the officer for failing to
respond to his call to the helpline
number?
(a) No, because Nandu did not contact the helpline.
(b) Yes, because the police did not take any action in response to the call made to
the helpline number.
(c) No, but the security guard has the right to file a complaint against the
officer in charge.
(d) Yes, because the police officer failed to act on a complaint for a cognizable
offence in any case.
102. Continuing with the similar circumstances as above, under what grounds can a
complaint be filed against the
police officer?
(a) For not registering an FIR immediately of the offence informed to the police.
(b) No complaint can be made as the information received does not disclose a
cognizable offence.
(c) For not conducting a preliminary inquiry, so as to ascertain if there is a
cognizable offence committed or not.
(d) For quoting the informant to register an offline FIR instead.
103. Dhanush and Sakshi married in 2006, but they seem too separated. This was the
result of Dhanush being very
impatient with Sakshi at the beginning of their marriage, but Sakshi remained
silent because she didn't want to
ruin things. Sakshi filed a domestic violence complaint against her husband after
ten years, a cognizable offence,
via email to Mumbai police, who are known to be very proactive in taking action.
The police did not take any
action even after sakshi made a personal call to the nearby police station. Decide
(a) A complaint can be registered against the officer in charge for not filing an
FIR.
(b) A complaint can be registered against the police officer for not making an
initial enquiry.
(c) A complaint cannot be filed against the officer in charge as it was made 10
years after the actual incident
happened.
(d) A complaint against the police officer can be made for not filing an FIR made
in respect of a cognizable
offence.
104. Chaman suffers from thanatophobia. He isterrified of being killed whenever
someone comes close to him. When
he was crossing a silent lane and saw a stranger approaching with a knife, he
immediately phoned the helpline
number and reported the threat to police. We'll be there in 20 minutes as place is
little far, according to the cops.
Chaman, fearful, called the DSP to complain about the officer's failure to arrive
on the scene immediately. Make
a decision?
(a) Chaman's complaint is valid because the officer should have arrived as soon as
possible.
(b) Chaman's complaint is invalid because he suffers from thanatophobia.
(c) Chaman's complaint is valid because he reported a cognizable offence.
(d) Chaman's complaint is invalid because no crime occurred and there was no
reluctance on the part of the
police to come to the scene.
105. Assertion: Cognizable offence means an offence for which police cannot make an
arrest without warrant; they
are usually of serious nature.
Reason: Anyone can report the commission of a cognizable offence either orally or
in writing to the police.
(a) Both A and R are false and R is not correct explanation of A.
(b) Both A and R are true and R is correct explanation of A.
(c) A is true but R is false.
(d) A is false but R is true.

. Page 29 of 40
SECTION - D: LOGICAL REASONING
Passage (Q.106-Q.110): Read the passage below and answer the questions that follow:
Sadly, there is no respite for the people of Malwa from the heavy toll that the
highly contaminated water in the
area is taking on their health, children’s growth, soil fertility and food
production. The abuse of the soil, rivers
and canals with chemical-laden fertilisers and pesticides, as also poisonous
industrial effluents seeping into the
natural resources, are playing havoc with the flora and fauna. According to various
studies, including the one
commissioned by the Centre’s Department of Science and Technology, there are
dangerous quantities of
fluorides, uranium, aluminium, lead and magnesium in as high as 80 per cent of the
groundwater, putting the
inhabitants at risk of carcinogenic diseases.
Despite desperate premonitions of impending disasters if the groundwater situation
is not urgently checked, the
Malwa region of Punjab seems to be headed for worse times. Numerous surveys in the
past couple of decades
have unanimously portended a calamitous scenario. The alarmingly depleting levels
of the water table and the
rising contamination of the water bodies with heavy metals have necessitated policy
overhauls.
Various high-level water commissions and authorities have been set up by successive
governments with the aim
of not only arresting the degradation of this precious resource but also restoring
it to its pristine condition.
However, every new study on the issue by academicians and other experts reveals
little improvement on the
ground. Clearly, most remedial steps have been knee-jerk reactions rather than fool
proof long-term solutions.
Similarly pitiful is the state of some sewage treatment plants. Even as officials
and politicians go around making
tall claims, the suffering residents helplessly see their near and dear ones
withering away in the toxic ecosystem.
A special Vidhan Sabha committee had recently suggested that a policy on
groundwater extraction and recharge
be framed and water conserved by agri-zoning and metering groundwater. Time for
effective action is running
out, for the doomsday of desertification is approaching fast.
106. Which among the following is true about the second paragraph of the passage?
(a) The paragraph introduces the readers to the main concern of the passage.
(b) Some suggestions have been made which are refuted in the subsequent paragraph.
(c) The paragraph contains some steps taken by the authorities to take note of the
problem.
(d) The steps taken by the authorities to solve the problem have been refuted by
the author.
107. If the information given in the passage is true, which of the following
statements finds support in the passage?
(a) The groundwater of the Malwa region is contaminated mostly due to natural
reasons.
(b) The authorities have shied away from solving the groundwater problem properly
and resorted to quick fix
solutions.
(c) Installation of RO plants in areas where the groundwater is contaminated is
enough to solve the given issue.
(d) A special Vidhan Sabha committee had recently initiated a policy on groundwater
extraction.
108. Which of the following has been used as a premise by the author in the
passage?
i. High quantities of fluorides can cause problems to the human body.
ii. A policy on groundwater extraction needs to be framed.
iii. The inhabitants of the area are at a risk of getting carcinogenic diseases.
(a) Only ii (b) Only i and ii (c) Only i and iii (d) All i, ii and iii
109. Which among the roles does the following statement play towards the passage?
“Nearly 1,000 RO plants installed
since 2009 by the government in villages that have been lying defunct for the last
few years.”
(a) This statement strengthens the arguments of the author.
(b) This statement weakens the arguments of the authorities.
(c) This statement weakens the arguments of the author.
(d) The statement neutralises the claims made by the author.

. Page 30 of 40
110. The author of the passage would not agree with which of the following options?
(a) Restoring the contaminated water to its natural state is a hard nut to crack.
(b) Proactive steps need to be taken by the authorities to solve the given issue.
(c) The suggestion given by the Vidhan Sabha is based on irrelevant arguments.
(d) Playing with the lives of flora and fauna is common in Punjab.
Passage (Q.111-Q.115): Read the passage below and answer the questions that follow:
External Affairs Minister S Jaishankar’s visit to Sri Lanka comes at a time when
the island nation is facing an
unprecedented economic crisis. In Colombo primarily for the seven-nation BIMSTEC
summit, the minister is
also holding talks with Lankan leaders and sealing bilateral pacts. Dwindling
foreign exchange reserves have led
to a currency devaluation in Lanka, causing critical shortage of food, fuel and
other essential items. The
beleaguered country has to repay debt of about $4 billion during the rest of this
year, way above its foreign
currency reserves which fell to $2.31 billion by February. The Covid-19 pandemic
derailed Lanka’s all-important
tourism industry, even as misgovernance and overdependence on China have also
contributed towards wrecking
the economy.
India has been prompt in lending a helping hand. New Delhi recently announced to
extend a $1-billion line
of credit, following a $400-million currency swap. An additional line of credit of
$1.5 billion to import essentials
is on the cards, while the Rajapaksa government is negotiating $2.5 billion in
credit support from China and has
sought restructuring of debt payments.
Sri Lanka is an important ally for India, considering its strategic location in the
Indian Ocean Region, where
China has been flexing its muscles on the military and economic fronts. Pursuing
its ‘Neighbourhood First’
policy, New Delhi must go all out to build goodwill in the region by doing a better
job than Beijing in bailing
out Sri Lanka. With no immediate relief or assistance expected from the World Bank
and the International
Monetary Fund, India’s eagerness to help a neighbour in dire straits is in stark
contrast to China’s opportunism,
exemplified by the circumstances under which Lanka had to hand over the Hambantota
port on 99-year lease.
Once the crisis subsides, Lanka will have to introspect about the reasons that made
things come to such a pass.
Colombo needs to pragmatically recalibrate its ties with Beijing to ensure that it
doesn’t fall headlong into the
debt trap all over again. At the same time, it should never undervalue the
importance of having India on board
as an all-weather friend.
(Source - https://www.tribuneindia.com/news/editorials/lankan-crisis-381866)
111. Which of the following reflects the central idea of the passage?
(a) India should help Sri Lanka to counter China for domination over the region.
(b) The crisis provides an opportunity for India to reduce ally’s dependence on
China.
(c) The present crisis is a result of China’s opportunism.
(d) The Covid-19 pandemic derailed Lanka’s significant Industries.
112. The author is most likely to agree with which of the following action by the
Indian government: -
(a) Focusing on India rather than playing a bet on Sri Lanka.
(b) Helping Sri Lanka in the crisis by providing them financial assistance.
(c) Grabbing the opportunity for its own benefit.
(d) Going forward to helping Sri Lanka and building goodwill with China.
113. “Sri Lanka is an important ally for India, considering its strategic location
in the Indian Ocean Region” Which
of these statements, if true, strengthens the statement?
(a) The Indian Ocean region has some minerals and natural resources within it.
(b) International trade in future is going to be dominated by Indian ocean region.
(c) Sri Lanka lacks the funds and money to prosper in the region.
(d) The present establishment in Sri Lanka is pro-China.

. Page 31 of 40
114. The author is most likely to agree with which of the following statements?
(a) The long-lasting crisis in Sri Lanka doesn’t seems perennial.
(b) China has not done anything to help Sri Lanka.
(c) Sri Lanka must introspect its bilateral relations with China.
(d) The relations between China and Sri Lanka have stained after Hambantota port
issue.
115. What purpose does the boldfaced statement in the passage serve?
(a) It is argument of the author that has been later substantiated in the passage.
(b) It is the premise of the author to support the arguments.
(c) It is the argument of the author which has been drawn on the basis of above
paragraph.
(d) It is the premise supporting the main idea of the passage.
Passage (Q.116-Q.120): Read the passage below and answer the questions that follow:
The recent arrest of a 25-year-old Muslim woman at Mudhol in Bagalkot district of
Karnataka for an innocuous
message on Pakistan’s Republic Day is yet another instance of the perverse misuse
of the law by authorities. If
it was the provision relating to sedition that was invoked mindlessly in the past,
including once for a play enacted
by primary schoolchildren, the latest one involves an alleged attempt at creating
enmity among different groups.
Kuthma Sheikh was granted bail on the same day of her arrest, but the incident is
no less disconcerting as it
indicates the ease with which members of the minority community can be arrested
without sufficient cause, often
at the behest of overzealous activists with a disproportionate clout in the
administration. In this case, the madrassa
student had said, “May God bless every nation with peace, unity and harmony” on
March 23, but a local Hindu
activist complained to the police that she was creating enmity among communities by
wishing people on Pakistan
Day. With unsurprising promptitude, the police booked her under penal sections
relating to promoting enmity
between different groups. As to how her wishes would have attracted either Section
153A or 505(2) of the IPC
is something only the police can explain. The district police have claimed that the
arrest was aimed at preserving
peace and maintaining order, but it is quite apparent that they acted in a cavalier
manner without ascertaining
whether there was any substance in the complaint.
Ever since a controversy broke out over girl students wearing the hijab, there
seems to be a tendency among
right-wing groups to foment trouble targeted at Muslims. These groups have called
for a ban on Muslim
traders and vendors doing business as part of temple fairs. Even though the State
government is citing a law
that prohibits non-Hindus from getting property in the vicinity of the temple on
lease, it is doubtful whether the
rules cover temporary stalls on special occasions. It is regrettable that the State
government is not doing enough
to stem the impression that its administration is hostile towards minorities.
Unwarranted arrests, especially for
trivial reasons and on communally motivated complaints, result in unfair
incarceration, ruined lives and
immensely delayed justice. For a regime that takes strong exception to strident
criticism about its human rights
and religious tolerance record, the Union government should be equally concerned
about the possible damage
that such incidents may cause to its global image. The Centre may not have anything
to do with law and order,
but it may have to advise certain States to restrain the police from perfunctory
use of the power to arrest to please
majoritarian groups and individuals.
[source: the Hindu, https://www.thehindu.com/opinion/editorial/without-sufficient-
cause-the-hindu-editorial#on-unwarranted-arrests-at-the-behest-of-majoritarian-
outfits/article65271197.ece ]
116. What is the central idea of the passage?
(a) Arrests without substance stifles individuals’ right to eloquently voice their
thoughts.
(b) Communal divide dents the reputation of regime as an upholder of democracy
globally.
(c) Authorities have blatantly been misguided by the activists under the garb to
avoid further tensions.
(d) The instance elucidates the deploring ties between India and Pakistan.

. Page 32 of 40
117. Which of the following can be drawn as an inference of the passage?
(a) Courts have failed to prevent the abuse of power by the authorities, which has
ultimately cost delayed justice.
(b) India still shows the prevailing signs of communal tensions, embers of which
can be witnessed in Hijab issue.
(c) Centre has an upper hand in the matters involving law and order to avoid
communal tensions to escalate
further.
(d) India has failed to cherish its ideology of being a secular state due to brisk
attitude of authorities.
118. Which of the following is most likely to be agreed with, in consonance of the
passage?
(a) Sore International relations have potential to cause regional rifts in their
respective countries.
(b) The current regime at the centre has been incessantly using sedition provision
to silence critics.
(c) Opposition regime in states is targeting minorities in their respective state
to please majority groups.
(d) Majority of activists alleging false accusations are from the majority groups.
119. What purpose does the boldfaced statement serve in the passage?
(a) It is an argument to support how the laws are being abused.
(b) It is a premise to support how the laws are being abused.
(c) It is a premise to support the fact that communal barrier still persists in the
country.
(d) It is an argument to support the fact that communal barrier still persists in
the country.
120. What can be most appropriate conclusion drawn from the passage?
(a) It is high time that False arrests should be curbed.
(b) Where there is a room for discretion, there exists arbitrariness.
(c) Custodial deaths resulting from such false arrests depicts failed justice.
(d) International conflicts seldom fan internal differences.
Passage (Q.121-Q.125): Read the passage below and answer the questions that follow:
Among the many interesting observations that Chinese foreign minister Wang Yi made
at a press conference in
Beijing on Monday, one was particularly telling. Noting that China- European Union
trade had exceeded $800
billion for the first time in 2021, Wang said that the cooperation between China
and Europe, “going through
decades of ups and downs, is deeply rooted in solid public support, extensive
common interests and similar
strategic needs. Such cooperation… cannot be reversed by any force”. Since Russia
invaded Ukraine, Beijing
has had to balance its “no limits” friendship with Moscow, its vital economic
relationships with the US and
Europe, and the need to be seen and accepted as a responsible power but without
yielding on its core belief —
that Americanunilateralism is the original sin. It has constantly finessed its
statements on the conflict to reflect
the fast-evolving ground situation. On Tuesday, Chinese President Xi Jinping called
for “maximum restraint to
prevent a humanitarian crisis” in a conference with French President Emmanuel
Macron and German Chancellor
Olaf Scholz, commendedtheir mediation efforts, and said China would coordinate with
France, Germany and
the EU, and work actively with the international community.
It is clear that Russia’s actions in Ukraine have left Beijing in a difficult
position, one thatit may not have
anticipated perhaps due to a belief, now misplaced, that Russian President Vladimir
Putin would be able to pull
off a surgical operation. The sanctions against Russia are going to hurt Beijing as
well. But it appears that China
also sees an opportunity in the present moment to project itself as a country that
believes in de- escalation and
conflict resolution and is prepared to work with other global powers to achieve
this. It seeks to play down the
image of a country that needles its neighbours and indulges in coercive Wolf
Warrior diplomacy.
Beijing still considers the US as its primary rival in its quest for global
supremacy. Wang’s deliberate casting of
the Quad grouping as an “Asian NATO” may have been anattempt to draw a comparison
between the Indo#Pacific and the conflict in Europe, but it contradicts its own
position that the two are not comparable. If anything,
in terms of economics, China is a prospective beneficiary of the war in Ukraine,
with the US preoccupied in
Europe.

. Page 33 of 40
121. The author’s arguments are strengthened by which of the following pieces of
evidence?
(a) China’s pro Russia stance can harm its relations with EU.
(b) China’s position on the Ukraine conflict would eventually improve its economic
standing.
(c) In recent times, India has emerged as a global leader in terms of economic
growth.
(d) Both a and b are correct.
122. What is the best representation of the central argument of the author in
thepassage?
(a) China wants to be seen as a responsible power in the Ukraine conflict.
(b) China’s ambition of becoming a global leader has been undermined by its
actions.
(c) China’s ambition to take centre stage in the world is becoming a reality
(d) China is sure to gain, even if it takes a temporary beating in the Ukraine
conflict.
123. What could be inferred based on the information given in the passage?
(a) China’s position in the Ukrainian conflict is different from that of India.
(b) The US presents a real challenge to the global supremacy of China.
(c) France and Germany’s stance on the conflict is not starkly different from that
of China.
(d) China’s antagonism against America will ultimately benefit the European Union.
124. What is the author trying to portray in the second paragraph of the passage?
Choose the best option.
(a) That China had conspired with Russia to defeat Ukraine.
(b) That China has an opportunity to establish a new world order.
(c) That the relations between China and Russia are set to deteriorate.
(d) That Russia is unlikely to defeat Ukraine comprehensively.
125. Which among the following can be said to be false based on the information
given in the passage?
(a) China has a vested interest in the conflict between Russia and Ukraine.
(b) China has never claimed that Quad and NATO are incomparable.
(c) Chinese President wants the conflict to not escalate beyond uncontrollable
limits.
(d) Chinese Foreign Minister is wary of India’s position on Ukraine.
Passage (Q.126-Q.130): Read the passage below and answer the questions that follow:
This week, India will complete two years since the Government embarked on what is
now considered the harshest
and quickest lockdowns in the world in a bid to block entrypoints for the
infectious COVID-19 virus. The
efficacy of that lockdown, both in terms of curbing infection rates (and mortality
rates) and the accompanying
hardships imposed on the population at large, can be debated at length. There is,
however, little debate over the
massive economic costs for the country. The Reserve Bank of India has underlined
that some of that damage to
India’s GDP is permanent. This has been linkedto businesses shutting shop for good,
labourers migrating home
(with many choosing not to return) and consumers turning increasingly reluctant.
[1] The rebuilding effort
remains a work in progress, although record tax collections would suggest that all
is well. Personal
consumption and spending on personal wants have dipped, even as essential goods
have reached pre-COVID
performance. Just as the virus appeared to be ebbing, triggering hopes of a revival
in consumer confidence, the
Russia-Ukraine conflict has thrown up fresh challenges, including high commodity
and crude oil prices.
Health-care costs are considered a key factor for pushing several middle and lower-
income households below
the poverty line, while high inflation affects all economic sectors. The Russia-
Ukraine situation has not only
catapulted gas, oil and coal prices higher but also fertilizers, wheat, corn, and
seed oil. For now, India’s oil
marketing companies, who the Government has allowed to determine the retail prices
of fuel, haveshown
extreme benevolence in holding rates at November 2021 levels and this may persist
till Parliament’s current
session ends. This is, however, not fiscally sustainable, just as the Finance
Ministry has argued that high global
commodity prices are not. A prolonged conflict in Europe could tip the global
economy into recession, even as
monetary policy missteps and social risks associated with high inflation, could
dampen growth. [2] On the other
hand, the Government’s robust direct tax collections that have surpassed even
revised estimates by ₹1.13

. Page 34 of 40
lakh crore, give it room to not just push forward the LIC share sale till market
volatility subsides but also
slash fuel taxes further, curb other inflationary pressures and expand the COVID-19
booster shots
coverage. Unless people get more certainty about the pandemic’s end- game and have
some money in their
hands, it would be difficult to spur consumption enough to reach the necessary next
stage of the recovery — a
revival in private investments.
126. If the information in the given passage is true, then which of the following
can be said to be true as well?
(a) Record tax collections is an indicator that nothing can destabilise a country’s
economy.
(b) High tax collections by the government give it an opportunity to raise
consumers’ confidence.
(c) The poor are impacted the most when the rates of their secondary needs are
increased unusually.
(d) The Pandemic has dented the economies of the countries for decades.
127. Out of the following, which piece of evidence would substantially weaken the
arguments of the author?
(a) The lockdown imposed by the government was not able to stop the spread of Covid
19.
(b) Migrant labourers have chosen not to return because they found better
opportunities to work in their
native places.
(c) The conflict between Russia and Ukraine has had a major impact on the
psychology of Indian consumers.
(d) The pandemic has not divided the line between the poor and the rich further, as
consumers.
128. The author would agree with which of the following statements?
(a) If some migrants had chosen to return to their workplaces, then the damage to
the economy would have been
less severe.
(b) The Russia-Ukraine conflict has implications that are being felt at places
other than the two countries.
(c) India’s oil marketing companies should be allowed to increase prices steeply
for their patience.
(d) The lockdown has caused irreparable damage to the country’s economy, with
little possibility of returning
to pre-pandemic levels.
129. The relationship between boldface statements is best depicted by which of
thegiven options?
(a) [1] can be used as a premise for the claim made in [2].
(b) [2] can be used as a premise for the claim made in [1].
(c) Both [1] and [2] could be used as a premise for one another.
(d) [1] and [2] contain claims that are independent of each other.
130. What role does the following statement play towards the passage?
“Migrantlabourers found better opportunities
to work in their native places”
(a) This statement is an assumption made by the author.
(b) This statement can be reasonably inferred from the passage.
(c) This statement weakens the argument of the RBI.
(d) This statement is a course of action.
131. In the question given below, two statements are given followed by conclusions:
I and II. You have to take the
given statements to be true even if they seem to be at variance from commonly known
facts. Read the conclusions
and decide which of the given conclusions logically follows from the given
statements disregarding commonly
known facts.
Statement 1: No dress is a cloth.
Statement 2: All clothes are colours.
Conclusion I: Some colours are dresses.
Conclusion II: No dress is a colour.
(a) Both I and II follow (b) Only I follows
(c) Either I or II follows (d) Neither I nor II follow

. Page 35 of 40
132. In the question given below, two statements are given followed by three
conclusions. You have to take the given
statements to be true even if they seem to be at variance from commonly known
facts. Read the conclusions and
decide which of the given conclusions logically follows from the given statements
disregarding commonly
known facts.
Statement 1: All words are ways.
Statement 2: Some ways are round.
Conclusion I: All rounds are words.
Conclusion II: Some rounds are words.
Conclusion II: All ways are round.
(a) Only II follows (b) None of the conclusions follow
(c) Either I or III follows (d) Both I and II follow

You might also like